Sei sulla pagina 1di 495

MANUAL DE TRABAJO DEL CURSO ENARM CMN SIGLO XXI

Pharmed Solutions Institute


Dr. Angel Mauricio Soriano Perez.

MANUAL DE TRABAJO DEL CURSO ENARM CMN SIGLO XXI


INTRODUCCION

Este manual de trabajo del curso para aprobar el ENARM; es el resultado de ms de 13 aos de estudio de los contenidos del ENARM desde el ao 2000 a
la fecha. La modalidad, contenido y caractersticas pedaggicas han cambiado de forma importante desde entonces con la versin moderna del ENARM.
Podemos mencionar que es la mejor.
Los cursos de preparacin para el ENARM que se imparte en diversas instituciones pblicas y privadas han tenido que adaptarse a estos cambios. Sin
embargo, no todos los cursos, ni los diferentes libros dedicados a la preparacin de los mdicos aspirantes, ha tomado con toda seriedad y
responsabilidad. Modificando y adaptndose a los cambios cruciales para as ofrecer una verdadera alternativa al mdico general que desea prepararse
para aprobar el ENARM.
Nuestro curso, los medios que son empleados y la metodologa han sido ampliamente analizados para ser un curso integral que ofrezca un medio
apropiado para ser un seleccionado nacional.
Agradecemos a todos los mdicos especialistas que participaron en la realizacin del presente manual de trabajo, as como a los participantes de los
diversos cursos.

Dr. ngel Mauricio Soriano Prez


Director General

No hay hombre ms digno de estimacin que el mdico que, habiendo estudiado la naturaleza desde su juventud, conoce las propiedades del cuerpo
humano, las enfermedades que le atacan y los remedios que pueden beneficiarle y que ejerce su arte.
Voltaire

CURSO ENARM CMN SIGLO XXI TEL: 36246001

Pharmed Solutions Institute

PGINA 2

MANUAL DE TRABAJO DEL CURSO ENARM CMN SIGLO XXI

CURSO ENARM CMN SIGLO XXI TEL: 36246001

Pharmed Solutions Institute

PGINA 3

MANUAL DE TRABAJO DEL CURSO ENARM CMN SIGLO XXI

CURSO ENARM CMN SIGLO XXI TEL: 36246001

Pharmed Solutions Institute

PGINA 4

MANUAL DE TRABAJO DEL CURSO ENARM CMN SIGLO XXI


MEDICINA INTERNA
1) HIPERTENSION ARTERIAL PRIMARIA Y SECUNDARIA
2) INSUFICIENCIA CARDIACA AGUDA Y CRONICA
3) VALVULOPATIAS.
4) MIOCARDITIS, ENDOCARDITIS Y PERICARDITIS
5) NEUMONIAS.
6) NEUMOPATIAS
7) HIPERTENSION PULMONAR Y COR PULMONAR
8) CARCER PULMONAR Y BRONCOPULMONAR
9) TRASTORNOS HIPOTALAMICOS E HIPOFISIARIOS
10) TRASTORNOS TIROIDEOS Y PARATIROIDEOS
11) TRASTORNOS SUPRARRENALES Y GONADALES
12) SOBREPESO, OBESIDAD Y DISLIPIDEMIAS
13) DIABETES MELLITUS Y COMPLICACIONES
14) ESTEATOSIS HEPATICA, CIRROCIS HEPATICA, HEPATOPATIAS
15) LEUCEMIAS, LINFOMA NO HODGKIN, LINFOMA HODGKIN
16) FIEBRE REUMATICA, ARTRITIS REUMATOIDE
17) OSTEARTROSIS, OSTEOCONDRITIS, GOTA, SINDROME DE SJOGREN
18) LUPUS ERITEMATOSO SISTEMICO, VASCULITIS
19) TUBERCULOSIS, PALUDISMO, DENGUE, LEPRA
20) VIH Y SIDA
21) CMV, VEB, HEPATITIS, RABIA
22) CANCER E INFECCIONES DEL SISTEMA NERVIOSO CENTRAL
23) CEFALEA, EPILEPSIAS Y ENCEFALOPATIAS
24) DEMENCIA VASCULAR, ALZHEIMER Y PICK, HUNTINGTON Y PARKINSON
25) ESCLEROSIS MULTIPLE, ESCLEROSIS LATERAL AMNIOTROFICA Y MIASTENIA GRAVIS
26) DEPRESION, ANSIEDAD, ESQUIZOFRENIA Y TRASTORNOS DE LA ALIMENTACION
COMPLEMENTARIOS
27)
28)
29)
30)
31)

TRIPANOSOMIASIS, LESHMANIASIS, ESQUISTOSOMIASIS, FILARIASIS, ONCOCERCOSIS


RICKETTSIAS, LEGIONELA, PSEUDOMONA AERUGINOSA, LISTERIA MONOCYTOGENES
CRIPTOCOCCIS, LEPTOSPIROSIS, ACTINOMICOCIS, NORCARDIA
BORDOTELA PERTUSIS, MORAXELLA CATARRHALIS, BRUCELA, FRANCISELLA TULARENSIS
STAFILOCCOCUS AUREUS, EPIDERMIDIS Y ESTREPTOCCOCUS PNEUMONIAE, HAEMOPHILUS INFLUENZAE,

CURSO ENARM CMN SIGLO XXI TEL: 36246001

Pharmed Solutions Institute

PGINA 5

MANUAL DE TRABAJO DEL CURSO ENARM CMN SIGLO XXI


HIPERTENSIN ARTERIAL PRIMARIA Y SECUNDARIA:
CIENCIAS BASICAS: Es el incremento sostenido de la tensin arterial diastlica >90 mmHg. La OMS la define como un trastorno cuya patogenia se
desconoce en muchos casos, que en ltima instancia causa aumento de la presin diastlica/sistlica, as como alteraciones del lecho vascular y
alteraciones funcionales de los tejidos afectados. SALUD PUBLICA: Prevalencia mundial: 1 billn de individuos, 7.1 millones de muertes al ao. OMS:
Seala que con una PAM >115mmHg, genera un 62% de los EVC y 49% de la enfermedad isqumica cardiaca. La padecen 50% de personas entre 60-69
aos y 3/4 de los mayores de 70 aos. La prevalencia de hipertensin sistlica se incrementa con la edad, a partir de los 50 aos es la forma ms comn.
PAD factor de riesgo CV ms potente hasta los 50 aos. A partir de entonces, la PAS >140mmHg es de importancia mayor como factor de riesgo de EVC.
En los ensayos clnicos, la terapia antihipertensiva se ha asociado con reducciones en incidencias de ictus de un 35-40 %, de infarto de miocardio de un
20-25 %, y de insuficiencia cardiaca en ms de un 50 %. Estudio Framingham: PA de 130-139/85-89 mmHg: 2 veces de incremento RR de enfermedad
cardiovascular comparados con los niveles por debajo de 120/80 mmHg. El riesgo de EVC comienza a partir de 115/75 mmHg se dobla con cada
incremento de 20/10 mmHg. ETIOLOGIA: HTA primaria es ms frecuente, la secundaria es causada por: estenosis de arteria renal, insuficiencia renal,
apnea del sueo, causas atribuibles a frmacos (AINEs, cocaina, anfetaminas, simpaticomimticos, anticonceptivos orales, corticoides), enfermedad de
las glndulas suprarrenales, feocromocitomas, hiperplasia cortical, sndrome de cushing, hipertiroidismo, hiperaldosteronismo, otras causas son la
preeclampsia, coartacin de la aorta, enfermeades cerebrales. Factores de riesgo mayores: Hipertensin, edad (H >55, M >65), Diabetes mellitus, LDL
elevado, colest total alto bajo HDL, TFG <60 ml/min, historia familiar de EVC prematura (H <55 M <65), microalbuminuria, obesidad (IMC > 30Kg/m2),
inactividad fsica, tabaco. Dao rgano diana: hipertrofia ventricular izquierda, angina, IAM, revascularizacin coronaria, insuficiencia cardiaca, ictus
ataque isqumico transitorio, demencia, enfermedad renal crnica, enfermedad arterial perifrica, retinopata. PATOGENIA: Multifactorial, interaccin
de factores prevenibles y no prevenibles, adems de hiperactividad del sistema nervioso simpatico, alteracin de catecolaminas, quimio y
barorreceptores. Alteraciones del sistema renina-angiotensina-aldosterona, altaracion de resistencias vascular, del volumen intravascular, constricion
arteriolar, e hipopotasemia y defecto natriuretico por capacidad disminuida de reducir la carga de sodio e hdrica. Aumento de sodio y calcio intracelular,
alteracin del transporte Na/K. CUADRO CLINICO: Medida apropiada de PA, verificacin en el brazo contralateral, examen del fondo de ojo, IMC,
auscultacin carotdea, abdominal y soplos femorales, palpacin de la glndula tiroidea, examen cardiaco y pulmonar completo, exploracin abdominal:
riones, deteccin de masas y pulsaciones articas anormales, palpacin de pulsos y edemas en extremidades inferiores, valoracin neurolgica, buscar
sntomas de rgano blanco, cefalea, acufenos, fosfenos, presion torcica, palpitaciones, fatiga, adinamia, cansancio, edema, alteraciones visuales,
alteraciones sensitivas. DIAGNOSTICO: Se recomienda realizar toma de TA a pacientes mayores de 35 aos con o sin sntomas sugestivos de HTA. La
clasificacin de la presin arterial segn la OMS: Normal; menor de 140/90 mmHg. Limtrofe: 140-160 y 90-95 respectivamente, HTA leve 140-180 y 90105; HTA moderada y severa > 180 y > 105 mmHg, o la JNC7: Normal <120/80, Prehipertensin 120-139/80-89, HTA estadio 1, 140-159/90-99, HTA
estadio 2 >160/>100mmHg. TRATAMIENTO: Primero modificar los estilos de vida, en presin normal y prehipertension no esta indicado el tratamiento
farmacolgico, IMC 18.5-24.9 (reduccin de 5-20mmHg, por cada 10Kg de disminucin de peso), NaCl <6 grs/da (reduce de 2-8mmHg), ejercicio minimo
30 min/dia (reduce 4-9mmHg), no alcohol, tabaco. En estadio 1, las tiazidas son de primera eleccin, se pueden considerar IECAS, ARA II, BBs, BCC. En
estadio 2 generalmente combinacin de dos frmacos tiazidicos con IECAS, ARAII, BBs. Los objetivos son: PAS y PAD <140/90 mmHg, (prevenir
complicaciones cardiovasculares). En DM o IR el objetivo es <130/80mmHg. Diureticos tiazidicos, son insuperables en la prevencin de complicaciones
cradiovasculares, enlentecen la desmineralizacin que ocurre en la osteoporosis, aunque pueden generar resistencia a la insulina en tejidos perifericos,
iniciar hidroclorotiazida genera natriuresis, depleciona el espacio intravascular, inhiben reabsorcin de Na+ y Cl- en el tbulo distal. IECAs: Bloquean de
forma competitiva la enzima que convierte angiotensina I en angiotensina II. Disminucin importante de angiotensina II y aldosterona, con estmulo
secundario de la sntesis de renina, no usar en pacientes con antecedentes de angioedema, tampoco si Cr >2.5 o FG <30ml/min, porque precipitan el
dao renal. Antagonistas de los receptores AT1: Ejercen un bloqueo competitivo unindose al receptor AT1 y evitando las acciones derivadas del estmulo
de ste. Simpaticoliticos: alfametildopa estimula los receptores 2adrenrgicos en el SNC, disminuyendo el tono simptico perifrico. Alfabloqueantes:
prazocin acta sobre receptores -1 postsinpticos. BBs; Metoprolol: Reduce gasto cardaco a travs de la frecuencia y del inotropismo cardaco, bloqueo
de la liberacin de renina hasta 60% a nivel renal, en pacientes con DM hay empeoramiento de la sensibilidad a la insulina y pueden enmascarar cuadros
de hipoglucemia. No usar en bloqueos cardiacos de 2do y 3er grado. Calcioantagonistas: verapamilo, diltiazem y nifedipino (dihidropiridinas de accin
corta, riesgo de IAM), tiles en hipertesion arterial aislada. En isquemia cardiaca: 1. Hipertensin y angina estable; BBs, BCC (accin larga), nitratos. 2.
SICA (angina inestable o IAM): BBs y IECAS. 3. Post-IAM: BBS, IECAs, antagonistas de la aldosterona. PRONOSTICO: Las causas de muerte: 50% fallecern
por enfermedad cardiovascular, principalmente por cardiopata isqumica, 33% por enfermedad cerebrovascular, 10-15% por insuficiencia renal, 2-7%
por otras causas. COMPLICACIONES: Insuficiencia cardiaca como resultado de la HTA y de la enfermedad coronaria isqumica, lo ideal es tratarala con
IECAs y BBs. Hipertensin resistente: fracaso en conseguir objetivos en pacientes que tienen adherencia teraputica a dosis completa de una pauta
apropiada con 3 farmcos que incluyan un diurtico. Una vez excluidas causas identificables de HTA. REHABILITACION: Disminuir la remodelacin
vascular (engrosamiento de la intima, media y adventicia) y la repercusin visceral (remodelado miocrdico, nefropata hipertensiva, retinopata
hipertensiva). INSUFICIENCIA RENAL CRONICA: Se determina por una filtracin <60ml/min, albuminuria >300mg/da, el objetivo del tratamiento es
enlentecer el dao renal y TA <130/80mmHg, altamente recomendados los IECAs y ARA II, por sus efectos favorables en la progresin de la enfermedad
renal. HIPERTENSION SECUNDARIA: ENFERMEDAD RENAL PARENQUIMATOSA; Datos de sospecha, nicturia, edema, aumento de creatinina, proteinuria,
hematuria. Se puede diagnosticar por ecografa y/o biopsia renal. HTA RENOVASCULAR: sospecha HTA en estadio 3 con IR progresiva, HTA refractaria,
edema agudo de pulmn recurrente, soplo abdominal, enfermedad vascular perifrica. Causa mas frecuente en adulto, estenosis ateroesclertica en
arteria renal, en pacientes jvenes la causa mas frecuente, son las displasias fibromusculares. Diagnostico; eco-doppler de arterias renales,
angiorresonancia, radiorrenograma con captopril. El tratamiento es una angioplastia con baln con o sin implante. HIPERALDOSTERONISMO PRIMARIO:
Exceso de produccin de aldosterona, datos de sospecha, hipokalemia espontnea, inducible con diurticos, HTA refractaria, hallazgo casual de una
masa suprarrenal. Diagnostico; aldosterona plasmtica y urinaria, test de fludrocortisona (valores >5ng/dl). Puede ser causado por un adenoma
productor de aldosterona (tx.: quirrgico), por una hiperplasia adrenal bilateral o hiperaldosteronismo idioptico (tx.: espironolactona), se puede hacer
TAC o centellograma. FEOCROMOCITOMA: es un tumor derivado de clulas cromafines, producen adrenalina y noradrenalina a veces dopamina, datos
de sospecha, triada palpapitaciones (HTA), cefalea, sudoracin. Puede haber arritmias y palidez durante episodios hipertensivos, la hipertensin
paroxstica es el signo ms frecuente (90-95%), pueden desencadenarse durante procedimientos invasivos diagnosticos, anestesia, ingestin de
alimentos que contengan tiramina. Diagnostico; catecolaminas plasmticas y urinarias, ac. Vainillin mandelico, metanefrinas urinarias, TAC. El tx.: es con
prazosin o doxazosin, para provocar vasodilatacin se puede agregar un BBs. SINDROME DE CUSHING: sospecha obesidad, estras, debilidad muscular,
edema, hiperglucemia. Diagnostico; cortisol libre urinario, test de dexametasona.
CASO CLINICO
Se trata de femenino de 52 aos, acude a consulta por presentar desde
hace 6 meses cefalea global de intensidad leve-moderada, que se

CURSO ENARM CMN SIGLO XXI TEL: 36246001

incrementa con los esfuerzos y bajo estrs laboral principalmente,


agrega que ha tenido episodios de mareo no relacionado a el
movimiento, asi como ruidos e intolerancia a la luz, las ltimas

Pharmed Solutions Institute

PGINA 6

MANUAL DE TRABAJO DEL CURSO ENARM CMN SIGLO XXI


semanas ha notado que le aprietan los zapatos, y se cansa con
facilidad al subir las escaleras en su trabajo. AHF: padre finado por IAM
sbito a la edad de 40 aos, madre actualmente viva con DM tipo 2,
dislipidemia e hipertensin arterial bajo tratamiento. APP: tabaquismo
positivo desde la edad de 20 aos a razn de 5 cigarrillos diarios,
alcoholismo social, vida sedentaria y obesidad grado I. AGO: desarrollo
eclampsia hace 30 aos y tomo anticonceptivos por 15 aos. A la EF:
IMC 32, TA 145/100mmHg. FC 93 lpm, FR 23, paciente con palidez de
tegumentos, manchas algodonosas y hemorragias en llama en fondo
de ojo, discreta ingurgitacin yugular al decbito dorsal, ruidos
respiratorios disminuidos, ruidos cardiacos normales, edema maleolar
++, laboratorios Hb 12, Hto 49, glucosa 116, creatinina 1.6, acido urico
9, colesterol 256, triglicridos 380, EGO: proteinuria +, bacterias ++,
Estudios de gabinete: tele de torax con discreta hipertrofia ventricular
izquierda, ECG con QRS amplios de V4, V5 y V6.
PREGUNTA
Cul de los siguientes criterios es el ms til para para establecer el
diagnostico?
RESPUESTA
a.- Cifras tensionales altas en repetidas ocaciones.
b.- Sospecha de alteraciones de rgano blanco.
c.- Factores de riesgo presentes.
d.- Alteraciones de estudios paraclinicos.
PREGUNTA
En que categoria se encuentra actualmente el paciente?
RESPUESTA
a.- Normal superior.
b.- Etapa 2.
c.- Etapa 1.
d.- Etapa 3.
PREGUNTA
Cul es la patologa diferencial ms probable que puede sospecharse
con los datos clnicos y laboratorios actuales?
RESPUESTA
a.- Patologa renal.
b.- Uso de estrgenos.
c.- Hipertensin vascular renal.
d.- Hiperaldosteronismo primario.
PREGUNTA
Cul de los auxiliares diagnosticos esta indicado para continuar con el
manejo integral del padecimiento?
RESPUESTA
a.- Realizar ecocardiograma.
b.- Realizar urografa excretora.
c.- Repetir BH, QS y EGO.
d.- Solicitar electrolitos sericos.
PREGUNTA
Que intervencin no farmacolgica proporciona un mayor impacto
para la reduccin de cifras tensionales de esta paciente?
RESPUESTA
a.- Dieta hiposodica.
b.- Dieta hipocalrica
c.- Dieta hipoproteica.
d.- Dieta normokalemica.
PREGUNTA
Cul de los siguientes farmacos presenta mayor utilidad para el
paciente?
RESPUESTA
a.- IECAs.
b.- Calcio antagonistas.
c.- Diureticos.
d.- ARA II.

CURSO ENARM CMN SIGLO XXI TEL: 36246001

PREGUNTA
Cul de los siguientes esquemas farmacolgicos presenta mayor
utilidad para el paciente?
RESPUESTA
a.- Metoprolol, hidroclorotiazida y nifedipina.
b.- Captopril, hidroclorotiazida y metoprolol.
c.- Lisonapril, amlodipino nifedipino.
d.- Losartan, hidroclorotiazida.
PREGUNTA
Cules de las condiciones no es causa de envio a segundo nivel a los
pacientes con hipertensin arterial de difcil control?
RESPUESTA
a.- Falta de recursos para continuar con el manejo integral del
paciente.
b.- Datos sugestivos de dao a rgano blanco.
c.- Requerir ms de 3 farmacos.
d.- Presencia de crisis hipertensivas.
PREGUNTA
Cul rgano blanco presenta mayor evidencia de dao en el paciente
actual?
RESPUESTA
a.- Retina.
b.- Rion.
c.- Corazn.
d.- Arterias.
PREGUNTA
Cules son los cambios bioqumicos ms probables por los datos
clnicos en el caso descrito?
RESPUESTA
a.- Aumento de angiotensina II.
b.- Aumento de endotelinas.
c.- Aumento de noradrenalina.
d.- Disminucin de tromboxano A2
PREGUNTA
Cul de los cambios funcionales no es dependiente del sistema
cardiovascular?
RESPUESTA
a.- Hiperactividad simpatica.
b.- Aumento del volumen sanguineo.
c.- Disminucion FNA.
d.- Expancin del volumen extracelular.
PREGUNTA
Cul de los elementos, es menos adecuado para garantizar el
adecuado control, vigilancia y seguimiento en este paciente?
RESPUESTA
a.- Toma de presin arterial al menos 2 veces al mes.
b.- Envio oportuno a segundo nivel al diagnostico inicial.
c.- Evaluacin oftalmolgica para manejo de retinopata.
d.- Solicitud de ecocardiograma.
PREGUNTA
Cul de los resultados de los ndices o clasificaciones siguientes es
ms firme para la referencia del paciente antes descrito a segundo
nivel de atencin?
RESPUESTA
a.- Clasificacin de keith, Wagener y Barker.
b.- Indice de KDOQI.
c.- ndice de Sokolow-Lyon.
d.- ATP III.
PREGUNTA

Pharmed Solutions Institute

PGINA 7

MANUAL DE TRABAJO DEL CURSO ENARM CMN SIGLO XXI


Cules son las complicaciones mas frecuentes en el paciente con
hipertensin arterial?
RESPUESTA
a.- Infarto agudo al miocardio.
b.- Accidente vascular cerebral.
c.- Insuficiencia cardiaca congestiva.
d.- Insuficiencia renal crnica.
PREGUNTA
Cul de los siguientes cambios histolgicos renales no se presentan
en la hipertensin arterial?
RESPUESTA
a.- Inadecuado desarrollo del sistema capilar.
b.- Esclerosis glomerular.
c.- Necrosis fibrinoide glomerular.
d.- Fibrosis intersticial.
PREGUNTA
Cules son los cambios anatmicos menos frecuentes que se
presentan de la retina en la hipertensin arterial?
RESPUESTA
a.- Papiledema del nervio ptico.
b.- Atrofia.
c.- Hemorragias.
d.- Ceguera.
PREGUNTA
Cul es el efecto farmacodinamico ms til para la nefroproteccion en
el tratamiento de la hipertensin arterial?
RESPUESTA
a.- Diurtico con ahorro de K.
b.- Inhibidores de la enzima de conversin de la AGT
c.- Antagonistas de receptores de AGT II.
d.- Bloqueadores de canales de calcio.
PREGUNTA
Cul es la estrategia ms til para la rehabilitacin de las alteraciones
cardiovasculares?
RESPUESTA.
a.- Control clnico y de los factores de riesgo.
b.- Entrenamiento fsico programado.
c.- Tratamiento psicolgico.
d.- Estudio de familiares de primer grado.
PREGUNTA
Cul es la estrategia ms til para la rehabilitacin de las alteraciones
nefrolgicas del caso actual?
RESPUESTA
a.- Mantener los niveles de tensin arterial estricta.
b.- Monitorizacin de KDOQI semestral.
c.- Reduccin de peso, lpidos y tabaco.
d.- Disminuir la actividad del sistema nervioso simpatico.
PREGUNTA
Cul es la accin ms importante para prevenir los factores de riesgo
de hipertensin arterial en el primer nivel de atencin?
RESPUESTA
a.- Mantener IMC menor a 25/Kg/m2.
b.- Dieta hiposodica, hipocalrica y normokalemica.
c.- Evitar tabaquismo, alcoholismo y otras toxicomanias.
d.- Ejercicio aerobico y manejo del estrs.
CASO CLINICO
Se trata de masculino de 78 aos de edad el cual se acude a consulta
por dolor articular en rodilla izquierda, durante la exploracin usted
identifica que sus cifras tensionales son las siguientes, TA 170/80
mmHg, repite la toma de presin y se mantiene semejante, el paciente
niega sintomatologa asociado a las cifra tensionales. El paciente

CURSO ENARM CMN SIGLO XXI TEL: 36246001

cuenta con antecedente de un episodio de isquemia cerebral


transitoria hace 6 meses.
PREGUNTA
Cual es conducta a seguir para el manejo de este caso?
RESPUESTA
a.- Reduccin de peso y sal.
b.- Hidralacina 25 mg cada 24 hrs.
c.- Captopril e hidralacina.
d.- Enalapril y lisonipril.
PREGUNTA
Cual es el objetivo sobre el ndice de masa corporal para el
tratamiento del paciente?
RESPUESTA
a.- Mantener el ndice corporal entre 20 y 25 Kg/m2.
b.- Mantener la tensin arterial sistlica al menos en 140 mmHg.
c.- Mantener la tensin arterial diastlica menor a 90 mmHg
d.- Disminuir el riesgo de IAM o EVC.
PREGUNTA
Cual es el tratamiento de eleccin farmacolgico considerando el
antecedente del paciente?
RESPUESTA
a.- Enalapril 10 a 20 mg.
b.- Losartan 50 mg
c.- Hidroclorotiacida 12.5 a 25mg.
d.- Amlodipino.
PREGUNTA
El paciente se encuentra estable actualmente con cifras tensionales de
120/80 mmHg. cual de las siguientes medidas de referencia debe
realizar para envio a tercer nivel?
RESPUESTA
a.- Realizar QS, BH, EGO.
b.- Laboratorios de rutina ms Rx de torax y electrocardiograma.
c.- Buscar primeramente dao a rgano blanco.
d.- No es necesario ya que fue un hallazgo reciente.
PREGUNTA
El paciente fue evaluado por cardilogo y volvi para continuar con
control, dentro de las medidas enviadas por el mismo fue monitorizar
la tensin arterial supina, sentado y de pie. Cual es la razn de esta
medida?
RESPUESTA
a.- Es una medida que garantiza las cifras.
b.- Buscar hipotensin ortostatica.
c.- Verificar el efecto del tratamiento.
d.- No tiene un efecto demostrable.
CASO CLINICO
Femenino de 51 aos de edad originaria de Campeche, acude a su
control mensual, antecedentes de Hipertensin arterial y diabete
mellitus desde hace 10 y 5 aos respectivamente, sus signos vitales
fueron TA 155/95 mmHg, FC 58 lpm, FR 27 rpm, laboratorios glucosa
en ayunas de 210, hemoglobina glucosilada 6, urea 17, creatinina 1,1
mg/dl, refiere haber presentado mareo ocacional, cefalea global
acompaado de sensacin de sueo, frio con mareo ocacional y disnea
de leve a moderados esfuerzos, se observa edema de miembros
inferiores. Su tratamiento es captopril, metoprolol y glibenclamida.
PREGUNTA
Se realiza ECG, considerando las manifestaciones que se han
presentado, que conducta teraputica considera ms apropiada
realizar de forma mediata?
RESPUESTA
a.- Incrementar los farmacos hasta meta de <120/80 mmHg.
b.- Retirar metoprolol continuar con captopril y glibenclamida.

Pharmed Solutions Institute

PGINA 8

MANUAL DE TRABAJO DEL CURSO ENARM CMN SIGLO XXI


c.- Mantener glibenclamida, retirar metoprolol e iniciar losartan.
d.- Agregar hidroclorotiazida al manejo actual.

inmediato, sin edema, pies sin lceras, reflejos osteotendinosos


normales, fuerza conservada, sensibilidad sin alteraciones.

PREGUNTA
Cuales son los objetivos ms importantes con el nuevo manejo
debido a los antecedentes?
RESPUESTA
a.- Disminuir el riesgo de IAM.
b.- Disminuir el riesgo a EVC.
c.- Disminuir el bloque b-adrenergico por riesgo a hipoglucemia.
d.- Disminuir el riesgo de progresin de nefropata.

PREGUNTA
De acuerdo al cuadro clnico del paciente, de qu tipo de hipertensin
se trata?
RESPUESTA
a.- Hipertensin arterial descontrolada
b.- Hipertensin arterial sistlica aislada
c.- Hipertensin arterial diastlica aislada
d.- Hipertensin secundaria

PREGUNTA
Dos aos despus la paciente continua en tratamiento, actualmente ha
surgido edema de miembros inferiores, fatiga de medianos esfuerzos,
adems de dos eventos de isquemia cerebral transitoria menor a 2
horas. Cual es la conducta a seguir?
RESPUESTA
a.- Indica estudios para identificar alguna complicacin a rgano
blanco.
b.- Modifica la dosis para mantener un mejor manejo.
c.- Indica mayores medidas generales de control de la tensin arterial y
diabetes mellitus.
d.- Considera suficientes criterios para enviar a segundo nivel.

PREGUNTA
Cul sera el manejo ms adecuado para este paciente?
RESPUESTA
a.- Beta bloqueadores
b.- Diurticos
c.- Calcio antagonistas no dihidropiridinas
d.- Calcio antagonistas dihidropiridinas

CASO CLINICO
Masculino de 22 aos de edad el cual es ingresado a urgencias debido
a la presencia de cefalea intensa, palpitacin, sensacin de muerte
inminente, diaforesis, refiere que ya le haba ocurrido esto
anteriormente pero no tan intenso, su tensin arterial fue de 140/100
mmHg. FC 115, FR 32, se observa agitado con dolor precordial resto de
exploracin sin datos por agregar, laboratorios de rutina sin datos
patolgicos.
PREGUNTA
Considerando la presencia del cuadro clnico cual es la conducta a
seguir mas adecuada.
RESPUESTA
a.- Evaluar metanefrinas /24 hrs.
b.- Medicina nuclar con MIBG.
c.- Prueba de supresin con clonidina.
d.- Prueba de supresin con glucagon.
PREGUNTA
Posterior a la realizacin del estudio anterior, se observo una imagen
isointensa y continuaron los sntomas, cual es la conducta aseguir.
RESPUESTA
a.- Realizar bsqueda en cuello, torax y abdomen.
b.- Identificar CAS/CARS
c.- Supresion con Fluhidrocortisona.
d.- Realizar urografa excretora
CASO CLINICO
Masculino de 55 aos de edad, con los siguientes antecedentes:
alcoholismo ocasional semanal sin llegar a la embriaguez, tabaquismo
2 cigarrillos al da desde los 18 aos, hipertensin arterial de 5 aos de
evolucin en manejo con metoprolol 100mg cada 12horas. Refiere
presentar astenia de 3 semanas de evolucin y desde hace
una semana se agrega cefalea holocraneana opresiva, con acfenos y
fosfenos, sin disnea, angor u otro sntoma acompaante. Laboratorio:
glucemia 124, colesterol 320, triglicridos 129, urea 1, creatinina 0.8.
EF. FC 56x, FR 2Ox, TA 162/62, peso 98kg, talla 1.63m. Masculino
alerta, hidratado, orientado, adecuada coloracin tegumentaria, con
hemorragia subconjuntival derecha, campos pulmonares bien
ventilados sin sibilancias ni estertores, ruidos cardiacos de buen tono e
intensidad, abdomen blando, depresible, no doloroso, con peristalsis
normoactiva, extremidades con pulsos normales, llenado capilar

CURSO ENARM CMN SIGLO XXI TEL: 36246001

PREGUNTA
Si diramos a este paciente nifedipino, le aumentaramos el riesgo
de?
RESPUESTA
a.- Incremento en la mortalidad por aparicin de IAM
b.- Incremento en la mortalidad por aparicin de EVC
c.- Incremento de desarrollar insuficiencia cardiaca
d.- Incremento de desarrollar insuficiencia renal aguda
CASO CLINICO
Masculino de 44 aos, procedente y residente de Chiapas, empresario.
Padecimiento actual con ocho aos de evolucin, refiere palpitaciones
en forma progresiva relacionada con actividad fsica, cefalea
holocraneana sin periodicidad establecida y relacionada con alimentos
ricos en grasa y caf o chocolate, diaforesis nocturna y prdida de
aprox. 7 kg en los 2 meses previos a su ingreso, hipertensin arterial
paroxstica en los ltimos seis meses tratado con atenolol 50 mg/d,
hidroclorotiazida-enalapril 1 comp. C/12 h, as como hiperglucemia en
los ltimos seis meses tratada con pioglitazona. Signos vitales: TA
168/110 mmHg, FC 82 x, FR 22 x, T 37, peso 82 kg, talla 178 cm, IMC
26, Cr 1.5. Hidratado con actitud libremente escogida.
PREGUNTA
De acuerdo al cuadro clnico cual es el diagnstico ms probable?
RESPUESTA
a.- Hiperaldosteronismo primario
b.- Hipertension renovascular
c.- Hipertiroidismo
d.- Feocromocitoma
PREGUNTA
Para confirmar el diagnstico, que estudios le solicita al paciente?
RESPUESTA
a.- Renina y aldosterona plasmticas
b.- Angiorresonancia y radorrenograma
c.- T4 y TSH
d.- Catecolaminas urinarias y Ac. Vainillin mandelico
PREGUNTA
El tratamiento ms adecuado para este paciente es?
REPUESTA
a.- Bloqueadores alfa (prazozin)
b.- Bloqueadores beta
c.- Diurticos
d.- Hormonas tiroideas

Pharmed Solutions Institute

PGINA 9

MANUAL DE TRABAJO DEL CURSO ENARM CMN SIGLO XXI


INSUFICIENCIA CARDIACA CRONICA Y AGUDA:
CIENCIAS BASICAS: La Insuficiencia Cardaca Crnica (ICC) puede ser definida como un sndrome clnico complejo que proviene de anormalidades
cardacas estructurales y/o funcionales, adquiridas o hereditarias, que conllevan al empeoramiento de la capacidad de llenado y eyeccin ventricular a
una velocidad congruente, con la necesidad de los tejidos en plenos metabolismo. SALUD PUBLICA: Es la tercera causa ms frecuente de ingresos
hospitalarios (5% de ingresos urgentes) y la primera causa de ingresos por enfermedad cardiovascular. Hay 15 millones de pacientes con IC. Disfuncin
ventricular asintomtica 4%, prevalencia de la IC: 2-3% y aumenta drsticamente a los 75 aos de edad. 10-20% en el grupo de pacientes de 70-80.
PATOGENIA: Causas de deterioro funcional ms comunes: dao o prdida de musculo cardiaco, isquemia aguda o crnica, aumento de la resistencia
vascular con hipertensin, taquiarritmias como la FA, miocardiopatas, valvulopatas, frmacos (BBs, antagonistas de calcio, antiarrtmicos),
endocrinopatas (DM, hipo e hipertiroidismo, sx de Cushing, feocromocitoma). El dao miocrdico, deteriora la capacidad de vaciado del ventrculo en
sstole y ello incrementa el volumen teledistlico y al estar el corazn insuficiente se genera un menor volumen de eyeccin, sin embargo, hay etiologa
desconocida. La enfermedad coronaria causa la IC en cerca del 70-80% de los pacientes, enfermedad valvular es origen del 10% de los casos,
miocardiopatas 10%. Los mecanismos neurohumorales pueden causar efectos indeseables como: vasoconstriccin prolongada, aumento de poscarga,
hipertrofia ventricular, dilatacin ventricular, excesiva retencin de Na y H2O, arritmias. Activacin del SN simptico: produce vasoconstriccin,
incremento en poscarga, incremento en el consumo miocrdico de O2, hipertrofia ventricular esto activa el SRAA=incremento en precarga (incremento
en trabajo ventricular). PNA: se produce en respuesta a dilatacin auricular, es una hormona contrarreguladora, vasodilatacin, incremento en excrecin
de Na y H2O, sus niveles reflejan severidad de la IC. DIAGNOSTICO: Sndrome clnico, el paciente presenta: Sntomas tpicos de IC (disnea en reposo o con
el ejercicio, fatiga, edema) + signos tpicos de IC (taquicardia, taquipnea, estertores pulmonares, IY, edema perifrico, hepatomegalia) + Evidencia de una
anomalia estructural o funcional del corazn en reposo (cardiomegalia, 3er ruidos, soplos cardiacos, anomalias en ECG, elevacin de PNA). Criterios de
Framingham: Mayores; disnea paroxstica nocturna u ortopnea, IY, estertores, cardiomegalia, edema agudo de pulmon, galope por tercer tono, PVC
>16mmHg, tiempo de circulacin >25sg, reflujo hepatoyugular. Criterios Menores; edema maleolar, tos nocturna, disnea de esfuerzo, hepatomegalia,
derrame pleural, reduccion de la capacidad vital, taquicardia >120 lpm, perdida de mas de 4.5Kg en respuesta al tratamiento. Para diagnstico de IC se
precisan 2 criterios mayores o uno mayor y 2 menores, en los menores deben descartarse otras causas. En ECG podemos observar; arritmias, isquemia,
hipertrofia de VI, bloqueos, microvoltajes, >120 lpm. En Rx de torax; congestin pulmonar, cardiomegalia, derrames, tienen valor predictivo solo si se
asocia sntomas concomitantes de IC. Los pptidos natrireticos (BNP y pro-BNP; hasta 93pg/ml en hombres y hasta 144pg/ml mujeres) son
biomarcadores para dx y tx, si tienen una concentracin normal=se excluye la enfermedad, una concentracin elevada=mal pronostico. El
ecocardiograma confirma el dx. CLASIFICACION: La ACC/AHA (basada en la estructura y dao a msculo miocrdico) Estadio A: con alto riesgo de IC.
Anomala estructural o funcional no identificada; sin signos ni sntomas. Estadio B: enfermedad cardiaca estructural desarrollada claramente en relacion
con la IC pero sin signos ni sintomas. Estadio C: IC sintomtica asociada a enfermedad estructural subyacente. Estadio D: enfermedad cardiaca
estructural avanzada y sntomas acusados de IC en reposo a pesar de tratamiento mdico mximo. La NYHA (severidad basada en sntomas y actividad
fsica): Clase I; sin limitaciones de la actividad fsica, el ejercicio fisico normal no causa fatiga, palpitaciones o disnea, Clase II; ligera limitacin de la
actividad fsica, sin sntomas en reposo, la actividad fsica normal causa fatiga, palpitaciones o disnea. Clase III; Acusada limitacion de la actividad fisica,
sin sintomas en reposo, cualquier actividad fisica provoca la aparicion de los sintomas, Clase IV; incapacidad de realizar actividad fisica, los sintomas de la
IC estan presentes en reposo y aumentan con cualquier actividad fisica. Otras: IC de reciente aparicin; nueva sintomatologa. IC transitoria; sintomtica
durante un periodo limitado (IAM, miocarditis). IC sistlica; signos y sntomas con FEVI <40%. IC diastlica; signos y sntomas pero con FEVI conservada
(>40-50%). DIAGNOSTICO DIFERENCIAL: Estados que semejan la sintomatologa de IC por cursar con gasto elevado: anemia, tiortoxicosis, embarazo,
septicemia, insuficiencia heptica, cortocircuitos A-V, enfermedad de paget, beriberi. TRATAMIENTO: Autocontrol, adherencia al tratamiento: solo 2060% cumple Tx farmacolgico y no farmacolgico, reconocimiento de sntomas, control del peso, dieta: restriccin de Na, restriccin de lquidos, no
alcohol (inotrpico negativo, hipertensa, arritmias). IECA (1A): a todos los pacientes con IC sintomtica y una FEVI 40%, ya que mejora la funcin
ventricular, reduce los ingresos hospitalarios, mejora la supervivencia, reduciendo la mortalidad y el riesgo de IAM. En pacientes hospitalizados, se
iniciar el tratamiento con IECA antes del alta, solo se administrara en pacientes con funcin renal adecuada y concentraciones sricas normales, no dar
en angiodema o hipotensin sintomtica. BBs (1A): en todos los pacientes con IC sintomtica y una FEVI 40%, mejoran la funcin ventricular, reducen
los ingresos por descompensacin, aumentan la supervivencia y hay mejora en el bienestar del paciente. Antagonista de aldosterona (1B): con una FEVI
35% e IC grave y sintomtica, sin hiperpotasemia disfuncin renal significativa (clase III y IV de NYHA), reducen ingresos hospitalarios, aumenta
supervivencia cuando se aaden al IECA. ARA II: pacientes con IC y FEVI <40%, que siguen sintomticos a pesar de recibir tratamiento ptimo con IECA y
BBs, excepto cuando incluya antagonista de la aldosterona. Pueden causar empeoramiento de funcin renal, hiperpotasemia e hipotensin sintomtica.
Hidralazina + isosorbide en sintomticos con FEVI 40%, intolerancia a IECA y ARA. Digoxina en IC y FA; controla la FC, en pacientes con ritmo sinusal
con IC sintomtica y FEVI 40, mejora la funcin ventricular y el bienestar, reduce ingresos, pero no tiene efecto sobre la supervivencia. Diurtico:
recomendado en IC, alivio de los sntomas y signos de congestin venosa pulmonar y sistmica (edema, IY, crepitantes, hepatomegalia, derrame pleural).
Deben prescribirse en combinacin con IECA/ARA. PRONOSTICO: El 50% fallece a los 4 aos y el 40% de los pacientes ingresados por IC fallece o
reingresa durante el primer ao. COMPLICACIONES: Insuficiencia cardiaca congestiva, arritmias, muerte sbita, edema agudo pulmonar, etc.
INSUFICIENCIA CARDIACA AGUDA (ICA): Inicio rpido de signos y sntomas secundarios a una funcin cardiaca normal, hay un gasto cardiaco bajo,
hipoperfusin tisular, incremento en la presin capilar pulmonar y congestin pulmonar. Puede ocurrir con o sin patologa cardiaca previa.
Cronotropismo o automatismo: propiedad de algunas fibras cardiacas miocrdicas para excitarse as mismo de forma rtmica y automtica (nodo sinusal
y AV). Dromotropismo o conductividad: capacidad de transmitir potenciales de accin siguiendo la ley del todo o nada y coordinadamente mediante
un sistema de clulas especializadas. Inotropismo o contractibilidad: propiedad mediante la cual la fibra miocrdica desarrolla fuerza o tensin
permitiendo su acortamiento. Posibilita la funcin de la bomba y uno de los determinantes de gasto cradiaco. Batmotropismo o excitabilidad: es la
capacidad de despolarizarse ante la llegada de un estimulo elctrico. Precarga: preestiramiento de la fibra miocardica antes de la contraccin. Postcarga:
tensin transmural de la pared ventricular que debe desarrollar el ventrculo, para expeler el volumen sistlico y es igual a la resistencia contra la cual se
contrae. Causas y factores precipitantes de falla cardiaca aguda: 1. Descompensacin de falla cardiaca crnica preexistente. 2. Sindrome coronario agudo
(IAM). 3. Crisis hipertensiva. 4. Arritmia aguda (TV, FV, FA, TSV).5. Regurgitacin valvular. 6. Estenosis aortica severa. 7. Miocarditis aguda severa. 8.
Tamponade. 9. Diseccin aortica. 10. Factores precipitantes no cardiacos: frmacos, sobrecarga de volumen, infecciones, lesiones cerebrales severas,
ciruga mayor, asma, abuso de drogas. 12. Sndromes de gasto elevado: tirotoxicosis, sepsis, anemia, cortocircuito. La descompensacin puede ser
transitoria e irreversible o inducir dao permanente; se puede asociar a disfuncin sistlica o diastlica, anormalidades del ritmo cardiaco, alteraciones
de la precarga y poscarga. CLASIFICACION: Killip y Kimball. I. Sin datos de falla cardiaca: sin datos clnicos de descompensacin. II. Falla cardiaca:
estertores, S3, hipertensin venosa pulmonar, estertores hmedos en mitad inferior de pulmones. III. Falla cardiaca severa: edema pulmonar franco,
estertores hmedos en la totalidad de ambos pulmones. IV. Choque cardiognico: hipotensin, hipoperfusion, oliguria, cianosis y diaforesis. Clasificacin
de Forrester (describe la clnica y el estado hemodinmico en IAM). Grupo A: Caliente-seco (Normal, perfusin y presin pulmonar normales). Grupo B:
caliente- hmedo (Edema pulmonar, perfusin normal, presin pulmonar aumentada). Hay que usar diurticos, nitroglicerina, nitroprusiato. Grupo L:

CURSO ENARM CMN SIGLO XXI TEL: 36246001

Pharmed Solutions Institute

PGINA 10

MANUAL DE TRABAJO DEL CURSO ENARM CMN SIGLO XXI


fro-seco (Shock hipovolmico, hipoperfusion y presin pulmonar disminuida). Administrar fluidos. Grupo C: fro-humedo (choque cardiogenco,
hipoperfusion, presin pulmonar aumentada). Para reducir la presin inotrpicos o vasopresores. DIAGNOSTICO: Basado en sntomas y hallazgos
clnicos: disnea en reposo o con el ejercicio, fatiga, edema, taquicardia, taquipnea, estertores pulmonares, IY, edema perifrico, hepatomegalia.
Auscultacion de estertores, S3, S4, murmullo, soplos. Si encontramos un ECG, y gasometra anormal, congestin en tele de torax, PNA elevado,
enfermedad corazn conocido o crnica, se debe realizar una ecocardiograma para confirmar la falla cardiaca, de lo contrario podra ser una enfermedad
pulmonar. TRATAMIENTO: Mantener SO2 entre entre 95-98% para mantener una DO2 y prevenir una disfuncin organica mltiple. Administrar altas
dosis de O2 a pacientes no hipoxmicos, puede generar disminucin del flujo coronario y del GC, incremento en la TA y RVP. Para la oxigencion se debe
considerar el CPAP, o NIPPV, ventilacin mecnica. Morfina: en etapas tempranas en asociacin con agitacin y disnea, induce venodilatacion y ligera
dilatacin arterial, reduce FC. Diureticos: si hay datos de congestin y sobrecarga hdrica. Vasodilatadores: primera lnea si hay normotensin y signos de
congestin, ya que disminuyen la precarga. Calcioantagonistas: no recomendados, si hay hipotensin estn contraindicados. Nitratos: disminuyen
congestion pulmonar, sin disminuir volumen, ni incrementar MVO2, producen diltacion arterial, disminuyen precarga del VI. Inotropicos: usar en
presencia de hipoperfusion y congestin como dobutamina; inotrpico positivo -1, iniciar con 2-3mg/kg/min hasta 15. Vasopresores: en choque
cardiogenico, si hay hipotensin y datos de bajo gasto que no responden a manejo con lquidos e inotrpicos, epinefrina no es recomendada. Digoxina:
refuerza, retarda, regula, en pacientes con IC sintomtica y FA, controla la FC, por ello esta inidicada como 1ra eleccin, impregnacin 10-15mcg/kg, 3
dosis, primera dosis; 50%. Mantenimiento 125-300mcg/da en 2 dosis, reacciones adversas, ginecomastia, bloqueos, disminucin de libido, bradicardia,
arritmias, en primeros sntomas de intoxicacin, tambin nasea, vmito, diarrea, confusin, mareo, fatiga, visin borrosa, fotofobia, xantopsia (ver
amarillo). En ECG el desnivel negativo del segmento, es un signos de accin digitalica o cubeta digitalica, no correponde a intoxicacin.
CASO CLINICO
Femenino de 73 aos de edad la cual cuenta con 20 aos de
hipertensin arterial y diabetes mellitus, con tratamiento mdico pero
mal apego farmacolgico y no farmacolgico, actualmente se
encuentra con el siguiente esquema hidroclorotiazida 100mg/d,
furosemida 20mg/d, enalapril 60mg/d, amlodipino 10mg/d, acido
acetilsalicilico 100mg/d, glipizida 20 mg dividido en dos dosis,
actualmente acude por mareo ocasional con cada de su altura en dos
ocasiones, se ha desorientado dos veces desconociendo a la personas.
A la exploracin fsica se observa paciente orientada, adecuado estado
de alerta, con leve desihratacion con facies pletricas, se observa
ingurgitacin yugular de 5 cm, campos pulmonares con estertores de
predominio en las bases, se escucha desdoblamiendo del segundo
ruido cardiaco, leve hepatomegalia, miembros inferiores con edema
+++, los cuales se encuentran calientes. Signos vitales: TA
105/80mmHg, FC 109 lpm, FR 31rpm. Se calculo una FE >45%.
PREGUNTA
Cules son las manifestaciones clnicas ms especificas para
establecer el diagnostico de la enfermedad?
RESPUESTA
a.- Ortopnea, disnea y tos noctura.
b.- Edema, fatiga y congestion torcica.
c.- Electrocardiograma y radiografia de torax.
d.- Ingurgitacion yugular, hepatomegalia y fraccin de eyeccin.
PREGUNTA
Que de Framingham para el dignosticos de IC tiene este paciente?
RESPUESTA
a.- 4 mayores y 2 menores.
b.- 4 mayores y 3 menores.
c.- 3 mayores y 2 menores.
d.- 3 mayores y 3 menores.
PREGUNTA
Cul es el factor de riesgo que puede conducir a un estado
descompensado en este caso?
RESPUESTA
a.- Incumplimiento del tratamiento.
b.- SAOS.
c.- Arritmia, anemia hemorragia.
d.- Antiarritmico de clase Ia y Ic.

PREGUNTA
Cules son los auxiliares diagnosticos mas tiles que no han sido
realizados en el paciente para identificar las complicaciones de la
enfermedad?
RESPUESTA
a.- Pruebas de funcin heptica.

CURSO ENARM CMN SIGLO XXI TEL: 36246001

b.- Electrolitos sericos.


c.- Pruebas de estrs.
d.- Ecocardiograma.
PREGUNTA
Cul
es
la
medida
farmacolgica
que
intervendr
predominantemente sobre la respuesta neurohumoral en la retencin
de sodio y liquidos?
RESPUESTA
a.- Sistema renina-angiotensina-aldosterona.
b.- Sistema arginina-vasopresina.
c.- Liberacin de pptido natriuretico auricular.
d.- Inhibe el sistema de transporte Na + Cl -.
PREGUNTA
Cual de los siguientes factores no es determinante del consumo de
oxigeno miocardico?
RESPUESTA
a.- Estrs parietal.
b.- Contractilidad.
c.- Frecuencia cardiaca.
d.- Fraccin de eyeccin.
PREGUNTA
Antes de definir el tratamiento farmacolgico. Cual de las causas
reversibles es menos probable se encuentra en el caso?
RESPUESTA
a.- Depresin miocrdica inducida por alcohol.
b.- Antiinflamatorio no esteroideo.
c.- Estado de poscarga elevado.
d.- Uso de bloqueador de canales de calcio.
PREGUNTA
Cual de las siguientes medidas no es la ms recomendable para el
actual caso clnico?
RESPUESTA
a.- Dieta con 5 a 6 g. de sal al da.
b.- Dieta sin sodio absoluta.
d.- Programa de gradual de ejercicio.
c.- Dieta normokalemica.
PREGUNTA
Cosiderando el estado actual de velocidad de filtrado glomerular del
paciente. Cual de los siguientes diureticos proporcionan un equilibrio
adecuado de natriuresis y caliuresis?
RESPUESTA
a.- Espironolactona.
b.- Hidroclorotiazida.
c.- Furosemida.
d.- Clortalidona.

Pharmed Solutions Institute

PGINA 11

MANUAL DE TRABAJO DEL CURSO ENARM CMN SIGLO XXI


PREGUNTA
Considerando el esquema farmacolgico actual del paciente que
medida farmacolgica es la mas adecuada para promover un menor
ndice de morbilidad y mortalidad?
RESPUESTA
a.- Incrementar la dosis de enalapril.
b.- Aadir losartan.
c.- Iniciar esquema con digoxina.
d.- Iniciar sildenafil.
PREGUNTA
Cul de los cambios neurohumorales que se presentan en la
insuficiencia cardiaca es menos probable en este caso?
RESPUESTA
a.- Aumento de la contractibilidad miocrdica.
b.- Aumento de la frecuencia cardiaca.
c.- Aumento del tono venoso.
d.- Disminucin del volumen sanguneo central efectivo.
PREGUNTA
Cul de los cambios funcionales que se presentan en el paciente con
insuficiencia cardiaca es menos probable en este caso?
RESPUESTA
a.- Volumen sistlico reducido.
b.- Poscarga excesiva.
c.- Depresin de la contractibilidad.
d.- Disminucin del volumen telediastolico.
CASO CLINICO
Masculino de 43 aos de edad el cual reingresa a urgencias debido a
dificultad respiratoria, disnea de minimos esfuerzos, tos incluyendo en
la noche, adems de dolor toracico cuando se presenta la disnea, este
dolor disminuye muy lentamente con el reposo, el paciente cuenta con
el antecedente de IAM egresando 15 dias antes de este evento. Se
observa reflujo hepatoyugular con hepatomagalia, se ausculta tercer
ruido. Sus contantes vitales son: IMC 31, FC 119 lpm, FR 31, TA 140/70
mmHg,
PREGUNTA
Cuales son los criterios de framingham presenta el paciente en este
momento?
RESPUESTA
a.- 3 menores y 4 mayores.
b.- 4 menores y 4 mayores.
c.- 3 menores y 3 mayores.
d.- 4 menores y 2 mayores.
PREGUNTA
Cual es condicin que se presenta tomando la clasificacin de killip y
Kimball.
RESPUESTA
a.- Clase funcional I.
b.- Clase funcional II.
c.- Clase funcional III.
d.- Clase funcional IV.
CASO CLINICO
Ingresa paciente de 39 aos de edad el cual ingresa a urgencias 4 horas
antes, por presencia sbita de dolor retrosternal acompaado de
presin toracicca sensacin de mucho peso sobre su pecho que se
presento en la madrugada, nausea, dificultad para respirar, refiere que
la noche anterior comio abundante y consumio alcohol hasta la
embriaguez, al ingreso se observa confuso, diafortico con palidez y
cianosis distal, ingurgitacin yugular, se asculta estertores cripitantes
bibasales, tercer ruido, TA 90/60 mmHg, FC 132 lpm, FR 36 rpm, Temp.
36.3 grados. Gasto urinario de < 20 ml/h, se observaron los siguientes
estudios.

CURSO ENARM CMN SIGLO XXI TEL: 36246001

PREGUNTA
Cual es la clase funcional de Killip que presenta el caso?
RESPUESTA
a.- Clase funcional Killip I.
b.- Clase funcional Killip II.
c.- Clase funcional Killip III.
d.- Clase funcional Killip IV.
PREGUNTA
Cual es el pronstico para la vida en este caso considerando la clase
funcional?
a.- Tasa de mortalidad del 6%.
b.- Tasa de mortalidad del 17%.
c.- Tasa de mortalidad del 38%.
d.- Tasa de mortalidad del 81%
PREGUNTA
Se calculo los siguientes resultados (GC) de 2 L/m2/min. Considerando
los elementos clnicos y el GC, en la curva de Starling. Cual es el
diagnostico?
RESPUESTA
a.- Insuficiencia cardiaca.
b.- Edema agudo pulmonar.
c.- Choque cardiogenico.
d.- Choque distributivo
CASO CLINICO
Se trata de masculino de 62 aos de edad diabtico e hipertensin
controlado con glibenclamida y enalapril a dosis minimas toxicas
respectivamente, fue fumador hasta los 45 aos. Hace 5 aos presento
infarto al miocardio tratado con trombolitico con buena respuesta,
acude a consulta debido cansacio, fatiga y dolor toracicco opresivo que
sede con el reposo, ocasionalmente el dolor toraccico es ardoroso o
urente, adems de palpitaciones ocacionales, disnea de medianos
esfuerzos, ortopnea as como fatiga, a la EF presenta ingurgitacin
yugular, frote carotideo, area cardiopulmonar con estertores
crepitantes finos, palidez de tegumento, cianosis distal, miembros con
edema de miembros inferiores, TA 140/70 mmHg, FC 93 lpm, FR 26
rpm, IMC 29.5.
PREGUNTA
Se realizarn los siguientes estudios de rutina hemoglobina glucosilada
8, creatinina 0.5, acido urico 7.0 mg/dl, colesterol HDL 27 mg/dl,
triglicridos 210 mg/dl, considerando el estado actual de caso clnico.
cual es la conducta a seguir?
RESPUESTA
a.- Solicitar BNP NT-proBNF.
b.- Solicitar ecocardiograma transesofagico.
c.- Solicitar ECG en reposo y holter.
d.- Solicitar prueba de estrs.
PREGUNTA
Cual es la conducta teraputica a seguir para disminuir el riesgo
cardiometabolico?
RESPUESTA
a.- Iniciar hidroclorotiazida
b.- Iniciar verapamilo.
c.- Iniciar betabloqueador.
d.- Iniciar atorvastatina
PREGUNTA
Se realizo ecocardiograma que reporta una FE > 45 %, el
electrocardiograma reporta extrasstoles ventriculares. cual es la
conducta farmacolgica mas adecuada para disminuir el remodelado
ventricular y mejorar la actividad elctrica cardiaca?
RESPUESTA
a.- Iniciar betabloqueador.

Pharmed Solutions Institute

PGINA 12

MANUAL DE TRABAJO DEL CURSO ENARM CMN SIGLO XXI


b.- Iniciar verapamilo.
c.- Iniciar losartan.
d.- Iniciar diltiazem.

c.- Hemodialisis.
d.- Soluciones metabolizadas.

PREGUNTA
El paciente se mantiene en buenas condiciones sin embargo 12 meses
despus acude a cita de control, se realizo electrocardiograma de
control, donde se apresia bloqueo de primer grado, el paciente agrega
que la monitorizacin ambulatoria de la presin fue adecuada, sin
embargo la opresin toracica se ha presentado de forma espontanea y
con mayor duracin, asi como disnea, mas frecuente. Cual es el
frmaco mas adecuado?
RESPUESTA
a.- Isosorbide 10 mg.
b.- Verapamilo 80 mg.
c.- Diltiazem 60 mg.
d.- Nitroglicerina 18 mg.
CASO CLINICO
Se trata de paciente femenino de 67 aos de edad, que acude a
consulta por cansancio, fatiga, dificultad para respirar, dolor toracico
que disminuye con reposo, disnea de moderados esfuerzos, edema de
miembros inferiores hasta 2/3 de la pierna, a la exploracin fsica se
observo palidez de piel y mucosas, llenado capilar lento, estertores
crepitantes leves, presencia de galope y trill. La paciente es hipertensa
desde hace 20 aos con moderado apego a tratamiento con captopril
50 mg/dia, hidroclorotiazida 25 mg/12 hrs. Ha presentado 2 crisis
hipertensivas, asi como un evento de insuficiencia mesentrica, sus
laboratorios EGO protenuria, glucosuria, uratos, densidad urinaria
disminuida, colesterol 289 mg/dl, triglicridos 720 mg/dl, creatinina
6.2 mg/dl, BUM 29, kalemia de 5.3, los signos vitales fueron TA
160/105 mmHg, FC 96, FR 28, IMC 32.
PREGUNTA
Se realizo un ecocardiograma donde se observa una FE de <40 %,
durante su ingreso la paciente presento volumen urinario de 20 ml/hr.
Considerando las caractersticas actuales cual es la clase funcional
que presenta el caso?
RESPUESTA
a.- Clase funcional I.
b.- Clase funcional II.
c.- Clase funcional III.
d.- Clase funcional IV.
PREGUNTA
Considerando las caractersticas actuales. Cual es el mtodo mas
adecuado para determinar el estadio de la IRC?
RESPUESTA
a.- Depuracion de creatinina de 24 hrs.
b.- Biopsia renal.
c.- Ultrasonografia renal.
d.- Urografia escretora.
PREGUNTA
Cual es la conducta farmacolgica ms adecuada considerando la
clase funcional?
RESPUESTA
a.- Verapamilo.
b.- Diltiazem.
c.- Telmisartan.
d.- Isosorbide.
PREGUNTA
Cual es la conducta inmediata a seguir para mejorar la precarga
disminuyendo los niveles de potasio?
RESPUESTA
a.- Furosemide.
b.- Dialisis peritoneal.

CURSO ENARM CMN SIGLO XXI TEL: 36246001

PREGUNTA
Cual de las siguientes patologias es la principal causa de IRC?
RESPUESTA
a.- Diabetes mellitus.
b.- Hipertensin arterial.
c.- Glomerulonefritis.
d.- Enfermedad renal poliqustica.
PREGUNTA
Cuales factores de riesgo aceleran el deterioro del funcionamiento
renal en el caso actual?
RESPUESTA
a.- Dislipidemia.
b.- Edad avanzada.
c.- Diabetes mellitus.
d.- Anemia.
CASO CLINICO
Masculino de 59 aos de edad, ingresa a urgencias debido a dificultad
para respirar, cansancio, fatiga, tos productiva desde hace 15 dias
intensificandoce la noche previa al ingreso actual, cuenta con
antecedentes de EPOC diagnosticado hace 10 aos, posterior a
tabaquismo, con tratamiento de bromuro de iprapropio, salbutamol y
betametasona, adems fue diagnosticado hipertenso hace 2 aos,
inicio con cambios del estilo de vida y dieta con restriccin de sodio,
captopril que fue cambiado 3 meses despus por incremento de tos,
desde entoces toma losartan, hidralazina. A la EF se observa facies
pletrica con hiperemia conjuntival, mucosas orales cianticas
moderadamente, se ausculta un soplo carotideo derecho,
ingurgitacin yugular grado II, estertores subcrepitantes bibasales, con
resonancia tipanica a la percusin, los ruidos cardiacos levemente
disminuidos, rtmico, abdomen con hepatomegalia 5 cm debajo del
borde costal.
PREGUNTA
Considerando el estado actual del paciente cual es la conducta mas
apropiada a seguir, para establecer la funcin cardiopulmonar?
RESPUESTA
a.- Espirometria.
b.- Ecocardiograma.
c.- Valor de natriuretico.
d.- Radiografia de torax.
PREGUNTA
Luego de realizar las pruebas anteriores se observo una FE <40 %, la
Rx. torax mostro un indice cardiotoracico patolgico, exudados
algodonosos bilaterales, trama brocovascular incrementada, gases con
PCO 49, PO2 85, potasio 6.3, Factor natriuretico disminuido, a la
exploracion fsica se auscultan estertores crepitantes, sibilancias
moderadas, a la percusin hiperresonancia, se alcanza apresiar galope,
cual es la conducta mas apropiada a seguir para disminuir la precarga,
considerando todos los factores?
RESPUESTA
a.- Retirar diurtico ahorrador de potasio por diurtico de asa.
b.- Iniciar con calcioantagonista dihidropiridinico de accin rpida.
c.- Incrementar la capacitancia vascular perifrica con nitritos.
d.- Restriccion hdrica, dieta libre de sodio, diurtico de asa.
CASO CLINICO
Se ingresa a urgencias a paciente femenino de 48 aos de edad,
originaria de Veracruz, empleada de agrcola, casada desde los 17
aos, gesta 5, para 3, aborto 1, cesarea 1, histerectoma a los 38 por
miomatosis uterina, no fuma ni toma bebidas alcoholicas,
aparentemente sana, no toma medicamentos, anteriormente presento
caries dentales pero fueron tratadas hace 7 meses sin embargo hasta

Pharmed Solutions Institute

PGINA 13

MANUAL DE TRABAJO DEL CURSO ENARM CMN SIGLO XXI


hace 3 meses que comenz padecimiento actual caracterizado por
fatiga, disnea de medianos esfuerzos, sensacin de ahogamiento de
predominio nocturno, es ingresada debido a que presenta tos con
esputo fino color rosa, disnea de leves esfuerzo, a la exploracin fsica
se observa paciente con estado grave, edematizada con cianosis
central y perifrica, ingurgitacin yugular, soplo mediodiastolico III/IV
el cual predomina en posicin erecta, edema de miembros plvicos
con lesiones isqumicas en dedos 1 y 2 de pie derecho, tratado y
contaminado con remedios locales, no se apercibe olor ni pus, los
signos vitales son los siguientes TA 125/85 mmHg, FC 102, FR 26,
Temperatura 37.5 C. La paciente recibe diagnostico de insuficiencia
cardiaca congestiva.
PREGUNTA
Cul es la causa ms probable para la insuficiencia cardiaca
congestiva de este caso?
RESPUESTA
a.- Endocarditis bacteriana con insuficiencia valvular.
b.- Estenosis de la valvula mitral.
c.- Mixioma auricular.
d.- Melanoma metastasico
PREGUNTA
Se estabiliza a la paciente y se realizan estudios de gabinete donde se
identifica una masa tumoral intracavitaria de 6 centmetros, dentro de
los estudios se realizaron TAC, IRM y cateterismo, 4 horas despus la
paciente presenta dificultad respiratoria intensa, presencia de tos
abundante con esputo asalmonelado, por lo que se intuba para
soporte ventilatorio, se observa cianosis perifrica con prdida de
pulsos distales de miembros plvicos, ausencia de ruidos abdominales
seguido de parada cardiaca en dos ocasiones, declarando su muerte 2
horas ms tarde. Cul fue la intervencin ms probable que culmino
con la embolia masiva en esta paciente?
RESPUESTA
a.- No se anticoagulo profilcticamente.
b.- El cateterismo cardiaco innecesario.
c.- El desprendimiento del mixioma.
d.- El paciente ya tena signos de embolia.
CASO CLINICO
Masculino de 74 aos, exfumador, DM tipo 2 (15 aos),
Hipercolesterolemia 10 aos, IAM anterior (5 aos). Enfermedad de
dos vasos no susceptible de revascularizacin. Ultimo cateterismo (6
meses) sin cambios. FE 35%. Ingresos repetidos por IC. Consulta por
disnea invalidante y aumento del edema en MMII, de instauracin
progresiva y 5 das de evolucin. No dolor torcico. No fiebre ni clnica
de infeccin respiratoria. No cambios en el tratamiento. Exploracion
fsica: Consciente y orientado. Marcado incremento del trabajo
respiratorio. Sat O2 83%. PA 95/55 mmHg; Fc 85 lpm, rtmico; 26 rpm,
Ingurgitacin yugular. Crepitantes diseminados en ambos campos,
espiracin prolongada, soplo sistlico politpico II-III/IV. Edema hasta
rodillas en MMII. Hemograma: Hb 11,2 g/dl. BNP 1600 pg/ml,
Troponina T normal.
PREGUNTA
De acuerdo al cuadro clnico del paciente Cul es la conducta
teraputica inicial ms indicada?
RESPUESTA
a.- Oxigenoterapia, IECA, lquidos parenterales
b.- Oxigenoterapia, diurtico, nitroglicerina
c.- Oxigenoterapia, diurtico, vasodilatadores
d.- Oxigenoterapia, BBs, vasopresores

Las cifras de BNP obtenidas en este paciente nos indican:


RESPUESTA
a.- Mal pronostico
b.- Nos excluye enfermedad
c.- Edema agudo pulmonar
d.- Insuficiencia cardiaca aguda
PREGUNTA
Al dar de alta a este paciente, qu medicamentos son indispensables
para reducir la mortalidad y los ingresos hospitalarios?
RESPUESTA
a.- Dihidropiridinas, diurtico
b.- Diurtico, IECA
c.- IECA, digoxina
d.- IECA, betabloqueador
CASO CLINICO
71 aos de edad, sexo femenino, 53 kg de peso, tratada por
insuficiencia cardiaca crnica durante tres aos, fue referida al hospital
por su mdico por sospechas de exacerbacin de disnea. Tres meses
atrs comenz a quejarse de cansancio progresivo, dificultad para
dormir, tos nocturna y falta de aliento tanto al caminar como al
realizar sus actividades diarias. Tambin indic periodos de malestar
en el pecho, el cual describi como palpitaciones en vez de opresin
o dolor. Posee antecedentes de fibrilacin auricular e hipertensin.
Medicamentos actuales: Hidroclorotiazida 25mg da, Furosemida 40mg
da, Enalapril 10 mg da. Al examinarla estaba plida y su temperatura
era de 37.3 C, pulso irregular de 130 latidos por minuto, y su P.A.
(acostada) era de 155/89 mmHg. Su pulso venoso yugular se elev 3cm
y presentaba edema con fvea en ambos pies y tobillos, y
crepitaciones pulmonares. Un ECG demostr fibrilacin auricular
rpida sin cambios isqumicos o signos de infarto. Un ecocardiograma
confirm una disfuncin ventricular sistlica izquierda con una fraccin
de eyeccin del 35%. Rx. De torax: cardiomegalia
PREGUNTA
De acuerdo a la clasificacin de NYHA, que grado de insuficiencia
cardiaca presenta la paciente?
RESPUESTA
a.- NYHA I
b.- NYHA II
c.- NYHA III
d.- NYHA IV
PREGUNTA
Cul medicamento sera el ms indicado en esta paciente para
mejorar su FA?
RESPUESTA
a.- Losartan
b.- Metoprolol
c.- Digoxina
d.- Hidroclorotiazida
PREGUNTA
Qu criterios de Framingham mayores cumple esta paciente para
diagnstico de insuficiencia cardiaca en este momento?
RESPUESTA
a.- Cardiomegalia, IY, estertores
b.- Cardiomegalia, tos nocturna, taquicardia
c.- Estertores, disnea de esfuerzo, tos nocturna
d.- Taquicardia, IY, estertores.

PREGUNTA

CURSO ENARM CMN SIGLO XXI TEL: 36246001

Pharmed Solutions Institute

PGINA 14

MANUAL DE TRABAJO DEL CURSO ENARM CMN SIGLO XXI


CARDIOPATIAS CONGENITAS (ADULTO)
DEFINICION: La incidencia de las formas moderadas a severas de las cardiopatas congnitas es de 6 por 1000 nacidos vivos, los pacientes sin manejo
farmacolgico y/o quirrgico temprano no llegan a la vida adulta, ya que con el tratamiento adecuado el pronostico es de 85% para llegar a la vida
adulta, la mayor parte de los adultos con cardiopatas congnitas no requieren tratamiento llegando a la vida adulta. Los defectos mas frecuentes son 1.defecto septal ventricular estrecho, 2.- defecto artrial septal secundario, 3.- estenosis pulmonar leve a moderada, 4.- valva aortica bicuspidea, 5.prolapso de valvula mitral. DIAGNOSTICO: Los adultos con cardiopatas congnitas presentan tres formas bsicas: con una historia de ciruga previa
paliativa o reparativa durante la infancia, 2.- con conocimiento de la cardiopata congnita sin intervencin, 3.- presentacin en la vida adulta sin
antecedentes de la infancia del padecimiento. La excepcin se presenta en los pacientes con persistencia de conducto arterioso leve. Cianosis en el
adulto debe buscar la causa: Eritrocitosis, deficiencia de hierro, sangrado, o cardiovascular. Cuando se presentan las siguientes alteraciones: Alteracion
de la funcin ventricular (es la mas frecuente que presente sntomas), arritmias (las atriales es la mas frecuente, en las ventriculares son secundarias a
fibrosis, dilatacin ventricular, y son causas de muerte sbita). Enfermedad por conduccin, (intriseco o posoperatorio de enfermedad de nodo SA con
disfuncin) se puede observar fraccin baja de eyeccin, disincronia, puede causar bloqueos. DEFECTO ATRIAL SEPTAL: 75 % es defecto secundario a
ostium encontrado en el adulto, se observa murmullo y alteraciones inespecficas de RX y ECG, presencia de segundo ruido, sobrecarga ventricular
derecha crnica, hipertensin pulmonar, arritmias atriales, raramente embolizacion paradoxica). Tratamiento: Especifico por alteracin. PREVENCION:
Endocarditis (es mas frecuente cuando hay antecedentes de cardiopata congnita), Embarazo (cuando hay antecedentes de lesiones estenoticas severas
pulmonar, hipertensin pulmonar, shunts derecha a izquierda, regurgitacin, vigilancia estrecha por riesgo de endocarditis durante el embarazo y
sobrecarga a las 20 a 22 SDG indicado ecocardiografia), Anticoncepcion (Los Estrogenos incrementan mas los riesgos para tromboembolico, usar
contracepcin con progestgenos); Ejercicio (identificar taquicardia inducida, evaluar la competencia cronotropica, se contraindica el ejercicio intenso
isomtrico). DEFECTO SEPTAL VENTRAL: Existen 4 tipos, la perimembranosa es la mas comn, la muscular es comn en la infancia con cierre
expontaneo, puede presentar murmullo holosistolico inversamente relacionado al tamao del defecto, pueden desarrollar el complejo Eisenmenger al
no tratarse. TRATAMIENTO: al desarrollar hipertensin pulmonar puede estar indicado sindanafil o anlogos de prostanglandinas. DEFECTO SEPTAL
ATRIOVENTRICULAR: Se presenta lesiones en los septos atriales y ventriculares clasificados en parcial o complejo, se acompaa comnmente de
subestenosis aortica, puede coexistir regurgitacin mitral. El ECG puede mostrar desviacin a la izquierda del eje, y bloqueo incompleto, lesin frecuente
en sndrome de down, requiere reparacin antes de los 6 meses de edad para prevenir la hipertensin pulmonar y consecuentemente el complejo
Eisenmenger. TETRALOGIA DE FALLOT: Es la mas comn de las cardiopatas congnitas cianogenas representa del 7 al 10 %, difcilmente paciente sin
antecedentes solo en forma minimas, pueden presentar alteraciones de las intervenciones realizadas durante la vida adulta. Las arritmias son las
manifestaciones mas frecuentes en la vida adulta, junto a la disfuncin ventricular derecha, Tratamiento con marcapaso y defibrilador interno.
ESTENOSIS PULMONAR AISLADA: Presentan estenosis infundivular subpulmonar pudiendo sobrevivir a la vida adulta, presente en sx de noonan,
Tratamiento con valvuloplastia con baln. OBSTRUCCION DEL TRACTO DE SALIDA DEL VENTRICULO IZQUIERDO: Obstruccion valvular bicuspidea,
subvalvular o supravalvular, puede presentarse afecciones en multiples niveles. COARTACION DE LA AORTA: Presente en el adulto con menor posibilidad
de reparacin, se presenta predominantemente distal al inicio de la subclavia izquierda, es silenciosa hasta que se presenta un evento sbito como
hipertensivo. TRANSPOSICION DE GRANDES ARTERIAS: La sobrevida a la vida adulta es casi imposible, se realiza un swich de vasos en la infancia, 50 %
de los casos desarrollan trasntornos sinusales, arritmias, regurgitacin valvular, falla ventricular derecha.
CASO CLINICO
Posteriormente a la correccin de la tetraloga de Fallot realizada con
19 aos de edad, con ampliacin del tracto de salida del ventrculo
derecho, valvotoma pulmonar y cierre de la comunicacin
interventricular, deliberadamente quedaron como defectos residuales,
la estenosis pulmonar al nivel valvular por un pequeo anillo pulmonar
y la comunicacin interatrial. Pero incluso as, permaneci con una
buena evolucin por 28 aos cuando surgi la primera complicacin
por embolia cerebral paradjica con un discreto accidente
cerebrovascular, sin consecuencias sensoriales o motoras. Adems,
hace un ao se suscit una complicacin arrtmica por fibrilacin atrial
con frecuencia cardaca elevada, que fue revertida con amiodarona y
choque elctrico. Recientemente, una nueva fibrilacin atrial, pero
ahora con una frecuencia ventricular baja (entre 50 a 60 lpm), y
insaturacin arterial (70%), posteriormente a la deambulacin en
pocos minutos, motiv la investigacin diagnstica y una conducta ms
precisa. Al examen fsico, el paciente se present en un buen estado
general, eupneico, enrojecido, con pulsos normales, pesando 114 kg,
altura de 170 cm, PA de 110/80 mmHg y FC de 60 lpm en ritmo de
fibrilacin atrial. La aorta se palpaba discretamente en la frcula. En el
precordio, no haba impulsos o deformidades y el ictus cordis no se
palpaba. Los ruidos eran hipofonticos y se auscultaba un soplo
sistlico, +/++ de intensidad, de eyeccin, rudo, en el 1, 2 y 3
espacios intercostales izquierdos en el borde esternal y en la frcula. El
hgado no se palpaba.
PREGUNTA
Cual de las siguientes condiciones es la ms probable causa del
estado actual del paciente?
RESPUESTA
a.- Estenosis pulmonar valvular moderada e hipertrofia del ventrculo
derecho
b.- Shunt de sangre de la derecha hacia la izquierda por comunicacin
interatrial

CURSO ENARM CMN SIGLO XXI TEL: 36246001

c.- Defectos residuales posteriores a la correccin de la tetraloga de


Fallot
d.- Insaturacin arterial grave a pequeos esfuerzos.
CASO CLINICO
Mujer de 69 aos, intervenida a los 48 aos por CIA tipo ostium
primum con disnea de medianos esfuerzos. Se cerr quirrgicamente
el defecto con parche de tefln y se corrigi, mediante sutura, la
hendidura de la valva anterior mitral. Precis implante de marcapasos
VVI por bloqueo auriculoventricular completo. Permaneci
asintomtica hasta 22 aos despus, cuando present dolor torcico
inespecfico y una prueba de esfuerzo interrumpida por la disnea.
Ecocardiogrficamente presentaba un ventrculo izquierdo hipertrfico
con funcin conservada e hipertensin pulmonar severa. La vlvula
mitral estaba fibrosada, con insuficiencia moderada e insuficiencia
artica ligera. Presentaba un gradiente sistlico fijo en el tracto de
salida del ventrculo izquierdo (TSVI) de 84 mmHg, ocasionado por una
membrana, que pareca partir de la valva anterior mitral. En el
cateterismo cardaco se diagnostic una estenosis severa del tronco de
la coronaria izquierda.
PREGUNTA
La ESA tras ciruga correctora de CIA de tipo ostium primum es
infrecuente, considerando las datos previos referidos cual de las
siguientes afirmaciones es menos probable?
RESPUESTA
a.- Afecta al 2-15% de los casos
b.- Suele diagnosticarse 6-8 aos tras correccin del defecto en la
infancia.
c.- Falta de seguimiento postoperatorio en adultos por evolucin ms
lenta.
d.- Su incidencia es de 60-70% y permanece silente durante pocos
aos.

Pharmed Solutions Institute

PGINA 15

MANUAL DE TRABAJO DEL CURSO ENARM CMN SIGLO XXI


CASO CLINICO
Una mujer de 43 aos de edad. Su historial mdico incluye la
reparacin de una transicin atrioventricular (AV), comunicacin
interventricular (D-AV) a la edad de 14 aos, que consiste en un ostium
primum AVSD y musculoso comunicacin interventricular restrictiva
(VSD), asociado con un velo anterior mitral hendida. Despus de su
ciruga, ella permaneci asintomtica hasta que tena 34 aos, cuando
experiment la fatiga progresiva. Una ecocardiografa transtorcica
mostr shunt residual, tricspide moderada e insuficiencia artica,
insuficiencia mitral moderada y estenosis subartica con un tracto de
salida del ventrculo izquierdo fijo (TSVI) gradiente pico de 52 mm Hg.
La estenosis subartica se agrav en los ltimos aos, alcanzando un
mximo gradiente sistlico de 100 mm Hg. Permaneci activa, pero
experiment episodios diaforticas ms frecuentes con actividad leve.

funcional III de la NYHA y la ecocardiografa puso de manifiesto presin


suprasistmica en el ventrculo derecho, comunicacin interventricular
residual pequea con cortocircuito bidireccional e insuficiencia
tricuspdea grado III/IV.

PREGUNTA
Cual de las siguientes alteraciones favorece ms la condicin actual
de ESA?
RESPUESTA
a.- Crestas subartica fibromuscular.
b.- Anormales vlvula AV izquierda.
c.- Cuerdas tendinosas de los archivos adjuntos.
d.- Insercin anmala de los msculos papilares del ventrculo
izquierdo y generalizada hipoplasia del LVOT.

CASO CLINICO
Mujer de 28 aos con atresia pulmonar y comunicacin
interventricular intervenida quirrgicamente a los 4 aos de edad,
practicndosele fstula de BT izquierdo clsica. A los 6 aos se le realiz
anastomosis de WC por estenosis de la fstula. A los 14 aos
presentaba cianosis progresiva severa y disnea de esfuerzo. El
cateterismo cardaco demostr estenosis severa de la fstula de BT e
hipoplasia de las arterias pulmonares, descartndose para ciruga
correctora. A los 26 aos, la paciente presentaba cianosis severa,
acropaquias y disnea de mnimos esfuerzos, decidindose la
realizacin de una nueva fstula paliativa, que no pudo completarse
por la aparicin de sangrado muy abundante durante la diseccin
quirrgica. Mediante angiografa se demostr obstruccin completa de
la fstula de BT en su tercio distal y estenosis severa de la anastomosis
de WC.

CASO CLINICO
Masculino de 20 aos con soplo cardaco, cianosis y cansancio
observados desde el nacimiento, con ntida acentuacin en los ltimos
aos. Sin control mdico, hematocrito de 83% y hemoglobina de 28 g.
Hemafresis repetidas disminuyeron el hematocrito para 73%.
Ninguna medicacin especfica haba sido administrada desde el
nacimiento. Examen fsico: Regular estado general, eupneico, cianosis
acentuada, pulsos normales. Peso: 42,6 Kg; Altura: 160 cm; PA: 105/65
mmHg; FC: 82 lpm; FR: 20 rpm. Saturacin O2: 75%. La aorta era
discretamente palpada en la frcula. En el precordio haba pulsos
sistlicos discretos, frmito sistlico ntido en todo el borde esternal
izquierdo. Ictus cordis no era palpado. Los ruidos cardacos eran muy
hiperfonticos y se auscultaba soplo sistlico acentuado en todo el
borde esternal izquierdo, con ntida irradiacin hacia el rea artica. El
hgado no era palpado y los pulmones eran limpios.
PREGUNTA
Considerando el cuadro clnico, antecedetnes y frecuencia de las
cardiopatas congenitcas, cual es la condicin mas probable del caso?
RESPUESTA
a.- Estenosis pulmonar.
b.- Doble va de salida de ventrculo derecho.
c.- Doble va de entrada de ventrculo nico.
d.- Estenosis pumonar aislada.
CASO CLINICO
Varn de 21 aos con atresia pulmonar y comunicacin
interventricular intervenido de anastomosis aortopulmonar de
Waterston-Cooley (WC) en el primer mes de vida. A los 2 aos se le
realiz fstula de Blalock-Taussig (BT) izquierdo modificado y a los 8,
ampliacin de arterias pulmonares con parche de Dacron,
implantndose adems un conducto no valvulado entre el ventrculo
derecho y las arterias pulmonares. A los 12 aos se cerr la
comunicacin interventricular mediante un parche en teja y se
reconstruy el conducto con tcnica de Danielson. Dos aos ms tarde,
el cateterismo cardaco demostr estenosis severa entre el tronco
pulmonar y ambas ramas, con presiones sistmicas en el ventrculo
derecho. Se realiz angioplastia con baln de ambas ramas, que fue
ineficaz. Nueve aos despus el paciente se encontraba en grado

CURSO ENARM CMN SIGLO XXI TEL: 36246001

PREGUNTA
Cual de las siguientes causas es menos frecuente en la estenosis e
hipoplasias localizadas de las arterias pulmonares secundarias a ciruga
paliativa?
RESPUESTA
a.- Constriccion progresiva.
b.- Ineficacia de angioplastia con baln.
c.- Distorsin de tejido fibroso en su interior.
d.- Crecimiento de tejido fibroso en su interior.

PREGUNTA
El tratamiento quirrgico de esta complicacin es tcnicamente difcil y
la dilatacin con baln habitualmente ineficaz por que razo?
RESPUESTA
a.- Retroceso elstico de la arteria.
b.- Fibrosis persistente suprapulmonar.
c.- Falta de angenten inmunomoduladores.
d.- Persistencia de las resistencias intracavitarias.
CASO CLINICO
Varn de 19 aos diagnosticado de tetraloga de Fallot severa a los 3
aos de edad, practicndosele fstula de BT izquierdo seis meses
despus. A los 5 aos se rechaz la ciruga correctora por presentar
hipoplasia severa de tronco y arterias pulmonares. Desde entonces
presentaba cianosis progresiva severa, disnea de esfuerzo,
acuclillamiento y sncopes. A los 9 aos se realiz ampliacin con
parche del tracto de salida del ventrculo derecho y del origen de
ambas ramas pulmonares, as como cierre de la comunicacin
interventricular con parche fenestrado, quedando con presin
sistmica en el ventrculo derecho, gradiente transpulmonar severo y
cortocircuito bidireccional ventricular.
PREGUNTA
Cual de las siguientes aseveraciones es correcta, al manejo de stens
en esta patologia?
RESPUESTA
a.- La dilatacin mediante stent no es un medoto seguro ni eficaz.
b.- La combinacin de stent y valvuloplastia con baln tiene mayor
eficacia y eficiencia.
c.- La dilatacin mediante stent es un mtodo seguro y eficaz de
tratamiento.
d.- En las estenosis de las arterias pulmonares en adultos con
cardiopatas congnitas operadas en la infancia no es til los Stens.

Pharmed Solutions Institute

PGINA 16

MANUAL DE TRABAJO DEL CURSO ENARM CMN SIGLO XXI


VALVULOPATIAS
CIENCIAS BASICAS: Estenosis valvular o dificultad para que la vlvula pueda abrirse. Insuficiencia valvular o imposibilidad de que la vlvula al cerrarse
ocluya completamente el orificio valvular con ello reflujo anormal de sangre. Focos de auscultacin: foco mitral o apexiano 5to. espacio intercostal izq.
con lnea medio clavicular; foco artico 2do. espacio intercostal con paraesternal der; foco tricuspdeo porcin inferior del esternn; foco pulmonar 2do.
espacio intercostal paraesternal izq. Causas: fiebre reumtica (infeccin de origen estreptoccico ms mecanismo inmunolgico afectando endocardio
valvular), endocarditis bacteriana (da lugar a formacin de vegetaciones en las vlvulas mutilndolas), malformaciones congnitas, degeneracin
mixomatosa (prolapso de los velos valvulares en insuficiencia), disfuncin y/o rotura de las cuerdas tendinosas (IAM). ESTENOSIS MITRAL: Valvulopata
ms frecuente. Su principal causa es la fiebre reumtica. Ms a mujeres. PATOGENIA: Disminucin del rea del orificio valvular mitral (normal 5cm), con
lo cual hay dificultad para el relleno del VI, durante la distole, esto va generando hipertrofia de la pared auricular, porque se requiere mayor presin
para mantener el flujo hacia el VI, pero adems hay bajo gasto. Se puede desencadenar una FA, puede haber estasis de sangre en la aurcula, generar un
trombo y este embolismo sistmico. rea inferior a 2cm, genera sntomas al ejercicio, rea <1cm genera elevacin de la presin auricular, aumento de
presin venosa y capilar pulmonar; hay insuficiencia cardiaca izquierda con hipertensin pulmonar y congestin pulmonar que genera disnea de reposo,
mas EAP. DIAGNOSTICO: Disnea de esfuerzo (sntoma principal), ortopnea, disnea paroxstica nocturna, edema agudo del pulmn (EAP), episodios de
infeccin respiratoria de repeticin, astenia por disminucin de gasto cardaco, datos de IC der. (Ingurgitacin yugular, hepatomegalia, ascitis) en fases
avanzadas, fascies mitral por rubicundez de las regiones malares, pulso parvo y latido de punta dbil (por cada de vol. sistlico), estertores pulmonares
hmedos (si hay EAP). Auscultacin: Refuerzo del 1er ruido, cierre brusco y enrgico de la vlvula mitral, soplo diastlico con sonoridad de retumbo (se
acenta despus del ejercicio), y reforzamiento presistlico inmediatamente antes del 1er ruido. ECG: Onda p mitral, si no existe FA. Rx: datos de
congestin venocapilar pulmonar, con dilatacin de ramas pulmonares y AI. Ecocardiograma, forma no invasiva mas til para dx. TRATAMIENTO: Dirigido
a aliviar sntomas de congestin pulmonar con diurticos, la FA (beta-bloqueadores, bloqueadores de canales de calcio) y anticoagulacin. La ciruga es lo
definitivo; comisurotoma o cambio valvular. COMPLICACIONES: EAP, hemoptisis, arritmias (FA); embolismo sistmico (fractura de trombo intraauricular,
causando EVC, isquemia mesentrica y de extremidades inferiores), afona (por compresin de la AI sobre el nervio recurrente). INSUFICIENCIA MITRAL:
Causa ms comn cardiopata reumtica, otras colagenosis, enfermedades degenerativas, causas congnitas, alteracin del anillo mitral, rotura de
cuerdas tendinosas, alteracionnde musculos papilares (IAM). Pacientes jvenes. PATOGENIA: La vlvula permite reflujo anormal de sangre desde VI a la
AI, durante la sstole ventricular, esto genera sobrecarga en aurcula izq. que lleva a congestin y edema pulmonar. Durante la distole hay sobrecarga de
volumen en VI, generndose una dilatacin compensatoria que genera IC izq. DIAGNSTICO: Astenia, fatigabilidad ceciente por descenso de GC disnea,
ortopnea, disnea paroxstica nocturna y/o EAP, ICC en fases avanzadas, latido de punta desplazado a la izq. y abajo, amplio poco sostenido, el ventrculo
grande regurgita, puede palparse trill sistlico apical. Auscultacin: 1er ruido apagado (por no cierre completo), soplo holosistlico (regurgitante, en
chorro de vapor), que se propaga hacia la axila, pulso arterial normoamplio y celer. ECG: onda p mitral y crecimiento de VI con R altas. Rx de trax:
dilatacin del VI y AI, con desplazamiento del bronquio principal izq. TRATAMIENTO: Reducir la pre y poscarga, para disminuir sntomas por congestin
pulmonar; el edema pulmonar con diurticos y nitratos; reduccin de la poscarga con IECAS. Qx definitiva, plastia o cambio valvular. ESTENOSIS
ARTICA: Es una obstruccin al tracto de salida de VI. Causas: degenerativa hombres de 60-70 aos, por calcificacin, forma mas comn es la congnita
(valva uni o bivalva), ateroesclerosis, reumtica. PATOGENIA: Dificultad para vaciar el VI, lo que genera aumento de la poscarga e hipertrofia concntrica
del VI (nos puede llevar a isquemia, arritmias y muerte sbita), que conduce a disminucin de la distensibilidad ventricular, aumentndose la presin
telediastolica del VI, mas dilatacin del VI, lleva a insuficiencia mitral, que lleva a IC retrgrada. DIAGNOSTICO: Asintomtica por mucho tiempo. Triada
clsica: angina de pecho (30-40%), insuficiencia cardiaca (20%) y sincope (10-15%, tras esfuerzo intenso), astenia progresiva primer sntoma, disnea de
esfuerzo, de reposo, palpitaciones, visin borrosa. Auscultacin: Latido de punta desplazado hacia la izq. y abajo por hipertrofia de VI, pulso parvo y
tardo, TAS disminuida con escasa modificacin de la TAD, trhill sistlico palpable en el borde esternal izquierdo. Chasquido clic de eyeccin despus del
primer ruido, soplo mesositlico de morfologa romboidal, por disminucin del flujo artico de la vlvula estenosada, cuando la presin intraortica
alcanza un determinado nivel. ECG: signos de crecimiento ventricular izq. con grandes ondas R en V5 y V6 y S profunda en V1 y V2, ondas T negativas por
la sobrecarga de presin y trastornos de la conduccin. Rx de trax: silueta cardiaca normal, porque la hipertrofia concntrica no la modifica, en fases
finales hay cardiomegalia. Ecodopler: nos permite valorar el rea valvular (rea valvular en cm 2 >1.5 leve; 1.0-1.5 moderada; < 1.0 severa) y el gradiente
transvalvular. TRATAMIENTO: De eleccin es quirrgico (prtesis mecnica). INSUFICIENCIA ARTICA: Causas fiebre reumtica, sfilis y endocarditis
bacteriana. PATOGENIA: Los velos valvulares articos, no tienen un cierre hermtico, lo que genera reflujo diastlico esto aumenta la precarga y genera
hipertrofia del VI, que puede llevar a IC, por otro lado el aumento en el consumo de O2 y la disminucin de la presin diastlica, lleva a isquemia,
frecuente en el Sx. de Marfan, diseccin artica entre otras. DIAGNOSTICO: Manifestaciones de IC o isquemia hasta 4 o 5 dcada de la vida, disnea en
reposo, ortopnea, disnea paroxstica nocturna, angina, palpitaciones, pulso capilar (signo de Quincke) se presiona sobre la ua se observa, como el borde
de la zona plida avanza y retrocede, durante la sstole y distole. Danza carotidea (signo de Corrigan), latido carotideo visible y muy amplio, ampliacin
de la TA diferencial por elevacin de la TA sistlica y disminucin de la diastlica, que incluso puede ser 0. Auscultacin: 1er y 2do ruidos apagados, soplo
pandiastlico decreciente (por la progresiva cada de la presin en la raz de la aorta durante la regurgitacin), de tono alto, suave y aspirativo,
irradindose a lo largo del borde esternal. ECG: Signos de crecimiento del VI, sobrecarga diastlica ondas T picudas y supradesnivel del segmento ST. Rx.
cardiomegalia con signos de crecimiento del VI y dilatacin de la raz aortica, ecodopler permite medir grado de ingurgitacin. TRATAMIENTO: Enfocado
a corregir sntomas de IC, oxigeno, diurticos, inotrpicos, IECAS. Qx definitivo, se recomienda en pacientes con dimetros ventriculares >50mm.
ESTENOSIS TRICSPIDEA: Condiciona sobrecarga de presin en sentido retrgrado sobre la aurcula y congestin venosa con clnica de IC der.
Auscultacin en foco tricspide soplo diastlico, con refuerzo presistlico y chasquido de apertura de la tricspide. INSUFICIENCIA TRICSPIDEA: Clnica
de IC der. por regurgitacin sistlica, hacia la aurcula. Auscultacin: soplo sistlico, similar al de la insuficiencia mitral, pero se ausculta mejor en el foco
tricspide., se incrementa durante la inspiracin, por aumentar esta condicin el retorno venoso. ESTENOSIS PULMONAR: IC der., sobrecarga de presin
del ventrculo der. Auscultacin: Foco pulmonar muestra un clic de eyeccin y un soplo sistlico de morfologa romboidal. INSUFICIENCIA PULMONAR:
IC der., regurgitacin y sobrecarga del VD. Auscultacin: soplo diastlico decreciente en foco pulmonar, equiparable en su gnesis al de la insuficiencia
aortica. CLAVES: La profilaxis con antibiticos es recomendada, si hay procedimientos invasivos, por alto riesgo de endocarditis infecciosa.
CASO CLINICO
Paciente de 66 aos de edad, con obesidad, hipertensin arterial y
dislipemia con diagnstico de estenosis artica grave y cardiopata
coronaria al realizarse ecocardiografas transtorcica y transesofgica;
se concluy estenosis artica grave (vlvula artica con gradientes
mximo de 68, medio de 46 y rea de 0,95 cm2) con funcin sistlica
conservada e hipertrofia concntrica del ventrculo izquierdo (septum
interventricular de 15 mm y pared posterior de 14 mm). El cateterismo

CURSO ENARM CMN SIGLO XXI TEL: 36246001

cardiaco demostr enfermedad coronaria (lesin oclusiva del 80% en


el tercio proximal de la descendente anterior y del 75% en el tercio
medio de la circunfleja).
PREGUNTA
Considerando los factores del paciente cual es la causa mas probable?
RESPUESTA
a.- Calcificacion valvular.

Pharmed Solutions Institute

PGINA 17

MANUAL DE TRABAJO DEL CURSO ENARM CMN SIGLO XXI


b.- Degenerativa.
c.- Cardiopatia reumtica.
d.- Fibrosis autoinmune.

c.- Embolia pulmonar.


d.- Infeccin

PREGUNTA
Cual es el pronstico del paciente conociendo la mortalidad anual de
esta patologia?
RESPUESTA
a.- 3 %.
b.- 6 %.
c.- 9 %.
d.- 12 %.
PREGUNTA
Cual de las siguientes etiologias es menos frecuente en este caso?
RESPUESTA
a.- Sindrome de Marfan.
b.- Hipercolesterolemia.
c.- Cardiopatia reumtica.
d.- Lupus eritematoso.
CASO CLINICO
Paciente de femenino de 65 aos de edad, se detect hipertensin
arterial (160/100 mmHg), fibrilacin auricular de inicio indeterminado.
Se inici tratamiento con atenolol 50 mg/da, enalapril 20 mg/da.
Desde el ltimo ao disnea progresiva con esfuerzos menores de los
habituales y palpitaciones frecuentes espontneos. En la auscultacin
cardaca se apreci arritmia completa por fibrilacin auricular, un soplo
holosistlico 4/6 de alta frecuencia en el foco mitral irradiado a la axila
y otro sistlico, de menor intensidad, en el borde esternal izquierdo
bajo, que aumentaba con la inspiracin. En la radiografa de trax se
apreci una cardiomegalia global grado III/IV con campos pulmonares
normales. El ECG demostr una fibrilacin auricular controlada, con
signos de crecimiento ventricular izquierdo y alteraciones secundarias
de la repolarizacin.
PREGUNTA
Cul es el manejo ms adecuado de la insuficiencia cardiaca?
RESPUESTA
a.- Amiodarona.
b.- Hidroclorotiazida.
c.- Digoxina.
d.- Nifedipina.

CASO CLINICO
Mujer de 83 aos, con prtesis mitral mecnica. Fibrilacin auricular
crnica. Ecocardiograma transesofgico: rea valvular de 0,5cm2,
hipertensin pulmonar de 75mmHg, fraccin de eyeccin del 65%,
anillo artico de 18 mm y prtesis mitral normofuncionante. La
paciente evolucion sin complicaciones. En la revisin de 1 y 3 meses
se encuentra en clase funcional I.
PREGUNTA
Cual es el agente etiolgico mas frecuente de esta patologa?
RESPUESTA
a.- Estreptococcos
b.- Estafilococcos.
c.- Treponema.
d.- Cocobacilos
CASO CLINICO ESTENOSIS VALVULA PULMONAR
Posteriormente a la correccin de la tetraloga de Fallot realizada a los
19 aos de edad de la paciente, con ampliacin del tracto de salida del
ventrculo derecho, valvotoma pulmonar y cierre de la comunicacin
interventricular, quedaron como defectos residuales, estenosis
pulmonar al nivel valvular por un pequeo anillo pulmonar y
comunicacin interatrial. Permaneci con una buena evolucin por 28
aos cuando presenta embolia cerebral por fibrilacin atrial.
PREGUNTA
Considerando el cuadro clnico cual de los siguientes frmacos es el
mas apropiado para mantener controlada la agregacin plaquetaria.
RESPUESTA
a.- Acetilsalicilico
b.- Clopidrogel.
c.- Acecumarol.
d.- Enoxoparina.
CASO CLINICO INSUFICIENCIA TRICUSPIDEA
Mujer de 59 aos con historia de implantacin de marcapasos
definitivo endocavitario por bloqueo auriculoventricular completo.
Present trombosis de vena cava, que origin un sndrome de vena
cava superior y requiri de la colocacin de un stent autoexpandible.
Present estenosis de la vena subclavia izquierda y tromboembolia
pulmonar. Es egresado luego de 30 dias de estancia hospitalaria.

PREGUNTA
Cul es el manejo ms adecuado para los trastornos del ritmo?
RESPUESTA
a.- Amiodarona.
b.- Hidroclorotiazida.
c.- Digoxina.
d.- Nifedipina.
PREGUNTA
Cual es la etiologa ms probable considerando los factores de riesgo
que presenta la paciente?
RESPUESTA
a.- Fiebre reumtica.
b.- Sindrome de lutenbacher.
c.- Enfermedad de fabry.
d.- Enfermedad de whipple.

PREGUNTA
Cual de los siguientes factores etiolgicos es mas frecuente en esta
patologa.
RESPUESTA
a.- Insuficiencia venosa profunda.
b.- Arterio esclerosis de grandes vasos.
c.- Insuficiencia tricuspidea.
d.- Anticoagulacion insuficiente.

PREGUNTA
Considerando la condicin de la paciente. Cual de las siguientes
causas es la ms frecuente de muerte?

CASO CLINICO
Paciente femenino de 20 aos, con antecedente de fiebre reumtica,
diagnosticada a la edad de 8 aos. Inicia padecimiento hace 8 meses, la
paciente refiere disnea de esfuerzo progresiva, y ortopnea que inicio
hace 2 das, astenia, y rubicundez en reas malares. Laboratorios: sin
alteraciones de importancia. A la auscultacin, estertores, primer ruido
intenso ("chasquido de cierre"), un chasquido de apertura se ausculta
inmediatamente despus del segundo ruido, despus retumbo
diastlico, que al final de la distole se acenta como refuerzo
presistlico, en ritmo sinusal.

RESPUESTA
a.- Embolia sistmica.
b.- Insuficiencia cardiaca.

PREGUNTA
Cul es el diagnstico ms probable en este paciente?
RESPUESTA

CURSO ENARM CMN SIGLO XXI TEL: 36246001

Pharmed Solutions Institute

PGINA 18

MANUAL DE TRABAJO DEL CURSO ENARM CMN SIGLO XXI


a.- Estenosis aortica
b.- Insuficiencia aortica
c.- Estenosis mitral
d.- Insuficiencia mitral
PREGUNTA
A qu factor es ms probable que se deba la disnea de esfuerzo que
presenta la paciente?
RESPUESTA
a.- Presin hidrosttica intravascular
b.- Hipertensin en la aurcula izquierda
c.- Presencia de congestin pulmonar
d.- Calcificacin de valvas
PREGUNTA
Cul es la arritmia que ms probablemente podra presentar esta
paciente?
RESPUESTA
a.- FV
b.- TVSP
c.- FA
d.- AESP
CASO CLINICO
Hombre de 56 aos de edad, con antecedente de tabaquismo desde
los 17 aos a razn de 12 cigarrillos al da, hipertenso, con
antecedente de IAM, hace 4 aos. Inicia su padecimiento actual hace 7
das con disnea, inicialmente desencadenada con los esfuerzos y desde
hace 2 das presentndose en reposo en forma paroxstica y
remitiendo espontneamente. El da de hoy inicia de forma sbita con
disnea, sin otros sntomas acompaantes, motivo por el que acude a
urgencias. A la exploracin fsica, FC 115x, FR30x, temp 36.0C, TA
100/60mmHg. Consciente, orientado, diafortico, inquieto, con palidez
de piel y tegumentos, mucosa oral hidratada, cuello en pltora yugular
grado II, campos pulmonares con estertores crepitantes, ruidos
cardiacos primer apagado segundo ruidos normales, con presencia de
tercer ruido, se ausculta soplo pansistolico mitral (en chorro de vapor),
irradiado a base, sin frote pericrdico, abdomen sin datos patolgicos,
extremidades fras, sin edema, pulsos dbiles. Laboratorio Glucosa
110mg/dl, Cr 0.9, BUN 18, CK 380.
PREGUNTA
Cul es el diagnstico ms probable en este caso?
RESPUESTA
a.- Estenosis aortica
b.- Insuficiencia aortica
c.- Estenosis mitral
d.- Insuficiencia mitral

PREGUNTA
Cul es el diagnstico ms probable en este caso?
RESPUESTA
a.- Estenosis aortica
b.- Insuficiencia aortica
c.- Estenosis mitral
d.- Insuficiencia mitral
PREGUNTA
Cul es la causa ms probable del sincope de este paciente?
RESPUESTA
a.- Aumento desproporcionado de la demanda de O2 miocrdico y
calcificacin del sistema valvular
b.- Disminucin de la distensibilidad arterial y ventricular
c.- Hipercolesterolemia e hipertrofia ventricular izquierda
d.- Imposibilidad de aumentar el GC e hipoperfusin cerebral
PREGUNTA
Qu mtodo diagnostico sera el ms adecuado para el diagnstico
definitivo?
RESPESTA
a.- Electrocardiograma
b.- Radiografa de trax
c.- Ecocardiografa transesofgica
d.- Fluoroscopa
CASO CLINICO
Paciente masculino de 35 aos de edad, el cual refiere buen estado de
salud, niega antecedentes patolgicos. nicamente refiere que su
padre muri a los 40 aos de muerte sbita. Hace 3 meses inicio con
disnea de medianos esfuerzos, mareos ocasionales, astenia,
palpitaciones, dolor torcico ocasional de leve a moderado.
Laboratorios dentro de parmetros normales. FC 105x, FR 22x, TA
90/75mmHg. A la exploracin fsica, se auscultan estertores, latido de
punta a la derecha y hacia abajo, pulso parvo y tardo. ECG, con signos
de crecimiento ventricular izquierdo, sobrecarga de presin y bloqueo
AV.
PREGUNTA
Considerando el cuadro clnico y antecedentes del paciente cul es la
causa ms probable?
RESPUESTA
a.- Insuficiencia tricuspidea
b.- Vlvula aortica bicspide
c.- Cardiopata reumtica
d.- Calcificacin valvular

Cul es la causa de que disminuya la fraccin de eyeccin en esta


paciente?
RESPUESTA
a.- Disminucin de la funcin y dilatacin progresiva del ventrculo
izquierdo (fracaso)
b.- Regurgitacin de sangre a la aurcula izquierda
c.- Sobrecarga del ventrculo izquierdo
d.- Falla de funcional de la vlvula aortica
PREGUNTA
Cul es la conducta teraputica ms adecuada para esta paciente?
RESPUESTA
a.- Baln de contrapulsacin intraartico
b.- Digoxina, diurtico, IECA
c.- Diurtico, digoxina, beta bloqueador
d.- Plastia mitral

CURSO ENARM CMN SIGLO XXI TEL: 36246001

CASO CLINICO
Paciente masculino de 65 aos de edad, diabtico de 20 aos de
evolucin e hipercolesterolemia. Actualmente con angina de pecho,
insuficiencia cardiaca, astenia progresiva, disnea de esfuerzo y en
reposo. Sufri sincope, al estar arreglando su jardn, 30 minutos antes
de llegar al servicio de urgencias. A la exploracin fsica pulso arterial,
que se palpa en cartidas, TA 80/50 con amplitud de pulso disminuida,
FC 98x, FR 22x, soplo sistlico irradiado a pex.

PREGUNTA
Qu otros defectos congnitos podra tener asociados este paciente?
RESPUESTA
a.- Coartacin aortica, comunicacin interventricular
b.- Comunicacin inerventricular y alteraciones auditivas
c.- Coartacion aortica, insuficiencia renal
d.- Hipoplasia de corazn izquierdo, insuficiencia renal

Pharmed Solutions Institute

PGINA 19

MANUAL DE TRABAJO DEL CURSO ENARM CMN SIGLO XXI


CARDIOMIOPATIA Y MIOCARDITIS
CIENCIAS BASICAS: Es un espectro de patologas con diversos mecanismo patognicos, con un final semejante a un sndrome congestivo de falla
cardiaca, el dao o padecimiento del miocardio puede ser primario o secundario, las causas de miocardiopatias mas frecuentes son: miocarditis
infecciosa viral (coxsaxkievirus, echovirus, HIC, Epstein barr, influenza, CMV. Bacteriano (Corynebacterium diphtheriae, streptococcus pyogenes,
staphylococcus aureus, haemophilus pneumoniae, salmonella spp, neisseria gonorrhoeae, leptospirosis, lyme, syphilis, brucelosis; Fungico (Candida spp,
arpergillus spp, histoplasmosis, blastomicosis, cryptoccosis, coccidioidomycosis; Parasitario (Toxoplasmosis, schistosomiasis, trchinosis). Dilatada
(desconocida); Infiltrativa (amiloidosis, sarcoidosis, hemocromatosis). MIOCARDITIS: Las causas mas frecuentes son procesos infecciosos virales del 1 al 9
% son coxsackie virus B, las manifestaciones son variadas, puede pasar asintomticas hasta un profundo choque cardiogenico, el antecedente de
afeccion viral de 7 a 10 dias y ataque al estado generalizado el 60 % de los casos, 35% presentan dolor torcico, otra presentacin son los bloqueos
cardiacos completos o taquicardia ventricular. Las manifestaciones clnicas incluyen fiebre, quicardia, signos de insuficiencia cardiaca, disminucin de
ruidos cardiacos, galope, murmullo por regurgitacin mitral, frote pericardico, laboratorios son leucocitosis, eosinofilia, incremento de IgM, IgG, CPK en
aproximadamente 10 %, el ECG taquicardia, alteracin de ST y cambios de la onda T, prolongacin QT, el ECOCG puede manifestar alteraciones de la
funcin ventricular, se clasifica en fulminante, agudo, crnico activo, crnico persistente. Sobrevida a 5 aos de 50 %. El tratamiento es de soporte,
diurticos, ECAs, bloqueadores antagonistas de aldosterona, la digoxina puede incrementar la expresin inflamatoria, AINEs no son utiles, son utiles;
esteroides con azatioprine o esteroide con ciclosporina. ENFERMEDAD DE CHAGAS La tripanosomiasis o enfermedad de chagas es mas comn en
america central y del sur, es causada por tripanosoma cruzi, la lesin cardiaca es mediada por inumunocomplejos. Se disemina via hematogena a varios
rganos y sistemas con una intensa reaccin inflamatoria, fiebre, sudoracin, mialgias y miocarditis. El 5 % son fatales, del 20 al 30 % permanecen
asintomticos en fase latente, los casos crnicos presentan fibrosis de miofibrillas causando cardiomegalia, falla cardiaca, bloqueos y arritmias. El
tratamiento es sintomtico, marcapasos y agente antiparasitarios- ENFERMEDAD DE LYME: Es causada por la infeccin con la espiroqueta borrelia
burgdorferi introducido por picadura, los sntomas iniciales del padecimiento es bloqueo cardiaco completo, puede verse disfuncin ventricular
izquierda, la miopsia muestra datos de miocarditis activa. MIOCARDITIS REUMATICA: Se puede observar durante la fiebre reumtica aguda seguido de
faringitis streptococcica del grupo A, El diagnostico clnico se hace con los criterios de JONES. Mayores; carditis, poliartritis, corea, eritema marginatm,
ndulos subcutneos, y evidencia de infeccin previa. Los criterios Menores son fiebre artralgias, fiebre reumtica, elevada sedimentacin eritrocitaria,
protena C-reactiva y prolongacin del intervalo PR. El diagnostico es con dos mayores y un mayor y dos menores, del 5 al 10 % desarrollan insuficiencia
cardiaca. Se presenta carditis, lesion valvular aortica predominantemente. ECG prolongacin de PR y cambios inespecficos del ST-T. Tx aspirina,
corticoides y 1.2 millones U de penicilina G benzatina. MIOCARDITIS NO INFECCIOSAS: La miocarditis hipersensitivas y son causadas por reaccin a
medicamentos, caracterizada por eosinofilia, infiltracin al miocardio de eosinofilos, clulas gigantes multinucleadas, y leucocitos, los frmacos mas
frecuentes son: metildopa, penicilina, tetraciclina y antituberculosos, pueden causar emadamente raras que generan miocariditis, progresiva sin
respuesta a tratamiento, mas frecuente en adulto jove, asociado a enfermedades autoinmunes 20 %, se observa histiocitos y linfocitos y eosinofilia, es
fatal solo ttx transplante. Cardiomiopata periparto, 1 en 3,000 partos, multiples factores, obesidad, gestacion multiple, preeclapmsia, hipertensin
crnica, px con 30 aos aproximadamente. La falla cardiaca es variable inicia en el tercer trimestre, ECG con hipertrofia ventricular izquierda, pronostico
reservado. Miocardiopatias por enfermedades neuromusculares, distrofias neuromusculares hereditarias, asociadas a cardiomiopatas Becker,
Duchenne, Steinert, distrofia miotonica, Friedreich, Barth. CARDIOMIOPATIAS POR ENFERMEDADES ENDOCRINAS: El exceso o disminucin de
hormonas tiroideas, en la tirotoxicosis se compromete eventualmente la funcin ventricular izquierda con dilatacin pudiendo llegar a falla cardiaca, El
feocromocitoma es otra causa con hipertensin, sudoracin, palpitaciones e hipotensin ortostatica, puede presentarse miocarditis inducida por
catecolaminas, puede llegar a fallar cardiaca o arritmias ventriculares malignas. La acromegalia presenta cardiomiopata en el 10 al 20 % de los casos por
el exceso de hormona de crecimiento que genera hipertrofia de miocitos que se fibrosan, con disfuncin diastlica y sistlica finalmente, tambin
degeneracin del nodo AV con bloqueo completo. MIOCARDIOPATIAS POR TOXINAS: El abuso crnico de ALCOHOL es un riesgo mayor para desarrollar
cardiomiopata congestiva, 45 % de todas las dilatadas, puede generar insuficiencia cardica, hipertensin y arritmias, el dao es directo, acetilaldehido y
metabolitos, asi como deficiencia nutrimentales, estimulacin simpatica y cofactores toxicos. La edad promedio de 30 a 55 aos, con historia de 10 aos
de consumo intenso, la fibrilacin atrial es la forma mas frecuente de preentacion luego la muerte sbita. COCAINA produce isquemia miocrdica,
infarto, espasmo coronario, arritmias cardiacas, muerte sbita, miocarditis y cardiomiopata dilatada. Mal pronostico, tratamiento sintomtico los
betabloqueadores puede disminuir espasmos. QUIMIOTERAPICOS: doxirrubicina, ciclofosfamida, generan citotoxicidad, la radiacin de mediastino, el
trastuzumab es un anticuerpo monoclonal se puede presentar miotoxicidad. La CLOZAPINA antipsicotico de uso crnico. Fenotiacinas, cloroquina, litio,
cobalto, hidrocarbonos, interferon alfa IL-2. CARDIOMIOPATIAS ASOCIADAS CON DEFICIENCIAS NUTRIMENTALES: Deficiencia de TIAMINA, resulta en
beri beri con sntomas caracteristicos de falla cardiaca, pronostico fatal sin tratamiento, Deficiencia de VITAMINA D, la deficiencia o el exceso aumenta el
riesgo, falta de absorsion de SELENIO, la disminucin de L-CARNITINA. CARDIOMIOPATIAS TAKO-TSUBO: La miocardiopata de Takotsubo es un
sndrome recientemente descrito atribuido al exceso de catecolaminas, probablemente relacionado con una hiperactividad simptica inducida por un
factor estresante de cualquier tipo. Son numerosas las enfermedades neurolgicas que se han descrito relacionadas con este sndrome: hemorragia
subaracnoidea, ictus, esclerosis mltiple, sndrome de Guillain-Barr, crisis miastnicas y crisis epilpticas. Del mismo modo, son numerosos los casos de
sndrome de Takotsubo asociados a enfermedades psiquitricas, dependencia a opioides, alcoholismo, trastorno manaco-depresivo, trastorno
depresivo, esquizofrenia. Tambin se han relacionado diversos factores emocionales (malas noticias, fiestas sorpresa, discusiones, divorcio, muertes
inesperadas...) y fsicos (ejercicio, neumotrax, hipoglucemia, ataque de asma, cirugas...). El sndrome de Takotsubo, descrito por primera vez en Japn
en 1991, se caracteriza por acinesia o discinesia de la porcin apical y media del ventrculo izquierdo que no corresponde con un nico territorio vascular
en ausencia de enfermedad coronaria, demostrada mediante cateterismo cardaco. El electrocardiograma puede mostrar elevacin del segmento ST o
inversin de la onda T. Como criterios de exclusin, entre otros, se encuentran antecedentes recientes de hemorragia intracraneal, feocromocitoma,
miocarditis, miocardiopata hipertrfica, hipertiroidismo, pancreatitis y envenenamiento. Como desencadenantes se han relacionado factores de estrs
tanto de tipo emocional como fsico. CARDIOMIOPATIA DILATADA: Es una causa frecuente de insuficiencia cardaca y es el diagnstico ms frecuente en
pacientes sometidos a trasplante cardaco. Desde el punto de vista clnico, la MD se caracteriza por dilatacin y disfuncin contrctil del ventrculo
izquierdo o de ambos ventrculos. La dilatacin ventricular es generalmente severa y se acompaa siempre de hipertrofia. La MD puede ser idioptica,
gentica/familiar, viral y/o inmune, alcohlica/txica, o asociada a otras cardiopatas en las cuales el grado de disfuncin miocrdica no se explicara por
una determinada sobrecarga hemodinmica o severidad de dao isqumico. Probablemente, el sndrome clnico de la MD representa un final comn al
que se llega a travs de mltiples mecanismos citotxicos, metablicos, inmunolgicos, infecciosos y familiares. Anatoma patolgica: Los estudios post
mortem demuestran habitualmente dilatacin de las cuatro cmaras cardacas, en especial los ventrculos, que se acompaa a veces de aumento de
grosor de la pared. Las vlvulas cardacas son intrnsecamente normales y es frecuente la presencia de trombos intracavitarios. Es tpica la presencia de
miocitos hipertrficos y muertos, sustituidos por fibrosis con variable afectacin del sistema de conduccin. Los componentes de la matriz extracelular
estn aumentados de forma no selectiva. Aunque la ausencia de clulas inflamatorias se utiliza como criterio para diferenciar la MD de la miocarditis, en
algunos casos de MD se detectan clulas T inflamatorias y clulas endoteliales activadas, sugiriendo la posible contribucin de un proceso inflamatorio

CURSO ENARM CMN SIGLO XXI TEL: 36246001

Pharmed Solutions Institute

PGINA 20

MANUAL DE TRABAJO DEL CURSO ENARM CMN SIGLO XXI


crnico en la patogenia de esta enfermedad. La etiologa y los mecanismos patognicos son desconocidos en alrededor de la mitad de los casos de MD.
Para explicar el dao miocrdico crnico y progresivo se han propuesto tres principales etiopatogenias: a) infeccin viral crnica del miocardio que
produce dao celular; b) alteracin de los mecanismos inmunes que conduce probablemente a una enfermedad autoinmune, y c) factores genticos que
seran directa o indirectamente responsables de la enfermedad. DIAGNSTICO: El estudio del paciente con MD debe enfocarse no slo al
establecimiento del diagnstico sindrmico, sino hacia la identificacin, por los mtodos de diagnstico habituales, de posibles causas tratables o
reversibles de la enfermedad. La historia clnica debe incluir preguntas relativas al posible consumo de alcohol y cocana, medicamentos, hbitos
nutricionales, estancias en zonas endmicas para infecciones, relacin con animales, embarazos recientes, transfusiones sanguneas, historia familiar de
MD, somnolencia diurna y exposicin profesional a txicos. Asimismo, la idea de que la MD idioptica es con frecuencia un problema gentico
hereditario debe ser tenida en cuenta en la prctica clnica, estudiando sistemticamente a los familiares de primer grado del paciente. En la mayora de
los pacientes la MD se manifiesta clnicamente entre los 20 y 60 aos de edad, aunque la enfermedad puede afectar tambin a nios y ancianos. Los
sntomas ms frecuentes son los de insuficiencia cardaca (disnea de esfuerzo progresiva, ortopnea, disnea paroxstica nocturna y edemas perifricos).
Otras formas de presentacin son la deteccin accidental de cardiomegalia asintomtica y los sntomas relacionados con arritmias, alteraciones de
conduccin, complicaciones tromboemblicas o muerte sbita. La exploracin fsica suele revelar diferentes grados de cardiomegalia y signos de
insuficiencia cardaca. La presencia de un galope presistlico (cuarto ruido) puede preceder a la aparicin de insuficiencia cardaca. El ritmo de galope
ventricular (tercer ruido) es la regla en los casos con descompensacin de la IC. Es frecuente la presencia de soplos sistlicos de insuficiencia mitral o,
menos frecuentemente, tricuspdea. Las dos causas ms frecuentes de muerte en pacientes con MD e insuficiencia cardaca son la muerte sbita y el
fallo de bomba progresivo. MIOCARDIOPATA HIPERTRFICA: La miocardiopata hipertrfica (MH) es una enfermedad con una importante
heterogeneidad en cuanto a su base gentica, manifestaciones clnicas y pronstico 93-96. Se caracteriza, fundamentalmente, por la presencia de una
hipertrofia ventricular de causa desconocida (generalmente de predominio septal) y por una excelente funcin sistlica 93-96. Los gradientes
intraventriculares dinmicos constituyen un rasgo primordial de esta entidad, pero sabemos que slo se detectan en una parte de los pacientes y buena
parte de las manifestaciones son consecuencia de las alteraciones diastlicas. El diagnstico clnico de los pacientes con MH sigue basndose en la
demostracin de una hipertrofia ventricular en ausencia de factores cardacos o sistmicos que la justifiquen. Los sntomas ms frecuentes -disnea,
angina, palpitaciones-, se manifiestan en ms de la mitad de los pacientes, pero son comunes en otras patologas cardiovasculares. Sin embargo, algunos
datos clnicos como, por ejemplo, disnea en presencia de una buena funcin sistlica o angina con coronarias angiogrficamente normales pueden
orientar hacia el diagnstico. Mucho ms especfica, sin embargo, sera la aparicin de sncope o presncope en individuos jvenes previamente
asintomticos. Un episodio de muerte sbita recuperada en un nio, joven, o adulto, sin duda obliga a descartar esta patologa. Tras la anamnesis, la
exploracin fsica dirigida puede ser diagnstica cuando se detecta una semiologa caracterstica de gradiente intraventricular dinmico, pero en caso
contrario, es poco reveladora. Con frecuencia, un electrocardiograma manifiestamente patolgico (ondas Q, hipertrofia ventricular, patrones de
preexcitacin o severas alteraciones de la repolarizacin), muchas veces en un individuo asintomtico, es la primera clave diagnstica.
CASO CLINICO (MIOCARDITIS)
Masculino de 38 aos de edad, sin antecedentes personales ni
familiares de inters, sin hbitos txicos, ni tratamiento habitual.
Refera un cuadro catarral de 7 das de evolucin tratado con
amoxicilina-clavulnico. Un da antes del ingreso present disnea
progresiva hasta la ortopnea, presentaba una tensin arterial
indetectable y una expectoracin espumosa, abundante y hemoptica,
junto a signos de insuficiencia respiratoria severa (taquipnea, cianosis,
crepitantes pulmonares generalizados y desaturacin del 70%). El ECG
mostraba una taquicardia sinusal a 180 lpm, con QRS ancho,
disociacin auriculoventricular y morfologa de bloqueo de rama
izquierda, criterios de taquicardia ventricular (TV) y, la radiografa
torcica, unos infiltrados algodonosos bilaterales. Analticamente,
salvo poliglobulia, acidosis metablica e intensa hipoxemia, no
presentaba otros hallazgos de inters incluyendo los enzimas
cardiacos.
PREGUNTA
Cual es la forma mas frecuente que debute esta patologia MCG?
RESPUESTA
a.- Taquicardia ventricular.
b.- Insuficiencia cardiaca.
c.- Infarto al miocardio.
d.- Bloqueo auriculoventricular completo el 5%.
CASO CLINICO
Paciente mujer de 18 aos de edad, natural y procedente de Lima.
Alergia a AINES. Acudi por presentar en forma brusca, malestar
general, nauseas, vmitos y deposiciones lquidas sin moco, ni sangre
(4 horas de inicio). En el examen de ingreso se constat que sus
funciones vitales estaban dentro de lmites normales. Se le diagnostic
gastroenterocolitis aguda y deshidratacin leve-moderada; en ese
sentido, y de acuerdo con los diagnsticos planteados, en el manejo
inicial se administr hidratacin endovenosa con cloruro de sodio al
9%, pargeverina clorhidrato 10mg y dimenhidrinato 50mg. Una hora
despus de la admisin, la paciente desarroll un cuadro de
hipotensin (presin arterial: 70/50 mmHg) y palidez marcada, recibi

CURSO ENARM CMN SIGLO XXI TEL: 36246001

tratamiento de soporte, pero ante el deterioro clnico paulatino la


paciente fue conducida a la unidad de cuidados intensivos.
PPREGUNTA
Cual es la imprension diagnostica inicial del caso?
RESPUESTA
a.- Estado de choque sptico.
b.- Estado de choque anafilctico.
c.- Estado de choque cardiogenico.
d.- Estado de choque hipovolemico.
CASO CLINICO (VIH + H1N1)
Un varn de 42 aos fue hospitalizado por fiebre y sntomas de dolor
torcico. Antecedente de (VIH), diagnosticado 15 aos antes del
ingreso. El recuento de clulas T CD4+ al ingreso era de 874 cl./l y la
carga viral era indetectable (< 50 copias/ml). En tratamiento con
tenofovir + didanosina + atazanavir + ritonavir, con una carga viral
indetectable en los ltimos 6 aos. El cuadro se inici con fiebre,
diarrea, mal estado general y mialgia de 1 semana de duracin. En los
2 das previos al ingreso sufri un dolor torcico punzante intenso y
progresivo, en reposo, con irradiacin a la espalda, que se agravaba
con el decbito y la inspiracin profunda. La intensidad del dolor
torcico disminuy tras la administracin de analgsicos intravenosos,
pero no con nitroglicerina sublingual. En la exploracin fsica, las
constantes vitales eran normales, excepto la temperatura corporal,
que era de 38 C. No presentaba signos de congestin sistmica o
pulmonar ni haba signos de bajo gasto ni dificultad respiratoria. Los
ruidos cardiacos eran normales. Los datos de laboratorio revelaron
leucocitosis con linfocitosis, protena C reactiva en 19 mg/ml y un pico
de troponina I cardiaca que alcanz 30 ng/ ml. La radiografa de trax
fue anodina. El ECG mostr un ritmo sinusal normal, con una elevacin
del segmento ST de 1 mm en las derivaciones DI, aVL y V4-V6.
PREGUNTA
Cual es el agente mas frecuente de esta patologia?
RESPUESTA
a.- Parvovirus B19.
b.- H1N1.

Pharmed Solutions Institute

PGINA 21

MANUAL DE TRABAJO DEL CURSO ENARM CMN SIGLO XXI


c.- VIH.
d.- Cocxackie B6.
CASO CLINICO (TAKOTSUBO)
Mujer de 43 aos con historia de epilepsia focal del lbulo temporal
izquierdo criptognica farmacorresistente, trastorno lmite de la
personalidad, etilismo crnico, infeccin por el virus de la hepatitis C.
Fue ingresada en Urgencias por presentar una crisis focal motora
secundariamente generalizada. Posteriormente, durante las primeras
24 horas de observacin, sufri cuatro crisis tonicoclnicas
generalizadas ms, la frecuencia cardaca durante la crisis descendi
hasta 35 latidos por minuto. Mostr tambin disnea y se objetiv en la
auscultacin cardiopulmonar crepitantes bibasales, bradicardia
extrema.
PREGUNTA
Considerando las manifestaciones clnicas cual de las siguientes
condiciones es la mas probable que presenta el paciente?
RESPUESTA
a.- Choque Distributivo.
b.- Choque Cardiogenico.
c.- Choque Hipovolemico.
d.- Choque Oculto.
CASO CLINICO (CHAGAS)
Mujer de 50 aos, procedente de zona rural. Present un cuadro
clnico de 15 das de evolucin consistente en fiebre, deposiciones
diarricas y, posteriormente, disnea, el cual la oblig a consultar en
varias ocasiones. La sintomatologa, especialmente la disnea, empeor,
por lo cual consult nuevamente, encontrndose en el examen fsico
signos claros de insuficiencia cardiaca. En la radiografa de trax se
7evidenci cardiomegalia y derrame pleural, y el ecocardiograma
report taponamiento cardiaco.
PREGUNTA
Cual es su conducta a seguir mas adecuada?
RESPUESTA
a.- Pericardiocentesis.
b.- Pericardiotomia.
c.- Diuretico, corticosteroide, b-bloqueador.
d.- Antiparasitario, pericardiocentesis.
CASO CLINICO
Hombre de 26 aos, procedente de zona rural. Consult por cuadro de
cinco das de fiebre, escalofros, dolores musculares, cefalea, artralgias,
astenia y adinamia. Se realiz estudio para hemoparsitos,
encontrndose ocasionales tripomastigotes. El ecocardiograma mostr
leve derrame pericrdico y leve dilatacin ventricular izquierda.
Adems, se encontr franca leucopenia (3.200 leucocitos/l) y
plaquetopenia (69.000 plaquetas/l).
PREGUNTA
Cual es la conducta farmacolgica especifica para el caso?
RESPUESTA
a.- Metronidazol.
b.- Albendazol.
c.- Benzonidazol.
d.- Prazicuantel.
CASO CLINICO (FEOCROMOCITOMA)

CURSO ENARM CMN SIGLO XXI TEL: 36246001

Una mujer de 41 aos con antecedente de HTA ligera no tratada acude


a urgencias de su hospital presentando dolor torcico de
caractersticas anginosas y elevacin del segmento ST en cara lateral.
Desde 3 aos antes sufra un trastorno de ansiedad que haba
requerido ingreso en una ocasin. En las ltimas semanas la paciente
sufra un gran estrs laboral. Presin arterial de 110/70 mmHg; tras
nitroglicerina sublingual, la clnica cedi y se normaliz el ECG. A las 12
h vuelve a tener dolor con elevacin del segmento ST, cateterismo
cardiaco normal, pero la ventriculografa muestra una disfuncin
medioventricular severa, con hipercontractilidad de los segmentos
basales y apicales. Se inici tratamiento con BB e IECA, manteniendo la
anticoagulacin y la aspirina. La paciente permaneci asintomtica,
con creatincinasa normal y troponina I de 1,4. A los 5 das del ingreso
se realiz una RM cardiaca que todava mostr una hipocinesia
medioventricular, sin realce tardo tras la administracin de gadolinio.
Al ingreso en nuestro hospital se haban solicitado catecolaminas en
sangre y en orina para una mejor caracterizacin del cuadro. stas
mostraron una elevacin supranormal (noradrenalina y adrenalina en
sangre > 5.000 y 190 pg/ml; noradrenalina y metanefrina en orina de
24 h, 582 y 5.386 g/24 h).
PREGUNTA
Cual es la conducta mas apropiada a seguir?
RESPUESTA
a.- Resonancia magnetica abdominal.
b.- Tomografia axial computada.
c.- USG abdominal.
d.- Tomografia helicoidal.
CASO CLINICO (PERIPARTO)
Mujer de 30 aos sin antecedentes de cardiopata que ingresa en
nuestro centro debido a un cuadro de insuficiencia cardiaca congestiva
un mes y medio despus de un parto sin complicaciones. Se realiz un
ecocardiograma en el que se objetiv dilatacin moderada del
ventrculo izquierdo (dimetro telediastlico, 64 mm; dimetro
telesistlico, 46 mm) junto con hipocinesia generalizada y fraccin de
eyeccin del ventrculo izquierdo del 32%; la aurcula izquierda estaba
ligeramente dilatada y las cavidades derechas, en el lmite superior de
la normalidad; se apreciaron regurgitaciones mitral y tricuspdea
moderadas-severas sobre vlvulas estructuralmente normales, con
estimacin de presin arterial pulmonar sistlica de 46 mmHg. Tras la
estabilizacin, se realiz una cardiorresonancia magntica, en la que se
confirm que el ventrculo izquierdo estaba moderadamente dilatado,
con fraccin de eyeccin del 37%, as como hipocinesia generalizada
del ventrculo derecho con funcin sistlica general severamente
deprimida (fraccin de eyeccin, 25%); en el estudio de retencin
miocrdica tarda de contraste con gadolinio, se observ un depsito
mesocrdico lineal a nivel septal extenso. En una coronariografa no se
objetivaron lesiones coronarias. Con el diagnstico de miocardiopata
periparto.
PREGUNTA
Cuales son los siguiente criterios no es til para establecer el
diagnostico de cardiomiopata periparto?
RESPUESTA
a.- Desarrollo de insuficiencia cardiaca en el ltimo mes de embarazo.
b.- Desarrollo en los 2 meses siguientes al parto.
c.- Ausencia de causa identificable para el desarrollo de insuficiencia
cardiaca.
D.- Ausencia de enfermedad cardiaca primaria en el ltimo mes de
embarazo.

Pharmed Solutions Institute

PGINA 22

MANUAL DE TRABAJO DEL CURSO ENARM CMN SIGLO XXI


ENDOCARDITIS
CIENCIAS BASICAS: La endocarditis infecciosa es una enfermedad causada por un agente microbiano que afecta la capa endotelial de estructuras
intracardiacas que invariablemente es fatal sin tratamiento. La infeccin ms frecuente reside en una o ms vlvulas que envuelven el endocardio mural,
miocardio y pericardio. Factores de riesgo: extraccin molar, ciruga periodontal, tonsilectomia, dilatacin esofgica, escleroterapia de varices esofgicas,
instrumentacin de via biliar, reseccin transuretral de prstata, litotripsia, dilatacin uretral, citoscopia, caterizacin cardiaca, implante de marcapasos,
los dispositivos intracardiacos o endovascular es una fuente de infeccin. SALUD PUBLICA: La mortalidad es del 25 % dentro de los 6 meses. La causa
inicial de EI era una complicacin de fiebre reumtica o de origen dental, actualmente la degeneracin de vlvulas cardiacas o dispositivos se han
incrementado. Los pacientes con DM, VIH, IRC esta en mayor riesgo de EI con riesgo nosocomial. De 70-75% de las endocarditis infecciosas estn
relacionadas con una cardiopata subyacente, cardiopatias de alto riesgo asociadas a EI: protesis valvulares, prolapso mitral, EI previa, ICC ciantica,
shunts pulmonares sistmicos. Cardiopatias de riesgo intermedio: prolapso mitral con insuficiencia, valvulopatia adquirida, miocardiopata hipertrfica,
cardiopatas congnitas. PATOGENIA: Llegada de microrganismos por via hematgena, la EI es una bacteremia persistente y continua endocardica o
endovascular. Las bacteriemias iatrogenas, en mucosas gingival, tracto respiratorio, GI y genitourinario. El agente patgeno lesiona estructuras cardiacas
y presenta capacidad de adhesin, y evitar la actividad inmunolgica del husped, generandoce vegetaciones (lesin caracterstica) en vlvulas, hay
acumulacin de detritus celulares, material trombotico y microorganismos. La etiologa depende de la puerta de entrada: cavidad bucal, piel y vas
respiratorias superiores=Streptococcus viridans, estafilococo, HACEK (Haemophilus, Actinobacillus, Actinomycetemcomitans, Cardiobacterium hominis,
Eikenella corrodens, Kingella Kingae). Tracto GI=Streptococos Bovis. Tracto genitourinario=Enterococcus. FORMAS CLINICAS: Endocarditis de vlvula
nativa, protsica, en adictos endovenosos, nosocomiales y con hemocultivos negativos (por tratamiento previo con antibiticos, microrganismos no
comunes o de crecimiento lento). DIAGNOSTICO: Clnico, la infeccion sistmica; se manifiesta con fiebre (80-85%), escalosfrios (40-75%), diaforesis
(25%), anorexia (25-55%), malestar general (25-40%), letargo, delirio, cefalea (15-40%), disminucin de peso (30%), mioartralgias, esplenomegalia (1550%), anemia, leucocitosis. En lesiones intravasculares; disnea (20-40%), tos (25%), dolor torcico (8-35%), debilidad focal, EVC isquemico (13-20%),
dolor abdominal, extremidades frias y dolorosas, soplos (80-85%), signos de insuficiencia cardiaca, manchas de Roth (4-10%), ndulos de Osler (7-10%),
lesiones de Janeway (6-10%), hemorragias en astilla (5-15%), EVC, aneurisma micotico, hematuria. En reacciones inmunitarias; mioartralgias (15-30%),
mialgias, tensosinovitis, artritis, signos de uremia, acropaquias (10-20%), proteinuria, hematuria, acidosis, factor reumatoide positivo. 1. Sospecha clnica,
2. Aislamiento del germen, 3. Ecocardiograma. Categorias diagnosticas de Duke: I) Endocarditis infecciosa definitiva: 1 Criterio patolgico
(microorganosmo en cultivo o histologa en la lesion y/o lesiones patolgicas vegetacin o absceso). 2 Criterio clnico (dos criterios mayores un criterio
mayor y tres menores cinco criterios menores). II) Endocarditis infecciosa posible. III) Diagnostico rechazado (firme alternativa diagnostica de las
manifestaciones resolucin de manifestaciones en menos de 4 dias de tratamiento falta de evidencia patolgica en ciruga o autopsia, con menos de
4 dias de antibiticos). Criterios mayores de Duke: 1) Hemocultivos positivos: microorganismo tpicos para endocartitis infecciosa, en dos hemocultivos
separados (Streptococos bobis, viridans, Sthaphylococcus, enterococcus, HACEK) hemocultivos peristentemente positivos, con microorganosmo
compatibles con endocarditis infecciosa separados por 12 hrs. 2) Evidencia de dao endocardico: A. Ecocardiograma positivo para masa oscilante en
ausencia de una explicacin anatmica, absceso, nueva deshiscencia parcial de valvula protsica. B. Nueva insuficiencia valvular (no es suficiente soplo
cambiante). Criterios menores de Duke: 1) Predisposicion, 2) Fiebre, 3) Fenomenos vasculares (embolos arteriales mayores, infartos pulmonares
spticos, aneurisma micotico, hemorragia intracraneal, hemorragia conjuntival, lesiones de Janawey), 4) Fenomenos inmunolgicos (glomerulonefritis,
ndulos de Osler, manchas de Roth, factor reumatoide), 5) Evidencia microbiolgica ( que no llena criterio mayor), 6) Ecocardiograma (que no llena
criterio mayor). Estudios para endocarditis de vlvulas nativas se inicia con ecocariograma transtoracica, para valvula protsica, se debe hacer
ecocardiograma transtoracica y transesofagica. TRATAMIENTO: Para endocarditis con valvula nativa debida a estreptococcus; en <65 aos y valores
normales de creatinina Penicilina G 12-20 U + Gentamicna 3mg/Kg/24h IV dividido en 2-3 dosis. Pacientes >65 aos, valores elvados de creatinina,
Penicilina G adaopatada a funcin renal o Ceftriaxona. Alrgicos a penicilinas o cefalosporinas, Vancomicina 30mg/kg/24hrs, si hay resistencias se puede
sumar gentamicina. Tratamiento de endocarditis en vlvula nativa por estafilococcus Oxacilina 8-12grs/24 + gentamicina. En alergia a la penicilina
vancomicina + gentamicina. En pacientes con valvula protesica: oxacilina+ rifampicina+ gentamicina. Tratamiento cuando los hemocultivos son negativos
o la terapia es urgente cuando no estn bien identificados Vancomicina 15mg/kg IV c/12 hrs por 5 semanas + gentamicina 1mg/kg IV c/8hrs por 2
semanas. Tratamamiento para endocarditis por hongo, anfotericina B, fluconazol o itraconazol. PREVENCION: Procedimientos dentales, orales,
respiratorios y esofgicos: Amoxicilina o ampicilina 2 gr VO o IV 2 hrs de 30min a una hora antes del procedimiento, alrgicos a penicilina usar
clindamicina 600mg o azitromicna 500mgs una hora antes del procedimiento.Procedimientos de tracto GI, y GU: ampicilina o amoxicilina 2 gr IV +
gentamicina 1.5mg/kg IV de 30min a 1 hora antes del procedimiento, si es de alto riesgo , si es moderado solo la ampi o amoxicilina. ENDOCARDITIS DE
VALVULAS NATIVAS: Aguda: se manifiesta con sndrome febril, de corta evolucin, con SRIS o sepsis, en los primeros 7 dias es cuando hay mayor
mortalidad. Subaguda: inicio insidioso, sndrome febril de semanas a meses, en ocasiones manifestaciones autoinmunes. Etiologia: Streptococo ssp 4565% (S. viridans 30-40%, nativo de orofaringe, entra por dao dental o gingival, primera causa en vlvulas protsicas tardias), Sthaphylococcus aureus 1027% (causa aguda , coloniza nasofaringe, factores de riesgo, dilisis, diabetes, quemaduras, VIH, usuarios de drogas, alteraciones dermatolgicas
crnicas), Enterococcus ssp 5-18% (E. fecalis, ingresa por manipulacin con sonda Foley o colonoscopia), Estafilococo coagulasa negativa de 1-3%
(principalmente en vlvulas protsicas, precozmente), bacilos gram negativos HACEK 1.5-13%. En usuarios de drogas IV, el principal agente es
Sthaphylococcus aureus, Estafilococo coagulasa positivo, la valvula mas afectada es la tricspide. La Salmonella es el principal agente en pacientes con
SIDA. INDICACIONES DE CIRUGIA en endocarditis de valvula nativa: 1. IC caradiaca por regurgitacin aortica o mitral aguda. 2. IRA aguda + taquicardia y
estenosis temprana de la valvula mitral. 3. Endocarditis por hongos (evidencia de disfuncin valvular y perisistencia de la infeccion 7-10 dias con tx.,
antimicrobiano adecuado, indicado por fiebre, leucocitosis, bacteriemia, sin causa no cardiaca atribuible). 4. Conformacion de absceso artico y/o
pseudoaneurismas del seno artico. 5. Embolia aortica recurrente a pesar de tratamiento antimicrobiano. 6. Infeccion por gramm negativos o
microorganismos con pobre respuesta al tratamiento y evidencia de disfuncin valvular. 7. Vegetaciones mviles >10mm. 8. Infeccion temprana de la
valvula mitral que puede normalmente ser reparada. 9. Hiperermia persistente mas leucocitosis y cultivos negativos.
CASO CLINICO
Mujer de 46 aos, afectada de nefropata lpica IV, en programa de
diaslisis peritoneal. Desde entonces mantiene marcadores positivos.
Tuvo un brote cutneo-articular, por lo que reciba con micofenolato
sdico a dosis de 180 mg y prednisona a dosis de 5 mg diarios. Ingresa
por disnea y malestar general progresivo, de 15 das de evolucin.
Dolor torcico en el hemitrax izquierdo que aumenta con la
inspiracin profunda y que mejora relativamente en anteversin. No
refiere sndrome febril, ni otra clnica acompaante. EF soplo diastlico

CURSO ENARM CMN SIGLO XXI TEL: 36246001

en el foco artico, irradiado a las cartidas, con roce pericrdico


importante, sin signos de fallo cardaco. El resto de la exploracin fsica
fue anodina. En la analtica destacan: leucocitos 21,3x103/mm3,
neutrfilos 92%, linfocitos 3,0%, PCR 17,73mg/dl, procalcitonina 4,84
ng/ml (<0,5). Anticuerpos antinucleares (IFI) 320-640 u arb (0-80),
anticuerpos anti-ADN (EIA) 1,3/ml (<10 U/ml), anticuerpos anti-ADN
(IFI) <80, anticuerpos cardiolipina (IgG) 3,5 U GPL/ml, IgM 2,5 U GPL/ml
(negativos), urea 127, creatinina 8,46 mg/dl, hemoglobina (Hb) 9,6

Pharmed Solutions Institute

PGINA 23

MANUAL DE TRABAJO DEL CURSO ENARM CMN SIGLO XXI


g/dl, hematocrito 29%. Tiempo de cefalina 27,9/30 segundos (29-31 s),
anticoagulante lpico positivo.
PREGUNTA
Cual es la conducta antibitica empirica mas adecuada?
RESPUESTA
a.- Vancomicina, ceftriaxona y dicloxacilina.
b.- Amilacina, doxiciclina y ceftaxidima.
c.- Vancomicina, ceftazidima y gentamicina
d.- Gentamicina, vancomicina y imipenem.
CASO CLINICO ENDOCARDITIS
Varn de 38 aos de ocupacin pintor, con signos de disnea, ortopnea,
edemas en miembros inferiores, adems de registros febriles aislados
de 2 meses de evolucin. Relat tambin la presencia de un soplo
cardaco desde haca 3 aos sin estudio, enfermedad periodontal,
caries dentales y otros antecedentes sin relevancia para el caso.
Negaba contacto con animales de cra o consumo de productos sin
pasteurizacin. Dos semanas antes de la consulta present tos con
expectoracin purulenta, que cedi espontneamente. Dos horas
antes de la consulta present dolor precordial, opresivo, intensidad
8/10, con irradiacin a brazo izquierdo. El ecocardiograma
transtorcico, la presencia de dilatacin de la aurcula izquierda, dos
imgenes ecodensas sobre cara ventricular de la vlvula artica
bicspide, compatibles con vegetaciones de 28,4 mm x 9 mm y de 24
mm x 6 mm cada una, insuficiencia valvular artica grave, ventrculo
izquierdo dilatado, hiperdinmico, con signos de sobrecarga de
volumen, funcin sistlica conservada y leve derrame pericrdico,
todos signos compatibles con endocarditis infecciosa.
PREGUNTA
Cual de los criterios es el indicado para endocarditis bacteriana?
RESPUESTA
a.- Criterios de Jones.
b.- Criterios de duke.
c.- Criterios de Baltazar.
d.- Criterios de Hermosillo.
PREGUNTA
Se aislo Brucella canis en los cultivos. Cual es la conducta teraputica
mas adecuada?
RESPUESTA
a.- Doxiciclina, rifampicina y trimetoprim + sulfametoxazol.
b.- Vancomicina, rifampicina y doxiciclina.
c.- Ceftriaxona, doxiciclina y trimetoprim + sulfametoxazol.
d.- Imipenem, doxiciclina y rifampicina.
CASO CLINICO
Paciente de 61 aos de edad con antecedentes de hemoptisis hace 20
aos, diabetes mellitus no insulnodependiente en tratamiento con
antidiabticos orales, faringitis crnica, anemia ferropnica en
tratamiento con hierro. No hipertensin arterial. Alrgico a penicilinas.
Adems est siendo tratado con antidepresivos. Exfumador. Bebedor
moderado. Acude por un cuadro de odinofagia que comenz hace una
semana. Desde hace 3 meses presenta febrcula, cansancio
generalizado y anorexia. Rectorragia aislada. No regurgitacin ni
pirosis. Hace aproximadamente 45 das le aparecieron manchas
violceas de pequeo tamao, no dolorosas y muy abundantes en
extremidades inferiores que fueron disminuyendo en intensidad y
tamao hasta llegar a desaparecer muchas de ellas; tambin se aprecia
un ndulo en pulpejo de tercer dedo de la mano derecha doloroso a la
palpacin. EF: En faringe hay lesiones candidisicas, enrojecimiento y
edema; dolor a la palpacin en regin submandibular y tiroidea.
Auscultacin cardiaca: ruidos cardiacos rtmicos; soplo pansistlico
eyectivo multifocal. Auscultacin pulmonar: ruidos respiratorios
conservados; no roncus, ni crepitantes ni sibilancias. BH: leucos
16.400, hemates 4,160.000, hemoglobina 10,9, hto 32,1, VCM 77,2,

CURSO ENARM CMN SIGLO XXI TEL: 36246001

plaquetas 360.000, TP 64%. Bioqumica: glucosa 123; funcin renal


normal; bilirrubina normal; triglicridos, colesterol y enzimas hepticas
normales; fosfatasa alcalina 306; LDH e iones normales; hierro 30.
PREGUNTA
Cul de los siguientes agentes, esperara
probablemente en el cultivo de la paciente?
RESPUESTA
a.- Staphylococcus aureus
b.- Grupo HACEK
c.- Streptococcus ssp
d.- Enterococcus ssp

encontrar

ms

PREGUNTA
Qu criterios mayores de Duke debe de cumplir esta paciente para
que sea una endocarditis bacteriana definitiva?
RESPUESTA
a.- 2 hemocultivos positivos separados de patgenos tpicos +
Ecocardiograma positivo con masa oscilante en ausencia de una
explicacin anatmica
b.- Fiebre + 2 hemocultivos positivos separados de patgenos tpicos
c.- Hemocultivos persistentemente positivos, con microorganosmos
compatibles + glomerulonefritis, ndulos de Osler
d.- Ecocardiograma positivo con masa oscilante en ausencia de una
explicacin anatmica + glomerulonefritis, ndulos de Osler
PREGUNTA
Cul es la conducta teraputica ms adecuada para este paciente?
RESPUESTA
a.- Penicilina G + Gentamicina por 2 semanas
b.- Ceftriaxona + Gentamicina por 2 semanas
c.- Vancomicina por 4 semanas
d.- Teicoplanina + Estreptomicina
CASO CLINICO
Se trata de un paciente de sexo masculino, de 51 aos que ingres a
urgencias por 10 das de evolucin de fiebre de 40C, asociada a
diaforesis y deposiciones lquidas sin moco ni sangre. Colecistectoma
por laparoscopia 41 das antes, que requiri hospitalizacin por 31 das
para el tratamiento de una infeccin del sitio operatorio con
compromiso, tratada con meropenem. No se haba realizado
monitorizacin hemodinmica invasiva y no tena antecedentes de uso
de drogas ilcitas intravenosas. EF: se encontraba alerta, orientado, con
FC 72 lpm, FR 16x, TA de 95/57mmHg, T38,5C y deshidratacin grado
I. Se auscult un soplo holosistlico en el foco artico, grado II/VI. En el
abdomen haba dolor a la palpacin en epigastrio, sin signos de
irritacin peritoneal. En los exmenes de laboratorio con anemia
normoctica, normocrmica, elevacin de la PCR y leve aumento de las
transaminasas, y fosfatasa alcalina y amilasa en rango normal.
PREGUNTA
Cual de siguientes factores de riesgo asociados para la patologia
actual es menos frecuente?
RESPUESTA
a.- Cardiopata congnita cianosante.
b.- Comunicacin interventricular.
c.- Conducto arterioso permeable.
d.- Presencia de vlvulas protsicas.
PREGUNTA
Entre los siguientes signos. Cual es el menos frecuente en la patologia
actual?
RESPUESTA
a.- Embolismo.
b.- Esplegnomegalia.
c.- Aneurisma micoticos.
d.- Lesiones en retina

Pharmed Solutions Institute

PGINA 24

MANUAL DE TRABAJO DEL CURSO ENARM CMN SIGLO XXI


PERICARDITIS
CIENCIAS BASICAS: El pericardio consiste en una capa visceral y una lamina parietal que entre si forma un espacio potencial, contiene 50 ml de plasma
infiltrado, sus funciones limitan la distencin cardiaca, facilita el acople e interaccion cardiaca, mantienen el volmen de presin cardiaca, la ergometra
del ventrculo izquierdo, adems de que lubrica, minimiza la friccion, regula la inercia, las fuerzas hidrosttica, es una barrera mecnica contra la
infecccion, presenta acciones inmunolgicas, vasomotoras, fibrinoliticas, modula la estructura del miocito, su funcin, asi como su expresin gnica, es
un vehiculo para frmacos y terapia gnica. No es escencial para la vida, sin embargo puede comprometerlo al presentar una patologia. PERICARDITIS
AGUDA: Es un estado agudo fibrinoso o sudativo por pericarditis, es un Sx caracterizado por dolor retroesternal y precordial izq., que se irradia a cuello y
trapecio izquierdo, en ocasiones dolor en epigastrio, aumenta en posicin supina, tos, inspiracin profunda, disminuye al reposo. Friccin o roce
pericardico, cambios ECG, depresin del segmento PR, en DII, aVF, V4 a V6, elavacion del segmento ST de concavidad superior, difusos, depresin de ST
en aVR y V1, alteraciones de la repolarizacion como en el IAM, aun cuando estos son difusos. RX puede ser normal.El diagnostico es clnico, un
ecocardiograma normal, no excluye el diagnostico, su utilidad es para detectar derrame pericrdico. Los marcadores inflamatorios puede incrementarse,
en caso de isoenzimas cardiacas es mas frecuente en epicarditis, pueden elevarse semejando IAM, la hospitalizacin de estos casos son para diferenciar
los casos. Tratamiento es reposo, acido acetilsalicilico 500mg c/6hrs, por 2 semanas, si no hay respuesta indometaciona 25-50mg. Prednisona solo en
caso de dolor intenso y fiebre alta o mas de 7 dias de evolucion trpida, hay que evitar su uso al mximo. En pericarditis benigna recidivante, usar
colchicina durante 1 ao. En la pericarditis de larga evolcuion (<1ao), usar azatriopina y pericardiectomia. Pericardiocentesis; infeccin aguda grave
asociada a evidencia de derrame pericrdico, presencia de taponamiento pericrdico severo, taponamiento perisitente o recidiva. El dolor permanece 1
a 2 dias, en caso viral se prolonga a mas de 4 dias o en caso de drogadiccin el dolor es mas severo, requiririendo corticoides sistmicos. DERRAME
PERICARDICO: Es el acumulo de transudado, exudado o hemosudado en el saco pericardico frecuente en las complicaciones de pericarditis aguda,
pericarditis crnica (>3 meses) frecuente en retencin de agua y sodio, o procesos inflamatorios crnicos, la presencia hemtica o sero hemtica es mas
frecuente en infecciones o inflamaciones, en pericarditis quilosa lesiones u obstruccion del conducto torcico, por coresterol, hipotiroidismo, artritis
reumtica, o tuberculososis. Hay aumento de la presin intrapericardica, puede presentarse por obstruccin de estructuras vecinas en la pericarditis
cronica, disfagia, tos, disnea, afona. El ECG; reduccin del voltaje QRS, aplanamiento de ondas T, casos sevros: alternacia lecetrica, la ecocardiografa
tcnica de eleccin para diagnostico. Rx: derrame pleural izquierdo, agrandamiento de silueta cardiaca, silueta cardiaca en forma globular en botella. La
TAC y el IRM son utiles para identificar la inoculacin tpica o atpica, la etiologa del derrame pericardico deber ser identificado por citologa, anlisis
inmunolgico o por biopsia guiada. La pericardiocentesis es diagnostica y teraputica, ya que evita la falla del corazn hemodinamicamente hablando,
disfuncin atrial y ventricular diastlica, generando pulso paradojico, hipotensin arterial, la fusin cardiaca decrese, hasta llegar al prolapso y falla
cardiaca por agotacion de los mecanismos compensatorios. TAMPONADE CARDIACO: Antecede derrame periocardico agudo o cronico, se observa como
una alteracin dinmica continua, con presiones de 10 a 20 mmHg como mximas, en los casos leves es asintomtica, en la moderada el disconfort y la
disnea, dolor toracico predominan, triada: ingurgitacion yugular, pulso paradjico, hipotensin arterial, tonos cardiacos apagados, se presenta bajo
voltaje en el ECG, durante la inspiracin disminuye. Cuadro severo, hipotensin, bajo gasto, taquicardia y taquipnea. Se debe realizar pericardiocentesis
cuando los sntomas los amerite, a los 50 ml ya iniciaran, sin embargo se recomienda cultivo, citologa. PERICARDITIS CONSTRICTIVA: Es una condicin
calcificante (pericardio fibrotico, engrosado y adherido) que limita la funcin diastlica de los ventrculos, idioptico principalmente, trauma cardiaco,
quirrgico o tuberculoso otras infecciones, neoplasias, radioterapia, falla renal y enfermedades del tejido conectivo como menor causa. La condicin
crnica congestivo asemeja enfermedad miocrdica y enfermedad heptica crnica. El paciente refiere fatiga, disnea, tos y ortopnea (por congestin
pulmonar), aumento de peso, disconfort abdominal y nausea. Puede llegar a ascitis, hepatoesplenomegalia, edema confundiendoce con cirrosis, signo de
Kussmaul, Knock, tercer ruido, pulso paradjico. QRS de bajo voltaje, cambios en P y T, fluter atrial es comn, la TAC e IRM define entre pericarditis
restrictiva y cardiomiopata contrictiva, pueden necesitar pericardiectomia con una mortalidad del 30 al 40 %. PERICARDITIS UREMICA: En insuficiencia
renal grave relacionada a la hiperazoemia. Asociada a dilisis insuficiente o hiperhidratacion. Trasplante renal se asocia a rechazo.
CASO CLINICO
Varn de 48 aos, fumador. Haba participado recientemente en la
recogida de restos de aves enfermas sacrificadas en granjas. Consult
en urgencias por dolor retroesternal que aumentaba con la inspiracin
profunda y la tos empeoraba con el decbito. En la exploracin fsica
destacaba el hallazgo de febrcula (37,5 C) y roce pericrdico. En el
electrocardiograma y en la radiografa de trax no se observaron en
aquel momento hallazgos patolgicos. Con el diagnstico de
pericarditis aguda el paciente fue remitido a su domicilio bajo
tratamiento con cido acetilsaliclico a dosis antiinflamatorias. Tras una
mejora inicial, consult diez das ms tarde por fiebre de 39 C, tos
con escasa expectoracin mucosa y dolor torcico de caractersticas
pericardticas, de tres das de evolucin.

alteraciones. Refiere en las 6 semanas antes del ingreso astenia


progresiva, con posterior aparicin de fiebre y tos seca. No presentaba
disnea, dolor torcico ni alteracin en la diuresis. En la exploracin
tena una presin arterial (PA) de 190/80 mmHg, estaba eupneica, con
presin venosa yugular elevada, auscultacin cardaca rtmica con roce
pericrdico, normoventilacin en ambos hemotrax y ausencia de
edemas. En la analtica, presentaba una hemoglobina de 7,7 mg/dl,
urea de 217 mg/dl, creatinina 4,5 mg/dl, potasio 3,6 mEq/l, saturacin
basal de O2 del 98%; en orina, tena proteinuria de +++ y 60
hemates/campo. En el ECG no haba alteraciones. En la radiografa de
trax se observ una gran cardiomegalia, se solicit una
ecocardiografa de urgencia, en la que se evidenci un derrame
pericrdico moderado-grave.

PREGUNTA
Cual de las siguientes manifestaciones es menos probable encontrar?
RESPUESTA
a.- Roce pericrdico.
b.- Alteracin difusa de la repolarizacin con aplanamiento de las
ondas T.
c.- Radiografa de trax con cardiomegalia e infiltrado pulmonar en la
lngula.
d.- Determinacin seriada de creatincinasa y de su fraccin MB
elevadas.

PREGUNTA
Cual es la conducta mas apropiada a seguir?
RESPUESTA
a.- Pericardiocentesis
b.- Pericardiectomia.
c.- Diuretico, esteroide.
d.- Esteroide, ciclofosfamida.

CASO CLINICO
Mujer de 60 aos con DM diagnosticada 3 aos, HTA de 10 aos de
evolucin. Un ao antes tena una creatinina de 0,8 mg/dl; FGE
(MDRD) >60 ml/min/1,73 m; sistemtico y sedimento de orina sin

CURSO ENARM CMN SIGLO XXI TEL: 36246001

CASO CLINICO
Mujer de 69 aos de edad que sufre un trauma torcico cerrado por
compresin antero posterior al quedar atrapada por las puertas de un
autobs y 30 das despus comienza a presentar falta de aire a los
esfuerzos, aumentando progresivamente hasta desencadenarse a los
pequeos esfuerzos. Ingresa en el hospital con diagnstico de

Pharmed Solutions Institute

PGINA 25

MANUAL DE TRABAJO DEL CURSO ENARM CMN SIGLO XXI


cardiopata isqumica. Se realiza ecocardiograma y se comprueba gran
coleccin lquida pericrdica.
PREGUNTA
Cual de las siguientes manifestaciones es mas frecuente encontrar
para establecer el diagnostico actual?
RESPUESTA
a.- Triada de Beck
b.- Signo de Kussmaul.
c.- Disminucin del voltaje de QRS.
d.- Elevacin de CPK-MB y Troponinas.
PREGUNTA
Considerando la fisiopatogenia del presente caso, que tipo de choque
es mas frecuente que presente?
RESPUESTA
a.- Choque Distributivo.
b.- Choque Obstructivo
c.- Choque Cardiogenito.
d.- Choque hipovolemico.
CASO CLINICO
Varn de 55 aos con recambio valvular mitral (1998) y sndrome
pospericardiotoma, que curs con fiebre y cansancio a mnimos
esfuerzos. Posteriormente qued asintomtico. En abril de 2004
present derrame pericrdico severo y taponamiento cardiaco.
Requiri pericardiocentesis. Tras sta, qued inicialmente
asintomtico. Reingres en julio de 2004 y refieri que tras el alta
clnica se sinti bien, pero que en pocas semanas comenz a sentir
cansancio a grandes y luego a moderados esfuerzos. En el momento de
su reingreso la presin arterial (PA) era 110/70 mmHg, sin pulso
paradjico, pero presentaba ingurgitacin yugular. La auscultacin
cardiaca era rtmica, con 70 lat/min y ruidos protsicos normales.
Presentaba una discreta hepatomegalia y edemas con fvea en los
miembros inferiores.
PREGUNTA
Cual de las siguientes afirmaciones no es correcta para el caso?
RESPUESTA
a.- Es una infrecuente forma de sndrome pericrdico.
b.- Donde la constriccin cardiaca ocurre en presencia de derrame
pericrdico significativo.
c.- Los pacientes presentan taponamiento cardiaco, sin sintomticos
pospericardiocentesis
d.- sta se resuelve slo tras pericardiectoma.
PREGUNTA
Cual de las siguientes manifestaciones no es propio de la pericarditis?
RESPUESTA
a.- Localizacin retrosternal sbito.
b.- Naturaleza pleurtica.
c.- Se exacerba con la inspiracin.
d.- Disminuye al inclinarse.
PREGUNTA
Cuales no son cambios que se presentan en ECG del paciente?
RESPUESTA
a.- Elevacin del segmento ST de forma cncava.
b.- Hay desarrollo de ondas Q.
c.- El voltaje de la onda R se mantiene.
d.- Depresin del segmento PR.

RESPUESTA
a.- Idiopatico.
b.- Traumatico.
c.- Bacteriano.
d.- Viral.
PREGUNTA
Cual de no es una secuela de la patologia del paciente?
RESPUESTA
a.- Taponade cardiaco.
b.- Pericarditis recurrente.
c.- Prericarditis restricitva.
d.- Pericarditis constrictiva.
CASO CLINICO
Mujer de 30 aos que, 15 das despus de un cuadro gripal, ingres en
nuestro hospital por presentar un cuadro de dolor precordial agudo
irradiado a omplato que aumenta con la inspiracin y al deglutir y se
alivia al incorporarse en la cama y ligero dolor en epigastrio. En el
momento de la exploracin la presin venosa yugular era normal.
Presentaba una tensin arterial (TA) de 130/70mmHg y una FC de 100
lpm. A la auscultacin cardaca presenta un suave roce en mesocardio.
Auscultacin pulmonar normal. Como exploraciones complementarias
se practic un ECG en el que se observaba una ligera elevacin del ST
en todas las derivaciones. La radiografa de trax presentaba un ndice
cardiotorcico normal. En los datos de analtica destaca leucocitosis
moderada y VSG de 50 mm en la primera hora. Se le practic tambin
un ecocardiograma observndose mnimo derrame pericardio.
PREGUNTA
Cul es la conducta teraputica ms adecuada para este paciente?
RESPUESTA
a.- Reposo, MONA, trombolizar
b.- Reposo, salicilatos
c.- Reposo, Morfina, oxigeno, nitratos
d.- Reposo, indometacina, paracetamol
PREGUNTA
Tres meses despus la paciente acude a Urgencias por presentar un
cuadro de similares caractersticas al ingreso previo, pero de
intensidad moderada, y adems se agreg fiebre de 38C. En este
ingreso se realizaron las siguientes exploraciones complementarias:
Mantoux, 3 baciloscopias en esputo, hormonas tiroideas, factor
reumatoide, ASLO, anticuerpos antinucleares, serologa a diferentes
virus y pruebas de funcin heptica y renal, que fueron todas negativas
o normales. Durante el ingreso se practic un nuevo ecocardiograma
en el que se observaba un derrame pericrdico leve. Cul es la
conducta teraputica ms adecuada a seguir?
RESPUESTA
a.- Reposo, indometacina, pericardiectomia
b.- Reposo, indometacina, colchicina
c.- Reposo, salicilatos, prednisona
d.- Reposo, aziatropina, pericardiectomia
PREGUNTA
Si la paciente presentara eventos similares aproximadamente cada 2
meses, en un ao. Cul sera el manejo ms adecuado a seguir?
RESPUESTA
a.- Reposo, indometacina, pericardiectomia
b.- Reposo, indometacina, colchicina
c.- Reposo, salicilatos, prednisona
d.- Reposo, aziatropina, pericardiectomia

PREGUNTA
Considerando los antecedentes del caso clnico cual es la etiologa
mas frecuente?

CURSO ENARM CMN SIGLO XXI TEL: 36246001

Pharmed Solutions Institute

PGINA 26

MANUAL DE TRABAJO DEL CURSO ENARM CMN SIGLO XXI


BRONQUITIS:
CIENCIAS BASICAS: La bronquitis aguda, es un termino que define a la inflamacin de la traquea, bronquios y bronquiolos. SALUD PUBLICA: Tiene
etiologa viral principalmente como; virus de la influenza tipo A y B, paraiinfluenza, virus sincitial respiratorio, adenovirus, rinovirus. Los agentes
infecciones no virales como; Mycoplasma pneumoniae, Bordetella pertusis, Chlamydia pneumoniae, Streptococcus pneumoniae, H. influenzae.
DIAGNOSTICO: Se caracteriza por tos con o sin esputo, por al menos 3 semanas de evolucin. Hacer historia clnica (investigando el uso de tabaco) y
exploracin fsica. Bronquitis aguda no complicada; tos <de 3 semanas, paciente inmunocompetente, afebril, no comorbilidades, adulto joven. Bronquitis
aguda complicada; tos >3 semanas, paciente inmunodeprimido, fiebre, comorbilidades, anciano. Criterios de probabilidad de bronquitis aguda; infeccion
respiratoria aguda, manifestada principalmente por tos, con o sin produccin de esputo, por al menos 3 semanas. No evidencia clnica de neumona,
descartar la presencia de resfriado comn, esofagitis por reflujo, asma aguda o exacerbacin de EPOC. TRATAMIENTO: Noprecisa tratamiento antibitico
la mayora de las veces, ya que la etiologa principal es viral, nicamente justificado con estudio microbiolgico. Dentro del manejo sintomtico primario
debe considerarse: 1. Dejar de fumar y evitar ambientes donde se fumen. 2. Buena hidratacin e incremento de la humedad. 3. Limitar la diseminacin
de la infeccion viral (lavado de manos).

NEUMONIA ADQUIRIDA EN LA COMUNIDAD (NAC), NEUMONIA INTRAHOSPITALARIA (NIH)


CIENCIAS BASICAS: NAC: Proceso infeccioso agudo del parnquima pulmonar adquirido fuera del entorno hospitalario, se relaciona con sntomas
y signos pleuropulmonares que acompaan casi siempre a infiltrados recientes observados en una radiografa de trax. Factores de riesgo: Edad>
65 aos, EPOC, tabaquismo, alcoholismo, Enf. cardiovascular, DM, hepatopata e Insuf. Renal, aspiracin, exposicin ambiental a aves, IVRS previas, Enf.
Estructural pulmonar, Obstruccin de la va area. NIH: Se desarrolla 48-72hrs de ingreso hospitalario y que no se encontraba en fase de incubacin en el
momento del internamiento. ETIOLOGA: Streptococcus pneumoniae 50-65%; Haemophilus influenzae 8-10%; Staphylococcus aureus 3-5% (abseso
pulmonar), otros Klebsiella (1%), Escherichia coli (1%), Moraxella catharralis, Mycoplasma pneumoniae,
TABLA 1 Pneumonia Severity Index
Caractersticas del paciente
Puntaje
legionella pneumophila, Chalmydia pneumoniae, pesudomona auregunosa, anaerobios (aspiracin,
Caractersticas demogrficas
obstruccin de va area). SALUD PUBLICA: Es una causa de frecuente de morbi-mortalidad en la
Sexo masculino
Edad en aos
poblacin general. Incidencia de 2-10 casos/1000 habitantes/ao. 20-35% requieren ingreso
Sexo femenino
Edad en aos 10
hospitalario. Mortalidad del 5% de hospitalizados y de un 35% ingresados a la UCI. La mortalidad oscila
Asilo
+10
entre un mximo del 61% para las NAC debidas a Pseudomonas y un 35% para las producidas por
Comorbilidades
Enf. Neoplsica
+30
enterobacterias, Staphylococcus aureus y las de etiologa mixta. Grupos de riesgo y agentes: Mayores
Enf. Heptica
+20
de 65 aos; Streptococcus pneumoniae, H. influenzae, Moraxella. EPOC, fumadores; S. pneumoniae, H.
Enf. Cardiaca
+10
influenzae. Ma catarhalis, pseudomona. Alcoholismo; S, penumoniae, anaerobios, Klebsiella. Aspiracion;
Enf. cerebrovascular
+10
anaerobios. Exposicion a aves; C. psittacci. IVRS; Influenza, S. aureus, S. pneumoniae. Obstruccion de la
Insuficiencia renal
+10
Exploracin fsica
via area; Anaerobios, S. pneumoniae. Asilos, esidencias; S. pneumoniae, H. influenzae. Las tasas de
Alteracin del estado mental
+20
mortalidad por NAC la sitan en el quinto lugar como causa ms frecuente de muerte en los pases
FR > 30rpm
+20
industrializados, tras las enfermedades cardiovasculares, neoplsicas y cerebrovasculares y la
P.sistlica <90mmHg
+20
enfermedad pulmonar obstructiva crnica (EPOC). 60-70% de todos los casos de NAC son originados por
Temp. <35C o >40C
+15
FC >125 lpm
+10
Streptococcus pneumoniae. En los ltimos aos se ha informado la aparicin de Staphylococcus aureus
Estudios de laboratorio y Rx
resistente a la meticilina como causa de infecciones extrahospitalarias graves. PATOGENIA: Vas de
pH arterial <7.35
+30
entrada: Aspiracin de organismos que colonizan la orofaringe (mecanismo ms comn, sobre todo en
BUN > 10.7 mmol/l
+20
ancianos, alcoholicos, residentes de asilos, DM; mas propensos a infecciones por gram(-), provenientes
Na < 130 mEq/l
+20
Glucosa >250 mg/dl
+10
del estmago, comida contaminada), inhalacin de aerosoles infecciosos (son partculas menores a
Hto. <30%
+10
10micras, como Coxiella, Legionella, Mycoplasma), diseminacin hematgena (por el embolismo
Derrame pleural
+10
sptico a partir de focos de infeccin distantes, adictos a drogas parenterales, endocarditis bacteriana,
TABLA 2 ESCALA DE RIESGO Y MORTALIDAD DE PNEUMONIA SEVERITY INDEX
colonizacin de catteres intravenosos, S. aureus. patgeno ms frecuente), inoculacin
Puntaje total
Riesgo
Clase
Mortalidad
Lugar de Tx.
directa (es muy rara, el microorganismo traspasa las barreras y alcanzan espacios
S/factores de riesgo
Bajo
I
0.1
Domicilio
alveolares). Hay factores que inhiben la actividad mucocoiliar (epitilio cilndrico ciliado
<70
Bajo
II
0.6
Domicilio
71-90
Bajo
III
2.8
Intrahospitalario
pseudoestratificado) como: humo del tabaco, el aire fro, frmacos (anestsicos), xido
91-130
Moderado
IV
8.2
Intrahospitalario
de azufre, xido de nitrgeno, fibrosis qustica. Factores que incrementan el riesgo para
>130
Severo
V
29.2
UCI
adquirir pseudomona; bronquiectasias, corticoesteroides, desnutricin, terapia de
antibitico por ms de 7 dias. DIAGNOSTICO: Cuadro clnico: Tos con o sin esputo, escalofros, fatiga, disnea y dolor torcico pleurtico, hemoptisis,
mialgias, en caso de legionella; puede existir sintomatologa gastrointestinal, en pacientes ancianos puede haber cadas y confusin. Exploracin fsica:
taquipnea, matidez a la percusin estertores y/o crepitancias, frmitos y egofona, pectoriloquia, estertores crepitantes, roce pleural. Rx de trax;
neumona tpica (causada usualmente por Streptococcus pneumoniae, mas en nios y
TABLA 3 CRITERIOS DE ADMISION A UCI; CRITERIOS DE NEUMONIA GRAVE
ancianos): consolidacin lobar y en neumona atpica (influenzae, mycoplasma,
Criterios menores
legionella, no detectables en tincin de gram (-), de ah su nombre, ms en jvenes)
FR >30rpm o necesidad de apoyo ventilatorio
infiltrados difusos: de no observar nada repetir Rx a las 24-48 hrs. Los estudios
Indice de PaO2/FiO2 <250
Infiltrados multilobulares o bilaterales
especiales dirigidos a un agente como hemocultivo, cultivos de esputo, tincin gram,
TAS <90mmHg
Ag urinario para legionellla y Streptococcus solo cuando: neumona extensa que
TAD <60mmHg
ingresa a UCI, falta de respuesta al tx., infiltrados cavitarios, leucopenia, alcoholismo
Criterios mayores
severo. Clasificacion radiolgica:1. Neumonia de espacio areo; viral, mycoplasma,
Ventilacin mecnica invasiva
Choque sptico >4hrs
Klebsiellla. 2. Neumonia lobar; K. pneumoniae, S. pneumoniae. 3. Neumonia
Aumento de opacidadpulmonar >50% en 48 hrs
intersticial; Micoplasma pneumoniae, virales, P. carinii. 4. Infiltrados nodulares;
Insuficiencia renal aguda
granulomas tuberculosos, granulomas micoticos. ESCALAS DE GRAVEDAD: tiles para
Diseminacion extrapulmonar
no sobreestimar la intensidad de los cuadros neumnicos y menor tasa de
Se requieren 2/3 menores y 1 / 2 mayores
hospitalizacin. Ver tabla 1, 2, 3. Criterios de hospitalizacin: NAC clasificacin grupo
CRITERIOS DE INGRESO A UCI
IV y V, NAC con criterios de gravedad, NAC con criterios de contraindicacion de
Frecuencia respiratoria >30/min
Presion arterial diastolica <60mmHg
tratamientop intradomiciliario. TRATAMIENTO: En la mayora de los casos el tx. es
Nitrogeno de urea >19mg/dl
emprico (A. pciente externo previamente sano, sin uso de antibiticos en los 3
Alteraciones en el estado de la conciencia
meses previos, usar un macrolido. B. paciente externo con comorbilidades, dar
Se requieren 2/4 criterios
levofloxacino o un betalactamico mas un macrolido. C.paciente hospitalizado, pero no
en UCI, levofloxacino o betalavctamico mas macrolido. D. paciente en UCI, cefotaxima o ceftriaxona mas azitromicina o levofloxacino. E. paciente con

CURSO ENARM CMN SIGLO XXI TEL: 36246001

Pharmed Solutions Institute

PGINA 27

MANUAL DE TRABAJO DEL CURSO ENARM CMN SIGLO XXI


sospecha de pseudomona, piperacilina/tazobactam, cefepime, imipenem o betalactamico mas aminoglucocido mas azitromicina. F. si se sopecha S.
aureus meticilino resistente agragar vancomicina o linezolid), pero debe ser activo frente a los patgenos ms frecuentemente implicados (S.
pneumoniae). La duracin del tratamiento minimo 5 das, despus de que el paciente permanezca por ms de 48-72 hrs asintomticas y no deben tener
mas de un signo de inestabilidad clnica asociado a NAC, antes de descontinuar el tratamiento. 7-10 dias en caso de Streptococcus; 10-14 para
Mycoplasma. Grupo I; pacientes sin historia de enfermedad cardiopulmonar y sin factores de riesgo; 1ra eleccin macrolidos, 2da eleccin doxicilina.
Grupo II; pacientes con enfermedad cardiopulmonar y con factores de riesgo; 1ra eleccin fluroquinolonas (moxifloxacino, levofloxacino) 2da. Eleccin Blactamico (cefotaxima, ceftriaxona, ampi/sulbactam). Grupo III; pacientes no admitidos a UCI que tienen lo siguiente: A: con enf.cardiopulmonar y otros
factores modificables y B: sin enf.cardiopulmonar y sin otros factores modificables; 1ra. Eleccin fluroquinolona, 2da., eleccin b-lactamico+ macrolido.
Grado IV: pacientes que ameritan ingreso a UCI. A: sin riesgo para pseudomona 1ra eleccin cefotaxima, ceftriaxona+ azitromicna o fluoroquinolona. B:
con riesgo para pseudomona; 1ra. Eleccin b-lactamico. Antineumococo y antipseudomona (piperacilina/tazobactam, cefepime, imipenem)+
ciprofloxacino o levofloxacino. 2da eleccin piperacilina/tazobactam, cefepime + aminoglucosidos y azitromicina. Antibiotico recomendado para
neumona atpica (legionella, mycoplasma) es claritromicna. Criterios de contraindicacin al tratamiento domiciliario. Ausencia de soporte social y
familiar. Existencia de comorbilidades asociadas a empeoramiento del pronostico, enf., neuromuscular, drogadiccin, hipoxemia, siatuacion psicolgica
inadecuada, intolerancia digestiva, hallazgos radiolgicos. PRONOSTICO: Criterios de gravedad; edad >75 aos, comorbilidad, TAD <60mmHg, TAS
<90mmHg, pulso >125/min, temp <35 o mas de 40, alteracin del estado mental, diseminacin extrapulmonar, leucocitos <4,000 o >30,000/mm3,
creatinina >1.2mg/dl, compromiso multilobar, cavitacin derrame pleural, hto >30%, hb <9mg/dl. COMPLICACIONES: Cavitaciones, abscesos, destruccin
antomica, destruccin bacteriana, derrames paraneumonicos complicados. PREVENCION: La primera medida es abandonar el habito tabquico. Vacuna
de neumococo se recomienda en >65 aos, o en sujetos de 2-4 aos con factores de riesgo; revacunacin cada 5 aos. Vacuna inactiva contra influenza;
>65 aos, 6-49 aos con factores de riesgo.; revacunacin anual. STREPTOCOCCUS PNEUMONIAE: Afecta lobulos inferiores 90%, generalmentre solo
afecta un lbulo, presenta broncograma areo, causa derrame pleural en un 10%. Klebsiella; causa NIH, broncoaspiracion, abscesos pulmonares,
derrame pleural, empiema, afecta lbulo superior derecho. S. aureus; infiltrados nodulares centrolobulares, distribucin segmentaria, causa NIH,
derrame pleural y abscesos pulmonares. P jiroveci; en inmunodeprimidos, infiltrados reticulares, bilateral simtrico, opacificacion progresiva y
homognea del parnquima pulmonar vidrio despulido, causa frecuente de neumatoceles y neumotrax espontaneo. Pseudomona; principal
neumona nosocomial, patrn nodular bilateral, neumona de focos multiples, principal causa de septicemia, absceso multiples y/o cavitaciones.
CASO CLINICO
Hombre de 68 aos de edad, con historia de consumo de licor cada fin
de semana hasta la embriaguez, cuatro das de tos con expectoracin
purulenta, disnea de moderados esfuerzos, fiebre de 38,5C, dolor tipo
pleurtico en el hemitrax derecho y automedicacin con una dosis de
dexametasona de 4 mg intramuscular dos das antes de la admisin.
Ingres febril, taquicrdico, taquipneico, hipo-xmico y en la
radiografa de trax presentaba signos de derrame pleural y
consolidacin en la base derecha. La evaluacin de laboratorio
muestra leucocitosis de 14.500 por mm3, neutrofilia de 96%,
hiperglucemia de 638 mg/dl, hiperazoemia (nitrgeno ureico en sangre
de 27,45 mg/dl). El paciente fue hospitalizado con diagnstico de
neumona grave extrahospitalaria IIIA, segn las guas nacionales de
manejo de esta enfermedad.
PREGUNTA
Cul es el agente etiolgico ms probable en este paciente?
RESPUESTA
a.- Mycoplasma pneumoniae
b.- Legionella pneumoniae
c.- Pseudomona
d.- Strepococcus pneumoniae
PREGUNTA
Qu criterios de neumona grave tiene este paciente, para ser tratado
intrahospitalariamente?
RESPUESTA
a.- leucocitosis, derrame pleural, tx con dexametasona
b.- Hiperazoemia, hiperglucemia, taquipnea
c.- Hiperazoemia, leucocitosis, consolidacin en base derecha
d.- Hiperglucemia, leucocitosis, tx. con dexametasona
CASO CLINICO
Hombre de 54 aos de edad, con historia de consumo de licor cada fin
de semana hasta la embriaguez, cuatro das de tos con expectoracin
purulenta, disnea de moderados esfuerzos, fiebre de 38,5C, dolor tipo
pleurtico en el hemitrax derecho y automedicacin con una dosis de
dexametasona de 4 mg intramuscular dos das antes de la admisin.
Ingres febril, taquicrdico, taquipneico, hipoxmico y en la radiografa
de trax presentaba signos de derrame pleural y consolidacin en la
base derecha. Leucocitosis de 14.500 por mm3, neutrofilia de 96%,
hiperglucemia de 638 mg/dl, hiperazoemia (nitrgeno ureico en sangre
de 27,45 mg/dl). El paciente progres rpidamente a falla respiratoria

CURSO ENARM CMN SIGLO XXI TEL: 36246001

hipoxmica, con choque sptico y, posteriormente, disfuncin


orgnica mltiple.
PREGUNTA
Considerando el cuadro clnico. a que grupo pertenece para
considerar tratamiento antibitico?
RESPUESTA
a.- Grupo I
b.- Grupo II.
c.- Grupo III
d.- Grupo IV.
PREGUNTA
Considerando el Pneumonia Severity Index que puntaje presenta el
caso?
RESPUESTA
a.- <70
b.- 71-90
c.- 91-130
d.- >130
PREGUNTA
Considerando la escala de riesgo y mortalidad de pneumonia severity
index. Que riesgo de motralidad presenta el caso?
RESPUESTA
a.- 0.6
b.- 2.8
c.- 8.2
d.- 29.2
CASO CLINICO
Varn de 34 aos, trabajador en la hostelera, fumador y bebedor
importante (con dependencia alcohlica). Fue trado a urgencias por
un traumatismo craneoenceflico con hematoma subdural. El paciente
ingres en la unidad de cuidados intensivos (UCI) intubado y
conectado a ventilacin mecnica, y recibi tratamiento antibitico
emprico con amoxicilina-cido clavulnico. La cifra de leucocitos en
sangre era de 12,05 103/l. En el primer da del ingreso se realiz un
broncoaspirado (BAS). En el cultivo creci una flora mixta respiratoria.
A los 8 das del ingreso present fiebre y secreciones purulentas
espesas, y en la radiografa de trax se objetiv una neumona en el
lbulo inferior izquierdo complicada con atelectasias bibasales. La

Pharmed Solutions Institute

PGINA 28

MANUAL DE TRABAJO DEL CURSO ENARM CMN SIGLO XXI


concentracin de leucocitos en sangre era en ese momento de 4,10
103/l.
PREGUNTA
Cual es el agente etiolgico mas probable para este caso?
RESPUESTA
a.- Pseudomona aeruginosa.
b.- Escherichia coli.
c.- klebsiella pneumoniae.
d.- Acinetobacter.

difuso sin irradiacin, fiebre hasta 38C, calofros y mialgias. Una


semana previa al inicio de sus sntomas cae al mar cerca de zona de
desage de alcantarillado, ingiriendo importante cantidad de agua.
Evaluado se le diagnostica amigdalitis aguda. Posteriormente presenta
tos, disnea progresiva, escalosfrios, mialgias. Paciente consiente,
desorientado, algo agitado, hidratado, perfundido, Ictericia de piel y
mucosas, pequeas adenopatas cervicales e inguinales, PA: 102/79,
FC: 103, FR 24, T: 38,8. A la auscultacin estertores crepitantes, roce
pleural, hepatoesplenomegalia moderada, hgado graso, Rx de trax:
9/01/07, Imagen sospechosa de condensacin paracardiaca derecha.

PREGUNTA
Cual es el tratamiento de primera intensin en este caso?
RESPUESTA
a.- Ceftriaxona, levofloxacino.
b.- Cefepime, ceftazidima.
c.- Imipenem, meropenem.
d.- Piperacilina/tazobactam

PREGUNTA
Cul es el diagnstico ms probable de este paciente?
RESPUESTA
a.- Sndrome febril
b.- Neumona
c.- Faringoamigdalitis crnica
d.- Bronquitis aguda

CASO CLINICO
Femenino de 84 aos. Paciente con demencia vascular de varios aos
de evolucin, con prdida total de la independencia funcional,
postrada en cama desde hace aproximadamente 3 aos, al cuidado
permanente de 2 de sus hijas, portadora adems de miocardiopata
dilatada, cumpliendo tratamiento en la actualidad con Digoxina,
Furosemida, Enalapril, AAS y baso activos cerebrales. Desde hace 10
das presenta dificultad para deglutir los alimentos, presentando
accesos de tos hemetizante durante las ingestas, y desde hace 48 hs.,
fiebre, dificultad respiratoria progresiva, somnolencia permanente y
expectoracin muco purulenta, nausea y vomito. T/A: 90/50mmHg, FC
126x, Temp 38.6.

PREGUNTA
De acuerdo al CURB 65. Qu probabilidad de morir tienen este
paciente?
RESPUESTA
a.- <2%
b.- 9%
c.- 20%
d.- 50%

PREGUNTA
Cul sera el agente etiolgico ms probable para este caso?
RESPUESTA
a.- Estreptococo pneumoniae
b.- Klebsiella
c.- Haemophilus influenzae
d.- Escherichia coli
PREGUNTA
Cul sera la conducta diagnostica ms adecuada a seguir en este
caso?
RESPUESTA
a.- Broncoscopia
b.- Tincion de gramm
c.- Aspirado transtraqueal
d.- Toracocentesis
PREGUNTA
De las siguientes cual es la razn menos probable por la que no se
debe administrar tratamiento domiciliario?
RESPUESTA
a.- Intolerancia digestiva
b.- Enfermedades subyacentes
c.- Signos vitales alterados
d.- Soporte familiar inadecuado
CASO CLINICO
Hombre de 66 aos sin antecedentes mrbidos relevantes. Ingresa,
con historia de 5 das de evolucin caracterizada por dolor abdominal

CURSO ENARM CMN SIGLO XXI TEL: 36246001

PREGUNTA
Cul sera el tratamiento emprico ms adecuado para este paciente?
RESPUESTA
a.- Macrolido mas betalactamico
b.- Fluoroquinolona respiratoria
c.- Betalactamico mas azitromicina
d.- Betalactamico ms piperacilina/tazobactam
CASO CLINICO LISTERIA
Mujer de 70 aos con cirrosis heptica por el virus de la hepatitis C, en
estadio B de Child, con trombosis portal, episodios de hemorragia
digestiva alta, descompensacin hidrpica y un ltimo ingreso por
ascitis e hidrotrax secundario, que se resolvi con tratamiento
diurtico. Acudi a urgencias por un cuadro de ascitis, edemas y
aumento de la disnea de 15 das de evolucin. En la auscultacin
pulmonar tena disminucin del murmullo vesicular en los dos tercios
inferiores del hemitrax izquierdo. En el hemograma destacaba
nicamente la trombopenia (76.000 clulas/l) y en la bioqumica, la
elevacin de la creatinina (1,9 mg/dl), bilirrubina (3,1 U/l),
gammaglutamiltranspeptidasa (71 U/l) y fosfatasa alcalina (268 U/l),
as como el descenso de la albmina (2,3 U/l). En la radiografa de
trax se objetivaba un derrame pleural que ocupaba los dos tercios
inferiores del hemitrax izquierdo y sin focos de compensacion.
PREGUNTA
Considerando el cuadro clnico. Cual de las siguientes complicaciones
es la ms probable que presenta este caso?
RESPUESTA
a.- Peritonitis.
b.- Neumonia.
c.- Sindrome hepato-renal.
d.- Derrame pericardico.

Pharmed Solutions Institute

PGINA 29

MANUAL DE TRABAJO DEL CURSO ENARM CMN SIGLO XXI


NEUMOCONIOSIS
CIENCIAS BASICAS: Definicin: Enfermedad producida por acumulacin de polvos minerales secundaria a la inhalacin crnica de los mismos. Los
materiales mas implicados son: asbesto, slice, carbn, berilio, hierro, el dao tisular que provocan depende de; tamao, tiempo e intensidad de
exposicin, estado inmunolgico, relacin inhalacin/ aclaramiento ciliar pulmonar. ASBESTOSIS: Hay 3 tipos de asbesto: amosita, asbesto azul
(cocidolite) y asbesto blanco (crisolito, el 90%del consumo de asbesto es bajo esta forma, es mas soluble y tiende a fragmentarse). Las formas en las que
se inhalan las fibras de asbesto es en anfibole y serpentinas. PATOGENIA: Exposicin directa, cuando se trabaja con materiales o en lugares donde hay
asbestos. Exposicin indirecta, cuando se vive o trabajo cercano a un lugar de concentracin de asbesto. Lo podemos encontrar en materiales para pulir
piedra preciosa, afiladores de piedra, resistencias de algunos electrodomsticos y algunos plsticos. La enfermedad se puede presentar como: placas
pleurales benignas 40%, enf., maligna 20% (mesotelioma maligno), asbestosis como tl 40%. Una vez inhalada las partculas, llegan a los espacios
alveolares y son fagocitados por lo macrfagos, si son menor a 3 m, si son mayores son fagocitadas incompletamente ( cuerpos de asbesto)e, ya
fagocitadas son trasportadas a los ganglios y despus a las pleuras, donde se depositan para formar las placas pleurales, los cuerpos de asbesto se
acumulan de manera progresiva en espacios areos e intersticio, las clulas afectadas liberan citocinas, fibronectina y colgena, causando as migracin
de otras clulas de defensa y favoreciendo proliferacin de fibroblastos, provocando zonas de regeneracin parenquimatosa, provocando la asbestosis,
es decir la fibrosis pulmonar afecta mas zonas inferiores y regiones subpleurales. DIAGNSTICO: Relacin clara entre exposicin de asbesto y fibrosis
pulmonar. El consumo de tabaco aumenta el riesgo de asbestosis. Cuadro clnico: lo primero es disnea, la cual se va haciendo progresiva, despus tos
seca pertinaz, pueden auscultarse estertores crepitantes basales, a veces hipocratismo. Rx.: opacidades irregulares que al principio de la enfermedad son
de localizacin perifrica y basal, hiperclaridad alrededor de silueta cardiaca. TAC: se puede ver fibrosis, como vidrio despulido, hasta lesiones fibroticas
con engrosamiento de los septos interlobares, engrosamiento de pleura viscerales. El patrn funcional observado en pacientes con asbestosis es
restrictivo. Se puede realizar lavado bronquioalveolar. SILICOSIS: Provocada por inhalacin de cristales de slice, en la mayora en sus formas de cuarzo,
las actividades de riesgo son la construccin, la fundicin, demolicin o reparacin de estructuras de concreto, taladrar piedras. PATOGENIA: Hay silicosis
aguda, donde se observa proteinosis y la crnica la mas frecuente, donde se observan ndulos con centro fibrotico, rodeado de parnquima con cmulos
de slice, que ha sido fagocitado por macrfagos. En la forma complicada llamada fibrosis masiva progresiva, se observan ndulos confluentes, que se
localizan principalmente en lbulos superiores, que se relacionan con una pobre calidad de vida. La silicosis se relaciona con una mayor susceptibilidad a
Tb, artritis reumatoide, esclerodermia, lupus y enf., renal progresiva. DIAGNSTICO: Cuadro clnico: sntoma principal disnea, la cual es progresiva, raros
otros sntomas. Rx: pequeas opacidades redondeadas, con distribucin casi siempre bilateral, de localizacin en lbulos superiores, puede haber
crecimiento de adenopatas hiliares, las cuales pueden calcificarse en una forma caracterstica conocida como "cascara de huevo". Funcionalmente la
silicosis se comporta con limitacin al flujo areo y con reduccin de la capacidad de difusin. ANTRACOSIS: Provocada por inhalacin crnica de carbn
masiva, la forma mas comn es el antracite. Existen 2 formas principales: la mcula o ndulo antracotico y fibrosis pulmonar masiva. PATOGENIA: En la
antracosis hay mayor susceptibilidad a presentar artritis reumatoide, y cuando se presentan ambas condiciones, se conoce como Sx., de Caplan, en el
cual debe haber un ndulo reumatoideo, que en algunas condiciones puede cavitarse debido a la necrosis central o bien calcificarse. DIAGNSTICO: La
macula o mancha de carbn suele ser una lesin de 5cm que puede observarse principalmente en regiones apicales, en ocasiones rodeadas de
parnquima que muestra enfisema centroacinar. Existen tantas neumoconiosis como diversidad de polvos minerales y son una causa importante de
morbimortalidad a nivel mundial.
CASO CLINICO
70 aos, minero de minas carbn jubilado, fumador de 30
cigarrillos/da hasta hace 10 aos, bebedor de l de vino al da y 2
cervezas. HTA en tratamiento con diurticos y dieta hiposalina.
Hipercolesterolemia en tratamiento con dieta. Desde hace 25 aos
presenta tos y expectoracin crnica de predominio matutino. Desde
hace 15 aos se asocia disnea de esfuerzo progresiva con aumento de
la disnea en relacin con las infecciones respiratorias. Presenta con
frecuencia expectoracin de color negro. Paciente que acude al
servicio de urgencias por un cuadro de aumento de su tos habitual,
aumento de su expectoracin habitual, siendo actualmente
mucopurulenta y aumento de la disnea que se hizo de reposo. Desde
hace una semana nota que se le hinchan los tobillos. Paciente
consciente, orientado. Ciantico. Taquipneico (30 respiraciones por
minuto). Hepatomegalia blanda y no dolorosa de 2 cm. Ingurgitacin
yugular +++. Edemas maleolares ++++. Tonos cardiacos rtmicos a 130
por minuto. Roncus y sibilantes diseminados por ambos campos
pulmonares.TA: 160/100. Hb 16, Hto 49, leucocitos 13500 con
aumento de neutrfilos en la formula leucocitaria. P02 52 PC02 40, PH
7.35. RX de torax: Signos de atrapamientoretroesternal y
retrocardiaco, engrosamientos peribronquiales. 3 imgenes de
aumento de densidad nodulares de 3-7 cmde dimetro en Lbulo
superior derecho y 2 en Lbulo superior izquierdo.
PREGUNTA
Se realizaron pruebas funcionales respiratorias con los siguientes
resultados FVC 60%, FEV1 30%, cual es la FEV1-FVC?
RESPUESTA
a.- 40
b.- 50
c.- 60

CURSO ENARM CMN SIGLO XXI TEL: 36246001

d.- 30
CASO CLINICO
Una mujer afroestadounidense de 29 aos de edad se presenta a
disnea de esfuerzo, tos crnica seca, dolor torcico leve, malestar
general y fatiga de 6 meses de evolucin. EF: T38C, FC 80lpm, FR
20rpm. El nico hallazgo en el examen de torax son sibilancias tenues.
En el examen ocular se advierte eritema bilateral en e borde
esclerocorneal. Laboratorio: Hb 13mg/dl, Calcio serico 12mg/dl, e
incremento en la IECA srica. La sedimentacin eritrocitaria es alta
80mm/h. Los resultado sde funcin pulmonar son: FEV1/CVF: 80%,
CPT: 3.7l (70% de lo predicho). Se realiza una biopsia pulmonar y el
departamento de anatoma patologiac reporta granulomas sin
caseificacin.
PREGUNTA
Cul es el diagnostico mas probable?
RESPUESTA
a.- Asbestosis
b.- Antracosis
c.- Sarcoidosis
d.- Silicosis
PREGUNTA
Cul es la causa mas probable de hipercalcemia?
RESPUESTA
a.- Calcitriol extrarrenal producido por los macrfagos activados en los
granulomas
b.- Acumulacion por la nrecrosis grasa
c.-Por la alteracin a nivel muscular
d.- A causa de la enfermedad pulmonar intesrticial

Pharmed Solutions Institute

PGINA 30

MANUAL DE TRABAJO DEL CURSO ENARM CMN SIGLO XXI


HIPERTENSION PULMONAR PRIMARIA (HAP)
CIENCIAS BASICAS: Grupo de enfermedades caracterizadas por un aumento en la resistencia vascular pulmonar que conduce a fallo ventricular
derecho, caracterizada por una proliferacin fibromuscular y remodelacin vascular endotelial, que da lugar a estenosis de la luz vascular. LEVE
<30mmHg, MODERADA 30-45mmHg, SEVERA >45mmHg. Hipertensin pulmonar >25mmhg en reposo y >30mmhg en el ejercicio. A partir de 50 aos
> 1mmhg por cada dcada de vida. SALUD PUBLICA: Incidencia anual es de 15 casos por milln de habitantes, con edad media de diagnstico de 36
aos (4ta-5ta dcadas de la vida). La HAP primaria (idioptica) 30-40%; predomina en la mujer. Los casos de HAP familiar representan el 10%. La HAP
asociada a fenmeno de Eisenmenger (cortocircuito der-izq), representa 30-35%. La hipertensin es la causa ms comn de cor pulmonale. PATOGENIA:
La obstruccin y obliteracin vascular, la reduccin vascular pulmonar, la acidosis e hipercapnia, la hipoxemia que lleva a policitemia y esta a su ves a
aumento de la viscosidad, todos estos factores juntos nos llebvan a un aumento en la resistencia vascular, que genera la hipertensin pulmonar, y esto a
dilatacin e hipertrofia ventriculat derecha lo que genera insuficiencia cardiaca derecha. La disfuncin endotelial juega un rol pivote y trascendental,
sobre todo por el imbalance entre las sustancias vasodilatadoras (xido ntrico, prostaciclina, PG- 1-2, pptido intestinal vasoactivo) y vasoconstrictoras
(Tx A-2, endotelina-1, serotonina, factor de crecimiento derivado de cel. endoteliales). En HAP familiar, se encuentran alteraciones en los receptores que
codifican para los factores transformadores de crecimiento tipo beta (BMPR-II), que regulan la angiognesis y apoptosis celular. CLASIFICACION DE LA
OMS: GRUPO I DANA POINT: Hipertensin arterial pulmonar; a) Hipertensin pulmonar primaria (espordica y familiar), b) HP asociada a Colagenosis,
Cortocircuitos izq.-der., H Portal, VIH, Producida por frmacos, crack, coicaina. GRUPO II DANA POINT: Hipertensin pulmonar venosa; Cardiopatas y
Valvulopatas izq. Compresin de venas pulmonares. Enfermedad Venoclusiva pulmonar. Caractersticas comunes: Aumento de la P Venosa. GRUPO III
DANA POINT: H. P. Asociada a neumopatias o hipoxemia; EPOC, Enf Intersticiales, Trastornos del sueo, Hipoventilacin alveolar. Grandes alturas.
Caractersticas; Mecanismo productor la Hipoxia. GRUPO IV DANA POINT: HTP por Trombosis o Embolia; Tromboembolismo Central o perifrico. GRUPO
V DANA POINT: HTP por Enf. vasculares y otras; Sarcoidosis. Esquistosomiasis, Hemangiomatosis capilar, Tratamiento: El de la enfermedad productora o
Trasplante pulmonar. Tambien se puede clasificar en: 1. Primaria, idioptica: HTP primaria (precapilar): se caracteriza por tener una presin de capilar
pulmonar venoso normal, es de causa desconocida y el diagnstico se hace por exclusin. Una disminucin de la actividad de canales de K del msculo
liso, con aumento del calcio intracelular y aumento del tono vascular. 2. Secundaria con causa explicable; HTP secundaria (postcapilar): se caracteriza
por tener una presin de capilar venoso aumentada y una causa que la explique: EPOC, obesidad. 3. Hereditaria. DIAGNSTICO: Sintomas: Falla
cardiaca derecha, el sntoma ms comn es disnea (60%), la presencia de ortopnea o disnea paroxstica nocturna, aumento del trabajo respiratorio en el
decbito, tos o sncope en pacientes con cor pulmonale con HTAP severa, tambin puede presentarse dolor abdominal y ascitis, as como edema de las
extremidades inferiores, datos de falla izquierda. Signos: Taquipnea, presin venosa yugular elevada, hepatomegalia y edema en las extremidades
inferiores, choque de la punta desviado, edema de pulmn agudo, cianosis, acropaquia. Puede haber ingurgitacin yugular, por contractilidad
aumentada de la de la AD, por falla ventricular derecha crnica. Auscultacin: segundo ruido, en su componente pulmonar desdoblado o acentuado,
datos de regurgitacin tricspidea, as como la presencia de galope sobre el precordio correspondiente al VD. El diagnostico de certezas solo puede
establecerse, segn la definicin mediante el cateterismo derecho, cateterizacin de Swan-Ganz, Gold standard y permite evaluar afectacin del
ventrculo izquierdo. ECG, tele de trax (Arteria descendente anterior derecha >17mm, abombamiento de la pulmonar y rectificacin, calcificaciones
lineales sobre arteria pulmonar derecha, hiperclaridad perifrica, agrandamiento de cavidades derechas, arco medio prominente, signos de patologas
asociadas (fibrosis pulmonar, enfisema, etc), ecocardiografa transtoracica (valorar afeccin valvular o miocrdica como pb. causa, as como
cortocircuitos izq-der). Pruebas de funcin respiratoria para determinar neumopatas. El gamagrama pulmonar ventilatorio-perfusorio es estudio clave
para detectar HAP tromboembolica crnica, confirmacin dx. con angiografa pulmonar. Tambien se puede hacer biopsia pulmonar. Biomarcadores: ANP
(pepetido natriuretico auricular), se origina en la auricula y se libera cuando hay distensin auricular. BNP (pptido natriureico cerebral), se orirgina en el
miocardio ventricular, y el estimulo para su liberacin es sobrecarga ventricular, proBNP estimula la natriuresis y la diuresis, acta como vasodilatador y
antagonista del sistema renina-angiotensina aldosterona, BNP >500pg/ml y pro-BNP >1.800pg/ml es igual IC muy probable (95%). CNP (pptido
natriuretico del endotelio), se origina en el endotelio y el estimulo para su liberacin es el estress endotelial. TRATAMIENTO: Si est limitada la actividad
fsica usar diurticos (mejora la funcin bi-ventricular, mejora sobrecarga, en exceso, disminuye el gasto cardiaco, disminuye perfusin renal : urea
bun creatinina), para edema perifrico, O2 suplementario si esta reducida la PO2 (a largo plazo mejora la supervivencia y disminuye morbilidad,
disminuye la vasoconstriccin pulmonar y RVP aumentando el gasto cardiaco) y anticoagulacin crnica con warfarina (objetivo INR= 2.0-3.0). Como los
vasoconstrictores juegan un papel importante, el tx. con vasodilatadores se basa en tales procesos, pero no se puede predecir quien responder sin
resistencia a los vasodilatadores orales como a los antagonistas de canales de calcio, ya que estos mejoran la sobrevida. Mediante una prueba se puede
predecir que pacientes tendrn respuesta sostenida a dichos agentes, se define una prueba positiva, cuando al administrar vasodilatadores endovenosos
de accin corta como adenosina o epoprostenol, existe una disminucin de 10mmHg o mayor en la presin arterial pulmonar media o como un descenso
en ms de 25% de las resistencias vasculares pulmonares sin presentar variacin en la presin arterial sistmica, solo responden 7-10% a dicho reto,
habr que monitorizar hipotensin o empeoramiento de la insf. Cardiaca der. Digoxina solo indicada en FA + ICC izquierda. Otros frmacos son los
prostanoides, son sustancias tanto vasodilatadoras como antiproliferativas, usados por varias vas, epoprostenol endovenoso, iloprost inhalado,
treprostinil subcutneo, el mecanismo de accin radica en estimular la produccin de AMP cclico como segundo mensajero induciendo vasodilatacin.
En pacientes con hipertensin arterial pulmonar existe un dficit relativo de xido ntrico, por lo que los inhibidores de fosfodiesterasas tipo 5, al
prolongar la vida media de los segundos mensajeros GMPc, prolongan el efecto vasodilatador y antiproliferativo del xido ntrico, como es el caso del
sildenafil, el cual es de 1ra lnea en el tx. por va oral de la hipertensin arterial pulmonar (Inhibidor de la fosfodiesterasa 5, mejora la clase funcional y
caminata de 6 minutos, vasodilatador,(relajante de musculo liso), mejora la o2%, mejora la sobrevida, 50 mg por va oral 2 veces por da o 25 mg 3
veces por da). Si persiste la Insf. Cardiaca derecha, debe considerarse el trasplante pulmonar bilateral. PRONSTICO: Deletreo y muy pobre. Sobrevida
de 2.5 aos a partir del dx. sin tratamiento. Supervivencia a 5 aos de 75-85% en pacientes con tx. Adecuado y adyuvante con combinacin o sinergismo
de frmacos con diferentes mecanismos de accin. Las variables ms importantes para establecer pronstico son presin media de la arteria pulmonar,
presin de AD y GC. Indicadores de mal pronostico: Edad <14 y >65 aos. Sncope, Angor, hemoptisis. Clase funcional I - II, sobrevida media 6 aos. Clase
funcional III, sobrevida media 2,5 aos. Clase funcional IV, sobrevida media 0,5 aos. Fenomeno de Reynold. COMPLICACIONES: congestin hepatica:
Higado en nuez moscada, cirrosis cardiaca, congestin esplenica, sindrome cardiorrenal, edema pulmonar, muerte subita, cardiopatia isquemica: SICA.
CASO CLINICO
Paciente varn de 58 aos de edad, ex fumador (con una dosis
acumulada de 20 paquetes-ao) y ex enolismo moderado, y con
antecedentes personales de hipertensin arterial sistmica,
linfangiectasias duodenales, epistaxis de repeticin y cirrosis heptica
enlica ChildA (MELD13) con un nico episodio de hemorragia
digestiva alta por sangrado de varices esofgicas. Tiene 2 hijas, una de

CURSO ENARM CMN SIGLO XXI TEL: 36246001

las cuales refera episodios de epistaxis de repeticin. Ingres por


clnica de infeccin respiratoria e insuficiencia respiratoria aguda con
una saturacin de O2 de 77% (FIO20,21). En la radiografa de trax se
observ un infiltrado en la base derecha que se interpret como una
probable neumona adquirida en la comunidad y se inici tratamiento
antibitico con fluoroquinolonas (levofloxacino).

Pharmed Solutions Institute

PGINA 31

MANUAL DE TRABAJO DEL CURSO ENARM CMN SIGLO XXI


PREGUNTA
Ante la persistencia de hipoxemia severa refractaria cual es la
conducta mas adecuada?
RESPUESTA
a.- Cambiar esquema antibitico.
b.- Realizar TAC.
c.- Realizar ecocardiograma doppler
d.- Realizar gamagrafia.

con disnea a moderados esfuerzos. Cuando consult en urgencias,


presentaba disnea de reposo, debilidad, astenia y refera haber tenido
un sncope. A la exploracin presentaba PA de 120/80mmHg, FC de
110 lat/min y SatO2 del 95% sin O2 suplementario. Se apreciaba
aumento del permetro del miembro superior derecho, doloroso a la
palpacin y sin signos de empastamiento. La analtica presentaba
perfiles bioqumico, heptico, renal, hemograma y coagulacin dentro
de la normalidad, excepto los dmeros D (2.500 ng/ml).

CASO CLINICO
Mujer de 40 aos, con xeroftalmia de 5 aos de evolucin a la que se
haban aadido en los ltimos 6 meses xerostoma y artralgias, sin
otras manifestaciones extraglandulares. Las pruebas complementarias
nos permitieron hacer el diagnstico de SS: ANA positivo, anti-SSa
positivo, anti-SSb positivo, test de Shirmer anormal, datos de
inflamacin crnica en biopsia de glndula salival menor y
gammagrafa de glndulas salivales compatible. Se realiz
ecocardiograma de rutina donde apareci por primera vez datos de
HTP ligera (32 mm Hg) que se confirm con cateterismo derecho. La
paciente present disnea de esfuerzo. En ecocardiografa se estim
una HTP de 46 mm Hg. Mediante TC torcica se descart la existencia
de fibrosis pulmonar y eventos tromboemblicos. Actualmente
persisten artralgias que ceden con analgsicos habituales y su disnea
est estable.

PREGUNTA
Cual es la conducta a seguir mas adecuada, para confirmar el
diagnostico para el caso actual?
RESPUESTA
a.- Realizar rx de torax.
b.- Realizar TAC de torax.
c.- Realizar Angio-TAC de torax.
d.- Realizar IRM de torax.

PREGUNTA
Cual de las siguientes afirmaciones es la incorrecta, respecto a la HTP
del caso?
RESPUESTA
a.- En asociacin con enfermedades del tejido conectivo CREST.
b.- En asociacin con LES
c.- Enfermedad mixta del tejido conectivo.
d.- La asociacin con SS es comn.
CASO CLINICO
Un hombre de 33 aos fue admitido, con disnea reciente y progresiva,
edema y prdida de peso de aproximadamente 10 kg. Dos hermanas a
los 11 y 14 aos de edad. Al EF, pareca "dbil" (subnutrido) (peso 60
Kg; altura 1,82 m), presentaba hepatomegalia leve y edema de
miembros inferiores. Un sonido alto de cierre de la vlvula pulmonar
era odo en el precordio. El examen de los pulmones revel ronquidos
leves y estertores finos en ambos lados, que desaparecieron
completamente despus de la administracin de diurticos. Los datos
de la cateterizacin cardaca derecha fueron compatibles con
hipertensin arterial pulmonar (HAP). xido ntrico inhalado y
sildenafila fueron eficaces en la reduccin de la resistencia vascular
pulmonar. Una acentuada elevacin en la presin de
enmentoencuamiento pulmonar fue registrada durante la inhalacin
del xido ntrico.
PREGUNTA
Cual es la conducta adecuada para establecer un diagnostico
definitivo?
RESPUESTA
a.- Gamagrama pulmonar.
b.- Ecodoppler pulmonar.
c.- Biopsia pulmonar abierta
d.- Tomografia axial computada.
CASO CLINICO
Mujer de 20 aos, de profesin camarera, que consult por cuadro de
disnea, edema en el miembro superior derecho y sncope. Tena
antecedentes de ser fumadora activa de 15 cigarrillos/da y padecer de
rinoconjuntivitis y asma bronquial. La paciente comenz un mes previo

CURSO ENARM CMN SIGLO XXI TEL: 36246001

CASO CLINICO
Femenino, 36 aos, con antecedentes de asma leve intermitente, 3
partos previos por va vaginal sin complicaciones. No fumadora, no
antecedentes tromboemblicos, no ingesta de anorexgenos, no
elementos clnicos sugestivos de afeccin conectivovascular. Comienza
en el ao 2012 con disnea de esfuerzo lentamente progresiva clase I.
En mayo de 2014 cursando gravidez de 35 semanas agrava su disnea
que se hace clase funcional III, agregando mareos, hemoptisis leve y
edemas de miembros inferiores. En ese momento se diagnostica
hipertensin pulmonar en estudio ecocardiogrfico, se descarta
enfermedad tromboemblica y se coordina cesrea sin incidentes. Del
examen fsico se destacaban ritmo regular de 90 pm, R2 aumentado en
foco pulmonar, ingurgitacin yugular sin hepatalgia y edemas
pretibiales. El examen pleuropulmonar al igual que el resto del examen
fsico no mostr alteraciones. ECG: ritmo sinusal de 100 pm, eje
elctrico a derecha, bloqueo incompleto de rama derecha. Radiografa
de trax: aumento del dimetro del tronco de arteria pulmonar.
Ecocardiograma doppler transtorcico: cavidades izquierdas normales
con FEVI 66%, dilatacin severa e hipertrofia del VD, desplazamiento
del septum interventricular, presin sistlica pulmonar estimada en 80
mmHg.
PREGUNTA
Cul es la conducta teraputica ms adecuada a seguir?
RESPUESTA
a.- Digoxina
b.- Beta-bloqueadores
c.- Sildenafil
d.- Hidralacina
PREGUNTA
Cul es la arritmia ms probable que pueda presentar la paciente?
RESPUESTA
a.- Fibrilacin ventricular
b.- Taquicardia ventricular
c.- Fibrilacin auricular
d.- Taquicardia sinusal
PREGUNTA
Cul es la conducta diagnostica ms adecuada de hipertensin
pulmonar, que adems evalua severidad y determinar pronostico?
RESPUESTA
a.- Angiografa
b.- Cateterismo cardiaco
c.- Resonancia magntica
d.- Ecocardiograma

Pharmed Solutions Institute

PGINA 32

MANUAL DE TRABAJO DEL CURSO ENARM CMN SIGLO XXI


COR PULMONALE
CIENCIAS BASICAS: Se define como el crecimiento de ventriculo derecho por dilatacin o hipertrofia, secundario a alteraciones anatomicas o
funcionales primarias del pulmn, su mecnica ventilatoria o ciruculacin pulmonar y puede conducir a insuficiencia cardaca derecha, como resultado
de un desorden respiratorio que produzca hipertensin pulmonar. La hipertrofia del ventrculo derecho es el cambio principal en el cor pulmonale
crnico y dilatacin del ventrculo en los casos agudos, ambos consecuencia de un incremento de presin en el ventrculo derecho del corazn.
ETIOLOGA: Para que una alteracin del corazn derecho sea clasificado como cor pulmonale, el origen de la anomala debe encontrarse bien en el
sistema de circulacin pulmonar o bien en el parnquima pulmonar, producindose en cualquier caso una hipertensin arterial pulmonar. Segn la
agudeza y la severidad, se puede distinguir: Cor pulmonale agudo: por estmulo repentino y severo, que produce dilatacin y fallo del ventrculo derecho,
pero no hipertrofia debido a embolismo pulmonar masivo, maligno, carcinomatosis miliar, linfangitis carcinomatosa, empeoramiento del cor pulmonale
crnico. Cor pulmonale crnico: por aumento progresivo y lento de la HTAP, que conduce a la hipertrofia y dilatacin del ventrculo derecho; puede
generarse a causa de: EPOC, bronquitis crnica y enfisema, en la que la alteracin de la estructura alveolar y la consecuente insuficiencia respiratoria
representa el inicio del cor pulmonale. Prdida de tejido pulmonar por razones traumticas o quirrgicas. Pneumocistitis terminal. Dicha restriccin
respiratoria causa hipoxemia, hipercapnia con subsecuente vasoconstriccin pulmonar e hipertrofia del ventrculo derecho. Sarcoidosis por fibrosis del
parenquima pulmonar en una pequea porcin de los pacientes (5%). FISIOPATOLOGA: Anatmicamente, el ventrculo derecho tiene una pared
delgada, con una compliancia elevada, por lo que est mejor preparado para adaptarse a variaciones de volumen que a variaciones de presin. Por ello,
cuando se produce un aumento de la presin pulmonar y un aumento de la resistencia vascular pulmonar, el ventrculo derecho no puede desarrollar la
fuerza suficiente como para superar dicho aumento, por lo cual se dilata (en los casos agudos) o se hipertrofia (en los casos crnicos), pudiendo llegar a
fallar. La severidad de la hipertensin pulmonar y el fallo del ventrculo derecho estn influidos por mltiples factores que pueden producirse de manera
intermitente, como hipoxia secundaria a alteraciones en el intercambio gaseoso, hipercapnia y acidosisn (inhiben la inmovilizacin del Ca, lo que
disminuye la contractilidad cardiaca), adems de cambios en la sobrecarga de volumen del ventrculo derecho que se producen con el ejercicio, con el
aumento del ritmo cardaco, la policitemia o la retencin de sales debido a una disminucin del gasto cardaco. Mecanismos celulares; Dao de clulas
endoteliales. Tnica media empieza a proliferar. Proliferacin de fibroblastos en vasos sanguneos y ventrculo derecho. Aglutinacin de plaquetas
y formacin de microtrombos en la pared ventricular y arterial pulmonar. Proliferacin de clulas inflamatorias. Los factores quer participan en la
genesisi del sor pulmonale son: 1. Hipoventilacion en los alveolos secundaria a la capacidad de la perdida retrctil del pulmn y aumento del volumen
residual. 2. Obtsruccion bronquial por broncoespasmo. 3. El aumernto de la resistenciaperoferica por la reduccin anatmica del lecho vascular. 4. La
disminucin funcional del calibre de los vasos y al aumento de la viscosidad sangunea. DIAGNOSTICO: El sntoma ms comn es disnea, debido a que
hay un aumento del trabajo necesario para respirar, a consecuencia de los cambios en la elasticidad del pulmn (como ocurre en la fibrosis) o a
modificaciones en la mecnica respiratoria (como sucede en la EPOC), y ambos pueden verse agravados por un incremento de la hipoxemia. La
hipoxemia pulmonar puede generarse por una disminucin de la permeabilidad de los capilares, por desequilibrios ventilacin-perfusin o por la
presencia de un shunt cardaco o pulmonar. La presencia de ortopnea o disnea paroxstica nocturna son raramente sntomas aislados de fallo cardaco
derecho. Pero si se presentan, indican que hay un aumento del trabajo respiratorio en el decubito. Tambin puede presentarse tos o sncope en
pacientes con cor pulmonale con HTAP severa, debido a la incapacidad del corazn derecho de bombear sangre a travs de los pulmones hacia el
corazn izquierdo. Tambin puede presentarse dolor abdominal y ascitis, as como edema de las extremidades inferiores. Signos: taquipnea, presin
venosa yugular elevada, hepatomegalia y edema en las extremidades inferiores. Adems, el ventrculo derecho puede palparse a lo largo del borde
izquierdo del esternn o en el epigastrio. En el cor pulmonale, la aparicin de cianosis es un evento tardo, secundario a una reduccin del gasto cardaco
con vasoconstriccin sistmica y desequilibrio ventilacin-perfusin en el pulmn. Dx., de gabinete y laboratorio igual que para hipertensin pulmonar.
EDEMA DE PULMN AGUDO: La causa ms comn del edema pulmonar agudo es el aumento de la presin capilar (desequilibrio de las fuerzas de
Starling) en la microvasculatura pulmonar, causando fuga plasmtica hacia el espacio intersticial. Cualquier evento que curse con hipertensin venosa
pulmonar ocasionar una congestin venosa y un aumentado volumen de sobrecarga al ventrculo derecho. La incapacidad del ventrculo derecho de
expandir para adaptarse a ese incrementado volumen aumenta an ms el volumen venoso, y por ende retrgrada, la presin capilar. La fuga de plasma
por una mayor presin hidrosttica en el capilar pulmonar que en el espacio intersticial acumula lquido en el parnquima pulmonar. COMPLICACIONES:
El cor pulmonale retrasa el flujo de la sangre desde la circulacin venosa hacia la arterial. En la hiperemia, la sangre se acumula en el sistema venoso,
incluyendo la vena heptica. La congestin prolongada de sangre en la regin centro-lobulillar del hgado conlleva a hipoxia y cambios grasos de los
hepatocitos perifricos, produciendo agrandamiento voluminoso del hgado, llegando a tener hasta el doble del volumen de sangre. El aspecto ciantico
combinado con islotes grasos amarillentos le da la apariencia al hgado de nuez moscada. Con el tiempo se agrega una acumulacin fibrtica
(cicatrizacin) que tiende a reducir al hgado de tamao y a endurecerlo. En los pulmones, la hiperemia produce ingurgitacin capilar, edema en el
intersticio pulmonar y, como consecuencia, fibrosis entre los alvolos y la matriz capilar, traducindose en trastornos de la relacin perfusin y
ventilacin pulmonar. El pulmn adopta una consistencia dura y (macroscpicamente) de color "rojo morena". El cor pulmonale puede tambin llevar a
insuficiencia cardaca congestiva (ICC), empeorando la respiracin por razn del edema pulmonar, hinchazn de las piernas debido a edema en la
periferia y hepatomegalia congestiva y dolorosa. La ICC es un indicador negativo en el pronstico del cor pulmonale. TRATAMIENTO: La eliminacin de la
causa es la intervencin ms importante. En el embolismo pulmonar, se apela a trombolisis (disolucin enzimtica del cogulo sanguneo), en particular
si hay trastornos del ventrculo derecho. En la EPOC, la terapia con oxgeno a larga duracin puede mejorar el cor pulmonale. Los principios generales de
tratamiento del cor pulmonale incluyen la reduccin del trabajo respiratorio, mediante ventilacin mecnica no invasiva, broncodilatadores y esteroides,
adems de tratar las infecciones subyacentes. Una oxigenacin adecuada (saturacin de oxgeno 90-92%) tambin permite reducir la resistencia vascular
pulmonar y reducir la demanda sobre el ventrculo derecho. Los pacientes anmicos necesitan una transfusin y si el hematocrito excede 65% debe
efectuarse una flebotoma para reducir la HTAP. Ver manejo para hipertensin pulmonar.
CASO CLINICO
Mujer de 48 aos, exfumadora de 10 paquetes/ao, diagnosticada de
sndrome de CREST e hipertensin pulmonar secundaria en
seguimiento en consulta de Neumologa desde hace seis aos. En la
actualidad, se encuentra en grado funcional III de la NYHA y est en
tratamiento con acenocumarol, sildenafilo, treprostinil subcutneo y
oxigenoterapia domiciliaria. Desde hace dos meses, presenta disnea de
mnimos esfuerzos (grado funcional IV de la NYHA) y edemas en
miembros inferiores, por lo que se aadi furosemida y
espironolactona al esquema teraputico. Hace una semana, comenz
con disnea de reposo y aumento de los edemas, por lo que se decidi

CURSO ENARM CMN SIGLO XXI TEL: 36246001

ingreso hospitalario. A la exploracin estaba afebril, taquicrdica y


taquipneica. A la auscultacin cardiaca presentaba un soplo sistlico
en foco pulmonar con refuerzo del segundo tono y en la auscultacin
pulmonar, crepitantes finos bibasales. En la exploracin del abdomen
se palpaba ascitis. En miembros inferiores presentaba edemas con
fvea hasta rodillas. Se realiz analtica sin alteraciones significativas y
radiografa de trax con aumento del ndice cardiotorcico e infiltrado
intersticial bilateral. En el ecocardiograma se apreciaba dilatacin
severa de cavidades derechas con regurgitacin tricuspdea severa y
PAPs de 59mmHg.

Pharmed Solutions Institute

PGINA 33

MANUAL DE TRABAJO DEL CURSO ENARM CMN SIGLO XXI


PREGUNTA
Cual de las siguientes aseveraciones no es correcta en el manejo de
esta patologia?
RESPUESTA
a.- El tratamiento de la hipertensin pulmonar se basa en tres pilares
fundamentales: medidas generales, anticoagulacin y tratamiento
vasodilatador.
b.- Entre las medidas generales estn evitar el ejercicio fsico que
provoque sntomas, las grandes altitudes, las situaciones de estrs
agudo y el embarazo.
c.- La anticoagulacin hay que iniciarla siempre que no exista
contraindicacin por el aumento del riesgo de trombosis derivado del
estado de hipercoagulabilidad de la hipertensin pulmonar.
d.- En todos los casos deber realizar vacunacin antigripal,
antineumococica y mantener profilaxias antibitica.
CASO CLINICO
Un hombre, albail de 50 aos de edad y agricultor fue admitido por
falta de aire y tos, de 4 aos de duracin, y edema bilateral inferior
durante 2 meses. En el EF dificultad respiratoria, una frecuencia
respiratoria de 50 rpm, pulso de 120 latidos por minuto, cianosis,
hipocratismo digital, pedal edema con fvea bilateral, a la izquierda
paraesternal tirn, p2 voz alta, grado 3 pansistlico murmullo, mxima
abajo a la izquierda borde esternal, aumento de la presin venosa
yugular, hepatomegalia, ascitis, as como las conclusiones del pecho de
parches sonidos respiratorios bronquiales y crepitantes.
PREGUNTA
Cual de las siguientes acciones no es adecuada para el caso?
RESPUESTA
a.- Oxgeno intermitente 4 l / min.
b.- Furosemida 80 mg cada 12 horas y espironolactona 50 mg.
c.- Ceftriaxona intravenosa y enoxaparina 40 mg subcutneo.
d.- Prednisolona 80 mg al da
CASO CLINICO
Paciente masculino de 58 aos de edad, con antecedente de
tabaquismo de 20 aos de evolucin, as como tuberculosis y
neumona tratadas, empleo albail. Consulta hace 10 das por cuadro
consistente, en disnea sbita que se present, por la madrugada
mientras dorma, no ceda con el reposo motivo por el cual el paciente,
acude al servicio de urgencias, recibiendo oxgeno y medicamentos
que no especifico, no refiere, cefalea, dolor en hombro izquierdo,
mareos emesis, ni dolor retroesternal. A la fecha refiere dolor urente,
difuso constante de intensidad leve, localizado en epigastrio que no
mejora con ninguna posicin, ni despus de las comidas, tos seca
crnica, refiere que en el hospital le diagnosticaron hipertensin, y
ulcera gstrica. EF: Orientado, consciente, con piel y tegumentos con
buena coloracin, T/A 110/90mmHg, FC 60x, FR 24X, presenta
ingurgitacin yugular, a la auscultacin, campos pulmonares bien
ventilados, con tiempo de espiracin prolongado, ruidos cardiacos
arrtmicos y disminuidos en intensidad. Se le realiza
electrocardiograma, el cual presenta datos de fibrilacin auricular e
hipertrofia del ventrculo derecho, bloqueo de rama derecha.
PREGUNTA
Cul es la conducta ms adecuada a seguir para confirmar el
diagnstico del paciente?
RESPUESTA
a.- Ecocardiograma ms cateterismo cardiaco derecho
b.- Ecocardiograma ms radiografa
c.- Tomografa mas radiografa
d.- Cateterismo cardiaco derecho ms radiografa

CURSO ENARM CMN SIGLO XXI TEL: 36246001

PREGUNTA
Cul es la causa ms probable, del diagnstico de esta paciente?
RESPUESTA
a.-Sndrome de apnea del sueo
b.- EPOC
c.- Hipertensin pulmonar primaria
d.- Obesidad
PREGUNTA
Cul es la conducta teraputica ms adecuada a seguir?
RESPUESTA
a.- Digoxina, morfina, expectorante
b.- Oxigeno, diltiazem, beta-agonista
c.- Oxigeno, diurtico, teofilina
d.- Digoxina, diurtico, teofilina.

CSO CLINICO
Mujer de 37 aos de edad sin antecedentes personales patolgicos
consulta a su medico de atencin primaria aquejando disnea de
esfuerxo acompaada de dolor torcico leve. Recientemente se
desmayo luego de subir un tramo de escaleras. Esta afebril la FC 90
lpm, FR rpm, TA 90/70mmHg, y la saturacin de oxigeno de 89%, en la
oximetris de pulso. El hallazgo principal en el examen cardiaco es un
componente pulmonar fuerte del segundo ruido cardiaco yun solplo
sistlico que se aprecia mejor en el borde esternal inferior. La
radiografia de torax revela campos pulmonares normales, con
cardiomegalia y vasos pulmonares agrandados. Las pruebas de funcin
ulmonar son normales, salvo por capacidad de difusin de 50%. Se le
diagnostico Hipertension pulmonar idioptica.
PREGUNTA
Cules serian los sntomas o signos menos probables de encontrar si
ya hubiera insuficiencia del hemicardio derecho?
RESPUESTA
a.- Distension de las venas del cuello
b.- Edema
c.- Hipotension
d.- Hepatomegalia

PREGUNTA
Cmo se define la hipertensin arterial pulmonar?
RESPUESTA
a.- Presion en arteria pulmonar >25mmHg en reposo y >30mmHg en
ejercicio
b.- Presion en arteria pulmonar >30mmHg en reposo y >35mmHg en
ejercicio
c.- Presion arterial media >130mmHg
d.- Presion arterial >80/40mmHg mas presin arterial pulmonar
>30mmHg
PREGUNTA
Cul es la conducta teraputica mas adecuada a seguir?
RESPUESTA
a.- Tratar morbilidad subyacente mas anticoagulacin, mas oxigeno y
mantener saturacin por arriba de 90%
b.- Tratar morbilidad mas tiazidas y un betabloqueador
c.- Tratar morbilidad subyacente mas ahorrador de K, mas
anticoagulante, oxigeno y mantener saturacin arriba de 92%
c.- Anticoagulacion mas tiazidas, oxigeno suplementario

Pharmed Solutions Institute

PGINA 34

MANUAL DE TRABAJO DEL CURSO ENARM CMN SIGLO XXI


CANCER DE PULMN
CIENCIAS BASICAS: Son tumores que surgen del epitelio respiratorio, los 4 tipos celulares principales son: No microciticos (Carcinoma de clulas
escamosas o epidermoide, adenocarcinoma, carcinoma de clulas grandes, el resto son indiferenciados. Carcinoma microcitico (clulas de avena).
Factores de riesgo: El principal tabaquismo, mostrando una relacin entre consumo y nmero de cigarrillos consumidos por da, as como por el inici y
tiempo de consumo dandonos el indice tabaquico que se calcula asi: N. de cigarrillos fumados al dia x aos de consumo entre 20. Indice tabaquico >20
paquetes/ ao; se considera con incremento en factor de riesgo para cncer pulmonar. Otros son asbesto, arsenico, niquel. Tambien los b-carotenos,
dieta rica en grasas mas obesidad, EPOC, tuberculosis. SALUD PUBLICA: Primer lugar de mortalidad en todos los tipos de cncer en el mundo occidental.
En Mxico la tasa de mortalidad para cncer pulmonar es de 125.2 por cada 100,000 en hombres y 48.8 por cada 100,000 en mujeres. Ms comun en
hombres de 50-70 aos. El 86% fallece dentro de los primeros 5 aos de diagnostico. PATOGENIA: El humo de tabaco contiene alrededor de 4800
compuestos; 60 son cancergenos. El mas estudiado es el benzopireno, que hace una transversion de guanina por tiamina y las nitrosaminas que median
reacciones de alquilacin de DNA. Se han detectado mutaciones en el gen p53 (cromosoma 17), el cual es un gen supresor de tumores, que al no
funcionar adecuadamente permite la sobrevida de elementos geneticamente daados, que conducen a la transformacin tumoral. Tambin hay
alteracines en el gen ras y myc. Existe un engrosamiento epitelial, un crecimiento celular desorganizado, nucleos atpicos, lo que es igual hiperplasia,
metaplasia escamosa, displasia, carcinoma in situ. CLASIFICACIN: Se dividen en benignas (2-5%) y malignas (95%). Malignas: Se dividen en 2: Cncer
broncogenico de celulas pequeas (20%; subtipos: carcinoma de cel. pequeas, mixto de cel. pequeas y grandes, combinado de cel. pequeas) y de
celulas no pequeas (80%; subtipos: adenocarcinoma 60%, escamoso o epidermoide 30%, carcinoma indiferenciado de cel. grandes <10%).
DIAGNOSTICO: Precoz, en pacientes mayores de 45 aos, fumadores, hacer citologa de esputo y radografia de torax, en dos proyecciones y TAC.
Paciente con sospecha; hacer historia clnica, BH, QS, PFH, deteccin de posibles METs. A todo paciente con mas de 3 semanas con tos, disnea,
hemoptisis, perdida de peso hacerles radiografia de torax. No hay un cuadro clnico especfico, puede haber tos (hasta 65%), hemoptisis, disnea, dolor
torcico, perdida de peso, fatiga. Hay otros signos y sntomas producidos por el crecimiento local del tumor como: obstruccin local de estructuras
adyacentes, crecimiento de ganglios, crecimiento en lugares distantes, efectos de productos sintetizados (sx., paraneoplasicos, el 30% caquexia,
anorexia, perdida de peso. 12% endocrinos como; hipercalcemia e hipofosfatemia, hiponatremia, alteraciones hidroelectroliticas). Si el crecimiento es
central o endobronquial predominan; tos, estridor y sibilancias, hemoptisis, disnea. Si el crecimiento es perifrico; dolor por afeccion pleural, tos,
absceso pulmonar. Manifestaciones clnicas por diseminacin regional como: 1. Obstruccin traqueal, 2. Compresion del esfago (disfagia), 3. Paralisis
del nervio larngeo recurrente (disfona), 4. Paralisis del nervio frnico, 5. Paralisis de nervios simptico (Sx. de Horner=ptosis, miosis, anhidrosis), 6. Sx.
de Pancoast; Es una forma de presentacin del carcinoma de pulmn (generalmente epidermoide), localizado en el surco pulmonar superior, cerca del
vrtice pulmonar, evolucin lenta, tendencia a infiltrar los linfticos subpleurales y por contiguidad raices de 8vo. cervical, 2do y 3er dorsal, cadena
simptica y estructuras seas. Clinica: Dolor en hombro irradiado a lo largo del brazo, parestesias (por infiltracin del plexo braquial), irradiacin a
hombro y codo, debilidad y atrofia de musc. intrnsecos, Sx. Horner, Rx. opacidad o engrosamiento unilateral en vrtice pulmonar, destruccion de 1ra y
2da., costilla. 7. Sindrome de vena cava superior; se debe a obstruccin vascular, inurgitacion de las venas yugulares, edema en esclavina, cianosis en
miembros superiores. El 75% de los derrames pleurales neoplsicos son secundarios a cncer pulmonar, son unilaterales, debido a disminucin de la
absorcin linftica por compromiso de ganglios mediastinicos. Sindromes esquelticos como acropaquias. Alteraciones de la coagulacin como sx. de
Trousseeau, CID. Puede haber metastatasis extratoracicas, el principal causante es el microcitico (90%) y adenocarcinoma (85%) y son a hueso, cerebro,
medula osea, heptica, ganglionares. En pacientes con signos y sntomas hay que establecer el diagnostico histolgico; broncoscopa, biopsia bronquial,
biopsia ganglionar, tumor de tejido blando. Rx., de trax; permite descubrir cncer pulmonar en asintomticos, los carcinomas escamosos y microciticos,
radigraficamente observamos masas centrales. En el adenocarcinoma y en el de clulas grandes, observamos masas perifricas. TAC; si detectamos
adenopatas <1cm benignas, 1-2cm indeterminados, >2cm positivo para malignidad, nos permite concoer actividad metastasica y til para estadificacin.
TRATAMIENTO: Para un tratamiento adecuado, se requiere de un diagnostico histolgico correcto. El tratamiento de urgencia, hay que resolver de
primera instacia el derrame pleural, neumotrax, hemoptisis, edema pulmonar, insuficiencia respiratoria, alteraciones en la mecnica respirtaoria. Tx.
Especifico; ciruga, quimioterapia + radioterapia + paliativo. Por regla general la afectacin de mediastino, significa que el tumor es inoperable. Los
tumores no microciticos, tienen la posibilidad de tratamiento curativo con ciruga a radioterapia sobretodo en etapas precoces, generalmente los
microciticos ya estn diseminados al momento del diagnostico, hay que dar quimioterapia, asociada a radioterapia. PREVENCION: Dejar de fumar
disminuye el riesgo, pero nunca vuelve a nivel de los no fumadores. ADENOCARCINOMA: No microcitico, cancer pulmonar mas frecuente (90% en
pacientes que fuman, 50% en casos nuevos, ex fumadores). La mayora se presenta en la periferia, tiende a invadir la pleura subyacente, puede asociarse
a linfoma de Hodgking, tiene gran capacidad metastasica. CANCER PULMONAR METASTSICO: El pulmon es un sitio muy frecuente para la implantacin
de metstasis de tumores pulmonares y extrapulmonares, hasta 30-40% de los canceres producirn metstasis a pulmn. Los tumores que mas afectan
son: cncer de mama, de prostata, gastrointestinales, rion. Ya sea por via hematgena, via linftica.
CASO CLINICO
Una mujer de 64 aos fue ingresada en el hospital por una disnea
creciente y dolor torcico. La paciente tena antecedentes de
carcinoma pulmonar no microctico y estaba recibiendo tratamiento.
Anteriormente haba fumado 4-5 cigarrillos al da durante un periodo
de hasta 10 aos. A la exploracin, se trataba de una mujer con
sensacin de enfermedad y emaciacin, que estaba clnicamente
anmica y presentaba signos de un derrame pleural izquierdo masivo
en la TC torcica. La frecuencia del pulso era de 85/min, la presin
arterial de 120/85, y no haba signos de insuficiencia cardiaca. La
radiografa de trax confirm el derrame unilateral izquierdo. La
hemoglobina era de 9,1g/100ml, con un frotis de sangre
normocrmico y normoctico. Los niveles sricos de ferritina, vitamina
B12 y folato eran normales. La velocidad de sedimentacin globular
fue de 90mm en la primera hora.
PREGUNTA
Considerando el caso clnico. Cual es el agente causal ms probable?
RESPUESTA
a.- Bordetela pertursis

CURSO ENARM CMN SIGLO XXI TEL: 36246001

b.- Streptococcus epidermidis.


c.- Staphylococcus aureus.
c.- Enteroccocus fecalis.
CASO CLINICO
Paciente de 69 aos, exfumador, diabtico y dislipmico con el
diagnstico de adenocarcinoma pulmonar obtenido mediante
fibrobroncoscopia que mostr una tumoracin en el orificio del
bronquio segmentario anterior del lbulo superior derecho. La
tomografa por emisin de positrones (PET-TC) informaba de una masa
hipercaptante (SUVmx. de 15g/ml) paramediastnica en el LSD
compatible con tumoracin maligna, una adenopata paratraqueal
inferior derecha con centro graso sin captacin de FDG y una lesin
paratraqueal
superior
derecha
de
8mm
discretamente
hipermetablica que se interpret como adenopata. Ante estos
hallazgos se estadific como T3NxM0. Al tratarse de un paciente
candidato a tratamiento quirrgico con intencionalidad radical, se
consider imprescindible la obtencin de muestras para descartar la
afectacin N2.

Pharmed Solutions Institute

PGINA 35

MANUAL DE TRABAJO DEL CURSO ENARM CMN SIGLO XXI


PREGUNTA
Cual de las siguientes mets es menos frecuente en este caso?
RESPUESTA
a.- Huesos
b.- Hgado,
c.- Tiroides.
d.- Glndulas suprarrenales.

PREGUNTA
Qu METs es menos probable encontrar en este caso?
REPUESTA
a.- Hgado
b.- Cerebro
c.- Hueso
d.- Mama

CASO CLINICO
Varn de 26 aos, sin hbitos txicos ni antecedentes patolgicos de
inters, que acudi a urgencias por disnea progresiva de una semana
de evolucin y expectoracin hemoptoica en las ltimas 48 h,
acompaadas de malestar general, distermia, dolor centrotorcico,
dolor en pelvis derecha y columna lumbar. Hemoglobina de 9 g/dl,
hematcrito del 27% y leucocitosis de 13.500 clulas/ l con
neutrofilia. En la bioqumica general se observaban patrn de
colestasis e hipoalbuminemia. La gasometra mostraba insuficiencia
respiratoria (pH: 7,45; presin arterial de anhdrido carbnico: 37
mmHg; presin arterial de oxgeno: 54 mmHg; HCO3: 27 mmol/l. En el
estudio de coagulacin se objetiv una actividad de protrombina del
50%, actividad de tromboplastina tisular de 17 s y fibringeno de 300
mg/dl. En la radiografa de trax se observaban un derrame pleural
derecho que ocupaba un tercio del hemitrax y patrn intersticial
bilateral, ms evidente en los lbulos superior derecho y medio.
PREGUNTA
Considerando el cuadro clnico cual es la conducta inmediata mas
adecuada?
RESPUESTA
a.- Toracocentesis
b.- Toracostomia diagnostica.
c.- Colocacion de sello.
d.- Drenaje endotoracico.
PREGUNTA
Considerando el cuadro clnico, cual es la clase mas probable?
RESPUESTA
a.- Cncer broncogenico de celulas pequeas.
b.- Carcinoma de clulas grandes.
c.- Adenocarcinoma escamoso.
d.- Carcinoma indiferenciado de celulas grandes.
CASO CLINICO
Paciente masculino de 75 aos que fue evaluado en consulta externa
por presencia de tos de 3 meses de evolucin as como expectoracin
hemoptoica de 1 mes de evolucin, asociada a toracodinia izquierda
inespecfica. Albail de profesin, tabaquista actual de 50
paquetes/ao. Hipertensin arterial controlada con enalapril
10mg/da. Al examen fsico presentaba: TA 120/80 mmHg, FC 90, FR
12, T 36. Ginecomastia bilateral, roncus bilaterales y sibilancias en
hemitrax izquierdo. No se constat hipocratismo digital ni signos
clnicos de fallo de bomba. El laboratorio mostr: Hematocrito 40%, GB
9900, VSG 20, PPD 0 mm, el hepatograma fue normal y el esputo para
coloracin de Ziehl Nielsen, grmenes comunes y hongos fue negativo.
En la radiografa de trax se observaba opacidad alveolar de aspecto
consolidativo sin signo de la silueta, en el lbulo inferior izq., en la
periferia. El paciente evolucion con prdida de 10 kg de peso y
persistencia de la expectoracin hemoptoica, por lo que se decidi su
internacin para estudio y tratamiento.
PREGUNTA
Cul es el diagnstico ms probable para este paciente?
RESPUESTA
a.- Adenocarcinoma de pulmn
b.- Carcinoma de clulas pequeas
c.- Carcinoma epidermoide
d.- Sndrome de pancoast

CURSO ENARM CMN SIGLO XXI TEL: 36246001

PREGUNTA
Hasta qu estadio el tratamiento es curativo, mediante ciruga?
RESPUESTA
a.- Estadio IIA
b.- Estadio IIIA
c.- Estadio IV A
d.- Estadio IV B
CASO CLINICO
Paciente femenina, de piel negra, 46 aos. Antecedentes patolgicos
familiares: padre fallecido por Infarto Miocrdico, madre fallecida
(Hipertensa). No refiere cirugas, transfusiones ni traumatismos.
Antecedentes patolgicos personales de Hipertensin Arterial.
Fumadora de 30 aos de evolucin, de dos a tres cajas de cigarros al
da. Seis meses antes de su ingreso, viene presentando aumento de
volumen de hombro derecho, por lo cual acudi primeramente al
ortopdico, este le observa uas en vidrio de reloj y dedos
hipocrticos, por lo cual la remite a consulta de Neumologa. Acude a
urgencias por dolor intenso y aumento importante de volumen de
hombro derecho e impotencia funcional. Refiere haber perdido
alrededor de 9 Kg de peso en los ltimos 6 meses. No tos, no falta de
aire. Al examen fsico inicial de Cuerpo de Guardia, se observan como
datos positivos: Aumento de volumen de hombro y miembro superior
derecho e impotencia funcional. Murmullo vesicular muy disminuido
hacia el vrtice derecho. Se indican complementarios, donde se
observa una imagen radiopaca irregular en vrtice del pulmn derecho
en la radiografa de trax anteroposterior y leucocitos 64,000 P: 0,80
Lf: 0,20 Hematcrito: 0,34. Durante su estancia en Sala de Medicina, se
le impone tratamiento para el dolor y la anemia; no cede el dolor, no
tos, no disnea. Se constata, das posteriores al ingreso, la aparicin de
adenopatas cervicales derechas no dolorosas, firmes, adheridas a
planos profundos, adems de esto una ptosis parpebral derecha con
discreta miosis de ese lado, no tos, no falta de aire.
PREGUNTA
Cul es el diagnstico ms probable para este paciente?
RESPUETA
a.- Adenocarcinoma de clulas no pequeas
b.- Carcinoma de clulas pequeas
c.- Sndrome de Pancoast
d.- Sndrome de Horner
PREGUNTA
Cul sera la histologa ms probable en este caso?
RESPUESTA
a.- Epidermoide
b.- Adenocarcinoma de clulas pequeas
c.- Carcinoma de clulas no pequeas
d.- Carcinoma escamoso
PREGUNTA
En cul de las siguientes patologas primarias es menos probable
encontrar el sndrome de Pancoast?
RESPUESTA
a.- Cncer de mama
b.- Cncer de esfago
c- Cncer de pleura
d.- Cncer de hueso

Pharmed Solutions Institute

PGINA 36

MANUAL DE TRABAJO DEL CURSO ENARM CMN SIGLO XXI


TRASTORNOS HIPOTALAMICOS E HIPOFISIARIOS
CIENCIAS BASICAS: El hipotlamo libera factores liberadores de hormonas hipofisiarias, asi como dopamina que inhibe la prolactina evitando su
liberacin, adems de la sntesis de arginina, vasopresina y oxitocina. El sndrome hipotalmico mas frecuente es la secresion anormarl de GnRH
consecuentemente reduccin de gonadotrofinas e hipogonadismo. Las causas mas comunes, esta el estrs, perdida de peso o ejercicio exesivo, el
tratamiento envuelve el manejo de la causa cuando es posible, la administracion pulsatile GnRH es usualmente afectivo. ADENOMA HIPOFISIARIO:
Neoplasias beninganas opriginadas en uno de los 5 tipos celular de la hipfisis anterior. Representa el 10% de todas las neoplasias craneales., son
tumores plurihormonales, secrecin autnoma hay escasa respuesta a vas de inhibicin. Clasificacion: Adenoma del somatotropo (hormona de
crecimiento), adenemoa de corticotropo (adrenocorticotrofina), adenoma del tirotropo (hormona estimulante del tiroides), adenoma del gonadotropo
(hormona foliculoiestimulante o luteinizante), prolactinoma (prolactina), adenoma no secretor (sin produccin hormonal). Efectos metablicos: cuando
afecta las regiones anteriores y preopticas del hipotlamo hay vasocontriccon paradjica, taquicardia e hipertermia (lesion del hipotlamo posterior),
lalesion en los nucleos ventromediales nos puede generar hiperfagia y obesidad, las lesiones hipotalmicas de lento crecimiento causan somnolencia,
obesidad, hipotermia y alteraciones emocionales. Efectos de compresin: cefaleas, perdida de visin por extensin suprasillar (compresin de quiasma
ptico), fenmeno de seccin de tallo (compresin del tallo de la hipfisis por tumo intrasillar, con afeccion de vaosos portales, interrupcin del paso de
hormonas hipotalmicas a la hipfisis, hiperprolactinemoa con dficit de otras hormonas), infiltracin lateral (afeccion del seno cavernosos, paralisis de
paraes cranelaes III, IV, VI), infiltracin hacia el techo del paladar. Campimetria por confrontacin, hemianopsia bitemporal, cortes de campos
homnimos lesion posquiasmaticos, cortes monoculares, lesion prequiasmatica, la perdida de color rojo es un signo temprano de presin sobre las vas
pticas. Tratamiento: en ausencia de hipersecrecin mantener en vigilancia, los grandes tumores hay que resecarlos por riesgo de infiltracin, ciruga
transesfenoidal, los efectos secundarios pueden ser diabetes inipida, hipopituitarismo transitorio, lesion de pares cranelaes, perforacin de tabique
nasal, alteraciones visulaes en 10%, perdida de liquido cefloraquideo, lesion de A. cartida, meningitis. Se puede dar radioterapia como coadyuvante a la
ciruga, puede haber secualeas como pedidad de olfato y gusto el seguimiento es de por vida. PROLACTINOMA: La prolactina se sintetiza en las clulas
lactotropas, mujeres de 10-25mcg/l, hombres 10-20mcg/l. El mecanismo de control central inhibitorio, es igual a la supresin de la liberacin de
prolactina por dopamina, los glococorticoides y la hormona tiroidea inhiben la secrecin de prolactina, incrementa con el ejercicio, el estrs y la comida,
en el embarazo y la lactancia. La prolactina induce y mantienen la produccin de leche, reduce la fucnion reproductora e inhibe el deseo sexual.
Hiperprolactinemia; sndrome de hipersecrecin hipofisiaria mas frecuente en hombres y mujeres, la causa mas frecuente es un adenoma hipofisiario
secretor, detecciones de >100mcg/l, puede haber elevaciones menores debidas a frmacos (metildopa, inhibe sintesis de dopamina), compresin de
tallo hipofisiario, hipotiroidismo, lesion del eje hipotlamo-hipofisiario (se altera la sintesisi de dopamina). Sintomas amenorrea, galactorrea, esterilidad,
disminucionm de densidad osea, disminucin del campo visual por defecto de compresin, en varones hay disminucin de la libido y perdida de la visin.
Diagnostico; niveles en ayunas >100mcg/L para macroedenomas, RMN. Tratamiento; agonistas de la dopamina como bromocriptina (accin breve, de
eleccin si se desea embarazo), cabergolina (accin prolongada, encogimiento del tumor), en pacientes hipotiroideos dar levotiroxina, la ciruga se
reserva para casos resistentes a ala dopamina o adenoma infiltrante con alteraciones visulaes. Los microadenomas (<1cm), no sulen infiltrar la regin
parasillar, los macroadenomas (>1cm), localmente infiltrantes, hay compresin de estructuras subayacentes. DEFICIENCIA DE GH: (Hormona del
crecimiento), causa cambios en el metabolismo de los lpidos, reduccin de masa muscular magra, hiperlipidemia, insuficiencia ventricular iozquierda,
desmineralizacin osea, alteraciones emocionales. Diagnostico: prueba de tolerancia a la insulina, es considerada como la prueba de oro para
diagnosticar deficiencia de hormona del crecimiento en adultos usando un umbral diagnostico de 0.3ng/l, en una prueba normal el cortisol y GH se
elevan despus de la insulina, pero en un paciente con hipopituitarismo, no se eleva ni el cortisol ni la GH, se mantienen constantes y bajas, Tambien se
puede pedir factor de crecimiento semejante a la insulina tipo I (IGF-I), se sintetiza en el hgado y otros rganos por estimulo de GH (puede estar
deficiente en cirrosis heptica y envejecimiento, Sx. de Laron es una ausencia de receptores hepticos para la hormona del crecimiento). Tratamiento:
GH 0.15-0.30mg/dl. SINDROME HIPOTALAMICO: Poco frecuente, ocurre en tumores, posterior a neurociruga por craneofaringeoma, o por infiltracin
como en la histiocitosis de clulas de Langerhans. La tpica presentacin es hiperfagia, con cambios de peso, prdida de sensacin, somnolencia, cambios
de conducta. Se debe regular los liquidos diarios y el balance. Los efectos metabolicos del dao hipotalmico por masa son frecuentemente alteracin
del apetito (hiperfagia y obesidad y anorexia), sed, adipsia, beber compulsivo, Temperatura (hipertermia, hipotermia), somnolencia y coma.
HIPOPITUITARISMO: Es una patologia que se caracteriza por la disminucin parcial o completa de las hormonas hipofisiarias anterior o posterior
(deficiencia de LH, FSH) o como producto de la interferencia del funcionamiento del hipotlamo sobre la actividad hipofisiaria. En el hombre puede
causar; infertilidad, impotencia, disminucin de la libido, disminucin de fuerza y masas musculares, de masa osea, eritropoyesisi disminuida, aumento
de arrugas cutneas. En la mujer pede causar; amenorrea, oligoamenorrea, anovulacin, baja libido, atrofia mamaria, osteoporisis, perdida de vello
pbico. Las causas mas frecuentes son los tumores hipofisiarios, tumores parahipofisiarios (craniofaringiomas, meningiomas, secundarios pecho,
pulmon, cordomas, gliomas) por radioterapia (hipofisiaria, craneal y nasofarngea) por infarto hipofisiario (sndrome de sheehan), Infiltrado hipofisiario
(sarcoidosis, linfociticas, hemocromatosis, histiocitosis de clulas de langerhans, enfermedad de Erdheim-Chester), sndrome de silla turca vacia, por
causa infecciosa, traumatica, sndrome de Kallman. ACROMEGALIA: Causa mas frecuente un adenoma de clulas somatotrofas, es de evolucin lenta,
hay un crecimiento excesivo, crecimiento del hueso frontal y mandibula, aumento del crecimiento de tejidos balndos, macroglosia, cardiomegalia, el
diagnostico es clnico si queda duda se solicita niveles sricos de IGF-I. Tratmiento anlogo de somatostanina como el octreotide o extirpacin quirrgica
transesfenoidal. DEFICIENCIA DE ACTH: Insuficiencia suprarrenal secundaria, fatiga, debilidad, anorexia, nauseas, vomito, hipoglucemia. La causa mas
frecuente es la interrupcin de la administracin de glucocorticoides. Tratamiento son glucocorticoides, prednisona. SINDROME DE CUSHING: Se debe a
un adenoma productor de ACTH, es un hipercortisolismo iatrogeno, clnica piel fina y quebradiza, obesidad central, hipertensin, cara de luna llena,e
strias violceas, fragilidad capilar osteoporosis. Diagnostico: determinacin de cortisol libre en oprina de 24 h. Tratamiento: extirpacin transesfeinodal
selectiva, el ketoconmazol, disminuye la concentracin de cortisol en pacientes con Cushing. DIABETES INSIPIDA: Producida por deficiencia absoluta o
parcial de vasopresina o por resistencia a ella, clnica poliuria (hasta 18 l/dia), polidipsia, hipernatremia, osmolaridad urinaria disminuida (<300mosm/kg)
y opsmolaridad plasmtica elevada (>290mosm/kg), la diabetes inspida central es la mas comn pero tambin hay nefrogenica. Diagnostico: densidad
urinaria <1.010, orina hipotnica, ADH disminuida. Se hace la prueba de vasopresina o pitressin,se dan 5U de vasopresina acuosa en 1000cc sol. Al 0.9% a
pasar en 1 hora, medir las unidades en miliosmoles, si se normalizan es diabetes inspida central. Tratamiento: reponer perdidas de agua con suero
fisiolgico al 0.9%, preparados hormonales como arginina-vasopresina via subcutnea, desmopresina en spray o intranasal. SIADH: (Sindrome de
secrecin inapropiada de ADH), dentro de susu criterios diagnosticos encontramos, Na plamatico <135mEq/l, osmolalidad plasmtica disminuida,
volumen extracelular normal, osmolalidad urinaria >100mOsm/Kg, Na urinario >40 mEq/l, ausencia de insuficiencia suprarrenal e hipotiroidismo.
(Normalmente la ADH, selibera para mejorar osmolaridad del plasma, si el paciente esta con una hiperosmolaridad del plasma, secreto ADH, para
retener agua y diluir plasma que esta hiperconcentrado), aqu lo que pasa es que aunque el plasma ya llego a su concentracin osmticas normales, se
sigue secrenatndo ADH, por eso es inapropiada. A este paciente la hiposmolaridad lo puede llevar a edema cerebral (movimiento de agua al interior
celular por gradiente osmtico), por expansin de volumen extracelular e hiponatremia dilucional. Una causa comn y la mas comn la hemorragia
subaracnoidea, otros como TCE. La clnica se relaciona directamente con la severidad de la hiponatremia: 130-135mEq/l sin sntomas. 125-130; anorexia,

CURSO ENARM CMN SIGLO XXI TEL: 36246001

Pharmed Solutions Institute

PGINA 37

MANUAL DE TRABAJO DEL CURSO ENARM CMN SIGLO XXI


nauseas, vomito, dolor abdominal. 115-125; agitacin, confusin, alucinaciones, incontinencia de esfnteres y otros sntomas neurolgicos. <155mEq/l;
convulsiones, coma, hipetension craneal. Tratamiento: quitar el factor desencadenanate, restriccin de liquidos, correcion de hiponatremia, frmacos
acuareticos, causan una diuressi acuosa, preservando electrolitos como mozavaptam, tixivapatam, solo en casoso severos o refractarios. Complicacion
ms importante de corregir una hiponatremia sbita nos puede dar mielinolisis pontina.
CASO CLINICO
Varn de 34 aos de edad, sin antecedentes, inici a los 31 aos con
dolor tpico, en regin supraorbitaria derecha, intenso, pulstil, de 3060 minutos de duracin, asociado a inyeccin conjuntival ipsilateral,
congestin nasal y en ocasiones rinorrea. Presentaba 2-3 ataques por
da en perodos que duraban de 7-10 das y se diagnostic con Cefalea
en Racimos CR episdica. El examen neurolgico no mostr datos
patolgicos y la imagen de resonancia magntica (RM) inicial no
mostr anormalidades. Se inici el tratamiento con verapamilo 240
mg/da y se administraba oxgeno y sumatriptn durante los ataques
desde el inicio del cuadro, con respuesta adecuada. Tres aos despus
se aadi disminucin de la agudeza visual (AV) y defecto en el campo
visual del ojo derecho. La evaluacin oftalmolgica report AV 20/100
y escotoma central en el ojo derecho y AV 20/20 sin alteracin del
campo visual en el ojo izquierdo.
PREGUNTA
Cual es su conducta a seguir mas adecuada para el caso?
RESPUESTA
a.- Realizar RX con nfasis en silla turca.
b.- Evaluacin de fondo de ojo con dilatacin.
c.- Resonancia magnetica contrastada.
d.- Perfil hormonal hipofisiario.
CASO CLINICO INCIDENTALOMAS
Mujer de 32 aos, con ciclos menstruales regulares, asintomtica y sin
tratamientos crnicos, a la que se le practic una RNM craneal en
febrero de tras un traumatismo craneoenceflico. Se objetiv un
adenoma hipofisario de 13 x 10 mm, intraselar, lateralizado hacia la
derecha. El campo visual y la exploracin oftalmolgica eran normales,
y las determinaciones de hormonas hipofisarias tambin. Tras
tratamiento con cabergolina (0,5 mg dos veces por semana) durante 3
meses. Ante la necesidad de un seguimiento indefinido,
previsiblemente largo por la edad de la paciente, se acord con ella la
exresis quirrgica del macroadenoma no funcionante, que se efectu
de forma total y selectiva, por va transesfenoidal sublabial.

b.- DI permanente.
c.- DI patrn bifsico (DI-SIADH).
d.- DI trifsico (DI-SIADH-DI).
CASO CLINICO (INFARTO HIPOFISIARIO)
Varn de 61 aos, con hipercolesterolemia e hipertensin arterial de
muchos aos de evolucin, controlada con medicacin. Se realizo
operacin de menisco de rodilla, bajo anestesia raqudea con 10 mg de
bupivacaina 0,5%. Durante la operacin la presin arterial sistolica
decreci desde 120 a 80 mm Hg, permaneciendo en ese nivel durante
tres horas y, el pulso, en 30-45 s/m. En el inmediato postoperatorio el
enfermo sufri una agudizacin grave de la cefalea, junto con un
cuadro confusional leve y ebre de 39. La cefalea era generalizada,
opresiva y no relacionada con la posicin ortosttica. Cuarenta y ocho
horas ms tarde el paciente presenta prdida de visin bilateral,
agitacin y mayor confusin mental. La determinacin de Na
plasmtico fue de 129 mmol/L.
PREGUNTA
Cual es la complicacin mas grave tras la correccin rpida del Na
plasmtico?
RESPUESTA
a.- Mielinolisis pontina
b.- Degeneracin pontino cerebelosa.
c.- Mielonolisis cortico basal.
d.- Hemorragia periacueductal.
CASO CLINICO
Masculino de 73 aos con una prdida de peso de 5 kg, fatiga, prdida
de apetito, desinteres por ambiente, falta de intensionalidad y tos
desde hace 2 meses, con expectoracin, Inform fumar un paquete de
cigarrillos por da durante los ltimos 50 aos. EF 162 cm de altura, 53
kg de peso, temperatura era de 35,2 C, el pulso 74/min y la presin
arterial 102/60 mmHg. Se auscultaron sibilancias y roncus audibles en
la zona basal izquierda. Se diagnostico cncer pulmonar, se agrego
posteriormente poliuria y polidipsia.

PREGUNTA
Cual es el porcentaje de reduccin de los incidentalomas?
RESPUESTA
a.- 60 %
b.- 30 %
c.- 20 %
d.- 10 %

PREGUNTA
Cual es su impresin diagnostica en este momento?
RESPUESTA
a.- Diabetes inspida.
b.- Sindrome de addison.
c.- Crisis hipotiroidea
d.- Panhipopituitarismo.

CASO CLINICO DIABETES INSIPIDA


Nio de cinco aos con un cuadro de pubertad precoz de origen
central, que durante el estudio etiolgico fue diagnosticado de una
lesin ocupante de espacio de localizacin supraselar El paciente fue
intervenido y posteriormente ingresado en la unidad de cuidados
intensivos peditricos para el control postoperatorio. Se administr
tratamiento antiinflamatorio con dexametasona y hormonal
sustitutivo con hidrocortisona. Transcurridas ocho horas desde la
intervencin el paciente present poliuria (diuresis de 1.080ml en 4
horas, 8ml/kg/h) y el control analtico revel: sodio plasmtico:
140mmol/l; osmolaridad plasmtica: 287 mosm/kg, con sodio urinario:
<3mmol/l y una osmolaridad urinaria: 108 mosm/kg.

CASO CLINICO ABCESO HIPOFISIARIO


Mujer de 35 aos, con antecedentes de asma bronquial y consumidora
de fin de semana de cocana inhalada, hbito no referido al ingreso,
que acude al Servicio de Urgencias por un cuadro de cefalea intensa de
predominio occipital, con nauseas, vmitos, fotofobia y fiebre de hasta
38 C. Se le practica una tomografa axial computarizada cerebral, que
no muestra hallazgos patolgicos, y una puncin lumbar, que evidencia
lquido cefalorraqudeo (LCR) con pleocitosis de predominio
polimorfonuclear e hipoglucorraquia. En la exploracin neurolgica, no
se evidenciaron hallazgos excepto rigidez de nuca con signos
menngeos positivos.

PREGUNTA
Considerando el cuadro clnico, cual de los siguientes diagnosticos
presuntivo es el mas frecuente?
RESPUESTA
a.- DI transitoria

CURSO ENARM CMN SIGLO XXI TEL: 36246001

PREGUNTA
Cual de la siguientes aseveraciones es incierta, en el padecimiento
actual?
REPUESTA
a.- La clnica puede ser indistinguible de la de un tumor hipofisario o
una infeccin del sistema nervioso central.

Pharmed Solutions Institute

PGINA 38

MANUAL DE TRABAJO DEL CURSO ENARM CMN SIGLO XXI


b.- Habitualmente, se presentan con insuficiencia pituitaria,
manifestada como hiperprolactinemia panhipopituitarismo.
c.- Las alteraciones visuales estn presentes en el 50-75%, la
heminopsia uni o bitemporal es el patrn hallado con ms frecuencia.
d.- La fiebre y la leucocitosis estn presentes hasta en el 46% de los
pacientes.
CASO CLINICO MACROADENOMA HIPOFISIARIO
Se trata de femenino de 27 aos de edad la cual acude a consulta
refiriendo visin borrosa en ambos ojos que le ha causado algunos
accidentes como tropezar o golpearse debido a que no alcanza a ver
los objetos, refiere adems que desde hace 2 aos aproximadamente
ha tenido cefalea que anteriormente se controlaba con la
administracin de AINES, en el examen del campo visual con
confrontacin digital se revela reduccin de la visin en la periferia
izquierda del ojo izquierdo y en la periferia derecha del ojo derecho, a
la exploracin general observa que la paciente presenta obesidad
grado I, acn e hirsutismo, la paciente agrega presentar alteraciones
menstruales.
PREGUNTA
Considerando los hallazgos en los campos visuales, donde es ms
probable que se ubique la lesin:
RESPUESTA
a.- Espacio suprasillar.
b.- Nervios pticos.
c.- Tracto posquiasmaticos.
d.- Cuerpos geniculados.
CASO CLINICO MACROADENOMA HIPOFISIARIO
Acude a consulta paciente masculino de 31 aos de edad el cual refiere
que desde hace 6 meses presenta cefalea continua que no sede a
AINES indicados por su mdico familiar, agrega que tuvo un accidente
automovilstico hace un mes, niega nausea o vomito, a la exploracin
fsica se observa con leve crecimiento de areas acras del cuerpo
principalmente en dedos y maxilar inferior, sus estudios de laboratorio
muestran dislipidemia e incremento de glucosa, finalmente refiere
disfuncin erctil.
PREGUNTA
Cul es la hormona responsable que ms probable se encuentre
alterada?
RESPUESTA
a.- ACTH
b.- PRL
c.- GH
d.- TSH.
CASO CLINICO
Paciente femenino, de 22 aos de edad, con antecedentes de
esclerodermia localizada desde los 6 aos de edad, por lo cual se trat
con reumatologa llevando tratamiento con D- Penicilamina hasta los
17 aos. Luego de cinco aos sin seguimiento acude a consulta por
presentar decaimiento y dolores articulares por lo que se decide
ingresar. Al interrogatorio se encuentra amenorrea de 5 aos de
evolucin (despus de una menarquia normal, con menstruaciones
normales hasta los 17 aos), adems de cefalea que a veces es
bitemporal, y en otras ocasiones es de localizacin frontoparietal que
se acompaa con trastornos visuales: visin borrosa, con disminucin
de la agudeza visual, en ocasiones visin doble cuando fija la mirada.
BH: 11.5mg/dl, LH: 2,2 UI/L, FSH: 7,31 UI/L, prolactina: > 5000,

CURSO ENARM CMN SIGLO XXI TEL: 36246001

campimetra: hemianopsia bitemporal. Rx de trax: no alteraciones


pleuropulmonares. Ultrasonido abdominal: rganos ginecolgicos muy
pequeos, el tero mide 27x21x31.
PREGUNTA
Cul es el diagnstico ms probable para esta paciente?
RESPUESTA
a.- Adenoma hipotalmico
b.- Adenoma hipofisario
c.- Hipertiroidismo
d.- Hipotiroidismo
PREGUNTA
Cul es la manifestacin clnica menos frecuente por la presencia de
hiperprolactinemia?
RESPUESTA
a.- Galactorrea
b.- Oligomenorrea
c.- Hirsutismo
d.- Infertilidad
PREGUNTA
Cul es la medida teraputica ms apropiada para este caso?
RESPUESTA
a.- Agonistas -adrenergicos
b.- Agonistas dopaminrgicos
c.- Agonistas colinrgicos
d.- Agonistas -adrenergicos
CASO CLINICO
Masculino de 19 aos de edad, de raza blanca, con antecedentes de
malnutricin, y traumatismo crneo enceflico grave, quien es
atendido por presentar poliuria y polidipsia, as como cefalea, de seis
meses de evolucin. Posteriormente, presenta vmitos, constatndose
al examen fsico signos de deshidratacin e hiponatremia en los
complementarios. Se ingresa con el diagnstico de una deshidratacin
hipertnica en el servicio de terapia intensiva.
PREGUNTA
Cul es la hormona ms probablemente afectada en este caso?
RESPUESTA
a.- Prolactina
b.- Vasopresina
c.- Corticotropina
d.- Hormona del crecimiento
PREGUNTA
Cul es la prueba diagnstica ms adecuada para este caso?
RESPUESTA
a.- Prueba de tolerancia a la glucosa
b.- Prueba de cortisol
c.- Prueba de vasopresina o pitressin
d.- Niveles de IGF-1
PREGUNTA
Cul es la medida teraputica ms adecuada para este caso?
RESPUESTA
a.- Prednisona
b.- Somatostatina
c.- Octreotide
d.- Desmopresina

Pharmed Solutions Institute

PGINA 39

MANUAL DE TRABAJO DEL CURSO ENARM CMN SIGLO XXI


TRASTORNOS TIROIDEOS
CIENCIAS BASICAS: Tiroides mayor glndula endocrina del cuerpo pesa 20grs, formada por los folculos tiroideos, en la luz del folculo esta el coloide
formado por tiroglobulina, rodeado por clulas A foliculares. La produccin de tiroxina (T4) 90% y triyodotironina (T3) 10%, se controla a travs de un
bucle de retroalimentacin endocrina clsica, se forman con la unin de la tiroglobulina ms iodo, mediante una peroxidacion. Parte de T3 es secretada
por tiroides, pero ms es producida por desyodacin de T4 en los tejidos perifrico su secrecion depende de TSH secretada en hipfisis posterior. Tanto la
T4 y T3 estn ligadas a protenas transportadoras (globulina de unin a la tiroxina (TBG) 80%, transtiretina, y albmina) en la circulacin. Aumento de los
niveles de T4 total y T3 con niveles libres normales se observan en los estados de aumento de protenas transportadoras (embarazo, los estrgenos,
cirrosis, hepatitis y trastornos hereditarios). Por el contrario, la disminucin de los niveles totales de T 4 y T3, con niveles libres normales se observan en la
enfermedad sistmica grave, enfermedad heptica crnica, y nefrosis. Solo la hormonas libre es la que esta biolgicamente disponible para lostejidos, la
unida a las protenas, es para aumehtar reservas circulantes y retrasar depuracin. Valores normales: T4; 64-154nmol/l, T3; 1.2-2.9nmol/l, T4 libre 1339nmol/l, T3 libre; 3.8-10nmol/l, TSH; 0.3-5.0.
HIPOTIROIDISMO (Hipofuncin de glndula tiroides)
PRIMARIO (tiroides)
TSH alta; T3,T4 baja

1.- Dficit de yodo: Principal causa a nivel mundial. 2.- Autoinmunitario: Tiroiditis de Hashimoto: 1ra causa donde no existe dficit de Iodo.
131
Tiroiditis atrfica. 3.- Iatrogena: Tiroidectoma total o subtotal, radiacin de cuello por tx. de linfoma o cncer, uso de I
. 4.Medicamentos: Amiodarona, litio, antitiroideos, ac. p-aminosalicilico, aminoglutecimida. 5.- Congnito: Disgenesia de tiroides (80-90%),
errores en hormonas tiroideas (10-15%), ectopia o hipoplasia de tiroides. 6.- Trastornos que infiltran tiroides: Amiloidosis, sarcoidosis,
hemocromatosis, esclerodermia.
H. TRANSITORIO
1.- Tiroiditis silenciosa (puerperio). 2.- Tiroiditis subaguda
SECUNDARIO
1.- Hipopituitarismo: Tumores de hipfisis, Qx, radiacin hipofisiaria, trastorno infiltrativo, traumatismos, Sx. de Shehann (necrosis
(hipfisis)
hipofisiaria post-parto por colapso circulatorio e isquemia, secundario a hemorragia obsttrica, caracterstica clnica, incapacidad para
TSH baja
lactar, por produccin insuficiente de prolactina, tamnien secrecin inadecuada de antidiuretica). 2.- Dficit o inactividad aislada de la TSH.
3.- Tratamiento con bexaroteno. 4.- Enf. Hipofisiaria: Tumores, traumatismos, trastornos infiltrativos, idioptico.
CUADRO
Debilidad, cansancio, intolerancia al frio, retardo mental, poco comunicativo, aumento de peso (almacenan glucosa como lpidos), disminucin de
CLINICO
apetito, aumento del peso (dificultad para bajar de peso), bradicardia, hipotensin, piel seca y spera, cada de pelo, extremidades fras, macroglosia
(por depsito de glucosaminoglicanos), menorragia, esterilidad, disfona, estreimiento, disminucin de reabsorcin tubular, derrame pericrdico, la
glucosa se almacena como grasa.
DIAGNOSTICO
Si TSH elevada; T4L y T4 normales = H. subclnico o leve Buscar Ac-TPO; positivo-tx. con T4. Negativo-vigilancia anual
Si TSH elevada; T4L y T4 disminuidas = H. clnico Buscar Ac TPO; positivo-autoinmunitario (presentes en 90-95%). Negativo-iatrogeno, otro. Si
TSH N o elevada; T4L Yt4 disminuidas = Orienta hacia causa hipofisiaria (H. secundario). TSH elevada marcador sensible de hipotiroidismo 1ro. Una
TSH NORMAL excluye una patologa primaria.
TRATAMIENTO
Primera eleccin levotiroxina (T4). Comienza su efecto hasta 4 sem., despus de su inicio. Dosis de 100-150g/24h VO, si hay cardiopata 12.525g, incrementando cada 6-8 semanas. Objetivo TSH normal; llegar al eutiroidismo (respuesta gradual aprox., 2 meses)
COMA
Forma ms SEVERA, urgencia mdica, alto ndice de mortalidad. Caracterstico alteraciones de las funciones mentales. Hipotensin, hipotermia,
MIXEMATOSO
bradicardia, convulsiones, hiponatremia, hipoglucemia. Factores de riesgo: sepsis, qx., anestesia, IAM, trauma, EVC. Se presenta en 8% F y 2% M.
TX.: levotiroxina 400g IV impregnacin; 100g IV c/24h para control. Hidrocortisona 50 mgs
CONGENITO
Prevalencia 1 de cada 4000, transitorio o permanente dependiendo si madre tiene AC frente a TSH-R. Cuadro clnico: hipotona, intolerancia a VO,
macroglosia, fontanela posterior amplia, ictericia prolongada. DX.: Tamiz neonatal, para evitar dao neurolgico permanente. TX.: Administracin
de T4, dosis 10.15g/kg/da
AUTOINMUNITARIO
Prevalencia en >60 aos. Incidencia 4 por cada 100 mujeres y 1 en cada 1000 hombres. Caracterstica clnica presencia de Bocio, irregular
y firme, frecuente asociado a DM, vitligo, LES, artritis reumatoide. Los sntomas se hacen evidentes con TSH >10mu/l

HIPERTIROIDISMO (Hiperfuncin de glndula tiroides)


PRIMARIO
(tiroides)
SECUNDARIO
(hipfisis)
TIROTOXICOSIS sin
hipertiroidismo

1.- Enf. Graves-Basedow. 2.- Bocio multinodular toxico. 3.- Adenoma toxico (estos 3 los ms frecuentes). 4.- Metstasis de cncer de tiroides.
5.- Mutacin activada del receptor de TSH. 6.- Estruma ovrico. 7.- Fenmeno de Jod-Basedow (exceso de I)
1.- Tumores que secretan gonadotrofina corionica con TSH baja. 2.- Tirotoxicosis en el embarazo con TSH baja. 3.- Tiroiditis subaguda. 4.Tiroiditis silenciosa
Estado hipermetabolico donde existe un exceso de hormonas T3 y T4 libres, no es sinnimo de hipertiroidismo, que es el resultado de un
exceso de la funcin tiroidea. Causas: Hipertiroidismo causadao por enf., de Graves, frmacos: amiodarona. 2.- radiacin a cuello por
linfomas o cncer. 3.- Infarto de un adenoma. 4.- Tirotoxicosis facticia (ingesta excesiva de T4)
CUADRO
Hiperactividad, nerviosismo, irritabilidad, disforia, palpitaciones, taquicardia sinusal (lo ms frecuente y que puede llevar a fibrilacin auricular),
CLINICO
hipertensin, insomnio, fatiga, debilidad muscular, prdida de peso, a pesar de aumento de apetito (por aumento de gluconeognesis), diarrea,
intolerancia al calor, piel caliente, transpiracin exagerada, poliuria, perdida de libido, temblor, miopata proximal, retraccin palpebral mirada de
asombro, ginecomastia, hipersexualidad, esterilidad
DIAGNOSTICO
TSH srica disminuida es un marcador sensible de hipertiroidismo causado por la enfermedad de Graves, los ndulos tiroideos autnomos,
tiroiditis, y el tratamiento con levotiroxina exgena. Nivel T4 elevada suficiente para tirotoxicosis.
TRATAMIE
Antitiroideos (tionamidas): Carbimazol, metimazol 10-20mg c/8-12h inicio; 2.5-10mg para control. Propiltiouracilo (PTU) (se puede usar en embarazo)
NTO
100-200mg c/6-8h inicio; 50-100mg para control, efectos secundarios rash, urticaria, fiebre, agranulocitosis (ms importante). Iodo ra diactivo;
destruccin de glndula tiroides indicado en recidivas, contraindicacin absoluta en embarazo y lactancia. Tiroidectoma; cuando falla tx. mdico y se
rehsan al Iodo, til en jvenes con bocios voluminosos, complicaciones; hemorragia, edema larngeo, hipoparatiroidismo, lesin de N. larngeo
recurrente. Funcin tiroidea debe ser checada cada 3-4 sem. despus de iniciar tx., con ajustes para mantener T 4 en niveles normales. Propanolol 2040mg VO, para disminuir taquicardia y sntomas adrenrgicos. Tormenta tiroidea: PTU 500mg impregnacin VO, rectal o por sonda nasogstrica, para
continuar PTU 200-300mg c/6h, propanolol y dexametasona.
GRAVESHasta 60-80% el ms comn. Principalmente mujeres. Edad entre 20-50 aos. Para su desarrollo se necesita la actividad inmunolgica de las
BASEDOW
inmunoglobulinas estimulantes de tiroides (TSI), son anticuerpos que estimulan a la TSH-R. Se puede desarrollar oftalmopatia por la alteracin de los
msculos extraoculares por infiltracin de linfocitos T activados, que inducen la liberacin de IL-1, IFN y TNF, que a su vez activan a los fibroblastos y
aumentando sntesis de glicosaminoglicanos, produciendo tumefaccin de los msculos. Cuando se comprime el N. ptico se produce edema papilar,
defectos en campos perifricos y ceguera. Puede haber dermopata; que es una placa indurada violcea o rosada piel de naranja. Tx. Antitiroideo
hasta por 12-18 meses para ver efecto, podemos esperar hasta 2 aos.
CRISIS TIROTOXICA O
Es rara, suele ocurrir despus de una enf. Estresante en pacientes no tratados, se caracteriza por taquicardia, fiebre, delirium, vomito
TORMENTA TIROIDEA
diarrea, deshidratacin, arritmias, coma. Exacerbacin de hipertiroidismo. Las concentraciones sricas de TG estn elevadas (su principal
funcin de la determinacin de Tg, es el seguimiento de cncer de tiroides)

La ecografa nos ayuda a ver lesion solitaria o multinodular. Aumento de tamao heterogeno como en tiroidis de Hashimoto. El gamagrama de tiroides,
permite visualizar directamente la glandula y cuantificar la capatacion de marcador radiactivo, diferenciamos entre enf. De Graves (capacin de manera
homognea, aumento de tamao), adenomas toxicos (reas focales de mayor captacin), bocio multinodular (aumento de tamao, multiples reas de

CURSO ENARM CMN SIGLO XXI TEL: 36246001

Pharmed Solutions Institute

PGINA 40

MANUAL DE TRABAJO DEL CURSO ENARM CMN SIGLO XXI


capatacion). NO CAPTA (ndulos frios), capatacion reducida de marcador suelen ser benignos, mas probabilidad de cncer, CAPTA (nodulos calientes),
prcticamente nunca son malignos. BOCIO MULTINODULAR TOXICO: Bocio voluminoso, por lo general en edad avanzada y se puede presentar con
fibrilacin auricular o palpitaciones, taquicardia, nerviosismo, prdida de peso. La exposicin reciente a yodo, a partir de los medios de contraste o de
otras fuentes, puede precipitar o exacerbar la tirotoxicosis; esto puede evitarse mediante la administracin previa de un frmaco antitiroideo. Dx:
Niveles de TSH bajos, T4 N o mnimamente elevado. Gammagrafa de la tiroides muestra una captacin heterognea con zonas mltiple de aumento y
disminucin de la captacin. TUMORES TIROIDEOS: Benignos=adenomas, malignos =carcinomas. Incidencia mundial de 0.5-10 casos por 100 000, ms en
mujeres, entre 40-50 aos, peor pronstico en extremos de la vida. Factores de riesgo: radiacin a cabeza y cuello en la infancia, ndulo tiroideo de 2-4
cm, signos de fijacin a estructuras vecinas, invasin a ganglios linfticos, parlisis de cuerdas vocales, antecedente de neoplasia endocrina mltiple tipo
2 (NEM2). Dentro de los carcinomas hay foliculares, papilares y anaplasicos de tiroides (raro, muy maligno y fatal). Carcinoma papilar tiroideo subtipo
ms frecuente (70-90%) y menos agresivo, es multicentrico, el antecedente principal es radiacin a cuello en la infancia, generalmente asintomtico en
ocasiones disfagia y voz ronca, sospecharlo en ndulos solitarios palpables.
CASO CLINICO BOCIO SIMPLE
Acude a la consulta externa paciente femenino de 25 aos de edad,
acude debido a que no produce suficiente leche para lactar tiene 8 dias
de puerperio, niega otra sintomatologia, se le explica la tcnica para
lactacin, sin embargo a la exploracin se observa incremento
simetrico de glandula tiroides, no se palpan ndulos ni zonas
dolorosas, no se identificas cambios de voz, la paciente es originaria de
Chiapas.
PREGUNTA
Cual la conducta a seguir mas adecuada para el caso?
RESPUESTA
a.- Realizar valoracin de THS y T4.
b.- Indicar suplementacion de yodo.
c.- Indicar alimentacin rica en yodo, cita 6 meses.
d.- Solicitar perfil de anticuerpos antitiroideos.
PREGUNTA
6 meses despus se envio a endocrinologa debido a que presento
incremento de peso, se mantuvo las mismas caractersticas de
volumen y consistencia de la glandula tiroidea, fue realizado un perfil
tiroide con los siguientes resultados T4 total baja, TSH normal e indico
100 mcg/dia, considerando la evolucin. cual de los siguientes
diagnostico diferenciales se debe causas no tiroideas?
RESPUESTA
a.- Sindrome de T4 baja.
b.- Bocio multinodular no toxico.
c.- Bocio multinodular toxico.
d.- Nodulo solitario hiperfuncionante.
CASO CLINICO HIPOTIROIDISMO
Paciente diagnosticada de hipotiroidismo subclnico y tratada
adecuadamente con tiroxina oral. Unos meses despus desarrolla
exoftalmos bilateral y simultneamente presenta anticuerpos sricos
anti-receptor de TSH positivos. Se suspende entonces el tratamiento
con tiroxina, y se comprueba que el hipotiroidismo ha progresado
hasta hacerse primario. Se establece el diagnstico de enfermedad de
Graves hipotiroidea.
PREGUNTA
Cual es la complicacin mas frecuente a largo plazo para el paciente
debido al tratamiento que se debe instaurar?
RESPUESTA
a.- Osteoporosis.
b.- Arteriosclerosis.
c.- Edema pulmonar.
d.- Dislipidemia.
CASO CLINICO HIPOTIROIDISMO AUTOINMUNE
Acude a consulta de revisin anual paciente femenino de 31 aos, la
cual cuenta con el diagnostico de bocio simple, refiere que ha
presentado auscencia de periodo menstrual en 4 ocaciones en el aos,
adems de presentar infecciones de vas areas altas en 2 ocaciones,
ha presentado cansancio, debilidad generalizada, incremento de peso.
A la exploracin fsica se observa bradipsiquia y bradilalica, tiene un
aspecto seco y aspero. Los signos vitales fueron TA 110/70 mmHg.

CURSO ENARM CMN SIGLO XXI TEL: 36246001

PREGUNTA
Se ingreso a la paciente debido al malestar generalizado e hipotermia,
cual es la conducta inmediata a seguir para establecer un diagnostico
presuntivo.
RESPUESTA
a.- BH, QS, EGO.
b.- Solicita TSH, T4.
c.- Solicitar Anticuerpos peroxidasa tiroideos.
d.- Biopsia de tiroides.
PREGUNTA
Se obtuvieron los siguientes resultados hematocrito 32 %,
hemoglobina 10.1, Leucocitos 5,900, glucosa 219 central, colesterol
310. Considerando los resultados cual es la conducta a seguir ms
adecuada.
RESPUESTA
a.- Iniciar levotiroxina 150 mcg/ dia.
b.- Solicitar perfil tiroideo con anticuerpos.
c.- Estabilizacion y posterior envio a segundo nivel.
d.- Administracin de prednisona 20 mg y levotiroxina 100 mcg/dia.
PREGUNTA
2 meses despus regresa la paciente para control por el primer nivel
de atencin, en su nota de contrarreferencia se encuentra el
diagnostico de tiroiditis de Hashimoto, considerando la evolucin del
paciente. Cual de las siguientes comorbilidades es la menos
probable?
RESPUESTA
a.- Diabetes mellitus.
b.- Enfermedad de Addison.
c.- Anemia perniciosa.
d.- Infiltracion leucocitaria hipofisiaria.
CASO CLINICO HIPERTIROIDISMO
Nia de 12 aos de edad, que desde hace 4-5 meses inicia con
nerviosismo, taquicardia, intolerancia al calor, sudoracin profusa e
impulsividad para comer, sin prdida de peso ni insomnio. Asimismo,
refiere abultamiento en la parte anterior del cuello, sin dolor ni otros
signos inflamatorios asociados. Su madre tiene hipotiroidismo en
tratamiento con L-tiroxina desde hace 7 aos. En la exploracin fsica
destaca una protrusin ocular bilateral y un bocio grado II, de
consistencia elstica, sin dolor ni otros signos inflamatorios. El estudio
de hormonas tiroideas mostr hipertiroidismo con anticuerpos
antitiroideos elevados. La ecografa tiroidea objetiv ambos lbulos
tiroideos aumentados de tamao (derecho 18 x 18,8 x 53mm,
izquierdo 15,9 x 18 x 51mm), heterogneos, con mltiples zonas
hipoecognicas de bordes imprecisos que confluyen y una marcada
hipervascularidad con Doppler.
PREGUNTA
Cual es la medida teraputica mas apropiada a largo plazo?
RESPUESTA
a.- Tiroidectomia parcial.
b.- Iodo radioactivo.
c.- Propiltiouracilo.

Pharmed Solutions Institute

PGINA 41

MANUAL DE TRABAJO DEL CURSO ENARM CMN SIGLO XXI


d.- Metimazol.
CASO CLINICO CRISIS TIROIDEA
Mujer de 66 aos de edad, que ingresa por proteinuria, edema
generalizado e hipertensin de reciente comienzo, refractaria a
tratamiento. En la analtica destacaban: urea 78 mg/dl, creatinina 1,1
mg/dl; aclaramiento de creatinina de 48 ml/min; protenas totales 4,6
g/dl; albmina 1,2 g/dl. Perfil proteico compatible con sndrome
nefrtico. Colesterol: 484 mg/dl. Triglicridos: 180 mg/dl. LDL: 386
mg/dl. Autoinmunidad y complemento dentro de la normalidad.
Inmunocomplejos circulantes negativos. TSH: 10,17 mU/l, T4: 0,78
mg/dl. Anticuerpos antimicrosomales: 84 U/ml. Anticuerpos
antitiroglobulina: 4 U/ml. Anticuerpos antirreceptor TSH: 1,6 U/l.
Proteinuria hasta 10 g/24 hrs. Se realiz biopsia renal eco-dirigida con
resultado anatomopatolgico de glomerulonefritis membranosa. Ante
el diagnstico de glomerulonefritis membranosa probablemente
secundaria a tiroiditis de Hashimoto, se comenz tratamiento
mediante levotiroxina, estatina y doble bloqueo del sistema reninaangiotensina-aldosterona.
PREGUNTA
Cual es la condicin que genera el edema en este caso?
RESPUESTA
a.- Proteinuria.
b.- Presion hidrosttica.
c.- Glomerulonefritis.
d.- Bloque ECA.
CASO CLINICO CRISIS TIROTOXICA
Femenino de 28 aos de edad que acude a urgencias debido a que
presenta irritabilidad, ideas delirantes de dao, taquicardia, diaforesis,
taquipnea, hipertermia, opresin torcica, vomito y diarrea as como
hipotensin arterial, adems presenta dolor abdominal difuso,
peristaltismo presente, la paciente presenta piel caliente, delgada,
pelo quebradizo y con cada de este, al llegar a urgencias se observa en
delirium, los familiares acompaantes refieren que la paciente tiene
antecedentes de enfermedad endocrinolgica y se encontraba en
espera de reseccin de ndulos tiroides.
PREGUNTA
Usted considera por el cuadro clnico y el antecedente referido que se
trata de una crisis tirotoxica, usted debe decidir cul es el tratamiento
ms adecuado de primera instancia:
RESPUESTA
a.- Emplear rodio radioactivo inmediatamente.
b.- Usted indica dexametazona.
c.- Emplea metoprolol y metimazol.
d.- Decide indicar Propiltiouracilo.
CASO CLINICO
Mujer de 28 aos acude a la consulta por presentar, de 2 semanas de
evolucin, palpitaciones y temblor en las manos. Cursa con embarazo
de 22 semanas por ultrasonido, ya que desconoce la fecha de su ltima
menstruacin. Niega prdidas vaginales y los movimientos fetales
estn presentes, refiere hbito intestinal diario. El control prenatal ha
sido adecuado y no se han encontrado alteraciones de importancia.
Antecedentes familiares tiene una madre hipertensa en tratamiento.
Antecedentes obsttricos muestran una menarquia a los 14 aos, los
ciclos regulares de 30 x 5 y el perfil obsttrico es G2 P1 A0, parto
anterior vaginal hace tres aos. EF: paciente en adecuadas condiciones
generales, irritable, piel caliente y sudoracin moderada, con talla 1,60
m, peso 62 kg, IMC 24,22 kg/m2, TA 130/85, FC 110 x', FR 18 x'. La
palpacin del cuello es normal. La auscultacin cardiopulmonar es
normal, excepto por la presencia de taquicardia. El examen abdominal

CURSO ENARM CMN SIGLO XXI TEL: 36246001

revela un tero grvido con altura de 20 cm y frecuencia cardiaca fetal


de 140 x' audible con doppler; feto nico. Al tacto vaginal se encuentra
una vagina normal con un cuello posterior, largo, cerrado, sin flujo ni
sangrado.
PREGUNTA
Cul es el diagnstico ms probable para esta paciente?
RESPUESTA
a.- Preeclampsia
b.- Corioamnioitis
c.- Hipertensin arterial crnica.
d.- Hipertiroidismo
PREGUNTA
Cul es la conducta teraputica ms adecuada para esta paciente?
RESPUESTA
a.- Alfa-metildopa
b.- Hidralazina
c.- Propiltiouracilo
d.- Metimazol
PREGUNTA
Cul es la causa menos probable de hipertiroidismo primario?
RESPUESTA
a.- Enfermedad de Graves
b.- Bocio multinodular toxico
c.- Adenoma toxico
d.- Tirotoxicosis gravdica
CASO CLINICO
Paciente femenino de 59 aos quien refiere ligera fatiga desde hace
ms de 10 aos y dificultad para bajar de peso. Sus estudios de
laboratorio mostraron TSH srica de 15 mU/L, T4 libre 0.6, colesterol
srico de 220 mg/dl, LDL de 140 mg/dl y anticuerpos antiperoxidasa
positivo. A la exploracin fsica no se encuentran alteraciones, excepto
por una tiroides firme y pequea con superficie ligeramente irregular y
ndulo firme, mvil, no doloroso, en parte superior de lbulo derecho
de aprox 1.5 cm.
PREGUNTA
Cul de las siguientes protenas es la ms probable que transporte la
mayor cantidad de hormonas tiroideas?
RESPUESTA
a.- Globulina de unin a la tiroxina
b.- Transtiretina
c.- Albumina
d.- Somatostatina
PREGUNTA
Cul es el diagnstico ms probable en este caso?
RESPUESTA
a.- Hipotiroidismo primario
b.- Hipotiroidismo secundario
c.- Hipertiroidismo primario
d.- Hipertiroidismo secundario
PREGUNTA
Qu efectos fisiolgicos es ms probable encontrar en este paciente?
RESPUESTA
a.- Almacena glucosa como lpidos, insominio, disminucin poderal de
peso
b.- Aumento de apetito, disminucin ponderal de peso
c.- Almacena glucosa como lpidos, aumento ponderal de peso,
disminuye apetito
d.- Aumento ponderal de peso, aumento de apetito, insomnio

Pharmed Solutions Institute

PGINA 42

MANUAL DE TRABAJO DEL CURSO ENARM CMN SIGLO XXI


GLANDULAS PARATIROIDES
CIENCIAS BASICAS: Las glndulas producen hormona paratiroidea (PTH), la cual controla las concentraciones de calcio y fosforo en la sangre, incrementa
la resorcin sea, aumenta la recaptacin de calcio y disminucin de fosfato, incrementa la absorcin gastrointestinal de calcio, e induce la conversin
renal de la vitamina D hasta su forma activa. HIPERPARATIROIDISMO PRIMARIO (HPP): Trastorno endocrino ms frecuente (despus de DM),
caracterizado por secrecin excesiva de hormona paratiroidea, que condiciona a hipercalcemia crnica, predomina en mujeres que en hombres (4:1),
entre 4-5ta dcada de la vida, origen desconocido, se ha descrito asociacin con irradiacin ionizante de cuello, las causas incluyen adenoma 85%,
hiperplasia primaria difusa o nodular 10-55%, y por ltimo el carcinoma <1%. El adenoma paratoiroideo: frecuentemente asociado a patrn familiar,
espordico por inversin del cromosoma 11 y mutacin del gen MEN-1, suelen ser masas nicas de color rojizomarrn con capsula bien definida,
principalmente de cel., principales y solo afectan una glndula. Hiperplasia primaria: Suele afectar las 4 glndulas, se manifiesta por aumento de las
clulas principales, los hallazgos clsicos de hiperplasia y la presencia de tejido adiposo es caracterstico: Carcinoma de paratiroides: infrecuente ms
comn en la 6ta. Dcada de la vida, bulto en cuello, grande (12g) dificultad al hablar o tragar, astenia y es muy comn este asociado a hipercalcemia
severa. PATOGENIA: Existen 3 mecanismos: cambio del punto de inflexin del sensor de calcio, mayor proporcin de cel. Activas reflejando la mayor
actividad secretora, y un aumento del nmero total de cel. Parenquimatosas. DIAGNOSTICO: 2 formas; con afeccin renal y la que manifiesta
principalmente una participacin sea. Cuadro clnico: cefalea frecuente, depresin, HTA, incapacidad para concentrarse, hipercalcemia (22%),
normocalcemico (8%). sea; dolor, deformidad, quistes y susceptibilidad a las fracturas, lo clsico es la ostetis fibrosa qustica. Renal; nefrolitiasis (38%),
nefrocalcinosis, disminucin del filtrado glomerular y diversas anomalas tubulares, acidosis tubular. Otras manifestaciones derivan de la hipercalcemia
crnica; debilidad muscular proximal, fatiga fcil y atrofia, depresin, cambios de la personalidad y psicosis, estreimiento, pancreatitis, osteoartritis
degenerativa y avulsin de tendones. Laboratorio: El HPP, es la causa mas frecuente de elevacin de calcio srico ambulatorio, hay que recordar la
asociacin de HPP a tumores, la determinacin de PTH en plasma es el mtodo dx., definitivo para discriminar un HPP de otras causas de hipercalcemia.
Las nicas hipercalcemias de origen no paratiroideo, donde la PTH esta elevada son los tratamientos con litio o diurticos tiazidicos. El aumento de
calcio filtrado a travs del glomrulo conduce a hipercalciuria y aunque la PTH produce un aumento de la reabsorcin tubular de calcio, el resultado de
ambos factores contradictorios condiciona a la aparicin de calcio en orina, que justifica la mayor frecuencia de litiasis. Rx de torax (masa mediastinica),
densitometra sea, TAC sesibilidad de 65%. TRATAMIENTO: Inicial, indicaciones; calcemia inferior a 11.5mg/ml, ausencia de sntomas en relacin directa
con la enfermedad o contraindicacin para ciruga. Aumentar ingesta hdrica, reducir calcio de la dieta y movilizacin inmediata, suprimir frmacos como
diurticos tiazidicos y carbonato de litio. Se pueden usar calcitonina y bifosfonatos orales (para inhibir los efectos de la PTH), para disminuir la absorcin
intestinal de calcio y en nefrolitiasis, es til la celulosa fosfato sdica. El tx., de HPP es exclusivamente quirrgico, sobre todo en sus fases sintomticas
con hipercalcemia o complicaciones viscerales. Son indicaciones absolutas de ciruga; calcemia >11mg/dl, hipofosfatemia grave, disminucion de la
funcin renal, urolitiasis metablicamente activa, sntomas objetivos de hipercalcemia , hipercalciuria >400mg/dl, calcificacin de tejidos blandos,
evidencia radiogrfica de ostetis fibroquistica, ulcera pptica o pancreatitis recidivante. HIPOPARATIROIDISMO: Trastorno causado por la hipofuncin
de las glndulas paratiroides, caracterizada por una muy baja concentracin de PTH, de calcio y un aumento en la concentracin de fsforo sanguneo. La
PTH es la hormona encargada de mantener los niveles normales de calcio en el suero sanguneo, favoreciendo su absorcin en el tracto digestivo por
medio de la activacin de la vitamina D3 en el rin con su subsiguiente migracin hacia en epitelio intestinal y la resorcin sea aumentando
indirectamente la actividad de los osteoclastos en el hueso, aparte del incremento de la captacin por parte de los tbulos distales de las nefronas que
reabsorben este mineral. CAUSAS: Extirpacin quirrgica accidental o intencional durante la tiroidectoma, siendo esta la causa ms frecuente. Dentro
de las causas no quirrgicas los procesos autoinmunes son los ms importantes (sndrome poliglandular autoinmune). La Ausencia o disfuncin
congnita de la glndula paratiroides la cual se ha relacionado con microdelecciones dentro del cromosoma 22q11 (sndrome de DiGeorge). Las
deficiencias de magnesio. La Hemocromatosis, causante de un cuadro de disfuncin de los rganos endocrinos debido al acumulo de hierro en el interior
de las clulas. Idiopticos. PATOLOGA: Siendo el calcio uno de los iones ms importantes en el mantenimiento de la homeostasia normal del cuerpo,
adems de estar implicado en la contraccin del msculo esqueltico estriado, liso y cardiaco, cualquier variacin en sus concentracin desencadena un
alto nmero de desordenes funcionales todo esto producido en el caso del hipoparatiroidismo por la disminucin del potencial de accin, lo que lleva al
aumento de la excitabilidad celular, favoreciendo en el caso del msculo la aparicin de tetania y el SNC de convulsiones. Adems de esto el calcio est
relacionado con los procesos de secrecin glandular favoreciendo la unin de las vesculas de secrecin a la pared celular promoviendo as su exocitosis,
la disminucin de este mineral afecta significativamente este suceso lo que se ve dilucidado en la reduccin de la sudoracin que presentan los paciente
que tienen este proceso patolgico. CLNICA: Los ms frecuentes pueden incluir: Disnea, dolor clico agudo, hormigueos, contracciones de distintos
grupos musculares. Son muy caractersticos las localizadas en algunos msculos del brazo y mano. Esta posicin incluso puede ser provocada
estimulando los msculos del brazo mediante una compresin con el manguito de un esfigmomanmetro, lo que se conoce como el signo de Trousseau.
Tambin aparecen frecuentemente cataratas, trastornos psquicos, y piel seca y otros trastornos trficos de la piel. Los trastornos mentales ms
frecuentes de la hipocalcemia aguda causada por el hipoparatiroidismo es delirium y la psicosis. En la hipocalcemia crnica son ms frecuentes los
sntomas de ansiedad, irritabilidad, labilidad emocional, depresin, psicosis, alteracin de la memoria y de la concentracin, deterioro cognitivo y a veces
retraso mental en funcin de la edad de comienzo. El compromiso intelectual aparece en el 50% de los casos de hipoparatiroidismo. Variantes clnicas:
Pseudohipoparatiroidismo: En este cuadro, se aaden trastornos constitucionales como talla corta, cara ancha, metacarpianos y metatarsianos cortos.
Pseudo-pseudohipoparatiroidismo: En el que slo aparecen los trastornos constitucionales descritos, sin alteracin del calcio. TRATAMIENTO:
Administracin de la hormona paratiroidea (PTH) pero principalmente de la administracin de calcio y vitamina D en caso de que se trate de la verdadera
forma del hipoparatiroidismo la cual presenta bajos niveles de calcio srico.
CASO CLINICO HIPOPARATIROIDISMO
Mujer de 77 aos que consulta por disnea progresiva de medianos
esfuerzos en el ltimo ao, ortopnea e hinchazn de los miembros
inferiores. Entre sus antecedentes no haba factores de riesgo
cardiovascular, no tena hbitos txicos ni reciba ninguna medicacin.
El examen fsico revel una presin arterial de 160/100 mmHg, leve
cianosis perioral e ingurgitacin yugular hasta ngulo mandibular. No
tena bocio ni adenopatas perifricas. Los tonos cardacos eran
rtmicos, sin soplos, y en la auscultacin pulmonar haba crepitantes
bibasales. Tena leves edemas maleolares bilaterales. La exploracin
neurolgica era normal, con negatividad de los signos de Trousseau y
Chvostek. En el hemograma destacaba macrocitosis y en la bioqumica,
un calcio corregido con la albmina de 4,2 mg/dl, fsforo 8 mg/dl,

CURSO ENARM CMN SIGLO XXI TEL: 36246001

magnesio 1,6 mg/dl y la funcin renal normal. La CPK fue de 739 con
CPK-Mb normal, GOT 305 y LDH de 1.038.
PREGUNTA
Cual de las siguientes manifestaciones esperara mas frecuente
encontrar en el caso?
RESPUESTA
a.- QT estrecho.
b.- QT alargado.
c.- T picudas.
d.- T aplanada.
CASO CLINICO HIPERPARATIROIDISMO

Pharmed Solutions Institute

PGINA 43

MANUAL DE TRABAJO DEL CURSO ENARM CMN SIGLO XXI


Varn de 21 aos, con cefalea y vmitos de 24h de evolucin. En la EF
slo se encontr un discreto edema y eritema amigdalar. Leucocitosis
con desviacin izquierda (14.700leucocitos/l con 85% neutrfilos). A
las 2448h del ingreso, el cuadro clnico empeor con la aparicin de
obnubilacin y somnolencia. En la nueva analtica, destacaba una
calcemia de 15,8mg/dl. Se administr fluidoterapia, corticoides y
aporte adicional de potasio y magnesio, evolucionando
favorablemente, con recuperacin del estado de consciencia. El
electrocardiograma se normaliz y la sintomatologa digestiva
desapareci en 24h. PTH 237pg/ml (965), calciuria 290mg/24h (535),
magnesio srico 1,63mg/dl (1,692,29), creatinina 0,46mg/dl (0,53
0,79).
PREGUNTA
Cual es el diagnostico mas probable?
RESPUESTA
a.- Adenoma de paratiroides.
b.- Hiperplasia de paratiroides.
c.- Carcinoma de paratiroides.
d.- Sindrome paraneoplasico.

PREGUNTA
Cul es el diagnostico mas probable para este caso?
RESPUESTA
a.- Mieloma multiple
b.- Hiperparatiroidismo primario
c.- Hipeparatiroidismo secundario
d.- Tumoracion de ovario izquierdo
PREGUNTA
Cul es la causa mas probable de la patologa de esta paciente?

PREGUNTA
En caso de la presencia de sndrome paraneoplasico. Cual es la fuente
mas frecuente?
RESPUESTA
a.- Rion
b.- Pulmon
c.- Cabeza
d.- Cuello
CASO CLINICO HIPERPARATIROIDISMO
Mujer de 38 aos con IRC secundaria a nefropata tubulointersiticial
crnica. A su ingreso destacaban niveles sricos de calcio de 12 mg/dL,
Fsforo 6,6 mg/dL, FA 2.490 UI/L, PTHi 1.450 pg/ml. Destacaba la
presencia de una tumoracin cervical de 3 x 2 x 1 cm situada en la zona
inferior del lbulo derecho de la glndula tiroides con 4 ganglios
linfticos palpables. Radiolgicamente se observaba resorcin
subperistica a nivel de las falanges proximales de ambas manos y una
estructura granular a nivel del crneo. Se ingres por un cuadro de
astenia, debilidad muscular y dolores seos generalizados.
PREGUNTA
Cual es la conducta a seguir mas adecuada para el caso clnico?
RESPUESTA
a.- Hidroxido de aluminio.
b.- Quelantes clcicos.
c.- Vitamina D.
D. Paratiroidectoma

RESPUESTA
a.- Hiperplasia primaria difusa o nodular
b.- Adenoma
c.- Carcinoma
d.- Hipertrofia
PREGUNTA
Cul de las siguientes patologas es menos probable que cause
hipercalcemia?
RESPUESTA
a.- Leucemia
b.- Deshidratacion
c.- Esclerosis multiple
d.- Enfermedad de Addison
PREGUNTA
Qu estudio seria mas adecuado solicitar para conformar
diagnostico?
RESPUESTA
a.- Niveles de fosfatasa alcalina
b.- Niveles de hormona paratiroidea
c.- Calcio en orina
d.- RNM
PREGUNTA
Cul es la indicacin menos probable para someter a tratamiento
quirrgico a esta paciente?

CASO CLINICO
Mujer de 48 aos de edad es llevada a la sala de urgencias con
confusin y dolor en el cuadrante inferior izquierdo del abdomen. Una
hora antes su esposo la encontr buscando por toda la casa un gato de
la familia, que haba muerto hace 4 aos. La paciente es incapaz de
describir el dolor, pero es evidente su inquietud y se sostinene el lado

CURSO ENARM CMN SIGLO XXI TEL: 36246001

izquierdo. El esposo mencina que la paciente recibi tratamiento para


un calculo renal en el mismo hospital nueve meses antes. Desde
entonces ha perdido alrededor de 6.8Kg y regularmente se queja de
fatiga y debilidad muscular. En la exploracin la TA 136/72mmHg, y la
FR 16 rpm. Se encuentra hpersensibilidad con defensa en el cuadrante
inferior izquierdo, ruidos intestinales intensos, asi como
hipersensibilidad en la parte baja de la espalda. La paciente esta alerta,
mas no orientada en cuanto a tiempo y lugar. Laboratorio: Na 152
mg/dl, K 3.2mg/dl, Ca 17.3 mg/dl, Fosfato serico 1.7 mg/dl, Cl 121
mg/dl, Bicarbonato 15.2 mg/dl, pH serico 7.2.

RESPUESTA
a.- Hipercalciuria >400mg/dl
b.- Evidencia radiogrfica de ostetis fibroquistica
c.- Hipofosfatemia grave
d.- Calcio serico >9mg/dl

Pharmed Solutions Institute

PGINA 44

MANUAL DE TRABAJO DEL CURSO ENARM CMN SIGLO XXI


TRASTORNOS DE GLANDULAS SUPRARRENALES Y GONADALES
CIENCIAS BASICAS: Las suprarrenales estn compuestas de medula y corteza est dividida en glomerular (produce aldosterona-regula Na y vol.
Extracelular por medio de la retencin de H2O, Na y la excrecin renal de K), fascicular (produce cortisol-aumenta niveles de glucosa, metabolismo de
protenas y lpidos, efecto inmunosupresor, antiinflamatorio, acta en SNC) y reticular (produce cortisol y esteroides andrognicos:
dihidroepiandrostendiona y androstendiona). En mdula las clulas cromafines secretan adrenalina y adrenalina, cuando la medula produce
catecolaminas en exceso conduce a feocromocitoma. HIPERFUNCION DE GLNDULAS SUPRARRENALES: SINDROME DE CUSHING: La causa ms comn
es la iatrognica, debido a la administracin de glucocorticoides. El Cushing endgeno resulta del exceso de produccin de cortisol (y otras hormonas
esteroideas). La mayor causa de hiperplasia suprarrenal bilateral secundaria es la hipersecrecin de Hormona adrenocorticotropica (ACTH) por la
pituitaria (Enf. de Cushing) o de fuentes ectpicas tales como carcinoma de cel. Pequeas de pulmn, carcinoma medular de tiroides o tumores de timo,
pncreas, ovario. Los adenomas y carcinomas de glndulas suprarrenales representan aproximadamente el 25% de los casos de sndrome de Cushing.
CUADRO CLNICO: Ms comn; obesidad central, HTA, osteoporosis, psicosis, acn, amenorrea y DM, pero inespecficas, ms especficos incluyen
hematomas, estras prpura, miopata proximal, la deposicin de grasa en la cara y las zonas interescapular (facies de luna y joroba de bfalo), y
virilizacin, piel delgada y frgil. La hipopotasemia y alcalosis metablica son prominentes, sobre todo con la produccin ectpica de ACTH.
DIAGNSTICO: Se requiere de demostracin del incremento de cortisol y supresin anormal de cortisol en respuesta a dexametasona. Para el cribado,
medir cortisol libre urinario en 24 hrs. TRATAMIENTO: Para adenoma o carcinoma suprarrenal requiere escisin quirrgica, dar glucocorticoides antes y
despus de la operacin, para regular el estrs. Metstasis y carcinomas irresecables se tratan con mitotano con incrementos graduales de 6g por da
dividido c/8-6h. En ocasiones la citoreduccin de carcinoma de pulmn, o reseccin de otros tumores, podemos remitir el Sx. de Cushing ectpico. Si las
fuentes de ACTH no pueden ser resecadas, hacer adrenalectoma bilateral total administrar ketoconazol (600-1200mg/dl), metirapona (2-3g/d), mitotano
(2-3mg/d), podran aliviar las manifestaciones por exceso de cortisol. HIPERALDOSTERONISMO: Es causada por hipersecrecin de aldosterona adrenal.
PRIMARIO hipersecrecin autnoma de aldosterona (suprarrenal) con supresin de los niveles de renina, puede ser debido a un adenoma suprarrenal
35% (productor de aldosterona=Sx. de Conn) o hiperplasia suprarrenal bilateral 60%, patogenia: aldosterona produce retencin renal de sodio y prdida
de potasio. Esto se traduce en la expansin de contenido de sodio del cuerpo, lo que lleva a la supresin de la sntesis de renina renal. La accin directa
de la aldosterona en la nefrona distal provoca la retencin de sodio y prdida de hidrgeno y los iones de potasio, lo que resulta en una alcalosis
hipocalemia, la aldosterona tiene efectos fisiopatolgicos en un rango de otros tejidos, causando fibrosis cardiaca, disfuncin vascular endotelial y
nefroesclerosis. El SECUNDARIO ocurre secundario a elevacin de los niveles de renina circulante, esto es tpico en falla cardiaca, cirrosis, sndrome
nefrtico, pero podra ser debido a estenosis de la arteria renal o tratamiento diurtico, muy raro reninoma. CUADRO CLINICO: Primario; tienen dolores
de cabeza e hipertensin diastlica. El edema es caractersticamente ausente, a menos que la insuficiencia cardaca congestiva o la enfermedad renal
est presente. Las prdidas de potasio urinario, puede causar debilidad muscular y fatiga, aunque los niveles de potasio pueden ser normales en
aldosteronismo primario suave. Tambin se puede producir alcalosis metablica e hipernatremia. DIAGNOSTICO: Se sugiere por la hipertensin y es
asociado con hipocalemia persistente, en un paciente sin edema, que no este recibiendo diurticos ahorradores de K. TRATAMIENTO: La ciruga puede
ser curativa en adenoma adrenal, pero no efectiva en hiperplasia suprarrenal, que se maneja con restriccin de Na y espironolactona. El secundario; es
tratado con restriccin de sal y correccin de la causa subyacente. HIPOFUNCIN DE GLNDULAS SUPRARRENALES: La insuficiencia suprarrenal primaria
es debida a falla de la glndula suprarrenal, mientras la insuficiencia suprarrenal secundaria es debida a falla de la produccin o liberacin de ACTH.
ENFERMEDAD DE ADDISON: Ocurre cuando ms >90% del tejido suprarrenal est destruido quirrgicamente, por enf. granulomatosa (Tb,
histoplasmosis, coccidioidomicosis, criptococosis), va autoinmune, metstasis bilateral, hemorragia bilateral, CMV,HIV, Amiloidosis, sarcoidosis.
CUADRO CLINICO: Fatiga, debilidad, anorexia, nausea y vmito, prdida de peso, dolor abdominal, pigmentacin cutnea y mucosa, deseo de sal,
hipotensin y ocasionalmente hipoglicemia, si hay deplecin del fluido extracelular se acenta la hipotensin. DIAGNOSTICO: La mejor prueba de
deteccin es la respuesta del cortisol 60 min despus de 250 g de ACTH IV o IM. Los niveles de cortisol deben exceder de 18 mg / dl 30 a 60 minutos
despus de ACTH. Si la respuesta es anormal, la deficiencia primaria y secundaria se puede distinguir mediante la medicin de aldosterona en sangre.
TRATAMIENTO: Hidrocortisona 20-30mg/d, dividido 2/3 en la maana y 1/3 en la tarde es el pilar de la sustitucin de glucocorticoides. La sustitucin de
mineralocorticoides es necesaria en la insuficiencia suprarrenal primaria. Durante la crisis suprarrenal se usan dosis altas de hidrocortisona (10mg/h
continuas IV o 100mg bolo IV tres veces al da). HIPOALDOSTERONISMO: Deficiencia aislada de aldosterona. Con produccin normal de cortisol, se
produce con hiporeninismo, como un defecto biosinttico hereditario, despus de la extirpacin de los adenomas secretores de aldosterona, y durante
el tratamiento prolongado con heparina. Hipoaldosteronismo hiporreninmico es ms frecuente en adultos con insuficiencia renal leve y la diabetes
mellitus en relacin con la hiperpotasemia desproporcionada. Fludrocortisona oral, restablece el equilibrio electroltico si la ingesta de sal es adecuada.
En IRC y falla cardiaca est aprobado el uso de furosemide. MASAS SUPRARRENALES INCIDENTALES (INCIDENTALOMA): Las masas suprarrenales son
comn encontrarlas en TAC o RMI. La mayora (70-80%), son no funcionantes y la probabilidad de un carcinoma suprarrenal es baja (<0.01%). El primer
paso es determinar el estado funcional y medir metanefrinas libres en plasma para deteccin de feocromocitoma. Si hay un tumor maligno conocido, hay
30-50%de posibilidades que el incidentaloma sea una metstasis. Evaluacin hormonal adicional debe incluir la prueba durante la noche dexametasona
1mg supresin de todas los puntos, la renina plasmtica, actividad / aldosterona ratio en pacientes hipertensos, DHEAS en mujeres con signos de exceso
de andrgenos y estradiol en hombres con feminizacin. FEOCROMOCITOMA: Son tumores adrenomedulares que secretan catecolaminas, son raros,
forman <0.1% de las causas de hipertensin, sin embargo es muy importante su diagnstico debido a: el desarrollo de una crisis hipertensiva fatal, la
reversin de todas las manifestaciones despus de remover el tumor quirrgicamente, la falta de eficacia a largo plazo de tratamiento mdico, la
apreciable incidencia de malignidad. Manifestaciones; hipertensin resistente a tx. convencional, sudoracin e intolerancia al calor (80%), palidez o
enrojecimiento, sentimiento de aprensin, pirexia, dolor de cabeza palpitante y constante (65%), parestesias, disturbios visuales, palpitaciones (65%),
dolor de pecho, hipotensin postural. La ciruga es curativa en la mayora de los pacientes. USOS CLINICOS DE GLUCOCORTICOIDES: Son usados en una
variedad de enfermedades tales como asma, artritis reumatoide y psoriasis. Es casi segura la aparicin de complicaciones (ganancia de peso, HTA, fascies
cushinoide, DM, osteoporosis, miopata, incremento de presin intraocular, necrosis sea isqumica, infecciones e hipercolesterolemia), por ello debe
valorarse riesgo-beneficio. Estos efectos secundarios pueden minimizarse mediante la eleccin cuidadosa de preparaciones de esteroides, alternando
das o interrumpiendo terapias; el uso de esteroides tpicos, inhalados, intranasales o drmicos siempre que sea posible. Altas dosis de estos pueden ser
requeridas durante periodos de estrs.
CASO CLINICO SINDROME DE CUSHING
Paciente de 49 aos de edad de sexo masculino con hipertensin
severa, cambios de humor, inestabilidad, alternando entre la
agresividad y la depresin. Se observ debilidad muscular progresiva,
oscurecimiento de la piel y aumento de la grasa abdominal. Sus

CURSO ENARM CMN SIGLO XXI TEL: 36246001

sntomas y pruebas de laboratorio confirmaron un sndrome de


Cushing dependiente de ACTH.
PREGUNTA
Cual es la conducta a seguir para determinar el origen?
RESPUESTA

Pharmed Solutions Institute

PGINA 45

MANUAL DE TRABAJO DEL CURSO ENARM CMN SIGLO XXI


a.- Radiografia de torax.
b.- Tomografia computarizada abdomen.
c.- Resonancia magnetica selar.
d.- USG suprarrenal.

elevada (0.244 nm, el valor normal: 0,014 a 0,083 Nm), y un bajo


niveles de renina plasmtica normal (0.021 pM, el valor normal: desde
0,008 hasta 0,3 pM). TC adrenal mostr un ndulo redondo de 1 cm
en la glndula suprarrenal izquierda (1,1 x 1,0 cm), sugestivo de
adenoma suprarrenal.

CASO CLINICO HIPERALDOSTERONISMO


Femenino de 49 aos de edad, hipertensa con debilidad generalizada
que implica sobre todo sus piernas. Con potasio srico 1.8mEq/L y
creatina fosfoquinasa 1753 U / L. En ese momento, se haba observado
aldosterona (336.73pg/ml ). La TAC abdominal mostr tumores
suprarrenales bilaterales, homogneos, hipodensas, derecha 2,29 cm y
0,92 cm de dimetro. Recibi adrenalectoma laparoscpica derecha y
un tumor de 2x2 cm fue extirpado.

PREGUNTA
Cual es el diagnostico mas probable de este caso?
RESPUESTA
a.- Hiperaldosteronismo primario.
b.- Hiperaldosteronismo secundario.
c.- Pseudoaldosteronismo.
d.- Hiperaldosteronismo terciario.

PREGUNTA
Cual de las siguientes alteraciones ECG es mas probable observar en
este caso?
RESPUESTA
a.- QT corto.
b.- T invertida.
c.- U normal.
d.- P aplanada.

PREGUNTA
La paciente se neg a un procedimiento quirrgico, cual es una medida
teraputica mas adecuada?
RESPUESTA
a.- Indicar amlodipino.
b.- Indicar espironolactona.
c.- Indicar ECA.
d.- Indicar furosemida.

PREGUNTA
Cual de las siguientes alteraciones acido-base es mas probable
observar en esta patologia?
RESPUESTA
a.- Alcalosis hipocalemica.
b.- Alcalosis hipocalcemica.
c.- Acidosis hipocalemica.
d.- Acidosis hipocalcemica.

CASO CLINICO ENFERMEDAD DE ADDISON


Paciente de sexo masculino, de 63 aos de edad, con antecedentes de
tabaquismo activo (40 paquetes/ao), hipertensin arterial crnica y
diabetes mellitus tipo 2 en tratamiento con hipoglicemiantes orales,
cardiopata coronaria y enfermedad arterial oclusiva crnica de
extremidades inferiores. Consult por cuadro de un ao de evolucin
de astenia, adinamia, baja de peso, hiperpigmentacin de piel y
mucosas, episodios de lipotimia e hipoglicemia sintomtica a pesar de
disminucin de la terapia hipoglicemiante y antihipertensiva.

CASO CLINICO
Un hombre de 53 aos de edad con antecedente de cncer de colon
tratado y dos masas en glndulas suprarrenales de 1 y 2 cm
respectivamente, el paciente presenta hipertensin sin control
adecuado tratado con amlodipino y telmisartan. Ingreso por cefalea
leve persistente, con TA 160/90 mmHg, con incremento de los ROTs,
no se observa edema. Se observo potasio serico de 2.6.

PREGUNTA
Cul es la conducta a seguir para establecer el diagnostico?
RESPUESTA
a.- Cuantificacion de cortisol.
b.- Cuantificacion de ACTH.
c.- Cuantificacion de CRF.
d.- Cuantificacion de DEAS.

PREGUNTA
Cual es el diagnostico mas probable del presente caso?
RESPUESTA
a.- Aldosteronismo secundario
b.- Aldosteronismo primario.
c.- Pseudoaldosteronismo.
d.- Pseudo-pseudoalteronismo

CASO CLINICO CRISIS ADRENAL


Hombre de 53 aos de edad, que consulto por nuseas, vmito y
diarrea, en el examen fisico se detecto hipotensin sostenida con
presiones sistlicas en el rango de 60 a 70 mmHg. Por lo cual se inicio
hidratacin intravenosa, sin embargo, el paciente persisti con una
presin arterial de 70/42 mmHg. Los exmenes paraclnicos
documentaron hipoglucemia, hiponatremia e hiperkalemia, por lo cual
se hace la impresin diagnstica de Enfermedad de Addison.

CASO CLINICO HP
Mujer de 29 aos de edad, con hipertensin no controlada e
hipopotasemia espontnea desde hace 4 aos. En la investigacin de
las causas secundarias de la hipertensin, la relacin de actividad de la
renina-aldosterona se elev en dos ocasiones separadas,
diagnosticando como hiperaldosteronismo primario.
PREGUNTA
Cual de las siguientes pruebas confirman el diagnostico?
RESPUESTA
a.- Infusion de solucin glucosada.
b.- Infusin de solucin salina.
c.- Infusion de solucin mixta.
d.- Infusion de soluciones hipertnicas.
CASO CLINICO
Una mujer de 68 aos de edad con nivel de calcio srico de (2,95 mM,
el valor normal: 2,25 a 2,75 mM) con un bajo nivel de potasio (1,7 mM,
el valor normal: 3.5 a 5.3 mM), un nivel de aldosterona plasmtica

CURSO ENARM CMN SIGLO XXI TEL: 36246001

PREGUNTA
Cual es la conducta a seguir mas apropiada en el caso?
RESPUESTA
a.- Soluciones meatabolicas y corticoides.
b.- Soluciones normotonica y corticoides.
c.- Soluciones fisiolgicas, catecolaminas y glucocorticoides.
d.- Soluciones metabolicas, catecolaminas y glucocorticoides.
CASO CLINICO FEOCROMOCITOMA
Paciente masculino de 62 aos con antecedentes de ser fumador, que
dos meses atrs haba comenzado con prdida del apetito y fiebre por
las tardes de 38 C, Mucosas hmedas hipercoloreadas, tonos
cardiacos rtmicos. FC 79 x. TA 160/100. Ultrasonido abdominal
imagen compleja ecolcida a predominio de cara anterior y porcin
superior del rin derecho de 76 mm. El trax ofrece datos de inters
como la presencia de una radioopacidad no homognea de contornos
irregulares, mas acentuad a la base pulmonar derecha, fina banda
radio opaca, que parte desde la regin perifrica hacia el hilio. 5 das

Pharmed Solutions Institute

PGINA 46

MANUAL DE TRABAJO DEL CURSO ENARM CMN SIGLO XXI


despus del ingreso, el paciente comienza a presentar cifras
tensionales que en algunas ocasiones eran muy altas y otras bajas; en
esa oportunidad se aprecia la presencia de lesiones dermatolgicas
papulares rojizas, comienza con lesiones purpuro-hemorrgicas y se
auscultan crepitantes bibasales. Tensin arterial (TA) 160/100
Frecuencia cardiaca (FC): 110 latidos/minuto.
PREGUNTA
Cual es el procentaje diagnostico de feocromocitomas de forma
directa?
RESPUESTA
a.- 5 %
b.- 10 %
c.- 15 %
d.- 20 %
CASO CLINICO
Femenino de 41 aos. Acude a consulta, por cefalea, se detecta una
hipertensin arterial severa: 220/120 mm/Hg. Electrlitos plasmticos
revelan sodio 145 mEq/L e hipokalemia: 3,3 mEq/L: la creatininemia
fue normal: 0,77 mg/dl, al igual que el examen de fondo de ojo. Una
ecotomografa abdominal no mostr alteraciones. Se hospitaliz
corroborndose valores de electrlitos plasmticos: Na 143 y K 2,4
mEq/L: La medicin de actividad de renina plasmtica (PRA) confirm
su supresin: 0,4 ng/ml/h y los niveles de aldosterona en suero
resultaron elevados: decbito 26,4 ng/dl (1-16), de pie 44,4 ng/dl (431). Con diagnstico bioqumico de hiperaldosteronismo primario, se
practic tomografa computada, que revel un ndulo suprarrenal
derecho hipodenso de 2,8x1,9 cm.
PREGUNTA
Cul es la conducta teraputica medica ms adecuada en este caso?
RESPUESTA
a.- Amilorida
b.- Espironolactona
c.- Cortisol
d.- Fludrocortisona
PREGUNTA
Cul es la conducta teraputica a seguir en este caso?
RESPUESTA
a.- Manejo con esteroides a bajas dosis
b.- Cortisol de por vida
c.- Uso de diurticos
d.- Extirpacin de glndula suprarrenal derecha
PREGUNTA
Cul es la funcin ms probable de la zona fascicular de la glndula
suprarrenal?
RESPUESTA
a.- Produccin de glucocorticoides, principalmente cortisol
b.- Produccin de mineralocorticoides, principalmente aldosterona
c.- Produccin de andrgenos, incluyendo testosterona
d.- Catecolaminas; adrenalina y noradrenalina
PREGUNTA
Cul es la causa menos probable que nos haga sospechar
hipertensin de origen renovascular?
RESPUESTA
a.- Hipertensin arterial sistolica ms diabetes mellitus
b.- Hipertensin moderada a severa, refractaria a tratamiento mdico
agresivo
c.- Hipertensin de inicio antes de los 20 aos o despus de los 80 aos
d.- Hipertensin moderada a severa, con insuficiencia renal progresiva

CURSO ENARM CMN SIGLO XXI TEL: 36246001

CASO CLINICO
Un paciente varn de 32 aos acude al Servicio de Urgencias, por un
cuadro clnico de astenia, aumento de fatiga con el ejerccio y anorexia
con prdida de peso de 8 Kg en los ltimos 4 meses. Administrativo de
una empresa de informtica, exfumador de cajetilla y media al da
desde hace 2 aos, no presenta alergias medicamentosas, no bebedor
ni otros hbitos txicos. Sin historia familiar de enfermedades
autoinmunes ni otros antecedentes de inters. Refiere episodios de
dolor abdominal difuso durante los citados meses, ms intenso en los
ltimos das. A los sntomas iniciales se agregaron nuseas y vmitos
intensos. Afebril, consciente y orientado, presenta en la exploracin
fsica una tensin arterial de 105/55 mmHg, taquicardia (115 lpm),
eupneico, discreta deshidratacin de piel y mucosas, abdomen blando
y depresible, doloroso de forma difusa, sin defensa a la palpacin ni
masas ni megalias. Destaca una hiperpigmentacin cutnea difusa de
color bronceado oscuro en codos, pliegues palmares y areolas
mamarias, y manchas azuladas negruzcas en paladar y mucosa gingival.
PREGUNTA
Cul es el diagnstico ms probable para este caso?
RESPUESTA
a.- Hiperaldosteronismo primario
b.- Sndrome de Addison
c.- Sndrome de Waterhouse-Friderichsen
d.- Hiperplasia suprarrenal
PREGUNTA
Qu anticuerpos solicitaramos al paciente por estar ms relacionados
con esta patologa?
RESPUESTA
a.- ANCAs
b.- ANA
c.- Anti-CCP
d.- Anti-DNA
PREGUNTA
Cul es la conducta teraputica ms adecuada en este caso?
RESPUESTA
a.- 0.5mg de betametasona
b.- 20mg de hidrocortisona
c.- 20mg de Fenoxibenzamina
d.- 1g de Metirapona
CASO CLINICO
Mujer de 40 aos de edad acude con el medico familiar para su
revisin de seguimiento. Dos meses antes durante una visita por una
infeccion viral de las vas respiratorias superiores, la TA fue de
140/90mmHg, por lo cual el medico le pidi que fuera al siguiente dia
para checar nuevamente su presin arterial. Hoy la presin arterial es
de 190/100mmHg, FC 80lpm, fr 14 RPM, La exploracin fsica no revela
hallazgos considerables. Laboratorio: Glucosa 100mg/dl, Na 160mg/dl,
K 2.5 mg/dl, pH serico 7.55, osmolalidad srica 275mosm(kg,
osmolalidad urinaria 530mosm/kg.
PREGUNTA
Cul es el diagnotico mas probable para esta paciente?
RESPUESTA
a.- Hipertension esencial
b.- Sindrome de Conn
c.- Hipoaldosteronismo
d.- Diabetes insipida

Pharmed Solutions Institute

PGINA 47

MANUAL DE TRABAJO DEL CURSO ENARM CMN SIGLO XXI


SOBREPESO Y OBESIDAD
CIENCIAS BASICAS:
La obesidad, incluyendo al sobrepeso como un estado premrbido, es una enfermedad crnica caracterizada por el
almacenamiento en exceso de tejido adiposo en el organismo, acompaada de alteraciones metablicas, que predisponen a la presentacin de
trastornos que deterioran el estado de salud, asociada en la mayora de los casos a patologa endcrina, cardiovascular y ortopdica principalmente y
relacionada a factores biolgicos, socioculturales y psicolgicos. SALUD PBLICA: Obesidad padecimiento costoso (25% mayor que una persona con peso
normal). Mxico ocupa 1er lugar en sobrepeso y obesidad (30% de la poblacin), 2do EU, en Mxico 31% de la poblacin infantil sufre de sobrepeso y
obesidad. CLASIFICACIN: En cuadro anexo. EVALUACIN: En la evaluacin se debe incluir el IMC (ndice de masa corporal), medicin de la
circunferencia de la cintura. Para IMC= peso en Kg/ talla2 mts (kg/m2), esta proporciona una medida precisa del contenido de grasa corporal, en adultos
sin importar gnero. El exceso de grasa abdominal, es un factor de riesgo por si solo, la medicin de la circunferencia abdominal (a nivel de crestas
iliacas, cinta ajustada sin comprimir piel, con una espiracin normal), es de particular importancia en pacientes categorizados como "normales" o con
"sobrepeso", pero no es necesario en pacientes con IMC >35kg/m2. Los hombres que presentan circunferencia abdominal >102 cm y mujeres con >88
cm, tendrn un mayor riesgo de desarrollar diabetes, dislipidemia, hipertensin y enf. cardiovasculares, enf. de vesicula biliar; estos pacientes deben ser
colocados en una categoras de riesgo superior, que aquella que les corresponde por IMC. La incidencia de cncer endometrial, de mama, prstata,
colorectal, esteatohepatitis, osteoartritis y gota en hombres y mujeres esta incrementado por la obesidad. PATOGENIA: Puede ser resultado de aumento
del aporte calrico o disminucin gasto energtico o la combinacin de ambos. La susceptibilidad a la obesidad es de naturaleza polignica, 30-50% de la
variabilidad de reservas de grasa puede ser determinada genticamente, este es el principal factor. Las causas secundarias de obesidad incluyen: enf.
Hipotalmica, hipotiroidismo, Sx. de Cushing e hipogonadismo. El aumento de peso tambin es inducido por frmacos, comn en aquellos que usan
antidiabticos, glucocorticoides, agentes psicotrpicos, estabilizadores del humor (litio), antidepresivos, antiepilpticos. Existen factores que al
combinarse con el sobrepeso y la obesidad, colocan al paciente en un altsimo riesgo de morir prematuramente, entre ellos: infartos, angina de pecho,
ciruga de arterias coronarias, ateroesclerosis, DM tipo II (esta coloca al paciente en riesgo absoluto muy alto). Tres o mas de los siguientes factores de
riesgo definitivamente aaden un riesgo absoluto: HTA, tabaquismo, elevacin del colesterol LDL, glucosa en ayuno alterada, antecedente de enf.
Cardiovascular prematura y edad (hombres >45 y mujeres >55). TRATAMIENTO:
En >25kg/m2 se les sugieren cambios en estilo de
IMC (Kg/m2) CATEGORIA OMS
CATEGORIA SEEDO
Riesgo de enfermedad
vida (dieta, comidas pequeas, frecuentes,
<18.5
Infrapeso
Peso insuficiente
desayuno, ejercicio. La prdida de peso se
18.5-24.9
Normopeso
Normopeso
recomienda en pacientes con IMC >30, en quienes
25.26.9
Sobrepeso
Sobrepeso grado I
tienen IMC 25-29.9 con factores de riesgo o
27-29.9
Sobrepeso grado II (preobesidad) Incrementado
aquellos que se encuentren en la categora "alto
30-34.9
Obesidad moderada Obesidad tipo I
Alto
riesgo" con base en la circunferencia abdominal.
35-39.9
Obesidad severa
Obesidad tipo II
Muy alto
El primer objetivo es una reduccin de 10% de la
40-49.9
Obesidad morbida
Obesidad tipo III (mrbida)
Extremadamente alto
masa corporal en un periodo de 6 meses (a una
>50
Superobesidad
Obesidad IV (extrema)
Extremadamente alto
velocidad de 0.5-1kg por sem), mediante una
reduccin calrica de 500-1000kcal/da, posteriormente enfocarse en mantenimiento de peso con la combinacin de dieta, actividad fsica y cambios en
la conducta. Esta fase de mantenimiento se define como una ganancia de peso no mayor a 3kg durante 2aos y una reduccin sostenida de la
circunferencia abdominal de por lo menos 4 cm. Se usan cambios en el estilo de vida y adicionar farmacoterapia: sibutramina (inhibidor de la recaptura
de norepinefrina y serotonina), produce prdida de peso de 5-9% en 12 meses, aunque produce alteraciones en la frecuencia cardiaca y presin arterial,
COFEPRIS pidi su retiro e 2010; orlistat (inhibidor de la lipasa intestinal= inhibe la absorcin de grasas), produce prdida de peso de 9-10%, en 12 meses
con cambios en el estilo de vida; metformina tiende a la disminucin del peso corporal. Si logramos la primer meta, nos enfocamos en la tercera fase que
es prdida de peso adicional. En pacientes que no es posible perder peso, la meta es prevenir mayor ganancia de peso. El tratamiento quirrgico: Estar
indicado segn la NOM exclusivamente en los individuos adultos con obesidad severa e ndice de masa corporal >40, o >35 asociado a comorbilidad
importante y cuyo origen en ambos casos no sea puramente de tipo endcrino. Deber existir el antecedente de tratamiento mdico integral reciente,
por ms de 18 meses sin xito; salvo ocasiones cuyo riesgo de muerte, justifique el no haber tenido tratamiento previo. Deber ser resultado de la
decisin de un equipo de salud multidisciplinario. Las cirugas para bajar de peso son A) Restrictivas (limita la cantidad de comida que el estmago puede
contener y frenar el vaciamiento gstrico), prdida significativa y sostenida por mas de 5 aos; aqu encontramos, la colocacin laparoscpica de la
banda gstrica ajustable, gastroplastia de banda vertical, , gastrectoma vertical en banda (qx. De emergencia). B) Cirugas que limitan la ingesta de
alimentos y alteran la digestin. La mas famosa es el puente gstrico tipo Y de Roux. PRONSTICO: Personas con obesidad mrbida viven entre 8-10
aos menos que quienes tienen un peso normal. Por cada 15kg arriba del peso ideal se incrementa el riesgo de muerte temprana en 30%. Incrementan
riesgo de morbilidad respecto a HTA, dislipidemia, DM tipo 2, EVC, apnea del sueo, enf. arterial coronara.
CASO CLINICO OBESIDAD
Varn obeso de 59 aos de edad con hemipleja izquierda. A los 54
aos de edad presento disnea intensa e hipertensin arterial
descontrolada. Recibi el diagnstico de hipertensin arterial a los 44
aos de edad. El examen fsico revel peso de 163,8kg, altura de 1,74
m, ndice de masa corprea 54,1 kg/m2, pulso de 84 lpm, presin
arterial de 200/110 mmHg. El shock de punta del corazn fue palpado
en el 6 espacio intercostal, hacia fuera de la lnea hemiclavicular
izquierda, y la ausculta no revel ruidos accesorios. Haba soplo
sistlico en rea mitral y borde esternal izquierdo. El abdomen estaba
voluminoso sin visceromegalias. Haba edema discreto de miembros
inferiores y los pulsos en estos miembros estaban disminuidos.
PREGUNTA
Cual es la conducta a seguir mas apropiadas?
RESPUESTA
a.- Cirugia bariatrica.
b.- Colocacin de banda gstrica.
c.- Orlistat mas sibutramina.

CURSO ENARM CMN SIGLO XXI TEL: 36246001

d.- Dieta estricta, orlistad y metformida.


CASO CLINICO OBESIDAD
Mujer de 39 aos con hipertensin arterial, en tratamiento con
candesartn, y obesidad grado III que comenz a tratar con
sibutramina 12 das antes. Desde el inicio de la toma de medicacin
refera cifras ms elevadas de presin arterial y palpitaciones. El da del
ingreso present en reposo dolor intenso retroesternal irradiado a la
extremidad superior izquierda y sudoracin de unos 15 min de
duracin, que cedi tras nitroglicerina sublingual, y lleg a urgencias
asintomtica. Los mximos sricos de creatincinasa y troponina T
fueron 388 UI/l (normal hasta 140) y 0,23 ng/ml (normal hasta 0,035),
respectivamente, con curva enzimtica tpica de infarto agudo de
miocardio. El electrocardiograma realizado sin dolor torcico fue
normal durante todo el ingreso. El ecocardiograma no mostr
alteraciones de la contractilidad. La coronariografa mostr coronarias
normales
PREGUNTA

Pharmed Solutions Institute

PGINA 48

MANUAL DE TRABAJO DEL CURSO ENARM CMN SIGLO XXI


Se programar para ciruga bariatrica ms liposuccin, considerando la
comorbilidad, cual es la complicacin aguda mas frecuente.
RESPUESTA
a.- Embolia grasa.
b.- Embolia pulmonar.
c.- Insuficiencia cardiaca.
d.- Sindrome de absorcin.
CASO CLINICO OBESIDAD MORBIDA
Se trata de una mujer de 50 aos de edad con obesidad mrbida (peso
105kg, talla 155cm e IMC de 43) y con antecedentes de trombosis
venosa profunda de repeticin. Ingres en Dermatologa por lesiones
cutneas ampollosas generalizadas con biopsia cutnea compatible
con eritema polimorfo que se atribuy al tratamiento anticoagulante.
Tras valoracin por el servicio de alergologa, se aconsej su retirada y
su no reintroduccin. Se indic tratamiento con heparina de bajo peso
molecular, desarrollando importantes hematomas dolorosos en el sitio
de la administracin, motivo por el que se suspendi.
PREGUNTA
Cual es la medida teraputica que presenta menos complicaciones
por las caractersticas del caso?
RESPUESTA
a.- Cirugia bariatrica.
b.- Colocacin de banda gstrica.
c.- Dieta y ejercicio.
d.- Orlistat.
CASO CLINICO OBESIDAD MORBIDA
Paciente varn de 60 aos, con sndrome de Alport, microhematuria y
proteinuria de 4gr/da, creatinina de 2 mg/dl, urea 123mg/dl, perdida
moderada de la audicin, hipertensin arterial tratada con enalapril 20
mg, obesidad con IMC=36,26 (P 120Kg, h 1,81mts), glucemias en
ayunas alteradas (menor a 126mg/dl)Hb glicosilada normal,
dislipidemia e hiperuricemia en tratamiento. Se indica dieta bajas en
protenas, hipocalrica y se agrega al tratamiento losartan en dosis de
75 mg/ da. Se realiza ciruga de by pass gstrico presentado en su
evolucin disminucin de 35 kg en total llegando a IMC de 25,6.
Control nefrolgico: creatinina 1,56mg/dl, proteinuria 0,3 g/dia, Urea
65mg/dl, normotension con enalapril 5 mg/d, se mantiene dosis
mnima de hipolipemiantes, normouricemia y normoglucemias.
PREGUNTA
Cual es la complicacin ms importante que presenta a largo plazo el
paciente?
RESPUESTA
a.- Insuficiencia renal.
b.- Hipotiriodismo.
c.- Sindrome de mala absorcin
d.- Anemia perniciosa

buena salud, aunque reconoce estra en peor forma fsica que hace 5
aos. Atibuye esto a un cambio de carrera que lo tienen trabajando en
un escritorio en lugar de sobre los pies todo el dia. Ha intentado
repetidas veces empezar un nuevo rgimen de ejercicio durante el ao
pasado, pero en general solo se las arregla para hacer ejecicio tres
veces cada mes. Come tres veces al dia, pero no opculta su gusto por la
comida chatarra. Bebe alcohol a diario; toma una o dos cervezas en la
comida. Fum una cajetilla de cigarrillos el dia durante 20 aos antes
de abndonar el hbito el ao pasado. La presin arterial es de
1260/90mmHg, el pulso de 90 por minuto y la FR 16 rpm. S u estatura
es de 160cm y pesa 90.7Kg. Su ultima revisin completa fue hace casi 3
aos, de modo que su medico recomienda un panel completo de
anlisis de sangres. Los resultados importantes son: Glucosa 145mg/dl,
Colesterol 208mg/dl, TG 410mg/dl, HDL 26mg/dl, LDL 173mg/dl, pH
serico 7.42.
PREGUNTA
Cul es el diagnostico mas probable para este paciente?
RESPUESTA
a.- Diabetes mellitus tipo 2
b.- Obesidad grado II
c.- Sindrome metablico
d.- Dislipidemia
PREGUNTA
Cul es la conducta teraputica menos apropiada para el control de la
presin arterial de este paciente?
RESPUESTA
a.- Calcioantaginistas
b.- Betabloqueadores
c.- Tiazidas
d.- IECAS
PREGUNTA
Cul es el IMC del paciente actualmente?
RESPUESTA
a.- 32,3Kg/m2
b.- 30,0 Kg/m2
c.- 34,2 Kg/m2
d.- 36,1mg/m2
PREGUNTA
Cul es la complicacin menos probable que puediera presentar este
paciente?
RESPUESTA
a.- Diabetes mellitus tipo 2 verdadera
b.- Apoplejia
c.- Infarto de miocardio
d.- Hepatopatia

CASO CLINICO
Hombre de 49 aos de edad acude con al edico de atencin primaria
para revbision fsica relacionada con el trabajo. En general muestra

CURSO ENARM CMN SIGLO XXI TEL: 36246001

Pharmed Solutions Institute

PGINA 49

MANUAL DE TRABAJO DEL CURSO ENARM CMN SIGLO XXI


DISLIPIDEMIAS
CIENCIAS BASICAS: Definicin: son un grupo de trastornos caracterizados por la presencia de concentraciones anmalas de lpidos (LDL, HDL,
triglicridos) en sangre. Son importantes por su contribucin en la gnesis de ateroesclerosis, por lo que se relacionan directamente con la enf.
cardiovascular. SALUD PUBLICA: Prevalencia en adultos mexicanos es de 30% para hipercolesterolemia, 40% hipertrigliceridemia y 50% para
hipoalfalipoproteinemia, siendo la combinacin de esta ltima mas hipertrigliceridemia, lo ms frecuente en el paciente diabtico. CLASIFICACION:
Primarias: Secundarias a causa de obesidad, diabetes mellitus, consumo de alcohol, anticonceptivos orales, glucocorticoides, falla renal, dao heptico e
hipotiroidismo, o empeorar las subyacentes.
CARACTERISTICAS DE LAS PRINCIPALES DISLIPIDEMIAS PRIMARIAS
HIPERCOLESTEROLEMIA AISLADA
HIPERTRIGLICERIDEMIA AISLADA
Hipercolesterolemi
LDL
colesterol
elevados;
Hipertrigliceridemia
VLDL elevadas, TG 250-750, leve
a familiar
colesterol total de 275-500mg/dl
familiar
incremento de colesterol <250mg/dl
Autosmica dominante
Autosmica
dominante.
Obesidad,
Puede ser a consecuencia de
hiperglicemia e hiperinsulinemia son
mutacin para el receptor LDL
caracteristicos
Xantomas tendinosos en edad
Asintomtico, xantomas eruptivos, puede
adulta
y
xantelasmas,
estar relacionado con incremento del
enfermedad vascular
riesgo
de
enfermedad
vascular.
Complicaciones:
cardiopata
Complicaciones:
Pancreatitis,
sx.
isqumica
metablico
Hipercolesterolemi
LDL elevada, colesterol total
Deficiencia
de
TG = >750
a polignica
<350mg/dl, defectos genticos y
lipoprotein
lipasa
Asintomtica recesiva, que a su vez
factores a,mbientales, dieta,
familiar
deteriora el metabolismo de los
edad, ejercicio.
quilomicrones, la acumulacin de estos
Asintomtica,
desarrollan
uede ser asociada a pancreatitis, dolor
enfermedad
vascular,
no
abdominal, hapatoesplenomegalia
xantomas

COMBINADA
Disbetalipoproteinemia
TG 250-500; VLDL elvado
Alteraciones en la Apo E
Autosmico recesivo, puede
haber Xantomas tuberosos o
palmares
Complicaciones:
cardiopata
isqumica y sx. metablico

DIAGNSTICO: Cuadro clnico: Mas asintomticas, datos que pueden orientar hacia el origen, xantomas tuberosos (ndulos, en codos y rodillas;
disbetalipoproteinemia, hipercolesterolemia familiar), xantomas tendinosos (en tendn calcneo y extensores de manos; LDL >300mg/dl), xantomas
eruptivos (borde eritematoso y centro blanquecino, confluentes; TG >1000mg/dl), el xantoma estra palmaris o depsito de lpidos en pliegues de las
manos (disbetalipoproteinemia), el xantelasma es una ppula o placa amarillenta en prpados de forma bilateral. Escrutinios a partir de 20 aos y
repetirse cada 5 aos, dx., se basa en lpidos sricos, con ayuno de 8-10hrs.
NIVELES NORMALES DE LIPIDOS
LDL <100 optimo; 100-129 casi
optimo; 130-159 limitrofe alto; 160189 alto; >190 muy alto

Colesterol total <200 normal; 200239 limitrofe alto; >240 alto

HDL
<40
(hipoalfalipoproteinemia);
normal; >60 alto

bajo
40-59

TG <150 normal; 150-199 limitrofe


alto; 200-499 alto; >500 muy alto

TRATAMIENTO: Determinar nmero de factores de riesgo cardiovascular (hombre >45, mujer >55 aos, historia en familiares de primer grado de
cardiopata isqumica a edad temprana hombre <55, mujer <65 aos, tabaquismo, HTA, HDL <40). El orden en que se deben perseguir las metas es:
primero colesterol LDL, luego triglicridos y al ltimo colesterol HDL. Es importante sealar que cuando hay TG >500mg/dl, la meta principal son estos,
debido al riesgo de desarrollar pancreatitis. El tx., no farmacolgico l( dieta, ejercicio, reduccin de peso), logra disminuir hasta 10% los niveles de
colesterol, cuando no se logra control iniciar tx. Farmacolgico; en general las estatinas son el frmaco de eleccin para la hipercolesterolemia, mientras
que los fibratos para hipertrigliceridemia. Hipoalfalipoproteinemia usar niacina y fibratos. hipercolesterolemia aislada, estatinas, colestiramina,
colestipol, niacina. Estatinas; Inhibidores de la hidroximetil glutaril coenzima A reductasa (prava, atorva, rosuva, sinvastatinas), tienen efecto
hipolipemiante, antiinflamatorio, mejoran funcin endotelial y disminuye la agregacin plaquetaria, sus efectos adversos mas importantes, miositis,
rabdomiolisis (cuando se usan con fibratos se prefiere gemfibrozil, para disminuir este riesgo), hepatotoxicidad, elevacin de CPK por arriba de 10 veces
el limite superior normal es una indicacin de suspensin de estatinas. Colestiramina y colestipol; quelantes de ac. biliar, disminuye LDL y aumenta HDL,
no usar en hipertrigliceridemia. Fibratos; reducen sntesis de VLDL y LPL, por el hgado (gemfibrozil, fenofibrato, clofibrato), efectos adversos nausea,
exantema, miopatia. Niacina; disminuye sntesis heptica de lipoproteina con Apo B es el mejor frmaco para elevar HDL, efectos adversos bochorno,
nusea, prediabetes, hiperuricemia.
CASO CLINICO DISLIPIDEMIA
Varon de 78 aos de edad con IRC secundaria a hipertension arterial y
diabetes mellitus (creatinina serica basal 1,5 mg/dl), exfumador con
hipercolesterolemia. Se diagnostico de cardiopatia isquemica con
enfermedad grave de un vaso, llevandose a cabo una angioplastia y
stent. Un mes despus de este procedimiento, el paciente acudio a
Urgencias por hematuria franca, detectando una elevacion de la
creatinina serica a 4,5 mg/dl, motivo por el que ingreso. En la
exploracion fisica se detecto una arritmia en la auscultacion cardiaca y
una cianosis en primer, tercer y quinto dedo del pie derecho. El EGO
mostraba sangre +++ y la cuantificacion de proteinas de 24 horas fue
de 1,2 g/dia egresa con colesterol 350, triglicridos 780, HDL 28.
PREGUNTA
Cul es la conducta teraputica a seguir al egreso ms adecuada?
RESPUESTA
a.- Bezafibrato y atorvastatina.
b.- Pravastatina, bezafibrato, metformida.
c.- Dieta, ejercicio, bezafibrato y pravastatina.
d.- Dieta, rehabilitacin, bezafibrato y pravastatina.

CURSO ENARM CMN SIGLO XXI TEL: 36246001

PREGUNTA
El paciente regresa a urgencias 4 semanas despus, somnoliento,
urmico, con CPK de 1870, hematura macroscpica, mialgias
generalizadas, considerando el cuadro, cual es su diagnostico actual?
RESPUESTA
a.- Insuficiencia Renal Aguda.
b.- Sindrome Hepatorrenal.
c.- Rabdomiolisis.
d.- Glomerulonefritis.
CASO CLINICO HIPERCOLESTEROLEMIA
Paciente de 69 aos de edad con antecedentes de HTA de carcter
moderada desde hace 8 aos aproximadamente, con enalapril 10 mg e
hidroclorotiazida 12,5 mg una vez por da. Consulta por cefalea a
predominio frontal, la cual se manifiesta desde hace varios das
acompaado de sensacin de inestabilidad, mareo que no sabe referir
exactamente pero si manifiesta sentirse muy extrao, con alteraciones
de la sensacin de la cara como hormigueo y falta de fuerza. Refiere
presentar en una ocasin perdida de la visin de ojo izquierdo durante
menos de 1 minuto.

Pharmed Solutions Institute

PGINA 50

MANUAL DE TRABAJO DEL CURSO ENARM CMN SIGLO XXI


PREGUNTA
Cual es su conducta a seguir en este momento para el caso actual?
RESPUESTA
a.- IRM crneo contrastada.
b.- Perfil lipidico.
c.- Analitica de electrolitos sericos.
d.- Iniciar antilipemiantes y oxigenadores.
CASO CLINICO DISLIPIDEMIA
Femenino de 58 aos, antecedentes de diabetes tipo II, con
diagnstico de dislipidemia. No presenta enfermedad vascular previa.
En tratamiento con metformina 850 mg/da, AAS 100 mg/da y
atorvastatina 10 mg/da, benzafibrato 200mg/dia, acude por dolor
muscular en miembros pelvicos. Padre y to diabtico, hermano con
dislipemia. EF: PA: 135/90, IMC: 29, Permetro cintura: 100 cm, Glu:
198, Col: 410, Tg: 560, HDL: 28, HbAic: 8.7, Io: 143/4.3, Cr: 2.1, CPK:
320, Microalb: 30.
PREGUNTA
Cual de los siguientes elementos son ms importantes para
considerar rabdomiolisis?
RESPUESTA
a.- Estatinas y fibratos.
b.- Dolor muscular.
c.- Elevacion de CPK.
d.- Valores de creatinina.
PREGUNTA
Cual es el objetivo principal en las dislipidemias?
RESPUESTA
a.- LDL
b.- Trigliceridos
c.- HDL
d.- Glucosa
PREGUTA
Considerando la respuesta anterior cual es la complicacin mas
importante para establecer el objetivo?
RESPUESTA
a.- Pancreatitis.
b.- Estado hiperosmolar.
c.- Infarto al miocardio.
d.- Enfermedad vascular.
CASO CLINICO DISLIPIDEMIA
Pte de 51 aos, diabtico tipo II con glibenclamida 5mg/d +
metformina 1700 mg/d y enalapril 20 mg/d. FRP: Sedentarismo y
Tabaquismo (15 cig/d), Antecedentes: Madre y hermano hipertensos,
EF: PA: 150/90 mmhg, Peso: 56 kg, Talla: 1.75 mts, Perimtro Cint.: 103
cm, Rx Tx: Cardiomegalia c/aorta elongada y calcificacin botn
artico. Hto: 44, Urea 31. Cr: 0.8. Glu: 118, HbA1c: 7.4; Col: 207, LDL:
433, TG: 408, HDL: 22; Ac.Urico: 7.8. GOT: 52; GPT: 38; Microalb: 40
mg/24hs.
PREGUNTA
Cual es el objetivo principal en la dislipidemia del caso?
RESPUESTA
a.- LDL
b.- Trigliceridos

CURSO ENARM CMN SIGLO XXI TEL: 36246001

c.- HDL
d.- Glucosa
PREGUNTA
Cual es medicamento de eleccin para incrementar HDL en el
presente caso?
RESPUESTA
a.- Niacina.
b.- Nicotinamina.
c.- Riboflavina.
d.- Hidroxicobalamina
CASO CLINICO
Varon de 40 aos de edad con antecedente de hipertensin arterial,
acude a clnica para su revisin mensual. Niega molestias recientes y su
interrogatorio y su exploracin fsica no revelan hallazgos obvios salvo
por presin arterial de 150/90mmHg. En la actualidad toma
hidroclorotiazida para el control de su hipertensin. Un perfil de
lpidos sistematico obtenido en ayunas revela colesterol serico toal de
250mg/dl, colesterol de lipoprotenas de baja densidad (LDL 200mg/dl)
y colesterol lipoprotenas de alta densidad (HDL 50mg/dl). No tiene
antecedentes de arteriopatia coronaria y no fuma.
PREGUNTA
Cul de las siguientes es la causa menos probable para esta
patologa?
RESPUESTA
a.- Alcoholismo
b.- Diabetes mellitus
c.- Enfermedad heptica
d.- Insuficiencia cardiaca
PREGUNTA
Cul es la conducta mas adecuada a seguir en este paciente?
RESPUESTA
a.- Cambios en el estilo de vida y estatinas
b.- Fibratos y cambios en el estilo de vida
c.- Fibratos mas estatinas
d.- Solo cambios en el estilo de vida

PREGUNTA
Cul es el factor de riesgo que menos influye en que el paciente
desarrolle arteriopatia coronaria prematura?
RESPUESTA
a.- Tabaquismo
b.- HDL <40mg/dl
c.- Hipertension arterial
d.- La edad del paciente
PREGUNTA
Despues del control del paciente. Cada cuando debemos hacer el
seguimiento de lpidos del paciente?
RESPUESTA
a.- Cada ao
b.- Cada 3 aos
c.- Cada 5 aos
d.- Cada 2 aos

Pharmed Solutions Institute

PGINA 51

MANUAL DE TRABAJO DEL CURSO ENARM CMN SIGLO XXI


DIABETES MELLITUS Y SUS COMPLICACIONES MICROANGIOPATICAS
CIENCIAS BASICAS: Definicin: Enfermedad sistmica cronico-degenerativa, de caracter heterogneo con grados variables de predisposicin hereditaria
y con participacin de diversos factores ambientales y que se caracteriza por hiperglucemia crnica debido a la deficiencia en la produccion o accion de
la insulina lo que afecta al metabolismo intermedio de los hidratos de carbono, protenas y grasas (def. NOM). Factores de riesgo: Edad, obesidad si esta
es central genera mas resistencia a la insulina, resistencia a la insulina (la acantosis nigricans), sexo (M60%; H40%), sedentarismo, dieta hipercalorica.
SALUD PUBLICA: 90-95% DM tipo 2, 5-10% DM tipo 1. Mortalidad 70 de cada 100,000. 7 de cada 10 diabticos, muere antes de cumplir la edad
promedio. Es la 7ma causa de muerte, principal causa de insuficiencia renal, causa de ceguera en personas de 20-74 aos. PATOGENIA: Convinacion
entre resistencia a la union a la insulina y una inadecuada respuesta secretora, que inicialmente las celulas b del pncreas incrementa. Crculo vicioso de
hiperglucemia (pospandrial), hiperinsulinismo, regulacion a la baja de receptores de insulina (perifricos). Otras causas de DM tipo 2 son pancreatitis,
trauma, infeccion, carcinoma y endocrinopatas entre otras. DIAGNSTICO: Clinico; poliuria, polidipsia, polifagia, perdida de peso, visin borrosa,
susceptibilidad algunas infecciones, fatiga, dolor abdominal, nauseas, vomitos. Conceptos: Glucosa anormal en ayuno= >100 y <125mg/dl, hiperglucemia
pospandrial= >140mg/dl, dos horas despus de la comida. Criterios diagnosticos; 1. Hemoglobina glicosilada >6.5%. 2. Glucemia plasmatica en ayuno
(8hrs) >126mg/dl (normal 60-100mg/dl), debe ser tomada en 2 ocasiones. 3. Glucemia plasmatica casual >200 mg/dl, en un paiente con sintomas
clasicos de hiperglucemia (normal 100-200 mg/dl). 4. Glucemia >200mg/dl a las 2 hrs despus de una carga oral de 75mg de glucosa (PTOG). Si tenemos
una glucosa <100mg/dl, pero hay glucosuria, realizar una PTGO. Prediabetes: Persona que tiene antecedente padre, madre o ambos con estado
metablico entre lo normal y la diabetes, glucosa anormal en ayuno 100-125 mg/dl, PTOG; 140-199 mg/dl. TRATAMIENTO: Nutricional; disminucion de
carbohidratos, incremento en fibra, disminucion de lipidos. Aumentar la actividad fsica al menos 150 min/semana como caminar o trotar (3 dias a la
semana, no mas de 2 dias seguidos). Perder 7% de peso corporal (dieta ms actividad fsica 2-4 meses, no control). 1. Modificaciones al estilo de vida +
metformina (usar pioglitazona por intolerancia), especial en pacietes con IMC > 35, >60 aos o con diabetes gestacional previa. Pacientes sin sobrepeso
se puede iniciar con sulfonilureas, no control. 2. metformina + sulfonilurea (1ra eleccion). Metformina + tiazolidendionas (2 eleccin) no control.
Metformina + sulfonilurea + insulina o metformina + tiazolidendionas + sulfonilureas. (tiazolidendionas contraindicadas en pacientes con insuficiencia
cardiaca o juntas con insulina). Sintomas marcados o glucemias o Hb glicosiladas muy elevadas iniciar con insulina con o sin agentes adicionales, si no hay
control con insulina a dosis mxima controlada, agregar un segundo frmaco oral. SULFONILUREAS (glibenclamida, vida media larga 10-16hrs); Farmaco
de eleccin en paciente no obeso, que no tolere metformina y que no tenga un riesgo especial de hipoglucemias, estimula las clulas beta del pncreas
para liberacin de insulina de los granulos secretorios hacia el torrente sanguneo, aumenta el consumo de glucosa a nivel perifrico. Hay que vigilar
funcin renal por riesgo de hipoglucemia (no dar con una depuracin <50, por el riesgo de acumulacon e hipoglucemia). BIGUANIDAS (metformina);
Inhiben la gluconeognesis heptica, aumentan la capatcion de glucosa por el musculo, aumento y actuivacion de una protein kinasaactivada por AMP
que induce la traslocacion del GLUT 4 y aumenta tambin la oxidacin de acidos grasos, aumenta la glucolisis anaerobia y produccin de ac., lctico.
INHIBIDORES DE -glucosidasa (acarbosa); retraso en la absorcin de hidratos de carbono complejos, reduccin de pico mximo de glucemia
pospandrial. TIAZOLIDINEDIONAS (pioglitazona); Produce aumento en la transcripcin de genes de las enzimas que normalmente son inducidas por la
insulina, aumento de utilizacin perifrica de glucosa. INCRETINAS (GLP1 y GIP); tienen como funcin aumentar la secrecin de insulina pospandrial, son
liberadas en el intestino y susu niveles aumentan en respuesta a la comida. COMPLICACIONES: En descompensacin y agudas; estado hiperosmolar,
cetoacidosis, hipoglucemia. A largo plazo; Nefropatia diabetica=IRC, la lesion renal, incrementa la TA capilar glomerular, esto incrementa la
permeabilidad y la filtracin de proteinas, se caracteriza por proteinuria persistente, HAS, disminucin de TFG, una de las primeras manifestaciones que
se asocia con microalbuminuria (30-300mg/24hrs) suele ser edema perimaleolar intermitente. Macroalbuminuria >300mg/24hrs. Estadios; dao renal
(DR), con funcin renal normal, TGF >95. Etapa 2 DR con diminucin leve de la funcin, TFG 60-89, etapa 3 DR con disminucin moderada de la funcin,
TFG 30-59, etapa 4 DR con disminucin severa de la funcin, TFG 15-20, etapa 5 insuficiencia renal, TFG <15, los IECA y ARA II son nefroprotectores y
antiproteinuricos, en pacientes con estadios 1 al 3 limitar ingesta de proteinas a 0.8gr/kg de peso ideal, no utilizar BCC dihidropiridinicos.
Retinopata=amaurosis; el manejo de la reinopatia consiste en control metabolico y oftalmolgico, la fotocoagulacin (destruye el tejido hipoxico) y
vitrectomia permiten mejorar el pronstico y reducir el riesgo de ceguera, el exudado blanco y algodonosos es la expresin de la oclusin capilar
reciente, la aparicin de neovasos en la superficie de la retina indica el estado proliferativo, Retinopatia diabtica no proliferativa (RDNP) leve; solo
microaneurismas, en RDNP moderada; exudados duros, microaneurismas, en la grave hemorragias y exudados algodonosos. En la retinopata
proliferativa; neovasos, hemorragias prerretinianas (h. vtrea), en la avanzada ya hay tejido cicatrizado. Neuropata perifrica = ulceras, malformacion de
Charcot. Polineuropatia simtrica distal (en guante y calcetn); es la mas frecuente, disestesias adormecimiento, dolor e hiperalgesia en miembros
inferiores, seguido de perdida de sensibilidad trmica y al tacto a estimulos dolorosos. Mononeuropatia; la mas comn es la paralisis de III par, comn en
ancianos, inicio rpido asociado a dolory autolimitada. Sindromes de atrapamiento; inico lento, progresivo, nervios, mediano, ulnar, radial, femoral, del
tnel del carpo, daignostico por electrofisiologa. Amiotrofia diabtica: neuropata motora proximal, debilidad en musculos proximales de los miembros
inferiores con incapacidad. Neuritis de fibras pequeas, de inicio sbito luego de aber inciado tratamiento con insulina, es de carcter quemante,
lacinante o punzante, parestesias, hormigueo, sensacin de frio. Neuropata autonmica = vejiga neurogenica, disautonomia cardiovascular
(hipertensin paradjica supina o nocturna), disfuncin sexual, gastroparesia (diarrea, estreimiento), piel (fra, anihidrosis, fisuras). El estrs oxidativo
es un factor determinate para la patognesis de la polineuropatia, por lo que es prioritario el control de la hiperglucemia. Tratamiento sintomtico del
dolor; se puede usar antidepresivos tricclicos (amitriptilina 25-150mg, conraindicada en glaucoma o arrittmias cradiacas, desipramina) son de primera
lnea en polineuropatia dolorosa, si estos no se toleran inhibidores de la racaptura de serotonina (paroxetina 20-40mg, citalopram, venlafaxina),
inhibidores de la recaptacion de serotonina y noradrenalina (duloxetina 60-120mg/dia, opcin final mas si se asocia a depresion). Carbamazepina, no se
recomienda de primera lnea, fenitohina se usa para el dolor neuropatico, en el servicio de urgencias, gabapentina de 300-3600mg/dia, cuando los
tricclicos no son tolerados, es de primera lnea, pregabalina 150-600mg/dia frmaco de primera eleccin si polineuropatia no dolorosa, solo calambres,
se puede sumar tramadol de 50-400mg/dia VO. PREVENCION: En diabeticos control glicemico con Hb glicosilada 2 por ao, si hay descontrol hacer cada
3 meses, es lo que tiene de memoria, valores <7% reduce retinopatia, nefropata y todas las complicaciones microvasculares. Pacientes asintomticos
adultos a cualquier edad, con sobrepeso u obesidad (IMC>25), con uno o mas factores de riesgo, hacer prueba cada 3 meses si salen normales.
Asintomtico con glucosa en ayuno >100 y <126; repetir glucosa en ayuno, se repite resultado = prediabetes, sale >126 hacer PTGO. Prediabetes;
seguimiento anual. Paciente con cifras normales sin factores de riesgo cada 3 aos. Deteccion de albuminuria una vez por ao y al momento del dx., toda
cifra superior 300mg/dl = albuminuria clinica o macroalbuminuria. Creatininemia por lo menos 1 vez por aos. Valoracion por oftalmologia una vez por
ao. La presencia de retinopata, no es una contraindicacin para el tratamiento cradioprotector con aspirina, ya que esta terapia no aumenta el riesgo
de hemorragia retiniana. Meta de colesterol LDL <100mg/dl, si hay riesgo cardiovascular <70mg/dl. PRONSTICO: Las variables que nos pueden hacer
predecir el peor pronostico estan relacionadas con el grado de sedentarismo, coexistencia de IC, descenso del aclaramiento de creatinina (nefropatia
principal), y de hemoglobina. REHABILITACION: Micro y macroalbuminuria, iniciar trtatamiento con IECAS o ARA II, para nefroproteccin (sin olvidar
medir niveles de K+), excepto en embarazadas. Reducir ingesta de proteinas de 0.8-1 g/kg/da. CLAVES: Biguanidas (Metformina) 1ra. Eleccion en
obesos, complicacion mas grave acidosis lctica. Insulina glargina, dura 24hrs, se mantienemantiene basal todo el dia.mantiene Insulina NPH, dura hasta

CURSO ENARM CMN SIGLO XXI TEL: 36246001

Pharmed Solutions Institute

PGINA 52

MANUAL DE TRABAJO DEL CURSO ENARM CMN SIGLO XXI


12hrs, dosis 0.15 UI/kg, 2/3 matutino y 1/3 vespertino. Efecto somogy; hiperglucemia reactiva matutina, secundaria a una hipoglucemia
nocturna.Insulina lispro ultrarapida, ideal para tx. de picos de insulina. Cuando no se logran las metas teraputicas, con el tratamiento actual agregar un
nuevo frmaco, en lugar de cambiarlo por otro. DIABETES TIPO 1: Representa el 5-10%, es un proceso autoinmune, gentica, hay autoanticuerpos
presentes en el 85-90%, en muchas ocasiones encontramos la presencia de cetoacidosis como la primera manifestacin en nios y adolscentes.
CASO CLINICO HIPOGLUCEMIA
Mujer de 78 aos, independiente en actividades de la vida diaria, con
escolaridad completa, tiene antecedentes de hipertensin arterial,
hipotiroidismo, hipoacusia y diabetes mellitus tipo 2 (DM 2) manejada
con glibenclamida 10 mg y metaformina 1.700 mg al da, con lo cual
lograba glicemias capilares adecuadas. Fue llevada por familiares al
Servicio de Urgencia, para evaluacin por compromiso cualitativo de
conciencia, no fluctuante, de ms de 24 hrs de evolucin, sin signos de
focalizacin y con recuperacin espontnea completa. Su familia
seal la presencia cada vez ms frecuente de episodios similares en
los ltimos 2 meses, indicando adems, alteraciones de la memoria
reciente, de la conducta y alteraciones autolimitadas de la marcha, sin
trastorno del control de esfnteres, con repercusin funcional por
prdida parcial de su autovalencia.
PREGUNTA
Cul es el factor ms probable que genere la hipoglucemia en este
paciente?
RESPUESTA
a.- La glucosuria.
b.- El hipoglucemiante.
c.- El antipertensivo.
d.- Depuracin renal
CASO CLINICO DIABETES MELLITUS E HIPERTENSION
Mujer de 70 aos de edad, hipertensa, diabtica de reciente
diagnostico durante un examen de rutina y cursando con bacteriuria
asintomatica, procur atencin mdica por tos seca, disnea,
inapetencia y desnimo. La paciente saba que era portadora de
hipertensin arterial desde la edad de 37 aos, de diabetes mellitus
desde los 56 aos y de hipertrigliceridemia. A los 56 aos de edad, se
quej de palpitaciones taquicrdicas con duracin de 20 minutos, sin
sncope. Haca uso de 75 mg de captopril, 50 mg de clortalidona, 600
mg de quinidina y 0,25 mg de digoxina. El examen fsico revel peso de
54 kg, altura de 1,49 m, presin arterial de 170/110 mmHg. En el
examen fsico, fue identificado soplo sistlico +/4+ en rea mitral. Lo
restante del examen fsico no revel alteraciones. El ECG mostr ritmo
sinusal, sobrecarga ventricular izquierda, infradesnivel de ST en V5 y
V6 y presencia de onda U.
PREGUNTA
Cual de los siguientes frmacos tiene mayor evidencia de efecto
cardioprotector en esta condicin del paciente?
RESPUESTA
a.- IECAs
b.- ARA II.
c.- Digoxina.
d.- Clortalidona.
PREGUNTA
Considerando las condiciones del caso clnico cual es la indicacin mas
adecuada para envio a olftalmologia?
RESPUESTA
a.- Disminucion de la funcin renal.
b.- Incremento de creatinina.
c.- Aumento de microalbuminauria.
d.- Presencia de fosfenos.
CASO CLINICO DIABETES
Se trata de masculino de 46 aos de edad el cual acude a consulta
debido a que ha presentado un proceso infecciosos urinario ya
tratado, refiere mejora completa posterior a una semana de
tratamiento antibitico, a la exploracin identifica un permetro

CURSO ENARM CMN SIGLO XXI TEL: 36246001

abdominal de 102 cms debido a ello realiza una glucosa perifrica


donde se obtiene 130 mg/dl.
PREGUNTA
Cual es la conducta a seguir para sustentar el diagnostico?
RESPUESTA
a.- Curva de tolerancia a la glucosa.
b.- Glucosa en ayuno.
c.- Determinar hemoglobina glucosilada.
d.- Glucosa pospandrial.
CASO CLINICO DIABETES MELLITUS E INFECCION
Varn de 47 aos con DM diagnosticada hace 9 aos, en tratamiento
actual con esquema insulnico intensivo (una dosis nocturna de
insulina glargina e insulina lispro previo a las comidas). Entre sus
antecedentes destacaban gran tabaquismo, dislipidemia mixta en
tratamiento irregular con ciprofibrato 100 mg/da y microalbuminuria
de 50 mg/12 h, sin hipertensin arterial en tratamiento con enalapril 5
mg/da. Consult en un centro de salud por sensacin febril no
cuantificada, calofros, odinofagia, nuseas y vmitos alimentarios de 2
das de evolucin. Se constat deshidratacin moderada, temperatura
axilar de 37,8C y glicemia capilar mayor de 400 mg/dl. Fue internado,
recibiendo solucin fisiolgica e insulina en esquema de minidosis.
Entre los exmenes de laboratorio obtenidos al ingreso destacaban
glicemia 549 mg/dl, creatinina 1,67 mg/dl, pH 6,91, BE -27, HC03 4,6
(en sangre arterial), cetonemia +++, natremia 141,7 mEq/L, kalemia
5,92 mEq/L, cloremia 98 mEq/L, amilasemia 48 U/L, hematocrito
40,5%, leucocitos 9100 mm3, VHS 38 mm/h y urocultivo negativo.
PREGUNTA
Que cambios electrocardiogrficos es probable esperar?
RESPUESTA
a.- QRS corto.
b.- P-R prolongado.
c.- T picudas.
d.- Ondas U.
PREGUNTA
Se envio urocultivo y hemocultivo donde se observo crecimiento de e.
coli cual es el manejo antimicrobiano mas adecuado considerando el
KDOQI actual?
RESPUESTA
a.- Ceftriaxona y vancomicina.
b.- Ampicilina y amikacina.
c.- Imipenem y gentamicina.
d.- Imipenem, vacomicina y gatifloxacina.
CASO CLINICO DIABETES MELLITUS E IRA
Hombre de 62 aos con antecedentes de hipertensin arterial, infarto
agudo de miocardio y DM tipo 2. Ingresado por pielonefritis con IR
aguda (MDRD 21 ml/min), interpretndose en el contexto de la propia
infeccin, agravada por el consumo de antiinflamatorios no
esteroideos (AINE) y persistiendo al ser dado de alta. Un mes despus
presenta lo siguiente (MDRD 13,83 ml/min), proteinuria de 5 g/24 h y
microhematuria persistente.
PREGUNTA
Cual es su conducta diagnostica mas til en este momento?
RESPUESTA
a.- Urocultivo.
b.- Citologia para malignidad.
c.- Fondo de ojo.
d.- Ecografia abdominal.

Pharmed Solutions Institute

PGINA 53

MANUAL DE TRABAJO DEL CURSO ENARM CMN SIGLO XXI


PREGUNTA
Considerando el MDRD actual en que grado de IR se encuentra?
RESPUESTA
a.- IR grado 4.
b.- IR grado 3.
c.- IR grado 2.
d.- IR grado 1.
CASO CLINICO DIABETES MELLITUS Y NEFROPATIA.
Varn de 58 aos, con hipertensin arterial en tratamiento
farmacolgico con regular control y diabetes mellitus tipo 2 en
tratamiento con insulina con buen control metablico. Con (IRC) y
microalbuminuria. Ingresa la tensin arterial era 139/87 mmHg, peso
de 81 kg, talla de 180 cm, urea de 61 mg/dl, creatinina 1,7 mg/dl,
aclaramiento de creatinina 44,37 ml/min, microalbuminuria 11,75
ug/min y hemoglobina de 5,5; albmina 4,2 g/dl, protenas totales 6,7
g/dL, colesterol total: 150 mg/dl, triglicridos: 168 mg/dL.
PREGUNTA
En que estadio de IRC se encuentra?
RESPUESTA
a.- Estadio 1.
b.- Estadio 2.
c.- Estadio 3.
d.- Estadio 4.
CASO CLINICO DIABETES MELLITUS
Varn de 69 aos, con historia de HTA, cardiopata isqumica, hernia
hiatal, gastritis crnica y diabetes mellitus tipo II, con pobre control
metablico habitual, tratada con hipoglicemiantes orales, alcoholismo
crnico, acude por debilidad muscular proximal en las cuatro
extremidades (dificultad para realizar tareas como abducir los brazos,
subir escaleras, levantarse desde una silla baja, caminar distancias
moderadas, etc.), de inicio insidioso, simtrico y lentamente
progresivo, acompaada de marcada inestabilidad para la marcha,
prdida de la musculatura distal en las extremidades y saltos
musculares, as como una acentuacin de las parestesias habituales
(pinchazos, adormecimientos, etc.) fundamentalmente en horarios
nocturnos. Paralelamente, se hallaba aquejada por disminucin
bilateral y rpidamente progresiva de la agudeza visual, astenia,
poliuria, polidipsia y disminucin de peso. Paciente vigil, bien
orientado, con deterioro cognitivo, con lenguaje conservado; marcha
ligeramente atxica; cuadriparesia flccida simtrica de predominio
proximal, arreflexia osteotendinosa generalizada, hipotrofia muscular
distal ms acentuada en las manos y antebrazos, acompaada de
fasciculaciones; signo de Romberg positivo.
PREGUNTA
Cual de los siguientes trastornos es mas probable que presenta
actualmente el paciente?
RESPUESTA
a.- Demencia por alcoholismo.
b.- Degeneracion cerebelosa.
c.- Polineuropatia diabtica.
d.- Degeneracion pontoolivocerebeloso
CASO CLINICO
Mujer de 63 aos remitida desde Atencin Primaria por obesidad
mrbida sin respuesta a tratamiento diettico. Antecedentes
personales: hipertensin arterial en tratamiento con 3 frmacos,
hipercolesterolemia, sndrome de apneas obstructivas del sueo
(SAHOS). Tratamiento farmacolgico: cido acetilsaliclico 100 mg/da,
alprazolam 0.5 mg/da, atorvastatina 40 mg/da, esomeprazol 40
mg/da, eprosartn 600 mg/da, hidroclorotiazida 12.5 mg/da. La
paciente tiene obesidad leve desde la juventud, con aumento
importante de peso a partir de la menopausia, alcanzando en la
actualidad su peso mximo (121 kg). Mltiples intentos dietticos

CURSO ENARM CMN SIGLO XXI TEL: 36246001

previos, bajo supervisin mdica, con xito parcial inicial (prdida de


hasta 10 kg de peso) y recuperacin ponderal posterior. No efecta
ejercicio de forma regular. En analtica de rutina efectuada se objetiva
glucemia de 156 mg/dl, que se confirma en segunda determinacin
analtica (glucemia: 161 mg/dl; HbA1c 8.3%). No clnica cardinal
hiperglucmica. EF: tensin arterial 130/80 mmHg; peso 121 kg; talla
165 cm; IMC 44.4 kg/m2; permetro de cintura 126 cm Leve acantosis
nigricans en cuello. Auscultacin cardiopulmonar: normal. No edemas.
Pulsos pedios y tibiales posteriores presentes y simtricos. No
alteraciones trficas.
PREGUNTA
Cul es la conducta teraputica ms adecuada a seguir en esta
paciente?
RESPUESTA
a.- Dieta y ejercicio fsico, sulfonilureas
b.- Dieta y ejercicio fsico, tiazolindendionas mas insulina
c.- Dieta y ejercicio fsico, biguanidas mas insulina
d.- Dieta y ejercicio fsico, biguanidas
PREGUNTA
Cul es la meta de hemoglobina glicosilada en esta paciente para
reducir las complicaciones microvasculares?
RESPUESTA
a.- <6.4%
b.- >7%
c.- <7%
d. <8%
PREGUNTA
A los 4 meses la paciente consulta por presentar poliuria, as como
refiere que est tomando ms agua de lo normal y que tienen mucho
apetito, desde hace un mes, se le toma una glucosa a las 13:00hrs la
cual nos reporta 305mg/dl. Cul sera la conducta ms adecuada a
seguir en este momento?
RESPUESTA
a.- Dieta y ejercicio fsico, sulfonilureas mas tiazolindendiona
b.- Dieta y ejercicio fsico, tiazolindendionas mas insulina
c.- Dieta y ejercicio fsico, biguanidas mas insulina
d.- Dieta y ejercicio fsico, biguanidas mas tiazolindendionas
PREGUNTA
Cmo actan las sulfonilureas, para mejorar los niveles de glucosa en
el organismo?
RESPUESTA
a.- Aumento de receptores de insulina, en monocitos, eritrocitos y
adipositos
b.- Inhiben gluconeogenesis heptica, aumenta captacin de glucosa
por el musculo
c.- Inhibicin reversible y competitiva de la alfa-glucosidasa
d.- Produce aumento en la transcripcin de genes de las enzimas que
normalmente son inducidas por la insulina
CASO CLINICO
Paciente de masculino de 53 aos de edad que asiste por consultorio
externo para control de diabetes, con el resultado de exmenes de
laboratorio solicitados con anterioridad. Es diabtico tipo 2 de 12 aos
de evolucin en tratamiento con metformina 1700 mg/da y
glibenclamida 10 mg/da. Otros antecedentes: hipertensin arterial,
obesidad, dislipemia, ex tabaquista y sedentario. El paciente refiere en
la consulta pesadez y fatiga, menciona que de 2 meses atrs han
disminuido sus comidas a la mitad, ya que se siente satisfecho cada vez
como menos cantidad de comida, asi mismoha disminuido un poco de
peso. EF; Peso: 85 kg Talla: 1.65 mts IMC: 31 kg/m2 TA: 140/75
mmHg. Laboratorios: glucosa 178 mg/dL, HbA1c 8,5%, depuracin de
cretinina de 24hrs 49ml/min, GOT 28 UI/L, GPT 49 UI/L, GGT 27 UI/L,
bilirrubina total 0,48 mg/dL, triglicridos 82 mg/dL, colesterol total 184
mg/dL, HDL colesterol 51 mg/dL, LDL colesterol 116.6 mg/d.

Pharmed Solutions Institute

PGINA 54

MANUAL DE TRABAJO DEL CURSO ENARM CMN SIGLO XXI


PREGUNTA
Cul es la conducta teraputica ms adecuada en este momento para
el paciente?
RESPUESTA
a.- Continuar con medicamentos actuales, aumentar a dosis mxima.
b.- Continuar con metformina, y cambiar glibenclamida por
pioglitazona
c.- Continuar con glibenclamida e iniciar insulina
d.- Continuar con metformina y glibenclamida e iniciar insulina
PREGUNTA
Con los datos clnicos que presenta el paciente. Con que complicacin
es ms probable que este cursando?
RESPUESTA
a.- Cncer de estomago
b.- Efectos secundarios de la sulfonilureas
c.- Neuropata autonmica
d.- Efectos secundarios de las biguanidas
PREGUNTA
Qu medidas ms probablemente nos ayudaran a reducir riesgos y
retrasar progresin de su padecimiento?
RESPUESTA
a.- Iniciar cardioproteccin con aspirina, as como IECA o ARA II
b.- Reducir ingesta de protenas de 0.8-1g/kg/da, medir niveles de
potasio, enviar a oftalmologa
c.- Iniciar IECA, reducir ingesta de protenas, enviar a oftalmologa
d.- Control de glucosa, reducir ingesta de protenas de 0,8-1g/kg/da,
medir nivel de potasio
CASO CLINICO
Femenino de 63 aos de edad hipertensa de 5 aos de evolucin,
diabtica tipo 2 desde los 43 aos, en mal control metablico habitual,
con glucosuria persistentemente positiva y retinopata diabtica
fotocoagulada. En el ltimo control analtico hace 6 meses presentaba
glucemia 198mg/dl, hemoglobina A1c 8.7%, urea 42mg/dl, creatinina
1.18mg/dl, FG 64ml/min, microalbuminuria. Est a tratamiento con
metformina, insulina glargina y rpida. En el control actual presenta TA
153/98 y en la analtica, glucemia de 116mg/dl, urea 41mg/dl,
creatinina 1.52mg/dl, FG 61ml/min, colesterol 274mg/dl (HDL 102
mg/dl, LDL 161 mg/dl) y microalbuminuria.
PREGUNTA
Ante esta situacin. Cul sera la complicacin ms probable?
RESPUESTA
a.- Riesgo aumentado para enfermedad renal crnica
b.- Dao renal estadio 2
c.- Dao renal con filtrado glomerular normal
d.- Dao renal estadio 1
PREGUNTA
Para control tensional en esta paciente. Cul sera la conducta
teraputica ms adecuada?
RESPUESTA
a.- ARA II
b.- Diurtico tiazidico

CURSO ENARM CMN SIGLO XXI TEL: 36246001

c.- Calcioantagonista
d.- Alfa bloqueador
PREGUNTA
Qu medicamento ms probablemente deberamos evitar en este
paciente, para evitar la progresin del dao renal?
PREVENCION
RESPUESTA
a.- Hiero
b.- IECA
c.- Estatinas
d.- AINES
CASO CLINICO
Paciente de 62 aos, casado, con dos hijas, empleado de una tienda de
repuestos mecnicos y de automvil, en la consulta su esposa nos
refiere, que est preocupada porque el paciente est gordo, fuma, a
veces lo veo raro y creo que algn da le va a dar algo. Se confirma
que el paciente es obeso (IMC 30); nos confirma que es fumador de 20
cigarrillos/da y presenta una TA de 156/98 mmHg (promedio de varias
tomas realizadas), las cifras de glucemia capilar oscilan alrededor de
170 mg/dl en ayunas y 225 postprandiales y en la analtica reciente
destacan: triglicridos 256 mg/dl, colesterol 235 mg/dl, HDLc 34 mg/dl,
LDLc calculado: 160 mg/dl , HbA1c: 8.1%. creatinina 1.3 mg/dl. urea 53
mg/dl y en orina existe glucosuria, siendo los cuerpos cetnicos y la
proteinuria negativos. Los pulsos perifricos y los reflejos
osteotendinosos son normales. A la vista de estos resultados
proponemos tratamiento con dieta hiposdica e hipocalrica de 1500
caloras y con glibenclamida un comprimido diario.
PREGUNTA
Cul es el objetivo de LDL, en este paciente y conducta teraputica
mas adecuada?
RESPUESTA
a.- <110mg/dl, pravastatina
b.- <70mg/dl, pravastatina
a.- <110mg/dl benzafibrato
b.- <70mg/dl benzafibrato
PREGUNTA
Cul es el objetivo de hemoglobina glicosilada y presin arterial en
este paciente?
RESPUESTA
a.- Hb glicosilada <8%, TA 140/90mmHg
b.- Hb glicosilada <7%, TA 130/80mmHg
c.- Hb glicosilada <6.5%, TA 130/80mmHg
d.- Hb glicosilada <6%, TA 140/90mmHg
PREGUNTA
Cul es la neuropata diabtica que ms probable presentara este
paciente?
RESPUESTA
a.- Neuropata motora proximal
b.- Neuropatia focal
c.- Polineuropatia simtrica difusa
d.- Neuropatia de fibras pequeas

Pharmed Solutions Institute

PGINA 55

MANUAL DE TRABAJO DEL CURSO ENARM CMN SIGLO XXI


ESTEATOSIS HEPATICA (EH):
CIENCIAS BASICAS: Acumulacin de lpidos histolgicamente visible de grasa, en el citoplasma de los hepatocitos puede haber diferentes grados de
inflamacin y fibrosis. Se puede clasificar de acuerdo a su etiologa en esteatosis alcohlica y esteatosis no alcohlica. Etiologia: sndrome metablico,
obesidad, diabetes tipo II e hiperlipidemia, rpida prdida de peso en los obesos, nutricin parenteral total, sndrome de intestino corto, gastroplastia,
hipolipoproteinemias, tirosinemi, enf. de Wilson, tratamiento con drogas como amiodarona, estrgenos, tetraciclinas, coticoides, tamoxifeno, nifedipino,
lipodistrofia, hgado graso del embarazo). SALUD PUBLICA: La esteatosis heptica no alcohlica es un padecimiento que afecta al 20-30% de la poblacin
general en varios pases; la prevalencia aumenta en los sujetos con obesidad mrbida 75-92%, mientras que en la poblacin peditrica es de
aproximadamente 13-14%. La forma progresiva de la esteatosis heptica no alcohlica, la esteatohepatitis, se estima que aproximadamente en 3-5% de
los casos puede progresar a cirrosis. En Mxico la prevalencia de esteatosis heptica, se informa
ETIOPATOGENIA
que el 30% de la poblacin rene criterios de obesidad y de ellos, ms de las dos terceras partes
ESTRS OXIDATIVO
pueden tener esteatosis en el hgado; por otra parte, la prevalencia promedio de DMT 2 es de
PEROXIDACION LIPIDICA
7%, y el 92% de estos pacientes tienen sndrome metablico y resistencia a la insulina.
CLASIFICACION: Segn el porcentaje de hepatocitos afectados, se clasifica en: 1) Leve; menos
ESTIMULOS
TNF-alfa
CELULAS
ANTIGENOS
de 25% de los hepatocitos afectados, 2) Moderada; de 25-50%, 3) Severa; ms de 50%.
QUIMIOTACTICOS
IL-6, IL-8
ESTRELLADAS
PROTEICOS
PATOGENIA: Resulta de un desequilibrio entre la sntesis hepatocitica de triglicridos a partir de
cidos grasos y de la secrecin desde el hepatocito bajo forma de lipoprotenas. La retencin de
INFLAMACION
FIBROSIS
H. MALLORY
triglicridos en los hepatocitos es requisito indispensable para el desarrollo de esteatosis
heptica. El metabolismo de los triglicridos en el tejido adiposo da lugar a la liberacin de cidos grasos libres a la circulacin y stos son captados por
los hepatocitos. El acumulo de lpidos en el hgado resulta de una prdida del balance entre la captacin, la sntesis, la salida y la oxidacin de los cidos
grasos libres. Estudios recientes demuestran que el flujo de cidos grasos libres provenientes del tejido adiposo y que llegan al hgado representa la
mayor fuente de la grasa intraheptica, 62% 82% de los triacilgliceroles del hgado y que la contribucin de lipognesis de novo que en sujetos
normales es menor del 5%, en pacientes con esteatohepatitis aumenta hasta ser del 26%. La insulina, inhibe el metabolismo de los triglicridos en el
tejido adiposo, aumenta la sntesis intraheptica de cidos grasos libres y de triglicridos e inhibe la beta oxidacin de los cidos grasos libres en los
hepatocitos. Los estados de hiperinsulinismo como la DMT2 y la obesidad disminuyen la sensibilidad tisular a la insulina, esta resistencia a la accin de la
insulina es el factor ms reproducible en el desarrollo de esteatosis heptica. Muchos casos son atribuidos al alcohol, su presencia se asocia con un
espectro de enfermedades tales como: ingesta de drogas, txicos, obesidad, dislipidemias, diabetes, caquexia y nutricin parenteral. En la esteatosis
heptica al igual que en otros padecimientos metablicos se altera la liberacin de adipocinas. ESTEATOHEPATITIS: Es una forma ms agresiva de
esteatosis heptica en la cul la infiltracin grasa del hgado puede llegar hasta el 2030%, se acompaa de intensa actividad necro-inflamatoria y puede
progresar a fibrosis y cirrosis heptica y eventualmente a insuficiencia heptica y hepatocarcinoma. De los factores que participan en la progresin de
hgado graso a esteatohepatitis se reconoce que tanto el sndrome metablico (Resistencia insulnica/hiperinsulinismo/DM tipo 2, Obesidad, Dislipemia
(TG >180 o HDL-col <40), HTA) como la resistencia a la insulina son factores predictivos independientes en esa progresin. DIAGNOSTICO: Clnico: puede
haber dolor en hipocondrio derecho y presentar una hepatomegalia lisa, firme, a veces dolorosa. Laboratorio: nada especifico, puede haber aumento en
las transaminasas y GGT pueden dar sospecha. Imagenologa: La ecografa es el mtodo mas accesible para el diagnstico de EH (se observa, aumento de
la ecogenicidad heptica hgado brillante, acompaado de una atenuacin del haz sonoro en los sectores profundos del hgado). En funcin de los
hallazgos clasificamos a los pacientes en tres grados: Grado 0: No presenta esteatosis heptica. Grado I: Presenta esteatosis heptica, pudiendo
distinguir la grasa periportal. Grado II: Presenta esteatosis heptica, con borramiento de la grasa periportal. Supone un mayor grado de acmulo graso.
TAC Y RM, diagnstico certero, pero altos costos y disponibilidad. Laparoscpico: observacin del tpico color amarillento de la superficie heptica. El
examen histolgico mediante puncin bipsica heptica, permite el diagnostico de EH, constituyndose en el patrn de oro en el diagnstico de esta
enfermedad. TRATAMIENTO: Cambios en el estilo de vida, para ello tenemos que considerar entre otras cosas el consumo de caloras y el tipo de
alimentos, la actividad fsica que ese paciente desarrolla, su situacin socioeconmica, etc. A la fecha, no hay un tratamiento farmacolgico que haya
demostrado ser de utilidad precisa, algunas modalidades descritas incluyen: medicamentos utilizados para bajar de peso, medicamentos que intentan
bloquear la resistencia a la insulina, agentes hipolipemiantes, antioxidantes, citoprotectores, adems de la ciruga baritrica. PRONOSTICO: La esteatosis
heptica simple parece tener un pronstico relativamente benigno mientras que aquellos con datos de esteatohepatitis o fibrosis en la biopsia del
hgado tienen un peor pronstico.
CASO CLINICO HIGADO GRASO.
Se trata de paciente masculino de 41 aos de edad con antecedentes
de obesidad grado II, dislipidemia, bajo tratamiento con bezafibrato,
pravastatina y orlistat adems refiere ingesta crnica de alcohol, fue
diagnosticado con esteatosis hace 6 meses y hernia hiatal con mal
apego a tratamiento, acude a consulta debido a dolor en cuadrante
superior derecho, tos con leve expectoracin, con disminucin de
ruidos respiratorios en bases de predominio derecho, se observa leve
tinte ictrico central. Signos vitales TA 130/100 mmHg, FR 27, FC 98,
Temperatura 37.6 grados. Se realiza tele de torax donde se observa
elevacin del diafragma de predominio derecho.
PREGUNTA
Cul es la conducta teraputica a seguir ms adecuada?
RESPUESTA
a.- Iniciar antibitico de amplio espectro.
b.- Realizar USG de via biliar.
c.- Indica bloqueadores de H.
d.- Realiza panendoscopia
CASO CLINICO

CURSO ENARM CMN SIGLO XXI TEL: 36246001

Paciente de sexo masculino de 53 aos de edad que asiste por


consultorio externo de Diabetologa para control con el resultado de
exmenes de laboratorio solicitados con anterioridad. Es diabtico tipo
2 de 12 aos de evolucin en tratamiento con metformina 1700
mg/da y glibenclamida 10 mg/da. Otros antecedentes: hipertensin
arterial, obesidad, dislipemia, ex tabaquista y sedentario. El paciente
refiere en la consulta pesadez negando dolor abdominal en u otros
sntomas relacionados. EF: Dolor a la palpacin profunda en
hipocondrio derecho, con ligera hepatomegalia, 2 traveses por debajo
del reborde costal. No presentaba signos de hepatopata crnica. Peso:
85 kg Talla: 1.65 mts IMC: 31 kg/m2 TA: 140/75 mmHg. GOT 137
UI/L, GPT 215 UI/L, GGT 90UI/L, FAL 370 UI/L, Bil T 0.98 UI/L, Gluc: 164
mg/dL, Col T 209 mg/dL, LDL 118 mg/dL, TG 239 mg/dL. El resto del
laboratorio no presentaba resultados relevantes
PREGUNTA
Cul es el diagnostico mas probable en este caso?
RESPUESTA
a.- Cirrosis heptica
b.- Hepatitis
c.- Esteatosis heptica
d.- Hepatomegalia

Pharmed Solutions Institute

PGINA 56

MANUAL DE TRABAJO DEL CURSO ENARM CMN SIGLO XXI


CIRROSIS HEPATICA Y HEPATOPATIAS:
CIENCIAS BASICAS: Hepatopatia crnica: claudicacin del hgado, que se torna incapaz de cumplir con sus funciones, el dao heptico crnico
anatmico-patologico se manifiesta como cirrosis. Cirrosis es una definicin histopatolgica, y tienen una variedad de causas, existe lesin hepatocelular,
fibrosis heptica, formacin de ndulos de regeneracin, dando como resultado decremento en la funcin heptica. Recordar: Factores de coagulacin
dependientes de la vitamina K son: II, VII, IX y X. SALUD PBLICA: Causas de cirrosis: alcoholismo (60-70%), esteatohepatitis no alcohlica (10-15%),
hepatitis viral crnica, VHB, VHC (10%), criptognica, hepatitis autoinmune y farmacos, cirrosis biliar, cirrosis biliar primaria (5%), colangitis esclerosante,
colangiopata autoinmune, cirrosis cardiaca (Sx. Budd-Chiari= ICD y pericarditis constrictiva), hemocromatosis, enfermedad de Wilson, deficiencia de
antitripsina 1, fibrosis qustica. PATOGENIA: El principal evento en la gnesis es el aumento en la produccin y depsito de tejido de colgeno (I, III, el
tipo I es ms raro), al ser estimuladas las clulas hepticas estrelladas expresan ms RNA mensajero para sintetizar colgeno, esto produce la fibrosis
heptica y ndulos de regeneracion, est en combinacin con distorsin vascular genera hipertensin portal, LA SANGRE a presin en el sistema portal
propicia su paso a otras vas que la derivan hacia la circulacin general, generando cortocircuitos, con posterior varices esofgicas, red venosa colateral,
hemorroides. La cirrosis se considera irreversible en estadios avanzados; es probable que la diferencia entre la fibrosis reversible e irreversible sea el
tipo de colgeno depositado (tipo I no es degradable, tipo III, s lo es). El tejido fibroso comprime los vasos y canalculos biliares, hay obstruccin biliar;
ictericia, por paso de bilirrubina conjugada a la sangre, retencin de sales biliares, con
baja absorcin de lpidos (vitamina K) que puede llevar a coagulopatia.
CLASIFICACIN: histopatolgica: micronodular, tpica de cirrosis alcohlica, se
caracteriza por un tamao uniforme de ndulos, <3mm de dimetro. La variedad
macronodular >3mm, se presenta secundaria a hepatitis viral o autoinmune.
DIAGNOSTICO: Cuadro clnico: Puede estar ausente, anorexia, fatiga, trastornos del
sueo, nausea, vmito, diarrea, dolor vago en cuadrante superior der. (70% hgado
palpable y firme), debilidad, fiebre, ictericia hepatocelular, amenorrea, impotencia,
infertilidad, angiomas en araas, eritema palmar, hipertrofia de glndulas paratiroides
y lagrimales, contractura de Dupuytren (fibrosis de la fascia palmar), lneas blancas en
la uas, hipoglucemia (descenso de depsitos hepticos de glucgeno), fetor
hepaticus, vasodiltacion (por accin
prolongada
de
oxido ntrico),
hepatoesplenomegalia, ascitis, sangrado gastrointestinal, encefalopata heptica (por
acumulo de elementos nitrogenados, amonio, mercaptano o fenol que generan
intoxicacin). Como hay una disminucin en la capacidad de dextoxificar los estrogens
en el hgado, se acumulan en el varon y nos da el Habito feminoide de Chvostec;
ginecomastia, atrofia testicular, araas vasculares. Puede haber mayor susceptibilidada
infecciones y bacteriemias por disminucin de la capacidad de las clulas de Kuppffer. Laboratorio: Anemia (microcitica, debida a prdida de sangre,
macrocitica, debida a deficiencia de folato; hemoltica llamada Sx. de Zieve), pancitopenia (hiperesplenismo), PT prolongado, hiponatremia, alcalosis
hipocalemica, intolerancia a la glucosa, hipoalbuminemia (edemas, ascitis, porque disminuye la presin oncotica del plasma con <2.5g/dl).
Hiperbilirrubinemia, aumento de AST, ALT, FA y globulinas, disminucin de factores de la coagulacin, K dependientes. USG abdominal muestra
hepatomegalia dependiente de lbulo izq., o caudado, descarta ascitis y hepatocarcinoma, permite evaluar, vena porta, esplnica y heptica. Diagnstico
definitivo a menudo depende de la biopsia de hgado (percutnea, transyugular o abierta). El Child-Pugh es usado para predecir la severidad de la cirrosis
y el riesgo de complicaciones. COMPLICACIONES: 1.-Hipertensin portal: Se define como el aumento sostenido de la presin hidrosttica en el sistema
venoso portal, que genera un gradiente de presin entre la vena porta y la vena cava inferior >5mmHg, lo cual favorece el desarrollo de circulacin
colateral y derivacin del flujo portal hacia la circulacin sistmica, es causada por aumento en la resistencia intrahepatica e incremento en el flujo
sanguneo esplcnico secundario a vasodilatacin, sus 3 principales complicaciones son: varices gastroesofgicas con hemorragia, ascitis e
hiperesplenismo. La vena porta est formada por vena gstrica isq., esplnica y mesentrica superior. 2.-Varices gastroesofgicas con hemorragia: Son
vasos colaterales generalmente con un gradiente de presin >12mmHg, para la prevencin primaria del sangrados se utilizan bloqueadores beta no
selectivos (propanolol). 3.-Ascitis: acumulacin de lquido en cavidad peritoneal, se considera la complicacin ms frecuente de la cirrosis, se asocia a
una mortalidad de 50% a los 2 aos, dx., por exploracin fsica y USG abdominal, se debe realizar paracentesis para determinar la albumina, protenas,
glucosa, LDH, conteo celular, tincin de Gramm y cultivo. El tx., consiste en restriccin de la ingesta de Na, diurticos (espironolactona, furosemida). 4.Peritonitis bacteriana espontanea (PBE): ocurre en 10-20% de pacientes con cirrosis y ascitis, factores de riesgo, protenas totales menores a 1g/dl,
antecedente de PBE y sangrado gastrointestinal, se manifiesta con fiebre dolor abdominal y encefalopata, dx., con conteo celular de liq. de ascitis debe
tener >250 neutrfilos/mm3 en el cultivo lo mas frecuente es E. coli, Klebsiella (70%), Enterococos y Streptococcus (30%). 5.- Encefalopata heptica. 6.Otros como: sndrome hepatorrenal tipo I, II, sndrome hepatopulmonar, hipertensin portopulmonar, malnutricin, coagulopata, fibrinlisis,
trombocitopenia, osteopenia, osteoporosis, anemia, hemolisis. TRATAMIENTO: La cirrosis heptica compensada no requiere de tratamiento, especifico,
solo dieta restringida en protenas (1-1.2 g/Kg), evitar consumo de alcohol y endoscopia al momento del dx., y peridica, cuando no hay varices
esofgicas o son muy pequeas, realizar cada 2 aos; con varices grado II y III iniciar bloqueadores , y cuando hay sangrado endoscopia cada 6-12
meses. El nico tratamiento que modifica claramente el pronstico es el trasplante heptico. Trasplante de hgado: Indicaciones: Nios; atresia biliar,
hepatitis neonatal, fibrosis heptica congnita, enfermedad de Angilles, enfermedad de Bylers, desordenes inherentes del metabolismo, enfermedad
de Wilsons, enfermedades de depsito lisosomal, Crigler-Najar tipo I, hipercolesterolemia familiar. Adultos; cirrosis biliar primaria y secundaria,
colangitis esclerosante primaria, hepatitis autoinmune, enfermedad de Carolis, cirrosis criptogenica, trombosis venosa heptica, hepatitis crnica con
cirrosis, hepatitis fulminante, malignidad hepatocelular primaria, esteatohepatitis no alcohlica Contraindicaciones: Infeccin extrahepatobiliar no
controlada, sepsis activa sin tratar, anomalas congnitas incorregibles que limitan la vida, abuso de sustancias o de alcohol, enfermedad cardiopulmonar
avanzada, malignidad extrahepatobiliar, colangiocarcinoma, SIDA. CIRROSIS ALCOHOLICA: Hombres de 60g/dia de alcohol, mujeres 30g/dia (por menor
volumen de distribucin, y poca cantidad de alcohol deshidrogenasa en la mucosa gstrica). Se debe documentar abuso de alcohol en la historia clnica,
asintomtica, clnica >10 aos de abuso de alchol. En este tipo de cirrosis la atrofia testicular debida a alteraciones hormonales o efecto toxico del
alcohol. Pueden presentar anemia hemoltica por efecto de la hipercolesterolemia, en la membrana eritrocitica, la cual genera acantocitos. Es
caracterstica la elevacin de AST sobre ALT, lo produce relacin AST/ALT >2. La biopsia heptica reporta necrosis, cuerpos de Mallory e infiltracin por
neutrfilos. HEMOCROMATOSIS: Genetica, aumento crnico de la absorcin de hierro, hay depsitos en hgado, pncreas, testiclos, hipfisis, corazn, la
ferritina srica esta elevada >1000ug/l, la hemoglobina aumntada al doble de lo normal. ENFERMEDAD DE WILSON: Hereditaria autosmica recesiva,
alteracin en el metabolismo del cobre, hay degeneracin de los nucleos de la base del cerebro y presencia de anillos pigmentados caf verdosos en la
periferis de la cornea. CIRROSIS BILIAR PRIMARIA (CBP): Enfermedad progresiva, con inflamacin y destruccin de conductos biliares intrahepticos, lo
cual produce colestasis crnica y cirrosis; predomina en mujeres (95%), entre 30-65 aos de edad, probablemente debida a un trastorno autoinmune

CURSO ENARM CMN SIGLO XXI TEL: 36246001

Pharmed Solutions Institute

PGINA 57

MANUAL DE TRABAJO DEL CURSO ENARM CMN SIGLO XXI


adems se asocia con frecuencia a enfermedades autoinmunes. Asintomticos, manifestacin inicial con frecuencia es prurito, predominio nocturno y se
asocia a piel seca, se puede ver ictericia, hiperpigmentacin, xantelasma, y xantomas, hepatoesplenomegalia, malabsorcin intestinal, astenia.
Laboratorio: anemia normocitica normocromica, eosinofilia, elevacin de FA (>600mU/l), GGT, AST, ALT (100-250mU/l), bilirrubinas normales al
principio, despus elevadas, hipergamaglobulinemia, anticuerpos antimitocondriales (AMA), presentes hasta en 95%, tienen sensibilidad de 95% y
especificidad de 98%. Anticuerpos antinucleares (ANA) en 70% de los casos. Tratamiento alivio del prurito (colestiramina, colestipol, fenobarbital),
correcion de las consecuencias de malabsorcin intestinal. El tx., especfico de la CBP se basa en administracin de esteroides y ac. ursodexosicolico (1315mg/Kg/24h), y en el trasplante heptico. COLANGITIS ESCLEROSANTE PRIMARIA (CEP): Inflamacin y fibrosis de los conductos biliares intra y
extrahepticos, los cuales se estenosan y obliteran, ocasionando cirrosis heptica. La CEP suele asociarse a colitis ulcerativa crnica inespecfica (40-80%),
LES y arttris reumatoide. Predomina en hombres. Presentan astenia progresiva, prurito e ictericia. Hay cirrosis, insuficiencia heptica e hipertensin
portal. El hallazgo caracterstico en la biopsia es la fibrosis concntrica periductal (en cascara de cebolla). No hay tratamiento especfico, se debe
considerar el trasplante heptico, ya que la supevivencia despus del dx., es de 12 aos DROGAS HEPATOTOXICAS: Arsenicos, vitamina A, cloruro de
vinilo, metotrexato, metildopa, nitrofurantoina, fenofibrato, sulfas, clotromoxaxol, amiodarona, paracetamol.
CASO CLINICO CIRROSIS
Hombre de 65 aos, con antecedentes de cirrosis heptica por
hepatopata crnica VHC (Child-Pugh 5). Habia sufrido un episodio de
hemorragia digestiva alta por sangrado de varices esofgicas, resuelto
con ligadura. Mientras pasea y de forma aguda presenta dolor intenso
a nivel hipogstrico. A la llegada del equipo de asistencia mdica
extrahospitalario el paciente ya est inconsciente (GCS 7), con signos
de mala perfusin perifrica, intensa palidez y situacin de shock. No
hay evidencia de hemorragia digestiva alta. Se procede a intubacin y
reposicin de volemia.
PREGUNTA
Cual de los siguientes sitios de sangrado en la cirrosis heptica es la
menos frecuente y muy grave?
RESPUESTA
a.- Hemo-retroperitoneo.
b.- Hemo-peritoneo.
c.- Varices esofagogastricas.
d.- Varices hemorroidales.
PREGUNTA
Considerando la respuesta anterior cuales signos y sntomas son
indicativos del sitio de sangrado?
RESPUESTA
a.- Dolor abdominal intenso de inicio brusco y distencin sbita.
b.- Distension abdominal sbita y signos de choque.
c.- Manifestaciones de hipovolemia y dolor abdominal.
d.- Signos de choque y dolor abdominal intenso.
CASO CLINICO IDIOPATICA
Paciente de sexo femenino de 61 aos de edad, con antecedentes de
diabetes mellitus tipo 2, hipertensin arterial y cirrosis heptica Child B
de etiologa no precisada, Consult por cuadro de una semana de
evolucin caracterizado por fiebre de predominio nocturno,
cuantificada hasta en 39C y precedida de un da de diarrea no
disentrica y autolimitada. Se evalu en Servicio de Urgencia, se
realizaron exmenes dentro de los cuales destacan: hemograma sin
leucocitosis ni desviacin a izquierda, pruebas de coagulacin,
electrolitos plasmticos y funcin renal normales. A los tres das
presenta espectoracion la cual se cultiva con resultado de L.
monocytogenes.
PREGUNTA
Cual es el tratamiento de primera eleccin ya que el paciente es
alrgico a la penicilinas, adems de la patologia de base?
RESPUESTA
a.- Cotrimoxazol.
b.- Eritromicina.
c.- Cloranfenicol.
d.- Tetraciclinas.
CASO CLINICO CIRROCIS HEPATICA ALCOHOLICA
Paciente de 38 aos ex-adicto a drogas por va parenteral sufre de
forma espontnea sin traumatismo previo dolor abdominal de
localizacin dorsal con irradiacin a ambos hipocondrios, hipotensin

CURSO ENARM CMN SIGLO XXI TEL: 36246001

arterial y anemizacin importante (5g/dl de hemoglobina), se realiza


TC abdominal ante la sospecha de rotura de anerurisma artico, y se
evidencia la existencia de rotura de tumoracin heptica
(hepatocarcinoma) en segmento VIII sobre un hgado cirrtico y
hemoperitoneo masivo. El paciente es intervenido quirrgicamente de
forma inmediata realizndose alcoholizacin de la lesin heptica y
sutura hemosttica.
PREGUNTA
Cul es el agente etiolgico ms probable.
RESPUESTA
a.- Virus tipo A
b.- Virus tipo B
c.- Virus tipo C
d.- Virus tipo D
CASO CLINICO
Masculino de 74 aos de edad, refiere cambios evacuatorios hace 2
meses, liquidas, amarillentas, cuatro por das, sin moco ni sangre, a la
semana se anexa al cuadro fiebre recurrente nocturna, no cuantificada
precedida de escalofros la cual atena con medio fsicos. A los 15 das
se suma alcuadro dolor abdominal de fuerte intensidad tipo clico,
Antecedente de esplenectoma comoresolucin a anemia hemoltica a
los 46 aos. Padre fallece a los 73 aos por CA. Gstrico. Hermanos: 12
Vivos, 3 fallecen por CA. Pulmn, CA. seo. y CA. Gstrico. Hijo
fallecido por CA. Testicular. Alcohol desde los 22 aos hasta los 46
aos los fines de semana tipo cerveza llegando a la embriaguez.
Tabquico: Desde los 14 aos hasta los 46 aos 10 paquetes por ao.
Refiere disminucin de 20 Kg aproximadamente en 2 meses. Cuello:
Adenopata cervical de 2 cm aproximadamente. Pulmn: Crepitantes
bilaterales difusos. Abdomen: Hepatometria 12/17/18 cm, bordes lisos
irregulares, Hemograma: WBC 9.7, HGB 10.3, PLT 386. NEUT: 73%,
LINF: 21,7%. BILIRRUBINA TOTAL 3,50 mg/dl. BILIRRUBINA DIRECTA:
2,12 mg/dl. FOSFATASA ALCALINA 200 UL. UROANALISIS: Bacterias
moderadas, leucocitos 4-8 x cpo. Trazas protenas, Pig. Biliares +++,
Bilirrubina +++. TGO: 198,3 U/L. TGP: 30,7 U/L. ALBUMINA 2,5;
PROTEINA C REACTIVA 36.
PREGUNTA
Considerando el cuadro, la evolucin y antecedentes, cual es la
condicin secundaria que presenta el paciente al padecimiento de
base?
RESPUESTA
a.- Cirrosis
b.- Hepatitis Viral
c.- Adenoma heptico
d.- Hiperplasia adenomatosa
PREGUNTA
Considerando la presencia de los adenomas hepticos, cual de las
siguientes afirmaciones cual es menos frecuente?
RESPUESTA
a.- Un adenoma hepatocelular es un tumor frecuente y no canceroso
del hgado.

Pharmed Solutions Institute

PGINA 58

MANUAL DE TRABAJO DEL CURSO ENARM CMN SIGLO XXI


b.- Los adenomas hepatocelulares inciden principalmenteen mujeres
en edad frtil.
c.- En general, no presenta sntomas, de modo que muchos de los
casos no se llegan a detectar.
d.- En casos moderados, una adenoma puede hacerse canceroso.
PREGUNTA
Relacionado al carcinoma hepatocelular, cual de las siguientes
afirmaciones es mas frecuente?
RESPUESTA
a.- Carcinoma hepatocelular en paciente con cirrosis.
b.- Carcinoma hepatocelular multifocal en paciente sin cirrosis.
c.- Un paciente con Sindrome de Bud-Chiari cronico con una lesion
nodular sospechosa.
d.- Mismo paciente con el uso de contraste, ayuda a delimitar mejor la
lesion.
CASO CLINICO CIRROCIS HEPATICA AUTOINMUNE
Mujer de 26 aos de edad. Acudi por un cuadro de una semana de
evolucin de dolor en hipocondrio derecho que se acompaa de
astenia, nuseas, coluria, acolia, y febrcula. En la exploracin fsica
slo destacaba la presencia de ictericia cutaneomucosa y un reborde
heptico doloroso a la palpacin a 4 cm de la arcada costal.
PREGUNTA
Cul es factor etiolgico ms probable?
RESPUESTA
a.- Viral.
b.- Alcohol.
c.- Oncologico.
d.- Autoinmune.
CASO CLINICO
Paciente masculino, 42 aos de edad, con antecedentes de
tabaquismo, alcohol espordico, hernioplasta umbilical; biopsia
heptica que reporto cirrosis heptica; familiar de primer grado con
hepatitis de causa desconocida, antecedente de varices esofgicas. Es
llevado por familiar a urgencias por cuadro agudo de confusin ms
euforia con posterior deterioro del sensorio de 24hrs de evolucin. Al
examen fsico se constata somnolencia alternada por episodios de
excitacin, confusin tmporo-espacial; flapping +; araas vasculares
en cara anterior de tronco, ascitis leve; edemas en miembros
inferiores. Laboratorio de ingreso: Bilirrubina Total: 3.27, B. Directa:
1.87, B. Indirecta: 1.4, FAL: 741, GGT: 504, GOT: 180, GPT : 148. TP:
44% Plaquetas: 65.000. Se solicita estudios para determinar etiologa
de la misma; Serologas virales VHC; VHB; HIV; VHA negativos
PREGUNTA
Qu grado de encefalopata heptica tiene este paciente?
RESPUESTA
a.- Grado I
b.- Grado II
c.- Grado III
d.- Grado IV
PREGUNTA
Cul es el compromiso de este paciente y su sobrevida a 2 aos?
RESPUESTA
a.- Enfermedad bien compensada, sobrevida 85%
b.- Compromiso funcional significativo, sobrevida 60%
c.- Enfermedad descompensada, sobrevida 35 %

CURSO ENARM CMN SIGLO XXI TEL: 36246001

d.- Dao irreversible, sobrevida 20%


PREGUNTA
Debido a que el paciente ya tiene ascitis Qu presin portal
esperaramos encontrar?
RESPUESTA
a.- <5mmHg
b.- >5mmHg
c.- >10mmHg
d.- >12mmHg
CASO CLINICO
Paciente varn, de 72 aos de edad, con antecedentes personales de
cirrosis heptica de origen mixto, alcohol y VHB, con clnica de
hipertensin portal (varices esofgicas), que ingresa en el servicio de
urgencias de nuestro hospital por presentar cuadro de ascitis. A su
ingreso el paciente refiere astenia, dolor ocasional en epigastrio,
sequedad de boca y equimosis fcil. La FC 92 lat/min, y la TA 160/90
mmHg. Su estado general era regular, con ictericia de piel y mucosas.
El abdomen presentaba circulacin colateral con repermeabilizacin
de vena umbilical y ascitis libre. En las extremidades presentaba
edemas III/IV bilaterales. En la analtica de rutina destacaba una
glucemia de 228 mg/dl y una urea de 64 mg/dl, Cr 1.3mg/dl. La
hemoglobina era de 15,4 g/dl, el hematocrito, del 45,1%, tena 4.600
leucocitos, 85.000 plaquetas por mm3, y una actividad de protrombina
del 22%.
PREGUNTA
Cul es la conducta teraputica ms adecuada a seguir?
RESPUESTA
a.- Paracentesis, espironolactona, restriccin de lquidos
b.- Espironolactona, restriccin de lquidos
c.- Vigilancia y restriccin de lquidos
d.- Paracentesis, furosemida
PREGUNTA
Cul es la medida ms adecuada para prevenirle un sangrado por sus
varices esofgicas?
RESPUESTA
a.- Bajar 25% la frecuencia cardiaca basal con propanolol
b.- Bajar el 25% la T/A con metoprolol
c.- Bajar el 15% la frecuencia cardiaca basal con propanolol
d.- Bajar el 15% la T/A con metoprolol
PREGUNTA
Cul es su probabilidad de vida a 5 aos para este paciente?
RESPUESTA
a.- <10%
b.- <50%
c.- >50%
d.- >70%
PREGUNTA
El paciente desarrollo en una siguiente vez peritonitis bacteriana
espontanea. Cul sera la conducta teraputica ms adecuada?
RESPUESTA
a.- Amikacina 10mg/kg c/24hrs
b.- Cefotaxima 2g c/8hrs
c.- Ceftriaxona 1g c/12hrs
d.- Vancomicina 1g c/12hrs

Pharmed Solutions Institute

PGINA 59

MANUAL DE TRABAJO DEL CURSO ENARM CMN SIGLO XXI


LEUCEMIAS
CIENCIAS BASICAS: Es un grupo de neoplasias malignas de las clulas precursoras hemticas, las cuales derivan de la celula multipotente (pluripotencial),
del progenitor mieloide (granulocitica) derivan, eritrocitos, megacariocitos, monocitos neutrfilos y del citoplasma plaquetas. Del progenitor linfoide
derivan los linfocitos T, B clulas NK. Se pueden clasificar en agudas y crnicas. Las primeras frecuentemente involucran a las clulas inmaduras o blastos,
tienen una rpida evolucin, son clulas no diferenciadas, es decir no funcionales. Las crnicas se caracterizan por la proliferacin de clulas maduras,
funcionales, crecimiento lento pero de mayor numero de celulas. SALUD PBLICA: Constituyen el 3% de las neoplasias de todo el mundo. La leucemia
mieloblastica aguda, es la ms comn en el adulto (80%). Edad promedio es de 65 aos. Solo un 10% en nios menores de 10 aos. Es responsable del
1.2% de todas las muertes en EU. PATOGENIA: En las agudas hay activacin de oncogenes MLL, Myc, ABL, BCL-2. DIAGNOSTICO: En general se
manifiestan con prdida de peso, astenia, adinamia, linfadenopata, hepatoesplenomegalia, palidez, petequias, infecciones frecuentes. En las agudas se
puede presentar clnicamente Sx. hemorrgico; trmbocitopenia, coagulopatia por consumo, Sx. infiltrativo; crecimiento de ganglios, bazo e hgado, Sx.
anmico; invasin tumoral de la MO. Se puede hacer uso de BH, aspirado y/o biopsia de MO y citogentica. TRATAMIENTO: Alquilantes, antimetabolitos,
sustancias que se fijan al ADN, corticoides. LEUCEMIA MIELOIDE AGUDA (LMA): Neoplasia de clulas precursoras mieloides de mdula sea en el que
clulas poco diferenciadas se acumulan en la mdula sea, circulacin y rganos perifricos. Los signos y
FAB PARA LEUCEMIAS MILOBLASTICAS AGUDAS
sntomas ocurren debido a la ausencia de clulas maduras normales producidas por la medula sea,
M0
LMA Minimamente diferenciada
incluyendo: granulocitos (sensibilidad a infecciones), plaquetas (susceptibilidad a sangrado) y eritrocitos.
M1
LMA Sin maduracin
Afecta tanto adultos como nios. Factores de riesgo; cloranfenicol, tabaco, benceno, radiacin previa,
M2
LMA Con maduracin
anomalas genticas, (trisoma 21, anemia de Fanconi, ataxia-telangiectasia), infecciones (HTLV-1), otras
M3
LMA Promieloctica (Riesgo de CID)
enfermedades hematolgicas (anemia aplasica, sndromes mielodisplasicos y familiares). CLASIFICACION:
M4
LMA Mielomonoctica
El grupo cooperativo Franco-Americano-Britnico (FAB) ha subdividido las LMA en ocho subtipos basados
M5
LMA Monoctica (Afeccion a SNC)
M6
LMA Eritroleucemia (Afeccion a SNC)
en la morfologa y la citoqumica. La OMS clasifica la leucemias mieloblasticas agudas en: 1.- LMA con
M7
LMA Megacarioctica
anormalidades genticas recurrentes. 2.- LMA con caractersticas mielodisplasicas. 3.- LMA relacionada a
tratamiento previo. 4.- LMA no especificadas en otra clasificacin. DIAGNOSTICO: Clnico: la mayora inicia con sntomas relacionados con la
pancitopenia, esto es, fatiga (>3meses), y palidez a causa de anemia o infecciones provocadas por la neutropenia y/o hemorragias secundaras a la
trombocitopenia epistaxis, hemorragia conjuntival, petequias, algunos refieren dolores seos en esternn y huesos largos. La presencia de fiebre se debe
por lo general a un proceso infeccioso activo relacionado con la neutropenia (<1000), entre los patgenos ms frecuentes E. coli, Pseudomonas y hongos
oportunistas, Candida, Aspergillus, Histoplasma. Los rganos mas comnmente afectados son: piel, SNC, hgado, timo y testculos. Despus de la
sospecha clnica se realiza: Biopsia y aspirado de medula sea; por lo general en cresta iliaca posterosuperior o columna vertebral, si hay alguna
contraindicacin el sitio de eleccin es el esternn. A las muestra se le realiza tincin de Wright-Giemsa y la de hierro, a la biopsia la de mieloperoxidasa,
sudan negro, esterasa inespecfica y hematoxilina-eosina, cuando son negativas se debe pensar en una LLA, no en una LMA. Microscpicamente es
posible observar los cuerpos de Aver en el citoplasma de clulas malignas, tanto en M2, M3, estos cuerpos son patognomnicos de los mieloblastos y
son de color rosa bermelln. Dx. Con 20% de linfoblastos indiferenciados, con marcadores de superficie de clulas mieloides. En la LMA, es necsaria la
presencia de 20% de blastos en medula osea para su diagnstico. Se pueden hacer estudios citogenticos, y de inmunofenotipo, BH, puede haber
leucocitosis extrema >80,000 (se puede presentar hemorragia cerebral por esta razn, por lo que hay que hidratar, alcalinizar la orina, alopurinol,
leucoferesis), el 80% presenta trombocitopenia de 100,00/mm3, anemia normocitica normocromica, neutrfilos de 15,000/mm3, 95% mieloblastos
circulantes en sangre. TRATAMIENTO: El objetivo es la remisin y consiste en dos fases: primera es la induccin se usa una antracilina ms citarabina,
(remisin hasta 80%). La segunda fase es de consolidacin, mismo esquema de la primera, si el paciente es joven o tienen factores favorables, en esta
fase se puede realizar trasplante de medula sea. En la M3, nica en que se usa adems la droga ATRA, con el objetivo de evitar la CID (por la liberacin
de procoagulantes de sus grnulos citoplasmticos). Transfundir eritrocitos y plaquetas. PRONOSTICO: Mal pronstico: Edad >60 aos, resistencia a
mltiples frmacos, historia de enfermedades hematolgicas, sndrome mielodisplasico o tratamientos previos, leucocitosis mayor 20,000/mm3,
variantes M0, M5, M6, M7, anormalidades cromosmicas. CUADRO TIPICO: 40 aos, dolor farngeo, gingivorragias (trombocitopenia), anemia, leucocitos
50,000, en frotis se observan blastos. LEUCEMIA MIELOIDE CRONICA (LMC): Caracterizada por esplenomegalia y produccin incrementada en el nmero
de granulocitos, debilidad, hiporexia, perdida de peso, molestias abdominales, hemorragias, purpura y fiebre en <25%, en la fase de crisis blastica es la
de peor pronstico. Mas de 90% presenta translocacin entre los cromosomas 9 y 22. Ocasionalmente se pueden encontrar leucocitos elevados, con
incremento de granulocitos y precursores de los mielocitos, las bandas y formas maduras predominan. En un estudio de rutina podemos encontrar
>30,000 leucocitos, la cuenta de basfilos podra estar entre 10-15%, plaquetas normales o ligeramente aumentadas (en la mitad de los casos). FASE
CRONICA: 2-4 aos, la fase acelerada es marcada por anemia desproporcionada con la actividad de la enfermedad o el tratamiento. Supervivencia en una
crisis blastica es menor a 4 meses. Tx. Para prolongar la vida Bisulfan VO 4-10mg/dia. LEUCEMIA LINFOCITICA AGUDA (LLC): Es mas comn en nios, las
tasas de remisin son hasta de 90%, en adultos es mas agresiva, el 80% se origina de clulas B. Diagnostico; clnica sangrado (30%), fiebre (25%) por mas
de 3 meses, dolor oseo, linfadenopatia, hepatoesplenomegalia. Tx. Vincristna, prednisona, antraciclinas. LEUCEMIA LINFOCITICA CRONICA (LLC):
Caracterizada por la acumulacin de linfocitos (B en 95%), no funcionales o incompetentes de origen monoclonal, por lo que es la leucemia mas comn.
Se considera que la LLC, un estadio del linfoma linfocitico de cel. Pequeas maduras. Se presenta en adultos entre 50 y 70 aos de edad con predominio
en varones. Es la leucemia crnica ms comn y constituye 30% de todas las leucemias. Se considera una enfermedad de adultos mayores. En la
actualidad no existen factores de riesgo, se cree que hay un patrn familiar. DIAGNOSTICO: Asintomticos, descubierta a veces en exmenes de rutina,
solo el 10% presenta los clsicos sntomas B (prdida de peso, fiebre y sudoracin nocturna) de los linfomas, al progresar se inflaman los ganglios
linfticos, hgado y bazo. Alrededor de 90% de los pacientes presenta linfadenopata. Laboratorio: BH con diferencial, citologa de flujo y frotis de sangre
perifrica, no es necesaria la biopsia de MO (si se hace; presenta hiopercelularidad de linfocitos). Para hacer el diagnostico el paciente debe presentar
cuenta de linfocitos B absolutos mayor a 5,000/microL. TRATAMIENTO: La mayora son asintomticos y sin evolucin del trastorno. Se usan agentes
alquilantes y terapia de anticuerpos monoclonales. Quimioterapia fludarabina Estadios de LLC: A: Ausencia de anemia y trombocitopenia y menos de 3
reas invadidas. B: Ausencia de anemia y trombocitopenia y ms de 3 reas linfoides invadidas. C: Anemia <10g/dl y/o trombocitopenia <100,000.
CASO CLINICO
Masculino de 65 aos de edad, trabajador en unas minas ubicadas en
Veracruz, refiere que en algunas ocasiones sinti ligero dolor en el lado
izquierdo del abdomen cuando realizaba algunos esfuerzos en su
trabajo, acude al consultorio mdico por presentar fiebre de 39,
gingivorragias y adems por presentar unos puntos rojos diseminados
en todo el cuerpo, refiri cansancio de aproximadamente 4 meses
atras y marcada debilidad. EF: T 39C, se constat petequias

CURSO ENARM CMN SIGLO XXI TEL: 36246001

diseminadas por todo el cuerpo especficamente en las extremidades,


ganglios sobre todo submaxilares derecho e izquierdo y toda la cadena
lateral del cuello de forma bilateral estaban inflamados, no dolorosos a
la palpacin, no formaban paquetes y la consistencia era fibroelstica.
Axilas: Se palparon ganglios en ambas axilas derecha e izquierda con
iguales caractersticas a los del cuello. Abdomen: A la palpacin
superficial y profunda se constat una tumoracin en el hipocondrio
izquierdo y otra en el hipocondrio derecho que se correspondi con el

Pharmed Solutions Institute

PGINA 60

MANUAL DE TRABAJO DEL CURSO ENARM CMN SIGLO XXI


bazo e hgado respectivamente, el paciente refiri ligera molestia pero
no dolor durante las maniobras de palpacin. Laboratorio: Hb 6 g /L,
Leucos 60,000, presencia de incontables clulas inmaduras en la
sangre (con desviacin a la izquierda), neutropenia, Trombocitopenia
83,000.
PREGUNTA
Cul es el diagnstico ms probable para este caso?
RESPUESTA
a.- Leucemia linfoctica aguda
b.- Leucemia mieloide aguda
c.- Leucemia mieloide crnica
d.- Leucemia linfoctica crnica
PREGUNTA
Cul de las siguientes medidas es menos efectiva para prevenir una
hemorragia cerebral en este paciente por leucostasis extrema?
RESPUESTA
a.- Hidratacin
b.- Alcalinizacin de orina
c.- Citarabina
d.- Leucoferesis
RESPUESTA
El paciente inicia a presentar datos de CID. Cul es el tipo ms
probable que este presentando este paciente?
RESPUESTA
a.- M1
b.- M3
c.- M5
d.- M7
PREGUNTA
Con que numero de plaquetas se hace transfusin de las mismas?
RESPUESTA
a.- <90,000
b.- <60,000
c.- <50,000
d.- <10,000
CASO CLINICO
Varon de 60 aos de edad se presenta con un medico familiar para un
examen fsico completo. Su ltima revisin mdica fue hace 2 aos,
por un cuadro de gripe. Explica que su salud ha estado relativamente
bien y que ha tenido muchos resfriado pero que en la actualidad, no
tiene problemas mdicos. Cuando se le interroga con mayor detalle
reconoce haber estado cada dia mas cansado durante los ltimos 6
meses y haber perdido alrededor de 9 Kg de peso durante los 2 aos
pasados. Insiste en que esta sano y que necesita perder peso, pero
niega algun rgimen de ejercicio y restricciones especificas en la dieta.
Los signos vitales estn dentro de lmites normales. El examen revela
ganglios linfticos palpables en cuello y axilas. Niega dolor a la
palpacin; los ganglios varial de 0.5 a 1 cm de dimetro. A la palpacin
de abdomen se palpa hgado 3 cm, por debajo de reborde costal, QS
con valores dentro de parmetros normales. BH, leucocitos
25,200/mm3, con 90% de linfocitos, Hto. 38%, Hb 10mg/dl.
PREGUNTA
Cul es el diagnostico mas probable para este caso?
RESPUESTA
a.- Leucemia linfoctica aguda
b.- Leucemia mieloide aguda

CURSO ENARM CMN SIGLO XXI TEL: 36246001

c.- Leucemia mieloide crnica


d.- Leucemia linfoctica crnica
PREGUNTA
Al hacer un frotis. Cules clulas serian las mas afectadas en este
caso?
RESPUESTA
a.- Celulas NK
b.- Celulas B
d.- Monocitos
d.- Plaquetas

PREGUNTA
Cules marcadores solicitaramos para apoyar nuestro diagnostico?
RESPUESTA
a.- CD20
b.- CD19
c.- CD8
d.- CD 50
PREGUNTA
Cul es el pronstico para este paciente si es diagnoticado ya en
etapa avanzada?
RESPUESTA
a.- 5 aos
b.- 10 aos
c.- 15 aos
d.- 2 aos
CASO CLINICO
Varon de 57 aos de edad es remitido con el hematlogo para un
estudio detallado de anemia grave. La concentracin de Hb es de
8.2mg/dl, pero los estudios clnicos no revelan signos de sangrado
oculto. Durante las semanas anteriores ha observado sangre en el
cepillo dental, casi a diario y se queja de presentar moretones con
mayor facilidad que antes. Se ha fatiogado cada vez mas y ha
restringido las horas que pasa en la empresa de tratamiento de
desperdicios que posee, porque a menudo tiene problemas para
respirar durante las horas de trabajo. A parte de la Hb baja, los
resultados de laboratorio importante son leucocitos de 3200/mm3,
recuento de reticulocitos de 0.8% y recuento plaquetario de
30,000/mm3. El frotis de sangre muestra multiples eritrocitos
multinucleados y un nmero mayor que lo normal de clulas
inmaduras.
PREGUNTA
Cual es el diagnostico mas probable para este caso?
RESPUESTA
a.- Leucemia linfoctica aguda
b.- Leucemia mieloide aguda
c.- Leucemia mieloide crnica
d.- Leucemia linfoctica crnica
PREGUNTA
Cul es la conducta teraputica mas adecuada a seguir?
RESPUESTA
a.- Fludarabina
b.- Vincristina, prednisona
c.- Citarabina, antraciclina
d.- Bisulfan

Pharmed Solutions Institute

PGINA 61

MANUAL DE TRABAJO DEL CURSO ENARM CMN SIGLO XXI


LINFOMAS
CIENCIAS BASICAS: Todas las neoplasias linfoides son malignas y derivan de una clula que ha sufrido mutacin maligna son monoclonales, tambin
alteran la inmunidad. 80-85%, tiene su origen en clulas B, los derivados de NK o histiociticas son raros. Se clasifican en Linfoma de Hodgking (EH) y
linfomas no Hodgkin (LNH). SALUD PUBLICA: LH: Constituye el 10% de todos los linfomas. Ms frecuente de 25 aos y >60 aos. LNH constituyen el 3%
de todas las neoplasias y son ms frecuentes que Los LH (60% de los linfomas en adultos, edad promedio 64 aos) El subtipo histolgico ms comn es el
difuso B de cel. Grande (LDGBC)s con incidencia en individuos infectados por el VIH de ms de 100 veces la incidencia en la poblacin en geral (25%),
seguido del linfoma folicular. LINFOMA DE HODGKIN (LH): Origen
ESTADIO I
Afecta una sola regin de ganglios linfticos
I
en cel. B activados del centro germinal Tienen una extensin
Afecta a un solo rgano extralinftico
IE
ordenada y localizada, raramente afecta ganglios mesentricos, rara
ESTADIO II
Afecta 2 o mas reas ganglionares en el mismo lado del diafragma
II
vez es extraganglionar. El de celularidad mixta es el subtipo
Afecta a un solo rgano extralinftico y sus ganglios regionales al
IIE
mismo lado del diafragma
histolgico ms frecuente en LH extranodales, mientras que el
ESTADIO III
Afecta reas ganglionares a ambos lados del diafragma
subtipo esclerosis nodular es predominante en los de afectacin
Puede acompaarse de afeccin esplnica
IIIS
nodal. PATOGENIA: Se sospecha del VEB como agente de
rganos extralinftico localizados
IIIE
transformacin maligna, pacientes con mononucleosis infecciosa,
ESTADIO IV
Afecta de forma difusa a uno o ms rganos extralinftico (hgado,
tienen mayor riesgo, se reconoce el genoma de VEB en 70%,
pulmn, medula sea) con o sin involucro ganglionar
tambin comn en casos de inmunodeficiencia. Caractersticas;
Agregar A cuando no se presentan sntomas y B si la persona refiere sntomas B
extensin ordenada, localizado, rara vez afecta ganglios
mesentricos, rara extensin extraganglionar, muy comn dolor en ganglios linfticos, despus del consumo de alcohol (<10%), en la variante esclerosis
nodular DIAGNSTICO: Tpico escenario paciente joven, masculino, linfadenopatas perifricas no dolorosas (70%), especialmente en regin cervical (6080%) o masa mediastinal. Cuadro clnico: Sntomas B: prdida de peso inexplicable de ms de 10%, en los ltimos 6 meses; fiebre inexplicable >38C, de
duracin mayor a 2 semanas; sudoracin nocturna profusa. Prurito en 10-15% de los casos, no es sntoma B. Fiebre Pel- Ebstein; fiebre cclica que dura
de 3-5 das, seguida de un periodo equivalente sin fiebre. Biopsia excisional ganglionar cel de Reed Sternberg 2 o ms ncleos con nuclolo prominente.
Inmunohistoquimica: positivos para CD15 y 30, nos ayuda a diferenciar entre LNH. PCR en ganglio detecta VEB en 60-80%. Tac de trax, abdomen y
pelvis, PET. ESTADIFICACION: La clasificacin mas usada es la Ann Arbor- Costwold. TRATAMIENTO: Radioterapia en estadios tempranos sin factores
desfavorables. Quimioterapia; tratamiento de eleccin para casos localizados (IA y IIA), 2 ciclos de quimioterapia con ABVD (adriamicina, vinplastina,
bleomicina, y decarbazina)+ radioterapia en caso de urgencias avanzadas ( sx. de vena cava superior, obstruccin de va area). Estadios III y IV quimio
combinada. El caso clnico caracterstico de este linfoma es un paciente masculino, en buen estado general, con linfadenopatia no dolorosa que al
realizarle una biopsia de ganglio se observan Cel. De Red- Stenmberg (que son binucleadas o multinucleadas), mas sntomas B. PRONOSTICO: Factores de
mal pronstico: Hb <11mg/dl, albmina <4g/dl, masculino, edad >45 aos, adenopata mediastinal, estadio IV, leucocitosis >15,000, linfocitosis <500,
histologa del tipo celularidad mixta, presencia de sntomas B. Despus de 3 aos sin enfermedad sobrevida >53%. Tasa de curacin 65%, para pacientes
con trastornos avanzados. LINFOMA NO HODGKING (LNH): Tumores slidos malignos de los tej. Linfoides, suelen provenir de cel. B (80%) Diseminacin
menos previsible, generalmente diseminada al realizar el dx., suele afectar ganglios mesentricos, frecuentemente afecta tej. extraganglionar. Entre los
factores de riesgo estn; inmunodefoiciencias (LNH sistmico puede afectar el miocardio, particularmente en pacientes inmunocomprometidos),
tratamientos con radioterapia o quimioterapia, enfermedades autoinmunes y enfermedades previas. Los linfomas no hodgkiniano son la causa ms
frecuente de linfoma de cabeza y cuello. Se clasifican en bajo grado (transformacin en agresivos, no curables, edad avanzada, sobrevida larga a 8 aos)
y agresivos: potencialmente curables, sobrevida corta si no remiten; cuadro clnico, sntomas B crecientes. Dentro de los de bajo grado; linfoma folicular,
sus clulas neoplsicas se parecen a cel. B normales, adenopatas indoloras generalizadas, incurables. LNH de bajo grado Linfoma de la zona marginal
(MALToma= tumores de los tej. linfoides asociados a mucosas) se origina, en bazo, ganglios y tej. extraganglionares, afectados por procesos inflamatorios
autoinmunitarios ( S. Sjogren, tiroiditis de Hashimoto) o no infecciosos (asociado a H. pylori). LNH agresivo en general, debilidad, fatiga, prdida de peso,
50% prdida de peso, hepato y esplenomegalia, hiperviscosidad (que genera mareos, sordera, cefalea, formacin de complejos y crioglobulinemia).
Linfoma difuso de cel. B grandes; aparicin de masas de rpido crecimiento, afecta una sola regin ganglionar o extraganglionar, primera manifestacin
tubo digestivo, piel, hueso cerebro. LNH agresivo Linfoma de Burkit; tumor de cel. B relativamente maduras, translocacin del gen c- myc en cromosoma
8, en tej. extraganglionar, el LB africano endmico; masa en mandbula, vsceras abdominales (riones, ovarios), agresivo pero responde a quimioterapia
fcil. Macroglobulinemia de Waldestrom; es una neoplasia de las clulas B, mas comn en adultos de edad avanzada, es incurable, las clulas tumorales
liberan IgM, lo que ocasiona hiperviscosidad, por el aumento de las inmunoglobulinas lo que causa alteraciones visules, cefaleas, mareos, sordera,
hemorragia, crioglobulinemia (precipitacin de globulinas por disminucin de la temperatura, se asocia a fenmeno de Ryanaud). Son caractersticas
para su diagnostico clulas con inclusiones PAS positivas, llamados cuepors de Russell- Dutcher. El tratamiento de la hiperviscocidad es la plamaferesis.
DIAGNOSTICO: Sospecha clnica, biopsia de ndulo u rgano involucrado. Estos se diagnostican por resultados de laboratorios anormales; citopenias,
hipercalciemia, hiperuricemia, proteinemia o LDH elevada) TRATAMIENTO: LNH; linfoma folicular R-CHOP (rituximab, ciclofosfamida, hidrixidaunomicina,
vincristina y prednisona). LNH agresivos; R-CHOP ms de 50% de curacin. En todos los linfomas en casos refractarios trasplante de medula sea.
PRONSTICO: LNH agresivos; tasa de remisin de 60-80%, para estadios II al IV. PREVENCION: Hombre almacenar espermas, mujeres no embarazarse en
2 aos posteriores.
CASO CLINICO
Una paciente de 27 aos, del sexo femenino, se present en la Sala de
Emergencias con palpitaciones de inicio sbito y disconfort torcico. La
presin arterial (PA) era de 68 x 40 mmHg y la frecuencia cardaca (FC)
era de 184 lpm. Al examen fsico, la paciente presentaba aumento de
volumen de la mama izquierda y masa abdominal difusa. El
electrocardiograma (ECG) de 12 derivaciones realizado en la admisin
revel taquicardia de complejo ensanchado con estndar de bloqueo
de ramo izquierdo y concordancia negativa en los electrodos
precordiales. Con base en la presentacin inicial, el diagnstico de
taquicardia ventricular monomrfica fue establecido y la paciente fue
sometida a cardioversin elctrica para el ritmo sinusal normal. El ECG
fue repetido y mostr ritmo sinusal normal, sin alteraciones del
segmento ST. La paciente recibi una infusin intravenosa de
amiodarona y entonces fue admitida en la Unidad de Terapia Intensiva

CURSO ENARM CMN SIGLO XXI TEL: 36246001

(UTI). La investigacin fue negativa para isquemia miocrdica o


embolia pulmonar. Una ecocardiografa transtorcica (ETT) revel
funciones ventriculares izquierda y derecha normales, derrame
pericrdico posterior difuso y engrosamiento del miocardio basalseptal, que no estaba presente en examen anterior realizado un ao
antes para investigacin de disnea durante actividad fsica.
PREGUNTA
Cul de las siguientes condiciones patolgicas previas es ms probable
para el desarrollo del LNH (LDGCB)?
RESPUESTA
a.- Carcinoma metastasico.
b.- Carcinoma pulmonar.
c.- Antecedente de VIH.
d.- Lupus eritematoso sistmico discoide.

Pharmed Solutions Institute

PGINA 62

MANUAL DE TRABAJO DEL CURSO ENARM CMN SIGLO XXI


PREGUNTA
En pacientes con LNH, cual es la presentacin mas comn de
compromiso cardaco?
RESPUESTA
a.-Dolor torcico
B.-Insuficiencia cardaca congestiva
C.-Derrame pericrdico
D.-Bloqueo atrioventricular
PREGUNTA
Cul es el manejo inicial de la taquicardia ventricular en este caso?
RESPUESTA
a.- Cardioversin elctrica
b.- Verapamilo
c.- Amiodarona
d.- Desfibrilacin
PREGUNTA
Cul es el medio diagnostico mas adecuado?
RESPUESTA
a.- Biopsia.
b.- IRM-f
c.- Frotis perifrico.
d.- BH.
CASO CLINICO LINFOMA DE HODGKIN EN ADENOIDES
Varn de 39 aos con antecedentes de amigdalectoma en infancia, sin
hbitos txicos referidos y abuela materna con leucemia linftica
crnica. El paciente es visto en consulta por insuficiencia respiratoria
nasal y ronquido asociado a fatiga diurna, ms acentuada en el ltimo
ao, sin otra clnica asociada. Es trabajador a turnos y conductor
profesional por lo que se solicita interconsulta en la Unidad de
Trastornos Respiratorios del Sueo, con diagnstico de trastorno de
sueo leve. En la exploracin se visualiza, por rinoscopia anterior, una
desviacin septal derecha con hipertrofia turbinal compensatoria. En
nasofibroscopia, se identifica una masa en cavum, compatible con
hipertrofia adenoidea. La rinomanometra demuestra mayor
resistencia area en fosa nasal derecha, sin gran mejora tras
vasoconstrictor, siendo el resto de la exploracin ORL normal. Dado el
agravamiento de la clnica en el ltimo ao y el tamao de la masa en
cavum, se realiza biopsia de la misma, informada como hiperplasia
folicular linfoide, y RM, objetivndose una masa de 2,3cm de dimetro
compatible con hipertrofia de tejido adenoideo. Teniendo en cuenta la
clnica referida y la exploracin realizada, el paciente es intervenido de
septoplastia con turbinectoma y adenoidectoma. El informe AP es de
linfoma de Hodgkin (LH) clsico rico en linfocitos, sin detectar
presencia de virus Epstein Barr (VEB).
PREGUNTA
Cul es la manifestacin clsica de Linfoma Hodgking?
RESPESTA
a.- Paciente joven, masculino, linfadenopatas perifricas
b.- Adulto mayor, masculino, afeccin extraganglionar
c.- Paciente joven, femenino, linfadenopatas perifricas
d.- Paciente joven, masculino, afeccin extraganglionar
PREGUNTA
Qu utilidad tiene hacer estudio inmunohistoquimico a este
paciente?
RESPUESTA

CURSO ENARM CMN SIGLO XXI TEL: 36246001

a.- Decidir tratamiento


b.- Diferenciar LNH bajo grado de un LNH agresivo
c.- Establecer un diagnstico diferencial con el linfoma no hodgkiniano
d.- Saber si es un tumor en cabeza y cuello
PREGUNTA
El paciente descrito presenta un LH, sin sntomas B, localizado en
adenoides, sin afectacin de otras regiones ni adenopatas asociadas.
En qu estadio de la clasificacin de Ann Arbor- Costwold se
encuentra?
RESPUESTA
a.- Estadio IV
b.- Estadio III E-A
c.- Estadio IE-A
d.- Estadio IE-B
CASO CLINICO
Femenino de 65 aos que acudi a nuestra consulta por referir,
aparicin de masa en cara interna de brazo izquierdo con crecimiento
rpido de hace 2-3 meses y escasamente doloroso, refiere 4 das con
fiebre y los mismos sin fiebre, en el ltimo mes. Astenia, adinamia y
prdida de peso. Relataba que haca unos nueve aos, haba
presentado una tumoracin en la misma localizacin del tamao de
una nuez por la que no consult creyendo que se trataba de un
quiste sebceo; sta desapareci espontneamente. Sus antecedentes
personales incluan una tonsilectoma, apendicectoma; siendo
fumadora de 10 cigarrillos/da; antecedente de mononucleosis,
menarquia a los 15 aos, con ciclos regulares: 28/3-5. G1P1A0 y
portadora de DIU. Como antecedentes familiares, slo cabe destacar
padre finado a los 47 aos debido a un cncer de pncreas. En la
exploracin fsica se apreciaba una adenopata axilar izquierda de unos
2 centmetros de dimetro de contornos lisos y redondea dos, fija en
profundidad; otra adenopata supraclavicular izquierda de unos 0,5
centmetros de dimetro, mvil, no adherida y masa en cara interna
del tercio proximal del brazo izquierdo dura, violcea.
PREGUNTA
Cul es el diagnstico ms probable para este caso?
RESPUESTA
a.- Cncer de pncreas
b.- Linfoma de Hodking
c.- Linfoma no Hodking
d.- Leucemia mielocitica aguda
PREGUNTA
Cul es la tasa de curacin en un paciente con estadio avanzado?
RESPUESTA
a.- 25%
b.- 45%
c.- 65%
d.- 85%
PREGUNTA
En qu estadio de la clasificacin de Ann Arbor se encuentra este
paciente?
RESPUESTA
a.- I
b.- II
c.- III
d.- IV

Pharmed Solutions Institute

PGINA 63

MANUAL DE TRABAJO DEL CURSO ENARM CMN SIGLO XXI


FIEBRE REUMATICA (FRA):
CIENCIAS BASICAS: Enfermedad inflamatoria ocurre como secuela alejada de una infeccin por estreptococo beta hemoltico grupo A, habitualmente
faringoamigdalitis o escarlatina. Su nombre proviene del dao articular, pero los daos ms importantes se producen en corazn. SALUD PBLICA: La
FRA aparece de 0.5-5% de pacientes que hacen faringoamigdalitis estreptoccica. Aparece especialmente entre 5-15 aos. Mortalidad <1% por carditis
grave. Aparicin favorecida por factores climticos (hmedo, frio) y socioeconmico (hacinamiento, pobreza). PATOGENIA: Existe una relacin entre
faringoamigdalitis y estreptococo grupo A y la fiebre reumtica que se sustentan en: 1. Relacin epidemiolgica sustentada entre estas infecciones y la
aparicin de casos de FRA. 2. Comprobacin inmunolgica de infeccin estreptoccica (antiestreptolisinas en suero). 3. Episodios primarios y secundarios
pueden prevenirse mediante antibitico precoz para la infeccin. Debido a que las toxinas del estreptococo producen inmunidad cruzada.
DIAGNOSTICO: Clnico; Cuadro febril insidioso, con aparicin de artritis (70-80%), es una poliartritis migratoria de grandes articulaciones. Carditis (4060%), generalmente dentro de las primeras semanas del episodio, si no hay carditis (infiltrado inflamatorio que afecta miocardio, endocardio y
pericardio) en los primeros meses el pronstico es muy bueno; puede manifestarse por soplos de insuficiencia mitral (75-80%) o aortica (30%), frote
pericardico, galope, cardiomegalia, congestin pulmonar, a veces son poso sintomticas. Corea de Syndenham (10-20%) que son movimientos
desordenados, descoordinados, inesperados e involuntarios. Eritema marginado y ndulos subcutneos de Meynet (<2-3%). Puede haber compromiso
del estado gnarl., artralgias. Laboratorio: Antiestreptolisinas (ASO) elevadas (desde los primeros das y duran hasta 6-8 semanas). VSG Y Protena C
reactiva elevadas. Se puede hacer cultivo farngeo para estreptococo. Histolgico: Los cuerpos de Aschoff son una confluencia de monocitos y
macrfagos rodeados por fibrosis se consideran tpicos de FRA. En ECG puede haber bloqueo AV simple, fenmeno no diagnostico de carditis. La
ecografa cardiaca es til para el diagnostico y el tratamiento de la carditis (valoracin de dao de vlvulas, y de insuficiencia cardiaca). CRITERIOS
DIAGNOSTICOS de Jones: Criterios mayores; poliartritis, carditis, corea, eritema marginado, ndulos subcutneos. Criterios menores; fiebre, artralgias,
FRA previa, VSG y PCR elevadas, prolongacin de PR. El diagnostico se hace con 2 criterios mayores 1 mayor y 2 menores, junto con alguna evidencia de
infeccin estreptoccica, tal como elevacin de las ASO o antecedente de escarlatina, cultivo farngeo positivo o prueba rpida de antgeno
estreptoccico positivo. TRATAMIENTO: Se realiza en 2 partes del episodio actual y prevencin de la recurrencia. Reposo en caso, de artritis o carditis
hasta recuperacin completa. El medicamento de primera eleccin para la artritis es la aspirina, segunda eleccin naproxen. En la carditis se pueden
utilizar corticoides. Prevencin primaria: Tratamiento adecuado a las faringoamigdalitis estreptoccicas (su diagnostico se basa marcadamente en
cuadros febriles y exudativos), con penicilina benzatinica 1,200,000U dosis nica o eritromicina VO por 10dias (en caso de alergia a penicilinas).
Prevencin secundaria: Uso prolongado de penicilina benzatinica 1,200,000U cada 4 semanas o sulfadiazina 1gr/dia (en caso de alergia a penicilinas). La
duracin de profilaxis depende de la gravedad y edad del paciente. El periodo mnimo recomendado en paciente con antecedente de FRA, sin carditis es
al menos de 5 aos, pero directrices reciente refieren 10 aos o hasta 21 aos de edad. PRONOSTICO: Los episodios de FRA tienden a remitir
espontneamente en 75%, antes de 6 semanas y un 90% antes de 12 semanas. El pronstico a largo plazo depender de la gravedad de la carditis,
aparicin de nuevos brotes y magnitud de dao valvular.
CASO CLINICO FA
Femenino de 30 aos, antecedente IAM, estenosis mitral con remplazo
biologico, tabaquismo, 4 gestas, con dificultad para hablar y
movimientos involuntarios en dedos, manos, boca y lengua haca 18
meses, disnea de moderado esfuerzo. Actualmente con fraccin de
eyeccin de 44% y prtesis biolgica mitral sin alteraciones
estructurales con reflujo mnimo. Inici cuadro de cefalea acompaada
de parestesia en miembro superior izquierdo y movimientos
involuntarios en manos, boca, lengua y dedos. Enseguida, present
convulsiones tnico-clnicas generalizadas y episodios de alucinacin
visual, delirium e intensa labilidad emocional. Al examen fsico, se
encontraba en regular estado general, taquicrdica (frecuencia
cardaca=130 latidos por minuto), presin arterial de 120x70 mmHg,
presencia de ruidos rtmicos con hiperfonesis de B1 y frote pericrdico
audible en mesosstole y proto y teledistole y estertores finos
crepitantes en ambas bases pulmonares.
PREGUNTA
Que clase funcional de la NYHA se encuentra el paciente?
RESPUESTA
a.- Clase I
b.- Clase II
c.- Clase III
d.- Clase IV

c.- Insuficiencia cardiaca.


d.- Edema agudo pulmonar.
PREGUNTA
Cual de las siguientes medidas farmacolgicas es la mas apropiada
para el manejo de sntomas neurolgicos?
RESPUESTA
a.- Fenitoina.
b.- Haloperidol.
c.- Biperideno.
d.- Clonacepam.
PREGUNTA
Cual es la causa de la carditis en la fiebre reumtica?
PREGUNTA
a.- Proteina M Streptococcica + N-Metil glucosamina con
cruzada con la miosina.
b.- Proteina M Streptococcica + N-Acetil-D glucosamina con
cruzada con la miosina.
c.- Proteina P Streptococcica + N-Acetil-D glucosamina con
cruzada con la miosina.
d.- Proteina P Streptococcica + N-Metil glucosamina con
cruzada con la miosina.

reaccin
reaccin
reaccin
reaccin

PREGUNTA
Cual es la complicacin neurolgica que se presento en caso?
RESPUESTA
a.- Acidente cerebro vascular.
b.- Corea de huntintong.
c.- EVC y corea.
d.- Corea de Syndeham.

PREGUNTA
Considerando los Criterios de Jones cual es la respuesta correcta en
este caso?
RESPUESTA
a.- 2 Criterios Mayores y 2 menores
b.- 1 Criterio Mayor y 2 menores.
c.- 2 Criterios Mayores y 1 menor.
d.- 3 Criterios Mayores y 2 menores.

PREGUNTA
Cual es la causa de muerte aguda ms frecuente en estos casos?
RESPUESTA
a.- IAM.
b.- Carditis.

CASO CLINICO FIEBRE REUMATICA


Femenino de 18 aos que acude a urgencias por poliartralgias en
rodillas y codos de 72 h de evolucin, asociadas a fiebre. Antecedente
de escarlatina hace 3 semanas, tratada con penicilina V 7 das y 3
episodios anuales de faringoamigdalitis estreptoccica en los ltimos 2

CURSO ENARM CMN SIGLO XXI TEL: 36246001

Pharmed Solutions Institute

PGINA 64

MANUAL DE TRABAJO DEL CURSO ENARM CMN SIGLO XXI


aos. A la exploracin, presenta impotencia funcional de rodilla
derecha, dificultad para la extensin, posicin en flexo, edema y signo
de la tecla positivo. Dolor a la palpacin en rodilla contralateral y
ambos codos, con menor inflamacin e impotencia funcional. Soplo
sistlico II/VI vibratorio multifocal. Con leucocitos 19.910/l,
neutrfilos 12.040/l; PCR 28,7mg/l y VSG 55mm.
PREGUNTA
Cul es la conducta a seguir en esta paciente?
RESPUESTA
a.- Succinato de eritromicina, va oral, por diez das.
b.- Amoxicilina, va oral, por diez das 1500 mg una vez al da por diez
das.
c.- Cefalexina, 500 mg cada ocho horas, va oral durante diez das.
d.- Clindamicina va hrs VO 600 mg al da c/6 dosis durante 10 das.
CASO CLINICO FIEBRE REUMATICA
Mujer de 31 aos, portadora de cardiopata reumtica, hasta entonces
asintomtica, refiri disnea de esfuerzo progresiva y palpitaciones aun
usando regularmente penicilina benzatina de tres en tres semanas. A
los 13 aos present artritis migratoria de grandes articulaciones tras
estreptococcia de la orofaringe, y a los 19 aos, en el 14 da de
puerperio, se quej de recurrencia de la artritis, detectndose soplo
cardiaco. El examen fsico revel taquipnea, con discreto edema de
miembros inferiores. Se constataron los siguientes datos: PA = 130/70
mmHg; frecuencia cardiaca de 120 rpm; ritmo cardaco regular a tres
tiempos con intenso soplo holosistlico en rea mitral (AM) y soplo
sistlico rudo en el rea tricspide (AT); y hepatomegalia dolorosa
discreta. Ecocardiograma doppler fue compatible con insuficiencia
mitral importante e insuficiencia tricspide moderada. Hemograma
detect anemia hipocrmica (hemoglobina = 11,1 g/dl), leucocitosis
discreta (13.000/mm3) y hemosedimentacin acelerada (37 mm/1
hora). Los niveles de antiestreptolisina O (AEO) fueron normales.
PREGUNTA
Cul es el tratamiento farmacologico para limitar el dao?
RESPUESTA
a.- Salicilatos dosis 30-60 mg/kg/da.
b.- Paracetamol 20 mg/Kg/dia.
c.- Naproxeno 750 mg/dia.
d.- Prednisona 2mg/kg/dia
PREGUNTA
Cul es la conducta teraputica mas adecuada seguir en este
momento?
RESPUESTA
a.- Suspender penicilina benzatinica y usar esteroides
b.- Continuar penicilina benzatinica y agragar vancomicina
c.- Suspender penicilina benzatinica
d.- Continuar con penicilina benzatinica
CASO CLINICO
Femenino de cinco aos de edad, que pesa 18 Kg, alrgico a penicilina.
Es admitida en el Servicio de Pediatra por referir fiebre de un mes de
duracin, alta no cuantificada de inicio sbito sin patrn caracterstico,
acompaado de escalofros, diaforesis, sin antecedentes de una
historia previa de infeccin respiratoria de vas altas. EF: Se le encontr
en mal estado general, nutricin buena, astenia, adinamia, soplo
holosistolico en chorro de vapor, acompaada de artritis migratoria,

CURSO ENARM CMN SIGLO XXI TEL: 36246001

afectando las rodillas y codos, pulso 86x', presin arterial 110/80, se


localiz abultamientos pequeos indoloros localizados sobre
prominencias seas (ndulos subcutneos), disnea de moderados
esfuerzos.
PREGUNTA
Qu criterios de Jones mayores presenta esta paciente?
RESPUESTA
a.- Poliartritis, astenia, infeccin de vas respiratorias
b.- Poliartritis, carditis, ndulos subcutneos
c.- Carditis, ndulos subcutneos, corea
d.- Infeccin de vas respiratorias, carditis, poliartritis
PREGUNTA
Cul es la conducta teraputica ms adecuada a seguir?
RESPUESTA
a.- Penicilina G benzatinica 600,000U mas esteroides
b.- Eritromicina 20-50mg/kg/dia mas esteroides
c.- Eritromicna 20-50mg/kg/dia
d.- Penicilina G benzatinica 1200 000U

PREGUNTA
La paciente mejoro con el tratamiento, la madre la lleva 10 semanas
despus a revisin y el mdico le solicita antiestreptolisinas. Cul es el
resultado esperado en este caso?
RESPUESTA
a.- Lo ms probable es que an salgan altas
b.- Lo ms probable es que estn normales
c.- Lo ms probable es que salgan negativas
d.- Lo ms probable es que estn bajas
CASO CLINICO
Se trata de paciente femenino de 30 aos de edad de tes blanca, con
dificultad para habler y movimientos involuntarios en dedos, boca y
lengua hacia 18 meses. Referia disnea de moderados esfuerzos desde
hace 1 ao. La paciente refiere infeccion de vas respiratorias hace 2
semanas, actualmente presenta fiebre, y refiere dolor en muecas y
tobillos, desde hace 3 dias.
PREGUNTA
Cul es el diagnotico ms probable en este caso?
RESPUESTA
a.- Artritis reumatoide
b.- Fiebre reumtica
c.- Insuficiencia cardiaca
d.- Corea
PREGUNTA
Se le realizo un ecocardiograma, el cual reporto insuficiencia mitral
Cul es el pronstico para esta paciente?
RESPUESTA
a.- Bueno para la funcin malo para la vida
b.- Malo para la funcin, bueno para la vida
c.- Reservado
d.- Malo para la funcin y la vida

Pharmed Solutions Institute

PGINA 65

MANUAL DE TRABAJO DEL CURSO ENARM CMN SIGLO XXI


ARTRITIS REUMATOIDE (AR):
CIENCIAS BASICAS: AR es la ms comn de las artritis inflamatorias. Es una enfermedad multisistmica crnica, de etiologa desconocida, caracterizada
por inflamacin persistente (sinovitis), y consecuentemente lesin y destruccin de las estructuras periarticulares y articulares. Participan factores
genticos y ambientales los cuales juegan un rol en el desarrollo de la enfermedad. SALUD PUBLICA: Se presenta 0.5-1% de la poblacin, en relacin
mujer: hombre 3:1. Asociada a HLA-DRA-4. Ms frecuente en cuarta y quinta
ANTIGENO
Respuesta inmunitaria-reaccin
dcada de la vida. PATOGENIA: En la AR los T CD4 son los principales para
DESCONOCIDO +
inflamatoria con activacin de
desencadenar engrosamiento sinovial e invasin de articulaciones y cartlagos. En
PREDISPOSICION
cel. Plasmticas y linfocitos T
AR el rgano blanco es la membrana sinovial. DIAGNOSTICO: Clnico; Poliartritis
GENETICA
crnica (>6sem) simtrica, rigidez matutina (>1h), inicia de forma insidiosa, afecta
principalmente manos (metacarpofalalgicas 90%; carpo 80%), rodilla (60%), pies
Infiltracin sinovial por linfocitos
(metarasofalangicas 50%), articulacin atlo-axoidea (40%; se manifiesta como
T CD4> T CD8 y monocitos
Manifestaciones generales
cervicalga). Sntomas generales: astenia, adinamia, hiporexia, afectacin musculopor citocinas secretadas
esqueltica variable. Exploracin fsica: desviacin cubital, ndulos de Bouchar
por los macrfagos
(interfalangicas proximales), sinovitis, dedos en cuello de cisne o botonero, hallux
Formacin de tejido de
valgus (miembros inferiores), hipotrofia en msculos interseos en manos (por
granulacin por activacin de
vasculitis), disminucin de fuerza prensil, derrame bilateral de rodillas de
fibroblastos (pannus) con
Destruccin articular y
predominio izq. (complicacin frecuente de AR, el quiste de Barker en hueco
hiperplasia de clulas mviles
sea por citocinas
poplteo asociado a ruptura e invasin muscular). Manifestaciones extraarticulares:
formadas en el pannus.
piel; ndulos reumatoides (25-50%). Hematolgicos; anemia normocitica
normocrmica (35-30%), trombocitosis (>50,000), trombocitopenia, linfadenopata. Heptico; transaminemia. Pulmn (30%), dolor pleurtico (25%),
ndulos pulmonares, arteritis pulmonar, enfermedad pulmonar intersticial. Ojos; queratoconjuntivitis Sicca (35%), ojo seco, epiescleritis, uvetis,
queratitis ulcerativa. Corazn; pericarditis, enf. Ateroesclertica avanzada. Neurolgico; neuropata perifrica, sndrome del tnel del carpo. Renal;
nefropata Laboratorio: Factor reumatoide (FR=presente hasta 65%): Auto-anticuerpos dirigidos contra la porcin de Fc de la IgG tiene sensibilidad de
70%, especificidad 80%; tambin presente en otros procesos infecciosos crnicos e inmunitarios (Tb, endocarditis, neoplasias, lupus esclerodermia). Nos
indica artritis seronegativa o seropositiva esta ltima se relaciona con forma ms agresiva de la enfermedad. Autoanticuerpos anti-pptido cclico
citrulinado (Anti-Pcc =presente hasta 95%), sensibilidad de 70% y especificidad 90-95%, al igual que FR se relaciona con formas ms agresivas de la
enfermedad. VSG PCR elevadas. Anti-DNA negativo (especifico para (LES). Radiolgico (PA de mano): Inflamacin de articulaciones metacarpofalngicas,
desviacin cubital, pulgar en Z, dedos en cuello de cisne, erosiones seas y osteopenia yuxtaarticular, luxaciones metacarpofalngicas proximales.
CRITERIOS DIAGNOSTICOS: 1. Rigidez matutina al menos de 1 hora de la mejora mxima. 2. Artritis de 3 o ms articulaciones. 3. Artritis de las manos. 4.
Artritis simtrica. 5. Ndulos reumatoideos (sobre prominencias seas, superficies extensoras o regiones yuxtaarticulares). 6. Factor reumatoide positivo.
7. Cambios radiolgicos. El diagnostico se hace con 4 de 7 criterios. COMPLICACIONES: Derrame lateral de rodillas de predominio izq., puede
presentarse quiste de Barker en hueco poplteo asociado a ruptura e invasin muscular. TRATAMIENTO: Metotrexate piedra angular 10 mgs inicial, se
puede aumentar hasta 25 mgs/sem. Prednisona no es de eleccin, se da en dosis bajas y no modifica el curso de la enfermedad. Los AINES se dan junto
con los FARMES (frmacos modificadores de la enfermedad). En artritis activa leve: Hidroxicloroquina (HCQ) o sulfazalazina (SSZ) ambos tardan 2-3
meses en iniciar su accin. Artritis activa moderada a severa: iniciar con metotrexate, y considerar prednisona. Si hay una respuesta inadecuada agregar
otro FARME (MTX + Abatacep MTX + leflunomide MTX + SSZ + HCQ MTX + Rituximab). Los biolgicos (anti-TNF) no son actualmente de 1ra.
Eleccin. PRONOSTICO: Empeoramiento progresivo de la salud, ya que daa permanentemente los huesos, cartlagos y articulaciones. Si FR y ANTI-PCC
muy elevados indican mal pronstico y mayor gravedad. PREVENCION: En los 2 primeros aos de inicio de la enfermedad dao severo e irreversible. Un
tratamiento precoz disminuye y previene el dao. Derivacin precoz ante sospecha dx. A reumatologa. Toda paciente requiere contar con radiografa de
manos y pies en AP y oblicuas. La suspensin del habito tabquico puede ayudar a prevenir el desarrollo de AR. CLAVES: Ndulos de Heberden en
interfalangicas distales nos habla de osteoartritis. Raro debut con poliartritis aguda con fiebre y linfadenopatas, pensar en rotavirus, si hay monoartritis
en extremidades inferiores pensar espondiloartropatias seronegativas aun mas si se acompaa de lumbalga. Sndromes especficos en pacientes con AR:
Sx. de Sjogren secundario= AR + Sx. SICCA (xerostoma, xerostalma). Sx. de Felty= AR+ esplenomegalia+ leucopenia. Sx. de Caplan= AR + ndulos
reumatoideos + neumoconiosis. Amiloidosis = AR de larga evolucin y grave proteinuria e Insf. Renal
CASO CLINICO AR
Se trata de un varn de 63 aos con antecedentes de AR de ms de 15
aos de evolucin bajo tratamiento. Antecedentes hipertensin
arterial, dislipidemia y cardiopata isqumica. Ingresa por sndrome
nefrtico observndose a la exploracin fsica: 100,7kg, regular estado
general, eupneico en reposo, saturacin del 96%; hbito cushingoide;
TA 130/70mmHg, T 37C; auscultacin cardiopulmonar normal;
abdomen globoso, blando y depresible; edemas con fvea hasta la raz
de los muslos, abdomen y manos; artritis bilateral de las articulaciones
metacarpofalngicas y limitacin a la flexo-extensin en ambos carpos;
deformidad de ambos tobillos y pies, ms el derecho. Entre las
exploraciones complementarias destaca anemia normoctica
normocrmica; VSG de 130 mm/1. hora; hiperfibrinogenemia
reactiva; urea de 112 mg/dl; creatinina de 3,8 mg/dl; aclaramiento de
creatinina de 45,4 ml/min; cido rico de 8,2 mg/dl, protenas totales
de 4,5 g/dl; protena C reactiva de 10,9 mg/dl; proteinograma con
aumento de fracciones alfa 1, alfa 2 y betaglobulinas y disminucin de
las gammaglobulinas; albmina de 1,5 g/dl y en el sistemtico de orina
se detecta proteinuria, microhematuria y abundantes cilindros
granulosos-hialinos y ms de 11 g de protenas en orina de 24 h.
PREGUNTA

CURSO ENARM CMN SIGLO XXI TEL: 36246001

Considerando el manejo habitual del padecimiento de base AR, cual de


las siguientes observaciones es mas probable que se encuentre
relacionado con el sndrome nefrotico?
RESPUESTA
a.- Uso crnico de AINES.
b.- Uso de inmunomoduladores.
c.- Presencia de depsitos inmunologicos.
d.- Presencia de amiloidosis secundaria a AR.
PREGUNTA
Considerando la pregunta anterior, cual de los siguientes estudios es
de mayor utilidad en este caso?
RESPUESTA
a.- Tomografia computada abdominal.
b.- Biopsia renal.
c.- Radiografia de torax.
d.- Ecocardiografia.
PREGUNTA
Cual de las siguientes observaciones referentes al diagnostico de AR es
de mayor utilidad?
RESPUESTA

Pharmed Solutions Institute

PGINA 66

MANUAL DE TRABAJO DEL CURSO ENARM CMN SIGLO XXI


a.- Los anticuerpos anti-CCP tiene un cociente de probabilidad para el
diagnostico de AR superior al de Factor Reumatoide y es pronostico de
la enfermedad.
b.- La evaluacin radiogrfica por mtodo de Sharp/van der Heijde
permite evaluar dao estructural para (erosion y pinzamiento
articular).
c.- Inflamacin articular con anticuerpos anti-CCP y factor reumatoide
elevadoes indicativo de enfermedad agresiva con mal pronostico.
d.- Se deber realizar IRM precoz para detectar erosiones, sinovitis o
derrame articular.
CASO CLINICO AR
Mujer de 68 aos de edad con antecedentes de AR de 20 aos de
evolucin, en tratamiento con AINE, glucocorticoides a dosis bajas (5
mg al da de prednisona) y metotrexate; prtesis bilateral de caderas y
rodilla derecha; hipertensin arterial, sndrome mielodisplsico en
tratamiento con eritropoyetina e insuficiencia renal crnica leve.
Ingresa por cuadro de diarreas de 8 semanas de evolucin, prdida de
peso de 20 kg y edema en los miembros inferiores. En la exploracin
fsica se aprecia: 45 kg de peso, regular estado general, tensin arterial
de 110/60mmHg y T 36C; auscultacin cardiopulmonar normal;
abdomen blando y depresible; edemas con fvea hasta raz de muslos
y en manos. Artritis de metacarpofalngicas bilaterales y limitacin a la
flexo-extensin en ambos carpos. En la analtica destaca anemia
normoctica normocrmica, VSG de 110 mm/1. h, urea de 46 mg/dl,
creatinina de 1,3 mg/dl, sistemtico de orina normal, con protenas en
orina de 24 h negativa, proteinograma con aumento de fracciones alfa
1, alfa 2, beta y gammaglobulinas y albmina disminuida.
PREGUNTA
Considerando el cuadro clnico, cual de los siguientes estudios es el
mas adecuado para una aproximacin dignostica?
RESPUESTA
a.- Tomografia abdominal.
b.- Endoscopia abdominal.
c.- Panendoscopia.
d.- Biopsia por colonoscopia.
PREGUNTA
Considerando el manejo habitual del padecimiento de base AR, cual de
las siguientes observaciones es mas probable que se encuentre
relacionado con el sndrome diarreico?
RESPUESTA
a.- Uso crnico de AINES.
b.- Uso de inmunomoduladores.
c.- Presencia de depsitos inmunologicos.
d.- Amiloidosis secundaria a AR.
CASO CLINICO AR
Paciente varn de 60 aos con artritis reumatoide de 6 aos de
evolucin en tratamiento con sulfasalazina 1.500mg/da y deflazacort
6mg/da, con buen control clnico y analtico de su enfermedad. Se
trata de un paciente fumador activo de 20 cigarrillos/da y con
cardiopata isqumica crnica. En una revisin habitual refiere la
aparicin de una tumoracin dolorosa en el hueco poplteo derecho,
cuyo dolor se incrementa con la bipedestacin prolongada. En la
exploracin fsica se objetiv una tumoracin en el hueco poplteo.
PREGUNTA
Considerando las complicaciones mas frecuentes en la AR cual de las
siguientes es la mas probable?
RESPUESTA
a.- Quiste de Baker.
b.- Aneurisma popitleo.
c.- Lipoma popitleo.
d.- Varice popitlea.

CURSO ENARM CMN SIGLO XXI TEL: 36246001

PREGUNTA
Cual de las siguientes aseveraciones relacionadas al diagnostico de AR
no es adecuado?
RESPUESTA
a.- BH, QS, Transaminasas, VSG, EGO, actividad, funcionalidad articular
y dao estructural.
b.- Compromiso en 3 articulaciones metacarpofalangicas o
metatarsofalangicas y rigidez matinal de ms de 30 minutos.
c.- La elevacin de la VSG y la PCR tiene estrecha relacin con la
actividad inflamatoria de la AR.
d.- La presencia de Factor Reumatoide confiere el diagnostico de
artritis reumatoide.
PREGUNTA
Cual es la conducta mas apropiada en el presente caso?
RESPUESTA
a.- Los AINES no debe reemplazar a los FARME.
b.- Considerando el uso de corticoides debe usarse calcio y vitamida D.
c.- El MTX es el frmaco de eleccin para AR en riesgo de enfermedad
persistente.
d.- En caso de falla con MTX se recomienda LEF SSZ anti-TNF.
CASO CLINICO
Una mujer de 40 aos de edad, con antecedentes de artralgia de 20
aos de evolucin con tratamiento intermitente de AINEs y sales de
oro, consulta por un nuevo cuadro de artralgias de manos y pies, con
rigidez matinal de media hora de duracin, y dolor punzante, EVA
6/10, que se exacerba en estados depresivos, estrs y temperaturas
bajas. Al interrogarla dirigidamente, refiere haber presentado astenia,
xerostoma y xeroftalmia. Refiere, adems, constipacin de 1 semana
de evolucin. Dentro de sus antecedentes ginecoobsttricos figura un
ciclo menstrual I/28, G1A0P1 y uso de mtodo anticonceptivo 1 vez al
mes. EF: Se aprecia una marcha enlentecida e inestable. Existe alopecia
difusa. Sin adenopatas palpables. Trax y abdomen normales, se
evidencia en sus muecas una dorsiflexin, limitacin de movimiento y
puo incompleto (75%). En rodillas, tiene el signo del tmpano y signo
de la ola positivos. Adems, presenta aumento de temperatura y de
volumen en la rodilla derecha.
PREGUNTA
Cul es el diagnstico ms probable para este caso?
RESPUESTA
a.- Fiebre reumtica
b.- Artritis reumatoide
c.- Espondilitis anquilosante
d. Lupus eritematoso sistmico
PREGUNTA
Cules son los anticuerpos ms especficos para esta patologa?
RESPUESTA
a.- Factor reumatoide
b.- Anti-PCC
c.- ANA
d.- Anti-DNA
PREGUNTA
Qu es lo ms probable que encontraramos en una radiografa, de la
mano de esta paciente?
RESPUESTA
a.- Sinovitis, destruccin en punta de lpiz, perdida de espacios
articulares
b.- Perdida de espacios articulares, erosiones seas, osteopenia yuxtaarticular, ndulos de Bouchar
c.- Perdida de espacios articulares, osteopenia yuxta-articular, ndulos
de Heberden
d.-Osteopenia yuxta-glomerular, destruccin en punta de lpiz,
ndulos de Bouchar

Pharmed Solutions Institute

PGINA 67

MANUAL DE TRABAJO DEL CURSO ENARM CMN SIGLO XXI


OSTEOARTRITIS (OA):
CIENCIAS BASICAS: Enfermedad degenerativa crnica y forma ms comn de enfermedad de las articulaciones, caracterizada por falla comn y
progresiva de todas las estructuras de la articulacin, cartlago, revestimiento, ligamentos y hueso. Acompaadas por el aumento de espesor y esclerosis
de la placa sea subcondral. SALUD PUBLICA: Mas de 70% de personas >70 aos, tienen evidencia radiogrfica de OA. Es considerada la causa de dolor e
incapacidad ms frecuente en ancianos. La incidencia de osteoartrosis de rodilla se ha calculado en 240 casos por 100, 000 habitantes y es una causa de
morbilidad y alto costo para la comunidad.. Su prevalencia es igual para ambos gneros entre los 45 y 55 aos, pero despus de los 55 aos es ms
frecuente en mujeres. Enfermedad articular mas frecuente en mujeres. PATOGENIA: Deterioro de la articulacin debido con frecuencia a tensin
mecnica, alteraciones biomecnicas y celulares que producen un desequilibrio entre la degradacin y la sntesis de los condrocitos, la matriz
extracelular y del hueso subcondral, lo que hace que el hueso que est bajo el cartlago falle, tiende afectar las articulaciones que se usan con frecuencia
(manos, columna, cadera, rodillas). Los 2 principales componentes del cartlago son: colgeno tipo 2 (provee fuerza tensil) y un proteoglucano. El OA es
caracterizado por deplecin gradual del proteoglucano, desplegando de la matriz el colgeno y perdiendo colgeno tipo 2, lo que nos lleva a incremento
de la vulnerabilidad. Dentro de la fisiopatologa ocurren cambios morfolgicos:
fragmentacin del cartlago, formacin de fisuras y ulceracin de la superficie articular,
CLASIFICACION DE LA OSTEOARTRITIS
eburneacin, la posterior proliferacin celular en acmulos. Cambios bioqumicos:
Primaria: localizada; Cadera
Rodilla
mayor cantidad de agua, sintesisi de fibras de colgeno pequeas y frgiles, menor
Apfisis espinosas y cuerpos vertebrales
concentracin de proteoglucanos y sulfato de queratn con aumento de la
(cervical,lumbar)
concentracin de sulfato de condritin que desencadena la produccin de
Mano (interfalangicas distales)
proteoglucanos de menor tamao y de menor capacidad para agregarse al cido
Pie (primer dedo)
hialurnico, que predispone a fragilidad. Cambios biomecnicos: perdida de la rigidez y
Generalizada: Manos
elasticidad del cartlago, Factores de riesgo: edad avanzada, tener familiares con OA,
Manos y rodillas
obesidad, lesiones en articulacin o uso repetitivo, defectos congnitos, densidad sea,
Secundaria: Displasica: condrodisplasias, displasias epifisiarias,
Desplazamiento congnito articular
ocupacin, enfermedades concomitantes. DIAGNOSTICO: Clnico; Dolor (empeora con la
Postraumtica: aguda, repetitiva, posoperatoria
actividad, mejora con el reposo), rigidez matutina breve (<30min), hinchazn
Insuficiencia estructural: osteonecrosis, osteocondritis
prominente, crujidos, rechinado de toda la articulacin, llevando a un menor
Posinflamatoria: artrits reumatoide y sptica
funcionamiento de la misma, puede haber aumento de temperatura local y de volumen
Endocrina y metabolica: acromegalia, hemocromatosis
articular, si hay derrame, deformidad (angulacion en varo o valgo), subluxacin en casos
Tejido conectivo: Hipermovilidad (Ehlers- Danlos)
avanzados, ndulos de Heberden (interfalangicas distales), Bouchar (interfalangicas
Formas
generalizadas:
Hiperparatiroidismo,
proximales).
acromegalia Laboratorio: Generalmente todo normal. VSG y PCR normales. Si se hace estudio de liquido sinovial, ligeramente turbio, no contienen
cristales y con leucocitos <2,000/microlitro y <25% de neutrofilos. Radiologa: Disminucin de espacio articular, esclerosis subcondral, osteofitos
marginales y quistes subcondrales. COMPLICACIONES: Derrame articular, herniacin capsular, bloqueo articular, tendinitis, estenosis espinal
espondilolistesis. TRATAMIENTO: Ningn tratamiento especifico. Objetivo reducir dolor y mejorar funcionamiento: 1. Medidas fsicas: prdida de peso y
ejercicio con lo que se disminuye dolor, mejora fuerza muscular y disminuye rigidez de la articulacin, tiles usar ortesis o bastones. Evitar sobrecarga y
trauma de articulaciones. Calor o frio local. Reposo articular 2. Medico: Tpico; capsaicina, lidocana, diclofenaco. Va oral AINES, paracetamol (primera
eleccin), se puede considerar el tramadol (200-300mgs dividido en 3 a 4 dosis) si el AINE no es efectivo. Los corticoides orales no estn indicados en OA.
En 2010 se aprob duloxetina para dolor musculo-esqueltico crnico (largo plazo). La diacerina es una quinona, que posee propiedades antioxidantes,
inhibiendo a IL-1 y metaloproteinasas, pero aun no es contundente su eficacia y seguridad. El Ac. Hialurnico intraarticular; mejora parcial y se usa para
retraso de reemplazo de rodilla. Los glucocorticoides intraarticulares son recomendados en los brotes agudos inflamatorios en las distintas guas de OA,
de rodilla, no debe administrase una inyeccin cada 3 o 4 meses. Ciruga: Artroscopa, artroplastia u osteotoma por angulacin (disminuye el dolor, en
pacientes no candidatos a artroplastia), en casos muy severos. Glucosamina/condroitina, aunque ampliamente vendidos, no estn aprobados por FDA,
su eficacia no esta establecida.
CASO CLINICO OSTEOARTITIS
Varn de 66 aos con antecedentes personales de EPOC severo con
hiperreactividad bronquial y bullas enfisematosas en ambos vrtices
pulmonares, en tratamiento con oxigeno domiciliario (15h al da) junto
a broncodilatadores, diurticos y corticoides (prednisona 10mg) a das
alternos. El paciente haba tenido varios ingresos hospitalarios por
reagudizaciones de EPOC. Refirie dolor en la regin lumbar central y en
ambas caderas con la deambulacin. Se realizaron exploraciones
complementarias que incluyeron una radiografa de pelvis y un TAC
lumbosacro que mostraron espndilolisis con espondilolistesis de L5 y
artrosis de cadera respectivamente, procesos a los que se atribuyeron
los sntomas. En la radiografa de pelvis inicial se observa
estrechamiento del espacio articular y esclerosis subcondral. Se
encuentra en silla de ruedas por la incapacidad para deambular.
Leucocitos 9,5103/mm3 (neutrfilos 48.4%), Hb 12mg/dl, Hto 36,
plaquetas 355103/mm3, VSG 32mm/h, glucosa 86mg/dl, urea
44mg/dl, creatinina 0,88mg/dl, colesterol 190mg/dl, triglicridos
116mg/dl, fosforo 4,2mg/dl, calcio total 9,5mg/dl, actividad de
protrombina 106%, INR 0,97, ratio tromboplastina 1,10, protena C
1,22, protena S 0,73, antitrombina III 0,99U/l. Los anticuerpos
antinucleares, anticuerpos antifosfolpido y el factor reumatoide
fueron negativos, as como la serologa para virus de hepatitis B, C y
VIH.

Considerando la entidad nosolgica descrita, asi como los reportes de


laboratorio, cual de los siguientes diagnosticos diferenciales es el mas
frecuente?
RESPUESTA
a.- Infeccioso.
b.- Metablico.
c.- Inflamatorio crnico.
d.- Neuropatas.

PREGUNTA

PREGUNTA
Cual de los siguientes criterior del presente caso, lo hace candidato a
artroplastia?

CURSO ENARM CMN SIGLO XXI TEL: 36246001

CASO CLINICO OA
Mujer de 74 aos de edad que acude por aumento de dolor en la
rodilla izquierda. Le administraron paracetamol ms ibuprofeno,
refiere incapaz para limpiar la casa debido a que el dolor empeore. Ella
usa un masaje tpico al acostarse y ha intentado usar unas compresas
fras, con beneficios limitados. Es hipertensa bajo tratamiento, En el
examen, su presin arterial es de 140/ 92 mm Hg, pulso es de 80
latidos por minuto. Ella es de 162 cm de alto y pesa 80 kg (ndice de
masa corporal de 30,5 kg/m2). Ella califica su dolor como 8 de 10
cuando est activo, y 5 de 10 en reposo. Camina con un bastn y cojea.
Sus resultados del examen rodilla izquierda no se han modificado.
Previo anlisis de rayos- x mostraron cambios degenerativos severos.
Resultados de creatinina de 90 mmol / L Aclaramiento de creatinina
estimado ( CrCl ) es entre 40 y 60 ml / min.

Pharmed Solutions Institute

PGINA 68

MANUAL DE TRABAJO DEL CURSO ENARM CMN SIGLO XXI


RESPUESTA
a.- Artrosis bicorpartamental.
b.- Artrosis patelofemora aislada.
c.- Artrosis media o lateral.
d.- Necrosis avascular.

a.- Radiculopata lumbar


b.- Osteoartritis
c.- Necrosis avascular
d.- Bursitis

PREGUNTA
Considerando el estudio de liquido sinovial para el caso, que
resultados es menos probable observar?
RESPUESTA
a.- Celular completa.
b.- Tincion Gram.
c.- Cultivo de liquido.
d.- Cristales de oxalato.
CASO CLINICO OSTEARTROSIS
Varn de 66 aos con antecedentes personales de EPOC severo con
hiperreactividad bronquial y bullas enfisematosas en ambos vrtices
pulmonares, en tratamiento con oxigeno domiciliario (15h al da) junto
a broncodilatadores, diurticos y corticoides (prednisona 10mg) a das
alternos. El paciente haba tenido varios ingresos hospitalarios por
reagudizaciones de EPOC. Desde comienzos del ao 2008 vena
refiriendo dolor en la regin lumbar central y en ambas caderas con la
deambulacin. Por este motivo fue atendido en la consulta de
reumatologa y rehabilitacin. Se realizaron exploraciones
complementarias que incluyeron una radiografa de pelvis y un TAC
lumbosacro que mostraron espndilolisis con espondilolistesis de L5 y
artrosis de cadera respectivamente, procesos a los que se atribuyeron
los sntomas. Leucocitos 9,5103/mm3 (neutrfilos 48.4%), Hb
12mg/dl, Hto 36, plaquetas 355103/mm3, VSG 32mm/h, glucosa
86mg/dl, urea 44mg/dl, creatinina 0,88mg/dl, colesterol 190mg/dl,
triglicridos 116mg/dl, fosforo 4,2mg/dl, calcio total 9,5mg/dl,
actividad de protrombina 106%, INR 0,97, ratio tromboplastina 1,10,
protena C 1,22, protena S 0,73, antitrombina III 0,99U/l. Los
anticuerpos antinucleares, anticuerpos antifosfolpido y el factor
reumatoide fueron negativos, as como la serologa para virus de
hepatitis B, C y VIH.
PREGUNTA
Cul de los siguientes criterios es el mas importante en el diagnostico
clnico?
RESPUESTA
a.- Rigidez clnica.
b.- Deformidad osea.
c.- Dolor articular.
d.- Prueba de McMurrey.
CASO CLINICO
Femenino de 68 aos de edad con peso de 97Kg, estatura de 1.52mts.
Antecedente de sndrome metablico, asma sin requerimientos de
medicacin, colecistectomizada. Actualmente presenta dolor a nivel
sacriiliaco y trocantereo izquierdo, solo en posicin supina y al
caminar, irradiado hacia rodillas e ingls, desaparece con el reposo y
no permite sueo adecuado, adems aumenta con frio. No cede con la
toma de AINEs. EF: Paciente orientada y consciente, con piel y
tegumentos con buena coloracin, dolor a nivel lumbar bajo izquierdo,
regin gltea ante movilizacin activa, Lasague negativo, dolor
exquisito a la compresin del trocnter.
PREGUNTA
Cul es el diagnstico ms probable para este caso?
RESPUESTA

CURSO ENARM CMN SIGLO XXI TEL: 36246001

PREGUNTA
Cul es la conducta teraputica ms adecuada a seguir?
RESPUESTA
a.- Esteroides intraarticulares
b.- Bloqueo anestsico
c.- Quirrgico
d.- Reposo absoluto
PREGUNTA
Cul es la prevalencia de esta patologa en mujeres mayores de 65
aos?
RESPUESTA
a.- 30%
b.- 85%
c.- 50%
d.- 10%

CASO CLINICO
Un medico de atencin primaria remite con el cirujano ortopdico a
una mujer de 65 aos de edad para evaluacin de dolor en la rodilla
derecha. Hace solo diez aos ella corria maratones, pero lentamente,
durante los 5 aos pasados, los achaquez empezaron a limitar su
movilidad. Informa que puede caminar solo alrededor de 800mts,
antes de que el dolor la oblogue a detenerse y descansar. Tambien
tiene rango de movimiento disminuido en la rodilla derecha en
comparacin con la izquierda. Las radiografas de la rodilla derecha
muestran formacin de osteofito, quistes oseos subcondrales y
estrechamiento del espacio articular. El recuento leucocitico es de
6000mm3 y la sedimentacin eritrocitica es de 8mm/h.

PREGUNTA
Cul es la causa ms probable de esta patologa?
RESPUESTA
a.- Infecciosa
b.- Autoinmune
c.- Mecanica
d.- Necrosis avascular

PREGUNTA
Cul es la conducta teraputica mas adecuada a seguir en este caso?
RESPUESTA
a.- Manejo conservador, antiinflamatorio
b.- Artroplastia de rodilla
c.- Esteroides intrarticulares
d.- Ac. Hialuronico intraarticular
PREGUNTA
Cul es el porcentaje mas probable de aparicin de esta patologa a la
edad de 65 aos aproximadamente?
RESPUESTA
a.- 20%
b.- 40%
c.- 60%
d.- 80%

Pharmed Solutions Institute

PGINA 69

MANUAL DE TRABAJO DEL CURSO ENARM CMN SIGLO XXI


OSTEOCONDRITIS
CIENCIAS BASICAS: Definicin clnica: Conjunto de trastornos seos de origen idioptico, caracterizados por trastornos en la osificacin endocondrial que
afecta tanto la condrogenesis como la ostogenesis (se presenta en centros seo en pleno crecimiento). Definicin radiolgica: Trastornos seos
idiopticos en relacin a su etiologa que comparten las siguientes caractersticas; 1. Predileccin por el esqueleto inmaduro. 2. Afeccin de epfisis,
apfisis o huesos epifisoides. 3. Cuadro radiolgico caracterizado por fragmentacin, colapso, esclerosis y posteriormente reosificacin y restitucin de
contorno seo. Se da en nios clnicamente sanos y en periodo de crecimiento rpido (3-15aos) y expuestos a traumatismos; sus localizaciones ms
frecuentes son las articulaciones que soportan carga. PATOGENIA: Idioptica. Son procesos cclicos que pasan por diversas etapas en su evolucin y que
pueden dejar secuelas por deformaciones producidas en el estadio de fragilidad de la osteonecrosis. Se describen 4 etapas de la necrosis: 1. La zona
vecina a la necrosis sufre un proceso de descalcificacin por hiperemia reactiva. La zona necrtica, en cambio no se descalcifica por falta de vasos que
drenen sus componentes minerales. Resulta as una mayor densidad del fragmento necrtico respecto al sano. 2. Ms tarde, la zona necrtica es
invadida gradualmente por tejido conjuntivo y vasos sanguneos, que van a retirar el tejido necrtico por reabsorcin. 3. Esas zonas aparecen rarefactas
en medio de la necrosis, fenmenos de sustitucin progresiva. 4. Cuando el proceso de reabsorcin y reconstitucin est avanzado, todo el fragmento
esta descalcificado para ir luego a la recalcificacin y restauracin sea. DIAGNOSTICO: Clnico; dolor al apoyo, hipersensibilidad a la presin a nivel del
hueso afectado. El inicio de la sintomatologa puede ser agudo o gradual y puede o no existir anamnesis de traumatismo. Radiolgico: No siempre guarda
relacin con la intensidad del cuadro clnico. Hay descalcificacin precoz con necrosis subsiguiente y finalmente los signos de reparacin. En las fases
precoces se observan zonas de densidad disminuida, que aumentan en intensidad y extensin; las epfisis se fisuran, se ensanchan presentando un
contorno borroso con aspecto deshilachado. Pueden observarse zonas de hueso necrtico denso. LEGG CALVE PERTHES: Necrosis avascular idioptica de
la cabeza femoral en nios. La teora ms aceptada de su origen es por insuficiente aporte sanguneo al ncleo epifisiario de la cabeza femoral, debido a
oclusin vascular sea de una rama importante de la circunfleja o de una rama terminal. Se presenta entre 3-10 aos de edad, mas en sexo masculino, con
mayor frecuencia unilateral. Tiene una evolucin radiolgica caracterstica: A) Estadio incipiente o de sinovitis: que dura de 1-3 semanas. B) Estadio
necrtico, asptico o avascular: dura varios meses a 1 ao. C) Estadio regenerativo o de fragmentacin: dura de 1-3 aos. D) Estadio residual, que
corresponde a la etapa de crecimiento y de reparacin definitiva. El objetivo del tratamiento es la contencin de la cabeza femoral dentro del acetbulo,
para que ella se rehaga en forma concntrica. OSGOOD-SCHLATTER: Osteocondritis de la tuberosidad de la tibia, apfisis tibial anterior. Tiene como
factor desencadenante un traumatismo. Ocurre entre los 17 y 20 aos, ms frecuente en masculinos y en deportistas sometidos a microtraumatismos
repetidos. Hay dolor a la presin, hinchazn en tuberosidad anterior de anterior de la tibia en extensin. KOHLER: Osteocondrosis del escafoides
tarsiano: Ms frecuente en hombres (5:1), incidencia entre 5 y 10 aos. Dolor dorsal en mesopie de meses a aos. RX: aumento perchado de densidad
con fragmentacin, en estadios avanzados; aplanamiento, fragmentacin, colapso y esclerosis. THIEMANN: Necrosis asptica de las epfisis basales de las
falanges. Igual en ambos sexos. Caracterizada por hinchazn progresiva y no dolorosa a nivel de la articulacin interfalangica (proximal de manos y pies).
Rx: irregularidad de epfisis de las falanges que aparecen esclerticas y fragmentadas, ocasionalmente el espacio articular se ve disminuido, base de la
falange ensanchada. PANNER: Osteocondrosis del epicondilo lateral de hmero (capitelum). Rx: fisuracin, aumento de densidad, disminucin del
tamao del capitelum, fragmentacin, resorcin. KIEMBOCK: Osteocondritis del semilunar. Es rara por debajo de los 15 aos, hay antecedente de un
traumatismo, se da mas del lado derecho y en los trabajadores manuales, las personas jvenes acusan dolor progresivo en mueca con rigidez de la
misma e hipersensibilidad local a nivel de semilunar. FREIBERG: Osteocondrosis de la cabeza de los metatarsianos. Ms frecuente en mujeres (5:1), tpico
en segunda dcada de la vida, 10% bilateral. Es rara la afectacin de ms de un dedo por pie. Presencia en orden decreciente dedo 2>3>4, muy raro en 1
y 5. SCHEUERMANN: Cifosis torcica rgida por acuamiento de los cuerpos vertebrales que ocurre en la infancia tarda. Rx: Cifosis torcica >45,
irregularidad de platillos vertebrales, estrechamiento del espacio intervertebral.
CASO CLINICO OSTEOCONDRITIS
Femenino de 69 aos de edad con cuadro clnico de 10 meses de
evolucin de inicio sbito; caracterizado por dolor tipo punzada en
rodilla izquierda que aparece luego de mantenerla en reposo
prolongado, sin predominio de horario, asociado a crepitacin, rigidez
e inflamacin de la rodilla; intensidad 5/10 en la escala visual anloga,
la cual cede a medicacin con acetaminofn. Fue valorada por
medicina general quien indico 4 ampollas de betametasona; sulfato de
glucosamina, diclofenaco, colestiramina; con mejora de sntomas. La
radiografa ntero-posterior de rodilla izquierda muestra una lesin
radio lcida subcondral, rodeada por un halo de esclerosis. Al examen
fsico, presenta crepitacin bilateral de rodilla de predominio izquierdo
con inflamacin moderada de rodilla izquierda. Sinovitis moderada en
rodilla izquierda.
PREGUNTA
Cual es su impresin dianostica?
RESPUESTA
a.- Osteocondritis Aseptica.
b.- Osteocondritis Disecante de Rodilla.
c.- Osteonecrosis.
d.- Osteocondritis osificante.

PREGUNTA
Cul es el diagnostico mas probable en este caso?
RESPUESTA
a.- Sinovitis txica
b.- Artritis septica
c.- Enfermedad de Legg-Calv-Perthes
d.- Enfermedad de Gaucher

PREGUNTA
Considerando la respuesta previa, cual la localizacion mas frecuente?
RESPUESTA
a.- Lateral del cndilo femoral medial.
b.- Rotula.
c.- Cndilo femoral externo.
d.- Meniscos discoideos.

CURSO ENARM CMN SIGLO XXI TEL: 36246001

CASO CLINICO
Un nio de 9 aos s remitido con el medico debido a un antecedente
de dolor intermitente en la cadera derecha. El dolor empez durante
un juego de futbol soccer, seis meses antes y se ha intensificado con el
tiempo. Aunque ha restringido la actividad y usa muletas, la cadera
derecha continua generando molestias, con cualquier actividad. No
existe antecedente de traumatismo, no hay antecedentes familiares
similares. EF: ino sano bien nutrido, signos vitales normales. No
muestra deformidades oseas, la rotacin interna y abduccin de la
cadera causan dolor intenso, en comparacin con el lado opuesto hay
disminucin de movimientos. Labaratorios normales. Rx de cadera
derecha: se aprecia radiotransparencia apolillada, sugerente de
osteopenia de epfisis y el cuello femorales. Una gammagrafa osea de
las extremidades inferiores demuestra signos de avascularidad de la
epfisis femoral derecha.

PREGUNTA
Cul es la conducta teraputica mas adecuada a seguir en este caso?
RESPUESTA
a.- Manejo conservador
b.- Aparatos ortopdicos
c.- Manejo quirrgico

Pharmed Solutions Institute

PGINA 70

MANUAL DE TRABAJO DEL CURSO ENARM CMN SIGLO XXI


GOTA:
CIENCIAS BASICAS: La gota se caracteriza por hiperuricemia y la presencia de episodios inflamatorios habitualmente monoarticulares y recurrentes con
lapsos asintomticos, que son intensos y autolimitados. La inflamacin est directamente relacionada con la presencia de cristales de urato monosdico
(UMS) intraarticular de forma constante. Hiperuricemia: Nivel plasmtico de acido rico >7.0mg/dl en hombres o ms de 6.0mg/dl en mujeres, surge de
causas en la disminucin de su eliminacin, de aumento de la produccin o de ambos. El acido rico es el producto final de la degradacin de los
nucletidos de purina, su produccin est relacionada con la concentracin intracelular de 5-fosforibosil-1-pirofosfato (PRPP), es el mayor determinante
de la velocidad de biosntesis del acido rico. El acido rico es excretado principalmente por el rin a travs de mecanismos de filtracin glomerular,
secrecin tubular y reabsorcin.SALUD PBLICA: Gota causa comn de artritis en hombres mayores de 40 aos y mujeres posmenopusicas. La
prevalencia de hiperuricemia en poblacin adulta es de alrededor de 15%. PATOGENIA: La enfermedad comienza con la formacin y la subsecuente
acumulacin de acido rico secundario a un aumento en su produccin y/o disminucin en su eliminacin (fase asintomtica). Posteriormente factores
desencadenantes como alcohol, diurticos, obesidad, trauma, pH acido, ambientales y otros ocasionan acumulacin de cristales de urato monosdico en
las articulaciones (artritis), riones (nefropata) y vas urinarias. El depsito de cristales produce liberacin de citocinas y activacin, as como
reclutamiento de clulas inflamatorias (PMN) y no inflamatorias (endotelio y sinoviocitos) y quimiotaxis. No se conoce a detalle el proceso completo de
la formacin de cristales pero algunos de los factores son: el acido rico tiene menor solubilidad a menor temperatura, por lo cual pueden formarse
cristales con mayor facilidad en lugares distales como dedos de los pies y de las orejas. La solubilidad de los UMS es menor cuando el pH es cido,
microtraumas repetidos, alteracin de la matriz extracelular. DIAGNOSTICO: Clnico; Artritis aguda, la ms frecuente y temprana manifestacin de gota,
inicialmente afecta una articulacin (suele localizarse en la primera metatarsofalangica = podagra), pero puede ser poliarticular en episodios tardos. La
gota aguda inicia por la noche con dolor severo, hinchazn, calor. El ataque generalmente desaparece espontneamente en 3-10 das. Aunque algunos
pacientes suelen tener solo un ataque, otros episodios recurrentes, con intervalos de variacin sin sntomas. La gota aguda puede ser precipitada por
exceso de purinas en la dieta, trauma ciruga, ingestin excesiva de alcohol, terapia hipouricemica y enfermedades serias como infarto al miocardio.
Artritis crnica; una proporcin de pacientes con gota pueden tener una sinovitis unilateral crnica, esta suele ser rara. Puede tambin presentarse con
tofos (agregados de UMS, rodeados por clulas de reaccin inflamatoria, ndulos, subcutneos, de aspecto amarillento y duros) periarticulares. Tofos
extraarticulares: Algunos estn en la cpsula del olecranon, hlix y antihlix del pabelln auricular, superficie cubital o antebrazo, tendn de Aquiles,
generalmente se desarrollan entre 5-10 aos despus del inicio de ataques de gota. Tenosinovitis. Nefropata por uratos: por depsito de UMS en el
intersticio y pirmides, puede causar IRC. Nefropata aguda por acido rico: causa reversible de falla renal aguda debida a precipitacin de los uratos en
los tbulos, los pacientes que reciben tratamiento citotxico para neoplasia estn en riesgo. Nefrolitiasis por acido rico: responsable de 10% de las
litiasis renales. CRITERIOS PARA LA CLASIFICACION de artritis gotosa aguda: A) La presencia de cristales de UMS caractersticos en el liquido sinovial (gold
estndar para dx) o bien B) Un tofo cuyo contenido de cristales de UMS este comprobado por medios qumicos o microscopia de luz polarizada o bien C)
La presencia de 6 de los siguientes 12 fenmenos: 1. Mas de un ataque de artritis aguda. 2. Desarrollo mximo de brote inflamatorio en el curso del da.
3. Ataque monoarticular. 4. Franco enrojecimiento del area articular. 5. Localizacin primera articulacin metatarsofalngica. 6. Ataque unilateral que
afecte la primera articulacin metatarsofalngica. 7. Ataque unilateral en una articulacin del tarso. 8. Tofo: sospecha o confirmacin. 9. Hiperuricemia.
10. Infamacin asimtrica en estudio radiogrfico. 11. Quistes subcutneos sin erosiones seas. 12. Cultivo negativo del liquido articular obtenido
durante un ataque agudo. Los ataques de gota agudos se relacionan con cambios en el nivel plasmtico de AU (tanto aumento como disminucin). La
identificacin de UMS en liquido sinovial es patognomnico, se ven al microscopio como agujas y birrefringentes. Radiolgico: En ataque agudos pueden
mostrar tumefaccin de tejidos blandos. Los tofos son radiotransparentes, aunque en ocasiones pueden tener calcificaciones en su interior, los cambios
caractersticos son edema difuso, prominencias nodulares, erosiones intraarticulares y extraarticulares con imagen de techo colgante y calcificaciones.
TRATAMIENTO: Dentro de la medidas generales dieta baja en alimentos ricos en purinas como: carnes, vsceras, piel, alimentos ahumados, rostizados,
los dos factores ms importantes que favorecen la hiperuricemia son: ingesta de alcohol y ayuno prolongado. Artritis gotosa aguda: Analgesia, AINES
(indometacina, diclofenaco, naproxen) primera eleccin. Colchicina efectiva dentro de las primeras 24hrs del ataque, contraindicada cuando hay
insuficiencia renal o citopenias. Dosis de 1-6 mg/da. Glucocorticoides intraarticulares; se debe descartar primero artritis sptica. Glucocorticoides
sistmicos; uso breve, considerado en pacientes con ataque de gota poliarticular. Agentes hipouricemicos: Indicados para iniciar a bajar el acido rico,
incluyendo recurrencias de artritis gotosa aguda, artritis poliarticular, gota tofosa, litiasis renal. No deben iniciarse en el ataque agudo, pueden precipitar
una llamarada y se debe considerar su uso concomitante con colchicina. Alopurinol: disminuye la sntesis de acido rico por inhibicin de la xantinooxidasa, reducir dosis en insuficiencia renal. Dosis de 100-300mgs, pueden darse hasta 800mgs. Uricosricos (probenecid, sulfinpirazona): Incrementan la
excrecin de acido rico, por inhibicin de la reabsorcin tubular (inefectivo en insuficiencia renal), no debera usarse en >60 aos de edad. Iniciarse una
vez que haya pasado el ataque agudo.
d.- Osteocondritis
CASO CLINICO
Varn de 43 aos de edad, hipertenso, diabtico, con sobrepeso,
PREGUNTA
alcohlico desde hace 21 aos. Hace 15 aos comienza con crisis de
Cul sera la comorbilidad ms importante en este para este
dolor e inflamacin monoarticular en miembros inferiores. En un
paciente?
principio presentaba 23 episodios anuales de monoartritis que
RESPUESTA
mejoraban con antitinfamatorios no esteroideos. El paciente no
a.- Diabetes
realizaba correctamente el tratamiento farmacolgico, ni las medidas
b.- Hipertrigliceridemia
higinico dietticas pautadas por su mdico de atencin primaria, por
c.- Hipertensin
lo que las crisis de monoartritis eran cada vez ms frecuentes,
d.- Sobrepeso
extendindose la afectacin a articulaciones de miembros superiores y
evolucionando el cuadro hacia oligoartritis y el dia de hoy amanecio
PREGUNTA
con dolor en 5 articulaciones diferentes agudo, tarsitis unilateral..
Cul sera la conducta diagnstica ms adecuada a seguir para
confirmar esta patologa?
PREGUNTA
RESPUESTA
Cul es el diagnstico ms probable para este caso?
a.- Anticuerpos anti-PCC
RESPUESTA
b.- Histologa cuerpos de Aschoff
a.- Osteoartrosis
c.- Radiologa, esclerosis subcondral, osteofitos marginales
b.- Artritis reumtica
d.- UMS en lquido sinovial
c.- Gota

CURSO ENARM CMN SIGLO XXI TEL: 36246001

Pharmed Solutions Institute

PGINA 71

MANUAL DE TRABAJO DEL CURSO ENARM CMN SIGLO XXI


SINDROME DE SJOGREN (SS):
CIENCIAS BASICAS: Sindrome Sjgren primario: Enfermedad autoinmune sistmica con disfuncin lagrimal y salival progresiva, las cuales pueden ser
sintomticas o asintomticas e incluir varios problemas extraglandulares. SS Secundario: Presencia de disfuncin lagrimal y salival con otras
enfermedades autoinmunes (Artritis reumatoide, Lupus, esclerodermia), no lleva anticuerpos Anti-Ro, ni Anti-La. SALUD PUBLICA: Prevalencia de 0.1-0.6
% de la poblacin. 90% sexo femenino relacin 9:1, M:H, entre 45 y 55 aos de edad. SS es un factor de riesgo para linfoma (daa los linfocitos).
PATOGENIA: Exocrinopata autoinmunitaria de etiologa desconocida. Probable etiologa infecciosa: Infeccin viral (Epstein Barr, cocxaquie, VIH).
Asociada frecuentemente a infeccin por Virus de Hepatitis C. Se asocia a HLA DR-3, HLA DR-8. Hay una infiltracin potrlinfocitos T CD4. DIAGNOSTICO:
Manifestaciones glandulares: Queratoconjuntivitis Sicca (xeroftalmia, sensacin de arena en los ojos). Xerostoma (resequedad oral): Manifestacin ms
frecuente (dificultad para pasar alimentos secos, sensacin urente, aumento de caries dentales). Crecimiento de glndulas partidas (20-30%), glndulas
salivales tumefactas. Xerotrquea. Manifestaciones extraglandulares: La manifestacin principal son las artralgias y datos de artritis no erosiva (25-85%)
(si es erosiva est asociado a artritis reumatoide). Fenmeno de Raynaud (13-62%). Tiroiditis autoinmune de Hashimoto. (10-20%). Acidosis tubular renal
distal tipo I (5-20%). Hepatitis o cirrosis heptica (2-4%). Pulmn: Enfermedad pulmonar intersticial (9-32%). Riegos elevado para linfoma no-Hodgkin,
linfoma MALT asociado a clulas B y macroglobulinemia de Waldenstrom. Laboratorio: Prueba de Schirmer positiva (menor o igual a 5 mm en 5 minutos,
severa entre 0-3mm, leve mayor a 8 mm). Tincin de rosa de bengala positiva . Biopsia de glndula salival menor (lacrimal o partida): muestra infiltrado
linfocitico. Autoanticuerpos: Anti-Ro (SSA) 56%, anti-La (SSB) 30%. si estn positivos indican comienzo precoz, mayor predisposicin a enfermedades
extraglandulares. ANAs (95%), FR (75%). CRITERIOS DIAGNOSTICOS: Clasificacin Colegio Americano de Reumatologa: 1.-Ojo seco. 2.-Boca seca (ambos
3 meses). 3.-Prueba de Schirmer o rosa de bengala positivo. 4.-Biopsia de glndula menor con infiltrado inflamatorio. 5.-Prueba objetiva de disminucin
de funcin de glndulas salivales. 6.-Anti-Ro o Anti-La positivos. Para hacer el diagnstico con 4 de 6 criterios. Diagnostico definitivo; criterios clnicos
mas biopsia de glandula salival. TRATAMIENTO: Xerostoma: higiene bucal, pilocarpina (estimula secrecin salival), ambroxol, agua, gel de ac. propionico.
Xeroftalma: lagrimas artificiales (metilcelulosa 0.5% 2-3 veces al dia), pilocarpina oral, si hay ulceras corneales usar ac. Borico en pomada y ocluir el ojo
afectado, evitar diurticos, antidepresivos, antihipertensivos por que generan mas sequedad. Manifestaciones extraglandulares: Artralgias/artritis:
Metrotexate, hidroxicloroquina, prednisona dosis bajas. Fenmeno de Raynaud: Proteccin contra el fro, nifedipino, losartan. Hay que sospechar de
linfoma si hay adenopatas y disminucin o desaparicin del factor reumatoide. Acidosis tubular renal dar bicarbonato. Caso clnico tpico: Mujer con
artritis reumatoide y se le seca el ojo, con 3 meses de evolucin, que le cuesta trabajo deglutir el alimento seco.
CASO CLINICO SX DE SJOGREN
Femenino de 14 aos de edad, acude a consulta por presentar
sangramiento en las encas desde hace varios meses, refiere la madre
que el estomatlogo le indic el cepillado correcto y buchadas de
manzanilla 3 veces al da. Examen fsico: Se observa presencia de
caries, encas edematosas, al sondeo bolsas de 4mm en el sector
anteroinferior (brecha edente de 3.1-3.2), perdidos por movilidad, no
restituidos por prtesis, sarro supragingival y manchas de origen
medicamentoso. Se aprecia pigmentacin melnica en la enca
adherida, hay xerostomia, lengua lisa y brillante e higiene deficiente.
PREGUNTA
Cual es la conducta a seguir en relacin al caso?
RESPUESTA
a.- Yodopovidona oral.
b.- Pilocarpina oral.
c.- Clorhexidina oral.
d.- Prednisona oral
PREGUNTA
Cul es la conducta mas adecuada a seguir para un diagnostico
defintivo?
RESPUESTA
a.- Prueba de Schirmer
b.- Criterios diagnsticos mas biopsia de glndula salival
c.- Biopsia de glandula salival
d.- Anti-Ro y Anti-La
PREGUNTA
Cul es la utilidad de la prueba de Rosa de Bengala?
RESPESTA
a.- Indica disminucin de la produccin lagrimal
b.- Acidez de las estructuras
c.- Tejido inflamado
d.- Epitelio desvitalizado
PREGUNTA
2 meses despus la paciente acude a revisin, y al exploracin fsica se
detecta ganglio detrs de la oreja aumentado de tamao asi como

CURSO ENARM CMN SIGLO XXI TEL: 36246001

partidas ligeramente aumentadas y la madre refiere que ha tenido


fiebre intermitente, y que tiene dificulatad para tragar los alimentos.
Cul es el diagnostico mas probable en este momento?
RESPUESTA
a.- Sindrome de Sjogren
b.- Linfoma
c.- Sobreinfeccion bacteriana
d.- Acalasia
CASO CLINICO SINDROME DE SJOGREN
Mujer hipertensa de 74 aos. Unas semanas antes comienza con
astenia, fatigabilidad y disnea progresiva, dolor torcico de
caractersticas inespecficas, hipotensin sintomtica y sncope; no
refera fiebre. En el momento del ingreso presentaba una presin
arterial sistlica (PAS) de 90 mmHg, un electrocardiograma con
taquicardia sinusal, bloqueo de rama derecha ms hemibloqueo
anterior izquierdo, con PR normal y cardiomegalia-congestin en la
radiografa de trax. Analtica bsica: creatinina 1,4 mg/dl, Na 121
mEq/l, K 5,6 mEq/l, aclaramiento de creatinina 95 ml/min,
microalbuminuria 9,4 mg/dl, troponina I 2,4 ng/ml, cLDL 78 mg/dl,
GOT-GTP 239 U/l, GGT 136 U/l, FA 154 U/l, LDH 904 U/l. Leucocitos 5,9
3 103/ l; hemoglobina 11 g/dl; plaquetas 233 3 103/ l; actividad de
protrombina 87%, VSG 133 mm en la primera hora y PCR 13,2 mg/dl.
Hemocultivos y serologas varias negativas. La ecocardiografa en el
momento del ingreso muestra un ventrculo izquierdo no dilatado, sin
alteraciones segmentarias de la contractilidad y disfuncin sistlica
severas, biventricular, con fraccin de eyeccin del ventrculo
izquierdo (FEVI) del 15%, sin otros hallazgos.
PREGUNTA
Considerando los hallazgos clnicos ms relevantes. Cual es el frmaco
de eleccin?
RESPUESTA
a.- Corticosteroides.
b.- Azatioprina
c.- Metotrexato
d.- Ciclofosfamida.

Pharmed Solutions Institute

PGINA 72

MANUAL DE TRABAJO DEL CURSO ENARM CMN SIGLO XXI


LUPUS ERITEMATOSO SISTEMICO (LES)
CIENCIAS BASICAS: Prototipo de enf. autoinmune, multiorganica, multisistmica, se asocia con la produccin de autoanticuerpos contra componentes
del ncleo, mediada por inmunocomplejos (hipersensibilidad III). SALUD PUBLICA: Ms en mujeres de 20-40aos. Relacin M:H es de 9:1, raza negra.
Lupus eritematoso sistmico 70% (con amenaza organica 35%), lupus cutneo l5%, Sx de sobreexposicion/Enf. mixta de tejido conectivo 10%, lupus
eritematoso inducido por drogas 5%. PATOGENIA: Enfermedad de etiologa desconocida.
Clasificacin de nefritis lupica segn la OMS
Participan para el desarrollo factores genticos como: HLA-DR2, DR3, DR4 (75%), deficiencia
(histopatolgica)
de complemento C1, C2, C4 (a mayor deficiencia mayor riesgo para nefritis lupica) la
TIPO I
Nefropata lupica mesangial mnima
formacin de complejos inmunes es lo que causa dao aorganico principalmente nefritis
TIPO II
Nefropata lupica mesangial
lupica y neurolupus, mayor incidencia en familiares de 1er grado. Factores ambientales:
TIPO III
Nefropata lupica focal
Rayos UVB (apoptosis de queratinocitos), medicamentos, infecciones (VEB), mimetismo
TIPO IV
Nefropata lupica difusa
molecular. Factores hormonales: estrgenos son permisivos de autoinmunidad, puerperio y
TIPO V
Nefropata lupica membranosa
anticonceptivos exacerban l LES. Factores inmunolgicos: falla en el control del sist.
TIPO VI
Nefropata lupica esclerosante (terminal)
Inmunolgico, produccin excesiva de autoanticuerpos. Se producen autoanticuerpos, se
forman complejos inmunes, disminuye la apoptosis de cel. T autoreactivas, hay actividad aumentada de linfocitos T cooperadores (CD4) y disminucin de
linfocitos T supresores (CD8); todo esto junto con los factores nos lleva a LES. Tiene un cuado variable de remisin-exacerbacin. DIAGNSTICO: Cuadro
clnico: Hay perodos de remisin y exacerbacin, sntomas inespecficos en 95%; fiebre, astenia, adinamia. Manifestaciones musculo-esquelticas son las
ms frecuentes (90-95%); artralgias (70-100%), artritis no erosiva (60%), pulgar en z, mialgias, hiperlaxitud, osteonecrosis avascular (dolor en cadera y/o
hombro). Mucocutaneas frecuentes (80-90%); lo mas comn presencia de lupus discoide (15-30%: cara, hombro, cuello, retroauricular), rash en alas de
mariposa (50%), eritema malar tiene la caracterstica de inestabilizarse e incrementarse con la exposicin al sol, fotosensibilidad (70%), ulceras orales
3
(40%), vasculitis cutnea y livedo reticularis. Hematolgicas frecuentes (80-90%); anemia (70%), leuco, linfopenia (<1500/mm ) y trombocitopenia,
linfadenopata y esplenomegalia. Neurolgicas (50-60%); Neurolupus, neuralgia, meningitis asptica, disfuncin cognitiva (50%), sx. neural orgnico
(35%), convulsiones y psicosis (20%), neuropata perifrica, EVC, cefalea. Cardiopulmonares; pleuritis (50%), derrame pleural (30%), neumonitis lupica (en
10%; complicaciones graves, mortalidad 60-70%), pericarditis (30%), endocarditis de Libman-Sack (insf. artica y mitral, por deposito de
inmunocomplejos es asptica, no hay bacterias, ni leucos). Renales frecuentes, glomerulonefritis (50%); principal factor que condiciona la sobrevida del
paciente, glomerulonefritis membranosa y mesangial, nefritis lupica factor de mal pronstico (mortalidad <10%) y es la principal causa de muerte en
lupus hasta 50%. Laboratorio: Perfil metablico (tienen aterogenesis acelerada y lpidos alterados), EGO (proteinuria), reactantes de fase aguda (PCR y
VSG), anticuerpos antinucleares (ANA; considerado gold standar para LEG, especificidad media), C3 y C4, anti-DNA ds ms especfico para lupus (
especificidad excelente), anti- histona ( relacionado con lupus inducido por drogas), anti sm (especificidad excelente), anti-Ro (SSA, se relaciona con lupus
neonatal, pedir en embarazadas), anti-La (SSB, si sale positivo protege contra dao renal). Anti-Ro y anti-La; son especficos pero son ms sensibles para
Sx. de Sjogren. Se pueden tomar Rx. De manos y pies, factor reumatoide, densitometra osea.
CRITERIOS DIAGNOSTICOS:
Cutneos
1. Rash malar: eritema malar fijo, plano o palpable
2. Rash discoide: parches eritematosos palpables, con ampliacin queratosica y taponamiento folicular, pueden existir cicatrices atroficas.
3. Fotosensibilidad: reaccin inusual a la luz solar, observado por mdico (de los mas especficos)
4. Ulceras orales: o nasofaringeas, usualmente indoloras, observadas por medico
Sistmicos
1. Artritis: no erosiva que involucre 2 o ms articulaciones perifricas, caracterizadas por hipersensibilidad, inflamacin o derrame.
2. Serositis: pleuritis (dolor pleurtico, o frote escuchado por medico) pericarditis (documentada por electrocardiograma, frote o derrame pericrdico)
3. Trastorno renal: proteinuria persistente (>0.5 g/d o ms de +++), o cilindros celulares de cualquier tipo.
4. Trastorno neurolgico: convulsiones o psicosis en ausencia de otras causas.
Laboratorio:
1. Trastorno hematolgico: Anemia hemoltica o leucopenia (>4000 en 2 ocasiones), linfopenia (<1500 en 2 ocasiones) o trombocitopenia (<100,000 en ausencia de drogas
causantes)
2. Trastorno inmunolgico: Anti-DNA o anti- sm, o anticuerpos antifosfolipido
3. Anticuerpos antinucleares: en ausencia de drogas conocidas, que puedan ser asociadas a lupus inducido por drogas.
SE DICE QUE UNA PERSONA TIENE LEG, SI PRESENTA 4 DE LOS 11 CRITERIOS DE FORMA SIMULTNEA O SERIADA EN EL CURSO DE LA ENFERMEDAD, DURANTE O EN
CUALQUIER PERIODO DE LA MISMA.

FARMACOS QUE INDUCEN LUPUS: Constituye el 10%; Hidralacina, procainamida, isoniacida, clorpromacina, d-penicilamina, quinidina, interferon alfa,
fenitoina, anti-TNF, predominan manifestaciones constitucionales y pleuropericarditis, no afeccin renal, ni neurolgica, 90% remite a los 3 meses de
suspender el frmaco, algunos requerirn esteroides o AINES. Asociado a HLA-DR4, anti- DNA negativos, ANA positivos, anti-histona positivo e
hipocomplementemia. LUPUS NEONATAL: Se produce en menos de 5% de los RN de madres portadoras de autoanticuerpos anti-Ro y anti-La, hasta en
25% de madre sanas con anti- Ro circulante, lesiones cutneas en areas fotoexpuestas a partir de los 2 meses de vida, bloqueo A-V completo
(complicacin ms grave del lupus neonatal) y alteraciones hematolgicas. TRATAMIENTO: Basado en el tipo y la severidad. AINES (Ibuprofeno 400800mg c/6-8hrs). Corticoesteroides (prednisona 1mg/Kg, en caso de dao orgnico agudo dar pulsoso de; metilprednisolona 1g/d x 3dias),
hidroxicloroquina (antimalaricos) y aspirina (estos autorizados por la FDA). No exponerse al sol, dietas sin alfalfa (incrementa la autoinmunidad).
Manifestaciones leves: AINES, esteroides a dosis bajas (>20mg/kg/da), hidroxicloroquina (400mg/da. efecto secundario toxicidad retiniana). Lesiones
cutneas, fotoproteccin (FPS>100), corticoides tpicos. Manifestaciones graves (neurolupus, glomerulonefritis, neumonitis, trombocitopenia, anemia
hemoltica) pulsos de metilprednisolona, prednisona, inmunosupresores como aziatropina, metotrexate (son ahorradores de corticoides). 1ra eleccin en
nefritis lupica ciclofosfamida (10-15mg/kg/da). Aziatropina; nefritis lupica y dao hematolgico. Metotrexate articulares (piedra angular en artritis
reumatoide), manifestaciones articulares 7.5-15mg/sem. Si no responden a la terapia inicial lo ms usado en terapia biolgica es rituximab (anti-CD-20).).
Anticoagulacin puede estar indicada en complicaciones tromboticas. PRONOSTICO: Con lupus eritematoso generalizado, la sobrevida a 5 aos es de
85% y a 10 aos es de 25%. Principal causa de muerte es la nefropata hasta 50%. Factores de mal pronostico; hipotensin arterial, Cr >1.4mg/dl,
nefropata lupica difusa, cambios crnicos en la biopsia renal, si se presentan evoluciona rapidamente a insuficiencia renal crnica. Del 50% que presenta
falla renal el 5-20% progresa a insuficiencia renal crnica.
CASO CLINICO LES
Mujer de 17 aos que padeca de LES. En el momento del diagnstico
debut con una nefritis lpica manifestada por un sndrome nefrtico,
con protenas en orina de 24 horas de 3,3 g, colesterolemia de 280

CURSO ENARM CMN SIGLO XXI TEL: 36246001

mg% y albuminemia de 1,8 g/dL, por lo cual recibi tratamiento con


metil-prednisolona succinato en bolos durante tres das (tres dosis), y
se mantuvo con prednisona 1 mg/kg/da, observndose mejora del
cuadro. Igualmente la paciente present una cefalea intensa que

Pharmed Solutions Institute

PGINA 73

MANUAL DE TRABAJO DEL CURSO ENARM CMN SIGLO XXI


calmaba parcialmente con analgsicos y sin signos de irritacin
menngea, que fue interpretada como un sntoma asociado al lupus.
Continu el tratamiento con prednisona hasta 15 das despus de su
egreso, pero disminuy a una dosis de 35 mg/da. Un mes despus, la
paciente acudi nuevamente para control. Los exmenes demostraron
deterioro de la funcin renal, asociado a un sndrome nefrtico
impuro, con hipertensin arterial (160/110). Se aument la dosis de
prednisona a 1 mg/kg de peso/da y se agreg captopril 50 mg cada 8
horas. La paciente refera persistencia de la cefalea frontoparietal,
opresiva, de fuerte intensidad, que ya no mejoraba con analgsicos y
que era exacerbada por los movimientos de la cabeza, sin otros signos
y sntomas asociados. A los pocos das del ingreso present fiebre de
40 C, nuseas, vmitos y fotofobia.
PREGUNTA
Cual es la conducta a seguir mas adecuada?
RESPUESTA
a.- Agregar diurtico, neuroproteccion y antibitico.
b.- Realizar punsion lumbar para citoquimico, citolgico y cultivo.
c.- Realizar IRM de crneo.
d.- Aumentar corticoide y FARME
PREGUNTA
Entre 30 y 50% de la morbilidad y mortalidad de los pacientes con
lupus eritematoso sistmico, son atribuidas a las infecciones. Cal de
las siguientes es la menos frecuente?
RESPUESTA
a.- SNC
b.- Piel.
c.- Pulmones
d.- Sistema genitourinario
CASO CLINICO LES
Mujer de 67 aos de edad diagnosticada de hipertensin arterial.
Ingres por sensacin nauseosa, infeccin urinaria y anemia. Anlisis:
Ht 27,8%; Hb 9,5g/dl. VSG 63 mm. Urea 182mg/dl; creatinina 4,7
mg/dl; calcio 9mg/dl; fsforo 4,2mg/dl, protenas totales 9g/dl.
Aclaramiento de creatinina (frmula de Cockroft-Gault): 17,73 ml/min.
Inmunoprotenas y complemento normal. Cadenas Kappa-s 774 mg/dl,
cadenas Lambda 392 mg/dl. En el proteinograma se observ un pico
de amplia base en regin Gamma, con aumento de IgG (193%) y
cadenas ligeras Kappa (191%) y Lambda (180%). ndice K/L= 1,97.
Cadenas ligeras en orina: cadenas Kappa 13,7 mg/dl (0-0,7); cadenas
Lambda 6,880 (0-0,39). Se realiz aspirado y biopsia de mdula sea,
que result normal. THS: 4,85 UI/ml, T4 libre 1,03 ng/fl; Ac anti-TPO
22,5 UI/ml; Ac anti-Tiroglobulina 115,3 UI/ml. PTH: 110 pg/ml. Orina
(tira reactiva): protenas 25 mg/l; sedimento: abundantes leucocitos.
Marcadores tumorales: normales. Serologa viral: negativa.
Autoanticuerpos: ANA +, anti-ADN positivos.
PREGUNTA
Considerando las caractersticas del caso clnico. Cal es el abordaje
teraputico mas adecuado?
RESPUESTA
a.- Metotrexate.
b.- Ciclosporina.
c.- Prednisona.
d.- Aziatropina.
PREGUNTA
El lupus eritematoso sistmico, es una enfermedad inflamatoria con
afectacin sistmica. Cal es el porcentaje de pacientes que
presentan signos de nefropata?
RESPUESTA
a.- Al menos 60%.
b.- Al menos 50 %.

CURSO ENARM CMN SIGLO XXI TEL: 36246001

c.- Al menos 40 %.
d.- Al menos 30%
CASO CLINICO LES
Mujer de 47 aos de edad con Nefropata lpica. AHP a los 17 aos
presenta Prpura Trombopnica que lleva a una esplenectoma, a los
19 aos de edad ingresa tras presentar un brote de artralgias, fiebre,
edemas y lesiones eritematosas en cara, siendo diagnosticado de LES.
En ese momento destaca la presencia de sndrome nefrtico con
alteraciones en el sedimento, una Cr 1.9mg/dl y en el estudio
cardiolgico se objetiva prolapso de la vlvula mitral. Se practica
biopsia renal; proliferacin mesangial difusa de distribucin irregular,
ncleos en carirorrexis, trombos intracapilares ocasionales y paredes
capilares engrosadas en asa de alambre. Las pequeas arterias, no
tienen alteraciones. La tcnica de inmunofluorescencia directa
demuestra la presencia de depsitos subendoteliales y mesangiales de
IgG, C3, C1q, IgM e IgA. Diagnstico: Nefritis lpica Clase IV. Se decide
tratamiento con corticoides y azatriopina, normalizndose la funcin
renal y negativizndose la actividad inmunolgica, persistiendo
proteinuria de 2 gr/24 h. A los 21 aos se detectan cifras elevadas de
tensin arterial. En su primer embarazo, presenta aborto espontneo
tras 2 meses de amenorrea. A los 22 aos se retira la inmunosupresin
en el curso de su segundo embarazo que llega a trmino, presentando
hipertensin arterial en el octavo mes. Inmediatamente tras el parto,
se presenta empeoramiento de su situacin con hipertensin arterial y
proteinuria de 3 gr/24 h, reinicindose el tratamiento con esteroides y
azatriopina, quedando la paciente asintomtica, sin proteinuria. A los
25 aos de edad comienza con crisis epilpticas detectndose en TAC
cerebral lesiones isqumicas mltiples y un mes despus, segundo
brote lpico con fiebre, afectacin articular, cutnea, reaparicin de
proteinuria con funcin renal normal y actividad inmunolgica. A los
36 aos de edad, es intervenida de aneurisma fusiforme de la arteria
cartida derecha. Un ao despus presenta un accidente cerebro
vascular con hemiplejia izquierda y recuperacin total, se observaron
dilataciones en cartida y aorta abdominal en estudios posteriores.
PREGUNTA
Considerando la comorbilidad del LES cual de las siguientes entidades
clnicas es mas probable considerando la evolucin del caso?
RESPUESTA
a.- Sindrome infeccioso
b.- Arteriosclerosis
c.- Sindrome antifosfolipidico
d.- Necrosis avascular sea
CASO CLINICO LUPUS ERITEMATOSO
Femenino de 32 aos de edad la cual se encuentra diagnosticada con
LES con 4 aos de evolucin con tratamiento irregular, ingresa a
admisin continua, hace 48 horas inicia sbitamente con estado
confusional agudo previamente con letargia, a la exploracin fsica se
observa ictrica, su estado no permite responder preguntas, sus signos
vitales se encuentra taquicardia, hipotensin, palidez y disnea, no se
auscultan alteraciones cardiorespiratorios, sin embargo en abdomen
se detecta esplegnomegalia, se realizan exmenes de rutina con
hemoglobina de 6 g/dl, leucos de 6,200 y plaquetas de 300,000,
bilirrubina 4,5 g/dl con reticulocitos de 19 % as como
microesferocitosis, resto sin datos por agregar.
PREGUNTA
Cual es la complicacin actual que presenta esta paciente que es
potencialmente mortal aguda?
RESPUESTA
a.- Sindrome hemoltico-uremico.
b.- Purpura trombocitopenica trombotica.
c.- Anemia hemoltica autoinmunitaria.
d.- Anemia Hemolitica microangiopatica.

Pharmed Solutions Institute

PGINA 74

MANUAL DE TRABAJO DEL CURSO ENARM CMN SIGLO XXI


VASCULITIS:
CIENCIAS BASICAS: Grupo heterogneo de enfermedades raras, que pueden ocurrir de forma independiente o como complicaciones de una enfermedad
previamente establecida. Involucran la vasculatura de cualquier rgano, la presentacin depende del calibre del vaso afectado. CLASIFICACION: Ver
cuadro anexo. TRATAMIENTO: Prednisona ms metotrexate o ciclofosfamida (CYC) en fases agudas, si remiten se da tratamiento de mantenimiento con
azatioprina (AZA) o metotrexate (MTX). ARTERITIS DE CELULAS GIGANTES (ACR): La ms comn
de las vasculitis sistmicas. Tambin llamada arteritis de la temporal o enf. de Horton, afecta a las
VASOS DOMINANTES
VASCULITIS
Grandes vasos
Arteritis de clulas gigantes (ACR)
ramas craneales del arco artico, especialmente a la arteria temporal. Ms comn en mayores de
Arteritis de Takayasu
50 aos, con pico mximo a los 80 aos. Ms frecuente en mujeres en relacin 2:1. CLINICO:
Medianos vasos
Poliarteritis nodosa (PAN)
Fiebre de bajo grado, fatiga, prdida de peso, mialgias, anorexia, cefalea (60%), hipersensibilidad
Enf. De Kawasaki (AT)
Pequeos vasos y
Granulomatosis de Weneger (GW)
o dolor de la arteria temporal (ausencia de pulso) y piel cabelluda. Afeccin de la arteria
arterias medianas
Sx. de Churg-Strauss (SCS)
oftlmica (20%), neuritis ptica, diplopa, amaurosis fugaz y ceguera (dar tx. mdico precoz
ASOCIADAS A ANCAS
Poliangitis microscpica (PA)
urgente; esteroides en dosis altas). Afeccin de la A. facial: claudicacin mandibular. CRITERIOS
Pequeos vasos
Purpura de Henoch-Scholein (PHS)
(leucocitoclstica)
Arteritis leucocitoclstica cutnea
DIAGNOSTICOS: 1. Edad >50 aos 2. Cefalea de reciente inicio 3. Hipersensibilidad de la arteria
temporal (piel cabelluda) o disminucin de la pulsacin 4. VSG >50mm/h 5. Biopsia granulomas y
vasculitis. Con 3 de 5 criterios se hace el diagnostico. DIAGNOSTICO: VSG (98%), PCR elevadas, anemia y alteracin de pruebas de la funcin heptica
(70%), biopsia de la arteria afectada (afeccin en parches, no es continua); infiltrado mononuclear, con presencia de granulomas y clulas gigantes.
TRATAMIENTO: Prednisona 1mg/kg/da, durante las primeras semanas con descenso gradual. ARTERITIS DE TAKAYASU (AT): Llamada enf. sin pulso o
sndrome del arco artico. Ms comn en menores de 40 aos y en mujeres. Proceso inflamatorio crnico granulomatoso, que afecta la aorta y sus
ramas, provocando generalmente sntomas isqumicos. CLINICA: Se divide en fases: fase I; periodo inflamatorio con fiebre, artralgias y prdida de peso,
trill artico o subclavio. Fase II; hipersensibilidad y dolor en vasos sanguneos, disminucin asimtrica en pulsos en extremidades, claudicacin,
hipertensin renovascular, sincope neurognico. Fase III; periodo fibrotico. La artera ms afectada hasta 93% es la subclavia. CRITERIOS DIAGNOSTICOS:
1. Edad <40aos de edad 2. Claudicacin de extremidades 3. Disminucin del pulso de la arteria braquial 4. Diferencia de presin arterial sistlica
>100mmHg en los brazos 5. Trill sobre arterias subclavia o aorta. 6. Arteriografa anormal. Con 3 de 6 criterios se hace el diagnostico. CLASIFICACION:
de acuerdo al sitio afectado: I: Ramas del arco artico. IIa: Aorta ascendente, arco artico y sus ramas. IIb: Aorta descendente, arco artico y sus ramas,
A. torcica descendente. III: A. torcica descendente, A abdominal y/o arterias renales. IV: A. abdominal y/o A. renales. V: Combinacin de los tipos IIb y
IV. DIAGNOSTICO: VSG Y PCR elevadas. Arteriografa=Gold estndar; oclusin, estenosis, irregularidad y aneurisma. La toma de biopsia es rara, se ve una
panarteritis focal, infiltrado celular con granulomas. TRATAMIENTO: Esteroides, MTX, Aziatropina y antiplaquetarios, en enfermedad refractaria
micofenolato y ciclofosfamida, anti-TNF. PANARTERITIS NODOSA CLASICA (PAN): Entre 65-75 aos. Inflamacin necrotizante de arteria de pequeo y
mediano calibre (respeta capilares y vnulas). Disminucin de la luz vascular. Se asocia a VHB 30% y a VHC 5% CLINICA: Insuficiencia renal, HTA (70%),
purpura papable (50%), mononeuritis mltiple (50%), dolor abdominal (40%), ICC, IAM, pericarditis (30%), raras afecciones pulmonares. CRITERIOS
DIAGNOSTICOS: 1. Prdida de peso >4Kg 2. Livedo reticularis 3. Dolor e hipersensibilidad testicular (orquitis) 4. Mialgias, debilidad, dolor de
extremidades (50-60%). 5. Mononeuropata o polineuropata 6. Presin diastlica >90mmHg 7. BUN elevado (>40mg/dl) o Cr mayor de 1.5 mg/dl 8. Virus
de hepatitis B 9. Arteriografa anormal (aneurisma, oclusin, estenosis de arterias viscerales) 10. Biopsia; vasculitis de mediano y pequeos vasos. Con 3
de 10 criterios se hace el diagnostico. DIAGNOSTICO: Angiografa, biopsia; infiltrado de PMN y necrosis fibrinoide (caracterstico ausencia de granulomas
y eosinofilos). TRATAMIENTO: Glucocorticoides, ciclofosfamida, antiviral si est asociada a VHB O VHC. La panarteritis nodosa microscpica, presenta los
mismo hallazgos que PAN, pero esta afecta a capilares y vnulas, la afectacin pulmonar es frecuente, no hay granulomas. GRANULOMATOSIS DE
WENEGER (GW): Enfermedad inflamatoria granulomatosa necrotizante con vasculitis sistmica. Predomina en adultos jvenes. Predomina en capilares y
vnulas. CLINICO: Vas areas superiores (95%); Sinusitis, otitis media y destruccin de tabique nasal. Pulmn (90%); Infiltrados pulmonares cavitados
bilaterales y no migratorios (si so migratorios puede ser un proceso infeccioso). Ndulos hemorrgicos y hemoptisis. Glomerulonefritis (75%); Focal y
segmentaria rpidamente progresiva. Ocular (50%); epiescleritis, uvetis. Neuropatas perifricas o craneales. CRITERIOS DIAGNOSTICOS: 1. Inflamacin
oral o nasal; ulceras orales, descarga nasal purulenta o hemorrgica 2. Radiografa de trax con presencia de ndulos, infiltrados fijos o cavitaciones 3.
Hematuria microscpica o restos de eritrocitos 4. Biopsia con inflamacin granulomatosa. Con 2 de 4 criterios se hace el diagnostico. DIAGNOSTICO: cANCA (Anticuerpos antiproteinasa-3) positivos 90%. TAC o Rx de trax; ndulos, infiltrados, cavitaciones. Elevacin de BUN, Cr, proteinuria, sedimento
urinario activo. Biopsia; inflamacin granulomatosa necrotizante de arteriolas, capilares y vnulas. TRATAMIENTO: Ciclofosfamida + prednisona, MTX o
aziatropina. TMP/SFX puede prevenir las recadas de infecciones de vas areas superiores. POLIANGITIS MICROSCOPICA (PA): Vasculitis necrotizante de
pequeos vasos, puede haber o no granulomas. CLINICO: Sntomas constitucionales; prdida de peso, fiebre (50%), fatiga, mialgias. Renal; hematuria,
glomerulonefritis rpidamente progresiva (90%). Pulmonar; tos y/o hemoptisis, hemorragia pulmonar (30%). Neurolgico; mononeuritis multiplex (40%).
DIAGNOSTICO: p-ANCA (anti-mieloperoxidasa) positivos 70%. Biopsia; inflacin pauci-inmune necrotizante de arteriolas capilares y vnulas, con o sin
granulomas o infiltrados eosinofilicos. TRATAMIENTO: Ciclofosfamida + aziatropina, MTX o aziatropina (ahorradores de esteroides). SINDROME DE
CHURG-STRAUSS (SCS): Inflamacin granulomatosa eosinofilica que involucra pulmn. A cualquier edad ms entre 30-40 aos. CLINICO: Asma de difcil
control (95%), enfermedad eosinofilica infiltrativa, vasculitis de pequeos vasos con granulomas. Neuropata; mononeuritis multiplex (60-70%),
parestesias en guante o calcetn, arteritis coronaria y miocarditis. Glomerulonefritis. CRITERIOS DIAGNOSTICO: 1. Asma 2. Eosinofilia (>10%) 3. Mono o
polineuropata 4. Infiltrados pulmonares migratorios transitorios 5. Anormalidades paranasales 6. Eosinofilia extravascular en la biopsia. Con 4 de 6
criterios se hace el diagnostico. DIAGNOSTICO: Dato cardinal para diagnostico: eosinofilia perifrica 80%. ANCA + (c-ANCA o p-ANCA) positivo en 50%.
Biopsia: microgranulomas, necrosis fibrinoide y trombosis de pquelas arterias y venas con infiltrados eosinofilicos. Rx. de trax: puede haber infiltrados
pulmonares. TRATAMIENTO: Ciclofosfamida + prednisona, MTX o aziatropina. PURPURA DE HENOCH-SCHONLEIN (PHS): La ms comn de las vasculitis
(leucocitoclstica) en nios (<16aos). Suele aparecer despus de una infeccin de tracto respiratorio o ingesta de frmacos, es mediada por IgA.
Producida por depsito de inmunocomplejos. CLINICO: Purpura palpable en superficies extensoras y glteos, poliartralgias, dolor abdominal tipo colico
(edema de pared, invaginacin intestinal poco frecuente). Nefritis; hematuria microscpica y proteinuria. Fiebre. CRITERIOS DIAGNOSTICOS: 1. Purpura
palpable 2. Edad menor de 20 aos al momento de la aparicin 3. Angina intestinal (dolor abdominal) 4. Biopsia; granulocitos en la pared de arteria o
venas. Con 2 de 4 criterios se hace el diagnostico. DIAGNOSTICO: Conteo plaquetario, elevacin de IgA srica. Biopsia de piel: vasculitis leucocitoclstica
con IgA y depsitos de C3 en la pared de los vasos. Biopsia renal; deposito mesangial de IgA. TRATAMIENTO: De sostn. Esteroides (1mg/kg) y frmacos
modificadores de enfermedad severa, con afeccion renal. CRIOGLOBULINEMIA: Vasculitis rara con depsitos inmunes de crioglobulinas (protenas que se
precipitan con el fro), afecta piel y glomrulo, se asocia con infeccin por VHC (90%). CLINICA: Discinecia, confusin, cefalea, y EVC isqumico, causado
por hiperviscocidad en tipo I. Estasis vascular; livedo reticularis, necrosis. Triada de manifestaciones en tipo II Y III; purpura, artralgias, mialgias. Sistemica
glomerulonefritis membranoproliferativas, neuropata perifrica. CLASIFICACION: Tipo I: Constituida por una sola inmunogloblina IgM.
Macroglobulinemia de Waldestron o mieloma. Tipo II: IgM monoclonal tipo K, que presenta actividad tipo factor reumatoide e IgG policlonal. Sx.
proliferativos. Tipo III: Constituida por 2 inmunoglobulinas (IgM-IgG) de carcter policlonal: Forman complejos Ag:Ac circulantes: procesos que cursan

CURSO ENARM CMN SIGLO XXI TEL: 36246001

Pharmed Solutions Institute

PGINA 75

MANUAL DE TRABAJO DEL CURSO ENARM CMN SIGLO XXI


con inmunocomplejos, infecciones crnicas, cirrosis. DIAGNOSTICO: Serologa viral, hipocomplementemia, crioglobulinas. TRATAMIENTO: Terapia
antiviral, esteroides, ciclofosfamida, plasmaferesis. SINDROME DE BEHCET: Vasculitis multisistemica, caracterizada por presencia de ulceras genitales y
orales recurrentes dolorosas. Se asocia con HLA-B51. CLINICO: Ulceras orales dolorosas (>3 episodios anuales), de base necrtica amarillenta, se curan en
1 o 2 sem sin dejar cicatriz. Ulceras genitales (80%), si dejan cicatriz. Foliculitis (80%), eritema nodoso (50%), erupciones acneiformes. Ocular;
manifestacin ms grave, uvetis posterior que puede condicionar prdida de visin (20%, tx. Agresivo con ciclosporina o anti-TNF). Puede haber
meningitis asptica, meningoencefalitis. DIAGNOSTICO: Fenmeno de patergia (patognomnico), se inocula sol. Inyectable lesin de >2cm es positiva.
Biopsias de ulceras. Fondoscopia. CRITERIOS DIAGNOSTICOS: 1. Ulceras aftosas orales recurrentes 2. Ulceras genitales recurrentes 3. Lesiones oftlmicas:
uvetis (con hipopin), escleritis, vasculitis retiniana, neuritis ptica. 4. Lesiones cutneas; ppulas foliculitis, eritema nodoso 5. Patergia positiva. El
diagnostico se hace con el N.1 y 2 ms del resto. TRATAMIENTO: Aziatropina, para ulceras (colchicina, dapsona, talidomida), corticoides, para uvetis la
ciclosporina. ENFERMEDAD DE KAWASAKI: Sindrome linfomucocutneo. Ms comn en menores de 5 aos (80%), ligero predominio en sexo masculino.
Etiologa desconocida. CLINICA: Cuadro exantemtico febril, que no responde a antibiticos y congestin conjuntival o no. Alteraciones labiales; eritema,
fisuras, sangrado con formacin de costras y lengua en frambuesa. Adenopatas cervicales dolorosas y exantema en tronco y extremidades. Formacin
de aneurismas (25%), vasculitis coronarias (forma ms mortal), carditis (80%). DIAGNOSTICO: Clnico, factores reactantes de fase aguda, angiografa
coronaria, no se asocia a ningn autoanticuerpo. CRITERIOS DIAGNOSTICOS: Fiebre persistente por al menos 5 dias, mas 2 de los siguientes: 1. Cambios
en extremidades inferiores 2. Exantema polimorfo 3. Inyeccin conjuntival bilateral 4. Cambios en labios y cavidad oral 5. Linfadenopata cervical.
TRATAMIENTO: Gammaglobulina IV (2g/Kg dosis nica o 400 mg/kg, durante 4 dias. ASA de 100-1500mg/da. Controversial uso de esteroides.
CASO CLINICO WEGENER
Paciente femenina de 22 aos de edad con historia de 2 meses de
evolucin con sinusitis, otitis media bilateral complicada con
mastoiditis, conjuntivitis bilateral, artralgia en hombros, codos, rodillas
y tobillos, tos con expectoracin sanguinolenta, debilidad generalizada
y cansancio fcil, asociado a fiebre no cuantificada sin predominio de
horario y a prdida de peso aproximadamente de 10% de su peso
habitual. Hace ms o menos tres semanas presenta cuadro de
odinofagia, lumbalgia, disuria, coluria y refiere un episodio de epistaxis
aislada. SV: Temperatura: 38,2 C, FC: 110 latidos/minuto, FR: 22
ciclos/minuto. Tensin Arterial: 120/90 mmHg. Radiografa de trax: se
identifica infiltrado de tipo intersticial de predominio derecho
parahiliar, en menor grado, este tipo de infiltrado se visualiza hacia la
base pulmonar izquierda. No hay signos de lesiones nodulares ni
cavitaciones. TAC: infiltrados nodulares en ambos campos pulmonares,
cavitacin en lbulo superior derecho, tenue patrn de vidrio
esmerilado posterobasal derecho y crecimientos ganglionares
precarinales, parahiliares y subcarinales.
PREGUNTA
Considerando el diagnostico del caso, cual de las siguientes
aseveraciones sobre su tratamiento no es adecuada?
RESPUESTA
a.- Pulsos de ciclofosfamida 1 g intravenoso (I.V.) mensuales por 6
meses y luego pulsos de ciclofosfamida 1 g I.V. cada 3 meses por un
ao.
b.- Pulsos de Metilprednisolona 1 g I.V. cada da por 7 das, luego se
administra 1 mg/kg/da por 6 semanas y posterior reduccin gradual.
c.- La combinacin de ciclofosfamida y metilprednisolona se utiliza
para inducir la remisin del cuadro.
d.- Leuprolide 11,5 mg I.M. cada mensual, para evitar amenorrea como
efecto secundario de la ciclofosfamida.
PREGUNTA
Segn los criterios diagnsticos de GW segn el American College of
Rheumatology (ACR), son: Sedimento urinario anormal (cilindros
hemticos o ms de 5 eritrocitos por campo de alto poder).
Radiografa de trax con hallazgos anormales (ndulos, cavidades o
infiltrados). Ulceras orales o secrecin nasal; y biopsia con inflamacin
granulomatosa. Cuantos criterios se requieren para confirmar el
diagnostico?
RESPUESTA
a.- 1.
b.- 2.
c.- 3.
d.- 4
CASO CLINICO DE CHUG STRAUSS
Paciente femenina, de 44 aos de edad, con antecedentes patolgicos
personales de asma bronquial de inicio tardo (35 aos), quien llevaba
tratamiento esteroideo regular en los perodos de crisis, y que

CURSO ENARM CMN SIGLO XXI TEL: 36246001

comenz 4 aos despus con entumecimiento y debilidad en ambos


miembros inferiores, por lo que se interpret como un sndrome
polineuroptico. A los 3 meses comenz con ligera mejora del cuadro,
fue ingresada en varias ocasiones con la misma impresin diagnstica
pero agrego gran debilidad en ambos miembros inferiores. Al
interrogatorio se constat: asma bronquial de inicio tardo precedida
de rinitis, fenmeno de Raynaud, diarreas frecuentes, con
antecedentes de gastritis. Al examen fsico: cuadriparesia con
predominio en miembros inferiores, con cambios trficos dstales,
dedos en martillo con aumento del arco plantar, disminucin del tono
muscular, as como abolicin de los reflejos osteotendinosos.
Eosinofilia marcada. El estudio de rayos X de trax mostr infiltrado
inflamatorio. El estudio de conduccin nerviosa (ECN) evidenci la
presencia de un sndrome polineuroptico. Se comenz tratamiento
con corticosteroides, metilprednisolona y se continu con prednisona
y vitaminoterapia, con lo cual la paciente mostr una mejora
significativa. El ciclo de tratamiento se repiti cada 2 meses. Al cabo de
6 meses la paciente se encuentra deambulando sin apoyo y con
discreta afeccin del ECN.
PREGUNTA
Se diagnostico SCS, considerando la patologia cual de las siguientes
manifestaciones no es frecuente observar?
RESPUESTA
a.- Vasculitis de los pequeos vasos.
b.- Granulomas vasculares y/o extravasculares.
c.- Eosinofilia perifrica que ocurre en los pacientes asmticos sin
historia de rinitis alrgica o sinusitis.
CASO CLINICO SCS
Paciente femenina, de 45 aos de edad, con antecedentes de asma
bronquial que comenz a los 27 aos. 3 meses antes inicio con lesiones
en piel, diagnosticando una vasculitis. Comenz con tratamiento
esteroideo con lo que mejor su cuadro de forma relativa.
Posteriormente la paciente comenz a presentar entumecimiento en
el miembro superior derecho y posteriormente en la extremidad
inferior contralateral, con ligera impotencia funcional. Despus
padeci de diarreas, edemas en la cara y en miembros inferiores y
cefalea de caracterstica vascular. Se encontr hipoestesia distal,
superficial y profunda, cambios trficos, con disminucin de reflejos
osteotendinosos, adems se encontr el fenmeno de Raynaud,
hipereosinofilia y aumento del valor de creatinina (210 ml/L). El
ecocardiograma mostr evidencias de pericarditis de pequea cuanta
y el rayos X de trax evidenci infiltrado inflamatorio. Se encontr
evidencia de mononeuropata multiplex. La TAC de crneo evidenci la
presencia de 3 pequeas lesiones hipodensas con caractersticas
vasculares. Se comenz tratamiento con metilprednisolona, no
teniendo una respuesta adecuada, solo con leve mejora sintomtica.
La paciente se mantiene actualmente bajo tratamiento
inmunosupresor, azatioprina 100/diarios.

Pharmed Solutions Institute

PGINA 76

MANUAL DE TRABAJO DEL CURSO ENARM CMN SIGLO XXI


PREGUNTA
Los criterios para la clasificacin del SCS 6 criterios: el asma, la
eosinofilia mayor que 10 % en recuento de leucocitos diferenciales, la
mononeuropata
o
polineuropata,
infiltrados
pulmonares,
anormalidad del seno paranasal y la biopsia de vasos sanguneos con
eosinofilia intra/extravascular. Cuantos criterios son necesarios para
establecer el diagnostico?
RESPUESTA
a.- 2.
b.- 3.
c.- 4.
d.- 5.
CASO CLINICO DE ARTERITIS DE CELULAS GIGANTES
Paciente varn de 62 aos que acude a consulta externa para una
exploracin rutinaria. En la exploracin oftalmolgica la agudeza visual
(AV) fue de 1 en ambos ojos (AO). Con pupilas isocricas y
normorreactivas, sin defectos aferentes, tensin por aplanacin de 16
mmHg en AO. En el estudio biomicroscpico anterior destacaba una
esclerosis cristaliniana moderada. En la retina del ojo izquierdo (OI)
haba dos manchas algodonosas asintomticas, una de 250 micras y
otra puntiforme a lo largo de la arcada temporal inferior. Un estudio
fsico completo no detect patologa. Los estudios de laboratorio
fueron normales salvo una protena C reactiva de 4,4 mg/dl, una
velocidad de sedimentacin glomerular (VSG) de 56 mm/1. hora.
PREGUNTA
Cual es la conducta a seguir mas adecuada para establecer el
diagnostico?
RESPUESTA
a.- Biopsia de la arteria temporal.
b.- Biopsia de la arteria cartida.
c.- Biopsia de la occipital.
d.- Biopsia de la oftlmica.
CASO CLINICO ACG
Mujer de 83 aos que acude a consulta por disminucin bilateral de la
agudeza visual. Entre sus antecedentes destacaba ciruga bilateral de
cataratas hace 5 aos. La AV era de 0,4 en el ojo derecho (OD) y 0,3 en
OI. La tensin por aplanacin fue de 18 mmHg en OD y de 16 mmHg en
OI, con pupilas isocricas y normorreactivas. En la exploracin
fundoscpica destacaba una esclerosis coroidea y una mancha
algodonosa de 20 micras de dimetro en el OD. La exploracin fsica
demostr una dificultad importantsima en la movilidad con
claudicacin de las extremidades inferiores, astenia, un leve soplo
carotdeo, cefalea en casco durante 8 meses y unas arterias
temporales endurecidas. Durante el estudio (en 4 das) la paciente
desarroll una neuritis ptica anterior en el OD y con velocidad de
sedimentacin glomerular de 94 mm/1. hora, protena C de 5,85
mg/dl, 400 Mil/mm3 plaquetas, anemia hipoproliferativa leve y
elevacin de la fosfatasa alcalina (134 UI/L).
PREGUNTA
Considerando las sintomatologia oftalmolgica cuales la causa mas
frecuente?
RESPUESTA
a.- Diabetes mellitus.
b.- Hipertensin arterial.
c.- Colagenopatas.
d.- Patologa tumoral.
CASO CLINICO DE ARTERITIS DE TAKAYASU
Mujer de 27 aos que consult por un cuadro de un mes de evolucin
de dolor en brazo izquierdo acompaado de parestesias y paresias
homoloterales que aumentaban con la actividad y cedan con el
reposo. Antecedentes: 12 aos antes: Fiebre Reumtica tratada con
penicilina benzatnica durante 7 aos, 4 aos antes: Embarazo ectpico
en trompa de Falopio con ooforectoma izquierda 3 aos antes:

CURSO ENARM CMN SIGLO XXI TEL: 36246001

Episodio de dolor lumbar de comienzo sbito durante una actividad


fsica tratado con corticoides que abandon por cuenta propia 2 aos
antes: Diagnstico de anemia, tratada con hierro intramuscular
durante un ao 1 ao antes: Disminucin de la agudeza visual y
fotofobia durante la actividad fsica que persiste hasta la actualidad.
Episodio de taquicardia y disnea de comienzo sbito. Se diagnostic
taquicardia sinusal y comenz tratamiento con atenolol 8 meses antes:
Cefalea de inicio sbito, intensa, holocraneana acompaada de
hipertensin arterial. EF: PA 140/60mmHg en brazo derecho, inaudible
en brazo izquierdo; FC 96 lpm; FR 18 cpm; T 36.1 C. Cabeza y Cuello:
pulsos carotdeos asimtricos con disminucin franca del lado
izquierdo, frmito carotdeo derecho, soplo carotdeo bilateral.
Cardiovascular: soplo sistlico 5/6 en focos artico y pulmonar que
irradia a cuello y aumenta con la inspiracin. Pulsos radial, femoral,
tibial posterior y pedio asimtricos, con disminucin en hemicuerpo
izquierdo. Abdomen: hgado palpable a 1cm del reborde costal,
Miembros: disminucin de la fuerza en hemicuerpo izquierdo. Tono,
trofismo, temperatura conservada. Hematocrito 36%, Hemoglobina 12
g/dL, Glbulos blancos 9300/mm3, Plaquetas 284000/mm3, Glicemia
112 mg/dL, Urea 37 mg/dL, Creatinina 0.6 mg/dL, Na 134 mEq/L, K 3.3
mEq/L, plasma no ictrico, Protenas totales 8.8 g/dL, Albmina 4.7
g/dL, Tiempo de protrombina 13.3 segundos, KPTT 33 segundos,
Velocidad de eritrosedimentacin 70 mm/1 hora.
PREGUNTA
Ante la presencia de un cuadro clnico con manifestaciones sistmicas,
anemia, velocidad de eritrosedimentacin (VES) elevada y alteracin
de la aorta y sus ramas, debe pensarse en aortitis, de las siguientes
patologas cual es la menos frecuente?
RESPUESTA
a.- Espondolitis anquilosante.
b.- Arteritis de clulas gigantes.
c.- Enfermedad de Behct.
d.- Sindrome de Marfan.
PREGUNTA
Considerando su diagnostico, cual es la indicacin mas frecuente para
revascularizar?
RESPUESTA
a.- Hipertensin arterial renovascular.
b.- Claudicacin severa de los miembros.
c.- Isquemia mesentrica
d.- Infarto agudo de miocardio
CASO CLINICO DE KAWASAKI
Un varn de 22 aos, consumidor ocasional de cocana y fumador de
20 cigarrillos/da, fue remitido a nuestro centro por dolor torcico. La
exploracin fsica y la analtica de control fueron normales. El
electrocardiograma mostr bloqueo de rama derecha con ondas q en
V1, V2 y V3. La radiografa de trax mostr una imagen calcificada,
redondeada superpuesta a la silueta cardiaca. En el ecocardiograma se
objetiv un ventrculo izquierdo dilatado y adelgazado, con hipocinesia
general ms marcada en el septo y la cara anterior, con disfuncin
sistlica severa (fraccin de eyeccin del 30%). Se realiz una
coronariografa que mostr oclusin completa de la arteria
descendente anterior distal a un aneurisma gigante calcificado y
oclusin completa de la coronaria derecha.
PREGUNTA
El diagnstico clsico de la EK se basa en la presencia de los siguientes
sntomas, cual no es frecuente?
RESPUESTA
a.- Fiebre.
b.- Alteraciones cutneas en las extremidades.
c.- Exantema polimrfico.
d.- Conjuntivitis bilateral exudativa.
CASO CLINICO

Pharmed Solutions Institute

PGINA 77

MANUAL DE TRABAJO DEL CURSO ENARM CMN SIGLO XXI


Paciente de 45 aos, que consulta por malestar general, erupcin
cutnea pruriginosa en cara anterior de ambas piernas, hipoestesia y
dolor urente en pie derecho y tos progresiva de 3 semanas de
evolucin. Refera antecedentes de sntomas asmatiformes
espordicos y rinitis crnica con plipos nasales. Al examen fsico
presentaba taquipnea, crepitaciones en campo pulmonar derecho,
edema bimaleolar, maculas eritematosas de 3-4mm en ambas piernas,
lesiones aisladas compatibles con purpura palpable en glteos,
hiprreflexia aquileana derecha e hipoestesia plantar derecha.
Laboratorios: Leucocitos 13,000 (N 56%, E16%, B0%, L22%).

d.- Aumento ganglionar cervical

PREGUNTA
Cul es el diagnstico ms probable para este caso?
RESPUESTA
a.- Granulomatosis de Weneger
b.- Arteritis de clulas gigantes
c.- Sndrome de Behcet
d.- Sndrome de Churg-Strauss

PREGUNTA
Cul es el pronstico de vida para esta paciente a 5 aos?
RESPUESTA
a.- 10%
b.-35%
c.- 50%
d.- 75%

PREGUNTA
Cunto criterios diagnsticos presenta esta paciente?
RESPUESTA
a.- 4
b.- 3
c.- 2
d.- 1

CASO CLINICO
Femenino de 74 aos, sin alergias conocidas y con antecedentes
mdicos de bronquitis asmtica, hipertensin arterial, portadora de
marcapasos (desde haca 5 aos), e intervenida quirrgicamente de
prtesis valvular mitral (haca 3 aos) y cncer de mama (dieciocho
aos antes). Segua tratamiento habitual con acenocumarol,
furosemida, bisoprolol, espironolactona, enalapril, pantoprazol y
hierro oral. La paciente refera clnica de tres meses de evolucin de
cefalea occipital unilateral, irradiada a zona frontal y temporal, de tipo
opresivo, que no ceda con analgsicos habituales y acompaados de
astenia, hiporexia, prdida ponderal de 10 Kg y claudicacin
mandibular intermitente. A la exploracin fsica, destacaba palidez de
mucosas. No se palpaban adenopatas. En el resto de la exploracin
fsica no se encontraron otros hallazgos significativos. La exploracin
neurolgica fue normal. Se solicitaron como pruebas complementarias
analtica de sangre y tomografa computarizada craneal. En la analtica
no exista anemia y s leucocitosis con neutrofilia. Asimismo se observ
una glucemia de 128 mg/dl. La PCR fue de 3,99 mg/dl, VSG fue de 83
mm/h. No se observaron otras alteraciones analticas.

PREGUNTA
Con que agentes esta relacionada esta patologa?
RESPUESTA
a.- Staphylococcus aureus, Streptococo pneumoniae
b.- Ascaris lumbricoides, trichinella
c.- Virus sincitial respiratorio, VHS
d.- E. coli, Salmonella typhi

CASO CLINICO
Adolescente femenina de 16 aos de edad, procedente del Estado de
Mexico, quien acudi a la consulta externa de medicina interna con
cuadro de 2 aos de evolucin caracterizado por aumento ganglionar
en la regin cervical, acompaado de fiebre no cuantificada, calosfros,
hiporexia y ataque al estado general. Durante ese tiempo se le realiazo
una biopsia bde ganglio cervical, con tejido granulomatoso compatible
con tuberculosis, fue manejada con tratamiento homeoptico. La
exploracin fisisca a su ingreso indico un perfil somatomtico normal.
Marcha claudicante. La tensin arterial 150/130-90. El fondo de ojo
con angiopata angiotnica. El examen del cuello con escrfula en
regin cervical y cicatriz lineal en cara lateral derecha de cuello. En la
regin precordial a la inspeccin con presencia de choque de la punta
del corazn y a la auscultacin desdoblamiento del 2 ruido, y
disminucin del pulso en arteria braquial.
PREGUNTA
Cul es el diagnstico ms probable para este caso?
RESPUESTA
a.- Granulomatosis de Weneger
b.- Linfoma no Hodking
c.- Arteritis de Takayasu
d.- Panarteritis nodosa clsica
PREGUNTA
Cul no forma parte de los criterios diagnsticos para esta patologa?
RESPUESTA
a.- Edad menos de 40 aos
b.- Claudicacin de extremidades
c.- Disminucin de pulso de arteria braquial

CURSO ENARM CMN SIGLO XXI TEL: 36246001

PREGUNTA
Qu arteria es la que ms probablemente encontraramos afectada al
hacer la arteriografa?
RESPUESTA
a.- Cartida comn
b.- Subclavia
c.- Arteria temporal
d.- Arteria renal

PREGUNTA
Cul es el diagnstico ms probable para esta paciente?
RESPUESTA
a.- Arteritis de Takayasu
b.- Panarteritis nodosa
c.- Meningioma
d.- Arteritis de clulas gigantes
PREGUNTA
La paciente repentinamente nos refiere que tienen visin doble y
minutos despus dice que no ve nada. Cul es la conducta teraputica
mas adecuada en este momento?
RESPUESTA
a.- Ciclofosfamida
b.- Ciruga laser
c.- Manitol
d.- Esteroides a dosis altas
PREGUNTA
Cul de los siguientes no es un criterio diagnstico?
RESPUESTA
a.- VSG >55mm/h
b.- Perdida de 10Kg de peso
c.- Cefalea de reciente inicio
d.- Edad >50 aos

Pharmed Solutions Institute

PGINA 78

MANUAL DE TRABAJO DEL CURSO ENARM CMN SIGLO XXI


TUBERCULOSIS
CIENCIAS BASICAS: Es una enfermedad infecciosa que suele afectar a los pulmones y es causada por el complejo Mycobacterium tuberculosis (M.
tuberculosis, M. bovis, M. africanum). La infeccin por M. tuberculosis suele ser asintomtica en personas sanas, dado que su sistema inmunitario acta
formando una barrera alrededor de la bacteria. SALUD PBLICA: Considerada una emergencia mundial por la OMS, es prioridad casos complicados en
los extremos de la edad, multifrmacorresistencia y vnculo con el sida y la diabetes, e inmunocomprometidos (neoplasias, quimioterapias, trasplantes).
OMS informa que 1/3 de la poblacin mundial est infectada por Mycobacterium tuberculosis. Cada ao: >10 millones de casos nuevos y 3.5 millones de
defunciones por tuberculosis. En frica y este de Europa: incidencia es de >300 casos/100,000 habitantes por ao. La tuberculosis que responde al
tratamiento estndar tiene una tasa de curacin >95%, en multiresistencia es de 50-80%. Menos de 30% de los inmunocompetentes logran la curacin y
>del 50% muere dentro de los primeros 5 aos. En multiresistencia la tasa de mortalidad es de 90%. PATOGENIA: La transmisin de bacilos tuberculosos
se produce bsicamente por va area (tambin ingestin de leche de vaca infectada, contacto con personas enfermas baciliferas o animales bovinos
enfermos). Las personas infectantes eliminan bacilos a partir de aerosoles (tos, expectoracin) y la infecciosidad depende del nmero de bacilos
eliminados y la susceptibilidad del husped. Las partculas aerolizadas que contienen bacilos, son suficientemente pequeas para eludir la 1 barrera
defensiva (aparato muco-ciliar), para alcanzar los alveolos pulmonares, donde comienza la multiplicacin de los bacilos, son fagocitados por lo
macrfagos alveolares, se liberan multiples citocinas proinflamatorias, TNF (ocasiona fiebre y caquexia), IL-1 y IL-6, adems liberan proteasas, urocinasa,
implicadas en la destruccin tisular. Secundariamente los bacilos son transportados por los propios macrfagos a los ganglios regionales donde se
produce la respuesta inmunitaria mediada fundamentalmente por los linfocitos T (inmunidad celular). Perodo de incubacin que oscila entre 6 a 8
semanas. Puede ocurrir que antes del desarrollo de la respuesta inmunitaria celular se produzca una diseminacin va linfo-hematgena que d lugar a
siembra de bacilos en diversos tejidos: zonas apicales de pulmn, vrtebras, epfisis de huesos largos etc. que condicionen la evolucin ulterior a
enfermedad progresiva tras perodos largos de latencia. En la mayora de los casos de infeccin tuberculosa, hay una destruccin rpida de bacilos y no
se produce enfermedad, el nico indicio residual es la positividad de la PPD. El granuloma se forma por la interaccin del M. tuberculosis, con la
respuesta inmune del husped y liberacin tisular de proteasas; al inicio se hallan neutrofilos, mas tarde necrosis caseosa (tpica de granulomas
producidos por micobacterias) y finalmente calcificaciones. El bacilo no siempre es eliminado y permanece inactivo, causando una infeccin latente.
INFECCION PRIMARIA: Afecta pulmn, se adquiere al inhalar bacilo, se
desarrolla despus de una exposicin inicial, tiende a autolimitarse. Se
necesitan de 50-200 microorganismos para producir la infeccin. De las
personas expuestas al bacilo 30% se infecta y 5% desarrolla
enfermedad. Los bacilos inhalados causan alveolitis, adenopatas y
linfagitis, lo que se conoce como complejo primario de Ghon (cicatrices
en pulmn). Clnica; hemoptisis, dolor pleurtico, disnea, fiebre,
diaforesis nocturna y prdida de peso. Complicacin: TB endobronquial.
REACTIVACION: Es progresiva se desarrolla de un foco previo de TB, el
cual puede producir TB pulmonar o extrapulmonar por via hematgena.
Ms comn en inmunocomprometidos (IRC, DM, esteroides,
desnutricin, deficiencia de vitamina D. Clnica; tos, prdida de peso,
fatiga, fiebre y diaforesis nocturna. Complicaciones agudas; hemoptisis
y neumotrax. Cavitaciones en (20-45%), manifestacin radiolgica
(focal o en parches) ms comn consolidacin. Los aspergillomas se
desarrollan dentro de la cavitacin (20%). Hasta en 5% de pacientes
reactivacin la manifestacin principal es el tuberculoma (centro:
material caseoso y periferia histiocitos epiteliales y cel. Gigantes
multinucleadas). DEFINICIONES (Estndares para la atencin de la TB en
Mxico): Caso de tuberculosis: Persona en quien se establece el
diagnostico de tuberculosis pulmonar o extrapulmonar y se clasifica por
bacteriologa o estudio histopatolgico en caso confirmado o no
confirmado. Caso de tuberculosis confirmado: Toda persona con cuadro
clnico compatible con tuberculosis pulmonar o extrapulmonar que
cumpla adems cualesquiera de los siguientes criterios: 1. Aislamiento
de Mycobacterium tuberculosis por cultivo. 2. Resultado positivo en la
baciloscopia. 3. Deteccion de genes de micobacterias por mtodos de biologa molecular (PCR o ampliacin de RNA). Caso de tuberculosis no
confirmado: Toda persona con cuadro clnico compatible con tuberculosis pulmonar o extrapulmonar sin confirmacin por baciloscopa, cultivo o
estudios de biologa molecular pero presenta uno o mas criterios como: 1. Cuantificacin de adenosin desaminasa (ADA): los niveles de ADA parab TB
peritoneal y pleural, estn >70U/ml; para SNC >7U/l. 2. Antecedente epidemiolgico de convivencia con un caso bacilifero. 3. Reactor a la PPD, con o sin
antecedente de BCG. 4. Respuesta favorable al tratamiento antituberculoso. 5. Estudio histopatolgico compatible con TB. Y alguno de los siguientes
estudios de gabinete: A. TB pulmonar; las radiografas de trax PA y lateral pueden mostrar imagen de sndrome del lbulo medio, derrame pleural,
ensanchamiento mediastinal o patrn miliar. B. TB ganglionar; ultrasonido con imgenes de material calcificado y lquido. C. TB del SNC; la TAC de crneo
puede mostrar datos de aracnoiditis e hidrocefalia; la radiografa de crneo, datos de hipertensin endocraneana. D. TB osea o enfermedad de Pott: las
rx., de columna vertebral AP y lateral muestran destruccin de las vertebras dorsolumbares y rotoxifoescoliosis. E. TB geniturinaria: urografa excretora
muestra imgenes tortuosas debido a rigidez de urteres. F. TB abdominal; USG o TAC muestran ascitis e imgenes compatibles con tabicaciones. La
laparoscopia muestra lesiones granulomatosas ascitis y fibrina. DIAGNOSTICO: Bsqueda entre contactos de un caso de TB y en grupos o poblaciones de
alto riesgo. La baciloscopa se debe realizar en A) En cualquier muestra clnica excepto orina. B) En todos los casos probables, entre los contactos de un
caso, en grupos o poblaciones de alto riesgo, pacientes dados de alta que acudan con tos productiva, en 3 muestras sucesivas de esputo. C)
Independientemente de la edad, en quienes clnica y radiolgicamente, se sospecha TB, si la primera serie de 3 hubiera sido negativa. D) En el control del
tratamiento antituberculoso, con una muestra cada mes, la ultima al terminar el tratamiento E) En caso de sospecha de TB extrapulmonar, F) Sospecha
de TB en nios. G) Pacientes en tx., estrictamente supervisado, en quienes al cuarto mes, persiste baciloscopia positiva. H) Para confirmar fracaso de
tratamiento. Cultivo: En caso de sospecha clnica y radiolgica de tuberculosis pulmonar con resultado negativo de 6 baciloscopas en esputo, para saber
si es farmacorresistente. Prueba de la tuberculina (PPD), con derivado de protena puirificado, tiene un papel limitado en el diagnostico de TB activa.
Reactor al PPD, persona que a las 72hrs presenta induracin intradrmica de 10mm o ms en el sitio de la aplicacin. En menores de 5 aos con o sin
BCG, recin nacidos, nios desnutridos y personas inmunodeprimidas, la induracin de 5mm o mas. Clnico; TB pulmonar activa son tos, a veces con

CURSO ENARM CMN SIGLO XXI TEL: 36246001

Pharmed Solutions Institute

PGINA 79

MANUAL DE TRABAJO DEL CURSO ENARM CMN SIGLO XXI


esputo que puede ser sanguinolento, dolor torcico, debilidad, prdida de peso, fiebre y sudoracin nocturna. Radiologico: Linfadenopata; alteracin
ms comn nios (90-95%), adultos (40%), ms frecuentes unilaterales, paratraqueal derecho e hiliar y su frecuencia disminuye con edad. La
combinacin de ganglios hiliares calcificados y focos de Ghon, se conoce como Complejo de Ranke (sugiere TB previa, histoplasmosis). En infeccin
primaria: se puede observar focos de consolidacin generalmente en lbulo medio o inferior, derrame pleural (60%), ms unilateral, se presenta 3-7
meses despus de la exposicin inicial. En reactivacin: afeccin del segmento apical y posterior de lbulos superiores (85%), la cavitacin secundaria a
necrosis caseosa, se observa como nivel hidroereo; se pueden ver atelectasias lobares. TAC: Los ganglios muestran hipodensidad central (necrosis
caseosa) y reforzamiento perifrica, que representa el anillo vascular de tejido granulomatosos inflamatorio que indica enfermedad activa. En pacientes
de alto riesgo, se pueden formar cavitaciones, diseminacin hematogena y tuberculosis miliar. TUBERCULOSIS MILIAR: Se da por una infeccin
diseminada por va hematogena. Afecta de 1-7%, frecuente en ancianos, lactantes e inmunocomprometidos. Suele manifestase con fiebre, con
frecuencia existen granulomas pulmonares, afeccion del SNC, en raros casos sx. de dificultad respiratoria aguda con mortalidad de 90%. En casos
crnicos caquexia, lesiones drmicas maculares o papulares y tuberculos coroides (2-3mm), estos ltimos patognomnicos de tuberculosis miliar. RX:
Mltiples imgenes nodulares <5mm distribuidos en ambos pulmones difusas bilaterales, engrosamiento de septos interlobulillares. TB Y VIH: La
inmunodepresin favorece manifestaciones atpicas y extrapulmonares o inusuales de TB, al iniciar el tratamiento antirretroviral hay mayor riesgo de
desarrollar TB. Pueden desarrollar un sndrome de reconstitucin inmunolgica cuando se inicia el antiretroviral, por lo que primero se debe iniciar el
tratamiento antifimico. Antes de indicar tratamiento, se debe
CUADRO 2. Tratamiento primario acortado
realizar prueba de tuberculina. Las reactivaciones en estos
FASE INTENSIVA
Diario, de lunes a sbado, hasta completar 60 dosis o 2 meses.
grupos son 20 veces mayor. La TB aumenta la replicacin viral
Administracin en 1 toma
y la progresin de la enfermedad VIH. La TB se presenta
FARMACOS:
Dosis:
Rifampicina (R)
600mgs
independiente de la cuenta de CD4. La TB miliar y del SNC son
Isoniacida (H)
30mgs
ms frecuentes con cuentas de CD4 <200 cel/mm3. Todos los
Pirazinamida (Z)
1,500-2000 mg
pacientes VIH positivos, sin datos clnicos de TB, deben recibir
Etambutol (E)
1,200mgs
quimioprofilaxis con isoniacida. TRATAMIENTO: Debe de ser
FASE DE SOSTEN
Intermitente, 3 veces por semana, lunes, mircoles y viernes.
estrictamente
supervisado. Quimioprofilaxis, primario
Hasta completar 45 dosis o 4 meses. Administrar en una toma
acortado y retratamiento. Se considera TB multiresistente,
Frmacos:
Dosis:
cuando no es susceptible a isoniacida, ni rifampicina
Isoniacida (H)
800mgs
administradas simultneamente. Quimioprofilaxis: Por 6 meses
Rifampicina (R)
600mgs
a los contactos menores a 5 aos, con o sin antecedente de
BCG. Por 6 meses a contactos de 5-14 aos de edad, no vacunados con BCG, en quienes se haya descartado TB. Por 12 meses a contactos de 15 aos o
ms con inmunosupresin (previo descarte de TB pulmonar o extrapulmonar). El frmaco a usar es la isoniacida a dosis de 10mg/kg, sin exceder de
300mgs, en una toma diaria VO, estrictamente supervisada. El tratamiento acortado estrictamente supervisado (TAES), se instituye a todo caso nuevo y
al que lo reanuda posterior al primer abandono. TAES por 25 semanas, hasta completar 105 dosis. En 2 etapas: fase intensiva; 60 dosis, de lunes a sbado
con HRZE (ver cuadro 1, medicamentos 1ra lnea). Fase de sosten; 45 dosis (ver cuadro 2, intermitente 3 veces a la semana con HR), con frmacos en
combinacin fija y etambutol separado. En el acprtado no se usa etreptomicina. En tuberculosis de SNC, miliar u osea, el tratamiento es por un ao
dividido en 2 etapas, fase intensiva 2 meses diario de lunes a viernes con HRZE, fase de sostn 10 meses aproximadamente, L, M, V con HR, se alarga por
estar en luigares de poca penetrancia del frmaco. PREVENCION: Vacunacin; BCG, de bacilos Mycobacterium bovis vivos atenuados (bacilo de Calmette
y Guerin) inmunidad activa contra la TB, dosis 0.1ml (contiene como mnimo 200 000UFC), intradrmica, brazo der., se aplica a todos los recin nacidos y
hasta los 14 aos de edad cuando se considera necesario, disminuye incidencia de TB en SNC. Contraindicada, en recin nacidos con <2Kg, personas
inmunodeprimidas (excepto en infeccin por VIH en estado asintomtico), fiebre >38.5. NOTAS: La TB ganglionar es la forma ms de TB extrapulmonar.
La TB vertebral (torcica) o Enf. de Pott es la manifestacin ms frecuente de TB sea (50%), mas en varones.
CASO CLINICO TB TUBERCULOSIS CRONICA
Se trata de paciente masculino de 46 aos de edad, originario de la
regin selvtica de Chiapas, campesino, analfabeta el cual acude a
consulta debido a que se encuentra fatigado, lo cual le impide trabajar,
agrega perdida de peso y fiebre frecuente, adems refiere que tienes
tos continua con esputo ocacionalmente con trazas de sangre, a la
exploracin fsica, no se observa cicatriz de BCG, refiere consumo de
mezcal diario y tabaquismo 3 a 5 cigarrillos diarios, el ndice de masa
corporal actual es de 20.5, signos vitales dentro de parmetros
normales, a la auscultacin torcica se apresian ruidos respiratorios
anforicos posteriores de predominio superior derecho con estertores
aislados en la misma regin, no se observa cianosis ni hipocratismo,
resto sin datos por agregar.
PREGUNTA
Cual es la conducta a seguir mas adecuada?
RESPUESTA
a.- Internamiento en aislamiento ambiental hasta 3 expectoraciones
negativas.
b.- Internamiento sin aislamiento.
c.- Realizar biopsia e interconsulta a oncologa.
d.- Realizar prueba PPD.
CASO CLINICO TUBERCULOSIS MAS VIH
Se ingresa a paciente masculino de 45 aos de edad el cual cuenta con
antecedente de ser portador de VIH positivo con cifras de CD4 de 85,
actualmente suspendi el tratamiento antirretroviral, el motivo del
ingreso es debido a dolor en la regin lumbar intenso el cual mostro

CURSO ENARM CMN SIGLO XXI TEL: 36246001

datos de ostemielitis, el paciente cuenta con el antecedente de


presencia de tos productiva con esputo amarillento y sanguinolento,
perdida de peso importante, por lo que previamente se realizao
baciloscopia siendo positiva para tuberculosis, apoyada por los
parmetro radiogrficos.
PREGUNTA
Cual es el tratamiento inicial mas adecuado.
RESPUESTA
a.- Isoniazida, rifampicina, etambutol y pirazinamida.
b.- Isoniazida, rifampicina, etambutol y pirazinamida mas terapia
antirretroviral.
c.- Isoniazida, rifampicina, etambutol, pirazinamida y estreptomicina.
d.- Isoniazida, rifampicina y etambutol.
CASO CLINICO TUBERCULOSIS CUTANEA.
Paciente de 56 aos, sexo femenino, con antecedente de
dermatomiositis, en tratamiento con hidroxicloroquina 200 mg,
metotrexato 10 mg y prednisona 30 mg diarios. Fue hospitalizada por
cuadro de 10 das de evolucin, con fiebre, compromiso del estado
general y prdida de peso, encontrndose crepitaciones bilaterales al
examen pulmonar. La TAC de trax mostr un infiltrado nodular
bilateral, extenso, con cavitaciones biapicales. La baciloscopia obtenida
por lavado bronquio-alveolar (LBA) result positiva y la reaccin de
polimerasa en cadena (RPC) positiva para Mycobacterium tuberculosis.
Adems lceras en los pulgares aparecidas cuatro semanas previas al
ingreso. Al examen fsico se observ una lcera de bordes bien
delimitados con fondo sucio y bordes necrticos, no dolorosa, ubicada

Pharmed Solutions Institute

PGINA 80

MANUAL DE TRABAJO DEL CURSO ENARM CMN SIGLO XXI


en el pulpejo del dedo pulgar izquierdo. S. aureus y
Candidaparapsilosis. La tincin de Ziehl Neelsen en tejido fue positiva.
PREGUNTA
Cual es el origen mas probable que cause las ulceras en pulgares.
RESPUESTA
a.- Farmacologica.
b.- Bacteriana.
c.- Autoinmune.
d.- Mixta.
CASO CLINICO
Se trata de paciente femenino de 70 aos de edad la cual acude a
consulta debido a que presenta desde hace varios meses, tos no
productiva, fatiga, prdida de peso, la paciente cuenta con
antecedente de tuberculosis hace mas de 20 aos la cual fue tratada,
se logra obtener esputo para cultivo asi como sangre para cultivo, los
reportes de laboratorio informan presencia de bacilos
acidorresistentes que indican complejo Mycobacterium avium, la
radiografia de trax reporta bronquiectasias y ndulos pequeos
esparcidos a lo largo del parnquima pulmonar. La paciente no cuenta
con otros antecedentes de importancia para el cuadro clnico.
PREGUNTA
Cul es el esquema de tratamiento ms adecuado?
RESPUESTA
a.- Claritromicina y etambutol.
b.- Claritromicina y rifampicina.
c.- Levofloxacino y rifampicina.
d.- Prizinamida, isonizida, rifampicina y etambutol.
PREGUNTA
Cul es el dato menos probable de una reactivacin de tuberculosis?
RESPUESTA
a.- Infiltrados
b.- Consolidacin
c.- Cavitacion
d.- Granuloma
CASO CLINICO
Femenino de 31 aos. No presenta alergias medicamentosas
conocidas, es fumadora de 15 cigarrillos/da, no padece hipertensin
arterial ni dislipemia. Tiene antecedentes mdicos VIH positivo y hace
2 aos se someti a quimioprofilaxis incompleta durante 2 meses por
contacto tuberculoso. Presenta adems antecedentes quirrgicos hace
9 aos de quiste hidatdico en pulmn izquierdo. La paciente consulta
por un cuadro de fiebre diaria de predominio vespertino desde hace 1
mes junto con sudoracin nocturna. Desde el inicio de la fiebre
presentaba tambin astenia sin prdida de peso, tos, disnea o cefalea.
EF: Temp 37.4, consciente y orientada, eupneica, bien hidratada y
perfundida. No presenta lesiones cutneas, pero s pequeas
adenopatas supraclaviculares y latero-cervicales bilaterales menores
de 0,5 cm. La auscultacin cardiaca muestra tonos rtmicos sin soplos y
la auscultacin pulmonar la disminucin global del murmullo vesicular
sin ruidos aadidos. El abdomen es blando y depresible, no se palpan
megalias, no presenta signos de irritacin peritoneal. La exploracin
neurolgica no muestra focalidad neurolgica ni signos menngeos. Se
le solicit desde la consulta de Atencin Primaria analtica y radiografa
(Rx) de trax, donde se apreciaba un patrn intersticial retculonodular de predominio en bases pulmonares con tractos fibrosos e
hilios aumentados de tamao y con ausencia de derrame pleural.

CURSO ENARM CMN SIGLO XXI TEL: 36246001

PREGUNTA
Cul es la sospecha diagnstica ms probable para este caso?
RESPUESTA
a.- Tuberculosis renal
b.- Tuberculosis miliar
c.- Tuberculosis sea
d.- Tuberculosis primaria
PREGUNTA
Cul es la conducta ms adecuada a seguir en este momento?
RESPUESTA
a.- Manejo con antiretrovirales y antifimicos de manera simultanea
b.- Manejo con antiretrovirales primero y posteriormente antifimicos
c.- Manejo antifimicos primero y despus de 1 ao iniciar
antiretrocirales
d.- Manejo con antifimicos primero y posteriormente los
antiretrovirales
PREGUNTA
Cul es la conducta teraputica ms adecuada a seguir para el
tratamiento de la TB en este paciente?
RESPUESTA
a.- Fase intensiva de 2 meses de lunes a sbado (RHZE) ms fase de
sostn por 10 meses, lunes, mircoles y viernes (RH)
b.- Fase intensiva de 2 meses de lunes a sbado (RHZE) ms fase de
sostn por 4 meses, lunes, mircoles y viernes (RH)
c.- Fase intensiva de 2 meses de lunes, mircoles, viernes (RHZE) ms
fase de sostn por 10 meses, lunes a sbado (RH)
d.- Fase intensiva de 2 meses de lunes, mircoles, viernes (RHZE) ms
fase de sostn por 4 meses, lunes a sbado (RH)
PREGUNTA
Despus de 4 meses de tratamiento la paciente le refiere al mdico
que de repente ve borroso, y en algunas otras ocasiones a confundido
el color de los objetos y un poco de dolor de cabeza. Qu situacin es
ms probable que est sucediendo?
RESPUESTA
a.- La tuberculosis se ha diseminado a SNC
b.- Efecto adverso de isoniacida
c.- Sospechar tumoracin en SNC
d.- Efecto adverso de etambutol
PREGUNTA
Con la paciente vive un hermano de 18 aos, el cual tiene leucemia y
se descart que tenga tuberculosis. Cul sera la conducta ms
adecuada a seguir?
RESPUESTA
a.- No se da quimioprofilaxis, solo vigilancia
b.- Quimioprofilaxis por 6 meses
c.- Quimioprofilaxis por 12 meses
d.- Se inicia TAES
PREGUNTA
Cul sera la conducta diagnostica ms adecuada en un contacto con
fuerte sospecha de tuberculosis, pero que tienen 6 baciloscopias
negativas?
RESPUESTA
a.- Realizar un cultivo de tejido afectado
b.- Vigilancia, no tiene tuberculosis
c.- Realizar PPD
d.- Realizar tomografa

Pharmed Solutions Institute

PGINA 81

MANUAL DE TRABAJO DEL CURSO ENARM CMN SIGLO XXI


PALUDISMO
CIENCIAS BSICAS: Enfermedad parasitaria febril aguda causada por protozoarios del genero plasmodium que se trasmite a los humanos por la picadura
de las hembras infectadas de los mosquitos del genero anopheles. Las especies del plasmodium causantes son ovale, malaria, vivax y falciparum; estas
dos ltimas la de mayor distribucin en el mundo, el ms mortal P. falciparum. Los mosquitos se cran en agua dulce de poca profundidad y tienen
preferencia por los humanos sobre los animales, su transmisin depende de condiciones climticas, como el rgimen de lluvias, la temperatura y
humedad. Enfermedad endmica en poblaciones con clima tropical. SALUD PUBLICA: OMS; Aparecen aproximadamente de 300-500 millones de casos
nuevos cada ao, de los cuales mas de un milln resultan fatales. El 90% de las muertes atribuibles al paludismo ocurre en frica en nios menores de 5
aos. En Mxico el principal agente etimolgico es P. vivax, algunos casos aislados por falciparum, en estados de la frontera sur. La transmisin en
Mxico se concentra en costas del Pacifico, Chiapas, Oaxaca, Chihuahua, Baja California y Yucatn. La tasa de mortalidad por paludismo se ha reducido
>25% desde el ao 2000 a nivel mundial. VIH/SIDA + paludismo=altos ndices de morbimortalidad en zonas de alta prevalencia. PATOGENIA: Inicia con el
vector (hembra anopheles) infectada, transfusiones, uso compartido de agujas hipodrmicas contaminadas. El ciclo de vida de plasmodium se divide en
asexual (2 etapas) que se realiza en el humano y sexual en el mosquito. Ciclo asexual, etapa heptica: tras la picadura del mosquito este inocula el
esporozoito existente en su saliva en la sangre del husped, a travs del torrente sanguneo llegan a los hepatocitos, aqu se multiplican posteriormente
se rompe el hepatocito, apareciendo un nuevo estadio el merozoito, vuelven al torrente sanguneo donde penetran al eritrocito. Inicia etapa eritrocitica:
comienza a alimentarse de la hemoglobina, apareciendo entonces el trofozoito, nuevamente por esquizogenesis se multiplica en el interior de los
eritrocitos, a los cuales rompe liberando nuevos merozoitos y as continua el ciclo, pero algunos se convierten en gametocitos masculinos y femeninos.
Ciclo sexual: se da en la hembra anopheles, la cual adquiere el plasmodium mediante la picadura al humano infectado con gametocitos en sangre. Estos
gametocitos forman un cigoto en el intestino medio del insecto, el cigoto madura hasta formarse un ovocineto, que migra a la glndula salival del
insecto. Al destruirse los eritrocitos hay liberacin de sustancias del parasito y de hemates a la circulacin sangunea. Los eritrocitos parasitados su
membrana se vuelve rgida y se forman protuberancias que favorecen su adherencia al endotelio vascular en capilares de rganos vitales, fenmeno
conocido como "citoadherencia", sobre todo en cerebro, la adherencia produce enlentecimiento del flujo sanguneo, aumento local de citocinas,
glicolisis anaerobia y acidosis lctica. DIAGNOSTICO: Clnica: Los sntomas aparecen a los 7 das (10-15 das), de la picadura del mosquito, lo primero es
fiebre, acompaada de cefalea, escalofros, debilidad, insomnio, artralgias, mialgias y vomito. Toda persona sospechosa de paludismo con cuadro
escalofros (dura de 15-30 min, pulso dbil y rpido, piel algo ciantica), fiebre (inicio brusco puede llegar hasta 41.5 grados, puede delirios, convulsiones,
taquicardia, hipotensin) y sudoracin, que resida o provenga de reas endmicas se le debern realizar examen microscpico de gota gruesa de sangre
y a los contactos de los casos. La enfermedad tiende hacia la cronicidad, en donde existen periodos de recadas y latencia por aumento de la parasitemia.
Todos los nios de zonas endmicas con enfermedad grave suelen presentar anemia grave, esplenomegalia, sufrimiento respiratorio relacionado con la
acidosis metablica o paludismo cerebral. En adulto es comn afeccin multiorganica. Laboratorio: Realizar frotis de sangre, finas y de gota gruesa,
identificar parsitos y cuantificarlos, el porcentaje de eritrocitos con parsitos dar la medida de la gravedad del la enfermedad. Tcnicas
inmunocromatograficas, detectan el Ag del parasito en sangre. Se puede usar PCR para detencin de estos. Pacientes que sufren complicaciones o
enfermedad severa sobre todo por P. falciparum, pueden presentar hiperparasitemia >100 000 parsitos/microl de sangre con un >5-10% de eritrocitos
parasitados, hemoglobina debajo de 7g/dl, ictericia. COMPLICACIONES: Malaria cerebral, insuficiencia renal, fiebre biliosa hemoglobinurica, anemia
severa, edema pulmonar, dao heptico, hemorragia, coagulopatia. TRATAMIENTO: Actualmente se dispone de la combinacin de artesunato (4mg/kg)
y mefloquina en una sola tableta para falciparum, no complicado. Para P. vivax, el tratamiento de eleccin para cura radical es primaquina (0.25-0.5
mg/kg), es esquizonticida mas cloroquina (25mg/kg), son tabletas de 150mgs, por 3 das. La OMS considera que el frmaco de eleccin para P. vivax es
cloroquina, las combinaciones de artesunato, se recomiendan en casos de P. vivax resistente a cloroquina. La quinina, es un ezquizonticida hemtico muy
eficaz, gametocida eficaz contra P. vivax y P. malarie, inactiva frente a P. falciparum. Artemisina por via parenteral accin rpida empleado para P.
falciparum resistente a cloroquina, sus derivados son artesunato y artemer ambos en presentacin via oral y parenteral. La ONU recomienda no emplear
artemisina en monoterpia, porque acelera la resistencia, debe tratar e agregarse siempre algn otro antipaldico. Si no se trata en las primeras 24 hrs el
paludismo por P. falciparum, puede agravarse llevando a la muerte. En caso de P. vivax y ovale, pueden producirse recidivas clnicas semanas o meses
despus de la infeccin inicial, aunque el paciente haya abandonado la zona paldica. Estos nuevos episodios se deben a formas hepticas "durmientes"
del parasito (inexistentes en falciparum, malariae) y para lograr curacin completa es obligatorio tratamiento dirigido a esas formas hepticas.
PREVENCIN: La lucha antivectorial es el medio principal de reducir la transmisin del paludismo. Algunas medidas son: mosquitos tratados con
insecticidas (peritroides), fumigacin de interiores con insecticidas, vigilancia continua y eliminacin del mismo.
CASO CLINICO PALUDISMO
Masculino de 35 aos de edad residente del Estado de Mexico, inicia
padecimiento 9 dias posteriores a su regreso de la Ribera del Rio Niger
en Mal donde permaneci 30 dias, cursando con un cuadro clnico
caracterizado por fiebre de 40 C, de predominio vespertino y
nocturno terciada que se controlaba con paracetamol 500 mg cada 8
hrs. Al cuadro clnico se agrego estreimiento, disfagia, tos no
productiva, cefalea 8/10 holocraneana y perdida ponderal de 4 kg en
una semana. EF. Temp.39.2 C FC 127, FR 24, TA 122/72 mmHg,
somnoliento, orientado en tres esferas, mucosas secas, ruidos
cardiacos aumentados en frecuencia sin agregados, campos
pulmonares bien ventilados, abdomen distendido, doloroso a la
palpacin media y profunda en ambos hipocondrios, timpnico a la
percusin.
PREGUNTA
Cul es la conducta diagnostica que nos puede ayudar a cuantificar la
gravedad del paciente?
RESPUESTA
a.- El nmero de parasitos libres
b.- El porcentaje de eritrocitos parasitados
c.- El porcentaje de hepatocitos parasitados
d.- El Antigeno del parasito en sangre

CURSO ENARM CMN SIGLO XXI TEL: 36246001

PREGUNTA
A los 5 dias se observa ictericia y se realiza una Biometria hemtica la
cual reporta Hb 6.5mg/dl, Hto 28%, leucocitos 4,000. De que especie
es ms probable que se trate este caso?
RESPUESTA
a.- P. vivax
b.- P. malarie
c.- P. falciparum
d.- P. ovale
PREGUNTA
Cual es la conducta teraputica mas adecuada a seguir en el caso?
RESPUESTA
a.- Artesunato/mefloquina.
b.- Cloroquina.
c.- Cloroquina/mefloquina.
d.- Mefloquina
PREGUNTA
Cul patologa subyacente, le causara mayor morbilidad al paciente?
RESPUESTA
a.- Diabetes mellitus

Pharmed Solutions Institute

PGINA 82

MANUAL DE TRABAJO DEL CURSO ENARM CMN SIGLO XXI


b.- SIDA
c.- Tuberculosis
d.- Insuficiencia cardiaca
PREGUNTA
6 meses despus el paciente acude a revisin, clnicamente todo se
encuentra dentro de lo normal, sus laboratorios sin alteraciones. Cul
es el riesgo de recidiva para este paciente?
RESPUESTA
a.- No hay riesgo de recidiva
b.- 50% de recidiva a los 2 meses
c.- 10-20% de recidiva a los 6 meses
d.- Recidiva a las 2-3 semanas
CASO CLINICO
Ingresa paciente masculino de 48 aos de edad, ingeniero que acaba
de regresar de convencin en medio oriente, presenta fiebre,
diaforesis, ataque al estado generalizado, fatiga y adinamia, as como
obnubilacin, a la exploracin se observa hepato y esplegnomegalia
leve a moderada, es ingresado para realizar estudios encontrando
paludismo por plasmodium falciparum, parasitemia de 7 %,
hematocrito de 21 %, bilirrubina de 7.8 mg/100 ml, creatinina 2.6
mg/100 ml. Se ingresa posteriormente a cuidados intensivos, se realiza
asistencia ventilatoria y se indica neuroproteccin debido a la
gravedad del cuadro.
PREGUNTA
Cual es el mejor esquema de tratamiento para este caso?
RESPUESTA
a.- Quinina intravenosa.
b.- Quinidina Intravenosa.
c.- Artesunato Intravenoso.
d.- Cloroquina.

PREGUNTA
Cul es la conducta diagnostica ms adecuada a seguir en este
momento?
RESPUESTA
a.- PCR
b.- Tincin Gramm
c.- Gota gruesa
d.- BAAR

PREGUNTA
Cul es la especie ms probable en este caso?
RESPUESTA
a.- P. vivax
b.- P. falciparum
c.- P. malarie
d.- P. ovale
PREGUNTA
Cul es la conducta teraputica ms adecuada para la cura radical de
este paciente?
RESPUESTA
a.- Artesunato/mefloquina
b.- Cloroquina/primaquina
c.- Cloroquina/mefloquina
d.- Primaquina/artesunato

PREGUNTA
Qu complicacin seria menos probable observar en este caso?
RESPUESTA
a.- Insuficiencia renal
b.- Fiebre biliosa hemoglobinurica
c.- Coagulopatia
d.- Cirrosis heptica

PREGUNTA
Qu efecto adverso es menos frecuente encontrar tras la
administracin del medicamento?
RESPUESTA
a.- Visin borrosa
b.- atrofia de N. ptico
c.- Hipotensin
d.- Acidosis lctica

PREGUNTA
Cul es la forma infectante para el humano?
RESPUESTA
a.- Merozoito
b.- Trofozoito
c.- Esporozoito
d.- Esquistozito
CASO CLINICO
Masculino de 35 aos de edad, mexicano, residente de Tepotzotln
Estado de Mxico, soltero, diseador de jardines. El resto de sus
antecedentes sin importancia para el padecimiento actual. Inici 9 das
posteriores a su regreso del Oaxaca donde permaneci 30 das,
cursando con un cuadro clnico caracterizado por fiebre de ms de
40C de predominio vespertino y nocturno, que se controlaba
parcialmente con paracetamol 500 mg cada 8 h, antiinflamatorios no
esteroideos y antibiticos no especificados. Al cuadro clnico se
agregaron estreimiento, disfagia, tos no productiva, cefalea con

CURSO ENARM CMN SIGLO XXI TEL: 36246001

intensidad 8/10 holocraneana y prdida ponderal de 4 kg en una


semana, por lo que acudi a este hospital. A su ingreso, la exploracin
fsica revel temperatura de 39.2C, diaforesis, FC 127 x min, FR 24 x
min, TA 122/72mmHg, somnoliento, orientado en las tres esferas,
mucosas secas, ruidos cardiacos aumentados en frecuencia sin
agregados, campos pulmonares bien ventilados, el abdomen se
encontr distendido, peristalsis sin alteraciones, doloroso a la
palpacin media y profunda en ambos hipocondrios, timpnico a la
percusin, extremidades sin alteraciones aparentes. Laboratorios Hb
7.9mg/dl, Hto 29.9%, plaquetas 103,000, leucocitos 6,100, glucosa
117mg/dl, Cr 0.78.

PREGUNTA
A pesar de dar el tratamiento adecuado, el paciente no evoluciona
adecuadamente, su hematocrito sigue descendiendo as como sus
plaquetas y los cuadros febriles an no se controlan. Cul es la
conducta teraputica ms adecuada a seguir en este momento?
RESPUESTA
a.- Artesunato Intravenoso
b.- Mefloquina intravenosa
c.- Cloroquina intravenosa
d.- Quinina intravenosa

Pharmed Solutions Institute

PGINA 83

MANUAL DE TRABAJO DEL CURSO ENARM CMN SIGLO XXI


DENGUE:
CIENCIAS BASICAS: Enfermedad viral de carcter endmico-epidemico. Producido por un arbovirus (flaviviridae). Transmitido por Aedes aegypti (vector),
compuesto de ARN. El cual tiene 4serotipos (DE 1, 2, 3, 4), el 2 es el serio tipo ms peligroso. Cada serio tipo proporciona una inmunidad especfica para
toda la vida, as como inmunidad cruzada a corto plazo. Dos clases de dengue el clsico y el hemorrgico. SALUD PBLICA: Mxico ocupa el segundo
lugar en casos y defunciones de Amrica, despus de Brasil, mayormente entre 10-19 Aos, ligeramente ms en sexo femenino.OMS; Entre 50-100
millones por ao en el mundo, 500mil hospitalizados. 50mil muertes en ms de 100 pases y 2 mil millones de personas en riesgo. Mayor incidencia en
pocas de lluvia. En Mxico hay condiciones que propician la presencia de mosquitos, temperatura lluvias constantes, condiciones inadecuadas de
saneamiento e higiene. PATOGENIA: EL virus ingresa al organismo por la picadura del artrpodo se replica en los macrfagos y monocitos, lo que
produce supresin de la medula sea. El periodo de incubacin es de 5-7 das. Se desarrolla una respuesta inmune que acaba en el periodo de virema e
induce produccin duradera. Existen 3 etapas: 1. Febril; se da entre el 3-6to das, es variable, presencia de virus en sangre (virema). En esta etapa no es
posible reconocer si el paciente va a evolucionar a la curacin espontnea o si es el comienzo de un dengue grave. Al disminuir la fiebre, el dolor
abdominal se hace intenso y mantenido, se puede observar derrame plural o ascitis. 2. Crtico; momento de mayor frecuencia de instalacin de choque.
Hay extravasacin de plasma (vasodilatacin), su manifestacin ms grave el choque, grandes hemorragias digestivas, alteraciones hepticas y quiz de
otros rganos, ascitis o derrame pleural derecho o bilateral, aumento de hematocrito (la mxima elevacin de este coincide con el choque) y
disminucin de plaquetas. 3. Recuperacin: evidente mejora del paciente a veces hay sobrecarga de liquido, as como coinfeccion bacteriana. Esta etapa
es la de mayor riesgo de aparicin de complicaciones, hay que vigilar y controlar al paciente 48hrs posteriores al cese de la fiebre, los signos de alarma
son: dolor abdominal intenso y continuo, vmitos frecuentes, somnolencia o irritabilidad, derrame seroso (en peritoneo, pleura o pericardio), sangrado
de mucosa, hepatomegalia (>2cm), aumento rpido de hematocrito y disminucin rpida de plaquetas. DIAGNOSTICO: Dengue clsico: Inicio repentino,
fiebre (de menos de 7 das) bradicardia, pulso lento, exantema mculas puntiforme, rinitis, tos, ardor farngeo, adenopatas, fatiga, cambios en el sentido
del gusto, anorexia, cefaleas, mialgias, artralgias, dolor seo y retroorbitario, congestin conjuntival, edema palpebral, petequias, equimosis,
miocarditis. Dengue hemorrgico: Mas grave, incremento en permeabilidad vascular. El choque depende de mltiples factores como la presencia de
anticuerpos no neutralizantes. Dengue grave; shock hipovolemico por fuga de plasma, diestres respiratorio por acumulacin de lquidos, sangrado grave,
dao orgnico importante. BH; leucopenia y trombocitopenia, a veces transaminasas elevadas. ELISA, para determinacin de anticuerpo IgM e IgG
antidengue, se puede tomar prueba pareada una en fase aguda y otra en convalecencia. PCR o ELISA se usan para deteccin de antgenos virales.
Anticuerpos IgM especficos para virus de dengue: Si es negativo se descarta el diagnostico de dengue; si es positivo es un caso probable de dengue, se
requiere una segunda muestra para confirmar mediante prueba de neutralizacin; negativo=se descarta caso de dengue, constante=caso de dengue
anterior, positivo=se confirma dengue. CASO PROBABLE DE FIEBRE HEMORRGICA POR DENGUE: Toda persona que adems de un cuadro de probable
fiebre por dengue desarrolle fiebre persistente y uno o ms de los siguientes: A) Datos de fuga de plasma (ascitis, derrame pleural, edema,
hipoalbuminemia). B) Datos de fragilidad capilar (petequias, equimosis, hematomas). C) Hemorragias (gingivorragia, hematemesis, metrorragia). D)
Tombocitopenia <100 000 plaquetas/ml3 o hemoconcentracin. CLASIFICACIN: Del dengue hemorrgico: Grado I; fiebre y sntomas constitucionales no
especficos, prueba del torniquete positiva (nica manifestacin hemorrgica). Grado II; grado I ms petequias, epistaxis, hematemesis, melena. Grado
III; insuficiencia circulatoria (taquicardia, hipotensin, disminucin de la presin diferencial de pulso). Grado IV; choque, pulso y tensin arterial no
detectable. TRATAMIENTO: No especifico, solo de sostn. El dengue sin signos de alarma, ni comorbilidades, manejo ambulatorio (reposo, aislamiento
de mosquitos, lquidos, paracetamol, no aspirina, AINES, ni corticoides, evaluar mejora). Dengue con comorbilidades, estricto ambulatorio o internacin
en sala general (hidratacin va oral o IV con cristaloides a 2-3 ml/kg, mas medidas anteriores, buscar signos de alarma hasta 48hrs de cese de fiebre,
hemograma diario). Dengue con signos de alarma, sin criterios para dengue grave, (obtener hematocrito antes de expandir al paciente, fisiolgica o
Ringer 10ml/kg/hr, en una hora, repetir hto despus de cada carga y evaluar signos de alarma, mejora reducir goteo, si empeora, aumenta hto y caen
plaquetas, manejarlo como dengue grave). Pacientes con signos de alarma y criterios para dengue grave (tomar hto, iniciar SF o Ringer lactato a 20ml/kg
en 15-30 min, evaluar si hay mejora disminuir a 10ml/kg, si mejora ir disminuyendo lquidos, de no mejorar continuar con coloides, no mejora
considerar uso de drogas vasoactivas, si paciente mejora y hto baja, indica sangrado y requiere transfundir, tx. De hemorragia segn criterio) Soluciones
intravenosas (para reponer prdidas producidas por extravasacin de plasma, a veces se agrava por perdidas del exterior; sudoracin, vomito, diarrea)
y requerimientos transfucionales. PREVENCIN: No se dispone de vacunas, ni antivirales especficos. La nica forma para controlarlo, es mediante el
control del mosquito del dengue y evitar su picadura. Ms de 50% de criaderos de mosquitos se sita, en tiraderos de llantas y panteones.
CASO CLINICO DENGUE
Femenino de 18 aos de edad originaria de Tepic, Nayarit camarera,
ingreso por hipotensin e hipotermia, refiere que 48 hrs previas al
ingreso inicio con cuadro de cansancio, fiebre, dolor osteomuscular
generalizado, dolor retro-ocular y mal estado general, Hg 14.6 Htc
44.7, plaquetas de 51,000/mm y leucocitos 15,200/mm se enviaron
muestras para serologa, dos horas despus la Hb 13.7, Htc 43%,
plaquetas de 31,000 a las 5 horas la tensin arterial era de 80/50, FC
100, FR 24, FC 100, temperatura de 35 C.
PREGUNTA
Cul es la condcuta teraputica mas adecuada a seguir?
RESPUESTA
a.- Cristaloides y aminas.
b.- Solucion coloides, aminas vasoactivas.
c.- Aminas vasoactivas, hemoderivados.
d.- Cristaloides, aminas y hemoderivados.
PREGUNTA
Cul es el dato menos probable que nos indique gravedad en esta
paciente?
RESPUESTA
a.- palquetas bajas
b.- Descenso de hematocrito

CURSO ENARM CMN SIGLO XXI TEL: 36246001

c.- TA 80/50mmHg
d.- Dolor retro-ocular
PREGUNTA
Dias despus el paciente, continuo con fiebre a pesar de manejo,
presenta ascitis y derramme pleural, petequias y contina con
trombocitopenia. Ante que cuadro es ms probable que nos
encontremos?
RESPUESTA
a.- Dengue hemorrgico
b.- Fiebre hemorrgica por dengue
c.- Dengue clsico
d.- Complicaciones por dengue
CASO CLINICO DENGUE
Masculino de 38 aos de edad el cual inicio 5 dias previo a su ingreso
con debilidad, hiporexia, fiebre, nasea y vomito, el paciente
presentaba dolor torcico, TA 40/30 mmHg, FC 120, FC 24, Temp 35. La
piel marmrea, mucosas secas, Rx de trax con datos de derrame
pleural, torax hipoventilados, dolor epigstrico, Hb 12.8, Htc 51 %,
plaquetas 79,000, leucocitos 10,500.
PREGUNTA
Que criterios de gravedad de dengue grave presenta?

Pharmed Solutions Institute

PGINA 84

MANUAL DE TRABAJO DEL CURSO ENARM CMN SIGLO XXI


RESPUESTA
a.- 1.
b.- 2.
c.- 3.
d.- 4.

a.- Demostracin o prueba de NS1+.


b.- Elevacin cuatro veces o ms de ttulos de IgG o IgM contra algn
antgeno de virus de dengue en muestras pareadas de suero.
c.- Presencia de antgeno circulante en sangre del virus de dengue
determinada mediante ELISA, o de antgeno en hgado por
inmunohistoqumica (inmunoperoxidasa) o inmunofluorescencia.
d.- Deteccin de secuencias genmicas de virus de dengue mediante
prueba de PCR.

PREGUNTA
En que grado de Dengue se encuentra el paciente?
RESPUESTA
a.- Grado I
b.. Grado II
c.- Grado III
d.- Grado IV
PREGUNTA
Se pasaron soluciones y posteriormente coloides al paciente sin
observar mejora, Cul seria la conducnta mas adecuada a seguir en
este momento?
RESPUESTA
a.- Transfusion sangunea
b.- Aminas vasoactivas
c.- Continuar con coloides
d.- Pasar plasma
CASO CLINICO DENGUE
Se trata de una mujer de raza negra, de 21 aos de edad, que present
un cuadro febril agudo de 39-40 C, seguido en los das sucesivos de
una erupcin cutnea pruriginosa generalizada. Tras buscar asistencia
en un centro peditrico de su regin, se diagnostic clnica y
serolgicamente de dengue clsico y se trat sintomticamente con
paracetamol. Una semana despus, una vez que los sntomas de la fase
aguda se encontraban en una fase casi completa de regresin,
comenz a sentir dificultad para articular el lenguaje y para deambular,
disfagia, debilidad muscular progresiva en los cuatro miembros y
disminucin del nivel de conciencia.
PREGUNTA
Cual es la conducta diagnostica mas apropiada para identificar la causa
de las complicaciones.
RESPUESTA
a.- IRM.
b.- TAC.
c.- Pruebas de funcionamiento heptico.
d.- Biometia hemtica.
CASO CLINICO DENGUE
Se trata de una mujer de 50 aos fumadora ocasional. Entre sus
antecedentes personales de inters, destacaba la realizacin de un
viaje turstico de nueve das de duracin a la Rivera Maya, quince das
antes del inicio de los sntomas. Durante el viaje la paciente, sufri
numerosas picaduras de mosquitos. En las 48 horas previas a la
consulta, la paciente se autotrat con cido acetilsaliclico (AAS) como
antipirtico y como tratamiento sintomtico de la cefalea. En la
exploracin fsica la paciente estaba consciente, orientada y febricular
(37,6C), TA=110/70. Exista rubor facial e inyeccin conjuntival
bilateral as como la presencia de un exantema hemorrgico,
simtrico, mculo-papular en tronco y extremidades, especialmente en
las inferiores, donde era confluente, y que no se modificaba a la
vitropresin
PREGUNTA
Cual es la conducta mas adecuada para establecer el diagnostico.
RESPUESTA

CURSO ENARM CMN SIGLO XXI TEL: 36246001

CASO CLINICO
Mujer de 21 aos, residente en Argentina, que realiz un viaje a Brasil,
donde permaneci 1 semana. Al segundo da de su estada presenta
diarrea acuosa que se autolimita en 48 hs y lesiones maculopapulares
pruriginosas en miembros inferiores. Al sptimo da regresa a
Argentina y dos das posteriores presenta fiebre de 38C, acompaada
de mialgias, cefalea frontal y vmitos. A las 72 hs del inicio de la fiebre
present hematemesis, melena, epistaxis y dolor abdominal
epigstrico que motivan la hospitalizacin. El examen fsico mostr T
axilar de 37,6C, F.C: 90/min, F.R: 20/min, TA: 110/60 mm de Hg.,
mltiples petequias en paladar y conjuntivas, restos hemticos en
fosas nasales, prueba del lazo positiva y lesiones mculo-papulares en
ambas piernas de 1,5 cm de dimetro pruriginosas, sin
visceromegalias. Exmenes complementarios: Hematocrito: 31,6%,
glbulos blancos 7.840/mm3 (Neutrfilos 18%, Linfocitos 60%,
monocitos 18%), con linfocitos medianos hiperbasfilos, plaquetas
94.000/mm3, Test de Coombs negativo.
PREGUNTA
Cul es el diagnstico ms probable para esta paciente?
RESPUESTA
a.- Paludismo
b.- Salmonella Typhi
c.- Dengue
d.- Staphylococcus aureus
PREGUNTA
Cul es la conducta teraputica ms adecuada a seguir?
RESPUESTA
a.- Cristaloides, aminas, cloroquina, primaquina
b.- Cristaloides, transfundir plasma, oxacilina
c.- Aminas vasoactivas y coloides
d.- Cristaloides, Hemoderivados
PREGUNTA
Con que prueba diagnstica si es negativa, descartaramos la
patologa de esta paciente?
RESPUESTA
a.- IgM
b.- IgG
c.- Gota gruesa
d.- PCR
PREGUNTA
Despus de 5 horas que es revalorada la paciente se encuentra con un
hto de 38, Hb 13mg/dl plaquetas de 45,000, tensin arterial de
90/50mmHg, FC 98x, y en radiografa de trax se observa derrame
pleural. Cul es la conducta ms adecuada a seguir en este momento?
RESPUESTA
a.- Coloides y hemoderivados
b.- Ingreso a UCI
c.- Cristaloides, aminas vasoactivas
d.- Sala general, cristaloides

Pharmed Solutions Institute

PGINA 85

MANUAL DE TRABAJO DEL CURSO ENARM CMN SIGLO XXI


LEPRA (MYCOBACTERIUM LEPRAE):
CIENCIAS BASICAS: Enfermedad granulomatosa multisistmica crnica causada por Mycobacterium leprae parasito intracelular obligado, no mvil, no
espolurado, que afecta hgado, piel, mucosas de las vas respiratorias altas, testculos (disfuncin sexual, atrofia), ojos y nervios perifricos. SALUD
PBLICA: Actualmente existen zonas de alta incidencia en frica, Asia y Amrica Latina. En Mxico 21 estados presentan incidencia, los de mayor son
Sinaloa, Michoacn, Nuevo Len, Guerrero y Nayarit. La razn hombre: mujer 2:1, ms comn en >25 aos. PATOGENIA: El modo de transmisin e
invasin sigue siendo controvertido. Es de notar que pacientes con lepra-lepromatosa la mayor diseminacin del bacilo al ambiente se realiza a travs de
la descarga nasal, el estornudo y la tos; por ello se plantea que la principal ruta de infeccin es la via area (microgotas), la diseminacin a travs de la
piel es menos importante. Incubacin de 2-4 aos o ms. Para adquirir la infeccin se requiere la convivencia estrecha y prolongada con un enfermo
bacilifero y con inmunidad celular disminuida identificada. DIAGNOSTICO: Clnica; En piel maculas, ppulas, ndulos, placas o infiltraciones, afectando
especialmente macrfagos. La hipopigmentacin o eritema de la piel con dficit sensorial (hiperestesia, hipoestesia o anestesia), es uno de los signos
clnicos ms importantes en el diagnostico, se observan en la palma de la mano o planta del pie, y los origina una infeccin de la bacteria en las fibras
nerviosas (engrosamiento de troncos nerviosos perifricos), alteraciones motoras. El dao al sistema nervioso es una caracterstica, a pesar de no tener
habilidad locomotora, puede moverse en el endotelio a travs del tejido conectivo y alcanzar las clulas de Schwan. Las pruebas inmunolgicas son muy
limitadas, no existe un examen confirmatorio, baciloscopia positiva con resultado completo, deteccin de anticuerpos anti-PGL-I, donde se presentan
grandes cantidades de IgM. ELISA, pero tienen sensibilida y especificidad baja. El diagnostico histopatolgico es obligatorio para el pronstico y de esta
manera favorecer el tratamiento, se pueden obsrevar infiltrados en la dermis, hipodermis y rganos internos tales como clulas de Virchow, que son
macrfagos con muchos bacilos y gotas de lpidos en su citoplasma, con apariencia espumosa. En secreciones nasales y cutneas el hallazgo del bacilo
aislado en forma de globia puede ser detectado cpon coloracin de Zieehl Neelsen. CLASIFICACION: Para fines de control sanitario los casos se clasifican
en: Multibacilares (MB); los lepromatosos, dimorfos y con infiltracin difusa . Paucibacilares (PB); los tuberculoides e indeterminados. Clasificacin
clnica:
Caractersticas

Numero de lesiones

5 LEPROMATOSA
(progresa a necrosis y ulceracin)
Maculas, ppulas, ndulos, infiltracin
difusa
Numerosas

3
BODERLINE
LEPROMATOSA
Maculas, ppulas, ndulos,
infiltracin
Muchas

4 BODERLINE
BODERLINE (dimorfa)
Placas y lesiones en forma de
cpula y sacabocados
Muchas

Distribucin de las
lesiones
Definicin de las
lesiones

Simtricas (afecta ms labios, encas,


dorso de lengua, paladar duro)
Imprecisas, difcil
de definir la
enfermedad y la afeccin de la piel

Con tendencia a la simetra

Evidentemente Asimtricas

Imprecisas delimitadas por


los bordes externos

Imprecisas delimitadas por los


bordes externos mal definido

Sensibilidad
Bacilos
en
las
lesiones en la piel

No afectada
Muchas globias

Disminuida
Muchos

Disminuida
Muchos

Tipo de lesin

2 BODERLINE TUBERCULOIDE

1 TUBERCULOIDE

Placas de infiltrados

Placas de infiltrados

nica, usualmente con lesiones


satlites o >5 lesiones
No difusas y asimtricas

<5 lesiones

Bien definidas, bordes bien


delimitados, eritematosas o
cobrizas
Ausente
+/-

Bien definidas bordes


bien delimitados

Asimtricas

Ausente
Negativa

TRATAMIENTO: Los esfuerzos globales para controlar la lepra mediante la poliquimioterapia han permitido disminuir significativamente el nmero de
pacientes infectados. Sin embargo la deteccin de casos nuevos y las estrategias de control no han logrado la eficacia esperada. Casos multibacilares usar
dosis mensual supervisada de rifampicina, clofazimina, dapsona y dosis diaria autoadmnistrada de dapsona y clofazimina. Para casos paucibacilares usar
en dosis mensual supervisada (como mnimo 6 dosis mensuales) rifampicina y dapsona y dosis diaria autoadministrada (mnimo 162 dosis) de dapsona.
Usar hasta desaparicin de lesiones.
CASO CLINICO LEPRA
Masculino de 54 aos de edad, constructor, nacido en Tamaulipas y
radicado en Veracruz. Presenta dermatosis diseminada caracterizada
por ndulos de diferentes tamaos desde 3 x 3 hasta 8 x 8 mm,
algunos de coloracin de la piel y otros eritematosos en tronco y
extremidades superiores e inferiores de 8 meses de evolucin.
PREGUNTA
Cul es el diagnostico mas probable?
RESPUESTA
a.- Lepromatosa.
b.- Borderline lepromatosa.
c.- Borderline tuberculoide.
d.- Tuberculoide.
CASO CLINICO LEPRA
Masculino de 59 aos de edad originario de boca de rio, acude por
presencia de dermatosis diseminada con ndulos de 5 a 8 mm en cara,

tronco, miembros torcicos y plvicos de un ao de evolucin, con


manchas residuales hipercrmicas, con eritema mnimo en ambas
extremidades plvicas. Se detect alopecia en tercio distal de ambas
cejas y madarosis bilateral. La investigacin seriada de bacilos alcohol
cido resistentes en linfa y moco nasal fueron positivas. Se agregaban
descamacin plantar y onicodistrofia de dedos gordos de ambos pies.
El paciente cursaba con reaccin leprosa tipo II adems de calosfros,
artralgias maleolares y epistaxis de repeticin. La exploracin fsica
detect alteraciones de sensibilidad y motoras en dedos de ambas
manos e insuficiencia venosa en ambos miembros plvicos.
PREGUNTA
Que tipo es?
RESPUESTA
a.- Lepromatosa.
b.- Borderline lepromatosa.
c.- Borderline tuberculoide.
d.- Tuberculoide.

TRIPANOSOMIASIS (ENFERMEDAD DE CHAGAS Y DEL SUEO):


CIENCIAS BSICAS: Chagas o trypanosomosis americana, es una infeccin sistmica causada por el protozoario flagelado Trypanozoma cruzi. Su
distribucin en las zonas ms marginadas del continente americano y su curso crnico, degenerativo y muerte repentina, hacen de esta enfermedad una
gran carga para la economa y la salud. Es una zoonosis en la que participan un gran nmero de reservoros vertebrados (perros, ratas, gatos, zarigueyas)
y transmisores triatominos (chinche). SALUD PUBLICA: Elevada prevalencia e incurabilidad. Se estima que en la regin de las Amricas, se presenta en
21 pases, afecta a unos 7-8 millones de personas, y se encuentra en riesgo de adquirir la infeccin aprox. 7-8 millones de personas, con 56,000 nuevos
casos anuales y 12,000 muertes por ao. En Mxico es endmica en el estado de Veracruz. PATOGENIA: La infeccin se transmite principalmente por la
"chinche besucona" otros modos de transmisin son; transfucional, congnito, trasplante de rganos y oral (alimentos contaminados con heces del
artrpodo). La chinche durante la picadura defeca en la piel del hospedero, despus de ingerir los parsitos (en su forma de tripomastigoto til para dx.)
en la sangre de un ser humano, estos se transforman en tripomastigotes metaciclicos que son la forma infectante, estos son excretados en las heces del
parasito, que al ponerse en contacto con la conjuntiva o laceracin en la piel, con el rascado se facilita la entrada del patgeno al torrente circulatorio.
Las primeras clulas invadidas por los tripomastigotes son los histiocitos, donde se transforman en amastigotos (intracelulares, replicativo) y desarrollan

CURSO ENARM CMN SIGLO XXI TEL: 36246001

Pharmed Solutions Institute

PGINA 86

MANUAL DE TRABAJO DEL CURSO ENARM CMN SIGLO XXI


varios ciclos de fisin binaria. Casi cualquier clula puede ser invadida, a excepcin de las neuronas. La infeccin persiste en el cuerpo humano de por
vida. Las Reactivaciones en pacientes con VIH, nos pueden dar abscesos enceflicos. DIAGNOSTICO: Espectro clnico en 3 fases: Fase aguda (70% se
encuentran en esta fase); la incubacion de 14 dias y dura de 2-4 meses, asintomtica, se caracteriza por alta parasitema e invasin tisular
multiparenquimatosa, puede presentarse el llamado "chagoma de inoculacin", ndulo subcutneo con adenitis regional en el sitio de la picadura,
cuando el sitio de entrada es la conjuntiva se conoce como signo de Romaa (edema bipalpebral unilateral, conjuntivitis y linfadenitis preauricular), esta
fase se manifiesta con fiebre, linfadenopatas, hepatoesplenomegalia y mal estado general, se pueden presentar complicaciones como miocarditis aguda,
meningoencefalitis, 5% de nios fallece en esta etapa. Fase indeterminada; duracin variable sin parasitemia detectable, se han reportado anomalas
anatmicas y funcionales, serologa positiva. Fase crnica; hay compromiso visceral irreversible: cardiomiopata chagasica (mocardiopata dilatada,
arritmias severas, tromboembolia, bloqueos, principal causa de muerte), o de tubo digestivo, con la mayor frecuencia en intestino o esfago. Las
manifestaciones incluyen disnea de grandes a pequeos esfuerzos, palpitaciones, edema en miembros inferiores, dolor torcico, cuadros sincopales. La
destruccin de clulas ganglionares de la submucosa de tracto digestivo, dan lugar aperistalsia, retencin de residuos y dilatacin de rganos
(megaesofago-neumonia, megacolon-sepsis), puede haber disfagia, pirosis, dispepsia. Laboratorio: en etapa aguda bsqueda de Trypanozoma cruzi en
sangre con anticoagulante, por examen directo y tincin de extendidos de sangre con Giemsa. En etapa crnica, la parasitemia es transitoria, por ello
diagnostico mediante el hallazgo de anticuerpos circulantes de T. cruzi, con hemaglutinacin directa, ELISA, inmundo fluorescencia, adems contemplar
microscopia de gota gruesa o extendida, PCR, xenodiagnostico indirecto y hemocultivo. Estudios para funcin cardiaca; ECG, ecocardiografia, Holter,
cateterismo, biopsia. TRATAMIENTO: Beznidazol; 5-7.5 mg/kg/da en fase aguda por 60 dosis, casos congnitos y pacientes en etapa crnica, efecto
adverso neuropata y leucopenia. Nirfurtimox; 8-10mg/kg/dia en 4 dosis al dia 90-120 dosis, en fase aguda (70% de curacin) y crnica (20% de
curacin). Dichos medicamentos, sin embargo no logran erradicar el parasito, ni previenen la progresion de la enfermedad una vez que esta se ha
establecido. En la mocardiopata, se utilizan digitalicos, antiarritmicos, diurticos, marcapasos, trasplante cardiaco. PREVENCIN: La mejor solucin a
largo plazo, consiste en la eliminacin de los transmisores mediante el mejoramiento de viviendas, cuya construccin a base de adobe permite la
aparicin de grietas que albergan a los insectos, empleo de insecticidas (piretroides), medidas de educacin en salud. Tamizaje obligatorio en bancos de
sangre y a mujeres embarazadas en regiones endmicas. CIENCIAS BASICAS: Tambin llamada Enfermedad del sueo, es una parasitosis transmitida por
un vector. ocurre en frica es transmitida a los seres humanos por la picadura de la mosca tsetse (mosca de la fruta), que ha contrado la infeccin de
personas o animales que albergan los parasitos patgenos para el ser humano. Las personas ms expuestas personas que se dedican a la agricultura, la
pesca, la ganadera o la caza. Trypanosoma brucei, rhodesiense, gambiense causa 95% de los casos de esta enfermedad y es una infeccin crnica,
generalmente asintomtica, los sntomas aparecen cuando la enfermedad ya esta muy avanzada, en la etapa de afeccin al SNC. Trypanosoma brucei
rhodesiense causa 5% y causa una infeccin aguda, tambin afecta SNC. SALUD PBLICA: en los ltimos 10 aos, mas del 70% de los casos notificados
ocurrieron en la Repblica Democrtica del Congo, en 2010 esta reporto ms de 500 casos nuevos por ao. PATOGENIA: Transmisin principal por
picadura de mosca, tambin atraves de la placenta, con agujas contaminadas. En la primer etapa, los tripanosomas se multiplican en los tejidos
subcutneos, la sangre ya la linfa; se conoce como etapa hemolinfatica y se caracteriza por episodios de fiebre, cefalea, dolores articulares y prurito. En
la segunda etapa, los parasitos atraviesan la barrera hematoencefalica e infectan el SNC. DIAGNOSTICO: Fase I: Chancro tripanosomico, linfadenopatia
generalizada indolora, esplenomegalia importante, endarteritis con infiltracin de por parasitos y linfocitos, miocarditis, pancitopenia, IgM elevada,
lesion histica mediada por inmunocomplejos. Fase II invasin al SNC; indiferencia progresiva y somnolencia diurna, mirada indiferente, lenguaje confuso,
ataxia, temblor, rigidez, coma y muerte.Exudado de chancro al microscopio, tinion de Giemsa. Tamizaje de una posible infeccin (pruebas serologicas y
exploracin Clinica), diagnostico de la presencia del parasito, puncin lumbar y estudio del LCR, pleocitosis, tripanosomas, protenas elevadas.
TRATAMIENTO: Depende de la etapa, cuanto mas pronto se inicie, mejores las probabilidades de curacin. En la primera etapa: suramina o eflornitina,
alterantivo pentamidina. Tratamiento en la segunda etapa deben ser medicamentos que atraviesen la barrera hematoencefalica como: melarsoprol,
eflornitina, nifurtimox.
CASO CLINICO
Femenino que ingresa por astenia, somnolencia de varios das de
evolucin, sin determinar tiempo, acompaada de fiebre de 38.5 C,
las mucosas estn hipocoloreadas, sin otros sntomas y signos
acompaantes que ligeras adenopatas cervicales bilaterales, una
punta de bazo y discreta rigidez de nuca. Al da siguiente de su ingreso
se constata fiebre de 39 y 40 C, est aptica, astnica, deja de comer
y de hablar pasa a un estado estuporoso y en horas de la madrugada
tiene convulsiones tnico-clnicos generalizadas, se observan
temblores que se repetan igual que las convulsiones, a pesar de las
medidas antitrmicas y anticonvulsinantes; se muestra quejumbrosa
en oportunidades, posteriormente se comporta muy intranquila y con
gran agitacin psicomotora; cae en una fase ms profunda del coma,
no responde sino a estmulos profundos, tiene rigidez de
descerebracin. Se constata Kerning positivo y marcada rigidez nucal.

CASO CLNICO CHAGAS


Masculino de 42 aos de edad, agricultor. Motivo de consulta por
presentar hace mas menos 15dias dolor de barriga con sensacin de
plenitud abdominal, estreimiento, dolor a la deglusion, edema de
miembros plvicos, casi todos los dias faltade aire se siente cansado
con dolor en el pecho que aparece a pequeos esfuerzos aunque no es
de mucha intensidad y desaparece con el reposo. Por lo cual se decide
su ingreso paramejor estudio. Refiere que hace mucho tiempo lo pico
un insecto y tuvo inflamacin en un ojo pero no sabe con exactitud
que insecto fue ni la fecha. Condiciones de la vivienda: pesima. Casa
con piso de tierra agua de pozo y letrina a distancia. EF: disnea,
cardiovascular, dolor precordial simple ligero a moderado que
disminuye con el reposo, edema en miembros inferiores, astenia, dolor
en toda la regin abdominal, colicos, disfagia, constipacin, disuria,
cefalea, anorexia.

PREGUNTA
De acuerdo a la etapa. Cual es el tratamiento de eleccin?
RESPUESTA
a.- Nifurtimox.
d.- Pentamidina.
c.- Suramina.
d.- Prazicuantel.

PREGUNTA
Cual es el pronstico del caso?
RESPUESTA
a.- Malo para la funcin y malo para la vida
b.- Malo para la funcin y bueno para la vida.
c.- Bueno para la funcin y bueno para la vida.
d.- Bueno para la funcin y malo para la vida.

LESHMANIASIS
CIENCIAS BASICAS: Enfermedad tropical, casada por el protozoo leishmania, intracelular obligado del humano, produce lesiones a nivel cutneo,
mucocutaneo y visceral. Los insectos transmisores son dpteros del genero Lutzomyia (phleboyomus) el cual adquiere la infeccin del humano y
reservorios como roedores, canidos y primates. Tambin se puede adquirir por contacto con material de una lesin, trasplante de rganos, transfusin y
a travs de placenta. Enfermedad emergente oportunista, asociada a SIDA. SALUD PBLICA: Se estima que 12 millones de personas se encuentran

CURSO ENARM CMN SIGLO XXI TEL: 36246001

Pharmed Solutions Institute

PGINA 87

MANUAL DE TRABAJO DEL CURSO ENARM CMN SIGLO XXI


infectadas, con unos 1-2 millones de casos nuevos al ao. La falta de sistemas de vigilancia y el subdiagnostico, hacen difcil estimar la real incidencia y el
ndice de letalidad. Ms de 90% de casos de leishmaniasis visceral se presentan en Bangladesh, Sudn, Sudn del Sur, Etiopia y Brasil. En Mxico en
Chiapas, Oaxaca, Tabasco, Veracruz, Puebla y Guerrero. PATOGENIA: Periodo de incubacin de semanas a meses, L. donovani. El promastigote
metacclico extracelular (forma infectante), es introducido en la piel del hospedero a travs del piquete de la mosca hembra. Los parsitos son
fagocitados en la piel por macrfagos y clulas de Langerhans y activan el complemento. Aunque muchos promastigotes son destruidos por los
leucocitos PMN, unos pocos se transforman en amatigotes que es intracelular (forma replicativa), se multiplica por divisin binaria, en los fagolisosomas
dentro de fagocitos mononucleares de los hospederos. Las clulas infectadas se rompen y los amastigotes se diseminan a los tejidos, induciendo a
produccin de citocinas proinflamatorias. La leishmaniasis cutnea difusa se presenta como consecuencia de factores inmunes del hospedero, asociado a
ciertas especies del parasito. La forma visceral (kala-azar, coloracin gris) es la ms severa, causada por L. donovani. DIAGNOSTICO: Clnica; Se
consideran 2 cuadros clnicos cutneos: la leishmaniosis cutnea localizada (LCL), generalmente circunscrita al sitio de inoculacin, gracias a la respuesta
inmune celular protectora, en Mxico conocida como ulcera de los chicleros (redondeada de borde elevado y bien definido, indurado, cubierta por una
costra amarillenta, cuando esta se desprende, fondo de tejido de granulacin limpio), con frecuencia afectando el pabelln auricular y la leishmaniasis
cutnea diseminada (LCD), caracterizada por pobre respuesta inmune y diseminacin no controlada en piel, caracterizada por lesiones nodulares con
gran numero de parsitos, diseminadas en toda la piel excepto cuero cabelludo, regin inguinal y axilar, genitales, plantas y palmas. Fiebre, palidez,
anorexia, prdida de peso, tos, vomito, diarrea hepatoesplenomegalia, linfadenopatas, sangrado gingival, epistaxis, equimosis, anemia; posteriormente
taquicardia, ictericia, distencin abdominal, ascitis, edema, sangrados importantes, hiperpigmentacion, lesiones verrucosas no ulceradas y alopecia.
Laboratorio: trombocitopenia, anemia normocitica normocrmica, leucopenia. Hipoalbuminemia, hipergammaglobulinemia. Tincin de Giemsa o
Romanosvsky. Biopsia de bazo e hgado (amastigotes). Raspado, biopsia, improntas y extendido de lesiones y ganglios linfticos. La intradermorreaccin
de Montenegro, prueba de hipersensibilidad celular a antgenos de Leishmania, es un mtodo indirecto. TRATAMIENTO: En casos graves la mortalidad
sin tratamiento es de 100%. La primera lnea esta constituida por los antimoniales pentavalentes: antimoniato de meglumina 20mg/kg/dia (variedad
visceral, cutnea), mitelfosina 50-100mgs por 28 dias y estibogluconato de sodio (pentostam). Otros frmacos sistmicos utilizados son: anfotericina B
(efecto adverso dao a rin) 1mg/kg/dia cada 3 dias, pentamidina, paromomicina, sitamaquina.
CASO CLINICO
Mujer de 70 aos que consulta por la aparicin de una ppula en la
frente, algo pruriginosa, que sigue creciendo y a la que se van
aadiendo lesiones similares en su periferia, pese al tratamiento
emprico con corticoides, sin sospecha diagnstica. La lesin se
considero como placa eritematopapulosa, la biopsia de la lesin
demuestra una dermatitis granulomatosa no necrotizante. La reaccin
en cadena de la polimerasa (PCR) diagnostica una leishmaniasis
cutnea (LC).

PREGUNTA
Cual es el tratamiento mas adecuado para el caso?
RESPUESTA
a.- Antimoniato de meglumina.
b.- Anfotericina B.
c.- Paromomicina.
d.- Sitamaquina.

ESQUISTOSOMIASIS
CIENCIAS BASICAS: Enfermedad parasitaria crnica causada por duelas sanguneas (trematodos) del genero Schistosoma. Corren el riesgo de contraer la
infeccin las personas que realizan actividades agrcolas, domesticas o recreativas que las exponen a aguas infestadas. La falta de higiene y las
actividades ldicas hacen que los nios sean especialmente vulnerables a la infeccin. SALUD PBLICA: Es prevalente en las regiones tropicales y
subtropicales, en especial en comunidades sin acceso a agua potable. El nmero de personas tratadas aumento de 12,4 millones en 2006 a 33,5 millones
en 2010. Al menos 230 millones de personas necesitan tratamiento cada ao. PATOGENIA: Las personas se infectan cuando las formas larvarias del
parasito, liberadas por caracoles de agua dulce (son el reservorio, donde maduran los huevos), penetran en la piel durante el contacto con aguas
infestadas. En el interior del organismo, las larvas se convierten en esquistosomas adultos, que viven en los vasos sanguneos, emigran a las venas
abdominales (vena porta- mesentrica superior e inferior) donde se instalaran definitivamente y se reproducirn es aqui donde las hembras ponen sus
huevos. Algunos de esos huevos salen del organismo con las heces o la orina y continan el ciclo vital del parasito. Otros quedan atrapados en los tejidos
corporales, donde causan una reaccin inmunitaria (granulomatosa y fibrotica) y un dao progresivo en sitios de acumulacin de huevos, en pared
intestinal e higado. Hay dos formas principales: Esquistosomiasis intestinal (Schistosoma mansoni, japonicum, mekongi) y esquistosomiasis urogenital
(Schistosoma haematobium). DIAGNOSTICO: Clnico; Los sntomas son causados por la reaccin del organismo al huevo y no por el gusano en si mismo.
En la forma intestinal, dolor abdominal, diarrea y sangre en las heces, inflamacin intestinal, ulceraciones, poliposis, microabscesos, en casos avanzados,
hepatoesplenomegalia (colgeno y tejido fibroso con la consiguiente hipertensin portal), ascitis e hipertensin portal. El signo clsico de la forma
urogenital es la hematuria, puede haber disuria, poliuria, en casos avanzados son frecuentes la fibrosis de la vejiga y los urteres, as como las lesiones
renales. El cncer de vejiga es otra posible complicacin tarda. En las mujeres puede haber lesiones genitales, hemorragias vaginales, dispareuna y
ndulos vulvares. En el hombre trastornos de las vesculas seminales y prstata, en ambos puede llevar a la infertilidad. En nios puede causar anemia,
retraso de crecimiento y problemas de aprendizaje. Laboratorio: Deteccin de huevos del parasito en muestras de heces (frotis en glicerina con azul de
metileno) u orina, en la forma urogenital puede haber microhematuria. TRATAMIENTO: El prazicuantel 40mg/kg dosis nica VO, es el nico tratamiento
disponible contra todas las formas de esquistosomiasis. COMPLICACIONES: Deposito de huevo en SNC, por posible migracin embolica, epilepsia focal o
generalizada tnico clnica, mielitis transversa. Caso clnico tpico, persona que fue a nadar y tiene lesion de entrada en miembros inferiores.
CASO CLINICOS
Nio de ocho aos procedente de guerrero, que acude para estudio de
hematuria macroscpica al final de la miccin y proteinuria de ms de
un ao de evolucin. Refiere baos anteriores frecuentes en ro. No
tiene antecedentes personales ni familiares a destacar. La exploracin
fsica fue normal. Los anlisis de laboratorio muestran un hemograma
con anemia microctica, con Hb 10,9 g/dL, Hto 32,8%, VCM 74,9 fL y
eosinofilia con 767 eosinfilos/L; VSG 40 mm/h. La bioqumica
muestra urea y creatinina e iones normales. Estudio inmunolgico con
niveles de inmunoglobulinas y complemento dentro de la normalidad.
El sistemtico de orina muestra hematuria microscpica y presencia de
cristales de oxalato clcico. La ecografa abdominal fue informada

CURSO ENARM CMN SIGLO XXI TEL: 36246001

como hidronefrosis grado II de pelvis renal izquierda. La investigacin


de Schistosoma sp. en orina fue positiva visualizndose huevos de S.
haematobium. El coprocultivo fue negativo. Las serologas para
Plasmodium falciparum, Entamoeba histolytica, Schistosoma sp., y
virus de las hepatitis A, B y C fueron negativas. El tratamiento se
realiz mediante dos dosis de praziquantel a 20 mg/kg/dosis separadas
por 12 horas. El paciente no acudi a posteriores revisiones.
PREGUNTA
Cual de las siguientes manifestaciones es ms caracteristica en esta
patologia?
RESPUESTA

Pharmed Solutions Institute

PGINA 88

MANUAL DE TRABAJO DEL CURSO ENARM CMN SIGLO XXI


a.- Presencia de huevesillos en heces.
b.- Positivo en frotis perifrico.

c.- Hepato y esplenomegalia.


d.- Hematuria y alteracin renal.

FILARIASIS
CIENCIAS BASICAS: La filariasis linftica, es causada por la infeccin por nematodos de la familia Filarioidea. Hay tres tipos de estos gusanos filiformes:
Wuchereria bancrofti, que es responsable del 90% de los casos; Brugia malayi, que causa la mayora de los casos restantes; B. timori, que tambin causa
la enfermedad, conocida generalmente como elefantiasis, es una enfermedad tropical desatendida. La infeccin humana se produce por la transmisin
de unos parsitos denominados filarias a travs de los mosquitos. SALUD PBLICA: En el mundo hay ms de 1300 millones de personas de 72 pases en
riesgo de sufrir esta enfermedad. En la actualidad hay ms de 120 millones de personas infectadas, y unos 40 millones estn desfigurados e
incapacitados por la enfermedad. Aproximadamente un 65% de los infectados viven en la Regin de Asia Sudoriental, el 30% en la Regin de frica, y los
dems en otras zonas tropicales. La filariasis linftica afecta a ms de 25 millones de hombres con enfermedad genital y a ms de 15 millones de
personas con linfedema. Como la prevalencia de la enfermedad y la intensidad de la infeccin estn relacionadas con la pobreza. PATOGENIA: Cuando
un mosquito (cullex, anopheles, aedes), que contiene larvas en estado infectivo pica a una persona deposita los parsitos en la piel. Desde ah las larvas
invaden el organismo, migrando hacia los vasos linfticos y tejido subcutneo donde se desarrollan y transforman en gusanos adultos en el sistema
linftico. Los gusanos adultos se alojan en el sistema linftico y alteran el sistema inmunitario. Estos gusanos viven entre 6 y 8 aos, y a lo largo de su vida
producen millones de pequeas larvas (microfilarias) que circulan en la sangre. Cuando el gusano muere produce obstruccin linftica. DIAGNOSTICO: La
infeccin suele adquirirse en la infancia, pero las manifestaciones dolorosas y muy desfigurantes de la enfermedad aparecen ms tarde. Mientras que los
episodios agudos de la enfermedad causan discapacidad transitoria, la filariasis linftica produce discapacidad permanente. La filariasis linftica adopta
formas asintomticas, agudas y crnicas. La mayora de las infecciones son asintomticas y no presentan signos externos. A pesar de ello daan el
sistema linftico, los riones y el sistema inmunitario. El linfedema crnico (tumefaccin de los tejidos), o elefantiasis (engrosamiento de la piel), se
acompaa a menudo de episodios agudos de inflamacin local de la piel y de los ganglios y los vasos linfticos. Algunos de esos episodios son causados
por la respuesta inmunitaria del organismo contra el parsito. Sin embargo, la mayora se debe a infecciones bacterianas cutneas porque las defensas
normales se han deteriorado debido al dao linftico. Es frecuente la afectacin de las mamas y de los rganos genitales (hidrocele). En contramos
microfilarias en sangre o en liquido de hidrocele (aplicacin de formaldehido al 2%, ELISA. USG gusanos vivos en ganglios y escroto 80% (Signos de danza
de la filaria). TRATAMIENTO Y PROFILAXIS: El rgimen recomendado es administracin conjunta de dos medicamentos en dosis nicas: albendazol (400
mg) ms ivermectina (150-200 mcg/kg) en zonas donde la oncocercosis (ceguera de los ros) tambin es endmica, o citrato de dietilcarbamazina (6
mg/kg, fiebre, escalosfrios, mialgias y artralgias) en zonas donde la oncocercosis no es endmica. Estos medicamentos eliminan las microfilarias del
torrente sanguneo y matan a la mayora de los gusanos adultos. Para interrumpir totalmente la transmisin de la infeccin es necesario que la
farmacoterapia colectiva dure entre 4 y 6 aos. Se recomienda que los pacientes con discapacidad crnica (elefantiasis, linfedema o hidrocele)
mantengan una higiene rigurosa y tomen las precauciones necesarias para prevenir las infecciones secundarias y la agravacin de la enfermedad. Para
eleiminacion la OMS recomienda farmacoterapia colectiva. La obstruccin se debe quitar por medios mecnicos.
CASO CLINICO
Mujer de 33 aos originaria de chiapas, ama de casa y agricultora,
quien refiere enfermedad insidiosa de 8 aos de evolucin,
caracterizado por la aparicin de lesiones papulares que pustulizan,
localizadas a predominio de tronco y extremidades, acompaadas de
prurito y en ocasiones dolor pungitivo. Antecedentes: padeci
paludismo y dengue, Al examen fsico, adelgazada, con leve palidez de
piel y mucosas, mltiples mculas hipercrmicas en todo el cuerpo. Se
palpan ppulas sin flogosis en axilas, espalda y muslo izquierdo. A la
vitropresin elimina secrecin blanquecina con parsito filiforme de 3
cm. de longitud con cabeza y cola. No adenopatas. No edemas. No

visceromegalia. El resto no contributorio. Su hemograma mostraba


Hemoglobina 8.5gr% Leucocitos 6,100 A0, S58, E24, B0, M6, L12.
PREGUNTA
Respecto al tratamiento. Cual de los siguientes esquemas es el mas
adecuado?
RESPUESTA
a.- Levamisol 150 mg diarios durante 3 meses ms Piperacina 400 mg
diarios 45 dias.
b.- Albendazol 800 mg diarios por 45 das en dos cursos.
c.- Albendazol (400 mg) ms ivermectina (150-200 mcg/kg).
d.- Citrato de dietilcarbamazina (6 mg/kg).

ONCOCERCOSIS
CIENCIAS BSICAS: Conocid como ceguera de los Rios. Parasitosis del hombre causada por onchocerca volvulus (nematodo), que afecta piel, ojos
llegando a producir ceguera, transmitida por insectos hematofagos del genero Simulium. SALUD PUBLICA: En regiones subtropicales del frica occidental
y en reducidas reas del Yemen y Amrica Central. En Mxico se encuentran 3 focos endmicos en Oaxaca y norte de Chiapas. Segunda causa de ceguera
infecciosa en tdo el mundo. PATOGENIA: La Onchocerca volvulus es una filaria, la hembra vivpara, libera embriones mviles y activos (microfilarias,
atraviesan los ndulos y llegan a los tejidos dermicos), en vez de huevos, algunas son ingeridas por una hembra de simulido, la mayora muere y es
eliminada por fagocitosis. Los adultos viven en ndulos fibrosos u oncocercomas subcutneos, aunque algunos se han localizado adheridos al periostio,
oquedades seas, localizados mas frecuente en salientes oseas. En Mxico, los ndulos se ubican fundamentalmente en la cabeza y tronco, mientras que
en frica es ms comn en cintura plvica. La oncocercosis cutnea resulta de mecanismos originados por el desplazamiento de las microfilarias y de las
secuelas resultantes de reacciones inflamatorias acumuladas. DIAGNOSTICO: Clnica; las primeras manifestaciones son cutneas irritacin, prurito,
edema e hipertermia localizadas, la piel se engrosa y hay erupciones papulares y ligeros cambios en la pigmentacin. El prurito se intensifica y el rascado
causa excoriaciones que se infectan secundariamente, zonas de hiperpigmentacion y despigmentacion puede presentarse liquenificacion (epidermis
engrosada, formas nodulares y descamacin). La migracin continua y prolongada de las microfilarias, junto con la respuesta inflamatoria origina la
perdida de elasticidad cutnea y explica la fascies leonina. Al engrosamiento de la piel mas perdida de elasticidad se debe la paquidermitis. En ojo
podemos encontrar las microfilarias en humor vtreo y acuoso, afectan regiones anterior y posterior de ambos ojos, en cornea se presenta queratitis
punteada (corta evolucin) despus queratitis esclerosante que opacas permanentemente la cornea, la visin perifrica se ve reducida ya la funcin
visual limitada. Tambin se produce uveitis, atrofia del nervio ptico y alteraciones en retina, que causan ceguera irreversible. Epidemiologa: individuo
con residencia permanente o antecedentes de visita en rea endmica. Parsitologico; confirma impresin clinica y se realiza mediante la observacin de
microfilarias que emergen de biopsias cutneas o directamente por la observacin de microfilarias en la cmara anterior y posterior del ojo con lampara
de hendidura, deteccin de gusano adulto dentro de un ndulo. Laboratorio: eosinofilia. PCR en piel, ELISA, Western Blot. TRATAMIENTO: Para eliminar
a los adultos de O. volvulus, la extirpacin de los ndulos subcutneos palpables, continua siendo el procedimiento de eleccin. Actualmente se utiliza la
ivermectina 150mcg/kg en una sola dosis cada 3 meses, eficaz contra las microfilarias, en piel y ojo y el efecto perdura 8 meses. En algunos pases se ha
utilizado un esquema mixto, con doxiciclina (esterilidad de la hembra). PREVENCION: Ivermectina cada 6-12 meses para prevenir contagios.

CURSO ENARM CMN SIGLO XXI TEL: 36246001

Pharmed Solutions Institute

PGINA 89

MANUAL DE TRABAJO DEL CURSO ENARM CMN SIGLO XXI


CASO CLINICO
Se trata de una paciente de 24 aos de edad, que acudi por presentar
dolor y fotofobia en OD. En la exploracin, se apreci la existencia de
una uvetis anterior, por lo que se comenz un tratamiento con
midriticos y antiinflamatorios. A los pocos das acude a urgencias de
Oftalmologa, donde se ve una disminucin del cuadro uvetico
anterior, pero, se descubre la existencia de una vitritis con un foco de
coriorretinitis parapapilar inferior, que afecta a la papila, con exudados
algodonosos y hemorragias, as como perivasculitis zonal, observada
tambin en la AFG; el fondo de ojo del Ol era normal. Ante este
cuadro, la paciente ingresa con la sospecha de coriorretinitis por
citomegalovirus por lo que se inicia tratamiento sistmico con
ganciclovir IV. Sin antecedentes patolgicos de inters, haba recibido
tratamiento para una infeccin urinaria haca un mes y tena un
antecedente de anexitis, ya curada. En el estudio analtico inicial
apareca una eosinofilia mayor de 1.000/mm3.
PREGUNTA
Cual es la conducta a seguir mas adecuada?
RESPUESTA

a.- Ascultacion con lmpara de hendidura.


b.- Frotis sanguneo.
c.- Verificar los valores de eosinofilia.
d.- Iniciar ivermectina.
CASO CLINICO
Paciente masculino de 62 aos, natural y procedente de oaxaca, de
oficio agricultor, quien consult por presentar tumoraciones drmicas
en miembros superiores e inferiores, prurito generalizado, cefalea,
artralgias y disminucin de la agudeza visual, con presencia de ndulo
subcutneo, Bh eosinofilia importante.
PREGUNTA
Cual de los siguientes mtodos diagnosticos es mas adecuado para
establecer el diagnostico?
RESPUESTA
a.- Verificar por lmpara de hendidura.
b.- Biopsia del nodulo.
c.- Frotis de sangre.
d.- Eosinofilia persistente.

RICKETTSIASIS
CIENCIAS BSICAS: Son bacterias estrictamente intracelulares gramm negativas, que se transmiten a travs de vectores artrpodos (garrapatas, pulgas y
piojos) a diferentes mamferos que actan como reservorios. SALUD PBLICA: Sus caractersticas epidemiolgicas, su alta infectividad y elevada
mortalidad de algunas, pueden hacer que se conviertan en armas biolgicas ideales. CLASIFICACIN: Existen 2 tipos de vasculitis que en funcin de ellas
podemos diferenciar en 2 grupos: grupo de las fiebres manchadas; afectan endotelio, capa intima y media, entre ellas Fiebre manchada de Montaas
Rocosas (en Amrica, transmitida por garrapatas causada por R. rickettsia), Fiebre Botnosa mediterrnea (causada por R. coronii, transmitida por
garrapata de perro). El grupo de las fiebres tficas; afectan endotelio y capa intima entre ellas Tifus o exantema epidmico (causado por R. prowazekii,
transmitido por piojos)y tifus murino (causado por R. typhi, transmitido por pulgas). PATOGENIA: La rickettsia penetra en el organismo, tras la picadura,
o atraves de una solucin de continuidad contaminada con heces de los artrpodos (ingresan a travs del piquete a la hora des rascado), o bien a travs
de la conjuntiva o vas respiratorias, rpidamente alcanza torrente sanguneo, all se une a la clula endotelial y se producen cambios en la conformacin
de la membrana que facilitan la fagocitosis de la bacteria, esta rompe la membrana fagosomica y escapa libre a citosol aqu se produce la multiplicacin
(intracelular)a travs de un mecanismo de expulsin activo, se produce inoculacin en clulas contiguas al rea infectada, tambin hay diseminacin
linftica. Y este ciclo, ocasiona la aparicin de mltiples focos de vasculitis y un estado procoagulante endovascular erupcin, hemorragias, trombosis y a
veces gangrena). La destruccin celular y liberacin de rickettsias explican la fiebre, las lesiones capilares son la base del colapso vascular y alteracin del
estado de conciencia. El aumento de permeabilidad capilar, produce extravasacion de lquidos, y protenas, dando edema, y agravamiento del estado de
choque. DIAGNOSTICO: Clnico; fase de prdromo; malestar general, cefalea y febrcula. Fase de estado; fiebre elevada sostenida de 2-3 semanas,
cefalea intensa, vrtigos ataque al estado general con gran postracin, piel seca, estado de conciencia alterado con alternancia de estupor y delirio, tos
seca, nausea, vomito. Fase de exantema; mculas eritematosas en axilas y flancos que se extienden al tronco y finalmente a extremidades. En la
eruptiva, es comn que exista oligura, taquicardia desproporcionada a la temperatura, polipnea. Complicaciones; gangrena por obstruccin. Serologico:
IgM e IgG aparecen despus de 1 semana a 10 das, y duran de 3-4 meses. Existencia de antgenos con prueba de Weil-Flix. En Mxico prueba de RuizCastaeda. TRATAMIENTO: primera eleccin tetera inclinas, clorhidrato de tetraciclina 40mg/kg dividida c/6 o 8 o 12 hrs VO o parenteral, doxiciclina 24mg/kg/da (dosis mxima 200mg/ dia en dos dosis), mantenerse 3 das despus de remisin trmica. Alternativa cloranfenicol, el tratamiento debe
continuar hasta 24 hrs despus de haber desaparecido la fiebre. El estado de choque, hiponatrema, edema y coagulopatia de consumo; infusiones de
albmina, heparina (la sangre total y plasma contraindicados por CID). Los medicamentos solo detienen el desarrollo, de las rickettsias sin destruirlas, no
son raras las recadas.
CASO CLINICO
Paciente masculino de 8 aos de edad, que inici 5 das previos con
presencia de fiebre intensa de hasta 40C de difcil control, dolor
farngeo y tos productiva, astenia y adinamia, llevado con mdico 4
das previos quien recomienda trimetoprim/sulfametoxazol a dosis de
7mg/kg/da (8 dosis), ambroxol y clorfenamina con paracetamol para
tratamiento decuadro respiratorio. El paciente no muestra mejora
clnicamente, e incluso aparecen mltiples zonas de erupcin en lapiel
con eritema y petequias, pruriginosas, principalmente encara y que se
han ido extendiendo a tronco y extremidades.Con patrn tipo
centrpeto. Al no encontrar mejora es trado a revisin mdica a la
consulta externa de con diagnstico probable de enfermedad de
Kawasaki. El cuadro clnico inici posterior a una visita de establo y en
contacto con ganado bovino y ovino, se desconoce s tenan vectores
ya que el paciente luce descuidado pues no vive con los padres. Tiene
hbitos de higiene regulares. Bao cada tercer da pero en vacaciones
hasta cada semana. No muestra cartilla, desconoce cules vacunas le
faltan. Tuvo varicela a los 4 aos. El paciente se observ hipotrfico,
consciente y cooperador, con buena coloracin de piel y lapresencia de
una erupcin maculo papular eritematosa, descamativa, petequial o

CURSO ENARM CMN SIGLO XXI TEL: 36246001

micromacular en parpados, as como otras regiones de cara, pecho,


dorso, abdomen y extremidades, respetando palmas y plantas de pies.
Kerning y Brudsinskinegativos y sin otros datos de meningismo; cuello
simtricocon un ganglio palpable en regin media, mvil no doloroso
menor a 1cm. Hemorragia en flama conjuntival bilateral, sin secrecin
conjuntival ni prurito, mucosa nasal conrinorrea hialina, mucosa oral
bien hidratada y faringe hipermica, sin exudados. Trax sin dificultad
respiratoria, con estertores gruesos bilaterales transmitidos, sin
problemas a la ventilacin. Precordio rtmico sin soplos y pulsos
perifricos normales. Abdomen blando, levemente distendido, con
peristalsis normal, sin hepatomegalia ni esplenomegalia.
PREGUNTA
Cual es el agente etiolgico mas probable?
RESPUESTA
a.- Rickettsia prowazekii
b.- Rickettsia typhi
c.- Rickettsia rickettsii
d.- Rickettsia coronii

Pharmed Solutions Institute

PGINA 90

MANUAL DE TRABAJO DEL CURSO ENARM CMN SIGLO XXI


CASO CLINICO
Se trata de masculino de 19 aos de edad ingresado al servicio de
urgencias presentando fiebre, cefalea intensa, dolos abdominal difuso
pero ligero, pero con mialgias intensas, al examen fsico se observa con
38,5 c, FC 112, TA 120/70 su estado general es adecuado en
crecimiento y desarrollo con diaforesis, se mantiene alerta y orientado,
se realizan estudios de laboratorio y gabinete de rutina donde se
observa nicamente plaquetas con 84,000, se realiza puncion lumbar
donde se observa auscencia de eritrocitos, protenas y glucosa
normales, solo se observa 5 monocitos. Como antecedente de

importancia refiere regresar de un campamento y realizar escalado de


montaas, actualmente continua en el servicio de medicina interna.
PREGUNTA
Considerando los hallazgos asi como los antecedentes usted integra un
diagnostico, y debe iniciar tratamiento, cual es el ms adecuado:
RESPUESTA
a.- Rifampicina.
b.- Doxiciclina.
c.- Vancomicina.
d.- Ceftriaxona.

ENFERMEDAD DEL LEGIONARIO (LEGIONELLA PNEUMOPHILA):


CIENCIAS BSICAS: Llamada as porque afecto a los asistentes de una reunin de la Legin Americana, celebrada en un hotel de Filadelfia (1976).
Legionella bacteria gran negativa, de forma bacilar crece en aguas (entre 20- 50) naturales, lagos, ros, arroyos, lodos. Supervivencia en aire es corta
SALUD PBLICA: Legionella es causa frecuente de neumona comunitaria y nosocomial, con una mortalidad que oscila entre el 3-50%. CLASIFICACIN:
Fiebre de pontiac; El 95%, la forma ms leve y la ms frecuente se parece a una "gripe", no produce neumona es autolimitada, con buena evolucin.
Enfermedad del Legionario; De 1-5%, es la forma ms grave, provoca neumona y afecta al estado general. La mayora de casos se dan en forma
espordica. PATOGENIA: Factores de riesgo: uso previo de corticoides, e inmunosupresores, neoplasias, trasplante, dilisis, sexo masculino, mayores de
50 aos, tabaquismo, EPOC, va de contagio parece ser a travs de aerosoles de gotas de agua contaminada que son aspirados o inhalados. Estas gotas
preceden de reservorios de agua infectados por legionella (aparatos de refrigeracin, condensados de agua, duchas, sistemas de canalizacin y
distribucin de agua que sirve de reservorio en hospitales, edificios). Sospechar en un contexto epidemiolgico adecuados datos clnicos, un habito
tabquico importante, la ausencia de supera a antibiticos betalactamicos y la observacin de escasos microorganismos en un esputo llenos de
neutrofilos son caractersticos. Solicitar antgeno de legionella en orina. DIAGNOSTICO: Clnico: Inicio brusco con fiebre alta de predominio matutino, con
afeccin general, tos no productiva, esputos, escalofros, cefalea, presencia de manifestaciones extrapulmonares (diarreas o confusin mental),
hiponatrema y elevacin de la creatincinasa. La neumona por legionella se incluye dentro de las neumonas de presentacin graves. TRATAMIENTO: De
eleccin clsico es la eritromicina (efectos secundarios; flebitis, alteraciones gastrointestinales, ototoxicidad), por ello se puede utilizar claritromicina
(500mg c/12hrs 10-14 das). Tambin til azitromicina (500mg/da). La experiencia con fluoroquinolonas como levofloxacino mas en caos graves o
neumona intrahospitalaria (500mg/da en los primeros das c/12 hrs e intravenoso) y ofloxacino son positivas.
CASO CLINICO
Varn de 46 aos, fumador de 40 paquetes al ao y ciruga de
colesteatoma con parlisis facial perifrica derecha residual. Una
semana antes de la clnica refiere estancia en un hotel del Levante.
Acude al mdico de guardia del centro de salud por fiebre de ms de
39 C, sin otra sintomatologa acompaante y con exploracin fsica
normal. Se le pauta tratamiento sintomtico con paracetamol. Al da
siguiente acude a su mdico de cabecera por persistencia de la fiebre,
no existiendo variaciones en la exploracin. Se decide continuar con el
tratamiento prescrito el da anterior y observacin domiciliaria. A los
cinco das tras persistir la fiebre, y aparecer disnea y tos con
expectoracin hemoptoica, el paciente es remitido al Servicio de
Urgencias hospitalario, donde se le practican diversas pruebas
complementarias: hemograma (16.430 leucocitos con 93%
neutrfilos), bioqumica (Na 134, PCR 340), GAB (pH 7.46, PO2 60,
PCO2 30), Rx de trax (infiltrado alveolar bilateral con derrame pleural
derecho).
PREGUNTA
Considerando el agente infeccioso presuntivo, cual es la conducta
diagnostica mas apropiada?
RESPUESTA
a.- Realizar Hemocultivo.

b.- Baciloscopia de Esputo.


c.- Antigeno en Orina.
d.- Inmunoglobulinas.
LEGIONELA
Se trata de paciente masculino de 57 aos de edad el cual acude al
servicio de urgencias debido a tos intensa no productiva y fiebre, el
paciente cuenta con los siguientes antecedentes, originario de
guanajuato, empleado de la construccin, alcoholismo y tabaquismo
positivo desde hace 40 aos, actualmente bajo tratamiento por EPOC y
cirrosis heptica, se realizan estudio de laboratorio y gabinete de
rutina donde se observa un reporte con disminucin de la respuesta
inmunolgica esperada, la radiografia de torax presenta un patrn
caracterstico, sin embargo el cuadro respiratorio bajo se ha
intensificado por lo que se traslada a terapia intensiva.
PREGUNTA
Cual es tratamiento antibitico ms apropiado para este caso.
RESPUESTA
a.- Azitromicina.
b.- Levofloxacina.
c.- Trimetoprim/sulfametoxazol.
d.- Penicilina.

PSEUDOMONA AERUGINOSA (PAE):


CIENCIAS BSICAS: Es una bacteria bacilo Gramm negativo, aerobio estricto, dispersa en el ambiente, emergente relevancia como patgeno oportunista
causante de infecciones en pacientes hospitalizados (equipos de ventilacin mecnica, soluciones de limpieza, instrumental, medicamentos), crticos y/o
inmunodeprimidos (quemados, ventilados, fibrosis qustica, EPOC, pos quirrgicos).m La PAE causa infecciones en la mayora de las partes del cuerpo, se
adapta rpidamente al tracto respiratorio y es el lugar ms frecuente de infeccin por PAE (principal causante de la neumona asociada a la ventilacin
mecnica NAR). SALUD PUBLICA: Diseminada prcticamente en toda la geografa mundial, es cosmopolita. Tasa de mortalidad atribuible a pseudomoma
es de 34%. PATOGENIA: Factores de riesgo; inmunodeprimidos. La fuente de origen puede ser hematogena o como puerta de entrada en un sitio de
puncin o a travs de vas centrales o catteres de dilisis que pueda tener el paciente al parecer la lesin inicial provocada por la P. aeruginosa al
epitelio respiratorio y otras mucosas esta mediada por pili o fimbrias y por un exapolisacorido mucoide conocido como alginato. Existen receptores de
estas adhesinas en las clulas epiteliales. El microorganismo produce diversas enzimas extra celulares como la proteasa alcalina, elastasa, fosfolipasa,
citotoxina y exoenzimas A y S, la alteracin de los tejidos del husped por estos productos bacterianos, crea las condiciones necesarias para la
proliferacin e invasin bacteriana y la consiguiente destruccin del tejido. Infecciones seas (fracturas, cirugas traumatolgicas) y articulares
(osteomielitis vertebral), por bacteriemia, inoculacin directa dentro del hueso y por diseminacin contigua desde otro sitio de infeccin. Infeccin en

CURSO ENARM CMN SIGLO XXI TEL: 36246001

Pharmed Solutions Institute

PGINA 91

MANUAL DE TRABAJO DEL CURSO ENARM CMN SIGLO XXI


SNC (meningitis, infecciones subdurales o extradurales) secundarias a ciruga y a trauma de crneo o bacteriemias. Infeccin urinaria secundaria a cuerpo
extrao (calculo, atenta, sonda) o a estenosis. Una situacin importante es la infeccin en tejidos quemados. DIAGNOSTICO: El cuadro clnico es el de un
paciente sptico, el nico punto diferente con otros tipos de sepsis por gramm negativos es la presencia de lesiones cutneas llamadas ectima
gangrenoso, que se ve mas frecuente en pacientes neutropenicos. Lesin maculopapular, pequea, roja, dolorosa, mal circunscrita, que comienza de
color rosa, se oscurece hasta volverse prpura y finalmente negra y necrtica. Debe sospecharse neumona nosocomial en un paciente con uno infiltrado
radiolgico nuevo o progresivo asociado a datos clnicos sugestivos de infeccin; fiebre, esputo purulento o leucocitosis, tras diagnostico sindromatico,
intentantar confirmacin microbiologa. . La caracterstica ms importante en laboratorio es la produccin de pigmento llamado piocianina, que le da un
color azulado. Obtencin de material respiratorio (BAL, aspirado traqueal, minibar) TRATAMIENTO: Las pseudomonas son productoras de enzimas MBL
(metalo-b-lactamasas) capaces de resistir prcticamente a todo tipo de antibitico, por ello su actividad es devastadora. Basndose en esquema para
neumonas nosocomiales A) inicio reciente y sin factores de riesgo, cualquier gravedad: ceftriaxona, fluroquinolonas, ampicilina- sulbactam o ertapenem
B) inicio tardo o factores de riesgo, cualquier gravedad: beta-lactamico antipseudomonico (carbenicilina, ticarcilina, mezlocilina, piperacilina) mas
fluoroquinilona o aminoglucosidos mas linezonid o vancomicina. Antibiticos mas utilizados cfepime (1-2 g cada 8 hrs IV), ceftazidime ( 2g c/8hrs IV),
imipenem (500mgs c/6 o 1g c/8hrs IV), meropenem (1g c/8hrs), gentamicina (7mg/kg/da IV), amikacina (20 mg/kg/da IV), levofloxacino (750mg/da IV),
colistin 100mg c/8hrs, piperacilina/tazobactam (4.5g c/6hrs IV). No hay un esquema especifico lo que realmente cambia la mortalidad es el tratamiento
emprico inicial adecuado.
CASO CLINICO
Un varn de 27 aos, diabtico, fue internado por fiebre, compromiso
de conciencia y rigidez de nuca aparecidos tras tres das de cursar con
una infeccin respiratoria aguda. Se le diagnostic una
meningoencefalitis asptica (LCR de aspecto claro, con presin de 12
cm H2O; leucocitos: 200/mm3; tincin de Gram directa y cultivo
bacteriano: negativos) y recibi ceftriaxona ms vancomicina y
corticoterapia (dexametasona, 10 mg iv cada 6-horas durante 4 das).
Inicialmente ingres en una UCI y estuvo en ventilacin mecnica,
siendo desconectado a los 4 das. Desarroll fiebre y tos productiva
tras una semana de evolucin. Una Rx de trax detect un infiltrado
pulmonar bilateral. Tres das ms tarde, una segunda Rx revel la
presencia de mltiples abscesos pulmonares con niveles hidro-areos.
PREGUNTA

Cual es la conducta diagnostica mas adecuada?


RESPUESTA
a.- TAC de torax.
b.- Toracostomia.
c.- Cultivo de esputo.
d.- Hemocultivo.
PREGUNTA
Se obtiene cultivo positivo para P. aeruginosa, cual es la conducta
teraputica asociada mas apropia?
RESPUESTA
a.- Linezolid.
b.- Meropenem.
c.- Amikacina.
d.- Ciprofloxacina.

LISTERIOSIS (LISTERIA MONOCYTOGENES):


CIENCIAS BSICAS: Bacteria Gramm positiva anaerobia, facultativa intracelular, puede causar infecciones invasoras muy graves. Principales reservorios
son el suelo, forrajes el agua, los silos y tracto gastrointestinal en aves, peces y mamferos incluyendo el hombre. Suele considerarsele un patgeno
oportunista. SALUD PUBLICA: Distribucin universal, relativamente resistente a la refrigeracin, la sequedad y el calor extremo. Afecta a
inmunodeprimidos, pacientes con enfermedades crnicas debilitantes mayores de 60 aos, cirrticos, embarazadas en (en la Ciudad de Mxico 1
por 4000) y neonatos (1 por 1500). Contina causando una elevada morbilidad y una preocupante mortalidad. PATOGENIA: la infeccin se adquiere
mediante el consumo de Alimentos contaminados, carnes, pescado y vegetales, crudos y lcteos no pasteurizados y productos refrigerados, los recin
nacidos la adquieren a travs de la placenta o del canal de parto infectado. Puede sobrevivir en el entorno gstrico, colonizar el intestino y cruzar la
barrera hematoencefalica y maternofetal, con sus protenas de superficie internalinas, se favorece su invasin celular por un mecanismo tipo zipper, en
que la bacteria se hunde progresivamente en la superficie celular, despus estas se lizan y escapan, la protena polmerizadora de activa, le permite
azucares en el citosol celular durante su replicacin intracelular. DIAGNOSTICO: Clnica; La ms habitual es la listeriosis gastrointestinal; importante
puede causar brotes de toxiinfeccion alimentaria, puede ser asintomtica o sufren deposiciones acuosas, nauseas, vmitos, cefalea, artromialgias y
fiebre, sntomas que se limitan a 2 das, salvo que estn inmunocomprometidos. Listeriosis invasora; enfermedad gestacional y neonatal,
frecuentemente fiebre sin foco aparente, la gravedad de Listeriosis materna radica en el tercer trimestre, ya que puede haber aborto, muerte fetal
intrautero, prematuro dad, sepsis, muerte neonatal. Es fundamental hacer hemocultivos a toda mujer embarazada con fiebre, aunque semiologa
parezca banal. Los neonatos pueden sufrir sepsis, microabscesos y granulomas diseminados (granulomatosis infantoseptica), infeccin respiratoria o
meningitis, que conlleva una mortalidad y morbilidad importantes como la hidrocefalia persistente en el caso de meningitis. Infeccin del SNC; pueden
tener meningitis, meningoencefalitis o cerebritis. Endocarditis es una rara y grave complicacin de la bacteriemia (forma clnica mas frecuente en
inmunodeprimidos, sin foco identificable, ocurre con fiebre, deteriroro rapido y a menudo fulminante) por listeria. El diagnostico definitivo es
aislamiento en sangre, LCR, liquido articular, placenta. TRATAMIENTO: La mayora de los antibiticos incluyendo penicilinas son bacteriostticos contra
la listeria, los aminoglucosidos, glucopeptidos y cotrimoxaxol son bactericidas. Neonatos y formas graves; cotrimoxaxol 20mg/kg/da, en 4 dosis o
gentamicina 5-7mg/kg/da en dosis nica por 3 das. Infeccin del SNC ampicilina 400mg/kg/ da en 6 dosis, asociada a gentamicina (si no hay dao renal)
o ampicilina y cotrimoxaxol (mas eficacia, menos nefrotoxicidad). En embarazadas ampicilina mejor opcin.
CASO CLINICO
Femenino de 33 aos con fiebre de una semana de evoluacion, de tipo
intermitente, mialgias, artralgia y escalofros, acude a consulta a
primer nivel donde no se integra proceso infeccioso, la paciente se
encuentra embarazada con 21 SDG por FUM, a la exploracin se
observa con mucosas orales deshidratadas, durante su ingreso
presenta nausea, cefalea y diarrea, al tercer dia se observa con
somnolencia con inversin del ciclo vigilia sueo, posteriormente
refiere sensacin de salida de liquido transvaginal, al tacto no se
observa elementos ovulo placentarios.

CURSO ENARM CMN SIGLO XXI TEL: 36246001

PREGUNTA
Cual es la conducta teraputica mas adecuada sobre el agente casusal
del caso?
RESPUESTA
a.- Ampicilina.
b.- Gentamicina.
c.- Cotrimoxaxol.
d.- Ceftriaxona.

Pharmed Solutions Institute

PGINA 92

MANUAL DE TRABAJO DEL CURSO ENARM CMN SIGLO XXI


CRIPTOCOCOSIS (CRYPTOCOCCUS NEOFORMANS):
CIENCIAS BSICAS: Micosis sistmica aguda, subaguda o crnica. Causada por el hongo levaduriforme Cryptococcus neoformans. La forma pulmonar es
generalmente transitoria y leve, la inclusin del sistema nervioso central con meningitis es la forma ms familiar de la micosis. SALUD PBLICA:
Espordico. La meningoencefalitis tiene mortalidad de 100%, en no tratados y 40% en los tratados. PATOGENIA: La infeccin se adquiere al inhalar el
hongo (presente en heces de palomas y sus nidos). El SIDA es el principal factor predisponente, tambin uso de esteroides, neoplasias; la DM, cirrosis y
leucopenia aumentan el riesgo de padecer la enfermedad. Se transmite en la forma de basidiosporas, presentes en el ambiente por tiempos cortos y son
infecciosas para el humano y otros animales, despus de su diseminacin pueden sintetizar material plisacarido y convertirse en levaduras
encapsuladas. Al entrar por vas respiratorias altas, el hongo coloniza el rbol bronquial, por ello inicia como una enfermedad pulmonar, que se
disemina a piel, huesos, vsceras, abdominales y SNC. DIAGNOSTICO: Clnica; asintomtica, a veces dolor torcico, tos, fiebre, infiltrados intersticiales o
lobeares y derrame pleural; las lesiones pulmonares se caracterizan por inflamacin granulatiza, la diseminacin hematogena permite que infecte el SNC
(sustancia gris cortical y ganglios basales). La meningoencefalitis, los datos clnicos estn relacionados con el desarrollo de lesiones granulomatosas en las
meninges, se presenta con nausea, cefalea, irritabilidad, confusin, demencia, visin borrosa, fiebre, rigidez de cuello y parlisis de nervios craneales. En
el 10% aparecen lesiones cutneas (papilas, lceras, abscesos). Laboratorio: cultivo en Sabouraud, forma colonias de color crema plido. Antgeno
manan se detecta en 33% de los casos. Biopsia para diagnostico de criptococosis pulmonar. LCR vemos levadura encapsulada, hipoglucorraquia,
pleocitocis linfocitica y aumento de protenas. TRATAMIENTO: Anfotericina B a 0.7mg/kg/da y 5-fluorocitocina a 100mg/kg/da durante 6-10 semanas.
CASO CLINICO
Paciente femenina de 35 aos con antecedentes de presentar desde
haca dos meses cefalea, irritabilidad, prdida de la memoria, cambios
de personalidad, alteraciones del sensorio, dificultad para caminar por
disminucin de la fuerza muscular en el miembro inferior izquierdo y
ataxia de la marcha. EF: presenta alteraciones del sensorio con
somnolencia, hemiparesia izquierda ligera a predominio crural, ataxia
de la marcha y rigidez nucal y lesiones vesiculosas herpticas en la
regin peribucal. El fondo de ojo mostraba borramiento de los bordes
temporales de ambas papilas. Estando ingresada se queja de vrtigo y
presenta varios vmitos. Se le realizan los siguientes exmenes de
laboratorio: Hemoglobina: 122 g/l, Hematocrito: leucocitos: 6.1x109
g/l, neutrfilos: 59, linfocitos: 40, monocitos: 1; eritrosedimentacin:
120 min; glicemia: 4.8 mmol/l; creatinina: 212 mmol/l, TGP: 11 unid,
urocultivo: normal, sin crecimiento; tomografa axial computarizada
(TAC) de crneo simple: sin alteraciones evidentes; Lquido
cefalorraqudeo (LCR) citoqumico claro y transparente, clulas: 11,
pandy (-), protenas: 25, glucosa 1.9 mmol/l.
PREGUNTA
Cual es la conducta diagnostica mas adecuada para el presente caso?
RESPUESTA
a.- Serologia e inmunolgica.
b.- Elisa y west-blot.
c.- Resonancia magnetica con contraste.
d.- Cultivo y prueba de tinta china en LCR.
PREGUNTA

El paciente es positivo para VIH, al termino del tratamiento habitual


cual de las siguientes alternativas para continuar la profilaxis
secundaria es mas adecua?
RESPUESTA
a.- Fluconazol 200 mg / dia
b.- Ketoconazol 400 mg / dia.
c.- Itraconazol 100 mg / dia
d.- Clortrimazol 300 mg / dia.
CASO CLINICO MENINGOENCEFALITIS CRIPTOCOCCICA
Se trata de paciente femenino de 38 aos de edad la cual acude al
servicio de urgencias debido a que presenta fiebre, cefalea y dolor en
la regin de la nuca, como antecedentes de importancia la paciente
trabaja en un aviario, no es diabtica ni hipertensa, aparentemente
sana antes de este episodio, a la exploracin fsica se observa
conciente, irritable, orientada, facies algicas, se realiza TAC de cabeza
el cual no revela datos de importancia, se realiza puncin lumbar
reportndose presin de apertura de 20 cmH2O, leucocitos 15 clulas
(90 % monocitos), protenas de 0.6 g/L, glucosa de 50mg/dl y tincin
positiva con tinta de la India.
PREGUNTA
Cual es el tratamiento mas adecuado para el caso.
RESPUESTA
a.- Anfotericina.
b.- Fluconazol.
c.- Cexfriaxona.
d.- Vancomicina.

LEPTOSPIROSIS (LEPTOSPIRA INTERROGANS):


CIENCIAS BSICAS: Se considera la zoonosis ms frecuente, la leptospira interrogans, es la nica espiroqueta que infecta al hombre, gramm negativo,
aerobio y mvil. Frecuente en campesinos, granjeros, taladores, cazadores, trabajadores de cultivo de arroz y militares, actualmente tambin en nios
(natacin cabotaje, ciclismo, caza deportiva, animales domsticos infectados). PATOGENIA: Las leptospiras en agua o tierra contaminada por orina de
animales infectados, penetran a la piel a travs de cortaduras o abrasiones, membranas, mucosas y conjuntivas, se disemina a travs de la sangre a todos
los rganos y sistemas incluyendo el SNC. Los fenmenos hemorrgicos son secundarios a vasculitis severa con dao endotelial. Los riones presentan
nefritis intersticiales, necrosis tubular y permeabilidad capilar anormal que acompaa a hipovolemia y puede llegar a IRC. La ictericia es secundaria al
necrosis centrolobulillar y proliferacin de clulas de Kupffer, con disfuncin hepatocelular. Miocarditis y uvetis ocurren por invasin tisular.
DIAGNOSTICO: Clnica; leptospirosis anicterica (85-90%), incubacin de 7-12 das, inicia fiebre, cefalea, mialgias calosfros severos y mal estado general,
linfadenopatias hepatoesplenomegalia. Durante este periodo pueden aislaras las leptospiras en sangre o LCR. Entran en perdidos asintomtico de 1-3
das e inician ahora la llamada fase inmune o desarrollo de anticuerpos IgM especficos, puede reaparecer la fiebre, cefalea y vmitos y se puede
desarrollar meningitis asptica, existe aumento de clulas en el liquido cefalorraquideo, primero predominando los PMN y despus los mononucleraes,
las protenas estn elevadas, glucosa normal. Puede o no haber irritacin meninges, esta fase dura de 3-40 das, y ya no se pueden aislar las leptospiras
de sangre y/LCR, pero si en orina. Leptospirosis ictericia (sx. de Weill); forma grave y de mayor mortalidad, los pacientes pueden desarrollar ictericia pero
no dao renal, lo usual es que haya ictericia y elevacin de urea y creatinina, as como elevacin de transaminasas, proteinuria, hematuria, cilindruria,
puede haber anemia, trombocitopenia, leucocitosis, hemorragia, Asi como neumonas hemorrgica. Laboratorio observacin de leptospiras es el
estndar de oro para el diagnostico, la bsqueda en orina es la ms confiable. Las pruebas en suero para deteccin de anticuerpos de la fase aguda (1-2
semanas) y su elevacin en 4 tantos durante la fase convaleciente (2semanas despus), incluyen la prueba de aglutinacin al microscopio, confirman
diagnostico serolgico. Una sola determinacin de ttulos de 1:800 o mayores sugieren fuertemente la enfermedad. Analizar LCR en pacientes con
meningitis. La radiografa de trax puede mostrar cardiomegalia, edema agudo pulmonar asociado a miocarditis o infiltrados en parches asociados a
hemorragia alveolar por capilaritis. TRATAMIENTO: La penicilina G sdica (20-24 millones U/da c/4-6hrs), doxiciclina (100mg c/12hrs IV) o eritromicina
(500mg c/6hrs IV) Son eficaces para detener la infeccin y el dao orgnico. La doxiciclina se usa para prevencin en dosis semanal de 200mg.

CURSO ENARM CMN SIGLO XXI TEL: 36246001

Pharmed Solutions Institute

PGINA 93

MANUAL DE TRABAJO DEL CURSO ENARM CMN SIGLO XXI


CASO CLINICO LEPTOSPIROSIS
Femenino de 29 aos la cual labora como asistente de veterinario,
refiere que hace 10 dias inicio con fiebre, escalofros, cefalea, nauseas,
dolor muscular, se automedica con frmacos sintomticos, mejorando
su estado general sin embargo regresan los sntomas y agregandoce
ictericia, a la exploracin clnica se presenta 38.9 C temperatura, FC
1010, TA 140/90 mmHg, la saturacin por oximetro 92 %, a la
percusin se presenta dolor heptico, se persibe leve crecimiento, sin
esplenomegalia, los estudios de laboratorio BUN de 64, creatinina 3.6,
bilirrubina de 64, AST 86, ALT 103, fosfatasa alcalina de 390, leucocitos
11 000, 13% de bandas y 80 % polimorfonucleares, hematocrito 33% y

plaquetas de 145, puncion lumbar revela pleocitosis, TAC de torax


muestra inflitrados en flama difusos compatibles con hemorragia
pulmonar.
PREGUNTA
Cual es diagnostico ms probable?
RESPUESTA
a.- Neumonitis intersticial aguda.
b.- Leucemia mieloide aguda.
c.- Fiebre por Streptobacillus moniliformis.
d.- Infeccion por Leptospira interrogans.

ACTINOMICOSIS
CIENCIAS BASICAS: Es causada por bacterias anaerbicas o microaerfilos, gramm positivos principalmente del gnero Actinomyces (por ejemplo, A.
israelii), que est caracterizada por formacin de absceso, fibrosis tisular, las regiones ms afectadas son la cervicofacial, torcica y plvico-abdominal
(generalmente infecciones polimicrobianas). Actinomicosis se asocia con mala higiene dental, absceso dentario, uso de dispositivos anticonceptivos
intrauterinos (DIU), y la inmunosupresin. SALUD PBLICA: Su incidencia est disminuyendo, probablemente como resultado de una mejor higiene
dental y un inicio ms temprano del tratamiento antibitico. PATOGENIA: Los agentes de actinomicosis son miembros de la flora oral normal. La
enfermedad se presenta slo despus de la interrupcin de la barrera de la mucosa. Infeccin local se extiende en forma contigua en una manera lenta y
progresiva, haciendo caso omiso de los planos del tejido. Las paredes fibrticas de la masa a menudo se describen como "madera. DIAGNOSTICO: La
enfermedad oral - cervicofacial: La infeccin comienza como una inflamacin de los tejidos blandos, abscesos, o masa, a menudo en el ngulo de la
mandbula con extensin contigua al cerebro, espina dorsal cervical, o en el trax. El dolor, la fiebre, y leucocitosis son variables. Enfermedad torcica:
Produce dolor en el pecho, fiebre y prdida de peso. Radiografa de trax muestra una lesin de masa o la neumona. Enfermedad cavitaria o adenopata
hiliar pueden ocurrir, y > 50 % de los puntos tienen engrosamiento pleural, derrame o empiema. Las lesiones cruzan fisuras o pleura y pueden implicar el
mediastino, el hueso contiguo, o la pared torcica. Enfermedad abdominal: El diagnstico es difcil y no se puede hacer hasta meses despus del evento
inicial (por ejemplo, diverticulitis, ciruga intestinal). La enfermedad generalmente se presenta como un absceso, masa o lesin fijado al tejido
subyacente y, a menudo se confunde con el cncer. Fstulas en la pared abdominal, en la regin perianal, u otros rganos pueden desarrollar e imitar la
enfermedad inflamatoria intestinal. Participacin del tracto urogenital puede presentarse como pielonefritis o absceso perirrenal. Actinomicosis plvica
se asocia a menudo con DIU. La presentacin es indolente y puede seguir a la extraccin del dispositivo se presenta fiebre, prdida de peso, dolor
abdominal y sangrado vaginal anormal. Endometritis progresa a masas plvicas o abscesos tuboovrico. Actinomicosis pueden afectar el tejido msculoesqueltico, tejido blando, o el SNC y puede difundir va hematgena, ms comnmente a los pulmones y el hgado. Aspiraciones, biopsias, o la escisin
quirrgica puede ser necesaria para obtener el material para el diagnstico. La identificacin microscpica de los grnulos de azufre en pus o tejidos
establece el diagnstico. Necrosis central de las lesiones con los neutrfilos y los grnulos de azufre es prcticamente de diagnstico de la enfermedad.
Por lo general requieren de 5-7 das, pero puede tardar 2-4 semanas para convertirse en positiva, incluso una sola dosis de antibitico puede afectar el
rendimiento de los cultivos. TRATAMIENTO: Requiere un tratamiento prolongado. Tratamiento IV durante 2-6 semanas (por lo general con la penicilinasamoxicilina, penicilina G, ampicilina) seguido de tratamiento oral durante 6-12 meses (por ejemplo, con penicilina o ampicilina) se sugiere para la
infeccin grave y enfermedad voluminosa. En particular la enfermedad de la regin oral y cervicofacial, se puede curar con un tratamiento ms corto. En
alrgicos eritromicina o azitromicina. Si el tratamiento se prolonga ms all del punto de la resolucin de la enfermedad medible (cuantificada mediante
TAC o RM), se minimiza la recada. Agentes alternativos adecuados incluyen las tetraciclinas (por ejemplo, minociclina, 200 mg/da administrado por va
IV o PO cada 12 horas) o clindamicina (2,7 g /d administrado por va IV cada 8 horas).
CASO CLINICO
Mujer de 53 aos, con antecedentes de amigdalectoma en la infancia
y sin manipulaciones dentales recientes. Consult por sensacin de
cuerpo extrao farngeo derecho de 1 ao de evolucin, con expulsin
por boca de material blanquecino al menos en tres ocasiones. El
otorrinolaringlogo extrajo un cmulo de detritus del lecho amigdalar
derecho, y apareci en la zona adyacente una pared de aspecto
inflamatorio. El anlisis anatomopatolgico de la muestra inform de
masa (1,51,51cm). Durante los 6 meses posteriores de seguimiento
present tres episodios de expulsin con la tos, de masas de
aproximadamente 1cm de iguales caractersticas. A la exploracin
fsica destacaba la visualizacin repetida de un cmulo de detritus
blanco en una cavidad en el polo superior del lecho amigdalar derecho,
cavidad atribuible a una cicatrizacin anmala de la amigdalectoma
previa. La higiene bucal era buena. Serologa para el virus de la
inmunodeficiencia humana, negativa. La tomografa computarizada
cervical identific una prominencia inespecfica a nivel amigdalar
derecho, sin objetivar trayectos en partes blandas ni adenopatas.
PREGUNTA
Se realiza cultivo y es positivo para actinomyces israelii, cual es la
conducta mas adecuada para minimizar recaidas?

RESPUESTA
a.- Amoxicilina.
b.- Clindamicina.
c.- Penicilina.
d.- Tetraciclina.
PREGUNTA
Cual de los siguientes factores de riesgo es mas frecuente para la
presencia actinomicosis craneofacial?
a.- Otitis media.
b.- Sinusitis cronica.
c.- Absceso dentario.
d.- Amigdalas hipertrficas.
PREGUNTA
En el caso de la actinomicosis abdominopelviana, cual de los siguientes
factores es el mas frecuente?
RESPUESTA
a.- Cuerpo extrao.
b.- Aborto sptico.
c.- EPI.
d.- LUI.

NOCARDIASIS
CIENCIAS BASICAS: Nocardias son actinomicetos aerbicos saprfitos comunes en el suelo (participa en la degradacin de materia organica). Varias
especies estn asociadas con la enfermedad humana. N. asteroides es la especie ms comnmente asociada con la enfermedad invasiva en el hombre.
N. brasiliensis se asocia ms con lesiones cutneas localizadas. SALUD PBLICA: En los Estados Unidos, 1.100 casos de infeccin por nocardia se

CURSO ENARM CMN SIGLO XXI TEL: 36246001

Pharmed Solutions Institute

PGINA 94

MANUAL DE TRABAJO DEL CURSO ENARM CMN SIGLO XXI


producen cada ao, de los cuales 85 % son pulmonares o sistmicos. El riesgo de enfermedad es mayor de lo habitual entre las personas con deficiencia
de inmunidad mediada por clulas, por ejemplo, que asocia con el linfoma, el trasplante, la terapia con glucocorticoides, o SIDA. Las tasas de mortalidad
son altas entre los pacientes con nocardiosis del cerebro. PATOGENIA: La neumona y la enfermedad diseminada siguen a la inhalacin de micelios.
Nocardiosis causa abscesos con infiltracin de neutrfilos y necrosis. DIAGNOSTICO: Clnico; Enfermedad pulmonar suele ser subaguda, presentando en
das o semanas. Enfermedad extrapulmonar se documenta en > 50 % de los casos, y un poco de afectacin pulmonar es evidente en el 80 % de los
pacientes con enfermedad extrapulmonar. Una tos productiva importante de pequeas cantidades de esputo espeso purulenta, fiebre, anorexia, prdida
de peso y malestar general son comunes, disnea, hemoptisis y dolor pleurtico son menos comunes. La radiografa de trax; suele mostrar infiltrados
nodulares nicos o mltiples de diferentes tamaos que tienden a cavitacin. Empiema se observ en un tercio de los casos. La infeccin puede
diseminarse a los tejidos adyacentes, como el pericardio o el mediastino. Enfermedad extrapulmonar normalmente se manifiesta como abscesos
subagudos en el cerebro, la piel, los riones, los huesos y / o musculares. Algunos abscesos forman fstulas y pequeas cantidades de descarga de pus,
pero no las de los pulmones o el cerebro. Los abscesos cerebrales son generalmente supratentorial, a menudo multiloculada, pueden ser nicos o
mltiples y tienden a enterrarse en los ventrculos o extender en el espacio subaracnoideo. Enfermedad despus de la inoculacin transcutnea es la
celulitis subaguda puede presentar 1-3 semanas despus de una lesin en la piel (a menudo contaminada con tierra). Sndrome linfocutneo: Una lesin
piodrmica se desarrolla en el sitio de inoculacin, con ulceracin central y secrecin purulenta. Esta lesin suele drenada por ndulos SC a lo largo de
los vasos linfticos. Esta forma se asemeja a la esporotricosis. Actinomicetoma: Son formas nodulares de hinchazn en el sitio de traumatismo local, por
lo general en los pies o las manos. Forma de fstulas y la descarga de drenaje seroso o purulento que puede contener grnulos que consisten en masas de
micelio. Las lesiones, que se extienden poco a poco a lo largo de los planos faciales involucrar a la piel, tejido subcutneo y hueso, pueden provocar
graves deformidades. Infecciones oculares: queratitis por lo general sigue a un trauma ocular. La endoftalmitis puede ocurrir despus de la ciruga del ojo
o en la enfermedad diseminada. Exmenes en los que se identifica a la bacteria; broncoscopia, cultivo de esputo o pus (en agar enriquecido con extracto
de levadura y carbn activado), se pueden ver filamentos gramm positivos, biopsia de piel, cerebro, pulmn (tambin aspiracin pulmonar percutnea,
lavado broncoalveolar). TRATAMIENTO: Las sulfonamidas son los frmacos de eleccin, Trimetropim (10-20mg/kg) y sulfametoxaxol (50-100mg/kg). Para
enfermedad grave, los niveles sricos de sulfonamidas deben ser monitoreadas y mantenidas a 100-150 mg /mL. Una vez que la enfermedad est
controlada, la dosis de trimetoprim-sulfametoxazol puede ser reducida por 50%. Las pruebas de sensibilidad pueden guiar tratamientos alternativos.En
enfermedad del SNC; minociclina (200-400mg). Sindrome de celulitis linfocutanea; linezolid (1200mg). Actinomicetoma amikacina (10-15mg/kg),
cefotaxima (6g), ceftriaxona (1-2g), imipenem (2g). La terapia para la nocardiosis debe continuar mientras el paciente permanezca inmunodeprimido. Los
abscesos cerebrales que son grandes o no responde a los antibiticos deben ser aspiradas. Las recadas son comunes. El paciente debe ser seguido por al
menos 6 meses despus de la terapia se haya completado.
CASO CLINICO
Mujer de 42 aos, diestra, portadora de poliglobulia diagnosticada 15
aos antes y catalogada como primaria luego de exhaustivos estudios.
Tratada con cido acetil saliclico en las ocasiones que se le realiz
flebotomas, tuvo mala tolerancia a las mismas. Un mes previo fue
sometida a un procedimiento de extraccin e implante dentario,
presentando 10 a 15 das despus un cuadro de inicio y progresin
insidiosos, en el curso de 15 das, y caracterizado por cefalea, vmitos,
fiebre, a lo que se agregaron posteriormente alteraciones de la
agudeza visual y campimtricas y en las ltimas cuarenta y ocho horas,
depresin de la vigilia. En el examen fsico, la paciente se presentaba
confusa, por momentos excitada, con guio a la amenaza abolido a
izquierda, paresia de VI par izquierdo, parlisis facial central izquierda,
paresia leve de miembros izquierdos, con hiporreflexia y signo de
Babinski. En la paraclnica de laboratorio el hematocrito fue de 6,79 %,
hemoglobina 20,9 mg/dL, VCM 90,1, HBCM 30,7%, leucocitos
12,800/mm3, PMN 80%, linfocitos 17%, eosinfilos 1%, basfilos 2%,
plaquetas 150/mm3, VES 1 mm 1 hora

PREGUNTA
Cual es la conducta diagnostica mas adecuada a seguir?
RESPUESTA
a.- Hemocultivo.
b.- IRM crneo.
c.- TAC crneo.
d.- Serologia
PREGUNTA
La RM con gadolinio evidencia proceso expansivo cortico-subcortical
con anillo grueso irregular de realce con centro necrtico. La
orientacin diagnstica imagenolgica RM orient a un absceso
cerebral, se realizo drenaje y cultivo, que resulto positvo a Norcardia
asteroides. Cual es la conducta teraputica mas adecuada?
RESPUESTA
a.- Linezolid (1200mg).
b.- Minociclina (200-400mg).
c.- Amikacina (10-15mg/kg)
d.- TMP/SMX (3.200/640mg)

TOS FERINA O COQUELUCHE (BORDETELLA PERTUSSIS)


CIENCIAS BSICAS: Bordetella pertussis causa tos ferina, una infeccin aguda del tracto respiratorio, caracterizada por accesos de tos paroxiticos
acompaado de estridor al final de la inspiracin. B. pertussis es un bacilo aerobio Gramm negativo. SALUD PBLICA: La tos ferina es altamente
contagiosa. En los hogares, las tasas de ataque son el 80% de los contactos no vacunados y 20% entre los contactos inmunizados. La tos ferina sigue
siendo una causa importante de morbilidad y muerte infantil en los pases en desarrollo. En los Estados Unidos, la incidencia ha aumentado lentamente
desde 1976, particularmente entre los adolescentes y adultos. Tos persistente de duracin> 2 semanas en un adulto puede ser debido a B. pertussis en
12-30% de los casos. La morbilidad y la mortalidad severa se limitan a lactantes <6 meses de edad. La incidencia en Mxico es de 1-3 por 100,000
habitantes. PATOGENIA: Transmisin por va respiratoria, al estar en contacto con gotas de secreciones, el periodo de contagiosidad se presenta
durante el periodo catarral y la primera semana del inicio de la fase paroxstica. B. pertussis fastidioso se adhiere a las clulas epiteliales ciliadas
(tropismo) de la nasofaringe, donde prolifera y se dirige al epitelio traqueobronquial, es internalizado por las clulas epiteliales, pero no penetra clulas
submucosas, ni invade el torrente sanguneo, se multiplica localmente, y produce una amplia gama de toxinas y productos biolgicamente activos como
toxina pertussis (la cual acta como exotoxina y penetra al torrente sanguneo, induce linfocitosis, activacin de islotes pancreticos), hemaglutininas
filamentosa (adherencia a clulas de epitelio), pertactina (adherencia y altamente inmunogenica), BrkA (adherencia y resistencia al complemento),
adenilciclasa (inhibe funcin fagocitica). DIAGNOSTICO: Enfermedad prolongada de tos. Los sntomas suelen ser ms graves en los bebs y nios
pequeos. Tres fases: La fase catarral; posterior a periodo de incubacin de 7-10 das es similar al resfriado comn (rinorrea, estornudos, lagrimeo, tos
leve progresiva) y tiene una duracin de 1-2 semanas y es el periodo de mayor contagiosidad. La fase paroxstica o de estado; los accesos de tos son
paroxsticos (5-10 golpes de tos), caractersticamente si inspiracin y suelen acompaarse de vomito y cianosis y terminar con un estridor inspiratorio
prolongado que le da el nombre de tos coqueluche. El nmero de accesos le da la gravedad, llegando a 15-20 por da, sin son frecuentes, puede
presentarse sudacin, lasitud y somnolencia y son de predominio nocturno, tiene una duracin de 2-6semanas. Fase convaleciente: inicia cuando los
accesos de tos dejan de ser espasmdicos y dura varias semanas, durante la fase de accesos pueden exacerbarse si el paciente presenta infeccin en vas

CURSO ENARM CMN SIGLO XXI TEL: 36246001

Pharmed Solutions Institute

PGINA 95

MANUAL DE TRABAJO DEL CURSO ENARM CMN SIGLO XXI


areas superiores. En cuadros ms severos predominan datos de dificultad respiratoria, cianosis y apnea, ms que accesos de tos. Laboratorio:
Aislamiento de Bordetella pertussis es 100% especifico, la posibilidad de positividad declina si la muestra se toma >2 semanas del inicio de la
sintomatologa y la muestra para el cultivo se toma de la nasofaringe posterior. PCR a aumentado su sensibilidad, pero no se usa como mtodo exclusivo
y no sustituye al cultivo. Deteccin de anticuerpos fluorescentes (DAF), til como prueba de deteccin rpida, pero la sensibilidad es baja. Serologa;
demostracin de una elevacin sustancial 4 veces los ttulos de anticuerpos contra diferentes antgenos. COMPLICACIONES: La ms frecuente y causas
de muerte es la neumona bacteriana secundaria (S. peneumoniae, H. influenzae), otras son crisis convulsivas y encefalopata TRATAMIENTO: Del 8090% de pacientes con tos ferina sin tratamiento depuran espontneamente a B. pertussis de la nasofaringeo en las 3-4 semanas. Manejo principal ente
de soporte. La eritromicina es el antibitico de eleccin, preferentemente en forma de estolato a dosis de 40mg/kg/da por 14 dias. Alternativas
claritromicina por 7 das igual efectividad y mejor tolerancia. Azitromicina y TMP/ SFX. En menores de un mes se da azitromicina (eritro y claritromicina
estn contraindicadas). Para la tos se puede manejar benzonatato o salbutamol en nebulizador. PROFILAXIS POSTEXPOSICION: Contactos asintomticos
de un caso ndice que se encuentre dentro de los 21 das del inicio de los accesos de tos, deber recibir profilaxis con antibiticos antes mencionados.
PREVENCIN: Vacuna combinada de toxoide tetnico y difterico (DPT). En Mxico A partir de 1999, poco despus de la introduccin de la vacuna triple
viral, se introduce una vacuna pentavalente (DTwP-hepatitis B-Hib), en tres dosis a los 2,4 y 6 meses de edad y refuerzos con DPT a los 4 aos de edad,
para que se adquiera inmunidad completa se requieren todas las dosis. A partir de 2007, se efectu un cambio biolgico a una vacuna pentavalente a
base de vacuna acentuar de pertussis (DTaP-IPV-Hib).
CASO CLINICO
Una mujer de 64 aos ingresada por una disnea creciente y dolor
torcico. La paciente tena antecedentes de carcinoma pulmonar no
microctico y estaba recibiendo tratamiento. Anteriormente haba
fumado 4-5 cigarrillos al da durante un periodo de hasta 10 aos. A la
exploracin, se trataba de una mujer con sensacin de enfermedad y
emaciacin, que estaba clnicamente anmica y presentaba signos de
un derrame pleural izquierdo masivo en la TC torcica. La frecuencia
del pulso era de 85/min, la presin arterial de 120/85, y no haba
signos de insuficiencia cardiaca. La radiografa de trax confirm el
derrame unilateral izquierdo. La hemoglobina era de 9,1g/100ml, con
un frotis de sangre normocrmico y normoctico. La velocidad de
sedimentacin globular fue de 90mm en la primera hora. Se realiz
una aspiracin pleural, en la que se obtuvieron 1.500ml de lquido
turbio. Dada la persistencia de las colecciones de lquido pleural, 2 das
despus de la segunda aspiracin se coloc un drenaje de trax. El
nivel de protenas fue de 35g/l y la concentracin de glucosa, de
0,3mmol/l. No se observaron clulas malignas, pero en la PCR se
demostr la presencia de B. pertussis. La paciente no present fiebre
ni manifestaciones clnicas de infeccin, y el recuento leucocitario en
sangre fue normal. Adems, no hubo clnica de diarrea y los
coprocultivos, urinocultivos y hemocultivos fueron repetidamente
negativos.
PREGUNTA
Cual es la conducta antibitica mas apropiada para el caso?
RESPUESTA
a.- Claritromicina.
b.- Azitromicina
c.- TMP/ SFX
d.- Eritromicina.
CASO CLINICO
Lactante de 47 das sin antecedentes de inters que acude a la
consulta de su pediatra por un cuadro de tos y mucosidad nasal de

cuatro das de evolucin, habindose hecho la tos ms persistente en


forma de accesos repetidos en los que le han observado cianosis
perioral y que ocasionalmente terminan con un vmito; se encuentra
afebril y con una reduccin en la ingesta de leche materna. En la
exploracin fsica se objetiva un buen estado general, con buena
coloracin y perfusin perifrica, sin signos de dificultad respiratoria y
auscultacin cardiopulmonar normal; se decide dejar en observacin,
durante la cual se objetiva uno de estos episodios en los que presenta
un acceso con varios golpes de tos durante la espiracin y con una
inspiracin ruidosa en forma de gallo al finalizar el acceso,
acompaado de cianosis facial.
PREGUNTA
Cul es el agente etiolgico ms probable para este caso?
RESPUESTA
a.- Moraxella catarralis
b.- Bordetella pertusis
c.- Franciciella tularensis
d.- Streptococo pneuoniae
PREGUNTA
Cul es la complicacin menos frecuente para este caso?
RESPUESTA
a.- Hemorragias subcojuntivales
b.- Sincope
c.- Neumotorax
d.- Petequias en cara u trax
PREGUNTA
Cul es la conducta teraputica ms adecuada para este paciente?
RESPUESTA
a.- Azitromicna 10mg/kg
b.- Eritromicina 20mg/kg
c.- TMP/SFX 15mg/kg
d.- Amoxicilina 30mg/kg

MORAXELLA CATARRHALIS
CIENCIAS BSICAS: Moraxella catarrhalis es un coco gram-negativo, que se asemeja a Neisseria. En el gnero Moraxella podemos identificar cuatro
especies: M. catarrhalis, M. caviae, M. ovis, M. cuniculi. Forma parte de la flora normal de las vas respiratorias superiores. SALUD PUBLICA: M.
catarrhalis coloniza hasta el 50% de los nios sanos (muy relacionados con IRA) y hasta el 3-7% de los adultos sanos. Las tasas de infeccin pico son a
finales de invierno / principios de primavera. Generalmente no se le considera un agente primario en las infecciones del tracto respiratorio inferior, esto
cambia al referirse a individuos mayores de 50 aos, donde es considerado un patgeno primario de vas respiratorias bajas. Se puede decir que es un
agente oportunista que se aprovecha de las condiciones predisponentes del husped para causar enfermedad y formar parte de los patgenos humanos
emergentes. DIAGNOSTICO: La otitis media y sinusitis: M. catarrhalis es la tercera causa ms frecuente de otitis media en los nios y es importante
aislarlo de los casos de sinusitis aguda y crnica. Conjuntivitis en neonatos. Traqueobronquitis purulenta, bronquitis, neumona: La mayora son
pacientes > 50 aos de edad y con EPOC (a menudo con el cncer de pulmn tambin). Laringitis, queratinitis, uretritis. Los sntomas son de leves a
moderados, la enfermedad invasiva (por ejemplo, enfisema) es poco frecuente. Laboratorio: Se cultivan muestras de esputo, sangre, secreciones
bronquiales, aspirado transtraqueal, lavado broncoalveolar y biopsia pulmonar. Un mtodo de diferenciacin con la Neisseria, es que la Moraxella
Catarrhalis no fermenta los carbohidratos y produce DNasa. Adems produce Butirato esterasa, que constituye la base de la prueba fluoromtrica rpida
para identificarla. En la tincin aparecen bacilos, cocobacilos o cocos pequeos Gramnegativos. TRATAMIENTO: Este microorganismo presenta una alta
resistencia a beta-lactmicos como la penicilina, ampicilina y amoxicilina. Se puede usar amoxicilina/ac. clavulnico, cefalosporinas de segunda y tercera

CURSO ENARM CMN SIGLO XXI TEL: 36246001

Pharmed Solutions Institute

PGINA 96

MANUAL DE TRABAJO DEL CURSO ENARM CMN SIGLO XXI


generacin, TMP-SMZ (mezcla de una parte de trimetoprim y cinco partes de sulfametoxazol). Esta bacteria presenta sensibilidad a la quinolonas,
eritromicina, tatraciclina, amikacina, imipenem, meropenem, cloranfenicol.
CASO CLINICO
Masculino de 15 aos, fue ingresado con una historia de 3 das de
dolor de cabeza y de 2 das de petequias generalizadas, nuseas y
vmitos. Temperatura 36,9 , FC 93 lpm y la TA 125/82 mmHg. Estaba
alerta y se quej de rigidez en el cuello. No hubo ninguna secuela
motora o sensorial, y el resto de su examen fsico sin complicaciones.
Leucocitos perifricos de 30.290 / l (96 % neutrfilos), hemoglobina de
15,4 g / dl, y plaquetas de 151.000 / l . CSF tena un aspecto turbio con
un aumento de la concentracin de protena de 575 mg / dl, leucocitos
de 35.500 / l ( 90 % de neutrfilos ) y glucosa de 10 mg / dl.

CASO CLINICO
Masculino de 81 aos, con antecedentes de cncer de pncreas y
cirrosis heptica por virus de la hepatitis C. Fue ingresado por inversin
del ciclo vigila sueo. A su ingreso, tena ictericia esclertica.
Leucocitos 6950 / l (73% neutrfilos), hemoglobina de 12,8 g / dl,
plaquetas de 60.000 / l. LCR estaba claro con protena de 38 mg / dl,
glbulos blancos de 9/L, glucosa de 76 mg / dl, nitrgeno ureico /
creatinina 41.8/2.2 mg / dl, amonaco de 287 mg / dl, bilirrubina total
de 3,1 mg / dl.

PREGUNTA
Cual es la conducta antibitica mas apropiada para el caso?
RESPUESTA
a.- Ampicilina y acido clauvulanico.
b.- Ceftazidima y netilmicina.
c.- Quinolonas e imipenem.
d.- Eritromicina y meropenem.

PREGUNTA
Cual es la conducta antibitica mas apropiada para el caso?
RESPUESTA
a.- Ampicilina.
b.- Ceftaxima.
c.- Imipenem.
d.- Eritromicina.

BRUCELOSIS
CIENCIAS BSICAS: La Brucella es una bacteria gramm negativa, viven en el interior de las clulas del sistema fagocitico mononuclear
(intracelularfacultativo), por ello tiene un curso recidivante, tratamiento difcil y prolongado. Todas las especies de Brucella tienen como reservorio las
ubres o tero gestante de las hembras y los genitales del macho. La B. melitensis (caprinos y ovinos), es el agente causal ms importante, seguido de B.
abortus (bovinos) y B. suis (porcinos) y B. canis (de perros). La brucelosis es una zoonosis sistmica en el humano puede afectar cualquier rgano o
sistema, generalmente adopta un curso crnico, ataque al estado general, baja letalidad. SALUD PBLICA: OMS, la considera la zoonosis de mayor
distribucin en el mundo. Cada ao ocurren medio milln por lo menos de casos nuevos. En Mxico existe en todo el territorio nacional predominando
en un rea triangular con la base en la frontera norte y el vrtice en el centro. PATOGENIA: La brucelosis se transmite por ingestin, inhalacin o
exposicin percutnea o mucosa, la enfermedad en los seres humanos por lo general se asocia con la exposicin a animales infectados o sus productos,
ya sea en lugares de trabajo (por ejemplo, el trabajo de los
BRUCELOSIS
TUBERCULOSIS
mataderos, la agricultura) o el contexto familiar (por ejemplo, el
SITIO
Lumbar y otros
Dorsolumbar
consumo de alimentos, especialmente productos como leche no
LAS VRTEBRAS
Mltiple o contiguas
Contiguo
pasteurizada, queso, mantequiilla) contaminados. Ya en el organismo
DISQUITIS
Tarde
Temprano
son fagocitados por leucocitos, PMN, macrfagos tisulares, donde
CUERPO
Intacto hasta el final
Morfologa de perdida temprana
COMPRESION
DEL
Raro
Comn
pueden sobrevivir y multiplicarse. Una vez rebasada la barrera
CANAL
linftica, llegan a la circulacin sistmica, y de esta manera son
EPIFISISTIS
Anterosuperior Signo de
General: las regiones de disco
transportados a los diferentes rganos del sistema fagocitico
Poms
superior e inferior , central,
mononuclear (hgado, bazo, medula sea, ganglios). La persistencia
subperistico
de Brucella dentro de las clulas es debido a la inhibicin de la fusin
OSTEOFITOS
Anterolateral (pico de
Inusual
loro)
fagosoma-lisosomal, la degranulacion y la activacin del sistema
DEFORMIDAD
En cua poco comun
Cua anterior, giba
mieloperoxidasa. La liberacin de las bacterias de las clulas
ABSCESOS
Pequeos,
bien
Prdida Comn y discreta, apfisis
necrticas puede sobrepasar la capacidad fagocitica. Las
PARAVERTEBRALES
localizados
transversa
manifestaciones clnicas estn determinadas en gran parte por la
RECUPERACIN
Esclerosis,
todo
el
variable
liberacin de una endotoxina y el grado de hipersensibilidad a los
cuerpo
antgenos brucelares.
DIAGNOSTICO: Clnico: El perodo de
incubacin de 1 semana hasta varios meses es seguido por el desarrollo de la fiebre ondulante, sudores, aumento de la apata, fatiga y anorexia y
sntomas inespecficos como dolor de cabeza, mialgias y escalofros. La brucelosis menudo se presenta con uno de los tres patrones: una enfermedad
febril similar pero menos grave que la fiebre tifoidea, fiebre y monoartritis aguda, tpicamente de la cadera o de la rodilla, en un nio pequeo (artritis
sptica), o la fiebre de larga duracin y baja de la espalda o dolor en la cadera en un hombre mayor (osteomielitis vertebral). Brucelosis puede causar
linfadenopata, hepatoesplenomegalia, epididimoorquitis, compromiso neurolgico y el absceso focal. El microorganismo se cultiva con xito en el 50-70
% de los casos, pero la cultura de identificacin suele tardar hasta 6 semanas. El diagnostico de certeza es el aislamiento de brucella, en hemocultivo o
cultivo de medula sea. Fiebre prolongada acompaada de ataque articular nos hace sospecharla. Ensayos de aglutinacin para IgM son positivos en la
infeccin temprana. Los ttulos individuales de 1:160 y 1:320-1:640 son diagnsticos en reas no endmicas y endmicas, respectivamente. La
brucelosis se debe distinguir de la tuberculosis, y si esta distincin no es posible, el rgimen debe adaptarse para evitar la monoterapia inadvertida para
la tuberculosis. Brucelosis tiende a causar menos destruccin del hueso y la articulacin de la tuberculosis. TRATAMIENTO: Se recomienda como primera
eleccin estreptomicina a una dosis de 750 mg a 1 g al da (o gentamicina a 5-6 mg / kg al da) durante 14-21 das ms doxiciclina en una dosis de 100 mg
C/12 hrs durante 6 semanas. Alternativa: TMP/SFX + Rifampicina por 6 semanas. Enfermedad compleja (por ejemplo, enfermedad neurolgica
significativa o endocarditis) requiere al menos 3-6 meses de tratamiento con mltiples agentes. Alternativa: rifampicina (600-900 mg / d) ms doxiciclina
(100 mg bid) durante 6 semanas. El trimetoprim /sulfametoxazol, se puede dar en vez de doxiciclina - por ejemplo, a los nios o las mujeres
embarazadas. La recada se produce en aproximadamente el 30 % de los casos, por lo general debido a la falta de cumplimiento. El paciente debe
controlarse por lo menos 2 aos.
CASO CLINICO
Un paciente varn de 20 aos de edad, que se acababa de alistar en el
ejrcito, se present refiriendo tos, expectoracin, dolor punzante en
el hemitrax izquierdo, sudoracin nocturna, anorexia, fiebre

CURSO ENARM CMN SIGLO XXI TEL: 36246001

ondulante y disnea de casi 10 das de evolucin. Tambin refiri la


prdida de 3kg de peso en el ltimo mes. Presentaba fiebre de 38,4C y
su frecuencia respiratoria era de 22/min. La auscultacin torcica
revel estertores inspiratorios basilares bilaterales. Se detect un

Pharmed Solutions Institute

PGINA 97

MANUAL DE TRABAJO DEL CURSO ENARM CMN SIGLO XXI


recuento de leucocitos de 27.930/mm3 y una velocidad de
sedimentacin de 68mm/h. El valor de protena C reactiva era de
141mg/dl. La radiografa de trax mostr un infiltrado neumnico en la
zona media derecha y una opacidad homognea en la zona inferior
izquierda. Se document el crecimiento de Brucella spp. en el
hemocultivo.
PREGUNTA
Cual es la conducta antibitica mas apropiada para el caso?
RESPUESTA
a.- Rifampicina
b.- Estreptomicina
c.- Gentamicina
d.- Doxiciclina
CASO CLINICO
Varn de 74 aos haba trabajado, la mayor parte de su vida, en
criaderos de aves de zonas rurales del rea Metropolitana. Su cuadro

clnico se haba iniciado aproximadamente tres meses antes con


compromiso moderado del estado general, fiebre vespertina y prdida
de peso, que en ese momento cuantificaba en diez kilos. Slo se
verific la existencia de fiebre irregular, alrededor de 38C, y
decaimiento. Se inici estudio del sndrome febril prolongado. El
hemograma mostraba anemia moderada (hematocrito 34,1%,
hemoglobina 11,3 gr%), leucocitos en rango normal, neutrofilia de
76%) sin desviacin a izquierda y VHS de 36 mm/h. Albuminemia (3,3
gr%), la protena C reactiva moderadamente elevada.
PREGUNTA
Cual es la conducta antibitica mas apropiada para el caso?
RESPUESTA
a.- Rifampicina
b.- Estreptomicina
c.- Gentamicina
d.- Doxiciclina

TULAREMIA
CIENCIAS BASICAS: Las infecciones humanas causadas por Francisella tularensis, que es una bacteria facultativa intracelular, se producen a travs de la
interaccin cuando el insecto muerde, o chupa sangre (especialmente las garrapatas y las moscas tabanid), animales salvajes o domsticas (por ejemplo,
conejos salvajes, ardillas), o el medio ambiente. El organismo puede persistir durante meses en el barro, agua, y los cadveres de animales en
descomposicin. Ms de la mitad de los casos en Estados Unidos se producen en Arkansas, Oklahoma y Missouri. PATOGENIA: La F. tularensis entra en la
piel o las membranas mucosas a travs de mordeduras o rasguos inaparentes o se adquiere por inhalacin o ingestin. Su patogenicicdad se debe a su
capacidad para sobrevivir dentro de macrfagos no estimulados. Los macrfagos fagocitan con facilidad a la F. tularensis pero esta resiste la muerte
frente a los radicales derivados del oxigeno, una respuesta neutrofilica local contraresta la infeccin, esta respuesta evoluciona a una ulcera.
DIAGNOSTICO: El perodo de incubacin es de 2-10 das. La tularemia a menudo comienza con un comienzo agudo de fiebre, escalofros, dolor de cabeza
y mialgias. Uno de varios sndromes puede desarrollar: Tularemia Ulceroglandular / glandular (75-85 % de los casos). El sello es una induracin, eritema,
lceras que no sanan 1-3 semanas duraderos (forma ulceroglandular) que comienza como una ppula pruriginosa, ulcera, bordes drsticamente
marcados y un exudado amarillo, y desarrolla una base de negro. Una lesin primaria de piel puede no ser evidente en el 5-10% de los casos (forma
glandular). Linfadenopata est relacionada con la ubicacin de la picadura de la garrapata; linfticos inguinales / femoral estn afectadas con mayor
frecuencia en los adultos debido a la frecuencia de picaduras en las piernas. Los ganglios linfticos pueden llegar a ser fluctuante y drene
espontneamente. Tularemia oculoglandular: Infeccin de la conjuntiva, por lo general por el contacto con los dedos contaminados, resultados en
conjuntivitis purulenta con adenopata regional y dolor debilitante. Doloroso linfadenopata preauricular es nico a la tularemia. Orofarngea y
gastrointestinal: Adquirida por ingestin, la infeccin se puede presentar con faringitis y adenopata cervical, ulceraciones intestinales, adenopatas
mesentricas, diarrea, nuseas, vmitos y dolor abdominal. Tularemia pulmonar: La infeccin se adquiere por inhalacin o por va hematgena. El
paciente se presenta con una tos no productiva, disnea, dolor torcico pleurtico, infiltrados irregulares o lobar bilaterales, derrame pleural y empiema
ocasionales en la radiografa de trax. Tularemia tifoidea: Consta de fiebre y signos de sepsis y sin hallazgos focales. Laboratorio: Tincin policromtica de
muestras clnicas (tincin de poca ayuda). Serologa mediante microaglutinacin o la prueba de aglutinacin en tubo. Un solo ttulo de 1:160 o un
aumento de cuatro veces en el ttulo despus de 2-3 semanas se consideran positivos. La cultura es difcil y supone un riesgo importante para el personal
de laboratorio. Mtodos de reaccin en cadena de la polimerasa (PCR) se han utilizado para detectar el ADN de F. tularensis en muestras clnicas.
TRATAMIENTO: La gentamicina se considera el frmaco de eleccin para adultos (5 mg / kg al da C/12hrs) y nios (2,5 mg / kg tres veces al da o 5 mg /
kg c/12) con tularemia. Estreptomicina (1 g cada 12 horas) tambin es eficaz, pero no es tobramicina. Desaparicin de la fiebre por lo general ocurre
dentro de 2 das, pero la curacin de las lesiones de la piel y los ganglios linfticos puede tardar 1-2 semanas. Late supuracin de ganglios linfticos
puede ocurrir, con el tejido necrtico estril. Enfermedad leve a moderada responden rpidamente a tratamiento pueden ser tratados durante 5-7 das,
de lo contrario, el tratamiento se administra durante 7-10 das. Las alternativas incluyen tetraciclinas o cloranfenicol (tasas de recada de hasta el 20 %).
Las fluoroquinolonas han demostrado ser prometedores, pero los ensayos clnicos estn pendientes.
CASO CLINICO
Se trata de un paciente varn de 36 aos, trabajador en un matadero
industrial de aves. Entre sus aficiones destaca la caza de liebres,
conejos y jabales, adems de su hobby como taxidermista (su ltima
pieza disecada fue un zorro). No tiene perros ni gatos y niega haber
recibido transfusiones o tener otras enfermedades en curso. El
paciente acudi al mdico por presentar fiebre de 40 grados,
sudoracin, quebrantamiento general, mialgias, odinofagia y cefalea.
Se interpret el cuadro clnico como un sndrome gripal y se trat con
paracetamol. La fiebre persisti durante los siguientes 3 das, y por ello
acudi al hospital. Tras una exploracin fsica y una radiografa de
trax normal, continu con el mismo diagnstico y tratamiento.
Aproximadamente una semana despus del comienzo de los sntomas,
present una tumoracin dolorosa en la axila derecha que aument de
tamao progresivamente hasta alcanzar los 5 cm de dimetro. De las
pruebas complementarias que all se le realizan destacamos las
siguientes: la radiografa de trax es normal, la analtica presenta
neutrofilia con desviacin a la izquierda y la ecografa axilar derecha
revela un conglomerado de adenopatas de 5 x 3 cm.

CURSO ENARM CMN SIGLO XXI TEL: 36246001

PREGUNTA
Cual es la conducta antibitica mas apropiada para el caso?
RESPUESTA
a.- Rifampicina
b.- Estreptomicina
c.- Gentamicina
d.- Doxiciclina
CASO CLINICO
Mujer de 49 aos remitida desde otro centro para estudio de masa
hiliar derecha. Como antecedentes personales presenta una DM tipo 2.
Trabaja como ganadera y refiere contacto con liebres. Inicialmente el
motivo de consulta fue fiebre, adenopata femoral y sndrome
constitucional. Exploracin fsica dentro de la normalidad En la
analtica destaca GPT 152 UI/L y GGT 151 UI/L con leve hipoxemia al
ingreso con FiO2 0,21 (pO2 56 mmHg). En Rx de trax masa hiliar
derecha de aproximadamente 4 cm con infiltrado a nivel de LSD. Se
realiz broncoscopia observndose inflamacin en la carina de divisin

Pharmed Solutions Institute

PGINA 98

MANUAL DE TRABAJO DEL CURSO ENARM CMN SIGLO XXI


entre el segmento anterior y apical del LSD. Test de microaglutinacin
para Francisella tularensis de 128. Repetido a las tres semanas se
alcanza un ttulo superior a 1048
PREGUNTA
Cul es la conducta teraputica ms adecuada a seguir?
RESPUESTA
a.- Estreptomicina
b.- Amikacina
c.- Eritromicina
d.- Ampicilina
PREGUNTA
De las formas clnicas. Cul sera la menos probable observar en el
paciente?

RESPUESTA
a.- Ulceroglandular
b.- Oculoglandular
c.- Glandular
d.- Pulmonar
PREGUNTA
Cul es la mortalidad para este paciente?
RESPUESTA
a.- 2%
b.- 5%
c.- 10%
d.- 30%

INFECCIONES POR ESTAFILOCOCOS


CIENCIAS BASICAS: Los estafilococos son cocos gram-positivos que forman racimos de uvas en la tincin de Gramm, son catalasa positivo (a diferencia de
los estreptococos), no mviles, aerbicos y anaerobias facultativas. Staphylococcus aureus, se distingue de otros estafilococos por su produccin de
coagulasa, es la especie ms virulentas, causando la enfermedad a travs de ambos mecanismos no mediadas por toxinas y mediada por toxina. Los
estafilococos coagulasa negativos (ScoN) son menos virulentas que S. aureus, pero son causas importantes y comunes de las infecciones de dispositivos
de prtesis. S. epidermidis con mayor frecuencia causa la enfermedad, este organismo
DEFINICIONES CLINICAS PARA SINDROME DE CHOQUE TOXICO
es un componente normal de la piel, orofarngea, y la flora vaginal. S. saprophyticus es
Temperatura > 38.9
una causa de las infecciones urinarias. Otras dos especies de ScoN, S. lugdunensis y S.
Exantema macular difuso
schleiferi, son ms virulentas y causan infecciones graves tales como vlvulas nativas
Descamacin 1-2 semanas posteriores al inicio de la enfermedad
endocarditis y osteomielitis. SALUD PBLICA: S. aureus es responsable 25 a 35 % de los
(+en palmas y plantas)
casos de endocarditis bacteriana. Las tasas de mortalidad oscilan entre 20 a 40% a
Hipotensin: Presin sistlica <90mmHg. Cada ortosttica de la
pesar de la disponibilidad de antibiticos eficaces. PATOGENIA: Una vez que se
presin diastlica > 15mmHg de acostado a sentado, sincope
ortosttico o mareo.
presenta la infeccin, la invasin local y sistmica ocurre por va hematgena y
Afeccin multisistmica: 3 o ms de las siguientes
liberacin de toxinas. Localmente los organismos pueden invadir o necrosar el tejido y
Gastrointestinal; vomito o diarrea al inicio de la enfermedad
ocasionar una potente respuesta inflamatoria (como resultado de abrasiones,
Muscular; mialgia severa o elevacin de CPK, 2 veces arriba de lo
quemaduras, herida, inyecciones o a travs de catteres), mediada principalmente por
normal
PMN (respuesta primaria). La formacin de abscesos (microcpsula polisacrido
Mucosas; hiperemia en conjuntiva, orofaringe, vagina
antifagoctica, facilita la evasin de las defensas del husped, til en la formacin de
Renal; nitrgeno ureico o creatinina por lo menos dos veces arriba
absceso) es comn, con un centro necrtico que est formado de pus y una pared de
de lo normal o sedimento urinario con piuria (>5 leucos x campo),
fibrina, que dificulta la penetracin de antibiticos, y protege de defensas del husped,
en ausencia de infeccin de vas urinarias
Heptica; bilirrubina total, transaminasas 2 veces por arriba de lo
las bacterias se adhieren formando una biopelcula similar a la formada por los ScoN. La
normal
infeccin puede diseminarse localmente por la formacin de trayectos sinuosos y
3
Hematolgica: plaquetas <100,000/mm
abscesos secundarios. La diseminacin hematgena puede infectar articulaciones,
SNC; desorientacin o alteraciones en la conciencia sin focalizacin,
vlvulas cardiacas y otros tejidos. Para el desarrollo del choque sptico los monocitos y
en ausencia de fiebre o hipotensin
macrfagos tienen el papel principal, liberan TNF , IL-1, IL-6, IL-8, posterior al contacto
Criterios de laboratorio: Estudios con resultado negativo:
con el estafilococo, peptidoglocano o acido lipoteicoico, en respuesta a esto las vas del
Cultivos de sangre, faringe, LCR (excepto positivo en sangre para S.
complemento y coagulacin se activan, se metaboliza el acido araquidnico y le libera
aureus)
Ttulos de anticuerpos para leptospira o antiestreptolisinas o
el factor de activacin de plaquetas; provocndose hipotensin, fiebre, fuga capilar,
sarampin
CID y disfuncin multiorgnica. Los estafilococos coagulasa negativos (ScoN), se
Clasificacin de caso:
adhieren a cuerpos extraos (a travs del acido teicoico en su pared celular y la
Probable; 5 de los 6 datos clnicos descritos
produccin de slime que recubre el material extrao) y provocan infecciones difciles
Conformado; Los 6 datos clnicos descritos anteriormente,
de tratar (flebitis hasta bacteriemia y choque sptico) sin retirar el material protsico
incluyendo descamacin, a menos que el paciente fallezca antes de
(catteres). La bacteriemia por estafilococo puede complicarse con endocarditis o
la descamacin
sndrome sptico. El estafilococo se adhiere vidamente a la clula endotelial a travs
de interacciones adhesina-receptor. Algunos productos de estafilococos son considerados como superantigenos, que provocan enfermedades graves. La
toxina-1 del sndrome de choque toxico, se une a todas la regiones variables de la clula. DIAGNOSTICO: Infecciones de piel y tejidos blandos; imptigoinfeccin superficial de piel, buloso (30%) o no buloso (70%), este inicia con una ppula eritematosa, que se presenta en lesiones previas por picadura de
insectos, varicela, eczema, trauma o abrasiones, la ppula progresa a vescula y despus a costra mielisrica, la topografa de las lesiones es alrededor de
nariz y boca raro en extremidades. Foliculitis- pequeas ppulas o pstulas centradas en el folculo piloso y rodeado de eritema. Fornculos-ndulo
profundo y doloroso de color rojizo con formacin de pus, forma pequeos abscesos por la necrosis del aparato polisebaceo. Paroniquia-infeccin que
rodea el borde una ua de mano o pie. Celulitis, matitis, infecciones de heridas quirrgicas, conjuntivitis purulenta, endoftalmitis. Endocarditis
infecciosa-en asociacin con el uso de drogas inyectables, vlvula protsica, nosocomial, los pacientes tienen fiebre alta, aspecto txico, y dolor torcico
pleurtico y producir esputo purulento que a veces sangriento. La radiografa de trax puede revelar embolia sptica: lesiones pequeas y perifricas.
Pericarditis como consecuencia de una diseminacin contigua de una infeccin respiratoria, es rara. Infecciones en huesos y articulaciones; osteomielitis
derivada de la diseminacin hematgena o propagacin contigua desde un sitio de tejido blando (por ejemplo, lceras diabticas o vasculares).
Osteomielitis hematgena en nios frecuentemente en la metstasis de huesos largos y se presenta con fiebre, dolor de huesos, y la renuencia a
soportar peso. Leucocitosis, aumento de la VSG y hemocultivos positivos son tpicos. Osteomielitis hematgena en los adultos es a menudo vertebral. La
osteomielitis por infecciones de tejidos blandos contiguos se sugiere por la exposicin del hueso, un trayecto fistuloso drenaje, la falta de cura, el drenaje
continuo. S. aureus es tambin causa comn de artritis sptica, asociada con el trauma o ciruga, o se debida a la diseminacin hematgena, hacer
hemocultivo y puncin articular, la evaluacin de liquido sinovial debe incluir cultivo, tincin de gramm, cuenta celular (75% de PMN), glucosa (2/3 de la
srica) y protenas. Infecciones respiratorias; otitis media, traquetis bacteriana, neumona nosocomial-se produce principalmente en pacientes
intubados, en unidades de cuidados intensivos, tienen un mayor volumen de esputo purulento, fiebre y nuevos infiltrados pulmonares y puede

CURSO ENARM CMN SIGLO XXI TEL: 36246001

Pharmed Solutions Institute

PGINA 99

MANUAL DE TRABAJO DEL CURSO ENARM CMN SIGLO XXI


desarrollar dificultad respiratoria, rara neumona de la comunidad. Infecciones en SNC; meningitis-por extensin de un foco paramenngeo, durante la
bacteriemia se puede producir cerebritis y posteriormente formacin de un absceso cerebral. La incidencia de bacteriemia durante la siembra
metastsica se ha estimado para ser tan alta como 31 %. Los huesos, las articulaciones, los riones y los pulmones son los ms comnmente infectados.
Las infecciones por SARM pueden tener muchas presentaciones inusuales (por ejemplo, la fascitis necrotizante, neumona necrtica, sepsis, prpura
fulminante) que reflejan el aumento de la virulencia de las cepas CA MRSA. ENFERMEDADES MEDIADAS POR TOXINAS; El sndrome de shock txico
(TSS): Se presentaba en mujeres menstruando que utilizaban tampones de gran absorcin. En la actualidad se conoce que cualquier paciente con
infeccin por estafilococo puede presentar TSS, as en la actualidad <50% de casos reportados se relacionan al uso de tampones y alrededor de 10% se
relacionan a diversas infecciones por estafilococo (empiema, bacteremia, abscesos, infecciones quirrgicas). Ms comn en mujeres jvenes. El sndrome
se relaciona con la enterotoxina F o exotoxina pirognica C o toxina-1 del sndrome de choque toxico, pero las endotoxina A a la E pudieran contribuir a
la presentacin del sndrome; puede manifestarse con signos y sntomas leves hasta una enfermedad fatal rpidamente progresiva, inicio sbito de
exantema difuso (difuso, escalatiniforme, predomina en aras de flexin), fiebre, alta, vomito, diarrea, cefalea, faringitis, mialgias, alopecia, hipotensin,
falla multiorgnica, por la pobre perfusin tisular y dao directo de los mediadores, falla hematolgica. Los cambios hemodinmicas incluyen
disminucin en el tono vasomotor y fuga no hidrosttica del liquido del espacio intravascular al intersticial. Las complicaciones que pueden llevar a la
muerte son choque refractario, falla renal oligrica, arritmia ventricular, CID y pulmn de choque. La intoxicacin alimentaria: La inoculacin de la toxina
de S. aureus, por los manipuladores de alimentos, coloniza los alimentos. La toxina crece despus en los alimentos que promueven el crecimiento, como
la crema, ensalada de papas, jamn, pollo o carne procesada. La toxina estable al calor no se destruye. Inicio de la enfermedad es rpida y explosiva, que
ocurre dentro de 1-6 h de la ingestin de alimentos contaminados. Los principales signos y sntomas son nuseas y vmitos, pero pueden ocurrir diarrea,
hipotensin y deshidratacin. La fiebre est ausente. Los sntomas se resuelven en 8-10 h. El sndrome de piel escaldada estafiloccica (SSSS): Tambin
conocida como enfermedad de Ritter, caracterstico la piel se desprende como si se hubiese quemado, afecta ms frecuentemente a los recin nacidos y
los nios. La piel es frgil, se puede formar bullas de pared delgada que contienen lquido, su manejo es control de electrolitos y lavado de manos
estricto as como antibiticos. El signo de Nikolsky la friccin leve sobre la piel produce un desprendimiento de las capas superficiales de la piel. La bula
tpica se rompe deja una rea denudada dejando una superficie hmeda y finalmente ocurrir descamacin. TRATAMIENTO: xito inicial con penicilinas
semisintticas resistentes a las penicilinasas como meticilina, oxacilina, nafcilina, cloxacilina y dicloxacilina. Estos agentes fueron utilizados ampliamente,
hasta la aparicin de cepas resistentes a meticilina (SARM), actualmente se ha descontinuado su uso debido a que produce nefritis intersticial. La
nafcilina y oxacilina se usan en otros pases. En Mxico, la dicloxacilina (250mg a 1 gr en adultos y de 50-400mg/kg/da en nios dividido en 4 dosis), es
uno de los frmacos de primera eleccin para el tratamiento de estafilococos. Una alternativa son las cefalosporinas de primera generacin (cefalozina,
cefalotina), segunda generacin (cefuroxima). En infecciones severas se utiliza la terapia combinada, generalmente con un aminoglucosido para sinergia.
Los antibiticos combinados con un inhibidor de beta-lactamasa (ac. Clavulanico, sulbactam, tazobactam), tambin pueden utilizarse en el tratamiento
para infecciones por estafilococo sensible a meticilina, pero no son la primera eleccin. La vancomicina no debe utilizarse en forma rutinaria, porque es
caro, toxico y requiere una estrecha vigilancia por efectos secundarios, lo ms importante es que favorece la aparicin de cepas resistentes. A pesar de
los reportes de resistencia a meticilina, en infecciones comunitarias por S. aureus, donde se sospeche que la resistencia es elevada, debern emplearse
otras alternativas como clindamicina o trimetropim/sulfametoxaxol. Es indispensable, que se tomen cultivos antes del inicio de tratamiento, para
efectuar modificaciones de acuerdo a los resultados de sensibilidad y la respuesta clnica del paciente. Cuando el paciente tiene una infeccin severa y
sntomas de sepsis, lo fundamental es reconocer la causa y quitar los factores precipitantes, el apoyo en un servicio de terapia intensiva y el tratamiento
emprico con antibiticos de acuerdo a la sospecha clnica. En el Sndrome de choque toxico est recomendado: dicloxacilina + aminoglucocido,
cefalotina + aminoglucocido, aminoglucocido + glicopeptidos (amikacina-vancomicina), linezolid. En infecciones de SNC, se ha utilizado tanto el
tratamiento sistmico como el intraventricular, la limitacin es la penetracin adecuada del frmaco al SNC, los medicamentos habitualmente utilizados
son dicloxacilina, oxacilina y con vancomicina los niveles no son ptimos. Para tratamiento intraventricular se ha utilizado gentamicina o amikacina.
CASO CLINICO
Paciente varn de 11 aos de edad, procedente de Chancay,
readmitido despus de 19 das de ser sometido a ciruga electiva por
un quiste seo aneurismtico en el fmur izquierdo, realizndole
curetaje seo e injerto autlogo de cresta iliaca. Fue dado de alta al
sptimo da. Posterior al alta, present fiebre no cuantificada y
malestar general; persistiendo febril los siguientes doce das, motivo
de su readmisin en el servicio de Traumatologa. No tena
antecedentes importantes. El nio luca en buen estado general,
presentando fiebre entre 38 y 38.5 C. Al examen se evidenciaba
discreto aumento de volumen y dolor a nivel sacro y glteo izquierdo,
sin otros signos de flogosis.Las heridas operatorias habian cicatrizado.
El resto del examen fsico era normal. El hemograma presentaba
10,200 leucocitos, sin desviacin izquierda. No tuvo estudio de VSG ni
proteina C reactiva. El examen ecogrfico de partes blandas a nivel
sacro-glteo izquierdo revelaba distorsin de las partes blandas en los
planos subcutneos y musculares adyacentes a la herida operatoria,
sin evidenciar colecciones. En la tomografa pelvica se evidenci
imgenes compatibles con abscesos en partes blandas en pelvis
izquierda. No se procedi a puncin ni drenaje. Cont con solo un
cultivo, el cual fue positivo a S. epidermidis
PREGUNTA
Cual es la conducta teraputica para el agente etiolgico aislado?
RESPUESTA
a.- Cefalotina.
b.- Dicloxacilina.
c.- Ciprofloxacina.
d.- Tazobactam.

CURSO ENARM CMN SIGLO XXI TEL: 36246001

CASO CLINICO
Femenino de 63 aos, inmunocompetente, que acude por aparicin de
un bulto en la cara interna del tercio superior del muslo derecho. En la
TAC se observaron extensas colecciones de contenido lquido, desde la
pelvis hasta el compartimento intramuscular de los msculos psoasilaco derecho y obturador derecho. A travs de la regin inguinal
derecha, estas se extendan hasta el tercio medio del muslo derecho;
las colecciones de mayor tamao se localizaban en el muslo y se
asociaban a reaccin peristica femoral, y llegaron a medir hasta 11
cm de dimetro mayor. Por cultivo del material de drenaje se
diagnostic una infeccin por Staphylococcus aureus resistente a la
meticilina (SARM).
PREGUNTA
Cul es la condcuta teraputica mas adecuada a seguir?
RESPUESTA
a.- Rifampicina y minociclina.
b.- Vancomicina y gentamicina.
c.- Ceftriaxiona y amikacina.
d.- Dicloxacilina y vancomicina.
CASO CLINICO
Se trata de un paciente de 79 aos con enfermedad renal crnica
estadio 5 secundaria a nefroangioesclerosis y/o nefropata diabtica,
en terapia renal sustitutiva con dilisis peritoneal continua
ambulatoria. Como otros antecedentes presenta diabetes mellitus
tipo 2 de larga evolucin, hipertensin arterial y anemia perniciosa. A
los 27 meses del inicio de la tcnica, el paciente acude a urgencias con

Pharmed Solutions Institute

PGINA 100

MANUAL DE TRABAJO DEL CURSO ENARM CMN SIGLO XXI


clnica y recuento de clulas en el lquido de drenaje, compatibles con
una peritonitis. Se inicia el protocolo emprico que incluye vancomicina
y ceftazidima, Se enva muestras para cultivo. Se presentan
posteriormente 3 episodios de peritonitis con un intervalo de 15 dias
minimo entre cada cuadro.
PREGUNTA
Cual es la conducta mas adecuada a seguir?
RESPUESTA
a.- Cambiar de tipo de catter.
b.- Vancomicina intraperitoneal.
c.- Daptomicina IV.
d.- Cambiar a hemodilisis.
CASO CLINICO
Paciente varn de 44 aos, con infeccin por virus de
inmunodeficiencia humano (VIH) diagnosticada en marzo de 2005, sin
terapia antirretroviral (linfocitos T CD4/+n 286/mm3, carga viral
89.000 copias/mi). Semanas previas a la consulta realiz viaje a
Uruguay y mantuvo contactos sexuales con ms de una pareja. Una
semana antes del ingreso comenz con aumento de volumen labial
diagnosticndose infeccin por virus Herpes simples, indicndose
aciclovir 400 mg/12 h vo. Al da siguiente, se agreg ciprofloxacino 500
mg c/12 h por progresin de lesin. Evolucion con fiebre 38C, tos y
dolor torcico. Radiografa de trax revel neumona intersticial
bilateral indicndose hospitalizacin, la que el paciente rechaz. Se
indic levofloxacino 500 mg/da vo. A las 48 h consult en por
agravamiento de su cuadro clnico y fue hospitalizado. Ingres febril
38C, presin arterial 120/70 y taquicardia de 110x'. Al examen facial
presentaba aumento de volumen flegmonoso del labio superior y en
examen pulmonar crpitos bsales bilaterales. Hemograma con
hematocrito 40%, leucocitos 14.300/mm3, baciliformes 4%, VHS 69 y
PCR 24,7 mg/dL. Radiografa y tomografa axial computarizada (TAC)
de trax mostraron pleuroneumona bilateral. Puncin pleural dio
lquido con pH 7,4, glucosa 5 mg/dL, 26.800 clulas, 61,5% neutrfilos.
En cultivos de secrecin de labio superior y lquido pleural se aisl
Staphylococcus aureus resistente a B-lactmicos, ciprofloxacino,
eritromicina y resistencia inducible a clindamicina.
PREGUNTA
Cual es la conducta teraputica mas adecuada para el caso?
RESPUESTA
a.- Vancomicina y rifampicina.
b.- Vancomicina y gentamicina.
c.- Ceftriaxiona y amikacina.
d.- Dicloxacilina y vancomicina.
CASO CLINICO
Paciente varn de 23 aos, estando en chiapas sufri picadura de
insecto en tobillo derecho. En las horas siguientes, desarroll aumento
de volumen, dolor y eritema en la zona, asociado a fiebre. Evaluado
por mdico, se le indic cefadroxilo 500 cada 12 h vo. A las 24 h
present drenaje espontneo de abundante pus de la herida. Cuatro
das ms tarde consult por persistencia de lesin inflamatoria del
tobillo. Al examen de ingreso se constat paciente afebril, presin
arterial 134/70 mmHg, FC 64x'. En tobillo derecho haba aumento de
volumen eritematoso con lcera central. El hemograma mostr
hematocrito 36,8%, leucocitos 6.680 x mm3, baciliformes 3%, VHS 60 y
PCR 6,3 mg/ dL. El cultivo fue positivo para Staphylococcus aureus
resistente a B-lactmicos, sulfametoxazoltrimetropim y ciprofloxacino.
PREGUNTA
Cual es la conducta teraputica a seguir mas adecuada para el caso?
RESPUESTA
a.- Cefazolina y clindamicina intravenosa.
b.- Clindamicina via oral.
c.- Ceftriaxiona y amikacina.
d.- Dicloxacilina y vancomicina.

CURSO ENARM CMN SIGLO XXI TEL: 36246001

CASO CLINICO
Paciente varn de 39 aos, motociclista. Tres meses previo al ingreso,
comenz con furnculos en trax, cuello y extremidades inferiores las
que se trataron con diversos antimicrobianos va oral, con respuesta
parcial y aparicin de nuevas lesiones. El estudio microbiolgico de una
lesin del muslo, identific Staphylococcus aureus resistente a
cloxacilina y sensible a clindamicina, la que recibi va oral, sin
respuesta. Por progresin de lesin en muslo izquierdo con celulitis
extensa. Al examen fsico destacaba lesin pustulosa de 4 cm de
dimetro en muslo izquierdo, rodeada de una zona eritematosa
extensa que comprometa el tercio inferior del muslo. El hemograma
mostr hematocrito 40%, leucocitos 6.600 x mm3, baciliformes 2%,
VHS 32 y PCR 6 mg/dl.
PREGUNTA
Cual es la conducta teraputica a seguir mas adecuada para el caso?
RESPUESTA
a.- Vancomicina y clindamicina intravenosa.
b.- Clindamicina via oral.
c.- Ceftriaxiona y amikacina.
d.- Dicloxacilina y vancomicina.
CASO CLINICO
Se trata de paciente femenino de 31 aos de edad la cual acude a
consulta debido a que presenta disnea grave, desorientada,
confundida con tos productiva y fiebre importante, el familiar refiere
no tener enfermedades, ser prcticamente sana, soltera y abogada de
profesin, agrega que hace una semana presento un cuadro gripal,
caracterizado por fiebre, cefalea, mialgias y malestar generalizado, sin
darle importancia solo toma medidas generales mejorando, dos das
despus presenta ataque al estado generalizado, con dificultad
respiratoria, a la exploracin fsica se presenta confusa, diafortica,
disneica, con temperatura de 39.6 C, pulso de 130 lpm, TA 90/60
mmHg, FR 39 se ingresa y monitoriza con oximetro que reporta 89 %
de saturacin se coloca oxigeno al 100% sin cambios, la auscultacin
pulmonar con sonidos respiratorios anforicos predominante en lbulo
inferior izquierdo, la TAC revela necrosis del lbulo inferior izquierdo.
Se realiza intubacin y soporte de lquidos, se sospecha Staphylococcus
aureus como agente causal.
PREGUNTA
Cual es tratamiento mas adecuado antes del reporte de los cultivos?
RESPUESTA
a.- Doxiciclina.
b.- Meticilina.
c.- Vancomicina.
d.- Trimetroprim/sulfametoxazol.
CASO CLINICO
Se trata de masculino de 34 aos de edad el cual tiene catter para
hemodilisis, actualmente con renopatia terminal, acude a consulta
debido a que presenta fiebre y dolor intenso en la regin baja de la
espalda, a la exploracin fsica se observa ansioso, irritable, diafortico
as como ataque al estado generalizado sin embargo signos vitales
120/80, FC 89, FR 21, temperatura 38.7 C, el sitio donde se encuentra
colocado el catter se observa rojo y caliente, no se observan
secreciones ni olor, la biometra hemtica reporta 16 700 con 13 % de
bandas, se realiza IRM donde se reporta lesiones compatibles con
osteomielitis vertebral.
PREGUNTA
Cual es agente etiolgico ms probable?
RESPUESTA
a.- Streptococcos.
b.- Stafilococcos.
c.- Mycoplasmas.
d.- Tuberculosis.

Pharmed Solutions Institute

PGINA 101

MANUAL DE TRABAJO DEL CURSO ENARM CMN SIGLO XXI


CASO CLINICO
Se trata de masculino de 75 aos de edad el cual cuenta con
antecedentes tales como diabetes mellitus de ms de 25 aos de
evolucin, actualmente acude por segunda ocasin a consulta debido a
que presenta una ulcera del pie diabtico de 2 meses de duracin la
cual no ha respondido a tratamiento previo con metronidazol y
dicloxacilina sin especificar dosis, a la exploracin fsica se observa
lesin con olor ptrido, se trata de una lesin de 2,6 cm con pus, la
cual se drena e identifica lesin que llega al hueso con una
profundidad de 2.5 cm, se realiza cultivo y tincin, sin embargo se
debe iniciar tratamiento, la tincin reporta flora mixta.

CASO CLINICO
Femenino de 23 aos de edad, acude a consulta refiriendo fiebre,
escalofros asi como dolor de espalda y malestar generalizado. Cuenta
con antecedente de trastorno bipolar y abuso de sustancias,
actualmente bajo tratamiento sin embargo con recaidas. A la
exploracin fsica se observa lesiones tipo puncion en pliegue cutneo
en antebrazo por probable inyeccin intravenosa de sustancias, al
parecer de drogas sin aceptarlo la paciente, los signos vitales son 38.5
C de temperatura, FC 110 lpm, FR 24, tensin arterial de 110/60
mmHg, Rx de torax con varios infiltrados nodulares perifricos con
cavitaciones.

PREGUNTA
Cul es el manejo farmacolgico en espera de los resultados del
cultivo.
RESPUESTA
a.- Clindamicina.
b.- Ampicilina/sulbactam.
c.- Metronidazol.
d.- Vancomicina.

PREGUNTA
Cul es la patologa ms grave que la paciente presenta.
RESPUESTA
a.- Neumonia
b.- Endocarditis.
c.- Septicemia.
d.- CID.

INFECCIONES ESTREPTOCOCICAS:
CIENCIAS BSICAS: Son un genero de los ms importantes como patgeno para el ser humano, son bacterias esfrica, erticos facultativos, no forman
esporas, catalana negativos e inmviles. El Streptococcus pyogenes, es el agente etiolgico mas frecuente de faringoamigdalitis aguda, pudiendo
ocasionar secuelas no supurativas, tales como la fiebre reumtica y la glomerulonefritis post- estreptoccicas. Otro el Streptococcus pneumoniae. De
acuerdo con el tipo de hemolisis de eritrocitos que producen, en beta (hemolisis completa), alfa (hemolisis parcial) y gamma (sin hemolisis). La primera la
presentan prcticamente todo el S. pyogenes y la segunda los S. pneumoniae y viridans. SALUD PUBLICA: S. pyogenes incidencia mayor en raza blanca,
en adolescencia y sexo masculino. La faringoamigdalitis ms frecuentes en zonas templadas y fras y en invierno, mientras el imptigo y la piodermitis lo
son en los climas tropicales y en meses calurosos. PATOGENIA: La fuente de infeccin son las secreciones de nariz, garganta y piel de enfermos, la
transmisin se realiza persona a persona, mediante secreciones respiratorias al toser o estornudar o por transferencia manual (piel con traumatismo
previo). Incubacin de 2-4 das, la fase aguda constituye el periodo de mayor contagiosidad. Ingresa a vas respiratorias superiores y se disemina, tiene
especial afinidad por el sistema linftico y el lugar de implantacin es el tejido linfoide de faringe. Se extiende por los linfticos o por continuidad a otras
reas cercanas, causando complicaciones tales como adenitis cervical, absceso periamigdalino o retrofaringeo, sinusitis, otitis. Para el desarrollo de
escarlatina, la infeccin farngea es por una cepa productora de toxina eritrogenica, la cual es antignica y estimula produccin de anticuerpos
protectores (por eso solo da un cuadro). ESTREPTOCOCOS BETA HEMOLITICO DEL GRUPO A o Streptococcus pyogenes: Tiene una estructura celular
compleja, su protena M parece ser su principal factor de virulencia, las cepas ricas en ellas son resistentes a la fagocitosis. Elabora productos
extracelulares; la toxina eritrogenica es la responsable del exantema en la escarlatina, su produccin es inducida por un bacteriofago. La estreptomicina
O y la S ambas txicas para los eritrocitos y otras clulas, incluso miocardio. La elevacin de niveles serios de anti estreptomicina O. Es un buen indicador
de infeccin estreptoccica reciente. Por evidencia seroepidemiologica, se considera que ciertos serotipos, causantes de faringoamigdalitis, estn ms
relacionados con fiebre reumtica, los de piel no causan fiebre reumtica pero ambos son nefritogenicos (glomerulonefritis). DIAGNOSTICO: Clnica;
faringoamigdalitis, forma txica con fiebre alta, nausea y vmitos, comienzo agudo, odinofagia, cefalea, malestar, faringe hiperemica, exudado blancoamarillento en parches, adenopatas anterior dolorosa. Escarlatina; fiebre elevada, vomito, dolor farngeo y cefalea, 12-48 hrs despus aparece exantema
y el enantema (lengua=fresa roja, paladar, amgdalas y faringe) caracterstico. El rash se inicia en el tronco y se generaliza con rapidez, se presenta con
una erupcin con fondo eritematosa y a la vez puntico ene, con seccin tctil de aspereza (papel de lija), respetando la cara. Siempre se presenta la
descamacin en mayor o menor grado. Imptigo; vescula superficial con poco eritema e indolora, progresiva pstula con costra gruesa, con secrecin
meliserica, mas en extremidades y comisuras nasales y bucales. Erisipela; poco frecuente, celulitis caracterizada por una lesin eritematosa, con
engrosamiento de la piel, dolorosa y con aumento de temperatura, borde bien definido. Otras infecciones son; piodermitis, ectima, celulitis o fscitis
necrozante. El diagnostico de faringoamigdalitis se fundamenta en la triada amgdalas con exudado, odinofagia y adenopatias cervical anterior dolorosa,
en un paciente que NO presenta tos, o sntomas nasales. El dx., de certeza aislamiento de S. pyogenes en cultivo de exudado farngeo, aglutinacin en
ltex y coaglutinacion, detectan antgeno. En la escarlatina el rash, y se puede hacer cultivo y antiestreptolisinas. TRATAMIENTO: Debido a que S.
pyogenes sigue siendo sensible a penicilina, esta es el tx., de eleccin. Nios menores de 4 aos; penicilina procainica 400,000UI c/24 hrs 2-3 das + p.
Benzatinica 600,000UI, en una sola dosis, despus de la procainica o penicilina benzatinica 600,000UI dosis nica. Nios mayores de 4 aos y adultos;
penicilina procainica 800, 000UI c/24hrs 2-3 das + P. Benzatinica 1,200,000UI dosis nica despus de la procainica o penicilina benzatinica 1,200,000UI
dosis nica. El medicamento sustituto en alergia a penicilinas es eritromicina oral 30mg/kg/da, en dos o tres dosis diarias durante 10 das (dosis mxima
2 gramos). En caso de miosotis y fascitis necrozante, se recomienda penicilina, clindamicina y amikacina, hasta corroborar agente, si es pyogenes
continuar solo penicilina y clindamicina.
CASO CLINICO
Varn con 43 aos de edad, diabtico sin control, que 24 horas antes
present una contusin accidental contra el respaldo de la cama en la
zona periorbitaria derecha, por lo que acudi a consultorio, donde se
le realiz una curacin y prescribi tratamiento con anti-inflamatorios
(diclofenaco sdico). En las horas siguientes curs con fiebre,
compromiso de conciencia y vmitos. Al examen fsico se describi un
paciente de aspecto txico, con los siguientes signos vitales: frecuencia
cardaca: 130/min, frecuencia respiratoria: 49/min, presin arterial:
77/55 mm Hg, T rectal: 37,8 C, escala de Glasgow neurolgico: 14.

CURSO ENARM CMN SIGLO XXI TEL: 36246001

Destacaba un gran edema periorbitario bilateral con eritema y calor en


la zona palpebral y en la mejilla derecha. Se hospitaliz con el
diagnstico de celulitis preseptal, inicindose tratamiento con
dicloxacilina endovenosa.
PREGUNTA
Cual de los siguientes agentes infecciosos es mas probable aisla en
este caso?
RESPUESTA
a.- Estreptococos del grupo A

Pharmed Solutions Institute

PGINA 102

MANUAL DE TRABAJO DEL CURSO ENARM CMN SIGLO XXI


b.- Estreptococos del grupo B
c.- Estreptococos del grupo C
d.- Estreptococos del grupo D
CASO CLINICO
Varn de 62 aos, sin antecedentes clnicos de inters, que ingres por
presentar en las 48-72 h previas un cuadro de fiebre elevada,
sudoracin profusa, escalofros, mialgias y posteriormente cefalea
frontooccipital intensa, postracin y hematuria macroscpica. No
presentaba sntomas gastrointestinales, infecciosos, traumatismos ni
intervenciones quirrgicas previas. En la exploracin fsica destacaba
rigidez de nuca y afectacin del estado general. No se encontraron
signos de afectacin cardiovascular ni digestiva. En los estudios
analticos destacan la presencia de leucocitosis con desviacin
izquierda, inicio de patrn analtico de coagulacin intravascular
diseminada (CID) y leucocituria con nitritos positivos en orina. Se
realiz puncin lumbar, obtenindose lquido claro a presin normal
cuyo examen mostr: leucocitos 1.200 cl./ml, protenas 8 g/l, glucosa
9 mg/dl, Gram: se identificaron cocos grampositivos en cuyo cultivo se
aisl S. bovis biotipo II
PREGUNTA
Cual de los siguientes grupos agentes infecciosos es mas probable
aisla en este caso?
RESPUESTA
a.- Estreptococos del grupo A
b.- Estreptococos del grupo B
c.- Estreptococos del grupo C
d.- Estreptococos del grupo D
CASO CLINICO
Mujer de 59 aos con cuadro de 72 horas de evolucin con astenia,
anorexia y deterioro del nivel de conciencia que se acompaaba en las
ltimas 24 horas de disnea. Se observo con insuficiencia respiratoria
parcial y una fibrilacin auricular con respuesta ventricular rpida e
inestabilidad hemodinmica secundaria motivo por el que se decidi el
traslado a la Unidad de Cuidados Intensivos (UCI). Ingresa a UCI
obnubilada, hipotensa (tensin arterial: 90/60 mmHg), en fibrilacin
auricular a 130 lpm, taquipnica a 30 rpm y con saturacin arterial de
oxigeno de 90% segn pulsioximetra respirando con mascarilla al 50%
de fraccin inspirada de O2. En la exploracin fsica destac
nicamente la existencia de ingurgitacin yugular bilateral e
hipoventilacin en la base pulmonar izquierda. A nivel analtico llam
la atencin la existencia de leucocitosis con 15.300 leucocitos,
neutrofilia y una importante desviacin izquierda. Electrocardiograma
con fibrilacin auricular rpida sin alteraciones de la repolarizacin.

Radiografa de trax con cardiomegalia y borramiento de ambos


hemidiafragmas. Se canaliz un catter venoso central y un catter
PICCO, objetivndose un patrn hemodinmico compatible con sepsis
(gasto cardiaco elevado con resistencias vasculares sistmicas
disminuidas) y unos valores de presin venosa central elevados.
PREGUNTA
Cual de los siguientes grupos agentes infecciosos es mas probable
aisla en este caso?
RESPUESTA
a.- Estreptococos del grupo A
b.- Estreptococos del grupo B
c.- Estreptococos del grupo C
d.- Estreptococos no A no B.
CASO CLINICO
Paciente de 35 aos, sexo femenino, no fumadora, fue sometida a
tiroidectoma total por neoplasia folicular de tiroides y fue dada de alta
en buenas condiciones generales a las 48 h. Consult 5 dias despues
por malestar general, odinofagia, cefalea, sensacin febril, escalofros,
dolor cervical, tos productiva y expectoracin mucopurulenta, siendo
admitida a sala de cuidados generales. En el examen fsico destacaba
paciente en regulares condiciones generales, lcida, orientada, FC: 90
lat/min, PA: 106/60 mmHg, FR: 24 resp/min, T: 37,8C. Dolor a la
palpacin en la zona de herida operatoria de la regin cervical
anterior, faringe sana, examen cardiopulmonar y abdominal normales.
Se plante el diagnstico de infeccin de herida operatoria y se inici
tratamiento con ceftriaxona 1 g/da EV. La paciente evolucion febril y
con dificultad respiratoria progresiva, requiriendo aporte de oxgeno al
35% para corregir el trastorno del intercambio gaseoso (Pa02/Fi02:
259). En el hemograma destacaba leucopenia (2.700/mm3) con
desviacin a izquierda (13% baciliformes), PCR: 5,4 mg/dL, GSA: pH:
7,46, Pa02: 54,4 mmHg, Sa02: 89%, PaC02: 27,5 mmHg, B. act: 19,6
mEq/L. La radiografa de trax mostr opacidades parenqui-matosas
en lbulo superior derecho, lbulo medio, lngula y lbulo inferior
izquierdo.
PREGUNTA
Cual de los siguientes grupos agentes infecciosos es mas probable
aisla en este caso?
RESPUESTA
a.- Estreptococos del grupo A
b.- Estreptococos del grupo B
c.- Estreptococos del grupo C
d.- Estreptococos no A no B.

INFLUENZA (HAEMOPHILUS INFLUENZAE):


CIENCIAS BASICAS: Infeccin contagiosa de origen viral, considerada una de las causas ms importantes de infecciones de vas respiratorias. La
relevancia de esta infeccin radica en su frecuente asociacin a complicaciones y muerte en poblaciones susceptibles. Este grupo de virus es capaz de
asociarse a epidemias y pandemias con consecuencias devastadoras. El virus de la influenza puede afectar la mucosa nasal, faringe, bronquios y a veces
hasta los alveolos pulmonares. Los sntomas son parecidos a los del catarro comn o resfriado, sin embargo son ms severos y su inicio es generalmente
abrupto. El Haemophilus influenzae es un pequeo gramm negativo, ARN, cocobacilo, pleomrfico. La influenza es ocasionada por 3 virus (influenza
A,B,C). Las cepas con una cpsula de polisacrido son ms importante clnicamente, causan la enfermedad invasiva sistmica, principalmente en bebs y
nios <6 aos de edad. Cepas no tipificables de H. influenzae, que estn no encapsulado, provocan la enfermedad mediante invasin local de superficies
mucosas. SALUD PBLICA: La enfermedad generalmente requiere atencin mdica y hospitalizacin, contribuyendo sustancialmente a prdidas
econmicas y exceso en nmero de hospitalizaciones y muerte. Por lo menos 4 pandemias han afectado a la humanidad en el siglo XIX, y 3 en el XX. El
virus de la influenza A se caracteriza por causar enfermedad moderada a grave; tiende a afectar a todos los grupos etarios y tiene la caracterstica de
afectar tanto aves como cerdos. El virus B de la influenza cuadros clnicos menos graves, y tradicionalmente produce infeccin durante la infancia. La
capacidad del virus de la influenza A y B de sufrir cambios antignicos graduales en sus dos antgenos de superficie: la hemaglutinina y la neuraminidasa,
complica la vacunacin contra esta enfermedad. Afecta a todos los grupos etarios, pero principalmente a <2aos y >65aos. Ocurre principalmente
durante los meses de invierno y principios de la primavera. En Mxico la neumona e influenza representan una de las mayores causas de morbilidad y
mortalidad. En >65 aos de edad, la neumona e influenza se reportan como la sptima causa de mortalidad. PATOGENIA: Los virus de influenza se
diseminan de persona a persona a travs de los actos de toser o estornudar de personas infectadas o enfermas con los virus de influenza, gotitas en el
aire o fmites, el periodo de incubacin es de 1-4 das, la patognesis de la replicacin del virus y su relacin al desarrollo de manifestaciones clnicas no
ha sido completamente descifrada, no obstante la replicacin viral ocurre primariamente en el epitelio columnar del aparato respiratorio, pero puede
ocurrir en cualquier segmento a lo largo del tracto respiratorio. La regeneracin del epitelio columnar toma de 3-4 semanas, tiempo en el cual las
manifestaciones pulmonares pueden persistir. DIAGNOSTICO: Clnico; Inicio agudo de fiebre, mialgia, cefalea, debilidad extrema, tos no productiva,

CURSO ENARM CMN SIGLO XXI TEL: 36246001

Pharmed Solutions Institute

PGINA 103

MANUAL DE TRABAJO DEL CURSO ENARM CMN SIGLO XXI


odinofagia y rinitis, estos sntomas ocurren en 50-70% de las infecciones. Puede haber dolor retroesternal, fotofobia, dolor abdominal y diarrea, en
ancianos puede no haber fiebre solo delirio y postracin, en nios puede asociarse a otitis media y traqueobronquitis. Podemos encontrar linfadenopata
cervical. Las definiciones clnicas para diagnosticar influenza varan de 63-78% de sensibilidad y de 55-71% de especificidad. Laboratorio: Aislamiento del
virus en muestras de exudado farngeo o nasofarngeo obtenido dentro de los primeros 3 das del inicio de sntomas. Se considera al cultivo viral (permite
tipificar y caracterizarlo antignicamente), como el estndar de oro seguido de conformacin por inhibicin de la hemaglutinacin. Las tcnicas
serolgicas mas recuente usadas son la fijacin de complemento y la inhibicin por hemaglutinacin. COMPLICACIONES: Neumona primaria por
influenza o neumonas secundarias bacterianas (S. pneumoniae, S. aureus), sinusitis bacteriana, bronquitis, traqueobronquitis y otitis media. Puede
ocurrir muerte en 0.5-1 en 1,000 casos. Otras; convulsiones, encefalopata por el virus, o encefalopata asociada a la utilizacin de salicilatos (sndrome
de Reye= encefalopata aguda acompaada de degeneracin grasa visceral), miositis, miocarditis y pericarditis. Puede haber diversas manifestaciones a
nivel de SNC, incluyendo mielitis transversa y Sx. de Guillan Barr. Meningitis que se asocia con una alta morbilidad, 6 % de los pacientes tienen una
prdida auditiva neurosensorial, una cuarta parte tienen algunas secuelas importantes, la mortalidad es de 5 %. Epiglotitis, que ocurre en nios mayores
y en ocasiones en adultos, implica la celulitis de la epiglotis y tejidos supraglticos que comienza con dolor de garganta y progresa rpidamente a la
disfagia, salivacin, y obstruccin de va area. TRATAMIENTO: Reposo en casa, analgsico o antiinflamatorios que no sean de la familia de los salicilatos,
no utilizar antibiticos profilcticamente. Los derivados de amantadino; amantadina (>10 aos 100mgs c/12 hrs >65 100mg c/24 hrs) y rimantadina (>10
aos 100mgs c/12 hrs >65 100mg c/24 hrs) y los inhibidores de la enzima neuraminidasa; oseltamivir (>13 aos 75mg c/12 hrs) y zanamivir (>10 aos de
edad 10mg c/12 hrs), los cuatro han demostrado utilidad clnica reduciendo la duracin de los sntomas. Meningitis en adultos por H. influenzae tipo B:
ceftriaxona (2 g cada 12 h durante 1-2 semanas). Meningitis por H. influenzae tipo B en nios: ceftriaxona (75-100 mg / kg por da, divididos en dos dosis
cada 12 h) y dexametasona (0,6 mg / kg por da en cuatro dosis divididas durante 2 das a la iniciacin del tratamiento con antibiticos para prevenir
prdida de la audicin). La epiglotitis: ceftriaxona (50 mg / kg al da durante 1-2 semanas). PREVENCION: Vacunacin anual de personas en grupos de
alto riesgo, utilizando la vacuna inactivada trivalente por su eficacia y baja reactogenicidad, los grupos son los siguientes; vacunacin a personal de
salud, personal que labora en asilos de ancianos, personas que viven en contacto intradomiciliario con personas con alto riesgo de desarrollar
complicaciones, mujeres con embarazo de alto riesgo (2do y 3er trimestre), personas mayores de 65 aos, personas entre 50-64 aos (enfermedades
cronicodegenerativas), pacientes de cualquier edad con padecimientos crnicos (principalmente respiratorios y cardiovasculares), vacunacin de nios
de 6-59 meses y viajeros (que van a los trpicos o al Hemisferio Sur). La vacuna se administra por va intramuscular, en regin deltoidea (nios regin
anterolateral de muslo). Contraindicada en alergia al huevo o a otros componentes de la vacuna o cuadro febril agudo.
CASO CLINICO
Femenino de 46 aos con antecedentes de asma bronquial tipo III,
para lo cual lleva tratamiento con salbutamol (spray). Acudi por
presentar fiebre de 39 0C y trastornos de la conciencia. Cinco das
antes haba presentado una crisis asmtica acompaada de un cuadro
febril asociado con sntomas respiratorios altos, que coincidi con un
cuadro respiratorio alto en su nieto pequeo atendido por ella; por lo
anterior fue vista en su rea de salud donde le indicaron tratamiento
con penicilina. El da antes present vmitos, en nmero de 12, de tipo
bilioso y acompaados de nuseas, con mucha tos y abundante
secrecin nasal verdosa y fiebre. En el momento de ser valorada en el
servicio de urgencia llamaba la atencin los trastornos de conciencia
que presentaba esta paciente, dados por la agitacin psicomotora y el
estado de estupor. Al examen fsico se constat rigidez nucal y signos
de irritacin menngea. Se le realiz puncin lumbar (PL) de donde se
obtuvo un lquido cefalorraqudeo (LCR) de aspecto turbio, as como
otros exmenes complementarios, incluidos hemogramas seriados. El

examen citoqumico del LCR mostr los valores siguientes: 435


leucocitos/mm3 a predominio de polimorfonucleares (PMN), hemates
8/mm3, glucosa en 1,0 mmol/L y protenas totales en 0,95 g /L. En el
examen bacteriolgico del lquido se inform pleomorfismo bacteriano
gramnegativo. En el leucograma se constat una cifra de 16 000
leucocitos con predominio de PMN. El cultivo reporto haemophilus
influenzae en el LCR.
PREGUNTA
Cual es la conducta mas apropiada a seguir con el diagnostico
presuntivo del caso?
RESPUESTA
a.- Rivavirina y amantadina.
b.- Ceftriaxona va endovenosa
c.- Ceftriaxona y amikacina IV.
d.- Cloranfenicol y ampicilina.

SINDROME DE INMUNODEDICIENCIA ADQUIRIDA (SIDA); VIRUS DE LA INMUNODEFICIENCIA HUMANA (VIH)


CIENCIAS BASICAS: SIDA; Alteraciones inmunitarias profundas, infecciones bacterianas recurrentes o por grmenes oportunistas y a aparicin de formas
comunes de neoplasias malignas, causada por VIH -1 (Amrica, Europa, Caribe), VIH-2 (oeste de frica). El VIH pertenece a la familia de retrovirus
(poseen enzima transcriptasa reversa) citopaticos, no transformantes denominados lentivirus. Las regiones codificadores del VIH son los genes:
gag=poliproteina precursora, que es escindida para protenas de la capside, pol=transcripatasa inversa, integrasa y proteasa, env=protenas de la
cubierta. SALUD PUBLICA: Los primeros casos de SIDA, fueron reconocidos en los Angeles en 1981. Segn el CENSIDA, los casos acumulaos suman, hasta
junio del 2007, 112,830, de los cuales 2,720, ocurren en el grupo de los menores de 15 aos. Proporcin hombre: mujer 6:1, en el ao 2003, para el
grupo ms afectado (25-39 aos). El riesgo de transmisin vertical del VIH-1 de una madre infectada al producto vara entre 11-50%. La transmisin
sexual en hombres aun es predominantemente homo-bisexual. PATOGENIA: La infeccin por VIH, puede adquirirse por transmisin sexual (80-90%)
exposicin parenteral a sangre o derivados (por agujas en usuarios de drogas, o trabajado a de la salud), y de madres infectadas a sus productos,
durante el perodo perinatal. Rutas menos frecuente a travs de trasplante de rganos, tejidos y semen contaminado. El riesgo de adquirir infeccin
depende del tipo de exposicin (transfusin de un donador seropositivo es de 100%, exposicin nica a aguja contaminada es de 0.4%). La transmisin
del VIH de la madre al nio adems de in tero, puede ocurrir intraparto, por exposicin del producto a la sangre materna o las secreciones genitales
infectadas, esta transmisin se puede evitar mediante estrategias de profilaxis con antiretrovirales durante el embarazo y parto as como mediante la
realizacin de cesrea programa en mujeres infectadas con VIH-1, con carga viral de >1,000 copias, a la semana 38 de gestacin antes de que se haya
desencadenado el TDP y de que se presente ruptura de membranas. Existe un riego adicional por lactancia materna del 14% en casos de infeccin
establecida y 29% de infeccin primaria. No hay evidencia que el VIH se pueda transmitir entre los miembros de una familia, a travs de contacto
estrecho o por compartir utensilios. El VIH se une a la clula blanco (tiene tropismo por los T CD4), que tiene un receptor de alta afinidad de la
glicoproteina de la envoltura gp120 del virus, lo que permite su unin, una vez unido se produce la fusin de su envoltura externa del virus a la
membrana celular a travs del gp41 con lo que el VIH entra a la clula, pierde su envoltura proteica, libera su ARN y las protenas codificadas por la
regin Pol, como la transcriptasa inversa, en el citoplasma. Esta enzima dirige la construccin de una cadena de ADN sobre la plantilla de ARN viral, y en
segundo paso cataliza la copia de la cadena recin sintetizada para producir un ADN de cadena doble (proviral). Este ADN proviral se integra en el ADN
cromosmico de la clula husped. Despus de su integracin, el ADN proviral del VIH pasa a un estado productivo, en el cual el ADN proviral se
transcribe en ARN viral y ARN mensajero, el cual codifica la si resist de protenas virales, necesarias para la replicacin viral, mediante el uso de las

CURSO ENARM CMN SIGLO XXI TEL: 36246001

Pharmed Solutions Institute

PGINA 104

MANUAL DE TRABAJO DEL CURSO ENARM CMN SIGLO XXI


funciones metablicas de la clula husped. Los estadios finales comprenden el proceso de ensamblaje o encapsulacin viral y la protrusin en la
superficie celular; en esta etapa el virus adquiere una envoltura lipidica, siendo finalmente liberado para infectar a otras clulas y repetir el ciclo. Durante
la fase asintomtica de VIH (Clnica latente), los niveles circulantes del virus y las cuentas de CD4 permanecen esencialmente constantes (equilibrio); el
grado de infeccin de las clulas es muy semejante al grado de muerte de las clulas infectadas. Con la replicacin viral, la clula experimenta en pocos
das degeneraciones balninformes (tumefaccin celular por alteracin
Sistema de clasificacin para la infeccin por VIH/SIDA revisada en 1993. (categora clnica)
en la permeabilidad de la membrana), lo que conduce finalmente a la
Nmero de clulas CD4
A
B
C
> = 500/mm3
A1
B1
C1
muerte
200-499/mm3
A2
B2
C2
celular. Las clulas infectadas pueden volverse ms susceptibles a la
<200/mm3
A3
B3
C3
sper infeccin por otros patgenos, lo cual puede llevar ms
rpidamente a la deplecin de clulas T, que son las clulas crticas de la activacin de la respuesta inmunitaria especifica, tanto celular como humoral.
Esta inmunosupresin adquirida, explica la gran susceptibilidad a presentar infecciones por grmenes poco frecuentes, como neoplasias malignas. No
solo los linfocitos CD4 se afectan, tambin monocitos y macrfagos pueden ser infectados por el VIH. La afeccin de los macrfagos alveolares, explica la
susceptibilidad a neumona por Pneumocystis jiroveci. La alteracin funcional de las clulas fagociticas tambin compromete a los neutrfilos PMN, lo
cual explica la susceptibilidad a grmenes piogenos. Tambin hay alteracin en linfocitos B, las cuales consisten en activacin policlonal de estas clulas,
por lo que se presenta hipergammaglobulinemia haciendo susceptible a grmenes encapsulados como Streptococcus pneumoniae y Haemophilus
influenzas. DIAGNOSTICO: Clnico: La enfermedad de SIDA ocurre cuando el husped infectado, ya con graves alteraciones en sus mecanismos de
defensa, no puede controlar microorganismos oportunistas o neoplasias malignas que rara vez causan enfermedad en el individuo inmunocompetente.
Las manifestaciones especificas ms frecuentes son: infeccin primaria=sndrome retroviral agudo, dermatolgicamente eritema maculopapular y
ulceracin mucocutanea, adems, fiebre, faringitis, linfadenopata, artralgias, mialgias, anorexia, nusea, prdida de peso, diarrea intermitente. A nivel
neurolgico puede haber; Encefalopata; cuando
ESTADIOS CLNICOS DE LA INFECCIN POR EL VIH POR LA CDC 1993
VIH llega a SNC, en la fase temprana hay
ESTADIO A
ESTADIO B: Denotan progresin de
ESTADIO C: Define un caso de SIDA por medio de las entidades
disminucin de la capacidad de concentracin,
la enfermedad, pero que por s
clnicas asociadas que se presentan
perdida de la memoria y lentitud mental,
solas no clasifican al paciente
hipertrofia, ataxia, temblores, en la fase tarda
como enfermo de SIDA
Infeccin
o
Sintomtico (sin condicin A ni C)
Candidiasis de bronquios, trquea o pulmones
incrementan todos los sntomas y signos, puede
Sndrome
Candidiasis orofarngea o vaginal,
Candidiasis esofgica
haber convulsiones y psicosis, hay datos de
retroviral
por ms de un mes o que
Cncer cervical invasivo
extrema atrofia cerebral y son comn es los
agudo (SRA)
responde
pobremente
al
Coccidiodomicosis diseminada
cambios en la sustancia blanca. La incidencia de
Infeccin
tratamiento
Criptococosis crnica intestinal (>1mes)
gastroenteritis es hasta de 77% cursan con diarrea
asintomtica
Leucoplasia vellosa de la lengua
CMV en cualquier rgano (excepto hgado, bazo o ganglios)
Linfadenopata
Displasia cervical severa o
Encefalopata asociada a VIH
(crnica o intermitente) en algn momento, la
generalizada
carcinoma in situ
Herpes simplex, causando ulceraciones crnicas por ms de un
causa ms frecuente es Cryptosporidium. La
persistente
Enfermedad plvica inflamatoria
mes
infeccin por mycobacterium tuberculosis es la
(LGP)
Herpes zoster en ms de una
Histoplasmosis diseminada
mayor complicacin oportunista que afecta a los
dermatoma o por ms de 2
Isosporosis crnica por ms de un mes
episodios
Sarcoma de Kaposi
adultos con VIH, con cualquier recuento de CD4 y
Purpura
trombocitopenica
Linfoma de Burkitt, inmunoblastico o linfoma primario de
puede incrementar el riesgo de complicaciones y
idioptica
cerebro
mortalidad asociada a VIH. Muchas de las
Neuropata perifrica
Mycobacterium tuberculosis pulmonar o extrapulmonar
infecciones oportunistas en el adulto son
Listeriosis
Mycobacterium avium intracellulare
Angiomatosis bacilar
Otras micobacterias diseminadas
secundarias a reactivacin de patgenos
Sndromes
constitucionales
Neumonia por P. jiroveci
oportunistas como: Mycobacterium avium,
(diarrea por mas de 1 mes, fiebre
Leucoencefalopatia multifocal progresiva
candida, criptococcus (meningitis con CD4 < 50
prolongada
de
origen
Septicemia recurrente por Salmonella
cels/mcl),
herpes
simple,
toxoplasma,
desconocido)
Toxoplasmosis cerebral
histoplasma, CMV (coriorretinitis, perdida de
Sndrome de emaciacin con prdida de peso de >4.5 kg o
ms de 10% del peso usual.
visin, infiltrados amarillentos y/o hemorragias).
Toxoplasma (causa mas comn de dficit focal del SNC, cefalea confusin, < 50cels/mcl, TAC lesiones multiples en anillos, tx. Primetamina+ sulfadiazina+
leucovorin por 6 sem), candida albicans (candidiasis esofagiga definitoria de SIDA, tx. Fluconazol 14-21 dias) adquiridos previamente a la infeccin por
VIH. De acuerdo al INDRE, las infecciones oportunistas ms frecuentes en pacientes con SIDA en Mxico son: 1. Cndidiosis (37.6%) 2. Neumona por
Pneumocystis jiroveci (12.2%). Tuberculosis (11.1%) herpes simple (7.4%), y Herpes zoster (7.2%). Otras manifestaciones caractersticas son las
neoplasias malignas. El Sarcoma de kaposi (neoplasia mas comn en VIH, ndulos, placas, alargamiento de linfonodos), se observa principalmente en
adultos, siendo su presentacin poco frecuente en nios. Dentro de este grupo se incluyen las neoplasias del SNC, como los linfoms no Hodking del
cerebro (200 veces ms comn en pacientes con VIH, origen de cel. B, sntomas fiebre, diaforesis, perdida de peso). Laboratorio: la demostracin de
anticuerpos anti-VIH puede realizarse por varios mtodos, como ELISA ms utilizado, sensibilidad 95%, especificidad 99%, en las primeras 8-12 semanas
puede dar negativo , de ah que si ELISA sale positiva, se recomienda realizar una segunda prueba ms especifica (prueba confirmatoria) y Western el
cual reacciona con los antgenos virales, esta prueba tiene especificidad del 100%. Se considera como persona infectada por VIH o serio positiva aquella
que presente: a) dos resultados positivos de pruebas de Tamizaje positivos (ELISA, aglutinacin o pruebas rpidas) y pruebas confirmatorias de Western
Blot positiva. b) dos resultados. Pruebas positivas de Tamizaje, con cuadro clnico sugerente de infeccin y c) alguna prueba suplementaria positiva
(cultivo, Ag p24, PCR). EN infecciones oportunistas, Diagnostico de SIDA: infeccin declaratoria y CD4 <200/ml. NEUMONA POR Pneumocystis jiroveci:
Inicio insidioso, fiebre, tos no productiva, disnea, hipoxemia, linfadenopata generalizada, diaforesis nocturna. Rx., de trax infiltrados intersticiales
bilaterales, neumtorax espontneos, DHL elevada, CD4 <200cel/ml, lavado bronco alveolar 85-90% sensibilidad, tratamiento de eleccin TMP/SFX
15mg/kg/d 3-4 dosis por 21 das, se ha demostrado que agregar esteroides al manejo incrementa la sobre ida en pacientes con procesos moderadosseveros. Se debe dar profilaxis con cuentas de CD4 menores de 200/mm3, suspenderla si alcanzan >200. CLASIFICACIN: Ver cuadro anexo.
TRATAMIENTO: Los antiretrovirales (ARV) se debe iniciar en adultos asintomticos con linfocitos CD4 <500-l y en aquellos pacientes sintomtico,
independientemente de la cifra de estos, en sndrome de infeccin aguda, en infeccin crnica (sintomtica, o asintomtica con CD4 <350/l o
decreciendo y VIH RNA >50,000 copoias por ml o incrementando), py profilaxis postexposicion. Actualmente se dispone de los siguientes AVR: 1. Inhibido
res de la transcriptasa reversa anlogos de ncleosidos; zidovudina 200mg c/8hrs o 300 c/12hrs, toxicidad anemia, granulocitopenia, miopatia,
didanosina 200 mg c/12hrs, toxicidad pancretitis, neurpatia perifrica, lamivudina, estavudina, abacavir, emitricitabina 2. Inhibidores no ncleosidos
de la transcriptasa reversa: nevirapina (NVP), toxicidad sx. Steven Johnson, sedacin, hepatitis, efavirenz (EFV). Inhibidores de la proteasa; indavir (IDV)
800mg VO c/8hrs, toxicidad sabor metlico, nausea, dolor abdominal, ritonavir (RTV), squinavir, amprenavir, lopinavir. Inhibidores de fusin: efuvirtide
o T-20. EN pacientes con SIDA la fibra es un motivo frecuente de consulta y hospitalizacin, determinar si la infeccin no se debe a bacterias, los agentes

CURSO ENARM CMN SIGLO XXI TEL: 36246001

Pharmed Solutions Institute

PGINA 105

MANUAL DE TRABAJO DEL CURSO ENARM CMN SIGLO XXI


frecuentes del proceso infecccioso pueden ser Streptococcus pneumoniae, Haemophilus, Salmonella, Staphilococcus. Tuberculosis: algunos
medicamentos antifimicos interaccionan con los ARV, en este caso retrasar el inicio de los ARV de 2-8 semnas. Se puede presentar el sndrome de
reconstitucin inmune: 8-43% de pacientes que inician terapia antirretroviral con Tb activa; se manifiesta con fiebre, linfadenopata, empeoramiento de
infiltrados pulmonares, derrame pleural.
CASO CLINICO
Se trata de masculino de 31 aos de edad el cual acude a consulta a
solicitud de su esposa, ya que ha presentado cambios en su estado
cognitivo y alteraciones en sus actividades de la vida diaria, el paciente
cuenta con antecedentes de VIH positivo de diagnostico reciente, la
esposa del paciente ha notado una disminucin significativa de las
actividades de su esposo en la casa, ha sido sancionado en su trabajo,
adems se encuentra con afecto embotado, con disminucin de la
atencin, no logra concentrarse, pierde el nucleo del discurso,
actualmente su cuenta de CD4 es de 380 con una carga vrica de 78
000/ml, el paciente presenta a la exploracin fsica signos vitales sin
alteraciones, el examen neurolgico no muestra datos de focalizacin,
fondo de ojo normal, minimental 22/30, fue enviado a IRM con atrofia
cerebral desproporcionada e inespecfica sin lesiones focales.
PRECUNTA
Considerando el cuadro clnico. Cual es la conducta a seguir ms
apropiada?
RESPUESTA
a.- Terapia antirretrovirica.
b.- PCR para virus JV en LCR.
c.- PCR para micobacteria en LCR.
d.- VDRL para LCR.
CASO CLINICO
Paciente de 30 aos, procedente de la ciudad de pergamino, provincia
de Buenos Aires, que es ingresado por presentar fiebre, astenia, tos
productiva, disnea progresiva y sudoracin nocturna. Heterosexual,
niega consumo de alcohol y drogas ilcitas, conoce su condicin de VIH
positivo desde hace 7 aos. En enero de 2006 se le diagnostica
candidiasis esofgica. Recibi diversos esquemas de terapia
antirretroviral de gran actividad (TARGA) con mala adherencia y
consecuentes fracasos teraputicos.
PREGUNTA
Cul es el agente causal ms probable en este caso?
RESPUESTA
a.- Cryptococcus neoformans.
b.- Aspergillus fumigatus.
c.- Mycoplasma kansasii.
d.- Pneumocystis jiroveci
CASO CLINICO
Paciente de 54 aos, soltero sin conducta sexual de riesgo con
antecedentes de HTA y Giardiasis, haba sido atendido desde hace 6
meses por presentar 6-7 episodios diarios de diarreas lquidas con
flemas, pujos y tenesmo. coproparasitoscopico donde se evidenci
quistes de giardia lambia y se le puso tratamiento con metronidazol
con mejora clnica. A los pocos das reapareci el cuadro diarreico con
toma del estado general, astenia, anorexia, prdida de peso de ms de
8Kg. EF: se encontr palidez cutnea mucosa, disminucin del panculo
adiposo, dolor abdominal ligero a la palpacin de flanco y fosa ilaca
derecha. En ocasiones signos de deshidratacin moderada que
requirieron
tratamiento
de
rehidratacin
endovenoso. Complementarios Hb: 9.2 g/L, leucocitos 3.2 x 109/L,
Segmentados:0.40 Linfocitos:0.57, Eosinfilos: 0.03, Glicemia 5.2
mmol, Creatinina: 53 mol/L. Dosificacin de inmunoglobulinas y

Complemento srico: normal, Antgeno de superficie para hepatitis B:


negativo, Anticuerpo para virus C: negativo, Serologa para VIH:
negativa, Serologa para Sfilis: no reactiva, Ultrasonido abdominal:
normal, Colonoscopa: signos de colitis inespecfica a nivel de rectosigmoide, Trnsito intestinal acelerado con signos radiolgicos de
duodeno-yeyuno-ileitis. Se sigui por consulta al paciente con
tratamiento con Vitamina B12, Multivit, cido flico y fumarato
ferroso. Rx: compatible con neumona muestra infiltrados intersticiales
bilaterales, por pneumocistis jiroveci. Al no existir mejora clnica se
indica nuevamente serologa para VIH siendo positiva. Conteo de
linfocitos CD4+ 48 clulas/L y carga viral 57000 copias.
PREGUNTA
En qu categora clnica se encuentra el paciente en este momento?
RESPUESTA
a.- A2
b.- B3
c.- C3
d.- C1
PREGUNTA
Qu alteraciones gasomtricas es ms probable encontrar en este
paciente, por afeccin respiratoria?
RESPUESTA
a.- Hipoxemia + acidosis respiratoria
b.- Hipoxemia + alcalosis respiratoria
c.- Acidosis metablica + hipoxemia
d.- Alcalosis metablica mas acidosis respiratoria
PREGUNTA
Cul es la conducta teraputica ms adecuada a seguir para su cuadro
respiratorio?
RESPUESTA
a.- Eritromicina
b.- Vancomicina
c.- Trimetropim/sulfametoxaxol
d.- Ceftriaxona
PREGUNTA
Cul sera la conducta teraputica de segunda lnea ms adecuada si
el paciente fuera alrgico al primero?
RESPUESTA
a.- Amoxicilina/Ac. Clavulanico
b.- Dapsona
c.- Ciprofloxacino
d.- Estreptomicina
PREGUNTA
Si este paciente con infeccin crnica por VIH estuviera asintomtico.
Cul sera la indicacin ms acertada para iniciar terapia
antiretroviral?
RESPUESTA
a.- CD4 <350 clulas/l, HIV RNA >5000 copias/ml
b.- CD4 500-700 clulas/l, HIV RNA >3000 copias/ml
c.- Profilaxis postexposicin
d.- Presencia de infeccin por M. tuberculosis

CITOMEGALOVIRUS (CMV)
CIENCIAS BASICAS: CMV es un herpes virus, que hace a las clulas infectadas 2-4 veces el tamao de las clulas circundantes. Como resultado de la
primoinfeccin se establece una infeccin latente o persistente. El virus permanece principalmente en clulas endoteliales y leucocitos durante toda la
vida del individuo a menos que la reactivacin se desencadene por la inmunidad celular deprimida (por ejemplo, en receptores de trasplante o
infectados por el VIH. Enfermedad de inclusin citomeglica (clulas gigantes, con inclusiones que semejan un ojo de lechuza) SALUD PBLICA:

CURSO ENARM CMN SIGLO XXI TEL: 36246001

Pharmed Solutions Institute

PGINA 106

MANUAL DE TRABAJO DEL CURSO ENARM CMN SIGLO XXI


Enfermedad por CMV se encuentra en todo el mundo. En los Estados Unidos, aprox. 1 % de los recin nacidos estn infectados. La seroprevalencia en
pases desarrollados va de 30-70% y en pases en vas de desarrollo alcanza hasta 90% en la edad adulta. Los homosexuales masculinos tienen mayor
seropositividad 94% PATOGENIA: El virus se puede transmitir por la leche materna, saliva, contacto sexual (virus en semen y secreciones cervicales), va
transplacentaria, transfusin sangunea, transfusin de rganos. La transmisin requiere el contacto repetido o prolongado. La infeccin se establece en
las clulas del hospedero, en la lnea mieloide, que se transforma en el reservorio para infecciones posteriores. DIAGNOSTICO: Clnico; La infeccin
congnita por CMV; se produce en aproximadamente el 5 % de los fetos infectados en el contexto de la infeccin por CMV materna primaria durante el
embarazo. Presentan petequias, hepatoesplenomegalia e ictericia. Otros hallazgos; microcefalia con o sin calcificaciones cerebrales, retraso del
crecimiento intrauterino, prematuridad y coriorretinitis. Los hallazgos de laboratorio incluyen pruebas anormales de la funcin heptica (PFH),
trombocitopenia, hemlisis, y el aumento de los niveles de protenas del fluido cerebroespinal (CSF). La tasa de mortalidad es del 20-30% de los lactantes
con enfermedad grave, los sobrevivientes tienen dificultades intelectuales o auditivas. Infeccin perinatal, se adquiere por la lactancia o contacto con
secreciones maternas infectadas. La mayora de pacientes son asintomticos, pero las infecciones oportunistas neumonitis intersticial y otras pueden
ocurrir, particularmente en los bebs prematuros. Mononucleosis; es el sndrome ms comn del CMV en huspedes inmunocompetentes. El perodo
de incubacin vara de 20 a 60 das. Los sntomas duran 2-6 semanas e incluyen fiebre, fatiga profunda y malestar general, mialgias, dolor de cabeza, y
esplenomegalia, faringitis y adenopatas cervicales son raras. Los hallazgos de laboratorio incluyen linfocitosis relativa con > 10 % de linfocitos atpicos. El
aumento de los niveles sricos de transaminasas y fosfatasa alcalina as como anormalidades inmunolgicas (por ejemplo, la presencia de crioglobulinas
o aglutininas fras) pueden ser evidentes. La recuperacin es completa, pero astenia postviral puede persistir durante meses. La excrecin de CMV en la
orina, las secreciones genitales, y/o la saliva pueden continuar durante meses o aos. CMV es el patgeno viral ms comn e importante que complica el
trasplante de rganos, la infeccin por CMV es un factor de riesgo de prdida del injerto y muerte. El riesgo de infeccin es mayor 1-4 meses despus del
trasplante, pero la retinitis puede ocurrir ms tarde. El rgano trasplantado es un riesgo especial, por ejemplo, neumonitis por CMV suele seguir un
trasplante de pulmn. Alrededor del 15-20% de los receptores de trasplante de mdula sea va a desarrollar neumona por CMV, 5-13 semanas despus
del trasplante, con una tasa de letalidad del 84-88 %. El riesgo de enfermedad grave se puede reducir por la profilaxis o terapia antiviral preventiva. El
CMV es un patgeno importante en pacientes con infeccin por el VIH cuyo recuento de clulas CD4 + han cado por debajo de 50 a 100 / l. En esta
configuracin, el CMV causa retinitis, colitis y enfermedad diseminada. Pacientes inmunodeprimidos con infeccin por CMV desarrollan fiebre, malestar
general, anorexia, fatiga, sudores nocturnos y artralgias o mialgias. Taquipnea, hipoxia, y tos no productiva. La participacin del tracto gastrointestinal
puede ser localizada o extensa, con lceras que pueden sangrar o perforar. La hepatitis es comn. SNC, con mayor frecuencia afectan a pacientes
infectados por el VIH, retinitis grave, encefalitis, polirradiculomielopata. Retinitis por CMV puede dar lugar a ceguera. Las lesiones comienzan como
pequeas reas blancas de necrosis retiniana granular, con el desarrollo posterior de las hemorragias y edema retiniano. Infeccin grave se asocia con
virema persistente y compromiso multiorgnico. Amplia necrosis suprarrenal se ve a menudo en la autopsia. El cultivo viral (diagnostico de certeza) y
los anticuerpos monoclonales se utilizan para detectar el antgeno temprano inmediato de CMV. La deteccin de antgenos de CMV en los leucocitos de
sangre perifrica o de ADN de CMV en sangre. Los anticuerpos no pueden ser detectados por un mximo de 4 semanas de la infeccin primaria, y los
ttulos pueden permanecer elevados durante aos. IgM puede ser til en el diagnstico de infeccin aguda. Cuando sea posible, los donantes
seronegativos deben ser utilizados para los receptores de trasplante seronegativos. TRATAMIENTO: Ganciclovir (o valganciclovir, el profrmaco oral de
ganciclovir ) se usa como tratamiento y como profilctico, produce tasas de respuesta del 70-90 % entre los infectados por el VIH, pacientes con retinitis
por CMV o colitis. La terapia de induccin con ganciclovir (5-12mg/kg/da IV 2-7 semanas) o valganciclovir (900 mg/24hrs VO) se da durante 14-21 das.
En los receptores de trasplante, se ha demostrado reduccin de la presencia de CMV, cuando se administra tratamiento profilctico a base de
vanganciclovir cuando el paciente es susceptible. Actualmente se puede utilizar el frmaco durante los primeros 100 das postrasplante. La dosis de
induccin es de 900mg c/12 hrs y despus 900mg c/24 hrs. Otros frmacos activos contra CMV son: El foscarnet es activa contra la infeccin por CMV,
pero se reserva para casos de insuficiencia a ganciclovir o intolerancia a causa de sus efectos txicos, que incluyen la disfuncin renal, hipomagnesemia ,
hipopotasemia , hipocalcemia , y parestesia. Este medicamento debe ser dado a travs de una bomba de infusin, y su administracin debe ser
monitoreado de cerca. Un rgimen de induccin de 60 mg / kg cada 8 horas o 90 mg / kg cada 12 horas durante 2 semanas es seguido por los regmenes
de mantenimiento de 90-120 mg / kg al da. Cidofovir (en casos de retinitis) tiene una intracelular larga vida media. Los regmenes de induccin de 5 mg /
kg por semana durante 2 semanas son seguidos por los regmenes de mantenimiento de 3-5 mg / kg cada 2 semanas. Cidofovir produce nefrotoxicidad
grave por lesin de las clulas del tbulo proximal. El uso de la hidratacin salina y probenecid reduce este efecto adverso.
CASO CLINICO
Hombre de 53 aos, sometido a trasplante renal hace un mes, que
acude a urgencias por cuadro febril con temperatura de 38,5 C de 24
horas de evolucin con escalofros, dolor abdominal leve difuso,
astenia, anorexia y disminucin subjetiva del volumen de diuresis. El
cuadro progres con un aumento del dolor abdominal, aparecieron
nuevos picos febriles junto con cuadro diarreico, con melena, asociado
a deterioro neurolgico, hepatoesplenomegalia, as como alteracin
funcin heptica, anemia y trombocitopenia. Analtica: GOT/GPT
135/156 U/l; LDH 558 U/l; sodio 130 meq/l, fibringeno 133 mg/dl,
hemoglobina 9,2 g/dl, y hematocrito del 26,8% y plaquetas de 48.000
l con leucocitos normales (5.500 l con frmula normal), aumento
muy importante de triglicridos (738 mg/dl), deterioro progresivo de
funcin renal (creatinina entorno a 4-5 mg/dl). Se solicitaron nuevas
pruebas en las que destacaron: antgeno precoz CMV positivo con PCR
de CMV mayor a 100.000 copias/ml.
PREGUNTA
Considerando la complicacin, cual es la medida profilctica mas
apropiada?
RESPUESTA
a.- Ganciclovir 200 mg por 100 das.
b.- Aciclovir 200 mg por 75 dias.
c.- Cidofovir 500 por 100 dias.

CURSO ENARM CMN SIGLO XXI TEL: 36246001

d.- Oseltanmivir 100 mg por 74 dias.


CASO CLINICO
Femenino de 42 aos. Manifiesta 8 meses con herpes recurrente en
boca. Ingresa por padecimiento de 2 meses con tos seca en accesos y
disnea rpidamente progresiva. Adems dolor torcico bilateral, fiebre
hasta 39C y prdida de peso de 14kg. La recurrencia de la lesin
herptica le ocasionaba disfagia y odinofagia. Al examen fsico
presentaba placas blanquecinas en orofaringe; la exploracin del trax,
con disminucin del ruido respiratorio y estertores finos. Al aire
ambiente la saturacin de oxgeno era del 86%. El reporte de la
gasometra arterial con oxgeno suplementario al 70% fue: pH 7,30,
pCO2 40,5mmHg, pO2 132mmHg, HCO3 19,5mmol/l, exceso de base
5,8mmol/l, saturacin de oxgeno al 97,9%. ndice de oxigenacin (IO)
de 188. Laboratorio: linfopenia de 600clulas/mm3, Hb 11,8gr/dl,
deshidrogenasa lctica de 971UI/l y albmina 3,3gr/dl. La Rx de trax
presentaba opacidades bilaterales en parche con vidrio deslustrado y
neumomediastino, por lo cual, en el diagnstico diferencial se incluy
inmunosupresin asociada a VIH y neumona por P. jirovecii (PJP). El
anlisis para VIH por ELISA fue POSITIVO, se confirm por Western
Blot. Se le realiz broncoscopia con biopsia transbronquial y lavado
broncoalveolar (LBA). El estudio histopatolgico se reporta en la figura
1. Recibi tratamiento con Trimetoprim/Sulfametoxasol y Prednisona
en dosis de reduccin por 21 das. Se somete a LBA cuyo estudio de

Pharmed Solutions Institute

PGINA 107

MANUAL DE TRABAJO DEL CURSO ENARM CMN SIGLO XXI


patologa confirma la presencia de hemorragia alveolar reciente y
activa . Adems, por rt-PCR se documenta infeccin por CMV
PREGUNTA
Considerando la complicacin, cual es la medida terapeutica mas
apropiada?

RESPUESTA
a.- Ganciclovir.
b.- Aciclovir.
c.- Cidofovir.
d.- Gamaglobulina.

MONONUCLEOSIS INFECCIOSA (VIRUS DE EPSTEIN-BARR):


CIENCIAS BSICAS: Enfermedad causada por el virus de Epstein-Barr (VEB), pertenece a la familia de los gammaherpesvirus (DNA bicatenario), tambin
llamada enfermedad del beso. El VEB se descubri al observar en el microscopio biopsias de linfoma de Burkitt, un hallazgo fortuito lo asocio a
mononucleosis infecciosa. Se trasmite por secreciones de oro faringe y por contacto sexual. SALUD PUBLICA: Distribucin mundial, con mayor
frecuencia en mujeres adolescentes y adultos jvenes. La incidencia a nivel mundial se aproxima al 100%. La enfermedad recurrente es una fuente de
contagio y puede causar diseminacin asintomtica. PATOGENIA: El VEB infecta las clulas epiteliales de la cavidad oral donde se multiplica en los
linfocitos, causa lisis celular, pasa a la saliva y se disemina en el tejido linftico, es mitogeno para las clulas B y permanece en ellas en forma latente. La
inmunidad celular permanente participa en el control de la infeccin. DIAGNOSTICO: Clnica; en nios generalmente es subclnica, a veces, faringitis,
otitis, adenopatas cervicales. Periodo de incubacin de 10-14 dias, en adultos jvenes fiebre, adenopatas en 2 cadenas ganglionares, exantema
maculopapular, faringitis, fatiga, anorexia, petequias en paladar y frecuentemente hepatoesplenomegalia. Serologa, buscar aglutininas heterofilicas
(titulo >40) o identificacin de cuerpos de inclusin intracelular. En infeccin aguda se observa IgM e IgG contra el antgeno de la casi de viral (anti-VCA),
adems del anticuerpo contra el antgeno temprano (anti-EA). COMPLICACIONES: Puede originar trastornos neurolgicos como: meningoencefalitis,
parlisis de Bell, sndrome de Guillan-Barre, mielitis transversa, neuritis perifrica, anemia hemoltica, CID. Se le a asociado carcinoma nasofarngeo,
linfoma de Burkitt y leucoplasia oral vellosa. TRATAMIENTO: No especifico, solo sintomtico, reposo para evitar rotura esplndida. Corticoides, si hay
obstruccin de va area.
CASO CLINICO
Varn de 19 aos de edad, estudiante, tres semanas antes del ingreso
haba comenzado con fiebre, adenopatas laterocervicales bilaterales,
rash cutneo y dolor farngeo. Haba sido seguido de forma
ambulatoria, pero se decidi su ingreso ante la intensa afectacin del
estado general. Presentaba leucocitosis (24.500cel/l) con predominio
de linfocitos (71%) que en el frotis tenan aspecto atpico en un alto
porcentaje, trombopenia ligera (106.000plaquetas/l), alargamiento
discreto de los tiempos de coagulacin (INR 1,7) y datos de colostasis
(bilirrubina directa 3,1mg/dl). Tena un ttulo alto (>1/256) de IgM
frente al antgeno de la cpside del VEB. El estudio serolgico de CMV,
VHA, VHB y VIH era negativo. Una TAC de trax y abdomen puso de
manifiesto la presencia de esplenomegalia, hepatomegalia, ascitis y
derrame pleural bilateral. Se instaur tratamiento con
metilprednisolona (1mg/kg/da) que se mantuvo durante 30 das. Tras

su inicio remiti la fiebre y se produjo una lenta, pero progresiva


mejora en un periodo de 35 das. El paciente fue dado de alta a los 10
das, pero en el seguimiento posterior refera debilidad y dificultad
para la concentracin durante tres meses ms. Los parmetros
hepticos se normalizaron a los 6 meses del inicio de los sntomas.
PREGUNTA
La manifestacin ms frecuente de este padecimiento es una
mononucleosis severa o fatal, cual de los siguientes cuadros mas
frecuente observar como diagnostico diferencial?
RESPUESTA
a.- Sndrome hemofagoctico asociado a virus.
b.- Trastornos linfoproliferativos.
c.- Disgammaglobulinemias (hipogammaglobulinemia).
d.- Anemia aplstica.

HEPATITIS
CIENCIAS BASICAS: Definicin: Infeccin sistmica que afecta principalmente el hgado (inflamacin). Causada por los virus hepatotroficos (A,B,C,D,E)
que es la causa ms comn y por otros virus (VEB, CMV, coxackievirus, etc), alcohol, drogas, hipotensin e isquemia y enfermedades de la va biliar.
Hepatitis aguda: Enfermedad inflamatoria de hgado, de menos de 6 meses de evolucin, por lo general produce anorexia, ictericia, coluria, y
alteraciones en pruebas de funcin heptica. Hepatitis crnica: inflamacin persistente de hgado, de ms de 6 meses de evolucin, puede desarrollarse
de manera favorable o condicionar cirrosis heptica. Hepatitis fulminante: necrosis masiva del hgado, se manifiesta por encefalopata heptica aguda,
coagulopata, insuficiencia renal y coma, aparece en una enfermedad de hgado con menos de 8 semanas de evolucin. SALUD PUBLICA: 85% de los
pacientes con hepatitis A, tienen recuperacin clnica y bioqumica aprox. a los 3 meses. Cerca de 5% de la poblacin mundial est infectada con VHB, en
EU, es responsable de 5-10% de los casos de hepatopata crnica y cirrosis. El riesgo de transmisin por puncin con aguja es VHB se transmite en 30%
de las exposiciones, VHC en 3% y VIH en 0.3%. La prevalencia mundial es de 0.5-2%, en Mxico es de 1.4% La evolucin natural de la infeccin por VHC es
la hepatitis crnica activa (hasta 74%). El riesgo de presentar carcinoma hepatocelular en pacientes con cirrosis por VHC es de 1-4% por ao.
DIAGNOSTICO: Cuadro clnico: Manifestaciones comunes: Malestar, nausea, vmito, diarrea, fatiga, febrcula seguida de orina oscura, ictericia y
hepatomegalia dolorosa puede ser subclnica y detectarse por niveles elevados de AST y ALT. Manifestaciones poco comunes: vasculitis, artritis, neuritis
ptica, anemia aplasica, aplasia de serie roja, y mielitis transversa. HEPATITIS VIRALES AGUDAS Y CRONICAS: HEPATITIS A: El VHA, es picornavirus RNA no
cubierto. Principal va de transmisin fecal-oral ( alimentos contaminados, guarderas), poblacin de riesgo son nios y adolescente, la infeccin no suele
ser grave (autolimitada), seguido del proceso se producen anticuerpos contra el virus de la hepatitis A, lo cual confiere inmunidad. Incubacin promedio
de 30 das, la hepatitis se produce por dao secundario de la respuesta del sistema inmune del husped mediada por dao citotxico por linfocitos T
CD8 y clulas NK. Puede presentarse desde asintomtica hasta insuficiencia heptica aguda. DIAGNOSTICO: IgM anti-VHA en una muestra de suero de
convalecientes aguda o temprana, en la exposicin pasada se caracteriza por anticuerpos anti-IgG positivos con anti-IgM negativos (las cuales se
negativizan a los 6 meses), elevacin de ALT, AST, bilirrubinas y FA. TRATAMIENTO: Medidas de soporte, cuando hay complicaciones como falla heptica
fulminante, valorar terapia intensiva y trasplante heptico. PREVENCION: Inmunizacin pasiva posexposicin, inmunoglobulina humana 0.2 ml/Kg IM,
proteccin por 6 meses, indicado en pacientes que planeen viajar a zona endmica, o contacto ntimo con alguien infectado con VHA. Inmunizacin
activa: Vacuna contra VHA (1ml IM y refuerzo a los 6 y 12, 0.5ml para nios) indicado en pacientes con enf. heptica crnica, varones homosexuales,
drogadictos, personal de salud que trabaje con VHA. HEPATITIS B: El VHB, es hepadnavirus DNA, est compuesto por la polimerasa con actividad de
transcriptasa reversa, una proteincinasa rodeada por el antgeno del ncleo (HBcAg) y una envoltura que contienen el antgeno de superficie
glucoproteico (HBsAg). El HVB puede causar accin citopatica directa y a travs de respuesta inmune. Factores de riesgo son; contacto sexual, uso de
drogas intravenosas (20%), transfusiones, contaminacin con suero o leche materna, principal va perinatal. Periodo de incubacin de 40-150 das, puede
presentarse desde asintomtica hasta hepatitis fulminante (<1%) Hepatitis aguda: malestar general, nausea, vomito, anorexia, dolor en hipocondrio der.,
ictericia que dura de 1-3 meses (70% anictericos). Hepatitis crnica: Ag de superficie positivo por mas de 6 meses, concentraciones sricas de DNA >10

CURSO ENARM CMN SIGLO XXI TEL: 36246001

Pharmed Solutions Institute

PGINA 108

MANUAL DE TRABAJO DEL CURSO ENARM CMN SIGLO XXI


copias/ml, aumento de aminotransferasas persistentes, biopsia heptica que muestre inflamacin. DIAGNOSTICO: HBsAg en suero infeccin aguda o
crnica (si persiste por mas de 6 meses). IgM anti-HBc (indica infeccin aguda o reciente). La prueba ms sensible para la deteccin de HVB DNA en suero
(actividad viral replicativa en suero). Paciente con AntiHBs es una persona sana vacunada. Las transaminasas pueden aumentar hasta 1000 o 2000 y si
permanecen por ms de 6 meses se vuelve crnica. TRATAMIENTO: En aguda es de sostn, en la crnica, suprimir la replicacin del virus con el fin de
evitar progresin a cirrosis y cncer (realizar tamizaje de alfafetoproteina y ultrasonido heptico cada 6 meses). El interfern alfa-2b, 5-10 millones de U,
diarias, 3 veces a la semana SC, por 16 sem. Actualmente se usa alfa-2b pegilado, se aplica una vez por semana a una dosis de 180mg SC por 48 sem. La
lamivudina (3TC), inhibe la transcriptasa reversa, disminuyendo la carga viral, otros adefovir y entecavir. PREVENCION: Vacunacin administrar HBsAg,
para estimular la produccin de antiHBs, en individuos no infectados, efectividad >95%; vacunacin universal en todos los recin nacidos, as como en
trabajadores de la salud, paciente s en hemodilisis, familiares y parejas sexuales de pacientes con HBsAg. Vacunacin via IM a los 0, 1, 6 meses.
HEPATITIS C: El VHC, es RNA monocatenario. Periodo de incubacin de 2 meses. Factores de riesgo, uso de drogas IV, transfusin sangunea antes de
1990, la transmisin vertical es infrecuente, est mas relacionada con una coinfeccin con VIH-1, en la madre. La transmisin sexual tiende a ser menos
frecuente que otros virus (VIH), por la menor carga viral en lquidos y tejidos genitales. Las clulas blanco del VHC son los hepatocitos y lis linfocitos B. La
sintomatologa aparece aprox. De 6-8 sem de la exposicin, puede pasar asintomtico, ictrica, malestar general, nausea. La infeccin crnica se
caracteriza por presentar periodos prolongados sin manifestaciones, la aguda progresara a crnica, con diversos grados de hepatitis, fibrosis y cirrosis;
entre los factores que promueven la progresin clnica se observan el consumo de alcohol, coinfeccin con VIH-1 o VHB, gnero masculino, y edad
mayor al momento de infeccin. Otras manifestaciones estn asociadas a enf. autoinmuines (tiroideas, DM). DIAGNOSTICO: La prueba de rutina es la
serolgica mediante ensayo inmunoenzimtico, que detecta anticuerpos a partir de 4-10 sem posteriores a la infeccin. El inmunoblot se usa como
prueba confirmatoria. Los estudios moleculares se basa en deteccin del RNA viral mediante PCR, pueden ser cuantitativos (limite menor de deteccin
100 copias/ml) o cualitativos, la carga viral nos ayuda con la evolucin y respuesta a tx. Los anticuerpos anti-VHC se pueden detectar en >97% de las
personas a los 6 meses posteriores a la exposicin. La biopsia es el estndar de oro para determinar la actividad de VHC, es el nico predictor pronstico
de la progresin. TRATAMIENTO: Indicado en: carga viral positiva, biopsia heptica con fibrosis > grado 1 o enfermedad heptica compensada (CHILD A o
B). Contraindicaciones: cirrosis descompensada, embarazo, enf. psiquitrica, consumo activo de drogas, enf. pulmonar o cardiaca grave, DM
descontrolada, enf. autoinmunes. Frmacos: interfern alfa en monoterapia o combinado con ribavirina; la combinacin confiere 55% de respuesta viral
sostenida. HEPATITIS D: El VHD, es RNA monocatenario, requiere la presencia del VHB para su supervivencia y replicacin, la forma ms comn de
contagio es por compartir agujas en personas que usan drogas IV, tambin transmisin sexual y perinatal. La coinfeccin aguda es la infeccin por
exposicin simultnea a VHB y VHD. La superinfeccin es la exposicin del VHD, en un individuo ya infectado con VHB. El diagnstico con IgM e IgG
antiVHD o por la deteccin de RNA de VHD en suero. HEPATITIS AIUTOINMUNE: Sospechar ante una hepatitis que presente aumento de globulinas
plasmticas y autoanticuerpos especficos; es una enfermedad necroinflamatoria crnica del hgado de causa desconocida, parece haber predisposicin
gentica (HLA-B8, DR3 y DR5). La hepatitis autoinmune se divide en 3 de acuerdo con los anticuerpos en suero: Tipo I: Clsica o lupoide, ms comn en
mujeres jvenes, 70% menores de 40 aos, los anticuerpos que se distinguen son los antinucleares (ANA), antimusculo liso (AML), antiactina y
antineutrofilos (pANCA) e hipergammaglobulinemia. Tipo II: Ms comn en Europa, en especial en nios, aqu encontramos annticuerpos
anticromosomas de hgado y rin tipo 1 (anti-LKM1), asociada a enf. inmunolgicas, tiende a progresar rpido a cirrosis. Tipo III: 90% mujeres entre 2040 aos, se observa anticuerpo anti-SLA/LP, amplia relacin con tiroiditis autoinmune. La hepatitis autoinmune es de progresin lenta, como hepatitis
aguda adems, mltiples telangiectasias, estras, cutneas, acn, hirsutismo, hepatomegalia, artralgias y amenorrea. Para tx se usan glucocorticoides de
sntesis (prednisona, metilprednisolona), tambin aziatropina en biterapia con cortoicoesteroides (xito de 80-90%). HEPATITIS ALCOHOLICA: Se
presenta con niveles de aminotransferasas <300-500 con AST: ALT >2.1 y GGT elevada, y deficiencia concomitante de vitamina B12, los gravemente
afectados presentan al inicio fiebre, hepatomegalia, leucocitosis, ictericia y coagulopatas, asi como manifestaciones de hipertensin portal, es comn
que desarrollen cirrosis. El pronstico se ve en particular afectado por la presencia de elevadas concentraciones de bilirrubina (>12mg/dl), ascitis e
insuficiencia renal. Tx.: indicado en pacientes con encefalopata, administracin de prednisona, pentoxifilina por un mes disminuye mortalidad e
incidencia de sx. hepatorrenal. TOXICIDAD POR PARACETAMOL: Se presenta con dosis mayores de 10 gr, puede presentarse en pacientes con
desnutricin o alcoholismo con 2-6 gr, parece proveer un reservorio de grupos sulfidrilo y esto obliga a los metabolitos txicos o estimulacin de la
sntesis de glutatin heptico si la falla heptica se presenta en las primeras 4 hrs, el tx.: consiste en la administracin de carbn activado o colestiramina
y lavado gstrico. La N-acetilcisteina debe ser iniciado dentro de las primeras 8 hrs de la ingestin, pero puede ser efectivo incluso en las 24-36hrs
despus de la sobredosis, se prefiere va oral con una dosis carga de 140mg/Kg y despus 70mg/Kg cada 4 hrs por 17 dosis. Otros medicamentos que
producen hepatitis son: amiodarona. Azoles, isoniacida, metildopa, fenitoina, rifampicina, sulfas. Hay que observar al paciente de 72-96hrs para evaluar
dao heptico.
CASO CLINICO HEPATITIS A
Varn de 33 aos no fumador con el nico antecedente personal de
hipercolesterolemia a tratamiento con dieta. Realiza ejercicio fsico de
forma regular y su trabajo es sedentario. Refiere malestar general
acompaado de cefalea, astenia y fiebre vespertina de hasta 39,2 C,
acolia y coluria. En la EF no se observa ictericia, se auscultan niveles
hidroaereos aumentados, Refiere haber comido ostras vivas una
semana antes, las PFH se encuentran en parmetros elevados, mas un
patrn colestasico, se realiza serologa resultado positivo a virus a
hepatitis A (IgM positiva e IgG negativa).
PREGUNTA
Cual es la complicacin mas frecuente observar en esta patologia?
RESPUESTA
a.- Hepatitis recidivante.
b.- Hepatitis colestsica.
c.- Hepatitis aguda fulminante.
d.- Sndrome post-heptico.
PREGUNTA

CURSO ENARM CMN SIGLO XXI TEL: 36246001

Considerando la fisiopatogenia del caso, en cuanto tiempo se puede


eliminar el virus por las heces?
RESPUESTA
a.- Una semana antes de la clnica.
b.- Durante la presentacin de la clnica.
c.- Una semana despus de la clnica.
d.- dos semanas despus de la clnica.
PREGUNTA
Habitualmente la acolia y la coluria se presenta cuantos das antes de
la ictericia?
RESPUESTA
a.- 1 a 5 dias.
b.- 6 a 10 dias.
c.- 11 a 15 dias.
d.- 16 a 20 dias.
CASO CLINICO HEPATITIS B
Hombre de 27 aos sin antecedentes mrbidos, consult por
compromiso del estado general y fiebre de 5 das, con posterior
aparicin de ictericia. Se hospitaliz con bilirrubina total de 13 mg/dl,

Pharmed Solutions Institute

PGINA 109

MANUAL DE TRABAJO DEL CURSO ENARM CMN SIGLO XXI


aminotransferasa oxalactica de 1.977 mU/ml y pirvica de 5.975
mU/ml, fosfatasas alcalinas de 157 U/L, y tiempo de protrombina 15%.
Del panel viral result positivo un HBsAg. Evolucion con compromiso
de conciencia hasta Glasgow 8, con tomografa computada (TC) de
cerebro normal. Fue recibido en la Unidad de Cuidados Intensivos con
Glasgow 5 sospechndose broncoaspiracin. Se conect a ventilacin
mecnica, se iniciaron antibiticos y N-acetil cistena. La TC de trax
revel neumona, y la de cerebro edema cerebral leve. La
ecotomografa abdominal muestr hgado normal y ascitis leve. Dentro
del estudio etiolgico presenta serologa para virus hepatitis C y A y
estudio de autoinmunidad (anticuerpos y recuento de
inmunoglobulinas) negativos. Se repiti el estudio del VHB destacando
aparicin de anticuerpos contra el HBsAg (antiHBs) y negativizacin del
HBsAg, con anticore positivos, IgM y total. Se realiz ELISA para VIH,
que result positivo, y recuento de CD4 de 634 clulas/ul. La carga viral
(CV) del VHB fue de 140 copias/ml, y la del VIH 16.900 copias RNA/ml.
12 dias despus fallece el paciente.
PREGUNTA
Cual de las siguientes observaciones sobre el presente caso clnico
explica mas adecuadamente la complicacion del paciente?
RESPUESTA
a.- Falla Hepatica Fulminante por manejo inoportunamente
diagnosticada y tratada.
d.- La falla hepatica por VHB con HIV ocurre en aproximadamente 20
%.
c.- En la falla heptica, mientras mayor es la respuesta del hospedero
mayor es su probabilidad.
d.- En estados de inmunodepresin como es el tener VIH avanzado, el
desarrollo de Falla Hepatica sea infrecuente.
PREGUNTA
Considerando la evolucin del caso, cual es el periodo necesario para
considerar falla heptica fulminante.
RESPUESTA
a.- Dentro de las 72 horas.
b.- Dentro de la primera semana.
c.- Dentro de las primeras 8 semanas.
d.- Dentro de las primeras 12 semanas.
PREGUNTA
De las siguientes observaciones relacionadas a esta patologia cual es la
menos probable?
RESPUESTA
a.- Mexico se encuentra dentro de los pases de baja seroprevalencia.
b.- Se estima que hay almenos de 110,000 portadores crnicos.
c.- El VIH tiene una mayor virulencia que el VHB.
d.- La infeccin crnica por el VHB despus de una exposicin aguda es
del 5 %.
PREGUNTA
La infeccin crnica por el VHB, tiene cuatro fases, de acuerdo a la
presencia o ausencia del HBeAg, carga viral, nivel de ALT y hallazgos
histolgicos, cual de las siguiente no es correcta?
RESPUESTA
a.- Inmunotolerancia.
b.- Portador activo.
c.- Inmunoeliminacion.
d.- Hepatitis crnica HBeAg negativa.
CASO CLINICO HEPATITIS C
Masculino de 70 aos con hepatitis crnica por el VHC genotipo 1 de
cronologa incierta, infeccin pasada por el VHB y bebedor de unos 60
gramos de alcohol al dia. Se observo elevacin de GGT y FA, en la
ecografa/TAC abdominal convencional en fase venosa se puso de
manifiesto una tumoracin de 7 cm de caractersticas solidas, en
lbulo heptico izquierdo, sobre el hgado de morfologa normal, la
biopsia revelo que se trataba de un CHC, sin presencia clnica ni

CURSO ENARM CMN SIGLO XXI TEL: 36246001

paraclinica de cirrosis previa y el valor de la alfafetoproteina era de


6.9, se realizo tratamiento QX.
PREGUNTA
Considerando el caso clnico cual es la tasa de sobrevida al ao del
presente caso?
RESPUESTA
a.- del 15 %.
b.- del 45 %.
c.- del 75 %
d.- del 95%.
PREGUNTA
Relacionado a la patologia descrita en el caso previo, cual de las
siguientes aseveraciones no es cierta?
RESPUESTA
a.- La hepatitis C es una hepatopata crnica que cursa silenciosa y
tiene una prevalencia de 1 a 1.9 en Mxico.
b.- La muerte por infeccin crnica de VHC estn esencialmente
relacionadas con la descompensacin heptica, enfermedad heptica
terminal y el carcinoma hepatocelular.
c.- Las pruebas serolgicas actuales permiten distinguir entre infeccin
aguda, crnica o resuelta, por lo cual se puede llevar un buen
monitoreo.
d.- La infeccin por el VHC se ha situado como la principal causa de
hepatopata crnica, cerca del 27 % de los casos de cirrosis y el 25 % de
los casos de hepatocarcinomas.
PREGUNTA
Cual de las siguientes aseveraciones no es correcta en el diagnostico
de infeccin por VHC?
RESPUESTA
a.- Durante la fase aguda de la infeccin la dereccion de acido nucleico
puede generar inadecua interpretacin de resultados.
b.- Con una prueba anti-VHC positiva se debe realizar prueba PCR-RNAVHC.
c.- Se debe solicitar prueba de RNA del VHC en los pacientes que se
esta considerando la administracin de tratamiento antiviral.
d.- Se debe solicitar una prueba anti-VHC en pacientes con VIH para
determinar comorbilidad.
CASO CLINICO HEPATITIS AUTOINMUNES.
Mujer de 62 aos que refiere, desde hace 13 aos, mltiples episodios
de visin borrosa en ambos ojos con recuperacin posterior completa
de la agudeza visual tratada con esteoides, TA 100/70mmHg, Glucosa
84 mg/dl, con antecedente de una hermana con el mismo cuadro,
presin intraocular normal, inicia cuadro actual con dispepsia, vomito,
prurito, hiporexia, astenia, GOT 110, GPT 90, GGT 110,
hipergammaglobulinemia (IgG 1620), ANA 1/320, AMA negativos,
Antigeno de superficie VHB, Anticuerpo anti HBs, Anticuerpo anti-HBc
y Anticuerpo anti-VHC negativos, USG heptico normal.
PREGUNTA
Considerando las manifestaciones clnicas asi como los estudios de
laboratorio y gabinete, cual es la conducta mas adecuada?
RESPUESTA
a.- Repetir estudios serolgicos ms sensibles y especficos.
b.- Realizar biopsia heptica percutnea.
c.- Mantener en observacion y repertir estudios en 6 meses.
d.- Manejo conservador y corticoides.
PREGUNTA
Considerando la respuesta anterior, los antecedentes familiares y
personales cual es el diagnostico mas probable?
RESPUESTA
a.- Hepatitis B.
b.- Hepatitis A.
c.- Hepatitis autoinmune.

Pharmed Solutions Institute

PGINA 110

MANUAL DE TRABAJO DEL CURSO ENARM CMN SIGLO XXI


d.- Hepatitis Ideopatica.
CASO CLINICO HEPATITIS ALCOHOLICA
Masculino de 64 aos con historia de multiples ingresos por
alteraciones del estado de conciencia debido a insuficiencia heptica y
atrofia cortical, acude 14 dias despus su ultimo internamiento por
alteracin del estado de conciencia, por fiebre, desorientacin y dolor
abdominal, urea 103, creatinina 2,6. LDH 699, CPK 864, GOT 57, GPT
46. La gasometra revela una acidosis respiratoria. En la radiografia se
observa torax enfisematoso e importante cardiomegalia sin signos
neumnicos ni deramme pleural.
PREGUNTA
Se diagnostico como neumona, considerando el caso, cual de los
siguientes agentes es mas frecuente?
RESPUESTA
a.- Streptococcus neumonae.
b.- Stafilococcus aureus.
c.- Listeria monocitogenes.
d.- Campylobacter jejuni.
CASO CLINICO HEPATITIS ALCOHOLICA
Masculino de 54 aos con DM, artrosis lumbar y hepatopata
alcoholica, acude por dolor abdominal generalizado, fiebre, perdida de
20 kg en 6 meses. Leucocitosis a expensas de neutrofilos, acompaado
de trobocitosis, glucosa 263, urea 185, creatinina 1,2. LDH 204 y PCR
358. La radiografia de torax no muestra imgenes de condensacin.
PREGUNTA
Se diagnostico como neumona, considerando el caso, cual de los
siguientes agentes es menos frecuente?
RESPUESTA
a.- Streptococcus neumonae.
b.- Stafilococcus aureus.
c.- Listeria monocitogenes.
d.- Campylobacter fetus.
CASO CLINICO HEPATITIS POR INTOXICACION.
Masculino de 18 aos de edad que consulta de rutina, los laboratorios
de rutina reportaron elevacin de GOT de 56 y el GPT de 78 las cuales
fueron incrementando lentamente desde hace 16 meses. GGT estaban
discretamente aumentada y el resto de parmetros hepticos eran
normales. No haba serologa positiva a virus de la hepatitis B y C. Los
autoanticuerpos para descartar hepatopata autoinmune eran
negativos. No hay antecedentes de de hemotransfusion ni quirrgicos.
Hace ejercicio y como nico antecedente refiere tomar finasterida 1
mg diario para evitar la alopecia desde hace 2 aos.

PREGUNTA
Cual es la conducta mas apropiada a seguir?
RESPUESTA
a.- Retirar finasterida.
b.- Realizar USG heptico.
c.- Realizar biopsia heptica.
d.- Repetir serologa.
CASO CLINICO AMD
Varn de 60 aos con antecedentes de hipertensin arterial, adenoma
de prstata e hiperuricemia ocasional. Haba sido diagnosticado de
miocardiopata dilatada idioptica haca 5 aos y segua tratamiento
con digoxina, ibopamina, quinapril, espironolactona, hidralacina,
dicumarina, furosemida y nitratos transdrmicos, con lo que se
mantena en CF-II. Presentaba adems fibrilacin auricular paroxstica
por lo que espordicamente haba recibido AMD por va oral (200
mg/da). No tena antecedentes de enolismo. Nunca se detectaron
alteraciones de la funcin heptica. Ingres por un cuadro de
taquicardia ventricular sostenida a 190 por minuto con morfologa de
bloqueo de rama derecha. La arritmia curs con regular tolerancia
clnica y sin constatarse en ningn momento signos de compromiso
hemodinmico (hipotensin, oliguria, etc.). Se haba restablecido el
ritmo sinusal por medio de un bolo de 100 mg de lidocana y se haba
instaurado tratamiento con AMD intravenosa a dosis de 1.200 mg/da
durante los 7 das previos. A su llegada a la unidad coronaria el
paciente presentaba estabilidad hemodinmica con presin arterial de
120/80 mmHg y frecuencia cardaca irregular a 90 por minuto con
pulsos normales. Se apreciaban a la auscultacin cardaca un tercer
sonidos y un dbil soplo holosistlico en foco mitral. No haba edemas,
ingurgitacin yugular o alteraciones en los ruidos respiratorios.
Destacaba especialmente una ictericia franca en piel y conjuntivas, y
una evidente depresin del nivel de conciencia con flapping-tremor.
No se apreciaron hepatomegalia, ascitis ni ningn estigma de
hepatopata crnica. En el electrocardiograma se observaba fibrilacin
auricular a 90 por minuto con bloqueo de rama izquierda. La
radiografa de trax presentaba cardiomegalia y signos de hipertensin
venocapilar pulmonar. En la analtica al ingreso se observaba como
nico dato destacable una alanina-aminotransferasa (ALT) de 52,4
kat/l (normal: 0,46-0,77).
PREGUNTA
Cual es la conducta mas apropiada a seguir?
RESPUESTA
a.- Retirar amiodarona.
b.- Realizar USG heptico.
c.- Realizar biopsia heptica.
d.- Repetir serologa

RABIA:
CIENCIAS BSICAS: Zoonosis causada por Rabdovirus, se trasmite al hombre principalmente por la saliva de animales infectados (y en periodo de
transmisin) a partir de una mordedura, rasguo o solucin de continuidad en la piel o mucosas, se presenta como una encfalomielitis de curso agudo.
PATOGENIA: La velocidad con se manifiesta la rabia, depende de la cepa del virus, de la concentracin de receptores para el virus en las clulas nerviosas
del msculo esqueltico, de la magnitud del inoculo, de la internacin en el sitio de entrada y de la proximidad de la lesin al SNC. Cuando la va de
entrada es transcutanea o epidrmica, el virus se queda un tiempo en el sitio de inoculacin. En ese lapso puede ocurrir una primera replicacin en las
clulas nerviosas de las placas musculares ms cercanas a la herida y con ello aumenta la carga viral, despus el agente infeccioso se desplaza de 8-mm
por da, avanzando por continuidad al infectar nuevas neuronas del SNP. Por endocitosis o fusin de membranas. El camino que sigue el virus de SNP al
SNC se denomina diseminacin centrpeta. Una vez en el SNC la infeccin es irreversible, conduciendo invariablemente a la muerte del individuo. La
multiplicacin del virus en el encfalo inicia en el sistema lmbico y luego al resto del cerebro, produciendo edema, congestin vascular, infiltracin
directa de linfocitos e hiperemia de las leptomeninges, despus de haber completado su invasin al cerebro comienza la diseminacin centrifuga, en la
cual el virus regresa a los rganos con alta intervencin nerviosa, cuando llega a glndulas salivales, se difunde por el nervio trigmino, lo que inicia la
eliminacin del virus a travs de la saliva. En perros la eliminacin de virus por saliva es de 3-10 das antes de que se manifiesten los primeros signos
clnicos, esto es importante ya que el animal esta asintomtico, pero ya es infectivo. CURSO CLNICO: Periodo de incubacin: Va desde la exposicin
hasta inicio de signos clnicos, depende de la cantidad de virus inoculados y sitio anatmico (cabeza 30-48 das, mano 40-59 das, pierna 38-72 das). Las
primeras manifestaciones se presentan de 1-3 meses despus de la agresin. Prodromo y primeros sntomas: Dura de 1-20 das, sntomas neurolgicos
como ansiedad y agitacin que son precedidos por malestar general, parestesias en el sitio de la lesin, fiebre, anorexia, vomito, dolor de cabeza, letargo
y fiebre. Periodo neurolgico agudo: Signos de dao a SNC, se puede presentar como rabia paraltica o furiosa. La paraltica es ms comn en ganado
bovino y roedores, la furiosa en mamferos carnvoros que son reservorios de la enfermedad. Los signos y sntomas para ambas son: fiebre, rigidez de la

CURSO ENARM CMN SIGLO XXI TEL: 36246001

Pharmed Solutions Institute

PGINA 111

MANUAL DE TRABAJO DEL CURSO ENARM CMN SIGLO XXI


nuca, parestesias, fascculacion muscular, convulsiones generalizadas y focalizadas, hiperventilacin e hipersalivacion, la presencia de hidrofobia que es
patognomnico de rabia, dura de 1-5 min y se espacia por periodos de lucidez, es producto de un reflejo exagerado para proteger el tracto respiratorio,
ms que como espasmo de faringe y laringe. En esta fase evoluciona hasta la afeccin del SNC, que incluye hiperactividad, desorientacin, delirio,
alucinaciones, convulsiones, rigidez de nuca y progresan hasta que se presenta parlisis irregular de msculos respiratorios lo que conduce a estado de
coma. Durante el periodo de transicin entre la fase neurolgica y el coma se ostentan lapsos de lucidez y locura, respiracin rpida e irregular, ya
posteriormente parlisis generalizada particularmente en la zonas inervadas por los nervios craneales y en el msculo cardiaco, se paralizan los msculos
respiratorios y sobreviene la muerte. DIAGNOSTICO: Se pueden hacer muestras postmortem (biopsia 10-20g de cerebro, 1-3 g de hipocampo o 5-10g de
medula espinal) en animales y seres humanos y antemortem (biopsia de cuero cabelludo, impronta de cornea, saliva suero y liquido cefalorraquiedeo),
en seres humanos, tomar muestras de 7-14 das de iniciados los signos neurolgicos. La tcnica de rutina para diagnostico de rabia es
inmunofluorescencia directa, otras tcnicas auxiliares son determinacin en LCR, ELISA. TRATAMIENTO: Observar al perro o gato 10 das posteriores a la
agresin. La atencin inmediata de la lesin es; lavar con agua y jabn de forma abundante, desinfectar con agua oxigenada o tintura de yodo, si
requiere sutura aplicar primero inmunoglobulina antirrbica humana y se aproximan los bordes, valorar aplicacin de antibiticos y toxoide tetnico en
heridas contaminadas o punzantes, secar con gasa y cubrir. Si animal agresor no se localiza iniciar tratamiento antirrbico: A) Exposicin leve; una dosis
de vacuna en los das 0, 3, 7, 14, 28, en la regin deltoidea por va IM. B) Exposicin grave; aplicar inmunoglobulina antirrbica humana lo ms cerca al
da de la agresin (20 UI/kg), la mitad alrededor de la herida y el resto IM. Tambin debemos aplicar la vacuna antirrbica humana, si el animal sigue
sano durante 5 dias posteriores se suspende la vacuna, de lo cobtrario prodeguir. PREVENCIN: El control en las poblaciones y la variacin parenteral y
oral de reservorios silvestres y domsticos es ahora el mejor tipo de prevencin.
CASO CLINICO
Nia de 12 aos ingresa al Hospital General OHorn de la Ciudad de
Mrida, referida de la localidad de Tekax, Yucatn, por una historia de
dolor en miembro torcico derecho por aparente traumatismo que no
haba cedido al tratamiento mdico, sin ms antecedentes de
importancia. La afeccin continu con limitacin funcional de la
mueca y disminucin de la fuerza de dicha extremidad superior, la
cual fue progresiva, hasta acompaarse de dolor torcico con
limitacin de la respiracin, fiebre, odinofagia, presentando aerofobia
e hidrofobia y cambios en la conducta que se hicieron mas evidentes,
por lo que se sospech de una encefalitis por virus rbico. La nia
contino con deterioro general que hizo necesaria la asistencia
ventilatoria, con evolucin trpida y datos clnicos de muerte cerebral
declarada 12 dias despus.
PREGUNTA
Cual es la conducta a seguir?
RESPUESTA
a.- Profilaxia con inmunoglobulina a contactos.
b.- Vacunacion antirrbica canina en cerco sanitario.
c.- Cerco epidemologico con vacunacin antirrbica a contactos.
d- Busqueda de perros sospechos para rabia.

CASO CLINICO
Se trata de masculino de 25 aos de edad dedicado al campo, refiere
que al estar trabajando fue mordido por perro desconocido, el cual no
pudo capturar, se conoce de casos de rabia en la zona en animales no
domesticados, acude a consulta una hora despus del hecho.

CURSO ENARM CMN SIGLO XXI TEL: 36246001

CASO CLINICO
Nia de 12 aos de edad, referida de la localidad de Tekax, Yucatn,
por una historia de dolor en miembro torcico derecho por aparente
traumatismo que no haba cedido al tratamiento mdico, sin ms
antecedentes de importancia. La afeccin continu con limitacin
funcional de la mueca y disminucin de la fuerza de dicha extremidad
superior, la cual fue progresiva, hasta acompaarse de dolor torcico
con limitacin de la respiracin, fiebre, odinofagia, presentando
aerofobia e hidrofobia y cambios en la conducta que se hicieron mas
evidentes. La nia contino con deterioro general que hizo necesaria la
asistencia ventilatoria, con evolucin trpida y datos clnicos de
muerte cerebral.
PREGUNTA
Cul es el diagnostico mas probable para esta paciente?
RESPUESTA
a.- Meningitis bacteriana
b.- Encefalitis por virus rbico
c.- Meningitis asptica
d.- Encefalitis herpes virus

PREGUNTA
Cual de los siguiente vectores no puede transmitir el virus?
RESPUESTA
a.- Canes.
b.- Felinos.
c.- Murcielagos.
d.- Roedores.

PREGUNTA
Cul es la conducta ms apropiada a seguir en este caso?
RESPUESTA
a.- Lavar la herida y vigilancia.
b.- Lavar la herida y administracin de toxoide.

c.- Lavar la herida, administracin de toxoide y aplicar globulina


inmunitaria humana.
d.- Lavar la herida, administrar toxoide, aplicar globulina inmunitaria
humana y vacuna de clulas diploides humanas.

PREGUNTA
Cul es el periodo de incubacin que se presenta hasta en 95% de los
casos?
RESPUESTA
a.- 3 semanas a 4 meses
b.- 2 meses a 1 ao
c.- 5 meses a 1 ao
d.- 3 semanas a 4 semanas
PREGUNTA
Cul es la va de entrada ms probable para este caso?
RESPUESTA
a.- Manipulacin de lquidos o tejidos animales
b.- Ingesta de bacterias aerolizadas
c.- Contaminacin de una herida por saliva de animal enfermo
d.- Transmisin por fmites

Pharmed Solutions Institute

PGINA 112

MANUAL DE TRABAJO DEL CURSO ENARM CMN SIGLO XXI


NEOPLASIA DEL SISTEMA NERVIOSO CENTRAL
EPIDEMIOLOGIA: la incidencia de los tumores del SNC oscilan entre el 5.70 y el 9.63 en hombres y entre el 4.71 y el 6.95 en mujeres. Sin embargo, en
todas las series los tumores ms frecuentes son los gliomas, y dentro de ellos los glioblastomas. En cuanto a la edad de presentacin, los tumores del
SNC muestran una distribucin bimodal, con un pico en la edad peditrica y un aumento progresivo de la incidencia en los adultos, hasta alcanzar su
mximo entre la sexta y la sptima dcada de la vida. La incidencia combinada de tumores del SNC en EEUU fue de 6,6 casos nuevos por 100.000
habitantes-ao, en tanto que la mortalidad se estim del 4,7 por 100.000 habitantes/ao. CLASIFICACIN: El grado I incluye tumores de bajo potencial
proliferativo y posibilidad de curacin con extirpacin quirrgica completa. El grado II incluye tumores con capacidad infiltrativa y capacidad de
recurrencia y progresin a mayor grado de malignidad, a pesar de presentar un bajo nivel de actividad proliferativa (por ejemplo, los gliomas de bajo
grado pueden progresar a gliomas anaplsicos y/o glioblastoma).
El grado III incluye tumores con evidencia histolgica de
malignidad, como atipia nuclear y alta actividad mittica. El grado
IV incluye tumores con evidencia histolgica de malignidad (atipia
nuclear, alta actividad mittica, necrosis, proliferacin
microvascular) y comportamiento clnico agresivo con rpida
progresin, recurrencia a pesar de tratamiento intensivo y muy
alta mortalidad. Algunos tumores de grado IV como los
meduloblastomas y tumores de clulas germinales son
rpidamente fatales si no se tratan, pero se asocian a altas tasas
de respuesta y supervivencia con tratamiento adecuado. Aunque
no son requisitos indispensables, la capacidad de infiltracin
difusa del tejido sano adyacente y la propensin a diseminacin
leptomenngea (e incluso extraneural) son caractersticas de
algunos tumores de grado IV. DIAGNSTICO: Los tumores del
sistema nervioso suelen presentarse con sntomas neurolgicos
de instauracin progresiva, aunque tambin es posible la
presentacin aguda (por ejemplo, crisis comiciales o dficit
neurolgico agudo secundario a hemorragia intratumoral). As mismo, los sntomas pueden ser generalizados (por aumento inespecfico de la presin
intracraneal) o focales (dependientes de la localizacin tumoral).La cefalea es el sntoma de presentacin ms frecuente de los tumores cerebrales,
aunque como sntoma aislado slo se da entre un 2 y un 16% de los casos. Las crisis comiciales son el segundo sntoma en frecuencia, pudiendo
desarrollarlas hasta un 35% de pacientes. Son especialmente habituales en gliomas de bajo grado. Pueden tener un inicio focal y reflejar la localizacin
del tumor, aunque en ms de la mitad de los casos se produce una generalizacin secundaria y el inicio focal puede ser muy difcil de identificar en base a
criterios clnicos. Otros sntomas frecuentes de presentacin son la disfuncin cognitiva, las nauseas y vmitos, la disfuncin endocrina, los sntomas
visuales y los sntomas focales dependientes de la localizacin tumoral. DIAGNSTICO POR IMAGEN: La tcnica de neuroimagen de eleccin para el
diagnstico y seguimiento de los tumores del sistema nervioso es la resonancia magntica (RM). La tomografa computarizada (TC) presenta limitaciones
importantes en esta patologa, como son su peor resolucin anatmica y la presencia de artefactos de fosa posterior. A pesar de ello, la TC constituye
una tcnica muy til para la evaluacin de complicaciones hemorrgicas agudas, efecto masa y extensin del edema vasognico asociado al tumor. Es
importante recordar que la TC craneal puede aportar falsos negativos en el diagnstico inicial de un tumor cerebral, en situaciones como tumores
pequeos o con poca densidad celular, masa tumoral isodensa, ausencia de cambios patolgicos significativos en la barrera hematoenceflica, o
estabilizacin de la barrera hematoenceflica por uso previo de corticoides. Existe un conjunto de caractersticas radiolgicas que pueden orientar hacia
el grado de malignidad en la TC, como son: nmero, forma y tamao de las lesiones, localizacin, mrgenes del tumor, presencia de quistes,
calcificaciones o hemorragias, presencia de necrosis intratumoral, edema perilesional, desplazamiento de estructuras de la lnea media y captacin o no
de contraste la lesin. En la TC la imagen de un tumor puede ser hipo, iso o hiperdensa. Diagnstico anatomopatolgico y tcnicas de biologa molecular:
Una vez establecida la sospecha clnica y radiolgica, el diagnstico de certeza de una neoplasia cerebral requiere el estudio directo del tejido tumoral
obtenido mediante biopsia o reseccin quirrgica. El objetivo es confirmar el diagnstico de un tumor y establecer su naturaleza, tambin dar
informacin pronstica e incluso predictiva de respuesta al tratamiento. TRATAMIENTO: La ciruga suele ser el primer paso en el tratamiento de los
tumores primarios del sistema nervioso. En toda ciruga debe plantearse cul es el beneficio esperable, que deber compararse con el riesgo existente
para poder indicar la intervencin. En determinados tumores (meningiomas, neurinomas, gliomas grado I de la OMS) la ciruga puede ser curativa por s
misma si se extirpa totalmente la lesin. En gliomas de grados II, III y IV, la evidencia acerca del beneficio de la ciruga sobre la supervivencia es limitada,
pero hoy en da se acepta que los pacientes con una extirpacin amplia tienen mayor supervivencia. La radioterapia (RT) es un tratamiento til y eficaz
en mltiples neoplasias primarias y secundarias del sistema nervioso central. Se basa en la induccin de dao en el ADN y orgnulos de las clulas
malignas, con lo que consigue producir apoptosis y reduccin de la masa tumoral. Sin embargo, es necesario reducir al mximo el rea de parnquima
sano tratado, pues causa tambin desmielinizacin, dao neuronal y cambios vasculares en el tejido normal. La radiociruga permite tratar de forma muy
precisa volmenes pequeos con mnima afectacin del parnquima circundante. Sin embargo, la radiociruga tiene un papel muy limitado en el
tratamiento de los tumores primarios, siendo su principal indicacin las metstasis cerebrales. Quimioterapia y Agentes Biolgicos: se basa en tres
pilares fundamentales, que son la reseccin quirrgica, la radioterapia y la quimioterapia. El papel de esta ltima ha cambiado en los ltimos aos, al
pasar de su uso limitado en determinados tumores (meduloblastomas, tumores germinales) a formar parte la temozolomida del tratamiento estndar de
primera lnea del glioblastoma, el tumor primario cerebral ms frecuente. Adicionalmente, cada vez ms estudios muestran que temozolomida es activa
en otras neoplasias como los gliomas de bajo grado y los oligodendrogliomas. Adems de la temozolomida, otros frmacos quimioterpicos usados en el
tratamiento de los tumores cerebrales primarios incluyen, entre otros, las nitrosoureas (BCNU, CCNU), procarbazina, vincristina, derivados del platino,
metrotrexate y citarabina. Tratamiento sintomtico: Las crisis comiciales pueden aparecer hasta en el 40% de los casos en el momento del debut de la
enfermedad, y hasta en el 60% a lo largo de su evolucin. Todo paciente diagnosticado de un tumor cerebral que ha sufrido alguna crisis comicial debe
recibir tratamiento antiepilptico. Su uso de forma profilctica es sin embargo controvertido, y la recomendacin actual de la American Academy of
Neurology (AAN) es no iniciar tratamiento antiepilptico en pacientes con tumores cerebrales que no hayan presentado nunca crisis. En caso de ser
necesario tratamiento antiepilptico, deben seleccionarse frmacos con bajo potencial de interacciones (no inductores) y con un buen perfil de efectos
secundarios. El edema peritumoral contribuye significativamente en el deterioro clnico de los pacientes con tumores primarios y metstasis cerebrales.
El tratamiento fundamental del edema peritumoral son los corticoides, siendo el ms utilizado en este contexto la dexametasona, por su buena difusin
hacia el parnquima cerebral y su mnima actividad mineralocorticoide. Su uso debe estar guiado por la clnica del paciente, manteniendo siempre la
menor dosis posible para el control de los sntomas e iniciando su retirada progresiva posteriormente para evitar los efectos secundarios del tratamiento

CURSO ENARM CMN SIGLO XXI TEL: 36246001

Pharmed Solutions Institute

PGINA 113

MANUAL DE TRABAJO DEL CURSO ENARM CMN SIGLO XXI


prolongado con corticoides. La trombosis venosa profunda y el tromboembolismo pulmonar ocurren hasta en el 30% de los pacientes fuera del periodo
periquirrgico. Entre los factores que los favorecen se encuentran la disminucin de la movilidad de los miembros particos, decbito prolongado, uso
de quimioterapia, liberacin de factores procoagulantes por parte del tumor y el uso de tratamientos antiangiognicos. Se recomienda el uso de medias
de compresin elstica o de mecanismos de compresin mecnica secuencial a todos los pacientes tras reseccin quirrgica. Las heparinas de bajo peso
molecular son el tratamiento del evento tromboemblico, al haberse comprobado que el riesgo de complicaciones hemorrgicas es bajo. La fatiga, las
dificultades cognitivas y la depresin y ansiedad son tambin sntomas frecuentes que pueden tener un gran impacto en la calidad de vida del paciente.
CASO CLINICO Astrocitoma
Caso clnico. Paciente de 19 aos, masculino, consulta por crisis de
cefalea intermitente desde hace 5 meses, que aumentaron en
frecuencia e intensidad agregndose vmitos explosivos 3 das previo a
la consulta. Al examen neurolgico destac Glasgow 15, dismetra de
extremidad superior derecha y leve paresia facial izquierda. Se le
realiz una tomografa computada de encfalo que mostr tumor de
tipo astrocitoma e hidrocefalia. Se realiz reseccin del tumor y la
biopsia definitiva confirm el diagnstico preoperatorio y determin
bordes libres de tumor.
PREGUNTA
Cual es la sobrevida a 10 aos del caso clnico?
RESPUESTA
a.- Ms del 90%
b.- Ms del 80 %
c.- Ms del 70%
d.- Ms del 60 %.
CASO CLINICO Glioblastoma multiforme
Femenino de 60 aos de edad quien tiene el antecedente de
tiroidectoma total, cinco meses previos a su ingreso por ndulo
tiroideo reportado por histologa como hiperplasia multinodular
multifocal. Su padecimiento tena un ao de evolucin con cefalea
holocraneana persistente, hemiparesia corporal izquierda, as como
parestesias mismo lado; presentaba adems, cacosmia y dficit
campimtrico visual ipsilateral. La exploracin neurolgica demostr
funciones mentales superiores conservadas. Nervios craneales:
olfacin conservada, fondo de ojo sin papiledema, hemianopsia
homnima izquierda. Extremidades: hemiparesia corporal izquierda
4/5, reflejos exaltados y Babinski ipsilateral, marcha partica, no se
encontraron alteraciones sensitivas del orden interoceptivo,
propioceptivo o exteroceptivo; sin afeccin menngea o cerebelosa.
PREGUNTA
De acuerdo al cuadro clnico. Qu rea es ms probable que esta
afectada?
RESPUESTA
a.- Corteza
b.- Nucleos basales
c.- Cerebelo
d.- Puente
CASO CLINICO Ependimoma
Femenino de 59 aos con antecedente de dificultad para la marcha de
mas de 5 aos de evolucin, acompaada de dolor tipo radicular de
miembros inferiores, el dolor era intenso tanto en la region
sacrolumbar como en sus extremidades inferiores, que se
incrementaban con las maniobras de valsava. Desde el inicio de los
sntomas requiri de andarea para demabular, limitando la marcha a
pocos metros y presentndose un cuadro de urgencia esfentiriana
urinaria y fecal desde hacia 15 meses. Referia dos episodios de
empeoramiento, los cuales fueron tratados con AINES, corticoides e
inmunoglobulina sistmica, con minima mejora.
PREGUNTA
Cual es el diagnostico diferencia mas frecuente en este caso?
RESPUESTA
a.- Esclerosis multiple.
b.- Esclerosis lateral amniotrofica.
c.- Mielitis transversa.

CURSO ENARM CMN SIGLO XXI TEL: 36246001

d.- Sindrome raquimedular traumatico.


CASO CLINICO Meduloblastoma
Varn de 17 aos y 10 meses, que acudi a su mdico por cefalea,
vmitos y desorientacin. Muri 2 aos despus tras extensin
medular y metstasis sea. TC craneal con contraste i.v. en el que se
observa una masa hipercaptante del vermix que invade el 4 ventrculo
con pequeos focos qusticos/necrticos. RM axial en secuencia T2 y
sagital en T1 con contraste, en el que se observa una masa muy
hiperintensa con edema perilesional en T2, que se tie de manera
parcheada, comprime el 4 ventrculo generando intensa hidrocefalia
obstructiva.
PREGUNTA
Cual de las siguientes aseveraciones respuesto a los meduloblastomas
es menos probable?.
RESPUESTA
a.- La localizacin del meduloblastoma desmoplsico presenta ms
frecuentemente una localizacin fuera de la lnea media (50%).
b.- La hemorragia es un hallazgo de imagen frecuente en los
meduloblastomas, pero se presenta aproximadamente en el 10%
c.- Se ha descrito que el nico subtipo que presentaba ligeras
diferencias en su apariencia con RM era el desmoplsico, que
tiende a presentar edema marcado (75%).
d.- Se observan signos radiolgicos clsico de hidrocefalia obstructiva.
CASO CLINICO Meduloblastoma
Nia de 16 aos que acude por cefalea, vmitos, diplopia y sndrome
cerebeloso. TC con contraste en el que se objetiva una masa
hiperdensa de 4 cms en el vermix cerebeloso, con realce heterogneo
y focos qusticos de necrosis. Masa que comprime el tronco,
hiperintensa en T2 axial e hipointensa en T1 coronal con contraste, de
bordes bien definidos, con realce parcheado, y que genera dilatacin
ventricular.
PREGUNTA
Cual de las siguientes aseveraciones respecto a los meduloblastomas
es menos probable?
RESPUESTA
a.- La apariencia clsica en TC es una masa vermiana hiperdensa, bien
delimitada, con edema vasognico circundante ligero o moderado.
b.- Presencia de hidrocefalia y realce homogneo con contraste.
c.- La presencia de calcificaciones y reas necrtico/qusticas son
comunes.
d.- La hemorragia es el signo mas frecuente.
PREGUNTA
Cual de las siguientes aseveraciones respecto a los meduloblastomas
es menos probable?
RESPUESTA
a.- La apariencia tpica en RM es hipointensa en secuencias
potenciadas en T1.
b.- La aparencia tpica en RM Isointensa en secuencias potenciadas en
T2.
c.- Respecto a la substancia gris cerebral, observndose mayores
grados de heterogeneidad en la RM que en TC.
d.- Presentan captacin intensa del gadolinio de forma principalmente
uniforme.
CASO CLINICO Meningioma

Pharmed Solutions Institute

PGINA 114

MANUAL DE TRABAJO DEL CURSO ENARM CMN SIGLO XXI


Un hombre de 61 aos consulta por un cuadro progresivo de 3
semanas de evolucin de debilidad de extremidades inferiores e
inestabilidad de la marcha. No refiere haber presentado fiebre ni
perdida de peso, ni historia previa de incontinencia o disfuncin vesical
o anal, convulsiones o trauma. Sin embargo, tiene antecedentes de
abuso de cocana e hipertensin, tratada con lisinopril. Al examen
fsico, presenta signos vitales normales. Cuando se realiza el examen
neurolgico, se evidencia debilidad de ambas extremidades inferiores,
hiperreflexia, y debilidad muscular bilateral (puede hacer movimientos
en contra de la fuerza de gravedad y ante una leve resistencia).
Presenta marcha atxica. Se continu el estudio con una RMN de la
mdula espinal, que, en fase T1, muestra una lesin a nivel de de T8,
extradural y que comprime la mdula. La RMN de cerebro muestra, en
fase T1, mltiples lesiones homogneas. Al realizar una excisin de la
lesin medular, el paciente no vuelve a presentar debilidad muscular
en las extremidades inferiores.
PREGUNTA
Los meningiomas son los tumores benignos del sistema central ms
comunes, totalizando entre un 15 a 20% de todos los tumores
primarios cerebrales. Cual es la edad mas frecuente de presentacin?
RESPUESTA
a.- 10 a 20 aos.
b.- 20 a 40 aos.
c.- 40 a 60 aos.
d.- 60 a 80 aos.
CASO CLINICO Hemangioblastoma
Se trata de una paciente mujer de 58 aos que consult por una
cervicalgia crnica. La paciente present desde dos meses antes de la
consulta episodios de parestesias de miembros superiores que se
exacerbaban con las maniobras de valsalva. El examen fsico de la
paciente fue normal. Se le realiz resonancia magntica de columna
cervical, donde se observ una gran lesin qustica que se extenda
desde C2 hasta C3. Con el contraste se observ una captacin de un
ndulo en la superficie medular que estaba en estrecha relacin con el
quiste intramedular. La paciente fue llevada a ciruga, donde se le
realiz un abordaje por va posterior. Se le realiz laminectoma C2 y
C3, y al abrir la dura se encontr un ndulo tumoral vascular en la
pared medular. Con tcnicas microquirrgicas se hizo reseccin de la
lesin espinal medular, y al retirar la lesin se produjo drenaje del
quiste. En el postoperatorio la paciente present mejora de la
sintomatologa. El estudio de patologa report un hemangioblastoma
espinal.
PREGUNTA
Los tumores intramedulares de la columna espinal son raros, cuales su
frecuencia dentro de los tumores del sistema nervioso central?
RESPUESTA
a.- 1 a 2 %.
b.- 2 a 8 %.
c.- 8 a 10 %.
d.- 10 a 12 %.
PREGUNTA
Se pueden encontrar a lo largo de toda la mdula espinal, cual de los
siguientes es el mas frecuente?
RESPUESTA
a.- Ependimomas.
b.- Astrocitomas
c.- Hemangioblastomas
d.- Menigiomas.
CASO CLINICO Germinoma
Varn de 16 aos con historia de cefalea, episodios de prdida
transitoria del conocimiento de dos aos de evolucin asociadas a
prdida bilateral de la visin desde 4 meses. Como antecedente
familiar informa hermano mayor fallecido a los 19 aos con

CURSO ENARM CMN SIGLO XXI TEL: 36246001

enfermedad metastsica por Osteosarcoma de fmur. El TAC


demuestra tumoracin expansiva en ubicacin supraselar, que se
refuerza con el medio de contraste. Le realizan craneotoma para
reseccin biopsia de la tumoracin con remisin de muestra a
patologa con el diagnstico de glioma del quiasma.
PREGUNTA
Cual es la localizacin ms predominante de esta patologia?
RESPUESTA
a.- Pineal.
b.- Supraselar.
c.- Tlamo.
d.- Tercer ventrculo.
CASO CLINICO Adenoma pituitario
Una mujer de 52 aos consulta por pigmentacin de las uas de un
ao de evolucin. No presentaba sntomas sistmicos y era
inmunocompetente. Se le haba diagnosticado hace 12 aos un
macro-adenoma pituitario productor de ACTH, que haba sido tratado
con neurociruga endoscpica transnasal. Sin embargo, la ciruga no
produjo la cura, por lo que se realiz una adrenalectoma bilateral, y la
paciente reciba mineralo-corticoides y glucocorticoides. Al exmen
fsico, algunas de las uas de los dedos y de los pies presentaban una
banda de melanoniquia longitudinal, mientras que otras manifestaban
pigmentacin gris negruzca difusa. No se observaron cambios de color
en piel ni mucosas. L os estudios de laboratorio mostraron que los
niveles plasmticos de ACTH eran altos, de 100 pg/mL (normal 10-50
pg/mL). Las imgenes de resonancia magntica (MRI) confirmaron
vestigios de adenoma pituitario.
PREGUNTA
Cual de las siguientes afirmaciones relacionadas a la melanoquia es
menos probable?
RESPUESTA
a.- Las drogas ms comunes que causan melanoniquia son los agentes
quimioterpicos.
b.- Las enfermedades endcrinas que causan incremento de los niveles
circulantes de ACTH.
c.- Los melanomas son la principal causa de melanoquia.
d.- Frecuentemente los pinealomas no presentan melanoquia.
CASO CLINICO Prolactinoma
Paciente de 35 aos que presenta desde hace 10 aos amenorrea,
esterilidad, y galactorrea. A veces alteraciones visuales. Sensibilidad en
las mamas. Disminucin del inters sexual, dolor de cabeza,
infertilidad. Se le hizo un TAC y se hall un prolactinoma, al tratarla con
dopamina, (carbegolina) el microadenoma remiti.
PREGUNTA
Cual es el porcentaje de cambio de microadenomas a macroadenomas
productores de prolactina?
RESPUESTA
a.- 2 %.
b.- 3 %.
c.- 4 %.
d.- 5 %.
CASO CLINICO Craneofaringioma
Preescolar masculino de 2 aos y 11 meses de edad hospitalizado
como desnutrido severo marasmtico. Al momento de ingreso la
madre refera un padecimiento de dos meses de evolucin
caracterizado por vmitos post-prandiales persistentes, hiporexia,
disminucin subjetiva y progresiva de peso adems de compromiso del
estado general. Es producto de primer embarazo, nacido de trmino
por parto institucional sin complicaciones de una madre de 24 aos sin
antecedentes patolgicos de importancia, con llanto inmediato al
nacer y desarrollo psicomotor adecuado hasta el ao de edad, a partir
del cual se percibi falta de desarrollo del lenguaje receptivo y

Pharmed Solutions Institute

PGINA 115

MANUAL DE TRABAJO DEL CURSO ENARM CMN SIGLO XXI


expresivo adems de retraso del desarrollo motor caracterizado por
imposibilidad de ponerse de pie y caminar sin apoyo. Al examen fsico
de ingreso se encontraba en mal estado general y nutricional. El
indicador peso para la talla era menor a -3 desvos estndar (DE),
exista emaciacin visible y palidez mucocutnea generalizada.
Neurolgicamente llamaba la atencin una respuesta verbal
inapropiada, imposibilidad de bipedestacin y aparente afectacin
visual manifestada por imposibilidad del paciente de seguir objetos
con la mirada. Su permetro ceflico fue de 48 cm. (adecuado para la
edad) y las pupilas eran isocricas reactivas al estmulo luminoso. El
informe histopatolgico conrm el diagnstico de craneofaringioma.
PREGUNTA
Con respecto al diagnostico, cual de las siguientes aseveraciones en
menos probable?
RESPUESTA
a.- Tiene un leve predominio en el sexo femenino.
b.- Es un tumor de origen epitelial de la regin sellar que se forma a
partir de los restos embrionarios de la bolsa de Rathke.
c.- El tumor puede limitarse a la silla turca, o bien puede extenderse a
travs del diafragma sellar y comprimir la va ptica, la protuberancia o
el tercer ventrculo, produciendo hidrocefalia.
d.- Los craneofaringiomas representan entre el 5 y el 10% de todos los
tumores intracraneales que se presentan durante la niez.
CASO CLINICO Schwannoma
Mujer de 51 aos que padeca desde haca cuatro meses episodios de
repeticin de amaurosis fugax en su ojo izquierdo. Mediante
tomografa axial computerizada se diagnostic una masa orbitaria
circunscrita intraconal izquierda sospechosa de hemangioma
cavernoso. Se le practic una orbitotoma lateral izquierda
resecndose una masa rojiza encapsulada, sugestiva de hemangioma
cavernoso. Microscpicamente estaba compuesta por reas de clulas
fusiformes dispuestas en empalizada (Antoni tipo A), entremezcladas
con reas qusticas donde las clulas estaban rodeadas por una matriz
mixoide (Antoni tipo B).
PREGUNTA
Cual es la edad de presentacin mas frecuente de este padecimiento.
RESPUESTA
a.- entre 20 y 30 aos.
b.- entre 20 y 40 aos.
c.- entre 20 y 50 aos.
d.- entre 20 y 60 aos.
CASO CLINICO
Se trata de una mujer blanca, de 59 aos, que se presenta a nuestro
servicio con una historia de dificultad progresiva para la marcha de 5
aos de evolucin, acompaada de dolores radiculares en miembros
inferiores. El dolor era intenso tanto en la regin sacrolumbar como en
sus extremidades inferiores, incrementndose con las maniobras de
Valsalva. Desde el inicio de los sntomas requiri de andadores para
poder deambular, limitndose la marcha a pocos metros y
presentndose un cuadro de urgencia vesical y fecal durante 15 meses.
Refera dos cuadros posteriores de empeoramiento de su cuadro
clnico, lo cuales se instauraron en 45 semanas, presentndose
mejora ligera en ambos casos con el tratamiento. Debido a estas
manifestaciones acude al consultorio mdico, recibiendo tratamiento
son esteroides y antiinflamatorios. La paciente recibe varios
tratamientos durante dos aos con esteroides e Inmunoglobulina
intravenosa, refiriendo mejora transitoria de los sntomas tanto
motores como sensitivos; la enfermedad contina progresando y
acude a nuestro servicio solicitando una nueva valoracin.
PREGUNTA
Cul es la conducta diagnostica ms adecuada en este momento?
RESPUESTA

CURSO ENARM CMN SIGLO XXI TEL: 36246001

a.- Tomografa
b.- Resonancia magntica
c.- Radiografa
d.- Electromigrafia
PREGUNTA
En estudio realizado aparece una masa larga, homognea, que inclua
cono y cauda equina, causando expansin del canal raqudeo y
produciendo un aspecto festoneado, de los mrgenes posteriores de
los cuerpos vertebrales Cul es el diagnstico ms probable para este
caso?
RESPUESTA
a.- Poliradiculoneuropatia desmielinizante
b.- Glioma
c.- Ependimoma
d.- Meningioma
PREGUNTA
Cul es la causa ms probable del cuadro clnico de la paciente?
RESPUESTA
a.- Compresin tumoral
b.- Edema secundario
c.- Trastorno de circuitos corticales
d.- Afeccin a corteza
CASO CLINICO
Un hombre de 61 aos consulta por un
cuadro progresivo de 3 semanas de
evolucin de debilidad de extremidades
inferiores e inestabilidad de la marcha. No
refiere haber presentado fiebre ni prdida
de peso, ni historia previa de incontinencia
o disfuncin vesical o anal, convulsiones o
trauma. Sin embargo, tiene antecedentes de abuso de cocana e
hipertensin, tratada con lisinopril. Al examen fsico, presenta signos
vitales normales. Cuando se realiza el examen neurolgico, se
evidencia debilidad de ambas extremidades inferiores, hiperreflexia, y
debilidad muscular bilateral (puede hacer movimientos en contra de la
fuerza de gravedad y ante una leve resistencia). Presenta marcha
atxica. No tiene evidencia de dficit sensorial, de pares creaneanos o
dismetra. Se le realizan exmenes de laboratorio, sin hallazgos
patolgicos. Se continu el estudio con RMN de cerebro muestra, en
fase T1, mltiples lesiones homogneas ver imagen.
PREGUNTA
Cul es el diagnstico ms probable para este caso?
RESPUESTA
a.- Ependimoma
b.- Glioma
c.- Meningioma
d.- Oligodendroglioma
PREGUNTA
Cul es la localizacin ms frecuente de estos tumores?
RESPUESTA
a.- Supratentoriales
b.- Infratentoriales
c.- Conducto raqudeo
d.- Tallo cerebral
PREGUNTA
Cul es el origen ms probable de estos tumores?
RESPUESTA
a.- De oligodendroglia
b.- clulas del aracnoides
c.- Races nerviosas
d.- Clulas de duramadre

Pharmed Solutions Institute

PGINA 116

MANUAL DE TRABAJO DEL CURSO ENARM CMN SIGLO XXI


INFECCIONES DEL SISTEMA NERVIOSO CENTRAL
CIENCIAS BASICAS: Constituyen una emergencia mdica. Su ubicacin anatmica en un espacio seo sellado que no permite una expansin fcil ante un
proceso inflamatorio difuso contribuye a que las altas posibilidades de dao neurolgico ocurran por efecto
mecnico, como en los sndromes de herniacin. Otros factores como la competencia inmunolgica de cada
individuo, la penetracin y concentracin de los agentes antimicrobianos en el sistema nervioso, la edad y las
dificultades diagnsticas contribuyen en la evolucin. La edad de aparicin ms frecuente de meningitis desde
la infancia hasta la segunda y tercera dcada. Por otro lado ha aumentado la proporcin de infecciones
nosocomiales. El lquido cefalorraquideo (LCR) es el pilar fundamental en el diagnstico de la mayora de las
infecciones del SNC. Las caractersticas iniciales del anlisis citoqumico pueden orientar a un diagnstico especfico aunque se requiere la confirmacin
de ste por otros medios. Sin embargo una buena correlacin clnica inicial es suficiente para tomar una conducta teraputica. En general las
caractersticas tpicas de los diferentes tipos de infeccin menngea de acuerdo con el citoqumico se muestran en la Tabla. Los cultivos y el Gram del LCR
continan siendo los exmenes de eleccin en las meningitis bacterianas. La
administracin de antibiticos orales antes de la puncin lumbar no altera
ninguno de los parmetros (conteo celular, protenas, glucosa) en el LCR pero
s puede disminuir el porcentaje de neutrfilos y la probabilidad de resultados
positivos en el cultivo o el Gram. El Gram tiene una sensibilidad cercana a 6090% y una especificidad de 100%. Los hemocultivos identifican el germen
causal en 80% de los casos de neumococo, 90% de los de meningococo y 94%
en los casos de Haemophylus influenza. Las pruebas de ltex para
identificacin de antgenos de los grmenes son rpidas requiriendo entre 10
a 15 minutos para realizarlas y sin un entrenamiento riguroso. Adems tienen
la ventaja de tener pocos falsos positivos. Por ejemplo, para criptococo puede
haber falsos positivos ante la presencia de factor reumatoideo. En el caso de
meningitis por Histoplasma capsulatum, las pruebas son muy sensibles pero la
especificidad es baja por tener reactividad cruzada con criptococo, candida y
Coccidiodes inmitis. La identificacin de anticuerpos especficos en el LCR es de gran utilidad. En la neurosfilis un diagnstico definitivo se obtiene con la
demostracin de positividad del VDRL en el LCR. Una prueba reactiva en cualquier ttulo es diagnstica de neurosfilis. Aunque la prueba es muy
especfica, la sensibilidad sin embargo vara entre 30-70% y en muchos casos el diagnstico se basa en la presencia de pleocitosis o protenas elevadas en
el LCR en un paciente con VDRL reactiva en sangre. En los pacientes con VIH las dificultades nacen debido a que puede haber pleocitosis con aumento de
protenas en el LCR debido a la neurosfilis o por la misma infeccin por el VIH. En estos casos tiene mucha utilidad el FTA-ABS (Treponemal antibodyabsortion test) y el test de hemoaglutinacion para Treponema pallidum (MHA-TP) los cuales son muy sensibles para el diagnstico de neurosfilis. Un
resultado no reactivo descarta el diagnstico de neurosfilis en los pacientes con infeccin por VIH. La PCR es la prueba ms til para el diagnstico de
meningitis y encefalitis viral. Adems tiene la ventaja de cuantificar el cido nucleico en las muestras de LCR con lo cual se puede determinar la
progresin de la enfermedad y la respuesta al tratamiento. La prueba se fundamenta en realizar mediante un sistema trmico cclico una copia y
amplificacin de hasta un milln de veces del contenido de ADN del germen presente en el LCR. La adenosin deaminasa (ADA) es una enzima que est
asociada con enfermedades que producen una respuesta inmunolgica celular; es de mucha ayuda para el diagnstico de meningitis TB. Sin embargo,
pueden observarse resultados positivos de esta prueba en linfomas con compromiso menngeo, sarcoidosis, neurobrucelosis y hemorragia
subaracnoidea. Tambin puede haber falsos negativos. Los diferentes estudios muestran una sensibilidad y especificidad para el diagnstico de 90% con
ttulos mayores de 10 unidades internacionales por litro (UI/L) siendo los ttulos de corte entre 5-10 UI/L. Los resultados de la ADA pueden elevarse en
las dos primeras semanas de tratamiento. MENINGITIS: Presencia de inflamacin menngea originada por la reaccin inmunolgica del husped ante la
presencia de un germen patgeno en el espacio subaracnoideo. La meningitis es el sndrome infeccioso ms importante del sistema nervioso central. El
compromiso parenquimatoso adyacente a las meninges definir la presencia de meningoencefalitis (encfalo), meningoencefalomielitis (encfalo y
mdula), meningomielorradiculitis (encfalo, mdula y races nerviosas). Los diferentes tipos de meningitis tienen diferente origen y los patgenos
responsables en la mayora de los casos son predecibles, lo cual permite en muchas situaciones clnicas el inicio de una terapia emprica mientras se
obtiene la confirmacin del germen involucrado. La identificacin de grmenes que pueden tener influencia de tipo endmico es de importancia para las
medidas profilcticas y epidemiolgicas en determinadas poblaciones (por ej: meningococo). Los diferentes tipos de meningitis se pueden definir de
acuerdo con el perfil clnico, hallazgos de LCR y la etiologa. MENINGITIS ASPTICA: Por lo general es de curso benigno y en la mayora de los casos de
etiologa viral, puede haber causas no infecciosas. Tiene un curso clnico bifsico. Primero preceden a su aparicin los sntomas o signos de una afeccin
viral respiratoria, gastrointestinal o en la piel, presentndose posteriormente la fase menngea con sntomas ms especficos como cefalea, fiebre y
signos menngeos. Son poco frecuentes los sntomas de compromiso enceflico como convulsiones o alteracin del estado mental. En algunos pases
tiene predileccin estacional y es ms frecuente en nios. Los enterovirus son los principales agentes causales (entre 55-75%) de los casos de meningitis
asptica y hasta en 95% de los casos cuando se identifica el patgeno. El virus de la parotiditis fue considerado en alguna ocasin responsable de muchos
casos de meningitis asptica pero la incidencia ha declinado desde el uso de la vacuna; sin embargo una meningitis oculta se presenta en ms de la mitad
de los casos de parotiditis y llega a ser sintomtica en solo 30%. Mas an, puede ocurrir la meningitis sin parotiditis. Los herpes virus raramente causan
meningitis; sin embargo el virus del herpes simple (VHS) es el responsable de 1 a 3% de todos los casos de meningitis asptica. De los dos tipos de herpes
simple (VHS 1 oral - VHS 2 genital) el VHS 2 produce meningitis en 11% a 33 % de las personas en el momento de la infeccin genital primaria. El VIH
puede causar una meningitis asptica principalmente en el perodo de la infeccin primaria y durante la seroconversin pero incluso puede producirla
muy temprano en las fases iniciales de la infeccin por VIH. Raramente la tuberculosis se puede comportar como una meningitis asptica autolimitada.
La PCR es de gran utilidad con una especificidad del 100% y el resultado es disponible en pocas horas. El manejo es prcticamente de soporte y de alivio
de los sntomas. MENINGITIS SPTICA (bacteriana) implican una mayor morbi-mortalidad y es una emergencia neurolgica. La tasa general de
mortalidad es de 25% y de morbilidad hasta 60%. Presentacin clnica: los sntomas clsicos de presentacin incluyen cefalea, fiebre, escalofros,
alteracin de la esfera mental y la presencia de meningismo. En los neonatos predominan los sntomas inespecficos como irritabilidad, vmito, letargia,
dificultad respiratoria y sntomas gastrointestinales, siendo el meningismo poco frecuente. Los ancianos presentan con mayor frecuencia alteracin en el
estado mental asociado a fiebre; la presencia de cefalea y meningismo resulta extica. La meningitis por meningococo tiene frecuentemente una
presentacin dramtica y puede progresar como una enfermedad fulminante muriendo el paciente en pocas horas. Muchas veces los pacientes tienen
un rash cutneo petequial en el tronco y las extremidades inferiores. Este puede, por contigidad de las lesiones, formar reas extensas de equimosis. La
prpura fulminante es una forma grave de manifestacin de la sepsis por meningococo; es causada por coagulacin intravascular diseminada y en
muchas ocasiones causa necrosis distal en las extremidades. Los marcadores de riesgo incrementado de muerte en los pacientes con meningitis por

CURSO ENARM CMN SIGLO XXI TEL: 36246001

Pharmed Solutions Institute

PGINA 117

MANUAL DE TRABAJO DEL CURSO ENARM CMN SIGLO XXI


meningococo son: la presencia de ditesis hemorrgica, signos neurolgicos focales y personas mayores de 60 aos. En general muchos sndromes
neurolgicos diferentes al meningismo pueden ser la forma de presentacin de las meningitis bacterianas. La presencia de signos neurolgicos focales
por compromiso cerebrovascular, hipertensin endocraneana, alteracin en pares craneales, crisis convulsivas y estado confusional son algunos de ellos.
Tratamiento: El LCR es el principal soporte diagnstico en la escogencia de la terapia adecuada, pero en muchas ocasiones la puncin lumbar debe
diferirse en aquellos casos de pacientes en estupor o coma, con signos neurolgicos focales y ante la presencia de crisis convulsivas. En estos casos las
neuroimgenes estn indicadas inicialmente para descartar complicaciones asociadas con la meningitis o hacer un diagnstico diferencial. Una
escanografia de crneo (simple) normal permite una mayor seguridad en la realizacin de la puncin lumbar en estos casos y evitar as complicaciones.
Cabe anotar que ante la sospecha diagnstica debe iniciarse una terapia emprica mientras se realiza la puncin lumbar. La terapia antibitica no variar
significativamente el LCR en las primeras 48 horas. La terapia emprica se escoger de acuerdo con los grupos de edad, la sensibilidad antibitica de los
grmenes. El Gram puede ser una gua inicial de utilidad pero no es tan eficaz en todos los casos; las pruebas inmunolgicas son de gran ayuda para un
diagnstico especfico y rpido del germen. El Streptococcus pneumoniae (neumococo) hoy por hoy es el principal germen patgeno en la meningitis
bacteriana en cualquier grupo de edad. Penicilina cefalosporina con vancomicina. El Haemophylus influenzae es un coco bacilo gramnegativo. El nico
husped natural de este germen es el humano y la transmisin persona-persona ocurre por la va respiratoria. El trauma de crneo reciente, la ciruga
neurolgica previa, las sinusitis paranasales, las otitis media y las fstulas del LCR son los factores de riesgo ms importantes en adultos para padecer una
meningitis por H. influenzae. El tratamiento antibitico es ceftriaxona cefotaxima, por lo tanto son los frmacos de eleccin. La Listeria monocitogenes
es un germen beta-hemoltico facultativamente anaerobio y grampositivo. Los grupos de edad ms frecuentemente involucrados son los neonatos
menores de un mes y los adultos mayores de 60 aos. En este grupo la listeria es la causa de 20% de las meningitis. Los principales factores de riesgo son
el embarazo, inmunosupresin y edad avanzada. El tratamiento de eleccin es la ampicilina o la penicilina por un tiempo de cuatro semanas en los
pacientes inmunosuprimidos. En los pacientes alrgicos a las penicilinas el trimetropim-sulfa-metoxazol es la alternativa ms recomendada. El
Estreptococo del Grupo B es la causa ms frecuente de sepsis neonatal y es una causa importante de infeccin bacteriana invasiva en los adultos. Es el
germen causal en 70% de los casos de meningitis bacteriana en los neonatos menores de un mes, mientras que en adultos es responsable de menos de
5%, es altamente susceptible a la penicilina y a la ampicilina siendo las drogas de primera eleccin. Muchos recomiendan el uso concomitante con
gentamicina para el tratamiento de la meningitis neonatal. La Neisseria meningitidis (meningococo) es una bacteria gramnegativa encapsulada que
aparece en pares en el extendido. Generalmente coloniza la nasofaringe de manera asintomtica y la transmisin ocurre de persona a persona mediante
secreciones respiratorias. Los individuos esplenectomizados y con deficiencias de
complemento tienen un alto riesgo de infeccin por meningococo. Causa 60% de las
meningitis bacterianas entre la poblacin de 2 a 18 aos de edad y va declinando su
frecuencia hasta 5 % en los mayores de 60 aos. Es el responsable en 1/3 de los casos
de meningitis en menores de 2 aos. El frmaco de eleccin es la penicilina o la
ampicilina. La ceftriaxona y el cefotaxima tienen tambin una excelente respuesta y
alcanzan buenas concentraciones en el LCR. Los contactos cercanos en pacientes con
meningococemia tienen un riesgo significativamente alto para desarrollar la infeccin
semanas despus de la exposicin. MENINGITIS TUBERCULOSA: Su epidemiologa ha
cambiado significativamente aun en pases en donde la incidencia haba disminuido de
manera importante. Este hecho se explica por la epidemia reciente de infeccin por
VIH y por la resistencia que ha desarrollado el bacilo a las terapias convencionales.
Manifestaciones clnicas: con fines pronsticos se reconocen tres estadios de la
enfermedad. a) Estadio 1: el paciente est consciente y no presenta signos neurolgicos focales. b) El estadio dos: el paciente est confuso pero no est
en coma, y tiene signos neurolgicos focales como hemiparesia o parlisis de pares craneales. c) Estadio tres: el paciente se encuentra en coma o en
estupor; tiene compromiso mltiple de pares craneales, hemipleja o parapleja. En los nios es ms frecuente identificar la fase inicial de infeccin a
nivel respiratorio, mientras que en el adulto en muchos casos no se puede identificar y puede pasar mucho tiempo desde la infeccin inicial y la aparicin
del compromiso del sistema nervioso. Es frecuente la presencia de hiponatremia en adultos encontrndose en 45% de los casos y se debe a una
secrecin inadecuada de hormona antidiurtica (SIDHA). En los nios aunque pueden manifestarse tambin la cefalea, fiebre y meningismo, es comn la
presencia de hidrocefalia como primera manifestacin de la infeccin por TBC. Diagnstico: el diagnstico recae principalmente en el LCR. La pleocitosis
a expensas de linfocitos, las protenas altas y la glucorraquia significativamente baja son los hallazgos ms frecuentemente encontrados. En algunos
casos hay un predominio polimorfonuclear en las fases iniciales de la infeccin, que tambin suele observarse cuando se inicia el tratamiento. Este
ltimo coincide con el empeoramiento clnico que se observa en algunos pacientes cuando se inicia la terapia y es muy caracterstico de la meningitis
TBC. Por lo general hay un viraje posterior al predominio de linfocitos pero en algunas ocasiones puede persistir el predominio de polimorfonucleares
denominndose meningitis neutroflica persistente. Se ha encontrado un mayor contenido de protenas en el estadio tres de la enfermedad. Como ya se
coment anteriormente el cultivo y los extendidos para BK tienen una mayor probabilidad de positividad cuando se estudian muestras repetidas en altos
volmenes. La ADA y la PCR recientemente han sido de gran ayuda para un diagnstico ms rpido y en los casos de cultivos negativos. Las
neuroimgenes brindan apoyo en el diagnstico. En la RMC se puede observar el compromiso de las meninges basales que realzan al inyectar el
gadolinio; los sitios ms comprometidos son la fosa interpeduncular, la cisterna ambiens y la regin quiasmtica. Este realce es ms frecuentemente
observado en los pacientes VIH positivos. Las neuroimgenes tambin son de ayuda para el diagnstico de hidrocefalia, la presencia de vasculitis por la
infeccin y para caracterizar los granulomas. Complicaciones: Algunas de ellas son de tipo cerebrovascular por compromiso de las arterias intracraneales
debido a una panarteritis, producto de la infiltracin de las paredes de los vasos por el exudado inflamatorio. Lo anterior produce infartos cerebrales. El
territorio carotdeo es el ms frecuentemente comprometido. Una complicacin metablica que puede agravar el cuadro clnico es la hiponatremia por
SIDHA; sta debe corregirse rpida y lentamente para evitar el riesgo de mielinolisis pntica. Se han informado casos de siringomielia muchos aos
despus de la infeccin inicial. Tratamiento: va orientado a eliminar las formas intra y extracelulares del bacilo. Resultan importantes dos factores para
evitar la resistencia: el primero, utilizar mltiples frmacos y el segundo la adherencia al tratamiento. Por lo general el inicio del tratamiento es emprico.
La isoniacida y la pirazinamida son bactericidas y penetran las meninges inflamadas o no inflamadas alcanzando buenas concentraciones bactericidas. Las
concentraciones que alcanzan en el LCR la estreptomicina intramuscular, la rifampicina y el etambutol apenas sobrepasan las concentraciones
inhibitorias mnimas para la micobacteria; adems no penetran las meninges no inflamadas. Mientras que la rifampicina es bactericida, el etambutol y la
estreptomicina son tuberculostticos. Se aceptan dos tipos de rgimen: uno corto de seis meses, con cuatro frmacos. En los primeros dos meses:
isoniacida 300 mg, rifampicina 600 mg, pirazinamida 1,5 gr al da y estreptomicina 1g IM (500 mg en ancianos o con peso menor de 50 kg). Luego un
perodo de cuatro meses con isoniacida y rifampicina dos veces por semana o diario. En los nios este segundo perodo debe ser de 10 meses. En los
casos de resistencia se debe incluir otro medicamento como etambutol en vez de estreptomicina y seguir luego con dos drogas (isoniacida y rifampicina)
por 9 a 18 meses. MENINGITIS POR CRIPTOCOCO: En general todas las meningitis por hongos se consideran que ocurren en pacientes

CURSO ENARM CMN SIGLO XXI TEL: 36246001

Pharmed Solutions Institute

PGINA 118

MANUAL DE TRABAJO DEL CURSO ENARM CMN SIGLO XXI


inmunosuprimidos. La va de entrada en la mayora de los casos es a travs de las vas respiratorias superiores, ingresando en forma de levadura en el
alvolo, all puede producir un proceso inflamatorio que puede ser completamente asintomtico o producir sntomas, la presentacin clnica de las
meningitis por hongos en general es insidiosa, de instauracin subaguda o crnica; se presenta cefalea, fiebre, escalofros, alteracin del estado general y
cambios mentales en un paciente por lo general con factores de riesgo para adquirir la infeccin (SIDA, cncer, terapia inmunosupresora, trasplante de
rganos). Diagnstico: requiere de sospecha clnica en los pacientes con riesgo y recae principalmente en el aislamiento del criptococo en el LCR. El LCR
muestra generalmente un aumento moderado de la presin; es de aspecto claro y hay pleocitosis que puede alcanzar desde 20 hasta 1.000 clulas por
milmetro cbico. El predominio celular es de linfocitos, la glucosa es frecuentemente baja (alrededor de 30 mg/dl) y las protenas son elevadas entre 50
a 1.000 mg/dl; la tinta china puede mostrar la presencia del criptococo. La prueba debe hacerse con un control negativo de agua y as estar seguro que la
tinta no est infectada por espora. La prueba serolgica de ltex en el LCR es muy sensible para el diagnstico; se puede cuantificar y es de gran ayuda en
el seguimiento. Adems es rpida y de fcil interpretacin. Tratamiento: Primera opcin anfotericina B a la dosis de 0,7 mg/kg/da ms 5 flucytocina a la
dosis de 25 mg/kg cada seis horas por va oral hasta que los cultivos sean negativos o haya mejora clnica (entre cuatro a seis semanas). Posteriormente
se debe continuar una terapia de mantenimiento con fluconazol por ocho o diez semanas; pero si el paciente es inmunosuprimido (SIDA, trasplantado,
cncer, etc.) esta terapia de mantenimiento es de por vida.
CASO CLINICO Listeria monocytogenes
Paciente de 30 aos, sexo masculino, en tratamiento con prednisona
por Lupus Eritematoso Sistmico, que comenz con un cuadro de
aproximadamente dos semanas de evolucin caracterizado por
compromiso progresivo del estado general, cefalea holocraneana de
intensidad progresiva, fiebre y diaforesis vespertina por lo que
consult. Se le realiz puncin lumbar, obtenindose LCR de aspecto
opalescente, cuyo estudio citoqumico fue compatible con meningitis
bacteriana; el cultivo demostr desarrollo de un bacilo Gram (+)
pequea, cuya identificacin microbiolgica confirm Listeria
monocytogenes.
PREGUNTA
Cual es la conducta teraputica mas apropiada a seguir?
RESPUESTA
a.- Cefotaxima mas vancomicina.
b.- Ampicilina mas gentamicina.
c.- Trimetroprima mas sulfametoxazol y ampicilina.
d.- Gentamicina mas imipenem.
CASO CLINICO L. M
Paciente de 87 aos, sexo masculino, hipertenso, portador de
marcapasos definitivo, adems de multiinfarto cerebral y bronquitis
crnica. Ingres con historia de tos productiva y fiebre de
aproximadamente 1 semana de evolucin, agregndose
posteriormente desorientacin progresiva y franco compromiso de
conciencia que motiv hospitalizacin. Desde su ingreso, muy grave,
en sopor, con signos menngeos esbozados y neumona confirmada
tanto clnica como radiolgicamente. Se manej con antibioticoterapia
agresiva y de amplio espectro para neumona adquirida en la
comunidad con sospecha de aspiracin; a pesar de lo anterior
evoluciona a la falla orgnica mltiple.
PREGUNTA
Cual es la conducta teraputica mas apropiada a seguir?
RESPUESTA
a.- Cefotaxima mas vancomicina.
b.- Ampicilina mas cefotaxima.
c.- Ampicilina mas levofloxacina.
d.- Gentamicina mas imipenem.
CASO CLINICO Menigitis Aseptica
Un varn de 56 aos de edad, con antecedentes de meningitis
criptogena y meningitis asptica que presenta hace 10 dias fiebre,
escalofros, rigidez en el cuello y dolor de cabeza. El EF temperatura
36.9 c, FR de 121 lpm. Pareca incmodo, pero sin rigidez de nuca
franca. LCR revelo eritrocitos 34 clulas/mm3, 154 clulas/mm3,
leucocitos (77 % de neutrfilos , 12 % de linfocitos, histiocitos 10 % ), la
glucosa de 63 mg / dl , la protena de 82 mg / dl. TAC de la cabeza
revela estable, leve prominencia difusa del sistema ventricular. El
paciente es alrgico a la penicilina.
PREGUNTA

CURSO ENARM CMN SIGLO XXI TEL: 36246001

Cual es la conducta teraputica mas apropiada a seguir?


RESPUESTA
a.- Cefotaxima mas vancomicina.
b.- Meropenen mas aciclovir.
c.- Amikacina mas levofloxacina.
d.- Tratamiento sintomatico.
CASO CLINICO Meningitis por criptococo
Varn de 51 aos, ingres por cefalea de 4 semanas y somnolencia. La
EF con deterioro neurolgico con 10 de Glasgow. Se realiz una
puncin lumbar (PL): 40clulas/mm3, el 95% mononucleares (MN);
glucosa: 4mg/dl, protenas: 0,68mg/l. En la tincin de tinta china se
apreciaron clulas levaduriformes. El ttulo de antgeno criptoccico en
plasma era superior a 1/256 y en lquido cefalorraqudeo (LCR),
superior a 1/1.024. No se midi la presin intracraneal. En el cultivo de
LCR se aisl C. neoformans, se inici tratamiento con anfotericina B i.v.
a dosis de 4mg/kg/da y 5 flucitosina oral a dosis de 100mg/kg/da. El
paciente desarroll sordera bilateral como complicacin. A los 12 das
reingres con afectacin de VII par craneal izquierdo.
PREGUNTA
Cual es la conducta teraputica mas apropiada a seguir?
RESPUESTA
a.- Anfotericina B iv.
b.- Meropenem i.v. a dosis de 1g/8h.
c.- Fluconazol 400mg/da/VO.
d.- Linezolid i.v. a dosis de 600mg/12h.
CASO CLINICO Streptococcus pneumoniae
Agricultor, del sexo masculino, de 64 aos, admitido en Urgencias con
un cuadro de fiebre baja, mialgias, artralgias e hiporexia, haca ya tres
semanas y evolucionando haca ya tres das con tos, disnea y
compromiso del estado general. A su llegada, estaba taquipneico,
depletivo, con vmitos, confusin mental y agitacin. Presentaba de
rigidez cervical terminal. Estertor crepitante en la base del hemitrax
derecho. Soplo diastlico en foco mitral +2/+6, sin irradiacin y soplo
sistlico en foco artico +3/+6, con irradiacin cervical. Leucograma de
18.700 cels/mm y 6% de bastones. La radiografa de trax mostr
opacidad alveolar en la base derecha. El paciente evolucion con
insuficiencia respiratoria aguda, siendo necesario la realizacin de
intubacin orotraqueal y ventilacin mecnica. Fue realizada
tomografa computadorizada (TC) de crneo que no arroj
alteraciones, y la puncin lumbar con lquido cefalorraqudeo
presentando 78 cels/mm, siendo 87% polimorfonucleares,
hipoglicorraquia.
PREGUNTA
Cual es la conducta teraputica mas apropiada a seguir?
RESPUESTA
a.- Ceftriaxone mas vancomicina.
b.- Meropenen mas aciclovir.
c.- Amikacina mas levofloxacina.
d.- Bencilpenicilina benzatinica.

Pharmed Solutions Institute

PGINA 119

MANUAL DE TRABAJO DEL CURSO ENARM CMN SIGLO XXI


CEFALEAS Y DOLOR CRANEO-FACIAL
SALUD PUBLICA: Mas comn prevalencia en las mujeres, 18%, hombres, 6%. Edad de inicio promedio 19 aos. 46 % tienen antecedentes familiares. El
riesgo de que un nio desarrolle migraa 70% si ambos padres estn afectados, el 45%, cuando uno de los padres afectados. Una condicin hereditaria
dominante rara, migraa hemipljica familiar, debido a una mutacin en el cromosoma 19 que codifica para una subunidad del canal de calcio
dependiente de voltaje. La arteriopata cerebral autosmica dominante con infartos subcorticales y leucoencefalopata (CADASIL) puede presentarse con
migraa hemipljica y progreso a una encefalopata isqumica. PATOGENIA: La migraa es un trastorno neurovascular en un individuo genticamente
predispesto. Predisposicin es una inestabilidad dentro de la red trigminovascular originada en el tronco cerebral, en particular, el mesencfalo dorsal
y dorsolateral puente de Varolio. Proyecciones difusas del locus ceruleus y de la corteza cerebral resultan en deterioro del flujo sanguneo cortical
cerebral que causa la propagacin de la depresin asociada con auras migraosas. DIAGNOSTICO: Clnico; La migraa es una cefalea episdica
generalmente asociada con nuseas ( vmitos) y fotofobia. Puede estar precedida por sntomas neurolgicos focales. El aura no necesariamente puede
ser seguida por dolor de cabeza (anteriormente conocido como equivalentes de migraa). 30 % puede tener otros dolores de cabeza que coexisten, por
ejemplo tensin y dolores de cabeza por uso excesivo analgsicos. Caractersticas de dolor de cabeza: Unilateral en 2/3 de los pacientes y bilateral en
1/3. Dolor que se siente detrs o a lo largo del ngulo interno del ojo o regiones frontotemporal. Irradia a espalda, occipucio o el cuello. Sitio de dolor de
cabeza puede ser tanto ipsilateral o contralateral a la perturbacin neurolgica focal. Algunos pacientes pueden quejarse de dolor del miembro
ipsilateral del lado del dolor de cabeza. Carcter del dolor de cabeza es aburrido en el inicio y ms tarde palpitante (que aumenta con cada pulsacin).
Otros pacientes slo pueden describir un dolor de cabeza constante, o incluso un ligero dolor de cabeza embotada. Empeora con el movimiento.
Caractersticas Aura: Auras visuales incluyen: alucinaciones visuales, escotomas, y espectros de fortificacin (zig-zag lneas se asemejan a una muralla
cuando se ve desde arriba). Por lo general, blanco y el brillo se mueven a travs del campo visual, dejando un rea de discapacidad escotomas
centellantes. Otros fenmenos visuales incluyen destellos de luz (fotopsia). Nota: la epilepsia del lbulo occipital provoca alucinaciones que son
circulares o de formas geomtricas y colores. Auras sensoriales son, por ejemplo parestesias generalmente positiva en lugar de adormecimiento, que se
reparten en minutos u horas (5 %). Otras auras: hemiparesia (de minutos u horas), disfasia, alucinaciones olfativas y gustativas, y la distorsin de las
partes del cuerpo, tales como inflamacin de la lengua. Disparadores de migraa: El estrs y la relajacin despus de estrs. Falta o el exceso de sueo
acostumbrados. Trauma (especialmente en nios).Estimulacin sensorial: reflejos, luces parpadeantes, los olores. Los hbitos alimentarios y la
alimentacin: falta de una comida (hipoglucemia). Los alimentos como el vino tinto, queso, chocolate. Aditivos alimentarios: el glutamato monosdico.
Ejercicio. El exceso de calor y la deshidratacin. Drogas: vasodilatadores como la nitroglicerina. Variantes de migraa: Migraa vertebrobasilar; sntomas
del tronco cerebral: diplopa, vrtigo, incoordinacin, ataxia y disartria se producen en posteriores ataques de migraa. Tambin puede ser desmayos o
prdida de consciencia debido a la participacin de la formacin reticular del cerebro medio. En casos severos un estado de estupor o coma puede durar
una semana (estupor migraa). La mayora de los casos estn asociados con otros sntomas vertebrobasilar. Migraa oftalmopljica: Paresia extra-ocular
el tercer nervio es el ms afectado. Paresia puede durar das o semanas. Excluir una lesin compresiva tales como aneurisma de la arteria comunicante
posterior. Migraa retiniana: Resultado de la constriccin de las arteriolas retinianas, daa la visin en un ojo y se asocia con dolor de cabeza por detrs
del mismo ojo. Vrtigo benigno recurrente. Migraa que tiene anormalidades del sistema vestibular. Ataques de vrtigo acompaadas por tinnitus,
sordera y cefalea que puede responder con terapia antimigraosa. Criterios abreviados de Sociedad Internacional de Cefaleas (IHS) para la migraa:
Migraa sin aura; a) Dolor de cabeza una duracin de 4 horas hasta 3 das. b) Nuseas/vmitos y/o sensibilidad a la luz y el ruido. c) Dos de los siguientes:
dolor unilateral dolor moderado o grave intensidad, agravacin por la actividad fsica sencilla, dolor palpitante. Migraa con aura; Al menos 3 de los
siguientes a) tronco cerebral focal reversible o disfuncin cortical. c) aura se desarrolla durante > 4 minutos o 2 auras sucesivas. d) cada aura < 60 min. e)
cefalea < 60 min siguiente aura. Los criterios sugeridos para la migraa crnica o transformada: diario o casi a diario (> 15 das / mes) dolor de cabeza > 1
mes. Promedio de duracin de la cefalea > 4 horas / da (sin tratar). Al menos uno de las siguientes: antecedentes de migraa de IHS. Historia de
aumento de frecuencia de cefalea con la disminucin de la gravedad de caractersticas migraosas durante al menos 3 meses. Ataques superpuestas de
dolor de cabeza que cumpla con todos los criterios de la IHS, excepto duracin. MIGRAA EN EL EMBARAZO: Consejos para no toma de drogas. Para
evitar las nuseas y los vmitos relacionados con el embarazo como resultado hipoglucemia y deshidratacin: comer pequeos bocados carbohidratos
frecuentes, ingesta adecuada de lquidos. Descanso adecuado. Minimizar la exposicin al frmaco, especialmente durante el primer trimestre. Si es
posible, deje los medicamentos profilcticos. Tratamiento agudo: paracetamol, en todos los trimestres y durante la lactancia, aspirina: probablemente
seguro pero la precaucin a corto plazo debido a riesgo de hemorragia postparto, hemorragia neonatal, y cierre prematuro del ductus arterioso. Evitar
durante la lactancia de riesgo del sndrome de Reye y el sangrado en la mortalidad infantil. triptanos, no se recomienda,preparaciones de ergotamina
contraindicado en el embarazo y la lactancia. MIGRAA MENSTRUAL: Gatillo hormonal es la exposicin a los altos niveles de estrgenos seguido por una
cada en los niveles. La liberacin de prostaglandinas uterinas que ocurren alrededor de la menstruacin un mecanismo adicional. 60 % de las mujeres
reportan un aumento en la frecuencia de la migraa en torno a la menstruacin. 14 % tiene exclusivamente migraa relacionada con la menstruacin.
cido mefenmico 500 mg 3-4 veces al da o naproxeno 500 mg dos veces 1-2 das antes de dolor de cabeza y de la duracin del perodo (de -2 a 3 das
de la menstruacin). Ergotamina, 1 mg (o 1/2 supositorios) durante el periodo vulnerable. Naratriptn, 1 mg bd o frovatriptn 2,5 mg bd durante 3-5
das. TRATAMIENTO: Analgesia simple con antiemticos: si las nuseas y los vmitos no son un sntoma importante como la motilidad gstrica se ve
afectada durante un ataque de migraa. La aspirina, 900-1200 mg (disuelto) + metoclopramida, 10 mg o domperidona, 10 - 20mg. Los medicamentos
alternativos incluyen paracetamol, 1000 mg y los AINE por ejemplo, ibuprofeno, naproxeno, diclofenaco. Triptanos: Todas las drogas de esta clase
(sumatriptn 50-100mgs, zolmitriptn 2.5mgs, naratriptn 2.5mg, rizatriptn 5-10mgs) tienen una alta eficacia con un mximo de 70 % que tiene una
respuesta dentro de 2 horas y el 40 % es libre de dolor a las 2 horas. Zolmitriptan y rizatriptn disponibles como obleas no tienen una accin ms rpida.
Sumatriptan disponible como aerosol nasal e inyeccin. Las drogas funcionan mejor cuando se toma temprano, pero no durante el prdromo o de la fase
de aura. Recurrencia de la cefalea a las 12-24 horas se produce en el 30%. Consejo habitual es tomar una dosis adicional de los triptanos, quizs
combinarlo con un AINE. Si no hay respuesta intente otra triptanos. El uso excesivo puede dar lugar a dolores de cabeza de rebote de 10 %.
Contraindicaciones: enfermedad arterial coronaria, enfermedad cerebrovascular, hipertensin no controlada, enfermedad vascular perifrica,
insuficiencia heptica significativa, y el embarazo. Efectos secundarios: molestias en el pecho o sensacin de pesadez, la mandbula, el hombro y rigidez
del cuello, parestesia, fatiga y mareos. Interacciones con otros medicamentos: evitar IMAO. Propranolol concentraciones sricas de rizatriptn. Por lo
tanto, utilizar dosis de 5 mg. El sndrome de la serotonina posible cuando se utiliza con los ISRS. Preparaciones de ergotamina: Aun as un papel para la
consideracin de tartrato de ergotamina en, aquellos pacientes que no toleran a los agonistas 5 - HT. Solo o en combinacin con cafena 1-2 mg puede
administrarse por va oral en el inicio o puede ser utilizado en aquellos pacientes que tienen sntomas premonitorios tales como ansiedad, bostezo, o la
fatiga. Tambin puede ser administrado por inhalador o supositorio. La sobredosificacin causa nuseas, dolor de cabeza de rebote, y la vasoconstriccin
perifrica. La dosis mxima recomendada por semana es de 10 mg. Dihidroergotamina (DHE) usaron por va intravenosa en pacientes con migraa
intratable en dosis de 0,3 a 1,0 mg cada 8 horas hasta una dosis total de 10mg. PREVENCION: La profilaxis de la migraa; Dolor de cabeza diario til para
controlar la frecuencia y patrones, por ejemplo, relacin de los periodos, los fines de semana, el uso excesivo de analgsicos y de triptanos. La profilaxis
ineficaz si el uso excesivo de medicamentos. Evite los desencadenantes dietticos slo en el 10 %. Considere la profilaxis si 2 ataques al mes o un

CURSO ENARM CMN SIGLO XXI TEL: 36246001

Pharmed Solutions Institute

PGINA 120

MANUAL DE TRABAJO DEL CURSO ENARM CMN SIGLO XXI


ataque prolongado que afecta el estilo de vida. Medicamentos profilcticos: betabloqueantes (propanolol, timolol, nadolol, atenolol). Los 5-HT2
antagonistas; pizotifen, ciproheptadina, metisergida, amitriptilina especialmente si la migraa asociada con cefalea tensional. Valproato sdico.
Topiramato. Gabapentina.
CASO CLINICO
Paciente 27 Aos, femenino, refiere haber comenzado con cefaleas
hace 12 hs, de carcter pulstil, unilateral, que no cede con AINES,
refiere nauseas que atribuye a los medicamentos, esta cefalea es
distinta de otros episodios sufridos previamente donde el dolor era
como una presin en zonas parietales y nuca.
PREGUNTA
Cual es el diagnostico mas adecuado en este paciente?
RESPUESTA
a.- Migraa.
b.- Cefalea tipo-tensin.
c.- Cefalea en racimos (cluster).
d.- Cefalae autonmica trigeminal.
CASO CLINICO
Paciente masculino, 62 aos, antecedentes de HTA, tabaquismo, EPOC,
que es trado por la familia quienes refieren que el paciente comenz
hace das a consumir AINES por cefalea de reciente comienzo,
posteriormente present conductas poco habituales y desde ayer se
encuentra desorientado.
PREGUNTA
Considerando los antecedentes y cuadro clnico, cual de los siguientes
son factores es el menos crticos para el pronostico?
RESPUESTA
a.- Cefalea subaguda y/o cefalea progresiva que empeora en el tiempo
(meses).
b.- Cefalea Nueva o distinta de la Habitual.
c.- Cualquier cefalea de maxima severidad en su inicio.
d.- Nueva cefalea en mayores de 40 aos.
CASO CLINICO
Paciente femenino, 42 aos que refiere cefalea sbita, intensa,
explosiva, siente que le estalla la cabeza, refiere haber tenido hace
unos das atrs un episodio similar aunque de menor intensidad, que la
obligo a guardar cama por unas horas.
PREGUNTA
Cual es el claso diagnostico mas probable?
RESPUESTA
a.- A trauma de crneo y/o cuello.
b.- A desrdenes vasculares craneales o cervicales.
c.- A desorden intracraneal no vascular.
d.- A sustancias o a su supresin.
CASO CLINICO
Paciente de 43 aos que acude a urgencias de atencin primaria por
cefalea. En el ltimo ao refiere cefalea basal diaria (Escala Visual del
dolor (EVA) 5-6/10) de caracterstica tensional, sin nuseas, fotofobia
ni fonofobia, y aunque no limita por completo su actividad ya que se
ha acostumbrado a ella, tiene episodios de reagudizacin ms
intensos, que la obliga a dejar sus labores, como en esta ocasin.
ltimamente estos episodios son ms frecuentes, y los relaciona con
problemas de salud en un hijo. Refiere adems que toma analgsicos
prcticamente a diario y por rachas combinando paracetamol,
ibuprofeno y lorazepam, desde hace casi ao y medio. Adems est en
tratamiento desde hace 2 aos con escitalopram.
PREGUNTA
Cual es el diagnostico mas probable?

CURSO ENARM CMN SIGLO XXI TEL: 36246001

RESPUESTA
a.- A trauma de crneo y/o cuello.
b.- A desrdenes vasculares craneales o cervicales.
c.- A desorden intracraneal no vascular.
d.- A sustancias o a su supresin.
CASO CLINICO
Femenino de 24 aos, casada, ocupacin actual comerciante. Paciente
refiere que hace 2 das presenta cefalea vespertina, caracterizada por
ser de tipo pulstil, unilateral, con una intensidad de 7/10, que dura
aproximadamente cuatro horas, que se intensifica con el movimiento
de la cabeza, se acompaa de nusea que no llega al vmito, y le
lastima la luz. Este dolor se exacerbo porque dice que no ha habido
buenas ventas. Refiere como causa aparente que los dolores le
empezaron hace una semana previa al inicio de su ciclo menstrual los
cuales eran de leve intensidad actualmente nota que el dolor
aumentado en intensidad. Para lo cual ha estado automedicandose
con advil que no refiere dosis, con lo que ceda el dolor a una
intensidad de 5/10. Manifiesta que estando con la crisis ya no puede
seguir realizando sus actividades diarias, y siente la necesidad de
acostarse y apagar la luz con lo cual el dolor disminuye.
PREGUNTA
Cul es el diagnstico ms probable para este caso?
RESPUESTA
a.- Trastorno de ansiedad
b.- Cefalea tensional
c.- Episodio de ansiedad
d.- Migraa
PREGUNTA
Cul sera la conducta teraputica ms adecuada a seguir en este
momento?
RESPUESTA
a.- Ibuprofeno
b.- Sumatriptan
c.- Paroxetina
d.- Fluoxetina-diacepam
PREGUNTA
La paciente regresa 2 meses despus y refiere que las cefaleas han
continuado, que han disminuido muy poco y que en estos dos meses
ha presentado 8 episodios, la paciente refiere que hace 1 mes le
hicieron un electrocardiograma y le dijeron que tena un bloqueo, no
da ms datos. Cul es la conducta teraputica ms adecuada a seguir
en este momento?
RESPUESTA
a.- Diltiazem mas amitriptilina
b.- Metoprolol mas amitriptilina
c.- Propanolol ms nortriptilina
d.- Verapamilo mas amitriptilina
PREGUNTA
3 meses despus a pesar de tratamiento la paciente nos refiere que no
ha notado cambio a pesar de tomar el tratamiento como se le indico.
Cul es la opcin teraputica ms adecuada en este momento?
RESPUESTA
a.- Valproato de sodio
b.- Esteroide
c.- Carbamacepina
d.- Fluoxetina

Pharmed Solutions Institute

PGINA 121

MANUAL DE TRABAJO DEL CURSO ENARM CMN SIGLO XXI


EPILEPSIA:
CIENCIAS BASICAS: Se define como la tendencia a tener convulsiones recurrentes. La epilepsia es una manifestacin de la enfermedad cerebral
subyacente. Convulsiones simples o aquellos que ocurren durante la enfermedad aguda no deben ser clasificados como epilepsia. Causas: Desconocido
en dos tercios de los casos .En Reino Unido, las encuestas de la comunidad muestran: Enfermedad cerebrovascular, 15 %; tumor cerebral, 6 %;
relacionados con el alcohol, 6 %; post-traumtico, 2 %; trastornos genticos, 1 %. Otras causas
CLASIFICACION DE EPILEPSIA
incluyen la esclerosis del hipocampo y malformaciones corticales y vascular. En los trpicos, la
EPILEPSIAS GENERALIZADAS Y SNDROMES:
Idiopticas de aparicin relacionada con la edad
neurocisticercosis es una causa comn. SALUD PUBLICA: 50/100 000/ao. 1 de cada 200
a)Epilepsia de ausencia infantil
tienen epilepsia activa (en el Reino Unido, 350.000) .Mayor incidencia en los pases en
b)Epilepsia mioclonica juvenil (JME)
desarrollo. En Mxico las crisis convulsivas de origen tardo son en su mayora debidas a
c)Epilepsia con crisis tnico-clnica
Sintomticas
y criptognicas
cisticercosis. CLASIFICACION: Bsica en generalizada (50 %) y focal (50 %), subdividido en
a)Sndrome de West
categoras etiolgicos: Idioptica (predisposicin gentica con el desarrollo normal, el examen
b)Sndrome de Lennox- Gastaut
y EEG). Sintomtico (anomala estructural). Criptognicas (anomala estructural supone, pero
c)Epilepsias con ausencias mioclnicas
Sintomticas
no probado). Subdivisiones: 1. Parcial simple; alteracin motora, sensitiva, autonmica o
a)Encefalopata mioclnica
psquica sin alteracin del estado de conciencia. 2. Parcial compleja; alteracin del estado de
EPILEPSIAS FOCALES Y SNDROMES
conciencia, puede comenzar con prdromos o con sntomas motores, sensitivos, autonmicos
Idioptica de aparicin relacionada con la edad
a)Epilepsia infantil benigna con puntas centrotemporales
o psquicos, presenta automatismos y es seguida por un periodo de confusin (estado
b)Epilepsia de lectura
posictal). 3. Parcial secundariamente generalizada: Inicia con alteraciones motoras, sensitivas,
Sintomtica:
autonmicas o psquicas, seguidas de prdida del estado de conciencia, con aumento de tono
a)Epilepsia con convulsiones parciales simples, complejas o
secundariamente generalizadas, que surgen de cualquier parte de la
muscular, contracciones rtmicas (clnicas), al final paciente comatoso y recuperacin lenta.
corteza
Puede haber incontinencia y/o mordida de lengua. 4. Generalizada de ausencia: Inicio sbito
b)Epilepsia parcial continua
con periodo breve de desconexin con el medio y recuperacin rpida, tono muscular puede
c)Sndrome caracterizado por la activacin especifica
Epilepsias
aumentar o disminuir, se acompaa de automatismos o movimientos clnicos leves. 5.
EPILEPSIAS INDETERMINADAS (FOCAL O GENERALIZADA)
Generalizada tnico-clnica: Perdida sbita del estado de alerta, acompaada de
Epilepsias con puntas continuas y actividad de las ondas en el sueo
contracciones tnico clnica generalizadas y hay estado posictal. STATUS EPILEPTICO:
Presencia de una crisis convulsiva continua o crisis repetitivas, discretas y con alteracin del estado de conciencia en el periodo interictal, el lmite de
tiempo es de 5 min. La causa ms frecuente es abstinencia a frmacos anticonvulsivantes o falta de apego al tratamiento. PATOGENIA: Es producida por
el disparo sincrnico y sostenido de una poblacin de neuronas cerebrales. Las estructuras ms susceptibles al desarrollo de crisis convulsivas
recurrentes son la corteza motora y la formacin del hipocampo y complejo amigdalino del sistema lmbico. DIAGNOSTICO: Clnico; Los ataques son
estereotipados, eventos paroxsticos. El diagnstico es clnico, testimonios son cruciales. Por lo general, seguido de un perodo de somnolencia. Ver ' La
prdida de la conciencia ". Desencadenantes incluyen: alcohol, la fatiga, privacin del sueo, infecciones, hipoglucemia, el estrs, luces estroboscpicas
(epilepsia fotosensible), lectura, el agua caliente (raro). Ausencias infantiles; raras despus de 10 aos de edad. F> M. Breve prdida de la conciencia
varias veces al da, accionado por la hiperventilacin, remiten en la edad adulta. EEG caracterstico; 3 Hz puntas y olas, sin fotosensibilidad. Epilepsia
mioclnica juvenil (JME): inicio antes de los 30 aos, sacudidas mioclnicas por la maana, ausencias tpicas, convulsiones tnico-clnicas generalizadas.
EEG tpico generalizado con puntas y ondas puede haber o no fotosensibilidad, remisin raro. Las convulsiones parciales: complejas asociado con
anormalidad estructural que subyace, por ejemplo, esclerosis del hipocampo, automatismos (relamerse los labios, la masticacin, la deglucin,
movimientos manuales estereotipados). Dj vu y jamais vu, auras olfativas (desagradable). Comportamiento inusual o emocionalidad. En aquellos
pacientes con epilepsia previamente diagnosticada y que presenta una crisis epilptica, se debern revisar los niveles sricos del frmaco, obtener
electrlitos, BH, pruebas de funcin heptica y renal, as como panel toxicolgico, si todo sale normal, pensar en la modificacin teraputica, aumentar
dosis mxima, disminuir dosis de 1er frmaco e iniciar un segundo. Pacientes que no tengan historia previa de epilepsia, solicitar BH, electrolitos, calcio,
magnesio, glucosa, funcin heptica renal, EGO y panel de toxicologa, si salen normal o negativo hacer RMN o electroencefalograma, para buscar causas
focales de las crisis (tumor, EVC, neuroinfeccin, trauma, enfermedad degenerativa) y TIPO DE EPILEPSIA
1RA. LINEA
2DA. LINEA
Valproato (200mg aumentar c/2
Levotiracetam o
tratar la causa especfica as como considerar tratamiento anticonvulsivo. Si todo sale Generalizadas
sem. maximo 2.5 grs) o
Topiramato
normal se puede pensar en crisis idiopticas. TRATAMIENTO: Recomendaciones
Lamotrigina (25 mg/d, mximo
(25mg/d mximo
generales; Evite las actividades peligrosas, por ejemplo, nadar solo, el montaismo.
400mgs)
400mg/d)
Carbamacepina (200mg/d) o
Levotiracetam o
Tome duchas en lugar de baos. Convulsiones simples: No hay un tratamiento a menos Focal
Lamotrigina o
Topiramato
que exista un alto riesgo de recurrencia, por ejemplo, EEG anormal como en JME o un
Valproato
MRI anormal. Si los factores precipitantes (por ejemplo, alcohol), la evitacin pueden Mioclonicas, atnicas Valproato o
Levotiracetam
Lamotrigina
(250mgs/d)
prevenir la recurrencia. Despus de un solo ataque no provocado, el riesgo de
Etosuximida
recurrencia es del 24%, sin causa y EEG normal y 65 % si se asocia con una anormalidad Ausencia
Valproato o
neurolgica + EEG anormal. Profilaxis: No hay indicacin para iniciar el tratamiento en
pacientes con lesiones en la cabeza, craneotoma, tumores cerebrales, a menos que se presentan convulsiones. La resistencia al tratamiento
farmacolgico se presenta hasta en 20% de los casos. En focos epileptgenos bien definidos se debe considerar el tratamiento quirrgico con reseccin
del mismo. Estatus epilptico: de 0-5 minutos. Documentacion clnica, diagnostico correcto, identificacin del tipo de SE, breve anamnesis y exploracin
fsica. Medidas bsicas; mantener funcin cardiorespiratoria, mon itorizacion de cosntantes vitales, via IV permeable, realizacin de exmenes de
laboratorio y administracin de antiepilpticos, si es posible documentacin con EEG. De 6-10 minutos, segn sospecha clnica (etilismo, dficit
nutyricional, hipoglicemia), tiamina IM (100mg) , glucosa IV (50cc al 50% adulto, 2-4cc/kg al 25% nios). Finalicacion del SE clnico y elctrico (tratamiento
farmacolgico). Diazepam IV (2mg/min hasta que cedan, mximo 0.3mg/kg), loracepam IV (0.1mg/kg, ritmo 2mg/min hasta que cedan). Puede constituir
el nico tratamiento si las crisis ceden y la causa de SE es corregida. Si no es asi asociar fenitoina IV (15-20mg/kg, en suero salino, ritmo 50mg/min en
adultos y 1mg/kg/min en nios). Minuto 30-40 Status epilptico resistente, ingreso UCI opciones: diacepam en perfusin continua, fenobarbital IV,
fenitoina IV. Minuto 60 SE refractario, ingreso a UCI, intubacin orotraqueal, ventilacin mecnica, acceso vascular central, monitorizacin EEG y
anestesia general. Diagnostico y tratamiento etiolgico; completar anamnesis y exploracin fsica detalladas, exmenes complementarios TAC, RNM,
LCR, pero nunca posponer el tratamiento, para asi corregir y prevenir hipoglicemia, alteraciones hidroelectrolticas y equilibrio acido-base, hipotensin,
hipoxia, edema cerebral, HIC, fiebre.
CASO CLINICO
Hombre de 62 aos, debuta con crisis de disnea, visin borrosa, cada
al suelo, movimientos repetitivos de los brazos y estado confusional de
duracin total de 10 minutos. Fue tratado con fenitona y cido

CURSO ENARM CMN SIGLO XXI TEL: 36246001

valproico con lo que permaneci estable, presentando slo 3 aos


despus nueva crisis con similares caractersticas. Comienza a notar
cambios conductuales, desinters, irritabilidad y comportamiento
obsesivo. Se hospitaliza por cuadro de compromiso de conciencia

Pharmed Solutions Institute

PGINA 122

MANUAL DE TRABAJO DEL CURSO ENARM CMN SIGLO XXI


progresivo de varios das de evolucin, agitacin psicomotora,
desorientacin, desviacin de la mirada hacia arriba y derecha,
temblor del mentn y parpadeo. Al ingreso se encontr adems
distonas de las extremidades y mantencin espontnea de posturas
de apariencia catatnica, fenmenos que cedieron con el uso de
diazepam endovenoso. Durante esta hospitalizacin present varios
otros episodios catatoniformes que duraban horas o das y que en
algunas oportunidades se asociaron a mioclonas faciales y de la mano
derecha. El EEG se mantuvo persistentemente alterado con actividad
lenta generalizada. Se interpret clnicamente como un cuadro
epilptico, con buena respuesta a los anticonvulsivantes. El estudio
imagenolgico mostr una atrofia frontal bilateral. El estudio de
laboratorio obtuvo una amonemia alta (44 mol/L) que fue
interpretada como secundaria al uso de valproato, por lo que debi
modificarse el tratamiento anticonvulsivante.
PREGUNTA
La asociacin de demencia y epilepsia obliga a pensar en diagnsticos
diferenciales. Cual de los siguientes es ms frecuente?
RESPUESTA
a.- Porfiria aguda intermitente.
b.- Neuroacantocitosis.
c.- Dficit de vitamina B12.
d.- Enfermedad de Lyme.
CASO CLINICO
Se trata de un paciente de 45 aos. Su padre haba sufrido crisis
epilpticas no tipificadas que fueron tratadas durante la infancia y la
adolescencia y que posteriormente desaparecieron. No hay otros
antecedentes destacables. A los 4 aos de edad comenz a presentar

CURSO ENARM CMN SIGLO XXI TEL: 36246001

episodios que consistan en unasensacin de sabor raro y de


familiaridad con lo que le rodeaba u ocurra en ese momento, de
segundos a un minuto de duracin, seguidos de prdida del
conocimiento con actividad convulsiva tonicoclnica generalizada de 1
a 3 min, que le mantenan en estado poscrtico unos 10 min. La
frecuencia de las crisis fue de mensual a trimestral durante la infancia
y desaparecieron en la adolescencia. Sigui tratamiento con
fenobarbital y fenitona, que hubo de mantenerse al persistir de forma
ocasional los episodios de sabor y familia ridad. A los 23 aos
comenz con episodios de desconexin del medio de 1a 3 min de
duracin, sin previo aviso y sin prdida de tono, en los que realizaba
automatismos orodeglutorios y con ambas manos, sin finalidad;
ocasional. Fue ingresado por una intensas crisis, aunque la exploracin
neurolgica, el EEG y la TAC craneal resultaron normales. Despues
ingres nuevamente por episodios subintrantes; en el EEG se detect
un foco de puntas y punta-onda lenta temporal izquierda. Seguido en
la consulta de neurologa en los ltimos 12 aos, ha mantenido un
curso fluctuante en cuanto a la frecuencia e intensidad de las crisis,
desde diarias hasta transcurrir tres meses sin presentar ninguna
PREGUNTA
Qu tipo de crisis presentaba el paciente a la edad de 4 aos y
adolescencia?
RESPUESTA
a.- Crisis generalizadas
b.- Crisis parciales simples con sntomas sensoriales especiales
c.- Crisis parciales complejas con aura
d.- Crisis de ausencia

Pharmed Solutions Institute

PGINA 123

MANUAL DE TRABAJO DEL CURSO ENARM CMN SIGLO XXI


ENCEFALOPATIAS:
TRASTORNOS TOXICO-METABOLICOS: El sistema nervioso puede verse daado cuando el fallo de un rgano permite el acmulo de sustancias txicas
que en circunstancias normales son eliminadas del organismo. Tambin como consecuencia de drogas o toxinas exgenas o endgenas, disfuncin de
mecanismos homeostticos o por el dficit de sustratos indispensables. Las principales herramientas de trabajo sern la historia clnica y la exploracin
fsica. El curso de los sntomas (monofsico, progresivo o con recaidas), la presencia de sntomas sistmicos, enfermedades previas o coexistentes as
como el uso de frmacos, hbitos dietticos o txicos y la exposicin a txicos sern de gran importancia en la identificacin del sndrome y su causa.
Hallazgos de laboratorio, estudios ms especficos (autoanticuerpos, estudio del LCR) o pruebas de imagen entre otros nos permitirn completar la
aproximacin etiolgica. Las encefalopatias se desarrollan habitualmente de manera insidiosa y la norma en todas ellas es la alteracin del estado
mental, en ocasiones de forma tan sutil que puede no detectarse con exploraciones rutinarias. La lesin del sistema reticular y la corteza cerebral
inducirn diferentes grados de alteracin en la orientacin, memoria, percepcin, capacidad de concentracin, juicio o planificacin y ejecucin de
tareas. Los trastornos metablicos sistmicos y la exposicin a toxinas provocarn ms frecuentemente degeneracin axonal. La musculatura ocular ser
la ms frecuentemente afectada as como los msculos flexores del cuello y de cinturas. Las enfermedades de los msculos se presentarn con debilidad,
dolor y fatiga. Dficits nutricionales y txicos exgenos sern los responsables ms frecuentes y el mecanismo patognico en ocasiones est bien
definido. ENCEFALOPATIA HIPXICO-ISQUMICA: El cerebro es un rgano con alto requerimiento metablico y por ello muy susceptible a dao por
deprivacin del flujo sanguneo. Las reservas de glucosa, glucgeno, ATP y posfocreatina se deplecionan a los 10-12 minutos de la isquemia. Despus de
15 minutos de isquemia global con parada cardiaca ms del 95% del tejido cerebral estar daado. Ms de la mitad de los supervivientes a una parada
cardiaca presentan algn grado de dao cerebral permanente. La patogenia del dao cerebral depender de la etiologa del mismo y el pronstico del
mecanismo implicado. Cualquier mecanismo conduce finalmente a una necrosis y apoptosis neuronal irreversible. Cambios bioqumicos diversos, dao
mitocondrial precoz, alteracin de citoesqueleto neuronal y la activacin de receptores de Glutamato son aspectos funcionales subyacentes al dao
celular tras un insulto hipoxico-isqumico cerebral. El dao neuronal puede ocurrir durante el periodo inicial de la isquemia (como consecuencia de los
cambios bioqumicos y funcionales), en el periodo de reperfusin (como consecuencia de la formacin de radicales libres y dao txico continuo), por
deterioro del flujo sanguneo (al reanudarse la circulacin espontnea y como consecuencia de una microcirculacin alterada) y como dao diferido
(secundario a desmielinizacin). ENCEFALOPATA HEPTICA: La enfermedad heptica crnica o aguda puede acompaarse de sntomas
neuropsiquitricos (y neuromusculares) conformando distintos grados de encefalopata. Es aceptado el papel del amonio como factor clave dentro de
una cascada de sucesos entre los que aparece alteracin de neurotransmisores, excesiva produccin de glutamina o estrs oxidativo. El edema de
astrocitos resultante se postula como la base fisiopatologa subyacente en la forma de HE aguda y crnica. Factores como la hiponatremia o sedantes
como las benzodiacepinas influyen en la aparicin de encefalopata heptica incluso con cifras de amonio en sangre normales. El aumento de
osmolaridad intracelular causado por hiperamonemia produce reduccin de los picos de colina y mioinositol y un aumento en el pico de glutamina y
glutamato. Las anomalas metablicas se correlacionan con la severidad clnica y son reversibles tras tratamiento. Las opciones teraputicas dependen
del estadio e instauracin de la encefalopata heptica. El objetivo ser reducir la produccin de amonio, aumentar su fijacin y excrecin, controlar los
sntomas neurolgicos presentes y modificar el shunt porto-sistmico. Bromocriptina o flumazenilo cayeron en desuso frente a lactulolosa o rifaximina.
En ocasiones el trasplante de hgado ser la opcin teraputica a considerar. ENCEFALOPATIA URMICA: Los signos de encefalopata en pacientes con
enfermedad renal no siempre obedecen al fracaso renal progresivo y pueden sumarse al dao neurolgico secundario a tratamientos inmunosupresores,
dilisis o trasplante renal. La fisiopatologa permanece por determinar. Trastornos del equilibrio acido-base, alteracin en las concentraciones de agua y
electrolitos, metabolismo anmalo del calcio y glndulas paratiroides o cambios en las concentraciones plasmticas de distintos neurotransmisores estn
presentes y determinan los sntomas neurolgicos detectables.El tratamiento principal de la encefalopata urmica es la dilisis. As mismo existen dos
sndromes neurolgicos relacionados con sta, el sndrome del desequilibrio de la dilisis y la demencia de la dilisis. ENCEFALOPATIA HIPOGLUCEMICA:
Glucemia menor de 30mg/dl, o periodos prolongados de hipoglucemia, para que se produzca dao irreversible. La intensidad del metabolismo cerebral
explica la vulnerabilidad particular del cerebro frente a desrdenes metablicos. Ante la posibilidad de una disminucin importante de glucosa en el SNC,
la actividad de los centros cerebrales superiores disminuye para reducir las necesidades de energa. La hipoglucemia provoca respuestas nerviosas y
hormonales para aumentar la produccin de glucosa en el hgado, y reducir su uso en rganos no nerviosos. Se caracteriza por sntomas de estimulacin
del sistema nervioso simptico o de SNC, provocados por una concentracin plasmtica de glucosa anormalmente baja. Los sndromes hipoglucmicos
pueden producirse por frmacos o sustancias como insulina, alcohol o sulfonilureas, y con menos frecuencia por salicilatos, propanolol, pentamidina,
disopiramida, hipoglicina A. Clnica: confusin, convulsiones, estupor, coma. ENCEFALOPATIA HIPERCAPNICA: Debido a enfermedades respiratorias
crnicas retenedoras de CO2, como fibrosis pulmonar, enfisema. Clnica: Presenta los sntomas de hipertensin craneal, cefalea holocraneal o frontal
intensa, edema de papila, somnolencia hasta coma. ENCEFALOPATA SPTICA: Disfuncin cerebral difusa o multifocal asociada con una infeccin
sistmica, sin evidencias de infeccin intracraneal, y que no puede ser atribuida a otros factores tales como efectos farmacolgicos o disturbios
metablicos. La encefalopata de la sepsis puede ser clasificada como encefalopata sptica o precoz, que se presenta antes de que se produzca la
disfuncin orgnica mltiple, o encefalopata tarda, que es acompaada por fallo orgnico mltiple, hipotensin y otros fenmenos sistmicos.
Probablemente se origine por la accin de mediadores inflamatorios en el cerebro o por una respuesta citotxica de las clulas cerebrales a estos
mediadores. Los efectos de la sepsis sobre el cerebro son detectables en cerebros previamente sanos, pero son amplificados en casos con lesin cerebral
concomitante, como luego de la injuria traumtica o la hemorragia subaracnoidea. La causa ms comn de encefalopata en los pacientes con
enfermedades mdicas crticas, habiendo sido descrita en el 9 al 71% de todos los pacientes crticos que sufren sepsis. TRASTORNOS CARENCIALES. Ante
cualquier trastorno del SN adquirido, siempre debe tenerse en consideracin una carencia vitamnica ya que el tratamiento sustitutivo administrado al
inicio del cuadro puede hacer remitir la sintomatologa, mientras que la administracin tarda, cuando las lesiones anatomopatologicas ya estn
establecidas, no evitara que el paciente quede con secuelas. Los sndromes ms importantes y frecuentes son debidos a la carencia de las vitaminas de
grupo B y en general son secunmdarios a estados carencialesde malnutricin, alcoholismo o patologa del sistema digestivo que causen malabsorcin.
ENCEFALOPATIA DE WERNICKE-KORSAKOFF: Es un sndrome neuropsiquitrico agudo que aparece como consecuencia del dficit de tiamina (B1) y se
asocia con una morbi-mortalidad significativa. Las causas de deplecin de tiamina y los mecanismos responsables son mltiples. Las lesiones se localizan
de forma simtrica a nivel periacueductal, sustancia gris subependimaria del III ventrculo, vermis cerebeloso, tubrculos mamilares, ncleos
hipotalmicos, porcin medial y dorsal del tlamo o ncleos vestibulares. El cuadro tpico clnico de confusin mental con disminucin del nivel de
conciencia. Puede o no haber parlisis oculomotora (VI) o nistagmo, oftalmopleja y trastorno de la marcha aparece tan slo en el 16% de los
pacientes. Puede haber crisis comiciales y si no se trata al paciente aparece miosis arreactiva a la luz, sntomas vegetativos, piramidalismo, coma y
muerte. Una correcta aproximacin facilita el diagnstico, la prevencin en paciente con factores o situacin clnica predisponentes y mejora el
pronstico con el adecuado soporte de tiamina parenteral 500mgs en 100ml de SF a pasar en 30 min 3 veces al dia. Los pacientes pueden superar la fase
aguda con secuelas como el sndrome amnsico de Korsakov. DEFICIENCIA DE VITAMINA B12: Cianocobalamina, La metilcobalamina actua como factor
indispensable para la enzima sintetasa (transformacin de homocosteina a metionina), ruta indispensable para el metabolismo del cido flico y sntesis
del ADN. La metinina es esencial para la sntesis de colina y fosfolpidos. La deficiencia se manifiesta con anemia megaloblastica, glositis, atrofia de

CURSO ENARM CMN SIGLO XXI TEL: 36246001

Pharmed Solutions Institute

PGINA 124

MANUAL DE TRABAJO DEL CURSO ENARM CMN SIGLO XXI


mucosa intestinal, ocasionalmente vaginitis. Degeneracin combinada subaguda de la medula (alteracin de cordones posteriores y laterales, con lo que
presenta trastornos sensitivos, parestesias que ascienden desde los pies hasta el tronco, a la que se aade una ataxia), alteraciones cognitivas/demencia
(irritabilidad, psicosis), polineuropatia mixta sensitivo-motora. En casos de malabsorcin o anemia perniciosa: 1000mg por va IM diaria durante 2
semanas. Luego seguir con 1000mg IM al mes. DEFICIENCIA DE ACIDO NICOTINICO: Niacina esencial para sntesis de NAD y NADP. La deficiencia de
niacina o su precursor el triptfano, provoca la pelagra. En el mundo occidental la pelagra es muy rara y se observa en alcohlicos, en el sndrome
carcinoide y enfermedad de Harnup. Clnica: triada clsica es dermatitis, diarrea y demencia, puede haber cuadro neurolgico sin manifestaciones
cutneas. De inicio es una encefalopata inespecfica (sndrome confusional, alteracin del nivel de conciencia, ataxia y mioclonias) que evoluciona a
demencia. Tratamiento, administrar entre 100-250mg VO tres veces al da durante 5 das. La administracin de 325 mg de aspirina media hora antes
previene el flushing facial. DEFICIT DE VITAMINA E: Debida a defecto de malabsorcin intestinal, atresia de vas biliares. Se manifiesta como una
degeneracin espinocerebelosa (ataxia, sndrome piramidal), ms oftalmopleja externa, retinitis pigmentaria, polineuropata. Tratamiento
administracin de vitamina E hidrosoluble por VO o IV.
CASO CLINICO
Paciente masculino, 42 aos de edad, nacionalidad italiana, con
antecedentes de tabaquismo, alcohol espordico, hernioplasta
umbilical; biopsia heptica cirrosis heptica; familiar de primer grado
con hepatitis de causa no filiada. Motivo de ingreso: cuadro agudo de
confusin mas euforia con posterior deterioro del sensorio de 24 hs de
evolucin ms catarsis negativa en la ltima semana. Al examen fsico
se constata somnolencia alternada por episodios de excitacin,
confusin tmporo-espacial; flapping +; spiders en cara anterior de
tronco, ascitis grado II; edemas en miembros inferiores. Laboratorio de
ingreso: Bilirrubina Total: 3.27/B.Directa: 1.87/B.Indirecta: 1.4 / FAL:
741/ GGT: 504/GOT : 180/GPT : 148. TP: 44% Plaquetas: 65.000/mm/
Factor V: 34%. Se solicita estudios para determinar etiologa de la
misma; Serologas virales (VHC; VHB; HIV; VHA negativos)FAN; ASMA;
AMA negativos;Ferremia:169 microg/dL(38-158);
Ferritina184
nanogr/ml (8-110);Saturacin de Transferrina 98,8%(20-50);
Transferrina 171 microg/dL(180-350). Ceruplasmina 18 mg/dL (5-30);
Cupruria 24h 424 microgramos (0-60); Lmpara de hendidura; anillos
de Kayser- Fleischer.
PREGUNTA
Considerando el cuadro clnico, que grado de encefalopata presenta?
RESPUESTA
a.- Grado I.
b.- Grado II.
c.- Grado III.
d.- Grado IV.
CASO CLINICO
Mujer de 52 aos con antecedentes de alcoholismo. Es trada al
Servicio de Urgencia por cuadro de tres das de compromiso de
conciencia cuali-cuantitativo, vmitos y diarrea. Al ingreso se constata
paciente confusa, inatenta, poco cooperadora. Al examen cognitivo
(Mini Mental, Test del Reloj, Test de Generacin de Palabras, Test de
trecho de digitos) destaca alteracin de la atencin y memoria
episdica, con amnesia antergrada y retrgrada. El resto de las
funciones cognitivas estaban relativamente conservadas. Se observa
confabulacin durante el examen mental. En el examen fsico
neurolgico se encontr dismetra y nistagmo bilateral. La tomografa
computada de cerebro y los parmetros de laboratorio bsicos eran
normales.
PREGUNTA
Cual es la conducta teraputica mas apropiada?
RESPUESTA
a.- Cianocobalamina.
b.- Tiamina.
c.- Niacina.
d.- Riboflamina.
CASO CLINICO
Mujer de 60 aos sindrome mielodisplsico tipo anemia refractaria
simple en tratamiento, presenta cefalea hemicranea izquierda, pulsatil,
severa, sbita (2 dias) agrega fotopsias, nauseas y mareo, no relevante,
a la EF se observa confusin, desorientacin, distractibilidad,
alteracin de la memoria a corto plazo. La resonancia nuclear

CURSO ENARM CMN SIGLO XXI TEL: 36246001

magntica cerebral evidencia: Alteracin en la intensidad de la seal


de la sustancia blanca con extensin hacia las fibras U de predominio
posterior, principalmente en la regin parieto-occipital bilateral y
simtrica, con compromiso parcial de la corteza.
PREGUNTA
Cual es el diagnostico mas probable en este caso?
RESPUESTA
a.- Enfecefalopatia hipoxico-isquemico.
b.- Encefalopatia de wernicke-korsakoff.
c.- Encefalopata heptica.
d.- Encefalopatia hipercapnica.
CASO CLINICO
Paciente del sexo femenino de 26 aos de edad, refiere a los 10
minutos de histerorrafia, sentirse mal diciendo que le est dando
sueo y que el dolor no ha cedido y la tensin arterial comienza a
disminuir y la frecuencia cardiaca, la paciente se encontraba
ligeramente ciantica y con tensin arterial de 80/40 torr y frecuencia
cardiaca de 60 p.m. procedo a intubar sin aplicacin de frmacos
inductores ni relajantes y se continu con la administracin de oxgeno
al 100% a 5 l/min, observndose movimientos de la bolsa que
corresponden a inspiracin y espiracin del automatismo respiratorio,
sin embargo se aplica ventilacin asistida. Posteriormente la paciente
presenta paro cardiorespiratorio detectado con ausencia del pulso
carotdeo, por lo cual se comienza a realizar maniobras de resucitacin
cardiopulmonar y administramos 1 mg de epinefrina sin respuesta
positiva por lo que se procede a administrar otro miligramo ms de
epinefrina, saliendo del paro la paciente con tensin arterial de 130/90
y frecuencia cardiaca de 150 p.m., comienza a disminuir nuevamente
la frecuencia cardiaca y la presin arterial, por lo que administramos
0.6 mg de atropina y dopamina a dosis respuesta; posteriormente, a
pesar de tener la paciente tensin arterial de 130/90 torr con apoyo
farmacolgico con una frecuencia cardiaca de 150 p.m. la paciente
contina con cianosis por lo que decidimos pasarla a terapia intensiva,
practicndose una gasometra arterial a su ingreso la cual nos reporta
los siguientes parmetros: pH 7.27, PCO2 31, PO2 252, CO2 total
14.90, HCO3 14, EB -11.40, Sat. O2 99.50%. Se practica una
tomografa cerebral entre 36 y 48 horas despus la cual nicamente
observa edema cerebral. Se extuba dos das despus de su ingreso con
respuesta neurolgica Glasgow 7; a las veinticuatro horas, presenta
crisis convulsivas tnico-clonicas de una hora de duracin por lo que
sedan profundamente a la paciente y la intuban nuevamente, para que
varios das despus se declare coma y lesin cerebral con estado
vegetativo.
PREGUNTA
Cual es el diagnostico mas probable en este caso?
RESPUESTA
a.- Enfecefalopatia hipoxico-isquemico.
b.- Encefalopatia de wernicke-korsakoff.
c.- Encefalopata heptica.
d.- Encefalopatia hipercapnica.
CASO CLINICO

Pharmed Solutions Institute

PGINA 125

MANUAL DE TRABAJO DEL CURSO ENARM CMN SIGLO XXI


Mujer de 69 aos de edad que presentaba como antecedentes CBP
estadio IV, osteoporosis, fracturas vertebrales y hernia de hiato.
Realizaba tratamiento con fosamax, omeprazol, atorvastatina, calcio,
atarax, colestiramina, hierro y rifaximina. Consult por un cuadro
progresivo de unos 6 meses de evolucin de fallos mnsicos, temblor y
cierta torpeza para caminar. La exploracin fsica fue normal salvo por
tinte subictrico y algunas sibilancias aisladas. No exista
hepatomegalia. En cuanto a la exploracin neurolgica se encontraba
bien cognitivamente con un Minimental de 26/30 a expensas de fallos
en la memoria reciente. Pares craneales normales. No exista rigidez,
acinesia, alteraciones de los reflejos posturales, aunque s un discreto
temblor fino distal en miembros superiores bilateral y simtrico. La
marcha era cautelosa sin clara disminucin de braceo y se detect una
hiperreflexia generalizada con aumento del rea reflexgena. No
exista dficit motor o sensitivo. Los reflejos plantares fueron flexores.
En cuanto a las pruebas complementarias, el hemograma presentaba
una discreta pancitopenia (3.550 leucocitos/l, 116.000 plaquetas/l y
3,75 millones de hemates/l). El estudio de coagulacin mostr un
tiempo de protrombina normal estando el de tromboplastina en 48,2s.
En la bioqumica destacaban datos de hepatopata con una bilirrubina
total de 1,72mg/dl, bilirrubina directa 1,15mg/dl (0-0,30), elevacin de
las transaminasas con aspartato aminotransferasa en 262 u/l, alanino
aminotransferasa 128 u/l, fosfatasa alcalina 146 u/l, gammaglutamil
transpeptidasa 87 u/l.
PREGUNTAS
De acuerdo al cuadro clnico Cul es la complicacin ms probable
que este presentando en este momento?
RESPUESTA
a.- Hemorragia subaracnoidea
b.- Evento isqumico cerebral
c.- Encefalopata
d.- Meningioma
PREGUNTAS
Qu enfermedades hay que investigar por estar ms probablemente
relacionadas con la cirrosis biliar primaria de esta paciente?
RESPUESTA
a.- Infecciosas
b.- Autoinmunes
c.- Crnico-degenerativas
d.- Endocrinas
PREGUNTA
Cul es la conducta teraputica ms adecuada para este caso?
RESPUESTA
a.- Esteroides ms ac. ursodexoxicolico
b.- Trasplante heptico
c.- Quirrgico
d.- Metronidazol y lactulosa
CASO CLINICO
Mujer de 42 aos con antecedentes de alcoholismo, hipertensa, con
diagnstico de cncer de mama hace 1 ao, es trada al servicio de
urgencias por cuadro de 3 das de compromiso de conciencia cualicuantitativo, vmitos, diarrea, el familiar refiere que lleva semanas
ingiriendo muy poco alimento. Al ingreso se constata paciente confusa,
inatenta, poco cooperadora, tienen tendencia a la somnolencia, pero
conserva la capacidad de mantenerse despierto. Al examen cognitivo,
destaca alteracin de la atencin y memoria episdica, con anmenisa
antergrada y retrograda. El resto de las funciones cognitivas estaban
relativamente conservadas. En el examen neurolgico se encontr
dismetra y nistagmo bilateral, oftalmoplejia y polineuropatia. La
tomografa de cerebro y los parmetros de laboratorio bsicos todos
fueron normales.

CURSO ENARM CMN SIGLO XXI TEL: 36246001

PREGUNTA
Cul es la conducta teraputica ms adecuada a seguir en este
paciente?
RESPUESTA
a.- Cianocobalamina
b.- Tiamina
c.- Niacina
d.- Riboflavina
PREGUNTA
Cul es la alteracin ms precoz e importante que se puede presentar
en esta patologa?
RESPUESTA
a.- Regin periventricular del tlamo
b.- Atrofia de cuerpos mamilares
c.- Agrandamiento de tercer ventrculo
d.- Desmielinizacin de sustancia gris profunda
PREGUNTA
Cul es la causa menos probable de que este paciente presente esta
patologa?
RESPUESTA
a.- Alcoholismo
b.- Hipertensin
c.- Inanicin
d.- Cncer
PREGUNTA
Cul es la incidencia de esta patologa?
RESPUESTA
a.- 1-2%
b.- 3-5%
c.- 10%
d.- 0.1-0.5%
CASO CLINICO
Varon de 61 aos de edad es llevado a urgencias, por su hija debido a
dificultad para caminar. Ella informa que dos das antes advirti que
los pasos de su padre eran muy cortos y que no caminaba una
distancia mayor de 1.5 mts sin tambalearse. Supuso que esos sntomas
eran resultado de otra intoxicacin etlica, pero al no resolverse con el
tiempo decidio buscar atencin medica. EF: temperatura bucal 37C,
pulso de 84/min, TA 122/74mmHg. El paciente parece distrado y solo
esta orientado en cuanto a persona. El examen neurolgico revela
nistagmo horizontal bilateral en la mirada lateral, sensacion disminuida
al dolor y la vibracin por debajo de las rodillas y marcha con base
amplia con pasos lentos y cortos. La concentracin de alcohol en
sangre es de 18mg/dl.
PREGUNTA
Cul es el diagnostico mas probable para este caso?
RESPUESTA
a.- Hemorragia cerebelosa
b.- Hematoma subdural
c.- Encefalopatia de Wenicke
d.- Beriberi
PREGUNTA
Qu es mas probable esperar en este paciente si no se trata o se trata
de manera inadecuada?
RESPUESTA
a.- Sindrome de Korsakoff
b.- Hemorragia cerebelosa
d.- Demencia de Pick
c.- Corea de Syndeham

Pharmed Solutions Institute

PGINA 126

MANUAL DE TRABAJO DEL CURSO ENARM CMN SIGLO XXI


DEMENCIAS:
CIENCIAS BASICAS: La demencia es un sndrome clnico caracterizado por perdida adquirida de habilidades cognitivas y emocionales, lo suficientemente
importante para interferir con el funcionamiento diario y la calidad de vida. Tambin puede acompaarse de otras manifestaciones neuropsiquitricas
tales como alteraciones motoras, de la conducta, depresin, ansiedad, alucinaciones y/o delirium. El trmino demencia no implica una causa o proceso
patolgico especfico, es de etiopatogenia heterognea y puede ser mixta. Existen ms de 55 enfermedades que pueden causar demencia. La definicin
de demencia segn Clasificacin de la demencia de acuerdo al DSM IV se divide de la siguiente manera: Demencia en enfermedad de Alzheimer; de inicio
temprano, de inicio tardo, mixta o atpica, inespecfica. Demencia vascular; de inicio agudo, multi-infarto, vascular subcortical, mixta: subcortical y
cortical. Demencias propias de otras enfermedades; Enfermedad de Pick, Enfermedad de Creutzfeldt-Jakob, Enfermedad de Huntington, Enfermedad de
Parkinson, asociada a infeccin por VIH, asociada a otras enfermedades. Demencia no especificada. As mismo una vez hecho el diagnstico de demencia
se puede clasificar: Leve: cuando afecta actividades de la vida diaria sin causar problemas para la independencia del paciente. Moderado: cuando
condiciona incapacidad para vivir de manera independiente. El individuo no recuerda informacin bsica acerca de su vivienda, actividades recientes o el
nombre de ciertas personas familiares a l. Severo: Se caracteriza por la absoluta incapacidad de retener nueva informacin y solo se recuerdan
fragmentos de experiencias o conocimientos pasados. El paciente no reconoce a sus familiares. SALUD PUBLICA: La E. Alzheimer en 2050 podra llegar al
30% de la poblacin mundial. A medida que se prolonga la expectativa de vida tambin aumenta la prevalencia de enfermedades como las demencias. La
frecuencia de E. Alzheimer se duplica cada 5 aos apartir de los 60a. A los 60 aos 1%, entre 80-84 aos de 16%, >de 85aos de 30-40%. Existen factores
de riesgo no modificables como la edad siendo este el ms importante para el desarrollo de demencia. En los pacientes con trastorno del aprendizaje
como Sndrome de Down las demencias se presentan ms temprano. Estudios de prevalencia muestran un mayor ndice de demencia en mujeres que en
hombres, especialmente en enfermedad de Alzheimer (2:1). El nmero de casos de demencia vascular es mayor en hombres que en mujeres aunque
stas ltimas suelen igualarlos en edades ms avanzadas. La historia familiar de demencia en un pariente de primer grado aumenta cuatro veces el
riesgo de desarrollar demencia. Demencia vascular es la segunda causa de demencia despus del Alzheimer representando hasta el 18% de las
demencias, se han descrito prevalencias que varan desde el 3 al 21%. DIAGNOSTICO: Para un diagnstico clnico confiable la prdida de la memoria
debe estar presente al menos durante seis meses; si la duracin de las manifestaciones es menor, el diagnstico es presuntivo. La severidad de la
demencia se establece por el grado de decremento cognitivo o de la memoria, cualquiera que sea el ms predominante. Cuando existen episodios
concomitantes de delirium el diagnostico de demencia debe ser diferido. El diagnstico de las demencias se debe realizar por un especialista experto en
el tema y podra abordarse de la siguiente establece en dos pasos: se deber hacer solo tras una extensa evaluacin que debe incluir: historia clnica,
evaluacin del estado cognitivo, examen fsico y otros estudios apropiados, una revisin de los medicamentos tomados por el paciente en orden de
descartar efectos adversos que afecten el funcionamiento y minimizar su uso. ENFERMEDAD DE ALZHEIMER: Sndrome clnico adquirido y progresivo
caracterizado por afectar las funciones intelectuales superiores. Deterioro de la memoria en el corto y largo plazo. Presencia de por lo menos uno de los
siguientes dficits cognitivos: Afasia, Agnosias, Apraxia y/o alteraciones de funciones ejecutivas. Alteracin de conducta: Laboral y social. No debe existir
alteracin de nivel de conciencia. En su patogenia exiten Hiptesis colinrgica: hay prdida de neuronas colinrgicas en corteza e hipocampo con
aumento de accin de butirilcolinesterasa. Hiptesis glutamatergica: L-glutamato neurotransmisor exitatorio SNC, interviene en los procesos de
memoria, aprendizaje y plasticidad neuronal, el aumento parece jugar un rol principal en la patognesis y en el dao producido por isquemia. Se genera
por la produccin anormal y acumulacin de amiloide, aumenta la produccin de A1-42 que es el que se deposita. Se generan las placas de amiloide
que desencadenan, formacin de ovillos neurofibrilares (por hiperfosforilacin de protena Tau). Oxidacin y peroxidacin de lpidos. Excitotoxicidad por
glutamato, inflamacin, activacin de la cascada que lleva a la apoptosis. La muerte neuronal lleva al dficit de neurotrasmisores. En las guas de
diagnstico, se describe de manera didctica 10 signos de alerta para pensar en Alzheimer: 1. Prdida de la memoria 2. Dificultad para realizar tareas
familiares 3. Problemas con el lenguaje 4. Desorientacin en tiempo y espacio 5. Juicio empobrecido 6. Problemas con el pensamiento abstracto 7.
Prdida de cosas 8.
Cambios en el nimo o el comportamiento 9.
Cambios en la personalidad 10. Prdida de la iniciativa Alteraciones
neuropsiquitricas y motoras. Conforme la enfermedad avanza se presentan apraxias y con ello la incapacidad funcional para las actividades cotidianas,
como asearse y vestirse, puede haber desinhibicin y agresividad, las alucinaciones son poco frecuentes y si se presentan el diagnstico debe orientarse
en otra direccin, sin embargo pueden desarrollar delusiones, entre ellas se ha descrito el sndrome de Capgras hasta en el 10% de los pacientes, donde
generan la idea de que su cuidador ha sido sustituido por un impostor, paralelamente hay alteraciones en el patrn de sueo vigilia, sntomas motores
extrapiramidales como rigidez y sacudidas mioclnicas, hasta una etapa final de mutismo y postracin. Tratamiento: Inhibidores de la Acetilcolinesterasa:
Donepecilo, rivasigmina, galantamina. Inhibidores de los receptores NMDA: Memantina. Estabilizadores de membrana: Lamotrigina, carbamacepina,
topiramato. Delirios y alucinaciones: Risperidona, olanzapina, haloperidol. Depresin y ansiedad: citalopram, paroxetina, sertralina. DEMENCIA
VASCULAR: Se ha acuado el trmino de dao cognitivo vascular para describir a la prdida de funciones cognitivas asociada a otras manifestaciones
neurolgicas y que tienen como base el dao vascular. Factores de riesgo, la edad, el bajo nivel de escolaridad, la enfermedad hipertensiva, el dao
cerebral vascular previo, especialmente infartos cerebrales, el padecer diabetes, obesidad, sndrome metablico, niveles elevados de homocistena y la
ateroesclerosis. Clasificacin: Demencia por dao cortical o enfermedad multi infarto relacionada generalmente con el dao de grandes vasos (23%) y la
causada por dao subcortical o enfermedad difusa de la sustancia blanca relacionada a dao de pequeos vasos (50%). La regin ms afectada son los
ganglios basales, dentro del dao subcortical se hace referencia a tres tipos de entidades, 1) infartos lacunares (16%), la distribucin puede variar, sin
embargo el dao se ha descrito con mayor frecuencia en la regin frontal, 2) demencia talmica (8%) donde el dao puede venir desde la arteria basilar y
3) la encefalopata subcortical ateroesclertica o sndrome de Binswwanger donde hay gran dao a la sustancia blanca. El dao vascular, como ya se
coment puede ser secundario a infartos en grandes vasos corticales o a nivel subcortical predominantemente ya sea por hemorragias o principalmente
por eventos vasculares isqumicos donde la hipoperfusin lleva a la gliosis y al dao en la sustancia blanca que posteriormente se manifestar como
leucoaraiosis, otro tipo de dao es la isquemia crnica subcortical que da afeccin principalmente a nivel periventricular. Edad de inicio es en entre los
50 y 59 aos, con otro pico entre los 60 y 69 aos de edad. Clinica: pueden desarrollan alteraciones en la marcha, afectacin de primera neurona motora,
bradicinesia, rigidez, alteracin en los esfnteres, sndrome pseudobulbar, alteraciones de la personalidad y del estado de nimo, afasia, agnosias y
alteraciones en la memoria. Establecer criterios diagnsticos definitivos ha sido complicado, las ms utilizadas son las de Hachinski, Rosen. Tratamiento:
donepezilo con dosis inicial de 5mg al da y de mantenimiento de 10-20mg despus de 4 semanas, rivastigmina con dosis inicial de 1.5mg, incrementando
de 1.5mg diarios hasta lograr la dosis de mantenimiento de 6mg o parches de 4.6mg por da con dosis paulatinas con un mximo de 9.6 mg/da. Dado
que se trata de una demencia que se puede prevenir, es importante dar tratamiento a los factores de riesgo. DEMENCIA FRONTOTEMPORAL: Tercera
demencia ms comn despus de la Demencia tipo Alzheimer. La prevalencia general es de 17,6 por 100.000 habitantes. La edad de presentacin es
aproximadamente a los 58 aos. Afecta por igual a hombre y mujeres. En el 30-50 % de los casos existe una historia familiar de Demencia
frontotemporal (DFT) principalmente en la variante conductual, mientras que en la demencia semntica y la afasia progresiva no fluentes menos
frecuente. El trmino de Enfermedad de pick se reserva para casos de DFT con inclusiones positivas teidas de plata denominados cuerpos de Pick.
Debido a su asociacin de atrofia en lbulos temporales y frontales. La mutacin caracterstica se encuentra en el gen que codifica a la protena tau en el
cromosoma 17, cuyo patrn es de herencia autosmico dominante. Esta protena pertenece a la familia de las MAPT (protenas tau asociadas a

CURSO ENARM CMN SIGLO XXI TEL: 36246001

Pharmed Solutions Institute

PGINA 127

MANUAL DE TRABAJO DEL CURSO ENARM CMN SIGLO XXI


microtbulos) y tiene participacin en la regulacin de la polimerizacin y ensamble de las subunidades de tubulina, por lo que es indispensable para
determinar la estabilidad y funcin axonal. La DFT es una enfermedad progresiva con inicio insidioso, en etapas iniciales presentan ms sntomas
conductuales, sin afectar la memoria ni las capacidades visuoespaciales. Puede ser espordica o familiar. Pueden presentar diversas alteraciones de
acuerdo al sndrome de DFT, sin embargo se caracteriza por la presencia de afasia, alteraciones en la conducta y del lenguaje. Las alteraciones en el
lenguaje pueden existir con o sin demencia asociada, aunque se estima que la mayora de los pacientes desarrollara demencia en etapas tardas de la
enfermedad. Existe un deterioro de la funcin ejecutiva que incluye la planificacin, organizacin, flexibilidad, juicio, resolucin de problemas con
conservacin de la percepcin visual y habilidades espaciales. Existe tambin una incapacidad para reconocer emociones, en particular las negativas
como la ira, el miedo, tristeza. La orientacin en tiempo y lugar, el recuerdo libre demorado son alteraciones frecuentes de la DMF, sin embargo estn
presentes tambin durante las primeras etapas de la enfermedad de Alzheimer, su distincin sigue siendo difcil mediante pruebas neuropsicolgicas.
CASO CLINICO
Se trata de masculino de 73 aos de edad el cual es originario de
Mrida, actualmente vive solo, padece diabetes mellitus desde hace 15
aos, hipertensin arterial, triglicridos, colesterol y acido rico
elevados, al interrogatorio se desconoce la posologa para sus
padecimientos, refiere que le indicaron glibenclamida, bezafibrato,
enalapril, alopurinol y pravastatina, el familiar refiere que desde hace
mas de dos aos presenta dificultad para recordar las cosas que est
haciendo, se ha vuelto desconfiado, esconde las cosas para evitar que
se las roben, pobreza de pensamiento, pensamiento con bloqueo e
ideas de dao y robo.
PREGUNTA
Debido a las mltiples entidades nosolgicas que presenta el paciente.
Cual es complicacin neurolgica ms probable?
RESPUESTA
a.- Demencia vascular es la ms probable.
b.- Demencia de Alzheimer es la ms probable.
c.- La demencia por priones es la ms probable.
c.- La demencia de Pick es la ms probable.
CASO CLINICO
Masculino con neumona por aspiracin, con antecedente de prdida
progresiva de funciones mentales tipo demencia, previamente
presento cambio de conducta, prdida de peso y cefalea continua de
tres meses de duracin. A la EF alerta, mutista, el examen neurolgico
no revela alteraciones sensoriales, pares craneales sin afecciones sin
embargo presenta mioclonos a la estimulacin y expontaneos
incluyendo durante el sueo, los estudio de laboratorio no muestran
datos de importancia, el liquido cefalorraqudeo normal al igual que la
TAC de cabeza.
PREGUNTA
Cul es el agente causal ms probable en este caso?
RESPUESTA
a.- Viral ADN.
b.- Viral ARN.
c.- Fungico.
d.- Prion
CASO CLINICO
Se trata de femenino de 84 aos de edad la cual ingresa a consulta por
presencia de dificultad para deglutir los alimentos, presentando
accesos de tos hemetizante durante las ingestas, y desde hace 48 hs.,
fiebre, dificultad respiratoria progresiva, somnolencia permanente y
expectoracin muco purulenta ftida, se observa prdida total de la
independencia funcional, postrada en cama desde hace
aproximadamente 3 aos, al cuidado permanente de 2 de sus hijas,
portadora adems de miocardiopata dilatada, cumpliendo
tratamiento en la actualidad con Digoxina, Furosemida, Enalapril, AAS
y vasoactivos cerebrales.
PREGUNTA
Considerando los antecedentes y al cuadro clnico, cual es el
diagnostico mas probable?
RESPUESTA
a.- Demencia vascular.
b.- Demencia seil.

CURSO ENARM CMN SIGLO XXI TEL: 36246001

c.- Demencia alzheimer.


d.- Demencia de pick.
CASO CLINICO
Masculino de 69 aos de edad con antecedentes de HTA de carcter
leve -moderada desde hace 8 aos aproximadamente adems de
Hipercolesterolemia desde hace 10 aos segn refiere, para lo cual
ingiere como medicacin, Enalapril 10 mg e hidroclorotiazida 12,5 mg
una vez por da, complementando dicho tratamiento con dieta
hiposdica e hipocolesterolemica. Manifiesta que no cumple
estrictamente con la dieta descripta pero que siempre ingiri la
medicacin antihipertensiva. Consulta el da de hoy por cefalea a
predominio frontal, la cual se manifiesta desde hace varios das
acompaado de sensacin de inestabilidad que no sabe referir
exactamente pero si manifiesta sentirse muy extrao, situacin que no
es habitual en el pero, que lo inquieta y preocupa por lo cual decidi
concurrir a la consulta medica, por propios medios. Manifiesta que
ltimamente no control su T A como se le haba indicado y que
tampoco respet la dieta habitual. Como antecedente importante
refiere hipercolesterolemia leve. Adems expresa que siempre llev
una vida activa, que sola concurrir desde el pueblo hasta su domicilio
rural en bicicleta hasta hace unos 10 aos aproximadamente.
PREGUNTA
Considerando los antecedentes y al cuadro clnico, cual es el
diagnostico mas probable?
RESPUESTA
a.- Demencia vascular.
b.- Demencia seil.
c.- Demencia alzheimer.
d.- Demencia de pick.
CASO CLINICO
Paciente de 73 aos de edad que presenta temblor de reposo de
predominio en miembro superior derecho, rigidez, bradicinesia y
alteracin de reflejos posturales. En la ltima visita, la familia que lo
acompaa relata que a lo largo de los ltimos 6-8 meses, lo vienen
notando triste, poco comunicativo, "perezoso" y con escasa ilusin a la
hora de iniciar actividades nuevas. Han notado que pasa horas
ensimismado, sin llegar a concluir la tarea que estuviera realizando. Se
muestra incapaz de programar las actividades que ha de realizar al da
siguiente o las realiza a destiempo, de forma desorganizada. Parece
mostrarse poco reactivo ante los problemas, disgustos o alegras
cotidianas, como si nada le importara. Se muestra olvidadizo, tiende a
usar dietarios y notas cada vez con mayor asiduidad, sin que esto haya
impedido el haber olvidado citas importantes o tareas inexcusables en
su negocio.
PREGUNTA
Considerando los antecedentes y al cuadro clnico, cual es el
diagnostico mas probable?
RESPUESTA
a.- Demencia por parkinson.
b.- Demencia por prionopatia.
c.- Demencia alzheimer.
d.- Demencia de frontotemporal.
CASO CLINICO

Pharmed Solutions Institute

PGINA 128

MANUAL DE TRABAJO DEL CURSO ENARM CMN SIGLO XXI


Mujer de 56 aos, con 12 aos de escolaridad, diestra, duea de casa,
separada, con una convivencia estable. Inicia un delirio paranoide pero
asociado a una gran pasividad (era necesario escogerle la ropa,
abandon las labores domsticas). Se presento descontrol urinario
ocasional. En el examen mostraba una enferma vigil, prescindente, sin
contacto visual, que no expresaba afectos y colaboraba poco al
examen (lo que impidi evaluar el campo visual y la sensibilidad).
Exista gran hipokinesia pero sin rigidez ni movimientos anormales; la
metra era normal. Exista enganche digital pero no prehensin forzada
ni succin. En el Minimental test de Folstein (MM) obtuvo slo 10/30
puntos, fracasando en Orientacin, Atencin y clculo, Lectoescritura y
Dibujo, con respeto relativo de la memoria. Al preguntarle la fecha dijo
"a ver... a ver... a ver"; lo mismo respondi al pedirle restas. Fue capaz
de leer la orden escrita, pero no la cumpli; en vez de una frase
escribi su nombre, y el pedirle copiar el dibujo recibi el lpiz pero no
rindi, limitndose a leer una y otra vez fragmentos del test. En la
prueba de Matrices Progresivas Coloreadas (MPC) repeta las
instrucciones pero no renda y musitaba en forma ininteligible; obtuvo
0/12 puntos. En Aprendizaje verbal (AV) deca "ya..." y se quedaba
mirando al frente, con rostro inexpresivo, sin repetir las palabras
propuestas. En Memoria semntica (MS) evoc 2 nombres de animales
en 60 segundos.
PREGUNTA
Considerando los antecedentes y al cuadro clnico, cual es el
diagnostico mas probable?
RESPUESTA
a.- Demencia por parkinson.
b.- Demencia por prionopatia.
c.- Demencia alzheimer.
d.- Demencia de frontotemporal.
CASO CLINICO
Femenino de 47 aos acude por fallas de la memoria, errores en la
denominacin y en actividades domsticas. Aparecieron cambios de
conducta, se puso:"confianzuda", caprichosa, golosa, floja. Se negaba a
colaborar en actividades domsticas, e insista en forma majadera en
diversos temas. Meses despus fue evaluada con mayor detalle. En su
casa era independiente para vestirse, alimentarse y asearse, pero a
veces se orinaba. No era capaz de retener informaciones nuevas, a
veces no reconoca a sus familiares, y mostraba una proximidad
excesiva frente a los extraos. Tena prdida de libido y se mostraba
inquieta e irritable. Se mostraba gil y el examen neurolgico bsico
era normal, no existan reflejos primitivos. Su actitud era inadecuada
por animo subido, proximidad excesiva y gran tendencia al payaseo.
Por ejemplo, en una oportunidad interrumpi el examen diciendo
"aprate abuelito por favor! porque tengo que ir a hacerle la comida a
mi marido". No mostraba ninguna preocupacin por sus rendimientos
y con frecuencia no colaboraba; al pedirle interpretar proverbios slo
dijo "no s!". Al denominar cometa errores caprichosos, pero que
sugeran una afasia amnsica. Por ejemplo, ante el dibujo de un
paraguas dijo riendo "es un sostn!" y otra vez "es una bicicleta!",
pero ante el dibujo de un sof dijo "es para sentarse". No existan
defectos fonolgicos ni morfosintcticos. Al pedirle interpretar el
proverbio "a quien madruga Dios le ayuda" dijo "a mi marido? No, mi
marido tiene negocio y yo trabajo con l, yo de cajera y l de
vendedor...". Al dictarle "baj a la tierra" lo repiti bien pero escribi con buena letra- "monono nonino ninino". No exista apraxia
constructiva (aunque a veces el dibujo era muy descuidado), oral ni
ideo-motora; segn su familia se vesta bien, pero seleccionando mal
su vestimenta. Fracas en clculo oral y escrito; exista una agnosia
digital visual y desorientacin derecha-izquierda -ni siquiera fue capaz
de decir cual era su mano derecha.
PREGUNTA
Considerando los antecedentes y al cuadro clnico, cual es el
diagnostico mas probable?
RESPUESTA

CURSO ENARM CMN SIGLO XXI TEL: 36246001

a.- Demencia por parkinson.


b.- Demencia por prionopatia.
c.- Demencia alzheimer.
d.- Demencia de frontotemporal.
CASO CLINICO
Mujer de 60 aos de edad, sin antecedentes de inters, que acude a la
consulta acompaada de sus familiares los cuales la notan rara,
olvidaba recados que le dejaban o los transmita a destiempo, a veces
de forma reiterada a la misma persona. En varias ocasiones vena de la
compra habiendo olvidado artculos bsicos que precisaba. Termin
por llevar una lista exhaustiva de la que antes no dependa, pero an
as se liaba con los cambios monetarios y en ltimo trmino tena que
ir siempre acompaada, sobre todo cuando acuda a realizar alguna
tarea bancaria. La paciente refera en ocasiones que ya no serva para
nada y a veces se pona triste, olvidaba ingredientes para cocinar o a
veces los condimentaba demasiado. EF: Paciente alerta, con cierta
indiferencia ante el motivo de preocupacin de sus familiares. Escasa
fluidez verbal. Minimental 22 Fallos en orientacin temporal (no
recordaba da del mes ni de la semana), no recordaba 3 palabras a los
cinco minutos. Fallos en la repeticin de dgitos en orden inverso y en
clculo mental simple. Fallo en la comprensin y ejecucin de rdenes
escritas y en copia de grficos. Ausencia de signos menngeos. Pupilas,
oculomotricidad, fondo de ojo y resto de pares craneales sin
alteraciones. TA: 150/80 mm de Hg. T: 36.5 . Ausencia de bocio y
adenopatas en cadenas ganglionares cervicales. Auscultacin
carotdea cervical y cardiorespiratoria sin hallazgos. Abdomen normal.
No existan trastornos trficos ni edemas en miembros.
PREGUNTA
Cul es el diagnstico ms probable en este caso?
RESPUESTA
a.- Enfermedad de Creutzfeldt-Jakob
b.- Demencia frontotemporal
c.- Demencia de cuerpos de Lewi
d.- Enfermedad Alzheimer
PREGUNTA
Cul es la conducta teraputica ms adecuada a seguir con esta
paciente?
RESPUESTA
a.- Inhibidores de acetilcolinesterasa
b.- Inhibidores de los receptores de NMDA
c.- Inhibidores de recaptura de serotonina
d.- Estabilizadores de nimo
PREGUNTA
Por cunto tiempo es ms probable que le sea til este medicamento
a la paciente?
RESPUESTA
a.- Por 10 aos
b.- Todo el tiempo
c.- De 1-3 aos
d.- Solo los primeros meses
PREGUNTA
Se le solicita una tomografa de crneo. Qu es lo ms probable que
esperaramos encontrar?
RESPUESTA
a.- Atrofia de predominio parietal bilateral
b.- Atrofia del putamen
c.- Atrofia de lbulos fronto-medial, temporal anterior
d.- Atrofia occipital y frontal
PREGUNTA
Cules la prevalencia actual en Latinoamrica?
RESPUESTA
a.- 1-2%

Pharmed Solutions Institute

PGINA 129

MANUAL DE TRABAJO DEL CURSO ENARM CMN SIGLO XXI


b.- 3-6%
c.- 8-10%
d.- 0.3-0.9%

b.- Atrofia de predominio frontal con dilatacin de astas ventriculares


frontales
c.- Atrofia corteza y caudado
d.- Atrofia occipital y frontal

CASO CLINICO
Masculino de 62 aos de edad, sin antecedentes personales de inters
y con padre fallecido tras demencia a los 73 aos, acude a consulta
acompaado de sus familiares los cuales se mostraban poco seguros o
imprecisos a la hora de sealar el momento aproximado de
instauracin de los sntomas. Refieren que su padre dejaba de
comportarse en las reuniones sociales con la correccin de la que haca
gala, pero lo atribuan inicialmente a que ellos mismos le haban
recriminado previamente una comportamiento tan "pulcro". Sin
embargo, empezaba a descuidar su forma de vestir y ltimamente
tambin se mostraba bastante irregular en el cuidado de su higiene
personal. A pesar de ello, mantena independencia en el desempeo
de sus actividades. Unos 6 meses antes de la consulta, haba sido
multado en sendas ocasiones por exceso de velocidad y por circular en
direccin prohibida. En los 3 ltimos meses fumaba excesivamente
(ms de 3 cajetillas al da), y abandonaba los cigarrillos encendidos en
los platos mientras coma. Cuando se le reprenda dicha actitud, rea
de forma sonora, haca comentarios jocosos y pareca olvidarlo en
poco tiempo, reincidiendo en esta actitud. Su lenguaje fue
reducindose a unas cuantas frases que repeta a veces de forma
estereotipada. El examen neuropsicolgico result bastante difcil por
falta de colaboracin, respuestas imprecisas y precipitadas a los
diversos tems. Se mostraba bien orientado tmporo-espacialmente,
sin alteraciones nominativas ni en la repeticin del lenguaje. El clculo
mental simple estaba preservado. Fallaba en interpretacin de
similitudes entre diversos objetos y pensamiento abstracto. Reduccin
en test de fluidez verbal. Se evidenciaban reflejos primitivos de lnea
media (palmomentoniano y de prensin). El resto del examen no
mostraba datos de inters salvo un discreto aumento del tono
muscular en los cuatro miembros.
PREGUNTA
Cul es el diagnstico ms probable para este caso?
RESPUESTA
a.- Demencia de cuerpos de Levi
b.- Demencia Alzheimer
c.- Demencia vascular
d.- Demencia frontotemporal
PREGUNTA
Qu sustancias ms probablemente participen en la formacin de
inclusiones en este caso?
RESPUESTA
a.- beta amiloide
b.- TAU y ubiquitina
c.- alfa-sinucleina
d.- Binswanger
PREGUNTA
Se le solicita una tomografa. Qu es lo ms probable que esperamos
encontrar en este caso?
RESPUESTA
a.- Atrofia de putamen

CURSO ENARM CMN SIGLO XXI TEL: 36246001

CASO CLINICO
Masculino de 76 aos de edad, con sobrepeso, fumador de 30
cigarrillos al da y en tratamiento con enalapril por hipertensin
arterial, con cumplimiento irregular del mismo. Se remite a consulta
por dificultades para la marcha de 5 meses de evolucin, que le han
ocasionado diversas cadas. Unos 3 meses antes tuvo un cuadro de
debilidad en brazo y pierna derechos que se resolvi parcialmente en
unas semanas. Le prescribieron tratamiento con aspirina, pero lo
suspendi por molestias gstricas. Posteriormente, estuvo varios das
con dificultades para la articulacin de las palabras y reapareci la
torpeza en la mano derecha. En los 2 ltimos meses presenta episodios
de incontinencia esfinteriana. Ha tenido dificultades deglutorias
fundamentalmente con lquidos, que le han producido accesos de tos.
Los familiares advierten adems prdida de inters en sus actividades
previas, habiendo abandonado la mayor parte de sus aficiones. En
ocasiones su familia lo encuentra como "ausente", poco comunicativo.
EF: Paciente alerta, con escaso lenguaje espontneo, mal articulado.
Desorientado parcialmente en tiempo. No hay signos menngeos.
Pupilas iguales y reactivas a la luz y acomodacin, ligera limitacin en
mirada vertical hacia arriba. El fondo de ojo no muestra papiledema,
pero hay signos de cruce arteriovenoso. Hemiparesia derecha (4+/ 5)
con hiperreflexia generalizada y aumento de tono fundamentalmente
en extremidades inferiores. Reflejo cutneo-plantar derecho extensor.
No se constata dficit sensitivo. Ligera dismetra derecha en prueba
ndice-nariz. Ausencia de temblor u otros movimientos involuntarios.
Bipedestacin estable. Marcha a pequeos pasos, con discreto
aumento de base de sustentacin.
PREGUNTA
Cul el diagnstico ms probable para este caso?
RESPUESTA
a.- Evento cerebral vascular isqumico
b.- Evento cerebral vascular hemorrgico
c.- Demencia vascular
d.- Demencia de Pick
PREGUNTA
Cul sera el manejo preventivo ms adecuado para este caso?
RESPUESTA
a.- Rivastigmina
b.- Dejar de fumar, bajar de peso y control de la presin
c.- Manejo de clopidrogel
d.- Heparina
PREGUNTA
Cul es la causa menos probable del cuadro clnico que presenta el
paciente?
RESPUESTA
a.- Sndrome de Binswagener
b.- Infarto lacunar
c.- Encefalopata subcortical
d.- Participacin de priones

Pharmed Solutions Institute

PGINA 130

MANUAL DE TRABAJO DEL CURSO ENARM CMN SIGLO XXI


TRASTORNOS DEL MOVIMIENTO Y ATAXIAS:
CIENCIAS BASICAS: Los trastornos del movimiento constituyen una patologa relativamente frecuente en la prctica mdica diaria, tanto neurolgica
como en Atencin Primaria, y se pueden presentar como estados hipercinticos o hipocinticos. El prototipo de trastorno hipocintico es la enfermedad
de Parkinson y tambin la representa la patologa ms frecuente dentro de los trastornos del movimiento. En general, los trastornos del movimiento
tienen como sustrato anatmico comn los ncleos de los ganglios basales. Adems, entre la lista de efectos secundarios relacionados con una gran
variedad de frmacos de uso comn entre la poblacin, est el desarrollo de trastornos del movimiento. SALUD PUBLICA: La enfermedad de Parkinson
se presenta con la misma incidencia en todo el mundo y afecta por igual a ambos sexos. En el 5 al 10% de las personas que sufren dicha patologa, sta
aparece antes de los 40 aos de edad, en su forma precoz. La edad media de inicio es aproximadamente de 65 aos. Sndrome de piernas inquietas
afecta al 5-10% de las personas entre 18 y 65 aos, y aumenta hasta el 15-20% en los mayores de esa edad. Enfermedad de Huntington suele iniciarse de
forma insidiosa hacia la cuarta dcada de la vida, tiene una prevalencia de 7-10 casos/100.000 habitantes y afecta por igual a ambos sexos ENFERMEDAD
DE PARKINSON: Idioptica, afeccin degenerativa que combina una bradicinesia asimtrica, tpica facies inexpresiva, hipocinesia y rigidez (rueda
dentada), algunas veces acompaadas de temblor en reposo y alteraciones posturales. El temblor, inicialmente intermitente, suele comenzar en una
mano para extenderse luego a las extremidades restantes y, a veces, a los msculos de la cara y del cuello. El temblor clsico de la enfermedad de
Parkinson es regular y rtmico. Est presente en reposo y disminuye o desaparece con el movimiento de la zona afectada. El temblor aumenta con la
ansiedad y cesa durante el sueo. Con frecuencia, el cuadro se acompaa de alteraciones en el intelecto, aunque de forma leve, lo que incluye
afectaciones cognitivas, de percepcin, de memoria y de expresin. Un 15% puede verse afectado de demencia, sobre todo en los casos en los que la
enfermedad se ha iniciado tardamente. La afeccin del estado emocional, con tendencia a la depresin, afecta hasta un 40% de los casos. Pueden
presentar, en el 40% de los casos, sntomas de disfuncin vegetativa, que incluyen estreimiento, a veces grave, hiperhidrosis, sofocaciones y
alteraciones en la miccin (incontinencia, polaquiuria y enuresis nocturna). La sialorrea es frecuente y se debe al defecto de la deglucin. Los pacientes
pueden afectarse de forma diferente debido a una combinacin de factores genticos y ambientales, entre los que podemos citar: virus, toxinas, el
consumo de aguas no potables, la vitamina E y el hbito tabquico (el tabaquismo no influye de forma negativa al igual que el consumo de vitamina E),
una vez ms, presente, como responsable de la puesta en marcha de un pro- ceso patolgico. Los familiares en primer grado de consanguinidad,
duplican el riesgo de desarrollar la enfermedad (un 17% de probabilidad, a lo largo de su vida), comparndolos con la poblacin general. Los criterios de
diagnstico clnico tienen una sensibilidad de 80% y una especificidad del 30%, comparado con el patrn de oro del diagnstico de la autopsia. La
patologa primaria consiste en la prdida progresiva de clulas de la sustancia negra del tallo cerebral donde se produce el neurotransmisor dopamina. El
tratamiento est obligado a reemplazar la prdida de dopamina. Tambin se ven afectados varios sistemas de neurotransmisores catecolaminrgicos.
Las pruebas, como la RMC, suelen ser normales. Por el contrario, en las pruebas de neuroimagen funcional, como el PET y el SPECT cerebral, se puede
observar una reduccin asimtrica de la captacin estriatal de 18- fluorodopa o de DAT (transportador de dopamina), respectivamente, alteraciones que
no se observan en pacientes con temblor esencial o con parkinsonismo inducido por frmacos. El tratamiento mdico suele ser eficaz y debe de
continuarse durante el resto de la vida del ciudadano. Se fundamenta en las siguientes medidas: Anticolinrgicos (con la intencin de corregir la
hiperfuncin de las clulas colinrgicas del ncleo estriado y actan bloqueando los receptores muscarnicos centrales)-Amantadina (posee propiedades
anticolinrgicas y dopaminrgicas, ya que estimula la liberacin de dopamina en las terminaciones nerviosas) -L-DOPA (frmaco ms eficaz, traspasa con
facilidad barrear hematoenceflica)-Otros frmacos dopaminrgicos-Medidas generales-Tratamiento quirrgico PARKINSONISMOS: producidos por
ejemplo medicamentosos, reciben el nombre de secundarios o sintomticos. Los secundarios incluyen aquellos producidos por infecciones (encefalitis
letrgica), intoxicaciones (manganeso, monxido de carbono, MPTP), administracin de frmacos (neurolpticos, cinarizina), tumores, traumatismos,
infartos cerebrales profundos y calcificaciones de los ganglios basales. Estos sntomas se inician en la quinta o sexta dcadas de la vida y son de evolucin
progresiva. La falta de equilibrio y las molestias visuales constituyen los sntomas iniciales ms frecuentes. SNDROME DE PIERNAS INQUIETAS:
Trastorno neurolgico del movimiento, caracterizado por la aparicin de molestias inespecficas en la parte inferior de las piernas durante el reposo,
cuando el paciente est sentado o tumbado, es especialmente frecuente durante el inicio del reposo nocturno, cuando el paciente se acuesta. Consiste
en una sensacin desagradable y difcil de calificar, en forma de parestesias, disestesias, hormigueos, pinchazos, desasosiego o dolor que lleva al
paciente a la necesidad irresistible de mover las piernas (inquietud motriz), levantarse, pasear, darse masajes e incluso baos con agua fra, en un
intento de alivio pasajero. Hay movimientos peridicos en las extremidades, en el que el paciente realiza movimientos bruscos de las piernas, repetitivos
y estereotipados, cada 10-60 seg., a veces durante toda la noche, involuntariamente. Al impedir un descanso nocturno adecuado puede manifestarse
con cansancio y somnolencia diurna e insomnio. Los frmacos con actividad dopaminrgica y en concreto los agonistas dopaminrgicos (pergolida,
pramipexol y ropinirol), se consideran el tratamiento de eleccin. Ropinirol ha demostrado en varios estudios que reduce los movimientos peridicos de
las piernas, y mejora la eficiencia del sueo en relacin con el perodo basal. COREAS: Significa baile o danza. Se utiliza para designar movimientos
involuntarios, irregulares, de duracin breve y de baja amplitud, que no son predecibles y no tienen una finalidad aparente; fluyen de una parte del
cuerpo a otra sin una secuencia definida. Suelen localizarse en las partes distales de los miembros, generalmente en las manos o en la cara, y suelen
asociarse a hipotona o a impersistencia motora: dificultad para mantener una contraccin muscular sostenida. Los sndromes coreicos suelen
relacionarse con lesiones o alteraciones funcionales del neoestriado (caudado y putamen). Su fisiopatologa, desde el punto de vista neuroqumico,
responde a una situacin de hiperactividad dopaminrgica. Esto viene apoyado por la eficacia teraputica de los frmacos antidopaminrgicos y por la
capacidad de agravar o incluso inducir estos trastornos, que poseen los agonistas de la dopamina. ENFERMEDAD DE HUNTINGTON: Es la forma de corea
hereditaria ms frecuente. Enfermedad neurodegenerativa que se transmite de forma autosmica dominante con penetrancia completa, precisando un
solo gen defectuoso en uno de los progenitores. Los hijos de una persona afectada, tienen el 50% de probabilidad de padecer la enfermedad. Hay atrofia
cortical relacionada con el grado de evolucin de la enfermedad, y tambin atrofia estriatal, fundamentalmente a nivel del ncleo caudado. Las neuronas
afectadas por el proceso degenerativo son las de mediano tamao. La disfuncin estriatal, contraria a la que caracteriza a la enfermedad de Parkinson, es
la responsable de que aparezcan las hipercinesias; y la degeneracin neocortical y del ncleo caudado es la responsable de que aparezca demencia. El
cuadro clnico incluye sntomas motores, trastornos psiquitricos y alteraciones cognitivas: Sntomas motores: movimientos coreicos, que afectan
inicialmente a la parte distal de las extremidades y posteriormente se van extendiendo a la musculatura craneal, farngea y larngea. Las anormalidades
en la motilidad ocular, con dificultad para seguir con la mirada un objeto mvil, posturas distnicas axiales y de las extremidades, as como disfagia y
disartria coreica por movimientos coreoatetsicos de los labios, la lengua, el paladar y el diafragma. Trastornos psiquitricos: en ocasiones son la primera
manifestacin de la enfermedad, cambios en la personalidad, depresin, apata, agitacin, mana, alucinaciones, insomnio. En casos graves es frecuente
el suicidio; ms del 30% lo intentan durante el desarrollo de la enfermedad. Dficit de la memoria, la atencin, la concentracin y el aprendizaje; pueden
llegar a una demencia con afectacin de las funciones frontales: trastorno de la atencin y de las funciones ejecutivas, sin afasia, apraxia o agnosia. El
diagnstico de sospecha se realiza mediante la clnica, la historia familiar positiva y la neuroimagen. La confirmacin por gentica molecular, que tiene
una sensibilidad diagnstica del 98% y puede utilizarse incluso en fase presintomtica. No existe un tratamiento especfico. Se estn realizando ensayos
con GABA, baclofen, ACTH y antagonistas selectivos de los receptores D2. COREA DE SYDENHAM: Es un cuadro de corea aguda, que aparece
generalmente en la infancia en relacin con una infeccin por el estreptococo betahemoltico. Est considerada como uno de los criterios mayores de la

CURSO ENARM CMN SIGLO XXI TEL: 36246001

Pharmed Solutions Institute

PGINA 131

MANUAL DE TRABAJO DEL CURSO ENARM CMN SIGLO XXI


fiebre reumtica. Su fisiopatologa parece estar relacionada con un proceso autoinmune, en el que los anticuerpos antibacterianos provocan reacciones
cruzadas con los antgenos tisulares en el cuerpo estriado. Los sntomas pueden aparecer hasta seis meses despus de una infeccin por el estreptococo,
que a veces no se consigue documentar y que hace el diagnstico ms difcil.
CASO CLINICO
Mujer de 58 aos que presenta un cuadro de torpeza generalizada,
lentitud y temblor postural en miembros superiores de dos aos de
evolucin. Dos aos presenta de forma fluctuante: conducta impulsiva,
cambios de carcter con violencia y agresividad, ataques de llanto
repentino y comportamiento infantil. Presenta asimismo dificultades
en concentracin, poca capacidad de comprensin y razonamiento y
dificultad para hablar y expresarse. En ocasiones confunde ideas
simples, tambin nombres de objetos y personas. Comienza a perder
las rutinas de su vida diaria, mantiene poca relacin con el entorno y
muestra falta de cuidado personal. En la exploracin neurolgica se
objetiva: clara alteracin en prueba de secuencias alternantes y en
patrones motores alternantes unilaterales (triple maniobra de Luria),
lenguaje muy pobre con carencia de sustantivos y confusiones
frecuentes, desorientacin parcial en tiempo y espacio, movimientos
extraoculares pobres (aunque sin limitacin) junto rigidez y
bradicinesia generalizadas.
PREGUNTA
Cual es el diagnostico mas probable?
RESPUESTA
a.- Enfermedad de Huntington
b.- Enfermedad de Sydenham.
c.- Enfermedad de Parkinson.
d.- Enfermedad de Piernas inquietas.
CASO CLINICO
Paciente de 76 aos que acude a la consulta diciendo que sufre de
temblor en las manos, sobre todo la derecha, desde hace meses. Lo
presenta de forma constante, No refiere otra sintomatologa. El
paciente presenta temblor en ambas manos, sobre todo la derecha, de
reposo, que mejora con la realizacin de movimientos voluntarios,
inexpresividad facial, dificultad para levantarse del asiento y lentitud al
caminar. La movilizacin de las extremidades muestra aumento del
tono en las piernas y rigidez en rueda dentada en los brazos. Se trata,
por tanto, de un paciente que consulta por temblor en las manos.
PREGUNTA
Considerando el diagnostico cual es el tratamiento mas apropiado?
RESPUESTA
a.- Pergolida.
b.- Penicilina.
c.- Baclofen.
d.- L-dopa.
CASO CLINICO (EH)
Masculino de 54 aos, referido por presentar intentos suicidas
reiterativos (3 intentos) en un lapso de 2 meses. El ltimo de ellos
realizado en las ltimas horas, al arrojarse a un pozo, slo
provocndose ligerascontusiones en diversas partes del cuerpo. El
padecimiento inicia aproximadamente hace 4 aos. De forma
paulatina comienza a presentar miedo a la obscuridad, que cada vez
aumentaba en intensidad, desde hace dos aos se da la aparicin de
movimientos involuntarios en la frente (gesticulaciones), y sin motivos
aparentes comienza con decaimiento del estado del nimo,
aislacionismo, mutismo por das, para luego regresar a su estado
normal, present adems irritabilidad fcil, as como ideas de
desesperanza, refiriendo que su vida no tena sentido, transcurre
aproximadamente un ao y seis meses, con poca afectacin en su vida
laboral, social y de familia, durante este tiempo comienza a presentar
algunos movimientos involuntarios en manos y en cuello de tipo
espasmdicos, hacindose ms evidentes los movimientos en cara,
dichos movimientos se presentan de forma peridica y repetitiva.

CURSO ENARM CMN SIGLO XXI TEL: 36246001

PREGUNTA
Cual es el diagnostico mas probable?
RESPUESTA
a.- Enfermedad de Huntington
b.- Enfermedad de Sydenham.
c.- Enfermedad de Parkinson.
d.- Enfermedad de Piernas inquietas
CASO CLINICO
Varn de 56 aos con antecedentes de hipertensin arterial esencial
desde los 40 aos, en tratamiento con enalapril, 20 mg al da;
amigdalectoma y apendicectoma en la infancia. En cuanto a sus
hbitos txicos, ha sido fumador de unos 15 cigarrillos al da hasta
hace 5 aos y consume habitualmente alcohol. Hace 5 aos atrs,
comenz con problemas de estreimiento moderado, dificultad para
deglutir lquidos y problemas leves en la deambulacin, especialmente
identificados como dificultad para continuar la marcha en algunas
ocasiones. Ninguno de estos sntomas le preocup lo suficiente ni le
incapacitaba gravemente para acudir al mdico. Coincidiendo con esta
clnica, y aunque su aparicin fue progresiva, present bradicinecia y
necesidad de esfuerzo abdominal para realizar la miccin, flujo
miccional dbil y discontinuo. El examen neurolgico, discreto temblor
de reposo en mano derecha, con ligera rigidez. El discurso espontneo
era escaso, hipofnico, con pobre articulacin, pero bien estructurado
y sin transformaciones semnticas, fonticas o parafasias. No haba
defectos en repeticin ni en nominacin. El resto del examen solo
evidenciaba escaso braceo derecho durante la marcha y una tendencia
a la inestabilidad postural tras la maniobra del empujn.
PREGUNTA
Cul es el diagnstico ms probable en este caso?
RESPUESTA
a.- Enfermedad de Parkinson
b.- Enfermedad de Huntington
c.- Corea de Syndeham
d.- Demencia de cuerpos de Lewy
PREGUNTA
Cul es el manejo ms adecuado para esta patologa?
RESPUESTA
a.- levodopa-carvidopa
b.- Baclofen, risperidona
c.- Antibitico,
d.- Risperidona, memantina
PREGUNTA
Qu medicamento tendra cierta utilidad, para ayudar con la
bradicinecia que presenta el paciente?
RESPUESTA
a.- Penicilina
b.- Haloperidol
c.- Diazepam
d.- Amantadina
PREGUNTA
Cul es la base fisiopatolgica ms probable para esta patologa?
RESPUESTA
a.- Atrofia frontotemporal
b.- Prdida progresiva de clulas de la sustancia negra del tallo
cerebral
c.- Prdida neuronal y gliosis del ncleo caudado
d.- Formaciones de alfa-sinucleina

Pharmed Solutions Institute

PGINA 132

MANUAL DE TRABAJO DEL CURSO ENARM CMN SIGLO XXI


ENFERMEDADES NEUROMUSCULARES (ENM)
CIENCIAS BASICAS: Son enfermedades de carcter gentico y generalmente hereditario que afectan a la musculatura y al sistema nervioso. Tambin se
conocen con el nombre de miopatas. Su aparicin puede producirse tanto en el nacimiento como en otras etapas de la vida. Las ENM son trastornos de
la unidad motora, cuyos principales sntomas son la debilidad muscular, la fatiga, calambres, el dolor, problemas articulares y la rigidez. Su distribucin
suele ser simtrica, a diferencia de lo que ocurre en las enfermedades que afectan al sistema nervioso central (SNC). La debilidad de los msculos
respiratorios es la causa bsica que conduce al fracaso de la bomba ventilatoria, y que se traduce en hipoxemia e hipercapnia. Sin embargo, el
compromiso del sistema respiratorio no es igual en todas las entidades sino que est determinado por el grado de afeccin muscular y por la progresin
de la enfermedad. 1. Distrofias musculares: afectan predominantemente al msculo estriado y son debidas a un defecto alguno de las protenas que
forman parte de la fibra muscular, ya sean estructurales o enzimticas (ejemplos son la distrofina calpana, merosina y emerina, entre otras). 2.
Miopatas distales. 3. Miopatas congnitas. 4. Distrofia miotnica de Steinert. 5. Miotonas congnitas. 6. Parlisis peridicas familiares. 7.
Enfermedades musculares inflamatorias. 8. Miositis osificante progresiva. 9. Miopatas metablicas. 10. Enfermedades de la unin neuromuscular. 11.
Amiotrofias espinales. 12. Neuropatas hereditarias sensitivo-motoras (enfermedades de Charcot-Marie-tooth). ESCLEROSIS LATERAL AMIOTRFICA
(ELA): Tiene una incidencia anual de 1-2 casos/100.000 habitantes y su sustrato patolgico es la degeneracin de las neuronas motoras de la mdula
espinal, el tronco cerebral y el crtex motor. Clnicamente se caracteriza por espasticidad e hiperreflexia al inicio de la enfermedad, pero a medida que
progresa se establecen otros sntomas, como amiotrofia asimtrica, debilidad muscular, fasciculaciones y sndrome bulbar. La debilidad de los msculos
respiratorios, fundamentalmente de los intercostales y del diafragma, es la causa de la hipoventilacin, y los sntomas respiratorios aparecen cuando la
enfermedad est muy evolucionada, lo ms frecuente es que la insuficiencia respiratoria se presente en casos de ELA claramente establecida como
consecuencia de la progresin natural de la enfermedad o de forma aguda, precipitada por una infeccin de tracto respiratorio. De forma ocasional, la
insuficiencia respiratoria puede ser la primera manifestacin de la ELA; en estos casos la lesin afecta fundamentalmente a las motoneuronas del nervio
frnico localizadas en la mdula cervical, y la afeccin bulbar implicar un grave compromiso de la va area superior. ESCLEROSIS MLTIPLE (EM): Esta
caracterizada por la inflamacin crnica (reaccin linfocitaria y mononuclear) y destruccin selectiva de la mielina del SNC (desmielinizante), importante
con conservacin de los axones, el sistema nervioso perifrico est a salvo. Hay dao de los oligodendrocitos con proliferacin astroc ca. Los hallazgos
anteriores junto con la prdida de la mielina cons tuyen la placa de desmielinizacin. Aunque pueden verse en la mdula espinal, el tallo cerebral y el
nervio ptico, la localizacin ms frecuente es la periventricular (90%). Estas tambin pueden encontrarse en la sustancia gris, principalmente en
localizacin subpial. Las placas aisladas raramente miden ms de 1,5 cm. Etiologa se piensa que es autoinmune, con la susceptibilidad determinada por
factores genticos y ambientales. SALUD PBLICA: La prevalencia es muy variada en las latitudes norte la enfermedad es ms frecuente. Por ejemplo en
algunas zonas de Escocia es de 309 por cada 100.000 habitantes, mientras que en el Norte de los Estados Unidos es de 58 por cada 100.000 habitantes y
en Japn, frica y Sudamrica desciende hasta cuatro personas por cada 100.000 habitantes. La incidencia de la enfermedad es casi 1,8 veces mayor en
mujeres que en hombres. Compromete ms la raza blanca y la edad de aparicin ms frecuente est entre los 20 y los 40 aos. ASPECTOS CLINICOS: Lo
ms comn son los ataques recurrentes de disfuncin neurolgica focal, por lo general duran semanas o meses. Los pacientes con la forma de recada
remisin tienen en promedio un episodio de disfuncin neurolgica por ao. Esta es quizs la clasificacin clnica ms reciente de esclerosis mltiple:
Asintomtica (hallazgos incidentales en RMN de cerebro) y la sintomtica que son: 1. Recada remisin; se caracteriza por ataques recurrentes de
disfuncin neurolgica general con o sin recuperacin entre los ataques, no se observa la progresin de la discapacidad neurolgica. Representa el 85 %
de los casos de EM de nueva aparicin. 2. Secundariamente progresiva; no siempre se presenta inicialmente como recada remisin sino que evoluciona
para ser de forma progresiva. 3. Primariamente progresiva; se caracteriza por una progresin gradual de la discapacidad desde el inicio sin ataques
discretos, el 15% de los casos de EM de nueva aparicin. 4. progresiva con exacerbacin; es una forma rara que comienza con un curso de primaria
progresiva, pero se producen recadas despus superpuestas. La EM es una enfermedad crnica, 15 aos despus del diagnstico, slo el 20 % de los
pacientes tienen ninguna limitacin funcional, de un tercio a la mitad habrn progresado y requerirn ayuda para la deambulacin. Los sntomas pueden
desarrollarse de minutos a horas en 40% de los enfermos, durante varios das en 30% y lentamente durante semanas a meses, en 20% de los pacientes.
En la mayor serie de pacientes con EM y disfuncin respiratoria, las complicaciones respiratorias aparecieron a los 9,5 aos de las manifestaciones
neurolgicas y se atribuyeron a debilidad muscular, disfuncin bulbar, trastornos del control respiratorio, hiperventilacin paroxstica y apneas
obstructivas. En las lesiones agudas de la mdula espinal, las complicaciones respiratorias dependen de la extensin y la localizacin de la lesin y se
deben a la interrupcin de la inervacin. Se consideran lesiones altas las que afectan a C1 y C2, y bajas las que se producen entre C3 y C8. Las
motoneuronas que inervan al diafragma se originan entre C3 y C5, por lo que las lesiones medula- res situadas por encima de C3 implican una parlisis
total de la musculatura respiratoria, mientras que en las lesiones entre C3 y C5 la parlisis muscular es parcial. DIAGNOSTICO: Algunos pacientes se
presentan inicialmente con deterioro neurolgico lentamente progresivo. Los sntomas a menudo empeoran transitoriamente con fatiga, el estrs, el
ejercicio o el calor. Las manifestaciones de la EM normalmente incluyen debilidad y/o sntomas sensoriales que implican una extremidad, dificultades
visuales, alteraciones de la marcha y la coordinacin, urgencia o frecuencia urinaria y fatiga anormal. Participacin Motor puede presentarse como una
pesada rigidez dbil o torpe extremidad. Hormigueo localizado, "alfileres y agujas" y sensaciones "muertos" son comunes. La neuritis ptica puede
resultar en visin borrosa, especialmente en el campo visual central, a menudo con dolor retroorbital acentuado por el movimiento del ojo. La
participacin del tronco cerebral puede causar diplopa, nistagmos, vrtigo o dolor facial, entumecimiento, debilidad, hemiespasmo o mioquimia
(ondulacin contracciones musculares). Ataxia, temblor, disartria y siempre son representativas de la enfermedad de las vas cerebelosas. Sntoma de
Lhermitte, una sensacin momentnea elctrica similar al shock provocado por la flexin del cuello, indica enfermedad en la mdula espinal cervical.
CRITERIOS DIAGN STICOS: El diagnstico de esclerosis mltiple es de predominio clnico, y existen algunos criterios, como los de Schumacher, que
todava permanecen vigentes. Con el advenimiento de nuevos estudios paraclnicos, como los potenciales evocados (PE), RMN (revela las reas brillantes
multifocales en las secuencias ponderadas en T2 en > 95 % de los pacientes, a menudo en una ubicacin periventricular) y el estudio de LCR (Hallazgos
pleocitosis linfocitaria leve 5-75 clulas (25 %), bandas oligoclonales (75-90 %), elevacin de IgG 80 %) TRATAMIENTO: En la actualidad, para los brotes o
periodos de exacerbacin de la enfermedad se utilizan la hormona adrenocorticotropa (ACTH) y otros corticoides. La ACTH puede administrarse
siguiendo un protocolo de dosis elevadas de la medicacin que incluye 80 UI diarias intramusculares o intravenosas por siete das, seguidas de 40 UI
intramusculares o intravenosas por cuatro das, finalizado con 20 UI por tres das. En la actualidad, se u liza en la mayora de los centros la
me lprednisolona, a dosis altas, de 500 mg a 1 g IV diarios por tres a cinco das. Se recomienda administrar esta medicacin en infusin, disolvindola en
100 ml de dextrosa al 5%, para ser administrada en 30 minutos a 1 hora. Si la sintomatologa del paciente persiste despus de un mes a pesar de este
tratamiento, se recomienda repetir el ciclo de metilprednisolona, seguido por la administracin de 1 mg/kg de peso de prednisona interdiaria,
reducindola en el curso de 8 a 14 semanas. El interfern beta (reduce las tasas de exacerbacin anual aprox. 30 % y tambin reduce el desarrollo de
nuevas lesiones de MRI) ha demostrado ser muy til en el tratamiento de la esclerosis mltiple especialmente en la forma de recadas y remisin, ha sido
recomendado por la Academia Americana de Neurologa en sus guas de manejo. Terapia sintomtica: la espasticidad puede responder a la terapia fsica,
lioresal (20-120 mg/d), diazepam (2-40 mg/d), tizanidina (8-32 mg/d), dantroleno (25-400 mg/d) y clorhidrato de ciclobenzaprina (10-60 mg/d).
Disestesia puede responder a la carbamazepina (100-1200 mg/da), fenitona (300 mg/d), gabapentina (300-3.600 mg/d), pregabalina (50-300 mg/d) o

CURSO ENARM CMN SIGLO XXI TEL: 36246001

Pharmed Solutions Institute

PGINA 133

MANUAL DE TRABAJO DEL CURSO ENARM CMN SIGLO XXI


amitriptilina (50-200 mg/d). El tratamiento de los sntomas de la vejiga se basa en la fisiopatologa subyacente investigados con pruebas urodinmicas:
hiperreflexia vesical es tratado con restriccin de lquidos por la noche y la miccin frecuente, y si esto no funciona, anticolinrgicos como oxibutinina ( 515 mg/d) puede ser juzgado; hiporreflexia es tratado con el betanecol colinrgica drogas (10-50 mg c/8hrs), y disinergia debido a la prdida de la
coordinacin entre la pared de la vejiga y los msculos del esfnter se trata con anticolinrgicos y cateterismo intermitente. La depresin debe ser
tratada agresivamente.
: Esta enfermedad se observa en pacientes con trastornos linfoprolifera vos,
como la enfermedad de Hodg in, leucemia linfoc ca y linfosarcomas. Tambin puede presentarse en pacientes con sndrome de inmunodeciencia
adquirida (SIDA). El JC papovavirus ha sido encontrado en la mayora de los casos de leucoencefalopa a mul focal progresiva. La enfermedad cursa en
forma subaguda con cambios de personalidad y demencia. Otros sntomas muy comunes son hemianopsia, disartria y ataxia para la marcha. El curso de
la enfermedad es progresivo e inexorablemente lleva al paciente a la muerte en un periodo de algunos meses. El LCR puede ser normal, la TAC de
cerebro y la RMC ayudan mucho para el diagnstico y muestran el compromiso de la sustancia blanca. Un diagnstico definitivo de la enfermedad solo
puede hacerse por patologa.
: Clnicamente se caracteriza por una paraparesia o cuadriparesia de evolucin rpida, con sntomas
seudobulbares, que se manifiestan por disartria y disfagia, en pacientes con
CRITERIOS DE SCHUMACHER:
cambios electrolticos importantes. La causa real de la mielinolisis no es
Examen neurolgico que evidencie compromiso del sistema nervioso
clara, pero se han postulado teoras vasculares y metablicas. El cuadro se
Evidencia clnica de 2 o ms lesiones del sistema nervioso
observa en alcohlicos y pacientes a quienes se les corrige rpidamente una
Compromiso principalmente de la sustancia blanca
hiponatremia. Esta entidad tambin se ha visto asociada a otras patologas,
El compromiso del sistema nervioso debe seguir el siguiente patrn: dos o ms
episodios de la enfermedad con una duracin no menor de 24 hrs y usualmente
como la enfermedad de Wilson, la cirrosis, las nefropatas y la enfermedad
menos de un mes, o una lenta progresin de la enfermedad por un periodo hasta
de Wernicke. Histolgicamente la lesin compromete toda o parte de la base
de 6 meses.
del puente. Puede afectar el tegmento pn co y ascender hasta el
Inicio de los sntomas en edades comprendidas entre 10-50 aos
mesencfalo, produciendo desmielinizacin importante. Para diagnos car la
Los signos y sntomas no deben ser mejor explicados por otra enfermedad
enfermedad son de gran importancia los potenciales evocados audi vos y la
Esclerosis mltiple definitiva apoyada en clnica
Segn clnica, la esclerosis mltiple definitiva deber reunir una de estas
RMC. El tratamiento de estos pacientes deber hacerse con solucin salina,
condiciones:
administrada con prudencia, y restriccin de lquidos. ENCEFALOMIELITIS
2 episodios de la enfermedad y evidencia de 2 lesiones separadas
DISEMINADA AGUDA (ADEM): Fulminante, a menudo devastadora,
2 episodios de la enfermedad, evidencia clnica de una lesin y paraclnica de otra.
enfermedad desmielinizante que tiene un curso monofsico y puede estar
asociada con la inmunizacin antecedente o infeccin. Los signos de enfermedad neurolgica diseminada son siempre presentes (por ejemplo,
hemiparesia o tetraparesia, respuesta plantar extensora, perdidos o reflejos tendinosos hiperactivos, prdida de la sensibilidad, y la participacin de
tronco cerebral). Puede presentarse fiebre, dolor de cabeza, meningismo, letargia progresando al coma y convulsiones. Pleocitosis, generalmente 200
clulas/l, es comn. RM puede revelar extensa realce de la materia blanca en el cerebro y la mdula espinal. El tratamiento inicial es con glucocorticoides
a dosis altas. Los pacientes que no responden pueden beneficiarse de un curso de la plasmafresis o inmunoglobulinas.
CASO CLINICO (EM)
Paciente de 45 aos con un cuadro clnico que comenz hace 2 aos
con ligera disminucin de la fuerza muscular, debilidad e inestabilidad
para la marcha, tuvo una evolucin desfavorable y en la actualidad
muestra falta de coordinacin, alteraciones visuales, rigidez muscular
y trastornos del habla. Acude a nuestro centro para realizarse
Resonancia Magntica por Imgenes de Crneo y Columna cervical,
encontrndose las siguientes alteraciones, en la Resonancia Magntica
por Imgenes (RMI) de Crneo (Secuencias T1, T2 y Flair axial),
observamos imgenes ovoides, algunas miden 6 mm de dimetro,
perpendiculares al eje mayor de los ventrculos que se observan
isointensas y algunas hipointensas en T1 y que en T2 y Flair se
muestran hiperintensas, localizadas nivel del cuerpo calloso, en la
sustancia blanca profunda periventricular y a nivel de la protuberancia
y bulbo; por las alteraciones antes descritas se solicita realizar
Resonancia Magntica por Imgenes (RMI) de columna cervical
(secuencias T1 y T2 sagital y axial T2), donde visualizamos a nivel de
mdula reas focales hiperintensas con muy ligero efecto de masa y
de caractersticas similares a las del encfalo.
PREGUNTA
Considerando la clasificacin de esta patologa. Cual es el grado mas
probable del caso?.
RESPUESTA
a.- 1.
b.- 2.
c.- 3.
d.- 4.
CASO CLINICO (MPC)
Masculino de 68 aos de edad quien, previamente sano, sufre un
accidente de trnsito al ser atropellado; producindose politrauma
severo y fractura de base de crneo, fractura expuesta del tercio
medio de la tibia derecha y trauma de la rodilla izquierda. Al ingreso
Glasgow 3 se describe pupila izquierda media no reactiva,
sospechando un III par craneal compresivo se realiza TC que corrobora
la fractura y demuestra una hemorragia subaracnoidea sin hematoma;

CURSO ENARM CMN SIGLO XXI TEL: 36246001

se instaura tratamiento con fenitona y diurticos; se considera que


presenta un III par craneal perifrico traumtico. En 12 horas Glasgow
7. A los 2 das movilizaba las 4 extremidades y abra los ojos al llamado,
no acataba rdenes; posteriormente su sensorio mejor manteniendo
un Glasgow de 14. Durante la estancia el paciente recibi varios
esquemas de antibiticos, debido a neumona nosocomial asociada a
ventilacin mecnica la cual requiri los tres primeros das. Al mes de
estancia presenta progresivo descenso de la natremia con valores
desde 126 mmol/l, hasta 108 a las dos semanas cuando se inici el
tratamiento de reposicin; concomitantemente el paciente tena
progresivo deterioro del sensorio. Durante las primeras horas la
correccin se logr llevar a un ritmo adecuado, pero no ocurri as a
las 12 horas de reposicin que se pas de un sodio de 116 a 125
mmol/l en 5 horas. A los 2 das el paciente nuevamente conciente pero
se anota que presenta dificultad para la deglucin; luego presenta un
cuadro de diarrea asociada a antibiticos que lo hace llegar a una
natremia de 138 mmol/l.
PREGUNTA
Cual es la velocidad de correccin de sodio para evitar esta
complicacin del caso?
RESPUESTA
a.- 05 mmol/l/dia.
b.- 10 mmol/l/dia.
c.- 15 mmol/l/da.
d.- 20 mmol/l/dia.
CASO CLINICO (LMP)
Paciente de sexo masculino, 44 aos, diestro, infeccin por VIH, en
tratamiento antirretroviral. Presentaba una poblacin linfocitaria con
CD4 inferior a 160 elementos/mm3. Comienza con un sndrome
confusional, alteraciones conductuales y cefalea moderada. A los
pocos das de su ingreso se percibe prdida de fuerza de sus miembros
izquierdos y alteraciones del habla que se agravaron en el curso de los
das. No se detecta fiebre. El examen mostraba un paciente irritable,
con desorientacin temporal y espacial, tendencia al mutismo,
sndrome tnico frontal bilateral predominando a derecha. No se

Pharmed Solutions Institute

PGINA 134

MANUAL DE TRABAJO DEL CURSO ENARM CMN SIGLO XXI


detect sndrome menngeo ni fiebre. La TC craneal en fase de estado
mostr una lesin frontal derecha y de cuerpo calloso, con
compromiso menor del lbulo frontal izquierdo que incida
especialmente sobre la sustancia blanca, sin efecto de masa y con
realce con el contraste. La RM enceflica delimit con mayor exactitud
las lesiones observndose lesiones de baja seal en T1 y de aumento
de seal en T2. Se resuelve practicar biopsia estereotxica, Hay
acmulos de histiocitos de citoplasmas microvacuolados por
fagocitosis de restos mielnicos lipdicos, Existe moderada astrogliosis
con astrocitos de aspecto bizarro smil neoplsicos, con ncleos
grandes, algunos multilobulados que no se acompaan de aumento de
densidad celular. La oligodendrogla muestra una cariomegalia
hipercuomtica y algunos de ellos tienen gruesas inclusiones
intranucleares acidfilas 'en vidrio esmerilado' que contienen el
antgeno viral. Hay escaso o nulo exudado inflamatorio perivascular
linfo-plasmocitario".
PREGUNTA
Considerando la clnica y los hallazgos histolgicos, cual es el
diagnostico mas apropiado?
RESPUESTA
a.- Leucoencefalopa a mul focal progresiva.
b.- Encefalomielitis diseminada aguda.
c.- Mielinlisis pn ca.
d.- Escleroris lateral amniotrofica.
CASO CLINICO (EAD)
Paciente de 21 aos, con antecedentes de pan-colitis ulcerosa, en
tratamiento esteroidal con prednisona 60 mg/da y Mesalazina 1 gr c/6
hrs durante 5 meses previos al ingreso. Das previos al diagnstico
complet tratamiento con ciprofloxacino por sndrome diarreico
presuntamente bacteriano, sin confirmacin etiolgica y con respuesta
favorable. Ingres al Servicio de Urgencia por cuadro de 48 horas de
evolucin de compromiso de conciencia caracterizado por
desorientacin temporo-espacial y somnolencia, adems de
deposiciones sanguinolientas. Al examen fsico general presentaba
parmetros hemodinmicos dentro de rangos normales, afebril y como
nico hallazgo se describe candidiasis orofarngea. Al examen
neurolgico el paciente se encuentra vigil, con tendencia a la
somnolencia, desorientado en tiempo y espacio, nistagmo vertical,
prueba de pequea paresia positiva a derecha, y signo de Brudzinski.
Resto del examen sin hallazgos. Se realiza TAC de cerebro que no
muestra hallazgos patolgicos y se realiza puncin lumbar que da
salida a lquido cefalorraqudeo claro, con protenas 74,8/mm3,
glucorraquia normal, glbulos rojos 2/mm3, leucocitos 760/ mm3 de
predominio linfoctico. Por antecedente de inmunosupresin crnica y
candidiasis orofarngea al ingreso, se inicia tratamiento antibitico con
ampicilina+ceftriaxona+aciclovir y corticoides en dosis de estrs
(Hidrocortisona 100mg c/8 hrs). Evoluciona en buenas condiciones
generales, estable desde el punto de vista hemodinmico y
recuperando conciencia a las 48 horas. Se realiza Resonancia
magntica que muestra imgenes con compromiso talmico bilateral
asimtico asociado a compromiso capsular posterior izquierdo con
discreta captacin de contraste.
PREGUNTA
Considerando la clnica y los estudios de imagen, cual es el
diagnostico mas apropiado?
RESPUESTA
a.- Leucoencefalopa a mul focal progresiva.
b.- Encefalomielitis diseminada aguda.
c.- Mielinlisis pn ca.
d.- Esclerosis lateral amniotrofica.
CASO CLINICO ELA

CURSO ENARM CMN SIGLO XXI TEL: 36246001

Se trata de masculino de 65 aos de edad el cual acude debido a que


desde hace cuatro meses presenta debilidad distal asimtrica, adems
refiere contracturas musculares y fasciculaciones que se presentan con
los movimientos voluntarios, a la exploracin fsica se observa
debilidad extensora de las manos as como dificultad para mover la
lengua y la cara, los reflejos de estiramiento muscular se observan
incrementados, la exploracin de la sensibilidad no se observan
alteraciones, al examen mental se encuentra pensamiento y lenguaje
con curso y contenido adecuado, humor y afecto conservados, resto
de funciones mentales superiores estn intactas.
PREGUNTA
Cul es el diagnostico clnico ms probable?
RESPUESTA
a.- Esclerosis lateral amniotrofica.
b.- Esclerosis multiple.
c.- Mielitis transversa.
d.- Sndrome de guillain-barre.
CASO CLINICO
Paciente femenina de 45 aos de edad, consulta por presentar un
cuadro clnico de aproximadamente un ao de evolucin, fatiga, visin
borrosa o doble, cambios en la coloracin de las manos (palidez,
cianosis y eritema), con el fro, sensacin de manos edematosas,
disfagia baja, regurgitaciones cidas, tos seca, disnea y constipacin,
as como debilidad y parestesias de predominio en miembros
inferiores, lo que en ocasiones le dificultaba incluso ponerse de pie as
como urgencia urinaria. A la exploracin clnica presenta disartria,
dismetra y disdiadococinesia, disminucin de fuerza muscular en
miembro plvico izquierdo 3/5, resto de extremidades 5/5, Babinski
presente bilateral, prueba de Romberg positiva
PREGUNTA
Cul es el diagnstico ms probable en este caso?
RESPUESTA
a.- Leucoencefalopata multifocal progresiva
b.- Encefalomielitis diseminada aguda
c.- Miastenia gravis
d.- Esclerosis mltiple
PREGUNTA
Cul sera la conducta teraputica ms adecuada seguir en este caso?
RESPUESTA
a.- Acetato de glatiramer mas metilprednisolona
b.- Metilprednisolona mas interfern
c.- Succinilcolina mas corticoesteroides
d.- Oxibutinina mas acetato de glatiramer
PREGUNTA
Cul es la alteracin que ms probablemente este causando su
problema visual?
RESPUESTA
a.- Iridociclitis
b.- Neuritis retrobulbar
c.- Uvetis posterior
d.- Edema macular
PREGUNTA
Cul de los siguientes es menos probable que nos sea de utilidad para
mejorar la espasticidad?
RESPUESTA
a.- Baclofen
b.- Diazepam
c.- Carbamacepina
d.- Tizanidina

Pharmed Solutions Institute

PGINA 135

MANUAL DE TRABAJO DEL CURSO ENARM CMN SIGLO XXI


MIASTENIA GRAVIS (MG):
CIENCIAS BASICAS: Enfermedad autoinmune. Se presentan anticuerpos contra el receptor de acetilcolina (AchR) que interrumpen la funcin de la
acetilcolina en la unin neuromuscular presentando en debilidad muscular. Las complicaciones asociadas a falla respiratoria son la causa de muerte en
paciente con Miastenia Gravis. SALUD PUBLICA: La incidencia es de menos de 1 caso por 100 000. Es ms comn en mujeres durante la segunda y
tercera dcada de la vida. Sin embargo en la Sptima y Octava dcada de la vida es ms comn en hombres. Los Afroamericanos tambin tienen una
incidencia alta. PATOGENIA: La contraccin muscular ocurre cuando la acetilcolina es liberada por una fibra nerviosa y se une al receptor de acetilcolina
(AchR) de una fibra muscular. En la MG, la fibra nerviosa es normal, sin embargo, el nmero y funcin de los AchR nicotnicos del msculo esqueltico
estn disminuidos. Los sntomas aparecen cuando el nmero de AchR esta disminuidos por debajo del 30% de lo normal. Los msculos cardiaco y liso no
se afectan por que ellos tiene diferente antigenicidad que el msculo esqueltico. La hiperplasia del timo o el timoma se ha visto en el 75% de los
pacientes. Por lo tanto, se sospecha que el timo es el sitio de produccin de anticuerpos, pero el estmulo que inicia el proceso autoinmune es
desconocido. La morbilidad por la debilidad muscular intermitente incluye la neumona por aspiracin y cadas. Los sntomas empeoran en el da
progresivamente con el uso repetitivo de grupos musculares como leer o masticar. Los nervios craneales son los ms comnmente afectados, resultando
en ptosis, diplopa, debilidad facial, disfagia y disartria. La respuesta pupilar a la luz permanece intacta. La debilidad muscular de las extremidades
proximales empeora con el movimiento y mejora con el descanso. Los reflejos tendinosos profundos pueden estar disminuidos pero nunca ausentes. No
hay dficit en la funcin cerebelar y sensorial. La crisis miastnica es el grado ms severo de MG por que la debilidad extrema de los msculos
respiratorios resulta en falla respiratoria requiriendo soporte ventilatorio. Otras enfermedades que se pueden presentar con sntomas similares son el
botulismo, hipotiroidismo y lesiones de la masa intracraneal. DIAGNOSTICO: incluyen la prueba de edrofonium, electromielograma con estimulacin
repetitiva del nervio y prueba serolgica para anticuerpos anti AchR. El cloruro de edrofonium es un inhibidor de la colinesterasa de corta accin que
incrementa la cantidad de acetilcolina en la unin neuromuscular. Este incremento vence el bloqueo de los receptores y la ptosis, oftalmopleja y la
debilidad muscular se corrige en 30 segundos pero regresa a lo basal en 1 a 2 minutos. La prueba de anticuerpos contra AchR es la prueba ms especfica
para MG. El aspecto ms importante para los mdicos del departamento de urgencias es que en todos pacientes gravemente enfermos, la prioridad es
establecer y mantener una ruta area y asegurar respiracin adecuada. La Intubacin endotraqueal por la induccin rpida de la sucesin y intubacin
(IRS) puede ser necesario es antes de ser capaz de diferenciar miastenia gravis contra una crisis de colinergica. El preferido es la Succinilcolina. Los
pacientes con MG AchRs resistentes al succinilcolina y dosis ms altas (2 mg/kg) debe ser usado para inducir la parlisis. Los agentes no despolarizantes
como (rocuronium o vecuronium) son los agentes paraliticos preferidos. Estos agentes no causan la parlisis prolongada en una dosis de 50% la dosis
recomendada. La dosis en adultos de edrofonium es 2 mg IV lento. Sin presentar ninguna respuesta ni ningn efecto adverso de tipo colinergico,
administrando 8 mg IV lento para una dosis total de 10 mg, las contraindicaciones del edrofonium es obstruccin gastrointestinal u obstruccin del
trecho urinario. DISTROFIA MUSCULAR PROGRESIVA, O ENFERMEDAD DE DUCHENNE: Es una enfermedad de herencia recesiva ligada al cromosoma X
causada por mutaciones en el gen de una protena del citosqueleto: la distrofina. Es una miopata proximal que comienza en la infancia y en la que,
inicialmente, se afectan los msculos de las extremidades inferiores (cintura pelviana), de modo que el paciente refiere dificultad para correr o saltar y, a
medida que progresa, afecta a los msculos de las extremidades superiores y del tronco; as, los enfermos a los 12 aos de edad son dependientes por
completo. Tambin es caracterstica de la enfermedad la miocardiopata primaria, que cursa de forma asintomtica o con trastornos del ritmo cardaco.
La mayora de estos enfermos presentan un cierto dficit intelectual, y los pacientes suelen fallecer alrededor de los 20 aos por complicaciones
respiratorias o cardacas. Muchas enfermedades neuromusculares son genticas, lo que significa que hay una tendencia familiar o existe una mutacin
en los genes. Algunas veces, pueden ser provocadas por un trastorno del sistema inmunolgico. La mayora de ellas no tiene cura. El objetivo del
tratamiento es mejorar los sntomas, aumentar la movilidad y el lapso de vida.
CASO CLINICO (MG)
Masculino de 23 aos de edad el cual acude a consulta debido a que
desde hace un mes presenta debilidad generalizada, dificultad para
caminar y alzar objetos, estas manifestaciones predominan en la
maana, durante el interrogatorio refiere dormir bien, no consumo de
drogas alcohol, no cuenta con enfermedades neurodegenerativas en
la familia, actualmente se encuentra terminando la licenciatura pero le
preocupa estos sntomas, a la exploracin fsica usted observa ptosis
bilateral, y disminucin de fuerza generalizada.
PREGUNTA
Cul es el diagnostico ms probable?
RESPUESTA
a.- Esclerosis multiple.
b.- Esclerosis lateral amniotrofica.
c.- Sindrome de Guillain-Barre.
d.- Miastenia gravis.
CASO CLINICO
Una mujer de 30 aos de edad acude con su medico de atencin
primaria debido a visin borrosa y fatiga. Poer la maana no presenta
sintomasa, pero aparecen y se hacen progresivamente mas intensos
hacia el final del dia. El examen fsico revela frecuencia cardiaca de
90lpm, y TA 115/75mmHg. Las evidencias en el examen oftalmolgico
son ptosis simtrica y respuestas pupilares intactas en ambos lados. Es
evidente la debilidad de los musculos de las dos manos, pero solo
despus de contracciones multiples. El examen sesitivo resulta por
completo normal y los reflejos tendinosos profundos estn intactos.
PREGUNTA

CURSO ENARM CMN SIGLO XXI TEL: 36246001

Cul es el diagnostico mas probable en este caso?


RESPUESTA
a.- Esclerosis lateral amiotrofica
b.- Fibromialgia
c.- Esclerosis multiple
d.- Miastenia gravis
PREGUNTA
Qu enfermedad es menos probale que este relacionada con esta
patologa?
RESPUESTA
a.- Hipertiroidismo
b.- Artritis reumatoide
c.- Insuficiencia cardiaca
d.- Lupus eritematoso sistemico
PREGUNTA
Cul es la conducta mas adecuada a seguir en este caso?
RESPUESTA
a.- Potenciales evocados
b.- Prueba de edrofonio
c.- RNM
d.- Electromiografia
PREGUNTA
Cul es la conducta teraputica mas adecuada a seguir en este caso?
RESPESTA
a.- Inhibidores de la acetilcolinesterasa
b.- Timectoma
c.- Esteroides
d.- Hormona adrenocorticotropa

Pharmed Solutions Institute

PGINA 136

MANUAL DE TRABAJO DEL CURSO ENARM CMN SIGLO XXI


DEPRESION:
CIENCIAS BASICAS: Trastorno del estado de nimo con repercusin en distintos mbitos de la persona. Existe una alteracin a nivel emocional, con un
sentimiento intenso de tristeza, desesperanza, abandono, inutilidad o culpa. En otras ocasiones, nimo irritable. A nivel del contenido del pensamiento,
predomina el pesimismo, con abundantes preocupaciones, ideas negativas repetitivas, angustia y falta de inters con desesperanza en relacin al futuro.
Desde el punto de vista somtico se producen alteraciones del sueo y del apetito, con prdida de peso, astenia y alteraciones digestivas que son
expresin de la disfuncin vegetativa. A nivel conductual se produce una disminucin del rendimiento, con afectacin de la memoria, la atencin y la
capacidad de concentracin, apata, disminucin de la libido y tendencia al aislamiento social. Tanto en el trastorno depresivo como en el sndrome
confusional agudo (SCA) puede existir una afectacin de las funciones cognitivas con disminucin de la atencin y del ciclo vigilia-sueo. Cabe destacar la
depresin con rasgos psicticos, donde el paciente pierde el contacto con la realidad, combinando sntomas de depresin y psicosis, con aparicin de
alucinaciones o delirios. Habitualmente, el contenido de stos es coherente con la depresin, con una temtica relacionada con ideas de culpa, induccin
al suicidio, etc. Existen varios tipos de episodios depresivos en funcin de su intensidad y curso: el trastorno depresivo mayor, que generalmente aparece
de forma episdica, y el trastorno distmico, menos intenso y con curso crnico. El episodio depresivo mayor tiende a la recuperacin completa, y puede
darse de forma aislada en la vida de un sujeto. Sin embargo en general es una enfermedad con tendencia a la cronicidad y resulta ms incapacitante que
otras enfermedades mdicas como la artritis o la diabete. Etiopatogenia, podra ser el modelo biopsicosocial, que resultara de la integracin de tres
perspectivas diferentes, la biolgica, la psicolgica y la social. Los sistemas de neurotransmisin serotoninrgica, noradrenrgica, dopaminergica y
peptidrgica estn implicados en la depresin. La serotonina produce la inhibicin o la activacin de la neurotransmisin en el sistema nervioso central.
Desde las neuronas de los ncleos dorsales y caudales del rafe se extienden numerosas proyecciones serotoninrgicas hacia reas cerebrales asociadas a
sntomas depresivos. Se ha observado disminucin de los niveles de serotonina en el LCR de pacientes con depresin. El sistema noradrenrgico est
tambin implicado en la depresin. El sistema dopaminrgico implica reas cerebrales de las que depende el comportamiento y funciones fisiolgicas
alteradas en la depresin. La dopamina puede intervenir en la depresin sobretodo en la mana, ya que algunos agonistas dopaminrgicos se han
asociado a la aparicin de mana y los antagonistas son eficaces en su tratamiento. CRITERIOS DSM-IV-TR DEL EPISODIO DEPRESIVO MAYOR: Cinco o ms
de los siguientes sntomas (entre los que debe cumplirse los dos primeros) que representan un cambio respecto a la actividad previa, durante un periodo
mnimo de dos semanas. Uno de los sntomas debe ser el estado de nimo depresivo, o la prdida de inters o de la capacidad para el placer. Estado de
nimo depresivo. Disminucin del inters o la capacidad para el placer. Aumento o prdida importante de peso. Insomnio o hipersomnia casi cada da.
Agitacin o enlentecimiento psicomotor casi cada da. Fatiga o prdida de energa casi cada da. Sentimientos de inutilidad o de culpa excesivos o
inapropiados. Disminucin de la capacidad de pensar, concentrarse o indecisin, casi cada da. Pensamientos recurrentes de muerte, ideacin suicida
recurrente. El tratamiento en todo tipo de depresiones el uso de frmacos antidepresivos puede resultar beneficioso, no obstante hay que valorar el
riesgo-beneficio especialmente en pacientes con otras patologas y en pacientes ancianos. Una excepcin es la depresin bipolar, por lo que es
importante hacer una buena exploracin sobre los antecedentes personales de episodios manacos e hipomanacos, y de la historia familiar del paciente.
En los pacientes bipolares, antes de prescribir un antidepresivo es necesario administrar un frmaco aprobado como estabilizador del estado de nimo.
Tratamiento de la depresin: Inhibidores selectivos de la recaptacin de serotonina (ISRS), los inhibidores selectivos de la recaptacin de noradrenalina
(ISRSN), los moduladores selectivos duales de los receptores de la noradrenalina y serotonina (NaSSA), inhibidores de la recaptacin noradrenrgica
(IRN), inhibidores duales de la recaptacin de la noradrenalina y dopamina (IRNaDa), Inhibidores de la monoaminooxidasa (IMAO) que comprenden
varios frmacos. Los nuevos antidepresivos en su conjunto han dado lugar a mejoras en la farmacoterapia de la depresin por su facilidad de uso, menos
efectos secundarios y mayor seguridad en sobredosis.
CASO CLINICO
Mujer de 69 aos, trada a la consulta por su hija, por presentar
sensaciones de abandono persistente, temor a quedarse sola en su
casa, cambios del humor , insomnio intermedio y final (logra conciliar
el sueo, pero se despierta en la madrugada, a veces ya no logra volver
a dormir), la invaden pensamientos catastrficos, miedos sobre todo
cuando su hija sale del domicilio a trabajar, piensa que algo malo le
puede suceder y no soporta quedarse sola, llora constantemente,
presenta labilidad emocional, su comportamiento ha regresionado de
ser una persona muy independiente, a ser alguien que exige
proteccin constantemente,
llegando
inclusive a
agotar
emocionalmente a su familiar, la sintomatologa se evidencia despus
de recuperarse de una crisis hipertensiva, la hija refiere que desde
entonces su madre ya no fue la de antes. Antecedentes: Siempre fue
una persona independiente, su madre falleci cuando era una
adolescente y su padre la abandono con sus tres hermanos, era la
segunda de cuatro hermanos, se vio obligada a trabajar desde muy
joven y resolva los problemas cotidianos sin dificultad llegando
incluso a la sobreproteccin con sus familiares, su experiencia de
pareja al parecer fue complicada, brinda pocos datos al respecto,
decidi vivir sola con su hija la cual tuvo estando soltera, Hasta su
adultez sufri de cuadros migraosos con frecuencia
PREGUNTA
Cual es el diagnostico mas probable del caso presentado?
RESPUESTA
a.- Trastorno distimico.
b.- Episodio depresivo.
c.- Trastorno depresivo.
d.- Trastorno de ansiedad.
CASO CLINICO

CURSO ENARM CMN SIGLO XXI TEL: 36246001

Mujer de 58 aos. Lleva ms de 20 aos en tratamiento psiquitrico y


ha sido valorada por ms de un psiquiatra durante su evolucin. Desde
los 20 aos de edad, ella se reconoce triste. Excepto por breves
temporadas, cada una de las cuales raramente ha durado ms de un
mes, en las que se nota prcticamente bien. La intensidad de la
sintomatologa nunca ha sido severa. En general, con ciertos reparos,
reconoce que siempre ha seguido desarrollando las tares domsticas.
Desde hace aos es su hija quien se responsabiliza de la casa, y ella
procura ayudarle. Se queja de tristeza, desnimo al afrontar el da
desde que se levanta. Pero sobre todo una pobre confianza en s
misma. Dificultad para concentrarse en cualquier tarea que con
dificultad emprende.
PREGUNTA
Cual es el diagnostico mas probable para el caso?
RESPUESTA
a.- Trastorno bipolar.
b.- Depresin mayor.
c.- Trastorno Distmico.
d.- Depresin Psictica.
CASO CLINICO
Mujer de 43 a. acude al Servicio de Urgencias. Se siente cansada y sin
fuerzas para vivir. Responde con lentitud a las preguntas, en un tono
de voz bajo. Dice encontrarse muy triste desde hace dos meses, lo dice
con precisin. Ha perdido inters por la vida, se nota distinta, como si
no le importara nada, ni siquiera lo que hagan su marido o sus hijas,
aunque no quiere verlos sufrir. Tiene muy poco apetito, ha perdido
casi 5 kg en un mes. Tarda mucho en quedarse dormida y se despierta
varias veces por la noche. Se siente intil y un estorbo en casa. Piensa
en la muerte, aunque no quiere quitarse la vida, porque ello va contra
sus creencias religiosas. Nunca antes le haba ocurrido algo parecido.

Pharmed Solutions Institute

PGINA 137

MANUAL DE TRABAJO DEL CURSO ENARM CMN SIGLO XXI


b.- Indica trombolisis, oxigenoterapia y analgesia.
c.- Indica anxiolitico, trombolisis, oxigenoterapia y analgesia.
d.- Indica ansioltico y oxigenoterapia.

PREGUNTA
Cual es el diagnostico mas probable para el caso?
RESPUESTA
a.- Trastorno bipolar.
b.- Depresin mayor.
c.- Trastorno Distmico.
d.- Depresin Psictica.
CASO CLINICO DEPRESION
Femenino de 35 aos de edad la cual acude a consulta por un resfriado
comn, a la exploracin se observa con la mirada fija, callada,
disminucin de la atencin y memoria a corto plazo disminuida, a la
exploracin fsica se observa evasiva, hipoactiva, descuidada en su
aspecto, hace 3 aos se divorcio y actualmente solo tiene trabajos
temporales y ya ha perdido dos por faltas injustificadas, refiere que ha
comenzado a ingerir bebidas alcohlicas hasta la embriaguez.
PREGUNTA
Usted considera que la paciente presenta un episodio depresivo y
considera iniciar tratamiento anterior a su derivacin a psiquiatra.
Cul seria el ms adecuado?
RESPUESTA
a.- Imipramina.
b.- Fluoxetina.
c.- Duloxetina.
d.- Reboxetina
CASO CLINICO
Ingresa paciente masculino de 32 aos de edad al servicio de
urgencias, se observa agitado, diafortico, ansioso con sensacin de
ahogo, refiere dificultad para respirar con dolor torcico, y
epigastralgia, refiere que presenta temor a morir, refiere que su padre
muri hace 4 aos por infarto al miocardio as como su hermano
mayor el cual tena 36 aos de edad, a la exploracin fsica se observa
obesidad, TA 120/85, FC 102, FR 23, el paciente es agente de ventas y
se encuentra bajo presin continua, decide enviar estudios de
laboratorio y gabinete, observando alteraciones en el ECG sin tener los
resultados de laboratorio.
PREGUNTA
Tomando en cuenta el cuadro clnico y la evidencia de laboratorio y
gabinete. Cul es el abordaje inicial?
RESPUESTA
a.- Indica oxigenoterapia y analgesia.

CURSO ENARM CMN SIGLO XXI TEL: 36246001

CASO CLINICO
Varon de 81 aos de edad es llevado por su hija con el geriatra, ella
esta preocupada acerca de su comportamiento reciente. Ella dice que
su padre ha llorado con frecuencia y que no come bien. El reconoce
haberse sentido abatido a ltimas fechas y ha dicho a sus hijos que no
quiere ser una carga para ellos, y que estran mejor sin l. Antes de las
ltimas semanas an era activo y jugaba pker con regularidad con
otros compaeros en la institucin para jubilados. Comenta que no ha
estado de humor para eso durante las dos semanas pasadas y se
encuentra durmiendo todo el dia. Niega experimentar sntomas
maniacos o alucinaciones auditivas o visuales, Los resultados de
examen fsico son en escencia normales, aunque ha perdido 4.5Kg de
peso, desde la ultima visita y muestra lentitud psicomotora. Los
medicamentos que toma incluyen un IECA, una tiazida.
PREGUNTA
Cul es el diagnostico mas probable para este caso?
RESPUESTA
a.- Episodio depresivo
b.- Depresion mayor
c.- Trastorno bipolar
d.- Trastorno distimico
PREGUNTA
En cual de las siguientes situaciones del paciente habra que tener un
estudio mas profundo?
RESPUESTA
a.- No he estado de humor para eso
b.- Se encuentra durmiendo todo el da
c.- Estaran mejor sin m
d.- Llora con frecuencia y no come bien
PREGUNTA
Cul es la conducta teraputica mas adecuada para este paciente?
RESPUESTA
a.- Inhibidores de la recaptura de serotonina
b.- Antidepresivos tricclicos
c.- Inhibidores de la monoaminooxidasa
d.- Terapia electroconvulsiva

Pharmed Solutions Institute

PGINA 138

MANUAL DE TRABAJO DEL CURSO ENARM CMN SIGLO XXI


ANSIEDAD:
CIENCIAS BASICAS: Estado afectivo fisiolgico que consiste en tener una sensacin difusa de aprensin. Es una sensacin desagradable y vaga, de
aparicin aguda y transitoria, que se acompaa de una activacin del sistema nervioso autnomo. Este estado acta como mecanismo de alerta y
adaptacin ante una situacin amenazante para la integridad fsica o psquica del individuo, o percibida como tal. La ansiedad patolgica constituye un
estado de ansiedad que se desencadena sin un estmulo amenazante, a diferencia del miedo, que constituye una situacin de ansiedad provocada por un
estmulo realmente amenazante. SALUD PUBLICA: Al menos 20% de los pacientes vistos en atencin primaria sufren de 1 o mas trastornos mentales
como condiciones primarias o comorbilidades. Se falla en diagnosticar y tratar entre 55-75% de las condiciones psiquitricas comunes tales como
ansiedad y depresin. 1 de cada 20 personas puede tener trastorno de ansiedad. Relacion 2:1 mujer, hombre, inica en la juventud o adolscencia y se
manifiesta entre 40-45 aos. CLASIFICACION: Trastornos de ansiedad segn la DSM-IV TR: 1. Ataques de pnico (crisis de ansiedad, crisis de angustia,
panic attac ). 2. Agorafobia Trastorno de angustia sin agorafobia (F41.0) Trastorno de angustia con agorafobia (F40.01) Agorafobia sin historia de
trastor
DIAGNOSTICO DIFERENCIAL DE TRASTORNOS DE ANSIEDAD
TIEMPO
no de
DESENCADENANTE
MANIFESTACIONES
OBSERVACIONES
DURA
PERIODICIDAD DE
angust
CION
SINTOMAS
ia
Fobia
No
Episodios espordicos,
Estimulo fbico (presencia o
Miedo a un objeto o situacin determinada. Temor o
Evitacin de la situacin que
especifica
min/hrs
anticipacin)
sntomas ansiosos
provoca el temor
(F40.0
Fobia social
No
Episodios
Situaciones sociales o escrutinio social (presencia o
Miedo a colocarse en una situacin vergonzosa
Evitacin de la situacin
0). 3.
espordicos,
anticipacin). Temor
angustioso a
en un medio social. Temor o sntomas ansiosos.
que provoca el temor
Fobia
min/hrs
determinadas circunstancias, reales o imaginarios
Preocupacin de que el desempeo pueda ser
Preocupacin
por
que
el
desempeo
sea
casa
de
causa
de
humillacin
o
burla
espec
humillacin.
fica
Trastorno
1 mes
Episodios
Aparicin brusca de un medio intenso.
Temblor, sudoracin, miedo a morir, sensacin de
1 de 3: ansiedad anticipatoria.
(F40.0
de pnico
espordicos 5-20
Ninguno aparente, pero el sujeto lo
frio/calor, nauseas, sensacin de asfixia, marea,
Preocupacin constante,
min
puede
atribuir
a
mltiples
causas
miedo
a
perder
el
control,
dolor
de
pecho
Ataques
Repercusin de los AP. Cambio
2)4.
de pnico (AT)sbitos inesperados y recurrentes
conductual significativo
Fobia
Ansiedad
6
La mayor parte del
Aparicin progresiva y permanente
Fatiga, inquietud, dificultad para dormir,
El individuo est preocupado
social
generalizad
meses
da casi todos los
de sntomas de ansiedad, sin una
irritabilidad, tensin muscular,
permanentemente sin motivos aparentes y no
a (TAG)
das
causa real que los provoque
preocupaciones, nerviosisimos excesivos
lo puede controlar. 3 de 6: intranquilidad,
(F40.1
cuya intensidad han debilitado el control
fatigabilidad, concentracin, irritabilidad,
). 5.
sobre ellos
tensin muscular, alteracin en el dormir
Trasto
Obsesivo
No
Min7hrs
Obsesiones que pueden ser das o
Cada obsesin lleva asociada una compulsin o
Un ejemplo es el miedo al
compulsivo
pensamientos que se repiten, y no
conducta que compensa la angustia que provoca
contagio, cuya composicin
rno
(TOC)
desaparecen de le mente del individuo,
correspondiente es lavarse las
obsesi
aunque lo intente por todos los medios
manos rpidamente
voEstrs
Aparece en aquellos individuos que se
Perdida del sueo, irritabilidad, sobresalto, falta de
Provoca alteraciones en la
postraumti
han
visto
expuesto
a
un
evento
concentracin.
Re
exoperimentracion
de
la
situacin
vida familiar, laboral y social
compu
cos
traumtico, que involucra un dao fsico
traumtica: recuerdos intrusivos recurrentes, sueos
del individuo
lsivo
(EPT)
o psicolgico extremo
estresantes recurrentes, flashbacks, temor ansiedad o
(F42.8
reacciones neorovegetativas intensas al exponenrsea
situaciones u objetosrelacionados con el evento.
). 6.
Trastorno por estrs postraumtico (F43.1) 7. Trastorno por estrs agudo (F43.0). 8. Trastorno de ansiedad generalizada (F41.1) 9. Trastorno de
ansiedad debido a enfermedad mdica (F06.4) 10.Trastorno de ansiedad inducido por sustancias Adems de estos trastornos, agrupados bajo el rtulo
trastornos de ansiedad, en la DSM-IV TR se incluye finalmente un trastorno de ansiedad infantil, el trastorno de ansiedad por separacin. PATOGENIA:
Existen varias teoras, teora del locus cueruleos (aumento de descarga de nucleos noradrenergicos del SNC), teora metabolica (cambios metablicos
aberrantes, inducidos por la infusin de lactato), teora de la falsa alarma por asfixia (hipersensibilidad de los receptores de diasoxido de carbono del
tronco cerebral), teora benzodiacepinica del GABA (funcin anormal de los receptores, que produce una disminucin de la actividad inhibidora, la
inhibicin ligera del sistema gabaergico, genera ansiedad, insomnio, hiperreactividad a estimulos no amenazadores y aumenta el nivel de alerta).
Podemos encontrar GABA que es el freno, puede estar disminuido, GLUTAMATO que es el acelerador, tiene una actividad aumentada, la noradrealina
nos puede dar agitacin pscomotriz (sirve para la atencin). Sujetos que toman benzodiacepinas de manera crnica, pierden el temor a la muerte y ms
fcilmente se suicidan. La ansiedad genera edema mitocondrial, lo que lleva a aumento de glucosa y de la frecuencia cardiaca. DIAGNOSTICO: Angustia
normal= reactiva, breve, motivadora, no sufrimiento, no desajuste. Angustia patolgica= autnoma, prolongada, paralizante, sufrimiento, desajuste. La
manifestacin de la ansiedad se basa en tres componentes clnicos fundamentales: un componente somtico (sntomas vegetativos), uno cognitivo
(sensacin de nerviosismo, aceleracin, percepciones de malestar, etc) y uno motor (temblor, etc). La gran mayora de los pacientes llega por insomnio o
dolor torcico, no puede respirar, manos tiesas, cefalea, no por ansiedad. A mayor numero de sntomas, mayor es la ansiedad. El estimulo de la ansiedad
es tan potente que puede enera una arritmia letal y llevar a la muerte. Tambien eleva adrenalina y las personas pueden responder agresivamente.
TRASTORNO DE ANSIEDAD DE TIPO ADAPTATIVO: Malestar subjetivo de ansiedad, interfiere en el funcionamiento social, secundario a crisis vital
sucedida en el mes anterior, duracin menos a 6 meses (Por ejemplo en un trauma raquimedular, va a tener necesidades que no va a poder satisfacer
por si mismo, la capacidad de adapatacion). TRASTORNO DE PANICO: Crisis de ansiedad recurrente y espontanea, puede haber palpitaciones, mareos,
disnea, despersonalizacin, desrealizacion, hay temor a morir, perder el control o enloquecer, asi como ansiedad anticipatoria (me va a dar, me va a dar),
ocurre varias veces al dia, se relaciona mucho con consumidores de coca y alcohol. TRASTORNO OBSESIVO-COMPULSIVO (TOC): Obsesiones tpicas;
contaminacin, orden y sexuales. Se trata de pensamiento, impulsos o imgenes recurrentes y persistentes que se experimentan en algn momento del
trastorno como intrusos e inapropiados y causan ansiedad y malestar significativo. Los pensamientos impulsos o imgenes no se reducen a simples
preocupaciones excesivas sobre problemas de la vida real. Los obsesivos saben que son obsesivos, el sujeto sabe que es irracional su comportamiento, la
persona intenta ignorar o suprimir estos pensamientos, que son producto de su mente. Compulsiones; comportamiento o actos mentales de carcter
repetitivo que el individuo se ve obligado a realizar en respuesta a una obscesion o con arreglo a ciertas reglas que debe seguir estrictamente. El objetivo
de estos comportamientos u operaciones mentales es la prevencin o reduccin del malestar o la prevencin de algn acontecimiento o situacin
negativa. Add. La anorexia tiene un componente TOC, el anorxico no solo se preocupa se imagina las caloras. TRASTORNO DE ANSIEDAD
GENERALIZADA (TAG): La ms frecuente y predominante. Ansiedad y preocupaciones excesivas, sobre una amplia gama de acontecimientos o
actividades que se prolongan por ms de 6 meses, le resulta difcil controlar este estado constante de preocupacin. Hay ansiedad y preocupacin
asociada a 3 o mas de los siguientes sntomas: inquietud o impaciencia, fatigabilidad fcil, dificultad para tener la mente en blanco o concentrase,
irritabilidad, tensin muscular, alteraciones del sueo. Areas afectadas; incremento del metabolismo occipital, temporal y frontal, cerebelo y talamo,
incremento de actividad metabolica de ganglios basales por hipervigilancia, el locus coeruleos esta relacionado con la ansiedad tal vez mediante el
sistema autonmico como taquicardia, dilatacin pupilar, temblor, diaforesis. Por dficit serotoninergico, puede haber alteraciones en el sexo, apetito y
agresin y las alteraciones en la noradrenalina causan alteracin en concentracin, inters, memoria. TRATAMIENTO: En el trastorno del pnico se
podrn utilizar ansiolticos, y para prevenir la crisis se administrarn antidepresivos. Este tratamiento tendr una duracin de 6-12 meses. El trastorno

CURSO ENARM CMN SIGLO XXI TEL: 36246001

Pharmed Solutions Institute

PGINA 139

MANUAL DE TRABAJO DEL CURSO ENARM CMN SIGLO XXI


obsesivo-compulsivo deber ser tratado por el psiquiatra, fluvoxamina es de primera eleccion. El tratamiento consiste en la combinacin de un frmaco
junto a terapia psicolgica de modificacin de conducta. Las fobias; hay que distinguir el tipo, si es especfica, se puede abordar en atencin primaria con
un tratamiento semejante al de la ansiedad; pero si es grave, como la agorafobia, fobia social generalizada o algn tipo de fobia simple, ser tratada por
el especialista. Es importante recordar que si nosotros bloqueamos e incrementamos el efecto GABA con benzodiacepinas, vamos a incremetar los
trastornos de memoria, por ello no debemos hacer uso de ellas indiscriminadamente, le iria peor al paciente con ansiedad (si se usa no mas de 8
semanas). Lo mas recomendable es yugular la ansiedad lo mas breve posible, iniciando con benzodiacepinas paralelo al antidepresivo (alprazolam,
diazepam- en agudo, crisis, manejo breve. Loracepam, clonazepam de 0.5-3, oxacepam, mantenimiento crnico). Los inhibidores duales (de recaptura de
serotonina y noradrenalina), son de primera eleccin, para el tratamiento de trastorno de ansiedad generalizada como venlafaxina y fluoxetina, aunque
cursan con un periodo de ansiedad al principio (periodo de adaptacin). Inhibidores de la recaptura de serotonina (ISRS) tratamiento de 1-2 aos
ansiedad; paroxetina, pnico venlafaxina.
CASO CLINICO
La paciente, nacida en 1981. Vena padeciendo ataques de angustia
episdicos recidivantes desde el ao 2003, sobre todo cuando se
encontraba sola. Tampoco era capaz de recorrer trayectos largos en
tren ni en metro. Las crisis comenzaron tras el fallecimiento de su
abuelo, con el cual haba tenido supuestamente una relacin especial,
ya que siempre haba sufrido los problemas de relacin de sus padres.
Ms o menos al mismo tiempo se rompi el antebrazo por una cada,
que requiri una complicada operacin. Esto acrecent an ms sus
crisis de angustia. Por lo dems, refiri que gozaba de buena salud, no
fumaba ni beba alcohol, aunque descuidaba las comidas por motivos
de tiempo. Dorma bien, necesitaba de 7 a 8 horas de sueo diarias, en
caso contrario no se encontraba en forma.

PREGUNTA
Cual es el diagnostico mas probable en el caso descrito?
RESPUESTA
a.- Trastorno de personalidad.
b.- Trastorno mental secundario a lesin neurolgica.
c.- Trastorno de la conducta secundaria a lesin neurolgica.
d.- Trastorno de control de impulsos.

PREGUNTA
Cual es el diagnostico mas probable en el caso descrito?
RESPUESTA
a.- Trastorno de pnico
b.- Trastorno de estrs
c.- Trastorno de ansiedad
d.- Trastorno obsesivo compulsivo
CASO CLINICO
Femenino, de 52 aos, ama de casa y madre de cinco hijos ya adultos.
La relacin con su marido ha perdido mucho con los aos pero se
resiste a iniciar la separacin. Se ha preocupado en exceso por muchos
motivos: sus hijos, su madre, sus nietos... Algunas frases tpicas suyas
son: Ha llegado Joaqun?; Estn bien los cros?; No salgan hasta muy
tarde, que nunca se sabe qu puede pasar en la noche. El menor de
sus hijos tiene ya 20 aos y se ha acostumbrado a las continuas
advertencias y sugerencias para prevenir males posibles. Tambin se
ha acostumbrado a llamarla por telfono en mitad de la noche cuando
sale con los amigos para informarle de que no pasa nada, que todo va
bien. Reconoce que le resulta difcil dejar de preocuparse tanto por
todos y por todo. Le resulta muy difcil concentrarse en otra cosa que
no sean los peligros que acechan a los suyos, confundiendo con
frecuencia el hecho de que un peligro sea posible, con el hecho de que
sea probable. Tiene dificultades para dormir y mucha tensin muscular
acumulada. Se resista a reconocer que lo suyo era un problema de
ansiedad porque los peligros son reales.
PREGUNTA
Cual es el diagnostico mas probable en el caso descrito?
RESPUESTA
a.- Trastorno de pnico
b.- Trastorno de estrs
c.- Trastorno de ansiedad
d.- Trastorno obsesivo compulsivo
CASO CLINICO
Varn de 33 aos, que acude por episodios de agitacin psicomotriz en
domicilio, heteroagresividad hacia familiares. Ingresa mostrndose
hostil, irritable, suspicaz y autorreferencial. Episodio de agitacin
psicomotriz en Urgencias y heteroagresividad dirigida a familia y
mobiliario. Es necesaria contencin farmacolgica y mecnica hasta
remisin de la clnica actual (Olanzapina i.m) y se cursa ingreso

CURSO ENARM CMN SIGLO XXI TEL: 36246001

involuntario (nula conciencia de enfermedad, riesgo alto para el


paciente y terceros, necesidad de completar estudio) Conductas de
riesgo desde la adolescencia (carreras de coches, consumo de txicos
THC y cocana). Personalidad premrbida impulsiva segn su familia y
con escasa tolerancia a las normas. Soltero. Padre de una nia, con la
que mantiene escasa relacin. En 2002, a la edad de 23 aos, en
contexto de intoxicacin enlica, sufre accidente de trfico con
importantes secuelas orgnicas, politraumatizado (incluido
traumatimo craneoencefalico grave) requiriendo hospitalizacin
durante 70 das. Permanece en coma segn historia clnica durante 15
das hasta recuperacin total del nivel de conciencia. Al da de hoy no
secuelas fsicas.

CASO CLINICO
Persona de 30 aos, que en el curso de menos de 6 meses, acude a la
consulta y observamos que el ndice de Quetelet es menos (peso en
Kg/altura al cuadrado) que 17,5 (normal de 19 a 24). Ha perdido 10
kilos o ms en ese tiempo, presenta amenorrea de tres o ms ciclos
consecutivos, y un miedo intenso a ganar peso.
PREGUNTA
Cual es el tratamiento mas adecuado para el caso descrito?
RESPUESTA
a.- Fluvoxamina.
b.- Fluoxetina.
c.- Sertralina.
d.- Clorimipramina.
CASO CLINICO
Paciente mujer de 42 aos acude, acompaada, por presentar episodio
de agitacin psicomotriz y trastorno de conducta con
heteroagresividad fsica y verbal en entorno familiar. Se procede a su
ingreso para aclaracin diagnstica y contencin de la situacin, con
orientacin diagnstica de sndrome maniforme. Psiquitricos: A los 21
aos debuta con un episodio depresivo, cuya sintomatologa cedi en
pocos meses y fue tratado por Mdico de Atencin Primaria. Tras un
segundo episodio depresivo, presenta a los 24 aos un primer cuadro
maniforme que requiri ingreso hospitalario y cuya orientacin
diagnstica al alta fue de trastorno bipolar, episodio actual maniaco.
PREGUNTA
Cual es el tratamiento mas adecuado para el caso descrito?
RESPUESTA
a.- Litio
b.- Olanzapina.
c.- Lamotrigina.
d.- Valproato.

Pharmed Solutions Institute

PGINA 140

MANUAL DE TRABAJO DEL CURSO ENARM CMN SIGLO XXI


ESQUIZOFRENIA Y TRASTORNOS RELACIONADOS:
CIENCIAS BASICAS: Trastorno mental severo que afecta al pensamiento, las emociones y el comportamiento. Es la forma ms frecuente de trastorno
mental severo y afecta a una persona de cada 100. Es poco frecuente antes de la pubertad y ms comn de 15 a 35 aos. Con frecuencia dura toda la
vida. Un nio que tiene un padre con esquizofrenia tiene una probabilidad entre 10 de desarrollar esquizofrenia. Ganglios basales; integracin de
informacion sensorial, prefrontal y dorsolateral; funciones especificas, sist. Lmbico; alteracin de emociones, hipocampo; memoria procesos. Zona
critica para las funciones ejecutivas (Yo-dorsolateral); si esta alterado hay inflexibidad, problemas de planeacin, para la solvencia, campo de direccin
de conducta. (Super yo-orbitofrontal); su alteracin produce deshinibicion, hipersexualidad, labilidad emocional, agresividad, reduce el autocontrol y la
vigilancia. (Id- vental); su alteracin genera mutismo, acinetico, abulia, no hay conducta espontanea, alteracin de la iniciacin de la conducta. SALUD
PUBLICA: 1-2% universalmente acepatada, 0.7-1.4% a lo largo de la vida, Relacion 1:1 hombre: mujer. Hay 1, 100,000 personas afectadas en Mexico.
Inicio ewntre los 18-21 aos mas comnmente, es uno de los trastornos mas incapacitantes. CLASIFICACION: Formas clnicas: Esquizofrenia paranoide.
Esquizofrenia catatnica. Esquizofrenia hebefrnica/desorganizada. Esquizofrenia residual. Existen dos formas de grupos: positivos y negativos,
normalmente tienen una mezcla de los dos. Sntomas positivos: Delirios, trastorno del pensamiento, alucinaciones, lo ms comn es oir voces. Sntomas
negativos: inactivas, solitarias y parecer desmotivadas. Parecen perder inters y motivacin y pueden dejar de lavarse regularmente o cuidarse de una
forma apropiada. Normalmente no pueden concentrase en un trabajo o en el estudio. PATOGENIA: Hay Muchas hiptesis, pero ninguna es suficiente
para expliucar todos los sntomas, en esquizofrenia estn alteradas estas 4 proyecciones; Tubero- infundibular, disminucin en meso-cortical, aumento
en meso-limbico (genera alucinaiones, distorsion del pensamiento por aumento de dopamina), y nigro-estriado DIAGNOSTICO: En promedio hay un
periodo de 2 aos aprox., en el que ya se pueden identificar sntomas del inicio de la enfermedad, antes de establecerse el diagnostico. Prdromos;
aislamiento, despersonalizacin, alteraciones de la conducta, cambios en el autocuidado, baja en rendimiento laboral y acadmico, alteracin del
lenguaje, sntomas depresivos, aislamiento, mayor retraimiento social, sensibildad aumentada a los sonidos, comportamiento excentricco, expresin
inadecuada de los sentimientos, consumo de sustancias, discurso difcil de seguir, preocupacin por temas extravagantes, empobrecimiento de discurso,
ideas de referencia y/o premoniciones, sentimientos persitentes de desrealizacion, alteraciones de la percepcin, falta de inters sexuial, anhedonia,
ideas suicidas, bloqueo del pensamiento, baja de peso apetito disminuido, cambios en la motilidad, ideas obsesivas, impulsividad. Historia clnica;
presentan rasgos de personalidad extrao-excentrico, ansioso-temeroso. Si se presentan estos 3 sindromes hay ezquizofrenia: Sindrome de pobreza
psicomotora (corteza prefrontal, dorsolateral, estriado); pobreza de lenguaje, disminucin de movimientos espontaneos, expresividad facial fija, pobre,
falta de respuesta afectiva, falta de inflexiones vocales, contacto visual pobre. Sindrome de desorganizacin (cortex prefrontal, ventromedial, cngulo,
lbulo parietal y talamo); afecto inadecuado, contenido de lenguaje pobre, tangencialidad, divagacin, presin verbal, distractibilidad, incoherencia
verbal, alteracin del curso del pensamiento. Sindrome de distorsion de la realizad (lbulo temporal); alucinaciones auditivas, delirios de persecucin, de
referencia, alucinaciones visuales, desrealizacion, alteracin del contenido del pensamiento, pensamiento sonoro, eco, robo, transmisin, insercin,
comando. TRASTORNO EZQUIZOTIPICO: Comportamiento excntrico, anomalas del pensamiento y de afectividad, no hay sntomas predominantes o
caracteristicos, puede haber afectividad fra o vacia, anhedonia, extraos, excntrico, peculiares, retraimiento social, ideas de referencia, extravagantes y
preocupaciones autsticas, ideas paranoides o suspicacia (de ser Dios, tener poderes), rumiaciones obsesivas (habla en voz baja solo o acompaado), a
menudo sobre contenidos dismorficos sexuales o agresivos. Pueden tener episodios casi psicoticos, ocasionales y transitorios, con alucinaciones visuales
y auditivas intensas eideas pseudo-delirantes, sin provocacin externa (comunicarse con el mas alla, hablar con muertos, predecir que algo va a pasar,
predecir el futuro). TRASTORNO DELIRANTES PERSISTENTES: Ideas delirantes, relacionadas entre si, normalmente son muy peristentes, pueden durar
hasta el final de la vida del individuo. El contenido del tema o conjunto de ideas delirantes es muy variable, a menudo es de persecucin, hipocondriaco o
de grandeza, litigio o de celos, poner de manifiesto la conviccin de que una parte del propio cuerpo esta deformada, piensan que despiden mal olor, o
que se es homosexual. Lo mas caracterstico es que no se presenta otra psicopatologa, pero pueden aparecer de modo intermitente sntomas
depresivos y, en algunos casos, alucinaciones olfatorias y tctiles. Las voces alucinatorias, los sntomas ezquizofrenicos tales como las ideas delirantes de
ser controlado, el embotamiento afectivo y la presencia de una enfermedad cerebral son incompatibles con este diagnostico. TRASTORNOS PSICOTICOS
AGUDOS y transitorios; 1. Comienzo agudo (<2sem), se define como un cambio desde un estado sin caractersticas psicticas a otro claramente anormal
y psictico en un periodo de 2 semnas o menos, el comienzo agudo es signo de buen pronostico (cuanto mas sbito sea el inicio, mejor ser el
desenlaze), 2. Presencia de sndromes tpicos que son; el estado rpidamente cambiante y variable polimorfo, la presencia de sntomas esquizofrnicos
tpicos, 3.- presencia de estrs agudo. Ejemplo; Dios habla a travs de el, esta posedo, le salen estigmas. TRASTORNO DE IDEAS DELIRANTES
INDUCIDAS; Trastorno de ideas delirantes poco frecuentes, compartido por dos a mas personas que comparten estrechos lazos emocionales. Solo uno
de los afectados padece un autentico trastorno psictico. En el otro o los otros las ideas delirantes son inducidas y normalemnte remiten cuando se les
separa. El trastorno psictico del individuo dominante suele ser un esquizofrenia, pero no es algo, ni necesario ni constante. Tanto las ideas delirantes
originales de la persona dominate como las inducidas en la otra, son crnicas de naturaleza persecutoria o de grandeza. Son transmitidas de esta manera
en circunstancias extraordinarias poco frecuentes. Casi siempre las dos personas son familares cercanos, aislados de entorno por su lengua, cultura o por
factores geogrficos. Las personas en las cuales las ideas delirantes son inducidas suelen ser tambin dependientes o tienen una relacin de servidumbre
con la que padece la psicosis genuina. Un ejemplo alienacin parental (madre; poner en contra a uno de los hijos hacia el padre). TRASTORNOS
EZQUIZOAFECTIVOS: Los sntomas afectivos como los esquizofrnicos (aplanamiento afectivo) son destacados y se presentan durante el mismo episodio
de la enfermedad, preferiblemente de forma simultane o al menos con pocos das de diferencia entre unos y otros, los sntomas afectivos aparecen
superpuestos o forman parte de una enfermedad ezquizofrenica preexistente, los cuales coexisten o alternan con otros tipos de trastornos de ideas
delirantes persistentes, se clasifican bajo la categora adecuada. El episodio de enfermedad no satisface las pautas, ni de esquizofrenia, ni de episodio
depresivo o maniaco. TRATAMIENTO: Antipsicticos convencionales (tpicos): haloperidol (2da eleccin, pero primera eleccin en evento agudo,
bloqueo de sist. limbico), levomepromacina (distorcion de la realidad), perfenacina (distorsion de la realidad), pimocide, zuclopentixol, sulpiride,
tioridacina. Antipsicticos de nueva generacin (atpicos): clozapina, risperidona (1ra eleccin, a larago plazo, en la parte cronica), olanzapina (2da.
Eleccin), quetiapina, ziprasidona, amisulpiride y aripiprazol. Tratamiento esquizofrenia resistente: clozapina (vigilar agranulocitosis). Los antipiscticos
tratan parte de los sntomas de la enfermedad, mejoran el funcionamiento y calidad de vida y evitan recaidas. D2 incisivos tiles en desorganizacin,
duales (risperidona) en pobreza motora, para yugular una crisis psictica haloperidol y levomepromacina. Antes de cambiar de antipsictico esperar al
menos 3-4 semanas a dosis teraputicas: Evaluar el riesgo de efectos secundarios a corto y largo plazo. Especial importancia tiene la dicinesia tarda y el
sndrome neurolptico maligno. Evaluar la respuesta subjetiva al tratamiento. Reforzar siempre necesidad de tratamiento y evaluar conciencia de
enfermedad. Para la sintomatologa extrapiramidal (SEP) se deben dar anticolinrgicos como el biperideno (Akineton), en caso de distona aguda dar va
intramuscular y el cuadro revierte a los pocos minutos. Para la acatisia (sensacin interna de inquietud, dificultad para estar sentado, mover las piernas,
etc) se puede utilizar propanolol. En algunos casos tambin responde a anticolinrgicos (biperideno) o dosis bajas de benzodiacepinas. La acatisia es
especialmente preocupante porque se asocia a abandono del tratamiento, agresividad y suicidio. TEC especialmente en pacientes con catatona, gran
riesgo de suicidio o importante componente afectivo. Terapia conductual: Tiene en cuenta las deficiencias que tienen los pacientes. Habilidades sociales
en las que se ensea a los paientes a relacionarse, buscar trabajo, hacer una entrevista, utilizar medios de transporte, etc. Terapia orientada a la familia

CURSO ENARM CMN SIGLO XXI TEL: 36246001

Pharmed Solutions Institute

PGINA 141

MANUAL DE TRABAJO DEL CURSO ENARM CMN SIGLO XXI


(sistmica): Informacin, identificacin de factores precipitantes de recada, reducir emocin expresada. Importancia de asociaciones de familiares.
Terapia grupal: Centrada en planes de la vida diaria, intercambio de experiencias, conciencia de enfermedad, resolucin de problemas. Psicoterapia
individual: Su eficacia se aada a la del tratamiento farmacolgico. Importancia de la relacin teraputica.

CASO CLINICO
Femenino de 55 aos, viuda reciente, inica hace 5 aos despus de la
muerte de su esposo por cncer pulmonar. Luego de colocar una
antena de televisin, refiere preocuparse de que se este
contaminando por rayos de la antena, cubri la ventana para evitar lo
rayos, pero posteriormente inicia con voces, que no le disgustaban
porque eran amables, posteriormente comienzan a ser agresivas y
adems la antena le enviaba rayos asus genitales, para excitarla, lo que
le molestaba.
PREGUNTA
Cul es la impresin diagnostica ms probable para este caso?
RESPUESTA
a.- Esquizofrenia hebefrenica
b.- Esquizofrenia simple
c.- Esquizofrenia paranoide
d.- Esquizofrenia desorganizada

PREGUNTA
Cul es el diagnostico mas probable?
RESPUESTA
a.- Esquizofrenia hebefrenica
b.- Esquizofrenia catatnica
c.- Esquizofrenia paranoide
d.- Esquizofrenia simple
CASO CLINICO
Femenino de 26 aos de edad, desempleada, acudi voluntariamente
por dolor de cabeza, desde hace unos se ha vuelto agresiva y ha
cambiado su personalidad, destrozaba cosas en su casa, golpeaba
vidrios, insultaba a la madre con lenguaje obsceno, hablaba sola y
actuaba como si hablara con otra persona, la paciente refiere que la
familia quiere hacerle dao, le envenenaban la comida, deca que era
tutankamon, a la exploracin se observa sucia, disgregada, bloqueos
con sntomas de sneider.

PREGUNTA
Cul es el tratamiento inmediato mas adecuado?
RESPUESTA
a.- Haloperidol
b.- Perfenacina
c.- Levomepromacina
d.- Ziprasidona

PREGUNTA
Cul es el diagnostico mas probable?
RESPUESTA
a.- Esquizofrenia hebefrenica
b.- Esquizofrenia catatnica
c.- Esquizofrenia paranoide
d.- Esquizofrenia indiferenciada

PREGUNTA
Cul es el tratamiento a largo plazo mas adecuado?
RESPUESTA
a.- Risperidona
b.- Olanzapina
c.- Quetiapina
d.- Zisaprida
CASO CLINICO
Femenino de 16 aos de edad. Estudiante, dejo de ir a la escuela
sbitamente, se comportaba ansiosa y agitada peleando consus
padres. Se siente perseguida y aseguraba que unos ratones le coman
el estomago y que haba serpientes cerca de la cama, la paciente fue
ingresada y tratada con haloperidol mejorando significativamente
luego de 3 meses de internamiento. Sin embargo 9 meses depues dejo
el tratamiento por mejora.
PREGUNTA
Cul es su impresin diagnostica ms probable?
RESPUESTA
a.- Esquizofrenia desorganizada
b.- Esquisofrenia simple
c.- Esquizofrenia hebefrenica
d.- Esquizofrenia paranoide
PREGUNTA
Alos 21 aos reingresa por presencia de ansiedad, insomnio,
alucinaciones visuales y comportamiento agresivo sbito hacia su
madre. Incoherente, se prendio fuego despus de rociar la cama con
parafina Cul es la conducta psiquitrica a seguir?
RESPUESTA
a.- Haloperidol y levomepromacina
b.- Haloperidol y clonacepam
c.- Haloperidol y diacepam
d.- Haloperidol, levomepromacina y diacepam

CURSO ENARM CMN SIGLO XXI TEL: 36246001

CASO CLINICO
Femenino de 22 aos ingresa a urgencias, trada por la polica, debido
a que estando, en la parada de autobs, trato de estrangular a una
nia de 9 aos, algunos transentes ayudaron a sujetarla y la polica
intervino, en ese momento la paciente no se movia, no hablaba, se
encontraba rigida con la mirada salvaje.

CASO CLINICO
Masculino de 26 aos, soltero, nini, vive con sus padres, es llevado por
sus padres debido a que no se asea, no sale de la casa, rechazo social,
tiene antecedentes de haber consumido, canabis, LSD, herona y
anfetaminas, a los 18 aos dejo el trabajo, se hizo introvertido, perdi
el inters por las cosas, sin sentido del deber, lo corrieron del servicio
militar, se negaba a levantarse de la cama y comia muy poco, hablaba
poco, con rumiaciones suicidas
PREGUNTA
Cul es el diagnostico mas probable?
RESPUESTA
a.- Esquizofrenia simple
b.- Esquizofrenia indiferenciada
c.- Esquizofrenia paranoide
d.- Esquizofrenia hebefrenica
CASO CLINICO
Masculino de 30 aos, soltero, vive con sus padres, es enviado al
especialista por dificultad para dormir nervioso y ansioso, refiere que
hace 2 meses estaba en un baile y estaba parado viendo a quien
invitaba a bailar, pero no se animaba, se gano un premio y se
aterrorizo al momento para recibir el premio, solo bailo con una chica
y ya no quiso hacerlo mas y este ultimo se molesto. A la exploracin se
observa suspicaz y a disgusto, su lenguaje era vago, complejo,
disgregado y circunstancial, referia creer en lo extrasensorial, la
telepata y encontraba mensaes en las cosas.
PREGUNTA
Cul es el diagnostico mas probable para este caso?

Pharmed Solutions Institute

PGINA 142

MANUAL DE TRABAJO DEL CURSO ENARM CMN SIGLO XXI


RESPUESTA
a.- Trastorno de ideas delirantes
b.- Esquizofrenia paranoide
c.- Trastorno ezquizotipico
d.- Trastorno esquisofreniforme
CASO CLINICO
Masculino de 43 aos de edad, casado, 2 hijos desde hace 9 meses
presenta depresin y dolores de cabeza, refiere que estos surgieron
por los problemas en su matrimonio, refiere que su esposa tiene una
aventura amorosa con otro hombre, referia que era con el dueo de
una librera, nunca lo haba visto, la seora lo negaba, lo ha
encontrado buscando pruebas entre sus cosas, asi como la ropa
interior para buscar resto de una relacin.
PREGUNTA
Cul es el diagnostico mas probable en este caso?
RESPUESTA
a.- Trastorno de ideas delirantes
b.- Esquizofrenia paranoide
c.- Trastorno esquizotipico
d.- Trastorno esquizofreniforme
CASO CLINICO ESQUIZOFRENIA
Se trata de paciente de 24 aos de edad el cual ingresa a urgencias por
referir dolor en la regin cervical lo cual le dificulta a la movilizacin de
la cabeza, adems refiere el acompaante que ha incrementado su
aislamiento social, como antecedentes de importancia el paciente se
encuentra diagnosticado con esquizofrenia y actualmente recibe
tratamiento con Haloperidol 5 mg cada 8 horas, clonacepam 2 mg cada
24 horas y levomepromazina 25 mg en la noche, agrega que hace 15
dias sali de internamiento psiquitrico por brote psictico con una
estancia de 3 semanas, no hay una red de apoyo adecuada. A la EF
observa rigidez en miembros superiores, reflejos osteotendinoso
incrementado, signo de rueda dentada positivo, adems de sialorrea
leve.
PREGUNTA
Cual es la conducta a seguir en el paciente?
RESPUESTA
a.- Reducir levomepromazina.
b.- Reducir haloperidol.
c.- Administrar biperiden.
d.- Administrar trihexifenidilo
CASO CLINICO
Masculino de 24 aos, se encuentra en urgencias por referir
inestabilidad emocional, taquicardia y opresin en el pecho, temor a
ser secuestrado debido a sus conocimientos especiales, los familiares
agregan que desde los 18 aos dejo la escuela, comenz a
comportarse extrao se extravi 3 semanas, regresando sucio,
mencionando que fue a buscar a Dios para hablar con l, a la EF: se
observa falta de contacto visual, mutista selectivo, disperso, con
rumiaciones, sus constantes vitales se encuentran normales.

CURSO ENARM CMN SIGLO XXI TEL: 36246001

PREGUNTA
Cul es la conducta teraputica de segunda eleccin ms adecuada?
RESPUESTA
a.- Haloperidol 10 mgs y levopromacina 25 mg IM
b.- Risperidona 2mg cada 12 hrs
c.- Olanzapina 5mgs cada 12 hrs
d.- Haloperidol 5mgs cada 12 hrs
PREGUNTA
Se yugulan sntomas motivo de ingreso se realiza cambio en la
teraputica para mayor apego, as para disminuir la conducta autstica.
Cual ser el frmaco a elegir?
RESPUESTA
a.-Haloperidol 10mgs en 24 hrs VO
b.- Levopromacina 25 en 24 hrs VO
c.- Quetiapina 200mg cada 12 hrs VO
d.- Risperidona 3 mg cada 12 hrs VO
PREGUNTA
Cul es el diagnstico ms probable?
RESPUESTA
a.- Esquizofrenia paranoide
b.- Esquizofrenia hebefrenica
c.- Esquizofrenia desorganizada
d.- Esquizofrenia simple
PREGUNTA
El paciente regresa en 6 meses con un nuevo cuadro psictico, se
ingresa a urgencias, el familiar refiere que el paciente no quiere tomar
el medicamento por los efectos adversos, de los siguientes manejos
coadyuvantes. Cual es el ms adecuado para contrarrestar los efectos
indeseables?
RESPUESTA:
a.- Difenhidramina 25 mgs
b.- Trihexifenidilo 5mg cada 8 hrs
c.- Clonacepam 2 mg cada 24 h
d.- Biperiden 25 mgs cada 24 h
PREGUNTA
Considerando la naturaleza del frmaco agregado. Cul de las
siguientes manifestaciones no son propias de este grupo
farmacolgico?
RESPUESTA
a.- Pupilas dilatadas, piel y mucosa oral secas
b.- Elevacin de la temperatura corporal
c.- Bradicardia y bloqueos
d.- Distension abdominal y retencin urinaria
PREGUNTA
Cul de las siguientes no es una contraindicacin del biperideno?
RESPUESTA
a.- Hipertrofia prosttica benigna
b.- Temblor esencial idioptico
c.- Sindrome de QT largo
d.- Enfermedad de Alzheimer

Pharmed Solutions Institute

PGINA 143

MANUAL DE TRABAJO DEL CURSO ENARM CMN SIGLO XXI


TRASTORNOS DE LA ALIMENTACION:
ANOREXIA NERVIOSA: Se caracteriza por la negativa para mantener el peso corporal normal, lo que resulta en un peso corporal <8 % del peso esperado
para la edad y altura. Signos y sntomas: sensacin de fro, la piel, el cabello, las uas quebradizos, alopecia , lanugo, acrocianosis , edema, bradicardia ,
hipotensin, agrandamiento de las glndulas salivales, el vaciamiento gstrico lento, estreimiento, enzimas hepticas elevadas, anemia normocitica
normocromica, leucopenia, aumento de nitrgeno de urea en sangre , aumento de la creatinina, hiponatremia, hipopotasemia, baja la hormona
luteinizante y la hormona folculo-estimulante con amenorrea secundaria, hipoglucemia, la hormona estimulante de la tiroides normal con baja tiroxina,
aumenta el cortisol plasmtico. Caractersticas de diagnstico: Rechazo a mantener el peso corporal igual o por encima del valor mnimo normal. (Esto
incluye un fracaso para lograr la ganancia de peso esperada durante un periodo de crecimiento que conduce a un peso corporal anormalmente bajo).
Miedo intenso a la ganancia de peso o la grasa. Distorsin de la imagen corporal (por ejemplo, sensacin de grasa a pesar de un peso objetivamente bajo
o minimizar la gravedad de bajo peso). La amenorrea (Este criterio se cumple si los perodos menstruales se producen slo despus de hormonas, por
ejemplo, la administracin de estrgenos). Tratamiento: Recuperacin del peso a 90% de peso predicho es el objetivo principal. La intensidad del
tratamiento inicial, incluyendo la necesidad de hospitalizacin, se determina por el peso actual, la rapidez de la prdida de peso reciente, y la gravedad
de las complicaciones mdicas y psicolgicas. Desequilibrios electrolticos graves deben ser identificadas y corregidas. Recuperacin nutricional casi
siempre se puede lograr con xito por la va oral. Las caloras se pueden aumentar gradualmente hasta lograr un aumento de peso de 1-2 kg/sem (30004000 kcal /d). Las comidas deben ser supervisadas. La ingesta de vitamina D (400 UI/d) y calcio (1.500 mg/d) debe ser suficiente para reducir al mnimo la
prdida de hueso. La ayuda de psiquiatras o psiclogos suele ser necesario. No hay medicamentos psicotrpicos. Las complicaciones mdicas en
ocasiones se producen durante la realimentacin, la mayora de los pacientes transitoriamente retienen el exceso de lquido, a veces resultando en
edema perifrico. La insuficiencia cardaca congestiva y la dilatacin gstrica aguda se han descrito cuando la realimentacin es rpida. Elevaciones
modestas transitorias en los niveles sricos de enzimas hepticas ocurren ocasionalmente. Los niveles bajos de magnesio y fosfato deben ser
reemplazados. La mortalidad es del 5% por dcada, ya sea de hambre crnica o el suicidio. BULIMIA NERVIOSA: Se caracteriza por episodios recurrentes
de atracones seguidos de conductas compensatorias anormales, tales como vmitos autoinducidos, abuso de laxantes o ejercicio excesivo. Peso es en el
rango normal o por encima. Signos y sntomas: agrandamiento de las glndulas salivales, la erosin dental, hipopotasemia, hipocloremia, alcalosis
(vmitos) o acidosis (de abuso de laxantes). Caractersticas de diagnstico: Los episodios recurrentes de atracones de comida, que se caracterizan por el
consumo de una gran cantidad de alimentos en un corto perodo de tiempo y la sensacin de que la alimentacin est fuera de control. Comportamiento
inapropiado recurrente para compensar la ingesta compulsiva, como el vmito autoinducido. La aparicin de tanto el trastorno por atracn y el
comportamiento compensatorio inapropiado al menos dos veces por semana, en promedio, durante 3 meses. Preocupacin excesiva por la forma y el
peso corporal. Nota: Si se cumplen simultneamente los criterios diagnsticos para la anorexia nerviosa, slo se da el diagnstico de anorexia nerviosa.
Puede ser tratada de forma ambulatoria. La terapia cognitiva conductual y la fluoxetina (Prozac) son tratamientos de primera lnea. La dosis de
tratamiento recomendada para la fluoxetina (60 mg/d) es ms alta que la normalmente utilizada para tratar la depresin. Tanto la anorexia nerviosa y la
bulimia nerviosa presentan principalmente en mujeres jvenes previamente sanos que se convierten en demasiado preocupados con la forma y el peso
corporal. Atracones y las purgas comportamiento que puede estar presente en ambas condiciones, con la distincin fundamental entre los dos que
descansa sobre el peso de la persona. Las caractersticas diagnsticas.
CASO CLINICO ANOREXIA NERVIOSA
Femenino de 21 aos de edad la cual acude a urgencias secundaria a
perdida del estado de alerta, refiere compaantes que se encontraba
en clases cuando cayo de su propia altura sin causa aparente, a la
exploracin fsica se observa mucosas orales deshidratadas, palides
generalizada, piel reseca, cabello quebradizo y de cada fcil, se
administra glucosa intravenosa y mejora su estado, observa un estadio
tanner no correspondiente a la edad y al interrogatorio refiere pirosis,
disfagia y vomitos ocacionales con trazas de sangre.
PREGUNTA
Cual es diagnostico ms probable de la paciente?
RESPUESTA
a.- Anorexia nerviosa.
b.- Hipoglucemia.
c.- Desequilibrio hidroelectrolitico.
d.- Gastritis crnica.
CASO CLINICO
Una joven de 19 aos de edad acude con su medico de atencin
primaria para un examen de rutina. Niega molestias actuales. Mide
160cm de estatura y pesa 52Kg. El examen fsico revela temperatuira
de 37C, pulso de 82/min y presin arterial de 118/70mmHg. Tambien
muestra cierta erosion del esmalte dental, agrandamiento bilateral
leve de las partidas y tejido cicatrizal en la segunda y tercera
articulaciones metacarpofalangicas derechas. Los hallazgos de
laboratorio importantes son: Na 135mEq/l, K 2.8 mEq/l, Cl 33mEq/l,
amilasa srica 130U/L
PREGUNTA
Cul es el diagnostico mas probable en este caso?

CURSO ENARM CMN SIGLO XXI TEL: 36246001

RESPUESTA
a.- Anorexia nerviosa
b.- Desequilibrio 144idroelectroltico
c.- Bulimia nerviosa
d.- Deficiencia de vitaminas
PREGUNTA
Cul es la prevalencia de esta patologa en la primera y segunda
patologia?
RESPUESTA
a.- 5-10%
b.- 10-20%
c.- 2-5%
d.- 20-30%
PREGUNTA
Cul es menos probable que este relacionado con esta patologa?
RESPUESTA
a.- Trastornos de la personalidad
b.- Ansiedad
c.- Abuso de sustancias
d.- Depresion
PREGUNTA
Cul es la complicacin menos probable que presente esta paciente?
RESPUESTA
a.- Erosin y caries dental
b.- Osteoporosis
c.- Dilatacion y rotura gstrica
d.- Esofagitis crnica

Pharmed Solutions Institute

PGINA 144

MANUAL DE TRABAJO DEL CURSO ENARM CMN SIGLO XXI

URGENCIAS
1)
2)
3)
4)
5)
6)
7)
8)
9)
10)
11)
12)
13)
14)
15)
16)
17)
18)

MUERTE SUBITA, RCP


ESTADO CHOQUE
URGENCIAS Y EMERGENCIA HIPERTENSIVA.
TAPONAMIENTO CARDIACO, DISECCION DE LA AORTA, ANEURISMA AORTICO.
ANGINA ESTABLE, ANGINA INESTABLE, ANGINA DE PRINZTMETAL.
SICA I, SICA II
TRASTORNOS DEL RITMO (ARRITMIAS CARDIACAS)
ANAFILAXIA Y ALERGIAS
TROMBOSIS VENOSA PROFUNDA, TROMBOEMBOLIA PULMONAR.
SINDROME DE INSUFICIENCIA RESPIRATORIA AGUDA, EDEMA AGUDO PULMONAR.
CRISIS ASMATICA Y EPOC AGUDIZADO.
INSUFICIENCIA RENAL AGUDA, GLOMERULOPATIAS AGUDAS.
TRASTORNOS ACIDO-BASE AGUDOS.
TRASTORNOS ELECTROLITICOS AGUDOS.
COMPLICACIONES AGUDAS DE DIABETES MELLITUS
ISQUEMIA CEREBRAL TRANSITORIA, ACIDENTE VASCULAR CEREBRAL.
CRISIS CONVULSIVAS Y ESTATUS EPILEPTICO.
TRASTORNO BIPOLAR, DELIRIUM, PSICOSIS, SUICIDIO.

CURSO ENARM CMN SIGLO XXI TEL: 36246001

Pharmed Solutions Institute

PGINA 145

MANUAL DE TRABAJO DEL CURSO ENARM CMN SIGLO XXI


MUERTE SBITA (MS):
CIENCIAS BSICAS: Muerte (ausencia de signos vitales) inesperada sin sntomas precedentes la mayora de las veces o que en caso de existir estos,
ocurren pocos segundos antes de que la muerte sobrevenga y sin causa traumtica aparente o que la explique. Son 4 las arritmias letales, las de ritmos
descargables; fibrilacin ventricular (FV), taquicardia ventricular sin pulso (TVSP), y las de ritmos no descargables; asistolia y actividad elctrica sin pulso
(AESP). SALUD PBLICA: En EU de Norteamrica hay hasta 300mil muertes por ao. En Mxico entre 33-53 millones anuales, en general ligadas a
enfermedad isqumica cardiaca. La cardiopata coronaria aumenta 4-6 veces el riesgo de MS. En 60-70% de los casos de MS, por evento cardiovascular;
1. Cardiopata coronaria (isqumica) con o sin antecedentes conocidos, es responsable del 70-80% (causa numero uno mundial). 2. Cardiopata
estructural (isqumica, miocardiopatia hipertrfica, dilatada, cardiopatias valvulares, congnita o adquirida, miocarditis aguda, tumores cradiacos,
sarcoidosis, amiloidosis) 15-20% 3. Arritmias 5% asociadas a fenmenos elctricos primarios o del sistema excitoconductor, en corazones sanos (sx. de
QT largo, sx. de Brugada, sx. Wolf-Pakinson-White, taquicardia helicoidal). Algunos de estos pacientes concentran una historia familiar positiva, que
puede ser el nico elemento que permita identificarlos como grupo de riesgo. La FV causa >90% de MS.
PATOGENIA: Existe un descontrol total del corazn, debido a la presencia del fenmeno R sobre T (R/T), el
cual se presenta cuando llega un estimulo en la parte final de la repolarizacin. Es de importancia el periodo
refractario relativo (PRR) en el, si llega un estimulo que rebase el umbral, el msculo se excita en su zona
lbil, se desencadena una arritmia cardiaca y puede sobrevenir la muerte. (PRA=aunque el estimulo sea muy
fuerte las clulas del miocardio no se excitan). Los que tienen ms riesgo de sufrir un fenmeno de R/T son:
post IAM, porque se quedan cicatrices y estas son focos ectpicos que desencadenan estmulos, en los
hipertensos con miocardiopata hipertrfica. Solo 2 arritmias descontrolan el corazn porque producen el
fenmeno de R/T (en ECG, vemos una extrasstole ventricular) y lo pueden llevar a la muerte; la taquicardia
ventricular (ondas regulares >180 lpm, no hay ondas P, complejos anchos) y la fibrilacin ventricular
(desorden y ritmo catico). Ninguna de las 2 produce gasto cardiaco, el primer signo es el sincope, el cerebro
se desconecta (no llega oxigeno, ni glucosa), tambin se puede presentar una convulsin como manifestacin
de anoxia. La persistencia de la falta de riego lleva a la destruccin irreversible y al fallecimiento del
individuo. As tenemos isquemiaTVFVMS. Muerte sbita es= Condicin subyacente (cardiopata
coronaria/estructural) + Susceptibilidad individual (inestabilidad elctrica, no todos los pacientes con SCA
hacen FV solo el 15%) + Evento gatillante (isquemia, trastorno HE o acido-base, frmaco, trastorno elctrico primario). DIAGNOSTICO: La MS cardaca no
tiene unos sntomas previos como tal, pero una serie de posibles factores pueden influir en su aparicin: 1. Haber tenido un episodio de estas
caractersticas anteriormente o que tenga familiares con antecedentes 2. Insuficiencia cardiaca. 3. Haber sufrido un IAM. 4. Arritmias cardacas.
TRATAMIENTO: Efectivo para frenar la MS cardaca, es la desfibrilacin precoz. Consiste en realizar una descarga elctrica en el corazn, a travs de unas
palas o parches, con la que se pretende reiniciar la actividad elctrica del corazn. En el caso de no disponer de un desfibrilador, o de no saber utilizarlo,
se puede realizar una RCP. Tambin existe la posibilidad de implantar un desfibrilador cuando se detecta, por ejemplo, un sndrome de Brugada
(elevacin del ST en V1 a V3 y morfologa similar a la de un bloqueo de rama derecha, episodios sincopales, que pueden llevar a FV y MS), o cuando el
porcentaje de sangre bombeada por el corazn en cada latido, denominada 'fraccin de eyeccin', sea <40%. PREVENCION PRIMARIA: Identificar grupos
de riesgo; GRUPO DE MAYOR RIESGO: A). Cardiopata coronaria sintomtica: con IAM en evolucin o reciente. Angina inestable. B). Cardiopata
estructural asociada a cardiopata coronaria, ICC FEVI <35%. C). Con arritmias ventriculares, definen riesgo de MS : TVNS inducible o espontnea, TVS
inducible o espontnea. En este grupo siempre debe corregirse isquemia, ya sea mdicamente o con procedimientos de revascularizacin. GRUPO DE
RIESGO INTERMEDIO: A).Cardiopata estructural adquirida (HTA, valvular, coronaria) o congnita. En general son pacientes cardipatas sintomticos o
asintomticos, pero sin elementos coronarios activos ni arritmias ventriculares de alto riesgo, se benefician de terapia farmacolgica que detiene o
aminora progresin de cardiopata demostrado por evidencia (IECA, espironolactona, bloqueadores AT2. B). GRUPO BAJO RIESGO: Poblacin general con
factores de riesgo cardiovasculares que determinan finalmente aparicin de cardiopata coronaria, estructural o ambas. Fundamental manejar factores
de riesgo: HTA, DM, sedentarismo, tabaquismo, dislipidemias. Intervenciones teraputicas tanto farmacolgicas como no farmacolgicas aplicadas a esta
poblacin son de alto impacto. TRATAMIENTO: Si el paciente tiene una TV o FV, desfibrilar, para que el corazn retome su ritmo, se puede dar RCP pero
con CAB no con ABC (primero compresiones, para generar gasto cardiaco, llenar coronarias y perfundir corazn y cerebro). Solo los grupos de mayor
riesgo se benefician en trminos de prevencin primaria de DI (desfibrilador implantable), pacientes con FEVI <30%, tambin es la terapia preferida en
una gran proporcin de pacientes sobrevivientes a MS. El resto de los pacientes debieran recibir terapia farmacolgica que se ha demostrado que
disminuye el riesgo de muerte atribuible a MS, como amiodarona (prolonga el PRA, reduce posibilidad de fenmeno R/T) o betabloqueadores, estos
ltimos adems han demostrado que reducen mortalidad global. Esta estrategia es independiente del manejo adecuado del paciente con cardiopata
demostrada con frmacos que mejoran sobrevida y progresin de enfermedad como IECA, antagonistas de Angiotensina 2, espironolactona, etc. Se debe
evaluar al paciente sobreviviente de MS: 1. Verificando si tiene factores de riesgo o factores gatillante para corregirlo inmediatamente post-RCP. 2.
Buscando cradiopatias subyacentes y 3. Busqueda de arritmias de alto riesgo, para ello hay que realizar un Holter de 24-48hrs que es el de eleccin,
podemos encontrar como hallazgos complejos ventriculares prematuros, TVNS (TV no sostenida), TVPS (polimorfa sostenida) o TVMS, TPSV o trastornos
de la conduccin.
b.- Compresiones toracicas
CASO CLINICO
c.- Administracion de amiodarona.
Se trata de paciente masculino de 61 aos de edad el cual ingresa a
d.- Masaje carotideo.
urgencia por perdida del estado de despierto de forma sbita,
produciendoce cada desde su propia altura. Se iniciaron maniobras de
PREGUNTA
resucitacin con recuperacin luego de dos descargas con 360 j,
Cual de los siguientes factores de riesgo es el ms grave para
presentando nuevamente perdida del estado de despierto. Tiene como
predisponer al paciente de presentar el estado actual?
antecedentes diabetes mellitus 2 desde hace 15 aos, tratado con
RESPUESTA
insulina glargina, actualmente en protocolo de dialisis peritoneal con
a.- Insuficiencia renal crnica.
KDOQI 5, y un infarto hace 6 meses.
b.- Infarto al miocardio antiguo.
c.- Diabetes mellitus de larga evolucin.
PREGUNTA
d.- Hipomagnesemia crnica.
Considerando la presentacin del cuadro clnico y el ECG. Cual es la
conducta a seguir?
CASO CLINICO
RESPUESTA
Se trata de paciente masculino de 71 aos de edad el cual ingresa a
a.- Descarga de 360 j.
urgencias por perdida del estado de despierto, al ingreso se observa

CURSO ENARM CMN SIGLO XXI TEL: 36246001

Pharmed Solutions Institute

PGINA 146

MANUAL DE TRABAJO DEL CURSO ENARM CMN SIGLO XXI


con palidez generalizada, hipotrmico, sin respuesta a estimulos y sin
pulso palpable, se activa el cdigo de alerta y se inicia con
compresiones toracicas por parte de medico de guardia, se realiza dos
ciclos de compresiones con leve respuesta y se observa trazo fibrilar en
ECG.

RESPUESTA
a.- Neumotorax a tensin.
b.- Sobredosis por drogas
c.- Tromboembolismo pulmonar.
d.- Hipovolemia.

PREGUNTA
Cual es la conducta a seguir inmediata mas adecuada considerando el
trazo?
RESPUESTA
a.- Desfibrilar a 360 j.
b.- Reiniciar compresiones.
c.- Administracin de amiodarona.
d.- Intubacin orotraqueal inmediata.

CASO CLINICO
Varn de 23 aos en parada cardiorrespiratoria no recuperada tras
protocolo de reanimacin cardiopulmonar avanzada, por lo que se
activa cdigo de donante en asistolia, mientras se mantiene el masaje
cardaco con ventilacin mecnica y se conecta al paciente a
circulacin extracorprea. En la radiografa de trax que se realiza en
el proceso de validacin pulmonar se evidencia masa mediastnica
radioopaca de localizacin inferior y media. Se aprecian signos de
broncoaspirado, por lo que se desestima la extraccin pulmonar.

PREGUNTA
Considerando la etiologa de la FV. Cual es la mas probable en este
paciente?
RESPUESTA
a.- Cardiopatia coronaria.
b.- Infarto agudo al miocardio.
c.- Hipertrofia ventricular.
d.- Cardiomiopatia dilatada.
CASO CLINICO
Se trata de paciente masculino de 21 aos de edad, es ingresado a
urgencias debido a que perdi el estado de alerta mientras jugaba
futbol, no referio sntoma alguno solo se cayo en la cancha, a la
exploracin se observa ligera palidez de tegumentos, llenado capilar
de 2 segundos, pulso lento y dbil, FC 90, FR 22, TA 110/70 mmHg,
responde a ordenes y refiere no recordar lo que paso. Como
antecedente de importancia refiere que ocacionalmente se ha sentido
mareado desde los 12 aos, pero nunca se haba desmayado, de nio
le mencionaron que tena un soplo que se quitara posteriormente,
cabe mencionar que su padre muri a los 31 aos.
PREGUNTA
Considerando el cuadro clnico. Cual es la etiologa ms probable?
RESPUESTA
a.- Miocardiopatia hipertrfica.
b.- Sindrome de QT largo.
c.- Displasia arritmogenica.
d.- Cardiopatia coronaria.
PREGUNTA
Considerando la condicin subyacente. Cuales son las medidas mas
adecuada para evitar otro evento del mismo orden potencialmente
mortal?
RESPUESTA
a.- Evitar eventos gatillantes.
b.- Considera susceptibilidad individual.
c.- Estudiar antecedentes personales.
d.- Modificacin de estilo de vida.
CASO CLINICO
Ingresa a urgencias maculino de 21 aos de edad el cual se observa
ansioso, agitado con actitud alucinada, a la exploracin se encuentra
diafortico con palidez generalizada, sbitamente el paciente pierde el
estado de alerta y presenta parada cardiaca, se activa el cdigo de
alerta e inicia compresiones torcicas, los familiares solo refieren que
lo golpearon en una fiesta, 10 minutos despus presento los sntomas.
No hay respuesta, no se detecta pulso, ni constantes vitales. Se
descarga con 360 j, se mantiene soporte por 20 minutos ms.
PREGUNTA
No se presive pulso. Cual de las siguientes causas es la ms probable
en este caso?

CURSO ENARM CMN SIGLO XXI TEL: 36246001

PREGUNTA
Cul es la etiologa ms probable de este caso?
RESPUESTA
a.- Aspiracion de contenido gstrico por maniobras.
b.- Neumonia.
c.- Edema agudo pulmonar.
d.- Tromboembolismo.
CASO CLINICO
Masculino de 52 aos, camionero de profesin, sin hbitos txicos
reconocidos, con sobrepeso y antecedentes de hipercolesterolemia
bien controlada con dieta y estatinas. Estando en su domicilio viendo
la televisin sufre un episodio de prdida sbita de conciencia con
cada al suelo. Ante la ausencia de pulsos perifricos se inician
maniobras de RCP bsicas hasta la llegada de los servicios mdicos de
emergencia que constatan la presencia de fibrilacin ventricular con
lnea de bajo voltaje por lo que se inician maniobras de RCP avanzada
que logran estabilizar la funcin cardiorrespiratoria por lo que se
decide traslado a centro hospitalario. A su llegada a urgencias se
realiza EKG donde se observa bloqueo de rama derecha con elevacin
del segmento ST en las derivaciones V1, V2 y V3. Los familiares
refieren antecedente familiar de muerte sbita.
PREGUNTA
A qu arritmia del sistema exitoconductor est ms probablemente
asociada la fibrilacin ventricular del paciente?
RESPUESTA
a.- Sndrome de Wolf-Parkinson White
b.- Sndrome de Brugada
c.- Sndrome de QT largo
d.- Miocardiopata hipertrfica
PREGUNTA
Cul sera la conducta teraputica ms adecuada para prevenir la
muerte sbita de este paciente?
RESPUESTA
a.- Amiodarona
b.- Lidocana
c.- Desfibrilador implantable
d.- Marcapasos
PREGUNTA
De no dar el tratamiento adecuado que probabilidades de recurrencia
de un nuevo episodio de fibrilacin ventricular tiene este paciente en
los 2 siguientes aos?
RESPUESTA
a.- 100%
b.- 50%
c.- 33%
d.- 80%

Pharmed Solutions Institute

PGINA 147

MANUAL DE TRABAJO DEL CURSO ENARM CMN SIGLO XXI


RCP BASICA Y AVANZADA:
RCP BASICA: Ante una potencial victima de MS siempre BUSCAR RESPUESTA: verificar la ausencia de respuesta (inconsciencia), ponerla en decbito
dorsal, mirando hacia arriba, tomarla de los hombros y hablando en voz alta al odo, buscar alguna respuesta (apertura ocular, emisin de sonidos o
movimiento). PEDIR AYUDA: Como la FV es el ritmo ms frecuente y tratable en la MS, si el evento fue extrahospitalario y hay otras personas presentes,
el reanimador debe identificar a una de ellas y solicitarle que active el sistema de respuesta medica de urgencias indicando que hay una vctima de MS y
la necesidad de un desfibrilador dando la direccin exacta del evento. Si el reanimador esta solo debe pedir la ayuda el mismo aun si esto implica dejar a
la victima solo por algunos segundos. Intrahospitalario, el reanimador debe activar el sistema local de respuesta mdica de urgencias (compuesto por 6
personas, timbre de alarma, cdigo azul). Si hay un desfibrilador presente selo en este momento. La RCP bsica se mantiene hasta la llegada de un
desfibrilador o hasta que reinicie ventilaciones espontaneas. Debe evitarse realizar movimientos bruscos que puedan producir o agravar lesiones
presentes, especialmente de la columna cervical. Si la vctima no responde las posibilidades son que este en coma, este en paro respiratorio o paro
cardiorrespiratorio (PCR). Para determinarlo se debe continuar con la evaluacin en forma ordenada.
: Permeabilizar la va area de la
vctima. Durante el PCR la victima pierde complemente el tono muscular, asociado al efecto de la gravedad la lengua cae hacia atrs ocluyendo la va
area, siendo esta la principal causa de obstruccin. La maniobra ms efectiva para permeabilizar la va area es extender la cabeza y elevar del mentn.
Si existe o se sospecha lesin de columna cervical solo se debe realizar la elevacin del mentn manteniendo la cabeza en posicin neutra.
: Con la va area abierta el reanimador debe mirar, escuchar y sentir (MES), buscando la respiracin, mirando hacia el trax de la misma,
en un tiempo igual o menor a 10 segundos. Si la victima tiene una respiracin adecuada el reanimador mantiene la va area abierta y mientras espera la
llegada de ayuda mdica avanzada inicia la bsqueda de la causa y reevala peridicamente a la vctima. La presencia de respiracin adecuada implica
presencia de circulacin. En ocasiones las victimas de MS presentan movimientos respiratorios muy superficiales o respiracin agnica que no deben ser
confundidos con respiracin adecuada y el reanimador debe actuar de la misma forma que si no hubiera respiracin.
: Se debe
checar ritmo cada 5 ciclos. El pulso carotideo ha mostrado ser impreciso. La FV progresa hacia asistolia rpidamente, disminuyendo la probabilidad de
xito con la desfibrilacin en un 10% con cada minuto de retraso, la
ALGORITMO RCP AVANZADA
RCP bsica disminuye el porcentaje a un 4% por cada minuto,
1.-Paro cardiorrespiratorio, iniciar RCP bsico
aumentando el periodo en el cual el ritmo es desfibrilable y por lo
tanto reversible. Durante los primeros minutos de una MS la sangre
3.- FV/TVSP
NO
9.- Asistolia/AESP
SI
(resetear corazn)
2.- Checar es un
(reperfundir coronarias)
de la victima contiene suficiente oxigeno para cumplir con la
ritmo a desfibrilar?
demanda. Adems, la demanda de oxigeno es menor ya que la
circulacin tambin esta disminuida hasta 1/3 parte de lo normal. El
4.- Dar una descarga (12010.- Reiniciar RCP por 5 ciclos.
problema principal es entonces falta de circulacin de sangre ms que
200J-bifasico o 360JAdmon.: adrenalina 1mg IV, repetir
falta de oxigenacin de la misma. Por lo tanto, si la persona esta
monofasico. Reiniciar RCP
cada 3-5 min. Considerar atropina
inconsciente y sin respiracin el reanimador debe iniciar la RCP con 30
1mg IV en asistolia o AESP lenta,
5 ciclos
repetir cada 3-5 min, hasta 3 dosis
compresiones torcicas externas seguidas de 2 ventilaciones
5.Checar
es
un
5 ciclos
artificiales. Una excepcin para el uso precoz de la desfibrilacin es la
ritmo a desfibrilar?
presencia de un PCR prolongado (>5 minutos) donde la RCP bsica,
11.- Checar es un
aumentan el xito de la desfibrilacin. Existe un pequeo porcentaje
ritmo a desfibrilar?
de pacientes que tienen una muerte sbita de etiologa hipoxica
6.- Continuar RCP mientras
carga desfibrilador. Dar
(obstruccin de va area por cuerpo extrao) en quienes si se tiene la
12.- Si asistolia, ir a cuadro 10.
una nueva descarga.
Si actividad elctrica, checar
sospecha o certeza se podra privilegiar las ventilaciones artificiales y
Reiniciar RCP. Admon.:
pulso: Si no hay ir a cuadro 10.
empezar
la
RCP
con
estas
y
continuar
con
adrenalina 1mg IV repetir
Si hay, iniciar cuidados postcompresiones.
: Se realizan colocando el
cada 3-5 min durante RCP
resucitacin.
taln de una mano en el centro del trax entre ambas tetillas con la
5 ciclos
otra mano encima y los dedos entrelazados. Con los codos
7.- Checar es un
13.- Cuidados post-resucitacin
completamente extendidos se inician las compresiones empujando
ritmo a desfibrilar?
fuerte y rpido. El mayor determinante de xito con la desfibrilacin
es la presin de perfusin coronaria. Las compresiones torcicas
8.- Continuar RCP mientras carga desfibrilador. Dar una nueva descarga. Reiniciar
efectivas logran entregar una pequea pero critica cantidad de
RCP. Considerar antiarrtmico, amiodarona: 300mg IV, considerar 150 adicional o
oxigeno al cerebro y corazn, suficiente para mantener una presin de
lidocana 1mg/kg. Considerar Magnesio 1-2g IV en torsade de pointes. Luego 5 ciclos
perfusin coronaria. Compresin torcica externa efectiva: 1.de RCP, ir a cuadro 5
recuencia 100-120 x min (con via area definitiva), 30x min (via area
no definitiva). 2. Profundidad entre 4-5 cm. 3.- Equilibrar el tiempo de compresin con el de descompresin del trax (50 y 50%). 4.- Permitir el retorno
del trax a su expansin total durante la fase de descompresin. 5.-Minimizar las interrupciones en la compresin torcica externa. Las compresiones
torcicas son seguidas de 2 ventilaciones artificiales de 1 segundo cada una.
: 1.- Boca a boca: Abriendo la va area el reanimador toma
una respiracin normal y a continuacin sella su boca alrededor de la de la victima e insufla lo suficiente como para elevar el trax de la vctima. 2.Ventilacin con barreras de proteccin: estas son dispositivos plsticos con una vlvula unidireccional que en teora evitan la transmisin de
enfermedades infectocontagiosas (no demostrado). 3.- Boca a nariz: Se utiliza cuando es imposible ventilar a travs de la boca (lesiones bucales). 4.Mascara-vlvula-bolsa: logra oxigenacin y ventilacin efectivas pero requiere de entrenamiento y prctica frecuente. En los primeros minutos de la RCP
la mscara-vlvula-bolsa es igual de efectiva y segura que un tubo endotraqueal. La intubacin endotraqueal no tiene lugar en la RCP bsica. Si hay dos o
ms reanimadores presentes deben rotarse las compresiones torcicas cada 2 minutos para garantizar que estas sean efectivas. Despus de la
desfibrilacin (200J, 200-300J, 360J si es bifsico o cargas fijas de 360J si es monofsico) las compresiones torcicas y ventilaciones artificiales se
mantienen por 2 minutos. Si la desfibrilacin fue exitosa y se logro un ritmo de perfusin se suspende la RCP bsica y se inicia la avanzada segn
necesidad. En cambio, si la desfibrilacin no fue exitosa se siguen alternando ciclos de compresin torcica con ventilacin artificial por 2 minutos y
nuevos anlisis del ritmo por el DEA y descargas si este las indica. RCP AVANZADO: A (Airway): asegurar la va area: En el pasado se prefera la
intubacin oro-traqueal (IOT), pero la incidencia de complicaciones es muy alta cuando es realizada por inexpertos. Las alternativas serian el Combitube
esfago-traqueal y la mascara larngea, pero no estn disponibles. La IOT permite mantener permeable la va area, aspirar secreciones, administrar O 2
en altas concentraciones, es una va alternativa para la administracin de medicacin y cuando se insufla el manguito, protege la va area de la

CURSO ENARM CMN SIGLO XXI TEL: 36246001

Pharmed Solutions Institute

PGINA 148

MANUAL DE TRABAJO DEL CURSO ENARM CMN SIGLO XXI


aspiracin. Se debe minimizar el tiempo de interrupcin de las compresiones (10seg). Si no fue exitosa, asegurar un intervalo adecuado de compresiones
y ventilaciones. Si el paciente tiene un ritmo con pulso, controlar con saturometria. Complicaciones de OIT: ruptura de dientes laceracin
traqueofarngea, ruptura traqueal, perforacin faringoesofagica, aspiracin de contenido gstrico, intubacin bronquial. B (Breathing): comprobacin de
la localizacin del tubo a travs de mtodos clnicos (visualizacin de expansin torcica y auscultacin) y complementarios (detectores de CO2). C
(Circulation): Obtener un acceso venoso de grueso calibre (antecubital), conectar a monitor y administrar drogas segn ritmo. Cuatro ritmos producen el
paro cardiaco: la FV, TV sin pulso, asistolia y AESP. Las drogas se administran en bolo seguidas de 20 ml de solucin fisiolgica y elevar el brazo. Algunas
pueden administrarse por el tubo endotraqueal; atropina, lidocana, adrenalina y naloxona, pero a 2 veces la dosis y seguida por 10ml de SF y 3-4
ventilaciones. Los fluidos deben ser administrados cuando se sospecha hipovolemia. Administrar bicarbonato solo si hiperkalemia persistente.
Diagnsticos diferenciales: considerar las posibles causas descompensantes o que complican la resucitacin. Valorar causas reversibles regla de las 5Hs
(hipovolemia, hipoxia, H+ (acidosis), Hiper/hipocalemia, hipotermia) y las 5Ts (txicos, taponamiento cardiaco, neumotrax a tensin, trombosis
coronaria, TEP). Farmacos en reanimacin: Adrenalina 1mg, vasopresina 20UI, atropina 1mg, amiodarona 150mg (FV/TV refractaria), lidocana al 1-2%,
dopamina 200mg/5ml, dobutamina 250mg/20ml.
CASO CLINICO
Masculino de 48 aos, diabtico e hipertenso, acude a su consulta de
medicina familiar al hospital y cuando se dirige a recabar su ficha de
consulta cae al suelo inconsciente, circunstancialmente usted pasa por
el lugar y se dispone a brindar auxilio.

espontneamente. Sin terapia posterior y asintomtico. Estando en


reposo, presenta paro cardaco que fue atendido inmediatamente por
familiares con maniobras bsicas de resucitacin cardiopulmonar,
siendo trasladado en un lapso de 5 a 10 min al Hospital local donde al
ingreso se observa actividad elctrica sin pulso.

PREGUNTA
Cul es la primera accin en la secuencia de Reanimacin
hospitalaria, en este caso?
RESPUESTA
a.- Iniciar compresiones
b.- Iniciar ventilaciones
c.- Solicitar ayuda y un monitor desfibrilador
d.- Uso de desfibrilador

PREGUNTA
Cual es su conducta a seguir inmediatamente?
RESPUESTA
a.- Iniciar compresiones
b.- Iniciar ventilacin
c.- Activar sistema de emergencia.
d.- Verificar pulso.

PREGUNTA
Despus de 5 minutos el paciente es llevado a la Sala de Urgencias.
Durante ese lapso y en el trayecto se contina con las maniobras de
RCP bsico. Al arribar a Urgencias, al paciente se le colocan las paletas
del desfibrilador y se observa una imagen ondulatoria de amplitud y
contornos variables, catico con ms de 250 lpm. No se distinguen
complejos QRS, segmento ST ni ondas T. Cul es la carga con la que se
inicia la desfibrilacin en este paciente?
RESPUESTA
a.- 150-200 J Bifasico, 360 J Monofasico
b.- 200 J Monofsico, 360 J Bifasico
c.- 360 J Bifasico
d.- 150 J Monofasico
PREGUNTA
Despus de dar una descarga, seguido de 5 ciclos (2min) de RCP y
haber colocado una va central. Qu medicamento es el ms
adecuado para este caso?
RESPUESTA
a.- Amiodarona 150mgs IV
b.- Adrenalina 1mg va IV o intraosea
c.- Amiodarona endotraqueal
d.- Adrenalina endotraqueal
PREGUNTA
Se verifica registro y persiste el trazo despus de 3 dosis de
medicamento, se da una nueva descarga, seguida de 5 ciclos de RCP.
Cul es el siguiente medicamento ms adecuado en este momento
para el paciente?
RESPUESTA
a.- Amiodarona 300mgs IV
b.- Adrenalina 1mg va IV
c.- Amiodarona endotraqueal
d.- Adrenalina endotraqueal

CASO CLINICO
Masculino de 44 aos de edad se recupera en launidad de cuidados
intensivos luego de un procedimiento toracoscpico, cuando una
enfermera descubre que ha dejado de respirar. La evaluacin revela
que no existen latido cardiaco, ni pulso palpables.
PREGUNTA
Cul es el mecanismo ms probable del paro cardiaco?
RESPUESTA
a.- Fibrilacion ventricular
b.- Taquicardia ventricular sin pulso
c.- Actividad elctrica sin pulso
d.- Asistolia
PREGUNTA
Cul es la conducta mas adecuada a seguir en este momento?
RESPUESTA
a.- Intubacion endotraqueal
b.- Desfibrilacion
c.- Compresiones del trax
d.- Activacion del sistema medico de urgencia
PREGUNTA
En el registro se observa fibrilacin ventricular, adems de la descarga
de 360J, Cul es el medicamento mas adecuado en este momento?
RESPUESTA
a.- Adrenalina IV 1mg
b.- Vasopresina IV 40 unidades
c.- Atropina IV 1 mg
d.- Dobutamina

CASO CLINICO
Paciente de 47 aos, con antecedente de un episodio de fibrilacin
auricular en que no tuvo mayor estudio y revirti a ritmo sinusal

CURSO ENARM CMN SIGLO XXI TEL: 36246001

Pharmed Solutions Institute

PGINA 149

MANUAL DE TRABAJO DEL CURSO ENARM CMN SIGLO XXI


ESTADO DE CHOQUE:
CIENCIAS BASICAS: Es la reduccin de la perfusin tisular sistmica, con decremento de la disponibilidad de O2, que lleva a lesion celular, inicialmente
reversible y si se prolonga irreversible. Estado metablico en el cual las demandas energticas celulares de oxigeno, exceden el aporte, hay disfuncin
celular, orgnica y sistmica. SALUD PUBLICA: Mortalidad mayor al 20%. Corresponde al 1% de las atenciones en el servicio de urgencias. En el shock
anafilctico cerca de 60% se muere por obstruccin de la va area superior, el resto por hipotensin y arritmias. PATOGENIA: Varios factores influyen
como: desproporcin entre el volumen circulante y el lecho vascular. Trastorno en la circulacin capilar. Trastorno en el intercambio gaseoso. Hipoxia de
los tejidos y acidosis metablica. A 8ml/kg/min se inicia la isquemia celular. Es importante conocer la PIC= 10-12mmHg, la presin intrabadominal=
12mmHg, la presin arterial media (PAM), va de 70-100mmHg, es importante para saber si la perfusin es adecuada a nivel generalizado, asi
.
TIPOS
Definicion

HIPOVOLEMICO
Es la hipoperfusin con
hipoxia
celular,
secundaria a la perdida
de volumen circulante,
intra o extravascular

CARDIOGENICO
TAS <90mmHg o <30 por debajo de la
basal si se conoce por >30 min., que
no responde a la administracin de
lquidos y que es secundaria a una
disfuncin cardiaca

Signos
Sintomas

Hipotensin
arterial,
hipotermia, taquicardia,
palidez de tegumentos

Subtipos

Hemorrgico
No hemorrgico:
-3er
espacio
(pancreatitis, vasculitis,
quemados)
-Perdidas GI
-Perdidas urinarias (DM
inspida)

Hipotensin, IY, congestin pulmonar


y hepatica, galope o 3er ruido, pulso
paradjico y/o disminucin de
transmisin de ruidos cardiacos. IC
<2.2 l/min/m2. Palidez, oliguria,
retardo en llenado capilar. En IAM:
dolor torcico, disnea, diaforesis,
nasea, vmito.
Es la mxima expresin de la insuf.
cardiaca por disfuncin ventricular,
que se acompaa de disminucin
persistente y progresiva del GC,
acompaado de estado de choque e
hipoperfusin tisular, llevado a
disfuncin celular y falla multiorganica

Causas

Se
da
como
consecuencia de la
perdida
sbita
de
volumen intravascular

RVP
GC
PVC

Aumentado
Disminuido (<5 L/min)
Disminuida (<8 cm de
H2O)
Baja (<8 mmHg)

PCP

OBSTRUCTIVO
Obstruccin mecnica al
flujo de sangre que
ocasiona una poscarga
disminuida
y
una
inadecuada
perfusin
tisular

DISTRIBUTIVO
Estado de emergencia, existe una vasodilatacin, que
conlleva a una hipoperfusin tisular generalizada, porque
hay poca precarga. La falta de respuesta a manejo hdrico
y/o vasopresores, establece el diagnostico definitivo de
choque distributivo
Por lo general hipertrmico, taquicardico (>120 lpm),
taquipneico e hipotenso. En el neurognico dependiendo
nivel de lesin puede haber hemiplejia superior con
incapacidad para movilizacin y sensibilidad de
extremidades, y en ocasiones insuficiencia respiratoria.
Dao medular bajo, hemiplejia inferior con falta de
sensibilidad en extremidades e incontinencia de esfnteres.

El ms comn es el inducido por IAM,


otros:
contusin
miocrdica,
miocardiopata, rotura de cuerdas
tendinosas,
CIV,
aneurisma
ventricular, arritmias. Hasta en 7580% la disfuncin proviene del VI
Aumentado
Disminuido
Aumentada (>15 cm de H2O)

Lo ms frecuente TEP, otras


neumotrax a tencin,
tamponade
cardiaco,
estenosis mitral o aortica
aguda,
pericarditis
obstructiva
Normal
Disminuido
N, aumentada/disminuida

Sptico: hipotensin inducida por sepsis que no responde a


tratamiento con lquidos o vasopresores
Anafilctico: reaccin aguda y generalizada de gran
severidad que puede ser potencialmente mortal.
Neurognico: Alteracin hemodinmica por interrupcin de
la va simptica, lo que genera una vasodilatacin
permanente, antecedente de trauma, con compromiso del
SNC, el GC puede estar comprometido o no, ms comn
traumatismo medular (perdida del tono, lesion espinal)
Por infecciones, reacciones alrgicas o inflamatorias
severas, el ms comn es el sptico.
Anafilctico por: antibiticos, analgsicos, alimentos,
picadura de insectos, medios de contraste.
Neurognico por: accidentes vehiculares (40%), cadas,
violencia, nios durante deporte o actividades recreativas.
Disminuido
Aumentado
Normal o disminuida

Aumentada (>15 mmHg)

N, aumentada/disminuida

Disminuida

Es importante diferenciar entre shock sptico y sepsis. Sndrome de respuesta inflamatoria sistmica (SRIS): Cuando se presenta dos ms de los
siguientes signos; 1. Distermia: >38C o <35C. 2 Taquicardia: >90min. 3. Taquipnea: >20min o PaCO2 <32mmHg. Leucocitos: >12,000 o <4,000 con 10%
de bandas. Sepsis: Cuando un paciente tiene SRIS y un foco infeccioso Manifestaciones
evidente, con de
cultivo hemtico
positivodeobajo
negativo. Sepsis
severa: Hipotensin
Manifestaciones
Inestabilidad
ICC
gastoAccesos venosos perifricos,
hemodinmica
inducida por sepsis que responde al manejo de lquidos y/o vasopresores. ESTRATEGIAS
DE TRATAMIENTO:
son la primera
oliguria,
llenado
Hipotensin
arterial,
opcin para tratamiento de choque hipovolmico (catteres 14,16 o 18),Disnea,
sitios polipnea,
preferidos safenaPalidez,
externa,
yugular
externa, pliegue
de ambos
antebrazos.
estertores
capilar prolongado, reaccin
PAS <90mmHg, IC < 2.2
Los catteres cortos no debern emplearse en venas centrales a menospulmonares,
que sea indispensable.
Accesos
venosos
centrales,
indispensables
en
todas las
2
pltora
adrenrgica, piloereccin,
L/min/m , PCP
formas de choque, las vas preferidas de acceso central son: yugular interna
subclavia. Mtodos
de vigilancia,
oxigenacin >15mmHg
tisular, catter de presin
yugular,yS3
diaforesis,
cianosis perifrica
venosa central (PVC) intraarterial en aurcula derecha, para vigilancia de estado nutricional: BH, QS, orina de 24 hrs, vigilancia respiratoria, gasometra
arterial. Apoyo hemodinmica: orientado a la optimizacin de la oxigenacin, mediante frmacos; 1. Terapia inotrpica: corroborar una adecuada PVC y
DIAGNOSTICO CHOQUE CARDIOGENICO
PCP antes de iniciar tratamiento inotrpico. Una dosis inicial de
CLASIFICACION
DE CHOQUE
dobutamina (5-10 /kg/min), recomendacin de inicio en
HIPOVOLEMICO HEMORRAGICO
GRADO
I
II
III
IV
presencia de choque cardiognico. 2. Terapia vasopresora: de
PERDIDA/ML
750
750-1500
1500-2000
>2000
primera lnea dopamina cuando hay RVP disminuidas y GC
PERDIDA %
15%
15-30%
30-40%
>40%
aumentado, cuando no existe respuesta con dosis elevadas de
FC
<100
>100
>120
>140
dopamina (>20/kg/min), se recomienda norepinefrina o
FR
14-20
20-30
30-40
>40
epinefrina. La noradrenalina es un medicamento vasoconstrictor
T/A
NL
NL
Disminuido
Disminuido
potente, no genera tantas arritmias, es de primera eleccin en
P. pulso
Normal
Disminuido
Disminuido
Disminuido
DIURESIS
>30ml/h
20-30
5-15
0
choque septico. La vasopresina puede ser til en pacientes
Edo. mental
Ansiedad
Ansiedad
Confusin
Letargo
refractarios a norepinefrina, y se recomienda en el choque en fase
Rest. liquida
Cristaloides
Cristaloides
Crista +sangre
Crista +sangre
de vasodilatacin (choque sptico). La adrenalina es de primera
eleccin en choque anafilctico, en bolos subcutneos 3. Terapia vasodilatadora: su uso debe ser sustentado en monitoreo invasivo avanzado, que
provea medicin de RVS. Lquidos y fluidos: cristaloides; las mas empleadas solucin salina isotnica y Hartman, clnicamente puede esperarse de 100200ml de expansin, despus de infundir 1 litro de cristaloides isotnicos. Las soluciones hipertnicas tienen un potencial benfico como incremento en
la contractilidad miocrdica y vasodilatacin precapilar; sin embargo, pueden conducir a estados hiperosmolares con facilidad. Coloides; en Mxico se
cuenta con 4 como albumina (frasco de 50cc con concentracin al 20%), gelatina (haemacel), dextranes, almidn (hidroximetilalmidon al 10%). Un litro
de albumina al 5%, produce una expansin de plasma de 500-1000 ml. Un litro de almidn expande 700ml. Tanto gelatinas como almidones son
inductores de ditesis hemorrgica, en forma dosis dependiente. El dextran compromete dramticamente la hemostasis. Sangre y hemoderivados: no se
ha demostrado beneficio en la perfusin tisular, al transfundir pacientes en estado de choque con niveles de hemoglobina de 8-10mg/dl. Las siguientes
condiciones clnicas podrn beneficiarse de hemotransfusion, taquicardia importante, desaturacin venosa mezclada importante, disfuncin cardiaca,
enfermedad arterial coronaria coexistente, acidosis lctica persistente, incapacidad otros marcadores de hipoperfusin celular. No existe un nivel de

CURSO ENARM CMN SIGLO XXI TEL: 36246001

Pharmed Solutions Institute

PGINA 150

MANUAL DE TRABAJO DEL CURSO ENARM CMN SIGLO XXI


hemoglobina que indique hemotransfucion con el sustento de mejorar el aporte sanguneo a los tejidos. Las complicaciones de hemotransfusion masiva,
lleva a mayor riesgo de infecciones y reacciones anafilcticas, puede llevar a SIRA, aqu se llama TRALI, puede dar a partir de las 6 hrs de trasnfundido.
Apoyo ventilatorio; en todo momento asegurase del adecuado funcionamiento de la va area, siendo la permeabilidad e integridad de la misma una
condicin indispensable para cumplir con la meta de oxigenacin sistmica. Los elementos que pueden orientar para el empleo de asistencia en la
ventilacin son: administrar puntas nasales, mascara facial, cnula farngea o mascarilla con reservorio en caso de contarse con ella, en cualquier
situacin clnica que sugiera estado de choque. Criterios recomendados para decidir intubacin endotraqueal, con manejo de ventilador mecnico en un
paciente en estado de choque; hipercarbia >32mmHg, taquipnea >20 rpm, cianosis distal con alguno de los elementos anteriores, acidosis respiratoria
con pH=7.2, paciente con relacin PaO2/FiO2 <150, paciente con sospecha de quemadura de va area. Para quelar citratos se usa gluconato de calcio.
METAS DE REANIMACION: En 6 hrs; PAM>65, disminuir el lactato, uresis >0.5ml/kg/h, PVC 8-12 cmH2O, Svo2 >70%. MARCADORES SERICOS DE
CHOQUE: Lactato (1.5-2mmol/l); es un reflejo de hipo perfusin tisular, producto de la glicolisis anaerobia, valores por arriba de 4mmol/l, se asocia con
un 90% de mortalidad, por arriba de 2.5 mmol/l alta incidencia de falla orgnica mltiple (FOM), su medicin nos sirve como factor pronostico. Dficit de
base; aproximacin global del grado de acidosis e indirectamente el grado de perfusin tisular, nos orienta sobre la severidad de choque: a) -2 a -5meq/l
leve, b)-6 a -14 moderado, c) mayor a -15.
CASO CLINICO CHOQUE HIPOVOLEMICO
Se trata de paciente masculino de 23 aos de edad el cual ingresa a
urgencias presentando orificio de entrada en flanco derecho, por arma
de fuego, sin orificio de salida, a la exploracin fsica se observa con FC
125 lpm, llenado capilar lento, FR 31 rpm, se observa ansioso con
tendencia a la confusin, su gasto urinario fue de 20 ml/h.
PREGUNTA
Considerando las manifestaciones clnicas observadas asi como las
variables vitales. En que clase se encuentra el paciente?
RESPUESTA
a.- Clase I.
b.- Clase II.
c.- Clase III.
d.- Clase IV.
PREGUNTA
Cual es la conducta mas adecuada para mantener el volumen
circulante?
RESPUESTA
a.- Solucin salina 10 ml/kg/hora.
b.- Solucin hartman 15ml/Kg/hora.
c.- Crioprecipitados.
d.- Concentrado eritrocitario y cristaloide.
PREGUNTA
Una hora despues, el paciente presenta los siguientes constantes
vitales, FC 105 lpm, FR 21 rpm, gasto urinario de 35 ml/h, se encuentra
ansioso, pero no confundido, la hemoglobina fue de 12 y hematocrito
de 37, plaquetas de 120,000, asi como DHL 312, AST 435, tiempos de
coagulacin levemente incrementados, cual es la conducta a seguir en
este momento?
RESPUESTA
a.- Mantener observacin estrecha.
b.- Preparar para laparatomia exploratoria.
c.- Incrementar volumen circulante con paquete globular.
d.- Vasopresores, volumen y LAPE.
PREGUNTA
Se realizo LAPE con reparacin heptica con buena respuesta, fue
necesario crioprecipitados, plaquetas y concentrado eritrocitario, se
administro 3 gramos de gluconato de calcio, cual es la razn de esta
indicacin?
RESPUESTA
a.- Alteracin de coagulacin.
b.- Mantener estable la membrana.
c.- Evitar arritmias.
d.- Disminuir la adhesividad plaquetaria.
CASO CLINICO CHOQUE HIPOVOLEMICO
Se trata de paciente femenino de 48 aos de edad, la cual fue
ingresada posterior a accidente automovilstico y volcadura, era
copiloto y fue la nica sobreviviente, se observa confundida,
desorientada, agitada, se presenta diaforesis, con palidez de mucosas,

CURSO ENARM CMN SIGLO XXI TEL: 36246001

su llenado capilar lento, FC de 138 lpm, FR 34 rpm, TA 100/65 mmHg,


minutos despus se estableci un Glasgow de 8, distensin abdominal
e inestabilidad toracicca, su gasto urinario fue de 20 ml/h, Campos
pulmonares con matidez a la percusin bibasal, no hay ingurgitacin
yugular, pero hay equimosis importante sobre area del cinturn de
seguridad, en la radiografia porttil se observo fractura de 5 costillas
del lado izquierdo.
PREGUNTA
Cual es la medida inmediata mas adecuada en este momento?
RESPUESTA
a.- Intubacin endotraqueal.
b.- Solucin hartman, vasopresores.
c.- Solucion salina hipertnica.
d.- Sedacion y relajacin muscular.
PREGUNTA
Posterior al decisin anterior se administra 3 paquetes de
concentrados eritrocitarios, 3 paquetes de plasma fresco y 2 litro de
solucin hartman, sin embargo el gasto urinario es de 25 ml/hr, la
tensin arterial se mantiene en 95/60 mmHg durante la primera hora
de estancia. Cual es la conducta a seguir mas adecuada?
RESPUESTA
a.- Albumina y expansores.
b.- Infusion de dopamina 5 g/Kg/min. (Inicial noerepinefrina)
c.- Infusion de dobutamina 10 g/kg/min.
d.- Crioprecipitados y gluconato de calcio.
PREGUNTA
Cual es el objetivo mas importante de la conducta farmacolgica
tomada anteriormente?
REPUESTA
a.- Mantener el gasto urinario.
b.- Incrementar la fraccin de eyeccin.
c.- Mantener la tensin arterial permisible.
d.- Favorecer el inotropismo.
PREGUNTA
El paciente presenta estabilidad en la tensin arterial 95/70 mmHg y
gasto urinario de 35 ml/hr, se prepara para laparatoria exploratoria,
tomando en cuenta la decisin farmacolgica anterior, cual de los
siguiente efectos no favorece al caso?
RESPUESTA
a.- Incremento mayor del 25 % sobre la tensin arterial basal.
b.- Aumento del flujo sanguneo renal.
c.- Disminucin de la resistencia vascular perifrica.
d.- Disminucin de la demanda de O2 del miocardio.
CASO CLINICO ESTADO DE CHOQUE OCULTO
Se trata de paciente masculino de 19 aos de edad el cual ingresa a
urgencias posterior a accidente automovilstico, refiere cefalea y dolor
lumbar leve, permaneci en observacin, se coloco catter perifrico
18 fr y solucin glucosada al 5 % para 8 hrs, a las 4 horas posterior al
ingreso refiere ansiedad, sensacin de mareo, a la EF se observa

Pharmed Solutions Institute

PGINA 151

MANUAL DE TRABAJO DEL CURSO ENARM CMN SIGLO XXI


palidez de tegumentos, diaforesis, torax sin datos patolgicos,
abdomen con ruidos peristlticos disminuidos, SV con TA 100/70
mmHg, FC 105 lpm, FR 32 rpm, no se apresia compromiso oseo.
PREGUNTA
Se realizan realiza QS, BH, RX de torax y abdomen, Hb 10.1, Hto 30, Rx
de abdomen con niveles hidroaereos, imagen despulida, cual es la
conducta a seguir ms adecuada?
RESPUESTA
a.- Solucin hartman 10 ml/kg/hora.
b.- Solucin salina 0.45 % 10 ml/kg/hora.
c.- Solucin mixta 10 ml/kg/hora.
d.- Solucin glucosada al 10 % 10 ml/Kg/hora.
PREGUNTA
Posteriomente se administra 3 concetrados eritrocitarios y 2 paquetes
de plasma fresco, la tensin arterial se mantiene estable en 100/70
mmHg sin embargo la ursis fue de 40 ml/hora, se observa auscencia
de ruidos peristlticos, se observa estado de conciencia adecuado,
alerta y tranquilo. cual no es un objetivo primordial en este momento
del caso?
RESPUESTA
a.- Elevar la tensin arterial media por arriba de 95 mmHg.
b.- Manterne la hemostasia con gluconato de calcio.
c.- Estabilizar hemostasis por sospecha de hemorragia.
d.- Mantener un adecuado volumen urinario.
PREGUNTA
Cual es el origen menos probable de la perdida del volumen circulante
del caso actual?
RESPUESTA
a.- Contusin cardiaca.
b.- Rotura esplenica.
c.- Hemorragia heptica.
d.- Hemorragia retroperitoneal.
PREGUNTA
Se realizo tomografa donde se confirmo hemorragia retroperitoneal y
lactato elevado, cual de las siguientes complicaciones de alto volumen
de reposicion es menos frecuente?
RESPUESTA
a.- Acidosis hipernatremica.
b.- Sndrome de fuga capilar.
c.- Perdida de la estabilidad de un coagulo
d.- Disminucin de la precarga con aumento de FE.
CASO CLINICO CHOQUE ANAFILACTICO
Mujer de 34 aos sin antecedentes patolgicos, no hbitos txicos, no
alergias a frmacos conocidas. Present cefalea tensional en
tratamiento con cido acetilsaliclico con episodios de urticaria aguda
en relacin con el consumo de marisco, tratado y remitido hace 24 hrs.
4 horas despus apareci dolor faringeo, tumefaccin facial, de manos
y brazos, con disminucin de la diuresis. A las pocas horas de la
hospitalizacin present una alteracin hemodinmica con taquicardia
sinusal a 140 pulsaciones por minuto e hipotensin.
PREGUNTA
Considerando el cuadro clnico cual es la conducta inmediata a seguir,
la paciente se encentra con vas de acceso venoso?
RESPUESTA
a.- Administracin de volumen con cristaloides.
b.- Administracion de aminas vasoactivas.
c.- Colocacin de catter largo.
d.- Administracin de epinefrina.
PREGUNTA
Posteriormente se deterior el nivel de conciencia y se produjo un
paro respiratorio que precis intubacin y ventilacin mecnica. Se

CURSO ENARM CMN SIGLO XXI TEL: 36246001

consigui la estabilizacin hemodinmica con dopamina y expansores


de volumen, crioprecipitados y plasma fresco, cual es la complicacin
ms probable?
RESPUESTA
a.- Coagulacion por hemodilucin.
b.- Sindrome compartimental.
c.- Coagulacion intravascular por consumo.
d.- Coagulopatia dilucional.
PREGUNTA
Considerando el cuadro clnico y la evolucin del caso, cual de las
siguientes causas es la menos probable?
RESPUESTA
a.- Reacciones a frmacos.
b.- Reacciones a alimentos.
c.- Reacciones transfusionales.
d.- Reacciones a antgenos inhalados.
PREGUNTA
El da 11 de estancia en cuidados intensivos se introduce tratamiento
con linezolid por va intravenosa, 600mg/12h por aislamiento de
Enterococcus faecium resistente a meticilina, sensible a linezolid, en el
empiema pleural bilateral. El resto de tratamiento es: enoxaparina
40mg/24h (11 das), propofol 300mg/h y meperidina 15mg/h en
perfusin continua (como sedoanalgesia) (10 das), imipenem 1g/8h
(10 das), amikacina 1,2g/24h (5 das). A las 48 h del inicio de linezolid
se observan crisis comiciales en relacin con movimientos
tonicoclnicos preferentemente de extremidades superiores y tronco
sin llegar a generalizarse, transitorios (pocos minutos de duracin) y
repetidos a lo largo del da. Considerando la evolucin y los nuevos
sntomas cual es su impresin?
RESPUESTA
a.- Choque anafilctico.
b.- Reaccion adversa a medicamento.
c.- Choque sptico.
d.- Sindrome serotoninergico maligno.
PREGUNTA
Se observa tambin hiperreflexia, discreta acidosis metablica y
empeoramiento grave de la fiebre, leucocitosis, marcadores de
inflamacin y estado hemodinmico (hipotensin). Ante la sospecha
de RAM se sustituye meperidino por fentanilo y se aade midazolam,
10mg/h, en perfusin continua al esquema de sedoanalgesia para la
mejora del cuadro y noradrenalina para el soporte hemodinmico.
Considerando las variable vitales, hipotensin refractaria cual de las
siguientes medidas es mas adecuada?
RESPUESTA
a.- Incrementar aminas vasoactiva.
b.- Hemodialisis.
c.- Retirar linezolid y administrar metilprednizolona.
d.- Concentrado eritocitario y plasma fresco congelado.
PREGUNTA
Los cultivos microbiolgicos fueron negativas con la excepcin de los
drenajes torcico y abdominal, en los que se aisl Candida spp. Se
administro anfotericina B liposomal en dosis de 4mg/kg/da. Linezolid
fue tambin suspendido y se paut daptomicina, 350mg/24h. Tras el
cambio de tratamiento las mioclonas e hiperreflexia mejoraron antes
de 24 h, hasta desaparecer por completo al segundo da. La fiebre,
leucocitosis e hipotensin mejoraron tambin progresivamente,
aunque la acidosis metablica persisti. Cual es la complicacin mas
probable?
RESPUESTA
a.- Dudoso por acidosis metabolica.
b.- Coagulopatia por dilucin.
c.- Alergia al frmaco.
d.- Falla organica multiple.

Pharmed Solutions Institute

PGINA 152

MANUAL DE TRABAJO DEL CURSO ENARM CMN SIGLO XXI


PREGUNTA
El diagnstico de sndrome serotoninrgico suele ser difcil de realizar
por la presencia de sntomas inespecficos y comunes a otras
entidades. Esta problemtica puede ser todava mayor en el caso de un
paciente crtico y polimedicado como el que nos ocupa, donde los
sntomas pueden ser atribuidos a mltiples factores de la enfermedad
o del abordaje farmacolgico. Cual es la base principal para
diagnosticar este caso?
RESPUESTA
a.- Inicio brusco.
b.- Recuperacin tras suspensin del frmaco.
c.- La presencia de fiebre, hiperreflexia y acidosis metabolica.
d.- Criterios diagnsticos de Sternbach y Dunkley.
CASO CLINICO CHOQUE CARDIOGENICO
Se trata de un varn de 74 aos que ingres por ebre y deterioro del
estado general de meses de evolucin. Como antecedentes
personales, cabe destacar gastritis crnica atrca, cardiopata
reumtica con afectacin artica y mitral y brilacin auricular con
tratamiento anticoagulante. En la exploracin fsica no haba
alteraciones signicativas. En la analtica, a resear una creatinina de
4,17 mg/dL no conocida y un INR de 7. En los hemocultivos realizados,
crecieron en 2/3 L. monocytogenes. En el estudio de la bacteriemia, se
solicit un ecocardiograma transtorcico que puso de maniesto una
verruga en la vlvula artica, con fraccin de eyeccin deprimida,
KDOQ III.
PREGUNTA
Se complet el estudio con una RMN craneal, en la que se observaron
mltiples infartos cerebrales, y con una ecografa abdominal en la que
se apreci un hgado de estasis. La evolucin fue mala, desarrollando
insuciencia cardiaca por lo que se decidi realizar ciruga cardiaca el
7 da. En la intervencin, se encontraron vegetaciones y abscesos que
afectaban tanto a la vlvula artica como a la mitral, el paciente
presento hipotensin refractaria, sin respuesta a aminas vasopresoras,
hipotermina, piel marmrea. Cual es el tipo de choque mas probable
en este momento del caso?
RESPUESTA
a.- Choque cardiogenico.
b.- Choque sptico.
c.- Choque restrictivo.
d.- Choque oculto.
CASO CLINICO CHOQUE SEPTICO
Femenino de 63 aos, con antecedentes de hipertensin arterial
(HTA), diabetes mellitus (DM) tipo II, dislipidemia, infecciones urinarias
y clicos nefrticos de repeticin, que presenta un cuadro de cinco
meses de evolucin de dolor abdominal de predominio en fosa ilaca
izquierda, vmitos y fiebre intermitente. Acude en diversas ocasiones a
Urgencias, siendo diagnosticada de infeccin urinaria y clico
renoureteral. Recibiendo manejo farmacolgico empirico a dosis y
tiempo adecuado, debido a la persistencia del dolor se realiza
ecografa abdominal, objetivndose ndulo en segmento IV heptico y
quistes renales corticales.
PREGUNTA
Ante dichos hallazgo. Cual es la conducta mas adecuada a seguir?
RESPUESTA
a.- Bh completa, QS, EGO, Hemocultivo y urocultivo.
b.- TAC de trax, abdomen y pelvis con contraste.
c.- IRM-F torax y abdomen con marcadores inflamatorios.
d.- Biopsia heptica y renal.
PREGUNTA
A los dos meses acude de nuevo a Urgencias por dolor intenso en
hipocondrio derecho y mal estado general. En la EF destaca signo de
Murphy positivo e hipotensin severa, creatinina 3,68mg/dl, urea
96mg/dl, transaminasa glutmico oxalactica (GOT) 55 UI/l,

CURSO ENARM CMN SIGLO XXI TEL: 36246001

transaminasa glutmico-pirvica (GPT) 92 UI/l, protena C reactiva


(PCR) 451mg/dl. Se realiza TAC de abdomen sin contraste: probable
colecistitis, elevacin de hemidiafragma derecho, atelectasia basal
derecha, hemangiomas hepticos, granuloma esplnico y quistes
renales. La paciente presenta deterioro significativo hemodinamico,
con obnulizacin, hipotensin, taquicardia, taquipneica, mal
perfundida con signos de abdomen agudo cual es la conducta a seguir?
RESPUESTA
a.- Preparar para LAPE.
b.- Iniciar dopamina.
c.- Iniciar dobutamina.
d.- Ingresar a UCI.
PREGUNTA
Se realizan laboratorios con los siguientes resultados: hemoglobina
8,7g/dl, leucocitos 2.900 (neutrfilos 77%), actividad de protrombina
54%, PCR 359mg/ml, procalcitonina 3,8ng/dl, cido lctico 18mg/dl,
creatinina 2,9mg/dl, urea 88mg/dl, GOT 184 UI/l, GPT 111 UI/l,
fosfatasa alcalina 229 UI/l, gasometra arterial pH 7,28, pCO2 41; pO2
66, HCO3 19, saturacin arterial de O2 91%, cual es la conducta a
seguir menos apropiada ante el estado de choque sptico del caso?
RESPUESTA
a.- Flunitracepam y pancuronio e intubacin orotraqueal con conexin
a ventilacin mecnica.
b.- Estabilizacin hemodinmica con cristaloides y perfusin de FVA
(noradrenalina y dopamina).
c.- Correccin de coagulacin con concentrado plaquetario,
crioprecipitados y plasma fresco.
d.- Correccion intensiva de valores gasomtricos y perfusin de FVA
(dobutamina y vasopresina).
CASO CLINICO CHOQUE SEPTICO
Mujer de 66 aos que ingresa por cuadro de shock sptico secundario
a pielonefritis aguda derecha con disfuncin multiorgnica (DMO).
Requiri apoyo hemodinmico, terapia sustitutiva renal y ventilacin
mecnica prolongados. Precis dobutamina (20 mcg/kg/min) y
noradrenalina (2-2,5 mcg/kg/min) durante una media de 6 semanas.
Recibi adems terlipresina (1mg/4h) y azul de metileno. Se
administr hidrocortisona y antibioterapia dirigida. Las dosis elevadas
y prolongadas de aminas vasoactivas provocaron vasoconstriccin
distal intensa con necrosis isqumica digital que precis la amputacin
de todos los dedos de los miembros inferiores. La retirada de las
medidas de soporte orgnico fue lenta as como la recuperacin
posterior. La estancia en UVI fue de 120 das. En planta comenz con
dolor en ambas caderas que limit la rehabilitacin. No haba
antecedentes de traumatismo o cada. Las pruebas de imagen
informaron de lesiones compatibles con necrosis avascular de caderas.
PREGUNTA
Cuales de los siguientes criterios diagnosticos es menos til para
identificar sepsis?
RESPUESTA
a.- PAM < 60 mmHg PAS < 90 mmHg.
b.- Hipertermia y/ hiportermia.
c.- Volumen urinario <20 ml/hora.
d.- Velocidad de llenado capilar.
PREGUNTA
Cuales son los valores analticos es de poca utilidad dentro de las
primeras horas del padecimiento para confirmar el diagnostico de SRIS
y los procesos spticos, adems de los datos clnicos?
RESPUESTA
a.- Hiperglucemia > 120 mg/dl sin DM.
b.- Procalcitonina.
c.- Leucopenia < 4,000.
d.- Proteina C reactiva 2DS mayor a lo normal.
PREGUNTA

Pharmed Solutions Institute

PGINA 153

MANUAL DE TRABAJO DEL CURSO ENARM CMN SIGLO XXI


Cual de los siguientes analticos no es un elemento del sistema PIRO
para estimar pronostico del paciente?
RESPUESTA
a.- Grado de agresin de la infeccin.
b.- Velocidad de instauracin.
c.- Numero de rganos con disfuncin
d.- Intensidad de la respuesta.

antecedentes y estado actual cual es la conducta a seguir para


establecer un agente etiolgico.
RESPUESTA
a.- Ecocardiograma transesofagico.
b.- RMN craneal por sntomas neurolgicos.
c.- Cultivo con antibiograma.
d.- BH y tiempos de coagulacin.

PREGUNTA
Cul de las siguientes no es una meta adecuada en el tratamiento de la
sepsis y choque sptico?
RESPUESTA
a.- Presion venosa central (PVC) 8-12 mmHg.
b.- Presion arterial media (PAM) > 65 mmHg.
c.- Gasto urinario (GU) 0.3 ml/kg/hr.
d.- Saturacion de sangre venosa (SATVO2) > 70 %.

CASO CLINICO
Hombre de 48 aos, con antecedente familiar de cardiopata
coronaria, tabquico y dislipidmico sin tratamiento farmacolgico.
Present intenso dolor precordial mientras jugaba ftbol, consultando
en un servicio de urgencia donde present paro cardiorrespiratorio por
fibrilacin ventricular. Fue tratado con mltiples desfribilaciones y
maniobras de reanimacin durante 25 min. El electrocardiograma
(ECG) mostr supradesnivel del segmento ST en la pared anterior. Se
traslad al Hospital para estudio hemodinmico de urgencia, con IAM
de 9 h de evolucin. Al ingreso se encontraba intubado con asistencia
ventilatoria mediante amb, hemodinmicamente inestable
requiriendo infusin de noradrenalina a 0,1 g/kg/min. El ECG mostr
onda Q y supradesnivel del segmento ST hasta 3 mm en VI a V3.
Troponina I: 292 ng/mL (normal <0,05). Su perfil hemodinmico
mostr: presin arterial de 80/63 mmHg, IC 2.0 L/min/m2, PCP
23mmHg. Se administraron lquidos durante 1 hora sin respuesta. A la
auscultacin, pulso paradjico, IY, disminucin de la transmisin de
ruidos cardiacos.

PREGUNTA
Cual de los siguientes criterios de reanimacin del paciente con sepsis
grave se encuentra fuera de rango?
RESPUESTA
a.- Soluciones cristaloides > 1000 ml/30 min 300 a 500 ml coloides.
b.- Trasfusin de concentrados eritrocitarios para lograr un
hematocrito > de 30 % con ScvO2 baja.
c.- Administracion de plasma fresco congelado con elevacin de TP,
INR TPT.
d.- Perfusion de dobutamina 50 mcg/kg/min
PREGUNTA
Cual de los siguientes parmetros guarda mayor relacin de la
principales complicaciones sin embargo menos especifico para vigilar
al caso clnico actual?
RESPUESTA
a.- Proteina C reactiva.
b.- Hiperlacticidemia.
c.- Procalcitonina.
d.- Creatinina serica
PREGUNTA
Cual de los siguientes parmetros guarda menor sensibilidad en
relacin con el sndrome de respuesta inflamatoria sistmica?
RESPUESTA
a.- Proteina C reactiva +/-1DS mayor.
b.- INR > 1.5
c.- TTPa > 60
d.- Trobocitopenia < 100,000 celulas/mm3.
CASO CLINICO CHOQUE HIPOVOLEMICO
Se trata de una mujer de 58 aos que ingres por ebre y cefalea de
aparicin brusca. Dentro de sus antecedentes personales, destacaba la
presencia de prtesis mecnica mitral, artica secundaria a
valvulopata reumtica, y nefrectoma derecha. Se encontraba
recibiendo tratamiento anticoagulante. A la exploracin fsica,
nicamente llamaba la atencin cierto grado de rigidez nucal. En la
analtica inicial, leve deterioro de su insuciencia renal de base
(creatinina de 1,83 mg/dL).
PREGUNTA
Ante tales manifestaciones, se realiz una TAC craneal as como
puncin del LCR que no presentaron alteraciones. Considerando los

CURSO ENARM CMN SIGLO XXI TEL: 36246001

PREGUNTA
En qu clasificacin de Forrester se encuentra esta paciente?
RESPUESTA
a.- Grupo A
b.- Grupo B
c.- Grupo L
d.- Grupo C
PREGUNTA
Cul es la conducta teraputica ms adecuada para tratar a esta
paciente?
RESPUESTA
a.- Baln intraartico de contrapulsacin (BIAC) y terapia de
reperfusin lo ms pronto posible con intervencin coronaria
percutnea
b.- Fibrinolisis seguida de estabilizacin farmacolgica intensiva
c.- Cateterismo y angiografa coronaria y estabilizacin farmacolgica
d.- Balon intra-aortico de contrapulsacion (BIAC) y estabilizacin
farmacolgica intensiva
PREGUNTA
Qu estrategias son las ms adecuadas para prevenir la muerte
celular miocrdica as como evitar la alteracin del estado metablico
en este paciente?
RESPUESTA
a.- IECA, digoxina, Inhibidores de complemento
b.- Beta bloqueadores, Bloqueadores de la sintasa de xido ntrico,
Hipotermia
c.- Nitroglicerina, betabloqueadores, Bloqueadores de la sintasa de
xido ntrico, Hipotermia
d.- Inhibidores de complemento, Bloqueadores de la sintasa de xido
ntrico, Hipotermia

Pharmed Solutions Institute

PGINA 154

MANUAL DE TRABAJO DEL CURSO ENARM CMN SIGLO XXI


CRISIS HIPERTENSIVA:
CIENCIAS BASICAS: Aumento sbito de la presin arterial, habitualmente por arriba del estadio 2 (>160/>100mmHg), y que puede (o no), estar
acompaada a rgano blanco. Las crisis hipertensivas incluyen a las emergencias y urgencias. Emergencia hipertensiva: Elevacion de presin arterial
>179/>109 que se acompaa de dao de rgano agudo, que requiere rpido descenso de TA, en un intervalo de horas. Usualmente requiere internacin
en unidad de cuidados crticos con medicaciones por va parenteral. En estos casos la TA cumple un rol patognico en la gnesis y progresin del cuadro
clnico. Debemos recalcar que la emergencia est denida por la gravedad del cuadro clnico y no por los valores de TA, usualmente la TAD es mayor 120
mmHg. Los rganos que son afectado ms frecuentemente son el cerebro (EVC 24.5%, encefalopata 16.3%, hemorragias intracerebrales o
subaracnoideas 4.5%), corazn (EAP 22.5%, ICC 14.3%, IAM 12.0%, diseccin aortica 2.0%), grandes vasos, rin y tero gravdico (eclampsia 4.5%,
preeclampsia grave), siendo ms frecuente la afectacin nica en el 83% de los casos, pero en porcentaje variable se pueden presentar ms de un dao
de rgano blanco. Urgencia hipertensiva: cuando se produce una elevacin brusca de la presin arterial, tambin puede ser en relacion a las cifras
tensinales habituales, sin dao a organo diana, pero con sintomatologa significativa. El objetivo teraputico sera reducir gradualmente la PA (en 2448h) con medicacin oral ya que si se baja muy rpidamente puede favorecerse la hipoperfusin de rganos diana. Debe distinguirse de una pseudocrisis
hipertensiva, que es la elevacin de la PA reactiva a una situacin de stress o dolor, hipercapnia, hipoglucemia, estado posictal. Hipertensin severa
aislada: Elevacin de la TAD >120 mmHg, que requiere descenso de valores de TA en lapso de das con medicacin y usualmente sin internacin. Por
definicin se debe establecer previamente la ausencia de dao de rgano blanco, hay poca evidencia que el control de la TA, en un lapso de tiempo
corto sea beneficiosos. SALUD PBLICA: La Hipertensin Arterial (HTA) es un proceso crnico que presenta una prevalencia de ms del 25% en la
poblacin occidental adulta. Aproximadamente un 1-2% de los pacientes hipertensos desarrollaran una crisis hipertensiv en algn momento de su vida.
El tratamiento y el adecuado seguimiento de estos pacientes aumentan su esperanza de vida y disminuye la incidencia de complicaciones. PATOGENIA:
T/A = (gasto cardiaco) (resistencias vasculares perifricas). Para que la presin arterial aumente se requiere desequilibrio en esta relacin, como falla de
mecanismos autorreguladores, alteraciones en propiedades contrctiles del musculo arteriolar, activacin del sistema SRAA (angiotensina II, radicales de
O2, disminucin de oxido ntrico), vasoconstriccin y aumento de RVP, aumento de permeabilidad vascular, proliferacion celular, activacin de cascada
de coagulacion. El rpido desarrollo de dao de rgano agudo de las emergencias hipertensivas es atribuido a la falla en el mantenimiento de la normal
autorregulacin de los lechos vasculares de rganos de choque (cerebro, rin, retina y corazn) y por otro lado un aumento abrupto de las resistencias
vasculares sistmicas. Conduciendo a una injuria endotelial con necrosis fibrinoide de arteriolas. Establecindose el ciclo isquemia, depsito plaquetario,
y falla de la autorregulacin con deterioro clnico del paciente. DIAGNOSTICO: Historia clnica: historia de la HTA (edad de inicio, tiempo de evolucin,
grado de control, cifras basales, tratamiento, sntomas acompaante, evolucin hasta el momento del ingreso, hbitos higinico-diet cos (alcohol,
drogas), enfermedades asociadas, factores de riesgo cardiovascular, antecedentes familiares: HTA, enfermedades cardiovasculares, renales, obesidad,
dislipemia). Cuadro clnico; cefalea, mereo, vomito, alteraciones visulaes, dolor torcico, dificultad repiratoria. Exploracin fsica: Priorizar en primer
lugar la bsqueda de dao a rgano blanco. Medicin de la PA. Peso, talla, IMC y permetro abdominal. Exploracin sistemtica (cardiovascular,
pulmonar, abdominal, neurolgica, ocular, cutnea, cuello y palpacin tiroidea). Palpacin de pulsos perifricos (en las cuatro extremidades) y
centrales valorando simetras y amplitud. Fondo de ojo (hemorragias, exudados blandos, papiledema). Pruebas complementarias: En urgencia
hipertensiva: sistemtico de orina, ECG (isquemia, arritmia, hipertrofia VI), Rx posteroanterior y lateral de trax. En emergencia hipertensiva:
hemograma, bioqumica con CPK, CPK-MB, troponina I (si existe sospecha de SCA), sistemtico de orina, ECG, gasometra arterial, si existe sospecha de
EAP o IC, fondo de ojo (grado de retinopata), Rx posteroanterior y lateral de trax, TAC craneal (si existe sospecha de ictus), ecocardiograma (si hay
cardiopa a o sospecha de disfuncin cardiaca), TAC tracoabdominal (si existe sospecha de aneurisma disecante de aorta). TRATAMIENTO: Si dao a
rgano blanco: Emergencia hipertensiva, Objetivo: Disminuir la PAM en un 25% en un periodo comprendido entre minutos y 1ra hora o hasta niveles
seguros (excepto en la diseccin artica), reduccin de PAD 10-15% o 110mmHg entre 30-60min, meta es 160/100-110mmHg en la 2-6ta hora, evitar
tratamiento sublingual, hospitalizacin, ABC soporte vital avanzado, si es necesario. Medicamentos: 1. LABETALOL (diseccin aortica, IAM, sindromes
coronarios agudos): bloqueador mixto (musc. Cardiaco), (musc, liso), dosis de impregnacin 20mg, seguida por una dosis incremental de 20 a 80mg
con intervalos de 10 min hasta conseguir el efecto deseado (mximo 300mg). Reducen la mortalidad asociada a arritmias, no produce taquicardia refleja,
no afecta el FSC, ni la funcin renal, contraindicado en ICC, bloqueo cardiaco, feocromocitoma y asma. 2. NICARDIPINA (encefalopata hipertensiva,
hemorragia intracraneal, posoperatorio, embarazo): es un calcio antagonista (dihidropiridina) con alta selectividad vascular y fuerte actividad
vasodilatadora cerebral y coronaria. La infusin inicial es de 5mg/h con aumentos de 2.5mg/h cada 15min, hasta un mximo de 15 mg/h, contraindicado
en cirrticos y renales, causa enrojecimiento facial. 3. NITROPRUSIATO DE SODIO (EVC, isquemia, encefalopata, hemorragia subaracnoidea): de eleccin
en la mayora de las emergencias hipertensivas, es un vasodilatador arterial y venoso que disminuye la postcarga y precarga. IV de 0.5-10 mg/kg/min
(inicial 0.5-1.0), accin en 3-5min. Es extremadamente potente, til monitoreo intraarterial, puede generar toxicidad por cianuros y tiocianato (nausea,
vomito, clicos, fsciculaciones, convulsiones, psicosis) a altas dosis o por el uso prolongado vasodilatador cerebral por lo que aumenta el FSC y puede
provocar aumento de la presin intracraneana. 4. NITROGLICERINA (isquemia cardiaca y edema pulmonar): es un venodilatador potente, disminuye
precarga y presin telediastolica del VI, peligroso en pacientes con deterioro de perfusin renal y/o cerebral. La dosis en infusin es de 5-200ug/min
iniciar con 20ug y aumentar cada 5 min. 5. HIDRALACINA (embarazo): vasodilatador arterial directo de accin predominante arteriolar, droga de eleccin
en crisis hipertensivas en embarazo. IM 5 mg de incio, repitiendo cada 20 min de 5-10mgs, lupus li e, despus de consumirlo de 3-6 meses. Eclampsia,
metildopa, MgSO4. 6. ISOSORBIDE; donador de oxido ntrico, evita la isquemia tisular, aumenta el flujo coronario. NO dao a rgano blanco: Urgencia
hipertensiva: Si PAS 210 mmHg o PAD 120 mmHg. Objetivo: Disminuir la presin arterial media (PAM) en un 20% en un periodo de 24-48 horas o
disminuir la PAD a valores < 120 mmHg. La disminucin debe ser gradual a fin de prevenir isquemia orgnica (cardiaca, cerebral). Reposo 5-10 minutos
en lugar tranquilo y en decbito supino. Valorar ansioltico suave (benzodiacepina) si ansiedad/nerviosismo. Comprobar el nivel de PA. Es importante
saber si el paciente estaba recibiendo tratamiento antihipertensivo previo o no: Si estaba a tratamiento y lo ha abandonado, hay que reiniciarlo de
nuevo, pero si con na con la medicacin hay que aumentar la dosis del que toma o asociar nuevos frmacos. Si no est a tratamiento y es necesario
medicacin antihipertensiva, debe emplearse un frmaco de accin lenta (ej. captopril) evitando los de accin rpida porque pueden inducir isquemia en
los rganos diana (nifedipino sublingual). Nunca descender la PA por debajo de las cifras habituales del paciente. Tratamiento en primer nivel con
medicamentos va oral: 1. INHIBIDORES DE LA ENZIMA CONVERTIDORA DE ANGIOTENSINA: a. Captopril, a dosis de 25 mg, puede utilizarse por va oral o
sublingual comenzando su accin a los 15-30 minutos, hasta un mximo a los 50-90 minutos, durando su accin 4-6 horas. Si a pesar del tratamiento la
PA 210/120 mmHg se puede repetir la dosis 2-3 veces a intervalos de 30 minutos hasta un mximo de 100 mg. b. Enalapril: 10mg cada 12 horas, dosis
mxima 40mg al da 2. BETABLOQUEADORES: a. Metroprolol: 50-100mg cada 12 horas b. Atenolol: 50mg cada 12-24 horas 3. ANTAGONISTAS DE LOS
RECEPTORES DE ANGIOTENSINA II: a. Losartan: 50mg cada 12 horas, b. Candesartan: 8-16mg cada 24 horas, c. Telmisartan: 40-80mg cada 24 horas. 4.
CALCIOANTAGONISTAS: a. Amlodipino: 5-10mg cada 24 horas, b. Nifedipino: 30-60mg cada 24 horas. 5. Diur cos azdicos: a. Hidroclorotiazida b.
Clortalidona. COMPLICACIONES: Encefalopata hipertensiva: Ocurre como resultado de un abrupto y reversible secundario al incremento brusco de la
PAM (160mmHg), que excede los lmites de autorregulacin de las arterias de resistencia de pequeo calibre en el cerebro hay cefalea intensa, vomito,
somnolencia, confusin, puede ocurrir convulciones, dficit neurolgicos focales, el edema de papila es frecuente, retinopata hipertensiva grave, coma.

CURSO ENARM CMN SIGLO XXI TEL: 36246001

Pharmed Solutions Institute

PGINA 155

MANUAL DE TRABAJO DEL CURSO ENARM CMN SIGLO XXI


La reduccin inmediata en un 30-40% revierte el vasoespasmo, pero la disminucin excesiva debe ser evitada para prevenir mayor isquemia cerebral,
que ocurre cuando la TA cae por debajo del lmite inferior de la autorregulacin. El rgimen estndar es nitroprusiato IV, hasta reduccin de 25% de
PAM, en una hora (buscar PAM minima de 110mmHg, tambin nitroglicerina, labetalol, nicardipina. Isquemia cerebral aguda: Caracterizado por dficit
neurolgico de ms de 24 hrs de aparicin en ausencia de alguna otra causa vascular, se manifiesta por alteraciones visuales, afasia, paresia,
hemiparesia, hemiplejia, ataxia-hemiparesia, disartria. Hemorragia intracerebral, 80% de los pacientes son hipertensos crnicos, aumenta el riesgo con
cifras elevadas de presin diastlica. Enfermedad vascular cerebral; el 85% sin isqumicos, no se recomienda disminucin sbita de la TA, la nica
excepcin es cuando esta asociada a diseccion aortica, pacientes con TA sostenida de 185/110mmHg no indicar tratamiento trombolitico. En EVC
isqumico candidato a terapia trombolitica (ventana de 3-4.5h) nicamente con alteplase, objetivo TA <180/105, para evitar una hemorragia, reducir 520%. En EVC hemorrgico TAS 200 o TAD >110mmHg, o TAM >130mmHg disminuir 25% en 6-12 hrs, un incremento en la PIC causa hipertensin refleja
(triada de Cushing; bradicardia, hipertensin y alteraciones respiratorias) debido a que hay gran liberacuion de catecolaminas, por lo que hay una
respuesta vagal que lleva a bradicardia para frenar, ruptura de arterias perforantes en nucleos basales; Tratamiento labetalol (reajusta la autorregulacin
cerebral) y nicardipino de primera lnea, otros IECA, se debe evitar el nitroprusiato por el descenso sbito, los vasodilatadores aumentan la PIC.
Hemorragia subaracnoidea, irrupcin sbita de sangre al espacio subaracnoideo craneal o espinal, ms comn en ganglios basales 40-50%, sustancia
blanca subcortical 20%, talamo 15%. Diseccin proximal de aorta; es una urgencia quirrgica, reduccin en 5-10 min, hay dolor precordial, alteracin del
pulso, soplo de insuficiencia aortica, dficit neurolgico, en Rx de trax se observa ensanchamiento mediastinico, la finalidad es la disminucin de la
fuerza de eyeccin y las resistencias vasculares perifricas, requiere tx. IV agresivo, usar un beta bloqueador mas un vasodilatador (labetalol, esmolol +
nitroprusiato hasta PAS de 100-120mmHg). IAM; esta indicada la disminucin inmediata de la TA para prevenir dao miocrdico, nitroprusiato puede
taquicardia refleja y no favorece la circulacin colateral, la nitroglicerina es la de eleccin mejora precarga y poscarga y mejora circulacin corornaria (la
ventana para trombolizar a un paciente con elevacin de ST es de 12 h ideal 6 h). EAP; incremento de TA con falla del ventrculo izquierdo, en falla
cardiaca diastlica los nitratos y diuyreticos son de primera eleccin, en falla cardiaca sistlica podemos usar diurticos, digital, dobutamina,
levosimendan. Insuficiencia renal; en crnica calcioantagonistas, diurticos, en aguda, nitroprusiato de Na, nicardipina.
CASO CLINICO HIPERTENSION ARTERIAL
Se trata de paciente femenino de 68 aos de edad, acude a consulta
por presentar cefalea global, acufenos y fosfenos, mareo, refiere
adems parestesias de miembro toracicco, adems hormigueo facial, a
la exploracin fsica se observa ansioso y levemente desorientado, TA
180/120 mmHg, FC 132 lpm, FR 41. Se enva al servicio de urgencia
donde se le administra nifedipino subligual antes de enviar a segundo
nivel.
PREGUNTA
Cual de las siguientes complicaciones por la maniobra realizada es
menos probable para el caso.
RESPUESTA
a.- Infarto al miocardio.
b.- Acidente cerebrovascular.
c.- Edema agudo pulmonar.
d.- Bloqueo cardiaco.
CASO CLINICO CRISIS HIPERTENSIVA CON DAO A ORGANO BLANCO
Se encuentra paciente de 42 aos de edad el cual se conoce hipertenso
con buen apego a tratamiento, tiene antecedentes de endarteritis de
takayasu, refiere cefalea global, con acufenos y fosfenos, refiere
amaurosis fugax y dolor retrosternal ardoroso y opresivo, se observo
soplo carotidea, disminucin de la fuerza muscular, disminucin de la
sensibilidad en de miembros, disminucin de pulsos, con hipotermia
perifrica, se observa desviasion de choque de punta, galope, sin de
edema pulmonar e imagen de ensanchamiento mediastinal. Sus
constantes vitales fueron TA 210/119 mmHg, FC 123 lpm, FR 31 rpm,
PREGUNTA
Cual de las siguientes complicaciones es la mas probable que se
encuentra en este caso?
RESPUESTA
a.- Insuficiencias cardiaca aguda.
b.- Diseccin de la aorta.
c.- Bloqueo AV de tercer grado.
d.- Encefalopatia hipertensiva
PREGUNTA
Cual es el objetivo principal del manejo de la presin arterial
considerando la complicacin con la que cursa actualmente?
RESPUESTA
a.- Disminuir la capacitancia coronaria.
b.- Disminuir el volumen de eyeccin.
c.- Disminuir el volumen circulante.
d.- Disminuir la poscarga.

CURSO ENARM CMN SIGLO XXI TEL: 36246001

CASO CLINICO CRISIS HIPERTENSIVO CON DAO A ORGANO BLANCO


Paciente masculino de 61 aos de edad hipertenso mal controlado y
diabetes mellitus, acude a urgencias por cefalea intensa, vomitos
intenso, el paciente presenta datos compatibles con delirium
caracterizado por agitacin psicomotriz, presento vomito importante,
su alteraciones neurologas no fueron focalizados y se presento
lentamente, durante la exploracin clnica presento disartria y
hemiparesia, los signos vitales TA 205/120 mmHg, cual es la
complicacin mas probable que presento el paciente?
RESPUESTA
a.- Hemorragia cerebral.
b.- Encefalopatia hipertensiva.
c.- Edema pulmonar.
d.- Meningoencefalitis.
PREGUNTA
Cual es la meta sobre la hipertensin arterial?
RESPUESTA
a.- Disminuir 25 % de la diastlica.
b.- Disminuir 50 % de la diastlica.
c.- Disminuir 25 % de la sistlica.
d.- Disminuir 50 % de la sistlica.
PREGUNTA
Cual es tratamiento de primera eleccin en este caso
RESPUESTA
a.- Betabloqueadores.
b.- Nitroglicerina
c.- Nitroprusiato.
c.- Calcioantagonistas.
PREGUNTA
Se agrego al tratamiento diuretico al manejo, el paciente presenta rahs
cutneo en alas de mariposa, cual es la causa de la presencia de este
sindrome de lupus Like?
RESPUESTA
a.- Hidroclorotiazida.
b.- Clorotiacida.
c.- Furosemide.
d.- Hidralacina.
PREGUNTA
El paciente diminuyo los sntomas durante la estancia en sala de
choque, 8 horas despus presento incremento de la tensin arterial
nuevamente sin datos de focalizacin, con datos sugestivos de

Pharmed Solutions Institute

PGINA 156

MANUAL DE TRABAJO DEL CURSO ENARM CMN SIGLO XXI


isquemia miocrdica. Cul es el tratamiento mas adecuado en este
momento?
RESPUESTA
a.- Nitroprusiato.
b.- Clonidina.
c.- Nitroglicerina.
d.- Labetolol.
CASO CLINICO
Varn de 56 aos, camionero, fumador de 40 cigarrillos al da,
antecedentes de hipertensin de 12 aos de evolucin mal controlada
y dislipidemia. Refiere cefalea frontotemporal que incremento en
intensidad, que apareci hace 2hrs acompaadas de visin borrosa,
dolor torcico, dificultad respiratoria, vomito en dos ocasiones as
como desorientacin en tiempo y persona. Exploracin fsica TA
200/130mmHg, FC 100x, FR 24x, Temp 36.5. Dficit neurolgico
PREGUNTA
Cul es la conducta teraputica ms apropiada para evitar dao
irreversible en este paciente?
RESPUESTA
a.- Nitroprusiato de Na
b.- Enalaprilat
c.- Nitroglicerina
d.- Hidralacina
PREGUNTA
Con este medicamento que porcentaje de presin buscamos
disminuir y en qu tiempo?
RESPUESTA
a.- 15% en la primera hora
b.- 25% en la primera hora
c.- 30% en las 2 primeras horas
d.- 20% en las 2 primeras horas

c.- Diuretico de asa.


d.- Dihidropiridina
PREGUNTA
Cul es el riesgo mas probable que puede presentar el paciente,
secundario al tratamiento establecido en el traslado?
RESPUESTA
a.- Evento cerebral vascular
b.- Isquemia cerebral transitoria
c.- Infarto agudo al miocardio
d.-Edema gaudo del pulmn
CASO CLINICO
Masculino de 60 aosde edad con antecedente de arteriopatia
coronaria, estado de vasculopata perifrica despus de injerto de
derivacin de arteria femoral-arteria tibial posterior, osteoartritis y un
antecedente de hipertensin de 3 aos de evolucin, acude a la clnica
para revisin sitematica de la presin arterial. No ha tenido molestias
desde su ltima revisin hace 6 meses. Los signos vitales son:
temperatura 37C, TA 160/95mmHg, pulso 7/min, frecuencia
respiratoria 10/min. La exploracin fsica es marcada por un solploo
abdominalque se ausculat a la derecha de la lnea media a nivel del
ombligo. Los valores de laboratorio son: Na 140mEq/l, K 4,1mEq/l, Cl
98 mEq/l, bicarbonato 23 mEq/l, BUN 22mg/dl, Cr 1.8 mg/dl, glucosa
110mg/dl.
PREGUNTA
Cul es el diagnostico mas probable en este caso?
RESPUESTA
a.- Sindrome de Cushing
b.- Hiperaldosteronismo
c.- Feocromocitoma
d.- Estenosis de arteria renal
PREGUNTA

PREGUNTA
Cul es la complicacin ms probable que presentara el paciente si
usamos el medicamento por mucho tiempo?
REPUESTA
a.- Vasoconstriccin a nivel cerebral
b.- Disminucin del FSC
c.- Acumulacin de calcio
d.- Acumulacin de cianuro y tiocianato
CASO CLINICO CRISIS HIPERTENSIVAS
Masculino de 56 aos de edad, inicio con cefalea, visin borrosa, con
dolor toracico opresivo, el paciente es diabtico e hipertenso con mal
apego a tratamiento, la diastlica se observo en 115, refiri
disminucin de la agudeza visual, con necrosis fribinoide, con
hemorragia de la retina, y exudados algodonosos, sus antecedentes
fueron tabaquismo, anemia hemoltica, con cardiomegalia sin datos de
edema pulmonar. Uremia, Anteriormente presento hemorragia
cerebral hace 2 aos.
PREGUNTA
El paciente recibi previamente al traslado recibi dihidropiridina
sublingual, cual es el tratamiento mas adecuado en este caso?
RESPUESTA
a.- Nitroprusiato de sodio.
b.- Nitroglicerina.

CURSO ENARM CMN SIGLO XXI TEL: 36246001

Cul es la causa mas probable del diagnostico del caso?


RESPUESTA
a.- Enfermedad ateroesclertica
b.- Displasia fibromuscular
c.- Alteracion Congenita
d.- Tumoracin
PREGUNTA
Cul es la conducnta teraputica mas adecuada para este caso?
RESPUESTA
a.- Prazosina
b.- Angioplastia coronaria con colocacin de endoprotesis
c.- Disminuir esteroides gradualmente
d.- Espironolactona
PREGUNTA
Cul es la complicacin menos probable en este paciente a
consecuencia de la hipertensin?
RESPUESTA
a.- Enfermedad renal crnica
b.- Isquenia cerebral transitoria
c.- Insuficiencia heptica
d.- Insuficiencia cardiaca

Pharmed Solutions Institute

PGINA 157

MANUAL DE TRABAJO DEL CURSO ENARM CMN SIGLO XXI


HIPERTENSIN MALIGNA:
CIENCIAS BASICAS: Es una hipertensin arterial rpidamente progresiva caracterizada, desde el punto de vista anatomopatolgico, por lesiones
arteriolares graves y clnicamente por la existencia de presin arterial muy elevada severa, hemorragias y exudados retinianos (retinopata) y, a menudo
aunque no necesariamente, edema de papila (papiledema). TAD >140mmHg. La encefalopata hipertensiva es la consecuencia de un aumento sbito y
generalmente intenso de la PA que cursa con intensa cefalea y alteraciones variables del nivel de conciencia y que es reversible al reducir la presion.
Puede acompaar una hipertensin maligna, pero tambin puede aparecer sin sta. Se presenta con ms frecuencia en individuos previamente
normotensos (embarazadas con eclampsia, jvenes) que en hipertensos crnicos cuyas cifras de PA aumentan progresivamente. SALUD PBLICA: Si la
hipertensin arterial afecta alrededor de la quinta parte de la poblacin general, la hipertensin maligna apenas alcanza el 1 % de los hipertensos. Se
asocia a tabaquismo intenso. El 80 % de las hipertensiones malignas corresponden a una nefropata o patologa vascular renal. PATOGENIA: Rapida
instauracion. Las lesiones vasculares que se producen conllevan consecuencias isqumicas tanto a nivel de rgano (rin) como a nivel de la propia pared
vascular, establecindose un crculo vicioso entre isquemia y PA a ambos niveles. Al 75 % de los pacientes con hipertensin maligna se les detecta una
actividad renina plasmtica elevada, con el consiguiente aumento de la secrecin de aldosterona. La hipersecrecin de renina secundaria a los cambios
vasculares renales producidos por la elevacin intensa de la PA es el mecanismo que sostiene el crculo vicioso al producir, a travs de la angiotensina ll,
una ulterior vasoconstriccin renal con ms isquemia y ms secrecin de renina. Adems en este momento se produce una natriuresis de presin
(intensa), con la consiguiente reduccin del volumen intravascular, que puede a su vez tambin estimular la secrecin de renina y la actividad del sistema
nervioso simptico. Las lesiones arteriolares de la hipertensin maligna afectan a muchos rganos: encfalo, pncreas, corazn, intestino, bazo y rin.
En el rin las lesiones ms caractersticas son la necrosis fibrinoide de las pequeas arteriolas, por deposito de protenas plasmticas en el subendotelio
y la endarteritis proliferativa (hiperplasia o proliferacin miointimal) se caracteriza por un engrosamiento de la ntima, constituido por clulas musculares
lisas, restos de membrana basal y mucopolisacridos que conlleva un importante estrechamiento de la luz vascular con el consiguiente aumento de la
resistencia al flujo de sangre e isquemia parenquimatosa. La lesin ms frecuentemente observada en el rin humano es una arteriosclerosis
hiperplsica y una esclerosis glomerular isqumica, y la tpica hiperplasia miointimal se observa con relativa frecuencia en el rin de individuos de raza
negra. DIAGNOSTICO: Sntomas diversos como malestar general, cefaleas, visin borrosa, confusin, somnolencia, nuseas, vmitos, disnea, hematuria y
oliguria (sntomas de uremia). La cefalea es, sin duda, el sntoma ms frecuente y precoz, y no es raro que preceda al resto de la sintomatologa unos seis
meses. En el 75 % de los casos el cuadro se acompaa de prdida de peso, como resultado de la isquemia del msculo esqueltico y la prdida excesiva
de sal y agua por parte del rin que se observa en las fases iniciales de la hipertensin maligna. Acostumbra a presentarse en individuos jvenes o de
mediana edad, y slo excepcionalmente es una forma de presentacin de una hipertensin en edades avanzadas. En una Rx de torax, podemos observar
cardiomegalia y signos de insuficiencia cardiaca. COMPLICACIONES: 1. Encefalopata hipertensiva: alteraciones neurolgicas, normalmente transitorias y
fluctuantes, reversibles al controlar la PA, que se manifiesta en forma de cefalea intensa, confusin mental, disfagia, alteraciones visuales y convulsiones.
Puede tambin manifestar alguna focalidad (hemiparesia), en cuyo caso la posibilidad de una hemorragia intracraneal debe ser considerada,
patolgicamente existe edema cerebral ms o menos generalizado. El descenso brusco de la PA durante el tratamiento en estos pacientes con
hipertensin maligna puede producir isquemia cerebral por hipoperfusin, si se alcanzan niveles por debajo de una PA media de 100-120 mmHg pues
por debajo de estas cifras el paciente es incapaz de autorregular su flujo cerebral. 2. Insuficiencia renal: que recibe el nombre de nefroesclerosis
maligna, puede presentarse clnicamente como una insuficiencia renal progresiva o como una insuficiencia renal aguda, fases iniciales estos enfermos
pueden presentar poliuria y polidipsia, con hiponatremia e hipopotasemia. Dada por la natriuresis de presin y el hiperreninismo e
hiperaldosteronismo secundario a la isquemia renal, la hipertensin maligna puede clnicamente presentarse como un sndrome nefrtico agudo. 3.
Anemia hemoltica microangioptica: es la traduccin de un dao endotelial generalizada. Verdadera retinopata hipertensiva (grado lll-IV). En el fondo
del ojo grado lll el calibre arterial es muy inferior al venoso (espasmo arterial generalizado), con espasmos focales, apariencia de hilo de plata en
cuanto al reflejo luminoso, con desplazamiento venoso, cruces en ngulo recto y dilataciones venosas distales. Tpicamente aparecen hemorragias y
exudados. PRONSTICO: La principal causa de muerte era la hemorragia cerebral o la insuficiencia cardaca. Con un tratamiento inmediato y eficaz es
posible lograr una supervivencia superior al 70 % a los cinco aos, con una remisin, generalmente parcial, del cuadro tanto clnico como biolgico.
TRATAMIENTO: El objetivo del tratamiento es bajar la PA para evitar la aparicin o agravacin de la repercusin orgnica sin llegar, durante las primeras
24 horas, a niveles de normotensin. 1ra eleccin nitroprusiato de Na, enlaprilat, calcioantagonista (cuando haya falla renal o niveles de Cr >1.5, reducir
TA el 25% en la primera hora, en las 2 a 6 hrs no menor a 160/100mmHg, monitorizacin invasiva de TA.

CASO CLINICO
Se trata de un paciente varn de 31 aos, fumador de 20 cigarrillos al
dia, desde los 17 aos de edad, hipertenso descontrolado, es remitido
por una prdida brusca de visin en OD de unas 12 horas de evolucin.
No refiere antecedentes de inters. En la exploracin oftalmolgica se
objetiv una agudeza visual (AV) de movimiento de manos en OD y de
1,2 en OI. En el examen funduscpico se encontraron en ambos ojos
mltiples exudados algodonosos, hemorragias retinianas en llama y
borramiento parcial de la papila. La visin de movimiento de manos en
OD se justificaba por la importante afectacin macular, con exudados
algodonosos y hemorragias a este nivel. EF: 230/140mmHg, FC 110x,
FR21x, temp 36.8C, no datos de encefalopata. Cr 5 mg/dl,
proteinuria +++.
PREGUNTA
Cul sera la principal sospecha diagnstica en este caso?
RESPUESTA
a.- Hipertensin maligna
b.- Emergencia hipertensiva
c.- Insuficiencia renal aguda
d.- Urgencia hipertensiva

CURSO ENARM CMN SIGLO XXI TEL: 36246001

PREGUNTA
Cul es la conducta teraputica ms adecuada para este paciente?
RESPUESTA
a.- Nitroprusiato de Na, reducir TA 25% en la primera hora en las 2 a 6
hrs no menor a 160/100mmHg
b.- Calcioantagonista, reducir TA 25% en la primera hora en las 2 a 6
hrs no menor a 160/100mmHg
c.- Nitroglicerina, reducir TA 25% en la primera hora en las 2 a 6 hrs no
menor a 160/100mmHg
d.- Esmolol, reducir TA 25% en la primera hora en las 2 a 6 hrs no
menor a 160/100mmHg
PREGUNTA
Si el paciente no presentara dao renal Cul sera el medicamento de
primera eleccin?
RESPUESTA
a.- Nitroprusiato de Na
b.- Calcioantagonista
c.- Nitroglicerina
d.- Esmolol

Pharmed Solutions Institute

PGINA 158

MANUAL DE TRABAJO DEL CURSO ENARM CMN SIGLO XXI


TAPONAMIENTO CARDIACO:
CIENCIAS BSICAS: Urgencia vital, es la compresin del corazn que resulta de la acumulacin de lquido, coagulos, pus o sangre en el saco pericrdico y
que produce un severo trastorno hemodinmico, como resultado de derrame, traumatismo o rotura cardiaca. La compresin cardaca lleva a una
disminucin del gasto cardaco y de la presin arterial en un espectro variable en el que el paciente severamente comprometido puede estar en shock
cardiognico. Liquido pericrdico normal aprox. de 15-50ml, efecto lubrica, equilibrio de fuerza gravitacional e hidrosttica, inmunolgico, fibrinolitico. El
pericardio por su rigidez influye fundamentalmente en la funcin diastlica del corazn, limita la funcin del llenado del corazn, la presion
intrapericardica (-5 a +5mmHg) es habitualmente negativa, aunque tambin hay una presin
CAUSAS MDICAS DE TAPONAMIENTO CARDACO
transmural. CLASIFICACION: Agudo; acumulacin repentina de liquidos 200ml. Cronico; acumulacin
Taponamiento cardaco agudo
Hemopericardio
de lquido progresivo, bien tolerado de 1-2litros PATOGENIA: Shock obstructivo, que genera falla en el
Ruptura de la pared libre del ventrculo postinfarto
llenado diastlico, producto de la compresin del corazn y las estructuras circundantes, pierden
de miocardio (mortalidad 90%)
distensibilidad y el llenado de la bomba es inadecuado. En condiciones normales la presin venosa
Aneurisma artico roto a cavidad pericrdica
perifrica es superior a la presin venosa central y a su vez sta es mayor que la presin
Traumatico 1ra causa (arma de fuego, arma blanca,
intrapericrdica en 5 Torr. Esta diferencia de presiones permite y favorece el flujo sanguneo continuo
trauma de torax cerrado)
desde el sistema venoso hacia el corazn derecho. Al acumularse lquido en el espacio pericrdico se
Diseccin artica al pericardio
eleva la PVC y disminuye el gradiente entre la presin de la aurcula derecha y el sistema venoso
Neoplasias (CA metastasico de pulmn y seno,
linfoma, leucemias, mesotelioma)
perifrico; cuando la presin intrapericrdica llega a 10 Torr se iguala a las presiones venosa central y
Tratamiento anticoagulante
venosa perifrica, y de este punto en adelante, si se eleva la presin intrapericrdica, las otras dos
Postoperatorio
ciruga
cardaca
(fugas,
tambin. La consecuencia de esta igualdad en las presiones es que el ventrculo derecho pierde la
anicoagulacion, aneurismas, disecciones)
presin efectiva de distensin y se colapsa, tanto por accin directa del lquido intrapercrdico como
Pericarditis aguda o cronica
por la prdida de flujo desde la aurcula derecha. En estas condiciones, el flujo hacia la circulacin
Bacteriana
pulmonar, as como el volumen diastlico que llega al ventrculo izquierdo, disminuyen y se reduce el
Tuberculosa
gasto cardaco, lo que lleva a isquemia cardiaca,. En esta situacin los mecanismos homeostticos para
Urmica
Enfermedades del tejido conectivo (LES, AR, fiebre
tratar de mantener la presin de perfusin de los rganos vitales son tres: la utilizacin del volumen
reumtica, poliarteritis)
residual diastlico en el corazn, el aumento de la frecuencia cardaca y la vasoconstriccin arteriolar a
Taponamiento cardaco crnico
nivel del msculo estriado y el rin, aumento de resistencias perifricas, aumento de inotropismo.
Causas infecciosas. Tuberculosis. Algunas parasitosis.
Cuando se aade lquido al espacio pericrdico la presin aumenta en forma lenta, mientras la
Causa no infecciosa. Urmico. Neoplsica,
membrana pericrdica permanece distensible, pero una vez que se encuentra a tensin la presin
posirradiacion Mixedematoso. Sndrome de Dressler.
aumenta en forma rpida. Cuando la acumulacin de lquido es aguda, la capacidad de distensibilidad
Sndrome postpericardiectoma. Quilopericardio
pericrdica (por el tejido conectivo denso) es muy baja, cuando la acumulacin de lquido es lenta,
como ocurre en los procesos crnicos, el pericardio se va adaptando y aumenta en forma progresiva su distensibilidad, por lo cual recibe mayores
cantidades. Las presiones existentes en el corazn derecho son mucho mas bajas que las del corazn izquierdo, por lo que cualquier aumento en la
presin dentro del compartimento pericrdico, afectara antes y en mayor grado a las cavidades derechas que a las cavidades izquierdas
(interdependencia biventricular), solo se afectan en condiciones de sobrecarga o expansion de volumen, lo que da como resultado el pulso parodojico.
DIAGNOSTICO: Ingurgitacion yugular severa, la cual disminuye en forma visible durante la sstole, los ruidos cardacos disminuyen y llegan en ocasiones
a no ser audibles, latido apical palpable, disminucin de gasto cardiaco (frialdad, debilidad muscular y diaforesis) y congestion venosa, aumento de PVC.
La presin arterial se encuentra generalmente disminuida y convergente (presin arterial diferencial menor de 30 mmHg) y el paciente est taquicrdico.
El pulso paradjico es un signo casi constante, lo mismo que el signo de Kussmaul. Disnea, congestion heptica, dolor sordo o constrictivo, oliguria.
Triada de Beck: hipotensin, elvacion de PVC, ruidos cardiacos velados. La Rx., del trax puede haber cardiomegalia, evidencia de derrame pericrdico y
procesos patolgicos primarios que son causa de tapnamiento, como neoplasia o tuberculosis. El ECG, taquicardia sinusal, bajo voltaje y anomalas
inespecficas de la repolarizacin ventricular como la elevacin cncava del segmento ST, depresin de PR, (signos de pericarditis) y la alternancia
elctrica, la cual indica la presencia de derrame pericrdico, pero no es concluyente. El ecocardiograma es el mtodo diagnstico que ms informacin
aporta. En primer lugar, detecta la presencia de derrame pericrdico y permite hacer una aproximacin de su cantidad movimiento de tabique IV hacia la
izquierda en inspiracion. La manera ms simple de documentar el taponamiento cardaco es por medio de un catter colocado en la aurcula derecha,
con el cual se pone en evidencia el aumento de la PVC y se puede obtener una curva que revele los cambios antes mencionados en el pulso venoso. Con
el catter de Swan-Ganz se pueden obtener, adems, curvas de presin de la arteria pulmonar, mediciones de la presin pulmonar en cua y una
estimacin del gasto cardaco por termodilucin. TRATAMIENTO: pericardiocentesis; alivio de emergencia, drenaje de derrame pericrdico. Se prefiere la
ciruga en los casos de hemopericardio en que se desea evitar la repeticin de hemorragias y est indicada en la pericarditis purulenta, con el fin de
realizar drenaje completo y controlar la infeccin. La pericardiocentesis con gua ecocardiogrca u otro po de imagen, como la fluoroscopia o la TC, es
buena alternativa en prcticamente todas las enfermedades pericrdicas. No debe olvidarse la probabilidad de recurrencia del taponamiento cardaco
cuando se trata con pericardiocentesis, por lo cual deber repetirse el procedimiento o hacer ciruga. Son de utilidad medidas teraputicas de sostn
como los lquidos intravenosos, en especial en casos de hemopericardio traumtico agudo, cuando el paciente est hipovolmico. Esta medida aumenta
el volumen circulante, incrementa las PVC, pulmonar y la presin arterial. En ningn caso deben aplicarse vasodilatadores, como el nitroprusiato de sodio
por va intravenosa. La pericardiocentesis tiene ventajas como su rpida aplicabilidad, la facilidad de estudiar el lquido, la factibilidad de combinarla con
estudios hemodinmicos y su eficacia en las 2/3 partes de los casos. Complicaciones; puncion ventricular o auricular, arritmias, hemoperitoneo,
perforacin de estomago o colon, puncion de arteria coronaria. Se aconseja como premedicacin a este procedimiento la administracin de atropina (0,8
a 1,0 mg) por va intravenosa o intramuscular, a fin de prevenir reacciones vasovagales. Previa asepsia con alcohol yodado y bajo anestesia local (en los
casos urgentes se omite la anestesia) se efecta la puncin 5 cm por debajo de la punta del apndice xifoides y 1 cm a la izquierda de la lnea media, con
la aguja en ngulo de 45 grados y dirigida hacia el hombro izquierdo. Se ejerce succin continua mientras se introduce la aguja, detenindose cuando se
perciba una sensacin de vencimiento de una resistencia lo cual indica la entrada al espacio pericrdico, hasta que se obtenga lquido o cuando se
sientan las pulsaciones cardacas transmitidas a la aguja, lo que significa que se ha tocado el miocardio y se debe retirar un poco la jeringa. En
condiciones electivas puede emplearse el electrocardiograma para evitar lesiones del miocardio. La sangre obtenida no coagula porque los movimientos
del corazn la desfibrinan rpidamente; si la sangre obtenida se coagula es porque se puncion una cavidad del corazn. La dobutamina, administrada
para revertir la hipotensin, es tericamente ideal.
CASOS CLINICOS
Mujer de 81 aos, sin antecedentes de inters, que ingres con el
diagnstico de insuficiencia cardiaca desencadenado por infeccin
respiratoria, con un cuadro inespecfico de 1 mes de evolucin, de

CURSO ENARM CMN SIGLO XXI TEL: 36246001

malestar y astenia, y los das previos al ingreso haba aparecido disnea


de esfuerzo, que llegaba a ser de reposo. Refera tos escasa, sin
expectoracin y sin fiebre. Laboratorios: hemoglobina, 11,9 g/ dl, con
hematocrito, 36,8%, y leucocitosis, con 16,8. 705,109 plaquetas/l, y

Pharmed Solutions Institute

PGINA 159

MANUAL DE TRABAJO DEL CURSO ENARM CMN SIGLO XXI


fibringeno, 464 mg/dl. La radiografa de trax mostraba
cardiomegalia con ligera redistribucin en ambas bases. En el ECG,
reduccin en el voltaje del QRS y un aplanamiento difuso de las ondas
T. A las 24 h present un deterioro clnico con datos clnicos y
hemodinmicos, hipotensin, pulso paradjico y datos de presin
venosa elevada.
PREGUNTA
Cul es la conducta a seguir mas apropiada?
RESPUESTA
a.- Pericardiocentesis.
b.- Pericardiectomia urgente.
c.- Manejo conservador.
d.- Correccin del estado hemodinamico.
PREGUNTA
Cul seria la premedicacin mas adecuada para llevar a cabo este
procedimiento?
RESPUESTA
a.- Morfina por via intravenosa
b.- Atropina por via intravenosa
c.- Lidocaina local
d.- Antibiotico intravenoso
CASO CLINICO
Mujer de 69 aos de edad que sufre un trauma torcico cerrado por
compresin antero posterior al quedar atrapada por las puertas de un
autobs y 30 das despus comienza a presentar falta de aire a los
esfuerzos, aumentando progresivamente hasta desencadenarse a los
pequeos esfuerzos. Ingresa en el hospital con diagnstico de
cardiopata isqumica. Se realiza ecocardiograma y se comprueba gran
coleccin lquida pericrdica que se evacua parcialmente mediante
pericardiocentesis, pero al profundizarse los sntomas de hipovolemia
se decide realizar pericardiectoma anterior radical de urgencia a
travs de una toracotoma antero lateral izquierda y se constata el
pericardio engrosado y tenso. Se comprueba el diagnstico por
puncin pericrdica transoperatoria por la presencia de sangre y se
procede a descomprimir lentamente la coleccin intrapericrdica. Se
completa la pericardiectoma y se diagnostica una gran contusin
miocrdica con movimiento cardiaco lento. La evolucin
temporalmente
es
satisfactoria
con
estabilizacin
del
electrocardiograma aunque tres meses despus sufre infarto del
miocardio y fallece.
PREGUNTA
Cual es el porcentaje de pacientes con aplastamiento torcica
desarrollan tamponade cardiaco?
RESPUESTA
a.- 20%.
b.- 30 %.
c.- 40 %.
d.- 50 %.
PREGUNTA
Cul es el dato clnico menos probable de observar en este caso?
RESPUESTA
a.- Ruidos cardiacos disminuidos
b.- Hipotension
c.- Signos de Kussmaul
d.- Pulso paradjico
CASO CLINICO

CURSO ENARM CMN SIGLO XXI TEL: 36246001

Masculino de 23 aos de edad es llevado en ambulanciaa la sala de


uregencias tras haber sido apualado en el trax durante una ria. En
el interrogatorio inicial revela que no es alrgico, no toma
medicamentos y que comio unas tres horas antes. Se queja de dolor
intesno en el trax y de dificultad para respirar. Los signos vitales son:
T 37C, TA 85/50mmHg durante la espiracin y 60/palpable durante la
inspiracin, pulso de 120/min, frecuencia respiratoria de 20/min y
saturacin de oxigeno de 96% en aire ambiente. La exploracin fsica
muestra masculino bien desarrollado y bien nutrido, con ropa
desgarrada y manchada con sangre y sin datos de otros traumatismos
penetrantes o no, incluso traumatismo craneoenceflico. El examen de
cuello muestra presin venosa yugular de alrededor de 15cm H2O, la
traquea esta en la lnea media, no hay crepitacin subcutnea, ni
heridas obvias en el cuello. La auscultacin de torax revela campos
pulmonares limpios y bien ventilados. Se observa una herida lineal de
2 cm , aproximadamente a 2 cm a la izquierda del esternona nivel del
pezn. Los ruidos cradiacos son distantes y existe taquicardia, pero el
ritmo cardiaco es regular, resto de exploracin sin alteraciones. La
radiografi de trax muestra campos pulmonares limpios, silueta
cardiaca moderadamente agrandada.
PREGUNTA
Cul es el diagnostico mas probable en este caso?
RESPUESTA
a.- Diseccin de aortca aguda
b.- Taponamiento cardiaco
c.- Neumotrax a tensin
d.- Infarto agudo de miocardio
PREGUNTA
Se le realiza un electrocardiograma al paciente Qu es lo mas
probable de observar en este estudio?
RESPUESTA
a.- Supradesnivel del segmento ST
b.- Extrasistoles ventriculares
c.- Disminucion global del voltaje
d.- Onda p mitral
PREGUNTA
Cul es la conducta diagnostica que nos aportara mas datos para
confirmar el diagnostico?
RESPUESTA
a.- Ecocardiograma
b.- Tomografia
c.- Resonancia
d.- Ultrasonografia FAST
PREGUNTA
Cul es la conducta teraputica inmediata mas adecuada a seguir?
RESPUESTA
a.- Administracion de liquidos
b.- Pericardiocentesis
c.- Toracostoma
d.- Manejo conservador
PREGUNTA
Cul es la complicacin ms probable tras la realiozacion del
procedimiento?
RESPUESTA
a.- Puncin ventricular o auricular
b.- Arritmias
c.- Hemoperitoneo
d.- Puncin de arteria coronaria

Pharmed Solutions Institute

PGINA 160

MANUAL DE TRABAJO DEL CURSO ENARM CMN SIGLO XXI


DISECCION DE AORTA:
CIENCIAS BASICAS: Consiste en la separacin de la capa media de la pared aortica en la que penetra sangre
PACIENTES DE ALTO RIESGO DE
precedente de la aorta con lo que se establece una falsa luz que puede comprimir la luz verdadera del vaso. Entre
DISECCION AORTICA
Enfermedad vascular ateroesclertica
ambas encontramos el colgajo intimal. Esta separacin es consecuencia de una rotura o perforacin en la intima
Anuloectasia artica
(puerta de entrada) que se propaga distalmente. La adventicia puede contener inicialmente el sangrado o
Aneurisma artico
evolucionar a la rotura, Alrededor de 95% de roturas ocurren en la aorta ascendente, distal a la vlvula aortica.
Ulcera aortica
Dependiendo de la localizacin y la extensin pueden aparecer un taponamiento cardiaco, hemotorax, una
Calcificacin de la intima
Alteraciones genticas (conectivopatias)
insuficiencia aortica o un sndrome de mala perfusin. Este ultimo segn los troncos arteriales afectados, se puede
Sndrome de Marfan
manifestar como un sndrome coronario agudo, una afeccin neurolgica o visceral (por afeccin de troncos
Sndrome de Ehlers-Danlos
supraarticos, arteriales medulares y ramas viscerales), o una isquemia de los miembros inferiores. Los factores
Sndrome de Turner
predisponentes mas frecuentes son: hipertensin (72%) y ateroesclerosis (31%). En pacientes menores de 60 aos
Enfermedad de Noonan
Osteognesis imperfecta
encontramos generalmente un aneurisma de aorta, una ciruga cardiaca previa, un sndrome de Marfan o una
Enfermedades congnitas
vlvula aortica bicspide. DIAGNOSTICO: Clnico: el dolor es la caracterstica ms importante, es una sensacin de
Coartacin aortica
desgarro agudo e intenso, migratorio. La intensidad del dolor es constante, por lo que se diferencia del dolor
Valvula artica unicspide o bicspide
secundario a IAM. Un dolor retroesternal con irradiacin a cuello es caracterstico de la afeccin de la aorta
Enfermedades degenerativas
Hipertensin
ascendente, mientras que el dolor dorsolumbar orienta hacia una afeccin de la aorta torcica descendente. El
Causas traumticas
sincope puede estar causado por dolor intenso, la obstruccin de los vasos cerebrales, la activacin de
Lesin por desaceleracin
barorreceptores articos o un taponamiento cardiaco. En funcin de los vasos afectados habr manifestaciones de
Lesin penetrante
mala perfusin. Una diseccin origina una obstruccin dinmica de los troncos supraarticos, que se puede
Enfermedades inflamatorias
manifestar con un cuadro de isquemia cerebral y asimetra o ausencia de pulsos en extremidades superiores. El
Sfilis
Arteritis de clulas gigantes
desgarro de las arterias intercostales se puede mostrar como un dficit motor en los miembros inferiores. El
Iatrogenia
compromiso de las arterias viscerales con dolor abdominal (isquemia medular) o con aligoanuria (arterias renales).
Cateterismo cardiaco
Los pulsos femorales pueden estar disminuidos o ser asimtricos. Otras manifestaciones menos frecuentes son
Canulacin artica o femoral previa
Baln de contrapulsacin intraartico
hemoptisis, sndrome de vena cava superior, obstruccin de la va area y sndrome de Horner (compresin por el
aneurisma o hematoma en el cayado), disfona. La ausencia de pulsos, presencia de sincope previo, signos
neurolgicos, estado de Shock y hemotorax se consideran factores de mal pronstico. El diagnostico de diseccin aortica aguda debe considerarse en
todo paciente con un cuadro de dolor abdominal agudo, isquemia de las extremidades, en caso de sincope o un cuadro de insuficiencia cardiaca aguda
no explicada. Recientemente se ha publicado el score de Diseccin Aortica en Urgencias, valora la presencia de dolor en espalada, una relacin toracomediastinica mayor a 30%, una insuficiencia aortica y un dimetro artico mayor de 30mm por ecografa. La sensibilidad y especificidad de este score es
de 93 y 77% respectivamente, si estn presentes 3 o ms de estos signos. Si se sospecha una diseccin aortica y no hay disponibilidad diagnostica ni de
tratamiento, debe asegurarse la remisin rpida del paciente. Rx de trax; suele mostrar ensanchamiento mediastinico o derrame pleural asociado. ECG:
es normal en 30%, cambios isqumicos, signos de hipertrofia VI. Analtica sangunea: anemia ligera, leucocitosis, trombopenia, alteracin de pruebas de
funcin heptica, elevacin de Cr srica. Elevacin de reactantes de fase aguda. Acidosis
metablica en casos de mala perfusin. Especficos: Ecocardiografa transtorcica (ETT), identifica
diseccin aortica proximal, una insuficiencia aortica asociada y un taponamiento cardiaco,
permite evaluar la funcin miocrdica y medir los dimetros de la raz aortica. Ecocardiografa
transesofgica (ETE), mas sensible y especifica, confirma diseccin en un 90% de los casos y
permite localizar la rotura intimal en la mayora de los casos y valorar el flujo sanguneo en la
verdadera y falsa luz. TAC, tiene sensibilidad superior al 95% y una especificidad del 87%, da
informacin precisa de los diferentes dimetros de los segmentos articos. Permite evaluar, con
precisin la extensin de la aorta afectada y mostrar la implicacin de las arterias viscerales e
iliacas. Tiene como inconveniente la nefrotoxicidad, el contraste y la necesidad de desplazar al
paciente. TRATAMIENTO: Actuacin inmediata: La afectacin de la aorta ascendente requiere
ciruga, mientras que la diseccin de la aorta descendente precisa abordaje mdico en ausencia
de complicaciones. Farmacolgico: Antes de la realizacin de cualquier medida diagnosticoteraputica el paciente debe estar monitorizado con ECG, medida de presin arterial (cada 5min)
y canalizacin de una va venosa de buen calibre. La analgesia contribuye a estabilizar al paciente
y en, general se da opiceos a dosis bajas (cloruro mrfico 3 mg IV cada 5-10 min), para evitar la
progresin de la enfermedad y el riesgo de rotura aortica, se debe disminuir el estrs parietal
artico y controlar la presin arterial (objetivo TAS entre 90-110 mmHg, si se mantiene la
diuresis). Los frmacos ms usados como tratamiento antiimpulso, son los bloqueadores
selectivos como: de vida corta esmolol IV carga 0.5mg/kg en 2-5 min, mantenimiento 0,10-0,20 mg/kg/min, vida intermedia atenolol IV, dosis carga 2.5
mg, mantenimiento 0,15 mg/kg/dia. No selectivos como: propanolol dosis de carga 0.5-1 mg en 5 min, mantenimiento 0.05-0,015 mg/kg cada 4-6 hrs y
los bloqueadores 12 (labetalol IV de vida media larga en dosis crecientes 0.5-4 mg/min). En caso de hipertensin arterial se puede iniciar labetalol de
primera eleccin. Si se requiere de un control adicional de la presin arterial se suelen asociar nitroprusiato, nitratos, agonistas de calcio (nimodipino IV)
o incluso IECAS IV, Si los betas bloqueadores estn contraindicados (EPOC) hay que pensar en verapamilo o diltiazem. El tratamiento farmacolgico
crnico se reserva para los pacientes ya intervenidos con diseccin tipo B no complicada y requiere un control estricto de la presin arterial,
mantenindola por debajo de 130-135/80mmHg. El pilar del tratamiento son los bloqueadores beta, asociando IECA o antagonista de calcio, si se precisa
o el paciente no tolera lo bloqueadores beta. Se debe realizar revisiones peridicas y pruebas de imagen, se recomiendan a los 1, 3, 6, y 12 meses y
posteriormente anuales. Tratamiento quirrgico: Objetivo salvar la vida del paciente previniendo la rotura de la aorta torcica ascendente en el
pericardio. El tratamiento de la diseccin aortica tipo A es quirrgico (I B), excepto en situaciones de edad avanzada, comorbilidad importante o dao
neurolgico. La diseccin tipo B tiene un mejor pronostico. Su abordaje inicial es mdico y se reserva el tratamiento quirrgico en la fase aguda a las
disecciones complicadas. TIPO A: Reemplazar la aorta ascendente disecada, con un injerto sinttico de dacrn, la zona resecada debe incluir la zona con
la rotura intimal. La mortalidad quirrgica vara entre un 10-20% en funcin de la edad, la comorbilidad y la extensin de la diseccin. TIPO B: El
tratamiento mdico tienen buenos resultados, este se centra inicialmente en el control de la presin arterial mediante bloqueadores beta y
vasodilatadores, y el alivio del dolor. Una vez superada la fase aguda, se continan los bloqueadores beta orales y vasodilatadores. Un seguimiento
radiolgico cada 6 meses ser necesario para detectar precozmente una evolucin trpida. La ciruga queda limitada a: dolor torcico persistente o
recurrente, expansin aortica, hematoma periartico, hematoma mediastnico. En la actualidad, las intervenciones endovasculares en la diseccin

CURSO ENARM CMN SIGLO XXI TEL: 36246001

Pharmed Solutions Institute

PGINA 161

MANUAL DE TRABAJO DEL CURSO ENARM CMN SIGLO XXI


aortica aguda tipo B suelen limitarse a aliviar las complicaciones con riesgo de muerte. ANEURISMA AORTICO (AA): CIENCIAS BASICAS: Se definen
como la dilatacin focal de la arteria que supone un aumento de ms de 50% del dimetro esperado, basado en medidas medias obtenidas en estudios
con TAC en poblacin general. En el caso de la aorta abdominal correspondera a un dimetro superior a 3 cm. Tambin se habla de AA como una
dilatacin localizada de por lo menos 1.5 veces mayor al dimetro normal de la arteria; puede ser sacular o fusiforme, y ambas estn adyacentes a un
segmento de arteria normal. Esta patologa se ha convertido en el motor del tratamiento quirrgico preventivo de la ciruga vascular. Decimos que existe
un aneurisma verdadero cuando afecta a las tres capas histolgicas; pero cuando la ntima y la media estn rotas y la dilatacin es a expensas solo de la
adventicia, entonces hablamos de seudoaneurisma. La aorta abdominal es el vaso donde se localizan con ms frecuencia los aneurismas arteriales. Hay
aneurismas articos abdominales (AAA) y aneurismas articos de torcica descendente (AAT). SALUD PBLICA: De los aneurismas articos 80% se ubican
en el abdomen y tienen una presencia en la poblacin de la tercera edad de 2 a 7%. Es mayor en hombres que en mujeres, en una proporcin de 4:1. Los
factores de riesgo, hipertensin arterial, tendencia familiar (se relaciona con cromosoma , autosmico, predisposicin gentica), tabaquismo, diabetes,
infecciones, dislipidemia y los relacionados con la ateroesclerosis, aumentan su prevalencia significativamente. El riesgo proporcional para las personas
que tienen un familiar de primer grado con AAA, es de 6:1. Como en un elevado porcentaje son asintomticos, y la ruptura es la primera manifestacin
clnica, la mortalidad real es muy alta. La probabilidad de ruptura al ao del AAA menor de 5 cm es de 0.4-5.4%, entre 5-6cmde 20-25%, y de >7cm entre
40-80%. PATOGENIA: El dimetro mximo en adultos corresponde en su origen a 3 cm, disminuyendo caudalmente, siendo de 2.5 cm a nivel de la
porcin de aorta descendente torcica, hasta 1.8 a 2 cm en la porcin abdominal de la misma. La pared vascular est formada por la n ma delgada,
compuesta de endotelio, capa subendotelial de tejido conjuntivo y una capa interna elstica; la tnica media, de clulas musculares lisas y matriz
extracelular y una adventicia formada fundamentalmente por tejido conectivo, que engloba los vasa vasorum y la inervacin del vaso. Debido a su
exposicin contina a la presin pulstil y fuerzas de sujecin, est particularmente expuesta a sufrir lesiones secundarias a trauma mecnico, sobre todo
en los casos de desaceleracin, siendo mayor el riesgo de ruptura y de aparicin de dilataciones aneurismticas. Desde el punto de la fsica, hay dos
factores que generan los AA: la excesiva aplicacin de una fuerza interna y la inadecuada resistencia del material. Se piensa que el origen de los AAA y
AAT inespecficos, es una disminucin en la cantidad de elastina de la pared arterial, normal es de 12%, porcentaje que baja a 1% o menos en los
pacientes con aneurismas detectado a travs de un marcador gentico; el aumento de la actividad de la elastasa y de la colagenasa afectan al desarrollo
del aneurisma. Los AAA y AAT principalmente, se deben a atero y arteriosclerosis; en menor proporcin trastornos del tejido conjuntivo, traumatismos,
sndrome de Marfan, infeccin, necrosis qustica de la media y arteritis. Tambin pueden ser por fibrodisplasias o iatrognicos. El AAA inflamatorio, que
representa 10% de todos los aneurismas articos, puede deberse a una infeccin o a alguna otra forma oscura de arteritis. La evolucin natural de los
aneurismas no tratados es hacia la expansin y la ruptura siguiendo la Ley de Laplace. CLASIFICACION: En los AAT (Crawford), tiene en cuenta la longitud
de la aorta afectada, resultando en cuatro patrones que determinan la extensin de la intervencin quirrgica, el resultado del tratamiento y la
naturaleza de las complicaciones. Tipo I; est afectada la mayor parte de la aorta descendente torcica y la parte proximal de aorta abdominal. Tipo II; el
aneurisma afecta gran parte de la aorta descendente y la mayor parte o toda la aorta abdominal. Tipo III; afecta la aorta torcica distal y la totalidad de
aorta abdominal. Tipo IV la mayor parte de la aorta abdominal, incluyendo el segmento de vasos viscerales. DIAGNOSTICO: Clnico; Una minora presenta
dolor a la palpacin abdominal o ms comn dolor lumbar confundible con clico renal. Un bajo nmero de pacientes se presentan con dolor abrupto
abdominal y en la regin lumbar. Al examen fsico se puede palpar, en algunos casos, una masa pulstil en el abdomen. La ruptura en el aneurisma es
una forma de presentacin con alta mortalidad, hay extravasacin de sangre, la cual puede ser masiva (hacia la cavidad peritoneal) o contenida (en el
retroperitoneo). La trada de hipotensin, dolor abdominal y masa pulstil en el abdomen, es observada slo en 15% de los pacientes con aneurisma
abdominal roto, donde 78% de ruptura es hacia el retroperitoneo, y slo 22% se rompe en su pared anterior hacia la cavidad peritoneal. Otra
manifestacin es la embolizacin distal de material trombo co que ocupa el saco aneurism co, y esto ocurre en 3 a 5% de los pacientes. La presencia de
soplos a la auscultacin debe hacernos sospechar enfermedad oclusiva visceral o de aorta terminal y ms raramente la presencia de una stula aortocava
con datos de falla cardiaca. El estudio fsico debe completarse con palpacin de pulsos en extremidades, y en algunos casos pueden estar disminuidos o
ausentes. Las presentaciones clnicas pueden ser de un abdomen agudo por ruptura del AAA, embolias distales o sangrado de tubo digestivo cuando se
comunica la ruptura al duodeno. En los AAT los datos de compresin a bronquios, nervios o tubo digestivo pueden hacer sospechar su presencia. Dx. De
imagen: La mayora de los AA se diagnostican en forma incidental (80% de AAA), desde la Rx simple en trax, que deforma el mediastino, o bien, la
sombra de la aorta torcica descendente se hace ms evidente. En la Rx de abdomen, adems de las calcificaciones se puede observar borramiento de
las lneas del msculo psoas. Asimismo, tanto en una evaluacin ultrasonogrca (medicin de aneurisma, no sensible para hemorragia retroperitoneal,
ni para la medicin de la distancia que separa el aneurisma de las arterias renales o las arterias iliacas), TAC, RNM. TRATAMIENTO: La mayora de los
centros quirrgicos estn de acuerdo en que todo aneurisma de 5 cm o mayor debe ser operado en forma electiva o tratado en forma endovascular. La
modificacin de los factores de riesgo es fundamental, en el manejo medico de los AAA, la hipercolesterolemia y la HTA deben controlarse
adecuadamente. Los betabloqueadores (propanolol) han sido considerados como terapia para reducir el riesgo de expansin y rotura del aneurisma. Una
vez sospechado el diagnostico debe colocarse un acceso venoso de grueso calibre e infusin de cristaloides, evitando o corrigiendo la situacin de shock
inicial; esta medida est directamente relacionada con la morbimortalidad. La mayora de los autores coinciden en que deber tener como finalidad
mantener tensin arterial sitolica en torno a 80-100mmHg. Manejo endovascular: se desarrollo para hacerla va transfemoral, siendo un procedimiento
combinado, a travs de la cual pasa la prtesis montada del sistema para ser colocada bajo fluoroscopia a nivel del aneurisma. La eleccin de los
pacientes es la clave del xito. Existe un protocolo estricto que debe incluir mediciones precisas del dimetro de las arterias iliacas.
CASO CLINICO DISECCION DE LA AORTA
Se trata de paciente masculino de 46 aos de edad, gerente de una
tienda departamental, contador, con antecedente de tabaquismo
positivo (una cajetilla diaria) es hipertenso desde hace 5 aos con mal
apego a su tratamiento, acude a urgencias debido presencia de dolor
torcico que se irradia a la espalda, refiere que nunca haba
presentado un dolor as de intenso, que inicio hace 6 horas por la
maana y se retiro de su trabajo porque comenz a sudar y a
presentar nauseas sin llegar al vomito, el dolor se ha vuelto lacerante,
el trazo electrocardiogrfico se observa elevacin ST de 1 mm en DII y
aVF el paciente se encuentra taquicardico con pulso carotideo salton.
PREGUNTA
Cul es el diagnostico mas probable hasta el momento?
RESPUESTA

CURSO ENARM CMN SIGLO XXI TEL: 36246001

a.- Infarto al Miocardio.


b.- Diseccion aortica.
c.- Prolapso valvular.
d.- Ruptura papilar.
CASO CLINICO
Se presenta el caso de un paciente de sexo masculino, de 78 aos de
edad, que habita un sector rural. Tiene el antecedente de padecer de
HTA crnica que al momento de la consulta no se encontraba en
control, ni segua ningn tratamiento farmacolgico. Refiere presentar
un hbito tabquico activo, aproximadamente 10 paquetes/ao.
Consulta por presentar un cuadro de dolor torcico de inicio sbito de
aproximadamente 4 horas de evolucin, localizado en la regin dorsal
interescapular de moderada intensidad (EVA 5/10), asociado a
nuseas, vmito. Al examen fsico: orientado, reactivo con GCS: 15,

Pharmed Solutions Institute

PGINA 162

MANUAL DE TRABAJO DEL CURSO ENARM CMN SIGLO XXI


diafortico, plido, sin disnea, con cifras tensionales elevadas y
taquicrdico (Presin arterial: 230/118 Presin arterial media: 155,
Frecuencia cardaca: 105) que refiere cambios en las caractersticas del
dolor, localizacin y aumento de su intensidad. El dolor se localiza en la
regin precordial y epigstrica, de carcter punzante transfixiante con
EVA 7/10. Se realizan dentro de los exmenes de laboratorio: enzimas
cardacas (Creatinin-kinasa total: 124, Creatinin-kinasa MB: 21,
Troponina I: 0,02) y dimero D (valor: 254) encontrndose sus valores
dentro de rangos normales.
PREGUNTA
Cul es el diagnstico ms probable en este paciente?
RESPUESTA
a.- Taponamiento cardiaco
b.- Diseccin artica
c.- Emergencia hipertensiva
d.- IAM
PREGUNTA
Cul de los siguientes factores predisponentes es el menos
relacionado con diseccin aortica?
RESPUESTA
a.- Sndrome de Marfan
b.- Envejecimiento
c.- Embarazo
d.- Hipertensin arterial
PREGUNTA
Para disminuir la fuerza de eyeccin y disminuir las resistencias
vasculares perifricas, cual es la conducta teraputica ms adecuada?
RESPUESTA
a.- Labetalol ms Nitroprusiato hasta PAS 100-120mmHg
b.- Metoprolol mas Enalaprilat hasta PAS 100-120mmHg
c.- Isosorbide mas Nitroprusiato hasta PAS 100-120mmHg
d.- Labetalol mas calcioantagonista hasta PAS 100-120mmHg
CASO CLINICO ANEURISMA ABDOMINAL 1/3
Se trata de paciente masculino de 59 aos de edad el cual se
encuentra bajo tratamiento por padecer hipertensin arterial desde
hace 10 aos, adems hipercolesterolemia, actualmente recibe
clortalidona, atenolol y pravastatina, se encuentra apegado a
tratamiento sin embargo continua fumando casi una cajetilla diaria.
Acude a su consulta de revisin durante la cual le comenta que ha
sentido una bolita que pulsa a la altura del ombligo y quiere descartar
un padecimiento cancergeno ya que en la familia hay esos
antecedentes, a la exploracin se encuentra con sus constantes vitales
controladas, ninguna alteracin cardiopulmonar, al revisar abdomen
palpa una masa depresible no dolorosa pero pulsatil a nivel de cicatriz
umblical de al menos 3 dedos de longitud.
PREGUNTA
Cul es su conducta a seguir?
RESPUESTA
a.- Solicita USG.
b.- Solicita TAC.
c.- Solicita IRM.
d.- Envia para LAPE.

PREGUNTA
Cul es el diagnostico mas probable para este caso?
RESPUESTA
a.- Aneurisma de artico
b.- Diseccion aortica
c.- Estenosis de arteria renal
d.- Carcinoma pancreatico
CASO CLINICO ANEURISMA ABDOMINAL
El paciente regresa con los resultados de la tomografa solicitada
donde se reporta aneurisma de la aorta abdominal de 4.5 cm
infrarrenal, el paciente agrega que recuerda que desde hace algunos
meses ha notado dolor ocasional de espalda que ha sido tratado por su
medico familiar con diclofenaco y medidas locales.
PREGUNTA
Cul es la conducta mas apropiada a seguir en este caso?
RESPUESTA
a.- Realizar estudios para intervencin quirrgica.
b.- Continuar con diclofenaco.
c.- Vigilar al paciente.
d.- Vigilancia y antiagregante plaquetario.
ANEURISMA ABDOMINAL
El paciente ha presentado algunas ocasiones episodios de descontrol
hipertensivo, continua su habito de tabaquismo, los niveles de
colesterol no han cedido, el paciente refiere que ha cursado con
mareos ocasionales acompaados de zumbido de odos y luces
brillantes, ha mejorado su alimentacin pero refiere aumento del dolor
de espalda y sensacin de presin interna como si le pulsara, debido
a los cambios observados se repite la TAC abdominal donde se reporta
un incremento en las dimensiones del aneurisma de la aorta
abdominal pasando de 4.5 cm hace 2 aos a 6.7 cm.
PREGUNTA
Considerando las nuevas evidencias el paciente es sometido a ciruga.
Cul es el criterio ms importante para el tratamiento elegido?
RESPUESTA
a.- El tamao del aneurisma.
b.- La presencia de dolor persistente.
c.- La presencia de descontrol hipertensin.
d.- Incremento de placas ateromatosas.
PREGUNTA
Cul es la incidencia de esta patologa?
RESPUESTA
a.- 0.1-05%
b.- 2-4%
c.- 4-9%
d.-10-13%
PREGUNTA
Cul es el factor de riesgo menos relacionado con esta patologa?
RESPUESTA
a.- Ateroesclerosis
b.- Tabaquismo
c.- Hipertension
d.- Alcoholismo

ANGINA ESTABLE, INESTABLE Y PRINZTMETAL

CURSO ENARM CMN SIGLO XXI TEL: 36246001

Pharmed Solutions Institute

PGINA 163

MANUAL DE TRABAJO DEL CURSO ENARM CMN SIGLO XXI


CIENCIAS BASICAS: La angina estable es un sndrome clnico caracterizado por malestar en el pecho, mandbulas, hombros, espalda o brazos, que
aparece con el ejercicio o estrs emocional, dura de 2-5 minutos, remite espontneamente, con el descanso o con la administracin de nitroglicerina, las
manifestaciones clnicas aparecen cuando al menos hay una oclusin del 70 % de la luz del vaso afectado. SALUD PUBLICA: Mortalidad mundial segn la
OMS: 1. Enfermedades cardiovasculares (cradiopatia isqumica), 2. Enfermedad vascular cerebral, 3. Enfermedades respiratorias inferiores, 4. EPOC.
Incidencia de 5-8% de la poblacin en general. El principal factor de riesgo es la ateroesclerosis. CLASIFICACION: De cardiopata isqumica; 1.- Angina
crnica estable. 2.- Sindrome isqumico coronario agudo (sin elevacin de segmento ST [angina inestable/IAM sin elevacin de ST], con elevacin de
segmento ST). 3. Muerte sbita. PATOGENIA: Aterotrombosis por ateroesclerosis y complicaciones tromboticas asociadas, tambin por enfermedad
inflamatoria crnica sistmica o clnicas locales, caracterizada por placas ricas en lpidos en la pared de las arterias de mediano y gran calibre. A las placas
con escaso contenido lipdico y recubrimiento fibroso grueso formado por clulas de musculo liso son las denominadas estables, condicionan estenosis
limitante del flujo sanguneo (habitualmente cuando obstruyen mas de 70% de su luz), son el sustrato de la angina crnica estable. Las placas con alto
contenido lipdico son las denominadas vulnerables o inestables, por su propensin a complicarse mediante la fisura, erosin o rotura de la placa, en
cuya superficie se forma un trombo que desencadena un evento clnico agudo. Conforme la ateroesclerosis coronaria progresa, la placa de ateroma se
deposita en las paredes y puede crecer de manera lenta y progresiva de forma excntrica (remodelado positivo), acumulndose gran cantidad de
contenido trombogenico, sin condicionar gran limitacin al flujo coronario (sin estenosis angigrafica). Desequilibrio entre oferta y demanda miocrdica:
puede ser por isquemia miocrdica regional por inadecuada perfusin coronaria, en presencia de unas demandas miocrdicas de oxigeno aumentadas,
ocurre como umbrales de esfuerzo predecibles y fijos, tiende a recurrir y es completamente reversible mediante reposo. Las arterias coronarias
presentan un endotelio disfuncionante: constriccion paradjica de la arteria con el esfuerzo fsico o el estrs emocional. Mayor limitacin al flujo.
Consecuencias de la isquemia celular: Isquemia causa aumento del Na citosolico, las corrientes tardas de Na y canales inetrcambiadores Na-H en
acidosis metabolica por isquemia. Sobrecarga de calcio, causa principal del deterioro de la relajacin ventricular (disfuncin diastlica) en isquemia
miocrdica. Aumento de la tensin parietal en distole, el lecho vascular coronario se comprime y limita el flujo miocrdico. Inestabilidad elctrica en el
miocardio isqumico. SINDROME CARDIACO X: Angina microvascular (isquemia miocrdica demostrada) hay espasmo microvascular, disfuncin
endotelial (factores posibles de isquemia miocrdica) hiperreactividad coronaria, aumento de pptido vasocosntrictor endgeno, dficit de tono
simptico, cosntriccion prearteriolar y liberacin de adenosina, exceso de potasio extracelular, aumento de la percepcin del dolor. Mecanismo de la
angina: isquemia miocrdica, reduccin de ATP, metabolismo anerobio, acidosis tisular, hay liberacin de mediadores como lactato, serotonina,
histamina, bradicinina, adenosina, metabolitos oxigenados, hay dolor precordial, cervical, mandibular, susceptibilidad psicosocial, paciente ansioso,
deprimido. ANGINA ESTABLE: La angina de pecho crnica estable es un sindrome clnico caracterizado por dolor, malestar precordial secundario a
isquemia miocrdica y sin caractersticas clnicas de inestabilidad. Angina tpica confirmada cumple 3 de las siguientes caractersticas; malestar
retroesternal de duracin y tipo caracteristicos, causado por ejercicio o estrs emocional, cede con reposo y/o nitroglicerina. Angina atpica probable;
cumple 2 de las caractersticas citadas. Dolor torcico no cardiaco; cumple una o ninguna de las caractersticas citadas. CLASIFICACION funcional por
Canadian Cardivascular Society (CCS): I: Actividad fsica habitual, como pasear o subir escaleras, no causa angina. Solo ejercicio intenso. II. Moderada
limitacin a actividad diaria. Permite caminar mas de 200m o subir mas 1 piso a velocidad normal. III. Marcada limitacin de la actividad diaria. La angina
aparece al caminar menos de 200m o al subir un piso a velocidad normal. IV. Incapacidad para llevar a cabo cualquier actividad fsica sin molestias.
Angina en reposo. DIAGNOSTICO: Clinico y se caracteriza por una molestia en la zona retroesternal que puede variar desde cierta pesadez a un dolor
intenso, sin que haya una relacin directa entre el grado de malestar y el grado de isquemia subyacente, generalmente menos de 10 minutos, de forma
tpica la angina aparece con el ejercicio o en circusntancias que aumentan el trabajo cardiaco, como el estrs emocional, el frio intenso o una comida
copiosa, dura unos pocos minutos y cede con el reposo o tras la toma sublingual de nitroglicerina. En la mayora de los casos las lesiones estenticas
graves de las arterias epicrdicas: limitan el flujo coronario, aparecen con actividades que aumentan el consumo de oxigeno de miocardio (ejercicio,
estrs emocional, taquicardia o el aument de la presin arterial), ver conveniencia de realizar una angiografa coronaria. El concepto de estabilidad
implica per se un buen pronostico a corto y mediano plazo. Es predecible sin variacin en el tiempo y autolimitada cuando disminuye el consumo de
oxigeno del miocardio. Es una manifestacin tardia de la isquemia miocrdica. ECG: 50% ausencia de isquemia miocrdica, alteraciones de
repolarizacion, depresin de ST. Con un ECG alterado, realizar ecocardiograma, angiografa isotpica, RM, en el paciente incapaz de realizar esfuerzo
fsico, hay frmacos que incrementan el trabajo cardiaco para evaluar isquemia subyacente (dobutamina, adenosina). Prueba de esfuerzo convencional,
poca especificidad en hipertrofia de VI y en el paciente revascularizado, tcnica barata, aporta informacin adicional sobre la capacidad funcinal del
paciente, realziar en pacientes sintomticos con sospecha de cardiopata isqumica, contraindicada en pacientes que requieran cateterismo de urgencia.
TRATAMIENTO: Valorar factores de riesgo, comorbilidades, factores precipitantes (emergencia hiprtensiva), arritmias, anemia, valvulopatias (estenosis
aortica), contraindicaciones del tratamiento medico o revascularizacin, el mejor predictor de pronostico es el FEVI. Betabloqueadores adrenrgicos: 1ra
eleccin en paciente con infarto previo o mala funcin ventricular, reduccin de episodios de angina, mejoran la capacidad funcional, en paciente con ICC
reducen la mortalidad. Si no es suficiente para reducir la isquemia usar antagonistas de Ca dihidropiridinico, nitrato de accin rpolongada, nicorandil
(inhibidor de canales de K), ivabradina (inhibidor de la corriente tardia de los canales de Na), usar si la FC esta >60lpm. Alivio inmediato de dolor, nitratos
sublinguale 0.6mg y pueden repetirse cada 5 minutos. Prevencion secundaria: dieta y ejercicio, no fumar controlar factores de riesgo, dislipidemia
objetivo de LDL <100 (estatinas primera opcin), AAS para todos los pacientes, clopidrogel para queienes no toleren ASS, IECA en caso de mala funcin
ventricular o riesgo moderado de episodios vasculares. Revascularizacion miocrdica: cuando el control de la angina con tratamiento medico no es
suficiente, en pacientes con alto riesgo de complicaciones isqumicas, en pacientes con isquemia grave, mala funcin ventricular, lesiones de tronco
comn de CI la ciruga ofrece mejor pronostico, revascularizacin percutnea, mayor facilidad inicial, sin embargo a mediano y largo plazo se requiere
repetir el procedimiento. ANGINA INESTABLE: Angor, en reposo o ejercicio minimo, duracin mas de 10 min y menos de 20 min, intenso, aparicin
reciente (menos de 1 mes), patrn de intensidad progresiva. Sindrome isqumico coronario agudo: disminucin de aporte de oxigeno al miocardio,
ruptura de placa ateroesclertica de una arteria coronaria; inflamacin, trombosis, vasoconstriccin, microembolizacin. Causas potencialmente
mortales dedolor torcico: SICA, TEP, diseccion aortica, neumotrax espontaneo, miopericarditis. PATOGENIA: Falta de oxigeno por percusin
insuficiente, secundaria a desequilibrio entre el aporte y la demanda de oxigeno, la causa mas frecuente es la ateroesclerosis de las arterias coronarias
epicrdicas. Fisiopatogenia; hay un obstruccin mecnica progresiva (ateroesclerosis), ruptura o esrosion de la placa ateroesclertica con un trmbo no
oclusivo sobreaadido mecnica progresiva o por obstruccin dinmica (espasmo coronario). DIAGNOSTICO: Cuadro clnico, ECG de 12 derivaciones,
biomarcadores necrosis. Dolor torcico retroesternal (tpico), dolor en epigastrio, irradiacin a cuello, hombro izquierdo o brazo izquierdo, disnea y
epigastralgia (mas frecuente en mujeres), descarga adrenrgica: diaforesis, piel palida, fra, taquicardia sinusal. Coqueteo vagal: hipotenion, nausea,
vomito, sensacin de miccin. Estertores en las bases pulmonares, 3er, 4to ruido cardiaco. Presentacion atpica: Jovenes <40aos, tercera edad >75
aos, diabticos, mujeres puede haber dolor epigstrico, dolor tipo picada, indigestin de origen reciente, dolor quemante o pleurtico, disnea
progresiva. ECG: Sospecha de dolor torcico de causa coronaria, hacerlo en lo sprimeros 10 minutos: 1. SCA con elevacon de ST o bloqueo de rama
izquierda nuevo. 2. SCA con infradesnivel del ST o inversin de la onda T. 3. ECG normal o inespecfico.En angina estable (onda T negativa de ramas
simtricas). Angina inestable (elevacin de segmento ST con onda T negativa simtrica, depresin del segmento ST). Coronariografia: lesion como una

CURSO ENARM CMN SIGLO XXI TEL: 36246001

Pharmed Solutions Institute

PGINA 164

MANUAL DE TRABAJO DEL CURSO ENARM CMN SIGLO XXI


estenosis excntrica con bordes fenestrados o protruyentes y un cuello estrecho, representan la rotura de la placa ateroesclertica, la presencia de
trombo o ambas, 5% estenosis de arteria CI, 15% tres arterias coronarias, 30% afeccion de dos vasos, 40% afeccion de un solo vaso, 10% no tiene
estenosis coronaria critica. Rx solo identifica cuadros de congetion pulmonar. Escala de TIMI: Edad >65aos 1 punto, mas de3 factores de enfermedad
coronaria 1 punto, angina grave 1 punto, estenosis coronaria significativa 1 punto, cambios del segmento ST 1 punto, marcadores sricos elevados 1
punto, consumo de aspirina en los ltimos 7 dias 1 punto. Alto riesgo: 5 puntos, riesgointermedio 3-4puntos, riesgo bajo 0-2 puntos. TRATAMIENTO:
Medidas generales; hospitalizacin, monitoreo, analgesia efectiva, oxigeno suplementario, reposo en cama. Si hay una CRISIS (MONA); morfina, oxigeno,
nitratos, aspirina. Rescate de perfusin coronaria. Antitrombotico; acido acetilsaliclico, clopidrogel, enoxaparina, heparina. Antiisquemico: nitroglicerina,
betabloqueadores, antagonistas de calcio: diltiacem/verapamilo, IECA. ANGINA DE PRINZMETAL: Angina variante, incremento transitorio del tono
vasomotor o vasoespamo, demostrado de manera convinecente por coronariografa, VASOESPASMO, disminucin intensa, sbita y transitoria en el
dimetro de una arteria coronaria epicardica (o arteria grande del tabique), hay isquemia miocrdica intensa, complicacin que aparece en ausencia de
cualquier incremento en la demanda de O2. Mas comn en paciente joven, sin factores de riesgo coronario clsico, salvo en grandes fumadores,
molestia anginosa extremadamente intensa, puede haber sincope, desnivel negativo del ST, arritmias graves: bloqueo AV y taquicardia ventricular, los
sntomas generalemnte aparecen entre la media noche y las 8am, se agrupan en 2 o 3 episodios en un lapso de 30-60 minutos, hay una relacin estrecha
entre el estrs emocional y los peisodios de vasoespasmo coronario por un desequilibrio simptico y emocional. DIAGNOSTICO: Clinico, ECG,
coronariografia (aspecto clave para el dx. Del desnivel episdico del ST, junto con el dolor. , ecocardiograma con estimulacin con dobutamina. Prueba
de provocacin: prueba con ergonovina, hiperventilacin, acetilcolina. Aspecto clave del tratamiento antagonista de calcio como tratamiento nico en
combinacin con nitratos de accin prolongada, los nitratos eliminan o impiden la isquemia a travs de su efecto de vasodilatacin directa de las arterias
coronarias, los calcioantagonistas, extremadamente efectivos en la prevencin del espasmo coronario que acompaan a la angina coronaria. Prazocina
bloqueante selectivo del receptor adrenrgico alfa, tambin ha demostrado ser efectivo. Nicorandil, vasodilatador que actua sobre el tono arterial
coronario a travs de la activacin de los canales de potasio. PRONOSTICO: En varones y mujeres con una presentacin clnica inicial de angina estable, la
incidencia de infarto de miocardio no fatal y muerte por enfermedad coronaria a los 2 aos era del 14,3 y el 5,5% en varones y del 6,2 y el 3,8% en
mujeres, respectivamente. La tasa anual de mortalidad vara entre el 0,9 y el 1,4% por ao 5-9, con una incidencia anual de infarto de miocardio no fatal
del 0,5% y el 2,6%. COMPLICACIONES: En caso de no obtener buenos resultado o si continua la evolucin trpida de la angina se podr realizar
angioplastia o ciruga de derivacin cardiaca, sin haber evidencia de sobrevida mayor con cualquier tcnica.
CASO CLINICO
Se trata de paciente masculino de 52 aos de edad, el cual acude a
consulta externa debido a que desde hace 6 meses presenta opresin
toracicca, refiere que considera que es por las presiones que tiene en
el trabajo y en casa se encuentra en proceso de divorcio, refiere que el
dolor dura menos de 5 minutos, desapareciendo respirando
profundamente y reposando, agrega que ltimamente ha presentado
dolor epigstrico acompaado de nauseas independiente del malestar
en el torax para lo cual emplea anticidos casi diario, cuenta con
antecedentes de importancia por tabaquismo y alcoholismo semanal
hasta la embriaguez, asi como padre finado por IAM, madre viva con
hipertensin arterial con tratamiento, tiene habitos alimenticios
inadecuados, ricos en grasas y comida rpida, por el trabajo y no
realiza ejercicio a la exploracin fsica se observa con leve rubicundez
facial, aliento al parecer alcoholico, presenta signo de Levine al
interrogar sobre el sitio del dolor, queratosis seborreica importante y
xantomas, al parecer tercer ruido en area precordial y estertores
subcrepitantes bibasales. Signos vitales TA 150/95 mmHg, FR 28 rpm,
FC 104 lpm.
PREGUNTA
Cules son las manifestaciones clnicas de la enfermedad ms
importantes para considerar un dianostico presuntivo?
RESPUESTA
a.- Presencia del signo de Levine.
b.- Dolor torcico.
c.- Disminucin del dolor con el reposo.
d.- La duracin del dolor.
PREGUNTA
Cul es la manifestacion clnica que no es criterios dignosticos para
establecer la enfermedad del paciente?
RESPUESTA
a.- Molestia toracica subesternal.
b.- Inicio por estrs o ejercicio.
c.- Disminuye con nitroglicerina.
d.- Sexo, mas frecuente en mujer.
PREGUNTA
Considerando la clasificacin de la enfermedad cual de los siguientes
diagnosticos es el mas probable para el caso actual?
RESPUESTA
a.- Angina definitiva.

CURSO ENARM CMN SIGLO XXI TEL: 36246001

b.- Angina probable.


c.- Angina posible.
d.- Angina no posible
PREGUNTA
Considerando la clasificacin funcional de la Canadian cardiovascular
society cual es la clase actual del paciente?
RESPUESTA
a.- Clase I.
b.- Clase II.
c.- Clase III.
d.- Clase IV.
PREGUNTA
Antes de realizar al paciente una coronariografia se encontraron los
siguientes resultados, previos cual de ellos no es criterio?
RESPUESTA
a.- Depresin del segmento ST >1 mm.
b.- Prueba de esfuerzo anormal.
c.- Sintomatologia presente.
d.- Resultados inconclusos.
PREGUNTA
Durante las pruebas se encontraron los siguientes resultados de
laboratorio en el paciente: Creatinina 1,24, Glucosa 224, Colesterol
310, Trigliceridos 420, HDL 29, Urea 41, Na 138, K 3,42, Troponina T-U:
2,8, CK 105, CK-MB 1,67, ALT-GPT 19, Hemograma y coagulacin
normal. En relacin a datos de laboratorio obtenidos cuales son los
objetivos teraputicos ms importantes?
RESPUESTA
a.- Incrementar el HDL.
b.- Disminuir triglicridos y colesterol.
c.- Revascularizacin inmediata.
d.- Buscar protena C de alta sensibilidad.
PREGUNTA
Cules son las medidas teraputicas no farmacolgicas inicial en este
padecimiento, para disminuir la demanda miocrdica de oxigeno?
RESPUESTA
a.- Propanolol.
b.- Hidroclorotiazida.
c.- Losartan.
d.- Enalapril

Pharmed Solutions Institute

PGINA 165

MANUAL DE TRABAJO DEL CURSO ENARM CMN SIGLO XXI


PREGUNTA
Considerando los datos de lpidos y el informe de la coronariografia,
que medida farmacolgica es necesario agregar para estabilizar la
placa?
RESPUESTA
a.- Clopidrogel 75 mg cada 12 hrs.
b.- Atorvastatina 40 mg cada 12 hrs.
c.- Enalapril 20 mg cada 12 hrs.
d.- Aspirina 150 mg cada 24 hrs.
PREGUNTA
Cules son los siguientes esquemas teraputicos secuenciales
antianginoso tiene un efecto favorable sobre la presin arterial, la
contractilidad y la frecuencia cardiaca ms adecuado?
RESPUESTA
a.- Prazocin 1 a 2 mg cada 24 hrs.
b.- Tamsulosina 0.4 mg cada 24 hrs.
c.- Clonidina 0.075 a 0.150 mg cada 24 hrs.
d.- Propranolol 40 a 80 mg en 24 hrs.
PREGUNTA
Cual de las siguientes comorbilidades es la menos frecuentes en la
angina tpica como es el caso?
RESPUESTA
a.- EPOC.
b.- DM.
c.- Espasmo vasocoronario.
d.- Disfuncin erctil.
PREGUNTA
Considerando la fraccin de eyeccin del paciente, cual frmaco calcio
antagonista puede usarse con cuidado comparado con el resto?
RESPUESTA
a.- Amlodipino.
b.- Verapamilo.
c.- Diltiazem
d.- Nifedipina
PREGUNTA
Considerando la fraccin de eyeccin del paciente, cual frmaco calcio
antagonista puede usarse con confianza comparado con el resto?
RESPUESTA
a.- Nitroglicerina.
b.- Isosorbide.
c.- Nitrito de amilo.
d.- Nitrito de potasio.
PREGUNTA
Considerando la patogenia de la angina de pecho cual de los siguientes
sntomas estn menos relacionados a la isquemia del miocardio y si a
la necrosis miocrdica?
RESPUESTA
a.- Disconfor o dolor en el pecho.
b.- Maxilar inferior brazo.
c.- Hombros y espalda.
d.- Elevacin de CPK-MB.
PREGUNTA
Se le solicito al paciente que usara un diario para registra el dolor, cual
es la razn fisiopatolgica que el paciente presente mayor frecuencia
de dolor en la maana?
RESPUESTA
a.- Debido a presencia de apnea.
b.- Debido al ritmio endgeno de cortisol.
c.- Debido al ritmo endgeno de secresin de catecolaminas.
d.- Debido a la insensibilidad coronaria a vasoconstrictores.

CURSO ENARM CMN SIGLO XXI TEL: 36246001

PREGUNTA
Considerando la escala de Duke Treadmill Scores para calificar el riesgo
estratificado no invasivo cul es la calificacin que presenta
actualmente?
RESPUESTA
a.- Riesgo alto (3 % de mortalidad anual).
b.- Riesgo muy alto (5 % de mortalidad anual).
c.- Riesgo intermedio (1-3 % de mortalidad anual).
d.- Riesgo bajo (menor a 1% de mortalidad anual).
CASO CLINICO
Se trata de masculino de 68 aos de edad al cual se encuentra
diagnosticado y tratado por padecer angina tpica desde hace 5 aos,
sin embargo de 6 meses a la fecha el disconfort del torax ha sido
moderado y en ocasiones durante el reposo, acude al servicio de
urgencias debido a que presenta opresin toracicca acompaado de
dolor en hombro izquierdo que se iradia a el brazo, agrega que se
encontraba discutiendo cuando comenz las molestias, no cedi al
reposo y duro ms del tiempo habitual, cuenta con los antecedentes
de hipertensin arterial desde hace 20 aos con tratamiento actual
con captopril 25 mg cada 12 hrs, hidralazina 25 mg /12 hrs, hace 5
aos fue agregado propranolol, pravastatina y aspirina para el manejo
de su angina inicialmente estable, a su ingreso se observa diafortica,
con palidez generalizada, ansiosa, con cianosis distal, sus constantes
vitales fueron TA 105/70 mmHg, FC 101 lpm, FR 29 rpm.
PREGUNTA
Considerando el cuadro clnico cul es diagnostico ms probable hasta
este momento?
RESPUESTA
a.- Angina tpica descompensada.
b.- Angina atpica inicial.
c.- Infarto agudo al miocardio.
d.- Insuficiencia cardiaca por angina previa.
PREGUNTA
Cul de los siguientes auxiliares diagnostico es mas til en este
momento para mantener una conducta clinica adecuada?
RESPUESTA
a.- Buscar biomarcadores de isquemia.
b.- Realizar electrocardiograma.
c.- Coronariografia.
d.- Programar prueba de esfuerzo con dobutamina.
PREGUNTA
Cul cual es su clase funcional con la clnica observada?
RESPUESTA
a.- Clase I.
b.- Clase II.
c.- Clase III.
c.- Clase IV
RESPUESTA
Cul es la conducta inmediata ms importante a seguir?
RESPUESTA
a.- Morfina, oxigeno, nitritos y antiagregantes.
b.- Diuretico, morfina, verapamilo y oxigeno.
c.- Diuretico, antiaginoso y trombolisis.
d.- Morfina, nitrito, trombolisis, enoxoparina.
CASO CLINICO
Masculino de 60 aos, remitido a la consulta externa de cardiologa
por el mdico de familiar por dolor torcico al caminar desde hace
medio ao, es empleado de banca, sin alergias conocidas, fumador
activo de una cajetilla al da desde 40 aos, hipertenso mal controlado
con tratamiento farmacolgico. Tambin sufre dislipemia y DM. Niega
antecedentes familiares de cardiopata. Su tratamiento habitual es
amlodipino 5mg, simvastatina 40mg y metformina 850mg. Refiere que,

Pharmed Solutions Institute

PGINA 166

MANUAL DE TRABAJO DEL CURSO ENARM CMN SIGLO XXI


desde aproximadamente 4 meses antes, tena dolor torcico
retroesternal de caractersticas opresivas, irradiado al cuello, con
sensacin acompaante de falta de aire, y que apareca cuando
realizaba esfuerzos moderados como subir ms de dos pisos de
escaleras o en momentos de estrs laboral importante; que esta
molestia desapareca tras reposar unos 5min, y que el nivel de
esfuerzo al que se iniciaba no se haba modificado. Refera molestias
en ambas pantorrillas cuando caminaba ms de cuatro o cinco
manzanas, y su mujer explic que, desde haca aos, ronca y se queda
dormido leyendo el peridico. EF: constitucin pcnica, con una
obesidad de predominio central (IMC=35,8). Su presin arterial era de
155/95mgHg y la frecuencia cardiaca, 89 lpm. La auscultacin cardiaca
revel tonos apagados, rtmicos, con soplo suave en la punta, sin
frmito asociado. Ambos hemitrax se expandan con normalidad y no
haba ruidos agregados a la auscultacin respiratoria.

Cul es la conducta teraputica ms adecuada a seguir en este caso?


RESPUESTA
a.- Oxigeno + Morfina + IECA + Heparina
b.- Oxigeno + ASA + Clopidogrel + Morfina + nitroglicerina + bisoprolol
c.- Oxigeno + Enoxaparina + Morfina + nitroglicerina + bisoprolol
d.- Morfina + oxigeno + nitroglicerina + enoxaparina

PREGUNTA
En qu clasificacin de la Canadian Cardiovascular Society se
encuentra este paciente?
RESPUESTA
a.- Clase I
b.- Clase II
c.- Clase III
d.- Clase IV

PREGUNTA
Cul es el diagnstico ms probable de este paciente?
RESPUESTA
a.- Angina estable
b.- Angina inestable
c.- IAM sin elevacin de ST
d.- Coartacin aortica

PREGUNTA
Cul sera la conducta diagnostica ms adecuada a seguir en este
paciente?
RESPUESTA
a.- Resonancia magntica
b.- Prueba de esfuerzo con dobutamina
c.- Prueba de esfuerzo convencional
d.- Radiografa de trax
PREGUNTA
Cul sera la conducta teraputica ms adecuada para este paciente?
RESPUESTA
a.- Dieta baja en sal, dejar de fumar + ASA + statina + IECA +
metformina + betabloqueador + nitratos
b.- Dieta baja en sal y grasas, dejar de fumar, ASA + fibrato + ARA II +
betabloqueador + nitratos
c.- Clopidrogel + ASA + statina + betabloqueador + Bloqueadores de los
canales de calcio
d.- ASA + fibrato + statina + bloqueadores de los canales de calcio
CASO CLINICO
Paciente del sexo femenino, 70 aos, blanca, requiri atencin de
emergencia cardiolgica quejndose de dolor torcico retroesternal,
en aprieto, de fuerte intensidad, con irradiacin para regin cervical y
mandbula, iniciada al reposo. En la historia mdica previa de la
paciente, se constataron diagnstico de hipertensin arterial sistmica
de cinco aos, en tratamiento con captopril 25 mg a cada 8 horas,
dislipidemia, DM, con control a travs de dieta, con antecedente de
IAM hace 3 aos, historia familiar positiva para enfermedad arterial
coronaria. La paciente refera tabaquismo pasivo y negaba etilismo
actual o pasado. Al examen fsico inicial, se hallaba en regular estado
general, con aspecto de dolor, lcida, orientada y coherente, mucosas
hmedas, piel plida, eupneica, leve diaforesis. Auscultacin cardaca y
pulmonar normales, y se obtuvieron los siguientes resultados: TA
100/60 mmHg y FC 80 lpm. Ante la hiptesis diagnstica de angina
inestable, se recolectaron enzimas cardiacas CK 330, y se efectu ECG,
el cual muestra FA con respuesta ventricular media a 120 lpm,
llamativo descenso de ST en cara anterolateral.

PREGUNTA
En las siguientes situaciones debemos tener cuidado al administrar
nitroglicerina, excepto en:
RESPUESTA
a.- Taquicardia (>120 lpm)
b.- Bradicardia (<50 lpm)
c.- Hipotensin arterial
d.- Disnea (23 rpm)

CASO CLINICO
Paciente de 40 aos, e historia de hipertensin arterial, fumadora de
10 cigarrillos al dia, y refiere tomar medicamentos antimigraosos a
veces una vez por semana, lleg a urgencias por la maana, con dolor
torcico opresivo, continuo, en reposo, con intensidad de 7/10,
irradiando para el cuello, que apareco posterior a una discusin
familiar. Despus de la realizacin de anlisis sanguneos y un ECG
(mostro depresin del segmento ST), se inici el tratamiento
convencional antiisqumico. Los niveles de Troponina I eran de 18
UI/L. Tras 24 horas de vasodilatores va IV, hemodinmicamente
estable y libre de sntomas, se realiz una ACP (coronariografiaia
percutnea), mostrando ausencia de estenosis epicrdica significativa.
El ecocardiograma transtorcico (ETT) mostr fraccin de eyeccin (FE)
normal (70%), aparato valvular normal y va de salida del ventrculo
izquierdo normal.
PREGUNTA
Cul es el diagnstico ms probable para este paciente?
RESPUESTA
a.- Angina estable
b.- Angina inestable
c.- Angina de Prinzmetal
d.- IAM con elevacin de ST
PREGUNTA
Qu frmacos son ms eficientes en la prevencin del espasmo
arteriocoronario de la angina variable de este paciente?
RESPUESTA
a.- Betabloqueadores
b.- Bloqueadores alfa
c.- Calcioantagonistas
d.- Salicilatos
PREGUNTA
Considerando la patologa del caso. Qu factor es el ms relacionado?
PREGUNTA
a.- Migraa
b.- Dislipidemia
c.- Esfuerzo
d.- Tabaquismo

PREGUNTA

CURSO ENARM CMN SIGLO XXI TEL: 36246001

Pharmed Solutions Institute

PGINA 167

MANUAL DE TRABAJO DEL CURSO ENARM CMN SIGLO XXI


SINDROME CORONARIO AGUDO (SICA) CIENCIAS BASICAS: Conjunto de cuadros clnicos por lo que se pone de manifiesto de forma aguda la
isquemia miocrdica secundaria en general, pero no exclusivamente, por ateroesclerosisis coronaria, sin embargo tienen su base en la erosion de una
placa ateroesclertica de las arterias coronarias, lo cual activa la coagulacin culminando en la formacin de un trombo oclusivo. SALUD PBLICA:
Cardiopatia isqumica primera causa de morbimortalidad. Ms en hombres que en mujeres. SICA SEST hasta 65.2% y SICA CEST hasta 38.2%. Hay 2.5
millones de ingresos a urgencias. Factores de riesgo tabaquismo (64%), hipertensin (50%), diabetes mellitus (42%), hipercolesterolemia, antecedente de
infarto, insuficiencia cardiaca, EVC. La arteria mayor afectada es la artereia descendente anterior (65%), y la arteria coronaria derecha (53%).
DIAGNOSTICO: Historia de dolor torcico (transfictivo, retroesternal con irradiacin a brazo, cuello o mandbula) y malestar, sntomas asociados,
simpticos, vegetativos (diaforesis, nausea, vomito, sensacin de miccin), Bezold-Jarish (infarto en cara inferior, bradicardia extrema, hipotensin,
apnea), el cual tienen una variacin circadiana, de predominio en madrugada (por el ciclo del cortisol). Se pueden presentar sntomas atpicos en adultos
mayores, diabticos, posoperados, se puede presentar disnea, fatiga, mareo, sincope. Hallazgos especficos de dao: Disfuncion ventricular izquierda;
taquicardia, estertortes crepitantes, taquipnea, S3. En disfuncin/ruptura de musculo papilar; soplo mitral. Extension al ventrculo derecho; signos de
Kussmaul, pltora yugular, hipotensin, hemitorax limpios. Dao severo del VI; 40% de la masa miocardica, CLASIFICACIN DE KILLIP Y KIMBALL, grado I
(sin insuficiencia cardiaca-mortalidad 6%), grado II (estertores, S3, mortalidad 17%), grado III (edema pulmonar-mortalidad 38%), grado IV (choque
cradiogenico-mortalidad 81%). Registro electrocardiogrfico, de acuerdo a las reas afectadas: DII, DIII, aVF: Inferior. V1, V2: Septal. V1, V2, V3:
Anteroseptal. V2, V3: Apical. V1-V4: Anterior. V1 a V6, DI-aVL: Anterior extenso. DI, aVL: Lateral alta. V5, V6: Lateral baja. DII, DIII, aVF cambios en V1:
Posterior. DII, DIII, aVfF, V4r, V1: ventrculo derecho. Biomarcadores de necrosis; mioglobina elevacion 1-4 hr, pico mximo a las 6-7 hrs, dura hasta 24
hrs. CK-mb elevacin 3-12 hrs, pico mximo a las 24 hrs, permanece hasta 48-72hrs. Troponina I elavacion 3-12hrs, pico mximo a las 24 hrs, permanece
de 5-10 dias. Troponina T elevacin 3-12 hrs, pico mximo 12hrs a 2 dias, permanece de 48-72hrs. Nos orienta sobre IAM primeras 12 hrs y reinfarto=CKMB, IAM de 2-14 dias la troponina I, T. PRONOSTICO: Predictores de mortalidad temprana; edad avanzada, Killip y Kimball alto, FC elevada, localizacin
anterior. SICA I (IAM SEST): Desarrollo de isquemia hasta necrosis miocrdica debida a inadecuado aporte de oxigeno, secundario a disminucion del flujo
sanguneo coronario. Fisiopatogenia: es casusado por ruptura o erosin de una placa ateromatosa con formacin de trombo plaquetario que genera
consecuentemente obstruccion predominantemente parcial del flujo sanguneo coronario. Es un fenmeno dinamico que trata de mantener un balance
entre la demanda del miocardio y la demanda que se observa durante la angina inestable y infarto miocrdico sin elevacin del segmento ST, las cuales
incluyen, 1.- reduccin de la luz arterial coronaria debido a un trombo no oclusivo que se desarrolla seguido a ruptura o erosion de una placa
aterotrombotica. 2.- severo estrechamiento de arteria coronario sin espasmo o trombosis por arteriosclerosis progresiva o con reestenosis con mas de 6
meses posterior a intervencin coronaria percutnea. 3.- Espasmo focal intenso de segento epicardico coronario arterial (prinzmetal o angina variante)
causando obstruccion dinmica del lumen arterial. 4.- Diseccion coronaria arterial (causado posparto) 5.- En presencia de factores precipitantes
extrnsecos a las arterias coronarias que limitan la perfusin miocrdica, causando una alteracin sbita de la demanda miocrdica de oxigeno como
(sepsis, fiebre, taquicardia) o reduccin de flujo coronario (hipotensin) o disminucin de la entrega de oxigeno al miocardio (hipoxia, anemia severa,
etc). SICA II (IAM CEST): Desarrollo agudo de necrosis miocrdica transmural debida a isquemia, por inadecuado aporte de oxigeno, secundario a
disminucin del flujo sanguneo coronario. Definicion universal: Sintomas de isquemia miocrdica, biomarcadores de necrosis miocrdica, sustento
electrocardiografico y sustento en el ecocardiograma. Fundamentos electrocardiogrficos: 1. Nueva elevacin del ST en punto J, en dos derivaciones
contiguas. 2. >0.2 mV en hombre y >0.15 en mujeres en V2 y V3, > de 0.1mV en otras derivaciones. Presencia de Q en V1-V3, >30ms/0.2mV, en DI, DII,
aVF, V4-V6, dos derivaciones contiguas. El BCRIHH, con cualquiera de los criterios de Sgarbosa que son: 1. Elevacin del ST >1mm concordante a QRS (V5,
V6, DI, aVL. 2. Depresin de ST >1mm derivaciones derechas (V1-V3). 3. Elevacion de ST >5mm discordante al QRS. Resolucin ST 2 semanas. IAM
inferior 95%, IAM anterior 40%. Elevacin ST persistente 60% aneurisma ventricular anatmico. SICA CEST 80% Q. BCRIHH mortalidad 36%, BCRDHH 12%,
ambos 50%. Fisopatogenia: Las placas ateromatosas consideradas como susceptibles a la erosion, en el caso del SICA 1, principalmente la centrolipidicas
y con una delgada capa de fibrina, la desestabilizacin y consecuente rotura de la placa es consecuencia principal de la circulacin de factores
inflamatorios locales y sistmicos, asi como los cambios fisiolgicos de las arterias coronarias por demanda u otros fenmenos, al presentarse lisis de la
delgada capa de fibrina se activa la cascada de coagulacin, adhesin y agregacin plaquetaria por Von Willebrand, glucoproteinas IIb/IIIa, tromboxano
A, epinefrina, serotonina y factor activador de plaquetas, aumentando la afinidad de la protrombina y el fibringeno convirtindose en trombina y fibrina
respectivamente consecuentemente acumulo de eritrocitos y plaquetas formando un trombo el cual no necesariamente es oclusivo, de ello depender
en gran medida las modificaciones del ECG. La elevacin de ST es un trastorno de repolarizacion, si en el ECG vemos el signo de bandera, esta sociado a
lesion e infarto y aun podemos recuperar tejido, cuando ya hay onda Q, ya no hay oportunidad de recuperar tejido. TRATAMIENTO: Inmediato; Morfina
2-4mg IV (primera elecccion) o nalbufina SC. Oxigeno si saturacin es <90%, a 3- 4 litros/min por 6h, nitroglicerina (venodilatador) pacientes con
molestia isqumica, 3 dosis de 0.4mg (Mex. 0.8mg) IV las primeras 48h, se da en pacientes ansiosos o agitados, hay que tener cuidado en infarto inferior
o derecho, hipotensin, bradicardia y evitar si hay uso previo0 de inhibidores de laq fosfodiesterasa (sildenafil). Aspirina, reduce la mortalidad hasta un
50%, debe ser sin capa entrica, contraindicada en alergia, sangrado activo, ingesta de acenocumarina, enfermedad heptica grave. Clopidrogrel, es un
inhibidro reversible de adenosindifosfato 300mgs dosis nica. Tratamiento
Elevacion de ST
conjunto: Betabloqueadores, si se usa en las primeras 24 hrs reduce hasta 29% la
> 12h
Aspirina
< 12h
2
mortalidad, disminuye la progresin y el consumo de O , mejora el flujo
betabloqueador
sanguneo coronario al permitir periodos diastlicos largos (metoprolol 5mg IV,
max. 15 mg o VO 50-200mg/dia, tambin se puede usar esmolol). IECA/ARA II,
Elegible para
Terapia
No candidato a
mejoran la remodelacin ventricular y sobrevida, VO en las primeras 24 h,
terapia
fibrinolitica
terapia de
sobretodo en IAM anterior, congestion pulmonar o FEVI <40%. Bloquean sistema
fibrinolitica
reperfusin
contraindicada
RAA, reduciendo la precarga, el estrs parietal sistlico, mejoran la funcin
sistlica, reducen el consumo de O2, limitan la extensin del infarto, de primera
lnea el IECA, si no se tolera entonces ARA. Calcioantagonistas, no
Angioplastia
Otras terapias
dihidropiridinas, tiles cuando hay isquemia recurrente, contraindicacin para
primaria o ciruga
mdicas: IECAS
corornaria
Nitratos,
betabloqueador, y cuando no hay disfuncin del VI. Heparina no fraccionada, es
Anticoagulantes
un mucopolisacarido que inhibe la agreacion plaquetaria, y la actividad de
trombina y factor X, tiene una vida media de 60-90 minutos, iniciar con bolo de
60 UI/kg (Mximo 5000 UI), continuar con infusin de 12UI/kg/hr, no mas de 1000UI por hora, mantener TTP entre 60 y 80 seg. La heparina de bajo peso
molecular (enoxaparina), vida media mas larga, ya no se usa tanto, dosis 1mg/kg c/12hrs, ajustar en pacientes con IRC y en >75aos. Estatinas, usar dosis
altas en las primeras 48 h, ya que tienen efectos pleiotropicos como, antitrombotico, antiinflamatorio, inmunomodulador (atorvastatina, pravastatina,
sinvastatina 40mg). IAM con elevacin de ST, reperfusion inmediata. Terapia de reperfusion: Reperfusion, rpida, adecuada y sostenida del la arteria
relacionada con el evento. Angioplastia, tiene una efectividad de 80-90% de permeabilizacin y es la de primera eleccin (si se dispone de transporte
para trasladar al paciente en 20 a 30 minutos. La angioplastia primaria con colocacin de stent en los primeros 90 min de ingreso al hospital. Debe

CURSO ENARM CMN SIGLO XXI TEL: 36246001

Pharmed Solutions Institute

PGINA 168

MANUAL DE TRABAJO DEL CURSO ENARM CMN SIGLO XXI


hacerse en pacientes con elevacin de segmento ST, infarto posterior verdadero y BCRIHH con inicio de sntomas<12 horas si se puede efectuar <90
minutos (puerta-baln). Pacientes <75 aos con <36 horas de inicio y <18 horas de shock. Si la duracin de sntomas es 3 horas y el tiempo p-b < 90
minutos se prefiere ACTP. Si el tiempo p-b es >90 min se prefiere fibrinlisis. Si la duracin de sntomas es >3 horas se prefiere ACTP. Segunda eleccin
terapia fibrinolitica, con un 50-70% de repermeabilizacion (nicamente en caso de que la angioplstia no este disponible), la meta es lograr un flujo TIMI 3
a los 90 minutos. La anigioplastia de rescate se usa solo cuando hay fracaso en la fibrinlisis, se encuentra dentro de las primeras 12h. Criterios de
reperfusion: Descenso de segmentos de ST menor de 50%, remisin de dolor, lavado enzimtico, arritmias de reperfusion (bradicardia, bloqueo AV,
complejo prematuro). Fibrinolisis: Estreptocinasa (puede causar hipotensin), alteplase (el mejor), reteplase, tenecteplasa. Contraindicaciones absolutas
de fibrinolisis: Embarazo, diseccin Artica, tumor fstula intracraneana, trauma ciruga de crneo, EVC hemorrgico de por vida, ciruga Mayor
Litotripsia <14 das, HAS descontrolada >210/110mm Hg, puncin de un Vaso no Comprimible, hemorragia Activa de cualquier localizacin, hemorragia
tubo digestivo renal <1 mes. El riesgo de hemorragia con la fibrinlisis la podemos ver en varios minutos, el efecto de la reperfusion lo podemos ver
desde 5 min hasta 30min max. En el ECG, despus de este tiempo ya no podemos esperar mejora, la meta es un flujo TIMI 3 por angiografa.
Tratamiento antiplaquetario adjunto para angioplastia primaria: Aspirina, clopidogrel, inhibidores de la GPIIb/IIa (Abciximab). Para fibrinlisis: Aspirina,
clopidrogel. COMPLICACIONES: La arritmias cardiacas sin una de las complicaciones mas importantes, asi como el choque cardiogenico, otras
complicaciones vienen del uso de fibrinoliticos.
CASO CLINICO SICA I
Mujer de 66 aos, hipertensa de 10 aos de evolucin con
atorvastatina y enalapril actualmente con adecuado control, acudi
por opresin torcica en reposo de 2 h de evolucin. Al ingreso se
observo con leve palidez de tegumentos con sensacin de ahogo y
disnea aun con oxigeno suplementario, TA de 165/88mmHg, obesidad
troncular y soplo sistlico II/VI en focos de la base. El ECG mostr
elevacin del ST en cara inferior y de V2 a V6. Se realiz fibrinolisis, se
encontraba en las primeras 2 hrs de evolucin. Se administr
enoxaparina, cido acetilsaliclico, nitroglicerina y morfina. Un
ecocardiograma urgente demostr acinesia del pex y la cara anterior,
y se estim una fraccin de eyeccin del 40%. Se observ un ligero
derrame pericrdico posterior.
PREGUNTA
Cual de las manifestaciones del cuadro clnico es el menos importante
para la decisin para realizar fibrinlisis en este caso?
RESPUESTA
a.- Antecedente de angina inestable.
b.- El tiempo de inicio del cuadro clnico.
c.- La auscencia de factores de riesgo a la fibrinlisis.
d.- La presencia de cambios electrocardiogrficos.
PREGUNTA
Cul de los antecedentes del caso es el de menor importancia para el
actual IAM?
RESPUESTA
a.- Hipertension arterial previa.
b.- Antecedente de angina.
c.- Sobrepeso.
d.- Uso de atorvastatina subterapeutica.
PREGUNTA
Considerando los cambios electrocardiogrficos, cuales de las arterias
es la menos probable que se encuentre involucrada?
RESPUESTA
a.- Desendente anterior (ramos septales).
b.- Desendente anterior (ramos diagonales).
c.- Circunfleja.
d.- Desendente posterior.
PREGUNTA
Cul es la complicacin mas probable considerando el sitio de
compromiso coronario?
RESPUESTA
a.- Bloqueos de rama.
b.- Disfuncin ventricular izquierda.
c.- Insuficiencia cardiaca.
d.- Hipotensin.
PREGUNTA

CURSO ENARM CMN SIGLO XXI TEL: 36246001

Considerando las caractersticas del paciente, cual de los siguiente


frmacos favorece la reduccin de la extensin del dao miocrdico,
ruptura miocrdica o reinfarto?
RESPUESTA
a.- Nitritos.
b.- Betabloqueadores.
c.- IECAS.
d.- Calcioantagonistas.
PREGUNTA
Cul de las siguientes manifestaciones bioqumicas no espera
encontrar en el paciente en cuestin?
RESPUESTA
a.- Creatinincinasa se eleva entre las 4 y 8 hrs despus del comienzo de
la sintomatologia.
b.- La fraccin MB de la CPK a partir de las 3 del comienzo del infarto.
c.- La troponina I se eleva a partir de que se inicia la sintomatologia.
d.- La mioglobina es la ultima en enzima que se eleva.
CASO CLINICO SICA I
Varn de 66 aos ingresado por IM lateral evolucionado, no tratado
con fibrinolticos por criterios de tiempo. Tras 72 h sin complicaciones,
bajo tratamiento con nitratos. En el sptimo da de ingreso present
TV polimorfa sincopal que precis cardioversin elctrica urgente. En
las horas siguientes se objetivaron numerosos episodios de taquicardia
sostenida y no sostenida, con un comportamiento incesante y
degeneracin ocasional en fibrilacin ventricular (FV), precisando
mltiples choques externos. Estos episodios no eran precedidos de
bradicardia significativa, prolongacin anormal del intervalo QT,
cambios en el segmento ST o dolor torcico. Los valores de las enzimas
cardacas dentro de rango normal.
PREGUNTA
Cual es el manejo farmacolgico mas adecuado.
RESPUESTA
a.- Lidocaina.
b.- Amiodarona.
c.- Verapamilo.
d.- Propanolol.
CASO CLINICO SICA II
Hombre de 48 aos, con antecedente familiar de cardiopata
coronaria, tabquico y dislipidmico sin tratamiento farmacolgico.
Present intenso dolor precordial mientras jugaba ftbol, consultando
en un servicio de urgencia donde present paro cardiorrespiratorio por
fibrilacin ventricular. Fue tratado con mltiples desfribilaciones y
maniobras de reanimacin durante 25 min. El ECG no mostro datos
sugerentes de infarto. Al ingreso se encontraba intubado con
asistencia ventilatoria mediante amb, hemodinmicamente inestable
requiriendo infusin de noradrenalina a 0,1 g/kg/min. 2 horas
despus se observo cambios en el ECG mostr onda Q y supradesnivel
del segmento ST hasta 3 mm en VI a V3. Troponina I: 292 ng/mL
(normal <0,05).

Pharmed Solutions Institute

PGINA 169

MANUAL DE TRABAJO DEL CURSO ENARM CMN SIGLO XXI


d.- Estado procoagulante y proinflamatorio.
PREGUNTA
Cual de los criterios es ms determinante para considerar infarto al
miocardio sin elevacin del segmento ST comparado con angina
inestable?
RESPUESTA
a.- Cambios bioqumicos.
b.- Presentacin sintomtica.
c.- Cambio electrocardiogrficos sin elevacin de ST.
d.- Factores de riesgo previos.
PREGUNTA
Considerando el tipo de manifestaciones clnicas, electrocardiogrficos
y biomarcadores cual de las siguientes conductas teraputicas no se
recomienda en el IAM SESST.
RESPUESTA
a.- Reperfusion inmediata.
b.- Inhibidores de receptores de glucoproteinas.
c.- Heparina de bajo peso molecular.
d.- Angioplastia coronaria percutnea.
PREGUNTA
Cul es la complicacin ms probable del paciente.
RESPUESTA
a.- Choque cardiogenico.
b.- Arritmia mortal.
c.- Ruptura de musculos papilares.
d.- Falla organica multiple
CASO CLINICO
Hombre de 72 aos, con historia de hipertensin de 22 aos de
evolucin tratada con enalapril 40 mg, diabetes mellitus 2 desde hace
20 aos tratada con glibenclamida 50 mg, hipercolesterolemia
manejada con atorvastatina 20 mg e hipertrigliceridemia tratada con
200 mg, actualmente se encuentra en cardiologa por dolor torcico
recurrente, fue admitido con quejas de "opresin en el pecho", que se
irradio a mandibula, hombro y brazo izquierdo de 105 minutos de
duracin que sedio reposo absoluto y nitroglicerina sublingual, el
comienzo fue sbito y acompaado de disnea, nauseas. Se
encontraron los siguientes resultados patolgicos 140/95mmHg, FC
107 lpm, FR 29 rpm, temperatura 37.8 grados, glicemia 235 mg/dl,
Colesterol 429 mg/dl, Trigliceridos 650 mg/dl. Electrolitos sericos, K
5.5, El ECG realizado en la admisin mostr fibrilacin atrial con
frecuencia ventricular rpida y Segmento ST y Onda T normales. El
examen fsico no mostr alteraciones significativas, pero hubo
elevacin de biomarcadores cardacos, con pico de troponina I de 1,84
ng/ml y CKMB-masa 13,4 ng/ml. El diagnstico inicial no revel infarto
agudo de miocardio con elevacin del Segmento ST, y la angiografa
coronaria se muestra. El ecocardiograma revel una FE >40.
PREGUNTA
Considerando la sintomatologa observada en el paciente que clase
funcional killip y Kimball.
RESPUESTA
a.- Clase funcional Killip I.
b.- Clase funcional Killip II.
c.- Clase funcional Killip III.
d.- Clase funcional Killip IV.
PREGUNTA
La presencia de DM acelera el proceso de aterognesis a travs de
diversos mecanismos, cual de los siguientes mecanismos es mayor en
el paciente actual?
RESPUESTA
a.- Anomalas en concentraciones y la composicin de las
lipoprotenas.
b.- Asociacin con la hipertensin.
c.- Oxidacin lipdica.

CURSO ENARM CMN SIGLO XXI TEL: 36246001

PREGUNTA
Cul es los paraclinicos en el primer nivel de atencin es de mayor
utilidad para confirmar el diagnstico de la insuficiencia cardaca en el
paciente?
RESPUESTA
a.- ECG.
b.- Banda sin fin.
c.- Ecocardiograma.
d.- Arteriografia.
PREGUNTA
Cual de los siguientes factores comorbidos presenta la mayor
importancia para desencadenar un evento coronario en el paciente.
RESPUESTA
a.- Diabetes mellitus.
b.- Hipertensin arterial.
c.- Dislipidemia.
d.- Tabaquismo.
PREGUNTA
Cual de los siguientes datos patolgicos en el paciente presenta la
mayor importancia para desencadenar un evento coronario en el
paciente.
RESPUESTA
a.- Niveles de glucosa.
b.- Hemoglobina glucosilada.
c.- Trigliceridos
d.- Colesterol.
PREGUNTA
Cual es la conducta a seguir con los siguientes criterios es el menos
importante en este paciente para envio inmediato a tercer nivel?
RESPUESTA
a.- Dolor toracico sugerente de angor pectoris.
b.- Dislipidemia.
c.- Hipertensin arterial.
d.- Diabetes mellitus.
PREGUNTA
Cual de las siguientes medidas debe iniciar antes de enviar al paciente
a segundo nivel cuando existe alta sospecha de cardiopata isqumica?
RESPUESTA
a.- Dieta y ejercicio progresivo.
b.- Acetilsalicilico y pravastatina.
c.- Diuretico y betabloqueador.
d.- Estudios bsicos y gabinete.
CASO CLINICO
Paciente de masculino de 77 aos de edad, con antecedente de
hipertensin de 13 aos de evolucin, as como diabetes mellitus y
dislipidemia, tabaquismos desde hace 15 aos, con IMC de 30 Kg/m2,
el cual sufri un EVC hace 2 meses, actualmente se presenta con dolor
torcico opresivo de 35 minutos con intensidad 8/10, que se irradia a
brazo izquierdo, y tambin refiere dolor en cuello, refiere pezantes y el
dolor no mejora con nada, nuseas sin llegar al vmito y diaforesis de
leve a moderada. El familiar refiere que dolor inicio mientras se
encontraba cambiando la llanta de su automvil, asi como un
internamiento hace 3 meses a consecuencia de un EVC. A la EF: FC: 90
lpm, FR: 25 rpm, TA: 130/80 mmHg, Temp: 37, Sat: 87%, Dolor: 9/10,
Llenado capilar: 2-3 seg, Estado de despierto: alerta. Qumica
sangunea: Gluc 125, BUN 18, Creat 0.75. ELECTROLITOS SERICOS: Na
135, K 4, Cl 110, Ca 8.5, P 4.5, Mg 1.98. Troponina 3.5. Se le realiza un
ECG, en el cual se observa elevacin de ST de 2mm en DII, DII y aVf.
PREGUNTA
Cul es conducta teraputica ms adecuada para este caso?

Pharmed Solutions Institute

PGINA 170

MANUAL DE TRABAJO DEL CURSO ENARM CMN SIGLO XXI


RESPUESTA
a.- Terapia antitrombotica y antisquemica precoz
b.- Terapia de reperfusion inmediata mediante fibrinolisis
c.- Terapia antiagregante y antisqumica precoz
d.- Angioplastia coronaria transluminal percutnea (ACTP)
PREGUNTA
Cul es la conducta teraputica inicial mas adecuada en este caso?
RESPUESTA
a.- Oxigeno, nitroglicerina sublingual, aspirina, morfina, abciximab
b.- Oxigeno, isosorbide sublingual, heparina no fraccionada, morfina
c.- Oxigeno, nitroglicerina, heparina no fraccionada, tenecteplase
d.- Oxigeno, nitroglicerina, morfina, aspirina, tenecteplase
Cul es la causa ms probable por la que le daramos un
betabloqueador a este paciente?
RESPUESTA
a.- Prevenir regurgitacin mitral
b.- Prevenir arritmias ventriculares
c.- Prevenir un bloqueo de rama de Haz de Hiss
d.- Prevenir un bloqueo AV
CASO CLINICO
Hombre de 55 aos, con antecedente de hipercolesterolemia familiar,
fumador de 5 cigarillos/dia desde los 25 aos, e Hipertensin Arterial
sin tratamiento diagnosticada hace 20 aos. Hoy, despus de una
discusin en el trabajo, presenta dolor epigstrico, por lo que ingiere
anticidos, sin disminuir el dolor. Al pasar los minutos, siente dolor
opresivo retroesternal, intensidad 8 /10, que no cede con el reposo,
acompaado de abundante sudoracin. Por persistencia del dolor (van
6 horas), consulta en el Servicio de Urgencia. Peso 67 kg. P/A 140/ 90
T 36.5, FC =96, Conciente, lucido, orientado, angustiado, plido y
sudoroso. Examen Cardaco: Normal. Rx Torax: Cardiomegalia
moderada, hilios sin congestin, BUN 36, Crea 1,5, Glicemia 101,
Colesterol 345, Triglicridos 310, Hemograma normal, CK total = 580%,
CK- MB = 30, ECG= Ritmo sinusal, hipertrofia de vemtriculo izquierdo,
supradesnivel del segmento ST en D1- AVL, V3, V4 V5, T (-) en D2, D3,
AVF.

PREGUNTA
Cul es el manejo inicial de este paciente?
RESPUESTA
a.- Fibrinolisis con tecneteplase
b.- Morfina, clopidrogel, nitroglicerina
c.- Aspirina, nalbufina, nitroglicerina
d.- Fibrinolisis con estreptoquinasa
PREGUNTA
Cul es el tratamiento conjunto ms adecuado para este paciente?
RESPUESTA
a.- Betabloqueador, IECA, Heparina no fraccionada
b.- Betabloqueador, ARA II, calcioantagonista
c.- Calcioantaonista, IECA, enoxaparina
d.- ARA II, heparina no fraccionada
PREGUNTA
De acuerdo al curso del cuadro clnico y tiempo de evolucin En este
momento la conducta ms adecuada a seguir es?
RESPUESTA
a.- Fibrinolisis con estreptocinasa
b.- Fibrinolisis con tecneteplase
c.- Angioplastia coronaria con colocacin de Stent
d.- Anticoagulacin

PREGUNTA
Cul es conducta teraputica ms adecuada para este caso?
RESPUESTA
a.- Terapia antitrombotica y antisquemica precoz
b.- Terapia de reperfusion inmediata mediante fibrinolisis
c.- Terapia antiagregante y antisqumica precoz
d.- Angioplastia coronaria transluminal percutnea (ACTP)
PREGUNTA
Cul es la conducta teraputica inicial ms adecuada en este caso?
RESPUESTA
a.- Oxigeno, nitroglicerina sublingual, aspirina, morfina, abciximab
b.- Oxigeno, isosorbide sublingual, heparina no fraccionada, morfina
c.- Oxigeno, nitroglicerina, aspirina, heparina no fraccionada,
estreptoquinasa
d.- Oxigeno, nitroglicerina, morfina, aspirina, tenecteplase
PREGUNTA
Cul es la prevencin secundaria ms adecuada post-IAM, para este
caso?
RESPUESTA
a.- Estatinas para manejo de LDL
b.- Tiazida para manejo de hipertensin
c.- ASA, para prevencin de EVC
d.- Beta-bloqueador para taquicardia

CURSO ENARM CMN SIGLO XXI TEL: 36246001

CASO CLINICO
Masculino de 55 aos de edad con antecedente de
hipercolesterolemia, suspendido el tx., con estatinas por temor a sus
efectos secundarios, HTA para la que realiz tx., con betabloqueantes
que fueron retirados por disfuncin erctil. Cardiopata isqumica y
tras episodios de ardor retroesternal se realiz un estudio de perfusin
que mostr una zona de isquemia posteroinferior equivalente a 10%,
no reversible al esfuerzo que fue interpretada como probable necrosis
antigua, tratamiento actual Diltiazem 240 mg/da, Enalapril 10 mg/da.
Actualmente: Estando previamente bien, hoy a las 13:30, despus de
comer, comienza con dolor retroesternal opresivo irradiado a zona
interescapular y a brazo izquierdo, acompaado de mareo y ligero
cortejo vegetativo. Refiere haber tomado una aspirina (500 mg de
AAS). A su llegada al hospital haban transcurrido 4h, persiste dolor
torcico, sin disnea. Exploracin general: TA 121/90 mmHg. Fc 70 lpm.
SO2 96%. Afectado, plido aunque bien perfundido e hidratado. No IY,
ni edemas en EEII. Pruebas complementarias: ECG, se observa
elevacin de ST (signos de bandera) en DII, DIII, Avf. Rx Trax: sin
alteraciones valorables. Analtica: Creatinina 1,66/ Glucosa 143/ Urea
39/ Na 138/ K 2,82/ Troponina T ultrasensible 75,5/ CK 155/ ALT-GPT
27/ Hemograma normal/ INR 0,93.

PREGUNTA
Pasados 90 minutos tras la administracin de fibrinoltico, aunque el
paciente no refiere dolor, en el ECG persiste la elevacin del ST e
incluso mayor. Cul es la conducta ms adecuada a seguir?
RESPUESTA
a.- Angioplastia primaria
b.- Angioplastia de rescate
c.- Angioplastia facilitada
d.- Colacin de baln
PREGUNTA
Si el paciente tiene una adecuada reperfusin Qu es lo menos
probable encontrar en el paciente?
REPUESTA
a.- Descenso del segmento ST menor de 50%
b.- Disminucin de dolor
c.- Arritmia de reperfusin
d.- Presencia de onda Q

Pharmed Solutions Institute

PGINA 171

MANUAL DE TRABAJO DEL CURSO ENARM CMN SIGLO XXI


ARRITMIAS CARDIACAS:
CIENCIAS BASICAS: La arritmia cardiaca se define como la alteracin del ritmo cardiaco establecido como normal entre 60-100 lpm. Menor a 60 lpm, ser
bradicardia y si es mayor a 100 lpm, ser taquicardia. La arritmia ms frecuente es la fibrilacin auricular. El fenmeno R/T se da en en el periodo
refractario relativo en mitad de fase 3 y 4 de potencial de accin. Recordando en reposo la celula tiene -90mV, en la fase 0 entra Na (despolarizacin), en
fase 1 cierre de canales rapidos de Na y hay +20mV, fase 2 apertura de canales lentos de Ca, fase 3 salida de K y se hace la onda T, fase 4 canales
NaKATPasa, hiperpolarizacion onda U, el potencial se hace mucho mas negativo entre -94 y -96mV. SALUD PUBLICA: La prevalencia de FA oscila 0,7-

Ritmo rpido o lento


Ritmo regular o irregular
QRS estrecho o ancho
nda p
Relacin p/QRS
Diagnostico probable
Manejo

Lento
Regular
Estrecho
Presente
Todas las ondas p conducen
BRADICARDIA SINUSAL
Causa ms frecuente: sndrome del seno enfermo

Ritmo rpido o lento


Ritmo regular o irregular
QRS estrecho o ancho
nda p
Relacin p/QRS
Diagnostico probable
Manejo

Lento
Regular
Estrecho
Presente
Todas las ondas p conducen, PR alargado (>0.21
mseg)
BLOQUEO AV de 1
Habitualmente no precisa tratamiento

17,6%, La prevalencia de las otras 2 taquiarritmias supraventriculares ms importantes, el flutter auricular y la taquicardia paroxstica supraventricular
(TPSV), es mucho ms baja. La incidencia del flutter auricular es de 88 por 1,000,000 de personas/ao. ACTUACION INICIAL EN SERVICIO DE
URGENCIAS: Hay que sistematizar nuestra actuacin para no pasar por alto aspectos esenciales. 1. Realizar ECG; verlo con calma y valorar los siguientes
aspectos: a) si el ritmo es rpido o lento, b) si el ritmo es regular o irregular, c) si el QRS es estrecho o ancho, d) la presencia o no de ondas p, e) la
relacin de p/QRS, esto nos orientar la arritmia a la entidad etiolgica ms probable. 2. Monitorizar las constantes vitales: como la tensin arterial,
saturacin de oxigeno, frecuencia cardiaca y respiratoria, vigilar la perfusin tisular, para detectar precozmente los signos de bajo gasto cardiaco. Todas
las alteraciones teraputicas deben documentarse con tiras de registro de ECG. 3. Canalizar una va venosa perifrica y administrar oxigeno: es necesario
Ritmo rpido o lento
Ritmo regular o irregular
QRS estrecho o ancho
nda p
Relacin p/QRS
Diagnostico probable
Manejo

Lento
Regular
Estrecho
Hay ondas p con ritmo regular
Ninguna onda p conduce, no hay relacin p/QRS
BLOQUEO AV 3 COMPLETO
Derivar a servicio de urgencias, precisara marcapasos

ya que el shock puede no detectarse precozmente y la mala perfusin perifrica dificulta obtener una via. El oxgeno lo administraremos en funcin de la
oxigenacin, para evitar hiperoxigenacin. 4. Valorar estabilidad hemodinmica-los signos adversos y que indican mala evolucin son; a) shock-palidez,

Ritmo rpido o lento


Lento
Ritmo regular o irregular
Regular
QRS estrecho o ancho
Estrecho
nda p
Presente
Relacin p/QRS
Alargamiento progresivo del PR hasta una onda p no conduce
Diagnostico probable
BLOQUEO AV 2 WENKEBACK (MOBITZ I)
Manejo
Habitualmente no precisa tratamiento

Ritmo rpido o lento


Ritmo regular o irregular
QRS estrecho o ancho
nda p
Relacin p/QRS
Diagnostico probable
Manejo

Lento
Regular
Estrecho
Presente
PR, constante, hasta que una onda p no
conduce
BLOQUEO AV 2 MOBITZ II
Derivar a servicio de urgencias, precisara marcapasos

sudoracin, por aumento de actividad simptica, bajo nivel de conciencia, por disminucin de FSC o hipotensin arterial. b) sincope-por disminucin de
FSC. c) insuficiencia cardiaca-edema pulmonar o fallo del VI o ingurgitacin yugular, hepatomegalia por fallo de VD. d) isquemia miocrdicar aumento de
la demanda de oxigeno del miocardio. Si el paciente tolera bien la arritmia probablemente no haga falta hacerle nada o bien solo frmacos. Si la tolera
mal, al final precisara una cardioversin elctrica. Dos aspectos importantes en atencin primaria: primero no debemos ser ms agresivos con nuestra
actuacin que la propia arritmia lo es con el paciente y en segundo lugar, los frmacos antiarrtmicos combinados o a dosis altas pueden ser
arritmognicos: para evitar efectos secundarios es recomendable seguir la mxima de un paciente, un solo antiarrtmico. Adenosina y ATP son los
nicos que escapan a esta mxima. RITMO SINUSAL: Todo lo que no cumpla con ritmo sinusal es una arritmia; 1. Onda P (+) en DII, 2. onda p precede a
QRS, 3. Intervalo PR 120-200mseg, 4. FC 60-100lpm, 5. RR regular. Por otra parte revisar que la P sea negativa en aVR, si es positiva esta mal tomado o
hay dextrocardia, si P negativa en DII, no es ritmo sinusal. DIAGNOSTICO DIFERENCIAL DE BRADICARDIAS: Ver cuadros anexos. MANEJO DE LAS
BRADICARDIA: Tratar las bradicardias, mal toleradas o que cumplan criterios de inestabilidad que se numeran a continuacin: 1. Tensin arterial sistlica
<90mmHg. 2. Frecuencia cardiaca <40 lpm. 3. Bradicardia sintomtica. 4. Bradicardias con QRS ancho. Los medicamentos de que se dispone para su
manejo son; ATROPINA: 0,5mg IV (dosis inferiores producen un efecto paradjico). Se puede repetir cada 2 minutos, dosis mxima 3 mgs. Usar con
precaucion en el sndrome coronario agudo, ya que puede aumentar el grado de isquemia cardiaca; en transplantados cardiacos causa bloqueo AV de
alto grado o paro sinusal. Si tras la medida anterior el paciente no mejora o hay riesgo de asistolia, precisara un marcapasos transcutneo, por lo que hay
que avisar a servicios de emergencia. Las situaciones que aumentan el riesgo de que la bradicardia derive en asistolia son: asistolia reciente, paro
ventricular >3seg, BAV Mobitz II, BAV 3 con QRS ancho. Si no disponemos de servicio de emergencias a corto plazo, el paciente se inestabiliza, podemos
administrar una perfusin de ADRENALINA a 1mg en 100cc de SF en 10 min. Si no mejora podemos golpear rtmicamente (60 veces por minuto), con
nuestro puo en el margen inferior izquierdo del esternn, a modo de marcapasos externo. Si la causa de la bradicardia es una intoxicacin con
betabloqueadores o antagonistas de calcio, administraremos GLUCAGON 1mg.
DIAGNOSTICO DIFERENCIAL DE TAQUICARDIAS: Ver cuadro
anexos.Recordar que el ciprofloxacino, haloperidol, pueden generar sndrome de QT largo, el verapamilo esta contraindicado en el sndrome de
preexitacion. Cuando se presenta un bigeminismo en el ECG, hay riesgo de hacer un fenmeno de R/T y hacer una arritmia letal que puede llevar a

CURSO ENARM CMN SIGLO XXI TEL: 36246001

Pharmed Solutions Institute

PGINA 172

MANUAL DE TRABAJO DEL CURSO ENARM CMN SIGLO XXI


muerte sbita. La nica indicacin para desfibrilar a un paciente inestable vivo, es la taquicardia ventricular helicoidal o Trasade de Pointes, paciente
estable medicamento, inestable cardiversion (sincronizar con una R). En un paciente que tiene QRS estrecho y esta estable se puede dar masaje
acrotideo solo por 10 segundos, si no se revierte se puede hacer uso de adenosina. BRDHH: QRS mas de 120mseg, con empastamiento en meseta,
complejos rsR en V1, S empastada en V4-V6, causas en comunicacin interauricular, padecimientos crnicos que cursan con hipertensin pulmonar,
comn en post-cirugia correctiva de Tetralogia de Fallot, en miocardiopatas dilatadas Enfermedad de Chagas. BRIHH: QRS ensanchado >120mseg, R
empastada inversin secundaria de onda T en DI, aVL, V5 y V6, S empastada en V1, causas 90% asociado a hipetrofia ventricular izquierda, pacientes con
cardiopata isqumica y miocardiopata hipertrfica.

CURSO ENARM CMN SIGLO XXI TEL: 36246001

Pharmed Solutions Institute

PGINA 173

MANUAL DE TRABAJO DEL CURSO ENARM CMN SIGLO XXI


La cardioversion electrica sincronizada, para revertir una arritmia debe ser sincronizada sobre la onda R para evitar el periodo refractario que sigue a la
onda T, ya que podriamos provocar una taquicardia ventricular, de efectos deletereos. Previamente debemos sedar al paciente con 10-15 mgs de
midazolam intravenoso. La energia necesaria para la cardioversion de una arritmia de QRS ancho o una FA (son las mas frecuentes) es de 70-120J si
usamos un desfibrilador bifasico o 100J si es monofsico. En cambio para cardiovertir arritmias de QRS estrecho o un flutter auricular la energia a palicar
es de 120-150J en desfibriladores bifasicos y 200J en los monfasicos. Si la primera descarga no es eficaz, se puede incrementar la energia (50J) en las
siguientes y si hay dificultad para sincronizar el choque, podemos proceder a choque no sincronizado.
CASO CLINICO BLOQUEO AV
Se trata de paciente masculino de 31 aos de edad, licenciado en
derecho, originario del Estado de Tabasco, soltero pero con pareja
sexual, de orientacin homosexual, con prcticas de riesgo de los 19 a
los 27 aos, actualmente es negativo a VIH y mantiene una proteccin
adecuada, consume bebidas embriagantes casi cada semana y tambin
presenta tabaquismo positivo, no ms de 5 cigarrillos diarios, cuenta
con antecedentes de importancia dengue a los 17 aos y
apendicetoma a los 19, se le est realizando evaluacin para ingresar
a laborar en el sector gubernamental, el paciente refiere excelente
estado de salud sin embargo reconoce no estar en forma, no practica
actividades fsicas y su trabajo es muy sedentario, refiere uso de
drogas con uso recreativo y experimental de los 21 a los 26 aos, solo
agrega sentir brincos en el corazn en ltimas fechas, tal vez mareo
ocasional que no le ha resultado de importancia, a la exploracin fsica
se encuentra un IMC de 24, aparenta buen estado general, resto
normal, los parmetros de laboratorio normales pero el ECG del
chequeo muestra un trazo donde se prolonga progresivamente el
intervalo PR de 0.20 a 0.28.
PREGUNTA
Cul de los siguientes diagnsticos es el ms probable en este
hallazgo?
RESPUESTA
a.- Bloqueo AV de 1er grado.
b.- Bloqueo AV de 2do grado Mobitz I.
c.- Bloqueo AV de 2do grado Mobitz II.
d.- Bloqueo AV de 3er grado.
CASO CLINICO BLOQUEO AV
Se trata de paciente femenino de 72 aos de edad, originaria de
Guanajuato, viuda, dedicada a su casa, actualmente bajo tratamiento
por insuficiencia cardiaca por hipertrofia ventricular secundaria a
hipertensin arterial de 30 aos de evolucin, cuenta adems con
aterosclerosis e insuficiencia intestinal crnica, refiere familiar que
encontr a la paciente confundida, desorientada y posteriomente
pierde la conciencia trasladndole a urgencias donde se ingresa a
choque, el trazo electrocardiogrfico se observa ondas P que no
conducen y latidos de escape ventricular de forma variable (disociacin
auriculoventricular). Se reanima a la paciente y se prepara para
colocacin de marcapasos, previamente se estableci el diagnostico de
un Bloqueo AV de 3er grado.
PREGUNTA
Cul de las siguientes condiciones es la causa ms probable del
bloqueo de este paciente?
RESPUESTA
a.- Uso de digitalico.
b.- Trastorno electroltico.
c.- Uso de diurtico.
d.- Uso de antihipertensivo.
CASO CLINICO FIBRILACION AURICULAR 1/2
Se encuentra de guardia en el servicio de urgencias donde llega por la
tarde paciente femenino de 49 aos de edad la cual refiere que desde
la noche anterior inicio con molestia en el torax con disconfort,
ansiedad y sensacin de que su corazn latia muy rpido, aun asi
acudi al trabajo, pero se sali porque le faltaba el aire y presencia
mareo continuo, teniendo que descansar repetidamente, la paciente
es madre soltera, con tres hijos, emigra al Estado de Mxico hace 15
aos por disfuncin familiar severa con violencia fsica, tiene

CURSO ENARM CMN SIGLO XXI TEL: 36246001

antecedente de histerectoma total abdominal con ooforectomia


bilateral por presencia de cncer cervicouterino, adems de recibir
quimioterapia y radioterapia hace 5 aos con excelente respuesta y
controles adecuados, adems cuenta con antecedente de procesos
infecciosos recurrentes de vas areas altas y recuerda escasamente
que le diagnosticaron fiebre reumtica a la edad de 23 aos, pero ya
no continuo su atencin por sentirse mejor, a la exploracin fsica se
encuentra palidez de tegumentos, cianosis peribucal, no hay datos de
ingurgitacin yugular, se ve confusa, al examen mental esta
desorientada, agitada, incongruente, sntomas se duran solo minutos
sus ruidos cardiacos estn disminuidos, constantes vitales son TA
150/60 mmHg, FC 145 lpm, FR 31 rpm, normotermica, la paciente
refiere que fue diagnosticada recientemente con hipertensin arterial
y prescribieron metoprolol 100 mg cada 12 horas, el ECG presenta
ausencia de onda P, con presencia de ondulaciones irregulares y de
pequea amplitud, pero complejos QRS de 80 a 100 por minuto.
PREGUNTA
Cul es el manejo ms apropiado para evitar complicaciones agudas
para el caso?
RESPUESTA
a.- Propafenona.
b.- Amiodarona.
c.- Digoxina.
d.- Verapamil.
CASO CLINICO FIBRILACION AURICULAR) 2/2
Luego de 6 horas de ingreso la paciente presenta datos de focalizacin
neurolgica, caracterizado por desviacin de la comisura labial,
disartria y dislalia, parestesias de miembro torcico contralateral a la
cara, se observa disminucin de fuerza en el mismo sitio, a las 24 horas
posteriores se observa recuperacin paulatina de sensibilidad y fuerza
de miembro torcico y discreta mejora en la cara, al parecer se trato
nicamente de una isquemia cerebral transitoria, actualmente se
encuentra con antiarritmico, anticoagulante y neuroprotector,
continua en terapia.
PREGUNTA
Cul es la causa ms probable que condujo a esta arritmia a la
paciente?
RESPUESTA
a.- Uso reciente de Betabloqueador.
b.- Valvulopatia crnica.
c.- Sindrome paraneoplasico.
d.- Cardiomiopatia metastasica.
CASO CLINICO FLUTER AURICULAR
Se encontr en la via publica a sujeto de 50 aos aproximadamente el
cual se encuentra en condicin de calle, se observa desalineado, con
nula higiene, con aliento alcohlico, se observan lesiones drmica sin
tratamiento, el paciente se encuentra desorientado, confundido, con
tendencia a perder el equilibrio con palidez generalizada con prdida
de conciencia por segundos, se toman muestras para laboratorio, se
coloca ECG y se observa un trazo caracterizado por ausencia de ondas
p con ondas regulares dentadas en derivaciones DII, DIII y aVF
particularmente negativas.
PREGUNTA
Cul es la conducta a seguir ms adecuada?
RESPUESTA
a.- Cardioversion.

Pharmed Solutions Institute

PGINA 174

MANUAL DE TRABAJO DEL CURSO ENARM CMN SIGLO XXI


b.- Colocacion de marcapasos.
c.- Administracion de Amiodarona.
d.- Administracion de Digoxina.

conduce a travs del nodo AV, de manera que una onda P se bloquea,
el segmento PR es constante, QRS estrecho.

CASO CLINICO BRADICARDIA


Varn de 67 aos. Como antecedentes personales de inters destacan
hipertensin arterial, diabetes mellitus tipo 2, dislipemia, bronquitis
crnica, clicos nefrticos de repeticin, ciruga de hemorroides y de
hernia umbilical. Tratamiento habitual con insulina aspart,
metformina, omeprazol, tiaprida y lorazepam. Ingresa en nuestro
centro hospitalario para estudio por posible neoplasia de cabeza de
pncreas. Encontrndose hemodinmicamente estable, se inicia pauta
con octreotide 100g/8h subcutnea. A las 24 horas presenta
bradicardia sinusal a 40 lpm con presin arterial (PA) de 120/60 mmHg,
sin sintomatologa asociada. En el ECG se observa bradicardia sinusal y
hemibloqueo de rama anterior izquierda, realizan ecocardiograma
transtorcico sin alteraciones significativas y colocan Holter de 24
horas. Presenta deterioro clnico con bradipsiquia e inestabilidad de la
marcha; PA, 90/45 mmHg y 35 lpm. Valorada tira de ritmo en planta
presenta bradicardia sinusal a 37 lpm y hemibloqueo de rama anterior
izquierda descrito. Tras administracin de atropina hasta 3mg persiste
bradicardia sintomtica.
PREGUNTA
Cul es la conducta a seguir ms adecuada?
RESPUESTA
a.- Cardioversion.
b.- Colocacion de marcapasos.
c.- Administracion de Amiodarona.
d.- Administracion de adrenalina.
CASO CLINICO
Mujer de 83 aos, hipertensa de 20 aos de evolucin, en tratamiento
con losartan, diabtica de 15 de evolucin, en tratamiento con
insulina. Acude al servicio de urgencias con palpitaciones de una hora
de evolucin, sin dolor torcico, ni disnea, sin mareo, ni cortejo
vegetativo. EF: FC 190x, FR 18x, TA 130/90mmHg. ECG: muestra
arritmia, con ritmo regular, se aprecia QRS estrecho regular
PREGUNTA
Cul es la conducta teraputica ms adecuada a seguir en esta
paciente?
RESPUESTA
a.- Cardioversin
b.- Amiodarona
c.- Desfibrilacin
d.- Adenosina
PREGUNTA
Si la paciente a las 2 hrs de ser ingresada no mejora y por el contrario
inicia con inestabilidad hemodinmica y continua con trazo
electrocardiografico. Cul es la conducta teraputica ms adecuada a
seguir?
RESPUESTA
a.- Cardioversin
b.- Amiodarona
c.- Desfibrilacin
d.- Adenosina
PREGUNTA
Un ao despus se le realiza un nuevo electrocardiograma el cual
muestra, de forma sbita un estmulo supraventricular que no se

CURSO ENARM CMN SIGLO XXI TEL: 36246001

RESPUESTA
Cul es el diagnstico ms probable en este momento?
RESPUESTA
a.- Bloqueo AV de 1er grado
b.- Bloqueo AV de 2 grado Mobitz I
c.- Bloqueo AV de 2 grado Mobitz II
d.- Bloqueo AV de 3er grado
CASO CLINICO
Femenino de 19 aos remitida a la consulta de Cardiologa por haber
observado durante una exploracin rutinaria una auscultacin cardiaca
arrtmica sin soplos. Al interrogarla, la paciente refera haber
presentado en dos ocasiones durante los ltimos 10 meses, sendos
episodios consistentes en visin borrosa, cefalea, palidez cutnea e
hipersudoracin seguidos de sensacin de debilidad muscular y mareo,
que haban remitido de forma progresiva al tumbarse y colocar las
piernas en alto. En ninguno de los mismos lleg a perder la
consciencia, y sealaba que en una ocasin haba sucedido tras una
extraccin sangunea para analtica y en la otra tras un traumatismo
doloroso en el brazo derecho. Adems, de forma ocasional, ha
padecido episodios de escasos minutos de duracin de dolor torcico
izquierdo no irradiado definido como pinchazos, sin relacin con la
actividad fsica ni cortejo vegetativo acompaante, y con remisin
espontnea sin haber interferido en sus actividades cotidianas. No
sealaba otra sintomatologa cardiovascular, y sus antecedentes tanto
personales como familiares carecan de inters. Se constat una
auscultacin cardiaca arrtmica sin soplos ni otros hallazgos
significativos en el resto de la exploracin fsica. Se realiza un ECG el
cual muestra ritmo sinusal a 81 latidos por minuto, con un eje QRS a
80, duracin del intervalo PR (150 ms), QRS y onda p normales.
PREGUNTA
De las siguientes caractersticas. Cul es la menos probable que nos
oriente a que se trata de una taquicardia sinusal?
RESPUESTA
a.- Onda P positiva en DII
b.- Toda onda P, precede a un QRS
c.- R-R regular
d.- Onda P positiva en aVR
PREGUNTA
Cul es la causa menos probable de taquicardia sinusal?
RESPUESTA
a.- Ejercicio
b.- Insuficiencia cardiaca
c.- Hipertiroidismo
d.- Cafena
PREGUNTA
Cul es la causa ms importante por la que se debe dar tratamiento si
se detecta bigeminismo?
RESPUESTA
a.- Riesgo de hacer un fenmeno R/T, desencadenar una arritmia letal
b.- Riesgo de desarrollar taquicardia sinusal
c.- Riesgo de hacer un fenmeno R/T, desencadenar un bloqueo de
segundo grado
d.- Riesgo de desarrollar fibrilacin auricular

Pharmed Solutions Institute

PGINA 175

MANUAL DE TRABAJO DEL CURSO ENARM CMN SIGLO XXI


DISFUNCION ORGNICA MLTIPLE (DOM):
CIENCIAS BSICAS: Es la disminucin potencialmente reversible en la funcin de uno o ms rganos, que son incapaces de mantener la homeostasis sin
un sostn teraputico. El trmino disfuncin implica un proceso continuo y dinmico en la prdida de la funcin de un rgano, que va de menos a ms,
siendo la etapa final en la claudicacin de la funcin de dicho rgano lo que denominamos falla. El Sndrome de Disfuncin Orgnica Mltiple (SDOM) se
considera como un conjunto de sntomas y signos de patrn diverso que se relacionan en su patogenia, estn presentes por lo menos durante 24 a 48
horas y son causados por disfuncin orgnica, en grado variable, de dos o
ms sistemas fisiolgicos, con alteracin en la homeostasis del
organismo, y cuya recuperacin requiere mltiples medidas de soporte
avanzado. Esta definicin adems de permitir un diagnstico clnico
precoz, nos da la oportunidad de intervenir activa y rpidamente para
tratar de revertir el proceso que origin el dao. Asimismo debemos
definir el SDOM primario cuando ocurre una vez que cualquier sistema
orgnico falla, en forma evidente o sea como resultado directo de una
lesin conocida, por ejemplo, luego de un episodio de insuficiencia
respiratoria aguda por lesin pulmonar despus de un trauma tisular
global o contusin pulmonar (En la forma ms comn de SDOM, los
pulmones son los rganos predominantemente afectados y con
frecuencia el nico sistema implicado, hasta etapas tardas de la
enfermedad), la insuficiencia renal aguda por rabdomilisis y a la
coagulopata por politransfusin. El SDOM secundario es el que se
presenta en el contexto de una respuesta inflamatoria sistmica, o
consecuencia de la respuesta del husped a una agresin englobndose
en el Sndrome de Respuesta Inflamatoria Sistmica (SRIS), es el nexo que
media entre la injuria (infecciosa o no) y el SDOM. El SRIS y el SDMO son
la va final comn de muerte en pacientes severamente enfermos o lesionados, en los que la tecnologa avanzada existente en las UCI ha posibilitado su
resucitacin. SALUD PUBLICA: Las cifras de mortalidad para SIRA por s solo es de 40 al 50%, una vez que se agrega disfuncin orgnica sistmica, la
mortalidad se incrementa hasta el 90%. Sus complicaciones son la causa ms frecuente de mortalidad en UCI no coronarias ETIOLOGA: Inicialmente se
pens que ste cuadro era la expresin fatal de una sepsis incontrolada, sin embargo posteriormente se ha demostrado que puede ocurrir en ausencia
de infeccin. La infeccin no es una condicin indispensable para el desarrollo de FOM. Plantea la existencia de una activacin masiva de mediadores de
la inflamacin como consecuencia del dao tisular. La incidencia del SDOM en una poblacin de pacientes hetereognea mixta (patologas de tipo
mdico y quirrgico) vara entre 7 y 15%.En pacientes con trauma su incidencia puede ser hasta del 35%, y mucho menor en pacientes luego de ciruga
cardiaca electiva, alrededor del 3%. Es difcil encontrar un elemento causal simple para el SDMO, existiendo en la mayora de los pacientes varias causas
que se potencian: 1. Traumatismos multisistmicos graves. 2. Postoperatorio. 3. Inestabilidad hemodinmica. 4. Infecciones severas. 5. Pancreatitis
aguda. 6. Quemados. 7. Necesidad de ventilacin mecnica prolongada. 8. Hemorragia gastrointestinal. 9. Diseccin, ruptura o reparacin de aneurisma
artico. 10. Perforacin gastrointestinal. 11. Enfermedad inflamatoria intestinal. 12. Nutricin parenteral prolongada. 13. Ciruga valvular cardiaca. 14.
Transfusiones masivas. 15. Coagulacin intravascular diseminada. PATOGENIA: Se ha acumulado suficiente evidencia de la existencia de una
participacin activa de la cascada inflamatoria en la gnesis de cuadros de DOM. Es as como noxas de origen infeccioso no infeccioso son capaces de
iniciar el proceso inflamatorio, activando vas comunes que a manera de respuesta inespecfica, permiten al husped reaccionar frente a la injuria. Esta
respuesta inflamatoria se verifica bsicamente a cuatro grandes niveles: 1. Activacin del sistema Monocito Macrfago. 2. Activacin del sistema del
Complemento. 3. Activacin del sistema Retculo Endotelial. 4. Activacin de la Cascada de la Coagulacin. Sin embargo, todas estas vas estn
relacionadas entre s, pudiendo activarse y potenciarse unos a otros desde cualquier punto de esta red inflamatoria. Es el caso de pacientes
politraumatizados en donde la cascada se inicia por la activacin del sistema del Complemento y cascada de la Coagulacin, o en las vasculitis en donde
el dao inflamatorio endotelial sera el gatillador del proceso. Existe suficiente evidencia que avala la activa participacin de la respuesta inflamatoria en
la gnesis de la DOM, sin embargo todos los intentos por bloquear o modular esta respuesta ha fracasado. Luego del fenmeno inicial de up regulation
de citoquinas proinflamatorias (TNFa, IL-1, IL-2, IL-6, IL-8) o concomitantemente con l, se liberan especies anti-inflamatorias (receptores solubles de
TNFa, IL-1 ra e IL-10) en concentraciones 30 a 100.000 veces superiores que la de las molculas proinflamatorias. Esto podra ser responsable de la
disminucin de la activacin observada en Monocitos humanos ante la exposicin a infecciones de severidad creciente. Es as como el paradigma actual
de la respuesta inflamatoria presente en cuadros de SIRS, sepsis o MOD es la interaccin sistmica entre mediadores pro y anti inflamatorios que
determinarn en definitiva el dao celular la recuperacin. En el dao celular, ltimo elemento en el que radica la DOM, una de las teoras que mayor
peso tiene en la actualidad indicara que, como consecuencia de la activacin de la cascada inflamatoria, a nivel intracelular la activacin de mediadores
determina un stress oxidativo. Es este stress oxidativo el que es capaz de activar un complejo de mediadores intracelulares denominado "Complejo
Nuclear Factor kb", el cual est compuesto por tres sub unidades; una molcula pequea denominada p50, una mayor, la p65 y una sub unidad
inhibitoria. Como consecuencia del stress oxidativo este complejo es fosforilado, liberando la sub unidad inhibitoria y permitiendo al dmero p50-p65
libre, migrar al ncleo y unirse al DNA en los sitios que promueven la codificacin de citoquinas y otras sustancias proinflamatorias. Diversos trabajos han
demostrado que la administracin de antioxidantes y que la prevencin del stress oxidativo mitocondrial suprimen la activacin de Nuclear Factor kb.
DIAGNOSTICO: Clnico: Los registros de temperatura deben ser centrales o rectales. La fiebre se produce por el efecto de las citoquinas IL-1 y TNF
liberados por los macrfagos activados. Aunque la fiebre y los escalofros son tpicos, algunos pacientes que desarrollan infecciones bacterianas
sistmicas estn debilitados y no exhiben cambios llamativos (por ejemplo, escalofros) al comienzo de una infeccin. La causa de la hipotermia es menos
conocida y su presencia se asocia con mal pronstico. En los pacientes crticos, los mismos sntomas y signos caractersticos de sepsis pueden aparecer
durante la inflamacin sistmica de origen no infeccioso, por lo que el diagns co y la denicin de la severidad del proceso sp co pueden ser di ciles.
Durante los l mos aos se ha buscado un marcador clnico o de laboratorio capaz de iden car a los pacientes con sepsis, diferencindolos de los
portadores de otras patologas que tambin cursan con SIRS. Entre ellos podemos mencionar: Procalcitonina (PCT); posible marcador de SIRS en
respuesta a infeccin. Niveles mayores de 10 ng/mL (sepsis) y a veces superiores a 100 ng/ mL(shock sptico). Protena C reactiva (PCR); Es un marcador
sensible pero tardo y de baja especificidad. Recuento leucocitario; La leucocitosis se interpreta habitualmente como evidencia de posible infeccin, pero
no es un marcador sensible ni especfico. La activacin de la coagulacin es un hecho comn en el curso de la sepsis, con consumo de factores, aumento
del dmero D y sobre todo disminucin de la actividad de los anticoagulantes naturales. Varias citoquinas proinflamatorias, sobre todo los niveles
plasmticos de la IL-6 e IL-8, han mostrado correlacin con el pronstico en diversos estudios efectuados en pacientes crticos. Pero su determinacin
tiene varios inconvenientes: alto costo, la vida media de las citoquinas es muy corta y las concentraciones varan rpidamente por lo que es difcil su

CURSO ENARM CMN SIGLO XXI TEL: 36246001

Pharmed Solutions Institute

PGINA 176

MANUAL DE TRABAJO DEL CURSO ENARM CMN SIGLO XXI


interpretacin. No se sugiere su utilizacin en la prctica clnica. John Marshall crea un Score de Disfuncin Orgnica, en el cual considera por medio de
un mecanismo de puntuacin, la magnitud de la disfuncin de cada uno de los 6 sistemas orgnicos analizados, otorgndoles un valor entre 0 y 4 segn
la magnitud de la disfuncin. Este sistema de puntuacin se correlacion estrechamente con la mortalidad intra UCI tanto cuando se utilizaban los
valores obtenidos durante las primeras 24 horas desde la admisin como cuando se aplicaba durante la estada. Jean Louis Vincent y cols introducen la
versin Europea de los Scores de Disfuncin (SOFA). Los scores nos permiten hacer un pronstico objetivo sobre la evolucin del paciente. En la prctica
clnica, cuando tenemos ms de tres rganos en falla por ms de 48 horas, la mortalidad se eleva por sobre el 90%, debiendo plantearse seriamente y
con la familia la posibilidad de limitar el tratamiento en curso. En estos casos, tendrn ms posibilidades de sobrevida aquellos pacientes ms jvenes y
sin patologa debilitante previa. TRATAMIENTO: Hasta la fecha la terapia de la Disfuncin Orgnica contina siendo una terapia de soporte, una vez
eliminada la injuria inicial (drenaje de colecciones, terapia antibitica, remocin de tejidos necrticos, restablecimiento de una perfusin adecuada, etc.).
Existen algunos estudios recientes que mostraran el beneficio de terapias de hemofiltracin de alto flujo, como una forma de modular la respuesta
inflamatoria, removiendo aquellas sustancias que se encuentran en exceso. Todos los pacientes con choque sptico deben tener acceso venoso
adecuado para la reanimacin con volumen. Un catter venoso central tambin puede ser usado para monitorear la presin venosa central y evaluar el
estado del volumen intravascular. Una sonda vesical permanente debe ser usada para monitorear la produccin de orina, marcador de una adecuada
perfusin renal y el gasto cardaco. Los pacientes que desarrollen choque sptico requieren cateterismo cardaco derecho en la arteria pulmonar a travs
del catter de Swan-Ganz. Este catter proporciona una evaluacin precisa del estado de volumen de un paciente que est sptico. Mediante la
evaluacin de la funcin ventricular tanto derecha como izquierda, monitoreo de los cambios hemodinmicos, adems de brindar informacin
pronstica. La mayora de los pacientes que se encuentren spticos desarrollan dificultad respiratoria como una manifestacin del estado de choque. La
disfuncin pulmonar de la sepsis (SDRA) tambin puede ocurrir. Estos pacientes necesitan intubacin y ventilacin mecnica para la asistencia
respiratoria ptima.
CASO CLINICO
Se trata de paciente femenino de 24 aos de edad tercigesta con
cesrea y aborto previos, sin antecedentes de importancia, quien
ingresa al servicio de urgencias con embarazo de 36 semanas de
gestacin + Preeclampsia severa con cifras tensinales de hasta
190/120mmHg. Presenta adems actividad uterina irregular sin datos
de vasoespasmo ni datos de eminencia de eclampsia ni afectacin
heptica. Frecuencia cardiaca fetal 135 latidos por minuto. Se
hospitaliza y maneja con hidratacin, antihipertensivo perifrico,
calcioantagonista y sulfato de magnesio. Se solicitan laboratoriales.
Hemoglobina: 14.2, Hematocrito: 46.2, Plaquetas: 215,000, TP: 12.6,
TPT: 36.1, Glucosa: 88, Creatinina: 0.8, Acido rico: 5.2, Bilirrubina
Total: 0.75, Bilirrubina Directa: 0.16, Bilirrubina Indirecta: 0.59, Alanina
Amino Transferasa (TGP): 35, Aspartato Amino Transferasa (TGO): 20,
Deshidrogenasa Lctica (DHL): 350, Examen General De Orina (EGO):
Protenas 30mg. Se estabilizan cifras tensinales, motivo por el cual se
decide interrupcin del embarazo va abdominal. Se ingresa a
quirfano con hipotensin arterial de hasta 80/40, recuperndose con
carga de solucin cristaloide y bolo de efedrina. Se inicia ciruga y al
entrar a cavidad abdominal se encuentra hemoperitoneo de 600cc. Se
realiza cesrea Kerr, obteniendo producto nico vivo hipotnico. Se
ampla incisin y se realiza revisin heptica, encontrndose ruptura
heptica del lbulo derecho. Se prosigue para realizar
empaquetamiento. Paciente en choque hipovolmico, reanimada con
soluciones cristaloides coloides y tres paquetes globulares, as como
aminas (dopamina y dobutamina). Se administran esteroides a dosis de
choque. No se cuenta con plasma ni derivados plaquetarios, motivo
por el cual se traslada a tercer nivel al servicio de terapia intensiva con
sedacin y analgesia con midazolam y fentanil y relajacin con
vecuronio. Sin embargo, fallece a las 24 horas por falla orgnica
mltiple.
PREGUNTA
Cual de las siguientes observaciones relacionadas al DOM es ms
frecuente en la patologa de base?
RESPUESTA
a.- Disminucin de la fraccin de eyeccin.
b.- Disminucin de la relacin PaO2/FiO2.
c.- Ictericia, plaquetopenia, alteracin de las PFH.

CURSO ENARM CMN SIGLO XXI TEL: 36246001

d.- Ileo paralitico, ulcera por estrs, colecistitis aguda alitiasica.


CASO CLINICO
Joven de 16 aos, originario de Tampico, Tamaulipas, previamente
sano. Inici su enfermedad con ataque al estado general, mialgias,
calosfros y fiebre, presentando en pocas horas deterioro
multisistmico. Ingres a la UTI por cuadro de sepsis grave, choque
sptico, DOM (cardiovascular, hematolgica y respiratoria) y sndrome
de insuficiencia respiratoria aguda (SIRA), con ndice de presin arterial
de oxgeno/fraccin inspiratoria de oxgeno (PaO2/FiO2) < 100 mmHg,
Acute Physiology and Chronic Health Evaluation II Score (APACHE II) de
28 puntos y Sequential Organ Failure Assessment (SOFA) de 13 puntos.
Se implementa
PREGUNTA
Cul es la mortalidad para este paciente de acuerdo a los rganos y
sistemas afectados por DOM?
RESPESTA
a.- 20%
b.- 40%
c.- 60%
d.- 80%
PREGUNTA
Cul es la causa menos probable para desarrollo de DOM?
RESPUESTA
a.- Infeccin
b.- Arritmia
c.- Trauma
d.- Isquemia
PREGUNTA
Cul de los siguientes datos es menos probable que nos oriente a falla
cardiovascular?
RESPUESTA
a.- PAS menos 90mmHg
b.- PAM menor de 60mmHg
c.- lactato elevado
d.- PAD menos de 80mmHg

Pharmed Solutions Institute

PGINA 177

MANUAL DE TRABAJO DEL CURSO ENARM CMN SIGLO XXI


COAGULACIN INTRAVASCULAR DISEMINADA (CID):
CIENCIAS BSICAS: Sndrome patolgico adquirido, trombohemorrgico que se produce como resultado de una activacin excesiva del sistema de la
coagulacin y alteraciones de la fibrinlisis y conlleva el depsito masivo de fibrina en la microcirculacin y lesin orgnica. La CID se produce como
consecuencia de una activacin excesiva, del mecanismo hemosttico, de la incapacidad de los inhibidores fisiolgicos para neutralizar la coagulacin y
de inhibicin de la fibrinlisis mediada por el inhibidor del activador del plasminogeno de tipo endotelial (PAI-1). La formacin de trombos de fibrina y su
depsito en los pequeos vasos (microangiopatia trombotica) produce disfuncin multiorgnica, que consume plaquetas y factores de coagulaciones
consecuentemente desencadena una fibrinlisis secundaria. Otros trminos que suelen utilizarse para denominar este sndrome son: sndrome de
desfibrinacin, coagulopata por consumo y sndrome de activacin sistmica de la coagulacin. Frecuencia de CID en condiciones asociadas trauma
severo 50-70%, sepsis o infeccin severa 30-50%, feto muerto y retenido 50% (>5 sem), embolia de liquido amnitico 50%, abruptio placentae 50%,
HELLP 15%, neoplasias, anormalidades vasculares, circulacin extracorprea, falla heptica severa, shunt arterio-venoso, rechazo de trasplante.
PATOGENIA: La presencia de una condicin asociada a CID. Como la sepsis o el trauma, inducen la activacin del sistema de coagulacin a travs de la
liberacin de citoquinas, como IL-6, IL-1 y/o el factor de necrosis tu oral (TNF); integrados como parte de la respuesta inflamatoria e incrementando la
cantidad de factor tisular asociado a factor VII. La
liberacin o exposicin en alta concentracin de
sustancias procoagulantes, como el factor tisular en el
SNC. Como ocurre en el traumatismo de crneo, puede
desencadenar un trastorno coagulopatico local y/o
generalizarse en una CID. La activacin del sistema de
coagulacin, con exceso de trombina y el subsiguiente
deposito de fibrina en la microcirculacin, favorecen la
agregacin plaquetaria y el consumo de factores de
coagulacin. Los glbulos rojos quedan atrapados en las
mallas de fibrina, otros sufren ruptura mecnica al
pasar entre la malla de fibrina (responsable de la
esquistocitosis). La microtrombosis intravascular,
promovida en la micro-vasculatura parenquimatosa y sistmica, generan un aumento de plasmina, que degrada el fibringeno, la fibrina y otros factores
de la coagulacin; perpetundose el circulo vicioso de coagulacin- fibrinlisis. El funcionamiento suboptimo de los sistemas anticoagulantes naturales,
protena C o antitrombina, desbordados por el incremento de trombina y plasmina, permiten que estas circulen libremente por el torrente circulatorio.
El consumo de los factores de la coagulacin y plaquetas, junto a la hiperfibrinolisis favorecen la aparicin de hemorragias. FORMAS DE PRSENTACION:
A) segn el tiempo de instauracin y progresin: aguda y crnica. B) Por compromiso y extensin: localozada, sistmica. C) Por sus manifestaciones:
hemorrgica (reaccin hemoltica trasnfucional, leucemia promielocitica), trombtica (sndrome de Trousseau). DIAGNOSTICO: Clnico; hemorragia,
hemorragias cutneo mocosas generalizadas (epistaxis, hematuria, petequias, equimosis, gingivorragia, hemorragia digestiva, hemorragia en SNC),
hemorragia tras incisin quirrgica, heridas, catteres, o punciones vasculares,
trombosis, prpura fulminante, acrocianosis perifrica, gangrena de extremidades,
anemia microangiopatica, en SNC-delirio, coma, en pulmn-diestress, en rininsuficiencia renal aguda, necrosis cortical, en tubo digestivo-lceras duodenales.
La CID aguda se manifiesta como hemorragias, trombosis o ambas. Las hemorragias
suelen ser de presentacin aguda, en mltiples sitios, especialmente de puncin
venosa o arterial, y frecuentemente asociada a petequias y prpura. La CID crnica
se presenta de manera ms solapada, con menores manifestaciones clnicas y de
laboratorio, debe considerarse en paciente con patologas asociadas a esta entidad
(neoplasias, feto muerto retenido, aneurisma de aorta, hemangioma, trasplante,
HELLP, reacciones alrgicas), que presentan alteraciones leves de los parmetros de
hemostasia o que sangran excesivamente frente a intervenciones quirrgicas. El
diagnostico de CID se fundamenta en el cuadro clnico: 1. Identificacin de la
entidad causal de CID. 2. Valoracin de los sangrados y/o trombosis. 3. Alteraciones coagul-mtrica de laboratorio. Pruebas de hemostasia bsicas para
diagnostico de CID: 1. Recuento de plaquetas, un recuento bajo o en descenso progresivo, es una determinacin sensible aunque no especifica. El 98%
de casos de CID presenta plaquetopenia y el 50% se encuentra por debajo de 50,000 mm 3. 2. Producto de degradacin del fibringeno (PDF) y dmero D
(DD), frecuentemente estn elevados como manifestacin de hiperfibrinolisis secundario al incremento de plasmina; son sensibles (95%) aunque poco
especficos ya que otras condiciones como el trauma, ciruga reciente o tromboembolismo venoso pueden incrementarlos, al igual que el deterioro de la
funcin renal o heptica, por una dos induccin de su eliminacin. Aun no se han estandarizado los puntos de corte considerados incremento moderado
y alto necesarios para el score diagnostico de CID. 3. Tiempo de pro trombina (TP) y tiempo de tromboplastina parcial (TTP), se encuentran prolongados
en el 50-60% de los casos, aunque cerca de la mitad de los pacientes pueden presentar tiempos normales o incluso acortados por la presencia de
factores de coagulacin activados como la trombina o el factor X. 4. Fibringeno, su descenso es poco sensible (28%), se encuentra sobre todo en
estadios avanzados de la enfermedad. 5. Frotis de sangre perifrica, la presencia de esquistocitosis, suele encontrarse en el 50% de los casos. No existe
un mtodo diagnostico estndar pero se han desarrollado scores diagnostico. La sensibilidad del score de CID manifiesta la Sociedad Internacional de
Hemostasia y Trombosis (ISTH) es de 91% y tiene una especificidad del 97%. TRATAMIENTO: La clave es la correccin de la enfermedad desencadenante
y dar soporte. No se debe posponer la medida teraputica requerida, por intentar corregir un parmetro de coagulacin alterado. Hemoderivados; no
estn indicados para corregir parmetros de laboratorio, pero si en presencia de sangrado activo, previo a intervenciones invasivas o en aquellos con alto
riesgo de sangrado. Plasma (15 o 30ml/kg), plaquetas (una unidad de plaquetas cada 10 kg de peso en pacientes sangrando con recuentos <50,000
mm3), cri precipitados (cuando fibringeno esta < 1g/l, sin respuesta a plasma), concentrado de factores (el concentrado de complejo protrombnico,
solo tiene factores vitamina K dependientes y carece de otros factores como el factor V, no pudiendo corregir todos los factores deficientes en la CID). El
factor VII activado recombinante se ha utilizado en casos de sangrado que compromete la vida (su eficacia y seguridad aun no est definida).
Indicaciones de heparina en CID, debido a que la CID est caracterizada por una importante activacin de la coagulacin, el tratamiento anticoagulante
podra ser racional. No hay ensayos clnicos randomizados que demuestren que la heparina mejora los resultados clnicos y en la CID aguda podra
agravar hemorragias.

CURSO ENARM CMN SIGLO XXI TEL: 36246001

Pharmed Solutions Institute

PGINA 178

MANUAL DE TRABAJO DEL CURSO ENARM CMN SIGLO XXI


CASO CLINICO CID
Varn de 66 aos acude a urgencias por clnica de disnea a mnimos
esfuerzos de 10 das de evolucin y fiebre de hasta 39C. Niega tos y
expectoracin. Inici tratamiento antibitico de forma ambulatoria sin
presentar mejora. A su llegada presenta insuficiencia respiratoria
grave que requiere FiO2 elevadas. Presenta rpida progresin de la
insuficiencia respiratoria y a las 24h precisa intubacin orotraqueal y
ventilacin mecnica. La radiografa de trax inicial muestra un
infiltrado alveolar en la base izquierda con aparicin de infiltrados
pulmonares bilaterales en controles posteriores. Se realiza
cateterizacin de la arteria pulmonar que descarta fallo cardaco, con
presin capilar pulmonar inferior a 18mmHg, y que muestra signos de
hipertensin pulmonar moderada. Analticamente, destaca la aparicin
de coagulacin intravascular diseminada (D-dmero mayor a
20.000ng/ml, trombocitopenia mxima de 47.000/l y tiempo de

CURSO ENARM CMN SIGLO XXI TEL: 36246001

protrombina mximo de 1,58 ratio), fracaso renal agudo con cifras de


creatinina de hasta 2,33mg/dl y elevacin de la lactato deshidrogenasa
(LDH) con valores mximos de 2.305U/l.
PREGUNTA
Cual de los siguientes antecedentes esta mas relacionado con el
estado del paciente?
RESPUESTA
a.- Diabetes mellitus.
b.- Hipertension arterial.
c.- Cardiopatia isqumica.
d.- Enfisema pulmonar.
PREGUNTA

Pharmed Solutions Institute

PGINA 179

MANUAL DE TRABAJO DEL CURSO ENARM CMN SIGLO XXI


ALERGIA Y ANAFILAXIA:
CIENCIAS BASICAS: Reaccin de hipersensibilidad generalizada o sistmica, grave y que amenaza la vida, consideraremos que la anafilaxia es una
reaccin alrgica grave de instauracin rpida y potencialmente mortal. Desde el punto de vista clnico se trata de un sndrome complejo,
desencadenado por mecanismos inmunolgicos o no, con aparicin de sntomas y signos sugestivos de liberacin generalizada de mediadores de
mastocitos y basofilos, tanto a nivel cutneo, como en otros rganos. Las causas ms frecuentes son medicamentos (antibiticos betalactamicos, AINES)
y medios diagnostico (46-62%), alimentos (22-24%), picaduras de insectos (8-14%), factores fsicos (3-4%), otros (incluye ltex 7%), idioptica (3-5%). La
importancia relativa de cada uno de ellos vara considerablemente en funcin de la edad, as los alimentos (huevo, leche, frutos secos, pescado y
marisco) son la causa ms comn en nios y los frmacos son ms frecuentes en adultos. SALUD PUBLICA: Cifras de incidencia entre 3,2-30 por 100,000
personas al ao, con una mortalidad entre 0,05-2% del total de las reacciones. En el Shock anafilctico la incidencia vara entre 3,2-10 por 100,000
personas /ao y al menos 1 % es mortal. DIAGNOSTICO: Sospecha clnica; cuando aparece de una manera aguda (en minutos a pocas horas), un
sndrome rpidamente progresivo que afecta a la piel y/o mucosas y que se acompaa de compromiso respiratorio y/o circulatorio. Como las mayora
de las anafilaxias cursan con sntomas cutneos (>80%), con este criterio al menos un 80% de las anafilaxias serian identificadas. Sin embargo existen
presentaciones menos tpicas, como las que presentan exclusivamente hipotensin. Tambin se ha descrito que las manifestaciones digestivas se asocian
con una mayor gravedad. Por ltimo es importante tener en cuenta que la concurrencia de una exposicin a un alrgeno potencial o conocido para el
paciente apoya el diagnostico. Por ello se han establecido criterios, con ellos se espera identificar ms del 95% de las anafilaxias. Cuando existe afeccin
cardiovascular con hipotensin se habla de shock anafilctico. La dificultad en el diagnostico estriba en que no hay signos o sntomas patognomnicos; lo
que s es tpico es la rpida progresin en la gravedad e intensidad de los sntomas. EVALUACION DE GRAVEDAD: Se relaciona con la rapidez en la
progresin de los sntomas, con el tipo de antgeno, va de entrada y rganos afectados. Factores relacionados como edad avanzada, presencia de
patologa respiratoria o cardiovascular asociada, tratamiento con IECAS o betabloqueantes o una mastocitosis de base, se han asociado con reacciones
graves y mayor mortalidad, por tanto es fundamental la evaluacin ABCD. Las anafilaxias graves vienen definidas por la presencia de cianosis, saturacin
O2 <92%, hipotensin, confusin, hipotona, prdida de conciencia o incontinencia. Las anafilaxias moderadas presentan signos o sntomas que sugieren
afeccin respiratoria, cardiovascular o GI, como disnea, estridor, sibilancias, nauseas, vmitos, mareo, sudoracin, opresin torcica, sensacin de
garganta ocupada o dolor abdominal. Las manifestaciones cutneas (eritema, urticaria, angioedema) no se consideran criterios de gravedad.
Laboratorio: Los niveles plasmticos de histamina (se metaboliza rpidamente) y de triptasa total, esta es la prueba mas til para el diagnostico de
anafilaxia, puede elevarse en muestras obtenidas entre los 15 y 180 min del comienzo de los sntomas. Se aconseja un mnimo de 3 muestras seriadas, la
concentracin normal en suero es inferior a 13.5 /l, una elevacin de al menos 2 veces lo normal, es sugestivo de anafilaxia. TRATAMIENTO: 1.
Valoracin inmediata del sistema cardiovascular y respiratorio, si el paciente est en paro las medidas de resucitacin cardiopulmonar deber ser de
extrema urgencia, ante la inminencia del shock colocar al paciente recostado con los pies elevados. 2. La primera droga a administrar es la Adrenalina IM
es la va de eleccin para el tratamiento inicial, obtiene unas concentraciones plasmticas mas rpidas y elevadas que la va SC, presenta un mayor
margen de seguridad que IV. En solucin acuosa y una dilucin de 1:1000 y a una dosis de 0.01 ml/kg. Subcutnea dosis mxima de 0.3 ml. cuando la
anafilaxia fue secundaria a una inyeccin o a un piquete siempre y cuando no sea en la cabeza o cuello deber de recibir una segunda administracin e
0.01 ml/ kg. sin pasar de 0.3 ml en la zona del piquete, esto reduce de una manera considerable la absorcin el antgeno. Esta dosis de Adrenalina puede
ser repetida cada 15 o 20 minutos dependiendo de la evolucin mxima de 3 dosis. La administracin IV de adrenalina tiene un alto riesgo de arritmias,
sin embargo Cuando la evolucin es mala y hay shock evidente o Colapso vascular, podr administrarse a una dosis de 0.1 ml de adrenalina 1:1000
diluida en 10 ml de solucin fisiolgica (para quedar en una dilucin de 1:100,000) y administrar IV durante un periodo de 15 a 20 minutos. Considrese
esta medida cuando el paciente est en riesgo de perder la vida. 3. Torniquete En caso de que la reaccin de anafilaxia sea debido a un piquete de
insecto o por una inyeccin se utilizara un torniquete cercano al sitio del piquete o inyeccin, este podr liberarse 1-2 minutos cada 10 minutos. 4.
Oxigeno siempre que sea posible hay que administra oxigeno a los pacientes con cianosis, disnea, sibilancias por espasmo bronquial, a una dosis de 3 a 5
lts. por minuto. 5. Antihistamnicos el Benadryl (difenhidramina) a una dosis de 1-2 mg por kilo deber ser administrada IV lentamente en un tiempo no
menor de 15 minutos , puede ser administrada IM o hasta va oral, otra opcin es el clorfeniramina IM, IV u oral a dosis de 0.35mgxkgx24 hs el
antihistamnico deber de continuarse por va oral cada 6 hs para prevenir recurrencia de la reaccin principalmente de urticaria y angioedema,
principalmente para administracin oral los antihistamnicos de la nueva generacin pueden ser muy buena opcin y continuarlos cuando menos por un
lapso de una semana. Corticosteroides .- La administracin temprana despus de la adrenalina y del antihistamnico es de mucha ayuda para prevenir la
recurrencia de los sntomas, y la fase tarda, la dosis inicial de la hidrocortisona es de 7-10 mg/kg. y posteriormente 5 mg/kg. cada 6 hs o
Metilprednisolona 2-3 mgx kg. y no debern ser descontinuados cuando menos por 3 a 4 das. Cuando a pesar de las medidas enunciadas el paciente no
mejora, persiste hipotenso o con problemas de dificultad respiratoria, deber de internarse inmediatamente y de preferencia en una unidad de cuidados
intensivos para la administracin de: 1. Lquidos parenterales de solucin glucosada- fisiolgica inicialmente a una dosis de 30 ml por kilo en una carga
para la primera hora, deber de repetirse de acuerdo a su evolucin para mantener una presin por encima de 50 mmHg. 2. Vasopresores: cuando a
pesar de las soluciones y manejo persiste hipotenso deber de manejarse con vasopresores bitartrato de norepinefrina diluir en 250 ml (glucosado 5% y
fisiologico ) 1 ml y pasar 0.5 ml /minuto. Dopamina 15 mcg /kg/minuto IV, preferible sobre la norepinefrina en los pacientes con insuficiencia cardiaca.
Broncodilatadores.- Regularmente la misma adrenalina alivia la presencia de broncoespasmo sin embargo cuando est presente a pesar de esta, La
nebulizacion de salbutamol 0.5 ml de la solucin al 5% ,en 2 ml de agua y dos de fisiolgico para nebulizar durante 8-10 minutos y repetir cada hora las
primeras tres horas es de muchisimo beneficio para mejorar la permeabilidad de las vas areas por broncodilatacion. En caso de que el broncoespasmo
persista deber de administrarse AMINOFILINA a una dosis de 5-7 mg por kilo en solucin cuando menos 30 ml para pasar en 30 minutos y
posteriormente una dosis de 5 mg por kilo para administrar cada 6 hrs durante 30 minutos. Intubacin y traqueostomia.- Cuando a pesar del manejo el
paciente no mejora y no se puede restablecer la permeabilidad de las vas respiratorias por el edema deber de intentarse la Intubacin endotraqueal, Y
NUNCA RETRASAR LA DESICION DE ESTE PROCEDIMIENTO SI SE REQUIERE para el caso de la traqueostomia deber de ser realizada por personal
calificado y en sala de operaciones de ser posible. Terapia de soporte.- Despus de que el paciente ha sido estabilizado, deber de mantenerse la terapia
de mantenimiento con lquidos y drogas por el tiempo en que se requiera y las funciones vitales se hayan reinstalado. Esto puede ser de pocas horas a
varios das, en los caso ms severos la muerte puede ocurrir dentro de los primeros treinta minutos, con el manejo usualmente la recuperacin es
completa, a menos que haya sufrido durante la crisis un infarto al miocardio o dao al SNC. Un hecho es que en estos casos las reacciones subsecuentes
con el mismo antgeno pueden dar lugar a reacciones mas severas y mas rpidas, por lo que un pilar muy importante del manejo de la anafilaxis es la
Prevencin.
CASO CLINICO
Se trata de una paciente de sexo femenino de 26 aos de edad, que
consult por un cuadro clnico de cinco meses de evolucin
consistente en ppulas eritematosas, prurito intenso y angioedema

CURSO ENARM CMN SIGLO XXI TEL: 36246001

asociado al contacto con el fro y, en algunas ocasiones, asociado a


disnea. Los sntomas se presentaban con ambiente fro o con agua fra
y sucedan en el rea expuesta. Cuando se baaba en piscina o con
agua fra, la reaccin era generalizada; cuando lavaba platos o se

Pharmed Solutions Institute

PGINA 180

MANUAL DE TRABAJO DEL CURSO ENARM CMN SIGLO XXI


lavaba las manos, la reaccin era local. Cuando ingera bebidas fras,
no presentaba edema en lengua ni disfagia, pero s reaccin en las
manos al sostener la botella o el vaso fro.
PREGUNTA
Cual es el tratamiento farmacologico sistmico mas adecuado?
RESPUESTA
a.- Hidroxicina.
b.- Loratadina.
c.- Prednisona.
d.- Clorfenamina.
CASO CLINICO ANAFILAXIS
Se trata de una mujer de 77 aos, obesa, hipertensa y diabtica no
insuln-dependiente. Haba sido intervenida quirrgicamente en varias
ocasiones sin sufrir accidentes anestsicos, y no era alrgica conocida a
frmacos. Tras la induccin anestsica aparecio inmediatamente un
broncoespasmo que dificultaba la ventilacin mecnica, (la gasometra
arterial durante el episodio agudo mostr acidosis respiratoria severa
con pH: 7,12; pCO2: 71 mmHg; pO2: 63 mmHg; HCO3: 23 mmol/l; EB: 7,7 mmol/l; SaO2 83% con FiO2 al 100% ), hipotensin extrema
(tensin arterial sistlica 50 mmHg) y finalmente disociacin
electromecnica que requiri resucitacin cardiopulmonar. Durante el
episodio aparecieron eritemas en pliegues, habones diseminados y
enrojecimiento facial, y la monitorizacin cardaca mostr una
bradicardia a 30 latidos por minuto (lpm), y una elevacin de ST
localizada en la cara inferior y anterolateral baja en el
electrocardiograma (ECG).

PREGUNTA
Cul es la reaccin de hipersensibilidad ms probable en este caso?
RESPUESTA
a.- Tipo I
b.- Tipo II
c.- Tipo III
d.- Tipo IV
PREGUNTA
Cul sera la conducta teraputica ms adecuada para este paciente?
RESPUESTA
a.- Difenhidramina
b.- Prednisona
c.- Clorfenamina
d.- Adrenalina
CASO CLINICO
Paciente de 60 aos con paresia progresiva del miembro superior
izquierdo desde hace 35 das, asociada a crisis convulsiva. El cuadro
sugiere la posibilidad de una lesin neoplsica, por la lenta y
progresiva evolucin y la asociacin de una crisis convulsiva del adulto
sin antecedentes de epilepsia. Por lo que el estudio indicado es RM
enceflica con contraste. Tiene como antecdedentes hipertenso de 10
aos de evolucin en tratamiento con captopril. Asmtico en control
actualmente. Posterior al paso de medio de contraste, inicia con rubor,
urticaria, nausea y vomito en una ocasin, adems refiere opresin
torcica.

PREGUNTA
Cual es el tratamiento farmacolgico inicial para manejar la anafilaxia
presentada?
RESPUESTA
a.- Metilprednisona.
b.- Prednisona.
c.- Adrenalina.
d.- Dexametazona.

PREGUNTA
Cul es el factor de riesgo menos probable para desarrollar una
reaccin al medio de contraste?
RESPUESTA
a.- Asma
b.- Alergia a medicamentos
c.- Atopia
d.- Alergia alimentaria

CASO CLINICO
Paciente de 8 aos con antecedente de Eczema durante la infancia,
superado en la edad escolar. Rinitis y conjuntivitis durante la estacin
polnica, desde la edad escolar, antecedentes familiares interrogados y
negados. La madre refiere se encontraban en una fiesta que estaba
comiendo una palanqueta de cacahuate e inicio repentinamente con
sntomas, Padecimiento actual: obstruccin nasal y leve edema
larngeo, combinado con ojos irritados y pruriginosos, reaccin en piel
maculopapular prurginosa.

PREGUNTA
Qu medicamento es necesario suspender, por aumentar el riesgo de
nefrotoxicidad de los medios de contraste?
RESPUESTA
a.- Metformina
b.- Captopril
c.- Benzafibrato
d.- Aspirina

PREGUNTA
Cul es la inmunoglobulina ms importante en la gnesis de la
alergia?
RESPUESTA
a.- IgA
b.- IgM
c.- IgE
d.- IgD

CURSO ENARM CMN SIGLO XXI TEL: 36246001

PREGUNTA
Cul es la premedicacin ms probable administrar a este paciente?
RESPUESTA
a.- Hidrocortisona, adrenalina
b.- Prednisona, hidrocortisona
c.- Metilprednisolona, difenhidramina
d.- Hidrocortisona, ranitidina

Pharmed Solutions Institute

PGINA 181

MANUAL DE TRABAJO DEL CURSO ENARM CMN SIGLO XXI


ENFERMEDAD TROMBOEMBOLICA VENOSA = TVP (Trombosis venosa profunda) + TEP (Tromboembolia pulmonar aguda)
CIENCIAS BASICAS: Triada de Virchow, base para TVP y TEP=Lesin o trauma endotelial, disminucin del flujo sanguneos o estasis, hipercoagulabilidad o
estado tromboflico. Se requiere un ambiente reolgico propicio = velocidad menor de retorno venoso y corriente local con rotaciones elipsoidales y casi
estticas. Factores predisponentes: Inmovilizacin prolongada 2-4 das (25%), antecedente previo de TVP (19%), neoplasia maligna (17%), ciruga
(ortopdica de rodilla 70%, abdominal) o traumatismo + miembros inferiores y huesos largos, en los ltimos 3 meses (13%), uso de anticonceptivos
orales (4%), IAM, ICC, edad > 40aos, embarazo, parto y puerperio, obesidad, venas varicosas, viajes largos. Dentro de los estados hipercoagulables;
estn las trombofilias primarias hereditarias (deficiencia de antitrombina, protena C y S, mutacin del factor V de Leiden) y adquiridas (sndrome
antifosfolipido secundario a LES). SALUD PUBLICA: Afecta aprox. a 2 millones de personas. Incidencia de 145 por 100,000. La TVP va de un caso por cada
10,000 de adultos jvenes y 1 por cada 100 de adultos mayores. En personas de 65 a 69 aos es de 1.8 habitantes por cada 1,000 y aumenta a 3.1 en el
grupoentre 85-89 aos. TEP Causa frecuente de mortalidad en pacientes hospitalizados (5-10%). Toda TEP, tiene su origen en una TVP o por lo menos el
80%. TEP masiva 0.3%. PATOGENIA: La TVP Se inicia con mayor frecuencia en las piernas (venas gemelares y soleas). SI hay dao a nivel endotelial se
activa la cascada de coagulacin, el trombo se asienta sobre el seno valvular, las corrientes de flujo cambian localmente, disminuye el calibre de la vena,
se produce estenosis y mayor fuerza de rozamiento, que permite el crecimiento del trombo y posteriormente adherencia, al disminuir el retorno venoso
de la extremidad, hay acumulacin de lquido y aumento de la presin hidrosttica, lo que lleva a Edema, en la zona donde ocurre la oclusin aumenta la
extraccin de O2, de los tejidos lo que produce cianosis. Lo peligroso no es que se asiente, si no que siga creciendo y se fragmente y produzca embolismo
que llega a obstruir la circulacin arterial pulmonar (TEP) en diferentes grados. En la TEP en estadios tempranos puede tener un GC normal o levemente
elevado, hay hipoxemia (disminucin de la relacin ventilacin/ perfusin), esto aumenta la precarga, generando un gradiente de presin que llevara a
isquemia subendocardica y finalmente falla cardiaca derecha con la consiguiente disminucin de la distensibilidad del VI. En la TEP masiva no hay
mecanismos compensatorios, manifestndose clnicamente como taquicardia sinusal y finalmente como hipotensin arterial sistmica, que llevara a la
muerte en poco tiempo. DIAGNOSTICO: TVP: No puede ser afirmado, ni excluido por clnica, la mayora asintomtico. En parte distal a sitio de oclusin;
edema, dolor, calor local, cianosis (+ en regin plantar), circulacin colateral, cordn venoso palpable. En caso severo flegmasa cerulea dolens (edema
severo, cianosis marcada, flictenas y puede llegar a gangrena por compromiso de retorno linftico). Signo d Homans: dolor en parte alta de pantorrilla al
realizar dorsiflexion del tobillo. Maniobra de Olow: apretar pantorrilla posterior en busca de dolor. TEP: El dx. requerir de un alto grado de sospecha
clnica, basndose en; a) uno o mas factores de riesgo, b) comorbilidades (ej. cncer), c) sntomas y signos clnicos, d) hallazgos paraclinicos, e)
demostracin objetiva de hipoxemia y trombosis. Sintomatologa como: disnea sbita sostenida, taquicardia, hipotensin arterial sistlica (<100),
sncope, dolor precordial o colapso cardiorrespiratorio, se ha correlacionado con obstruccin mayor a 30%; otros sntomas diaforesis, tos hemoptisis,
edema de MsPs. ECG; proporciona datos inespecficos; taquicardia sinusal, eje de QRS >90, T negativa profundas en V1-V4. Gasometra; hipoxemia y
alcalosis respiratoria aguda. Dmero D (DD, es un producto de degradacin de los puentes de fibrina, que se libera a la circulacin), se eleva cuando existe
un proceso trombotico, si el valor es <500ng/ml y sospecha clnica baja permite excluir el dx. de TVP. Valores elevados de DD nunca sern suficientes
para dx. de evento tromboembolco, la ausencia de elevacin del DD, prcticamente descartara un evento trmboembolico. Ecodopler; primera
eleccin en sospecha de TVP proximales. Flebografa; Gold Standart para dx. de TVP para todo territorio venoso ( contraindicado en insf. renal,
hipertiroidismo, embarazo, alergia a medio). En TEP el gammagrama ventiltorio-perfusorio, es el estudio inicial. La angiografa pulmonar selectiva
actualmente se mantiene como "estndar de or" para el dx. definitivo de TEP. Escala de Wells; estratificacin de riesgo para trombosis proximales; 1.
Cncer +1; 2. Parlisis o inmovilizavion de miembros inf. +1. 3. Encamamiento > 3 das +1. 4. Dolor localizado +1. Aumento de tamao de miembros
inferiores +1. Edema en pitting unilateral +1. Dilatacin venosa superficial +1. Otros dx. posibles -2. Wells >2 Alta probabilidaf; <2 Baja probabilidad.
TRATAMIENTO: HBPM actualmente es la piedra angular y estndar de oro para tx. inicial, tiene como mecanismo de accin la unin con la antitrombina
III e inhibiendo indirectamente a la trombina, previniendo la formacin de trombo adicional y facilitando la fibrinlisis endgena. Vida media 12hrs. SC.
No requiere monitoreo, puede usarse en tx. ambulatorio, reduce mortalidad. Se puede usar durante embarazo porque no atraviesa la placenta.
Enoxaparina 1mg/kg/12hrs. HNF (heparina no fraccionada), precisa control con TTpa, riesgo de trombocitopenia y osteoporosis. Iniciar de forma
simultanea con anticoagulacin y antagonistas de vitamina k VO , ya que los anticoagulantes orales tienen efecto procoagulante los primeros 3 das,
anticoagulacin oral (acenocumarol) control con INR; mantener de 2-3, primero, segundo y tercer da acenocumarol mas enoxaparina, en cuarto da
podemos retirar la enoxaparina, el tiempo de tratamiento con acenocumarinicos, depender del tipo d evento y la coexistencia de factores de riesgo. La
terapia fibrinolitica con estreptocinasa, urocinasa o factor activador de plasminogeno, se reserva e indica solo en TEP masiva y datos clnicos de
inestabilidad hemodinmica. PREVENCION: Paciente con factor de riesgo transitorio: anticoagular por 3 meses (ej. embarazo). Paciente con TVP
idioptico o estado protrombotico: anticoagular 6-12 meses. Paciente con TVP recurrente o persistencia de algn factor de riesgo: anticoagular
indefinidamente. En qx. ortopdica anticoagular por 30 das, en qx. gnral. tratamiento hasta anticoagulacin completa. Profilaxis: 1.- Riesgo bajo; qx. sin
complicaciones en <40 aos o qx. menos de 30 min. en >40 aos sin factores de riesgo en ambos casos. Recomendaciones: no tx. antitrombotico, solo
deambulacin precoz e hidratacin adecuada. 2.- Riesgo moderado: qx. gnral en > 40a y duracin > 30 min. o <40 aos n tx. con anticonceptivos.
Incidencia de TVP distal 10-40%; proximal 2-10%; TEP mortal 0.1-0.7%. Recomendaciones: HBPM c/24 hrs o HNF 5000 U c/8-12 hrs SC, 2 hrs antes de la
intervencin. Si puncin peridural, aplicar 8 hrs antes, por riesgo de hematoma peridural que puede expandirse y causar parapleja. 3.- Riesgo elevado:
qx. gnral o urolgica n >40aos con historia de TVP o TEP o en qx. abdominal o pelvica para tx. de neoplasia o qx. ortopdica mayor en miembros
inferiores. Incidencia de TVP distal 40-80%; proximal 10-20%; TEP mortal 1-5%. Recomendaciones: Prtesis de cadera o rodilla, HBPM c/12 hrs SC.
profilctico cada 12 hrs. teraputica cada 24 hrs. PRONOSTICO: Depender de la sospecha clnica, la instalacin de la profilaxis, y recordar que una TVP
hasta en 80%, precede a una TEP
CASO CLINICO
Se trata de paciente masculino de 62 aos de edad el cual acude a
consulta debido a la presencia de dolor en miembros inferiores,
generando dificultad para caminar. Cuenta con antecedentes de
tabaquismo positivo hasta la actualidad refiere que con menor
intensidad que antes comenzando a los 20 aos de edad, hace dos
aos ha presentado leve mareos y en una ocasin fue internado por
dificultad para movilizar su cuerpo, pero se recupero sin secuelas
aparentes.
PREGUNTA
Considerando la clasificacin de Rutherford, cual es la categora que
presenta el paciente?

CURSO ENARM CMN SIGLO XXI TEL: 36246001

RESPUESTA
a.- 2.
b.- 3.
c.- 1.
d.- 4.
PREGUNTA
Considerando las manifestaciones clnicas del paciente, cual es la
causa menos probable de este caso?
RESPUESTA
a.- Compresin de una raz nerviosa.
b.- Proceso artrtico inflamatorio.
c.- Sindrome compartamental crnico.

Pharmed Solutions Institute

PGINA 182

MANUAL DE TRABAJO DEL CURSO ENARM CMN SIGLO XXI


d.- Insuficiencia venosa.
PREGUNTA
Cual es la conducta a seguir mas adecuada para establecer una
teraputica adecuada en el paciente para identificar factores de riesgo
modificables?
RESPUESTA
a.- Buscar niveles de glucosa, colesterol y triglicrido.
b.- Suspensin de tabaquismo absoluta.
c.- Holter para identificar origen cardiolgico.
d.- Ejercicio secuencial programado.
CASO CLINICO TEV
Acude a consulta paciente femenino de 34 aos de edad, originaria de
yucatan la cual tiene como antecedentes de importancia 7 semanas de
posparto y obesidad grado II, viajo mas de 12 horas por carretera para
visitar a familiares, solicita atencin debido a presencia de dolor e
inflamacin de miembro plvico derecho, a la exploracin se observa
dicha extremidad con inflamacin, hipersensibilidad y caliente al tacto,
con signo de Hoffman positivo, sus signos vitales son estables TA
115/75 mmHg, FC 81, FR 17, temperatura 37 C, se realiza doppler
apoyando el diagnostico de trombosis venosa, se ingresa para
tratamiento.
PREGUNTA
Cul es el manejo farmacolgico ms adecuado para esta paciente?
RESPUESTA
a.- Heparina IV
b.- Enoxoparina.
c.- Warfarina.
d.- Acecumarol.
PREGUNTA
Cual es la complicacin mas importante que debe evitarse en el
paciente?
RESPUESTA
a.- Tromboembolia pulmonar aguda.
b.- Evento embolico fatal subsecuente.
c.- Sindrome compartamental.
d.- ECV.
PREGUNTA
La paciente fue ingresada para tratamiento y realizacin de estudios
mas especficos, se colocaron medias compresivas, sin embargo luego
de tres das de estancia inicia con eritema en miembro plvico derecho
acompaado de hipertermia local, edema y dolor, se toman tiempos
de coagulacin, pero 3 horas despus antes de tener los tiempos de
coagulacin, presenta disnea sbita, tos y ansiedad intensa a la
exploracin se ausculta cuarto ruido cardiaco del lado derecho,
cianosis y sincope.
PREGUNTA
Considerando las reglas predictivas de Wells para trombosis venosa
profunda, que calificacin presenta el paciente en este momento?
RESPUESTA
a.- 2.
b.- 3.
c.- 4.
d.- 1.
CASO CLINICO
Se trata de paciente femenino de 38 aos de edad con antecedentes
de importancia para el padecimiento actual, usuaria de anticonceptivo
orales desde hace 8 aos, tabaquismo positivo desde los 18 aos,
actualmente con IMC 32. Acude a urgencias refiriendo que hace 24
horas regreso de un viaje de 12 horas de camino, el cual realizo en
auto ella era la conductora, refiere que comenz con dolor en las
pantorrillas de predominio izquierdo, con leve dificultad para caminar,

CURSO ENARM CMN SIGLO XXI TEL: 36246001

posteriormente se presenta malestar generalizado, dificultad para


respirar motivo por el cual se traslada al hospital, a la exploracin fsica
se observa diafortica con dolor torcico, disnea, sus constantes vitales
son 95/60 mmHg, FC de 115 lpm, FC de 30 rpm, SaO2 de 90%, campos
pulmonares si datos por agregar, se realiza radiografia de torax, y
medicin de gases arteriales siendo anormales.
PREGUNTA
Cual es el manejo mas apropiado para el caso?
RESPUESTA
a.- Enoxaparina.
b.- Heparina.
c.- Warfarina.
d.- Tecneplasa
CASO CLINICO
Paciente masculino de 68 aos de edad, ocupacin Ingeniero civil
retirado, casado con lugar de nacimiento y residencia DF. Con
antecedente de IAM trombolizado y posteriormente revascularizado,
hipotiroidismo (2011) en tratamiento con levotiroxina 50 mcgs /da;
Trombosis venosa profunda recurrente en miembros plvicos, Cncer
de pncreas diagnosticado en Julio 2011 evidenciado por TC que
evidenci una lesin en el cuerpo de pncreas con presencia de mets,
cerebrales, hepticas y sacro, infiltracin a tronco celaco y arteria
heptica y esplnica. Paciente que inicia su padecimiento actual hace
10 das al presentar aumento de volumen de pie derecho acompaado
de eritema y dolor al apoyar, present cambios en la coloracin del pie
derecho, debido a la persistencia de la sintomatologa acude a
urgencias de este instituto donde lo encontraron con FC 102x, TA:
71/57, temp 35.5, Sat 90% a la exploracin fsica se encontraron
silbilancias difusas, en abdomen se encuentra tumoracin palpable en
epigastrio dolorosa a la movilizacin miembros superiores integros con
llenado capilar de 2; miembro inferior derecho con edema +++ eritema
y aumento de volumen de pie derecho, con llenado capilar de 5.
Dentro de sus laboratorios de ingreso destaca Hb 9.4 VGM91.8
HCM30.5 Leucos 16.9, neutrfilos 85.9 plaq 50,000, cr 2.85 BUN 43.7
INR1.8 TTP36.2 Ego con leucos 13.5 Hb positivo Bacterias 12.6 Nitritos
negativos VSG 53 PCR29.84. Se realiz USG doppler de miembro
plvico derecho que evidencio una trombosis femoral aguda y una
trombosis crnica de la safena magna,
PREGUNTA
Cul es el factor predisponente ms importante en este paciente para
desarrollar trombosis venosa profunda?
RESPUESTA
a.- Colocacin de catter venoso central
b.- Fibrilacin auricular
c.- Cncer de pncreas
d.- Tiempo prolongado en cama
PREGUNTA
Cul de los siguientes factores es menos probable que requiera
antitrombotico profilctico?
RESPESTA
a.- Paciente menor de 40 aos, drenaje de absceso perianal.
b.- Paciente menor de 40 aos en tratamiento con anticonceptivos
c.- Ureterostomia, paciente menor de 40 aos
d.- Apendicitis, paciente mayor de 40 aos
PREGUNTA
Cul es la conducta teraputica ms adecuada para este paciente?
RESPUESTA
a.- Heparina de bajo peso molecular
b.- Antagonista de la vitamina K
c.- Enoxaparina
d.- Filtro de vena cava inferior
CASO CLINICO

Pharmed Solutions Institute

PGINA 183

MANUAL DE TRABAJO DEL CURSO ENARM CMN SIGLO XXI


Femenino de 59 aos de edad, con antecedente de artroplastia
bilateral de rodillas, recibiendo posterior al evento quirrgico,
tratamiento con heparina de bajo peso molecular hasta 15 das previos
a su ingreso. Acude al Servicio de Urgencias donde refiere ataque al
estado general, astenia, adinamia y disnea progresiva llegando incluso
a la ortopnea, de 48 horas de evolucin. A la exploracin fsica se
encuentra con TA 110/70 mmHg, FC 120 lpm, FR 24, rpm y saturacin
de oxgeno de 76% al aire ambiente, sin otros hallazgos relevantes. Se
le realiza electrocardiograma (ECG) que muestra taquicardia sinusal,
con datos de sobrecarga de ventrculo derecho y presencia de patrn
S1Q3T3. La radiografa de trax con datos de hipertensin venocapilar
pulmonar y amputacin de la arteria pulmonar derecha. Laboratorios
con Dmero D 4935 ng/dL, resto en lmites normales. Se decide su
ingreso a la Unidad de Cuidados Coronarios, sin apoyo de aminas, y se
solicita Doppler de miembros inferiores que muestra la presencia de
trombo oclusivo en vena femoral comn izquierda.
PREGUNTA
Cul de los siguientes parmetros es menos probable que indique
inestabilidad hemodinmica?
RESPUESTA
a.- Necesidad de reanimacin cardiopulmonar
b.- PAS >90mmHg
c.- Dilatacin del ventrculo derecho
d.- Necesidad de perfusin de catecolaminas para mantener una
adecuada perfusin orgnica
PREGUNTA
Cul de las siguientes es ms probable que sea una contraindicacin
absoluta para trombolizar?
RESPUESTA
a.- Hipotensin arterial mal controlada (210/110)
b.- Ciruga mayor <10 das
c.- Neoplasia intracraneana
d.- Endocarditis
PREGUNTA
Cul es la conducta teraputica ms adecuada a seguir en este
paciente?
RESPUESTA
a.- Tecnecteplase
b.- Acenocumarina
c.- Alteplase
d.- Estreptoquinasa
CASO CLINICO
Una mujer de 85 aos de edad en quien dos meses antes se parctico
intervencin quirurgicapara tratar cncer de colon es llevada a la
clnica por su hija, porque se ha quejado de hinchazn dolorosa en la
pierna izquierda. La paciente dice haberse sentido por lo dems bien,
salvo por dolor, hichazon y enrojecimiento en la parte alta de la
pantorrilla izquierda, de dos semanas de evolucin. Niega antecedente
de problemas similares y traumatismo agudo y repetitio del rea; doce
que de hecho desde que se le opero, ha evitado hacer ejercicio o
caminar innecesariamente. Tambien niega otro antecedente medico o
quirrgico; toma atenolol y aspirina a diario. Los signos vitales son T
37.2C, TA 135/90mmHg, pulso 60/min, frecuencia respiratoria
12/min. La exploracin fsica no muestra distensin venosa yugular;
cardiopulmonar sin compromiso, abdomen sin alteraciones. La
pantorrilla izquierda muestra edema, signo de Godete, y

CURSO ENARM CMN SIGLO XXI TEL: 36246001

aproximadamente 4 cm de circunferencia mas que la pierna derecha.


Esta eritematosa en la cara posterior e hipersensible a la palpacin. Los
exmenes neurolgicos y de piel no revelan hallazgos importantes.
PREGUNTA
Cul es el diagnostico mas probable en este caso?
RESPUESTA
a.- Celulitis
b.- Quiste poplteo
c.- Trombosis venosa profunda
d.- Tromboflebitis superficial
PREGUNTA
Cul seria la conducta diagnostica mas adecuada a seguir en este
caso?
RESPUESTA
a.- Venografia con medio de contraste
b.- Ultrasonografia con compresin
c.- Pletismografia de impedancia
d.- Venografia con resonancia magntica
CASO CLINICO
Una mujer de 50 aos de edad acude a la sala de urgencias quejndose
de falta de aliento, con dolor a la inspiracin de inicio repentino. Seis
semanas antes se le practico reemplazo total de rodilla y se recupera
sin complicaciones, aunque no ha asistido a sus sesiones de
fisioterapia prescritas. Los medicamentos que toma son oxicodona
para tratar el dolor y hormonoterapia de reemplazo para controlar los
sntomas perimenopausicos. La temperatura es de 38C, el pulso de
120/min, la frecuencia respiratoria es de 30/min, la TA 110/75mmHg y
la saturacin de oxigeno es de 89% mediante oximetra de pulso. La
radiografia de torax resulta normal. Un electrocardiograma muestra
taquicardia sinusal sin otras anomalas.
PREGUNTA
Cul es el diagnostico mas probable para este caso?
RESPUESTA
a.- SDRA
b.- Embolia pulmonar
c.- Pericarditis
d.- Sepsis
PREGUNTA
Cul seria la conducnta diagnostica mas adecuada a seguir en este
caso?
RESPUESTA
a.- Angiografia pulmonar
b.- Gamagrafia pulmonar V/Q
c.- Tomografia computarizada
d.- Ecografia doppler
PREGUNTA
Cul es la conducnta teraputica mas adecuada a seguir en este caso?
RESPUESTA
A.- Heparina de bajo peso molecular
b.- Heparina no raccionada
c.- Acenocumarol
d.- Estreptocinasa

Pharmed Solutions Institute

PGINA 184

MANUAL DE TRABAJO DEL CURSO ENARM CMN SIGLO XXI


SNDROME DE INSUFICIENCIA RESPIRATORIA AGUDA (SDRA):
CIENCIAS BSICAS: Antiguamente denominado sndrome de dificultad respiratoria del adulto, es un trastorno que se caracteriza por una insuciencia
respiratoria aguda hipoxmica debida al edema pulmonar causado por el aumento en la permeabilidad de la barrera alveolocapilar. El SDRA constituye la
manifestacin ms grave de una serie de respuestas al dao pulmonar agudo; estas respuestas traducen las complicaciones de una reaccin sistmica
ms extensa a la inflamacin o agresin agudas. Se dice que ocurre un dao pulmonar agudo cuando se observa una hipoxemia grave, de comienzo
agudo, y opacidades bilaterales difusas en una radiografa anterior de trax, despus de excluir una hipertensin en la aurcula izquierda o en los
capilares pulmonares. El SDRA se diferencia del dao pulmonar agudo por la gravedad de la hipoxemia y se define como una relacin igual o menor a 200
mmHg entre la pO2 arterial y la fraccin de oxgeno en el aire inspirado (PaO2/FIO2). El dao pulmonar agudo y el SDRA deben considerarse como las
manifestaciones primeras y ms fciles de reconocer de un trastorno sistmico infeccioso o inflamatorio. El pulmn adquiere una enorme importancia en
la agresin sistmica por que recibe todo el gasto cardaco y porque las alteraciones de su funcin se reflejan rpidamente en clnica. Ms de la mitad de
los casos comienzan en las primeras 24 horas desde la primera agresin. El dao pulmonar agudo se asocia ntimamente a trastornos que producen un
dao alveolar directo o un dao indirecto a travs del lecho capilar pulmonar como e indicen a SDRA: Lesin directa del epitelio alveolar,
aspiracin, infeccin difusa, semiahogamiento, inhalacin txica, contusin de la va respiratoria, dao pulmonar indirecto, sndrome sptico,
traumatismo no torcico grave, derivacin cardiopulmonar. SALUD PBLICA: La probabilidad del SDRA vara segn la causa desencadenante, desde un
13 % en los casos de sobredosis farmacolgicas hasta un 43 % en la sepsis. PATOGENIA: El dao pulmonar agudo es la consecuencia de la expresin
excesiva, sin ningn tipo de regulacin, de las respuestas inflamatorias generales y habituales a la infeccin, la agresin o ambas. El dao afecta al
epitelio alveolar y al endotelio capilar pulmonar, consecuencia del acontecimiento que inicia la cascada tan compleja de reacciones celulares y
bioqumicas. Estos acontecimientos atraviesan tres etapas: 1. Iniciacin; en la que el factor desencadenante activa la cascada celular. 2. Amplificacin:
en la que se reclutan y activan las clulas efectoras. 3. Dao; fase en la que los acontecimientos se expresan en los tejidos. El dao lo producen los
acontecimientos celulares asociados a los neutrfilos, macrfagos, monocitos y linfocitos, que sintetizan diversas citocinas; estas, a su vez, determinan
una activacin, quimiotaxis y adherencia celular. Las clulas activadas producen una serie de mediadores in amatorios, como los oxidantes, las
proteasas, las caninas, los factores de crecimiento, los neuropp dos, los activadores de la cascada del complemento, la coagulacin intravascular y la
fibrinlisis. La caracterstica fisiopatolgica del SDRA es un aumento en la permeabilidad vascular a las protenas, que determinan la falta de oposicin al
gradiente hidrosttica; por eso incluso elevaciones discretas de la presin capilar (producidas por una sobrecarga de lquidos por va IV o la disfuncin
cardiaca caracterstica de la sepsis) aumentan de forma considerable el edema intersticiales y alveolar. Este efecto aditivo de la permeabilidad y los
factores hidrosttica, se ilustra como la mayor gravedad del proceso en las zonas declives. El aumento entre la relacin del tejido pulmonar y el gas
situado en estas zonas determina que las presiones de cierre alveolar excedan de las presiones transpulmonares locales y se produzca un cierre y un
colapso alveolares. La tendencia al colapso se agudiza por la disminucin cuantitativa de la sntesis de sustancia tensioactiva, debido a la agresin de los
neumocitos de tipo II y a otras anomalas en el tamao, composicin y metabolismo del resto de la sustancia tensioactiva depositada. Estas zonas
atelectsicas del pulmn contribuyen a reducir la distensibilidad del pulmn en su conjunto, pero las zonas de pulmn, no situadas en declive, poseen
propiedades mecnicas y de intercambio gaseoso normales. Gran parte de la ventilacin y del intercambio gaseoso se desplaza hacia estas regiones
pulmonares intactas; para establecer un smil, es como si la funcin respiratoria del adulto fuera suplida por un par de pulmones de un beb. Dada la
distensibilidad menor de los pulmones, es necesario que los msculos respiratorios generen altas presiones inspiratorias, con lo que aumenta el trabajo
de la respiracin. Esta mayor carga mecnica explica la fatiga de los msculos respiratorios, la disminucin consiguiente de los volmenes corrientes y el
empeoramiento del intercambio gaseoso. La hipoxemia y la estimulacin de los receptores del parnquima pulmonar rgido determinan un aumento de
la frecuencia respiratoria., una disminucin del volumen corriente y un deterioro del intercambio gaseoso. La resistencia en las vas respiratorias
aumenta porque el volumen minuto debe sostenerlo un menor nmero de vas respiratorias ventiladas y por el estrechamiento de las vas causado por el
exceso de lquidos y el broncoespasmo. El intercambio gaseoso se caracteriza por relaciones ven lacin-perfusin bajas y un cortocircuito amplio,
acompaado de un gran espacio muerto. El cortocircuito obedece a la atelectasia, colapso vascular, mal funcionamiento de la sustancia tensioactiva y
atenuacin de la vasoconstriccin hipxica. El aumento del espacio muerto se debe a la obstruccin y obliteracin del lecho capilar pulmonar.
DIAGNOSTICO: Clnico; al inicio puede no mostrar sntomas, ni signos respiratorios. El signo ms precoz es el aumento de la frecuencia respiratoria,
seguido poco despus de disnea. En el periodo inicial, la determinacin de los gases en sangre arterial muestra disminucin de la Po2 a pesar de la
disminucin de la Pco2, por lo cual aumenta la diferencia de oxgeno alveolo arterial. En este estado inicial, la administracin de oxgeno produce un
incremento significativo de la Po2 arterial. La exploracin fsica puede ser poco llamativa, aunque se pueden auscultar algunos estertores inspiratorios
finos. Radiolgicamente, los campos pulmonares pueden ser claros o mostrar slo mnimos infiltrados intersticiales focales. A medida que avanza la
enfermedad, el paciente se torna ciantico, con disnea y taquipnea crecientes. Pueden hacerse intensos los estertores, que se oirn fcilmente en todos
los campos pulmonares, junto a zonas de ruido tubricos; la radiografa de trax muestra extensos infiltrados intersticiales y alveolares de carcter
bilateral y difuso. En este momento, la hipoxemia no se puede corregir simplemente aumentando la concentracin de oxgeno en el aire inspirados y hay
que comenzar con la ventilacin asistida. En este estadio ms avanzado, el mecanismo principal de la hipoxemia arterial es el cortocircuito de la sangre
desde la derecha a la izquierda, a travs de los alveolos colapsados u ocupados. El diagnostico de IRA se basa fundamentalmente en la determinacin de
gases arteriales al encontrar PaO2 <50 torr o PaCO2 >50 torr. Las manifestaciones nicas de hipoxemia o hipercapnia, sirven para el reconocimiento de la
presencia de anormalidades importantes en el recambio gaseoso, mas no para el diagnostico, ya que pueden ocurrir tardamente o aun faltar en
presencia de IRA. La falla respiratoria puede clasificarse en 2 tipos: Tipo I: llamada tambin oxigenatoria o hipoxemica; hipoxemia con PaCO2 normal o
bajo. Gradiente alveolo-arterial de O2 incrementado. Tipo II: denominada as mismo ventiladores o hipercapnica; hipoxemia con PaCO2 elevado y
gradiente alveolo-arterial de O2 normal. TRATAMIENTO: Tratamiento de sostn de la hipoxemia. Para alcanzar una Pao2 de 60 mmHg (saturacin de O2
de aprox. el 90 %) debe emplearse el mtodo ms sencillo y la fraccin ms baja de oxgeno en el aire inspirado. Los niveles ms altos apenas aaden
ms oxgeno y, en cambio, conllevan el riesgo de toxicidad pulmonar por esta sustancia. Los tres mtodos fundamentales para la oxigenacin, en orden
de eficacia creciente, son las gafas nasales blandas, las mascarillas faciales simples y las mascarillas faciales con una bolsa de reserva inspiratoria. Parece
razonable empezar con flujos moderados (5-10 L/min de O2 al 100 %) y controlar la gasometra arterial; el flujo y la concentracin de O2 se ajustan en
funcin de los resultados. Soporte ventilatorio mecnico; en presencia de SDRA, estas medidas poco enrgicas no suelen mantener una oxigenacin
adecuada, sino que se requiere la intubacin endotraqueal y la respiracin asistida con un respirador volumtrico. El motivo de aplicar respiracin
asistida a un enfermo que hiperventila no es incrementar la ventilacin sino el volumen pulmonar medio, abriendo las vas respiratorias previamente
cerradas y mejorando la oxigenacin. El objetivo de la ventilacin en el SDRA se basa en proporcionar un soporte fisiolgico para el intercambio gaseoso
y evitar las consecuencias mecnicas desfavorables de la intervencin. Como el pulmn sufre un dao heterogneo en el SDRA, el ajuste de los
volmenes corrientes para lograr este objetivo en todo el pulmn acarrea el riesgo de sobredistender o romper los alveolos menos afectados y ocasionar
la entrada de aire fuera del alveolo (barotraumatismo). Para evitar esta ultima complicacin hay que restringir las presiones de distensin alveolares
mediante el uso de volmenes corrientes relativamente pequeos (aproximadamente 6 a 10 mL/kg de peso corporal magro) y ajustar la presencia
respiratoria, de tal suerte que el volumen minuto se asocie a un pH superior a 7.25 7.30. Dado el volumen tan reducido del pulmn aireado, se

CURSO ENARM CMN SIGLO XXI TEL: 36246001

Pharmed Solutions Institute

PGINA 185

MANUAL DE TRABAJO DEL CURSO ENARM CMN SIGLO XXI


necesitan frecuencias elevadas para alcanzar un volumen minuto adecuado. Tratamiento de la sepsis que causa SDRA: Identificar y tratar cualquier
proceso infeccioso, y Tratar de controlar la respuesta inmunitaria, sin regulacin, que provoca el dao pulmonar agudo. Todo foco localizado de sepsis se
debe drenar de inmediato. La mortalidad quirrgica es elevada, pero si no se drena un foco purulento, lo ms probable es que el enfermo
muera. Complicaciones: Las medidas empleadas para tratar el SDRA pueden asociarse a graves complicaciones. La toxicidad del oxgeno, debido al uso
prolongado de una Fio2 superior al 50 %, y la hidratacin excesiva empeoran a veces los infiltrados pulmonares. Un volumen y presin corrientes
elevados, por parte del respirador, son causa de barotraumatismo. La ventilacin mecnica prolongadas predispone a la neumona hospitalaria. El SDRA
y los trastornos subyacentes favorecen las fstulas boncopleurales y la coagulacin intravascular diseminada. La fibrosis pulmonar, la hipertensin
pulmonar irreversible y la insuficiencia multiorgnica sugieren un mal pronstico en los enfermos con SDRA.
CASO CLINICO
Se trata de paciente masculino de 54 aos de edad el cual es ingresado
a urgencia traido por ambulancia desde una distancia de 4 horas de
camino, refieren los paramdicos que es sobreviviente a accidente
automovilstico, al parecer era el conductor, a la exploracin
conciente, orientado, con facies algicas, se observa contusion en
esternn y parrilla costal de predominio izquierdo, discreta disnea,
hematomas multiples en miembros pelvicos, se encuentra fractura de
femur no expuesta, signos vitales TA 105/80 mmHg, FC 105, FR 32. Fue
estabilizado y se procedi a realizar ciruga de femur, se traslado a piso
al 3 dia el paciente inicia con fiebre, disnea progresiva, edema de
miembros inferiores, se realizo radiografia de torax sin observarse
hallazgo patolgicos, el electrocardiograma mostro un eje de QRS
mayor a 90 oC y ondas T negativas en V1 a V4.
PREGUNTA
Cual es su impresin diagnostica mas probable con el cuadro clnico del
paciente hasta el momento?
RESPUESTA
a.- Tromboembolia pulmonar.
b.- SIRA.
c.- Embolia pulmonar grasa.
d.- Neumotorax.
PREGUNTA
Se realiza tratamiento anticoagulante sin aparente mejora, a las 12
horas posteriores se incrementa la disnea y ortopnea, intolerancia al
decbito y ansiedad generalizada, tos con expectoracin, dificultad
respiratoria con uso de musculos accesorio, se realiza Rx de torax
porttil con sospecha de edema agudo de pulmon, cuales de los
siguientes hallazgos no corresponden a ese diagnostico?
RESPUESTA
a.- Opacidades alveolares diseminadas y bilaterales.
b.- Redistribucin del flujo pulmonar.
c.- Distribucin en ala de mariposa.
d.- Lineas B de Kerley.
CASO CLINICO
Masculino de 66 aos con antecedentes de neoplasia pulmonar,
tratado con quimioterapia y radioterapia, que queda libre de
enfermedad, con aparicin posterior de una metstasis cerebral sobre
la que se realiza reseccin completa pendiente de iniciar radioterapia
holocraneal. Acude a urgencias por clnica de disnea a mnimos
esfuerzos de 10 das de evolucin y fiebre de hasta 39C. Niega tos y
expectoracin. No destaca ningn antecedente epidemiolgico de
inters. Inici tratamiento antibitico de forma ambulatoria sin
presentar mejora. A su llegada presenta insuficiencia respiratoria
grave que requiere FiO2 elevadas, por lo que ingresa en la unidad de

CURSO ENARM CMN SIGLO XXI TEL: 36246001

crticos de nuestro centro. Presenta rpida progresin de la


insuficiencia respiratoria y a las 24h precisa intubacin orotraqueal y
ventilacin mecnica. La radiografa de trax inicial muestra un
infiltrado alveolar en la base izquierda con aparicin de infiltrados
pulmonares bilaterales en controles posteriores. Se realiza
cateterizacin de la arteria pulmonar que descarta fallo cardaco, con
presin capilar pulmonar inferior a 18mmHg, y que muestra signos de
hipertensin pulmonar moderada. Analticamente, destaca la aparicin
de coagulacin intravascular diseminada, fracaso renal agudo con
cifras de creatinina de hasta 2,33mg/dl y elevacin de LDH. Se inicia
tratamiento antibitico de forma emprica con piperacilina-tazobactam
y azitromicina, y se aaden posteriormente corticoides. Dada la mala
evolucin, con persistencia de insuficiencia respiratoria grave y
empeoramiento de los infiltrados pulmonares de causa no filiada, se
realiza TC torcica que muestra afectacin parenquimatosa bilateral
muy extensa en vidrio deslustrado, con un aspecto radiolgico que
indica dao alveolar difuso o proceso infeccioso de caractersticas
atpicas. Se sospecha probable infeccin por el virus influenza A (H1N1)
se realiza frotis nasal y farngeo, y la PCR es positiva.
PREGUNTA
Cul de los siguientes criterios es menos probable, que nos sea de
utilidad para diagnosticar SIRA?
RESPUESTA
a.- Inicio agudo
b.- Presin capilar pulmonar >18mmHg
c.- PaO2/FiO2 <200
d.- Infiltrados pulmonares difusos
PREGUNTA
Cul mecanismo fisiopatolgico es menos probable que se halla
presentado en el paciente para el desarrollo de SIRA?
RESPUESTA
a.- Permeabilidad capilar alterada
b.- El paso de lquido supero el drenaje linftico
c.- Se altera la relacin V/Q
d.- Aumento de surfactante
PREGUNTA
Cul es la conducta teraputica ms adecuada para este paciente?
RESPUESTA
a.- Garantizar transporte de oxgeno, puntas nasales,
tazobactam/piperacilina
b.- Ventilacin mecnica, con presiones de volmenes bajas,
oseltamivir
c.- Garantizar transporte de oxgeno, mediante ventilacin mecnica,
oseltamivir
d.- Ventilacin mecnica, reemplazo de surfactante, oseltamivir

Pharmed Solutions Institute

PGINA 186

MANUAL DE TRABAJO DEL CURSO ENARM CMN SIGLO XXI


EDEMA AGUDO PULMONAR (EAP):
CIENCIAS BSICAS: El EAP no es en s una enfermedad, pero frecuentemente es una complicacin de alguna otra anormalidad, la cual puede tener origen
en los pulmones o ms comnmente, en rganos extrapulmonares, (corazn). El edema pulmonar constituye una verdadera urgencia mdica, y por ello
debe identificarse rpidamente. El cuadro puede ser de origen cardiognico o no cardiognico. En el EAP cardiognico, un fallo de bombeo hace que la
sangre retroceda a la circulacin pulmonar. Cuando la presin hidrosttica capilar pulmonar supera la presin osmtica coloidal, el lquido es impulsado
fuera de los capilares pulmonares hacia el espacio intersticial y el alveolo. En el de origen no cardiognico, las paredes de las clulas capilares
pulmonares se vuelven ms permeables. Constituye una de las urgencias ms graves en pacientes con enfermedades del corazn izquierdo. Ocurre por
aumento de la presin capilar pulmonar secundaria a estenosis mitral o insuciencia cardiaca izquierda, de ah que tambin se le conozca como edema
pulmonar cardiognico. PATOGENIA: El EAP es el acumulo excesivo de lquido extravascular en el pulmn, ya sea en el intersticio (edema intersticial) o
en el alveolo (edema alveolar). Para mantener seco el intersticio pulmonar funcionan varios mecanismos delicados: 1. Presin osmtica superior a la
presin capilar pulmonar; Las fuerzas hemodinmicas bsicas opuestas son la presin capilar pulmonar (PCP) y la presin osmtica del plasma. En los
individuos normales la PCP oscila entre los 7 y los 12 mmHg, siendo la presin osmtica del plasma de 25 mmHg aproximadamente, por lo que .esta
fuerza tiende a impulsar el lquido de regreso a los capilares. 2. Tejido conjuntivo y barreras celulares relativamente permeables a las protenas
plasmticas; La presin hidrosttica acta a travs del tejido conjuntivo y la barrera celular, que en circunstancias normales son relativamente
impermeables a las protenas plasmticas. 3. Extenso sistema linftico; El pulmn posee una extensa red linftica que puede aumentar su flujo en 5 a 6
veces cuando se encuentra con un exceso de agua en el inters cio pulmonar. Cuando los mecanismos normales para mantener el pulmn seco funcionan
mal o estn superados por un exceso de lquidos el edema ende acumularse. Las causas de EAP son: Cardiognicas: IAM, arritmias
cardiacas, insuficiencia ventricular izquierda grave, shoc cardiognico. No cardiognicas: Inhalacin de gases irritantes, neumona por aspiracin, Shock
sptico, embolia grasa, sndrome de distrs respiratorio del adulto, administracin rpida de lquidos intravenosos, sobredosis de barbitricos u opiceos.
DIAGNOSTICO: Agitacin, angustia, nerviosismo. La valoracin del edema agudo del pulmn a nivel respiratorio es laborioso; disnea (el edema pulmonar
dificulta la distensibilidad pulmonar y hace ms difcil la expansin de los pulmones). Hipoxemia (debido al deterioro del intercambio gaseoso).
Existencia de hipoxia con hipocapnia (hiperventilacin) ya que elimina gran cantidad de CO2 al respirar tan dificultosamente, desarrollando una alcalosis
respiratoria. Taquipnea 30-40 min (debido a la hipoxia el paciente respira ms de prisa). Crepitantes y sibilancias (al pasar el aire a travs de las vas
areas llenas de liquido, durante la inspiracin). Valoracin cardiovascular: Taquicardia y aumento de la presin arterial (para compensar el dficit de
aporte de oxigeno, el sistema simptico aumenta la frecuencia cardiaca). El sistema nervioso simptico provoca vasoconstriccin que aumenta la presin
arterial. Distensin de las venas yugulares. Esputo espumoso teido de sangre (debido a la presin, algunos vasos pulmonares se rompen haciendo que
los hemates pasen a los pulmones). Piel plida, fra y sudorosa (para conservar el oxigeno, el organismo deriva sangre arterial desde la piel hacia rganos
vitales). Tercer ruido cardiaco (ritmo de galope), producido cuando durante el principio de la distole la sangre entra de prisa en el ventrculo izquierdo
distendindolo y da lugar a vibraciones anormales. En el trax puede observarse el empleo de los msculos accesorios de la respiracin (tiros
intercostales). Puede haber estertores audibles a distancia y sin necesidad del estetoscopio. Buscar intencionadamente la presencia de ritmo de galope
que permi r establecer el diagns co de insuficiencia cardiaca, mientras que con la auscultacin de un ritmo de Duroziez haremos el diagnstico de
estenosis mitral. La radiografa de trax es el estudio ms til para el diagnstico en conjunto con la historia clnica del EAP, ya que muestra la imagen
correspondiente al edema alveolar como una opacidad diseminada en ambos campos pulmonares. Cuando los grados de hipertensin venocapilar
pulmonar son menores, podrn observarse otros datos como son: la redistribucin de flujo a los vrtices, la cisura interlobar visible, las lneas B de erley
o un moteado no difuso y en ocasiones la imagen en alas de mariposa; la presencia de cardiomegalia orientar hacia el diagns co de insuciencia
cardiaca, mientras que la ausencia de ella hablar de disfuncin diastlica. El electrocardiograma ayuda a identificar la posible cardiopata subyacente o
los factores desencadenantes, como un infarto del miocardio o una taquiarritmia como la fibrilacin auricular. TRATAMIENTO: El manejo teraputico de
esta grave complicacin tiene tres objetivos: 1. Disminuir la presin venocapilar: 2. Mejorar la ventilacin pulmonar. 3. Tratamiento de la enfermedad
causal. Medidas que disminuyen la hipertensin venocapilar: a). Sentar al paciente al borde de la cama con las piernas pendientes para disminuir el
retorno venoso al corazn. b). Aplicacin de torniquetes rotatorios en tres de las cuatro extremidades, con la fuerza necesaria para que no desaparezca
el pulso arterial y se cambia cada 15 minutos a la extremidad que se encuentra libre (sangra seca). c). Isosorbide o nitroglicerina por va sublingual. La
administracin de una tableta de cualquiera de estos dos frmacos tiene un poderoso efecto vasodilatador que reduce drsticamente el retorno venoso
y la congestin pulmonar. d). La administracin intravenosa de furosemida a razn de 20 a 60 mg promueve con gran rapidez la movilizacin de lquidos
del intersticio pulmonar hacia el rin, por lo que se reduce con gran eficiencia la congestin pulmonar. e). Nitroprusiato de sodio.- Este poderoso
vasodilatador mixto se administra por va intravenosa a razn de 0.3 a 0.8 microgramos/Kg/minuto, cuando el EAP es consecutivo a insuficiencia
ventricular por crisis hipertensiva. En estos casos, esta medida es de eleccin para el tratamiento del edema pulmonar. El efecto arteriodilatador reduce
significativamente la postcarga y con ello mejora la funcin ventricular y el gasto cardiaco, y por otro lado, el efecto venodilatador reduce el retorno
venoso al corazn y la congestin pulmonar. Medidas que mejoran la ventilacin pulmonar: a). Administracin de oxgeno mediante puntas nasales a
razn de 4 litros por minuto. b). Aminolina 1 ampolleta de 250 mg muy lenta por va intravenosa; su efecto broncodilatador mejora la ven lacin
pulmonar as como su efecto diur co potencia la accin de la furosemida. Tratamiento de la enfermedad causal: a). La mayora de los pacientes que
presentan EAP por estenosis mitral mejoran con las medidas antes anotadas. Cuando a pesar de ellas contina el cuadro clnico, se requiere la intubacin
del paciente para administrarle asistencia mecnica a la ventilacin pulmonar, especialmente la presin posi va respiratoria nal (PEEP) que evita el
colapso de las vas respiratorias pequeas y con ello asegura la ven lacin alveolar. En estas condiciones, el paciente deber ser llevado al quirfano
para realizar comisurotoma mitral de urgencia, que en estos casos es la nica medida que alivia el cuadro de edema pulmonar. b). Cuando la
insuciencia ventricular izquierda es la causa, se requiere la digitalizacin rpida con ouabana o lanatsido C, adems de las medidas para reducir la
presin capilar y mejorar la ventilacin pulmonar. c). El edema pulmonar que es causado por disfuncin diastlica, usualmente se presenta en la
evolucin de un infarto del miocardio agudo; debe ser tratado con las medidas que reducen la presin capilar pulmonar asociada a aquellas otras que
mejoran la ventilacin pulmonar y ambas a su vez, a las que reducen el efecto de isquemia miocrdica como lo son la administracin de
betabloqueadores (propranolol 10 a 20 mg cada 8 horas) o calcio antagonistas (diltiazem 30 mg cada 8 horas), ya que estos frmacos reducen el efecto
que la isquemia tiene sobre la relajacin ventricular. En efecto, cuando la relajacin se hace ms rpida y completa, se reduce significativamente la
presin diastlica del ventrculo izquierdo y con ello la hipertensin venocapilar. Estabilizacin hemodinmica: Se considera una buena respuesta si a los
15 a 30 minutos del inicio del tratamiento, mejora la disnea, la frecuencia cardiaca, disminuyen los estertores pulmonares, aparece una coloracin ms
oxigenada de la piel y se produce una buena diuresis, despus de la cual se le puede ofertar al paciente potasio, 1 ampolleta por va oral con agua para
compensar las prdidas de de este ion o adicionrselo a la infusin de la venoclisis
CASO CLINICO
Mujer de 25 aos de edad, primigesta, admitida por cuadro de
preeclampsia severa, con presiones arteriales > 160/110 mmHg,

CURSO ENARM CMN SIGLO XXI TEL: 36246001

manejada con sulfato de magnesio y labetalol intravenoso sin obtener


adecuada respuesta, por lo que se realiza cesrea, que culmina con
recin nacido sano, se traslada posteriormente a unidad de cuidados

Pharmed Solutions Institute

PGINA 187

MANUAL DE TRABAJO DEL CURSO ENARM CMN SIGLO XXI


intensivos (UCI) para manejo con infusin de vasodilatadores y
vigilancia neurolgica por riesgo elevado de eclampsia. Al ingreso en
UCI present presin arterial de 161/120 mmHg, taquicardia,
inadecuado patrn respiratorio, saturacin de oxgeno arterial (SaO2)
del 89%, crepitantes bibasales y presin venosa central (PVC) de 16
mmHg. Se realiza radiografa de trax que muestra cefalizacin del
flujo, con hilios pulmonares congestivos.
PREGUNTA
Cul es el diagnotico ms probable para este caso?
RESPUESTA
a.- Embolia pulmonar
b.- Edema agudo de pulmn
c.- SDRA
d.- Neumotrax
PREGUNTA
Cual es la conducta mas adecuada para el caso?
RESPUESTA
a.- Ventilacion mecnica.
b.- Antibioticoterapia profilctica.
c.- Furosemide y nitroglicerina.
d.- Sindenafil
PREGUNTA
A pesar de todas las medidas iniciales, no se observa gran mejora y la
saturacin se encuentra en 85%. Cul seria la conducta mas adecuada
a seguir en este caso?
RESPUESTA
a.- Puntas nasales
b.- Ventilacion mecnica con PEEP
c.- Casco ceflico
d.- Anticoagulacion
CASO CLINICO
Paciente de 68 aos de edad, fumador con criterios clnicos de EPOC,
que acudi al servicio de urgencias por disnea progresiva hasta hacerse
a mnimos esfuerzos de 2 semanas de evolucin. No refera
traumatismo torcico. Radiolgicamente, se diagnostic un
neumotrax izquierdo. Se insert un tubo de drenaje torcico, se
reexpandi completamente el pulmn y mejor as la clnica del
paciente. Tres horas ms tarde tuvo un deterioro clnico consistente en
dificultad respiratoria grave. Se auscultaban estertores hmedos de
gruesa burbuja en el hemitrax izquierdo. Una nueva RX de trax
mostr imgenes compatibles con edema pulmonar unilateral. Fue
preciso conectarlo a ventilacin mecnica y que recibiera tratamiento
vasoactivo.
PREGUNTA
Cual es la etiologa mas frecuente del caso?
RESPUESTA
a.- Arritmia cardiaca.
b.- Isquemia miocrdica.
c.- Insuficiencia cardiaca.
d.- Complicaciones del parnquima pulmonar
PREGUNTA
Qu trastorno acido-base, es mas probable observar en este
paciente?
RESPUESTA
a.- Alcalosis respiratoria
b.- Acidosis respiratoria
c.- Alcalosis metabolica

CURSO ENARM CMN SIGLO XXI TEL: 36246001

d.- Acidosis metabolica


PREGUNTA
Cul es la conducnta teraputica mas adecuada para disminuir la
presin venocapilar?
RESPUESTA
a.- Nitroprusiato de sodio
b.- Furosemida
c.- Isosorbide
d.- Propanolol
CASO CLINICO
Paciente varn de 69 aos, que ingresa por cuadro de disnea
paroxstica nocturna. Antecedentes personales: cardiopata isqumica,
diabetes Mellitus tipos II controlada con dieta, tratamiento actual:
Aspirina infantil y Coronur 20 mg/12 horas. Estando previamente bien
(el paciente llevaba una vida activa y estaba asintomtico), esta noche
se despierta sbitamente con sensacin de dificultad respiratoria (el
paciente dice que no poda respirar, que le faltaba el aire), que se
acompaa de frialdad, palidez y gran sudoracin. El cuadro ha sido de
instauracin brusca (mientras dorma) y ha empeorado de manera
muy rpida en pocos minutos. No refiere dolor torcico. Despus se ha
aadido tos con expectoracin muy lquida de color sonrosado. EF:
Paciente consciente y orientado, colaborador pero muy inquieto e
intranquilo. Palidez de piel y mucosas, sudoracin y frialdad y
piloereccin. Tiene gran dificultad respiratoria que hace que el
paciente permanezca sentado en el borde de la cama y no tolere el
decbito. Respiracin muy ruidosa audible a distancia. Intenso tirage
intercostal y supraclavicular y respiracin abdominal manifiesta.
Auscultacin pulmonar: ruidos crepitantes inspiratorios y espiratorios
intensos y difusos. Auscultacin cardiaca: taquicardia rtmica. No
soplos. Exploracin de otros rganos y aparatos normal. Constantes
vitales: TA 160/90; Pulso 120 p/m; Temp. 36C. FR 32 resp/m.
Radiografa de trax: Patrn alveolar difusa de carcter centrfugo y
cardiomegalia importante. Gases arteriales respirando basalmente: pH
7,48 ; PCO2 32; PO2 56; HCO3 24
PREGUNTA
Cul es el diagnstico ms probable para este paciente?
RESPUESTA
a.- Aspiracin bronquial de contenido digestivo
b.- Agudizacin grave del asma (AGA)
c.- Edema aguda de pulmn de origen cardiognico (EAP)
d.- Neumona bilateral
PREGUNTA
Cul es la causa menos probable de edema agudo del pulmn no
cardiogenico?
RESPUESTA
a.- Choque sptico
b.- Neumonia
c.- Sobredosis de barbitricos u opiceos
d.- Cncer de pulmn
PREGUNTA
Cul es la fisiopatologa ms probable para el desarrollo del
diagnstico del paciente?
RESPUESTA
a.- Presin onctica de plasma menor a la presin capilar pulmonar
b.- Presin onctica de plasma mayor a la presin capilar pulmonar
c.- Presin hidrosttica menor a la capilar pulmonar
d.- Presin hidrosttica mayor a la capilar pulmonar

Pharmed Solutions Institute

PGINA 188

MANUAL DE TRABAJO DEL CURSO ENARM CMN SIGLO XXI


ASMA
CIENCIAS BASICAS: Definicin: Obstruccin reversible al flujo areo producida por inflamacin bronquial, as como por respuesta a diversos estmulos
externos o mediadores endgenos. La mayora de los asmticos tienen enfermedad durante la infancia. El asma ocupacional puede resultar de gran
variedad de productos qumicos, incluyendo disocianato de tolueno y anhdrido trimeltico, y esto puede ser de aparicin en el adulto. Diferentes
disparadores pueden empeorar la sintomatologa como alrgenos inhalados, en especial a individuos sensibles, infecciones virales, bloqueadores Badrenrgicos pueden empeorar los sntomas de asma y por lo general se deben evitarse, el ejercicio entre otros como la contaminacin, exposiciones
ocupacionales, cambios bruscos de temperatura, ERGE y estrs. SALUD PUBLICA: Prevalencia en Mxico 10% de la poblacin; 80% son nios menores de
8 aos, del total de casos; 12% tienen entre 9-17 aos y 8% adultos. La mayora de los asmticos son atpicos y a menudo tienen asma y rinitis alrgica.
La mayora de los asmticos tienen enfermedad durante la infancia. PATOGENIA: En el asma la respuesta del rbol bronquial consiste en disminucin del
calibre de su luz originada por 3 fenmenos; 1) constriccin de la musculatura bronquial (broncoconstriccin). 2) edema de la mucosa e infiltracin
celular (inflamacin). 3) incremento de la secrecin bronquial (moco). Los mecanismos anatomopatologicos ms representativos son hipertrofia de
musculo liso bronquial, transformacin de fibroblastos a miofibroblastos y depsito de colgena subepitelial (fenmeno de remodelacin), todo esto
lleva a largo plazo a disminucin de la funcin pulmonar. DIAGNOSTICO: Cuadro clnico: Sntomas respiratorios comunes son sibilancias, disnea y tos en
accesos, opresin torcica, con predominio nocturno y/o matutino. Exploracin fsica: Depende gravedad podemos observar taquipnea, uso de msculos
accesorios, cianosis, voz entrecortada, diafortico, taqucardico, rudeza respiratoria. La evidencia de rinitis alrgica, sinusitis, reaccin en la piel podran
asesorarnos. Pruebas de funcin pulmonar: La prueba ideal es la espirometra (medimos resistencia al flujo areo), nos reportara un patrn obstructivo
con reduccin de FEV1 (volumen espiratorio forzado en el primer segundo) y CVF (capacidad vital funcional), si hay sntomas de asma y la espirometra es
normal se pueden realizar pruebas de metacolina, histamina o provocacin con ejercicio. La tasa de flujo espiratorio mximo (PEF) puede ser utilizado,
para seguir el control del asma objetivamente en casa. Rx. de trax: Usualmente normal, en exacerbaciones agudas podemos identificar neumotrax,
sobredistencin por atrapamiento areo, horizontalizacin de arcos costales, abatimiento de hemidiafragmas, infiltrados pulmonares eosinofilicos.
CLASIFICACION Y TRATAMIENTO: Asma extrnseca, pacientes en o que puede demostrarse una reaccin Ag-Ac como desencadenante del proceso,
mediadad por IgE, generalmente atpica u ocupacional. Asma intrnseca no se encuentra Ag especfico, comienza en vida adulta, se puede asociar a
plipos nasales, sinusitis, idiosincrasia, AS o AINES.
GRADO
CARACTERISTICAS
TX. RESCATE
TX. MANTENIMIENTO
Los
efectos
INTERMITENTE
Sntomas menos de 1 vez por semana
Los agonistas 2 adrenrgicos, relajan el
Baja dosis de antiinflamatorios
secundarios de
Exacerbaciones de corta duracin
musculo liso (salbutamol) de corta accin,
los B2 agonistas
Sntomas nocturnos no ms de 2 veces al mes
cuantas veces sea necesario
FEV1 o PEF >80% del valor predicho
son temblores
Variabilidad en el PEF o FEV <20%
musculares y
PERSISTENTE
Sntomas ms de 1 vez por semana pero menos de 1 vez al da
Los agonistas 2 de corta accin, cuantas
Agonista B, de larga accin (salmeterol,
palpitaciones.
LEVE
Exacerbaciones que pueden afectar la actividad y el sueo
veces sea necesario
formoterol) + antiinflamatorios en baja dosis
Sntomas nocturnos ms de 2 veces por mes
Los
agentes
FEV1 o PEF >80% del valor predicho
antiinflamatori
Variabilidad en el PEF o FEV1 <20 a 30%
os
MODERADO
Sntomas diarios
Los agonistas B de corta accin, cuantas
Esteroide inhalado dosis baja (u otro
principalmente
PERSISTENTE
Exacerbaciones que afectan la actividad y el sueo
veces sea necesario
antiinflamatorio
a
dosis
alta)
+
Sntomas nocturnos ms de 1 vez a la semana
broncodilatador de larga accin = reducen
usados son los
Uso diario de inhaladores con b agonistas de accin corta
las exacerbaciones y proporciona excelente
esteroides
FEV1 o PEF 60 a 80% del valor predicho
tx. a largo plazo
inhalados
Variabilidad en el PEF o FEV1 >30%
INTENSA
Sntomas
diarios
Los
agonistas
B
de
corta
accin,
cuantas
Esteroide inhalado dosis media (u otro
(inhiben
la
PERSISTENTE
Exacerbaciones frecuentes
veces sea necesario. El uso excesivo
antiinflamatorio
a
dosis
alta),
infiltracin
Sntomas frecuentes de asma nocturna
indica un control inadecuado
broncodilatador de accin larga
celular
y
FEV1 o PEF <60% valor predicho
Variabilidad en el PEF o FEV1 >30%
disminuyen el
edema, efecto de 3 a 6 hrs). Son la primera lnea en pacientes que no se encuentran en crisis, son los mas efectivos en el control del tx. del asma, el
efecto local ms importante es la candidosis oral, y ronquera los ms usados, budesonina, fluticasona y beclometasona, reducen sntomas durante el
ejercicio, y sntomas nocturnos. Los antileucotrienos (montelukast, zafirlukast), beneficiosos solo en algunos puntos. La adrenalina produce disminucin
de la resistencia muscular sobre el rbol bronquial. Otros brocodilatadores incluyen los anticolinrgicos (que se dan principalmente en EPOC) y la
teofilina es un inhibidor de la fosfodiesterasa que aumenta el adenosin monofosfato ciclico en los neutrfilos, que tienen efecto brocodilatador y
antiinflamatorio, pero debido a su toxicidad asociada, no se usa comunmente. CRISIS ASMATICAS: Caracterizado por aumento del trabajo respiratorio,
tos, opresin torcica y sibilancias; usar salbutamol o terbutalina, micronebulizados, durante 1 h continua para romper el broncoespasmo. Esteroides
endovenosos, metilprednisolona o prednisona para romper cascada de inflamacin.
CASO CLINICO (ASMA)
Mujer de 40 aos, fumadora con una exposicin tabquica de 20
paquetes-ao. Trabajaba en una empresa metalrgica desde haca 3
aos. Desde haca un ao presentaba clnica de disnea, tos y sibilancias
de predominio nocturno, que mejoraba los fines de semana y perodos
vacacionales. Requiri un ingreso por un episodio de broncospasmo y
al reincorporarse a su lugar habitual de trabajo se reiniciaron dichos
sntomas. Con ttulos de IgE srica total de 59 U/ml. El estudio de la
funcin pulmonar fue normal, mostrando una capacidad vital forzada
de 4,17 l (114%), volumen espiratorio forzado en el primer segundo
FEV1% del 79% y prueba broncodilatadora negativa. Se efectu una
prueba de metacolina, que fue negativa, con PC20 superior a 16 mg/ml
(concentracin de metacolina que caus un descenso del FEV1 basal
del 20%).
PREGUNTA
Considerando el estado actual que grado de asma presenta el caso?
RESPUESTA
a.- Intermitente.

CURSO ENARM CMN SIGLO XXI TEL: 36246001

b.- Leve persistente.


c.- Moderado persistente.
d.- Intenso persistente.
PREGUNTA
Considerando la gravedad del cuadro, cual es el tratamiento mas
adecuado para este caso?
RESPUESTA
a.- Esteroide inhalado dosis media (u otro antiinflamatorio a dosis
alta), broncodilatador de accin larga.
b.- Esteroide inhalado dosis baja (u otro antiinflamatorio a dosis alta) +
broncodilatador de larga accin.
c.- Agonista B, de larga accin (salmeterol, formoterol) +
antiinflamatorios en baja dosis.
d.- Baja dosis de antiinflamatorios.
CASO CLINICO (ASMA)
Paciente de 63 aos de edad. Diabetes mellitus tipo 2 diagnosticada
hace un mes, en tratamiento. Exfumador. Sobrepeso (IMC:27). Asma

Pharmed Solutions Institute

PGINA 189

MANUAL DE TRABAJO DEL CURSO ENARM CMN SIGLO XXI


bronquial de diez aos de evolucin, en tratamiento combinado con
un anticolinrgico de larga accin (bromuro de tiotropio), un
antagonista leucotrinico (Montelukast), corticoides orales e
inhalados. Hernia hiatal. Estenosis artica leve degenerativa.
Hiperplasia benigna de prstata. Acude al Servicio de Urgencias por
clnica de quince das de evolucin con fiebre de predominio
vespertino, malestar general, sudoracin profusa, astenia, prdida de
peso, vmitos, dolor epigstrico y somnolencia creciente adems
disuria, polaquiuria, tenesmo o urgencia miccional. Destaca un
episodio de odontalgia diez das antes en relacin con absceso
peridental, resuelto con antibioterapia oral (amoxicilina-clavulnico).
EF: TA de 89/57 mm Hg, temperatura 39,3C. El paciente se mostraba
consciente, orientado globalmente, con deshidratacin cutneomucosa y palidez cutnea, aunque con mucosas normocoloreadas.
Destaca un pequeo hematoma en la cara interna del brazo izquierdo.
Auscultacin cardaca era rtmica, FC (120 1pm), FR (24 rpm),
hipoventilando
en
ambas
bases
pulmonares
con
hepatoesplenomegalia. Ligeros edemas en tercio distal de ambas
piernas.
PREGUNTA
Considerando el estado actual que grado de asma presenta el caso?
RESPUESTA
a.- Intermitente.
b.- Leve persistente.
c.- Moderado persistente.
d.- Intenso persistente.
PREGUNTA
Considerando la gravedad del cuadro, cual es el tratamiento mas
adecuado para este caso?
RESPUESTA
a.- Esteroide inhalado dosis media (u otro antiinflamatorio a dosis
alta), broncodilatador de accin larga.
b.- Esteroide inhalado dosis baja (u otro antiinflamatorio a dosis alta) +
broncodilatador de larga accin.
c.- Agonista B, de larga accin (salmeterol, formoterol) +
antiinflamatorios en baja dosis.
d.- Baja dosis de antiinflamatorios.
PREGUNTA
En caso de requerir tratamiento de rescate, cual es la opcin mas
adecuada para el caso?
RESPUESTA
a.- Los agonistas B de corta accin, cuantas veces sea necesario.
b.- Los agonistas B de corta accin, cuantas veces sea necesario. El uso
excesivo indica un control inadecuado.
c.- Los agonistas 2 de corta accin, y de larga accin cuantas veces
sea necesario.
d.- Los agonistas 2 adrenrgicos, relajan el musculo liso (salbutamol)
de corta accin, cuantas veces sea necesario.
CASO CLINICO (ASMA + SCS)
Varn de 40 aos, diagnosticado de asma bronquial desde nio. Desde
hace un ao hace frecuentes visitas al servicio de urgencias del
hospital por disnea. Ha sido tratado con esteroides inhalados y orales,
adrenrgicos beta-2 de corta y larga duracin, y antibiticos. En su
historia actual cabe destacar que desde unos das antes de su ingreso
en el hospital presentaba tos, disnea y fiebre. A la EF, disneico, trax
hiperinsuflado y tiraje con disminucin de ruidos respiratorios y
presencia de lesiones cutneas eritematosas y vesiculosas en pie
izquierdo, hombros y trax. Las radiografas de trax presentaron una
condensacin en el lbulo superior derecho en dos focos y otra en el
lbulo superior izquierdo menos densa. Leucocitos 15.000/l
(eosinfilos 31%, segmentados 44%). Segn la espirometra, sus
valores de flujos espiratorios mximos en el primer segundo oscilaron
entre 800 y 2.650 ml, es decir, entre el 23 y el 71% del valor predicho.
El servicio de dermatologa inform una consulta sobre las lesiones

CURSO ENARM CMN SIGLO XXI TEL: 36246001

cutneas como probablemente secundarias a alergia inespecfica. Se


trat con esteroides orales a dosis medias decrecientes, mejorando de
su clnica respiratoria y desapareciendo las lesiones cutneas y las
alteraciones radiolgicas, elaborndose el diagnstico de probable
SCS.
PREGUNTA
Considerando el estado actual que grado de asma presenta el caso?
RESPUESTA
a.- Intermitente.
b.- Leve persistente.
c.- Moderado persistente.
d.- Intenso persistente.
PREGUNTA
Considerando la gravedad del cuadro, cual es el tratamiento mas
adecuado para este caso?
RESPUESTA
a.- Esteroide inhalado dosis media (u otro antiinflamatorio a dosis
alta), broncodilatador de accin larga.
b.- Esteroide inhalado dosis baja (u otro antiinflamatorio a dosis alta) +
broncodilatador de larga accin.
c.- Agonista B, de larga accin (salmeterol, formoterol) +
antiinflamatorios en baja dosis.
d.- Baja dosis de antiinflamatorios.
PREGUNTA
En caso de requerir tratamiento de rescate, cual es la opcin mas
adecuada para el caso?
RESPUESTA
a.- Los agonistas B de corta accin, cuantas veces sea necesario.
b.- Los agonistas B de corta accin, cuantas veces sea necesario. El uso
excesivo indica un control inadecuado.
c.- Los agonistas 2 de corta accin, y de larga accin cuantas veces
sea necesario.
d.- Los agonistas 2 adrenrgicos, relajan el musculo liso (salbutamol)
de corta accin, cuantas veces sea necesario.
PREGUNTA
El paciente presenta datos sugerentes del sndrome de Churg-Strauss
(SCS), cual de las siguientes aseveraciones no es correcta para el
diagnostico de SCS?
RESPUESTA
a.- Sndrome hipereosinoflico de origen desconocido.
b.- Caracterizado por una vasculitis necrosante con granulomas
extravasculares.
c.- Infiltrados eosinfilos y de polimorfonucleares en la pared de vasos
de mediano y pequeo calibre.
d.- Es muy poco frecuente, afecta sobre todo a menores de 30 aos.
PREGUNTA
Clsicamente el SCS se presenta en 3 fases clnicas, cual no es una de
ellas?
RESPUES
a.- Prodrmica (afeccin de vas areas altas y presencia de asma).
b.- Eosinoflica (eosinofilia perifrica e infiltrados pulmonares).
c.- Vascultica (afeccin sistmica).
d.- Necrotica (afeccion necrtica sistmica terminal).
PREGUNTA
Cual de los criterios para el diagnstico establecidos por el American
College of Rheumatology (ACR) para SCS no son ciertos?:
RESPUESTA
a.- Asma.
b.- Eosinofilia perifrica mayor del 10%,
c.- Radiculopatias y Polineuritis.
d.- Infiltrados pulmonares transitorios.

Pharmed Solutions Institute

PGINA 190

MANUAL DE TRABAJO DEL CURSO ENARM CMN SIGLO XXI


ENFERMEDAD PULMONAR OBSTRUCTIVA CRONICA (EPOC) AGUDIZACIONES:
CIENCIAS BASICAS: La denicin de exacerbacin de EPOC no est bien establecida y no existen criterios universalmente aceptados, aunque
generalmente suele describirse como el empeoramiento agudo de los sntomas respiratorios en un paciente con EPOC. La combinacin de sntomas
descrita por Anthonisen y cols. en 1987 (incremento de la disnea, aumento de la produccin y purulencia del esputo) ha sido habitualmente aceptada
para la definicin de exacerbacin pero realmente slo
CLASIFICACION DE LAS EXACERBACIONES DE LA EPOC Y TRATAMIENTO ANTIBIOTICO RECOMENDADO
representa una parte del global. Por ello nos parece ms
GRUPO
DEFINICION
MICROORGANISMOS
TRATAMIENTO ORAL
TRATAMIENTO IV
I
EPOC leve
<4 reagudizaciones/
Amoxi+clavulanico
adecuada la denicin propuesta por Rodrguez Roisn:
Menor
de
65
aos
ao
Telitromicina
Empeoramiento mantenido de la situacin de un paciente con
No comorbilidad
H. influenzae
Levo o moxifloxacino
EPOC, desde un estado basal, por encima de las fluctuaciones
S. pneumoniae
Azitromicina
M. catharralis
diarias, que es agudo en su comienzo y que necesita modificar su
IIa
EPOC moderado o
Igual +
Levo o moxifloxacino
Levofloxacino
tratamiento habitual. Es importante destacar que la tendencia a
grave
enterobacterias
Telitromicna
Ceflasporina 3 y 4
sufrir agudizaciones repetidas es caracterstica de la EPOC. Los
No riesgo para P.
Amoxi+clavulanico
Amoxi +
aeruginosa
clavulanico
pacientes que sufren un nmero elevado de agudizaciones en un
IIb
EPOC moderado o
>4 reagudizaciones/
Ciprofloxacino
Betalactamico
periodo de tiempo siguen con esa tendencia y sufrirn
grave
ao
Levofloxacino
parenteral con
agudizaciones en el futuro. La mayora de las agudizaciones (50Riesgo de P.
Igual IIa + P.
actividad
aeruginosa
aeruginosa
antipseudomona
70%) se debe a infecciones, fundamentalmente bacterianas,
Desnutricion
mientras que el resto puede estar ligado a factores diversos entre
los que se encuentran la polucin, el incumplimiento del rgimen terapu co o la toma no controlada de medicacin depresora del SNC,
hiperrreactividad bronquial, TEP, neumotrax, sedantes. PATOGENIA: La infeccin supone la causa ms frecuente de las agudizaciones (entre el 50-70%
de las agudizaciones podran ser atribuidas a infecciones respiratorias). La explicacin de que en muchas de las mismas no podamos determinar su
etiologa puede deberse a la existencia de infecciones bronquiales no detectadas con las tcnicas diagnsticas habituales. En este sentido, el dato clnico
que mejor indica que la exacerbacin est producida por una infeccin bacteriana es la coloracin amarillo-verdosa del esputo. En cuanto a los
patgenos bacterianos se ha observado que estn presentes entre el 25-80% de las exacerbaciones; pueden tener un papel primario en el desarrollo de
esta o ser una sobreinfeccin de un proceso viral inicial. El Haemophilus influenzae es la bacteria aislada con mayor frecuencia en todas las series (2/3 de
las exacerbaciones en general), seguido del Streptococcus pneumoniae, Moraxella catarrhalis y Pseudomona aeruginosa. Se ha demostrado que un 30%
de las muestras en las que se asla H. in uenza pertenecan a pacientes fumadores ac vos, lo que implica que los fumadores, por presentar una menor
movilidad de los cilios, pueden presentar infecciones respiratorias con mayor frecuencia. Estos mismos grmenes se han obtenido en cultivo de esputo y
en diversas muestras de ms fiabilidad. DIAGNOSTICO: Clnico; Los sntomas ms comnmente referidos por los pacientes con EPOC exacerbada son:
incremento de la disnea, aumento del volumen o purulencia del esputo, aumento de la tos, sntomas referidos a vas areas superiores (odinofagia),
opresin torcica o escasa tolerancia al ejercicio. Adems, se puede aadir la presencia de fiebre, taquicardia, taquipnea, cianosis y sntomas derivados
de la eventual presencia de insuciencia respiratoria. En ocasiones aparece sintomatologa cardiovascular (taquicardia, hipo hipertensin, arritmias y
signos de insuciencia cardiaca derecha) sntomas neurolgicos (cefalea, irritabilidad, depresin, cambios en el ritmo de sueo, obnubilacin y
encefalopata). En la exploracin fsica los datos a resaltar son: uso de la
musculatura accesoria de la respiracin, movimientos paradjicos de la caja
torcica, aparicin o empeoramiento de cianosis, edema perifrico,
alteraciones auscultatorias (sibilancias, roncus, crepitantes), signos de
insuficiencia cardiaca derecha o en casos graves inestabilidad hemodinmica
y disminucin del estado de alerta. El diagnstico de la exacerbacin de la
EPOC es eminentemente clnico. Las pruebas diagnsticas que a continuacin
se enumeran van encaminadas a establecer la severidad de la exacerbacin o
a descartar procesos que pueden simular la exacerbacin y que requieren un
tratamiento especfico. Gasometra arterial; La insuficiencia respiratoria (IR)
se define como la presencia de niveles de PaO2 menor de 55 mmHg y/o PaCO2
mayor de 50 mmHg. El pH es muy til para valorar el grado de hipoventilacin
aguda insertada sobre la hipoventilacin crnica. En la acidosis respiratoria
aguda sin compensacin renal, el pH cae 0,08 por cada 10 mmHg de subida
de la PCO2. En la acidosis respiratoria crnica con compensacin renal normal,
el pH cae 0,03 por cada 10 mmHg de subida de la PaCO2. Hay un grupo de
pacientes con EPOC que tienen valores gasomtricos alterados de forma
basal; en estos casos hablamos de IR cuando se observa un cada de la PaO2
igual o mayor a 10-15 mmHg de su valor basal, y/o cualquier valor de hipercapnia asociado a un pH 7,30. No est indicada la realizacin de gasometra
de entrada en todos los pacientes con exacerbacin; en los casos leves se puede controlar el nivel de oxigenacin determinando la saturacin de O2 con
pulsioximetra. Solo realizaremos determinacin de gases en sangre arterial en aquellos casos de exacerbacin moderada o grave, o en pacientes leves
con saturaciones de O2 bajas. RX de trax, ECG y otras tcnicas de imagen. Los estudios radiolgicos convencionales tienen valor para identificar o
descartar diagnsticos alternativos de enfermedades que pueden cursar con sntomas similares a los de la exacerbacin de la EPOC. El ECG es til para
demostrar la presencia de hipertrofia de ventrculo derecho, arritmias o cardiopata isqumica. El diagnstico del EPOC agudizado en el entorno
extrahospitalario es fundamentalmente clnico: aumento de la disnea, acompaado o no de aumento de la tos, expectoracin o cambios en coloracin
del esputo. TRATAMIENTO: Manejo de exacerbacin; el paciente EPOC agudizado puede ser tratado o bien de forma domiciliaria o en el mbito
hospitalario. Como norma general el paciente EPOC levemoderado ser tratado de forma ambulatoria como primera opcin, salvo si la causa de la
exacerbacin requiere ingreso o si la evolucin no ha sido favorable tras tratamiento ambulatorio. El paciente EPOC grave o muy grave generalmente
requerir ingreso hospitalario, ya que la exacerbacin se asocia con frecuencia a complicaciones de peor pronstico como insuficiencia respiratoria,
insuficiencia cardiaca, neumona, TEP. Indicaciones para ingreso hospitalario en EPOC agudizado: EPOC grave o muy grave, edad avanzada, presencia de
comorbilidad asociada (incluida neumona, arritmia cardiaca, insuficiencia cardaca congestiva, DM, insuficiencia renal o heptica), mala respuesta al
tratamiento ambulatorio, insuficiencia respiratoria (Sat de O2 <90% o PaO2 <60 mmHg), taquipnea > 25 rpm, uso de musculatura accesoria, hipercapnia,
fiebre alta (> 38,5), disminucin del nivel de consciencia Respiratorias De ingresar se usan dosis elevadas de broncodilatador en forma nebulizada
(asociacin de agonista 2 de accin corta (2.5-10mgs) + bromuro de ipratropio (0.5-1mg) cada 4-6hrs), oxigeno suplementario si insuficiencia
respiratoria. Antibioticoterapia si EPOC grave. Glucocorticoides; metilprednisolona IV 0.4-0.6mg/kg cada 6 hrs durante 3-4 das, posteriormente se

CURSO ENARM CMN SIGLO XXI TEL: 36246001

Pharmed Solutions Institute

PGINA 191

MANUAL DE TRABAJO DEL CURSO ENARM CMN SIGLO XXI


reducir en pauta descendente y se pasara va oral cuando sea posible. Indicaciones de ingreso en UCI: Insuficiencia respiratoria aguda en la que a pesar
de FiO2 40 % la PaO2 es menor de 60 mmHg. Hipercapnia progresiva con / sin acidosis respiratoria. Apnea. Fatiga muscular respiratoria
CASOS CLINICOS
Mujer de 84 aos de edad con asma bronquial desde la infancia e
hipertensin arterial en tratamiento con espironolactona y amlodipino.
Un mes antes de su ingreso present una reagudizacin respiratoria, y
se le prescribi por vez primera un compuesto inhalado cada seis
horas, que asociaba bromuro de ipratropio y salbutamol. 2 dias
despus ingresa a urgencias debido a que presenta tos productiva
escasa con dificultad respiratoria, sibilancias crepitantes, diaforesis,
palidez generalizada, cianosis central, se realiza tele de torax donde se
observa imgenes sugerentes de consolidacin y reforzamiento de la
trama broncovascular, rectificacin de diafragma, sin datos de
derrame pleural.

PREGUNTA
Cul es la conducta teraputica medica ms adecuada para este
paciente?
RESPUESTA
a.- Salmeterol/tiotropio
b.- Salmeterol/fluticasona/tiotropio
c.- Trasplante pulmonar
d.- Eritromicina/Salmeterol
CASO CLINICO
Masculino de 75 aos de edad que se presenta con su medico, por que
ha disminuido su tolerancia al ejercicio y falta de aliento. Tienen
antecedente de tabaquismo de 50 cajetillas/ao. El examen fsico
revela trax en tonel, frecuencia cardiaca de 75/min y frecuencia
respratoria 20/min. El examen cardiaco resulta normal; no se
encuenyra distensin venosa yugular. En el examen de torax se
advierte disminucin difusa de los ruidos respiratorios den ambos
campos pulmonares, con sibilancias espiratorias dbiles. La
saturaciopn de oxigeno es de 91%, PO2 65mmHg, HCO3 35mEq/l. Una
radiografia de torax revela hiperinflacin y ampollas subpleurales.

PREGUNTA
Cual es la conducta a seguir mas apropiada?
RESPUESTA
a.- Iniciar con antibitico de amplio espectro.
b.- Microneubilizacion con teofilina.
c.- Aminofilina.
d.- Prednisona.
PREGUNTA
Cul es la principal causa de agudizacin en esta paciente?
RESPUESTA
a.- Infeccion bacteriana
b.- Polucion
c.- Hiperreactividad bronquial
d.- TEP
CASO CLINICO
Masculino de 43 aos, bebedor moderado, fumador de 40 cig/da y
trabajador de una empresa de elaboracin y procesado de fibra de
vidrio, con obesidad. Hace 2 meses sufri un proceso catarral y desde
entonces presenta disnea que ha aumentado progresivamente
asociada a tos escasamente productiva. EF; Constantes vitales
normales, consciente, orientado y colaborador. Auscultacin cardaca:
rtmica, sin soplos. Auscultacin pulmonar: disminucin del MV en
ambos campos pulmonares. Roncus dispersos. Abdomen: globuloso,
blando, depresible y sin visceromegalias. EEII: pulsos conservados, no
edemas ni signos de flebitis ni trombo. Paciente con diagnostico de
EPOC.
PREGUNTA
Al realizar una espirometria a este paciente. Que ndice de FEV1/FCV,
nos indicara limitacin al flujo areo y EPOC?
RESPUESTA
a.- 0.60 0.65
b.- 0.70 - 0.75
c.- 0.80 - 0.85
d.- 0.85 - 0.90
PREGUNTA
Cul es la conducta teraputica angular o prioritaria inicial en el
manejo de este paciente?
RESPUESTA
a.- Dejar de fumar, broncodilatadores, esteroides inhalados
b.- Vacunacin, rehabilitacin, ansiolticos

CURSO ENARM CMN SIGLO XXI TEL: 36246001

d.- Oxigeno, ventilacin mecnica


c.- Mucoliticos, antioxidantes, inmunoterapia oral

PREGUNTA
Cul es el diagnotico ms probable en este caso?
RESPUESTA
a.- Enfisema
b.- Bronquitis crnica
c.- Neumona
d.- Bronconeumonia
PREGUNTA
Cul es la conducnta teraputica menos adecuada en este paciente?
RESPUESTA
a.- Albuterol
b.- Ipratropio
c.- Teofilina
d.- Cese de tabaquismo
PREGUNTA
Cul la medida teraputica mas til para disminuir la mortalidad del
paciente?
RESPUESTA
a.- Rehabilitacion pulmonar
b.- Oxigenoterapia contina
c.- Teofilina
d.- Cese de tabaquismo
PREGUNTA
Cul es la causa ms probable por la que deberamos hospitalizar al
paciente?
RESPUESTA
a.- Edad del paciente
b.- Comorbilidad asociada
c.- Insuficiencia respiratoria
d.- Mala respuesta a tratamiento ambulatorio

Pharmed Solutions Institute

PGINA 192

MANUAL DE TRABAJO DEL CURSO ENARM CMN SIGLO XXI


INSUFICIENCIA RENAL AGUDA (IRA) Y GLOMERULOPATIAS AGUDAS:
CIENCIAS BASICAS: Se denomina IRA a la reduccin brusca, en horas o das, de la funcin renal; se produce una disminucin del filtrado glomerular y un
acmulo de productos nitrogenados sricos (urea y creatinina en sangre) con incapacidad para regular la homeostasis (equilibrio cido-base e
hidroelectroltico). Aunque se suele asociar a una disminucin de la diuresis (IRA oligrica), hasta un 40% de los casos no cursan con oliguria e incluso
puede existir poliuria (IRA no oligrica). La IRA suele presentarse como una complicacin de enfermedades graves previas, apareciendo entre el 5 al 30%
de enfermos hospitalizados. CLASIFICACION: IRA PRERRENAL O FUNCIONAL (60-70%); Inadecuada perfusin renal que compromete el filtrado
glomerular; sera, por tanto, una respuesta fisiolgica a la hipoperfusin renal pero el parnquima renal est ntegro. Es reversible si se acta sobre la
causa desencadenante de manera precoz. Causas de IRA prerrenal son: hipovolemia (hemorragias, perdidas GI, perdidas renales, secuestro de lquidos al
espacio extravascular), disminucin del gasto cardiaco (ICC, TEP, hipertensin pulmonar), vasodilatacin perifrica (sepsis, anafilaxia, antihipertensivos,
anestesia), vasoconstriccin renal (hipercalcemia, norepinefrina, ciclosporina, anfotericina B, sndrome hepatorrenal), alteraciones de las respuestas
autorreguladoras renales (inhibidores de las PG, como los AINES, y/o IECAS). IRA RENAL, PARENQUIMATOSA O INTRNSECA (25%); Dao en las
estructuras anatmicas; se clasifica segn la estructura primariamente daada: glomrulos, tbulos, intersticio o vasos renales. Se dividen las causas de
IRA intrnseca en 4 apartados: 1.Necrosis Tubular aguda (NTA): Causa ms frecuente de IRA intrnseca (70%), afecta a las clulas tubulares renales,
desde lesiones mnimas hasta necrosis cortical, la lesin de los tbulos renales puede ser por mecanismos isqumicos (cursa con oliguria) principalmente
o txicos (aminoglucosidos, cefalosporinas, contrastes radiolgicos, AINES, anestsicos, toxinas endgenas), puede cursar con diuresis conservada,
incluso aumentada. 2. Lesin glomerular: glomerulonefritis agudas y rpidamente progresivas, hipertensin maligna, vasculitis, sndrome hemolticourmico, purpura trombocitopnica trombtica, toxemia del embarazo, esclerodermia. 3. Lesin tubulointersticial: reacciones alrgicas a frmacos
(antibiticos, AINES, diurticos), infecciones (legionella, leptospira, CMV, candidas). 4. Lesin de grandes vasos: obstruccin de arterias renales (placa
ateroesclertica, trombosis, embolia), obstruccin de venas renales (trombosis, compresin). IRA POSRENAL U OBSTRUCTIVA (5%): Lesiones que
produzcan un obstculo en la va urinaria que impida la salida de la orina formada, provocando un aumento de presin que se transmite
retrgradamente, comprometiendo el filtrado glomerular. Pueden ser lesiones extrarrenales de urteres, pelvis (litiasis, tumores, fibrosis), vejiga (litiasis,
cogulos, tumores, prostatismo, vejiga neurgena), uretra (estenosis, fimosis) o tambin lesiones intrarrenales (depsito de cristales, cogulos, cilindros).
Para que estas causas produzcan una IRA es necesario que la obstruccin sea grave, prolongada y que afecte a tracto urinario distal (meato uretral
externo, cuello de la vejiga) o bien a los urteres de manera bilateral o unilateral en paciente con un nico rin funcionante. DIAGNSTICO: Clnica de
las diferentes formas de IRA depender de las causas desencadenantes. As, en la forma prerrenal destacaran las manifestaciones de reduccin
verdadera de volumen (sed, hipotensin, taquicardia, disminucin de la presin venosa yugular, disminucin de peso, sequedad de piel y mucosas) o de
reduccin efectiva de volumen (en este caso la exploracin revelar signos de hepatopata crnica, insuficiencia cardaca avanzada, sepsis). Es
importante destacar que en estos casos la IRA desaparece rpidamente tras reestablecer la perfusin renal. En el caso de la forma renal o intrnseca hay
que investigar la presencia de isquemia renal prolongada (shock hipovolmico, shock sptico, ciruga mayor). En estos casos existe oliguria o incluso
anuria (diuresis diaria < 100 ml). La probabilidad de que estemos ante un cuadro de NTA aumenta an ms si la IRA persiste a pesar del
reestablecimiento de la perfusin renal. La posibilidad de IRA nefrotxica requiere el estudio de los medicamentos que ha recibido recientemente el
paciente, exposicin a contrastes radiolgicos (sobre todo si se han realizado en pacientes de riesgo: ancianos, diabetes mellitus, deshidratacin previa,
mieloma mltiple). Las toxinas tambin pueden tener un origen endgeno como la mioglobina (tras una rabdomilisis, por destruccin muscular aguda)
o hemoglobina (tras hemolisis grave). Destacar que en la NTA por txicos la diuresis suele estar conservada. En la forma posrenal la causa ms frecuente
en el varn es la obstruccin del cuello de la vejiga por una enfermedad prosttica (hiperplasia o carcinoma).La diuresis fluctuante es caracterstica de la
uropata obstructiva. Laboratorios: BH: urea, creatinina, glucosa, iones. CK. La caracterstica fundamental de la IRA es la aparicin de uremia aguda de
rpida aparicin. A nivel prctico se considera que esto ocurre cuando la creatinina plasmtica aumenta 0,5 mg/dl/da durante varios das. Si la IRA
ocurre en el seno de una IRC, se considera que el aumento debe ser mayor de 1 mg/dl/da. La creatinina es ms fiable que la urea para la el diagnstico
de IRA. Tambin puede calcularse el grado de disfuncin renal detectando el deterioro del aclaramiento de
creatinina. Para ello, en urgencias, se puede usar esta frmula, de aclaramiento de creatinina (Ccr), es una
prueba aceptada como medida de filtrado glomerular. El valor normal de Ccr es de 100-120 ml/min. En el caso de IRA el Ccr calculado debe reducirse un
50%. Existir hiperpotasemia en casos de IRA oligrica o en estados hipercatablicos, como sucede en la hemlisis, rabdomiolisis y en los casos de lisis
tumoral. La hipopotasema se da en las formas poliuricas. La hiponatremia es tambin un hallazgo frecuente. Un manejo incorrecto del paciente, con un
aporte excesivo de agua en proporcin a la de sodio, puede agravar an ms la hiponatremia. El aumento del cido rico es caracterstico de la IRA
aunque habitualmente es moderado y asintomtico. Suele existir hipocalcemia, hiperfosforemia e hipermagnesemia. La severidad de estas alteraciones
ser paralela a la del dao renal que las ha ocasionado. Hemograma: Puede tener gran importancia en el diagnstico diferencial entre IRA e IRC: as si
aparece una anemia normoctica normocrmica, estar ms en concordancia con una IRC. Gasometra arterial: El patrn cido-base ms frecuente del
fracaso renal agudo es la acidosis metablica ya que el rin es incapaz de eliminar los cidos fijos no voltiles. Volumen urinario: debido a las
variaciones en la diuresis de las distintas formas de IRA en general no tiene gran valor diagnstico, aunque si sirve para clasificar la IRA como oligrica y
no oligrica Sedimento urinario: En la IRA prerrenal el sedimento no contiene clulas pero si cilindros hialinos. En NTA existen cilindros granulosos,
pigmentados y de clulas epiteliales, generalmente en asociacin con hematuria microscpica. Proteinuria: suele verse en la NTA, es de tipo tubular y
menor de 1 gr/24 h. Sistemtico de orina: el estudio de iones, urea, creatinina, osmolaridad y densidad junto al sedimento urinario son fundamentales
para el diagnstico difrencial de IRA prerrenal de NTA. ECG: Puede orientar hacia trastornos electrolticos sobre todo la hiperpotasemia (prolongacin del
intervalo PR, ensanchamiento del QRS y aplanamiento de la onda T) o hipocalcemia. La Rx. simple de abdomen informa sobre la existencia de litiasis
radiopaca y el tamao y silueta renal y con la radiografa de trax se puede valorar la existencia de sobrecarga de lquidos (edema agudo de pulmn).
Ecografa abdominal: diferencial de la IRA. Se puede descartar patologa obstructiva as como visualizar el tamao renal, dato muy importante para
distinguir entre IRA e IRC. TRATAMIENTO: Manejo de IRA prerrenal. Dieta rica en hidratos de carbono y con aporte de protenas de alto valor biolgico
entre 0,6-0,8 gr/Kg/da. Monitorizar tensin arterial, frecuencia cardiaca y medicin de la ingesta y prdida de agua y sal. El mecanismo de control ms
simple es el peso diario. El sondaje vesical ser necesario si se precisa la medicin de diuresis horaria. Canalizar va perifrica y central y monitorizar
presin venosa central (PVC), para ajustar el aporte de lquidos a una PVC entre 4-8 cmH2O. Si no existe contraindicacin, se puede realizar una
rehidratacin rpida (en unos 30 min) con 500-1000 ml de SF, controlando la presin arterial, la PVC y vigilando la respuesta clnica y diurtica. Si existe
una prdida hemtica grave se usar concentrado de hemates. Una vez corregida la volemia, el volumen urinario aumenta y se debe continuar con
reposicin de lquidos a ritmo de diuresis. Si hay una mala evolucin en observacin ser necesaria la consulta con el nefrlogo que valorar la indicacin
de dilisis. Manejo de NTA: una serie de medidas conservadoras pueden, sino evitar la necesidad de dilisis, si al menos transformar una IRA oligrica en
otra no oligrica lo que permitir un mejor manejo del enfermo. Para ello, una vez adoptadas las medidas generales del apartado anterior se forzar la
diuresis por medio de diurticos y dopamina. As se puede intentar el uso de furosemida a dosis de 20 a 40 mg cada 6 horas, segn los valores de diuresis
y creatinina. Tambin pueden usarse diurticos osmticos como el manitol al 20%, a dosis de 80 ml cada 6-8 horas, en funcin de la respuesta. La
dopamina a dosis de 3-5 g/kg/min puede ser otra herramienta til para mejorar la perfusin renal. Para ello se diluye una ampolla de 200 mg en 250 de

CURSO ENARM CMN SIGLO XXI TEL: 36246001

Pharmed Solutions Institute

PGINA 193

MANUAL DE TRABAJO DEL CURSO ENARM CMN SIGLO XXI


glucosado y se perfunde a 5-10 gotas/min. El mecanismo de la dopamina es aumentar el filtrado glomerular y favorecer la accin de los diurticos. Se
deben usar antagonistas de los receptores de histamina H-2 para la prevencin de hemorragias digestivas. No se recomienda el uso de antibiticos de
manera profilctica. Manejo de la IRA posrenal: Definitivo es desobstruir. De manera transitoria se proceder al sondaje vesical en el caso de patologa
prosttica. En caso de obstruccin ureteral con hidronefrosis ser necesario la realizacin de nefrostomia percutanea. Las indicaciones de dilisis en la
IRA son: hiperpotasemia o hipontremia graves, acidosis metablica con bicarbonato plasmtico menor de 10 mEq/l, sobrecarga de lquidos con edema
pulmonar o insuficiencia cardiaca, pericarditis urmica, encefalopata urmica, ditesis hemorrgica urmica y azotemia severa (urea > 250 mg/dl o
creatinina > 10 mg/dl). GLOMERULOPATIAS AGUDAS (GNA): Se caracteriza por inicio abrupto de hematuria macroscpica, oliguria, falla renal,
disminucin sbita de las tasa de filtracin glomerular con retencin de sodio y agua, manifestando edema e hipertensin y proliferacin de clulas
endocapilares del glomerulo. La proteinuria vara ampliamente en este sndrome y por lo general es menos de 3 g/dl. La principal es la post-infecciosa,
donde el dao glomerular resulta de un compromiso inmune, desencadenado por una variedad de infecciones bacterianas, virales o de protozoarios. La
ms comn es la postestreptoccica, afecta a nios entre los 2 y 10 aos, predomina ligeramente en hombres. Slo ciertas cepas nefritognicas de
estreptococo se asocian con la glomerulonefritis. La variedad ms comn de glomerulonefritis postestreptoccica es usualmente despus de una
infeccin farngea con estreptococo beta-hemoltico del grupo A. En los pases desarrollados, la nefropata IgA es, en la actualidad, la ms frecuente de
las glomerulonefritis primarias. La glomerulonefritis postestreptoccica es una enfermedad aguda y reversible, caracterizada por recuperacin
espontnea en la mayora de los pacientes. Tpicamente la hematuria gruesa y el edema se presentan entre 7 das a 12 semanas despus de la infeccin
estreptoccica. La resolucin espontnea de las manifestaciones clnicas es por lo general rpida. La diuresis reaparece en una a dos semanas y la
concentracin de creatinina srica retorna a nivel basal dentro de las cuatro semanas. La hematuria microscpica desaparece dentro de los seis meses
pero la proteinuria leve permanece esttica en 15 % de los pacientes despus de 3 aos. El hallazgo de anticuerpos contra antgenos estreptoccicos
proporciona evidencia de infeccin reciente, pero no es diagnstica de glomerulonefritis postestreptoccica. Son los ms solicitados los anticuerpos
antiestreptolisina o, antiestreptoquinasa, antihialuronidasa y antinicotinamina dinucletidasa. El tratamiento de la glomerulonefritis postestreptoccica es
de soporte, enfocado a la sobrecarga de fluidos con dieta hiposdica (2.4 g/sodio/da); la hipertensin responde a diurticos de asa (furosemide) y
antihipertensivos tipo IECA, pero se deben dar con precaucin si hay insuficiencia renal con hipercalemia. La terapia antimicrobiana temprana del
paciente con infeccin farngea o de la piel, puede prevenir la propagacin de la infeccin estreptoccica y atena la severidad de la glomerulonefritis
postestreptoccica, pero no previene el desarrollo de la misma. SNDROME NEFRTICO: Criterios diagnsticos Proteinuria >3-3,5g/24 horas o una
relacin protena: creatinina >3-3,5 (ambas en mg/dL) en una muestra aislada. Albuminemia <25 g/L (hipoalbuminemia). Edemas perifricos. Es
frecuente la hiperlipidemia severa (colesterol total >385 mg/dL). Es consecuencia del aumento de la permeabilidad para las protenas y es expresin de
una enfermedad glomerular cuando existe una alteracin funcional o morfolgica de la barrera de ltracin. Nunca aparece en las enfermedades
extraglomerulares. El sndrome nefrtico constituye el motivo ms frecuente de realizacin de biopsia renal en todos los grupos de edad.
Manifestaciones clnicas: La prdida de protenas en orina origina una serie de alteraciones que constituyen las caractersticas del sndrome nefrtico: 1.
Hipoalbuminemia: aparece cuando la capacidad de sntesis heptica se ve superada por las prdidas urinarias de albmina y el catabolismo renal. 2.
Edema: suele ser la primera manifestacin; aparece en partes blandas y en casos graves puede comportar ascitis, derrame pleural y anasarca,
perorbitario, en miembros inferiores. Es debido a la reabsorcin de sodio y agua secundaria a la disminucin de la presin onctica capilar o estimulada
directamente por la proteinuria. 3. Hiperlipidemia con aumento de las cifras de colesterol total, lipoprotenas de densidad baja (LDL), muy baja (VLDL) e
intermedia (IDL), lipoprotena A y, con menos frecuencia, hipertrigliceridemia y descenso de las cifras de lipoprotenas de alta densidad (HDL). La lipiduria
se maniesta con aparicin de cilindros grasos en el sedimento urinario. 4. Prdida de inmunoglobulinas y de factores del complemento, que conlleva
una tendencia especial a la aparicin de infecciones (peritonitis espontneas, celulitis, e infecciones pulmonares, menngeas y digestivas). 5. Trombosis,
con una incidencia de entre el 5 y el 60%; suele aparecer en venas renales y de extremidades inferiores, aunque tambin pueden afectarse territorios
arteriales. 6. Hipertensin arterial, que aparece en el 42,5% de los pacientes. 7. Insuciencia renal aguda, que se observa con ms frecuencia en
pacientes de edad avanzada, con hipoalbuminemia grave, tratados con dosis altas de diurticos. Causas de sndrome nefrtico: Enfermedad glomerular
primaria; esclerosis glomerular segmentaria focal, enfermedad glomerular membranosa, enfermedad glomerular con cambios minimos, enfermedad
glomerular membranoproliferativa (ej: IgA). Causas secundarias de sndrome nefrtico: DM, LES, amiloidosis, mieloma multiple, agentes antimicrobianos,
AINES, penicilamina, VIH, hepatitis B y C, mycoplasma, sfilis. La biopsia renal est indicada en el estudio del sndrome nefrtico del adulto, excepto en
pacientes diabticos en los que no se sospeche otra enfermedad distinta de la nefropata diabtica. En nios con sndrome nefrtico, en principio no est
indicada la realizacin de biopsia renal (el 90% de los casos se debe a glomerulonefritis de cambios mnimos), salvo en caso de sndrome nefrtico
resistente a los corticosteroides o con recidivas frecuentes. El tratamiento general comprende: reduccin del edema, control de la presin arterial y del
perl lipdico y prolaxis de la trombosis. SNDROME NEFRTICO: Se caracteriza por edema, oliguria, hematuria (con cilindros hemticos en el sedimento
urinario), disminucin del ltrado glomerular e hipertensin arterial, normalmente de instauracin aguda. Tpicamente se ha relacionado con la forma de
presentacin de la glomerulonefritis aguda posinfecciosa, sobre todo asociada a infecciones estreptoccicas. Tambin puede aparecer en otras
afecciones. Manifestaciones clnicas: Aparece como macrohematuria en el 30-50% de los casos, con datos de hematuria glomerular. En la
glomerulonefritis aguda posestreptoccica se presenta tras 2-3 semanas de una infeccin faringoamigdalar o 4-6 semanas de una infeccin cutnea. La
hipertensin arterial suele ser moderada, con edemas en prpados y extremidades inferiores. En algunos casos puede producirse insuciencia cardaca
congestiva. La proteinuria es inferior a 1-2 g/24 h. Es fundamental la realizacin de una historia clnica completa, interrogando sobre antecedentes
familiares, manifestaciones clnicas sistmicas, infeccin reciente o consumo de frmacos. En la exploracin fsica valoraremos la presencia de edemas,
hipertensin arterial o datos de insuciencia cardaca. La ausencia de manifestaciones clnicas sistmicas, el antecedente de infeccin, la
hipocomplementemia y ttulos elevados de anticuerpos antiestreptolisina establecen usualmente el diagnstico de glomerulonefritis aguda
posestreptoccica. La biopsia renal estara indicada si existiera algn dato que hiciera dudar del diagnstico o en caso de insuciencia renal progresiva. Si
se sospecha otro tipo de infecciones o se observan manifestaciones clnicas sistmicas, estara justicado el estudio serolgico, la realizacin de cultivos y
el estudio de autoanticuerpos o inmunoglobulinas. Tratamiento: consiste en tratar la infeccin, o especcamente el trastorno subyacente si se debe a
otra entidad. Las causas ms comunes de sndrome nefrtico: Enfermedades sistmicas; LES, endocarditis, absceso viscerales, nefritis por cortocircuito,
crioglobulinemia, poliarteritis nodosa, vasculitis por hipersensibilidad, granuloma de Wegener, purpura de Henoch-Schonlein. Enfermedades renales;
glomerulonefritis aguda postinfecciosas, glomerulonefritis membranoproliferativa, nefropata por IgA. El tratamiento general del sndrome nefrtico se
basa en el manejo adecuado del volumen y el control tensional. Para el control del edema se emplean diurticos de asa y restriccin hidrosalina. En caso
de insuciencia cardaca o edema agudo de pulmn, se emplea oxgeno, morna y nitroglicerina. El control tensional se consigue con bloqueadores del
sistema renina-angiotensina y diurticos. En caso de uremia grave, hiperpotasemia y/o insuciencia cardaca refractaria puede estar indicada la dilisis.
CASO CLINICO INSUFICIENCIA RENAL
Varn de 18 aos, cuidador de animales, refiere un cuadro febril de
larga evolucin; en dicho ingreso desarrolla un fracaso renal agudo de

CURSO ENARM CMN SIGLO XXI TEL: 36246001

evolucin subaguda con creatinina plasmtica (crp) de 6 mg/dl. Refera


fiebre elevada de dos meses de evolucin, que haba aparecido una
semana despus de sufrir una dudosa picadura de garrapata. La fiebre

Pharmed Solutions Institute

PGINA 194

MANUAL DE TRABAJO DEL CURSO ENARM CMN SIGLO XXI


se presentaba diariamente sin predomino horario y con sudoracin
nocturna. Haba recibido distintos tratamientos antibiticos de forma
emprica. En el momento del ingreso llevaba 20 das sin tratamiento
antibitico persistiendo el cuadro febril. Radiografa de trax: al
ingreso, normal; un mes despus presenta pequeos infiltrados
pulmonares bibasales con imgenes alveolares. La proteinuria que
inicialmente fue de rango no nefrtico (1-2 g/24 h), posteriormente
alcanza el rango nefrtico.
PREGUNTA
Cul es la manifestacin clnica menos probable de encontrar en este
momento en el paciente?
RESPUESTA
a.- Derrame pleural
b.- Anasarca
c.- Hipertension
d.- Hematuria
PREGUNTA
Cual es la conducta a seguir mas adecuada para establecer el
diagnostico final?
RESPUESTA
a.- Gamagrama pulmonar.
b.- Biopsia renal.
c.- Urocultivo.
d.- Hemocultivo.
CASO CLINICO
Femenino de 34 aos, con antecedentes de madre y padre diabtico,
cuadro de faringoamigdalitis hace una semana, nos refiere que el dia
de ayer por la maana corri un maratn de 10K, para ganar una
apuesta. Acude al servicio de urgencias refiriendo que la noche
anterior orino en 2 ocasiones rojo oscuro, fatiga, dolor muscular,
nuseas y vomito en una ocasin. EF: orientada y consciente, mucosas
con deshidratacin moderada, T/A 100/60mmHg, FC 78x, FR 18x, Hb
12.5mg/dl, glucosa 165mg/dl, Cr 2.0mg/dl, Na disminuido.
PREGUNTA
Cul es el diagnstico ms probable en este caso?
RESPUESTA
a.- Glomerulonefritis rpidamente progresiva
b.- Insuficiencia prerrenal
c.- Necrosis tubular aguda
d.- Glomerulonefritis postestreptococica

PREGUNTA
Egresa paciente luego de haber resuelto su cuadro actual. Cul de los
siguientes factores es el ms til para evitar un nuevo episodio?
REPUESTA
a.- Controles cada 6 meses de glucosa en ayuno
b.- No usar aminoglucocidos por ms de 5 das
c.- Hidratacin adecuada e inicio gradual de ejercicio
d.- Evitar ejercicio, dieta alta en protenas
PREGUNTA
Cul es la fraccin excretada de Na (FeNa), que ms probable
encontraramos en este caso?
RESPUESTA
a.- 6%
b.- 1%
c.- 0.5%
d.- 0.3%
CASO CLINICO
Paciente de 75 aos con antecedente de hipertensin arterial,
diabetes mellitus y dislipidemia controlados en tratamiento con
losartan, metformina mas insulina, benzafibrato y prazosina. Es llevado
por familiar a urgencias el cual nos refiere que el paciente presenta
visin borrosa, tiene 48hrs sin orinar, ni defecar, sudoracin leve, no
pudo dormir durante la noche, dificultad para tomar los objetos y un
poco desorientado, as como dolor moderado en regin de hipogastrio
. EF: Buena coloracin de tegumentos, desorientado en espacio, con
T/A 150/90mmHg, FC 90lpm, FR 18X, cardiopulmonar sin
compromiso, abdomen blando y depresible, ligero dolor y dureza en
hipogastrio, resto sin alteraciones. Laboratorio: glucosa 140mg/dl, Cr
2.0mg/dl, colesterol 180mg/dl, trigliceridos 149mg/dl, TGO, TGP
normales.
PREGUNTA
Cul es el diagnstico ms probable para esta paciente?
RESPUESTA
a.- Delirium
b.- Insuficiencia renal aguda
c.- Cetoacidosis diabtica
d.- EVC

PREGUNTA
Cul es la conducta teraputica ms adecuada a seguir?
RESPUESTA
a.- Dilisis peritoneal
b.- Bicarbonato y potasio
c.- Reposicin volumtrica
d.- Penicilina G Benzatinica
PREGUNTA
Cul de los siguientes factores de riesgo es ms importante interrogar
a la paciente?
RESPUESTA
a.- Tratamiento con aminoglucocidos
b.- Consumo de esteroides
c.- Consumo de metilfenidato
d.- Bajo consumo de soluciones hidratantes
PREGUNTA
Cul de las siguientes protenas es ms probable que participe en la
fisiopatogenia del diagnstico en este caso?

CURSO ENARM CMN SIGLO XXI TEL: 36246001

RESPUESTA
a.- Hemoglobina y disminucin de flujo sanguneo renal
b.- Albumina y drogas nefrotxicas
c.- Mioglobina y disminucin de flujo sanguneo renal
d.- Hemoglobina, reaccin antgeno-anticuerpo

PREGUNTA
Cul es la conducta teraputica ms adecuada a seguir en este
momento?
RESPUESTA
a.- Haloperidol
b.- Colocacin de sonda vesical
c.- Sol. Fisiolgica, insulina, bicarbonato
d.- Anticoagulante
PREGUNTA
El paciente acude 3 meses despus por dolor de moderado a severo,
secundario a diagnstico de osteoartrosis. Cul sera el manejo ms
adecuado para este paciente?
RESPUESTA
a.- Tramadol
b.- Diclofenaco
c.- Dexametasona
d.- Paracetamol

Pharmed Solutions Institute

PGINA 195

MANUAL DE TRABAJO DEL CURSO ENARM CMN SIGLO XXI


TRASTORNOS ACIDO-BASE AGUDOS
CIENCIAS BASICAS: Cuando se habla de equilibrio acido-base en realidad se hace referencia a la regulacin de la concentracin de hidrogeniones en los
lquidos corporales. Pequeos cambios en la concentracin de iones hidrogeno pueden producir grandes alteraciones en las reacciones qumicas
celulares, aumentando algunas e inhibiendo otras; por este motivo la regulacin de la concentracin de iones de hidrogeno es uno de los aspectos ms
importantes de la homeostasis. El pH sanguneo arterial se
TRASTORNOS PRIMARIOS Y RESPUESTAS COMPENSADORAS
mantiene entre 7.35-7.45, cualquier desviacin de este rango
TRASTORNO
ALT. PRIMARIA
RESPUESTA COMPENSADORA
implica desequilibrio acido-base. El pH plasmtico se refiere
Acidosis metablica
[HC3]
PaCO2 desciende 1,2mmHg por cada 1
habitualmente a la relacin entre las concentraciones de
mEq/l de aumento de la [HC3]
bicarbonato/cido carbnico. El CO2, en presencia de anhidrasa
Alcalosis metablica
[HC3]
PaCO2 aumenta 0,7mmHg por cada 1
mEq/l de descenso de la [HC3]
carbnica (AC), se hidrata de la siguiente forma: CO2 +
Acidosis respiratoria
PaCO2
Aguda: [HC3] aumenta 1mEq/l por cada
H2O H2CO3 H+ + HCO3 . Al valor de pH arterial por debajo
10mmHg de aumento de PaCO2
de 7.35 se denomina acidemia y cuando est por arriba de 7.45,
Crnica: [HC3] aumenta 3,5mEq/l por
alcalemia; la alteracin que cursa con niveles de pH por debajo de
cada 10mmHg de aumento de PaCO2
7.35 se le denomina acidosis y la que cursa con niveles de pH por
Alcalosis respiratoria
PaCO2
Aguda: [HC3] desciende 2mEq/l por
arriba de 7.45 se denomina alcalosis. Factores de riesgo: Se
cada 10mmHg de desciende de PaCO2
recomienda tener alto ndice de sospecha
para detectar
Crnica: [HC3] desciende 5mEq/l por
oportunamente trastornos del equilibrio acido-base en las
cada 10mmHg de aumento de PaCO2
Valores normales: sangre arterial pH 7,36-7,44; PaCO2 36-44mmHg; [HC3] 22-26mmHg
personas con factores de riesgo para desarrollo. El equilibrio cidoSangre venosa pH 7,31-7,37; PaCO2 42-50mmHg; [HC3] 23-27mmHg
base requiere la integracin de tres sistemas orgnicos: Hgado,
Pulmones, Rin. El hgado metaboliza las protenas produciendo iones hidrgeno (H+), el pulmn elimina el dixido de carbono (CO2), y el rin
generando nuevo bicarbonato (HCO3). Un cido es una sustancia capaz de donar un H+ y una base una sustancia capaz de aceptarlo. Por tanto, la acidez
de una solucin depende de su concentracin de hidrogeniones [H+]. SISTEMAS DE AMORTIGUAMIENTO: El funcionamiento celular requiere mantener
la concentracin de H+, del lquido extracelular (LEC) en lmites muy estrechos (el pH compatible con la vida esta entorno a 6.80-7.80). Dado que los
procesos metablicos generan gran cantidad de cidos, el organismo necesita neutralizar y eliminar los H+ para mantener constante el pH (-log [H+]) del
LEC. Para ello, dispone de varios medios: 1. Amortiguadores plasmticos. Bicarbonato El H+ se une al HCO3- en forma reversible, cuando el aporte o la
produccin de H+ aumenta, la reaccin se desplaza hacia la derecha, con lo que incrementa la cantidad de H+ que es captado por el amortiguador, lo que
minimiza los cambios de la H+. El HCO3- representa el 50% de la capacidad amortiguadora del plasma. Si la produccin de cidos no voltiles excede la
excrecin, el HCO3- disminuye y la H + aumenta, resultando en acidosis metablica, por otra parte si la excrecin de HCO3- es mayor que la produccin el
HCO3- aumenta y la H+ disminuye resultando en alcalosis metablica.
+
H + HCO3- H2CO3
H2O + CO2- Hemoglobina (Hb), Protenas y
fosfatos. Existen otros sistemas de amortiguamiento como la Hb,
protenas y fosfatos, los cuales proveen de sitios adicionales de unin
de H+ y por lo tanto amortiguamiento. La Hb proporciona el 30% de la
capacidad amortiguadora del plasma, el restante 20% lo comparten
las protenas y los fosfatos (13 y 7% respectivamente). 2. Respuesta
respiratoria El segundo sistema de amortiguamiento que hace frente
a los trastornos del equilibrio AB es el pulmonar; la disminucin en el
pH acta estimulando quimiorreceptores en el tallo cerebral con
incremento en la ventilacin minuto y eliminacin del CO2. 3.
Respuesta renal. El rin es el tercer sistema de amortiguamiento,
para mantener el equilibrio AB, stos deben de excretar aniones de
los cidos no voltiles y reabsorber el HCO3-, esto lo logran por medio
de tres mecanismos: reabsorcin o excrecin del bicarbonato
filtrado, excrecin de acidez titulable y excrecin de amoniaco. En
definitiva, segn la ley de accin de masas, la acidosis (aumento de H +) puede producirse por una disminucin del bicarbonato (acidosis metablica) o
por un aumento de la PaCO2 (acidosis respiratoria); y la alcalosis (disminucin de los H+) por un aumento del bicarbonato (alcalosis metablica) o por una
disminucin de la PaCO2 (alcalosis respiratoria). As pues la alcalosis o la acidosis son estados en los que existe un acumulo de bases o cidos. Por otra
parte en todo trastorno acidobsico se producen necesariamente respuestas compensadoras (renales en trastornos respiratorios y respiratorios en
trastornos metablicos), que intentan mantener normal el pH. ANION GAP: La neutralidad qumica del plasma se mantiene por el equilibrio entre
aniones (Cl 103; HCO3 24; otros aniones 10; protenas 17 =154) y cationes (Na 143; K 4.5; Ca 5.0; Mg 1.5 = 154) excretados y retenidos. El nmero total
de Cationes plasmticos debe igualar a los aniones. Los aniones medidos en el laboratorio son inferiores a los cationes medidos. Esta diferencia se
denomina ANION GAP. El anion gap normal es de 12+/- 2. Una AG superior indica la presencia anormal de cidos. AG = cationes suma de aniones. AG=
Na (Cl + HCO3). Segn el valor de anin GAP calculado las acidosis metablicas pueden dividirse en dos grupos: Con anin GAP elevado (por adicin de
cidos fijos). Con anin GAP normal o hiperclormicas (por perdida de Bicarbonato). El aumento en la concentracin de cloro se debe a que si el
bicarbonato desciende, el Cloro aumenta para mantener la electroneutralidad del medio. Correccin del anin GAP: Segn valor de albmina: por cada
g/dl de albmina por encima de 4 se suma al anin GAP calculado 2 puntos y, por cada g/l por debajo de 4 se restan 2 puntos.
ACIDOSIS METABOLICA
Ej: Ph: 7.32; PCO2: 45; HCO3: 16 (aguda).Ph:7.34; PCO2: 28;
HCO3: 18 (subaguda). Ph: 7.36; PCO2: 34; HCO3: 19 (crnica)

Clnica

Causas

Anorexia, Fatiga,
Deshidratacin,
Confusin,
Letargia, Estupor,
Coma
Taquicardia,
Hipotensin
arterial sistmica,

Anin GAP elevado (normocloremicas)


Cetoacidosis diabtica
Acidosis lctica
Insuficiencia renal
Intoxicaciones (salicilatos, etanol, metanol,
formaldehido)
Rabdomiolisis
Anin GAP normal (hipercloremicas)

MANIFESTACIONES CLINICAS
ACIDOSIS RESPIRATORIA
ALCALOSIS METABOLICA
Ej.: Ph: 7.24; PaCo2: 55; HCO3:
Ej: Ph: 7.48; PCO2: 35; HCO3: 26
23 (aguda). pH 7.33; PaCO2: 53;
(aguda). Ph: 7.48; PCO2: 50; HCO3: 28
HCO3: 28 (subaguda). pH: 7.35,
(subaguda). Ph: 7.45; PCO2: 50; HCO3:
PaCo2: 50, HCO3: 28 (crnica)
35 (crnica)
Clnica
Causas
Clnica
Causas
Confusin,
Coma,
Convulsiones,
Alteraciones
del
ritmo
cardiaco,
Hipotensin
arterial,

CURSO ENARM CMN SIGLO XXI TEL: 36246001

EPOC,
neumona,
asma, SDRA
Sedacin
Hipoventilacin
mecnica
Enfermedad
neuromuscular

Apata,
Vomito,
Bulimia,
Confusin,
Arritmias
cardiacas,
Hiperreflexia,
Clonus,

Perdidas
digestivas
(vomitos, aspiracin
nsaogastrica, diarrea)
Diurticos
Hiperaldosteronismo
1ro y 2ro.
Dieta pobre en sal
Hipercalcemia

Pharmed Solutions Institute

ALCALOSIS RESPIRATORIA
Ej: Ph:7.50; PaCO2: 30; HCO3:23
(aguda). Ph: 7.46; PaCo2: 27;
HCO3: 19 (subaguda)
Clnica

Causas

Vrtigo,
Mareo,
Ansiedad,
Euforia,
Alucinaciones,
Alteraciones
del estado de
conciencia,

Ansiedad
Hipoxia
Hiperventilacin
Dolor
Fiebre
Enfermedad del
tronco cerebral

PGINA 196

MANUAL DE TRABAJO DEL CURSO ENARM CMN SIGLO XXI


Taquipnea,
Disnea, Fatiga de
msculos
respiratorios
Respiracin
de
Kussmaul

Perdidas digestivas de bicarbonato (diarrea,


fistulas pancreticas, biliares)
Perdidas renales de bicarbonato (acidosis
tubulares renales, hioperparatiroidismo,
frmacos; acetazolamida, anfotericina,
ciclosporina)

Cefalea

(Guillan-Barre,
esclerosis
mltiple, crisis
miastenica)
Neumonia
Derrame pleural
masivo

Convulsiones,
Hiporreflexia

Hipopotasemia
Administracion
de
bicarbonato sdico
Transfusiones masivas
de sangre

Mioclonus,
Asterixis,
Taquicardia

CASO CLINICO ACIDOSIS LACTICA


Mujer de 81 aos con hipertensin arterial, dislipemia, diabetes tipo 2
y miocardiopata dilatada (fraccin de eyeccin [FE] del 30%).
Tratamiento habitual: telmisartn, metformina a dosis de 850 mg/8
horas, atorvastatina, bezafibrato y omeprazol. Acude a urgencias por
diarrea mucosanguinolenta y vmitos de una semana de evolucin
acompaados de oligoanuria en las ltimas 24 horas. TA, 120/70
mmHg, FC de 95 lpm, temperatura 36 C. Glasgow 12, desorientacin
temporoespacial y bradipsiquia, sin signos de focalidad.

enfermedad del ndulo sinusal, claudicacin intermitente y con


valvulopata artica, estenosis carotdea y cardiopata isqumica
intervenida. Creatinina de 1.56 mg/dl. En la exploracin fsica destaca
soplo carotdeo derecho, pulsos poplteos dbiles y ausencia de pulsos
tibiales y pedios como hallazgos patolgicos. BH con (Hb 10.4 mg/dl,
VCM 91 fl, HCM 30.6 pg y CHCM 31 gr/dl), y en la bioqumica
plasmtica potasio 3.7 mEq/l. En la gasometra venosa: (ph 7,461) y
(CO3H 29.5 mEq/l), con (pCO2 medida 42.3 mmHg, con PCO2 estimada
de 43,85 +/- 2 mmHg).

PREGUNTA
Cual es la prueba mas apropiada para establecer el diagnostico agudo
del paciente?
RESPUESTA
a.- Realizar TAC.
b.- Realizar IMR.
c.- QS y BH.
d.- Gases arteriales.

PREGUNTA
Cual es el manejo ms adecuado para corregir los gases?
RESPUESTA
a.- Disminuir Fi02
b.- Disminuir FR.
c.- Disminuir volumen ventilarotio.
d.- Disminuir volumen residual.

CASO CLINICO ACIDOSIS LACTICA


Analtica: hemoglobina 11,7 g/dl, leucocitos 18.030 (78,9%
neutrfilos), plaquetas 307.000, glucosa 68 mg/dl, urea 133 mg/dl,
creatinina 6,89 mg/dl, sodio 134 mEq/l, potasio 4,4 mEq/l, pH 6,89,
pCO2: 29 mmHg, bicarbonato 6,9 mmol/l, calcio inico 3,85 mg/dl,
anin gap 28. Coagulacin normal. Orina: pH 6, creatinina 71 mg/dl,
proteinuria 400 mg/dl, 100 hemates/campo, 60 leucocitos/campo,
cuerpos cetnicos positivos.
PREGUNTA
Cul es la conducta teraputica ms adecuada.
RESPUESTA
a.- Bicarbonato.
b.- Solucion salina.
c.- Solucion glucosada.
d.- Dobutamina.
CASO CLINICO ACIDOSIS METABOLICA
Varn de 75 aos, hipertenso, con enfermedad pulmonar obstructiva
crnica. Consult por cuadro de infeccin respiratoria y deterioro
funcional asociado consistente en apata, somnolencia y perodos de
agresividad. A la exploracin afebril, desorientado, con tendencia al
sueo, taquipnea superficial, roncus dispersos y crepitantes en la base
izquierda, con imagen radiolgica sugerente de bronquiectasias
sobreinfectadas. Cloro (114 mEq/l), con funcin renal y resto de iones
normales (sodio: 138 mEq/l; potasio: 4,2 mEq/l), y una gasometra
arterial (pH: 7,24; pCO2: 33 mmHg, pO2: 67 mmHg; bicarbonato: 17
mmol/l, y exceso de bases (EB): 9,1 mmol/l). El valor del anin GAP
(diferencia entre el valor del sodio srico y la suma del cloro y el
bicarbonato) fue 7 mEq/l.
PREGUNTA
Cul es el estado clnico acido base del paciente?
RESPUESTA
a.- Acidosis metabolica.
b.- Alcalosis metabolica.
c.- Acidosis respiratoria.
d.- Alcalosis respiratoria.
CASO CLINICO ALCALOSIS METABOLICA
Paciente de 61 aos de edad ex-fumador, con hipertensin arterial
(HTA), diabetes mellitus tipo II, dislipemia, portador de marcapasos por

CURSO ENARM CMN SIGLO XXI TEL: 36246001

CASO CLINICO
Mujer de 27 aos en seguimiento por Endocrinologa por trastorno de
la conducta alimentaria tipo bulimia nerviosa (10 aos de evolucin).
Presento un episodio de sncope que se atribuy a crisis comicial en el
contexto de desequilibrio hidroelectroltico. Actualmente segua
tratamiento con BOIK asprtico: 1-0-1, 1-0-0, Magnesio Boi: 2-02,
Multicentrum: 2-0-0. Se solicita valoracin por el servicio de Nefrologa
por trastornos hidroelectrolticos a pesar de tto. Los especialistas en
Endocrinologa y Psiquiatra estn convencidos que la paciente toma
la medicacin y no miente. La paciente niega consumo de laxantes,
diurticos y vmitos alimenticios. Se quiere descartar la posibilidad de
algn trastorno tubular que origine el cuadro que presenta la paciente.
A la Exploracin fsica, la paciente est eupneica, normocoloreada. TA:
110/70. FC: 72 lpm Afebril. AC: rtmica. AP: MVC. Abdomen: blando y
depresible. No dolor a la palpacin. No megalia, sin presencia de
edemas. Pruebas complementarias: A su llegada a Urgencias: BQ:
Glucosa 103 Urea 38 Cr 0.6 Na 138 K 2.9 Cl 99 Ca 9,4 Mg 0.7.
Gasometra venosa: pH: 7.44 PCO2: 49 HCO3: 33.3. Analtica de orina:
Na 89mmol/l K 75 mmol/l Cl: 135 mmol/l.
PREGUNTA
Cul es el diagnstico ms probable para esta paciente?
RESPUESTA
a.- Alcalosis respiratoria aguda
b.- Alcalosis metablica compensada
c.- Alcalosis metablica aguda
d.- Acidosis respiratoria compensada
PREGUNTA
Cul es la causa menos probable de la hipokalemia de la paciente?
RESPUESTA
a.- Acidosis tubular renal
b.- Hiperaldosteronismo
c.- Acidosis metablica
d.- Vmitos
PREGUNTA
Cul es el ANION gap que presenta esta paciente?
RESPUESTA
a.- AG= 10
b.- AG= 6
c.- AG= 12
d.- AG= 2

Pharmed Solutions Institute

PGINA 197

MANUAL DE TRABAJO DEL CURSO ENARM CMN SIGLO XXI


TRASTORNOS ELECTROLITICOS AGUDOS:
CIENCIAS BASICAS: El rin desempea un papel esencial en el equilibrio hidrosalino del organismo porque regula la excrecin de sodio y del agua. La
osmolalidad del sector extracelular es estrictamente una funcin de la natremia. La natremia depende de: el capital sdico, el capital potsico y el agua
TIPOS DE SOLUCIONES INTRAVENOSAS
total. En situaciones normales los riones regulan la osmolalidad srica entre 295 a 300
SOLUCION
CONCEPTO/ACCIONES
mosm/kg. de agua. La homeostasis del agua depende de la normalidad del mecanismo
ISOTNICOS
de la sed y de la secrecin de la hormona antidiurtica (ADH) por la hipfisis posterior.
Suero fisiolgico
Expansor de volumen extracelular para la
Sobre estos ltimos mecanismos interactan la presin arterial, la osmolalidad celular y
al 0.9%
hipovolemia. Un exceso puede provocar hipervolmia
srica, la angiotensina II y las prostaglandinas. La osmolalidad srica se puede estimar a
Na: 154mEq/l
y acidosis por exceso de cloro. Solo aporta iones cloro
partir de las siguientes formulas: Osmolalidad srica= 2Na (mEq/L)+ K(mEq/L)+Urea
Cl: 154mEq/l
y sodio. Se puede usar en hiponatremia. nico que se
308 mOm/l
puede usar conjuntamente con sangre y derivados
(mg/dl)/2.8 +Glucosa (mg/dl)/18 o si la urea y la glucosa son normales se puede utilizar
Suero glucosado
Igual que SF al 0.9%, pero la mezcla de dextrosa en
la siguiente formula simplificada: Osmolalidad= (Na+10) x 2. La osmolalidad srica se
al 5%
sangre provoca hipotonicidad
expresa en mosm/kg. de agua. Las principales manifestaciones de un trastorno en el
278 mOm/l
Aporta muchas caloras (170/l)
equilibrio del agua son la hiponatremia y la hipernatremia. HIPONATREMIA: Sodio (Na+)
Ringer-lactato
Contienen mltiples electrolitos y en concentraciones
(Hartmann)
similares a la plasmtica. Para tratar hipovolemia
<135 mEq/L o de 135 mmol/L y como consecuencia existir disminucin de la
El lactato se metaboliza rpido y se convierte en
osmolalidad plasmtica. Sus causas pueden ser: hipovolmicas con sodio corporal total
bicarbonato (riesgo de alcalosis)
disminuido y se produce prdida de lquidos y sodio extracelular como en el caso de la
Ringer-lactato
Aporta 170 caloras por litro y agua libre
fase polirica de la IRA, uso de diurticos, vmitos, diarreas, sudoracin profusa, fstulas
glucosado al 5%
No se debe utilizar para reponer volumen, pues
digestivas y quemaduras; hipervolmicas con sodio corporal total aumentado. Existe
diluye el plasma y altera concentraciones
HIPOTONICOS
exceso de lquidos y de sodio extracelular como se observa en la insuficiencia cardiaca
Suero fisiolgico al
Se emplea para potenciar la eliminacin de
congestiva, sndrome nefrtico, insuficiencia renal y cirrosis heptica; endocrinas como
0,45%
solutos por los riones debido a su cantidad
en el caso de la secrecin inapropiada de la hormona antidiurtica por tumores,
Na: 76.5mEq/l
de agua libre y para reponer el lquido
traumatismos, SIDA y en la enfermedad de Addison. Clnica: aparece cuando la natremia
Cl: 76.5mEq/l
extracelular
es inferior a 120 mEq/L de manera cronica y se caracterizan por anorexia, nuseas,
153 mOm/l
Tratamiento de la hipernatremia y de otros
estados hiperosmolares
vmitos, calambres musculares, hipotermia, letargo, confusin, convulsiones y coma.
Solo aporta cloro y sodio
Con natremias de 128-130mEq/l, sntomas pueden ser graves si se produce de manera
Si se emplea la formula con dextrosa 5%, se
aguda (<48hrs), tienen alto riesgo de edema cerebral. Laboratorio: electrolitos en sangre
puede provocar hipertonicidad en relacin con
y orina, osmolalidad plasmtica y urinaria, glicemia, protenas y lpidos plasmticos. Se
el plasma
debe recordar que 100 mg/dl de glicemia superior a lo normal disminuye el sodio srico
HIPERTONICOS
Suero fisiolgico al 3%
Para tratar hiponatremias en situaciones
en 1.6 mEq/L. Tratamiento, se debe tratar la causa desencadenante de manera tal que
Na: 513mEq/l
graves
en los estados hipovolmicos se utilizan soluciones fisiolgicas. Cuando existe retencin
Cl: 513mEq/l
Precaucin por su contenido alto en sodio y
hidrosalina y edema se restringen los lquidos y el sodio y se administran diurticos tipo
1,026 mOm/l
cloro, que pueden dar lugar a exceso de
furosemida y en caso de sobrehidratacin, restriccin de la ingesta de lquidos. En la
volumen intravascular y edema de pulmn
Suero fisiolgico al 5%
Igual que el anterior pero ms concentracin
prctica para la correccin de una hiponatremia aguda hipovolmica se debe elevar el
de sodio y cloro
sodio srico de 1 a 2 mEq/l x hora y se calcula el dficit de sodio segn la siguiente
formula: (Na+ plasmtico deseado-Na plasmtico real) x 0.6 x Kg peso=dficit de sodio corporal total (en mEq). Se utilizarn soluciones salinas isotnicas
al 0.9% que contienen 154 mEq/L. Tambin en la prctica para tratar una hiponatremia severa en un paciente con insuficiencia renal y realizarla a una
velocidad de 0.5 mEq/L/hora de elevacin de la natremia ha sido utilizada la hemodilisis continua venovenosa con un lquido de dilisis con un
contenido de sodio menor que lo normal. La hiponatremia crnica puede ocasionar mielinolisis pontina si se corrige rpidamente. En las formas agudas
puede elevarse el Na+ hasta 2mEq/l, mientras en las formas crnicas no debe superar los 1-1.5mEq/l. El uso de furosemida favorece la eliminacin de
agua libre, siendo especialmente til en estados edematosos o si la osmolalidad en orina es >400mOsm/kg. En el sndrome de secrecin inadecuada de
vasopresina (SIADH): el tratamiento crnico se basa en la restriccin de lquidos debe recordarse que: 1) la restriccin de lquidos afecta a todos los
lquidos no solo al agua. 2) el grado de restriccin depende de la diuresis ms las perdidas insensibles. 3) suelen ser varios das antes de que se modifique
la osm. 4) no debe restringirse el Na+, frmacos solo para casos refractarios, antagonista V2 (tolvaptan). HIPERNATREMIA: Sodio (Na+)>145mmol/l, y por
tanto existir un aumento de la osmolalidad plasmtica superior a 300 mosm/L. Menos frecuente que hiponatremia, ms comn en nios y adultos
mayores. Puede ser resultado de perdida de agua (frecuente) o de un aporte de sodio (raro). La prdida de agua desencadena dos mecanismos para
evitar la hipernatremia: la sed y la liberacin de vasopresina (ADH). Siempre hay que tener en mente la sed es primordial, ya que incluso la mxima
secrecin de ADH puede no lograra retener agua suficiente para compensar las prdidas si no se aumenta el aporte de agua. La hipernatremia con
hipervolemia es poco frecuente y se produce por una ganancia neta de sodio. Es mucho ms frecuente encontrar situaciones de normo o hipovolemia.
Sus causas pueden ser: euvolmicas con Na+ normal y se observa en los estados febriles severos, hiperventilacin pulmonar; hipovolmica con Na
disminuido se observa en la deshidratacin por sudoracin profusa, uso de manitol y soluciones glucosadas, diarreas, quemaduras, diabetes
inspida(central o nefrognica su diferencial mediante test de deprivacin de agua, midiendo la ADH plasmtica); hipervolmica con Na aumentado la
cual se observa cuando se utilizan en forma indiscriminada soluciones de bicarbonato de sodio, cuando se utilizan la carbenicilina y los esteroides y en el
coma hiperosmolar no cetognico. Clnica: sed, astenia, debilidad muscular, somnolencia, irritabilidad, letargia, confusin, temblor, convulsiones y coma.
Laboratorio: sodio srico y osmolalidad plasmtica. El tratamiento consiste en administrar soluciones glucosadas al 5% en los pacientes deshidratados
con sodio normal. En los pacientes con volumen intravascular aumentado se deben utilizar los diurticos ms soluciones glucosadas al 5%. En la diabetes
inspida central se utilizar la vasopresina exgena y en nefrognica tiazidas. En forma prctica, para calcular la cantidad de lquido a administrar se
utiliza la siguiente formula: Agua corporal total (ACT) real = peso corporal (Kg) x 0.6.
Na+ plasmtico real/Na+ plasmtico deseado X ACT real = ACT
deseada. ACT deseada- ACT real = dficit de lquido. Es prudente administrar el 30% de la solucin calculada en las primeras 24 horas. La correccin
rpida de hipernatremia puede producir edema cerebral, convulsiones, dao neurolgico permanente e incluso la muerte. Para disminuir riesgo se
aconseja corregir Na+ lentamente. En casos graves (>170mEq/l), no corregir a menos de 140mEq/l en primeras 48-72hrs y en formas crnicas no debe
+
+
corregirse mas de 8-12 mEq/l. HIPOPOTASEMIA: Potasio (K ) <3.5 mEq/L. La hipokalemia es leve cuando el K se encuentra entre 3.5 a 2.8 mEq/L,
moderada de 2.8 mEq/L y severa menor de 2.8 mEq/litro. Causa son: vmitos, diarreas, fstulas gastrointestinales, uso de diurticos, diuresis osmtica,
fase polirica de la IRA, uso de esteroides, hiperaldosteronismo primario o secundario, acidosis tubular renal, alcalosis. Clnica: decaimiento, letargia,
predisposicin a la intoxicacin digitlica, anorexia, nuseas, vmitos, distensin abdominal, leo paraltico, paresias, hiporeflexia osteotendinosa,
rabdiomilosis la cual puede originar IRA. Se deben solicitar el potasio srico y urinario as como los gases arteriales. Tratamiento correccin de la
hipokalemia, cuando el K+ srico es superior a 2.8 mEq/L se utilizar la va oral sobre la base de alimentos ricos en potasio sobre todo frutas ctricas, de
manera tal que la ingesta diaria sea de unos 40 a 120 mEq. S el K+ srico es inferior a 2.8 mEq/L se utilizar la va IV a razn de 40 mEq de KCL por litro en
solucin fisiolgica. Es prudente un tratamiento preventivo de la hipopotasemia en pacientes que reciban diurticos, esteroides, digitlicos, as como en

CURSO ENARM CMN SIGLO XXI TEL: 36246001

Pharmed Solutions Institute

PGINA 198

MANUAL DE TRABAJO DEL CURSO ENARM CMN SIGLO XXI


la fase polirica de la IRA y en pacientes que reciban tratamiento para una cetoacidosis diabtica. HIPERPOTASEMIA: Potasio (K+) >5.5 mEq/L y
constituye una verdadera emergencia mdica ya que el paciente corre el riesgo de morir por paro cardiaco. La hiperkalemia puede ser leve hasta 6.5
mEq/L, moderada hasta 7.5 mEq/L y severa superior a 7.5 mEq/L. Las causas ms frecuentes dehiperkalemia son: insuficiencia renal aguda y crnica, uso
de diurticos ahorradores de potasio, insuficiencia suprarrenal aguda o crnica (enfermedad de Addison), hipercatabolismo, acidosis metablica y la
pseudohiperkalemia por muestra de sangre hemolizada, trombocitosis y leucocitosis por encima de 100.000 mm3. Se deben solicitar potasio srico y
electrocardiograma, ya que ste es fundamental para evaluar la hiperkalemia. Cuando es leve al ECG se observan ondas T altas picudas y simtricas,
cuando es moderada se observa ensanchamiento del complejo QRS, disminucin de la amplitud de la onda P y prolongacin del intervalo PR y cuando es
grave desaparicin de la onda P, bradicardia, extrasistoles ventriculares, fibrilacin ventricular y asistolia. Tratamiento, adems del preventivo (evitar
frutas ctricas, evitar uso de diurticos ahorradores de potasio, no administrar sangre almacenada por largo tiempo). Moderada: se utilizan resinas de
intercambio catinico (poliestirensulfonato clcico) que eliminan el potasio del tubo digestivo. Diurticos. Grave: glucosa intravenosa junto con insulina
de accin rpida (10UI de insulina en 500ml de glucosa al 10%). La insulina favorece la entrada de potasio a la clula y la glucosa previene la aparicin de
hipoglucemia. Administracin de bicarbonato de sodio va intravenosa para corregir la acidosis. Furosemida: 40 mg IV a repetir cada 4 horas o
bumetanida 1 mg IV cada 4 hrs. Gluconato de calcio: 10 ml al 10% IV en 20 minutos. Bicarbonato de sodio: 60 ml IV rpidamente cada 8 horas. Solucin
glucosada al 10% 250 ml ms 10 unidades de insulina cristalina IV. Sulfonato de poliestireno sdico (Kayexalate) intercambia sodio por potasio a nivel
intestinal en enema a retener 50 a 100 gramos en 100 a 200 ml de agua o por va oral 20 a 40 gramos ms 20 ml de sorbitol al 50%. De ser necesario y en
medio especializado hemodilisis o dilisis peritoneal. HIPOCALCEMIA: Calcio (Ca 2+) <8,5mg/dl. Causas: Hipoparatiroidismo: una de las causas ms
frecuentes de hipocalcemia crnica, dficit de vitamina D, hipomagnesemia (en esta situacin se suprime la secrecin de PTH), insuficiencia renal,
hipoalbuminemia (calcio circulante se halla unido a la albumina), pancreatitis (aumenta la lipolisis, los acidos grasos se unen a calcio y disminuye),
politransfusiones sanguneas (elcitrato utilzado como anticoagulante se une al calcio), alcalosis, abuso de laxantes o sndromes de malaabsorcion. Clinica:
hiperreflexia, calambres musculares, la tetania por irritabilidad neuromuscular, es el signo clnico fundamental. La tetania se puede poner de manifiesto
mediante dos maniobras: Signos de Chevostec (estimular nervio facial; contraccin de musculos faciales), signo deTrousseau (se infla manguito de
presin por encima de presin arterial sistlica y se produce un espasmo carpal. Adormecimiento alrededor de la boca, hormigueo, espasmo farngeo,
convulsiones, psicosis, demencia. Tratamiento, tratar la causa, en situaciones agudas, se administra gluconato de calcio de forma intravenosa. Cuando
hipocalcemia es crnica aporte de vitamina D y suplementos de calcio oral. HIPOMAGNESEMIA: Es una situacin muy frecuente en los pacientes
gravemente enfermos, de sintomatologa vaga e inespecfica y frecuentemente se asocia a la hipocalcemia. Los niveles sricos de magnesio normales son
de 1.4 a 2 mEq/l. Puede observarse cuando se utilizan diurticos, en los alcohlicos crnicos, en la pancreatitis aguda, en la hipomagnesemia familiar con
hipercalciuria, intoxicacin digitlica, IAM, insuficiencia cardiaca. Clnica: disfagia, diplopa, debilidad, psicosis, depresin, nistagmo, signos de Chevostek
y Trousseau, hiperreflexia osteotendinosa y convulsiones. El tratamiento consiste en administrar magnesio a la dosis de 2 gramos de sulfato de magnesio
diluidos en 100 ml de solucin glucosada al 5% IV para pasar en 5 a 10 minutos y se puede repetir cada 8 horas por 5 das. HIPERMAGNESEMIA: Se
acompaa de arreflexia, parlisis respiratoria y paro cardiaco se trata con calcio IV 100 a 200 mg en 20 minutos.
CASO CLINICO
Femenino de 70 aos y 35 kg de peso. La paciente tena insuficiencia
cardiaca secundaria a miocardiopata multivalvular, siendo portadora
de una doble prtesis de Bjrk artica y mitral con anuloplastia
tricuspdea, con grado funcional III-IV de la NYHA. As mismo tena
implantado en zona epicrdica un marcapasos VVI Medtronic por
enfermedad del seno sintomtica. Como enfermedades no
cardiolgicas tena una leucotrombopenia secundaria a hepatopata
crnica por VHC. Se encontraba en tratamiento con digoxina,
acenocumarol y furosemida. Acude a urgencias por aumento
progresivo de su disnea habitual hasta hacerse de reposo, ortopnea de
2 almohadas y aumento progresivo de sus edemas en miembros
inferiores, es diagnosticada de insuficiencia cardiaca descompensada
por disfuncin del marcapasos, recambiando el generador y dndola
de alta con dieta sin sal, pobre en grasas, furosemida 40 mg/12 horas,
espironolactona 100 mg/24 h, enalapril 5 mg/24 h, digoxina y
anticoagulantes orales. Es valorada por nosotros en visitas a domicilio
por desorientacin con desconexin del medio que la paciente lo
defina como mente en blanco. Adems refera angustia y temor. No
presentaba cefalea, nuseas, vmitos, calambres musculares ni
disminucin de los reflejos. No tena aumento de su disnea habitual ni
edemas en MMII, ni dolor torcico. Le realizamos una analtica de
urgencias objetivando un K: 5,2 mmol/l y un Na: 122 mmol/l, glucosa
156mg/dl, con el resto de los parmetros en los lmites de la
normalidad.
PREGUNTA
Cul es la osmolaridad plasmtica que tienen este paciente?
RESPUESTA
a.- 262.6
b.- 280.3
c.- 320.4
d.- 290.0
PREGUNTA
Cul es el sitio donde ms probable se reabsorbe la mayor cantidad
de sodio?

CURSO ENARM CMN SIGLO XXI TEL: 36246001

RESPUESTA
a.- Tbulo contorneado proximal
b.- Tbulo contorneado distal
c.- Asa de Henle
d.- Tbulo colector
PREGUNTA
Qu porcentaje de sodio se excreta por orina normalmente?
RESPUESTA
a.- 1%
b.- 2%
3.- 3%
4.- 4%
CASO CLINICO
Mujer de 43 aos de edad presenta una costilla fracturada. Tiene dolor
oseo difuso y dificultad para peinarse, ponerse de pie y subir las
escaleras. Dice haber tenido heces voluminosas y ftidas y perder
alrededor de 4.5Kg, en el transcurso de los 6 meses pasados, aunque
su apetito ha sido normal. Muestra ingestin alta de calcio y
exposicin normal a la luz solar. Toma un polivitamnico a diario. El
examen fsico indica abdomen normal, debilidad de los musculos
proximales e hipersensibilidad sobre la columna vertebral y lasd
costillas. La marcha es de pato. El examen neorologico por lo dems
resulta normal. Los estudios de la sangre muestran electrolitos,
funciones renales y hepticas normales. Laboratorio: Ca 7.5mg/dl,
fosforo 1.6mg/dl, Mg 1.3mg/dl, albumina 3.2mg/dl, hormona
paratiroidea 96pg/ml, calcitriol 28pg/dl.
PREGUNTA
Cules son las causas posibles de hipocalcemia?
RESPUESTA
a.- Deficiencia de Magnesio
b.- Deficiencia de hormona paratiroidea
c.- Hepatopatia crnica
d.- Mala absorcin de vitamina D

Pharmed Solutions Institute

PGINA 199

MANUAL DE TRABAJO DEL CURSO ENARM CMN SIGLO XXI


HIPOGLUCEMIA, HIPERGLUCEMIA
HIPOGLUCEMIA: Niveles de glucemia < 50 mg/dL reversible con la administracin de glucosa. Se trata de una urgencia muy peligrosa ya que puede
ocasionar lesiones irreversibles e incluso la muerte por falta de aporte de glucosa a nivel de SNC. La hipoglucemia es la complicacin aguda ms
frecuente del tratamiento de la DM. Ms del 35% de pacientes con DM tipo 1 experimentan durante la evolucin de la enfermedad, al menos, un
episodio de hipoglucemia grave que requiere atencin por otra persona. La incidencia de hipoglucemia grave en los pacientes con DM tipo 2 es de 1 a 3
por 100 pacientes por ao. Esta frecuencia se triplica en pacientes mayores de 75 aos, con ingresos hospitalarios recientes y que reciben mltiples
frmacos. Si bien la incidencia de hipoglucemia es inferior en pacientes con DM tipo 2 que en los de tipo 1. Ante una situacin de disminucin de las
concentraciones de glucosa en sangre se produce una serie de acontecimientos tendentes a normalizar la glucemia. En sujetos diabticos que sufren
episodios repetidos de hipoglucemia se ha observado que el umbral de respuesta secretora de catecolaminas, as como de la aparicin de sntomas
autnomos, desciende de forma importante. Estos pacientes no presentan sntomas neuroglicopnicos hasta que sus valores de glucemia descienden a
valores en torno a 40 mg/dl. Es decir, estos pacientes soportan bien valores bastante bajos de glucemia (mecanismo de adaptacin cerebral), lo que
resulta perjudicial, ya que le introduce en un crculo vicioso de hipoglucemias no reconocidas. ETIOLOGA: La causa ms frecuente de hipoglucemia por
exceso de insulina es la producida por antidiabticos orales (ms frecuente las sulfonilureas) e insulina exgena, seguida de la discutible hipoglucemia
reactiva y los casos de hiperinsulinismo endgeno por tumores (insulinoma y tumores extrapancreticos productores de factores insulin-like). Pacientes
diabticos: Dosis excesivas de insulina o sulfonilureas. Dieta inadecuada (horarios alterados, baja ingesta,...). Exceso de ejercicio fsico. Enfermedades
que disminuyen las necesidades de insulina: Insuficiencia renal (aumenta la vida media de los hipoglucemiantes), heptica, dficits hormonales.
Interacciones medicamentosas (salicilatos, clofibratos, fenilbutazona, sulfinpirazona). Estrs. Pacientes no diabticos: Hiperinsulinismo endgeno
(insulinoma, frmacos). Comidas poco frecuentes. Hipoglucemia postprandial postciruga gstrica. Abuso de alcohol. Una causa inusual, pero con una
incidencia cada vez mayor, es la producida por fenmenos autoinmunes contra la insulina, su receptor o la propia clula beta, conocido como sndrome
autoinmune por insulina (tiroideas como tiroiditis de Hasimoto y enfermedad de Graves, etc). Otras causas: La sintomatologa propia de la hipoglucemia
reactiva se presenta en el perodo postprandial, especialmente si se ha efectuado una ingesta de hidratos de carbono de absorcin rpida. El insulinoma
es un raro tumor neuroendocrino con una incidencia aproximada de 1 a 2 por cada milln de habitantes/ao. Tpicamente se presenta como un tumor
solitario, pero puede aparecer de forma multifocal en el contexto de una neoplasia endocrina mltiple tipo 1 o como un tumor metastsico en el
insulinoma maligno. En el diagnstico de un insulinoma, la confirmacin bioqumica de hiperinsulinismo debe preceder a cualquier tcnica de
localizacin. La sospecha clnica es importante para su diagnstico, y as, es fundamental la existencia de la triada de Whipple: la clnica sugestiva de
hipoglucemia debe asociarse con unos niveles de glucosa bajos en sangre y mejorar con la administracin de glucosa. Con la sospecha clnica de
insulinoma la prueba diagnstica de eleccin es un ayuno de 72 horas. Esta prueba tiene que realizarse bajo supervisin mdica y por tanto ser
necesario hospitalizar al enfermo. Por ltimo, ante una hipoglucemia inexplicable, debemos pensar en la posibilidad del diagnstico de hipoglucemia
facticia para realizar las pruebas de confirmacin pertinentes y as evitar otras que pudieran resultar innecesarias. DIAGNOSTICO: Sntomas
adrenrgicos: Sudoracin, palpitaciones, ansiedad, temblor, hambre, predominan cuando la glucemia
desciende rpidamente pero pueden enmascararse con la toma de B-bloqueantes o si existen
neuropatas. Sntomas neuroglucopnicos: trastornos de conducta, agresividad, confusin focalidad
neurolgica, somnolencia, convulsiones, coma. Exploracin fsica: TA, T, FC, FR, SAT O2, glucemia capilar,
hidratacin o perfusin cutnea, fetor etlico, nivel de conciencia, focalidad neurolgica o crisis
convulsivas. Glucemia capilar , BH, QS, electrolitos, osmolaridad, coagulacin, EGO, ECG: podran
producirse por el efecto directo de la hipoglucemia, el aumento en la secrecin de adrenalina, la
hipopotasemia secundaria o la disfuncin autonmica, todo ello sobre un posible sustrato de cardiopata:
alteraciones de la repolarizacin, como depresin del segmento ST, aplanamiento e inversin de la onda T
y prolongacin del intervalo QT, trastornos del ritmo, como taquicardia y bradicardia sinusal, bloqueo
auriculoventricular. TRATAMIENTO: LEVE: nivel de conciencia conservado y tolera va oral se
administrarn lquidos azucarados con 20gr de glucosa. MODERADO: disminucin leve del nivel de
conciencia o no tolera la va oral: glucagn 1mg (1amp) IM o SC (pauta habitual en domicilio) o seguir
pauta siguiente. En un estudio se recomiendan 20 gramos de D-glucosa como lo ms eficaz, ya que
corrigen la hipoglucemia de grado moderado a grave en 20 minutos, sin causar hiperglucemia prolongada.
A los adultos diabticos en estado de coma u otros pacientes que tienen coma de causa imprecisa se les
debe administrar 50 ml de una solucin de glucosa al 50 % en bolo intravenoso, despus de obtener sangre para estudios apropiados. En caso de
confirmarse la hipoglucemia, se debe iniciar venoclisis con solucin de glucosa al 5, al 10 o al 20 %. La glucosa intravenosa continua por 4 a 6 horas es
necesaria para la mayor parte de las reacciones hipoglucmicas. GRAVE: disminucin del nivel de conciencia severo, coma o PCR: Va venosa perifrica y
administrar glucosa hipertnica (glucosmon*) 30ml al 33% (3 amp) y sueros glucosados (SG) al 10%, si no posible canalizar va: glucagn 1mg (1amp) IM
o SC, si tras 1000ml de SG continua con cifras bajas de glucemia administraremos hidrocortisona 100mg y 1mg de glucagn a cada litro de solucin,
adrenalina 1mg diluido 1/1000 SC, protocolo de RCP si precisa. En pacientes alcohlicos administrar previamente tiamina 100mg, IM. Mantenimiento:
Control horario de glucemia capilar hasta cifras 120mg/dl y a partir de entonces cada 4-6 horas durante 24 horas. Continuar con SG 5-10% hasta
tolerancia oral. HIPERGLUCEMIA: Se define como el nivel srico superior a 140mg/dl. La liberacin de glucgeno por parte del hgado y los msculos,
combinado con la glucognesis y el metabolismo de los cidos grasos libres, aumentara el nivel de glucosa en sangre. El que este aumento alcance el
nivel de hiperglucemia depender de la capacidad del organismo para utilizar la glucosa a nivel celular. Hiperglucemia grave, es la elevacin de la
glucemia plasmtica casi siempre superior a 250mg/dl, que se asocia con alteracin de otros rganos en el organismo. Las dos principales causas de
deterioro en la utilizacin de glucosa son los medicamentos y la diabetes. Tambin puede producir hiperglucemia cualquier enfermedad que aumente el
estrs corporal. El estrs aumenta la produccin de citosinas y hormonas contrarreguladoras de insulina (catecolaminas, cortisol, glucagn y hormonas
del crecimiento) que alteran el metabolismo de los carbohidratos, incluidas la resistencia a la insulina, la gluconeognesis, glucogenolisis y la disminucin
de la secrecin de la misma debida al mal funcionamiento de la clulas beta del pancretica. Otros factores que participan en la elevacin de glucosa son:
la administracin exgena de glucocorticoides a dosis elevada, el tratamiento subptimo del control glucmico por falta de estrategias o algoritmos de
manejo (pobre adherencia 20-40%) y la nutricin enteral o parenteral, infecciones (20-50%), embarazo, entre otros. La hiperglucemia se ha asociado con
aumento de la respuesta proinflamatoria, una funcin alterada del sistema inmunitario, disfuncin en la quimiotaxis de neutrfilos con la consecuente
alteracin en la fagocitosis, disfuncin endotelial, estado protrombtico, dao neuronal asociado con la isquemia cerebral y con aumento del estrs
oxidativo. Los pacientes diabticos pueden presentar hiperglucemia si no se aplican su insulina o no toman hipoglucemiantes orales, se alteran las
restricciones dietticas o estn sometidos algn tipo de estrs ya sea fisco o emocional. Clnica: polidipsia, polifagia, poliuria, frecuentemente
acompaadas de prdida de peso y fatiga. La hiperglucemia es un problema que se manifiesta en un porcentaje importante de pacientes hospitalizados y
constituye un factor de riesgo para IAM, infarto cerebral, sepsis, infecciones nosocomiales, insuficiencia cardiaca y renal. Adems, complica una gran

CURSO ENARM CMN SIGLO XXI TEL: 36246001

Pharmed Solutions Institute

PGINA 200

MANUAL DE TRABAJO DEL CURSO ENARM CMN SIGLO XXI


variedad de enfermedades, aumenta el tiempo de estancia hospitalaria y es un factor de riesgo independiente de complicaciones que producen una
importante morbilidad y mortalidad en los servicios hospitalarios, sin mencionar su gran impacto econmico. La administracin de insulina puede
prevenir varios de los efectos adversos asociados a hiperglucemia. La insulina puede suprimir la generacin de especies reactivas de oxgeno, inducir
vasodilatacin, inhibir la lipolisis, reducir los cidos grasos libres, inhibir la agregacin plaquetaria y disminuir la respuesta inflamatoria. Por lo tanto la
insulina juega un papel primordial en la prevencin de los riesgos asociados con la hiperglucemia. Si la hiperglucemia no se trata aparecer: cetoacidosis.
En la valoracin posterior hay que controlar los medicamentos que puedan causar hiperglicemia: corticoides, anticonceptivos, antihipertensivos
(furosemida, tiacidas, cido etacrinico).
CASO CLINICO HIPOGLUCEMIA
Mujer de 36 aos con diabetes de larga evolucin con polineuropata,
retinopata y nefropata. Padeci adems un sndrome de Cushing por
microadenoma hipofisario que requiri ciruga, una infeccin por VHC
crnica e HTA, para la que haba recibido tratamiento con amlodipino
10mg/24h. Intolerancia al ortostatismo y sncopes, que se agrav en
las dos ltimas semanas, por lo que la paciente haba estado confinada
en la cama. A la exploracin se observo desorientacin, agitacin,
desviacin de la comisura labial acompaada de ptosis parpebral
unilateral, ausencia de reflejos osteotendinosos en las extremidades
inferiores y perdidas de control de impulsos.

PREGUNTA
Se intenta pero no se puede canalizar a la paciente Cul es la
conducnta mas adecuada a seguir en este caso?
RESPUESTA
a.- 20 gr de D-glucosa
b.- Glucagon 1mg IM
c.- Glucosa al 33% IM
d.- Hidrocortisona 100mgs
PREGUNTA
Cul es la meta de glucosa en este paciente?
RESPUESTA
a.- 80mg/dl
b.- 100mg/dl
c.- 120mg/dl
d.- 140mg/dl
CASO CLINICO
Se trata de un paciente varn de 67 aos que hace 13 aos fue
diagnosticado de diabetes mellitus tipo 2 e inici tratamiento con
metformina 850 mg en el desayuno y la cena (1-0-1), hasta que hace
cuatro aos present deterioro de su control metablico, con una
hemoglobina glucosilada (HbA1C) de 8,4%, por lo que se decidi aadir
glimepirida hasta los 6 mg/da como dosis final. En la analtica de hace
cuatro meses se le detect un cociente albmina/creatinina (CAC) de
310 mg/g, con filtrado glomerular (FG) de 52 ml/m/1,73 m2. Acude a
consulta para recoger una nueva analtica y revaluacin. Adems, en
los ltimos dos meses ha presentado un par de episodios de
sudoracin y se queja de mareos ocasionales.

CURSO ENARM CMN SIGLO XXI TEL: 36246001

PREGUNTA
Qu es lo mas probable que le este sucediendo al paciente?
RESPUESTA
a.- Efecto secundario de glimepirida
b.- Episodios de hipoglucemia
c.- Manifestaciones de insuficiencia renal
d.- Efecto secundario de metformina
PREGUNTA
Si la insuficiencia renal del paciente sigue avanzando a Estadio 4. Cul
es la conducnta teraputica mas adecuada a seguir?
RESPUESTA
a.- Quitar metformina y glimepirida y usar acarbosa
b.- Quitar solo la glimepirida y anexar insulina
c.- Quitar metformina y glibenclamida y usar insulina
d.-Quitar metformina y usar glibenclamida

PREGUNTA
Cual es la conducta a seguir mas apropiada?
RESPUESTA
a.- Glucosa al 50 %.
b.- Ringer latato.
c.- Solucion harman.
d.- Solucion mixta.

PREGUNTA
Qu estadio de insuficiencia renal presenta el paciente?
RESPUESTA
a.- Estadio 1
b.- Estadio 2

c.- Estadio 3
d.- Estadio 4

CASO CLINICO
Mujer de 46 aos con hipertensin arterial, diabetes mellitus
insulinodependiente con insuficiencia renal crnica en dilisis
peritoneal que ingresa en planta de medicina interna con el
diagnstico de cetoacidosis diabtica. Se inicia tratamiento y el
segundo da se reinicia la dilisis peritoneal con su pauta habitual. El
cuarto da presenta disminucin del nivel de conciencia, aumento de
presin arterial, sudoracin profusa y al final una parada respiratoria
por lo que es intubada. La glucemia capilar es de 122mg/dl y la
gasometra arterial muestra una glucemia de 20mg/dl. Se administra
glucosa intravenosa. Al da siguiente se decide reducir la sedacin,
presentando una hora ms tarde convulsiones clnicas, desviacin de
la mirada y taquipnea. La glucemia capilar en ese momento es de
59mg/dl.
PREGUNTA
Cual es la conducta mas adecuada a seguir para establecer el
diagnostico de esta complicacin?
RESPUESTA
a.- Realizar EEG.
b.- Realizar PEV.
c.- Realizar TAC.
d.- Realizar IRM
PREGUNTA
Entre otros estudios se realiz un EEG en donde se observa signos de
sufrimiento cerebral difuso de grado muy severo y una TC craneal que
descarta afeccin intracraneal aguda. Se decide no retirar el lquido de
dilisis peritoneal e iniciar dilisis convencional. La escala de Glasgow
fue de 4, Cual es el diagnostico mas probable en este momento?
RESPUESTA
a.- Muerte cerebral.
b.- Encefalopatia hipoglucemica.
c.- Acidente vascular cerebral.
d.- Encefalopatia hipoxico isqumica.

Pharmed Solutions Institute

PGINA 201

MANUAL DE TRABAJO DEL CURSO ENARM CMN SIGLO XXI


ESTADO HIPEROSMOLAR HIPERGLUICEMICO (EHH), CETOACIDOSIS DIABETICA (CAD):
CIENCIAS BASICAS: Las complicaciones hiperglucemicas son causantes de un gran nmero de hospitalizaciones en diabticos, presentndose entre 4 y 8
por 1000 pacientes con diabetes, con una mortalidad que vara entre 4 y 20%. De su adecuado
DIAGNOSTICO DIFERENCIAL ENTRE CETOACIDOSIS DIABETICA
diagnstico y tratamiento depender una considerable disminucin de las hospitalizaciones y
(CAD) Y ESTADO HIPEROSMOLAR HIPERGLUCEMICO (EHH)
PRESENTACION CLINICA
CAD
EHH
das de estancia en esta poblacin. PATOGENIA: Tanto la CAD como EHH, son el resultado de la
Dolor abdominal
+++
combinacin del dficit absoluto o relativo de insulina y el aumento de las hormonas
Hiperventilacion
+
contrareguladoras. Este estado lleva a un aumento de la produccin heptica de glucosa y
Vomito
+++
disminucin en el consumo perifrico, produciendo un aumento exagerado de la glucosa
Deshidratacin
++
+++
sangunea; esta a su vez induce glucosuria, diuresis osmtica y deshidratacin. En los pacientes
Signo de Kussmaul
+++
Alteracin del edo. de alerta
+
+++
con DM tipo I el dficit severo de insulina y el aumento en las catecolaminas, cortisol y hormona
Aliento cetonico
++
del crecimiento estimulan la lipasa sensible a hormonas, aumentando la produccin de cidos
Poliuria
+
+++
libres derivados de triglicridos, los cuales son metabolizados en el hgado y convertidos en
Polidipsia
+
+
cuerpos cetnicos, proceso estimulado por el aumento de glucagn y el dficit de insulina que
Fiebre
+
+
Prdida de peso
+
+
activa la enzima carnitina palmitoil transferasa I que permite la entrada de los cidos grasos
Taquicardia
+
libres a la mitocondria donde se convierten en cido -hidroxibutirico, acetona y cido actico.
DIAGNOSTICO
Estos cuerpos cetnicos son los responsables de la acidosis en los pacientes con DM1. El Glucemia
>250mg/dl
>600mg/dl
hidroxibutirico y el acetoactico se disocian, produciendo exceso de iones de hidrogeno que
Cetonuria
+++
+
consumen el bicarbonato, disminuyendo sus niveles sricos. A medida que se van acumulando
Bicarbonato srico
<15
>15
pH
<7.30
>7.30
los cuerpos cetnicos, el pH sanguneo desciende. Cuando el pH desciende hasta cerca de 7.2, el
Brecha aninica Na- (Cl+HCO3)
>12
<12
centro respiratorio eleva frecuencia e intensidad de las respiraciones (respiracin de Kussmaul)
Osmolaridad
>320
en el intento de eliminar con las reparaciones el exceso de cido carbnico. La acetona un
2Na+(glu/18)+(BUN/2.8)
componente de los cuerpos cetnicos, tambin se elimina a travs de la respiracin, lo que
origina un olor caracterstico afrutado en el aliento del paciente. Los riones a su vez tambin eliminan el exceso de cuerpos cetnicos, dando lugar a la
cetonuria. Como la cetoacidosis diabtica altera la homeostasia, el organismo intenta restablecer el orden mediante una serie de mecanismos
compensatorios. Para compensar la diuresis osmtica, el centro de la sed del cerebro estimula al paciente para que beba ms. Los riones comienzan a
segregar renina, ponindose en marcha el sistema renina angiotensina, aldosterona, este complejo proceso hace que los riones reabsorban sodio y
agua. Tambin la hipfisis posterior segrega hormona antidiurtica que ayuda tambin a conservar el agua y el sodio. La diuresis osmtica deriva de la
hiperglucemia lleva a un dficit severo de lquidos que puede estar entre 5 y 7 litros. Adems se produce tambin dficit de cloruro de sodio entre 310mmol/kg, los niveles de sodio pueden estar falsamente alterados por la hiperglucemia presente. El potasio se encuentra tambin severamente
disminuidos, sin embargo los niveles de potasio en suero pueden estar normales o incluso elevados durante el episodio, debido a la acidosis y a la
hiperglucemia presente, disminuyendo severamente el potasio intracelular. Otros elementos que pueden estar alterados son el fosfato, el magnesio y el
calcio. CAUSAS: Diagnostico de novo, infecciones, enfermedades intercurrentes, falla en la aplicacin de insulina o en la toma de medicamentos orales
(corticoides), excesos alimentarios, ciruga,
traumatismo, desconocida. En los diabticos
diagnosticados, su causa desencadenante suele ser
una situacin estresante que incrementa las
necesidades de insulina, aunque tambin puede
obedecer a una descompensacin de la
enfermedad por no haber seguido correctamente
el tratamiento prescrito CLINICA Y DIAGNOSTICO:
En cuadro. Iniciar con Bh completa y gases
arteriales, EGO, glucosa plasmtica, BUN,
electrolitos,
cretinina,
electrocardiograma.
TRATAMIENTO: El objetivo principal e inicial es la
correccin del dficit hdrico para expandir el
volumen intra y extracelular y asegura una
adecuada perfusin renal. Si no existen trastornos
cardiacos se inicia con solucin salina al 0.9%
normal; si el sodio es mayor de 155meq/l se
recomienda utilizar solucin salina al 0,45 normal.
COMPLICACIONES: La aparicin de edema cerebral
es raro afortunadamente, mltiples factores
influyen en su origen, incluyendo la aparicin de
idiosmoles que causan un gradiente y una
desviacin del agua hacia las clulas, la terapia con
insulina por si sola aumenta la entrada de
sustancias osmticamente activas en el espacio
intracelular y un aumento rpido del dficit de sodio. Sndrome de dificultad respiratoria del adulto; esta complicacin puede producirse por el aumento
en el volumen de agua en los pulmones y una disminucin en la adaptabilidad pulmonar. Acidosis metablica hipercloremica; el mecanismo principal es
la perdida de cetocidos por la orina, los cuales son requeridos para la generacin de bicarbonato.

CASO CLINICO
Una mujer de 76 aos que no se conoca diabtica ingres por una
descompensacin hiperglucemica cetosica. Durante el ingreso sufri 4
crisis parciales motoras con generalizacin secundaria, de inicio motor
hemicorporal izquierdo y generalizacin tnico-clnica, con una
duracin de entre 1 y 3 minutos, con recuperacin del nivel de

CURSO ENARM CMN SIGLO XXI TEL: 36246001

conciencia entre las crisis y en un periodo de 5h, seguidas de un


intenso dficit poscrtico hemisfrico derecho, con hemiparesia,
hemihipoestesia, hemianopsia y heminegligencia izquierdas,
recuperndose en las siguientes 24h. La analtica mostr una glucemia
de 596mg/dl pH 7.30, glucosuria y cetonuria, con osmolalidad en suero
calculada de 318 mOsm/l. Se inici tratamiento con anticonvulsivante),
sin recurrencia de las crisis. Se le realiz una RM craneal 18 horas

Pharmed Solutions Institute

PGINA 202

MANUAL DE TRABAJO DEL CURSO ENARM CMN SIGLO XXI


despus de la primera crisis, donde se apreci una hipointensidad en
sustancia blanca subcortical parietal derecha en T2 con ligera
hiperintensidad cortical en FLAIR, tenue captacin giriforme de
contraste y leve restriccin de la difusin en dicha localizacin. El
electroencefalograma (EEG) evidenci un foco de ondas delta
frontotemporal derecho.

conciencia, por lo que su mujer le ha administrado agua con azcar


repetidamente. No disnea ni dolor torcico. No disuria. Exploracin
general: TA 111/52, FC 50, T 36, SatO2 98% DTX: HI. Consciente y
orientado, colaborador, taquipnea sin tiraje, mal hidratado, mala
coloracin cutneo-mucosa, cardiopulmonar normal, abdomen:

PREGUNTA
Considerando las complicaciones que se presentan en esta
patologa cual es su pronostico mas probable?
RESPUESTA
a.- Bueno ya que se resolvi adecuadamente.
b.- Moderado posibles secuelas.
c.- Puede continuar con crisis convulsivas.
d.- Para evitar recurrencias es conveniente dejar
anticonvulsivo.
CASO CLINICO
Mujer de 42 aos con antecedentes de tiroiditis crnica de
Hashimoto, 3 partos normales, sin macrosoma y una
enfermedad mixta del tejido conjuntivo (EMTC), ante la
aparicin de una poliartritis de pequeas articulaciones,
esclerodactilia, fenmeno de Raynaud, miopata, anticuerpos
antinucleares (+) 1/5120 y anticuerpos antiENA (++++). Una ta
era diabtica tipo 2. Usaba levotiroxina 50 g/da, prednisona
7,5 mg diarios y MTX 25 mg intramusculares semanales. Hace
tres meses, se espaci la dosis de MTX a 25 mg cada dos
semanas, apareciendo debilidad muscular progresiva, mialgias y
posteriormente sed y poliuria. La glicemia fue 286 mg/dl (glicemias
previas siempre normales), prescribindose dieta y glibenclamida 5
mg/da. Tres semanas despus consult en un Servicio de Urgencia por
sed intensa. La glicemia fue 550 mg/dl con cetonemia (-). Se aument
la glibenclamida a 10 mg/da e inici metformina 850 mg y
antiinflamatorios. Una semana despus, consult nuevamente por
vmitos, sed, poliuria, mialgias, artralgias y compromiso importante
del estado general. Fue hospitalizada, destacando deshidratacin
marcada, normotensin y polipnea. IMC: 27,3 kg/mt2. La piel de la
cara estaba acartonada, enrojecida, violcea y tena aspecto
cushingoide. No exista acantosis nigricans. El tiroides se palpaba
normal y el examen cardiopulmonar y abdominal era normal. Exista
falta de fuerzas en las extremidades. La glicemia era 414 mg/dl,
cetonemia (++), hemoglobina glicosilada A1C 12%, exista acidosis
metablica (pH 7,0, bicarbonato 2,9 mEq/L) con lactacidemia normal,
creatinina 1,3 mg/dl y potasio 2,6 mEq/L. Se administr insulina
cristalina subcutnea y luego en bomba de infusin en dosis
crecientes, hasta 520 unidades en 24 h, sin lograr controlar la
hiperglicemia ni la cetoacidosis. Recibi bicarbonato de sodio y
potasio. Se descart la existencia de una infeccin. A pesar de la
administracin de grandes dosis de insulina cristalina, persistieron la
hiperglicemia y la cetosis.
PREGUNTA
Considerando las condiciones del caso cual de las siguientes medidas
serian mas tiles para identificar la resistencia al tratamiento?
RESPUESTA
a.- Buscar anticuerpos antiinsulina.
b.- Buscar anticuerpor antireceptores.
c.- Buscar niveles de CD8.
d.- Buscar anticuerpos antimicrosomales tiroideos
CASO CLINICO
Masculino de 81 aos. Antecedentes personales: DM en tratamiento
con insulina (NPH), HTA en tto con Valsartn, cada de su propia altura
hace 4 das, sin requerir hospitalizacin. Ingreso en Mayo por
Hipoglucemia y se suspendi la insulina inicindose tx con Metformina
+ Sitagliptina. Comienza hace 48h. con dolor abdominal difuso,
nuseas y vmitos de repeticin; con escasa ingesta. Deposiciones
diarias de consistencia dura. No fiebre. Disminucin del nivel de

CURSO ENARM CMN SIGLO XXI TEL: 36246001

blando y depresible, doloroso a la palpacin de forma generalizada, sin


signos de irritacin peritoneal, peristaltismo de progresin. EEII: no
edemas ni signos de TVP. SNC sin focalidad, funciones superiores
conservadas (un poco lento/obnubilado). Analtica: Creatinina 2,52,
Glucosa 1.045, Urea 158, Na 131, K 6,85, Cl 104 Amilasa 59 Lactato 54
Hemates 4,10 Hb 13,1 VCM 99,6 Plaquetas 352 INR 0,94,
Leucocitos 11.800 (N 80%). Gasometra venosa: pH 6,96 pCO2 19
Bicarbonato 4,3 Exceso de Bases -26,3, Orina: Leucos y Nitritos (-),
Glucosa 1000, C .Cetnicos 50. ECG: arritmia sinusal, complejos
supraventriculares. BRD, T picudas. RX trax: sin hallazgos patolgicos.
PREGUNTA
Cul es el diagnstico ms probable para este paciente?
RESPUESTA
a.- Estado hiperosmolar hiperglucemico
b.- Cetoacidosis diabtica
c.- Cetosis por inanicin
d.- Acidosis hipercloremica
PREGUNTA
Cul es la causa ms probable del diagnstico de este paciente?
RESPUESTA
a.- Trauma
b.- Frmacos
c.- Proceso infeccioso
d.- IAM
PREGUNTA
Cul es el anin GAP para este caso?
RESPUESTA
a.- 27.7
b.- 17.2
c.- 12.7
d.- 22.7
PREGUNTA
Cul es la complicacin menos probable de la acidosis metablica
grave?
RESPUESTA
a.- Alteracin de contractilidad miocrdica y gasto de VI disminuido
b.- Acidosis paradjica del SNC

Pharmed Solutions Institute

PGINA 203

MANUAL DE TRABAJO DEL CURSO ENARM CMN SIGLO XXI


c.- Vasoconstriccin de arteria pulmonar
d.- Coma y vasodilatacin cerebral
PREGUNTA
Cul es la conducta teraputica inmediata ms adecuada a seguir?
RESPUESTA
a.- Administrar insulina
b.- Corregir electrolitos
c.- Corregir glucosa
d.- Hidratacin

CASO CLINICO
Femenino de 65 aos que ingresa al servicio de urgencia por deterioro
del nivel de conciencia. Hija refiere que inicia hace 48 horas,
caracterizado por nuseas ms vmitos acuosos y abundantes en tres
ocasiones, lo que se acompaa posteriormente de prdida del
conocimiento. APP: refiere HTA de 13 aos controlada con enalapril;
DM II de 13 aos de evolucin controlada inicialmente con
glibenclamida y en los ltimos 4 meses controlada con insulina
irregularmente; tumor cerebral benigno; sndrome ansioso controlado
con clonazepam; IVU a repeticin. EF: afebril estuporosa, plida,
pupilas isocricas reactivas, prpados cerrados con reflejo corneal
conservado; mucosas secas, con evidentes signos de deshidratacin
severa; ruidos cardiacos taquicrdicos, choque de punta en sexto
espacio intercostal fuera de la lnea medio clavicular, pulso rpido y
dbil; taquipnea campos pulmonares hipoventilados; abdomen
blando, depresible, discreto dolor; con reflejos osteotendinosos
normales, fuerza muscular disminuda, respuesta al dolor, abre los ojos
al dolor, Glasgow de 10, signo del pliegue positivo. PA: 100/54mmHg.
Analitica: Glu 886mg/dl, urea 238mg/dl, Hb 15.10mg/dl, Hto 40.50%,
Na 120mEq/l, K 5,9mEq/l, Ca 8.4 mE/l, amilasa 167, lipasa 238, pH7.4,
pCO2 32mmHg, pO2 109mmHg, HCO3 18, EB -5.
PREGUNTA
Cul es el diagnstico ms probable para este paciente?
RESPUESTA
a.- Cetoacidosis diabtica
b.- Acidosis lctica
c.- Estado hiperosmolar hiperglucemico
d.- Hiperglucemia por estrs

CASO CLINICO
Femenino de 65 aos que ingresa al servicio de urgencia por deterioro
del nivel de conciencia. Hija refiere que inicia hace 48 horas,
caracterizado por nuseas ms vmitos acuosos y abundantes en tres
ocasiones, lo que se acompaa posteriormente de prdida del
conocimiento. APP: refiere HTA de 13 aos controlada con enalapril;
DM II de 13 aos de evolucin controlada inicialmente con
glibenclamida y en los ltimos 4 meses controlada con insulina
irregularmente; tumor cerebral benigno; sndrome ansioso controlado
con clonazepam; IVU a repeticin. EF: afebril estuporosa, plida,
pupilas isocricas reactivas, prpados cerrados con reflejo corneal
conservado; mucosas secas, con evidentes signos de deshidratacin
severa; ruidos cardiacos taquicrdicos, choque de punta en sexto
espacio intercostal fuera de la lnea medio clavicular, pulso rpido y
dbil; taquipnea campos pulmonares hipoventilados; abdomen
blando, depresible, discreto dolor; con reflejos osteotendinosos
normales, fuerza muscular disminuda, respuesta al dolor, abre los ojos
al dolor, Glasgow de 10, signo del pliegue positivo. PA: 100/54mmHg.
Analitica: Glu 886mg/dl, urea 238mg/dl, Hb 15.10mg/dl, Hto 40.50%,
Na 120mEq/l, K 5,9mEq/l, Ca 8.4 mE/l, amilasa 167, lipasa 238, pH7.4,
pCO2 32mmHg, pO2 109mmHg, HCO3 18, EB -5.
PREGUNTA
Cul es el diagnstico ms probable para este paciente?
RESPUESTA
a.- Cetoacidosis diabtica
b.- Acidosis lctica
c.- Estado hiperosmolar hiperglucemico
d.- Hiperglucemia por estrs
PREGUNTA
Cul es la mortalidad para este paciente, de acuerdo al cuadro?
RESPUESTA
a.- 15%
b.- 30%
c.- 45%
d.- 60%

PREGUNTA
Cul es la mortalidad para este paciente, de acuerdo al cuadro?
RESPUESTA
a.- 15%
b.- 30%
c.- 45%
d.- 60%

PREGUNTA
Cul es el factor precipitante ms probable para este caso?
RESPUESTA
a.- Tumor cerebral benigno
b.- Gastroenteritis
c.- Iniciacin de la insulina
d.- IVU de repeticin

PREGUNTA
Cul es el factor precipitante ms probable para este caso?
RESPUESTA
a.- Tumor cerebral benigno
b.- Gastroenteritis
c.- Iniciacin de la insulina
d.- IVU de repeticin
PREGUNTA
Con qu solucin es ms adecuado corregir la hidratacin de este
paciente?

CURSO ENARM CMN SIGLO XXI TEL: 36246001

RESPUESTA
a.- Solucin de NaCl 0.45%
b.- Solucin de NaCL 0.9%
c.- Solucin dextrosa 5%
d.- Solucin de Ringer-lactato

PREGUNTA
Con qu solucin es ms adecuado corregir la hidratacin de este
paciente?
RESPUESTA
a.- Solucin de NaCl 0.45%
b.- Solucin de NaCL 0.9%
c.- Solucin dextrosa 5%
d.- Solucin de Ringer-lactato

Pharmed Solutions Institute

PGINA 204

MANUAL DE TRABAJO DEL CURSO ENARM CMN SIGLO XXI


ISQUEMIA CEREBRAL TRANSITORIA (ICT):
CIENCIAS BASICAS: Se define como un episodio breve de disfuncin neurolgica causado por dao cerebral focal o isquemia retiniana, con signos que
tpicamente duran hasta una hora y sin evidencia de infarto cerebral agudo. SALUD PUBLICA: Alrededor de 15-20% de los pacientes con un infarto
cerebral tienen historia de ICT. Recientemente se ha demostrado que la ICT es un fuerte predictor a corto plazo de infarto cerebral, enfermedad
cardiovascular y muerte. Presentar un ICT conlleva un riesgo de ACV en el primer mes de 8% y al ao de 5%, junto con 5% de riesgo de infarto de
miocardio al ao. PATOGENIA: Se basa especialmente en un mecanismo de produccin aterotrombotico, que sin duda es el principal responsable de la
mayora de los casos de ICT. Otros mecanismos como el cardioembolismo, la arteriopata no ateroesclerosa y las vasculopatas han sido descritas
tambin como ICT. Las observaciones iniciales mediante examen oftalmoscpico de la amaurosis monocular transitoria, sirvi para ilustrar el
compromiso del flujo sanguneo en las arterias retinianas y el rompimiento de las columnas venosas en patrn cuadrado, con material "blanco" que
obstrua la arteria retiniana. La razn ntima relacionada con las lesiones en ICT tiene que ver con la estenosis vascular y ulceracin de placas
aterosclerticas con formacin de trombos, con subsecuente embolizacin de material de fibrina y plaquetas desde estos sitios aterosclerticos, como el
arco artico, cartida interna extracraneal e intracraneal y la arteria cerebral media y la vertebral. Igualmente, la formacin de cogulos de fibrina y
glbulos rojos formados en la circulacin rpida o las cavidades cardiacas tambin cuenta como causal de las lesiones en ICT. Recientemente, el estudio
para el tratamiento de ACV Agudo defini las anormalidades cardiacas de alto y mediano riesgo de embolizacin. Las anormalidades de alto riesgo
(mayor de 5% anual) son: la fibrilacin auricular (FA), vlvulas protsicas, enfermedad reumtica cardiaca, endocarditis bacteriana, mixoma auricular y
cardiomiopata dilatada. Los de riesgo moderado (2% anual) son: personas mayores de 65 aos sin factores de alto riesgo; y pacientes con riesgo bajo
(1% anual) los menores de 65 aos y sin factores de riesgo concomitantes. CARACTERISTICAS CLINICAS: Debe referirse exclusivamente al principal
territorio arterial afectado, es decir carotideo o vertebrobasilar (tallo cerebral). La caracterstica principal es la constelacin de signos y sntomas de
dficit neurolgico focal que son: alteraciones de la conciencia o sincope, mareo monosintomtico, amnesia o confusin aislada, crisis convulsivas,
vrtigo aislado, diplopa aislada, escotomas cintillantes, disfagia aislada, disartria aislada, incontinencia. Los sntomas aislados no deben ser considerados
como eventos de ICT. En el territorio carotideo las ms frecuentes son: dficit motor, sensitivo o trastornos del lenguaje e incluyen; disfuncin sensitiva y
motora de extremidades contralaterales, seguidas de afeccin motora o sensitiva pura o bien con menos frecuencia disfasia o afasia aislada. Duracin de
sntomas menos de 15 min. La amaurosis fugax o ceguera monocular transitoria (visin borrosa, nebulosa o vidrio empaado) es una de las
manifestaciones clnicas tradicionales de enfermedad carotidea ateroesclerosa, se atribuye a embolismo arteria-arteria, duran de 1-5 min., y rara vez
exceden los 15 min. La visin se restablece por completo al terminar el evento. En el territorio vertebrobasilar las manifestaciones pueden ser ms
variadas lo que provoca con mucha frecuencia que sntomas aislados se confundan con ICT. Lo ms recuente es debilidad o torpeza que pueden cambiar
de un lado a otro, alteraciones sensitivas que pueden ser bilaterales, hemianopsia homnima o ceguera total transitoria, ataxia, diplopa; o bien por lo
menos dos de los siguientes; disartria, diplopa, vrtigo o disfagia. DIAGNOSTICO: Se obtiene por historia clnica, ya que al momento de la exploracin es
frecuente que el paciente se encuentre recuperado completamente. Es importante realizar un adecuado examen fsico general buscando cardiopatas o
enfermedades de origen vascular y un examen neurolgico exhaustivo ya que permiten identificar signos persistentes. Laboratorio: Nos ayuda a
identificar causas metablicas como hipoglicemia, hiponatremia y trombositosis, una VSG elevada puede sugerir endocarditis bacteriana o arteritis
temporal. Dado que una de las causas de ICT es la embolia de origen cardiaco, uno de los estudios iniciales debe ser un ECG, que puede revelar fibrilacin
auricular o IAM silente. La TAC y RM pueden revelar patologas que simulan a ICT. El doppler carotideo o angioIRM permiten identificar enfermedad
ateroesclerosa. TRATAMIENTO: La presencia de ICT ofrece la oportunidad de iniciar tratamientos que limiten el posible inicio de un infarto cerebral. No
existe tratamiento especfico para la ICT, debe individualizarse en base a los factores de riesgo y alteraciones encontradas en cada paciente. En la HTA
reducciones de 10mmHg en sistlica, as como 5 en diastlica se asocian con un 30-40% de reduccin de riesgo de EVC, por ello es uno de los principales
elementos en la prevencin de nuevos eventos de ICT y de infarto cerebral. Los niveles de glucosa en pacientes con ICT deben ser <126mg/dl, y en casos
ya confirmados de DM, el tx., con hipoglucemiantes orales y/o insulina debe ser iniciado de inmediato y control estricto. Manejo farmacolgico:
Antiagregantes plaquetarios est indicado en los casos de ICT de origen no cardioemblico ya que se ha demostrado disminucin en el riesgo de EVC
recurrente en alrededor de 22%. Los aprobados como medida de prevencin secundaria son: aspirina (75-325mg/d), clopidogrel y la combinacin de
aspirina-dipiridamol de liberacin prolongada. El tratamiento con anticoagulantes orales est indicado en los casos de ICT secundarios a fibrilacin
auricular (ICT cardioemblica) u otras patologas cardiacas potencialmente embolignas. Se sugiere mantener valores de INR en promedio de 2.5 (23).Otro grupo de alto riesgo en quienes est indicada la anticoagulacin son los pacientes con ICT in crescendo. En estos casos la presentacin de los
episodios de ICT es repetitiva, ya que se ha observado cese de la sintomatologa de manera dramtica despus del inicio de la anticoagulacin. La
endarterectoma carotidea o stent carotideo se reserva para pacientes con criterios establecidos para dicho manejo. Dado el alto riesgo de desarrollar
isquemia cerebral en las horas o das que siguen al ICT, esta debe considerarse como una urgencia neurolgica. Los objetivos de su evaluacin oportuna
incluyen: confirmar que se trata de ICT, definir su mecanismo de produccin, para establecer un tratamiento adecuado.
CASO CLINICO
Mujer de 73 aos sin factores de riesgo vascular que acudi a
urgencias por presentar cuadro de inicio sbito de disminucin de la
movilidad en hemicuerpo izquierdo con cada al suelo. A su llegada a
urgencias estaba consciente con desviacin conjugada de la mirada a
la derecha y hemipleja completa izquierda, hemihipoestesia y reflejo
de Babinsky izquierdo presente. TC craneal sin lesiones evidentes. Se
aplic tratamiento tromboltico a los 150min del inicio de los sntomas.
En su evolucin en las primeras horas se objetiv clara mejora de la
movilidad de hemicuerpo izquierdo, persistiendo al alta hemiparesia
izquierda.

CASO CLINICO
Varn de 75 aos, dislipidemia en tratamiento hipolipemiante como
nico factor de riesgo cardiovascular. Trado a urgencias por cuadro de
hemiparesia izquierda, parlisis facial central izquierda, hipoestesia en
hemicuerpo izquierdo, disartria y Babinsky izquierdo. TC crneo sin
alteraciones. Se administr tratamiento tromboltico a 150 min de
evoluci. Posterios al tratamiento se mantiene semiologa neurolgica,
con TC de control a las 24h de la trombolisis con infarto en territorio
de arteria cerebral media con edema citotxico y desviacin de lnea
media. Se aadi tratamiento con manitol, con mejora del nivel de
conciencia

PREGUNTA
Cul es la conducta a seguir para mejorar la secuela del paciente?
RESPUESTA
a.- Rehabilitacin fsica.
b.- Rehabilitacion neuropsicolgica.
c.- Se evolucin es favorable.
d.- Rehabilitacin en casa.

PREGUNTA
Cul es el tiempo mximo para administracin de trombolisis.
RESPUESTA
a.- 100 minutos.
b.- 150 minutos.
c.- 200 minutos.
d.- 250 minutos.

CURSO ENARM CMN SIGLO XXI TEL: 36246001

Pharmed Solutions Institute

PGINA 205

MANUAL DE TRABAJO DEL CURSO ENARM CMN SIGLO XXI


EVENTO VASCULAR CEREBRAL (EVC)
CIENCIAS BASICAS: Comprenden un conjunto de trastornos de la vasculatura cerebral que conllevan a una disminucin del flujo sanguneo en el cerebro
(flujo sanguneo cerebral o FSC) con la consecuente afectacin, de manera transitoria o permanente, de la funcin de una regin generalizada del
cerebro o de una zona ms pequea o focal, sin que exista otra causa aparente que el origen vascular. La enfermedad cerebrovascular trae como
consecuencia procesos isqumicos o hemorrgicos, causando o no la subsecuente aparicin de sintomatologa o secuelas neurolgicas. La hipertensin
arterial (HTA) es el principal factor de riesgo de la enfermedad cerebrovascular. Epidemiologa: Actualmente la cifra de muertes por ECV supera los 5
millones anuales, lo que equivale a 1 de cada 10 muertes. Las enfermedades
cerebrovasculares ocupan el tercer lugar como causa de muerte en el mundo
occidental, despus de la cardiopata isqumica y el cncer, y la primera causa de
invalidez en personas adultas mayores de 65 aos. La enfermedad cerebrovascular fue
tambin la quinta causa principal de prdida de productividad, medido por los aos de
vida ajustados por discapacidad. Ello incluye los aos de prdida de productividad por
razn de muerte o distintos grados de discapacidad. Las mujeres son ms propensas a
padecer ECV, sobre todo cuando han perdido el factor protector estrognico. Factores
de Riesgo: El consumo de cigarrillos es el factor de riesgo modificable ms poderoso
que contribuye a la enfermedad cerebrovascular, independiente de otros factores de
riesgo. Otros factores de riesgo demostrados son la hipertensin arterial y la diabetes
mellitus. Por su parte, las dislipidemias suelen ser factores de riesgo ms importantes
en la enfermedad coronaria que en la cerebrovascular. CLASIFICACIN: Segn el
tiempo de evolucin de la ECV se agrupan en: Accidente isqumico transitorio (AIT). Es
cuando los sntomas de la focalidad neurolgica se recupera en menos de 24 horas sin
secuelas, de inicio sbito que por lo general dura menos de 15 minutos. Dficit
isqumico neurolgico reversible (RIND). Es cuando la duracin del dficit persiste por
ms de 24 h, pero los sntomas desaparecen en un plazo de 7 a 21 das, sin secuelas.
Accidente cerebrovascular (ACV) establecido: el dficit neurolgico de origen vascular persiste y no se modifica por ms de tres semanas desde su
instauracin y deja secuelas. ACV estable: el dficit neurolgico persiste sin modificaciones por 24 horas (en los casos de origen carotdeo) o 72 horas (en
los casos de origen vertebrobasilares), pudiendo luego evolucionar hacia un RIND o ACV establecido. ACV en evolucin o progresivo: la focalidad
neurolgica aumenta y el cuadro empeora o aparece nueva clnica en 24 a 48 horas. ACV con tendencia a la mejora o secuelas mnimas: son casos con
un curso regresivo de modo que la recuperacin al cabo de 3 semanas es mayor al 80%. Lo ms frecuente es dividirlos en dos grupos segn el
mecanismo, as suele ser de tipo isqumico o hemorrgico: Isqumico (85% de los casos). Trombtico (Infarto lacunar: oclusin de vasos cerebrales
pequeos ocluyendo la irrigacin sangunea de un volumen pequeo de tejido cerebral. Infarto de un gran vaso sanguneo. Emblico (Cardioemblico: la
embolia proviene del corazn, con frecuencia, de la aurcula cardaca). Arteria-arteria. Criptognico: la oclusin de un vaso intracraneal sin causa
aparente. Hemorrgico: (Intraparenquimatoso, Subdural, Epidural, Subaracnoideo). Tambin pueden ser clasificados en funcin del rea en el cerebro
donde ocurre el mayor dao o por el territorio vascular afectado y el curso clnico del trastorno. Etiologa: Trombosis en pacientes con arteroesclerosis,
la hemorragia cerebral hipertensiva, el accidente isqumico transitorio, el embolismo y la rotura de aneurismas. Las malformaciones arteriovenosas, la
vasculitis y la tromboflebitis tambin causan con frecuencia ECV. Otras causas menos frecuentes incluyen ciertas alteraciones hematolgicas como la
policitemia y la prpura trombocitopnica, los traumatismos de la arteria cartida, los aneurismas disecantes de la aorta, la hipotensin arterial
sistmica y la jaqueca con dficit neurolgico. Fisiopatologa: El flujo sanguneo cerebral (FSC) es aproximadamente 15-20% del gasto cardaco total, de
550 - 750 ml de sangre por cada kg de tejido por cada minuto. La disminucin o interrupcin del flujo sanguneo cerebral produce en el parnquima
cerebral dao celular y funcional que empeora con el transcurrir del tiempo. Penumbra isqumica: En el tejido cerebral donde ocurre la isquemia
resultado de la disminucin del FSC, se distinguen dos zonas: Un ncleo isqumico de isquemia intensa: A los 10 s de isquemia se aprecia prdida de la
actividad elctrica neuronal por alteraciones en los potenciales de membrana notable en el EEG. A los 30 s se observa fallo de la bomba sodio-potasio
con alteraciones en el flujo ionico y desequilibrio osmtico con prdida de la funcin neuronal y edema citotxico. Al cabo de 1 min y por el predominio
de la gluclisis anaerbico, aumenta a niveles letales la concentracin de cido lctico y los mediadores de la cascada isqumica. Despus de 5 min se
aprecian cambios irreversibles en los orgnulos intracelulares y muerte neuronal. Rodeando a este ncleo isqumico evoluciona la llamada penumbra
isqumica donde el efecto de la disminucin en el flujo sanguneo cerebral, el cual ha descendido a niveles crticos alrededor de 15 a 20 mL/100 g/min,7
no ha afectado la viabilidad celular. La extensin del rea de penumbra depende del mejor o peor funcionamiento de la circulacin colateral.
Vulnerabilidad celular: Las neuronas ms sensibles a la isquemia son las clulas de la corteza cerebral, del hipocampo, el cuerpo estriado y las clulas de
Purkinje del cerebelo. De las neuroglas, se afecta primero los oligodendrocitos, los astrocitos (gliosis reactiva) y por ltimo la microgla. Las clulas del
endotelio vascular son las ltimas en ser afectadas. Tambin se ha notado una menor concentracin de clulas progenitoras endoteliales en pacientes
con enfermedad cerebrovascular. Factores de influencia: Ciertos factores participan en el dao cerebral progresivo, como el calcio, acidosis lctica,
radicales libres, glutamato, el factor de adhesin plaquetaria y la descripcin gentica del individuo. El dao por isquemia cerebral se ver mayor o
menor dependiendo tambin de: El estado del flujo sanguneo cerebral regional; El tiempo que dura la oclusin vascular; El funcionamiento de la
criculacin colateral; El grado de vulnerabilidad celular frente a la isquemia; La presencia de sustancias vasoactivas como cidos grasos y radicales libres
en la zona afectada; Hiperglicemia; Hipertermia; Los valores de la tensin arterial; El grado de hipoxia. CUADRO CLNICO: La presentacin clnica de la
ECV se da fundamentalmente de dos formas: aguda, manifestada por el accidente cerebrovascular y la hemorragia subaracnoidea; y una forma crcnica
manifestada por demencia y epilepsia. Sntomas neurolgicos como debilidad, cambios del lenguaje, visin o cambios en la audicin, trastornos
sensitivos, alteracin del nivel de conciencia, ataxia, u otros cambios en la funcin motora sensorial. La enfermedad mental puede tambin producir
trastornos de la memoria. DIAGNSTICO: Ante la sospecha de enfermedad cerebrovascular, se necesita identificar la lesin y su ubicacin y obtener
informacin sobre el estado estructural del parnquima del cerebro y su condicin hemodinmica como consecuencia de la lesin. La evaluacin
neuropsicolgica de sujetos con dao producido por enfermedad cerebrovascular est enfocada en conocer las funciones afectadas y depende del tipo
de evento. En el infarto cerebral se estudia por imgenes radiolgicas los aspectos topogrficos de la lesin, especialmente antes de las 24 horas del
inicio del trastorno sbito. Los ms utilizados son la tomografa computarizada, la resonancia magntica y el estudio del flujo sanguneo regional
cerebral. Tratamiento: El tratamiento debe ser individualizado, segn las condiciones de cada paciente y la etapa de la enfermedad cerebrovascular,
sopesando los riesgos frente a los posibles beneficios. En general, hay tres etapas de tratamiento: la prevencin del accidente cerebrovascular; la terapia
provista inmediatamente despus de la persona sufrir un accidente cerebrovascular; y la rehabilitacin del paciente despus de sufrir el accidente
cerebrovascular. Para la prevencin de eventos cerebrovasculares, un estudio demostr que el uso de ramipril era efectivo en pacientes de alto riesgo
con o sin hipertensin arterial o ECV previo. La vitamina E no parece ser efectivo en reducir el riesgo de accidente cerebrovascular fatal o no fatal.

CURSO ENARM CMN SIGLO XXI TEL: 36246001

Pharmed Solutions Institute

PGINA 206

MANUAL DE TRABAJO DEL CURSO ENARM CMN SIGLO XXI


Terapia farmacolgica: El tratamiento mdico est destinado a reducir los riesgos y/o complicaciones de un accidente cerebrovascular a corto y largo
plazo. El uso de antitrombticos se indica tan pronto como se ha descartado una hemorragia intracraneal. Las enfermedades cerebrovasculares no
cardioemblicas, no hemorrgicas son tratadas con agentes antiplaquetarios, en lugar de la anticoagulacin oral como tratamiento inicial. La aspirina,
administrada entre 50-325 mg diarios, o combinada con dipiridamol de liberacin prolongada, y el clopidogrel (75 mg diarios) son las opciones
recomendadas de primera lnea. La combinacin de aspirina y clopidogrel, que bloquea la activacin de la plaqueta por el difosfato de adenosina, puede
ser superior a la aspirina sola en la reduccin de riesgo de eventos cerebrovasculares isqumicos. La aspirina en combinacin con el clopidogrel aumenta
el riesgo de hemorragia y no se recomienda combinada de forma rutinaria para los pacientes con accidente isqumico transitorio. Para casos con
enfermedad cerebrovascular cardioemblico como la fibrilacin auricular, prtesis de las vlvulas cardacas o prolapso de la vlvula mitral, se indica la
anticoagulacin a largo plazo, principalmente con heparina no freccionada, obteniendo un INR de 1,5-2,5. La administracin de 325 mg diarios de
aspirina se recomienda para aquellos que no pueden tomar anticoagulantes orales. En casos de infarto agudo de miocardio con trombo ventricular
izquierdo puede combinarse la anticoagulacin oral con aspirina hasta 162 mg diarios. En pacientes con miocardiopata dilatada tambin se indican
anticoagulantes orales o algunos clnicos consideran iniciar la terapia antiplaquetaria. Normalmente no se recomienda aadir agentes antiplaquetarios a
la warfarina en casos con enfermedad reumtica de la vlvula mitral, a menos que el paciente tenga una embolia recurrente a pesar de tener un INR
teraputico. En casos de calcificacin del anillo mitral se suele administrar tratamiento antiplaquetario. Los pacientes con regurgitacin mitral pueden
recibir warfarina o aspirina. Cerca de un 4-28% de los pacientes con hemorragia intracerebral presentan convulsiones, las cuales pueden ser rpidamente
controladas con una benzodiazepina, como lorazepam o diazepam, acompaado de fenitona o fosfenitona. El uso de una terapia anticonvulsiva de
manera profilctica en todos los casos de hemorragia intracerebral es controvertido, ya que no hay ensayos controlados que han demostrado un
beneficio claro. La tromblisis con activador tisular del plasmingeno se ha definido como el tratamiento de primera lnea del infarto isqumico agudo,
pero debe ser administrada durante las tres horas posteriores al accidente cerebrovascular. Control de la tensin arterial: Aunque no hay estudios
controlados que definan los niveles ptimos de presin arterial en pacientes con ECV, el seguimiento de los niveles de presin arterial es importante. Se
cree que la presin arterial muy elevada puede conducir a nuevas hemorragias y/o la expansin de un hematoma intracraneal. Por otra parte, la bajada
sbita de una presin arterial elevada puede comprometer la perfusin cerebral. Las dos excepciones al manejo conservador de la hipertensin arterial
son posterior al uso de activador del plasmingeno tisular y ante un concomitante infarto de miocardio. El nicardipino, labetalol, esmolol, y la hidralazina
son agentes que pueden ser utilizados cuando sea necesario el control de la presin arterial. No se suele usar el nitroprusiato ya que puede elevar la
presin intracraneal. La American Heart Association publica las siguientes pautas para el tratamiento de la hipertensin arterial: Si la presin arterial
sistlica es> 200 mmHg o la presin arterial media (PAM) es > 150 mmHg, se considera la reduccin agresiva de la presin arterial con la infusin
intravenosa continua con valoraciones frecuentes de la presin arterial (cada 5 min). Si la presin arterial sistlica es > 180 mmHg o PAM es > 130 mmHg
y hay evidencia o sospecha de hipertensin intracraneal (PIC), entonces se considera la vigilancia de la PIC y se reduce la presin arterial con el uso de
medicamentos por va intravenosa de manera intermitente o continua para mantener la presin de perfusin cerebral > 60-80 mmHg. Si la presin
arterial sistlica es > 180 o PAM es > 130 mmHg y no hay pruebas ni sospecha de elevacin de la PIC, se considera una modesta reduccin de la presin
arterial (PAM diana de 110 mmHg o presin arterial deseada de 160/90 mmHg) con controles de la presin arterial cada 15 minutos. Pronstico: La
complicacin ms catastrfica de los diversos tipos de enfermedad cerebrovascular es la isquemia sbita e irreversible de alguna parte del cerebro, es
decir, el accidente cerebrovascular, especialmente frecuente en ancianos. La severidad vara, desde la recuperacin total de las funciones cerebrales y
aquellas que del cerebro dependen, en menos de 24 horas, hasta la discapacidad severa y la muerte. La mortalidad por ECV isqumico en el primer mes
oscila entre el 17 y 34%, mientras la del hemorrgico puede ser dos veces mayor.
CASO CLINICO
Mujer de 73 aos sin factores de riesgo vascular que acudi a
urgencias por presentar cuadro de inicio sbito de disminucin de la
movilidad en hemicuerpo izquierdo con cada al suelo. A su llegada a
urgencias estaba consciente con desviacin conjugada de la mirada a
la derecha y hemipleja completa izquierda, hemihipoestesia y reflejo
de Babinsky izquierdo presente. Puntuacin en la escala NIHSS de 16.
TC craneal sin lesiones evidentes. Tras consentimiento informado
escrito se aplic tratamiento tromboltico a los 150min del inicio de los
sntomas.
PREGUNTA
Considerando la sintomatologia, cual es la arteria mas probablemente
involucrada?
RESPUESTA
a.- Arteria cerebral anterior izquierda.
b.- Arteria cerebral media Izquierda.
c.- Arteria cerebral anterior derecha.
d.- Arteria cerebral media derecha.
PREGUNTA
Cul es la medida teraputica mas adecuada para prevenir un EVC
izquemico mas adecuada?
RESPUESTA
a.- Aspirina y ramipril
b.- Aspirina y clopidrogel
c.- Clopidrogel y enoxaparina
d.- Clopidrogel y isosorbide

CASO CLINICO

CURSO ENARM CMN SIGLO XXI TEL: 36246001

Varn de 75 aos, con dislipemia en tratamiento con hipolipemiante


como nico factor de riesgo cardiovascular. Trado a urgencias por
cuadro de hemiparesia izquierda, parlisis facial central izquierda,
hipoestesia en hemicuerpo izquierdo, disartria y Babinsky izquierdo. TC
crneo sin alteraciones. NIHSS 19. Tras consentimiento informado
escrito se administr tratamiento tromboltico con 150min de
evolucin desde el inicio de los sntomas.
PREGUNTA
Considerando la sintomatologia, cual es la arteria mas probablemente
involucrada?
RESPUESTA
a.- Arteria cerebral anterior izquierda.
b.- Arteria cerebral media Izquierda.
c.- Arteria cerebral anterior derecha.
d.- Arteria cerebral media derecha.
CASO CLINICO
Varn de 54 aos fumador y con episodio compatible con accidente
isqumico transitorio dos aos antes. Trasladado a urgencias por
hemipleja derecha de 5 horas de evolucin, objetivndose plejia
completa de la extremidad superior y paresia de la extremidad inferior,
NIHSS 7.
PREGUNTA
Cual es la conducta a seguir mas apropiada al caso?
RESPUESTA
a.- Tratamiento sintomtico.
b.- Tratamiento trobolitico.
c.- Tratamiento antiplaquetario.
d.- Tratamiento anticoagulante.

Pharmed Solutions Institute

PGINA 207

MANUAL DE TRABAJO DEL CURSO ENARM CMN SIGLO XXI


CASO CLINICO
Mujer de 52 aos fumadora, sin otro factor de riesgo que fue
trasladada a urgencias por hemipleja derecha y afasia de instauracin
brusca. Destac a su ingreso hemipleja completa derecha y Babinsky
derecho, NIHSS de 18. Se realiz TC craneal sin que se objetivaran
alteraciones. Tras realizar consentimiento informado se aplic
trombolisis a los 150min del inicio de los sntomas. A las 24h persista
en la misma situacin neurolgica, realizando TC de control que
mostraba infarto isqumico extenso con edema y efecto masa, por lo
que se asoci tratamiento antiedematoso. Pese a ello 48h despus
present rpido deterioro neurolgico, con nuevo TC mostrando
progresin del edema y signos de enclavamiento de tronco,
desestimndose ciruga descompresiva. Finalmente la paciente fue
incluida en protocolo de donacin de rganos.
PREGUNTA
Considerando la sintomatologia, cual es la arteria mas probablemente
involucrada?
RESPUESTA
a.- Arteria cerebral anterior izquierda.
b.- Arteria cerebral media Izquierda.
c.- Arteria cerebral anterior derecha.
d.- Arteria cerebral media derecha.
CASO CLINICO
Masculino de 44 aos, con obesidad, dislipidemia e hipertensin
arterial. El motivo de su ingreso fue cuadro caracterizado por afasia
global y paresia de hemicara derecha de 20 minutos de evolucin. En
la TAC de crneo se observ, en la fase simple, imagen de
hiperdensidad en el trayecto de la cerebral media izquierda y en
ncleo lenticular. Electrocardiograma en ritmo sinusal y sin
alteraciones del ritmo y/o la conduccin. Ecocardiograma transtorcico
normal y en el ultrasonido carotideo se observ placa ateromatosa en
el origen de la arteria cartida interna derecha. Con el diagnstico de
evento vascular cerebral isqumico en evolucin y con una presin de
220/120mmHg
PREGUNTA
Cul es la conducta teraputica ms adecuada a seguir en este
momento?
RESPUESTA
a.- Enalapril VO 5mg
b.- Activador tisular de plasminogeno
c.- Nitroglicerina
d.- Tecnecteplase
PREGUNTA
Cul es la presin de perfusin cerebral normal?
RESPUESTA
a.- 5-15mmHg
b.- 20-35mmHg
c.- 50-100mmHg
d.- 100-150mmHg

dificultades para encontrar las palabras durante el discurso. El examen


neurolgico mostr una afasia de Broca moderada y una prdida
parcial de la sensibilidad del brazo y la pierna derechos. El examen del
fondo de ojo era normal. No exista sndrome confusional ni signos de
meningismo. La paciente presentaba un historial oncolgico de larga
evolucin. Fue diagnosticada de carcinoma de crvix uterino a los 26
aos, y de neoplasia mamaria a los 43. Un mes antes del episodio
actual se realiz una tomografa computarizada toracoabdominal
debido a un incremento de los marcadores tumorales en sangre. ste
mostr una masa ovrica, mltiples adenopatas abdominoplvicas y
un tromboembolismo pulmonar bilateral.
PREGUNTA
Cul es la frecuencia de presentar EVC dentro de los primeros 6
meses?
RESPUESTA
a.- 10 al 20 %
b.- 20 al 30 %
c.- 30 al 40 %
d.- Ms del 40 %
PREGUNTA
Qu tipo de evento vascular nos da la mayor mortalidad?
RESPUESTA
a.- EVC isqumico
b.- EVC hemorrgico
c.- ICT
d.- TVP
CASO CLINICO
Se trata de paciente femenino de la tercera dcada de la vida, que no
cuenta con antecedentes de importancia para su padecimiento actual,
siendo lo nico relevante la prdida de la visin en la mitad derecha de
los campos visuales de ambos ojos (hemianopsia homnima derecha).
PREGUNTA
Qu estructura es ms probable que esta afectada?
RESPUESTA
a.- Lesin de las vas visuales por detrs del quiasma ptico.
b.- Lesion de vas visuales por delante del quiasma ptico
c.- Fibras visuales de la cintilla ptica
d.- Corteza cerebral
PREGUNTA
Qu arteria es ms probable que este afectada?
RESPUESTA
a.- Cerebral anterior izquierda
b.- Cerebral posterior
c.- Cerebral media
d.- Cerebral anterior derecha
PREGUNTA
Qu lobulos ms probablemente se daaron?
RESPUESTA
a.- Lobulo occipital y talamo homolateral
b.- Lobulo frontal y talamo homolateral
c.- Lbulo temporal y talamo contralateral
d.- Lobulo occipital y talamo contralateral

PREGUNTA
Cul es la causa menos probable de EVC isqumico?
RESPUESTA
a.- Cardioembolico
b.- Aterotrombotico
c.- Indeterminado
d.- Pequeos vasos
CASO CLINICO
Mujer de 53 aos de edad que fue evaluada debido a un trastorno del
lenguaje de 12 h de evolucin e inicio brusco. La paciente refera

CURSO ENARM CMN SIGLO XXI TEL: 36246001

Pharmed Solutions Institute

PGINA 208

MANUAL DE TRABAJO DEL CURSO ENARM CMN SIGLO XXI


CRISIS CONVULSIVA, ESTATUS EPILEPTICO
CIENCIAS BASICAS: El estado epilptico (EE) es considerado como la mxima expresin de la epilepsia, debido a su importante morbilidad y mortalidad
tanto a corto como a largo plazo. Se debe recordar que definir epilepsia
implica la presencia de dos o ms crisis y no necesariamente el paciente con
EE deber ser diagnosticado con ep lepsia, principalmente cuando este se
presenta como manifestacin clnica o subclnica de una enfermedad
sistmica (por ejemplo, hipoglucemia), que al tratar se no se vuelve a
presentar. La definicin clsica de EE se centra en el tiempo de duracin y en
la cantidad de eventos
presentes en el paciente,
siendo
historicamente
definido como aquella
condicin con >30 minutos
de actividad epilptica
continua o dos o ms crisis
secuenciales sin recuperar completamente el estado de alerta entre las crisis. Con el paso de los aos,
la duracin que fue aceptada para EE tiende a disminuir de 30 minutos en las guas del Epilepsy
Foundation of Americas Wor ing Group on Status Epi lepticus a 10-20 minutos en el Veterans Affairs
Status Epilepticus Cooperation Study, y ms recientemente por Lowenstein a 5 minutos. La razn de
esta ltima definicin radica en el hecho de que una crisis tn co clnica generalizada (CTCG) tpica
raramente dur >5 minutos y la terminacin espontnea es ms difcil despus de este tiempo. A
mayor duracin de la crisis menor respuesta a FAE (Farmacos antiepilpticos) y aumenta el riesgo de
dao neuronal. SALUD PUBLICA: El 30% de los pacientes que debutan con EE sern epilpticos y de los
pa cientes con diagnstico establecido de epilepsia, el 5 a 15% sufren de EE en algn momento de su
vida. De ellos 0.5 a 1% presentan al menos un episodio cada ao. CLASIFICACION: Ver tabla anexa. PATOGENIA: La principal causa del EE son niveles
bajos de FAE en sangre en pacientes con epilepsia crnica (34%), causas sintomticas remotas (24%), eventos vascula res cerebrales (22%), anoxia o
hipoxia (~10%), causas metablicas (~10%), alcohol y supresin de drogas (~10%).En una revisin en Mxico las causas ms frecuentes de EE fueron la
presencia de lesiones perinatales 30%, cisticercosis intracraneal 20%, EVC 7%, trauma craneal 5%, neuroles 5%, idiopticos 30%, esclerosis tubero sa 5%
y tumores intracraneales 2.5%. En el EE es bien conocido que existe una alteracin en los mecanismos que normalmente detienen e in citan una
descarga epilptica tpica. Dicha inhibicin es mediada por los receptores del acido gama ami nobutrico (GABA), un neurotrasmisor inhibitorio, que es
responsable de la terminacin normal de una crisis. Por otro lado, la activacin del receptor N-metil-D aspartame (NMDA) por el neurotrasmisor
excitatorio glutamato es necesario para la propagacin de la acti vidad epilptica. El mecanismo por el que el EE induce muerte neuronal es originado
por la excesiva liberacin de glutamato, lo cual activa los receptores post-sinpticos de NMDA ocasionando la entrada de calcio dentro de la neuro na
con el subsecuente dao
neuronal (citotoxicidad). Esto
lleva a una cascada de eventos
que terminan en la muerte
celular. DIAGNOSTICO: Cuando
el paciente ha sufrido un
cuadro sugestivo de EETCG
llega
a
presentar
manifestaciones
que
son
facilmente identificables, como
son movimiento t nicos,
clnicos, mioclnicos, atnicos,
alteracin del estado de alerta,
etc. As mismo, existen
hallazgos como traumatismos
en cualquier parte del cuerpo,
dilatacin
de
pupilas
o
relajacin de esfnteres que
hacen ms sugestivo el
diagnstico. En otros casos el
paciente llega al servicio de
urgencias
estuporoso
o
somnoliento.
En
ambos
escenarios el primer paso
durante
la
evaluacin
diagnstica es la comprobacin
de que el paciente mantiene
constantes vitales dentro de la
normalidad y de que la
oxigenacin es la adecuada.
Cuando
el
paciente
se
mantiene
somnoliento
o
confuso puede ser difcil
determinar clnicamente si est en un estado posictal, un estado epilptico no convulsivo o un deterioro de conciencia de etiologa no epilptica. Es en
este escenario donde el estudio electroencefalogrfico (EEG) tiene un gran peso para la evaluacin diagnstica ya que la presencia de actividad

CURSO ENARM CMN SIGLO XXI TEL: 36246001

Pharmed Solutions Institute

PGINA 209

MANUAL DE TRABAJO DEL CURSO ENARM CMN SIGLO XXI


epileptiforme continua en el registro no deja duda sobre la causa de la alteracin del estado de conciencia del paciente. De la misma forma, el EEG nos
ser de gran ayuda en el diagstico diferencial de crisis no epilpticas, como en el caso de algunos trastornos del movimiento, sncope o pseudocrisis. El
siguiente paso, una vez establecido el origen epilptico del episodio ser determinar la causa de la crisis, lo cual ser posible en una de cada seis perso
nas.25 El origen de las crisis puede estar en trastornos precipitantes agudos que requieren una intervencin inmediata o trastorno preexistente que no
requiere esa rpida actuacin pero que puede influir en el pronstico y, por tanto, en la desicin de iniciar o no tratamiento. La historia clnica y la
exploracin fsica sern las que determinen las pruebas de imagen o de laboratorio que se van a realizar para determinar la etiologa de las crisis. En
pacientes con EE se describen cambios en la seal de resonancia magntica (IRM) de localizacin cortical sin una distribucin arterial determinada. La
hiperintensidad en difusin (DWI) se corresponde con edema citotxico, y en T2* (T2WI) con edema vasognico; la hiperseal en FLAIR traduce edema
mixto. La presencia de edema cerebral en el EE explica porque puede aparecer en la tomografa axial computarizada (TAC) un rea hipodensa con
borramiento de surcos y/o prdida de diferenciacin corticosubcortical. En IRM con contraste se observa alteracin de la barrera hematoenceflica y
esto explica para algunos autores el componente de edema vasognico y constituir la justificacin fisiopatolgica para considerar tratamiento con
corticoesteroides en esta patologa. La mejora clnica y electroencefalogrfica como respuesta al tratamiento se corresponde con la reversibilidad de los
cambios de seal en IRM y el hecho de que las regiones hiperintensas en DWI y T2 WI con bajo coeficiente de atenuacin de difusin se normalizan,
indican que estos cambio no predicen necrosis, como sucede cuando estn causados por isquemia cerebral. La IRM DWI puede tener un valor pronstico
al informar sobre la extensin y la localizacin de las alteraciones de la seal y el posible dao estructural. TRATAMIENTO: El EE es una emergencia
neurolgica. Tomando en cuenta la evidencia actual se sabe que mientras ms se tarde en tratar el EE, menor respuesta farmacol gica encontraremos.
A mayor duracin es ms dficil responder a las demandas metablicas del cerebro y pueden aparecer diferentes complicaciones neurolgicas y
sistmicas, en otras palabras, tiempo es cerebro. De 0 a 5 minutos es prioritario la estabilizacin de la va area, capacidad ventilatoria y hemodinmica
de nuestro paciente asegurando as el mantenimiento de los signos vitales. Al minuto 5 se busca estabilizar los signos vitales, agregando la
monitorizacin por gasometra de la oxige nacin y de ser necesario dar soporte ventilatorio. Se coloca una va intravenosa permeable con solucin
salina (soluciones glucosadas precipitan la fenitoina). Se toman muestras de sangre para medir niveles de FAE en suero (si los toma), glucosa srica,
electrolitos, urea y de considerar necesario perfil toxicolgico, estudios de imagen de trax, abdomen, cerebro (TAC o IRM) as como puncin lumbar.
Despus de controlados estos parmetros debe iniciarse a la mayor brevedad posible la administracin de FAE (5 a 10 minutos), as como tiamina en
caso que se sospeche dficit nutricional. Ver cuadro anexo de tratamiento.

CASO CLINICO
Femenino de 14 aos, abuelo paterno con DM2, abuela materna con
HAS, to abuelo paterno sufre epilepsia, padre con HAS, es trada por
familiar que refiere que al ir a despertarla no reacciona a estimulo
verbal y fsico, posterior inicia crisis convulsiva, refiriendo sacudida del
cuerpo seguida de extensin, sin especificar tiempo, a su trayecto al
hospital se mantiene consciente, en urgencias se recibe en estado
post-ictal obnubilada, plida, afebril, hidratada, Glasgow 12, presenta
crisis tnico-clonicas generalizadas de 2 minutos, cardiopulmonar sin
compromiso, resto sin alteraciones.

Paciente masculino de 24 aos de edad que tiene una historia de crisis


convulsivas generalizadas tnico clnicas, as como crisis parciales
complejas que iniciaron a los 4 aos de edad. Su primera CGTC fue
durante una enfermedad febril (39.5 C), estuvo sin tratamiento y
asintomtico, y a los 9 aos de edad repite la CGTC, desde entonces
recibe tratamiento con carbamacepina. Sin embargo en el ltimo mes
presenta de 4-6 episodios con la mirada fija, automatismos,
desconexin, que dura aprox. 3-5 minutos, seguidos por un perodo de
confusin que dura 2 a 3 minutos. Sus CCTC no se han vuelto a
presentar.

PREGUNTA
Cul es la conducta ms adecuada a seguir en este momento?
RESPUESTA
a.- Valproato de magnesio
b.- Toma de glucosa
c.- Benzodiazepina
d.- Anticomicial

PREGUNTA
Si este paciente requiriera intubacin. Qu medida es la ms
adecuada a seguir antes de realizar procedimiento?
RESPUESTA
a.- Soporte vasopresor
b.- Toma de glucosa sangunea
c.- Iniciar benzodiacepina
d.- Examen neurolgico

PREGUNTA
Una hora despus la paciente presenta nueva crisis convulsiva que
duro 6 min. Cul es la conducta ms adecuada a seguir en este
momento?
RESPUESTA
a.- Midazolam IM
b.- Tiopental mas midazolam IM
c.- Loracepam IV mas fenitoina
d.- Ac. Valproico mas carbamazepina
PREGUNTA
Cul es la probabilidad de que esta paciente sea epilptica?
RESPUESTA
a.- 15%
b.- 30%
c.- 45%
d.- 60%

PREGUNTA
Para yugular sus crisis se agreg fenitoina. Qu efecto adverso grave
es ms probable si hacemos una administracin rpida?
RESPUESTA
a.- Hipotensin
b.- Arritmias
c.- Depresin respiratoria
d.- Pancreatitis
PREGUNTA
En qu solucin debemos diluir la fenitoina, para evitar su
precipitacin?
RESPUESTA
a.- Sol. Salina 0.9%
b.- Sol. Hartmann
c.- Sol. Glucosada al 5%
d.- Sol. Ringer-lactato

CASO CLINICO

CURSO ENARM CMN SIGLO XXI TEL: 36246001

Pharmed Solutions Institute

PGINA 210

MANUAL DE TRABAJO DEL CURSO ENARM CMN SIGLO XXI


COMA Y MUERTE CEREBRAL:
CIENCIAS BASICAS: Dentro de las alteraciones agudas de la vigilia; la obnubilacin significa literalmente embotamiento mental o torpeza, se aplica a
enfermos con reduccin leve o moderada del estado de vigilia. Es distintivo en ellos lograr la reaccin de despertar con estmulos sonoros ms o menos
intensos. En el estupor el paciente no responde y se despierta solo con estmulos verbales o dolorosos repetidos y fuertes, tras los cuales vuelve a
sumirse en un sueo profundo. En el coma, ningn estmulo despierta al sujeto; incluso la estimulacin dolorosa no provoca respuestas intencionadas y
pueden producir postura refleja de decorticacin o descerebraciones, se debe a una disfuncin cerebral orgnica difusa. Definicin de COMA; Mxima
degradacin del estado de conciencia. Sndrome clnico caracterizado por una prdida de las funciones de la vida de relacin y conservacin de las de la
vida vegetativa, como expresin de una disfuncin cerebral aguda y grave. Teasdale y Jennet definen el coma como la incapacidad de obedecer rdenes,
hablar y mantener los ojos abiertos. PATOGENIA: La alerta se mantiene por el Sistema Reticular Activador Ascendente (SRAA) localizado entre el tercio
medio de la protuberancia y la porcin ms alta del mesencfalo. Este sistema es una estructura polisinptica que se puede afectar por procesos
intrnsecos del tallo cerebral que lo destruyen, por procesos extrnsecos que lo comprimen o desplacen y por procesos metablicos que lo alteran o
inhiben. Una lesin hemisfrica produce coma directamente por su volumen o de manera indirecta por compresin, isquemia o hemorragia en el
COMA POR LESIN ANATMICA
mesencfalo y tlamo. Esta lesin por crecimiento radial crea un cono de presin transtentorial y comprimen el
SUPRATENTORIALES (15 A 20 %)
SRAA en la parte rostral del tronco enceflico. CLASIFICACION: Existen mltiples clasificaciones del estado de
Intracerebrales: Hemorragia cerebral,
coma. Segn el porcentaje de dao cerebral, el sitio y la causa del coma tenemos, ver cuadro. DIAGNSTICO: La
Hemorragia intraventricular, Infarto
historia clnica de los pacientes en coma sigue siendo el elemento fundamental de su diagnstico. Ante todo es
cerebral extenso (arterial o venoso),
preciso interrogar a las personas que traen al enfermo para extraer los datos posibles sobre las circunstancias de
Tumores, Infecciones (Encefalitis
focal, absceso cerebral)
aparicin del evento. El inicio del cuadro puede ser sbito (paro cardiaco, hemorragia o embolias cerebrales) o

Extracerebrales:
Tumores,
progresivo (intoxicaciones, tumores, trombosis cerebral, meningoencefalitis, encefalopata heptica,
Hidrocefalia, Hemorragia intracraneal
encefalopata urmica). Un TCE reciente puede sugerir un hematoma epidural o una contusin cerebral y si es de
postraumtica (epidural, subdural),
ms tiempo orienta hacia un hematoma subdural crnico. Se recogern los antecedentes inmediatos y antiguos,
Empiema subdural
INFRATENTORIALES (10 A 15 %)
el antecedente reciente de cefalea puede orientar hacia una masa expansiva intracraneal (tumor, hematoma,
Oclusin basilar
absceso); el de epilepsia, a un coma postcrtico y la existencia de focalidad neurolgica, a un tumor o isuqemia
Hematoma subdural y extradural de la
cerebral. Exploracin fsica siempre debe realizarse completa por aparatos, ya que puede aportar datos muy
fosa posterior
tiles. Nivel de conciencia la profundidad del coma se explora aplicando al paciente estmulos de intensidad
Hemorragia pontina primaria
creciente (verbal, tctil y dolorosa) y se clasificar segn la mejor respuesta obtenida durante la exploracin. Para
Hemorragia cerebelosa
Infarto cerebeloso
valoracin del estado de COMA utilizamos la escala de Glasgow. A todo paciente en coma se le deber realizar un
Malformaciones arterio venosas del
estudio analtico bsico para descartar una causa metablica de coma (Diabetes mellitus, hipoglucemia, coma
tronco
urmico, encefalopata heptica o coma por diselectrolitemia). Ante la sospecha clnica de coma exgeno y en
enceflico
todos los casos de coma sin diagnstico evidente debera realizarse un estudio toxicolgico de sangre y orina. Es
Aneurisma de la arteria basilar
Abscesos
aconsejable determinar la presencia de alcohol, barbitricos, benzodiacepinas, antidepresivos tricclicos,
Granulomas
fenotiazinas y opiceos. TRATAMIENTO: Medidas generales: Asegurar la oxigenacin, dada la necesidad de un
Tumores primarios o metastsicos
aporte continuo de oxgeno al cerebro es necesario priorizar y garantizar la funcin respiratoria. En caso de coma
Mielinolisis central pontina
profundo se puede intubar al enfermo como profilaxis de la broncoaspiracin. Mantener la circulacin: Con el fin
COMA POR LESIONES DIFUSAS (TXICOde mantener flujo sanguneo cerebral adecuado. Si deterioro hemodinmico, obrar segn corresponda con
METABLICAS, 65 A 75 %)
EXGENOS
aporte de volumen, drogas vasoactivas, etc. Evitar hipotensin brusca en caso de emergencia hipertensiva y
Frmacos
coma. No bajar TA diastlica por debajo de 100 mmHg. Si alcoholismo crnico o desnutricin se administrar 100
Txicos
mg intramuscular y 20 mg endovenoso de Tiamina y luego 50 mL de dextroza al 50 % IV. (25 gramos). Si se
Trastornos fsicos
administra esta antes de la Tiamina se puede precipitar una encefalopata de Wernicke. Otras medidas;
ENDGENOS
Hipoglicemia
Vaciamiento del contenido gstrico: Sonda nasogstrica a bolsa previo lavado gstrico. Sonda vesical: Medir
Hipoxia: Disminucin de la tensin de
diuresis horaria. Considerar antdotos y corregir causas: Tratar las causas de reversibilidad inmediata. Glucosa 50
oxgeno: PaO2 35mmHg, enfermedades
% IV. si hipoglucemia sospechada o constatada. Tiamina (B1) 100 mg IM si alcoholismo. Flumazenilo 0,25 mg IV. si
pulmonares, alturas, Hipoventilacin.
sospecha intoxicacin con benzodiacepinas. Fisostigmina 5 mL = 2 mg. Administrar lentamente 1 mp. cada 30Disminucin del contenido sanguneo de
60 min. en intoxicaciones graves por antidepresivos tricclicos Naloxona, 1 mL = 0,4 mg. Dosis: 10 mcg/kg o 400
oxgeno: Anemia, intoxicacin por CO2,
metahemoglobulinemia
mcg /dosis nica, si se sospecha intoxicacin por Opiceos. Tratamiento del edema cerebral: Dexametasona 10
Shoc : Cardiognico, hipovolmico o
mg IV y seguir con dosis de 4 mg IV. cada 6 horas. Manitol al 20 %: 1g/kg IV. en 20 min. Iniciar cuando se aprecia
sptico
deterioro rostrocaudal. Evitar soluciones hipotnicas y de glucosa puras- Tratamiento anticonvulsivo: Diazepam 2
Alteraciones metablicas: Hiper o
mg/min. IV. hasta 20 mg, Clonazepam 1mg/min. IV. hasta 6 mg, Fenitoina (250 mg-5 mL) 2,5 cc IV. c/8 horas.
hiponatremia,
hipercalcemia, hiper o hipomagnesemia,
MUERTE CEREBRAL: Es la expresin con la que se designa la perdida de todas las funciones del encfalo. Se
acidosis metablica o respiratoria, hiper
declara cuando los reflejos del tronco cerebral, las respuestas motoras y la actividad respiratoria estn ausentes
o hipoosmolaridad, hipofosfatemia
en un individuo comotaso normotrmico, con lesin cerebral masiva e irreversible, que no haya recibido drogas o
frmacos que acten en el sistema nervioso central y que no tenga otros factores que contribuyan que contribuyan al cuadro clnico=coma estructural,
origen conocido, dao irreversible, hemodinamicamente estable, temperatura mayor de 34C, ventilacin y oxigenacin adecuada, ausencia de
enfermedades metablicas importantes, ausencia de frmacos o drogas que acten sobre el SNC. Esta definicin permite certificar la muerte aunque se
encuentre funcionando la mayor parte de sus rganos del cuerpo. Solo es posible establecer este diagnstico en el medio hospitalario (unidades de
reanimacin o de cuidados intensivos), y requiere un protocolo extremadamente riguroso, la intervencin de personal experto y la realizacin de
pruebas confirmatorias objetivas y fiables. Adems, para aumentar la seguridad, se fija un periodo de observacin cuya duracin depende de la edad del
paciente. PATOGENIA: La muerte cerebral implica siempre la lesin estructural del encfalo, en gran parte de los casos suele ser primaria, inicialmente
localizada y supratentorial, por ejemplo: hemorragia intracerebral espontanea, traumatismo cerebral, infarto cerebral, hemorragia subaracnoidea y
tumores cerebrales. Menos de 15 % de las lesiones enceflicas son globales, supratentoriales e infratentoriales, casi siempre secundarias a un paro
cardiaco o respiratorio que provoca anoxia enceflica y con menor frecuencia debidas a infeccin del sistema nervioso central y edema celular
ocasionando por txicos o trastornos hidroelectrolticos. Se produce edema global del parnquima enceflico, con el consiguiente aumento de la presin
intracraneal, que iguala a la presin de perfusin enceflica. Esto ocasiona paro circulatorio intracraneal y conduce a infarto enceflico total y a isquemia
global del encfalo; el estadio final es la necrosis de toda la masa enceflica=muerte cerebral. La prdida de funciones del tronco del encfalo sigue casi
siempre a una secuencia rostrocaudal. Se inicia en el mesencfalo y finaliza en el bulbo. Su fase clnica final suele distinguirse por la cada brusca y
significativa de la presin arterial sistmica, tanto sistlica como diastlica. El paro circulatorio intracraneal completo puede ocurrir simultneamente o
ms tarde, pero nunca antes. DIAGNOSTICO: Exige una certeza absoluta, por lo cual debe seguirse el protocolo sistemtico, estricto y riguroso. Los tres
pilares son: a) conocer la causa de la lesin enceflica, b) descartar los trastornos que pudieran simular la muerte enceflica (condiciones

CURSO ENARM CMN SIGLO XXI TEL: 36246001

Pharmed Solutions Institute

PGINA 211

MANUAL DE TRABAJO DEL CURSO ENARM CMN SIGLO XXI


hemodinmicas, metablicas, farmacolgicas y toxicas actuales o relativamente recientes y c) efectuar una exploracin neurolgica reglada. La ley
General de Salud, dedica su ttulo decimocuarto a la donacin, trasplantes y prdida de vida. Especficamente los artculos 343 y344 del captulo IV se
refieren a la perdida de vida. Artculo 344: La muerte cerebral se define como la ausencia total e irreversible de todas las funciones cerebrales. Se puede
considerar muerte cerebral cuando existen los siguientes signos: I. Perdida permanente e irreversible de conciencia y de respuesta a estmulos
sensoriales. II. Ausencia de automatismo respiratorio. III. Evidencia de dao irreversible del tallo cerebral, manifestado por arreflexia pupilar y ausencia
de movimientos oculares en pruebas vestibulares y de respuesta a estmulos nociceptivos. En la legislacin mexicana se ha determinado que la muerte
cerebral es irreversible y que puede considerar el fin de vida. De acuerdo con los criterios del artculo 344, no hay diferencia entre el estado vegetativo
persistente y la muerte cerebral. El personal de salud debe involucrarse en la creacin de una normatividad y lenguaje comn sobre muerte cerebral,
mientras que la ley debe adecuarse a los avances tecnolgicos y humansticos.
CASO CLINICO
Varn de 48 aos que acude a nuestro por nuseas y malestar de 2
das de evolucin. Desarrolla rpidamente coma, acidosis metablica
severa (pH 6,8) con anin GAP elevado e infiltrado pulmonar bilateral
que requiere soporte ventilatorio invasivo. Se realiza TAC craneal, que
es normal. En las siguientes 12 horas presenta rigidez de
descerebracin. En un nuevo TAC se observa hipodensidad putaminal
bilateral y edema cerebral difuso. Ante la sospecha de intoxicacin por
metanol se inicia tratamiento con dilisis y etanol. La evolucin
neurolgica es desfavorable, producindose muerte enceflica en 24
horas. Posteriormente se documentaron niveles de metanol de 0,4g/l.
PREGUNTA
Cual de las siguientes pruebas es menos til para establecer el
diagnostico de muerte cerebral?
RESPUESTA
a.- EEG
b.- PEV
c.- Retiro de apoyo ventilatorio.
d.- IRM
CASO CLINICO
Paciente joven de sexo femenino que tras consumir 50mg de xtasis
ingresa con una encefalopata hiponatrmica e insuficiencia
respiratoria aguda severa por edema pulmonar, destacndose una
natremia de 109mEq/l y una osmolaridad urinaria de 360mOsm/l. Se
realiz la correccin de la hiponatremia mediante la infusin de
cloruro sdico hipertnico al 3%, presentando una mala evolucin con
fallo respiratorio y shock. A las 9h la ecografa doppler transcraneal
evidenci un patrn compatible con muerte enceflica. La necropsia
mostr edema cerebral con signos de herniacin y hepatizacin del
parnquima pulmonar.
PREGUNTA
Cual es la casua ms probable de esta condicin?
RESPUESTA
a.- Sndrome de secrecin inadecuada de hormona antidiurtica.
b.- Prdida de fluidos hipotnicos (hipertermia, sudoracin).
c.- Prdida de la capacidad de dilucin urinaria por lesin tubular renal
proximal.
d.- Reabsorcin de fluidos hipotnicos desde la luz intestinal.
CASO CLINICO
Una mujer de 29 aos cursando la 17ma semana de su primer
embarazo, sin complicaciones, salvo por una ot tis media supurada
tratada con antibiticos por va oral, consult al servicio de
emergencias por cefalea de 4 das de evolucin que haba empeorado
en las ltimas horas. Durante la consulta present prdida del
conocimiento y del tono muscular que inmediatamente progres al
esta do de coma. Inicialmente presenta anisocoria y luego miosis
pupilar sin respuesta al estmulo luminoso. La tomografla computada
mostr un hematoma en el he misferio cerebeloso izquierdo con
pasaje de sangre a los ventrculos cerebrales; la ecografa obsttrica
mostr v talidad fetal.
PREGUNTA
Al ingreso a la unidad de cuidados intensivos presen taba hipotonia
generalizada, ausencia de reflejos osteotendinosos y reflejos

CURSO ENARM CMN SIGLO XXI TEL: 36246001

oculoceflicos patolgicos. Adems desarroll hipotensin arterial.


Cul es el medicamento mas adecuado en este momento?
RESPUESTA
a.- Dopamina
b.- Dobutamina
c.- Epinefrina
d.- Adrenalina
PREGUNTA
Alas 12 hrs del ingreso se le realizo un electroencefalograma, que
mostr ausencia de actividad cortical, y potenciales evocados auditivos
y somatosensitivos sin obtenerse respuesta evocadacortical ni del
tronco del encfalo. Durante 56 das se mantuvo con sostn vital
artificial, correccin de dficit hormonal, nutricin enteral y
tratamiento de las infecciones. Durante la 25 semana de embarazo
tuvo un paro cardiaco. Cul es la conducta teraputica mas adecuada
a seguir en este caso?
RESPUESTA
a.- Practicar una cesrea
b.- Dar RCP y continuar embarazo
c.- Evitar manejo adicional
d.- Verificar vitalidad fetal
CASO CLINICO
Paciente de 37 aos, consulta en hospital comarcal por cefalea de 48h
de evolucin acompaada de vmitos proyectivos las ltimas 24h. Mal
control de la cefalea con analgesia convencional + mrficos. Se decide
TC craneal urgente. AP: sin alergias medicamentosas conocidas.
Comunicacin interventricular. ltima ecocariografa: FEVI 68%.
Pequea CIV en septum basal sin repercusin. Antecedentes familares:
historia familiar de transaminasitis, hermana muerta de
hepatocarcinoma a los 39 aos; padre con hepatocacarcinoma 4cm
pendiente de tratamiento. Exploracin Clnica: GCS inicial 15, sin
focalidades neurolgicas. PICNR, movimientos oculares conservados.
Nistagmus rpido que se agota de forma espontnea. Colaborador. No
meningismo TC : gran masa sellar-supraselar e infraselar de 4.4 x 4 cm
de dimetro, de caractersticas infiltrativas, que incluye las cartidas
cavernosas y destruye el clivus y se extiende a ambos senos
cavernosos y esfenoidales. Compatible con proceso neoformativo.
Probable diagnostico: meningioma aunque en contra est la extensin
de la destruccin osea, a considerar tambin como schawanoma o
cordoma. Paciente que ingresa por lesin expansiva en clivus junto con
deterioro progresivodel nivel de conciencia (GCS 6) Tendencia HTA y
empeoramiento progresivo NRL con midriasis bilateral y perdida
reflejos de tronco. Paciente sin sedo-analgesia y tras administrar
flumazenilo y naloxona, no presenta ninguna respuesta motora,
pupilas midriticas arreactivas y ausencia reflejos corneales. Nuevo TC:
swelling masivo.
PREGUNTA
Cul es el diagnotico ms probable para este caso en este momento?
RESPUESTA
a.- Sindrome compresivo
b.- Muerte enceflica
c.- METS generalizadas
d.- Isquemia cerebral

Pharmed Solutions Institute

PGINA 212

MANUAL DE TRABAJO DEL CURSO ENARM CMN SIGLO XXI


DELIRIUM:
CIENCIAS BASICAS: Alteracin fluctuante de la atencin, conciencia y cognicin. La causa siempre es orgnica y multifactorial, el inicio agudo y subagudo
y el curso fluctuante a lo largo del da, con alternancia de intervalos lucidos diurnos y empeoramiento nocturno. Delirium tremens: (cuadro confusional
agudo) secundario a la privacin alcohlica. Complicacin grave del sndrome de abstinencia alcohlica. Este aparece en casos de dependencia a alcohol,
entre 4-12 horas despus de la ltima ingesta. La ingesta habitual de alcohol para desarrollarlo es muy variable. SALUD PUBLICA: Su incidencia aumenta
su con la edad y es ms prevalente en la poblacin anciana. Es mucho ms frecuente a nivel hospitalario. Prevalencia hospitalaria 14-56%, mortalidad
hospitalaria 25-33%. Rpercusion sobre el pronstico vital y funcional, relacin con el envehecimiento y la demencia. Es la forma de presentacin de
numerosas enfermedades. Factores predisponentes: alteracin de la agudez visual, enfermedad grave
CUADRO CLINICO
(APACHE II <16), deterioro cognitivo (MME <24, 6 meses), deshidratacin. Factores precipitantes: 3 o
DELIRIUM
DEMENCIA
mas frmacos, yatrogena, catter urinario, contencin fsica, malnutricin <3gr. PATOGENIA: No est
Comienzo
Sbito
Insidioso
Curso
Fluctuante
Estable
bien definida, pero el sustrato bsico, consiste en un desorden generalizado del metabolismo cerebral
Conciencia
Disminuida
Normal
y la neurotransmisin que afecta a estructuras corticales y subcorticales encargadas de mantener el
Atencin
Alterada
Normal, salvo en
nivel de conciencia y la atencin. Originado por hiperreactivacin de receptores NMDA, hiperactividad
globalmente
casos graves
del sistema Noradrenrgico y Dopaminrgico. Hipoactivacin del sistema GABA. En esta disfuncin se
Cognicin
Alterada
Deteriorada
encuentran implicados neurotransmisores como acetilcolina, dopamina, GABA y serotonina, los cuales
globalmente
globalmente
Alucinaciones
Visuales
A menudo ausentes
unos por exceso y otros por defecto, justifican los sntomas. Multifactorial, interrelacin entre factores
Ilusiones
Fugaces
A menudo ausentes
predisponentes intrnsecos (envejecimiento cerebral, patologa orgnica cerebral, episodios previos de
Orientacin
Deteriorada
Deteriorada
delirium, dficit funcionales y factores estresantes) a un paciente vulnerable y factores precipitantes
Lenguaje
Incoherente
Pobre,
externos como: infecciones respiratorias y urinarias, alteraciones metablicas, endocrinopatas,
perseveracin
trastornos cardiopulmonares y gastrointestinales, el perioperatorio y medicamentos; principalmente
Enf. orgnica
Siempre
Ausente
sedantes hipnticos (BZD) anticolinrgicos (difenhidramina, hidroxizina, levodopa, amantadina,
escopolamina, atropina, amitrptilina, imipramina, antiespasmdicos), antiarritmicos (propanolol, digoxina, procainamina, lidocana), antihipertensivos
(reserpina, metildopa), psicotrpicos (amitriptilina, litio, benzodiacepinas, neurolpticos, opiceos, hipnticos), antagonistas H2 (ranitidina, cimetidina,
famotidina). Tambin hay que recordar los estados de intoxicacin por frmacos, en estrecha relacin con los cambios farmacocinticas y
farmacodinamicos asociados a la edad, como disminucin de la masa corporal con aumento de materia grasa, disminucin de la filtracin glomerular y
aclaramiento de la creatinina, deterior de funcin heptica y disminucin de protenas, que hacen que dosis teraputicas sean toxicas para ancianos.
Otros factores son las lesiones que produce la hospitalizacin (deprivacin del sueo, interrupcin de rutinas, cambios del ambiente, inmovilizacin y uso
de restricciones fsicas etc). DIAGNOSTICO: Es clnico; la anamnesis y exploracin fsica son fundamentales, caractersticas principales: inicio agudo y
curso fluctuante, inatencin, pensamiento desorganizado, alteracin de nivel de conciencia, debemos revisar el nivel de conciencia, atencin (respuesta
a estmulos, destruido, respuesta a instrucciones), memoria (perturbacin de memoria, inmediata y reciente), orientacin (esfera temporal, segunda de
espacial y personal), pensamiento y lenguaje (lenguaje, irrelevante y repetitivo, incoherente, con circunloquias, la capacidad de abstraccin es nula),
percepcin (interpretacin errnea de un objeto real, alucinaciones, principalmente visuales) conducta (actividad psicomotora disminuida, con apata e
inmovilidad o bien inquietud y agitacin), estado afectivo (euforia o ansiedad, que alternan con apata, indiferencia y depresin), ciclo sueo-vigilia
(insomnio con empeoramiento de la confusin durante la noche e hiperinsomnia durante el da). Las pruebas complementarias bsicas y las tcnicas de
imagen permiten hacer una valoracin etiolgica, como BH, QS, VSG, Funcin renal, heptica, electrolitos, glucosa, calcio, EGO, determinacin de niveles
de medicamento, ECG, radiografa de trax. TIPOS: Delirium hiperactivo (29%), delirium hipoactivo (29%); son pacientes mas graves y con peor
pronostico, estancias mas largas con las consecuentes ulceras por presin), delirium mixto (hiperactivo-hipoactivo). TRATAMIENTO: Medidas
preventivas, ya que reducen la incidencia y reduccin del cuadro clnico. Eliminar medicamentos innecesarios, alteraciones metablicas, hidratacin y
nutricin, psicoeducacion a la familia, proporcionar orientacin del paciente, remover lneas, venoclisis y otros equipos que reduzcan la movilidad,
mantener higiene del sueo, estimulacin durante el dia, preferencia de permanecer en casa, asilo u hospital. La utilizacin de psicofrmacos requiere la
valoracin riesgo beneficio, estn indicados, en sntomas como trastornos de la conducta e insomnio, los neurolpticos son de eleccin para el control de
la agitacin haloperidol es de 1ra eleccin 5-10mg es suficiente para iniciar, se puede repetir 5 mg cada 30min si la agitacin es incontrolable, es posible
duplicar la dosis cada 30 min, hasta el control de sntomas precaucion con sedacin e hipotencion. Los de ltima generacin o atpicos (risperidona,
olanzapina, clozapna), provocan menos efectos extrapiramidales y han demostrado similar eficacia al haloperidol, por lo que su utilizacin se est
extendiendo en la actualidad, no hay que olvidar sus efectos adversos como hipotensin ortostatica, sedacin, acatisia y agranulocitosis en el caso de
clozapina. Las benzodiacepinas constituyen la medicacin indicada en casos de delirium secundarios a sndromes de abstinencia por sedantes y alcohol y
el control del insomnio. Paciente alcohlico que acude a urgencias con sntomas de delirium tremens establecido: Este tipo de pacientes requiere ingreso
hospitalario en una unidad de medicina interna o de cuidados intensivos. Desde el punto de vista psiquitrico, puede valorarse a necesidad de
contencin mecnica debido a la agitacin, que en algunos casos puede ser extrema y que debe ser de 5 puntos. Debe mantenerse al paciente en un
ambiente bien iluminado, tranquilo, con escasos estmulos sensoriales: Dosis de ataque de 20mg de diazepam por va intramuscular, repitiendo cada 30
minutos hasta conseguir la sedacin Posteriormente se administrarn 10-20 mg cada 6h que se irn reduciendo de manera progresiva; 50-75mg de
cloracepato por va intramuscular, repitiendo cada 30 minutos hasta conseguir la sedacin y posteriormente, 50mg cada 6 horas reduciendo
progresivamente. Tambin Vitamina B1 tiamina.
CASO CLINICO
Masculino de 79 aos de edad el cual se encuentra diagnosticado con
demencia tipo Alzheimer desde hace 5 aos, actualmente permanece
solo en casa, con supervisin nocturna nicamente, se observa con
signos de desnutricin, con mal higiene generalizada, a la exploracin
fsica se observa con ruidos cardiopulmonares adecuados, se observa
con moderada agitacin psicomotriz que se alterna con periodos de
somnolencia, no hay datos de focalizacin, se realiza BH y EGO sin
datos de procesos infecciosos, se mantiene en observacin por la

CURSO ENARM CMN SIGLO XXI TEL: 36246001

noche los sntomas se agudizan con presentacin de ideas delirantes


de dao y alucinaciones visuales.
PREGUNTA
Cul es la conducta a seguir en este paciente?
RESPUESTA
a.- Realizar IRM de Craneo.
b.- Administrar haloperidol.
c.- Realizar hemocultivo.
d.- Cuantificacion de electrolitos sericos.

Pharmed Solutions Institute

PGINA 213

MANUAL DE TRABAJO DEL CURSO ENARM CMN SIGLO XXI


TRASTORNO BIPOLAR (TB):
CIENCIAS BASICAS: Es una enfermedad psiquitrica recurrente y severa que consiste en la alteracin cclica y recurrente del estado de nimo, entre
episodios de depresin (depresin intensa desesperanza), de mana (felicidad extrema) y mixtos (depresin y exceso de actividad en una fase maniaca).
Estos cambios duran normalmente varias semanas o meses. SALUD PUBLICA: Existe una prevalencia a lo largo de la vida de 1,6% para trastorno bipolar
tipo I y 0.5% para tipo II. La prevalencia por trasto en personas mayores de 65 aos que se
encuentran en la comunidad se estima en el 0.1-1%. Generalmente empieza durante o
despus de la adolescencia (antes de los 19 aos). Es raro que empiece despus de los 40
aos de edad. En relacin con el comportamiento suicida en TB, la prevalencia del intento
suicida fue de 17% para tipo I y 24% para tipo II. Tiende a ser una condicon permanente a
lo largo de la vida caracterizada por altas tasas de recaidas, ansiedad (75%) comorbida,
trastorno por abuso de sustancias (49%), disfucnion y mortalidad prematura debida
especialmente a la alta tasa de siucidio. ESPECTRO BIPOLAR: Bipolar I: mania y episodios
depresivos. Bipolar II hipomana y depresin. Bipolar II.5 depresion con temperamento
ciclotmico. Ciclotimia. Bipolar III depresiones repetidas con hipomana inducida por
frmacos. Bipolar.5w. abuso de sustancias (con el fin de inducir hipomana). Bipolar IV
depresin con temperamento hipertmico. PATOGENIA: Existe un dficit de 27% de
interneuronas en la corteza cerebral e hipocampo y una alta expresin de genes
proapoptticos como Bax, BAD, caspasa-9 y 3, y un descenso en la expresin de genes
antiapoptticos como BNDNF y Bcl-29. Posibles factores etiolgicos: Endgeno o
constitucional relacionado con el biotipo pcnico en un 70%. Hereditario, es de tipo
autosmico dominante. Biolgico (catecolaminicas, indolaminicas). Toxico (alcohol,
cocana, anfetaminas. Hormonales (tiroides, hipfisis, gnadas). Psicolgico (shocks
emocionales). La fisiopatologa, esta sin duda mediada por una red de circuitos lmbicos,
estriatales y fronto-corticales interconectados, as como por sistemas colinrgicos,
catecolaminrgicos y serotoninrgicos de neurotransmisores, adems de alteraciones de
neuroplasticidad y resiliencia celular.
DIAGNOSTICO: Manifestaciones generales;
alteraciones cognitivas, motoras, de impulsos, bsqueda de excitacin, emocin y peligro,
deshinibicion, aburrimiento, violencia, hostilidad, irritabilidad, negativismo, resentimiento,
DX. DIFERENCIAL
recelo, hostilidad verbal, culpabilidad. Impulsividad; conducta precipitada, falta de
ENTRE DEPRESION Y
planificacin, decisiones no meditadas, tendencia a actuar sin pensar, predisposicin hacia
TRASTORNO
reacciones rapidas no planificadas a estimulos internos o externos, sin previcion de las
BIPOLAR
consecuencias negativas de esta conducta para el propio individuo o para los dems.
Busqueda de sensaciones, novedad, susceptibilidad al aburrimiento, suponen la necesidad
de variedad, sensaciones fsicas a travs del peligro y la aventura. El diagnostico de bipolaridad se hace cuando se observa, que la depresin alterna con
un episodio maniaco., de intensidad variable y que alternan de manera cclica en el tiempo. Generalmente la depresin aparece primero y en pocas
ocasiones el primer episodio de trastorno bipolar es de tipo maniaco. La depresin es el sntoma dominante en el trastorno bipolar, la depresin es 3-5
veces mas frecuente que la mania. Caracteristicas sugestivas del espectro bipolar; comienzo en edad temprana, alta recurrencia (al menos 5 episodios),
comienzos y terminacin de episodios abruptos, depresin postparto, marcadas estaciones, rasgos atpicos (hiperinsomnia y comer en exceso), humor
hostil y labil, sntomas hipomaniacos aislados durante la depresin, ciclar con antidepresivos, refractario (por lo menos 3 antidepresivos de distintas
clases), historia familiar de bipolaridad, historia familiar de trastornos del animo en 3 generaciones. RIESGOS: Catastrofe econmica, muerte accidental,
delicuencia, hiperconsumo, riesgo profesional, baja adherencia, suicidio. TRATAMIENTO: La meta es dar una respuesta adecuada a ambas fases de la
enfermedad (mana, depresin), prevenir la frecuencia y mejorar la calidad.Paciente bipolar que debuta con mania; antipsictico atpico es primera
eleccin, estabilizador del estado de animo, carbamazepina. Episodio maniaco en un paciente bipolar; antipsictico atpico de prmera eleccin
(olanzapina, quetiapina, risperidona). Mantenimiento; carbonato de litio de primera elccion (en 3er nivel), en primer nivel acido valproico, otros
lamotrigina. Depresion bipolar aguda; lamotrigina, quetiapina, olanzapina/fluoxetina. PRONOSTICO: 97.5% de los pacientes bipolares evidencian
remisin de los sntomas clnicos agudos en los 24 meses posteriores al inicio del tratamiento, pero solo un 37.5% recupera de forma equiparable la
funcionalidad. Hay ajuste deficitario en cuanto a las relaciones familiares, matrimonio, actividad de ocio y actividades laborales. El 30-60% no alcanzaron
la recuperacin ocupacional y social, despus de un episodio afectivo y 273 no consiguieron la recuperacin funcinal completa. Medicamentos que
pueden causar mania o depresin; Aciclovir, ac. Nalidixico, ac. Valproico, alcohol, amantadina, anfetaminas, anticonceptivos orales, antidepresivos,
AINES, entre otros.
CASO CLINICO
Paciente de 17 aos, portadora de un trastorno bipolar, trastorno de
personalidad limtrofe y con antecedente de abuso de marihuana,
cocana, xtasis y herona. Despus de cuatro das de ser dada de alta
desde una clnica psiquitrica, donde estuvo dos meses en tratamiento
con clozapina, cido valproico, carbonato de litio y lorazepam, inici un
cuadro de confusin, letargia, conducta catatnica, rigidez, mialgias y
fiebre, que mantena por cinco das, razn por la que ingres a la
Unidad de Cuidados Intensivos (UCI) con el diagnstico presuntivo de
meningoencefalitis. En el examen fsico se constataron fiebre,
taquicardia, normotensin, rigidez principalmente del esqueleto axial,
y gran agitacin psicomotora. El hemograma de ingreso mostr 17.000
leucocitos/mm3 con 14% de baciliformes, plaquetas normales, VHS de

CURSO ENARM CMN SIGLO XXI TEL: 36246001

31, PCR de 19,8 mg/dl, CK de 920 mg/dl con fraccin MB de 22 mg/dl.


Se tomaron hemocultivos, cultivo de orina, radiografa de trax y
puncin lumbar que fueron normales, pese a lo cual recibi
antibiticos por sospecha de un cuadro pulmonar aspirativo.
PREGUNTA
Cul es la complicacin mas probable que presenta el caso?
RESPUESTA
a.- Sindrome serotoninergico maligno.
b.- Sindrome neurolptico maligno.
c.- Sndrome neurotxico por litio.
d.- Encefalopatia por hepatotoxicidad.

Pharmed Solutions Institute

PGINA 214

MANUAL DE TRABAJO DEL CURSO ENARM CMN SIGLO XXI


PSICOSIS Y SUICIDIO
CIENCIAS BASICAS: La Asociacin Psiquitrica Americana estableci la diferenciacin entre trastornos psicticos y trastornos no psicticos, puesto que,
ante una agresin exgena, es posible hallar, cuadros acreedores del trmino psicosis. Tambin se estableci la distincin entre trastorno "orgnico" y
trastorno "sintomtico". Esta subdivisin se apoya en el hecho de que en los trastornos orgnicos existira una lesin histopatolgica cerebral de carcter
permanente, con sintomatologa crnica y estable. En los trastornos sintomticos existira una alteracin fisiopatolgica cerebral sin lesiones orgnicas
pesquisables, perturbando el organismo en forma aguda y generalmente transitoria, con un cuadro clnico agudo y reversible. En algunos casos, si la
afeccin se prolongase o fuera demasiado intensa, o si existiera una labilidad previa del terreno, este cuadro podra evolucionar hacia un trastorno
orgnico. De este modo, el trastorno sintomtico sera la manifestacin nerviosa central de una enfermedad sistmica o de una intoxicacin que no
habra producido una lesin histopatolgica por el momento. As, aunque la sintomatologa confusional es preferentemente de origen "sintomtico" y la
semiologa demencial de procedencia "orgnica" es posible encontrar cuadros confusionales en una demencia, alteraciones irreversibles de memoria
despus de algunos delirium tremens o cuadros neurotiformes en muchos trastornos "orgnicos" o "sintomticos". Por esta razn, "trastornos
orgnicos". 1.- Demencia de aparicin senil o presenil. 2.- Trastornos mentales orgnicos inducidos por sustancias psicoactivas. 3.- Trastornos mentales
orgnicos asociados con trastornos fsicos del eje III, o cuya etiologa es desconocida. Dentro de estos: a.- Delirium, b.- Demencia, c.- Sndrome amnsico,
d.- Trastorno delirante orgnico, e.- Alucinosis orgnica, f.- Trastorno orgnico del estado de nimo, g.- Trastorno orgnico de ansiedad, h.-Trastorno
orgnico de la personalidad. La proclividad o la resistencia a hacer una psicosis somtica vara mucho de un sujeto a otro. Los factores individuales que
intervienen en la etiopatogenia son: psquicos, endgenos y somticos. Los factores ms influyentes son los de orden somtico, dentro de los cuales
debe considerarse: la va de penetracin de la noxa, los dispositivos bsicos en la defensa metablica e inmunitaria y el estado del sistema nervioso. Un
factor individual importante de considerar es la edad del enfermo. En los nios, las psicosis sintomticas son mucho ms frecuentes que entre los adultos
y a menudo adoptan la forma de "delirium". Entre los adolescentes abundan las psicosis sintomticas esquizofreniformes. A medida que la edad es ms
avanzada, aumenta la frecuencia del sndrome de Korsakow. Generalmente el EEG no se altera en las psicosis endgenas, mientras que a menudo
muestra anomalas persistentes de tipo difuso o central en las psicosis sintomticas, las alteraciones EEG en las psicosis sintomticas carecen de
especificacidad, varan segn las oscilaciones del nivel de conciencia. A medida que progresa el compromiso de conciencia aumenta la desorganizacin
del trazado y la lentificacin y la amplitud de la actividad bioelctrica. PSICOSIS EXOGENAS AGUDAS: Se refiere a las perturbaciones psiquitricas agudas
que aparecen en relacin directa con alteraciones somticas: afecciones cerebrales agudas, traumatismo y heridas cerebrales; epilepsia, enfermedades
generales (infecciones, intoxicaciones, anemias, etc.); trastornos endocrinos; puerperio; intoxicaciones por sustancias extraas al organismo (alcohol,
anfetaminas, etc.). Para que se produzca la perturbacin psiquitrica, las alteraciones mencionadas o las noxas que de ella provengan deben actuar
sobre el cerebro, de manera directa o indirecta. Bonhffer estableci los fundamentos del concepto de psicosis exgena y demostr que los cuadros
psicopatolgicos resultantes se reducen a un escaso nmero a los que denomin "tipos de reaccin exgena aguda", destacando los siguientes
sndromes o "formas de presentacin": estado delirioso, epileptoide-angustioso, crepuscular, amencial, alucinsico y estuporoso. El concepto de tipos de
reaccin exgena aguda lleva implcita la idea de una falta de especificidad en la relacin existente entre la naturaleza de la enfermedad somtica y el
cuadro psicopatolgico resultante. Esto significa que enfermedades corporales completamente diferentes pueden dar lugar al mismo cuadro
psicopatolgico. El sntoma central de estos trastornos es el enturbiamiento de conciencia en sus diversos grados (excepto en el caso de la alucinosis).
Bonhoffer tambin mencion las "formas de transcurso" de las reacciones exgenas agudas, es decir, conjunto de sntomas enmascarados entre los
otros, pertenecientes al sndrome hiperestsico emocional o al sndrome de Korsakow. El sndrome hiperestsico-emocional incluye: "aspecto fatigado y
marchito, color pajizo de la cara, hiperestesia a los ruidos, a la luz y el dolor, tristeza, mal humor, cefalea, mareos, astenia de predominio vesperal y
nocturno, labilidad de la memoria, labilidad emotiva, susceptibilidad a flor de piel, incapacidad de concentrarse, tendencia marcada a las
autorreferencias paranoicas las cuales nunca se configuran acabadamente, insomnio o sueo muy interrumpido, no reparador y con actividad onrica
laboral o terrorfica. El sndromde Korsakow est constituido por: desorientacin en tiempo y lugar, falso reconocimiento y fabulacin de perplejidad.
DIAGNOSTICO: Estos cuadros se inician bruscamente, aunque pueden no ser evidentes desde el principio cuando son de ligera intensidad. Generalmente
son reversibles si remite la patologa subyacente, pero algunos pueden evolucionar hacia un sndrome orgnico crnico, como cuando una psicosis post
traumtica aguda deja al descubierto una demencia o cuando la encefalopata de Wernicke se transforma en un sndrome amnsico duradero. Los
cuadros clnicos resultantes se deben fundamentalmente a la perturbacin de la funcin cerebral normal por alteraciones bioqumicas, elctricas o
mecnicas. Los sntomas comunes ms importantes son los siguientes: Alteracin del nivel de conciencia, alteraciones psicomotoras, trastornos del
pensamiento, alteraciones de la memoria, alteraciones perceptivas, alteraciones emocionales. Otras manifestaciones que aparecen en estadas poco
intensas de la enfermedad: sntomas neurotiformes de apariencia depresiva, hipocondraca, fbica, histrinica, paranoide o esquizofreniforme. Los
trastornos cuantitativos de conciencia corresponden a la obnubilacin, que agrupa los cuatro trastornos cuantitativos, desde el ms leve al ms grave:
embotamiento (disminucin o retardo en el ritmo de las elaboraciones psquicas. Fatigable; la captacin de los estmulos es trabajosa porque la
percepcin es lenta, imperfecta, imprecisa y carente de nitidez; se altera la memoria), somnolencia (tendencia al sueo, pesadez, tiende a quedarse en la
cama y dormitar en el da. Hay disminucin del ritmo alfa al EEG) o adormecimiento, sopor (slo logra despertarse parcialmente, reacciones
psicomotoras rudimentarias y presenta respuestas reflejas. El EEG muestra ondas delta) y coma (EEG tiende a ser isoelctrico). El trmino obnubilacin
significa que la conciencia est nublada o empaada en relacin con la funcin de alerta. TRASTORNOS CUALITATIVOS DE CONCIENCIA: 1.- ESTADO
CREPUSCULAR: Lo ms caracterstico de este trastorno es circunscribirse slo a ciertas manifestaciones de la vida psquica, durante el cual el paciente es
cogido por un afecto exaltado que tie de tal manera el campo de su vivenciar que da la impresin de que toda la actividad psquica del sujeto se
concentra slo en aquello que tiene relacin con su rabia, su angustia, su odio o su xtasis. Presenta ilusiones y alucinaciones, principalmente visuales
aunque tambin auditivas. El afecto desmandado puede llevar al enfermo a incurrir en actos de violencia o a estados beatficos de xtasis. Los estados
crepusculares en general son breves, de minutos hasta das, raramente duran semanas o meses. Debido al fondo obnubilatorio presente en todo estado
crepuscular, hay amensia parcial o total del episodio vivido, pero mientras est dentro de l, el enfermo recuerda lo que ha sucedido. Se tiende a
denominar estado crepuscular epilptico a todo cuadro psictico epilptico con compromiso de conciencia. La psicosis epilptica que presenta
fenmenos deliriosos no es un estado crepuscular epilptico, sino un estado delirioso epilptico. Los estados crepusculares aparecen con mayor
frecuencia en la epilepsia, en las psicosis sintomticas y en la histeria. La embriaguez patolgica es la forma de estado crepuscular ms frecuente de las
psicosis sintomticas, y se presentan generalmente en epilpticos, orgnicos o psicpatas, con dosis pequeas de alcohol. Son estados crepusculares
desorientados acompaados de agitacin psicomotora, con amnesia posterior. Todas las causas de hipoxia cerebral pueden provocar un estado
crepuscular. Los estados crepusculares de causa orgnica son desorientados. Pueden ser debidos a repercusiones patolgicas de las crisis epilpticas, el
edema cerebral, el proceso fundamental del cerebro, o bien a dosis excesivas de anticonvulsivantes. 2.- LA AMENCIA: Hay formas alucinatorias,
catatnica e ideofugal-incoherente. La obnubilacin es un embotamiento leve. La psicomotrocidad da la impresin de intencionalidad, aunque los actos
son muy cambiantes e incompletos. El pensamiento es incoherente y la percepcin de la realidad dificultosa. El sentimiento que coge al paciente es de
miedo, desamparo y angustia, que impregna todo el cuadro clnico. El miedo expectante lo lleva a sobresaltarse al primer ruido, originando as
percepciones deliriosas paranodeas. El desamparo se observa en la expresin llorosa y los sentimientos de soledad y abandono mencionados por el

CURSO ENARM CMN SIGLO XXI TEL: 36246001

Pharmed Solutions Institute

PGINA 215

MANUAL DE TRABAJO DEL CURSO ENARM CMN SIGLO XXI


paciente. El enfermo est muy angustiado y perplejo, presentando de forma caracterstica cambios afectivos bruscos. A ratos da la impresin de que se
sumerge en un mundo de vivencias deliriosas, alucinaciones y falsos reconocimientos, todo esto vivido con gran sentimiento de realidad, que lo lleva a
arrancar, defenderse, gritar, hacer movimientos bruscos, tornarse negativista. Dentro del cuadro, el enfermo recuerda difcilmente lo sucedido horas
antes o el da anterior; una vez pasado el episodio puede haber amnesia completa o recuerdos fragmentarios y borrosos. Dura semanas o meses. 3.ALUCINOSIS: Se caracteriza por la presencia de alucinaciones auditivas en forma de voces, un delirio persecutorio, en directa relacin con las voces,
generalmente no se presenta alteracin de conciencia (el paciente est orientado, atento y reflexico). El contenido de las voces es habitualmente de
reproche y condenacin, lo que en una atmsfera de intensa angustia lleva muchas veces al paciente al suicidio. Tambin se presentan voces que
discuten entre s, que comentan los propios actos o repiten lo que el enfermo piensa. Habitualmente se presenta en el alcoholismo y puede durar das o
semanas, pero tambin puede orientarse hacia la cronicidad. Conviene subrayar algunas diferencias entre la alucinosis alcohlica y el delirum tremens:
1.- El delirium tremens generalmente es desencadenado por abstinencia previa. La alucinosis suele desencadenarse por un aumento del consumo. 2.- En
el delirium tremens hay alteracin de conciencia y la alucinosis aparece con conciencia clara. 3.- En el delirium tremens las alucinaciones son
principalmente visuales y en la alucinosis hay alucinaciones auditivas. 4.- El correlato somtico agudo del delirium tremens no se encuentra presente en
la alucinosis. La alucinosis tambin es conocida como "delirio alucinsico de los bebedores", de Kraepelin o "alucinosis aguda de los bebedores" de
Wernicke. 4.- EL ESTADO EPILEPTOIDE ANGUSTIOSO: Para Roa; "el ncleo de la exacerbacin lo forma casi siempre un sentimiento indefinible de miedo,
desamparo o soledad, con oscurecimiento de la situacin y viva peticin mmica de ayuda. El enfermo demanda auxilio con las manos, se reclina sobre
alguien, abre desmesuradamente los ojos, emite algunas interjecciones ansiosas, palidece o enrojece y se recupera a los pocos minutos. En lugar de
peticin anhelante de ayuda puede haber violencia y agresividad. La exacerbacin, que puede terminar en una crisis convulsiva de acento epilptico,
puede repetirse muchas veces, sobre todo en presencia de terceros. El estado bsico entre las exacerbaciones es de euforia banal, con precipitacin en
las respuestas, sentimiento ntimo de depresin, falta de concentracin y quizs cierta pesadez corporal. Este estado dura das, semanas o meses. A
veces precede a los primeros ataques de la epilepsia genuina, acompaa los tumores enceflicos o matiza el curso de la histeria y la esquizofrenia". 5.- EL
ESTADO ESTUPOROSO: El estado estuporoso con compromiso de conciencia es una de las formas de manifestacin de un estado delirioso o crepuscular,
en su inicio o durante el transcurso. "los estados estuporosos se dan con claridad u oscuridad de conciencia. El primer caso es muy propio de la
esquizofrenia catatnica". Los enfermos que quedan de pie, sentados o acostados, ya en posiciones fijas, ya acomodndose ligeramente de vez en
cuando y sin obedecer rdenes. Algunos caen en gatismo urinario y fecal, otros en retencin y unos terceros evacuan si son llevados al retrete. No rara
vez el estado estuporoso es el comienzo de un estado delirioso, crepuescular, amencial o uno de los momentos del curso de dichos estados. 6.- EL
DELIRIO AGUDO O DELIRIUM: ya explicado en tema anterior. SUICIDIO. CIENCIAS BASICAS: Diversos estudios dan cuenta del incremento de suicidios
relacionados con la vejez en muchos pases, sobre todo los industrializados. Son factores de riesgo la ancianidad, el sexo masculino, la raza blanca, una
historia familiar de suicidios, los conflictos familiares y de vidas severas y recientes, los trastornos de personalidad y la enfermedad mental. SALUD
PUBLICA: En relacin con la gente joven se cita que la proporcin de suicidas entre ancianos es 50% mayor. Adems, las autopsias psicolgicas de las
vctimas revelan que ms de 90% presentaba un trastorno psiquitrico en el momento de su muerte. ETIOLOGIA: Las causas habituales para suicidio son:
los trastornos afectivos; de ellos, el ms frecuente es la depresin, con una prevalencia aproximada de 50-60%; le siguen los trastornos por abuso de
sustancias 19-60% y la esquizofrenia 2-13%. Tambin se cita el trastorno de personalidad como factor de riesgo para suicidio. Con respecto a este ltimo
los rasgos caracterolgicos conectados con la conducta suicida son rigidez, autosuficiencia, dificultad para aceptar la ayuda de los dems y narcisismo. De
los problemas existenciales, relacionados con el suicidio, se describen diferencias entre los jvenes y los viejos. En los primeros los principales factores
desencadenantes de estrs, previos al suicidio, estn relacionados con el trabajo, las finanzas y las dificultades en las relaciones interpersonales.
Mientras tanto, entre los adultos mayores el suicidio se comete en el contexto de enfermedades mdicas, generalmente crnicas o invalidantes y
prdidas. El suicidio en esta etapa de la vida puede ser silente, expresado mediante conductas pre suicidas como dejar de comer o medicarse. Sin
embargo, es comn el acto impremeditado e impulsivo, en ocasiones precipitado por acontecimientos menores que no alertan a la familia. Es usual que
el suicida elija mtodos drsticos tales como, el ahorcamiento o las armas de fuego. Tambin hay quienes antes del suicidio, mientras estn deprimidos,
cometen homicidios: para estos casos se cita una incidencia de 0.4-0.9 por 100, 000. Lo referido destaca la importancia del reconocimiento precoz y el
tratamiento oportuno del trastorno afectivo, adems de la adopcin de medidas preventivas. Por ltimo, es necesario subrayar que cerca de un tercio o
la mitad de los suicidas hicieron contacto con psiquatras antes del acto, y que aproximadamente tres cuartas partes de ellos lo hicieron con un mdico
general en el mes previo al acto, u horas antes de suicidarse. La ideacin suicida supone entre el 5-20% de las consultas psiquiatricas urgentes
(media=10%). 1% de admisiones a urgencias en hospitales generales. El 2% fallecen tras la tentativa. El suicidio supone 8,5 por mil fallecidos. Mtodos:
Suicidio consumado: ahorcamiento, precipitacin, ahogamiento. Tentativas: 90% son sobredosis farmacolgicas. Mas intentos de suicidio en mujeres
(9/1), ms suicidios consumados en hombres (3/1). DIAGNOSTICO: Evaluacion de riesgo suicida de (SAD person): El paciente tiene ideacin suicida?: SI
valorar (cada item positivo de los siguientes vale 1 punto): Sexo: varn. Edad: < 19 > 45 aos. Depresin clnica. Intentos de suicidio previos. Abuso de
alcohol. Trastornos cognitivos. Bajo soporte social. Plan organizado de suicidio. Sin pareja estable. Enfermedad somtica. TRATAMIENTO: Modalidad de
intervencin: 0-2: Alta al domicilio con seguimiento ambulatorio. 3-4: Seguimiento ambulatorio intensivo. Considrese el ingreso. 5-6: Hospitalizacin si
no hay control familiar adecuado. 7-10: Hospitalizacin. Elevado riesgo de suicidio. Abordaje verbal del paciente suicida General: Preocuparse por las
necesidades del paciente y transmitir que se intenta ayudarle. Identificarse y explicar los objetivos. Mostrarse sereno, firme, seguro y honesto. Evitar
conductas hostiles, desafiantes, amenazantes o irrespetuosas. Especfico (hablar de la problemtica suicida). Cuidar la contratransferencia (evitar o
controlar reacciones de angustia, preocupacin, hostilidad). Insistir en la dimensin temporal (la tendencia suicida como crisis delimitada en el tiempo).
Resaltar la ambivalencia de toda conducta (vivir vs.o morir como cuestin normal). Intervencin farmacolgica a corto plazo en el paciente suicida.
Tratamiento especfico a corto plazo: Ansiedad: Benzodiacepinas de vida media media/larga (diazepam). Insomnio: Benzodiacepinas vida media corta
(Lorazepam). No utilizar frmacos potencialmente letales. No utilizar antidepresivos en urgencias (incrementan el riesgo de suicidio)
CASO CLINICO
Ingresa a urgencias femenino de 27 aos la cual se encuentra bajo
tratamiento por trastorno de personalidad limite, ha sido tratada por
mltiples psiquiatras y esquemas teraputicos, al ingresar el esposo de
la paciente la encontr en la baera con cortes superficiales en ambas
muecas y falta de respuesta a los estmulos sin embargo respira
superficialmente, en la sala de urgencias usted identifica respiracin
lenta y superficial sin percibir olores especficos, observa ROTs
disminuidos, con respuesta ha estmulos dolorosos, su tratamiento
actual es clonacepam, clozapina y citalopram.

CURSO ENARM CMN SIGLO XXI TEL: 36246001

PREGUNTA
Considerando el manejo farmacolgico cual sera la mejor intervencin
para revertir el efecto de una dosis excesiva ingerida en este intento
suicida.
RESPUESTA
a.- La administracin de naloxona est indicada por polifarmacia.
b.- Un agonista antagonista parcial competitivo como flumacenil.
c.- Naltrexona es un frmaco que revierte la dosificacin de
benzodiacepinas.
d.- La paciente debe recibir biperiden para revertir los efectos de
clozapina

Pharmed Solutions Institute

PGINA 216

MANUAL DE TRABAJO DEL CURSO ENARM CMN SIGLO XXI

GINECOLOGIA Y OBSTETRICIA
1) ANTICONCEPCION, INFERTILIDAD Y ESTERILIDAD.
2) AMENORREAS Y ENDOMETRIOSIS.
3) CERVICOVAGINITIS Y ENFERMEDAD PELVICA INFLAMATORIA.
4) ENFERMEDADES DE TRANSMISION SEXUAL.
5) INFECCION POR VIRUS DEL PAPILOMA HUMANO, DISPLASIAS.
6) CANCER CERVICOUTERINO Y OVARICO.
7) SANGRADO UTERINO ANORMAL Y DISFUNCIONAL.
8) SINDROME DE OVARIO POLIQUISTICO.
9) MASTOPATIA FIBROQUISTICA Y CANCER DE MAMA.
10) MENOPAUSIA Y CLIMATERIO
OBSTETRICIA
1) CONTROL PRENATAL Y DEL EMBARAZO.
2) EMBARAZO ECTOPICO. AMENAZA DE ABORTO, ABORTO, INCOMPETENCIA CERVICAL.
3) HIPEREMESIS GRAVIDICA, ENFERMEDAD TROFOBLASTICA.
4) HIPERTENSION ARTERIAL Y ENFERMEDAD HIPERTENSIVA DEL EMBARAZO.
5) PREECLAMPSIA Y ECLAMPSIA.
6) DIABETES MELLITUS, INTOLERANCIA A LA GLUCOSA, DIABETES GESTACIONAL.
7) POLIDRAMNIOS, OLIGODRAMNIOS, PLACENTA PREVIA, DESPRENDIMIENTO DE PLACENTA.
8) ANEMIAS, IVU, PARTO PRETERMINO, PARTO NORMAL Y CESAREA.
9) COMPLICACIONES DEL PUERPERIO, HEMORRAGIA OBSTETRICA, PLACENTA ACRECTA.
10) SINDROME DE TENSION PREMENSTRUAL, PSICOSIS POSPARTO Y DEPRESION POSTPARTO.

CURSO ENARM CMN SIGLO XXI TEL: 36246001

Pharmed Solutions Institute

PGINA 217

MANUAL DE TRABAJO DEL CURSO ENARM CMN SIGLO XXI


PLANIFICACION FAMILIAR, ANTICONCEPCION Y CONTRACEPCION:
CIENCIAS BASICAS: Planificacin familiar: es el derecho de toda persona a decidir de manera libre y responsable e informada sobre el nmero y
espaciamiento de sus hijos y a obtener la informacin especializada y los servicios idneos. Mtodos anticonceptivos: procedimientos que se utilizan
para impedir el embarazo en forma temporal o permanente y se clasifican de la siguiente manera; Temporales hormonales (orales, inyectables, parches)
y no hormonales (barrera, naturales, DIU). Definitivos vasectoma, salpingoclasia. CRITERIOS para iniciacin y seguimiento de todos los mtodos
anticonceptivos; Conocer la eficacia del mtodo, ventajas y desventajas, efectos colaterales y complicaciones, prevencin de ETS, seguimiento.
Evaluacion de un mtodo de planificacin familiar a travs del ndice de Pearl, que es Tasa de embarazo=N de embarazos x 1200/meses-mujer de uso.
Se considera efectivo cuando es menor a 10
METODO
Orales
combinados

DESCRIPCION
Estrgeno (etinilestradiol o mestranol)
y progestgeno (norentindrona o
acetato de norentindrona o
noretinodrel o norgestrel o
levonorgestrel)

Orales
minipastillas

Contiene nicamente progesterona


(levonorgestrel), minidosis 350mcg

Anticoncepcin
emergencia

Levonorgestrel 1,5 mg. Se toman para


prevenir embarazo, hasta 5 das
despus de la relacin sexual sin
proteccin
Cilindros (silastic, etonorgestrel)o
capsulas pequeas y flexibles que se
colocan debajo de la piel del brazo,
contienen nicamente progestgeno
Sistema trasndrmico (norelgestrinoma
y etinilestradiol)
Se cabia cada semana x 3 sem, la sem 4
no se utiliza

Implantes
Subdrmicos y

Parches

Inyectable de
progestgeno

Inyectable
mensual

Dispositivo
intrauterino
(DIU)
Condn
masculino

Condn
femenino

Diafragma
(capuchn de
ltex blando)

Espermaticida

Coito
interrumpido

METODOS ANTICONCEPTIVOS
COMO FUNCIONA
EFICACIA
Evita la liberacin de vulos
>99% (uso
por los ovarios (ovulacin)
correcto y
sostenido)
92% como se usa
comnmente

Hace ms espeso el moco


del cuello uterino, lo que
impide entrada de
espermatozoides y previene
la ovulacin. Modifican
endometrio para disminuir
probabilidades de
implantacin
Evita la ovulacin
1ra dosis dentro de las 72
hrs poscoito y 2da dosis, 12
hrs despus de la primera
Mismo mecanismo de las
pastillas de progestgeno

>99% (uso
correcto y
sostenido)
92% como se usa
comnmente

OBSERVACIONES
Disminuye el riesgo de cncer endometrial y
ovrico: no tomar en la lactancia. Reduccin de la
frecuencia de spotting. Disminucin del riesgo de
EPI (cambia la consistencia del moco y protege).
No usar en Ca de mama, endometrio, CaCu,
hepatopatas, varices extensas, mujeres de 3540aos
Puede usarse durante la lactancia: tomar todos los
das a la misma hora
Pueden inducir amenorrea a lo largo del tiempo

Reduce de un 60%
as un 90% el
embarazo

No altera el embarazo si este ya se ha producido


No protegen de ETS. Causan ms efectos
colaterales

>99%

Debe ser insertado y extrado por personal


sanitario, se puede usar por 3-5 aos, segn el
tipo. Puede generar spotting

>99%

Menor perdida sangunea, ciclos mentsruales


predecibles, disminuye acn, hirsutismo,
disminuye riesgo de ca de ovario y de endometrio.
Contraindicado en flebitis, enf. Coronaria, EVC, ca
de mama, tumores heptico, embarazo
Al cesar el uso, la fecundidad tarda en reaparecer
(1-4meses); Puede haber spotting. Indicado en
nuligestas, nulparas o multparas, antes del 1er
embarazo aun cuando estn contraindicados los
estrgenos (VPH), purperas lactando (usar
despus de 6ta sem).
Por ser de depsitos tienen ms efectos adversos
Hemorragias vaginales comunes. Indicado en
nuligestas, nulparas y multparas, antes de 1er
embarazo, posaborto, postparto y postcesarea, si
no est lactando. Contraindicado en la lactancia en
primeros 6 meses posparto, embarazo o sospecha
de l, en ca mamario o de crvix, enf.
Tromboembolica, hepatopatas
Disminuye la frecuencia del clico menstrual y los
sntomas de endometriosis amenorrea, en un
grupo de usuarias

IM profunda cada 2-3 meses. Se inicia


en los primeros 7 das del ciclo.
Enantato de noretisterona
200mg/1ml=c/2meses. Acetato de
medroxiprogesterona=c/3
meses150mgs/3ml
IM profunda cada mes, se inicia en
primeros 5 das del ciclo, contiene
estrgeno y progestgeno (Cipronato
de estradial +Acetato de
Medroxiprogesterona. Valerianato de
estradiol+ Enantato de norestisterona)

Mismo mecanismo de
pastillas de progestgeno
solo

>99%

Mismo mecanismo de
anticonceptivos orales en
combinacin. Su proteccin
anticonceptiva se extiende
hasta 33 das

>99% (uso
correcto y
sostenido)
97% como se usa
comnmente

Dispositivo de plstico flexible y


pequeo que contienen una asa o
cubierta de cobre y se inserta en el
tero
Vaina o cubierta que envuelve el pene
erecto

El cobre daa los


espermatozoides e impiden
que se junten con el ovulo

>99%

Forma una barrera que


impide el encuentro de los
espermatozoides con el
ovulo
Forma una barrera que
impide que los
espermatozoides y el ovulo
se junten
Forma una barrera que
impide que los
espermatozoides y el ovulo
se junten

98% usado de
manera correcta y
sostenida
85% como se usa
90%

Tambin protege de las ETS, en particular la


causada por VIH

90%

Funcionan rompiendo la
membrana de las clulas del
esperma, matndolas o
enlenteciendo su
movimiento.
Evita que los
espermatozoides penetren

70-80%

El riesgo de embarazo aumenta, si no se usan


espermaticidas juntos. No interfiere la lactancia.
Puede introducirse hasta 6 hrs antes. Puede
brindar cierta proteccin contra algunas ITS.
Extremadamente raros: sndrome de shock txico
Uso correcto en cada acto sexual. Pueden
utilizarse como mtodo principal o como mtodo
de respaldo. Pueden irritar el pene o la vagina,
alergia local. Favorecen IVU. No usar pacientes con
SIDA o VIH.
Es uno de los mtodos menos eficaces, porque a
menudo resulta difcil determinar el momento,

Vaina o forro que se adapta


holgadamente a la vagina; esta hecho
de un material plstico transparente,
fino y suave
Cubre el crvix. Tambin hay
diafragmas de plstico y silicona
disponibles. El borde contiene un
resorte firme y flexible que mantiene el
diafragma en su lugar
Nonoxinol-9.Detergentes no inicos.
Los espermicidas se introducen
profundamente en la vagina poco
antes de tener sexo.
El hombre retira el pene fuera de la
vagina y eyacula fuera de esta, con lo

CURSO ENARM CMN SIGLO XXI TEL: 36246001

96% uso de
manera correcta

Tambin protege de las ITS, en particular la


causada por VIH

Pharmed Solutions Institute

PGINA 218

MANUAL DE TRABAJO DEL CURSO ENARM CMN SIGLO XXI


Mtodos que
requieren
conocimiento
de la
fecundidad y/o
abstinencia
peridica
Amenorrea en
la lactancia

Vasectoma

Salpingoclasia

que el semen no tienen contacto con


los genitales externos de la mujer
Mtodo de calendario (Ogino-Knauss):
temperatura corporal basal (TCB),
mtodo del moco cervical (Billings),
sintotrmico (TCB+moco cervical)

en la vagina, con lo que se


evita la fecundacin
La pareja evita tener
embarazo, evitando tener
coito, sin proteccin
durante los das fecundos
generalmente, mediante la
abstinencia

73% como se usa

preciso de la retirada

70-80% uso de
manera correcta
63% como se usa
comnmente

El uso correcto y regular exige la colaboracin del


varn. Cuando hay infeccin vaginal, es difcil
interpretar el moco. Por lo menos hay que
observar 6 ciclos menstruales, mientras
abstinencia o mtodo de barrera. De las 2 fases
del ciclo menstrual, solo la 2da es la constante

Proteccin prolactina. Para mujeres


recin paridas que no han vuelto a
menstruar, exige el amamantamiento
exclusivo da y noche, de una criatura
menor de 6 meses
Se bloquean o cortan los conductos
deferentes que transportan los
espermatozoides desde los testculos

Impide que los ovarios


liberan vulos (ovulacin)

99% si se aplica de
manera correcta
98% como se usa
comnmente

Esta basado en el efecto natural del


amamantamiento sobre la fecundidad

Impide que haya


espermatozoides en el
semen eyaculado

Tarda en actuar unos 3 meses debido a que


quedan espermatozoides almacenados; no afecta
el funcionamiento sexual del hombre

Se bloquean o cortan las trompas de


Falopio

Los vulos no pueden


juntarse con los
espermatozoides

>99%. Despus de
la evaluacin del
semen a los 3
meses
>99%

CASO CLINICO
Se trata de paciente femenino de 23 aos de edad la cual acude a
consulta para planificacin familiar, solicita un mtodo anticonceptivo,
refiere estar casada desde hace 3 meses y desea no embarazarse por
lo menos en 3 aos ms, niega antecedentes heredofamiliares como
hipertensin, diabetes mellitus, as como otras relevante, al
interrogatorio niega tabaquismo y alcoholismo, no toma
medicamentos, a la exploracin fsica no se agregan datos de
relevancia, IMC 24, TA 100/65 mmHg, FC 78, FR 19. Aparentemente la
paciente se encuentra sana.
PREGUNTA
La paciente solicita un mtodo anticonceptivo oral, cul sera el
mtodo de contracepcin ms apropiado para esta paciente desde el
enfoque de riesgo, as como en funcin del tiempo de espera que
quiere para embarazarse.
RESPUESTA
a.- No hay contraindicacin para el mtodo anticonceptivo oral por
carencia de factores de riesgo.
b.- No hay contraindicacin para el mtodo anticonceptivo oral ya que
es seguro y efectivo por el tiempo que desea usarlo.
c.- Igualmente podra usarse anticonceptivo inyectable mensual o
trimestral.
d.- Considerando su edad y inicio de vida sexual activa, un mtodo de
barrera sera la mejor opcin.
PREGUNTA
Despues de estar usando el metodo anticonceptivo actual, por 1 ao,
la paciente refiere amenorrea de 4 meses, prueba de embarazo
negativa. Cul seria la conducta mas adecuada en este momento?
RESPUESTA
a.- Parches
b.- DIU
c.- Implante
d.- Metodo de barrea
CASO CLINICO
Paciente femenino de 28 aos de edad la cual se encuentra en posaborto de 5 dias, refiere que ya no presenta sangrado, acude a solicitar
mtodo anticonceptico, refiere que no quiere tener mas hijos por lo
menos en 3 aos, tiene 2 hijos, anteriormente empleaba hormonales
orales y durante este mtodo se embarazo, no cuenta con
antecedentes de cervicovaginitis, ni EPI.
PREGUNTA
Cual de los siguientes mtodos de planificacin familiar es el mas
adecuado para este caso?
RESPUESTA

CURSO ENARM CMN SIGLO XXI TEL: 36246001

Es fundamental que sea una eleccin voluntaria y


con conocimiento de causa

a.- Hormonal inyectado.


b.- Implante hormonal.
c.- Presenvativo.
d.- Dispositivo intrauterino.
PREGUNTA
La paciente acude ha consulta por cambios del flujo menstrual, el cual
refiere es obscuro y ha presentado manchado intermenstrual, a la
exploracin se obseva moco cervical con sangre moderada, cual es la
conducta a seguir?
RESPUESTA
a.- Retirar el DIU.
b.- Indicar un AINES.
c.- Indicar antibitico.
d.- Indicar DOC.
PREGUNTA
La paciente acude 5 aos despus por flujo abundante con olor
desagradable, a la exploracin se observa un proceso compatible con
cervicovaginitis, agrega que hace un ao se divorcio por infidelidad por
parte del esposo y actualmente ya tiene otra pareja, cual es la
conducta a seguir ms adecuda.
RESPUESTA
a.- Retirar el DIU.
b.- Cambio de mtodo a hormonal.
c.- Indicar acido mefenamico.
d.- Indicar antibioticoterapia y preservativo.
PREGUNTA
La paciente acude 6 meses despus por dolor abdominal en fosa iliaca
izquierda que se irradia a hipocondrio y hacia tras, a la palpacin se
presenta dolor abdominal de predominio plvico, acude adems
porque su DOC presento cambios inflamatorios inespecficos, cual es la
conducta a seguir por en este caso?
RESPUESTA
a.- Retirar DIU por probable EPI.
b.- Retirar DIU por resultado de DOC.
c.- Retirar DIU por probable embarazo.
d.- Retirar DIU por cambio de mtodo.
CASO CLINICO
Se trata de paciente femenino de 22 aos de edad la cual acude a
solicitar mtodo de planeacin familiar, actualmente vive en casa con
sus padres, refiere un IVSA a los 20 aos, agrega que ha utilizado
preservativo pero lo considera con fallas, adems no desea que sus
familiares se enteren que tiene vida sexual activa, por otro lado
menciona que mantiene una relacin de dos aos de duracin y no
desea embarazarse, a la exploracin fsica se observa con leve

Pharmed Solutions Institute

PGINA 219

MANUAL DE TRABAJO DEL CURSO ENARM CMN SIGLO XXI


sobrepeso, con hirsutimo y acn presente, refiere que sus periodos
menstruales son irregulares.
PREGUNTA
Considerando la solicitud de la paciente y su juicio clnico. Cul sera
su conducta a seguir?
RESPUESTA
a.- Considera suficiente la informacin y hallazgos para indicarle el
mtodo solicitado.
b.- Considera que necesita mayor informacin por lo que enva a
realizar un USG.
c.- Considera que debe realizar un PIE por la irregularidad menstrual.
d.- Considera que debe esperar el prximo periodo para tomar la
decisin con la paciente.
PREGUNTA
De acuerdo al cuadro clnico Cul seria el mtodo anticonceptivo mas
adecuado si ya no hubiera forma de adquirir mas informacin?
RESPUESTA
a.- DIU
b.- Implante
c.- Hormonales orales
d.- Hormonales inyectanles
CASO CLINICO
Masculino de 31 aos de edad el cual acude para realizarse
vasectoma, refiere que tiene una paternidad satisfecha, a la
exploracin clnica se observa aparentemente sano, a la exploracin
genital se observa bolsa escrotal con aspecto normal, a la palpacin
hay leve dolor y se aprecia una masa irregular depresible, tortuosa,
refiere el paciente que desde los 25 aos se haba percatado, sin
embargo no le prest importancia solo ocasionalmente le duele,
principalmente cuando camina.
PREGUNTA
Considerando las manifestaciones clnicas, cual es la conducta a
seguir respecto al procedimiento que solicita el paciente?
RESPUESTAS
a.- No presenta contraindicacin de relevancia para poder realizar la
vasectoma.
b.- Debe descartarse un proceso neoplasico antes de realizar la
vasectoma.
c.- Debe realizar un USG para descartar un varicocele antes de realizar
la vasectoma.
d.- Debe cuantificar factores tumorales antes de realizar la vasectoma.
CASO CLINICO
Femenino de 21 aos de edad la cual se encuentra en procedimiento
cesarea transoperatorio, actualmente se realiza el procedimiento por
cesrea previa y periodo intergenesico de 10 meses, su primer
embarazo fue a los 16 aos el cual fue interrumpido mediante
legrado, actualmente es su tercer hijo y refiere maternidad satisfecha
al preguntarle, sin embargo refiere que es muy joven an, y podra
utilizar un dispositivo intrauterino.
PREGUNTA
Considerando el caso y tomando en cuenta la condicin quirrgica en
la que se encuentra la paciente. Cul sera la mejor opcin de esta?
RESPUESTA
a.- Solicitar autorizacin y colocar DIU ya que la paciente es joven aun.

CURSO ENARM CMN SIGLO XXI TEL: 36246001

b.- Solicitar autorizacin y realizar OTB ya que la paciente tiene riesgo


obsttrico.
c.- Le explica brevemente las ventajas y la dice que podra pensarlo.
d.- El GO contina el procedimiento y la podra realizarse OTB por
endoscopia posteriormente.
CASO CLINICO
Paciente femenina de 38 aos, con gestas 3, partos 2, cesreas 1, fecha
de ltimo parto hace 1 ao, es diabtica de 5 aos de evolucin,
fumadora de 5-8 cigarrillos al da, as como hipertensa de reciente
diagnstico, acude a su unidad de medicina familiar, para orientacin
sobre mtodos de planificacin familiar, desea control al menos por 3
aos.
PREGUNTA
De acuerdo a los antecedentes y factores de la paciente, que mtodo
anticonceptivo seria de primera eleccin en ella?
RESPUESTA
a.- Anticonceptivos orales
b.- Dispositivo intrauterino de cobre
c.- Dispositivo intrauterino Mirena
d.- Implante
PREGUNTA
La paciente regresa 6 meses despus refiriendo dolor tipo clico, as
como spotting Cul sera el mtodo anticonceptivo de segunda
eleccin para esta paciente que le ofrecera?
RESPUESTA
a.- Anticonceptivo oral
b.- Dispositivo intrauterino de cobre
c.- Dispositivo intrauterino Mirena
d.- Implante
CASO CLINICO
Se trata de paciente femenina de 36 aos de edad, en estudio por alta
sospecha de cncer, con antecedente familiar de cncer de mama en
la madre, que tiene 6 meses de posparto, actualmente en buen estado
general, con estudios de laboratorio normales. La paciente se refiere
actualmente asintomtica.
PREGUNTALA
La paciente solicita mtodo de planificacin familiar, cual es el ms
adecuado para este caso?
REPUESTA
a.- Dispositivo intrauterino de cobre
b.- Hormonales inyectables
c.- Mtodos de barrera
d.- Hormonales orales
PREGUNTA
Al mes acude la paciente a revisin de DIU, refiriendo dismenorrea
leve, as como flujo vaginal moderado, con olor suigeneris. Por lo que
se le realizan cultivos los cuales dan negativo para trichomona,
positivo para chlamydia trachomatis y negativo para gardnerella. Cul
es la conducta ms adecuada a seguir?
REPUESTA
a.- Antibioticoterapia
b.- Retiro de DIU y antibioticoterapia
c.- Antibioticoterapia y uso de preservativo
d.- Retiro de DIU y inicio de hormonal

Pharmed Solutions Institute

PGINA 220

MANUAL DE TRABAJO DEL CURSO ENARM CMN SIGLO XXI


INFERTILIDAD, ESTERILIDAD
CIENCIAS BASICAS: Esterilidad; incapacidad de una pareja para lograr la cocenceppcion. La esterilidad se clasifica en: primaria (cuando la pareja, tras un
ao de relaciones sin tomar medidas de proteccin, no ha conseguido un embarazo) y secundaria (la de la pareja que, tras la consecucin del primer hijo,
no logra una nueva gestacin tras 2 o ms aos de intentarlo). El tiempo mnimo a partir del cual se habla de esterilidad se fija en un ao de relaciones
sexuales con deseo de descendencia. Infertilidad; incapacidad reproductiva. As, se denomina infertilidad primaria la que padece una pareja que
consigue una gestacin que no llega a trmino con un recin nacido normal, e infertilidad secundaria cuando, tras un embarazo y parto normales, no se
consigue una nueva gestacin a trmino con recin nacido normal. Infecundidad; incapacidad de una pareja de llevar a termino un embarazo. Existe un
momento de mxima fecundidad en los primeros meses (alrededor de 20-30% de embarazos por ciclo) de forma que al cabo de tres ciclos, dos terceras
partes de las mujeres han logrado el embarazo. Globalmente, la tasa acumulada de embarazo entre parejas de fecundidad probada es del 90%
aproximadamente a los 12 meses y de alrededor del 95% a los 24 meses. SALUD PUBLICA: Entre un 12 y un 20% en las parejas hay imposibilidad de
tener un hijo. La edad de la mujer es importante: se estima que las posibilidades de que una mujer de 20-24 aos no se quede embarazada estn entre el
4-5%, a los 35 aos estaran alrededor del 14%, a los 40 aos la esterilidad en la mujer rondara cifras del 65-70%. Este incremento se ve agravado por el
hecho de que tambin aumentan las tasas de aborto, que a partir de esta edad son aproximadamente del 71%. La esterilidad e infertilidad suponen una
situacin carencial, esta carencia puede incidir negativamente en el desarrollo de la persona, produciendo frustracin y desmoralizacin. PATOGENIA:
Un 40% de los casos de esterilidad son de causa masculina: por alteraciones en el mbito testicular, obstruccin de conductos, patologas en la prstata,
alteraciones en la eyaculacin o ereccin y alteraciones en el semen. Otro 40% se debe a causas femeninas, como la menopausia precoz, la
endometriosis, las obstrucciones o lesiones de las trompas de Falopio, anomalas uterinas y cervicales o los problemas ovulatorios. El 20% restante
corresponde a causas mixtas o combinadas, en las cuales ambos cnyuges son responsables. Aborto de repeticin: Posiblemente pocos problemas
ginecolgicos resulten tan frustrantes, tanto para la pareja como para el mdico, se aplica en la actualidad a aquellas situaciones en las que se han
producido al menos dos abortos consecutivos o ms de dos alternos. FACTOR VAGINAL: las infecciones que se presentan pueden reprimir el coito (por
dispareunia), las infecciones causadas por Chamydia trachomatis, ureaplasma urealyticum, mycoplasma, traen problemas de esterilidad, ya que ejercen
accion nociva sobre los espermatozoides, ovulos y embriones, hay que hacer toma de cultivos, tambien buede haber defectos Mullerianos; vagina
tabicada, duplicacion de cervix, vaginismo. FACTOR CERVICAL: incopentencia istimico-cervical, los espermatozoides reciben una captacion por parte del
moco cervical, les facilita el ingreso a la cavidad uterina durante la etapa fertil, las criptas cervicales actuan como depositos de espermatozoides, de la
cuales pueden continuar el paso de los mismos en direccion a las salpinges, tomar citologia vaginal, exudado, frotis y cutivos, colposcopia, prueba de
dilatadores. FACTOR UTERINO; responsable del 5-10% de los casos de infertilidad; por presenia de polipos, miomas intracavitarios y algunas de las
malformaciones Muellerianas (bidelfo, bicorne). Aqu se proporciona un endometrio adecuado para la implantacion del ovulo fertilizado, efectua
multiples cambios adaptativos ante el embrion-feto en desarrollo. FACTOR TUBO-PERITONEAL: responsable de 15-20% de todos los casos de infertilidad
congenitas (agenesia, estenosis, atresias, duplicaciones y orificios accesorios en salpinges), adquiridas (infecciosa, endometriosis, procesos adherenciales,
tumorales), la laparoscopia es el elemento diagnostico terapeuico mas valioso. FACTOR ENDOCRINO-OVARICO: Responsable del 25-30% de los casos de
infertilidad, patrn menstrual regular y patrn menstrual irregular, ello se valora con medicion de progesterona, biopsia de endometrio, debe ser
tomada en la fase secretoria intermedia en los dias 21 a 23 del ciclo menstrual, realizar determinaciones de T3, T4 y TSH, en pacientes con ciclo
irregulares se debe realizar una determinacion de FSH, LH y E2 en los dias 35 del ciclo menstrual, la relacion FSH/LH es 2/1, si las cifras estan en >6m/ml
se hara diagnstico de falla ovarica, el estradiol debera encontrase entre 30-45pg/ml, debe ser determinado entre los dias 3 y 5 del ciclo. FACTOR
INMUNOLOGICO: es bajo solo representa de 3-5% de todos los casos. Las causas que podemos buscar en el varon son: infecciones, problemas
inmnologicos, tumores, alteraciones neurlgicas, ambientales, toxicas, idiopatica, disfuncion erectil, solicitar Ac anti-espermatozoides Ig-G e IgM en suero.
FACTOR INFECCIOSO: moco cervical contaminado, capacidad bacteriostatica del endometrio, diseminacion hematogena. Agudo: periodos en que el
endometrio es mas susceptible, dolor abdominal, fiebre, secrecion purulenta, anaerobios (B.fragilis, peptoestrptococos), aerobios (E. coli, S. aureus. N.
gonorreae, Paureginosa), virales (CMG, herpes virus), mycoplasma, clamidia, parasitos, toxoplasma, ricketsias. Cronicos:endometritis tuberculosa (puede
haber hipomenorrea, oligomenorrea, ataque al estado general, granulomas multiples no caseosos, adherencias gruessas, en la parte superficial den
endometrio, hacer biopsia dirigida), por clamidia (asintomatico, dao tubario). FACTOR MASCULINO: obstruccion de conductos (ETS), hipogonadismo
hipogonadotrofico, insuficiencia testicular primaria, idiopatica, criptoorquidea, endocrinopatia, diabetes mellitus (eyaculacion retrograda), fibrosis
quistica, hernias inguinales, trasplante renal y cirugia escrotal, medicamentos ( alfabloqueantes, fentolamina, metildopa, guanetedina, reserpina), orqitis,
prostatitis, varicocele, gonadotoxinas (calor, tabaco, radiacion, metales pesados, disolventes o pesticidas, cefalosporinas, TMP-SFX, quinolonas,
penicilinas, esteroides anabolizantes) DIAGNOSTICO DE LA PAREJA ESTRIL: Gnerales; BH, QS, EGO, urocultivo, rx. De torax, antigenos contra hepatitis.
Pruebas diagnsticas esenciales son: Evidencia de que la mujer ovula, para lo cual es suficiente comprobar que menstra cada 28 7 das. Anamnesis e
historia clnica para deteccin de enfermedades concomitantes (diabetes, hipertensin, alteraciones tiroideas, etc.) e historial mdico familiar sobre
patologas y enfermedades hereditarias. Evidencia de una reserva ovrica suficiente: se determina sistemticamente, de forma basal (tercer da del
ciclo), la FSH (hormona folculoestimulante) y el estradiol srico. Deeterminar prolactina, ya que la hierprolactinemia se encuentra entre 10-30% decasos
de infertilidad. Debe encontrase <20ng/ml. Evidencia de que el semen es normal, para lo cual se realiza un seminograma. Evidencia de que el canal
genital es normal, para lo cual basta inicialmente con una histerosalpingografa (HSG) y una ecografa transvaginal que explore tambin los ovarios, en
busca de quistes endometrisicos o endometriomas y miomas uterinos fundamentalmente. La HSG es una prueba molesta, por lo que siempre se realiza
tras evidenciar un espermiograma normal o suficiente para la realizacin de coitos programados o inseminaciones artificiales, muestra alteraciones
morfologicas de la cavidad y area corneal. La RNM tienen una precision insuperable en focos endometriosiscos. Alteraciones en el espermiograma: para
tomar un analisis de semen hay abstinencia de 2-3 dias, envase limpio por masturbacion, examinar a la hora siguientes, se valora la concentracin, la
motilidad y la morfologa de los espermatozoides. En ocasiones se objetiva una ausencia de espermatozoides en el eyaculado (azoospermia), por lo que
se deber recurrir a la biopsia testicular para completar el estudio y obtener espermatozoides testiculares o de epiddimo con los que poder trabajar a
posteriori. Oligoespermia densidad menor de 20millones/ml, intensa cuando es menor de 5 millones/ml, asociarse a varicocele, hipogonadismo,
microdelecciones del cromosoma Y. Atenospermia: motilidad escasa. Valores recomendados por OMS: volumen: 1,5-5,0ml, pH; >7.2, viscocidad; <3
(escala 0-4), espermatozoides; >20 millones/ml, N. total de espermatozoidez; >40 millones/eyaculado, porcentaje de motilidad >50%, progresion
progresiva; >2 (escala de 0-4), Morfologia normal; >50% normal, aglutinacion de espermatozoides <2 (escala de 0-3). Laparoscopia: para valorar factor
tubo-peritoenal, es el elemento mas valioso para el diagnostico y tratamiento mas valioso, nos ayuda a corroborar la permeabilidad de las salpinges y
permeabilizar las mismas. TRATAMIENTO: Si todas las exploraciones resultan normales se indica la realizacin de coitos programados con estimulacin
de la ovulacin (si el problema es una disfuncin ovulatoria, como ocurre en los ovarios poliqusticos) o de inseminaciones artificiales (cuando
supuestamente los ciclos de la mujer han sido regulares). La experiencia demuestra que la inseminacin conyugal proporciona tasas acumuladas de
embarazo > 60%, por lo se aconseja instaurar este tipo de terapia durante 4 ciclos. Si la mujer no gesta, el camino a seguir depender mucho de su edad,
pero en casi todos los casos se decide practicar fecundacin in vitro (FIV) para estudiar ambos gametos en el laboratorio y la interaccin de ambos
(fecundacin y desarrollo embrionario). En el 20% de los casos en los que las inseminaciones fallan y se realiza fecundacin in vitro, en un primer ciclo

CURSO ENARM CMN SIGLO XXI TEL: 36246001

Pharmed Solutions Institute

PGINA 221

MANUAL DE TRABAJO DEL CURSO ENARM CMN SIGLO XXI


tras fallo de inseminaciones, se realiza con la mitad de los vulos FIV y con la otra mitad ICSI o microinyeccin intracito plasmtica de espermatozoides
(1/2 FIV- 1/2 ICSI), lo que permite decidir sobre la tcnica ms adecuada en el futuro y al menos asegurar fecundacin en la mitad de los ovocitos, para
poder realizar transferencia embrionaria y rentabilizar el ciclo con posibilidad de gestacin. FRMACOS EMPLEADOS: La terapia farmacolgica
contempla las siguientes opciones: 1) Citrato de clomifeno: Se emplea por va oral. Su accin se basa en la unin del frmaco con los receptores
estrognicos de la hipfisis, que provoca el bloqueo de la retroalimentacin negativa del estradiol. Ello induce un incremento de la secrecin de
gonadotropinas por la hipfisis. Es un frmaco de fcil uso y barato, pero el efecto antiestrognico a nivel endometrial y del cuerpo lteo puede ser
responsable de las bajas tasas de implantacin que permite obtener. 2) Gonadotropinas humanas: Las primeras estimulaciones de la ovulacin se
realizaron con gonadotropina hipofisaria humana, pero tuvo que ser abandonada por la aparicin de la enfermedad de Creutzfeldt-Jakob. Aos ms
tarde se aislaron gonadotropinas en las orinas de las mujeres menopusicas (gonadotropina corinica humana o HMG) con escasa pureza y conteniendo
FSH y hormona luteinizante (LH). Desde entonces ha habido una intensa investigacin farmacutica para aislar FSH de alta pureza sin carga de LH y con
posibilidad de administracin subcutnea, y desde 1998 existe la FSH recombinante obtenida por ingeniera gentica. En breve saldr al mercado LH
recombinante. 3) Anlogos de la GnRH: Administrados en distintas fases del ciclo menstrual y combinados con la estimulacin de la ovulacin con
gonadotropinas, proporcionan una mayor sincrona en el crecimiento de los folculos, impiden el pico espontneo de LH que provocara una luteinizacin
prematura de los mismos (impiden la ovulacin espontnea) y disminuyen el nmero de cancelaciones de procedimientos. Con su uso se han mejorado
las tasas de fecundacin, de nmero de ovocitos obtenidos y ovocitos maduros obtenidos, as como de las tasas de gestacin. Su mecanismo de accin es
en dos tiempos: en un primer momento por su unin al receptor estimulan la produccin (efecto flare up), para que horas despus desensibilicen los
receptores por saturacin (hipofisectoma mdica reversible). 4) Inductores de la ovulacin HCG: La HCG u hormona del embarazo farmacolgica induce
la ovulacin 37 horas tras su administracin. Obtenida de la orina de mujeres embarazadas, su accin es equivalente a la de la LH hipofisaria. La LhRH
recombinante est en fase de ensayo clnico. 5) Antagonistas de la GnRH: Bloquean los receptores de la GnRH, suprimiendo la secrecin de
gonadotropinas por la hipfisis de forma inmediata y evitando el efecto flare up provocado por el uso de anlogos de la GnRH. En breve aparecern en el
mercado farmacutico espaol preparados de administracin subcutnea.
observa con clasificacin de Taner grado 3, refiere presencia de
CASO CLINICO INFERTILIDAD
leucorrea frecuente ha recibido tratamiento, a la exploracin observa
Femenino de 37 aos de edad la cual acude a consulta debido a que no
Talla 1.47 mts, Peso 48 kg. Adems cuello alado, torax en escudo.
ha logrado embarazarse, refiere que lleva 4 aos de intentndolo, con
vida sexual activa desde hace 5 aos con la misma pareja, no usa
PREGUNTA:
mtodo anticonceptivo, como antecedentes refiere menarca a los 19
Cul es la condicin ms frecuente de origen gentico que presenta
aos, gesta 0, para 0, abortos 0, a la exploracin fsica se observa con
con los datos observados en este caso?
caracteres sexuales secundarios adecuados, a la exploracin GO sin
RESPUESTA:
secrecin vaginal blanquecino sin olor desagradable, acude con
a.- Sindrome de Kallman.
resultados de Papanicolaou con datos inflamatorios crnico,
b.- Sindrome de Turner.
colposcopia negativo para VPH y perfil hormonal normal.
c.- Sindrome de Klinifelter.
c.- Sindrome XXX.
PREGUNTA:
Tomando en cuenta el perfil hormonal de la paciente que intervencin
CASO CLINICO
en primera instancia tomara usted:
Acude pareja (mujer de 28aos de edad, con obesidad y DM
RESPUESTA:
descontrolada y hombre de 38 aos) al servicio de ginecologa por
a.- Iniciar clomifeno 5 mg cada 24 hrs durante 14 das.
referir, deseo de embarazo. AGO: G:0, A:0. Ciclos menstruales de 28x5,
b.- Solicitar una espermatobioscopia.
IVSA 18 aos. Refieren 8 meses de convivencia, con relaciones sexuales
c.- Realiza USG de anexos para buscar causa obstructiva.
una vez por semana, sin mtodo de planificacin familiar.
d.- Indicar medroxiprogesterona 25mg y etinilestradiol 2mcg im.
PREGUNTA
CASO CLINICO INFERTILIDAD
De acuerdo al caso cual sera la conducta ms adecuada a seguir?
El resultado de laboratorio reporta 40 % de espermatozoides viables,
RESPUESTA
tomando en cuenta estos resultados, decide continuar el abordaje
a.- Iniciar protocolo de infertilidad
mediante estudio de ultrasonografia plvica, donde se reporta ovarios
b.- Iniciar clomifeno
de caractersticas adecuadas, sin embargo se reporta alteraciones de
c.- No cumple con criterios
oviductos, la paciente agrega que en los ltimos 2 aos ha presentado
d.- Mejorar factores de riesgo
cuadro repetidos de infecciones cervicovaginales por lo que indica un
PREGUNTA
cultivo vaginal, recibiendo resultado positivo para gardnerella y
Despus de un ao, regresan al servicio de ginecologa, con deseo de
cladmidia tracomatis resistente a doxiciclina.
embarazo, actualmente la paciente refiere galactorrea, flujo
transvaginal leve. Cul es la conducta diagnostica ms adecuada a
PREGUNTA:
solicitar?
La paciente es referida a biologa de la reproduccin, donde se indica
RESPUESTA
histerosalpingografia, la cual reporta obstruccin bilateral del 75 % de
a.- Valores de estradiol y hormona luteinizante en da 3-5 del ciclo
oviductos, tomando en cuenta los ltimos reportes de laboratorio y
b.- Valor de progesterona en el dia 21-23 del ciclo
gabinete, cul sera la complicacin ms importante para conseguir la
c.- Valor de folculo estimulante y luteinizante en das 21-23 del ciclo
concepcin en este caso?
d.- Valor de progesterona en el da 3-5 del ciclo
RESPUESTA:
a.- Nivel de espermatozoides viables disminuido.
PREGUNTA
b.- Secuelas de Enfermedad Plvica Inflamatoria.
Qu cultivos de base le solicitara a esta paciente dentro del
c.- Resistencia a los antibiticos de primera lnea.
protocolo de infertilidad?
d.- La condicin mixta como causa de la infertilidad.
RESPUESTA
a.- Chlamydia, mycoplasma, ureaplasma
CASO CLINICO ESTERILIDAD
b.- Chlamydia, trichomonas, gardnerella
Femenino de 31 aos de edad la acude a consulta debido a que
c.- Mycoplasma, candida, trichomona
presenta amenorrea secundaria, agrega que desea embarazarse, como
d.- Mycoplasma, trichomona, gardnerella
antecedentes cuenta con menarca a 21 aos con oligomenorrea, se

CURSO ENARM CMN SIGLO XXI TEL: 36246001

Pharmed Solutions Institute

PGINA 222

MANUAL DE TRABAJO DEL CURSO ENARM CMN SIGLO XXI


AMENORREAS
CIENCIAS BASICAS: Ausencia temporal o permanente de menstruacin. Factores asociados para una correcta menstruacin: secrecin pulsatil de GnRh
en hipotlamo, secrecin hipofisiaria de FSH y LH, ovarios que contengan folculos y capaces de secretar estrgenos y progesterona, correcto
funcionamiento de los mecanismos de retroalimentacin, aparato genital femenino correctamente desarrollado y sin obstrucciones para la
menstruacin. Correcto funcionamiento de los
mecanismos de retroalimentacin, aparato
genital femenino correctamente desarrollado y
sin obstruccin para la menstruacin.
AMENORREA PRIMARIA: se define como la
ausencia de menstruacin a los 14 aos de edad
cuando coincide con la falta de la aparicin de los
caracteres sexuales secundarios (infantilismo
sexual) y ausencia a los 16 aos en presencia de
desarrollo sexual. La amenorrea primaria
frecuentemente se acompaa de infantilismo
sexual y talla baja, lo que hace sospechar en
primer trmino la existencia de una disgenesia
gonadal, 1RA CAUSA de amenorreas primarias
(perdida de folculos ovricos durante la
embriognesis o en los primeros aos
prepuberales, ausencia de clulas germinales en
las gonadas alteraciones cuantitativas en
cromosomas sexuales, genitales internos
hipoplasicos, infertilidad); adems si existen malformaciones somticas se hace aparente un sndrome de Turner el mas frecuente (talla baja constante,
velocidad de crecimiento disminuida, pliegue en cuello, amplio espacio entre pezones, maduracin osea normal, infantilismo sexual; utero pequeo y
ausencia de gonadas) Sx. de Swyer o dsgenesia gonadal XY (no presenta pubertad, ni menarquia, ausencia de desarrollo de caracteres sexuales
secundarios, presencia de utero y fonadas rudimentarias con tendencia a transformacin tumoral). En otras ocasiones se puede encontrar el
antecedente en la infancia de un tumor hipofisario que fue sometido a ciruga y/o radiacin. En forma excepcional la amenorrea primaria se encuentra
asociada a una talla normal, desarrollo mamario, pero ausencia de vello sexual e infantilismo sexual con dispareunia; todo ello hace el diagnstico de un
sndrome de testculo feminizante Sx., de Morris (enfermedad recesiva ligada al cromosoma X, insensibilidad a andrgenos, cariotipo 46XY, produccin
de testosterona a nivel normal o superior al varon, inestabilidad a andrgenos por dficit de la formacin de receptores celulares), es la 3RA CAUSA de
las amenorreas primarias. Agenesia del conducto de
Muller o Sx. de Rokitansky-Kuster-Hauser;
SEGUNDA CAUSA mas frecuente de amenorrea
primaria, hay utero rudimentario no canalizado o
solo cuernos uterinos, cariotipo 46XX. Finalmente,
aunque raro, es que se presente en forma normal el
desarrollo de los caracteres sexuales secundarios,
pero no la menarquia y esto hace sospechar una
malformacin congnita del tero y la vagina o un
himen imperforado (hematocolpos, hematmetra,
hematosalpinge, criptomenorrea). La confirmacin
diagnstica
utiliza
esencialmente
estudios
citogenticos y de imagenologa, pasando a
segundo trmino los hormonales. Se puede
establecer este diagnstico de acuerdo al algoritmo
de la figura 1. En los casos en que se presenta
amenorrea y adems ausencia de caracteres
sexuales secundarios, el diagnstico se establece en
base a los resultados de laboratorio y el estudio del
cariotipo. La pubertad retardada es una entidad
familiar
caracterizada
por
hipogonadismohipogonadotrpico, por lo tanto los antecedentes
familiares detallados ayudan a establecer esta
etiologa, ya que es difcil de distinguir entre la falla
hipotalmicao hipofisaria. Otra entidad conocida
como sndrome de Kallmann, la cual se asocia a anosmia, tambin puede causar hipogonadismo-hipogonadotrpico. Los casos de hipogonadismohipergonadotrpico y amenorrea primaria estn causados por disgenesia gonadal o falla ovrica prematura. Sindrome de silla turca vacia; alteracin
congnita que da como consecuencia que la hipfisis quede aplanada y separada del hipotlamo. AMENORREA SECUNDARIA: es la desaparicin de la
menstruacin, por al menos 3 ciclos, en una mujer que haya tenido varios ciclos previos. La duracin de la amenorrea secundaria es muy variable y
puede ser permanente. La primera posibilidad de amenorrea secundaria es el embarazo como norma clnica; una vez descartado lo anterior, las causas
ms frecuentes son el sndrome de ovarios poliqusticos Sx de Stein-Leventhal: tipo I; muy raro, amenorrea secundaria de instalacin progresiva,
despus de largos periodos de oligoamenorrea, asociado con esterilidad primaria y tendencia a la obesidad. Tipo II; mas frecuente; oligomenorrea,
sndrome premestrual e insuficiencia ltea, es la causa mas frecuente de amenorrea por anovulacin. Tumores de ovario productores de andrgenos
(androblastoma). Los trastornos hipotalmicos que son diagnsticos por exclusin (eliminando la hiperprolactinemia debida coincidiendo o no con un
tumor hipofisario, as como un trastorno tiroideo). Una prueba que mantiene su vigencia y utilidad es la administracin de progesterona para inducir un
sangrado menstrual y cuando esto ocurre se confirma la presencia de un nivel aceptable de estrgenos circulantes; en cambio cuando no hay
menstruacin la continuacin diagnstica es el ciclo completo de estrgenos y progesterona, si consigue la menstruacin entonces se puede sospechar

CURSO ENARM CMN SIGLO XXI TEL: 36246001

Pharmed Solutions Institute

PGINA 223

MANUAL DE TRABAJO DEL CURSO ENARM CMN SIGLO XXI


un defecto severo de produccin endgena de estrgenos. Si un ciclo combinado de estrgenos y progesterona no consigue inducir la menstruacin se
debe sospechar una ausencia virtual del tero por sinequia uterina ocasionada por un legrado (Sx. Asherman, destruccin de endometrio), traumatico o
una infeccin postparto o ms raro por tuberculosis genital, radioterapia, esquistosomiasis uterina. Panhipopituitarismo: insuficiencia hipofisiaria global
consecuencia de isquemia o infarto, sndrome de Sheehan despus de hemorragia postparto, o por ciruga o radioterapia. En la actualidad aparece ms
frecuentemente la menopausia precoz ya sea quirrgica o idioptica que se puede diagnosticar con la prueba de estrgno-progesterona y si es necesario
con las determinaciones de FSH y estradiol circulantes. Los exmenes de laboratorio de mayor utilidad como una segunda etapa a la clnica y orientados
por sta son la prolactina (PRL), las gonadotropinas (LH, FSH), TSH (hormona estimulante del tiroides) y de menor valor estradiol y andrgenos. Los
avances en la imagenologa son ahora de gran ayuda para conocer la hipfisis y los rganos intraplvicos (ovarios, tero, endometrio y anexos). Existen
rutas diagnsticas que han permitido la elaboracin de algoritmos tiles en la clnica. Se ha establecido un algoritmo para llegar a este diagnstico
despus de haber descartado el embarazo como potencial diagnstico, como se muestra en la figura 2. Hipotiroidismo: Cuando existe enfermedad
tiroidea se presentan cambios menstruales antes de presentarse la amenorrea, como en el caso de hipotiroidismo leve, donde se puede presentar
disminucin o aumento del sangrado menstrual,
pero despus del tratamiento del hipotiroidismo se
restablece el ciclo menstrual aunque despus de
varios meses. Hiperprolactinemia: En las pacientes
con elevadas concentraciones de PRL (> 100 ng/mL),
galactorrea, cefalea y trastornos visuales se deben
realizar estudios de imagenologa para confirmar un
tumor de hipfisis, ya que el prolactinoma es la
principal causa de trastorno de la hipfisis anterior.
En caso de encontrarse un microadenoma (< 10 mm) se deben tratar con agonistas dopaminrgicos, adems de vigilar y tratar el problema de
infertilidad. Los macroadenomas tambin deben tratarse con agonistas dopaminrgicos o extirparlos quirrgicamente a travs de reseccin
transesfenoidal. Cuando se ha excluido el adenoma, se debe tener presente la posibilidad de hiperprolactinemia causada por medicamentos, aunque los
medicamentos no logran concentraciones elevadas como un adenoma, es decir, las concentraciones siempre estarn por debajo de 50 ng/mL, la
suspensin del medicamento mejorar este trastorno. Otras causas menos frecuentes la constituyen: la estenosis cervical, la fibrosis obstructiva o los
plipos. Falla ovrica prematura (hipoestronismo 17-estradiol <50pg/ml) y menopausia precoz: Est caracterizada por hipogonadismo
hipergonadotrpico, causado por deplecin de los folculos ovricos que lleva al hipoestrogenismo y finalmente amenorrea destruccin folicular
iatrogena por radioterapia, quimioterapia, virus. Esta entidad est asociada a trastornos autoinmunes, por esta razn los exmenes de laboratorio
resultan normales, por lo tanto debe tratarse la enfermedad de fondo (ej. enfermedad de Addison). Patologia iatrogena por uso de frmacos:
antidepresivos, neurolpticos, antimeticos, cimetidina, alfametildopa, anestsicos, estrgenos. La obesidad y la anorexia nerviosa tambin generan
amenorrea, amenorrea del deportista. En el diagnostico podemos tener: Gonadrotrofinas aumentadas= FSH>40 y LH >25Mui/ml= Trastorno reside en
ovarios. Gonadotrofinas disminuidas= FSH<5 y LH <5mUI/ml= Tratsorno del sistema hipotlamo-hipofisis. TRATAMIENTO: Pacientes que desean
embarazo y que la exploracin descubre falata persistente de la ovulacin: citrato de clomifeno y gonadotrofinas. Pacientes que no desean embarazo:
gestagenos, acetato de medroxiprogesterona a dosis de 10mg/dia, durante 10 dias en la segunda mitad del ciclo para evitar el riesgo de hiperplasia o
carcinoma de endometrio, anticonceptivos. Bromocriptina en casos de amenorrea asociados a galactorrea y niveles de prolactina elevada sin tumor
hipofisiario.
CASO CLINICO
Se trata de femenino de 24 aos de edad la cual acude a consulta
debido a que anteriormente se encontraba bajo tratamiento con
clomifeno ya que desea embarazarse sin embargo no sabe el
diagnostico establecido, tiene un USG donde se observa gnadas con
hipoplasicas adems de niveles hormonales gonadales por debajo de la
media.
PREGUNTA
Considerando los datos observados, anatmicos y fisiolgicos. Cul es
la indicacin ms adecuada para favorecer la ovulacin y de esta
manera mejorar la posibilidad de embarazo?
RESPUESTA
a.- Debemos primeramente realizar una prueba con bromocriptina.
b.- La groserelina nos favorecera una estimulacin gonadal.
c.- El citrato de clomifeno es el mas indicado para estimular
hipotalmicamente.
d.- Por protocolo deber iniciarse manejo con gonadotrofinas.
CASO CLINICO
Se trata de femenino de 24 aos acude por presentar alteraciones en
su ciclo menstrual, refiere periodos menstruales irregulares que le
duran uno o dos das y en ocasiones solo es manchado, a la
exploracin fsica se observa paciente con deficiencia en caracteres
sexuales femeninos que corresponden a menor edad aparente contra
la cronolgica.
PREGUNTA
Cul es el patrn hormonal ms probable que esperara encontrar?
RESPUESTA
a.- Aumento de FSH y LH con estrgeno y progesterona alta.

CURSO ENARM CMN SIGLO XXI TEL: 36246001

b.- Aumento de FSH y LH con estrgeno y progesterona baja.


c.- Andrgenos elevados y estrgenos y progesterona normal.
d.- Andrgenos normales con estrgeno y progesterona baja.
CASO CLINICO
Se trata de paciente femenino de 21 aos de edad la cual acude por
falta de menstruacin durante los ltimos 6 meses, niega vida sexual
activa, sus antecedentes GO son menarca 11 aos, G: 0, a la
exploracin se encuentra caracteres sexuales normales para edad y
sexo, se observa sobrepeso por IMC, niega flujo, refiere que en los 3
ltimos aos sus periodos menstruales han sido irregulares, niega
cefalea u otros sntomas.
PREGUNTA
Cul de los siguientes diagnsticos es el ms probable?
RESPUESTA
a.- Microadenoma PRL productor.
b.- Adenoma GH productor.
c.- Microadenoma ACTH productor.
d.- MEN.
CASO CLINICO
Paciente femenina de 26 aos, hipertensa, quien consulta por
amenorrea de 1 ao de evolucin asociado a dolor plvico
intermitente Ha tenido 3 gestas, dos partos, 1 aborto, 2 hijos vivos, y 1
hijo muerto. En abril del 2009 le realizan legrado intrauterino por un
alumbramiento incompleto. Luego en el mes de junio del mismo ao
consulta por hemorragia vaginal donde realizan ultrasonido plvico el
cual revela un aborto incompleto por lo que posteriormente le realizan
un aspirado manual endouterino (AMEU). En un control ecogrfico
posterior por ausencia de menstruacin de un ao de evolucin se le

Pharmed Solutions Institute

PGINA 224

MANUAL DE TRABAJO DEL CURSO ENARM CMN SIGLO XXI


realiza un ultrasonido plvico el cual evidencia reas hper-ecognicas
en la cavidad uterina y abundante lquido, tambin se observa
ausencia de endometrio y reas hiperecognicas que se extienden
desde la pared posterior a la cara anterior del tero.
PREGUNTA
Cul es el diagnstico ms probable de esta paciente es?
RESPUESTA
a.- Sndrome de Swyer
b.- Sndrome Sheehan
c.- Sndrome de Asherman
d.- Sndrome de Rokitansky-Kuster
PREGUNTA
Cul es el antecedente ms importante y comn para desarrollar este
sndrome?
RESPUESTA
a.- Hemorragia vaginal
b.- legrado
c.- amenorrea
d.- Hipertensin
PREGUNTA
2 aos despus la paciente contino con amenorrea, se agregaron
galactorrea, sensibilidad en las mamas, disminucin del inters sexual,
cefaleas. Considerando la complicacin, cual sera la conducta
diagnostica ms adecuada?
REPUESTA
a.- Hormona foliculoestimulante (FSH)
b.- Hormonas tiroideas
c.- Prolactina
d.- Hormona gonadotrofica CASO CLINICO
Mujer DE 34 aos de edad, G3, P3, se encuentra en los 20 dias
posparto luego de parto vaginal complicado por hemorragia de 1500ml
consecutiva a atona uterina. No hubo respuesta a la administracin de
oxitocina o de meytilergonovina, pero si un episodio de hipotensin
sostenida de 10 minutos de duracin que vari desde 62/32mmHg a
71/25mmHg y requiri 10 unidades de eritrocitos aglomerados. En el
segundo dia posparto se encontr una concentracin de Cr 1.8mg/dl
que alcanzo un mximo a 3.5mg/dl, al sptimo dia postparto y desde
entonces volvi a la basal. En la actualidad la paciente se siente bien
pero acude con su obstetra por incapacidad para producir leche.
PREGUNTA
Cul es el diagnostico mas probable en este caso?
RESPESTA
a.- Sindrome de Asherman
b.- Adenoma hipofisiario
c.- Prolactinoma
d.- Sindrome de Sheehan
PREGUNTA
Cul de los siguientes es menos probable encontrar en esta
patologa?
RESPUESTA
a.- Amenorrea
b.- Hipotiroidismo
c.- Hipogonadismo
d.- Hipertensin
PREGUNTA
Es posible que esta paciente se pueda volver a embarazar mas
adelante?
RESPUESTA
a.- No, por la baja produccin de hormonas y la amenorrea
b.- Si, solo hay que dar inductores de la ovulacion

CURSO ENARM CMN SIGLO XXI TEL: 36246001

c.- No, por disminucin de la hormona estimulante del tiroides


d.- Si, solo hay que administrar prolactina
CASO CLINICO
Una adolescente de 17 aos acude a la consulta externa de pediatra
por amenorrea primaria. A la exploracin fsica se reconoce un
fenotipo femenino con cuello alado, asimetra facial, flexin de
cabeza y cuello hacia la izquierda, discreta escoliosis y se aprecia un
desarrollo sexual externo completo con estadio de Tanner V, los
genitales externos de apariencia normal, pero se observ atresia
vaginal con fondo a 1 cm. Se le solicita perfil hormonal en el que no se
encontraron alteraciones. El ultrasonido hecho por el gineclogo
mostr una masa plvica que se diagnostica como hematmetra. Se
interroga a la paciente que refiere dolor abdominal peridico. En una
radiografa de la columna cervical y torcica se observ la presencia de
hemivrte bras y escoliosis y una audiometra tonal y con potencia les
evocados auditivos de tallo cerebral, se le encuentra hipoacusia
sensorial izquierda profunda.
PREGUNTA
Cul es el diagnostico mas probable para este caso?
RESPUESTA
a.- Sindrome de Swyer
b.- Sndrome de Mayer-Rokitansky-Kster-Hauser
c.- Sindrome de Turner
d.- Sindrome de Morris
PREGUNTA
Cul es la causa ms probable de ausencia de utero ovarios?
RESPUESTA
a.- Degeneracion de conductos Wolffianos
b.- Degeneracion de conductos Mullerianos
c.- Disgenesia gonadal
d.- Produccin de hormona antimulleriana
CASO CLINICO
Adolescente de 14 aos, que consulta por amenorrea primaria, con
antecedentes de desarrollo puberal normal, telarquia a los 11 aos y
pubarquia a los 10 aos, sin menarquia. Relata historia de dolor
plvico de aproximadamente un ao de evolucin, cclico, de
intensidad moderada a severa que se irradia a zona lumbar, sin otras
molestias. No refiere antecedentes mrbidos de importancia ni
cirugas previas. Al examen: paciente eutrfica, mucosas rosadas,
mamas Tanner V. Abdomen: blando, depresible, con tumor pelviano
de consistencia aumentada que alcanza hasta el ombligo. Vulva: vello
pbico Tanner IV, meato uretral normal, no se evidencia orificio
himeneal, mucosa himeneal abombada, protruyente, de aspecto
violceo.
PREGUNTA
Cul es el diagnostico mas probable en este caso?
RESPUESTA
a.- Sindrome de Kallman
b.- Himen imperforado
c.- Sindrome de ovario poliquistico
d.- Criptomenorrea
PREGUNTA
Cul es la conducta diagnostica mas adecuada a seguir?
RESPUESTA
a.- Ultrasonido
b.- Radiografia
c.- Perfil hormonal
d.- Tomografia

Pharmed Solutions Institute

PGINA 225

MANUAL DE TRABAJO DEL CURSO ENARM CMN SIGLO XXI


ENDOMETRIOSIS
CIENCIAS BASICAS: Es una de las enfermedades ms enigmticas que afecta a las mujeres en edad reproductiva, se dene por la presencia de tejido
endometrial (epitelio glandular y estroma) fuera de la cavidad uterina (ectpico), localizndose habitualmente sobre la supercie peritoneal y sobre el
ovario. Otras posibles localizaciones de esta enfermedad son: sistema gastrointestinal, pulmn, sistema genitourinario, pared abdominal. Puede
manifestarse por trastornos funcionales, relacionados con esterilidad, es un distrofia benigna, es hormonodependiente, la menopausia es una forma de
curacin fisiologica, y el tratamiento sustitutivo la reactiva. Las localizaciones mas frecuentes son: ligamentos utero-sacros, fondo de saco posterior,
fondo de saco vesicouterino, ovario (el mas frecuente), tubas uterinas, extragenialtes. SALUD PUBLICA: Se estima en 2% en la poblacin ene general.
Aunque la prevalencia se estima que entre el 20-90% de mujeres con dolor plvico y/o infertilidad presentan esta enfermedad. En mujeres
asintomticas, la prevalencia se sita entre 3-30%. Un 10% en el grupo de eda de los 30-40 aos de edad. Factores de riesgo: antecedentes familiares,
intervenciones quirrgicas ginecolgicas u obsttricas. Factores favorecedores; reflujo menstrual, DIU, entorno hormonal, factores inmunolgicos
(deposito de IgG y C3) y por ultimo la reaccin autoinmunitaria alimenta un estado de inflamacin crnica local endometrial responsable de la
infertilidad de la paciente. CLASIFICACION: Estadio I (Mnima, 1-5 puntos): Implantes superficiales, principalmente en el ovario. Adherencias delgadas.
EstadioII (Leve 5-15 puntos): Implantes superficiales en el revestimiento plvico y el ovario. Estadio III (Moderada, 16-40 puntos): Implantes profundos en
el revestimiento plvico y el ovario. Adherencias densas y firmes. Estadio IV (Severo, >40puntos): Implantes profundos en los ovarios y el revestimiento
plvico. Adherencias densas en los ovarios, trompas, e intestinos. Fondo de saco Douglas obliterado. Endometriomas. PATOGENIA: Teorias: 1)
Metaplasia celmica: el revestimiento epitelial de la cavidad celmica parecera tener un potencial de metaplasia en tejido endometrial por efecto de
irritaciones repetidas (infecciosa, hormonal o por reflujo menstrual). El epitelio celmico, que surge de una capa de clulas epiteliales del conducto
mulleriano, tambin se diferencia en epitelio pleural y peritoneal y en clulas de la supercie de los ovarios. Este hecho puede explicar las localizaciones
inusuales de la endometriosis. 2) Teora de los restos embrionarios: Esta teora fue acreditada por Von Rec linghausen, quien se bas en la identicacin
de restos del conducto Wolano en el ligamento ancho y porciones antero-laterales de la vagina y el crvix. reas adyacentes a los conductos
mullerianos y duplicaciones del sistema mulleriano, permitiran que clulas de origen mulleriano se conviertan en endometrio funcionante. La
endometriosis tiene su mayor incidencia en mujeres de a partir de los 25 aos. 3) Diseminacin linftica y vascular: Sampson fue el primero en sugerir la
diseminacin linftica y hematgena de la endometriosis al encontrar tejido endometrial en venas de pacientes con adenomiosis. Posteriormente Halban
conrm esta teora postulando la viabilidad de las clulas endometriales que entraban a travs de la membrana basal de los vasos sanguneos y
linfticos. Estas clulas endometriales mediante la formacin de micrombolos provocaran el desarrollo de la enfermedad en lugares a distancia. 4)
Teora de la implantacin: Segn los hallazgos de Sampson el contenido menstrual con fragmentos de endometrio pasa a travs de las trompas de
Falopio alcanzando la cavidad peritoneal en lo que conocemos como menstruacin retrgrada. Ese endometrio posteriormente se implanta en la
supercie peritoneal y en los rganos del abdomen y de la pelvis, donde en los prximos ciclos menstruales se produce el sangrado menstrual.
DIAGNOSTICO: Se hace aparente en los aos de la etapa reproductiva debido a que las hormonas producidas por el ovario estimulan al endometrio. Los
sntomas son intensos en el periodo premenstrual y moderados al trmino de la menstruacin, por lo tanto, el dolor plvico es el sntoma ms frecuente,
pero adems puede aparecer dolor de espalda, dispareunia, dolor a la defecacin, dolor al cambio de posicin y disuria. La mayora de las mujeres en
quienes se ha diagnosticado endometriosis ha tenido previamente sndrome de vejiga irritable. La asociacin de endometriosis con infertilidad es debido
a las adherencias en la cavidad peritoneal, lo cual distorsiona la anatoma plvica y
trastorna la liberacin del vulo; sin embargo, los trastornos anatmicos no son el
nico motivo de la infertilidad, ya que se ha descrito que la endometriosis causa baja
reserva ovrica y de la calidad del vulo. Hay signos que pueden estar ausentes, pero
deben estar incluidos ndulos en la parte posterior de la vagina, dolor al movimiento
uterino, tero en retroversin y fijo, y dolor de masa anexial por endometriomas. El
diagnstico de la endometriosis se basa en la visualizacin del endometrio ectpico por
laparoscopia (lesiones planas nodulares o qusticas tienen una coloracin azulada, rojas,
blanquecino, conenido achocolatado) o laparotoma, acompaado de confirmacin
histolgica de macrfagos con hemosiderina, epitelio endometrial y glndulas o
estroma endometrial. El ultrasonido intravaginal ha sido el de mayor uso en el
diagnstico de lesiones retroperitoneales y uterosacras, pero no tiene la exactitud para
identificar lesiones peritoneales o endometriomas pequeos. Medicion de CA-125,
aunque es poco especifico, positivo >30UI/ml, mas en estadios III y IV. TRATAMIENTO:
Quirrgico consiste en la ablacin del tejido endometrial ectpico a travs de
laparoscopia, aumenta la fertilidad en mujeres con endometriosis, aunque se ha reportado que la exresis de los quistes endometrisicos otorga un
mejor resultado en la disminucin del dolor comparado con la eletrofulguracin y drenaje del tejido endometrial ectpico. Por otro lado, la histerectoma
acompaada de salpingo-ooforectoma bilateral se considera el tratamiento definitivo para la endometriosis severa, aunque en estudios retrospectivos
se ha observado que 10 % de las mujeres con histerectoma y salpingectoma bilateral tuvieron recurrencia de los sntomas, mientras que las mujeres
que solo tenan histerectoma presentaron recurrencia de los sntomas en 62 % de los casos. La vertiente actual cuando hay infertilidad es tratar la
endometriosis aunque sta sea mnima, debido a que los cambios presentes por este tejido ectpico son a nivel inmunolgico y en los rganos genitales,
lo que trastorna todo el proceso de la concepcin. El tratamiento mdico, se busca un clima hipoestrogenico para atrofiar los implantes como el danazol
600mg diarios por 6 meses (inhibidor a nivel del eje hipotlamo.hipofisiario, suprimiendo la produccin de GnRH, FSH y LH, esteroides ovricos por
inhibicionde hormonas esteroidogenicas). Para el dolor incluye analgsicos (AINES), anticonceptivos orales, anlogos de la hormona liberadora de
gonadotropinas (GnRH = leuprolide, goserelina, triptorelina, nafarelina), se ha demostrado que el uso de GnRH para el tratamiento del dolor causado por
endometriosis no mejora el efecto que los anticonceptivos o los progestgenos, adems, tiene el inconveniente de causar hipoestrogenismo como efecto
secundario, que cesa con la suspensin del tratamiento, efectos secundarios bochornos, insomnio, disturbios emocionales, favorecen la aparicin de
osteoporosis. Recientemente se han utilizado los inhibidores de aromatasa con respuesta prometedora para tratar la endometriosis, con lo cual se ha
visto efecto sobre el dolor plvico y la produccin de endometrio ectpico, ya que bloquea la produccin en secuencia de ciclooxigenasa-2 (COX2),
produccin de prostaglandinas y produccin local de estradiol por el endometrio ectpico, el cual es estrgeno-dependiente.
CASO CLINICO ENDOMETRIOSIS
Femenino de 41 aos de edad. Entre sus antecedentes mencion el
diagnstico de tuberculosis intestinal tres aos antes, siendo tratada
durante 8 meses con tratamiento especfico tetra-asociado, periodos
menstruales regulares y un parto por cesrea a los 32 aos. Se

CURSO ENARM CMN SIGLO XXI TEL: 36246001

hospitalizo por cuadro de varios aos de evolucin, con frecuentes


episodios de dolor abdominal difuso, distensin abdominal, flatulencia,
nauseas, vmitos y constipacin, mayor durante ciclos menstruales. El
cuadro que se intensifico durante el ltimo ao, asocindose
espordicamente deposiciones con estras de sangre viva y perdida de

Pharmed Solutions Institute

PGINA 226

MANUAL DE TRABAJO DEL CURSO ENARM CMN SIGLO XXI


4 a 5 Kg de peso. Al ingreso, paciente en regulares condiciones
generales, presin arterial 120/80 mmHg, frecuencia cardiaca 88
latidos por minuto, temperatura 36,6 C, el abdomen se encontraba
distendido, timpanizado a la percusin, chapoteo en flanco y fosa iliaca
derecha y borborigmos generalizados a la auscultacin. Los
laboratorios, rayos X de trax, abdomen y ecografa abdominal de
ingreso fueron normales, exceptuando la sangre oculta en heces que
fue (+). Las revisiones ginecolgica y proctolgica no encontraron
alteraciones.
PREGUNTA
Cul es el estadio en el que se encuentra la paciente?
RESPUESTA
a.- Estadio I (Mnimo) - 1-5 puntos.
b.- Estadio II (Leve) - 6-15 puntos.
c.- Estadio III (Moderada) - 16-40 puntos.
d.- Estadio IV (Severa) - ms 40 puntos.
CASO CLINICO ENDOMETRIOSIS
Una paciente de 47 aos que consult por dolor intenso, de carcter
cclico, de aproximadamente 1 ao de evolucin, localizado en la FID.
Los dolores comenzaban 3 o 4 das antes de cada menstruacin y
precisaba de grandes dosis de analgsicos para su control. Como
antecedentes de inters refera una salpingectoma y una cua ovrica
izquierdas por va laparoscpica, probablemente por quistes
endometrisicos, as como la reseccin por histeroscopia de un
endometrioma uterino 3 aos antes. En las exploraciones
complementarias, que incluyeron resonancia nuclear magntica y
trnsito gastrointestinal baritado, nicamente destac un
endometrioma en ovario izquierdo de 17 mm de dimetro, que el
servicio de ginecologa desestim para ciruga. Ante el diagnstico de
dolor crnico recurrente en FID de origen desconocido y dados los
antecedentes mdico-quirrgicos de la paciente, se plante realizar
una laparoscopia exploradora, en la que se evidenci un quiste seroso
ovrico izquierdo y un apndice vermiforme normal, pero no se
apreciaron tumores ni lesiones intraabdominales, por lo que se opt
por llevar a cabo la apendicectoma profilctica.
PREGUNTA
Cuales son las evidencias diagnosticas de laboratorio y gabinete mas
frecuentes?
RESPUESTA
a.- Niveles plasmticos de CA 125.
b.- Glndulas endometriales.
c.- Estroma endometrial
d.- Macrfagos con inclusiones de hemosiderina
CASO CLINICO ENDOMETRIOSIS
Enferma de 34 aos de edad que consulta por clnica de sndrome
miccional y sensacin de disconfort hipogstrico de varios meses de
evolucin con urocultivos realizados en Atencin Primaria negativos.
En una ocasin tuvo un episodio de hematuria autolimitada y no tiene
antecedentes mdicos ni ginecolgicos de inters, salvo un parto
vaginal no instrumentado. En los estudios analticos practicados no se
observan hallazgos dignos salvo una discreta anemia y una elevacin
mnima de la V.S.G. La ecografa abdominal practicada se informa
como compatible con la normalidad, por lo que se decide la prctica de
una cistoscopia donde se aprecia una lesin qustica en pared posterior
no sugerente de neoformacin papilar tpica. En la R.T.U. practicada se
informa como endometriosis vesical
PREGUNTA
Cul es la conducta ms adecuada para este caso?
RESPUESTA

CURSO ENARM CMN SIGLO XXI TEL: 36246001

a.- Danazol a dosis de 600 a 800 MG.


b.- Anlogos de GnHR 3.5 MG IM mensual.
c.- Medroxiprogesterona oral.
d.- Anticonceptivos hormonales combinados.
CASO CLINICO
Paciente de 27 aos de edad, sin antecedentes patolgicos relevantes,
ni hbitos ecotxicos, control natal con DIU hace 2 aos, con dos
gestaciones previas, culminadas con cesreas, la ltima hace 5 aos.
Desde hace 3 aos nota un pequeo tumor a nivel del extremo
izquierdo de la cicatriz de Pfannenstiel, que aumenta discretamente de
tamao y se hace doloroso coincidentemente con cada ciclo
menstrual. A la palpacin se constata un tumor a nivel de la pared
abdominal anterior, sobre el recto anterior izquierdo de 4 cm de
dimetro poco mvil, doloroso, que no desaparece con la contraccin
de los msculos abdominales. Se realiza una ecografa que muestra
una imagen slida, isoecognica, mal delimitada, a nivel de la pared. El
laboratorio no se aparta de lo fisiolgicamente normal.
PREGUNTA
Cul sera la conducta teraputica mdica, no quirrgica mas
adecuada?
REPUESTA
a.- Hormonales combinados
b.- Anlogos de la GnRH (leuprolide, nafarelina)
c.- Danazol
d.- Progestinas
PREGUNTA
Con estos medicamentos se logra disminuir la sintomatologa, debido
a que actan produciendo?
RESPUESTA
a.- Hipoestronismo y amenorrea
b. Inhibicin de enzimas esteroidognicas
c.- Interfiere el desarrollo folicular
d.- efecto bloqueante ejercido sobre la produccin hipotalmica de la
hormona liberadora de gonadotrofina (GnRh)
PREGUNTA
La complicacin ms frecuente de la patologa de este caso es?
RESPUESTA
a.- Dismenorrea
b.- Dolor plvico
c.- Esterilidad
d.- Infertilidad
PREGUNTA
Una mujer multigravida de 29 aos de edad acude con su gineclogo
por tener dolor plvico, dispareunia, clicos y dolor de espalda
intensos durante la menstruacin. En el examen fsico la frecuencia
cardiaca es de 75/min y la presin arterial de 125/80mmHg. El examen
de la pelvis revela cuello uterino normal, con hipersensibilidad del
frnix posterior en el examen bimanual. No se palpan masas. Los
cultivos para N. gonorroeae y Chlamydia resultan negativos al igual
que una prueba para detectar embarazo.
PREGUNTA
Cul es la complicacin menos probable en este caso?
RESPUESTA
a.- Adenomiosis
b.- Infecundidad
c.- Obstruccion intestinal
d.- Peritonitis

Pharmed Solutions Institute

PGINA 227

MANUAL DE TRABAJO DEL CURSO ENARM CMN SIGLO XXI


VULVOVAGINITIS
CIENCIAS BASICAS: Son un grupo heterogneo de alteraciones de la parte baja del aprato genital femenino, caracterizado por sntomas inespecficos
(flujo vaginal, ardor, prurito vulvar). Vaginitis: Corresponde a la enfermedad inflamatoria del crvix y la vagina, causada con mayor frecuencia por
agentes microbiolgicos. Vaginitis recurrente: Presentacin de cuatro o ms episodios en un ao. Exudao vaginal fisiolgico: consiste en secreciones
cervicales y vaginales, clulas epiteliales y flora bacteriana (levaduras, gram + y -, bacterias facultativas y anaerobias). El pH vaginal es acido entre 3.8 y
4.4 debido a que contiene 0.3-0.5 de acido lctico, un exudado vaginal fisiolgico no suele tener olor, ni causar prurito, ardor o alguna molestia, la
cantidad es variable. La flora vaginal normal se compone predminantemente de la especie de Lactobacillus (bacilo de Doderlein), un baston grampositivo
aerobio protector del microambiente vaginal. Existen, no obstante mas de 136 cepas de lactobacilos, mantienen el pH normal mediante la produccin de
acido lctico, tambin producen perxido de hidrogeno. SALUD PUBLICA: La vaginosis o vaginitis bacteriana se ha encontrado en el 15-19% de las
pacientes ambulatorias. Durante el embarazo, las cervicovaginitis se asocian en 11.5 % con el parto pre trmino y en 11.6 % con ruptura prematura de
membranas y como factor predisponente para esterilidad de origen tubrico y cncer cervicouterino. PATOGENIA: Dentro de la etiologa de las
cervicovaginitis, 22.6 % de los casos son producidos por Gardnerella vaginalis, 19.1 % por Candida spp., 7.8 % por Candida albicans y 1.5 % por
tricomonas, estas en conjunto causan el 90% de los casos. Otras causas que forman el 10%, son: alrgicas (a la protena del semen), irritativa (ducha
vaginal, jabones, desodorantes, medicamentos tpicos). La mucosa cervical y vaginal responde a la infeccin con una reaccin inflamatoria que se
caracteriza por el dao a las clulas superficiales. Este dao conduce a la descamacin y ulceracin, que causan una reduccin del espesor epitelial
debido a la prdida de clulas de la capa superficial y de parte de la capa intermedia (que contienen glucgeno). En las capas ms profundas, las clulas
sufren tumefaccin con infiltracin de neutrfilos en el espacio intercelular. La superficie del epitelio se halla cubierta por desechos celulares y
secreciones inflamatorias mucopurulentas. Existe congestin del tejido conjuntivo subyacente con dilatacin de los vasos superficiales y papilas
estrmicas hipertrficas y dilatadas. VAGINOSIS BACTERIANA (VB): Es la causa ms frecuente de exudado vaginal y de mal olor de vagina. La flora
bacteriana normal, es reemplazada por cocobacilos gramnegativos (Gardnerella vaginalis) y anaerobias (prevotella, movilluncus, mycoplasma). La causa
de esta disbacteriosis es desconocida, y se asocia con la existencia de mltiples parejas sexuales, duchas vaginales y prdida de Lactobacillus, lo que
provoca una elevacin del pH vaginal. Criterios diagnsticos de Amsel: 1) Flujo homogneo, delgado blanco-grisceo. 2) pH >4.5, 3) prueba de aminas
positiva, 4) clulas clave en frotis en seco. Con 3 de 4 criterios son diagnostico; con sensibilidad de 73% y especificidad de 94%. La tincin de Gramm es
el gold estndar. Con los criterios de Nugent la calificacin de <7 correponde a flora normal y de 7-10 correponden a vaginosis bacteriana. Tratamiento:
metronidazol 500mg VO c/12h por 7 dias, metronidazol 0.75% 5 g intravaginal c/24h por 7 dias. Regimen alterno metronidazol 2g VO DU o clindamicina
300mg VO c/12h por 7 dias. En el embarazo: metronidazol 250mg VO c/8h por 7 das o clindamicina 300mg VO c/12 por 7 das. VAGINITIS POR
TRICHOMONAS: La infeccin por T. vaginalis constituye una de las ETS ms frecuentes en el mundo, en la embarazada se ha asociado a parto prematuro
y recin nacido de bajo peso. Se identifica en 30-80% de parejas masculinas de mujeres infectadas, la mayora se mantienen en estado de portador solo
el 10% presenta uretritis no gonoccica. Produce leucorrea profusa, espumosa, amarillo-verdosa y maloliente, eritema vulvar, puntilleo hemorrgico en
el cuello uterino, con abundantes polimorfonucleares, pH alcalino (>4.5) y prurito vaginal. Diagnostico, en frotis en fresco se observan tricomonas
mviles en 50-80%, el mtodo mas sensible es el cultivo. Tratamiento eficaz es el metronidazol DU 2 g (1ra eleccin) o dosis mltiples de 500mg c/12h
por 7 das, se ha mostrado eficaz en ms del 90% de las pacientes 8util tambin en el embarazo), al igual que los nuevos compuestos tinidazol y
ornidazol. VULVOVAGINITIS POR CANDIDA: La colonizacin vaginal por Candida es relativamente frecuente. Muchas de ellas tambin presentan
colonizacin en la zona anorrectal. No presenta sin sintomatologa.Hay inflamacin vulvar y vaginal, fisuras y existencia de un exudado adherente a la
mucosa, blanquecino y amarillento, con grumos (cottage cheese), no fetido. El pH vaginal se mantiene en 4,5. Factores predisponentes la diabetes, el
embarazo, el uso de contraceptivos orales, la obesidad, el empleo reciente de antimicrobianos y la utilizacin de corticoides. Diagnostico con cultivo.
Tratamiento tpico: Existen numerosos tratamientos tpicos mediante aplicacin intravaginal. Referimos solo algunos: 1. Butoconazol 3% en crema, 5 g
durante 3 das. 2. Clotrimazol 100 mg en vulos vaginales, 2 vulos durante tres das. 3. Clotrimazol 500 mg en vulos, una sola aplicacin. 4. Miconazol
100 mg un supositorio vaginal al da durante 7 das. 5. nistatina 100.000 unidades en tableta vaginal, una tableta durante 14 das. Tratamiento oral
fluconazol 150 mg, dos dosis separadas 72 horas. DIAGNOSTICO: Es importante hacer historia clnica minuciosa, interrogar sobre las caractersticas de
flujo, cambio de pareja sexual recientemente, nmero de parejas sexuales, tollas vaginales, duchas vaginales, uso de tampones, tratamientos previos,
etc.. Fortis en seco: Laminilla N.1; aplicar secrecin vaginal+2gotas de sol. Salina 0.9%, observar en campo seco, vemos flagelados mviles con
abundantes PMN, caracteristicos de tricomoniasis o bacterias cocoides y celulas clave (cel.epiteliales descamativas cubiertas por bacterias,
Gardnerella), caractersticas de vaginosis bacteriana. Laminilla N.2; secrecin vaginal+ KOH al 10%, al microscopio vemos levaduras o pseudohifas de
candida, caractersticas. Laminilla N.3; secrecin vagunal + tincin gramm, vemos morfotipos gramm variables de vaginosis bacteriana. Pruebas de las
aminas, se realiza durante la preparacin de la laminilla 2, es positiva cuando se desprende un olor a pescado, y es altamente sugestiva de vaginosis
bacteriana. Los cultivos son tiles en la confirmacin de diagnostico de Candida y tricomonas, en caso de vaginosis bacteriana no hay ningn cultivo que
tenga la sensibilidad y especificidad que lo haga til, la tencion de papanicolao tampoco es confiable para el diagnostico de estos procesos.
CASO CLINICO CERVICOVAGINITIS BACTERIANA
Mujer de 30 aos, acude a centro de salud, refiriendo una secrecin
maloliente vaginal, ardorosa o quemante durante 4 meses. La paciente
fue diagnosticada por primera vez con tricomoniasis vaginal por su
mdico de atencin primaria y que haba recibido cinco ciclos de
metronidazol 500 mg por va oral dos veces al da, la mayor duracin
del tratamiento fue de 10 das. Su marido se trat con 3 das y 7 ciclos
de un da de metronidazol. Ella admiti haber tenido otra pareja sexual
que no haban recibido tratamiento con metronidazol. Su ltimo curso
de metronidazol fue de 1 mes antes de su presentacin. No haba
antecedentes de otras enfermedades de transmisin sexual. Ella
estaba tomando anticonceptivos orales. En el examen, vulva con
eritema, edema y excoriacin. El examen con espculo, la pared
vaginal era eritematosa, edematosa y con secreciones anormales.
PREGUNTA
Cul es al hallazgo mas frecuente en esta patologa?
RESPUESTA
a.- Desaparicin de lactobacilos y bacterias anaerobias.

CURSO ENARM CMN SIGLO XXI TEL: 36246001

b.- Clulas epiteliales con bordes mal definidos.


c.- Abundantes cocobacilos.
d.- Presentacin de cuatro o ms episodios en un ao.
PREGUNTA
Cul seria la alternativa teraputica as adecuada en esta paciente?
RESPUESTA
a.- Azitromicna
b.- Ceftriaxona
c.- Clindamicina
d.- Gentamicina
CASO CLINICO CERVICOVAGINITIS CANDIDA
Paciente de sexo femenino, 28 aos de edad, sin antecedentes
patolgicos previos. Consult por episodios repetidos de
vulvovaginitis. Estos episodios haban comenzado dos aos antes, se
repetan casi mensualmente y aport estudios microbiolgicos previos,
tres de los cuales mostraron pseudohifas y elementos levaduriformes
brotantes en el examen microscpico directo, con desarrollo de

Pharmed Solutions Institute

PGINA 228

MANUAL DE TRABAJO DEL CURSO ENARM CMN SIGLO XXI


Candida spp. Sus ciclos menstruales eran regulares y no tena
antecedentes de embarazos previos. Como mtodo anticonceptivo
empleaba un dispositivo intrauterino. Fumadora y bebedora. No tena
antecedentes de diabetes, ni haba recibido antibiticos, solo refiere
tomar medicamentos en sus cuadros agudos de artritis reumatoide. El
examen fsico no acus anormalidades y ginecolgico apreci aumento
de la secrecin vaginal, congestin de la mucosa y pseudomembranas
blanquecinas en vulva y vagina.

Se trata de paciente femenino de 23 aos de edad la cual acude a


consulta debido a que presenta desde hace tres semanas secrecin
vaginal blanquecina de mal olor, refiere relacin sexual con nuevo
compaero sin proteccin, cuenta con antecedente de uso de
anticonceptivo orales, menarca 10 aos, gesta 1, para 0, abortos 1,
tabaquismo y alcoholismo positivos desde los 17 aos, a la exploracin
se observa secrecin blanca leve que se encuentra recubriendo la
vagina de forma uniforme con olor a pescado.

PREGUNTA
Cul es la conducta ms adecuada para este caso?
RESPUESTA
a.- Metronidazol 500 mg cada 24h, va vaginal 7 das.
b.- Nistatina 100 mil unidades cada 24h, Va vaginal. 14 das.
c.- Itraconazol 200 mg cada 24 h, va bucal. 3 das.
d.- Metronidazol 500 mg cada 12h va vaginal 7 das.

PREGUNTA
Cual es tratamiento ms adecuado para este caso?
RESPUESTA
a.- Metronidazol 2 g VO dosis nica.
b.- Metronidazol 500 mg VO cada 12 hrs x 7 dias.
c.- Fluconazol 100 mg VO dosis nica.
d.- Clindamicina 300 mg VO cada 12 hrs por 5 dias.

PREGUNTA
Cul es el factor predisponente que ms influye en esta paciente?
RESPUESTA
a.- Tabaquismo
b.- Alcoholismo
c.- Artritis reumatoide
d.- Consumo de esteroides

CASO CLINICO
Mujer de 35 aos que consulta por presentar desde hace 6 meses
prurito vulvar recurrente asociado a ardor y quemazn. Visit a varios
profesionales habiendo recibido tratamientos locales y/o sistmicos
con antimicrobianos (antibacterianos y antimicticos). Desde hace 4
das nota prurito vulvar y vaginal intenso, as como flujo blancogrisceo. Refiere tambin disuria terminal, sin tenesmo vesical, ni
polaquiuria. AGO: menarca 12 aos, ritmo 5x28, G: 3, P: 3, FUP hace 3
aos. Pareja estable hace 15 aos. Tiene pH vaginal de 4.6 y prueba de
aminas positiva.

CASO CLINICO CERVICOVAGINITIS TRICOMONIASIS


Paciente femenino de 42 aos de edad present una historia de 10
meses de evolcuion de descarga vaginal persistente por el que haba
recibido ciclos intermitentes de eritromicina. Ella se encontr que
tienen tricomoniasis y se le prescribi metronidazol 400 mg dos veces
al da durante 7 das. Ella neg cualquier actividad sexual posterior. Su
compaero sexual anterior tambin fue tratado con metronidazol. T.
vaginalis se aisl repetidamente en el seguimiento con cultivos, sobre
las prximas semanas a pesar de nuevos cursos de metronidazol
intravaginal tanto oral como tpica.
PREGUNTA
Cul es la conducta ms adecuada para este caso?
RESPUESTA
a.- Metronidazol 500 mg cada 12 h, va bucal. 7 das
b.- Metronidazol 500 mg cada 24h va vaginal. 7 das.
c.- Nistatina, 100 mil unidades cada 24h, Va vaginal. 14 das
d.- Metronidazol 500 mg cada 24h, va vaginal, 7 das.

PREGUNTA
Qu estudios nos orientaran ms hacia este diagnstico?
RESPUESTA
a.- Frotis en fresco con solucin salina al 0.9% ms tincin de Gramm
b.- Frotis en fresco ms KOH al 10% ms cultivo
c.- Frotis en fresco con solucin salina al 0.9% ms cultivo
d.- Frotis en fresco ms KOH al 10% ms tincin de Gramm

CASO CLINICO CERVICOVAGINITIS TRICOMONIASIS


Una mujer de 28 aos refiere que durante los ltimos tres aos, la
haba estado tratando sin xito una recurrente descarga vaginal
desagradable. En su primera visita a la clnica, Trichomonas vaginalis se
encontr en una preparacin hmeda de las secreciones vaginales y se
indico metronidazol 200 mg tres veces al da durante una semana fue
recetado. Durante los prximos seis meses que tena tricomoniasis
sintomtica recurrente.
PREGUNTA
Cul es la conducta ms adecuada para este caso?
RESPUESTA
a.- Bao diario.
b.- Enjuague anogenital al medio da.
c.- Uso de toallas sanitarias solo durante la menstruacin.
d.- Evitar el uso diario de pantiprotectores y tampones.
CASO CLINICO CERVICOVAGINITIS GARDNERELLA VAGINALIS

CURSO ENARM CMN SIGLO XXI TEL: 36246001

PREGUNTA
Cul es el diagnstico ms probable para este caso?
REPUESTA
a.- Vaginosis bacteriana
b.- Tricomoniasis
c.- Candidiasis vaginal
d.- Gonorrea

CASO CLINICO
Mujer de 26 aos de edad comprometida, presenta secrecin vaginal
ftida. Niega prurito vulvar o dispareunia, aunque ha observado una
secrecin delgada grisasea. El examen fsico revela una mujer afebril,
de buen aspecto, un tanto ansiosa con presin arterial de
125/85mmHg y frecuencia cardiaca de 8/min. Un examen plvico
muestra secrecin acuosa, grisasea con pH>4.5. Al mezclar la secrecin
con hidrxido de potasio se despide un olor a pescado.
PREGUNTA
Cul seria la complicacin mas probable al estar tomando el
tratamiento para esta patologa?
RESPUESTA
a.- Resequedad en la piel, tras exposicin solar
b.- Rash cutneo
c.- Reaccion tipo disulfiram, tras consumo de alcohol
d.- Nauseas y vomito

Pharmed Solutions Institute

PGINA 229

MANUAL DE TRABAJO DEL CURSO ENARM CMN SIGLO XXI


ENFERMEDADES DE TRANSMISION SEXUAL (ETS)
CIENCIAS BASICAS: Son conocidas tambin como enfermedades venreas (en honor a la diosa romana Venus que es el equivalente a la diosa griega del
amor y la belleza Afrodita) y su definicin obedece al modo de transmisin de las mismas ya que se adquieren mediante el contacto sexual: vaginal, anal
u oral. Todas las conductas sexuales que involucran contacto con otra persona o con los fludos de la misma se considera como factor de riesgo para la
adquisicin de ETS. SALUD PUBLICA: En Mxico y tambin a nivel mundial, las ETS han representado un problema de salud pblica; a nivel nacional son
una de las diez primeras causas de morbilidad, con un promedio de 220,000 casos anuales. La frecuencia de ETS en la poblacin general flucta entre 0.1
y 0.5%; en cambio, para la poblacin considerada como de alto riesgo, se calcula una frecuencia que va de 10 a 20%. La poblacin expuesta se compone
en su mayor parte de jvenes, el 34% corresponde al periodo entre la 2da y 3ra dcada de la vida, seguido por el grupo de 25 a 44 aos que integran dos
dcadas en un 41%. La relacin hombre mujer fue de 2,8:1. GONORREA O BLENORRAGIA (Neisseria gonorrhoeae): la cual tiene tropismo por las
mucosas, el humano es el nico husped conocido. Puede causar uretritis, cervicitis, epididimitis, faringitis, proctitis, EPI y en los casos graves
diseminacin sistmica. La transmisin ms frecuente es mediante el acto sexual vaginal, anal u oral y se hace a travs del contacto de la superficie
mucosa de un individuo infectado. Tambin puede ser contagiada mediante fmites a otras reas, por ejemplo a un ojo con la mano. La transmisin
vertical ocurre cuando la madre infecta al recin nacido mediante el paso por el canal del parto, el producto adquiere la infeccin manifestndose como
conjuntivitis, vulvovaginitis e inclusive neumona, por lo tanto, en estos casos estara indicado el parto por cesrea. Patogenia: Este microorganismo se
adhiere a la superficie de los epitelios uretral, endocervical, vaginal, rectal, farngeo e incluso a los espermatozoides humanos y a las clulas ciliadas de
las trompas de Falopio, pueden expresar simultneamente varios tipos de cadenas de LPS antignicamente diferentes, que median en gran parte su
toxicidad. Las manifestaciones aparecen de 2-5 das posterior al contacto de sexual. Se inician con leve molestia uretral, seguida de disuria de grado
variable y a veces urgencia miccional. Hay eritema y edema del meato urinario y salida de material purulento por l, sin embargo la infeccin puede ser
asintomtica en 5-7% de los afectados. Puede complicarse con epididimitis, prostatitis, tromboflebitis, absceso periuretral o fstulas, vesiculitis seminal,
estenosis uretral y esterilidad. En mujeres los sntomas suelen aparecer entre 7-21 das despus de la infeccin, disuria, descarga vaginal y a veces fiebre.
En los hombres, lo ms comn es que el paciente presente una uretritis anterior con disuria (que es ms frecuente durante la maana) y una descarga
uretral purulenta caracterstica. La complicacin local ms comn es secundaria al ascenso de los microorganismos y se presenta como una salpingitis
aguda o EPI en el 10%-20% de los casos. El diagnstico de gonorrea es fundamentalmente clnico y se comprueba con la identificacin de N. gonorrhoeae
(diplococo gram negativo). Uretritis se hace con el hallazgo de 4 o ms PMN por campo por 1000X; el de uretritis gonoccica en tanto es altamente
sugerente por la presencia al de cocobacilos gram negativos en granos de caf. Cuando el examen directo es negativo y la sospecha es alta, debe
realizarse el cultivo en medio de Thayer Martin, medio enriquecido. Tratamiento: en no resistentes: Penicilina procainoca 4.8 UI millones IM DU,
precedida de probenecid 1g VO. Resistencia: ceftriaxona 500mg IM DU, espectinomicina 2 g IM DU, Alergicos: Ciprofloxacino 500mg DU, clrhidrato de
tetraciclina 500mgs c/6hrs por 7 dias. Neiseria meningitidis: practicas sexuales orogenitales o anogenitales. Si se comprueba la infeccin conjunta con C.
trachomatis el paciente puede ser tratado con azitromicina o doxiciclina. Es importante mencionar que debe darse tratamiento a las parejas sexuales de
los pacientes, y recordarles que no pueden tener relaciones sexuales hasta que se haya completado el tratamiento. CERVICITIS Y URETRITIS NO
GONOCCICA (UNG) (Chlamydia Tracomatis y Mycoplasmas; M. hominis, M. genitalum, Ureaplasma urealyticum): Estos son los patgenos ms
frecuentes. Corresponden al 40 a 50% de las uretritis. En un 30% de los casos no se encuentra el agente etiolgico. La infeccin asintomtica, secrecin
uretral o cervical hialina, disuria, dispareunia, cervicitis, bartolinitis, se debe descartar con antecedente de N. gonorroeae. Mycoplasma hominis: fiebre
posparto, meningitis neonatal, bacterenia, pielonefritis, EPI, absceso plvico, artritis sptica. M. genitalum: asociado a EPI. Chlamydia trachomatis es un
parasito intracelular obligado, fecuencia de 3-6%, relacionado por su cronicidad conm embarazo ectpico (toxicidad directa por estallido celular y de
respuesta inmune, lo cual es responsable del dao tubario), EPI, Sx Fitz-Huch-Curtis (perihepatitis producida por una peritonitis secundaria al ascenso de
bacterias, como resultado de una EPI), en feto neumona y conjuntivitis en mujeres puede provocar dao tubario e infertilidad. La edad pico de la
infeccin por C. trachomatis es la adolescencia tarda y primeros aos de la tercera dcada. El uso de anticonceptivos orales y la ectopa cervical son
factores de riesgo para infeccin por C. trachomatis, es un patgeno intracelular obligado de tamao pequeo. El perodo de incubacin va de 2-3
semanas. Clsicamente se describe una secrecin uretral escasa de tipo muco-serosa, fundamentalmente matinal. Hay leve disuria y prurito y escasa
congestin del meato uretral. Puede ir desde un cuadro asintomtico a un cuadro similar a la uretritis gonoccica. El crvix en fresa es caracterstico de
chalamydia. Diagnstico: C. trachomatis; Aislamiento en cultivos celulares (celuas McCoy o HeLa) a partir de raspado endocervical y/o uretral, pruebas
serolgicas en bsqueda de anticuerpos, hibridizacion in situ. Mycoplasma; se incuba en caldo de arginina y caldo de urea, si hay viraje del indicador se
siembra en medio de arginina y agar E. Tratamiento: Clorhidrato de tetraciclina 500mg cada 6hrs por 7 dias o doxiciclina 100mg c/12hrs minimo 7 dias
(evitarse asolearse, fotosensibilizacin) o Azitromicina 1g DU. Dar tratamiento a parejas sexuales. Mujeres embarazadas o lactancia: Azitromicina 1 g VO
DU o Eritromicina 500 mg VO c/6 h por 7 das Recurrencia y persistencia: Metronidazol 2 g VO DU o 500 mg c/12 h por 7 das + eritromicina 500 mg VO
c/6 h por 7 das. SIFILIS (Treponema pallidum): Tambin como les, mal glico, es una enfermedad crnica adquirida por contacto sexual (aunque
tambin debe tenerse en cuenta la infeccin prenatal), la lesion primaria es indolora, nica, bien delimitada, apenas elvada, fondo limpio, rojo, liso, sin
sangrado, exudado, ni supuracin, consistencia indurada (chancro duro), la adenopata inguinal no inflamatoria, los ganglios conservan individualidad,
indoloros, agrupados, alrededor de uno de mayor tamao (Signo de la Plyade). Es una enfermedad intermitente que tiene perodos de actividad; fase
primaria; chancro primario duracin 3-6semanas y desaparece. Secundaria; exantema y lesiones en mucosas, puntilleo rojizo o marrn en palmas y
plantas, fiebre, adenomegalias, dolor farinego, alopecia, cefalea, perdida de peso, mialgias, adinamia (etapa dosde se es altamente contagioso).
Desaparecen con tratamiento, si no hay una fase latente (aqu permanece el 70%): con ausencia de signos y sntomas con estudios serolgicos positivos,
periodo mas infectante. Terciaria: hay muy pocos microorganismos y una respuesta inmune celular aumentada, en esta etapa puede afectar SNC (510%), ojos, cardivascular (80-85%), hgado, osteoarticular (dificultad para coordinar movimientos, ataxia sifiltica, paralisis, amaurosis progresiva y
demencia), debido a la respuesta de hipersensibilidad retardada se producen reas de inflamacin local y formacin de gomas en el tejido afectado. El
Treponema pallidum es una espiroqueta, incapaz de sobrevivir fuera del husped, la transmisin ocurre mediante la penetracin de la espiroqueta en las
membranas mucosas o abrasiones en las superficies epiteliales, el tiempo de incubacin desde la exposicin a la aparicin de las lesiones primarias es en
promedio de 3 semanas con un rango de 10 a 90 das. La sfilis latente, se divide en temprana (cuando dura menos de un ao) o tarda (duracin de mas
de un ao) y tpicamente encontramos serologa positiva con anticuerpos especficos al T. pallidum y el paciente se encuentra asintomtico. Diagnostico:
Identificacion del treponema por el estudio de campo oscuro en el frotis del chancro o aspirado de ganglio, VDRL, es serolgico de escrutinio.
Confirmatorio anticuerpos antitreponmicos IgM: FTA-abs IgM. Tratamiento: sfilis primaria con Penicilina benzatinica 2.4millones UI IM por 10-14 dias,
alrgicos con doxiciclina 100mgs c/12hrs por 14 dias (en tardia hasta 28 dias), tetraciclina 500mg c/6hrs por 7 dias. Neurosifilis: Penicilina G cristalina
20millones UI IV c/4hrs por 20 dias. El rgimen alternativo es doxiciclina 200mg diariamente por 28-30 das. LINFOGRANULOMA VENREO (Chlamydia
trachomatis): Tambin conocido como enfermedad de Durand-Nicolas-Favre o como linfogranuloma inguinal. En Mxico es poco frecuente con una tasa
de incidencia de 0.2% por cada 100000, sin embargo en algunos pases de frica y Asia se considera una enfermedad endmica. La infeccin ocurre
despus del contacto directo con la piel o las membranas mucosas de la pareja sexual infectada. El microorganismo penetra en una solucin de
continuidad; viaja por los vasos linfticos hasta los ganglios donde se replica dentro de los macrfagos e inicia la respuesta inflamatoria. A pesar que la

CURSO ENARM CMN SIGLO XXI TEL: 36246001

Pharmed Solutions Institute

PGINA 230

MANUAL DE TRABAJO DEL CURSO ENARM CMN SIGLO XXI


mayora de los casos son predominantemente por contacto sexual, tambin hay casos de infeccin por accidentes de laboratorio, fomites o contacto
directo. El linfogranuloma venreo ocurre en tres etapas: Primera, que puede ser asintomtica y pasar desapercibida, aparece una ppula, vesicula no
dolorosa en el sitio de inoculacin que rpidamente se convierte en pstula y se ulcera (chancro de inoculacin) con resolucin espontnea de la misma
en una semana aproximadamente. Segunda etapa se presenta de 2 a 6 semanas y consiste en linfadenopata inguinal, inicia 1 ganlglio duro empastado,
indoloro o poco doloroso, fiebre y malestar general, psterirmente reaccin de ganglios vecinos, confluyen en masa que se extiende a espina iliaca
anterosuperior, afeccion ganglios femorales, la piel invadida se engruesa y adhiere a ganglios, aspecto violceo, piel de naranja, lesion caracterstica
bubn situado por arriba y debajo de ligamento inguinal, separado por depresin lineal signo del surco. Finalmente la tercera, proctitis, estenosis
rectal, absceso perirrectales, fistulas, edema genital grave. Diagnostico: La aspiracin del material purulento de los bubones es la mejor muestra.
Tratamiento: Doxiciclina 100mg c/12hrs por 21 dias, en embarazadas eritromicina 500mgs c/6hrs por 21 dias, azitromicina 1g cada semana por 3
semanas. El drenaje de los bubones puede ayudar en la disminucin de la sintomatologa y prevenir la formacin de lceras. CHANCROIDE (Haemophilus
ducreyi): Es conocido tambin como chancro blando o ulcus molle. El periodo de incubacin es entre 3 y 10 das. Se localiza principalmente en genitales y
regin perianal. En hombres se puede encontrar en prepucio, en el surco coronal o en el frenillo, y no es raro que exista un edema importante del
prepucio. En mujeres en introito vaginal, aunque tambin se encuentra en el crvix o el rea perianal. La caracterstica de estas lceras es que son
blandas (de ah su nombre) con fondo sucio amarillento y rpidamente se ulcera necrosante, adenopata inguinal unilateral, plastrn inflamado con la
piel que cubre eritematosa y dolorosa bubn. Las lesiones se pueden autoinocular a otras reas del cuerpo muy fcilmente. Diagnostico: frotis Gram,
cultivo en agar chocolate adicionada con sangre del enfermo. Tratamiento: Azitromicina 1g VO DU, ceftriaxona 250mgs c/12hrs por 3 dias, eritromicina
500mgs c/6hrs por 7 dias. GRANULOMA INGUINAL (DONOVANOSIS): El agente etiolgico es Calymmatobacterium granulomatis, una bacteria
pleomrfica gramnegativa; sin embargo, y dada la similitud filognetica que tiene con el gnero Klebsiella, se ha propuesto el cambio de nombre
a Klebsiella granulomatis. En Mxico es muy rara, y los casos que se logran detectar son en personas que han viajado a zonas endmicas. El modo de
transmisin es primariamente sexual, aunque la infeccin tambin puede adquirirse mediante fmites, va oral-fecal por alimentos contaminados o a
travs del canal del parto. El periodo de incubacin 3-40dias . La lesin inicial es una ppula firme o nodulo suave que rpido crece y se ulcera. Una vez
que se ha formado la lcera, inicia la destruccin del tejido adyacente con exudado maloliente y presencia de tejido necrtico, aspecto granulomatoso
rojo, sangra con facilidad, con el tiempo fibrosis e hiperplasia epitelial vegetante, aos despus lesiones queloides y elefantiasis genitales externos y
fistulas. Diagnstico: identificacin de cuerpos de Donovan (que son colonias de C. granulomatis) dentro de los macrfagos en un frotis de la lcera
teido con Giemsa, Wright o en biopsia de las lesiones. Tratamiento: doxiciclina 100mg c/12hrs por 3 semanas o hasta que las lesiones hayan cicatrizado
por completo, puede agregarse genamicina. Como alternativa se puede emplear azitromicina 1gr va oral una vez a la semana durante 3 semanas. As
tambin se puede utilizar ciprofloxacino 750mg dos veces al da durante por lo menos 3 semanas, eritromicina 500mg c/6hrs. HERPES SIMPLE: El virus
del herpes simple es un microorganismo patgeno que causa infeccin orolabial (VHS-1) y genital (VHS-2) caracterizadas por erupciones vesiculares
primarias y recurrentes. Las infecciones por el VHS-1 se presentan con gran variabilidad y la regla es que se observen hasta en el 80% de los casos en
forma asintomtica; se desconoce el nmero exacto de personas infectadas ya que se considera que ms del 95% de la poblacin mundial tiene
anticuerpos contra este virus que contrajo en el 1er ao de vida. En Mxico se desconoce la prevalencia de la infeccin por el VHS tipo 2 que es
considerado como ETS, el espectro clnico del herpes simple genital va desde la primo- infeccin primaria que raramente se observa para posteriormente
darnos los episodios de recurrencia (prurito o ardor donde aparecern las vesculas)que son variables en frecuencia e intensidad. En el sitio de entrada
del virus, que generalmente son los genitales externos aparecen vesculas que se agrupan sobre una base eritematosa, stas fcilmente progresan a
pstulas y lcera, son superficiales y aparecen de 2-7 dias del contacto sexual. No es infrecuente encontrar durante la primoinfeccin una balanitis
erosiva, vulvitits o vaginitis. En las mujeres las lesiones tambin pueden encontrarse en crvix, nalgas y perineo. Se requiere de un diagnstico certero
que es fundamentalmente clnico, recordando que cada episodio es autorresolutivo en 3 a 7 das. Cultivo del virus, PCR, citologa o prueba de Tzanck
menos sensible. Tratamiento: generalmente sintomtico con analgsicos, aplicacin de compresas fras, y medidas adecuadas de higiene. Debe hacerse
hincapi en explicar amplia y detalladamente la evolucin de la infeccin, es decir que habr periodos de recidivas. Primer episodio: Aciclovir 400mg
c/8hrs por 7-10 dias o valaciclovir 1g c/12hrs por 7-10 dias. Brotes recurrentes: Aciclovir 800mgs c/12hrs, famciclovir 500mg c/12hrs. Debemos hacer
hincapi en que los antivirales por va tpica son totalmente ineficaces. VIRUS DE MOLUSCO CONTAGIOSO: Produce Hiperplasia e hipertrofia de los
queratinocitos de la epidermis, incremento de receptores para factor de crecimiento epidrmico, las celuas que primero se afectan son las de la capa
basal epidermis, por la mitosis celular se van desplazando los estratos mas superficiales, pueden ser pseudoquistes, pedunculado, gigante (>1.6cm).
Incubacion 14-60 dias, lesion papula o nodula de 2-5mm, alcanzan de 5-10mm entre 6-12 semanas; con umbilicacin central, firmes, palidas de color
carne, aspecto perlado, nucleo central material hmedo viscoso, indoloras. Diagnostico: clnico, biopsia (masa esfrica hialina eosinofilicas). Tratamiento:
puede haber resolucin espontanea en 4 meses, destruccin de lesiones con curetaje, crioterapia, electrodesecacion, agentes qumicos: imiquimod,
acido triclorocetico 80%.
CASO CLINICO SIFILIS
A primigesta de 24 aos de edad con amenorrea secundaria sin
determinar fecha y dolor abdominal y lumbar que irradia en su muslo
derecho. Durante los tres meses anteriores, el paciente descrito
sangrado vaginal irregular y flujo vaginal acuoso. IVSA a los 14 aos de
edad y se observ un mnimo de 15 parejas sexuales anteriores.
Resultados positivos de examen toxicolgico de drogas ilegales.
Durante los ltimos 6 aos, los resultados del cultivo y serologa
document cuatro infecciones previas por separado con Chlamydia
trachomatis, Neisseria gonorrhoeae, herpes simplex virus 2 y
Treponema pallidum, respectivamente. El paciente inform que
complet el tratamiento para las cuatro infecciones.
PREGUNTA
Cul es el padecimiento infeccioso que ms probable presenta?
RESPUESTA
a.- Sifilis
b.- Gonorrea
c.- Herpes simple
d.- Cervicitis

CURSO ENARM CMN SIGLO XXI TEL: 36246001

CASO CLINICO GONORREA.


Acude a consulta paciente femenino de 24 aos de edad, refiere que
inicia padecimiento hace 2 semanas aproximadamente caracterizado
por flujo amarillento, en cantidad moderada, agrega que su esposo
tiene secrecin amarillenta proveniente del meato urinario la cual es
ardorosa.
PREGUNTA
Cual es la conducta a seguir ms apropiada para el manejo de
incapacidad y referencia?
RESPUESTA
a.- Incapacidad y referencia a segundo nivel.
b.- 7 dias de incapacidad y referencia a segundo nivel.
c.- Iniciar el tratamiento e incapacidad de 24 hrs.
d.- Incapacidad hasta resolucin del padecimiento.
CASO CLINICO
Paciente mujer de 21 aos remitida a nuestro hospital por su mdico
de familia con el diagnstico de conjuntivitis aguda purulenta bilateral,
por no responder al tratamiento tpico con la asociacin de

Pharmed Solutions Institute

PGINA 231

MANUAL DE TRABAJO DEL CURSO ENARM CMN SIGLO XXI


gentamicina y dexametasona ni con la asociacin de neomicina,
polimixina y gramicidina. Como antecedentes personales la paciente
refiere que es alrgica a las cefalosporinas. En la exploracin presenta
clnica en ambos ojos de edema palpebral severo, abundante secrecin
purulenta, y epitelitis con lesin superior yuxtalmbica en la crnea de
ojo derecho (OD) que implica capas profundas del estroma, no
observndose sta en el ojo izquierdo (OI). Se remiten muestras de la
secrecin conjuntival al servicio de microbiologa, y se pauta
tratamiento emprico con ofloxacino y la asociacin cloranfenicol y
dexametasona tpicos en horas alternas, con limpieza previa de las
secreciones con suero fisiolgico, y por va enteral con ciprofloxacino,
betametasona y dexclorfeniramina. En el cultivo se identifica Neisseria
Gonorrhoeae, y el antibiograma demuestra resistencia al
ciprofloxacino y tetraciclinas, la paciente es alrgica a las
cefalosporinas.

CASO CLINICO
Masculino de 44 aos, alrgico a la penicilina, acude a nuestra consulta
con su mujer porque desde hace 2 semanas presenta una lcera en el
glande no dolorosa y adenopatas inguinales. Niega haber tenido
relaciones sexuales extramatrimoniales.
PREGUNTA
Cual es la conducta a seguir?
RESPUESTA
a.- Al negar relaciones sexuales extramatrimoniales podemos descartar
enfermedades de transmisin sexual.
b.- Se trata de una sfilis primaria, trataremos con una dosis de
penicilina G benzatina 2,4 MU i.m.
c.- Entre el diagnstico diferencial de la lcera genital hay las
enfermedades de transmisin sexual. Debemos pedir serologas para
sfilis.
d.- Al tratarse de una lesin cutnea derivamos al paciente al
dermatlogo de referencia.
PREGUNTA
Si finalmente fuera diagnosticado de sfilis primaria, cual es la
conducta prescriptiva mas adecuada?
RESPUSTA
a.- Administraremos una dosis de penicilina benzatina 2,4 MU i.m.
b.- Administraremos penicilina benzatina 2,4 MU i.m. una
administracin semanal por 3 semanas.
c.- Doxiclina 100 mg va oral dos veces al da por 14 das.
d.- Doxiciclina 200 mg oral dos veces al da durante 28 das.
CASO CLINICO
Un hombre de 30 aos de edad, con presencia de sangrado con moco
por via rectal varias veces al dia por tres semanas. La sigmoidoscopia
reporto mucosa granular con varias lceras a 5-7 cm distales del recto.
La biopsia rectal reporto colitis irregular severamente activa con
criptas distorsionadas semejante a la criptitis de la enfermedad de
Crohn. El historial clnico adicional revel que el paciente tuvo
relaciones sexuales anales sin proteccin durante el viaje a travs de
Europa tres meses antes. Se confirm la presencia de C. trachomatis.

CURSO ENARM CMN SIGLO XXI TEL: 36246001

CASO CLINICO
Masculino de 28 aos de edad, VIH positivo se presenta con sangrado
rectal. La anoscopa revel una lcera en el borde anal. Las biopsias
mostraron fragmentos de piel anal con ulceracin, tejido linfoide y
tejido de granulacin inflamatorio histolgicamente sospechoso de
una enfermedad inflamatoria del intestino. La sigmoidoscopia mostr
una mucosa eritematosa en el recto distal con pequeas reas de
exudados blancos y pliegues apilados. Las biopsias mostraron proctitis
linfohistiocitario grave y criptitis. Las pruebas serolgicas fueron
positivos para C. trachomatis .
PREGUNTA
Cual es la conducta teraputica a seguir mas adecuada?
RESPUESTA
a.- Tetraciclina
b.- Doxiciclina
c.- Eritromicina.
d.- Ciprofloxacino.

PREGUNTA
Cual es la conducta mas adecuada a seguir?
RESPUESTA
a.- Agregar cloranfenicol.
b.- Agregar vancomicina.
c.- Agregar imipenem.
d.- Agregar doxiciclina.

PREGUNTA
Cual es la conducta teraputica a seguir mas adecuada?
RESPUESTA
a.- Tetraciclina

b.- Doxiciclina
c.- Eritromicina.
d.- Ciprofloxacino.

CASO CLINICO
Mujer de 48 aos, nulpara, con antecedente de un embarazo ectpico
tubario derecho, que ingresa para una histerectoma electiva por
adenomiosis sintomtica, complicada por una anemia secundaria
crnica. Se realiz histerectoma total, con conservacin de ovarios,
por va laparoscpica, sin incidentes. Una vez terminada la ciruga, se
realiz la exploracin de la cavidad abdominoplvica, incluyendo los
rganos y paredes, tal como lo hacemos en forma metdica y rutinaria
en toda ciruga laparoscpica. Al observar la superficie heptica, se
encontraron adherencias en "cuerda de violn", entre el hgado y la
pared abdominal. Durante el postoperatorio, se le comunic a la
paciente el hallazgo y se pregunt a la paciente por antecedente de
dolor abdominal en hipocondrio derecho, ante lo cual relat no
tenerlo. Sin embargo, record haber sido tratada con antibiticos, por
flujo vaginal persistente 15 aos antes.
PREGUNTA
Cual es el diagnostico diferencial mas probable, considerando los
hallazgos del caso?
RESPUESTA
a.- Sndrome de Beckwith Wiedemann.
b.- Sndrome de Fitz-Hugh-Curtis.
c.- Sndrome de Simpson-Golabi-Behmel.
d.- sndrome de Klippel-Trenaunay-Weber
CASO CLINICO
Una mujer de 23 aos present ojo irritado y rojo durante 4 semanas,
con sensacin de prdida parcial de la audicin en el odo derecho
durante 5 das, refiriere adems flujo vaginal de color amarillo durante
1 semana. Se diagnostico conjuntivitis folicular del ojo derecho,
cervicitis mucopurulenta, y un derrame en el odo derecho confirmado
por timpanometra. Audiometra de tonos puros mostraron prdida de
audicin conductiva en el lado derecho. Se aislo C. trachomatis del ojo
derecho, el cuello uterino, y la nasofaringe, pero no del aspirado del
odo medio.
PREGUNTA
Cual es la conducta teraputica a seguir mas adecuada?
RESPUESTA
a.- Tetraciclina
b.- Doxiciclina
c.- Eritromicina.
d.- Ciprofloxacino.

Pharmed Solutions Institute

PGINA 232

MANUAL DE TRABAJO DEL CURSO ENARM CMN SIGLO XXI


ENFERMEDAD PELVICA INFLAMATORIA (EPI)
CIENCIAS BASICAS: Comprende las alteraciones inflamatorias e infecciosas que afectan los rganos genitales situados en la pelvis menor. Incluye
combinaciones de cervicitis, endometritis, salpingitis, anexitis, parametritis (ligamentos de soporte) y pelviperitonitis, as como abscesos tubo ovricos.
SALUD PUBLICA: Es ms frecuente y grave en mujeres de 15-25 aos. 1 de cada 4 EPI experimentan secuelas. Su importancia radica no slo en la
morbilidad aguda, sino en su capacidad de producir esterilidad, gestacin ectpica, recidivas y dolor abdominal crnico. Tasa de incidencia anual de 17.2
por mil mujeres entre 15-44 aos. Afecta cada ao del 1-2% de mujeres jvenes sexualmente activas. PATOGENIA: La EPI es la complicacin ms
frecuente de las ETS bacterianas. Suele ser polimicrobiana. Los patgenos ms frecuentes son: Neisseria Gonorrhoeae (5-36%) y Chlamydia Trachomatis
(17-70%), Aerobios (Estreptococos del grupo B, Escherichia coli 30%, Gardenella vaginalis,
Mycoplasma hominis 37-75%), anaerobios
(Peptoestreptococos, Bacteroides fragilis 60%), y un 5% de los casos patgenos respiratorios (Haemophilus influenzae 5%, Streptococcus pneumoniae,
Streptococcus pyogenes). Las bacterias pueden acceder a la porcin superior del aparato genital por diseminacin de rganos adyacentes infectados
(apendicitis, diverticulitis), por diseminacin hematgena de focos distantes (tuberculosis) y fundamentalmente por diseminacin ascendente
transuterina a partir del tracto genital inferior. En los primeros estados de la enfermedad, la luz de la trompa permanece abierta con lo cual se permite
que este exudado salga por la fimbria hacia la cavidad plvica, lo que produce peritonitis plvica. Como resultado de esta inflamacin peritoneal, las
estructuras adyacentes se comprometen en el proceso inflamatorio. La tensin de oxgeno en estos tejidos inflamados y necrticos disminuye lo que
favorece el crecimiento de flora anaerobia. La destruccin del tejido con la posterior degradacin lleva a la formacin de abscesos. Los factores de riesgo:
Edad <25 aos, mltiples compaeros sexuales, ETS, no utilizacin de mtodos de barrera, historia previa de EPI, historia de vaginosis, cervicitis,
dispositivos intrauterinos, solo tiene relacin con la EPI en los 3 meses posteriores a la insercin por la manipulacin, abortos, instrumentacin uterina,
ciruga cervical, trmino de embarazo. CLASIFICACIN: Podemos diferenciar cuatro estadios Monif: Grado I (leve-no complicada): sin masa anexial, no
datos de abdomen agudo. Grado II (moderada-complicada): con masa anexial o absceso trompas y/o ovarios, con o sin signos de irritacin peritoneal.
Grado III (grave): absceso tubo-ovarico roto o pelvi peritonitis, con datos de respuesta sistmica. Clasificacion laparoscpica: Leve; eritema, edema, las
trompas se mueven libremente, no hay exudado purulento. Moderada; eritema, edema mas marcado, material purulento evidente. No hay movimiento
libre de las trompas. La fimbria puede no ser evidente. Severa; presencia de piosalpinx y/o absceso. DIAGNOSTICO: puede cursar con los siguientes
sntomas: dolor abdominal bajo (95%), y su intensidad vara de totalmente ausente (EIP silente) a muy intenso en cuadros con un componente peritoneal
importante. Aumento del flujo vaginal, flujo de caractersticas anormales (74%). Sangrado anormal (intermestrual, poscoital) (45%). Sntomas urinarios
(35%), como disuria y polaquiuria. Sntomas digestivos (14%), como nauseas, vmitos, diarrea. Signos: dolor a la movilizacin del cuello, dolor anexial en
la exploracin vaginal bimanual (99%). En el examen con espculo observamos cervicitis y descarga endocervical purulenta (74%). Fiebre (> 38 C) (<
47%). Masa plvica: sugiere absceso tuboovrico (ATO). Signos de peritonitis. Las pacientes con infeccin por Chlamydia pueden cursar con salpingitis
subclnica o subaguda, con secuelas de adherencias e infertilidad. Hay que buscar datos de de SRIS, al menos 2; T>38 o <36, taquicardia (>901pm),
taquipnea (>20lpm o PaCO2 <32mmHg), leucocitos >12 000 o >4 000. Ante la sospecha de EPI debemos realizar: Hemograma y bioqumica general, con
determinacin de VSG y PCR, test de embarazo en orina o HCG srica, tomas vaginales y endocervicales para deteccin de gonococo y clamidias.
Ecografa, sobre todo en su forma transvaginal, ya que se obtendr informacin sobre la existencia o no de abscesos tuboovricos, liquido libre. La
laparoscopia (estndar de oro) es el nico mtodo fiable para el diagnstico, que permite una visin directa y la toma de cultivos. Biopsia endometrial;
endometritis. Serologa completa. Criterios de Hager modificados por la SEGO, son todos los criterios mayores y de al menos uno menor. CRITERIOS
MAYORES: Dolor en abdomen inferior. Dolor a la movilizacin del cervix.
Dolor anexial a la exploracin abdominal. Historia de actividad sexual en
los ltimos meses. Ecografa no sugestiva de otra patologa. CRITERIOS
MENORES: Temperatura > 38C. Leucocitosis >10500 /l. VSG elevada.
GRAM de exudado intracervical con diplococos intracelulares (gonococo),
cultivo positivo para N.Gonorrhoeae o C.Trachomatis, M. hominis,
Ureaplasma, gramm negativos, anaerobios. SINDROME FITZ-HUGHCURTIS: Es una perihepatitis, se asocia con EPI, principalmente por
Chlamydia, hay desarrollo de adherencias y fibrosis perihepaticas, dolor
agudo en cuadrante superior derecho, no hay alteracin de enzimas
hepticas. TRATAMIENTO: Ante la sospecha de EPI se debe instaurar
tratamiento antimicrobiano emprico, para prevenir las secuelas. En las
pacientes con leve-moderado ambulatorio con: ofloxacino 400mg VO
c/24h o lecofloxacino 500mg c/24h por 14 dias mas metronidazol 500mg
c/12hrs por 14 dias o clindamicina 450mg c/6h por 14 dias. Guias
internacionales. Cefotaxima 2g IM DU + 1g de probenecid VO DU (1RA
ELECCIN) mas doxiciclina 100mg VO c/12 h por 14 dias Ceftriaxona
250mg IM DU + probenecid 1g + doxiciclina 100mg VO c/12h por 14 dias
(mismo tratamiento para la pareja sexual). Alternativa azitromicina 2g. Las pacientes que no respondan al tratamiento antibitico ambulatorio en 48 h,
deben ser hospitalizadas Ceftriaxona 250mg IM + doxiciclina 100mg c/12h + metronidazol 400mg c/12h por 14 dias clindamicina 900mg IV c/8h +
gentamicina 2mg/kg inicial posterior 1.5/kg c/8h. Cefoxitina 2g IV c/6h cefotetan 2g IV c/12h + doxiciclina 100mg IV o VO c/12h. Alternativa: Aztreonam
2-4g/dia IV + clindamicina 600mg IV c/6h. Este rgimen debe administrarse hasta 48 horas despus de que la paciente muestra mejora clnica evidente.
En caso de presencia de DIU es preciso la extraccin y cultivo del mismo. Si existe absceso mayor de 8 cm se proceder al drenaje quirrgico. En
ocasiones, es necesaria la histerectoma y la anexectoma. Las parejas sexuales de las pacientes con EIP 2 gr. de Azitromicina DU VO Ceftriaxona 250mg
IM DU y Doxiciclina 100mg/12 horas vo. 7 das. CRITERIOS DE INGRESO: Falla de tratamiento por falta de mejora posterior a los 3 dias de tx.
Incapacidad para el seguimiento o tolerancia de los antibiticos orales. Embarazo. Enfermedad grave, fiebre alta, vomito. Urgencia quirrgica que no
puede ser excluida. Abceso tubo-ovarico.
CASO CLINICO ENFERMEDAD PELVICA INFLAMATORIA
Paciente de 19 aos de edad con antecedentes de rara malformacin
uterina y vaginal, consistente en tero doble, el derecho rudimentario,
con hipoplasia cervical y agenesia de vagina. Fue intervenida a los 16
aos, practicando extirpacin de hemitero rudimentario, creacin de
neovagina con piel del abdomen, identificacin de crvix y colocacin
de dispositivo intrauterino (DIU). Se comprob durante la intervencin

CURSO ENARM CMN SIGLO XXI TEL: 36246001

la existencia de ovarios y trompas normales. Posteriormente sigui


controles y presentaba ciclos regulares con menstruaciones normales.
Inicia coitos sin uso de preservativo, practicando controles citolgicos
sin hallazgos patolgicos. El episodio motivo de esta presentacin se
inicia tras la ltima regla, como cuadro de inicio brusco de dolor
hipogstrico, leucorrea y fiebre de 38 C. USG con pioslpinx. Se parti
de una analtica inicial de 25.000 leucocitos/ml, protena C reactiva de

Pharmed Solutions Institute

PGINA 233

MANUAL DE TRABAJO DEL CURSO ENARM CMN SIGLO XXI


63,6 mg/dl y cultivos del DIU positivos para Streptococcus spp.,
Proteus spp. y Escherichia coli, con urocultivo negativo.
PREGUNTA
Cul de los siguientes sntomas clnicos es menos frecuente al
dianostico de esta patologa?
RESPUESTA
a.- Uso de DIU.
b.- Dolor abdominal.
c.- Leucorrea.
d.- Sangrado irregular.
CASO CLINICO ENFERMEDAD PELVICA INFLAMATORIA.
Acude a consulta femenino de 33 aos de edad, refiere que hace 10
das fue diagnosticada con enfermedad inflamatoria, menciona que ha
tomado el tratamiento de forma irregular, regreso debido a que desde
hace 24 horas inicio con dolor abdominal intenso, que se incrementa
cuando camina o hace algn movimiento, fatiga, adinamia, a la
exploracin fsica observa TA 110/70, FC 89, FR 23, Temperatura de
38.5 grados, mal estado generalizado, abdomen con datos de irritacin
peritoneal, decide ingresarla y enva estudios de laboratorio y gabinete
debido a que considera que la paciente presenta una complicacin:
PREGUNTA
Cul es la complicacin ms probable que presenta la paciente?
RESPUESTA
a.- Peritonitis espontanea
b.- Absceso tubo-ovrico
c.- Salpingitis aguda.
d.- Ooforitis aguda.
CASO CLINICO ABSCESO TUBOVARICO
Una mujer de 35 aos de edad, con alcohol crnico y abuso de nicotina
fue admitida con un historial de dos das de evolucin de dolor en fosa
iliaca izquierda, as como las temperaturas febriles de 38.8C y los
parmetros de inflamacin elevados. Una historia previa de
tuberculosis fue negada por el paciente. En un examen por ultrasonido
se observo lesin de 5x6 cm masa qustica en el rea del anexo
izquierdo.
PREGUNTA
Cul es el agente etiolgico mas frecuente?
RESPUESTA
a.- Neisseria g.
b.- Chalmydia.
c.- Mycoplasma.
d.- Ureaplasma .
CASO CLINICO
Paciente de 45 aos que desde hace 3 aos sufre metrorragias, que
han sido tratadas por su gineclogo. Desde hace 5 das sufre fuerte
dolor a nivel del hipocondrio izquierdo, leucorrea leve, dispareunia que
la paciente relaciona con una citologa que le han practicado; desde
ese da ha ingresado varias veces en el servicio de urgencias del
hospital siendo tratada de dicho dolor. Refiere 2 parejas sexuales. A la
exploracin crvix doloroso a la movilizacin y se palpa masa anexial
aprox. de 4cm, sin datos de irritacin peritoneal.
PREGUNTA
Cul es el agente etiolgico ms probable en el diagnstico que se
sospecha en esta paciente?

CURSO ENARM CMN SIGLO XXI TEL: 36246001

RESPUESTA
a.- Neisseria gonorrhoeae
b.- Chlamydia trachomatis
c.- Mycoplasma hominis
d.- Ureaplasma urealyticum
PREGUNTA
En qu clasificacin de Monif se encuentra esta paciente?
REPUESTA
a.- G I leve: no complicada
b.- G II moderada: complicada
c.- G III grave
d.- G IV moderada: no complicada
PREGUNTA
Cul es la conducta diagnostica menos adecuada para este caso?
REPUESTA
a.- Ultrasonido plvico
b.- Biopsia de endometrio
c.- Laparoscopia
d.- Radiografa de pelvis

CASO CLINICO
Una mujer de 19 aos de edad acude a la sala deurgencias por dolor en
el abdomen. Comenta que el dolor es mas intenso en la parte baja del
abdomen. Ha tenido parejas sexuales multiples y rara vez usa condon
desde que toma anticonceptivos orales. Advirtio aumento de dolor con
el coito durante las semanas anteriores. El ultimo periodo menstrual
fue hace 5 dias. En el examen fsico la temperatura es de 39C, la
presin arterial es de 110/70mmHg y la frecuencia cardiaca de 80/min.
El abdomen es blando pero hipersensible al tacto en los cuadrantes
inferiores. El examen plvico revela secrecin purulenta del orificio
cervical e hipersensibilidad con el movimiento del cuello uterino. Una
coloracin Gramm muestra diplococos negativos.
PREGUNTA
Cul es la conducta teraputica mas adecuada a seguir en esta
paciente?
RESPUESTA
a.- Ceftriaxona mas doxiciclina
b.- Clindamicina mas metronidazol
c.- Ceftriaxona mas clindamicina
d.- Doxixiclina mas clindamicina
PREGUNTA
Cul es la complicacin menos probable que pueda presentar esta
paciente?
RESPUESTA
a.- Perihepatitis gonoccica
b.- Dolor plvico crnico
c.- Absceso tuboovarico
d.- Endometriosis
PREGUNTA
Cul es el agente causal ms probable en este caso?
RESPUESTA
a.- Neisseria Gonorrhoeae
b.- Chlamydia Trachomatis
c.- Estreptococos del grupo B
d.- Mycoplasma hominis

Pharmed Solutions Institute

PGINA 234

MANUAL DE TRABAJO DEL CURSO ENARM CMN SIGLO XXI


VIRUS DEL PAPILOMA HUMANO (VPH), DISPLASIA, CANCER CERVICOUTERINO (CACU)
CIENCIAS BASICAS: Los papilomavirus son pequeos virus de DNA, de doble cadena, cuya actividad transformante se explica principalmente por la
actividad de sus oncoprotenas E6 y E7. Estas protenas se unen a un sin nmero de reguladores celulares importantes en el control de procesos
biolgicos como: la apoptosis, proliferacin celular, estabilidad cromosmica, transcripcin de genes (oncogenes y genes supresores de tumor),
diferenciacin celular y la respuesta inmunolgica, entre otros. El VPH se encuentra ampliamente distribuido en todo el mundo y provoca un amplio
espectro de enfermedades epiteliales, desde verrugas a papilomas en los epitelios de distintas mucosas (lesiones preinvasores de crvix, la principal), ya
que poseen un trofismo especfico. Metaplasia: aparicin de epitelio planopoliestratificado en el endocervix, generalmente en respuesta fisiolgica a la
agresin. Ectopia o eritroplasia: Tejido o epitelio cilndrico por debajo del orificio cervical externo, mas frecuente en pacientes con ACO. SALUD PUBLICA:
En Mexico el CACU es la segunda neoplasia mas comn. El VPH est fuertemente asociado al desarrollo de displasia, neoplasia intraepitelial, y cncer del
cuello uterino. Ms del 95% de los cnceres de crvix poseen DNA de VPH de alto riesgo. 1% de las displasias leves evolucionan a invasividad. De 5-15%
de las displasias graves progresa a cncer en 3 aos. El virus puede estar latente hasta 10-15 aos y no ocasionar cambios. El papilomavirus tipo 16 es el
ms prevalente de los VPH oncognicos, responsable de ms de la mitad de los tumores, mientras que el papilomavirus tipo 18 est involucrado en el
20% de los mismos. Factores de riesgo del carcinoma de endometrio: edad>50aos, obesidad, nuliparidad, edad temprana de primer embarazo,
hipercolesterolemia, hipertensin, diabetes. Factores de riesgo del carcinoma de crvix: promiscuidad sexual (>20 parejas sexuales a lo largo de vida),
inicio precoz, relaciones sexuales, ETS, tabaco (muy importante), multiparidad, periodo corto entre la menarca e inicio de vida sexual activa (por mayor
numero de parejas sexuales). PATOGENIA: Dependiendo del tipo de VPH y de la lesin clnica, puede transmitirse por contacto cutneo, relacin sexual,
transmisin perinatal, por escamas infectadas directa o indirectamente, y posiblemente por fmites. El perodo de incubacin vara desde tres semanas a
8 meses, con un promedio de tres meses. En cuanto al genotipo del VPH y su localizacin ms frecuente en piel se tiene: Verruga Vulgar: 2,4,7, verruga
plantar: 1,4, verruga plana: 3, 10, 28, 41, epidermodisplasia Verruciforme: 5, 8, 9, 12. En Mucosas (bajo riesgo): Papilomas anogenitales, cervicales,
orofaringe y tracto respiratorio: 6, 11, 30, 34. En Mucosas (alto riesgo): papilomas anogenitales y cervicales, papulosis Bowenoide, displasia cervical,
orofaringe, cncer cervical y anogenital: 16, 18, 31, 33, 35. Slo un 10% de las infecciones por VPH tiene manifestacin clnica, ya sea en la forma de
verrugas, papilomas o displasias. El virus se replica en el estrato granuloso y es detectado en el estrato crneo, no as en el estrato basal. La infeccin por
VPH no tiene rol oncognico per se, sino que juega un rol que es potenciado por factores fsicos y qumicos. En los carcinomas no se encuentran
partculas virales activas, pero s su DNA y sus genes tempranos. Los productos de los genes E5, E6 y E7 tienen actividad oncognica, ya que sus protenas
estn involucradas en el control del ciclo celular y estimulan la proliferacin o interfieren con la diferenciacin de clulas infectadas. El blanco de estas
protenas virales est representado por las protenas retinoblastoma (Rb), y p53. Normalmente retinoblastoma acta inhibiendo la transcripcin de
genes como c-myc, ras, entre otros que estn encargados de la proliferacin celular, de manera que la inactivacin de Rb provoca una replicacin celular
descontrolada. La p53 se encarga de promover la transcripcin de genes para reparar el DNA daado o inducir apoptosis, de modo que su inactivacin
provoca la prdida de la capacidad de bloquear la proliferacin celular como respuesta al dao del DNA. El resultado es la inestabilidad gentica y el
desarrollo de mutaciones crticas que favorecen el desarrollo de tumores. En la zona de transicin (unin escamocolumnar) de exocervix (plano
poliestratificado) y endocervix (cilndrico) hay dos epitelios, por eso es un lugar ideal, para el virus del VPH. DIAGNOSTICO: Clnica; La principal
manifestacin clnica son las verrugas del rea genital externa y el condiloma acuminado. El condiloma acuminado se presenta como lesiones papulares
con superficie lobulada e irregular, de color rosado oscuro, con prolongaciones digitiformes con aspecto de coliflor. El nmero de lesiones es variable, su
tamao ve de 2mm a 1 cm, pero si son numerosas pueden confluir comprometiendo grandes reas genitales que se traumatizan durante el coito. En la
mujer se ubican con mayor frecuencia en los labios mayores y menores, parte posterior del introito, cltoris, monte de Venus, paredes vaginales y en el
cuello uterino. En el hombre son comunes en la cara interna del prepucio, en el frenillo y en el surco balanoprepucial. Con menor frecuencia se
presentan en el glande y en el meato uretral. Las lesiones perianales y rectales pueden verse en varones homosexuales o pueden ser el resultado de la
propagacin perineal en las mujeres. En personas que practican el sexo oral puede encontrarse condilomas orales. En el embarazo e inmunodeprimidos,
especialmente los transplantados renales las lesiones tienden a ser ms numerosas y exuberantes. El VPH puede infectar el epitelio escamoso de crvix,
vagina, vulva, perin, pene y regin perianal, y determinar lesiones como verrugas genitales, condilomas acuminados, lesiones precancerosas
intraepiteliales y cncer. La deteccin de HPV va de 40 a 90% en las neoplasias intraepitelial (NIE) de bajo grado, a 95% en los pacientes con cncer
invasor. Los VPH tambin han sido implicados en el desarrollo de tumores malignos en sitios distintos a la regin anogenital, como ano, vagina, vulva y
pene e incluso cavidad bucal, pero con una fraccin atribuible considerablemente menor a la del cncer de crvix, en el cual virtualmente el 100% de los
cnceres son causados por VPH. Laboratorio y gabinete: Papanicolaou se utiliza para detectar cambios celulares o clulas anormales en el cuello uterino,
(estas clulas anormales pueden ser precncer o cncer, u otras cosas) es una prueba de screening. Se extraen clulas del cuello uterino y se procesan.
Luego se observan con un microscopio para ver si las clulas son normales o si se pueden observar cambios en ellas. La prueba de Papanicolaou es una
excelente prueba para encontrar clulas cancerosas y clulas que se pudieran convertir en cncer. El resultado anormal ms comn en la prueba de
Papanicolaou se llama ASC-US. Las clulas ASC-US generalmente no indican precncer, pero tampoco son del todo normales. Actualmente con la captura
de hibridos, es ms fcil detectarlo, solo nos dice si hay virus o no. Hasta 7 aos antes, a diferencia del papanicolao normal donde a veces lo
encontramos en NIC I, porque no siempre vemos los coilocitos, este detecta lesion los hibridos no. Las lesiones subclnicas deben ser visualizadas con
colposcopa y con la aplicacin de cido actico al 3-5% que ayuda a delimitar la lesin mediante la reaccin blanco actica. Sin embargo es un examen
de baja especificidad, ya que se altera en otras enfermedades, como por ejemplo el lquen plano, la candidiasis etc. En todas las mujeres con condiloma
acuminado debe realizarse citologa con tcnica de Papanicolau anual. Colposcopia: identifica lesiones sopechosas, delimita la zona de transformacin,
determina extensin de la enfermedad, til para toma de biopsias en zonas sospechosas. Clasificacion por hallazgos colposcopicos: Grado I; insignificante
no sospechoso, epitelio aceto-blanco delgado, vasos no atpicos, distancia intercapilar pequea. Correlacion histolgica: metaplasia escamosa y LIEBG.
Grado II: significativo, sospechoso, epitelio aceto-blanco mayor opacidad y grosor, sin vasos atpicos, distancia intercapilar aumentada. Correlacion
histolgica: NIC II y NIC III. Grado III: Altamente significativo, muy sospechoso, epitelio aceto-blanco grueso irregular y opaco, vasos dilatados irregulares
y atpicos, distancia intercapilar variable, contorno superficial irregular. Correlacion histolgica: LIEAG e invasin temprana. Todas las lesiones que
encontremos por colposcopia se biopsian. Debe biopsiarse verrugas resistentes al tratamiento, atpicas o pigmentadas. CLASIFICACION: Bethesda, para
pronostico y tratamiento: LIEBG (lesin intraepitelial de bajo grado): IVPH y displasia leve (NIC I). LIEAG (lesion intraepitelial del alto grado): NIC II, NIC III.
ASCUS: Clulas que no son claramente displasicas, cambios inflamatorios. 20-50% desarrollan displasia. Se da seguimiento como si se hubiera
descubierto un NIC. TRATAMIENTO: Qumico; Podofilino al 10%-30%-45% en solucin alcohlica, aplicacin por profesional mdico en lesiones de
genitales externos y perianales. La aplicacin debe repetirse semanalmente por 3 a 4 semanas hasta la desaparicin de las lesiones. Si no mejora
considerar otra posibilidad diagnstica o la presencia de una cepa ms agresiva. La aplicacin de grandes cantidades de podofilino puede provocar
toxicidad sistmica. Est contraindicado en el embarazo y la lactancia. Podofilotoxina al 0.5% Puede aplicarse directamente por el paciente dos veces al
da por 3 das. No requiere lavarse como la podofilina. Se descansa 4 das y luego se repite por 3 das ms. Las respuestas se ven habitualmente a las 6
semanas. Su eficacia es similar a la de la podofilina, pero tiene menos toxicidad sistmica. Aproximadamente la mitad de los pacientes muestra algn
grado de inflamacin, quemaduras o erosiones. cido Tricloroactico al 80-90%, aplicacin local por el mdico 2-3 veces por semana por un mximo de

CURSO ENARM CMN SIGLO XXI TEL: 36246001

Pharmed Solutions Institute

PGINA 235

MANUAL DE TRABAJO DEL CURSO ENARM CMN SIGLO XXI


tres semanas. Se forma una erosin que sana en unas 3 semanas sin cicatriz. Puede usarse como terapia combinada con podofilino al 40% en pacientes
inmunodeprimidos y es el tratamiento de eleccin en el embarazo. Tratamientos Fsicos; Crioterapia (Nitrgeno Lquido), cura aproximadamente el 90%
de las lesiones, aunque a veces se requiere varias aplicaciones. Electrociruga Eventualmente desaparecen todas las lesiones, aunque el 20-30%
desarrolla nuevas lesiones en los bordes quirrgicos o en sitios alejados. Extirpacin Quirrgica; 1) Lser. 2) Inmunomoduladores: factor
inmunomodulador inductor de la sntesis de interfern gamma, TNF alfa, IL 1,6,8,10 y factor estimulante de colonias granulocticas. Estimula la
inmunidad celular y carece de actividad antiviral directa in vitro. La presentacin en crema al 5% se utiliza en el tratamiento de los condilomas y verrugas
planas, se aplica cada 48 horas y se deja actuar por 2 a 4 horas. La respuesta se observa a las seis semanas. Es bien tolerado incluso en pacientes
inmunodeprimidos, aunque dos tercios de los pacientes presentan eritema y ardor. Produce curacin en el 60% de los casos y las mujeres responden
mejor que los hombres. 3) Imiquimod (Aldara). Tratamiento especfico: NIC I o LIEBG: Tratar procesos infecciosos concomitantes y repetir papanicolao y
colposcopia, a intervalos de 3-4 meses con examen plvico. Terapia ablativa en personas poco confiables para seguimiento. En pacientes
inmunocompetentes hay remisin hasta en 50%. NIC II, NIC III LIEAG: Terapia ablativa la criociruga elimina lesin hasta 95%, ciruga lser de CO2,
vaporizacin del tejido tratado con xito de 95%, se hace en lesiones que se extienden. Escisin quirrgica con asa diatermica, solo cuando esta
confinada la lesin al crvix. Cono cervical: Se realiza cuando hay cncer in situ, con deseos de conservar la fertilidad, de lo contrario se realizara
histerectoma. La curacin es de 87-97%. Tambin se realiza cuando hay ca microinvasor, menor a 3mm, sin compromiso linfovascular. Con este
procedimiento el seguimiento es ms riguroso, realizar citologa cada 3 meses. Histerectomia: Extrafascial, cuando no est a discusin la fertilidad. En
casos de ca in situ el seguimiento de la citologa es trimestral primer ao, posteriormente anual. LESIONES PREINVASORAS: Abarcan solo epitelio, sin
membrana basal. NIC I, II, III (Neoplasia intraepitelial cervical). DISPLASIA: Cambios morfolgicos celulares, ncleos hipercromaticos, aumento de la
relacin ncleo y citoplasma, mayor ndice mittico. CANCER CERVICOUTERINO: EL CaCu ocupa los primeros lugares como causa de muerte por cncer
en mujeres mexicanas. Constituye el 34.2% de las neoplasias malignas. Una infeccin persistente de virus del papiloma humano (VPH) de tipos virales de
alto riesgo oncognico, es el factor etiolgico principal en el desarrollo de esta neoplasia. Se conoce que solamente una pequea fraccin de lesiones
cervicales infectadas con VPHs de alto riesgo evoluciona a lesiones de alto grado o cncer. Las mujeres sexualmente activas, de cualquier edad, pueden
infectarse con VPHs oncognicos. Sin embargo, el cncer de crvix invasor en mujeres jvenes infectadas con virus oncognicos es raro y la prevalencia
de VPH en mujeres de 40 aos o mayores no se correlaciona con la alta tasa de cncer cervical. Es la persistencia de VPHs oncognicos lo que da lugar al
desarrollo de lesiones precancerosas y potencialmente al cncer invasor, lo que puede llevar varios aos para su desarrollo. El cncer de crvix ocurre en
dos formas predominantes: carcinoma epidermoide y adenocarcinoma, carcinoma adenoescamoso 1.7%, carcinoma verrugoso, carcinoma
indiferenciado de clulas pequeas, tumor carcinoide, melanomas. El tipo histolgico ms comnmente encontrado en las mujeres es el carcinoma
epidermoide (90% de los casos) y est ms frecuentemente asociado al VPH 16. El adenocarcinoma 3.7% es el segundo tipo histolgico ms comn y
aunque el VPH tipo 16 tambin es el ms frecuente, la proporcin de los genotipos 18 y 45 aumenta significativamente en este tipo de tumores. Los
estudios de citologa, que incluyen a la prueba Pap convencional, se utilizan para detectar lesiones precancerosas, pero no es suficiente para detectar
infecciones por VPH. Es claro que, el adenocarcinoma es ms difcil de detectar por Pap que el carcinoma escamoso. Manifestaciones clnicas: sangrado
(lo ms importante), predominantemente poscoital, tambin puede ser intermenstrual y posmenopusico (siempre pensar en cncer cervical y despus
cacu), flujo seroso, ftido ocasionado por la necrosis tumoral, dolor plvico (territorio de nervio citico), edema de extremidades inferiores, dolor
lumbar, pueden presentarse fistulas por infiltracin a vejiga o recto. En estadios avanzados; prdida de peso, anemia, sndrome urmico. Marcadores
tumorales: Antigeno del carcinoma de clulas escamosas, lo podemos encontrar hasta en 50% de los primarios y 75% en los recurrentes, tienen baja
utilidad. El diagnstico de esta neoplasia se realiza con estudio histopatolgico mediante una biopsia dirigida, ya sea mediante colposcopia en caso de no
observarse una lesin o mediante toma directa si existe tumor visible. Los estudios de extensin a realizar en cada caso, depender de la etapa clnica
obtenida en el examen inicial Estadificacion segn la FIGO; ver cuadro anexo. TRATAMIENTO: Recomendaciones de tratamiento en el Instituto Nacional
de Cancerologa 1) Cncer Cervico uterino In Situ; Preservacin de Fertilidad, cono teraputico. Fertilidad satisfecha (<50 aos o premenopausia): cono
teraputico o histerectoma Tipo I con preservacin de anexos. Fertilidad satisfecha (>50 o post menopausia): cono teraputico o histerectoma Tipo I
con salpingo-ooforectoma bilateral. 2) CaCu IA 1; Cono teraputico o histerectoma tipo I. Preservacin de anexos en <50 aos o premenopusica
Salpingo-ooforectomia bilateral en >50 aos o postmenopusica. 3) CaCu IA 2; Histerectoma Radical tipo II. Preservacin de anexos en <50 aos.
Linfadenectoma plvica en caso de PVL. Braquiterapia radioterapia externa a pelvis total (dosis total de 75-80Gy a punto A) en caso de pacientes
mdicamente inoperables. 4) CaCu IB1: Histerectoma Radical tipo II (en tumores menores de 2 cm) y linfadenectoma plvica bilateral. Histerectoma
Radical tipo III y linfadenecto ma plvica. Preservacin de anexos en <50 aos o premenopusicas. En caso de contraindicacin mdico quirrgica la
opcin teraputica es la Radioterapia externa a pelvis total + braquiterapia (dosis total de 80 85Gy a punto A). 5) CaCu IB2IVA. Radioterapia externa a
pelvis total concomitante con quimioterapia basada en cisplatino + braquiterapia (dosis total >85Gy a punto A). Cis platino 40 mg/m2 en forma semanal.
Exenteracin plvica para pacientes con fstula vesico-vaginal y/o recto-vaginal sin infiltracin a la pared plvica y con ECOG I y K ms del 90%. Nota: En
caso de paciente con hidronefrosis co locacin de catter JJ o Catter de nefrostoma antes de iniciar el tratamiento con radioterapia. Con fstula vsicovaginal no exenterables derivacin urinaria. Con fstula recto vaginal no exenterable colostoma. 6) Ca Cu II, III, IVB; Tratamiento sistmico paliativo,
radioterapia paliativa exclusiva, superviviencia a 5 aos 65-75%, 30-50% y 20% respectivamente. Vigilancia: 80-90% presentan recaidas en los 2 primeros
aos. Exploracin plvica y citolgica; mensual o bimensual el primer ao. Cada 2-3 meses el segundo ao. Cada 4-5 meses el tercer ao. Cada 6 meses el
cuarto ao y quinto ao. Cada ao a partir del sexto ao. PAPULOSIS BOWENOIDE (PB): Es una forma de cncer espinocelular in situ asociado al VPH. Se
caracteriza por mltiples ppulas asintomticas, ligeramente solevantadas, de 2 a 20 mm que pueden confluir formando placas de color marrn violceo
y con tenue descamacin, ubicadas en la base del pene, vulva y perin en pacientes jvenes. Se considera un factor de alto riesgo de desarrollo de cncer
de crvix. Su curso es habitualmente benigno. CONDILOMA ACUMINADO GIGANTE: o tumor de Buschke-Lowenstein es una lesin precancerosa. Se
caracteriza por condilomas de gran tamao que causan dao tisular local. Son ms frecuentes en la ingle, regin perianal y en el surco balanoprepucial.
No metastisa, se maneja con ciruga y crioterapia y recurre con frecuencia. Si se detecta la presencia de VPH 16 o 18 debe considerarse la progresin a la
malignidad.
CASO CLINICO DISPLASIA CERVICAL
Mujer de 29 aos de edad la cual refiere sangrado poscoital la cual ha
resultado levemente dolorosa, como antecedentes presento menarca
a los 9 aos, IVSA a los 14 aos, ha tenido 4 parejas sexuales, es
usuaria de mtodo hormonal oral, agrega que ha presentado varios
cuadros de cervicovaginitis, la ultima DOC presento cambios
inflamatorios, tabaquismo y alcoholismo positivo. A la exploracin
ginecolgica se observa crvix afresado, doloroso a la movilizacin.

CURSO ENARM CMN SIGLO XXI TEL: 36246001

PREGUNTA
Cul es la conducta a seguir?
RESPUESTA
a.- Repetir DOC.
b.- Realizar colposcopia.
c.- Enviar a clnica de displasias.
d.- Realizar citologa de base liquida.

Pharmed Solutions Institute

PGINA 236

MANUAL DE TRABAJO DEL CURSO ENARM CMN SIGLO XXI


CASO CLINICO
Se trata de una paciente femenina de 22 aos de edad, con
antecedentes de salud aparente (no enfermedades crnicas, no
transmisibles), es miembro de una familia disfuncional, madre
alcohlica y padre alcohlico, consumidor de opiceos. Antecedentes
de una vida sexual activa y desorganizada, con cambio frecuentes de
parejas y relaciones fortuitas con desconocidos sin ninguna proteccin,
que refiere que desde hace aproximadamente 3 das se palpa algo
extrao en sus genitales, suave y hmedo, de color rosado, indolora
(referido por la paciente), adems refiere mucho escozor y flujo
vaginal abundante de color blanquecino.

c.- Histerectoma tipo I con preservacin de anexos


d.- Histerectomia radical tipo II

PREGUNTA
Qu serotipos de VPH, nos pueden causar verrugas genitales y son de
bajo riesgo?
RESPUESTA
a.- VPH 3, VPH, 10
b.- VPH 11, VPH 16
c.- VPH 46, VPH 18
d.- VPH 16, VPH 28

PREGUNTA
Qu tipo histolgico sera el ms probable y comn en este caso?
RESPUESTA
a.- Epidermoide
b.- Clulas escamosas
c.- Papulosis Bowenoide
d.- Adenocarcinoma

PREGUNTA
Se le realiza citologa vaginal con tcnica de papanicolao a la paciente y
reporta clulas AS-CUS. Cul es la interpretacin de las mismas?
RESPUESTA
a.- Clulas precancerosas, cambios inflamatorios
b.- Clulas de displasia leve, lesin intraepitelial de bajo grado
c.- Clulas que no son claramente displasicas, cambios inflamatorios
d.- Clulas de displasia moderada, lesin intraepitelial de alto grado
PREGUNTA
Qu estudio complementario sera el ms adecuado de realizar a la
paciente?
RESPUESTA
a.- Toma de biopsia de lesin
b.- Colposcopia y aplicacin de cido actico
c.- Cultivo cervicovaginal
d.- Toma de biopsia de crvix
CASO CLINICO
Mujer de 38 aos de edad. Gesta 6, Para 5, Aborto 1. Es referida por
presentar desde hace un mes sangrado genital de color rojo oscuro y
olor ftido. De sus antecedentes destaca inicio de vida coital a los 16
aos, tres compaeros sexuales y antecedente de ta materna con
cncer de cuello uterino. nica citologa cervical hace 12 aos y
enfermedad plvica inflamatoria hace cuatro aos. A la exploracin
clnica general se evidencia paciente en buenas condiciones generales,
afebril, hidratada, eupneica, consciente y piel plida. Al examen se
evidencia una lesin sangrante en labio anterior del cuello uterino, de
6x6 cm, clnicamente sospechosa de malignidad. Se toma muestra para
citologa crvico-vaginal y biopsia dirigida por colposcopia. Se solicitan
ecograma obsttrico y exmenes paraclnicos. La paciente refiere
paridad satisfecha.
PREGUNTA
Con los estudios realizados se diagnostic carcinoma in situ. Cul es la
conducta teraputica ms adecuada para este caso?
RESPUESTA
a.- Cono teraputico
b.- Histerectoma tipo I con salpingooforectoma bilateral

CURSO ENARM CMN SIGLO XXI TEL: 36246001

PREGUNTA
Para vigilancia de esta paciente, la exploracin plvica y citologa
vaginal el primer ao se realiza cada
RESPUESTA
a.- Cada 3 meses
b.- Cada 4 meses
c.- Cada 6 meses
d.- Cada 10 meses

CASO CLINICO
Femenino de 25 aos. Maestra. AHF: Abuela materna cncer de mama
a los 60 aos. AGO. M: 12 aos, IVSA 23 aos, G 0, P 0. Padecimiento
actual: 6 meses de evolucin con distensin abdominal y dolor
ocasional de poca intensidad que se incrementa al sentarse, con
alternancia de polihipermenorrea y amenorrea desde hace 1 ao. EF:
Buenas condiciones generales, cardiopulmonar sin compromiso,
abdomen con dolor en fosa iliaca izquierda a la palpacin media y
profunda. Se palpa tumor de 8 cm dependiente de anexo izquierdo, sin
ascitis ni implantes en fondos de saco. Se realiz laparotoma
exploradora con ooforectoma izquierda. La anatoma patolgica
inform disgerminoma.
PREGUNTA
Cul es la conducta teraputica ms adecuada, si la paciente desea
conservar la fertilidad?
RESPUESTA
a.- Radioterapia
b.- laparotomia
c.- Quimioterapia y cono
d.- Vigilancia
PREGUNTA
Qu marcadores tumorales seran los ms adecuados de solicitar en
este caso?
RESPUESTA
a.- AFP, LDH
b.- CA 125, hCG
c.- AFP, hCG
d.- AFP, CA 125
PREGUNTA
Con que trastorno hidroelectroltico es ms probable encontrar en
esta paciente?
RESPUESTA
a.- Hipernatremia
b.- Hiperpotasemia
c.- Hipercalcemia
d.- Hiponatremia

Pharmed Solutions Institute

PGINA 237

MANUAL DE TRABAJO DEL CURSO ENARM CMN SIGLO XXI


CANCER OVARICO
CIENCIAS BASICAS: Entre los cnceres del tracto genital femenino, el de ovario es el tercero en frecuencia por detrs del cncer de crvix y de
endometrio, pero es responsable de casi la mitad de las muertes por cncer genital, debido a la dificultad para el diagnstico precoz. El ovario puede
originar tumores histolgicamente muy diferentes y siempre ha sido un problema su clasificacin porque la patogenia est menos clara que en otros
tumores. Factores de riesgo: Adquieren cada vez ms importancia los antecedentes familiares y las mutaciones hereditarias como las del BRCA1 y BRCA2,
igualmente se ha descrito concentraciones elevadas del oncogn HER2/neu y mutaciones en el p53, hiperestrogenismo, uso de anticonceptivos,
disminuye con la paridad. SALUD PUBLICA: Los tumores ovricos constituyen el tercer grupo de tumores en la mujer. Aproximadamente 1 de cada 10
fminas lo tendr a lo largo de la vida, la mayora de las cuales requerir de una evaluacin quirrgica. Estos se presentan desde edades tempranas hasta
avanzadas. La experiencia clnica revela la alta incidencia en la etapa del climaterio, comprendida entre los 35 y 65 aos de edad. CLASIFICACIN: La
OMS clasifica los tumores ovricos de acuerdo con el tejido de origen. Ver cuadro anexo. Es necesario puntualizar que los tumores borderline o
tumores en el lmite de malignidad histolgica, son formas tumorales con entidad propia, no un estadio evolutivo. Histolgicamente se diferencian de los
benignos por: estratificacin epitelial, aumento de la actividad mittica, atpia nuclear con ausencia de invasin estromal, y porque pueden metastatizar.
PATOGENIA: En los ovarios pueden desarrollarse tumores benignos o malignos, qusticos o slidos, de tipo epitelial, conjuntivo, funcionantes o no o
teratomatosos. Los tumores ms frecuentes no son neoplsicos, sino que derivan del desarrollo folicular; son formaciones qusticas originadas por un
estmulo anormal del folculo o alteraciones en el proceso de involucin. Los verdaderos procesos neoplsicos presentan una constitucin histolgica
muy diversa, de origen epitelial, le continan los procedentes del estroma gonadal y, finalmente, los tumores de las clulas germinales. Se conoce que
los tumores benignos del ovario no constituyen un grupo bien definido y algunos de ellos pueden malignizarse en su evolucin. Aproximadamente de 75
a 85 % de los tumores son en principio benignos. Hoy da, los factores causales del cncer de ovario son desconocidos, aunque se plantea que en el
mbito celular es el resultado de la acumulacin de mltiples defectos genticos menores. Los estudios epidemiolgicos han permitido enunciar 3
teoras con respecto a las causas de esta enfermedad: La divisin celular ininterrumpida y la continua regeneracin del epitelio ovrico con cada
ovulacin proporcionan la oportunidad para una mutacin y transformacin maligna por la estimulacin de los ovarios y por hormonas gonadotrpicas,
que de forma mantenida pueden inducir a la malignizacin. La exposicin del ovario a diferentes agentes carcinogenticos. Los eventos moleculares que
dirigen el desarrollo del cncer epitelial de ovario son desconocidos. A pesar de esto, en investigaciones epidemiolgicas se han identificado algunos
factores causales en la carcinognesis del cncer de ovario: factores endocrinos, ambientales y genticos, aunque se incluyen otros, tales como:
nuliparidad, historia familiar, menarquia temprana, menopausia tarda, raza blanca y aumento de la edad. La incidencia especfica aumenta segn la
edad de la paciente, con un pico en la sexta dcada de la vida. Es infrecuente en mujeres menores de 40 aos. El riesgo de que se presente esta
enfermedad disminuye con la paridad, con el uso de contraceptivos hormonales y la lactancia; tambin parece que la histerectoma o la ligadura de
trompas con conservacin de los ovarios disminuyen la posibilidad de aparicin. El cncer de ovario con historia familiar constituye apenas 2,5 % del
total. Al respecto, se han identificado 3 tipos de cnceres hereditarios, a saber: 1. Cncer en sitio especfico: Existe alto riesgo si hay cncer en 2
familiares de primer grado (50 % de probabilidades de ser afectado). 2. Sndrome de cncer familiar mama-ovario: Patrn de transmisin dominante;
asociado a los cromosomas 16q y 17q. Se considera este sndrome cuando hay 5 casos de cncer de mama y ovario en la familia, incluyendo 2 de cada
uno de estos 3. Cncer colorrectal no polipsico (sndrome de Lynch de tipo II): Incluye mltiples adenocarcinomas, cncer de colon familiar, de ovario,
endometrio, mama, as como de los sistemas gastrointestinal y urinario. DIAGNOSTICO: Entre los datos clnicos de sospecha figuran: crecimiento lento
del abdomen, sntomas compresivos, dolor abdominal agudo o subagudo, sobre todo cuando se complican por torsin (frecuente en los benignos),
rotura, hemorragia o infeccin, rara vez datos indirectos de actividad especial endocrina o metablica, tales como: pubertad precoz, hemorragia uterina
disfuncional, amenorrea, hirsutismo, tirotoxicosis, sndrome de Cushing, galactorrea, policitemia, hipoglucemia e hipercalcemia, por citar algunos. Por lo
general, los sntomas que ms refieren las pacientes incluyen cierto malestar abdominal, seguido de distensin abdominal, a causa de la presencia de
ascitis o una masa tumoral que paulatinamente aumenta de tamao, as como tambin sntomas gastrointestinales (como nuseas, dispepsias, saciedad
precoz y constipacin); los urinarios son menos frecuentes, al igual que la metrorragia. Estos sntomas tardos ocurren con ms frecuencia en mujeres
premenopusicas. Es difcil de diagnosticar en etapas tempranas, pues las manifestaciones clnicas son escasas y fcilmente se confunden con las de
otras afecciones, adems, no es fcil crear un sistema de deteccin precoz (tamizaje).
CASO CLINICO CANCER DE OVARIO
Se trata de paciente femenino de 51 aos de edad la cual es originaria
del estado de Mxico, comerciante ambulante, cuenta con
antecedentes de familiares con cncer que ya murieron, no especifica
qu tipo de cncer, acude a consulta debido a que desde hace varios
meses ha presentado prdida de peso de ms de 15 % total, agrega
fatiga, malestar generalizado, refiere que se realiza deteccin
oportuna de cncer mediante papanicolao hace dos aos sin datos de
importancia y deteccin de cncer de mama sin datos patolgicos,
acude a urgencias debido a que se encuentra con distencin
abdominal importante, se realiza paracentesis donde se encuentra
datos sugerentes de adenocarcinoma, se realiza rastreo tomografico
de mama y pelvis sin encontrar datos patolgicos, sin embargo se
encuentra CA-125 elevada, posteriormente se toma biopsia dirigida a
ovarios con reporte de patologa positivo para cncer ovrico.
PREGUNTA
Cul es la conducta ms apropiada para seguir?
RESPUESTA
a.- Cirugia citorreductora.
b.- Cirugia citorreductora mas cisplatino y paclitaxel.
c.- Mastectomia bilateral y ooforectomia bilateral.
d.- Mastectomia y ooforectomia bilateral mas tamoxifen y radiacin
CASO CLINICO
Una mujer de 55 aos de edad posmenopusica, G1, P1, acude con su
medico de atencin primaria por experimentar fatiga, molestias

CURSO ENARM CMN SIGLO XXI TEL: 36246001

abdominales y meteorismo. Dice tener nausea y anorexia leves, pero


no vomitos. En el examen fsico la temperatura es de 38C, FC 85 lpm,
FR 18 rpm, y la TA 130/80mmHg. El examen del abdomen revela
distensin, matidez cambiante, una onda de liquido e hipersensibilidad
leve al tacto, sin masas. No se encuentra nada obvio en exmenes de
cabeza, cuello, cardiovascular, trax y musculoesqueletico. El examen
plvico revela genitales externos y cuello uterino normales, pero se
palpa una masa nodular fija en los anexos izquierdos. La ecografa
transvaginal revela una masa de 5 cm con elementos qusticos y
solidos en el ovario izquierdo y liquido en el fondo de saco. No se
aprecian anomalas en ovario derecho.
PREGUNTA
Cul es el diagnostico mas probable en este caso?
RESPUESTA
a.- Quiste tuboovarico
b.- Absceso tuboovarico
d.- Cancer de ovario
c.- Endometriosis de ovario
PREGUNTA
Cul es el factor de riesgo menos probable para esta patologa?
RESPUESTA
a.- Nuliparidad
b.- Uso de anticonceptivos orales
c.- Procreacion retrasada

Pharmed Solutions Institute

PGINA 238

MANUAL DE TRABAJO DEL CURSO ENARM CMN SIGLO XXI


SANGRADO UTERINO DISFUNCIONAL (HUD)
CIENCIAS BASICAS: La hemorragia uterina anormal es la causa ms comn de prdida hemtica en la mujer en edad reproductiva. Las mujeres con
hemorragia pueden padecer anemia crnica, dolor plvico e incapacidad, enfrentando de tal forma un problema mdico debilitante que afecta de
manera adversa sus responsabilidades laborales y familiares. HUD: se define como aquella en la
que se producen cambios en la frecuencia del ciclo menstrual, en su duracin, o en la cantidad de la
prdida sangunea; su diagnstico es de exclusin, lo que obliga al clnico a descartar inicialmente
alguna patologa orgnica (Tabla 1). sta puede catalogarse en 2 grandes rubros: la debida a causas
orgnicas (hemorragia uterina anormal) y la que es producida por anovulacin (hemorragia uterina
disfuncional). Las principales causas de hemorragia uterina disfuncional o anovulatoria se enlistan
en la Tabla 2. SALUD PUBLICA: HUD, es la segunda causa de consulta ginecolgica, despus de la
infecciones cervicovaginales, es la principal causa de hemorragia en la mujer adulta. Se estima que
alrededor de 10 millones de mujeres en Mxico sufren hemorragia uterina y anualmente, solo 6
millones de ellas buscan atencin mdica. CLASIFICACION: Por la FIGO sistema PALM-COEIN: El
sistema bsico engloba cuatro categoras que se definen por criterios estructurales visualmente
objetivos (PALM: plipos, adenomiosis, leiomioma y malignidad o hiperplasia); cuatro afecciones no estructurales (COEI: coagulopatia, disfuncin
ovulatoria, disfuncin endometrial, iatrognica), que no estn relacionadas con anomalas estructurales y una (N), reservada para las afecciones no
clasificables. La categora leiomioma (L), se subdivide en las pacientes que tienen al menos un mioma submucoso y que tienen miomas que no tienen
efecto en la cavidad endometrial. PATOGENIA: En relacin a su fisiopatologa, en un ciclo anovulatorio el cuerpo lteo no se desarrolla, el ovario pierde
su capacidad para producir progesterona pero la produccin estrognica contina; esta situacin trae
como consecuencia una proliferacin endometrial sin una descamacin inducida por progesterona
que culmine en una menstruacin. El resultado clnico de esta eventualidad es una hemorragia no
cclica, impredecible e inconsistente en cuanto a volumen. La estimulacin estrognica continua y sin
oposicin (de la progesterona) produce un crecimiento endometrial inestable excesivamente
vascular, sin la suficiente capa estromal que lo soporte, tornndolo frgil y vulnerable; en este
contexto la descamacin endometrial es irregular, prolongada e impredecible. En el endometrio de
las mujeres con hemorragia uterina se han encontrado grandes cantidades de prostaglandinas (PGE2
y PGF2) cuando se les compar con mujeres con ciclos menstruales regulares. Incluso existe
evidencia deque en los trastornos de la homeostasis (coagulacin anormal) la proporcin de
PGE2/PGF2 y la de prostaciclina (PGI2)/tromboxano (TXA2) estn elevadas. Estas prostaglandinas
estn presentes, tanto en el endometrio como en el miometrio, y el mecanismo exacto por el cual
producen prdida sangunea elevada an es especulativo. DIAGNOSTICO: La hemorragia uterina
disfuncional es un diagnstico de exclusin, por lo cual el clnico debe descartar inicialmente cualquier patologa orgnica o endocrinolgica. Para una
adecuada evaluacin clnica conviene estratificar por edad a las pacientes, debido a que las de mayor edad incrementan el riesgo de patologas malignas
o premalignas. EDAD REPRODUCTIVA (19 A 39 AOS DE EDAD): Aproximadamente, entre un 6 a 10% de las mujeres con HUD tienen hiperandrogenismo
con anovulacin crnica (sndrome de ovarios poliqusticos), lo cual incluye trastornos en el ciclo menstrual, hirsutismo y obesidad (IMC>25 kg/m2). El
65% de las mujeres con hirsutismo y anovulacin crnica son obesas. En nuestro pas, el 37.4% de las mujeres tienen sobrepeso y el 34.5% obesidad, por
lo que al sumar ambas prevalencias, tenemos un 71.9% de mujeres de 20 aos y mayores (esto es en mujeres en edad reproductiva) con trastornos en la
alimentacin. En las mujeres con obesidad, irregularidades menstruales (oligo u anovulacin) y datos de hiperandrogenismo deber descartarse
sndrome de ovarios poliqusticos. En mujeres con una rpida progresin de hirsutismo acompaada de virilizacin, debe sugerir al clnico descartar un
tumor suprarrenal. En la mayora de los casos, la cuantificacin de los niveles de testosterona, de sulfato de dehidroepiandrosterona y de 17hidroxiprogesterona, puede guiarnos al diagnstico. La evaluacin debe considerar tambin la valoracin de biometra hemtica, una prueba de
embarazo, de los niveles de prolactina y de la hormona estimulante del tiroides (TSH). La evaluacin debe considerar tambin la existencia de embarazo,
hiperprolactinemia y trastornos Tiroideos. Cuando se sospecha de falla ovrica precoz la estimacin de los niveles de FSH sern de utilidad. La
anovulacin es la causa ms frecuente de amenorrea en las mujeres que experimentan amenorrea secundaria. La anovulacin crnica que resulta de una
disfuncin hipotalmica se diagnostica por niveles bajos o normales de FSH, y puede ser el resultado de estrs fisiolgico, ejercicio en exceso o prdida
de peso. A las mujeres con amenorrea que presentan una prueba negativa de embarazo, niveles normales de FSH, TSH y prolactina podemos catalogarlas
como anovulacin. MUJERES EN EDAD REPRODUCTIVA TARDA (DE LOS 40 AOS HASTA LA MENOPAUSIA): La incidencia de HUD se incrementa de
manera paralela con la edad, y los ciclos anovulatorios continuos representan la declinacin de la funcin ovrica. En estas mujeres, la causa ms
frecuente de hemorragia uterina no es precisamente la hiperplasia endometrial ni el cncer endometrial, sino las patologas intracavitarias como plipos
endometriales y miomas submucosos. En las mujeres de esta edad con HUD a las cuales se les ha descartado inicialmente alguna causa orgnica, y que
persisten con episodios de hemorragia a pesar de haberse instaurado un tratamiento adecuado, debern revalorarse en la bsqueda de causas malignas
o premalignas. El clnico no debe olvidar la estrecha relacin de algunos de los parmetros clnicos presentes en la mujer con HUD, como la obesidad y la
anovulacin en la gnesis del cncer endometrial. La biopsia de endometrio es una tcnica sencilla, relativamente simple y que puede realizarse en el
consultorio; las pacientes menores a 35 aos de edad con HUD que no respondan a la terapia mdica instaurada son candidatas a la biopsia de
endometrio y las mujeres mayores de 40 aos de edad y HUD debern someterse a una biopsia de endometrio. La histeroscopia de consultorio es una
tcnica de visualizacin directa que detecta un alto porcentaje de anormalidades intracavitarias. Ultrasonido transvaginal (UTV), la sonohisterografa
(SHG) y la histeroscopia de consultorio en la deteccin de lesiones intracavitarias, se reportan una sensibilidad y especificidad del UTV en un 56.3 y 100%;
para la SHG 72% y 87.5%; finalmente, para la histerosopia un 100% y 100%. Los autores concluyen que la certeza diagnstica de la SHG es equiparable a
la histeroscopia. Sin embargo, se reporta menos dolor con la SHG que con la histeroscopia. Desafortunadamente, es una tcnica que requiere
entrenamiento, y el instrumental necesario para su realizacin es caro. El objetivo de la evaluacin de la cavidad uterina en las mujeres con HUD incluye
la deteccin de lesiones focales (miomas o adenomiosis) y alteraciones a nivel endometrial (engrosamiento endometrial o plipos). El ultrasonido
transvaginal (UTV) es una herramienta til para el diagnstico de lesiones focales como miomas, ya que logra detectar alguna patologa hasta en un 97%
de los casos en mujeres premenopusicas, sin embargo, pierde certeza diagnstica en lesiones intracavitarias. La evaluacin por UTV del endometrio que
resulte en un grosor mayor de 18 mm sugiere, fuertemente, alguna patologa a este nivel. TRATAMIENTO: No existe evidencia suficiente para afirmar
que el uso de anticonceptivos orales (AOC), solos o comparados con otros tratamientos (AINES, danazol, DIU) sean benficos para la mujer con HUD en
relacin a la mejora de su sintomatologa. No existen ventajas con el uso de progestinas para el control de HUD si se les compara con danazol, AINES o
con el dispositivo impregnado con levonorgestrel; sin embargo, el uso de progesterona por 21 das parece reducir de manera significativa la prdida
sangunea en pacientes con HUD, por lo que este rgimen se puede administrar de manera inmediata para el control de la hemorragia y solo como un

CURSO ENARM CMN SIGLO XXI TEL: 36246001

Pharmed Solutions Institute

PGINA 239

MANUAL DE TRABAJO DEL CURSO ENARM CMN SIGLO XXI


tratamiento temporal. Es insuficiente la evidencia para sealar que el uso de DIU-IL sea la mejor opcin, en comparacin con el uso de noretindrona
continua en las mujeres con HUD; pero se reportarn mayores efectos adversos en el grupo de DIU-IL. El danazol es una mejor alternativa para el
tratamiento de HUD, en comparacin a placebo, progestinas, AINES y AHO, pero presentan mayores efectos adversos. Debido al pequeo nmero de
trabajos, no es posible emitir una recomendacin adecuada para su uso en la prctica clnica en las pacientes con HUD. Tratamiento quirrgico: El
legrado uterino instrumentado es la intervencin diagnstica o teraputica ms empleada en nuestro medio, no cuenta con la suficiente evidencia
cientfica sobre su utilidad, sola o comparada contra intervenciones. La ablacin endometrial solo est indicada en mujeres con paridad satisfecha y que
no desean histerectoma total abdominal. Si se le compara con el tratamiento mdico, resulta ser significativamente ms efectiva para controlar la
hemorragia a 4 meses de seguimiento, sin evidencia de efectividad a ms largo plazo. No hay diferencia en un ao respecto a la calidad de vida, al
compararse con el DIU-IL. Para las mujeres que tienen satisfecho su deseo reproductivo, y en las que se han utilizado las diferentes opciones teraputicas
(tanto mdicas como quirrgicas) sin haberse encontrado una respuesta satisfactoria a su problema, la histerectoma representa la mejor opcin al ser
curativa y mejorar la calidad de vida, aunque tenga mayor riesgo de complicaciones.
CASO CLINICO SANGRADO UTERINO
Mujer de 27 aos con antecedentes de migraa y colecistectoma
laparoscpica 2 aos antes, presenta alteraciones en la cantidad y
frecuencia del ciclo menstrual asi como flujo intermenstrual
recurrente, como antecedentes la paciente le fue realizada cesarea por
placenta de implantacin baja total hace 6 meses, hay antecedentes
familiares de cncer endometrial y quistes ovricos.
PREGUNTA
Cual es la conducta a seguir para establer un escrutinio diferencial?
RESPUESTA
a.- USG enfocndose en anexos.
b.- Realizar PIE.
c.- Realizar evaluacin de gonadotrofina.
d.- Biopsia endometrial.
CASO CLINICO SANGRADO UTERINO
Paciente de 26 aos de edad se presenta a la consulta refiriendo que
desde hace un ao posterior a un parto via vaginal inicia padecimiento
caracterizado por alteraciones de su periodo mentrual, refiere que
anteriormente era regular pero ahora no, adems agrega infecciones
vaginales frecuentes, es usuaria de DIU su DOC fue normal a la
exploracin presenta leve dolor a la movilizacin de crvix.
PREGUNTA
Cul es la conducta a seguir?
RESPUESTA
a.- Iniciar mtodo de barrera.
b.- Realizar USG.
c.- Retirar DIU.
d.- Envio a colposcopia.
CASO CLINICO
Femenino de 52 aos, casada, ama de casa. Madre con hipertensin
arterial sistmica. Padre finado por cardiopata no especificada. Abuela
materna finada por cncer de mama. Consumo de tabaco positivo 20
cigarrillos diarios durante 30 aos, suspendido hace 6 aos. AGO:
Menarca; 9 aos, ritmo 28x8, IVSA; 19 aos. G: 6, P: 2, A: 1, C: 3.
ltima citologa vaginal hace 9 meses, reportada como sin datos
patolgicos. Mastografa hace 2 aos, solo recuerda fue negativa para
malignidad. Inici su padecimiento actual hace 10 aos, tiempo en el
que ha presentado ciclos menstruales irregulares, con amenorrea de
hasta 6 meses y sangrados uterinos anormales en varias ocasiones, de
tipo hipermenorrea, acompaados de dolor abdominal en hipogastrio,
tipo clico, sin irradiaciones, exacerbantes o atenuantes, de intensidad
mxima 6/10, con cese del dolor y de las hemorragias de forma
espontnea tras 8 das de sangrado. Cuenta con ecografa que reporta,
tero en anteversin, de 9.5 X 7.0 X 5.5 cm, con gran ndulo subseroso
localizado en cara posterior del cuerpo uterino, es hipoecgenico,
mide aprox: 5.1x4.3x3.8 cm. Se visualiz otro ndulo, este intramural,
de menor tamao, en fundus uterino.
PREGUNTA
Cul es el diagnostico menos probable para esta paciente?

CURSO ENARM CMN SIGLO XXI TEL: 36246001

RESPUESTA
a.- Plipos endometriales
b.- Cncer de cuerpo uterino
c.- Miomatosis uterina
d.- Endometritis crnica
PREGUNTA
La paciente se niega a procedimiento definitivo. Cul es la opcin
teraputica ms adecuada en este caso?
RESPUESTA
a.- Anlogos de la GnRh
b.- Levonorgestrel in situ
c.- Medroxiprogesterona
d.- Acetato de leuprolide
PREGUNTA
Cul de las siguientes no es un factor de riesgo que aumente la
exposicin a estrgenos para desarrollar miomatosis uterina?
RESPUESTA
a.- Menarca temprana
b.- Menopausia tarda
c.- Tiempo prolongado de hormonales orales
d.- Fumar
CASO CLINICO
Una mujer de 70 aos de edad, G3, P3, informa de dos meses de
manchado vaginal intermitente sin dolor. Paso por la menopausia a los
52 aos de edad y uso hormonoterapia de reemplazo hasta hace 3
aos. El antecedente personal patolgico de importancia es de
diabetes tipo 2. Mide 1.70mts y pesa 82 Kg.En el examen fsico esta
afebril, con frecuencia cardiaca de 75/min y presin arterial de
135/80mmHg. En el examen con especulo los genitales externos son
normales, mientras que las paredes de la vagina estn resecas y
atrficas. No existen lesiones en el cuello uterino y se observa una gota
de sangre en el orificio. Serealiza ecografa intravaginal en el
consultorio que revela una lnea endometrial de 7 mm, asi como frotis
y biopsia endometrial las cuales se envan a patologa.
PREGUNTA
Cul es el diagnostico mas probable para esta paciente?
RESPUESTA
a.- Carcinoma endometrial
b.- Hiperplasia endometrial
c.- Miomatosis uterina
d.- Polipos benignos
PREGUNTA
Cul es el factor de riesgo mas importante para este caso?
RESPUESTA
a.- Diabetes mellitus
b.- Hormonoterapia de 240estitucin
c.- Obesidad
d.- SOP

Pharmed Solutions Institute

PGINA 240

MANUAL DE TRABAJO DEL CURSO ENARM CMN SIGLO XXI


MIOMATOSIS UTERINA
CIENCIAS BASICAS: Los miomas son una enfermedad benigna comn del tero, son tumores que se inician en una sola clula de msculo liso uterino y
pueden crecer en cualquier parte del tero bajo la influencia de factores de crecimiento local, ctocinas y hormonas sexuales; ocasionan trastorno a nivel
endometrial. La miomatosis uterina es la presencia de uno o varios tumores benignos formados por fibras musculares del tero (matriz) y se considera la
tumoracin ms frecuente del aparato genital femenino. Tambin se le denomina leiomiomas o fibromas uterinos. SALUD PUBLICA: La prevalencia de la
miomatosis uterina en mujeres en edad frtil se calcula entre 20 y 40%, siendo ms comn en mujeres afroamericanas, y contando como factores de
riesgo la edad, nuliparidad, tabaquismo y otros. Su incidencia acumulada para la edad de 50 aos es de ms del 80% en mujeres afroamericanas y 70% en
mujeres caucsicas. Se han asociado con infertilidad en 5 a 10% de los casos, y se estima que se encuentran como factor nico presente en 1 a 3% de los
casos. CLASIFICACION: Se ha clasificado a los miomas uterinos como submucosos si distorsionan la cavidad uterina, intramurales si residen
predominantemente dentro de la pared miometrial uterina y subserosos si protruyen fuera de la superficie uterina. El nmero y localizacin de los
miomas correlaciona con los sntomas y el efecto en la fertilidad. Existen otras localizaciones poco comunes: cervicales (en el cuello de la matriz),
intraligamentarios (dentro de los ligamentos que sostienen al tero) o incluso miomas parsitos, que son aqullos que al ir creciendo van tomando el
aporte sanguneo de estructuras vecinas, pudiendo incluso llegar a quedar completamente separados del tero. PATOGENIA: Predisposicin del sitio
anatmico a formar miomas. Se cree que son causados por una excesiva estimulacin de los estrgenos, que actan sobre una o varias clulas de las
fibras musculares del tero susceptibles a responder en forma exagerada, dando lugar a la proliferacin desmedida y la formacin de tumores. Se piensa
que es un tumor estrgeno-dependiente, ya que crece durante la edad reproductiva y en ocasiones revierte o disminuye en la menopausia cuando ya no
hay el estmulo estrognico. El uso de hormonales orales en forma constante incrementa su desarrollo DIAGNOSTICO: Depender en gran medida de la
localizacin, tamao y nmero de los miomas: Subserosos: Su sintomatologa se relaciona con la compresin a rganos vecinos. Si comprimen la vejiga,
pueden dar sntomas de infeccin urinaria (ardor al orinar, miccin frecuente y en poca cantidad, etc.), o incluso incapacidad para contener la orina. Si
comprimenal intestino grueso podrn ocasionar constipacin, colitis, dolor durante la evacuacin o incluso oclusin intestinal. La compresin sobre las
arterias o venas de la pelvis condicionar el desarrollo de varicosidades, inflamacin de miembros inferiores o sensacin de dolor y de pesantez en las
piernas. Intramurales: Interfieren con la contractilidad uterina, comprimen los plexos venosos y arteriales y, a la larga, condicionan la presencia de
menstruaciones prolongadas y abundantes, sangrado nter menstrual y dolor durante la menstruacin, adems de estar relacionados en muchos casos
con esterilidad. Submucosos: Deforman el interior de la cavidad uterina y son los que con mayor frecuencia ocasionan aumento en el sangrado
menstrual (hiper-poli menorrea) adems de clicos. Son un importante factor causal de esterilidad y pueden crecer tanto que salen a travs del cuello
uterino (miomas abortados). La exploracin bimanual clnica es el segundo paso para un diagnstico adecuado, pudiendo detectarse un crecimiento
uterino exagerado o la delimitacin de la tumoracin. De ah la importancia de que la paciente haya vaciado la vejiga antes de la exploracin y de que se
encuentre en una posicin cmoda, con los msculos abdominales completamente relajados, para permitir un examen adecuado. Siempre que se
sospeche la presencia de miomas, est indicada la realizacin de un ultrasonido plvico, lo que permitir determinar las dimensiones del tero, as como
el nmero, localizacin, tamao y relacin del (o los) miomas con estructuras cercanas. Si despus de los puntos anteriores existe alguna duda
diagnstica, se podr recurrir a otros estudios de gabinete como son: histerosalpingografa, tomografa axial computarizada, y realizar diagnstico
diferencial con algn otro tipo de tumor benigno o maligno de la cavidad abdominal. TRATAMIENTO: Miomectoma: Consiste en la reseccin nicamente
de los miomas a travs de una incisin en la pared abdominal (si son tumores de gran tamao) o mediante ciruga de mnima invasincomo lo es la
laparoscopia. Esta miomectoma se indica siempre que la paciente tenga deseos de embarazos futuros y sea menor de 35 aos. Hasta 80% de las mujeres
refiere disminucin de los sntomas despus de la miomectoma. Histerectoma: Es la extirpacin total del tero, tambin a travs de una incisin
abdominal o por ciruga laparoscpica. Ser el procedimiento de eleccin en aqullas que no deseen tener ms hijos, estn cerca de la menopausia o si el
crecimiento tumoral ha sido muy acelerado. Tratamiento Hormonal: El objetivo es no bloquear al endometrio, el objetivo ser regular la tortuosidad de
los vasos, la estimulacin a nivel endometrial y que el endometrio este creciendo de manera regular en toda la cavidad uterina. Se puede usar ergotrate
(vasoconstrictor especifico), los primeros das del ciclo, en cada menstruacin o ac. Mefenamico (seguro). Lutoral o provera en la segunda parte del ciclo.
En miomatosis uterina de grandes elementos, donde no hay sangrado no se da tratamiento. Se pueden utilizar hormonas inhibidoras de la produccin de
estrgenos en forma paliativa o si existe alguna contraindicacin para realizar los procedimientos anteriores, tambin se utilizan si se desea disminuir
el tamao de los tumores previo a la ciruga o si coexiste endometriosis severa. El xito del tratamiento mdico-hormonal es menor del 10%.
CASO CLINICO SANGRADO UTERINO
Mujer de 49 aos con 3 hijos, viene a verlo debido en a que presenta
perodos abundantes, desde hace dos aos ha notado que esto se ha
incrementado de tal forma que le ha generado incomodidades ya que
afirma que inunda la toalla los primeros das y duran hasta 10 dias
adems agrega cansancio crnico, astenia y adinamia. A la exploracin
fsica se observa palidez de tegumentos, uas quebradizas, pelo frgil
asi como piel fra. A la exploracin GO se observa sangre obscura en
vagina con algunos coagulos, dolor a la palpacin y movilizacin
uterina, refiere que en su anterior DOC se reportaron cambios
inflamatorios recibiendo tratamiento el cual no especifica. Cuenta con
OTB, finalmente refiere sangrado poscoital.
PREGUNTA
Cul es la conducta a seguir?
RESPUESTA
a.- Realizar USG.
b.- Biometria hemtica y ferritina serica.
c.- Histeroscopia.
d.- Biopsia endometrial.

CURSO ENARM CMN SIGLO XXI TEL: 36246001

CASO CLINICO SANGRADO UTERINO


Mujer de 31 aos de edad, refiriendo presencia de irregularidades de
su periodo menstrual, agrega que no se ha podido embarazar desde
hace 6 aos que inicio su vida sexual activa. No usa mtodo
anticonceptivo. Actualmente toma 37,5 mg de venlafaxina dos veces al
da por episodio depresivo que inicio hace un ao. Su examen fsico se
observa presencia de hematomas refiriendo que esto le ha ocurrido
desde la adolescencia, se observa adems un IMC de 34, con
hirsutismo en cara, espalda y piernas, adems vello pbico tipo
romboide.
PREGUNTA
Cul es la conducta a seguir?
RESPUESTA
a.- USG transvaginal.
b.- USG ginecologico abdominal.
c.- USG de glndulas suprarrenales.
d.- Biopsia endometrial.

Pharmed Solutions Institute

PGINA 241

MANUAL DE TRABAJO DEL CURSO ENARM CMN SIGLO XXI


SINDROME DE OVARIO POLIQUISTICO (SOP)
CIENCIAS BASICAS: El SOP es una de las alteraciones endocrinas ms comunes en la mujer, en edad reproductiva y es la principal causa de esterilidad por
anovulacin. Representa un trastorno heterogneo que se distingue por una combinacin de irregularidades menstruales, hirsutismo o acn y obesidad,
que suele diagnosticarse en la adolescencia, pero aparentemente tienen su origen en la vida intrauterina. Actualmente la relacin entre SOP y sndrome
metablico (obesidad, hipertensin, dislipidemia y disglucosis), y subsecuente diabetes mellitus tipo II, lo convierten en un grave problema de salud
pblica con un alto coste econmico. SALUD PUBLICA: Tiene una prevalencia de 4-8%. Tan solo en Estados Unidos se considera entre 7-10 millones de
mujeres afectadas por este padecimiento. Se ha asociado con un riesgo mayor de cncer de endometrio y probablemente de glndula mamaria. El 5065% de las pacientes con SOP son obesas y se considera que el 35-45% de ellas presentar intolerancia a la glucosa durante una curva de tolerancia a la
glucosa, con un riesgo de desarrollar DM tipo 2 del doble a lo esperado para su edad. PATOGENIA: En la aparicin de SOP existen componente sobre los
que no podemos influir: preconcepcionales (gentica) y postconcepcionales (peso al nacer o exposicin intratero a andrgenos) y otros aspectos que si
pueden ser modificables como son los hbitos de vida (dieta y ejercicio). Hay evidencias que parece seguir un patrn de herencia autosmico
dominante. En mujeres con SOP, se ha descrito una alteracin en el eje hipotlamo-hipfisis-ovario, que se distingue por aumento de la actividad del
hipotlamo que produce un mayor numero de pulsos de la hormona liberadora de gonadotrofinas (GnRH), lo que a su vez aumenta la hormona
luteinizante, cambiando la relacin LH/FSH a favor de la primera. Al predominar la LH se sintetizan preferentemente andrgenos en el ovario. La
resistencia a la insulina puede desempear una funcin central en la causa del sndrome: el musculo esqueltico es profundamente resistente, mientras
que otros tejidos, como el hipotlamo, la glndula suprarrenal y el ovario, conservan la sensibilidad a la insulina. La hiperinsulinemia compensadora
resulta en una disminucin en la globulina transportadora de esteroides sexuales (SHBG) y simultneamente sirve como estmulo trfico en la
produccin de andrgenos por parte de las glndulas suprarrenales y el ovario. La insulina tambin tiene efectos directos en el hipotlamo, ya que
estimula el apetito y la secrecin de gonadotrofinas. Existen otros efectos importantes de la insulina que contribuyen al hiperandrogenismo: inhibicin
de la produccin heptica de la SHBG y de la protena transportadora tipo de IGF-1 (1GFBP-1), lo cual aumenta las concentraciones circulantes de
andrgenos libres y produce una mayor actividad ovrica. DIAGNOSTICO: La mayora de los datos clnicos que aparecen en pacientes con SOP pueden
formar parte de otras patologas no necesariamente relacionadas con alguna disfuncin hormonal y no los hechos aislados deben siempre sugerir el
diagnstico. La severidad o la rpida evolucin debe alertar a los clnicos a la realizacin de un diagnstico diferencial ms extenso. Acn: Su
permanencia de los 20 aos en adelante debe considerarse sospechosa. Es imperativo interrogar acerca de irregularidades menstruales, hirsutismo o
cualquier otro dato de hiperandrogenismo. En mujeres con acn se ha encontrado datos de SOP hasta en el 45% de los casos. Hirsutismo: Se define
como el crecimiento excesivo del vello corporal terminal en mujeres, en reas anatmicas donde el desarrollo de los folculos depende de la estimulacin
andrognica: Tercio proximal en cara interna de muslos, abdomen, pecho, parte baja de la espalda y cara entre otros. El grado y la severidad se evalan
de acuerdo a la escala de Ferriman-Galwey. Virilizacin: La aparicin de hipertrofia clitordea, amenorrea prolongada, aumento de la musculatura, atrofia
de los senos, hirsutismo severo y habitus masculino obliga a descartar la presencia de hiperplasia adrenal, hipertecosis o tumores ovricos o adrenales.
Irregularidad menstrual y anovulacin: Las mujeres con SOP presentan grados variables de disfuncin ovulatoria, manifestada como oligomenorrea,
dismenorrea y amenorrea. Incluso, frecuentemente presentan infertilidad (17.5% vs 1.3% en normales). La relacin entre el exceso de insulina y la
anovulacin se atribuye en primer lugar a la hiperandrogenemia y en segundo a la estimulacin del inhibidor 1 del activador del plasmingeno (PAI-1).
Este inhibidor no solo regula la eliminacin de los depsitos de fibrina de los vasos sanguneos sino que a nivel del ovario, inhibe a las colagenasas
responsables de la ruptura folicular. Acanthosis Nigricans: Se trata de una hiperplasia hiperpigmentada de la piel, la cual aparece predominantemente en
el cuello y en pliegues cutneos como axilas y codos. La importancia de su deteccin radica en que su presencia correlaciona significativamente con los
estados de resistencia a la insulina e hiperinsulinemia compensatoria. Dependiendo de las poblaciones estudiadas aparece ligada a obesidad hasta en el
74% de los casos constituyndose como un factor de riesgo independiente para DM tipo 2. Aproximadamente el 30% de las pacientes con SOP lo
presentan. El SOP es un diagnstico de exclusin y no es necesaria la presencia de alteraciones en los niveles hormonales o quistes en los ovarios para
considerar su presencia. Escrutinio: Determinacin en ayunas y durante los primeros 7 das del ciclo de: a) LH y FSH en pool (el promedio de 3 muestras
recolectadas con 20 minutos de diferencia) y Prolactina. b) Testosterona total y libre. c) Dehidroepiandrosterona sulfato y 17 alfahidroxiprogesterona. En
el 66% de los pacientes se documenta una relacin LH:FSH (>2.5:1), la cual es caractersticade SOP. La prolactina debe ser determinada en la evaluacin
de cualquier paciente con amenorrea; muchas de las causas de hiperandrogenismo pudieran incrementarla pero valores >100 sugieren patologa
hipofisiaria. La testosterona total es la prueba ms validada para evaluar la presencia de tumores secretores de andrgenos: niveles >150 ng/dL
fuertemente sugieren la posibilidad de tumores ovricos o adrenales. Cuando la paciente desea procrear es fundamental determinar si los ciclos son
ovulatorios. Los niveles de progesterona <2 ng/mL despus del da 21 del ciclo son interpretados como anovulacin. Estudios de Imagen: La
ultrasonografa (USG) plvica es el mtodo de eleccin para la evaluacin inicial de los anexos. Caractersticamente se trata de folculos de 8 mm con
aumento del estroma central, aunque con la tcnica vaginal pueden detectarse quistes de 3-5 mm. Los quistes ovricos aparecen irregularmente en las
pacientes con SOP. El estudio de imagen de eleccin para el estudio de las suprarrenales es la tomografa axial computarizada de alta resolucin.
TRATAMIENTO: Prdida de peso. La obesidad es prevalente en mujeres con SOP, y se asocia con un empeoramiento de los sntomas. La prdida de peso
de un 5-7% con respecto al basal produce un descenso en la concentracin circulante de andrgenos, insulina y lpidos, lo cual se acompaa de una
mejora de la sintomatologa y de las posibilidades de presentar ciclos ovulatorios. Tratamiento hormonal. Los anticonceptivos hormonales combinados
proveen varios beneficios en las pacientes con SOP y durante mucho tiempo han sido la piedra angular del tratamiento. Restituyen los ciclos menstruales
con eficiencia y mejoran el hirsutismo en ms del 60% de los casos por su efecto inhibidor de la LH, lo que aumenta los niveles de SHBG. Tambin
suprimen el metabolismo de los andrgenos en las adrenales y disminuyen el nmero de receptores de la 5a-reductasa en la piel, lo que beneficia al
acn. En general, las progestinas protegen al endometrio oponindose a los efectos proliferativos de los estrgenos pero tienen un efecto negativo en
los parmetros metablicos ya que aumentan la resistencia a la insulina y los niveles de triglicridos. Agentes sensibilizadores de la insulina. Est
documentado que la disminucin en los niveles de insulina mediante el uso de este grupo de frmacos se acompaa de una mejora del cuadro clnico y
de las anormalidades metablicas. Metformina: Acta principalmente en el hgado inhibiendo parcialmente la gluconeognesis, lo que reduce en un 1725% la produccin de glucosa y en el msculo esqueltico donde incrementa la captacin de glucosa estimulada por insulina en un 29% y disminuye la
oxidacin de los cidos grasos en un 10-20%. En mujeres con SOP el metformn ha demostrado inducir la ovulacin y mejorar las posibilidades de lograr
un embarazo, incluso en pacientes sometidas a fertilizacin in vitro. La combinacin de metformn con anticonceptivos orales o acetato de ciproterona,
se ha acompaado de una mejora en los parmetros clnicos y metablicos. Tiazolidinedionas: En pacientes con SOP la troglitazona, el primer
medicamento del grupo y el ms estudiado (actualmente no disponible para su venta en Mxico) mejora la resistencia a la insulina disminuyendo la
dehidroepiandrosterona sulfato, la testosterona libre, la androstenediona y la LH, e incrementando la protena transportadora de hormonas sexuales.
Tratamiento de la anovulacin. El citrato de clomifeno, solo o en combinacin con la prdida de peso, sigue siendo, dada su seguridad y simplicidad, el
tratamiento de primera eleccin en la infertilidad de origen anovulatorio asociado al SOP. Acetato de ciproterona, para tratamiento de hirsutismo.
Agonistas de la GnRh. Los agonistas de la GnRH como el leuprolide, disminuyen la produccin ovrica de esteroides por supresin de LH y FSH. Este
tratamiento es altamente efectivo en mujeres con SOP severo o hipertecosis ovrica. Tratamiento Quirrgico. La reseccin en cua bilateral de los

CURSO ENARM CMN SIGLO XXI TEL: 36246001

Pharmed Solutions Institute

PGINA 242

MANUAL DE TRABAJO DEL CURSO ENARM CMN SIGLO XXI


ovarios es un procedimiento quirrgico que se realiz con xito en pacientes con SOP durante muchos aos. Esto provoca una reduccin en los niveles
de LH y produccin de andrgenos.
CASO CLINICO SINDROME DE OVARIO POLIQUISTICO.
Acude a consulta femenino de 23 aos de edad que presenta falta de
periodo menstrual y desea saber si esta embarazada, la paciente se
dedica a su casa, vive y tiene relaciones con su pareja desde hace 2
aos y quiere tener su primer hijo, refiere que nunca ha sido regular, a
la exploracin fsica se observa con una talla de 156 cm y peso de 74
kg, se observa acn en cara y espalda con hirsutismo, se observa
obscurecimiento de cuello y axilas.
PREGUNTA
Cul es la causa ms probable de la amenorrea en este caso?
RESPUESTA
a.- Hiperandrogenismo.
b.- Incremento de progesterona.
c.- Disminucin de estrgenos.
d.- Aumento de gonadotrofinas.
CASO CLINICO
Se trata de femenino de 27 aos de edad la cual cursa con esterilidad
primaria de dos aos de evolucin, sin diagnostico definitivo aun,
acude a la consulta debido a que se han acentuado sus sntomas, la
cual refiere presencia de periodos menstruales irregulares, a la
exploracin usted observa hirsutismo, acn en frente y espalda,
obesidad con datos de acantosis nigrans.
PREGUNTA
Enva a la paciente para realizar estudios de imagen, cual es el
diagnostico que espera encontrar ms probablemente:
RESPUESTA
a.- Hiperplasia Suprarrenal.
b.- Sindrome de Silla Turca Vacia.
c.- Sindrome de Ovarios Poliquisticos.
d.- Sindrome de Cushing.
CASO CLINICO
Paciente de 27 aos de edad, quien consulta porque desde la
menarquia ha tenido ciclos irregulares, caracterizados por la presencia
de oligo y amenorreas que oscilan entre 30 y 180 das. Ha presentado
en varias oportunidades tratamiento dermatolgico por la presencia
de acn. En varias ocasiones ha recibido anticonceptivos orales con
progestgenos antiandrognicos (ciproterona). Como antecedentes
importantes tiene un abuelo paterno diabtico. Su menarquia fue a los
13 aos y nunca ha estado embarazada. El examen fsico muestra una
paciente en buenas condiciones con talla de 1,63 m, peso 72 kg, ndice
de masa corporal de 27,1 kg/m2, TA 130/80, FC 72 por minuto. En el
cuello y las axilas se encuentra la presencia de acantosis nigricans. La
circunferencia abdominal es de 86 cm. En la espalda hay cicatrices de
acn y el ndice de Ferriman y Gallway da un puntaje de 10
PREGUNTA
Teniendo en cuenta el cuadro clnico de esta paciente, as como la
posible etiologa, usted considera que el tratamiento ms apropiado
sera:
RESPUESTA
a.- Usar inductores de la ovulacin, como citrato de clomifeno.
b.- Medroxiprogesterona de depsito.
c.- Tan solo dieta y ejercicio.
d.- Iniciar metformina.
PREGUNTA

CURSO ENARM CMN SIGLO XXI TEL: 36246001

Cul de las siguientes complicaciones es menos probable que


presente esta paciente?
RESPUESTA
a.- Infarto agudo de miocardio
b.- En caso de embarazo preeclampsia
c.- Evento cerebro-vascular
d.- Diabetes gestacional
PREGUNTA
Cul de las siguientes hipotesis de la patogenia de este padecimiento
guarda menor evidencia?
RESPUESTA
a.- Elevacin de LH que induce maduracin prematura del ovocito
b.- Hipopralactinemia y aumento de Globulina fijadora de hormonas
sexuales (SHBG)
c.- Insulinorresistencia que estimula la sntesis ovrica y suprarrenal de
andrgenos
d.- Obesidad genera resistencia a la insulina e hiperinsulinemia
compensatoria
QUESTIONS
Which of the following agents is considered a first line treatment for
hirsutism in patients with polycystic ovarian sndrome (PCOS)?
ANSWERS
a.- Clomiphene
b.- Lisinopril
d.- Metformin
c.- Sibutramine
CASO CLINICO
Mujer multigravida de 22 aos de edad que acude con su gineclogo
debido a menstruaciones irregulares y ampliamene espaciadas. La
menarca fue a los 14 aos de edad, pero rara vez ha habido ciclos
regulares. Durante el ao anterior solo hubo 3 menstruaciones
completas y una ocasin transcurrieron 6 meses entre los periodos. La
paciente tienen actividad sexual y usa condon para anticoncepcin. En
el examn fiico la temperatura es de 37C, la FC 80/min, la presin
arterial de 140/85mmHg. Mide 1.65mts y pesa 83Kg. Muestra acn y
algunos pelos oscuros en el labio superior y la barbilla. Los examens
con especulo y bimanual revelan genitales externos e internos
normales, sin hipersensibilidad, ni masas.
PREGUNTA
Cul es la complicacin menos probable en este caso?
RESPUESTA
a.- Hiperplasia endometrial
b.- Resistencia a la insulina
c.- Neoplasia endometrial
d.- Sindrome metabolico
PREGUNTA
Cul es la conducta diagnostica mas adecuada a seguir en este
momento?
RESPUESTA
a.- Ultrasonido
b.- Radiografia
c.- Prfil hormonal
d.- Prueba de supresin de progestina

Pharmed Solutions Institute

PGINA 243

MANUAL DE TRABAJO DEL CURSO ENARM CMN SIGLO XXI


MASTOPATIA FIBROQUISTICA (MFQ)
CIENCIAS BASICAS: Tambin conocida como enfermedad broqustica de la mama, cambios broqusticos, displasia mamaria y mama nodular dolorosa,
es una entidad clinicopatolgica crnica no maligna, que se evidencia como una zona dura, debido a la proliferacin del tejido conectivo, del epitelial y a
la presencia de quistes, de forma focal o difusa. Consisten en abultamientos circunscritos a la mama que pueden confundirse con neoplasias. La
diferencia es que son bilaterales y tienen variaciones en su tamao de acuerdo con la fecha del ciclo en que se examinan; su volumen es mayor en
etapa premenstrual. La naturaleza precancerosa de la MFQ es muy discutida, aceptndose que su transformacin maligna es baja. La MFQ simple no
tiene riesgo de malignizacin; s lo tiene la mastopata proliferativa con atipia. SALUD PUBLICA: Entre un 45-85% de las pacientes que acuden a una
consulta de mama lo hacen por este cuadro clnico. Constituye el proceso benigno ms frecuente de la mama. Produce sntomas incapacitantes en el 510% de las pacientes. Se observa, clnicamente, en un 50% de las mujeres adultas y se detecta en un 90% de los estudios histolgicos. Representa la
alteracin ms comn evidenciada en la mama de la mujer durante la madurez sexual. PAOGENIA: En el origen de la mastopata fibroqustica se
involucran factores hormonales, como: desequilibrio estrgeno-progesterona, con concentraciones en exceso de estrgenos y disminucin en las
concentraciones de progesterona, y concentracin elevada de prolactina en la sangre. DIAGNOSTICO: Clnicamente, se maniesta con molestias en una
o ambas mamas y con dolor a la palpacin. El dolor suele ser uni o bilateral, generalmente de carcter cclico, premenstrual. Otros sntomas son
nodularidad y/o induracin de forma variable y secrecin por el pezn; la telorrea serosa suele aparecer en la MFQ y en los quistes simples y suele ser
bilateral y plurioricial, igual que la de aspecto lechoso. La sintomatologa es frecuente que mejore con el embarazo y lactancia y tambin tras la
menopausia. A la exploracin, se palpa un engrosamiento en forma de placa o mltiples irregularidades en una parte de la mama, sobre todo en
cuadrantes superiores y externos, normalmente bilateral. Si hay un ndulo dominante, se presenta de forma regular, borde y supercie denidos y con
poca movilidad. Los sntomas en pacientes con mastopata fibroqustica coinciden en mastodinia y ndulos no adheridos en ambas mamas, que
aumentan de volumen en etapa premenstrual. Para su diagnstico, adems de la exploracin y clnica, contamos con las pruebas de imagen, cuyo
objetivo fundamental es conrmar o descartar la existencia de una lesin maligna. Las lesiones de mama se clasican mediante el sistema BI-RADS, que
caracteriza los hallazgos de la imagen en varias categoras que van desde la normalidad hasta la conrmacin histolgica de malignidad. Se considera el
idioma universal en el diagnstico de la patologa mamaria. Los ndulos descritos (BI-RADS 2) son los quistes simples demostrados en el estudio
ecogrco. La placa brosa se maniesta en el estudio mamogrco como una densidad focal asimtrica de mrgenes mal denidos (BI-RADS 3),
precisando de la ecografa para completar el estudio. Ecografa: lo ms habitual es que nos
podemos encontrar en la MFQ imgenes de similares caractersticas ecogrcas que en las
tumoraciones qusticas, diferencindose en su nmero y en su dimetro mximo. Atendiendo a su
dimetro mximo, dividimos la MFQ en: Tipo I (de pequeas formaciones): tumoraciones qusticas
de dimetro < 0,5 cm. Tipo II (de medianas formaciones): entre 0,5---2 cm. Tipo III (de grandes
formaciones): > 2 cm. Como todas las tumoraciones qusticas, ecogrcamente se presentan
redondeadas u ovaladas, con ecoestructura generalmente anecoica (BI-RADS 2), aunque pueden
presentar otros aspectos como hipoecognico con nos ecos internos, de contenido turbio o
espeso, a veces con nos tabiques en su interior (BI-RADS 3). Sus bordes estn bien denidos, su
eje mayor suele ser horizontal y, con gran frecuencia, estn presentes los signos indirectos
(refuerzo acstico posterior y sombra acstica lateral), pudiendo ser, a veces, bilobuladas. No
tienen ninguna seal Doppler interna. En muchas ocasiones, tras el estudio ecogrco, se modica
la categora BI-RADS previamente asignada por mamografa. Est indicada la realizacin de una
ecografa mamaria ante la presencia de una alteracin mamogrca para una caracterizacin ms
precisa, como es la presencia de una densidad asimtrica focal que, cuando no es palpable, la
lesin es categora BI-RADS 3. Tambin, ante la evidencia de una distorsin arquitectural en el seno
de reas densas de tejido mamario, catalogada como BI-RADS 4. En general, se puede armar que la RM tiene una alta sensibilidad y baja especicidad
en la evaluacin de las lesiones mamarias. Tambin se dispone de las pruebas diagnsticas intervencionistas, como la puncin-aspiracin con aguja na
(PAAF)/biopsia con aguja gruesa (BAG): brosis, proliferacin epitelial, adenosis, quistes y, en ocasiones, hiperplasia intraductal o lobulillar tpica o
atpica. Ante un cuadro clnico---eco y radiolgico dudoso (BI-RADS 3) o sugerente de malignidad (BI-RADS 4 y 5), hay que hacer una PAAF y/o BAG como
primera medida. La MFQ asintomtica no requiere seguimiento; la sintomtica se seguir con ecografa y/o mamografa. TRATAMIENTO: Los AINES
tpicos en gel como el piroxicam o diclofenaco al 2% deben ser considerados para el control localizado del dolor en el tratamiento de la mastalgia. Debe
iniciarse con el uso de preparados hormonales existentes para el tratamiento sintomtico, tales como los progestgenos (de uso tpico o por va oral o
vaginal), antiprolactnicos (bromocriptina), antiestrgenos, fundamentalmente el tamoxifeno, medicamento no autorizado en nuestro pas para el
tratamiento de la mastalgia pero s en otros pases, y antigonadotropos (danazol). Si dichos preparados fracasan, se adoptarn otras diversas medidas
paliativas. Actualmente, se propone alfa dihidroergocriptina por su alta efectividad, mnimas reacciones secundarias y fcil dosificacin. El alfa
dihidroergocriptina es un derivado hidrogenado del alfa ergocriptina, que es un alcaloide natural del Ergot utilizado en el tratamiento de la
hiperprolactinemia. Tiene un efecto dopaminrgico significativo en el cuerpo estriado y en la hipfisis y es capaz de disminuir las concentraciones
sricas de prolactina mediante la inhibicin de la adenilato-ciclasa dependiente de los receptores dopaminrgicos D2 en los lactotropos. PREVENCION:
La edad recomendable para el inicio del examen medico mamario rutinario es a partir de los 19 aos. Los pacientes con BCRA1 y BCRA2, deben inicar la
exploracin mdica mamaria entre 18-21 aos de edad. La autoexploracin aumenta la probabilidad de que la mujer encuentre anormalidad en las
mamas.
CASO CLINICO MASTOPATIA FIBROQUISTICA
Una mujer de 43 aos de edad con un historial de cambios
fibroqusticos de la mama, que se presentan con una masa palpable en
la mama derecha. No tena antecedentes familiares de cncer de
mama. El examen clnico revel una masa de 4x3 dolorosa, movil, bien
circunscrita en el cuadrante inferior interno de la mama derecha. La
mamografa mostr una masa bien delimitada con pared engrosada.
En la ecografa, la masa era hipoecoica y claramente definidas con
pared gruesa e irregular. La lesin fue extirpada. El examen
macroscpico mostr una lesin qustica bien delimitada de 3.3x1.7
cm de tamao, con aspecto gelatinoso espeso de material homogneo,

CURSO ENARM CMN SIGLO XXI TEL: 36246001

de color amarillo oscuro en su lumen. El espesor de la pared del quiste


fue de aproximadamente 3 mm, y la superficie interior tena
plegamientos papilares reminiscentes de papiloma qustica.
PREGUNTA
Cul es la diagnostico mas probable?
RESPUESTA
a.- Galactocele.
b.- Fibroma.
c.- Lipoma.
d.- Fibrosarcoma.

Pharmed Solutions Institute

PGINA 244

MANUAL DE TRABAJO DEL CURSO ENARM CMN SIGLO XXI


CANCER DE MAMA
CIENCIAS BASICAS: Es un padecimiento crnico, heterogneo con una evolucin irregular y lenta. Es la primera causa de muerte por neoplasia en la
mujer en el mbito mundial. SALUD PUBLICA: Cerca de 500 mil muertes cada ao a nivel mundial, de las cuales el 70% ocurre en pases en desarrollo. Las
tasas de incidencia tienen una variacin considerable en todo el mundo; las ms altas se encuentran en Europa y Norteamrica con cifras
estandarizadas de 99.4 por 100 mil mujeres. En cuanto a la magnitud actual del cncer de mama en Mxico, a partir de 2006 ste ocupa el primer lugar
de mortalidad por tumor maligno en las mujeres mayores de 25 aos, desplazando de esa posicin al cncer cervicouterino. En el ao 2010 la tasa
estandarizada de mortalidad fue de 18.7 por 100 mil mujeres de 25 y ms aos, lo que representa un incremento del 49.5% en los ltimos 20 aos.
PATOGENIA: Los factores de riesgo para el desarrollo del cncer de mama son los siguientes: A) Biolgicos: Sexo femenino, envejecimiento: A mayor
edad mayor riesgo, antecedente personal o familiar de cncer de mama en madre, hijas o hermanas, antecedentes de hallazgos de hiperplasia ductal
atpica, imagen radial o estrellada, as como carcinoma lobulillar in situ por biopsia. Vida menstrual mayor a 40 aos (menarca antes de los 12 aos y
menopausia despus de los 52 aos), densidad mamaria, ser portador conocido de los genes BRCA1 o BRCA2. B) Iatrgenos o ambientales: Exposicin a
radiaciones ionizantes, principalmente durante el desarrollo o crecimiento (in utero, en la adolescencia), tratamiento con radioterapia en trax. C)
Factores de riesgo relacionados con los antecedentes reproductivos: Nuliparidad, primer embarazo a trmino despus de los 30 aos de edad, terapia
hormonal en la perimenopausia o posmenopausia por ms de cinco aos. D) Factores de riesgo relacionados con estilo de vida: Alimentacin rica en
carbohidratos y baja en fibra, dieta rica en grasas tanto animales como cidos grasos trans, obesidad,
principalmente en la posmenopausia, sedentarismo, consumo de alcohol mayor a 15 g/da, tabaquismo.
DIAGNOSTICO: El cncer de mama en etapas iniciales se presenta de manera subclnica en la mayora
de los casos, es decir que solamente es detectable por estudios de imagen (mastografa, ultrasonido y
resonancia magntica), en menor proporcin por clnica (tumores palpables); sin embargo otra forma de
presentacin comn es como un tumor no doloroso que hasta en 30% se asocia a adenopatas axilares.
Los tumores localmente avanzados en nuestro pas representan 70% de las etapas clnicas al diagnstico,
pueden incluir cambios cutneos como edema, ulceracin, cutnides, as como afectacin de ganglios
como los supra e infraclaviculares homolaterales El carcinoma inflamatorio de la mama, un tipo de
presentacin poco comn pero de mal pronstico y que por lo general progresa rpidamente, se
caracteriza por una induracin difusa de la mama con eritema, edema y aumento de la temperatura
local en al menos un tercio de la glndula, en la mayora de los casos no existe una tumoracin franca
palpable. Todava menos frecuente es el diagnstico de cncer de mama por los sntomas de la
metstasis y no por el tumor primario. Es de suma importancia tener en cuenta dentro del abordaje
diagnstico los factores de riesgo del paciente, sus condiciones generales y antecedentes heredofamiliares. Estudios de imagen: La mastografa es hasta ahora el mejor mtodo de deteccin, tiene una sensibilidad diagnstica de 80 a 95%, aunque 10 a
15% de los tumores puede ser oculto sobre todo en mujeres con mamas densas (con el uso de mastografa digital mejora la sensibilidad diagnstica en
este grupo de pacientes). El ultrasonido es en algunos casos una herramienta complementaria para diferenciar masas qusticas de slidas, para
caracterizar lesiones benignas y malignas y como gua para la realizacin de biopsias de lesiones no palpables. La imagen por resonancia magntica (IRM)
con gadolinio tiene sensibilidad diagnstica de 94 a 100%, pero baja especificidad (37 a 97%) y valor predictivo positivo de 44 a 96%.Las indicaciones
actuales de este estudio son: a) como estudio de deteccin en mujeres con alto riesgo (como aquellas portadoras de mutaciones BRCA 1 y 2), b)
bsqueda de tumores ocultos mamarios de presentacin axilar, c) mujeres portadoras de implantes o prtesis mamarias, d) evaluacin de la respuesta al
tratamiento sistmico neoadyuvante, e) evaluacin complementaria para determinar multicentricidad y bilateralidad. Biopsia: El fundamento del
diagnstico del cncer de mama es la confirmacin histolgica del mismo, para esto se prefiere la realizacin de biopsias de mnima invasin con la
obtencin de material tisular que per mite determinar factores pronsticos y predictivos de suma importancia en el manejo integral de las pacientes, por
ejemplo la determinacin de receptores hormonales y de Her2/neu. El procedimiento de eleccin es la toma de biopsias con aguja de corte (trucut)
tanto en lesiones palpables como en las no palpables; esta forma diagnstica se asocia con una exactitud del 98.5% (7). En lesiones no palpables, la
biopsia debe ser realizada bajo la gua de algn mtodo de imagen (ultrasonido, mastografa, resonancia, etc). Las biopsias quirrgicas previo marcaje
(arpn, radiocoloide, etc.) estn indicadas cuando no es factible el diagnstico mediante un procedimiento menos invasivo. En la actualidad la biopsia
por aspiracin con aguja fina (BAAF) generalmente est reservada para la confirmacin de metstasis en adenopatas loco-regionales y tiene poca
utilidad como mtodo diagnstico en la lesin primaria. La evaluacin patolgica del cncer de mama debe incluir de manera indispensable tipo
histolgico, grado, permeacin vascular y linftica, tamao del tumor, mrgenes, nmero de ganglios y tamao de la metstasis ganglionar, estudios de
inmunohistoqumica que evalen la presencia o no de receptores hormonales para estrgenos y progesterona, Ki67 (>14%), la sobreexpresin del gen
ErbB2 (Her2/neu) o su amplificacin por FISH o CISH, adems de estudios complementarios como citoqueratinas y factores de crecimiento epidrmico,
etc. ESTADIFICACIN: La estadificacin del cncer de mama proporciona informacin respecto al pronstico y orienta el trata miento. Los estudios de
imagen en la actualidad son un complemento para evaluar el tamao del tumor, la presencia de los ganglios y las metstasis. Ver cuadro anexo.
TRATAMIENTO: Integral y multidisciplinario, los manejos locorregionales son ciruga y radioterapia en cualquiera de sus tres modalidades
(neoadyuvante, adyuvante y paliativa) y el tratamiento sistmico incluye la quimioterapia, la terapia endocrina y la terapia dirigida a blancos
moleculares. El tratamiento quirrgico del tumor primario en el cncer de mama ha pasado por mltiples modificaciones, en la actualidad se divide en
ciruga conservadora y mastectoma con sus mltiples variedades. . Las indicaciones para mastectoma incluyen: 1) contraindicacin para recibir radiote
rapia; 2) enfermedad multicntrica e 3) dificultad para obtener mrgenes adecuados y resultado cosmtico favorable despus de un intento de ciruga
conservadora. Existen varios tipos de mastectoma, la mastectoma total extirpa la totalidad del tejido mamario incluyendo el complejo areola-pezn
(CAP), en la mastectoma total preservadora de piel se realiza una incisin circundante a la areola y se remueve solo el CAP, la ventaja de esta modalidad
es favorecer los resultados cosmticos de la reconstruccin inmediata. La mastectoma radical modificada es un procedmiento realizado en nuestro pas
debido a los estadios localmente avanzados en los que realizamos el diagnstico en nuestras pacientes e incluye la reseccin de la totalidad del tejido y
piel mamaria as como la diseccin de los niveles ganglionares I y II, procedimientos ms extensos como la mastectoma Halsted estn reservados solo en
casos seleccionados. Por muchos aos, la diseccin de la axila ha sido el estndar de tratamiento. En la actualidad la diseccin del ganglio centinela es el
abordaje quirrgico de eleccin en los casos en que la axila es clnicamente negativa. Tratamiento Adyuvante; quimioterapia, eliminar la enfermedad
micrometastsica antes del desarrollo de clonas resistentes. La quimioterapia basada en antraciclinas disminuye 33% las recadas y 27% la mortalidad en
mujeres menores de 50 aos. Terapias biolgicas: Aproximadamente 15 a 25% de los cnceres de mama tendr sobreexpresin del gen HER2/neu
(ErbB2). El Trastuzumab es un anticuerpo monoclonal humanizado dirigido en contra de la protena HER2. Terapia Endocrina: Entre 50 y 70% de los
pacientes con cncer de mama, el tumor ser hormono sensible por lo que se podrn beneficiar de una de las siguientes modalidades de manejo:
Tamoxifn; ha demostrado una disminucin en el riesgo a recurrencia de 40% y en riesgo de muerte de 35%, as como reduccin en el desarrollo de otro
cncer de mama. Inhibidores de aromatasa: exclusivo de mujeres posmenopusicas, ha demostrado mejorar la supervivencia libre de enfermedad, no la

CURSO ENARM CMN SIGLO XXI TEL: 36246001

Pharmed Solutions Institute

PGINA 245

MANUAL DE TRABAJO DEL CURSO ENARM CMN SIGLO XXI


global. El tratamiento con radioterapia est indicado en todos los pacientes que hayan sido sometidos a ciruga conservadora. Tratamiento
Neoadyuvante Sistmico: Esta modalidad teraputica se ha utilizado desde hace muchos aos y se considera el estndar en los tumores localmente
avanzados. Se recomienda un esquema basado en taxanos y antraciclenos durante seis a ocho ciclos. El objetivo principal es facilitar las diferentes
modalidades quirrgicas. En trminos generales, la enfermedad metastsica puede clasificarse como visceral (pulmn, hgado y sistema nervioso central)
y no visceral (incluye hueso, tejidos blandos y derrame pleural), con supervivencia global para el primer grupo de 12 meses y para el segundo de tres a
cinco aos. Seguimiento: Consiste en valoracin clnica peridica (trimestral los primeros dos aos, del ao 3 al ao 5 es semestral, y posteriormente,
anual), los objetivos son: detectar recada local, regional o sistmica y la presencia de un segundo primario. Se debe realizar una masto grafa y tele de
trax anuales as como densitometria sea semestral (en posmenopusicas o tratadas con inhibidores de aromatasa). PREVENCION: La promocin de las
conductas favorables a la salud para la prevencin del cncer de mama puede disminuir hasta en 30% la incidencia en la poblacin. Se debe orientar a
las mujeres acerca de su responsabilidad en el autocuidado de la salud, disminuir los factores de riesgo cuando sea posible y promover los estilos de
vida sanos como: Dieta rica en frutas y verduras y baja en grasas animales. Prctica de ejercicio fsico moderado, que se asocia con una disminucin del
riesgo de cncer de mama en la poblacin general. Consumo de cido flico. Amamantar, por lo que debe incluirse entre las ventajas de la lactancia
materna. Mantener un adecuado ndice de masa corporal pues el elevado (> 30) se asocia con un incremento significativo en el riesgo de cncer de
mama en posmenopusicas. La prevencin secundaria para la deteccin del cncer de mama debe incluir la autoexploracin, el examen clnico y la
mastografa. La autoexploracin se debe recomendar a partir de los 20 aos; el objetivo es sensibilizar a la mujer sobre el cncer de mama, lograr que
tenga un mayor conocimiento de su propio cuerpo y que identifique cambios anormales para la demanda de atencin mdica apropiada. El examen
clnico debe ser practicado anualmente a partir de los 25 aos, por personal de salud capacitado en la exploracin de las mamas. Deteccin anual con
mastografa a partir de los 40 aos. Se recomienda el empleo de quimioprevencin, el empleo de tamoxifeno1-3 y exemestano se asocia con una
reduccin en el riesgo de carcinoma ductal invasor Para decidir el uso de estos agentes se deben tomar en cuenta las contraindicaciones relacionadas
con su empleo y los antecentes de la paciente tales como: Historia de eventos tromboemblicos o hiperplasia atpica del endometrio en el caso de
tamoxifeno o el diagnstico de osteopenia/osteoporosis o enfermedades cardiovasculares con el uso de exemestano. La ciruga profilctica debe tener
un abordaje multidisciplinario considerando que es una ciruga reductora de riesgo de cncer de mama contralateral. Los pacientes con antecedente de
radioterapia (enfermedad de Hodgkin en mediastino), susceptiblidad gentica, e historia de cncer de mama familiar son los grupo ms recomendados
para este tratamiento.
CASO CLINICO CANCER MAMA.
Se trata de femenino de 48 aos de edad originaria del estado de
puebla, con el antecedente de importancia de ser portadora de cncer
de mama actualmente bajo tratamiento con tamoxifen, alendronato,
acetato de megestrol, acude a urgencia debido a que presenta nauseas
y vomito, fatiga y cansancio generalizado que se acompaa con dolor
abdominal difuso, los sntomas no se presentan relacionados a ingesta
de alimento u otras actividades cotidianas, a la exploracin fsica se
observa con un ndice de masa corporal de 23, leve deshidratacin de
mucosas, palidez generalizada moderada, cardiorespiratorio sin
compromiso aparente, abdomen plano depresible difusamente
doloroso, no hay signos de irritacin peritoneal, ruidos intestinales
normales, los estudios de laboratorio revelan biometra normal,
qumica sangunea y electrolitos con los siguientes resultados: sodio
130 meq/L, potasio 4,8 meq/L, cloruro 98 meq/L, Bicarbonato 29
meq/l, BUN 15mg/dl, creatinina 0.7 mg/dl.
PREGUNTA
Cul es la conducta ms adecuada a seguir para establecer un
diagnostico con los datos que presenta el caso?
RESPUESTA
a.- Realizar laparoscopia.
b.- Cuantificar cortisol serico.
c.- Transito gastrointestinal.
d.- Endoscopia alta.
CASO CLINICO CANCER DE MAMA
Se trata de paciente femenino de 49 aos de edad la cual acude a
consulta debido a que desde hace 6 meses ha presentado dificultadas
para caminar y cadas frecuentes por lo mismo, cuenta con
antecedentes de importancia como alcoholismo social crnico,
tabaquismo positivo desde los 17 aos hasta la actualidad, hace 2 aos
fue realizada histerectoma por sangrado uterino anormal, sin
especificar el diagnostico histopatolgico, a la exploracin fsica se
observa incoordinacin, marcha inestables, nistagmus, disartria leve y
alteracin en la prueba dedo a nariz.
PREGUNTA
Considerando el historial clnico cual es la fuente primaria mas
probable en este caso?
RESPUESTA
a.- Cncer de mama.
b.- Cncer cervicouterino.

CURSO ENARM CMN SIGLO XXI TEL: 36246001

c.- Cncer de colon.


d.- Cncer pulmonar.
CASO CLINICO
Mujer de 38 aos, nuligesta, con menarquia a los 12 aos y frmula
menstrual FM=6/30. Antecedentes mdicos: escoliosis y pectum
excavatum. Acude a la unidad de urgencias, por endurecimiento en la
mama y telorragia. Haba notado el ndulo haca 1 ao. En la ecografa
mamaria no se vieron alteraciones entonces. Ahora, en las
mamografas, lesin en el cuadrante superoexterno (CSE) y en el
cuadrante inferoexterno (CIE) de la mama izquierda. No se identifican
con claridad alteraciones a nivel de la lesin palpable. Adenopata
inespecfica en la axila. En las mamografas hay dos reas extensas de
microcalcificaciones en cumulo, la 1.a en el CSE, que corresponde a la
lesin ms superficial y ms fcil de palpar; y la 2.a en el CIE, ms
extensa, en donde hay una rea donde hay perdida de la estructura
normal.
PREGUNTA
De acuerdo a la descripcin de las lesiones A qu grado de la
clasificacin de BIRADS corresponde?
RESPUESTA
a.- 2.
b.- 3.
c.- 4.
d.- 5.
PREGUNTA
Cul es la conducta diagnostica ms adecuada a seguir para el
diagnstico definitivo de esta paciente?
RESPUESTA
a.- Exploracin, Mastografa, biopsia
b.- Exploracin, ultrasonido, mastografa
c.- Exploracin, ultrasonido, biopsia
d.- Exploracin, mastografa, resonancia
PREGUNTA
Cul es el diagnstico ms probable de este caso?
RESPUESTA
a.- Carcinoma ductal infiltrante
b.- Carcinoma lobulillar infiltrante
c.- Carcinoma medular
d.- Carcinoma mucinoso

Pharmed Solutions Institute

PGINA 246

MANUAL DE TRABAJO DEL CURSO ENARM CMN SIGLO XXI


MENOPAUSIA Y CLIMATERIO
CIENCIAS BASICAS: Menopausia: es un evento natural, inevitable en la vida de la mujer, que lleva al cese de la funcin reproductora. Menopausia
literalmente significa "cese permanente de la menstruacin", se debe a la prdida de la funcin folicular ovrica y el diagnstico se hace en forma
retrospectiva despus de un perodo de amenorrea de 12 meses. El promedio de edad de la menopausia en estudios realizados en E.U. y Europa vara
entre 48 y 52 aos. Cifras similares se han reportado en frica, Mxico y Japn. El factor ms importante que determina la edad de la menopausia es el
nmero de folculos ovricos, cuyo nmero es determinado en la fase temprana de la embriognesis. Climaterio o perimenopausia: comienza varios aos
antes de que se produzca la menopausia, incluye el perodo inmediatamente anterior a la menopausia (cuando comienzan las manifestaciones
endocrinolgicas, biolgicas y clnicas) como mnimo se prolonga hasta el primer ao siguiente a la menopausia. Postmenopausia: se refiere al perodo
que comienza a partir de la menopausia, si bien este momento no
se puede determinar hasta que se hayan observado doce meses de
amenorrea espontnea. Se ha utilizado el trmino de menopausia
quirrgica para referirse al cese de la menstruacin posterior a la
histerectoma sin ooforectoma, pero se recomienda utilizarlo
nicamente cuando la operacin va acompaada de la extirpacin
de ambos ovarios con o sin tero. SALUD PUBLICA: Debido a
diferentes factores como la disminucin de la mortalidad materna y
de las enfermedades infecciosas, la esperanza de vida se ha
prolongado y actualmente es de ms de 70 aos. Esto ha provocado
un aumento del nmero de mujeres que llegan a la menopausia y
de los aos que viven como postmenopusicas ya que, si tomamos
en consideracin que la edad en que se produce aqulla es
alrededor de los 50 aos. PATOGENIA: Durante la transicin de la
etapa reproductiva a la no reproductiva, la mujer experimenta una serie de eventos, los cuales se inician varios aos antes de que ocurra la menopausia
(cese de la menstruacin). Generalmente despus de los 40 aos la fase folicular del ciclo se acorta, los niveles de estradiol, inhibina y de folculoestatina
disminuyen y los de la FSH comienzan a elevarse. Posteriormente los niveles de la hormona luteinizante (LH) tambin aumentan aunque en menor
proporcin que la anterior. Los folculos disminuyen y es ms frecuente la ovulacin prematura y la fase ltea insuficiente con disminucin de la
produccin de progesterona, lo que produce un exceso de estrgenos en relacin a la hormona anterior. Cerca de la menopausia, los ciclos menstruales
a menudo son irregulares con anovulacin intermitente, los niveles de las gonadotrofinas son errticas, es frecuente el hiperestrogenismo relativo con
disminucin de la progesterona. Clnicamente esto se manifiesta con sangrados vaginales irregulares, pueden existir perodos de amenorrea con valores
de FSH y LH elevados, similares a los hallados en la menopausia, pero cuando se produce la ovulacin, los niveles hormonales pueden volver a lmites
normales. Finalmente, los folculos ovricos dejan de responder a las FSH y LH; el estradiol desciende a menos de 20 pg/mL, la progesterona es
indetectable y clnicamente hay cese de la menstruacin. Ver cuadro anexo. Despus de la menopausia cambia el origen y la naturaleza del estrgeno
circulante, ya que durante la vida reproductiva predomina el estradiol y en la postmenopausia la estrona. El primero es producido por el ovario y la
segunda proviene en su mayora de la conversin perifrica de los precursores andrognicos en el tejido adiposo, msculo e hgado. DIAGNOSTICO: El
climaterio se asocia con signos y sntomas tpicos relacionados con la deficiencia de estrgenos. Tpicos: Amenorrea, bochornos, crisis de sudoracin,
alteraciones del tracto genitourinario. Generales: Alteraciones psicolgicas, alteraciones emocionales, alteraciones psicosomticas. Sntomas
vasomotores: Los bochornos o sofocos se definen como un aumento en la percepcin del calor dentro del cuerpo, se manifiestan por una elevacin en la
temperatura cutnea, vasodilatacin perifrica, aceleracin transitoria de la frecuencia cardaca y modificaciones en la actividad electrodrmica. Se
inician varios aos antes de la menopausia, aun en mujeres con ciclos regulares, generalmente aumentan conforme stos se vuelven irregulares,
alcanzando su mxima intensidad y mayor frecuencia 1 a 2 aos despus de la menopausia y pueden persistir hasta por 5 15 aos despus de sta. Los
episodios nocturnos de sofocos se refieren como crisis de sudoracin. Debido al mismo origen embriolgico de vagina, trgono uretral y vejiga, las vas
urinarias inferiores presentan cambios atrficos por falta de estrgenos. Son comunes las quejas tanto de sntomas urinarios (disuria, infecciones
urinarias frecuentes) como los propios de vaginitis atrfica: prurito, sequedad vaginal, dispareunia, sangrado vaginal etc. Las mujeres que no reciben
tratamiento con estrgenos, al paso del tiempo pueden desarrollar uretritis causada por atrofia de la mucosa uretral que produce disuria y vaginitis
atrfica. La atrofia de la vagina produce adelgazamiento del endotelio, prdida de los pliegues, acortamiento de la misma y disminucin de la lubricacin,
lo que se relaciona con dispareunia. Antes de la menopausia las clulas epiteliales de la vagina son ricas en glicgeno, el cual es metabolizado por
lactobacilos, lo que contribuye a formar un ph cido que protege contra las infecciones, posterior a la misma se pierde este mecanismo protector y
puede presentarse mayor frecuencia de infecciones as como tendencia a las ulceraciones. Sntomas psicolgicos, somticos y emocionales: ansiedad,
aumento de la tensin, cambios de humor, depresin, irritabilidad, cansancio, insomnio, dificultad para concentrarse, ataques de pnico, cefalea, dolores
articulares, palpitaciones, parestesias. Despus de la menopausia, el adelgazamiento del epitelio de la vagina no afecta la funcin sexual cuando hay
relaciones sexuales en forma regular. En los casos de atrofia severa o vaginitis atrfica puede existir dispareunia. Recientemente se ha sugerido que la
falta de estrgenos que se produce durante la menopausia contribuye al desarrollo de la enfermedad de Alzheimer y se ha reportado que la terapia de
reemplazo con estrgenos puede ser til para prevenir o retardar el inicio de esta enfermedad. La osteoporosis y la enfermedad cardiovascular
constituyen las principales causas de morbimortalidad en las mujeres despus de la menopausia. Varios estudios han reportado alteraciones en el
metabolismo de lpidos, principalmente disminucin de los niveles plasmticos de lipoprotenas de alta densidad (HDL-c) y aumento de lipoprotenas de
baja densidad (LDL-c) tanto en mujeres con menopausia fisiolgica como quirrgica. Despus de la enfermedad cardiovascular, la osteoporosis es el
problema ms significativo relacionado con la deficiencia de estrgenos. TRATAMIENTO: Farmacolgico: 1) Terapia hormonal (TH): se debe ofrecer de
primera lnea con estrgenos estrgeno/progestageno. Es la ms eficaz para el control de sntomas vasomotores (hasta en 75%) y atrofia urogenital del
climaterio, as como prevencin de la osteoporosis. Esta contraindicada en las pacientes con manifestaciones clnicas de hipoestronismo en la
perimenopausia o post menopausia. Terapia cclica: estrgenos de 1-25 das ms progestgeno los ltimos 10-14 das de haber iniciado los estrgenos.
Ciclico-combinado: estrgenos 1-25 das y progestgenos de 1-25 das junto con estrgenos. Continuo cclico: estrgenos diariamente y progestgeno
durante 14 das durante 2-6 meses. Continuo combinado: estrgeno ms progestgeno. Diariamente sin descanso. Intermitente combinado: estrgenos
diariamente y progestgenos 3 dias con 3 de descansos. La principal funcin de los progestgenos es la proteccin endometrial, ya que los estrgenos
solos, tienen riesgo de desarrollo de cncer. Los estrgenos utilizados en la TH solos o combinados ms conocidos son: Orales; Estrgenos conjugados
derivados de equinos simples o combinados con acetato de medroxiprogesterona (0.625mg), estradil 1mg, etinilestradil + acetato de noretindrona.
Estradiol-drosperinona. Transdermica: estradiol-levonorgestrel, estradiol-acetato de noretindrona. Al utilizarse un progestgeno es recomendable que
sea por lo menos 12-14 das de cada mes. En Mxico el acetato de clormadinona es uno de los progestgenos que se usan frecuentemente como terapia

CURSO ENARM CMN SIGLO XXI TEL: 36246001

Pharmed Solutions Institute

PGINA 247

MANUAL DE TRABAJO DEL CURSO ENARM CMN SIGLO XXI


combinada en el sndrome climatrico. 2) Terapia no hormonal: inhibidores de la recaptura de serotonina y norepinefrina. Clonidina, gabapentina o
veraliprida. Cuando haya contraindicacin para hormonal.
CASO CLINICO MENOPAUSIA Y CLIMATERIO
Una mujer de 48 aos de edad, con menstruaciones regulares
mensuales refiere frecuentes despertares nocturnos durante los
ltimos 3 aos, seguido por una gran dificultad para volver a dormir.
Estos despertares inexplicables ocurrieron 2-3 veces por noche 3
noches a la semana. La interrupcin del sueo causaba somnolencia
diurna excesiva, dificultad para concentrarse en el trabajo, por lo que
se encuentra irritable. Ella haba estado tomando anticonceptivos
orales diariamente por cerca de 12 aos. Le prescribieron durante 6
meses trazodona antes de dormir sin resultado favorable y
posteriomente 4 meses de amitriptilina al acostarse ifualmente
ineficaz.
PREGUNTA
Cul es la conducta a seguir ms adecuada?
RESPUESTA
a.- Terapia hormonal de remplazo.
b.- Clonacepam.
c.- Venlafaxina.
d.- Zolpiden.
CASO CLINICO
Una mujer de 58 aos de edad ces de tener sus perodos menstruales
a los 49 aos. En ese entonces present bochornos leves, que se
resolvieron con el tiempo sin necesidad de tratamiento. Sin embargo,
ahora presenta sntomas que empeoran progresivamente y se queja
de dispareunia, picazn del introito y ardor, que afectan su desempeo
sexual, la paciente informa que el inicio de quejas urogenitales fue
abrupto y se asoci con descarga y dolor plvico o abdominal. Pide
terapia hormonal para aliviar los sntomas.
PREGUNTA
Antes de considerar el reemplazo de estrgeno, Cul de los siguientes
hallazgos de laboratorio debe obtenerse de esta paciente?
RESPUESTA
a.- Hormona foliculoestimulante (FSH)
b.- Estradiol
c.- Estradiol y FSH
d.- Cultivo vaginal
PREGUNTA
Qu alteraciones seria menos frecuentes encontrar en esta paciente,
si se da terapia hormonal por 1 ao?
RESPUESTA
a.- Tromboembolismo
b.- Accidente vascular cerebral
c.- Cncer de mama
d.- Osteoporosis
PREGUNTA
Qu tratamiento mdico no hormonal, sera ms adecuado para
disminuir los sntomas vasomotores?
RESPUESTA
a.- Gabapentina
b.- Veralipride
c.- Homeopatia
d.- Vitamina E
CASO CLINICO
Mujer de raza caucsica, de 47 aos de edad, que es remitida a la
consulta de ginecologa para evaluar la necesidad de terapia hormonal
sustitutiva. La paciente refiere sofocos intensos e intermitentes
adems de insomnio durante el ltimo ao, y sequedad vaginal y
dispareunia en los ltimos 3 meses. No hay informacin sobre su

CURSO ENARM CMN SIGLO XXI TEL: 36246001

patrn menstrual debido a que la paciente ha sufrido una his


terectoma por tero miomatoso (sin ooforectoma) hace 10 aos.
Desconoce la edad de la menopausia materna y es fumadora de unos
10 cigarrillos/da. No tiene historia ni personal ni familiar de
enfermedad coronaria, tromboembolismo venoso o cncer de mama.
PREGUNTA
Cul es el diagnotico ms probable en este caso?
RESPUESTA
a.- Hipertiroidismo
b.- Hiperprolactinemia
c.- Perimenopausia
d.- Embarazo
PREGUNTA
Cul es la caracterstica mas probable que nos ayudara a orientar el
diagnotico?
RESPUESTA
a.- Que el cese de la menstruacin sea brusco
b.- Que la ultima menstruacin vaya seguida de un ao de amenorrea
c.- Que la ultima mesntruacion vaya seguida de 6 meses de amenorrea
d.- Que la amenorrea vaya acompaada, cambios en el estado de
animo, sudoraciones, palpitaciones

PREGUNTA
Cul hormona nos seria de mas utilidad para el diagnotico de la
paciente?
RESPUESTA
a.- Hormona foliculoestimulante
b.- Hormona luteinizante
c.- TSH
d.- Prolactina
CASO CLINICO
Una mujer de 72 aos de edad, postme_nopusica desde hace 20
aos, se queja de irritacin y ardor vaginales y vulvares asociados con
una descarga persistente. Ha probado varios tratamientos con
frmacos de venta libre, incluso varias duchas vaginales comerciales y
agentes antimi_cticos, sin resultados. Durante el examen se observan
reas excoriadas en la vulva. La vagina est eritematosa, con una
mucosa aplanada y brillante con reas de ulceracin. Cul es la
patologa mas probable de esta paciente?
RESPUESTA
a.- Vaginitis atrfica
b.- Vulvovaginitis monilial crnica
c.- Vulvovaginitis bacteriana
d.- Incontinencia urinaria
CASO CLINICO
Una mujer de 54 aos de edad ces de presentar perodos
menstruales hace unos 18 meses. Todava sufre de bochornos y ahora
ha desarrollado sequedad y molestias vaginales. Parece ser una buena
candidata para recibir terapia de reemplazo de estrgeno para el alivio
de sus sntomas menopusicos.
PREGUNTA
Cul de las siguientes pruebas sera la ms importante antes de iniciar
el tratamiento con estrgeno?
RESPUESTA
a.- Perfil de lpidos
b.- Mamograma de seleccin
c.- Cultivo de orina
d.- Perfil hormona

Pharmed Solutions Institute

PGINA 248

MANUAL DE TRABAJO DEL CURSO ENARM CMN SIGLO XXI


CONTROL PRENATAL Y DEL EMBARAZO
La mayora de los daos obsttricos y los riesgos para la salud de la madre y del nio pueden ser prevenidos, detectados y tratados con xito, mediante
la aplicacin de procedimientos normados para la atencin, entre los que destacan el uso del enfoque de riesgo y la realizacin de actividades
eminentemente preventivas y la eliminacin o racionalizacin de algunas prcticas que llevadas a cabo en forma rutinaria aumentan los riesgos.
ESPECIFICACIONES: En la atencin a la madre durante el embarazo y el parto debe de vigilarse estrechamente la prescripcin y uso de medicamentos,
valorando el riesgo beneficio de su administracin. La unidad de atencin deber disponer de un instrumento que permita calificar durante el embarazo,
el riesgo obsttrico en bajo y alto, el cual servir para la referencia y contrarreferencia. CONTROL PRENATAL: Las actividades son; Elaboracin de historia
clnica. Identificacin de signos y sntomas de alarma (cefalea, edemas, sangrados, signos de infeccin de vas urinarias y vaginales). Medicin y registro
de peso y talla, as como interpretacin y valoracin. Medicin y registro de presin arterial, as como interpretacin y valoracin. Valoracin del riesgo
obsttrico. Valoracin del crecimiento uterino y estado de salud del feto. Determinacin de biometra hemtica completa, glucemia y VDRL (en la
primera consulta; en las subsecuentes dependiendo del riesgo), la embarazada debe ser informada a cerca del propsito de las pruebas de laboratorio.
Determinacin del grupo sanguneo ABO y Rho, (en embarazadas con Rh negativo y se sospeche riesgo, determinar Rho antgeno D y su variante dbil
D), se recomienda consultar la Norma Oficial Mexicana para la disposicin de sangre humana y sus componentes, con fines teraputicos. Examen
general de orina desde el primer control, as como preferentemente en las semanas 24, 28, 32 y 36. Deteccin del virus de la inmunodeficiencia
adquirida humana VIH en mujeres de alto riesgo (transfundidas, drogadictas y prostitutas), bajo conocimiento y consentimiento de la mujer y referir los
casos positivos a centros especializados, respetando el derecho a la privacidad y a la confidencialidad. Prescripcin profilctica de hierro y cido flico.
Prescripcin de medicamentos (slo con indicacin mdica: se recomienda no prescribir en las primeras 14 semanas del embarazo). Aplicacin de al
menos dos dosis de toxoide tetnico rutinariamente, la primera durante el primer contacto de la paciente con los servicios mdicos y la segunda a las
cuatro u ocho semanas posteriores, aplicndose una reactivacin en cada uno de los embarazos subsecuentes o cada cinco aos, en particular en reas
rurales. Orientacin nutricional tomando en cuenta las condiciones sociales, econmicas y sociales de la embarazada. Promocin para que la mujer
acuda a consulta con su pareja o algn familiar, para integrar a la familia al control de la embarazada. Promocin de la lactancia materna exclusiva.
Promocin y orientacin sobre planificacin familiar. Medidas de autocuidado de la salud. Establecimiento del diagnstico integral. Con el apoyo de los
datos anteriores, se deben establecer los criterios de referencia para la atencin de las gestantes a las unidades de primero, segundo y tercer niveles. La
unidad de atencin debe proporcionar a la mujer embarazada un carnet perinatal que contenga los siguientes datos: identificacin, antecedentes
personales patolgicos, evolucin del embarazo en cada consulta, resultados de exmenes de laboratorio, estado nutricional, evolucin y resultado del
parto, condiciones del nio al nacimiento, evolucin de la primera semana del puerperio, factores de riesgo y mensajes que destaquen la importancia de
la lactancia materna exclusiva, planificacin familiar y signos de alarma durante el embarazo. Se utilizar ste, como documento de referencia y
contrarreferencia institucional. Para establecer el diagnstico de embarazo no se deben emplear estudios radiolgicos ni administrar medicamentos
hormonales. El control prenatal debe estar dirigido a la deteccin y control de factores de riesgo obsttrico, a la prevencin, deteccin y tratamiento de
la anemia, preeclampsia, infecciones crvicovaginales e infecciones urinarias, las complicaciones hemorrgicas del embarazo, retraso del crecimiento
intrauterino y otras patologas intercurrentes con el embarazo. La unidad de salud debe promover que la embarazada de bajo riesgo reciba como mnimo
cinco consultas prenatales, iniciando preferentemente en las primeras 12 semanas de gestacin y atendiendo al siguiente calendario: 1ra. consulta: en el
transcurso de las primeras 12 semanas. 2a. consulta: entre la 22 - 24 semanas. 3a. consulta: entre la 27 - 29 semanas. 4a. consulta: entre la 33 - 35
semanas 5a. consulta: entre la 38 - 40 semanas. La prolongacin del embarazo despus de las 40 semanas requiere efectuar consultas semanales
adicionales con objeto de vigilar que el embarazo no se prolongue ms all de la semana 42. DETECCIN DE FACTORES DE RIESGO: 1) Asociacin entre
exposicin a sustancias toxicas y efectos adversos en la madre o el feto (partos prematuros, hipertensin preeclampsia, bajo peso al nacer). 2) El exceso
de alcohol tienen efecto adverso en el feto, por lo que se sugiere que la mujer embarazada limite su consumo. 3) El tabaquismo se ha asociado a un
incremento en el riesgo de mortalidad perinatal, muerte sbita infantil, RPM, embarazo ectpico, placenta previa, parto prematuro, bajo peso al nacer,
desarrollo de labio y paladar hendido; por lo cual debe ser evitado. 4) El fumar marihuana se asocia a bajo peso al nacer, muerte perinatal, parto
prematuro, evitar su uso. 5) Se ha demostrado la utilidad de la suplementacin con Yodo en poblaciones con alto riesgo de cretinismo. 6) Tamizaje para
preeclampsia, con toma de presin, pruebas de orinas para detectar proteinuria, capacitacin para reconocer los sntomas de alarma de preeclampsia. 7)
Las vacunas con virus vicos atenuados estn contraindicadas durante la gestacin y solo se recomienda el toxoide tetnico. 8) Los profesionales de la
salud deben estar alertas sobre datos que orienten hacia violencia intrafamiliar, ya que se presenta hasta en 17% en el embarazo y ello dificulta la
atencin de la mujer. CONTROL EN LAS CONSULTAS: De acuerdo a guas de prctica clnica: La primera cita debe ser antes de las 12 semanas: Identificar
mujeres con riesgo alto de resultado adversos del embarazo (DM, HTA, preeclamsia, sndrome de Down, nefropatas, enfermedades de la colgena).
Descartar incompatibilidad a grupo y Rh, anemia hipocromica y megaloblasrtica; sfilis. Solicitar grupo y Rh, BH, EGO y VDRL. Deteccin de VIH y hepatitis
B. Bsqueda de bacteriuria asintomtica y protenas en orina. Envi de pacientes con factores de riesgo a segundo nivel. Calcular fecha probable de parto
por fecha de ltimo periodo menstrual, en caso de duda sobre la edad gestacional solicitar ultrasonido. Calcular y registra IMC y presin arterial. A las 16
semanas: revisar, discutir y registrar los resultados de todos los exmenes, plantear el diseo de cuidados. El embarazo no complicado, dar nueva cita, el
embarazo complicado o factores de riesgo, enviarlas a segundo nivel. En hemoglobinas menores de 11g/dl considerar suplemento de hierro. Medir y
registrar presin arterial. En 18-20 semanas: Debe realizarse un ultrasonido para detectar anormalidades estructurales. Para las pacientes en que se
encuentre la placenta cerca del OCI, realizar otro USG a la semana 36 medir y registra presin arterial. En 25 semanas en primigestas: Medir fondo
uterino, presin arterial y descartar la presencia de protenas en orina. En semana 28: un nuevo examen para detectar anemia y clulas atipias. Nivel
menor a 10.5 g/dl considerar implementacin de hierro. Ofrecer vacuna anti Rh de ser necesario. Medir fondo uterino, presin arterial y descartar la
presencia de protenas en orina. Realizar tamiz de diabetes gestacional. En 31 semanas: medir fondo uterino, presin arterial y descartar la presencia de
protenas en orina. Identificar mujeres embarazadas que requieren cuidados especiales. En semana 34: ofrecer segunda dosis de vacuna anti Rh en
mujeres Rh negativo. Medir fondo uterino, presin arterial y descartar la presencia de protenas en orina. En pacientes con tratamiento por anemia
revisar sus estudios de laboratorio de control, ajustar tratamiento. En embarazos con evolucin normal informar fecha probable de parto con medidas
preventivas para embarazo posmaduro y deteccin de preeclamsia. En semana 36: medir fondo uterino, presin arterial y descartar presencia de
protenas en orina. Determinar la posicin fetal, en caso de duda realizar USG. Mujeres con producto en presentacin plvica envi a ginecoobstetricia.
En semana 37: se debe descartar presentacin anormal del producto y asegurar atencin oportuna para evitar pos madurez. En semana 38: medir fondo
uterino, presin arterial y descartar la presencia de protenas en orina. Insistir en medidas preventivas de embarazo pos maduro. En semana 40 o ms:
medir fondo uterino, presin arterial y descartar presencia de protenas en orina, envi a urgencias GO para evaluar induccin. La ganancia de peso total
durante el embarazo en una mujer sana 7-18 Kg, se relaciona con neonatos de 3-4 kg. El incremento en el IMC se ha asociado con preeclampsia y con
cesrea. El peso y la estatura se deben medir y registrar en cada cita de atencin prenatal. Se debe calcula el IMC. En cada cita se debe determinar la
altura de fondo uterino para detectar productos mayores a menores para su edad gestacional; de preferencia utilizar tablas estandarizadas y la
aplicacin correcta de la tcnica. Aunque la auscultacin del foco fetal confirma que el producto est vivo, parece no ofrecer otros datos; en general
mdicos y enfermeras suponen que es satisfactorio para el paciente. Se recomienda registrar la frecuencia del foco fetal en cada consulta. El examen

CURSO ENARM CMN SIGLO XXI TEL: 36246001

Pharmed Solutions Institute

PGINA 249

MANUAL DE TRABAJO DEL CURSO ENARM CMN SIGLO XXI


plvico se ha utilizado para identificar condiciones clnicas como: anormalidades anatmicas, ETS, evaluar tamao de la pelvis de la mujer y evaluar el
crvix uterino, incompetencia cervical o para predecir parto pretermino. Sin embargo la RPM aumenta 3 veces ms cuando se realiza comparacin con
las mujeres en las que no se realiza exmenes plvicos. No se recomienda el tacto vaginal en forma rutinaria, pues no predice con exactitud la edad
gestacional, nacimientos pre trmino o desproporcin cefalo-pelvica y si favorece la RPM.
CASO CLINICO PLANEACION DE EMBARAZO.
Se trata de femenino de 23 aos de edad la cual acude a consulta por
odinofagia, malestar generalizado, cefalea, mialgias y artralgia, a la
exploracin fsica observa amgdalas hiperemicas e hipertrficas con
secrecin blanquecina, adenopata cervical, refiere que se enferma con
poca frecuencia, durante el interrogatorio la paciente le menciona su
inters por quedar embarazada ya que lleva casada 2 aos y as lo
desea, usted amplia su interrogatorio dirigido y no encuentra hallazgos
de importancia.
PREGUNTA
Cul es la mejor conducta a seguir con esta paciente para tener las
mejores condiciones para tener un embarazo con adecuada
planeacin?
RESPUESTAS
a.- Es conveniente indicar suplemento de Hierro y Vitaminas para
prevenir defectos.
b.- Realizar estudios tales como BH, QS y EGO para descartar
padecimiento no visibles.
c.- Realizar Papanicolaou y colposcopia para descartar virus del
papiloma humano.
d.- Realizar serologa para ETS y VIH ya que es una etapa donde se
podran evitar la transmisin al producto.
CASO CLINICO PREVENCION Y ATENCION DE VIH Y ETS
Se trata de paciente femenino de 29 aos de edad con amenorrea de
10 semanas, se realiza PIE resultando positivo, al interrogatorio la
paciente refiere nauseas, cefalea, vomito, prdida de peso
aproximadamente 5 kilos, refiere tabaquismo y consumo de alcohol
ocasionalmente, agrega que pocas ocasiones acude a consulta mdica,
sus antecedentes GO son Gesta 2, Para 1, Abortos 1, refiere que su
primer embarazo no fue planeado, actualmente vive con su segundo
esposo, el cual consume drogas inyectables, tienen 16 meses de
relacin, lo conoci en su trabajo donde labora como mesera, el
esposo es originario de Tijuana, a la exploracin fsica palpa una masa
en mama derecha la cual no es dolorosa, esta superficial, en espalda y
muslos observa equimosis en proceso de resolucin, la paciente refiere
que se pego accidentalmente, a la exploracin vaginal se observa
leucorrea con olor desagradable.
PREGUNTA
Considerando los factores de riesgo y la sintomatologa observada
cual es su conducta a seguir para identificar otros riesgos no visibles,
potencialmente prevenibles?
RESPUESTAS
a.- Realiza una bsqueda intencionada de ETS y VIH por los datos
indirectos aportados de la paciente.
b.- Enva interconsulta a trabajo social para identificar violencia
intrafamiliar ya que las lesiones que observa no se explican con la
informacin proporcionada.
c.- Indica suplemento de hierro y complejo vitamnico siguiendo la
normatividad.
d.- Considera que no tiene informacin necesaria para modificar su
manejo prenatal de forma distinta al de rutina.
CASO CLINICO CONTROL DE HAS
Se trata de paciente de 41 aos de edad la cual refiere 5 meses de no
presentar periodo menstrual, sin embargo decide acudir a consulta
porque tiene ardor al orinar y cefalea ocasional de intensidad leve, la
paciente refiere presentar estrs laboral y posiblemente perdida del
empleo, cuenta con antecedente de Hipertensin Arterial
diagnosticada a los 38 aos, actualmente tratada con ARA con buena
respuesta, a la exploracin fsica usted identifica crecimiento de

CURSO ENARM CMN SIGLO XXI TEL: 36246001

pezones, a la palpacin abdominal identifica presencia de tero a la


altura de cicatriz umbilical, ausculta y percibe frecuencia cardiaca fetal
de 140 lx.
PREGUNTA
Cules son los factores de riesgo ms importantes que se deber
mantener controlado por el embarazo actual?
RESPUESTA
a.- Hipertensin arterial preexistente.
b.- Preeclapsia.
c.- Glomerulopatia
c.- Descontrol hipertensivo.
CASO CLINICO EDAD GESTACIONAL
Paciente femenino de 21 aos de edad la cual acude a consulta debido
a que presenta un atraso del periodo menstrual de 4 semanas, refiere
nausea, mareo leve en dos ocasiones, cefalea pulstil en regin
occipital, sensibilidad mamaria, al interrogatorio refiere IVSA a los 20
aos sin mtodo contraceptivo, a la exploracin vaginal se observa
reblandecimiento del cuello uterino, a la exploracin de fondo uterino
se palpa a nivel de snfisis pbica.
PREGUNTA
Integrando signos y sntomas cual sera la edad gestacional mas
probable para esta paciente?
RESPUESTA:
a.- 4-6 semanas de gestacin.
b.- 6 a 12 semanas de gestacin.
c.- 12 a 18 semanas de gestacin
d.- 16 a 20 semanas de gestacin.
PREGUNTA
Hasta que semana es ideal no prescribirle medicamentos a esta
paciente?
RESPUESTA
a.- Semana 8
b.- Semana 10
c.- Semena 12
d.- Semana 14
PREGUNTA
La paciente dejo de acudir a sus consultas, reinicia y en la consulta el
fondo uterino se palpa a la altura de la cicatriz umbilical. Cuntas
semanas es mas probable que tenga la paciente?
RESPUESTA
a.- 16 semnas
b.- 20 semanas
c.- 24 semanas
d.- 28 semanas
CASO CLINICO CONTROL PRENATAL.
Acude a la consulta paciente con 10 semanas de amenorrea, refiere
que se realizo una prueba de embarazo en casa, resultando positiva,
refiere nausea matutina y vomito ocasional, durante la exploracin se
observa crecimiento mamario y pigmentacin, se observa lnea alba
pigmentada, la paciente presenta tabaquismo positivo y un ndice de
masa corporal de 17, usted indica medidas correctivas a estos factores
de riesgo.
PREGUNTA:
Considerando los factores de riesgo cual es su conducta a seguir para
mantener un buen control prenatal?
RESPUESTA

Pharmed Solutions Institute

PGINA 250

MANUAL DE TRABAJO DEL CURSO ENARM CMN SIGLO XXI


a.- Incrementa la dieta y vigila su estado nutricional para obtener un
producto con peso adecuado.
b.- Indica a la paciente que definitivamente debe suspender el
consumo de tabaco por el riesgo de bajo peso al nacer.
c.- Envia a la paciente a realizar BH, QS para valorar su estado general y
como protocolo segn la norma.
d.- Envia USG para identificar la edad gestacional y el estado del
producto ya que hay riesgos para cierre de tubo inadecuado.
PREGUNTA
Cada cuanto se debe citar a esta paciente para su control prenatal?
RESPUESTA
a.- Cada 5 semanas
b.- Cada 4 semanas
c.- Cada 3 semanas
d.- Cada 2 semanas
.
CASO CLINICO VACUNACION EN EMBARAZO
Paciente originaria de Chiapas la cual actualmente vive en el estado de
Mxico, refiere tener 27 aos, acude para control prenatal, presenta 8
semanas de amenorrea, niega uso de mtodo anticonceptivo, sus
antecedentes GO, gesta 3, para 2, abortos 0, menarca a los 12 aos,
IVSA a loa 19 aos.
PREGUNTA
Considerando la edad gestacional y el correcto control prenatal, cual
es la inmunizacin adecuada para esta paciente?

finado por Cncer no especificado. AGO: M: 11 aos, CM: 28-30x 34 das , dismenorrea no incapacitante, IVSA a los 21 aos , PS: 1, G2,
P1,C0, A 0. FUP: hace 2 aos, Papanicolau: ltimo el 13/ 02/13 con
resultados normales. DOCMA: hace un ao (-), MPF: ninguno. G: 1
(25/05/05), producto nico vivo femenino, peso de 2840grs. Talla de
46 cm. PC 34CM. Apgar: 8/9. Capurro de 38 semanas de gestacin.
Gesta actual: FUM: 23/08/13, FPP: 30/05/14, dx de embarazo durante
el 2 mes de embarazo mediante prueba inmunolgica. Control
prenatal: 4 citas. Inicia a las 14SDG. Consumo de cido flico y
fumarato ferroso desde el segundo mes de embarazo. Inmunizaciones
durante el 2 mes de embarazo: Toxide tetnico.
PREGUNTA
Qu medidas son las que ms corresponden realizar en esta paciente,
de acuerdo al caso?
RESPUESTA
a.- Identificacin de signos y sntomas de alarma, determinacin de
VIH, VDRL
b.- Identificacin de signos y sntomas de alarma, Revisar EGO, aplicar
2da de toxoide tetnico
c.- Identificacin de signos y sntomas de alarma, determinacin de
VIH, aplicar toxoide tetnico
d.- Identificacin de signos y sntomas de alarma, determinacin de
VDRL, grupo y Rh

RESPUESTA
a.- Usted indica vacuna de influenza.
b.- Toxoide tetnico.
b.- Vacuna de Hepatitis.
c.- Vacuna DPT

PREGUNTA
Cul sera el rango de peso ms adecuado que debera ganar la
paciente durante todo el embarazo?
RESPUESTA
a.- 9-18Kg
b.- 15-22Kg
c.- 5-12Kg
d.- 16-23Kg

PREGUNTA
Cundo citaria a esta paciente para su segunda consulta?
RESPUESTA
a.- A la semana 12
b.- A la semana 22
c.- A la semana 14
d.- A la semana 28

PREGUNTA
A qu semanas de gestacin citara nuevamente a esta paciente?
RESPUESTA
a.- 35 semanas de gestacin
b.- 38 semanas de gestacin
c.- 40 semanas de gestacin
d.- 41 semanas de gestacin

PREGUNTA
En su segunda consulta la paciente registra una glucosa de 132mg/dl.
Cul es su conducta a seguir mas adecuada?
RESPUESTA
a.- Inciar insulina
b.- Envio a segundo nivel
c.- Iniciar metformina
d.- Esperar a la siguiente consulta y hacer una nueva glucosa

CASO CLINICO
Femenino de 26 aos de edad de escolaridad primaria, G3, P1, C1, con
antecedente de cesarea hace 2 aos por preeclampsia severa,
evolucionanado con hipertensin crnica y dao renal secundario.
Tuvo seguimiento de patologa crnica por medicina interna, desde
hace 6 meses no acude a sus consultas. Reinicia atencin mdica por
amenorrea; se diagnstica embarazo de 10 semanas de gestacin. Se
inicia manejo conjunto con Nefrologa por datos bioqumicos de
insuficiencia renal, se medica con hidralazina y diurtico. Inicia con
crisis hipertensivas a las 22 SDG, se diagnostica como preclampsia
severa, presenta hemorragia cerebral por hipertensin y fallece.

PREGUNTA
Cada cuanto se debe citar a esta paciente por su riesgo?
RESPUESTA
a.- Cada 2 semanas
b.- Cada 3 semanas
c.- Cada 4 semanas
d.- Cada 5 semanas
CASO CLINICO
Femenino de 23 aos de edad con dx de embarazo de 33.3 SDG que
acude a su 4 cita de control prenatal refirindose asintomtica.
AHF: Madre
con DM2 e IRC en
DPCA.
Abuelo
paterno

CURSO ENARM CMN SIGLO XXI TEL: 36246001

PREGUNTA
Se pudo haber evitado la muerte de esta paciente con la interrupcin
del embarazo?
RESPUESTA
a.- Si, pues no hubiera evolucionado a hemorragia cerebral
b.- No, el curso de la patologa iba a ser el mismo
c.- Si, pero era decisin de la paciente
d.- No, el embarazo no influia en su patologa de base

Pharmed Solutions Institute

PGINA 251

MANUAL DE TRABAJO DEL CURSO ENARM CMN SIGLO XXI


EMBARAZO ECTOPICO (EE)
CIENCIAS BASICAS: Se define como la implantacin del vulo fecundado fuera de la cavidad uterina. Representa una de las causas ms importantes de
abdomen agudo en ginecologa. Alrededor del 95% de los embarazos ectpicos se localizan en la trompa (ampular 79,6 %, istmico 12,3 %, fmbrico 6,2 %,
intersticial 1,9 %). Las formas no tubricas del embarazo ectpico incluyen el embarazo cervical (0,1%), el embarazo ovrico (0,5%), el embarazo
abdominal (1,3%), el embarazo cornual (3%), el embarazo intraligamentario (0,1%), y el embarazo en cuerno uterino rudimentario. SALUD PUBLICA: El
embarazo ectpico es la causa ms frecuente de morbilidad y mortalidad materna en el primer trimestre de la gestacin, siendo responsable del 9% de
las muertes durante este periodo. En cuanto a la edad, afecta en el 482% a mujeres entre los 30-39 aos y en el 368% entre los 20-29 aos,
representando las nulparas el 344% y las que han tenido 1-2 hijos el 23,6%. PATOGENIA: Est determinada por todos aquellos factores que impiden o
dificultan la migracin normal del huevo hacia la cavidad uterina. Se pueden distinguir entre: 1) Factores tubulares: pueden ser debidos a) EPI y dentro
de ella la causada por la salpingitis gonoccica y la TBC genital (asintomticas en el 80-90% de casos) y la salpingitis posabortiva. Deterioran la actividad
ciliar y la motilidad tubrica y generan adherencias en la mucosa tubrica que conducen a la estenosis. b) ciruga tubrica previa contra la infertilidad
(21% riesgo relativo), c) ciruga tubrica para la esterilidad permanente (9.3%), d) endometriosis tubrica: favorece la nidacin precoz, e) anomalas
congnitas: hipoplasias (observada en la exposicin intratero al dietilestilbestrol) o trompas excesivamente largas y tortuosas, f) alteraciones
funcionales del peristaltismo tubarico o de la actividad ciliar. 2) Factores ovulares: aquellos que conducen a la; a) Nidacin precoz del huevo, bien sea
debido a huevos excesivamente pesados que maduran con rapidez y no pueden progresar, o al desprendimiento precoz de la capa pelcida del huevo
capacitando la implantacin, b) Transmigracin: se ha sugerido la posibilidad de la hipermigracin del huevo y su implantacin en la trompa
contralateral. En el 20% de los casos el cuerpo lteo se encuentra en el ovario contralateral al de la implantacin, c) Tcnicas de Reproduccin Asistida
(TRA): de todos los embarazos clnicos resultantes de las TRA, el 5-5,7% fueron ectpicos y el 1% fueron embarazos heterotpicos, debido a la
hiperestimulacin ovrica y a las implantaciones mltiples resultantes de la transferencia de varios embriones. Algunos autores afirman que la
implantacin ectpica est estrechamente ligada a la insuficiencia del cuerpo lteo, y que esta insuficiencia es un factor patognico comn a muchas
anomalas de la reproduccin, entre ellas algunas
cromosomopatas y abortos. Una vez implantado el huevo
en la trompa, las vellosidades invaden rpidamente el
endosalpinx, alcanzando la parte tubrica y el peritoneo.
Esta penetracin va acompaada de una proliferacin
vascular y de un hematoma peritubarico e hematosalpinx,
que a menudo afecta la trompa contraleteral, y que se
propaga hacia el extremo distal de la trompa, entre la
pared y la serosa. A partir de aqu puede evolucionar hacia
el: 1) Aborto tubarico, el huevo carece de vellosidades
suficientes y est implantando en el segmento distal de la
trompa. Se atrofia, se desprende y es expulsado por el orificio peritoneal de la trompa. Esta expulsin suele ir acompaada de hemorragia moderada,
que ocupa el fondo de saco de Douglas formando hematocele. En raras ocasiones puede convertirse secundariamente, en embarazo abdominal. El
aborto tubrico se da con ms frecuencia en la implantacin ampular y fmbrica de la trompa. 2) Rotura tubarica, las vellosidades son suficientes y
enrgicas, pero el aumento de la presin intatubarica llega a dificultar el riego sanguneo de la trompa, lo que produce necrosis tisular que conduce a la
ruptura por corrosin vascular (no por estallido). Esta rotura suele acompaarse de hemorragia intensa, que da lugar al hemoperitoneo y shock
hipovolmico, ya que suele afectar las anastomosis de las arterias tubricas con la uterina ascendente. La rotura tubrica se da con ms frecuencia en la
implantacin stmica e intersticial de la trompa. La metrorragia se manifiesta generalmente antes de la rotura, debido a la insuficiencia placentaria
(esteroidognesis reducida), que precede las manifestaciones clnicas. 3) Ectpico viable, lo habitual del embarazo ectpico es que progrese hasta el
segundo o tercer mes, momento en que se interrumpe, y rara vez llega a ser viable. DIAGNOSTICO: En algunas ocasiones, la interrupcin del embarazo
es tan precoz, que se resuelve por reabsorcin espontnea, sin llegar a dar manifestaciones ni de amenorrea. El embarazo ectpico accidentado no
precisa mucha pericia diagnstica, la paciente suele presentar un cuadro de schock hipovolmico y dolor abdominal agudo, confirmndose el
diagnstico en quirfano, previa puncin fondo de saco de Douglas. Las manifestaciones clnicas del embarazo ectpico no accidentado en evolucin, se
caracterizan en el 90% de los casos por prdidas hemticas irregulares y dolor abdominal. La triada clsica compuesta por dolor, sangrado y masa anexial
solo se presenta en 45% de las pacientes. A la exploracin fsica 30% de pacientes, no presenta sangrado transvaginal, y 10% pueden tener masa anexial
palpable y 10% tener un examen plvico normal. La localizacin ms frecuente del embarazo ectpico es en la tuba uterina, manifestndose con dolor y
sangrado, sin evidencias de embarazo intrauterino y con una masa anexial. El sangrado del embarazo tubario es lento, gradual y algunas pacientes
pueden permanecer hemodinamicamente estables a pesar de hemoperitoneos de 1000-1500ml. En toda paciente en edad reproductiva y con vida
sexual que presenta retraso menstrual, dolo y sangrado transvaginal debe descartarse la presencia de embarazo tubario a travs de ultrasonografa
plvica transvaginal y determinacin srica de -HCG. La deteccin por ultrasonido transvaginal de una masa anexial, combinada con una concentracin
de -HCG de 1000mUI/ml tienen una sensibilidad de 97%, una especificidad de 99% y unos valores predictivos positivo y negativo de 98% para
diagnstico de embarazo tubario. Para la deteccin temprana del embarazo tubario temprano el US transvaginal ofrece mayor sensibilidad que el plvico
abdominal por lo cual debe realizarse el primero si se cuenta con el recurso de lo contrario utilizar el pelvicoabdominal. Ante la duda diagnostica de
embarazo tubario, la evaluacin laparoscpica est indicada. Los niveles sricos de -HCG por si solos no diferencian un embarazo tubario de un
intrauterino. Las determinaciones seriadas de -HCG tienen 36% de sensibilidad y 65% de especificidad para la deteccin de embarazo tubario. Los
rangos de -HCG de la zona de discriminacin son de 1500-2000mUI y estos se asocian con la presencia ultrasonografica del saco gestacional
intrauterino. La determinacin srica -HCG es ms sensible que la determinacin en orina. TRATAMIENTO: Puede ser mdico, quirrgico y expectante.
El tratamiento mdico consiste en uso de metotrexate con o sin cido folinico, est recomendado en pacientes con: estabilidad hemodinmica,
embarazo tubario no roto, sin datos de sangrado activo intra-abdominal, pacientes con niveles sricos de -HCG menores de 2000mUI/ml, tamao del
saco gestacional menor a 3.5cm, ausencia de latido cardiaco embrionario. Existen tres protocolos de administracin de metotrexate para el tratamiento
de embarazo ectpico monodosis, doble dosis y multidosis. Se recomienda monodosis ya que los resultados son similares y con menos toxicidad.
Contraindicaciones absolutas: lactancia, disfuncin renal, heptica, pulmonar o hematolgica, hipersensibilidad a metotrexate, enfermedad acidopptica. Relativas: saco gestacional mayor a 3.5cm, actividad cardiaca embrionaria. El tratamiento quirrgico puede consistir en laparoscopia o
laparotoma. Sin indicaciones para el tratamiento quirrgico y embarazo tubario: pacientes no candidatas a tratamiento mdico con metrotexate, falla al
tratamiento mdico, embarazo heterotopico con embarazo intrauterino viable, pacientes hemodinamicamente inestables, pacientes con inestabilidad
hemodinmica deben ser estabilizadas antes de entrar a quirfano, en pacientes con embarazo tubario hemodinamicamente estables, se recomienda el
abordaje laparoscpico, siempre y cuando se cuente con el recurso material y humano necesarios. Ante las siguientes circunstancias se considera de
primera eleccin la laparotoma exploradora: Antecedentes de ciruga abdominal, presencia de adherencias plvicas, inexperiencia del cirujano

CURSO ENARM CMN SIGLO XXI TEL: 36246001

Pharmed Solutions Institute

PGINA 252

MANUAL DE TRABAJO DEL CURSO ENARM CMN SIGLO XXI


laparoscopia. Las indicaciones de salpingectomia incluyen: dao severo de la tuba uterina, embarazo tubario recurrente en la misma tuba uterina,
sangrado persistente despus de la salpingostomia, embarazo tubario mayor a 5cm, embarazo heterotopico, pacientes con paridad satisfecha. El manejo
expectante del embarazo tubario no est claramente establecido. En pacientes con embarazo tubario que son Rh negativo no sensibilizadas, se
recomienda la aplicacin de inmunoglobulina anti D a razn de 250UI. Un embarazo ovrico es difcil de distinguir de un quiste hemorrgico de cuerpo
lteo, incluso durante el acto quirrgico.
CASO CLINICO EMBARAZO ECTOPICO
Se trata de femenino de 19 aos de edad la cual se encuentra en
control prenatal, acude a su consulta de control, refiriendo que desde
hace una semana presenta dolor en fosa iliaca derecha, sensacin de
pesantez y distencin abdominal, agrega que ha tenido dificultad para
la evacuacin, refiere que el da previo a la consulta observa sangrado
leve transvaginal, la paciente es ingresada a urgencias por sospecha de
embarazo se realiza PIE resultando positiva, por lo que es ingresada
para observacin ya que el sangrado no ha disminuido y continua el
dolor abdominal.
PREGUNTA
Cul es la su conducta a seguir, considerando la evolucin del
paciente?
RESPUESTA
a.- Realizar Bh en busca de anemia y leucocitosis.
b.- Realizar USG transvaginal.
c.- Realizar cuantificacin de Gonadotrofinas.
d.- Cuantificacin de progesterona.
PREGUNTA
Cul es la localizacin menos frecuente de esta patologa?
RESPUESTA
a.- Tubaria
b.- Cervical
c.- Ovario
d.- Abdominal
CASO CLINICO
Paciente de sexo femenino de 23 aos de edad, que ingreso con un
cuadro clnico de ms o menos 3 das de evolucin el cual se
caracteriza por presentar mal estado general, alzas trmicas no
cuantificadas, escalofros, dolor pungitivo a nivel lumbar y polaquiuria
sin disuria hace aproximadamente 1 mes. Cursa con amenorrea de 7
semanas, ingresa portando un parcial de orina con leucocitos 22 por
campo, hemograma con leucocitosis de 14.900 con segmentados de
89% una fraccin beta cualitativa positiva. Antecedentes GinecoObstetricos; G1, C1. Menarca a los 12 aos con ciclos regulares cada 28
das, duracin entre 4 a 5 das, en moderada cantidad,
dismenorreicos. IVSA a los 16 aos, reconoce dos parejas sexuales,
conoce mtodos de planificacin familiar utilizo depoprovera durante
seis meses. Al examen fsico la paciente se encuentra consiente,
orientada, con mucosas hmedas y rosadas. SV: FC: 98 x, PA: 110/80
mmHg, FR: 34 x, T: 39. Se realiza ecografa transvaginal y se visualiza
a nivel cervical imagen redondeada que mide 5cm con halo
refringente, presencia de vescula vitelina
PREGUNTA
A partir de que semana es ms probable que sea audible la frecuencia
cardiaca fetal por USG transvaginal?
RESPUESTA
a.- En semana 5-6 de gestacin
b.- En semana 6-7 de gestacin
c.- En semana 7-9 de gestacin
d.- En semana 9-10 de gestacin
PREGUNTA
Cul es la conducta teraputica ms adecuada a seguir en este caso?
RESPUESTA
a.- Histerectomia abdominal
b.- Metotrexate

CURSO ENARM CMN SIGLO XXI TEL: 36246001

c.- AMEU
d.- LUI
PREGUNTA
Cul es la contraindicacin menos probable para usar metotrexate en
la paciente?
RESPUESTA
a.- Inmunodeficiencia
b.- Trombocitopenia moderada
c.- Embarazo intrauterino
d.- Niveles de hCG >5000mIU/ml
PREGUNTA
Cul es la incidencia de embarazo ectpico?
RESPUESTA
a.- 10%
b.- 5%
c.- 8%
d.- 2%
CASO CLINICO
Paciente de 28 aos de edad quien acude al servicio de consulta
externa del Hospital, por presentar manchado escaso transvaginal de
dos das de evolucin, sin referir dolor. Con antecedentes ginecoobsttricos de: Menarca a los 13 aos, dos embarazos, el primero
obtenido por cesrea debido a una desproporcin cfalo-plvica, y el
segundo, el que est en discusin. Cuenta con historia de cervicitis
crnica tratada con vulos de Ketanserina, Miconazol y Metronidazol,
criociruga, as como enfermedad inflamatoria plvica tratada con
Clindamicina.Refiere no haber usado mtodo anticonceptivo en los 2
ltimos aos. En la exploracin por especuloscopa se detecta escaso
manchado transvaginal. Se realiza ultrasonido transvaginal
encontrndose embarazo de 8.0 semanas de gestacin, localizado en
la cicatriz de cesrea previa, con actividad cardiaca fetal de 150 latidos
por minuto.
PREGUNTA
Cul es la conducta teraputica mas adecuada a seguir en este caso?
RESPUESTA
a.- Dejar evolucionar el embarazo
b.- Realizar una laparotoma, obteniendo el producto
c.- Hacer uso de Metotrexate
d.- Pacticar un LUI
PREGUNTA
Cul es la causa menos probable del embarazo ectpico de esta
paciente?
RESPUESTA
a.- Enfermedad pelvca inflamatoria
b.- Salpingitis
c.- Cesarea previa
d.- Uso excesivo de antimicticos y antibiticos
PREGUNTA
Cul seria el diagnostico diferencial menos probable en este caso?
RESPUESTA
a.- Amenaza de aborto
b.- Apendicitis
c.- Diverticulitis
d.- Quiste de ovario

Pharmed Solutions Institute

PGINA 253

MANUAL DE TRABAJO DEL CURSO ENARM CMN SIGLO XXI


ABORTO, AMENAZA DE ABORTO
CIENCIAS BASICAS: Aborto es la expulsin o extraccin del producto de la concepcin antes de las 22SDG, el feto habr de pesar menos de 500gr. SALUD
PUBLICA: A mediados de los aos 90 se calculaba una cifra anual de un milln 700 mil abortos, estimndose que alrededor de la mitad eran inducidos Se
presenta aborto espontaneo en 15% de los embarazos reconocidos y del 13-26% de los no reconocidos. En general 80% de los abortos espontneos
ocurren en las primeras 12 semanas de gestacin. PATOGENIA: Factores de riesgo: Edad materna avanzada (en mayores de 40 aos se duplica el riesgo).
Aborto recurrente (ms de dos prdidas consecutivas aumenta el riesgo al 43%). Perodo intergensico largo (mayor a 10 aos). Bajo peso y sobrepeso
materno. Intervalo de ovulacin-implantacin prolongado (mayor a 72 horas). Bajo nivel de folatos durante la gestacin (nivel srico </= 2.19ng/ml).
Alcohol (no ha sido establecido nivel seguro en ninguna etapa del embarazo). Cocana. Cafena (consumo de ms de 100mg por da = 4 a 5 tazas).
Cigarrillo (padre o madre): consumo de ms de 10 cigarrillos por da. AINES (consumo cerca a la concepcin por su mecanismo de accin sobre las
prostaglandinas). Etiologa: Anomalas genticas 50-60% (la ms frecuente, aneuploidas: trisomas autosmicas 52%, monosoma X 19%, poliploidas
22% y otras 7%). Anomalas endocrinas 10-15%, desprendimientos corio-amniticos 5-10%, crvix incompetente 8-15%, implantacin anormal de la
placenta 5-15%, infecciones maternas agudas 3-5%: listeria monocitogenes, toxoplasma, parvovirus B19, rubeola, herpes simplex y CMV. Anomalas
inmunolgicas: 3-5%. Anomalas anatmicas uterinas: 1-3% (malformaciones: septo uterino, leiomiomas, adherencias). Causas desconocidas: < 5%.
DIAGNOSTICO: Clnico; amenorrea + sangrado vaginal + dolor hipogstrico. Historia clnica. Examen fsico: revela la fuente de sangrado. Ecografa
transvaginal: hallazgos de saco vitelino normal y actividad cardaca fetal (5,5-6 SDG). Es tranquilizador en etapas tempranas de la gestacin. El
diagnostico definitivo de embrazo intrauterino no viable puede ser hecho con los siguientes criterios: ausencia de actividad cradica en un embrin con
una longitud caudal >5mm, saco vitelino irregular y ausencia de polo fetal con un dimetro promedio de saco gestacional >25mm transabdominal o
18mm transvaginal. Cuando cualquiera de estos hallazgos este presente repetir el US en 1 semana: muy probablemente mostrara la perdida gestacional.
Estudios de laboratorio: Medicin de -HCG 8mediciones seriadas). RECOMENDACIONES GENERALES: Realizar ecografa transvaginal despus de la
expulsin espontnea o manejo mdico. Observacin al menos 30 min posterior al procedimiento, en caso de curetaje. Inmunoglobulina anti-D (despus
de evacuacin o al momento del diagnstico en caso de manejo expectante) con incompatibilidad Rh. Vacunacin antirrubola y sarampin posaborto si
no ha sido vacunada. Solicitar VDLR. La menstruacin debe reorganizarse en 6 semanas. Los niveles de B-hCG retornan a lo normal a las 2 o 4 semanas
posaborto. Iniciar anticoncepcin de inmediato. AMENAZA DE ABORTO: Es la presencia de hemorragia genital indolora, dolor leve suprapbico, crvix
cerrado, tero de tamao adecuado para la edad gestacional y actividad cardaca fetal demostrada por ecografa. Diagnstico: Amenorrea secundaria,
prueba inmunolgica de embarazo positiva, presencia de vitalidad fetal por ultrasonido, hemorragia de magnitud variable, dolor tipo clico en
hipogastrio de magnitud variable, volumen uterino acorde con amenorrea, sin dilacin cervical. Solicitar prueba de embarazo y bsicos. Tratamiento:
Disminuir la actividad fsica, psquica y sexual, reposo en cama, hidratacin va oral, se puede administrar antiespasmdicos. Si continua con hemorragia
referir a segundo nivel de atencin. No hay intervenciones teraputicas para prevenir la prdida en el 1er trimestre. Se hospitaliza a todas las pacientes
que continen con sangrado transvaginal. Realizar -HCG y ultrasonido plvico para completar evolucin clnica. Hidratacin oral y parenteral. ABORTO
INEVITABLE: Sangrado abundante, contracciones uterinas dolorosas, crvix abierto, tejido gestacional puede visualizarse a travs del orificio interno, y
ruptura prematura de membranas antes de las 20 semanas. Variedad que hace imposible la continuacin de la gestacin generalmente por la existencia
de hemorragia genital intensa o ruptura de membranas, aun sin modificaciones cervicales o actividad uterina reconocible. Diagnstico: Dolor tipo clico
en hipogastrio, volumen uterino igual o menor que por amenorrea, hemorragia abundante o ruptura de membranas, puede o no haber dilacin cervical.
Manejo quirrgico: curetaje (preferiblemente evacuacin por aspiracin) Manejo expectante: realizar en paciente estable, sin signos de infeccin y con
gestaciones menores a 13 semanas. La expulsin espontnea ocurre en las primeras 2 semanas del diagnstico en el 80% (puede prolongarse a 3 o 4
semanas). Manejo mdico con misoprostol: I trimestre: 800mcg va vaginal cada 6 a 12 horas, hasta 3 dosis, u 800mcg sublingual cada 3 a 4 horas hasta 3
dosis Iniciando II trimestre: 400 mcg vaginal entre 13 y 15 semanas, y 200mcg vaginal entre 16-20 SDG. No olvidar las reacciones adversas del
misoprostol: diarrea y dolor abdominal (10%), cefalea, constipacin, flatulencia, vomito, dispepsia y en <1% ansiedad, anafilaxia, fiebre, trombocitopenia,
hipotensin. ABORTO EN EVOLUCION: Se caracteriza por la presencia de hemorragia genital persistente, actividad uterina reconocible clnicamente y
modificaciones cervicales incompatibles con la continuidad de la gestacin. Diagnstico: expulsin del tejido ovular inminente, dolor tipo clico
progresivo en intensidad y frecuencia, volumen uterino menor que por amenorrea, hemorragia persistente, dilatacin cervical. Tratamiento:
Hospitalizacion, se realizara la AMEU y el LUI como procedimientos de primera y segunda opcin respectivamente. ABORTO INCOMPLETO: Cuando ha
ocurrido la expulsin de una parte del huevo y el resto se encuentra an en cavidad uterina. Diagnstico: expulsin parcial del producto de la
concepcin, hemorragia y dolor tipo clico de magnitud variable, dilacin cervical y volumen uterino no acorde con amenorrea. Tratamiento: Legrado si
el dimetro del tejido retenido es igual o mayor a 15mm, para evitar complicaciones hemorrgicas e infecciosas; adems, es la eleccin si ocurre antes de
12 semanas. Algunos estudios han sugerido que el manejo expectante del aborto incompleto en pacientes estables puede ser seguro, o se puede dar
manejo mdico con Misoprostol 600 mcg va oral, o 400 mcg va sublingual, dosis nica y control en 7 das. ABORTO COMPLETO o consumado: Aquel en
el que la expulsin del huevo ha sido total. Diagnostico expulsin completa del producto de la concepcin, disminucin de la hemorragia y el dolor, se
favorece el cierre del orificio cervical. Tratamiento: Generalmente no necesita manejo adicional, si se tienen duda manejarlo como aborto incompleto.
ABORTO DIFERIDO o huevo muerto retenido: Se presenta cuando habiendo ocurrido la muerte del producto de la concepcin, no se expulsa en forma
espontnea. Esta entidad presupone un lapso entre la muerte ovular y la elaboracin del diagnstico. Generalmente existe el antecedente de amenaza
de aborto. Diagnstico: volumen uterino menor que por amenorrea, ausencia de vitalidad fetal (latido cardiaco fetal), no hay modificaciones cervicales.
Los abortos del segundo trimestre de este tipo pueden asociarse a trastornos de la coagulacin. Tratamiento: Manejo quirrgico: curetaje
(preferiblemente evacuacin por aspiracin) Manejo expectante: realizar en paciente estable, sin signos de infeccin y con gestaciones menores a 13
semanas. La expulsin espontnea ocurre en las primeras 2 semanas del diagnstico en el 80% (puede prolongarse a 3 o 4 semanas). Manejo mdico con
misoprostol: I trimestre: 800 mcg va vaginal cada 6 a 12 horas, hasta 3 dosis, u 800 mcg sublingual cada 3 a 4 horas hasta 3 dosis Iniciando II trimestre:
400 mcg vaginal entre 13 y 15 semanas, y 200 mcg vaginal entre 16-20 SDG. No olvidar las reacciones adversas del misoprostol: diarrea y dolor
abdominal (10%), cefalea, constipacin, flatulencia, vomito, dispepsia y en <1% ansiedad, anafiulaxia, fiebre, trombocitopenia, hipotensin. ABORTO
HABITUAL: Es la perdida repetida y espontanea del embarazo en 3 o ms ocasiones, o de 5 embarzos en forma alterna. Requiere por ser de origen
multifactorial, de una adecuada valoracin clnica y estudios de laboratorio y gabinete para determinar su causa, de un tratamiento especializado y
posteriormente de una atencin prenatal precoz y con calidad durante todo el embarazo. ABORTO SEPTICO: Cualquiera de las variedades anteriores a
las que se agrega infeccin intrauterina. Diagnstico: Escurrimiento de mal olor e incluso secrecin hemato-purulenta a travs del crvix. Fiebre sin
ningn otro sitio evidente de infeccin, hipersensibilidad suprapubica, dolor abdomino-pelvico a la movilizacin del crvix y tero, ataque al estado
general. El examen fsico se encuentra taquicardia, taquipnea, dolor hipogstrico, tero friable y doloroso con crvix dilatado. Es una complicacin
frecuente de abortos ilegales. Agentes involucrados: staphylococcus aureus, bacilos gram negativos, cocos gram positivos, infecciones mixtas,
organismos anaerobios y hongos. Clasificacin: Leve: infeccin ovular. Moderado: respuesta inflamatoria sistmica secundaria a enfermedad localizada.
Severo: sepsis - inflamacin sistmica + bacteriemia + Choque sptico sndrome de falla multiorgnica. Tratamiento: Referir al paciente a segundo nivel
de atencin, independientemente de la edad gestacional. Soporte hemodinmico. Antibiticos de amplio espectro; Leve: cefalosporina de 1 generacin.

CURSO ENARM CMN SIGLO XXI TEL: 36246001

Pharmed Solutions Institute

PGINA 254

MANUAL DE TRABAJO DEL CURSO ENARM CMN SIGLO XXI


Moderado: clindamicina + gentamicina, Ampicilina-sulbactam. Severo: penicilina cristalina + clindamicina (600mgs IV c/8h) + gentamicina (80mg IV o IM
c/8h). Hidrocortisona 3 gr IV dosis inicial y repetir 1g IV c/8h en las siguientes 24h. Si no hay respuesta histerectoma o anexohisterectoma. Evacuacin
del contenido uterino. Profilaxis - ttanos
CASO CLINICO ABORTO
Paciente femenino de 24 aos de edad, originaria de Campeche, en
comunidad rural productora de aves, acude debido a que presenta
amenorrea secundaria de 8 semanas de evolucin, ella refiere que
desde hace tres das ha presentado dolor lumbosacro y presencia de
sangre roja rutilante, por lo que se mantiene en reposo; cuenta con los
antecedentes GO de Gesta 4, para 0, abortos 3, ella refiere que sus
embarazos han terminado en abortos a las pocas semanas.
PREGUNTA
Tomando en cuenta el cuadro clnico actual y los antecedentes GO,
cul es su conducta a seguir para establecer una aproximacin
diagnostica?
RESPUESTA
a.- Realizar USG en bsqueda de anomalas uterinas.
b.- Buscar anticuerpos antifosfolipidicos y anticardiolipinas.
c.- Prueba TORCH.
b.- Determinacin de gonadotrofina corionica.
CASO CLINICO TORCH
Se trata de femenino de 23 aos de edad que acude a control prenatal,
actualmente cuenta con 22 semanas de gestacion, refiere que es
empleada de una granja y que ha presentado molestias tales como
dolor en la regin lumbo-sacra que se extiende hacia la regin
abdominal baja, cuenta con antecedentes de dos abortos expontaneos
y por eso se mantiene preocupada cuando presenta estas molestias.
PREGUNTA
En caso de llegar a trmino el producto y considerando la exposicin
laboral, cul sera la complicacin ms probable en el Recien Nacido?
RESPUESTA
a.- Malformacion Cardiaca.
b.- Sordera.
c.- Malforacion Oftalmica.
d.- Malformacion Renal.
CASO CLINICO AMENAZA DE ABORTO
Se trata de femenino de 25 aos de edad la cual acude al servicio de
urgencias con dolor en ambas extremidades inferiores, clicos, ardor al
orinar, presencia de manchado transvaginal, refiere que tiene tres das
con esta sintomatologa y decidi acudir porque se incrementaron los
sntomas, al interrogatorio refiere contar con 6 semanas de embarazo
por FUM, actualmente bajo control prenatal con mdico familiar, sus
antecedentes GO son Gesta 2 para 1, Abortos 0, refiere que el
embarazo previo fue sin complicaciones al igual que el parto, niega
toxicomanas ni presencia de otra patologa.
PREGUNTA
Luego de ingresar a la paciente a observacin, se realiza cuantificacin
de gonadotrofinas observandoce dentro del rango, disminuyendo la
sintomatologa al mantenerse en reposo, cual es la conducta a seguir?
RESPUESTA
a.- Mantener a la paciente en observacin debido a que puede
progresar la amenaza de aborto.
b.- Iniciar con medicacin antiespamodica y vigilancia en casa con cita
abierta.
c.- Dar indicacin de reposo absoluto y signos de alarma con cita
abierta.
d.- Indicar antibitico, indicar reposo, dar signos de alarma y cita
abierta.

CURSO ENARM CMN SIGLO XXI TEL: 36246001

CASO CLINICO ABORTO


Se trata de paciente femenino de 28 aos de edad la cual acude al
servicio de urgencias ya que refiere que aproximadamente 4 horas en
el trayecto a su trabajo presenta dolor intenso en regin lumbar con
irradiacin a miembros plvicos, y sangrado transvaginal moderado,
refiere que cuenta con 9 semanas de embarazo aparentemente
normal y bajo control mdico, al ingreso a urgencias observa el cuello
reblandecido y 2 centmetros de dilatacin, y salida de material, se
indica USG transvaginal y se observa saco amnitico abierto sin
presencia determinada de producto en su interior.
PREGUNTA.
Cul es su conducta a seguir?
RESPUESTA
a.- Envio a segundo nivel.
b.- Ingreso para vigilancia estricta.
c.- Manejo ambulatorio.
d.- Determinacin de hGC.
CASO CLINICO
Femenino de 24 aos de edad gestante de 8 semanas
aproximadamente. Antecedentes ginecolgicos, Menarqua: 13 aos,
ciclo menstrual duracin de 28 das, menstruacin de 4 das, irregular y
de abundante cantidad. Mtodos anticonceptivos: ninguno. Refiere
que su cuadro clnico inici en horas de la maana al realizar sobre
esfuerzo domstico, inicialmente presento dolor plvico de leve
intensidad luego evoluciono con presencia de sangrado transvaginal de
moderada cantidad y presencia de cogulos, acompaado de dolor
leve en la regin lumbosacra, que se intensific e irradi hacia el
hipogastrio siendo de tipo espasmdico y que no cesaba. Es ingresada
a urgencias, donde le detectan restos embrionarios, dilatacin cervical
de 2 cm.
PREGUNTA
Cul es el diagnstico ms probable?
RESPUESTA
a.- Aborto inevitable
b.- Aborto en evolucin
c.- Aborto incompleto
d.- Aborto diferido
PREGUNTA
De acuerdo al diagnstico Qu caractersticas es menos probable
encontrar en esta paciente?
RESPUESTA
a.- Expulsin parcial del producto de la concepcin
b.- Hemorragia y dolor tipo clico de magnitud variable
c.- Dilatacin cervical y volumen uterino no acorde con amenorrea
d.- Hipersensibilidad a la movilizacin del crvix
PREGUNTA
Cul es la conducta teraputica ms adecuada?
RESPUESTA
a.- LUI
b.- AMEU
c.- Administrar clindamicina ms gentamicina
d.- Administar penicilina ms gentamicina

Pharmed Solutions Institute

PGINA 255

MANUAL DE TRABAJO DEL CURSO ENARM CMN SIGLO XXI


INCOMPETENCIA CERVICAL (IC)
CIENCIAS BASICAS: Es la entidad clnica en la cual el cuello uterino es incapaz de ejercer su funcin hasta el trmino del embarazo, con perdida repetida
del producto en el segundo trimestre (parto prematuro). Un crvix incompetente es aquel que, por una anomala estructural o funcional, es incapaz de
mantener un embarazo hasta su trmino. En realidad esta incompetencia del crvix se localiza en el orificio cervical interno y el istmo, de ah que
muchos hablen de insuficiencia stmico cervical. SALUD PUBLICA: Su incidencia vara entre el 0,005 y el 1% de todos los embarazos, y se considera la
principal causa de abortos tardos. PATOGENIA: Se caracteriza por la dilatacin cervical pasiva y por la sucesin de prdidas gestacionales durante el
segundo trimestre. No se acompaa de metrorragia ni de rotura prematura de membranas. Aunque no suele encontrarse una historia obsttrica clsica
en estas pacientes, deben investigarse los factores etiolgicos previos. En realidad la IC es un sndrome al cual se puede llegar por diferentes causas:
clsicamente se admite la 1) Causa congnita; puede darse en los casos de malformaciones uterinas (mujeres expuestas al dietilestilbestrol durante su
embarazo). 2) Causa traumtica. En los casos de partos traumticos vaginales instrumentales o no, pueden producirse desgarros que afecten al canal
cervical y sean la causa de la IC. Otras veces son los legrados con dilatacin los responsables del cuadro. Por ltimo pueden incluirse en este apartado las
intervenciones quirrgicas sobre el crvix; cada da son ms frecuentes las conizaciones en edad de procrear y, dependiendo de la tcnica y el tamao
del cono, la secuela de la IC puede producirse del 5 al 15% de estas mujeres. 3) Causa oculta. Incluimos aqu todas las IC sin aparente lesin pero a pesar
de lo cual se producen de forma reiterada abortos del segundo trimestre o partos muy prematuros. Son posiblemente casos en los cuales hay una
alteracin histoqumica de los tejidos que constituyen el cuello y de una forma muy especial del colgeno. DIAGNOSTICO: Es una entidad en la que el
diagnstico precoz es fundamental, ya que cuando se objetiva una protrusin cervical de membranas en reloj de arena, el pronstico es peor. En la
anamnesis es esencial buscar factores de riesgo de la incompetencia cervical, como antecedentes de partos distcicos con frceps o ventosa, en los que
se hayan podido producir desgarros cervicales extensos que hayan dejado al crvix incompetente para sus funciones; dilataciones cervicales traumticas
para legrados por abortos o en interrupciones voluntarias de embarazo, o intervenciones quirrgicas sobre el crvix, como conizaciones o tratamientos
destructivos por patologa cervical. En algunos casos, la etiologa del crvix incompetente no es de origen traumtico sino congnito, entidad en realidad
extremadamente rara y a cuyo diagnstico se llega tras constatar la existencia de una incompetencia cervical sin antecedentes lesivos cervicales. La
incompetencia cervical congnita se ha relacionado con la toma materna de dietilestilbestrol durante la gestacin y con la existencia de posibles
malformaciones uterinas. La ecografa transvaginal tiene un papel de primer orden en la valoracin de las caractersticas cervicales en el diagnstico de
incompetencia cervical y en la prediccin de parto prematuro, que afecta al 8-10% de los embarazos con o sin rotura prematura de membranas. Su
aportacin al diagnstico de la modificacin cervical sin la existencia de dinmica uterina es fundamental. Desde el punto de vista ecogrfico se ha
demostrado que cuanto menor es la longitud cervical ms posibilidades de parto prematuro existen. En la evaluacin ecogrfica cervical, entienden las
siguientes como imgenes de normalidad: crvix formado, mayor de 30 mm; OCI cerrado, menor de 5 mm; canal cervical con bordes paralelos; canal en
forma de cono, con angulacin dirigida al OCI, y ausencia de herniacin de la bolsa amnitica. Estos criterios de normalidad siempre se deben tener en
cuenta en la evaluacin cervical para realizar el diagnstico de incompetencia cervical, adems del diagnstico ecogrfico, que tambin es fundamental
en el control y seguimiento de esta patologa. Clsicamente, el diagnstico de incompetencia cervical se realizaba mediante exploracin, por la deteccin
de repetidas prdidas gestacionales o de antecedentes traumticos cervicales. Hoy da, la cervicometra y la observacin del orificio cervical interno (OCI)
mediante ecografa son imprescindibles para su diagnstico y seguimiento. Se sabe que la longitud cervical no es constante durante todo el embarazo,
sino que disminuye a medida que ste avanza. Para la apertura del OCI debern considerarse los valores patolgicos que, antes de la semana 28, sean
inferiores a 30 mm y superiores a 10 mm. TRATAMIENTO: 1) Tratamiento antes del embarazo: Es siempre quirrgico y tiene como objetivo reconstruir la
normal anatoma del cuello. Se han propuesto diferentes tcnicas: traquelorragia, reduccin del calibre stmico cervical, reparacin de desgarros y
cerclaje. Todos estos procedimientos han sido prcticamente desechados dados los malos resultados obtenidos. 2) Tratamiento durante el embarazo. Se
ha propuesto tratamiento mdico y tratamiento quirrgico. El tratamiento mdico consiste en reposo, abstinencia sexual y administracin de tocolticos
o de progesterona, en este tipo de tratamiento no ha demostrado, ningn tipo de eficacia, por lo que a excepcin del reposo, hoy da no se utiliza.
Tratamiento quirrgico, es de eleccin y consiste en el cerclaje. Hay numerosas tcnicas de cerclaje pero las utilizadas casi exclusivamente son: la de Mc
Donald y Shirodkar, ambas consisten en hacer un lazo alrededor del cuello por va transvaginal y el perodo ms apropiado para su realizacin es entre
las semanas 13 y 16 del embarazo. Excepcionalmente se preconiza la realizacin del cerclaje por va transabdominal. Esta tcnica se realiza en el
quirfano, ya que se requiere el uso de anestesia general, buenas condiciones de asepsia y control ecogrfico para valorar la correcta realizacin del
cerclaje. El cerclaje cervical se debe realizar si la gestacin es inferior a 20 semanas y el diagnstico est confirmado mediante ecografa. Si la gestacin
es superior a 26 semanas, el cerclaje no estara indicado en ningn caso. La actitud correcta que se debe adoptar es estar en reposo absoluto, llevar a
cabo un control exhaustivo del embarazo y valorar toclisis mientras se realiza maduracin pulmonar fetal con betametasona. El cerclaje puede ser
utilizado en tres circunstancias clnicas: cerclaje en casos diagnosticados de insuficiencia stmico-cervical, el cerclaje en caso de IC oculta (cuando los
datos ecogrficos se alteran haciendo temer el parto pretrmino) y, por ltimo, lo que se denomina cerclaje de emergencia que es aquel que se realiza
cuando ya existe dilatacin e incluso prolapso de las membranas en vagina. Cuando se practica el cerclaje entre las semanas 12 y 18 cuando an no
existe dilatacin, se logra un feto viable del 70 al 80% de los casos, los resultados suelen ser mejores cuando ms precozmente se realiza. El cerclaje est
contraindicado en las malformaciones fetales graves, cuando existen contracciones, en los casos de coriamnioitis y es muy discutible cuando hay rotura
de las membranas. Existen algunas publicaciones de abortos tardos de uno de los gemelos en la que se ha realizado cerclaje logrando prolongar la
gestacin hasta la viabilidad del feto. Estos casos son excepcionales y la indicacin muy discutible.
CASO CLINICO INCOMPETENCIA CERVICAL
Se trata de paciente femenino de 25 aos de edad la cual tiene 21
semanas de gestacin actualmente bajo control prenatal, refiere
presentar clicos tipo menstrual niega sangrado transvaginal, sin
embargo el dolo se extiende a las piernas y regin sacra, que
disminuye al reposo, se realizo USG con diagnostico de embarazo de
22 SDG, al tacto se percibe remblandecimiento de cuello uterino, no se
observa dilatacin cervical, por lo que se enva a domicilio con
indicacin de reposo absoluto, a la semana vuelve la paciente a
revisin, donde refiere presencia de leve sangrado transvaginal y
clicos de las mismas caracterstica, se realiza valores de
gonadotrofina con datos normales, al tacto se observa un cm de
dilatacin y remblandecimiento de cuello cervical.
PREGUNTAS

CURSO ENARM CMN SIGLO XXI TEL: 36246001

Considerando el cuadro clnico y considerando las complicaciones


potenciales durante el embarazo, Que procedimiento es conveniente
realizar?
RESPUESTA
a.- Indica antibitico profilctico.
b.- Envio a segundo nivel para cerclaje cervical.
c.- USG para establecer la viabilidad fetal.
d.- Evaluacin de fibronectina fetal.
CASO CLINICO CERCLAJE CERVICAL
Femenino de 31 aos de edad la cual acude a consulta para control
prenatal, refiere 4 semanas de retraso menstrual, muestra un reporte
de laboratorio con PIE positivo, se encuentra ansiosa debido a que ha
presentado 3 abortos espontneos, fue enviada a GO para ser
diagnosticada, es referida a la consulta de control prenatal en

Pharmed Solutions Institute

PGINA 256

MANUAL DE TRABAJO DEL CURSO ENARM CMN SIGLO XXI


medicina familiar con diagnostico de incompetencia cervical y con
tratamiento de cerclaje cervical.
PREGUNTA
Con el actual tratamiento, cual es la probabilidad de llegar a un
embarazo a trmino con producto vivo?
RESPUESTA
a.- 70 a 80 %
b.- 60 a 70 %
c.- 80 a 90 %
d.- 50 a 60 %.
PREGUNTA
En que semana es ideal retirara el cerclaje cervical?
RESPUESTA
a.- Semana 28 de gestacin
b.- Semana 32 de gestacin
c.- Semana 35 de gestacin
d.- Semana 37 de gestacin
CASO CLINICO
Paciente femenino de 32 aos. En 2007 acude a consultar por
infertilidad primaria de 1 ao. Durante el interrogatorio reere ciclos
menstruales irregulares, y niega otros antecedentes. A la exploracin
fsica destaca la presencia de obesidad e hirsutismo; crvix con
cervicitis leve. Mediante ecografa transvaginal se observa tero y
crvix de caractersticas normales, ovarios con mltiples quistes en
rueda de carreta. Histerosalpingografa normal. Se diagnostica
Sndrome de Ovario Poliqustico y se inicia tratamiento con inductores
de la ovulacin, dieta y metformina. Al no lograrse embarazo se realiza
laparoscopia diagnstica y se encuentra endometriosis leve, la cual es
cauterizada. Posteriormente, se diagnostica cervicitis erosiva por lo
que se realiza criocauterizacin cervical y se resuelve el padecimiento.
Se suspende tratamiento por 2 aos y en 2010 se inicia protocolo de
hiperestimulacin ovrica controlada con inseminacin intrauterina, se
realizan un total de 4 ciclos sin lograr embarazo. Se efecta
fertilizacin in vitro y se logra embarazo de tres embriones en primer
intento, el cual evoluciona de manera favorable hasta la semana 16
cuando se diagnostican membranas en reloj de arena.
PREGUNTA
Cul es la conducta teraputica ms adecuada a seguir?
RESPUESTA
a.- Reposo hidratacin, tocioliticos
b.- Esteroides, reposo y tocolisis
c.- Cerclaje cervical
d.- Interrupcin de embarazo y antibioticoterapia intravenosa
PREGUNTA
Cul es la complicacin menos probable, que presentara esta
paciente?
RESPUESTA
a.- Corioamnioitis
b.- Trombosis venosa profunda
c.- Ruptura de membranas
d.- Desgarros cervicales
PREGUNTA
La incidencia de la incompetencia stmico cervical es de?
RESPUESTA
a.- 0.2-1%
b.- 3.5-10%
c.- 15-20%

CURSO ENARM CMN SIGLO XXI TEL: 36246001

d.- 3-5%
CASO CLINICO
Paciente de 34 aos de edad, casada, con antecedentes de
incompetencia cervical, colecistectoma laparoscpica debido a
colecistitis crnica litisica de cinco meses de evolucin y
timectomizada a los 18 aos de edad por miastenia gravis, sin
tratamiento farmacolgico actual. Al momento de su estudio cursaba
la 24 semana del quinto embarazo. Los partos previos fueron
vaginales, menores de 25 semanas, de hijos nacidos vivos pero que
fallecieron antes de la primera semana de vida. No padece
hipertensin arterial ni otra enfermedad crnica, y carece de
antecedentes de tabaquismo o alcoholismo. Tipo de sangre: AB Rh(-).
PREGUNTA
En que etapa es mas adecuada colocar el cerclaje a la paciente?
RESPUESTA
a.- Semana 10-12 de gestacin
b.- Semana 14-16 de gestacin
c.- Semana 20-24 de gestacin
d.- Semana 28-30 de gestacin
PREGUNTA
Cul es la causa menos probable por la que se debera retirar el
cerclaje antes de tiempo?
RESPUESTA
a.- Datos de infeccion intrauterina
b.- Rotura de membranas
c.- Metrorragia
d.- Contracciones uterinas
PREGUNTA
Qu patologa seria conveniente investigar por la presencia de partos
prematuros repetidos?
RESPUESTA
a.- Lupus eritematoso sistmico
b.- Hipertiroidismo
c.- Sindrome antifosfolipdico
d.- Isoinmunizacion
CASO CLINICO
Paciente sana de 33 aos, con un embarazo gemelar monocorial
biamnitico de 22 semanas. No hay flujo genital patolgico. No hay
protrusin de membranas al examen especuloscpico. Su seguimiento
ultrasonogrfico previo no ha mostrado evidencia de desarrollo de una
transfusin feto-fetal y ambos fetos crecen concordantemente. Sin
embargo, el examen sonogrfico seriado del cuello uterino ha
mostrado una disminucin de la longitud del canal que se ha
estacionado en los 4 mm desde la semana 20. El examen de la regin
cervical no muestra otros hallazgos relevantes. Una amniocentesis del
primer saco arroj resultados compatibles con ausencia de
infeccin/inflamacin intrauterina a las 20 semanas, fecha desde la
que se encuentra hospitalizada. Se queja de contractilidad uterina
ocasional que ha tendido a aumentar en los ltimos das, sin que ello
haya modificado significativamente el examen sonogrfico cervical.
PREGUNTA
Cul es la conducta teraputica mas adecuada a seguir?
RESPUESTA
a.- Cerclaje cervical
b.- Uso de Nifedipino
c.- Reposo absoluto
d.- Uso de indometacina

Pharmed Solutions Institute

PGINA 257

MANUAL DE TRABAJO DEL CURSO ENARM CMN SIGLO XXI


HIPEREMESIS GRAVIDICA
CIENCIAS BASICAS: Las nuseas y vmitos son una situacin clnica muy frecuente en el embarazo que afecta al 75-80 % de las gestantes (1-2 vmitos al
da con tolerancia correcta a la ingesta, que inician sus sntomas entre la semana 6 y la 9 de gestacin). La hiperemesis gravdica representa el espectro
ms grave de una situacin clnica que aparece de forma gradual. Son pacientes que han ido empeorando clnicamente de forma progresiva hasta
presentar vmitos persistentes con intolerancia parcial o total a la ingesta y con prdida ponderal superior al 5%. SALUD PUBLICA: El espectro ms grave
de esta situacin clnica conocido como hiperemesis gravdica es mucho menos frecuente y afecta en realidad al 0,5-2 % de todas las gestaciones. No
existe consenso acerca de la definicin de la enfermedad, pero la mayora de las definiciones incluyen vmitos persistentes en ausencia de otras
patologas que los expliquen con prdida ponderal importante (superior al 5% del peso inicial). PATOGENIA: La etiologa de la hiperemesis gravdica es
desconocida, aunque diferentes causas han sido propuestas: Niveles elevados de Beta-hCG y estrgenos, la masa placentaria (las mujeres con
embarazos mltiples o enfermedad trofoblstica presentan con mayor frecuencia sintomatologa de nuseas y vmitos), factores psicolgicos y factores
familiares (mujeres que tienen madres o hermanas que hayan presentado nuseas y vmitos tienen ms probabilidad de presentarlas). La hieremesis
parce relacionarse con concentraciones sricas altas o en aumento rpido de gonadotropina corionica, estrgenos o ambos. Se ha demostrado que el
receptor hCG/LH el cual se encuentra presente en las clulas del cuerpo lteo del ovario produciendo el estmulo de la progesterona sobre la decidua
para la comunicacin inicial entre el blastocisto y el endometrio, adems se encuentra presente en distintas reas del cerebro como en el hipocampo y
tallo cerebral lo cual explica la hipermesis gravdica. Tambin se ha postulado que la HCG causa hipermesis gravdica estimulando la va secretora del
tracto gastrointestinal superior. Se ha informado que las mujeres con enfermedad grave tienen aumento 1.5 veces de la probabilidad de tener un feto
del sexo femenino, lo que apoya la hiptesis de los estrgenos (retraso del vaciamiento gstrico y enlentecimiento de la motilidad gastrointestinal).
DIAGNOSTICO: Es muchas veces un diagnstico de exclusin y bsicamente clnico: 1. Paciente que explica varios vmitos al da en relacin o no con las
comidas. Intolerancia total o parcial a la ingesta. 2. Ausencia de otra sintomatologa: No fiebre, no dolor abdominal, no cefalea, no alteraciones
neurolgicas. No todas las pacientes que vomitan al inicio de la gestacin presentan una hiperemesis 3. Alteraciones analticas: Hemograma
(hemoconcentracin), Alteraciones electrolticas (hiponatremia, hipopotasemia, hipocloremia y alcalosis metablica), alteracin del perfil heptico
(elevacin leve de GOT y GPT < 300U/L, y Bilirrubina < 4 mg/dl) y alteracin del perfil tiroideo (aumento de la T4 y la TSH). Ecografa: Las pacientes con
embarazos mltiples o enfermedad trofoblstica presentan vmitos con mayor frecuencia. PREVENCION: Existen diferentes estrategias de prevencin
de las nuseas y los vmitos del embarazo. 1. La toma de complejos multivitamnicos de manera periconcepcional han demostrado disminuir la
incidencia de nuseas y vmitos del embarazo. As que parece razonable recomendar su administracin sobretodo en aquellas pacientes que han
presentado nuseas y vmitos en gestaciones anteriores. 2. Comidas frecuentes, poco abundantes (repartir la ingesta en un mnimo de 5 comidas de
menor cantidad), slidas y evitar las comidas muy grasas y picantes. TRATAMIENTO: Farmacolgico: Puesto que la aparicin de la hiperemesis gravdica
se produce de manera escalonada en la mayora de las pacientes, la manera de tratarlas es ir aadiendo los frmacos que se refieren a continuacin en el
orden en el que se citan, a medida que la sintomatologa vaya empeorando. 1. Doxilamina 10 mg + piridoxina 10 mg: La dosis habitual es un comprimido
cada 6-8 horas, aunque se puede ajustar la dosis en funcin de la sintomatologa hasta alcanzar una dosis mxima de 70 mg/d. Hay que tener en cuenta
que ante la persistencia de sintomatologa en una franja horaria determinada la dosis a aumentar es la inmediatamente anterior (p. Ej. si persisten las
nuseas y vmitos matutinos hay que aumentar la dosis de la noche). Importante tener en cuenta la somnolencia como efecto secundario de la
doxilamina. 2. Aadir Dimenhidrinato 50-100 mg/4-6 horas: La va de administracin puede ser oral o rectal. Dosis mxima 400 mg /d. 3. Aadir
Metoclopramida 5-10 mg/8 horas: La va de administracin es oral en comprimidos o suspensin. Este frmaco puede causar sintomatologa
extrapiramidal (espasmos en cara, cuello y lengua). Cuando persiste la clnica a pesar de la asociacin de 2 ms tratamientos y la paciente presenta
intolerancia total a la ingesta, nos encontraremos ante el espectro ms severo de la enfermedad y ser necesario un ingreso hospitalario para
rehidratacin endovenosa. 4. Dieta famis y fluidoterapia: Suero Glucosado 10 % 500 cc /8 horas alterno con Ringer Lactato o Suero Fisiolgico 500 cc/812 horas hasta corregir el trastorno electroltico. En pacientes que requieren fluidoterapia y que presentan clnica de vmitos durante ms de 3 semanas
se recomienda aadir suplementos de tiamina (vitamina B1) intravenosa a dosis de 100 mg/d durante 2-3 das 1 vial de 100 mg/d. En pacientes que
presenten alteracin en el ionograma se asociar 10-20 mEq de ClK en cada suero glucosado durante 24-48 h o hasta corregir el trastorno
hidroelectroltico 5. Aadir Metoclopramida 5-10 mg/8 horas IV. 6. En casos resistentes a todos los tratamientos mencionados o que requieren varios
ingresos, adems se puede asociar: a. Ondansetrn 8 mg/12 h IV. b. Metilprednisolona 16 mg/8h va oral o endovenosa durante 3 das, seguido de dosis
decrecientes durante 15 das hasta desaparicin de los sntomas (Prednisona vo: 40 mg/da 1 da, 20 mg/da 3 das, 10 mg/da 3 das y 5 mg/da 7 das).
COMPLICACIONES: La encefalopata de Wernicke es poco frecuente pero se asocia a las principales causas de mortalidad en las pacientes, se presenta
por la deficiencia de tiamina posterior a 3 semanas de vmitos persistentes caracterizada por la triada de anormalidades oculares, confusin y ataxia de
la marcha se presenta en el 46.9% de los casos.
b.- Rehidratacin parenteral, piridoxina
CASO CLINICO
c.- Piridoxina mas meclizina, lquidos VO segn tolerancia de la
Paciente de 35 aos, multpara, cursando embarazo de 8.5 semanas.
paciente
Ingresa al Hospital, con historia de 5 semanas de evolucin
d.- Lquidos VO mas metoclopramida
caracterizada por vmitos alimentarios abundantes, aproximadamente
quince episodios diarios, asociado a ictericia, dolor abdominal difuso,
PREGUNTA
coluria y baja de peso cuantificada en seis kilos. Previo al ingreso
En qu semana esperaramos que desaparezcan las nuseas?
consult en dos oportunidades al servicio de urgencia por presentar
RESPUESTA
deshidratacin, secundaria a los vmitos. Al momento de su ingreso,
a.- Entre 12-14 semanas de gestacin
destacaba paciente ictrica y deshidratada, con resto del examen fsico
b.- Entre 16-18 semanas de gestacin
segmentario sin alteraciones. Se solicitan exmenes de laboratorio
c.- Entre 20-22 semanas de gestacin
dentro de los que destacaban VHB y VHC no reactivos, cetonuria
d.- Entre 24-26 semanas de gestacin
positiva, TSH 0.03 y T4 de 17.6, y pruebas hepticas: FA, transaminasas
elevadas. Se realiza ecografa obsttrica que muestra gestacin de 8.6
PREGUNTA
SDG, resto normal.
Qu trastorno acido-base y electroltico sera ms probable encontrar
en esta paciente?
PREGUNTA
RESPUESTA
Cul es la conducta teraputica inicial ms adecuada a seguir?
a.- Acidosis metablica, hipercloremia
RESPUESTA
b.- Acidosis respiratoria, hipernatremia
a.- Rehidratacin parenteral, y reiniciar alimentacin VO segn
c.- Alcalosis metablica, hiponatremia
tolerancia de la paciente
d.- Alcalosis metablico, hipernatremia

CURSO ENARM CMN SIGLO XXI TEL: 36246001

Pharmed Solutions Institute

PGINA 258

MANUAL DE TRABAJO DEL CURSO ENARM CMN SIGLO XXI


ENFERMEDAD TROFOBLASTICA GESTACIONAL (ETG)
CIENCIAS BASICAS: Las anormalidades proliferativas del trofoblasto (citotrofoblasto, sincitiotrofoblasto y trofoblasto intermedio) conocidas como
Enfermedad Trofoblstica gestacional, describe un grupo de patologas relacionadas con la fertiolizacion y se derivan de una proliferacin anormal del
trofoblasto de la placenta humana (hiperplasia) y del genoma paterno, con una contribucin materna ocasional, tendencia variable a la invasin local y a
metstasis, y cuyo denominador comn es la hipersecrecin de beta HCG y responden a la quimioterapia. La ETG comprende a la mola hidatiforme (MH),
mola completa y mola parcial, las cuales se consideran
enfermedades benignas. La neoplasia trofoblastica gestacional
(NTG) incluye a la mola invasora, coriocarcinoma y tumor del
sitio placentario (TSP), los cuales se consideran patologas
malignas. SALUD PUBLICA:
La MH se presenta
aproximadamente en 1 de cada 1000 embarazos en EUA y
Europa; en Japn la incidencia es 3 veces mayor. En Mxico es
de 2.4 por cada 1000 embarazos. La incidencia de mola invasora
ocurre en 1 de cada 40 embarazos molares y en 1 de cada
150,000 embarazos normales. PATOGENIA: Factores de riesgo y
probabilidad de desarrollar ETG: mola previa 50%, aborto previo
25%, embarazo ectpico previo 5%, embarazo previo de
trmino 20%. En mujeres mayores de 40 y 50 aos, la incidencia
de NTG es de 40% y 50% respectivamente. La ETG son
alteraciones de la gestacin, generadas por anormalidades en el
desarrollo del tejido trofoblastico con caractersticas biolgicas
y patolgicas particulares, puesto que son lesiones en las cuales hay representacin del genoma paterno, lo que las diferencia de los tumores no
gestacionales. CORIOCARCINOMA: Tumor maligno del epitelio trofoblastico. Est compuesto por sincitio y citotrofoblasto anaplasico y clulas gigantes
multinucleadas, que invade y puede dar metstasis a sitios distantes. En el coriocarcinoma la invasin vascular ocurre tempranamente resultando en
metstasis hacia el pulmn, vagina, cerebro, rin, hgado y aparato gastrointestinal. MOLA HIDATIFORME (MH): Se caracteriza histolgicamente por la
presencia de vellosidades corinicas con grado variables de proliferacin trofoblstica anormal y edema estromal (degeneracin hidrpica), adquiriendo
la morfologa de bandas y cmulos de vesculas que confieren el tpico aspecto de racimos de uvas. Dentro de los factores de riesgo se pueden
mencionar: Los extremos de la vida reproductiva y el antecedente de Mola previa. MOLA COMPLETA: Ausencia de tejido embrionario o fetal.
Degeneracin hidrpica de vellosidades coriales y son avasculares, hiperplasia trofoblastica difusa con atipia difusa y marcada del trofoblasto en el sitio
de la implantacin molar. El factor de riesgo ms asociado, es la edad, ya que mujeres mayores de 40 aos tienen un riesgo 5-10 veces mayor, que
mujeres jvenes. El sntoma y signo clnico ms importante en el embarazo molar es el sangrado o manchado vaginal, que se presenta entre un 89 a 97%
de las pacientes, esta patologa se debe sospechar cuando encontramos una Beta-hCG mayor de 100.000 UI en embarazos tempranos. Otro signo
importante es una altura uterina mayor a la esperada para la edad gestacional, lo cual se produce por una amplia proliferacin trofoblastica secundaria a
un elevado nivel de Beta-hCG; esto ocurre en cerca del 50% de las pacientes. La ausencia de actividad cardiaca fetal ocurre a pesar del tamao uterino
francamente suprapbico. Los quistes tecaluteinicos, estn presentes hasta en un 46% de los embarazos molares completos, y generalmente son
mayores de 5 cm (6-12 cm), pero pueden verse hasta de 20 cm de dimetro, suelen ser bilaterales, su contenido es serosanguinolento, y su aparicin se
explica por hiperestimulacin ovrica de la -hCG circulante. La hipermesis gravdica tambin es ms frecuente en estos embarazos, se ha identificado
hipertiroidismo (7%). MOLA INVASORA: (coriocracinoma destruens) La definen el crecimiento trofoblstico excesivo y su capacidad invasora local, con
una penetracin extensa por los elementos trofoblsticos, que incluyen vellosidades completas, en el miometrio (sin involucrar el estroma endometrial)
e incluso hasta el peritoneo parametrios y cpula vaginal adyacente. Las metstasis a distancia son excepcionales. MOLA PARCIAL: Presencia de tejido
embrionario o fetal. Hiperplasia trofoblastica focal, variabilidad marcada en el tamao de las vellosidades con importante edema y cavitacin,
presentando inclusiones prominentes en el estroma trofoblastico de las vellosidades, presentndose ocasionalmente atipia focal y leve de trofoblasto en
el sitio de implantacin. La presentacin clnica del embarazo molar parcial es similar a la de un aborto incompleto dado por sangrado genital y dolor
tipo clico en hipogastrio de hecho, el diagnstico, en su gran mayora, se realiza en el estudio histopatolgico. TUMOR DEL SITIO PLACENTARIO: Es un
tumor extremadamente raro. Se origina en el sitio de implantacin de la placenta y deriva de clulas de trofoblasto intermedio de la placenta, las cuales
se identifican por la secrecin de lactogeno placentario y pequeas cantidades de fraccin beta de HCG. No contiene vellosidades coriales.
DIAGNOSTICO: Los sntomas son ms evidentes en la mola completa, y se presentan entre el primero y el segundo trimestre. Los sntomas clsicos de la
mola completa son: la hemorragia vaginal (97%), el aumento del tamao uterino de manera ms rpida del habitual (50%); la hipertensin arterial en
embarazos menores de 24 semanas (27% de las pacientes con mola completa), embolismo pulmonar (2% en pacientes con mola completa), tirotoxicosis
(2-7%). La mola hidatiforme parcial no produce el cuadro clnico espectacular de la mola completa, generalmente se presentan los sntomas de aborto
(amenaza de aborto o aborto incompleto), y se puede establecer el diagnostico despus del estudio histopatolgico del material obtenido. Las
caractersticas histopatolgicas permiten hacer el diagnstico diferencial de la diversidad de las presentaciones de ETG. Cuando exista sangrado uterino
anormal por ms de 6 semanas posteriores a cualquier embrazo, descartar ETG. Luego de la sospecha clnica el examen de primera lnea es la ecografa.
La mola completa tiene un aspecto ecogrfico vesicular caracterstico en lluvia de nieve. En el embarazo molar parcial se puede ver como una imagen
indistinguible de un aborto incompleto, aunque se pueden observar espacios qusticos focales en el tejido placentario y aumento del dimetro
transversal del saco gestacional; si se encuentran los 2 hallazgos el valor predictivo positivo para la mola parcial es de 90%. La ecografa asociada a la
dosificacin de BhCG, cuando esta es mayor de 82350 mUI/ml y no existe actividad cardiaca visible en el embrin da una probabilidad de estar frente a
una mola hidatiforme del 89%. La medicin de los niveles sricos de BhCG con valores mayores de lo esperado para la edad gestacional son
fundamentales para el diagnstico. La hiperemesis gravdica y la aparicin de preeclampsia temprana son otros de los cuadros clnicos que pueden
presentarse. En la evaluacin inicial se debe hacer bsqueda rpida de enfermedad metastsica, solicitando RX de trax y pruebas de funcin heptica y
renal. TRATAMIENTO: El tratamiento de la MH tiene dos aspectos: evacuacin y seguimiento. La evacuacin inmediata de la mola preferiblemente se
debe realizar mediante aspiracin, previa maduracin cervical con misoprostol en caso de encontrarse un crvix cerrado. La evacuacin debe realizarse
en instituciones de segundo o tercer nivel de atencin, lo de eleccin es el AMEO, o histerectoma en bloque, con reserva de sangre, uso de oxitcicos en
caso de ser necesario, en casos especiales evaluacin ecogrfica transoperatoria y aplicacin de inmunoglobulina humana antiD si es Rh negativo. La
quimioterapia profilctica en MH ha sido controversial, est indicada en aquellas mujeres que tienen molas de alto riesgo. La histerectoma puede ser
una opcin preferible al curetaje por aspiracin en caso de mujeres con paridad satisfecha y en especial en mujeres mayores de 40 aos o ms, dada la
mayor frecuencia de enfermedad trofoblstica maligna en este grupo etareo. Aunque la histerectoma no elimina del todo la posibilidad de metstasis,
reduce apreciablemente la enfermedad recurrente. Los quistes tecalutenicos grandes y sintomticos se pueden aspirar con gua ecogrfica. Una

CURSO ENARM CMN SIGLO XXI TEL: 36246001

Pharmed Solutions Institute

PGINA 259

MANUAL DE TRABAJO DEL CURSO ENARM CMN SIGLO XXI


alternativa del AMEO es el LUI, considerando el mayor riesgo de perforacin uterina. En MH con altura uterina similar a 16SDG o mayor existe riesgo de
embolizacion pulmonar. Para descartar el desarrollo de NTG, las pacientes se siguen luego de la evacuacin molar con niveles de -hCG semanales, el
seguimiento debe realizarse cada 2 semanas con examen clnico completo y mediciones seriadas de BhCG hasta tener 2 mediciones seguidas negativas;
luego cada 3 meses durante un ao. En la enfermedad trofoblstica benigna la regresin de la BhCG ocurre entre los das 11 y 229, con un promedio de
74 das. Encontrar niveles sricos de 25.000 mUI/ml a la cuarta semana es signo de alta acividad trofoblstica.
CASO CLINICO ENFERMEDAD TROFOBLASTICA
Acude a la consulta femenino de 36 aos de edad a solicitar mtodo de
planificacin familiar ya que tiene 2 aos de su ultimo parto y no se ha
cuidado debido a que su esposo se encuentra trabajando en estados
unidos y va a regresar pronto, la paciente es originaria de Michoacn y
tiene estudios de primaria, dentro de sus antecedentes cuenta con
historia de tabaquismo positivo, ingesta de alcohol ocasional, presento
menarca a los 10 aos, gesta 4, para 3, abortos 1, refiere que el ultimo
embarazo fue hace 4 aos, le realizaron un legrado porque no se haba
formado el producto y presento sangrados al inicio con salida de tejido
como burbujas, le indicaron volver a consulta y ya no regreso, ahora ya
no quiere otro embarazo. A la exploracin fsica la paciente solo se
observa con obesidad y levemente desalineada.
PREGUNTA
Considerando que la paciente presento con mucha probabilidad una
enfermedad trafoblastica, Que mtodo de planificacin es el ms
adecuado?
RESPUESTA
a.- Inyectable combinado mensual.
b.- Oral con microdosis.
c.- Mtodo de barrera.
d.- Obstruccin tubarica bilateral.
CASO CLINICO
Paciente primigesta de 28 aos de edad, sin antecedentes de inters,
remitida a Urgencias a las 17.4 semanas de embarazo por elevacin de
la tensin arterial, que en el momento del ingreso alcanzaba valores de
hasta 185/130 mmHg. En la exploracin general se objetiv disnea,
edemas maleolares bilaterales y tero aumentado de tamao como
gestacin de 7 meses. La analtica mostraba alteraciones compatibles
con un cuadro de preeclampsia de comienzo precoz: proteinuria,
plaquetopenia, hiperamoniemia y aumento de enzimas hepticas. En
la ecografa transabdominal se confirm la existencia de una gestacin
nica evolutiva; feto con biometra fetal concordante con gestacin de
14 semanas, sin malformaciones evidentes y con pliegue nucal
aumentado. La placenta presentaba mltiples quistes anecognicos y
un grosor mximo de 8cm. A nivel de fondo de saco de Douglas se
visualizaron ovarios aumentados de tamao, de 104x69mm, a
expensas de mltiples luteomas. Se realiz determinacin srica de HCG, cuyo valor alcanz 714.061 U/l. Ante estos hallazgos se procedi
al ingreso de la paciente en la unidad de patologa del embarazo,
instaurndose tratamiento con oxgeno en puntas nasales, alfa-metildopa (500mg/8h) e hidralacina (25mg/8h).
PREGUNTA
Cul es la conducta ms adecuada a seguir en este caso?
RESPUESTA
a.- Evacuacin por va abdominal y seguimiento
b.- Quimioterapia
c.- Histerectomia en bloque
d.- Continuar embarazo con vigilancia estrecha
PREGUNTA
Cul es el diagnostico mas probable en este caso?
RESPUESTA
a.- Mola completa
b.- Mola parcial
c.- Mola invasora

CURSO ENARM CMN SIGLO XXI TEL: 36246001

d.- Coriocarcinoma
PREGUNTA
Cul sera la complicacin asociada menos probable en esta paciente?
RESPUESTA
a.- Preeclampsia
b.- Cardiopata isqumica
c.- Hipertiroidismo
d.- Anemia
CASO CLINICO
Paciente femenina de 22 aos de edad, primigesta, con antecedente
de amenorrea de siete semanas, nusea, vmito, hemorragia
transvaginal durante cinco das y dificultad respiratoria (disnea). Por lo
que acudi a clnica privada, donde se realiz ultrasonido obsttrico el
cual mostr embarazo molar sin evidencia de saco vitelino, ni
estructuras embrionarias. Se realiz legrado uterino instrumentado sin
complicaciones aparentes. Una semana despus persisti con disnea,
se agreg tos productiva con esputo hemoptoico, por lo que se solicit
radiografa de trax, que mostr infiltrado bilateral y radio opacidad
parahiliar derecha, por lo cual se decidi trasladar al Servicio de
Urgencias. Se recibe en el Servicio de Ginecologa, con integridad
neurolgica, palidez de tegumentos y aumento del trabajo respiratorio
evidente por taquipnea, aleteo nasal y uso de msculos accesorios de
la respiracin, sin estertores. Tacto vaginal con orificio cervical cerrado
formado, doloroso a la movilizacin y palpacin de fondos de saco, con
moderados loquios serohematicos no ftidos, no se delimita fondo
uterino. Laboratorio: gasometra arterial con hipoxemia, elevacin de
azoados, hiperuricemia y anemia con hemoglobina de 7.9 a 3.9 g y
plaquetopenia de 195 000 a 126 000/mm3, beta-HCG en suero de 73
676 mUI/mL$.
PREGUNTA
Cul es el diagnostico mas probable para este caso?
RESPUESTA
a.- Sepsis
b.- Coriocarcinoma
c.- Cancer cervicouterino
d.- Neumonia
PREGUNTA
Qu marcador no seria de ms utilidad en este caso?
RESPUESTA
a.- AFP
b.- hCG
c.- CA 125
d.- CA 19-9
PREGUNTA
Cul seria la conducnta teraputica mas adecuada a seguir en este
caso?
RESPUESTA
a.- Quirurgico
b.- Radioterapia
c.- Quimioterapia
d.- Conservador

Pharmed Solutions Institute

PGINA 260

MANUAL DE TRABAJO DEL CURSO ENARM CMN SIGLO XXI


ESTADOS HIPERTENSIVOS EN EL EMBARAZO
CIENCIAS BASICAS: Definicin: Se habla de hipertensin en el embarazo cuando la tensin arterial diastlica es > 90mmHg y la sistlica es >140mmHg, o
un incremento en la TA sistlica de por lo menos 30mmHg del valor basal o de diastlica de por lo menos 15mmHg sobre el valor basal. Proteinuria:
Excrecin urinaria de protenas mayor de 30mg/dl o ms en tiras reactivas (se requieren 2 determinaciones o ms en un lapso de 6 horas en ausencia de
infeccin). 300mgs o ms en una coleccin de orina en 24h. SALUD PUBLICA: Los trastornos hipertensivos durante la gestacin, son la primera
complicacin mdica en muchos pases del mundo, constituyendo una de las principales causas de morbilidad y mortalidad materna, fetal y neonatal. 710% de los embarazos se complican con HTA. La preeclampsia/eclampsia origina el 70% de los estados hipertensivos, y el 30% lo representan pacientes
con hipertensin crnica preexistente durante el embarazo. En Mxico su prevalencia vara entre 7-10% de la poblacin gestante, existe mayor
incidencia antes de los 20 aos y despus de los 35; aproximadamente 75% de los casos corresponden a primigestas. CLASIFICACION, de los trastornos
hipertensivos durante el embarazo: A) Enfermedad hipertensiva inducida por el embarazo: Preeclampsia leve. Preeclampsia severa. Inminencia de
eclampsia. Eclampsia. Sndrome de HELLP. Preeclampsia recurrente. Enfermedad hipertensiva no clasificable (imposibilidad de clasificar por carecer de
elementos necesarios o por haberse instituido tratamiento previo a su estadificacion). Hipertensin transitoria (HTA despus de la semana 20 o en las
primeras horas posparto sin otros signos de preeclampsia). B) Enfermedad vascular crnica hipertensiva: Hipertensin sistmica esencial (HTA
independiente de la gestacin o anterior a las 20 semanas y que persiste ms de 6 semanas posparto y que no sea a consecuencia de lesin de alteracin
anatmica o funcional renal). Hipertensin crnica con enfermedad hipertensiva agregada (HTA previa al embarazo agregndose preeclampsia, puede
haber elevacin de cido rico igual o mayor de 6mg/dl). De acuerdo a la Clasificacin del Colegio Americano de Obstetricia y Ginecologa, se identifican
4 categoras: HIPERTENSION CRONICA: Coexistencia de hipertensin de novo antes de las 20 SDG, al igual que si tiene el antecedente de hipertensin
preexistente antes del embarazo. Puede ser: primaria (esencial) o secundaria (renal, suprarrenal, etc). Las mujeres con hipertensin crnica tienen un
riesgo de 10-20% de desarrollar preeclampsia. Es conveniente, adems, en las pacientes con hipertensin crnica, establecer si ya existe dao en algn
rgano blanco antes del embarazo, incluyendo hipertrofia ventricular izquierda, retinopata y/o enfermedad renal. En canto a los medicamentos,
suspender el tratamiento con medicamentos del grupo de los IECA, as como ARA II. Existen comunicaciones sobre restriccin del crecimiento fetal
intrauterino, oligohidramnios, insuficiencia renal y muerte neonatal. Hay que reemplazarlos por alfametil-dopa, labetalol o los antagonistas de calcio.
HIPERTENSION ARTERIAL CRONICA CON PREECLAMPSIA SOBREAGREGADA: Pacientes con hipertensin crnica, que presentan descompensacin de las
cifras tensionales y aparicin o incremento de proteinuria despus de las 20 SDG. El pronstico es peor que con cada entidad por separado. Criterios
diagnsticos: Aparicin de proteinuria por primera vez, despus de las 20 semanas de gestacin. Incremento sbito de la proteinuria en pacientes que la
presentaban antes del embarazo. Incremento sbito de la presin arterial en la gestante que la tena bien controlada. Trombocitopenia (menos de 100
000 plaquetas/mL). Incremento de enzimas hepticas por encima de valores normales. HIPERTENSION GESTACIONAL: Presin arterial sistlica mayor o
igual a 140mmHg y/o presin arterial diastlica mayor o igual a 90mmHg en una mujer previamente normotensa, despus de la semana 20 de gestacin.
En dos tomas separadas por un intervalo de 4 a 6h entre una y otra con ausencia de proteinuria y se recupera despus del parto. Las mujeres con
hipertensin gestacional con inicio antes de la semana 34 de gestacin, tienen probabilidad de 35% de desarrollar preeclampsia.
CASO CLINICO HIPERTENSION ARTERIAL Y EMBARAZO
Paciente femenino de 37 aos de edad actualmente cursando su tercer
embarazo con 14 semanas por FUM, acude a visita de control prenatal
cuenta con 17 SDG por USG, antecedentes personales patolgicos de
pre-eclampsia, adems obesidad previa al embarazo, cuenta con
antecedentes familiares de diabetes y obesidad por parte de la madre
e hipertensin por parte del padre, el cual fallece por infarto al
miocardio. A la exploracin fsica se observa paciente gestante con
obesidad grado I, sin tratamiento previo, se enva EGO, QS y BH, donde
se observa triglicridos de 350, colesterol de 290, glucosa de 105
mg/dl, hemoglobina glucosilada de 7, tensin arterial en dos ocaciones
de 135/95 mmHg. Tratada con dieta hiposodica, hipocalorica y
restriccin de liquidos.
PREGUNTA
Cul es el factor ms importante que presenta la paciente, para
considerar Hipertension Arterial Pre-existente?
RESPUESTA
a.- Trigliceridemia.
b.- Hipercolesterolemia.
c.- Obesidad.
d.- Cifras tencionales altas.
CASO CLINICO ENFERMEDAD HIPERTENSIVA DEL EMBARAZO
Se trata de paciente femenino de 26 aos de edad la cual acude a
consulta de control prenatal actualmente contando con 21 semanas de
gestacin, refiere que ha presentado cefalea ocasional, mareo y
cansancio a la exploracin fsica se observa edema de miembros
inferiores, la consulta anterior usted envi estudios de EGO, QS y BH,
los datos de relevancia fue Hematocrito de 38, las constantes vitales
son de FR 18, FC 89, TA 160/90. MmHg. Con edema leve sin hematuria
ni proteinuria.
PREGUNTA.
Cul es la conducta a seguir mas adecuada?
RESPUESTA

CURSO ENARM CMN SIGLO XXI TEL: 36246001

a.- Restricin de liquidos y sal.


b.- Vigilancia estrecha de tencion arterial.
c.- Monitorizacin de vitalidad fetal.
d.- Iniciar con alfametildopa
CASO CLINICO
Una mujer de 21 aos embarazada consulta por hipertensin y
proteinuria a las 20 semanas de gestacin. Existen antecedentes de
preeclampsia durante su primer embarazo hace un ao atrs donde
tuvo un parto por cesrea de urgencia. Despus del parto su presin
arterial se normaliz y no recibi medicacin adicional. Su madre fue
hipertensa a partir de los 40 aos de edad, el resto de su familia es
sana. No tiene adicciones. EF: TA 145/100mmHg y como nico sntoma
presenta cefalea leve, FC 100 lpm, IMC 16,9 kg/m2. El resto del
examen fsico es normal. Hemograma mostr una anemia microctica
ferropriva moderada. Se detect proteinuria leve con un ndice
protena/creatinina de 40,6 mg/mmol (normal <30 mg/mmol durante
el embarazo). La excrecin urinaria de noradrenalina oscil entre
5.659 nmol/da y 9.601 nmol/da en varios registros.
PREGUNTA
Cul es el diagnostico mas probable?
RESPUESTA
a.- Hipertension esencial
b.- Hiperaldosteronismo primario
c.- Feocromocitmoma
d.- Glomerulopatia
PREGUNTA
Cul es la conducnta mas adecuada a seguir en este caso?
RESPUESTA
a.- Tomografia
b.- Radiografia
c.- Resonancia
d.- Ultrasonido

Pharmed Solutions Institute

PGINA 261

MANUAL DE TRABAJO DEL CURSO ENARM CMN SIGLO XXI


PREECLAMPSIA Y ECLAMPSIA
CIENCIAS BASICAS: Se define como la aparicin de hipertensin y proteinuria despus de la semana 20 del embarazo. Se suele acompaar de edemas
pero no es necesaria la presencia de stos para ser diagnosticada. Es una enfermedad caracterstica y propia del embarazo de la que se pueden tratar los
sntomas, pero slo se cura con la finalizacin del mismo y si no se trata adecuadamente puede ser causa de graves complicaciones tanto para la mujer
embarazada como para el feto. CLASIFICACION: Preeclampsia leve; presin arterial de 140/90mmHg o mas o elevacin de 30mmHg en la sistlica y
15mmHg en la diastlica, cuando se conocen las cifras basales previas, se presenta despus de la semana 20 hasta 30 das posparto, existe proteinuria de
ms de 300mg/24h, ausencia de sntomas de vasoespasmo. Preeclampsia severa; presin arterial de 160/110mmHg o ms, despus de la semana 20
hasta 30 das posparto, existe proteinuria mayor de 5g/24h, presencia de cefalea, acufenos, fosfenos, edema generalizado. Inminencia de eclampsia; se
establece el diagnstico cuando despus de la semana 20 de gestacin, parto o puerperio (no ms de 30 das), aparece uno o ms de los siguientes
datos: Presin arterial sistlica mayor a 185mmHg con presin arterial diastlica >115mmHg, proteinuria mayor a 10g, estupor, prdida parcial o total de
la visin, dolor epigstrico en barra, hiperreflexia generalizada. Preeclampsia recurrente; presencia de cualquiera de los tipos de enfermedad
hipertensiva inducida por el embarazo, que aparece por segunda ocasin o ms en embarazos consecutivos o no. PATOGENIA: Factores de riesgo:
Ausencia o deficiencia de control prenatal, desnutricin, obesidad, intervalo intergensico menor a dos aos, muer menor de 18 y mayor de 35 aos,
primigesta o nuligesta, preeclampsia/eclampsia en embarazos anteriores o antecdenetes familiares repetidos, hipertensin arterial crnica o cualquier
otro trastorno hipertensivo durante lagestacion, sobredistension uterina de cualquier origen (embarazo gemelar o multiple, polihidramnios), infeccion en
vas urinarias recurrentes, DM tipo 1 y 2, enfermedad renal previa o recurrente, enfermedad trofoblastica, enfermedades autoinmunes, factores
psicosociales. Es un estado de vasoconstriccin generalizado secundario a una disfuncin en el epitelio vascular, en lugar de la vasodilatacin propia del
embarazo normal. Ello se asocia a isquemia placentaria desde mucho antes de la aparicin del cuadro clnico, en lo que parece ser uno de los orgenes de
los factores txicos para el endotelio vascular. Dicha isquemia parece ser debida a una deficiente placentacin en la que no se producira la habitual
substitucin de la capa muscular de las arterias espirales uterinas por clulas trofoblsticas, que es lo que produce una vasodilatacin estable que
permite aumentar varias veces el caudal de sangre asegurando as el correcto aporte sanguneo a la unidad fetoplacentaria, su etiologa ltima que sigue
siendo desconocida, aunque vamos identificando factores. As podemos citar factores hereditarios por va materna pero tambin paterna, factores
inmunolgicos que explican que se produzca ms en la primigesta y ms si no ha estado expuesta antes al contacto directo con los antgenos paternos, a
factores nutricionales como la falta de ingesta de calcio en algunas poblaciones. Todo ello se manifiesta por un desequilibrio entre prostaglandinas
vasodilatadoras (prostaciclina) frente a las vasoconstricciones (tromboxano), quiz mediado por la aparicin de exceso de produccin de citoquinas
proinflamatorias (IL-2, TNF), y de produccin de lipoperoxidos capaces ambos de e inducir alteraciones endoteliales, esta disfuncin endotelial general,
que afecta a muchos rganos distintos como son el rin, el cerebro, el hgado pero en especial al sistema cardiovascular por lo que la hipertensin es su
expresin ms genuina, que con frecuencia se acompaa de edema. El vasoespasmo y el edema facilitan la reduccin del volumen plasmtico que lleva a
la hemoconcentracin caracterstica y a la hipercoagulabilidad. Todo ello hace que exista una hipoperfusin multiorgnica que empeora el cuadro a nivel
general y fetoplacentario. DIAGNOSTICO: Se establece el diagnostico de preeclampsia leve cuando se presentan 2 o mas de los siguientes signos: 1)
Presin sistlica >140mmHg o elevacin >30mmHg sobre la presin habitual. 2) Presin diastlica, >90mmHg o elevacin >15mmHg sobre la presin
habitual. Proteinuria. Edema. La medicin de la presin deber realizarse con la paciente sentada y requiere de dos tomas consecutivas con un intervalo
de 6h o mas en este lapso de horas debe permanecer en reposo. El incremento de proteinuria y de hipertensin en una paciente nefrpata o hipertensa
previa se denomina preeclampsia sobreaadida y el manejo clnico es parecido. Se establece el diagnostico de preeclampsia severa cuando se presentan
2 o ms de los siguientes signos: 1) presin sistlica >160mmHg y presin diastlica >110mmHg. 2) proteinuria >5g en orina de 24h o su equivalente en
tiras reactivas (ms de 3+). 3) Oliguria de menos de 500ml/24h. 4) Trastornos cerebrales o visuales. 5) Edema generalizado. Por otra parte tambin es
grave cuando aparece Sndrome de HELLP (plaquetopenia, elevacin de enzimas hepticas y hemolisis), de insuficiencia cardiaca (edema agudo de
pulmn), o de insuficiencia renal (creatinina>1,2 mg/dL), o dolor epigstrico. La inminencia de eclampsia se diagnostica con uno o ms de los siguientes
datos: 1) Presin arterial sistlica >185mmHg con presin arterial diastlica >115mmHg. 2) Proteinuria >10g. 3) Estupor. 4) prdida parcial o total de la
visin. 5) Dolor epigstrico en barra. 6) Hiperrreflexia generalizada. Por la gravedad de esta variedad clnica las pacientes deben ser manejadas como
eclmpticas. Todas suelen acompaarse de signos de afectacin fetal por insuficiencia placentaria crnica en forma de signos de restriccin del
crecimiento intrauterino o agudo con signos de Riesgo de Prdida de Bienestar Fetal. Con el agravante de que la situacin fetal suele empeorar al tratar
la hipertensin materna grave ya que al descender sus valores se disminuye la perfusin placentaria, y de que los frmacos administrados a la madre
dificultan la valoracin del estado fetal a travs del estudio de la frecuencia cardiaca fetal basal o test no estresante (NST en ingls), por lo que las
unidades que traten los casos graves tendran que disponer de la posibilidad de ecografa Doppler de forma continuada. TRATAMIENTO: Debe
recordarse que existe una hemoconcentracin a pesar de que puedan existir edemas, por lo que NO deben administrarse diurticos para tratarlos o para
disminuir la TA, excepto en los casos de edema agudo de pulmn o de oliguria. Por la misma razn tampoco se debe hacer una dieta hiposodica estricta.
Los mejores resultados actuales se deben a la posibilidad de terminar la gestacin en los casos graves a partir de la semana 32 o antes en caso necesario,
con buenos resultados perinatales gracias a la posibilidad de madurar los fetos con corticoides (no contraindicados con buen control en la preeclampsia),
y especialmente a las mejoras en el manejo neonatal de los prematuros extremos. Preeclamsia leve. Terminar la gestacin al llegar a trmino sin
sobrepasar la semana 40. No se ha demostrado que se mejore con reposo, ni con tratamiento hipotensor. Se debe controlar la aparicin de signos de
gravedad para poder iniciar el tratamiento en el momento oportuno. Antihipertensivos: alfametil-dopa 250-500mgs VO cada 8 h, si es preciso asociados
a hidralazina 10-50mgs VO c/8hrs. Preeclampsia grave. En todos los casos se debe: 1) Controlar la TA con hipotensores mantenindola de ser posible por
debajo de los valores indicados como de gravedad, pero por encima de 140/90 para no disminuir la perfusin placentaria en exceso. Se usara,
hidaralazina bolo inicial de 5 mg IV si diastlica >110mmHg, repetir en 20 minutos (5-10mg) de acuerdo a respuesta hasta un mximo de 30 mg (de no
contar con hidralazina se recomienda nifedipino VO 10mg, repetir cada 10-30min). Labetalol, 20 mg por va intravenosa, seguidos de 40 a 80 mg cada 10
minutos, hasta una dosis mxima de 220 mg. Alfametil-dopa 500mgs VO cada 6 h, si es preciso asociados a hidralazina 50mgs VO c/6hrs. No usar
betabloqueantes (atenolol) ni IECAs, ni ARA-II. Indicado para evitar edema cerebral dexametasona (16mg IV dosis nica). 2) Hacer prevencin del riesgo
de Eclampsia y la posible la hiperreflexia, con sulfato de magnesio (SO4Mg, la dosis de ataque es de 6g. en 100 de dextrosa al 5% pasar en 15 min.
Continuar con dosis de mantenimiento a 1-2 g por hora administrados en infusin IV continua (preparar sol. Glucosada de 900cc al 5% mas 10 ampolletas
de 1g. Pasar 100-200ml de solucin por hora igual a 24-48 gotas por minuto). En caso de intoxicacin utilizar 1 g de Gluconato de calcio IV diluido en
100cc de solucin fisiolgica y pasar en 15 min. 3) Vigilar la aparicin de otros signos de gravedad Sndrome de HELLP, CID clnica, insuficiencia cardiaca,
renal, etc. 4) controlar el bienestar fetal con NST peridico, perfil biofsico y Doppler para comprobar el grado de afectacin fetal y si existen signos de
redistribucin vascular, para poder indicar la extraccin fetal antes de la afectacin de los vasos venosos, momento que por lo que sabemos hasta ahora,
coincide con el inicio de la acidosis fetal. 5) Finalizar la gestacin: 1) A termino: en cuanto la situacin materna se estabiliza. 2) Pretrmino: >32 semanas,
en cuanto se compruebe la madurez pulmonar fetal o antes si hay indicacin materna o fetal. 3) Pretrmino <32 semanas: siempre que exista indicacin
materna por aparicin de complicaciones graves o fetales con signos de redistribucin ante la hipoxia. 28-32 semanas madurara con corticoides y
valoracin del riesgo de prematuridad correspondiente a cada semana y peso fetal frente a los signos de redistribucin ante la hipoxia con afectacin de

CURSO ENARM CMN SIGLO XXI TEL: 36246001

Pharmed Solutions Institute

PGINA 262

MANUAL DE TRABAJO DEL CURSO ENARM CMN SIGLO XXI


vasos venosos o indicacin materna. A partir de la semana 30 probablemente nos inclinaremos por la extraccin fetal, con feto madurado. ECLAMPSIA.
CIENCIAS BASICAS: Se denomina as a las convulsiones similares a una crisis epilptica aparecida en el contexto de una preeclamsia e hiperreflexia,
despus de la semana 20 de gestacin, parto o puerperio en ausencia de otras causas de convulsiones. Suponen un riesgo vital y pueden ocurrir, antes,
durante o despus del parto. TRATAMIENTO: 1) Soporte vital (va area, constantes, va intravenosa...). 2) Tratamiento de la hipertensin: hidaralazina
bolo inicial de 5 mg IV si diastlica >110mmHg, repetir en 20 minutos (5-10mg) de acuerdo a respuesta hasta un mximo de 30 mg (de no contar con
hidralazina se recomienda nifedipino VO 10mg, repetir cada 10-30min). Labetalol, 20 mg por va intravenosa, seguidos de 40 a 80 mg cada 10 minutos,
hasta una dosis mxima de 220 mg. 3) Tratamiento anticonvulsivante con SO4Mg, la dosis de ataque es de 6g. en 100 de dextrosa al 5% pasar en 15 min.
Continuar con dosis de mantenimiento a 1-2 g por hora administrados en infusin IV continua (preparar sol. Glucosada de 900cc al 5% mas 10 ampolletas
de 1g. Pasar 100-200ml de solucin por hora igual a 24-48 gotas por minuto), ajustando los niveles a 4.8-9.6 mg/dl de magnesemia, siendo necesario
mantener los controles clnicos continuados (reflejo patelar, respiraciones /minuto, diuresis) para evitar que una sobredosificacin pueda producir un
paro cardiorrespiratorio. Esta medicacin se ha comprobado en estudios prospectivos y randomizados ms eficaz que cualquier otra, pero los lmites
teraputicos y txicos estn muy cercanos. Tambin est indicado el uso de diazepam, como droga de segunda eleccin, que actuara disminuyendo el
consumo de oxigeno por el tejido cerebral, pero que posee efectos nocivos sobre el feto (depresin respiratoria, hipotona). Las dosis recomendada son:
bolo de 10 mg por va IV y luego continuar con 50 mg en 500 cc de solucin dextrosada al 5%. 4) Acabar la gestacin en cuanto la situacin clnica
materna este estabilizada y lo permita. COMPLICACIONES: De preeclampsia/eclampsia; Sindrome de HELLP. Desprendimiento prematuro de placenta.
Insuficiencia renal aguda. Edema pulmonar agudo. Edema cerebral. Coagulacion intravascular diseminada. Ruptura heptica (Raro, alta letalidad, se
manifiesta por dolor epigastrico en barra o en el cuadrante superior derecho, irradiado a hombro, regin iterescapulovertebral derecha, en
hemicinturon, pudiendogeneralizarse a todo el abdomen). Hemorragia cerebral.
CASO CLINICO PRE-ECLAMPSIA
Se trata de femenino de 27 aos de edad la cual cuenta con 22
semanas de embarazo, acude a su tercera visita de control prenatal, al
revisar su expediente usted observa que ha presentado por tercera
ocasin de forma independiente una tensin arterial de 140/90, usted
realiza una prueba de rutina para apoyar su sospecha diagnostica de
pre-eclampsia, se observa edema de miembros inferiores, proteinuria,
cefalea constante.
PREGUNTA
Cual de las medidas no es adecuada?
RESPUESTA
a.- Manejo ambulatorio.
b.- Hidralacina 50 mg cada 12 hrs.
c.- Dieta hiposodia y restriccin de liquidos.
d.- Captopril 25 mg cada 24 hrs.
CASO CLINICO
Paciente de 17 aos primigesta soltera con embarazo de 36 semanas
por FUM confiable y 38 semanas por ecografa de segundo trimestre.
Antecedentes patolgicos: gastritis hace ms de 5 aos en
tratamiento, quirrgicos: niega, hospitalizaciones: niega, traumticos:
niega, ginecobstetricos: m: 13 aos, ciclos: 4/28, IVSA: 16, FUM: 17-042013. G1, P 0, A 0, C 0, controles prenatales 5 consultas, en las cuales
su presin 110/80mmHg. Refiere cuadro clnico de 1 mes de evolucin
caracterizado por edema de miembros inferiores, niega cefalea, niega
acufenos, niega fosfenos, refiere dolor en regin lumbar de moderada
intensidad no irradiado y disuria, refiere movimientos fetales
presentes, niega sangrado uterino transvaginal. EF: Orientada
consciente, palidez de tegumentos, con edema en miembros inferiores
++, y manos y cara +, con T/A: 140/90mmHg (1ra toma) y
145/90mmHg (2da), FC 80 lpm, Examen general de orina: protenas
33mg/dl, bacterias +, leucocitos +. BH: Hb; 11.8mg/dl. Glucosa
105mg/dl, Cr. 1.0, Tele de trax, sin alteraciones
PREGUNTA
Cul es el diagnstico ms probable para esta paciente?
RESPUESTA
a.- Hipertensin arterial gestacional
b.- Preeclampsia
c.- Eclampsia
d.- Hipertensin arterial crnica
PREGUNTA
De los siguientes parmetros. Cul es el ms probable que presente la
paciente?
RESPUESTA
a.- PAM >126mmHg
b.- Elevacin de enzimas hepticas

CURSO ENARM CMN SIGLO XXI TEL: 36246001

c.- Proteinuria de 300mg/l pero menos de 2gr


d.- Trombocitopenia
PREGUNTA
Cul es la conducta teraputica ms adecuada a seguir?
RESPUESTA
a.- Iniciar sol. Fisiolgica, alfa-metildopa y resolucin de embarazo
b.- Iniciar sol fisiolgico, hidralazina y sulfato de Magnesio
c.- Iniciar sol. Fisiolgica, difenilhidantona y resolucin de embarazo
d.- Iniciar sol. Fisiolgica, dexametasona y resolucin de embarazo
CASO CLINICO ECLAMPSIA
Se trata de paciente de 19 aos de edad la cual acude por primera vez
a consulta por amenorrea secundaria de 30 semanas por FUM, acude
hasta ahora ya que desconocan su embarazo en casa, la paciente
refiere mareo, dolor abdominal en barra, opresin de torax, cefalea e
irritabilidad, a la exploracin fsica se observa escleras hiperemicas,
leve rubicundez facial, rots incrementados y edema de miembros
inferiores, proteinuria, se mide fondo uterino con 28 cm, con
disminucin de actividad fetal, los signos vitales de la madre fueron FR
21, FC 96, TA 130/100 en 3 ocaciones.
PREGUNTA
Cul es la conducta inmediata a seguir con la paciente?
RESPUESTA
a.- Identificar la viabilidad fetal.
b.- Busqueda de sndrome de HELLP.
c.- Sulfato de magnesio 4 g IV.
d.- Prepara a la paciente para Cesarea.
CASO CLINICO ECLAMPSIA
Se trata de femenino de 27 aos con 32 semanas de gestacin que
llega al servicio de urgencias por presentar cefalea intensa, acufenos y
fosfenos, dolor abdominal en barra, adems dolor en regin
lumbosacra y sensacin de pesantes plvica, a la exploracin fsica se
observa fondo uterino de 28 cm, obesidad grado I, ROTs
incrementado, edema de miembros inferiores +++, al tacto observa
dilatacin de 2 cm, con presencia de moco con sangre, se observan
protenas en orina ++, nivel de conciencia disminida, leve
desorientacin temporo-espacial, el familiar refiere que la encontr en
el suelo con somnolencia.
PREGUNTA
Cul es la conducta a seguir de forma inmediata?
RESPUESTA
a.- Indica Sulfato de Magnesio.
b.- Indica Hidralacina.
c.- Preparar para Cesarea.
d.- Preparar para Parto.

Pharmed Solutions Institute

PGINA 263

MANUAL DE TRABAJO DEL CURSO ENARM CMN SIGLO XXI


CASO CLINICO ECLAMPSIA
Se trata de femenino de 18 aos de edad con 32 semanas de gestacin
por fecha de ltima menstruacin, sin atencin prenatal, es llevada a
urgencias por presentar en casa perdida del estado de alerta y con
movimientos tipo crisis convulsivas, a la exploracin se observa
somnolienta con respuesta a estmulos dolorosos, ha sido estabilizada
en sus constantes vitales, los resultados de laboratorios son los
siguientes, plaquetas de 45,000, proteinuria +++, DHL de 590, AST 239,
ALT 432.
PREGUNTA
Tomando en cuenta la gravedad del caso, cual es la mejor conducta a
seguir?
RESPUESTA
a.- Realizar Cesrea.
b.- Sulfato de magnesio.
c.- Estabilizar plaquetas.
d.- Maduracin Pulmonar.
CASO CLINICO
Paciente de 36 aos con historia gineco-obsttrica G3, P2, A0,
actualmente con embarazo de 34 semanas de edad gestacional por
amenorrea y por ecografa del primer trimestre, es atendida en
institucin de segundo nivel por presentar cefalea, visin borrosa,
nauseas. Al examen clnico, cifras de presin arterial elevadas 180/110
mmHg. Con diagnstico presuntivo de preeclampsia severa, realizan
los siguientes exmenes paraclnicos en el nivel de atencin inicial: Hb
12.3 gr/dl, Hto 38%, Recuento de plaquetas 70.000/mm3, Creatinina
1.1 mg/dl, proteinuria ++ en tirilla reactiva, ALT 810 U/l, AST 900
PREGUNTA
Cul es el diagnstico ms probable en este caso?
RESPUESTA
a.- Hipertensin gestacional
b.- Eclampsia severa
c.- HELLP
d.- Preeclampsia severa

PREGUNTA
Cul es el diagnotico mas probable para este caso?
RESPUESTA
a.- Preeclampsia leve
b.- preeclampsia moderada
c.- Eclampsia
d.- HELLP
PREGUNTA
Cul es la conducnta mas adecuada a seguir en este momento?
RESPUESTA
a.- Directo a cesrea
b.- Dejar evolucionar a parto
c.- Profilaxis para eclampsia
d.- Toma de laboratorios
PREGUNTA
Durante el puerperio, recupera la visin de manera progresiva,
logrando ser normal 4 das postparto. Evoluciona con cifras tensionales
elevadas, hasta 169/107 mmHg, requiriendo terapia antihipertensiva.
Cul es la incidencia de amaurosis en pacientes eclmpticas?
RESPUESTA
a.- 1-3%
b.- 3-14%
c.- 15-20%
d.- 0.1-0.8%
PREGUNTA
Cul es el pronstico para esta paciente?
RESPUESTA
a.- Bueno para la funcin y la vida
b.- Malo para la funcin, bueno para la vida
c.- Bueno para la funcin malo para la vida
d.- Malo para la funcin y la vida

PREGUNTA
Qu parmetro es menos probable encontrar en esta patologa?
RESPUESTA
a.- Incremento de la DHL >600U/l
b.- Proteinuria <2 g/l
c.- Presencia de esquistocitos
d.- Incremento de bilirrubina directa >1.2mg/dl
PREGUNTA
En qu clasificacin de Sibai se encuentra la paciente?
RESPUESTA
a.- Tipo I
b.- Tipo II
c.- Tipo III
d.- Tipo IV
CASO CLINICO
Primigesta de 24, que cursa un embarazo de 36.4 semanas de
amenorrea. Sin antecedentes mrbidos de importancia. El control
prenatal registra cifras tensionales normales previo a los ltimos tres
controles obsttricos donde se observa cifras de 130/80. El da previo a
su ingreso, acusa visin borrosa de 10 minutos de duracin, cediendo
espontneamente. El da de su ingreso debuta con amaurosis bilateral,
sndrome vertiginoso y cefalea, por lo que consulta en el Servicio de
Urgencia de Obstetricia, donde se decide su hospitalizacin en
preparto. Ingresa subfebril, hipertensa (170/110 mmHg), taquicrdica,
sudorosa, con amaurosis bilateral, edema facial (+), reflejos

CURSO ENARM CMN SIGLO XXI TEL: 36246001

osteotendneos exaltados simtricos (+++), edema extremidades


inferiores simtricos (++) y pulsos presentes en las cuatro
extremidades. El examen obsttrico revela tero en reposo, tono
normal, altura uterina de 29 cm, feto vivo. El tacto vaginal muestra
cuello cervical con ndice de Bishop 3. Monitorizacin fetal normal.

CASO CLINICO
Adolescente de 17 aos, blanca, sana, G3 P1 A2, a la que se haba
practicado una cesrea por desproporcin cefaloplvica a las 39,1
semanas de edad gestacional, despus de un embarazo sin
complicaciones, dada de alta a los 3 dias. A las 24 h acude con cefalea y
mareos. EF: FC 125 lpm; PA, 100/70mmHg y palidez cutaneomucosa.
Fue ingresada con diagnstico de anemia severa (Hto: 0,17%); se le
transfundieron glbulos sanguneos y se le dio el alta al cuarto da con
Hto 026% y tratamiento antianmico. Volvi a ingresar a los 9 das de
puerperio con sntomas similares, presentaba palidez cutaneomucosa
intensa, Hto 018%, Hb 59g/l; leucocitos, 14,4 109/l; segmentados,
077; linfocitos, 013; eosinfilos, 001; monocitos, 001; mielocitos, 003;
plaquetas, 300109/l; lmina perifrica: anisopoiquilocitosis, punteado
basfilo, fragmentocitos, grnulos txicos xxx; prueba de Coombs
negativa, prueba de paracoagulacin negativa; creatinina: 744 mOsm/l
y proteinuria: trazas. Ese da se constat PA de 140/100mm Hg.
PREGUNTA
Cul es el diagnostico mas probable para este caso?
RESPUESTA
a.- Sepsis
b.- Preeclampsia-eclampsia
c.- CID
d.- Meningitis

Pharmed Solutions Institute

PGINA 264

MANUAL DE TRABAJO DEL CURSO ENARM CMN SIGLO XXI


DIABETES MELLITUS, INTOLERANCIA A LA GLUCOSA, DIABETES GESTACIONAL
CIENCIAS BASICAS: Cambios fisiolgicos en el embarazo normal: El embarazo normal se considera un estado diabetognico o de resistencia progresiva al
efecto de la insulina, durante el primer trimestre y las etapas iniciales del segundo se eleva la sensibilidad a la insulina, lo que se ha atribuido a las
mayores concentraciones de estrgenos circulantes. Este fenmeno incrementa el depsito de energa, sobre todo en el tejido adiposo, con expansin
del mismo; pero a partir de las 24 a 28 semanas de gestacin aumenta paulatinamente la resistencia a la insulina, que puede alcanzar los niveles que se
observan en pacientes diabticos tipo 2. Ms evidente al final del segundo trimestre del mismo; el organismo de la madre se prepara almacenando
nutrientes y en esta etapa la sensibilidad a la insulina es mayor; sin embargo, en la segunda mitad de
la gestacin hay una resistencia progresiva al efecto de la insulina, lo que causa una liberacin de los
nutrientes (glucosa, los aminocidos, los cidos grasos los triglicridos y los oligoelemntos) para que
el feto tenga un desarrollo adecuado. Las clulas del pncreas elevan la secrecin de insulina en un
intento de compensar la resistencia a la insulina del embarazo, lo que origina pequeos cambios en
la concentracin de insulina en el curso de la gestacin, comparados con los grandes cambios en la
sensibilidad de la misma. El msculo esqueltico es el sitio principal para utilizar la glucosa corporal, y
junto con el tejido adiposo, empiezan a ser resistentes al efecto de la insulina, lo que es ms evidente
durante la segunda mitad del embarazo. Un embarazo normal se caracteriza por aproximadamente un 50% de disminucin en la disponibilidad de
glucosa mediada por insulina. Barbour seala un incremento en la secrecin de insulina hasta de 200% para tratar de mantener euglucmica a la madre.
Una gran cantidad de sustancias producidas por la placenta y por los adipocitos, (lactgeno placentario, hormona placentaria de crecimiento, prolactina,
hormona liberadora de corticotropina-cortisol, insulinasa, FNT y adipocitocinas [leptina, resistina, visfatina, adiponectina]), son las que reprograman la
fisiologa materna y causan este estado de resistencia a la insulina para dirigir los nutrientes hacia el feto en desarrollo, sobre todo en la segunda mitad
del embarazo. El lactogeno placentario se eleva unas 30 veces durante la gestacin. En los obesos hay una correlacin positiva entre el FNTa y el IMC e
hiperinsulinemia. RESISTENCIA A LA INSULINA: En esta resistencia a la insulina participan numerosas sustancias producidas por la placenta y por los
adipocitos, entre las cuales destacan la hormona de crecimiento placentaria y el FNTa, que por diversos mecanismos disminuyen el efecto de la insulina a
nivel intracelular. La primera aumenta la cantidad de la subunidad p85a de la PI-3K (fosfatidil-inositol 3 cinasa), lo que al final disminuye un factor de
trascripcin AKT que altera el desplazamiento de los transportadores de glucosa (GLUT) hacia la membrana celular y disminuye la entrada de glucosa a la
clula; el segundo altera la fosforilacin de la subunidad beta del receptor de insulina y del IRS-1, ya que fosforila residuos de serina y treonina en lugar
de los de tirosina. Este evento conduce a la mayor degradacin de ambos y modifica la accin de la insulina. Otro mecanismo implicado es que en el
musculo esqueltico el GLUT-4 esta disminuido en el tejido adiposo en mujeres embarazadas, y es an menor en la diabetes mellitus gestacional;
adems, est alterada la traslocacin de estos trasportadores. Los cambios moleculares en el adipocito durante el embarazo muestran reduccin en la
trascripcin del PPARg, receptor nuclear que regula la trascripcin de varios genes centrales en el metabolismo del adipocito (adiponectina, la
lipoprotein lipasa, la protena P-2 fijadora de cidos grasos intracelulares y la protena no acoplada mitocondrial). Barbour hall hasta 40 a 50% de
reduccin en el ARNm del PPARg (un factor que suprime considerablemente a este receptor nuclear es el FNTa) en embarazadas obesas y DM
gestacional; esto altera la concentracin de cidos grasos en la sangre, y las lipoprotenas ricas en triglicridos (VLDL) con mayor resistencia para el
efecto de la insulina ya que la adiponectina es un sensibilizador de esta hormona y estos cidos grasos entran a la clula como fuente energtica. SALUD
PUBLICA: La prevalencia de diabetes gestacional a nivel mundial se ha estimado en 7%. En Mxico se ha mencionado una frecuencia que vara entre 4 y
19% de la poblacin obsttrica. Respecto a la edad de la madre, se ha sealado que la incidencia es de 0.4 a 0.5% en menores de 25 aos y de 4.3 a 5.5%
en mayores de esa edad. Lo cierto es que la frecuencia de este trastorno se ha duplicado en la ltima dcada, en forma paralela a la llamada pandemia
metablica que afecta a las sociedades modernas. Ms de 90% de los casos de diabetes que complican a un embarazo son casos de diabetes gestacional.
La creciente prevalencia de diabetes, que se diagnostica en mujeres a edades tempranas, favorece la presencia de diabetes y embarazo. Las mujeres que
desarrollan diabetes gestacional tienen alto riesgo de desarrollar diabetes tipo II, en los 10 aos que siguen al embarazo. PATOGENIA: Factores de
riesgo: 1) Edad mayor de 25 o 30 aos. 2) IMC > 25 kg/m2 SC. 3) Antecedentes de hijos macrosmicos. 4) Diabetes mellitus en familiares de primer grado
5) Antecedentes de intolerancia a la glucosa. 6) Glucosuria. 7) Ganancia de ms de 20 kg de peso en la actual gestacin. 8) Antecedentes de problemas
obsttricos serios. 9) Peso bajo del feto al nacimiento (<10 percentil) para la edad gestacional. 10) Peso alto del feto al nacer (> del 90o percentil). 11)
Origen tnico de riesgo alto de obesidad o diabetes mellitus. Los ms importantes son: mayor edad en la madre, familiares de primer grado con diabetes
y mayor ndice de masa corporal pregestacional. En la segunda mitad del embarazo se requiere un estado fisiolgico de resistencia a la insulina para
dirigir los nutrientes almacenados en la madre hacia la unidad fetoplacentaria y dar un crecimiento adecuado al feto; sin embargo, cuando las mujeres
desarrollan diabetes mellitus gestacional, la resistencia a la insulina es ms acentuada, lo cual modifica el medio intrauterino y causa crecimiento
acelerado del feto, con riesgo elevado de macrosoma. Los datos disponibles en la bibliografa muestran un aumento lineal en el ndice de
complicaciones en el feto conforme se eleva la cifra de la glucosa en la sangre materna: ndices de mortalidad del feto de 4% cuando la glucosa es menor
de 100 mg y hasta de 24% cuando la cifra excede 150 mg. DIABETES PREGESTACIONAL: o preexistente se refiere a pacientes conocidas con diabetes
mellitus tipo I o II que se embarazan. Los hijos de mujeres con diabetes pregestacional tienen alto riesgo de sufrir malformaciones estructurales
congnitas, aborto espontaneo y restriccin del crecimiento intrauterino, lo que se debe a que la hiperglucemia y la hiperinsulinemia en el feto pueden
ser teratognicas, ya que existe una fuerte asociacin entre un pobre control glucmico en el periodo periconcepcional y el riesgo de tales
malformaciones. La incidencia de riesgo de estas malformaciones es de 6 a 8% y la mayora ocurre en el sistema nervioso central, el corazn, el sistema
urogenital y el tubo gastrointestinal. DIABETES GESTACIONAL: Es un padecimiento caracterizado por la intolerancia a los carbohidratos con diversos
grados de severidad, que se reconoce por primera vez durante el embarazo y que puede o no resolverse despus de este. Existe mayor riesgo de
macrosoma y polihidramnios. Se incrementa el riesgo de prematurez, preeclamsia/eclampsia, trauma obsttrico, cesrea y mortalidad perinatal.
DIAGNOSTICO: Escrutinio; prueba de escrutinio con 50 g de glucosa oral entre las semanas 24 a 28 de la gestacin. Una cifra de 140 mg con diabetes
mellitus gestacional o bien 130 mg a la hora, que identificara al 90%. Sin embargo, a pesar de tener la mayor sensibilidad (79%) y especificidad (87%) de
todas las pruebas de escrutinio disponibles, sta se reserva para pacientes con alto riesgo para padecer la enfermedad, ms que para la poblacin
general. Existen 4 formas de realizar el diagnostico de Diabetes Gestacional: 1) Glucemia en ayuno >126mg/dl en dos ocasiones. 2) Glucemia casual
>200mg/dl. 3) Prueba de tamiz con 50g con resultado >180mg/dl (170mg/dl en mujeres mayores de 30 aos). 4) Curva de tolerancia a la glucosa con
100g o 75 g. Se realiza el diagnostico al tener alterados 2 o ms de los siguientes valores: ver cuadro anexo. En caso de tener alterado un solo valor se
diagnostica intolerancia a los carbohidratos. COMPLICACIONES MATERNO-FETALES: En el feto; Macrosoma, muerte intrauterina, asfixia neonatal,
distocia de hombro y lesin nerviosa, hipoglucemia, hipocalcemia, ictericia, SIRPA, Taquipnea transitoria, cardiomiopata (hipertrofia septal),
eritrocitosis, trombosis, sobre todo de la vena renal. Cuando son adultos: obesidad, diabetes mellitus, alteraciones neuropsicolgicas. En la madre;
Preeclampsia, eclampsia, polihidramnios, desgarros perineales, mayor nmero de cesreas, riesgo de padecer diabetes mellitus II, parto prematuro.
TRATAMIENTO: La meta teraputica de glucosa sangunea durante el embarazo es: Glucemia central en ayuno entre 60-90mg/dl y menor de 140mg/dl
una hora pospandrial o menor a 120mg/dl a las 2 h pospandrial. El objetivo fundamental del tratamiento es mantener la normoglucemia con una dieta

CURSO ENARM CMN SIGLO XXI TEL: 36246001

Pharmed Solutions Institute

PGINA 265

MANUAL DE TRABAJO DEL CURSO ENARM CMN SIGLO XXI


adecuada, reduciendo el consumo de carbohidratos a 40-45% de la ingestin calrica total. Las pacientes con IMC superior a 30 kg/m2 de SC deben
disminuir su ingestin calrica en 30 a 33% o alrededor de 25 kcal/kg de peso corporal. El peligro de la restriccin calrica muy estricta en la embarazada
es la produccin de cetosis, que puede alterar el desarrollo psicomotor del feto. La actividad fsica adaptada a la fisiologa materna de acuerdo con la
edad gestacional. Si con los dos recursos teraputicos sealados no se obtiene un control adecuado, ser necesaria la administracin de insulina en
forma juiciosa para mantener cifras de glucosa dentro de lmites normales para la mujer embarazada. Se debe evitar un tratamiento intenso que cause
hipoglucemia, lo que ocurre hasta en 60% de las pacientes, en muchos casos asintomtica para la madre, pero con diversas repercusiones para el feto.
En pacientes con insulina la dieta fraccionada con una colacin nocturna, disminuye la probabilidad de hipoglucemias. Los anlogos de insulina de accin
rpida lispro y aspart son seguros en el embarazo y tienen algunos beneficios comparados con la insulina rpida, como, menos episodios de
hipoglucemia, un mejor control de la glucemia postpandrial y mayor satisfaccin del paciente. La insulina NPH es tambin segura y se utiliza en
combinacin con las anteriores. El esquema de tratamiento con insulina puede consistir en: Se indica el fraccionamiento de la dosis de insulina en dos
aplicaciones cuando: 1) Continua glucemia en ayuno elevada a pesar de incremento de la dosis de insulina matutina. 2) Se requiere ms de 20 U de
insulina para lograr control metablico. 3) Se adiciona insulina rpida cuando no se alcanza la meta teraputica de la glucemia pospandrial. Esquema:
Combinacin de insulina rpida e intermedia administrada 30min antes del desayuno y de la cena. Los requerimientos varan de 0.3-1.5 UI por kg de peso
real y de acuerdo a descontrol glucmico, as como a la presencia de sobrepeso, edad gestacional o enfermedades que modifiquen la glucemia como la
hipermesis gravdica o infecciones. La dosis total calculada se dividir en 2/3 pre-desayuno y 1/3 pre-cena (30 min antes de los alimentos). La razn ser
NPH/rpida 2/1 pre-desayuno y 1/1 pre-cena. Se recomienda iniciar con la mnima dosis de insulina e incrementarla gradualmente de acuerdo al
automonitoreo con glucemias capilares pre y postpandriales, por lo menos 3 veces al da. Los anlogos de insulina deben ser utilizados hasta que se
disponga de mayores datos o exista respuesta a las interrogantes que an existen acerca de su uso, como: teratogenicidad, incremento en el dao a la
retina o incluso a la formacin de anticuerpos y reactividad inmunolgica de la insulina. Slo dos de los frmacos hipoglucemiantes orales han sido
utilizadas: una es un secretagogo de insulina (glibenclamida) y otro un sensibilizador de insulina (metformina). Aunque diversos grupos de expertos
restringen su uso, los datos disponibles muestran que en algunas se logra un control adecuado, adems que se encuentran dentro de la clasificacin C o
D, lo que significa que nos son seguros. La elevacin posprandial de la glucosa causa alteraciones en el feto. Un frmaco que puede utilizarse es la
acarbosa, que se absorbe en muy poca cantidad y, segn algunos trabajos, ha logrado mejor control de la glucemia posterior a la ingestin de alimentos.
El control adecuado de la glucosa sangunea es el objetivo ms importante del tratamiento de la diabetes mellitus gestacional y, en teora, este control
ptimo disminuira la morbilidad y mortalidad maternofetal. PREVENCION: La Federacin Internacional de Diabetes y la Asociacin Americana de
diabetes proponen los siguientes puntos para el consejo preconcepcional: 1) En toda mujer en edad frtil en etapa reproductiva con diabetes, identificar
anualmente por interrogatorio directo el deseo de embarazo, en caso de no desearlo proporcionar consejo sobre mtodo anticonceptivo. 2) En la eta
preconcepcional y de embriognesis, lograr un control glucmico estricto con una cifra ideal de Hb glicosilada menor a 6.1%, evitando la presencia de
hipoglucemia, para disminuir la probabilidad de malformaciones. 3) Con base, en que a mayor dao vascular mayor riesgo de complicaciones maternofetales, definir el riesgo establecido si existe dao por micro o macroangiopatia utilizando la clasificacin de Priscilla White. 4) En caso de diabetes tipo I,
determinar la funcin tiroidea asociada, ya que hasta un 17% pueden cursar con esta co-morbilidad. 5) En el momento en que se establece el diagnostico
de embarazo, suspender hipoglucemiantes orales o insulina glargina si la paciente los estaba utilizando. 6) Suspender los IECA y ARA II, si la paciente los
estaba utilizando y usar alfametildopa, nifedipina, labetalol o hidralazina. 7) Mantener las cifras de tensin arterial por debajo de 130/80mmHg. 8)
Suspender estatinas y fibratos. 9) Suspender el consumo de alcohol y tabaco. 10) Iniciar cido flico tres meses antes del embarazo. No es recomendable
el embarazo si la mujer diabtica presenta cualquiera de las siguientes situaciones: HbA1c >10%. Cardiopata isqumica. Nefropata avanzada
(depuracin de creatinina <30ml/min y Cr srica >1.4mg/dl). Retinopata proliferativa.
CASO CLINICO DIABETES MELLITUS
Paciente femenino de 34 aos de edad la cual acude a consulta debido
a que presenta amenorrea de 8 semanas de gestacion, agrega que ha
presentado leve fatiga, anorexia con nausea matinal, usted enva
laboratorios de rutina presentando glucosa en sangre de 200 mg/dl y
hemoglobina glucosilada de 7.5 refiere la paciente que fue
diagnosticada con diabetes hace 5 aos por lo que recibe Tolbutamida,
adems recibe captopril por hipertensin arterial diagnosticada hace 3
aos. Al interrogatorio la paciente refiere antecedentes de madre
diabtica e hipertensa, sus antecedentes GO refiere menarca a 13
aos, gesta 3, para 2, abortos 0, adems refiere que su embarazo
anterior hace 5 aos presento un incremento de 14 kilogramos con
producto que preso 3,750 grs. Por va cesrea a las 36 semanas de
gestacin.
PREGUNTA
Cul es la conducta no adecuada a seguir?
RESPUESTA
a.- Suspender hipoglucemiente.
b.- Suspender IECA.
c.- Indicar interrupcin de embarazo.
d.- Indicar alfametildopa.
CASO CLINICO DIABETES GESTACIONAL
Paciente femenino de 21 aos de edad, la cual presenta 24 semanas de
gestacin, acude a control prenatal, refiere ardor al orinar, deseos
continuos de orinar, con urgencia, usted identifico un incremento de
peso hasta el momento a partir de su embarazo de 10 kilos. Sus
antecedentes presentan, obesidad previa al embarazo, padre diabtico
actualmente con insuficiencia renal en tratamiento con hemodilisis,
madre hipertensa bajo tratamiento.

CURSO ENARM CMN SIGLO XXI TEL: 36246001

PREGUNTA
Considerando el cuadro clnico. Cuales son las complicaciones que el
producto puede presentar?
RESPUESTA
a.- Producto Macrosomico.
b.- Diestres respiratorio.
c.- Hipoglucemia.
d.- Parto pretermino.
CASO CLINICO
Mujer de 34 aos de edad, originaria del Estado de Veracruz, con
antecedente de madre y abuela materna con DM 2. Secundigesta,
cursa con embarazo de 18 semanas de gestacin por fecha de ltima
regla. Acudi por vez primera vez, 2 semanas atrs (15.3 semanas de
gestacin) para control de su embarazo. A su ingreso se realiz una
prueba de tamiz de glucosa la cual se report en 168 mg/dl (una hora
post carga oral de glucosa). Una semana despus le fue realizada una
CTOG de 180 minutos cuyos resultados fueron: 0 = 107 mg/dl, 60 =
205 mg/dl, 120 = 178 mg/dl y 180 = 142 mg/dl. Una HbA1c realizada
en ese momento se report en 8.6 %.
PREGUNTA
Qu probabilidad tiene el feto de sufrir malformaciones genticas por
el descontrol en la madre?
RESPUESTA
a.- <5%
b.- 10-15%
c.- 20-25%
d.- 30-35%

Pharmed Solutions Institute

PGINA 266

MANUAL DE TRABAJO DEL CURSO ENARM CMN SIGLO XXI


PREGUNTA
En qu semana de gestacin es ms adecuado realizar el tamiz
metablico, en pacientes sin factores de riesgo?
RESPUESTA
a.- 13-18SDG
b.- 20-23SDG
c.- 24-28SDG
d.- 29-32SDG
PREGUNTA
Cul de los siguientes requisitos es el menos probable para realizar
una curva de tolerancia de glucosa?
RESPUESTA
a.- Por la maana con ayuno de 10-12 hrs
b.- Alimentacin sin restriccin calrica 3 das antes de la prueba
c.- Suspender alfa-metildopa
d.- No debe tener infeccin aguda
CASO CLINICO
Paciente de 36 aos de edad con normopeso, sin antecedentes
personales de inters ni familiares, salvo haber presentado DMG en la
semana 9 de gestacin en su primer embarazo en 1997. Present
hiperglucemias con anomala de la glucosa en ayunas en dos analticas
del primer trimestre, por lo que se realiz una curva de tolerancia a la
glucosa, cuyos valores fueron los siguientes: glucemia basal de 138
mg/dl y a los 60 min, 266 mg/dl, motivo por el cual no se efectu una
prueba de sobrecarga oral de glucosa (SOG) prolongada. Recibi
insulinoterapia por no alcanzar los objetivos glucmicos deseados
preprandiales y posprandiales, con lo que mantuvo un buen control
metablico durante ese embarazo, con concentraciones de
glucohemoglobina (HbA1c) del 4,8-5% durante la gestacin.
PREGUNTA
Qu probabilidad tiene esta paciente de desarrollar diabetes mellitus
tipo 2, posterior al embarazo?
RESPUESTA
a.- 20-30%
b.- 50-80%
c.- 30-50%
d.- 80-100%
PREGUNTA
Cul de las siguientes manifestaciones en el RN, es menos probable
que sea causado por la hiperinsulinemia en el feto?
RESPUESTA
a.- Cardiomiopatia obstrucctiva
b.- Taquipnea transitoria
c.- Hipoglucemia
d.- Cifras de bilirrubinas bajas
PREGUNTA
Cul de las siguientes es menos probable que sea de alto riesgo para
diabetes gestacional?
RESPUESTA
a.- Paciente con intolerancia a los carbohidratos
b.- Paciente con glucosuria actual
c.- Paciente de 24 aos con primo, tio maternos con diabetes mellitus
d.- Paciente con productos macrosomicos anteriores
CASO CLINICO
Paciente femenino de 34 aos, ama de casa, catlica. Paciente quien
acude a su primer control prenatal, con clculo para edad gestacional
de 24 semanas por ltima regla. Un examen de orina extrahospitalario
evidencia Glucosuria +++, sin cetonas. Antecedentes: madre con

CURSO ENARM CMN SIGLO XXI TEL: 36246001

diabetes mellitus 2, Menarquia 8 aos, Coitarquia 16 aos Ciclos


regulares (30 x 5 das). No usaba ningn metidi de planificacin
familiar. G: 2, C1 (masculino a las 40 semanas, peso 4,200Kg, refiere
que su hijo se encuentra bien. EF: PA 130/80, Fc 78x, Fr 18x T 37, peso
100 kg, talla 160 cms, IMC 39 kg/m2. Alerta, en buen estado general,
mucosas humedas, cuello mvil, sin adenopatas ni masas, trax
expandible, murmullo vesicular audible, corazn rtmico, abdomen
blando depresible, tero grvido, FU 20 cms, ocupado por feto nico
vivo, indiferente, FCF 140, genitales externos de aspecto normal, TV no
hemorragia, cuello firme posterior, leucorrea moderada, grumosa,
ftida, blanquecina, resto, miembros inferiores mviles, no edema,
resto EF normal.
PREGUNTA
Qu factor de riesgo es menos probable para pensar en diabetes
gestacional?
RESPUESTA
a.- Obesidad
b.- Antecedente familiar de diabetes
c.- Producto macrosomico
d.- Glucosuria +++
PREGUNTA
Qu exmen es de menos utilidad para evaluar la tolerancia a la
glucosa de la paciente?
RESPUESTA
a.- Test de OSullivan
b. Curva de tolerancia a la glucosa
c.- Glicemia en ayunas
d.- Hemoglobina glicosilada
CASO CLINICO
Vienen Mara y Javier, muy preocupados por el tema de "la diabetes".
Les tranquilizamos y les explicamos que es un problema relativamente
frecuente en el embarazo, sobre todo con los antecedentes de Mara,
pero que tomando las medidas adecuadas, no tiene porque repercutir
en el bebe ni en la mam en este embarazo. Es una "diabetes" que
"aparece y desaparece" con el embarazo. S le explicamos, sin
embargo, que esto si puede indicar mayor riesgo de Mara de
desarrollar algn tipo de alteracin al metabolismo de la glucosa
tiempo despus del parto.
PREGUNTA
Cul es la conducta teraputica mas adecuada?
RESPUESTA
a.- Dieta hipocalrica y autoanlisis de glucemia y cetonuria
b.- Dieta equilibrada y autoanlisis de glucemia y cetonuria
c.- Dieta equilibrada e insulina
d.- Dieta equilibrada y antidiabticos orales
PREGUNTA
En la siguiente visita a los 15 das, en los controles de autoanalisis
presenta glucemias basales de 70-80 mg/dl y postpandriales de 110
mg/dl. No ha incrementado de peso en las ultimas 2 semanas, y
presenta cetonuria positiva (1+). Ante la cetonuria positiva, Cul es la
conducta a seguir?
RESPUESTA
a.- Mantenemos el mismo tratamiento
b.- Insistimos en la dieta y actividad fsica, pues no creemos que lo est
haciendo bien
c.- Insulinizamos
d.- Aumentamos aporte calrico y de hidratos de carbono en la dieta

Pharmed Solutions Institute

PGINA 267

MANUAL DE TRABAJO DEL CURSO ENARM CMN SIGLO XXI


POLIDRAMNIOS Y OLIGOHIDRAMNIOS
CIENCIAS BASICAS: Se define como un volumen de lquido amnitico mayor a 1500-2000ml, secundario a un aumento en la produccin o deficiencia en
la eliminacion. Segn el ndice de Phelan, definimos polihidramnios con un ndice de lquido amnitico >25 cm (8-18 normalidad ecogrfica del ILA, 18-25
valores lmite superior, >25 polihidramnios). El lquido amnitico (LA) es el fluido que ocupa la cavidad amnitica y que contribuye en importantes
funciones para un adecuado desarrollo del embarazo. Sirve de proteccin para el feto frente a traumatismos, mantiene una temperatura adecuada,
permite el desarrollo de determinados rganos vitales y puede aportar informacin acerca del estado y madurez fetal. Su volumen vara a lo largo de la
gestacin. Aumenta de 50 ml en la semana 12 a 400 ml en la semana 20. Hacia la 38 semana puede alcanzar valores de 1000 ml y al trmino es
aproximadamente de 800 ml, oscilando entre 300 y 1500 ml. El LA renueva a un ritmo de 500 ml/hora y que en un perodo de 2-3 horas todo el
contenido de agua del LA se ha renovado. Se ha visto que la cuarta parte de esta circulacin se realiza a travs del feto y el cordn umbilical; y el resto, a
travs de las membranas ovulares y superficie placentaria. El intercambio de LA a travs del feto puede realizarse por las siguientes vas: aparato
digestivo, respiratorio, urinario y piel. Se calcula que el feto puede deglutir de 5 a 7 ml hora, cantidad escasa en proporcin a la circulacin total de
lquido amnitico. SALUD PUBLICA: Incidencia oscila entre 0.13-3.2% de los embarazos. Se asocia con una alta mortalidad perinatal (60%). PATOGENIA:
Se puede deber a causas Idioptico (65%), maternas (7%): DM (en estos casos, el polihidramnios se atribuye a diversas causas, como un aumento de la
osmolaridad del LA debido a una elevacin de la glucosa, una disminucin de la deglucin fetal y poliuria fetal por la hiperglucemia), isoinmunizacin al
grupo y Rh) como fetales (13%): del tracto gastrointestinal (atresia esofgica, atresia duodenal, agnatia, fisura palatina, labio leporino, artrogriposis
mandibular, obstruccin intestinal, hernia diafragmtica, pncreas anular, gastrosquisis, peritonitis meconial, onfalocele), defectos del tubo neural,
enfermedades cardiovasculares fetales, patologa del tracto urinario(rin poliqustico, obstruccin ureteroplvica), embarazo mltiple, hidropesa fetal
inmune y no inmune) y por ultimo causas placentarias (corioangioma placentario: el tumor benigno ms frecuente en la placenta). CLASIFACION: Segn
la severidad se clasifica de acuerdo al ILA en LEVE; cuando el ndice de lquido amnitico (ILA) se encuentra entre 25-30cm. MODERADO; se encuentra
entre 30.1-35cm. GRAVE; por arriba de 35 cm. Por su evolucin se clasifica en agudo (2%), de inicio sbito, aparece en el segundo trimestre, se asocia a
malformaciones congnitas no compatibles con la vida y rpidamente evoluciona a parto pretrmino. Crnico (98%), aparece en el tercer trimestre, se
asocia a factores maternos y es principalmente idioptico. DIAGNOSTICO: La sintomatologa generalmente es secundaria a la sobresdistensin
abdominal y uterina, y dependiendo del grado de extensin puede ir desde fatiga leve, hasta la presencia de disnea, cianosis y edema (miembros
inferiores, vulvar y de pared abdominal), aumento de peso, desarrollo de preeclampsia, as como trabajo de parto. Se sospecha ante la presencia de un
fondo uterino mayor al esperado, con abdomen a tensin y dificultad para palpar partes fetales, dificultad para auscultar foco fetal y palpacin subjetiva
de aumento de lquido amnitico. El USG nos brinda el diagnostico con la medicin de lquido amnitico, la tcnica ms utilizada es la de Phelan. La
medida de la cantidad del lquido amnitico se puede realizar utilizando diferentes tcnicas, las dos ms utilizadas son la mxima columna vertical y el
ndice de lquido amnitico (ILA). Mxima columna vertical: Se realiza midiendo la mxima columna vertical de lquido libre de partes fetales y de cordn
de manera vertical. Es el mtodo de eleccin en gestaciones mltiples y en gestaciones nicas de menos de 24 semanas. Se considera normal cuando es
superior a 2 cm en todas las edades gestacionales y cuando es inferior a 8 cm por debajo de la semana 20 o a 10 cm a partir de la semana 21. ILA: Es el
valor obtenido a partir de la suma de las mximas columnas verticales de lquido, libre de partes fetales o cordn umbilical, en cada uno de los cuatro
cuadrantes que se delimitan por la interseccin de dos lneas perpendiculares en el abdomen materno: la lnea media longitudinal con la lnea transversal
media entre la snfisis pbica y el fondo uterino. El transductor se coloca en posicin sagital y lo ms perpendicular posible al suelo. Se consideran
normales valores de ILA entre 5 y 25 centmetros. La medida del ILA ha demostrado tener una superior reproducibilidad y mejor deteccin de patologa
por lo que se considera tcnica de eleccin en la medida de la cantidad de lquido amnitico en las gestaciones nicas de ms de 24 semanas. Tambin el
USG puede detectar malformaciones fetales. Los exmenes de laboratorio son importantes para poder determinar una causa no idioptica; se debe
tomar en cuenta la curva de tolerancia a la glucosa, los anticuerpos anti toxoplasma, CMV, tipo sanguneo y Rh y el cariotipo fetal analizado a travs del
lquido amnitico. TRATAMIENTO: El manejo depende dela etiologa, la edad gestacional y la intensidad, llegando a requerirse amniocentensis
evacuadora en casos con compromiso materno. El medicamento de eleccin es la indometacina su uso se limita a partir de 28-32 SDG, es un inhibidor de
las prostaglandinas vasodilatadoras, por lo cual produce una vasoconstriccin arteriolar en el rin fetal disminuyendo el filtrado glomerular con la
subsecuente disminucin de la produccin de orina fetal que es la principal fuente de lquido amnitico a partir del segundo trimestre. La dosis usada
vara entre 1,5 y 3 mg/Kg por da, siendo la ms usada de 25 mg cada seis horas. Su empleo debe ser controlado por el riesgo de cierre del conducto
arterioso fetal. La va de interrupcin depender de las condiciones obsttricas y fetales. Debe monitorizarse el ILA bisemanalmente de tal modo que
cuando llegue a 18cm se suspende. Tambin es importante realizar ecocardiograma fetal cada semana, para observar el agujero oval, en caso de cierre
de este debe suspenderse la indometacina ya que el proceso generalmente es irreversible. En pacientes con polihidramnios esta incrementado el riesgo
de parto pretrmino, rotura prematura de membranas, prolapso de cordn, presentaciones distosicas, rotura uterina, atona uterina y desprendimiento
prematuro de placenta normoinserta. La amniocentesis es un procedimiento invasivo, que alivia las molestias maternas, provee lquido que puede
estudiarse (cariotipo, madurez pulmonar, gramm y cultivo), y al disminuir la compresin uterina, mejora el flujo de las arterias espirales. Las
complicaciones de la amniocentesis pueden ser parto pretrmino, DPPN, puncin del feto e infecciones que son raras. El tratamiento depender de la
severidad del caso. En casos leves puede utilizarse nicamente la indometacina. Cuando es moderado a severo, puede utilizarse la indometacina junto
con la amniocentesis. OLIGODRAMNIOS. CIENCIAS BASICAS: Se define como un volumen de lquido amnitico menor a 500ml. Presencia de un ILA
inferior a 5 o de una mxima columna vertical inferior a 2 cm. Se define como oligohidramnios severo/anhidramnios la presencia de una columna
mxima 1 cm. SALUD PUBLICA: Se reporta una incidencia de 5-37%. En el segundo trimestre de la gestacin la presencia de una RPM explica el 50% de
los casos de oligohidramnios, seguido por el RCIU y las malformaciones fetales en el 20% y el 15% respectivamente, siendo un 5% de los casos
idiopticos. PATOGENIA: Podemos dividir las causas de Oligohidramnios en tres grandes grupos: Causas fetales: Crecimiento intrauterino restringido
(CIR), gestacin cronolgicamente prolongada (GCP), infeccin fetal por CMV, obstruccin tracto urinario (ureteral bilateral, valvas uretrales posteriores),
patologa renal (agenesia renal bilateral, displasia renal multiqustica bilateral, riones poliqusticos) y defectos del tubo neural. Causas placentariasmembranas: rotura prematura de membranas (RPM). Causas maternas: medicacin materna (inhibidores de la sntesis de prostaglandinas, IECA).
DIAGNOSTICO: Datos clnicos sugestivos: Disminucin de la motilidad fetal, fondo uterino menor al esperado para la edad gestacional, copn fcil
palpacin de partes fetales, memebranas aplandas a la exploracin vaginal y con alteraciones en la fecuencia cradiaca fetal. Datos paraclnicos: El
mtodo de mayor sensibilidad es sin duda la ultrasonografa, ndice de lquido amnitico con tcnica de Phelan, un ndice menor de 5 se considera
oligohidramnios. Ante el diagnstico de oligohidramnios debemos realizar las siguientes pruebas diagnsticas: 1. Descartar RPM: Mediante anamnesis y
exploracin. Realizar PROM test si no existe hidrorrea franca. Si existe historia clnica sugestiva y el PROM test es negativo valorar la posibilidad de
instilacin de fluorescena intraamnitica mediante amniocentesis. 2. Descartar CIR: Valoracin del peso fetal estimado as como realizacin de estudio
Doppler. 3. Descartar malformaciones fetales: Estudio morfolgico dirigido a descartar la presencia de malformaciones nefro-urolgicas y del tubo
neural. 4. Descartar la infeccin fetal por CMV: serologas maternas (IgG/IgM), marcadores fetales ecogrficos (microcefalia, ventriculomegalia, focos
parenquimatosos hiperecognicos, hiperrefringencia intestinal). 5. Descartar toma de frmacos: inhibidores de la sntesis de prostaglandinas y IECAs.
TRATAMIENTO: No existe tratamiento especfico para esta complicacin por lo que la asistencia se basa en la vigilancia y teraputica de la causa que lo

CURSO ENARM CMN SIGLO XXI TEL: 36246001

Pharmed Solutions Institute

PGINA 268

MANUAL DE TRABAJO DEL CURSO ENARM CMN SIGLO XXI


origin y la edad gestacional en la que se encuentra. El manejo clnico del Oligohidramnios depende principalmente de la causa del mismo: A) En aquellos
casos en los que se diagnostique una RPM o un CIR se aplicara el protocolo especfico de cada patologa. B) En el caso de toma de frmacos se
interrumpir la toma de los mismos de forma inmediata. Si la paciente ha consumido inhibidores de la sntesis de prostaglandinas se realizar valoracin
del ductus arterioso. Si existiera una restriccin (IP<1 o insuficiencia tricuspdea significativa (holosistlica, 150 cm/s)) se realizara control cada 48 h
hasta su normalizacin. C) En aquellos casos en los que el feto presente una malformacin, se informar del pronstico de la misma y del riesgo de
hipoplasia pulmonar y en funcin de esta informacin los padres podran acogerse a la interrupcin legal del embrazo. Si los padres deciden seguir
adelante con la gestacin debemos realizar amniocentesis/cordocentesis para estudio de cariotipo y valorar el estudio de la funcin renal en orina fetal.
D) La evidencia disponible indica que las pacientes con oligohidramnios idioptico no presentan peores resultados neonatales en comparacin con la
poblacin con lquido amnitico normal. Manejo anteparto: en casos menores de 34 SDG se deben emplear esteroides para maduracin pulmonar y
pruebas de biestar fetal. Realizacin de Perfil biofsico y estudio Doppler, semanal hasta las 36.6 semanas y a partir de la semana 37.0 cada 72 horas.
Estimacin de peso fetal cada dos semanas. Finalizacin de la gestacin: Se mantendr una conducta expectante hasta las 40 semanas de gestacin si el
control de bienestar fetal es normal. Por encima de las 37 semanas ante condiciones cervicales favorables valorar la finalizacin de la gestacin. No
existe contraindicacin para el uso de prostaglandinas. Manejo del parto: Se debe realizar monitorizacin continua. Realizar amnioinfusin.
CASO CLINICO POLIDRAMNIOS
Una mujer de 20 aos de edad, gesta 1 para 0, con 28 SDG con un alto
riesgo debido a la presencia de un patrn de crecimiento fetal
discordante y embarazo gemelar. A la exploracin fsica se observo
altura del fondo uterino de 35 cm, palidez de tegumentos, edema de
miembros inferiores, TA 90/70 mmHg, el orificio externo del tero
cerrado, sin secresin vaginal anormal.
PREGUNTA.
Cul es su conducta a seguir?
RESPUESTA
a.- Envio a segundo nivel.
b.- Realizar preparativos para cesaria.
c.- Indica datos de alarma y continua ambulatoria.
d.- Suplementacin energtica y proteica.
CASO CLINICO POLIDRAMNIOS
Femenino de 24 aos de edad, gesta 3, para 2, actualmente con 28
SDG por USG que fue realizado hace 6, la paciente refiere que se
incremento el tamao del abdomen. Su embarazo haba sido sin
incidentes. No hay otras pruebas anormales durante embarazo.
Durante una visita para un chequeo de rutina 6 semana antes, una
ecografa revel amnitico normal, la ecografa al ingreso revel actual
polihidramnios grave, alcanzando un ndice de lquido amnitico de 40
cm. Un examen Doppler del cordn umbilical descubiertos bucles de
cordn mltiples envuelto alrededor del cuello fetal.
PREGUNTA.
Cul es su conducta a seguir?
RESPUESTA
a.- Incrementar el nmero de visitas.
b.- Envio a segundo nivel.
c.- Realizar prueba de grupo y Rh.
d.- Realizar BH, VDRL.
CASO CLINICO
Paciente de 39 aos de edad, secundigesta con una cesrea anterior
por producto macrosomico, con antecedente de diabetes mellitus tipo
II mal controlada, as como hipertensin arterial en control. En la
evaluacin inicial la paciente desconoca su FUM, por lo que el primer
ultrasonido que se realiz demostr un producto de 13 semanas, sin
alteraciones, la paciente se encontraba asintomtica. La paciente
inform de un antecedente de ser tratada por migraa con
carbamazepina desde haca 2 aos el cual lo haba descontinuado al
momento de la evaluacin. La paciente se presenta a urgencias por
presentar disnea leve, edema en miembros inferiores y en pared
abdominal, as como cefalea leve, dolor abdominal y contracciones
uterinas. A la exploracin fsica fondo uterino mayor para edad
gestacional, palpacin subjetiva de partes fetales y dificultad para
auscultar el foco fetal. Se realiz ultrasonido obsttrico el cual revel

CURSO ENARM CMN SIGLO XXI TEL: 36246001

un embarazo 31.2 semanas de gestacin con un ILA de 26.1 cm y


anatoma fetal normal. Los exmenes de laboratorio (glicemia, qumica
y hemograma) demostraron valores normales, su tipo sanguneo y Rh
fue O positivo.
PREGUNTA
De los antecedentes de la paciente. Cul es el menos probable
causante del diagnstico actual de la paciente?
RESPUESTA
a.- Diabetes mellitus tipo II
b.- Macrosomia previa
c.- Hipertensin arterial
d.- Mal control diabtico
PREGUNTA
Cul es la conducta teraputica ms adecuada a seguir en este caso?
RESPUESTA
a.- Indometacina mas inductores de madurez pulmonar
b.- Amniocentesis ms inductores de madurez pulmonar
c.- Diurticos ms inductores de madurez pulmonar
d.- Restriccin de agua y sal ms inductores de madurez pulmonar
PREGUNTA
Qu complicacin materna, es menos probable encontrar en este
caso?
RESPUESTA
a.- Disfuncin uterina
b.- Hemorragia puerperal
c.- Coriamnioitis
d.- Ruptura prematura de membranas
CASO CLINICO OLIGODRAMNIOS
Femenino de 28 aos, gesta 3, para 1, cesarea 1, acude a consulta a las
34 SDG. Presenta un USG realizado a las 24 SDG, con peso fetal
aproximado de (650 g) apropiados para la edad gestacional. Despus
del examen, el paciente no recibi atencin prenatal durante dos
meses. Se realiz nuevamente un USG donde observ restriccin del
crecimiento intrauterino 1,440 g, con bajo nivel de liquido amnitico,
temperatura 36,0 grados, la frecuencia del pulso 84/min, frecuencia
respiratoria 20/min y la presin arterial 120/80 mmHg.
PREGUNTA.
Cul de las siguientes observaciones es falsa respecto a esta
patologa?
RESPUESTA
a.- Constituye un elemento accesorio del feto
b.- Depende del desarrollo, crecimiento y maduracin.
c.- Su volumen vara fisiolgicamente.
d.- El apgar depende del volumen del liquido amnitico.

Pharmed Solutions Institute

PGINA 269

MANUAL DE TRABAJO DEL CURSO ENARM CMN SIGLO XXI


PLACENTA PREVIA (PP)
CIENCIAS BASICAS: Implantacin en el segmento inferior del tero, ocluyendo a veces el orificio cervical interno (OCI). Las hemorragias del embarazo
son causales de importante morbimortalidad. Se las divide en aquellas que afectan la primera mitad y segunda mitad de ste. Entre las ltimas destacan
la placenta previa, DPPNI y rotura uterina. En el puerperio se agregan el acretismo, y la inercia uterina. La hemorragia del post parto por anomalas de la
insercin placentaria es la principal indicacin de histerectoma (HT) obsttrica. SALUD PUBLICA: La hemorragia obsttrica es la causa ms importante de
mortalidad materna en los pases desarrollados. Su incidencia es de 3,8/1.000 embarazos (rango: 1,4 a 7/1.000). Para nulpara, la incidencia es de 0.2%,
mientras que en multparas, puede ser mayor a 5% y la tasa de recidiva es de 4% a 8%. El factor de riesgo ms importante para placenta previa es tener
una cesrea previa. La placenta previa ocurre en el 1% de las embarazadas despus de una cesrea. La incidencia despus de 4 o ms cesreas se
incrementa a 10% Se asocia a riesgo significativo de hemorragia, hospitalizacin, transfusin, parto prematuro, alta frecuencia de cesrea, y de HT post
cesrea. La mortalidad perinatal (MPN) est aumentada 3 a 4 veces, dada principalmente por parto prematuro. CLASIFICACION: Basada en la localizacin
relativa de la placenta en el OCI: 1) COMPLETA O TOTAL: cuando la placenta cubre enteramente el OCI. 2) PARCIAL: cuando la placenta cubre parte pero
no todo el OCI. Es la ms peligrosa. 3) MARGINAL: cuando el borde de la placenta est en contacto con el OCI, pero sin cubrirlo. 4) DE INSERCIN BAJA:
cuando la placenta est localizada cerca (a 3 cm.), pero no est en contacto con el OCI. PATOGENIA: El segmento inferior es una regin inadecuada para
la insercin placentaria, por presentar: a. Endometrio: de menor grosor que determina una decidua ms delgada y con menor vasculatura, por lo que la
placenta tiende a ser ms extendida, aplanada e irregular, con escaso desarrollo de tabiques entre cotiledones. Debido a lo anterior el trofoblasto puede
invadir decidua, pared uterina (ms delgada) y an rganos vecinos (acretismo placentario). La insercin placentaria en el segmento inferior permite que
acte como tumor previo. b. Musculatura: menos fibras musculares en relacin al segmento superior y con mayor cantidad de fibras colgenas, lo que lo
hace distensible, pero con menos potencia para colapsar vasos sanguneos, dificultando la hemostasia si hay desprendimiento parcial de la placenta y por
supuesto, durante el alumbramiento. c. Membranas: en el borde placentario son ms gruesas y menos elsticas, existe mayor frecuencia de RPM. d.
Cordn: Por la atrofia de cotiledones, secundario al desarrollo insuficiente de decidua, es frecuente la insercin velamentosa del cordn. La placenta
previa sangra slo si se desprende. Las causas de desinsercin son: En el embarazo: Por el crecimiento uterino y por las contracciones de Braxton-Hicks,
se forma y desarrolla el Segmento uterino Inferior, cuya capacidad de elongacin es mayor y supera al de la placenta; En el parto: Por las contracciones
del trabajo de Parto que causan la formacin del canal cervico-segmentario y la
dilatacin cervical. DIAGNOSTICO: hemorragia genital indolora, con expulsin de
sangre liquida, roja rutilante, que aparece en forma brusca e inesperada, la
mayora de las veces en reposo e incluso durante el sueo. El 35% de las
pacientes con placenta previa presentan el primer episodio de hemorragia
genital antes de la semana 30, 33% entre las semanas 30 y 35, y 32% despus de
la semana 36. El promedio de edad estacional en el que aparece el primer
episodio de hemorragia es de 29, no se asocia con mortalidad materna y se
resuelve espontneamente. A veces es posible escuchar el soplo placentario
bajo. El sangrado genital est presente en el 80% de los casos; hemorragia +
dinmica uterina en un 10-20% y un 10% son asintomticas, detectadas slo por
ecografa. En el examen fsico, la anemia materna se correlaciona con la cuanta
de la metrorragia. A la palpacin, el tero presenta consistencia normal. Si existe
trabajo de parto, la dinmica es normal. El dolor, es el habitual durante las
contracciones. Como ya dijimos, la Placenta Previa acta en la cavidad uterina
como tumor previo, por lo que las presentaciones altas son frecuentes, al igual
que las presentaciones de tronco, nalgas (30%) y las posteriores (15%). A la
inversa, una presentacin de vrtice, con buen apoyo en la pelvis, nos debe
hacer pensar en otra causa de metrorragia. La fetocardia es normal. Es norma en
todo estudio ultrasonogrfico informar la localizacin de la placenta y su relacin
con el orificio cervical interno (OCI). Actualmente el diagnstico de la mayora de
las PP es realizado mediante ultrasonografa rutinaria del segundo trimestre, por va transvaginal (TV). El USG TV en cualquier edad gestacional debe
considerarse el Gold Standard para diagnosticar PP, ya que la va transabdominal (TA) presenta inconvenientes en la correcta localizacin placentaria,
describindose hasta 60% de relocalizacin por va TV. La va TV tiene una sensibilidad de 87,5%, especificidad de 98,8%, y un valor predictivo positivo
de 93,3%, y valor predictivo negativo de 97,6%. Adems, la va TV ha demostrado ser segura en presencia de metrorragia estable, al no agravar el
sangrado vaginal. TRATAMIENTO: El manejo mdico de la placenta previa sangrante es hospitalario, se debe efectuar evaluacin completa de los
factores etiolgicos que favorecen la presencia del sangrado: infecciones cervicales, infecciones urinarias o vaginales, sobredistensin uterina, factores
precipitantes de la actividad uterina de pretrmino. Los exmenes paraclnicos incluirn: hemograma, hemoclasificacin, gram y cultivo de secrecin
vaginal, parcial de orina (urianlisis), urocultivo, VDRL, ecografa transabdominal y/o transvaginal para confirmar el diagnstico y evaluar la biometra
fetal, as como determinar el peso fetal; en caso de que se considere muy posible la terminacin del embarazo, monitora fetal electrnica bisemanal.
Tratamiento quirurgico o cesarea: La manera de terminar el embarazo esta basado en el juzgamiento clnico sumada la exploracin ecografca. Una
placenta con un borde placentario a menos de 2 cm del orificio cervical externo es probable que requiera una cesrea, especialmente si esta es posterior.
Recomendacin B. Recientemente un estudio prospectivo observacional que incluy 63 pacientes con placenta previa , se demostr que en todas a las
que se le dio parto vaginal , la distancia del borde placentario al orificio cervical interno fue de 2 cm cuando esta era anterior y 3 cm cuando esta era
posterior. Nivel de evidencia IIb.
CASO CLINICO PLACENTA PREVIA
Se trata de femenino la cual cuenta con 36 semanas de gestacion la
cual inicia con trabajo de parto desde hace 6 horas, actualmente
presenta contracciones cada 3 minutos aproximadamente y con
duracin de 50 a 60 segundos, a la exploracin se observa producto
transverso, a la exploracin vaginal se observa 10 % de borramiento y
1-2 centmetros de dilatacin, no se palpan estructuras oseas, sin
embargo se observa sangre fresca y en cantidad moderada, la paciente
cuenta con antecedentes gineco-obstetricos de cesarea previa con
periodo intergenesico de 10 meses.

CURSO ENARM CMN SIGLO XXI TEL: 36246001

PREGUNTA
Considerando el cuadro clnico, cual es la complicacin ms probable
que se presentara en este caso.
RESPUESTA
a.- Desprendimiento de Placenta.
b.- Ruptura Uterina.
c.- Sufrimiento Fetal.
d.- Cesarea por Placenta Previa.

Pharmed Solutions Institute

PGINA 270

MANUAL DE TRABAJO DEL CURSO ENARM CMN SIGLO XXI


DESPRENDIMIENTO PREMATURO DE PLACENTA NORMOINSERTA (DPPNI)
CIENCIAS BASICAS: DPPNI, abruptio placentae o accidente de Baudelocque es el proceso de separacin parcial o total de la placenta de su lugar de
insercin normal que corresponde al fondo uterino, se produce a partir de las 20 semanas de gestacin hasta antes del nacimiento del feto, pudiendo
ocurrir incluso durante el trabajo de parto. SALUD PUBLICA: Aproximadamente 1% de los partos. El 50% ocurre en embarazos con sndrome hipertensivo
del embarazo (SHE). Existe un riesgo de recurrencia que va desde 5.5 hasta 30 veces. La tasa de mortalidad perinatal es entre un 20 y un 35%,
dependiendo de la severidad del cuadro y de la edad gestacional y determinados bsicamente por la prematuridad y la hipoxia. Actualmente la
mortalidad materna es infrecuente, pero la morbilidad es comn y
puede ser severa. Casi todas las complicaciones maternas graves del
DPPNI son consecuencia de la hipovolemia, y de la patologa de base
asociada al DPPNI. CLASIFICACION: Grado 0: asintomtico y
generalmente se puede diagnosticar en periodo postparto. Grado I:
pacientes quienes presentan solo hemorragia vaginal. Grado II:
pacientes que se presenta con hemorragia vaginal, hematoma
retroplacentarios, sensibilidad uterina (a veces sin hipertona), y
signos de sufrimiento fetal. Grado III: paciente que presenta
hemorragia vaginal, hematoma retroplacentario, sensibilidad uterina
(con o sin hipertona), Choque materno, muerte fetal y signos de
coagulopatia de consumo. La evidencia ha mostrado que hay ms
casos de Desprendimiento de placenta Grado III asociado a
Hipertensin arterial en el embarazo que en pacientes que no la
padecen y que adems el resultado perinatal era ms pobre en estas
pacientes. PATOGENIA: La gran mayora es multicausal y plantean la
posibilidad de un mecanismo isqumico a nivel decidual como factor involucrado, asociado con patologas variadas. De estas la ms comn es la
Enfermedad Hipertensiva asociada al Embarazo, con la cual se presentan los casos de DPPNI ms graves (45% de los casos). Otros trastornos descritos:
Antecedente de DPPNI en embarazos previos, RPM, Traumatismo abdominal grave, descompresin uterina brusca (polihidramnios o salida del primer
gemelar), leiomiomas uterinos, consumo de cocana, bajo incremento ponderal materno, tabaquismo. Causas directas: Constituyen entre el 1% y 5% de
todas las causas y especialmente se refieren a: trauma directo (accidentes de trnsito, cadas violentas, etc.), disminucin sbita del volumen uterino
como puede suceder despus de una perdida rpida y abundante de lquido amnitico o el parto del primer gemelo, o un cordn umbilical
anormalmente corto (lo cual puede ocurrir usualmente durante el parto). La causa precisa que conduce al abrupcio de placenta en la mayora de los
casos es desconocida. La formacin del hematoma retroplacentario originado por el DPPNI produce la separacin de la placenta lo que deteriora el
intercambio feto-materno dando lugar a una prdida del bienestar fetal o incluso a la muerte fetal si el desprendimiento es >50%. Tambin la lesin de
los vasos placentarios induce la produccin de sustancias vasoactivas (prostaglandinas y endotelina 1) que pueden originar por si mismas una
disminucin del intercambio gaseoso fetomaterno materno. Las prostaglandinas y la distensin uterina originada por el hematoma, provocan una
hipertona uterina que es tpica del DPPNI. La formacin del cogulo retroplacentario supone una hemorragia materna oculta. Adems, el hematoma
retroplacentario (HRP) y la infiltracin sangunea de la pared muscular estimulan las contracciones colapsando el retorno venoso, y persistiendo el aporte
arterial, con presiones superiores a las del tero, por lo que el hematoma sigue creciendo. Si el cogulo es de gran tamao dar lugar a hipotensin y
shock materno. Adems el secuestro de factores de coagulacin en la formacin del cogulo y el paso de tromboplastina al torrente circulatorio materno
podrn determinar la aparicin de una coagulacin intravascular diseminada. El hematoma retroplacentario es el principal responsable de la clnica y de
las complicaciones maternas y fetales del DPPNI. DIAGNOSTICO: El motivo de consulta es el dolor abdominal, de comienzo brusco, intenso y localizado
en la zona de desprendimiento, que se generaliza a medida que aumenta la dinmica uterina y se expande el HRP. Existe compromiso del estado general,
palidez taquicardia, pero las cifras de presin pueden aparentar normalidad si existe SHE. La hemorragia genital (78%) es rojo oscura, sin cogulos o muy
lbiles; es posterior a la presencia del dolor y decididamente menor que el compromiso del estado general, ya que la sangre proveniente del HRP debe
buscar camino, separando las membranas de la pared uterina para salir al exterior. El sangrado es de inicio sbito y cuanta variable (lo que no guarda
necesariamente relacin con la gravedad del cuadro). Segn su ubicacin, si el HRP aumenta, es posible observar en horas que el tero crece. El dolor
uterino se expresa con reblandecimiento y dolor de espalda en el 66% de los casos. La irritabilidad del tero va progresando: contracciones uterinas,
polisistola e hipertona (20%), contractura, palpndose finalmente un tero de consistencia leosa, tpico de este cuadro. Es difcil palpar al feto y
precisar su presentacin debido a la irritabilidad uterina. La auscultacin muestra sufrimiento fetal (60%) o muerte fetal (15-35%). En el tacto vaginal
dilatacin cervical, la que progresa rpidamente debido a la hiperactividad uterina. Las membranas estn tensas y al romperse, el lquido amnitico
presenta color vinoso, al estar mezclado con sangre y hemoglobina procedente del HRP. Ecogrfico. Su utilidad es limitada, ayudando al diagnstico
diferencial con la placenta previa dado que permite descartarla. Tambin tiene utilidad para el seguimiento de los hematomas retroplacentarios en
pacientes con DPPNI asintomticos en el segundo trimestre inicios del tercero. Un nuevo campo de investigacin es el intento de mejorar la
sensibilidad de este mtodo diagnstico mediante el uso del doppler, para ver la vascularizacin y reas de perfusin placentaria. Pruebas diagnsticas
complementarias. Se han buscado pruebas que nos ayuden en el diagnstico, ninguna de ellas es de utilidad clnica. Signos cardiotocogrficos, un buen
registro no es motivo de tranquilidad, pues se puede producir un rpido deterioro del estado fetal. Niveles de Ca125. Niveles de dmero-D.
Trombomodulina. Es un marcador de dao endotelial. -fetoprotena en suero materno. Se encuentra aumentada. TRATAMIENTO: La interrupcin de la
gestacin se realizar por la va ms rpida sin tener en cuenta la edad gestacional, el tero debe ser evacuado lo ms rpido posible, la decisin de
realizar parto vaginal depender de que las condiciones obsttricas sean ideales para una terminacin rpida y sobre todo cuando existan signos de
coagulopata, se realizar tacto vaginal para estimar las horas que faltan para la terminacin del parto, de acuerdo a las condiciones halladas
(borramiento, dilatacin, altura de la presentacin). Adems con el tacto, si hay trabajo de parto, se descarta la presencia de placenta previa. Se
romper las bolsas de las aguas tan pronto como sea posible aun sin tener en cuenta el mtodo de parto a emplearse, con sta maniobra disminuye la
presin intraamnitica, se reduce la extravasacin sangunea y se abrevia significativamente la duracin del parto. Si ste no hubiera comenzado, se
puede intentar la induccin mediante la infusin intravenosa continua de oxitocina. Ambas conductas, parto espontneo o inducido, estn justificadas
por la gran rapidez con que se produce el parto en estas pacientes. Si en el transcurso del trabajo de parto se constata una alteracin de los latidos
fetales, se interrumpir el trabajo de parto y se indicar la cesrea sin dilacin. La cesrea abdominal se realiza: Si la condicin de la paciente se agrava, si
el feto est vivo, para prevenir su muerte, si fracasa la induccin del parto o si el parto se prolonga, en los casos ms graves, cuando se sospecha una
apopleja uterina, ya que permite no solo evacuar el tero con rapidez, sino tambin decidir si ste podr conservarse o no. La precaucin del obstetra
no termina con la evacuacin del tero, ya que debe controlarse todava la correcta retraccin uterina, debiendo recordar la posibilidad de instalacin de
un cuadro de atona. En los casos severos de apopleja, o si despus del alumbramiento no hubiera respuesta a los ocitcicos y contina la hemorragia

CURSO ENARM CMN SIGLO XXI TEL: 36246001

Pharmed Solutions Institute

PGINA 271

MANUAL DE TRABAJO DEL CURSO ENARM CMN SIGLO XXI


por atona uterina se efectuar inmediatamente la histerectoma. El puerperio inmediato es el momento ms crtico de esta complicacin. Se pondr
especial atencin en: Mantener la volemia y las constantes hematolgicas con soluciones salinas y sangre preferentemente fresca. Se evitaran los
expansores plasmticos, por ser antiagregantes plaquetarios. Control estricto de signos vitales. Control de la diuresis horaria. Observacin rigurosa de los
valores de crasis sangunea. COMPLICACIONES DE LA FORMA GRAVE: Accidente de Couvelaire: apopleja uteroplacentaria; La sangre derramada invade
la pared uterina con zonas de extensas hemorragias miometriales que disocian los haces musculares y puede difundirse hacia el tejido subperitoneal,
trompas, ovarios y a veces ligamentos anchos. A causa de esta infiltracin sangunea el miometrio pierde su propiedad contrctil. CID; Se da en el 30 %
en los DPPN masivos con muerte fetal. Se produce por el paso de tromboplastina a la circulacin materna y activacin de la coagulacin. La baja
concentracin de fibringeno plasmtico depende del atrapamiento de fibrina en el hematoma retroplacentario, esto provoca que la hemorragia sea
incoercible y la sangre no coagule. Insuficiencia Renal Aguda; l colapso circulatorio perifrico y la sobredistencin uterina provocan isquemia en las zonas
corticales de los riones. Shock Hipovolmico; Se produce por la hemorragia brusca y masiva, con o sin exteriorizacin de la sangre por los genitales.
Necrosis de la Hipfisis: (Sndrome de Sheehan). La hipfisis anterior puede sufrir una necrosis total o parcial debido a trombos o espasmos de los vasos
del sistema porta. Se manifiesta en el puerperio por agalactia como sntoma inicial, ya que son las clulas secretoras de prolactina las que primero se ven
afectadas por la isquemia, posteriormente se evidencian los sntomas de esfera gonadal, tiroidea y suprarrenal. Aunque es poco frecuente.
CASO CLINICO DESPRENDIMIENTO DE PLACENTA
Se trata de femenino de 38 aos, con 40 semanas de gestacin la cual
acude a consulta debido a que inicia trabajo de parto, refiere que
desde hace 2 horas inicia con dolor en la regin abdominal baja tipo
contracciones, adems de presencia de sangrado leve pero continuo, a
la exploracin fsica usted palpa contracciones de 2 a 3 en 10 minutos
con una duracin de 40 segundos aproximadamente, al realizar tacto
encuentra borramiento del 40 % y dilatacin de 3 cm, sin embargo
presenta sangrado importante, las constantes vitales de la madre se
encuentra dentro de parmetros normales, no as del producto el cual
observa frecuencia cardiaca aproximadamente de 160 a 180 lpm.
PREGUNTA
Cul de los siguientes factores de riesgo esta altamente relacionado
con desprendimiento de placenta normoinserta?
RESPUESTA
a.- Mayor de 35 aos de edad.
b.- Muerte Materna.
c.- Cesarea.
d.- Miomectomia.
PREGUNTA
Qu grado de DDPI presenta la paciente?
RESPUESTA
a.- Grado I
b.- Grado II
c.- Grado III
d.- Grado 0
CASO CLINICO
Paciente de 37 aos de edad, acude por abdominalgia de 1 da de
evolucin, a las 25,3 semanas de gestacin por una rotura prematura
de membranas (RPM). Las ecografas y analticas practicadas hasta ese
momento eran normales. Como antecedentes obsttricos destacan
una cesrea (incisin transversal) 2 aos antes, a las 28 semanas de
gestacin por riesgo de prdida del bienestar fetal y un aborto
espontneo previo. A la exploracin presenta abdomen ligeramente
doloroso a la palpacin profunda en hipogastrio y en el tacto vaginal se
aprecia cuello uterino acortado un 20% y cerrado. La paciente refiere
prdida de lquido por genitales externos, que se confirma con el
espculo con el que se observa salida de lquido amnitico claro a
travs del crvix. En la ecografa abdominal se observa feto nico con
latido cardaco positivo, biometras de 26 semanas, placenta anterior
normoinserta y lquido amnitico en cantidad normal. Al tratarse de
una RPM pretrmino de 25 semanas, se deriva a nuestro centro previa
administracin de una dosis de betametasona 12 mg intramuscular.
PREGUNTA

CURSO ENARM CMN SIGLO XXI TEL: 36246001

Cul de los siguientes factores de riesgo esta altamente relacionado


con desprendimiento de placenta normoinserta?
RESPUESTA
a.- Mayor de 35 aos de edad.
b.- Ruptura prematura de membranas
c.- Cesarea.
d.- Miomectomia.
CASO CLINICO
Femenino de 38 aos, G5 P2 C1 A1. Embarazo de 33 semanas de
gestacin. Sin antecedentes familiares de importancia, no ha asistido a
controles prenatales. Acude por primera vez al hospital, al servicio de
urgencias, refiere haber estado realizando sus quehaceres domsticos
cuando sin el menor esfuerzo comenz a sangrar de forma rojo
brillante de leve a moderado, no dolorosa, comenta haber presentado
dos episodios similares de sangrado en las ltimas 3 semanas, todos de
forma leve. EF: FC 78x, FR 20x, T/A 120/80. Con buen estado general,
ligera palidez de tegumentos, hidratada. Cardiopulmonar sin
compromiso aparente, abdomen: globoso a expensas de tero
gestante, a la palpacin blando y con leve dolor, se encuentra una FCF
de 135 latidos.
PREGUNTA
Cul es el factores de riesgo que ms probable influyo en el desarrollo
de la placenta previa?
RESPUESTA
a.- Multiparidad
b.- Edad >35%
c.- Antecedente de aborto
d.- Raza negra
PREGUNTA
Cul es la conducta teraputica ms adecuada a seguir en este caso?
RESPUESTA
a.- Hospitalizacin y cesrea corporal
b.- Hospitalizacin, inductores de madurez pulmonar, vigilancia
c.- Manejo ambulatorio y reposo en casa
d.- Hospitalizacion por 48hrs, y alta con reposo en casa
PREGUNTA
Se le toman los siguientes laboratorios a la paciente glucosa 85mg/dl,
Cr 0.9, Hb 9.5mg/dl, leucocitos 9,000, EGO; normal. Cul es la
conducta a seguir en este momento?
RESPUESTA
a.- Evitar tacto vaginal, sulfato ferroso va oral, USG transvaginal
b.- Evitar tacto vaginal, transfusin sangunea, perfil biofsico
c.- Transfusin sangunea, reposo en casa
d.- Perfil biofsico, sulfato ferroso va oral, evitar tacto vaginal

Pharmed Solutions Institute

PGINA 272

MANUAL DE TRABAJO DEL CURSO ENARM CMN SIGLO XXI


ANEMIAS EN EL EMBARAZO
CIENCIAS BASICAS: La anemia es el problema hematolgico ms comn en el embarazo. Es referida como un proceso dilucional secundario al aumento
del volumen plasmtico. Sin embargo existen deficiencias nutricionales, hemlisis y otras enfermedades que pueden causar anemia significativa y ser
capaces de afectar a la madre como al feto. La anemia de acuerdo a la OMS en la mujer embarazada se define como la concentracin de hemoglobina al
nivel del mar menor a 11g/dl y el hematcrito menor que 33 % durante el tercer trimestre de la gestacin. La anemia puede relacionarse con muerte
fetal nacimientos de bajo peso y anormalidades del feto. La anemia sin embargo puede ser un marcador de factores nutricionales, sociales o ambientales
ms que la causa de esos problemas. Cuando la Hb cae a niveles inferiores a 6-7g/dL se pueden tener efectos adversos en la madre y el feto. La anemia
menos severa (8-10g/dL) es de poco riesgo para la madre pero puede poseer mayor riesgo para el infante. Un Hto elevado puede asociarse no slo con
partos prematuros, baja de peso fetal o muerte perinatal sino tambin con hipertensin materna y toxemia. SALUD PUBLICA: Existe una prevalencia de
anemia entre las mujeres embarazadas del 42%.4. En Mxico se encontr anemia en el 21.6% de 500 mujeres embarazadas. Tomando en cuenta mujeres
con embarazo normal y complicado se encontr la prevalencia del 22.4%. Ciertos grupos de mujeres estn en mayor
riesgo para desarrollar anemia durante el embarazo, probablemente en parte a factores familiares, sociales,
econmicos, nutricionales y a falta de cuidado prenatal. CLASIFICACION: Durante la gestacin, creemos til tener en
cuenta que las anemias que acompaan al embarazo pueden ser agrupadas en 2 categoras: 1. Directamente
relacionadas con la gestacin: a) Ferropnicas, b) Megaloblsticas, c) Hipoplsicas. 2. Que no guardan relacin directa
con la gestacin: a) Anemias por hemates falciformes. b) Otras anemias hemolticas y raras. ANEMIA FERROPENICA: En pases en desarrollo el 83-95%
de las mujeres embarazadas anmicas tienen deficiencia de hierro. El embarazo y el parto representan una prdida de1 a 1,3 g de hierro, que se extrae
fundamentalmente de los depsitos de hierro en el sistema reticuloendotelial y en el parnquima heptico, en forma de hemosiderina o ferritina. Con
frecuencia, las embarazadas enfrentan estas necesidades con las reservas de hierro exhaustas. Entre los factores que llevan a ello se encuentran:
menstruaciones abundantes, embarazos con escaso periodo intergenesico, dietas con bajo contenido en hierro, embarazos anteriores sin un adecuado
suplemento frrico, partos con sangramientos durante el alumbramiento o el puerperio, parasitismo intestinal, baja absorcin del hierro y otros. Estas
anemias son pobres en signos y por lo general, son asintomticas; puede observarse palidez cutaneomucosa y cierta tendencia a la fatiga. Las formas
ms severas presentan un sndrome anmico dado por: laxitud, "cansancio de muerte, irritabilidad, astenia, nerviosismo, cefalea, anorexia y otros. En
los casos de anemias muy severas alteraciones del apetito, pirosis, ardor lingual y bucal, flatulencia, constipacin y es posible la aparicin de glosistis. En
ocasiones puede haber manifestaciones de insuficiencia cardiacas y cardiomegalia. A veces las pacientes pueden tener dolores de tipo neurlgico,
adormecimiento de las extremidades, sensacin de hormigueo, trastornos vasomotores y otros. Al realizar el examen fsico, se detecta palidez
cutaneomucosa; las uas de las manos, y a veces las de los pies, aparecen opacas y sin brillo, y se rompen con facilidad. Con frecuencia, la auscultacin
permite escuchar soplos anmicos funcionales. Diagnstico: durante la atencin prenatal el estudio sistematico de la Hb y Hto que deben hacerse cada
6-12 semans, permitir el diagnostico precoz de anemia. Si la hemoglobina est por debajo de 110 g/L, se considera que hay anemia.
Independientemente de la clasificacin dada por la OMS (1991), creemos que desde el punto de vista prctico conviene considerar 3 grados: ver cuadro
anexo. Las anemias ferriprivas se caracterizan por tener: 1. Hemoglobina y hematocrito disminuidos. 2. Hierro srico disminuido. 3. Discapacidad total
elevada. 4. ndice de saturacin disminuido. 5. Protoporfirina eritrocitaria elevada. 6. Lmina perifrica normoctica hipocrmica. Tratamiento
profilctico: administracin de hierro VO desde la primera consulta prenatal. Ingestin diaria de 60mg de hierro elemental, en pacientes con feto nico.
Debe administrarse en forma de sales ferrosas: 1. Sulfato ferroso: tableta de 300 mg = 60 mg de Fe elemental. 2. Gluconato ferroso: tableta de 300 mg =
36 mg de Fe elemental. 3. Fumarato ferroso: tableta de 200 mg = 65 mg de Fe elemental. La administracin de hierro debe proveer al organismo la
cantidad suficiente de este elemento para lograr la regeneracin de la hemoglobina y para la reserva. Esto se consigue administrando 2 a 3 tabletas
diarias de sales ferrosas durante el embarazo y hasta 6 meses despus del parto. Tratamiento curativo: La VO es de eleccin siempre que sea posible en
dosis de 600 a 1 200 mg/da, que equivaldran a 120 o 180mg, preescrito en 1-2 tabletas media hora antes de desayuno, almuerzo y comida, ya que es
preferible separarlo de los alimentos. Puede indicarse adems la administracin de 100mg de cido ascrbico diariamente. La administracin durante las
comidas presenta mejor tolerancia, aunque es menor su absorcin. Las sales de hierro no deben acompaarse de leche, t, caf o huevo, ya que stos
interfieren en su absorcin. El tratamiento debe ser sostenido por lo menos de 2 meses despus de normalizado el hematocrito y la hemoglobina. Una
forma prctica de calcular la dosis total de hierro en miligramos sera: Hemoglobina normal-hemoglobina de la paciente 255 mg de Fe. La va IM sera
necesaria en las circunstancias siguientes: 1. Intolerancia gstrica al hierro oral. 2. Cuando este contraindicada su administracin como en los casos de
gastritis, lcera, diverticulosis y otras afecciones digestivas. 3 Falta de respuesta al tratamiento oral. 4. Sndrome de malabsorcin intestinal. 5. Anemia
intensa (85 g/L o menos) despus de las 34 semanas. ANEMIA MEGALOBLASTICA: En el embarazo, existe un aumento de necesidades de cido flico y
Vit., B12 para la sntesis del ADN y del ARN, debido al rpido crecimiento celular del embrin y del feto en desarrollo. La anemia megaloblatica del
embarazo es causada por deficiencia de cido flico y vit., B12. La gestante tambin puede sufrir una deficiencia de cido ascrbico, que se asocia con la
de cido flico. En la lmina perifrica se encuentra macrocitosis, punteado basfilo (policromatofilia), leucocitos de Pitaluga y macroplaquetas. La
determinacin de cido flico ofrece cifras por debajo de 4g/l. la medula osea es megaloblastica. La deficiencia de cido flico y de cido ascrbico
determinan un aumento de las complicaciones infecciosas de madre, abortos, partos prematuros, RPM y otros. Tratamiento: profilctico 1. cido flico:
1 mg/da (tabletas). 2. cido ascrbico: 200 mg/da (tabletas). Tratamiento especfico: Se administra ac. flico 5-10mg/da. Adems del cido flico debe
administrarse hierro en dosis teraputica, ya que la transformacin de la mdula sea megaloblstica en normal, requiere gran cantidad de hierro.
Antes de iniciar el tratamiento debe buscarse la existencia de una infeccin que condicione la anemia (la infeccin urinarias es la mas frecuente) y puede
hacerla refractaria a tratamiento. Una caracterstica de esta anemia es su remisin espontnea despus del parto. ANEMIA HIPOPLSICA: Se le relaciona
con el embarazo, y se considera por algunos como una manifestacin de toxemia. Es rara y de gravedad variable. Puede tener remisiones parciales o
completas y en algunas ocasiones, desaparecer espontneamente despus del parto. Puede provocar muerte fetal y parto pretrmino. Diagnstico: la
anemia es de desarrollo rpido, con palidez, fatiga y taquicardia. Las manifestaciones clnicas dependen de los grados de la anemia, la granulocitopenia y
la trombocitopenia. Puede haber formas globales, con cada de los 3 sistemas o formas parciales con la afeccin de 1 solo de ellos. Laboratorio:
fundamentan el diagnstico las determinaciones de: 1. Hemoglobina (muy baja). 2. Hematocrito (reducido). 3. Trombocitopenia. 4. Hierro srico
(elevado). 5. Mdula sea hipocelular con depresin selectiva o de los 3 sistemas (pancitopenia). Tratamiento: Para tratar la anemia hipoplsica se
dispone de recursos muy limitados. El tratamiento con hierro, cido flico y vitamina B12 ha resultado ineficaz. En el aspecto mdico, se aconseja: una
serie de medidas para prolongar la vida de la paciente, como son: 1. Transfusiones de concentrados de glbulos rojos, si la anemia fuera lo fundamental.
2. Transfusin de plaquetas. 3. Administracin de antibiticos (no profilcticos y con antibiograma). 4. Administracin de anablicos: nerobol: 1 a 3
mg/kg por va oral, diariamente. ANEMIA POR HEMATES FALCIFORMES O DREPANOCITEMIA: de acuerdo con los sntomas clnicos de la Hb.S parece
trasmitirse por medio autosmicos recesivo (solo estn afectados los hemocigotos SS), se considera ms bien el resultado de herencia autosmica
codominante, puesto que el genotipo heterocigoto ATS (rasgo de clulas falciformes), puede bajo ciertas circunstancias, producir morbilidad grave e
incluso la muerte. En los individuos SS, la hemlisis es consecuencia del secuestro y de la destruccin de los hemates falciformes en el sistema
reticuloendotelial, as como su destruccin intravascular originada por trauma mecnico; de ah que la anemia crnica sea la regla. La hemoglonina S y la

CURSO ENARM CMN SIGLO XXI TEL: 36246001

Pharmed Solutions Institute

PGINA 273

MANUAL DE TRABAJO DEL CURSO ENARM CMN SIGLO XXI


hemoglobina C son resulta do de la sustitucin del cido glutmico en la posicin 6 de la cadena de la globina, por valina y lisina respectivamente. En la
crisis drepanoctica se produce el fenmeno de deformacin semilunar de los hemates y puede hacerse irreversible. El feneomeno de falciformes es
debido a que la propiedad fundamental que diferencia a la hemoglobina S de la hemoglonina A es la baja solubilidad de la primera en su forma
desoxigenda, lo cual provoca su precipitacin debido a la formacin de polimeros, agergados constituidos por la formacin de varias molculas de
hemoglobina S, que forman un gel semislido que se extiende a lo largo de los hemates y da lugar a la defiormacion caracterstica.
CASO CLINICO ANEMIAS
Femenino de 16 aos de edad originaria de Tabasco, la cual acude por
cansancio, fatiga, adinamia, actualmente cursa con embarazo de 28
semanas de gestacion, en control de rutina muestra Hematocrito de 33
y hemoglobina de 10, desde la semana 20 fue diagnosticada con
anemia, fue suplementada con hierro, sin embargo comparado con el
estudio anterior no hay mejora, actualmente se observa edema de
miembros plvicos ++,
PREGUNTA
Considerando los datos de laboratorio, cual es su conducta a seguir
para resolver el presente caso?
RESPUESTA
a.- Transfusin Sangunea.
b.- Complejo B
c.- Hierro.
d.- Realizar un estudio de tincin.
PREGUNTA
Cul es el factor menos probable que haya influido en este cuadro?
RESPUESTA
a.- Menstruaciones abundantes
b- Embarazos con escaso periodo intergenesico
c.- Parasitismo intestinal
d.- Perdidas hepticas reiteradas
PREGUNTA
En que semana del embarazo es mas probable que el volumen
plasmtico, alcanze el mximo?
RESPUESTA
a.- Semana 10
b.- Semana 20
c.- Semana 30
d.- Semana 38
PREGUNTA
Qu complicacin es menos probable de observar de continuar con
estas cifras en la biometra hemtica?
RESPUESTA
a.- Hipoxia fetal
b.- Retardo en el crecimiento
c.- Afecciones cardiacas
d.- Infecciones de vas urinarias
CASO CLINICO
Mujer de 24 aos, AGO: menarqua a los 12 aos; ritmo menstrual de
28x3-4 das, en ocasiones con cogulos; inicia vida sexual activa a los
22 aos y cursa el 6 mes de su primer embarazo de evolucin normal
aunque tuvo emesis gravdica leve durante el primer mes pero no
acude a control gineco-obsttrico hasta el sexto mes, das antes de la
consulta hematolgica refiriendo entonces astenia, somnolencia y se le
observa plida. En la consulta de hematologa se confirman tales datos
y adems menciona pica y pagofagia de varios meses o incluso un
ao de evolucin. Signos vitales normales a la exploracin, observando
pelo frgil y platoniquia, se tiene la impresin de palpar el bazo al
inspirar profundo. Citologa hemtica: Hemoglobina (Hb): 10.5 g/dL;
Hematocrito:
0.32/l;
Eritrocitos:
5.000.000/mm3;
Volumen
eritrocitario medio: 70 fl; Hemoglobina corpuscular media: 26pg;

CURSO ENARM CMN SIGLO XXI TEL: 36246001

Concentracin de hemoglobina corpuscular media: 30g/dL; Leucocitos


y frmula diferencial normales; Plaquetas: 480.000/mm3
PREGUNTA
Cual es el diagnostico mas probable?
RESPUESTA
a.- Anemia normoctica normocrmica
b.- Anemia megaloblstica normocrmica
c.- Anemia microctica normocrmica
d.- Anemia microctica hipocrmica
PREGUNTA
De los siguientes datos de laboratorio, Cul es el menos til para
orientarnos sobre agotamientop de las reservas de hierro?
RESPUESTA
a.- Ferritina srica <12mg/dl
b.- Saturacion de transferrina <16%
d.- Numero de sideroblastos <10%
c.- Hemoglobina <12mg/dl
PREGUNTA
Cul es la conducnta teraputica mas adecuada a seguir en este caso?
RESPUESTA
a.- 60mg de Hierro y 500mg de ac. Flico al dia
b.- Transfusion 274sangunea
c.- Hidratacin intravenosa
d.- Complejo B
CASO CLINICO
Paciente de 25 aos de edad que acude a su mdico de cabecera para
confirmar una gestacin. Acabada la exploracin fsica, el mdico
decide practicarle una prueba de embarazo y anlisis general,
hemograma completo incluido. En el dictamen de laboratorio se
constata la existencia de una anomala hematolgica, sin especificar
ms, motivo por el cual el mdico prefiere remitir a la paciente
directamente a la consulta externa del hospital del rea. En la primera
visita la paciente trae el informe de los anlisis practicados, en el que
destaca, adems de la positividad de la prueba de embarazo, la
presencia de una discreta anemia microctica e hipocroma con
bioqumica normal, excepto por la ferritina srica, en el lmite inferior
de los valores de referencia. La exploracin fsica es normal, salvo por
una muy discreta palidez de piel y mucosas. No hay visceromegalias ni
adenopatas. El mdico que la atiende decide practicarle una serie
analtica ms completa, cuyos resultados confirman la sospecha
diagnstica: hemates, 5.360 1.012/l; hemoglobina, 119 g/l;
hematcrito, 0,37 L/L; VCM, 69 fl; HCM, 22,1 pg; concentracin de
HCM, 321 g/l; amplitud de distribucin eritrocitaria, 15%; plaquetas,
178 109/l;
PREGUNTA
Cul es el diagnostico mas probable en este caso?
RESPUESTA
a.- Talasemia
b.- Anemia ferropnica
c.- Anemia megaloblastica
d.- Anemia hipoplasica

Pharmed Solutions Institute

PGINA 274

MANUAL DE TRABAJO DEL CURSO ENARM CMN SIGLO XXI


INFECCIONES DE VIAS URINARIAS (IVU)
CIENCIAS BASICAS: La infeccin urinaria (IVU) es una entidad clnica que se asocia frecuentemente al embarazo; las variantes clnicas son la bacteriuria
asintomtica, la cistouretritis y la pielonefritis. Durante la gestacin se producen una serie de cambios fisiolgicos que aumentan el riesgo de presentar
infecciones del tracto urinario: Dilatacin ureteral secundario a la accin de progesterona y a la compresin uterina, reflujo vesico-ureteral, estasis
vesical, aumento del filtrado glomerular con glucosuria y amnioaciduria con elevacin del pH urinario. SALUD PUBLICA: La infeccin urinaria es una de las
complicaciones mdicas ms frecuentes del embarazo, nicamente superada por la anemia y la cervicovaginitis; si no es diagnosticada y adecuadamente
tratada, puede llevar a un incremento significativo en la morbilidad en la madre y el feto. El 20% de las embarazadas presentan una infeccin vas
urinarias bajas (ITU) en el curso de la gestacin. Un 10% de los ingresos hospitalarios en gestantes se deben a IVU. La bacteriria asintomtica no tratada
es un factor de riesgo de pielonefritis, bajo peso al nacer y amenaza de parto prematuro. CLASIFICACION: La infeccin en el embarazo es clasificada
segn el sitio de proliferacin bacteriana, de la siguiente manera: Bacteriuria asintomtica=Infeccin urinaria baja. Cistitis, Pielonefritis=Infeccin urinaria
alta. PATOGENIA: Constituyen factores de riesgo de desarrollar una infeccin urinaria en la gestacin los siguientes factores: 1. Bacteriria asintomtica
2. Historia de ITU de repeticin 3. Litiasis renal 4. Malformaciones uroginecolgicas 5. Enfermedades neurolgicas (vaciado incompleto, vejiga
neurgena) 6. Reflujo vesico-ureteral 7. Insuficiencia renal. 8. Diabetes mellitus 9. Infeccin por Chlamydia trachomatis 10. Multiparidad 11. Nivel
socioeconmico bajo. Entre los diferentes agentes etiolgicos que se conocen como causantes de las IVU, la Escherichia coli, procedente de la flora
enterobacteriana es el microorganismo ms habitual y causante de la mayor parte de estas infecciones (80-90% casos). Le siguen por orden de
importancia: Proteus mirabilis, Kelbsiella pneumoniae, Enterococcus spp y Staphylococcus saprophyticus. A mayor edad gestacional, mayor probabilidad
de Gram positivos principalmente SGB. BACTERIRIA ASINTOMTICA: Se denomina bacteriuria asintomtica (BA), a la presencia de bacterias en orina
cultivada (ms de 100.000 colonias por ml) sin que existan sntomas clnicos de infeccin del tracto urinario. La mayora de bacteriurias asintomticas se
dan en el primer trimestre de la gestacin. Aparece en un 5-6% de los casos. El 25% de las bacteririas asintomticas no tratadas desarrollaran una
pielonefritis aguda vs el 3% de las tratadas. Hasta un 30% de las bacteririas asintomitcas tratadas presentan una recada a pesar del correcto
tratamiento antibitico. Diagnstico: es microbiolgico: cultivo orina 100.000 unidades formadoras de colonias (UFC). El diagnstico se establece
mediante el urocultivo cuantitativo. La muestra de orina debe ser obtenida bajo determinadas condiciones de asepsia: despus de limpieza de genitales
externos y recogida del chorro medio de la miccin. NO es necesario para realizar un urocultivo el sondaje vesical. Si el resultado del urinocultivo es de
orina contaminada, debe repetirse, haciendo hincapi en las medidas de asepsia para una correcta toma de la muestra. Tratamiento: Normalmente se
dispone de antibiograma. Utilizar el antibitico de espectro ms reducido: empezar por amoxicilina, despus amoxicilina-clavulnico....etc. En caso de NO
disponer de antibiograma se propondr como tratamiento emprico de la bacteriria asintomtica: Amoxicilina-clavulnico 500 mg/8 h vo x 7 dias. En
caso de alergia a betalactmicos: Fosfomicina trometamol 3 g vo (unidosis o pauta corta dos dosis separadas por 3 das). Otras alternativas teraputicas
de mayor espectro que deben reservarse para casos de resistencias sn: Cefuroxima 250 mg/12 h vo x 7 das Cefixima 400 mg/24 h vo x 7 das En
caso de alergia a betalactmicos: Nitrofurantona 50-100 mg/6 h vo x 7 das. Seguimiento: Comprobar curacin con urinocultivo a los 15 das despus de
tratamiento. En caso de recidiva, actuar segn antibiograma y si no se dispone de l, ampliar el espectro. Repetir urinocultivo cada trimestre de
gestacin. CISTITIS AGUDA: Es un sndrome caracterizado por una clnica de urgencia miccional, disuria y dolor suprapbico en ausencia de sntomas de
afectacin sistmica (fiebre) y dolor lumbar. Aparece en un 1.3% de las gestaciones. La mayora de cistitis agudas se presentan en el segundo trimestre
del embarazo. Diagnstico: clnica sugestiva y cultivo orina positiva (100.000 UFC). Puede aparecer hematuria macro/microscpica. Recordar normas de
recogida de muestra de orina para urinocultivo: NO requiere necesariamente de sondaje vesical, pero debe recogerse en determinadas condiciones de
asepsia: despus de limpieza de genitales externos y recogida del chorro medio de la miccin. Tratamiento: idealmente segn antibiograma especfico,
pero puede tratarse empricamente si la clnica es muy sugestiva y no se dispone de resultado de urinocultivo. El tratamiento suele ser emprico porque
no solemos disponer de antibiograma Cefuroxima 250 mg/12 h vo x 7 das. Si alergia a betalactmicos: Fosfomicina trometamol 3 g vo (unidosis o pauta
corta dos dosis separadas por 3 das). Si disponemos de antibiograma, se debe seguir la misma estrategia teraputica que en el caso de bacteriria
asintomtica y empezar con el antibitico de menor espectro: Amoxicilina 500 mg/8 h vo x 7 das. Amoxicilina-clavulnico 500 mg/8 h vo x 7 dias.
Cefixima 400 mg/24 h vo x 7 daS. En caso de alergia a betalactmicos: Nitrofurantona 50-100 mg/6 h vo x 7 das. Comprobar curacin con urinocultivo a
los 15 das despus de tratamiento. En caso de recidiva, actuar segn antibiograma y si no se dispone de l, ampliar el espectro. Repetir urinocultivo cada
trimestre de gestacin. PIELONEFRITIS AGUDA: Infeccin del parnquima renal que aparece en 1-2% de las gestaciones y cuya prevalencia aumenta al
6% en gestantes en las que no se ha realizado cribado de bacteriria asintomtica durante el embarazo. El 80-90% de las pielonefritis aparecen en el 2,
3 trimestre de la gestacin y en el puerperio. La va de entrada ms frecuente es la ascendente a travs de la va urinaria aunque en pacientes
inmunodeprimidos la va de diseminacin puede ser la hematgena (principalmente en diabticos, tratamiento con corticoides, enfermedades
sistmicas...). La localizacin ms frecuente es la afectacin renal derecha (50% de los casos); en un 25% de casos la localizacin es izquierda y en un
25%, bilateral. Si la paciente tuvo una bacteriria asintomtica, es posible que el germen causante de la pielonefritis sea el mismo. Clionica; Dolor
costovertebral con puopercusin lumbar positiva, fiebre, nuseas y vmitos, en general, NO cursa con clnica de cistitis aguda. Tratamiento
ambulatorio: Si NO cumple criterios de hospitalizacin, es posible el control ambulatorio en dispensario de curas de enfermera (CENF) cada 24h: De
eleccin ceftriaxona 1 g /24 h ev o im, en caso de alergia a betalactmicos, gentamicina 240 mg/24h ev o 80 mg/8h im. Si mejora clnica del cuadro
febril, a las 48-72 h podr pasarse a terapia oral. Si se dispone de antibiograma, pasar el antibitico de menor espectro: amoxicilina, despus amoxicilinaac. Clavulanico. Si NO se dispone de antibiograma, iniciar tratamiento emprico con cefuroxima 250mg/12 h vo hasta cumplir 14 das de tratamiento.
Slo se utilizar como antibitico alternativo la cefixima 400 mg /24 h va oral x 14 das en caso de sospecha de resistencias a la cefuroxima (por
antibiogramas de urinocultivos anteriores). En caso de persistencia de sndrome febril y no respuesta clnica tras 72h (3 das) de tratamiento mdico
ambulatorio, indicar ingreso hospitalario. Tratamiento hospitalario: En general el tratamiento de la pielonefritis ser ambulatorio a excepcin de aquellas
pacientes que cumplan algunos de los criterios de ingreso hospitalario: Criterios de ingreso hospitalario fiebre 38C, sepsis, deshidratacin, edad
gestacional 24 semanas, cuadro clnico de amenaza de parto prematuro, pielonefritis recurrente, comorbilidad, intolerancia oral, fracaso de
tratamiento ambulatorio tras 72 h (3 das), no posibilidad de tratamiento ambulatorio. 1.- Hidratacin agresiva: Primer da: Sueroterapia a una perfusin
de 150 ml/h. Segundo da perfusin de100 ml/h. 2.- Tratamiento parenteral antibitico hasta 48 horas afebril: En gestante: Primera eleccin: 1.Ceftriaxona 1 g/24 h IV Alergia a betalactmicos: 2.- Gentamicina 80 mg/8 h IV. En purpera: Primera eleccin: 1.- Ceftriaxona 1 g/24 h ev + ampicilina 1
g/6 h ev (se debe cubrir tambin el Enterococo faecalis). Alergia a betalactmicos: 2.- Gentamicina 240 mg/24 h IV. En casos de sepsis y/o pielonefritis
con sospecha de grmenes multiresistentes o pacientes con sondajes o vas con tiempo prolongado: El tratamiento antibitico deber cubrir
Pseudomona auroginosa u otros grmenes multiresistentes: 1.- Ceftacidima 1 g/8 h IV. Alergia a betalactmicos: 2.- Amikacina 15 mg/kg/24h IV. En el
caso de sospecha de infeccin por Enterococco faecalis aadir ampicilina 1 g/6 h IV o iniciaremos tratamiento en monoterapia con piperacilinatazobactam 4g/8 h IV.
CASO CLINICO INFECCIONES DURANTE EL EMBARAZO

CURSO ENARM CMN SIGLO XXI TEL: 36246001

Mujer de 25 aos de edad con 34 semana de gestacin. Antecedentes


de aborto espontneo cuatro aos antes y un parto haca dos aos.

Pharmed Solutions Institute

PGINA 275

MANUAL DE TRABAJO DEL CURSO ENARM CMN SIGLO XXI


Haca tres das inicia con disuria, prurito, urgencia urinaria, dolor
suprapubico, con descarga vaginal grumosa.
PREGUNTA
Cual es el tratamiento antibitico mas adecuado para este caso?
RESPUESTA
a.- Ampicilina.
b.- Cefalexina.
c.- Nitrofurantoina.
d.- Trimetropin con sulfametoxazol.
CASO CLINICO INFECCION DURANTE EL EMBARAZO
Paciente de 17 aos, nulpara, con antecedentes mrbidos de
bacteriuria asintomtica con tres episodios de cervicovaginitis. Ingres
a las 31 semanas de gestacin por trabajo de parto prematuro. Al
ingreso se le practic una amniocentesis obtenindose lquido
amnitico con meconio y estudio citoqumico sugerente de infeccin
intraamnitica. Se tom cultivo del lquido amnitico y se inici
tratamiento. El trabajo de parto fue acelerado con oxitocina,
resolvindose por va vaginal.
PREGUNTA
Cual de las siguientes complicaciones maternas fetales se asocia ms
frecuentemente?
RESPUESTA
a.- Corioamnionitis.
b.- Pielonefritis.
c.- Sepsis puerperal.
d.- Urosepsis.
CASO CLINICO
Paciente de 17 aos primigestante con embarazo de 36 semanas por
fum cierta y confiable y 38 semanas por ecografia de segundo
trimestre. Quien refiere cuadro clinico de 1 mes de evolucion
caracterizado por edema de miembros inferiores, niega cefalea, niega
acufenos, niega fosfenos, refiere dolor en region lumbar de moderada
intensidad no irradiado y disuria, refiere movimientos fetales
presentes. Niega genitorragia, niega hidrorrea.
PREGUNTA
Cual es la conducta a seguir mas adecuada?
RESPUESTA
a.- Ceftriaxona.
b.- Gentamicina
c.- Amoxicilina.
d.- Cefuroxima.
CASO CLINICO
Paciente de 26 aos de edad, acude a urgencias, cursa con embarazo
de 31 semanas por amenorrea; refiere dolor plvico y lumbar de 2 das
de evolucin, acompaado de disuria, hematuria al final de la miccin,
tenesmo vesical, ha presentado fiebre no cuantificada. Refiere haber
acudido a Mdico particular quien indic manejo con Pirifur, con
mejora parcial. El da de hoy se agrega, dolor obsttrico, sensacin de
endurecimiento abdominal, salida de moco. AHF: madre con DM2 e
insuficiencia renal. AGO; Menarca a los 11 aos, Ritmo 30x5, IVSA 17
aos, GIII AI PI (Aborto de 10-11 semanas por causa desconocida que
amerito LUI), DOC Hace 4 aos Negativo a Cncer. Refiere 4 consultas
en su UMF para control prenatal donde se le realiz exmenes de
sangre y orina que le reporto su Mdico familiar dentro de parmetros
normales. A la exploracin fsica se encuentra con T.A. 100/70mmHg,
FC 91xmin, FR 22xmin, Temp 37.7C. Peso 68kg Talla 1.56m. Consiente,
tranquila, ligero rubor facial, buen estado de hidratacin,
cardiopulmonar sin compromiso, abdomen globoso con FU de 28cm,

CURSO ENARM CMN SIGLO XXI TEL: 36246001

con producto nico, con FCF de 157x, ceflico, longitudinal. Al tacto


vaginal con crvix posterior, semiblando, con 80% de borramiento, 12cm de dilatacin, membranas integras, presentacin rechazable, no
salida de lquido, leucorrea blanquecina no ftida en cantidad
moderada. Extremidades simtricas con datos de insuficiencia venosa
perifrica GI, edema bimaleolar.
PREGUNTA
Cul es la conducta teraputica ms adecuada para esta paciente?
RESPUESTA
a.- Nitrofurantoina tab
b.- Ampicilina ms gentamicina
c.- Terbutalina
d- Indometacina
PREGUNTA
Conque es menos probable que se asocie la bacteriuria asintomtica
sin tratamiento en el embarazo?
RESPUESTA
a.- Ruptura prematura de membranas
b.- Parto pretermino
c.- Corioamnioitis
d.- Productos con bajo peso al nacer
PREGUNTA
En qu semana est indicado solicitar el urocultivo de tamizaje
durante el embarazo?
RESPUESTA
a.- 8-10SDG
b.- 10-12SDG
c.- 12-16SDG
d.- 16-28SDG
CASO CLINICO
Mujer de 27 aos de edad, G4, P1, con 30.6 semanas de gestacin,
acude a urgencias por cefalalgia, escalosfrios y disuria que ahan
empeorado durante los dos das anteriores. Hace dos das la
temperatura era de 37.8C, pero ha aumentado, la ultima vez tuvo
38.8C. Niega visin norrosa, dolor abdominal o mareo. No esta segura
de haber tenido secrecin vaginal reciente, aunque advirti manchado
leve e intermitente durante las emanas anteriores. Ademas refiere que
desde inicio del segundo trimestre, ha tenido que orinar con
frecuencia de modo que no cree que el empeoramiento reciente haya
sido anormal. EF: T 38.4C, FC 108 lpm, TA 155/95mmHg. El abdomen
es blando sin hipersensibilidad suprapbica. Refiere dolor leve en
ambos angulos costovertebrales. Laboratorio: Hb 11.8mg/dl,
Leucocitos 157,000, plaquetas 255000, Na 140mEq/l, K 3.7mEq/l, Cr.
0.7mg/dl.
PREGUNTA
Cul es el diagnostico mas probable en este caso?
RESPUESTA
a.- Cistitis
b.- Pielonefritis
c.- Uretritis
d.- Infeccion cervicovaginal
PREGUNTA
Cul seria la prueba de mas utilidad en el diagnostico?
RESPUESTA
a.- Hemocultivo y EGO
b.- Exudado cervico-vaginal
c.- Ultrasonido
d.- Urografia excretora

Pharmed Solutions Institute

PGINA 276

MANUAL DE TRABAJO DEL CURSO ENARM CMN SIGLO XXI


PARTO PRETERMINO
CIENCIAS BASICAS: El diagnstico de parto pretrmino se obtiene en pacientes con 37 semanas (o un peso igual a mayor de 500g, que respira o
manifiesta signos de vida) y que presentan contracciones dolorosas y regulares que ocurren al menos cada 10 minutos. Esto debe estar asociado con
dilatacin cervical y/o descenso. El diagnstico se realiza al demostrar una dilatacin progresiva del crvix debido a la presencia de contracciones entre
las semanas 20-37, lo que incluye hasta la semana 37. La definicin se puede dividir a saber: Pretrmino: 33-37 semanas. Pretrmino moderado: 28-32
semanas. Pretrmino extremo: 20-27 semanas. SALUD PUBLICA: El parto pretrmino es la mayor causa de mortalidad perinatal en el mundo
desarrollado y ocurre en aproximadamente un 7% de todos los nacimientos. Los nios pretrmino presentan causas severas de morbilidad como
dificultad respiratoria del recin nacido, hemorragias intraventriculares, displasia broncopulmonar y enterocolitis necrotizante. Tambin se pueden
presentar complicaciones a largo plazo como parlisis cerebral, perdida visual y auditiva. Cerca de un 30% de los parto pretrmino son idiopticos y
espontneos. PATOGENIA: La incidencia del parto pretrmino es generalmente ms alta en los grupos socioeconmicos bajos en comparacin con los
grupos de mayor ingreso econmico. De manera que se han establecido relaciones entre la incidencia de amenaza de parto, parto pretrmino y algunas
condiciones asociadas con los problemas socioeconmicos, a saber: Bajo cumplimiento de la consulta prenatal, edades maternas extremas (adolescentes
y madre aosa), fumadoras, condiciones laborales extremas, pobre educacin materna, estrs en la vida diaria, ansiedad relacionada al embarazo,
violencia domstica. Ante la presencia de un parto pretermino se debe buscar de forma intencionada la presencia de infecciones a travs de urocultivo y
cultivo de secrecin vaginal con el objetivo de especificar el uso de antibitico. Es necesario corroborar por USG la edad gestacional. Para establecer el
pronstico de sobrevida del neonato y sus posibles complicaciones. El factor de riesgo ms importante para predecir parto pretermino es el antecedente
de parto pretermino. Las pacientes con antecedente de parto pretermino, deben recibir consejo preconcepcional o atencin medica temprana en el
embarazo. DIAGNOSTICO: No hay datos clnicos especficos y la exploracin de crvix es subjetiva e inexacta. Identifique los factores de riesgo de parto
pretrmino en la historia clnica de las mujeres en control. Algunos sntomas tempranos sugestivos de amenaza de parto pretrmino son: Dolor
abdominal bajo y/o calambres, dolor lumbar, presin plvica, flujo vaginal aumentado, manchado o sangrado. Sntomas definitivos son: actividad uterina
regular acompaada de borramiento y dilatacin del crvix. Criterios diagnsticos: A) actividad uterina regular entre las 20 y 37 semanas de gestacin
ms contracciones uterinas clnicamente documentadas (1/10 minutos, 4/20 minutos o 6/60 minutos). B) Dilatacin cervical igual o mayor a 2cm. C)
Borramiento cervical igual o mayor a 80%. Laboratorio y gabinete: la evaluacin ultrasonografica de la longitud cervical y prueba de fibronectina fetal
tienen elevado valor predictivo negativo; por lo tanto, el uso nico o de ambas, permite determinar qu tipo de pacientes no requieren tocolisis. En
pacientes con riesgo de parto pretermino la medicin de la longitud cervical se realizara entre la semana 20.1-34 de gestacin y fibronectina fetal entre
las semanas de 24-34 de gestacin. La amniocentesis puede ser usada en mujeres con amenaza de parto pretermino para valorar la madurez pulmonar
fetal e infeccin intraamniotica. FIBRONECTINA FETAL: La FNf es una glicoprotena producida por la membrana corinica y est localizada en la decidua
basal adyacente al espacio intervelloso. Su objetivo principal parece el de una molcula de adhesin (pegamiento), la cual favorece la unin entre las
membranas corionicas con la decidua materna contigua. Puede ser encontrada, normalmente, en SCV en las primeras 22 SDG. Virtualmente, nunca es
encontrada en la misma secrecin entre las 24 y 34 SDG, a menos que el crvix haya desarrollado borramiento y dilatacin prematura, generalmente en
asociacin con contracciones uterinas sintomaticas. Existe una fuerte asociacin entre la expresin de FNf en SCV y PP. Numerosos ensayos han
mostrado una asociacin entre la presencia de FNf y nacimientos pretrmino, as como una reduccin en el riesgo de nacimientos pretrmino cuando el
resultado de esta prueba es negativo Los beneficios con el uso de la prueba de FNf(1) son: Disminucin de ingresos hospitalarios y de la duracin de los
das de hospitalizacin. Identificacin ms apropiada de pacientes que necesitan esteroides y tratamiento con agentes tocolticos. Disminucin del uso
de agentes tocolticos en pacientes que no estn en riesgo de nacimiento pretrmino. Reduccin del estrs y ansiedad para la mujer embarazada y su
familia debido a la tranquilidad de no realizar una transportacin u hospitalizacin innecesaria. Las indicaciones para FNf son: embarazos de 24 a 34 SDG,
sntomas de parto pretrmino (contracciones uterinas regulares >6 por hora y/o presin plvica) membranas amniticas intactas, crvix <3 cm de
dilatacin y bienestar fetal documentado. TRATAMIENTO: Los antibiticos no parecen prolongar la gestacin y solo debe usarse para la profilaxis del
estreptococo del grupo B en pacientes en las cuales el parto es inminente. No est definido un tocolitico de primera eleccin para el manejo del parto
pretermino. Las drogas tocoliticas pueden prolongar el embarazo de 2-7 das permitiendo la administracin de esteroides para mejorar la madurez fetal
y considerar la referencia de la madre a un tercer nivel. Usar de terbutalina 0.25mg subcutnea cada 20min a 3 h controlando intervalos regulares de la
presin arterial y la frecuencia cardiaca en la madre, en conjunto con un control estricto de lquidos. Se recomienda el uso de indometacina 100mg rectal
de dosis inicial, con 25-50mg VO o VR cada 4-6 horas por un mximo de 48h. Indicado si la paciente presenta enfermedad cardiovascular,
hipertiroidismo, DM o hipertensin. Solo utilizar en embarazos con menos de 32 SDG. La administracin de 2 dosis de 12mg de betametasona o
dexametasona IM, con 12-24h de intervalo, se recomienda como esquema de induccin de madurez fetal. La edad gestacional recomendada para la
apliacion de corticoide para induccin de madures pulmonar fetal en pacientes con riesgo de parto pretermino es de 24-34 SDG. Los tocoliticos no deben
utilizarse como terapia de mantenimiento de repeticin. Tratamiento no farmacolgico: El reposo en cama y la hidratacin no parecen mejorar la
incidencia de parto pretermino, por lo cual no debe ser una recomendacin rutinaria. El internamiento prenatal para guardar reposo no ha mostrado ser
efectivo en disminuir el parto pretermino y la morbilidad perinatal, por lo cual se recomienda individualizar cada caso. El reposo absoluto puede
presentar efectos adversos en las mujeres y debe evitarse en la medida de lo posible. Sera motivo de referencia a tercer nivel, embarazos con fetos
potencialmente viables (27-32SDG), en caso que la unidad de referencia no cuente con unidad de terapia intensiva neonatal. Contraindicaciones de la
inhibicin del parto pretrmino; La inhibicin del parto pretrmino est contraindicada en las siguientes condiciones cuando el parto es inminente o
cuando otros factores obsttricos indican que el parto no debe ser retrasado, a saber: Retardo del crecimiento intrauterino, pre-eclampsia fulminante,
abrupto placentario severo, sufrimiento fetal, corioamnioitis severa ante una ruptura de membranas, anormalidad mortal del feto, desarrollo de efectos
adversos severos con el tratamiento. Evite inhibir el parto si est en el mejor inters de la madre o el nio. Debe detenerse el tratamiento si el parto
contina a pesar de la terapia. PREVENCION: En general se dice que cerca del 50% de los partos pretrmino son potencialmente prevenibles. Algunas
estrategias utilizadas para la reduccin de los factores de riesgo incluyen estrategias basadas en la poblacin, a saber: Preparacin para el parto: estado
nutricional, evitar el tabaco y drogas. Ambiente de apoyo. Dar mayor poder a la mujer en su consulta prenatal, incluyendo apoyo continuado, adecuada
respuesta a preguntas e informacin para evitar factores de riesgo. Educacin al paciente sobre el reconocimiento de los sntomas tempranos (dolor
abdominal bajo, dolor lumbar, presin plvica, flujo vaginal aumentado, sangrado o manchado). Tratamiento de las infecciones vaginales sintomticas
antes de las 32 semanas. Tamizaje por vaginosis bacteriana en mujeres con antecedente de parto pretrmino. Cerclaje cervical en mujeres con historia
de incompetencia cervical.
CASO CLINICO PARTO PRETERMINO
Se trata de paciente femenino de 29 aos de edad la cual acude a
urgencias refiriendo contar con 34 semanas de gestacin por FUM, no
cuenta con cuidados prenatales, agrega que este es su tercer
embarazo, previamente dos partos, agregando que no presenta

CURSO ENARM CMN SIGLO XXI TEL: 36246001

complicaciones en sus embarazos o partos previos, a la exploracin


usted observa borramiento del 100% y dilatacin de 8 cm, decide dar
continuidad al trabajo de parto, ya que considera que en caso de
enviarla a unidad de mayor complejidad colocara en riesgo al binomio.

Pharmed Solutions Institute

PGINA 277

MANUAL DE TRABAJO DEL CURSO ENARM CMN SIGLO XXI


PREGUNTA
Cul es la causa ms probable por la que realizaramos una
fibronectina fetal en esta paciente?
RESPUESTA
a.- Solo si la paciente tuviera menos de 3 cm de dilatacin
b.- Solo si tuviera menos de 30 semanas de gestacin
c.- Solo si tuviera menos de 4 contracciones regulares
d.- Solo si hubiera sufrimiento fetal agudo

PREGUNTA
Cul es la contraindicacin menos probable para inhibicin de parto
pretermino?
RESPUESTA
a.- Coriamnioitis
b.- Abrupto placentarie severo
c.- Retraso en el crecimiento
d.- Sufrimiento fetal agudo

PREGUNTA
Cul es la conducta teraputica mas adecuada a seguir en este
momento?
RESPUESTA
a.- Uso de tocoliticos
b.- Parto
c.- Cesrea
d.- Administracion de esteroides

CASO CLINICO
Paciente femenino de 39 aos de edad, soltera con AGO de G3, P2, A0,
C0, acude al servicio de urgencias por presentar contracciones uterinas
de leves a moderadas aproximadamente de una a dos en lapso de 10
min., desde hace 1 hora, actualmente cursa con embarazo de 33
semanas de gestacin, niega antecedentes de importancia y llevar
control prenatal en centro de salud mensualmente, nicamente refiere
infecciones de vas urinarias en 3 ocasiones en los ltimos 2 meses, las
cuales fueron tratadas, actualmente solo con poliuria, tenesmo vesical.
EF: Consciente y orientada, cardiopulmonar sin compromiso, signos
vitales T/A 115/80mmHg, FC76x, FR 18x, abdomen globoso a
expensas de PUVI con FU a 30cm, longitudinal ceflico, con FCF de
145x, al tacto vaginal, borramiento de 60%, dilatacin 2 cm.
Laboratorio

PREGUNTA
Considerando que la paciente cursa con un trabajo de parto
pretermino, cual es la complicacin ms frecuente que el producto
puede presentar y por lo tanto tendr que enviar a un segundo nivel
posteriorior al nacimiento?
RESPUESTA
a.- Hemorragia Periventricular.
b.- Enfermedad de Membrana Hialina.
c.- Enterocolitis Necrotizante.
d.- Sepsis Neonatal.
CASO CLINICO
Paciente de 34 aos de edad, casada, con antecedentes de
incompetencia cervical, colecistectoma laparoscpica debido a
colecistitis crnica litisica de cinco meses de evolucin y
timectomizada a los 18 aos de edad por miastenia gravis, sin
tratamiento farmacolgico actual. Al momento de su estudio cursaba
la vigsima cuarta semana del quinto embarazo. Los partos previos
fueron vaginales, menores de 25 semanas, de hijos nacidos vivos pero
que fallecieron a las pocas horas de nacidos, el alumbramiento fue
normal, con una placenta discoide, a la revisin se obserrvaron dos
zonas discoides, hemorrgicas adems signos de envejecimiento
placentario.
PREGUNTA
Cual de las siguientes medidas presentan mayor efecto para modificar
el curso de esta entidad?
RESPUESTA
a.- Cerclaje temprano.
b.- Reposo absoluto.
c.- Antibiotico profilctico.
d.- Progesterona micronizada
PREGUNTA
A las 2 semanas la pacienre regresa con dolor suprapbico, fiebre, el
USG muestra retardo en el crecimiento fetal, a la EF: diltacion de 4cm,
y borramiento de 40%. Cul es la conducta mas adecuada a seguir en
este momento?
RESPUESTA
a.- Reposo absoluto
b.- Uso de antibitico
c.- Terapia tocolitica
d.- Resolucin de embarazo

CURSO ENARM CMN SIGLO XXI TEL: 36246001

PREGUNTA
Cul es la conducta teraputica ms adecuada a seguir en este caso?
RESPUESTA
a.- Hospitalizacin y sulfato de magnesio
b.- Hospitalizacin y terbutalina
c.- Hospitalizacin e indometacina
d.- Hospitalizacin y antibitico
PREGUNTA
Si le tomaramos una fibronectina fetal a esta paciente. Cul sera la
utilidad ms probable?
RESPUESTA
a.- Nos proporciona datos de inmadurez fetal
b.- Nos proporciona datos de borramiento y dilatacin prematura
c.- Nos indica sufrimiento fetal agudo
d.- Nos orienta sobre proceso infeccioso en la madre
PREGUNTA
Cules son las sustancias ms probablemente encargadas de
desencadenar las contracciones uterinas en este caso?
RESPUESTA
a.- IL-6, IL-8
b.- Prostanoides
c.- Cortisol
d.- Proteasas
PREGUNTA
En la semana 36 regresa la paciente nuevamente con contracciones
uterinas, de intensidad de moderada a severa, de 3 en10 minutos, de
duracin de 30 seg aprox., al tacto vaginal crvix con dilatacin de 5cm
y borramiento del 60%, Cul es la conducta terapeutica mas
adecuada en este momento?
RESPUESTA
a.- Hospitalizar y dejar evolucionar
b.- Hospitalizar e induccin de madurez fetal
c.- Hospitalizar e iniciar terapia tocolitica
d.- Hospitalizar y cerclaje uterino

Pharmed Solutions Institute

PGINA 278

MANUAL DE TRABAJO DEL CURSO ENARM CMN SIGLO XXI


PARTO NORMAL
CIENCIAS BASICAS: Se define como aquel de comienzo espontneo, de bajo riesgo desde el comienzo del trabajo de parto, hasta la finalizacin del
nacimiento el nio nace en forma espontnea, en presentacin ceflica, entre las 37-41 SDG. Luego tanto la madre como el nio estn en buenas
condiciones. Presentacin: parte del feto que se presenta al canal del parto y es capaz de desencadenar el TDP. El 95 % de las presentaciones son
ceflicas, 4 % son presentaciones podlicas o pelvianas, que pueden ser de cara, frente y muy rara, de hombros. Situacin: relacin que existe entre el
eje axial del feto y el eje del tero, por lo que existen dos tipos de situacin: longitudinal y transversa. Posicin fetal de una presentacin particular:
relacin existente entre el dorso del feto y el lado derecho o izquierdo de la madre. En la variedad de posicin en cada tipo de presentacin va a tener un
punto de referencia. En la presentacin ceflica el punto de referencia es el occipucio; en la presentacin de cara es el mentn y en la pelviana o podlica
es el sacro. TRABAJO DE PARTO (TDP): Por definicin, el comienzo es espontneo, por lo tanto se inicia en la casa, y es la propia mujer y su familia
quienes controlan sus condiciones y evolucin durante las primeras etapas. Causas que desencadenan el TDP; 1. Musculares. 2. Hormonales. 3.
Nerviosas. 4. Placentarias. 5. Fetales. Reflejo de Ferguson 1: La distensin progresiva del tero estimula los receptores locales que por va neuronal
estimulan los ncleos supraopticos y paraventriculares que liberaran oxitocina. Reflejo de Ferguson 2: La mayor distencin del segmento inferior y el
borramiento y dilatacin cervical llevara nuevos estmulos reflejos hacia la neurohipofisis lo que estimulara la secrecin de oxitocina. FASES DE TDP:
FASE 1 LATENTE: Ocurre al final del embarazo hasta el inicio de las contracciones del TDP. FASE 2 ACTIVA: Se compone de 3 perodos: 1) Perodo de
dilatacin: comprende desde el comienzo de la labor del parto hasta la dilatacin completa (10 cm). Es la fase ms larga del TDP y tiene una duracin de
8-12 h en las nulparas y de 6-8h en las multparas. Consta de 2 fases: A) Fase latente: comienza con las contracciones y se prolonga hasta los 2,5 cm de
dilatacin; es de unas 8 horas de duracin y no debe pasar de 20 horas en nulparas y de 14 en multparas. B) Fase activa: fase de aceleracin inicial, fase
de mxima inclinacin, fase de desaceleracin. 2) Perodo de expulsin: comprendido desde la dilatacin completa hasta el nacimiento del beb y vara
desde pocos minutos hasta 2h, sobre todo en las nulparas. 3) Perodo de alumbramiento: transcurre desde el nacimiento hasta la expulsin de la
placenta, y se espera que esto ocurra en un perodo de 30 a 40 min. Algunos plantean un cuarto perodo que sera la hora despus del alumbramiento en
que existe gran peligro de que ocurra una hemorragia; pero la mayora asume que ste sera la prolongacin del tercer perodo. Es difcil determinar en
muchas ocasiones cundo comenz la labor de parto, pero por regla general esto ocurre cuando las contracciones tienen una frecuencia cada 2 3 min
de aparicin entre ellas y duran de 30 a 45seg, y como resultado existen modificaciones del cuello uterino. Los 3 factores que contribuyen a que las
contracciones borren y dilaten el cuello son: 1. La presin hidrosttica de la bolsa de las aguas. 2. La traccin que sobre las fibras del cuello uterino
ejercen las del cuerpo. 3. La dilatacin activa por la disposicin espiral de las fibras musculares del tero. La bolsa de las aguas, impulsada por la presin
hidrosttica, se introduce en el orificio cervical interno al comienzo de la dilatacin, y al mismo tiempo expulsa el tapn mucoso endocervical. El
sangramiento que ocurre en un parto normal se encuentra entre los 400 a 500 mL. Se plantea que despus del alumbramiento el tero se contrae y se
retrae para mantener la hemostasia por medio del estrechamiento de las fibras musculares que comprimen los vasos venosos, a lo que se ha
denominado ligaduras vivientes de Pinard. CONTRACCIONES UTERINAS: La medida de la contraccin uterina y su registro grfico tiene mucha
importancia en la clnica obsttrica moderna, porque constituyen la base de un control ms cientfico del seguimiento del TDP, que recibe el nombre de
monitorizacin obsttrica. Se ha determinado que la intensidad de las contracciones en el parto normal es de 30-50 mmHg y el tono muscular normal
est comprendido entre 8 y 12 mm. Antes de las 30 SDG: Tipo A, poca intensidad y limitado a pocas reas del tero (lvarez). Tipo B, Braxton Hicks
mayor intensidad, baja frecuencia y se propagan. Preparto en ultimas semanas: Aumento de las contracciones de Braxton Hicks, se caracteriza por la
maduracion progresiva del cuello uterino. Cuando el cuello presenta 2cm de dilatacin o ms, la onda contrctil normal comienza en las zonas
denominadas marcapasos ubicadas cerca de la implantacin de las trompas y se caracteriza por el triple gradiente descendente (TGD). 1. Sentido
descendente 2. Duracin: disminuye a medida que se aleja de los marcapasos 3. La intensidad disminuye a medida que se aleja de los marcapasos.
Clnica de las contracciones durante el TDP: Frecuencia: comienzo c/ 20 o 25 minutos final c/ 2-3 minutos. Duracin: Comienzo 15 a 20 segundos final 4045seg, intensidad: Aumento a medida TDP, hay dolor: por la dilatacin y contracciones. MECANISMO DEL PARTO: En la mayora de los partos se realiza
por parte del objeto del parto (el feto) una serie de evoluciones que van a ir sorteando los diferentes dimetros y curvatura de la pelvis sea y de las
partes blandas del canal, y que tendr su mecanismo bien establecido en cada una de las presentaciones. Por ser la presentacin de vrtice la ms
frecuente, describiremos estos mecanismos: 1) Acomodacin Estrecho Superior: para facilitar su entrada en la excavacin plvica, la cabeza se reduce
por flexin y cabalgamiento de los huesos del crneo y orienta sus dimetros mayores anteroposteriores hacia el dimetro mayor del estrecho superior.
Al comienzo del parto la cabeza colocada transversalmente se halla en actitud indiferente y presenta su dimetro frontooccipital de 12 cm al estrecho
superior de la pelvis. En virtud de las contracciones uterinas y de la resistencia que ofrece el estrecho superior, la cabeza se flexiona y sustituye su
dimetro anteroposterior por otro menor, el suboccipitofrontal de 10,5 cm. 2) Encaje y descenso: La contraccin uterina hace que la cabeza realice un
movimiento de lateroflexin, en virtud del cual el parietal posterior desciende a la excavacin pelviana y rebasa el promontorio, mientras el parietal
anterior est detenido al nivel del pubis (asinclitismo posterior). Al descender la cabeza por debajo del promontorio y alojarse el parietal posterior en la
concavidad del sacro, permite el descenso a su vez del parietal anterior hacia la cavidad pelviana. Cuando ambos parietales han franqueado el estrecho
superior de la pelvis se puede decir que la cabeza se ha encajado. La cabeza tambin puede encajarse en asinclitismo anterior e incluso sinclticamente.
3) Rotacin interna: Al llegar aqu se encuentra con una situacin completamente distinta: los dimetros mayores del estrecho inferior son los
anteroposteriores, por lo cual tiene que realizar una rotacin de 90 para acoplar as sus dimetros mayores (anteroposteriores) a los del estrecho
inferior que tambin son los anteroposteriores. 4) Desprendimiento: La cabeza tiene que vencer la resistencia del perineo y de la extremidad inferior del
sacro y del coccix. El sacro puede realizar movimientos muy limitados, no mayores que 5 a 6 mm en su parte inferior (nutacin y contranutacin del
sacro); sin embargo, el cccix puede retropulsar 30-40mm la cabeza va dilatando el anillo vulvar y aparecen, en secuencia, las eminencias frontales, la
raz de la nariz, la boca, y, por ltimo, el mentn, hasta completar su desprendimiento total, mientras la cabeza se est desprendiendo en sentido
anteroposterior, los hombros, con dimetro biacromial y situados en el estrecho superior, van descendiendo, en la excavacin pelviana (encajamiento de
los hombros). 5) Rotacin externa: Los hombros, que han llegado en transversa al estrecho inferior, tienen que realizar un movimiento de rotacin
interna de 90 para llevar su dimetro mayor biacromial al dimetro mayor anteroposterior del estrecho inferior. Los hombros, al realizar su rotacin
interna en el estrecho inferior, hacen que la cabeza, fuera de la vulva y sin nada que la obstaculice, realice su rotacin externa. Sexto tiempo: expulsin
de los hombros y deslizamiento del resto del cuerpo. Una vez expulsados los miembros inferiores del feto por la hendidura vulvar, termina el mecanismo
del parto. PERODO DE EXPULSIN: Cuando la dilatacin se ha completado y la cabeza llega al plano perineal, la parturienta experimenta deseos de
pujar en cada contraccin y se abomba el perineo, por lo que sabemos que la paciente est en perodo expulsivo. Se colocar a la paciente en posicin
ginecolgica en una mesa de partos. Auscultar frecuencia cardaca fetal. Asepsia del perineo, la vulva y la raz de los muslos. Colocarn los paos estriles
y medias ginecolgicas. Asepsia del mdico (manos). Se vestir con bata estril, gorro, tapabocas, botas, guantes estriles. La mano izquierda se coloca
sobre el saliente de la cabeza y con la mano derecha se cogen, entre el pulgar por un lado y los dedos ndice y medio por el otro, las eminencias frontales
a travs del perineo tenso. Con ambas manos se controla la salida de la cabeza para evitar que sta lo haga bruscamente y produzca un desgarro
perineal. Aunque la proteccin del perineo es conveniente, no debe prolongarse exageradamente. En estos casos se realizar su infiltracin local con
lidocaina a 1% u otro anstesico similar y se proceder a realizar la episiotoma mediolateral derecha o izquierda. La episiotoma siempre es ms

CURSO ENARM CMN SIGLO XXI TEL: 36246001

Pharmed Solutions Institute

PGINA 279

MANUAL DE TRABAJO DEL CURSO ENARM CMN SIGLO XXI


ventajosa que un desgarro perineal (no abusar de este proceder). Una vez expulsada la cabeza fetal, se aspiran las flemas de las fosas nasales de la
orofaringe mientras se va produciendo espontneamente el resto del mecanismo del parto. Si existiera alguna circular del cordn, se libera. Despus que
dejamos de sentir las pulsaciones en el cordn umbilical, se pinza ste con tres pinzas de Kocher y se secciona entre las dos ms prximas. ATENCIN
DEL PERODO DE ALUMBRAMIENTO: Una vez terminado el perodo de expulsin, debe asistirse con especial atencin al alumbramiento. Se observarn
los signos clnicos del desprendimiento placentario: 1. Elevacin del tero por encima del ombligo y desviacin hacia la derecha; esto se puede observar
a travs de las cubiertas abdominales Signo de Kstner, el cual consiste en el ascenso del cordn umbilical al colocar nuestra mano por encima de la
snfisis del pubis y rechazar el tero hacia arriba, lo que significa que la placenta an est adherida al tero. En caso de que la placenta se haya
desprendido, el cordn no se mover. 3. Descenso espontneo de la pinza colocada en el cordn al nivel de la vulva. El desprendimiento placentario por
lo general ocurre espontneamente en los 10 min siguientes a la expulsin del recin nacido, pero se debe esperar hasta 30 min antes de actuar. Una vez
desprendida la placenta. Una vez expulsada la placenta se debe evitar el desgarro de las membranas, para lo cual se le imprimen movimientos de torsin
a la placenta (maniobra de Dublin), de manera que las membranas se hagan ms resistentes a los desgarros y vayan saliendo lentamente, pero ntegras.
Despus de extrada la placenta se procede a su revisin.
CASO CLINICO TRABAJO DE PARTO
Se trata de femenino de 29 aos de edad, la cual acude a urgencias
debido a que desde hace 10 horas inicia con contracciones dolorosas,
la paciente cuenta con antecedentes GO gesta 3, para 2, abortos 0, a
la exploracin fsica se observa producto ceflico, occipito anterior
derecho identificado por palpacin abdominal y tacto vaginal, se
observa a nivel de segundo plano, se encuentra 100 % de borramiento
y dilatacin de 7 cm, la actividad uterina se observa 3 a 4
contracciones cada 10 minutos con duracin de 50 a 60 segundos,
durante la exploracin observa presencia de cordon umblical y liquido
amnitico de caractersticas adecuadas, el producto presenta
frecuencia cardiaca de 150 con descensos variables durante la
contraccin de menos 20 latidos.
PREGUNTA
Considerando el cuadro clnico, cual es la conducta a seguir?
RESPUESTA
a.- Realizar Bloqueo y Realizar Cesarea.
b.- Realizar Bloqueo y Continuar Trabajo de Parto.
c.- Continuar Trabajo de Parto.
d.- Indicar Tocoliticos.
CASO CLINICO PARTO
Se trata de femenino de 19 aos de edad la cual es originaria del
Estado de Mxico la cual acude a urgencias, refiere que desde hace 6
horas presenta dolor abdominal los cuales son espordicos,
aproximadamente cada 10 minutos, de intensidad moderada, agrega
que desde hace 4 horas arrojo moco con sangre por va vaginal,
durante la exploracin se encuentra cuello cervical reblandecido, con
un centmetro de dilatacin con producto libre, usted observa
presencia de secrecin verdosa con olor desagradable, la madre refiere
que ha presentado cuadro repetidos de infecciones cervicovaginales
con presencia de VPH por colposcopia reportado a las 18 semanas de
gestacin, sin tratamiento, actualmente cuenta con 40 semanas de
gestacin.

PREGUNTA
Cul es el factor ms importante que busca para tomar una decisin
quirrgica?
RESPUESTA
a.- Calcificacin placentaria.
b.- Caractersticas del lquido amnitico.
c.- Cantidad de lquido amnitico.
d.- Caractersticas del cordn umbilical.
CASO CLINICO
Paciente de 15 aos, primigestante Natural y residente en Zona rural
del Estado de Oaxaca, con embarazo de 38 semanas, por ecografa de
la semana 28 , FUM no confiable, Sin controles prenatales, sin otros
paraclnicos diferentes a la ecografa realizada. Consulta por cuadro
clnico de 4 horas de evolucin de presentar inicio de actividad uterina
aproximadamente 2 contracciones cada 10 minutos de 45 segundos de
duracin, de moderada intensidad, asociado a salida de tapn mucoso
sanguinolento por vagina, con movimientos fetales presentes, sin otras
perdidas vaginales. AGO: M: 13 G:1 A:0 P:0 sin planificacin, ciclos
irregulares. EF: Paciente en buenas condiciones generales, afebrl no
disnea FC: 74 FR: 16 PA: 110/70. Abdomen: tero grvido AU: 33cm,
con actividad uterina de 2 /10/40 segundos de buena intensidad,
fetocardia de 135/ min, movimientos fetales presentes, longitudinal,
ceflico, dorso derecho. Genitales externos sanos, TV: crvix anterior,
semiblando, longitud de 1cm, dilatacin de 2 cm, ceflico, estacin de
-1, membranas integras, pelvis ginecoide. Ext: edema de miembros
inferiores grado I.
PREGUNTA
En que fase de trabajo de parto se encuentra esta paciente?
RESPUESTA
a.- Fase latente
b.- Fase activa
c.- Periodo expulsivo
d.- Fase de aceleracin inicial

PREGUNTA
Considerando el cuadro actual, cual es la conducta a seguir?
RESPUESTA
a.- Esperar inicio de trabajo de parto.
b.- Inducir trabajo de parto.
c.- Prepara a la paciente para cesrea.
d.- Conducir el trabajo de parto.

PREGUNTA
En que momento es menos probable hospitalizar a esta paciente?
RESPUESTA
a.- Borramiento completo
b.- Dilatacion de 2 cm
c.- 2 o mas contracciones en 10 minutos
d.- Ruptura de membranas

CASO CLINICO PARTO POSTERMINO


Se trata de paciente femenino de 32 aos de edad la cual acude por
tercera vez a consulta debido a que se encuentra embarazada y no se
ha sentido bien, cuenta con antecedente de G; 3 Para; 2 Abortos 0; al
interrogatorio se identifica 42 semanas de gestacin por FUM, sin
embargo no hay inicio espontaneo de trabajo de parto, por lo indica
USG.

PREGUNTA
Qu es lo menos probable de observar en esta paciente si tuviera una
fase latente prolongada?
RESPUESTA
a.- Incrementa el riesgo de cesrea
b.- Ruptura prematura de membranas
c.- Exposicin a meconio
d.- Incremento en depresin neonatal e ingreso a UCI.

CURSO ENARM CMN SIGLO XXI TEL: 36246001

Pharmed Solutions Institute

PGINA 280

MANUAL DE TRABAJO DEL CURSO ENARM CMN SIGLO XXI


CESREA
CIENCIAS BASICAS: La cesrea es la intervencin quirrgica que tiene como objetivo extraer el producto de la concepcin y sus anexos ovulares a travs
de una laparotoma e incisin de la pared uterina. TIPOS DE CESREA: A) Segn antecedentes obsttricos de la paciente: Primera: Es la que se realiza por
primera vez. Iterativa: Es la que se practica en una paciente con antecedentes de dos o ms cesreas. Previa: es cuando existe el antecedente de una
cesrea anterior. B) Segn indicaciones: Urgente: Es la que se practica para resolver o prevenir una complicacin materna o fetal en etapa crtica.
Electiva: Es la que se programa para ser realizada en una fecha determinada por alguna indicacin mdica y se ejecuta antes de que inicie el trabajo de
parto. C) Segn tcnica quirrgica: Corporal o clsica. Segmento corporal (Tipo Beck). Segmento arciforme (Tipo Kerr). CORPORAL O CLASICA: La incisin
es vertical se realiza en el cuerpo uterino. Sus indicaciones ms frecuentes son: cncer crvico-uterino invasor, embarazo pretrmino, situacin fetal
transversa con dorso inferior, histerorrafia corporal previa, procesos adherenciales o varicosos importantes en el segmento inferior, placenta previa en
cara anterior, cesrea posmortem, miomatosis uterina de grandes elementos y cuando despus de la cesrea se realizar una histerectoma. Sus
desventajas son: apertura y cierre ms difcil, mayor hemorragia, adherencias ms frecuentes, histerorrafia menos resistente que puede hacerse
dehiscente durante un nuevo embarazo. SEGMENTO CORPORAL (Beck): La incisin es vertical y se realiza sobre el segmento y parte del cuerpo uterino.
Sus principales indicaciones son: embarazo pretrmino, embarazo gemelar, situacin fetal transversa con dorso inferior, presentacin plvica, placenta
previa en la cara anterior del tero, anillo de retraccin e histerorrafias corporales previas. Las desventajas de esta tcnica no difieren de la anterior.
SEGMENTO ARCIFORME O TRANSVERSAL (Kerr): Es la tcnica quirrgica ms usada por sus mltiples ventajas. Al realizarse una incisin transversal del
segmento inferior tiene las ventajas de producir menos hemorragia, y permitir una fcil apertura y cierre de la pared uterina, formacin de cicatriz
uterina muy resistente con poca probabilidad de dehiscencia y ruptura en embarazos subsecuentes y as como pocas adherencias postoperatorias.
INDICACIONES DE LA OPERACIN CESREA: La indicacin para realizar una operacin cesrea puede ser por causas: Maternas, Fetales o Mixtas. A)
Causas maternas; 1. Distocia de partes seas (desproporcin cefaloplvica): Estrechez plvica. Pelvis asimtrica o deformada. Tumores seos de la pelvis.
2. Distocia de partes blandas: Malformaciones congnitas. Tumores del cuerpo o segmento uterino, crvix, vagina y vulva que obstruyen el conducto del
parto. Ciruga previa del segmento y/o cuerpo uterino, incluyendo operaciones cesreas previas. Ciruga previa del cervix, vagina y vulva que interfiere
con el progreso adecuado del trabajo del parto. 3. Distocia de la contraccin: Hemorragia (placenta previa o desprendimiento prematuro de la placenta
normoinserta). Patologa materna incluyendo nefropatias, cardiopatas, hipertensin arterial o diabetes mellitus, etc. B) Causas fetales: Macrosoma fetal
que condiciona desproporcin cefaloplvica. Alteraciones de la situacin, presentacin o actitud fetal. Prolapso de cordn umbilical. Sufrimiento fetal.
Malformaciones fetales incompatibles con el parto. Embarazo prolongado con contraindicacin para parto vaginal. Cesrea posmortem. C) Causas
mixtas: Sndrome de desproporcin cefaloplvica. Preeclampsia/eclampsia. Embarazo mltiple. Infeccin amnitica. Isoinmunizacin materno-fetal.
INDICACIONES MS FRECUENTES: Las principales indicaciones de la operacin cesrea son: 1. Desproporcin cefaloplvica: La desproporcin
cefaloplvica ocurre en las siguientes circunstancias: Pelvis estrecha, deformada u obstruida y feto normal con buena actitud. Pelvis normal y feto
grande, anmalo (gigantismo o hidrocefalia) o con mala actitud. Pelvis y feto normales con dimetros fetoplvicos limtrofes, despus de una prueba de
trabajo de parto infructuoso. El diagnstico de la DCP se establece mediante una valoracin clnica adecuada de los estrechos superior, medio e inferior
de la pelvis materna y su relacin con el feto, con el apoyo de estudios de inmagenologia. Ante la duda diagnstica se deber utilizar la prueba de trabajo
de parto. 2. Cesrea previa: la estrategia de permitir un parto vaginal a pacientes con cesrea previa bajo condiciones controladas, logra un incremento
en los nacimientos por va vaginal sin complicaciones. Contraindicaciones para permitir el parto vaginal en mujeres con cesrea previa: Antecedente de
dos o ms cesreas previas segmento arciforme Antecedentes de cesrea previa tipo segmento-corporal, corporal o presencia de otras cirugas uterinas.
Antecedente de cesrea previa reciente (menor a 6 meses). Antecedentes de ruptura uterina o dehiscencia de histerorrafia. Pelvis no til clnicamente 3.
Sufrimiento fetal: Los factores causales del sufrimiento fetal pueden ser reversibles (hipotensin materna, efecto Poseiro, hipercontractilidad por uso de
oxitcicos), o irreversibles (causa fetal, placentaria o del cordn umbilical). Se considerarn los siguientes parmetros para evaluar la frecuencia
cardaca: Basal (normal): 120-160 lpm. Bradicardia: leve: entre 110-119 lpm, moderada: entre 100-109 lpm, severa: por debajo de 100 lpm. Taquicardia:
leve: entre 160-169 lpm, moderada: 170-179 lpm, severa: mayor de 180 lpm. En las unidades de atencin que disponen de equipos de cardiotocografa
se registrar la frecuencia cardaca fetal y su relacin con las contracciones uterinas. Las principales alteraciones que pueden observarse son: Bradicardia
menor de 100 lpm. mantenida durante 5 minutos o ms. Presencia de desaceleraciones tardas o dips tipo II en el 30% o ms de las contracciones.
Desaceleraciones variables graves (menos de 70 lpm de ms de 60 seg. de duracin) en el 50% de las contracciones. Variabilidad de la curva de 5 lpm o
menos por ms de 30 minutos. Taquicardia mayor de 170 lpm que persisten por ms de 10 min. Patrn sinusoidal (con amplitud menor de 5 lpm), sin
observarse movimientos fetales, ni aceleraciones. Se considerar la operacin cesrea, cuando exista la asociacin de dos o ms de las alteraciones de la
frecuencia cardaca fetal antes sealadas. 4. Ruptura prematura de membranas: Es la salida de lquido amnitico a travs de una solucin de continuidad
de las membranas ovulares en embarazos mayores de 20 semanas y/o por lo menos 2h antes del inicio del trabajo de parto. La conducta obsttrica en
los casos de ruptura prematura de membranas, es la siguiente: a) Menor de 26 semanas de gestacin: interrupcin del embarazo por va vaginal con el
consentimiento de la mujer. b) Entre 26 y 28 semanas de gestacin: cuando se cuente con los recursos mnimos para la atencin del neonato, se realizar
operacin cesrea. c) Entre 29 y 32 semanas de gestacin: se realiza operacin cesrea. d) Entre las semanas 33 y 36 de gestacin: se valorarn las
condiciones obsttricas y se resolver el embarazo por la va ms favorable. 5) Presentacin plvica: ocurre en el 3-4 % del total de los embarazos. En la
mayora de las unidades obsttricas, esta modalidad de presentacin constituye una indicacin de cesrea (excepto cuando la paciente acude en periodo
expulsivo). ANTIBIOTICOTERAPIA EN LA OPERACIN CESREA: Como en otras intervenciones quirrgicas, la realizacin de la operacin cesrea conlleva
riesgo de infeccin; la frecuencia y severidad de las infecciones depende de las condiciones en las que se realiza y con base en ello, puede considerarse
como: a) Limpia: Se considera que una operacin cesrea es limpia cuando se realiza en condiciones aspticas, sin defectos de la tcnica, ni lesiones en
los aparatos gastrointestinal o urinario. No hay reaccin inflamatoria en la vecindad y la cavidad uterina no est contaminada. En sta no hay necesidad
de utilizar antibiticos. b) Contaminada: Se considera de este modo cuando existen: Trabajo de parto de ms de 6h. Ms de 6 tactos vaginales.
Membranas rotas entre 6 y 24 hrs. Antecedentes de aminiocentesis o instrumentaciones. Alto grado de dificultad en la tcnica quirrgica. Anemia severa
(Hb. menor de 9 mg /dl). Lquido amnitico meconial. Se utilizarn antibiticos de manera profilctica desde el momento en el que se realiza el
pinzamiento del cordn umbilical. c) Sptica: Cuando hay evidencia de infeccin clnica, supuracin o presencia de materia fecal; en esta categora se
incluyen las cesreas con uno o ms de los siguientes factores: Cuadro febril. Lquido amnitico ftido o caliente. Ruptura de las membranas ovulares de
ms de 24h. Cesreas con alto grado de dificultad en la tcnica quirrgica. La antibioticoterapia se mantendr por va endovenosa 72h y se completar el
esquema segn la evolucin de la paciente. REQUISITOS PREOPERATORIOS: Antes de la operacin cesrea debern cubrirse los siguientes requisitos
preoperatorios: Historia clnica perinatal completa, incluyendo exmenes de laboratorio y gabinete. Consentimiento informado de la mujer o de un
familiar responsable, y firma de la autorizacin para realizar la operacin cesrea. En el caso de la oclusin tubaria bilateral, por ser un mtodo
anticonceptivo permanente se requiere adems la firma de consentimiento de la paciente en el formato institucional especfico. Nota preoperatoria en
el expediente clnico en la que se fundamente la indicacin de la operacin cesrea. CUIDADOS PREOPERATORIOS Y POSTOPERATORIOS:
Preoperatorios incluyen: Valoracin preanestsica de acuerdo a la NOM-170-SSAl-1998, Para la Prctica de Anestesiologa. Ayuno preoperatorio de 8h o
ms (cuando el caso lo permita). Rasurado suprapbico y aseo completo, con especial nfasis en la vulva y perineo. Colocacin de sonda vesical.

CURSO ENARM CMN SIGLO XXI TEL: 36246001

Pharmed Solutions Institute

PGINA 281

MANUAL DE TRABAJO DEL CURSO ENARM CMN SIGLO XXI


Disponibilidad de sangre segura y compatible. Postoperatorios consistirn en: Medicin horaria de signos vitales durante las primeras cuatro horas y
posteriormente cada 8h hasta el alta de la paciente. Vigilancia de la diuresis durante las primeras 12h y del inicio de la miccin espontnea, despus de
retirar la sonda vesical. Administracin de lquidos por va oral despus de 8h; una vez iniciado el peristaltismo intestinal se indicar dieta blanda.
CASO CLINICO CESAREA
Acude a consulta paciente femenino gestante con 39 semanas de
gestacin por USG la cual refiere tener 12 horas con dolor en regin
abdominal y lumbo-sacra, agrega que cuenta con antecedentes de
gesta 4 para 2 cesrea 1, aborto 0, a la exploracin identifica producto
podlico con borramiento del 30 % y dilatacin de 2 cm, la paciente
cuenta con antecedentes de polidramnios en embarazo previo con
producto con atresia gastroesofagica con muerte perinatal, enva a su
casa y cita en 6 horas, a la exploracin nuevamente observa a la
paciente la cual se encuentra fatigada, se observa actividad uterina con
3 contracciones en 10 minutos aproximadamente, con duracin de 50
segundos, la paciente refiere ruptura de membranas con presencia de
liquido verdoso, usted explora al producto el cual se encuentra
transversal con frecuencia cardiaca de 100 a 120 lpm.
PREGUNTA
Cul es su conducta ms adecuada a seguir?
RESPUESTA
a.- Prepara a la paciente para cesarea.
b.- Prepara a la paciente para conduccin.
c.- Espera a que el trabajo de parto continue.
d.- Inicia manejo con tocolitico.
CASO CLINICO
Paciente de 26 aos, de profesin abogada, casada, primigesta,
cursando un embarazo de evolucin fisiolgica de 36 semanas, sin
factores de riesgo. En la visita a su obstetra en un control habitual, ella
le recuerda que haba una conversacin pendiente sobre la va del
parto. En las primeras semanas de embarazo ella le haba planteado su
temor por el parto normal ya que ha tenido que tratar
profesionalmente con algunas mujeres que presentaron
complicaciones producto del parto o no se les indic a tiempo una
cesrea.
PREGUNTA
Cul sera la causa ms probable por la que se le realizara la cesrea
a esta paciente?
RESPUESTA
a.- Que llegara a la semana 41 y no presentara trabajo de parto
b.- Deseo explcito de la paciente de resolucin va cesrea
c.- Ruptura prematura de membranas
d.- Presencia de doble circular de cordn a cuello por USG
PREGUNTA
Por no tener factores de riesgo ni maternos, ni fetales, se decide la
resolucin por parto vaginal. Cul momento es el mas apropiado para
realizar amniotoma si se hiciera a esta paciente?
RESPUESTA
a.- En fase latente del primer periodo de trabajo de parto
b.- En fase activa del primer periodo de TDP
c.- En el segundo periodo de TDP
d.- En periodo expulsivo

CURSO ENARM CMN SIGLO XXI TEL: 36246001

PREGUNTA
Cunto tiempo esperaramos en promedio como normal, durante el
segundo periodo de parto (expulsivo) en esta paciente?
RESPUESTA
a.- 1 hora
b.- 10 minutos
c.- 30 minutos
d.- 80 minutos
CASO CLINICO
Femenino de 28 aos soltera, con embarazo de 37 semanas de
gestacin. AGO: G3, P2, A0, C0, PS 3, IVSA 16 aos, refiere control
irregular de embarazo en centro de salud. Acude a urgencias por
referir contracciones uterinas de intensidad leve, que se presentan de
2 a 3 en aproximadamente 30 minutos. EF: Orientada y consciente
cardiopulmonar sin compromiso, abdomen globoso a expensas de
PUVI, con FU a 32cm, presentacin ceflico, dorso izquierdo, FCF
157x, en genitales se observan vesculas en regin vulvar, as como
una ulcera en canal de parto, dolorosa, al tacto vaginal borramiento
del 70% y dilatacin de 4-5cm, en plano 2 de Hodge, resto sin
alteraciones. Al interrogatorio nos refiere un cuadro similar en tercer
trimestre para el cual recibi tratamiento, no especifica cual.
PREGUNTA
Cul es la conducta teraputica ms adecuada a seguir en esta
paciente?
RESPUESTA
a.- Cesrea corporal
b.- Parto con induccin
c.- Parto con conduccin
d.- Cesrea Kerr
PREGUNTA
Posterior a resolucin de embarazo. A dnde es ms probable que
enven al producto?
RESPUESTA
a.- Alojamiento conjunto
b.- Cunero patolgico
c.- UCI
d.- Aislamiento
PREGUNTA
A las 48 hrs la paciente, en pase de visita, refiere prurito y sensacin de
quemazn en rea de genitales y secrecin transvaginal
blancogrisacea, de olor desagradable. Cul de las siguientes medidas
es la ms adecuada a seguir en este momento?
RESPUESTA
a.- Metronidazol mas ganciclovir
b.- Revision de cavidad uterina
c.- Itraconazol y ganciclovir
d.- Metronidazol mas Aciclovir

Pharmed Solutions Institute

PGINA 282

MANUAL DE TRABAJO DEL CURSO ENARM CMN SIGLO XXI


COMPLICACIONES DEL PUERPERIO
CIENCIAS BASICAS: Se entiende por puerperio al periodo de tiempo que sigue al momento del parto durante el cual los cambios fisiolgicos del
embarazo, trabajo de parto y parto se resuelven y la mujer retorna al estado pregravdico y en la glndula mamaria se producen una serie de cambios
fisiolgicos que permiten la lactancia. Usualmente este periodo de tiempo es de 6 semanas. En el puerperio inmediato se aconsejan las siguientes
acciones, que pueden disminuir el riesgo de complicaciones: Vigilancia de la adecuada involucin uterina; masaje uterino. Lactancia precoz, limpieza del
pezn. Insistir en la higiene adecuada de la episiorrafia o de la herida quirrgica si la paciente tuvo una cesrea. Deambulacin precoz (en las primeras 24
horas) para disminuir la posibilidad de presentacin de eventos tromboemblicos y mantener activo el trnsito intestinal y vesical. La paciente puede
iniciar su alimentacin idealmente 2 horas despus de un parto vaginal; no antes porque dentro de este lapso pueden ocurrir las complicaciones
inmediatas del posparto que requieran anestesia. La revisin de la paciente en puerperio se puede realizar a la semana del posparto, cuando se puede
identificar cualquier anomala e iniciar el mtodo de planificacin familiar. CLASIFICACION: Cronolgicamente se divide en: Puerperio inmediato:
primeras 24h posparto. Puerperio mediato: desde el segundo hasta el dcimo da posparto. Puerperio tardo: desde el da 11 hasta el da 42 posparto.
HEMORRAGIA POSPARTO: prdida sangunea mayor de 500cc luego del parto vaginal y 1000cc luego de la cesrea. De una manera objetiva ha sido
descrito como la disminucin del hematocrito en 10% o ms en comparacin con el valor inicial. Su diagnstico se debe realizar de una manera oportuna
para darle un manejo adecuado. La frecuencia de la hemorragia posparto es de 3% 4% en las pacientes que han tenido un parto vaginal, hasta 9% en
quienes han tenido cesrea. Todo esto aumenta si est asociado a factores de riesgo. Usualmente se clasifica en dos tipos: Hemorragia posparto
inmediata: se llama as la que tiene lugar en las primeras 24 horas del puerperio; la mayora ocurre en las primeras 2 horas. sta es la hemorragia ms
frecuente y la ms importante. Hemorragia posparto tarda: Se presenta a partir del segundo da del puerperio hasta 6 semanas despus del parto. La
frecuencia oscila entre 5 a 15 %. A. HEMORRAGIA TEMPRANA: 1. RETENCIN DE PLACENTA: El principal mecanismo del alumbramiento est dado por la
contraccin uterina y la formacin del hematoma retroplacentario. El tiempo recomendado para que se produzcan los mecanismos del alumbramiento
es de 15 a 20 minutos; ms tiempo se considera retencin de la placenta. El manejo activo del alumbramiento se ha considerado una manera adecuada
para evitarla (se realiza la maniobra de Brandt: el descenso de la pinza que lig el cordn indica cuando empieza a desprenderse la placenta), uso de
oxitcico desde el tercer estadio del parto. 2. PLACENTA ANORMALMENTE ADHERIDA: Es un diagnostico que se realiza desde el tercer trimestre o en el
momento del parto. Consiste en una adherenciaanormal de las vellosidades a la pared uterina; existe una deficiencia o ausencia de la decidua basal, y es
difcil que se realice el alumbramiento normal, con la consecuente retencin de placenta. Se obtiene una placenta con sangrado de difcil control y
precoz. Clasificacin: creta: la placenta se encuentra adherida al endometrio ncreta: penetran las vellosidades en el miometrio Prcreta: la invasin
alcanza la serosa peritoneal y puede llevar a ruptura del tero. ATONIA UTERINA: Se define por la inadecuada involucin uterina, por la no contraccin
miometrial y cambios vasculares intramiometrales y ausencia de un adecuado sistema coagulacin que permita su control. Es la responsable del 80 % de
todas las hemorragias posparto, considerando que en el embarazo a trmino la suplencia sangunea de la placenta es de aproximadamente 600 a 800
mL/min. Se debe estar preparado para realizar esta reposicin de la volemia y desencadenar los mecanismos de adaptacin materna. Factores de riesgo:
Durante el embarazo: Multigestantes, infeccin intraamniotica, fibromas uterinos, sobredistencion uterina, polihidramnios. Macrosomia fetal. Durante
el trabajo de parto: Trabajo de parto precipitado, trabajo de parto prolongado. Relajacin uterina por medicamentos como sulfato de magnesio,
halogenados anestsicos, nitroglicerina. Hemorragia del tercer trimestre (diagnstico final del embarazo) Tratamiento: 1. Inicialmente reemplazar la
volemia con soluciones cristaloides y dependiendo del volumen de sangre perdido y el estado clnico de la paciente, administrar glbulos rojos
empacados. 2. Cuando la hemorragia se presenta antes de que se expulse la placenta se debe realizar un alumbramiento artificial. 3. Si ya ha salido la
placenta, entonces se procede a revisar el tero y el canal de parto. 4. La aplicacin de tero-tnicos: Oxitocina 10 a 20 U en infusin continua. Los
efectos adversos son poco: dolor con las contracciones, nausea, vmito, en casos extremos intoxicacin hdrica. Prostaglandinas. Disponible aplicacin
de Misoprostol (anlogo sinttico de la prostaglandina E 1). Dosis: 5 tabletas de 200 microgramos por va rectal, con muy buenos resultados reportados
en la literatura. Efectos adversos: hipertermia transitoria, broncoespasmo. Vasopresina: efecto vasoconstrictor: en casos extremos de no respuesta y si
est disponible. Inyeccin directa no vascular de 0.2 unidades (dilucin de 20 unidades en solucin salina 10occ, aplicar 1 cc ). INVERSIN UTERINA: Es
la invaginacin del tero sobre s mismo. El fondo del tero es impulsado hacia abajo y la parte superior de la cavidad endometrial se expone a travs
del cuello uterino y aparece en los genitales externos. El control y manejo incorrecto del alumbramiento es el principal factor desencadenante de esta
complicacin. Es muy raro que ocurra espontneamente. Se produce al traccionar en forma brusca y sostenida el cordn umbilical, combinado con la
presin sobre el fondo uterino, tratando de abreviar el tercer perodo del parto. Su presentacin es sbita, acompaada de la hemorragia, dolor plvico
intenso, hasta el shock neurognico por la gran tensin de todo el complejo de ligamentos de sostn uterino hasta la zona genital externa. Cuando
ocurre inmediatamente despus de la expulsin de la placenta se debe tratar: Colocar el tero en forma rpida de una manera anatmica. Introducir una
mano en la vagina y hacer presin suave y directa hacia arriba en el mismo eje del estrecho plvico superior. Aplicar Clorhidrato de Ketamina IV por parte
del anestesilogo para una relajacin materna. Estimular la contraccin uterina con infusin continua de oxitocina y masaje sobre el fondo uterino. Si se
produce la inversin antes de la salida de la placenta se debe considerar una normalidad de la insercin placentaria y manejarse como acretismo
placentario. B. HEMORRAGIA TARDA: SUBINVOLUCIN UTERINA: Es una detencin o retardo de la involucin. La subinvolucin va acompaada de una
prolongacin del periodo de eliminacin de los loquios y a veces, de una hemorragia uterina irregular o excesiva. Clnicamente el tero se encuentra de
mayor tamao, ms blando que lo normal para un momento determinado del puerperio. Entre las causas reconocidas de subinvolucin estn la
retencin de fragmentos de placenta y la infeccin plvica. Diagnstico: Examen clnico detallado. Realizacin de ecografa trasvaginal. Tratamiento: Si se
confirman restos placentarios: curetaje. tero tnico: (antes descritos). Si se diagnostica infeccin inicio de antibiticos de amplio espectro. HEMATOMA
VULVAR: Se diagnostica fcilmente por el intenso dolor y la aparicin sbita de un tumor sensible, fluctuante e intenso, de tamao variable y cubierto
por una piel cuya coloracin es diferente de lo normal, violceo. Puede presentarse tambin en el posparto inmediato. Su manejo es la Incisin y
evacuacin de sangre, seguidas de las ligaduras de los puntos que sangran. La cavidad debe obliterarse empleando una sutura adecuada, usualmente
catgut cromado 2 /0. Se administran antibiticos de amplio espectro si hay signos de infeccin. INFECCIN PUERPERAL: Las infecciones plvicas son las
ms comunes y serias complicaciones del puerperio. Se define como: temperatura mayor de 38 grados, excluyendo las primeras 24 horas. Las causas
ms frecuentes de fiebre en el puerperio son endometritis, infeccin urinaria y mastitis. ENDOMETRITIS: Es la forma ms comn de infeccin puerperal,
se relaciona con la decidua, el miometrio adyacente y los tejidos parametriales. Su va de Infeccin es ascendente, endgena y polimicrobiana (Gram
positivos del endometrio en el 40% de los casos, los grmenes aerobios gram negativos se aslan del endometrio en el 22%, Las bacterias anaerobias
representan el 38% de los patgenos, La Chlamydia Trachomatis es responsable del 2 al 4% de la endometritis tarda, hasta 6 semanas posparto.
Frecuencia de endometritis posterior a cesrea 7 a 30 veces ms frecuentecomparada con el parto vaginal. Depende de los factores de riesgo asociados.
Puede ser hasta un 90% si no se usan antibiticos profilcticos. Complica el 5 a 6 % de las cesreas electivas y el 22 a 85% de las urgentes Los sntomas
iniciales aparecen generalmente en las primeras 48 a 72 horas. La paciente consulta usualmente a los 4 a 5 das despus del parto por fiebre alta de 39 a
40 grados, (este es el signo cardinal de la endometritis, de ah que el aumento de la temperatura en el puerperio, se debe considerar endometritis hasta
que se pruebe lo contrario), sntomas generales, taquicardia, escalofro, expulsin de loquios que tienen un aspecto ajamonado (color rosado brillante) y
de muy mal olor .Se queja de dolor abdominal, principalmente en hipogastrio. La involucin del tero puede estar retardada. Si se encuentra fiebre de

CURSO ENARM CMN SIGLO XXI TEL: 36246001

Pharmed Solutions Institute

PGINA 283

MANUAL DE TRABAJO DEL CURSO ENARM CMN SIGLO XXI


38.5 a 39 grados centgrados durante las primeras 24 horas posparto, acompaada de distensin abdominal, taquicardia y loquios claros acuosos, se
debe considerar el diagnstico de endometritis por Estreptococo hemoltico del grupo B o del grupo A; el cuadro clnico puede progresar a celulitis
plvica, peritonitis y septicemia. Tratamiento: Teniendo en cuenta la microbiologa, los antibiticos usados deben ser de amplio espectro. Para casos
leves, usualmente despus de parto vaginal, un antimicrobiano de amplio espectro por va oral puede ser suficiente. Hay mejora en 48 a 72 horas en el
90% de las pacientes. Los esquemas pueden ser: Penicilina procanica. 800.000 unidades IM cada 12 a 24 horas Ampicilina 500 mg a 1 gr. Va oral cada
6 horas, Cefalospornas de primera generacin. 500 mg a 1 gr. Va oral, cada 6 horas, ms Un Aminoglicsido tipo Gentamicina 160 mg/da. Sin
embargo, para casos moderados a severos, incluyendo la mayora de aquellos que tuvieron parto por cesrea, se inicia terapia parenteral. Hospitalizar y
utilizar un esquema biconjugado triconjugado Emplear antibiticos de amplio espectro: Penicilina Cristalina 5.000.000 Unidades IV cada 6 horas
Cefalospornas de primera generacin 1 gramo IV cada 6 horas Ampicilina 1 gr., IV cada 6 horas, ms Gentamicina 80 mg IV cada 8 horas 240 mg IV
IM. , dosis nica al da Este esquema no cubre Bacteroides Fragilis, no previene las complicaciones de tromboflebitis plvica y absceso plvico, hay un
mayor tiempo de hospitalizacin. En un 20 y 40% de los casos posoperatorio de cesarea no responden a este tratamiento. Por eso se adiciona un tercer
antibitico que cubra anaerobios. Metronidazol 1 gr., inicial seguido 500mg IV va oral cada 8 horas Ornidazol 1 gr., cada 12 horas IV Clindaminica
600 cada 6 horas a 900 mg IV cada 8 horas (no slo cubre anaerobios, tambin estafilococo, estreptococo y Chlamydia), con este esquema slo se aade
la Gentamicina. INFECCIN E LA HERIDA QUIRRGICA: La incidencia de infeccin de la incisin abdominal luego de la cesrea es de un 5 a 15% con un
promedio de 7%. Cuando se administra antibiticos profilcticos la incidencia es de un 2% o menos. Los factores de riesgo para la infeccin de la herida
abdominal son: Obesidad, diabetes, terapia con esteroides, inmunosupresin, anemia y pobre hemostasia con formacin de hematoma. El tratamiento
consiste en drenaje quirrgico y antibiticos. Evaluar que la fascia este intacta, si no el cierre secundario se debe realizar. TRATAMIENTO: manejo
quirrgico; Taponamiento uterino: es necesario cuando el tratamiento quirrgico no sea posible en el sitio actual o la paciente este demasiado inestable
para una ciruga. Requisitos: 1. Eliminar todo el tejido placentario a excepcin de los fragmentos correspondientes a una placenta creta. 2. Descartar
una rotura y una inversin uterina. 3. Usar previamente los oxitcicos y el masaje uterino. 4. Descartar otra causa de sangrado. 5. Un ayudante expone el
cuello traccionando los labios anteriores y posteriores. Se emplea una pinza para extraer restos ovulares con el fin de empacar gasa seca en el fondo
uterino de modo que quede bien apretada, luego se completa el procedimiento dejando comprimida la vagina. 6. Durante el procedimiento se debe
aplicar una infusin continua de oxitocina. 7. La paciente debe recibir antibiticos 8. Se remite con sonda vesical abierta. 9. Si fracasa no debe intentarse
nuevamente, y se debe realizar una laparotoma sin prdida de tiempo. Si este procedimiento falla, se pasa al siguiente paso: ligadura de los vasos
uterinos: La arteria es ligada donde corre al lado del borde uterino, por encima del segmento uterino inferior. Si es posterior a una cesrea, la ligadura se
realiza 2 a 3 cm. por debajo del nivel de la incisin uterina. Puede ser necesario descender la vejiga. La arteria y la vena pueden ser ligadas pasando una
aguja 2 a 3 cm. medial a los vasos plvicos, incluyendo la mayora del grosor del miometrio, y atravesando el ligamento ancho en el rea avascular lateral
a los vasos. Histerectoma: Desafortunadamente no garantiza el control de la perdida sangunea severa. El sangrado puede persistir de las superficies
plvicas debido a alteraciones de la coagulacin combinado con el trauma de la manipulacin prolongada. Se debe continuar con empaquetamiento
abdominal. Empaquetamiento abdominal: Aplicacin de compresas estriles en la cavidad plvica, para controlar el sangrado contino de las superficies
peritoneales cuando ya se realiz la histerectoma y se inici una Coagulopata. Son retiradas 24 horas luego de corregida la Coagulopata. Embolizacin:
de la arteria uterina / iliaca interna: cuando las opciones quirrgicas se han agotado y existe la posibilidad de intervencin por radiologa.
CASO CLINICO CORIOAMNIONITIS
Acude a consulta paciente con 48 horas posparto, refiere que presenta
dolor en regin plvica de caractersticas punzante e intensidad
moderada a severa, agrega que presenta cefalea continua, malestar
generalizado, fatiga, adinamia, cuenta con antecedentes GO de gesta 3
para 3 abortos 0, refiere que presento flujo amarillento en varias
ocasiones en este ultimo embarazo, a la exploracin se observa TA
100/70, FC 89 FR 24, Temperatura de 38.5 grados, a la palpacin
refiere dolor, al tacto se identifica liquido serosanguinolento ftido y
tero con flacidez, decide ingresar a la paciente para manejo de la
complicacin posparto que presenta (corioamnionitis), a las 3 horas de
ingreso se presenta dificultad para respirar, a la auscultacin se
encuentra estertores bilaterales y ataque al estado generalizado, TA de
70 /50 FC 102, FR 31, Temperatura de 36.1 grados.

orientada, con buena coloracin de tegumentos, FC 95x, FR 22x, T/A


130/70mmHg, con dolor a la palpacin en hipogastrio y fosas iliacas de
leve a moderado, tero se palpa a nivel de cicatriz umbilical. En la
analtica en sangre destacaba una protena C reactiva de153 mg/L y
leucocitos 15.5200/mm3, con ligera desviacin a la izquierda, hto 45%,
glucosa 200mg/dl, Cr 0.9mg/dl. Pruebas de funcin heptica normales.

PREGUNTA
Considerando la gravedad del cuadro cual es la complicacin ms
probable en esta paciente?
RESPUESTA
a.- Tromboembolia pulmonar.
b.- Coagulacion intravascular diseminada.
c.- Sepsis abdominal.
d.- Choque distributivo.

PREGUNTA
Cul es la conducta ms adecuada a seguir en este momento?
RESPUESTA
a.- Antibitico emprico (cefuroxima), aminas vasoactivas
b.- Lquidos IV, toma de cultivos, antibioticoterapia emprica
(cefotaxima), control de glucemia
c.- Toma de cultivos, lquidos IV, antibiticos de acuerdo a cultivos
d.- Lquidos IV, valorar transfusin, antibioticoterapia emprica
(cefotaxima), control de glucemia

CASO CLINICO
Mujer de 33 aos de edad, con obesidad, quien llevo un control
prenatal inadecuado, la paciente nos refiere infecciones de vas
urinarias repetida. Acude a Urgencias a las 40 semanas de gestacin,
por presentar dinmica uterina y rotura prematura de membranas, se
decidi practicar una cesrea, por desproporcin cefaloplvica,
administrndose de profilaxis antibitica cefazolina endovenosa, se
obtuvo masculino sano de 4.050 g de peso al que la madre opt por
dar lactancia materna. A los 3 das present fiebre de hasta 38,5C y
dolor plvico, as como flujo blanco grisceo. EF; paciente consciente

CURSO ENARM CMN SIGLO XXI TEL: 36246001

PREGUNTA
Cul es el diagnstico ms probable en este momento?
RESPUESTA
a.- Sndrome de respuesta inflamatoria sistmica
b.- Sepsis puerperal leve
c.- Sepsis puerperal severa
d.- Shock sptico

PREGUNTA
Si le tomramos una hCG a esta paciente. En que da empezara a ser
negativa?
RESPUESTA
a.- 5-7 das pos cesrea
b.- 7-11 das pos cesrea
c.- 11-16 das pos cesrea
d.- 16-20 das pos cesrea

Pharmed Solutions Institute

PGINA 284

MANUAL DE TRABAJO DEL CURSO ENARM CMN SIGLO XXI


HEMORRAGIA OBSTETRICA
CIENCIAS BASICAS: Se define como aquel sangrado que se produce a travs de la vagina en cualquier momento del embarazo y cuya presencia no est
justificada. Durante las primeras 20 SDG el aborto, el embarazo ectpico y la mola hidatiforme, son las principales causas de sangrado transvaginal. Las
hemorragias que se presentan en el tercer trimestre del embarazo suelen ser todo un reto, pues estos sangrados son los que con mayor frecuencia
ponen en riesgo la vida de la madre y del feto. SALUD PUBLICA: Hemorragia obsttrica causa importante de morbimortalidad materna y perinatal. La
hemorragia obsttrica se ha identificado como la causa nica ms importante de decesos maternos en todo el mundo; explica casi la mitad de los
fallecimientos postparto. En Mxico en el 2005, las hemorragias durante el embarazo se
encuentran como la segunda causa de mortalidad materna con un porcentaje de 26.1%.
CLASIFICACION: 1) Hemorragias preparto: pueden deberse a embarazo ectpico, aborto
espontaneo, enfermedades del trofoblasto, lesiones vaginales y cervicales locales, trastornos de la
coagulacin, cncer. A la mitad del embarazo se deben: anomalas de la implantacin placentaria
(placenta previa), desprendimiento prematuro de placenta normoinserta (DPPNI), trabajo de
parto pre trmino y 2) Hemorragias posparto: trauma del canal del parto, desgarros cervicales,
atona uterina, inversin uterina, retencin de restos placentarios. PLACENTA ACRECTA. CIENCIAS
BASICAS: El acretismo placentario es la insercin anormal de parte o de toda la placenta, con
ausencia total o parcial de la decidua basal y anormalidad de la caduca verdadera con penetracin
de las vellosidades coriales al miometrio. Es una de las principales causas de hemorragia
obsttrica, principalmente del post parto inmediato, y que condiciona un importante riesgo
demorbimortalidad materna, por la patologa misma, como tambin por el tratamiento aplicado.
SALUD PUBLICA: La incidencia de placenta acreta est aumentando, especialmente por la mayor incidencia de operacin cesrea; el mayor riesgo de
morbimortalidad materna disminuye con un adecuado diagnstico prenatal y planificando la interrupcin del embarazo con un equipo quirrgico
multidisciplinario, en una institucin que disponga de banco de sangre. La edad media de las pacientes fue de 31 aos, y el grupo de mayor riesgo fue el
de 35 a 39 aos con 26% de acretismo placentario. CLASIFICACION: Existen tres variedades de adherencia anormalde la placenta: Placenta acreta: es la
adherencia anormal de una parte o de la totalidad de la placenta a la pared uterina, sin que las vellosidades coriales penetren el miometrio. Placenta
increta: las vellosidades coriales penetran el miometrio. Placenta percreta: es la penetracin de los elementos coriales hasta sobrepasar la serosa del
tero, pudiendo alcanzar rganos vecinos. Por su extensin se reconocen tres tipos: a) Focal: solo involucra pequeas reas de la placenta; b) Parcial:
uno o ms cotiledones se involucran en el proceso; c) Total: la superficie completa de la placenta esta anormalmente adherida. PATOGENIA: Los
principales factores de riesgo identificados para acretismo placentario de acuerdo a los antecedentes son: Edad (mayores de 30 aos), Gestas (multparas
de 2 a 3), Placenta previa con antecedente de cesrea (35%), Placenta previa con antecedente de 3 o ms cesreas (67%), Historia de legrado uterino (18
a 60%), Antecedente de extraccin manual placentaria. Historia de retencin placentaria. Antecedente de infeccin intramnitica. Se piensa que el
acretismo placentario es debido a una ausencia o deficiencia de la capa de Nitabuch, que es una capa de fibrina que se ubica entre la decidua basal y el
corion frondoso. Benirschke y Kaufman, sugieren que esta situacin es consecuencia de una falla en la reconstitucin del endometrio/decidua basal,
especialmente despus de la reparacin de una operacin cesrea. El acretismo placentario puede llevar a hemorragia obsttrica masiva, y como
consecuencia alteraciones de la coagulacin por prdida, y porcoagulacin intravascular diseminada. DIAGNOSTICO: Bsicamente el diagnstico se limita
al empleo de mtodos como es el ultrasonido y la resonancia magntica. Sin embargo, el diagnstico definitivo de acretismo placentario es por medio de
histopatologa, al comprobar la invasin de las vellosidades coriales en el miometrio. Ultrasonografa. El ultrasonido es una til herramienta para el
diagnstico de acretismo placentario as como de sus distintas variantes en el segundo y tercer trimestre del embarazo. Finberb y Williams, establecieron
en 1992, la utilidad de esta tcnica de imagen; entre los criterios diagnsticos establecidos fueron: prdida de la zona hipoecoica miometral
retroplacentaria, adelgazamiento o disrupcin de la serosa uterina hiperecoica y la interfaz con la vejiga, la presencia de masas exofticas, presencia de
una gran rea de sonolucencias placentarias. Alfa-feto protena fetal srica materna. Actualmente se ha propuesto que existe una anormalidad en la
interfaz placeta-tero, lo cual promueve una fuga de alfa-feto protena fetal hacia la circulacin materna. Histologa. Los criterios microscpicos para el
diagnstico de acretismo placentario es la ausencia de la decidua basal. Sin embargo su diagnstico no resulta del todo fcil por la distorsin que ocurre
en el momento del alumbramiento, salvo que se deje la pieza in situ y se realice una histerectoma y as se obtenga la muestra del tejido. En el caso de
placenta increta o percreta el diagnstico es ms fcil y consiste en la presencia de tejido trofoblstico en el espesor uterino o en la estructura invadida.
TRATAMIENTO: El principal tratamiento es la histerectoma obsttrica total abdominal, debido a su frecuente asociacin con placenta previa y cicatrices
de cesreas previas .Este procedimiento es un desafo a las habilidades quirrgicas del mdico obstetra, por lo que es necesario conocer otras
alternativas de tratamiento. Manejo conservador. Es una medida que consiste en evitar la histerectoma, al menos en ese momento, y tratar de
preservar la fertilidad. Existen diversas modalidades de manejo conservador,que incluyen: Dejar la placenta en su lugar (totalmente o en fragmentos).
Reseccin del lecho placentario y su reparacin. Extraccin y legrado obsttrico. Empleo de medicamentos asociados a cualquiera de los anteriores
puntos. Empleo de algn medio que cause isquemia (embolizacin, ligadura de vasos, etc.) del lecho placentario. El metotrexate es un medicamento
quimioterpico el cual se encuentra catalogado dentro del grupo de los antagonistas de los folatos. Su empleo en el acretismo placentario tiene como
fundamento la efectividad en contra del trofoblasto proliferativo. Sin embargo, de forma ms reciente se ha argumentado que despus del nacimiento
del feto la placenta detiene su divisin y por lo tanto pierde su utilidad.
CASO CLINICO
Paciente femenino, de 24 aos de edad, con dos embarazos, una
cesrea. La paciente desconoca la fecha de la ltima menstruacin,
por ultrasonido se determin embarazo de 16.3 semanas. No se
reportaron antecedentes patolgicos de importancia. Acude a
urgencias debido a dolor abdominal de 48 horas de evolucin, que se
acentuaba en la regin hipogstrica, ms vmito de contenido
gastrobiliar en cuatro ocasiones; con sntomas urinarios (disuria,
polaquiuria, tenesmo vesical), sin sangrado transvaginal. La TA 110/60
mmHg, FC 110 lpm y FR 20 por minuto. La paciente estaba consciente,
hidratada, con campos pulmonares con buena entrada y salida de aire,
con taquicardia y sin ruidos agregados. A la palpacin profunda del
abdomen se encontr sin datos de irritacin peritoneal, con fondo
uterino de 17 cm. A la especuloscopia el cuello uterino central estaba

CURSO ENARM CMN SIGLO XXI TEL: 36246001

cerrado, sin sangrado transvaginal. En el hospital, sbitamente la


paciente tuvo diaforesis, con aumento del dolor abdominal, tensin
arterial no audible, pulso ausente, FC 130 lpm. En el tamiz
ultrasonogrfico se le encontr lquido libre en la cavidad, con feto
nico, intrauterino, sin latido cardiaco, placenta fndica anterior. Los
estudios de laboratorio reportaron: leucocitos 13,950/mm3,
hemoglobina 8.09 g/dL, hematcrito 22.46, plaquetas 201,000/mm3
PREGUNTA
Con que porcentaje de sangrado seria irreversible el shock de la
paciente?
RESPUESTA
a.- >20%
b.- >35%

Pharmed Solutions Institute

PGINA 285

MANUAL DE TRABAJO DEL CURSO ENARM CMN SIGLO XXI


c.- >40%
d.- >45%

a.- Dehiscencia de cicatriz previa


b.- Multiparidad
c.- Estimulacion excesiva con oxitocicos
d.- Desproporcion cefalo-pelvica

PREGUNTA
Cul es la conducta ms adecuada seguir en este momento?
RESPUESTA
a.- Oxitocina
b.- Reposicin de volumen
c.- Ligadura de arteria uterina
d.- Histerectoma

CASO CLINICO

PREGUNTA
La paciente sin duda tendr que pasar a laparotoma. Qu medida
sera menos til en este caso para contener el sangrado y ayudar a
evitar la histerectoma?
a.- ligadura de arterias uterinas
b.- Baln de Bakri
c.- Ligadura de arterias ovricas
d.- Empaquetamiento
CASO CLINICO
Mujer de 28 aos de edad sin antecedentes de importancia, cursaba
con embarazo de 20 semanas, con diagnstico de tero didelfo e
implantacin placentaria en cavidad menor; embarazo de evolucin
normal. Inici el padecimiento el mismo da de su ingreso. Estuvo en
un temazcal y poste riormente recibi masaje, dos horas despus
inici el dolor abdominal sbito en mesogastrio e hipogastrio, con
nusea e incluso un vmito de contenido gastroalimentario. El dolor
aument de intensidad (hasta ser de 8 en una escala de 10) y
experiment salida de sangre transvaginal. Acudi al departamento de
urgencias donde se inici manejo para amenaza de aborto; se solicit
valoracin para ciruga por dolor abdominal y leucocitosis de
21000/ml. Exploracin fsica: estupor cardiopulmonar con frecuencia
cardiaca de 150 latidos/min, abdomen con franca disten sin, datos de
irritacin peritoneal y matidez generalizada. Presin arterial: 70-30
mmHg.
PREGUNTA
Cul es el diagnotico mas probable en este caso?
RESPUESTA
a.- Hipotonia uiterina
b.- Placenta percreta
b.- Rotura uterina
c.- Placenta acreta
PREGUNTA
Cul es la conducnta teraputica mas adecuada a seguir en este caso?
RESPUESTA
a.- Laparotoma exploradora
b.- Metotrexate
c.- Extraccion y legrado
d.- Embolizacion de uterinas
PREGUNTA
Cul es la incidencia de esta patologa?
RESPUESTA
a.- 1-2%
b.- 0.07-0.1%
c.- 3-5%
d.- 0.3-0.5%
PREGUNTA
Cul es la causa mas probable de esta patologa?
RESPUESTA

CURSO ENARM CMN SIGLO XXI TEL: 36246001

Mujer de 20 aos de edad con antecedente de aborto espon tneo un


ao atrs. Neg estar embarazada e incluso refiri ltima fecha
menstrual. Acudi a urgencias por dolor ab dominal sbito, en
mesogastrio, asociado con sensacin de necesidad de evacuar. Refiri
dos evacuaciones diarreicas y desvanecimiento. Lleg a urgencias con
frecuencia respi ratoria de 130 min y presin arterial de 90-60 mmHg.
Frecuencia cardiaca 100 por minuto, abdomen con resistencia
muscular, dolor a la palpacin superficial media y profunda, con datos
de irritacin peritoneal y peristalsis ausente.La tomografa de
abdomen mostr lquido libre en cavidad.
PREGUNTA
Cul es el diagnotico ms probable para este caso?
RESPUESTA
a.- Apendice perforada
b.- Rotura uterina
b.- Diverticulitis
d.- Cirrosis heptica
CASO CLINICO
Paciente de 31 aos, cuartigesta (1 aborto y 2 cesreas, la primera de
ellas por no progresin del parto y la segunda por presentacin
podlica y cesrea anterior). Sin antecedentes familiares de
importancia. Asmtica. El embarazo fue controlado ambulatoriamente.
Analtica normal. Se realizaron 5 ecografas acordes con amenorrea. En
la semana 15 se realiz amniocentesis diagnsti ca con resultado
normal, diagnosticndose en ese momento una placenta previa
oclusiva total. En las siguientes ecografas persiste el diagnstico. A las
29 semanas acude de urgencia por dinmica uterina que requiere
hospitalizacin y tratamiento tocoltico con ritodrine. A las 35 semanas
de amenorrea acude nuevamente de urgencia por dinmica uterina.
PREGUNTA
Cul es la conducta terapeutica mas adecuada a seguir en este
momento?
RESPUESTA
a.- Reposo y tocoliticos
b.- Laparotomia
c.- Evolucion a parto
d.- Histerectomia
PREGUNTA
Durante la extraccin fetal se objetiva sangrado importante, con
extraccin placentaria muy dificul tosa, siendo prcticamente
imposible su desp gamiento en la cara anterior, con la existencia de
tejido placentario invadiendo miometrio, y que se extiende a plica
vsico-uterina, cara posterior de vejiga y cara anterior de vagina,
pasando a anes tesia general y precisando mltiples transfusiones.
PREGUNTA
Cul es la conducnta a seguir en este momento?
RESPUESTA
a.- Histerectomia total
b.- Empaquetamiento
c.- Histerooforectomia
d.- Histeractomia en bloque

Pharmed Solutions Institute

PGINA 286

MANUAL DE TRABAJO DEL CURSO ENARM CMN SIGLO XXI


SINDROME DE TENSION PREMENSTRUAL.
CIENCIAS BASICAS: La American Psychiatric Associationlo denomina DDPM (desorden disfricopremenstrual) o DFL (desorden de fase ltea).Se
caracteriza por ser una entidad con sintomatologa somtica y afectiva (trastornos anmicos) que produce disfuncin e incapacidad significativa.
Posiblemente el DDPM no slo representa el extremo ms grave de la sintomatologa premenstrual sino quizs una modalidad de trastorno depresivo o
de ansiedad. Durante la vida "reproductiva" de la mujer, son frecuentes una serie de cambios tanto fsicos como psicolgicos (molimia premenstrual),
que acompaan a los distintos ciclos menstruales; tan slo, una pequea proporcin de mujeres (3-5%) llegan a presentar el llamado sndrome
premenstrual (SPM) definido como la recurrencia cclica de una combinacin de malestar fsico, psicolgico y/o cambios en la conducta presentes en la
fase lutenica (aumento de estrgenos y progesterona) de la mayora de los ciclos, que remiten al final de la menstruacin, con un intervalo libre de
sntomas de al menos una semana por ciclo y de tal severidad que producen un deterioro de las relaciones interpersonales y/o interfieren con otras
actividades cotidianas. SALUD PUBLICA: 3-10% de las mujeres en edad reproductiva son asintomticas. 30-90% presentan sntomas premenstruales
leves. 20-30% presentan sntomas moderados a graves que alteran su forma de vida. 3-8% padecen sntomas graves e invalidantes. Las causas por las
que las mujeres solicitaban ayuda mdica eran la depresin severa, ftiga y disminucin de la concentracin. PATOGENIA: En la actualidad es
desconocida; se han propuesto tanto biolgicas como psicolgicas para explicar el origen del SPM. A) Hiptesis biolgicas: postulan que los sntomas
pueden surgir como resultado de alteraciones neuropsicolgicas causadas por la accin a nivel del SNC de esteroides gonadales, neurotransmisores o
sustancias neuromoduladoras. Ciertos hallazgos fisiolgicos apoyan el origen orgnico-biolgico: Alteraciones de la conductividad cutnea. Cambios a
nivel del EEG ciclo-dependiente, con aparicin de ondas alfa y aumento de la duracin de la fase REM durante el sueo, siendo directamente
proporcional a la severidad de los sntomas. Otros hallazgos en el EOG (Electrooculograma). Se ha intentado relacionar cambios hormonales con
alteraciones en las monoaminas (5HT, NA), centrndose en algunos trabajos en las alteraciones del sueo y de la termoregulacin (aumento de la
temperatura corporal nocturna). Se sabe que la serotonina (5HT) disminuye los niveles de estrgenos y de progesterona al inhibir la LHRH, y que la
noradrenalina (NA) modula los receptores de progesterona situados en los ncleos septales y en la amgdala (implicados en la termorregulacin,
conducta sexual y emociones). Algunos autores han encontrado otros hallazgos como: Alteraciones en los niveles de prolactina, aumento de los niveles
de prostaglandinas (PG), disminucin de los niveles de PG, disminucin de los niveles de Piridoxina, B) Hiptesis psicolgicas: El SPM es ms frecuente en
mujeres con un elevado nivel de neuroticismo, que a su vez muestran un pobre ajuste psicolgico y una gran sensibilidad a la frustracin. Dada la
frecuencia de sntomas afectivos (disforia, ansiedad e irritabilidad) y la ciclicidad inherente en su definicin, se ha hecho hincapi en la relacin de este
cuadro y de otros trastornos psiquitricos (ciclotimia y distimia). En este sentido, se han dado tres teoras bsicas que intentan relacionarlos: 1.Episodios afectivos tempranos, anteriores a la menarqua, que determinaran las caractersticas de los cambios del humor posteriores (menstruales). 2.Cambios en el humor menstrual pueden servir como desencadenante de otras patologas
afectivas mayores en personas predispuestas genticamente. 3.- O la teora de la
"indefensin aprendida. DIAGNOSTICO: Las pacientes informan 1 ms de los siguientes
sntomas afectivos o somticos durante los 5 das anteriores a la menstruacin durante los
ltimos 3 ciclos. Los sntomas mejoran dentro de los 4 das de ocurrida la menstruacin, sin
recurrencia por lo menos hasta el da 13 del ciclo. Irritabilidad (85%), angustia (83%) y
labilidad emocional (77%) son los sntomas ms reportados. Segn Sampson (1989) deben cumplirse algunas caractersticas para su diagnstico:
Periodicidad de los sntomas, aumento de la gravedad en la medida que el ciclo progresa, existencia de un perodo postmenstrual de 7 das, como
mnimo, libre de sntomas, alivio al inicio de la menstruacin y desaparicin al cabo de 3 das posteriores al comienzo, presencia de los sntomas durante
3 ciclos consecutivos como mnimo, con posibles variaciones de intensidad, interferencia de los sntomas con el trabajo, las actividades sociales y las
relaciones interpersonales. Criterios diagnsticos del SPM o del Trastorno Disfrico Premenstrual (son criterios de investigacin)del DSM: A) Cinco (o
ms) de los siguientes sntomas durante la mayor parte del da de la ltima semana de la fase ltea de la mayora de los ciclos menstruales del ltimo
ao, que empiezan a remitir 2 das despus del inicio de la fase folicular y que desaparecen completamente en la semana siguiente a la menstruacin,
teniendo en cuenta que al menos uno de estos sntomas debe ser alguno de los cuatro primeros: 1) estado de nimo deprimido, sentimientos de
desesperanza e ideas de autodesaprobacin acusadas; 2) ansiedad, tensin, sensacin de agobio o de estar "al limite"; 3) labilidad emocional evidente;
4) enfado, irritabilidad o aumento de los conflictos interpersonales de forma acusada y persistente; 5) prdida del inters por las actividades cotidianas;
6) sensacin subjetiva de dificultad de concentrarse; 7) letargia, fatigabilidad fcil o falta evidente de energa; 8) cambios significativos del apetito,
atracones o antojos por determinadas comidas; 9) hipersomnia o insomnio; 10) sensacin subjetiva de estar rebasada o fuera de control; 11) otros
sntomas fsicos como hipersensibilidad o aumento del tamao mamario, dolores de cabeza, molestias articulares o musculares, sensacin de hinchazn
o ganancia de peso. B) Estas alteraciones interfieren acusadamente con el trabajo, la escuela, las actividades sociales habituales o las relaciones
interpersonales. C) La alteracin no representa una simple exarcebacin de sntomas de otro trastorno. D) Los criterios A, B y C deben ser corroborados
por tcnicas de valoracin diaria y prospectiva de los sntomas en al menos dos ciclos sintomticos consecutivos. TRATAMIENTO: El objetivo principal del
tratamiento es disminuir los sntomas de SPM o DDPM para que la paciente pueda realizar una vida normal, lo cual incluye entre otras medidas:
Modificaciones de los hbitos alimentarios (Nivel B), vitaminoterapia, frmacos, psicoterapia (Nivel C). Educacin: es importante que la mujer conozca la
fisiologa del ciclo menstrual para comprender lo que le est ocurriendo. El ACOG recomienda los cambios en el estilo de vida como la 1ra lnea de
tratamiento. Dieta: Se recomienda restringir el consumo de: Sal (3g/da) para reducir la retencin de agua, azcares refinados, alimentos con
Metilxantinas para no aumentar la ansiedad y la irritabilidad, tabaco y alcohol, carnes rojas, se recomienda incrementar la ingesta de: Carbohidratos
complejos, pescados y aves como fuente de protenas, fibras, vegetales, legumbres y cereales. Ejercicio fsico: Aerbicos: caminata, trote, natacin,
remo, ciclismo y danza (nivel C), meditacin, yoga. Es controvertido y debe reservarse a las pacientes que no mejoran con las medidas mencionadas.
Vitaminoterapia: Vitamina B6, la administracin diaria de 50 100 mg podra tener efectos beneficiosos sobre los sntomas depresivos.(nivel B).
Vitamina E: 400 a 600 UI/da. Mineraloterapia: El calcio y el magnesio son cofactores en la sntesis de neurotransmisores, lo que sugiere una posible
asociacin entre SPM y la deficiencia de serotonina (nivel B). AINE Mejoran la mastalgia, la dismenorrea, la migraa y los dolores musculares.
Anticonceptivos Hormonales: Se ha sugerido un efecto beneficioso sobre el bienestar psicolgico y se los ofrece como una buena alternativa para el
tratamiento (nivel B). FDA aprob un esquema teraputico de ACO con bajas dosis de EE y DRSP con 24 comprimidos activos y 4 placebos para los
sntomas fsicos y emocionales, mediantela reduccin de la fluctuacin hormonal a lo largo del ciclo y las propiedades antiandrognicas y
antimineralocorticoideas. Joffe y colaboradores informaron que los ACO no modifican los cambios de humor premenstrual, pero, en aquellas mujeres
con antecedentes de depresin, tienen un gran riesgo de empeorar estos sntomas. Agonistas de GnRH: Producen alivio significativo de las
manifestaciones de la enfermedad, pero inducen sntomas menopusicos que limitan su uso en la mayora de las pacientes (nivel B). Inhibidores de las
Gonadotrofinas: El danazol parece ser efectivo en el tratamiento del SPM, especialmente en el alivio de la mastalgia (por accin directa) y de la
irritabilidad. No se recomienda en pacientes con depresin y ansiedad. La dosis propuesta es de 200 mg dos veces por semana, slo en la fase ltea.
Gestgenos: No se pudo demostrar su efectividad para el tratamiento del SPM, no se recomienda su uso (nivel A). Diurticos: Las candidatas para el uso
de diurticos son aquellas pacientes que manifiestan aumento de 1.5 kg de peso durante el perodo premenstrual, tensin mamaria y otros sntomas de

CURSO ENARM CMN SIGLO XXI TEL: 36246001

Pharmed Solutions Institute

PGINA 287

MANUAL DE TRABAJO DEL CURSO ENARM CMN SIGLO XXI


retencin hdrica. La espironolactona es el diurtico recomendado, ya que es un agonista de la aldosterona. La dosis es de 25 mg, 2-3 veces por da,
durante los das 18 al 26 del ciclo. Inhibidores Selectivos de la Recaptacin de Serotonina (ISRS): Primera lnea para el tratamiento del DDPM, pues son
muy eficaces para el alivio de los sntomas fsicos y emocionales (nivel A).Fluoxetina, sertralina, citalopram, paroxetina) actan sobre el humor, la
saciedad y la agresividad.Son eficaces cuando se usan en forma continua o slo en fase premenstrual.
CASO CLINICO SINDROME DE TENSION PREMENSTRUAL
Se trata de femenino de 21 aos de edad la cual acude a consulta
debido a que refiere mastalgia y sensacin de plenitud mamaria,
irritabilidad, inestabilidad emocional, refiere que desde los 17 aos se
presento ms intensos los sntomas, agrega dismenorrea ocacional con
rechazo al periodo menstrual, al interrogatorio usted observa
irregularidades menstruales, la paciente presenta un IMC de 18.
PREGUNTA
Cul es la conducta ms adecuada para este caso?
RESPUESTA
a.- La conducta mas adecuada es iniciar un esquema de contraceptivo
oral.
b.- Iniciar un Inhibidor Selectivo de la Rescaptura de Serotonina.
c.- Indicar la calendarizacin de la sintomatologa previa a la terapia.
d.- Derivar a la paciente al servicio de psiquiatra.
PREGUNTA
Cul es la conducnta teraputica mas adecuada a seguir para el
manejo de la inflamacin?
RESPUESTA
a.- Ac. Mefenamico
b.- Ibuprofeno
c.- Diclofenaco
d.- Acido acetilsalicilico
CASO CLINICO
Paciente de 39 aos, madre de dos hijos, primer parto con cesrea
por falta de dilatacin, el segundo con frceps. Sus molestias haban

CURSO ENARM CMN SIGLO XXI TEL: 36246001

comenzado siete aos antes, en el perodo premenstrual,


manifestndose como dolores de cabeza. Experimentaron con el
tiempo progresiva prolongacin e intensificacin. Por la jaqueca
consult a neurlogo, quien pidi un electroencefalograma, que
result normal. Le fueron recetados analgsicos. Se presentaron
molestias mamarias que se hicieron francamente severas y
duraban casi todo el mes. Tuvo diagnstico de Mastopata Fibrosa
Bilateral de grado moderado. Fue tratada con un derivado sinttico de
la progesterona durante diez das cada mes; hubo alivio parcial de las
molestias mamarias. Por otra parte, se le diagnostic "lcera al
cuello del tero", lo que se trat con dos cauterizaciones, sin
proporcionar ningn beneficio a nivel general.
La paciente
manifestaba sufrir molestias en las relaciones sexuales y adems
frigidez, razn por la cual procuraba evitarlas. Sus molestias generales
persistan y los sntomas mamarios se acentuaron. Al exmen fsico se
observaba una paciente decada, palidez de piel y mucosas, lengua
sucia, conjuntivas irritadas, mirada triste. Adems distensin
abdominal, edema (hinchazn) de manos y prpados, piel y cabello
secos. Las mamas presentaban aumento de volumen, congestin,
sensibilidad, ardor y aumento de la red venosa.
PREGUNTA
Cual es la ducta a seguir mas adecuada?
RESPUESTA
a.- Acido mefenamico.
b.- Ibuprofeson.
c.- Progesterona
d.- Citalopram

Pharmed Solutions Institute

PGINA 288

MANUAL DE TRABAJO DEL CURSO ENARM CMN SIGLO XXI


PSICOSIS Y DEPRESION POSPARTO
CIENCIAS BASICAS: La psicosis posparto o puereral se define como un episodio de mana o de psicosis precipitado por el nacimiento de un beb: Es una
enfermedad severa cuyas caractersticas clnicas incluyen todas las variantes de la mana, psicosis cicloides o esquizofreniformes. Se caracteriza por
prdida de contacto con la realidad, agitacin, confusin, alucinaciones vividas, delirios paranoides y comportamiento violento. Muy excepcionalmente
se ha descrito suicidio e infanticidio. SALUD PUBLICA: El riesgo general es de 1-2 casos por cada 1.000 nuevos nacimientos. Se suelen describir algunos
factores de riesgo. As, se ha comprobado que el 50-60% de los casos ocurren en primparas, y que en el 50% de los casos se asocian a complicaciones
psiquitricas perinatales. En cuanto a los antecedentes familiares, un 50-60% de las mujeres afectadas tienen antecedentes familiares con trastornos
afectivos. El riesgo est muy aumentado en las mujeres que tienen antecedentes personales de trastornos del humor unipolares o bipolares
(especialmente un trastorno bipolar I). En estos pacientes el riesgo pasa de 2/1000 al 20 a 25%. Las enfermedades psiquitricas del posparto son 5 veces
ms frecuentes que las del embarazo, de ah la hiptesis del papel protector del embarazo frente a las enfermedades mentales graves. Esta patologa
psiquitrica del posparto debuta en la primera semana que sigue al parto en el 40% de los casos, y en el primer mes postparto en el 80% de los casos.
PATOGENIA: Los factores que pueden influir en el inicio de un cuadro de las caractersticas que nos ocupa se pueden agrupar en: 1.- Factores heredoconstitucionales: Se insiste en el papel del "terreno", siendo el grupo de familia con historia de bipolaridad el que ofrece las relaciones genticas ms
significativas. Cuando una mujer ha presentado crisis manaco depresivas, existe un elevado porcentaje de riesgo de recada despus del parto. 2.Factores biolgicos: Los trastornos endocrinos que aparecen antes, durante e inmediatamente despus del parto son de una amplitud y rapidez nicas.
En efecto, en los das siguientes al parto las concentraciones plasmticas de estrgenos y progesterona caen de manera considerable para alcanzar tasas
100 veces ms bajas que aquellas que existen en el preparto. 3.- Factores psicodinmicos: autores hacen intervenir el rechazo del nio como uno de los
factores esenciales de las melancolas puerperales: rechazo
consciente (nio no deseado) o ms a menudo inconsciente, que
emana de la ambivalencia profunda que marca la relacin madre e
hijo, "rechazo combatido tanto por la culpabilidad como por el
amor maternal. Entre los factores psicosociales figura, en primer
trmino, el conflicto provocado por la aparicin de una tercera
persona en el grupo familiar. La madre tiene que asumir ahora una
mayor responsabilidad. Tiene que dejar de ser hija para ser ella, a
su vez, madre. DIAGNOSTICO: Clinico; Las psicosis puerperales se
caracterizan por depresin e ideas delirantes que aparecen
despus del parto. El estado de nimo puede estar elevado o
deprimido y con frecuencia es lbil; puede tambin haber
agitacin, aumento de la actividad e insomnio. La enferma puede
ver al recin nacido como la personificacin del diablo o como el
Mesas; tambin pueden existir alucinaciones auditivas que pueden ser rdenes, en ocasiones para que la paciente mate al recin nacido. Pueden
producirse infanticidios. El cuadro clnico tiene unas caractersticas especficas: a) Rpido cambio de sintomatologa, b) Labilidad de humor, c) Signos
confusionales. Los sntomas iniciales ms frecuentes son ansiedad, cansancio, humor depresivo, disturbios del sueo, alteraciones del comportamiento,
agitacin catatnica, ilusiones y alucinaciones. El comienzo ocurre en un perodo muy poco tiempo despus del parto y existe una serie de
precisiones:Deben ser excluidas las psicosis asociadas con eclampsia, infeccin, deprivacin de alcohol o con problemas del puerperio. Los pacientes con
trastornos delirantes monosintomticos y con psicosis psicgena deben ser excluidos. Si se decide incluir como tal a las psicosis depresivas, debieran
restringirse a aquellas que presentan delirios, alucinaciones, confusin, estupor, catatona o sntomas maniformes transitorios. El intervalo entre el parto
y la aparicin de la psicosis puede ser de 14 das, un mes o tres meses. No es necesario que la madre est libre de patologa psiquitrica durante el
embarazo. El DSMIV no contiene criterios diagnsticos especficos para este trastorno. El diagnstico puede hacerse cuando la psicosis ocurre en
estrecha relacin temporal con el nacimiento del nio. Los sntomas caractersticos son ideas delirantes, dficits cognoscitivos, alteraciones motoras,
alteraciones del estado de nimo y, de forma ocasional, alucinaciones. El contenido del material psictico hace referencia al embarazo y la maternidad.
TRATAMIENTO: Los tratamientos farmacolgicos son comparables a aquellos utilizados para los sndromes similares que aparecen fuera del contexto del
posparto. Al comienzo, es necesario un tratamiento farmacolgico enrgico, y en la mayora de los casos, se recomienda la hospitalizacin. Dependiendo
de cada mujer, suelen prescribirse antipsicticos y estabilizadores del nimo. Por otra parte, se tiende a utilizar benzodiacepinas para controlar los
episodios de agitacin y de ansiedad. Si la paciente no responde favorablemente a la farmacoterapia o los sntomas van en aumento, puede ser
necesaria la terapia electro convulsiva. Un estudio demuestro que las fenotiazinas son ms utilizadas en las mujeres que presentan una psicosis del
posparto. La psicoterapia debe estar centrada en las reas particularmente conflictivas. Hay que ayudar a la paciente a aceptar su rol femenino y la rabia
y los celos que surgen a raz de la dependencia que tiene respecto de su madre. Va de suyo la necesidad de tener en cuenta al grupo familiar entero.
Tambin puede estar indicado el introducir algunos cambios en la estructura del hogar. DEPRESION POSTPARTO (DPP): CIENCIAS BASICAS: Dentro de la
nomenclatura psiquitrica, la DPP es definida rigurosamente como un trastorno/episodio depresivo de inicio en el postparto. En el Manual Diagnstico y
Estadstico de los Trastornos Mentales (DSM IV-TR) la clasificacin es la siguiente: F32.x Trastorno depresivo mayor, episodio nico (296.2x). SALUD
PUBLICA: Existen diferentes investigaciones a nivel mundial sobre la prevalencia de este padecimiento, las cuales fluctan entre el 6 y 34%. En nuestro
pas, los estudios reportan una prevalencia que va desde 13.9 hasta 32.6%. A pesar de esta situacin, se podra hablar de que cerca del 20% de las
mujeres que dan a luz pueden presentar sntomas de DPP. En cuanto a la recurrencia, existe una probabilidad del 10 al 35%. PATOGENIA: Hasta el
momento, no se conocen las causas de la depresin postparto (DPP); nicamente se han podido observar factores de riesgo, este padecimiento se puede
presentar incluso cuando la madre no ha mostrado ningn antecedente. Factores de riesgo: Historia pasada de psicopatologa (especialmente depresin
y ansiedad), presencia de alteraciones psicolgicas durante el embarazo, DPP previa, complicaciones obsttricas durante el embarazo, el parto o despus
del mismo (principalmente embarazos de alto riesgo), embarazo no planeado y/o deseado, falta de apoyo familiar y/o social, problemas familiares y/o
sociales, pobre relacin marital, abandono o separacin de la pareja, no contar con un compaero, pareja presente, pero que no proporcione apoyo
emocional, falta de redes de apoyo, situaciones estresantes, dicultades econmicas, abuso fsico, emocional o sexual en la infancia, fallecimiento,
abandono o separacin de un familiar o ser querido, mala relacin con la madre, antecedente de prdidas perinatales, sensibilidad a los cambios
hormonales, alteracin en la funcin tiroidea, dicultades en la lactancia, mala salud del recin nacido, beb difcil de cuidar. Suele comenzar en
cualquier momento posterior al parto, ya sea das, semanas, meses y hasta un ao despus. Puede afectar en cualquier tipo de parto o nmero de
embarazo. DIAGNOSTICO: En relacin a la intervencin, el primer paso es detectarla, especialmente en sus inicios, cuando se perciban sntomas o se
presenten signos de alarma, como quejas psicosomticas, cansancio, dolores, mltiples llamadas para pedir ayuda, o se observe que la mujer est triste,
preocupada o con miedo. A continuacin, es necesario canalizarla con un especialista (psiclogo, terapeuta o psiquiatra) para que sea evaluada y reciba
el tratamiento adecuado, buscando el apoyo multidisciplinario y una atencin integral. Sntomas depresin posparto: Estado de nimo deprimido,

CURSO ENARM CMN SIGLO XXI TEL: 36246001

Pharmed Solutions Institute

PGINA 289

MANUAL DE TRABAJO DEL CURSO ENARM CMN SIGLO XXI


tristeza o llanto persistentes, disminucin del inters o de la capacidad para disfrutar de casi todas las actividades, cambios en los patrones de
alimentacin y sueo (principalmente insomnio), cansancio o fatiga crnica, ansiedad que puede llegar hasta los ataques de pnico, sentirse abrumada,
indefensa e incapaz, autodesvalorizacin y autorreproches, generalmente relacionados a su competencia como madre, dicultad, reticencia o
imposibilidad de buscar ayuda y apoyo para s misma, irritabilidad, problemas de atencin, concentracin y memoria, dicultad para vincularse con el
beb, sentimientos de ambivalencia hacia el hijo, sobreproteccin del menor, dicultades en la lactancia, dicultad y/o imposibilidad de cuidar al recin
nacido, pensamientos obsesivos y conductas compulsivas, relacionados con el hijo (p. ej. revisarlo constantemente cuando est dormido para ver si
respira) y con su cuidado (p. ej. lavar y desinfectar varias veces sus artculos), pensamientos negativos hacia el beb, especialmente creer que lo puede
daar, sentimientos de culpa, pensamientos de daarse a s misma o al beb, pensamientos atemorizantes, ideacin suicida. TRATAMIENTO: El
tratamiento tiene dos opciones de intervencin teraputica: 1) terapia (individual o de grupo) y/o 2) medicacin. En aquellos casos en que se presenta
una depresin de moderada a grave, lo ideal es combinar ambas. La psicoterapia, ya sea interpersonal o con un enfoque cognitivo-conductual, debe
basarse en los siguientes aspectos: a) Educacin e informacin a la mujer, la pareja y los familiares sobre la DPP. b) Control, disminucin y extincin de
los sntomas. c) El nuevo rol como madre, las dificultades, mie dos o angustias que esto puede generar. d) Uso de estrategias para proteger, mejorar o
fortalecer el vnculo madre-hijo, as como la relacin de pareja. En esta ltima, se buscar una adecuada comunicacin, as como tener espacios de
calidad sin el beb. e) Promover un buen funcionamiento familiar, especialmente cuando existan otros hijos. f) Disminucin de las situaciones
estresantes actuales. g) Bsqueda y fortalecimiento de redes de apoyo (familiares, sociales e institucionales). Tratamiento medico: En cuanto al
tratamiento psiquitrico, dependiendo de cada caso, los medicamentos ms usados son los antidepresivos tricclicos y los inhibidores selectivos de la
recaptacin de serotonina (ISRS), para combatir los sntomas depresivos. Por otra parte, se tiende a utilizar benzodiacepinas para tratar la ansiedad y el
insomnio. Si la mujer tiene una historia previa de DPP, lo ms recomendable es iniciar la medicacin inmediatamente despus del parto, para prevenir
una posible recaida. Cuando la mujer con DPP no recibe tratamiento, puede presentar dificultades y estrs para cuidar al recin nacido, lo que tiende a
generar problemas en el vnculo materno-infantil. Dichos bebs pueden llegar a presentar retraso en el desarrollo psicomotor, temperamento difcil o
irritabilidad, as como mostrar un apego inseguro. As mismo, se ha observado que algunos no son tan atentos y activos, y sonren menos, lo que puede
generar dficits en el rea social y cognitiva. Con una intervencin adecuada, la DPP es temporal y se puede alcanzar una completa recuperacin.
CASO CLINICO
Femenino de 28 aos, casada desde haca cinco aos, estaba
esperando el nacimiento de su primer hijo. Nunca haba tenido ningn
problema emocional particular y haba estado siempre fsicamente
bien. El parto de una nia de 3,2 kg, saludable y sin complicaciones,
dej cansada pero feliz durante los tres das siguientes a la paciente.
Dej el hospital con nimos y todo fue bien durante la primera semana
en casa. Repentinamente, se puso ansiosa y no era capaz de estar
quieta. Empez a pensar que alguien intentaba hacer dao al beb.
Reconoca que era ridculo pero no poda parar de pensar que alguien
estaba intentando envenenar al beb o entrar en la casa. Empez a
preocupar al marido al decir cosas extraas, como que haban tenido
nios gemelos, en vez de una nica nia. Algunas veces se encontraba
normal, mientras que otras, estaba inquieta y preocupada con sus
ideas extraas. Durante la semana siguiente se encontr tan mal que
no pudo hacerse cargo del beb y en ocasiones, no poda ni levantarse
por las maanas. Se senta culpable de todo, no poda dormir por las
noches y no tena apetito. Con frecuencia, estaba confundida, no
sabiendo dnde estaba o lo que le estaba pasando.
PREGUNTA
Cual es la conducta farmacolgica mas adecuada?
RESPUESTA
a.- Clorimipramina.
b.- Risperidona.
c.- Clonacepam.
d.- Valproato de sodio.
CASO CLINICO
Femenino de 27 aos, Un mes antes de la internacin abandon a sus
hijos y a su marido porque no los reconoca como propios. El esposo
refiere que not 30 das despus del parto que la paciente comenzaba
a desatender a sus hijos, hablaba en lenguas extraas, no dorma, se
enojaba con facilidad y cantaba a los gritos durante todo el da
cnticos religiosos. En la misma poca comienzan las huidas del hogar.
Examen al ingreso: Desconfiada, alucinada, escucha la voz de un
hombre que dice cosas feas. Refiere sentir dolores de cabeza y de
vientre, sensacin que vivencia como una agresin de ese mismo
hombre. Reticente. Pueril. Con conductas impulsivas de huida. Durante
la internacin la paciente est excitada, con desorientacin y falsos
reconocimientos. Por momentos permanece sin ningn contacto con
el entorno y al rato est atenta y dispuesta al dilogo, persisten las
alucinaciones. Logorreica, con logoclonas, responde a todo estmulo
del ambiente, lee lminas de las paredes, toca historias clnicas, quita
hojas de las plantas, intensa actividad motora sin objeto. Insomne,

CURSO ENARM CMN SIGLO XXI TEL: 36246001

canta o grita de da y de noche. La afectividad est polarizada hacia el


placer con estados anmicos fluctuantes. Escaso capital ideativo.
Ocurrencias delirantes polimorfas, msticas, erticas y megalmanas.
Cree ser la Virgen Mara, tiene falsos reconocimientos con el personal
del servicio mdico.
PREGUNTA
Se le ingreso e inicio tratamiento antipsicoticco, a las 48 aos refiere
acatisia y rigidez, cual es la conducta ha seguir mas adecuada?
RESPUESTA
a.- Clonacepam.
b.- Trihexifenidilo.
c.- Biperiden.
d.- Difenidramina
CASO CLINICO
Paciente femenina de 36 aos de edad, con 5 dia de puerperio, parto
normal, producto a trmino, primer hijo, madre soltera, que vive con
su propia madre nicamente, la cual es divorciada desde hace 20 aos.
La madre de la paciente la lleva a consulta por presentar conductas
extraas, refiere que no se ha baado desde que salio del hospital, la
observa aptica, callada, poco interesada en el hijo, y finalmente
decidio ir a consulta al escuchar a su hija decir tal vez estaramos
mejor si no estubieramos a la EF se observa paciente desalineada,
desaseada, poco cooperadora, desinteresada, niega ideas suicidas ni
homicidas pero acepta haber dicho lo que la madre refiere, se apresia
melancolica mas que ansiosa.
PREGUNTA
Cual es la conducta a seguir mas adecuada?
RESPUESTA
a.- Ingreso psiquitrico involuntario.
b.- Envio a consulta externa de psiquiatra.
c.- Imipramina 50 mg cada 8 hrs.
d.- Haloperidol 5 mg cada 8 hrs
PREGUNTA
Cual de los siguientes antidepresivos es el mas adecuado
considerando la sintomatologa del caso?
RESPUESTA
a.- Velanfaxina.
b.- Paroxetina.
c.- Escitalopram.
d.- Fluvoxamina

Pharmed Solutions Institute

PGINA 290

MANUAL DE TRABAJO DEL CURSO ENARM CMN SIGLO XXI

PEDIATRIA
1)
2)
3)
4)
5)
6)
7)
8)
9)
10)
11)
12)
13)
14)
15)
16)
17)
18)
19)
20)
21)
22)
23)
24)
25)
26)
27)
28)
29)
30)
31)
32)
33)

SX DE ADAPTACION PULMONAR, TAQUIPNEA TRANSITORIA DEL RN, ASPIRACION POR MECONIO


SX DE DIFICULTAD RESPIRATORIA TIPO I (EMH), NEUMONIA INTRAUTERINA Y NEONATAL
SEPSIS NEONATAL, ENTEROCOLITIS NECOTRIZANTE, MENINGITIS NEONATAL
HIPOGLUCEMIA, HIPERBILIRRUBINEMIAS, ENCEFALOPATA HIPERBILIRRUBINEMICA
ATRESIA ESOFAGICA, FISTULA TRAQUEOESOFAGICA, HERNIA HIATAL, ESTENOSIS PILORICA
CARDIOPATIAS CONGENITAS Y ADQUIRIDAS
KWASHIORKOR, MARASMO, AVITAMINOSIS
OBESIDAD, VACUNACION, CONTROL DEL NIO SANO
DERMATITIS DE CONTACTO, DE PAAL, SEBORREICA, HERPETIFORME
IMPETIGO, ERISIPELA, PIODERMA, TB CUTANEA, VITILIGO
HERPES SIMPLE, ZOSTER, VERRUGAS VIRALES, MOLUSCO
FOLICULITIS, PEDICULOSIS, ESCABIASIS, HIPERQUERATOSIS HIPEDERMOLITICA
TIAS, PITIRIASIS VERSICOLOR, CANDIDIASIS, MICETOMA
EXANTEMATICAS, HEPATITIS
TORCH, PSORIASIS, ACNE
URTICARIA, PRURIGOS, ERITEMA PALMAR, ERITEMA NODOSO
PURPURA DE SCHNLEIN-HENOCH, ANEMIAS
TUMORES DEL SNC, MENINGITIS
LEUCEMIAS, RETINOBLASTOMA, INFLUENZA
OTITIS, SINUSITIS, FARINGITIS, AMIGDALITIS
EPIGLOTITIS, LARINGOTRAQUEITIS, BRONQUITIS
BRONCONEUMONIA, NEUMONIAS, TUBECULOSIS PULMONAR
ALERGIAS, ASMA, FIBROSIS QUISTICA
URETRITIS, CISTITIS, PIELONEFRITIS, IVU
SINDROME NEFRITICO, SINDROME NEFROTICO.
GASTROENTERITIS, PARASITOSIS, ERGE
DESHIDRATACION, LIQUIDOS Y ELECTROLITOS
QUEMADURAS, ACCIDENTES
PICADURAS, MORDEDURAS
INTOXICACIONES, ENVENENAMIENTOS
TRAUMATISMO CRANEOENCEFALICO
CEFALEA Y EPILEPSIAS
ENURESIS, ENCOPRESIS, TDAH, TRASTORNOS DEL DESARROLLO

CURSO ENARM CMN SIGLO XXI TEL: 36246001

Pharmed Solutions Institute

PGINA 291

MANUAL DE TRABAJO DEL CURSO ENARM CMN SIGLO XXI


SNDROME DE ADAPTACIN PULMONAR (SAP):
CIENCIAS BASICAS: Los cambios que implican adaptarse a la nueva vida se producen de manera simultnea desde el nacimiento con la primera
respiracin y terminan aproximadamente a las 15hrs de vida, denominndose perodo de transicin. Involucra cambios cardiorrespiratorios,
metablicos, neurolgicos y hormonales. Los componentes del perodo de transicin son: Esfuerzo respiratorio inicial, aumento unas cinco veces en
el flujo linftico pulmonar, depuracin del lquido pulmonar desde la va area a los canales linfticos y vasculares pulmonares, establecimiento de
un rea de superficie pulmonar estable, reduccin de los altos niveles de resistencia vascular pulmonar, aumento del flujo sanguneo pulmonar,
aumento de la presin de oxgeno arterial, cierre del conducto arterioso. La eliminacin del lquido pulmonar se lleva a cabo en 2-6hrs. Signos vitales
normales: FR: 40-60 rpm, FC: 100-180 lpm si esta despierto y 70-80 al estar dormido, temperatura de piel 36-36.5C, rectal 36.5-37.5. PRIMER
PERODO DE REACTIVIDAD: 30-60 minutos de vida. Movimientos de succin y deglucin. Temblores finos en extremidades y mandbula. Cierre y
apertura de los prpados. Movimientos espasmdicos rpidos y breves de los globos oculares. Reflejo espontneo de Moro. Llanto de inicio y
detencin sbita. Cianosis leve y breve. Enrojecimiento con el llanto. Abundantes secreciones en vas areas. FR 100x, FC 200, quejidos
inspiratorios. Estertores bilaterales. TA elevada. Peristalsis ausente. Hipotermia. PERODO DE SUEO O TRANQUILIDAD: Despus de los 30-60
minutos, dura hasta las 3 horas de vida. Puede permanecer dormido, pero responde a estmulos en forma brusca. FR es rpida y superficial.
Desaparecen los quejidos inspiratorios y estertores. FC regular 120-140x, desciende 100 por breves perodos. TA se normaliza. Aparece la peristalsis.
Se observan ondas peristlticas. Temblores y sacudidas espontneas. SEGUNDO PERODO DE REACTIVIDAD: Ocurre entre las 2-6hrs de vida, es
ESCALA DE SILVERMAN- ANDERSON
breve o persiste por 15hrs. Se reanuda la actividad motora,
SIGNOS
0
1
2
exagerada. Inicia nuevamente taquicardia, polipnea. Respiraciones
Quejido espiratorio
Ausente
Aduble con estetoscopio
Audible sin estetoscopio
peridicas. Secreciones abundantes. Reflejo nauseoso, vmitos,
Respiracion nasal
Ausente
Dilatacion
Aleteo
Retraccion costal
Ausente
Dbil
Marcada
meconio. Muy sensible estmulos externos. Inestabilidad
Retraccion esternal
Ausente
Hundimiento de la punta
Hundimiento del cuerpo
vasomotora. CAUSAS QUE ALTERAN LA ELIMINACIN DEL LQUIDO
Concordancia ToracoExpansion de ambos
Hundimiento de torax y
Discordancia
PULMONAR: Nacimiento por cesrea, parto inducido, prematurez,
abdominal
en inspiracion
abdomen
0 puntos: sin asfixia ni dificultad respiratoria.
nacimiento sin trabajo de parto, asfixia, drogas en las madres, hijo
Recin nacido con 1 a 3 puntos: con dificultad respiratoria leve. (casco ceflico)
de madre diabtica. El parto es un fenmeno fisiolgico, que nos
Recin nacido con 4 a 6 puntos: con dificultad respiratoria moderada. (casco ceflico)
Recin nacido con 7 a 10 puntos: con dificultad respiratoria severa. (ventilador)
ayuda a eliminar el lquido pulmonar, al pasar el producto por el
canal de parto, para comenzar con el intercambio gaseoso. CAUSAS QUE ALTERAN LA ADAPTACIN PULMONAR: Hipoxia y acidosis, hipotermia,
perodo expulsivo prolongado, RN con dificultad respiratoria, RN de trmino o pretrmino limtrofe, Sileverman <3, acrocianosis, elevacin de la FC,
remite entre las 8 y 12 hrs. DIAGNOSTICO: Rx de Trax es normal o con leve aumento de la trama vascular, los gases sanguneos con acidosis
respiratoria e hipoxia leves. DIAGNOSTICO DIFERENCIAL: Taquipnea transitoria del RN, enf., de membrana hialina, cardiopatas. TRATAMIENTO:
Ambiente trmico neutro (primero, antes que canalizarlo) para reducir el consumo de O2 (con ello no hara hipoglucemia, ni tampoco se agravara la
asfixia) aspiracin de secreciones, oxigenoterapia: mantener la PaO2 entre 50-80mmHg. Va oral en cuanto sea posible, si no, iniciar liquidos
intravenos con glucosada al 10% a requerimientos.
CASO CLINICO SINDROME DE ADAPTACION PULMONAR
Se trata de paciente masculino RN de 60 minutos, obtenido por
cesarea iterativa, dems de presentar sufrimiento fetal, por
disminucin de actividad intrauterina y disminuciones de FC de hasta
110 lpm, se observa con movimientos muy finos, al inicio presento
cianosis leve y breve principalmente periungeal y oral, se apresiaron
secresiones orales abundantes, a la exploracin fsica se auscultan
estertores bilaterales normales, peristaltismo poco audible y
temperatura de 36.1 grado, se apresi expulsin de secresiones con
tinte meconial durante la exploracin.
PREGUNTA
Considerando el cuadro clnico. Cual es su conducta a seguir?
RESPUESTA
a.- Mantener al paciente en observacin continua.
b.- Buscar psobles patologas ocultas.
c.- Realizar radiografia de torax.
d.- No hay datos patolgicos son adaptativo.
PREGUNTA
Cual es la conducta teraputica mas apropiada en este momento
del caso?
RESPUESTA
a.- Mantener ayuno hasta identificar causa
++++
c.- Administrar oxigeno por casco ceflico 40 %.
d.- Gasometria y oximetra de pulso.
PREGUNTA
Cual de los siguientes parmetros gasomtrico no es normal
esperarlo en el caso?
RESPUESTA
a.- pH < 7.34
b.- PaCO > 45 mmHg

CURSO ENARM CMN SIGLO XXI TEL: 36246001

c.- HCO3 > 40mEq/L


d.- SpO2 < 95
PREGUNTA
Se realizaron electrolitos sericos por continuar con rechazo al
alimento y secresion moderada oral, todos los resultados dentro de
parmetro normales. Cual de los siguientes diagnosticos
diferenciales
de acidosis respiratoria es ms probable de
presentarse?
RESPUESTA
a.- Ventilacin inadecuada alveolar.
b.- Desordenes musculares.
c.- Defectos pulmonares.
d.- Trastornos de las vas areas.
PREGUNTA
Considerando los valores gasomtricos y de eletrolitos. Cual es la
causa de la modificacin del anio gap, del caso?
RESPUESTA
a.- Error de laboratorio.
b.- Toxinas no identificadas.
c.- Mielomas.
d.- Exceso de soluciones.
PREGUNTA
El paciente presenta persistencia de los sntomas por 3 horas ms, se
reporta que incrementa con la alimentacin, con dificultad continua
succionando, usted considera realizar laboratorios. Cual de los
siguientes resultados es el menos probable encontrar en este caso?
RESPUESTA
a.- Aumento de la trama vascular.
b.- Presencia de infiltrados pulmonares.
c.- Acidosis respiratoria.
d.- Hipoxemia leve.

Pharmed Solutions Institute

PGINA 292

MANUAL DE TRABAJO DEL CURSO ENARM CMN SIGLO XXI


TAQUIPNEA TRANSITORIA DEL RECIEN NACIDO (TTRN)
CIENCIAS BASICAS: Tambien llamado Sindrome de Avery o pulmn hmedo diestres respiratorio tipo II. Es un proceso respiratorio no infeccioso
que se presenta con ms frecuencia en los recin nacidos de termino o cercanos a trmino o prematuros grandes, donde hay retencin de lquido
pulmonar. Se inicia en las primeras horas y se caracteriza por la presencia de taquipnea (FR >60x), insuficiencia respiratoria y aumento de
requerimiento de oxgeno, con niveles de CO2 normales o ligeramente aumentados. Es un proceso generalmente benigno y autolimitado que se
resuelve aproximadamente de 24-72h. Factores que favorecen la aparicin de TTRN: Maternos; asma, DM, madre con drogadiccin, tabaquismo,
manejo abundante de lquidos, sedacin por tiempo prolongado, RPM >24hrs. Recin nacido; nacimiento por cesrea sin trabajo anterior, parto
precipitado, nacimiento cercano a trmino o termino, asfixia perinatal, sexo masculino, macrosomia, demoras en el pinzamiento del cordn
umbilical, sedacin materna excesiva, embarazo mltiple. SALUD PUBLICA: Incidencia: 1-2% de los recin nacidos. Abarca del 35-50% de los casos de
insuficiencia respiratoria no infecciosa de los recin nacidos que ingresan a los servicios de neonatologa. PATOGENIA: 3 factores: 1. Retraso en la
resorcin de lquido pulmonar fetal 2. Inmadurez pulmonar 3. Deficiencia ligera de surfactante. Al momento del nacimiento, el epitelio pulmonar del
RN que durante el embarazo es un activo secretor de cloro (Cl-) y liquido hacia los alveolos, tiene que cambiar para convertirse en un activo
absorbedor de sodio (Na++) y liquido con el objeto de remover este ltimo, que est condicionado por la presencia de catecolaminas secretadas
durante el trabajo de parto que estimulan los llamados canales epiteliales de Na ++. La TTRN es el resultado de alveolos que permanecen hmedos
al no producirse esta reabsorcin en forma adecuada. El nio nacido por cesrea o el que nace precipitadamente por va vaginal tiene mayor riesgo
de tener exceso de lquido pulmonar como resultado de no haber experimentado las fases de la labor y la falta de exposicin a las catecolaminas
mencionadas. El resultado final son alveolos que retienen lquido (comprometiendo el intercambio gaseoso que favorece la hipoxemia), el cual se
acumula poco a poco en el intersticio, hasta que es removido por los vasos linfticos o pasa al torrente sanguneo. El acumulo de lquido produce
edema intersticial transitorio y disminucin de la distensibilidad pulmonar, siendo esto ltimo la causa de taquipnea (compensatoria), y colapso
parcial bronquiolar que condiciona a su vez atrapamiento areo. Durante el transcurso de las siguientes horas el lquido es removido
progresivamente, mejorando la oxigenacin y disminuyendo la frecuencia respiratoria. En este sndrome hay alveolos per fundidos, mal ventilados
(hipoxia) (hipercapnia). DIAGNOSTICO: Taquipnea es >60, pero estos nios van a tener de 80-120 rpm, dificultad respiratoria de cualquier grado
evaluado de acuerdo a la escala de Silverman, aumento de dimetro antero-posterior de torax (se puede observar en rx. De torax), cianosis en casos
graves (cede con FiO2 baja de 0.3-0.4), campos pulmonares sin estertores. Tiende a resolverse de 12-24h pero puede persistir hasta 72h. Laboratorio
y gabinete: Prenatal; amniocentesis de liquido amnitico con relacin lecitina/esfingomielina L/E= 2:1 riesgo bajo, <1:1 riesgo elevado. Gasometra
arterial; hipoxemia, CO2 en limite normal o ligeramente aumentado, acidosis respiratoria compensada o hipoxemia leve. Radiografa de trax;
aumento de trma vascular, pero vemos imgenes algodonosas quese quitan a las 24h. Imgenes de atrapamiento areo (rectificacin de arcos
costales, herniacin de parnquima, hiperclaridad, aumento de espacios intercostales, aplanamiento de diafragmas), cisuritis, congestin parahiliar,
cardiomegalia aparente (descratar cardiopata congnita). Si despus de 72 hrs la taquipnea no remite o incrementa pensar en otra patologa y/o en
complicaciones. Parametros normales e nio de termino de gasometra; PO2= 80-95mmHg, HCO3=24-26, pCO2= 35-45pH= 7.32-7.38. Prametros para
intubar a un nio PO2= <50, PCO2 >50. TRATAMIENTO: Mantener en todo momento ambiente trmico neutro. La forma e inicio de alimentacin se
determina de acuerdo al estado clnico de RN (respiracion normal 40-60rpm), asi tenemos Silverman menor a 3 y FR 60-80 rpm; come. Silverman
menor a 3 y FR 80 rpm; Sonda orogastrica. Silverman >3 y FR >80 rpm; ayuno y liquidos parenterales. El volumen de lquidos IV ser de acuerdo a las
necesidades para la edad gestacional y el peso. Si existiera alguna patologa asociada a la TTRN el inicio y forma de alimento ser a juicio del mdico.
Farmacolgico: oxigenoterapia; habitualmente no requieren FiO2 mayores al 40% (si es muy alto se puede causar retinopata del RN). El objetico es
obtener saturaciones por plsometria en el rango de 90-92%. No se recomienda el uso de medicamentos ya que no existe evidencia suficiente de su
eficacia y seguridad en RN. La administracin postnatal de epinefrina con el fin de estimular la reabsorcin de lquido pulmonar, ha sido motivo de
estudios experimentales sin que exista al momento evidencia, NO aplicar. Los esteroides antenatales a madres entre las 34-37 SDG, podra tener un
efecto benfico al disminuir la morbilidad respiratoria de los RN, aun se requieren estudios para establecer su recomendacin. Criterios para
ingresar a UCI: Intubado, acidosis mixta, cardiopata asociada, hipoglicemias pristentes. COMPLICACIONES: Hipoxemia, hipertensin arterial
pulmonar. PRONOSTICO: Muy favorable, remite de 2-7 dias, la regla es que remita a las 72hrs, las anomalas radiolgicas sulen desaparecer a las 4872hrs.
CASO CLINICO TTRN
Se trata de paciente masculino de 15 horas de vida extrauterina, el
paciente de 35 SDG, naci via cesarea, se administro maduradores a
la madre debido a trabajo de parto distcico con ruptura de
membranas 12 horas antes de la intervencin quirrgica, a la
exploracin fsica se observo taquipnea de 101 rpm, y signo
moderados de dificultad respiratoria.
PREGUNTA
Considerando un propable diagnostico clnico realizado. Cual de los
diagnosticos diferenciales es menos probable encontrar?
RESPUESTA
a.- Sindrome de adaptacin pulmonar.
b.- Sindrome de aspiracin de meconio.
c.- Enfermedad de membrana hialina.
d.- Neumonia neonatal hospitalaria.
PREGUNTA
Se decide realizar radiografia de trax. Cul de los siguientes datos
es menos probable encontrar para confirmar el diagnostico?
RESPUESTA
a.- Broncograma areo.
b.- Hilio congenstivos.
c.- Derrame cisurales.
d.- Sobredistencion pulmonar.

CURSO ENARM CMN SIGLO XXI TEL: 36246001

PREGUNTA
De los factores de riesgo inducido para la taquipnea trasitoria del
recin nacido. Cual es la mas probable en el caso clnico?
RESPUESTA
a.- Sexo masculino
b.- Grandote.
c.- Cesrea.
d.- Sedacin materna.
PREGUNTA
Cual de los fenmenos fisiopatolgicos no es el ms adecuado para
el caso?
RESPUESTA
a.- Falta de absorcin de lquido amniotico.
b.- Falta administracin de esteroides.
c.- Deficienca ligera de surfactante.
d.- Edema pulmonar transitorio.
PREGUNTA
Considerando el diagnotico actual, el cual muestra singos de
dificultad respiratoria, resultados de laboratorio normales al
momento, FiO2 < 0.40 descartando los diagnosticos diferenciales.
Cul conducta a seguir en este caso es menos adecuada?

Pharmed Solutions Institute

PGINA 293

MANUAL DE TRABAJO DEL CURSO ENARM CMN SIGLO XXI


RESPUESTA
a.- Monitoreo de signos vitales.
b.- Monitoreo de gases por puncion cada 6 horas.
c.- Mantener glucosa y electrolitos.
d.- O2 por casco al 70 %

d.- Colocacion de factor surfactante.


CASO CLINICO TAQUIPNEA TRANSITORIA DEL RN
RN femenino de 31 SDG "despus de un embarazo sin
complicaciones, con peso de 1480 g, y Apgar de 2/6. Fue obtenido
por cesaria electiva por cesaria previa por periodo intergenesico de 6
meses. A los pocos minutos de nacer, su frecuencia respiratoria fue
de 61, con significativo aleteo nasal, uso prominente msculos
accesorios y retracciones subcostales. Su ritmo cardaco era normal
(121 latidos / min). Haba signos de cianosis.

PREGUNTA
Tomando en cuenta la evolucin del caso cual es su pronstico?
RESPUESTA
a.- Bueno.
c.- Malo.
b.- Fatal.
d.- Excelente.

PREGUNTA
La paciente se encuentra en un hospital rural de concentracin.
Cual es su conducta a seguir?

CASO CLINICO TAQUIPNEA TRANSITORIA DEL RN


Un RN de 3,6 kg naci a las 37 semanas de gestacin hijo de
diabtica con un agente hipoglicmico oral. Obetnido por cesrea
electiva con Apgar de 6/9. Desarrollo taquipnea inmediatamente
despus de su nacimiento y requirio oxgeno suplementario. En la
gasometria presento; PO2 de 57 mm Hg, PCO2 de 52 mm Hg, y pH de
7,31. El nio se mantuvo en oxgeno por campana. A las 2 horas el
paciente se encuentra hipotnico, ciantico, con saturacin del 70 %
(Normal mas 88-92%, ideal 95%). La Rx de observa rectificacin,
hiperclaridad e incremento de los espacios intercostales y
congestin parahiliar.
PREGUNTA
Cual es la medida inmediata a seguir?
RESPUESTA
a.- Intubacion orotraqueal.
b.- Alimentacin por sonda orogastrica.
c.- Mantener un ambiente neutro.
d.- Realizar medidas de reanimacin.
PREGUNTA
Cual de las siguientes datos es menos probable para el diagnostico
radiolgico?
RESPUESTA
a.- Aumento de la trama vascular con imgenes algodonosas.
b.- Liquido en cisusras interlobales.
c.- Abatimiento de diafragma.
d.- Cardiomegalia.
PREGUNTA
Cual de los siguientes diagnosticos diferenciales es mas frecuente?
RESPUESTA
a.- Sepsis.
b.- Neumona
c.- Asfixia perinatal
d.- Cardiopata congenita.
CASO CLINICO TAQUIPNEA TRANSITORIA DEL RN
Un RN de 3,4 kilos naci a las 40 semanas de gestacin con madre
diabtica tratada con insulina. El nio fue obtenido por cesrea.
Puntuacin de Apgar fue 5/6. Se presento taquipnea una hora
despus en observacin, el nio fue colocado en oxgeno con casco
ceflico. Las determinaciones de gases en sangre arterial en ese
momento eran de PO2 52 mm Hg, pCO2 de 48 mmHg, y pH de 7,31.
PREGUNTA
Cual es la conducta inicial a seguir?
RESPUESTA
a.- Iniciar con bicarbonato.
b.- Verificar laboratorios.
c.- Realizar estudios de gabinete

CURSO ENARM CMN SIGLO XXI TEL: 36246001

RESPUESTA
a.- Trasladarla a una unidad de cuidados intensivos.
b.- Vigilancia estrecha ya que su evolucin es buena.
c.- Los datos radiogrficos no son importantes.
d.- Verificar el diagnostico inicial
CASO CLINICO
Recin nacido a trmino, de 38 semanas de edad gestacional, hijo de
madre diabtica, que nace por cesrea por estar en podlica. A las
18hrs de vida extrauterina, comienza con distrs respiratorio, con un
test de Silverman de 6 y 82 rpm y necesidades de oxgeno de 0.35.
Se instaura CPAP, se realiza gasometra a la hora de vida, la cual solo
muestra ligera hipoxemia.
PREGUNTA
Debido a los parmetros respiratorios presentes en este paciente
cual es la conducta a seguir?
RESPUESTA
a.- iniciar alimentacin al seno materno
b.- Iniciar alimentacin por sondaorogastrica
c.- Ayuno y lquidos parenterales
d.- Lquidos parenterales
PREGUNTA
Considerando el probable diagnstico, que datos radiolgicos
esperara encontrar en este paciente?
RESPUESTA
a.- Normal con leve aumento de trama vascular
b.- Imagen en vidrio despulido
c.- Atrapamiento de aire, cisuritis, cardiomegalia
d.- Infiltrados en parche, atelectasias
PREGUNTA
Cul de los factores es el ms probable causante de la taquipnea en
el paciente?
RESPUESTA
a.- Retencin de lquidos en los alveolos al no producirse la
reabsorcin
b.- No usar ventilador
c.- Deficiencia ligera del surfactante
d.- Edema intersticial y disminucin de la distensibilidad pulmonar
PREGUNTA
De acuerdo a la escala de Silverman presente en este paciente, que
grado dificultad respiratoria presenta?
RESPUESTA
a.- Sin dificultad respiratoria
b.- Leve
c.- Moderada
d.- Severa

Pharmed Solutions Institute

PGINA 294

MANUAL DE TRABAJO DEL CURSO ENARM CMN SIGLO XXI


SINDROME DE ASPIRACION MECONIAL (SAM)
CIENCIAS BASICAS: Se manifiesta con distrs respiratorio grave y es producido por la aspiracin de lquido amnitico teido con meconio intra tero
o intra parto. Constituye una causa de morbimortalidad en el recin nacido, principalmente en el nio a trmino y postrmino. Factores de riesgo:
Embarazo postrmino, preeclampsia- eclampsia, hipertensin materna, DM materna, oligohidramnios, tabaquismo intenso, puntaje de Apgar (<3)
bajo a los 5 minutos, hipoxia aguda intraparto, hipoxia perinatal crnica, frecuencia cardiaca fetal anormal, RN pequeos para la edad gestacional.
SALUD PUBLICA: Se reporta con frecuencia lquido meconial hasta en 14% de RN (sin ser patolgico). 10% presentan sndrome de aspiracin por
meconio. Mortalidad 12 % y adems deja secuelas. La expulsin de meconio rara vez es antes de las 37 semanas. Puede ocurrir 30% de los
embarazos que continan despus de las 42 SDG. PATOGENIA: 1. Evacuacin del meconio in-tero: La asfixia y otras formas de estrs intrauterino
pueden causar un aumento del peristaltismo intestinal, con relajacin del esfnter anal externo y evacuacin de meconio. El efecto de la hipoxia
intrauterina sobre el peristaltismo y el tono esfinteriano parece aumentar con la edad gestacional. 2. Aspiracin de meconio: Despus de la
evacuacin de meconio en el lquido amnitico, las respiraciones jadeantes (boqueadas) del feto asfixiado, ya sea in tero o durante el trabajo de
parto, pueden determinar la aspiracin del lquido amnitico meconial hacia las vas areas grandes del pulmn. El meconio espeso provoca
obstruccin de la va area (total o parcial), lo que ocasiona dificultad respiratoria. En las reas de obstruccin total se desarrollan atelectasias; en
cambio, en las reas de obstruccin parcial ocurre un fenmeno valvular que ocasiona atrapamiento areo e hiperinsuflacin pulmonar. El
atrapamiento areo aumenta el riesgo de escape de aire del 21 al 50%. Finalmente, se desarrolla neumonitis intersticial y qumica (por iirritacion del
meconico compuesto de descamacin de la piel, acidos biliares, protenas), con edema bronquiolar y estrechamiento de las vas areas de pequeo
calibre, tambin hay disfuncin del surfactante. La ventilacin desigual debida a reas de obstruccin parcial y neumonitis quimica sobreagregada
produce retencin severa de dixido de carbono e hipoxemia. La resistencia vascular pulmonar aumenta como resultado de la hipoxia, la acidosis y
la hiperinsuflacin de los pulmones. El aumento de la resistencia vascular puede conducir a un cortocircuito de derecha a izquierda auricular o ductal
y a una mayor desaturacin. CLASIFICACION: De Cleary- Wiswell: a) LEVE dificultad respiratoria que requiere menos de 40% de oxigeno en las
primeras 48 h. b) MODERADA requerimiento de mas de 40% de oxigeno por mas de 48 h. c) SEVERA ventilacin mecnica por mas de 48h asociada a
hipertensin pulmonar persistente. DIAGNOSTICO: Clnico; Los sntomas dependen de la severidad de la lesin hipxica y de la cantidad y de la
consistencia del meconio aspirado. Es frecuente que su inicio clnico sea precoz (no mejora aun con oxigeno), progresivo a lo largo de 12 a 24 horas,
con hipoxemia, dificultad respiratoria, taquipnea, un RN con <40 lpm (FC normal despierto 100-180lpm, dormido 70-80lpm)se considera en paro,
cianosis persistente, hipoxemia, de saturaciones frecuentes, labilidad en el manejo clnico. Los neonatos con lquido amnitico teido con meconio
suelen mostrar signos de posmadurez; son pequeos para la edad gestacional y tienen uas largas, piel descamada teida con pigmento amarillo o
verde y cordn umbilical teido de meconio (15min), uas manchadas (4-6 h), mancha o unto sebceo (12h). Laboratorio; niveles de gases en
sangre arterial revelen hipoxemia. La hiperventilacin puede producir alcalosis respiratoria en los casos leves; pero los lactantes con enfermedad
grave suelen manifestar acidosis respiratoria con retencin de dixido de carbono, debido a obstruccin de la va area y neumonitis. Elevacion de
lactato y troponina srica. La radiografa de trax tpica muestra infiltrados pulmonares heterogneos (irregulares y en parches (panal de abejas)),
hiperinsuflacion de los campos pulmonares, diafragma aplanado, atelectasias segmentadas o lobares. TRATAMIENTO: En la sala de parto; el obstetra
deber aspirar en contenido de la nariz y oro faringe antes del parto de trax con perilla. Si hay aspiracin de meconio y el RN no est vigoroso
(respiracin inadecuada, tono muscular disminuido, FC <100 lpm), est indicada aspiracin traqueal directamente, antes de que establezca las
respiraciones. Introducir laringoscopio y utilizar sonda de aspiracin de 12F a 14F para aspirar la boca y la faringe posterior y as visualizar la glotis.
Introducir el tubo endotraqueal en la trquea, conectar a equipo de aspiracin, aspirar a medida que se retira la sonda. Por otra parte aumentar la
concentracin de oxigeno inspirado, monitorear gases sanguneos, CPAP (presin de la va area positiva continua) individualizar cada caso,
ventilacin mecnica, ventilacin de alta frecuencia (ventiladores oscilatorios o jet). Corticoesteroides: no hay evidencia. En recin nacidos con SAM
masivo e hipertensin pulmonar persistente, el manejo con ventilacin de alta frecuencia y xido ntrico ha mejorado su pronstico; la incorporacin
de oxigenacin con membrana extracorprea, (no disponible an en nuestro medio) se muestra como un arma prometedora en el manejo de este
tipo de pacientes. La inactivacin del surfactante por el propio meconio y/o por la coexistencia de edema pulmonar, hace suponer que algunos de
estos RN se beneficiaran de la administracin de surfactante. En este sentido, estn en marcha estudios corroborativos para demostrar la
efectividad de esta nueva arma teraputica. COMPLICACIONES: Neumona por aspiracin de meconio, neumonitis, hipoxemia, di estrs respiratorio,
acidosis metablica, obstruccin mecnica de las vas areas, escape areo: 10-20% neumotrax (causa la mortalidad mas importante) o
neumomediastino, hipertensin pulmonar persistente del recin nacido, hemorragia pulmonar, displasia broncopulmonar. PRONSTICO: Poco
satisfactoria, hay 5 veces mayor riesgo de hipotona, puede presentar paralisis cerebral infantil, alteraciones en el neurodesarrollo, crisis epilpticas.
El 2% de RN con SAM mueren. Se considera que un bebe cae en paro con <40lpm.
CASO CLINICO SINDROME DE ASPIRACION DE MECONIO
RN femenino de trmino obtenido por parto vaginal espontneo con
un peso al nacer de 3.500 g. el monitoreo intraparto no revelaron
evidencia de sufrimiento fetal. Poco despus del parto, fue
ingresado en una unidad de cuidados intensivos debido a la
presencia de meconio, vmitos y succin debil, su temperatura era
inferior a 36,0 C, el pulso fue de 148 lat / min, y su frecuencia
respiratoria era de 72 respiraciones / min. El tiempo de protrombina
y el tiempo parcial de tromboplastina activada eran 20,7 y 54,6 s,
respectivamente. La orina y sangre se sometieron a la cultivo. La
radiografa de trax mostr infiltrados y rayas gruesas del campo
pulmonar derecho.
PREGUNTA
Considerando la gravedad del cuadro. Cual es la complicacin ms
probable en esta paciente?
RESPUESTA
a.- Sepsis.
b.- Acidosis respiratoria.
c.- Neumonia.
d.- Coagulopatia.

CURSO ENARM CMN SIGLO XXI TEL: 36246001

PREGUNTA
Cual de las siguientes manifestaciones es mas importante para el
pronostico?
RESPUESTA
a.- La cantidad de meconio aspirado.
b.- La edad gestacional.
c.- Manifestaciones de neumonitis qumica.
d.- Manifestaciones de posmadurez.
PREGUNTA
Cual de las siguientes manifestaciones es mas frecuente encontrar
en esta patologia?
RESPUESTA
a.- Infiltrados.
b.- Hiperinsuflacion.
c.- Atelectasia semegtadas o globales
d.- Puede haber neumotrax.
PREGUNTA

Pharmed Solutions Institute

PGINA 295

MANUAL DE TRABAJO DEL CURSO ENARM CMN SIGLO XXI


Cual de las siguientes complicaciones es la menos frecuente en esta
patologa?
RESPUESTA
a.- Hipertension pulmonar persistente.
b.- Acidosis con lactato bajo.
c.- Neumonitis qumica.
d.- Sindrome de escape areo.
CASO CLINICO
Recin nacido masculino, obtenido por cesrea de emergencia, del
cuarto embarazo de una madre de 39 aos sin control prenatal,
diabtica, a las 41 semanas de edad gestacional debido a presin de
160/115mmHg, desproporcin cefalo-plvica y sufrimiento fetal
agudo, que se diagnostic una hora antes del nacimiento mediante
monitoreo fetal. Al nacimiento se evidenci lquido amnitico con
meconio espeso, impregnacin de meconio en uas, cordn
umbilical y prpados, peso 4700 gramos, Apgar 6 y 7. Requiri
intubacin endotraqueal y aspiracin, obtenindose lquido de color
amarillo oscuro. El paciente fue admitido a la unidad de terapia
intensiva neonatal.
PREGUNTA
De acuerdo al diagnstico que complicacin es la que da la
mortalidad ms importante del caso?
RESPUESTA
a.- Neumonitis
b.- Parlisis cerebral infantil
c.- Neumotrax
d.- Hipoxemia
PREGUNTA
Se le toma una radiografa de trax. Que esperaramos ms
probablemente encontrar en el estudio de este caso?
RESPUESTA
a.- Infiltrados en parche, hiperinsuflacion
c.- Broncograma areo
d.- Hiperinsuflacion, hilio congestivos
c.- Neumona
PREGUNTA
Si hay aspiracin de meconio y el recin nacido no est vigoroso
cual es la conducta teraputica ms adecuada, antes de que
establesca las respiraciones normales con el llanto?
RESPUESTA
a.- Uso de corticoesterioides
b.- Aspiracin traqueal directamente con sonda de 12-14 F
c.- Ayuno
d.- Ambiente neutro
CASO CLINICO
Recien nacido producto de la IV gesta. Madre con diabetes
gestacional mal controlada, 34 aos, peso de 98 kgs. y talla de 1.65
m. Con 42 SDG por FUM, peso fetal por ultrasonido: 3800 g. Se
refiere RPM de 24 hrs, desaceleraciones, lquido amniotico meconial
espeso. Impregnado en meconio, incluyendo uas y cordn
umbilical. Al minuto con: Pobre esfuerzo respiratorio, cianosis
generalizada, FC: 150xm, tono muscular disminuido, reactivo a la
estimulacin.
PREGUNTA
Cul es el factor de riesgo materno menos probable en esta
patologa?

CURSO ENARM CMN SIGLO XXI TEL: 36246001

RESPUESTA
a.- Multiparidad
b.- Obesidad materna
c.- Embarazo postermino
d.- Diabetes gestacional
PREGUNTA
Cul es el Apgar en este momento para este paciente?
RESPUESTA
a.- 3
b.- 5
c.- 7
d.- 10
PREGUNTA
Cul es el factor ms importante para riesgo de infeccion en el
producto?
RESPUESTA
a.- Hijo de madre diabtica
c.- SAM
b.- RPM de 24 hrs
d.- 42 semanas de gestacin
PREGUNTA
Se realiza gasometra la cual reporta: Gasometra Arterial, pH: 7.22,
pC02: 75, pO2: 44, HC03: 12, DB:-10, Lactato: 6. Cul es el
diagnostico mas probable en este caso?
RESPUESTA
a.- Acidosis mixta no compensada
b.- Acidosis respiratoria
c.- Alcalosis repiratoria
d.- Acidosis metabolica

CASO CLINICO
Recin nacido de sexo femenino de 38 semanas por fecha de ltima
regla, pequea para edad gestacional, asimtrico, de parto
abdominal por sufrimiento fetal, de madre segundigesta nulpara sin
control prenatal, en quin se decide cesrea por detectarse
bradicardia fetal y lquido meconial espeso aproximadamente 15
minutos antes del parto. La nia nace deprimida luego de aspiracin
oronasal intraparto, realizndose maniobras de reanimacin en el
postparto inmediato que incluye aspiracin de meconio de trquea,
respiracin manual por tubo endotraqueal (TET) y con oxgeno
resultando con APGAR: 3(1)-9(10)-10(15). El examen inicial
(tiempo de vida: 30 min.) muestra: peso: 2140 gras. Talla:46 cm. FC:
160 x min, FR: 72 x min, mal estado general, despierta, distress
respiratorio moderado, piel normal. Aparato respiratorio: aleteo
nasa, retracciones costales, trax abombado, score de Silverman: 7,
subcrpitos en ambos campos pulmonares. Cardiovasculares:
normal, Abdomen: no visceromegalia, SNC: alerta, hipotona, mala
succin, moro dbil.
PREGUNTA
Cul seria la conducnta teraputica mas adecuada a seguir en este
momento?
RESPUESTA
a.- Ampicilina mas gentamicina
b.- Cefotaxima mas amikacina
c.- Vancomicina
d.- Levofloxacino mas eritromicina

Pharmed Solutions Institute

PGINA 296

MANUAL DE TRABAJO DEL CURSO ENARM CMN SIGLO XXI


SINDROME DE DIFICULTAD RESPIRATORIA (ENFERMEDAD DE MEMBRANA HIALINA)
CIENCIAS BASICAS: Tambien llamado atelectasia pulmonar difusa, el sndrome de dificultad respiratoria por deficiencia de surfactante es la principal
patologa respiratoria entre los recin nacidos en prematuros (>incidencia a menor numero de semanas); ocupa un papel preponderante por su alta
morbimortalidad. Se caracteriza por inmadurez del desarrollo anatmico y fisiolgico pulmonar del recin nacido prematuro, cuya caracterstica es la
deficiencia de surfactante que causa de manera progresiva, atelectasia pulmonar difusa e inadecuado intercambio gaseoso. Inicia al nacer o poco
despus y es de carcter progresivo, por dficit de surfactante cuya mxima gravedad se observa entre las 24-36h de vida. La funcin del
SURFACTANTE es disminuir la tensin superficial, es decir evita el colapso alveolar, estabilizando y manteniendo la superficie del alveolo
relativamente libre de lquido, adems de aumentar la elasticidad pulmonar, disminuye el trabajo de la respiracin, disminuye el tono precapilar,
protege a las superficies alveolares de la tensin oxidativa. El surfactante esta compuesto en 85% por fosfolpidos, fosfatidilglicerol y fosfatidilcolina,
dipalmitoilfosfatidilcolina, lpidos naturales, protenas A,B, C, D. Se sintetiza en la semana 16-25 de gestacin, en la semana 20-22 de gestacin se
originan y reconocen los neumocitos tipo I (intercambio gaseoso) y II (secretan surfactante). Factores de riesgo aumentado: prematurez, masculino,
predisposicin familiar, cesrea sin trabajo de parto, asfixia perinatal, corioamnioitis, diabetes materna. Riesgo disminuido: estrs intrauterino
crnico (RPM prolongada, hipertensin materna, uso de narcticos, RCIU), no haber recibido esteroides prenatales, hormona tiroidea, agentes
tocoliticos. SALUD PUBLICA: Es una de las primeras causas de ingreso a las unidades de terapia intensiva neonatal a nivel mundial. Se presenta en
15-50%. La incidencia exacta de esta patologa es difcil de precisar, en Mexico 1% de RN vivos, aprox., 60% en menores de 28SDG, 15-20% de 3236SDG, 5% en mayores de 37 SDG, 20% en RN con peso de 1000-1500gr. PATOGENIA: Inmadurez estructural del pulmn (torax), persistencia del
conducto arterioso, aumento de la permeabilidad alveolar-capilar, pared torcica demasiado distensible, disminucin en la produccin y secrecin
de surfactante, cortocircuito, presin intratracica disminuida. La frecuencia respiratoria se encuentra elevada por lo que a pesar del volumen
corriente (Vt) disminuido, la ventilacin minuto inicialmente esta incrementada. Debido a la deficiencia o disminucin en la cantidad o calidad del
surfactante pulmonar la mayor parte del pulmn est colapsado o poco ventilado y la mayor parte de la ventilacin alveolar se deriva a una regin
muy pequea del pulmn lo que conlleva a una disminucin de la capacidad residual funcional (CRF). Asimismo la distensibilidad est muy
disminuida, no tanto por el trax del recin nacido que es fcilmente distensible, sino que por los pulmones que con esta deficiencia de surfactante
llegan a tener mediciones de la distensibilidad menor al 30% de lo normal. DIAGNOSTICO: Los signos pueden manifestarse desde los primeros
minutos de vida o despus de algunas horas, y por lo general son de incremento gradual. Aunque en ocasiones estos signos de dificultad respiratoria
son menos marcados debido a la debilidad de la musculatura respiratoria, lo que los llevar rpidamente a una falla respiratoria con hipoventilacin
y apnea (conocido como respuesta paradjica a la hipoxemia). Signos de dificultad respiratoria, perisistente o que progresa en las primeras 24-48h,
con necesidad cada vez mayor de O2, cianosis generalizada, volumen torcico disminuido, hipoventilacion bilateral, taquipnea (>60 rpm) perisistente
y progresiva, tiraje intercostal, retraccin xifoidea, disociacin toracoabdominal, aleteo nasal, y quejido espiratorio, este ltimo uno de los ms
frecuentes y es motivado por el cierre de la glotis en su afn de realizar un auto PEEP (presin positiva al final de la espiracin) para conservar los
alvolos abiertos y aumentar el volumen residual pulmonar para un adecuado intercambio gaseoso. Edema de miembros inferiores e hipotensin.
Pruebas de madurez fetal lecitina/Esfingomielina (L/S) >2:0 = Madurez, <2:0 = Inmadurez. Riesgo bajo de padecer SDF 2:1, riesgo alto 1:1.
Gasometria arterial: hipoxemia, hipercarbia, acidosis repiratoria. Prueba de aspirado gstrico; se indica en dificultad respiratoria progresiva, peso
<1500g, 30 minutos siguientes a nacer. Se realiza con 1ml de aspirado gstrico + 1ml de alcohol etlico al 95% en tubo de ensaye y agitar; es positivo
si se observan burbujas (hay surfactante), si no hay burbujas es negativo (no hay surfactante). Diagnostico radiolgico: I. Infiltrado reticulogranular.
II. Broncograma areo + infiltrado reticulogranular (no rebasa silueta cardiaca). III. Broncograma areo + ensanchamiento del torax, horizontalizacion
de arcos costales. Infiltrado reticulogranular (rebasa la silueta cradiaca). IV. Imagen de vidrio despulido con ausencia de broncograma areo
pulmn blanco (por bronquiolos edematizados). A la auscultacin de campos pulmonares encontraremos disminucin del murmullo vesicular
habitualmente en forma bilateral. TRATAMIENTO: Ambiente trmico neutro, mantener PAM media, de acuerdo a la edad, equilibrio
hidroelectrolitco. Evitar acidosis; administrar Bicarbonato (1-3mEq/kg/do) con pH <7.1, o dficit de base -15. Para un buen transporte de O2, el hto
debe estar >40%. Adems de los signos de dificultad respiratoria se puede presentar cianosis central que obligar al uso de oxigenoterapia en
cualquiera de sus modalidades y que podr variar desde los casos leves que solo requieran apoyo con oxgeno en fase de casco ceflico o bien casos
moderados y severos que requerirn CPAP o ventilacin mecnica en su diversas modalidades y segn lo requiera cada paciente. La finalidad del
soporte ventilatorio es lograr un adecuado intercambio gaseoso reclutando los alvolos colapsados por el dficit de surfactante, y de esta manera
mejorar la acidosis, la hipoxemia y la hipercapnea. Este reclutamiento alveolar se logra manteniendo una presin positiva continua al final de la
expiracin (PEEP,) debido a que el llamado PEEP fisiolgico es de 2, se recomienda rangos por arriba de 4 cmH 2O. Esteroides: elevan la relacin L/S,
induce a neumocitos tipo II a que secreten surfactante, aceleran parmetros de desarrollo pulmonar, incrementan el numero de cuerpos laminares,
mayor sntesis de fosfolpidos de la sustancia tensoactiva, numero de recepetores b-adrenergicos, produccin de colgeno y elastina, disminuye el
riesgo de broncodisplasia pulmonar. SURFACTANTE: Sintetico accin de 15-30 min a dosis de 5ml/Kg (67.5mg), via intratraqueal. Natural accin
inmediata, dosis 4ml/kg (100mg). Profilaxis en los primeros 30 min, en los pretermino en 15 min. Rescate: primeras 2h de vida, 3 dosis con intervalo
de 6-8 h. Se administra en forma lenta, con la cabeza en posicion neutra, se aplica 1/3 de dosis y luego se voltea cabeza a ambos lados. Criterios de
administracion: peso entre 600-1750gr, radiografia compatible con SDR, necesidad fase III de VM, menos de 8 h de VEU, sin datos de hemorragia
pulmonar activa, cateterizacin de vasos umbilicales. La complicacin del surfactante es la hemorragia pulmonar. COMPLICACIONES: Acidosis,
hipocarbia (displasia pulmonar, leucomalacia periventricular) desequilibrio HE, insuficiencia renal, PCA, insuficiencia cardiaca, hipertensin
pulmonar, barotrauma, hemorragia peri-intraventricular, infecciones, broncodisplasia pulmonar, retinopata del prematuro (estas dos ultimas se
pueden generar por administracin excesiva de oxigeno). Evitar la hipotermia reduce la mortalidad. Hipocapnia aumenta el riesgo de paralisis
cerebral infantil.
CASO CLINICO SX DE DIFICULTAD RESPIRATORIA
RN masculino de edad gestacional de 39 semanas, de 3,8 kg hijo de
madre diabtica el cual se obtuvo por cesaria previa administracin
de esterioides a la madre, el cual requiri de estimulacin vigorosa
cursando con hipotermia que mejoro en incubadora, se observo
posteriormente, aleteo nasal, retraccin esternal, quejido
respiratorio, cianosis y polipnea, la Rx mostro un patro retculo
granular difuso bilateral y disminucin de la expansin pulmonar,
PaO2 < 50 mmHg.
PREGUNTA

CURSO ENARM CMN SIGLO XXI TEL: 36246001

Cual es la conducta teraputica mas adecuada a seguir?


RESPUESTA
a.- Presin positivo nasal cotinua.
b.- Oximetria de pulso.
c.- Intubacin traqueal.
d.- Ventilacin mecnica.
CASO CLINICO
RN de 31 semanas que presenta dificultad respiratoria. Tiene 2 horas
de nacido por parto vaginal. La dificultad respiratoria tiende a
incrementar. Madre diabtica, salvo la edad gestacional no se

Pharmed Solutions Institute

PGINA 297

MANUAL DE TRABAJO DEL CURSO ENARM CMN SIGLO XXI


identificaron otras complicaciones durante el embarazo. EF: Temp.
37.5 C, TA: 86/58, FR: 60/min, FC:148/min. Marcadamente
taquipneico, con retracciones supraesternales y supraclaviculares,
aleteo nasal, quejido. Faringe normal. CP normal. Usted decide
intubacin endotraqueal, toma laboratorios y RX de trax: Infiltrado
bilateral, difuso, vidrio despulido en ambos pulmones, sin datos de
atrapamiento de aire.
PREGUNTA
Cual de los siguientes resultados de la gasometra es menos
frecuente observar?
RESPUESTA
a.- Hipoxemia.
b.- Hipercapnia.
c.- Acidosis metablica.
d.- Acidosis respitoria.
CASO CLINICO
Se trata de RN femenino de 29 semanas de gestacion, que peso
1,700Kg. Hijo de madre diabtica, fumadora, obtenido por va
cesrea, debido a que la madre refiere no sentir movimientos fetales
desde la maana, a la exploracin se detecta sufrimiento fetal. A las
2 horas inicia con cianosis generalizada y dificultad respiratoria
progresiva, se observa taquipnea, tiraje intercostal, disociacin
toracoabdominal. A la auscultacin de campos pulmonares,
disminucin de murmullo vesicular.
PREGUNTA
A las 8 hrs se le toma tele de trax, que esperaramos observar en
el caso?
RESPESTA
a.- Infiltrado reticulogranular, broncograma areo
b.- Infiltrados en parche
c.- Horizontalizacin de costillas, aplanamiento de diafragmas
d.- Atelectasias
PREGUNTA
Para evitar la acidosis cuando debemos administrar bicarbonato a
este paciente?
RESPUESTA
a.- Dficit de base de -15 y pH <7.1
b.- Dficit de base de -10 y pH >7.1
c.- pH <7.3 y dficit de base -15
d.- pH >7.3 y dficit de base -10
PREGUNTA
Cul es la contraindicacin para administrarle surfactante a este
paciente?
RESPUESTA
a.- Peso entre 600-1750Kg
b.- Necesidad de fase III de VM
c.- Menos de 8hrs de VEU
d.- Hemorragia pulmonar
CASO CLINICO
Lactante masculino de 2 hrs de nacido, tienen dificultad respiratoria
en la sala de cuna. Nacio a las 34 semanas de gestacin de una
madre de 25 aos de edad, G1, P1, que tuvo un embarazo no
complicado y valores de laboratorio prenatales normales. La
puntuaciones de apgar fueron 6 y 8 a los uno y cinco minutos
respectivamente. En el examan esta afebril con FC 170 lpm, FR
80rpm, TA 65/40mmHg, saturacin de oxigeno al 94%.

Cul es el diagnotico ms probable para este caso?


RESPUESTA
a.- Taquipnea transitoria del recin nacido
b.- Sindrome de dificultad respiratoria
c.- Apnea transitoria del recin nacido
d.- Escape de aire pulmonar
PREGUNTA
Cul es la causa ms probable de esta patologa?
RESPUESTA
a.- Edema pulmonar debido a resorcin y eliminacin tardas del
lquido pulmonar fetal
b.- Deficiencia de surfactante
c.- Neumotrax
d.- Atelectasias
PREGUNTA
Se le adminstra oxigeno a 4 litros por minutos, Cul es la
complicacin mas probable para este caso?
RESPUESTA
a.- Broncodisplasia pulmonar
b.- Hemorragia periventricular
c.- Barotrauma
d.- Hipertension pulmonar

CASO CLINICO
Se trata de recin nacido de 31 semanas que presenta dificultad
respiratoria.Tiene 2 horas de nacido por parto vaginal. La
dificultadrespiratoria tiende a incrementar. Madre diabtica, salvola
edad gestacional no se identificaron otras complicaciones durante el
embarazo. EF: Temp. 37.5C, TA: 86/58, FR: 60/min, FC:148/min.
Marcadamente taquipneico, con retracciones supraesternales y
supraclaviculares, aleteo nasal, quejido. Faringe normal.
Laboratorios: Hb 19 g/dL (nl 17-22), leucocitos nles, EGO normal. RX
de trax: Infiltrado bilateral, difuso.
PREGUNTA
Cul es la conducta teraputica mas adecuada a seguir en este
caso?
RESPUESTA
a.- Casco ceflico
b.- Intubacion endotraqueal
c.- Puntas nasales
d.- Hidratacion intravenosa
PREGUNTA
Cul es el diagnotico mas probable para este caso?
RESPUESTA
a.- Neumonia
b.- Cardioparia congnita
c.- Sindrome de dificultad repiratoria tipo I
d.- Taquipnea transiotoria
PREGUNTA
Qu parmetro esprariamos encontara en este paciente?
RESPUESTA
a.- L/ E >2:0
b.- L/E <2:0
c.- L/E <1:1
d.- L/E 2:1

PREGUNTA

CURSO ENARM CMN SIGLO XXI TEL: 36246001

Pharmed Solutions Institute

PGINA 298

MANUAL DE TRABAJO DEL CURSO ENARM CMN SIGLO XXI


NEUMONIA NEONATAL (NNN)
CIENCIAS BASICAS: Ocurre como una complicacin de infecciones connatales o de infecciones nosocomiales. Neumona intrauterina: se adquiere va
intrauterina o durante el paso de canal de parto. Neumona nosocomial: se adquiere en su estancia hospitalaria, procesos invasivos, intubacin,
asistencia ventilatoria, permeabilizacin de vasos, despus de las 72 h. Neumona adquirida durante el nacimiento, por contaminacin durante el
paso a travs del canal de parto, aspirar meconio o liquido amnitico infectado. La neumona puede aparecer 2-4 semana de vida. Factores de
riesgo: maternos; RPM, IVU materna dentro de 15 das antes del parto, colonizacin vaginal patolgica, corioamnioitis. En el RN; menor dimetro de
rbol bronquial y escaso desarrollo de aparato ciliar, bajos niveles de IgM, complemento, opsoninas y funcin linfocitaria, trauma de la va area
(intubacin prolongada, aspiraciones profundas) y presencia de meconio en va area. Factores de riesgo para neumona nosocomial: peso <1500g,
periodo prolongado de hospitalizacin, maniobras invasivas multiples, hacinamiento. , contaminacin del respirador, lavado inadecuado de manos,
colonizacin de nariz y garganta. Microorganismo de la flora normal del RN: en los primeros 3 dias en nariz-Sthaphylococcus aureus, garagantaEstreptococcus hemolticos, recto-E. coli y lactobacillus. SALUD PUBLICA: La incidencia en el RNT es menor de 1% y en el RNPT de 10-20% (el aparato
ciliar no elimina mococ o bacterias), dependiendo de cada UCI Neonatal. El pulmn es el rgano que con mayor frecuencia se compromete en las
infecciones que se desarrollan en las primeras 24 horas de vida. Mortalidad: 20-50%, segn la edad gestacional y complicaciones asociadas. La
neumona nososcomial es la 2da causa de infeccion adquirida en el hospital con incidencia de 6.8-36%. PATOGENIA: Etiologa: NNN de inicio precoz
(primeras 48h a una semana de vida) = Complejo TORCH, E. Coli, Streptococo del grupo B, listeria y menos frecuentemente agentes virales (herpes
simple, CMV, influenza, rubola, ADV y echovirus. NNN de inicio tardo (siguientes 3 semanas) = Staphylococos, Klebsiella, Pseudomona, Enterovirus,
E. Coli, Cndida Albicans y algunos virus como CMV (por transfusiones). Etiologa transplacentaria: Sifilis, Listeria monocytogenes, herpes,
colonizacin materna estreptococcus agalactiae, Chlamydia trachomatis. Etiologia posnatal: Complejo TORCH, estreptococo B, E. coli, Klebsiella.
Etilogia nosocomial: Estafilococo, enterobacterias, Pseudomona, Candida. La neumona congnita se adquiere por via hematgena. DIAGNOSTICO:
Clnica; Inespecfica, puede causarse bito en lasprimeras 24h. Se manifiesta por dificultad respiratoria (polipnea, quejido y cianosis) asociado o no
a un signo clnico de infeccin (palidez, mal llenado capilar, alteraciones de termorregulacin, etc.) que se agrava rpidamente si no se instaura un
tratamiento adecuado. Datos clnicos sistmicos: letargo o irritabilidad, distermia, taquicardia o bradicardia, rechazo al alimento, residuo gstrico o
vomito, distensin abdominal, hepatomegalia, palidez, ictericia. Manifestaciones pulmonares: taquipnea, apnea, aleteo nasal, tiraje intercotsla o
xifoideo, quejido respiratorio, disociacin toracoabdominal, tos, estertores, cianosis. Considerar factores de riesgo descritos. Laboratorio:
hemograma (leucocitos fuera de rangos normales con predominio de neutrfilos en infecciones bacterianas, linfocitario o de monocitos en
infecciones virales, eosinfilos por chlamydia trachomatis), protena C reactiva, gasometra, glicemia, calcemia, Test Ltex, antgenos capsulares (en
orina, LCR). Parametros sugerentes de un proceso infeccioso: leucocitosis >20,000, leucopenia <5,000, relacin bandas/neutrfilos anormal >2.0,
plaquetopenia >100,000 Cultivos: de sangre, secrecin traqueal (tiles si se toman precozmente). Radiografa de trax. Inespecfico, consolidacin
difusa, parches blaterales con broncograma areo, infiltrado denso alveolar en uno o ambos pulmones, un dato importante es la persistencia de la
imagen radiolgica por mas de 48-72h (es raro ver consolidaciones). Chalmydia trachomatis: se adquiere la infeccion al pasar por el crvix y vagina,
sntomas a las 3-6 semnas de vida, ocasinalmente hemorragia pulmonar asociada, conjuntivitis en 35%, puede haber sibilancias, ausencia de fiebre, y
eosinofilia con aumento de IgM, en la radiografia hay infiltrado perihiliar difuso, torax hiperinsuflado, derrames y raro consolidacin. El tratamiento
es con eritromicina 10mg/kg por 14 dias c/6h. Listeria monocytogenes: Es un bacilo hemoltico gramm positivo, hay enfermedad febril en la madre.
Mortalidad 100%. Radiografia; patrn reticulonodular difuso, tratamiento ampicilina + aminoglucocido. Klebsiella y S. aureus pueden causar
microabscesos y neumatoceles. SINDROME DE FUGA DE AIRE: Neumomediastino (imagen de vela hinchada por el aire, un lbulo o varios del timo
que se elevan por arriba del corazn, hiperclaridades lineales o verticales, halo alrededor del corazn), neumotrax en 1-2% de los neonatos
(hiperclaridad perifrica, ausencia de arborizacin vascular), enfisema intersticial pulmonar, neumopericardio, neumoperitoneo, mancha de
meconio mas probabilidades. TRATAMIENTO: Medidas generales: monitorizacin, ambiente trmico neutral, balance H-E y cido base. Antibiticos:
inicia terapia de amplio espectro y luego, si existe confirmacin adecuar terapia especfica segn la epidemiologa de cada unidad de RN. Neumona
intrauterina: ampicilina (100-200mg/kg/dia)-amikacina (7.5mg/dia) 7-10 das. Neumona nosocomial: vancomicina 75 mg/kg/dia), cefotaxima (3050mg). NNN inicio precoz: Ampicilina (100-200mg/kg/dia) y Gentamicina (4-5mg/kg/dosis). NNN inicio tardo: Cloxacilina y Amikacina. Estafilococo;
dicloxacilina + aminoglucocido. Estrepotococo del grupo B; penicilina + aminoglucocido. Pseudomona; ceftazidima + ticarcilina. Infecciones
anaerobias; metronidazol (7.5-30mg/kg/dia). Manejo respiratorio segn gravedad, oxigenoterapia, ventilacin mecnica y otras medidas ms
recientes cuando se obtengan (xido ntrico, intubarlo). COMPLICACIONES: Choque sptico. Foco infeccioso en meninges, articulaciones.
Hipertensin pulmonar.
CASO CLINICO
Paciente masculino 3 dias que presenta datos de dificultad
respiratoria con rechazo a la alimentacin, su nacimiento fue a las 35
semanas de gestacion, cabe destacar presencia de conjuntivitis,
parto distocico, por via vaginal, la radiografia es inespecficas, solo
un infiltrado denso alveolar, la imagen fue persistente, los
laboratorios mostraron eosinofilia, no recibi factor surfatante pero
recibi oxigeno al 100 %, cabe destacar que mientras se encontraba
en unidad de cuidados intensivos presento distencin abdominal.
PREGUNTA
Cual es el agente etiolgico mas probable?
RESPUESTA
a.- Pseudomona
b.- Candida
c.- E. coli
d.- Cloacale

NEUMONIA NEONATAL TARDIA


Se trata de paciente masculino de 4 semanas de nacido, el cual
nacin prematuro por ruptura de prematuras de membrana el cual
presento taquinea transitoria del recin nacido, inicia
progresivamente con irritabiliad
PREGUNTA
Cual es el agente causal mas frecuente en esta patologia?
RESPUESTA
a.- klepsiella.
b.- Stafilococo.
c.- Psudomona.
d.- E. coli.

PREGUNTA
Cual es la conducta farmacologa a seguir considerando que los
cultivos aun no tienen datos para ser dirigido?
RESPUESTA

CURSO ENARM CMN SIGLO XXI TEL: 36246001

a.- Vancomicina 74mg/Kg y cefotaxima 50mg/kg


b.- Cefotaxima 20 mg/ Kg y metronidazol 30 mg/Kg
c.- Ampicilina 100 mg /kg mas amikacina 15mg/kg
d.- Doxiciclina 15 mg/kg mas gentamicina 15 mg/kg

CASO CLINICO NEUMONIA INTRAUTERINA


Se trata de recin nacido de 3 horas de nacido, el cual nace
prematuro, de madre adolecente con cervicovaginitis y parto

Pharmed Solutions Institute

PGINA 299

MANUAL DE TRABAJO DEL CURSO ENARM CMN SIGLO XXI


distcico, inicia con taquipnea, dificultad respiratoria, con aleteo
nasal, con disociacin toraco abdominal, con mala tolerancia a la via
oral, con hipotermia, Los datos radiogrficos de control se observo
persistencia de imgenes radiolgicas en ambos campos pulmonares
tipo infiltrado, con sndrome de fuga, el recuento de leucocitos
fueron predominio eosinofilos.
PREGUNTA
Cual es el agente etiolgico mas probable?
RESPUESTA
a- Clanmydia
b.- Klepsiella
c.- Candida
d.- Garnerella
PREGUNTA
Cual es el tratamiento farmacolgico en espera de los resultados de
cultivo?
RESPUESTA
a.- Ampicilina y amikacina.
b.- Ampicilina y gentamicina.
c.- Ceftriaxona y vancomicina.
d.- Imipenem y Cefotaxima
CASO CLINICO
Masculino de 35 semanas, que peso 2,800Kg, nacido por parto
distcico, con RPM de 18hrs, apgar 6/8. Madre con cervicovaginitis
recurrentes. A las 3 semanas de VEU, inicia con taquicardia, tiros
intercostales, retraccin xifoidea, disociacin toracoabdominal, tos,
irritabilidad, rechazo al alimento, ictericia. A la auscultacin
estertores y sibilancias. BH con Hb 10mg/dl, leucocitos 25,000 con
predominio de eosinofilos. Se realiza rx de trax la cual muestra
infiltrado perihiliar difuso y datos de hiperinsuflacion.

dilatacin) y expulsin de lquido amnitico con tinte meconial. Tuvo


RPM 1h 20 min antes del ingreso. Se indic anestesia de conduccin
con peridural continua y monitorizacin electrnica que revel una
variabilidad menor de 5 LCF, una frecuencia cardaca fetal media de
150 lpm y una desaceleracin variable prolongada (hasta 120 LCF
por minuto durante 3 min). Se reevalu luego de nueva
desaceleracin (60 LCF por minuto durante 7 min) y se encontr 8 a
9 cm de dilatacin, presentacin ceflica en plano II y LA con grumos
de meconio con leve mal olor. Se traslad inmediatamente a sala de
partos con dilatacin completa y presentacin ceflica en tercer
plano. Se obtuvo un recin nacido de 3,440g masculino, con muy
mal olor, con Apgar 1/3. El RN evolucion grave, con compromiso
del SNC, encefalopata hipxica isqumica grado III, asociado a shock
sptico precoz, con compromiso multiorgnico y necesidad de apoyo
ventilatorio y de drogas vasoactivas.
PREGUNTA
Cual es la complicacin ms probable en esta paciente?
RESPUESTA
a.- Coagulacion intravascular diseminada.
b.- Falla multiple
c.- Neurosepsis
d.- Neumonia asociada
CASO CLINICO
Se trata de paciente de 35 semanas de gestacion de 5 dias de nacido,
con ruptura prematura de membrana el cual requiri reanimacin
intensiva debido a pagar 2/6, se observo fiebre materna durante el
trabajo de parto, la madre vive en medio rural, apnea, crisis
convulsivas, vomito, distermia, taquicardia, disnea y deterioro,
hiporreactivo con piel marmrea, distencin abdominal, se
diagnostico sepsis neonatal.
PREGUNTA
Cuales de los diagnosticos diferenciales es el menos frecuente?
RESPUESTA
a.- Sindrome de adaptacin del recin nacido.
b.- Taquicardia transitoria del recin nacido
c.- Hemorragia ventricular
d.- Neumonitis

PREGUNTA
Cul es el agente etiolgico ms probable para este paciente?
RESPUESTA
a.- Listeria monocytogenes
b.- Klebsiella pneumoniae
c. Chlamydia trachomatis
d. Ureaplasma urealyticum
PREGUNTA
Cul es la conducta teraputica ms adecuada para este paciente?
RESPUESTA
a.- Eritromicina por 14 das
b.- Ampicilina-gentamicina por 14 das
c.- Vancomicina por 14 dias
d.-Dicloxacilina-gentamicina por 14 das
PREGUNTA
Generalmente estos pacientes cursan con infeccin en?
RESPUESTA
a.- Piel
b.- Conjuntiva
c.- vas urinarias
d.- gastrointestinal

PREGUNTA
Cual de los siguientes valores de laboratorio es ms probable
esperara?
RESPUESTA
a.- PCR elevada y neutrofilia
b.- Leucocitosis con eosinofilia
c.- Linfocitosis mas plaquetopenia
d.- VSG elevada con leucopenia.
PREGUNTA
Cual es la conducta farmacolgica empirico de primera eleccion en
espera de los resultados de cultivos?
RESPUESTA
a.- Ampicilina mas gentamicina.
b.- Ampicilina mas amikacina.
c.- Inmoglobulina y cefotaxima.
d.- Ampicilina y FEC

CASO CLINICO
Primigesta de 15 aos de edad, con gestacin de 40 semanas,
presenta trabajo de parto en fase de dilatacin rpida (5 cm de

CURSO ENARM CMN SIGLO XXI TEL: 36246001

Pharmed Solutions Institute

PGINA 300

MANUAL DE TRABAJO DEL CURSO ENARM CMN SIGLO XXI


SEPSIS NEONATAL
CIENCIAS BASICAS: Se define como un Sndrome de Respuesta Inflamatoria Sistmica (SRIS) en la presencia o como resultado de infeccin probada o
sospechada durante el primer mes de vida extrauterina. Los parametros de SRIS de 0-7 dias son: FC >180 o <100, FR >50 (>40 de 8-30 dias),
leucocitosis >34,000 (>19,500 o < 5000 de 8-30 dias), temp <36 o >38.5, banda >10%. Hipotension por debajo del percentil 5 para la edad, presencia
de al menos 2 de los criterios. CLASIFICACION: Temprana; del nacimiento a las 72h de vida, adquisicin transplacentaria, via ascendente o por el
canal de parto. Tardia; a partir de las 72h en adelante, adquisicin hospitalaria, meningitis, osteoartriris, enterocolitis necrotizante. Dependiendo de
la gravedad podemos tener: 1. SEPSIS= SRIS + hemocultivo positivo. 2. SEPSIS GRAVE= Sepsis + disfucion organica, hipotensin o hipoperfusion. 3.
CHOQUE SEPTICO= Sepsis grave con hipotensin que no responde a carga de liquidos. 4. Falla organica multiple. SALUD PUBLICA: Incidencia: 1 a 5
por cada 1000 nacidos vivos. 4 millones mueren de sepsis en las primeras 4 semanas. Cerca de un 85% de los pacientes requieren ventilacin
mecnica. 25% de los sobrevivientes presentan secuelas neurolgicas. Factores de riesgo: Sexo masculino, prematurez y bajo peso al nacer, ruptura
de membranas> 18 hrs, reanimacin al nacer y procedimientos invasivos, fiebre materna periparto, corioamnioitis. PATOGENIA: Etiologa: Sepsis
neonatal temprana: E. Coli, Klebsiella sp, Enterococcus sp. Streptococcus agalactiae y Listeria monocytogenes. Sepsis neonatal tarda:
Staphylococcus coagulasa negativa, staphylococcus aureus, Enterococcus sp, E Coli, Klepsiella, Candida spp, Streptococcus del grupo B. La presencia
de los mediadores hormonales e inmunolgicos que interviene directamente en la modulacin de la respuesta inflamatoria a la infeccin, definen
ms claramente los mecanismos involucrados en la respuesta inflamatoria aguda a la infeccin, adems de la respuesta de anticuerpos, la activacin
de las dos vas del complemento, la participacin del sistema de coagulacin, la respuesta fagocitica mononuclear, polimorfonuclear y de la
activacin linfocitara, ciertos mediadores que son sintetizados principalmente por monocitos, linfocitos, fibroblastos y clulas endoteliales actan
de manera determinante en la regulacin y modulacin de la respuesta inflamatoria inmunolgica a la infeccin. Las citocinas actan como
mediadores de la comunicacin intercelular y efectora de la activacin de funciones de las clulas que tiene receptores para estas protenas. Las
cuales son: interleucinas, interferones y factores de crecimiento. Segn sea su funcin se pueden encontrar las de actividad proinflamatoria (IL-1, IL6, TNFalfa, IL-2, IL-8, IL-12) otras antiinflamatorias o inhibitorias (Antagonistas, Receptores solubles, IL-4, IL-10, IL-13) y factores de crecimiento (GMCSF, G-CSF, M-CSF, IL-3, IL-6, IL-5, IL-7). La magnitud de citosinas proinflamatorias que se producen en etapa temprana de la sepsis se relaciona con
la intensidad del dao pulmonar. DIAGNOSTICO: El diagnstico temprano y oportuno de sepsis neonatal no es fcil porque las manifestaciones
clnicas son inespecficas y pueden avanzar rapidamente a estados ms avanzados. Los signos de alarma identificados por la OMS son los siguientes:
convulsiones, rechazo al alimento, dificultad respiratoria, hipoactividad, polipnea. Algunas de las principales inespecificas son: distermias, dificultad
respiratoria, irritabilidad, ictericia, apneas (con ms frecuencia en prematuros), hepatomegalia, letargia, palidez, oliguria, cianosis, piel marmrea,
crisis convulsivas, hepatoesplenomegalia, hipotensin arterial. En las especficas encontramos: fontanela abombada, distensin abdominal, aumento
de volumen articular, dificultad respiratoria. En sepsis temprana: 2 o mas datos de SIRS + cultivo positivo en sangre, orina o LCR, radiografia de torax
sugestiva de proceso neumnico, evidencia de foco infeccioso localizado; podra PCR>10, RPM de>12h, plaquetas < 100,000. En sepsis tardia:
nososcomial con la siguiente escala; temperatura >38.2= 5 puntos, protena C >14=5, neutrfilos >50%=3, nutricin parenteral >14. En sepsis
tarda se debe incluir adems urocultivo. En casos de infecciones localizadas (por ejemplo osteoartritis), se debe cultivar el sitio de infeccin.
TRATAMIENTO: Primera eleccin: Ampilicina- amikacina. El manejo emprico inicial de antibiticos debe hacerse con base en la experiencia de cada
hospital, siempre teniendo en cuenta el patrn de resistencia y sensibilidad. En sepsis neonatal temprana el tratamiento debe iniciarse con
ampicilina y un aminoglucsido (gentamicina amikacina), en ocasiones especiales se puede sustituir el aminoglcosido por cefotaxima, sobre todo
si existe la sospecha de neuroinfeccin (est demostrado que esteriliza el LCR con mayor rpidez). En recin nacidos con sepsis tarda adquirida en la
comunidad, es posible utilizar el mismo esquema, sin embargo en sepsis nosocomial, el tratamiento debe estar orientado a combatir los
microorganismos presentes en cada institucin. La evidencia actual de ensayos clnicos controlados aleatorizados no apoya el uso rutinario de
inmunoglobulina intravenosa y factor estimulante de colonias de granulocitos. El uso de pentoxifilina como adyuvante en el manejo de sepsis reduce
la mortalidad en neonatos pretrmino, sin embargo debido a debilidades metodolgicas de los estudios al respecto, no es adecuado utilizarlo de
manera rutinaria hasta la obtencin de mejor evidencia. Dependiendo de las condiciones clnicas del paciente en el caso de sepsis grave o choque
sptico se deber proporcionar apoyo ventilatorio, suministro de lquidos, aminas e incluso corticoesteroides en el caso de hipotensin refractaria a
las mismas o en caso de sospecha de insuficiencia suprarenal. Es indispensable realizar la correccin del equilibrio cido base y proporcionar apoyo
calrico y nutricional ya sea por va enteral o parenteral segn sea el caso. Pacientes con riesgo de sepsis: iniciar tratamiento revalorar 48 hrs. (el
95% de las infecciones se presenta en las primeras 24 hrs). Si no hay desarrollo de cultivos, biometra hemtica normal y PCR normal suspender el
tratamiento.
d.- Neumona
CASO CLINICO
Se obtiene recin nacida viva, de sexo femenino; llor y respir al
PREGUNTA
nacer, con Apgar de 8-9, peso de 2,400 gramos y talla de 48
Cul es la conducta teraputica ms apropiada en espera de
centmetros. EF: nicamente presenta equimosis en prpado
resultados de cultivos?
izquierdo y en hemicuello derecho, siendo normal el resto de la
REPUESTA
exploracin. Permaneci en la cuna tres das con evolucin normal.
a.- Vancomicina + cefotaxima
Al cuarto da de nacida, presenta fiebre de 39 C, irritabilidad,
b.- Ampicilina + gentamicina
succin dbil, piel marmrea, indiferencia al medio, hipoactiva,
c.- Vancomicina + gentamicina
hiporreactiva,
fontanela
anterior
normotensa,
reflejos
d.- Ampicilina + cefotaxima
osteotendinosos aumentados, pupilas con adecuada respuesta a la
luz, respiracin superficial; se observa ictrica (++). FC 185x, FR 50x,
PREGUNTA
Temp 38.2. BH leucocitos 35,000, bandas >10%, Neutrofilos 55%,
48hrs despus de estabilizar al paciente e iniciar antibitico, se
plaquetas 145,000, PCR 12mg/dl.
encuentra asintomtico y los cultivos de orina, LCR y sangre son
negativos, la BH se normalizo y la PCR bajo a 9mg/dl. Cul es la
PREGUNTA
conducta teraputica ms adecuada a seguir?
Cul es el diagnstico ms probable en este paciente?
REPUESTA
REPUESTA
a.- Apoyo calrico nutricional
a.- Sepsis temprana
b.- Aplicacin de esteroides
b.- Meningitis
c.- Suspender antibiticos
c.- Sepsis tarda
d.- Inmunoglobulinas intravenosas

CURSO ENARM CMN SIGLO XXI TEL: 36246001

Pharmed Solutions Institute

PGINA 301

MANUAL DE TRABAJO DEL CURSO ENARM CMN SIGLO XXI


ENTEROCOLITIS NECROTIZANTE (ECN)
CIENCIAS BASICAS: Se presenta como un sndrome gastrointestinal y sistmico, es una necrosis por coagulacin e inflamacin del intestino del
lactante. SALUD PUBLICA: En especial en prematuros, incidencia 0.3-2.5 por 1000 RN vivos y 1-5% de los RN ingresados a unidades neonatal, 90% en
prematuros, 10% en termino, en 90% se presenta sntomas en los 3-10 dias de nacidos. Aumenta la incidencia en los menores de 1500g, hasta un 210%. La edad gestacional media oscila en torno a las 31 semanas. Mortalidad de 10-30%. Factores de riesgo: prematurez, caterizacion umbilical,
hipotensin, hipotermia, bacterias (E. coli, proteus, klebsiella, enterobacter, pseudomona, clostridium, coronavirus, rotavirus), hipoxia-isquemiareperfucion, alimentacin enteral agresiva, policitemia, hiperviscosidad, uso de cocana materna. PATOGENIA: Mecanismo multifactorial se produce
lesion en la mucosa intestinal y desencadena la enfermedad sistmica. Entre los factores propuestos implicados en la patognesis de la ECN se han
descrito la prematuridad, alimentacin lctea, inestabilidad hemodinmica, infeccin y alteracin de la mucosa intestinal. Solo la prematuridad y la
alimentacin lctea tienen una base epidemiolgica consistente. En prematuros, la predisposicin de estos nios pudiera explicarse por la
inmadurez de su tracto gastrointestinal con funcin luminal limitada que conlleva una absorcin parcial de carbohidratos y grasas, existe mayor
permeabilidad de la mucosa, deficiencia de IgA, inadecuada de macrfagos, neutrfilos y citoquinas, por ello hay peristaltismo disfuncional, as como
traslocacion y proliferacin bacteriana a la pared
intestinal, fermentacin de sustratos que lleva a
neumatosis, que conlleva a un proceso inflamatorio y
eso es la via final del dao del tubo digestivo y
sistemico. La lesin mucosa puede provenir de
distintos procesos, incluyendo la asfixia o la isquemia
secundaria a la derivacin de la sangre lejos del
intestino. Se desconoce la especificidad de este
proceso. DIAGNOSTICO: El hallazgo ms precoz suele
ser un cambio en la tolerancia alimentaria en un nio
prematuro, con buena evolucin hasta ese momento y
que comienza a presentar restos gstricos. Los
sntomas sistmicos asociados son inespecficos, desde
aparicin de apneas, alteracin del patrn respiratorio, distermia, inestabilidad hemodinmica con bradicardias, hasta hipotensin, letargia o shock
sptico y CID. Desde el punto de vista gastrointestinal, la ECN se presenta con distensin abdominal, restos gstricos (>30-50%), abdomen doloroso,
dibujo de asas intestinales, cambios de coloracin de la pared abdominal (signo de Cullens), vmitos, diarrea o hematoquecia. En la forma grave se
puede agregar piel color terroso, choque, sangrado generalizado, eritema periumbilical. En perforacin: cambos de color en abdomen, ausencia de
gas abdominal, ausencia de neumatosis intestinal. El curso de la enfermedad vara segn recin nacidos. Con mayor frecuencia aparece como: 1)
una presentacin fulminante, rpidamente progresiva de signos congruentes con necrosis intestinal y sepsis, y 2) como una presentacin lenta,
paroxstica, de distensin abdominal, leo y posible infeccin. Las zonas mas afectadas son ileon y colon proximal. Suele encontrarse intestino
dilatado, con paredes muy delgadas y algunas zonas hemorrgicas con depsito de fibrina. Se encuentran habitualmente perforaciones y zonas de
necrosis transmural sobre el borde antimesenterico. Laboratorio y gabinete: La radiografa abdominal (confirma diagnostico) suele revelar un patrn
gaseoso, edema de la pared intestinal, la posicin fija de un asa en estudios seriados, la aparicin de una masa, la neumatosis intestinal (95%) ms
frecuente en cuadrante inferior derecho, el aire venoso portal o heptica, la neumobilia o el neumoperitoneo (cuando hay perforacin intestinal).
Estudios sanguneos; anemia, trombocitopenia, la acidosis metablica persistente y la hiponatremia refractaria grave son la triada ms frecuente y
ayudan a confirmar el diagnstico, puede haber plaquetopenia, hipocalcemia. La trombocitopenia se asocia a necrosis intestinal y empeoramiento
clnico. El anlisis de las heces para detectar sangre e hidratos de carbono, si los hay, se ha utilizado para diagnosticar a los recin nacidos afectos de
ECN, basado en los cambios de la integridad intestinal. La malabsorcin de los hidratos de carbono, segn refleja un Clinitest positivo (cuerpos
reductores) en las heces, puede ser un indicador frecuente y precoz de ECN en el marco de los signos. Otros hallazgos bioqumicos inespecficos son
la elevacin srica de PCR y alfa-1-glicoproteina o de la alfa-1-antitripsina en heces. Ante la posibilidad de sepsis deben recogerse cultivos
(hemocultivo, cultivo de LCR, coprocultivo). Si se sospecha Clostridium son necesarios cultivos especficos y determinacin de toxina. La visualizacin
directa de la mucosa entrica mediante endoscopia. Debe considerarse la ultrasonografa como mtodo til en el diagnostico cuando existe
sospecha clnica, no confirmada por radiologa. TRATAMIENTO: IA: antibitico 3 dias, ayuno de 4-14 dias. IB, IIA; ayuno, antibitico 7-10 dias. IIB;
antibiticos, bicarbonato. IIIA; anibioticos, bicarbonato, ventilador, paracentesis. IIIB; ciruga, laparotmia, drenaje peritoneal, reseccin de asas
necrticas, ostoma vs cierre primario. Tratamiento antibitico parenteral de 10-14 dias con ampicilina (100-200mg/kg/da c/12hrs, en menores de
2,000Kgs 50mg) + gentamicina (4-5mg/kg/dosis) (o cefotaxima 80-100mg/kg/dia), vancomicina en casos de catteres centrales o infeccion
estafiloccica. En perforacin intestinalo peritonitis usar metronidazol (7.5-30mg/kg/dia) o clindamicina (10-15mg/kg/dia). Medico: AYUNO, sonda
orogastrica, signos vitales, medicin de permetro abdominal, soporte respiratorio, circulatorio, liquidos, laboratorio, nutricin parenteral,
antibioterapia y correccin de alteraciones hematolgicas y/o electrolticas que pudieran estar presentes. Deben suspenderse todos los frmacos
relacionados como posibles factores de riesgo. Como medidas de soporte se incluye la asistencia respiratoria temprana ante la aparicin de apneas o
patrn respiratorio acidotico, la correccin de la acidosis, hiponatremia o trombopenia. La acidosis metablica persistente es un indicador de
progresin de la lesin intestinal e incluso necrosis. Se debe asegurar una perfusin y transporte de oxigeno adecuados, con un aporte suficiente de
lquidos y manteniendo el nivel de hematocrito >35%. Puede ser necesario el uso de agentes inotrpicos. La dopamina a dosis bajas puede ser de
ayuda para mejorar la perfusin sistmica y aumentar el flujo mesentrico. Habitualmente la ampicilina y gentamicina constituyen el tratamiento
adecuado, debiendo asociar clindamicina o metronidazol ante sospecha de grmenes anaerobios, si bien la clindamicina ha sido asociada con un
aumento de la frecuencia de estenosis postenterocoliticas. La dieta y la antibioticoterapia han de mantenerse durante 10-14 das, con introduccin
progresiva de aporte enteral, con frmulas hipoosmolares de hidrolizado de protenas. Quirrgico: El momento ideal sera aquel en que se ha
producido una gangrena intestinal pero todava no existe perforacin, ni peritonitis secundaria. Indicada en pacientes que progresa con necrosis de
pared intestinal y perforacin debe ser urgente debido al rpido deterioro. La decisin es clara en aquellos en que destaca la presencia de
neumoperitoneo. El empeoramiento progresivo nos alerta sobre necrosis intestinal como: signos de peritonitis como edema y eritema de pared,
masa abdominal, trombopenia y acidosis persistentes. Paracentesis: un resultado positivo es altamente especfico de necrosis intestinal.
COMPLICACIONES: Estenosis, fistulas intestinales, sndrome de intestino corto.
tolerancia al destete, por lo cual se inicia estmulo enteral con
CASO CLINICO ENTEROCOLITIS NECROTIZANTE
adecuada tolerancia. Los laboratorios iniciales hemograma normal y
Al segundo da de vida presenta resolucin de su dificultad
PCR negativa. Se contino vigilancia clnica y aumento progresivo de
respiratoria con disminucin de requerimientos de oxgeno y
la va oral. A los 5 das de vida presenta cuadro de episodio emtico

CURSO ENARM CMN SIGLO XXI TEL: 36246001

Pharmed Solutions Institute

PGINA 302

MANUAL DE TRABAJO DEL CURSO ENARM CMN SIGLO XXI


de contenido alimentario sin otra sintomatologa el cual se
interpret como reflujo gastroesofgico. Al da siguiente presenta
deterioro clnico dado por taquicardia, distensin abdominal,
persistencia de episodios emticos postprandiales, residuo gstrico
del 70%, deposiciones con sangre macroscpica e hipoglicemia.
PREGUNTA
Considerando los hallazgos clnicos. Cual es el estadio en el que se
encuentra?
RESPUESTA
a.- Enterocolitis necrotizante I
b.- Enterocolitis necrotizante II
c.- Enterocolitis necrotizante III
d.- Enterocolitis necrotizante IV
PREGUNTA
Cual es la conducta a segur mas adecuada en este momento?
RESPUESTA
a.- Se suspende la va oral, BH, QS, cultivos, ampicilina y
gentamicina.
b.- Ampicilina + amikacina, Bh, QS, cultivos, ayuno.
c.- Alimentacion parenteral, antibioticoterapia empirica, BH, QS,
cultivos.
d.- Ampicilina, cefotaxima, alimentacin por sonda, BH y QS.
PREGUNTA
Tres das despus el paciente presenta hipotensin e hipoperfusin
asociado a falla respiratoria por lo que requiere soporte inotrpico y
ventilatorio. Presenta acidosis metablica, trombocitopenia y
alteracin en
pruebas de coagulacin, cultivos con E.coli
multiresistente. Cal es la conduca a seguir en el paciente?
RESPUESTA
a.- Transfusin de plasma y plaquetas.
b.- Crioprecipitados, albumina y cristaloides.
c.- Piperacilina tazobactam y nutricin parenteral.
d.- Piperacilina tazobactam, amikacina y nutricin parenteral.
CASO CLINICO ENTEROCOLITIS NECROTIZANTE
Recin nacido varn, con antecedentes de parto de pretrmino a las
32 semanas de gestacin, con peso de 1 300 g y Apgar 8 y 10 al
nacer. Estando relativamente bien, a las 72 horas de vida comienza
con rechazo alimentario y distensin abdominal. Un da ms tarde se
agregan vmitos biliosos, deposiciones con sangre e inestabilidad
hemodinmica.
PREGUNTA
Se realizo una radiografia para confirmar diagnostico. Cal de los
siguientes datos no esta presente en esta?
RESPUESTA
a.- Dilatacin de asas intestinales de colon e intestino delgado.
b.- Engrosamiento de paredes y extensa neumatosis intestinal.
c.- Mltiples imgenes areas pequeas, en aspecto de "burbujas".
d.- Neumoperitoneo.
PREGUNTA
Considerando las imgenes observadas. Cal es diagnostico final y
complicaciones asociadas?
RESPUESTA
a.- Enterocolitis necrotizante complicada, con perforacin intestinal
y neumoperitoneo secundario.
b.- Enterocolitis necrotizante complicada, sin perforacin intestinal
ni neumoperitoneo secundario.
c.- Enterocolitis necrotizante simple, con preforacion intestinal.
d.- Enterocolitis complicada con peritoneo secundario.
PREGUNTA
Cual de las siguientes complicaciones agudas es mas frecuente?
RESPUESTA

CURSO ENARM CMN SIGLO XXI TEL: 36246001

a.- Necrosis intestinal con perforacin


b.- Peritonitis secundaria.
c.- Formacin de abscesos intraabdominales.
d.- Coagulacin intravascular diseminada.
CASO CLINICO ENTEROCOLITIS NECROTIZANTE
Se trata de un paciente varn de 5 meses de vida que ingres en
nuestro hospital por presentar vmitos proyectivos en la mitad y al
final de las tomas, acompaados de una prdida de peso de 350g en
el transcurso de una semana. Como antecedentes destacaba que era
un exprematuro moderado de 30 semanas de gestacin que al
quinto da de vida desarroll una EN con neumatosis gstrica e
intestinal, por lo que precis una reseccin de 10cm del yeyuno y
25cm del leon. Tras la intervencin quirrgica present varios
episodios de retencin gstrica y vmitos, con resolucin
espontnea y adecuada tolerancia posterior, por lo que se le dio el
alta a su domicilio.
PREGUNTA
Cual es la conducta a seguir en este caso, con la sintomatologia
presente cual es estudio de primera eleccin para identificar la causa
de la conducin actual?
RESPUESTA
a.- Trnsito esofagogastroduodenal.
b.- Radiografia de abdomen.
c.- Ultrasonograma abdominal.
d.- Endoscopia digestiva.
CASO CLINICO
Recin Nacido masculino pretrmino de 36 semanas con peso
adecuado para la edad gestacional 2,460kgs. Madre de 30 aos,
producto de tercera gestacin, embarazo gemelar, controles
prenatales positivo. Antecedente de preeclampsia en embarazos
anteriores y hallazgos de NIC III en citologa vaginal. A los 5 das de
vida presenta cuadro de episodio emtico de contenido alimentario
sin otra sintomatologa el cual se interpret como RGE. Al da
siguiente presenta deterioro clnico dado por taquicardia, distensin
abdominal, persistencia de episodios emticos postprandiales,
residuo gstrico del 70%, deposiciones con sangre macroscpica,
equimosis periumbilical.
PREGUNTA
Cules son los principales agentes etiolgicos en estos pacientes?
RESPUESTA
a.- Pseudomonas, Clostridium perfringens
b.- Klebsiella, E.coli
c.- Clostridium perfringens, S, epidermidis
d.- E. coli, Clostridium perfringens
PREGUNTA
Posteriormente presenta acidosis metablica, as como
trombocitopenia leve y celulitis abdominal. Cul es la conducta
teraputica mas adecuada en este paciente?
REPUESTA
a.- Administracin de antibitico, bicarbonato, ayuno
b.- Administracin de antibitico y ayuno
c.- Administracin de antibitico, bicarbonato, paracentesis, ayuno
d.- Administracin de antibitico, bicarbonato, ayuno, ciruga
PREGUNTA
Si el paciente evoluciona a peritonitis o perforacin intestinal
Cules son los antibiticos de primera eleccin?
RESPUESTA
a.- Ampicilina, ciprofloxacino, metronidazol
b.- Cefotaxima, ampicilina, metronidazol
c.- Ampicilina, gentamicina, metronidazol
d.- Ampicilina, clindamicina, metronidazol

Pharmed Solutions Institute

PGINA 303

MANUAL DE TRABAJO DEL CURSO ENARM CMN SIGLO XXI


MENINGITIS NEONATAL
CIENCIAS BASICAS: Es una enfermedad infecciosa con secuelas a corto y largo plazo, siendo su tratamiento costoso, con aumento de los das de
hospitalizacin y nmero de das recibiendo antibiticos, se considera neonatal de 28 dias a 6 semanas. Meningitis bacteriana: sndrome clnico
compatible con meningitis, ms aislamiento de un organismo por cultivo del lquido cefalorraqudeo o aislamiento de un organismo por cultivo de
sangre y anormalidad del LCR consistente con infeccin bacteriana. SALUD PUBLICA:
COMPONENTE
MENINGITIS
ENCEFALITIS
MENINGITIS
Frecuencia variable. 25-30% de los casos de sepsis neonatal se complica con meningitis.
BACTERIANA
VIRAL
HERPES
Meningitis bacteriana tiene incidencia hasta de 1 x 1000 nacidos vivos.Fcatoresde
Leucocitos
>1000
100-500
10-10,000
riesgo; infeccion perinatal e intrauterina, prematurez, RPM, peso bajo al nacimiento,
Neutrofilos
>50
<40
<50
Glucosa
<30
>30
<30
aspiracin inhalacin.
CLASIFICACION: Meningitis temprana: Primera semana
Proteinas
>100
50-100
>75
transmisin vertical (grmenes localizados en el canal vaginal materno) grmenes
Eritrocitos
0-10
0-2
10-500
involucrados, Streptococcus del grupo B, E. coli, Listeria monocytogenes, Enterococcus.
Meningitis tarda: Nosocomial adquirida en la comunidad grmenes involucrados, transmisin horizontal Gram negativos, Estafilococos.
PATOGENIA: Etiologa: gram (-): E. coli, Klebsiella (30-50%), Serratia marcescens, Proteus, Enterobacter, Neisseria, Pseudomona. Gram (+):
Streptococo (30-40%), Enterococos, Staphylococcus aureus, estafilococo coagulasa negativo. Otros: Lysteria monocytogenes (10%). Es la
diseminacin hematgena en la mayora de los casos, sin embargo se producen por propagacin por contigidad como consecuencia de la
contaminacin de defectos del tubo neural, fstulas congnitas o de heridas penetrantes causadas al obtener muestras de sangre, partos spticos o
domiciliarios. La cerebritis y los infartos spticos son frecuentes en la meningitis bacteriana. La formacin de abscesos, la ventriculitis, la hidrocefalia
y los derrames subdurales aparecen ms a menudo en recin nacidos que en nios mayores. Los hallazgos patolgicos son similares entre los
diferentes agentes bacterianos, las bacterias gramm tienen lipopolisacaridos, y las gramm +, producen ac. Teicoico, que son inductores de
inflamacin y con ello se daa la barrera hematoencefalica. El ms comn en autopsias es exudado purulento en meninges y superficie ependimal
de los ventrculos. Se evidencia tambin inflamacin perivascular y gran respuesta inflamatoria. Hidrocefalia y encefalopata no infecciosa puede ser
demostrada en aproximadamente 50% de los neonatos con meningitis. La efusin subdural raramente ocurre en neonatos. Varios grados de flebitis
y arteritis de vasos intracraneales pueden presentarse. Ventriculitis puede ser demostrada en virtualmente todos los neonatos y en 75% al momento
del diagnstico. Parte de la fisiopatologa de debe a inmadurez inmunitaria, disminucin de actividad de complemento, de clulas T, produccin de
inmunoglobulinas, se puede adquirir meningitis por el ambiente, y se inicia en las mucosas (rinitis). Corioamnionitis clnica: presencia de fiebre
materna >38 C, con dos o ms de los siguientes hallazgos: taquicardia fetal (>160), sensibilidad uterina a la palpacin, descarga vaginal ftida o
leucocitosis materna. DIAGNOSTICO: Clnica: fiebre o hipotermia 60%, Irritabilidad 60%, rechazo a alimentacin/vomito (en proyectil), 48% crisis
convulsivas (generalizadas, si es focalizada, sospechar absceso cerebral o neumonia), 42% dificultad respiratoria 33% apnea 31% diarrea 20%
fontanela abombada 25%, rigidez de nuca 13% (hacer dx. Diferencial con hemorragia). La sintomatologa de un paciente con meningitis es muy
inespecfica y puede ser muy similar a la de
un paciente con sepsis y las manifestaciones
como fontanela abombada, alteracin del
estado de conciencia, convulsiones y coma,
cuando se presentan se asocian a pobre
pronostico; por lo que es importante que
dentro de la evaluacin del neonato
infectado se incluya la puncin lumbar
incluso en la ausencia de signos neurolgicos evidentes. Laboratorios: 1. BH, recuento de leucos y porcentaje de PMN: <20% >80% ndice
inmaduros/ maduros totales: 0.2 - Plaquetas <100.000 2. PCR: Es significativo cuando esta sea > 10mg/dl 3. Glicemia: si presenta valor <40mg/dl
>120mg /dl 4. Hemocultivos: Siendo positivos hasta un 50 %. Citoqumico, Gram. Diagnostico por puncion de lumbar; LCR muestra de 10-30ml en
prematuros y 40ml en RN de trmino, realizarle, tincin de GRAM, cultivo de LCR: 1) Glucosa menor a dos tercios la glucosa srica medida por
glucometria simultnea. 2) Relacin glucosa LCR/ srica mayor de 0.5. 3) Protenas: Se considerar positivo para infeccin valor mayor de 90 mg/dl
para recin nacido a trmino y de ms de 150md/dl en prematuros. 4) Celularidad: Mayor de 10 clulas blancas, hacr cuenta basoluta de neutrfilos
(CAN). 5) Cultivo positivo. Contraindicaciones para la puncin lumbar (LCR: 10-30 ml prematuro, RN termino 40 ml): compromiso cardiorespiratorio,
CID, lesiones en el sitio de la puncin, prematuros extremos (menores de 1000 gramos). Imagenologa: Hacerla para detecta complicaciones,
pacientes choqueados, falla respiratoria, dficit neurolgico, cultivo + despus de 48-72 Hs con apropiada terapia. TAC para descartar abscesos.
Contraindicaciones para puncion lumbar; aumento de presin intracraneal, hiperemia o infeccion en sitio de puncion, paciente chocado o inestable,
TAC con crneo hipertensivo, hemorragia IV, plaquetas <150,000. Si el paciente es lactante de 2-3 meses y presenta crisis epilptica y fiebre es
meningitis hasta no demostrar lo contrario. TRATAMIENTO: Sepsis temprana: ampicilina+amikacina o gentamicina (2.5mg/kg/dosis) primera
eleccion. Sepsis tarda: cefotaxima+amikacina. Especfico para el agente identificado. Otros: Meropenem, Ticarcilina, Vancomicina. Meningitis
temprana: ampicilina (100mg/kg/dosis c/12hrs) mas cefotaxima (50mg/kg/dosis c/12hrs), meningitis nosocomial, ampicilina mas cefotaxima mas
vancomicina (30-45mg/kg/dia). Duracin terapia: 14-21 das. Esteroides: ayuda a la penetracin del antibitico en la barrera hematoencefalica, se
debe pasar 30 min. antes que el antibitico, tambin ayuda a disminuir el proceso inflamatorio. Inmunoglobulina (400mg/kg/dosis). Aunque el
manejo final deber basarse en el resultado del antibiograma de las muestras de los hemocultivos, el tratamiento emprico iniciado con penicilinas
ms una cefalosporina de tercera generacin y/o un aminoglucsido tipo gentamicina preferencialmente, despus de haberse tomado las muestras,
es el que ha mostrado la mejor de las respuestas clnicas. El tratamiento endovenoso deber continuarse con el antibitico apropiado para el
germen aislado, hasta dos semanas despus de la negativizacin del hemocultivo. COMPLICACIONES: Choque (inmediatas), Ventriculitis 20%
(mediata), absceso cerebral 13%, hidrocefalia 24% (tardia), perdida auditiva 12%, ceguera cortical <10%, crisis tardas 19%, la mas frecuente es
retraso en el neurodesarrollo hasta 26%. Desafortunadamente, las secuelas severas a largo plazo son frecuentes (12% a 29% de los afectados),
especialmente en las infecciones asociadas a Streptococcus del grupo B y a bacilos gram negativos, an en los casos en los que el tratamiento ha sido
oportuno y adecuado. Mortalidad de 20-40%, las secuelas en los sobrevivientes es de 50%.
CASO CLINICO MENINGITIS NEONATAL
Paciente masculino 10 dias de nacimiento, que se obtuvo en casa sin
control prenatal, ingresa por diarrea, llanto, e irritabilidad, vomito en
proyectil, en casa, refiere la madre que solo fue en una ocacion, al
ingreso se observo hipotnico, llanto agudo, mal estado
generalizado, con tono cervical no se observo rigidez, pero fontanela

CURSO ENARM CMN SIGLO XXI TEL: 36246001

abombada, su peso fue de 2050 grs, laboratorios con 150,000 de


plaquetas, antecedentes de rinorrea hialina, se presento 35 grados,
irritabilidad, durante la exploracin se observo pedaleo y chupeteo,
dificultad respiratoria.
PREGUNTA

Pharmed Solutions Institute

PGINA 304

MANUAL DE TRABAJO DEL CURSO ENARM CMN SIGLO XXI


Cul es la conducta diagnostica mas adecuada para conducir su
teraputica para el caso?
RESPUESTA
a.- Hemocultivo
b.- Puncion lumbar
c.- Tomografia
d.- Biometria hemtica.
PREGUNTA
Se encuentra en espera de resultados, sin embargo el paciente se va
deteriorando. Cal es la agente etiologico mas probable para dar
una teraputica dirigida?
a.- H influenza
b.- E Coli
c.- Listeria monocitogenes
d.- Estreptoco beta hemolito.
PREGUNTA
Cual es la conducta farmologica ms adecuada considerando la
etiologa?
RESPUESTA
a.- Ampicilina, amikacina.
b.- Ampicilina mas metronizadol.
c.- Ampicilina mas vancomicina.
d.- Ampicilina mas cefotaxima.
PREGUNTA
Se administro esteroides en el paciente. Cal es la razn mas
correcta para realizar esta conducta en el caso?
RESPUESTA
a.- Disminuir los efectos de los factores inflamatorios.
b.- Favorece la penetracin del frmaco
c.- Disminuir la probabilidad de crisis convulsivas.
d.- Prevenir el edema cerebral.
PREGUNTA
Cual es la secuela mas frecuente que los pacientes con esta
patologia desarrollan?
RESPUESTA
a.- Empiema cerebral
b.- Retraso en el neurodesarrollo
c.- Hidrocefalia.
d.- Paralisis cerebral.
CASO CLINICO
Recin nacido masculino de 2 das de vida extrauterina, de 36
semanas, peso 2,600Kg. Madre curso con IVU en ltimo trimestre,
obtenido por parto, previa ruptura de membranas de 14 hrs. Control
prenatal irregular. El paciente presenta crisis convulsiva, tiros
intercostales, aleteo nasal, retraccin xifoidea), fontanela
abombada, diarrea, rechazo al alimento, irritabilidad. Laboratorios
PCR 20mg/dl, Leucocitos 38,000, plaquetas 99,000, glucosa 38mg/dl
PREGUNTA
Cul es la contraindicacin ms probable en este paciente para
posponer la puncin lumbar?
RESPUESTA
a.- Hipoglucemia
b.- Prematuro extremo
c.- Leucocitosis
d.- Datos de dificultad respiratoria
PREGUNTA
Si se estabiliza al paciente la muestra de LCR de cuantos mililitros
debe ser para este paciente?
RESPUESTA
a.- 20ml
b.- 40 ml

CURSO ENARM CMN SIGLO XXI TEL: 36246001

c.- 5ml
d.- 50ml
PREGUNTA
Cules son los sntomas ms comunes de encontrar en meningitis?
RESPUESTA
a.- Alteraciones en la termorregulacin, irritabilidad
b.- Dificultad respiratoria, diarrea
c.- Rigidez de nuca, fontanela abombada
d.- Apnea, crisis convulsivas
CASO CLINICO INFECCION NEONATAL
Recin nacido de 12 das de vida con fiebre de 38,5C de una hora de
evolucin y rechazo de la alimentacin. Como antecedentes; ruptura
espontnea de membranas intraparto, parto instrumental con
frceps a las 41 semanas de gestacin y peso al nacimiento de 3880
g. Apgar 8/9. Comenz su esquema de inmunizacin. Alimentacin
con lactancia materna exclusiva. La exploracin fsica mostr
decaimiento y regular perfusin perifrica con constantes vitales
normales. La analtica inicial mostr una determinacin de protena
C reactiva cuantitativa (PC-R) 133,4 mg/L, con un hemograma
coagulado y un sedimento de orina normal. La puncin lumbar
result hemorrgica. Ingres con diagnstico de sndrome febril sin
foco y se inici antibioterapia emprica con ampicilina y cefotaxima
intravenosas tras la toma de cultivos. En el hemograma repetido se
observaron 5800 leucocitos/mm3, con frmula normal, 13,4 g/dl de
hemoglobina, 32 000 plaquetas/mm3. A las 18 h del ingreso
empeor su estado general y present crisis convulsivas
generalizadas que requirieron tratamiento con fenobarbital y
perfusin de midazolam. En la evolucin desarroll afectacin
sistmica con hipotensin arterial, acidosis metablica,
hiponatremia, oliguria y plaquetopenia grave; requiri ventilacin
mecnica convencional, transfusin de hemoderivados, soporte
inotrpico con dopamina y diurtico con furosemida. El tercer da
del ingreso se inform el aislamiento en el hemocultivo de
Streptococcus pyogenes sensible a penicilina, por lo que se
suspendi la ampicilina y se mantuvo el tratamiento con cefotaxima
intravenosa.
PREGUNTA
Cual es la causa ms probable de las crisis convulsivas?
RESPUESTA
a.- Hemorragia cerebral.
b.- Alteracion hidroelectrolitica.
c.- Neurosepsis.
d.- Encefalopatia hipoxico-isquemica.
PREGUNTA
Se le realiza una puncion lumbar nuevamente, Qu esperaramos
encontrar en este?
RESPUESTA
a.- Neutrofilos <40, glucosa >50, protenas de 80, eritrocitos 0,
leucocitos 100
b.- Neutrofilos >60, glucosa <40, protenas de 100, eritrocitos 2,
leucocitos 1100
c.- Neutrofilos <40, glucosa <30, protenas 100, eritrocitos 400,
leucocitos 3000
d.- Neutrofilos <40, glucosa >50, protenas 30, eritrocitos 2,
leucocitos 2000
PREGUNTA
Cul es la complicacin tardia menos probable que presente este
paciete?
RESPUESTA
a.- Prdida auditiva
b.- Ceguera cortical
c.- Absceso central
d.- Retraso en neurodesarrollo

Pharmed Solutions Institute

PGINA 305

MANUAL DE TRABAJO DEL CURSO ENARM CMN SIGLO XXI


CASO CLINICO INFECCION NEONATAL
Neonato de 11 das de vida, previamente sana, ingresada por
dificultad respiratoria, mala coloracin, rechazo de tomas y
decaimiento. Constantes vitales; TA de 95/65 mmHg, FR de 160 lpm,
FR de 90 rpm y saturacin de oxgeno del 94% con FiO2 del 30%. Se
auscultan ruidos crepitantes e hipoventilacin basal izquierda. La
radiografa de trax evidencia infiltrado basal izquierdo.

PREGUNTA
Cual es el tratamiento empirico de primera eleccin?
RESPUESTA
a.- Ampicilina mas gentamicina
b.- Ampicilina mas amikacina
c.- Amikacina mas cefotaxima.
d.- Vancomicina, ampicilina mas cefotaxima.
PREGUNTA
Al paciente se le administro esteroides. Cal es el objetivo
farmacolgico para esta indicacin?
RESPUESTA
a.- Como medida antiedema.
b.- Para sinergismo con inmunoglobulina.
c.- Sinergismo con los antibiticos.
d.- Estabilizacin de membrana.

PREGUNTA
Cual es la conducta a seguir?
RESPUESTA
a.- Amoxicilina mas gentamicina.
b.- Trimetoprim mas sulfametoxazol.
c.- Cetriaxona.
d.- Vancomicina.
CASO CLINICO MENINGITIS NEONATAL
Se trata de paciente que se encuentra en la UCIN con diagnostico de
sepsis neonatal, prematuro, bajo peso y masculino con 10 dias de
nacido, inicia con llanto continuo, irritable, sin rigidez de nuca, se
observa hipotnico, rechazo al alimento, vomito en proyectil.
Antecedentes con aspiracin de meconio con sufrimiento e hipoxia,
por parto prolongado y distocia, se observo disminucin de
complemento y linfocito,

PREGUNTA
Cual de las siguientes complicaciones es menos frecuente esperar?
RESPUESTA
a.- Ventriculitis
b.- Hidrocefalia.
c.- Prdida auditiva y ceguera.
d.- Sindrome de Rett.

PREGUNTA
Cual es el estndar de oro para el dianostico mas adecuado?
RESPUESTA
a.- Hemocultivo y urocultivo.
b.- Liquido cefaloraquideo.
c.- Biometria hemtica y EGO.
d.- Marcadores inflamatorios.

PREGUNTA
Condiderando los criterios para definir sepsis, es un SRIS, aunado a
un agente infeccioso en el RN?
RESPUESTA
a.- Horas de nacido, >130 FC.
b.- FR ms de 50
c.- Leucocitos ms de 16,500.
d.- Temperatura 37.8 y 10% de bandas 10 %

PREGUNTA
Cual es el agente etiolgico mas probable en el caso clnico actual?
RESPUESTA
a.- E. coli
b.- Klebsiella
c.- Pseudomona
d.- Salmonela

PREGUNTA
Cual es el agente causal mas probable en este caso especifico?
RESPUESTA
a.- Listeria monocitogenes.
b.- Stafilococco
c.- Enterococcus.
d.- Candida A.

PREGUNTA
Cuales son las siguiente manifestaciones es la que presenta mayor
frecuencia y mayor valor diagnostico?
RESPUESTA
a.- Fiebre e irritabiliad
b.- Vomito y crisis convulsivas.
c.- Alteraciones metabolicas
d.- Fontanela abombada.

CASO CLINICO
Masculino de 2 das de edad, hijo de madre de 27 aos, G3, P3, con
una edad gestacional de 38 semanas por FUM y 41 por Capurro que
fue controlada en 4 ocasiones de forma prenatal. Madre con
antecedente de flujo vaginal de color amarillento das antes del
parto, con fiebre antes y en periodo postparto inmediato mas dolor
en hemiabdomen inferior. El parto fue vaginal conducido. Al nacer se
le califica Apgar: 9 al minuto, 9 a los 5 minutos. Peso al nacer de
3700 g, talla: 51 cm. Los padres notan dificultad respiratoria
asociando posteriormente hipoactividad, rechazo al alimento y
aumento de la dificultad respiratoria. EF: fontanela anterior de 1 cm
por 1 cm, suturas cabalgadas. Con quejido inspiratorio, retracciones
subcostales, aleteo nasal, roncus y crpitos bilaterales. Hgado a 4
cm bajo reborde costal, petequias en abdomen. Neurolgico no
valorable por sedacin. Llenado capilar lento, pulsos presentes. Se
decide intubar por fallo respiratorio ameritando soporte ventilatorio
oscilador de alta frecuencia. La radiografa de trax al ingreso
mostr infiltrados intersticiales difusos bilaterales abundantes.

PREGUNTA
Cuales son los valores que no es probable observar en este
paciente?
RESPUESTA
a.- Protena de aumentada
b.- Glucosa 2/3 del central.
c.- Leucocitos elevados
d.- Acido lctico disminuido.
PREGUNTA
Cual es la complicacin anatomopatologia mas frecuente?
RESPUESTA
a.- Hemorragia ventricular.
b.- Ventriculomegalia
c.- Encefalomacia.
d.- Ventriculitis.

CURSO ENARM CMN SIGLO XXI TEL: 36246001

PREGUNTA
Cul es el diagnotico ms probable en este caso?
RESPUESTA
a.- Neumonia neonatal
b.- Sepsis neonatal
c.- Coagulacion intravascular diseminada
d.- Sindrome de dificultad respiratoria tipo I

Pharmed Solutions Institute

PGINA 306

MANUAL DE TRABAJO DEL CURSO ENARM CMN SIGLO XXI


HIPOGLUCEMIA
CIENCIAS BASICAS: Hipoglicemia fisiolgica: en periodo posnatal inmediato 1-2hrs, produccin neonatal de glucosa 6-9mg/hg/min. Hipoglucemia
neonatal: <45 mg/dl (2.5 mmol/L) para prematuros como de trmino y a cualquier edad extrauterina. Hipoglucemia neonatal transitoria: se auto
limita durante los primeros 7 das de vida extrauterina, es consecuencia de reserva energtica limitada, excesivo consumo perifrico, agotamiento
precoz de reservas energticas y a inmadurez del sistema hipotlamo-hipofisiario, responsable de la secrecin de hormonas contra regulacin.
Hipoglucemia neonatal persistente o recurrente: <45 mg/dl en 3 ocasiones o persiste por ms de 72 hrs a pesar de tratamiento con soluciones
glucosadas a flujo de 12-16mg/kg/min, para alcanzar normoglicemia y perisiste >7dias. Hipoglucemia severa: aporte glucosada >10 ml/kg/min
persisten cifras menores de 45 mg/dl. SALUD PUBLICA: En RN a trmino la incidencia est en un rango de 5-7% y puede variar entre valores de 3,215 % en recin nacidos pretrmino. Hijos de madre diabtica 40%. Es uno de los problemas metablicos mas frecuentes en UCIN. Casi el 40% de los
neonatos hijos de madre diabtica tienen hipoglucemia. Factores de riesgo: Madre diabtica durante el embarazo o ingestin de betabloqueadores
o hipoglucemiantes orales. Prematurez. Peso bajo para la edad gestacional (<2,500kgrs). Peso grande para la edad gestacional, defectos de la lnea
media. Deteccin tarda y manejo inadecuado de la hipoglucemia neonatal tiene un impacto sobre el desarrollo. Causas de hipoglucemia transitoria:
Estrs perinatal, septicemia, asfixia, hipotermia, policitemia, choque. PATOGENIA: La etiologa ms frecuente esta asociada a incremento de la
utilizacin de glucosa (hijo de madre diabtica, policitemia, drogas maternas, nesidiblastosis, Sx. Beckwith), a un aporte inadecuado de glucosa
endgeno o exgeno (prematurez, RCIU, ayuno prolongado, estrs asfixia, shock, exanguineotransfusion, deficiencia hipotalmica) o a una
combinacin de ambos. Una de las causas ms frecuetes en el RN ligada a hiperinsulinismo fetal es el hijo de madre diabtica mal controlada. En
estos nios la hipoglucemia frecuentemente se produce a las 4-6 hrs despus del nacimiento, tienen una reduccin incrementada de insulina
secundaria a sensibilidad aumentada de clulas beta del pncreas a la glucosa; que persiste durante varios das despus del parto. Sntomas clsicos
se deben a activacin del sistema nervioso autonomo con la liberacin de catecolaminas (hormona contrarreguladora de la hipoglucemia).
Deprivacin de glucosa al cerebro con alteracin funcin neurolgica, hay entrada de calcio en la celula y muerte neuronal, las reas mas afectadas
es la corteza cerebral superficial. La administracin antenatal de clopropamida, benxotiazidas, beta-simpaticomimticos, propanolol o la
administracin de glucosa a la madre a alta concentracin y supresin brusca de su administracin puede inducir insulinismo fetal transitorio y por lo
tanto hipoglucemia neonatal. La eritroblastocisis fetal en RN con incompatibilidad Rh y el sndrome de Beckwith-Wiedemann, se caracteriza por RN
con macrosomia, onfalocele, macroglosia, visceromegalia e hipoglucemia. Se ha explicado la hipoglucemia por hipertrofia de las clulas beta del
pncreas y por lo tanto hiperinsulinismo. Otras causas: por deficiencias hormonales: dficit de hormonas del crecimiento, deficiencia tiroidea.
Causas hereditarias: enfermedad de orina de jarabe de arce, intolerancia a la fructosa, galactosemia. DIAGNOSTICO: Apnea, hipotona, reflejo de
succin inadecuada, irritabilidad, llanto agudo, somnolencia, respiraciones irregulares, cianosis, temblores, palidez, crisis convulsivas, letargia,
cambios en el nivel de conciencia, inestabilidad de la temperatura, coma. Triada de Whipple: Caractersticas clnicas, glucosa srica baja, resolucin
de sntomas con correccin de glucemia. Estndar de oro: Determinacin enzimtica de los niveles de glucosa en laboratorio por el mtodo de la
hexocinasa. Tamizaje; RN con factores de riesgo asitomatico, medir 2-3 hrs posnatales, hijo de madre diabtica desde la primera hora, de
nacimiento, RN sintomtico inmediatamente. TRATAMIENTO: Debe iniciarse una alimentacin precoz en las dos primeras horas de vida (de
preferencia en los primeros 30-60 minutos) y establecerse intervalos de alimentacin cada 2-3 horas. Paciente: Con glucosa entre 30-44mg/dl
asintomtico, iniciar via oral, checar glucosa 30-60 min, despus de la alimentacin. Paciente asintomtico o glucosa <30mg/dl o sintomtico, iniciar
bolo de dextrosa al 10% 2ml/kg, iniciar infusin de glucosa 4-6mg/kg/min, iniciar via oral lo antes posible (10-15ml/kg). En caso de continuar con
hipoglucemia realizar incrementos en la infusin de glucosa de 2 en 2 hasta 14mg/kg/min. Iniciar hidrocortisona 5mg/kg/d cuando el aporte de
glucosa sea >15mg/kg/min, glucagn 0.1mg/kg IM, diazoxido 15mg/kg/d, octreotide 5-10mcg/kg c/6-8hrs, pancreatectomia subtotal. Cada vez que
se detecte hipoglucemia asintomtica se deber administrar bolo de solucin glucosada al 10%. Si a pesar de tratamiento apropiadamente
instaurado, el paciente persiste con hipoglucemia durante 3 determinaciones consecutivas, se recomienda referencia a tercer nivel en cualquiera de
las siguientes situaciones: persistencia de hipoglucemia a pesar de recibir aporte de glucosa kilo minuto de 10 mg/kg/min. Cuando exista
hipoglucemia asociada a: Antecedente familiar de muerte neonatal sbita o sndrome de Reye, crisis convulsivas o alteracin del estado de
consciencia asociados a hipoglucemia, alteraciones de la termorregulacin, defectos de la lnea media, exoftalmos, micropene. En hipoglucemia
refractaria o persistente: Glucagn (0.1mg/kg IM, mximo 1 mg) o hidrocortisona. PREVENCION: Inicio temprano de la alimentacin 30-60min
posnatal, alimentar a libre demanda con lactancia materna (10-12 veces), completar con formula PRN, preservacin de temperatura corporal, si hay
contraindicacin para via oral, inicra soluciones parenterales dentro de la primera hora de vida. COMPLICACIONES: Alto riesgo para complicaciones
neurolgicas, reduccin del scopre motor, mental y en la escala de desarrollo de Bayle, incidencia de paralisis cerebral o alteracin en el
neurodesarrollo incrementado hasta 3.5 veces
CASO CLINICO HIPOGLUCEMIA
RN de 36 SDG obtenido por cesrea por desprendimiento de
placenta. Al nacimiento presenta Apgar 8/9, peso en -0,89 DE,
longitud en +1,75 DE, sin datos del permetro ceflico. Con 48 horas
de vida ingresada por ictericia en su hospital de referencia presenta
hipoglucemias no cetsicas (glucemias entre 25-45mg/dl),
insulinemia mxima registrada: 7U/ml coincidiendo con glucemia
de 33mg/dl.
PREGUNTA
Cual es la conducta a seguir para identificar la causa de la
hipoglucemia?
RESPUESTA
a.- Verificar Diabetes Mellitus en la madre
b.- Realizar escaneo abdominal.
c.- Verificar funcionamiento heptico.
d.- Verificar funcionamiento suprarrenal.
CASO CLINICO HIPOGLUCEMIA

CURSO ENARM CMN SIGLO XXI TEL: 36246001

Se trata de reciencia nacido de 24 horas de edad, presenta en


cunero las siguientes alteraciones: reflejo de succion inadecuado,
crisis convulsivas letargia, inestabilidad de la temperatura.
PREGUNTA
Cual es la conducta mediata mas adecuada para este caso?
RESPUESTA
a.- Administracion rectal de diacepam.
b.- Administracin oral de glucosa.
c.- Intubacion orotraqueal.
d.- Glucosa perifrica.
CASO CLINICO HIPOGLUCEMIA PERSISTENTE
Se trata de paciente masculino de 37 semanas de gestacion obtenido
por cesarea, la madre presento cuadro de pre-eclampsia, luego de
24 horas de nacido presenta disminucin de la succion, letargia, falta
de respuesta a estimulos con disminucin de tono muscular, asi
como alteraciones de la temperatura con tendencia a la hipotermia,
los laboratorios reportaron 35 mg/dl de glucosa perifrica, resto de
datos de laboratorio y gabinete dentro de parmetro normales, sin

Pharmed Solutions Institute

PGINA 307

MANUAL DE TRABAJO DEL CURSO ENARM CMN SIGLO XXI


embargo posterior a la administracin de glucosa mediante catter
perifrico presenta nuevos cuadro de hipoglucemia.

minutos. A las 36 horas de vida se documenta hipoglicemia de 30


mg/dl

PREGUNTA
Cual de los siguientes antecedentes es menos frecuente para la
patologia que presenta el paciente?
RESPUESTA
a.- Displasia de clulas beta.
b.- Enfermedad de orina de arce.
c.- Galactosemia.
d.- Hipotiroidismo congnito

PREGUNTA
Cul es la conducta teraputica mas adecuada a seguir?
RESPUESTA
a.- Infusin endovenosa de glucosa a 4,8 mg/kg/min.
b.- Insulina NPH
c.- Administracion oral de glucosa
d.- Administracion en carga con glucosa al 50%

CASO CLINICO
Femenino de 37 semanas de edad gestacional, peso al nacer de
4,300 gms. APGAR de 7/9. Naci por cesrea de madre trigesta de 23
aos de edad, en aparente buen estado de salud con antecedente de
un parto eutcico y de un aborto por causa desconocida. Se
desconocen cuadros maternos de inmunizaciones previas al
embarazo. Control prenatal irregular. Desde las primeras horas de su
nacimiento present temblores generalizados, anorexia y crisis
convulsivas, succion dbil, hipotonia. La glucosa srica de control
seriado fueron de menos 40 mg/dl y persistieron por ms de siete
das con concentraciones elevadas de insulina y requerimientos de
glucosa que excedan 15 mg/kg/min.
PREGUNTA
Cul es el diagnstico ms probable en este paciente?
REPUESTA
a.- Nesidioblastosis
b.- Policitemia
c.- Sndrome de Wiedemann-Beckwith (macrosomico, macroglosia,
onfalocele, visceromegalia, hipoglucemia)
d.- Hipertiroidismo
PREGUNTA
Despus de ir incrementando aporte de glucosa de 2 en 2mg/kg/min
y llegar hasta 15mg/kg/min, se agreg Hidrocortisona a 5mg/kg/d,
as como glucagn y octreotide Cul es la conducta teraputica ms
adecuada a seguir en este momento?
RESPUESTA
a.- Insulina NPH
b.- Pancreatectoma subtotal
c.- Funduplicatura
d.- Insulina lispro
PREGUNTA
Con que antecedentes deberamos enviar a esta paciente a tercer
nivel?
REPUESTA
a.- Exoftalmos, hijo de madre diabtica, uso de drogas por la madre
b.- Defectos en la lnea media, hijo de madre diabtica, asfixia
perinatal
c.- Uso de drogas por la madre, exoftalmos, defectos en la lnea
media
d.- Alteracin en la termorregulacin, defectos en la lnea media,
exoftalmos

CASO CLINICO
Se trata de un recin nacido de pretrmino de 35 semanas, pequeo
para la edad gestacional, de sexo masculino, hijo de padres sanos,
no consanguneos, cuyo embarazo curs con oligoamnios y retardo
del crecimiento intrauterino. La madre recibe terapia corticoidal
prenatal, parto es por cesrea por sufrimiento fetal agudo, con peso
al nacer de 1 768 gramos, talla de 40 centmetros, apgar 9 a los 5

CURSO ENARM CMN SIGLO XXI TEL: 36246001

PREGUNTA
Persisten cifras de glicemias bajas, y a los 13 das de vida se obtiene
una muestra crtica (glicemia de 26 mg/dl con una carga de glucosa
de 15,2 mg/kg/min) con los siguientes resultados: insulinemia 14,2
mU/ml, hormona de crecimiento 16 ng/ml, cortisol 5,8 ug/dl, TSH
7,8 UI/ml, T4 7,9 ug/dl, amonemia 0,82 mg/ml, lactacidemia 30
mg/%, perfil bioqumico normal, cuerpos cetnicos en orina
negativos, acylcarnitinas normales. Se realiz test de glucagn con
una glicemia basal de 37 mg/dl y de 52 mg/dl luego de 45 minutos
del glucagn, demora debida a dificultades tcnicas en la toma de
muestra. Cul seria la conducnta mas adecuada a seguir en este
momento?
RESPUESTA
a.- Continuar con glucosa intravenosa a 15,2mg/Kg/min
b.- Iniciar glucocorticoides
c.- Insulina NPH
d.- Glucosa al 50%
PREGUNTA
Cul es el diagnotico ms probable en este caso?
RESPUESTA
a.- Hipertiroidismo
b.- Nesidioblastosis
c.- Hiperinsulinismo neonatal peristente
d.- Hipoglicemia transitoria
CASO CLINICO
RN femenina a trmino, pequea para la edad gestacional, de 2 das
de vida, quien presenta movimientos tnico-clnicos generalizados,
succin dbil, e hipotona, refractarios a tratamiento. Madre no
diabtica. Al examen fsico: Peso: 2.100 gr, talla: 48 cms. Piel con
leve tinte ictrico. Hipoactiva, con llanto agudo. Laboratorio:
Glicemia central 7 mg/dL y capilar: 13 mg/dL, Insulina 30,8 mU/mL,
Cortisol 5,68 g/dL, Hormona de Crecimiento 25,8 ng/mL. Perfil
tiroideo, gasometra y hemograma normal, bilirrubina elevada.
Recibe aporte de dextrosa a razn de 8 mg/kg/min ms un bolus de
dexametasona (0,6 mg/stat). A las 12 horas de su ingreso y luego de
iniciar la primera dosis de hidrocortisona (5 mg/kg/da) present:
Glicemia basal 13 mg/dL, Insulina basal 16,8 mU/mL, Triglicridos:
160 mg/dL, Colesterol 87 mg/dL, C-HDL 39 mg/dL. Estuvo
hospitalizada durante 2 semanas con aporte continuo de dextrosa a
razn de 9 mg/kg/min e hidrocortisona; evoluciona
satisfactoriamente, con disminucin progresiva de la necesidad de
aporte de glucosa y de esteroides. Se egresa con glicemia de 50
mg/dL e insulina de 3 U/mL.
PREGUNTA
Cul es el diagnostico mas probable para este caso?
RESPUESTA
a.- Hiperinsulinismo congnito
b.- Deficit de hormonas contrareguladoras
c.- Errores innatos del metabolismo
d.- Hipoglucemias transitoria

Pharmed Solutions Institute

PGINA 308

MANUAL DE TRABAJO DEL CURSO ENARM CMN SIGLO XXI


HIPERBILIRRUBINEMIA Y ENCEFALOPATIA HIPERBILIRRUBINEMICA
CIENCIAS BASICAS: La ictericia es una de las condiciones ms comunes que requieren atencin mdica en los RN y se refiere a la coloracin
amarillenta de piel, mucosas y fluidos corporales (LCR, lagrimas, saliva), causada por la fijacin de bilirrubina en el tejido graso subcutneo;
generalmente, se observa cuando los niveles sricos de bilirrubina son mayores o iguales a 5-7 mg/dl. La
hiperbilirrubinemia se refiere al aumento de los niveles de bilirrubina total en sangre ( 2-5 mg/dl). En la
mayora de los casos, suele ser benigna, y autolimitada, pero por el efecto neurotxico de la bilirrubina,
los neonatos de riesgo deben ser vigilados para evitar hiperbilirrubinemia severa que produzca
alteraciones neurolgicas como encefalopata aguda y kernicterus, los cuales son causas prevenibles de
parlisis cerebral. Los neonatos amamantados son ms propensos a desarrollar, ictrica fisiolgica
(aparece despus de las 24 hrs y desaparece antes de los 10 das) en la primera semana de vida y cerca
del 10% son ictricos al mes de vida. Ictericia patolgica; cuando inicia en las primeras 24 hrs, se
acompae de otros sntomas, duracin mas de 10-15 dias la bilirrubina aumente >5mg/dl diarios,
sobrepase los 12mg% o 14mg% en neonatos a trmino y pretermino, la fraccin directa sea >1mg/dl o
dure ms de una semana en el RN a trmino. SALUD PUBLICA: La ictericia fisiolgica es una situacin
frecuente, aparece despus del segundo da de vida como expresin de una condicin fisiolgica hasta en
un 80% de los prematuros y en un 60% de los a trmino. Generalmente resuelven 7-10 dias. Ictericia
patolgica en 6% de RN. Factores de riesgo: mayor prdida de peso (ms de 5%), sexo masculino., edad
gestacional <35 semanas, cefalohematomas, equimosis, deshidratacin, trauma obsttrico, TORCH, hepatitis neonatal, drogas (estreptomicina,
cloranfenicol, sulfas, gentico (Sx. Gilbert, Crigler Najjar tipo I, II, defectos estructurales del eritrocito, policitemia). Factores maternos:
incopatibilidad ABO y/o Rh, alimentacin al seno materno por inhibicin competitiva de la glucoronidiltransferasa heptica, drogas (diacepam,
oxitocina), diabetes gestacional. PATOGENIA: El 70-80% de la bilirrubina se forma a apartir de degradacin del grupo hem en el sistema
reticuloendotelial, 20-30% se origina de la miglobina, citocromo y
oxido ntrico. La principal causa de la apricion de la ictrica fisologica
en el RN es la inmadurez del sistema enzimtico del hgado, a esto se
le suma una menor vida media del glbulo rojo, la poliglobulia (el RN
produce el doble de bilirrubina que el adulto), la extravasacin
sangunea frecuente, el RN reabsorbe gran parte de la bilirrubina a
travs de la circulacin enterohepatica y la ictrica por lactancia. Se
debe (etiologa) a aumento en la secrecin: enfermedad hemoltica,
incompatibilidad materno-fetal, anemias hemolticas hereditarias
(defecto de membrana, hemoglobinopatas), anemias hemolticas no
hereditarias (hemolisis inducida por vitamina K, talasemias). Por
disminucin en la secrecin: disminucin de la capatacion heptica
(inadecuada perfusin de sinusoides hepticos, deficiencia de
protena Y y Z), disminucin de la conjugacin heptica (deficiencia o
inhibicin enzimtica, sx. de Lucey-Driscoll), inadecuado transporte
fuera del hepatocito, obstruccin biliar, extravasacon de sangre
(cefalohematoma, hemorrgias, deglutida), aumento en la circulacin
enterohepatica (estenosis pilrica, ayno prolongado, Hirschprug),
induccin de TDP con oxitocina. Por combinacin de reduccin en su
eliminacin y aumento en su produccin: sepsis, TORCH, SDR.
ICTERICIA POR INCOMPATIBILIDAD DE FACTOR Rh: Es la causa ms
frecuente de ictrica patolgica y el 97% de los casos se debe a isosensibilizacion para el antgeno Rh D. Un alto ttulo materno de anticuerpos antiDi 64 de se asocia a un riesgo elevado de hiperbilirrubinemia severa para los recin nacidos. La
administracin de profilctica de inmunoglobulina Anti-D. DIAGNOSTICO: Ictericia en tegumentos
aparece en sentido cefalo-caudal (criterios de Kramer). Es conveniente valorar la presencia de coluria y
acolia, ya que son datos de gran valor diagnstico. La presencia de hepatomegalia precoz es sugestiva
de infeccin prenatal o de enfermedad hemoltica por incompatibilidad Rh; cuando esta es dura y de
aparicin ms tarda, har pensar en la posibilidad de afectacin heptica primitiva (hepatitis, atresia), si
bien en estos casos el resto de signos clnicos son distintos. La esplenomegalia har sospechar que acta
como un foco hematopoytico extramedular o bien que el RN padece una infeccin prenatal con
manifestaciones clnicas. Por la frecuencia con la que se presenta al tercer da de vida una
hiperbilirribinemia secundaria a la reabsorcin de hematomas, se debern buscar colecciones de sangre
extravasada. Otro sntoma frecuentemente asociado a hemolisis es la hipoglicemia, como resultado de
la hiperpalsia pancretica. La presencia de petequias y purpura sugieren la posibilidad de infeccin
connatal, edema generalizado (hidrops fetal). Laboratorio: Hiperbilirrubinemia severa 20-24mg/dl y
critica de 25-30mg/dl. Dosaje de bilirruibinemia total y directa, si hay elevacin de bilirrubina indirecta,
sugiere hemlisis. Si hay elevacin de la bilirrubina directa, sugiere enfermedad hepatobiliar. Reaccin
de Coombs directa e indirecta. Hematocrito y hemoglobina (para valorar la presencia de anemia
asociada). Recuento de reticulociotos. El diagnostico se establece cuando se encuenra: >4mg/dl de BI en
sangre de cordon umbilical, >6mg/dl en las primeras horas de vida, >10mg/dl de BI en las primeras 24
hrs de vida, >13mg/dl de BI en las primeras 48 hrs, >15mg/dl de BI en cualquier momento.
TRATAMIENTO: Las alternativas son: 1. FOTOTERAPIA (mecanismos de accin fotoisomerizacion
geomtrica, ciclisacion intramolecular, oxidacin); la de eleccin y ms difundida, destruccin fsica de la
bilirrubina en productos ms pequeos y polares para ser excretados, el RN debe estar desnudo, cubrir
los ojos, y control trmico, actualmente existe fototerapia en fibra ptica, indicada cuando la BI
aumenta como >5mg/dl en <24hrs, >7-8 a las 24 hrs, >12-13 a las 48hrs, >17-18mg/dl a las 72 hrs. 2.
Exanguinotransfusion; recambio sanguneo, til para disminuir cifras de bilirrubina, corregir anemia y retiro de anticuerpos que generen hemolisis.

CURSO ENARM CMN SIGLO XXI TEL: 36246001

Pharmed Solutions Institute

PGINA 309

MANUAL DE TRABAJO DEL CURSO ENARM CMN SIGLO XXI


Indicada en encefalopata aguda hiperbilirrubinemica, aumento de la tasa de bilirrubina srica >0.5mg/100ml/h cn fototerapia intensiva, RN con
hemolisis, hidrops, fracaso de fototerapia, con sangre en cordon umbilical con Hb<10-11g/dl, bilirrubina de cordon >4mg/dl, reticulocitos >5%,
coombs directo 3-4 cruces. PROFILAXIS: Fototerapia en RN con peso menos a 1000g, independiente de edad. Administraciona la madre de
fenobarbital, antes del parto, para inducir la actividad de la glucoronil transferasa, utilizacin de inmunoglobulina en periodo prenatal y neonatal con
enfermedad hemoltica severa. Ictericia familiar no hemoltica con kernicterus: Ictericia despus del nacimiento, con signos de insuficiencia heptica,
por elevacin de BI, por lo que con frecuencia se presenta kernicterus, el estudio enzimtico demostrara dficit de glucoroniltransferasa, el tipo I
(Crigler-Najjar) presenta cifras de bilirrubina >30mg/dl, responde mal a mtodos de induccin enzimtica, y suele condicionar ictericia nuclear, es
autosmica recesiva. El genotipo II (Enfermedad de Arias) responde a fenobarbital y la herencia es autosmica dominante. Enfermedad de Gilbert; la
deficiencia enzimtica es de menor intensidad, hay insuficiencia funcional de protenas Y y Z. Lucey-Driscoll; aumento de inhibidores de la
gluconoconjugacion presentes en los sueros materno y fetal. TOXICIDAD: Los signos clnicos de neurotoxicidad aguda son apata, somnolencia o
insomnio, junto con la alteracin de los potenciales evocados auditivos, pero que luego revierten, una vez que los valores de bilirrubina descienden.
ENCEFALOPATIA HIPERBILIRRUBINEMICA. CIENCIAS BASICAS: Es la coloracin amarilla de los ganglios basales y nucleos del tallo cerebral,
producida por impregnacin de bilirrubina indirecta no unida a albumina, descrita en autopsias de RN fallecidos con severa ictericia. El kernicterus es
la secuela ms importante de la encefalopata bilirrubnica. SALUD PUBLICA: Su incidencia aumento con las nuevas polticas de alta prematura, esto
causa mayor riesgo de complicaciones debidas a ictericia temprana no detectada. Se presneta en 8% de pacientes con BI de 19-24mg/dl, en 33% con
cifras de25-29mg/dl, en 73% con cifras de 30-40mg/dl. PATOGENIA: Hay varias situaciones que alteran la barrera hematoencefalica y facilitan la
entrada de bilirrubina al SNC, como son; bajo peso al nacer, hipoglucemia, asfixia neonatal, acidosis metablica, hemolisis, hipotermia-frio,
hipoalbuminemia (hay desplazamiento por aniones organicos, endgenos y exgenos y aumenta de bilirrubina acida, por unin de bilirrubina-anion,
es irreversible y puede genera un ambiente hiperosmolar, que lleva a muerte celular), drogas que compiten por la unin a albumina, diestres
respiratorio. La bilirrubina no conjugada penetra en el cerebro y acta como una neurotoxina, a menudo sta se asocia con condiciones que
dificultan la funcin de la barrera hemato-enceflica (ejemplo, sepsis), inhibiendo varios procesos bioqumicos muy importantes, como la
fosforilacin oxidativa de las mitocondrias y la sntesis proteica. Mecanismo de neurotoxicidad por bilirrubinas; dao en mebrana celualr y
organelos, alteracin en uso de glucosa, fosforilacion oxidativa, ATP, sntesis de DNA, protenas , neurotrnsmisores, transporte de iones, funcin
sinptica. DIAGNOSTICO: Puede ser asintomtico en prematuros pequeos. Se caracteriza por atetosis, sordera neorosensorial parcial o completa,
limitacin de la mirada vertical, dficit intelectual, displasia dental. En el RN termino sntomas en 2-5to dia, en RNPT hasta el 7mo dia. Clinica; pobre
succion, tono muscular fluctuante, y posterior hipertona, llanto agudo, posicin de opistotonos, ojos en sol naciente, convulsiones tnicas o clnicas
generalizadas y en algunos casos coma o muerte. ENCEFALOPATIA AGUDA POR BILIRRUBINA: Tincion con bilirrubina 7-10 dias, globo palido, nucleo
subtalamico, hipotlamo, hipocampo, sustancia negra, nervios craneales III, VII, VIII, formacin reticular, cerebelo, clulas de astas anteriores de
medula. Hay lesion neuronal perdida neuronal, mineralizacin, astrocitosis. Kernicterus; 15% sin signos definidos, alteraciones de conciencia (f.
reticular), de tono y movimiento (globo palido y n. subtalamico), alimentacin y llanto (tallo). Clinica dos primeros das fase I: succion, estupor e
hipotona leves, hipomotilidad, covulsiones. Fase II: mitad de primera semana estupor leve, irritable, tono variable (retrocolis-opistotonos al
estimulo), alimentacin minima, llanto alto, fiebre 80%. Fase III avanzada: pasada la primer semna, estupor profundo a coma, retrocolis-opistotonos,
alimentacin nula, llanto estridente, crisis oculogiras, respiraciones irregulares, fiebre, la mayora de la lesion es irrversible y evoluciona
ancefalopatia crnica y muerte. COMPLICACIONES: Atetosis en miembros superiores, alteraciones de la deglucin, fonacin y movimiento facial,
corea, balismo, distonia, mirada hacia arriba, paralisis de mirada. ENCEFALOPATIA CRONICA: Anormalidades auditivas las ms frecuentes, perdida de
percepcin de tonos altos, retardo en lenguaje, dficit intelectual. El diagnostico macrosocpico es la coloracin amarilla, el microscpico vemos,
picnosis, vacuolizacion del citoplasma, destruccin de mitocondrias y ap. De Golgi, degeneracin atrfica, desmielinizacion. PRONOSTICO:
Mortalidad en el periodo neonatal a 10 primeros meses 10%, secuelas neurolgicas irrversibles 70% (paralisis cerebral coreoatetosica, oftalmoplejia,
hipoacusia neurosensorial, displasia de esmalte dental, dficit cognitivo).
CASO CLINICO
Neonato masculino de 4 das de vida, madre de 32 aos de edad, G1,
A 0. Nacimiento a las 38 semanas parto normal. Apgar de 8-9, peso
de 3400 gramos, talla de 50 cm y permetro ceflico de 35cm.
Alimentado al seno materno hasta la actualidad. Es llevado al
servicio de urgencias por presentar tinte ictrico que la madre le
not hace 24 horas. Se le realizan estudios encontrando: Hb 13, Hto
42, grupo sanguneo de la madre O positivo, grupo sanguneo del
neonato O positivo, bilirrubina total 6mg/dl a expensas de la
indirecta. Paciente activo, reactivo, FC 120x, FR 48x.
PREGUNTA
Cul es el diagnstico ms probable de este paciente?
RESPUESTA
a.- Ictericia por endocrinopata
b.- Ictericia por anemia hemoltica
c.- Ictericia fisiolgica
d.- Ictericia por lactancia materna
PREGUNTA
De acuerdo a la cifra de bilirrubina, en que zona de Kramer, se
encuentra este paciente?
RESPUESTA
a.- Zona 1
b.- Zona 2
c.- Zona 3
d.- Zona 4

CURSO ENARM CMN SIGLO XXI TEL: 36246001

PREGUNTA
Si este paciente presentara una hiperbilirrubinemia severa. Qu
alteraciones neurolgicas podra presentar?
RESPUESTA
a.- Paralisis cerebral, Kernicterus
b.- Encefalopatia, meningitis
c.- Meningitis, kernicterus
d.- Encefalopata aguda, kernicterus
CASO CLINICO ICTERICIA
RN presenta ictericia, que se hizo evidente en el segundo da de la
vida. Naci de padres no relacionados a los 36 semanas de gestacin
con un peso al nacer 2800 g, despus de embarazo y el parto
expontaneo y normal aparentemente. Al examen fsico, paciente
estaba activo y cmodo. Sus signos vitales eran estables. En
recuento de admisin de sangre completa fue en normal lmites.
PREGUNTA
Cul es la conducta mas adecuada a seguir?
RESPUESTA
a.- Alta y medidas en casa
b.- Se ingresa para fototerapia
c.- Verifica niveles de bilisrrubinas
d.- Verifica grupo y factor en ambos padre
CASO CLINICO
RN de 35 SDG, vigoroso, peso 2,400 g, permetro ceflico 34 cm, talla
47 cm, sin patologa perinatal, alimentado a pecho direcho exclusivo,

Pharmed Solutions Institute

PGINA 310

MANUAL DE TRABAJO DEL CURSO ENARM CMN SIGLO XXI


alta a las 48 horas, regreingresa a los 6 dias de vida por ictericia
generalizada, con peso al ingreso de 2,130 grs. Succion vigorosa pero
breve e ineficaz, escasas diuresis y deposiciones. Bilisrrubinas totales
32,52 mg/dl, bilisrrubina indirecta 32 mg/dl, Bh normal, glicemia
normal. La EF se observa leve temblor distal
PREGUNTA
Cual es la conducta mas adecuada a seguir en este caso?
RESPUESTA
a.- Colchon de fibra ptica de luz contina
b.- Exanguinotransfusion
c.- Fototerapia
d.- Luz halogenada
CASO CLINICO
Madre de 25 aos, sana, ORh +, VDRL -. Primigesta, nulpara,
embarazo bien controlado, sin complicaciones. Rotura espontnea
de membranas de cinco horas con lquido amnitico claro a las 35
semanas de gestacin. Se realiza induccin con oxitocina. Parto
vaginal en ceflica, recin nacido de sexo femenino, 35 semanas de
edad gestacional, vigoroso, peso 2.540 g, permetro ceflico 34 cm,
talla 47 cm, ORh +, Coombs -, VDRL -. Sin patologa perinatal,
alimentada a pecho directo exclusivo, alta a las 48 h, peso al alta
2.400 g. Reingresa a los 6 das de vida por ictericia generalizada.
Peso al ingreso 2.120 g. BT 32,52 mg/dl, BI 32 mg/dl, leucocitos
12.200mm3, 50% PMN, 330.000 plaquetas, Hto63%, Hb 21,1g/l.
Reticulocitosis 0,8%. Paciente con irritabilidad, hipertona y
opistotonos. Temp 38 C.

materna durante los primeros das y ahora est con frmula de


primer semestre. No hubo controles prenatales, pero la madre no
relata problemas durante el embarazo, ni abuso de drogas o alcohol.
Es el primer embarazo de una madre de 22 aos, que relata tener
una buena salud y que como nico antecedente relata haber sido
tratada para Chlamydia hace un ao. La situacin econmica es
difcil. El parto fue vaginal a las 38 semanas y el beb fue vacunado
al nacer contra Hepatitis B; ambos fueron dados de alta del hospital
al da siguiente. Al llegar al servicio de urgencias los signos vitales
eran estables, se not ictericia leve en conjuntivas y hepatomegalia
tambin leve. Con descamacin en la piel. En Rx. De miembros
inferiores so observa radiolucides en tibias.
PREGUNTA
Cules el diagnotico mas probable para este caso?
RESPUESTA
a.- Sifilis congnita
b.- Infeccion por citomegalovirus
c.- Infeccion por herpes simple
d.- Toxoplasmosis congnita
PREGUNTA
Cul dato presente en el paciente apoya mas el diagnostico?
RESPUESTA
a.- Descamacion en piel
b.- Antecedente de clamydia en la madre
c.- Radiolucides en la radiografia de piernas
d.- Vacunacion para hepatitis B

PREGUNTA
Qu secuelas son las ms probables que se presenten en este
paciente?
RESPUESTA
a.- Sordera, trastornos motores y problemas de la conducta
b.- Trastornos motores, hipotona, encefalopata
c.- Problemas en la conducta, hipertona, sordera
d.- Hipotona, encefalopata, problemas en la conducta

PREGUNTA
Cul seria la conducta teraputica mas adecuada a seguir en este
caso?
RESPUESTA
a.- Penicilina
b.- Rivabirina
c.- Aciclovir
d.- Amikacina

PREGUNTA
Cul es la conducta teraputica ms adecuada en este paciente?
RESPUESTA
a.- Fototerapia intensiva
b.- Exanguineotransfusin
c.- Fenobarbital
d.- Mesoporfirina

CASO CLINICO
Recien nacido de 35 SDG, vigoroso, peso 2,400 g, permetro ceflico
34 cm, talla 47 cm, sin patologa perinatal, alimentado lactancia
materna exclusiva, alta a las 48 horas, reingresa a los 6 das de vida
por ictericia generalizada, con peso al ingreso de 2,130 grs. Succin
vigorosa pero breve e ineficaz, escasas diuresis y deposiciones.
Bilirrubinas totales 32,52 mg/dl, bilirrubina indirecta 32 mg/dl, Bh
normal, glicemia normal. La EF se observa leve temblor distal.

PREGUNTA
Cul es la causa ms probable del Kernicterus?
RESPUESTA
a.- Neurotoxicidad de la bilirrubina directa
b.- Neurotoxicidad de la bilirrubina indirecta
c.- Neurotoxicidad de la bilirrubina total
d.- Disfuncion de la barrera hematoencefalica

PREGUNTA
Cual es la conducta mas adecuada a seguir en este caso?
RESPUESTA
a.- Colchn de fibra ptica de luz contina.
b.- Exanguinotransfusion.
c.- Fototerapia.
d.- Luz halogenada

CASO CLINICO
Nio de 8 das de vida que ingresa por ictericia, elevacin de
transaminasas y hepatomegalia. Los padres no relatan vmito,
diarrea, fiebre o dificultad respiratoria; si relatan un brote
maculopapular que ahora est en fase de descamacin. El paciente
ha estado comiendo normalmente y estuvo alimentado con leche

CURSO ENARM CMN SIGLO XXI TEL: 36246001

PREGUNTA
Cul es la causa ms probable en este caso?
a.- Bilirrubina no conjugada libre no unida a albmina
b.- La unin bilirrubina a la albumina
c.- bilirrubina conjugada unida a albumina
d.- bilirrubina no conjugada

Pharmed Solutions Institute

PGINA 311

MANUAL DE TRABAJO DEL CURSO ENARM CMN SIGLO XXI


ATRESIA ESOFAGICA (AE) Y FISTULA TRAQUEOESOFAGICA (FTE)
CIENCIAS BASICAS: Anomala congnita (entre3.5ta semana se forma esfago) en la cual la porcin media del esfago est ausente (atresia); en un
porcentaje importante se acompaa de una comunicacin anormal entre la traquea y el segmento distal del esfago llamada fistula
traqueoesofgica. La atresia de esfago es una malformacin incompleta de la luz esofgica. Se considera una verdadera emergencia quirrgica. La
variante ms frecuente es la tipo C de la clasificacin de Gross o la III de Voght en 87% de los casos. SALUD PUBLICA: La atresia esofgica con o sin
fstula traqueoesofgica es una alteracin que aparece en 1:3,000 a 1:4,500 recin nacidos. El 95% de la AE tiene fstula traqueoesofgica asociada.
El 50% de los casos se asocia con otras malformaciones congnitas y existe riesgo de recurrencia de 2 a 3%. En virtud que en Mxico nacen
aproximadamente 2 millones de nios por ao, se estima que cada ao hay entre 500 y 600 casos nuevos de nios con atresia de esfago. Factores
de riesgo y agravantes del pronstico: malformaciones congnitas, neumonas, bajo peso al nacer. Se encuentran anomalas cromosmicas entre 6
10% de los casos; las ms frecuentes son las trisomas 18 y 21, prematurez (34%), sexo masculinopolihidramnios materno (85%), defectos cardiacos,
genitourinarios o gastrointestinales. CLASIFICACION: Clasificacin de Voght (ver cuadro): AE TIPO I (5-8%): Ambos cabos esofgicos ciegos sin fstula
traqueoesofgica: Es de fcil diagnostico en el embarazo por que cursa con polihidramios y ausencia de imagen gstrica. Abdomen excavado por
falta de pasaje de aire al intestino. Ambos cabos se encuentran muy separados entre s. AE TIPO II (0.5-1%): Fstula traqueoesofgica superior y cabo
inferior ciego. AE TIPO III (80-85%): Fstula traqueoesofgica inferior y cabo esofgico superior ciego: En stos pacientes el reflujo gastroesofgico
puede ser lesivo para los pulmones, son propensos a sufrir neumonas qumicas. AE TIPO IV (3%): Fstula traqueoesofgica en ambos cabos del
esfago. AE TIPO V (3%): Fstula en H o N. Es una fstula traqueoesofgica sin atresia de esfago: El diagnostico se realiza en la infancia ya que
aparecen los sntomas en ese periodo. AE TIPO VI (0.-1%): Estenosis esofgica aislada. Clasificacin de Gross: A. Atresia esofgica sin fstula, 3-5%. B.
Atresia esofgica con fstula proximal, 2%. C. Atresia esofgica con fstula distal, 80-90%. D. Atresia esofgica con fstula proximal y distal, 3-5%. E.
Fstula traqueoesofgica sin atresia, en H 6%. F. Estenosis esofgica. PATOGENIA: Malformaciones congnitas asociadas: Cardiacas (35%):
comunicacin interventricular, conducto arterioso persistente, tetraloga de fallot, coartacin de aorta. Gastrointestinales (20%): ano imperforado,
atresia duodenal, malformaciones intestinales. Genitourinarias (20%): reflujo ureteral, agenesia renal. Msculo esqueltico (13%): vertebrales,
costales, defecto de las extremidades y del sistema nervioso central (10%). La asociacin de algunas de estas anomalas se denomina asociacin
VACTERL (Vertebrales, Anoreactales, Cardiacas, Traqueales, Esofagicas, Radiales, renales y de extremidades (L; limbs en inlges) y se presenta con una
frecuencia de 10%. El origen de la AE es poco claro an pero se atribuye a una alteracin en la migracin de los pliegues laterales o a una detencin
del crecimiento en el momento de la evaginacin. En la mayor parte de los casos el esfago posterior no se separa totalmente de la trquea, lo que
da lugar a distintas variedades de fstula traqueoesofgica o a hendidura. Fstula traqueoesofgica (TEF): conexin entre el esfago y la trquea,
permite que la comida (desde el esfago) entre en los pulmones (aspiracin). La comida en los pulmones puede causar neumona (referida a
neumona por aspiracin), lo cual puede ser muy serio. El tipo ms difcil para diagnosticar es la fstula traqueoesofgica tipo H, donde el esfago no
termina cerrado, pero hay una fstula o conexin entre la trquea y el esfago, el aire que entra al estmago (desde la trquea) y la comida puede
entrar a los pulmones. El aire en el estmago puede causar hinchazn en el abdomen del beb y puede hacer que el beb est molesto.
DIAGNOSTICO: El diagnstico prenatal se establece con ultrasonografa y puede sospecharse durante el embarazo por polihidramnios, incapacidad
de identificar el esfago fetal o ausencia de burbuja gstrica. Diagnostico en sala de parto, la complicacin al paso de la sonda (12-14 Fr) para
verificar la permeabilidad del esfago; sialorrea, distensin abdominal, dificultad respiratoria, tos o cianosis son algunas manifestaciones en el recin
nacido. Al darle de comer: vmitos/atragantamiento , regurgitacion, crisis de sofocacin con tos tras las tomas lo que sugiere fstula superior. Puede
apreciarse abdomen escavado a la inspeccin o abdomen distendido si fstula inferior. Radiografia; toraco-abdominal, con sonda radiopaca o medio
de contraste hidrosoluble, el estudio muestra el fondo de saco, ciego, del esfago atrsico. Broncoscopia; localizacin, tamao ycatacteristicas de la
fistula. TRATAMIENTO: El adecuado ambiente trmico, el suministro de lquidos, glucosa y electrlitos, la posicin semifowler. Drenaje del cabo
proximal con sonda de doble lumen (sonda de Repogle) para aspiracin constante, antibiticos penicilina G sdica cristalina 50 000 U/Kg si existe
neumona, iniciar doble esquema de antibitico (ampicilina mas aminoglucocido). Intervencin quirrgica, reestablecer la continuidad del esfago.
Gastrostoma y esofagectoma: Plastia esofgica, plastia esofgica y cierre fstula, sustitucin esofgica, cierre fstula, realizar esofagograma 8-10 dias
despus de la ciruga. COMPLICACIONES: Preoperatorio; bronconeumona, atelectasia. Posoperatorio Inmediatas; neumotrax, mediastinitis,
dehiscencia de la anastomosis. Tardias; fistula traqueoesofagica recurrente, estenosis de la anastomisis, reflujo gastroesofgico (de lo mas comn),
estrecheses anatmicas, traqueomalasia, dismotilidad esofgica. PRONOSTICO: Influye el bajo peso al nacer, presencia de neumonia,
malformaciones congenitas asociadas
b.- Colocar sonda de doble lumen
CASO CLINICO
c.- Prepara para ciruga
Recin nacida, gemela I, de madre sana de 35 aos de edad,
d.- Aplicacin de oxigeno a 3 lt/x
producto del segundo embarazo, de 35 semanas de duracin; el
embarazo anterior termin con aborto espontneo en el segundo
PREGUNTA
trimestre, seis aos antes. El embarazo actual tuvo adecuado control
Cul es el diagnstico ms probable en este caso?
prenatal, recibi cido flico, calcio, hierro y polivitaminas; se
RESPUESTA
complic por amenaza de aborto en el primer trimestre, pero se
a.- Estenosis pilrica
recomend reposo. Tres das antes del parto la madre tuvo infeccin
b.- Atresia esofgica
de las vas urinarias, por lo que se hospitaliz para ofrecer
c.- Oclusin intestinal
tratamiento y adicionalmente recibi esquema de tres dosis de
d.- Enterocolitis necrotizante
inductores de la maduracin pulmonar; tuvo rotura de membranas
cuatro horas antes del parto, por lo que bajo bloqueo peridural se
PREGUNTA
realiz la cesrea; se obtuvo la gemela I, quien recibi reanimacin
Cul es el manejo inicial para esta paciente?
con presin positiva intermitente durante 20 segundos; la
RESPUESTA
calificacin de Apgar fue 6/8, peso de 2,000 g, con abundante
a.- Evitar las prdidas de calor, la colocacin semisentada a 45,
salivacin por boca, que requiri aspiracin frecuente, dicultad a la
aspiracin de fondo de saco ciego, correccin de hipoglucemia e
aspiracin de secreciones, a la primera toma de leche presento tos,
hipocalcemia, soluciones parenterales.
disnea y cianosis leve.
b. Posicin semifowler, aspiracin permanente de secreciones, del
cabo proximal, correccin de trastornos electrolticos
PREGUNTA
c.- Aspiracin de fondo de saco ciego, la correccin oportuna de
Cul es la conducta a seguir inmediata mas adecuada?
hipoglucemia e hipocalcemia, el aporte de soluciones parenterales.
RESPUESTA
d.- Evitar prdidas de calor, la correccin de hipoglucemia e
a.- Colocar en silla porta-bebe a 45 grados.
hipocalcemia, el aporte de soluciones parenterales.

CURSO ENARM CMN SIGLO XXI TEL: 36246001

Pharmed Solutions Institute

PGINA 312

MANUAL DE TRABAJO DEL CURSO ENARM CMN SIGLO XXI


HERNIA HIATAL CONGENITA
CIENCIAS BASICAS: Se debe al cierre incompleto de la membrana pleuroperitoneal (abertura o dehiscencia congnita del diafragma) o al retorno
prematuro del intestino a la cavidad abdominal con herniacin de las vsceras abdominales a la cavidad torcica. SALUD PUBLICA: La hernia
diafragmtica congnita ocurre entre 1 de cada 4,000-5,000 recin nacidos vivos. La mortalidad vara entre un 40-70% dependiendo el grado de
hipoplasia pulmonar y el desarrollo de hipertensin pulmonar. La relacin hombre: mujer es de 1,8:1. El 50-60% presentan malformaciones
asociadas, sobre todo defectos cardiacos (9-23%), defectos del tubo neural (28%), trisomas 18,13,21 la malrotacion intestinal es una constante en
hernias voluminosas. PATOGENIA: Existen dos teoras acerca de la embriognesis de la hernia diafragmtica: 1. Crecimiento pulmonar anormal que
produce desarrollo diafragmtico anormal. 2. Defecto diafragmtico con hipoplasia pulmonar secundaria. La etiologa de la hernia diafragmtica no
es clara, han sido reportados casos familiares sugiriendo predisposicin gentica. En 20% de los casos se ha presentado polihidroamnios y es de mal
pronstico. Talidomida y quinidina se han reportado como causantes de hernia diafragmtica en humanos. Esta patologa se asocia prcticamente
siempre con efecto de masas, que se manifiesta en el feto por desplazamiento del mediastino, compresin pulmonar y reduccin del tejido
pulmonar antes de la semana 16, momento en el que desarrollo bronquial es completo determina reduccin del nmero de bronquios y alveolos del
pulmn en desarrollo que conlleva a la hipoplasia pulmonar. El pulmn hipoplsico en hernia diafragmtica presenta una reduccin cuantitativa y
cualitativa de surfactante y un compromiso de la distensibilidad pulmonar. CLASIFICACION: De acuerdo a su localizacin anatmica en congnitas y
adquiridas, las congnitas son: 1. HERNIA POSTEROLATERAL DE BOCHDALEK es el tipo ms comn representa 85-90%, siendo la mayor ubicacin
izquierda 80%, derecha 15%, y mixta 2%, asociada a polihidramnios 70-75%. 2. HERNIA DE MORGAGNI, 3. HERNIA PERITONEOPERICARDICA.
Adquiridas: las postraumticas. DIAGNOSTICO: Algunos neonatos asintomaticos y se desarrollan sin problemas excepto cuando por alguna razn por
lo general una infeccin respiratoria o problemas gastrointestinales se descubre en un estudio radiolgico que tienen hernia diafragmtica. Clinica:
dificultad respiratoria progresiva, taquipnea, retraccin, quejido, cianosis, tortax asimtrico, distensin torcica unilateral, abdomen excavado,
disminucion o abolido del murmullo vesicular de hemitrax afectado, ruidos hidroaereos en hemitrax ipsilateral, ruidos cardiacos desplazados al
lado contralateral. El diagnstico prenatal se realiza por ecografa, se basa en la visualizacin de rganos abdominales en el trax y el signo
ecogrfico distintivo es una masa ocupada por lquido inmediatamente por detrs de la aurcula y el ventrculo izquierdos, en la parte inferior del
trax visualizando en una vista transversal. Otros signos ecogrficos que hacen sospechar el diagnstico son la ausencia del estmago en el
abdomen, desplazamiento del mediastino, permetro abdominal fetal pequeo y polihidramnios. Radiografia: imgenes multiquisticasdel lado
afectado, derivacin de mediastino y corazn hacia el lado contralateral. TRATAMIENTO: Inmediato; intubado y ventilado, esta contraindicado uso
de oxigenacin con bolsa manual (ECMO=oxigenacin con membrana extracorprea), sonda bucogastrica, oximetra de pulso. El tratamiento incluye
terapias como el uso de corticosteroides, terapia con surfactante. En tercer nicel de atencin, la sedacin, relajacin evita vasoconstriccin
pulmonar. Tratamiento de hipertensin pulmonar; tolazolina, nitroglicerina, nitroprusiato de sodio, oxido ntrico, vasodilatacin pulmonar.
Quirurgico: HERNIA DE BOCHDALEK: Puede ser de presentacin espordica o familiar. El defecto ocurre cuando la membrana pleuroperitoneal no se
fusiona con las otras porciones. La fusin de la membrana pleuroperitoneal ocurre primero del lado derecho y luego del izquierdo. Adems la
proteccin del hgado en el lado derecho, hace que sea ms frecuente la hernia de Bochdalek del lado izquierdo. La hipoplasia pulmonar se debe a
que los pulmones se encuentran en la fase glandular del desarrollo y el intestino torcico ocupando espacio, impide el normal desarrollo. Por lo
tanto existe una disminucin del lecho vascular y de la segmentacin bronquial. Es frecuente la mal rotacin de los intestinos debido a que los
intestinos se desplazan al trax antes de la fijacin del ciego al cuadrante inferior derecho del abdomen. HERNIA DE MORGAGNI: Causada por un
defecto en la unin entre las porciones central y lateral del diafragma anteromedial del diafragma, orificio herniario a nivel retroxifoideo, se
diagnostican a cualquier edad en forma accidental al efectuar un examen radiolgico de trax por otra patologa, la mayora se diagnostica en la
edad peditrica por presentar dicultad respiratoria, en la edad adulta, la mayora de las veces en forma incidental. Su patogenia es desconocida,
aunque actualmente se sugiere una etiologa multifactorial en la que se implican factores hereditarios en relacin con otros sndromes
malformativos como los sndromes de Down, Turner, Prader-Willi y Noonan. PENTALOGIA DE CANTRELL: Onfalocele de lnea media superior, hernia
diafragmtica anterior, esternn hendido, ectopia cordis, defectos intracardiacos. PRONOSTICO: Mortalidad de 80% en etapa prenatal, 40% en
etapa posnatal, moratlidad 100% con hipoplasia pulmonar bilateral intensa, sntomas desde el nacimiento, hipertensin pulmonar intratable.
COMPLICACIONES: Displasia broncopulmonar, hiperreactividad de la via area, enfisema lobar en pulmn ipsilateral a la hernia, reflujo
gastroesofgico, retraso en crecimiento pondoestatural.
CASO CLINICO
Lactante de 5 meses de edad, femenino, eutrfico, con historia de
cuadros de aparente dolor abdominal a repeticin. Consult por
presentar sntomas respiratorios, fiebre e irritabilidad, se solicit
radiografa de trax en proyecciones anteroposterior y lateral que
muestra reduccin del parnquima pulmonar en hemitrax derecho
y desplazamiento del mediastino y del corazn hacia la izquierda.

PREGUNTA
Cul es la conducta diagnostica mas adecuada?
RESPUESTA
a.- Serie gastroesofgica.
b.- Radiografia de torax.
c.- Tomografia de abdomen.
d.- Endoscopia superior.

PREGUNTA
Cul es la conducta diagnostica siguiente mas adecuada?
RESPUESTA
a.- Serie gastroesofgica.
b.- Serie gstrica completa.
c.- Tomografia de abdomen.
d.- Resonancia magntica.

PREGUNTA
Cual de las siguientes manifestaciones es menos frecuente en esta
patologa?

CASO CLINICO
Recin nacido varn, que nace por Cesrea, SGB +, Transaminasas
Maternas elevadas. Apgar 6/7, Peso 3670 gramos, talla 51
centmetros. Presenta, en el postparto inmediato, dificultad
respiratoria con cianosis, aleteo nasal, quejido, tiraje intercostal,
crepitantes bibasales, disminucin de entrada de aire y del murmullo
vesicular en hemitorax Izquierdo.

CURSO ENARM CMN SIGLO XXI TEL: 36246001

RESPUESTA
a.- Insuficiencia Respiratoria Severa desde el nacimiento.
b.- Disminucin o ausencia de murmullo vesicular.
c.- Ruidos hidroareos (RHA) en trax.
d.- Desplazamiento de ruidos cardiacos al lado contralateral.
PREGUNTA
Cul es la trisoma que no se ha relacionado en esta patologa?
RESPUESTA
a.- X.
b.- 18.
c.- 21.

Pharmed Solutions Institute

PGINA 313

MANUAL DE TRABAJO DEL CURSO ENARM CMN SIGLO XXI


ESTENOSIS HIPERTROFICA DEL PILORO
CIENCIAS BASICAS: Tambin se le ha denominado estenosis pilrica hipertrfica infantil, para diferenciarla de la estenosis adquirida que se observa
en el adulto, se ha propuesto que tiene herencia autosmica dominante. La estenosis hipertrfica del ploro es la causa ms frecuente de ciruga en
los lactantes menores de 2 meses, superada solamente por las hernioplastias. SALUD PUBLICA: Se estima que hay entre 1-5 por cada 1,000 recin
nacidos vivos. 7% asociado a malformaciones: malrotacin intestinal, uropata obstructiva, atresia esofgica, hernia hiatal. Se presenta entre las 2 y
8 semanas de edad, con un pico entre las 3 y las 5 semanas. Es 4 a 5 veces ms comn entre varones que en mujeres, con una mayor incidencia en
primognitos. PATOGENIA: Etiologia multifactorial, existe una reducida expresin se la sintetasa de oxido ntrico neuronal (relajante de musculo
liso), se han desarrollado algunas teoras relacionadas con un desequilibrio neurohormonal, o de mediadores neuroendocrinos, en el control del
tono del esfnter pilrico, en la produccin de gastrina y la motilidad del estmago. Una propuesta es una descoordinacin entre el peristaltismo
gstrico y la relajacin pilrica, lo que lleva a una contraccin gstrica contra un ploro cerrado, que causara hipertrofia en el msculo pilrico. Otras
teoras proponen una elevacin en las concentraciones de gastrina (hipergastrinemia), debido a un aumento hereditario en el nmero de clulas
parietales de la mucosa gstrica que llevan a un ciclo de aumento en la produccin de cido gstrico, contracciones cclicas peridicas en el ploro y
vaciamiento gstrico lento, esto lleva a hipertrofia e hiperplasia de las fibras musculares del esfnter pilrico. Administracin de eritromicina/
azitromicina los primeros das de vida, se ha encontrado que la administracin de macrlidos es factor de riesgo. El hbito de fumar materno se ha
reportado como posible factor de riesgo para estenosis pilrica. Otras investigaciones sealan que al estudiar muestras de las capas musculares
depacientes afectados de estenosis pilrica, en comparacin con controles, se hanencontrado cantidades disminuidas de las terminales nerviosas y
de los neurofilamentos. DIAGNOSTICO: El sntoma ms caracterstico son los vmitos posprandiales, no biliosos, progresivos hasta ser incluso en
proyectil o explosivo, que eventualmente impiden la alimentacin adecuada del lactante, en la tercera a octava semana de vida. Es es un nio con
distensin abdominal, ansioso, succiona continuamente sus manos y se queda con hambre. Conforme continan los sntomas el paciente adelgaza
(pierde peso) y se torna hambriento, toma con avidez la leche, pero rpidamente la vomita. Algunos pacientes presentan ictericia (10%) de grado
variable. Tambin se pueden encontrar signos clnicos de deshidratacin como mucosas con saliva filante o secas, depresin de la fontanela, llenado
capilar lento y signo del pliegue presente. El abdomen no est distendido; cuando la emaciacin ha progresado, es posible observar las ondas
peristlticas en el epigastrio. El signo patognomnico es la masa palpable en el cuadrante superior derecho del abdomen, en forma de aceituna u
oliva, que corresponde al ploro engrosado (oliva pilrica 70-90%), localizada en la unin del epigastrio con hipocondrio derecho. Signo del bibern:
al tomar lquidos por bibern se observan ondas peristlticas. Signo de la pelota de golf: ondas peristlticas de izquierda a derecha hacia la zona
pilrica. El hallazgo ms caracterstico es la alcalosis metablica, por la prdida de hidrogeniones a travs del contenido gstrico; con el vmito se
pierde tambin cloruro lo que lleva a hipocloremia e hipocaliemica. En la estenosis del ploro el hemograma est normal; se describe sin embargo
que puede haber hemoconcentracin por la deshidratacin. Por ello puede encontrarse elevacin de la creatinina; la densidad urinaria tambin
puede estar elevada, sin datos de infeccin. La radiografa simple de abdomen muestra distensin de la cmara gstrica con poco o escaso aire
distal. La serie esfago gastroduodenal y ultrasonido (en caso de no palpar la oliva) muestra un estrechamiento del canal pilrico (cola de ratn) con
una o varias imgenes de cuerda; es el medio de contraste que dibuja los pliegues hipertrofiados de la mucosa pilrica; se describe tambin un
efecto de masa que se proyecta hacia el antro gstrico (signo del hombro), se puede ver imagen de doble o triple riel. En la actualidad, el mtodo
ms utilizado para confirmar el diagnstico es el ultrasonido abdominal (primera eleccin para descartar membrana duodenal fenestrada, pncreas
anular), se logra evidenciar engrosamiento de la capa muscular del ploro mayor de 2.5 mm, aumento en la longitud del canal pilrico mayor de 15
mm y estrechamiento del canal; se logra visualizar tambin hipertrofia de la mucosa en grados variables, corte axial Imagen ojo de bovino dona
Imagen tiro al blanco. La endoscopia tiene sensibilidad y especificidad 100%. TRATAMIENTO: Se deben corregir los trastornos hidroelectrolticos
(alcalosis, hipocloremia, hipokalemia y deshidratacin), posicin semifowler, sonda bucogastrica. Se pueden suministrar bolos de solucin de cloruro
de sodio al 0,9 % a 20 mL/kg para restituir volumen y electrolitos a una velocidad de 120-150ml/kg/min. Luego se puede dejar una solucin de
mantenimiento, con dextrosa al 5 % y cloruro de sodio al 0,45 % o al 0,9 %, segn el centro hospitalario, adems se debe agregar cloruro de potasio
a razn de 20 mEq/L (3.4mEq/kg/da) una vez asegurada la diuresis. La estabilizacin puede demorar unas 24 a 48 h. La ciruga no es urgente; si el
paciente es llevado a sala de operaciones con alcalosis metablica se aumenta el riesgo de apnea posoperatoria; adems, el estrs quirrgico puede
empeorar los trastornos electrolticos. Nio con electrlitos normales menos del 5% de la deshidratacin ciruga inmediata. Nios con alcalsis
metabolica e hipocloremia, pueden requer 48hrs o mas de hidratacin parenteral y correccin hdrica. Vitamina K: 0.4 mg/kg en nios con peso
inferior a 2.5 kg. > 1 ao 5-10 mg/da por va I.V. o I.M. Ranitidina 1 mg/kg/dosis cada 8 hrs. Una vez compensado el paciente se lleva a cabo la
Piloromiotoma de Fredet- Ramsted, se realiza el corte de la serosa y luego de la capa muscular circular del ploro, sin cortar la mucosa; luego se deja
sin suturar la muscular y se cierra la serosa. La operacin generalmente se hace por laparotoma, aunque tambin se puede a travs de tcnicas
laparoscpicas. COMPLICACIONES: Apnea posoperatoria, hipoglicemia, obstruccin posoperatoria (vmitos), perforacin de la mucosa duodenal,
miotoma incompleta, eventracin de la herida quirrgica; el fallecimiento ocurre en menos del 1 % de los casos, no requiere rehabilitacin, el riesgo
para el otro hijo es de 4%
PREGUNTA
CASO CLINICO
Qu trastorno hidroelectrolticos es ms probable encontrar en
Lactante de 5 semanas de edad, con peso de 3,700Kg, es llevado a
este caso?
consulta por padecimiento actual de una semana de evolucin con
RESPUESTA
vmito no biliar progresivo en chorro y que se relaciona con el
a.- Acidosis metabolica + hiponatremia
antecedente de disminucin de volumen urinario de 24hra de
b.- Alcalosis respiratoria + hipercloremia
evolucin. El lactante se encuentra activo y vido de alimento. EF:
c.- Alcalosis metablica + hipocloremia
Depresin de la fontanela anterior y disminucin de la turgencia
d.- Acidosis respiratoria + Hipernatremia
cutnea. Hay abdomen en batea y al realizar la prueba de
alimentacin se observa onda peristltica en epigastrio.
PREGUNTA
PREGUNTA
Cual de los siguientes diagnosticos diferenciales es el ms frecuente
Cul es el diagnstico ms probable?
en esta patologa?
RESPUESTA
RESPUESTA
a.- Gastroenteritis vrica
a.- Intolerancia a la lactosa
b.- Reflujo gastroesofgico
b.- Reflujo gastroesofgico
c.- Infeccin de las vas urinarias
c.- Meningitis
d.- Estenosis pilrica
d.- Volvulus

CURSO ENARM CMN SIGLO XXI TEL: 36246001

Pharmed Solutions Institute

PGINA 314

MANUAL DE TRABAJO DEL CURSO ENARM CMN SIGLO XXI


CARDIOPATIAS CONGENITAS
CIENCIAS BASICAS: Corazn primer rgano que alcanza desarrollo (de 3ra-8va semana de gestacin) funcional completo, el da 17 de gestacin
escuchamos latido cardiaco. Definicin: Conjunto de enfermedades caracterizadas por anomalas estructurales del corazn o de los grandes vasos
intratorcicos. En la transicin
CARDIOPATIAS POR FISOPATOGENIA (por orden de incidencia)
de la circulacin fetal a la
CORTOCIRCUITOS
neonatal existen 3 estructuras
Derecha a izquierda (CIANTICAS)
ACIANTICAS
nicas: el conducto venoso, el
Sangre pobremente oxigenada pasa a la circulacin
Propician aumento del volumen circulatorio en el sistema pulmonar,
agujero oval y el conducto
sistmica, ello condiciona a hipoxemia y cianosis
a expensas de una disminucin del gasto cardiaco sistmico, no
cianosis pero si hipertrofia del VD y vasos pulmonares
arterioso. La sangre oxigenada
Flujo
pulmonar
Flujo pulmonar disminuido
Flujo
pulmonar
aumentado
Flujo pulmonar disminuido
llega de la vena umbilical a la
aumentado
(cortocircuito de I-D)
(obstructivas)
vena cava inferior a travs del
Transposicin
de
Tetraloga de Fallot (5-7)
Comunicacin interventricular CIV
Estenosis pulmonar
conducto venoso, evitando el
grandes vasos TGV (3-5)
Estenosis pulmonar critica
(25-30)
Estenosis aortica
hgado. Gracias a la anotoma
Tronco arterioso (1-2)
o atresia pulmonar
Comunicacin interauricular CIA (6Coartacion aortica
de la aurcula derecha, la
Ventrculo nico
Anomalia de Ebstein
8)
mayor parte de la sangre
Hipoplasia de VI
Atresia tricuspidea
Conducto arterioso persistente CAP
proveniente de la vena cava
CAVP
Atresia pulmonar
(6-10)
inferior (sangre oxigenada) se
Defecto de cojinetes endocardicos
dirige al corazn izq. Por medio
del agujero oval, mientras que la sangre proveniente de la vena cava superior (sangre desoxigenada), se dirige al ventrculo derecho a travs de la
vlvula tricspide. La sangre que irriga el pulmn tienen un contenido de oxigeno sumamente bajo y esto produce una constriccin intensa de los
vasos pulmonares que eleva la resistencia vascular. El ventrculo der. bombea la sangre hacia la arteria pulmonar; sin embargo la alta resistencia
pulmonar hace que la sangre fluya principalmente a travs del conducto arterioso hacia la aorta descendente. Los cambios fisiolgicos y mecnicos
que ocurren al nacer y que propician el cierre del conducto arterioso son: la disminucin de la resistencia pulmonar (por la distencin alveolar y el
incremento del contenido de oxgeno en la sangre), la secrecin de bradicinina por el endotelio pulmonar y disminucin de la contraccin srica de
las prostaglandinas placentarias. Junto con el flujo pulmonar, estos cambios incrementan el volumen y la presin en la aurcula izq., rebasando la de
la aurcula derecha y cerrando el agujero oval. SALUD PUBLICA: Su incidencia es del 1%, lo que las coloca entre las malformaciones ms frecuentes.
Segunda causa de mortalidad en menores de 5 aos. En cerca de 90% de los casos no existe una causa identificable, pero existen ciertos factores
como como la prematurez, cromosomopatas, infecciones congnitas, diabetes, alcohol, entre otros. CLASIFICACION: Ver cuadro anexo.
TETRALOGIA DE FALLOT: Cardiopata congnita ms comn de las cianticas, despus del periodo de lactancia (3-6 por 10,000). Es una
malformacin ventrculo-arterial, implica obstruccin del tracto de salida del VD (estenosis pulmonar infundibular); CIV amplia; hipertrofia del VD y
dextroposicin de la aorta con cabalgamiento sobre la CIV, todo esto lleva a obstruccin del flujo del VD, hacia la arteria pulmonar y cortocircuito de
derecha a izquierda. 15-20% tienen defectos extracardiacos asociados. Leve no muestran cianosis hasta despus del ao, en grave cianosis 83-6
meses) poco despus de nacimiento, de no corregirse el paciente presenta color azulado, acropaquias de pies y manos (dedos en palillo de tambor,
hipocratismo 10-12 meses) y disnea, pueden sufrir ataques hipercianticos paroxsticos con el esfuerzo que lo pueden llevar al sincope, si se ponen
en cunclillas mejora su estado (incrementa resistencias perifricas y favorece el paso de sangre de VD hacia la pulmonar, mejora la oxigenacion. La
viabilidad del producto depende del conducto arterioso y del agujero oval. Se desarrollan crisis de hipoxia (defecar, llanto, toma de bibern) cuando
la hipertrofia ventricular derecha provoca mayor obstruccin pulmonar y mayor cortocircuito de recha a izquierda, ante una demanda incrementada
de oxigeno. A la ausculatacion soplo sistlico expulsivo 2 espacio intercostal, soplo sistlico en 4to espacio intercostal, irradiacin en barra. Rx:
corazn con aspecto de bota (zapato sueco), trama vascular pulmonar disminuida, pulmonar excavada, corazn normal. ECG: hipertrofia del VD y
desviacin del eje elctrico, Ecocardiograma: Estndar de oro, para dx y gravedad. TRATAMIENTO: Ciruga correctiva (despus de 3-4 meses de
edad, asintomticos o con cianosis leve de 3-4 meses) de Rastelli, inicialmente PG E1 para mejorar sntomas. Crisis de hipoxia (cursan con mortalidad
de 50% en primer evento) prevencin con propanolol 2-4mg/kg/dia, policitemia con ASA, salinoferesis, prevencin de endocarditis bacteriana,
diagnostico y tratamiento para anemia por defuiciencia de hierro. TRANSPOSICION DE LOS GRANDES VASOS: Prevalencia 5%. Aqu la arteria
pulmonar nace del VI y la aorta del VD. Mas frecuente en varones, hijos de madre diabtica, aosa, macrosomicos. Esta asociado a dextrocardia,
situs inversus, asplenia, necesaria la presencia de CIV, CIA y/o PCA, solo asi se hace la mezcla de sangre de los 2 circuitos, si hay una cominicacion
grande, cianosis leve despus de la primera semana de vida, comunicacin pequea, hipoxemia arterial severa, acidosis metabolica, insuficiencia
cardiaca, hipoglicemia, hipocalcemia, se asocia tambin con estenosis infundibular pulmonar. La cianosis y la taquipnea suelen ser evidentes en las
primeras horas o das de vida y representan una urgencia mdica. La Rx: normal o datos de hiperflujo pulmonar, cardiomegalia imagen en forma de
huevo. Ecocardigrafa es estndar de oro para dx. Tratamiento: Dar PG E1 para mantener abierto el conducto arterioso, la correccin quirrgica
(septostomia con baln tipo Rashkind, si no se prevee ciruga inmediata) final es la conmutacin o switch arterial, en las 3 primeras semnas de vida.
Pronostico: Sin tratamiento el 30% fallece a los 7 dias, 50% al mes, y 90% al ao. Con tratamiento quirrgico la mortalidad es de 10%, complicaciones
posquirrgicos arritmias, insuficiencia coronaria. TRONCO ARTERIOSO: Defecto en el desarrollo del tronco arterioso en aorta y arteria pulmonar,
dejando un tronco arterial nico que nace del corazn, para ambas circulaciones. COMUNICACIN INTERVENTRICULAR: Cardiopata congnita ms
comn (25%) en la infancia, durante los dos primeros aos de vida casi la mitad de estos defectos se cierran espontneamente y la mayora lo hace
para los 10 aos. Asociado a trisoma 13-15 (90%), trisoma 18 (99%), trisoma 21 (50%), XO o Turner (35%), VATER, Sx. alcohlico fetal, fetopata
diabtica, asociado a PCA y CIA. Se dividen en; perimembranoso (90%) y muscular (5%), infundibular y trabecular. El cuadro clnico depende de la
cantidad de sangre que fluye hacia la circulacin pulmonar, en la mayora es pequeo y permanecen asintomticos. Defectos de <10mm no
producen sintomatologa, defectos mayores sobrecarga de VD, incremento del flujo sanguneo pulmonar, hipertensin pulmonar, insuficiencia
cardiaca congestiva: disnea, taquicardia, galope, hepatomegalia, dificultad para la alimentacin, infecciones pulmonares recurrentes. Auscultacin
soplo holosistlico regurgitante, a veces frmito. La Rx: cardiomegalia a expensas de cavidades derechas y aumento de trama vascular, pulmonar
aumentada, ecocardiograma, cateterismo cardiaco si hay duda diagnostica. La CIV muscular es menos comn y su cierre espontaneo es ms
probable. Tratamiento: Anticongestivos diurticos (furosemida, espironolacona), antihipertensivos (captopril, sildenafil), digital, profilaxis para
endocarditis bacteriana. Intervencionista cierre con dispositivos. Ciruga, solo si hay retraso en el crecimiento, grandes defectos o hipertensin
pulmonar, cierre a cualquier edad, si no responde a tratamiento medico. Pronostico: 30-40% de las CIV cierran en los 2 primeros aos de vida, 50% a
los 10 aos, con repercusin hemodinmica 5-10%. COMUNICACIN INTERAURICULAR: Cardiopatia congnita ms comun en el adulto (1 de cada 3
casos). En cualquier parte del septo, los principales tipos son: ostium secundum (50-70%), ostium primum (30%) y seno venoso (10%), el grado de
cortocircuito y mezcla de sangre depende del tamao del defecto, generalmente no presentan sntomas hasta 4ta dcada de vida, la presentacin
ms comn es la falla cardiaca derecha, por sobrecarga de volumen de cavidades derechas, incremento de flujo sanguneo pulmonar e hipertensin

CURSO ENARM CMN SIGLO XXI TEL: 36246001

Pharmed Solutions Institute

PGINA 315

MANUAL DE TRABAJO DEL CURSO ENARM CMN SIGLO XXI


pulmonar. Sindromes asociados Sx. Holt-Oram, Sx. Down, Sx. Pierre Robin. Clinica: dificultad respiratoria, taquipnea, cianosis al alimentarse datos de
insuficiencia cardiaca, a la auscultacin, S2 con desdoblamiento fijo, por hiperflujo pulmonar, un soplo de eyeccin de bajo grado, pulsos perifricos
normales o amplios, habitualmente es asintomtica, los pacientes son delgados, con fotiga ligera y discreta falta de aire, infecciones respiratorias
bajas frecuentes. Radiografia: diltatacion de auricula y ventriculo derechos, arteria pulmonar dilatada, flujo pulmonar aumentado, boton artico
excavado, en ECG ondas p altas, BRDHH (90%), ecocardiograma. Tratamiento: Anticongestivos con diurtico y antihipertensivos, cierre percutneo
con dispositivo Amplatzer, correccin quirrgica de defectos grandes a edad de 4-5 aos o antes si hay repercusiones, contraindicada cuando la
relacin de las resistencias arteriales pulmonar: sistmica es mayor a 0.7. Requiere profilaxis para endocarditis bacteriana. Pronostico: cierre
espntaneo en 40% en los primeros 4 aos de vida, para evolucionar con hipertensin ulmonar, insuficiencia cardiaca congestiva, arritmias,
embollias paradjicas, muerte sbita. CONDUCTO ARTERIOSO PERSISTENTE: Es la cardiopata congnita mas frecuente en Mexico, mayor incidencia
en prematuros y en mujres, habitualmente relacionado con otras malformaciones cardiacas. Se asocia con trisoma 21 (55%), 13, 18. Conducto
arterioso esta situado entre la arteria pulmonar izquierda y la aorta descendente. Su cierre normal ocurre 12-15hrs post nacimiento, el cierre
anatmico o fisiolgico 4-8 semanas (88%), si no sucede la sangre aortica se desva hacia la arteria pulmonar, el cierre despus de la infancia es
infrecuente, el principal factor de riesgo es la premadurez e infeccin materna por rubeola. Los conductos grandes pueden producir presin de pulso
amplio, pulsos perifricos saltones, insuficiencia cardiaca y retraso en el crecimiento a largo plazo. Auscultacin: soplo caracterstico es sistlico;
suele describirse como ruido de maquinaria, en defecto grandes puede haber insuficiencia cardiaca, infecciones respiratorias de repeticin,
retardo en crecimiento, precordio hiperactivo. La Rx: incremento en la trama vascular y una arteria pulmonar ensanchada, cardiomegalia izquieda.
Ecocardiograma estudio de eleccin, sobrecarga diastlica del ventrculo izquierdo. Las probabilidades de que cierre espontneamente son escasas,
para ayudar a que se cierre se da indometacina (inhibidor de la sintesis de PG) o ibuprofeno dosis unica. En casos extremos es necesaria la ciruga
correccin con Coil, o cierre con toracotomia, ya que disminuye el riesgo de endocarditis infecciosa. COARTACION DE LA AORTA: Causada por el
estrechamiento de la aorta en algn punto desde el cayado hasta la bifurcacin de las arterias iliacas. Este trastorno se relaciona con vlvulas
aorticas bicspides (85%). Relacin importante en nias con Sx. de Turner (15%). Hay dos tipos: coartacin juxtaductal antes conocida como del
adulto y la infantil coartacin preductal. Cuadro clnico depende de la localizacin; la presin arterial esta aumentada en los vasos proximales a la
coartacin y conlleva una disminucin distal a la estenosis, signo clsico es la disparidad de la presin arterial entre los brazos y las piernas,
hipertensin arterial sistmica, crisis hipertensiva, soplo eyectivo G2 interescapular. La Rx: congestin pulmonar, cardiomegalia, signos de Roessler
(costillas con muescas). Ecografa cardiaca estndar de oro para dx. El tratamiento de la forma infantil se administra PG E1, manejo de
hipertensin, la ciruga correctiva (aqngioplastia con baln reseccin y anastomisis) es necesaria, en la juxtaductal se deben controlar la
hipertensin y la falla cardiaca y despus realizar la ciruga. Pronostico: mayor sobrevida mientras la correcion se mas temprana de preferencia antes
de los 8 aos., riesgo de recoartacion 50%. ENFERMEDAD DE EBSTEIN: Malformacion congnita de la valvula tricspide (valavas desplazadas hacia el
VD), VD atrializado, hipoplasia funcional del VD, auricula derecha dilatada, las vlvulas estn adheridas al septum. Se afectan la valcula septal y la
posterior que desplazan y deforman al VD. Menos de 1% de las cardiopatas congnitas, se asocia con ingesta de litio en el embarazo, se asocia con
sndrome de WPW. Clinica casos leves disnea, fatiga, cianosis durante el ejercicio. Casos severos cianosis intensa, insuficiencia cardiaca congestiva
derecha, crisis de hipoxia, hipocratismo digital, desdoblamiento fijo de R2, soplo sistoloco suave de regurgitacin, episodios de arritmias, algunos
cursan asintomticos hasta la adolscencia (sincope, taquicardias), se ha descrito mareos, perdida transitoris de la visin, signos de embolia cerebral.
R silueta cardiaca en garrafa, cardiomegalia extrema, ndice cardiaco >75%. ECG: ondas P muy altas y ensanchadas, BRDHH, sndrome de WPW,
ecocardiograma insuficiencia tricuspidea, VD pequeo, AD grande, CIA. Neonatos con cianosis severa, infusin de PGE1, inotrpicos, correccin de
acidosis metabolica, insuficiencia cardiaca congestiva, diurticos, digital, taquicardia supraventricular, digoxina propanolol. Insuficiencia cardiaca
refractaria a tratamiento anuloplastia tricuspidea, sustitucin protsica de VT y cierre de CIA, operacin tipo Fontan. Complicaciones: arrtimias, TSV
paroxstica, ICC, absceso cerebral, endocarditis infecciosa.
CASO CLINICO
Una RN de 19 das de vida es admitida en la sala de emergencias en
estado grave. En el examen fsico, la paciente se presentaba
hipotnica, con hipotermia, gimiente, ciantica con aleteo de la
nariz, taquipneica (80 rpm), con tiraje y severa retraccin subcostal y
esternal, murmullo vesicular MV presente con crepitantes difusos,
roncus, taquicrdica (190 lpm), pulso dbil, ritmo cardaco regular
(RCR), ruidos cardacos normofonticos (RCNF), soplo sistlico
+++/4+, abdomen distendido, hgado a 4 cm por debajo del reborde
costal derecho. La paciente es remitida en la Unidad de Cuidados
Intensivos Neonatales y se realiz intubacin orotraqueal (IOT), con
FiO2 al 100%, establecindose un acceso perifrico e inicindose
correccin con solucin de bicarbonato de sodio debido a la
presencia de acidosis metablica, diseccin venosa de la vena axilar
derecha y la siguiente medicacin: furosemida, dobutamina,
milrinona, fentanilo y midazolam. Se realizaron exmenes (PCR
negativa, cultivo de orina con presencia de Staphylococcus coagulasa
resistente a ampicilina/penicilina), se realiz una radiografa de trax
que mostr la presencia de cardiomegalia.
PREGUNTA
Cul es su diagnostico mas probable?
RESPUESTA
a.- CIV.
b.- CIA.
c.- CAP
d.- TGV
CASO CLINICO

CURSO ENARM CMN SIGLO XXI TEL: 36246001

Un lactante de cinco meses de edad fue llevado a la consulta de su


pediatra de Atencin Primaria, pues la madre refera que en los
ltimos das se cansaba y sudaba mucho coincidiendo con las tomas.
El embarazo, el parto y el periodo neonatal haban sido normales, y
no presentaba antecedentes mdicos quirrgicos personales,
nicamente la madre refera infecciones de vas respiratorias. En la
exploracin fsica, el pediatra constat taquipnea con leve tiraje
subcostal, saturacin de oxgeno del 98%, taquicardia y un soplo
pansistlico regurgitante grado 3/6 audible en mltiples focos, con
mxima intensidad a nivel del tercer-cuarto espacio intercostal
izquierdo y galope. Peso bajo para la talla, por lo deriv al lactante a
Urgencias del hospital, dada su sospecha diagnstica.
PREGUNTA
Cul es el diagnstico ms probable en este caso?
RESPUESTA
a.- Comunicacin interventricular
b.- Comunicacin interauricular
c.- Tetraloga de Fallot
d.- Persistencia de conducto arterioso
PREGUNTA
Cul de los siguientes sndromes no est relacionado con esta
patologa?
PREGUNTA
a.- Sndrome de Turner
b.- Trisoma 13
c.- Sndrome de Edwards
d.- Sndrome de Klinefelter

Pharmed Solutions Institute

PGINA 316

MANUAL DE TRABAJO DEL CURSO ENARM CMN SIGLO XXI


PREGUNTA
Qu alteraciones en la fisiologa desencadenan el cuadro clnico del
paciente?
RESPUESTA
a.- Sobrecarga de VD, hipertensin pulmonar
b.- Dilatacin de aurcula derecha, hipertensin pulmonar
c.- Cardiomegalia, hipertensin de VI
d.- Dilatacin de aurcula derecha e hipertrofia de VI
CASO CLINICO
Nio de 3 meses, con cianosis al llorar y defecar. Sin antecedentes
perinatolgicos de importancia. Al mes le diagnosticaron un soplo.
Lactante eutrfico con desarrollo adecuado. Presenta frecuencias
cardaca y respiratoria normales para su edad, cianosis central leve
(SO2AA 87%), precordio calmo, pulsos simtricos en los 4 miembros,
de adecuada amplitud, sin hepatomegalia. Se ausculta en foco
pulmonar un segundo ruido nico y reforzado, y un soplo sistlico
eyectivo rudo que irradia a las axilas. No se percibe tercer ruido. La
radiografa torcica muestra una silueta cardiotmica de tamao
conservado (silueta en sueco), arco medio izquierdo excavado, punta
levantada y flujo pulmonar algo disminuido. ECG con signos de
sobrecarga de presin de cavidades derechas.
PREGUNTA
Cul es el manejo teraputico ms adecuado para prevenir las crisis
de hipoxia en este paciente?
RESPUESTA
a.- IECA
b.- Beta bloqueador
c.- Diurtico
d.- Digital
PREGUNTA
Si el paciente presenta un primer episodio de crisis hipoxica. Cul es
su pronstico?
RESPUESTA
a.- 50% de mortalidad
b.- 80% de mortalidad
c.- 10% mortalidad
d.- 30% de mortalidad
PREGUNTA
Qu manifestaciones clnicas podramos observar en este paciente
al cumplir 12 meses de edad?
PREGUNTA
a.- Ingurgitacin yugular, disnea, hipocratismo digital
b.- Policitemia, taquicardia, hipocratismo digital
c.- Hipocratismo digital, policitemia, encuclillamiento
d.- Encuclillamiento, ingurgitacin yugular, disnea
CASO CLINICO
Nio de 15 aos, en el cual se descubre casualmente un soplo
sistlico en foco pulmonar, con desdoblamiento fijo del segundo
ruido, por lo que es enviado a su cardilogo. Como antecedentes
familiares de inters presentaban, un abuelo que haba sufrido una
muerte sbita y dos hermanos asmticos. En el momento del
diagnstico estaba totalmente asintomtico, sin disnea de esfuerzo,
mareos, sncopes ni palpitaciones. Haca deporte con asiduidad, sin

CURSO ENARM CMN SIGLO XXI TEL: 36246001

presentar fatiga precoz. En la exploracin fsica se constata un


desarrollo normal para su edad, con buena coloracin e hidratacin
de piel y mucosas. En la auscultacin cardaca presenta un soplo
sistlico eyectivo en foco pulmonar, con desdoblamiento fijo del
segundo ruido. El electrocardiograma puso de manifiesto ritmo
sinusal, bloqueo auriculoventricular de primer grado y signos de
crecimiento del ventrculo derecho. La radiologa de trax evidenci
prominencia del tronco de la arteria pulmonar, con pltora,
dilatacin de aurcula y ventrculo derecho, arteria pulmonar
dilatada, botn artico excavado.
PREGUNTA
Cul es el diagnstico ms probable en este caso?
RESPUESTA
a.- Transposicin de los grandes vasos
b.- Drenaje venosos anmalo
c.- Comunicacin interauricular
d.- Comunicacin interventricular
PREGUNTA
Cul de las siguientes complicaciones es menos frecuente
encontrar en el caso?
RESPUESTA
a.- Hipertensin pulmonar
b.- Insuficiencia cardiaca congestiva
c.- Arritmias, muerte sbita
d.- Hipertensin arterial
PREGUNTA
Qu riesgos tiene este paciente si se le realiza la correccin
quirrgica en este momento?
RESPUESTA
a.- Mayor morbilidad, mayor porcentaje de resolucin incompleta,
persistencia de arritmias o disfuncin ventricular
b.- Mayor morbilidad, hipertensin arterial pulmonar, bloqueo de
primer grado
c.- Fibrilacin auricular, resolucin incompleta, hipertensin
pulmonar
d.- Persistencia de arritmias, bloqueo de primer grado, hipertensin
pulmonar
CASE CLINICAL
You are called to the nursery to see neuborn with hypotonia and
syndromic features. The infant has a relatively flat occiput, upslanted
palpebral fissures, a flat nasal bridge with epicanthal folds, and a
small mouth whith a protruding tongue. The hands are both short
and wide, with incurving 5th digitis. A heart murmur is appreciated
at the left sternal border.
QUESTIONS
Which of the following is the most likely diagnosis?
ANSWERS
a.- Turner Syndrome
b.- Trisomy 13
c.- Trisomy 21
d.- Trisomy 18

Pharmed Solutions Institute

PGINA 317

MANUAL DE TRABAJO DEL CURSO ENARM CMN SIGLO XXI


DESNUTRICION ENERGETICO-PROTEICA
CIENCIAS BASICAS: La nutricin es el conjunto de funciones que se realizan en multiples secuencias coordinadas o concatenadas entre si y atraves
de las estructuras moleculares, que constituyen los organelos celulares en todas y cada una de las clulas de la economa. La desnutricin es un
estado patolgico, inespecfico, sistmico y potencialmente reversible, originado por la deficiente incorporacin de los nutrimentos a las clulas del
organismo presentado con diversoso grados de intensidad y varias manifestaciones clnicas de acuerdo con factores ecolgicos. SALUD PUBLICA:
Presente en 6 millones de nios menores de 5 aos, desnutrcion crnica en 226 millones y aguda en 183 millones. PATOGENIA: Primaria ingestin
insuficiente de alimentos, ndole social, poltica, econmica, ambiental, psicolgico y cultural. Secundaria a procesos digestivos o excretorios que
caracterizan la nutricin una vez ingeridos los alimentos. El ciclo de desnutricin-infeccin; incrementa la morbi-mortalidad, diarrea, sarampin,
enfermedades respiratorias, desencadenan anorexia, disminucin de la absorcin de nutrimientos, incremento de perdidas urinarias de nitrgeno y
electrolitos, aumento del gasto energtico basal. CLASIFICACION: Leve, moderada, grave; Marasmo, Kwashiorkor, marasmo-Kwashiorkor.
Clasificacion Wellcome% Kwashiorkor 80-60%, Marasmo <60%. DIAGNOSTICO: Universales se encuentran siempre independientemente de la
desnutricin energtico proteica, presentan retraso en el crecimiento y no dficit de este, estos son.; disfuncin (aplanamiento de vellosidades
intestinales, sndrome de mala absorcin), dilucin (osmolaridad baja, Na, K, albumina, cloro, zinc bajos) y atrofia (disminucin de masa muscular).
Circunstanciales no necesariamente presentes, desencadenados por circunstancias ecolgico-ambientales estos son; edema, cada del cabello,
petequias, hipotermia, piel seca (xerosica), piel seborreica (acn y fisuras), piel tipo pelagroso (eritema y dermatitis pelagrosa, descamacin,
lesiones hipercromicas), lesiones purpuricas, frialdad, cianosis, piel marmrea, red capilar muy visible, teleangiectasias, ascitis, o anasarca, red
venosa colateral, cabello de crecimiento lento, lacio, delgado, decolorado, quebradizo signo de la bandera, uas distroficas, lengua lisa, escarlata,
magenta. Agregados no se deben a la desnutricin pero estn asociados, sndrome de privacin social, grave alteracin emocional e intelectual;
anorexia, diarrea. Evaluacion del estado nutricional; peso, talla, circunferencia media del brazo con la banda de colores de Shakirs, en preescolares,
cuando no se conoce su edad (normal >14cm verde, desnutricin leve-moderada 12.5-14cm naranja, desnutricin grave <12.5cm rojo), emaciacin
visible, temperatura corporal, frecuencia cradiaca, repiratoria, pulso, signos y sntomas de deficiencia nutrimental, EGO, urocultivo, electrolitos
sricos, QS, BH, coprocultivo, coproparasitoscopico, , funcion renal y heptica, diuresis horaria, PFH, radiografia de torax. Factor antropomtrico es
%peso/edad x 100 = Normal 90-100%, leve grado I 76-90%, Moderada grado II 60-75%, moderada grado III <60%. BALANCE NEGATIVO DE
NUTRIMIENTOS; caracterizado por disminucin de tejido adiposo-perdida de peso, perdida de reservas protenicas, detiene crecimiento,
alteraciones hormonales, incapacidad para mantener la temperatura corporal, desventaja para las infecciones, hipoglucemias sintomticas y
asintomticas, reduccin de la ingestin de hidratos de carbono, disminucin enzimtica de lactosa, disminucin de la liberacin de
insulina/intolerancia a la glucosa, bajas concentraciones de lipasa pancretica, atrofia de intestino delgado, diarrea, infecciones intestinales giardia
lamblia, disminucin de colesterol y triglicridos, hiponatremia clsica dilucional, asi como potasio, calcio, fosforo, zinc, la disminucin de
proteinasmusculares afecta el musculo cardiaco, , hipotensin, disminucin del pulso, bradicardia, disminucin de consumo de oxigeno, falla
cardiaca por disfuncin ventricular y muerte, reduccin en la capacidad mxima de concentrar orina, aplanamiento de vellosidades del borde en
cepillo, alteraciones de la digestin y absorcin de lactosa y sacarosa, neutrfilo normales o elevados, reserva medular disminuida, disminuye C3, IL,
Ig, los rganos linfaticvos se atrofian, 70% se presenta anemia por hierro (hipocromica, microcitica), hgado graso (kwashiorkor), debido a
incremento del deposito de triglicridos, salida de lipoprotenas, alteraciones en la beta oxidacin y toxicidad del hepatocito, pncreas atrfico,
disminucin de los neurotransmisores, mielinizacion, numero de neuronas, apata, irritabilidad, reduccin de tono, los musculos respiratorios
pierden peso, alteraciones en el intercambio gaseoso , valores de albumina en deficiencia leve 3.5-3g, moderada de 3-2.5g, severa <2.5g.
TRATAMIENTO: La desnutricon grave dentro del hospital. COMPLICACIONES: Hipoglucemia, anemia, hipotermia o fiebre, infeccion local o sistmica,
deshidratacin, desequilibrio electroliico. SEVERAMENTE DESNUTRIDOS: P/E <60%, T/E <70%, emaciacin en reas glteas, estructuras oseas
visibles, MUAC (circunferencia media del brazo) <11cm. Vitamina A 200 000UI, Ac. Flico 1.5mg, zinc 2mg/kg/d, cobre 0.3mg/kg/d, hierro 3mg/kg/d.
Aporte hdrico; 100ml/kg/d en n ios con presencia de edama, 130ml/kg/d en nios sin edema. Desnutricion energtico proteica leve; 150-200
Kcal/kg/dia, 2-4 proteinas/kg/dia, 5-6 comidas al dia. Marasmo 20kcal/kg/dia, 1-4 proteinas/kg/dia, 120-150ml/kg/dia. Kwashiorkor; 50kcal/kg/dia,
<4 g de protenas/kg/dia, liquidos 100ml/kg/dia, icrementos de 35kCal/kg/dia. MARASMO:
Inicio temprano, primer ao, destete primario, emacido, irritable, evolcuion crnica, tejido
muscular y adiposo muy disminuido, hgado normal, sin edema, piel seca, protenas casi
normales, hemoglobina disminuida. Caracterizada clnicamente por emaciacin severa
(flaqueza exagerada) del tejido subcutneo, msculos y grasa, que da lugar a un aspecto,
envejecido o arrugado, debida a la carencia de protenas, su causa primaria es el aporte
inadecuado de caloras en la dieta alimentaria, si diferencia en la relacin protenas/energa.
Su peso corporal puede reducirse hasta menos del 80% de su peso normal para su altura.
Como la disminucin del aporte energtico no puede compensar el requerimiento, se utiliza
grasa corporal como sustrato de energa, con la consiguiente disminucin del tejido celular
subcutneo. Como hay un dficit de aporte tanto de protenas como de energa, el
fenmeno de adaptacin eleva niveles de glucagn y cortisol y disminuye la insulina con los
efectos orgnicos pertinentes. Quizs las causas precipitantes ms importantes del marasmo
son las infecciones y enfermedades parasitarias de la infancia. En los casos graves la prdida
muscular es obvia: las costillas sobresalen; el estmago, en contraste con el resto del
cuerpo, puede ser protuberante; la cara tiene una caracterstica simiesca (como un mono); y
las extremidades inferiores son muy delgadas. El nio parece ser slo piel y huesos.
Emaciacin: Los msculos siempre se encuentran muy disminuidos, la piel cuelga en arrugas,
sobre todo alrededor de las nalgas y los muslos. Cuando se toma la piel entre el ndice y el
pulgar se nota la ausencia de la capa habitual de tejido adiposo. Estado de alerta: raramente son desinteresados como los que sufren kwashiorkor.
Los ojos profundamente hundidos les dan una apariencia bastante despierta, menos infeliz e irritable, el nio por lo general tiene buen apetito. Los
nios con marasmo a menudo se chupan las manos violentamente o la ropa o cualquier otra cosa a su alcance. Algunas veces emiten sonidos de
succin, algunos nios son anorxicos. La materia fecal puede ser suelta, pero no es una caracterstica constante de la enfermedad. Tratar
infecciones, deshidratacin y trastornos del aparato circulatorio que frecuentemente son letales y provocan una alta mortalidad si son ignorados,
aplicacin de una dieta equilibrada, en la que el componente ms importante sea la leche, que en un principio se administra de forma diluida y poco
a poco se va haciendo ms concentrada. Una vez que la ingestin de protenas es adecuada, se aaden caloras en forma de azcar y cereales. El
tratamiento debe ser establecido poco a poco para lograr la readaptacin de las funciones metablicas e intestinales en los seres humanos.
KWASHIORKOR: Segundo ao, destete tardo, irritable, aptico, evolucin aguda, tejido adiposo presente, hepatomegalia, dermatosis alteraciones

CURSO ENARM CMN SIGLO XXI TEL: 36246001

Pharmed Solutions Institute

PGINA 318

MANUAL DE TRABAJO DEL CURSO ENARM CMN SIGLO XXI


psicolgicas, disminucin de hemoglobina y protenas, recuperacin breve, mas grave, mas frecuente, dficit grave de protenas, ingesta calrica
inadecuada, dficit de vitaminas y minerales . La desnutricin tipo edematosa es ms frecuente en lactantes con dficit de protenas pero adecuada
ingesta de carbohidratos y se caracteriza por deplecin de protenas sricas, principalmente la albumina, que tiende a ser ms severa. La gravedad
o grado de desnutricin se determina siguiendo la clasificacin de la OMS, basada en el grado de prdida de peso: Desnutricin grado I, hay prdida
del 15-25% de peso. Grado II, hay prdida de 25-40% de peso. Grado III, hay prdida de ms de 40% de peso. Es ms comn en las zonas rurales de
pases en vas de desarrollo y afecta principalmente a los nios de 2 aos de edad, asociado al periodo de destete e inicio de dieta rica en almidn y
pobre en protenas. Afecta aproximadamente una dcima parte de los nios <5aos. Clnico; es
anorxico, irritable, con facies abotagada cara de luna llena, edema peri orbitario y edema con
fvea en extremidades inferiores y manos, abdomen globoso con hepatomegalia (hgado graso) y
dilatacin de asas intestinales. La piel es seca (dermatosis) y el cabello se cae fcilmente, perdida
del inters y del apetito. El peso generalmente es adecuado para la edad. El edema, signo cardinal
del kwashiorkor, tpicamente inicia en el dorso de los pies y en las piernas. Despus se extiende a
otras partes del cuerpo incluyendo manos, antebrazos, espalda, extremidades superiores y en
casos severos en la cara, principalmente en las mejillas y alrededor de los ojos. Generalmente
corresponde al 5-20% del peso corporal. Puede desarrollarse ascitis (por ausencia de protenas en
la sangre) o derrame pleural como manifestaciones tardas del edema .La hipoglicemia es un
fenmeno comn en pacientes con kwashiorkor, aunque tambin se ha descrito en pacientes con
emaciacin severa. Tericamente la hipoglicemia puede estar causada por alteracin en la
produccin heptica endgena de glucosa o por aumento de la eliminacin de la misma. Esta
complicacin aporta de manera importante a la mortalidad. Para realizar el diagnstico de
desnutricin severa, la OMS define malnutricin aguda severa como una circunferencia superior
del brazo (MUAC por sus siglas en ingles) < 11.5 cm, un puntaje z de peso para la talla (P/T) < -3 o la presencia de edema bilateral pedio.
CASO CLINICO
Nio de 3 aos de muy bajo nivel socioeconmico, Cursa desde hace
dos aos con poliuria, polidipsia, astenia, y perdida de peso. Ingresa
por presentar estado de estupor y signos de deshidratacin severa.
Peso: 9 kg, Talla: 92 cm. Frecuencia respiratoria: 25 por minuto, T
corporal y PA: normal. Malas condiciones generales, marcada palidez
muco-cutnea, perdida de tejido subcutneo, secrecin purulenta
en conductos auditivos, lesiones ulcerosas en lengua, membranas
blanquecinas en cavidad oral, quelitis sangrante.
Abdomen globoso, distendido sin megalias, edema +++.
PREGUNTA
Considerando el estado de desnutricin y las complicaciones que
potencialmente desarrollar, cual de los siguientes es el mas
probable?
RESPUESTA
a.- Alcalosis metabolica
b.- Acidosis metabolica
c.- Acidosis respiratoria.
d.- Alcalosis respiratoria.
PREGUNTA
Cul es su grado de desnutricin?
RESPUESTA
a.- Grado I.
b.- Grado II.
c.- Grado III.
d.- Grado IV.
PREGUNTA
Cul es la complicacin endocrinolgica mas probable para el caso?
RESPUESTA
a.- Hipogonadismo hipogonadotrofico.
b.- Diabetes mellitus.
c.- Hipotiroidismo yodoprivativo.
d.- Cetoacidosis diabtica.
CASO CLINICO
Paciente masculino, de 3 aos de edad, es llevado a la sala de
urgencias por presentar: diarreas ftidas, vmito y fiebre. La madre
refiere que es el penltimo hijo de cinco hermanos, el ms pequeo
tienen 4 meses de edad. Refiere que el paciente naci prematuro y
que tena problemas para succionar, por lo que le puso leche infantil
desde el primer da de vida. Pesa 9 kg y talla 92cm. A partir de los 6
meses comenz una alimentacin basada en agua de arroz y otro

CURSO ENARM CMN SIGLO XXI TEL: 36246001

tipo de alimentos porque careca de los recursos econmicos para


adquirir leche, carne y otros alimentos poco proteicos. EF: Paciente
consciente, aptico e irritable, plido, eupneico, luce crnicamente
enfermo. Se observa cara con fascies abotagada, signos de lesin
ocular, pelo seco y quebradizo, pelo con manchas escamosas,
hepatomegalia y miembros inferiores edematizados.
PREGUNTA
Cul es el manejo inicial para este paciente?
RESPUESTA
a.- Hemoderivados, lquidos a 100ml/kg/da
b.- 50kcal/kg/dia, lquidos a 100ml/kg/da
c.- Lquidos a 200ml/kg/da, sol. Glucosada al 5%
d.- 200Kcal/kg/da, lquidos a 200ml/kg/da
PREGUNTA
Cul es el grado de desnutricin del paciente?
RESPUESTA
a.- Grado I
b.- Grado II
c.- Grado III
d.- Grado IV
PREGUNTA
De las siguientes manifestaciones Cul es la de mejor pronstico
para el paciente?
RESPUESTA
a.- Albumina <2g/dl
b.- Insuficiencia cardiaca
c.- Bronconeumonia grave
d.- Hepatomegalia
CASO CLINICO
Un varn de 6 meses de edad, nacido a trmino, que presenta
desnutricin severa. Naci de una madre alimenta normalmente,
tras un embarazo sin complicaciones y peso al nacer 2,680 g,).
Exclusivamente amamantado y pareca estar bien alimentado. En el
segundo mes de vida, empez a mostrar falta de crecimiento, sin
antecedentes de vmitos o diarrea. Dos semanas ms tarde,
desarrollo erupcion difusa no pruriginos. A la exploracin se oservo
pelo escaso con un aspecto plateado rubio. Se observa irritable, con
tos no productiva. Estaba alerta pero caquctico severamente. Su
peso (3.245 kg), longitud (51,5 cm), y circunferencia de la cabeza
(37,5 cm).

Pharmed Solutions Institute

PGINA 319

MANUAL DE TRABAJO DEL CURSO ENARM CMN SIGLO XXI


PREGUNTA
Cul de lo siguientes hallazgos de laboratorio es menos probable
encontrar?
RESPUESTA
a.- Anemia normoctica.
b.- Leucopenia.
c.- Hipoalbuminenia.
d.- Neutrofilia.
PREGUNTA
Cul de las siguientes dosificaciones esta contraindicada por
generar mayor deterioro respecto al estado nutricional?
RESPUESTA
a.- Vitamina A 5mg primera dosis luego 1mg/dia.
b.- Zinc 6mg /kg/dia.
c.- Cobre 0.3 mg/Kg/dia.
d.- Hierro 3mg/Kg/dia. A partir de la segunda semana.
PREGUNTA
Cual de las siguientes complicaciones se presenta ms
frecuentemente en el paciente desnutrido que por general pasa
desapersibidas?
RESPUESTA
a.- Infeccin de vas urinarias.
b.- Otitis media.
c.- Tuberculosis.
d.- Guardiasis.
CASO CLINICO
Un varn de 11 meses alimentado exclusivamente a seno materno,
vive en zona rural y es el ultimo hijo de 8 en total, madre dedicada al
hogar con educacin primaria, reingres 10 das despus de un
episodio previo de gastroenteritis con diarrea durante 2 das y
deshidratacin, Despus de la rehidratacin estaba por debajo del
3er percentil para el peso. 2 semanas despus de la admisin la
diarrea es severa. Una semana ms tarde la diarrea era con sangre,
fiebre. Egresa una semana despus de tratamiento intrahospitalario.
PREGUNTA
Cual es la conducta a seguir?
RESPUESTA
a.- Suplementar alimentacin.
b.- Integracin a la dieta familiar.
c.- Alimentacion cada dos horas suplementada.
d.- Referencia a segundo nivel.
PREGUNTA
El paciente anterior reingresa por presentar irritabilidad, llanto sin
lagrimas e incontrolable, hipotnico, hipotrmico con lienzo hmedo
y reporta hipoglucemia la cual es controlada oralmente.
Considerando la comorbilidad de la desnutricin cual es la conducta
profilctica a seguir.
RESPUESTA
a.- Administrar trimetropin sulfametoxazol.
b.- Administrar cefotaxima.
c.- Administrar ampicilina ms gentamicina.
d.- Administrar cloranfinicol IV.

Considerando el estado hdrico del paciente, los antecedentes y la


comorbilidad cual es su manejo de mantenimiento de liquidos?
RESPUESTA
a.- 100 ml/kg/dia.
b.- 130 ml/kg/dia.
c.- 160 ml/Kg/dia.
d.- 180 ml/Kg/dia.
PREGUNTA
El paciente es egresado por mejora despus de 7 dias de
tratamiento, con respecto a la recuperacin ponderal usted
suplementa micronutrientes y aporte proteico de 1 a 1.5 g/kg/dia
hasta 4 a 6 g/kg/dia. Cul es el incremento ponderal que esperara
usted con las indicaciones al egreso considerando que lo debe de
citar semanalmente?
RESPUESTA
a.- 100 g semanal.
b.- 150 g semanal.
c.- 200 g semanal.
d.- 250 g semanal.
CASO CLINICO
Se trata de paciente masculino de 7 meses de edad, el cual es
prematuro, madre muri post-cesrea. Alimentado con sustituto de
leche pero diluidos, por falta de recursos econmicos. EF: Temp
axilar 34.7, irritable y muy llorn, paciente extremadamente
delgado, con el abdomen prominente, con intensa emaciacin,
perdida de grasa subcutnea, costillas muy marcadas, cara de viejito,
la piel cuelga en pliegues, retraso del crecimiento, quien adems
cursa con infeccin de vas respiratorias.
PREGUNTA
Cul es el diagnstico ms probable para este paciente?
RESPUESTA
a.- Kwashiorkor
b.- Desnutricin moderada
c.- Marasmo
d.- Marasmo- Kwashiorkor
PREGUNTA
Cul es la conducta ms adecuada a seguir en este momento?
RESPUESTA
a.- Iniciar con 100kcal/kg/da, y lquidos 120-150ml/kg/dia
b.- Iniciar con 100kcal/kg/da, y lquidos 120-150ml/kg/dia, mas
hemoderivados
c.- Elevar temperatura con medios fsicos, Iniciar con 100kcal/kg/da,
y lquidos 120-150ml/kg/dia, descartar hipoglicemia
d.- Elevar la temperatura con medios fsicos, inicar antibitico,
hemoderivados
PREGUNTA
Cul es la prueba de tamizaje rpido ms adecuada para identificar
a los nios en riesgo de desnutricin energtico-proteica?
RESPUESTA
a.- IMC
b.- Puntaje Z
c.- Banda de colores Shakirs
d.- Determinacin de albumina

PREGUNTA

CURSO ENARM CMN SIGLO XXI TEL: 36246001

Pharmed Solutions Institute

PGINA 320

MANUAL DE TRABAJO DEL CURSO ENARM CMN SIGLO XXI


AVITAMINOSIS
CIENCIAS BASICAS: Dficit de vitaminas, ya sea por falta de ingesta, absorcin inadecuada a nivel intestinal, su mala utilizacin metablica o el
aumento de la demanda. Como las vitaminas participan en las reacciones como coenzimas, diversos procesos metablicos podran verse afectados.
Las vitaminas son nutrientes esenciales, orgnicos, de bajo peso molecular, imprescindibles para el crecimiento, desarrollo y mantenimiento del
organismo, y que el hombre necesita adquirir a travs de los alimentos. Aunque los alimentos de origen animal pueden ser fuentes inmediatas de
vitaminas, las plantas son sus principales suministradores, a causa de su mayor capacidad de sntesis de los precursores metablicos de las
vitaminas. Los requerimientos de vitaminas dependen de la composicin de nutrientes de la dieta y de las diferentes condiciones de vida del
individuo. Asimismo, deben adecuarse a las poblaciones en particular, a sus caractersticas fsicas, gastos de energa, fuentes alimentarias, hbitos,
actitudes alimentarias y a las deficiencias nutricionales detectadas. Las vitaminas son absorbidas, en general, de 20 a 95 %; as por ejemplo, la
vitamina A se absorbe de un 70 a 90 %, la vitamina D entre 80 y 90 %, la vitamina C entre 80 y 95 % y la vitamina B6 entre 95 y 98 %, mientras que la
vitamina E solo se absorbe entre 20 y 40 % y los carotenos entre 20 y 50 %.
VITAMINAS LIPOSOLUBLES

VITAMINA
A

K
Filoquinona

DEFICIENCIA
XEROFTALMIA (caracterizada por una sequedad del globo ocular y una falta de brillo en la
superficie ocular, se acompaa de una leve prdida de visin, especialmente por la noche).
Afecta a clulas y rganos de todo el cuerpo; los cambios resultantes de la arquitectura
epitelial reciben la denominacin de "metaplasia queratinizante", la cual se produce con
relativa rapidez en las vas respiratorias y urinarias. Fotofobia, xeroftalmia. En los pases en
desarrollo la carencia de este nutriente tiene carcter grave como para provocar daos
oculares permanentes en unos 13 000 000 de nios y ceguera a 5 000 000, causante de 25 %
de las muertes en la poblacin infantil;
Estimula la absorcin
RAQUITISMO: Caracterizada por reblandecimiento de los huesos del crneo, asociado
intestinal de Ca.
generalmente a un agradamiento exagerado de este, el trax se aplasta trasversalmente, el
Condiciona depsito
esternn es prominente y las articulaciones de la rodilla se abultan; en las extremidades
de Ca y P en los
inferiores, por la accin del peso corporal, aparecen incurvaciones de los huesos largos y
huesos
desviaciones del eje longitudinal de los mismos. Esta deformacin se manifiesta cuando los
nios empiezan a andar; es frecuente la aparicin de fracturas de hueso y la movilidad de las
articulaciones es muy amplia debida a la laxitud de los ligamentos. Son frecuentes las
lesiones musculares debido a la debilidad de los msculos. Otras manifestaciones del
raquitismo son: abultamiento del vientre, retraso en la denticin, implantacin defectuosa
de los dientes, disminucin de la talla en relacin con la normal. Craneo tabes, rosario
raqutico, torax en paloma, caput, piernas patisambas. TRATAMIENTO: Se basa en la
administracin diaria de vitamina D por va oral. La dosis ser de 2 500 a 5 000 U (62 a 125
mg), y puede llegarse a 10 000 U/da (250 mg), lo que corresponde a una cantidad de 5 a 20
gotas de preparado que contenga 10 000 U/mL, durante 6 u 8 semanas.
Impide la autooxidacin de los cidos
Puede ocasionar anemia hemoltica (destruccin de los glbulos rojos de la
grasos insaturados. Impide deterioro
sangre), degeneracin muscular y desrdenes en la reproduccin.
de las membranas celulares
Intervienen en la sntesis de
Pueden producirse hemorragias nasales, en el aparato digestivo o el genitoprotrombina (coagulacin)
urinario.

C
Ac.
Ascrbico

Antioxidante,
importante para la
sntesis de colgeno

B1
Tiamina

Coenzima
que
interviene en las
reacciones
de
transferencia
de
grupos aldehdo de
dos carbonos
Constituyente de las coenzimas FMN
y FAD que intervienen en el
metabolismo
energtico
como
transportadores de H+ y electrones
Coenzima que intervienen en las
desaminaciones. Relacionada con el
metabolismo de las protenas
Forma parte de las coenzimas NAD y
NADP que interfiere

E
Tocoferol

B2
Riboflavina

B6
Piridoxina
Niacina
Ac.nicotinico
Ac. Flico
B12
Cobalamina

FUNCION
Intervienen en la
percepcin
visual.
Necesaria para el
mantenimiento de los
tejidos epiteliales

VITAMINAS HIDROSOUBLES
ESCORBUTO; Se caracteriza por una debilidad muscular progresiva, dolores en los brazos y
en las piernas, adelgazamiento progresivo, palpitaciones, aceleracin de pulsaciones
cardiacas y sensacin de ahogo; son frecuentes las hemorragias especialmente en las encas,
que estn hinchadas y sangran con facilidad y en la piel aparecen manchas sanguneas; los
dientes caen precozmente; es constante la anemia y diversas alteraciones seas que se
ponen en evidencia radiolgicamente. La resistencia a las infecciones est muy disminuida.
Rosario escorbutico
BERIBERI: Se caracteriza por trastornos nerviosos, circulatorios y generales (astenia,
debilidad general, marcha tambaleante, anemia). Si el dficit es severo puede aparecer el
sndrome de Korsakoff, caracterizado por la prdida de memoria y confusin o la
encefalopata de Wernicke, trastornos oculares, confusin. Muerte por falla cardiaca

Coenzima que interviene en el


metabolismo de los cidos nucleicos
Forma parte de una
coenzima
necesaria en el metabolismo de las
protenas y de cidos nucleicos

FUENTES DIETETICAS
Vegetales
verdes,
legumbres frescas

Leche,
mantequilla

huevos,

Vegetales
tomates,
vegetales

verdes,
aceites

Frutos
ctricos,
verduras frescas

Levaduras, germen y
salvado
de
arroz,
hgado, carnes

Lesiones en la piel boca y ojos, fotofobia, dermatitis seborreica, visin


borrosa, quilosis

Mismas que B1, leche,


queso, huevos

El dficit importante provoca irritabilidad, debilidad, mareos, depresin,


neuropata perifrica y espasmos, alteraciones del crecimiento, acrodinia y
anemia.
PELAGRA: Las principales manifestaciones son dermatitis, inflamacin y dolor
en la boca y en la lengua, diarrea, marcha tambaleante, anemia. Dermatitis,
diarrea, demencia
Anemia

Levaduras,
verduras
frescas, leche, carne,
huevos
Trigo integral, levadura
de cerveza, verduras,
hgado

Anemia perniciosa. Megaloblastica, neuritis

Carne, leche. Huevo,


pescado

TRATAMIENTO: El tratamiento se basa en la identificacin de la vitamina o vitaminas deficitarias, para luego proceder a su reposicin mediante
suplementos vitamnicos y raramente, en sndromes de mala absorcin, mediante cargas inyectables.
CASO CLINICO

CURSO ENARM CMN SIGLO XXI TEL: 36246001

Se trata de un paciente masculino de 1 ao 3 meses al ingreso. Los


antecedentes heredo familiares eran negativos. Como antecedentes

Pharmed Solutions Institute

PGINA 321

MANUAL DE TRABAJO DEL CURSO ENARM CMN SIGLO XXI


perinatales se document: Madre de 17 aos. Se trataba de
suprimera gesta por cesrea por parto distsico con lquido
amnitico mecanizado. Se clasific al recin nacido de trmino
adecuado para la edad gestacional. Su peso al nacer fue de 2720 g.
Permetro Ceflico 33 cm y APGAR 9-9. Sus inmunizaciones estaban
completas. Segn la historia de la madre se document un retardo
de desarrollo psicomotor. Los mdicos tratantes consideraron que
su estado de desnutricin poda ser una causa contribuyente. La
alimentacin se describi como deficiente, con mucha leche o
atoles. Descrpcin del estado socio-econmico: Familia inmigrante,
matrimonio joven, con baja escolaridad y problemas econmicos
importantes. La madre ama de casa y padre recolectar de caf sin
trabajo fijo.) Cambios frecuentes de vivienda. Motivo de ingreso:
Enfermedad diarreica aguda de 8 das de evolucin, con 3 o 4
deposiciones por da, verde-amarillenta, sin moco, sin sangre.
Vmitos desde 5 das prevos al ingreso de contenido alimentario. Al
ingreso se plante como problema falla para progresar con un peso
de 8.3 kilogramos, talla de 73 centmetros y retardo del desarrollo
psicomotor. Se describieron datos cnicos de deshidratacin
moderada. Se describe al paciente en este ingreso irritable, aptico,
hipoactivo, con lesiones de piel se describen como
hiperpigmentadas en todo el cuerpo, mltiples, con distribucin de
predominio en cara, en forma de alas de mariposa y en las manos
con distribucin de "guantes", las cuales se desprendan fcilmente
con el roce, hiperqueratosis, resequedad en la piel, fisuras
generalizadas de predominio en labios y en codos, glositis,
hiperplasia gingival, flictenas en brazo izquierdo, bajo reborde costal
y edema en mano y piel. Se describe tambin hgado a 4 centmetros
bajo el reborde costal.
PREGUNTA
Considerando la sintomatologa. Cul es el dficit vitamnico mas
probable?
RESPUESTA
a.- Vitamina A.
b.- Complejo B.
c.- Vitamina C.
d.- Vitamina D.
CASO CLINICO
Femenino de 6 aos de edad, que acude por resfriado comn y a la
exploracin se observa dificultad en la marcha, con sensacin
dolorosa en el lado derecho de la cadera y refiere la madre que se ha
cado espontneamente en alguna ocasin, se encuentra sin
vacunacin, desarrollo psicomotor dentro de lmites.
PREGUNTA
Cual es su deficiencia vitamnica mas probable en este caso?
RESPUESTA
a.- Vitamina A.
b.- Complejo B.
c.- Vitamina C.
d.- Vitamina D.
CASO CLINICO
Femenino de 6 aos de edad, fue hospitalizada por cierta dificultad
en la marcha que fue notada desde aproximadamente uno y medio
mes antes, conjuntamente con sensacin dolorosa en el lado
derecho de la cadera y cadas ocasionales. Producto de un embarazo
complicado con toxemia y parto a trmino, pesando 3,225Kg al
nacer. Haba padecido de sarampin, tos ferina, infecciones de vas
areas repsiratorias superiores ocasionales, as como de diarreas
espordicas. No haba sido vacunada y su desarrollo psicomotor
habla estado dentro de los lmites normales. Seis hermanos, dos
hembras y cuatro varones, uno de los cuales fue hospitalizado
conjuntamente que ella, por desnutricin con edemas. Al examen
fsico encontramos una nia con desarrollo y nutricin pobre, plida,
con expresin triste e indiferente. 3,225. (menos del 3er. percentil) y

CURSO ENARM CMN SIGLO XXI TEL: 36246001

talla 104cm (menos del 3er. percentil). Cabeza normocfala. Trax


en paloma, ruidos cardiacos rtmicos. Abdomen blando, sin palparse
rganos ni masas. Genitales normales. Extremidades inferiores con
genu valgun e hiperextensin en rodillas. Sistema nervioso sin
alteraciones. Piel amarillenta.
PREGUNTA
Cul es el diagnstico ms probable en este caso?
RESPUESTA
a.- Raquitismo
b.- Escorbuto
c.- Hipercalcemia
d.- Pelagra
PREGUNTA
Cules alteraciones seria menos probable encontrar en este caso?
RESPUESTA
a.- Piernas patizambas, enanismo
b.- Dermatitis, diarrea, demencia
c.- Surco de Harrison, rosario raqutico
d.- Debilidad de tobillo, fracturas asintomticas
CASO CLINICO
Lactante menor masculino de 2 meses y 4 das de vida, que acudi a
consulta de emergencia por un cuadro clnico de 21 das de
evolucin caracterizado por deposiciones liquidas amarillo verdosas
con mucosidad sin sangre en numero de mas o menos 5 al da y un
da antes a su ingreso increment la frecuencia de las deposiciones y
se agregan estras de sangre; por este motivo acudi a medico
general quien prescribe: ampicilina, paracetamol, propinox
clorhidrato y maleato de clorfeniramina y por la noche, se torna
irritable, con llanto intenso y continuo, presenta rigidez de cuello y
dorso y epistaxis. Ante la persistencia de llanto e irritabilidad acude
al servicio de emergencia del centro de pediatra Albina R. de Patino,
donde se hospitaliza. Antecedentes: embarazo sin complicaciones,
parto cesrea por distocia de presentacin a las 38 semanas de
gestacin, recibi vitamina K al nacer, peso nacimiento: 3.200 g y
talla de nacimiento: 50 cm. Alimentacin: lactancia materna
exclusiva. Al examen de ingreso destac: irritabilidad, fiebre (38,8C
rectal), palidez mucocutnea, respiracin de Kusmaul, signos de
deshidratacin moderada, fontanela abombada y tensa, cuello con
rigidez de nuca, signo de Kernig positivo y Brudzinsky negativo.
PREGUNTA
Cul es el diagnostico mas probable en este caso?
RESPUESTA
a.- Meningitis
b.- Sepsis
c.- Deshidratacion severa
d.- Acidosis metabolica
PREGUNTA
Laboratorio: Hb: 6,53 g/dl, glbulos blancos: 24.700 por mm3 con
46% de segmentados, plaquetas 619.000 mm3, protena C reactiva
4.8 mg/dl, LCR con protenas 502,3 mg/dl, glucosa 46,7 mg/dl,
leucocitos 5 por mm3 y eritrocitos 13.2000 por mm3. Uroanlisis con
hema-turia microscpica. TP y TPTa prolongados. Resto de los
exmenes dentro de parmetros normales. En las horas siguientes a
su ingreso, se evidenci un sangrado profuso por los lugares de
venopuncin y present signos de choque hipovolmico.
PREGUNTA
Cul es el diagnotico ms probable en este caso?
RESPUESTA
a.- Coagulacion intravascular diseminada
b.- Enfermedad hemorrgica de RN por deficiencia de Vitamina K
c.- Leucemia linfoblastica aguda
d.- Infeccion viral

Pharmed Solutions Institute

PGINA 322

MANUAL DE TRABAJO DEL CURSO ENARM CMN SIGLO XXI


CONTROL DE NIO SANO
CIENCIAS BASICAS: Crecimiento: aumento de las dimensiones del cuerpo debido a la multiplicacin e incremento de tamao individual de las
clulas, hipertrofia, hiperplasia, incremento en la sustancia intercelular. Leyes del crecimiento: Ley de la velocidad, de la direccin (cefalo-caudal),
del ritmo, de la secuencia, de la continuidad. Periodos del crecimiento: recin nacido (primeros 28
dias), lactante menor (28 dias a 12 meses), lactante mayor (12-24 meses), preescolar (2-6 aos),
escolar (6-10 aos nias, 6-12 aos nios), adolescente (10-16 aos nias, 12-18 aos nios).
Desarrollo: proceso dinamico de adquisiscion de funciones en un tiempo determinado, diferenciacin
de clulas y tejidos yla complejidad creciente de la estructura, tanto organica como funcional, con la
adquisicin de nuevas capacidades a travs de un proceso de maduracin, abarca funciones,
adaptaciones, destrezas psicomotoras, relaciones afectivas y socializacin. CONSULTAS; <28dias se
dan 2 consultas a los 7 y 28 dias. De 1-12meses de vida una consulta por mes. De 1-3 aos se dan 4
consultas, cada 3 meses. De 3-5 aos se dan 2 consultas, una cada 6 meses. En mayores de 6 aos es
una consulta por ao. En cada consulta se evalua peso, talla, PC, IMC (EVALUACIN DE
CRECIMIENTO), TA, P/E, T/E, P/T, IMC/E, alimentacin, neurodesarrollo, habitos, esquema de
vacunacin, actividad fsica. Hay que hacer deteccin oportuna de trastornos de aprendizaje, lenguaje, comunicacin, desarrollo emocional. PESO:
RN 3,000Kg, 1-4 meses 750gr/mes, 4-8 meses 500gr/mes, 8meses-2aos 250gr/mes. TALLA: RN 50cm, 1mes-1ao 75cm (2cm/mes), 1-2aos 82cm
(0.8cm/mes), 2-12 aos (edad x 6.5) +70, 4 aos 1 metro, 12 aos 1.50 metros. PERIMETRO CEFALICO: RN 35cm, 3 meses 40cm, 6 meses 44cm, 1
ao 46cm, 3 aos 50cm, hasta los 2-3 aos de edad. IMC: es el parmetro que mejor define la obesidad en el nio y adolscentes, despuesde los 2
aos hacer IMC= peso en Kg/talla2 y percentilar con las tablas, sobrepeso >percentil 75, obesidad >percentil 85, obesidad grave >percentil 97, en los
adultos el IMC tiene limitaciones. La presin arterial en los nios se debe examinar en forma anual a partir de lo 3 aos y en los siguientes casoso
especiales: crisis convulsivas, infecciones de vas urinarias, edema, hematuria, sospecha de nefropata o cardiopata. EVALUACION DEL
DESARROLLO: en la primera consulta se identifican factores de riesgo, desarrollo psicomotor, movimiento grueso, fino y adaptable, lenguaje,
desarrollo personal y social, se puede hacer uso de la escala de Denver es una prueba
diseada para evaluar el desarrollo en menors de 6 aos, una prueba anormal= 2 o mas
reas, con 2 puntos no aprobados o 1 area, con 2 puntos no aprobados. Normal=todas las
reas aprobadas o 1 area con 1 punto no aprobado. En menores de 5 aos por norma
oficial mexicana hay que valorar; lenguaje, social, coordinacin, motora. Normal= ejecuta
conductas correspondientes a edad cronolgica, limtrofe=ejecuta conductas a edad
inmediata anterior, anormal=no ejecuta conductas correspondientes a su edad, ni a la
inmediata anterior. Valorar reflejos de desarrollo: ver en cuadro anexo.Desarrollo
psocomotor normal: 1 mes; no sostiene la cabeza, manos empuadas, fija la mirada en
objetos de 30 a 40 cm, se sobresalta con el sonido, llanto consolable, reconoce la voz de la
madre. 2 meses; levanta cabeza en posicin prona, sostn parcial de cabeza, manos abiertas, sigue objetos a 180, sonrisa social y balbuceos. 4
meses; sostn ceflico, inica rodamiento, control mano-ojo-boca, agarra objetos, se arrulla y vocaliza, identifica a la madre. 8 meses; se sienta solo,
mete las manos para no caerse, inica gateo, pasa objetos de una mano a otra, dice monoslabos (ma-pa), llora y hay angustia por la separacin. 10
meses; gateo alterno, inica bipedestacin, seala con el ndice, presenta pinza fina, imita onomatopeyas, busca objetos que se le caen, se lleva
objetos a la boca. 12 meses; inica marcha con ayuda, toma y da objetos, emite 3-5 palabras, ayuda a vestirse, empieza a usar la cuchara. 18 meses;
subes escaleras sin alternar u sube a una silla, empieza a omer solo, hace trpode, forma frases, seala las partes del cuerpo, ayuda a desvestirse,
patea pelota, usa el tenedor y maneja bien la cuchara. 24 meses; sube y baja escaleras con alternancia, garabatea, imita lneas, forma oraciones
cortas, ejecuta indicaciones sencillas, participa en juegos de grupo y juegos simblicos. Maduracion sexual; precoz antes de los 9 aos, retardada
despus de los 14 aos. Hay que hacer valoracin de la audicin; con reflejo oculo-giro y cocleo-palpebral, valoracin visual, valoracin ortopdica,
valoracin dental. ALIMENTACION: Lactancia materna, minimo 6 meses, la OMS recomienda hasta 2 aos de edad, los beneficios que aporta son;
inmunolgico, idoneidad de nutrimentos, maduracin de SNC, SGI, disminuye riesgo de infecciones, disminuye riesgo de cncer en la madre, apego,
econmico, disponibilidad, control natal. Biberones; 0-3 meses de 6-8 al dia, 3-6 meses 5, 6-9 meses 4, de 9-12 meses 3, de 12-24 meses 2, no
exceder 750ml de leche en 24 hrs. Ablactacion; apartir de los 4-6 meses, se sugiere 3 alimentos y 2 colaciones. A los 6 meses verduras, 7 meses
frutas, 8 meses cereales, 9 meses carnes blancas, leguminosas, carnes rojas, 10 meses, pescado blanco, yema de huevo, quesos, 11 meses
embutidos, 12 meses incorporacin a dieta familiar, kiwi, huevo entero, chocolate, ctricos, 24 meses frutas secas. Leyes de la alimentacin; ley de la
suficiencia, de la totalidad, del equilibrio, de la idoneidad, variabilidad, pureza. Requerimientos diarios de liquidos; RN 65-70ml/kg, 2-3 semnas 120150ml/kg, lactante de menos de 10Kg 150-180 ml/kg, nio mayor de 10kg 2500ml/m2SC. Requerimientos calricos; los primeros 10 Kg 100Kcal/kg,
los siguientes 10kg 50Kcal/kg, despus de los 20 Kg 20kCal/kg. CRITERIOS DE REFERENCIA A SEGUNDO NIVEL: desnutricin grave, obesidad,
sobrepeso, talla baja, retraso psicomotor, permetro ceflico alterado.
CASO CLINICO
Milagros es una nia de 10 meses, la cual es llevada por la madre al
consultorio de pediatra para su control mensual. En la consulta se
obtienen los siguientes datos: peso actual: 6,750 Kg, Talla: 72,5
cm, Per. Ceflico: 46,7 cm, PB: 150 mm, Pl. tricip: 10 mm
PREGUNTA
Cul sera el peso ideal para este lactante menor?
RESPUESTA
a.- 6,000Kg
b.- 8,000Kg
c.- 10,00Kg
d.- 12,000Kg
PREGUNTA

CURSO ENARM CMN SIGLO XXI TEL: 36246001

De acuerdo a la edad del paciente. Qu es menos probable


observar del desarrollo psicomotor?
RESPUESTA
a.- Se sienta solo, inicia gateo
b.- Dice monoslabas
c.- presenta pinza fina
d.- Llora y hay angustia por la separacin
PREGUNTA
De acuerdo a la edad del paciente Cul es el alimento menos
adecuado en su dieta?
RESPUESTA
a.- Verduras
b.- Frutas
c.- Carnes blancas
d.- Cereales

Pharmed Solutions Institute

PGINA 323

MANUAL DE TRABAJO DEL CURSO ENARM CMN SIGLO XXI


SOBREPESO, OBESIDAD
CIENCIAS BASICAS: Obesidad: Enfermedad sistemica crnica recurrente y compleja, multifactorial, caracterizada por acumulacin anormal u exceso
de grasa, es prevenible, genera muchos riesgos para la salud, su asociacin con enfermedades que ocupan los primeros lugares dentro de los
cuadros de morbi-mortalidad de la poblacin, como son la cardiopata isqumica, la hipertensin arterial, la DM tipo 2, las enfermedades
cerebrovasculares y algunos tipos de cncer. El sobrepeso que acompaa a la adiposidad excesiva puede producir diversos trastornos como son los
problemas ortopdicos y la reduccin de la capacidad de trabajo fsico. Todo esto, unido a consideraciones estticas, produce, adems, trastornos
psicolgicos que se derivan no solo de la distorsin de la autoestimacin, sino de las relaciones sociales del individuo. SALUD PUBLICA: Prevalencia
en nios 5 a 11 aos 46%, en 2010 43 millones de nios menores de 5 aos tenan sobrepeso. Primer lugar obesidad infantil mundial. En los
adolescentes, la obesidad constituye la causa principal de hipertensin mantenida y se ha sealado que aproximadamente el 25 % de los
adolescentes hipertensos obesos pueden sufrir complicaciones. Fatores de riesgo para obesidad: padres con obesidad (riesgo para los hijos hasta de
80%), nios macrosomicos, pequeos para edad gestacional, ambientes obesogenicos, alimentos disponibles, altamente calricos, y bajo gasto de
energa, patrones de alimentacin con dietas de alto valor calrico, sedentarism, madre con jornada de trabajo de tiempo completo, exceso de
tecnologa en casa, antecedentes de enfermedades metablicas, acantosis nigricans, TA>90, IMC >a percentil 97, circunferencia cintura >90cm.
CLASIFICACION: Obesidad; GRADO I: IMC: 30-34.9 KG/m2, GRADO II: IMC 35- 39.9 KG/m2, GRADO III: 40 KG/m2. Sobrepeso; SOBREPESO I: IMC 2526.9 KG/m2, SOBREPESO II: IMC 27-29.9 KG/m2. PATOGENIA: El depsito de grasa en exceso es multifactorial. Obesidad como una forma de mala
nutricin por exceso, bsicamente cuantitativa (con un balance energtico positivo). Desbalance energtico, entre aporte y utilizacin de grasas, de
carcter neuroendocrino, metabolico, gentico, ambiental y psicgeno. En el obeso hay una mayor eficiencia trmica, es menor la cantidad de
energa perdida en forma de calor y mayor la acumulada en las reservas en forma de grasa. Tambin suele ser menor la energa utilizada en actividad
fsica. La dieta es determinante en la regulacin energtica y de hecho constituye el principal factor desencadenante del desequilibrio entre el
ingreso y el gasto de energa. Las prcticas de alimentacin del lactante han sido con frecuencia relacionadas con el desarrollo de la obesidad: la
prevalencia de obesidad parece ser mayor en nios que recibieron lactancia artificial y en aquellos con destete temprano o en los que se
introdujeron slidos antes del tercer mes. DIAGNOSTICO: En la etapa de lactante el incremento fisiolgico de la grasa corporal puede ser excesivo,
por lo que se produce un aumento en el tamao de las clulas adiposas, pero la probabilidad de que la hiperadiposidad persista hasta la adultez es
muy baja. Sin embargo, cuando esta aparece entre los 5 y l0 aos de edad se observa un incremento gradual en la adiposidad y es frecuente que
persista a lo largo de la adolescencia y se mantenga en la adultez, caracterizndose por el incremento del nmero de adipocitos. En aquellos sujetos
en los cuales la grasa se acumula en el tronco (obesidad superior o androide) se ha observado una mayor asociacin con la presencia de
enfermedades crnicas degenerativas, lo cual tambin se observa cuando se produce un incremento de la grasa visceral (obesidad central). Cuando
la grasa tiende a acumularse en las extremidades, particularmente en el tren inferior (inferior o ginecoide), la probabilidad de asociacin de la
obesidad con dichas afecciones es mucho menor. En la piel del abdomen de sujetos obesos moderados o severos suelen verse estras atrficas.
Igualmente pueden observarse lesiones de Acantosis nigricans, localizados sobre todo en la piel del cuello y las extremidades. En el sistema
osteomioarticular pueden verse genu valgo, coxa vara y deslizamientos epifisarios de la cabeza del fmur. En los obesos son frecuentes los
trastornos psicolgicos, en ocasiones como elementos causales y en otras, como resultado de la distorsin que sufre el sujeto de su imagen y de los
conflictos que determina su condicin dentro de su grupo. En una proporcin de escolares y adolescentes es posible encontrar hipertensin arterial,
por lo general, ligera a moderada, hipercolesterolemia, hipertrigliceridemia y elevacin de LDL y VLDL, as como cada de las HDL. Tambin puede
observarse intolerancia a la glucosa e incluso hiperglicemia, manifestaciones que suelen responder favorablemente al tratamiento de la obesidad.
Antropometria: estatura, peso, IMC, signos vitales, circunferencia cintura-cadera (>90cm hombres, >80cm mujeres), se mide en el punto mdio entre
el borde inferior de la ultima costilla y el borde superior de la cresta iliaca, con nio de pie, en espiracin profunda. En nios debemos tomar en
cuenta la PUNTUACION Z; que es un valor antropomtrico, si es puntaje z >2desviaciones estndar es sobrepeso, obesidad es puntaje z>3
desviaciones estndar. INDICE DE MASA CORPORAL o ndice de Quetelet, segn la expresin; IMC= peso (kg)/talla m2): entre percentil 85-95 es
indicativo de sobrepeso en nios, encima de percentil 95 es obesidad en nios. IMC= >30 kg/m2 = obesidad adultos. Pero, al no tomar en cuenta la
talla del sujeto, puede incluir como obesos a individuos normales de talla elevada o excluir a individuos de baja talla. El mtodo ms difundido ha
sido la determinacin del peso corporal y su relacin porcentual con el peso ideal o esperada para la talla del sujeto. Otro procedimiento de utilidad
prctica es la medicin de los espesores de los pliegues cutneos en diversas zonas del cuerpo con un comps destinado al efecto; las ms utilizadas
son los pliegues tricipital, bicipital, subescapular y suprailaca. Estos pliegues brindan informacin sobre la cantidad de grasa subcutnea, la que
expresa indirectamente el estado de las reservas energticas del organismo. SINDROME DE CARPENTER Obesidad, retraso mental, puente nasal
plano, sindctilia, craneosinostosis, hipogonadismo. SINDROME DE PRADER WILLI Hipotona neonatal, baja estatura, hipogonadismo, hiperfagia
compulsiva, retraso mental, obesidad, cromosoma 15. TRATAMIENTO: El objetivo, es lograr un cambio en la composicin corporal con reduccin
del peso en grasa y mantenerla despus dentro del rango adecuado para su edad y sexo. El tratamiento se sustenta en 3 pilares fundamentales: La
dieta, el ejercicio, la induccin de cambios de conducta, farmacolgico: limitado en nios. Las dietas muy bajas en caloras solo deben hacer especial
manejo en obesidad y nunca ser la terapia indicada en nios. Reducir azcares, agua, frutas y cereales granos enteros, leche descremada, no
comidas rpidas. Educar a las personas para lograr que ellas distribuyan el consumo de energa diario de forma tal que el 20 % se consuma en el
desayuno, el 20 % entre comidas y el 30 % en almuerzo y comida, respectivamente. Solo al constituirse ese hbito con carcter familiar podr
ejercerse influencia sobre los miembros ms jvenes de la familia. Tratamiento farmacolgico metformina: obesos adolescentes no diabticos pero
con comorbilidad, hiperinsulinemia o resistencia a la insulina. Efectos secundarios: nuseas, vmito Ovarios poliqusticos. Medicamentos
aprobados: fentermina, anfepramona a corto plazo y alrgo plazo orlistat. Tratamiento quirrgico: Ciruga gstrica y ciruga baips: reduce la
comorbilidad. ltima opcin la ciruga bariatrica. PREVENCION: Promover la lactancia materna extendindola por el mayor tiempo posible, y con
carcter exclusivo hasta los 5 meses. No introducir ningn otro alimento en la dieta del lactante hasta esa fecha. Cuando se inicie la ablactacin, no
acostumbrar al beb al consumo excesivo de alimentos dulces. COMPLICACIONES: Hipertension arterial, dislipidemias, hiperinsulinemia, alteracion
del VI, asociacin con leucemias, cncer de mama, vesicular, pncreas, prstata, colon, esfago. En nios obesos; DM2, desplazamiento epifisiario,
esteatosis heptica, apnea obstructiva del sueo, ovarios poliquisticos.
CASO CLINICO OBESIDAD
Nia de 10 aos de edad, acompaada por su madre para una visita
de nio sano. El peso del paciente, la altura y los signos vitales,
fueron 42,1 kg, su altura es de 1,40 m, y su presin arterial es de
98/50 mmHg. IMC 21,6 kg/m2, que est en el percentil 91o para la
edad y el gnero.

CURSO ENARM CMN SIGLO XXI TEL: 36246001

PREGUNTA
Cul es el factores de riesgo ms importante que buscara para
establecer un diagnostico adecuado?
RESPUESTA
a.- Familiares con sobrepeso y obesidad.
b.- Interrogar sobre tiempo de ver televisin.
c.- Actividad fsica.

Pharmed Solutions Institute

PGINA 324

MANUAL DE TRABAJO DEL CURSO ENARM CMN SIGLO XXI


d.- Peso al nacimiento.
PREGUNTA
Cul es la comorbilidad ms frecuente que el paciente desarrollar
en caso de persistir el sobrepeso?
RESPUESTA
a.- Diabetes mellitus.
b.- Esteatosis heptica.
c.- Baja autoestima.
d.- Desplazamiento epifisiario.
PREGUNTA
Cul es el exmen no bsico para pesquizaje intensional de
complicaciones?
RESPUESTA
a.- Perfil lipidico.
b.- Valores antropomtricos.
c.- Glucosa en ayunas e insulina.
d.- USG por poliquistosis ovrica.
PREGUNTA
Considerando el cuadro clnico. Cul es la medida mas adecuada?
RESPUESTA
a.- Sibutramina.
b.- Orlistat.
c.- Metfomina
d.- Dieta y estilo de vida.
PREGUNTA
Cual no es un criterio de referencia para segundo nivel inmediato
para el tratamiento?
RESPUESTA
a.- Sobrepeso u obesidad con retrazo mental.
b.- Lesion hipotalmica por cualquier causa.
c.- Falla en el tratamiento instaurado.
d.- Falla en el crecimiento y desarrollo.
CASO CLINICO OBESIDAD
Un nio nacido de padres no relacionados, sanos despus de un
embarazo a trmino y el parto normal (peso al nacer 3200 g,
longitud 50 cm, 34,5 cm OFC). Fue hipotnica en los primeros meses
de vida, pero no se observaron problemas de alimentacin en un
principio. Excesivo aumento de peso con aumento del apetito y el
comportamiento de bsqueda de alimento se observ a los 30
meses de edad. A los 5 aos, su peso era de 26 kg (> 3 SD), longitud
112 cm (1 SD) y OFC 48 cm (-2DE) a los 9 aos y su peso es de 72 kg
(> + 6 SD), longitud 144 cm (+2 SD) y OFC 52,5 cm (-0,5 SD).
Distintivos rasgos faciales incluyen braquicefalia, cara plana, la
frente alta, hipertelorismo, nariz antevertidas, labio superior
delgado, prognatismo, manos cortas, sindactilia de los dedos 2-3, y
genitales anormales (criptorquidia e hipospadias).

gomitas de azcar La joven madre y su hijo son los primeros en


entrar al consultorio. Ella dice estar interesada en iniciar un control
de nio sano, ya que la maestra la haba mandado a llamar para
platicar con ella acerca de la apata que mostraba el infante ante el
desarrollo de actividad fsica y por el alto consumo de carbohidratos
simples y complejos que tena durante el recreo. En la exploracin
fsica se observ que el muchachito presentaba un aspecto de
tipologa claramente pcnica, con la presencia de reas
hiperpigmentadas de aspecto aterciopelado en los pliegues del
cuello y ambas axilas. Asimismo, los panculos adiposos abdominal,
mamarios y suprailiacos resultaron ser voluminosos cuando se
evaluaron mediante la palpacin. Al ubicar el peso del escolar en la
tabla de percentiles de peso para la edad, el valor cay por encima
del percentil 95.
PREGUNTA
Cul es la conducnta teraputica mas adecuada a seguir?
RESPUESTA
a.- Dieta y cambio en el estilo de vida
b.- Metformina
c.- Programar para colocacin de banda 325strica
d.- Cirugia bariatrica
PREGUNTA
Cul es la complicacin menos probable en este paciente?
RESPUESTA
a.- Esteatosis 325eptica
b.- Apnea obstructiva del sueo
c.- Hiperinsulinemia
d.- Leucemia
CASO CLINICO
Nia de 14 aos de edad que fue referida a nuestro servicio por
presentar polidipsia, poliuria y sobrepeso. La paciente tena el
diagnstico de sobrepeso desde los 4 aos de edad y dislipidemia
desde los 8 aos. Inicialmente, recibi tratamiento con dieta y
ejercicio de manera irregular. A los 11 aos fue diagnosticada de
diabetes por vulvovaginitis, poliuria, polidipsia y glicemia en ayunas
de 282 mg/dl. La dieta y ejercicio continuaban siendo irregulares.
Adems, tena el antecedente familiar de madre, padre y abuela
materna con DM2. El examen fsico mostr obesidad abdominal
(IMC 33.6) y acantosis nigricans severa a nivel del cuello y otros
pliegues cutneos. Su desarrollo segn la escala de Tanner era de 4.
Los exmenes auxiliares en ayunas mostraron: glicemia de 269
mg/dl, insulina de 29 mIU/mL (VN: 6-27), pptido C de 4,6 ng/dl (VN:
1,1-5,0), hemoglobina glicosilada A1c (HbA1c) de 9,2% (VN: <6,5),
colesterol total de 154 mg/dl (VN: <170), HDL colesterol de 38 mg/dl
(VN: > 34), LDL colesterol de 94 mg/dl (VN: < 100) y triglicridos de
109 mg/dl (VN: < 200). Los autoanticuerpos anti-GAD y anti-ICA
fueron negativos.
PREGUNTA
Cul es la conducnta teraputica mas adecuada a seguir en este
momento?
RESPUESTA
a.- Dieta y ejercicio
b.- Iniciar metformina
c.- Iniciar insulina
d.- Iniciar Pioglitazona

PREGUNTA
Cul es la conducta a seguir?
RESPUESTA
a.- Tratamiento farmacolgico.
b.- Enviar a segundo nivel.
c.- Enviar a tercer nivel.
d.- Tratamiento quirrgico.
CASO CLINICO
Escolar de 9 aos de edad acompaado por su madre. De primera
impresin, parece tener sobrepeso y se encuentra comiendo

CURSO ENARM CMN SIGLO XXI TEL: 36246001

Pharmed Solutions Institute

PGINA 325

MANUAL DE TRABAJO DEL CURSO ENARM CMN SIGLO XXI


VACUNACION
CIENCIAS BASICAS: Las vacunas son sustancias que nos ayudan a prevenir muchas enfermedades infecciosas, las cuales son contagiosas y provocan
discapacidad grave o incluso la muerte. Cartilla Nacional de Vacunacin (0 a 19 Aos): Es el documento Oficial para dar seguimiento al esquema de
vacunacin a los nios menores de 5 aos, escolares y adolescentes hasta los 19 aos de edad. Siempre que se acude a vacunacin se debe de
entregar para ver el esquema o anotar la vacuna
que se est aplicando en ese momento. BCG:
Vacuna contra la tuberculosis. Dosis nica de 0.1
ml,
RN, hasta los primeros 5 aos,
va
intradrmica, en la regin deltoidea del brazo
derecho, sin prueba tuberculnica previa, deja una
cicatriz en el brazo posterior a su aplicacin.
Contraindicaciones: Fiebre > 38.5 C. RN peso
inferior 2 kg, dermatitis progresiva, pacientes con
cuadro clnico de SIDA, Tx Transfusiones, o
inmunoglobulina, esperarn cuando menos tres
meses para ser vacunadas, no aplicar durante el
embarazo, no aplicar BCG con profilcticas de
medicamentos antituberculosos. HEPATITIS B: Se
aplican 3 dosis, 10 mcg en 0.5 ml. IM profunda, en
la cara anterolateral externa del muslo izquierdo en
los menores de 18 meses de edad, a partir de los
18 meses de edad, en la regin deltoidea del brazo
derecho , la primera al nacer y a los 2 y 6 meses de
edad. Previene este tipo de hepatitis, que afecta
principalmente al hgado. Contraindicaciones:
Fiebre > 38.5 C. RN peso inferior 2 kg, enfermedad
grave, Tx Transfusiones, o inmunoglobulina,
esperarn cuando menos tres meses para ser
vacunadas, hipersensibilidad a cualquier componente. PENTAVALENTE: Es la vacuna que previene la difteria, tos ferina y ttanos, adems tambin a
la poliomielitis y a las bacterias del Haemophilus Influenzae del tipo b, que provocan neumonas y meningitis. Se aplica en 4 dosis a los 2, 4, 6, y 18
meses de edad. Se aplica por va IM profunda, las primeras tres dosis deben aplicarse en el tercio medio de la cara anterolateral externa del muslo
derecho. A los 18 meses se debe aplicar en la regin deltoidea del brazo izquierdo. ROTAVIRUS: Previene la gastroenteritis o la diarrea causada por
el rotavirus en sus formas graves. Se aplican 3 dosis 0.1 ml sobre la parte interna de las mejillas, a los 2, 4 y 6 meses de edad y nunca despus de los
8 meses de edad. Contraindicaciones: Hipersensibilidad despus de la administracin de la vacuna, antecedente de enfermedad gastrointestinal
crnica, malformacin congnita no corregida en el tracto gastrointestinal. SABIN: Ayuda a prevenir la poliomielitis. Se aplican 2 gotitas en los
menores de 5 aos a partir de los 6 meses como dosis adicional (de 0.1 ml) en cada Semana Nacional de Salud, siempre y cuando hayan recibido dos
dosis previas de vacuna de poliovirus inactivada, uso de esta vacuna en actividades de control de casos y brotes. Contraindicaciones: Infeccin por
VIH, tener inmunodeficiencia congnita, tener hemopata o tumor slido o terapia inmnunosupresora. NEUMOCCICA: Contra el neumococo. Se
aplican 3 dosis, 0.5 ml, a los 2, 4 y 12 meses de edad. Va IM en el tercio medio de la cara anterolateral externa del muslo derecho.
Contraindicaciones: Procesos febriles de ms de 38.5 C, con enfermedad grave, reaccin alrgica severa (anafilctica) hacia un componente de la
vacuna incluyendo toxoide diftrico, antecedentes de
Sndrome de Guillain-Barr, antecedentes de
hipersensibilidad al ltex. No revacunar antes del
tiempo establecido, ya que puede provocar
reacciones locales severas. Aplicar un refuerzo antes
del tiempo establecido puede provocar el desarrollo
de neumona neumocccica posvacunal severa.
ANTI-INFLUENZA: Previene el virus de la influenza y
se aplica a nios de 6 a 35 meses de edad desde
octubre a febrero, 0. 25 ml IM basto lateral
izquierdo. En la primera ocasin se aplican dos dosis
con intervalo de 1 mes y despus cada ao.
Contraindicaciones: No aplicar a personas con alergia
a las protenas del huevo o a algn componente de la
vacuna, fiebre, haberse aplicado el biolgico en
menos de un ao, en lactantes menores de 6 meses,
en personas que hayan tenido una reaccIn de
hipersensibilidad,
incluyendo
una
reaccin
anafilctica al huevo o a una dosis previa de vacuna
contra influenza, ante la presencia de fiebre mayor o
igual a 38.5C, pacientes con antecedente de
sndrome de Guillain Barr. VACUNA TRIPLE VIRAL
(SRP): Previene el sarampin, la rubeola y las
paperas. Se aplica 2 dosis de 0.5 ml de vacuna
reconstituida: 12 meses de edad y los 6 aos de
edad va SC en el rea superior externa del trceps
del brazo Izquierdo, inscritos o no en primer ao de primaria. Contraindicaciones: Fiebre > 38.5, alrgicos al huevo, neomicina, antecedente de
aplicacin de inmunoglobulina o transfusiones 3 meses previos a la vacunacin, neoplasias, inmunodeficiencias, tratamiento con corticoesteroides,

CURSO ENARM CMN SIGLO XXI TEL: 36246001

Pharmed Solutions Institute

PGINA 326

MANUAL DE TRABAJO DEL CURSO ENARM CMN SIGLO XXI


citotxicos, embarazo. Poco frecuentes, pueden presentarse: Parotiditis uni o bilateral, despus de doce das y dura menos de cuatro das,
meningitis asptica, entre la segunda y la cuarta semanas posteriores a la vacunacin, prpura trombocitopnica. DPT: Sirve como un refuerzo que
previne a la difteria, tos ferina y ttanos. Se aplica dosis de 0.5ml a los 4 aos, se aplica por va IM profunda en la regin deltoidea del brazo
izquierdo. Contraindicaciones: No mayores a 5 aos, reaccin anafilctica a una aplicacin previa, antecedentes de crisis convulsivas o alteraciones
neurolgicas, fiebre > 38. 5C. Toxoide Tetnico Diftrico (TD): Se aplica a las personas desde los doce aos hasta los adultos mayores, hombres y
mujeres, especialmente a las embarazadas, dosis de 0.5ml. Previenen el ttanos en los recin nacidos y en los adultos. Las personas no vacunadas o
con esquema incompleto de vacuna pentavalente o DPT recibirn al menos dos dosis con intervalo de 4 a 8 semanas entre cada una, y revacunacin
cada 10 aos. Contraindicaciones: No suministrar a personas con hipersensibilidad a algn componente de la vacuna, a personas con
inmunodeficiencias, a excepcin de la infeccin por el VIH asintomtica, fiebre superiores a 38.5C y enfermedades graves, historia de reaccin grave
de hipersensibilidad o eventos neurolgicos relacionados con la aplicacin de una dosis previa, las personas transfundidas o que han recibido
inmunoglobulina, debern esperar tres meses para ser vacunadas, salvo en aquellos casos de traumatismos con heridas expuestas, ya que puede
aplicarse, simultneamente con antitoxina, independientemente de transfusin o aplicacin de inmunoglobulina. EVENTOS TEMPORALMENTE
ASOCIADOS A LA VACUNACIN ETAVS: Se define como eventos temporalmente asociados a la vacunacin a todas aquellas manifestaciones clnicas
que se presentan dentro de los 30 das posteriores a la administracin de una o ms vacunas y que no pueden ser atribuidos inicialmente a alguna
entidad nosolgica especifica (para la vacuna Sabin el periodo puede ser hasta de 75 das y para la vacuna BCG hasta seis meses un ao). Falsa
contraindicacin para todas las vacunas: Reaccin local de leve a moderada, enfermedad leve con o sin fiebre, tratamiento antimicrobiano actual,
fase de convalecencia de alguna enfermedad, exposicin reciente a enfermedades infecciosas, historia de alergia a penicilina u otras alergias
inespecficas o el hecho de que sus familiares tengan dichas alergias, alergia a protenas del huevo, a neomicina manifestada slo por rash.
VACUNACION
Un nio de 24 meses de edad, naci a trmino, cuenta con el
antecedente que despus de haber recibido la vacuna
antipoliomieltica, fiebre (38,5-40C), al da siguiente presento
vmitos y convulsiones tnico-clnicas. El lquido cefalorraqudeo
mostr pleocitosis y aumento moderado de protenas. El LCR fue
positivo por PCR para enterovirus y una pleocitosis. Las muestras de
heces tomadas los das 5 y 9 fueron positivos para enterovirus, que
se caracteriz posteriormente como poliovirus tipo 3.

CASO CLINICO
Se trata de recin nacido femenino de trmino de 2 meses de vida,
acude a medicina preventiva para aplicar esquema de vacunacin.

PREGUNTA
Cul es la conducta a seguir?
RESPUESTA
a.- Aplicar SRP
b.- Vacuna pentavalente acelular.
c.- Evitar DPT a los 4 aos.
d.- Revacunacin anual.
CASO CLINICO
Una nia previamente sana de 6 meses de edad es llevada por sus
padres a la consulta por presentar, fiebre de hasta 38,9C de un da
de evolucin, adems de referir que le hacen falta unas vacunas, la
madre comenta que la de neumococo. Uno de los hermanos ha
presentado durante estos das una infeccin respiratoria alta y el
otro present varicela el mes anterior. La paciente recibi la semana
anterior varias vacunas, entre las que estaban la
3ra de
pentavalente acelular, la 3ra de hepatitis B. Dentro de la
sintomatologa de la paciente destaca un aumento en la irritabilidad.
PREGUNTA
De acuerdo a su edad. Qu otras vacunas se le deben aplicar ya?
RESPUESTA
a.- 3era de rotavirus y 1ra de influenza
b.- 2da de rotavirus y 1ra de SRP
c.- 1ra de neumococo y 3era de rotavirus
d.- 2da. De rotavirus y 1ra de influenza
PREGUNTA
La madre refiere las siguientes condiciones de su hija Cul es la
razn ms probable por la que no aplicaramos la vacuna en este
momento?
RESPUESTA
a.- Fiebre mayor a 38.5
b.- Tratamiento antimicrobiano actual
c.- Alergia a protena de huevo manifestada solo por rash
d.- Exposicin reciente a enfermedades infecciosas

CURSO ENARM CMN SIGLO XXI TEL: 36246001

PREGUNTA
A qu edad le dice usted a la madre que le toca la vacuna de
neumococo?
RESPUESTA
a.- 10 meses
b.- 12 meses
c.- 18 meses
d.- 4 aos

PREGUNTA
Cuntas vacunas le deben aplicar a este paciente?
RESPUESTA
a.- 1ra de hepatitis, rotavirus, 2da de pentavalente
b.- 2da de hepatitis, 1ra de pentavalente, rotavirus y neumococo
c.- 2da de hepatitis, 1ra de pentavalente, rotavirus
d.- 1ra de hepatitis, 2da de pentavalente e influenza
CASO CLINICO
Nio de 8 aos sano y vacunado correctamente de lactante. En el
hospital de referencia en el transcurso de una infeccin vrica con
mnima alteracin heptica, se le practica serologa frente a hepatitis
B siendo negativos los marcadores: Anti-HBs, Anti-HBcy AgHBs.
PREGUNTA
Qu haramos con este paciente?
RESPUESTA
a.- Aplicar una dosis de recuerdo de la vacuna antihepatitis B
b.- Aplicar una nueva serie de 3 dosis de la vacuna anterior
c.-Repito serologa dentro de 1 mes por si fuera un falso negativo
d.- No es necesario realizar nada
PREGUNTA
Y si a este nio se le hubieran realizado los marcadores por contacto
con una jeringuilla abandonada. La actitud sera la misma?
RESPUESTA
a.- No cambio de actitud y hago serologa en 1 mes
b.- Aplico una nueva serie de 3 dosis de la vacuna antihepatitis B
c.- Aplico una dosis de inmunoglobulina antihepatitis B
d.- Aplico una dosis de inmunoglobulina antihepatitis B junto a una
dosis de vacuna antihepatitis B y continuo con otras 2 dosis de
vacuna al mes y a los 6 meses

Pharmed Solutions Institute

PGINA 327

MANUAL DE TRABAJO DEL CURSO ENARM CMN SIGLO XXI


DERMATITIS DE CONTACTO
CIENCIAS BASICAS: La dermatitis de contacto o eczema de contacto constituye un sndrome motivado por la reaccin cutnea de una sustancia
aplicada en la piel. Puede ser una reaccin aguda (eccematoza) o crnica (liquenificacion) y est causada por un irritante primario o por un
mecanismo de sensiblizacin a un alrgeno tpico. SALUD PUBLICA: Es una de las enfermedades de la piel ms frecuentes, alrededor de 5-10% de
todas las consultas dermatolgicas, 25-50% son debido a enfermedad ocupacional El 80% son irritativas y el 20% alergicas. Principales
sensibilizantes: Medicamentos (antibiticos, sulfonamidas, mercuriale, antihistamnicos, anestsicos, psoralenos), cosmticos (parafenilenodiamina,
perxidos, formaldehido, colorantes azoados, perfumes), metales (niquel, cromo, cobalto), ropas y zapatos (telas sintticas, hules, cueros), plsticos
(epoxi, resinas, acrlicos, nylon), remedios caseros (ajo, limn, yerbas, naranja). Fcatores predisponentes: humedad, sudoracin, higiene personal
deficiente, traumatismo fsico simultaneo, regin anatmica. CLASIFICACION: Irritativa, alrgica, fototoxicidad (UVA 320-400nm), fotoalergia
(reaccin patolgica a la luz), urticaria de contacto. PATOGENIA: Cualquier sustancia u objeto que este en contacto con la piel puede ocasionar una
dermatitis de contacto. El mecanismo puede ser: A) Alrgico, por el modelo clsico de inmunidad celular (hipersensibilidad tipo IV). Es necesaria fase
de sensibilizacin, fase de latencia y de reexposicin o desencadenante. El antgeno suele ser un hapteno que una vez en la epidermis, previa unin a
una protena, conforma un antgeno completo que, mediante clulas de Langerhans, es presentado a los linfocitos T que proliferan como clulas T
efectoras y de memoria penetrando en la circulacin sangunea. En la fase de desencadenamiento, 48 hrs despus de la exposicin, los linfocitos
sensibilizados reconocen a los alrgenos dando lugar a una transformacin blstica y proliferacin clonal con liberacin de citocinas mediadoras de
la inflamacin. Es posible la sensibilizacin por va oral o parenteral. El fenmeno de autoeczematizacion por protenas epidrmicas es frecuente en
lesiones en piernas. B) Irritativo que origina una inflamacin de la piel secundaria a la exposicin de un agente irritante, que romepe la barrera, hay
prdida de agua, descamacin y liquenificacion, en concentracin y tiempo suficiente o contactos repetidos del mismo que da lugar a lesin celular
sin que exista mecanismos inmunolgicos. C) Fototoxicas: en todas las personas desde la primera vez, depende de la concentrancion y tiempo de
permanencia, es aguda y violenta, no hay factores inmunolgicos, no hay IDES, no hay transferencia pasiva, las lesiones son monomorficas (v del
escote, cara, dorso de manos). D) Fotoarlergias: poco comn, requiere susceptibilidad, periodo de latencia mas de 48 hrs, eccema, reaccin
inflamatoria. DIAGNOSTICO: Localizacin variada y siempre en relacin al agente causal. La clnica puede ser: 1. Aguda, con eritema, edema,
vesculas, ampollas y en casos extremos necrosis, existe prurito quemante. Eczema, es la lesion elemental de la dermatitis aguda, caracterizado por
vesculas, costras, fondo eritematoso, piel aspecto lloroso. 2. Subaguda, con placas eritematosas con descamacin. 3. Crnica con lesiones en placas
liquenificadas, secas y zonas de descamacin y excoriacin, costras hematicas. A veces grietas y fisuras, en casos muy intensos y de exposicin
prolongada al antgeno, pueden aparecer lesiones a distancia o ides (lesiones a distancia de la dermatitis original, predominan en partes
expuestas, tienden a ser simtricas y bilaterales, ppulas, vesculas). Urticaria por contacto: inmunolgica mediada por IgE, ronchas, eritema o solo
prurito. El diagnstico se basa en la clnica, profesin y hbitos del paciente. Las pruebas epicutneas sirven para diferenciar el eczema de contacto
de origen irritativo del de origen alrgico y, en esta caso identificar el antgeno. Los diferentes tipos de eczemas o dermatitis de contacto tienen los
mismos rasgos histopatolgicos y van a depender de la fase evolutiva en la que se biopsien. Como rasgos generales evidenciaremos espongiosis,
exocitosis y en dermis superficial infiltrado perivascular, fundamentalmente de tipo linfocitario. Pruebas de parche, 48hrs, 2 lecturas.
TRATAMIENTO: No aplicarse nada en la piel, eliminar toda clase de pomadas, remedioscaseros, objetos, ropas cuando se sospecha de ellas. Evitar
jabones y detergentes. Dependera del agente causal en agudo si hay exudacin sulfato de cobre, cremas hidratantes, corticoide tpico, corticoide
sistmico. En la crnica corticoide tpico y emolientes. GPC: La piedra angular es hidratar, en piel seca y prurito dar emolientes, acetato de alumnio,
oxido de zinc, harina de soya, fomento de manzanilla, cold cream, vaselina, aceites ricos en lpidos, glicerina. En dermatitis de contacto irritativa dar
betametasona, esteroides sistmicos en casos severos. Dermatitis de contacto aguda y crnica, primera eleccin esteroides tpicos, hidratantes,
emolientes, segunda eleccin inhibidores de la calcineurina. Esteroides sistmicos, aziatropina, alitretinoina, ciclosporina.
CASO CLINICO
Un nio de 12 aos, en buen estado general y afebril, consult, por
presentar una placa eritematosa nica, amplia, de bordes definidos,
sobre la que asentaban pequeas ppulas y pstulas, no fluctuante,
localizada en la regin lateral derecha del cuello, de 3 das de
evolucin; se acompaaba de leve prurito, pero era indolora. No
presentaba adenopatas regionales. Haba acudido a una guardia,
donde se le indic cefalexina en dosis de 100 mg/kg/da; en el
momento de la consulta cursaba el primer da de tratamiento. En el
interrogatorio se destacaba que, 48 horas antes a la aparicin de las
lesiones, el nio haba participado en actividades recreativas al aire
libre en la colonia de verano del club, y no se haba aplicado
repelente.

pequeas agrupadas de 1 cm de dimetro, especialmente a nivel de


las mejillas. Los padres reeren que hace 2 das acudieron a
vacacionar en playa donde al nio se le aplic tatuaje temporal de
henna negra; no haba recibido manejo mdico.
PREGUNTA
Cul es la causa ms probable de la IDES en este paciente?
RESPUESTA
a.- Irritacin por contacto directo
b.- Sensibilizacin a distancia
c.- Por corticoestropeo
d.- Por fotosensibilizacion

PREGUNTA
Cual es la conducta a seguir mas adecuada en este momento?
RESPUESTA
a.- Continuar con antibitico por 5 dias.
b.- Corticoides topicos.
c.- Prednisona oral.
d.- Cetirizina nicamente

PREGUNTA
Cul es la complicacin ms probable si no se trata a este paciente
y evoluciona de manera crnica?
RESPUESTA
a.- Eccema
b.- Ptiriasis rosada
c.- Hipopigmentacion
d.- Liquenificacin

CASO CLINICO
Paciente masculino de 10 aos de edad, aparentemente sano, que
acude a consulta por dermatosis diseminada a extremidad superior
derecha y cabeza, afectando cara externa de brazo con placa
eritemato eccematosa ardorosa con erosiones hmedas
parcialmente cubiertas por elementos costrosos melicricos de 5 por
4 cm de tamao; se observa dicultad de la movilidad del miembro
superior afectado. En cara se evidencian placas papulo-rojizas

PREGUNTA
Cul es la medida teraputica ms apropiada en este caso?
RESPUESTA
a.- Fomentos secantes, esteroide tpico de baja potencia,
antihistamnico oral.
b.- Fomentos secantes, aziatropina, emoliente
c.- Hidratar, tacrolimus, anthistaminico oral
d.- Esteroide tpico de baja potencia, emoliente, tacrolimus

CURSO ENARM CMN SIGLO XXI TEL: 36246001

Pharmed Solutions Institute

PGINA 328

MANUAL DE TRABAJO DEL CURSO ENARM CMN SIGLO XXI


DERMATITIS DEL PAAL
CIENCIAS BASICAS: Dermatitis del paal, dermatitis amoniacal, rozadura. Es una erupcin inflamatoria aguda del rea cubierta por el paal.
SALUD PUBLICA: Afecta con mayor frecuencia a recin nacidos y lactantes, con un pico de incidencia a los 9 a 12 meses. Se presenta adems en
nios mayores y adultos con incontinencia urinaria y/o fecal secundaria a alteraciones urolgicas o neurolgicas. La prevalencia es del 7 al 35% en
poblacin peditrica, ocupando el 3 a 4 causa de consulta dermatolgica en nios. Factores de riesgo, mal aseo de la piel, paal hmedo,
detergentes, suavizantes para la ropa, automedicacin, inmunodeficiencias, maceracin, desnutricin, sudoracin, evacuaciones acidas.
CLASIFICACION: Irritativa, intertriginosa, sifiloide, vesiculopostular o vegetante. PATOGENIA: Koblenzer clasific la dermatitis de esta zona en tres
grupos: Grupo 1: Enfermedades que se producen en forma independiente del uso de paales (epidermolisis bulosa, histiocitosis de clulas de
Langerhans, etc.). Grupo 2: Enfermedades agravadas por el uso de paales (dermatitis atpica, dermatitis seborreica, psoriasis, etc.). Grupo 3:
Enfermedades provocadas por el uso de paales, en nios no predispuesto y como consecuencia directa de su uso, es el ms frecuente y ocupar
nuestra revisin. Lo ms caracterstico es la dermatitis de contacto por irritante primario en la cual la humedad persistente provoca maceracin de la
piel; lo anterior aunado a la friccin generada por los movimientos del beb altera la funcin de barrera y permite la accin de irritantes generan
activacin de las clulas de Langerhans, vasodiltacion y edema intersticial, los irrtantes como son: heces (proteasas, lipasas y sales biliares), orina
(urea y amoniaco), jabones, lociones, cremas y productos empleados para el lavado de los paales; por lo tanto, los irritantes actan en forma
secundaria en piel ya daada previamente. Por otro lado, la piel as daada, es susceptible de infeccin por Candida albicans con mayor frecuencia
(33%). Etiologia: Candida albicans (50%), Sthaphylococcus aureus, estreptococ, enterobacterias. DIAGNOSTICO: Dermatitis eritematosa simple, es el
ms frecuente. Caracterizada por eritema y papulas que son lesiones elementales, otras exulceraciones (por agresin), escamas, vesculas, costras y
liquenificacion. Afecta reas convexas (en W), respetando los pliegues, genitales, glteos, ingles, hipogastrio. Variedad sifiloide ppulo-erosiva o
poserosiva de Sevestre-Jaquet, es una forma moderada de la dermatitis eritematosa simple, se presenta sobre las superficies convexas en que se
observan ppulas del color de la piel o eritematosas, generalmente menores de 5 mm de dimetro, que por accin de la friccin o maceracin se
erosionan en la cpula, pueden volverse violceas y liquenificadas, aspecto umbilicado o en crater. Dermatitis perianal: Limitada a esta rea, se
manifiesta como eritema, erosiones o pstulas; es ms frecuente en el periodo neonatal. Puede deberse a cambios qumicos (alteracin del pH en
procesos diarreicos o enteropatas) o infecciosos (dermatitis estreptocccica perianal, parasitosis). El diagnstico es clnico; puede solicitarse
examen directo con KOH y y biopsia, adems cultivo para corroborar infeccin mictica. La biopsia se reserva a casos resistente al tratamiento, en
que se sospecha otra enfermedad de base. TRATAMIENTO: Medidas preventivas, como son el cambio frecuente de paal, recomendndose paales
desechables con geles absorbentes (alginatos), de mayor tamao, para evitar el roce y facilitar la absorcin; realizar el aseo del rea con agua,
dejando el rea sin paal el mayor tiempo posible. Las pastas con talco y xido de zinc son tiles como aislantes y para reducir la friccin y se aplican
posterior a cada cambio de paal. En las lesiones con ms de 72 horas de evolucin o datos de infeccin por candida debe aplicarse algn agente
antimictico (itracanozal, fluconazol), 2 veces al da, por 3 semanas. En caso de imptigo segundario se recomienda el uso de antibiticos tpicos o
sistmicos. La utilizacin de corticoesteroides deber ser racional, restringindose a casos resistentes a las terapias convencionales, utilizando
esteroides de baja potencia y nunca ms de 2 semanas, como hidrocortisona al 1% o triamcinolona o.025% c/12hrs. COMPLICACIONES: Candidosis:
es la complicacin ms frecuente, compromete los pliegues inguinales e interglteos y se caracteriza por ppulas y pstulas sobre una base
eritematosa, intensa, brillante, con bordes netos, elevados, geogrficos y escama blanquecina; asimismo lesiones satlites constituidas por ppulas
y pstulas que se presentan en la periferia. Se ha demostrado colonizacin por C. albicans en la dermatitis de la zona del paal con ms de 72 horas
de evolucin (45-75%) Corticoestropeo. Imptigo secundario: Agregndose pstulas y costras melicricas. Granuloma glteo infantil: Ppulas y
ndulos rojizos o purpricos de 0.5 a 4 cm, en regin perianal y gltea; se ha asociado a irritacin y friccin crnica, C. albicans y uso de esteroides
fluorinados.
CASO CLINICO
Nia de 4 meses de edad que la trae su padre porque al ir a
cambiarle el paal le ha visto que tena la zona muy eritematosa y
adems al limpiarle la nia se queja.
PREGUNTA
Considerando el agente etiolgico mas frecuente. Cul es la
conducta especifica mas adecuada?
RESPUESTA
a.- Desecantes
b.- Emolientes
c.- Antifungico
d.- Antihistaminico
CASO CLINICO
Paciente de 12meses de edad de sexo femenino, con dermatitis en
el rea del paal de 10 das de evolucin. Clnicamente se present
como una placa eritematosa de lmites difusos, ligeramente
sobreelevada, con ppulas, vesculas, pstulas y lesiones escoriadas.
Localizacin: rea que cubre el paal. Como sntoma relevante
presentaba prurito intenso. Antecedentes neonatolgicos: G1 P1 A0.
Recin nacido de trmino con peso adecuado para la edad
gestacional. Peso de nacimiento 3,450Kg.

Cul es la medida teraputica inicial ms adecuada?


REPUESTA
a.- Cambio frecuente de paal ms aplicacin tpica de xido de zinc
b.- Cambio frecuente de paal ms aplicacin de emoliente
c.- Cambio frecuente de paal ms humectante
d.- Cambio frecuente del paal ms lubricante
PREGUNTA
Cul es la etiologa ms probable en este caso?
RESPUESTA
a.- Staphilococcus epidermidis
b.- Candida albicans
c.- Estreptococo
d.- Enterobacterias
PREGUNTA
Cul es la complicacin menos probable que presentara este
paciente, de no recibir tratamiento?
RESPUESTA
a.- Impetigo
b.- Candidiasis
c.- Psoriasis
d.- Granuloma infantil cutneo

PREGUNTA

CURSO ENARM CMN SIGLO XXI TEL: 36246001

Pharmed Solutions Institute

PGINA 329

MANUAL DE TRABAJO DEL CURSO ENARM CMN SIGLO XXI


DERMATITIS SEBORREICA
CIENCIAS BASICAS: Es una dermatosis inflamatoria eritemato-descamativa de evolucin crnica y recurrente que se localiza en cara, cuero
cabelludo, pliegues y regiones medio-torcicas e interescapulares (zonas con mayor cantidad de glndulas sebceas activas). Tambin se le conoce
como eczema seborreico o pitiriasis seca y esteatoide. SALUD PUBLICA: Enfermedad frecuente en pases desarrollados, no existiendo diferencias de
gnero. La dermatitis seborreica infantil aparece en la lactancia, habitualmente entre la 2 y la 8 semanas de vida Puede afectar tanto a nios
como a adultos o ancianos. PATOGENIA: La causa es desconocida. Se consideran factores predisponentes, genticos, emocionales, Atpicos,
neurolgicos, bacterianos, endocrinos, alimentarios, medicamentosos, estrs y alcoholismo. Tambin se ha relacionado con la disminucin de cidos
grasos esenciales y biotina. Aunque se conoce que existe un cambio cuantitativo y cualitativo en la composicin del sebo con transformacin de
triglicridos en cidos grasos ms irritantes con mayor cantidad de colesterol y menor de escualeno, no se sabe cul es la causa de la citada
alteracin aunque se implica en ello la accin del Pityrosporum ovale, levadura lipfila que forma parte de la flora cutnea normal y se convierte en
oportunista con potencial patgeno. En pacientes con SIDA hay disminucin de linfocitos CD4. En nios se desconoce la causa aunque se sabe que
del 15-35% de os pacientes durante la infancia padecieron dermatitis seborreica, padecen en la edad adulta, psoriasis o dermatitis atpica. Un grupo
de autores suecos arma que la dermatitios seborreica infantil (DSI) podra deberse a un dcit de la enzima d-6-desaturasa, que convierte el cido
linolico en cido gammalinolnico (AGL). Este dcit pare ce normalizarse hacia los 6 7 meses de edad, coincidiendo con la desaparicin de la DSI.
La asociacin entre las levaduras de Malassezia y la patogenia de esta enfermedad ha sido causa de controversia desde los aos setenta. Evidencia
reciente relaciona a la dermatitis seborreica con metabolitos del hongo y con la induccin de citocinas. El hallazgo de las levaduras en escamas de los
pacientes es lo habitual. Las especies ms frecuentes en cuero cabelludo son: M. globosa y M. restricta, y en rostro M. globosa y M. furfur.
DIAGNOSTICO: En todas las reas, las lesiones de DSI consisten en placas eritematosas de tinte rosado, con morfologa redondeada o incluso
circinada, con bordes bien denidos, aisladas o con uentes, que se cubren de una descamacin amarillenta de aspecto grasiento, que merece la
denominacin de seborreica. La DSI no se acompaa de sntomas generales ni prurito. Para muchos autores, la forma ms precoz de aparicin de la
DSI es la costra lctea. Otros, consideran que la costra lctea es una entidad diferenciada, sin ninguna relacin con la DSI. Sea como fuere, la costra
lctea se desarrolla en la primera o segunda semanas de vida, y se maniesta como una gran placa descamativa, con escamas grasientas y
adherentes, de color amarillento, bajo las cuales se puede apreciar un eritema ms o menos vivo. Asienta sobre todo en el vrtex, y puede
extenderse hacia reas frontales con menor frecuencia, puede ocupar una amplia supercie del cuero cabelludo. Las lesiones clsicas de DSI suelen
afectar, adems del cuero cabelludo, a la frente, pabellones auriculares, pliegues retroauriculares, nasogenianos, cejas, parpados y cuello. Tambin
puede extenderse a el tronco, con predominio en la zona periumbilical, axilas, ingls, trax anterior y espalda En los miembros, suele afectar a los
pliegues antecubitales y poplteos. En ocasiones, puede ser un trastorno generalizado, de tipo eritrodrmico. La afectacin de la zona del paal es
muy frecuente, especialmente en las reas inguinales, pliegue interglteo y genitales externos. El trastorno conocido como eritrodermia
descamativa de Leiner cursa con eritrodermia descamativa seborreiforme, retraso del crecimiento y diarrea. No hay estudios a gran escala para
establecer la histopatologa de la DSI, aunque parece similar a la DS del adulto. Se observa paraqueratosis focal, acantosis y espongiosis. En la
desembocadura de los folculos pilosos dilatados se observan polimorfonucleares neutrlos. La dermis supercial muestra un inltrado
linfohistiocitario perivascular. TRATAMIENTO: No tienen tratamiento definitivo. Dependiendo del grado de afectacin se puede utilizar desde
jabones no alcalinos o sustitutivos del jabon, a lociones o crema a base de cido saliclico, azufre, selenio, ketoconazol u otros derivados azolicos,
ciclopiroxolamina, terbinafina, hidrocortisona, tacrolimus. A nivel general se puede complementar con biotina a altas dosis.

CASO CLINICO
En la revisin de un bebe al mes de nacido, la madre refiere que el
nio presenta costras que le sale sobre todo en la cabeza y en la
frente y cejas.

RESPUESTA
a.- Psoriasis
b.- Dermatitis atpica
c.- Dermatitis seborreica
d.- Candidiasis

PREGUNTA
Cual de las siguientes medidas teraputicas es la menos adecuada?
RESPUESTA
a.- Aplicar aceites y sustancias emolientes.
b.- Champues con antifungicos.
c.- Dieta rica en biotina.
d.- Corticoide de baja potencia.

PREGUNTA
Cul es el origen ms probable para el cuadro de este paciente?
RESPUESTA
a.- Predominio de Th1, IL-2, IF-gamma, herencia
b.- Predominio de TH2, antecedentes de alergia, eosinofilia
c.- Alteracin cualitativa/cuantitativa de la secrecin sebcea
d.- Inmunosupresion, Candida albicans

CASO CLINICO
Lactante masculino de 3 meses de edad, el cual es llevado por madre
por presentar lesiones en pliegues nasolabiales, cuero cabelludo, la
madre refiere que el nio no se las toca, por lo que supone no le dan
comezn, refiere que le aplico una crema pero que no observo
mejora. Al examen fsico dermatolgico llam la atencin, la
presencia de mltiples lesiones en placas eritemato-escamosas,
escamas estas blanco amarillentas de aspecto oleoso, acompaadas
de gran inflamacin, que se localizan en cuero cabelludo, cara y
trax

PREGUNTA
Cul es la conducta teraputica ms adecuada para este caso?
RESPUESTA
a.- Ketoconazol 2% mas corticoesteroide tpico
b.- Loratadina mas ketoconazol al 2%
c.- Dicloxacilina ms corticoide tpico
d.- Ac. Saliclico ms corticoesterioide

PREGUNTA
Cul es el diagnstico ms probable para este caso?

CURSO ENARM CMN SIGLO XXI TEL: 36246001

Pharmed Solutions Institute

PGINA 330

MANUAL DE TRABAJO DEL CURSO ENARM CMN SIGLO XXI


DERMATITIS HERPETIFORME (DH)
CIENCIAS BSICAS: Es una enfermedad ppulovesicular, muy pruriginosa, crnica, de distribucin caracterstica, es apollosa subdermica autoinmune
determinada por IgA, asociada a enteropata por gluten. PATOGENIA: Se ha especulado sobre el papel de los anticuerpos del gluten en la
etiopatognesis de esta enfermedad. El gluten es una fraccin proteica remanente de la harina despus que ha sido lavada y eliminado el almidn.
Las principales fuentes de gluten en occidente son: trigo, centeno, cebada; el maz y la avenA. El 75% de los casos de DH tienen aplanamiento de las
vellosidades intestinales y 20% presenta infiltrado linfocitario en el epitelio como evidencia de enteropata sensible al gluten (ESG). Es bien
establecido, que las manifestaciones cutneas e intestinales mejoran con la dieta estricta, libre de gluten, sin embargo, el mecanismo fisiopatolgico
se desconoce. La asociacin inmunogentica apoya la naturaleza autoinmune de la DH a otras enfermedades autoinmunes. Aunque no se ha
descrito todava el antgeno diana, hay asociacin de la DH con otras enfermedades autoinmunes, como lupus eritematoso, enfermedades tiroideas
y dibetes mellitus. DIAGNOSTICO: La DH se ha reportado raramente en nios, pero la edad de presentacin es de 2 a 7 aos. Las lesiones cutneas
caractersticas son ppulas y placas eritematosas, urticarianas, edematosas sobre las cuales hay vesculas, distribuidas simtricamente en superficie
extensora de las extremidades. Los sitios caractersticos son: codos, rodillas, glteos, nuca y cuero cabelludo. Una sensacin de ardor o escozor suele
preceder las lesiones. El prurito es intenso haciendo que el paciente est inquieto. Hay lesiones secundarias al rascado como excoriaciones, costras
y mculas hiper e hipopigmentadas residuales. Las membranas mucosas no estn comprometidas. Sin embargo, se han descrito defectos en el
esmalte dental similar a lo observado en la enfermedad celaca, cambio de coloracin y textura de la superficie dentaria. Por lo menos un tercio de
los pacientes sufre de esteatorrea y anemia. El diagnstico se basa en criterios clnicos, histolgicos, as como la respuesta favorable a las sulfonas y
a la dieta libre de gluten. TRATAMIENTO: La dapsona a la dosis de 1.5-2 mg/kg/da, es efectiva para el control de las lesiones cutneas y del prurito
intenso. La suspensin de sta, produce la reaparicin rpida de las lesiones, por lo que el tratamiento debe ser combinado con la dieta libre de
gluten y disminucin progresiva y suspensin de la dapsona al lograr control, con lo que disminuye la aparicin de los efectos secundarios de la
droga: metahemoglobinemia, anemia hemoltica, agranulocitosis, y complicaciones neurolgicas. Antes de su uso, los niveles de glucosa-6-fosfato
deshidrogenasa deben ser determinados. En esos casos, en los que no se pueda utilizar la dapsona, se recomienda la sulfapiridina a 250 mg/da con
ajuste de acuerdo a respuesta teraputica. El curso de la DH en nios no est bien definido, la ESG persiste toda la vida, aunque con perodos de
remisiones espontneas en algunos casos.
CASO CLINICO
Paciente varn de 13 aos de edad que consulta por lesiones muy
pruriginosas, que se localizan fundamentalmente en codos, rodillas y
regin lumbosacra. Se ha tratado con diversas pomadas de
corticoides tpicos sin mejora. En la exploracin cutnea se observa
en los codos, rodillas y regin lumbosacra la presencia de ppulas
eritematosas, excoriadas y alguna pequea vescula intacta. Se
realiz biopsia cutnea de una de las lesiones y el estudio
histopatolgico demostr: microabscesos de neutrfilos a nivel de
papilas drmicas, El estudio de inmunofluorescencia directa
demostr depsito de Ig A granular en las papilas dermicas.

PREGUNTA
Considerando la patologa que presenta el caso. Cul de los
siguientes diagnosticos diferenciales es el menos probable?
RESPUESTA
a.- Dermatitis alrgica de contacto
b.- Dermatitis atpica
c.- Sarna
d.- Penfigoide ampolloso

IMPETIGO
CIENCIAS BASICAS: Es la infeccin bacteriana superficial y contagiosa ms frecuente en los nios, producida por Estafilococo Aureus, Estreptococo B
hemoltico del grupo A (pyogens) o ambos; puede ser primario en caso de invadir piel sana (generalemnte alrededor de orificios naturales, no
prurito, boca, fosas, nasales, pabellones auriculares) secundario (en cualquier parte del cuerpo, eritema, ampollas, pustilas, costras melicericas)
cuando existe dao de la barrera cutnea previamente, como ocurre en la dermatitis atpica, escabiosis, traumatismo. Lesion elemntal ampolla de
contenido claro, con halo eritematoso alrededor, a las pocas horas pustulas, que se rompen apareciendo un exudado seroso o seropurulento y
posteriormente costras. Imeptigo estafiolococico o ampollar y imptigo costroso o estrptococico. Estas infecciones pueden dividirse en: 1. infeccin
cutnea primaria 2. Infeccin secundaria de una enfermedad cutnea primaria y 3. lesiones cutneas como manifestacin de infeccin primaria en
otro rgano, ejemplo de este ltimo grupo tenemos: S.aureus causando sndrome de shock toxico y sndrome de piel escaldada estafiloccico, S.
pyogenes escarlatina, N. meningitides, sepsis por meningococo (petequias), S. thiphy, fiebre tifoidea (rosola tifoidea), Rickettsia conorii, Tifus
africano (rash macular) IMPTIGO CONTAGIOSO: Costroso, no ampollar o estreptococico, representa el 70% de los casos; predomina en nios de
todas la edades , aunque tambin se puede ver en adultos; el contagio se origina sobre todo por rascado. La infeccin puede ser originada por Strep.
Pyogenes (15%), Staph. Aureus (70 a 80 %) o ambos (15 a 100%). La cara es la zona de ms frecuente de localizacin, sobre todo en las uniones
mucocutneas, por ejemplo: introito nasal, boca y en todas las zonas periorificiales, manos, etc, pero como es contagioso, se puede ver en cualquier
zona corporal. La lesin inicial son ppulas eritematosas pequeas que rpidamente evolucionan a una vescula que se rompen rpidamente dando
paso a erosiones cubiertas por costras melicricas y lesin residual hipocrmica temporal (lesiones primitivas, ampollas, pstulas que al romperse
dejan salir exudado que se seca, costra melicerica), sin dejar cicatriz; si hay dolor es leve y puede presentar linfadenopata regional, pero no hay
sntomas sistmicos. Las lesiones suelen propagarse por autoinoculacin originando lesiones satlites. La resolucin es espontnea entre la segunda
y tercera semana de evolucin, pero debe ser tratada con prontitud y as se acorta el perodo de enfermedad y el contagio, as como las eventuales
complicaciones glomerulonefritis post estreptoccica, escarlatina psoriasis. Diagnstico: se basa en la clnica pero se puede confirmar con un gram
y cultivo. IMPTIGO AMPOLLAR: o estafilococico Representa el 30% restante, es observa ms frecuentemente en neonatos y lactantes. Es
ocasionado por la toxina exfoliativa tipo A del Staph. Aureus; esta toxina acta como una serina proteasa de desmoglena 1, ocasionando de esta
manera la acantlisis subcorneal que conduce a la formacin de ampollas. Las lesiones se pueden localizar en cualquier rea de la superficie
corporal, pero lo ms frecuente es alrededor de los orificios de la boca, nariz, rea genital y en las extremidades Clnica: pueden presentar fiebre
hasta de 40C, ampollas flcida, que se rompen con facilidad en el curso de 1 a 2 das, dejando erosiones cubiertas por una costra perifrica fina, de
ah que tambin se conozca como imptigo circinado; las lesiones aparecen usualmente agrupadas. Existe una tendencia a la curacin espontnea
sin cicatriz, pero pueden dejar hipopigmentacin residual temporal. Impetigo ampollosos de Ritter: Eritema, apollas, erosiones extensas, costras
melicericas, ataque al estado general, fiebre 40, recin nacido y lactantes, agente el estafilococo. TRATAMIENTO: Lavado con agua y jabon,
aplicacin de antisptico local como sulfato de cobre, agua de alibour, gluconato de clohexedine, clioquinol o bioformo, ac. Fusidico. Otras
opociones bacitracina, poliixina, gentamicina. Se puede dar rifampicina, eritromicina, NO esterioides. En lesiones muy diseminadas en nios
pequeos: dicloxacilina 100mg/kg/dia, eritromicina 30mg/kg/dia, o penicilina, oxacilina. Si no hay una buena respuesta clnica en forma rpida se

CURSO ENARM CMN SIGLO XXI TEL: 36246001

Pharmed Solutions Institute

PGINA 331

MANUAL DE TRABAJO DEL CURSO ENARM CMN SIGLO XXI


debera cultivar y realizar antibiograma. COMPLICACIONES: Raro septicemias, celulitis, glomerulonefritis. PENFIGO: Hacer dx diferencial, es una
enfermedad apollosa intraepitelial acantolitica de piel y mucosas, probable etiologa autoinmune, ampollas en unin dermo-epidermica, que
normalmente afecta a individuos de edad media y mayores, pero puede ocurrir raras veces en nios. Las dos variantes: Pnfigo vulgar (80%), pnfigo
foliceo, pnfigo inducido por frmacos, pnfigo paraneoplasico, pnfigo IgA. Los pacientes tienen autoanticuerpos que reaccionan con los
desmosomas, molculas de adhesin localizadas en la superficie de los queratinocitos. Los autoanticuerpos detectados son de la clase IgG.
Clnicamente, suele comenzar en la mucosa bucal con erosiones del epitelio. Las lesiones cutneas consisten en vesculas, ampollas flccidas que se
rompen con facilidad resultando en extensas erosiones. Se localiza predominantemente en cara, cuello, tronco, extremidades y reas
intertriginosas. Las erosiones causan dolor y ardor y secundariamente pueden infectarse con bacterias u hongos. El signo de Nikolsky es positivo
(eritema y formacin de ampollas como resultado de friccin o presin sobre la piel), signo de Asboe-Hansen (aumento perifrico del tamao de la
ampolla al presionar verticalmente su superficie), no es especifica. Se ha reportado onicolisis y distrofia ungueal. En el PV se evidencia la ampolla
intraepidrmica suprabasal, con acantolisis (desaparicin de puentes intracelulares) marcada. El piso de la cavidad est constituido por
queratinocitos basales y leve infiltrado inflamatorio, compuesto por eosinfilos. En lesiones tempranas los eosinfilos invaden la epidermis en
racimo. Este fenmeno se denomina espongiosis eosinoflica. La ampolla en el PF se localiza en la subcorneal de la epidermis. La acantolisis est
presente inmediatamente por encima o por debajo de las clulas de la capa granulosa. Este patrn se ha denominado panal de abejas y constituye
la herramienta diagnstica en estas enfermedades, cuya positividad alcanza hasta el 100%. Clulas Tzank, tiene 100% sensibilidad y 43%
especificidad. Se observan clulas acantoliticas (zona blanca). Las lesiones se alivian en 1-2 aos y dejan hiperpigmentacion psinflamatoria.
Enfermedad localizada pueden ser tratados con corticoesteroides tpicos. Sin embargo, la mayora de los casos desarrollan formas ms severas que
ameritan el uso de corticoesteroides sistmicos a altas dosis (2 a 3 mg/kg/da). Con la finalidad de disminuir los efectos adversos de stas, se usan
drogas adyuvantes como: azatioprina, ciclofosfamida, dapsona y metotrexato, sales de oro.
CASO CLINICO PENFIGO
Un nio de 12 aos de edad se present con lesiones generalizadas
en la piel eritematosas y costras de 4 meses de duracin. Las
lesiones aparecieron por primera vez en su cuero cabelludo y dentro
de unos das, eritrodrmica se generaliz y, a continuacin. Escalado
y exudacin tambin fueron vistos. Diferentes antibiticos y
esteroides tpicos fueron prescritos sin mejora significativa. El nio
tambin se quejo de prdida de la audicin desde un mes. En el
examen fsico, eritrodermia con escalamiento grave y maloliente con
descarga y leve queratodermia palmoplantar todo el cuero
cabelludo. Haba dos pequeas vesculas a lo largo del lado cubital
de la palma derecha. Las superficies de las mucosas y las uas eran
normales. Tena ganglios linfticos submandibulares. El conducto
auditivo estaba lleno de escamas y costras, el pabelln de la oreja
era sensible a la palpacin. Despus de quitar las costras, canal
auditivo externo se encontr que era roja e hinchada.
PREGUNTA
Cul es la conducta a seguir?
RESPUESTA
a.- Prednisona oral de 0.5 a 1 mg/kg al da.
b.- Dapsona dosis de 100 mg/da.
c.- Azatioprina es de 2.0 a 2.5 mg/kg,
d.- Ciclofosfamida a dosis oral de 1 a 3 mg/kg al da

PREGUNTA

CASO CLINICO
Nia de 2 aos, sin antecedentes de inters, con fiebre de 39,
afeccion al estado general, remitida por su pediatra, por presentar
desde haca un ao brotes sucesivos de lesiones vesiculosas y
ampollosas, con eritema, que curaban en el plazo de una semana a
diez das. En la exploracin fsica mostraba lesiones vesiculosas de
contenido seroso de aproximadamente 3 mm de dimetro en rea
perioral, perinasal, junto con costras melicericas, tambin reas de
erosiones extensas en piernas y en la zona perineal.
PREGUNTA
Cul es el agente etiolgico ms probable en este caso?
RESPUESTA
a.- Streptococo pyogenes
b.- Staphylococo aureus
c.- Haemophylus influenzae
d.- Streptococo epidermidis

CASO CLINICO
Se trata de escolar masculino de seis aos de edad quien es trado
por su madre por presentar lesiones ampollosas en las manos,
pecho, espalda y brazos de color amarillento con ardor e hipertermia
de 39.
PREGUNTA
Cual es la conducta a seguir inmediata mas adecuada?
RESPUESTA
a.- Iniciar antibitico.
b.- Administrar antipirtico.
c.- Hidratacin adecuada.
d.- Laboratorio de rutina.

Considerando la exploracin del caso cual es la conducta


teraputica para la etiolgica mas adecuada?
RESPUESTA
a.- Mupirocina.
b.- Acido fusidico.
c.- Penicilina.
d.- Amoxicilina

PREGUNTA
Cul es el diagnstico ms probable en este caso?
RESPUESTA
a.- Dermatitis herpetiforme
b.- Herpes simple
c.- Imptigo
d.- Pnfigo
PREGUNTA
Cul es el manejo ms adecuado para este paciente?
RESPUESTA
a.- Clioquinol, dicloxacilina 100mg/kg/dia
b.- Betametasona, dicloxacilina 100mg/kg/da
c.- Betametasona, dicloxacilina 30mg/kg/da
d.- Clioquinol, Eritromicina 30mg/Kg/da

ERISIPELA
CIENCIAS BASICAS: Es una Infeccin bacteriana profunda dermoepidrmica pero ms superficial que la Celulitis, localizada en dermis profunda y
zona superior del tejido celular subcutneo con afectacin linftica significativa. Casi siempre muy bien delimitada y con tendencia a progresar hacia
la periferia. Es una enfermedad potencialmente transmisible (penetra por una solucin de continuidad, grieta, fisura, rasurado, traimatismo) y
reviste mayor gravedad en los nios. SALUD PUBLICA: Factores de riesgo: linfedema, mastectoma, insuficiencia venosa, obesidad. Factores
predisponentes: focos infecciosos, traumatismos, eccema, tia de los pies, diabetes, desnutricin. PATOGENIA: Los agentes causales ms comunes

CURSO ENARM CMN SIGLO XXI TEL: 36246001

Pharmed Solutions Institute

PGINA 332

MANUAL DE TRABAJO DEL CURSO ENARM CMN SIGLO XXI


son: Streptococccus pyogenes (el principal) o Streptococcus beta-hemoltico del grupo A, con menor frecuencia por Streptococcus de los grupos B, C,
D o G, sobre todo en pacientes postquirrgicos con otras enfermedades. Los del grupo B suelen ser responsables de la erisipela del recin nacido.
Otros grmenes que pueden ocasionar erisipela: son Staphylococcus aureus, Pneumococcus, Klebsiella pneumoniae, Yersinia enterocoltica y
Haemophilus influenzae tipo b. La lesin comienza a travs de una pequea solucin de continuidad de la piel o de las mucosas, que constituye la
puerta de entrada y que en muchas ocasiones pasa inadvertida, por ejemplo en pacientes con ulceras, prurigo, intertrigo, tia pedis, etc paciente
diabticos o imunosuprimidos. De forma ms excepcional se debe a una diseminacin hematgena, procedente de focos a distancia, por ejemplo en
dientes o faringe. El periodo de incubacin es de 2 a 5 das, la localizacin ms frecuente son los miembros inferiores, puede localizarse en cara y
rea periumbilical en caso de los lactantes. DIAGNOSTICO: Precede a la lesiones cutneas sntomas como fiebre (40), escalofro, nuseas, cefaleas y
vmitos, malestar general y dolor local; las lesiones tienen comienzo abrupto y se presentan como una placa ardorosa, eritematosa de color rojo
intenso (brillante), indurada, edematosa, caliente, dolorosa a la palpacin y bien delimitada , su crecimiento es centrfugo con un borde activo y bien
definido, con la zona central de la placa aclarndose ; en algunas formas menos usuales se pueden presentar vesculas, ampollas, pstulas y lesiones
necrticas. Su topografa ms comn es en las extremidades inferiores (50%) y en la cara (30%) a nivel de las mejillas. Se acompaa con frecuencia
de adenopatas satlites regionales. Habitualmente las lesiones inician su involucin en 10-12 das descamndose la regin afectada sin dejar
cicatriz. Habitualmente el diagnstico es clnico pero si es posible obtener material de la puerta de entrada se debe realizar Gram y cultivo. Puede
haber leucocitosis. TRATAMIENTO: Antisepsia. Penicilina sdica cristalina, penicilina procainica (800,000U c/24hrs) 8-10 dias, mas benzatinica cada
semana por 3-4 semanas. 2da. Eleccin cefalosporinas, amoicilina/ac. Clavulanico. En cepas resistentes sulfas o clindamicina y en los pases con
cepas Meticilino- resistentes se pueden utilizar de acuerdo a estudios epidemiolgicos previos: Trimetoprim-sulfametoxazole, Clindamicina,
Rifampicina-, Linezolid y Vancomicina. Si hay alergia a penicilinas se puede usar eritromicina. Para evitar recidivas usar sultrim dos tabletas diarias
por 8-10 dias. COMPLICACIONES: incluyen bacteremia, tromcosis del seno cavernoso, abscesos, gangrena, linfangitis, celulitis, septicemia,
tromboflebitis y glomerulonefritis. Cuando se trata de brotes repetidos de erisipela la afeccin de los vasos linfticos que al fibrosarse originan
estados elefantisicos, conocidos como elefantiasis nostra. CELULITIS: Hacer dx., diferencial; Es una ulceracin previa, zona edematosa, e infiltrada
profunda, firme, hay crepitacin, es mas extensa, mas profunda (hipodermis), minimo eritema o ausente, no bordes delimitados, no elevacin de la
epidermis subyacente, dolor al tacto moderado o ausente, excepto en fascitis necrotizante. Su tratamiento es con dicloxacilina, vancomicina,
fosfomicina, en nios en la cara dar dicloxacilina mas cloranfenicol.
CASO CLINICO
Se trata de femenino de 2 aos de edad la cual cuenta con el
antecedente de haber jugado en la tierra y producirse escoriasiones
en miembros plvicos, a las 48 hrs posteriores inicia con fiebre no
cuantificada, enrojecimiento y edema en la zona, la cual se
encuentra caliente y eritematosa a la exploracin.
PREGUNTA
Cual es la conducta a seguir?
RESPUESTA
a.- Limpieza y tratamiento ambulatorio con penicilina.
b.- Debridacin y tratamiento hospitalario.
c.- Limpieza y alta con signos de alarma.
d.- Limpieza local, vendaje y penicilina

anteroexterna de la misma constituda por eritema, edema, cicatriz


postlesional impetiginizada de aproximademente 4 cms de longitud
acompaada de exudado purulento, y llanto al ponerla a caminar,
aumento de temperatura no cuantificada, disminucin del apetito.
A.P. Ha cursado con cuadros diarreicos amibianos en dos ocasiones
atentido en el servicio medico. faringoamigdalitis hace una semana
con cuadro gripal que hasta el momento no ha cedido.
PREGUNTA
Cul es el diagnotico mas probable para este caso?
RESPUESTA
a.- Celulitis
b.- Erisipela
c.- Linfangitis
d.- Sindrome estafiloccico de piel escaldada

CASO CLINICO
Femenino de 1 ao y medio de edad que es trada por familiar por
presentar dermatosis en pierna izquierda, predominando en la cara

PIODERMIA GANGRENOSA
CIENCIAS BASICAS: Fagadenismo geomtrico de Brocq, a pesar del nombre este cuadro de causa desconocida, dista mucho de ser una enfermedad
bacteriana, probablemente sea ms una inmunopatia que una verdadera infeccin aun cuando no pueda descartarse del todo la intervencin de
bacterias. PATOGENIA: Su etiologa es desconocida y se invocan alteraciones inmunolgicas aun no bien determinadas, se piensa en una reactividad
tisular local, necrtica a antgenos microbiana. DIAGNOSTICO: En cualquier rea del cuerpo sobre todo en las extremidades, aparece
repentinamente en personas en aparente estado de salud, un rea eritematosa que pronto se cubre de pstulas o ampollas y se ulcera rpidamente.
Esta ulceracin se extiende con contornos geogrficos, sus bordes son levantados y pueden aparecer abscesos en la periferia habitualmente
estriles, el fondo es sucio, sangrante o necrtico. En poco tiempo estas lesiones que profundizan, han abarcado extensas zonas de la piel. La
sintomatologa no corresponde a la intensidad del proceso, pero puede haber dolor y ataque al estado general del paciente. Es frecuente que este
proceso coincida con artritis, colitis ulcerosa, enteritis regional de Crhon o enfermedades hematolgicas con las cuales no hay necesariamente una
relacin causa-efecto. TRATAMIENTO: Es emprico, se han usado los corticoesteroides, la colchicina, la diaminodifenilsulfona, la clofazimina, la
talidomina. Algunos responden bien a determinados frmacos y otros casoso a otros o no responden. El proceso de repente y sin causa aparente se
detiene y deja cicatriz.
CASO CLINICO
Femenino de 17 aos que consult por lesiones ampollosas
dolorosas, de contenido claro, en rodilla derecha, acompaada de
fiebre con escalofros, de 1 semana de evolucin. Las mismas
lesiones aparecen gradualmente en toda la pierna derecha,
antebrazo derecho y glteo izquierdo, que revientan y dejan la piel
exulcerada. Ingres febril, estaba plida. En piel se observan
mltiples lesiones ampollosas exulceradas, con fondo fibrinoso, muy
dolorosas, ubicadas en rodilla y pierna derecha, otras ms pequeas
en mueca derecha y glteo izquierdo.

CURSO ENARM CMN SIGLO XXI TEL: 36246001

PREGUNTA
Cual es la conducta a seguir ahora?
RESPUESTA
a.- Limpieza y tratamiento ambulatorio con penicilina.
b.- Debridacin y tratamiento hospitalario con corticoides y
antibiticos.
c.- Limpieza y alta con signos de alarma.
d.- Limpieza local, vendaje y penicilina

Pharmed Solutions Institute

PGINA 333

MANUAL DE TRABAJO DEL CURSO ENARM CMN SIGLO XXI


TUBERCULOSIS (TB) CUTANEA
CIENCIAS BASICAS: La TB cutnea se considera en la mayora de los casos de reinfeccin, la primoinfeccin es excepcional, aunque posible
presentndose el complejo primario en la piel: ndulo cutneo, linfangitis y adenopatas. SALUD PUBLICA: Mycobacterium tuberculosis, quiz el
germen de mayor ubicuidad en el mundo y en el organismo pues no hay rgano que se escape a su agresin. La tuberculosis sigue siendo un
problema de salud pblica, especialmente en la forma pulmonar. La TB cutnea ocupa el 4to o 5to lugar, en cuanto a localizacin del bacilo de Koch.
Estadsticas mexicanos en varios aos muestran cifras que van de 1-3% de Tb cutnea por cada 114 pacientes y cerca de 20% son nios menores de
15 aos. PATOGENIA: Casi en todos los casos, los pacientes han recibido ya la primoinfeccin pulmonar inoculado con M. tuberculosis bovina y
hominis, presentando una respuesta positiva al PDD. A partir de este foco original mediante reinfeccionendogena.se diseminan los bacilos a la piel,
otras veces la reinfeccin es exgena por llegada de nuevos bacilos del exterior. De acuerdo con esta va de reinfeccin, de la virulencia del bacilo y
sobre todo de la respuesta inmunolgica (hipersensibilidad) del organismo ante el bacilo, se van a producir lesiones en la piel y que parecen
entidades diferentes. CLASIFICACION: de Latap, Escalona y Estrada; se consideran 2 grupos bsicos, el primer grupo est formado por entidades en
las cuales hay respuesta normrgica al bacilo. El segundo grupo comprende las llamadas tuberculoides, que como su nombre indica son lesiones
resultados ms de la respuesta exagerada, hiperrgica al bacilo. En este caso no es posible encontrar bacilos en las lesiones y la respuesta a los
antgenos bacilares es muy intensa. TB COLICUATIVA: Tambin conocida como escrufulosis o escrofulodermia, en Mxico es la variedad ms
frecuente. Se presenta sobre todo en nios y jvenes y predomina en las reas de poblacin ms desprotegidas. Es secundaria habitualmente a TB
de ganglios, huesos y articulaciones. Su topografa habitual es donde hay ganglios, como regiones supraclaviculares, las axilas, las ingles, uni o
bilateral. Puede estar en regin esternal, codos, rodillas. Las lesiones son siempre ndulos y gomas, refieren pequeas bolitas en el cuello, es decir
ganglios infartados, no dolorosos, no mviles, un buen da se fijan a la piel y esta se pone eritematosa e infiltrados y se inicia la formacin de un
ndulo, que se reblandece y se abre al exterior dejando salir pus de color amarillo claro y espeso, pronto toda la regin se convierte en un plastrn
endurecido (ndulos, gomas, abscesos fros), es posible que dejen cicatrices deformantes y retractiles. Puede haber febrcula vespertina o franca
fiebre, anorexia y adelgazamiento, y a veces sntomas de TB activa. TB VERRUGOSA: A menudo inoculada en personas que manejan material
contaminado como mozos de anfiteatro, matanceros, carniceros, laboratoristas. Se localiza en partes distales, de os miembros como los pies y las
manos y algunas veces en nalgas. En sitio de inoculacin aparece el tubrculo anatmico, que es ndulo verrugoso, que es punto de partida de las
lesiones que son siempre placas verrugosas o vegetante de tamaos variables circulares u ovales, bien limitadas y cuya superficie siempre es spera,
con costras melicericosanguineas, a veces se observan pstulas. En algunas placas se ve cicatrizacin central con crecimiento perifrico y puede
haber compromiso de los linfticos superficiales y profundos de la regin y producirse una estasis linftica que ocasiona edema y ms
verrugosidades. TB LUPOSA: Era muy frecuente en el siglo pasado. Es ms frecuente en nios y jvenes y su clsica topografa es en la cara; mejillas
y dorso de nariz, disposicin en mariposa, tambin puede afectar a los pabellones auriculares. La lesin fundamental es el ndulo pequeo
llamado lupoma que se cubre de escamas y verrugosidades que lo ocultan, de tal manera que se forman placas eritematosas, escamosas,
verrugosas, circulares, bien limitadas de crecimiento perifrico y cicatrizacin central y con tendencia a la ulceracin. En el centro solan producir
extensas y profundas destrucciones, el subtabique nasal se destrua y la nariz tomaba el aspecto de pico de loro. TB MILIAR AGUDA: Muy rara se
presentaba en nios y jvenes con TB avanzada y baja reactividad a los antgenos. Son pequeos ndulos rojizos acuminados, a veces ulcerado y
cubiertos de costras en diferentes partes del cuerpo, en especial en el centro de la cara, que cicatrizan espontneamente. TB NODULAR
PROFUNDA: Descrita por Bazin, con el nombre de eritema indurado, casi exclusiva de mujeres de localizacin nica en las piernas sobre todo en las
caras posteriores y caracterizada por ndulos profundos, de evolucin crnica y recidivante, muy dolorosos y que en su evolucin dejan zonas
atrficas depreimidas que deforman las piernas. TB NODULO NECROTICA: Es menos frecuente que la nodular profunda, aparece en personas
jvenes con topografa en partes salientes; codos, rodillas, nalgas y en ocasiones en la cara, sobre todo en los pabellones auriculares. La lesin es un
pequeo ndulo que sufre una necrosis central y as se cubre de una costra negruzca que al caer deja una cicatriz varioliforme. Son asintomticos y
evolucionan por brotes. TB MICRONODULAR: Es muy rara y pasa inadvertida, son ndulos de 1-2mm que forman placas de aspecto folicular en las
regiones lumbares u otras partes del tronco, son asintomticas. DIAGNOSTICO: Es ms bien de eliminacin, dado que la comprobacin del agente
causal no es fcil en todos los casos. El laboratorio nos puede ayudar sobre todo para descartar procesos que suelen semejarse a las lesiones
tuberculosas. Hallazgo del bacilo; puede hacerse por medio de baciloscopia y de biopsia difcil de encontrar en TB colicuativa y ulcerosa. El cultivo en
medios apropiados como Lowestein es difcil de lograr. Histopatologa; por definicin la imagen de las lesiones tuberculosas es el clsico granuloma
tuberculoide, constituido por linfocitos, clulas epiteloides y clulas gigantes tipo Langhans, resultado de la unin de las segundas., sin embargo no
es patognomnica de la tuberculosis, por lo que solo ser sugestiva de tuberculosis. Intradermoreaccin con PPD; no es diagnostica ya que solo nos
indica un contacto con el bacilo de Koch y un alto porcentaje de la poblacin da positivo. Tiene importancia cuando es negativa, lo cual significa que
no tienen un cuadro de origen tuberculoso. TRATAMIENTO: Se usan las mismas drogas que en la tuberculosis en general y a dosis semejantes. La
estreptomicina es muy til a dosis de 1 gr diario o cada tercer da, 50-60grs como dosis total. La isoniacida que se dice bacteriosttica se usa a 58mg/kg. El etambutol a dosis de 15-20mg/kg, es ms costoso y tienen efectos colaterales en nervio ptico, es reversible. Rifampicina a dosis de 1020 mg/kg, para un adulto se aconsejan 600mgs, para nio la mitad. No usar monoterapia. Tratamiento ideal es; 1 gr de estreptomicina cada tercer
da, 600mgs de isoniacida diarios o bien estreptomicina + etambutol. La TB colicuativa tarda de 3-6 meses en curar. La TB verrugosa y luposa
requieren menos tiempo. En la actualidad se est aplicando con xito un triple tratamiento, isoniacida, rifampicina, pirazinamida.
CASO CLINICO
Nia de 10 aos que consulta por una lesin en mejilla izquierda
seguida de la aparicin de adenopata cervical, de dos meses de
evolucin. Al examen fsico, se observa una pequea lcera de 1 cm
de dimetro, con leve descamacin perifrica y adenopata satlite.

Cual es la conducta a seguir?


RESPUESTA
a.- Biopsia de la lesin.
b.- PPD.
c.- BAAR.
d.- Cultivo de la lesin.

PREGUNTA

VITILIGO
CIENCIAS BASICAS: La triada de enfermedades con discromas son: melasma, ptiriasis alba, viltiligo. El vitligo esuna enfermedad crnica,
asintomtica, de causa desconocida. SALUD PUBLICA: Implicados factores genticos, neurolgicos, autoinmunitarios y psicolgicos. La padece 1-2%
de la poblacin mundial. El 50% se inicia antes de los 20 aos. En Mexico ocupa el 3ro-5to lugar como causa de consulta. Predomina mas en la mujer
y es raro despus de los 50 aos. CLASIFICACION: Generalizado; afecta mas del 75% de la superficie corporal total. Localizado; afecta a un segmento
corporal. Diseminado: afecta a 2 o mas segmentos, pero menos del 75% de la superficie corporal total. PATOGENIA: Existen teoras como las gnicas:

CURSO ENARM CMN SIGLO XXI TEL: 36246001

Pharmed Solutions Institute

PGINA 334

MANUAL DE TRABAJO DEL CURSO ENARM CMN SIGLO XXI


autosmico dominante, antecedentes familiares 40%. Teoria neuroectodermica; origen comn, fatores neuroendocrinos en hipfisis que
interrumpieran la melanogenesis. Teoria inmunolgica, se encontraron anticuerpos antimelanocitos, no confirmado. Teoria psicosomtica;
tendencia a la neurosis, dificultad para establecer relaciones. DIAGNOSTICO: Caracterizada por manchas hipocromicas y acromicas, cambio de color
en la piel, tonalidad blanco, mate uniforme, limites, no palpable, caf, blanco, rojizo, con bordes circunscritos, <1cm, diversos tamaos, figuras
irregulares, hay isloes de piel sana, manchas efelediformes, por lo general sin afeccion sistmica. La topografa es simtrica, afecta dorso de las
manos, muecas, antebrazos, contorno de ojos y boca, piel cabelluda, cuello, zona de genitales, pliegues de flexion, pueden estar restringidas a un
solo segmento (nuca, axilas, cintura, pene, vagina). Puede haber leucotriquia o poilosis, puede afectarse piel cabelluda, cejas, pestaas, vello
corporal. Hay que hacer dx. Diferencial con el Nevo de Suton. TRATAMIENTO: GPC: esteroide tpico potencia alta o muy potente menos a 2 meses
Tambien psoralenos y PUVA. En vitligo simtrico en adultos: pimecrolimus, cara, cuello, genitales. Fotoquimioterapia: N-UVB, PUVA mas Vit. D.
Tratamiento quirrgico: paciente sin actividad de las lesiones, fenmeno de Koebner (aparicin de lesiones tpicas en lugares que han sufrido
traumatismo: cicatrices, quemaduras). Tratamiento despigmentizantes: solo adulto vitligo severo, ter monobenzyl, hidroquinona methoxiphenol.
Psoralenos, ac.acetico, ac. Retinoico, ansiolticos, psicoterapia, esencia de lima, melanidina, PUVA psoralenos, luz UV. Esteroides tpicos baja
potencia. PRONOSTICO: Impredecible, variable, incurable, repigmenta en ocasiones. MAL DEL PINTO: Hacer dx. Diferencial; enfermedad
transmisible no venrea, crnica benigna, ERRADICADA, producida por Treponema herrejoni, este entra en la piel, de 7 dias a 3 meses aparece un
chancro pintoso, dura unos 5 meses, a los 12 meses lesiones diseminadas, se instalan lesiones discromicas permanentes, no se cura sola, antes
conocidas como jiotes, aparecen en piernas, pies, cara, antebrazos, papula de 1-3cm, rosadas escamosa, luego placa oval eritematoescamosa, halo
hipocromicos. Las lesiones tardas manchas leucomelanodermicas triangulo acromico.
CASO CLINICO
Nio de 7 aos, sin antecedentes de inters, es traido a consulta
porque desde hace 3 das le han notado unas manchas blancas cerca
de los genitales sin ningn otro sntoma. Exploracin fsica: Tres
mculas hipocrmicas, redondeadas, sin descamacin ni signos
inflamatorios asociados, a nivel de raz de pene y pubis, de 1-2 cm de
dimetro, piel aparentemente intacta, y una pequea rea de
leucotriquia en ceja izquierda. No adenopatas regionales ni
sntomas generales. Las lesiones aumentan en nmero y tamao en
las siguientes semanas, se extienden a lnea media perianal y ano,
con la lmpara de Wood se ve que las lesiones contrastan. Pruebas
complementarias: Hemograma e ndices eritrocitarios, bioqumica,
anticuerpos
(AC)
antitiroideos
(antimicrosomales
y
antitiroglobulina), TSH y T4L. Todo fue normal.
PREGUNTA
Cul es el diagnstico ms probable para este paciente?
RESPUESTA
a.- Pitiriasis versicolor
b.- Pitiriasis alba

c.- Vitligo
d.- Albinismo
PREGUNTA
Cul es la conducta teraputica ms adecuada para este caso?
RESPUESTA
a.- Hidrocortisona
b.- Sulfato de cobre
c.- Vioformo
d.- Mupirocina
PREGUNTA
La incidencia de esta
aproximadamente de?
RESPUESTA
a.- 10-20%
b.- 1-4%
c.- 8-10%
d.- 6-8%

patologa

nivel

mundial

es

HERPES SIMPLE
CIENCIAS BASICAS: Los virus de herpes se encuentran ampliamente distribuidos en la naturaleza y la mayora de las especies animales son
hospederos naturales de ms de uno. Se han aislado y caracterizado ms de 100, varios de los cuales afectan al humano: herpes simple tipo 1 y tipo
2 (VHS-1, VHS-2), varicela zoster (VVZ), citomegalovirus (CMV), Epstein Barr, (VEB), virus herpes humano 6 (VHH6), virus herpes humano 7 (VHH7),
virus herpes humano 8 (VHH8) y otros. SALUD PUBLICA: La infeccin por VHS-1 es frecuente en sitios hacinados y con condiciones precarias de
higiene se tienen porcentajes de 90% de la poblacin tienen anticuerpos antivirales El ms frecuente en nios. El VHS-1 en ubicacin oral se
transmite por saliva, besos, por compartir vasos, cepillos de dientes y en otras partes del cuerpo se debe a contacto del virus con la piel, se
autotransmite con frecuencia, principalmente a los ojos. La infeccin por VHS-2 depende de la actividad sexual. El VHS-2 se transmite por
secreciones vaginales, contacto sexual (prepuberes) y al neonato durante el paso por el canal de parto infectado. La posibilidad de que VHS-1 y VHS2 establezcan infecciones latentes con recidivas asintomticas favorece su transmisin, ya que un individuo infectado puede ser transmisor durante
toda su vida. Los virus infectantes se encuentran en el lquido de las vesculas. El 50% de los infectsdos por VHS 2, permanecen asintomticos.
PATOGENIA: La patogenia de VHS-1 y VHS-2 es similar, entran en contacto con una mucosa o piel con solucin de continuidad, el virus se adhiere a
la celula husped, se fusiona la envoltura con la membrana plasmtica y la capside entra en el nucleo celular. En el nucleo se da la transcripcin del
genoma viral, replicacin del DNA y ensamblaje de nuevas capsides, posterior es la diseminacionm en sitio de primoinfeccion, neuronas sensitivas a
travs del axn, nucleo neuronal y ah se queda en estado de latencia. La reactivacin es peridica, el virus es llevado a tejidos perifricos por
transporte axonal, con infeccin primaria generalmente asintomtica. El tipo de infeccin que resulta depende del estado inmune del individuo; los
sujetos susceptibles desarrollan infeccin primaria despus de la primera exposicin al virus. Sujetos seropositivos pueden ser reinfectados con virus
de otro tipo. VHS-1 y VHS-2 se transmiten por diferentes vas e infectan diferentes sitios del cuerpo. A grandes rasgos se considera que el HSV-1
infecta de la cintura para arriba y el VHS-2 de la cintura para abajo, sin embargo esta diferenciacin no es estricta. DIAGNOSTICO: La primoinfeccin
por VHS-1 (ORAL): Lesiones vesiculosas, agrupadas en racimo, sobre una piel eitematosa, ulceras y costras. Infeccion rpimaria; Fuente de contagio
familiar, puerta de entrada erosion o araazo, mas susceptible el nio, se presenta fiebre, malestra general, adenopatas, se localiza en cavidad oral,
ojo, extremidades. RECURRENCIA; debida a que el virus esta latente, conjunto con enfermedades debilitantes, infecciones, fiebre, estrs,
inmunosupresin, exposicvion solar, se localiza en cara, labios, regin perioral, mejillas, va precedido de dolor, quemazn o disestesias, SIN fiebre,
adenopatas, ni ataque al estado general. GINGIVOESTOMATITIS HERPETICA; Fiebre hasta 40C, odinofagia, lesiones ulceradas en la mucosa bucal y
gingival, adenopatas, halitosis, malestar general, dificultad para la alimentacin. Periodo de incubacin de 2-12 dias, duracin de sintomatologa de
2-3 semnas, lesiones bucales son vesculas-ulceras supoerficiales de base eritematosa, no pasan por fase de costra, un tercio de los pacientes
sintomticos hacen viremia. Infeccion primaria en adolescentes FARINGITIS O SX. MONONUCLEOSICO: lesiones ulceradas de base eritematosa en
amifgdalas y adenopatas submandibulares. Recurrencia; lesiones vesiculares orolabiales (borde del labio). Prodromo de 6hrs: dolor urente,
hormigueo, prurito, lesiones duran 48hrs, progresan a ulceras y en 3-4 dias costras, recuperacin completa en 10 das. PANADIZO HERPETICO:

CURSO ENARM CMN SIGLO XXI TEL: 36246001

Pharmed Solutions Institute

PGINA 335

MANUAL DE TRABAJO DEL CURSO ENARM CMN SIGLO XXI


Lesiones cutneas vesiculosas que afectan al pulpejo y regin periungueal de un dedo, hay dolor y puede haber adenopatas satlites, es
autoinoculativo, duracin mayor de 2 semanas, es frecuente en personal de salud. QUERATOCONJUNTIVITIS: Unilateral o bilateral, hay adenopata
preauricular, fotofobia, lagrimeo, edema palpebral, quemosis, lesiones tipo ramificacin dendrtica, puede progresar a ulcera corneal, opacificacion y
ceguera, recuperacin mayor a un mes, recurrencias comunes unilaterales. IINFECCION NEONATAL: via de contagio, transplacentaria, genital
(ascendente o canal de parto), infeccion primaria antes de las 20 SDG: TORCH. Herpes simple neonatal: 90% primoinfeccion materna durante la
gestacin, adquisiscion in utero. INFECCION CONGENITA: lesiones vesiculares y reas de cicatrizacin fibrosa en la piel, coriorretinitis, restriccin del
crecimiento, microcefalia o hidroanancefalia, compromiso heptico, pronostico mortal. ENCEFALITIS: Con o sin lesiones cutneas, en la 3ra semana
de vida, se presenta en 30% de neonatos con infeccion por HSV, mortalidad de 50%, y secuelas neurolgicas en un 100%. El aspecto clnico es muy
caracterstico con la aparicin de lesiones vesiculosas, agrupadas en racimo, sobre una piel eritematosa, que con el tiempo dan lugar a una costra. El
cuadro puede ir precedido de sensacin prodrmica de dolor, quemazn o disestesias de difcil descripcin e interpretacin por parte de nuestros
pacientes y no suele acompaarse de fiebre, mal estado general, ni adenopatas. VHS-2 (GENITAL): En general no ocurre en edad peditrica, cuando
aparece en un menor debe investigarse la posibilidad de abusos sexuales. La primoinfeccin es superponible a las formas orofacilaes, pero en la
regin genital. El cuadro tambin es agudo se acompaa, de dolor, fiebre, dificultad para la miccin y adenopatas. Las lesiones vesiculosas, erosivas,
tpicamente agrupadas en racimos nos darn sospecha diagnostica. El diagnstico de laboratorio econmico y rpido se realiza mediante la prueba
de Tzank que consiste en hacer una impronta de las clulas y teirlas con el colorante de Wright o Giemsa, y observar clulas fusionadas con varios
ncleos, sincitios, as como inclusiones nucleares de Cowdry. No es posible confirmar la presencia del virus por medio de esta tcnica ya que otros
virus producen el mismo efecto en las clulas, inclusive el VVZ. Actualmente se hace uso de tcnicas inmunoenzimticas, biolgicas, bioqumicas y de
biologa molecular para detectar anticuerpos o antgenos virales. La deteccin de anticuerpos solo es de utilidad para deteccin de la primo infeccin
y estudios epidemiolgicos. Para el diagnostico hacer aislamiento del virus en cultivo celular, es el mtodo diagnostico definitivo, raspado de
lesiopnes o secreciones genitales. Inmunoflorescencia directa con anticuerpos monoclonales. Tioncion de Tzank o papanicolao, baja sensibilidad y
especificidad. En RN cultivo viral de nasofaringe, conjuntiva ocular, orina LCR. TRATAMIENTO: Lavado de manos con agua y jabon, evitar reinocular
el virus o infectar a otras personas. Terapia estndar Aciclovir, acorta el periodo sintomtico y la duracin de eliminacin del virus.
Gingivoestomatitis Aciclovir 15mg/kg/do (max 200mg), VO 5 veces al dia por 5-10 dias, valaciclovir. Recurrencia de herpes labial; Aciclovir 1015mg/kg/do cada 8 hrs por 5 dias, valaciclovir. Panadizo aciclivor 200mgs 5 veces al dia por 10 dias. Herpes neonatal 20mg/kg/do IV c/8hrs por 1421 dias. Encefalitis; Aciclovir 15mg/kg/do cada 8 hrs por 21 dias, criterio para termino de tratamiento PCR negativa para herpes en LCR. Herpes
genital primer episodio y recurrencias; Aciclovir 200mg 5 veces al dia 0 40mg 3 veces al dia, valaciclovir 1 gr 2 veces al dia, sirve para impedir la
replicacin no su eliminacin. Tratamiento tpico; fomentos con sulfato de cobre 1/1000 o sulfato de zinc 1/100, antibitico tpico.
COMPLICACIONES: La ms frecuente es la impetiginizacion o sobreinfeccin bacteriana, fundamentalmente estreptoccica y estafiloccica. Las
lesiones oculares en particular, la queratitis puede llevar a ceguera.
CASO CLINICO
Femenino de 15 aos de edad la cual acude a consulta por presentar
desde hace dos das, vesculas agrupadas en el labio inferior, que se
transforman en pustulas y costras hemorrgicas, cuenta
antecedente de prurito y sensacin de hormigueo en la zona.
Comenta la paciente que en el ltimo ao se han repetido episodios
de vesiculas similares en varias ocacion, antecentes de importancia
actualmente embarazo de 22 semanas, ha tenido multiples parejas y
es madre soltera.
PREGUNTA
Cual es la conducta a seguir mas adecuada?
RESPUESTA
a.- Tratamiento antiviral doble esquema.
b.- Realizar USG para identificar anormalidades.
c.- Tratar y esperar parto nornal.
d.- Tratamiento especifico y programa cesarea oportuna.
CASO CLINICO
Nio de 10 das de vida RNT (39 semanas). Peso: 3.900 kg nacido de
parto vaginal. Madre de 27 aos sana. Padre de 30 aos cursando un
cuadro gripal. Consulta en la guardia por presentar fiebre 38 y 3
lesiones eritematovesiculosas en el trax y 3 cerca de cavidad oral. El
nio se encontraba en buen estado general, sin antecedentes
perinatales significativos.

PREGUNTA
Cul sera la conducta diagnostica ms adecuada a seguir en este
caso?
a.- Cultivo viral del recin nacido (lesiones cutneas)
b.- Cultivo endocervical de la madre en bsqueda de Herpes Simplex
c.- Test de Tzanck
d.- Tincin de Gramm
PREGUNTA
De haber ocurrido la infeccin primaria antes de la 20SDG. Qu
patologa sera ms probable de sospechar?
RESPUESTA
a.- Herpes simple tipo I
b.- Herpes simple tipo II
c.- Herpes neonatal
d.- TORCH
PREGUNTA
Qu manifestaciones clnicas sera menos probable observar en un
recin nacido que curso con infeccin congnita?
RESPUESTA
a.- Coriorretinitis
b.- Microcefalia
c.- Acropaquias
d.- Restriccin del crecimiento

HERPES ZOSTER
CIENCIAS BASICAS: El virus de la varicela (VVZ) origina varicela y su reactivacin causa herpes zoster. SALUD PUBLICA: Ms frecuente en el adulto
que en la infancia (0.74 casos por 1000 individuos al ao). Suele ocurrir en nios que pasaron varicela precoz (primeros 2 aos de vida). El virus se
transmite por contacto directo, por gotitas de aerosol y por va rea en comunidades; tambin puede haber contagio por contacto con vesculas
cutneas. Es altamente transmisible. Se dispone de eficaz vacuna atenuada. PATOGENIA: Se caracteriza por una infeccin neurolgica y
dermatolgica, posterior a un episodio de varicela (primoinfeccin) en los ganglios sensitivos espinales y craneales (donde esta latente el VVZ). Una
vez reactivado, el virus se mueve a lo largo de las fibras sensitivas hasta l rea de piel que inervan, que usualmente afectan de 1-3 dermatomas.
Trigmino rama oftlmica- causa queratitis o amaurosis, en ganglios geniculados causa perdida de capacidad gustativa y paralisis facial. Dicha
reactivacin ocurre, por lo general, una vez en la vida, sobre todo en pacientes de la tercera edad o en aquellos inmunodeprimidos (por HIV, cncer,
tratamiento con corticoesteroides, frmacos inmunosupresivos, por ejemplo). DIAGNOSTICO: Las lesiones cutneas consisten en lesiones papulosas
iniciales agrupadas en racimos sobre la piel eritematosa, las ppulas se convierten en vesculas en 12-24 hrs, estas pueden confluir y formar ampollas
y luego estas se rompen y dan lugar a costras.unilateral, rara vez rebasa la lnea media, dolor urente, aumenta con la edad. La erupcin cura en 7-14

CURSO ENARM CMN SIGLO XXI TEL: 36246001

Pharmed Solutions Institute

PGINA 336

MANUAL DE TRABAJO DEL CURSO ENARM CMN SIGLO XXI


das y suele acompaarse de dolor. El dolor puede ser prodrmico, es decir preceder a las lesiones cutneas, acompaarlas e incluso persistir
durante meses tras la resolucin clnica dermatolgica, en este ltimo supuesto se habla de neuralgia postherptica, esta implica la prdida de
autonoma y una calidad de vida pobre. La infeccin por varicela puede ocasionar ocasionalmente que el virus atraviese la placenta, dando lugar al
sndrome de varicela congnita que puede ser leve o severo. Cuando la madre ha estado en contacto con casos de varicela es recomendable
determinar el nivel de anticuerpos anti-varicela. El diagnstico se basa en la prueba de Tzank, con las dificultades mencionadas; el virus se puede
aislar de las vesculas y recientemente se utilizan tcnicas de biologa molecular para su identificacin. COMPLICACIONES. La neuralgia
postherptica. En individuos inmunocomprometidos puede dar lugar a enfermedades progresivas y graves, principalmente encefalitis post-infeccin
y neumona, en algunos casos fatales. TRATAMIENTO: Resultados satisfactorios se han obtenido con inhibidores de la sntesis del ADN (aciclovir y
derivados), sin embargo se requieren dosis ms altas que las recomendadas para HSV-1 y 2, clorfenamina, hidroxicina, Aciclovir 800mgs VO 4 veces
al dia. HERPES ZOZTER EN LA INFANCIA: Las principales caractersticas son: 1. Suele presentarse en nios que presentaron varicela en los 2 primeros
aos de vida. 2. Su curso es benigno y sin complicaciones. 3. Es poco doloroso (no prdromos), no suele complicarse con neuralgia postherpetica. 4.
Se recomienda investigar existencia de inmunodepresin. 5. El tratamiento oral con antivirales solo est indicado en casos seleccionados, pero debe
ser lo ms precoz posible.
CASO CLINICO
Masculino de 6 meses de edad quien manifest una dermatosis
diseminada al tronco, regin inguinal derecha y la cara lateral
interna del muslo derecho en el tercio distal, asi como en el tercio
medio y proximal de la pierna. La dermatosis era unilateral,
constituida por vesculas agrupadas sobre una base eritematosa y
escasas costras hemticas que confluida entre si, formando varias
placas de tamao variable y de evolucin aguda y dolorosa, se
realizo diagnostico clnico de herpes zoster.
PREGUNTA
Cual es la conducta a seguir menos apropiada?
RESPUESTA
a.- Aciclovir 12 mg/kg/dosis.
b.- Fomentos con sultafo de cobe y oxido de cinc.
c.- Profilaxis antibitica profilctica.
d.- Cerco sanitario a contactos.
CASO CLINICO
Paciente masculino 70 aos de edad, con dolor en zona lateral
externa de muslo derecho, irradiado a rodilla. Sin traumatismo
previo. Antecedentes: Ex fumador (Abuso de tabaco), hemorragia en
conjuntiva por traumatismo, prostatitis crnica focal, catarata senil.
Dolor de 3 dias, de evolucin en miembro inferior derecho, el cual se
intensifica por la noche y es urente, durante el da ha cedido. Niega
prdida de fuerza muscular. Nunca le ha sucedido. EF: se observan
vesculas (pequeas), agrupadas en racimos que siguen trayecto de

nervio crural y citico, rodeadas de un halo rojo, hipostesia a nivel de


L2-L3 (cara externa de muslo derecho). Lasegue (-). Reflejo patelar
normal.
PREGUNTA
Cul es la etiologa ms probable para este caso?
RESPUESTA
a.- Virus de herpes simple
b.- Poxvirus
c.- Virus de herpes zoster
d.- Virus de inmunodeficiencia humana
PREGUNTA
Cules son las complicaciones ms probables en este caso?
RESPUESTA
a.- Encefalitis post-infeccin, neumona, neuralgia
b.- Neuralgia, insuficiencia renal, neumona
c.- SIDA, dermatomicosis, neuralgia
d.- Encefalitis post-infeccin, insuficiencia renal
PREGUNTA
Cul es la conducta diagnostica ms definitiva a seguir?
RESPUESTA
a.- Tincin de Gramm
b.- Prueba de parche
c.- Aislamiento de virus en cultivo celular
d.- Prueba de Tzanck

VERRUGAS VIRALES
CIENCIAS BASICAS: Son los virus del papiloma humano capaces de producir las verrugas virales y algunos cuadros relacionados con la
carcenognesis. Se han descrito ya 25 tipos de virus. La palabra verrugosidad indica solamente una lesin dura, anfractuosa, seca que
histolgicamente presenta hiperqueratosis y papilomatosis. La vegetacin en cambio es blanda, anfractuosa, friable y solo presenta la papilomatosis,
por ello estas lesiones solo se presentan en zonas hmedas de la piel y en mucosas. Las verrugas virales son neoformaciones epiteliales benignas que
pueden afectar a cualquier persona, nios, adultos, hombres y mujeres, constituyendo una de las 5 entidades que con ms frecuencia se ven en la
consulta. Su transmisibilidad es baja y sabemos que son autoinoculables. Un estado inmunolgico deteriorado permite sui extensin y persistencia.
VERRUGAS VULGARES: Son los populares mezquinos y son las ms frecuentes. Predominan en los nios. Son neoformaciones verrugosas solitarias
o mltiples, de superficie anfractuosa, secas, duras, de color de la piel o ms oscuras, semiesfricas y bien limitadas, presentan en su superficie un
fino puntilleo oscuro. Son indoloras a menos que se les traumatice y su desaparicin es espontnea y no deja secuelas, habituales en extremidades
superiores. VERRUGAS PLANAS o juveniles: Son muy pequeas y numerosas, no ms de 1mm y muy aplanadas, levantan de la superficie, as es que
a primera vista parecen machas. De color de la piel, llegan a sumar ms de 100, localizadas de preferencia en las mejillas aunque tambin pueden
verse en extremidades superiores; dorso de manos o antebrazos. Tambin son asintomticas. VERRUGAS PLANTARES: Se les conoce vulgarmente
como ojos de pescado y como su nombre o indica se presentan en la planta del pie, la presin de peso del cuerpo las hunde en la gruesa capa
cornea, por lo que solemos ver la base de pirmide, y ello explica el dolor que las acompaa, nicas o mltiples, con pequeas hemorragias
postraumticas en su superficie. VERRUGAS ACUMINADAS: Tambin llamadas condilomas acuminados o papilomas venreos, la mayora se
adquieren por la relacin sexual, en nios a veces por desaseo. Asientan en sitios hmedos y calientes, alrededor de orificios naturales; glande,
labios mayores y menores, ano recto, boca, no se produce hipertrofia como en otras verrugas, la lesin toma ms bien un aspecto vegetante, como
coliflor, pueden ser pequeas a penas salientes (crestas en el trmino) o alcanzar dimensiones monstruosas, deformando la regin, se maceran e
infectan y adquieren un olor muy desagradable. Debemos diferenciarlas siempre de condilomas planos o siflides papuloerosivas, muy ricas en
treponemas. Son ppulas aplanadas y erosionadas y no neoformaciones vegetantes, no tiende a persistir, evolucionan hacia la desaparicin. Los
condilomas acuminados no tienden a la involucin, persisten y crecen. VERRUGAS FILIFORMES y digitiformes: Se presentan en los pliegues: cuello
axilas, ingles en forma de neoformaciones alargadas, como hilos oscuros, algunas pediculadas o con prolongaciones transparentes. Se discute su
etiologa viral. PRONOSTICO: Aunque se menciona el papel oncognico de algunas variedades de parvovirus, no debe tomarse a las lesiones
verrugosas comunes como precancerosas, sin embargo en el caso de lesiones acuminadas, se piensa que pueden relacionarse con el cncer

CURSO ENARM CMN SIGLO XXI TEL: 36246001

Pharmed Solutions Institute

PGINA 337

MANUAL DE TRABAJO DEL CURSO ENARM CMN SIGLO XXI


cervicouterino y se ha observado que las mujeres que presentan esta neoplasia tienen con ms frecuencia verrugas acuminadas. TRATAMIENTO:
Involucin excepto en las acuminadas.Los mezquinos involucionan espontneamente. La destruccin de pocas lesiones puede hacerse con
electrofulguracin o congelndolas con nitrgeno lquido. Las acuminadas requieren aplicacin de soluciones de podofilina al 20-40% en solucin
alcohlica o en aceite, protegiendo tejidos vecinos con vaselina o colodin. Igualmente son molestas las elctrofulguraciones de las verrugas
plantares, pues solo se destruye una mnima porcin y es intil poner al paciente en cama varios das, cuando la recidiva es la regla. Ya se sabe que
las verrugas tienden a la involucin espontneamente, est probado cientficamente y con estudios estadsticos. Debe tenerse en mente que las
verrugas virales tarde o temprano desaparecern, no as las cicatrices que perduran toda la vida.
CASO CLINICO
Se trata de masculino de 17 aos con dermatosis localizada en cara,
rea de la barba con extensin al cuallo constituida por numerosas
neoformaciones (90) que varia de 1 a 3 mm de dimetro con
proyecciones digitiformes que dan aspecto verrugoso y papiliforme,
del color de la piel, solo presentaba prurito ocasional, estudios de
rutina dentro de parmetros normales.

PREGUNTA
Cual es la consuta a seguir mas adecuada?
RESPUESTA
a.- Electrofulguracion.
b.- Nitrgeno lquido.
c.- Podofilina.
d.- 5-fluorouracilo.

MOLUSCO CONTAGIOSO (MC)


CIENCIAS BASICAS: Es una patologa cutnea benigna ocasionada por un virus DNA de la familia de los poxvirus. SALUD PUBLICA: El MC tiene una
distribucin mundial y una incidencia de 4-5%. Los estudios epidemiolgicos realizados sugieren que la transmisin podra estar relacionada con
factores tales como la humedad y calor del clima, la pobreza, hacinamiento y la falta de higiene. Sin embargo, en otros trabajos se apoya ms la idea
del contacto entre personas, ya que se ha encontrado que se puede desarrollar en el 35% de los miembros expuestos de una familia y que en la
mayora de los casos las familias poseen excelentes estndares de higiene. En nios la transmisin se produce por contacto piel-piel, por fmites o
por auto inoculacin. La infeccin por MC ocurre en todos los grupos etreos, y al parecer ha venido aumentando en la ltima dcada. La mayor
incidencia se observa en nios menores de 5 aos, por el mayor contacto fsico casual entre ellos, y la auto inoculacin. La infeccin en lactantes es
rara, esto se explica por la persistencia de anticuerpos maternos. Otro pico en la incidencia se observa en adultos jvenes, debido a la propagacin
secundaria al contacto sexual. El MC est incrementndose cada vez ms dentro de las enfermedades diagnosticadas en la poblacin sexualmente
activa. La transmisin en adultos normalmente es va sexual. Esta va se apoya en la localizacin mayoritaria encontrada en el rea genital, la
frecuencia de encontrar las parejas afectadas. Otras vas posibles han sido a travs de saunas, masajes, ciruga. La prevalencia en poblacin con VIH,
es tan alta como del 5-18%. PATOGENIA: El virus del MC es uno de los ms grandes que causan enfermedad en los humanos, el vibrin maduro es
una partcula que mide 150x350 mm2. El genoma, al igual que en los dems poxvirus, es una nica molcula lineal de DNA de doble hlice. Estudios
recientes del DNA de los molluscum aislados confirman la presencia de dos diferentes subtipos, MCV I y MCV II, con genomas de 185 kb y 195 kb
respectivamente. Las lesiones producidas por cada subtipo son indistinguibles. Todos los MC aislados de un mismo paciente pertenecen al mismo
genotipo as como los aislados de los miembros infectados de su familia. Se ha observado la existencia de MC en pacientes inmunocomprometidos.
La mayora de los pacientes muestran una deficiencia en su funcin o nmero absoluto de linfocitos T. Un grupo heterogneo de enfermedades se
han descrito en asociacin al MC: atopia, neoplasias, estados yatrgenos de inmunosupresin, los pacientes atpicos un incremento en la
susceptibilidad a la infeccin cutnea viral poda estar relacionado con la piel eccematosa que abrira una puerta de entrada a la colonizacin vrica.
DIAGNOSTICO: El periodo de incubacin de la infeccin es de 19 a 90 das (7dias a 6 meses en RN). Las lesiones se inician como neoformaciones
(ppulas) que miden generalmente de 2 a 6 mm, aunque pueden llegar a medir 3 cm, son hemisfricas, cupuliformes, lisas, del color de la piel o
perladas, traslucidas, algunas (20%) tienen una umbilicacin central; la base es levemente eritematosa y son de consistencia firme, cuando se
rompen,algunas pueden contener material blanquecino y cremoso. Se localizan en cualquier parte del cuerpo e incluso pueden afectar mucosas,
generalmente se agrupan en un rea especfica, pero pueden estar diseminadas en personas infectadas con el virus de la inmunodeficiencia humana
adquirida, siendo un marcador de enfermedad avanzada, el material es blanquecino y cremoso. En el caso de los nios las lesiones se localizan
normalmente en cara, tronco, brazos y piernas a diferencia de los adultos jvenes, las lesiones tienen predileccin por genitales, abdomen y cara
interna de los muslos. Se pueden propagar por auto inoculacin, pero es tpico que se resuelvan espontneamente en pocos meses. Las lesiones son
asintomticas en la mayora de los pacientes, aunque en el 10% de los casos puede haber prurito y desarrollarse una reaccin eccematosa. El
diagnstico se hace clnicamente y en algunos casos dudosos puede efectuarse biopsia con tincin H-E donde se encuentran los cuerpos de molusco
(inclusiones intracitoplasmticas grandes) o de Hendersen-Paterson; el 90% de los pacientes posee Ac tipo IgG. Puede realizarse microscopa
electrnica, PCR, Elisa, e inmunohistoqumica. TRATAMIENTO: Enfermedad autolimitada que eventualmente se resolver en la mayora de los
pacientes pero en nios atpicos e inmunosuprimidos la evolucin es larga y trpida. Primera eleccin es imiquimod 5% crema cada 2 dias hasta
que desaparezcan las lesiones (aprox. 4 sem). Curetaje: Es un mtodo sencillo y disponible de forma rpida. Se realiza con una cureta simple,
doloroso por lo que se recomienda el uso de algn anestsico tpico previo a la realizacin del mismo. Tiene la ventaja de proveer de una muestra
de tejido para la confirmacin del diagnstico en caso de dudas. Criociruga: Es un mtodo eficiente y rpido para el tratamiento de las lesiones. El
nitrgeno lquido se aplica sobre cada lesin por pocos segundos. Se repiten las sesiones cada 2 a 3 semanas segn sea necesario. Entre las
desventajas se encuentran el dolor, hiper o hipo pigmentacin y cicatriz residual. Ac. Saliclico 15% mas podofilina 25% con vehiculo de colodin
elstico 1 vez por semana. El podofilotoxina es uno de los componentes activos de la resina de podofilina, es ms seguro que la podofilina y puede
ser usado por el paciente en casa. La aplicacin recomendada es 0,05 ml de podofilotoxina al 5% en etanol tamponado en lactato dos veces al da
por 3 das. Est contraindicado durante el embarazo.
CASO CLINICO MOLUSCO
Un varn de 10 aos de edad, nacido del matrimonio consanguneo
fue llevado con mltiples lesiones elevadas blancas como perlas en
todo el cuerpo desde hace 6 meses, asintomticas que se fueron
generalizando. No haba antecedentes de mltiples y recurrentes
lesiones llenas de pus en el pecho, las ingles, junto con infecciones
pulmonares recurrentes. No haba antecedentes personales o
familiares de atopia. No haba antecedentes de ningn dentales,
problemas seos, neurolgicos o cualquier facies distintivas. De los

CURSO ENARM CMN SIGLO XXI TEL: 36246001

tres hermanos, un hermano ms joven tena antecedentes de


lesiones de la piel similar en todo el cuerpo.
PREGUNTA
Cual es la conducta a seguir?
RESPUESTA
a.- Crioterapia y curetaje.
b.- Conducta expectante.
c.- Nitrito acidificado.
d.- Imiquimod tpico.

Pharmed Solutions Institute

PGINA 338

MANUAL DE TRABAJO DEL CURSO ENARM CMN SIGLO XXI


FOLICULITIS
CIENCIAS BASICAS: Inflamacion aguda perifolicular de origen infeccioso, principalmente en reas donde abunda el pelo, con pelo en el centro y no
deja cicatriz. CLASIFICACION: Puede ser primaria o secundaria a escabiosis, pediculsois, rasurado, uso de alquitranes. PATOGENIA: El agente causal
ms frecuente es Staphylococcus aureus, pero tambin pueden infectar bacterias Gram negativas S. Aureus coagulasa negativo, pseudomona
aeuruginosa, pytirosporum. Los folculos pilosos estn en toda la superficie de la piel excepto en palmas y plantas, pero son abundantes sobre todo
en piel cabelluda, zonas de barba y bigote, axilas, pubis. DIAGNSTICO: Es clnico y de ser necesario se realizar gram y cultivo. Se presentan en
cualquier zona pilosa, sobre todo en ambiente hmedo, maceracin, falta de higiene, aplicacin del emoliente oclusiva y drenaje de heridas
adyacentes y abscesos, uso de agentes oclusivos o presencia de infecciones adyacentes, y ms frecuentemente en pacientes con inmunosupresin
local sistmica. Se presentan clnicamente como una pstula de base eritematosa, apenas de unos milmetros, a nivel de cada folculo, en
ocasiones se dejan atravesar un pelo, son de color amarillento se abren rpidamente y dejan salir una gotita de pus que al secarse formara una
costra melicerica, rodeado de halo eritematoso. En general no es un proceso pruriginoso, ni doloroso .Las lesiones se extioenden rpidamente en
superficie y mientras unas van curando, otras nuevas aparecen. Al desaparecer no dejan cicatriz. Se localizan con mayor frecuencia en zonas de pelo
terminal; si la infeccin se extiende en profundidad formar Abscesos. A las formas ms superficiales tambin se les denomina Imptigo de
Bockhart (genera pustulas pequeas en la salida del folculo a menudo piel cabelluda de nios). TRATAMIENTO: En primer lugar eliminar factores
desencadenates (fomentos de sulfato de cobre y aplicacin de toques con solucin yodada al 1%), antispticos (mupirocina, ac. Fusidico, bacitracina)
2-3 veces al dia por <7 dias, antibiticos tpicos y sistmico (para evitar recidiva dicloxacilina 1-2g/dia por 10 dias, penicilina benzatinica 1200 000
cada 8 dias, TMP/SFX, por 3-4 semanas) sabiendo que el agente causal es Staph. Aureus. FURUNCULOSIS: Hacer dx. Diferencial este es una ataque al
folculo mas profundo, violento, intensa reaccin perifolicular, inicia en una zona eritematosa muy dolorosa, forma pustula o pequeo absceso, muy
doloroso. Sale pus y deja al final una cicatriz. Puede haber fiebre o adenopatas. MILIRIA: estado oclusivo no inflamatorio de las glndulas
sudorparas, vesculas pequeas.
CASO CLINICO
Nia de 8 aos de edad. Consult por aparicin en los 3 das previos
de un brote de lesiones papuloeritematosas, algunas de aspecto
seudovesiculoso o pustuloso, pruriginosas, en el tronco (de
predominio en caderas y glteos) y las extremidades inferiores. Con
la sospecha inicial de exantema periflexural asimtrico de la infancia
frente a picaduras, se inici tratamiento con antihistamnico por va
oral y metilprednisolona por va tpica, con mejora aparente de las
lesiones. Seis das ms tarde consult por aparicin progresiva de
nuevas lesiones de similares caractersticas. En la exploracin fsica
present mculas hiperpigmentadas marrones en la zona abdominal
inferior y en la regin anterior de ambos muslos, junto con mltiples
ppulas eritematoedematosas, algunas con centro pustuloso, en la
zona lumbar y en ambos glteos y otras con aspecto vesiculoso en
costado izquierdo. La nia negaba ir a piscinas, o hacer cualquier
tipo de actividad recreativa relacionada con agua.

principalmente. La zona facial y el cuero cabelludo estaban


respetados. La paciente no presentaba fiebre ni afectacin del
estado general. En la anamnesis se pudo constatar el antecedente
de bao en una piscina de vinilo de pequeo tamao, adquirida por
los padres recientemente. El llenado de la piscina se haba llevado a
cabo con agua del grifo y la piscina se hallaba situada en el jardn de
la vivienda. La paciente se baaba en la piscina a diario y a las 48
horas del inicio de los baos comenzaron a aparecer las lesiones.

PREGUNTA
Cul es el agente causal mas frecuente?
RESPUESTA
a.- Staphylococcus aureus
b.- Pseudomona aeruginosa.
c.- Estreptococos
d.- Proteus.

PREGUNTA
Cul es el agente etiolgico ms probable en este caso?
RESPUESTA
a.- Staphylococcus aureus
b.- Pseudomona aeuruginosa
c.- Virus de varicela zoster
d.- Trichophyton tonsurans

CASO CLINICO
Una nia de 9 aos sin ningn antecedente mdico de inters acudi
a nuestra consulta por presentar un cuadro de lesiones cutneas
discretamente pruriginosas. En la exploracin se observaron costras
melicricas y pstulas, muchas de ellas con un halo eritematoso
inflamatorio, a nivel de tronco y gluteos. Se apreci un gran nmero
de lesiones distribuidas en tronco, extremidades y glteos

PREGUNTA
Cul es la conducta teraputica ms adecuada para este caso?
RESPUESTA
a.- Sulfato de cobre ms dicloxacilina
b.- Sulfadiazina argentica mas perxido de benzoilo
c.- Medidas generales y vigilancia
d.- Higiene ms miconazol

PREGUNTA
Cul es el diagnstico ms adecuado para este caso?
RESPUESTA
a.- Foliculitis
b.- Tia corporis
c.- Furunculosis
d.- Varicela

PEDICULOSIS
CIENCIAS BASICAS: Es una ectoparasitosis ocasionada por los piojos, los cuales no reconocen barreras geogrcas, socioeconmicas ni culturales. Es
la parasitacin del hombre y los animales por insectos del genero pediculus del cual existen 3 principales: el P. capitis (pediculosis de piel cabelluda),
el P. vestimenti (pediculosis del vestido y el cuerpo) y el P. pubis o ladilla (en vello pbico). El piojo se alimenta succionando sangre; su saliva
contienen sustancias vasodilatadoras y anticoagulantes. Los huevos del piojo, conocidos como liendres, estn finamente adheridos a pelos
individuales. SALUD PUBLICA: Es un problema cosmopolita. Su alta prevalencia ha sido relacionada, con la limitada disposicin de agua y deficientes
prcticas de aseo personal. Hay preferencia con el sexo femenino relacionado con el cabello largo, ya que estos facilitan las formas de transmisin,
tambin los fmites y contacto sexual. Aproximadamente 6-12 millones de casos/ao se presentan entre nios de 3-12 aos de edad en Estados
Unidos. En Mxico se report una prevalencia entre 18-33%, en algunas poblaciones. PATOGENIA: Son insectos sin alas, depositas de 7-10
huevos/dia, tienen un cemento adherente (liendres), y maduran en 8 dias, desarrollo completo de 17-25 dias, mviles, caminan 23 cm/min,
sobreviven 6-20hrs fuera del ser humano. Topografia: cabeza, cuerpo y pubis. DIAGNOSTICO: PIEL CABELLUDA; en regin occipital y posauricular,
rara vez en barba, prurito intenso y excoriaciones, MUCHAS liendres y pocos parasitos adultos, puede generarse imptigo, linfadenitis occipital
regional dolorosa, erupcin urticariana y maculopapular en hombros y tronco. PEDICULOSIS DEL CUERPO: Tronco y cuello, abdomen, se presentan

CURSO ENARM CMN SIGLO XXI TEL: 36246001

Pharmed Solutions Institute

PGINA 339

MANUAL DE TRABAJO DEL CURSO ENARM CMN SIGLO XXI


ppulas, costras hemticas, manchas eritematosas, excoriaciones lineales, prurito intenso. PEDICULOSIS DEL PUBIS: en regin pubica, mslos, axilas,
pestaas, hay prurito, escoriaciones y se transmiten por contacto sexual. TRATAMIENTO: 1ra eleccin piretrinas naturales o aletrinas y sintticas o
permetrinas. 2.- Organofosforados (malation) y 3.- Organoclorados (lindano), La permetrina al 1% en forma de locin o crema, es el tratamiento de
eleccin para la pediculosos, nico piretroide en crema aprobado por FDA, ha mostrado una gran eficacia y baja toxicidad, su mecanismo de accin
es la neurotoxicidad por prolongacin de la activacin de los canales de sodio causando una despolarizacin sostenida. Otros scabisan shampoo 1%,
aplicar durante 10 minutos y retralo con el bao, repetir de 7-10 dias, por un periodo >2meses, no utilizar en crisis convulsivas, ni en embarzo y
lactancia. Lindano 1% (Herklin), es un inhibidro de neurotransmisin, es de segunda eleccin, hay que colocar la locin en la noche, friccionar,
retirar al dia siguiente posteriormente y aplicar shampoo, repetir al 5to dia. Malathion 0.5%, inhibe la acetilcolinesterasa, en mayores de 6 aos, se
aplica en pelo seco por 8-12hrs, repetir de 7-10 dias. Ivermectina aumnta el flujo de iones de cloro a travs de las memebranas neuronales
causando paralisis, no usar en <15KG y embarazo, la dosis es de 200mcg/kg/dia repetir de 10-20 dias, las tabletas son de 6 mg, no aprobado por FDA.
Para las liendres vaselina 30g mas xilol 30 gotas, uso externo, tambin acido actico (vinagre blanco) 20cc, agua destilada 100cc, uso externo.
CASO CLINICO PEDICULOSIS
Nia de 2 aos que tras una extensa pediculosis en la cabeza tratada
con 2 ciclos de permetrina al 1% en locin acude por parasitacin en
las pestaas. La exploracin mediante lmpara de hendidura pone
de manifiesto la presencia de algunas liendres y de un piojo maduro
(Pediculus humanus capitis) adheridos a las pestaas. Se realiza
retirada directa de gran parte de los parsitos y se inicia tratamiento
con vaselina en pomada 3 veces al da durante una semana. No
vuelve a presentar recurrencias tras el tratamiento.

PREGUNTA
Cual es la conducta a seguir?
RESPUESTA
a.- Permentina 1%.
b.- Piretroides.
c.- Permetrina 5 %.
d.- Malathion 0.5 %.

ESCABIASIS
CIENCIAS BASICAS: Tambin llamada sarcoptosis, sarna, roa. Es una enfermedad altamente transmisible, cosmopolita, cuyo agente etiolgico es el
caro Sarcoptes scabiei var. hominis, del grupo Arachnida, del orden Astigmata. Se le asocia a estratos socioeconmicos bajos en los que adems es
frecuente observar higiene y nutricin deficientes. SALUD PUBLICA: Se transmite principalmente por contacto directo, estrecho (hacinamiento), sin
descartar la evidencia de transmisin por fomites (por ejemplo, ropa de cama o personal en infestaciones severas) y contacto sexual. Debe tenerse
en cuenta el contacto frecuente en algunas instituciones (asilos, escuelas, cuarteles, hospitales). Los perros y gatos y otros animales tambin
padecen la parasitosis. Brotes epidmicos cclicos cada 10-15 aos. Poblacion de riesgo: promiscuidad, invierno, estancias, hospitales, asilos,
inmunocompromiso, VIH. PATOGENIA: El artrpodo adulto mide 1-3 mm de longitud, tiene un cuerpo aplanado, ovalado y 4 pares de patas. La
hembra fertilizada se instala en la superficie de la piel pone de 10-40 huevos que maduran en 1 semana y en un lapso de tiempo menor a una hora
excava un tnel en el estrato crneo y granuloso. Puede desplazarse unos 2-5 cm/min sobre la piel clida. El promedio de vida del artrpodo es de
unos 60 das. La respuesta inflamatoria y los productos del ectoparsito son responsables, en gran medida, del prurito. Las larvas liberadas alcanzan
la madurez unos 15 das despus de la oviposicin; los nuevos adultos copulan y el ciclo se repite; al cabo de varias semanas la infestacin todava es
baja los signos y sntomas leves. Se requieren unos 15-20 caros adultos y su diseminacin para provocar el intenso prurito. DIAGNOSTICO: Los
sntomas iniciales son leves y habitualmente se atribuyen a la picadura de algn insecto. El prurito es un sntoma cardinal, con agravamiento
nocturno (reaccin de hipersensibilidad retardad tipo IV a saliva, huevos, materia fecal de los caros, escoriaciones y costras hemorragicas). El
rascado, que inicialmente alivia al paciente porque destruye los tneles y algunos parsitos se erige como una forma de diseminacin de los mismos
(y elimina las lesiones primarias, constituidas por tneles y vesculas escasas y puntiformes, con caros, y ppulas pequeas, circunscrita, elevada de
bordes bien definidos de contenido solido, <1cm). Los tneles son delgados, elevados, curvados o en forma de S y miden 2-20 mm de longitud. Su
localizacin ms frecuente se encuentra en pliegues interdigitales, muecas, en los bordes de manos y pies, superficies extensoras de codos y
rodillas, pene, escroto, pliegues mamarios e incluso pezones, glteos, axilas, cintura y pliegues poplteos (zonas incluidas por las "lneas de Hebra",
divisiones imaginarias a nivel de hombros y rodillas). En nios pequeos tambin se ubican con frecuencia en palmas de las manos (signos del
cirujano, pustulas y costras melicericas en los dedos lo que impide cerrar las manos), plantas de los pies, cabeza y cuello, con vesculas y pstulas.
Adems pueden presentar ndulos en la zona del paal y en la axilar. En adultos tambin es posible identificar ndulos escabiticos. Las lesiones
secundarias dominan el cuadro clnico y son ocasionadas por el rascado, infeccin bacteriana secundaria y la automedicacin: pstulas,
excoriaciones, eccema, costras. Se desarrollan ndulos en codos, axilas, pene, escroto (masas firmes, oscuras, que persisten durante meses an en
ausencia de infestacin activa). Datos de eczema inflamatorio se atribuyen a la automedicacin. En los lactantes pede ser generalizado, aunque
respetan cara si hubiera lesiones serian IDES. Diagnstico epidemiolgico: se puede tener como apoyo la prueba de cinta adhesiva, raspado de la
piel, biopsia por rasurado, microscopia de bajo poder, dermatoscopia. Cuadros similares en el ambiente familiar o comunal, en instituciones. Biopsia
de piel. Infiltrado inflamatorio compuesto por eosinfilos, linfocitos, histiocitos. Pueden llegar a observarse larvas, ninfas, hembras. Se ha reportado
el empleo de PCR en pacientes con probable escabiasis y exmenes dermatolgico y microscpico negativos. TRATAMIENTO: Debe ser aplicado a
todos los miembros de la familia y contactos cercanos. Incluye aseo personal y de la ropa. Se sugiere lavar la ropa en agua caliente y guardar en
bolsas de plstico aqullo que no pueda lavarse durante 2 semanas. Antibiticos por va sistmica en el tratamiento de imptigo bacteriano
secundario y anthistamnicos para el control del prurito. La dosis nica de ivermectina es efectiva, en casos severos se aconsejan hasta tres
administraciones, c/1 - 2 semanas en >5aos >15Kg dar 100mcg/kg 1 dosis con cuiracion de 70%, 200mcg/kg curacin de 74%, si se dan dos dosis
con intervalo de 2 semanas la curacin es de 100%, este medicamento inhibe la sntesis neuromusculat del insecto. Los agentes tpicos incluyen:
Permetrina al 5% (Scabisan) es de primera elccion cada 24hrs se retira de 8-14 hrs, no usar en asmticos. Lindano 1% es de segunda eleccin es
organoclorado, no usar en crisis convulsinas, lactancia, ni embarazo menor a 2 meses. Benzoato de bencilo en emulsin del 10-25% para adultos
(Hastilan) por 2-3 noches continuas, reirarse al dia siguiente con agua y jabon, no usar durante el embarazo, lactancia y menores de 5 aos.
Crotamitn en crema (Eurax), muy eficaz ante el prurito. El lindano ha sido prohibido en varios pases. La FDA reporta que el 70% de las reacciones
adversas por lindano son de origen neurolgico, e incluyen ataxia, desorientacin, tremores, convulsiones y muerte. La exposicin crnica deriva en
efectos sistmicos graves y/o mortales. En pacientes con escabiasis noruega se utiliza ivermectina y tratamiento tpico simultneamente,
anthistamnicos y emolientes. En embarazadas manteca benzoada, azufre precipitado, blsamo de Per, frotar 20 minutos dejar actuar por 30
minutos, lavar con agua y jabon. FORMAS ATIPICAS: ESCABIOSIS NODULAR: En 7%, son lesiones persistentes, en escroto, ingles, axilas se atribuye a
uso de esteroides. SARCOPTOSIS COSTROSA O NORUEGA: Se presenta en sujetos con inmunodepresin (SIDA, virus linfotrpico o leucemia),
ancianos institucionalizados, en pacientes con trastornos neurolgicos, principalmente sndrome de Down, demencia, trastornos nutricionales y

CURSO ENARM CMN SIGLO XXI TEL: 36246001

Pharmed Solutions Institute

PGINA 340

MANUAL DE TRABAJO DEL CURSO ENARM CMN SIGLO XXI


enfermedades infecciosas. Las lesiones, hiperqueratsicas, son extensas, semejantes a las que se presentan en la psoriasis, con escaso prurito en la
mayor parte de los casos. Hay formacin de costras gruesas que involucran manos y pies, escamas grisceas y costras en tronco y extremidades,
descamacin en la zona facial, lesiones semejantes a verrugas en dedos y sitios de trauma, y cada profusa de cabello. COMPLICACIONES: imptigo
secundario, hiperinfeccin. Las infecciones bacterianas secundarias se relacionan con frecuencia con Streptococcus del grupo A y Sytaphylococcus
aureus, por lo que se estima que la ectoparasitosis es un factor de riesgo importante en zonas endmicas en el desarrollo de glomerulonefritis,
fiebre reumtica, celulitis y otras infecciones invasivas.
CASO CLINICO
Paciente de sexo masculino, de 15 meses de edad, previamente
sano. Consulta a su pediatra de cabecera por presentar ppulas
eritematosas en mueca izquierda. Algunas de ellas se tornaron
costrosas y, en pocos das, se sumaron ppulas, placas y pequeos
ndulos eritematosos con escamas en axila derecha, regin
supraumbilical y axila izquierda. Se encontraba en buen estado
general y afebril. Acude con la tia quein adems tiene las mismas
lesiones, el menor vive con su madre.
PREGUNTA
Cual de las siguientes medidas es conveniente prescribir como final
de tratamiento mas adecuado?
RESPUESTA
a.- Cefalexina por va oral
b.- Permetrina al 5% (crema fluida)
c.- Hidroxicina 2 mg/kg/da 3 veces/da.
d.- Aplicacin diaria de mometasona (crema) en los ndulos.
PREGUNTA
Si no hubiera ndulos en el paciente. Cul seria el tratamiento de
primera eleccin?
RESPUESTA
a.- Permetrina 1%
b.- Permetrina 5%
c.- Lindano 1%
d.- Crotramiton
PREGUNTA
Cul es la conducta teraputica mas adecuada para la madre del
menor?
RESPUESTA
a.- Permetrina
b.- Lindano
c.- Azufre en ungento
d.- Benzoato de bencilo
CASO CLINICO
Un paciente de 14 aos, con diagnstico de sndrome de Down y
antecedente de cardiopata congnita corregida quirrgicamente,
consult a nuestro servicio por una dermatosis de 2 meses de
evolucin con ligero prurito. En el examen fsico se evidenciaban

placas hiperqueratsicas, adherentes y costrosas, color amarillogrisceo, simtricas, con fisuras profundas y dolorosas, que
asentaban sobre una base eritematosa en las manos, el tronco, las
axilas, el abdomen y el dorso. Tambin presentaba un
engrosamiento de las lminas subungueales en ambas manos y
ppulas aisladas y costras en el tronco y los miembros inferiores.
PREGUNTA
Cul es el diagnstico ms probable en este caso?
RESPUESTA
a.- Psoriasis
b.- Sarna noruega
c.- Escabiosis nodular
d.- Hiperqueratosis epidermolitica
PREGUNTA
Cul es la conducta teraputica ms adecuada para el agente
etiolgico en este caso?
RESPUESTA
a.- Lindano
b.- Permetrina al 5%
c.- Urea
d.- Crotamiton 10%
PREGUNTA
Cul es la conducta teraputica ms adecuada para las lesiones
hiperqueratosicas?
RESPUESTA
a.- Urea al 10% ms ac. Lctico al 5% tpico ms vaselina
b.- Mometasona mas vaselina
c.- Clioquinol mas neomicina crema ms vaselina
d.- Crotamiton al 10%
Tres meses despus, presenta tos productiva y manchas caf con
leche, as como recidiva del cuadro. Cul es su conducta a seguir?
RESPUESTA
a.- Conteo de CD4
b.- Cuantificacin de inmunoglobulinas
c.- Cuantificaciones anticuerpos antinucleares
d.- Biopsia de piel

HIPERQUERATOSIS EPIDERMOLITICA
CIENCIAS BASICAS: Tambin llamada Eritrodermia ictiosiforme congnita ampollosa, es una genodermatosis infrecuente, con patrn de herencia
autosmico dominante, aunque 50% de los pacientes experimentan mutaciones de novo. Se distingue por eritrodermia y ampollas secundarias a
traumatismos mnimos, que se manifiestan desde
el nacimiento y que evolucionan a ictiosis
generalizada a lo largo de la vida,
predominantemente en zonas de flexion. SALUD
PUBLICA: No hay predileccin por sexo o raza. Se
manifiesta desde el nacimiento en un caso por
cada 200,000 a 300,000 nacidos vivos.
PATOGENIA: La causa son los defectos en las
citoqueratinas que forman los filamentos
intermedios. Las citoqueratinas afectadas son la
K1 y la K10, las cuales predominan en los estratos
suprabasales de la epidermis. Cuando las citoqueratinas experimentan mutaciones puntuales, se asocian con la hiperqueratosis epidermoltica y la
eritrodermia ictiosiforme congnita ampollosa, y generan clulas suprabasales que se fragmentan fcilmente, con epidermis hiperproliferativa e
hiperqueratsica. Forman parte del tbulo de las glndulas sudorparas ecrinas y son marcadores de queratinizacin por excelencia. La
hiperqueratosis y la ictiosis subsecuentes que se observan en estos pacientes se deben a un mecanismo de compensacin para proteger contra la

CURSO ENARM CMN SIGLO XXI TEL: 36246001

Pharmed Solutions Institute

PGINA 341

MANUAL DE TRABAJO DEL CURSO ENARM CMN SIGLO XXI


fragilidad cutnea, y as disminuir la probabilidad de formacin de ampollas. CLASIFICACION: Formas generalizadas; eritematoampollar y
eritematoqueratosica. Formas circusncritas: lineal, en placas, queratodermia palmoplantar. DIAGNOSTICO: Se basa en manifestaciones clnicas y
confirmacin histolgica.caracterizada por lesiones hiperqueratosicas o verrugosas o queratodermia palmoplantar. Aparece desde el nacimiento e
inicia como eritrodermia generalizada, con fragilidad cutnea que causa descamacin, as como ampollas, incluso con traumatismos menores, en la
mayora de los casos. Las ampollas son superficiales y grandes, al romperse ocasionan grandes zonas de exulceracin. En los pliegues de flexin
aparecen ulceraciones superficiales y zonas extensas de piel denudada. Debido a la interrupcin de la barrera epidrmica, en estos nios existe un
mayor riesgo de infecciones severas, desequilibrio hidroelectroltico y sepsis. Hay disminucin gradual de la eritrodermia y de las ampollas, que a los
pocos meses evolucionan a placas hiperqueratsicas, de aspecto verrugoso, en las zonas de flexin, la piel cabelluda y los glteos. La hiperqueratosis
inicia a los tres meses de edad, con engrosamiento poco evidente de la piel o con escamas finas y pequeas. Con la edad algunas de las escamas se
van haciendo ms gruesas (adquieren aspecto de cartn corrugado) y la probabilidad de manifestar ampollas disminuye, aunque incluso 20% de
los pacientes sigue teniendo ampollas hasta la vida adulta. La hiperqueratosis se agrava con el sol y el calor. La afeccin de las palmas y las plantas
ocurre en 60% de los pacientes y se manifiesta con grados variables de hiperqueratosis, asociada, sobre todo, con mutaciones de CK1. La
queratodermia palmoplantar puede originar fisuras y contracturas recurrentes y dolorosas, que pueden ocasionar incapacidad funcional. La biopsia
de piel, confirma el diagnostico si observamos epidermis con hiperqueratosis ortoqueratsica, acantosis irregular moderada con reas de
degeneracin de queratinocitos, espinosos, dermis con papilomatosis y focos mnimos de linfocitos perivasculares. La inmunohistoqumica muestra
anormalidades de expresin de la citoqueratina 1 o 10. TRATAMIENTO: La piedra angular del tratamiento son los emolientes tpicos. Existen
reportes de mejora clnica con dosis altas de betacarotenos, retinoides sistmicos o tpicos, glicerina a 10%, cido lctico, alfa-hidroxicidos,
calcipotriol y urea; sin embargo, no existe un consenso sobre la administracin de estos frmacos. Los agentes queratolticos son poco tolerados por
estos pacientes debido a que las escamas se desprenden en bloques extensos. El sobrecrecimiento bacteriano es comn, sobre todo por
Staphylococcus aureus, secundario a la acumulacin y maceracin de la escama, por lo que pueden usarse jabones antibacterianos suaves o
limpiadores con clorhexidina.Los agentes queratoliticos son poco tolerados por estos pacientes debido a que las escamas se desprenden en bloques
extensos. El sobrecrecimiento bacteriano es comn, sobro todo por Staphylococcus aureus, secundario a la acumulacin y maceracin de la esca,a,
por lo que pueden usarse jabones antibacterianos suaves o limpiadores conm clohexidina. PRONOSTICO: Es bueno para la vida, sin embargo los
pacientes pueden padecer alteraciones funcionales, secundarias a queratodermia palmoplantar y contracturas en flexin de las manos.
HIPERQUERATOSIS PALMO-PLANTAR: hacer dx. Diferemcial, es una genodermia autosmica, hay engrosamiento de la capa cornea en regiones
palmoplantares, aisladas o como parte de sndromes, inica con eritema, engrosamiento de la cornea, acompaado de descamacin y fisuras.
PAVILLON-LEFEVRE; queratodermia aplmoplantar con periodontosis, pedida prematura de dientes, genodermatosis autosomica, que altera el
metabolismo de la vitamina a, inica en el primer ao de edad, placas hiperqueratosicas en el dorso de articulaciones interfalangicas, codos, rodillas y
otras areas, denticion normal, destruyen los ligamentos periodontales en 4-5 aos, caracterizado por adoncia, hiperhidrosis, distrofia ungueal, pelo
normal, puede haber calcificaciones intracraneales en la duramadre, tratamiento cremas saliciladas 1-10%, urea 10-30%, lociones de ac. Retinoico.
CASO CLINICO
Lactante mujer, de un ao de edad, sin antecedentes patolgicos de
inters, que consulta por aparicin progresiva de lesiones cutneas a
nivel palmoplantar. El cuadro se inici a la semana del nacimiento
con discreto eritema en dicha localizacin que evolucion
progresivamente hacia placas hiperqueratsicas amarillentas,
simtricas y confluentes. En el momento de la consulta presentaba
una hiperqueratosis difusa palmoplantar bien delimitada por un
borde eritematoso en los mrgenes laterales de las manos y los pies,
sin extensin a la superficie dorsal de estos.

PREGUNTA
Cual es la conducta teraputica mas adecuada a seguir?
RESPUESTA
a.- Envio a segundo nivel.
b.- Tratamiento farmacolgico.
c.- Tratamiento quirrgico.
d.- Terapia fotodinmica

TIAS
CIENCIAS BASICAS: Bajo el trmino dermatofitosis se engloban una serie de dermatosis producidas por un grupo determinado de hongos que se
conocen como dermatofitos (hongos prasitos de queratina) y que se caracterizan por parasitar y digerir, las estructuras crneas (piel, uas y pelo).
Tambin se conocen como micosis superficiales (no pasan de la cornea) o tias y comprenden seis grupos, en funcin del rea corporal afecta. Las
micosis profundas penetran en la queratina, incluyendo rganos internos como micetomas, esporotricosis, coccidioidomicosis, actinomicosis,
histoplasmosis. SALUD PUBLICA: Distribucion mundial, tia de cabeza 98%, tias del cuerpo 3-28%, cualquier edad, tia de pies e ingle 17-50%.
Generos: trichophyton, Microsporum, Epidermophyton. T. rubrum 36-52%, es el agente mas frecuente en todo el mundo, M. canis 14-24%, T.
tonsurans 15-18% (tia capitis). T. mentagrophytes. El contagio es persona a persona, por animales, artculos contaminados a travs del suelo y
albercas. Fcatores predisponenetes: humedad, calor, diabetes, corticoesteroides, calzado cerrado, mala higiene. TIA DE LA CABEZA: Casi exclusiva
de nios, afecta a los cabellos de cuero cabelludo que se rompen a nivel del ostium folicular (quebradizos o deformados), lo que determina zonas de
alopecia. Predomina en escolares y preescolares por M. canis 40%, por T. tonsurans 60%. Hay no inflamatorias en donde a afectacin puede
realizarse implicando a todos los pelos de una zona determinada, pelos podados, zona redondeada de pseudoalopecia (tias tonsurantes
microspricas) o a unos pelos s y otros no, es decir pelo sano y pelo enfermo granos de polvora (tias tonsurantes tricofticas). Existen procesos
inflamatorios en forma de foliculitis supurativa, no exclusiva de cuero cabelludo, conocidas como Querion de Celso, que estn producidas por
hongos zooflicos (T. rubrum en 75%) y que presentan exudacin abundante a nivel folicular especialmente cuando se presiona la placa (signo de la
espumadera). Dependiendo de la intensidad del componente inflamatorio podremos encontrar, o no, la existencia de alopecia cicatricial. El
tratamiento de primera eleccin es la terbinafina. Nios 10mg/Kg/dia por 6 semanas, terbinafina 250mg/dia por 12 semanas Otra opcin es
itraconazol VO 5mg/da por 4 semanas, itraconazol 100mg/kg 6 semanas. Tambien griseofulvina 20mg/kg de peso 2-3 meses, chamopu de disulfuro
de selenio 25%. Querion de Celso: causado por M. canis y T. mentagrophytes, principamente en cabeza, es un plastrn inflamatorio, doloroso, con
multiples pustulas, abscesos, ulceras y costras melicericas, adenopata satlite, no fiebre, causa alopecia, sin tratamiento da alopecia definitiva,
tratamiento con prednisona 0.5mg/kg/dia por 1-2 semanas, griseofulvina 10-20mg/kg/dia por 4-6 semanas. Terbinafiba, itraconazol. TIA DEL
CUERPO: Cualquier zona corporal no incluida en esta clasificacin puede incluirse en este grupo. Su clnica es muy evocadora en forma de lesiones
papulosas que tienden a crecer de forma excntrica determinando crculos en los que existe una zona central sana o discretamente escamosa, y
un borde inflamatorio, activo, papuloso o pustuloso. Al igual que en cuero cabelludo pueden existir formas inflamatorias cuando el agente etiolgico
corresponde a un hongo de origen animal. Se puede usar terbinafina tpica, cada 24 hrs por 4 semanas. Alternativa, miconazol. TIA CRURAL: Se
caracteriza por la aparicin de placas anulares, a nivel inguinal. Es excepcional en nios pero s suele afectar a jvenes que practican deporte de

CURSO ENARM CMN SIGLO XXI TEL: 36246001

Pharmed Solutions Institute

PGINA 342

MANUAL DE TRABAJO DEL CURSO ENARM CMN SIGLO XXI


forma habitual, y no es rara su asociacin a tia de los pies. Su posible transmisin por contacto sexual es siempre necesario tenerla en cuenta. TIA
DE LAS MANOS: Es un proceso poco frecuente, y excepcional en la edad peditrica, que casi siempre aparece en mujeres con trastornos
circulatorios, diabetes o en profesiones que maceran sus manos bien por el uso continuado de guantes o por precisar mantenerlas en agua. Clinica
placas de eritema con descamacin y maceracin interdigital, vesicula o ppulas muy pruriginosa. T. rubrum 70%, T, mentagrophytes 25%.
Tratamiento con terbinafina tpica. 2 semanas. TIA DE LOS PIES: Rara en la etapa infantil pero frecuente en la pubertad, y en ocasiones asociada a
tia crural. Es debida a la parasitacin por dermatofitos en el pie pudiendo adoptar morfologas variables. Forma interdigital (pliegues de 4to y 5to
dedo): incluida dentro del cuadro genrico de pie de atleta se caracteriza por la aparicin de descamacin y fisuracin a nivel de los espacios
interdigitales, anhidrosis con descamacin, vesculas o plas, muy pruriginosa. El proceso puede ser poco llamativo cuando la infeccin es exclusiva
de dermatofitos (Tinea pedis simplex) o ser muy inflamatoria o exudativa cuando existe infeccin combinada de dermatofitos y bacterias (Tinea
peds complex). Examen microscpico directo con hidrxido de potasio 20%; observamos filamentos largos delgados o gruesos, artroconidias,
blastoconidias, onicomicosis candida. Tratamiento con ternbinafina tpica por 2 semanas. Onicomicosis; uas con fragilidad, engrosamiento, estras,
coloracin amarillenta o caf oscura, la inasion es del borde libre hasta el cuepo de la ua, matriz, son opacas, secas, el causante es T. rubrum hasta
75%. Puede haber infeccion en uas por candida, el ataque es desde la matriz hacia el borde. Tratamiento: terbinafina tpica por 2 semanas en uas
12-16 semanas, alternativa fluconazol. TIA DE LAS UAS: Es un proceso habitual en las consultas de dermatologa peditrica. El patrn diagnstico
es la presencia de una hiperqueratosis subungueal que determina un levantamiento de la ua del lecho (onicolisis). Generalmente la lesin comienza
en el pliegue subungueal distal (onicomicosis subungueal distal), aunque puede hacerlo en el proximal (onicomicosis subungueal proximal) o incluso
acabar afectando a toda la ua (onicomicosis distrfica total). En contadas ocasiones se produce una parasitacin exclusivamente de la lmina
ungueal que presenta una coloracin blanquecina (Onicomicosis blanca superficial). SEGUNDO NIVEL: Dermatitis graves, proceso necrtico
agregado, tratamientos refractarios.
CASO CLINICO TIA CORPORIS
Nia de 7 aos, que consult de urgencias por aparicin progresiva
en los ltimos 10 das de una placa anular eritemato-descamativa
muy pruriginosa, de 3 x 3cm de dimetro y crecimiento centrfugo,
con borde levemente sobreelevado. Clnicamente planteaba el
diagnstico diferencial entre una tinea corporis y un eczema
numular. Para confirmar el diagnstico se realiz un examen directo
de escamas cutneas, obtenidas despus de raspar el borde de la
lesin con un bistur e incubadas con 0,5ml de KOH al 30% sobre una
lmina portaobjetos.
PREGUNTA
Cuales son los factores de riesgo ms importante para sospechar de
este padecimiento?
RESPUESTA
a.- Deportes y recreacin en equipo.
b.- Exposicin a lugares hmedos sin protecin
c.- Compartir objetos personales.
d.- Uso de ropa sintetica.
PREGUNTA
Cual de los siguientes factores de riesgo es el ms frecuente que
estn relacionados con las tias?
RESPUESTA
a.- Uso de corticoides.
b.- Inmunocompromiso.
c.- Queratodermia.
d.- Ictiositocis hereditaria.
PREGUNTA
Cual es la conducta teraputica ms adecuada?
RESPUESTA
a.- Terbinafina 250 mg por 12 semanas.
b.- Itraconazol 10 mg/Kg/dia
c.- Fluconazol 150 mg VO por 24 semanas.
d.- Miconazol crema al 2 %
PREGUNTA
Al revalorar la paciente posteriormente la paciente continua con el
cuadro clnico con complicaciones, cual de las siguientes no es
indicativo de envio a segundo nivel.
RESPUESTA

a.- Diseminacion dentro de las 4 primeras semanas.


b.- Presencia de onicomicosis comorbida.
c.- Alteracin de enzimas hepticas.
d.- Inmunosupresion de origen idioptico.
CASO CLINICO
Se trata de paciente femenino de 9 aos de edad, el cual acude a
clases de natacin 3 veces por semana, se encuentra en tratamiento
con esteroides por alergia no especificada, la madre refiere que
tiene un perrito de mascota en casa. La madre la lleva a consulta por
presentar pseudoalopecia, descamacin, eritema y prurito en varias
reas de la cabeza pequeas e irregulares, y que su cabello se ha
vuelto quebradizo.
PREGUNTA
Cul es el agente etiolgico ms probable en este caso?
RESPUESTA
a.- Trichopyton rubrum
b.- Trichopyton tonsurans
c.- Mycrosporum canis
d.- Epidermophyton flocusum
PREGUNTA
Cul es la conducta teraputica ms adecuada a seguir en este
caso?
RESPUESTA
a.- Griseofulvina20mg/kg
b.- Itraconazol 3mg/kg/dia
c.- Ketoconazol 10mg/kg/dia
d.- Prednisona 0.5mg/kg/dia
PREGUNTA
A los 5 meses es llevada nuevamente por la madre, refiere que el
cuadro remiti, a la exploracin presenta, un plastrn inflamatorio
doloroso, con presencia de pstulas y costras melicericas en regin
occipital, con presencia de alopecia. Cul es la conducta teraputica
ms adecuada en este momento?
RESPUESTA
a.- Griseofulvina20mg/kg, prednisona 0.5mg/kg/da
b.- Itraconazol 3mg/kg/dia, prednisona 0.5mg/kg/da
c.- Ketoconazol 10mg/kg/dia, betametasona 0.5mg/kg/da
d.- Prednisona 0.5mg/kg/dia, dicloxacilina 50mg/kg/da

PTIRIASIS VERSICOLOR
CIENCIAS BASICAS: Es una infeccin mictica del estrato crneo de la piel, es la micosis ms superficial que se conoce, causada por Malassezia furfur
(ptiriosporum furfur), es un hongo levaduriforme y lipfilo, esun hongo de la flora normal de la piel. Adems del clima, se han sealado otros
factores predisponentes como aplicacin local de corticoesteriodes, desnutricin, recambio lento de la epidermis y predisposicin gentica. SALUD
PBLICA: La infeccin se presenta a partir de la adolescencia y es raro encontrarla en personas de edad avanzada. Esta micosis es ms frecuente en

CURSO ENARM CMN SIGLO XXI TEL: 36246001

Pharmed Solutions Institute

PGINA 343

MANUAL DE TRABAJO DEL CURSO ENARM CMN SIGLO XXI


personas que habitan lugares de clima clido y hmedo, tales como las regiones costeras tropicales, en donde la frecuencia puede llegar a ser hasta
del 50%. Predomina en varones, cualquier edad (20-45aos), lactantes. Factores predisponentes: calor, sudoracin, humedad, ropa ajustada,
material sintetico, aplicacin de grasas, glucocorticoides sitemicos, sndrome de Cushing, diabetes, inmnosupresion, tratamiento con tetraciclinas.
PATOGENIA: El agente etiolgico es M. globosa que puede encontrarse como especie nica o asociada con a otras especies, entre ellas M.
sympodialis, M. slooffiae. DIAGNOSTICO: Las lesiones generalmente son asintomticas, se inician como discretas manchas eritematosas, de 2-4mm
hasta 1-2cm, cubiertas con una escama muy delgada (como salvado), que pronto toman dos aspectos hipocromiantes o hipercromiantes, adems
son confluentes, de distribucin centrpeta. En Mxico predomina la primera, son manchas un poco ms claras que la piel raras veces acromicas,
lenticulraes confluentes, bien delimitadas de bordes no activos como deshilachados, con esa fina escama y prcticamente sin prurito. Se localiza con
mayor frecuencia en el tronco, cuello, cara, regiones periauriculares piel cabelluda y los brazos, aunque se pueden observar en otras regiones
corporales de acuerdo a los factores predisponentes de cada paciente. No se requiere biopsia, con tincin de heatoxilina y eosina o colorante de
Gomori, se manifiestan levaduras y filamentos en la capa cornea o infundbulo del pelo. Si se rapa con la ua o con cuchilla queda una marca en la
piel es el signo de Bernier o del Uazo. Examen directo con potasa 20-40%, se observan las hifas, asociadas a la levadura esfrica, espaguetis y
albndiga. Cinta adhesiva se observan esporas de 3-6mm en racimos de uvas y filamentos cortos. En lmpara de Wood, se ven color verde amarillo,
donde la piel afectada o una fluorescencia amarilla oro. TRATAMIENTO: lociones, cremas con acido saliclico y azufre 1-3%, ungento de Whitfield,
toques de yodo 1%, disulfuro de selenio 2.5% champ, hiposulfito de sodio 20%, propilenglicol 50%, ac. Retinoico, imidazoles como miconazol,
clotrimazol, ketoconazol 200mg/dia por 10 dias a un mes, itraconazol 100mg/dia por 15 dias-1mes. GPC: Champ flutrimazol 1%(1ra eleccin) o
ketoconazol al 2%, terbinafina 1% solucin c/12hrs, clotrimazol 1% crema c/12hrs por 2 semanas. Lesiones extensas fluconazol 400mg dosis nica es
mejor que clotrimazol 1% crema en pacientes con recurrencia. CRITERIOS para enviar a segundo nivel: dermatosis atpica y difcil de diagnosticar,
asociada a inmunosupresin, falla al tratamiento habitual. PTIRIASIS ALBA: Hacer dx. Diferencial; es benigna, frecuente en nios, son maculas
hipopigmentadas redondas u ovales asintomticas, 0.5-5cm de dimetro, mas en cara y extremidades superiores. PTIRIASIS ROSADA: En ersonas
jvenes y sanas, son placas anulares con borde descamativo, luego multiples maculo-ppulas, color rosado, cara, tronco, dura 6-8 semanas, hay una
placa heraldo o madre. ERITRASMA; infeccion bacteriama por Microsporum minutissimun, hay manchas marrn rojizas definidas, esca,as irregulares,
en zonas intertriginosas, axilar, genital, pies.
CASO CLINICO
Masculino de 28 aos originario y residente del estado de Veracruz.
Presenta una dermatosis crnica y asintomtica, diseminada a
tronco y miembros superiores, constituida por manchas
hipocromicas algunas
eritematosas con escama fina en su
superficie, que confluye formando placas de diversos tamaos.
Refiere haber iniciado hace un ao su padecimiento y solo
ocasionalmente presenta prurito leves. No cuenta con antecedentes
de importancia para el padecimiento actual y el resto de la
exploracin fsica se encuentra sin alteraciones.
PREGUNTA
Cul es el diagnstico ms probable para este paciente?
RESPUESTA
a.- Vitligo
b.- Dermatosis seborreica
c.- Pitiriasis versicolor
d.- Dermatitis solar hipocromiante

PREGUNTA
Cul es la conducta teraputica ms adecuada para este caso?
RESPUESTA
a.- Esteroides tpicos, psoralenos
b.- Shampoo de flutrimazol local, itraconazol cap 100mgs/dia
c.- Lindano shampoo, permetrina 1%
d.- Dicloxacilina caspulas 2g/da, urea en crema
PREGUNTA
Cul es la conducta diagnostica ms probable de realizarle a este
paciente y que observaramos?
RESPUESTA
a.- Corte histolgico, observaramos nmero reducido de
melanocitos
b.- Examen directo con potasa, observaramos espagueti y
albndiga
c.- Histopatologa espongiosis eosinofilica
d.- Tincion de Gramm, bacilos

CANDIDIASIS
CIENCIAS BASICAS: Es una micosis, con parasitacin de piel, mucosas, vsceras causada por diversas especies de levaduras del gnero Candida.
Cualquier tejido puede ser afectado por lo que se presentan diversos cuadros clnicos, cada uno de ellos asociado directamente al estado
inmunolgico del paciente. Es frecuente que origine neumonitis, gastroenteritis o meningitis. SALUD PUBLICA: La distribucin geogrfica de esta
micosis es universal y ms de 70 % de ellas son producidas por C. albicans observndose un porcentaje mayor por el serotipo B. Los casos de
candidiasis sistmica estn relacionados a pacientes con severas deficiencias en su sistema inmune. Vive saprofita en mucosa oral, nasal, vaginal,
tracto gastrointestinal, se transforma en oportunista. Factores predisponentes: pH de las mucosas (infancia, embarazo), maceracin, humedad y
traumatismos, mal estado de dentadura y protesis, diabetes, obesidad, SIDA, leucemia, medicamentos. PATOGENIA: La C. albicans se ha
considerado el ejemplo ms clsico de lo que es un parasito oportunista, vive habitualmente en forma saprofita en las mucosas orales, nasal, vaginal,
tracto gastroinetstinal, en cambio no es habitual encontrarla en piel. El nio al nacer puede recibir este parasito al pasar por el tracto vaginal de la
madre. Cuando se convierte de saprofita a patgena, puede producir desde una simple algondoncillo hasta graves endocarditis o septicemia.
Factores predisponentes: 1. Fisiolgicos que suelen transformar el pH de las mucosas como la infancia y el embarazo. 2. Maceracin, humedad y
traumatismos, (pliegues interdigitales, submamarios, uas y rebordes ungueales). 3. Dermatosis inflamatorias previas (como dermatitis del paal
que se complica con candida). 4. Mal estado de la dentadura y prtesis. 5. Enfermedades metablicas (DM y obesidad). 6. Enfermedades
inmunosupresoras (leucemias, Enf. De Hodking, SIDA). 7. Medicamentos que alteran la flora bacteriana (corticoides, citotxica). DIAGNOSTICO: La
manifestacin ms conocida es el algodoncillo que aparece en la boca de RN por su bajo pH. Las lesiones son placas cremosas, blanquecinas, como
residuo de leche, que pueden afectar la mucosa de los carrillos, la lengua, el paladar, encas e incluso invadir toda la boca hasta la trquea o salirse
de la boca y producir fisuras cubiertas de material blanquecino en las comisuras labiales. Estas lesiones son dolorosas e impiden la alimentacin al
nio. La Candida albicans no es habitual en la piel, las lesiones a parecen a nivel de los pliegues: interdigitales en manos y pies, inguinales, region
submamaria, axilas, intergluteos y perin. Se trata de fisuras y erosiones eritematosas, maceracin, vescula y pstulas y algunas costras y escamas.
La perionixis y la afeccin de la ua se ha observado sobre todo en personas que mantienen mucho tiempo las manos dentro del agua, como
empacadores de frutas, pescados y mariscos. El reborde ungueal se observa inflamado, eritematoso, desprendido de la ua y esta se afecta
principiando de la matriz al borde libre hacia adentro. Poco aparecen estras en las uas, la cual se vuelve amarillenta y opaca y se empieza a
despulir. La lesiones son habitualmente superficiales, es poco frecuente que al igual que los dermatofitos se introduzcan ms all de la capa cornea,

CURSO ENARM CMN SIGLO XXI TEL: 36246001

Pharmed Solutions Institute

PGINA 344

MANUAL DE TRABAJO DEL CURSO ENARM CMN SIGLO XXI


dependiendo del estado inmunolgico del husped originndose granulomas tricofticos o candidsicos. Es fcil buscarla de manera directa,
colocando material recolectado en un portaobjetos con solucin de potasa al 20% y se podr encontrara las clsicas formas levaduriformes, que a
veces producen pseudofilamentos. El cultivo en medio Saboraud produce colonias tpicas de aspecto cremoso. TRATAMIENTO: Corregir enfermedad
de base. En boca usamos buches con agua de bicarbonato de sodio. Cambiar pH, acido actico, bicarbonato de sodio, localmente puede usarse
violeta de genciana al 1% que mancha mucho o la nistatina, antibitico que solo es activo para levaduras de Candida. Los imidazoles tanto tpicos
como sistmicos son altamente efectivos en candidiosis: miconazol, ketoconazol, clotrimazol, itraconazol
CASO CLINICO
Paciente femenino de 10 aos de edad en consulta por presentar
engrosamiento y cambio de coloracin en la ua del pulgar derecho,
al examen fsico, en cuero cabelludo, se observan tras placas de
alopcicas, con cicatrizacin en la periferia y escasas costras
serosanguinolentas en el centro, dos de estas placas se localizan
alrededor del apice del crneo y la tercera en regin parietal
derecha. En cavidad oral y labios se observan placas blanquecinas
que cubren el centro y los bordes de la lengua, asi como ambas

comisuras labiales; al remover estas placas se observa eritema y


fisuras de mucosas.
PREGUNTA
Cual es la conducta a seguir mas adecuada?
RESPUESTA
a.- Itraconazol.
b.- Miconazol.
c.- Ketoconazol.
d.- Clotrimazol.

MICETOMA
CIENCIAS BASICAS: Es una infeccin crnica de la piel y de los tejidos subyacentes con tendencia a afectar los huesos. Se caracteriza por un aumento
de volumen relativamente indoloro y fstulas (lesion elemental) a travs de las cuales se elimina pus y granos constituidos por filamentos. SALUD
PUBLICA: Predomina en varones 4:1, 60% campesinos descalzos, o que usan huaraches, edad de presentancion entre 16-40 aos (58%), 21% amas
de casa, acarreadores de azcar en el dorso de la espalda. Agentes etiolgicos: ACTINOMICETALES (en Mexico) 60%, eumicetomas 40%, Nocardia
brasiliensis 32%.. Actinomadura madurae 10.2 %. Streptomyces somaliensis 8%. N. asteroides, A. pelletieri, N. otitidiscaviarus. Los hongos
dermatiaceos forman granos negros visibles a simple vista, mientras que los de filamento hialino forman granos blancos o blanco-amarillentos. Los
agentes causales se encuentran en tierra, madera, espinas, a travs de soluciones de continuidad. PATOGENIA: El micetoma se adquiere por
inoculacin traumtica de agentes etiolgicos a travs de la piel. En Mxico existe gran nmero de cactceas, las espinas son un mecanismo de
infeccin, que probablemente tengan importancia en la transmisin de la enfermedad. Una vez que la bacteria o el hongo se localizan en los tejidos
y sobrevive (periodo de incubacin varias semanas, meses, aos), se presenta una inflamacin aguda a nivel local con llegada de PMN y activacin de
macrfagos, aunque la mayora de bacterias son fagocitadas y destruidas, algunas logran sobrevivir hasta 16 das y pueden reproducirse. Los agentes
no destruidos proliferan y forman colonias en el interior del tejido conocidas como "granos"; el desprendimiento de los pseudofilamentos de la
periferia del grano ocasiona una inflamacin continua con formacin de nuevas estructuras parasitarias y colecciones de pus, que al fusionarse
forman los trayectos fistulosos generalmente limitados por tejido fibroso; cuando las fstulas alcanzan la superficie, forman un ndulo que
posteriormente se reblandece y abre liberando pus conteniendo colonias parasitarias del agente. El tejido conjuntivo formado alrededor de los
microabscesos y de las fstulas produce una retraccin que ocasiona el aspecto deprimido de las fstulas antiguas. La presencia de micro abscesos,
fstulas, inflamacin, edema y fibrosis provoca externamente la deformidad y dureza del rea afectada e internamente obstruccin de la circulacin
por compresin mecnica y por arteritis, adems de que se extiende a la superficie, tejido subcutneo, musculos, huesos. Estos cambios histolgicos
influyen en la pobre respuesta teraputica en los pacientes con evolucin prolongada. DIAGNOSTICO: Por la naturaleza saprofita de los agentes
causales de micetoma que posibilita la infeccin principalmente por traumatismo en pie o pierna, hay aumento de volumen, tumefaccin,
deformacin del rea, fistulas que drenan un exudado seroso o purulento, parasitos granos. Puede afetar piel, hipodermis, huesos y a veces
vsceras. Examen directo; La observacin del material purulento que drena a travs de las fstulas permite identificar los granos macroscpicos en los
eumicetomas y, al examen microscpico, en la mayora de los actinomicetomas. Clinica hay incapacidad funcional, dolor, caludicacion permanente, a
veces no se presentan sntomas intensos, por lo que acuden en etapas tardas. Suele coexistir infeccion bacteriana, dolor, perdida de peso, anemia,
febrcula. En nios; minimicetomas nicos o multiples, sin edema, pocas fistulas en extremidades inferiores. Tienen una evolcuion lenta sin regresin
espontanea, se extiende en planos profundos, tejido subcutneo, musculos, huesos. Topografia; extremidades inferiores 64% PIE, pierna, rodilla,
muslo, pared abdominal 17-25%, esternn, dorso 17-25%, extremidades superiores 14%. Espalda, nuca, pie. Los micetomas mesodorsales pueden
afectar vertebras, medula espinal, paraplejia. Los micetomas laterodorsales; invaden pleura y pulmn. Cuando las biopsias se toman de sitios con
gran actividad parasitaria, los granos de todos los agentes se visualizan con relativa facilidad con la tincin de hematoxilina eosina (H-E). Los estudios
radiolgicos no permiten establecer el diagnstico preciso de esta patologa, sin embargo son de suma importancia para determinar el grado de
afeccin sea, permiten evaluar la respuesta teraputica y emitir un pronstico. Los cambios observables en estudios radiolgicos simples pueden
ser en los casos iniciales nicamente periostitis, o bien, destrucciones osteolticas extensas con formacin de geodas, destruccin de superficies
articulares y lisis en huesos del pie que ocasionan el aspecto de 'caries'. TRATAMIENTO: Debido a la posibilidad de que los micetomas sean causados
por hongos o por bacterias, antes de prescribir la teraputica es indispensable conocer el tipo de agente causal (mictico o bacteriano); esto puede
lograrse mediante a la observacin de las caractersticas de los granos en el examen directo y en el estudio histolgico. Eumicetoma: quirrgico la
extirpacin completa elimina el proceso no hay metstasis. Contraindicada la amputacin en actinomicetomas porque se favorece la metstasis,
DDS diaminodifenilsulfona 200mg/dia 2-3aos, sulfametoxipiridazina 500mg/dia 6 meses, TMP/SFX 800mg/160mg c/12hrs 1-2 aos junto con
estreptomicina rifampicina. Anfotericina B, griseofulvina 500-1000mg/dia, ketoconazol 400mg/dia, itraconazol 300mg/dia. La mayora de micetomas
causados por actinomicetales responden a este tratamiento y la mejora es evidente despus de dos meses de tratamiento. En opinin de algunos
investigadores, la fibrosis caracterstica que rodea los granos de los agentes etiolgicos, impide que se alcancen las concentraciones adecuadas para
la curacin y, en consecuencia, el tratamiento no es efectivo. El advenimiento de los compuestos azlicos proporcion nuevas alternativas para el
manejo de esta patologa, se hicieron pruebas con ketoconazol solo o asociado a ciruga; esta ltima alternativa mejor el pronstico de manera
substancial. En la ltima dcada, se ha utilizado el itraconazol a dosis de 300 a 400 mg por da, reportndose algunos casos de curacin y otros con
mejora. Algunos autores como Smith y Kutbi, mencionan que en casos de eumicetoma de corta evolucin la ciruga acompaada de tratamiento
antimictico es la nica posibilidad de curacin definitiva. HIDROSADENITIS: Hacer dx. Diferencial; es una inflamacin de las glndulas apocrinas que
produce obstruccin y rotura de los conductos sudoriparos apocrinos, en axila, ingle, ndulos, fistulas.
CASO CLINICO

CURSO ENARM CMN SIGLO XXI TEL: 36246001

Masculino de 8 aos, originario de zona rural de guerrero, fue


llevado a consulta por una dermatosis localizada en el tronco que se

Pharmed Solutions Institute

PGINA 345

MANUAL DE TRABAJO DEL CURSO ENARM CMN SIGLO XXI


extendia al toras y a la cara anterior del hombro derecho. La
dermatosis era de aspecto monomorfo y estaba constituida por
algunas fistulas, numerosos ndulos eritematoso de 0.3 a 2 cm
aproximadamente, telangiectasias y costras sanguneas y meliceras
que formaban una placa mal circunscrita, de 6 x 4
aproximadamente, crnica y asintomtica. Se realizaron cultivo los
cuales mostraron colonias de aspecto yesoso, rogosas y de color
blanco. La prueba de la hidrolisis de casena resulto positiva y la
colonia se identifico como Norcardia brasiliensis. No se observo
compromiso oseo a los rayos x.

PREGUNTA
Cual es el manejo mas adecuado para el caso?
RESPUESTA
a.- Anfotericina.
b.- Trimetroprim-sulfametoxazol.
c.- Itraconazol.
d.- Clotrimazol.

EXANTEMATICAS
CIENCIAS BASICAS: Grupo de infecciones sitemicas, con diferente grado de contagiosidad, que tienen como caracterstica principal la presencia de
una erupcin cutaena (exantema) simtrica y que se presenta con mayor frecuencia durante la edad peditrica. Elmexantema es una lesion visible
en iel debido a enfermedad, aparece abruptamente y afecta varias reas de la piel simultneamente. SALUD PUBLICA: Se ha reportado que del 65100% de los exantemas con fiebre en la edad peditrica tienen una causa infecciosa, y 72% tienen una causa viral y la mayora de las veces son
inofensivos. CLASIFICACION: De acuerdo a la morfologa de la lesion primaria: 1. Maculo-papular (sarampin, rubeola, exantema sbito,
mononucleosis infecciosa), 2. Petequial, purpuricos (purpura fulminas, sndrome de choque toxico, dengue) 3. Eritematoso difuso con descamacin
(fiebre escarlatina, staphylococcemia, eritema infeccioso, enfermedad de Kawasaki), 4. Vesiculopustular (varicela, sndrome pie-mano-boca,
sndrome de piel escaldada SSSS, sfilis congnita), 5. Nodular, 6. Urticariforme (enfermedad de Lyme, sndrome de Stevens Johonson).
ESCARALATINA: 3-15 aos. Streptococobeta hemoltico del grupo A (toxina eritrogenica), por contacto directo secreciones nariz, garganta, piel
fmites, asociado a infecciones farngeas, heridas (infecciones cutneas). Incubacion 1-7 dias. Clinica: Inicio repentino, fiebre (39-40 de 2-4 dias),
enantema (exudado membranoso en amgdalas y lesiones petequiales en paladar blando) y exantema, vomito, nauseas, dolor abdominal (20%),
odinofagia. Lesiones petequiales en zona antecubital, ingle y axila (signos de la pastia), exantema eritematoso generalizado (primeras 24h), es
macolopapular, desaparece a la presion, de textura aspera (piel de lija, gallina). Enrojecimiento de la cara con palidez perioral (triangulo de Filatov).
En boca maculas puntiformes rojas en uvula (manchas de Forcheimer). Entre el 1ro y 2do dia hay una capa blanca (saburra) en el dorso de la lengua,
papilas enrojecidas y aumentada de volumen (lengua en fresa blanca), 4-5to dia desaparece del borde lingual al centro, dejando descubiertas a las
papilas rojas (lengua en frambuesa), no afecta palmas, ni plantas. La descamacin inicia en cara al final del 7mo dia moldes epidrmicos.
Diagnostico: Bh; leucopenia con PMN, cultivo de faringe o piel, exudado farngeo es la prueba estndar sensibilidad de 90%. Antiestreptolisina O se
hace positiva entre 3-5 semnas de infeccion. Complicaciones: Adenitis cervical, otitis media, sinusitis, tardas; cardiolgicas o nefrolgicas.Tx:
penicilina V 50-100,000 U/kg/da c/6h durante 10 dias, amoxicilina 20-50mg/kg/dia, penicilina G benzatinica, nica dosis erradica <20Kg 600 000U,
>20Kg 1200 000U, opcin cefalexinas, alrgicos eritromicina30-50mg/kg/dia o clindamicina. Profilaxis esta indcada en contactos con penicilina
benzatinica o eritromicina. ERITEMA INFECCIOSO: Quinta enfermedad, 6-19 aos, geritricos. Parvovirus humano B19, inhalacion de aerosoles,
transfusin sangunea y vertical (congnita). Incubacion de 4-21 dias, contagio es de 6-11 dias previos al exantema. Clinica: puede habre fiebre,
defalea, conjuntivitis, faringitis. 1ra etapa: exantema eritematoso en frente y mejillas mejilla abofeteada, no doloroso, con incremento de
temperatura local, edema y palidez peribucal. 2da etapa: exantema papular, distal y simtrico en el tronco y piernas con apariencia de encaje. 3ra
etapa: erupcin recurrente por varias semanas, cambios de intensidad por estrs, frio, luz solar. Diagnostico: Clinico, serologa y PCR, IgM, la IgG solo
indica infeccion previa o inmunidad. Complicaciones: Anemia aplasica, idropesia fetal. ENFERMEDAD KAWASAKI: <5aos, es una vasculitis sistmica,
causa mas comn de cardiopata adquirida, superando a la fiebre reumtica, mas en primavera e invierno. Clinica: Criterios completos: Fiebre por
mas de 5 dias y mas de 4 dias con los siguientes signos, cambios orofaringeos (lengua en fresa, eritema difuso en mucosa orofaringea, eritema o
fisuras en los labios), adenopata cervical (>1.5cm de dimetros, unilateral), exantema polimorfo, cambios perifricos en extremidades (eritema o
edema en plantas y manos, descamacin periungueal en fase subaguda). FASE AGUDA: Inicia con fiebre y termona con defervescencia, promedio 11
dias (choque cardiogenico por disfuncin sistlica, regurgitacin mitral). FASE SUBAGUDA: Inicia con la resolucin de la fiebre y termina con la
resolucin del cuaro clnico, del dia 10 y dura 2 semanas (se pueden presentar aneurismas, principal causa de morbi-mortalidad) FASE
CONVALESCENCIA: 4-8 semanas despus del inicio de la enfermedad. La fiebre es perisistente, alta y no responde a tratamiento. Diagnostico:
Reactantes de fase aguda elevados, leucocitosis con predominio de neutrfilos, trombocitosis, hipoalbuminemia, transaminasas elevadas. Realizar
ECG y ecocardiograma a todos, por afeccion cardiaca multiple. SARAMPION: Mixovirus ARN, <2aos, personas no vacunadas, o con esquema
incompleto, VIH, desnutricin grave. Incubacion de 7-18 dias. Contagio: desde el periodo prodrmico, historia de contacto reciente (<21dias),
altamente contagiosa, persona por tracto respiratorio o conjuntivas en un lapso no mayor a 2 hrs, se contagia desde 4 dias antes de iniciar exantema
hasta una semana despus de aparicin del mismo. PRODROMO: Fiebre 38.5-39, Tos, coriza, catarro, conjuntivitis e irritabilidad, en casoso graves
zonas hemorrgicas en parpados inferiores (lneas de Stimpson). FASE ERUPTIVA: Exantea en progresin cefalo-caudal, dura 5 dias. FASE
EXANTEMATICA: Maculo-papular, rojizas, rojo pardusco, las manchas se borran a la presin, tiende a ser confluente, inica en regin retroauricular,
afecta palmas y plantas, hasta 2 dias despus el enantema aparece en la cara interna de los carrillos a la altura del segundo molar (manchas de
Koplik). FASE CONVALESCENCIA: Inicia despus del inicio del exantema. Exudado farngeo y muestra de orina para aislamiento y genotipificacion. En
primeros 5 dias de haber iniciado el exantema. IgM especifica del virus desde lops primeros dias hasta 4-6 semanas. Complicaciones: otitis media,
neumoias (60-70% de defunciones), laringotraqueitis, encefalitis, diarrea, vomito. Vitamina A 200,00UI en >1ao y 100,000UI en <1ao, con ello se
disminuyen complicaciones como diarrea y neumona. Vacunar a los contactos susceptibles en las siguientes 72h post-exposicin. RUBEOLA: 1-4
aos. Virus RNA, rubivirus, en primavera-verano. Contacto directo, inhalacin de aerosoles y vertical (congnita). Perosnas no vacunadas,
inmunodeficiencia, desnutricin grave. Incubacion 14-20 dias. Contagio: 10 dias antes y 7 dias despus de qie inicie el exantema. Acudir a escuela
hasta 7 dias despus de haber iniciado la erupcin. Clinica: PRODROMO: 2-4 dias antes de exantema hay fiebre de bajo grado, cefalea, anorexia,
conjuntivitis leve, coriza, manchas de Forsheimer en boca, exantema maculopapular rosado y se asocia a la presencia de adenomegalias con
localizacin caracterstica en regin retroauricular y cervico-suboccipital, las lesiones tienen una aparicin cefalocaudal que en ocasiones puede
llegar a ser confluentes, pruriginosas y pueden durar hasta 3 dias, desapareciendo en orden invertido a su aparicin, descamacin en harina fina.
Triada fiebre, exantema, hipertrofia ganglionar. Diagnostico: ELISA, inhibicin de hemaglutinacin, aislamiento de virus en secrecin nasal, sangre,
garganta, orina o LCR. Para prevenir rubeola congnita, vacunar en la etapa de lactante y revacunar a adolescentes y adultos en edad reproductiva.
Complicaciones: artritis, artralgias, hemorragias en encfalo, trombocitopenia, hepatitis. Notificacion inmediata en sospecha o con firmacion de
caso. EXANTEMA SUBITO: Sexta enfermedad, nios de 1-3 aos, herpes virus HVV-6, por secreciones respiratorias. Contagio: la transmiten durante
el periodo febril y una semana previa. Clinica: Fiebre alta que desaparece sbitamente cuando aparecen las lesiones drmicas, son unas lesiones

CURSO ENARM CMN SIGLO XXI TEL: 36246001

Pharmed Solutions Institute

PGINA 346

MANUAL DE TRABAJO DEL CURSO ENARM CMN SIGLO XXI


roseaseas 2-3mm de dimetro en el tronco que se extienden al cuello y las extremidades. Exantema se caracteriza por maculas eritematosas de
forma de almendra y ppulas en el tronco, cara, cuello y extremidades superiores. Tambien presenta enantema con papulas eritematosas en el
paladar blando y uvula (manchas de Nagayama). Debe sospecharse en lactantes o preescolares con fiebre elevada y crisis convulsivas, incluso sin la
presencia de exantema. Se autolimita, no requiere antiviral. Principal morbilidad son las crisis convulsivas (6-15%), durante la fase febril. VARICELA:
<10aos, Virus de varicela zoster (VVZ). Primavera invierno, por contacto directo con secreciones respiratorias o las lesiones drmicas y en menor
frecuencia por via vertical, Incubacion de 10-21 dias. Contagio: antecedente de contacto en los ltimos 21 dias. Clinica: 1-2 dias antes del exantema
hay fiebre malestar general, cefalea, anorexia, dolor abdominal. El exantema tiene aparicin cefalocaudal. Es maculopapular pruriginoso, rodeadas
por un halo eritematoso que rpidamente evoluciona a vesicula con apariencia de gotas de agua, se umbilican al mismo tiempo que el contenido
se vuelve turbio constituyendo a las pustulas, que posteriormente forman costras. Brites que permiten visualizar todos los estadios. Diagnostico:
Clinico. Tincin de Tzank de material vesicular, clulas gigantes multinucleadas, PCR, cultivos, ELISA, inmunofluorescencia. Complicaciones: Infeccion
bacteriana, meningitis, cerebelitis, raramente encefalitis, encefalomielitis, purpura trombocitopenica. PAROTIDITIS: Virosis (paramyxoviridae, RNA)
glandular que denota tumoracin de glndulas salivales, en particular las partidas, altamente contagiosa, afecta mas a nios de 5-15 aos, se
contagia persona a persona, fmites, orina, en saliva hasta 7 dias antes y 9 despues de surgida la tumefaccin (periodo infectante). Glandula
edematosa y aumentada de volumen, en caso de orquitis hay edema e infiltrado perivascular de linfocitos. Incubacion 14-21 dias. 5 a 7 dias antes
malestar general, dolor en partida, fiebre 38-39, tumefaccin de la glandula uni o bilateral, dolor refiere como de odo y a la masticacin, siendo
progresivo puede persistir hasta 2 semanas, adenomegalias en cadena cervical, meningoencefalitis (10%), epididimoorquitis (30-38%), ooforitis,
pancreatitis. Diagnostico: clnico, IgM positiva, persiste IgG por tiempo indefinido. Tx: sintomtico, hidratacin, analgsicos, compresas frias o tibias,
no sustancias acidas. Complicaciones: sordera, neuritis facial, mielitis, encefalitis, miocarditis, artritis. SINDROME DE PIE-MANO-BOCA: 6meses3aos, verano-otoo, Coxsackie A16, mecanismo de transmisin fecal-oral u oral-oral, por gotas de respiratorias es menos fecuente. Periodo de
incubacin de 2-14 dias. El periodo prodrmico dura de 2-4 dias, caracterizado por fiebre de bajo grado, anorexia, odinofagia y dolorabdominal. El
exantema vesculas ovaladas con localizacin dorsal y lateral de los dedos, asi como en la superficie palmar y plantar; que a menudo se disponen
paralelamente a los dermatomas, rodeados por un halo eritematosos, puede haber prurito o ser asintomtico, tambin se acompaa pornulceras
dolororsas en mucosas (paladar, lengua, carrillos). Desaparece de 5-10 dias de haber aparecido. La mucosa oral tienen vesculas que se erosionan y
forman ulceras, muy dolorosas. ENFERMEDAD DE LYME: Borreliosis, por Borrelia burgdorferi, se transmite por la mordedura de garrapatas, eritema
migratorio, circular, papular, no dolorosa, en el sitio de la mordedura, con diseminacin centrifuga, que progresa a lesiones generalizadas maculares,
se desarrolla de 3-16 semanas posteriores a mordedura, se acompaa de fiebre, cefalea, fatiga y artralgias.
CASO CLINICO
Paciente masculino de 2 aos, el cual es
llevado por la madre al centro de salud,
por referir que inicio hace 4 das con
febrculas de 37.5-37.9, comezn
moderada, y que no quera comer, el da
de hoy la madre refiere que inicio con
unas ronchas en el cuerpo, y que le toco
unas bolitas por detrs de la oreja y en
la nuca, las lesiones iniciaron en la cabeza
y despus empezaron a salir en el trax y
despus en las piernas. EF: paciente
orientado y consciente, piel y tegumentos
de buena coloracin, conjuntivitis leve, as como rinorrea, faringe
ligeramente hiperemia con presencia de petequias en paladar y las
siguientes lesiones en el cuerpo.

PREGUNTA
Con el cuadro clnico y la imagen. Cul es el diagnstico ms
probable?
RESPUESTA
a.- Sexta enfermedad
b.- Escarlatina
c.- Enfermedad de Kawasaki
d.- Eritema infeccioso

PREGUNTA
Cul es el diagnstico ms probable para este paciente?
RESPUESTA
a.- Varicela
b.- Rubeola
c.- Exantema sbito
d.- Sarampin
PREGUNTA
A partir de que da el paciente ya puede ir a la escuela y no
contagiar?
RESPUESTA
a.- Al tercer da de haber aparecido el exantema
b.- A partir del 8vo da de que apareci el exantema
c.- 15 das despus de aparecido el exantema
d.- 7 das despus de terminado el exantema
PREGUNTA
Cul es el periodo de incubacin para esta patologa?
RESPUESTA
a.- 14-20 das
b.- 30-60 das
c.- 5-7 das
d.- 10-15 das

CURSO ENARM CMN SIGLO XXI TEL: 36246001

CASO CLINICO
Nia de 6 aos con sntomas
catarrales y fiebre de 38,5
de 4 das de evolucin, no
objetiva foco bacteriano y se
realiza
tratamiento
sintomtico con ibuprofeno.
Cuatro das ms tarde la
madre lo trae debido a que
presento unas ronchas, pero
la madre refiere que se
agudiza con luz solar y el calor. A la EF: se observan las siguientes
lesiones de la imagen, con aumento de temperatura al tacto, no
dolorosas.

PREGUNTA
Cul es el agente etiolgico ms probable para este caso?
RESPUESTA
a.- Streptococo pyogenes
b.- Parvovirus B19
c.- Desconocida
d.- HHV 6
PREGUNTA
Cul es la conducta teraputica ms adecuada para esta paciente?
RESPUESTA
a.- Es benigna y autolimitada y no requiere tratamiento
b.- Penicilina procainica
c.- Sintomtico
d.- Antiviral

Pharmed Solutions Institute

PGINA 347

MANUAL DE TRABAJO DEL CURSO ENARM CMN SIGLO XXI


PREGUNTA
Qu complicaciones sera ms probable encontrar?
RESPUESTA
a.- Hematolgicas
b.- Renales
c.- Cutneas
d.- Pulmonares

RESPUESTA
a.- Sarampin
b.- Sexta enfermedad
c.- Quinta enfermedad
d.- Escarlatina
PREGUNTA
Cul es el periodo de incubacin de esta patologa?
RESPUESTA
a.- 20-30 das
b.- 1-7 das
c.- 14-21 das
d.- 5-20 das

CASO CLINICO
Nia de 4 aos que se queja de
odinofagia y tiene fiebre de 39 C,
nuseas y dolor abdominal leve. A la
exploracin: paladar blando y pilares
anteriores con lesiones petequiales,
la lengua como se aprecia en imagen
. Adems tiene un exantema
puntiforme, que corresponde al signo de Pastia.

PREGUNTA
Cul es la conducta teraputica ms adecuada a seguir en este
momento?
RESPUESTA
a.- Penicilina
b.- Antiviral
c.- Ibuprofeno
d.- Eritromicina

PREGUNTA
Cul es el diagnstico ms probable para este caso?

HEPATITIS
CIENCIAS BASICAS: Indica proceso inflamatorio del hgado caracterizado por hepatomegalia, anorexia, molestis abdominales fundamentalmente
gstricas, funcin heptica anormal, heces despigmentadas y orina oscura. Estas alteraciones pueden ser ocasionadas por infecciones producidas
por bacterias, virus o parsitos o bien por ingestin excesiva de alcohol o frmacos, trastornos de tipi autoinmunitario o toxico. La mayor parte de las
hepatitis son de origen viral. La hepatitis crnica se define como una lesin inflamatoria del hgado que puede progresar a lesin mas grave y a
cirrosis, las causas ms frecuentes son: virales, autoinmunes, toxicas y criptogenicas. En general se habla de cronicidad con una evolucin de ms de
6 meses, con excepcin del caso de las hepatitis autoinmunes, en las cuales un diagnstico ms temprano, permitir detectar el padecimiento antes
de que se desarrolle cirrosis. Se sospecha una hepatitis crnica ante una sospecha de recada de hepatitis aguda, persistencia de hepatitis aguda por
ms de 3 meses, datos clnicos de enfermedad heptica crnica y en pacientes asintomticos de alto riesgo, con historia de transfusiones mltiples,
como son pacientes con leucemias en remisin, hemofilia, hemoglobinopatas, IRC en programa de dilisis, y neoplasias en los que existe el riesgo de
hepatitis post- transfusional. SALUD PUBLICA: En Mxico que 95% de las hepatitis en nios menores de 215 aos de edad es producida por el virus
de hepatitis A. Cada ao se registran aproximadamente 1,4 millones de casos de hepatitis A en todo el mundo. La hepatitis A se asocia a falta de
agua salubre y a un saneamiento deficiente. Las epidemias se pueden propagar de manera explosiva y causar prdidas econmicas considerables.
Las mejoras del saneamiento y la vacuna contra la hepatitis A son las medidas ms eficaces para combatir la enfermedad. HEPATITIS POR VIRUS A:
La seccin heptica ms frecuente, pertenece al grupo de los picornavirus de los enterovirus, y su husped natural es el ser humano, el genoma est
constituido por RNA. La hepatitis A se presenta en nios entre los 3-12 aos de edad, con promedio en la edad preescolar. La va de transmisin ms
comn es la del tubo digestivo a travs de alimentos o bebidas que se contaminan con matera fecal que contienen el virus. La contaminacin por va
area no se ha confirmado. La transmisin tambin se da en los drogadictos que comparten agujas y jeringas para uso intravenoso. El mayor riesgo
de diseminacin del virus es en heces, ocurre 15 das posteriores al inicio de la enfermedad. El estado de portador no existe. La inmunidad con la
infeccin es permanente. No se ha informado hasta la fecha evolucin a la cronicidad. Clnica; Los nios menores de 2 aos de edad, cursan sin
ictericia en 90% de los casos. Antes de la aparicin de la ictrica se presentan los siguientes sntomas, febrcula, anorexia, vomito, debilidad, cefalea,
dolor difuso en abdomen y esplenomegalia. En la fase ictrica cuya duracin puede ser de 8 das a 4 semanas, se hace aparente coluria, hipocolia,
decaimiento y anorexia ms acentuadas, con presencia de tinte amarillento en conjuntivas y piel. Laboratorio: Aumento de la bilirrubinas a expensas
de la bilirrubina directa, las transaminasas, se encuentran elevadas, la colinesterasa y la colestasa estn disminuidas, las pruebas de floculacin de
protenas como es la turbidez al timol se encuentra elevada, ocurre aumento de la gammaglobulina con disminucin de albumina. Se encuentran
alteradas la concentracin de inmunoglobulinas los tiempos de protrombina y la concentracin de glucosa. Las pruebas serolgicas especficas como
es la deteccin de anticuerpos contra el virus de la hepatitis A por radioinmunoanlisis, sugieren infeccin reciente o convaleciente. Tratamiento:
Las medidas son de tipo sintomtico, ya que no existe medicamentos especficos para el virus de hepatitis A. Dieta; implica restriccin de protenas,
de grasas y aumento en los hidratos de carbono. Reposo; mnima actividad para mantener un flujo circulatorio esplacnico y heptico adecuado.
Aislamiento y proteccin a contactos; precauciones con el manejo de excretas durante el periodo ictrico y 2 semanas despus. Aplicacin de
gammaglobulina estndar al 16% a todo contacto a razn de 0.2 a 0.12 ml/kg por va IM en dosis nica como inmunizacin pasiva. Inmunizacin:
Debe aplicarse la vacuna contra hepatitis A en presentacin peditrica, dos dosis con intervalos de 1 mes con refuerzo a los 12 meses, la vacuna
disponible en Mxico precede de la cepa HM75 inactivada. Son 360 unidades en 0.5ml y se aplica va IM en nios mayores de 1 ao de edad. VIRUS
DE HEPATITIS D: Se trata de un virus RNA defectuoso, formado por RNA del agente, cubierto por antgeno de superficie del virus de la hepatitis B,
sin este virus no resulta infectante. Se transmite por va parenteral o va percutnea, en mucosas, transfusin sangunea, contacto sexual y etapa
perinatal. La diseminacin es por portadores y afecta ms al adolescente. Su periodo de incubacin es de 15-200 das. El 75% del diagnstico inicial
se hace en la fase cirrtica y por lo tanto tiende a la fase de cronicidad. En laboratorio a parte de la BH, transaminasas, bilirrubinas y EGO, debe
hacerse cuantificacin de anticuerpos antivirus de la hepatitis (anti-VHD) en donde se encuentran titulaciones mayores de 1:100, las titulaciones
de IgM se encuentran altas en fase temprana, adems se cuantifican anticuerpos del virus de la hepatitis B (Ags-VHB). Tratamiento: Se ha utilizado
el interfern sin obtener resultados satisfactorios. No existe vacuna alguna, sin embargo algunos autores sugieren usar la vacuna contra el virus de la
hepatitis B.
CASO CLINICO HEPATITIS
RN pretrmino de 36,6 semanas de EG, peso 2.555 g, por depresin
neonatal y riesgo infeccioso. APGAR 5/9. La madre present, 24 h

CURSO ENARM CMN SIGLO XXI TEL: 36246001

antes del parto, un cuadro febril acompaado de dolor abdominal.


Por la sospecha clnica de corioamnionitis recibi tratamiento

Pharmed Solutions Institute

PGINA 348

MANUAL DE TRABAJO DEL CURSO ENARM CMN SIGLO XXI


antibitico y se indujo el parto, que finaliz en cesrea con anestesia
general por no progresin.
PREGUNTA
Cual es la conducta a seguir mas adecuada?
RESPUESTA
a.- Egresar con vigilancia estrecha.
b.- Hemocultivo e inicio de antibitico.
c.- BH, QS, EGO.
d.- Analisis de LCR.
PREGUNTA
En las siguientes 48 h se deteriora su estado general, con acidosis
metablica, anemia, plaquetopenia (recuento mnimo 10.000/mcl) y
hemorragia digestiva baja, por lo cual se modifica el tratamiento
antibitico, para cubrir una posible enterocolitis necrozante, y se
suspende la alimentacin oral. En las 24 h posteriores desarrolla
coagulopata y fallo heptico; presenta petequias generalizadas y
sangrado importante por puntos de canalizacin venosa umbilical y
perifrica. Cual es la conducta a seguir?
RESPUESTA

a.- Transfusin de hemoderivados.


b.- Administracin de vitamina K (1 mg/kg/da) durante 72 h.
c.- Pruebas de funcin heptica.
d.- Administracion de albumina.
PREGUNTA
Se obtienen los siguientes resultados donde alcanza concentraciones
mximas de transaminasas GOT 957 U/L, GPT 237 U/L y GGT 144
U/L, FA 93 U/L, LDH 5031 U/L, bilirrubina directa: 1,66 mg/dl, con
disminucin de las protenas totales (3,75 g/dl) y albmina (2,19
g/dl). Por la sospecha clnica de viremia, y ante la posibilidad de
infeccin herptica, se inicia tratamiento con aciclovir. Cual de los
siguientes patologias es el mas frecuente evaluar para el diagnostico
diferencial?
RESPUESTA
a.- Hepatitis neonatal.
b.- Infeccion por VEB, CMV.
c.- Enterovirus y adenovirus.
d.- Toxoplasma, rubola y sfilis.

TORCH
CIENCIAS BASICAS: Las infecciones congnitas son aquellas transmitidas por la madre a su hijo antes del nacimiento. En este captulo se tratarn las
infecciones que clsicamente se han agrupado en el acrnimo TORCH T: toxoplasmosis, R: rubeola, C: citomegalovirus y H: herpes. Segn algunos
autores O correspondera a otras infecciones entre las que inicialmente se incluyeron varicela y sfilis, pero que en la actualidad pueden englobar
parvovirus B19, papilomavirus, malaria y tuberculosis. PATOGENIA: Todas ellas tienen rasgos comunes: La transmisin puede ocurrir por va
transplacentaria o por contacto directo con el patgeno durante el parto. La fuente de infeccin fetal es la viremia, bacteriemia o parasitemia que se
produce en la mujer embarazada durante una primoinfeccin, que suele ser ms infectiva para el feto, o durante una infeccin crnica.
DIAGNOSTICO: La enfermedad suele pasar inadvertida o ser paucisintomtica en la madre, salvo en madres inmunocomprometidas en las que estas
infecciones son ms frecuentes y graves. El diagnstico es serolgico o por tcnicas de biologa molecular (reaccin en cadena de la polimerasa o
PCR) o cultivo celular. La expresin clnica es similar en todas ellas, pero con amplio margen de variabilidad. En general cuando la infeccin ocurre
antes de las 20 semanas, es ms grave y ocasiona malformaciones mltiples. Si tiene lugar en pocas posteriores, durante el perodo fetal, puede ser
causa de prematuridad, bajo peso, alteraciones del sistema nervioso central, etc. Y si ocurre poco antes del parto puede presentarse en forma de
sepsis con mal estado general, ictericia, hepatoesplenomegalia, neumonitis y en la analtica sangunea suelen aparecer anemia y trombopenia.
Finalmente algunas de ellas pueden ser asintomticas en el perodo neonatal y producir secuelas sobretodo neurosensoriales en pocas posteriores
de la vida. TOXOPLASMOSIS: La infeccin materna por Toxoplasma gondii se adquiere principalmente por ingestin de quistes de vegetales y frutas
mal lavados o carne cruda o poco cocinada, al limpiar excrementos de gato (nico husped comprobado) o al realizar trabajos de jardinera sin
guantes. Slo un 10% de la mujeres inmunocompetentes que se infectan presentan sintomatologa, usualmente leve e inespecfica o puede dar lugar
a un cuadro mononuclesico. Se transmite al embrin o al feto durante la fase de parasitemia materna y est aceptado que esta transmisin slo
tiene lugar, en las gestantes no inmunocompetentes, durante la primoinfeccin. Cuanto ms precoz sea la infeccin en el embarazo menor ser el
riesgo de transmisin fetal (10-20% en el primer trimestre, 25-30% en el segundo y 60-80% en el tercero), pero las consecuencias para el feto sern
ms graves si la infeccin es precoz, que si se trasmite en fases tardas. El RN puede presentar varias formas clnicas: Una minora (5%) presentan una
forma sistmica inicial que aboca a una fase de secuelas con la ttrada sintomtica de Sabin (hidrocefalia, calcificaciones intracraneales,
convulsiones y coriorretinitis), en general se trata de infecciones adquiridas antes de las 20 semanas. Si la infeccin es tarda pueden objetivarse
meningoencefalitis, fiebre, hepatoesplenomegalia, ictericia, exantema, neumonitis y diarrea, y en la analtica sangunea suelen aparecer: anemia,
trombopenia y eosinofilia. Otros (10%) pueden presentar lesiones aisladas del SNC u oculares de pronstico variable. Alrededor del 85% de los RN
infectados estarn asintomticos al nacer, pero de ellos un 20-30% pueden desarrollar afectacin neurolgica y coriorretinitis a medida que el nio
crece (hasta los 20 aos) si no reciben tratamiento. Diagnstico: en la gestante el diagnstico se realiza mediante la deteccin de seroconversin o
aumento significativo de ttulos de anticuerpos IgG (en dos determinaciones separadas 2-3 semanas y realizadas en el mismo laboratorio), presencia
de IgG de baja avidez y aparicin de ttulos elevados de IgM determinados por enzima-inmunoensayo, que se empiezan a detectar unas 2 semanas
despus de la infeccin, alcanzan su pico mximo a las 4-6 semanas y posteriormente declinan hasta los 6-9 meses, aunque ttulos bajos de IgM
pueden persistir durante aos en algunas pacientes. En el recin nacido el diagnstico se realiza ante la presencia de IgM especfica, pero la
sensibilidad es inferior al 50%. En su ausencia el diagnstico se basa en el mantenimiento de las IgG una vez aclaradas las IgG maternas
transferidas a travs de la placenta (unos 6-12 meses, en funcin del ttulo) o mediante tcnicas de PCR en sangre y en LCR, aunque presentan una
sensibilidad muy baja. Tratamiento: a) De la embarazada: Desde la sospecha de infeccin hasta el diagnstico por PCR en lquido amnitico se le
administrar espiramicina. Si se confirma el diagnstico de infeccin fetal (PCR positiva y/o ecografa alterada) a partir de la semana 20, se
recomiendan ciclos de pirimetamina ms sulfadiacina y cido folnico en forma continua hasta el final del embarazo. Los resultados sobre la utilidad
del tratamiento durante la gestacin son contradictorios, pero estudios recientes demuestran que la administracin precoz de estos frmacos,
disminuye de forma significativa la transmisin ver tical del parsito. b) Del RN: Si la toxoplasmosis es manifiesta o si la IgM o la PCR resultan
positivas, se administrarn: pirimetamina (Daraprim) ataque 2 mg/kg/da en dos rante 2-6 meses y luego a das alternos; y sulfadiacina (Sulfadiazina)
100mg/kg/da en 2 dosis VO, y cido folnico (Leucovorin clcico) 10 mg/3 veces por semana, durante un ao.
CASO CLINICO
Recin nacido de trmino de sexo masculino, hijo de padres sanos
no consanguneos y madre de 28 aos de edad con antecedente de
aborto previo 6 aos atrs por huevo anembrionado. Embarazo
controlado con ecografas seriadas, a las 34 + 5 semana de gestacin

CURSO ENARM CMN SIGLO XXI TEL: 36246001

se pesquisa hidrocefalia marcada y se inici estudio de TORCH con


serologa materna positiva para infeccin activa.
PREGUNTA
Cul es la conducta teraputica mas adecuada a seguir?

Pharmed Solutions Institute

PGINA 349

MANUAL DE TRABAJO DEL CURSO ENARM CMN SIGLO XXI


RESPUESTA
a.- Espiramicina
b.- Pirimetamina mas sulfadiazina
c.- Ac. Folinico mas gentamicina
d.- Solo medidas de control

d.- Pirimetamina, mas gentamicina mas sulfadiacina

PREGUNTA
Paciente naci por cesrea programada a las 40 semanas debido a
macrocefalia, Apgar 8-9 con peso de nacimiento 4 070 gramos, talla
52 centmetros y permetro craneano PC: 39 centmetros (> + 2 DS).
En buenas condiciones se hospitaliz para estudio. Cul es la
conducnta teraputica a seguir en el paciente?
RESPUESTA
a.- Pirimetamina mas sulfadiacina mas leucovorin
b.- Ac. Folinico mas espiramicina
c.- Leucovorin mas pirimetamina

PREGUNTA
Durante el tratamiento se observa alza leve de transaminasas que se
mantuvieron estables, sin mayor compromiso heptico ni medular.
TAC cerebral sin contraste mostr hidrocefalia triventricular
acentuada y marcada atrofia del parnquima cerebral. Lquido
cfalo-raqudeo por puncin ventricular, mostr protenas 974
mg/dl, pandy +++, glucorraquia 26 mg/dl, glbulos blancos 8,
predominio mo-nomorfos y glbulos rojos 30. Cul seria la
conducnta mas adecuada a seguir?
RESPUESTA
a.- AINE
b.- Prednisona
c.- Derivacion ventricular
d.- Ampicilina mas gentamicina

PSORIASIS
CIENCIAS BASICAS: Es una enfermedad autoinmunitaria eritematodescamativa muy frecuente de evolucin crnica que cursa a brotes. Se
caracteriza por lesiones bien definidas con escamas tpicas blanco-nacaradas. SALUD PUBLICA: Afecta a 1-3% de la poblacin general.
Aproximadamente el 25% de los casos de psoriasis se inicia durante la infancia, y aumenta su incidencia con la edad hasta la adolescencia.
PATOGENIA: Es una enfermedad hereditaria aunque no se conoce todava el modo de herencia. Existe una predisposicin gentica compleja y
probablemente multifactorial. Hay una frecuencia aumentada de algunos haplotipos HLA. Los factores genticos influyen en el patrn de la psoriasis,
severidad y edad de inicio. El riesgo para los familiares de primer grado de un caso aislado es menor de 10%, pero si los dos padres tienen psoriasis
los hijos tienen un riesgo de un 50% de padecerla. Existen unos factores desencadenantes conocidos capaces de precipitar o exacerbar un brote. Las
infecciones (particularmente por Streptcoccus), el frio, el estrs emocional, los traumatismos cutneos repetidos y ciertos frmacos (cloroquina y
corticoides sistmicos). DIAGNOSTICO: Las lesiones de psoriasis infantil son placas eritemato-escamosas similares a las de la edad adulta, de tamao
y forma variables, con los mismos signos caractersticos al raspado y con tendencia tambin a desarrollar lesiones de psoriasis en las zonas de roce o
de rascados y traumatismos (fenmeno isomrfico de Koebner). La biopsia cutnea muestra una epidermis engrosada con acantosis y papilomatosis,
elongacin y edema de la dermis papilar y un adelgazamiento de la epidermis por encima de las papilas. Se han realizado numerosos estudios sobre
la calidad de vida en pacientes con psoriasis3, y se ha demostrado que repercute en actividades cotidianas y en el bienestar fsico y psicolgico de los
enfermos. TRATAMIENTO: Tpico: Emolientes y queratolticos: No tienen efectos secundarios, alivian el prurito y disminuyen la descamacin,
aunque es el tratamiento menos eficaz. Son tiles para conseguir eliminar escamas importantes y as facilitar la penetracin de otros productos ms
efectivos. Uno de los ms utilizados es la vaselina saliclica al 3-5%. El alquitrn de hulla es la brea ms utilizada; en nios se emplean los baos de
breas slo en casos muy extensos, por su efecto calmante y reductor. La antralina (dithranol) al 0,1% parece que es menos irritante y es til en
placas muy queratsicas; carece de toxicidad general. Se ha demostrado adems que los tratamientos en aplicaciones cortas son efectivos y menos
irritantes, mediante aplicaciones diarias de 10-30 min y aumentando progresivamente la concentracin del 0,1 hasta el 2%. Corticoides: tienen en la
psoriasis una triple accin: antiinflamatoria, inmunosupresora y antiproliferativa. Su uso se ve limitado por los efectos secundarios: efectos locales
como atrofia, estras, telangiectasias e hipertricosis a menudo irreversibles; las erupciones acneiformes. En pliegues, genitales y cara causan ms
efectos secundarios locales que en el resto del tegumento. Efectos sistmicos: la absorcin de corticoides potentes a travs de la piel puede
provocar un Cushing iatrognico e incluso detencin del crecimiento. La vitamina D como tratamiento para la psoriasis, dado que induce la
diferenciacin de queratinocitos y disminuye su proliferacin. Calcipotriol. Proporciona buenos resultados en nios sin alterar la concentracin
srica de calcio, siempre que no se sobrepasen los 50 g/semana. Inmunomoduladores tpicos: El tacrolimus y el pimecrolimus actan inhibiendo la
calcineurina y mejoran las placas de psoriasis poco hiperqueratsicas. Si son muy hiperqueratsicas, se requiere la aplicacin previa de un
queratoltico tpico para facilitar su absorcin. Psoralenos + radiacin ultravioleta A (PUVA): La PUVAterapia est basada en la interaccin entre la
radiacin ultravioleta A (UVA) y un agente fotosensibilizante qumico: los psoralenos, pero no se recomienda su empleo en nios menores de 10
aos, ya que se acumulan en el cristalino y si ste est an en desarrollo puede causar daos irreversibles; los rayos UVA penetran hasta el cristalino
y pueden inducir la aparicin de cataratas, por lo que hay que extremar la proteccin ocular durante la radiacin. La alta permeabilidad del cristalino
en los jvenes contraindica este tratamiento antes de los 12 aos. Por otra parte, son ya conocidos los efectos carcinognicos a largo plazo de la
radiacin UVA15-17. Tratamiento sistmico. Retinoides, metotrexato, ciclosporina. Tratamientos biolgicos: Son anticuerpos monoclonales o
protenas humanas recombinantes contra algunas citocinas: etanercept, infliximab, adalimumab (antifactor de necrosis tumoral), efalizumab. Hay
pocos casos en nios, aunque con buenos resultados, pero de uso experimental y todava no est aprobado.
CASO CLINICO PSORIASIS EN GOTA
Varn de 17 aos que, tras pasar una amigdalitis, presenta este
exantema cutneo. A la EF se observan papulas y pequeas placas
eritematosas con minima descamaciones distribuidas de forma
generalizada, pero principalmente en el tronco.
PREGUNTA
Considernado el tipo de psoriasis que presenta el caso. Cul de las
siguientes medidas es la ms adecuada para intervenir en el agente
casual?
RESPUESTA
a.- Corticoide tpico de mediana potencia, en excipiente no
demasiado graso.

CURSO ENARM CMN SIGLO XXI TEL: 36246001

b.- Realizar baos con derivados de alquitrn.


c.- Antibiotico especifico preferentemente por cultivo.
d.- Exposicin solar moderada y frecuencia.
CASO CLINICO
Varn de 28 aos, fumador de 60 cigarrillos al da, bebedor de
riesgo. Trabajador de la construccin. Antecedente familiar de
madre con espondilitis anquilosante HLA-B27+. Antecedentes
personales de clicos nefrticos y monoartritis de rodilla
seronegativa un ao antes. Acude a consulta por presentar una
herida en la pierna sobreinfectada tras un traumatismo; se trata con
amoxicilina ms cido clavulnico, y evoluciona de forma trpida.
Tres semanas ms tarde acude con una lesin en placa eritematosa,

Pharmed Solutions Institute

PGINA 350

MANUAL DE TRABAJO DEL CURSO ENARM CMN SIGLO XXI


con borde definido con una escama color plata, refiere que no haba
acudido antes porque tiene mucho trabajo. La prueba de
tuberculina, as como el cultivo de hongos y micobacterias, fueron
negativos. Con posterioridad aparecieron lesiones ms pequeas
con escamas de las mismas caractersticas en codos y cuero
cabelludo, un poco pruriginosas.
PREGUNTA
Cul es el diagnstico ms probable para este caso?
RESPUESTA
a.- Tia corporis
b.- Dermatitis atpica
c.- Psoriasis
d.- Pitiriasis liquenoide
PREGUNTA
Qu fenmeno presento este paciente que pudo ayudar al
desarrollo de su patologa?
RESPUESTA
a.- Signo de Nikolsky

b.- Signo de Auspitz


c.- Fenmeno de Koebner
d.- Fenmeno de Shwartzman
PREGUNTA
Cul es la conducta teraputica ms adecuada en este caso?
RESPUESTA
a.- Ac. Saliclico + Hidrocortisona al 1%
b.- Urea + hidrocortisona 1%
c.- Fluocinolona+ Neomicna
d.- Fluocinolona + Lindano
PREGUNTA
Si el paciente presenta otro cuadro de artritis. Cul sera el manejo
ms adecuado en este caso?
RESPUESTA
a.- Metotrexate mas etarnecep
b. Etarnacep mas diclofenaco
c.- Metotrexato mas tramadol
d.- Tramadol mas etarnecep

ACNE
CIENCIAS BASICAS: Sinnimos acn vulgar, acn polimorfo o acn juvenil. Es la inflamacin crnica folicular producida por retencin de sebo,
debida a obstruccin y distensin de la unidad polisebacea. SALUD PUBLICA: El acn vulgaris es la enfermedad ms comn de la piel, afectando al
80% de las personas entre los 11 y los 30 aos de edad en algn momento. Adems, a nivel de los adolescentes afecta ms del 85%. Puede persistir
por aos y generar cicatrices permanentes y conllevar un dao importante a nivel del desarrollo
psicosocial y un problema de autoestima. Aunque se observa en ambos sexos, predomina ligeramente
en el hombre. PATOGENIA: La susceptibilidad a la seborrea est determinada genticamente. Los
factores patgenos fundamentales son: queratinizacin folicular anormal, secrecin sebcea
aumentada, colonizacin bacteriana e inflamacin local. No hay influencia prctica de la dieta. Al llegar
la pubertad, por aumento en la sensibilidad de las glndulas sebceas, las hormonas andrgenas
generan aumento de tamao y de la secrecin de las mismas, lo que aunado a la hiperqueratosis de la
pared y el poro foliculares, originan la lesin fundamental e inicial del acn, el comedn (espinilla). Se
ha demostrado la presencia de Propionibacterium acnes, quien transforma los triglicridos del sebo en
cidos grasos libres, que tienen efecto irritante local, favorece la inflamacin por este mecanismo. Si la
inflamacin se realiza en la parte terminal del conducto folicular, a nivel de la epidermis, se origina una
pstula, pero si se hace en la parte proximal del folculo y en la misma glndula sebcea, se originan los
abscesos. Los factores psicgenos pueden agravar esta dermatosis de modo secundario, sobre todo en
aquellos pacientes que presentan excoriaciones y ulceraciones autoprovocadas. DIAGNOSTICO: Afecta
cara (99%), cuello y trax (espalda 60%, pecho 15%), sobre todo en su parte superior. En cuanto a la
morfologa, el acn se caracteriza por polimorfismo lesional encontrndose lesiones no inflamatorias
que son los comedones abiertos y cerrados; el comedn abierto se reconoce como una concrecin gris oscuro, caf o negro dentro de un orificio
folicular dilatado y el comedn cerrado es una pequea ppula del color de la piel. Las lesiones inflamatorias son ppulas eritematosas, pstulas y
ndulos. Las secuelas del acn son cicatrices y senos epiteliales. TIPOS: De lesiones acneicas en cuadro anexo. CLASIFICACION: De acuerdo al tipo de
lesin predominante, el acn se clasifican en: comednico, papuloso, pustuloso, qustico, conglobata (abscesos y fstulas), queloideo y fulminans
(ppulas, ndulos y qustes acompaados de datos de infeccin sistmica). De acuerdo a la gravedad del
acn, se clasifican en: a) ACNE LEVE: Predominan los comedones. b) ACNE MODERADO: Con predominio de
ppulas y pstulas inflamatorias. c) ACNE GRAVE: Que consta de ndulos (abscesos}) y de ppulas, pstulas
y comedones. TRATAMIENTO: medidas generales; explicar al paciente claramente la naturaleza de la
enfermedad, los posibles efectos adversos de la terapia y su duracin habitualemnte prolongada, para
evitar las expectativas a corto plazo y el abandono teraputico. Detectar posibles factopres gravantes o
predisponenetes (medicaments, sustancias qumicas). Desaconsejar la manipulacin de las lesiones e
insistir en una adecuada limpieza e higienen cutnea. Evitar el uso de cosmticos comedogenicos (cremas,
lociones, geles, maquillajes) y recomendar como alternativa interesante los cosmticos libres de grasa (oil
free). El tratamiento tpico es el pilar fundamental en el que se debe basar el manejo del acn en atencin primaria. Considerando la clasificacin
clnica anterior, el manejo se efectuar de la siguiente manera. Acn leve. Jabones neutros o azufrados para el lavado dos veces al da y aplicacin de
lociones desengrasantes como el licor de hoffman, a las que pueden agregarse cido saliclico, resorcina o azufre a 1 y 3%. Pueden usarse
comedolticos como retinoides tpicos o perxido de benzoilo. Acn moderado: puede usarse clindamicina o eritromicina tpicas, en solucin o gel,
dos veces al da o valorar el uso de antibiticos sistmicos aunado al tratamiento tpico. Acn grave: Los antibiticos sistmicos son de primera
eleccin para el acn moderado. Acn grave: los antibiticos sistmicos son de primera eleccin para el acn moderado a grave, los ms
recomendados son las tetraciclinas, eritromicina, minociclina y el TMP/SFX, por uno a tres meses. La diaminodifenilsulfona (DDS) da muy buenos
resultados en acn conglobata. La isotretinoina oral se utiliza en acn conglobata y ndulo qustico, pero su toxicidad es muy alta. Tratamiento
psiquitrico: La dismorfofobia es una entidad que se asocia con frecuencia en el acn y consiste en la percepcin exagerada de los padecimientos
cutneos. Los pacientes se preocupan por su fealdad imaginaria. Esto lleva a depresin clnica y a un trastorno obsesivo compulsivo o fobia social.
CASO CLINICO

CURSO ENARM CMN SIGLO XXI TEL: 36246001

Paciente de 15 aos de edad, de sexo masculino estudiante, que


consulta por presentar compromiso sistemico, con fiebre, astenia,

Pharmed Solutions Institute

PGINA 351

MANUAL DE TRABAJO DEL CURSO ENARM CMN SIGLO XXI


mialgias, anorexia y prdida de peso. El cuadro fue de inicio sbito;
presenta lesiones dermatolgicas que afectan pecho y espalda. Estas
lesiones son de aspecto granulomatosas que sangran fcilmente,
muy secretantes y necrosis dolorosas. Se solicitan anlisis de
laboratorio. Los resultados de estos estudios evidenciaron
leucocitosis, anemia, eritrosedimentacin elevada (VSG), cultivos
bacterianos de secrecin y sangre negativos.

Cual de las siguientes medidas tiene la menor evidencia dentro del


tratamiento de esta entidad?
RESPUESTA
a.- Eritromicina.
b.- Isotretinona
c.- Dapsona.
d.- Prednisona.

PREGUNTA

URTICARIA
CIENCIAS BASICAS: Es una reaccin vascular de la piel y mucosas caracterizada por ronchas, angioedema, donde ocurren procesos inflamatorios
importantes. SALUD PUBLICA: Frecuencia de 1-2% hasta 15 %, predomina en mujeres de 40-50aos, dermografismo en 5% de afectados, en 30% no
se conoce la causa, coincide 50% con angioedema.. CLASIFICACION: Urticaria Aguda <6 semanas de evolucin. Urticaria Crnica >6 semanas de
evolucin. Urticaria Aguda Recurrente: Episodios repetidos de brotes agudos con perodos asintomticos prolongados entre ellos. Otra; urticara,
dermografismo, edema angioneurotico. Inmunitaria y no inmunitaria. PATOGENIA: Alimentos: Huevo, leche, nueces man, mariscos, pescados,
fresas, kiwi, otros. Drogas: Penicilinas, ASA, AINES, vacunas, medios de contraste, productos sanguneos. Insectos: Abejas, avispas, hormigas.
Infecciones: Parsitos, bacterias, virus, hongos. Estmulos Fsicos: Frio, calor, presin, ejercicios, agua, sol. Aeroalergenos, alrgenos de contacto:
Polvo casero, plen, esporas, epitelios de mamferos. Enfermedades sistmicas LES, ARJ, enfermedad del suero, vasculitis, cncer, endocrinopatas,
Enf celaca. Otros A. Hereditario, amiloidosis, deficiencia de C3b, estrs, idioptico. El evento central en la urticaria-angioedema es la liberacin de
histamina desde unos grnulos preformados en las clulas cebadas. Existen mltiples factores que pueden provocar la degranulacion de las clulas
cebadas. En la urticaria aguda, el mecanismo principal es el de hipersensibilidad tipo I, IgE mediada, tambin conocida como la reaccin alrgica
clsica. En las primeras exposiciones que el paciente tuvo alrgeno se inici una respuesta Th2 mediada que resulto en la formacin de abundantes
cantidades de IgE especfica por parte de las clulas plasmticas especficas. Esta IgE es liberada a la circulacin sistmica y se fija de preferencia a
receptores de alta afinidad para IgE (FcR1), en la
superficie de la clula cebada . Un subsecuente contacto
con el mismo alrgeno provoca directamente el puenteo
de dos IgE especificas vecinas en la membrana de la CC, y
esto es la seal para la degranulacin de la CC. As se
liberan grandes cantidades de sustancias vasoactivas,
especialmente histamina, pero tambin leucotrienos,
factor derivado de plaquetas. Las respuestas principales
de estos mediadores son: prurito, vasodilatacin
(eritema), aumento en la permeabilidad vascular, reflejo
axonal. La exposicin al alrgeno de la urticaria puede ser
por cualquier va, incluyendo la oral, transdermica e
inhalatoria. URTICARIA AGUDA: Mas fecuente en nios.
Es frecuentemente por una reaccin alrgica mediada
por IgE a infecciones agudas generalmente del TRS y
drogas. Infecciones, virus: Enterovirus, Parainfluenza
1,2,3, Virus respirat. Sincitial, Adenovirus, rinovirus,
Influenza A y B, VEB, CMV, Herpes simple, Parvovirus
B19, Hepatitis A, B, C, Mycoplasma Neumoniae.
Parsitos: Oxiuro, Toxocara, Giardia lamblia. Bacterias:
Estreptococo beta hemoltico, H. pylori. URTICARIA
CRONICA: los factores fsicos como presin y fro son la
causa principal. Luego siguen infecciones: tracto urinario,
tracto respiratorio y las idiopticas. Existe un porcentaje
de Ac antitiroideos antiperoxidasa positivos en nios con
urticaria crnica y tambin hay mayor incidencia de enfermedad celaca en estos nios. DIAGNOSTICO: La urticaria se identifica por lesiones tpicas
(ronchas pruriginosas y angioedema) que tienen 3 caracteristicas bsicas: consisten en una inflamacin central eritematosa, levemente elevada
sobre el nivel de la piel, casi siempre rodeada por una piel eritematosa (reflejo axonal). Las lesiones son en esencia evanescentes, con una duracin
de menos de 24 hrs, que al desaparecer dejan una piel intacta. Con frecuencia son intensamente pruriginosas. La urticaria puede tomar diferentes
formas variando de lesiones puntiformes de 2 milimetros (point lesions), que cubren grandes reas de la piel (tpico aspecto de urticaria colinrgica),
lesiones aisladas de 1-2 cm o hasta lesiones confluyentes de varios centmetros, algunas pueden dar aspecto de piel de naranaja. Vasculitis
urticariana; son ronchas de larga evolucin, que dejan una pigmentacin violcea, fiebre, artropata, enfermedad o medicamento. Historia y examen
fsico. Pruebas para urticarias fsicas, prueba de suero autlogo, examen simple de orina y seriado de heces, hematologa completa y VSG, pruebas
tiroideas y Ac antitiroideos. TRATAMIENTO: 1. Medidas generales como evitar contacto con alrgeno, eliminar el alrgeno si es posible.
Implementar dieta de eliminacin. 2. Antihistamnicos: Primera lnea en el tratamiento de la urticaria. Actan por inhibicin competitiva de la
histamina bloquean receptores H1, control del prurito. Primera generacin sedantes (Dexclorfeniramina 0.15-0.2 mg/da QID, Hidroxicina 2 mg/da
QID, Ciproheptadina 0.25 mg/da BID, Difenidramina 5 mg/dia TID) y Segunda generacin No sedantes (Loratadina: 1-2 aos 1cc/dia; 2-5 aos
5mg/dia, Cetirizina : 6m-1ao 0,5 mg/dia; 2-5aos 2.5 mg/dia, Fexofenadina: 6-12 aos 120 mg/da, Ebastina : 6-12-aos 5-10 mg/dia,
Desloratadina: 2-5 aos 1,25 mg/dia; 6-11 aos 2,5mg, Levocetirizina: >2 aos 5/mg dia). Los de segunda generacin provocan liberacin de aminas
vasoactivas. Desloratadina: Tratamiento de rpida accin y bien tolerado por largo tiempo. Usado en casos de Urticaria crnica idioptica.
Levocetirizina: Enantimetro o metabolito activo de certirizina, receptor antagonista H1, rpido inicio de accin, pocos efectos colaterales, indicado
en rinitis alrgica y en UCI, efecto antiinflamatorio importante que aumenta su efecto teraputico en enfermedades alrgicas. Uso por tiempo
prolongado desde el ao de edad. Dosis: 5mg/dia. 3. Antileucotrienos: Segunda lnea en urticaria que no responde a antihistamnicos. Previene o

CURSO ENARM CMN SIGLO XXI TEL: 36246001

Pharmed Solutions Institute

PGINA 352

MANUAL DE TRABAJO DEL CURSO ENARM CMN SIGLO XXI


mejora los sntomas asociados a procesos mediados por leucotrienos C4, D4, y E4. Util en urticaria por medicamentos ASA y AINES. Montelukast 2-5
aos 4 mg/dia; 6-14 aos 5 mg/dia. Zafinlukast 5-11 aos 10 mg/BID. 4. Corticoides: Esteroides Sistmicos ayudan a disminuir la inflamacin. Solo
deben usarse en casos extremos y de angioedema, por corto tiempo para evitar efectos colaterales. Urticaria por presin y urticaria vasculitis.
Prednisona 1mg/Kg/dia por 5-7 dias. 5. Ciclosporina: En la tercera lnea de tratamiento. Es un inmunosupresor. Controles de funcionalismo renal.
Dosis: 3 mg/Kg de peso. 6. Omalizumab. Urticaria Aguda con sntomas sistmicos: Dificultad respiratoria-Evidencia angioedema, Adrenalina 1/1000
subcutnea a 0.01 mg/Kg, repetir cada 15 a 20 minutos, hidrocortisona 7-10 mg/Kg/dsis y luego instaurar ciclo corto, menor a 10 das,
clorfeniramina: IV. Dieta de exclusin de alimentos disparadores (man, pia, chocolate). Combinacin de anti-H1 (1ra y 2da) o Combinacin de antiH1 y H2 (o anti-LT).
CASO CLINICO
Paciente de 4 meses de edad, sin antecedentes patolgicos
conocidos, traido a consulta por presentar papulas generalizadas
pririginosas de 2 meses de evolucin, al EF mostraba papulas
eritematosas generalizadas en torax y miembros superiores e
inferiores que respetaban palmas y plantas. Dichas papulas, al ser
frotadas, tomaban aspecto de roncha rodeada por un halo
eritematoso. Segn refiri su madre sobre una de ellas se haba
formado previamente una ampolla, el paciente se encontraba afebril
y en buen estado general.
PREGUNTA
Cual es la conducta teraputica mas adecuada?
RESPUESTA
a.- Clorhidrato de hidroxicina.
b.- Maleato de clorfeniramina.
c.- Loratadina.
d.- Cetirizina.
CASO CLINICO
Nia de tres aos de edad, sin antecedentes personales ni familiares
de inters, que fue remitida por su pediatra a la consulta de
Alergologa por presentar, desde haca un ao, episodios recurrentes
de lesiones habonosas diseminadas pruriginosas, sin angioedema.
Los brotes aparecan cada uno o dos meses, solan persistir una
semana y a veces se acompaaban de odinofagia. Los padres no
relacionaban estos cuadros con ningn posible agente externo
desencadenante, entre ellos frmacos o alimentos. En la exploracin
fsica realizada durante su primera visita se objetiv una hipertrofia
de las amgdalas palatinas, sin otros hallazgos reseables. Las
pruebas cutneas con una batera estndar de neumoalrgenos
habituales de la regin (incluidos caros del polvo, plenes, hongos y
epitelios de animales), ltex, alimentos, panalrgenos vegetales
fueron negativas. La analtica general realizada (bioqumica,

hemograma, velocidad de sedimentacin globular, hormonas


tiroideas y orina) fue normal, as como las clases de
inmunoglobulinas sricas IgA, IgM e IgG. La cifra de IgE total fue de
104,5 UI/ml (valores normales de referencia, 100-120 UI/ml). Los
niveles de anticuerpos antiestreptolisina O fueron de 1571 UI/ml
(valor de referencia del laboratorio, 0-200 UI/ml). En el examen de
heces no se observaron parsitos.
PREGUNTA
Cul es la conducta teraputica ms adecuada a seguir en este
momento?
RESPUESTA
a.- Loratadina mas prednisona
b.- Penicilina G benzatinica
c.- Hidroxicina
d.- Bnezoato de bencilo
PREGUNTA
Cul de los siguientes medicamentos es menos probable que tenga
efecto sedativo?
RESPUESTA
a.- Difenhidramaina
b.- Bromfeniramina
c.- Cetirizina
d.- Hidroxicina
PREGUNTA
Cules son las caractersticas ms probables del anigioedema?
RESPUESTA
a.- Edema de dermis profunda, pruriginoso
b.- Edema de dermis superficial, no pruriginoso
c.- Edema sbito de dermis profunda y tejido celular subcutneo, no
pruriginoso
d.- Edema superficial y tejido celular subcutneo, pruriginoso

PRURIGOS
CIENCIAS BASICAS: El prurigo infantil o tambin llamado urticaria papular es una reaccin alrgica producida por la picadura de insectos. Esta
afeccin netamente peditrica no debe ser confundida con una alergia a los alimentos; se caracteriza por el prurito intenso de difcil control, con
brotes frecuentes y sucesivos que por lo general se autolimitan en el tiempo. SALUD PUBLICA: Afecta en su mayora a nios entre los dos y diez
aos, frecuentemente en los meses de primavera y verano. Cosmopolita, mas en pases tropicales, nivel socioeconmico bajo. Etiologia mas
frecuente: Chinche (Cimex lectularius), pulga (pulex irritans), moscos, tromnidias, garrapatas. TIPOS: La palabra prurigo define un proceso patolgico
cuya lesin elemental es la ppula y el sntoma principal es el prurito. El espectro clnico abarca un rango que va desde las ppulas (prurigo papular),
ndulos (prurigo nodular) entre otros tipos de prurigo podemos mencionar al actnico, el atpico o de Besnier, el de Sutton y el pigmentoso.
DIAGNOSTICO: Se observa una erupcin estacional y recurrente, con presencia de grupo de ppulas y vesculas pruriginosas de localizacin
frecuente en antebrazos, abdomen, rea lumbar, glteos, muslos y piernas. La respuesta va a depender del tipo de paciente y la calidad del inculo,
pudiendo tambin observarse formas ampollares. PRURIGO SIMPLE POR INSECTOS O URTICARIA PAPULOSA: La naturaleza alrgica de la condicin
fue demostrada cuando los cambios histolgicos producidos por picaduras de insecto a nivel experimental demostraron que eran idnticos a los de
la urticaria papular. Afecta principalmente a nios de 1 a 7 aos, siendo una de las causas ms frecuentes en la dermatologa peditrica, afecta igual
a ambos sexos y a cualquier etnia. Numerosos insectos causan prurigo, siendo los ms frecuentes el Cemex lectularius (chinche) produciendo la
Cimiasis; la pulga (Pulex irritans) que ocasiona la puliciasis y las picaduras por mosquitos (Culicidae). Los alrgenos presentes en la saliva del insecto
inducen una sensibilizacin del paciente con formacin de anticuerpos especficos. El tipo e intensidad de la reaccin originada por la picadura
depender de si el paciente ha estado ya expuesto al insecto y de la capacidad del husped de responder al estmulo antignico. Las lesiones
tempranas de prurigo por insecto se deben a una respuesta de hipersensibilidad tipo I causada por la liberacin de IgE. Posteriormente interviene un
mecanismo de hipersensibilidad tipo IV dependiente de linfocitos T que produce las lesiones tardas. El prurigo por insecto afecta cualquier rea
corporal, se presentan vesculas en la fase inicial posteriormente aparecen pequeas ppulas eritematosas, las lesiones son muy pruriginosas por los
que se observan costras hemticas por rascado. Se observan lesiones en diferentes estadios y evolucionan por brotes. CIMICIASIS: La chinche tienen
en su saliva anticoagulantes y anestsicos, puede vicir sin alimentos hasta 1 ao y se alimenta cada 10 dias, pone hasta 500 huevos, y vive en

CURSO ENARM CMN SIGLO XXI TEL: 36246001

Pharmed Solutions Institute

PGINA 353

MANUAL DE TRABAJO DEL CURSO ENARM CMN SIGLO XXI


colchones, ropa, pared, las lesiones son mas frecuentes en regin lumbar, nalgas, caras externas de las piernas, son simtricas (pesas), evolucin
crnica, hay ronchas, ppulas, costras hemticas y prurito. PULICIACIS: Estos causan lesiones ppulas y petequias dispersas, la complicacin mas
frecuente es imptigo o dermatitis de contacto. Las garrapatas laceran la epidermis y la abren, pueden transmitir bacterias, virus, espiroquetas,
topografa en cara, cuello, extremidades sus lesiones son ppulas con halo eritematoso o equimtico de >4cm o ampolla o zona de necrosis.
TRATAMIENTO: Topicos, como pasta lassar, linimento olocalcaneo, mentol, fenol, alcanfor, se pueden usar antihistamnicos como Hidroxicina
1mg/kg/dia, hay que hacer fumigacin del rea de la casa o lugar. Preventivo: benzoato de bencilo, tiamina. PRURIGO NODULAR: Es una dermatosis
crnica caracterizada por ndulos muy pruriginosos que aparecen principalmente en las superficies extensoras de extremidades inferiores, puede
desarrollarse luego de una picadura por insecto de otras formas de inflamacin localizada. La lesin por s misma es muy pruriginosa, lo cual es el
resultado de un crculo vicioso establecido por el rascado, trauma mecnico e infeccin, etc. El prurigo nodular puede ocasionalmente ser el
resultado del prurito que acompaa a algunas enfermedades sistmicas como los procesos obstructivos biliares, anemia, enfermedad renal crnica,
policitemia vera, DM, parasitosis, erupciones por droga y en los nios el ms comn el Linfoma de Hodgkins. Las lesiones del prurigo nodular
presentan un dimetro aproximado de 0,5-3cm, la superficie de las lesiones puede ser queratsica deprimida en el centro. La lesin inicial es
eritematosa y puede semejar a la urticaria, sin embargo todas las lesiones tienden a pigmentarse con el tiempo. El nmero de lesiones vara desde 2
a 200 de las cuales algunas mejoran espontneamente. PRURIGO DE SUTTON: Es una erupcin liquenoide de los codos en nios, pitiriasis de codos y
rodillas. Esta entidad clnica es una dermatosis eccematosa papular pruriginosa, se asocia a una historia personal de atpia o familiar, as como a
niveles elevados de IgE srica. Se ha descrito en nios de 3 a 13 aos, siendo la lesin inicial una ppula eritematosa de 1-2mm de dimetro.
PRURIGO DEL ATOPICO O DE BESNIER: Tambin llamado neurodermatitis, o eccema del lactante. Es la dermatosis ms frecuente en poblacin
peditrica. La prevalencia ha mostrado incremento en las ltimas dcadas, siendo del 18-20%. Es ms frecuente en reas urbanas de pases
industrializados, especialmente en inmigrantes provenientes de pases con menor prevalencia. Clinica: 1. Aguda. Caracterizada por ppulas y
vesculas muy pruriginosas, sobre un rea de piel eritematosa, asociada a escoriaciones, erosiones, exudado seroso y costras melicricas (eccema). 2.
Subaguda. Caracterizada por eritema, ppulas, descamacin y escoriaciones. 3. Crnica. Placas de piel engrosada, con liquenificacin y ppulas
fibrticas. Existen adems tres fases cronolgicas, con caractersticas especficas: 1. Lactante (2 semanas a 2 aos). Afecta la cara, predominando en
mejillas y respetando el tringulo central. Puede extenderse a piel cabelluda, pliegues retroauriculares y de flexin, tronco y nalgas; a veces puede
generalizarse. Predominan las lesiones de dermatitis aguda por lo que se le ha denominado eccema del lactante. Se puede asociar con dermatitis
seborreica en un 17%. Aparece por brotes, siendo frecuente la desaparicin de a los 2 aos de edad. 2. Escolar o infantil (3 a 14 aos). Afecta
pliegues de flexin (antecubitales y huecos poplteos), cuello, muecas, prpados, regin peribucal y genitales. Puede presentarse con lesiones
agudas o crnicas, evolucionando en brotes, con prurito intenso. Puede desaparecer (75-90%) o progresar a la ltima fase. 3. Adulto. (15 a 23 aos
de edad). PRURIGO ACTINICO: Es una erupcin papular o nodular frecuente, persistente, pruriginosa y escoriada de la piel expuesta al sol y en
menor medida de la no expuesta. Es frecuente en verano y en ocasiones no desaparece con el invierno, por lo general aparece en la niez y a veces
remite en la pubertad. Parece ser una variante persistente y algunas veces coexstente de la erupcin de lumnica polimorfica (ELPM) aunque sus
caractersticas clnicas son diferentes. La exposicin de radiacin UV sera el inductor del prurigo actnico (PA) ya que el trastorno es ms intenso en
primavera y en verano y las respuestas cutneas anormales a la radiacin estn presentes en dos tercios de los pacientes, ms a menudo con
radiacin UVA que con UVB. TRATAMIENTO: Las medidas generales son de gran importancia, la explicacin a los padres acerca del padecimiento, su
predisposicin atpica, la cronicidad y evolucin por brotes. Se recomienda adems el uso de pijamas largos, mosquiteros, insecticidas cuando no
estn los nios. Tpicamente se utilizan lociones antipruriginosas y esteroides tpicos. Por va oral se utilizan antihistamnicos no sedantes. En
algunas ocasiones se utiliza Tiamina por va oral, la cual se ha recomendado empricamente por mucho tiempo "tal vez sea un repelente natural que
se excreta por la piel", dosis 200mg a 600mg por da, lo ms importante es evitar la infeccin secundara. Se utilizan emolientes, capsaicina tpica,
radiaciones UV y sistmicamente Talidomida, la cual est contraindicada cuando hay riesgo de embarazo y de producir neuropata perifrica.
CASO CLINICO
Nia de 11 aos de edad que consulta por haber presentado
lesiones eccematosas en el brazo izquierdo, donde hace un mes, en
un puesto ambulante, le realizaron un tatuaje de henna de color
negro. A los 10 das, comenz con eritema y vesculas muy
pruriginosas, que cubran toda la zona del tatuaje. Las lesiones se
resolvieron tras un mes de tratamiento con corticoide tpico, con
persistencia de hipopigmentacin residual limitada a la zona del
tatuaje. La paciente se haca, desde hace tres veranos, tatuajes
transitorios de henna color marrn en la playa.
PREGUNTA
Cul de los siguientes factores esta mas relacionado con el
padecimiento?
RESPUESTA
a.- Infecciones de la piel.
b.- Exposicin a alrgenos.
c.- Irritantes como jabon, shampoo y cremas.
d.- Inhalacion ingestin de alrgenos.
PREGUNTA
Considerando el caso clnico. Cul es la conducta teraputica mas
apropiada?
RESPUESTA
a.- Indica emolientes.
b.- Corticoides tpicos de leve potencia.
c.- Corticoides tpicos de moderada potencia.
d.- Corticoides tpicos de alta potencia.

CURSO ENARM CMN SIGLO XXI TEL: 36246001

CASO CLINICO
Masculino de 8 meses de edad que es llevado por su madre por
presencia de tos y secresion nasal desde hace dos das y a la
exploracin se observa en cara, brazos enrojecimiento, la madre
agrega que le pica y que se presentan y luego desapareces solos, al
tacto la piel se siente aspera.
PREGUNTA
Cual es la conducta a seguir menos adecuada?
RESPUESTA
a.- Evitar ambiente caluroso, ropa de lana, platicos y otras fibras.
b.- Baarlo con agua templada y de preferencia de periodos largos
para mitigar lesiones.
c.- Aplicar crema hidratante y aceites varias veces al dia.
d.- Indicar crema con corticoide.
CASO CLINICO
Nino de 5 aos es trado por la madre a consulta, por presentar,
ronchas, ppulas y costras hemticas las cuales son muy
pruriginosas, localizadas en regin lumbar y glteos, algunas ppulas
se ven agrupadas en mancuerna, la madre refiere que el dia
anterior, estaba muy bien, sin lesiones, solonos refiere que salieron
de vacaciones y al siguiente da lo despert la comezn. La madre
refiere alrgicos negados.
PREGUNTA
Cul es el diagnstico ms probable para este caso?
RESPUESTA
a.- Escabiasis

Pharmed Solutions Institute

PGINA 354

MANUAL DE TRABAJO DEL CURSO ENARM CMN SIGLO XXI


b.- Prurigo
c.- Varicela
d.- Urticaria
PREGUNTA
Qu media la fase de hipersensibilidad tarda en este paciente?
RESPUESTA
a.- IgE e histamina
b.- IL1, IL6, TNF
c.- Linfocitos T, complejos inmunes, complemento

d.- Leucotrienos, tromboxanos


PREGUNTA
Cul es la conducta teraputica ms adecuada para este caso?
RESPUESTA
a.- Benzoato de bencilo mas hidroxicina
b.- Pasta lasar ms hidroxicina
c.- Lindano ms cetirizina
d.- Benzoato de bencilo mas cetirizina

ERITEMA POLIMORFO
CIENCIAS BASICAS: El eritema multiforme (EM) o eritema polimorfo es una condicin reactiva aguda, autolimitada, en ocasiones recidivante,
mediada inmunolgicamente, que afecta la piel y las membranas mucosas. Desencadenada fundamentalmente por el virus del herpes simple (VHS),
presenta lesiones en diana y un patrn histopatolgico inflamatorio. Se cataloga como una reaccin inflamatoria aguda, autolimitada, con diversos
niveles de gravedad y variado conjunto sintomtico. CLASIFICACION: Eritema multiforme maculopapuloso o eritema polimorfo menor (menor
intensidad en los cambios anatomopatologicos, escasa o nula participacin mucosa). Representa el 80 % de los casos. Eritema multiforme
vesiculoampolloso o eritema polimorfo mayor. Las formas graves, copn afeccion del estado general y posible participacin de rganos internos
corresponderan al Sindrome de Steven-Jhonson. La Necrolisis toxica epidrmica puede coinsiderase como un proceso aparte aunque algunos
autores lo consideran dentro del mismo espectro. SALUD PUBLICA: Predomina en adultos jvenes, de 20-40 aos, con afeccin de nios y
adolescentes en menos del 20% de los casos. Es algo ms frecuente en varones y no presenta predileccin racial. Se observa con mayor frecuencia
durante la primavera y verano, probablemente secundario a la reactivacin del VHS desencadenada por la exposicin solar. Entre un 25% y un 50%
de casos no puede establecerse el agente desencadenante. PATOGENIA: Se han reportado numerosos factores desencadenantes, entre los cuales se
encuentran infecciones, enfermedades malignas, enfermedades autoinmunes, radiacin, inmunizaciones, medicamentos (AINE, sulfonamidas,
anticonvulsivantes (30%), penicilinas, doxiciclina, tetraciclinas) y menstruacin. El VHS es la causa ms comn y est involucrado en un 80-90% de los
casos, con mayor asociacin al VHS tipo 1. Se obtiene una historia de herpes labial en un 50% de los pacientes en las 2 semanas previas al desarrollo
del EM; sin embargo, la lesin herptica puede ocurrir simultneamente o desarrollarse luego de las lesiones en diana. Los mecanismos patognicos
han sido estudiados en los casos de infeccin por el VHS. En la recurrencia, material gentico es fagocitado y transportado a la circulacin perifrica
por monocitos/macrfagos que portan el antgeno linfocitario cutneo, un antgeno que les permite transportarse a la piel, en donde se adhieren a
las clulas endoteliales de la microvasculatura drmica, y el ADN viral fragmentado es finalmente transferido a los queratinocitos del estrato basal y
de las capas inferiores del estrato espinoso. La inflamacin en la lesin cutnea se cree es debida a una respuesta inmune celular TH1 especfica
contra los queratinocitos que contienen el gen de la polimerasa de ADN (Pol), con la liberacin de citoquinas efectoras como el interfern (IFN-),
que lleva a la amplificacin de una respuesta inflamatoria no especfica a travs de clulas T autorreactivas y que resulta en la apoptosis de
queratinocitos aislados (necrosis celular satlite) y el resto de los hallazgos patolgicos que observamos en el EM. Finalmente, aditivos de alimentos,
como los benzoatos y terpenos, y agentes tpicos como los tatuajes y la dermatitis por contacto con hiedra venenosa, han sido reportados como
causas de EM. DIAGNOSTICO: Tal como indica su nombre de la enfermedad, ls lesiones clnicas son polimorfas e incluyen maculas, papulas y lesiones
vesiculo ampollosas. Las lesiones ms caractersticas muestran una apariencia anular concntrica en iris o en diana Generalmente no se
observan prdromos; algunos pacientes pueden presentar fiebre, cefalea, malestar y mialgias leves una semana antes de la erupcin. Las lesiones
cutneas se desarrollan en forma sbita, completndose el brote en 3-5 das. Las lesiones pueden diferir entre un paciente y otro, y variar durante la
evolucin de la enfermedad. Hay sensacin de prurito o ardor. La erupcin inicia como ppulas eritematoedematosas que simulan picadas de
insectos, que permanecen fijas por ms de una semana. Algunas lesiones aumentan de tamao, formando placas menores de 3cm, redondeadas, de
bordes bien definidos, que desarrollan anillos concntricos (eritema iris). La lesin en diana tpica consiste en una porcin central de color rojo sucio,
rojo vinoso o purprico, inmediatamente rodeada por una zona externa de color rojo intenso; le sigue un anillo edematoso plido, rodeado
finalmente de un halo eritematoso. La porcin central en ocasiones puede presentarse vsico-ampollar, formando el herpes iris de Bateman, o
adquirir un aspecto costroso. La distribucin es acral y simtrica, con predileccin por el dorso de las manos y superficies extensoras de las
extremidades superiores; con menos frecuencia se afectan las palmas, cara, cuello, tronco y extremidades inferiores. Se han descrito lesiones
siguiendo las lneas de Blaschko. Las lesiones mucosas se presentan en un 25-60%, y ocurren conjuntamente con las lesiones cutneas. La mucosa
oral es la ms afectada, principalmente la mucosa no queratinizada del tercio anterior. Inicia de forma rpida como edema y enantema que afecta
los carrillos, gingival y lengua, que progresa a erosiones superficiales con formacin de seudomembranas; en los labios se observan fisuras, sangrado
y costras serohemticas. El prurito, la sensacin de quemazn, as como la pobre alimentacin y baja ingesta de lquidos debida al dolor de las
erosiones mucosas, que puede ser severo, son causas importantes de morbilidad en el EM Laboratorio: No hay pruebas especficas para el
diagnstico. En casos severos puede encontrarse aumento de la velocidad de eritrosedimentacin, leucocitosis o elevacin de las pruebas de funcin
heptica. El diagnstico del eritema multiforme es clnico. Los cambios histolgicos dependen de la morfologa clnica, la duracin y el rea de la
lesin de donde se toma la biopsia. El hallazgo ms precoz es la apoptosis de queratinocitos. Se observa necrosis celular en satlite, con linfocitos
unidos a queratinocitos necrticos aislados. ERITEMA MULTIFORME MENOR: Originado principalmente por infecciones virales herpticas (herpes
simple) o Mycoplasma pneumoniae y solo en 1 % por productos farmacuticos. La erupcin se produce en el trmino de 12 a 24 horas y en la mitad
de los casos hay un prdromo similar a una infeccin de las vas respiratorias altas. Las lesiones cutneas tpicas son mculas rojo- azuladas, con 3
anillos concntricos denominados en diana o arco de tiro y una ampolla central, si bien pueden estar afectadas las mucosas. De hecho, 20 % de los
eritemas multiformes afecta a nios y adultos jvenes; su inicio es abrupto, con fiebre alta y sntomas prodrmicos intensos. Se observan lesiones
cutneas en el tronco, similares al eritema multiforme menor, pero ms extensas y necrosantes. Daa gravemente a ms de 2 mucosas y ocasiona
erosiones costrosas profundas en los labios; su evolucin es ms prolongada, especialmente con frmacos de vida media ms larga y sus
manifestaciones clnicas duran entre 3 - 6 semanas. Las formas menores de eritema multiforme desaparecen espontneamente en alrededor de 3
semanas sin secuelas. SNDROME DE STEVEN-JOHNSON: Es una forma grave de eritema multiforme habitualmente de ori gen txico. El grado de
afectacin cutnea vara desde formas con gran lesionalidad hasta casos con escasas o nulas lesiones cutneas. Sin embargo, se caracteriza por la
severa participacin de mucosas (oral, nasal, conjuntival, genital) y afectacin del estado general. NECRLISIS TXICA EPIDRMICA: La caracterstica
clnica ms importante es la formacin de grandes ampollas flcidas que condicionan el desprendimiento de lminas de epidermis, dejan do
extensas reas denudadas. Se trata de una situacin grave, en ocasiones mortal. TRATAMIENTO: La aplicacin del tratamiento puede mejorar el

CURSO ENARM CMN SIGLO XXI TEL: 36246001

Pharmed Solutions Institute

PGINA 355

MANUAL DE TRABAJO DEL CURSO ENARM CMN SIGLO XXI


cuadro sintomtico y acortar la evolucin del proceso; pero las formas mayores suelen ser mortales o causar graves secuelas. La tasa de mortalidad
para el sndrome de Stevens-Johnson se sita entre 1 y 5%, segn distintos autores. La existencia de una gran extensin de zonas denudadas, la edad
avanzada, la insuficiencia renal concomitante y la afectacin pulmonar empeoran el pronstico. Dado el curso autolimitado del proceso, en eritema
multiforme menor, solo se requiere tratamiento sintomtico. En los casos postherpeticos recurrentes puede plantearse tratamiento con Aciclovir u
otros agentes antiherpeticos. El Sndrome de Stevens Jhonson y La Necrolisis toxica epidrmica son procesos potencialmente graves que requieren
de ingreso del paciente y manejo en cuidados intensivos, en condiciones de asepsia. Existe controversia respecto al empleo de corticoides sistmicos
pues si bien suelen condicionar un alivio de sintomatologa subjetiva no hay evidencia de que reduzcan la morbimortalidad. En ocasiones se precisa
antibioticoterapia para combatir complicaciones spticas.
CASO CLINICO
Preescolar de 3 aos de edad, de sexo femenino, previamente sana,
que consulta por cuadro de dos das de evolucin, caracterizado por
fiebre, odinofagia y discreto compromiso del estado general.
Evaluada en el Servicio de Urgencia se diagnostic amigdalitis aguda
purulenta. Se indic penicilina benzatina 600 000 UI intramuscular
por 1 vez. Posterior a la administracin y en el transcurso del mismo
da, comenz con prurito generalizado, ms intenso en la regin
dorsolumbar. Horas ms tarde, la madre not aparicin de lesiones
maculares eritematosas en extremidades, cara y tronco, por lo cual
consult nuevamente, 48 horas despus. En ese momento, al
examen clnico se constat una paciente en buenas condiciones
generales con temperatura de 38,5 C axilar y lesiones papulares,
eritematosas, redondeadas, distribuidas en forma simtrica y
comprometiendo todo el cuerpo, ms marcado en cara y
extremidades inferiores. Las lesiones presentaban aspecto de anillos

concntricos, con un halo perifrico eritematoso y un rea ms


central plida y en algunas lesiones una pequea vescula. La mucosa
oral presentaba petequias escasas en forma aislada. No haba otras
mucosas comprometidas. El hemograma mostr GB 11 700 (75%
segmentados, 5% baciliformes), hematocrito 38%, plaquetas 384 000
x mm3. Adems VHS 80 mm x hr, PCR 10,9, creatininemia 0,61
mg/dl, nitrgeno ureico 0,22 mg/dl. Radiografa de trax y
sedimento de orina fueron normales.
PREGUNTA
Dentro de los siguientes diagnosticos cual es el menos probable?
RESPUESTA
a.- Eritema Multiforme.
b.- Stevens-Jonhson.
c.- Necrolisis Epidrmica Txica
d.- Vasculitis.

ERITEMA NODOSO
CIENCIAS BASICAS: Es un sndrome caracterizado por una erupcin cutnea nodular, eritematosa, caliente y dolorosa, que se localiza
preferentemente en la regin pretibial, aunque tambin en ocasiones en muslos, brazos, antebrazos y otras reas corporales. PATOGENIA: La
etiologa de este sndrome es mltiple, pero podemos globalmente dividirla en dos causas fundamentales: 1. De causa infecciosa: Bacterianas;
tuberculosis (bacilo de Koch), estreptococicas, por yersinia, salmonela, shigela, campilobacter, brucelosis, tularemia, enfermedad por araazo de
gato, rickettsias, clamidias, mycoplasma, leptospirosis. Virica; VEB, hepatitis B, parvovirus B19. Protozoos; Giardia lamblia, amebas, toxoplasma. 2.
De causa no infecciosa: Medicamentos; antoconceptivos orales, sulfamidas, penicilinas, salicilatos, TMP, bromuros, yoduros, sales de oro. Las
lesiones del eritema nodoso representan una reaccin frente a varios estmulos, se consideran como un tipo de vasculitis cutnea alrgica,
pudiendo, por lo tanto, ser consecutivas de cierto nmero de estmulos provocadores. DIAGNOSTICO: En ocasiones se acompaa de fiebre, con
aceptable estado general, puede haber artralgias de localizacin en grandes articulaciones de extremidades inferiores, que puede ser la
manifestacin de un proceso subyacente causa de esta patologa. Las lesiones cutneas, son bastante caractersticas; se trata de ndulos de 1-3 cm
de dimetro, dolorosos, indurados, brillantes, enrojecidos, calientes y sobreelevados. Aparecen en las reas pretibiales fundamentalmente, pero
tambin pueden tener otras localizaciones, como las nalgas, pantorrillas y tambin en las extremidades superiores. La forma evolutiva de estas
lesiones es bastante caracterstica; durante un periodo de varios das, se hacen prominentes y de color violceo; despus de 1-2 semanas van
disminuyendo su prominencia e induracin, modificando el color, pasando por purpura oscuro y al final desaparecen dejando un rastro parduzco, sin
dejar ulceracin, ni cicatriz. Estas lesiones aparecen por brotes, en un periodo entre 3-6semanas, para determinar desapareciendo definitivamente.
El diagnstico es clnico, pero debemos hacer el diagnostico etiolgico se refiere a la necesidad de descartar una enfermedad asociada; esto puede
ser en ocasiones difcil y se puede no llegar al mismo en 20% de los casos. Hay que practicar pruebas complementarias como PPD, analtica bsica
general, coprocultivo, serologas, cultivos, radiografa de trax, trnsito intestinal si es necesario, etc. TRATAMIENTO: Generalmente tienen una
resolucin espontanea, y la adopcin de medidas generales como el reposo y las medidas fsicas es suficiente. El tratamiento debe ir dirigido a tratar
la enfermedad de base que sea la causante de este trastorno. Las medidas generales van encaminadas a disminuir el dolor y el acortamiento de la
enfermedad. Dentro de ellas es importante insistir en el reposo y en la elevacin de las extremidades inferiores. Diversos frmacos antiinflamatorios
han demostrado ser tiles en el tratamiento del eritema nodoso, pero especialmente los salicilatos, la indometacina y el ibuprofeno. El empleo de
corticoides sitemicos se debe reservar para aquellos casos con manifestaciones sistmicas graves (artritis, fiebre muy elevada), enfermos con
sarcoidosis y pacientes con eritema nodosp persistente y ausencia de respuesta a los dems tratamientos mencionados. ERITEMA NODOSO
MIGRATORIO: Es una variante clnica de eritema nodoso, tambin denominada Paniculitis migratriz de Bfverstedt o Vilanova. Presenta ndulos
unilaterales; en general menos numerosos, dolorosos y persistentes que los de la forma clsica. Al progresar tienden a dividirse y extenderse en la
periferia, esto les confiere un patrn arciforme con bordes eritematosos, brillantes y un centro ms violceo o pardo.
CASO CLINICO
Masculino de 12 aos de edad acudi a consulta debido a que 15
dias previas inicio con lesiones eritematosa violcea nodulares y
dolorosas en los miembros inferiores de predomio en la parte
posterior de las piernas. De 1 a 2 cm de dimetro, no referia ingesta
de frmacos, fiebre, sntomas respiratorios, digestivos ni articulares,
en la anamnesis no se encontraron antecedentes personales ni
familiares de inters.

CURSO ENARM CMN SIGLO XXI TEL: 36246001

PREGUNTA
Considerando la etiologa mas frecuente de esta patologa. Cules
la conducta diagnostica mas apropiada?
RESPUESTA
a.- Biopsia.
b.- PPD.
c.- Hemocultivo.
d.- Panel viral

Pharmed Solutions Institute

PGINA 356

MANUAL DE TRABAJO DEL CURSO ENARM CMN SIGLO XXI


PRPURA DE SCHNLEIN-HENOCH (PSH)
CIENCIAS BASICAS: Es una vasculitis sistmica de vaso pequeo y curso habitualmente agudo. Es una vasculitis leucocitoclstica, de mecanismo
inmunologico la ms comn en la infancia. La causa es desconocida. Precedida, en ocasiones, por una infeccin de vas respiratorias que sugiere un
posible agente infeccioso desencadenante, siendo larga la lista de patgenos implicados. Otros desencadenantes pueden ser frmacos (penicilina,
ampicilina, eritromicina, quinina), alimentos, exposicin al fro o picaduras de insectos. SALUD PUBLICA: Afecta en el 90% de los casos a nios,
preferentemente varones (2:1), con una edad media de 6 aos. PATOGENIA: Es una vasculitis mediada por IgA de los pequeos vasos. Hay aumento
en la produccin de IgA, aumento de inmunocomplejos circulantes de IgA y depsitos de IgA en las biopsias de piel y de rin. La lesin renal de la
PSH es indistinguible histopatolgicamente de la nefropata por IgA de la enfermedad de Berger. Ambas pueden producir insuficiencia renal.
DIAGNOSTICO: Usualmente se presenta con la ttrada: prpura palpable, artritis, dolor abdominal y enfermedad renal. La prpura aparece en el
100% de los casos pero en una cuarta parte no es la manifestacin inicial. Se caracteriza por lesiones palpables de 2-10mm, de color rojo-violceo
(90%), que se concentran caractersticamente en los glteos y las extremidades inferiores aunque, en ocasiones, pueden afectar a los brazos y la
cara, y rara vez al tronco. La afectacin articular (50-85%) se manifiesta habitualmente como oligoartritis de miembros inferiores, afecta
principalmente la articulacin del tobillo y la rodilla. Las alteraciones gastrointestinales (50-70%) se caracterizan por dolor abdominal clico o
anginoso (empeora con la ingesta) que puede acompaarse de nuseas, vmitos, diarreas, rectorragias y melenas Los sntomas gastrointestinales se
deben al edema de la pared intestinal y a la hemoragia propia de la vasculitis, cuando estas lesiones son muy intensas pueden dar lugar a un infarto
intestinal y perforacion. En la gastroscopia y/o colonoscopia se observan petequias, hemorragias o erosiones. Son complicaciones posibles una
hemorragia intestinal grave y la invaginacin o perforacin intestinal. La enfermedad renal (20-50%), normalmente se presenta como una
glomerulonefritis moderada que cursa con hematuria, habitualmente microscpica, y con menos frecuencia proteinuria leve y raramente como
sndrome nefrtico, nefrtico y/o insuficiencia renal. En los adultos hay un mayor riesgo de afectacin renal crnica que puede llegar a insuficiencia
renal terminal. Aunque se trata de una vasculitis autolimitada hay recurrencias en la tercera parte de los casos. El diagnstico es clnico y se utilizan
los criterios del American College of Rheumatology: 1. Inicio de los primeros sntomas a los 20 aos o menos. 2. Lesiones purpricas sobreelevadas
sin trombocitopenia. 3. Dolor abdominal anginoso (empeora con comidas), vmitos, diarreas, rectorragias y melenas. 4. Biopsia cutnea: neutrfilos
peri o extravasculares, en arteriola y/o vnula. 5 Biopsia cutnea: neutrfilos en pared vascular de arteriola y/o vnula. La presencia de 2 criterios
clasifica de PSH con sensibilidad: 87,1% y especificidad: 87,7%. Las crioglobulinas, FR, ANA y ANCA suelen ser negativos o positivos a ttulos bajos. En
los adultos la biopsia de piel y el control de la posible afectacin renal son obligatorios. La biopsia cutnea muestra una vasculitis leucocitoclstica
que afecta a los pequeos vasos (arteriolas, vnulas y capilares). El hallazgo ms caracterstico es la infiltracin de neutrfilos de los pequeos vasos
de la dermis, con frecuente fragmentacin del ncleo de los mismos. TRATAMIENTO: Se resuelve espontneamente en el 94% de los nios y en el
89% de los adultos, por lo que el primer objetivo es tranquilizar al paciente y utilizar tratamiento sintomtico para la afectacin articular y el dolor
abdominal. El tratamiento con corticoides (1-2mg/kg) es controvertido. Una revisin sistemtica indica que los corticoides reducen la duracin del
dolor abdominal y disminuyen el riesgo de invaginacin intestinal, afectacin renal y recurrencias. Su uso en la PSH no complicada no mejor el
curso de la enfermedad, segn un ensayo clnico8. La afectacin renal grave precisara de corticoides, inmunosupresores y/o plasmafresis. Lo nico
que tiende la cronicidad es la nefritis.
CASO CLINICO
CASO CLINICO
Femenino de 11 aos de edad, ingres con cuadro clnico de dos das
Nio de 6 aos de edad, sin antecedentes personales ni familiares de
de evolucin con lesiones maculopapulares en ambos miembros
inters, que fue valorado en el servicio de urgencias por presentar,
inferiores acompaado de dolor abdominal difuso. Antecedente de
desde haca 24 h, lesiones cutneas maculopapulosas y purpricas
faringoamigdalitis, dos semanas antes de su ingreso la cual cedi con
distribuidas simtricamente por las nalgas, miembros inferiores y
administracin de amoxicilina ms ibuprofeno con evolucin
superiores asociadas a signos inflamatorios en las rodillas y tobillos,
favorable. EF: se evidenciaron lesiones maculopapulares palpables
muy indicativo de prpura de Schnlein-Henoch. Los anlisis de
sobrelevantadas en miembros superiores y ambos miembros
orina, de funcin renal y la presin arterial eran normales. Se inici
inferiores extendindose a regin gltea, el abdomen estaba blando,
tratamiento con antiinflamatorios no esteroides y se dio de alta.
depresible con peristaltismo conservado no doloroso a la palpacin
PREGUNTA
profunda. Laboratorio: leucocitos de 9200, hb 13,6, hto 40%, plt
Considerando el cuadro clnico. Cul de las manifestaciones es la
71300, segmentados 53%, linfocitos 47%, tiempos de coagulacin
menos frecuente para orientar el diagnostico de PSH?
dentro la normalidad, proteinuria de 24 h de 52mg/24 h. La
RESPUESTA
evolucin a los 2 das de internacin fue con dolor abdominal
a.- Fiebre prolongada de origen desconocido.
espasmdico acompaado de deposiciones melnicas, se descart
b.- Lesiones cutneas sugestivas.
patologa quirrgica con ecografa abdominal y evolucin favorable
c.- Artralgias, artritis, miositis y serositis.
del cuadro con inicio de corticoide va oral, habiendo mejoria de la
d.- Parametros de laboratorio de inflamacin.
sintomatologa abdominal.
PREGUNTA
Cual de los siguientes factores etiolgicos puede estar ms
relacionado con esta patologa?
a. Estreptococo del grupo A beta hemoltico o M. pneumoniae
b.- Virus de hepatitis A, citomegalovirus virus de Ebstein Barr.
c.- Vacuna del sarampin, influenza, rubola, neumococo.
d.- Antibiticos betalactmicos, macrlidos AINES.

PREGUNTA
Cul es el factor desencadenante ms probable en este paciente?
RESPUESTA
a.- Infeccin por Haemophilus
b.- Antecedente de tomar amoxicilina
c.- Infeccin por Estreptococo beta hemoltico
d.- Exposicin al frio

PREGUNTA
Cual de las siguientes manifestaciones es la menos frecuente para
hacer el diagnostico?
RESPUESTA
a.- Manifestaciones cutneas.
b.- Manifestaciones digestivas.
c.- Manifestaciones articulares.
d.- Manifestaciones renales

PREGUNTA
Cul de los siguientes criterios diagnsticos es menos probable para
esta patologa?
RESPUESTA
a.- Inicio antes de los 20 aos
b.- Dolor abdominal difuso, que puede asociarse a diarrea o
presencia de sangre
c.- Presencia de ulceras orales
d.- Biopsia con granulocitos en pared

CURSO ENARM CMN SIGLO XXI TEL: 36246001

Pharmed Solutions Institute

PGINA 357

MANUAL DE TRABAJO DEL CURSO ENARM CMN SIGLO XXI


ANEMIAS
CIENCIAS BASICAS: Es la disminucin de la hemoglobina o el hematocrito, por debajo de los valores normales de la localidad y de acuerdo a la edad y
sexo, con la consecuente hipoperfusin tisular de oxgeno. A travs de la BH (Hb, Hto, reticulocitos, VCM, CHCM), se obtienen los valores que
indicaran si corresponde a un sndrome anmico. SALUD PBLICA: En Mxico 67% de los menores de 2 aos presentan ferropenia y en edad escolar
34%. En escolares y adolescentes la falta de una ingesta adecuada de carne de res y en el caso de las mujeres con la menstruacin, favorecen esta
entidad. De 2-11% de los nios menores de 12 aos de edad presentan deficiencia de folatos y aquellos que ingieren vegetales verdes solo 0.22%.
PATOGENIA: La anemia puede deberse a falta de produccin (aplasia, nutricional), perdida (hemorragia) o destruccin de los eritrocitos (hemolisis).
DIAGNOSTICO: En el RN a trmino que presenta anemia al nacimiento o durante los primeros das de vida, clnicamente puede lucir bien y ser un
hallazgo por estudio rutinario de BH o presentar manifestaciones de leves a severas: lucir mal, palido con ingesta inadecuada de leche, irritable. En
los lactantes y preescolares puede observarse palidez, irritabilidad, somnolencia, astenia, menor incremento ponderal, infecciones de repeticin y en
algunos casos pica o geofagia. En escolares o adolescentes puede manifestarse adems cefalea: acufenos, fosfenos, lipotimia, palpitaciones, disnea,
parestesias, calambres. La palidez como dato objetivo de anemia, es valido en los lechos ungueales, ya que la mucosa oral o la conjuntiva pueden
manifestar otras alteraciones que la eclipsan y la palidez global puede ser expresin de otras alteraciones como estrs coartacin artica, estado de
choque. Se puede encontrar de acuerdo a la intensidad y cronicidad de la anemia, manifestaciones de insuficiencia cardiaca como: taquicardia,
diferencial amplia en la tensin arterial, edema de miembros inferiores, hepatomegalia. La ruta diagnostica inicia al determinar el porcentaje de
reticulocitos corregidos: arregenerativa o falta en la produccin cunado estn por debajo de 3% y regenerativa cuando estn por arriba de 3%.
Habitualmente las segundas tienen cifras pr arriba de este porcentaje y es debida a hemorragia aguda o hemolisis. ANEMIA FERROPENICA: La
mayor causa por falta de produccin de eritrocitos, en la mayor parte de los pases es la ferropenia. Se inicia con la deplecin de eritrocitos de hierro
(ferropenia latente), pasa por eritropoyesis ferropnica (inicialmente microcitosis y posteriormente hipocroma) y termina con la anemia. En el
neonato este evento es poco frecuente porque la reserva de hierro es directamente proporcional a su peso corporal. El prematuro no recibe la
cantidad de hierro suficiente de Fe y debe reponerse el mismo en las primeras semanas. Las complicaciones del embarazo que originan perdidas
sanguneas crnicas, disminuyen la dotacin de hierro al neonato, como la hemorragia vaginal, amniocentesis traumtica, varices aneurisma del
cordn umbilical, insercin velamentosa, placenta previa, desprendimiento prematuro de placenta. ANEMIA MEGALOBLASTICA: Hay una
eritropoyesis ineficaz, puede deberse a deficiencia nutricional por deficiencia de folatos y de vitamina B12. En el prematuro la deficiencia se presenta
en el primer mes de nacido y requiere reposicin de este elemento. Las reservas de folatos se mantienen por 2 semanas en cualquier etapa del
desarrollo, la ingesta de vegetales verdes mantiene estos niveles, por lo que los lactantes alimentados solo con leche, desarrollan esta deficiencia.
Algunos medicamentos interfieren con el metabolismo de los folatos como los barbitricos, difenilhidantoina, TMP/SFX, excepcionalmente pueden
llevar a anemia megaloblastica. En el caso de B12 el hgado almacena hasta por 6 meses, por lo que es raro observar anemia por este elemento.
ANEMIAS CONGENITAS: Como la aciduria ortica hereditaria y la anemia diseritropoytica hereditaria se presentan excepcionalmente. En los RN si
hay anemia normocitica, pudiese corresponder a infecciones de tipo congnito como la rubeola, parvovirus B19, CMV, toxoplasmosis, ingesta
demediacmentos durante el embarazo o bien aplasia pura de la serie roja congnita. En el resto de las edades peditricas debe descartarse aplasia
pura de la serie roja adquirida, enfermedades crnicas como colagenopatias, endocrinopatas, infeccin o insuficiencia renal. Anemia arregenerativa:
DFe acuerdo a las caractersticas morfolgicas de los eritrocitos, se estudia por la presencia de hipocroma, macrocitosis o normocitosis.
DISQUERATOSIS CONGNITA: (sndrome de Zinsser -Cole-Engman) es una enfermedad hereditaria caracterizada por hiperpigmentacin de la piel,
leucoplasia de las membranas mucosas y distrofia ungueal, que en aproximadamente la mitad de los casos se acompaa de pancitopenia con
hipoplasia medular. Se ha demostrado que esta enfermedad se transmite en forma recesiva ligada al cromosoma X, habindose observado en
algunas madres portadoras, lesiones de leucoplasia en la mucosa bucal. Tambin se han descrito otros probables patrones de transmisin que
incluyen la herencia dominante ligada al cromosoma X y autosmica dominante. Lo anterior marca una diferencia con la anemia de Fanconi que es
una enfermedad heredada con carcter autosmico recesivo y que adems se presenta asociada a mltiples alteraciones esquelticas y
cromosmicas. ANEMIAS HEMOLTICAS HEREDITARIAS: la alteracin en el eritrocito se puede presentar a diferentes niveles; a. Defectos en la
membrana del eritrocito. La forma ms frecuente es la esferocitosis hereditaria, la cual ocupa el primer lugar entre las anemias hemolticas en
Mxico. Otros variantes incluyen la eliptocitosis, ovalocitosis y la estomatocitosis hereditaria. b. Capacidad limitada para la sntesis de las cadenas
normales de la hemoglobina. En este grupo se incluyen las talasemias y las alteraciones en la secuencia de los aminocidos de las cadenas
polipeptdicas lo cual determina un gran nmero de variantes moleculares; las ms frecuentes de estas variantes son la hemoglobinopata S o
anemia africana (ocupa el segundo lugar en frecuencia dentro de las anemias hemolticas hereditarias en Mxico), la hemoglobina C, D y E. c.
Deficiencias enzimticas de los eritrocitos. Incluyen principalmente las deficiencias de piruvatoquinasa y de glucosa-6-fosfato deshidrogenasa.
ANEMIA HEMOLTICA AUTOINMUNE: Destruccin exagerada de eritrocitos, producida por una actividad inmune aberrante que se dirige contra los
glbulos rojos del propio husped. Clnicamente puede ser clasificada en primaria o secundaria, de acuerdo a si se halla asociada o no a otros
estados patolgicos. Estos ltimos pueden incluir infecciones virales o bacterianas, enfermedades asociadas con produccin de autoanticuerpos,
sndromes de inmunodeficiencia y neoplasias. Se desconoce el mecanismo por el cual el organismo forma anticuerpos contra sus propios glbulos
rojos, pero s se conocen los mecanismos inmunolgicos capaces de destruir los glbulos rojos in vivo. La anemia hemoltica autoinmune primaria
(AHAIP) se caracteriza por la presencia de anticuerpos capaces de actuar contra los eritrocitos del propio paciente. Los anticuerpos han sido
caracterizados como inmunoglobulinas G(IgG) o M(IgM). stas actan directamente contra los antgenos de la membrana del eritrocito o bien,
mediante la formacin de complejos inmunes dirigidos contra ella. En otros casos, la lisis de la membrana eritrocitaria se produce por activacin del
sistema del complemento. Habitualmente el inicio es agudo con descenso rpido en los niveles de hemoglobina. Asimismo, la mayor parte de los
pacientes presentan periodos breves de evolucin, menores de tres a seis meses. Clnica; decaimiento, anorexia, palidez, dolor abdominal e ictericia
de intensificacin progresiva; puede ocurrir adems emisin de orina de color pardo obscuro a consecuencia de la hemoglobinuria. A la exploracin
fsica puede observarse hepatomegalia y esplenomegalia de intensidad variable. Con menos frecuencia se manifiestan fiebre, tendencia
hemorrgica, disnea, taquicardia y linfadenopatas. Los hallazgos de laboratorio caractersticos de los pacientes con AHAIP: anemia, macrocitosis,
reticulocitosis, hiperbilirrubinemia indirecta y concentracin de hemoglobina libre en plasma en valores superiores al normal. TRATAMIENTO:
Anemia ferropenica sulfato ferroso: 20mg/kg/da, fumarato ferroso 15mg/kg/da, el tiempo de administracin va en relacin a corregir la anemia, y
completar las reservas, se puede uno guiar con la BH y suspender cuando ha desaparecido la microcitosis o bien completarse por 6 meses.
Excepcionalmente se requiere la administracin por va parenteral y solo cuando existe intolerancia gstrica. Anemia por deficiencia de folatos;
cido flico 1 mg por VO, por un mes. Anemia por deficiencia de B12, es conveniente su administracin por va parenteral, ya que la absorcin
intestinal es insuficiente, cuando hay deficiencia. Se indica 100mg IM al da por 10 das. En la anemia hemoltica congnita el consumo exagerado de
folatos por la reprodccion celular incrementada, requiere mantener niveles adecuados de este elemento. Se administra cido flico a 1mg/da VO
por da. Anemia hemolitca autoinmunitaria; se indican folatos por la misma razn que en la congnita y esteroides por va endovenosa si hay crisis
hemoltica o por VO prednisona, iniciando 2 mg/kg/dia, con disminucin paulatina cada 2 semanas, hasta completar 2 meses de tratamiento.

CURSO ENARM CMN SIGLO XXI TEL: 36246001

Pharmed Solutions Institute

PGINA 358

MANUAL DE TRABAJO DEL CURSO ENARM CMN SIGLO XXI


Tratamiento quirrgico; en el caso de talasemia o esferocitosis hereditaria, que cursen con esplenomegalioa o hemolisis intensa, est indicada la
esplenectoma, a partir de los 4 aos de edad. Tratamiento nutricional; en las anemias carenciales es necesario realizar cambios en los hbitos
alimenticios, como el inicio temprano de la ablactacin, con frutas, verduras y carne de res por lo menos 4 das de la semana y en especial las
mujeres adolescentes por las prdidas sanguneas debidas a la menstruacin. El trasplante alogenico de medula sea est indicado en anemia
hemoltica congnita, ya que permite la curacin en la mayor parte de los casos, principalmente en pacientes con alfa talasemia, beta talasemia
mayor y anemia falciforme.
CASO CLINICO
Lactante de 2 aos von 6 meses con letargia, hiporexia, retraso
ponderoestatural e involucin psicomotriz (hipotona, irritabilidad y
prdida de sonrisa social) de un mes de evolucin. Como
antecedentes personales destacaban un crecimiento intrauterino
retardado disarmnico, una serologa materna positiva para les
durante la gestacin y anemia perniciosa mal controlada. No haba
consanguinidad. La alimentacin era con lactancia materna
exclusiva. En la exploracin fsica presentaba un estado general
regular, hipoperfusin perifrica, palidez e ictericia mucocutneas,
un soplo sistlico eyectivo, taquicardia, papilitis lingual, aftas
palatinas.
PREGUNTA
Considerando el cuadro clnico cual de las manifestaciones es la
menos importante para el diagnostico.
RESPUESTA
a.- Antecedentes personales.
b.- Antecedentes maternos.
c.- Soplo sistlico eyectivo.
d.- Aftas palatinas.
PREGUNTA
Cuales son las manifestacin para hacer el diagnostico diferencial
para anemia ferropenica?
RESPUESTA
a.- Volumen corpuscular medio disminuido.
b.- Hemoglobina corpuscular media normal.
c.- Amplitud de la distribucin eritrocitaria disminuido.
d.- Recuento de plaquetas mormales o aumentadas.
PREGUNTA
Cual no es una indicacin para referir al paciente antes referido?
RESPUESTA
a.- Ferritina serica no concluyente.
b.- Perfiel de hierro no concluyente.
c.- Paciente con buen apego que no responde.
d.- Prdida sangunea aguda.
CASO CLINICO
Hombre de 64 aos de edad consulta al servicio de urgencias, por
presentar adinamia, fatigabilidad, palpitaciones desde hace
aproximadamente 2 meses, que se vienen exacerbando, hacindose
intolerables en los ltimos das, adems, disnea de esfuerzos. No hay
antecedentes de importancia. Al examen fsico se encuentra plido,
taquicrdico con frecuencia cardiaca de 90 por minuto y un soplo
pansistlico de tipo funcional. Laboratorio: Glbulos Rojos: 2.9.
1012/L, Leucocitos: 7.5. 109/L, PN: 65%, Linfocitos: 25%, Monocitos
10%, Hb 4.5 g/dl , Hto 14%, Plaquetas: 550. 109/L
PREGUNTA
Cmo esperaramos encontrar los reticulocitos en este paciente?
RESPUESTA
a.- Bajos porque la anemia debe ser microctica
b.- Aumentados porque las plaquetas se encuentran aumentadas
c.- Normales porque no se encuentra neutropenia
d.- Normales porque la anemia debe ser normocrmica

RESPUESTA
a.- Test de Coombs directo
b.- Test de autohemlisis in vitro
c.- cido flico
d.- Ferritina
PREGUNTA
Para precisar la causa de esta anemia. Cul es la conducta
diagnostica ms adecuada a seguir?
RESPUESTA
a.- Mielograma
b.- Biopsia de mdula sea
c.- Endoscopia de vas digestivas
d.- Sangre oculta en heces
PREGUNTA
Entre los diagnsticos siguientes. Cul es el ms probable para este
caso?
RESPUESTA
a.- Ingesta baja en hierro
b.- Sndrome paraneoplsico
c.- Enfermedad celiaca
d.- Microesferocitosis hereditaria
PREGUNTA
Cul es el tratamiento ms adecuado para este caso?
RESPUESTA
a.- Suplencia con el complejo de vitaminas B
b.- cido flico
c.- Dieta rica en protenas
d.- Hierro y vitamina C
CASO CLINICO
Se presenta el caso clnico de un adolescente de 16 aos de edad,
raza negra, que viva en una zona rural con microclima (fro), y acude
a urgencias con fiebre de 13 das de evolucin, para lo cual llev
tratamiento con penicilina 3 das, iniciada al quinto da de la
enfermedad, pero fue suspendida al presentar sntomas de
sudoracin y falta de aire, y sustituida por cefalosporina de primera
generacin por 3 das. Nuevamente apareci la falta de aire, en esta
ocasin ms intensa, adems de debilidad y tos hmeda escasa con
dolor torxico. Fue remitido a la UCI, con el diagnstico presuntivo
de neumona bacteriana y anemia moderada. Antecedentes
patolgicos personales: madre con hemoglobina A (HBA). EF: Palidez
cutneo-mucosa, disnea, tiraje intercostal, murmullo vesicular
disminuido hacia la base derecha con subcrepitantes, sibilantes
bilaterales sin ctero ni visceromegalia. Laboratorios: EGO aspecto
ligeramente turbio, 0 leucocitos y cilindros, ausencia de protenas.
Hb 7 g/L , Hto 0,29 l/L, leucocitos: 27,8 x 109/L, diferencial: juveniles
0,01, polimorfonucleares 0,83; linfocitos 0,10; eosinfilos 0,05
PREGUNTA
Cul es el diagnostico mas probable para este caso?
RESPUESTA
a.- Neumona grave con insuficiencia respiratoria
b.- Anemia hemoltica
c.- Anemia autoinmune secundaria a medicamentos
d.- Anemia ferropenica

PREGUNTA
Para complementar el estudio biolgico de la anemia de este
paciente. Cul es la conducta diagnostica ms adecuada a seguir?

CURSO ENARM CMN SIGLO XXI TEL: 36246001

Pharmed Solutions Institute

PGINA 359

MANUAL DE TRABAJO DEL CURSO ENARM CMN SIGLO XXI


LEUCEMIAS
CIENCIAS BASICAS: La leucemia aguda es el padecimiento ms frecuente en pediatra, no solo Mxico, sino en todo el mundo. Es un trastorno
gentico de las clulas hematopoyticas. La caracterstica principal de este padecimiento, es la proliferacin de la clula ms primitiva de la medula
sea, que puede ser desde la ms indiferenciada hasta la mayor grado de diferenciacin, proviene tanto de linfocitos B como T. Las leucemias
crnicas son mucho ms comunes en los adultos que en los nios. Suelen crecer ms lentamente que las leucemias agudas, aunque tambin son
ms difciles de curar. Las leucemias crnicas tambin se pueden dividir en dos tipos. SALUD PUBLICA: En el nio existen varios tipos de leucemias
siendo la ms frecuente leucemia aguda linfoblastica (LAL), comprende ms de 65% de los casos, le sigue la leucemia mieloblastica aguda (LAM) en
un 25% frecuencia (es la responsable del 30% de las muertes por leucemia en la edad peditrica) y la leucemia granulocitica crnica en menos de 5%
de los casos. Se estima alrededor de 1 caso por 2880 nios sanos,
en pacientes con sndrome de Down la frecuencia es de 1 en 95
casos. La OMS seala que en Mxico es ms frecuente en el sexo
femenino. El pico mximo de frecuencia es el alrededor de los 3-5
aos de edad. PATOGENIA: La etiologa de esta enfermedad es
desconocida, existen factores predisponentes como: radiacin (in
tero, con las placas simples de radiografa), anomalas
cromosmicas (Sx. Bloom, anemia de Fanconi, Sx de Down), otras
entidades (Sx. de Poland, de Rubistein-Taybi, neurofibromatosis),
frmacos (cloranfenicol, agentes citotoxicos), insecticidas, la
frecuencia de leucemia aguda es mayor en los familiares de
pacientes con LA. Las clulas leucmicas se pueden reproducir
rpidamente, y puede que no mueran cuando deberan hacerlo,
sino que sobreviven y se acumulan en la mdula sea, desplazando
a las clulas normales. Un tipo comn de anomala del ADN que
pueden dar lugar a la leucemia se conoce como translocacin
cromosmica. El ADN humano est empacado en 23 pares de
cromosomas. En una translocacin, el ADN de un cromosoma se
desprende y se une a un cromosoma diferente. El punto en el
cromosoma donde ocurre el desprendimiento puede afectar los
oncogenes o los genes supresores de tumores. Por ejemplo, una
translocacin vista en casi todos los casos de leucemia mieloide
crnica (CML) infantil y en algunos casos de leucemia linfoctica
aguda (ALL) infantil es el intercambio de ADN entre los
cromosomas 9 y 22, lo que conduce a lo que se conoce como
cromosoma Philadelphia. Esto crea un oncogn conocido como
BCR-ABL. Tambin se han descubierto en leucemias infantiles
muchos otros cambios en cromosomas o en genes especficos.
Algunos nios heredan mutaciones del ADN de uno de sus padres
que pueden aumentar su riesgo de desarrollar cncer.
CLASIFICACION: Citomorfolgica, la clasificacin Franco-americanabritnica (FAB) reconoce 3 tipos morfolgicos: L1; tamao clula pequeo, forma del nuclolo regular, nuclolos ausentes y relacin ncleocitoplasma es elevada. L2; poblacin celular ms heterognea e irregular, la relacin ncleo-citoplasma es mayor y a menudo se perciben nuclolos.
L3; clulas morfolgicamente idnticas a las de linfoma de Burkit, grandes, homogneas con nuclolo redondo y ovalado, as como presencia de
vacuolas. En los nios L1 80-85%, L2 15% y L3 menos de 3%. Clasificacion tipo celular afectado: Leucemia linfoctica crnica (LLC). Afecta a las
clulas linfoides y es por lo general de crecimiento lento. Hay ms de 15 000 casos nuevos de leucemia cada ao. A menudo, las personas que son
diagnosticadas con esta enfermedad son mayores de 55 aos. Casi nunca afecta a nios. Leucemia mieloide crnica (LMC). Afecta a clulas mieloides
y por lo general es de crecimiento lento al principio. Hay aproximadamente 5 000 casos nuevos de leucemia cada ao. Afecta principalmente a
adultos. Leucemia linfoctica (linfoblstica) aguda (LLA). Afecta a clulas linfoides y es de crecimiento rpido. Hay ms de 5 000 casos nuevos de
leucemia cada ao. La LLA es el tipo de leucemia ms comn entre nios pequeos. Tambin afecta a adultos. Leucemia mieloide aguda (LMA).
Afecta a clulas mieloides y es de crecimiento rpido. Hay ms de 13 000 casos nuevos de leucemia cada ao. Afecta tanto a adultos como a nios.
En forma general las LAL son positivas a la reaccin de PAS, mientras que las LAM son negativas. La clasificacin inmunolgica en que los linfoblastos
expresan antgenos correspondientes a las clulas linfocitarias B o T en distintas fases de maduracin, aproximadamente 80% de las LAL.
DIAGNOSTICO: Clnica; Inicio insidioso desde asintomtico o presentar sntomas y signos atribuibles a cuadros virales de vas respiratorias altas,
sospecha por descubrimiento incidental en una BH. Ms de 50% presentan sntomas generales como fiebre elevada, anorexia, adinamia y malestar
general con palidez de tegumentos. En 30% manifestaciones de hemorragia como petequias, equimosis y epistaxis. Las manifestaciones de dolor
seo son frecuentes y no siempre corresponden con los hallazgos radiolgicos. Las manifestaciones neurolgicas o de tumores es poco frecuente y
se ha asociado con algunos tipos de especficos de leucemias como las LAL de clulas T. Laboratorio: Presencia de anemia, trombocitopenia y las
cifras de leucocitos pueden ser normales. El estudio de medula sea por aspiracin es la prueba diagnstica (90%), preferentemente se deber
efectuar en la espina iliaca posterosuperior, sin embargo se puede realizar en la espina anterosuperior, en general se muestra celularidad
aumentada, con sustitucin casi por linfoblastos. Despus de realizar los frotis se deber teir con tincin de Wright. Si en la cuenta diferencial
existen ms de 25% de linfocitos, se establece el diagnostico de leucemia. Adems se deben realizar pruebas de funcionamiento heptico. Otro
estudio obligado para el diagnstico y seguimiento es el de LCR en el que se buscan linfoblastos preferentemente, protenas y glucosa. Es obligado
realizar estudio de inmunofenotipo para confirmar el diagnstico y linaje celular B o T. TRATAMIENTO: Para LAL comprende 3 fases de tratamiento,
va de la induccin a la remisin el objetivo es erradicar la carga de linfoblastos de la MO, al trmino de esta fase que dura 28 das, la MO debe
encontrase con menos de 5% de blastos, para considerar que el paciente est en remisin; la segunda fase es de consolidacin; la tercera fase es la
de mantenimiento en la que se indican reducciones con la finalidad de que al trmino de esta no exista enfermedad medible en la MO. La duracin
depender del grupo de riesgo si es de bajo o alto grado, actualmente para una LAL de alto riesgo es de 30 meses, mientras que opera leucemias de
bajo riesgo es alrededor de 24 meses. La base del tratamiento es la quimioterapia: Algunos de los medicamentos comnmente usados para tratar la
leucemia en nios incluyen: Vincristina (Oncovin, Daunorubicina, tambin conocida como daunomicina (Cerubidina), Doxorrubicina (Adriamycin),

CURSO ENARM CMN SIGLO XXI TEL: 36246001

Pharmed Solutions Institute

PGINA 360

MANUAL DE TRABAJO DEL CURSO ENARM CMN SIGLO XXI


Citarabina, tambin conocida como arabinsido de citosina o ara-C (Citosar), L-asparaginasa (Elspar), PEG-L-asparaginasa (pegaspargasa, Oncaspar),
Etopsido (VePesid, otros). Hay que dar profilaxis al SNC, que se establece con la aplicacin de quimioterapia intratecal. Actualmente se acepta que
la indicacin para trasplante de MO para el paciente de LAC es en aquellos que presentan una segunda remisin, esto quiere decir, pacientes que
tienen una recada a la MO; pacientes con factores de riesgo elevado como las alteraciones citogenticas del cromosoma Philadelphia t (9,22), t
(4,11) entre las ms importantes, la indicacin de trasplante debe ser una vez alcanzada la remisin. La ciruga tiene una funcin muy limitada en el
tratamiento de la leucemia en nios. Como las clulas leucmicas se propagan por toda la mdula sea y a muchos otros rganos a travs de la
sangre, no es posible curar este tipo de cncer con ciruga.
CASO CLINICO
Nio de 10 aos que acude a urgencias por dolor en hombro y codo
izquierdos desde hace 4 das, con signos de inflamacin en codo
izquierdo. El dolor en hombro izquierdo cede pero a las 24 h aparece
dolor en hombro derecho, con limitacin para la abduccin y
rotacin interna. No refiere traumatismos. El da anterior hizo
ejercicio (waterpolo). Hace un mes, present un episodio de ojo rojo
diagnosticado de conjuntivitis aguda, que cedi con colirio
antibitico. No refiere procesos infecciosos previos. Antecedentes
personales sin inters. En los antecedentes familiares destacan
madre y to materno con espondilitis anquilosante HLA B27 positiva.
Exploracin fsica: Temp. 38C. Afectacin discreta del estado
general. Inflamacin del codo izquierdo con derrame y limitacin
funcional de la flexo-extensin. Impotencia funcional de abduccin
de hombro derecho. El resto de la exploracin es normal.
PREGUNTA
Cual es la conducta a seguir?
RESPUESTA
a.- Realizar BH, QS, EGO.
b.- Realizar pruebas CPK, GPT, GOT, GGT y DHL.
c.- Realizar serologa para micoplasma, Borrelia, Yersinia
enterocolitica (serotipo O:3), parvovirus, VHA, VHC, RPR con rosa de
Bengala.
d.- Realizar artrocentesis e iniciar antibitico e anti-infamatorio.
PREGUNTA
Se obtiene los siguientes resultados: Hemograma: leucocitos
9.930/mm3 (neutrfilos 53,3%, linfocitos 35,8%, monocitos 6%,
eosinfilos 2,1%), hemoglobina 14,2g/dl, VCM 80,6, plaquetas
308.000/l, protena C reactiva 5mg/dl; VSG 26mm/h. Bioqumica:
CPK, GPT, GOT y GGT normales, factor reumatoide 14 U/ml;
anticuerpos antiestreptolisina (ASLO), inmunoglobulinas y C3 y C4
normales. Serologas para Micoplasma, Borrelia, Yersinia
enterocolitica (serotipo O:3), parvovirus, VHA, VHC, RPR con rosa de
Bengala, negativas. HLA B27 positivo. Radiografas de trax, codo
izquierdo, hombro derecho y sacro-ilacas, normales. Hemocultivos y
cultivo de lquido articular, negativos. Fue diagnosticado con artritis
reumatoide juvenil, se ajusta tratamiento, cual es la conducta a
seguir?
RESPUESTA
a.- Indica ibuprofeno.
b.- Indica prednisona.
c.- Inicia con sulfaxalazina.
d.- Inicia con cloroquina.

nuevo hemograma por fiebre que objetiva un 36% de blastos. Cual


es la conducta diagnostica a seguir.
RESPUESTA
a.- Aspirado de medula osea.
b.- Escaneo con resonancia magnetica.
c.- Frotis y gota gruesa.
d.- Realizar Gamagrama
CASO CLINICO
Lactante femenino de 18 meses, de edad con rechazo a la
bipedestacin, de 4 semanas de evolucin, peso 9,700Kg, talla 82cm,
en el percentil 10, la madre refiere que una semana a tras curso con
tos y rinorrea, sin fiebre, para el cual recibi medicacin. Presento
limitacin funcional de la rodilla derecha. EF: palidez de tegumentos,
as como petequias en regin anterior de trax, adenomegalia
cervical, adems haba llanto a la palpacin profunda de sus
extremidades. Laboratorios: Hb: 11,5g/dL; Leucocitos: 17820(Ne:
21%, Li 80%), Plaquetas: 143000. Blastos en sangre perifrica. LDH:
1463U/L. PCR: 0,1mg/dL.
PREGUNTA
Cul es el diagnstico ms probable?
RESPUESTA
a.- Purpura trombocitopenica trombotica
b.- Sindrome mielodisplasico
c.- Leucemia linfoctica aguda
d.- Leucemia linfoctica crnica
PREGUNTA
Cul de los siguientes es menos probable que forme parte de un
sndrome de lisis tumoral?
RESPUESTA
a.- Hiperuricemia
b.- Hiperpotasemia
c.- Hipernatremia
d.- Hipercalcemia
PREGUNTA
Cul de las siguientes es menos probable como medida para evitar
la lisis tumoral?
RESPUESTA
a.- Hiperhidratacin
b.- Factor estimulante de colonias de granulocitos
c.- Alcalinizacin de la orina
d.- Alopurinol

PREGUNTA
Tras 6 meses de seguimiento se inicia tratamiento con metotrexato
por persistencia de los sntomas. Pasados 10 meses comienza con
astenia y dolor abdominal. En control analtico se objetiva
neutropenia (320/l) y aumento de LDH hasta 588 U/l. Se realiza un

CURSO ENARM CMN SIGLO XXI TEL: 36246001

Pharmed Solutions Institute

PGINA 361

MANUAL DE TRABAJO DEL CURSO ENARM CMN SIGLO XXI


RETINOBLASTOMA (RB)
CIENCIAS BASICAS: Es un tumor intraocular embrionario de origen retiniano que se presenta generalmente en nios menores de 5 aos. Dejado a su
evolucin, el retinoblastoma es casi siempre fatal y un retraso en su tratamiento suele suponer un pronstico visual precario, de ah la
transcendencia del diagnstico precoz. SALUD PUBLICA: Representa aprox. El 3% de todos los canceres presentes en nios menores de 15 aos. Su
incidencia es de 1:15-20.000 recin nacidos vivos y su etiologa espordica en el 70-75% de los casos o hereditaria, en el 25-30%. De acuerdo con la
Direccin General de Epidemiologa de la Secretara de Salud, podra representar la neoplasia slida ms frecuente despus de los tumores del
sistema nervioso central. Generalmente aparece antes de los 2 aos de edad y 95% de los casos se diagnostican antes de los 5 aos. PATOGENIA: El
gen responsable, el RB1, est localizado en el cromosoma 13 y acta de forma dominante; es decir, en clulas donde las dos copias del gen estn
daadas. El 35-40% de los pacientes corresponden a casos hereditarios que son portadores de una mutacin germinal. De ellos, ms de dos tercios
representan nuevas mutaciones (mutacin de novo) sin historia familiar previa. Fenmenos inflamatorios y glaucoma, ambos secundarios a un
tumor que empuja hacia adelante el diafragma cristaliniano o a clulas tumorales que azolvan la malla trabecular. Los sitios afectados ms
frecuentes por metstasis son el SNC y el hueso (60%), dentro de la afecciono sea el sitio ms frecuente fue la rbita y huesos craneales (90%),
huesoso largos (18%). DIAGNOSTICO: Se presenta clnicamente con leucocoria 50-60% (reflejo blanco en la pupila), estrabismo y signos
inflamatorios, ms raramente con glaucoma, celulitis orbital, uveitis y hemorragia vtrea. El 60% de los RB son unilaterales con una edad media al
diagnstico de 24 meses, mientras que Los RB bilaterales son menos frecuentes, suelen ser multifocales y de ms temprana edad de comienzo (15
meses). En la mayora de los nios con tumores bilaterales, ambos ojos estn afectados al diagnstico. Slo en algunos casos de RB unilateral se
desarrolla un tumor contralateral ms tarde. Las caractersticas de inicio precoz, bilateralidad y multifocalidad apuntan a un probable origen
hereditario del proceso. Los retinocitomas o retinomas son tumores benignos resultantes de la regresin espontnea de retinoblastoma y que
raramente pueden ser descubiertos en la edad adulta como lesin residual de un RB que curs de forma subclnica en la infancia. Otros tumores
asociados: Los portadores de la mutacin germinal en RB1 tienen un exceso de riesgo de desarrollar otros tumores. La presencia de tumores
extraoculares (Pinealomas, osteosarcomas, sarcomas de tejido blando, melanomas) llamados segundos tumores primarios, se manifiesta en la
adolescencia o al inicio de la edad adulta, siendo el tiempo medio de aparicin del segundo tumor 10-13 aos. El diagnstico de RB se establece
mediante oftalmoscopia indirecta. La CT, MRI y ecografa sirven para confirmacin del diagnstico y estadio tumoral. En base a la focalidad tamao y
localizacin de los tumores existe un estadiaje de Reese-Ellsworth (en tabla adjunta) que evala el pronstico, I Muy favorable para la conservacin
de vista. II Favorable para la conservacin de vista. III Posible conservacin de vista. IV Desfavorable para la conservacin de vista. V Muy
desfavorable para la conservacin de la vista. TRATAMIENTO: La eleccin de la terapia adecuada depende no solo del estadio del tumor sino de
factores tales como la focalidad (unifocal, unilateral multifocal, o bilateral, localizacin del tumor dentro del ojo y edad del paciente. Las opciones
incluyen: Enucleacin est indicado en todos aquellos casos unilaterales que cumplen alguno de los criterios siguientes: (1) Ocupar ms de la mitad
del ojo. (2)Extensin al humor vtreo. (3)Desprendimiento total de retina. (4)Crecimiento de nuevos vasos en el iris y (5) Implicacin del tumor en
otras estructuras del ojo. La Radioterapia externa incrementa en un 50% la aparicin de otros tumores. Sin embargo, est indicado en RB bilaterales
no susceptibles de tratamiento local. Este mtodo se aplica en tumores mayores de 16 mm que se extienden a la rbita, cercanos a la fvea,
mltiples o cuando se extienden al humor vtreo. La radioterapia local puede ser efectiva en tumores grandes (>16 mm) no susceptibles a la
crioterapia o fotocoagulacin. Aunque reduce la probabilidad de aparicin de tumores inducidos, est asociado a retinopatas y papilopatas.
Fotocoagulacin, este tratamiento se aplica en tumores pequeos (menos de 3 mm de dimetro y 2 de grosor) que no implican el disco ptico o la
mcula. Llega a ser muy efectivo en tumores muy pequeos, con localizacin anterior al ecuador y en tumores donde la altura es igual o menor que
la mitad del dimetro de la base. Crioterapia, es efectiva para el tratamiento de tumores de 5mm de dimetro y 3mm de grosor. Habitualmente son
necesarias varias sesiones. Es de creciente inters el empleo de la Quimioterapia sistmica como terapia coadyuvante en nios previamente
candidatos a enucleacin o external beal radiation bilateral. El empleo de agentes alquilantes puede incrementar la incidencia de segundos tumores
y est asociado al desarrollo de leucemia no linfoblstica e infertilidad. RETINOBLASTOMA TRILATERAL: Es un sndrome bien reconocido que se
presenta en 5 a 15% de los pacientes con la forma hereditaria del retinoblastoma y que se define mediante la formacin de un tumor neuroblstico
de la lnea media intracraneal, tpicamente hasta ms de 20 meses despus del diagnstico del retinoblastoma. Los pacientes que son asintomticos
al momento del diagnstico de un tumor intracraneal tienen un pronstico ms favorable que aquellos pacientes sintomticos.
CASO CLINICO
Nia de 7 meses, previamente sana, presenta leucocoria de 2 meses
de evolucin, no hay presencia de dolor ocular ni sntomas
concomitantes. Al examen oftalmolgico se constata ausencia de
fijacin y esotropia del ojo derecho con ausencia de reflejo rojo.
PREGUNTA
Cual de los siguientes diagnosticos diferenciales es el mas
frecuente?
RESPUESTA
a.- Enfermedad de Coats.
b.- Catarata.
c.- Persistencia de vtreo primario hiperplsico.
d.- Estadio V de la retinopata de la prematuridad
CASO CLINICO
Paciente femenino de 1 ao y 7 meses de edad que acude al servicio
de oftalmologa peditrica, por presentar, edema y eritema
palpebral del ojo derecho, impidiendo su apertura, de 4 das de
evolucin y tratado con antibitico oral no especificado. La madre
refiere que aproximadamente a los 3 meses de edad not que la
lactante presentaba desviacin del ojo derecho haca afuera y un
brillo blanco en el ojo de la nia, como de gato. Sin embargo no
acudi a facultativo hasta los 8 meses de edad, el cul le diagnostica
estrabismo hereditario. Al no mejorar sus sntomas, decide acudir a
nuestro servicio.

CURSO ENARM CMN SIGLO XXI TEL: 36246001

PREGUNTA
Cul es el diagnostico mas probable para este caso?
RESPUESTA
a.- Pars planaris
b.- Displasia retiniana
c.- Catarata congnita
d.- Retinoblastoma
PREGUNTA
Cul es el pronstico, recibiendo un tratamiento adecuado y
oportuno?
RESPUESTA
a.- 10%
b.- 30%
c.- 50%
d.- 90%
PREGUNTA
Cul de las siguientes es menos probable que sea una indicacin
para enucleacin?
RESPUESTA
a.- glaucoma secundario
b.- siembra vtrea
c.- Uveitis
d.- Siembra vtrea

Pharmed Solutions Institute

PGINA 362

MANUAL DE TRABAJO DEL CURSO ENARM CMN SIGLO XXI


INFLUENZA
CIENCIAS BASICAS: Es una infeccin viral aguda de las vas respiratorias, altamente contagiosa, que puede afectar la mucosa nasal, la farngea,
bronquios y en ocasiones hasta los alvolos pulmonares. Los sntomas son parecidos a los del catarro comn o resfriado; sin embargo, son ms
graves y su inicio es generalmente abrupto. La gripe es causada por el virus de la influenza (virus de influenza A, B y C), el catarro comn es causado
por mltiples virus respiratorios (rinovirus, coronavirus, adenovirus, virus parainfluenza). El cuadro clnico de la influenza puede variar, observndose
desde un cuadro febril agudo leve hasta una infeccin pulmonar y/ocomplicaciones graves. SALUD PUBLICA: Puede tener un comportamiento
endmico, epidmico o de una pandemia. Las epidemias de influenza son responsables de 36 000 a 50 000 muertes por ao en pases como EUA en
los ltimos aos, afecta a todos los grupos etarios, pero principalmente a nios menores de dos aos y adultos mayores de 65 aos. PATOGENIA:
Los virus asociados a la influenza humana son miembros de la familia Orthomyxoviridae, tienen una morfologa helicoidal y su genoma est
constituido por una cadena segmentada de ARN, los virus influenza A y B causan infecciones respiratorias en humanos y animales, el virus A es causa
de pandemias. El virus de influenza C afecta en forma moderada a los nios y en forma ocasional a los adultos. Estos virus se distinguen por
variaciones antignicas en dos protenas estructurales (la nucleoprotena y la protena de la matriz). Los virus de la influenza A se clasifican en
subtipos de acuerdo a dos antgenos de superficie: hemaglutinina (H) y neuraminidasa (N). La hemaglutinina es considerada el antgeno mayor para
la cual est dirigida la produccin de anticuerpos neutralizantes y cuya funcin es la adhesin del virus mediante residuos de cido silico en la
superficie del epitelio respiratorio humano. La expresin de la neuraminidasa es menos abundante en la superficie viral y su papel es facilitar la
liberacin de viriones de clulas infectadas del hospedero. Para la clasificacin de este virus se utiliza el siguiente orden en la nomenclatura: el tipo,
el lugar en donde fue aislado, el ao de aislamiento, el nmero de identificacin del laboratorio y, en los aislamientos del virus de la influenza A, el
subtipo H-N (por ejemplo: A/Moscow/10/99 [H3N2]). Estudios de epidemiologa molecular de los virus de la influenza han demostrado que los
subtipos H1N1, H1N2, y H3N2 son los que han circulado en los ltimos aos. Las aves constituyen un reservorio potencial de intercambio gentico
para los virus de la influenza, lo que representa un riesgo latente de pandemias. Los virus de la influenza A tambin infectan a pollos, cerdos,
caballos y ocasionalmente a mamferos marinos. El periodo de incubacin es de 1 a 4 das (promedio de dos das). Los adultos pueden ser
infectantes un da antes de que los sntomas inicien hasta aproximadamente 3-5 das despus. Los nios pueden ser infectantes por 10 das o ms.
Las personas con estados de inmunosupresin pueden excretar los virus por perodos ms prolongados. El virus de la influenza A se caracteriza por
causar enfermedad moderada a grave. El virus B de la influenza causa cuadros clnicos menos graves que el tipo A y tradicionalmente produce
infeccin y enfermedad durante la infancia. La capacidad del virus de la influenza A y B de sufrir cambios antignicos graduales en sus dos antgenos
de superficie, la hemaglutinina y la neuraminidasa, complica la vacunacin contra esta enfermedad. El tipo de variaciones antignicas menores o
deslizamientos antignicos (antigenic drift) resulta de la acumulacin de mutaciones puntuales de los genes que transcriben para estas protenas.
DIAGNOSTICO: La influenza se caracteriza por el inicio agudo de sntomas y signos respiratorios que incluyen: fiebre (elevada de inicio abrupto), tos
seca, coriza, cefalea, odinofagia, ardor farngeo, mialgias, artralgias, y ataque importante al estado general (postracin H3N2). Estos sntomas
ocurren en 50 a 70% de las infecciones por influenza, tanto estacional como por el virus de influenza humana H1N1 2009. Menos comunes son:
fotofobia, dolor abdominal, nusea, vmito y diarrea. La duracin de la enfermedad sin complicaciones habitualmente es de una semana, aunque la
tos y debilidad pueden persistir por ms de 14 das. En particular, en los pacientes con infeccin por el virus A H1N1, se presentaron durante la
epidemia cuadros graves de neumona intersticial y progresin a pulmn de choque. Para la infeccin por el virus influenza A humana H1N1 2009, se
establecieron definiciones epidemiolgicas para la confirmacin de los casos: los confirmados son aquellos que tienen sntomas y signos arriba
mencionados y una prueba confirmatoria positiva que consiste en reaccin de polimerasa en cadena en transcripcin reversa (RT-PCR) en tiempo
real o cultivo viral. Un caso probable es una persona sintomtica que es positiva para influenza A por prueba rpida, pero negativa para H1 y H3 por
RT-PCR. Una persona sospechosa es aquella que no cumple con la definicin de probable o confirmado, no tiene prueba negativa para el virus
nuevo, y: Es un individuo previamente sano, menor de 65 aos de edad, hospitalizado por infeccin respiratoria baja. Vive en un lugar o estado
donde no hay casos confirmados pero viaj recientemente a donde existe uno o ms casos probables o confirmados. Tuvo contacto en los siete das
previos con un caso confirmado o probable. Laboratorio: Idealmente, la muestra de exudado farngeo, nasofarngeo, lavado nasal, aspirado
bronquial o traqueal debe tomarse en los primeros tres das del inicio de los sntomas. El cultivo viral es el estndar de oro, adems permite que el
virus sea tipificado y caracterizado antignicamente. Los medios de cultivo utilizados son huevos embrionados de gallina o el cultivo de rin canino
de Madin-Darby, el rin de chimpanc y otros. Tambin se puede realizar el diagnstico por determinaciones serolgicas al mostrar un incremento
de cuatro veces en la titulacin de anticuerpos contra influenza en una segunda muestra. El suero en la fase de convalecencia debe obtenerse entre
los das 10 a 21 del inicio del cuadro. Las tcnicas serolgicas ms frecuentemente empleadas son las de neutralizacin y la inhibicin por
hemaglutinacin. COMPLICACIONES: Neumona primaria por influenza o neumona bacteriana secundaria. La etiologa ms frecuente de la
neumona bacteriana es: Streptococcus pneumoniae en 48% de los casos, Staphylococcus aureus en 19%, y Haemophilus influenzae no tipificable en
11%. Otras complicaciones del tracto respiratorio incluyen: sinusitis bacteriana, bronquitis, traqueobronquitis y otitis media. En nios se ha asociado
a otras manifestaciones clnicas como son: convulsiones febriles, encefalopata por el virus o encefalopata asociada a la utilizacin de salicilatos
(sndrome de Reye), miositis, rabdomilisis, miocarditis y pericarditis. En pocos casos se informa de complicaciones en sistema nervioso central
como mielitis transversa y polirradiculoneuritis. TRATAMIENTO: Permanecer en casa, en reposo para mejorar los sntomas y evitar contagio a otras
personas. En casos leves a mderados; ofrecer lquidos abundantes, aumentar el consumo de frutas con mayor contenido de vitamina A y C. No se
recomienda suspender la lactancia materna si la madre enferma. Deben evitarse los lugares concurridos, as como cambios bruscos de temperatura,
tabaquismo y exposicin a contaminantes dentro de la casa. El control de la fiebre por medios fsicos, evitando en los nios el uso de salicilatos (por
la asociacin con el sndrome de Reye). Cuando amerite, se podrn administrar paracetamol o AINES con efecto antipirtico (ibuprofeno). No se
recomienda utilizar antibiticos profilcticos. Es importante hacer hincapi en los datos de alarma que sugieren el agravamiento y necesidad de
manejo hospitalario. En la edad peditrica los sntomas incluyen: fiebre persistente, tos productiva con expectoracin, dificultad para respirar,
rechazo al alimento, hipotona o convulsiones. Medicamentos con utilidad clnica al reducir la duracin de los sntomas cuando son empleados en las
primeras 48 horas del inicio de la enfermedad. Tambin pueden ser efectivos en la quimioprofilaxis. La amantadina y la rimantadina inhiben la
replicacin de los virus de influenza A, pero no los de influenza B, se administran por va oral y se utilizan para el tratamiento o quimioprofilaxis,
disminuyen la excrecin viral y reducen en promedio un da la duracin de la enfermedad. El tratamiento se recomienda por cinco das y cuando se
utilizan para quimioprofilaxis, son efectivos en 70-90%. Se recomienda el uso profilctico de la amantadina y rimantadina para contactos o
convivientes. Los inhibidores de la neuraminidasa (zanamivir, oseltamivir) tienen actividad contra influenza A y B, bloqueando el sitio activo de la
neuraminidasa, disminuyen la excrecin del virus y reducen la duracin de los sntomas de la influenza por 36 horas. En ensayos clnicos se ha
encontrado que el oseltamivir disminuye las complicaciones de las vas respiratorias bajas (neumona y bronquitis), disminuye uso de antibiticos y
el riesgo de hospitalizacin. PREVENCIN: La vacunacin anual de personas en grupos de alto riesgo de desarrollar complicaciones y sus contactos,
representa la principal estrategia. Las vacunas de mayor uso son producidas de virus crecidos en huevos embrionados inactivados con formaldehido
o propiolactona, pueden producirse de virus completos tratados con detergentes o de antgenos de superficie (hemaglutinina y neuraminidasa)

CURSO ENARM CMN SIGLO XXI TEL: 36246001

Pharmed Solutions Institute

PGINA 363

MANUAL DE TRABAJO DEL CURSO ENARM CMN SIGLO XXI


purificados. Son ms recomendadas las que contienen antgenos purificados, ya que dan menos reacciones alrgicas. Habitualmente contienen los
tres tipos virales recomendados por la OMS. Diversos estudios han demostrado una eficacia de 80% en nios y de 77% en adultos. Las vacunas
contra la influenza son recomendadas en Mexico para: a) Vacunacin a personas que pueden transmitir influenza a personas en grupos que estn
propensos a desarrollar complicaciones: 1) Vacunacin a personal de salud (personal mdico, enfermera, paramdico). 2) Personal que labora en
asilos de ancianos y casas de estancia. 3) Personas que viven en contacto intradomiciliario con personas que tienen alto riesgo de desarrollar
complicaciones (nios que viven con un adulto con asma). b) Vacunacin de mujeres con embarazo de alto riesgo: especficamente aquellas en el
segundo y tercer trimestre del embarazo por tener el mayor riesgo de complicaciones. c) Vacunacin de personas mayores de 65 aos de edad. Este
grupo presenta el mayor riesgo de hospitalizacin y muerte asociado a complicaciones por influenza. d) Vacunacin de personas de entre 50 a 64
aos. En este grupo se concentra la mayor pre-valencia de personas con condiciones de alto riesgo (enfermedades crnico-degenerativas). e)
Vacunacin de pacientes de cualquier edad con padecimientos crnicos. Estos padecimientos incluyen: enfermedades pulmonares (asma o
enfermedad pulmonar obstructiva crnica) o enfermedades cardiovasculares (insuficiencia cardiaca). As como aquellos con enfermedades
metablicas crnicas (diabetes mellitus, insuficiencia renal) y en pacientes inmunodeprimidos (VIH positivos o trasplantados). f) Vacunacin de nios
de 6 a 36 meses de edad. Se ha demostrado que en esta poblacin existe un riesgo aumentado de hospitalizacin por complicaciones asociadas a la
influenza. Aunque el riesgo en nios menores de seis meses es an mayor, la vacuna trivalente no est aprobada para su uso en este grupo. g)
Viajeros. Se recomienda en aquellos viajeros que no fueron vacunados en el ao precedente si planean viajar a los trpicos, o al hemisferio sur
durante los meses de abril a septiembre. Medidas generales: Las medidas no farmacolgicas que se recomiendan para ayudar a prevenir la
propagacin de enfermedades respiratorias como la influenza son: Cubrirse la nariz y la boca con un pauelo desechable al toser o estornudar. Tirar
el pauelo desechable a la basura despus de usarlo. Lavarse las manos frecuentemente con agua y jabn, especialmente despus de toser o
estornudar. Puede utilizarse alcohol-gel. Evitar tocarse los ojos, la nariz o la boca. Evitar el contacto cercano con personas enfermas. Al enfermarse,
debe permanecer en casa por siete das a partir del comienzo de los sntomas o hasta que hayan pasado 24 horas desde que desaparecieron. Esto
disminuye la posibilidad de infectar a otras personas y propagar ms el virus. Se deben seguir las recomendaciones de la Secretara de Salud con
relacin al cierre de escuelas, evitar frecuentar sitios con multitudes y tomar medidas de distanciamiento social.
CASO CLINICO INFLUENZA
Paciente de 17 aos con fiebre, tos y dolor de garganta, cefalea,
rinorrea y dolor abdominal, por lo que iniciaron tratamiento
sintomtico para rinofaringitis con respuesta no favorable por lo que
regresa a los 3 dias con empeoramiento de los sntomas.

RESPUESTA
a.- 60 mg cada 12 hrs por 5 das.
b.- 45 mg cada 12 hrs por 5 das.
c.- 75 mg cada 12 hrs por 5 das.
d.- 150 mg cada 12 hrs por 5 das

PREGUNTA
Cuales son los sntomas o signos cardinales para considerar caso
sospechoso de influenza?
RESPUESTA
a.- Fiebre con tos o dolor de garganta.
b.- Cefalea, rinorrea y coriza.
c.- Artralgias, mialgias y postracin.
d.- Dolor torcico, dolor abdominal y congestin nasal.

CASO CLINICO INFLUENZA


Nio de 10 aos que consult al servicio de urgencias despus de
presentar crisis convulsiva tonicoclnica generalizada (TCG) de 3 min
de duracin, despus de 4 das de fiebre, tos, inapetencia general y
fatiga; en el hospital curs febril, con debilidad, desorientado; unos
minutos despus present otra crisis TCG de 3 min, se controla las
crisis con mejora 12 horas despus presenta las siguientes
manifestaciones radiologicas Inflitrados pulmonares en ms de dos
cuadrantes, ndice de oxigenacin PaO2/FiO2 menor de 250 y
distensibilidad pulmonar disminuida.

PREGUNTA
Cuales son las carateristicas que presentan mayor complicaciones?
RESPUESTA
a.- Edad >60 aos y <2 aos.
b.- Enfermedad crnica o debilitante.
c.- Embarazo y primeros 6 meses postparto
d.- Pacientes que acudan a una segunda consulta por deterioro o sin
mejora clnica.
PREGUNTA
Se ingresa ingresa anteriomente referido por fiebre de 40 grados,
dolor abdominal, dolor toraxico con dificultad respiratoria, durante
su estancia hospitalaria presenta vmito y diarrea persistentes, TA
100/70, Cul es su conducta a seguir?
RESPUESTA
a.- Unidad mdica hospitalaria con aislamiento estndar,
precauciones de gotas y de contacto.
b.- Aislamiento domiciliario estricto, con listado de datos de alarma,
con indicaciones para reevaluar al da diguiente si fuera necesario.
c.- Aislamiento domiciliario estricto, con listado de datos de alarma.
d.- Compensacin de enfermedad crnica, manejo de
complicaciones y tratamiento antiviral (oseltamivir).

PREGUNTA
Cual es la conducta a seguir?
RESPUESTA
a.- 60 mg de oseltamivir cada 12 hrs por 5 das.
b.- Ceftriaxona 50-100 mg/kg/da IV c/24 hrs por 7-10 das.
c.- FiO2: el necesario para mantener PaO2 arriba de 60 mm Hg.
d.- Presin menor de 35 cm H2O y meseta menor de 30 cm H2 O.
PREGUNTA
Cul seria la conducnta preventiva que mas ayudara a este
paciente posteriormente, para evitar cuadros repetidos?
RESPUESTA
a.- Cubrirse la nariz y la boca con un pauelo desechable al toser
b.- Lavarse las manos frecuentemente con agua y jabn
c.- Al enfermarse, debe permanecer en casa
d.- Aplicacin de vacuna de influenza

PEGUNTA
Cual es el esquema terapeutico antiviral mas apropiado?

CURSO ENARM CMN SIGLO XXI TEL: 36246001

Pharmed Solutions Institute

PGINA 364

MANUAL DE TRABAJO DEL CURSO ENARM CMN SIGLO XXI


OTITIS
CIENCIAS BASICAS: La otitis media aguda (OMA), es la infeccin supurada del odo medio, que tienen un inicio sbito y de corta duracin; asimismo,
denota inflamacin de la cubierta mucoperistica del odo medio. La membrana timpnica inflamada se presenta opacificada, protruyente o con
ambas caractersticas. Segn su tiempo de evolucin la enfermedad puede subdividirse en a) aguda, cuando el proceso dura no ms de 3 semanas;
b) subaguda, cuando la infeccin perdura de 3 semanas a 3 meses, y c) crnica, cuando la enfermedad se prolonga por ms de 3 meses. SALUD
PUBLICA: La OMA es un problema mundial que afecta a uno de cada 4 nios menores de 10 aos y es la infeccin bacteriana ms frecuente en nios
menores de 5 aos. De 25-40% de las visitas peditricas en menores de 5 aos son por OMA. Hay estudios que indican que 80% de los preescolares
menores de 3aos, han tenido cuando menos un episodio de OMA. Hay mayor incidencia durante otoo e invierno., los nios que son llevados a
guarderas parecen ser especialmente susceptibles a padecer OMA, y los expuestos a humo de tabaco. CLASIFICACION: 1. Miringitis: cuando se trata
de una inflamacin de la capa externa de la membrana timpnica que puede ocurrir sola o asociada a una inflamacin del conducto auditivo
externo. 2. Otitis media aguda supurada: infeccin aguda del odo con exudado y de corta duracin. 3. Otitis media secretoria: presencia de lquido
en odo medio detrs de una membrana timpnica integra sin signos agudos o sntomas. 4. Otitis media crnica supurada: presencia de descarga
crnica del odo medio a travs de una perforacin de la membrana, a veces hay perforacin sin descarga. 5. Otitis media recurrente: presencia de 3
episodios de otitis media aguda en 6 meses, o 4 en un ao, o 2 cuadros diferentes con un mes entre ellos. PATOGENIA: Entre las funciones del odo
se incluyen la ventilacin y el equilibrio de las presiones atmosfricas, as como la proteccin ante el sonido y el drenaje de secreciones a la
nasofaringe. Cuando estos mecanismos se alteran, contribuyen a la proliferacin bacteriana, lo que desencadena la infeccin. La obstruccin tubaria
tambin inhibe el drenaje del lquido del odo medio a la nasofaringe, lo que contribuye al desarrollo de la infeccin. La falla de estos mecanismos
puede deberse a inflamacin, infecciones frecuentes, problemas alrgicos o neoplsicos. Aunado a ello, la trompa de Eustaquio de los nios es ms
corta, ms horizontal y tienen menor soporte cartilaginoso. En 2/3 partes de los pacientes con OMA pueden aislarse patgenos bacterianos del
lquido del odo. En 30-50% de los casos se encuentra Streptococo pneumoniae. El Haemophilus influenzae causa entre 20-27% de los casos de OMA.
Moraxella se aisla Moraxella catarhallis se isla en 7-23% de los casos. Los nios hospitalizados desarrollan grmenes del medio ambiente
hospitalario. Las infecciones virales representan el 41% de los casos de OMA. Los virus syncitial respiratorio (74%), parainfluenza (52%) e influenzae
(42%). Algunas patologas como anormalidades craneofaciales y el reflujo gastroesofgico se relacionan con la presencia de otitis media. El uso de
chupn es un factor de riesgo para otitis media recurrente. DIAGNOSTICO: clnica; se presentan como un cuadro de coriza, rinitis, fiebre, tos,
irritabilidad y anorexia. El sntoma ms fidedigno es la otalgia. Los nios muy pequeos no verbalizan el dolor pero se jalan las orejas o rehsan a
deglutir y lloran constantemente. Debe sospecharse cuando el dolor despierta a los nios por la noche. Exploracin del odo medio, mediante el uso
de un otoneumatoscopio (sensibilidad de 95% y especificidad de 80%), debe realizarse de manera que el conducto auditivo externo se encuentre
libre para visualizar correctamente la membrana timpnica y valora su movilidad, los cambios de coloracin y vascularizacin, as como la presencia
de niveles hidroaereos por detrs de la misma, debe realizarse son suavidad para no lastimar. La OMA se manifiesta clnicamente con retraccin,
eritema, disminucin del reflejo luminoso e hipomotilidad de la membrana timpnica; asimismo, se presenta una disminucin de la audicin en
rangos de baja frecuencia. La membrana timpnica, particularmente la pars flcida, se encuentra hiperemica, con la vascularidad visible, y las pars
tensa, abombada y turbia. En un estado avanzado de la enfermedad puede haber ruptura de la membrana timpnica, habitualmente en la pars
tensa, con salida de material francamente purulento, sanguinolento o seroso; la perforacin habitualmente es pequea, lo que la distingue de
etiologas ms raras (necrosante, TB, recurrencia de otitis), y una vez que la pus drena, los sntomas disminuyen de manera importante. De manera
ideal se debe realizar timpanometria para identificar diferencias en la presin dentro y fuera del odo y confirmar la presencia de lquido; asimismo la
reflectometria acstica es un mtodo no invasivo para identificar derrame de odo medio mediante el reflejo del sonido. TRATAMIENTO: Es
importante hacer notar que existen pocas herramientas que ayuden a la pediatra para formular una estrategia adecuada de tratamiento; sin
embargo, en recientes revisiones se ha observado que el uso temprano de la terapia antibitica mejora el curso clnico de la OMA, disminuyendo la
sintomatologa y la incidencia de complicaciones. La mayora de los expertos continua recomendando amoxicilina (primera eleccin) para iniciar
tratamiento emprico de todos los episodios de otitis media a dosis de 80-90mg/kg de peso da durante 5-10 das, la accin de este puede llegar a ser
limitada (20-30% son resistentes). La seleccin debe basarse en la concentracin de los mismos en el sitio de infeccin, en caso de resistencia,
enfermedad grave y H. influenzae y M catarrhalis, se recomienda amoxicilina/clavulanato( 90mg/kg/dia con 6.4 mg/kg/da dividido en 2 dosis). El
manejo conservador sugiere 10 das de antibioticoterapia, aunque hay resultados aceptables con 5-7 das de tratamiento. Si un paciente no mejora
con amoxicilina despus de 72hrs de manejo se debe cambiar de antibitico. El TMP/SFX tienen una elevada resistencia para Streoptococus
pneumoniae 30-44.6%, no se recomienda su uso. En pacientes alrgicos a betalactamicos, las alternativas por su adecuada concentracin en ido
medio son macrolidos y clindamicina. Todos los nios menores de 6 meses con diagnstico de OMA deben recibir tratamiento para evitar
complicaciones. No se recomienda el uso de gotas oticas, con o sin antibitico, gotas nasales, mucolticos, antihistamnicos, pues no aportan
beneficios al tratamiento de la OMA. La administracin de paracetamol o ibuprofeno es necesaria, para el manejo de la fiebre y el dolor.
Tratamiento quirrgico: Miringotomia; cuando hay otalgia severa o complicaciones intratemporales o intracraneales, optimiza la regulacin de las
presiones en el odo medio. COMPLICACIONES: Prdida auditiva (la mas comn), perforacin de la membrana timpnica, mastoiditis aguda,
laberintitis, parlisis facial, colesteatoma (acumulacin de tejido epitelial de crecimiento rpido que va destruyendo el hueso temporal y su
contenido). PREVENCION: La aplicacin de vacuna conjugada antineumococica, se asocia a una reduccin de la colonizacin nasofarngea por
Streptococo pneumoniae. La eficacia estimada de la vacuna heptavalente conjugada para evitar otitis media aguda es de 6%, para otitis media
recurrente 9%, hasta un 57% de las OMA causadas por neumococo y 90% de las otitis causadas por los serotipos de neumococo incluidos en la
vacuna hepatavalente. Se recomienda a los 2, 4 y 6 meses de edad y refuerzo entre los 13 y 15 meses.
CASO CLINICO OTITIS
Paciente varn de 6 aos de edad, sin antecedentes mdicos de
inters, con vacunacin correcta, incluida vacuna antineumoccica.
Cuadro catarral de 5 das de evolucin, con otalgia izquierda de 6
horas de evolucin, fiebre de hasta 39,9C axilar y parestesias en la
hemicara derecha, que posteriormente se extienden en menor
intensidad a la mano y la pierna derechas. En la exploracin clnica
mostraba rigidez de nuca, con signo de Brudzinsky positivo. El resto
de la exploracin neurolgica era normal. En la otoscopia se vean
ambos tmpanos hipermicos, no abombados, y no mostraba
despegamiento del pabelln auricular.

CURSO ENARM CMN SIGLO XXI TEL: 36246001

PREGUNTA
Cual es la conducta diagnostica mas adecuada?
a.- Biometria hemtica.
b.- Factores inflamatorios.
c.- TAC craneal.
d.- Rx de crneo.
CASO CLINICO OTITIS
Mujer de 2 aos de edad, sin antecedentes patolgicos, con
vacunacin correcta, incluida vacuna antineumoccica, que consulta
por cuadro de 4 das de evolucin de fiebre (38,8C axilar mxima),
odinofagia y tortcolis, con desviacin ceflica hacia abajo y la

Pharmed Solutions Institute

PGINA 365

MANUAL DE TRABAJO DEL CURSO ENARM CMN SIGLO XXI


izquierda. Tratada desde haca 24h con ibuprofeno va oral y calor
local, sin presentar mejora. En la exploracin presentaba desviacin
ceflica hacia abajo y hacia la izquierda, con intensa contractura del
esternocleidomastoideo izquierdo, sin otros hallazgos patolgicos.
La exploracin neurolgica era normal. En la otoscopia se vean
ambos tmpanos hipermicos, no abombados, y no haba
despegamiento del pabelln auricular. Se realiz un hemograma,
con leve leucocitosis (11.800 cel/L) sin desviacin izquierda, y PCR
de 67mg/l; el resto de la analtica era normal. Se realiz un
hemocultivo, negativo. Dada la persistencia del tortcolis, se realiz
una TC craneal, en la que se visualiz ocupacin de caja timpnica,
aditus, antro mastoideo y celdas mastoideas, por lo que se catalog
de otomastoiditis izquierda.

mamila y entr a la guardera hace 3 meses. Sus padres tienen


antecedentes de atopia. Sus inmunizaciones estn actualizadas, con
3 dosis de vacuna conjugada de neumococo. Su padecimiento actual
lo inicio hace una semana con rinorrea y obstruccin nasal y desde
hace 10 das fiebre e irritabilidad.

PREGUNTA
Cual es la conducta a seguir?
RESPUESTA
a.- Clindamicina y vancomicina
b.- Ceftriaxona mas gatifloxacina.
c.- Dicloxacilina y amikacina.
d.- Ampicilina y gentamicina

PREGUNTA
Se le administro azitromicina pero aun asi no mejoro. La exploracin
revela una membrana timpnica amarillenta y abombada con el
otoscopio neumtico se observa poca movilidad. Se encuentra
irrritable. Con peso y talla normales en sus percentilas. No hay otras
alteraciones asociadas a la EF. Cul es el diagnotico mas probable
en este caso?
RESPUESTA
a.- Sinusitis
b.- Rinosinusitis
c.- Meningitis
d.- Rinitis alrgica

CASO CLINICO
Se trata de un nio de 16 meses de edad quien tiene el antecedente
de presentar 2 cuadros de infeccin respiratoria desde hace 2 meses
tratadas con antibiticos desconocidos. Tiene an alimentacin con
mamila y entr a la guardera hace 3 meses. Sus padres tienen
antecedentes de atopia. Sus inmunizaciones estn actualizadas, con
3 dosis de vacuna conjugada de neumococo. Su padecimiento actual
lo inicio hace una semana con rinorrea, obstruccin nasal y desde
hace 10 das fiebre, irritabilidad, llanto, hiporexia.

PREGUNTA
Cul es el diagnotico ms probable en este caso?
RESPUESTA
a.- Sinisitis
b.- Rinitis alrgica
c.- Resfriado comn
d.- Otitis media aguda

PREGUNTA
Cul es el diagnstico ms probable para este caso?
RESPUESTA
a.- Rinitis alrgica
b.- Rinosinusitis
c.- Otitis media aguda
d.- Sinusitis

CASO CLINICO
Lactante de un mes y 20 das, hijo nico, que acude a la consulta de
Atencin Primaria por presentar una secrecin blanquecina en
conducto auditivo externo. Se realiza un cultivo de dicha secrecin,
que resulta positivo para SAMR. Se realiza un estudio familiar,
obtenindose cultivo nasal en ambos padres, que resulta negativo.
Al analizar la historia familiar, tenemos conocimiento de que la
madre haba presentado una herida quirrgica en el momento del
parto, cuyo cultivo fue positivo para SARM con el mismo
antibiograma que el hallado en el nio en el CAE, por lo que
suponemos que se ha producido una transmisin vertical de dicho
germen.

PREGUNTA
Cul es el agente etiolgico ms probable de acuerdo a edad y
cuadro clnico para este caso?
RESPUESTA
a.- Haemophilus influenzae
b.- Streptococo pneumoniae
c.- Virus sincitial respiratorio
d.- Parainfluenzae

PREGUNTA
Dado el agente etiolgico, Cul seria la conducnta teraputica mas
adecuada?
RESPUESTA
a.- Dicloxacilina
b.- Penicilina
c.- Ciprofloxacino
d.- Vancomicina

PREGUNTA
Cul es el factor de riesgo ms importante en este paciente para el
desarrollo de esta patologa?
RESPUESTA
a.- Asistencia a guardera
b.- Infeccin previa de vas respiratorias
c.- Edad de 16 meses
d.- Anormalidad anatmica

PREGUNTA
Cul es la entidd menos probable que podra generar este agente
de no dar tratamiento?
RESPUESTA
a.- Neumonia
b.- Osteomielitis
c.- Sepsis precoz
d.- Impetigo

CASO CLINICO
Se trata de un nio de 16 meses de edad quien tiene el antecedente
de presentar 2 cuadros de infeccin respiratoria desde hace 2 meses
tratadas con antibiticos desconocidos. Tiene an alimentacin con

CURSO ENARM CMN SIGLO XXI TEL: 36246001

Pharmed Solutions Institute

PGINA 366

MANUAL DE TRABAJO DEL CURSO ENARM CMN SIGLO XXI


SINUSITIS
CIENCIAS BASICAS: Es la infeccin de uno o varios senos paranasales (etmoidal, maxilar, frontales, esfenoidal) causada por la obstruccin del ostium
de drenaje. Secundario, entre otros factores, a la desviacin septal, adenoiditis o plipos, con produccin de secrecin mucopurulenta e inflamacin
de la mucosa (habitualmente implica siempre un grado de afectacin de la mucosa nasal=rinosinusitis) y sintomatologa de dolor dependiendo del
seno afectado. SALUD PUBLICA: La sinusitis es infrecuente en los menores de un ao, quiz por el menor desarrollo anatmico de los senos
paranasales en este grupo etreo. No se ha guardado relacin con predominio por raza o sexo. La remisin espontnea puede ser hasta de 70%.
CLASIFICACIN: 1. Sinusitis aguda; dura menos de cuatro semanas, los episodios se resuelven con tratamiento mdico, sin dao residual en la
mucosa. 2. Sinusitis sub-aguda; duran de cuatro semanas a tres meses. 3. Sinusitis crnica; tiene una duracin de ms de tres meses, con
alteraciones estructurales en la mucosa sinusal 6 episodios de sinusitis aguda al ao, con alteraciones en la TAC de senos paranasales. 4. Sinusitis
aguda recurrente; cuando el paciente presenta ms de 4 episodios de sinusitis aguda en un perodo de seis meses y existe adecuada respuesta al
tratamiento mdico, sin dao residual de la mucosa. PATOGENIA: Los siguientes son los principales factores de riesgos que conllevan a que se
presente sinusitis en la infancia: Las infecciones respiratorias de las vas areas superiores, inmadurez inmunolgica e inmuno-deficiencias,
hipertrofia adenoidea y la adenoiditis, asistencia a la sala cuna o jardn infantil, la exposicin al humo de los cigarrillos, contaminacin ambiental y la
alergia, cuerpos extraos, plipos y tumores nasales. El papel de la infeccin bacteriana como causa primaria de sinusitis crnica es controvertido,
siendo ms consecuencia de factores no infecciosos. Las infecciones vricas predisponen al desarrollo de la sinusitis bacteriana (80% de los casos),
mientras que el componente alrgico es significativo en el 20% restante. La sinusitis bacteriana casi siempre surge como complicacin de una de las
anteriores. La microbiologa de la sinusitis bacteriana aguda (SBA) es semejante a la de la otitis media aguda, los grmenes que con ms frecuencia
causan tanto la sinusitis aguda como crnica, independiente de la edad, en orden de importancia son: Virus: Adenovirus, influenza, parain-fluenza,
Estreptococo pneumoniae (30%), Haemophilus influenzae (20%), Moraxella catarrhalis (20%), Branhamella catarralis, Streptococo beta hemoltico,
Stafilococo aureus, Grmenes anaerobios. La sinusitis aparece posterior a la rinofaringitis aguda o crnica, presentando antecedentes de inflamacin
viral en vas areas superiores o inferiores. Si persiste por ms de diez das predispone a la inflamacin de la mucosa de los senos paranasales.
DIAGNOSTICO: Clnica; Existen dos situaciones clnicas que nos hacen sospechar una SBA: 1. Sntomas catarrales leves (rinorrea, tos diurna) que no
ha empezado a mejorar tras 10 das (criterio de Persistencia): es la presentacin ms habitual y debe diferenciarse de los catarros encadenados. 2.
Concurrencia de fiebre elevada ( 39 C) y rinorrea mantenidas 3 das (criterio de Gravedad) con afectacin del estado general. Sntomas especficos:
Se ve ms en nios mayores, dolor facial, cefalea, dolor periorbitario y anosmia entre otros (presentes en 1/3 de los casos). Sntomas inespecficos:
Secrecin nasal y obstruccin nasal uni o bilateral. Fiebre. Halitosis. Tos. Identificacin en el nio menor: Los sntomas son inespecficos y se suelen
manifestar como irritabilidad. Se identifica con fiebre alta, aspecto txico. Puede presentar celulitis periorbitaria o aumento de volumen ocular. La
rinorrea puede ser: Hialina. Mucosa. Purulenta. Sanguinolenta. Es de anotar que las caractersticas de las secreciones no se relacionan con el agente
etiolgico. En estos casos de diferenciacin es mas importante el tiempo de evolucin y el estado general del paciente. RINORREA HIALINA: Se ve en
los cuadros agudos, por edema y secundario a enfermedades sistmicas, principalmente rinofaringitis aguda, rinitis alrgica. Hay inflamacin de la
mucosa con aumento de la permeabilidad de los capilares, que produce un exudado seroso en la cavidad sinusal. Se presenta edema del estroma
con infiltracin celular, linfocitico y clulas plasmticas. RINORREA MUCOSA: Por aumento de las glndulas productoras de moco, la secrecin se
estanca creando un medio de cultivo excelente para el desarrollo de grmenes, en el estroma aparecen histiocitos y clulas productoras de moco. La
cavidad nasosinusal con la secrecin, va absorbiendo el aire del seno, creando presin negativa que conlleva a mayores alteraciones histolgicas.
RINORREA PURULENTA: La coloracin podr variar de amarillo, verdoso, marrn, mal oliente y de gran viscosidad, adherente a la mucosa. Hay
infiltrado de polimorfonucleares y de otras clulas plasmticas, histiocitos y fibroblastos. Hay obliteracin glandular, vascular y neural. RINORREA
SANGUINOLENTA: Debido a la ruptura vascular se presenta salida de sangre; aparecen granulomas de colesterol, que acta como factor irritativo
tisular. La patologa se encuentra en un nivel ms profundo y hay compromiso de los vasos sanguneos. La presencia de rinorrea prolongada de ms
de 10 das permite predecir alteraciones radiolgicas asociadas a una infeccin bacteriana (90% en edades entre 2 y 6 aos, y 70% en mayores de 6
aos). La presencia de un catarro comn sin sinusitis puede provocar alteraciones radiolgicas similares, que permanecen hasta dos semanas
despus de la mejora clnica. Adems no distingue entre una sinusitis bacteriana, viral, o de otras causas. Las pruebas de imagen deben reservarse a
casos de fracaso teraputico o empeoramiento de sntomas, y no estn recomendadas en menores de 6 aos. El tratamiento emprico estara pues
justificado sin realizar ninguna prueba previa. Los cultivos de secrecin nasal o exudado farngeo no tienen correlacin con los sinusales. La puncin
y aspiracin directa del seno es el patrn oro diagnstico, pero slo est indicada en contados casos hospitalarios (enfermedad grave con aspecto
txico, inmunodeficiencias y complicaciones supurativas intracraneales). Criterios diagnsticos, los cuales a su vez pueden ser mayores (Dolor,
presin facial. Congestin nasal. Obstruccin nasal. Rinorrea anterior o posterior. Hiposmia o anosmia. Cefalea) o menores (Halitosis. Fiebre. Dolor
dental. Fatiga. Tos. Otalgia). TAC es una importante ayuda imagenolgica para documentar la existencia del compromiso sinusoidal, y tiene unas
indicaciones muy precisas: Persistencia de la sintomatologa luego de manejo apropiado por 10-15 dias. Cronicidad de la sinusitis. Complicaciones
con celulitis facial o periorbitaria. Coexistencia con asma, datos de hipertrofia severa adenoidea rinitis o rinorreas persistentes. Sospecha de
sinusitis mictica. COMPLICACIONES: Celulitis periorbitaria y orbitaria, la trombosis del seno cavernoso, el absceso subperistico, el absceso
cerebral, y el empiema epidural y subdural. TRATAMIENTO: El uso de antibiticos inicialmente no est indicado en la mayora de los casos si no
presenta criterios de gravedad o persistencia. Como tratamiento sintomtico analgsico y antipirtico son efectivos paracetamol (10-15 mg/kg/dia)
e ibuprofeno (4-6mg/kg/ dosis cada 6-8hrs). Suelen ser necesarios pocos das. El uso de descon- gestionantes nasales sigue siendo motivo de
controversia, pero se concepta que inicialmente mejora el drenaje de secreciones provocando alivio sintomtico, siendo su uso no mayor de cinco
das, los de aplicacin tpica se encuentra Oximetazolina al 0.025- 0.05% en dosis de 2-3 gotas o aspersiones en fosas nasales cada 12 horas.
Corticoides intranasales: Se aconsejan cuando existen estados de recurrencia o cronicidad y coexiste rinitis alrgica sobre todo si son persistentes. En
algunos estudios se destaca el efecto antiinflamatorio local. Se encuentran entre otros: Furoato de Mometasona, Fluticasona, Budesonida durante
tres meses. Los corticoides orales no son usados excepto en casos de enfermedades asociadas como poliposis o crisis asmtica. Antihistamnicos
Podran dificultar el drenaje de secreciones. Si existe una base alrgica se recomienda su uso al final del manejo antibitico, en casos de estados
bacterianos. Se encuentran: Loratadina: Presentacin: 5mg/5ml a dosis de 0.2 mg/kg/da cada 12 horas Desloratadina: Presentacin: 2.5 mg/ 5ml: 15 aos de edad: 1,25mg una dosis dia. 6 A 12 Aos: 2,5 mg/da: 5 ml/d Cetirizina: 0.5 mg/kg/da cada 12 horas Levocetirizina: Presentacin: sol. Oral.
0.5 mg/ ml. Gotas: 5mg/ml. Su dosis es de 0.125 mg/kg. Cuando existen procesos infecciosos bacterianos los antibiticos son teraputica central
dentro del tratamiento a realizar. El antibitico de eleccin es la Amoxicilina. Se administra a una dosis de 45 mg/ kg/da, puede aumentarse a 8090mg/kg/dia. El fracaso teraputico por resistencia antibitica se debe en el caso del neumococo (40%) por cepas con susceptibilidad disminuida,
mientras que para Haemophilus influenzae (20%) y Moraxella catharralis (80%) son por produccin de betalactamasas. Por todo lo cual si no existe
mejora con el tratamiento inicial se utilizar de segunda lnea es la Amoxicilina/clavulanato. La dosis es 40-90 mg/kg/da. No obstante, puede ser
usado como primera eleccin en caso de riesgo alto de resistencia (edad menor de 2 aos, toma reciente de antibitico o zonas de alta prevalencia
de neumococo resistente). Otras alternativas de antibiticoterapia son: Cefuroxime a una dosis de 30mg/kg/da y el Cefprozil a 30 mg/ kg/da. Los

CURSO ENARM CMN SIGLO XXI TEL: 36246001

Pharmed Solutions Institute

PGINA 367

MANUAL DE TRABAJO DEL CURSO ENARM CMN SIGLO XXI


das de duracin del manejo antibitico varan de acuerdo a los autores. Se suele recomendar que sea entre 5-14 das. Son indicaciones de
derivacin hospitalaria urgente la afectacin severa del estado general (aspecto sptico, cefalea facial intensa), la sospecha de complicaciones
(craneales, endocraneales y oculoorbitarias, salvo la celulitis preseptal que puede tratarse inicialmente en Primaria) o tumor (sntomas unilaterales
persistentes como epistaxis, obstruccin y rinorrea, siendo la deformidad facial un sntoma tardo) y la existencia de un entorno familiar de riesgo
que no garantice cuidados generales, cumplimiento teraputico y vigilancia eficaz. En rinorrea persistente unilateral siempre se descartar la
presencia de un cuerpo extrao nasal.
CASO CLINICO SINUSITIS
Varn de 9 aos y 8 meses, sin antecedentes de inters, remitido al
servicio de Urgencias de Pediatra, desde su centro de salud por
cefalea y proptosis. Inicio del cuadro 4 das antes, como fiebre,
vmitos, cefalea y congestin nasal; diagnosticado en su centro de
salud de sinusitis y en tratamiento antibitico con
amoxicilina/clavulnico desde hace 48 horas, sin mejora de los
sntomas. Refiere cefalea intensa fronto-temporal derecha y edema
en prpado derecho que ha evolucionado hacia proptosis en las
ltimas horas. A la exploracin fsica, se evidencia ligera proptosis
del ojo derecho, con leve tumefaccin palpebral, as como una
obstruccin de la fosa nasal del mismo lado. En la exploracin
neurolgica solamente destaca una leve dificultad para la mirada
conjugada

PREGUNTA
Cual es la conducta farmacolgica a seguir?
RESPUESTA
a.- Cefotaxima, vancomicina y metronidazol.
b.- Vancomicina, metronidazol y ampicilina.
c.- Ampicilina, metronidazol y amikacina.
d.- Gentamicina, dicloxacilina y metronidazol

PREGUNTA
Cual es la conducta a seguir?
RESPUESTA
a.- Envio a pediatra.
b.- Realizar radiografia.
c.- Cambio de antibiticos.
d.- Agregar anti-inflamatorio esteroideo.

CASO CLINICO
Paciente femenina de 11 aos de edad, con antecedente de rinitis
alrgica, con un ao y medio de evolucin con obstruccin nasal, por
la que recibi tratamiento fuera de esta institucin con esteroides
nasales y antihistamnicos, sin respuesta clnica favorable. Acudi a
nuestra consulta por persistir el sntoma obstructivo nasal y refiere
recientemente no percibe el olor. Durante la exploracin con
nasofibroendoscopia flexible se encontr una lesin de consistencia
slida que se semejaba a la del cornete medio, recubierta con
mucosa de aspecto normal que ocupaba la totalidad de la cavidad
nasal derecha, sin datos de infeccin ni hemorragia, as como
escurrimiento
nasal
posterior.
Se
solicitaron
estudios
complementarios de imagen.

PREGUNTA
Cual de los siguientes criterios no amerita envio urgente a segundo
nivel?
RESPUESTA
a.- Edema, eritema o dolor facial.
b.- Alteraciones visuales.
c.- Datos menngeos.
d.- Falta de mejora y fiebre elevada.

PREGUNTA
Qu senos paranasales es ms probable que estn afectados de
acuerdo a su edad?
RESPUESTA
a.- Etmoidal y frontal
b.- Esfeniodal y etmoidal
c.- Maxilar y frontal
d.- Estmoidal y maxilar

PREGUNTA
Cul de las siguientes condiciones requiere un envio ordinario a
segundo nivel?
RESPUESTA
a.- Factores predisponentes.
b.- Infeccin oportunistas.
c.- Compromiso inmunolgico.
d.- Refractario al tratamiento.

PREGUNTA
Cul sera la conducta teraputica ms adecuada a seguir en este
caso?
RESPUESTA
a.- Reseccin endoscpica nasal ms amoxicilina
b.- Reseccin endoscpica nasal ms cefuroxima
c.- AINE ms eritromicina
d.- Esteroide ms azitromicina

PREGUNTA
Cul no forma parte del protocolo de envio a segundo nivel?
RESPUESTA
a.- Biometria hemtica.
b.- Eosinfilo en moco nasal.
c.- Radiografia de senos paranasales.
d.- Cultivo de moco nasal.

PREGUNTA
Qu agente etiolgico es ms probable causante de la patogenia de
este paciente?
RESPUESTA
a.- Haemophilus influenzae
b.- Streptococo pneumoniae
c.- Moraxella catharralis
d.- Staphilococcus aureus

CASO CLINICO SINUSITIS Y MENINGITIS


Nia de 10 aos que presenta alteraciones del habla y de la marcha,
adormecimiento, un vmito y fiebre. En tratamiento con
amoxicilina-cido clavulnico por amigdalitis diagnosticada hace 4
das. En la exploracin se observan signos menngeos, y en las
exploraciones complementarias un lquido cefalorraqudeo (LCR) con
2.840 leucocitos/ l (60 % polimorfonucleares); hemograma con
21.300 leucocitos/ l (77 % granulocitos) y velocidad de
sedimentacin globular (VSG), 35 mm/h. Se diagnostica de
meningitis bacteriana y se inicia tratamiento con penicilina
intravenosa.

CURSO ENARM CMN SIGLO XXI TEL: 36246001

PREGUNTA
Qu complicacin es ms probable que este presentando el
paciente?
RESPUESTA
a.- Sinusitis
b.- Absceso de tejidos blandos
c.- Mucocele
d.- Absceso preseptal

Pharmed Solutions Institute

PGINA 368

MANUAL DE TRABAJO DEL CURSO ENARM CMN SIGLO XXI


FARINGITIS, FARINGOAMIGDALITIS (FA)
CIENCIAS BASICAS: La faringitis es un proceso inflamatorio causado por diferentes microorganismos de observacin frecuente en la edad peditrica,
que involucra las membranas adyacentes de la faringe en forma aguda o crnica; en el 50 a 60 % de los casos puede identificarse el agente causal. En
la mayora, la etiologa es viral. Debera reservarse el trmino faringitis en sentido estricto a la infeccin primaria viral o bacteriana, circunscrita a
la faringe, y no a la hiperemia farngea que se produce en el transcurso de un cuadro catarral vrico de
vas altas (rinofaringitis aguda). SALUD PUBLICA: Afecta fundamentalmente a nios en edad escolar,
5-10 aos. Es ms prevalente en climas fros o templados y en los periodos de invierno y primavera. La
transmisin es por contacto estrecho persona-persona a travs de las secreciones. El 65-80% de las FA
agudas tienen una etiologa viral, y ocurre como parte de un catarro de la va area superior que
afecta, por lo general, a nios menores de 3 aos. PATOGENIA: En la mayora de los casos, el contagio
se produce por contacto directo a travs de la inoculacin de gotas o transmisin fsica de secreciones respiratorias infectadas. Los posibles agentes
causales de faringitis son mltiples y los podemos diferenciar en: A) VIRALES: Inespecfica suele tener un inicio gradual, con fiebre moderada,
faringodinia, tos irritativa de intensidad variable, poca afeccin del estado general, hiperemia variable, el cuadro suele resolverse en 3-6 das. A
veces puede ser sugestivo de infeccin especifica: los rino, corona, adeno y virus sincicial respiratorio (VSR) son los ms frecuentemente asociados a
cuadros de resfro comn, con malestar general, cefaleas, mialgias y conjuntivitis; los virus coxsackie y echovirus son los causales de la herpangina,
caracterizada por la presencia de pequeas vesculas (1-2 mm) sobre el paladar blando, la vula y los pilares amigdalinos anteriores, y acompaada
de un sndrome febril con odinofagia intensa, por lo general, en nios de corta edad. Puede asociarse a lesiones vesiculares con localizacin en
manos y pies del tipo del denominado sndrome mano-pie-boca. La infeccin por Herpes simple tipos 1-2 presenta lesiones vesiculosos o ulcerosas
en paladar, configurando una gingivoestomatitis, adenopata regional dolorosa y fiebre. En los pacientes con trastornos inmunolgicos se puede
observar como enfermedad mucocutnea crnica. El VEB, causante de la mononucleosis infecciosa, cursa en la mitad de los casos con una faringitis
exudativa acompaada de fiebre, decaimiento, cefalea, poliadenopatas no dolorosas y esplenomegalia que nos hace presumir el diagnstico. La
faringitis puede ser un elemento caracterstico de una primoinfeccin por el VIH; suele estar acompaada de linfadenopatas, a veces de
ulceraciones en las mucosas y sntomas generales como fiebre mialgias, artralgias e hipersomnia. B) BACTERIANAS: La presencia de rinorrea, tos,
ronquera, diarrea, conjuntivitis o la edad inferior a 3 aos hacen poco probable la etiologa bacteriana. La difteria es una enfermedad infrecuente en
esta poca, dada la efectividad de la vacuna existente y la elevada cobertura lograda en la poblacin. De todas maneras, hay que pensar en esta
etiologa ante la aparicin de una faringitis membranosa de color blanquecino claro a oscuro firmemente adherida a la mucosa amigdalina y
farngea, acompaada de un cuadro txico-infeccioso en un paciente con un esquema incompleto de vacunacin. La faringitis aguda por Neisseria
gonorrhoae ha aumentado en aos recientes; en la mayora de los casos, la infeccin es asintomtica y afecta a la poblacin sexualmente activa.
Cuando se aisla este germen en menores de edad, hay que pensar en la posibilidad de abuso sexual. En adolescentes, debemos investigar otras
enfermedades de transmisin sexual. Las cepas de estreptococos de los grupos C y G tienen un comportamiento similar a los del grupo A y se
asocian fundamentalmente a brotes epidmicos de origen alimentario comn (los huevos hervidos fros estn reconocidos como un vehculo
importante). En la faringitis por anaerobios, el compromiso por flora mixta (anaerobios y espiroquetas) es poco frecuente y suele producir un
exudado purulento con aliento ftido. En la forma invasiva periamigdalina puede haber compromiso vascular (tromboflebitis sptica de la vena
yugular); afecta con mayor frecuencia a los adolescentes o adultos jvenes. Dentro de las infecciones bacterianas, la ms frecuente (15-25%)
corresponde al ESTREPTOCOCO B HEMOLTICO DEL GRUPO A (EBHGA) o Streptococcus pyogenes: Se lo puede aislar durante todo el ao aunque
presenta 2 brotes anuales en otoo y primavera. La faringitis es de aparicin aguda; afecta mayormente a los nios entre los 5 a 15 aos, cursa con
fiebre elevada (>39C), odinofagia (intensa), dolor abdominal (por adenitis mesentrica) con nuseas y vmitos, adenopatas cervicales y un exudado
que cubre la faringe posterior y el rea amigdalina con edema de vula, a veces pronunciado, exantema escarlatiniforme o ambiente epidemiolgico
positivo. La infeccin con cepas capaces de producir toxina eritrognica provoca erupcin eritematosa caracterstica de la piel denominada
escarlatina. Esta misma toxina en huspedes comprometidos por otras infecciones, como varicela, puede producir el grave cuadro del shock txico
estreptoccico, con un riesgo de mortalidad del 50 %. Se puede presentar frecuentemente imptigo, celulitis. Menos frecuentes son neumonia,
osteomielitis, meningitis, fascitis necrotizante, las complicaciones supurativas: adenoflemn, adenitis supurada, absceso retrofaringeo, mastoiditis,
mediastinitis. Las complicaciones no supurativas: fiebre reumtica (FR), el riesgo de desarrollar durante una epidemia de EBHA en pacientes no
tratados es aproximadamente del 3%, la artritis posestreptocccica (pacientes con artritis y evidencia de infeccin estreptocccica que no cumplen
con los criterios para FR) y la glomerulonefritis aguda posestreptocccica (GNA) est ms relacionada a cepas nefritgenas, asociadas con mayor
frecuencia a infecciones de piel. DIAGNSTICO: En la mayora de los casos no es posible realizar un diagnstico etiolgico solamente sobre la base
de datos clnicos, el examen fsico de las estructuras farngeas debe ser minucioso, buscando elementos clinicos que puedan ayudar al diagnstico. El
cultivo de fauces en placas de agar sangre es el paso diagnstico definitivo (gold estndar), debe tomarse la muestra de faringe y amgdalas y, si no
puede sembrarse inmediatamente, puede quedar el hisopo seco en un tubo estril a temperatura ambiente por 24 horas hasta su siembra en los
medios correspondientes. La sensibilidad de los cultivos oscila entre el 73 al 100%. Es conveniente examinar los cultivos hasta las 48 horas
posteriores a su siembra para poder determinar su verdadera negatividad. Actualmente, los mtodos rpidos de deteccin de antgeno
estreptoccico son un adelanto en el diagnstico de faringitis. Las pruebas que emplean el sistema de aglutinacin de ltex son muy especficas (9597%) y moderadamente sensibles (75 al 90%) en comparacin con el cultivo de fauces, y se requiere de una tcnica relativamente sencilla, de bajo
costo y de informacin rpida. Las pruebas de inmunoensayo enzimtico de fase slida, inmunoensayo ptico y sondas de DNA dan resultados
similares aunque requieren de un laboratorio especializado y son ms costosa. La faringitis estreptoccica es rara antes de los 3 aos y excepcional
antes del ao. Sin embargo hay que recordar que nios que asisten a guardera pueden presentar a partir de los 18 meses, especialmente en brotes
epidmicos. Se ha demostrado que el uso de los mtodos rpidos en reas de emergencia ha logrado una reduccin significativa en la prescripcin
de antibiticos en faringitis del 70% y en un 93% en aqullos con cultivos negativos. A veces es necesaria la realizacin de otros anlisis para
confirmar el diagnstico etiolgico: cultivos especiales cuando se piensa en gonococo (medios de Thayer-Martin), difteria (medio de Loeffler),
tinciones especiales cuando se piensa en espiroquetas o fuso-bacterias, realizacin de hemocultivos ante cuadros severos de sepsis, pruebas
serolgicas especficas ante la sospecha de EBV, CMV, VIH, bsqueda de antgenos, PCR, cuerpos de inclusin o en cultivo de clulas. TRATAMIENTO:
FA viral: Sintomtico con paracetamol o naproxen de 3 a 5dias. Ante la enfermedad herptica en el paciente inmunosuprimido, se puede realizar un
tratamiento por va oral con aciclovir a200 mg, en 5 tomas por da, por 7 das. El mismo planteo se puede aplicar para la Influenza A con el uso de
Amantadina 6 mg/kg /da por va oral, o inhibidores de la neuraminidasa. Faringitis por EBHA: Antibitico de eleccin sigue siendo la penicilina V
(fenoximetilpenicilina potsica) VO por 10 das <12 aos: 250mg c/12 hrs, >12 aos: 500mg c/12 hrs. Peniclina G benzatina IM profunda <12 aos
600000U, >12 aos 1200000 U (hasta 27 kg de peso se indican 600.000 U; ms de 27 kg se indican 1.200.000 u. Es muy importante que su aplicacin
sea realizada por personal idneo y en nios mayores de 2 aos). Amoxicilina VO por 10 das 40-50mg/kg/da c/12 o 24 hrs (mximo 500mgs
c/12hrs). Ninguno erradica el 100% de los casos de faringitis por EBHA. No deberemos indicar antibiticos sin confirmar el diagnstico. Si por alguna

CURSO ENARM CMN SIGLO XXI TEL: 36246001

Pharmed Solutions Institute

PGINA 369

MANUAL DE TRABAJO DEL CURSO ENARM CMN SIGLO XXI


razn se inicia el tratamiento hasta tener confirmacin de los cultivos, debemos suspenderlo si fueran negativos. En pacientes alrgicos a penicilina,
el estolato de eritromicina a una dosis de 40 mg/kg/da, en 3 4 dosis diarias, cefadroxilo 30mg/kg/da c/12 hrs por 10 das. Azitromicina
10mg/kg/dia por 5 dias p 20 mg/kg/da por 3 das (mximo 500mgs dosis). Resistencia a macrolidos clindamicina 20mg/kg/da c/12 hrs por 10 das.
En la actualidad, slo azitromicina, cefadroxilo, cexima y cefdinir est aprobada por la FDA como tratamiento una vez al da para faringitis por
Streptococus en nios. Los episodios causados por Streptococcus beta hemoltico de los grupos C y G requieren del mismo tratamiento antibitico.
Los relacionados con el Arcanobacterium haemolyticum se pueden tratar con penicilina o macrlidos. Con respecto a este grupo de pacientes
portadores de EBHA asintomtico no existe riesgo de complicaciones (FR o GNA) y no se requiere tratamiento. No hay urgencia en iniciar un
tratamiento antibitico ante un caso de faringitis con sospecha de ser por EBHGA, dado puede demorarse hasta 9 das de comenzado los sntomas,
para evitar la complicacin no supurativa del Streptococcus pyogenes. En las faringitis por anaerobios, la droga de eleccin es la penicilina y puede
ser administrada por va endovenosa u oral de acuerdo a las circunstancias y necesidades del paciente; otros esquemas pueden ser propuestos,
como clindamicina, ampicilina-sulbactam. Puede ser necesario que paciente requiera un tratamiento quirrgico combinado, por ejemplo, de un
absceso periamigdalino. La difteria y el gonococo tienen tratamiento especfico. En la primera deber indicarse antitoxina diftrica y el antibitico
correspondiente. El tratamiento quirrgico (amigdalectoma) es una indicacin excepcional y se puede plantear ante la obstruccin de la va area,
absceso periamigdalino (por su alta recurrencia 20%) y discutido en FR y amigdalitis crnica. De realizarse extirpacin de la amgdala, es importante
su envo para completar el estudio anatomopatolgico a fin de poder descartar otras etiologa.
CASO CLINICO FARINGITIS
Paciente femenino de 24 meses de edad el cual inicia sbitamente
padecimiento caracterizado por irritabilidad, disminucin en la
ingesta de alimento, dificultad para dormir, leve cianosis peribucal
principalmente con el llanto, a la exploracin fsica se observa
estridor con respiracin superficial y dificultad respiratoria,
presentando vomito durante la revisin.
PREGUNTA
Cul es la conducta a seguir?
RESPUESTA
a.- Indica antibitico de amplio espectro y desinflamatorio.
b.- Indica manejo ambulatorio y signos de alarma.
c.- Suministra primera dosis y enva a segundo nivel.
d.- Sumistra oxigeno, antibiticos y antipirtico.
PREGUNTA
El paciente presenta leve mejora sin embargo durante las siguientes
24 horas disminuye volumen urinario mas vomito recurrente, se
maneja restitucin de liquidos sin respuesta adecuada, cual es la
manifestacin mas importante para envio a segundo nivel?
RESPUESTA
a.- Oliguria.
b.- Vomito.
c.- Examantema.
d.- Fiebre persistente.
PREGUNTA
Cual de los criterios no es por Centor para el diagnostico de
faringitis estreptoccica?
RESPUESTA
a.- Ausencia de tos.
b.- Linfadenitis.
c.- Exudado amigdalino.
d.- Inicio sbito.
CASO CLINICO AMIGDALITIS
Nia de 5 aos acude al servicio por presentar fiebre (38,6 C),
odinofagia, trismus leve, rechazo alimentario, dolor y leve
tumefaccin latero-cervical izquierda y cierta dificultad a la
movilizacin cervical. No presentaba signos de disnea. El cuadro
empez cuatro das antes. Se conoce alrgica a penicilina, portador
de tetraologia de fallot, acompaado de retrazo mental.
PREGUNTA
Cual es la conducta a seguir?
RESPUESTA
a.- Amoxicilina- clavulanico.
b.- Trimetropima + sulfametoxazol.
c.- Clindamicina.
d.- Cefepime

CURSO ENARM CMN SIGLO XXI TEL: 36246001

CASO CLINICO
Una nia de 10 aos, es llevada
por la madre a consulta por un
cuadro de varios meses de
evolucin de fatiga, prdida de
peso y disfagia. Refiere tambin
presentar mala calidad del sueo,
lo que la lleva a estar
somnolienta durante el da, bajando su rendimiento escolar. Al
preguntarle a la familia, refieren que ha comenzado a roncar de
forma muy notoria. Al interrogarla por antecedentes mdicos, la
paciente refiere ser sana, a excepcin de infecciones del tracto
respiratorio
superior
recurrentes.
No
ha
presentado
hospitalizaciones ni cirugas previas. EF: destaca la presencia de
respiracin ruidosa con la boca abierta, halitosis y se observa la
siguiente imagen. No refiere historia de fiebre ni linfadenopata.
PREGUNTA
Cul es el diagnstico ms probable en este caso?
RESPUESTA
a.- Absceso periamigdalino bilateral
b.- Amgdalas hipertrficas
c.- Mononucleosis infecciosa
d.- Faringoamigdalitis
PREGUNTA
Estara indicado realizar un exudado farngeo a esta paciente en
este momento?
RESPUESTA
a.- Si pues est cursando con un cuadro agudo sintomtico
b.- No, el diagnstico es clnico
c.- No, pues no detectaramos el agente etiolgico
d.- Si, pues es un procedimiento crnico
PREGUNTA
Cul es la conducta diagnostica ms adecuada a seguir en este
momento?
RESPUESTA
a.- Drenaje y antobioticoterapia
b.- Quirrgico
c.- Antibitico ms antiinflamatorio
d.- Antiinflamatorios y reposo relativo
6 meses despus es llevada nuevamente por la madre la cual la
refiere con fiebre de hasta 40, e inicio sbito de odinofagia, asi
como exudado en amgdalas. Cul es el agente etiolgico ms
probable en este momento?
RESPUESTA
a.- Streptococo beta hemoltico
b.- Haemophilus influenzae
c.- Virus sicitial respiratorio
d.- Streptococo pneumoniae

Pharmed Solutions Institute

PGINA 370

MANUAL DE TRABAJO DEL CURSO ENARM CMN SIGLO XXI


EPIGLOTITIS Y LARINGOTRAQUEITIS
CIENCIAS BASICAS: Tambin llamada supraglotitis es una celulitis bacteriana de la supraglotis, que puede obstruir completamente la va area. Se
reconoce como una entidad rpidamente progresiva, que pone en peligro la vida y que afecta predominantemente a nios. SALUD PUBLICA:
Clsicamente se presenta en nios de 2-6 aos, sin embargo se puede presentar a cualquier edad, Incluso en neonatos. Existe una mayor incidencia
en invierno y primavera. PATOGENIA: El Haemophilus influenzae tipo B es el microorganismo identificado en el 90% de los casos. Esta bacteria
puede ser parte de la flora normal de la nasofaringe o puede ser adquirido mediante transmisin respiratoria. Otros patgenos aislados son el
estreptococo B hemoltico de los grupos A y B, estafilococos, neumococos, Klebsiella, Haemophilus parainfluenzae, Pseudomonas y Candida. Cuando
el edema aumenta como resultado del infiltrado inflamatorio difuso, la va area se obstruye parcialmente, pero el moco puede obstruirla
completamente. Nio grave con aspecto toxico. La progresin es rpida en trmino de horas. DIAGNOSTICO: Clnica; los 3 signos ms comunes de
supraglotitis son fiebre, dificultad respiratoria e irritabilidad. El paciente muestra estridor inspiratorio, retracciones, sialorrea, adems de odinofagia
y disfagia. Generalmente no cursan con disfona, ni tos. El estridor se presenta cuando la obstruccin de la va area es casi completa. Asumen una
posicin apoyados hacia delante y como olfateando (posicin en trpode). La aspiracin de secreciones en la va area ya comprometida puede
desencadenar laringoespasmo. El diagnostico de epiglotitis se hace mediante observacin directa de la supraglotis, en un ambiente controlado,
generalmente en quirfano. Laboratorio, se puede tomar BH, en la que se observa leucositosis severa con neutrofilia. La utilizacin de radiografas es
controversial. Aunque no se cuestiona que la supraglotis, puede ser diagnosticada en una placa lateral de cuello (signo del pulgar), la pregunta es si
realmente es un estudio necesario y sobre todo seguro. TRATAMIENTO: Si se sospecha de una supraglotitis, el primer paso es evitar la estimulacin
del nio. Una vez que se establece el diagnostico hay que asegurar la va area. Por muchos aos se realizaba una traqueotoma, en cuanto se
diagnosticaba al paciente. Durante los ltimos 20 aos la intubacin endotraqueal ha sido el mtodo estndar de manejar la supraglotitis aguda. Se
recomiendan la intubacin de rutina en todos los pacientes con epiglotitis; en el momento de la intubacin se deben obtener muestras de la
epiglotis para cultivo y tomar hemocultivos. Se debe iniciar antibitico intravenoso; se maneja con ampicilina y cloranfenicol. Se recomienda como 2
lnea de eleccin el ceftriaxone (100 mg/kg/da), cefotaxime (100 mg/kg/da) o ampicilina-sulbactam (200 mg/kg/da); la duracin del tratamiento es
usualmente de 10 a 14 das, complementado con antibiticos orales. El tratamiento con corticoesteroide es copntroversial. Con el tratamiento
adecuado el edema de la supraglotis cede dentro de las 48-72 hrs, por lo que se puede extubar al paciente casi siempre al tercer da.
COMPLICACIONES: La neumona la ms frecuente y, en menor porcentaje, meningitis, adenitis cervical, pericarditis, artritis sptica y otitis media.
Pueden presentarse complicaciones secundarias a la hipoxia, como dao en el SNC, la liberacin de la obstruccin sbita de la va area puede
producir edema pulmonar. LARINGOTRAQUEOBRONQUITIS (CRUP). CIENCIAS BASICAS: Es un sndrome respiratorio clnico agudo, de inicio sbito,
de presentacin en edad peditrica. Caracterizado por la trada estridor larngeo inspiratorio, tos ronca o perruna (traqueal), y disfona. Hay signos
de dificultad respiratoria secundario a la obstruccin larngea o traqueal; trmino usado casi exclusivamente para referirse a la laringotraquetis de
origen viral, frecuentemente precedida de un episodio de coriza, rinorrea clara y leve aumento de la temperatura. Se considera una enfermedad que
se autolimita, sin embargo puede evolucionar a dificultad respiratoria grave. Algunos nios con historia de atopia, tienen un cuadro no precedido de
sntomas virales que se le ha llamado CRUP espasmdico. Ambos son tratados similarmente siempre y cuando los signos y sntomas sean los
mismos. SALUD PUBLICA: Afecta a nios entre 3 meses y 6 aos de edad, con un pico a los 2 aos. Ms frecuente en sexo masculino. Predomina en
otoo e invierno. Con una incidencia anual de 18 por 1.000 nios menores de 6 aos de edad. PATOGENIA: Los agentes causales ms frecuentes son
el virus parainfluenza (65%) tipo 1 (50%), 2 y 3; con menor frecuencia; adenovirus, sincitial respiratorio, influenza A y B, sarampin y
excepcionalmente; Mycoplasma, enterovirus, parotiditis, rinovirus y difteria. El padecimiento inicia en la nasofaringe, se extiende por el epitelio
respiratorio de la laringe y la trquea. El virus se adhiere directamente a la mucosa, mediante los macrfagos o los linfocitos, provocando
degranulacin con liberacin de mediadores inflamatorios preformados o de neo formacin, que ocasionan inflamacin difusa con eritema y edema
que en las cuerdas vocales, disminuye la movilidad y ocasiona disfona y participa en el estridor; en la trquea provoca tos ronca. En la regin
subgltica que es la porcin ms estrecha de las vas respiratorias superiores en el nio, con esqueleto cartilaginoso, el edema provoca disminucin
del calibre, con dificultad para el flujo areo, que se manifiesta por estridor inspiratorio. Si la enfermedad progresa se forman exudados fibrinosos y
pseudomembranas que aumentan la obstruccin. La extensin hacia los bronquios o alveolos, ocasiona laringotraqueobronquitis, o
laringotraqueobronconeumonitis respectivamente, frecuentemente son provocados por complicacin bacteriana de la enfemedad viral inicial.
DIAGNOSTICO: Se recomienda realizar el diagnostico de CRUP basados en las manifestaciones de la triada clnica: disfona, estridor larngeo
inspiratorio y tos traqueal; aunados a los siguientes antecedentes, inicio sbito, fase prodrmica de 12-48hrs previas con rinorrea, fiebre y tos no
traqueal. Suelen presentarse signos progresivops de dificultad respiratoria. Puede encontrase disminucin del murmullo vesicular por la obstruccin
y datos variables de dificultad respiratoria como agitacin por hipoxia, aleteo nasal, tiros supraesternales e intercostales, ms severos cianosis, tiros
supraclaviculares, inquieto y ansioso. Generalmente no se requieren estudios de laboratorio; BH leucocitos con elevacin moderada con predominio
de linfocitos. Se pueden solicitar cultivos de virus y pruebas serolgicas para identificar el virus causal. La radiografa de trax muestra como dato
caracterstico, estrechamiento de la columna area a nivel subglotico y el signo de punta de lpiz o en torre de capilla. CLASIFICACION: Forbes la
cataloga en 4 etapas de acuerdo a gravedad y progresin: Etapa 1; fiebre, disfona, tos traqueal y estridor larngeo inspiratorio al explorar al
paciente. Etapa 2; Estridor respiratorio continuo y signos de insuficiencia respiratoria. Etapa 3; manifestaciones clnicas de hipoxia e hipercapnia:
inquietud, ansiedad, palidez, diaforesis y taquipnea. Etapa 4; cianosis intermitente, cianosis permanente y paro respiratorio. Con el objetivo de
iniciar rpidamente el tratamiento, es conveniente calificar la gravedad de la enfermedad. Con la escala del cuadro adjunto. Tambin se puede
utilizar la siguiente valoracin: LEVE: tos traqueal intermitente, estridor solo cuando se agita, no en reposo, taquipnea menor de 40/min y
taquicardia menor 100/min. No hay inquietud, el nio esta hidratado y su estado mental es normal. MODERADA: Estridor audible en reposo, que
aumenta cuando se agita, tos traqueal constante, aumento del trabajo respiratorio con tiros, frecuencia cardiaca >100/min y respiratoria >40/min,
pero menor de 50. Puede mostrase exigente, pero se encuentra alerta y es confortado por sus padres. SEVERA: Se agrega al cuadro clnico anterior
hipoxia con cianosis, gran inquietud y alteraciones de la conciencia con empeoramiento de las manifestaciones de insuficiencia respiratoria.
TRATAMIENTO: En 90% de los casos es ambulatorio, menos de 10% se hospitaliza, apenas 1% llega a requerir apoyo ventilatorio. 1. Medidas
generales: reposo ambiente tranquilo, control de la temperatura, hidratacin adecuada. 2. Va area permeable: la manera tradicional efectiva es
humedecer el ambiente. El ambiente hmedo es tan efectivo como la nebulizacin fra, este ltimo es ms seguro evita quemaduras por el agua
caliente. La nebulizacin fra, humedece la secreciones, produce vasoconstriccin que ayuda a desinflamar la mucosa, tambin disminuye la
viscosidad de las secreciones mucosas de trquea y bronquios. Se recomienda agregar oxgeno al 30-40% para prevenir o tratar la hipoxia. 3.
Epinefrina (adrenalina): Se administra por nebulizacin simple, ha hecho que en la actualidad la traqueotoma no sea necesaria. Se cree que su
mecanismo de accin, es la estimulacin de los receptores adrenrgicos con la subsecuente vasoconstriccin de los capilares arteriales, lo que da
lugar a reabsorcin de lquido en lugar de fuga-, a partir del espacio intersticial, con la disminucin consecuente del edema larngeo y de
secreciones mucosas de trquea y bronquios. Debe vigilarse la aparicin de taquicardia e hipotensin. Deber administrase en un servicio de
urgencias solo a nios severamente enfermos (>5); se emplea al 2.25% en 3 ml de solucin salina normal durante 20 min, 0.25 ml en nios con peso

CURSO ENARM CMN SIGLO XXI TEL: 36246001

Pharmed Solutions Institute

PGINA 371

MANUAL DE TRABAJO DEL CURSO ENARM CMN SIGLO XXI


de <20Kg, 0.5ml si peso 21-39Kg y 0.75ml si el peso es >40Kg. 4. Corticoesteroide: La dexametasona es el ms aceptado y empleado, tiene accin
prolongada (V de 36-72 hrs). Su efecto antiinflamtorio disminuye el edema de la mucosa larngea entre 2-6 hrs despus de su administracin. La
dosis terapeutica vara de 0.15-0.6 mg/kg IM en una sola dosis. O.6mg/kg es la ms efectiva, sin pasar de 10 mg como dosis total. 5. Intubacion
endotraqueal: si no hay respuesta a las medidas anteriores, y se observa obstruccin progresiva de la via area, con signos de fatiga en el nio por
trabajo respiratorio laborioso que sugiera hipoxia o hipercapnia y alteracin de la conciencia. Los antibiticos no estn indicados, solo estn
reservados en aquellos pacientes que se sospeche una sobreinfeccin bacteriana, una laringotraqueitis bacteriana o una
laringotraqueobronconeumonitis, y se debe cubrir principalmente Staphylococcus aureus, Streptococcus pyogenes, Streptococcus pneumoniae, y
Haemophilus
CASO CLINICO EPIGLOTITIS
Nia de 4 aos, previamente sana, con fiebre y sntomas catarrales
de 24 horas de evolucin. Los padres la encuentran con quejido
respiratorio, a la exploracin clnica revela tiros intercostales,
cianosis peribucal, respiracin superficial, sibilancias audibles a
distancia y estado de alerta disminuido, llanto dbil pero disfonico.
PREGUNTA
Cual es la gravedad de la obstruccin de la via area?
RESPUESTA
a.- Leve.
b.- Moderada.
c.- Grave.
d.- Severa.
PREGUNTA
Considerando el cuadro clnico. Cual es la patologia mas frecuente
para establecer un diagnostico diferencial?
RESPUESTA
a.- Cuerpo extrao.
b.- Difteria larngea.
c.- Epiglotitis post-vacunacion.
d.- Traqueitis bacteriana.
PREGUNTA
Cul es la conducta a seguir?
RESPUESTA
a.- Iniciar tratamiento farmacolgico.
b.- Administracin de oxigeno.
c.- Traslado a hospital.
d.- Administracion de prednisona.
CASO CLINICO LARINGOTRAQUEITIS
Ingresa mascilino de 4 aos de edad el cual inicia padecimiento
sbito con estridor larngeo inspiratorio, refiere la madre que hace
dos das inicio con tos, rinorrea y fiebre no cuantificada, motivo por
el cual acudi a consulta en centro de salud donde fue indicado
tratamiento con analgsico, medidas generales y datos de alarma,
agrega que una hora antes del ingreso se incrementaron los
sntomas con dificultad para hablar, inquietud, llanto con disfona y
tos seca.
PREGUNTA
Cuantos criterios clnicos clsicos presenta el paciente
laringotraqueitis?
RESPUESTA
a.- 1
b.- 2
c.- 3
d.- 4

para

PREGUNTA
Cual es la conducta a seguir para establecer la diagnostico clinico?
RESPUESTA
a.- Biometria hematica.
b.- Radiografia de cuello.
c.- Administrar corticoides.
d.- Administrar adrenalina.

CURSO ENARM CMN SIGLO XXI TEL: 36246001

PREGUNTA
Cual es la conducta teraputica a seguir?
a.- Administracion de dexametasona .60mg/Kg DU.
b.- Budesonida inhalada.
C.- Fluquitasona.
d.- L-epinefrina nebulizada 1:1000
CASO CLINICO
Se trata de femenino de 22 meses de edad la cual es llevada a
urgencias por su madre debido a que la observa que no responde a
estimulos, el nico antecedentes es la presencia de tos, fiebre y
rinorrea que fue tratada con frmacos sintomticos, a la exploracin
fsica se observa paciente letrgica, hipotnica, con dificultad
respiratoria, estridor larngeo, palidez distal y cianosis central, FC 69,
FR 42.
PREGUNTA
Considerando la sintomatologia cual es el estado de la
laringotraqueobronqitis segn los criterios de Westley?
RESPUESTA
a.- Leve.
b.- Moderada.
c.- Severa.
d.- Amenaza la vida.
PREGUNTA
Cul es la conducta a seguir?
RESPUESTA
a.- Iniciar RCP.
b.- Intubacin endotraqueal.
c.- Ventilacin positiva intermitente.
d.- L-epinefrina SC
PREGUNTA
Cul es el agente etiolgico ms probable en este caso?
RESPUESTA
a.- Parainfluenza 1
b.- Virus sincitial respiratorio
c.- Parainfluenza 2
d.- Influenzae
CASO CLINICO
Una nia de 7 aos de edad es llevada al pediatra debido a fiebre de
39.7, dificultad para deglutir y babeo. En el transcurso de algunos
minutos luego de llegar al consultorio, presenta estridor inspiratorio
y dificultad respiratoria. Se toma una radiografia de cuello donde se
observa engrosamiento de pliegues epigloticos y ariepigloticos.
PREGUNTA
Cul es el agente etiolgico mas probable en este caso?
RESPUESTA
a.- Haemophilus influenzae
b.- Mycoplasma pneumoniae
c.- Virus de parainfluenza
d.- Virus sincitial respiratorio

Pharmed Solutions Institute

PGINA 372

MANUAL DE TRABAJO DEL CURSO ENARM CMN SIGLO XXI


BRONQUITIS
CIENCIAS BASICAS: La bronquitis aguda, desde un punto de vista fisiopatolgico, se refiere a la inflamacin aguda de la mucosa bronquial.
Clnicamente, no est muy bien definida, pero se trata de una infeccin respiratoria de etiologa viral que no suele durar ms de dos semanas. La
mayora de los autores estn de acuerdo en que la tos, con o sin expectoracin, es el sntoma ms comnmente observado. Es una enfermedad
adquirida en la comunidad autolimitada. SALUD PUBLICA: Afecta a nios en edad peditrica, ms frecuente en menores de 2 aos. No hay
estadsticas exactas. Ocupa la sptima causa de muerte infantil en el apartado de infecciones respiratorias junto con la bronquiolitis. PATOGENIA:
Los virus respiratorios: influenza, parainfluenza, virus respiratorio sincitial son los responsables de la mayora de los casos de bronquitis aguda. Otros
grmenes aislados ocasionalmente son: Bordetella pertusis, parapertusis, Mycoplasma pneumoniae y Chlamidia pneumoniae. Las bronquitis
ocasionadas por el virus de influenza A, son graves y frecuentemente producen pandemias. El virus entra por la nariz o por la boca, se replica en los
acmulos linfticos del anillo de Waldeyer; puede provocar viremia y
posteriormente se extiende ms abajo llegando a los bronquios, en la
mucosa bronquial se adhieren directamente a ella o bien son
transportados por macrfagos, a los cuales pueden destruir o no; estos
ltimos liberan mediadores inflamatorios ya elaborados y almacenados
en sus granulos o de una nueva elaboracin, los que provocan dao a la
mucosa, edema, inflamacin con disminucin de la motilidad ciliar,
aumento en la secrecin mucosa y disminucin de la luz bronquial, con
resistencia al flujo del aire, mayor cuanto menor es el nio, ya que hay
relacin directa entre la edad del paciente y el calibre de la luz
bronquial. Dependiendo del germen, de su virulencia, del tamao del
inoculo y de las condiciones del hospedero, pueden ocasionar necrosis
de las clulas superficiales de la mucosa de las vas respiratorias. El
periodo de incubacin va de 2-14 das segn agente etiolgico.
DIAGNOSTICO: Eminentemente es clnico; Estornudos, rinorrea hialina,
dolor, ardor farngeo y fiebre de moderada a severa. La tos es
inicialmente seca, ronca, por la irritacin de la mucosa farngea y
traqueal y posteriormente se vuelve hmeda, productiva; puede ser
hemetizante y frecuentemente se acompaa de dolor retroesternal. Al
examen fsico el paciente est inquieto, febril, se observa la mucosa
nasal edematosa y enrojecida, la faringe hiperemica con moco hialino.
A la auscultacin se escuchan estertores bronquiales diseminados en ambos campos pulmonares, es frecuente encontrar roncus y sibilantes. El
esputo o las flemas pueden ser transparentes, de color blanco, amarillo o verdoso; estos cambios de coloracin no estn relacionados con la
sobreinfeccin bacteriana, sino con la peroxidasa liberada por los leucocitos presentes por el moco. Por lo tanto, el moco verde no es motivo para
instaurar tratamiento antibitico. La enfermedad en los nios puede presentarse en tres fases clnicas: 1. Prdromos; 2-3 das, fiebre, estornudos,
rinorrea serosa, tos seca disfonica. 2. Estado; 4-6 das, fiebre, mal estado general y tos productiva. 3. Recuperacin; persistencia de la tos que se
vuelve paulatinamente seca y aislada, puede durar hasta 2 semanas. En caso de complicacin bacteriana, la enfermedad se prolonga y se agregan
mal estado general, tos seca molesta con dolor retroesternal y dificultad para respirar. Si el germen es Mycoplasma pneumoniae con frecuencia se
presenta neumona y complicaciones como derrame pleural, absceso pulmonar e insuficiencia respiratoria grave. Los exmenes de laboratorio solo
con fines de estudio o enseanza, o en caso de complicaciones: BH; leucocitosis leve con linfocitosis o neutrofilia. La radiografa de trax es normal o
bien muestra engrosamiento de la trama broncovascular y la imagen especfica en caso de complicacin. Se puede solicitar cultivo para virus a partir
de secreciones farngeas, tambin se pueden realizar pruebas serolgicas. Las pruebas de inmunofluorescencia en secreciones nasofarngeas son
altamente sensibles y son tiles en el diagnstico rpido. La PCR se ha convertido en el estndar de oro para diagnstico de infeccin pop B.
pertussis. TRATAMIENTO: Es sintomtico, medidas generales tendientes a mantener buena hidratacin, temperatura en lmites normales y si la tos
es frecuente y molesta, en primera instancia ambiente hmedo con nebulizador ultrasnico tibio con termostato. Si no se mejora la tos agregar
mucolticos como el ambroxol que se ha referenciado mejora la motilidad ciliar y facilita la recuperacin de la funcin de la mucosa as como la
eliminacin de las secreciones. En casos especficos y graves se recomienda el empleo de antivirales: por ejemplo amantadina en infecciones por
virus de la influenza o ribavirina si esta ocasionado por el virus sincitial respiratorio. Si se sospecha o confirma infeccin bacteriana
concomitantemente o complicando el cuadro inicial, debe emplearse el antibitico de eleccin para el germen de que se trate, si se sospecha tos
ferina (azitromicina), infeccin por Mycoplasma pneumoniae (un macrlido). Los broncodilatadores inhalados no estn indicados de manera
rutinaria, slo en aquellos casos en los que se asocie broncoespasmo. Sin embargo, hay nios con hiperreactividad bronquial que presentan un
mnimo e incluso no apreciable broncoespasmo a la exploracin, pero que responden de manera espectacular al broncodilatador inhalado, con
resolucin de la tos.
BRONQUITIS
Masculino de 3 aos de edad con sndrome de Down y cardiopata
congnita, es trado a urgencias por cuadro catarral de 7 das de
evolucin con tratamiento sintomtico el cual mejoro parcialmente
pero fue lentamente reagudizado, durante las dos semanas previas
cambio en dos ocaciones de tratamiento, existe el antecedente que
sus dos hermanos presentaban cuadro catarral. A las 12 h del
ingreso presenta empeoramiento progresivo de la dificultad
respiratoria con aparicin de fiebre, secreciones mucopurulentas y
tos, continua taquicardico y taquipnea.
PREGUNTA
Cual es el agente etiolgico mas probable de este caso?
RESPUESTA

CURSO ENARM CMN SIGLO XXI TEL: 36246001

a.- Influenza A, influenza B, parainfluenza.


b.- Virus sincitial respiratorio.
c.- Coronavirus, adenovirus y rinovirus.
d.- Bordetella pertusis, Mycoplasma pneumoniae y C pneumoniae.
PREGUNTA
Al solicitar una radiografia de torax a este paciente, Qu es mas
probable que observemos en la misma?
RESPUESTA
a.- Infiltrado reticulogranular
b.- Rectificacion de costillas y aplanamiento de hemidiafragamas
c.- Engrosamiento de trama broncovascular
d.- Infiltrados parahiliares

Pharmed Solutions Institute

PGINA 373

MANUAL DE TRABAJO DEL CURSO ENARM CMN SIGLO XXI


BRONQUIOLITIS
CIENCIAS BASICAS: Es una inflamacin de los bronquiolos, habitualmente de etiologa viral, aguda y frecuentemente contagiosa. Debido a la
inflamacin de los bronquiolos, se manifiesta por tos, sibilancias, disnea y taquipnea. Por lo general autolimitada. SALUD PUBLICA: Se presenta en
menores de 2 aos de edad, el 80% ocurre en menores de 1 ao (pico entre 6-8 meses). Predomina en el sexo masculino. Es ms frecuente en
invierno, pero puede presentarse a lo largo del ao. PATOGENIA: El germen patgeno ms frecuente encontrado en caso de bronquiolitis es el virus
sincitial respiratorio (VSR), hasta en 60% de los casos. El resto es ocasionado por parainfluenza, adenovirus, rinovirus y el virus de la influenza.
Tambin puede haber coexistencia entre VSR con Chlamydia trachomatis Y Mycoplasma pneumoniae, as como virus diferentes a los ya sealados,
los que en conjunto constituyen entre 5-10%. El VSR se introduce al organismo principalmente por contacto directo (se extienden hasta 2 m) con
secreciones nasales de enfermos; quien elimina virus hasta por 10dias despus del inicio del padecimiento. En gotas grandes el virus puede
sobrevivir y mantener contaminantes las superficies en las que se encuentre, su periodo de incubacin es de 4-6 das. Afecta el sistema respiratorio
por colonizacin y replicacin en la mucosa de los bronquios. En los nios pequeos los bronquiolos son el sitio primeramente afectado, mientras
que en nios mayores y adultos, se afectan las vas respiratorias superiores. Hay necrosis de las clulas ciliadas con proliferacin de clulas no
ciliadas como linfocitos, clulas plasmticas y macrfagos en las reas peribronquiales; se liberan mediadores inflamatorios, lo que ocasiona edema,
inflamacin y congestin de la submucosa con taponamiento de los bronquiolos por moco y detritus celulares; que en conjunto ocasionan
estrechamiento de las vas respiratorias perifricas. La respuesta pulmonar a estos cambios, es un aumento en la capacidad residual funcional,
aumento en la resistencia al paso de aire y aumento en los cortocircuitos pulmonares. Todos estos factores aumentan el trabajo respiratorio dando
lugar a alteraciones en el recambio de gases secundario a obstruccin y atelectasia de las vas areas. Tambin puede presentarse hipoxia y
retencin de CO2, secundarios a la alteracin de la relacin ventilacin/flujo, junto con hipovetilacin. DIAGNOSTICO: Es clnico, se presentan con
antecedente de 1-5 das con rinorrea profusa habitualmente hialina y congestin nasal, en algunos tos y fiebre leve a moderada. Al 4-5 da se agrega
dificultad para respirar con tos hmeda, taquicardia y sibilancias. El nio se muestra irritable y letrgico; en los muy pequeos se puede presentar
apnea, la enfermedad progresa con insuficiencia respiratoria grave (cianosis, actividad de msculos accesorios de la respiracin, disnea y taquipnea,
ms grave quejido, aleteo y tiro supraesternal) y ameritan intubacin. En cuadros severos y con varios das de evolucin puede haber
deshidratacin. Tambin hay signos de hiperinflacin (sobre distensin pulmonar), manifestados por aumento del dimetro anteroposterior del
trax y crecimiento de hgado y bazo que se palpan abajo del borde costal. A la auscultacin se pueden or sibilancias inspiratorias, disminucin de la
entrada de aire y estertores inspiratorios. Datos de gravedad por Shaw, para predecir evolucin y tratamiento: 1. Aspecto toxico. 2. SaO2 <95%. 3.
Edad gestacional menos de 34semanas. 4. Frecuencia respiratoria >70/min. 5. Atelectasia en la radiografa de trax. 6. Edad menor de 3 meses. Por
otro lado se puede hacer la determinacin de PaCO2, habitualmente se encuentra entre 30-35mm Hg. Valores entre 45-55mmHg son peligrosos e
indican falla respiratoria inminente. El criterio de oro para el diagnstico es el cultivo nasofarngeo positivo para VSR. Existen pruebas rpidas que
incluyen ELISA y tincin directa de anticuerpos fluorescentes, son ms rpidas y tienen especificidad y sensibilidad cercana al 90%. La serologa para
anticuerpos del virus tiene limitaciones. La determinacin de leucocitos y electrolitos solo debe hacerse si hay deshidratacin o se sospecha sepsis.
La gasometra se indica para valorar severidad de compromiso respiratorio. La radiografa de trax no es necesaria, puede ser til en los siguientes
casos: insuficiencia respiratoria leve a moderada, sibilancias recurrentes, aspecto toxico. La imagen puede mostrar horizontalizacin variable de las
costillas, con aumento del espacio intercostal, diafragmas abatidos, hiperinflacin (hiperclaridad), atelectasia, engrosamiento peribronquial e
infiltrados intersticiales difusos. Aprox 10% de las placas son normales y puede no haber correlacin con la severidad de la enfermedad.
TRATAMIENTO: Primero valoracin de la gravedad. Las medidas ms importantes son la hidratacin (si hay deshidratacin o hay bajo nivel de
eliminacin urinaria) y oxigenoterapia por puntas nasales, mascarilla o casco ceflico, debe medirse con frecuencia la saturacin de O2 con oximetro
de pulso para adecuar la cantidad de O2 a administrar. La aplicacin de nebulizacin de epinefrina racemica (mezcla de ismeros d y L) es aceptada,
mejora la saturacin de O2 y los datos clnicos de insuficiencia respiratoria, aunque en muchos pases no est disponible por lo que se usa la
adrenalina (L-epinefrina), con los mismos resultados, solo se usa en nios severamente enfermos y en quienes es inminente la intubacin traqueal.
La dosis recomendad es epinefrina racemica al 2.25% en 3 ml de sol. Salina normal durante 20 min. En nios con peso de <20Kg, 0.5ml si peso 2139Kg y 0.75ml si el peso es >40Kg (0.5ml de epinefrina racemica equivalen a 5ml de L-epinefrina). El empleo de esteroides y -agonistas en estos
pequeos no es aconsejable a luz de mltiples reportes. El nico antiviral acepado para este padecimiento es la ribavirina, el cual tienen actividad
virostatica, se recomienda en nios muy pequeos, inmunodeprimidos, con malformaciones cardiacas y los severamente enfermos, solo se emplea
en inhalacin y el personal de salud debe protegerse de aspirar el medicamento ya que se han descrito crisis de espasmo bronquial y
malformaciones congnitas. Hay reportes de empleo de factor surfactante, oxigenacin por membrana extracorprea (OMEC) y vitamina A.

CASO CLINICO
Se trata de masculino de 6 meses de edad con peso adecuado, con
antecentes, con diagnstico de reflujo gastroesofgico, en
tratamiento con Domperidona y Omeprazol. Vacunas completas.
Antecedentes familiares: Padre con antecedente de broncoespasmo
y atopa. Comenz 5 das previos al ingreso con tos y dificultad
respiratoria, por lo cual se evalu en la guardia de otro hospital,
indicndose tratamiento ambulatorio con broncodilatadores cada 4
hs. y Prednisona durante 5 das. El paciente evolucion sin mejora
clnica, intensificndose la dificultad respiratoria durante las ltimas
24 hs. y se agreg regular actitud alimentaria. No present fiebre.
Consult en la guardia, donde se lo encontr taquipneico, (fr) 70 por
minuto (pm), con (fc) de 145 pm, Sat 91% aire ambiente (aa), tiraje
subcostal e intercostal, regular entrada de aire bilateral y sibilancias
audibles.
PREGUNTA
Cual es la conducta a seguir inmediata a seguir es la menos
adecuada?
RESPUESTA
a.- Nebulizaciones con broncodilatadores

CURSO ENARM CMN SIGLO XXI TEL: 36246001

b.- Hidrocortisona a 10 mg/kg.


c.- Monitorizacion de saturacin perifrica.
d.- Evaluacin de gases arteriales.
PREGUNTA
Qu valor es mas probable que nos oriente a falla respiratoria
inminente en este paciente?
RESPUESTA
a.- PaCO2 25-30mmHg
b.- PaCO2 30-35mmHg
c.- PaCO2 35-40mmHg
d.- PaC02 45-50mmHg
PREGUNTA
Cul seria la conducnta teraputica mas adecuada a seguir
RESPUESTA
a.- Nemulizacion con broncodilatador
b.- Nebulizacion con adrenalina
c.- Nebulizacion con ribavirina
d.- Nebulizacion con esteroide

Pharmed Solutions Institute

PGINA 374

MANUAL DE TRABAJO DEL CURSO ENARM CMN SIGLO XXI


PREGUNTA
Cul es la conducta diagnostica mas adecuada?
RESPUESTA
a.- Radiografia de trax
b.- ELISA
c.- Biometria hematica
d.- Cultivo nasofaringeo
CASO CLINICO
Paciente masculino de 18 meses de edad, el cual es llevado por la
madre la cual refiere, que naci a las 35 SDG y que inicio hace 4 das
con rinorrea hialina, escurrimiento nasal, malestar e irritabilidad,
pero que no haba presentado fiebre, nicamente le dio loratadina
en gotas. Actualmente inicio con fiebre desde ayer, as como unos
silbidos que percibe cuando el nio respira, adems de tos. EF:
Palidez de tegumentos, activo y reactivo, rinorrea hialina, tos
productiva, dificultad respiratoria, con FR 45, FC 130x, T 38,2, a la
auscultacin de campos pulmonares estertores bronquiales leves,
as como sibilancias en la inspiracin, a la palpacin de abdomen con
dolor a la palpacin media, disminucin de movimientos
peristlticos, resto sin alteraciones.

presentar en las ltimas horas pausa de apnea, con cianosis


peribucal, de unos segundos de duracin, fiebre de hasta 38,3C
acompaado de tos, rinorrea serosa y estornudos. Madre con
catarro de vas altas en la actualidad, asmtica, no fumadora. Padre
fumador de un paquete de tabaco diario. Antecedentes personales;
Embarazo controlado de 40 semanas de gestacin, parto vaginal
eutocico, Apgar 9/10. Peso al nacimiento: 3120 g. Periodo neonatal
sin incidencias. Alimentacin: lactancia materna. EF: Peso: 4.980 g.
Talla: 58 cm. PC: 39 cm. Buen estado general. Bien hidratado y
perfundido. Tiraje subcostal, intercostal y supraclavicular; no aleteo
nasal. Auscultacin cardiaca: tonos puros y rtmicos a 155 lpm.
Auscultacin respiratoria: 60 rpm; disminucin del murmullo
vesicular en ambos campos pulmonares, con sibilantes espiratorios y
subcrepitantes predominantemente en bases; saturacin de oxigeno
del 96%. Abdomen: blando, depresible, no masas ni megalias. ORL:
sin hallazgos. Valoracin neurolgica normal. BH: leucocitos 11.300
(L: 61%; N: 26%; M:5%); Hb 10.5 g/dl; plaquetas 500.000 mm3. QS:
valores dentro de la normalidad, incluyendo PCR y procalcitonina.
Sedimento orina: negativo. PCR para B. pertussis: negativo. Virus
respiratorios en lavado nasofaringeo: positivo para virus respiratorio
sincitial.

PREGUNTA
Cul es el agente etiolgico ms probable en este caso?
RESPUESTA
a.- Streptococo pneumoniae
b.- Parainfluenza
c.- Haemophilus influenzae
d.- Virus sincitial respiratorio

PREGUNTA
Cul es su sospecha diagnostica ms probable?
RESPUESTA
a.- Bronquiolitis
b.- Neumona
c.- Insuficiencia cardiaca
d.- Bronconeumonia

PREGUNTA
Cul es la conducta teraputica ms adecuada a seguir en este
momento?
RESPUESTA
a.- Domicilio, paracetamol, lquidos, cita en 24 hrs
b.- Hospitalizacin, paracetamol, radiografa de trax, vigilancia
72hrs
c.- Domicilio, paracetamol, amoxicilina/ac. Clavlanico, cita en 48 hrs
d.- Hospializacin, paracetamol, claritromicina, vigilancia 48hrs

PREGUNTA
Se le toma una radiografa de trax al paciente. Qu es ms
probable observar en este caso?
RESPUESTA
a.- Atelectasias
b.- Imagen en punta de lpiz y datos de atrapamiento de aire
c.- Infiltrado miliar
d.- Infiltrado micronodular parahiliar y datos de atrapamiento de
aire

PREGUNTA
Tres das despus la madre acude al servicio de urgencias, refiere
que su nio, presento fiebre de 39, que se le marcan sus costillas al
respirar, y que respira muy rpido y pareciera que se ahogara, y no
quiere comer. EF: irritable, con diaforesis, taquipnea, as como
estertores y sibilancias moderados. Qu complicacin es ms
probable que este presentando el paciente en este momento?
RESPUESTA
a.- Empiema
b.- Sobreinfeccin bacteriana
c.- Derrame pleural
d.- Necrosis pulmonar

PREGUNTA
3hrs despus, al hacer valoracin con las escala de Wood- Downes,
tiene 8 puntos. Cul es la conducta a seguir en este momento?
RESPUESTA
a.- Hospitalizacin, oxigeno mas 2 agonista inhalado, ms una dosis
de adrenalina inhalada
b.- Recomendaciones y manejo en domicilio
c.- Oxigeno, dosis de 2 agonista y domicilio
d.- Hospitalizacin, oxigeno ms 1 dosis de adrenalina inhalada

PREGUNTA
Cul es la conducta teraputica ms adecuada a seguir en este
momento?
RESPUESTA
a.- Hospitalizacin, oxigeno, ibuprofeno, penicilina ms azitromicina
b.- Hospitalizacin, oxigeno, ceftriaxona
c.- Hospitalizacin, ibuprofeno, vancomicina
d.- Hospitalizacin, drenaje, vancomicina
CASO CLINICO
Lactante de dos meses y medio de vida con febrcula en das previos
y rechazo de las tomas, que acude al Servicio de Urgencias por

PREGUNTA
La evolucin es favorable, desapareciendo las pausas de apnea y
disminuyendo los signos de dificultad respiratoria, lo que permite
iniciar alimentacin oral a las 24 horas de su ingreso, que es bien
tolerada. El paciente puede ser dado de alta a domicilio a los 7 das
de su ingreso. El paciente tiene antecedente de cardiopata
congnita. Cul es la medida preventiva mas til, para disminuir los
cuadros de bronquiolitis?
RESPUESTA
a.- Salbutamol
b.- Palivizumab
c.- Metilprednisolona
d.- Rivabirina

BRONCONEUMONIA

CURSO ENARM CMN SIGLO XXI TEL: 36246001

Pharmed Solutions Institute

PGINA 375

MANUAL DE TRABAJO DEL CURSO ENARM CMN SIGLO XXI


CIENCIAS BASICAS: El trmino es introducido en 1837 por Seiffert, a travs de la idea de infiltrados pulmonares de origen broncognico o
bronquiolognico. Se ha relacionado con neumona multifocal (o lobulillar), ahora frecuentemente llamada de focos mltiples. Infeccin de la va
respiratoria producida por un microorganismo (bacteria o virus). Es una infeccin de inicio violento y repentino que produce inflamacin en
pulmones y bronquios, la cual genera trastornos respiratorios que si no son tratados oportunamente pueden conducir a la muerte. Es ocasionada
por virus o bacterias y afecta particularmente a nios, personas de edad avanzada y a pacientes con dao en su sistema inmunolgico
(inmunosuprimidos). SALUD PUBLICA: Los bebs que tienen entre 3-6 meses de edad son quienes resultan ms comnmente afectados, si bien esta
enfermedad puede aparecer hasta los dos aos. Se calcula que aproximadamente uno de cada tres bebs tendr bronconeumona en algn
momento durante su primer ao de vida. Es ms frecuente durante los meses de invierno. PATOGENIA: La bronconeumona es una lesin
secundaria que aparece generalmente como complicacin de una enfermedad (gripe, resfriados mal cuidados, bronquitis, bronquiolitis o tos ferina).
A diferencia de la neumona, no posee fases evolutivas y el exudado no contiene fibrina o tiene muy poca. La bronconeumona es causada por uno
de varios tipos de virus. En tres cuartos de los casos, es provocada por el virus sincitial respiratorio, por contacto directo con la tos de un enfermo.
Tumores pulmonares que evitan la ventilacin adecuada del aire a los pulmones, los cuales favorecen el establecimiento de bacterias. La
caracterstica dominante de la bronconeumona es la consolidacin parcheada del pulmn. DIAGNOSTICO: Clnica; Cuadro febril sin otro sntoma, o
bien como dolor de trax, o abdomen. Tos, decaimiento, inapetencia o rechazo al alimento, cuadro de resfro que empeora progresivamente,
insuficiencia respiratoria severa, taquipnea, hundimiento de costillas al respirar, aleteo nasal, quejido, retraccin xifoidea. Anatoma patolgica: Las
lesiones bronconeumnicas consisten en focos de condensacin pequeos, a veces, confluentes, gris rojizos, secos, finamente granulosos.
Frecuentemente, sin embargo, son poco notorios macroscpicamente y se manifiestan como zonas ligeramente levantadas, hipermicas, que se
descubren mejor por palpacin que por inspeccin. Estos focos pueden pasar inadvertidos macroscpicamente, no as en el examen del pulmn
fijado previamente. Los focos bronconeumnicos se encuentran frecuentemente en las regiones dorso-basales y laterales de los lbulos inferiores.
En el centro del pulmn los focos son mayores que en la periferia, donde tienden a ser ms densos. A menudo alcanzan la pleura, donde se
desarrolla entonces una pleuritis fibrinosa o purulenta. BRONCONEUMONA ESTAFILOCCICA: Se observa en el 31% de las autopsias. Corresponde
al 5% de las neumonas bacterianas. Mortalidad cercana al 20%. El tipo de reaccin inflamatoria es la inflamacin necrotizante y abscedante. Es una
afeccin secundaria a piodermitis, furunculosis, endocarditis, osteomielitis y otras. Los abscesos se forman a partir de embolias spticas en arteriolas
y capilares. El empiema y el pioneumtorax son acompaantes frecuentes. La neumona aergena, primaria o secundaria a bronquitis viral, se
observa como una neumona con infiltrados purulentos mal delimitados, confluentes con tendencia a la necrosis. El pronstico de esta forma es muy
malo, especialmente la forma primaria en lactantes. BRONCONEUMONA ESTREPTOCCICA: Se caracteriza por un exudado hemorrgico y
flegmonoso, pobre en fibrina. En la forma hematgena, se constituyen infiltrados maculares (bronconeumnicos) y simultneamente se desarrollan
flegmones pleurales, septales y perilobulillares. La forma aergena es ms frecuente y corresponde en verdad a una bronconeumona purulenta
confluente. La pleuritis purulenta y el empiema son tambin frecuentes. BRONCONEUMONIA POR ASPIRACION: Tambin predominan en lbulos
inferiores y ms frecuentemente al lado derecho. Los cuerpos extraos pueden producir una estasis de secreciones bronquiales, heridas de la pared
con necrosis y neumona purulenta consecutiva, a menudo gangrenosa. La circunstancia ms frecuente es la aspiracin de vmitos con contenido
gstrico, que produce una inflamacin necrotizante, bronquial y alveolar, por la accin corrosiva del cido. En la agona se puede producir aspiracin
de contenido gstrico y digestin pulmonar post-mortem (neumomalacia cida). Otra situacin frecuente es la aspiracin de lquido amnitico. El
feto respira in tero e inhala lquido amnitico normalmente. En la asfixia neonatal aparece meconio en el lquido amnitico y en el pulmn se
observan partculas de vrnix caseoso o de meconio y escamas crneas. Como es un fenmeno normal, la distincin suele ser difcil y, al final, la
diferencia es slo cuantitativa. Cuando la aspiracin es masiva, puede observarse una bronconeumona leucocitaria reactiva.
CASO CLINICO
Paciente de 17 dias de vida extrauterina que presenta fiebre,
irrabilidad, y ampollas en el cuerpo. EF febril, hidratado, lloroso e
intranquilo, ligeramente taquipneico con exantema de distribucin
centrpeta en estadios evolutivos de vesculas y pustulas distribuido
en cara, cuello y tronco. Orofaringe congestiva y disminucin de
murmullo vesicular bilateral en campos pulmonares. Rx de torax:
presencia de infiltrado bronconeumonico bilateral a predominio de
hilios y campos inferiores. Laboratorios leucos 11 mil, acidosis
metabolica compensada. VSG 1ah: 14 mm y PCR 0.03 mg/l.
PREGUNTA
Cual es la conducta teraputica mas adecuada a seguir en el caso?
RESPUESTA
a.- Oxigeno, liquidos y aines.
b.- Oxigeno, liquidos y electrolitos, vigilancia.
c.- Oxigeno, antibiticos, liquidos y aines.
d.- Oxigeno, antibiticos, liquidos y corticoides
PREGUNTA
Mientras examina el medico al paciente nota decremento del
fremito tctil, matidez a la percusin y disminucin de los ruidos
respiratorios, en el lbulo inferior derecho, sin desviacin de la
traquea, Cul es el dato mas representativo de este trastorno?
RESPUESTA
a.- Obstruccion bronquial
b.- Neumonia lobar
c.- Derrame pleural
d.- Neumotrax

CURSO ENARM CMN SIGLO XXI TEL: 36246001

CASO CLINICO
Nia de un ao 3 meses transferida a un hospital por distress
respiratorio y componente obstructivo bronquial de difcil manejo
con dficit ponderal. Durante su hospitalizacin se objetivan
episodios de atragantamiento con la alimentacin a predominio de
lquidos as como exacerbacin de la obstruccin bronquial que
mejora parcialmente con el tratamiento broncodilatador enrgico.
As mismo, la paciente tena el antecedente de haber sido operada
de atresia esofgica y fstula traqueoesofgica al nacer, historia de
tos asociado a la ingesta de leche y cuadros recurrentes de
obstruccin bronquial siendo diagnosticado y tratado como asma del
lactante. Examen fsico al ingreso: Peso: 8.3 kg., FC: 140 x min. FR: 66
x min. Aparato Respiratorio: Tiraje subcostal e intercostal, aleteo
nasal, sibilantes inspiratorios y espiratorios en ambos campos
pulmonares. Exmenes auxiliares: Hemograma: 24800 leucocitos,
Segmentados: 88%; Linfocitos: 9%, Monocitos: 3% Hb: 7.9 gr%, PCR:
64 mg/lt., plaquetas: 434,000. En la radiografa de trax se
evidenciaba un infiltrado difuso bilateral (alveolar e intersticial) as
como una atelectasia basal posterior izquierda y un enfisema
compensador en el lbulo superior del mismo lado.
PREGUNTA
Cul es la cuasa ms probable del cuadro de esta paciente?
a.- Streptocco pneumonia
b.- Aspiracion pulmonar cronica
c.- Virus sicitial respiratorio
d.- Malformacion congenita

Pharmed Solutions Institute

PGINA 376

MANUAL DE TRABAJO DEL CURSO ENARM CMN SIGLO XXI


NEUMONIAS
CIENCIAS BASICAS: Se define como la inflamacin y condensacin del parnquima pulmonar causada por un agente infeccioso o por factores no
infecciosos como la aspiracin de cido gstrico, cuerpos extraos e hidrocarburos; as como las reacciones de hipersensibilidad y las neumonitis
inducidas por frmacos o radiaciones. El la neumona existe reemplazo del contenido areo de los alveolos y conductos alveolares por clulas y
exudado inflamatorio, que se manifiesta por la presencia de sntomas y signos de infeccin aguda y la presencia de imgenes radiolgicas que
indican ocupacin alveolar. SALUD PUBLICA: En 2002 en Mxico se reportaron 189 806 casos de neumona. La mayora de los casos se reportaron en
nios de 1-5 aos. Su incidencia en este grupo de edad es muy elevada, y se reportan 10-40 casos: 1000 nios/ao. PATOGENIA: Durante los 3
primeros aos de vida las neumonas estn causadas por virus, especialmente por el sinsitial respiratorio (VSR) y los virus de la gripe, tambin virus
influenza A y parainfluenza tipo 1. A partir de los 3 aos pueden aislarse Mycoplasma pneumoniae y Chlamydia pneumoniae. Aunque los agentes
bacterianos se asocian menos frecuentemente con neumonas, se ha evidenciado que causan infecciones ms graves. Patgenos comunes:
Streptococo pneumoniae, Mycoplasma pneumoniae, Chlamydia pneumoniae, Coxiella burnetii, virus respiratoria. Patgenos no comunes:
Legionella pneumophila, Haemophilus influenzae, enterobacterias, flora saprofita bucal, Moraxalla catarrhalis. El evento que con mayor frecuencia
altera los mecanismos de defensa de la va area es una infeccin viral que altera las propiedades de las secreciones, inhibe la fagocitosis, modifica la
flora bacteriana y puede temporalmente romper de manera temporal al epitelio de la va area. Es comn que se presente una infeccin viral unos
das previos al desarrollo de neumona bacteriana; sin embargo, su presencia no es indispensable para que las bacterias induzcan enfermedad. Los
microorganismos que causan neumona se adquieren en la inmensa mayora de los casos, por va respiratoria y alcanzan las vas respiratorias bajas
por trayecto descendente desde las vas respiratorias altas. De manera inicial, el edema reactivo favorece la proliferacin de los microrganismos y
colabora en su diseminacin a porciones adyacentes al pulmn. Uno ms lbulos generalmente se involucran, en nios pequeos predomina un
patrn ms difuso que sigue la distribucin bronquial y se caracteriza por muchas reas de consolidacin alrededor de las vas respiratorias
pequeas, el dao permanente es raro. Los pacientes con inmunodeficiencias primarias o secundarias tienen un riesgo mayor de infeccin,
recurrencias y complicaciones. DIAGNOSTICO: Clnica; las manifestaciones son secundarias a la respuesta inflamatoria sistmica y local a la
infeccin; por consiguiente, son de dos tipos; generales como fiebre, malestar general escalofros y cefalea y respiratorios como tos, disnea,
taquipnea y anomalas en la auscultacin torcica. La intensidad de los sntomas as como la gravedad de la enfermedad varan segn el agente
etiolgico, ya que mientras unos dan lugar a una consolidacin pulmonar localizada, otros provocan una inflamacin ms difusa. Tambin depende
de la edad del paciente, especialmente en nios pequeos. La radiografa de trax es til para corroborar el diagnstico y descartar complicaciones.
En aproximadamente el 20% de las neumonas hay derrame pleural, que en una minora de nios evolucionara a empiema. Las neumonas atpicas
suelen ocasionar un infiltrado heterogneo y poco denso, con aspecto de vidrio despulido, que tiende a estar situado cerca del hilio, sobre todo en
los lbulos inferiores; a menudo las imgenes de ocupacin alveolar afectan varios lbulos, en ambos pulmones; as el patrn radiolgico mas
frecuente es el de un infiltrado parahiliar peribronquial unilateral o bilateral. Aunque se ve pocas veces es muy caracterstico de infeccin por
M.pneumoniae la presencia de unas imgenes reticulonodulillares localizadas en un solo lbulo inferior. Los hemocultivos tienen pobre sensibilidad
en las neumonas (10-30%). La deteccin de antgenos bacterianos en sangre y orina tienen utilidad diagnostica variada. Los estudios serolgicos,
tiles en epidemiologia, tienen escasa utilidad clnica. Las pruebas rpidas de deteccin de antgenos bacterianos en secreciones nasofarngeas
mediante inmunofluorescencia directa o ELISA resultan muy tiles para la identificacin de virus respiratorios, pero tienen disponibilidad limitada y
altos costos. INFECCIONES VIRALES; tpicamente rinorrea, estornudos y tos. Temperatura ligeramente elevada, podemos encontrar taquipnea, tiros
intercostales, aleteo nasal y el uso de msculos accesorios, las infecciones graves se acompaan de cianosis y dificultad respiratoria de moderada a
grave especialmente en lactantes. A la auscultacin de trax se puede evidenciar estertores y sibilancias. Las neumonas vricas tambin tienden a
presentar un patrn de infiltrado parahiliar peribronquial, ms o menos difuso a veces acompaado de atelectasias; puede haber imgenes
micronodulares difusas y sobredistensin pulmonar, tambin son posibles otras imgenes, como el aumento de densidad localizada, segmentario o
lobular. Es difcil identificar el agente etiolgico. INFECCION POR STREPTOCOCO PNEUMONIAE: Historia clsica de escalofros de inicio sbito,
seguidos por fiebre alta, tos y dolor de trax. Los lactantes presentan varios das de congestin nasal, inquietud e hiporexia de manera inicial. De
manera sbita presentan fiebre, astenia y aprensin, el paciente se ve enfermo, a la auscultacin de trax puede proporcionar pocos signos, pero
puede evidenciarse un rea de consolidacin por percusin, as como incremento de las vibraciones torcicas y estertores. Los nios ms grandes
tienen escalofros y fiebre despus de una infeccin respiratoria alta. Tambin tienen ataque al estado general, taquipnea, tos seca y en casos graves
alteraciones de la conciencia, a la auscultacin, buscar lo mismo que en los lactantes y adems buscar derrame por percusin. NEUMONIA POR
NEUMOCOCO: La BH, muestra formula blanca incrementada (15,000-40,000/mm3), con predominio de PMN. La gasometra muestra hipoxia con
hipercapnia. El neumococo puede ser aislado de las secreciones nasofarngeas en la mayora de los pacientes, sin embargo, debe tenerse en cuanta
que 10-15% de la poblacin sana es portadora. El aislamiento de la bacteria en sangre o en lquido pleural es diagnstico. En las radiografas se
evidencia consolidacin aun antes de la deteccin de la neumona. INFECCION POR H. INFLUENZAE TIPO B: Inicio ms insidiosos que aquellos que
estn infectados por neumococo y el curso de la enfermedad se prolonga por varias semanas, a la exploracin: taquipnea, dificultad para respirar,
matidez a la o percusin, estertores alveolares y bronquiales. Los nios pequeos pueden presentar derrame pleural. El diagnostico se establece por
la historia clnica, los sntomas y los hallazgos de la exploracin. Pueden encontrarse imgenes segmentarias, compromiso lobular simple o mltiple,
derrame pleural o neumatoceles. Diagnstico de certeza se establece con aislamiento del organismo de la sangre, lquido pleural o aspirado
pulmonar. COMPLICACIONES: Empiema y bacteriemia para la mayora de los agentes etiolgicos, artritis sptica en casos de infeccin por Hib y
Streptococo del grupo A, pericarditis, meningitis, osteomielitis, abscesos de tejidos blandos, pioneumotrax y neumatoceles en las infecciones por S.
aureus y celulitis, meningitis y artritis supurada en los casos de infeccin por Hib. TRATAMIENTO ESPECIFICO: Usualmente en caso de neumonas
por virus se requieren de medidas de soporte mnimas como la administracin de lquidos IV, oxgeno y en los casos graves ventilacin mecnica, los
nicos agentes especficos de que se dispone son la amantadina y la ribavirina en aerosol, las cuales son eficaces contra influenza A y ribavirina para
Virus sisitial respiratorio, se recomienda solo en nios con enfermedad grave, que tienen cardiopata congnita, displasia broncopulmonar,
enfermedad pulmonar crnica o que reciben tratamiento inmunosupresor. La neumonas causadas por neumococo son tratables con penicilina IV, el
frmaco de eleccin es la penicilina G (100000U/Kg/24 hrs). Debe considerarse el empleo de una cefalosporina de tercera generacin (cefotaxima
150mg/kg/24h, o ceftriaxona 75 mg/kg/24h), en el caso de que el S. peneumoniae sea resistente a la penicilina pero sensible a cefalosporinas. La
vancomicina (40mg/kg/24h) se debera emplear si el aislamiento es resistente a las anteriores. Aproximadamente el 20-30% de los neumococos son
resistentes a los macrlidos, En los casos de neumona por Streptococo del grupo A deben ser tratados con penicilina G (100 000 U/kg/24h). Se
emplea penicilina parenteral de manera inicial y debe completarse un tratamiento de hasta 2-3 semanas con penicilina por la via bucal. Si se
presenta empiema debe realizarse una toracocentesis con propsitos diagnsticos y para evacuar lquido. Para Staphylococus aureus dicloxacilina
100-200mg/kg/24h). En los casos de derrame pleural aunque sea mnimo o de pioneumotrax, se recomienda la instalacin de un drenaje tracico.
Para H. influenzae de tipo b debe incluir ceftriaxona (75mg/kg/24h) o cefotaxima 150mg/kg/24h). Los derrames y la pioartrosis requieren de
drenaje. TRATAMIENTO EMPIRICO: Se hace en funcin de la sospecha del agente etiolgico. En nios de 2-3 aos con cuadro clnico y radiolgico de

CURSO ENARM CMN SIGLO XXI TEL: 36246001

Pharmed Solutions Institute

PGINA 377

MANUAL DE TRABAJO DEL CURSO ENARM CMN SIGLO XXI


neumona y con buen estado general se puede tratar por VO con dosis altas de amoxicilina (80-90mg/kg/da) cada 8 h, asociada o no a ac.
Clavulanico. Si el estado general est afectado o vomita es preferible ingresarlo y tratarlo por va IV con cefotaxima (100-150mg/kg/da), ceftriaxona
(50-100mg/kg/da) o amoxicilina/ ac. Clavulanico (100mg kg/da), durante mnimo 48-72 hrs si hay buena evolucin seguir con amoxicilina oral por 710 das. En latantes menores de 3 meses la mejor opcin teraputica es la cefotaxima sola o asociada a ampicilina si se considera que listeria es una
posible causa, siempre por via IV. En el nio mayor de 3 aos con una neumona tpica, si ingresa por afectacin del estado general, compromiso
repiratorio o intolerancia digestiva, se tratara del mismo modo que si tuviera menos de 3 aos, pero con un especial consideracin al empleo de
penicilina G (100 000mg/kg/24h) IV si el cuadro clnico no es grave. Los macrlidos son el tratamiento de eleccin en los nios con neumonia por
Mycoplasma pneumoniae y Chlamydia pneumoniae, VO 7-10 dias o 3 dias si se utiliza azitromicina.
CASO CLINICO NEUMONIAS
Un nio de 5 aos de edad se present con fiebre y tos no
productiva durante 3 das. El examen fsico revel mltiples ndulos
linfticos cervicales. Fue tratado en su centro de salud durante 7 dias
con medidas generales, regresando a su escuela sin embargo a los 5
dias despus inicia con tos productiva, dificultad respiratoria,
cansancio, malestar generalizado, aleteo nasal y fiebre de 39.2
grados. Examen torcico se apresiaron estertores bronquiales y
broncoalveolares de predominio en la zona inferior. La saturacin de
oxgeno de 92% en aire ambiente. La radiografa de trax (RXT) al
ingreso mostr consolidacin del lbulo inferior derecho. Leucositos
de 13,500. Refiere el familiar que el esquema de vacunacin se
encuentra al corriente. Vive en zona rural y es el segundo hijo de 5.
PREGUNTA
Cual de los siguientes estudios solicita para tomar una decisin
diagnostica?
RESPUESTA
a.- Proteina C reactiva.
b.- Biometria hemtica.
c.- Cultivo de secresiones.
d.- Hemocultivo.
PREGUNTA
Cual es el criterio para tomar la radiografia de torax como factor
desicivo para tratamiento?
RESPUESTA
a.- Paciente ambulatorio.
b.- Sospecha de neumona.
c.- Fiebre alta con sitomas respiratorios.
d.- Presencia de estertores sin taquipnea.
PREGUNTA
Considerando el cuadro clnico cual es el agente causal ms
probable de este caso?
RESPUESTA
a.- Estreptococcus pneumoniae.
b.- Mycoplasma pneumoniae.
c.- Haemophilus influenza.
d.- Clamydia pneumoniae.
PREGUNTA
Cual es la conducta terapeutica mas apropiada?
RESPUESTA
a.- Amoxicilina 80-90 mg/kg/dia.
b.- Azitromicina 10 mg/kg dia.
c.- Eritromicina 50mg/Kg/dia.
d.- Penicilina procainica 400,000 UI cada 24 hrs IM.
PREGUNTA
Cual de los siguientes criterios es mas importante para el envio a
segundo nivel?
RESPUESTA
a.- Presencia de apneas.
b.- Dificultad respiratoria moderada.
c.- Falta de seguridad en casa.
d.- Deshidratacion.

Nio de 7 aos con fiebre y tos recurrentes durante 7 meses, sin


clnica de broncoespasmo, pero antecedente de infecciones de vas
respiratorias repetidas. Ha recibido tratamiento con varias pautas
antibiticas, con mejora parcial en algunas ocasiones. Durante los 7
meses, persiste hipoventilacin en base derecha y radiolgicamente,
infiltrado en lbulo medio derecho. El estudio inicial con pruebas de
funcin respiratoria, ionotest, Mantoux, inmunoglobulinas, cultivo
de esputo y estudio de reflujo gastroesofgico fue normal. A pesar
de las negativas iniciales, se admiti un episodio de atragantamiento
2 meses antes de iniciar el cuadro.
PREGUNTA
Cul sera la conducta diagnostica ms adecuada a seguir en este
momento?
RESPUESTA
a.- Tomografa de trax
b.- Resonancia magntica
c.- Espirometra
d.- Fibrobroncoscopa
PREGUNTA
Cul es el diagnstico ms probable en este caso?
RESPESTA
a.- Bronconeumonia
b.- Neumona
c.- Asma bronquial
d.- Neoplasia pulmonar
PREGUNTA
Cul es la posible causa en este caso?
RESPUESTA
a.- Sndrome de aspiracin asociado a descoordinacin deglutoria
b.- Defectos en la funcin inmunitaria
c.- Antecedente de asma bronquial
d.- Infecciones de vas respiratorias repetidas
CASO CLINICO
Paciente masculino de 6 aos de edad, que es trado por la madre al
servicio de urgencias tiene antecedente de asma sin control, nos
comenta que hace 3 semanas varios compaeros de su saln haban
tenido tos y catarro, lo refiere con tos la cual se incrementa con el
ejercicio, con historia de: 10 das con artralgia de codo derecho, 7
das tos productiva frecuente. 3 das iniciales: fiebre moderada y
cefalea EF: Nio orientado, consciente, con palidez de tegumentos +,
desnutricin leve, aumento de volumen doloroso en codo derecho,
sibilancias y estertores escasos, condensacin base derecha. Rx de
Trax: Condensacin pulmonar basal derecha e infiltrado intersticial
difuso. Hemograma: leucocitosis con neutrofilia, PCR 30.
PREGUNTA
Cul es el agente etiolgico ms probable en este caso?
RESPUESTA
a.- Streptococo pneumoniae
b.- Staphylococcus dorado
c.- Virus sincitial respiratorio
d.- Mycoplasma pneumoniae
PREGUNTA

CASO CLINICO

CURSO ENARM CMN SIGLO XXI TEL: 36246001

Pharmed Solutions Institute

PGINA 378

MANUAL DE TRABAJO DEL CURSO ENARM CMN SIGLO XXI


Cul es la conducta teraputica ms adecuada a seguir en este
caso?
RESPESTA
a.- Claritromicina 15mg/kg/da
b.- Amoxicilina/ac. Clavulanico 80mg/kg/da
c.- Cefuroxima 30mg/kg/da
d.- Azitromicina 12mg/kg/da
PREGUNTA
Cul es el estudio de eleccin para establecer el diagnstico
etiolgico?
RESPUESTA
a.- Hemocultivo
b.- PCR
c.- Cultivo de lquido pleural
d.- IgM especifica
CASO CLINICO
Masculino de 6 aos, con antecedente de atopia, es llevado por la
madre al servicio de urgencia por presentar fiebre de 39 durante 48
horas, persistente; asociado a dolor en miembro inferior izquierdo,
habones urticarianos y distensin abdominal. La madre refiere que
su hijo presenta un soplo en el corazn de nacimiento. EF: buena
coloracin de piel y tegumentos, se aprecia epicanto palpebral,
orejas displasicas, as como pliegue nico interfalangico. Aparato
respiratorio: Aleteo nasal, tiraje intercostal moderado, quejido
espiratorio, FR: 50 x, estertores leves. FC: 150 x. TA: 110/70 mmHg.
Pulsos pedos dbiles, llene capilar <2 segs, no cianosis distal.
PREGUNTA
Cul es el agente etiolgico ms probable en este caso?
RESPUESTA
a.- Staphylococus aureus
b.- Streptococo pneumoniae
c.- Clamydia pneumoniae
d.- Haemophilus influenzae

respiratorias repetidas. Ha recibido tratamiento con varias pautas


antibiticas, con mejora parcial en algunas ocasiones. Durante los 7
meses, persiste hipoventilacin en base derecha y radiolgicamente,
infiltrado en lbulo medio derecho. El estudio inicial con pruebas de
funcin respiratoria, ionotest, Mantoux, inmunoglobulinas, cultivo
de esputo y estudio de reflujo gastroesofgico fue normal. A pesar
de las negativas iniciales, se admiti un episodio de atragantamiento
2 meses antes de iniciar el cuadro.
PREGUNTA
Cul sera la conducta diagnostica ms adecuada a seguir en este
momento?
RESPUESTA
a.- Tomografa de trax
b.- Resonancia magntica
c.- Espirometra
d.- Fibrobroncoscopa
PREGUNTA
Cul es el diagnstico ms probable en este caso?
RESPESTA
a.- Bronconeumonia
b.- Neumona
c.- Asma bronquial
d.- Neoplasia pulmonar
PREGUNTA
Cul es la posible causa en este caso?
RESPUESTA
a.- Sndrome de aspiracin asociado a descoordinacin deglutoria
b.- Defectos en la funcin inmunitaria
c.- Antecedente de asma bronquial
d.- Infecciones de vas respiratorias repetidas
CASO CLINICO
Aparecen varios casos de neumona grave en una comunidad. Los
pacientes se presentan con fiebre alta; los especmenes de esputo
muestran muy pocos bastones gramnegativos pequeos en la
coloracin de Gram. El brote se rastrea hasta un vaporizador usado
en la seccin de frutas y verduras de un supermercado.

PREGUNTA
Cul es el patrn radiolgico ms probable en este caso?
RESPUESTA
a.- Consolidacin
b.- Focos mltiples
c.- Alveolo-intersticial
d.- Derrame pleural

PREGUNTA
Cul de los siguientes es el microorganismo causal ms probable de
este brote?
RESPUESTA
a.- Bordetella pertussis
b.- Haemophilus influenzae
c.- Legionella pneumophila
d.- Mycobacterium tuberculosis

PREGUNTA
Cul es la complicacin ms probable en este caso?
RESPUESTA
a.- Neumatocele
b.- Neumotrax
c.- Absceso pulmonar
d.- Empiema
PREGUNTA
A pesar de tratamiento el paciente presento rpidamente deterioro
e insuficiencia respiratoria que requiri ventilacin mecnica
presento bacteriemia, empiema. Cul sera el tratamiento ms
adecuado a seguir en este momento?
RESPUESTA
a.- Cefotaxima ms oxacilina
b.- Penicilina ms azitromicina
c.- Vancomicina
d.- Ceftriaxona mas sulbactam

PREGUNTA
Cul es el mecanismo mediante el cual el cuerpo ejerce control
sobre la respiracin?
RESPUESTA
a.- Los cuerpos artico y carotideo se consideran quimiorreceptores
centrales
b.- Los quimiorreceptores cenrales afectan la respiracin al detectar
directamente las concentraciones sanguneas de H
c.- Los quimiorreceptores perifricos estimulan la respiracin a
presin parcial de oxigeno de menos de 60mmHg
d.- Los receptores de estiramiento, irritacin, funcionan fuera de los
pulmones para regular la respiracin.

CASO CLINICO
Nio de 7 aos con fiebre y tos recurrentes durante 7 meses, sin
clnica de broncoespasmo, pero antecedente de infecciones de vas

CURSO ENARM CMN SIGLO XXI TEL: 36246001

Pharmed Solutions Institute

PGINA 379

MANUAL DE TRABAJO DEL CURSO ENARM CMN SIGLO XXI


RINITIS ALERGICA (RA)
CIENCIAS BASICAS: Es una enfermedad crnica definida como una hipersensibilidad de la mucosa nasal a sustancias extraas mediada por IgE. Los
sujetos que presentan RA requieren de exponerse a bajas dosis de alrgenos durante varios aos para desarrollar los sntomas. SALUD PUBLICA:
Tienen una prevalencia entre 15-20%. La rinitis estacional raramente se presenta en nios menores de 5 aos y la mayora de los pacientes con la
enfermedad presentan los sntomas antes de los 20 aos. En la Ciudad de Mxico, 14% de los nios cursan con rinitis alrgica. Hasta 78% de los
pacientes con asma tienen rinitis alrgica. PATOGENIA: Factores de riesgo; ablactacin temprana, deprivacin de alimentacin al seno materno,
contaminacin ambiental, tabaquismo materno durante el 1 ao de vida, antecedente de alergia en padres, historia familiar de rinitis alrgica,
higiene excesiva y niveles elevados de IgE a los 9 meses y a los 6 aos, pacientes asmticos.
Existen
aeroalergenos
ms
frecuentemente responsables de
ocasionar rinitis intermitente (plenes
de rboles, gramneas y malezas,
como as tambin esporas de hongos
del exterior), pero la persistencia de
estos antgenos vara segn las
estaciones del ao y la regin
geogrfica analizada, pudiendo un
alrgeno estacional generar una rinitis
persistente. Los principales antgenos
involucrados en la rinitis persistente
son los caros del polvo de habitacin.
La liberacin de mediadores qumicos
(histamina, leuicotrienos entre otros),
originados en la activacin del mastocito, sensibilizado con IgE especifica de alrgeno, es la responsable de los sntomas del paciente. La enfermedad
alrgica presenta dos etapas: a) sensibilizacin (predisposicin gentica), y b) la presencia de sntomas (interaccin husped-medio ambiente). En la
primera etapa, debe existir un husped predispuesto a inducir una respuesta inmunitaria a los alrgenos (Th2), susceptibilidad dependiente de que
se genere un arreglo gentico para producir IgE, capaz de sensibilizar a la clula mastocitaria. En la segunda etapa, el contacto de esta clula
sensibilizada con el alrgeno produce la activacin celular que desencadena el proceso inflamatorio en dos fases: inmediata (dependiente de
mediadores qumicos e IL) a los pocos minutos del contacto y tarda, dependiente del infiltrado celular (eosinfilos, neutrfilos, mastocitos). Esta
activacin del sistema por interacciones vasculares y neurognicas genera los sntomas. Los principales mediadores qumicos con un papel central
en el desencadenamiento de la sintomatologa son: Histamina: es el principal mediador en la fase inmediata de la reaccin alrgica posterior a la
provocacin antignica. Se almacena en los grnulos del basfilo y del mastocito. La histamina acta sobre los receptores H1 de varias clulas y
causa los principales sntomas de rinitis. Leucotrienos: son formados de novo desde el ac. araquidnico por va de la lipooxigenasa, liberados
principalmente por el mastocito en la fase temprana y por eosinfilos y neutrfilos en la fase tarda. Los leucotrienos producen bloqueo e
incremento de la secrecin, pero no estornudos. Citoquinas: son liberadas por los linfocitos T durante la reaccin de fase tarda y por el mastocito;
resultan importantes para mantener la inflamacin crnica. CLASIFICACION: La OMS recomienda utilizar la establecida por ARIA (Allergic Rhinitis and
Its Impact on Asthma), de acuerdo a duracin de los sntomas: intermitente (<4 das a la semana y <4 semanas al ao) o persistente (>4 das a la
semana y >4 semanas al ao). Rinitis intermitente comprende aproximadamente al 20% de los casos de rinitis alrgica y la rinitis persistente afecta al
80% restante. DIAGNOSTICO: Para un adecuado diagnstico es importante realizar una cuidadosa historia clnica, que permitir caracterizar la
sintomatologa para clasificar la rinitis. En el interrogatorio se debe poner nfasis en el anlisis de los sntomas: 1. Obstruccin nasal: respiracin
bucal crnica, falta de aire, hiposmia-anosmia, voz nasal, babeo, ronquido, disfuncin tubariataponamiento otico. 2. Rinorrea: ruidos nasales, tos
farngea, nausea, dilatacin abdominal por
aerofagia. 3. Prurito palatino y nasofarngeo:
frotamiento, cloqueo, grgaras. 4. Estornudos.
5. Otros: epistaxis, fatiga, irritabilidad, prurito
ocular, hiremia conjuntival, edema palpebral.
Examen fsico: La observacin directa permite
detectar signos caractersticos, denominados:
facies alrgica, cianosis infraorbitaria (ojeras),
pliegue palpebral supernumerario (signo de
Denie Morgan), pliegue transversal en la nariz
(producido por el prurito y la limpieza
constante). Esta facies puede estar asociada a
signos de respiracin bucal, como consecuencia
de la obstruccin nasal. Tambin son frecuentes
el edema y la hiperemia conjuntival. En el
diagnstico, es indispensable el examen de la
cavidad nasal por rinoscopia anterior; se debe realizar con especial atencin a la estructura sea del septum nasal, en busca de desviaciones o
deformidades que impidan el normal flujo de aire. La mucosa puede presentarse inflamada, plida, gris-azulada, con secrecin cristalina. Los
cornetes edematizados pueden obstruir, de manera total o parcial, las fosas nasales. Se analizarn las principales pruebas que permiten establecer
un diagnstico etiopatognico adecuado: Recuento de eosinfilos en sangre perifrica. Citologa nasal. IgE total y especfica. Pruebas cutneas.
Estudios complementarios especiales como: Rinofibroscopia, el mejor modo de examinar el interior de las fosas nasales es con la ayuda de un
espculo nasal y luz frontal o con otoscopio. Radiologa convencional: La frecuente asociacin entre la rinitis y las alteraciones estructurales de la va
area superior, asociadas u originadas en el proceso inflamatorio, pueden necesitar del estudio por imgenes. TRATAMIENTO: No farmacolgico:
Educacin, medidas de control ambiental. Evitar alrgenos. Farmacolgico: ver cuadros anexos. En los cuadros intermitentes con grado de gravedad
leve, los frmacos de eleccin son: antihistamnicos orales tpicos, solos o asociados a descongestivos. En las rinitis intermitentes de intensidad
moderada-grave y en las persistentes leves, los frmacos de eleccin son: corticoides inhalatorios, antihistamnicos orales y tpicos, o
descongestivos y cromonas.

CURSO ENARM CMN SIGLO XXI TEL: 36246001

Pharmed Solutions Institute

PGINA 380

MANUAL DE TRABAJO DEL CURSO ENARM CMN SIGLO XXI


ASMA
CIENCIAS BASICAS: Es una enfermedad crnica inflamatoria de las vas respiratorias (participan clulas cebadas, eosinfilos, linfocitos T, neutrfilos
y clulas epiteliales), que se caracteriza por obstruccin reversible o parcialmente reversible de stas, adems de hiperrespuesta a diferentes
estmulos y se caracteriza clnicamente por: tos, disnea y sibilancias. Es particularmente importante la reversibilidad de la obstruccin bronquial,
completa parcial, ya sea en forma espontnea o en respuesta a tratamiento. SALUD PUBLICA: Primer causa de ausentismo escolar, tiene un gran
impacto en la calidad de vida del que la sufre, que produce importantes alteraciones en economa y la dinmica familiar. Las nicas encuestas
realizadas en Mxico de l948 a l991 que aparecen en la literatura reportan una prevalencia de 1.2 a 12.5%. La atopa; trmino que define a algunas
enfermedades con niveles elevados de IgE asociados a una predisposicin gentica definida, constituye el factor epidemiolgico ms consistente
para el desarrollo de asma en la infancia. El asma es una causa rara de muerte infantil en nios y adolescentes. PATOGENIA: Factores de riesgo: 1.
Alrgenos (caros, mascotas, hongos, cucarachas), la exposicin temprana a ellos tiene una alta correlacin con asma. 2. Virus desencadenan 90%. 3.
Tabaquismo de los padres es otro factor que se ha comprobado en meta-anlisis que incrementa el riesgo de padecer asma. 3. Contaminacin,
ozono, monxido de carbono, bixido de nitrgeno, partculas suspendidas. 4. Sustancias qumicas. 5. Ejercicio, su importancia radica en que nos
indica indirectamente la presencia de inflamacin bronquial a pesar de que el nio no tenga sntomas en reposo. 6. Fro. 7. Emociones. 8.
Medicamentos, como AINES. La remodelacin de la pared de la va respiratoria explica la importancia de la inflamacin crnica recurrente en el
asma. Los factores desencadenantes los alrgenos inhalados inducen la activacin de las clulas cebadas y macrfagos, con la consecuente
liberacin de varios mediadores pro-inflamatorios, incluyendo leucotrienos, factores quimiotcticos y citocinas. Los antgenos procesados y
presentados por los macrfagos a los linfocitos Th0, bajo la influencia un patrn adecuado de citocinas, estimula la diferenciacin a un patrn de
citocinas Th2 lo que a su vez estimulan la liberacin de mayores cantidades de IL- 4 e IL- 5, los cuales a su vez causan la sntesis de IgE por los
linfocitos B y eosinofilia, respectivamente. Las citocinas derivadas de macrfagos tales como IL -1, TNF- alfa, INF- gama, activan las clulas
endoteliales, aumentando la expresin de las molculas de adhesin tal como ICAM 1 y VCAM 1. Ms aun la IL - 4 derivada de clulas T
selectivamente aumenta la expresin de VCAM 1. Esto permite la salida de leucocitos de la vasculatura a la mucosa de las vas areas. Las
consecuencias de este infiltrado de clulas inflamatorias incluyen la esfacelacin del epitelio respiratorio, anormalidades en el control autonmico
del tono de las vas areas, cambios en la funcin mucociliar y aumento de las respuestas bronquiales. Esta reaccin inflamatoria autoperpetuable
de esas clulas efectoras (eosinfilos y clulas cebadas) es capaz de sintetizar citocinas que posteriormente promueven un incremento del proceso
inflamatorio. En resumen: Contraccin muscular bronquial (broncoespasmo o broncoconstriccin), edema e infiltracin celular de la mucosa
bronquial (inflamacin) e incremento de la secrecin bronquial. DIAGNOSTICO: El diagnstico se basa en la presencia de signos objetivos de
obstruccin bronquial, bsicamente por medio de la exploracin fsica (signos de dificultad respiratoria, sibilancias, espiracin prolongada,
hipoventilacin, opresin torcica, etc.). Estos signos obstructivos deben de ser recurrentes, esto es, presentarse en forma de exacerbaciones
episdicas (crisis), aunque en grados ms severos los sntomas obstructivos pueden ser persistentes, y aun as, presentar episodios de agravamiento.
Otra caracterstica bsica del diagnstico es que la obstruccin bronquial es reversible, o al menos parcialmente reversible, a veces en forma
espontnea o en base a tratamientos con broncodilatadores y/ anti-inflamatorios. Adems de lo anterior, en la gran mayora de los casos se puede
documentar en la historia clnica el fenmeno de hiper-reactividad bronquial, esto es, el inicio la exacerbacin de signos y sntomas de reaccin
bronquial (tos, secrecin bronquial, sibilancias, disnea) a una diversidad de estmulos fsicos, qumicos emocionales (ejercicio, olores
penetrantes, humos, cambios de temperatura humedad ambiental, etc.). Funcional: FEM (Flujo Espiratorio Mximo) Este valor proporciona una
medicin simple y cuantitativa de la obstruccin de las vas areas. Se realiza con un flujmetro porttil. El monitoreo de FEM es una herramienta
clnica de gran valor en el consultorio, en el hospital y hogar del paciente ya que permite valorar: La respuesta al tratamiento durante una crisis
aguda, respuesta al tratamiento crnico, detectar el deterioro asintomtico de la funcin respiratoria, antes de que se vuelva ms grave. Identificar
factores desencadenantes como por ejemplo el ejercicio. Espirometra: prueba de broncodilatador. FEV-1 (Volumen Espiratorio Forzado en el primer
segundo): Es la fraccin de volumen que se expulsa en el primer segundo del esfuerzo respiratorio mximo, y normalmente representa
aproximadamente el 80% de la CVF. Una disminucin >20% de este volumen, es indicativa de un proceso obstructivo. Todos los pacientes con
diagnstico presuntivo de asma deben ser valorados con una espirometra, al menos al iniciar su tratamiento y, posteriormente, controles de
seguimiento dependiendo de cada caso. Laboratorio: BH con eosinofilia, niveles IgE, pruebas cutneas, citologa nasal, coproparasitoscpicos. Tele
de trax en busca de complicaciones, radiologa de senos paranasales, gasometra, pruebas de funcin pulmonar. CLASIFICACION DE ASMA: 1.
ASMA INTERMITENTE: Sntomas menos de una vez a la semana. Exacerbaciones breves. Sntomas nocturnos no ms de 2 veces al mes. FEV1 >80%,
variabilidad del FEV1 <20%. 2. ASMA LEVE PERSISTENTE: Sntomas >1 vez a la semana pero <1 vez al da. Exacerbacin afecta la actividad o sueo.
Sntomas nocturnos >2 veces al mes. FEV1/FEM >80 %, variabilidad de FEV1 20-30%. 3. ASMA MODERADA PERSISTENTE: Sntomas Diarios.
Exacerbaciones que afectan la actividad o sueo. Sntomas nocturnos >1 vez a la semana. Uso diario de agonistas B2 Inhalados de accin rapida.
FEV1 o PEF de 60-80%, variabilidad del FEV1 >30%. 4. ASMA SEVERA PERSISTENTE: Sntomas Diarios. Exacerbaciones frecuentes. Sntomas nocturnos
frecuentes. Limitacin de actividad fsica. FEV1 <60%,
Exacerbaciones
NO
Uno o ms/ ao
Una vez/sem
variabilidad FEV1 >30%. CLASIFICACION DE LAS CRISIS
ASMATICAS: Leve: limita actividades fuertes (ejercicio), habla sin problemas, no lo despierta. Flujometria: PEF > 80%, FC <100 lpm, sibilancias:
moderadas. Moderada: limita actividades habituales, habla con dificultad. Flujometra: FEM 60-80% FC 100-120 lpm, sibilancias: intensas. Severa:
incapacidad de realizar actividades, dificultad respiratoria en reposo, cianosis, habla palabras. Flujometra: < 60% FC >120 lpm, sibilancias: intensas.
Paro inminente: conciencia: confundido, tiros: respiracin paradojia, bradicardia, sibilancias: silencio. TRATAMIENTO: Medidas Generales: Control
ambiental, evitar exposicin a desencadenantes, vacuna anti influenza, evitar AINEs, reducir o eliminar la inflamacin. Las metas en el tratamiento
del asma son: a.- Control adecuado de los sntomas. b.- Prevenir las exacerbaciones. c.- Mantener FR dentro de la normalidad. d.- Mantener
actividad normal. e.- Evitar efectos adversos de medicamentos. f.- Prevenir el asma fatal. Los medicamentos utilizados son: ESTEROIDES INHALADOS:
Mejoran la funcin pulmonar, disminuyen la hiperreactividad bronquial, los sntomas y las exacerbaciones, mejoran la calidad de vida. Las dosis
recomendadas son: Beclometasona: nios 100-800 ug/da (leve-moderada:200, grave: > 500 dosis mxima: 1000). Budesonida: nios 100-400 ug/da
(leve moderada:200, grave: > 400, dosis mximas: 800). Fluticasona: nios: 100-200 ug/da (leve-moderada: 100, grave: > 200, dosis mximas: 500).
Ciclesonida : Dosis 100 a 1200 mcg; Va de administracin : Inhalado. Triamcinolona: Dosis 400 a 2000 mcg/da va de administracin: Inhalado
Reacciones secundarias: Candidiasis oral, disfona, tos (irritacin de la va area), prevencin: uso de espaciadores. ESTEROIDES SISTEMICOS: Esta
terapia se utiliza para el control de los pacientes con asma severa persistente. Han demostrado disminucin de signos patolgicos de inflamacin y
mejora de la hiperreactividad bronquial. Prednisona: 0.5-1 mg/kg/da dosis de reduccin. Deflazacort : 5-10 mg/kg/da dosis de reduccin.
Metilprednisolona: Dosis 4-6 mg/kg cada 6-8 hrs. Hidrocortisona: Dosis 4-6 mg/ cada 6-8 hrs. Efectos secundarios: Osteoporosis, DM, glaucoma,
obesidad, estras cutneas, supresin del eje hipotalmico, metilxantinas: broncodilatador cuyo mecanismo de accin est relacionado con la
inhibicin de la fosfodiesterasa. Infusin: 5 mg/kg/dosis IV 0.7-0.9 mg/kg/hr. Reacciones secundarias: Nauseas y vmito, cefalea, taquicardia,
arritmias. B2 AGONISTAS: Relajan la musculatura lisa bronquial, mejoran el aclaramiento mucociliar, disminuyen la permeabilidad vascular y la

CURSO ENARM CMN SIGLO XXI TEL: 36246001

Pharmed Solutions Institute

PGINA 381

MANUAL DE TRABAJO DEL CURSO ENARM CMN SIGLO XXI


liberacin de mediadores inflamatorios. Los ms utilizados son: Salbutamol: 200mcg PRN. Fenoterol: 50mcg PRN. Salmeterol: 25mcg cada 12 hrs.
Terbutalina: 500mcg PRN. B2 Agonistas y Esteroides: Disminucin de los sntomas y de crisis de asma nocturna, mejora la funcin pulmonar, reduce
el nmero de exacerbaciones. MODIFICADOR DE LEUCOTRIENOS: Cistenil leucotrieno, Montelukast, Pranlukast, Zafirlukast. ANTICOLINRGICOS:
Bromuro de Ipratropio y Bromuro de oxitropio, mecanismo de accin: Broncodilatador. Medicamentos controladores: Esteroides inhalados,
agonistas de larga accin, Anti leucotrienos, Xantinas. Medicamentos de rescate: agonistas de corta accin, anticolinrgicos. Crisis asmticas:
Salbutamol + bromuro de ipratropio, 2-4 disparos c/20min x hora. LEVE: 2-4 disparos cada 3-4 hrs, MODERADA: 6-10 disparos por 1 2 hrs, SEVERA:
10 disparos o ms. ESTADO ASMATICO: Forma letal del asma, caracterizado por ataques cada vez peor que no mejora con tratamiento apropiado,
que culmina con la insuficiencia de la vlvula pulmonar: empleo de msculos accesorios de la respiracin, pulso paradjico, cianosis. Criterios
hospitalizar. Terapia intensiva: Ataques frecuentes y repetidos, ltima vez asma grave que culmino en hospitalizacin. Consumo diario y excesivo de
broncodilatadores y corticoesteroides. Empleo de msculos accesorios de la respiracin. Pulso paradjico que rebasa los 18 mmHg en adolescentes
y 10 mmHg en nios. Cambios en la conciencia, cianosis. Neumotrax, neumomediastino. FEV1 o PEFR (velocidad de flujo espiratorio mximo)
menor de 20% de la cifra calculada. PaO2 < 60mmHg. PaCO2 > 40mmHg en presencia de disnea y sibilancias. Acidosis metablica. Anormalidades
electroencefalogrficas.
CASO CLINICO ASMA
Nia de 7 aos que acude a urgencias por cuadro de tos, dificultad
respiratoria de dos semanas de evolucin y pico febril 38C. con
antecedentes patolgicos de obesidad, roncadora habitual y
respiracin bucal. adenoidectomizada hace 2 aos por sospecha
clnica de SAHS sin control posterior. Peso 56kg (p>97), FC 135 lpm
(p>95), TA 109/70, SatO2 82% con FiO2 21%. Facies anmica,
implantacin baja del pelo, estrabismo y obesidad mrbida.
Aceptable estado general, normohidratada, palidez cutnea y
retracciones subcostales moderadas. Auscultacin cardiaca normal.
Auscultacin respiratoria con hipoventilacin generalizada
moderada, sibilantes espiratorios finos y roncus dispersos.
PREGUNTA
Cul es la conducta farmacolgica mas adecuada?
RESPUESTA
a.- Salbutamol, bromuro de ipatropio inhalados y corticoides
endovenosos.
b.- Salbutamol, corticoide, loratadina.
c.- Salbutamol, oxigeno, ambroxol y prednisona.
d.- Ambroxol, prednisona, Bromuro de ipatropio y oxigeno.
PREGUNTA
El paciente empeoro su cuadro clnico, con mayor dificultad
respiratoria. Se decide enviar a segundo nivel. Al ingreso se realiz
BH con discreta leucocitosis, bioqumica normal y gasometra capilar:
pH 7,31, PC02 52,3mmHg, PO254,7mmHg, HC0326mmol/l. Precisa
FiO2 inicial de 1 para mantener SatO2>92%. Presenta evolucin
lenta y respuesta parcial al tratamiento. Tras los primeros das de
ingreso persisten necesidades de oxigenoterapia elevadas (FiO20,5)
durante el sueo. En controles de gasometra destaca hipercapnia
nocturna (PCO2 56,3mmHg). La evolucin trpida obliga a nuevo
planteamiento
diagnstico
y
realizacin
de
pruebas
complementarias. Cuales fueron los criterios ms importantes para
el envio a segundo nivel?
RESPUESTA
a.- Duda diagnostica.
b.- Asma complicada.
c.- Asma no controlada con tratamiento.
d.- Antecedetes de exacerbaciones.
CASO CLINICO ASMA
Se trata de un adolescente de 13 aos y 90kg de peso con
antecedentes de asma sin tratamiento de fondo. Acude por
dificultad respiratoria de varias horas de evolucin sin que hubiera
recibido broncodilatadores previamente. Se constata una crisis
asmtica grave con taquipnea y retraccin costal marcadas, y se
inicia tratamiento con nebulizaciones de salbutamol (15mg en la
primera hora) y bromuro de ipratropio (500 mg/h) asociados a
metilprednisolona por va intravenosa (60mg). Transcurrida 1 h de

CURSO ENARM CMN SIGLO XXI TEL: 36246001

tratamiento, se realiza gasometra venosa que muestra acidosis


metablica con hiperlactacidemia.
PREGUNTA
Cul es la conducta a seguir?
RESPUESTA
a.- Mantener el tratamiento.
b.- Enviar a segundo nivel.
c.- Ingreso a la UCI.
d.- Realizar rx de torax.
PREGUNTA
Al ingreso hospitalario se mantiene tratamiento broncodilatador
(salbutamol y bromuro de ipratropio) horario. Tras 4h de ingreso, se
mantiene el trabajo respiratorio, la taquipnea y la taquicardia, con
saturacin de oxgeno del 92% y con fraccin inspiratoria de oxgeno
(FiO 2 ) del 35%. La radiografa de trax presenta hiperinsuflacin sin
condensacin neumnica, atelectasia o escape areo. La gasometra
muestra empeoramiento de la acidosis lctica. Se ingresa al paciente
en cuidados intensivos para el inicio de la ventilacin no invasiva tipo
presin positiva con dos niveles de presin con los parmetros
iniciales (presin positiva inspiratoria de la va area de 12; presin
positiva espiratoria de la va area de 6; FiO 2 del 40%), se mantiene
el tratamiento broncodilatador y se expande la volemia. Cul es el
pronostico esperado?
RESPUESTA
a.- Mejora ventilatoria.
b.- Descenso significativo de la FR.
c.- Normotenso, caliente y bien perfundido.
d.- Incremento de acido lctico.

CASO CLINICO
Un nio de raza blanca, de 15 aos de edad, con antecedente de
asma grave, llega a la sala de urgencias con dificultad respiratoria
obvia. Despues de la admisin y de multiples tratamientos con
nebulizador, presenta, nausea, vomito y debilidad. Los estudios
revelan concentracin de potasio de 2.6mEq/l y ondas U en el
electrocardiograma.
PREGUNTA
Cul de los siguientes medicamentos es mas probable que haya
desencadenado estos sntomas?
RESPUESTA
a.- Albuterol
b.- Ipratropio
c.- Prednisona
d.- Teofilina

Pharmed Solutions Institute

PGINA 382

MANUAL DE TRABAJO DEL CURSO ENARM CMN SIGLO XXI


FIBROSIS QUISTICA (FQ)
CIENCIAS BASICAS: Es la enfermedad gentica ms letal, de carcter recesivo, multisistmico y progresivo, afecta de preferencia a poblaciones
caucsicas. La enfermedad se caracteriza por el espesamiento del mucus producido por las glndulas exocrinas induciendo compromiso sinopulmonar con dao pulmonar progresivo, insuficiencia pancretica y por lo tanto sndrome de mala-absorcin, con consecuente desnutricin,
esterilidad masculina por atrofia de los conductos deferentes y elevacin de electrolitos en el sudor. El leo meconial siempre debe obligar a
descartar FQ. Dependiendo de las mutaciones involucradas existe una gran diversidad de formas clnicas. SALUD PUBLICA: Su incidencia vara de 1
por cada 3,000 a 1 por cada 8,000 nacidos vivos. PATOGENIA: El gen de la FQ se encuentra localizado en el brazo largo del cromosoma 7, codifica
una protena de 1480 aminocidos que se ha llamada Protena Transportadora de Transmembrana (CFTR), esta se localiza en el polo apical de las
clulas epiteliales. Es una glicoprotena cuya funcin es actuar como canal de cloro, est constituida por dos regiones transmembrnicas
(hidrofbicas) separadas por una regin de unin al ATP. La primera mutacin encontrada fue la F508, localizada en el dominio NBD1, presente en
alrededor del 75% de la poblacin caucsica. El resultado de todas las mutaciones detectadas que alteran la funcin de la CFTR es el mismo: la
imposibilidad de transportar cloruro. Cualquiera que sea la mutacin en el gen CFTR, cada paciente muestra las siguientes anormalidades en
distintos grados: A) Concentracin anormal de los iones en las secreciones de las glndulas serosas, manifestada por aumento en la concentracin
de cloro y sodio en el sudor. B) Incremento en la viscosidad de las secreciones de las glndulas secretoras de moco, asociado con obstruccin y
perdida secundaria de la funcin glandular. C) Aumento en la susceptibilidad a la colonizacin endobronquial crnica por grupos especficos de
bacterias (Staphylococcus aureus, H. influenzae, Pseudomona aeruginosa, Burkholderia cepacia). Se han descrito 6 clases de mutaciones: las I a III
son las ms comunes y generalemnrte se relacionan con insuficiencia pancretica. En Mexico se han identificado 46 diferentes mutaciones que
afectan a 77% de los cromosomas de la fibrosis qustica. DIAGNOSTICO: La mayora de los casos de FQ se manifiesta por la triada clsica: a)
enfermedad pulmonar obstructiva progresiva crnica con infeccin agregada, b) insuficiencia pancretica exocrina, c) elevacin en las
concentraciones de Cl y Na en el sudor. Los RN afectados rara vez muestran sntomas respiratorios, aunque los menores de 6 meses de edad pueden
experimentar taquipnea, sibilancias, incremento del trabajo respiratorio, sobresdistension del trax y atelectasias. En 10-20% de los pacientes, el
leo meconial puede ser la primera manifestacin de la enfermedad. Se produce por la implantacin de meconio deshidratado en el leo terminal,
con un cuadro de obstruccin intestinal. Puede sospecharse antes del parto por ecografa u ocurrir al nacimiento con distensin abdominal
progresiva y vmitos biliosos y falta o retardo en la eliminacin de meconio en las primeras 24-48hrs de vida. La radiografa de abdomen suele
revelar asas intestinales dilatadas con reas de aire mezclado con meconio deshidratado. En la mayora de los casos se encuentran una o ms
expresiones clnicas de la enfermedad, que se confirma con la prueba de sudor. Casi todos los pacientes exhiben enfermedad sinusopulmonar
crnica y el 85-90% tiene insuficiencia pancretica exocrina. Es necesario hace notar que debe considerarse como FQ clsica, aquella con niveles de
test del sudor altos, compromiso pulmonar severo, progresivo, de instalacin temprana, rinosinusitis, sndrome de malaabsorcin e infertilidad
masculina. Se ha denominado como FQ no clsica o enfermedad vinculada a FQ aquella con al menos una mutacin del gen de FQ, lo que le confiere
funcin parcial a la protena CFTR, generalmente no tienen sndrome de maladigestin por estar preservada la funcin exocrina y por lo tanto mejor
estado nutricional, con niveles de test del sudor discretamente altos o normales, enfermedad pulmonar de instalacin tarda y de menor severidad,
rinosinusitis y azoospermia, con clara mejor sobrevida. DIAGNOSTICO PREIMPLANTACIONAL: determinar caractersticas genticas del embrin a
partir de una sola clula obtenida mediante biopsia embrionaria, sin prejuicio para la viabilidad del mismo. Por consiguiente, se requiere la
obtencin de embriones mediante fecundacin in vitro. DIAGNOSTICO PRENATAL: Se analiza el ADN de clulas de vellosidades corionicas o lquido
amnitico. Se realiza si los padres son portadores o si existe un hermano con fibrosis qustica. ESTUDIO NEONATAL: Se basa en el hecho de que las
concentraciones sricas de tripsina de los enfermos con insuficiencia pancretica pueden ser incluso 8 veces mayores a lo normal, se analizan:
tripsina, tripsinogeno o complejo tripsina 1 anti-tripsina. El primer estudio se realiza entre el 1-5 das de vida; si es positivo se repite entre la segunda
y octava semanas si las concentraciones se mantienen elevadas, se hace la prueba de Gibson y Cooke y estudio gentico. Test del sudor: la
iontoforesis de pilocarpina por el mtodo de Gibson y Cooke, continua siendo el gold standard que permite medir los valores de sodio y cloro en el
sudor; en el tbulo de la glndula sudorpara est bloqueado el reingreso de cloro a la clula, por lo cual tampoco lo hace el sodio, tenindose un
sudor con mayor cantidad de estos electrolitos. Se considera los siguientes valores: Positivo: >60 meq/lt, Limtrofe: 40 a 59 meq/lt, Negativo: <40
meq/lt. I.- Test secretina pancreozimina: gold standard para medir funcin pancretica. ANATOMIA PATOLOGICA: Los hallazgos macroscpicos son
limitados y solo sugestivos de fibrosis qustica. Los hallazgos histopatolgicos son variables e incipientes en RN con afeccin principal a glndulas
mucosas, y se vuelven ms evidentes al avanzar la enfermedad los ms representativos: A. Pncreas: 93% de los casos tienen cambos
histopatolgicos. Antes de las 40 semanas puede verse normal, el volumen acinar va disminuyendo y puede ser 25% menor a los cinco meses de
vida. B. Tubo gastrointestinal: en RN, el leo meconial se ha asociado como hallazgo patolgico inicial de fibrosis qustica (15 a 20%). La atresia
intestinal (leon, yeyuno, o ambos) afecta a 15-25% de los casos de fibrosis qustica. La peritonitis meconial ocurre en 33 a 50% de los pacientes con
fibrosis qustica debido a la perforacin intestinal intrauterina. C. Hgado y vas biliares: 60% de los pacientes sufren alteraciones clnicas y
morfolgicas, como cirrosis biliar focal, manifestada por proliferacin de conductos biliares dilatados con material eosinoflico intraluminal y reas
de fibrosis irregular portal e infiltracin de linfocitos. D. Vas respiratorias: las manifestaciones morfolgicas principales son: hiperplasia de glndulas
submucosas bronquiales con moco espeso intraluminal, bronquiectasias, atelectasias, neumonas, obstruccin bronquial por moco y clulas
inflamatorias que expanden las vas areas y que se extienden al parnquima pulmonar. TRATAMIENTO: Son cinco los pilares bsicos del
tratamiento de esta enfermedad: 1) tratar la infeccin y la inflamacin, y reparar el aclaramiento mucociliar, 2) mantener un buen estado de
nutricin, 3) tratar la insuficiencia del pncreas exocrino, 4) iniciar fisioterapia respiratoria y 5) detectar y tratar de manera oportuna enfermedades
concomitantes (diabetes, hepatopata y osteopenia). El germen que ms frecuentemente (60%) se asla en el esputo de los enfermos de fibrosis
qustica es P. aeruginosa. Existe gran inters en los antibiticos aerosolizados. Los beneficios potenciales de administrar antibiticos va aerosol
incluyen: el depsito directo en el sitio endobronquial de la infeccin, la disminucin de la toxicidad, mejor relacin costo-beneficio y mejor calidad
de vida. En un estudio basado en evidencias de la Asociacin Americana de Fibrosis Qustica se recomienda, para tratar las manifestaciones
pulmonares, administrar alfa dornasa inhalada (Pulmozymes), que es una desoxirribonucleasa recombinante que acta como mucolticos,
degradando el ADN. El gasto de energa incrementado en el paciente con fibrosis qustica se debe a la insuficiencia pancretica, la malabsorcin de
nutrimentos y la inflamacin. La adecuada alimentacin favorece la sntesis proteica. ltimamente se ha autorizado la administracin de Kalydeco
(ivacaftor), el cual funciona como un potenciador, ya que facilita que el canal transporte ms cloro. Staphylococcus aureus meticilina sensible:
Cloxacilina: 100 a 200 mg/Kg/da (d), c/6h por 21 das, oral o endovenoso, segn la situacin del paciente, o Flucloxacilina: 50 a 75 mg/Kg/d, c/8h por
21 das, oral. Staphylococcus aureus meticilina resistente: Vancomicina: 50 mg/Kg/d, c/6h por 21 das, IV, o Linezolid: 150 a 300 mg c/12h por 21
das, oral o IV. Est indicado en caso de alergia a Vancomicina, no disponibilidad de vas ev o tratamiento ambulatorio. Haemophilus influenzae lactamasa negativo: Amoxicilina: 90 mg/Kg/d, c/12h por 14 das, oral. Haemophilus influenzae -lactamasa positivo: Amoxicilina-cido clavulnico:
90 mg/Kg/d, c/12h por 14 das, oral, o Cefotaxima: 100 mg/Kg/d, c/6h por 14 das, IV. Pseudomonas aeruginosa: Ceftazidima: 200 a 250 mg/Kg/d,
c/6h por 14 a 21 das, IV ms Amikacina: 20 a 30 mg/Kg/d ev, o Tobramicina: 10 mg/Kg/d IV. Alternativa de segunda lnea: Ciprofloxacina: 30

CURSO ENARM CMN SIGLO XXI TEL: 36246001

Pharmed Solutions Institute

PGINA 383

MANUAL DE TRABAJO DEL CURSO ENARM CMN SIGLO XXI


mg/Kg/d, c/12h oral (mximo 300 mg c/8h IV en infecciones severas) por 14 a 21 das.
CASO CLINICO
Se presenta el caso de un paciente de tres aos de edad, raza blanca
y sexo masculino; presenta, tos hmeda, vmitos flemosos,
antecedentes patolgicos personales de fibrosis qustica y
perinatales de parto eutcico a trmino, con peso al nacer de 6,8
libras, y apgar 7/9, por aspiracin de lquido meconial, no ctero, y
cada del cordn umbilical a los tres das. Lactancia materna hasta
los tres meses y lactancia artificial actual con leche evaporada.
Vacunacin actualizada y medio-ambiente social favorable. Present
antecedentes patolgicos personales de broncoaspiracin de
meconio al nacer y un ingreso al mes y medio por atelectasia,
adems de un segundo ingreso a los siete meses de nacido por
polipnea y no ganancia de peso, a pesar de alimentarse bien, as
como deposiciones pastosas con grasa segn refiri la mam, suda
mucho y el sudor es salado.
PREGUNTA
Cual es la conducta a seguir para establecer el diagnostico
secundario que presenta el paciente?
RESPUESTA
a.- TC abdominal.
b.- USG abdominal.
c.- IRM abdominal.
d.- LAPE.
CASO CLINICO
Paciente de tres aos de edad, raza blanca y sexo masculino; acudi
al Hospital por presentar, tos hmeda, vmitos flemosos, no
ganancia de peso, a pesar de alimentarse bien, as como
deposiciones pastosas con grasa segn refiri la mam, suda
mucho y el sudor es salado. antecedentes patolgicos perinatales
de parto eutcico a trmino, con peso al nacer de 6,8 libras, y apgar
7/9, por aspiracin de lquido meconial xxxx, no ctero, y cada del
cordn umbilical a los tres das. Lactancia materna hasta los tres
meses y lactancia artificial actual con leche evaporada. Vacunacin
actualizada y medio-ambiente social favorable.
PREGUNTA
Qu electrolito es el mas probable implicado en esta patologa?
RESPUESTA
a.- Sodio
b.- Potasio
c.- Calcio
d.- Cloro
PREGUNTA
Qu complicacin es la ms grave que odria presentar este
paciente?
RESPUESTA
a.- Diarrea cronica
b.- Infecciones pulmonares
c.- Infecciones respiratorias repetidas
d.- Infecciones a nivel renales
PREGUNTA
Qu agentes etiolgicos son los mas implicados en este dao?
RESPUESTA
a.- Staphylococcus aureus, Pseudomona aeruginosa
b.- E.coli, salmonella
c.- Streptococo, Haemophilus influenzae
d.- Proteus, E.coli
CASO CLINICO
Varn de dos aos y tres meses de edad que debut con fallo de
medro al mes de vida, con ingesta adecuada para la edad y el peso.
En ese momento, se objetiv edema de prpados superiores, palidez

CURSO ENARM CMN SIGLO XXI TEL: 36246001

cutnea, leve distrofia muscular en muslos y glteos, e hipotona de


miembros superiores y cervicoaxial. Se realizaron urocultivo
(negativo) y controles de tomas y peso, con glucemia capilar de 78
mg/dl. Durante uno de esos controles, en la consulta, realiz una
deposicin bien conformada, verdosa y brillante, por lo que se
solicit estudio de heces (negativo para hemoglobina, buena
digestin de hidratos de carbono y fibras musculares, se observaban
grasas). Se paut hidrolizado de leche de vaca, a raz de lo cual
empez a estar ms activo, con mejor tono muscular y ganancia
ponderal. Se solicitaron dos ionotest, que resultaron de 105 mmol/l
y 100 mmol/l de Cl. A los tres meses, se inici tratamiento con
pancreatina y, a los cuatro meses, con vitaminas A, D, E y K. El
estudio gentico mostr como resultado la alteracin
F508del/G542 .
PREGUNTA
Cul es el diagnotico ms probable en este caso?
RESPUESTA
a.- Gatroenteritis aguda
b.- Intucepcion
c.- Fibrosis qustica
d.- Enterocolitis
PREGUNTA
A los 14 meses, en una ecografa abdominal se visualiz barro en la
vescula biliar Cul es la conducnta a seguir mas adecuada?
a.- Quirurgico
b.- Colestiramina
c.- Acido ursodeoxiclico
d.- Pancreatina
PREGUNTA
Cul es la caracterstica menos probable en esta patologa?
RESPUESTA
a.- Disfuncin de las glndulas exocrinas
b.- Obstruccin crnica de las vas respiratorias
c.- Dao heptico
d.- Insuficiencia pancretica
CASO CLINICO
Femenina de 1 ao y 2 meses, producto de un primer embarazo,
peso al nacer de 3340 g y talla de 48 cm, APGAR 9/10, alimentada
con lactancia materna, ablactada al sexto mes. Sin antecedentes
heredo-familiares de importancia, con historia de 6 deposiciones
diarias diarreicas, amarillo verdosa y lquida desde su nacimiento.
Asocia prolapso rectal desde los ltimos 4 meses; inicialmente
intermitente y luego persistente. Al ingresar a nuestro centro
hospitalario se encuentra una paciente con un peso de 9,1 Kg, talla
72 cm, con prolapso rectal que se reduce fcil pero igualmente se
vuelve a prolapsar. Laboratorio; frotis de heces por parsitos y
clulas, pruebas de funcin tiroidea, hemograma, pruebas de
funcin heptica, azcares reductores y no reductores, Elisa HIV y
VDRL normales. No presentaba actividad de tripsina en heces, las
grasas en heces ++++, los valores de cloruros en sudor en 2 muestras
con 8 das de diferencia fueron de 85 mEq/L y 105 mEq/L
respectivamente. La Rx de trax mostr atrape areo e infiltrado
intersticial difuso bilateral.
PREGUNTA
Cul es la causa menos probable de prolapso rectal en esta
paciente?
RESPUESTA
a.- Hernia hiatal
b.- Diarrea aguda o cnica
c.- Colitis por 384antibiticos
d.- Polipos intestinales

Pharmed Solutions Institute

PGINA 384

MANUAL DE TRABAJO DEL CURSO ENARM CMN SIGLO XXI


INFECCION DE VIAS URINARIAS (CISTITIS, PIELONEFRITIS)
CIENCIAS BASICAS: El trmino infeccin de vas urinarias (IVUs) se aplica a una amplia variedad de trastornos que afectan el aparato urinario, indica
la presencia de bacteriuria significativa con o sin sintomatologa general de
infeccin y/o sintomatologa uretrovesical: desde infecciones asintomticas, hasta
aquellas que ponen en peligro la vida del enfermo como la pielonefritis. SALUD
PUBLICA: La IVU tiene mayor gravedad en los nios <1 ao de edad con frecuencia
aprox 1%. Despus de esta edad la prevalencia aproximada de 3% en nios y de 5 a
8% en nias. La tasa reportada de recurrencia es de 12-30%, con mayor
probabilidad en menores de seis meses, en caso de reflujo vesicoureteral (RVU)
grave y en aquellos con gamagrafa renal anormal al momento de la primera
infeccin. Entre un 8 y 40% de los menores de seis aos con IVU tienen RVU; otras
anormalidades comunes incluyen hidronefrosis, uropata obstructiva y doble
sistema colector. De un 10 a 65% de los de menores de dos aos presentarn
cicatrices renales. Estas ltimas se asocian con el desarrollo de hipertensin y
enfermedad renal terminal. Se ha encontrado que entre 10 y 25 % de los enfermos
con insuficiencia renal crnica, tienen como causa pielonefritis crnica.
CLASIFICACION: 1.- BACTERIURIA ASINTOMTICA: Pacientes que incidentalmente
presentan bacteriuria sin los sntomas clsico de IVU, para confirmar el diagnstico
se requieren 2 cultivos, que muestren el mismo microorganismo con recuento de 10 UFC o ms por milmetro de orina. 2.- INFECCIONES URINARIAS
BAJAS: Infecciones que no afectan por el momento el parnquima renal, constituyen el grupo de mayor frecuencia y requieren de un tratamiento
adecuado que cure las molestias del paciente y evite su diseminacin a las estructuras renales. Se subdividen en: Uretritis Aguda, Cistitis Aguda,
Prostatitis aguda, la cistitis aguda es la de mayor importancia en el paciente peditrico, esta es la afeccin de la vejiga que se manifiesta por disuria,
urgencia miccional, dolor supra pbico, incontinencia y orina ftida. La cistitis no produce fiebre, ni daos renales, la orina puede ser turbia incluso
con hematuria. 3.- INFECCIONES URINARIAS ALTAS: Corresponden a la pelvis renal, clices, parnquima renal. Su incidencia se incrementa en
pacientes con factores predisponentes como malformaciones obstructivas, RVU, estenosis de cuello vesical, y del meato urinario, etc. En otros casos
es causada por una diseminacin hematgena del germen. Se subdividen en: Pielonefritis aguda; representa el tipo ms grave de IVU, no solo
produce ms morbilidad, sino tambin mayor potencial para causar dao irreversible. En los nios de mayor edad con pielonefritis aguda se
manifiesta fiebre, dolor e hipersensibilidad en el flanco y se relaciona con piuria y urocultivos positivos. Los resultados de laboratorio casi siempre
revelan leucocitosis, VSG aumentada. Los RN y lactantes menores son los grupos de edad
con mayor riesgo de formacin de tejido cicatrizal renal posterior a pielonefritis.
Pielonefritis crnica; se caracteriza por alguna o todas las siguientes manifestaciones dolor
abdominal o en flanco, fiebre, mal estado general, nauseas, vomito, ictericias en los RN, y
en ocasiones diarrea. PATOGENIA: Aproximadamente 95% de las IVUs son causadas por
enterobacterias. La E. coli es responsable del 80 al 90%. El porcentaje restante puede ser
ocasionado por Staphylococcus saprophyticus, Enterococcus sp., Klebsiella sp.,
Enterobacter sp., Pseudomonas sp. y Proteus sp. De las 150 cepas de E. coli, diez de ellas
son responsables de la mayora de las IVU. Esta situacin est relacionada con la presencia
de factores bacterianos virulentos, como la alfa hemolisina (protena citoltica que lesiona
la membrana celular), siderforos (protena quelante de hierro que prolonga la vida de la
bacteria), y polisacridos capsulares (que disminuyen la activacin del complemento). La
presencia de fimbrias en las bacterias, que favorecen su adherencia al urotelio, tambin es
un factor de virulencia importante: El 91% de las cepas de E. coli que producen pielonefritis tienen fimbrias. Las bacterias pueden acceder al tracto
urinario a travs de cuatro vas: 1. Va ascendente, desde la uretra y la vejiga a los riones, la ms comn. 2. Hematgena: En pacientes
inmunocomprometidos en neonatos. 3. Linftica, desde el recto, colon y linfticos periuterinos, difcil de comprobar. 4. Directa, a travs de fstulas
rectovaginales a cualquier parte del tracto urinario. Los factores del husped que estn implicados en la presencia de bacteriuria se pueden
resumir en: 1. Edad: Los neonatos tienen mayor predisposicin a IVU por la inmadurez de su sistema inmunolgico. Adems existe una elevada
colonizacin periuretral en el primer ao de vida. 2. Colonizacin fecal, periuretral y
prepucial: El uso indiscriminado de antibiticos de cualquier tipo favorece la proliferacin
de cepas virulentas y multirresistentes. 3. Gnero: factores anatmicos inherentes a la
mayor accesibilidad de la vejiga a los grmenes en las nias que en los nios por tener la
uretra ms corta. 4. Genticos: Con mayor frecuencia los nios con IVU recurrentes tienen
en su epitelio urinario receptores, glucolpidos antgenos del grupo sanguneo P, que
facilitan la adhesin de las fimbrias o pili de E. coli. 5. Anormalidades genitourinarias: Estas
alteraciones deben ser identificadas tempranamente, ya que si no son corregidas a tiempo
pueden llevar a secuelas como cicatrices renales, prdida de parnquima renal, hipertensin arterial e insuficiencia renal crnica. Es por esta razn
que se recomienda el estudio imagenolgico en todos los pacientes que se presenten con IVU. DIAGNOSTICO: Clnica; Los sntomas y signos ms
comunes en menores de cinco aos por los que acuden a urgencias con el primer episodio de IVU son: fiebre 80%, irritabilidad 52%, anorexia 49%,
malestar 44%, vmito 42%, diarrea 21%. Los sntomas menos comunes (en menos de
20%): disuria, orina ftida, dolor abdominal, frecuencia y hematuria. La presencia de
fiebre >38C, bacteriuria y dolor lumbar sugiere pielonefritis, mientras que la presencia
de sntomas urinarios como disuria asociada a bacteriuria, pero no a sntomas sistmicos,
sugiere cistitis o IVU baja. En nios de dos a 12 aos de edad, en su primer episodio de
IVU, los sntomas ms frecuentes fueron disuria y urgencia en el 82%, dolor abdominal
35%, enuresis 45%, fiebre 26%, hematuria 20% y balanitis 20%. El ABC del diagnstico de
la infeccin urinaria se basa en: EGO, para reconocer la presencia de estearasa
leucocitaria, reduccin de nitratos a nitritos cuenta de clulas inflamatorias (ms de 10
clulas) y presencia de bacterias. Esta prueba tiene una sensibilidad de 75 a 90% y una
especificidad de 70 a 82%. Cultivo de orina. La limitante de este estudio es disponer de

CURSO ENARM CMN SIGLO XXI TEL: 36246001

Pharmed Solutions Institute

PGINA 385

MANUAL DE TRABAJO DEL CURSO ENARM CMN SIGLO XXI


una muestra adecuada para el proceso. Si la orina se obtiene de una bolsa colectora, la sensibilidad y especificidad son muy bajas ya que el 80% de
las muestras se hallaron contaminadas. Si la orina se obtiene por catter, la sensibilidad y especificidad son superiores a 70%; por puncin
suprapbica la presencia de cualquier nmero de colonias bacterianas permite asegurar el diagnstico El nmero de unidades formadoras de
colonias (UFC) necesarias para establecer el diagnstico de IVU est en funcin del tipo de muestra que se obtiene, ver cuadro 2 anexo.
Imagenologa. El estudio estndar es la urografa excretora con histograma miccional, aunque el ultrasonido lo ha ido desplazando gradualmente.
Este ltimo es una prueba de escrutinio para descartar malformaciones mayores. Sin
embargo, la sensibilidad y especificidad para el diagnstico de RVU son bajas en ambas. El
histograma miccional debe realizarse cuando el nio este afebril o el cultivo urinario sea
negativo. Los estudios con radioistopos pueden ser de utilidad para evaluar la funcin e
integridad renal en pacientes con pielonefritis. INFECCIONES RECURRENTES: La frecuencia
de recurrencias en el primer ao de vida es menor de 20% en nios y de menos de 30% en
nias; en nios mayores de un ao, es superior a 30%. En nias que han tenido ms de dos
episodios previos, la recurrencia puede llegar a 75%. TRATAMIENTO: Neonato. La
infeccin urinaria a esta edad es un problema de extrema gravedad, con alto riesgo de
sepsis, complicaciones, secuelas y muerte. Su tratamiento debe realizarse en el hospital. La
probabilidad de malformaciones de las vas urinarias es muy alta. El tratamiento
antomicrobiano para sepsis neonatal es el indicado. La asociacin de un betalactmico ms un aminoglucsido permite una cobertura de ms del
90% de los microorganismos involucrados ver cuadro 3 y 4. El tratamiento debe ser de 10 a 14 das. Cuando se pueda prescindir de la va
endovenosa, se puede recurrir a la intramuscular o a la va oral cuando sea posible. Cualquiera de las combinaciones es igualmente eficaz y segura; la
diferencia es principalmente el costo. En neonatos con hiperbilirrubinemia, no se recomienda la ceftriaxona. Los lactanetes menores de 3 meses con
IVU deben ser tratados con los mismos criterios. Otras edades. El tratamiento se debe guiar de acuerdo al sitio de la infeccin: INFECCIONES
URINARIAS BAJAS: CISTOURETRITIS; El tratamiento de preferencia debe ser por va oral. Pueden utiliozarse amoxicilina, TMP/SFX y nitrofurantoina,
ver cuadro 5. La diferencia en la respuesta clnica a la amoxicilina o a la amoxicilina/clavulanato no es significativa. Los perfiles de sensibilidad de E.
coli a TMP/SMX han mostrado un incremento progresivo en la resistencia. La duracin del tratamiento por va oral debe ser de 5 a 7 das 2. Los
tratamientos menores a 4 das fallan con mayor frecuencia. Tratamientos ms prolongados (mayor de 7 das) no tienen ventaja e incrementan el
riesgo de efectos adversos. INFECCIONES DE VAS URINARIAS ALTAS (PIELONEFRITIS): Menos de 20% de los pacientes requieren hospitalizacincin,
ya que son de mayor edad, toleran la va oral y tienen menos manifestaciones sistmicas. El tratamiento debe iniciarse por va parenteral por tres a
cinco das; debe continuar por va oral, hasta completar diez a 14 das. El cambio de va parenteral a oral se basa en la desaparicin de la fiebre y la
mejora del estado general. Los esquemas de tratamiento incluyen monoterapia con cefalosporinas de tercera generacin, aminoglucsidos,
fluoroquinolonas o la combinacin de un beta lactmico y un aminoglucsido. La elevada frecuencia de resistencia de las enterobacterias a la
amplicilina, TMP/SMX y a las fluoroquinolonas debe tomarse en cuenta en cada regin geogrfica, antes de considerarlos como opciones de
tratamiento para pielonefritis, ver cuadro 6.
CASO CLINICO
Paciente de 14 aos masculino, sin antecedentes personales, ingresa
por sndrome febril de 48 hs. de evolucin, acompaado de
intolerancia oral y dolor abdominal localizado en hipocondrio
derecho y epigastrio, de tipo continuo, con mala respuesta al
tratamiento sintomtico. El paciente ingresa en regular estado
general, impresiona enfermo, febril (39.5C), hemodinamicamente
estable, buena suficiencia cardiorrespiratorio, normohidratado y
taquicardico. Mecnica ventilatoria conservada, buena entrada
bilateral de aire, sin ruidos agregados. Ruidos netos, silencios libres,
bien perfundido, pulsos perifricos presentes y simtricos con
relleno ungueal conservado. El abdomen era blando depresible y
doloroso a la palpacin profunda en epigastrio e hipocondrio
derecho, con puo percusin derecha positiva, sin defensa ni
contractura muscular. Ruidos hidroaereos presentes. Resto del
examen fsico sin particularidades. Refiere diuresis y catarsis
conservada. Laboratorio de ingreso: Leucocitosis moderada a
predominio neutrofilico Glbulos blancos: 18.600 (N: 87/ L: 5)
Glbulos rojos: 4970000 Plaquetas: 162000, Funcin heptica
normal (TGO: 62 TGP:23), Funcin renal normal (Urea: 42 Creatinina:
0.9) Glicemia: 140. VES: 25 mm / hs. Orina completa con sedimento:
densidad 1030, pH 6, protenas (++++), hemoglobina (+++), clulas
(+), regular cantidad de leucocitos y piocitos. Abundantes hemates.
PREGUNTA
Cual es el agente etiolgico mas probable?
RESPUESTA
a.- Klebsiella.
b.- Escherichia Coli.
c.- Proteus.
d.- Peptuscoccus.

CURSO ENARM CMN SIGLO XXI TEL: 36246001

CASO CLINICO
Paciente de 15 aos con historia de 1 semana de decaimiento
progresivo, palidez, nuseas con vmitos ocasionales y cefalea
vespertina intensa. Dos das previos al ingreso se agreg edema
periorbitario, mayor decaimiento y se constat bajo volumen
urinario. Al momento de la admisin al hospital, la paciente
presentaba discreto edema facial y de extremidades inferiores,
palidez de piel y mucosas, presin arterial de 150/105 y frecuencia
cardaca de 82 x'. Entre los exmenes de ingreso destacaban
hematocrito 19,9%, leucocitos 9500 x mm3 (53% neutrfilos),
plaquetas 150.000 x mm3, velocidad de eritrosedimentacin 62
mm/h, protrombina 86%, TTPK 11,7'', nitrgeno ureico 106 mg/dl,
creatininemia 10,1 mg/dl, uricemia 6,9 mg/dl, calcio 7,4 mg/dl,
fsforo 6,7 mg/dl, sodio 136,6 mEq/l, potasio 6,6 mEq/l, cloro 107
mEq/l. Examen de orina: proteinuria 2 g/l, hemates incontables por
campo, cilindros hialinos, creos y gruesos con inclusiones
granulosas numerosos en la preparacin.
PREGUNTA
Cual es la conducta teraputica inmediata mas apropiada para el
caso?
RESPUESTA
a.- Diurtico, antihipertensivo, hemodilisis aguda y biopsia renal.
b.Diurtico,
antihipertensivo,
hemodilisis
aguda
y
metilprednisolona en bolos de 1 g al da.
c.Diurtico,
antihipertensivo,
hemodilisis
aguda,
metilprednisolona y biopsia.
d.- Diurtico, antihipertensivo, metilprednisolona en bolos de 1 g al
da.

Pharmed Solutions Institute

PGINA 386

MANUAL DE TRABAJO DEL CURSO ENARM CMN SIGLO XXI


SINDROME NEFRITICO Y NEFRITICO
CIENCIAS BASICAS: Es la expresin clnica de las glomerulonefritis sobre todo de las agudas, es decir, de un proceso inflamatorio renal con asiento
exclusivo o predominante en el glomrulo. En el desarrollo de dicho proceso, intervienen fundamentalmente los mecanismos de la respuesta
inmune, se caracteriza por edema, hematuria, oliguria e hipertensin arterial, en la mayora de los casos hay hipocomplementemia. Las
glomerulopatas agudas no se manifiestan necesariamente con el sndrome nefrtico. En efecto, pueden tambin presentarse como sndrome
nefrtico, sndrome urmico, hematuria recurrente y, ms an, pueden cursar en forma totalmente asintomtica. El sndrome nefrtico es una
asociacin de hallazgos clnicos y la expresin <<glomerulonefritis>> indica una lesin anatmica. Debe enfatizarse que estos trminos no
representan ninguna definicin etiopatognica de la enfermedad. PATOGENIA: El sndrome nefrtico puede presentarse en ausencia de etiologa
infecciosa; las causas ms frecuentes son lupus eritematoso sistmico, prpura de Henoch-Schnlein, glomerulonefritis por IgA, crioglobulinemia
esencial, sndrome de Guillain-Barr y nefritis pos-radiacin. La incidencia de GLOMERULONEFRITIS AGUDA POSESTREPTOCCICA (descrita en el
tema de glomerulopatias), ha disminuido en varios pases desarrollados, pero an puede considerarse como la causa ms frecuente de sndrome
nefrtico agudo a cualquier edad. Prcticamente en todos los casos existen evidencias de patogenia inmunolgica por formacin de complejos
inmunes, tales como disminucin de los niveles de complemento hemoltico en suero, deteccin glomerular de inmunoglobulinas y complemento de
aspecto granular y periodo de latencia entre la infeccin y las manifestaciones clnicas de lesin renal. La activacin del sistema de complemento
es inducida por la formacin de complejos antgeno-anticuerpo. DIAGNOSTICO: Clnica; edema, es uno de los signos cardinales, habitualmente se
localiza en cara, es matutino, blando, plido (fascies nefrtica), en nios pequeos suele encontrarse anasarca y en nios mayores o adolescentes es
pedio o pretibial. Hipertensin arterial: relevante porque en nios suele ser asintomtica, pero esta es la complicacin ms grave de la enfermedad
ya que puede evolucionar a crisis hipertensiva que aunado al edema agudo pulmonar, son las causas de muerte en la fase aguda. Hematuria: es un
hallazgo universal en el sndrome, es macroscpica hasta 60%, esta es obscura, indolora, total, sin cogulos. En el sedimento urinario hay cilindros
hemticos y eritrocitos dismorficos. Proteinuria: si hay suele ser <40mg/h/m2 SC, encontrndose en un 30-60% de los casos, es una proteinuria no
selectiva. Oliguria: la presencia de esta o de anuria es de mal pronstico, es el signo ms inconstante, cuando se presenta la glomerulonefritis es muy
grave, sobre todo cuando es secundaria a vasculitis, enfermedades autoinmunes, con lesiones de necrosis cortical y obliteracin de los capilares
glomerulares. TRATAMIENTO: Debe estar enfocado a las complicaciones sobre todo de la hipertensin arterial y sus consecuencias como la
encefalopata hipertensiva y la insuficiencia cardiaca, es decir, disminuir la sobrecarga hdrica a travs de diurticos de asa del tipo furosemida dosis
3-5mg/kg/dosis, vigilando frecuentemente el gasto urinario y balance de lquidos, si no hay resultado se debe emplear procedimiento dialtico. Para
coadyuvar al tratamiento de la hipertensin se podr usar vasodilatadores sanguneos del tipo de prazosina 1mg c/8-12hrs o hidralazina a
3mg/kg/da dividido en 3 dosis y ocasionalmente IECAS. El edema y la hipertensin arterial inician su descenso los primeros das del tratamiento y el
sndrome nefrtico desaparece en trminos de 5-10 das, aunque los datos pueden persistir hasta 4 semanas, de tal manera de que si la hematuria
macroscpica y la hipertensin arterial persisten despus de este periodo o bien existe un sndrome nefrtico asociado y sobre todo incremento de
la creatinina srica, son indicaciones precisas de biopsia renal. Los casos en los que se demuestra infeccin estreptoccica, deben recibir tratamiento
de erradicacin a base de penicilina. El esquema teraputico aconsejado es la aplicacin intramuscular de 800,000 unidades diarias de penicilina
procanica durante cuatro das, seguidas de una dosis de 1200,000 U de penicilina se puede emplear eritromicina oral por 10 das a dosis de 30
mg/kg/da. La erradicacin del estreptococo no influye sobre la lesin renal y slo tiene inters epidemiolgico para evitar la diseminacin del
germen. COMPLICACIONES: Sobrecarga hidrosalina, la insuficiencia cardiaca, encefalopata hipertensiva, uremia e hipercaliemia infecciones que
desempean un papel etiolgico en el desarrollo de la glomerulonefritis e infecciones sobreaadidas, insuficiencia renal irreversible. SINDROME
NEFROTICO (SN). CIENCIAS BASICAS: Es la asociacin de proteinuria masiva, hipoalbuminemia y edema y a esta triada se asocia habitualmente
hiperlipidemia, lipiduria y alteracin de la coagulacin y ocasionalmente hay hipertensin arterial, hematuria o falla renal. El sndrome nefrtico es
una de las expresiones clnicas de las glomerulopatias, es decir no hay SN en las tubulopatias o en las uropatas. SALUD PUBLICA: En EUA la
incidencia en nios menores de 16 aos es de 2 por cada 100,000 nios. El 74% de los nios (2-7 aos de edad) presenta sndrome nefrtico con
lesin glomerular mnima. Predomina en varones 2:1. CLASIFICACION: SN primario o idioptico: no se conoce una causa especfica y SN secundario:
LES, amiloidosis, purpura de Henoch-Schonlein, vasculitis necrosante, artritis reumatoide, DM, hipotiroidismo, enf. De Alport, sndrome nefrtico
congnito, enf. De Fabri, neoplasias, infecciones bacterianas (glomerulonefritis posestreptoccica, endocarditis bacteriana, nefritis por shunt, sfilis),
virales, hepatitis, drogas. PATOGENIA: La proteinuria masiva (>40mg/hora/m2 SC): se puede presentar por un dao anatmico o estructural de la
barrera glomerular (endotelio vascular del capilar glomerular, membrana basal glomerular y podocito) entre la sangre y el espacio urinario, lo que se
conoce como proteinuria no selectiva, es decir que en la orina se encuentra albumina, transferrinas y globulinas, habitualmente el dao se hace por
complejos inmunes va la activacin del complemento y bien por anticuerpos contra la membrana basal. Pero tambin puede haber un dao
fisiolgico, o sea, que se pierda la capa electronegativa de esta barrera y esto sucede en las etapas tempranas de la nefropata diabtica,
hipertensin arterial y algunas alteraciones inmunolgicas, la proteinuria de esta caracterstica se le conoce como proteinuria selectiva, es decir,
albuminuria. El edema: es la acumulacin del lquido en espacio intersticial, por alteracin en el equilibrio de las fuerzas de Starling. Hay tres
mecanismos que contribuyen a su formacin: la hipoalbuminemia que contribuye a disminuir la presin onctica del plasma, pero al mismo tiempo
debe haber incremento en la presin hidrosttica, esto hace que el paciente tengan balance positivo de Na y H 2O, adems si el paciente tiene una
lesin glomerular compleja, disminuye la tasa de filtracin glomerular que contribuye a la retencin de lquidos. Hiperlipidemia: se conoce que las
lipoprotenas de densidad baja y muy baja deben metabolizarse en partculas ms pequeas, para ingresar a los tejidos, este paso metablico es
realizado por una enzima lipoproteinlipasa, la cual es activada por un cofactor conocido como ApoII, esta es una pequea protena que se pierde en
la orina de los pacientes son sndrome nefrtico, por otro lado a nivel heptico no se capta las lipoprotenas de baja densidad y del mismo modo que
aumenta la sntesis de albumina, aumenta la de colesterol. Trastorno de la coagulacin: El tromoboembolismo es una complicacin grave del
sndrome nefrtico, es raro en nios, as comn en adolescentes. DIAGNOSTICO: Si se sospecha SN, lo primero es confirmar la proteinuria, de
manera cuantitativa a travs de proteinuria horaria con coleccin de orina de 24 hrs, es masiva cuando el resultado es >40mg/h/m2 de superficie
corporal, por lo difcil de la coleccin y la no indicacin de colocacin de sonda vesical para la coleccin, se ha planteado la realizacin de ndices
urinarios proteinuria/creatininuria >2.5 =proteinuria masiva. En sangre encontramos hipoproteinemia <5g/dl, hipoalbuminemia <2.5g/dl,
hipercolesterolemia, hipertrigliceridemia (elevacin de lipoprotenas de baja y muy baja densidad). BH, aumento de Hb, de Hto y tendencia a la
plaquetosis. Las inmunoglobulinas se encuentran disminuidas. TRATAMIENTO: Enfocado en tratamiento de complicaciones extrarrenales y de las
glomerulopatas. Para edema: reposo relativo, restringir de dieta la ingesta de Na y agua, solo se administrara la cantidad de lquidos por perdidas
insensibles y egresos, se restringen las protenas o se da una dieta normoproteica. El uso de diurticos solos o en combinacin con bloqueadores de
la aldosterona o con el uso de albumina. Furosemida 1-3mg/kg por dosis c/8h. Espironolactona 1-3mg/kg/dia. El uso de albumina humana pobre en
sal tiene indicaciones precisas las cuales son: edema refractario al uso de diurticos, si la ascitis o anasarca intervienen con la mecnica respiratoria o
si el edema representa una deformidad corporal importante. La dosis 0.5-1g/kg/dosis, si hay elevacin de creatinina srica no se debe aplicar este
medicamento. El uso de statinas y los anticoagulantes no estn indicados en pediatra. Los antibiticos sern de acuerdo a infecciones especificas

CURSO ENARM CMN SIGLO XXI TEL: 36246001

Pharmed Solutions Institute

PGINA 387

MANUAL DE TRABAJO DEL CURSO ENARM CMN SIGLO XXI


(de preferencia no aplicar IM). SINDROME NEFROTICO CON LESION GLOMERULAR MINIMA (LGM): Representa la situacin ms frecuente en
pediatra, la edad de presentacin e despus de 1 ao y antes de los 7 aos de edad, casi siempre presentan un sndrome nefrtico puro, es decir,
sin hematuria, sin hipertensin arterial (que son componentes del sndrome nefrtico), y la caracterstica ms importante es que estos pacientes
responden muy bien al uso de esteroides (corticosensible), se debe a una alteracin del equilibrio de linfocitos CD4 y CD8 con predominio de estos
ltimos los cuales liberan IL-2, la cual neutraliza la electronegatividad de la membrana basal glomerular con la proteinuria subsecuente. Tratamiento:
prednisona a 60mg/m2 de superficie corporal, dosis nica por la maana por 4 semanas, posteriormente 40mg/m2/SC das alternos por 4 semanas.
CASO CLINICO
Varn 3 aos, acude por: coluria, edema palpebral maana del
ingreso, abdomen globuloso y edemas de miembros pelvicos,
ganancia de peso, refiere la madre un cuadro catarral hace una
semana con T mx 37.6C, Peso: 19.4 kg Talla: 99 cm Sc: 0.74 m2,
Fc: 94 Fr: 20 T: 36.5C TA: 140/100 (p>95), Edemas pretibiales que
dejan fvea con ligero edema testicular. Hemo: Hemat: 3,68, Hb:
8.8, Hto: 26.2 %, VCM: 71, HCM: 24.4 pg Leucos: 19.680 (S 44%, L
46%, M7%), Plaq: 400.000. Bioq: Gluc: 94, Urea: 69, Creat: 0.68, PT:
6.07, Na: 131, K 5.36, PCR 17.7, Ca: 8.67, P: 5.13. Coag: Tp: 10.9, IQ:
100, Aptt: 26.7, Fibringeno: 420. Gasometra: pH: 7.42, pCO2: 32.5,
HCO3: 21.2, EB: - 3.2. Sedimento orina: Densidad 1020, pH 6.0, Prot
>1000. 200-250 hemat/campo (10% dismrficos), 4-6 leucos/campo,
escasos cilindros hialino-granulosos.
PREGUNTA
Cual de los siguientes factores es la mas adecuada para identificar
la evolucin del padecimiento de base?
RESPUESTA
a.- Diuresis
b.- Densidad de la orina.
c.- Fiebre.
d.- Tension arterial.
CASO CLINICO
Se trata de preescolar que es traido a la consulta debido a la
aparicin brusca de 48 horas de edema facial, asi como de miembros
plvicos edematosos, con escasa miccin urinaria. El nico
antecedente de importancia es un proceso infeccioso en garganta
que refiere la madre, tratato con medidas caseras, los laboratorios
de relevancia fueron los siguientes: Hipoalbuminemia (< 2,5 g/dl).
Proteinuria 42 mg/m2/hora y dislipidemia. A la EF se observa
decaimiento, anorexia, vmitos y dolor abdominal. TA nornal.
PREGUNTA
Cual de los siguientes factores es la mas adecuada para identificar
la evolucin del padecimiento de base?
RESPUESTA
a.- Diuresis
b.- Densidad de la orina.
c.- Fiebre.
d.- Tension arterial
CASO CLINICO
Mujer de 15 aos, con diagnstico realizado hace 6 aos de
Sindrome Nefrotico corticosensible (biopsia compatible con nefrosis
lipodea) en remisin los ltimos tres aos, actualmente en recada.
Consult por cefalea intensa, fotofobia, vmitos y edema
generalizado, realizndose una RM cerebral que mostr trombosis
de los senos venosos sagital superior, transverso y sigmoideo
derechos. No se realiz estudio de coagulacin.
PREGUNTA
Cul es el conducta inmediata mas adecuada a seguir en este caso?
RESPUESTA
a.- Iniciar heparina endovenosa
b.- Iniciar antiinflamatorios
c.- Iniciar analagesicos
d.- Iniciar Esteroides

CURSO ENARM CMN SIGLO XXI TEL: 36246001

PREGUNTA
Cual es la causa mas probable por la la albumina no se filtra a
travs de la membrana basal glomerular?
RESPUESTA
a.- Una combinacin de poros pequeos y carga negativa impiden la
filtracin de la albumina
b.- Una combinacin de poros pequeos y carga positiva impiden la
filtracin de la albumina
c.- La albumina se filtra libremente a travs de la membrana basal,
pero se reabsobe con facilidad a lo largo d ela nefrona
d.- La carga positiva de la membrana basal repele a la albumina
CASO CLINICO
Una nia de 8 aos de edad es llevada a la sala de urgencias por
miccin frecuente, sed constante y coloracin nueva y extraa de
los dientes. Recientemente tuvo un resfriado y su pediatra le
prescribi un antibitico que le dijo que poda daarle los riones.
Nolleva el frasco.
PREGUNTA
Cul de los medicamentos es mas probable que se le haya prescrito
a esta nia?
RESPUESTA
a.- Norfloxacino
b.- Penicilina G
c.- Demeclociclina
d.- Vnacomicina
CASO CLINICO
Masculino de 6 aos 8 meses, estudiante de 1 bsico, inicia con
cuadro clnico de +/- 1 mes de evolucin caracterizado por presentar
alzas trmicas no cuantificadas. Desde hace 3 sem. present tos
productiva con expectoracin, acompaado por dolor de garganta,
disfona, cefalea y mareos. Se evidencia masa palpable en cuello
unilateral izquierdo, doloroso a la palpacin, con calor, rubor. Desde
hace 2 semanas, presenta edema en cara y MI, acompaado de
orinas colricas, ftidas. Adems epistaxis en moderada cantidad.
EF: Paciente en mal estado general, afebril, quejumbroso con palidez
generalizada.PA 155/105 FR:42/min pulso:100 x min T:36.6.
Mucosas: Poco hmedas plidas, ojos: Leve edema en prpados,
conjuntivas plidas, boca: Orofaringe levemente congestiva, piezas
dentarias en regular estado de conservacin. Cuello: Adenopata
unilateral izquierda, dolorosa a la palpacin de ms o menos 4 cm de
, mvil doloroso a la palpacin, ganglios palpables en regin
supraclavicular de ms o menos 2 cm de , dolorosos a la
palpacin.Trax: Corazn: Rtmico, regular, normofontico. Pulmn:
Murmullo vesicular conservado en ambos campos pulmonares.
Abdomen: Blando, depresible, doloroso a la palpacin profunda, en
regin epigstrica. Puopercusin en fosa renal derecha +.
PREGUNTA
Cul es el diagnotico mas probable en ste caso?
RESPUESTA
a.- Glomeruloesclerosis
b.- Sindrome nefrtico/GMN post infecciosa
c.- Uretritis/cistitis
d.- Sindrome nefrtico/GMN post infecciosa

Pharmed Solutions Institute

PGINA 388

MANUAL DE TRABAJO DEL CURSO ENARM CMN SIGLO XXI


GASTROENTERITIS (GEA)
CIENCIAS BASICAS: La GEA suele considerarse como una inflamacin de la mucosa gstrica e intestinal producida por un agente infeccioso o toxico,
que se traduce clnicamente en un cuadro de diarrea de instauracin rpida, generalmente es infecciosa y autolimitada. GEA aguda: Disminucin de
la consistencia de las heces (blandas o lquidas) y/o un incremento en la frecuencia de evacuacin (ms de 3 en 24 horas) con o sin fiebre o vmitos,
de una duracin habitualmente menor a 7 das y nunca superior a 14 das. SALUD PUBLICA: Son la segunda causa de muerte infantil en el mundo, los
ms vulnerables son los menores de 5 aos. PATOGENIA: Etiologa; 1. Virus (rotavirus, virus de Norwalk, otros). 2. Bacterias (S. aureus, Bacilus
cereus, Clostridium perfirngens, difficile y botulinum, Salmonella, Shigella, Campylobacter, listerya monocytogenes, Vibrio cholerae, Yersinia,
Eschericgia coli). 3. Parasitos (Entamoeba hystolitica, Giardia lamblia, Isospora belli). 4. Malaabsorcion (intolerancia a la lactosa, enfermedad celiaca,
fibrosis qustica, intolerancia a protenas de la leche, y/o alimentos especficos). 5. Farmacos (antobioticos, laxantes, quimioterapia, anticidos,
diurticos). 6. Toxinas (hongos o setas, crstaceos, pescados o mariscos, conservadores de alimentos, metales pesados). 6. Otros (mala tcnica
alimenaria, condimentos, alimentacin inadecuada para edad, radioterapia). En Mexico, los agentes que ocasionan hasta 70% de los cuadros
diarreicos son: rotavirus, especies de Shigella, E. coli enterotoxignica, E. coli enteropatgena y Campylobacter. La infeccin se adquiere por la va
oral, a partir de un enfermo, de un portador asintomtico, o de un reservorio animal; con transmisin de forma directa, a travs de alimentos
contaminados o de vectores. Puede aparecer como un caso espordico o en brotes, con mayor frecuencia durante el verano. Los cuadros
espordicos son debidos a cualquiera de los agentes citados anteriormente, pero los brotes suelen ser producidos por Salmonella o por toxinas
estafiloccicas preformadas. Existen tres mecanismos de produccin de las manifestaciones clnicas: .1. Sntesis de toxinas: Que alteran los procesos
de manejo hidroelectroltico celular a travs del AMPc, inhibiendo la absorcin de los iones sodio y cloro, y estimulando la secrecin de cloro y
bicarbonato.2. Invasin directa de la mucosa intestinal: Destruyen el borde en cepillo y las clulas adyacentes, provocando inflamacin local y
ulceracin.3. Mecanismo mixto o no preciso: Por posible adherencia directa o secrecin aumentada de moco. Actan as agentes como Giardia
lamblia y Escherichia coli enteropatgena. DIAGNOSTICO: El objetivo de la valoracin clnica bes la distincin entre los casos leves, benignos y
autolimitados. Debe considerarse segn el tipo de diarrea, perdidas hdricas alcanzan entre 10-200ml/kg; que supone el 20% del peso corporal que
puede constituir una situacin de urgencia. Los factores a identificar para una valoracin correcta son: 1. Gravedad de la enfermedad. 2. Duracin de
la diarrea. 3. Contexto epidemiolgico, en el que aparece el caso. 4. Situacin del husped en cuanto a inmunidad y defensas. En primer lugar se
debe delimitar lo que se considera como diarrea: de acuerdo con los criterios establecidos, evacuaciones de 200g, casi siempre lquidos y aumento
de la frecuencia. En una primera evaluacin sern elegidos como potencialmente graves a los que presenten alguno de los siguientes signos:
deshidratacin importante con sensacin de sed o reduccin de diuresis; diarrea sanguinolenta; dolor abdominal o fiebre elevada. En la exploracin
fsica se consideran como datos que confirman la sospecha de severidad establecida en la anamnesis, los siguientes; obnubilacin o disminucin del
nivel de consciencia, sequedad de mucosas, hipotensin ortostatica, en la exploracin del abdomen, la presencia de resistencia, dolorimiento o
signos de peritonitis. Se consideran criterios de hospitalizacin; diarrea hemorrgica, signos de toxicidad sistmica (obnubilacin), fiebre alta,
deshidratacin severa, distensin abdominal. Por otra parte debe considerarse que algunas infecciones por Salmonella, Shigella, Yersinia y E. coli
enterohemorrgica, pueden complicarse con afectacin de otros rganos o sistemas: sndrome hemoltico urmico, PTT, Sndrome de Reiter,
pericarditis, glomerulonefritis. En cuanto a la duracin de la diarrea, la persistencia de la sintomatologa durante 4-5 das es un rasgo de severidad,
por asociarse a microorganismos ms virulentos. Pertenecen a grupos de alto riesgo los menores de 5 aos, por la situacin inmunitaria y defensas
del husped, para cuya atencin y proteccin se han elaborado normas y programas en diversas partes del mundo, sobre todo en los pases en vas
de desarrollo como Mxico. SIGNOS DE PELIGRO: No puede beber o amamantarse, vomita todo, tienen convulsiones, esta letrgico o inconsciente,
muestra ausencia o disminucin de los ruidos intestinales. TRATAMIENTO: Factores de mal pronstico: 1. Menor de 2 meses. 2. Desnutricin
moderada a grave. 3. Presencia de inmunodeficiencia. 4. Muerte de un menor de 5 aos en la familia. 6. Madre analfabeta o menor de 17 aos. 7.
Dificultad para el traslado al mdico si se agrava el nio. La enfermedad es casi siempre autolimitada y la mortalidad se relaciona con las
complicaciones, en las que la deshidratacin es la causa de muerte en el 70% de los casos. El adecuado manejo de la deshidratacin con VSO evita
90% de estas muertes, por eso las acciones contra la diarrea incluyen este recurso, impulsando su uso en los casos. Par el tratamiento es necesario
evaluar el estado de hidratacin del paciente, por lo que se deber considerar las siguientes definiciones operacionales. CASO SIN DESHIDRATACION:
Es aquel que presentan menos de cuatro evacuaciones liquidas en 24h, ausencia de vmito, sin signos clnicos de deshidratacin. CASO CON
DESHIDRATACION: Es aquel que presenta dos o ms de las manifestaciones clnicas siguientes: Inquieto o irritable, ojos hundidos, llanto sin lgrimas,
boca y lengua seca, saliva espesa, respiracin rpida, sed amentada, bebe con avidez, elasticidad de la piel, > a 2 seg., pulso rpido, llenado capilar
de 3-5seg., fontanela anterior hundida (lactantes). CASO CON CHOQUE HIPOVOLEMICO: Es aquel que presenta dos o ms de las manifestaciones
siguientes: inconsciente o hipotnico, no puede beber, pulso dbil o ausente, llenado capilar mayor de 5 seg. De acuerdo con la valoracin, el
manejo de los pacientes que presentan algn grado de deshidratacin se basa en 3 planes. PLAN A: Para pacientes con enfermedad diarreica sin
deshidratacin con atencin en el hogar. Continuar con alimentacin habitual, aumentar la ingestin de lquidos de uso cotidiano en el hogar, Sueros
de VSO, en los menores de 1 ao, ofrecer media taza (75ml) y en lo mayores de un ao una taza (150ml) y administrarlo a cucharadas o mediante
sorbos pequeos despus de cada evacuacin. Capacitar a la madre para reconocer los signos de deshidratacin y datos de alarma por
enfermedades diarreicas (sed intensa, poca ingestin de lquidos y alimentos, numerosas heces liquidas, fiebre, vmito y sangre en las evacuaciones)
con el propsito de que acuda nuevamente a solicitar atencin medica en forma oportuna. PLAN B: Pasar pacientes con diarrea y deshidratacin con
atencin en la unidad de salud; administra VSO 100ml/kg, en dosis fraccionada cada 30min durante 4 hrs por va oral. Si el paciente presenta vmito,
esperar 10 minutos e intentar otra vez la hidratacin oral ms lentamente. Al mejorar el estado de hidratacin, pasar a plan A. En caso contrario
repetir el plan B en 4 hrs, de no existir mejora pasar al plan C. PLAN C: Pacientes con choque hipovolmico por deshidratacin. Iniciar
inmediatamente administracin de lquidos por va IV, con solucin de Hartmann, si no se encuentra disponible, use sol. Salina al 0.9%, de acuerdo
con el siguiente esquema: Primera hora 50ml/kg, segunda hora 25ml/kg, tercera hora 25ml/kg. Evaluar al paciente continuamente. Si no mejora
aumentar la velocidad de infusin. Cuando pueda beber (usualmente 2-3hrs), administrar sobres de VSO, a dosis de 25ml/kg/h, mientras sigue
lquidos IV. Al completar la dosis IV, evaluar al paciente para seleccionar plan A o B, y retirar venoclisis, o repetir plan C. Si selecciona el plan A,
observar durante 2 hrs para asegurarse de que el responsable del paciente pueda mantenerlo hidratado con sobres de VSO, adems alimentarlo en
su domicilio. ANTIMICRONIANOS: No son tiles en el tratamiento de las enfermedades diarreicas en 90% de los casos, el manejo inadecuado de
antimicrobianos puede propiciar que la enfermedad se prolongue y ocasione resistencia bacteriana. Los antimicrobianos debe prescribirse en
pacientes con diarrea por: especias de Shigella, Vibrio cholerae, trofozoitos de Entamoeba histolytia o Giardia lamblia. Cuando el paciente presenta
fiebre, se deben utilizar medios fsicos para su control, hidratar al paciente, mantenerlo con ropa ligera y de ser necesario, darle un bao con agua
fra. Puede emplearse un antipirtico como acetaminofn a dosi de 30-40kg/dia en nios de 2 meses a 4 aos de edad, repartidos cada 4-6 hrs, sin
pasar de 5 en 24 hrs. COMPLICACIONES: Choque hipovolmico, desequilibrio hidroelectroltico, acido-base, ileo paralitico, diarrea persistente,
insuficiencia renal, neumatosis intestinal, infarto intestinal, perforacin intestinal, peritonitis. PREVENCION: Vigilancia de la cloracin del agua,
procedimientos de desinfeccin (potabilizcion, ebullicin, cloracin, yodacin), promocin de la lactancia materna (exclusiva los primeros 4-6 meses

CURSO ENARM CMN SIGLO XXI TEL: 36246001

Pharmed Solutions Institute

PGINA 389

MANUAL DE TRABAJO DEL CURSO ENARM CMN SIGLO XXI


de edad), mejorara las prcticas de ablactacin (a partir del 4to mes), promocin de la higiene en el hogar (lavado de manos, eliminacin adecuada
de excretas, manejo adecuado de heces en nios), vacunacin contra el sarampin, complemento con vitamina A.
CASO CLINICO GASTROENTERITIS
Un nio de 4 aos de edad, que inicia hace 2 dias con dolor difuso
abdominal con 5 evacuaciones semiliquidad, niega fiebre, nauseas o
vomito. A la exploracin fsica se observa alerta y reactivo, signos
vitales dentro de rango, incremento de movimientos intestinales y
se ausculta periltaltismo incrementado, las mucosas orales se
observan hidratadas, su llanto contiene lgrimas, se encuentra
irritable, como antecedente se encuentra en tratamiento para
desnutricin y la familia es de escasos recursos pero sus
inmunizaciones se encuentran al corriente.
PREGUNTA
Cual es la conducta a seguir?
RESPUESTA
a.- Solucion de rehidratacin oral.
b.- Antibioticos profilctico.
c.- Envio a casa con SRO.
d.- Proporcionar datos de alarma.
CASO CLINICO
Lactante de 9 meses, sin antecedentes patolgicos, ni alergias
conocidas, presenta irritabilidad con vmitos y diarreas de 12h de
evolucin, acompaados de fiebre de hasta 38,5. Ha realizado 10
deposiciones lquidas abundantes, no ftidas con moco y 5 vmitos,
el ltimo en la sala de espera. Exploracin fsica: algo decado,
plido, llora sin lgrimas, tiene la mucosa oral seca y los ojos
hundidos. El tiempo de recapilarizacin cutnea es de 3 segundos.
FC de 120 x, auscultacin cardaca normal, pulsos perifricos
bilaterales presentes y simtricos, aparato respiratorio normal,
abdomen: algo distendido y doloroso de forma difusa, no se palpan
masas, ni visceromegalias, no presenta signos de irritacin
peritoneal y el peristaltismo est aumentado. Exploracin
neurolgica: activo y reactivo, fontanela normotensa. Se realiza una
analtica sangunea que muestra los siguientes valores: Hemograma:
Hb 11,2 g/dl. HTC 40%. Plaquetas 256.000/mmcc., leucocitos 15.600
(60%l, 9%M, 20%n). PCR 36 mg/dl. Gasometra venosa: pH 7,19,
PCo2 46 mmHg, Po2 50 mmHg, Bic 11,3 mEq, Exceso de bas 12,4. Na
135 mmol/l, K 4,3 mmol/l, Ca 1,12 mmol/l, Cl 106 mmol/l. Glicemia
78 mg/dl y una radiografa abdominal en la que se observan
imgenes compatibles con dilatacin de asas intestinales y niveles
hidroareos.
PREGUNTA
Cul es el agente etiolgico ms probable en este caso?
RESPUESTA
a.- Salmonella
b.-Rotavirus
c.- E.coli
d.- Campylobacter

CASO CLINICO
Lactante varn de 3 aos de edad, nacido a trmino, hasta el
momento sano, con percentiles 50 de peso y 50 de talla. Desarrollo
psicomotor normal. Encontrndose previamente bien, comienza con
llanto intermitente de varias horas de duracin, dolor a nivel
abdominal con diarrea verdosa conteniendo restos de sangre y
abundante mucosidad, fetida. Pico febril de 38.3 C. EF: Peso 6460
g. Talla 61 cm. No se aprecian exantemas ni petequias. No rigidez de
nuca, signos menngeos negativos. Cardiopulmonar sin alteraciones.
Abdomen blando depresible con dolorimiento a la palpacin.
Laboratorios: Leucocitos: 21900 (73% N, 20% L); Hb 11 g/dl. Hcto:
32.7%; Plaq. 477000. Sedimento urinario normal. BQ: 82 mg/dl,
BUN: 7 mg/dl.
PREGUNTA
Cul es el agente etiolgico ms probable en este caso?
RESPUESTA
a.- Campylobacter jejuni
b.- Escherichia coli
c.- Salmonella tiphy
d.- Giardia lamblia
PREGUNTA
Cul es la conducta teraputica ms adecuada a seguir en este
caso?
RESPUESTA
a.- Eritromicina 30mg/kg/da por 5 das
b.- TMP-SFX 10mg/kg/da por 5 das
c.- Ampicilina 25 mg/kg/da por 5 das
d.- Metronidazol 15mg/kg/ da por 5 das
PREGUNTA
Cul es la prdida de peso, para pensar en una deshidratacin
severa?
RESPUESTA
a.- <3%
b.- 6%
c.- >9%
d.- 7%
CASO CLINICO
Un nio de 2 aos de edad, que inicia hace 2 das con dolor difuso
abdominal con 5 evacuaciones semiliquidas, niega fiebre, nuseas o
vomito. A la exploracin fsica se observa alerta y reactivo, signos
vitales dentro de rango, incremento de movimientos intestinales y
se ausculta periltaltismo incrementado, las mucosas orales se
observan hidratadas, su llanto contiene lgrimas, se encuentra
irritable, como antecedente se encuentra en tratamiento para
desnutricin y la familia es de escasos recursos pero sus
inmunizaciones se encuentran al corriente.

PREGUNTA
Cul es la conducta ms adecuada a seguir en este momento?
RESPUESTA
a.- Antiviral y hospitalizacion
b.- Iniciar antibitico y vida suero oral
c.- Hospitalizacion y vigilancia 6h
d.-En domicilio con antobioticoterapia
PREGUNTA
Qu complicacin es la menos probable que se presente en este
paciente?
RESPUESTA
a.- Desequilibrio hidroelectroltico
b.- Intolerancia a los azcares
c.- Enterocolitis necrotizante

CURSO ENARM CMN SIGLO XXI TEL: 36246001

d.- Edema agudo pulmonar

PREGUNTA
Cul es la conducta teraputica ms adecuada a seguir?
RESPUESTA
a.- Solucin de rehidratacin oral.
b.- Antibiticos profilctico.
c.- Envi a casa con SRO.
d.- Proporcionar datos de alarma
PREGUNTA
Cul es el agente causal ms probable para este caso?
RESPUESTA
a.- Escherichia coli

Pharmed Solutions Institute

PGINA 390

MANUAL DE TRABAJO DEL CURSO ENARM CMN SIGLO XXI


b.- Giardia lamblia
c.- Rotavirus
d.- Staphylococcus aureus
PREGUNTA
Con que cantidad de evacuaciones se considera como diarrea en
este paciente?
RESPUESTAS
a.- 5-10 g/kg/dia
b.- 10-15 g/kg/dia
c.- 15-20 g/kg/dia
d.- 20-25 g/kg/dia
CASO CLINICO
Femenino de 3 aos de edad la cual inicia 3 das previos a su ingreso,
la madre refiere diarrea con moco y sangre, as como fiebre alta no
cuantificada, esto se acompa de rinorrea y dificultad para conciliar
el sueo, a la exploracin fsica se observa, decada con aspecto
deteriorado y respuesta alterada irritable, signos vitales taquicardico
y taquipnea, se observa disminucin de la turgencia de la piel, se
ingresa a urgencias para tratamiento.
PREGUNTA
Cul es el volumen recomendado de SRO para este caso?
RESPUESTA
a.- SRO 30 ml/kg durante 4 hrs.
b.- SRO 50 ml/kg la 1era hora luego 30 ml/Kg.
c.- SRO mayor a 50 ml/kg durante 4 horas.
d.- Plan de hidratacin intravenoso
PREGUNTA
Cul es el agente etiolgico menos probable para este caso?
RESPUESTA
a.- E. coli enteroinvasiva
b.- Shigella dysenteriae
c.- E. coli enterotxigenica
d.- Campilobacter jejuni
PREGUNTA
Cul de los siguientes parmetros no esperaramos encontrar en un
coproparasitoscopico normal?
RESPUESTAS
a.- pH 8
b.- Azucares reductores <500mg
c.- Leucocitos <5 por campo
d.- Moco negativo
CSO CLINICO
Un nio de 9 aos de edad fue hospitalizado debido a dolor agudo
abdominal y vmitos. Fue diagnosticado inicialmente con
apendicitis. El da 3 despus de la admisin, aunque estaba
totalmente alerta y orientado, su examen se caracteriz por
empeoramiento presentando disfona, boca seca, ojos secos,
estreimiento, abdomen timpnico, retencin urinaria, taquicardia,
taquipnea y disnea.
PREGUNTA
Qu grado de deshidratacin presenta este paciente?
RESPUESTA
a.- Normal
b.- Deshidratacin leve
b.- Deshidratacin moderada
c.- Deshidratacin severa

CURSO ENARM CMN SIGLO XXI TEL: 36246001

PREGUNTA
Cul es la conducta teraputica ms adecuada a seguir?
RESPUESTA
a.- SRO 30 ml/kg durante 4 hrs.
b.- SRO 50 ml/kg la 1era hora luego 30 ml/Kg.
c.- SRO mayor a 50 ml/kg durante 4 horas.
d.- Plan de hidratacin intravenoso
PREGUNTA
Cul es el agente etiolgico ms probable en este caso?
RESPUESTA
a.- Giardia lamblia
b.- Campilobacter jejuni
c.- Entamoeba histolytca
d.- Salmonella tiphy

CASO CLINICO
A 16 meses de edad de sexo masculino present a la sala de
urgencias despus de tener dificultad para respirar y cianosis
peribucal en casa. Su enfermedad comenz tres semanas antes, con
diarrea sin fiebre, varios miembros de la familia tambin estaban
enfermos. Aunque la diarrea del paciente paro, continu hipoactivo
y prefiere gatear a caminar. Cuatro das antes, desarroll fiebre y en
el da de la admisin sus padres notaron repentina dificultad para
respirar y color azulado alrededor de los labios. Su temperatura era
de 40,5 C, frecuencia cardaca de 180 latidos / minuto, la
respiracin 62/minuto.
PREGUNTA
Cual es su conducta ms importante a seguir?
RESPUESTA
a.- Iniciar amoxicilina ms albendazol y paracetamol.
b.- Manejo de la fiebre por medios fsicos, observacin y signos de
alarma.
c.- Envio a segundo nivel, previo manejo de temperatura.
d.- Manejo de fiebre por medios fsicos ms paracetamol,
amoxicilina, albendazol y laboratorios.
PREGUNTA
El paciente presenta los siguientes datos 6 horas despus: saturacin
de oxgeno del 96%, disminucin del murmullo vesicular en la base
del pulmn izquierdo. Clulas blancas de 12,000, con neutrfilos
segmentados 58%, 13%, linfocitos, monocitos 15% 6%, 2% y
eosinfilos. El hematocrito fue del 37,5%, plaquetas 48,000,
velocidad de sedimentacin globular de 33 mm / hora y la protena C
reactiva 57,4 mg / dL.
PREGUNTA
Considerando los resultados de laboratorios y gabinete cual de las
siguientes patologas es ms probable?
RESPUESTA
a.- Parasitosis y neumona en paciente con factores de riesgo.
b.- Neumonia adquirida en la comunidad.
c.- Neumonia sin relacin a la parasitosis previa.
d.- Procesos independientes en paciente con inmunocompromiso.

Pharmed Solutions Institute

PGINA 391

MANUAL DE TRABAJO DEL CURSO ENARM CMN SIGLO XXI


PARASITOSIS
CIENCIAS BASICAS: Los parsitos intestinales afectan con ms frecuencia a los nios que a los adultos. Los viajes, la inmigracin y la adopcin
internacional han hecho que su incidencia aumente en los ltimos aos. Aunque los sntomas clnicos con frecuencia son inespecficos, algunos datos
de la anamnesis pueden orientarnos en la prediccin del microorganismo causante. DIAGNOSTICO: En general es preciso recoger tres muestras de
heces en das diferentes para tener una mayor seguridad en la deteccin
de los parsitos. scaris lumbricoides (nematodo): Al ingerir los huevos
infectivos, las larvas inician un ciclo en el organismo (circulacin,
pulmones, trquea, faringe, intestino). Los gusanos adultos de A.
lumbricoides viven en el intestino delgado. Diagnstico. Examen
parasitolgico de las heces donde se visualizan huevos del parsito.
Clnica, durante la migracin pulmonar: neumonitis generalmente
asintomtica en ocasiones sndrome de Lffler (infiltrados pulmonares
transitorios, fiebre, disnea, tos, sibilancias y eosinofilia). Fase intestinal:
generalmente asintomtica. Sntomas disppticos, diarrea. Raramente,
obstruccin intestinal y vlvulo. Posibilidad de colecistitis, colangitis,
absceso heptico, apendicitis, diverticulitis o pancreatitis por migracin
del parsito. Tratamiento. Mebendazol (100 mg/12 h/3 das o 500 mg a
dosis nica). Alternativas: pamoato de pyrantel, 10 mg/kg (mximo 1 g),
dosis nica; albendazol, 400 mg, dosis nica (nios entre 2-5 aos: 200
mg); piperazina, 75 mg/kg (mximo 3,5 g en adultos o 2,5 g en nios
entre 2-12 aos). La ivermectina tambin es eficaz. (200 g/ g dosis
nica). Enterobius vermicularis (oxiuro, nematodo): El vermes adulto
vive en el ciego y en las zonas adyacentes. La hembra migra hacia la zona
anal para hacer la puesta. Diagnstico. El examen parasitolgico de
heces slo es positivo en un 10-15% de los casos en los que se
identifican los huevos del parasito o gusano adultos. Mejores resultados
tiene el examen de cintas adherentes transparentes despus de su
aplicacin en los mrgenes del ano. En caso de exmenes repetidamente negativos y alta sospecha de la infeccin, realizar tacto rectal para la
obtencin de muestra fecal. Clnica: Asintomtico, prurito anal y perineal de predominio nocturno. En casos excepcionales, colitis, dolor abdominal y
granulomas peritoneales. Puede provocar vulvovaginitis en mujeres jvenes. Epidemiologia y profilaxis: Cosmopolita, transmisin mediante ropa y
sabanas contaminadas. Despus de ser ingeridos, los huevos de E. vermicularis liberan las larvas en el duodeno y migran hacia el leo. Medidas
higinicas bsicas: lavarse las manos despus de la defecacin. Se recomienda dar el tratamiento a toda la familia para interrumpir la transmisin.
Strongyloides stercoralis (nematodo): Los gusanos adultos de S. stercoralis viven en el intestino delgado. Tiene capacidad de autoinfestacin, base
de la larga persistencia en el organismo infectado (dcadas). Diagnstico. Examen parasitolgico de las heces. Cultivo para estrongiloides. Produce
hipereosinofilia perifrica (puede faltar en personas inmunodeprimidas). Demostracin de larvas en el aspirado duodenal obtenido mediante
fibrogastroscopia. Clinica durante la fase pulmonar, las larvas migratorias, pueden ocasionar infiltrados eosinonoflicos y tos (sndrome de Leffler).
Manifestaciones gastrointestinales: dispepsia, dolor abdominal, diarrea intermitente. Puede producir un sndrome de malabsorcin. Capaz de
autoinfestaciones. En personas inmunodeficientes puede producir un Sindrome de hiperinfestacin de pronstico grave y elevada mortalidad. Puede
producir manifestaciones cutneas: larva currens. Las personas se infectan cuando las larvas filariformes del nematodo penetran a travs de la piel,
en contacto con el suelo infectado. Despues de un ciclo dentro del organismo humano (piel, circulacin, pulmones, trquea, faringe), los gusanos
adultos viven en el intestino. Entamoeba hystolitica (protozoo): Es un patgeno, con capacidad invasiva y de diseminacin extraintestinal. En el
intestino afecta al intestino grueso. Diagnstico. Examen de parsitos en heces (visualizacin de trofozoitos y/o quistes); Deteccin de antgeno en
heces. Coprocultivo en medios especiales. Serologa. Examen anatomopatolgico de biopsia intestinal. Difcilmente se visualiza E. histolytica en el
pus de los abscesos hepticos, el cual tiene una coloracin amarronada (pasta de anchoas). Pruebas de imagen (ecografa, TC). La amibiasis puede
manifestarse como cuadro diarreico; molestias abdominales inespecficas;
disentera amebiana; colitis fulminante con desarrollo de megacolon txico;
ameboma. Puede afectar a zonas extraintestinales, siendo el absceso
heptico amebiano la localizacin ms frecuente que puede extenderse a
zonas contiguas (pleura, pulmn, pericardio, etc.); otras afectaciones son
mucho menos frecuentes (cerebrales, cutneas, genitales). Hay personas
asintomticas. Indicaciones quirrgicas. En casos de megacolon txico y
perforacin. Puncin aspirativa transcutnea de los abscesos amebianos,
slo indicado en aquellos abscesos de gran tamao con riesgo de rotura
hacia otras estructuras adyacentes. Giardia lamblia (protozoo): Produce
colonizacin del intestino delgado, con adherencia del parsito a la mucosa
intestinal. Diagnstico. Identificacin del parsito en las heces. Deteccin
de antgeno en heces (ELISA). Clnica. Asintomtica. Dispepsia, diarrea
crnica intermitente. Sndrome de malabsorcin sobre todo en pacientes
inmunodeprimidos (sida, dficit de IgA). Taenia saginata (cestodo): Reside
en el intestino delgado del husped. Diagnstico. Identificacin de las
progltides en las heces de los pacientes. No puede diferenciarse de la T. solium excepto con el recuento de las ramas uterinas de las progltides
grvidas. Clnica. Asintomtica en la mayora de los casos; sntomas disppticos. Anorexia, urticaria, prurito, cefalea, convulsiones. De forma inusual
pueden producirse complicaciones por la migracin de progltides a lugares como apndice, conducto biliar o pancretico. Epidemiologa.
Cosmopolita. Transmisin oral a partir del consumo de carne cruda (o poco cocida) de ganado vacuno contaminada. Taenia solium (cestodo): Vive
en el intestino delgado del husped. Clinica y diagnostico igual que T. saginata, La cisticercosis, una enfermedad que puede afectar al SNC y tambin
al globo ocular, es una de las complicaciones de la T. solium. Trichuris trichiura (nematodo): Los parsitos adultos de T. trichiura viven en el colon y
el recto, adheridos a la mucosa intestinal. Criterios diagnsticos. Examen parasitolgico de las heces, donde se visualizan los huevos del parsito.
Clnica. Puede ser una infeccin asintomtica, provocar molestias abdominales inespecficas; diarreas, colitis y prolapso rectal en nios. En los casos

CURSO ENARM CMN SIGLO XXI TEL: 36246001

Pharmed Solutions Institute

PGINA 392

MANUAL DE TRABAJO DEL CURSO ENARM CMN SIGLO XXI


de hiperinfestacin pueden provocar disentera. Trichinella spiralis (nematodo): La triquinosis es una enfermedad que se produce por la ingestin
de quistes de triquina en carne contaminada. A partir de ella se liberan larvas que penetran en la mucosa intestinal, y se desarrollan hasta el estadio
adulto (30-40h). A los 5 dias comienza la fase larviposicion, donde las larvas son liberadas en la mucosa y atreves de los linfticos acceden a la
circulacin general.
RESPUESTA
CASO CLINICO GIARDIASIS
a.- Enterobius vermiculares.
Nia de 9 aos de edad que presenta edema facial de predominio
b.- Ascaris lumbricoides.
matutino de 3 meses de evolucin, sin lesiones de urticaria ni otra
c.- Trichuris trichiura.
sintomatologa acompaante. En la anamnesis realizada no se
d.- Ancylostoma duodenale.
encontr relacin con alimentos, frmacos ni otros posibles agentes
PREGUNTA
desencadenantes, ni tampoco antecedentes personales ni familiares
Considerando la presencia de parsitos en tubo digestivo alto se
de inters alergolgico. En la exploracin fsica realizada destacaba
incrementa el sndrome de loeffler. Cul no es una manifestacin
un ligero sobrepeso y un edema en surcos nasogenianos.
que lo orienta?
RESPUESTA
PREGUNTA
a.- Prurito nasal.
Cual es la conducta a seguir mas adecuada?
b.- Parsitos en heces negativo.
RESPUESTA
c.- Rx de trax con infiltrados intersticiales.
a.- Iniciar antihistamnico, previa administracin de corticoide oral.
d.- Rectificacin nasal
b.- Solicita BH, QS, EGO.
c.- Solicita pruebas cutaneas, anticuerpos y perfil tiroideo.
CASO CLINICO
d.- Envia paciente a tercer nivel.
Una nia de 4 aos de edad con dolor abdominal y distensin desde
hace 6 das, segn su madre. Ella tambin tena antecedentes de
PREGUNTA
vmitos desde 3 das. El dolor de inicio gradual, clicos, seguido por
El paciente presento una leve mejora, sus estudios presentaron:
distensin abdominal y aumento en la intensidad del dolor y los
anlisis de orina y de sangre (hemograma, bioqumica general,
vmitos. El vmito era de color verdoso y contena partculas de
hormonas tiroideas, estudio de complemento, IgA, IgG e IgM) fueron
alimentos no digeridos. El nio fue vacunado parcialmente. En la
normales. Los anticuerpos antinucleares y antitiroideos fueron
exploracin fsica estaba mal alimentado con palidez, temperatura
negativos. La cifra de IgE total fue de 9 U/ml. Las pruebas cutneas
oral fue de 37,7 C, la presin arterial era de 98/60 mmHg, pulso
para hipersensibilidad inmediata con una batera estndar de
regular, con una frecuencia de 110 latidos por minuto, frecuencia
neumoalrgenos habituales de la zona, fueron negativas. Cual es la
respiratoria de 34 por minuto. El examen abdominal revel un
conducta a seguir?
abdomen distendido de todos los cuadrantes. En la auscultacin,
RESPUESTA
ruidos intestinales hiperdinmico eran audibles. Examen rectal
a.- Coproparacitoscopico.
revel heces con sangre y moco asi como varios gusanos vivos.
b.- Indica antihistaminico.
c.- Indica metronidazol
PREGUNTA
d.- Indica albendazol.
Considerando las manifestaciones. Cual es el agente etiolgico ms
probable asociado?
CASO CLINICO
RESPUESTA
Nio de 11 meses de edad con antecedentes de 1 da de
a.- Entamoeba histolytica.
movimientos lentos. Evacuaciones sin sangre, asociados con 10-15
b.- Giardia Intestinalis.
episodios de vmito y diarrea en 4 ocasiones. Se observa letrgico y
c.- Enterobius vermicularis.
con disminucin de uresis. Fue hidratado y se le dio va ceftriaxona y
d.- Dientamoeba fragilis.
metronidazol. El nio tuvo un episodio de vmitos donde encontr
un gusano. En el examen, pareca aburrido, con deshidratacin
PREGUNTA
moderada. Tena un pulso de 137, una frecuencia respiratoria de 28
Considerando el agente etiolgico que considero ms probable en el
por minuto, y una temperatura de 37 C. Su estatura era superior al
caso anterior. Cul es la conducta teraputica a seguir?
percentil 95, su peso estaba en percentil 5.
RESPUESTA.
a.- Iodoquinol 30-40 mg/kg/da vo (mx 2 g) en 3 dosis x 20 da,
PREGUNTA
albendazol 15 mg/kg/da vo x 5-7 das (max 400 mg).
Cul es el estado nutricional?
b.- Metronidazol 5 mg/kg vo tid (mx 750 mg/da) x 5 das,
RESPUESTA
nitazoxanida 200 mg x 3 das.
a.- Normal
c.- Tetraciclina 40 mg/kg/da (mx 2 g) vo en 4 dosis x 10 das,
b.- Desnitricin leve
metronidazol 20-40 mg/kg/da vo en 3 dosis x 10 das.
c.- Desnutricin moderada
d.- Albendazol 15 mg/kg/da vo x 5-7 das (max 400 mg),
d.- Desnutricin severa
metronidazol 5 mg/kg vo tid (mx 750 mg/da) x 5 das.
PREGUNTA
El paciente mejora su estado hidrioelectrolitico con SRO. Cul es
manejo farmacolgico mas adecuado?
RESPUESTA
a.- Albendazol.
b.- Mebendazol.
c.- Nitaxozanida.
d.- Metronidazol
PREGUNTA
Considerando el cuadro clnico. Cul de los siguientes helmitos
intestinales es ms probable?

CURSO ENARM CMN SIGLO XXI TEL: 36246001

PREGUNTA
El paciente presenta los siguientes datos 6 horas despus: saturacin
de oxgeno del 96%, disminucin del murmullo vesicular en la base
del pulmn izquierdo. Leucocitos 12,000, con neutrfilos
segmentados 58%. El hto 37,5%, plt 48,000, velocidad de
sedimentacin globular de 33 mm / hora y la PCR 57,4 mg /dL. Cul
de las siguientes patologas es ms probable?
RESPUESTA
a.- Parasitosis y neumona en paciente con factores de riesgo.
b.- Neumona adquirida en la comunidad.
c.- Neumona sin relacin a la parasitosis previa.
d.- Procesos independientes en paciente con inmunocompromiso

Pharmed Solutions Institute

PGINA 393

MANUAL DE TRABAJO DEL CURSO ENARM CMN SIGLO XXI


DESHIDRATACION, LIQUIDOS Y ELECTROLITOS
CIENCIAS BASICAS: Para fines de terminologa mdica, deshidratacin se define como el estado clnico consecutivo a la perdida de lquidos y solutos
en el cuerpo humano. Sin embargo, es posible encontrar deplecin corporal de agua sin prdida de solutos, de causas diversas, sin denominarse
deshidratacin. SALUD PUBLICA: La primera
causa de deshidratacin en el mundo es la
diarrea aguda con mil millones de episodios
anuales y ms de 2.5 millones de muertes
secundarias a deshidratacin. En Mxico la
tasa de mortalidad en 2005 fue de 21.6 por
100 000 habitantes en menores de 5 aos.
PATOGENIA: las principales causas de
deshidratacin, estn determinadas por dos
mecanismos: 1) Incremento en las prdidas
(Intestinales; vmitos, diarrea, sondas,
fistulas
intestinales.
Extraintestinales;
quemaduras, uso de diurticos, diuresis,
osmtica, poliuria, fiebre). 2) Falta de
aporte (por va oral, por vas parenterales).
Otras causas de deshidratacin son
cetoacidosis diabtica, diabetes inspida,
estrs postquirrgico y privacin de agua. La
distribucin de lquido en el cuerpo est
determinada por la edad. En el recin
nacido el lquido corporal total es de 7075%, pero va disminuyendo conforme avanza la edad hasta ser de 60% en el adulto. El lquido corporal total est distribuido a su vez en los espacios
intracelular y extracelular; este ltimo est conformado por el espacio intersticial y el espacio intravascular. La prdida de lquidos produce diferente
dficit en los compartimentos de los espacios extracelular e intracelular. En la deshidratacin aguda (menos de 2 das), la perdida de lquidos en su
mayora es a expensas del lquido extracelular (75%); mientras que en la deshidratacin prolongada, la perdida de lquidos es aproximadamente la
misma en ambos espacios. La osmolaridad plasmtica est dada por los diferentes solutos, de acuerdo con la siguiente frmula: Osmolaridad =
2(Na+ mEq/L) + (NUS en mg/dl)/2.8 + (glucosa en mg/dL)/18. La osmolaridad del plasma se modificara dependiendo de la causa de la deshidratacin
y del mecanismo de sta; por ejemplo, en pacientes con deshidratacin por diarrea aguda, puede haber mayor prdida de lquido que de solutos
(gastroenteritis por rotavirus) o mayor prdida de solutos que de lquido (clera); con base en lo anterior se produce deshidratacin isosmolar,
hiposmolar
o
hiperosmolar.
DIAGNOSTICO:
El
diagnostico
de
deshidratacin se establece mediante un
minucioso interrogatorio y examen fsico.
Sin embargo, el mdico o personal de
salud de primer contacto debe estar
entrenado para detectar datos clave.
Durante la valoracin de un paciente
deshidratado se debe poner especial
nfasis en tres aspectos: 1) Establecer el
grado o severidad de la deshidratacin. 2)
Determinar el tipo de deshidratacin, as
como sus complicaciones concomitantes.
3) Planear la forma de rehidratacin. Una
revisin sistematizada demostr que los
tres datos clnicos ms importantes para
identificar deshidratacin son: llenado
capilar prolongado, turgencia de la piel y
patrn respiratorio. Determinacin del tipo de deshidratacin. El tipo de deshidratacin est dado principalmente por la cuantificacin srica de Na+:
1) Deshidratacin isotnica (isonatrmica), con Na+ srico entre 130 y 150 mEq/L. 2) Deshidratacin hipertnica (hipernatrmica), con Na+ srico
mayor a 150 mEq/L. 3) Deshidratacin hipotnica (hiponatrmica), con Na+ srico menor a 130 mEq/L. El tipo de deshidratacin depender entonces
de la prdida que predomine: si la deplecin de lquido es menor que la de solutos, se presentar una deshidratacin hipertnica; y si es mayor la
prdida de solutos, ser hipotnica. En trminos de frecuencia, la isotnica representa 80% del total de las deshidrataciones, la hipotnica 15% y la
hipertnica 5%. Las manifestaciones de la hiponatremia se correlaciona con la velocidad de descenso del Na srico: si el decremento sucede en un
tiempo menor a 12h, se denomina hiponatremia aguda, la cual mostrara datos clnicos ms tempranamente que la desarrollada en un tiempo
mayor. Las manifestaciones graves son las del SNC: irritabilidad, vmitos, nausea, fasciculaciones, cefalea, e incluso crisis convulsivas o coma con
cifras menores a 120 mEq/L. Otros elementos a considerar en sujetos con deshidratacin son el estado acido-base y la hipokalemia. Estos pacientes
pueden pre4sentar acidosis metablica (pH <7.35 con HCO3 <29mEq/l), debido al incremento en las perdidas de bicarbonato; la disminucin en la
capacidad del rin para eliminar hidrogeniones y la ganancia de cido lctico por hipoperfusion que se presenta en la deshidratacin moderada y
severa. Las manifestaciones clnicas de la acidosis metablica son polipnea (como mecanismo de compensacin), depresin miocrdica, arritmias,
dilatacin arteriolar, entre otras. La hipokalemia (K<3mEq/l), es producida por 2 mecanismos, las perdidas incrementadas a travs de la heces y la
excrecin renal aumentada para mantener la homeostasis con Na. Las manifestaciones clnicas pueden ir de un paciente totalmente asintomtico,
hasta sujetos con leo, rabdomiolisis o arritmias. Laboratorio: El examen de orina muestra densidad especfica, generalmente mayor a 1.020, debido
al mecanismo homeosttico de resorcin de lquidos. En caso de registrar densidad especfica menor a 1.020, se deber descartar alguna nefropata
intrnseca, por la incapacidad de realizar esos mecanismos. La determinacin de electrolitos sricos (Na+, K+, Cl-), tiene especial importancia para
determinar el tipo de deshidratacin hipo o hipernatrmica. El potasio srico puede modificarse a su vez por estados de alcalosis o acidosis; si el pH

CURSO ENARM CMN SIGLO XXI TEL: 36246001

Pharmed Solutions Institute

PGINA 394

MANUAL DE TRABAJO DEL CURSO ENARM CMN SIGLO XXI


desciende 0.1 unidades por debajo de 7.35, el K+ srico incrementa 0.5 mEq/L y viceversa, siel pH aumenta 0.1 unidades, por arriba de 7.45,el K+
srico desciende 0.5 mEq/L. La determinacin srica de cloro ayuda generalmente a calcular la brecha aninica (anion gap), la cualpermite descartar
otras causas de acidemia, como acidemias orgnicas, o cetoacidosis diabtica. Se calcula mediante la siguiente frmula: Na+ - (Cl- + HCO3-). Tiene
como valor normal entre 8 y 12 mEq/L. TRATAMIENTO: Como ya se mencion, la diarrea aguda es la principal causa de deshidratacin en nuestro
pas. Par esta causa la OMS especficamente recomienda un plan a seguir con base en los datos clnicos que presente el nio. El plan A en pacientes
sin deshidratacin o deshidratacin leve, el plan B en casos de deshidratacin moderada y el plan C en enfermos con choque. PLAN A: Continuar con
alimentacin habitual, aumentar la ingestin de lquidos de uso cotidiano en el hogar, VSO, en los menores de 1 ao, ofrecer media taza (75ml) y en
lo mayores de un ao una taza (150ml) y administrarlo a cucharadas o mediante sorbos pequeos despus de cada evacuacin. Capacitar a la madre
para reconocer los signos de deshidratacin y datos de alarma por enfermedades diarreicas (sed intensa, poca ingestin de lquidos y alimentos,
numerosas heces liquidas, fiebre, vmito y sangre en las evacuaciones) con el propsito de que acuda nuevamente a solicitar atencin medica en
forma oportuna. Esta formula consta de Na 90mEq/l, K 20mEq/l, HCO3 30mEq/l, Cl 80mEq/l, glucosa 111mEq/l. la OMS actualmente recomienda el
uso global de esta frmula con sodio 75 mEq/L, osmolaridad 245 mOsm/L y glucosa 75 mEq/L. PLAN B: Pasar pacientes con diarrea y deshidratacin
con atencin en la unidad de salud; VSO 100ml/kg, en dosis fraccionada cada 30min durante 4h VO. Si el paciente presenta vmito, esperar 10
minutos e intentar otra vez la hidratacin oral ms lentamente se puede intentar infusin por sonda nasogstrica a razn de 20-30 mL/kg/h. Al
mejorar el estado de hidratacin, pasar a plan A. En caso contrario repetir el plan B en 4h, de no existir mejora pasar al plan C. PLAN C: Iniciar
inmediatamente administracin de lquidos por va IV, con solucin de Hartmann, si no se encuentra disponible, use sol. Salina al 0.9%, de acuerdo
con el siguiente esquema: Primera hora 50ml/kg, segunda hora 25ml/kg, tercera hora 25ml/kg. Evaluar al paciente continuamente. Si no mejora
aumentar la velocidad de infusin. Cuando pueda beber (usualmente 2-3h), administrar sobres de VSO, a dosis de 25ml/kg/h, mientras sigue lquidos
IV. Al completar la dosis IV, evaluar al paciente para seleccionar plan A o B, y retirar venoclisis, o repetir plan C. Si selecciona el plan A, observar
durante h para asegurarse de que el responsable del paciente pueda mantenerlo hidratado con sobres de VSO, adems alimentarlo en su domicilio.
La OMS recomienda la administracin de solucin de Ringer-lactato o sol. Fisiolgica en dosis de 20 mL/kg de peso hasta que el pulso, la perfusin y
el estado de conciencia regresen a la normalidad. En sujetos en quienes est contraindicada la va oral o tienen otras causas de deshidratacin se
sugiere un plan de tratamiento que se divide en cinco fases: La primera fase de la rehidratacin se debe llevar a cabo en los primeros 30 a 60
minutos (NaCl 0.9% 20 mL/kg en bolo). Su objetivo es la restitucin rpida de la deplecin de volumen circulante, en pacientes con choque
hipovolmico. En estos pacientes se inicia la reposicin de lquidos con bolos de 20 mL/kg de cristaloides. Para el manejo de choque hipovolmico en
nios y RN, existen las siguientes recomendaciones clnicas. La primera eleccin para la resucitacin inicial es la solucin salina; si se requieren
grandes cantidades de lquidos, es posible utilizar coloides sintticos por su mayor duracin en la circulacin y por ltimo el volumen inicial debe de
ser de 20 mL/kg; dependiendo de la respuesta clnica, se deber valorar el nmero de dosis necesarias y, en pacientes con problemas cardiacos o
recin nacidos, se recomiendan bolos de 10 mL/kg. La segunda fase se lleva a cabo dentro 1-9 hrs de iniciada la hidratacin del paciente. Su finalidad
es la restitucin parcial del dficit del espacio extracelular y del estado cidobase. El plan teraputico en esta fase es administrar 1/3 de lquidos de
mantenimiento diarios y del dficit de lquidos. La recrea fase se lleva a cabo a las 9hrs a 24h de haber iniciado la hidratacin. Su objetivo es la
restitucin de volmenes del los compartimentos EEC y EIC y del estado acido-base. Administrar 2/3 de los lquidos de mantenimiento diario y 1/3
del dficit de lquidos. La cuarta fase se lleva a cabo dentro de las 25 a 48 horas de iniciada la hidratacin. Su finalidad es la correccin total de
electrolitos, estado cidobase y volumen de lquidos, dar lquidos de mantenimiento y reponer las prdidas actuales, as como iniciar la va oral si
las condiciones lo permiten. La quinta fase se lleva a cabo, durante los 2-14 dias de haber iniciado la hidratacin. Su objetivo es restaurar el dficit
calrico y proteico. Esto se logra con una adecuada dieta por va oral. CORRECCIN DE TRASTORNOS ELECTROLTICOS Y CIDO-BASE
CONCOMITANTES: En pacientes con hiponatremia (Na <13mEq/l), se debe agregar a las soluciones de mantenimiento y a las del dficit la cantidad
necesaria de sodio para llevarla a niveles sricos normales.41 La correccin no debe ser ms rpida que 1 mEq por hora o 12 mEq/L en 24 horas,
debido al riesgo de producir cambios osmolares en el sistema nervioso central. La correccin se hace mediante la siguiente frmula (Na+ ideal - Na+
medido) x peso en kg x 0.6= mEq a administrar en 24 horas. A este resultado se suman los requerimientos normales. En pacientes con hiponatremia
sintomtica y Na srico <120mE/l, se debe incrementar el valor de Na+ por arriba de esta cifra, en un periodo de una hora, con solucin salina al 3%
de 5 a 10 mL/kg; esto elevar el Na+ srico de 4 a 8 mEq/L. En pacientes con hipernatremia (Na srico >145mEq/l), se debe corregir el dficit de
lquidos en un periodo mayor, llegando hacer hasta 48h, debido a que el descenso rpido de Na tambin puede causar cambios osmolares en el SNC,
provocando edema cerebral y mielinolisis pontina. La meta es descender el Na srico un mximo de 15mEq/dia (1mEq/h). La hipokalemia se debe
tratar solo con incrementos del aporte diario en pacientes asintomticos, aumentando la concentracin de potasio en soluciones a 60-80mEq/l; pero
en pacientes que presentan sintomatologa, como arritmias, debilidad muscular importante o dificultad respiratoria atribuible a esta, deben ser
estrechamente monitorizados y administrar KCL a razn de 0.1-0,2 mEq/Kg/h, y la hiopocalemia con alteraciones ritmo cardiaco que pongan en
peligro la vida, la infusin de K+ debe ser de 0.3 mEq/kg/h, siempre mediante monitorizacin de electrocardiografa estrecha y la administracin del
potasio IV mediante bomba de infusin. La acidosis metablica se trata desde el inicio con la restitucin de volumen; sin embargo, la administracin
parenteral de HCO3, esta indicada en pacientes con pH menor a 7.10 o HCO3- menor de 10 mEq/L, y se calcula por medio de la siguiente frmula:
HCO3 (mEq/L) = Dficit de base x peso(kg) x 0.3, debindose administrar al paciente de un cuarto a la mitad de la cantidad resultante de HCO3. Un
cuarto de la dosis equivale aroximadamente a 1mEq/kg (peso) de HCO3.
CASO CLINICO DESHIDRATACION (LIQUIDOS Y ELECTROLITOS)
Paciente masculino de 4 aos de edad con diagnostico de
gastroenteritis el fue tratado de forma ambulatoria, a las 6 horas
acude a urgenciar debido a que continuo con vomito en 8 ocaciones
y 5 deposiciones liquidas, a la exploracin fsica se observa ojos
hundidos, llanto sin lagrimas, somnoliento, hiporreactivo.
PREGUNTA
Cual de los siguientes signos es esta mas relacionado con el choque
descompensado?
RESPUESTA
a.- Taquicardia.
b.- Taquipnea.
c.- Tiempo de llenado capilar prolongado.
d.- Hipotensin.

CURSO ENARM CMN SIGLO XXI TEL: 36246001

CASO CLINICO
Mujer de 18 aos de edad con una infeccion gastrointestinal, acude
a la sala de urgencias con vomito y diarrea, de 5 dias de evolucin.
Hay deshidratacin aguda. Los estudios de laboratorio muestran pH
7.38, PCO2 de 37mmHg y HCO3 de 25mEq/l.
PREGUNTA
Cul es el estado acidobasico mas probable de esta paciente?
RESPUESTA
a.- Acidosis metabolica
b.- Alcalosis y acidosis metablicas mixtas
c.- Alcalosis y acidosis respiratorias mixtas
d.- Alcalosis respiratoria

Pharmed Solutions Institute

PGINA 395

MANUAL DE TRABAJO DEL CURSO ENARM CMN SIGLO XXI


QUEMADURAS
CIENCIAS BASICAS: Son lesiones traumticas primariamente de la piel pero que pueden afectar a otros tejidos, por accin de diversos agentes
fsicos, qumicos, biolgicos, con alteraciones, de enrojecimiento a destruccin total. Suelen acompaarse de una variedad de complicaciones que
cuando son graves o inesperadas pueden ocasionar el fallecimiento del paciente. SALUD PUBLICA: Las quemaduras constituyen un problema
mdico, psicolgico, econmico y social que involucra al mdico, psiclogo, econmico y social que involucra al mdico, al paciente, al familiar del
paciente y a la sociedad en s. En Mxico se calcula se atienden en hospitalizacin 10 000 pacientes. Ms de 66% de los pacientes quemados
recibidos en unidades especiales, son nios. Pacientes tienen el 77% de mortalidad cuando hay combinacin de quemadura de va area con
quemadura cutnea. PATOGENIA: El mecanismo de lesin ms comn es la escaldadura 72.7%, seguida del fuego directo 18.7%, por contacto 10.5%,
elctricas 8%. Otros: por ignicin, por inhalacin, quemaduras qumicas, por deflagracin, por frio, por radiacin. La quemadura se debe a la
transferencia de energa de una fuente de calor al organismo; no es homognea en las regiones corporales afectadas ya que se produce ms lesin
en el rea de mayor exposicin. En la parte central donde la transferencia de energa es mxima, ocurre una necrosis cutnea irreversible, la zona de
coagulacin. Alrededor de ella se dispone una zona que se caracteriza por una intensa reaccin inflamatoria, la zona de estasis; esta ara puede
evolucionar a la recuperacin pero si se presenta disecacin o infeccin puede evolucionar hacia una completa destruccin en cuyo caso incrementa
el rea de afeccin. En la periferia se encuentra la zona de hiperemia en
donde el dao celular es mnimo y la recuperacin espontanea es muy
rpida. CLASIFICACION: Por su profundidad; 1er GRADO (epidrmica o
eritematosa); Involucra epidermis, la piel luce eritematosa, de color rojo o
rosa brillante, seca, es progresivamente pruriginosa y presenta hiperalgesia.
Sana espontneamente sin dejar cicatriz. Ejemplo: quemadura solar. 2do.
GRADO SUPERFICIAL (de espesor parcial o flictenular); Involucra epidermis y
dermis papilar, se manifiesta por un color rojizo, es suave, hmeda y existen
flictenas, Cicatriz normalmente en un lapso de 10-14 das. 2do. GRADO
PROFUNDA; Involucra epidermis, dermis pailar y en profundidad variable la dermis reticular, pero dejan intactas las faneras epidrmicas profundas.
Color rojo moteado o blanquecinas, no existen flictenas y son hipoalgesicas. La cicatrizacin ocurre en 21-28 das, siendo frecuente la cicatrizacin
hipertrfica, requieren injertos. 3er. GRADO (espesor total); Son de color blanquecino, amarillo plido, momificacin, marrn o negro
(carbonizacin). El tejido tiene una apariencia acartonada y suelen observarse vasos trombosados. Injerto de piel >1 cm. Efectos sistmicos si es
extensa, puede haber amputaciones. Curan > 42 das. La cicatrizacin ocurre principalmente por contraccin y es deformante. Clasificacin por su
gravedad LEVE; Quemaduras de 2do grado superficial o profunda o ambas, de menos de 10% de SC. Quemadura de 3er grado de menos de 2 % de
SC. Sin afeccin de reas especiales. MODERADAS; 2do grado superficial o profundas o ambas entre 10-20% de SC. De 3er grado entre 2-10% de SC.
Sin afeccin de reas especiales. SEVERAS; 2do grado superficial o profundo >20% de SC. De 3er grado
>10%. Afeccin de reas especiales (cara, ojos, cuello, manos, pies, genitales, articulaciones y
pliegues). En menores de 1 ao. Inhalacin, corriente elctrica. Presencia de quemaduras ms
fracturas o traumas graves, enfermedades asociadas. DIAGNOSTICO: Valorar, gravedad, tipo y
extensin de la quemadura esta ltima con las Tablas del porcentaje de los segmentos corporales
segn edad de "LURD y BROWDER" o ms conocida como REGLA DE LOS "9". sta y otras formas de
medicin significa no solo conocerlas, sino prctica en su uso, por lo que una forma sencilla de saber la
extensin de una quemadura es utilizar la regla de la palma de la mano, la cual representa aprox., el
1% de SC. Datos clnicos que indican quemadura de vas areas, inhalacin o ambas; cianosis,
expectoracin carboncea, depsitos bucofarngeo de carbn, ampollas y cambios inflamatorios en
lengua y paladar, cambios de voz, ronquera, tos persistente, quemadura de cara, cejas y delas vibrisas,
historia de confinamiento cuando ocurri el fuego, estado de alerta alterado. El choque hipovolmico
puede desarrollarse rpidamente en los pacientes peditricos. Un nio con 10Kg de peso, con una
quemadura de 20% de extensin, tienen perdidas por evaporacin cercanas a los 475ml, equivalentes
al 60% de su volumen circulante. CRITERIOS DE HOSPITALIZACION: 1) Extensin de la quemadura en
una rea mayor del 10% (<5 aos con superficies mayores a un 5%) de la SCT. 2) Quemadura de cara,
cuello, rea glteogenital y eventualmente manos en quemaduras palmo digitales intermedias o profundas. 3) Quemadura elctrica de alto voltaje o
de bajo voltaje. 4) Quemadura circular de extremidades, trax o cuello. 5) Quemadura por cidos o lcalis. 6) Rescate desde un espacio cerrado con
ambiente invadido por humo (Sospecha de Quemadura Respiratoria). 7) Traumatismo mecnico importante asociado. 8) Enfermedad metablica o
sistmica asociada. 9) Sospecha de maltrato infantil. 10) Marginalidad o ruralidad extrema. 11) Caso social (analfabetismo o escasa escolaridad de
los padres o personas a cargo del nio, recursos econmicos escasos, etc.). TRATAMIENTO: Inmediatamente evolucin de vas areas y ventilacin
(edema masivo, obstruccin ventilatoria, lesin por inhalacin), estado hemodinmico, cuantificar tamao y profundidad de quemadura, agente
etiolgico, tiempo y lesiones asociadas, recordar mientras ms pequeo el paciente es ms susceptible. La evaluacin tradicional del dficit
neurolgico no debe olvidarse. Traslado del paciente. Tratamiento: Pacientes no intubados requieren terapia respiratoria agresiva con
broncodilatadores y humidificadores con mezclas de Heliox y O2, ventilacin mecnica, cmara hiperbrica. Corticoesteroides no reducen el edema de
la va area. Elevar la cabeza 30 grados en la resucitacin inicial disminuye el edema de la va area. Reposicin hdrica: <2 aos: >10% SCQ. >2 aos:
>15% SCQ. Calcular prdidas insensibles: (25 + %SCTQ) X (SCT m2). SCTm2 = Peso x 4 + 7/peso + 70. Frmula de Parkland y Baxter, se utiliza en las
primeras 24h, con Hartmann 24 despus coloides. 4 ML x KG x % SCT Quemada. La mitad del dficit total en las primeras 8h sol Hartmann. La
segunda mitad en las 16h restantes. Calcular volumen circulante 60cc x kg. Paciente de 8 kg que tiene una quemadura de 25% de su cuerpo y se
decide utilizar Parkland modificado para restituir lquidos. 4ml x kg x % SCQ, 4 x8 x 25 = 800 ml, pasar 400ml para 8h y 400ml para 16h. Formula de
Galveston: Primer da: 50% en 8h y 50% en 16h = 500ml de sol. Hartmann/kg/SCQ + 200ml de sol. Glucosada al 5%/kg/SCQ. El segundo da: 3750
ml/kg/ SCQ Hartmann + 1500 mil/kg/ SCQ glucosada 5% en 24h. Vigilar continuamente y mantener adecuada PVC (8 - 10), mantener TAM >
60mmHg, gasto urinario 0.5 - 1.0ml/kg/h, evitar administracin excesiva de lquidos, uresis (2ml/kg/h), vigilar hipotermia. Manejo de dolor con
narcticos, analgsicos, sedantes: Morfina 10 mg IV c/3-4 h. Nalbufina 10 mg IV c /3-4 h. NO administrar antibiticos empricos. Cuidado de
quemaduras: Sulfadiacina de plata (Silvadene), cubrirlas con sbanas estriles, curaciones quirrgicas (qx plstica). La infeccin en pacientes
quemados es la mayor causa de morbilidad, en algunos centros especializados, la sepsis es causante de 50-60% de las muertes. Se sabe que durante
las primeras 24h posteriores a la quemadura predominan grmenes gramm+ (S. aureus). Despus del final de la primera semana se inicia
colonizacin por gramm- (P. aeruginosa. La infeccin por anaerobios se ha a atribuido principalmente a Bacteroides. Se manejara antibitico acorde

CURSO ENARM CMN SIGLO XXI TEL: 36246001

Pharmed Solutions Institute

PGINA 396

MANUAL DE TRABAJO DEL CURSO ENARM CMN SIGLO XXI


a germen implicado. El diagnstico definitivo de infeccin de la lesin se basa fundamentalmente en el estudio histopatolgico por medio del cultivo
biopsia.
c.- Morfina, anibioticoterapia
CASO CLINICO QUEMADURAS
d.- Envi a segundo nivel
Masculino de 4 aos de edad ingresa a urgencias por presencia de
quemadura elctrica en la boca, refiere la madre del paciente que el
CASO CLINICO
nio se encontraba jugando y sbitamente comenz a llorar,
Paciente de 12 aos de edad, masculino, es llevado al servicio de
sealando la boca, agrega que solo estaba un cable de luz que
urgencias por personal paramdico desde su lugar de origen 5 horas
pareca mordido y levemente quemado, a la exploracin se observa
despus del accidente, por presentar quemadura elctrica producida
a paciente alerta, con facies algicas, llanto intenso, sus contantes
por descarga de 3500 Voltios de corriente alterna (AC), prdida de
vitales se encuentran dentro de parmetros normales, la mucosa
conciencia por tiempo no determinado, quemaduras de segundo
oral se encuentra lesin tipo quemadura que incluye comisura labial
grado superficial en trax anterior, quemaduras de segundo grado
que incluye piel peribucal.
profundo en cara anterior de muslos y carbonizacin de miembros
superior e inferior izquierdos, no refiere dolor. Al ingreso en nuestra
PREGUNTA
Unidad el paciente se encuentra consciente, orientado en tiempo y
Cual es su conducta a seguir?
espacio, con valoracin de Glasgow 15/15, hemodinmicamente
RESPUESTA
inestable, con soplo sistlico grado II/VI en foco mitral y artico,
a.- Lavar la herida y colocacin de gasas vaselinadas.
franca hematuria y proteinuria.
b.- Debridar lesin e ingreso a observacin.
c.- Envio inmediato a tercer nivel.
PREGUNTA
d.- Manejo conservador, con control del dolor.
Cul es la conducta inmediata a seguir en este caso?
RESPUESTA
CASO CLINICO
a.- Hidratacin hdrica, con Parkland la mitad en 3 horas y la otra
Lactante mayor de 1 ao 7 meses de edad, con antecedente de
mitad en 16 hrs
sndrome bronquial obstructivo recurrente, sufre Quemadura de
b.- Hidratacin hdrica, con Parkland la mitad en 8 hrs y la otra mitad
7%SCQ, en trax y hombro derecho al volcarse agua caliente, el nio
en 16 hrs
refiere mucho dolor en rea afectada. EF: se observa quemadura de
c.- Hidratacin hdrica, con Parkland la mitad en 8 hrs y la otra mitad
color rosado a rojo intenso, al hacer presin sobre la lesin esta se
en las siguientes 8 hrs
blanquea, con presencia de mltiples flictenas.
d.- Hidratacin hdrica, con Parkland la mitad en 3hrs y la otra mitad
en las siguientes 8 hrs
PREGUNTA
Qu grado de quemadura ms probablemente presente este
PREGUNTA
paciente de acuerdo a la descripcin?
Cul es la complicacin inmediata ms probable si la reposicin es
RESPUESTA
con balance negativo?
a.- Segundo grado superficial y profunda
RESPUESTA
b.- Primero y segundo grado superficial
a.- Sepsis
c.- Primero y segundo grado profunda
b.- Derrame pleural
d.- Segundo grado profunda
c.- Choque hipovolmico
d.- SIRS
PREGUNTA
Cul es la conducta teraputica inmediata a seguir en este caso?
PREGUNTA
RESPUESTA
Cul es la conducta ms adecuada en cuanto a diuresis para evitar
a.- Restitucin hdrica, antibioticoterapia
complicaciones?
b.- Aseo y debridacin
RESPUESTA
c.- Domicilio, antibioticoterapia
a.- Mantener diuresis a 1-2ml/kg/h
d.- Analgsico, debridacin
b.- Mantener diuresis a 0.5-1ml/kg/h
c.- Mantener diuresis a 3-4ml/kg/h
PREGUNTA
d.- Mantener diuresis a 5-6ml/kg/h
Cul es el medicamento ms adecuado para este paciente, para
disminuir el dolor?
PREGUNTA
RESPUESTA
A los 11 das de hospitalizacin se realiz al paciente una
a.- Metamizol rectal
gammagrafa sea en la que observamos falta de vascularizacin del
b.- Cloruro mrfico subcutneo
segmento tibio-peroneo de los tercios medio y distal de la pierna
c.- Lidocana
izquierda. Cul es la conducta ms adecuada a seguir en este caso?
d.- Ibuprofeno oral
RESPUESTA
a.- Revascularizacin por microciruga
PREGUNTA
b.- Amputacin de la extremidad
Cuatro das despus la madre nuevamente lleva al pequeo, refiere
c.- Hacer injerto por imbibicin
que tena un vaso de agua caliente, y se lo vaco en la cara,
d.- Dermafasciotomia
afectando su prpado y pabelln auricular derechos. Cul es la
conducta ms adecuada seguir en este momento?
RESPUESTA
a.- Hospitalizacin, morfina, Debridacin
b.- Envi a tercer nivel

CURSO ENARM CMN SIGLO XXI TEL: 36246001

Pharmed Solutions Institute

PGINA 397

MANUAL DE TRABAJO DEL CURSO ENARM CMN SIGLO XXI


PICADURAS DE ARAAS
CIENCIAS BASICAS: El envenenamiento por picadura de araa, es un problema importante de salud pblica. Se han reportado tasas de ms de
200,000 accidentes por ao debido a 3,000-5,000 por picadura de araa. Los accidentes por artrpodos venenosos en Mxico como las araas de los
gneros Loxosceles y Latrodectus. En todo el mundo existen cerca de 30,000 especies o subespecies. Las araas poseen en el abdomen glndulas en
donde se produce una sustancia proteica con la que forman
sus nidos y redes, es decir las telas de araa. PICADURA
POR CAPULINA: Latrodectus mactans. El lactrodectismo es un
problema frecuente, la araa capulina o viuda negra es
frecuente encontrarla aun en zonas cosmopolitas como el DF.
A pesar de ser un animal tmido, cuando es molestado
muerde una o ms veces. A diferencia del alacrn, la
mordedura no duele en el momento; sin embargo los
sntomas se inician 30-60min despus y se inician en el sitio
de la lesin, hipopigmentacin e inflamacin en el sitio de la
picadura y luego se diseminan, es caracterstico el dolor
articular, temblor y fasciculaciones prcticamente de todos
los msculos, calambres abdominales, diaforesis profusa,
angustia y la fascies lactrodectsmica y trismos. El veneno
desestabiliza los canales inicos con liberacin de
catecolaminas en las terminaciones adrenrgicas adems
deplecin de acetilcolina en nervios motores (despolarizacin
prolongada de los receptores postsinapticos). La ponzoa de
esta araa es una proteasa alfa-latrotoxina, que tiene
actividad neurotxica y una potencia 15 veces mayor a la del veneno de la serpiente cascabel. Tratamiento: Colocar un torniquete para evitar la
diseminacin de la toxina, traslado a clnica ms cercana. Analgsicos. Neostigmina o prostigmina 0.5 a 1 mg c/8-12 h por va parenteral. Por va oral
15 mg c/8h 2-3 das. Este frmaco inhibe la accin de la colinesterasa en los tejidos y la sangre, e impide la destruccin de la acetilcolina. Cortisona y
corticotropina, que ayudan a controlar los sntomas Generales. Suero aracmyn faboterapia, el 90% de los pacientes responde a una sola dosis; el
10% restante, responde a 2 3 frascos. Dosis 1 ampolleta IV lenta y los resultados son extraordinarios, los sntomas como dolor, calambres, angustia
y miedo, mejoran en los nios dentro de los primeros 30 min, y en los adultos solo un poco ms. Las dosis deben ser individuales y aplicar nuevas
dosis solo previa valoracin y persistencia de la sintomatologa. PICADURA POR ARAA VIOLINISTA: Loxosceles, El loxocelismo es el cuadro que se
presenta cuando una persona es picado por la araa llamada araa violinista o araa del rincn. De color marrn, con una mancha ms plano y tres
pares de ojos. 100 especies en todo el mundo. Mxico: L. boneti mide de 9-25 mm, es de color caf. Habita lugares obscuros, poco ventilados,
depsitos de madera. Se esconde en cuadros, muebles. Patogenia: El veneno, producido en poca cantidad, tiene una accin dermonecrotizante,
hemoltica, vascultica y coagulante. Acompaando el cuadro cutneo puede existir compromiso sistmico que se manifiesta por hemlisis.
Diagnstico: Loxoscelismo cutneo, es la forma de presentacin ms frecuente, oscila en 84-97% segn distintos autores. La picadura suele producir
dolor de poca intensidad inicialmente y, muchas veces, el paciente no sabe precisar cundo ocurri, pero tambin se ha descripto un dolor urente.
La lesin inicial se caracteriza por edema, eritema y, a veces, dolor urente. Evoluciona en 24-36 h a la caracterstica placa marmrea o livedoide
(reas intercaladas de palidez y equimosis) de bordes irregulares, bien definidos, circundada por eritema y edema indurado, dolorosa a la palpacin.
Ms tarde pueden aparecer vesculas y flictenas en zonas declive. Las lesiones presentan un patrn de necrosis hemorrgica gravitacional,
posiblemente relacionada con la accin conjunta de las esfingomielinasas y la hialuronidasa. Como sntomas generales del loxoscelismo cutneo
pueden presentarse nuseas, vmitos y fiebre. Entre el 5 y 7 da la lesin cutnea se circunscribe con la formacin de una escara negra, que
comienza a desprenderse a partir de la segunda semana. sta deja una lcera con fondo de granulacin que tarda varios meses en cicatrizar y puede
requerir ciruga reparadora. La forma edematosa es una variante clnica del loxoscelismo por picadura en una zona de tejidos laxos, habitualmente la
cara. Se caracteriza por eritema y edema sin evolucionar a la necrosis. Las lesiones cutneas, una vez establecidas, son muy difciles de tratar, aun
con un tratamiento precoz y adecuado, y pocas veces es posible detener su evolucin. Loxoscelismo cutneo-visceral: Se considera un cuadro ms
grave que el anterior caracterizado por hemlisis intravascular y coagulacin intravascular diseminada (CID) asociadas a la lesin cutnea
anteriormente descrita. Tratamiento: Inespecfico. Incluye tratamiento de sostn, asepsia de la lesin, profilaxis antitetnica en caso de requerirla y
analgsicos. Dapsona: 1-2 mg/kg/da una dosis, evita necrosis. Vigilar: leucopenia, IR, metaheoglobinemia. Aplicar: a) 4 mg de fosfato de
dexametasona por va subcutnea en el sitio de la picadura. b) 40 a 80 mg de acetato de metil-prednisolona por va intramuscular. c) Repetir el
tratamiento local con 21-fosfato de dexametasona, cuatro horas despus. d) Si continan apareciendo vesculas, repetir la dosis de metilprednisolona a las 24 horas. e) Si ocurre hemlisis intravascular: 50 mg de prednisolona por va intravenosa, seguidos de 25 mg cada 6 horas, hasta
que desaparezca la hemlisis. f) En caso necesario, previa valoracin general, 5,000 U de heparina cada 12 horas. g) Administrar Aracmyn Plus
(Faboterapia). PICADURA DE ALACRAN: Dentro de la clase Arachnida, hay artrpodos que llaman especialmente la atencin del hombre, ya que
desde tiempos inmemoriales le han causado lesiones traumticas, intoxicaciones, invalidez, enfermedades severas y muerte.1 La picadura de alacrn
est considerada como una urgencia mdica por la naturaleza que representa el ataque, que es la inoculacin de veneno que provoca la alarma en la
persona afectada, as como a sus familiares, ms an si la agresin fue a un menor. SALUD PUBLICA: Mxico es el pas de mayor diversidad de
escorpiones en el mundo, con siete familias, 90 gneros y 200 especies. Son siete las especies peligrosas; se encuentran en la zona occidental y
central del pas, los estados de Nayarit, Colima, Jalisco, Durango, Michoacn, Puebla, Guanajuato, Oaxaca son los ms importantes. El IMSS reporta
en su semanario epidemiolgico 60 000 casos por ao. PATOGENIA: Los Suffusus, Ticumanus, Limpidus, Noxius, Sculpturatus, Nayaritus, entre otros
son los responsables de las intoxicaciones severas en nuestro medio y las costas del pacifico son las ms afectadas. PATOGENIA: El veneno de los
alacranes est compuesto principalmente por polipptidos, enzimas proteolticas, protenas de bajo peso molecular, serotonina y aminocidos de
accin neurotxica. Es inoculado directamente a la vctima por va subcutnea y se disemina por la va hematgena; es eliminado por la orina y por la
secrecin biliar. CLASIFICACION: Por su estado clnico se distinguen 4 grados de envenenamiento: I. Dolor local, parestesias en la zona de la picadura.
II. Se agrega parestesias en reas remotas a la zona del piquete. III. Disfuncin somtica o de pares craneales. Inquietud, estremecimiento
involuntario, espasmos musculares. Midriasis, visin borrosa, diplopa, ceguera temporal, movimientos incoordinados, sialorrea, dificultad para
deglutir, fasciculaciones de la lengua, espasmo larngeo, habla farfullante. IV. Disfuncin somtica musculo esqueltica y de pares craneales.
DIAGNOSTICO: Signos vitales: taquicardia, taquipnea, hipertensin arterial, hipertermia. Laboratorio: no hay exmenes especficos. Tratamiento
general: Compresas fras en tanto recibe tratamiento especfico. Analgsico, antihistamnicos en caso necesario sintomticos por la hipertensin y

CURSO ENARM CMN SIGLO XXI TEL: 36246001

Pharmed Solutions Institute

PGINA 398

MANUAL DE TRABAJO DEL CURSO ENARM CMN SIGLO XXI


ansiolticos. Especfico: faboterapia con alacramyn, dosis inicial una ampolleta diluida en 5 ml de solucin inyectable aplicacin lenta en la vena, los
resultados se miden a travs de mejora de los sntomas antes de aplicar una segunda dosis.
CASO CLINICO PICADURAS VIUDA NEGRA
Una lactante de 17 meses de edad, present irritabilidad aguda,
llanto, dolor en el pie izquierdo. La madre de la paciente le quit los
zapatos y se encontr una viuda negra muerta en el zapato izquierdo
del nio. Se desarrollo rpidamente inflamacin del pie izquierdo. El
EF mostro presin arterial 145/103 mm Hg y taquicardia (frecuencia
cardaca: 160-180 latidos por minuto). El examen revel edema y
eritema en el pie izquierdo y los prpados, y ninguna lesin diana
clsica vista en el sitio. Se observaba con dolor significativo. La
glucosa srica fue elevada a 186 mg / dl, y su recuento de glbulos
blancos se aument a 17 000/mm3.
PREGUNTA
Cual es el pronstico de la aracnismos presuntivo del caso?
RESPUESTA
a.- Dolor agudo, opresin precordial, taquicardia, dificultad
respiratoria y sensacin de muerte inminente.
b.- Alteraciones electrocardiogrficas, hipertensin y edema de
pulmn.
c.- Sensacin punzante, dolor urente o quemante y edema local.
d.- Ppula, que puede evolucionar a una placa eritematosa o placa
marmrea.
PREGUNTA
El paciente fue tratado con diazepam, morfina y difenhidramina. Se
traslado a la unidad de cuidados intensivos peditricos ~ 5 horas
despus de la mordedura de la araa, se qued irritado y con dolor,
con un firme, pero no rgido, abdomen y notable eritema del pie
izquierdo. Cuales medidas son ms utiles previo al traslado a una
unidad de segundo nivel?
RESPUESTA
a.- Retirar todos los elementos que puedan comprimir y agravar el
edema en la zona afectada.
b.- En las lesiones localizadas en los miembros mantener posicin de
Trendelemburg.
c.- Realizar antisepsia local, evaluar la necesidad de profilaxis
antitetnica y utilizar analgsicos.
d.- Uso de corticoides y antibiticos.
CASO CLINICO
Lactante del sexo femenino de nueve meses de edad, llevada por la
madre al servicio de urgencias, de un hospital en Oaxaca, con un
cuadro clnico caracterizado por una vescula en pierna derecha dos
das previos al ingreso, y 24 horas despus una mancha eritematosa,
atribuyndola a piquete de insecto, agregndose dolor intenso,
limitacin para la movilizacin de la extremidad afectada y edema,
refiere vomito en una ocasin. A la exploracin fsica se apreci:
lactante en regulares condiciones generales, irritabilidad, leve
palidez de piel, hidratada, taquicrdica, abdomen normal, en tercio
medio de pierna derecha lesin eritematosa, de bordes mal
definidos, vesculas con tendencia a confluir sobre la zona
eritematosa, con aumento del calor local, y el tercio distal fro y
pulsos perifricos palpables. Se inici manejo con dicloxacilina y
amikacina con diagnstico presuntivo de celulitis. Un da despus, el
rea se torn violcea, con flictenas y la parte central denudada
hemorragica, que abarcaba un dimetro de 10 cm.
PREGUNTA
Cul su sospecha diagnostica ms probable en este caso?
RESPUESTA
a.- Ofidismo
b.- Loxoscelismo
c.- Latrodectismo
d.- Alacranismo

CURSO ENARM CMN SIGLO XXI TEL: 36246001

PREGUNTA
Cul es la conducta teraputica ms adecuada a seguir en este
momento, para evitar ms necrosis?
RESPUESTA
a.- Dexametasona
b.- Prednisolona
c.- Faboterapico
d.- Dapsona
PREGUNTA
De acuerdo al cuadro clnico de paciente. Con cuntos frascos de
faboterpico debemos iniciar?
RESPUESTA
a.- 1
b.- 2
c.- 3
d.- 4
PREGUNTA
A las 24 hrs el paciente presenta hemoglobinuria ++, y ligera
acrocianosis. Cul es la conducta ms adecuada a seguir?
RESPUESTA
a.- Continuar faboterpico ms Prednisolona por va intravenosa
b.- Continuar faboterpico ms metacarbamol por va intravenosa
c.- Metilprednisolona va intravenosa mas neostigmina
d.- Continuar faboterpico ms dapsona
CASO CLINICO
Masculino de 7 aos de edad, relat que se encontraba jugando en
el campo cuando instantes despus de levantar un tronco, una araa
negra le pic en el hombro izquierdo. 2 horas despus de la
picadura, intenso dolor muscular a nivel de regin dorso-lumbar y
abdomen. A su llegada a urgencias presentaba inquietud extrema,
sudoracin profusa, rubor facial y espasmos musculares. Sus
constantes vitales fueron: TA 120/75mmHg FC 92 lpm y T 36,3. EF:
roncus y sibilancias en ambos campos pulmonares; el abdomen
mostraba cierta contractura muscular que pareca voluntaria. En el
hombro izquierdo, en la zona de la picadura, se apreciaba un
pequeo eritema con algunas lesiones petequiales puntiformes e
hipoestesia. No se observaron adenopatas regionales. En la analtica
practicada destac en el hemograma una leucocitosis de 20.000
clulas con un 90 % de neutrfilos con el resto de las series
normales.
PREGUNTA
Cul es la especie que ms probable es la causante del cuadro
clnico del paciente?
RESPUESTA
a.- Lactrodectus
b.- Loxoceles
c.- Vipiridae
d.- Theriidae
PREGUNTA
Cul es la conducta teraputica ms adecuada a seguir en este
momento?
RESPUESTA
a.- Faboterpico 2 frascos ms metocarbamol
b.- Faboterpico 3 frascos ms Neostigmina
c.- Faboterpico 1 frasco ms cortisona
d.- Faboterpico 2 frascos ms neostigmina ms metocarbamol
PREGUNTA
Cul de las siguientes caractersticas es menos probable que
encontremos en la fascies lactrodectsmica?
RESPUESTA
a.- Trismo
b.- Epifora
c.- Oftalmoplejia
d.- Hiperemia conjuntival

Pharmed Solutions Institute

PGINA 399

MANUAL DE TRABAJO DEL CURSO ENARM CMN SIGLO XXI


MORDEDURAS
CIENCIAS BASICAS: MORDEDURA POR SERPIENTES VENENOSAS: Se denomina accidente ofdico al cuadro txico desencadenado por la inoculacin
de veneno a travs de la mordedura de serpientes que poseen esta
propiedad. Las serpientes venenosas, poseen una cabeza triangular
con foseta real (rgano termosensible que detecta radiaciones de
calor producidas por sus presas) ubicado en el hocico entre la nariz y
ambos ojos, y pupilas que se verticalizan con la luz. Cuerpo cubierto
por plaquetas escamadas que al tacto le imprimen rugosidad, con
dibujos de distintas configuraciones y de colores poco llamativos. La
distribucin de los sitios anatmicos de las mordeduras de serpientes
fue la siguiente: 72% en pies y tobillos, 14% en muslos, 13% en
manos, 1% en cabeza. SALUD PUBLICA: Aproximadamente se
considera 15% de las 3 mil especies de serpientes se consideran
peligrosas en el mundo. En Mxico del total de las serpientes, el
81.36% son inofensivas y el 18.64% son venenosas. En Mxico hay
dos especies consideradas como venenosas: Viperidae: accidentes
ofdicos 90%. Elapidae. El grupo etreo con mayor riesgo de afeccin
es entre los 15-44 aos de edad (48.75%). Las especies ms
frecuentemente asociadas a mordedura son: Cascabel (44.9%),
Nauyaca (42.8%), Corales (4%), otras especies (3.6%), se desconoce
(5.1%). PATOGENIA: Mecanismos de accin de los venenos, las
toxinas de los principales grupos de serpientes se clasifican:
Hemotoxinas (tipificados por la serpientes cascabel sudafribana
(Crotalus terrificus), neurotoxinas (como los de la serpiente de coral (Elapidae - Micrurus), miotoxinas, nefrotoxinas, necrotoxinas. Locales;
destruccin intensa de tejido, dolor intenso eritema, edema que comprometen la extremidad afectada. DIAGNOSTICO: Los signos y sntomas
locales: Huellas de la mordedura, dolor, edema, incapacidad funcional, equimosis, sangrado, vesculas, necrosis local y parestesias regionales.
Sntomas sistmicos: nuseas, vmito, taquicardia, dolor abdominal y torcico, somnolencia, prdida de la conciencia, hematuria, hipotensin y
estado de choque. Hematolgicos: lisis del eritrocito, Hb, Hto, plaquetas, TP, TPT, tiempo de sangrado, hematuria, melena, hematemesis, epistaxis,
hemoptisis. Renales: glomerulonefritis, endarteriris proliferativa progresiva, necrosis cortical, insuficiencia renal aguda. Neuromuscular: hormigueo,
salivacin, parestesias, debilidad, reflejos anormales, depresin del SNC (disnea, parlisis bulbar (4-7h), parlisis difusa (1-2h). Los signos y sntomas
evolucionan rpidamente y pueden ocasionar paro respiratorio. Pulmonar, edema pulmonar. El diagnstico definitivo de la intoxicacin por veneno
de serpiente requiere la identificacin de las caractersticas morfolgicas de la serpiente venenosa y la correlacin de las manifestaciones clnicas de
envenenamiento. Los signos de envenenamiento pueden variar dependiendo la cantidad de veneno inoculado, tiempo de evolucin, regin afectada
y nmero de mordidas. El 20% de las mordidas se reportan como fras o secas, es decir, no existe envenenamiento. Es recomendable siempre tomar
cultivos de la lesin por el riesgo de infecciones agregadas, incluyendo un cultivo para clostridium. Adems se deben monitorizar: BH, cuenta de
reticulocitos, EGO, Na, K, Cl, Ca, urea, Cr, gasometra arterial, Tiempo de sangrado, TP, TPT, fibringeno, plaquetas. CLASIFICACION: Las
manifestaciones clnicas y el grado de envenenamiento por mordedura de serpiente venenosa se clasifican dependiendo del nivel del edema y de los
siguientes datos agregados, de acuerdo a la clasificacin de Christopher-Roding para la familia Vipiridae y para la familia Micrurus. Ver cuadro 1 y 2.
TRATAMIENTO: Traslado inmediato a clnica ms cercana. Todos los nios se internan mnimo 24 h. Agua y jabn, vendaje seco y estril. No
antibiticos profilcticos. Elevacin de la extremidad para disminuir edema e inmovilizarla. Si se considera peligro de provocar reaccin alrgica
administrar difenidramina 1 mg/kg. No se recomienda aplicar succin, ya que no ha demostrado remover el veneno. Actualmente, est
contraindicado el uso de torniquetes, ya que complica la circulacin linftica del miembro afectado, condicionando una mayor morbilidad. Si se
dispone de bandas de presin se pueden utilizar en el miembro afectado a una presin de 50-70mmHg, inmovilizando el miembro y aplicando
vendaje para disminuir la perfusin del veneno. Evaluar desde el principio el permetro del miembro afectado. Esta accin debe registrarse cada 1520min, para evaluar progresin, ya que hay riesgo de desarrollar sndrome compartamental. Se debe estabilizar al paciente y administrar analgsico
para el dolor. En pacientes mordidos por serpiente Vipiridae se recomienda el uso de opioides en lugar de AINES, debido al riesgo asociado de
desarrollar coagulopatia o trombocitopenia. Tratamiento especfico: la administracin de faboterapicos ha demostrado una disminucin del tiempo
de hospitalizacin y recuperacin. Se recomienda que sea de acuerdo al grado de envenenamiento, segn la clasificacin de Christopher-Rodning,
para nios y adultos, y grado de envenenamiento para la familia Micrurus. Personas hipersensibles al antiveneno que presenten nauseas, vomito,
choque anafilctico administrar: Adrenalina 1x 1000 aplicado 0.5 ml cada 15 minutos segn requiera. Oxgeno, esteroides. Gluconato de calcio: No.
COMPLICACIONES EN NIOS MAS FRECUENTES: Necrosis tisular, coagulacin intravascular diseminada, sndrome compartamental.
Administracin de faboterpico en base a grado de envenenamiento de acuerdo a clasificacin de Christopher y Rodning para nios y adultos (vipiridae)
TRATAMIENTO
No requiere faboterpico
Aplicar 4 frascos, va IV. Durante la primera hora se puede diluir en 100-250ml de sol. Salina 0.9%. Observar 12-24h. Si an no se ha controlado. Aplicar 4 frascos diluidos
en 100-250ml de sol. Salina 0.9%. Pasar en infusin continua durante 1 hora. Valorar estado de paciente
II
Aplicar 5 frascos va IV, diluidos en 100-250ml de sol. Salina 0.9% durante la primera hora. Observar 12-24h. Si no hay control, aplicar 10 frascos via IV diluidos en 100250ml de sol. Salina 0.9% durante 1 h, cada 4-6 hrs, hasta completar tratamiento de 18 h.
III
Aplicar 6-8 frascos va IV diluidos en 100-250ml de sol. Salina 0.9% va IV, durante la primera hora. Observar 12-24h. Si no hay control, aplicar 6-8 frascos IV, diluidos en
100-250ml de sol. Salina durante una hora, cada 4-6h, hasta completar tratamiento de 18 h.
IV
Aplicar 25 frascos va IV, diluido en 100-250ml de sol. Salina 0.9% durante la primera hora. Observar 12-24h. Si no hay control, aplicar 6-8 frascos via IV diluidos en 100250ml de sol. Salina 0.9% durante una hora, cada 4-6 h, hasta completar tratamiento de 18 h.
No olvidar considerar la cantidad de lquidos administrados junto con los faboterapicos en la evaluacin de balance hdrico total.
Administracin de faboterpico en base al grado de envenenamiento por mordedura de serpiente Micrurus en nios y adultos
GRADO
TRATAMIENTO
LEVE
Aplicar 2 frascos va IV, diluidos en 100mlde sol salina en la primera hora. Si no hay control, continuar con 2 frascos la siguiente hora. Si hay control mantener en
observacin de urgencias por 12-24h
MODERADO
Aplicar 5 frascos via IV, diluidos en 100ml de sol. Salina durante la primera hora. Si no hay control, continuar con 2 frascos la siguiente hora. Si hay control observar 1224h.
SEVERO
Apliacr 8 frascos via IV, diluidos en 100ml de sol. Salina durante la primera hora. Si no hay control, continuar con 2 frascos en la siguiente hora. Si hay control
observacin en uregencias por 12-24h
GRADO
0
I

CURSO ENARM CMN SIGLO XXI TEL: 36246001

Pharmed Solutions Institute

PGINA 400

MANUAL DE TRABAJO DEL CURSO ENARM CMN SIGLO XXI


MORDEDURAS DE PERRO (RABIA). CIENCIAS BASICAS: La rabia es una zoonosis de los mamferos causada por el Lyssavirus (virus de la rabia), y se
transmite al hombre principalmente por la saliva de animales infectados, a partir de una mordedura, rasguo o una lamedura sobre mucosa o piel,
con solucin de continuidad. SALUD PUBLICA: El virus est distribuido en todo el mundo. Los perros constituyen el 54% de los trasmisores. Animales
silvestres (42%). Murcilago (4%). Es un problema de salud pblica por cuanto conlleva una mortalidad de 100%. La rabia se presenta en dos
modalidades, considerndose la rabia "urbana" cuando el reservorio predominante es el perro, y "silvestre", si los reservorios son mamferos,
especialmente los quirpteros (murcilagos hematfagos). PATOGENIA: Etiologa, el virus pertenece a la familia Rhabdoviridae. Gnero Lyssavirus.
La protena G es el principal componente antignico y especfico de grupo. Peplmeros es el otro antgeno viral importante, localizado en la
membrana y origina la formacin Ab. La cpside viral est formada por lipoprotenas. Ingresa al organismo por herida o mucosas expuestas a
secreciones que contienen virus vivo. Una primera replicacin en el sitio de la herida en las clulas musculares. Infecta los nervios sensoriales y
motores con una diseminacin centripeta hacia nervios perifricos a travs de axones y clulas de Schwann. A una razn de unos 3mm/hr hasta
alcanzar el SNC. Posteriormente la segunda replicacin se realiza en el sistema lmbico, una nueva replicacin en la neocorteza. La diseminacin a
travs de nervios perifricos hacia otros rganos o tejidos. La respuesta inmunitaria es insuficiente para prevenir la enfermedad, ya que por su
localizacin hace poco accesible a la accin de los mecanismos de defensa. Periodo de incubacin: 95% es inferior al ao. (1 -3 meses).
DIAGNOSTICO: En el periodo prodrmico que dura 1-2 das (fase melanclica), hay fiebre, cefalea, anorexia, fatiga, nusea, vmito. Alteraciones
psquicas: ansiedad, depresin, irritabilidad, agitacin, nerviosismo, aprensin, temor. En el periodo de excitacin que dura de 1-3 das:
hiperactividad, excitacin, desorientacin, hiperreflexia, alucinaciones, convulsiones, salivacin y espasmos laringeos y farngeos. En el periodo
paralitico que es una etapa breve 1 da: disminucin de los fenmenos de excitacin, parlisis de extremidades y pares craneales, alteracin del SNA
(lagrimeo, salivacin), parlisis muscular generalizada.(ascendente). Anatoma patolgica: La lesin patognomnica de la rabia son los corpsculos
de Negri. Son inclusiones citoplasmticas eosinfilicas presentes en el asta de Ammon, corteza y tronco enceflico, clulas de Purkinje. Diagnstico
definitivo: Aislamiento del virus: Saliva, LCR, tejido (cerebro). Demostracin serolgica. Demostracin del antgeno vrico. TRATAMIENTO: Vacunas
antirrbicas: obtenidas por cultivo de clulas diploides (HDCV), clulas VERO o fibroblastos de embrin de pollo (PCEC). Aplicacin, por va
intramuscular, de 3 dosis los das 0, 7 y 21 o 28, en la regin deltoidea. La primera dosis se cuenta como da 0. Cada dosis es de: en casos de vacuna
HDCV, 1 ml, en el caso de vacuna VERO, 0.5 ml, en el caso de vacuna PCEC, 1 ml. Esquema de vacunacin antirrbica en EXPOSICION LEVE: Una dosis
de vacuna, en los das 0, 3, 7, 14 y 28 (30), por va intramuscular en la regin deltoidea en adultos, y en nios pequeos tambin puede aplicarse en
la cara anterolateral externa del muslo. En exposicin grave: inmunoglobulina y la vacuna antirrbica humana, de preferencia el da cero. La
aplicacin de inmunoglobulina antirrbica humana, debe hacerse lo ms cercano al da de la agresin, de no ser as no importa el intervalo
transcurrido, administrndose en nios y adultos como sigue: Inyectar inmunoglobulina antirrbica humana, a razn de 20 U.I/ kg, como dosis
nica. Infiltrar alrededor de la herida, si sta lo permite por su localizacin y extensin la mitad de la dosis total que requiere el paciente. Aplicar el
resto por va intramuscular. PRONSTICO: Es muy grave. Iniciando la sintomatologa su letalidad es el 100%.
CASO CLINICO MORDEDURAS
Nia de 2 aos que acudi a urgencias por mordedura de serpiente
en el malolo tibial derecho, ocurrida haca 90min. Segn la familia,
la serpiente, de color grisceo, meda aproximadamente 30cm. En el
momento del ingreso presentaba edema en el dorso de pie con
extensin hasta el tobillo, calor local y pulso pedio y tibial posterior
presentes, con dolor a la palpacin y movilizacin. Se apreciaban dos
pequeas incisiones puntiformes separadas 1cm entre s en malolo
tibial.
PREGUNTA
Cual es la conducta a seguir?
RESPUESTA
a.- Vigilar posible sndrome compartimental
b.- Corticoide, analgsico y antibitico.
c.- Vigilancia, vendaje compresivo y administracin de suero
antiviperino.
d.- Corticoide, analgsico, gluconato calcio, suero anticrotalico.
CASO CLINICO
Femenino de 15 aos de edad que acude a la consulta por haber
sufrido hace cuatro horas una mordedura del perro de su hermana.
Entre los antecedentes personales de la paciente destaca que est
correctamente vacunada del ttanos (ltima dosis hace seis aos).
En cuanto a los antecedentes del animal, no ha presentado ninguna
enfermedad relevante y est bien vacunado. En la anamnesis la
paciente nicamente se queja de dolor en la zona de la mordedura.
A la exploracin se observan dos heridas poco profundas de unos 3
mm de dimetro en la cara dorsal del antebrazo derecho. El
sangrado por las heridas es escaso. No se observan alteraciones en
ninguno de los siguientes datos: sensibilidad y movilidad del
antebrazo y de los dedos de la mano; pulsos, coloracin y
temperatura del antebrazo.
PREGUNTA
Tras realizar la cura local de la herida, cual es la conducta a seguir
mas adecuada?
RESPUESTA

CURSO ENARM CMN SIGLO XXI TEL: 36246001

a.- Profilaxis con antibiticos tpicos.


b.- Profilaxis antibitica sistemica.
c.- Profilaxis antitetnica nicamente.
d.- Profilaxis inmunoglobulina antirrbica.

CASO CLINICO
Paciente de 10 aos de edad, masculino, raza negra que fue mordido
por una serpiente en la parte superior de la mano derecha, 3 veces
consecutivamente. Inmediato a la mordida el paciente recibi
tratamiento local no profesional y fue referido al centro de
asistencia mdica. Pasadas 10 horas llega al Hospital, donde recibe
atencin mdica. EF: importante edema, rubor y calor en el dorso de
la mano derecha que comienza a extenderse de inmediato al
antebrazo derecho predominando en todo momento los signos
inflamatorios, acompaado de fiebre que comienza a ser constante y
rebelde a los antitrmicos. Se inicia tratamiento inmediato por va
intravenosa y al no contar con el antdoto para la neutralizacin del
veneno de la serpiente se comienza con tratamiento antimicrobiano
de amplio espectro, analgsico y fluido terapia de mantenimiento
con cristales. A las 16 horas aproximadamente despus de la
mordida comienzan a aparecer en la mano grandes ampollas y
flictenas, algunas de las cuales mostraban lquido en su interior,
claro, transparente, dejndose ver con su salida espontnea al
exterior. En esta fase comienza el enfermo a presentar cambios en
su estatus neurolgico: se duerme con ms frecuencia, decado,
refiere intenso dolor con sensacin de ardor, incapacidad para beber
y comer; tambin presenta intenso edema en todo el miembro
superior derecho que se traslada de inmediato al trax, base del
cuello, avanzando el paciente hacia un estado crtico.
PREGUNTA
Cul es el diagnotico mas probable en este caso?
RESPUESTA
a.- Sindrome de shock toxico
b.- Necrolisis epidrmica toxica
c.- Sepsis
d.- Shock anafilactico

Pharmed Solutions Institute

PGINA 401

MANUAL DE TRABAJO DEL CURSO ENARM CMN SIGLO XXI


INTOXICACIONES
CIENCIAS BASICAS: Los agentes que ms afectan a los nios son: medicamentos, sustancias de uso en el hogar (productos de limpieza, quitagrasa y
plaguicidas), hongos y plantas (tienen que ver con costumbres de cada poblacin, medicina tradicional). Los grupos ms afectados son los lactantes y
adolescentes. Los AINES y la cabamazepina desplazaron al AAS y a los barbitricos como agentes comunes de intoxicaciones de nios. PATOGENIA:
El mecanismo de accin es accidental y por intento suicida. Factores predisponentes son: espacios reducidos, abundancia de txicos en casa, mal
almacenados y al alcance, muy importante es la ausencia fsica o emocional de ambos padres y todo esto se traduce como descuido y/o falta de
atencin para los nios y adolescentes. TRATAMIENTO: Para el abordaje del nio intoxicado, se toma en cuenta como en cualquier toro
padecimiento, el estado de gravedad (ATLS, ACLS), antes que el toxico responsable y posteriormente se busca evitar mayor absorcin
(descontaminacin) y al final revertir los efectos del toxico. Primero Estabilizar las constantes vitales: Aire, ventilacin, circulacin, deterioro
neurolgico, eutermia, mantener vigilancia estrecha de posibles alteraciones metablicas (hipoglucemia, desequilibrio acido-base, hipoxemia, flujo
urinario). Evitar mayor absorcin: 1. Va area: retirar al paciente de la zona contaminada y aplicar oxgeno a 100%. 2. Va cutnea: lavado generoso
y cuidadoso, con agua y jabn, el que hace lavado con proteccin mnimo guantes y cubre bocas. 3. Va digestiva: la va de entrada ms comn. En la
ingestin de custicos o derivados del petrleo, no usar medidas heroicas, el dao se establece antes de tener opcin de neutralizar, la accin del
lcali o acido; amen de la reaccin exotrmica que se presenta, incrementa el dao. Situacin semejante en el caso de hidrocarburos, sera
preferible la neumonitis qumica que el riesgo de broncoaspiracin con sus consecuencias. Despus de 4h es menos probable encontrar el toxico en
estmago y entonces el procedimiento debe cambiar. Vaciamiento gstrico (primeras 4 h de ingestin). Emesis: si se provoca lo ideal sera disponer
de jarabe de ipecacuana en todos los hogares y administra 10ml en lactantes y 15mlen escolares; pero si se tiene jabn de tocador y de acuerdo a la
edad, lo ms rpido posible dar 1-2 cucharadas de ralladura del jabn + 1-2 vasos de agua, el vmito se presenta en 10-20 min (recordar que no
debe hacerse en ingestin de custicos, ni hidrocarburos). Lavado gstrico: para mayores de 6 meses, recambios de 50-100ml, hasta obtener liquido
claro, antdoto local: en ocasiones al inicio o al final agregar 1g/kg/ de carbn activado. Gastrodialisis: as se denomina, al uso repetido de carbn
activado+ catrtico (carbn activado 0.5g/kg + Manitol al 20%, 2ml/kg) administrado por sonda nasogstrica c/6h, est contraindicado en ingestin
de custicos, hidrocarburos, leo paralitico, coma, sangrado digestivo. Catrticos: acelerar el trnsito intestinal es un procedimiento que
ocasionalmente se emplea ya que es sustituido por la gastrodilisis. Por ultimo revertir los efectos del toxico (antdotos): Azul de metileno (1mg/kg
dosis diluir en 50ml sol. Salina 0.9%, pasar en 1 h), cido ascrbico (1g dosis c/8h, diluir en 50-100ml para 1 h), Atropina (0.5mg dosis), Fisostigmina
(0.02-0.06mg/kg dosis, diluir en 10ml de sol. Inyectable, pasar muy lento), Naloxona (10/kg/infusin continua), Flumazenil (0.1mg/dosis IV directo,
se puede repetir cada 5min), Pralidozina (30/50 mg/kg diluir en 10ml sol. Glucosada 5%, pasar en 1 h) Atropina+ pralidozina (combinacin usada en
intoxicacin por organofosforados), N-acetilcistena (inicial 140mg/kg-sostn 70mg/kg cada 4h por sonda nasogstrica). Quelantes: D-penicilamina
(50mg/kg dia VO repartir en 3 dosis por 10 dias cada 10 dias. INTOXICACION POR AINES: Tienen en comn ser inhibidores de la sntesis de
prostaglandinas por su accin sobre la enzima ciclooxigenasa. Se emplean como analgsicos, antiinflamatorios, existe gran disponibilidad por lo que
son comunes en las intoxicaciones. El directamente involucrado es el naproxeno. Comparten en mayor o menor grado efectos txicos y afectan
aparatos como digestivo (nausea, vomito, dolor abdominal y sangrado), hematolgico (discrasias sanguneas, anemias, leucopenia, agranulocitosis),
cardiolgico (congestin por retencin de lquidos), neurolgico (debilidad, confusin, somnolencia, insomnio, visin borrosa, convulsiones), renal
(retencin hdrica, de sodio, potasio, falla renal), ginecolgicas (disfuncin uterina, sangrado). Tratamiento: El manejo inicial antes mencionado.
Administrar soluciones: 1500ml/m2 para mantener hidratacin, corregir desequilibrio H-E sol. Salina 0.9% (30ml/kg/dosis). Descontaminacin
vaciamiento gstrico, para evitar absorcin, dosis repetidas de carbn activado ms catrtico manitol al 20%. INTOXICACION POR CAUSTICOS:
Especialmente lo lcalis como el hidrxido de Na (sosa caustica, leja), es el toxico ms peligroso que pueda existir en el hogar. Es conocido que el
tiempo que tarda en producir el dao es de 10 seg y que produce necrosis con licuefaccin de las protenas, afectacin vascular del tejido quemado
que finalmente ocasiona isquemia con dao celular extenso, as la lesin es en profundidad con riesgo de perforacin. Cuando la lesin involucra
todas las paredes del esfago, la cicatrizacin produce estenosis con disfuncin esofgica que en el mejor de los casos la obstruccin se resuelve
con trasposicin de colon pero nunca volver a tener funcin normal. Tratamiento: esteroides, antibitico, estudio radiolgicos, endoscopia.
INTOXICACION POR ORGANOSFOSFORADOS: Grupo de plaguicidas de empleo frecuente en los hogares, se encuentran en varios insecticidas de uso
corriente. Actan bloqueando en forma irreversible la enzima acetilcolinesterasa que es la encargada de catalizar la acetilcolina, romper la molcula
en acetato y colina para su reutilizacin por las terminaciones nerviosas (axones). La presencia de cantidades importantes de acetilcolina en las
terminaciones nerviosas colinrgicas hacen que se presenten 3 sndromes; sndrome muscarinico (broncorrea, sialorrea, epifora, despee rectal y
vesical), sndrome nicotnico (temblores, fasciculaciones, calambres abdominales, debilidad), sndrome neurolgico (crisis convulsivas, miosis,
arreflexia, como). INTOXICACION POR PARACETAMOL: El paracetamol o acetaminofn es un derivado del paraaminofenol, tienen efecto
antipirtico y analgsico, pero sin apenas efecto aniinflamatorio. La absorcin digestiva (oral, rectal), es rpida consiguindose efectos teraputicos y
efecto clnico entre 30minuts y 2 h, despus de una dosis terapeutica de 10-15mg/kg cada 4h (dosis mxima 90mg/kg/dia). Normalmente el 90% del
paracetamol es conjugado en el hgado a glocoronidos o sulfatos, que son eliminados en la orina, y cerca de 2% es excretado en la orina sin cambios.
Aproximadamente el 3-8% es metabolizado en hgado por el complejo enzimtico P450, por procesos de oxidacin, esta ruta metablica crea un
metabolito reactivo toxico, N-acetil, paranezoquinona imina (NAPQI), el cual es rpidamente ligado al glutatin y desintoxicado, si este se acumula
se adhiere a las membranas celulares de los hepatocitos generando la muerte celular y la consecuencia necrosis heptica, tambin puede generar
falla renal aguda. La dosis requerida para producir toxicidad vara segn la funcin de la citocromo P450, se sugiere que con dosis mayor a 150200mg/kg en nios, es potencialmente toxico agudo. La toxicidad crnica se presenta si se ingiere ms de 4 gr de paracetamol al da, luego de 2-8
das. Tratamiento: Iniciar con ABC, lavado gstrico, se recomienda administrar en cada irrigacin la cantidad de 15ml/kg en los nios. El uso de
carbn activado, reduce una media de 52% la curva de concentracin de paracetamol, siempre que se administre dentro de la primera hora postingesta. Nios 0.5g/kg de peso corporal diluidos en 100ml de agua/sulfato de sodio. Menores de 12 aos: 250mg/kg de peso corporal disuelto en
200ml de agua (catrtico). El uso de catartatico est indicado en caso de dosis mltiples de carbn activado. Antdoto por excelencia es la NAC (N
acetilcisteina), cuyo efecto reside, sobre todo, en la posibilidad de regeneracin del glutatin, su indicacin debe ser lo ms precoz posible. Existen
varias pautas de tratamiento con NAC. Va oral: la dosis recomendada es una carga inicial de 140mg/kg seguida de 17 dosis de 70mg/kg cada 4 hrs.
Va intravenosa: dosis inicial de 140mg/kg en una hora. Cuatro horas despus se inician 12 dosis de mantenimiento de 70 mg/kg en una hora cada 4
hrs.
CASO CLINICO INTOXICACION BELLADONA
Se trata de una nia de once aos que presentaba: boca seca,
confusin, habla incoherente, incapacidad para reconocer a los
miembros de la familia, que tambin se presenta vmito
incontrolable, trastornos visuales, auditivos y alucinaciones visuales.

CURSO ENARM CMN SIGLO XXI TEL: 36246001

El examen clnico revel alteracin de la conciencia, coma la escala


de Glasgow (GCS) se evalu a 13/15, las pupilas eran iguales y
reactivas, present polipnea a 26 ciclos por minuto, pero estaba
afebril y hemodinmicamente estable, con ictericia mucocutnea,
reflejos tendinosos eran agudos y difusos. Recuento de sangre de

Pharmed Solutions Institute

PGINA 402

MANUAL DE TRABAJO DEL CURSO ENARM CMN SIGLO XXI


rutina completo, pruebas de funcin renal y heptica revel citolisis
heptica.
PREGUNTA
Cual es la conducta a seguir?
RESPUESTA
a.- Realizar lavado gstrico.
b.- Administrar diazepam 5 mg.
c.- Administrar anticolinergico.
d.- Administrar fisostigmina.
CASO CLINICO ENVENENAMIENTO
Nio de 8 aos de edad, en urgencias 2 horas despus de la
ingestin de una cantidad desconocida de un lquido. Los familiares
refieren que encontraron un frasco abierto, el padre se dedica al
campo, el paciente estaba asintomtico, y los resultados del examen
fsico fueron normales.
PREGUNTA
Cual es la conducta a seguir?
RESPUESTA
a.- Administrar carbn activado.
b.- Mantener en vigilancia.
c.- Enviar a casa y citar al dia siguiente.
d.- Enviar a segundo nivel.
PREGUNTA
El paciente recibi 2 dosis de carbn activado y observacin durante
la noche. Fue dado de alta con un cita. En casa, el paciente present
fatiga generalizada, ingesta oral nula, dolor abdominal,
hematemesis, y disminucin de la diuresis. Sus signos vitales eran
taquicardia leve, y un examen fsico revel sequedad de las
membranas mucosas y el eritema farngeo moderado que estaban
ausentes durante la evaluacin inicial. Cual es las siguientes
substancias es la ms probable que se encuentre involucrada.
RESPUESTA
a.- Organofosforados.
b.- Carbamatos.
c.- Organoclorados.
d.- Paraquat.

CASO CLINICO
Una nia de 13 aos de edad de raza blanca, con antecedente
personal de plipos nasales, presenta broncoconstriccion y
sibilancias graves luego de tomar una dosis grande de aspirina, al
confundirla con unmedicamento para el resfriado. En la sala de
urgencias se encuentra que tiene antecedente de alergia al acido
acetilsaliclico.
PREGUNTA
Cul de los medicamentos que siguen puede usarse con eficacia
para suspender la broncoconstriccion originada por la alergia a dicho
medicamento?
RESPUESTA
a.- Cromoln
b.- Dopamina
c.- Prednisona
d.- Zileutn
CASO CLINICO
Una nia de 8 aos de edad, es llevada a la sala de urgencias debido
a que presento dolor abdominal despus de 10-15 tabletas de hierro
con cubierta de azcar. Al poco tiempode haber llegado presenta
hematemesis. A la exploracin fsica se halla hipotensin y
taquicardia.
PREGUNTA

CURSO ENARM CMN SIGLO XXI TEL: 36246001

Cul es la conducnta teraputica mas adecuada a seguir en este


momento?
RESPUESTA
a.- Acido aminocaproico
b.- Deferoxamina
c.- Dimercaprol
d.- Acido etilendiaminotetraactico
CASO CLINICO
Masculino de 17 aos sin antecedentes personales patolgicos
importantes es llevado a la sala de urgencias por su esposa, quien
informa que el paciente ha babeado. Vomitado, respirado con
dificultad y sudado de manera incrontrolable durante la hora
anterior. La esposa refiere que no toma medicamentos, ni es
alrgico a nada y que trabaja como agricultor. Parece un poco
letrgico y ha tenido problemas para hablar debido a la falta de
aliento, la esposa refiere que estaban aplicando un plaguicida en la
tierra de su patrn. No existen antecdenetes familiares de sntomas
similares, contacto con enfermos, viajes recientes, ni cambios con la
dieta a ultimas fechas. EF: T 37C, TA 100/70mmHg, pulso 40/min,
FR 6 rpm. Se observan pupilas mioticas, ojos llorosos, exceso de
saliva en cavidad bucal y aliento con olor a ajo.
PREGUNTA
Cul es el diagnotico mas probable?
REPUESTA
a.- Intoxicacion por monxido de carbono
b.- Intoxicacion por organofosforados
c.- Intoxicacion por organoclorados
d.- Intoxicacion por lcalis
PREGUNTA
Cul es el dato clnico que no esperarioamos encontrar en este
caso?
RESPUESTA
a.- Angioedema
b.- Fasciculaciones
c.- Broncoespasmo
d.- Defecacion
PREGUNTA
Cul es la conducnta diagnotica mas adecuada a seguir en este
caso?
RESPUESTA
a.- Cloro en plasma
b.- Carboxihemoglobina
c.- Endoscopia
d.- Acetilcolinesterasa eritrocitica

CASO CLINICO
Masculino de 16 aos de edad, es llevado a la salade urgencias por
su madre, despus de haberlo encontrado en su cabaa de invierno
confuso y mareado, tambaleaba por toda la cabaa y se quejaba de
cefalea intensa. La madre refiere que debio haber trabajado mucho
para reparar la cabaa, por que los pisoso estaban barridos, y el
viejo calentador de queroseno funcionaba muy bien. EF: T37, pulso
86/min, TA 126/82mmHg, No esta orientado en cuanto a lugar o a
tiempo y presenta tez rubicunda.
PREGUNTA
Cul es la conducnta teraputica mas adecuada a seguir en este
caso?
RESPUESTA
a.- Oxigeno al 100%
b.- Aplicacin de Atropina
c.- N-acetilcisteina
d.- Esteroides

Pharmed Solutions Institute

PGINA 403

MANUAL DE TRABAJO DEL CURSO ENARM CMN SIGLO XXI


TRAUMATISMO CRANEOENCEFALICO (TCE)
CIENCIAS BASICAS: Lesiones del crneo y su contenido, provocadas por el contacto violento de un agente fsico contra la cabeza o por el choque de
la misma contra una superficie ms o menos dura, o por lo movimientos de aceleracin y desaceleracin sbitos que en forma directa o que por sus
complicaciones, puede llegar a producir la muerte, o dejar como secuelas grados variables de invalidez. Son frecuentes en pediatra debido a las
caractersticas psicomotoras del nio: inquietud, inexperiencia e inters de exploracin, la desproporcin de la cabeza con el cuerpo y su menor talla
en relacin con el adulto son condiciones que lo hacen ms vulnerable a accidentes. SALUD PUBLICA: Primera causa de hospitalizacin en ms de
30% de los casos, implica lesiones de crneo y su contenido. Son ms frecuentes en el sexo masculino por personalidad y caractersticas de
conducta. La etiologa ms frecuente en nios menores de dos aos son las cadas(traumatismo leves desde la cama, de una mesa o al iniciar la
deambulacin. En nios menores de un ao con TCE grave debe sospecharse maltrato. En nios mayores e 2 aos las causas habituales son
accidentes de trafico, bicicleta o deporte. PATOGENIA: El nio tienen alta frecuencia de sufrir dao neurolgico debido a que los huesos del crneo
son ms delgados y tienen menor proteccin, hay menor mielinizacion de las fibras nerviosas y el cerebro en desarrollo en repuesta al trauma ya que
puede desencadenar mayor edema e hipertensin intracraneana. La ventaja que el lactante tiene es que la presencia de fontanelas y la ausencia de
cierre de las suturas permite un poco de mayor elasticidad (compliance) craneal, que pudiera favorecer en algn momento la presencia de
hidrocefalia si el incremento del contenido intracraneal es paulatino y lento. Al tener contacto traumtico con la cabeza, se inicia una serie de
eventos que van a determinar la extensin de la lesin, as como su severidad y secuelas. Primarias: son el resultado directo de la energa fsica
transmitida durante el impacto y cuya gravedad depende del tipo y la cantidad de est traducindose como conmocin, contusin o laceracin
cerebral. Las fuerzas producidas por aceleracin lineal tienden a causar lesin focal, es decir, fracturas, contusin o laceracin. La aceleracin por
rotacin establece en el cerebro fuerzas de arrancamiento que alteran en forma difusa la funcin traducindose como conmocin, lesin axonal y
edema. Secundarias: se originan a partir de los procesos que incrementan los efectos de lesin primaria, son resultado de hipotensin sistmica,
hipercapnia e hipoxia, as como las alteraciones metablicas producidas por el traumatismo que dan mayor lugar a isquemia y edema cerebral. La
primera es hipoxia que se presenta inmediatamente del golpe y que puede aumentar por la obstruccin de vas areas o por fractura, luxacin del
cuello o trax. DIAGNOSTICO: Existen diferentes signos y sntomas de acuerdo al grado de lesin: cefalea (provocada por cambios en el flujo
cerebral), vomito (por hipertensin intracraneana), letargia, somnolencia, delirio, estupor o coma (por cambios a nivel de flujo cerebral), triada de
Cushing (bradicardia, irregularidades en la respiracin e hipertensin arterial sistlica por alteracin de perfusin cerebral), alteraciones de la
memoria, cambios de la personalidad, papiledema, alteraciones de pares craneales (midriasis del lado afectado), alteracin en escala de coma de
Glasgow, convulsiones o focalizacin, anisocoria y alteracin pupilar. Dependiendo de dao cerebral y rea afectada se puede encontrar
descerebracin o descorticacion, proporcional al grado de dao celular e isquemia que haya existido. Los movimientos oculares (oculoenceflicos y
oculovestibulares), evalan la regin media cerebral en el rea del sistema reticular ascendente y la respuesta motora usando 5 niveles
internacionales se usan: normal 5, debilidad moderada 4, debilidad severa 3, trazas de movimiento 2, flacidez 1. La exploracin externa de la cabeza
nos puede dar datos de fractura importante de la base del crneo la anterior: equimosis periorbitaria signo de mapache. Medio: rinorraquia,
otorragia o otorraquia. Posterior: Signo de Battle, equimosis retroauricular. La exploracin de pares craneales indican un proceso expansivo
intracraneal, asimetra de la respuesta motora de los miembros, indica distorsin de los pednculos cerebrales o bien compresin sobre el tallo
cerebral. Este examen se puede realizar en menos de 5 min y permite clasificar al paciente peditrico segn el grado de trauma. El valor de la
radiografa de crneo enel traumatismo craneal est siendo cada vez ms cuestionado, de forma que no se recomienda su uso en la mayora de
situaciones de trauma craneal si la TAC est disponible. CLASIFICACION: Trauma leve o GRADO I: Escala de coma de Glasgow 15-13 puntos, es aquel
paciente que posterior al trauma no presenta ninguna manifestacin clnica, o bien, tiene una perdida transitoria de la memoria que durara algunos
minutos u horas pudiendo estar asociado a prdida del estado de conciencia en forma tambin transitoria (segundos a minutos) y no existe por lo
general alteracin en la escala de coma de Glasgow, se ha asociado a sndrome pos conmocin donde el paciente presenta episodios de cefalea,
irritabilidad y falta de concentracin, en nios ms grandes se puede asociar agresin, ansiedad, baja atencin y cambio de conducta, se presenta
meses posteriores al trauma y es autolimitado. Se ha descrito una triada en trauma leve: somnolencia, irritabilidad y vomito; que ocurre en 48-72 h
posteriores al trauma se cree que es debido a leve torsin del tallo cerebral. Trauma moderado o GRADO II: Escala de coma de Glasgow de 129puntos, perdida del estado de alerta menor a 5 min, tendencia a la somnolencia sin dficit neurolgico. Trauma severo o GRADO III: Escala de coma
de Glasgow 8-3 puntos. Perdida del estado de alerta mayor a 5 min, incapacidad para obedecer rdenes, confusin mental, lenguaje incoherente e
inapropiado, anisocoria o lenta respuesta pupilar; la respuesta motora puede variar a la localizacin del dolor o posturas anormales. Muerte
enceflica o GRADO IV. Cabe mencionar que hay ciertas condiciones en un TCE leve que puede transformarlo en moderado o severo por lo que el
examen debe ser cuidadoso y si existe deterioro rpido de la escala de coma de Glasgow manifestando un conjunto de sndrome de deterioro
rostro-caudal. TRATAMIENTO: A la llegada a urgencias de un TCE, se debe realizar una valoracin rpida y ordenada de la situacin del paciente
(ABCD). Si existe alteracin de la conciencia o se presume, por la historia o la exploracin, que el TCE puede ser moderado o grave. Asegurar la
permeabilidad de la va area. Administrar oxgeno. Monitorizar al paciente. Canalizar una va venosa perifrica. Ante la presencia de inestabilidad
respiratoria y/o hemodinmica, se proceder con las maniobras de RCP. En los casos con alteracin de con ciencia importante (Glasgow < 9) ser
necesario asegurar la va area mediante la intubacin orotraqueal. Objetivos del tratamiento: PIC <20mmHg , PAM (normal para edad),
PPC > 50mmHg,
Sat O2 > 95% con PCO2 3540mmHg.
CRIERIOS
DE
HOSPITALIZACION
DE
TCE:
Alteracin de los signos vitales:
Convulsiones postraumticas. Funciones mentales alteradas. Perdida de la conciencia prolongada. Dficit de memoria persistente. Signos
neurolgicos focales. Fractura craneal deprimida. Fractura craneal basilar. Edema de piel cabelluda amplio. Cefalalgia severa persistente,
especialmente con rigidez de nuca. Vmitos persistentes. Fiebre inexplicable. Anormalidades neuroradiologicas que sugieran abuso infantil.
CASO CLINICO
Paciente escolar, masculino, blanco, de 11 aos de edad, con
antecedentes de salud, que mientras jugaba en su poblado natal, fue
atropellado por un vehculo, recibiendo traumatismos mltiples a
predominio craneal. Lleg al servicio de urgencias manejado por el
sistema integrado de urgencias mdicas, en estado de coma.
Examen fsico al ingreso: Coma moderado, pupilas isocoricas y
reactivas, hemiparesia izquierda, herida contusa frontal medial y
hacia la izquierda, con salida al exterior de tejido cerebral y lquido
cefalorraqudeo (LCR). Exmenes complementarios al ingreso:
Hemoglobina: 130 g/l, tiempo de coagulacin: 7 minutos, tiempo de
sangramiento: 2 minutos. Tomografa axial computarizada (TAC) al

CURSO ENARM CMN SIGLO XXI TEL: 36246001

ingreso: Se aprecia fractura del hueso frontal hacia la izquierda con


fragmentos que penetran el tejido cerebral, focos de contusin
hemorrgica diseminados por regiones frontales polares de ambos
hemisferios cerebrales.
PREGUNTA
Cual es el grado de TCE que presenta el caso?
RESPUESTA
a.- Grado I.
b.- Grado II.
c.- Grado III.
d.- Grado IV.

Pharmed Solutions Institute

PGINA 404

MANUAL DE TRABAJO DEL CURSO ENARM CMN SIGLO XXI


TUMORES DEL SNC
CIENCIAS: Representan la segunda causa de neoplasias en la infancia. Los tumores cerebrales primarios son un grupo diverso de enfermedades que,
constituyen el tumor solido ms comn de la niez. Los tumores ms frecuentes son tres: astrocitoma cerebeloso (33.9%), el meduloblastoma
(26.3%) y el glioma del tronco cerebral. Otros tumores importantes de considerar son el craneofaringeoma, tumeor de la regin pineal y
ependimoma. SALUD PUBLICA: En Mxico no existen estadsticas, pero se estima 12%. En el INP, estos tumores ocupan el tercer lugar. Los tumores
del sistema nervioso central (SNC) constituyen la segunda causa de muerte en los menores de 15 aos, tan slo superada por la leucemia.
PATOGENIA: Se desconoce la causa de los tumores cerebrales infantiles pero existe una serie de factores predisponentes como: 1.- Genticos.Neurofibromatosis, Esclerosis Tuberosa , Enfermedad de von Hippel-Lindau, Sndrome de Li-Fraumeni, Sndrome de Turcot y el Sndrome de Nevo
con Carcinoma de Clulas Basales, este ltimo y aquellos que cursan con Tumor de Wilms son ms propensos a desarrollar meduloblastoma. 2.Inmunolgicos.- Inmunosupresin congnita o adquirida, individuos que sern sometidos a transplante renal ya sea antes o durante el mismo tienen
350 veces ms riesgo de desarrollar Sarcoma de clulas reticulares y Linfomas, los pacientes con ataxia-Telangiectasia cursan con alteraciones
inmunolgicas y mayor riesgo de desarrollar neoplasias. 3.- Ambientales.- Exposicin a Hidrocarburos Aromticos, compuestos N-nitrosos, triazinas
e hidrazinas. Una asociacin menos documentada es el consumo, durante el embarazo de barbitricos, exposicin prenatal a rayos X, trauma,
infeccin y anestsicos. El cncer se produce por mutaciones de genes que regulan la proliferacin y muerte celular. Las mutaciones genticas
pueden ocurrir dentro de la lnea germinal o como mutaciones somticas exclusivamente dentro de clulas tumorales. Solo una pequea fraccin de
nios con Tumores cerebrales tienen mutaciones germinales adquiridas de sus padres o mutaciones nuevas. La causa de las mutaciones somticas
en la mayora de todos los tumores cerebrales son desconocidas. Existe evidencia de que el crecimiento y diseminacin de una neoplasia es
dependiente de la angiognesis (formacin de nuevos capilares sanguneos a partir del propio tumor lo que incrementa las posibilidades de
incorporacin de clulas neoplsicas a la circulacin). Cada incremento sucesivo en el volumen del tumor es precedido por una fase de angiognesis.
CLASIFICACION: De acuerdo a la localizacin: existe un predominio de los tumores infratentoriales (43.2%), seguido de los tumores supratentoriales
(40.9%), los de mdula espinal (4.9%) y de sitios mltiples (11%). Infratentoriales y, de estos, 75% estn situados en el cerebelo o cuarto ventrculo.
Entre los tumores infratentoriales comunes (fosa posterior) se encuentran los siguientes: 1.- ASTROCITOMA CEREBELAR: tumor de mejor pronstico,
generalmente de bajo grado. Existe un tumor especfico denominado astrocitoma Pilocitico Juvenil, de excelente pronostico con terapia quirrgica.
No requiere radioterapia, estos tumores se ubican en los hemisferios cerebelosos y generalmente son qusticos. La porcin tumoral en realidad es un
ndulo mural. La cavidad qustica visible en TAC es reactiva. 2.- MEDULOBLASTOMA: forma parte de los tumores neuroectodrmicos primitivos, por
originarse de esta clula primordial, son muy agresivos y por ende de mal pronstico, puede dar metstasis incluso va LCR, se origina en la primera
dcada de la vida en el techo del IV ventrculo, por lo cual puede causar hidrocefalia precoz, clnicamente se presenta con cefalea y vmitos, los
signos sern relacionados con un sndrome cerebeloso, un nio con frecuentes cadas puede cursar una ataxia. Es de los tumores que con ms
frecuencia presenta diseminacionm extraneural (menos del 4%), siendo el hueso, mdula sea, linfticos, hgado y pulmones los sitios ms comunes.
Tratamiento quirrgico, radio y quimioterapia. 3.- EPENDIMOMA: ocurren mas frecuente en la regin del torax. En raras ocasiones presentan
malignizacion y en muchas ocasiones presentan asociacin con quistes intramedulares. Es un tumor relativamente avascular y son encapsulados, es
decir tienen un plano de clivaje que permite su extirpacin completa y curacin de los pacientes. Un grupo de ellos crece en el interior de la medula
y otro en el cono medular. 4. GLIOMA DEL TALLO ENCEFLICO: puede ser de distintos grados, en general de mal pronstico dado su natuiraleza
infiltrativa. Puede estar a nivel alto manifestndose por sndrome cerebeloso e hidrocefalia. Pueden ubicarse a nivel bajo (bulbo medular),
manifestndose por compromiso multiple de pares craneales bajos y por compromiso motor y sensitivo de extremidades. La radioterapia asociada a
corticoides, mejoran la calidad de vida. 4. Tumor teratoide atpico. Los tumores supratentoriales incluyen aquellos que ocurren en la regin selar o
supraselar del cerebro y en otras reas del mismo. Los tumores selares y supraselares representan aprox., el 20% de los tumores cerebrales
infantiles, incluso los siguientes: 1. Craneofaringoma. 2. gliomas diencefalicos. 3 Tumores de clulas germinales. Otros tumores que ocurren de
forma supratentorial incluyen los siguientes: 1. Astrocitoma o glioma de bajo grado. 2. Astrocitoma de alto grado o maligno. 3. Glioma mixto. 4.
Oligodendroglioma. 5. Tumor neuroectodermico primario. DIAGNOSTICO: No existen cuadros patognomnicos para el diagnstico de tumores
cerebrales. Generalmente el inicio es insidioso y progresivo. Las manifestaciones clnicas de los tumores cerebrales infantiles depende de: 1.Localizacin del tumor (zonas elocuentes, obstruccin o desplazamiento) 2.- Tipo histolgico del tumor. 3.- Edad y desarrollo del paciente. Los
tumores intracraneales pueden manifestarse como dficit neurolgico focal, crisis convulsivas o no convulsivas, aumento de la presin intracraneal,
que puede deberse a efectos directos del tumor u obstruccin del lquido cefalorraqudeo. Un anlisis de los sntomas tempranos de los tumores
cerebrales infantiles mostr que los tumores supratentoriales se presentan con vmito en el 46 % de los casos, con cefalea en el 43% de los casos;
mientras que los Tumores Infratentoriales se presentan con dificultades para la coordinacin en el 59 %, vmito 76% y cefalea 56 %. Debe
sospecharse de un tumor cerebral en todo nio con datos clnicos neurolgicos insidiosos y progresivos. DESCRIPCION SINDROMTICA: I)
Hipertensin Intracraneana: este cuadro puede manifestarse de manera aguda o crnica: Agudo: < 1 ao incremento inesperado del permetro
ceflico, separacin de suturas, alteraciones del estado de alerta (somnolencia, irritabilidad) disminucin en la ingesta. En preescolares, escolares y
adolescentes puede manifestarse con cefalea y vmito generalmente matutino que se exacerba con maniobras de Valsava y disminuye en el
transcurso del da. El vmito puede ser en proyectil, irritabilidad, letargia, edema de papila, discromatopsia (prdida de la visin de colores),
escotomas centrales y la paresia del sexto par, este ltimo inespecfico de topografa y explicado por la longitud del trayecto (falso focalizador).
Crnico o Intermitente: irritabilidad, letargia, vmito, atrofia de papila o prdida progresiva de la visin, cambios conductuales, de personalidad, del
rendimiento acadmico, anorexia y prdida o ganancia ponderal. II) Sndrome cefallgico: La cefalea en tumores clsicamente se describe con un
curso subagudo de 4 a 6 meses de duracin, de tipo pulstil, continua o intermitente. Despierta al nio por la noche, frontal, occipital o universal, de
moderada intensidad medida por escala anloga visual del dolor, o porque el nio deja de hacer sus actividades. III) Crisis: Las crisis en los tumores
generalmente son focales y pueden asociarse a patrones bioeltricos anormales focales. IV) Sndrome cerebeloso: Este puede ser vermiano con
ataxia troncal o de la marcha, hemisfrico con lateropulsin derecha, izquierda o indistinta, dismetra, disdiadococinesia, lenguaje escndido
(disartria), reflejos osteotendinosos pendulares con o sin nistagmus horizontal. V) Neuropata craneal: El involucro de pares craneales dependiendo
de cul sea, es sugestivo de la topografa de la lesin sin embargo esto puede ser muy sutil en virtud de que los tumores son ms compresivos que
destructivos. Entendiendo por neuropata del III al XII par craneal. Los estudios necesarios para realizar el diagnostico son la TAC, siendo posible
realizar el diagnostico hasta en 90%, la imagen por IRM, mielografa, mielotomografa, en caso de tumor como el meduloblastoma y ependimoma:
siendo el diagnstico definitivo el estudio histopatolgico, obtenindose a travs de reseccin quirrgica, con una toma de biopsia y exresis del
tumor, as como por biopsias por esterotaxia. Existe una ruta crtica para la evaluacin de un nio con sospecha de neoplasia en SNC: Sospecha de
tumor primario estudios de laboratorio y gabinete interconsulta con neurociruga y oncologa procedimiento diagnostico-quirrgico
clasificacin histopatolgica estadificacion valoracin radioterapia valorar inicio de esquema de quimioterapia seguimiento.
TRATAMIENTO: Los cnceres dejados a su evolucin natural son incontrolables y mortales en la mayora de los casos. Se basa en la desaparicin de
los sntomas, as como el tratamiento especfico, por lo que es importante individualizar cada caso, algunos pacientes requerirn el empleo de

CURSO ENARM CMN SIGLO XXI TEL: 36246001

Pharmed Solutions Institute

PGINA 405

MANUAL DE TRABAJO DEL CURSO ENARM CMN SIGLO XXI


anticomisiales (Fenitona 20 mg/kg/dosis de impregnacin y mantenimiento 7 a 10mg/kg/da en dos dosis), as como de esteroides para el manejo
de hipertensin craneal, ya que algunos autores mencionan que los esteroides tienen efecto vaso gnico, aumentando la permeabilidad vascular,
corrigiendo e edema cerebral y por tanto mejorando el flujo circulatorio de forma transitoria. En cuanto al tratamiento especfico, la ciruga debe
realizarse por el especialista experto en patologa tumoral. En ocasiones los pacientes requieren colocacin de vlvulas ventriculoperitoneales para
el manejo urgente de la hidrocefalia y posteriormente la ciruga de reseccin que siempre es recomendable sea completa, pero suele ser imposible.
La radioterapia es una modalidad de tratamiento necesaria para los tumores malignos de SNC, los pacientes menores de 3 aos presentarn
detencin del crecimiento y la posibilidad de retardo psicomotor, disfuncin endocrina y radionecrosis. La quimioterapia es usada para las
neoplasias del SNC, pero no todas son sensibles debido a la barrera hemartoencefalica
CASO CLINICO
Paciente de 9 aos de edad, sexo masculino, que consult por
cuadro de 3 das de evolucin caracterizado por cefalea holocrnea
de tipo opresivo y vmitos explosivos aproximadamente 6 veces al
da. Durante la anamnesis la madre refiri que el paciente
presentaba crisis de cefalea intermitente hace aproximadamente 5
meses, las que con el tiempo fueron aumentando en frecuencia e
intensidad hasta constituir el cuadro de consulta actual. Al examen
fsico general el nio impresionaba en buenas condiciones generales,
y se apreciaron varias lesiones vesiculares y costrosas en todo el
cuerpo, compatibles con cuadro de varicela en sus ltimas etapas. Al
examen neurolgico destac paciente con un puntaje de 15 en la
Escala de Glasgow, dismetra de extremidad superior derecha y leve
paresia facial izquierda.
PREGUNTA
Cual es la conducta inmediata a seguir mas adecuada?
RESPUESTA
a.- Realizar puncion lumbar.
b.- Realizar tomografa computada.
c.- Medidas anti-edema.
d.- Colocacion de derivacin ventrculo peritoneal.
CASO CLINICO
Se trata de RN femenino, nacido por cesarea con 38 SDG por
Capurro, apgar 8/9, sin datos de hidrocefalea, permetro ceflico de
39.5 cm. Fontanelas anterior y posterior ampliar y plenas, con
diastasis de la sutura sagital, tono muscular cervical disminuido,
cardiopulmonar sin compromiso. Movilidad expontanea en las
cuatro extremidades, discreta hiperreflexia miotatica, babinski
espontaneo bilateral y reflejos primitivos globales disminuido, La TC
mostro hidrocefalia supratentorial sin edema transependimario,
dilatacin de tercer ventrculo y en la fosa posterior una imagen
hiperdensa ovalada de 46 x 42 mm que obliteraba el cuarto
ventrculo en su parte posterior, se reporto una lesin tumora
infratentorial probablemente intraaxial glial o extraaxial de origen
mesenquimatoso. La RM reporto imagen hiperdensa en relacin con
el parnquima cerebral en la regin pancerebelosa que infiltraba el
tallo cerebral en el puente, mesencfalo y lamina cuadrigeminal, que
protruia por la hendidura tentorial enceflica y borra las foliar
cerrebelosas. Por su efecto compresivo sobre acueducto de Silvio,
adems de ocacionar la dilatacin ventricular supratentorial con
hidrocefalia obstructiva.

PREGUNTA
Cul es la conducta diagnostica ms adecuada a seguir en este
caso?
RESPUESTA
a.- Radiografa de crneo
b.- TAC de crneo
c.- Arteriografa
d.- Espectroscopia
PREGUNTA
Cul es el diagnstico ms probable para este caso?
RESPUESTA
a.- Meduloblastoma
b.- Astrocitoma cerebeloso pilocitico
c.- Teratoma
d.- Ependimoma
PREGUNTA
Cul es la localizacin que esperaramos encontrar en este
paciente?
RESPUESTA
a.- Supratentorial
b.- Infratentorial
c.- Medula espinal
d.- Tentorial
PREGUNTA
Cul es el pronstico para este paciente a 5 aos, si se le realizo
ciruga y se le dio radioterapia?
RESPUESTA
a.- 41%
b.- 13%
c.- 80%
d.- 65%
CASO CLINICO
Masculino de 18 aos de edad, el cual ha sido diagnosticado con un
glioblastoma multiforme. El neurlogo tratante debe escoger un
medicamento apropiado como teraputica para el tumor.

PREGUNTA
Cual es el diagnostico mas probable?
RESPUESTA
a.- Astrocitoma congnito.
b.- Meduloblastoma congnito.
c.- Ependimoma congnito.
d.- Craneofaringeoma congnito.
CASO CLINICO
Paciente femenina de 6 aos de edad, con antecedentes patolgicos
personales de salud anterior, acude a la consulta de neurologa
referida de su rea de salud, porque desde hace 7 das comenz con
un cuadro de cefalea pulstil, generalizada en toda la cabeza, con

CURSO ENARM CMN SIGLO XXI TEL: 36246001

sntomas, en ocasiones, de vmitos e inestabilidad en la marcha, la


cual se fue intensificando en los ltimos tres das. Por tal motivo, se
le realiza un examen fsico general y neurolgico; se constata una
tortcolis derecha con una marcha atxica y lateralizacin hacia la
derecha y dismetra del miembro superior derecho. Se le indica un
fondo de ojo, detectndole un papiledema bilateral, con exudados y
microhemorragias.

PREGUNTA
Cul de los siguientes es mas probable que atraviese la barrera
hematoencefalica y sea mas efectivo para este paciente?
RESPUESTA
a.- Citarabina
b.- Etopsido
c.- Nitrosoureas
d.- Alcaloides de la vinca

Pharmed Solutions Institute

PGINA 406

MANUAL DE TRABAJO DEL CURSO ENARM CMN SIGLO XXI


MENINGITIS
CIENCIAS BASICAS: Es un proceso inflamatorio agudo del SNC causado por microorganismos que afectan a las leptomeninges (aracnoides y
piamadre). El sistema nervioso puede infectarse por diferentes microorganismos, incluyendo bacterias, virus, hongos, protozoos y helmintos. La
presentacin clnica de estas infecciones puede ser aguda, subaguda o crnica dependiendo de la etiologa, la virulencia del microorganismo y la
localizacin del proceso infeccioso. SALUD PUBLICA: Un 80% ocurre en la infancia, especialmente en nios menores de 10 aos. En la ltima dcada,
con la introduccin de nuevas vacunas frente a los grmenes causales ms frecuentes y con el desarrollo de antibiticos ms potentes y con buena
penetracin hematoenceflica, ha disminuido la incidencia y ha mejorado el pronstico de la infeccin, pero las secuelas y la mortalidad no han
sufrido grandes cambios. MENINGITIS VIRAL (95%): Los virus que se detectan con ms frecuencia en meningitis asptica son los enterovirus (EV),
seguidos de virus herpes simple (VHS) y varicela zoster (VZV). Entre los muchos tipos de EV causantes de brotes de meningitis, destacan Echovirus
30, 13, 6, 11 y 9, Coxsackie B5 y Coxsackie A9. Los EV son tambin la causa principal de meningitis y sepsis neonatal. PATOGENIA: El ms general
sera la colonizacin del SNC a travs de diseminacin hematgena del virus durante la primoinfeccin vrica. En el caso de los alfa herpesvirus un
segundo mecanismo, cuyo ejemplo ms tpico sera la encefalitis herptica, consistira en la invasin del SNC tras reactivacin de la infeccin latente
desde los ganglios nerviosos regionales a travs de las fibras nerviosas. DIAGNOSTICO: Las meningitis de carcter vrico suelen tener un curso ms
insidioso, con escasa afectacin del estado general, fiebre de intensidad generalmente moderada (en ocasiones ausente) y cefalea que puede ser
intensa. La puncin lumbar puede generar un alivio transitorio de los sntomas. En el caso de la meningoencefalitis vrica, se describe una triada
clsica: fiebre, cefalea y alteracin conciencia. El LCR aunque muetsra pleocitosis generalmente no rebasa de 200-500 celulas y de predomino de
mononucleares. Los estuidos serolgicos o de estudio de virus esclarecen el diagnostico. MENINGITIS BACTERIANA (5%): Se caracteriza por la
presencia de signos y sntomas menngeos y enceflicos con grados variables. La morbilidad en Mexico es de 50% de los casos, se presenta en nios
de 3 meses a 3 aos de edad y su mortalidad varia de 5-15 %, las secuelas son retraso psicomotor leve hasta retraso mental grave, sordera,
convulsiones e hidrocefalia, varan de acuerdo a edad, germen causal, oportunidad y eficacia del tratamiento. Agentes causales en RN a 3 meses son
bateras gramm negativas (E. coli y Klebsiella), estreptococo beta hemoltico del grupo B, Listeria monocytogenes y meningococos. De 3 meses a 5
aos son: Haemophilus influenzae del tipo B, Streptococo pneumoniae, Neisseria meningitides, neumococos y meningococos. PATOGENIA: La mayor
parte de los casos inician con una bacteriemia a partir de focos infecciosos, respiratorios, gastroinetstinales y urinarios, tambin ocurre diseminacin
directa de un foco paramenngeo por ejemplo: pansinusitis u otitis recurrente donde los grmenes involucrados son diferentes de acuerdo al
proceso infeccioso primario, tambin suele ocurrir por invasin directa como en el caso de fracturas de crneo, disrrafas de la lnea media,
procedimientos neuroquirurgicos, colocacin de catteres. Unas vez que ocurre la bacteriemia existe un proceso inflamatorio del endotelio de los
vasos cerebrales con aumento del tamao de los poros o puentes intercelulares, lo que permite el paso de molculas de gran tamao (protenas y
bacterias) al intersticio del parnquima cerebral (rotura de la barrera hematoencefalica) y se produce un edema cerebral vasgeno que repercute en
la entrega de oxgeno a las neuronas, as como edema cerebral de tipo citotxico, irritando las meninges. La inflamacin y el dao celular neuronal
causan metabolismo anaerbico con produccin de lactato, consumo de glucosa y elevacin de protenas en el LCR. DIAGNOSTICO: La
sintomatologa vara segn la edad del paciente en RN: inespecfica, sospechar en RN con sepsis, fiebre, hipotermia, irritabilidad, rigidez de nuca,
rechazo al alimento, vmito, diarrea y convulsiones (40%). Lactante: sospechar cuando hay infeccin respiratoria o gastrointestinal de varios das de
evolucin con fiebre persistente, vomito, irritabilidad, convulsiones, fontanela anterior tensa y abombada (es un signos tardio presente en 20%),
PARAMETRO
NORMAL
M. BACTERIANA
M. VIRICA
M. TUBERCULOSA
rechazo a la va oral o bien rigidez de nuca,
Presin de apertura
70-150mm
Alta
N/alta
Alta
hiperreflexia, signos de Kerning (dolor de espalda
Aspecto de LCR
Turbio o purulento
Claro
opalescente
con la extensin pasiva de la rodilla estando los
Glucosa (mg/dl)
60-80% de la
<66% de la sangunea
Normal
<40mg/dl
muslos flexionados) y Brudzinsky (flexin
glucemia
(hipoglucorraquia)
espontnea de los miembros inferiores al flexionar
Cels/mm3
<10 (MMN)
>1000 (PMN)
<300
<1000
pasivamente el cuello), de Babinski y cefalea. Los
Protenas (mg/dl)
<45
Aumentadas ++
N o aumentada+
Aumentada +++
(100-500mg/dl)
isgnos de focalizacin neurolgica hemiparesia,
cuadriparesia y paralisis facial pueden ocurrir temprana o tardamente en 15% de los casoso por trombosis venosa o arterial, secundaria e
inflamacin. Laboratorio, BH reporta leucocitosis con predominio de neutrfilos y bandas, as como Hb baja. Debe tomarse cultivo de orina,
electrolitos sricos para corregir el balance hdrico. QS, para verificar la cuantificacin de glucosa en sangre, pruebas de coagulacin, cuantificacin
de plaquetas. Puncin lumbar para citologa de LCR con raquimanometro para registrar la presin de apertura del LCR (normal 70-150mm). Se hacen
pruebas de diagnstico rpido como coaglutinacin en ltex que permite un buen porcentaje de sensibilidad u especificidad existiendo reactivos
para detectar H. influenzae tipo B, Streptococo pneumoniae, N. meningitidis, E. coli. Se debe tomar hemocultivo e iniciar de inmediato el
tratamiento antimicrobiano. La TAC est indicada en : 1. Pacientes con neuroinfeccin en quienes se sospecha alguna complicacin como higroma
subdural , absceso etc. 2. Pacientes con persistencia de alteraciones neurolgicas a pesar de un tratamiento integral adecuado. 3. En quienes
presentan datos de focalizacin, hipertensin intracraneana o infeccin por enterobacterias. TRATAMIENTO: Varia segn resultado de cultivo de LCR
y hemocultivo, sin embargo se sugiere iniciar tratamiento de acuerdo a la edad. RN: ampicilina 150-200mg/kg/da + gentamicina 7.5mg/kg/da
amikacina 22mg/kg/da cefotaxima 150-250mg/kg/da. Nios mayores de 3 meses: ceftriaxona 100mg/kg/da, cefotaxima 200mgs/kg/da,
cloranfenicol 75-100mg/kg/da. Preescolares y escolares: penicilina 250-300 000/kg/da + cloranfenicol 75-100mg/kg/da. MENINGITIS FIMICA:
Puede representar alrededor del 1% de todas las formas y es ms frecuente en poblacin de pases subdesarrollados, nios y pacientes infectados
por el VIH. La localizacin menngea puede producirse por va hematgena durante la primoinfeccin o la reactivacin, o bien por ruptura al espacio
subaracnoideo de un foco paramenngeo ya existente. En cualquier caso se produce una inflamacin granulomatosa de las meninges basales y
pequeos focos tuberculosos (focos de Rich) en las meninges, el cerebro o la mdula espinal. . Los sntomas ms frecuentes son fiebre y cefalea de
curso subagudo o crnico, vmitos, decaimiento, rechazo de las tomas, aadindose posteriormente disminuciones del nivel de conciencia y/o
alteraciones de la conducta, otros sntomas y signos de hipertensin intracraneal, afectacin de los pares craneales (III,VI,VII), hidrocefalia, y
diferentes formas de alteraciones neurolgicas focales. El cuadro clnico, por lo general es de instalacin lenta (2-3semanas) y por lo reguilar debe
existir un estudio de Combe positivo. MENINGITIS MICOTICA: Son raras evolucionan lentamente, se presentan a menudo en pacientes
inmunodeprimidos o con desnutricin crnica. El estudio citoqumico de LCR es indistinguible del de una meningitis fimica. Posponer puncin
lumbar: Inestabilidad hemodinmica. Insuficiencia respiratoria. Ditesis hemorrgica (<100.000 plaquetas). Hipertensin intracraneal confirmada.
Infeccin superficial en el lugar de la puncin.
CASO CLINICO MENINGITIS
Una adolescente de 14 aos previamente sana que acudi en estado
comatoso. Tres das antes haba comenzado con tos, fiebre, rinorrea
y cefalea ocasional, siendo diagnosticada de catarro y tratada con

CURSO ENARM CMN SIGLO XXI TEL: 36246001

antitrmicos y antihistamnicos. Unas horas antes del ingreso


present varios episodios de prdida de fuerza de los 4 miembros,
cefalea intensa y fiebre de 38 C. En el momento previo al ingreso la
madre la haba encontrado apoyada en la pared realizando

Pharmed Solutions Institute

PGINA 407

MANUAL DE TRABAJO DEL CURSO ENARM CMN SIGLO XXI


movimientos incoordinados de las 4 extremidades, sin respuesta a
estmulos verbales, mirada prdida y despus cada al suelo, prdida
de conciencia y relajacin de esfnteres. Temperatura, 37 C; presin
arterial, 130/70 mmHg; frecuencia cardaca, 110 lat./min; frecuencia
respiratoria, 19 resp./min; saturacin de oxgeno, 100 %; coloracin
normal de piel y mucosas; desconectada del medio; Glasgow de 8;
hipotona generalizada; reflejos osteotendinosos normales; pares
craneales normales; pupilas midriticas pero reactivas, y signos
menngeos negativos. El resto de la exploracin fsica fue normal.
Lquido cefalorraqudeo: protenas, 53 mg/dl; glucosa, 60 mg/dl;
leucocitos, 5/ ml.
PREGUNTA
Cual es el agente etiolgico mas frecuente considerando la edad y
el cuadro clnico?
RESPUESTA
a.- Streptoccoco del grupo B, E. coli listeria.
b.- Haemophilus influenzae, Straptococo pneumoniae E. coli.
c.- Streptococcus pneumoniae neisseria meningitidis.
d.- Haemophilus influenzae, neisseria meningitidis streptococcus
pneumoniae.
CASO CLINICO MENINGITIS
Lactante de 8 meses. En la ltima semana presentaba cuadro febril
de hasta 39C en el contexto de una gastroenteritis aguda. Tras
presentar respiracin superficial y bradicardia, precisa intubacin y
conexin a ventilacin mecnica. Se practica TAC craneal en el que
destaca hidrocefalia tetraventricular por lo que se decide colocacin
de derivacin ventricular externa con salida de lquido turbio con
glucosa ligeramente disminuida (0,35g/l) y protenas elevadas
(0,57g/l). La citologa muestra un perfil bacteriano ms por la
frmula que por el nmero total de clulas (240 clulas nucleadas
con 80% de segmentados y 20% de linfocitos). Destaca la emisin de
abundante orina transparente en una nia con deshidratacin leve
(3%) hasta ese momento y cuadro diarreico de una semana de
evolucin, que se intensifica a la vuelta de quirfano llegando a
presentar poliuria de 30cc/kg/h. Destacan hipernatremia de
153mEq/l, hiponatriuria de 11,9mEq/l, densidad urinaria de 1003,
osmolaridad urinaria de 110mosmol/kg, osmolaridad plasmtica de
318mosmol/kg y poliuria con acentuacin de la deshidratacin a
pesar de reposicin horaria de la diuresis.
PREGUNTA
Cual es la complicacin mas probable?
RESPUESTA
a.- Falla renal.
b.- Sindrome de secresion inapropiada.
c.- Diabetes inspida central.
d.- Necrosis hipofisiaria.
CASO CLINICO
Un nio de 11 meses de edad ingresa a la sala de urgencias con un
antecedente de letargo. Fue atendido por su pediatra 3 dias antes
por fiebre y sntomas de infeccion de vas urinarias. Se le diagnotico
otitis media y se le trato con amoxicilina por via oral. Esta maana se
torno irritable y estuvo menos activo que lo habitual. Ha vomitado 3
veces su gasto urinario esta notablemente disminuido. No tiene
diarrea la temperatura es de 40.2, FC 100lpm, FR 20rpm, TA
110/80mmHg. Esta letrgico y solo se le puede despertar con
estimulos dolorosos. EF: fontanela anterior, llena y tensa, las
memebranas timapanicas son rojas y abultadas. Las pupilas son
reactivas pero los ojos no enfocan a sus padres. Los reflejos
osteotendinosos profundos son hiperactivos.

CURSO ENARM CMN SIGLO XXI TEL: 36246001

PREGUNTA
Cul es el diagnotico ms probable en este caso?
RESPUESTA
a.- Meningitis
b.- Sepsis
c.- Tumor cerebral
d.- Encefalopatia
PREGUNTA
El LCR muestra 5000 leucocitos con 75% de neutrfilos, glucosa
30mg/dl, protena 300mg/dl, Qu patgenos estn implicados mas
probablemente?
REPUESTA
a.- Streptococo pneumoniae
b.- Escherichia coli
c.- Haemophilus influenzae
d.- N. meningitidis
PREGUNTA
La coloracin de Gramm revela diplococos gram positivos. Cul es
el agente etiolgico mas probable?
RESPUESTA
a.- H. influenzae
b.- Neumococo
c.- Meningococo
d.- Listeria
CASO CLINICO
Se nota que un recin nacido de 3 semanas de edad tienen
agrandamiento de la cabeza en un examen fsico sistematico. El dato
notorio en sus antecedentes neonatales es un episodio de meningitis
por estreptococo del grupo B que se resolvi luego de un periodo de
antibioticoterapia por via intravenosa.
PREGUNTA
Cul de los mecanismos que siguen es la causa mas probable de los
sntomas de este paciente?
RESPUESTA
a.- Acumulacion de sangre en el espacio subaracnoideo
b.- Decremento de la absorcin de lquido cefalorraqudeo por las
vellosidades aracnoideas
c.- Incremento de resorcin de lquido 408efalorraqudeo
d.- Produccion aumentada del plexo coroideo
CASO CLINICO
Una pareja acude a la sala de urgencias, con su hijo de 5 meses de
edad. Los padres informan que el nio nunca ha sido difcil de
calmar. Sin embargo la noche anterior empez a llorar
incesantemente, y ni cambiarle el paal, ni alimentarlo pudo
calmarlo. Esta maana se haba puesto muy letrgico y caliente y la
temperatura alcanzo 39.4. El medico de la sala de urgencias
examino al lactante y nota cierta rigidez de la nuca. El medico se
preocupa en extremo y solicita una pucion lumbar, la cual revela
presin de abertura y concentracin de protenas altas, glucosa
disminuida y muchos polimorfonucleares.
PREGUNTA
Cul es la causa mas probable de los sntomas de este paciente?
RESPUESTA
a.- Cryptococo neoformans
b.- Citomegalovirus
c.- Escherichia coli
d.- Neisseria meningitidis

Pharmed Solutions Institute

PGINA 408

MANUAL DE TRABAJO DEL CURSO ENARM CMN SIGLO XXI


CEFALEAS
CIENCIAS BASICAS: Es el concepto genrico para considerar el dolor de la cabeza y es el sntoma ms comn del hombre civilizado, puede ser de la
misma intensidad si su origen es benigno o maligno dependiendo de la tolerancia de cada individuo, por tanto la cefalea representa una incapacidad
relativa de la persona para manejar las incertidumbres de la vida, un sntoma que refleja ms una alteracin subyacente del pensamiento o de la
conducta, que una enfermedad del sistema nervioso y aunque es considerada frecuentemente una manifestacin frecuentemente asociada a otros
signos y sntomas que puede originarse frecuentemente asociada a otros aparatos y sistemas. Las cefaleas ms frecuentes son las de origen
orgnico. SALUD PUBLICA: La cefalea ocurre entre 26-45% de los pacientes en alguna etapa de la vida. La migraa tiene mayor frecuencia en
escolares con una relacin de 2:1 en nios comparado con nias. CLASIFICACION: De origen orgnico: relacionada con procesos infecciosos o
inflamatorios, que afectan vas areas superiores, crneo, TCE. De origen neurolgico: migraa y cefalea vascular, por neuroinfecciones,
hipertensin intracraneana de etiologa neoplsica en menores de 6 aos, y para el grupo de escolares y adolescentes, la migraa, cefalea tensional
y epilepsia. Clasificacin International Headache Society 2004: 1. Migraa 2. Cefalea tensional 3. Cefalea en racimos y otras cefaleas trigeminales 4.
Otras cefaleas primarias 5. Cefalea atribuible a TCE 6. Cefalea atribuible a problemas vasculares 7. Cefalea asociada a trastorno IC de origen no
vascular 8. Cefalea atribuible al abuso o depravacin de sustancias 9.Cefalea atribuible a procesos infecciosos 10. Cefalea atribuible a trastornos
metablicos 11. Cefalea o dolor facial asociado a alteraciones del crneo, cuello, ojos, odos, nariz, senos, dientes, boca u otras estructuras faciales o
TIPOS DE CEFALEA
MIGRAA

CEFALEA TENSIONAL
TUMOR CEREBRAL
NEUROINFECCIONES

MANIFESTACIONES
Unilateral o bilateral, de inicio hemicraneal. Pulstil, incremento gradual. El ejercicio lo incrementa.
Duracion de mas de 60 min, menos de 1 semana. Disminuye en la oscuridad y el sueo. Predomina en
sexo femenino
Ambos sexos. Bifrontal u occipital pulsatil. Dolor opresivo o pulstil, duracin de dias o semanas,
intermitente, asociado a fatiga y lo desencadena el estrs
Unilateral o bilateral, sin predominio de sexo. Pulsatil, la despierta el dolor. Frecuencia una vez en la vida,
aparicin sbita
Cefalea universal u occipital. Sin predominio de edad o sexo, puede despertarlo, irritabilidad o fiebre

SINTOMAS ACOMPAANTES
Nauseas, vomito, fotofobia, escotomos

Depresin, ansiedad, estrs


Papiledema, vomito, crisis epilptica, paresias
Ataque al estado general, crisis epilptica, signos de
irritacin menngea.

craneales 12. Cefalea atribuible a patologa psiquitrica 13. Neuralgias craneales 14. Cefaleas no clasificables. De acuerdo a evolucin: agudas,
subagudas y crnicas que por mecanismo directos e indirectos genera modificaciones en la homeostasis de manera que el sntoma remite una vez
resuelta la patologa que la condiciono. Clasificacin Migraa; Migraa sin aura 60-85%. Migraa con aura 15-30%. Migraa Basilar. Migraa
hemipljica familiar. Sndromes peridicos infantiles. PATOGENIA: El dolor de cabeza se origina por estimulacin de las estructuras intracraneales y
extracraneales sensibles a dolor que son: piel, tej., subcutneo, musculo, arterias y periostio del crneo. Estructuras del ojo, odo, dientes y cavidad
nasal. Senos venosos intracraneales. Regiones de la duramadre en la base del crneo, arterias durales interiores de la duramadre. Nervios craneales
trigmino, glosofarngeo, vago. Los 3 primeros nervios cervicales. La estimulacin de una o de varias de estas estructuras sensibles a dolor producen
cefalea a travs de uno o ms de los siguientes mecanismos: irritacin directa por inflamacin, compresin, traccin, desplazamiento, dilatacin y
distensin de las arterias. Teora vascular: es la teora ms antigua, producida por la vasodilatacin de vasos cerebrales, que provocara una
activacin de fibras nociceptivas de las arterias intracraneales. Se consideraba que la vasoconstriccin posterior era la responsable de los sntomas
que ocurren durante el aura. Teora neural: depresin propagada como posible explicacin
FARMACO
INDICACIONES
AAS
Todos los tipos de cefalea
para el aura. Se trata de la existencia de unas ondas de excitacin que se propagaban a los
Paracetamol (: 10-20 mg/kg)
largo de la corteza cerebral a razn de 3mm por segundo. Teora neurovascular: surge ya que
Ketorolaco
ninguna de las dos teoras anteriores logra explicar todos los fenmenos de la crisis de
Ibuprofeno
Profilctico
migraa. En esta teora el desencadenante inicial sera neuronal: hipometabolismoNaproxen
Migrana
Ketoprofeno
Cefalea tensional
hipoperfusin del hemisferio ipsilateral al dolor. Posteriormente, se producira la liberacin
Clonixinato de lisina
Migraa, profilctico
de algunos neuropptidos vasoactivos, como la sustancia P. Estas sustancias liberadas
Aceclofenaco
Todas las cefaleas
pueden dar lugar a la inflamacin neurgena, induciendo vasodilatacin y posterior
Clordiacepoxido
Cefalea postraumtica
extravasacin de protenas plasmticas. Papel de la protena C-fos: sta es una fosfoprotena
Tramadol
neoplasias
que regula la expresin de diversos genes neuronales. La serotonina (5- hidroxitriptamina) se
Propanolol
Migraa
ha considerado como el mediador principal en esta cascada de eventos. Se ha comprobado
Metoprolol
Prfilactico
Imipramina
Migraa
una marcada deplecin de la concentracin de serotonina en plaquetas, lo que sugiere que
Clorimipramina
Profilactico
sta fuera la causa de la migraa. Las teoras vascular y neuronal han contribuido al
Fluoxetina
Cefalea tensional
conocimiento de la enfermedad, pero son incompletas. Proponen que la migraa es el
Cinarizina
Migraa
resultado de interacciones entre el cerebro y el sistema circulatorio sobre un terreno
Flunarizina
Profilctico
genticamente predispuesto. DIAGNOSTICO: Historia clnica dirigida, exploracin fsica
Ac. Valproico
Migraa
Fenitoina
Profilactico
integral. Hay que recordar que un dolor de cabeza es variable en su intensidad, segn la
Topiramato
Asociasion con epilepsia
tolerancia al dolor que tiene cada persona, por eso no es posible conocerlo con precisin y
mucho menos cuantificarlo. La diversidad de sus causas se resume en cuadro anexo. Auras ms frecuentes: Escotoma binocular (77%). Distorsiones
de los objetos (16%). Escotoma monocular (7%). Manchas, balones, colores, arco iris, sndrome de Alicia en el pas de las maravillas.
CEFALEA AGUDA: Se trata de un dolor de cabeza nico de duracin corta en trminos generales, considerada menor de 2 h, y aunque puede ser el
inicio del complejo sintomtico de mltiples causas como infeccin de vas respiratorias, neuroinfecciones, ametropas, gastroinetsinales, neoplasias
del SNC, toxicomanas, HTA. CEFALEA RECURRENTE: Ocurre en forma peridica o recurrente con factores especficos desencadenantes y
caractersticas particulares, pero sin olvidarse de la patologa casi siempre relacionada con la localizacin en la cabeza, dentro de las causas: migraa,
cefalea tensional, en racimos, postraumtica, hipertensin intracraneana crnica, epilepsia, trastorno del sueo. CEFALEA CRONICA: Generalmente
cuando est relacionada con evolucin los problemas serios sistmicos bien originados en el SN, se requiere manejo integral e investigar en forma
prioritaria la causa y establecer un tratamiento especfico, no solo sintomtico como en los casos anteriores. En el caso de que se torne crnica pero
no evolucione puede deberse al uso crnico de frmaco, que estn indicados como desencadenantes de respuesta de rebote, causas: hipertensin
intracraneana, neoplasia del SNC, epilepsia, migraa, cefalea tensional, sinusitis, disfuncin de articulacin temporopmandibular. TRATAMIENTO:
Medidas generales; basada en A) Controlando, deben evitarse los alimentos o circunstancias que desencadena el dolor de cabeza. B) Deteniendo,
automedicacin nunca ser conveniente, debe evitarse la ingestin de frmacos para disminuir un problema asociado que prolonga los dolores de
cabezas como es la cefalea de rebote. C) Previniendo, si se sigue una vida sana, evitando factores desencadenantes de dolor de cabeza, no ingerir
frmacos no indicados. Algunas medidas generales han permitido disminuir la manifestacin como: 1. Aplicacin de frio en forma de compresas de
hielo, pao hmedo, alternado cada 10 min por 30 min. 2. En ocasiones los dolores de cabeza se incrementan con la luz, por lo que es recomendable
usar lentes oscuros, cerrar las cortinas o persianas. 3. En caso de que el dolor de cabeza se acompae de vmito, para evitar deshidratacin,
consumir abundantes lquidos. El tratamiento mdico tiene por propsito en control del sntoma, pero tambin la causa que lo genera, de manera

CURSO ENARM CMN SIGLO XXI TEL: 36246001

Pharmed Solutions Institute

PGINA 409

MANUAL DE TRABAJO DEL CURSO ENARM CMN SIGLO XXI


que pueden considerarse para el primer grupo a) analgsicos no narcticos, b) analgsicos narcticos, c) AINES, d) antidepresivos tricclicos. Para los
casos de migraa se divide el tratamiento farmacolgico en dos fases: Abortiva; para la etapa aguda donde la utilidad de los triptanos sea
comprobado y en menores de 6 aos de edad se recomienda el uso de zolmitriptano y solo en mayores de 6 aos se puede usar sumatriptan,
eletriptan. Profilactico: se recomiendan frmacos para espaciar los cuadros agudos como pueden ser los AINES del tipo ibuprofeno, meloxicam y en
su caso bloqueadores de Cox durante periodos mximos de 6-8 semanas, pero aunado a medidas higienicodieteticas.
CASO CLINICO CEFALEA
Nia de 13 aos de edad presenta cefalea intensa tras un proceso
vrico inespecfico cuatro semanas antes. Lo describa como una
pesadez continua que la incapacitaba para hacer vida normal, cuya
intensidad aumentaba en el transcurso del da y se atenuaba al
colocarse en decbito. Durante el da los sntomas fluctuaban, y
mejoraban tras la administracin de analgsicos. Dorma mal y no
tena nimos para emprender las actividades cotidianas, de manera
que que faltaba al colegio y si intentaba acudir, invariablemente
tena que ser recogida por su familia a media maana. Estaba
tomando analgsicos (ibuprofeno) haca cuatro semanas, tres veces
al da sin cambios en la sintomatologa.
PREGUNTA
Cual es la conducta a seguir mas adecuada?
RESPUESTA
a.- Cambiar de medicamento a narproxen 250 cada 12 hrs.
b.- Agregar ergotamia y cafena profilctica.
c.- Enviar a evaluacin neuropediatrica.
d.- Retirar AINES y medidas generales para disminuir estimulos.
CASO CLINICO
Un varon de 17 aos de edad acude con el medico debido a
cefalalgias que aparecen por la noche de manera sbita. Empiezan
en el ojo izquierdo y despus se generalizan hacia el lado izquierdo
de la cara. El consumo de alcohol puede precipitar los ataques, los
cuales duran menos de una hora. El paciente clasifica el dolor como
de 10/10 y multiples analgsicos de venta sin receta han dado
beneficios minimos.
PREGUNTA
Cul es el diagnotico mas probable para este caso?
RESPUESTA
a.- Cefalagia en racimos
b.- Cefalagia por uso excesivo de medicamentos
c.- Cefalea migraosa
d.- Cefalagia por tensin
PREGUNTA
Cul seria la conducta terapeutica mas adecuada a seguir en este
caso?
RESPUESTA
a.- Amitriptilina
b.- Sumatriptan
c.- Meloxicam
d.- Medidas higienicodieteticas
CASO CLINICO
Una mujer de 16 aos de edad acude con su medico de atencion
primaria quejndose de cefalalgias recurrentes de inico hace 3 aos,
pero que a ultimas fechas han sido mas frecuentes, ahora aocurren
alrededor de 3 veces al mes. Describe el dolor como punzante,
localizado en la sien izquierda y acompaada de nauseas y vomitos
ocasionalesy sensibilidad a luces brillantes y ruidos intensos. Cuando
se le interroga con mayor detalle reconoce algunos episodios de
percibir luces centellantes en el campo visual inferior derecho
alrededor de una hora antesde que aprezca el dolor. EF: revela
temperatura 37, FC 80 lpm, FR12 rpm, TA 120/80mmHg. Un
examen neurolgico no muestra dficit focales.

CURSO ENARM CMN SIGLO XXI TEL: 36246001

PREGUNTA
Cul es el diagnotico mas probable?
RESPUESTA
a.- Cefalea de Horton
b.- Cefalea tensional
c.- Neuralgia del trigemino
d.- Migraa con aura
PREGUNTA
Cul de los siguientes no seria un criterio diagnotico para esta
patologa?
RESPUESTA
a.- Agravamiento por actividad 410sica
b.- Calidad pulsatil
c.- Localizacion unilateral
d.- Sensacion de presin o congoja
PREGUNTA
Cul no es una medida de prevencin para esta paciente?
RESPUESTA
a.- Uso de betabloqueador
b.- Uso de amitriptilina
c.- Uso de triptanos
d.- Uso de verapamil
CASO CLINICO
Una nia de 3 aos y 8 meses de edad acude al Servicio de Urgencias
debido a la presencia de cefaleas periorbitarias derechas intensas,
acompaadas de congestin conjuntival, lagrimeo, rinorrea y
obstruccin nasal ipsilateral. Esta sintomatologa presentaba cuatro
meses de evolucin; se confirm una periodicidad de uno a dos
episodios cada mes, de predominio vespertino y una duracin de
entre 30 minutos y cuatro horas y media; los episodios se alivia ban
con la administracin de paracetamol. En ese momento, la paciente
presentaba un examen objetivo, que inclua un examen neurolgico
sin ninguna alteracin, a excepcin de lo descrito con anterioridad.
Se deriva a la consulta de neuropediatra y se le realiza una
resonancia magntica (RM) cerebral que no muestra ningn tipo de
lesin orbitaria, cerebral o del tronco cerebral.
PREGUNTA
Cul es el diagnotico mas probable para este caso?
RESPUESTA
a.- Glaucoma
b.- Neuralgia del 410rigmino
c.- Cefalea en racimos
d.- Sinusitis esfeniodal aguda
PREGUNTA
Qu medicamentos no son tiles para la profilaxis de esta
paciente?
RESPUESTA
a.- Flunaricina
b.- Loratadina
c.- Ergotamina
d.- Topiramato

Pharmed Solutions Institute

PGINA 410

MANUAL DE TRABAJO DEL CURSO ENARM CMN SIGLO XXI


EPILEPSIAS
CIENCIAS BASICAS: Desorden del cerebro caracterizado por una predisposicin perdurable para genera crisis epilpticas con las consecuencias
neurobiolgicas, cognitivas, psicolgicas y sociales de esta condicin. De acuerdo a la OMS, la epilepsia es la presentacin crnica, recurrente de
fenmenos paroxsticos por descargas elctricas anormales en el cerebro (crisis epilpticas) que tiene manifestaciones clnicas variadas y causas
diversas. Crisis convulsiva; representa un evento de inicio brusco, generalmente autolimitado, caracterizado por una actividad muscular excesiva,
pudiendo ser clnica (prdida de la conciencia con contracciones clnicas rtmicas de las cuatro extremidades. Posictal con confusin), tnica
(contraccin muscular sostenida, duran menos de dos minutos y tienen un periodo posictal de confusin) o mioclonica (contracciones sbitas y muy
breves de grupos musculares, a veces generalizadas, de tal forma que pueden hacer caer al sujeto. El EEG con polipuntas generalizadas). Las crisis
epilpticas; son la manifestacin clnica de una descarga anormal de una poblacin neuronal, generalmente pequea, localizadas ya sea en la corteza
cerebral o bien en la profundidad del parnquima cerebral, de forma repetida y crnica, con correspondencia electroencefalografica. SALUD
PUBLICA: Mxico su prevalencia oscila entre 1.8 a 2 % lo que representa ms de un milln de pacientes. El 76% de los epilpticos inician su
padecimiento antes de la adolescencia; por lo que la Organizacin Mundial de la salud (OMS) reconoce a la epilepsia como un problema de salud
pblica. Las epilepsias afectan aproximadamente 40-50 millones de personas alrededor del mundo. (1-2% de la poblacin mundial). Al menos 5
millones la padecen en Latinoamrica y 3 millones no reciben tratamiento. PATOGENIA: Etiologa diversa. Causas; accidente cerebrovascular, TCE,
neuro-infeccin, alteraciones metablicas, efecto de alcohol o drogas, deprivacin de alcohol, asfixia. Hiptesis para crisis generalizadas; Hiptesis
Neuroanatmica: (Gibbs y colab.) Aqu se invoca una alteracin cortical generalizada sumado a un dismetabolismo en los neurotransmisores
especialmente los inhibidores, como el GABA. Hiptesis micromorfologica: perdida neuronal, esclerosis hipocampal y/o cortical sumado a trastornos
corticales ocasionados por alteracin de la migracin neuronal. Esta migracin neuronal anormal afectara la conduccin trasmembrana de los iones
Ca+ y Na. Anomalas funcionales: constituidas por disminucin del consumo de glucosa y del flujo circulatorio parenquimatoso; posteriormente
comunicaron disminucin del nmero de receptores benzodiazepinicos. Para las crisis focales: Cicatriz cerebral: las distintas lesiones ocasionan una
cicatriz glial que actuara como foco irritativo, que inestabiliza elctricamente las membranas celulares. Este mecanismo excitador seria mediado por
los cidos glutmico y asprtico. Disminucin de los mecanismos inhibidores gabaergicos: debido fundamentalmente a prdida de neuronas
gabaergicas en las distintas lesiones. TIPOS: Las GENERALIZADAS, representan los sntomas derivados de la activacin de grupos de neuronas
extendidas en ambos hemisferios cerebrales; se clasifican como convulsivas: moclnicas, tnicas, clnicas y tnico-clnicas y las no convulsivas:
Ausencias y atnicas. Las PARCIALES (cuando los sntomas iniciales representan la activacin de un grupo de neuronas circunscritas a parte de un
hemisferio cerebral) y pueden tener sntomas: motores o sensoriales localizados a parte de un hemicuerpo, autonmicos o psquicos (ilusiones,
alucinaciones, pensamiento forzado, etc). DIAGNOSTICO: El diagnstico de las crisis epilpticas y de los sndromes epilpticos es de carcter clnico,
examen neurolgico, condiciones mdicas relacionadas: Retraso del desarrollo, cefalea tensional y/o migraa, trastornos psiquitricos, trastornos de
sueo. La confirmacin de los mismos se efecta a travs de los mtodos complementarios de diagnstico. Electroencefalograma. Es la prueba de
eleccin para demostrar el carcter epilptico de un paroxismo y es insustituible para definir muchos sndromes epilpticos. Los estudios de EEG con
privacin parcial de sueo, foto estimulacin y/u otras activaciones son tiles para discriminar actividad epileptiforme no visible en el EEG
convencional. Neuroimagenes. La TAC y RM son las tcnicas de eleccin para detectar lesiones estructurales del SNC, siendo la segunda ms sensible
y especfica, especialmente para el estudio de la esclerosis temporal. CRISIS CONVULSIVAS GENERALIZADAS: Esta forma es la ms frecuente y se la
conoce y denomina habitualmente como Crisis de grand mal, bsicamente se trata de movimientos tnico-clnicos generalizados, de presentacin
abrupta, con alteracin aguda de la conciencia, que duran pocos minutos y que generalmente cursan con tres fases definidas: Fase tnica: Perdida
de conocimiento brusca con cadas e hipertona muscular generalizada. Fase clnica: Movimientos alternativos de flexo-extensin, con sacudidas
rtmicas a nivel ceflico y los cuatros miembros simultneamente Es habitual que se acompae de mordedura de lengua y labios e incontinencia
urinaria. Fase poscrtica: Recuperacin paulatina de la conciencia, con amnesia de lo ocurrido, cefalea, dolores musculares difusos; al cabo de una
hora el paciente se halla habitualmente recuperado. ESTADO EPILEPTICO: El termino se utiliza para describir cualquier tipo de crisis continuas lo
suficientemente prolongadas que pueden producir dao neuronal. La liga Internacional contra la Epilepsia define al estado Epilptico como una
crisis que no muestra datos de recuperacin de lo que durara una crisis habitual, o crisis recurrentes sin recuperacin de alerta durante el periodo
interictal, o recuperacin de la funcin basal normal del sistema nervioso. Desde el punto de vista operativo se acepta una duracin mayor de 5
minutos como suficiente para iniciar tratamiento. CRISIS FEBRIL: Se puede identificar cuando un nio de 6 meses a 6 aos de edad tienen como
nica causa de las crisis febriles un trastorno gentico que le hace susceptible de manifestar crisis convulsivas exclusivamente cuando se tiene
hipertermia, mientras que las crisis desencadenadas por fiebre, el paciente frecuentemente tiene una causa bien establecida de epilepsia, incluso
ha tenido crisis sin fiebre y la hipertermia es solo otro factor precipitante de las crisis. Los criterios para identificar las crisis febriles simples
incluyen: inicio entre los 3 meses y los 5 aos de edad, no tienen antecedente familiar de epilepsia, no tienen patologa neurolgica definida ni datos
de neuroinfeccion, no haber presentado una crisis previa en estado afebril, tienen una duracin menor de 5 min y no presentan dficit neurolgico
postictal. Una crisis febril compleja es aquella que el inicio es parcial, su duracin es mayor de 15 min, con un periodo posictal prolongado, con ms
de 1 crisis en 24h o ms de una por episodio febril, y a la exploracin fsica muestra algn dficit neurolgico, en menores de 6 meses o en mayores
de 5 aos. Tratamiento: Manejo inicial ABC, controlar fiebre con medios fsicos y/o paracetamol 10-15mg/kg/dosis, realizar historia clnica, examen
fsico y exploracin neurolgica. En caso de crisis febriles complejas agregar Valproato de Magnesio a dosis crecientes, hasta llegar a 30mg/kg/da.
CRISIS DE AUSENCIA: a. Tpica: Desconexin del medio por pocos segundos durante los cuales el paciente se ve con la mirada perdida. Puede
presentarse la ausencia simple o acompaarse de fenmenos clnicos leves (parpadeo), automatismos (deglutir, chuparse los labios), fenmenos
atnicos (cada de la cabeza), fenmenos tnicos (contraccin de los msculos del tronco), fenmenos autonmicos (palidez, rubicundez,
piloereccin) No hay periodo posictal y el paciente recupera bruscamente el estado de alerta total. El EEG muestra descargas generalizadas de
complejos punta onda lenta de 3 ciclos por segundo. b. Atpicas: Desconexin del medio un poco ms prolongada que la tpica, frecuentemente
acompaada de fenmenos tnicos y recuperacin lenta hasta la alerta total. El EEG muestra complejos de punta o polipunta onda lenta de 2 2.5
ciclos por segundo. TRATAMIENTO ANTIEPILEPTICO: Crisis primariamente generalizadas convulsivas: Fenitoina (impregnacin de 18mg/kg/dosis
lentamente y vigilando la funcin cardiovascular y mantener con dosis de 5-7mg/kg/da, dividido en 2-3 dosis), Fenobarbital (impregnacin
20mg/kg/dosis y mantener en dosis 3-5mg/kg/da dividido en 1-2 dosis) o Valproato de Magnesio en dosis de 15mg/kg/dosis, seguido de 2550mg/kg/da. Crisis parciales simples, complejas o secundariamente generalizadas: Se prefiere utilizar Valproato de Magnesio o Carbamacepina a
dosis de 15-20mg/kg/da divido en 2-3 tomas al da. No convulsivas. Ausencias Tpicas y atpicas, primera eleccin valproato de magnesio (3060mg/kg/da), Etosuximida. 20-30 mg/kilo/da, fraccionada c / 8 h. Se debe evitar la administracin de benzodiacepinas cuando el evento convulsivo
ha terminado, ante la imposibilidad de evaluar el estado mental y modificar la exploracin neurolgica. En el caso de una crisis febril o afebril
prolongada se podr administrar diacepam IV a dosis de 0.3mg/kg/dosis (mximo 10 mg total) o a dosis de 0.5mg/kg/dosis y evaluar el tratarlo como
estado epilptico. Sndrome de West + Esclerosis tuberosa; vigabatrina. Sndrome de Lennox-Gastaut; topiramato, valproato de magnesio. Epilepsia
de ausencias infantiles y juveniles; valproato de magnesio (30-60mg/kg/da), lamotrigina. Epilepsia mioclnica juvenil; valproato de mg. Crisis

CURSO ENARM CMN SIGLO XXI TEL: 36246001

Pharmed Solutions Institute

PGINA 411

MANUAL DE TRABAJO DEL CURSO ENARM CMN SIGLO XXI


convulsiva fase ictal: Medicamentos primera lnea: benzodiacepinas Lorazepan, diazepam (60-80%). CRITERIOS DE HOSPITALIZACION: < 1 ao,
Glasgow < 15, datos de hipertensin endocraneana, meningismo, crisis dura ms de 15 minutos, recurrencia de las crisis convulsivas 12 horas.
Lamotrigina: primera lnea, atnicas, tnicas, ausencias, tnico clnicas generalizadas, crisis parciales. Segunda lnea en mioclonicas. Topiramato:
primera lnea, crisis tnico clnicas generalizadas, mioclnicas, parciales con generalizacin. Segunda lnea, ausencias, tnicas, atnicas. Vigabatrina:
crisis parciales con eficacia de 50%
CASO CLINICO CRISIS CONVULSIVAS Y EPILEPSIAS
Varn de 15 aos, ingres tras sufrir un episodio de parestesias en el
hemicuerpo izquierdo, junto con desviacin de la comisura bucal
hacia la izquierda, seguido de cefalea frontal intensa y pulstil,
nuseas y vmitos. Antecedentes: producto de un embarazo y parto
normales con un desarrollo psicomotor y del lenguaje adecuados,
con trastorno del aprendizaje. No present antecedentes familiares
de inters excepto que su abuelo (materno) sufra de migraas. A los
4 aos, tras un traumatismo craneoenceflico leve, sufri un cuadro
de inestabilidad y somnolencia de varias horas de duracin. A los 5
aos present en vigilia, de forma sbita, una hemiparesia aguda
izquierda asociada a disartria y confusin, que cedi en 10 minutos.
El examen fsico y neurolgico completo evidenci, una leve
dismetra del miembro superior izquierdo.
PREGUNTA
Cul de los siguientes diagnosticos es el ms probable?
RESPUESTA
a.- Crisis parciales simples.
b.- Crisis parciales complejas.
c.- Crisis generalizadas convulsivas.
d.- Crisis generalizadas no convulsivas.
PREGUNTA
Cual es el auxiliar diagnostico mas adecuado para idenficar la
causa?
RESULTADO
a.- MVEEG.
b.- EEG.
c.- IRM.
d.- TAC.
PREGUNTA
Cual es la conducta teraputica mas adecuada para el caso?
RESPUESTA
a.- Carbamazepina 10 a 20 mg/kg.
b.- Lamotrigina 200 a 400 mg/da administrados en dos dosis.
c.- Valproato de magnesio 60 mg/kg/da.
d.- Topiramato 100 mg/dia.
CASO CLINICO
Una nia de 2 aos previamente sana ingresa ala sala de urgencias
despus de una crisis convulsiva.
Vea televisin cuando
experimento rigidez de todo el cuerpo, seguida por convulsiones y
contracciones musculares espasmdicas de las cuatro extremidades.
De acuerdo con los padres la crisis convulsiva duro alrededor de
cinco minutos y estuvo seguida por un periodo posictal corto de
somnolencia. Habia tenido secrecin nsalas abundante todo un dia,
que la madre atribuyo a contacto con nios enfermos en la guardera.
En el examen la temperatura esde 39.9, FC 140 lpm, FR 22 rpm, TA
95/75mmHg, oximetra de pulso de 99%. La nia tiene buen aspecto
y no se encuentra en angustia aguda. El hallazgo importante a nivel
pulmonar son ruidos respiratorios disminuidos en la base del
pulmn derecho, resto sin importancia.

CURSO ENARM CMN SIGLO XXI TEL: 36246001

PREGUNTA
Cul es la conducta diagnostica menos adecuada a seguir en esta
paciente?
RESPUESTA
a.- Electrolitos y glucosa
b.- Biometria hemtica completa
c.- Puncion lumbar
d.- Resonancia magntica
PREGUNTA
Qu riesgo tienen esta paciente de sufrir epilepsia en la edad
adulta?
RESPUESTA
a.- 2.5%
b.- 5%
c.- 10%
d.- 15.5%
PREGUNTA
Cinco horas mas tarde la misma paciente reingresa a urgencias luego
de experimentar una crisis convulsiva tnicoclnica generalizada
durante 25 minutos. La temperatura es de 40.1. Cul es la
conducnta teraputica menos adecuada para el estudio de esta
paciente?
RESPUESTA
a.- Electroencefalograma
b.- Resonancia magntica
c.- Tomografia de crneo
d.- SPECT
CASO CLINICO
Un nio de 5 aos de edad es llevado a la sala de urgencias por su
madre despus de encotrarlo con convulsiones en el piso de la
cocina. Camino al hospital hubo otra crisis convulsiva. En el
momento de la admisin el paciente parace dbil y desorientado, y
no recuerda el suceso. Se administra una dosis de fenitoina para
profilaxis de crisis convulsiva. Una hora despus de que se
administra el frmaco, la concentracin plasmtica es de
50microgramos/ml; dos horas despus es de 30. Suponiendo que la
fenitoina tienen eliminacin de orden 0.
PREGUNTA
Cul es la concentracin mas probable a las 3 hrs de la
administracin?
RESPUESTA
a.- 5g/ml
b.- 10g/ml
c.- 15g/ml
d.- 20g/ml

Pharmed Solutions Institute

PGINA 412

MANUAL DE TRABAJO DEL CURSO ENARM CMN SIGLO XXI


ENURESIS Y ENCOPRESIS
CIENCIAS BASICAS: La enuresis pertenece al grupo de trastornos de la eliminacin siendo definido por el DSM-IVTR como el repetido vaciado de
orina en la ropa o en la cama, tanto si el vaciamiento es involuntario como intencionado. Esta conducta debe ocurrir dos veces por semana durante
3 meses por lo menos o que genere un malestar clnico significativo o altere el funcionamiento social o acadmico del nio. La enuresis es un
problema evolutivo de la miccin. La definicin ms aceptada por la comunidad cientfica establece que la enuresis consiste en la emisin
involuntaria y persistente de orina durante el da o la noche, despus de una edad en la que el nio ya debera haber aprendido a controlar la
miccin (5 aos) y no existen indicios de patologa. Se manifiesta si el nio no ha aprendido a evacuar voluntariamente la orina en los lugares
adecuados, es decir, si la miccin no se realiza bajo control del individuo. SALUD PUBLICA: El 15% aproximadamente de la poblacin infantil mayor
de 5--6 aos tiene enuresis primaria. La incidencia familiar es alta. Un 44% tienen un padre que fue enurtico en la niez y si ambos padres lo fueron
el nio tiene hasta un 77% de probabilidades de tener enuresis. TIPOS: Enuresis primaria: el nio no ha logrado control suficiente y puede
encontrarse en cualquier estadio del aprendizaje del control voluntario de la miccin. Enuresis secundaria: se trata de casos en los que despus de
un amplio periodo de control (ms de 6 meses) el nio recae, en ocasiones, asociado a un accidente, a periodos de hospitalizacin largos, a
nacimiento de hermanos o a prdidas familiares. La enuresis funcional (no organica) es el tipo ms comn (ms del 90%) y es aquella en la que los
nios no presentan ningn otro problema bien sea de origen congnito orgnico, infeccioso o traumtico. El nio no ha alcanzado todava control
sobre la miccin pero este problema no puede ser achacable a ninguna otra causa. Su desarrollo general (motor y verbal) es normal y
aparentemente slo muestra dificultades en el aprendizaje del control de la miccin. En los otros casos se habla de enuresis orgnica bien sea de
tipo congnito o traumtica. La distincin entre enuresis diurna y nocturna es obvia, sin embargo, su tratamiento plantea problemas diferentes.
PATOGENIA: Decimos que es un problema evolutivo porque todos nacemos con el reflejo de miccin, de forma que cuando la vejiga est llena se
relajan los esfnteres y se libera la orina, pero en el curso del desarrollo aprendemos a controlar el reflejo y a orinar en el lugar apropiado. El
momento a partir del cual controlamos la miccin es muy variable y por tanto el diagnstico de enuresis se establece arbitrariamente a partir de los
cinco aos. Teoras: Una de ellas es, que el sueo especialmente profundo de los nios que mojan la cama impide que las contracciones de los
msculos de la vejiga despierten al nio antes de iniciar la miccin. Sin embargo, los estudios electroencefalogrficos, los escneres y tomografas
cerebrales no han podido constatar diferencias que pudieran justificar la enuresis de algunos casos aunque algunos resultados de investigacin
apuntan a que los enurticos presentan un umbral para despertar ms elevado que los no enurticos. Tambin asociado al sueo se ha sealado la
posibilidad de que las contracciones vesicales fueran demasiado dbiles para provocar el despertar, al mismo tiempo que se ha propuesto que
durante el sueo profundo el nivel de contraccin de los esfnteres era deficiente. Se ha sugerido tambin que los casos de enuresis presentan una
capacidad funcional de vejiga disminuida y, por tanto, no pueden aguantar toda la noche sin evacuar la orina. La teora comnmente mantenida
respecto a la enuresis desde una perspectiva psicolgica es que sta es debida a un dficit de aprendizaje. El control de la miccin diurno y,
especialmente, nocturno es una habilidad compleja que comporta muchos pasos y requiere un correcto entrenamiento, lo que no quiere decir que
haya que ocuparse activamente de que el nio aprenda sino que hay que permitir que se den las condiciones para el aprendizaje. DIAGNOSTICO: Por
denticin, el nio enurtico debe tener examen fsico normal de vas excretorias, tanto en sus aspectos anatmicos como funcionales. Para hacer el
diagnstico en nios con enuresis es necesario tener en cuenta historia clnica detallada, examen fsico y estudios paraclnicos. Es indispensable que
se investigue como fue el entrenamiento del esfnter vesical y anal. Es necesario evaluar la situacin familiar, identificando la funcionalidad, eventos
relevantes, situacin escolar, dinmica con hermanos y compaeros de escuela. Estudios bsicos: urocultivo, EGO, qumica sangunea, ultrasonido
abdominal (vas urinarias). TRATAMIENTO: El primer paso es la educacin y apoyo a los padres, puntualizando en la paciencia. Siempre ha que
descartar causas orgnicas, de presentarse pasan a primer trmino. Las alternativas teraputicas son varias de las que se pueden emplear las
siguientes: modificacin conductual, entrenamiento de la capacidad vesical, distribucin de la ingesta de lquidos (aumentar consumo de lquidos
durante el da; 40% en la maana, 40% por la tarde y 20% al anochecer)y tratamiento farmacolgico. En lo que respecta al tratamiento motivacional,
es importante incluir la participacin del nio con el propsito de hacerlo responsable y a su vez darles un reconocimiento de las noches secas (llevar
calendario de das secos y das que mojo la cama). Farmacolgico: imipramina, inhibe la recaptura sinptica de noradrenalina y serotonina.
Posee actividad anticolinrgica dbil y accin antiespasmdica directa sobre musculo liso vesical, alterando la inervacin sinptica. Tambin se ha
demostrado que disminuye el tiempo de sueo MOR. Dosis para nios de 5-8 aos es de 25mg, para nios mayores de 50mg o a dosis de 0.91.5mg/kg/da. Se administra en una sola dosis por la noche cuando la dosis es mayor de 50mgs, debe fraccionarse en dos tomas. Durante
tratamiento evaluar frecuencia cardiaca y presin arterial, as como realizar ECG, para detectar eventuales anomalas de la conduccin. Otro
medicamento que se puede utilizar es la desmopresina. ENCOPRESIS. CIENCIAS BASICAS: Es difcil establecer un acuerdo unnime sobre este
concepto. Es un trastorno de la eliminacin. Consiste en la emisin fecal, voluntaria o no (incapacidad de controlar el esfnter anal), de forma
regular, sobre lugares socialmente inapropiados, generalmente la ropa. SALUD PUBLICA: El control esfinteriano se adquiere progresivamente con la
edad. En culturas occidentales son encopreticos alrededor de 5%. A los 16 aos no hay prcticamente ninguno, ya que la encopresis declina con la
edad en un porcentaje muy elevado (28%). El riesgo relativo de los nios respecto de las nias es de 3-6 veces superior a partir de los 4 aos de
edad. La encopresis secundaria iguala o supera ligeramente a la primaria. Los episodios de encopresis son ms frecuentes durante el da que por la
noche. La encopresis retentiva es la ms frecuente. CLASIFICACION: Segn su etiologa: FUNCIONAL O NO ORGANICA (no hay causas orgnicas
conocidas) y ORGNICA (puede ser provocada por trastornos gastrointestinales que cursan con diarrea, intolerancia a la lactosa, insuficiencia
pancretica, CUCI, lesiones posquirrgicas, disrafismo espinal oculto, en estos casos la incontinencia parece estar provocado por la fatiga de los
msculos del suelo plvico y del esfnter externo, despus de que el nio los haya contrado de forma continuada para preservar la continencia).
Segn la aparicin del problema: PRIMARIA O CONTINUA (ausencia de control desde que naci, mejor pronstico, son nios desatendidos,
regresivos, desinhibidos emocionalmente sin sentimiento de vergenza) y SECUNDARIA O DISCONTINUA (presencia de control por un periodo de
continencia de al menos un ao, coincide con sucesos estresantes, son nios sobreprotegidos, inhibidos emocionalmente, con rasgos obsesivos,
sentimientos de vergenza). Segn su fisiopatologa: RETENTIVA (con estreimiento y rebosamiento, se caracteriza por un ciclo de varios das de
retencin, una expulsin dolorosa, otro periodo de retencin que ocasiona distensin y desensibilizacin de la pared rectal con ello se pierde la
sensacin de la necesidad de defecar, aparece un megacolon psicgeno, las heces se acumulan y finalmente se defeca por rebosamiento) y NO
RETENTIVA (sin estreimiento, el problema puede ser consecuencia de entrenamiento inadecuado, reaccin fisiolgica ante el estrs ambiental o
una forma de evidenciar conductas de oposicin ante las normas establecidas). PATOGENIA: La defecacin es el eslabn final de todo el proceso
digestivo. Empieza con la ingestin de alimentos y contina por el tracto digestivo, y acaba en el intestino grueso que se divide en: ciego, colon y
recto. Estos ltimos son los responsables de la defecacin y la continencia. Al menos dos factores contribuyen a la continencia, la distribucin
anatmica de la parte terminal del intestino grueso (ngulo anorectal) y la contraccin tnica del musculo puborectal y del esfnter externo. Dichas
condiciones anatmicas intervienen para evitar el flujo fecal cuando la presin intraabdominal se eleva sbitamente, como sucede con los
estornudos, la risa o cambios posturales bruscos. Cuando cierta cantidad de heces entra en el recto y se llena, se origina un reflejo que inhibe la
contraccin tnica del esfnter interno, que se relaja, la presin del canal anal disminuye y las heces pueden descender. Ante la distensin rectal, el

CURSO ENARM CMN SIGLO XXI TEL: 36246001

Pharmed Solutions Institute

PGINA 413

MANUAL DE TRABAJO DEL CURSO ENARM CMN SIGLO XXI


sujeto experimenta sensacin de plenitud que se acompaa del deseo de defecar. Hay que recordar que este reflejo no se produce en los sujetos
con enfermedad de Hirschsprug, debido a la ausencia congnita de clulas nerviosas en algunos segmentos del colon, lo que provoca estreimiento
y obstruccin intestinal. La contraccin (relajacin) del esfnter externo ante la distensin del recto es una respuesta voluntaria que se aprende
durante el segundo ao de vida, ligado al desarrollo neuromuscular y al entrenamiento del nio en hbitos de eliminacin. Por lo tanto, la mayora
de los individuos con incontinencia fecal muestran anomalas en algunos de estos mecanismos: 1. Capacidad para percibir la distensin rectal
(sensibilidad anorectal). 2. Capacidad del recto para almacenar heces (acomodacin y adaptacin al bolo fecal): funcin de depsito. 3. Habilidad de
contraccin del esfnter externo. 4. Motivacin para ejecutar las respuestas adecuadas. En los bebes la defecacin es un proceso automtico
(relajacin de los esfnteres de forma refleja cuando el recto est lleno). DIAGNOSTICO: Puesto que la encopresis es un trastorno psicofisiolgico y
multicausado, conllevara la realizacin de una exploracin mdica y psicolgica. El principal objetivo es descartar la existencia de cuadros orgnicos
que puedan ser responsables de la incontinencia. En general el protocolo es: Historia clnica. Exploracin fsica (abdominal, perianal, tacto anorectal).
Anlisis de sangre especficos. Urocultivo. Estudios radiolgicos. Manometra anorrectal. Biopsia rectal. El objetivo de la evaluacin conductual
reside en: 1. Averiguar si se han adquirido buenos hbitos de higiene y defecacin. 2. Determinar factores ambientales, sociales y personales que
pueden estar interfiriendo. 3. Establecer relaciones funcionales entre estos. TRATAMIENTO: Consta de 2 fases: Fase inicial de limpieza fecal o
desimpactacin (enemas). Fase de mantenimiento (laxantes). Modificaciones dietticas (fibra). Ingestin de lquidos. El protocolo de Levine, inclua
una combinacin de tratamiento mdico y conductual. Respuesta no significativa a frmacos (imipramina). Tratamiento conductual: Identificar las
conductas objeto de modificacin fecal. Ensear conductas requisito para la defecacin. Instaurar hbitos rutinarios. Tcnicas empleadas:
reforzamiento positivo, reforzamiento negativo, castigo, psicoterapia, entrenamiento en hbitos de defecacin.
CASO CLINICO
Se trata de una paciente de 12 aos de edad, quien cursa sptimo
grado y vive con su madre y sus dos hermanos (un nio de 10 aos y
una nia de 5 aos), La madre refiere que desde hace 3 aos
presenta enuresis y enconpresis en cualquier momento y en
cualquier situacin, refiere la madre adems aislamiento social,
tendencia al mutismo, llanto fcil y somnolencia diurnia, refiere
adems hiporexia, irritabiliadd, ideas de muerte, minusvala y
desesperanza

complicaciones perinatales; el desempeo escolar ha sido


satisfactorio; ha tenido 2 episodios de IVUB; el padre refiere que el
presento el mismo problema cuando era nio. Exploracin fisica:
Peso: Talla: T: FC: 82 lpm FR: 25 rpm TA: 100/60 mmHg. Se palpan
masas probablemente fecales en fosa iliaca izquierda y rea
suprapubica, hay un agujero sacro sin hipertricosis localizada,
reflejos tendinosos profundos son normales, extremidades
simetricas, tono y fuerza uscular normales. Resto de la exploracin
es normal.

PREGUNTA
Cual es la conducta mas adecuada a seguir?
RESPUESTA
a.- Iniciar con imipramina.
b.- Enviar al psiclogo.
c.- Enviar al psiquiatra.
d.- Ingreso psiquitrico obligado.

PREGUNTA
Cul es la condunducta teraputica mas adecuada a seguir en este
caso?
RESPUESTA
a.- Psicoterapia
b.- Imipramina
c.- Antidepresivos
d.- Oxibutina

CASO CLINICO
Nio varn de cuatro aos que presenta problemas relacionados con
el control de esfnteres. La madre consulta porque el nio muestra
retencin voluntaria de heces y orina y rechazo total a sentarse en el
retrete. Refiere que el nio solicita y exige a los padres que le
pongan el paal, y solo entonces lleva a cabo las deposiciones.
PREGUNTA
Cual es la conducta mas apropiada a seguir?
RESPUESTA
a.- Psicoterapia conductual.
b.- Psicoterapia gestltica.
c.- Psicoterapia cognitiva.
d.- Psicoterapia psicoanaltica
CASO CLINICO
Femenino de 7 aos de edad, originaria de Chiapas y actualmente
radicando en Montemorelos, desde hace 6 meses; concurre a
consulta por orinarse en la cama. La madre refiere que: moja la
cama casi todas las noches, sin embargo tuvo un periodo de tres
meses de noches secas con orinas ocasionales entre los 5 y 6 aos de
edad. Durante el da tiende a tener urgencia y frecuencia de orinar,
con salida de orina ocasional en el camino al bao. Tiende a tener
aumento en la ingesta de lquidos en ocasiones. Su Madre tambien
ha notado manchas redondas de orina en su ropa interior.
Antecedentes: hija unica, naci por parto vaginal a trmino, sin
.

CURSO ENARM CMN SIGLO XXI TEL: 36246001

CASO CLINICO
Nio de 7 aos que ingresa por infeccin urinaria e insuficiencia
renal aguda con urea 191mg/dl, creatinina 6,3 mg/dl, potasio 6.3
mmol/L y acidosis metablica compensada. Antecedentes: seguido
por enuresis nocturna, con control diurno temporal y encopresis con
mejora tras tratamiento de desimpactacion y mantenimiento.
Antecedentes familiares: padre enuresis nocturna hasta los 8 aos.
Exploracin: Destaca palidez y edema facial. Abdomen: globo vesical
palpable. Pruebas complementarias: Hemograma: Series blanca y
plaquetaria normal. Hb: 9.1g/dl. PCR: 6,1 mg/dl. PCT: >10 mg /dl.
Ecografa Abdominal: Dilatacin pielocalicial bilateral (15 mm) y del
sistema excretor con vejiga de paredes trabeculadas y engrosadas
(de lucha).
PREGUNTA
Cul es la conducta teraputica mas adecuada a seguir en este
momento?
RESPUESTA
a.- Tratamiento sintomtico
b.- Sondaje vesical, antibitico intravenoso, fluidoterapia
c.- Antibiotico intravenoso
d.- Antibiotico, esteroides, fluidoterapia

Pharmed Solutions Institute

PGINA 414

MANUAL DE TRABAJO DEL CURSO ENARM CMN SIGLO XXI


TRASTORNO POR DFICIT DE ATENCIN E HIPERACTIVIDAD (TDAH)
CIENCIAS BASICAS: Es el trastorno psiquitrico ms frecuente en la infancia; es un proceso crnico con una alta comorbilidad que va a influir en el
funcionamiento del individuo en la edad adulta. El TDAH segn el DSM-IV (o trastorno hipercintico segn el CIE-10) se define como un determinado
grado de dficit de atencin y/o hiperactividad-impulsividad que resulta desadaptativo e incoherente en relacin con el nivel de desarrollo del nio y
est presente antes de los 7 aos de edad. Las manifestaciones clnicas deben persistir durante ms de 6 meses. El cuadro debe ser ms severo que
Criterios diagnsticos del TDAH y diferencias entre DSM-IV y CIE-10 (negrita CIE-10)
lo observado en otros nios de la misma edad,
Dficit de atencin:
el mismo nivel de desarrollo e inteligencia.
1.
A menudo no presta atencin suficiente a los detalles o incurre en errores por descuido en las tareas escolares,
Debe estar presente en varios ambientes
en el trabajo o en otras actividades.
como familia, escuela, amigos. Debe producir
2.
A menudo tiene dificultades para mantener (no mantiene) la atencin en tareas o en actividades ldicas
3.
A menudo parece no escuchar cuando se le habla directamente (lo que se le dice)
serios problemas en la vida diaria. SALUD
4.
A menudo no sigue instrucciones y no finaliza tareas escolares, encargos u obligaciones en el lugar de trabajo
PUBLICA: TDAH es un importante problema
5.
A menudo tiene dificultad (presenta alteracin) para organizar tareas y actividades
de salud pblica debido a varias razones: 1. Su
6.
A menudo evita (o muestra una fuerte aversin), le disgustan tareas que requieren un esfuerzo mental
sostenido
alta prevalencia (Utilizando los criterios del
7.
A menudo extrava objetos necesarios para tareas o actividades
DSM-IV la prevalencia se sita entre el 3-7%.
8.
A menudo se distrae fcilmente por estmulos irrelevantes
Con la CIE-10 la prevalencia es del 1,5% y los
9.
A menudo es descuidado en las actividades diarias
cuadros ms severos) 2. El inicio en etapas
Hiperactividad
1.
A menudo mueve en exceso manos o pies, o se remueve en su asiento
precoces de la infancia. 3. A ser un proceso
2.
A menudo abandona su asiento en la clase o en otras situaciones en que se espera permanezca sentado
incapacitante y crnico. 4. A la afectacin de
3.
A menudo corre o salta excesivamente en situaciones en que es inapropiado hacerlo
las diferentes esferas comportamentales
4.
A menudo tiene dificultades para jugar o dedicarse tranquilamente a actividades de ocio
5.
A menudo est en marcha o suele actuar como si tuviera un motor (Exhibe un patrn persistente de
(familiar, escolar y social), y 5. A su alta
actividad motora excesiva que no se modifica sustancialmente por el contexto o exigencias sociales)
comorbilidad, por lo que siempre deben
Impulsividad
investigarse otras patologas asociadas que
1.
A menudo precipita respuestas antes de haber sido completadas las preguntas
conlleven un gran riesgo para el futuro del
2.
A menudo tiene dificultades para guardar su turno (A menudo no espera en la cola o no guarda su turno en
juegos o situaciones de grupo)
nio, como son trastornos del aprendizaje,
3.
A menudo interrumpe o se inmiscuye en las actividades de otros (p.ej., se entromete en conversaciones o
problemas de conducta o trastornos
juegos)
emocionales, entre otros. La relacin
4.
A menudo habla en exceso (sin una respuesta apropiada a las limitaciones sociales)
varn/mujer vara segn los estudios desde
6/1 a 3/1. CLASIFICACION: El DSM-IV, describe 3 subtipos: 1. Subtipo con predominio inatento (inatencin). Entre 20-30% de los casos. 2. Subtipo
con predominio hiperactivo-impulsivo del 10-15%. 3. Subtipo combinado (cuando estn presentes los 3 tipos de sntomas). Del 50-75% de los casos.
Segn la CIE-10; es necesaria la existencia simultnea de los 3 tipos de sntomas: dficit de atencin, hiperactividad e impulsividad, constituyendo el
cuadro de alteracin de la actividad y la atencin. Reconoce adems una categora separada, el trastorno hiperquintico de la conducta.
PATOGENIA: Se trata de un trastorno multifactorial con una base neurobiolgica y predisposicin gentica que interacta con factores ambientales.
Las hiptesis sobre la etiologa abarcan diversas reas: gentica conductual (mayor prevalencia en pacientes con familiares con TDAH y otros
trastornos psiquitricos. En gemelos) y molecular (factores genticos, en relacin con mutaciones de varios genes que codifican para los
transportadores y receptores de la dopamina y el gen transportador de la noradrenalina, fallo en el desarrollo de los circuitos cerebrales, en que se
apoya la inhibicin y el autocontrol), factores biolgicos adquiridos (en periodo prenatal, perinatal y postnatal como exposicin intrauterina al
alcohol, nicotina y determinados frmacos [benzodiazepnias, anticonvulsivantes], prematuridad, bajo peso al nacer, encefalitis, traumatismos,
hipoxia, hipogluecemia), la neuroanatoma (implicacin del crtex prefrontal y ganglios basales [reas encargadas de regular la atencin], se
sugieren alteraciones en las redes corticales cerebrales frontales y frontoestriadas. Sistema atencional anterior: lbulo frontal. Sistema atencional
posterior: lbulo parietal y cerebelo), neuroqumicos (la NA y DA, son los neurotransmisores de mayor relevancia en la fisiopatologa y tratamiento
de TDAH, ambos implicados en la funcin atencional y la DA tambin en la regulacin motora), bioqumica cerebral, neurofisiologa (alteraciones de
la actividad cerebral como: reduccin del metabolismo/flujo sanguneo en el lbulo frontal, crtex, parietal, stratium, cerebelo), neuropsicologa y el
entorno psicosocial (severidad y expresin de los sntomas puede verse afectado a travs de la interaccin gen-ambiente. Factores como:
inestabilidad familiar, problemas con amigos, trastornos psiquitricos en los padres, paternidad y crianza inadecuada, relaciones negativas padreshijos, nios que viven en instituciones con ruptura de vnculos, adopciones y bajo nivel socioeconmico). DIAGNOSTICO: Los tres sntomas
esenciales del TDAH son: 1. El dficit de atencin (falta de perseverancia en la atencin o atencin dispersa), 2. La hiperactividad (excesivo
movimiento) y 3. La impulsividad (dificultad en el control de impulsos). El dficit de atencin guarda una mayor relacin con las dificultades
acadmicas y el logro de metas, mientras que la hiperactividad e impulsividad estn ms relacionadas con las relaciones sociales y los resultados
psiquitricos. Las manifestaciones clnicas varan en grado e intensidad segn la edad del paciente en el momento del diagnstico. El DSM-IV
requiere que estn presentes al menos 6 de los 9 sntomas tanto para el subtipo con predominio del dficit de atencin, como para el subtipo
hiperactivo-impulsivo. El TDAH de tipo combinado requiere una combinacin de ambos tipos de sntomas de dficit de atencin y de hiperactividad
impulsividad (6+6). Para la CIE-10 se requiere 6/9 sntomas de dficit de atencin adems de al menos 3/5 sntomas de hiperactividad y 1/4 sntomas
de impulsividad. La impulsividad constituye un sntoma importante en el DSM-IV pero no tanto en el CIE-10. Existen una serie de criterios
diagnsticos adicionales que se requieren para poder hablar de trastorno y no solo de sntomas transitorios: Edad de inicio. Algunos sntomas deben
haber estado presentes antes de los 6-7 aos. Duracin. Los criterios sintomticos deben haber persistido al menos durante los ltimos 6 mesesUbicuidad. Algn grado de disfuncin debida a los sntomas debe haber estado presente en 2 situaciones o ms (escuela, trabajo, casa). Disfuncin.
Los sntomas deben ser causa de una disfuncin significativa (social, acadmica o laboral). Discrepancia. Los sntomas son excesivos en comparacin
con otros nios de la misma edad, desarrollo y nivel de inteligencia. Exclusin. Los sntomas no se explican mejor por la presencia de otro trastorno
mental como ansiedad, depresin o esquizofrenia, entre otros. Adems, es imprescindible obtener registros de sntomas mediante cuestionarios
para evaluar la gravedad o intensidad de los sntomas y su presencia en varios ambientes. Cuestionarios como el Cuestionario de Evaluacin del
TDAH son muy tiles para obtener de forma rpida los sntomas que estn presentes, tanto desde el punto de vista de los padres como de los
profesores. TRATAMIENTO: Un plan de tratamiento individualizado y multidisciplinar para un nio con TDAH y su familia casi siempre debe incluir 3
aspectos fundamentales: 1. Entrenamiento a los padres: Psicoeducin sobre el TDAH. Entrenamiento en tcnicas de manejo conductual del nio. 2.
Intervencin a niveles acadmico y escolar: En el colegio. En casa. 3. Medicacin especfica para el TDAH. El uso de medicacin debe ser una parte
del plan de tratamiento inicial en la mayora de los nios en edad escolar y adolescentes con TDAH. El reciente estudio de tratamiento multimodal
del TDAH, del Instituto Nacional de Salud Mental, ha demostrado que un tratamiento farmacolgico cuidadoso y estandarizado se asocia con una
reduccin de sntomas significativamente mayor, en la mayora de los nios, que un tratamiento de intervencin psicosocial con diferentes tipos de
psicoterapia. Pero el grupo de terapia combinada con psicoterapia y medicacin obtuvo mejores resultados que el grupo de terapia con medicacin

CURSO ENARM CMN SIGLO XXI TEL: 36246001

Pharmed Solutions Institute

PGINA 415

MANUAL DE TRABAJO DEL CURSO ENARM CMN SIGLO XXI


sola en el porcentaje de nios en remisin. Tratamiento farmacolgico, los ms utilizados y de primera eleccin son los psicoestimulantes, una
segunda alternativa los antidepresivos tricclicos si existe comorbilidad con problemas afectivos y emocionales y podra utilizarse los inhibidores
selectivos de la recaptura de serotonina, en algunos casos en donde la impulsividad es muy evidente. Uso de medicaciones estimulantes en el
tratamiento del TDAH: El metilfenidato (dosis de 0.1mg/mg, para fines practicos se inicia con dosis bajas de 2.5mg, es el nico estimulante indicado
para el tratamiento TDAH, mejoran la atencin, el comportamiento en clase y el desarrollo acadmico a corto plazo, mejora la interaccin con
compaeros y disminuye el comportamiento impulsivo. Se presenta en comprimidos de 5,10 y 20 mg. Existen opciones farmacolgicas diversas
tiles en el manejo del TDAH solo o con comorbilidad, como los medicamentos no estimulantes (atomoxetina, antidepresivos tricclicos, agonistas
alfa adrenrgicos y modafinilo).
CASO CLINICO
Nia de 10 aos, remetida a psiclogo a peticin del colegio por
fracaso escolar. Se trata de una nia con buena capacidad intelectual
que aunque no le gusta el estudio, se aplica trabajando en los
deberes escolares, nunca ha mostrado comportamiento hiperactivo
ni han existido dificultades a la hora de obedecer las normas ni en
casa ni en el colegio; la nica queja de sus padres es lo distrada que
es (no se concentra, le mandas a hacer un recado y se entretiene en
cien mil cosas sin llegar a hacerlo, desorganizada, pierde las
cosas,etc) la considera una nia feliz, aunque comenta que sus
dificultades escolares hace que a veces se entristezca.
PREGUNTA
Cual es el diagnostico mas probable?
RESPUESTA
a.- TDAH.
b.- TDA.
c.- TDAH-R
d.- Trastorno del control de impulsos.
CASO CLINICO
Un nio de 8 aos de edad es remitido con el pediatra por
recomendacin de su maestra, debido a las dificultades que enfrenta
en la escuela. Segn la maestra ha tenido problemas para terminar
la tarea y suele cometer errores por descuido. Se distrae con
facilidad y se le debe decir constantemente que se siente y
permanezca en su lugar, aunque a menudo se le encuentra mirando
fijamente por al ventana. Tambien dice que es muy perturbador y
responde bruscamente a preguntas antes de que ella termine de
hacerlas. Sus padres lo describen como un nio feliz que disfruta
jugar con su hermano y sus amigos del vecindario. El se queja de que
no le gusta la escuela, salvo por el recreo y la clase de educacin
fsica. La evaluacin por el consejero escolar no mostro signos de
trastorno del aprendizaje.

CURSO ENARM CMN SIGLO XXI TEL: 36246001

PREGUNTA
Cul es el diagnostico mas probable para este caso?
RESPUESTA
a.- Trastorno de la personalidad antisocial
b.- Trastorno de dficit de atencin e hiperactividad
c.- Trastorno bipolar de laniez
d.- Trastorno desafiante opocicional
PREGUNTA
Cul es la prevalencia para esta patologa?
RESPUESTA
a.- 3-10% mas en nias
b.- 3-10% mas en nios
c.- 8-15% mas en nias
d.- 8-15% mas en nios
PREGUNTA
Qu medicamentos de segunda lnea son tiles para el manejo de
este paciente?
RESPUESTA
a.- Premolina, imipramina
b.- Amitriptilina, fluoxetina
c.- Lamotrigina, paroxetina
d.- Haloperidol, imipramina
PREGUNTA
Si le tomaramos una resonancia magntica a este paciente. Qu
anomalas es mas probable observar?
RESPUESTA
a.- Corteza temporal,
b.- Corteza biparietal
c.- Corteza prefrontal, ganglios basales y cerebelo menores o
asimtricos
d.- Corteza occipital

Pharmed Solutions Institute

PGINA 416

MANUAL DE TRABAJO DEL CURSO ENARM CMN SIGLO XXI


CIRUGIA
1) ESOFAGITIS, ACALASIA, REFLUJO ESOFAGICO Y ERGE.
2) ESOFAGITIS, DISPLASIAS DE ESOFAGO Y ESOFAGO DE BARRETT,
3) CANCER DE ESOFAGO.
4) VARICES ESOFAGICAS, RUPTURA ESOFAGICA
5) GASTRITIS, ULCERA GASTRICA Y ULCERA DUODENAL.
6) CANCER GASTRICO.
7) COLANGITIS, COLECISTITIS, COLEDOCOLITIASIS
8) CARCINOMA HEPATOCELULAR
9) CANCER DE PANCREAS, COLANGIOCARCINOMA,
10) PANCREATITIS AGUDA Y CRONICA
11) ISQUEMIA MESENTERICA, INFARTO MESENTERICO, DIVERTICULITIS.
12) APENDICITIS, POLIPOS, COLITIS, CHRON, CUCI
13) HERNIAS DIAFRAGMATICAS, HIATAL, DE PARED E INGUINAL
14) HEMORROIDES Y FISTULA ANAL
15) CANCER DE COLON
16) ORQUIEPIDIDIMITIS Y TORCION TESTICULAR.
17) CANCER TESTICULAR.
18) HIPERTROFIA PROSTATICA BENIGNA.
19) CANCER DE PROSTATA.
20) TCE, TRAUMA FACIAL Y TRAUMA RAQUIMEDULAR.
21) TRAUMA TORACICO ABIERTO Y CERRADO
22) TRAUMA ABDOMINAL Y PELVICO
23) PIE DIABETICO Y COMPLICACIONES QUIRURGICAS DE LA DM.
24) RETINOPATIA DIABETICA E HIPERTENSIVA.
25) URGENCIAS OFTALMOLOGICAS, PERFORACION, GLAUCOMA, UVEITIS
26) TRAUMA OTICO, OTITIS EXTERNA MALIGNA, NEURITIS VESTIBULAR, SINDROME DE MENIERE

CURSO ENARM CMN SIGLO XXI TEL: 36246001

Pharmed Solutions Institute

PGINA 417

MANUAL DE TRABAJO DEL CURSO ENARM CMN SIGLO XXI


ESOFAGITIS
CIENCIAS BASICAS: Anatomia: El esfago se origina a nivel del cartlago cricoides. En posisicion erecta mide aprox de 25-30cm, va de C6-T1. Esta
irrigado por las arterias tiroideas inferiores, por las bronquilaes, por ramas directas de la aorta, y arterias frnica y cornoaria estomaquica (gstrica
izquierda). Inervado por fibras motoras del nervio vago, tambin fibras preganglionares parasimpticas. Su inervacin intrnseca esta dada por el
plexo de Meissner (submucoso) y plexo de Auerbach (mienterico, entre la longitudinal y la circular). EEl (Esfinter esofgico inferior) mide de 3-5cm y
tiene tono de 10-20mmHg, esta formado por musculo liso, abrazado por el diafragama y es el principal componente de la barrera antirreflujo.
Esofagitis: Es un trmino general para cualquier inflamacin, irritacin o hinchazn del esfago, el tubo que va desde la parte posterior de la boca
hasta el estmago. PATOGENIA: Disminucin de la eficacia de los mecanismos antirreflujo esofgico, en particular del tono del EEI, presencia de una
Hernia hiatal por deslizamiento, eliminacin inadecuada o lenta del material refluido, reduccin de la capacidad de reparacin de la mucosa
esofgica por exposicin prolongada a jugo gstrico. DIAGNOSTICO: Clnica; Acidez y pirosis, que el enfermo localiza en el epigastrio y en la regin
retroesternal. Factores incrementan el riesgo de esofagitis: Consumo de alcohol, de cigarrillos, ciruga o radiacin en el pecho (ejemplo, tratamiento
para cncer pulmonar), tomar ciertos medicamentos sin mucha agua. Tecnica de eleccin para esofagitis es la endoscopia, no esta indicada en
pacientes con sntomas tpicos, esta indicada cuando hay sntomas de alrma, perdida de peso, anemia, hemorragia, disfagia, sntomas persistentes
progresivos. Clasificacin de Los Angeles para esofagitis: Grado A: Una o ms lesiones de la mucosa confinadas a los pliegues mucosos, cada uno < 5
mm. Grado B: Por lo menos una lesion de la mucosa > 5 mm, confinada a los pliegues mucosos pero sin continuidad en los bordes de dos de los
pliegues. Grado C: Por lo menos una lesion de la mucosa entre los lmites de dos pliegues de la mucosa, pero sin ser circuferencial. Grado D: lesion
circunferencial de la mucosa. Clasificacion de Savary-Miller para esofagotis por reflujo: Grado 0: sin lesiones. Grado I: eritema/erosiones aisladas.
Grado II: erosiones confluentes (no toda la circunferencia). Grado III: erosiones circunferenciales. Grado IV: complicaciones (ulcera, estenosis,
Barret). TRATAMIENTO: Especfico del tipo de Esofagitis que se trate, se utilizan medicamentos como Ranitidina, Famotidina, Omeprazol entre otros,
hay muy poca tendencia a la curacin espontanea. La etiologa ms frecuente es la ESOFAGITIS POR REFLUJO: Es consecuencia del RGE patolgico,
que produce alteraciones inflamatorias de la mucosa esofgica, 60% de los pacientes con sntomas tpicos no presenta alteracin alguna en la
endoscopia. Presente en 2-5% de la poblacin en general. Mas frecuente en hombres que en mujeres. Otras causas son: ESOFAGITIS
MEDICAMENTOSA: alendronato, tetraciclina, doxiciclina, ibandronato, risedronato y vitamina C. ESOFAGITIS CUSTICA: Injuria con agentes
custicos (cidos o lcalis). Las causas de la ingesta pueden ser accidentales, frecuentemente en nios, o voluntarias con fines suicidas,
generalmente en adultos. Deteccin temprana de probable perforacin (examen clnico + Rx trax y abdomen). No usar sonda nasogstrica ni
lavado. No usar agentes neutralizantes. No inducir el vmito. ESOFAGITIS EOSINOFLICA: es una entidad que se caracteriza por la infiltracin tisular
muy significativa, de eosinfilos.
CASO CLINICO
Masculino de 75 aos con antecedentes de hipertensin arterial que
ingres a la UCI con el diagnstico de shock. Present vmitos
alimentarios y pirosis durante las 24 horas previas al ingreso. La
familia refiri un episodio de fiebre y escalofros. Se coloc una
sonda nasogstrica observndose al aspirar un contenido en borra
de caf. No refera hematemesis o melena. El paciente fue
reanimado con infusin de cristaloides y requiri drogas
vasopresoras durante las primeras 24h (noradrenalina 0,1
mcg/kg/min). A su ingreso se realizaron hemocultivos y urocultivo,
ambos fueron positivos a las 48 h para Escherichia coli multisensible.
Se inici tratamiento emprico con ceftriaxona 1 gr cada 12 h que
luego se cambi segn sensibilidad y cumpli tratamiento
antibitico por 14 das. La analtica demostr leucocitosis y anemia
con hematocrito de 30%.
PREGUNTA
Cual de las siguientes medidas no es apropiada en este momento?
RESPUESTA
a.- Inhibidores de la bomba de protones por va endovenosa.
b.- Sucralfato por va oral.
c.- Metronidazol 500 mg/cada 8 h por siete das.
d.- Prednisona via oral
PREGUNTA
La presentacin clnica ms comn de esta patologa es?
RESPUESTA
a.- Sangrado digestivo alto
b.- Hematemesis.
c.- Vmitos en borra de caf
d.- Dolor epigstrico.
PREGUNTA
Cual de los siguientes diagnosticos diferenciales es menos
frecuente?
RESPUESTA
a.- Necrosis local secundaria a infecciones.
b.- Hematoma intramural del esfago.
c.- Lesion por custicos.
d.- Sndrome pilrico con reflujo severo

CURSO ENARM CMN SIGLO XXI TEL: 36246001

CASO CLINICO
Masculino de 42 aos, con obesidad, hipertenso (presin arterial
actual 150/80), dislipidemia, fumador de 15 cigarrillos al da. Acude a
consulta y refiere dolor retroesternal ardoroso moderado y como
que se vuelve la comida del estmago a la garganta, que le deja mal
sabor de boca. Refiere sntomas prcticamente todos los das y
aumentan despus de comidas copiosas e ingestin de bebidas con
gas. En ocasiones sntomas aparecen cuando se acuesta. Niega
disfagia o melena.
PREGUNTA
Cul es la conducta teraputica ms adecuada para iniciar a este
paciente, adems de las medidas higienico-dietticas?
RESPUESTA
a.- Procinticos
b.- Inhibidores de la bomba de protones
c.- Tiazidas
d.- IECA
PREGUNTA
El paciente acude 8 meses despus de suspender medicamento,
refiere continuar con malestar, coincidiendo con un periodo donde
ha estado ms estresado, con ms trabajo, fumando ms, pero
adems astenia, mal estar general y prdida de peso de 8 Kg y tos.
Cuenta con BH con Hb de 11.3g/dl. Qu complicacin, sera la ms
probable de encontrar en este paciente?
RESPUESTA
a.- Esfago de Barret
b.- Acalasia
c.- Pseudoacalasia
d.- Ca de esfago
PREGUNTA
Qu conducta diagnostica sera la ms adecuada para confirmar la
complicacin de este paciente?
a.- Ph metra
b.- Serie esofagogastrodueodenal
c.- Manometria
d.- Endoscopia

Pharmed Solutions Institute

PGINA 418

MANUAL DE TRABAJO DEL CURSO ENARM CMN SIGLO XXI


ACALASIA
CIENCIAS BASICAS: Es un trastorno primario de la motilidad esofgica caracterizado por la ausencia de peristlsis esofgica y por la relajacin
incompleta del EEI, afecta los dos tercios inferiores del esofago. La acalasia se clasifica en primaria o idioptica que comprende el mayor nmero de
casos, y en secundaria (neoplasias, infiltraciones de esfago, estmago, linfomas, enfermedad de Chagas. SALUD PBLICA: La prevalencia de la
enfermedad es de aproximadamente 10 casos por 100.000 hab. Su incidencia se ha mantenido bastante estable durante los ltimos 50 aos en
aproximadamente 0,5 casos por 100.000 hab por ao. Puede manifestarse a cualquier edad, sin predileccin racial y con igual frecuencia en
hombres y mujeres, pero la mayora de los casos es diagnosticada generalmente entre los 25 y 60 aos de vida. PATOGENIA: La regin principal
afectada es el plexo esofgico mientrico (Auerbach) que incluye zonas de respuesta inflamatoria en su mayor parte linfocitos T citotxicos CD3 y
CD8 positivos, un nmero variable de eosinfilos y los mastocitos, la prdida de clulas ganglionares y cierto grado de fibrosis. La etiologa principal
se debe a la perdida selectiva de las motoneuronas inhibitorias, liberadoras de pptido intestinal vasoactivo y xido ntrico del plexo mientrico. La
prdida de estas neuronas ocasiona el incremento en la presin basal, la relajacin incompleta del EEI y la desaparicin de la latencia y naturaleza
peristltica de la contraccin del cuerpo esofgico. Estas alteraciones conllevan a una disminucin de la propulsin esofgica y un incremento en la
resistencia del EEI, son las responsables de la dificultad de trnsito y de la disminucin del aclaramiento esofgico y son la base de los sntomas
clnicos y signos radiolgicos, endoscpicos y manomtricos de la enfermedad. En casos avanzados puede haber afeccin de las neuronas
colinrgicas, incluso a nivel central. DIAGNOSTICO: Clnico; El sntoma inicial y ms comn es la disfagia (90%), que al ser de origen motor y no
mecnico, es tanto a solidos como a lquidos, suele empeorar con las comidas rpidas y la tensin emocional. Puede haber dolor y tambin
regurgitacin (75%) la cual se debe a la retencin de alimentos no digeridos en la porcin inferior del esfago, esta situacin con el tiempo puede
desencadenar esofagitis y pirosis (40%), pero no es lo ms comn. Hay prdida de peso (60%) a lo largo de meses o aos, cuando esta es acelerada
se debe considerar la posibilidad de acalasia secundaria a una neoplasia maligna. Radiografa de trax se puede llegar a observar ausencia de cmara
gstrica y un nivel hidroareo en mediastino. Serie esofagogastroduodenal (EGD), es la primera tcnica diagnostic, muestra dilatacin esofgica
distal que termina en punta de lpiz, y en casos avanzados un esfago sigmoideo (Rx de trax mas EGD). La prueba con la sensibilidad ms alta en la
diagnostico (gold standard) de la acalasia es la manometra esofgica y el nico mtodo posible para el diagnstico de la enfermedad en sus fases
iniciales, en las que todava no se han producido los cambios morfolgicos detectables por mtodos radiolgicos o endoscpicos. Todas las
sospechas diagnsticas de Acalasia deben confirmarse mediante manometra. La especificidad diagnstica de la ME no es del 100%, es importante
destacar que el patrn manomtrico de la Acalasia es indistinguible del ocasionado por la obstruccin mecnica del EEI casi siempre por un tumor
que origina un cuadro denominado pseudoacalasia, que corresponde casi al 5% de los diagnsticos manomtricos de Acalasia y que debe ser
particularmente sospechado en pacientes ancianos o con una clnica rpidamente progresiva. La endoscopia es til para excluir las lesiones
orgnicas, en particular aquellas que son causas secundarias de Acalasia, como el carcinoma del cardias gstrico, entre otros. Durante el
procedimiento se observa a menudo dilatacin y atona del cuerpo esofgico, con tortuosidades en casos avanzados y que el lumen esofgico est
lleno de saliva y restos de comida, y la regin de la unin esfago gstrico est cerrada. TRATAMIENTO: No hay tratamiento especfico. Se
recomienda dieta blanda. Frmacos: anticolinrgicos resultan poco efectivos, nitritos o antagonistas de calcio antes de los alimentos, pero no son
muy efectivos. El sildenafil regula la funcin motora del EEI. La toxina botulnica tiene eficacia de 60% a los seis meses, se utiliza ms bien como una
medida temporal, antes de la ciruga. Por lo general a los pacientes menores de 50 aos, se les propone el quirrgico de primera instancia, de no
haber xito se pueden hacer dilataciones (tx no quirrgico de primera eleccin, despus de 3 dilataciones sin xito hay que hacer cirugia). A los
mayores de dicha edad se les recomienda dilataciones neumticas o la aplicacin de toxina botulnica (dura de 6-7 meses) de manera peridica, si no
hay respuesta se puede proponer tratamiento quirrgico. Despus del ERGE, la acalasia es el trastorno funcional ms frecuente del esfago que
amerita intervencin quirrgica. El alivio puede lograse mediante una rotura instrumental no controlada del musculo del esfnter o con una
miotoma quirrgica controlada, esta conlleva un bajo ndice de morbilidad y logra mejores resultados a largo plazo. La miotoma se puede
acompaar de funduplicatura parcial para disminuir los eventos de reflujo posquirrgico. En la prctica la mayora de los pacientes se somete a
dilatacin. Un riesgo inherente de la dilatacin neumtica es la rotura esofgica. La miotoma modificada de Heller se practica en todas las capas
musculares: se extiende en sentido distal sobre el estmago 1-2 cm por debajo de la unin y se prolonga 4-5cm en sentido proximal sobre el
esfago. La esofagectoma es el mejor tratamiento para los pacientes con disfagia y enfermedad benigna prolongada cuya funcin esofgica se
destruy por enfermedad o mltiples procedimientos quirrgicos. PRONOSTICO: Estos pacientes tienenhasta 7 veces mas de padecer cncer.
CASO CLINICO
Un hombre de 62 aos consulta por un cuadro de tres semanas de
duracin de disnea y vmitos explosivos alimentarios. El da de la
consulta despert con restos de comida en sus narinas y tos
persistente, dificultad progresiva para tragar lquidos y slidos.
Durante los ltimos dos aos, el paciente cuenta haber tenido una
prdida de 12 kilos, disnea ocasional y constipacin. Como
antecedentes, el paciente refiere padecer de RGE, EPOC y fumar un
paquete de cigarrillos al da por ms de 50 aos. Bebe alcohol, de
forma social, pero sin abusar de l. EF: se aprecia un leve apremio
respiratorio y mucosas secas. Su examen respiratorio evidencia un
murmullo pulmonar disminuido, con leves sibilancias espiratorias. Al
examinar abdomen, se constata un dolor moderado a la palpacin
en epigastrio.
PREGUNTA
Basndonos en la historia y el examen fsico del paciente, adems de
los exmenes solicitados, cul de los siguientes es el diagnstico
ms probable?
RESPUESTA:
a.- Acalasia
b.- Carcinoma esofgico
c.- Estenosis pptica
d.- Esclerodermia

CURSO ENARM CMN SIGLO XXI TEL: 36246001

PREGUNTA
Cul es la conducta diagnostica ms adecuada para este paciente?
REPUESTA
a.- Manometria
b.- Endoscopia
c.- pH metria
d.- Serie esofagogastroduodenal
PREGUNTA
El paciente recibe tratamiento mdico, pero 8 meses despus
continua con la misma sintomatologa. Cul es la conducta
teraputica no quirrgica ms eficaz para este paciente?
RESPUESTA
a.- Toxina botulnica
b.- Dilataciones neumticas
c.- Antagonista de calcio
d.- Endoscopia
PREGUNTA
Este paciente debe ser vigilado posteriormente mediante
endoscopia debido al riesgo de desarrollar?
REPUESTA
a.- Mega-esfago
b.- Estenosis esofgica
c.- Cncer de esfago
d.- Hemorragia

Pharmed Solutions Institute

PGINA 419

MANUAL DE TRABAJO DEL CURSO ENARM CMN SIGLO XXI


ENFERMEDAD POR REFLUJO GASTROESOFAGICO (ERGE) Y REFLUJO GASTROESOFAGICO (RGE)
CIENCIAS BASICAS: El RGE se refiere al paso de contenido gstrico hacia el esfago, en ausencia de nauseas, vomito o eructos, puede ser fisiolgico
(mximo 5 episodios al da, fenmeno diurno, de corta duracin, no evoca sntomas, no daa la mucosa esofgica) o patolgico. Se habla de ERGE
cuando el reflujo ocasiona sntomas o complicaciones, como consecuencia del RGE. La enfermedad por reflujo se manifiesta de diferentes formas:
ERGE no erosiva (70%), ERGE erosiva (25%), ERGE asociada a esfago de Barret (4%). La regurgitacin se define como el paso del contenido gstrico
hacia el esfago y/o hacia la boca sin esfuerzo alguno es decir, no hay contraccin del diafragma; es el regreso involuntario hacia la boca, de
comida o secreciones previamente deglutidas. SALUD PUBLICA: El reflujo gastroesofgico ocasiona cerca de 75% de los padecimientos esofgicos. El
ERGE se presenta en 3-4% de la poblacin general, con aumento en su prevalencia en embarazadas y adultos mayores. PATOGENIA: Los tres
mecanismos ms importantes que condicionan la incompetencia de la barrera anti-reflujo son: 1) Las relajaciones transitorias del EEI, 2) la
hipotensin del EEI, y 3) la alteracin anatmica de la unin condicionada por la presencia de hernia hiatal. Otros factores que interactan; la
ansiedad, la depresin o el estrs, de tal manera que estmulos mnimos en el esfago desencadenan dolor. La ingesta de alimentos irritantes y
grasas tambin pueden intensificar la percepcin del cido. La obesidad es un factor de riesgo para la ERGE. El reflujo cido estimula el nervio vago y
esto es la causa de la broncoconstriccin; as como el reflujo de pepsina es la causante de la enfermedad ulcerosa pptica. El tabaquismo incrementa
los sntomas de RGE. Aunque no hay estudios concluyentes, se ha demostrado que la ingesta en exceso de ctricos, bebidas carbonatadas, caf,
cafena, chocolate, cebolla, especies, comidas ricas en grasas, menta, yerbabuena, alimentos salados y la fibra de algunos tipos de pan, pueden estar
relacionados con el reflujo. Pueden aumentar los sntomas en la ingesta de medicamentos no esteroides y AINES. DIAGNOSTICO: Clnico; Los
sndromes esofgicos de la ERGE se caracterizan por la presencia de sntomas tpicos pirosis (ms caracterstico y frecuente) y regurgitaciones, e
incluso dolor torcico o asociados a lesiones esofgicas como: erosiones, estenosis, Barrett y adenocarcinoma. Las manifestaciones extra-esofgicas
ms comunes son tos (38%), dolor torcico (30%), disfona (22%), asma (5%), escurrimiento nasal posterior (5%). Al parecer la presencia de sntomas
tpicos est relacionada con reflujo cido, mientras que el reflujo no cido (bilis, aire, etc.) se relaciona con la aparicin de sntomas atpicos. El RGE
que afecta severamente a la laringe puede contribuir a la aparicin de cncer en la laringe. Al parecer el 46% de todas las apneas estn relacionadas
con episodios de reflujo. Se ha observado que la pobre calidad de sueo est relacionada a un gran nmero de eventos de reflujo mediante pHmetra. Dentro de las herramienta diagnosticas se cuenta con cuestionarios de sntomas que otorgan una puntuacin, con base en lo cual se
determina si el paciente tienen o no ERGE. Ensayo teraputico con inhibidores de bomba de protones (IBP) en pacientes no complicados, con una
dosis estndar por 1-2 semanas (omeprazol 40mgs c/12hrs, pantoprazol 80 mg c/12hrs). La desaparicin de los sntomas con el tratamiento y la
recada al suspenderlo confirma el diagnstico. En caso de pacientes con sntomas extraesofgicos se recomienda extender el tiempo de
administracin hasta tres meses. La sensibilidad y especificidad varan entre 75 y 92% y 55 al 90% respectivamente. La pH-metra es la prueba ms
sensible y especfica y est indicada en ERGE, que no respondi a tratamiento, en ERGE posquirrgico. Los estudios radiogrficos son tiles para
visualizar defectos anatmicos como: Presencia de hernia hiatal, anillos, estrecheces, pero no es til para valorar el reflujo ni la esofagitis. El
esofagograma es til para valorar el trnsito esofgico, en casos de disfagia posoperatoria o cuando se sospecha neoplasia, debe realizarse en el
segundo o tercer nivel de atencin en pacientes posquirrgicos de hernia hiatal, en los cuales hay disfagia. TRATAMIENTO: No recostarse justo
despus de las comidas, evitar alimentos que relajen el EEI, evitar ropa ajustada (mejora 20%). Los anticidos controlan sntomas leves de pirosis y
regurgitacin, pero no curan la esofagitis. Los inhibidores de la secrecin acida; son la piedra angular, los antagonistas H2 (ranitidina), logran
curacin de esofagitis en 50%. Los IBP logran curacin de 80% a 4 sem y cerca de 90% a 8 sem. El tratamiento de reflujo alcalino es a base de
sucralfato, colestiramina o hidrxido de aluminio. La ciruga antirreflujo est indicada en pacientes con esofagitis, manifestaciones extraesofgicas
de difcil control. Antes de la ciruga se debe realizar una manometra y, adems se sugiere corroborar el diagnostico por pH-metra en 24 hrs. El
procedimiento antireflujo ms frecuente es la funduplicacin de Nissen que solo usa el fondo gstrico para envolver el esfago, la funduplicacin se
calibra con un dilatador 60 francs y la longitud de la misma se limita a 1-2cm. Cuando la motilidad esofgica no es normal y la fuerza propulsiva no
es suficiente para vencer la obstruccin de una funduplicatura completa, est indicada una funduplicacin parcial. En pacientes con esfago corto
por estrechamiento, esfago de Barret o una hernia hiatal grande se realiza una gastroplastia de Collis como procedimiento para prolongacin
esofgica. La ciruga antirreflujo disminuye la aparicin de displasia pero no eliminan el riesgo de malignizacion, esta indicada en: ERGE refractaria,
esofagitis grado 2 o superior, estenosis que no se controla con dilataciones, hemorrgia, sntomas respiratorios, hernia hiatal paraesofagica.
CASO CLINICO REFLUJO GASTROESOFAGICO
Se trata de paciente femenino de 24 aos de edad, acude a consulta
por opsomenorrea y oligomenorrea, agrega que no tiene vida sexual
activa desde hace 2 aos, refiere adems que ltimamente se siente
fatigada y ha presentado mareos ocacionales, niega uso de
sustancia, agrega que ocacionalmente toma ranitidina, finalmente
refiere regurgitacin y pirosis. A la exploracin fsica se observa IMC
de 17, pelo quebradizo, palidez generalizada, llenado capilar de 3
segundos, los exmenes de laboratorio reportaron hemoglobina 10.
PREGUNTA
Cual de los siguientes diagnosticos es la menos frecuente en el caso
clnico?
RESPUESTA
a.- Gastritis erosiva.
b.- Esofagitis.
c.- Enfermedad por reflujo gastroesofagico.
d.- Anorexia nerviosa.
PREGUNTA
Luego de tratamiento con omeprazol 20 mg/da, presentando leve
mejora sin embargo no se presento mejora de la hemoglobina y el
hematocrito, a la exploracin se observa halitosis y hemorragia
suconjuntival, refiere que ha incrementado su ansiedad por
presiones laborales ya que su trabajo requiere mantenerse en talla

CURSO ENARM CMN SIGLO XXI TEL: 36246001

5, considerando la evolucin del paciente. cual es la patologia


asociada que esta modificando la respuesta?
RESPUESTA
a.- Anorexia purgativa.
b.- Esofagitis erosiva.
c.- Esofago de barret.
d.- Ulcera gstrica.
PREGUNTA
La paciente regresa a consulta de seguimiento 3 meses despues, su
tratamiento instalado fue de fluoxetina 20 mg/24 hrs, asi como
complejo b y hierro oral, los laboratorios reportaron hemoglobina 11
y hematocrito de 38, se realizo endoscopia donde se reporta una
lesin mucosa mayor de 5 mm, sin continuidad entre la parte ms
prominente de 2 pliegues mucosos, que grado presenta la paciente
de la Clasificacin de Los ngeles de la esofagitis.
RESPUESTA
a.- Grado A.
b.- Grado B.
c.- Grado C.
d.- Grado D.
PREGUNTA
Se indico cambios en el estilo de vida, tratamiento farmacolgico por
6 meses, a la exploracin se observo IMC de 23, se realizo una

Pharmed Solutions Institute

PGINA 420

MANUAL DE TRABAJO DEL CURSO ENARM CMN SIGLO XXI


endoscopia de control con el siguiente reporte, unin escamosocolumnar se encuentra desplazada proximalmente de color rojo de
la mucosa columnar el reporte histolgico reporto existencia de
neutrfilos y eosinfilos que infiltraban la mucosa y la lmina propia,
considerando las condiciones actuales, cual es la conducta a seguir.
RESPUESTA
a.- Vigilancia estrecha endoscopia anual.
b.- Funduplicatura.
c.- Fototerapia endoscpica.
d.- Esofagectomia.
CASO CLINICO ENFERMEDAD POR REFLUJO GASTROESOFAGICO
Masculino de 52 aos de edad que acude a consulta debido a que
presenta sensacin de quemazn retrosternal que se irradia de
manera ascendente a cuello y espalda, que ha identificado se
presenta entre media hora y una hora despus de la alimentacin,
acteriormente se presentaba solo cuando comia grasas e irritantes,
pero actualmente con casi cualquier alimento y solo se resuelve
tomando anticidos OTC. Antecedente de alcoholismo, tabaquismo
positivo, tos crnica y osteoartitis controlada con diclofenaco. A la
exploracin fsica se observa perdida del esmalte dental y halitosis.
PREGUNTA
Cul es la conducta a seguir este caso?
RESPUESTA
a.- Omeprazol 20 mg cada 12 hrs.
b.- Misoprostol 400 g cada 12 hrs.
c.- Sales de bismutos 10 mg cada 24.
d.- Malgadrato 5 ml cada 8 hrs.
CASO CLINICO ENFERMEDAD POR REFLUJO GASTROESOFAGICO
Varn de 46 aos, sin alergias conocidas, fumador de 12 cigarrillos al
da y bebedor de 40 g de alcohol diarios. Antecedentes personales
de apendectoma a los 14 aos. Hace cuatro aos fue medicado con
ansiolticos por haber padecido varias crisis de ansiedad. Desde hace
18 meses aqueja sensacin de quemazn en regin epigstrica que
se irradia hacia la parte central de trax y, ocasionalmente, hasta el
cuello. Las molestias aumentan despus de la ingesta as como en
decbito. Mejoran slo un poco, y temporalmente, con la toma de
anticidos. No refiere tener regurgitacin, disfagia, odinofagia ni
dolor torcico. Tampoco padece sntomas como nuseas o vmitos.
Su estado general es bueno sin haber presentado astenia, anorexia
ni adelgazamiento. Medicado por su mdico de cabecera con
ranitidina 300 mg por la noche, ha tenido una mejora parcial que
refiere como 3 en una escala de 0 a 10. A la exploracin fsica
encontramos un paciente consciente y orientado. Coloracin normal
de piel y mucosas. No se objetivan adenomegalias ni lesiones
drmicas. Auscultacin cardiaca y pulmonar sin alteraciones.
Abdomen blando y depresible; no se palpan ni se percuten masas ni
visceromegalias. Exploracin neurolgica sin alteraciones.
PREGUNTA
Ante la clnica observada en este paciente. Cual es el diagnostico
mas probable?
RESPUESTA
a.- Dispepsia funcional.
b.- Enfermedad por reflujo gastroesofgico.
c.- Esofagitis por cndida.
a.- Acalasia.
PREGUNTA
Cul sera la siguiente exploracin a realizar para confirmar el
diagnstico clnico?
RESPUESTA
a.- pH-metra esofgica de 24 horas.

CURSO ENARM CMN SIGLO XXI TEL: 36246001

b.- Esofagograma.
c.- Prueba de tratamiento emprico con omeprazol.
d.- Manometra esofgica.
PREGUNTA
El paciente continuo con manifestaciones clnicas, considerando este
punto. cual es la conducta diagnostica ms adecuada a seguir para
establecer finalmente un diagnostico?
RESPUESTA
a.- pH-metra esofgica de 24 horas.
b.- Endoscopia y, si sta fuese negativa, pH-metra esofgica de 24
horas.
c.- Endoscopia y, si sta fuese positiva, pH-metra esofgica de 24
horas.
d.- Manometra esofgica.
PREGUNTA
La pH-metra esofgica fue informada como reflujo gastroesofgico
cido cuantitativamente normal con un ndice bajo de correlacin
sintomtica, cual es el diagnostico considerando los datos
observado?
RESPUESTA
a.- Dispepsia funcional.
b.- Somatizacin.
c.- Ansiedad.
d.- Pirosis funcional
PREGUNTA
Considerando la direccin que ha tomado el caso actual, cul cree
que sera el tratamiento ms adecuado para este paciente?
RESPUESTA
a.- Alcalinos despus de las comidas.
b.- IBP a dosis habituales.
c.- IBP a dosis superiores (al menos doble).
d.- Antagonista de los receptores H2
CASO CLINICO
Masculino de 48 aos de edad acude con su medico de atencion
primaria por dolor ardorosos recurrente en el pecho despus de
comidas abundantes. El dolor empeora al reclinarse y disminuye con
tabletas de anticido de venta libre. Tambien informa un sabor acido
en la boca, y tos persistente durante los 6 meses anteriores. Ha
fumado 2 cajetillas de cigarros al dia durante 20 aos pasados. EF: T
37C, pulso 91/min TA 149/90mmHg. Mide 1.75mts y pesa 135 Kgs.
Los resultados del examen fsico son marcados.
PREGUNTA
Cul es el sntoma menos probable que debe esperar el paciente a
que aprezcan con el tiempo sin tratamiento?
RESPUESTA
a.- Esofago de Barret
b.- Disfagia persistente
c.- Sangrado de tubo digestivo alto
d.- Estenosis del piloro
PREGUNTA
Cul es el diagnotico inicial de esta patologa?
RESPUESTA
a.- Endoscopico
b.- Tomografico
c.- Clinico
d.- Ph metria

Pharmed Solutions Institute

PGINA 421

MANUAL DE TRABAJO DEL CURSO ENARM CMN SIGLO XXI


ESOFAGO DE BARRET (EB)
CIENCIAS BASICAS: Es la transformacin metaplsica del epitelio esofgico, en el estudio patolgico sin importar la extensin. La marca distintiva de
la metaplasia intestinal es la presencia de clulas caliciformes intestinales. Recodar que RGE provoca un proceso inflamatorio de la mucosa
esofgica, llamada Esofagitis por Reflujo. Esta ltima, predispone a complicaciones o secuelas crnicas como lceras, estenosis displasia, las que
son generalmente irreversibles y que en algn caso, como sucede en el EB, son lesiones consideradas como precancerosas o de mayor riesgo de
cncer. SALUD PUBLICA: El EB se presenta en hasta 10% de los pacientes con RGE crnico y ste se presenta con una frecuencia estimada de entre
20 y 40% de la poblacin. Es ms frecuente en el sexo masculino y en caucsicos, y la edad media al diagnstico es de 60 aos aproximadamente. Se
ha descrito tambin asociacin familiar. La estimacin del riesgo de adenocarcinoma es variable, va de 1 en 50 a 1 en 400 pacientes-ao. Cerca de la
tercera parte de los pacientes con EB presenta neoplasia maligna. PATOGENIA: Normalmente toda lesin del epitelio epidermoide (escamoso) cura
mediante regeneracin de clulas epidermoides. En el EB, por causas an desconocidas, el epitelio epidermoide lesionado es reemplazado por
epitelio columnar cilndrico pseudoestratificado. Inicialmente se postul un origen congnito, pero actualmente se acepta el EB como
una condicin adquirida, en la que una lesin intensa y sostenida sobre el epitelio del esfago distal, generalmente debida a RGE, provoca el
reemplazo del epitelio plano pluriestratificado normal por un epitelio columnar metaplsico, ms resistente al cido. El origen de este nuevo epitelio
seran clulas totipotenciales ubicadas en la capa basal o germinativa del epitelio esofgico. Estudios de pHmetra de 24 hrs muestran que los
pacientes con EB tienen mayor intensidad y frecuencia de exposicin al reflujo cido que el resto de los pacientes con RGE, lo que pudiera
relacionarse con presiones del esfnter esofgico inferior ms bajas y peristalsis esofgica menos activa. DIAGNOSTICO: No existe una clnica del
esfago de Barrett, los sntomas que presentan estos pacientes son los propios del reflujo gastroesofagico, o de sus complicaciones: Pirosis,
regurgitaciones, epigastralgias, dolor torcico retro-esternal que se irradia al cuello y brazos, asfixia nocturna, tos. La realidad de su diagnstico
depende esencialmente de la endoscopia y de la biopsia, con histologa. Biopsia se hace a nivel distal de la unin escamosa-columnar, y ella se
practica en el rea Inferior a la lnea z, que es la (unin escamo-columnar), El endoscopista reconoce el borde prximo de los pliegues gstricos aun
con la existencia de una Hernia Hiatal, es la marca endoscpica ms segura para identificar la unin mucosa escamosa columnar, adems el llamado
el epitelio intestinal, tambin debemos considerarlo como esfago de Barrett , por este motivo la nueva definicin de esfago de Barrett es: La
presencia de mucosa columnar, con metaplasia intestinal en el esfago inferior, incluida la mucosa que tapiza el esfnter esofgico inferior. La
importancia de este cambio de definicin que la metaplasia de Barrett, se ha convertido en un criterio histolgico ms que endoscopio, ya que solo
sera posible identificar la metaplasia intestinal, mediante el estudio histolgico, con esta nueva definicin se puede distinguir dos tipos de esfago
de Barrett: 1. El esfago de Barrett clsico o largo de 3 ms cm. 2. El esfago de Barrett corto incluso ultracorto menos de 3cm. La mucosa
escamosa, tiene una coloracin blanquecina rosa plido en cambio la mucosa columnar y tiene una coloracin rojiza. Seguimiento endoscpicobiopsia. Diferenciar 3 subgrupos: a) pacientes sin displasia, seguimiento endoscpico cada 2 aos. b) Pacientes con displasia leve o algunos de los
marcadores tumorales positivos; hacer biopsia en los 4 cuadrabtes cada 2 cm, se da tratamiento con omeprazol 40mg c/12 hrs por 3 meses y repetir
endoscopia, si el hallazgo se mantienen considerar ciruga antirreflujo , se har control endoscpico-bipsico cada 6 meses hasta que en dos
exploraciones consecutivas no encontremos displasia ni otros marcadores tumorales positivos, con lo cual se pasara a control anual como en el
grupo anterior. c) Pacientes con displasia severa, en los que se repite la endoscopa-biopsia inmediatamente, ya que la progresin a cncer ocurre
entre 2-3 aos, continuar con vigilancia endoscpica, tx, Esofaguectoma, confirmada por dos patologos. Y debe plantearse seriamente el
tratamiento quirrgico u otras opciones teraputicas si hay contraindicacin a la ciruga. TRATAMIENTO: La esofagitis por reflujo es el primer
eslabn en la progresin de la enfermedad hacia el EB. Esta progresin slo puede evitarse haciendo que el material refluido pierda su agresividad o
que no se produzca el reflujo. Medidas higieno-dietticas comunes a todos los pacientes y el empleo de una medicacin (IBP, por tiempo prolongado
y altas dosis) que ha mostrado ser muy eficaz. En teora, la ciruga antirreflujo, si consigue evitarlo sera el mejor mtodo de tratamiento de la
esofagitis y de profilaxis del EB. Sin embargo, la ciruga slo se reserva para los pocos casos en los que fracasa el tratamiento mdico o en
circunstancias individuales especiales. Hasta ahora, ni el tratamiento mdico ni el quirrgico del RGE han demostrado en forma consistente una
regresin significativa del epitelio metaplsico o disminucin del riesgo de cncer esofgico, ni la terapia mdica ni la quirrgica evitan la necesidad
del seguimiento endoscpico-bipsico para detectar preczmente displasia o cncer de la mucosa metaplsica. Tratamiento quirrgico:
Funduplicatura de Nissen; que consiste bsicamente en el cierre del hiato esofgico, la seccin de vasos cortos para movilizar el fondo gstrico y la
colocacin de puntos que toman el fondo gstrico anterior, la pared muscular el esfago y el fondo gstrico posterior. Las complicaciones de esta
tcnica son la disfagia, el desgarro de la sutura con prdida de la vlvula, y la imposibilidad de eructar por hipercontinencia de la vlvula. Para
prevenir estas complicaciones existen variantes de la tcnica de Nissen. No son pocos los autores que proponen la Esofagectoma como nico
tratamiento con posibilidades de curacin, por supuesto asociada a alta morbi-mortalidad operatoria. Para pacientes con Barrett largo y con larga
expectativa de vida, buen desempeo funcional y capacidad fsica y psquica de tolerar la ciruga, debe considerarse la extirpacin quirrgica
mediante la esofagectoma o la tcnica de Merendino, como primera eleccin, en centros con experiencia y que acrediten sus resultados de
aceptablemente baja morbimortalidad. Para los casos que no deseen ciruga, o para aquellos con pobre reserva funcional, las posibilidades son dos:
tratarlos en un protocolo de ablacin endoscpica o ser sometidos a vigilancia endoscpica-histolgica. La primera debe considerarse an en etapa
de investigacin pues no se ha demostrado fehacientemente su rol teraputico en el largo plazo, no obstante sus innegables ventajas de buena tasa
de eliminacin del epitelio alterado en el corto y mediano plazo, manteniendo inalterada la continuidad anatmica del tubo digestivo superior. La
segunda ciertamente es la conducta menos agresiva y probablemente de mayor atractivo para los pacientes, pero de ms riesgo de progresin a
cncer y obliga en todo caso a varias endoscopas cada ao. COMPLICACIONES: Ulcera de Barret, estenosis de Barret, Adenocarcinoma (0-5-1%),
asociado mayormente a Barret largo.
CASO CLINICO
Un hombre de 67 aos de edad, acude por disfagia a alimentos
slidos y lquidos, tos de predominio nocturno, cuenta con
antecedente de ERGE con tratamiento irregular, agrega que ha
perdido 10 kilos en los ltimos tres meses, niega sangrado sus
constantes vitales se encuentran dentro de parmetros normales.
PREGUNTA
Cual es su conducta a seguir?
RESPUESTA
a.- Realizar pH-metria.
b.- Realizar manometra.
c.- Realizar serie gastroesofgica.

CURSO ENARM CMN SIGLO XXI TEL: 36246001

d.- Realizar endoscopia.


PREGUNTA
Se realizaron diversos estudios dentro de los cuales se encontra una
endoscopia la cual mostr una lesin plana, de color rojizo en la
mitad de esfago. Esta lesin no se tio con solucin de Lugol. Cual
es su impresin diagnostico en este momento?
RESPUESTA
a.- Adenocarcinoma.
b.- Esofagitis por reflujo.
c.- Acalasia.
d.- Esofago de barrett

Pharmed Solutions Institute

PGINA 422

MANUAL DE TRABAJO DEL CURSO ENARM CMN SIGLO XXI


PREGUNTA
Considerando su diagnostico. cual es el tratamiento mas apropiado
para el caso?
RESPUESTA
a.- Dilatacion neumtica.
b.- Funduplicatura de nissen.
c.- Esofagectomia.
d.- Terapia fotodinmica.
PREGUNTA
Luego del tratamiento indicado, 3 meses despus el paciente
presenta dolor severo, disfagia a liquidos con perdida de 5
kilogramos. cual es la conducta a seguir?
RESPUESTA
a.- Dilatacion neumtica.
b.- Funduplicatura de nissen.
c.- Esofagectomia.
d.- Terapia fotodinmica.
CASO CLINICO ESOFAGO DE BARRETT
Luego de realizarse una endoscopia una paciente de 41 aos
presenta incremento de los cuadros de reflujo gastroesofagico de
predominio nocturno, la endoscopia reporto lesiones displasicas en
el tercio distal del esfago y mucosa gstrica despulida, la paciente
agrega disminucin de peso corporal debido a que come poco por la
sensacin ardorosa a la deglucin, existen antecedentes de cncer
en la familia.
PREGUNTA
Cul es el tratamiento que presenta mejor pronstico?
RESPUESTA
a.- Fototerapia.
b.- Ciruga.
c.- Radioterapia.
d.- Quimioterapia.

PREGUNTA
Cual es el objetivo del tratamiento medico del esfago con
metaplasia columnal?
RESPUESTA
a.- Regresin de la metaplasia columnar.
b.- Retrasar la progresin.
c.- Disminuir la displasia columnar.
d.- Evitar el adeno carcinoma.
PREGUNTA
Cual es el tratamiento quirrgico cuando la biopsia reporta
metaplasia columnar?
RESPUESTA
a.- Fototerapia.
b.- Funduplicatura.
c.- Esofagectomia.
d.- Ablacin.
CASO CLINICO
Paciente masculino de 75 aos, diabtico descompensado son
sntomas de poliuria, polidpsia, fuma una cajetilla de cigarros al da y
es alcohlico social, con antecedente de reflujo gastroesofgico
crnico evidenciado por la frecuencia de presentacin de sntomas
tpicos (agruras y pirosis), con un tiempo de evolucin promedio de
casi 5 aos. Se le indica una endoscopia de esfago con toma de
biopsia, la cual nos reporta displasia leve.
PREGUNTA
Adems de la displasia leve, los estudios nos reportan globlet cells,
cual es el diagnostico mas probable?
RESPUESTA
a.- Adenocarcinoma
b.- Carcinoma epidermoide
c.- Esfago de Barret
d.- Leiomioma

PREGUNTA
Cual es la conducta si hay displasia leve?
RESPUESTA
a.- Endoscopia y biopsia.
b.- Endoscopia y biopsia confirmada.
c.- Endoscopia y biopsia seriada.
d.- Endoscopia sin biopsia.
CASO CLINICO ESOFAGO DE BARETT
Se trata de masculino de 41 aos de edad el cual acude a consulta
debido a que presenta pirosis, disfagia y regurgitacin, tratado con
omeprazol por un ao a dosis adecuada.
PREGUNTA
Qu complicaciones mas frecuente que esperara en este caso?
RESPUESTA
a.- Ulcera de barett.
b.- Estenosis de barett
c.- Adherencia
d.- Cncer de esofago
PREGUNTA
En paciente con esofagitis y esfago de barett, cual ser la conducta
a seguir mas adecuada?
RESPUESTA
a.- Reparacin quirurgia y funduplicatura.
b.- Repetir y corroborar con biopsia.
c.- Corroborar con dos patlogos.
d.- Ablacion con fototerapia.

CURSO ENARM CMN SIGLO XXI TEL: 36246001

PREGUNTA
Con cual tipo de barret anatomica esta mas frecuente relacionado
con presentacion de Ca?
RESPUESTA
a.- Barett corto.
b.- Barett largo.
c.- Barett bajo.
d.- Barett alto.

PREGUNTA
Cul es la conducta teraputica inicial ms adecuada para este
paciente?
RESPUESTA
a.- IBP por 3 meses
b.- Esofagectoma
c.- Endoscopia cada 3 meses
d.- Inhibidores H2
PREGUNTA
El paciente termina su tratamiento y a los 6 meses se le realiza una
nueva endoscopia con toma de biopsia, la cual reporta displasia
grave. Por lo que se manda con un nuevo patlogo para nueva toma
de biopsia y este confirma nuevamente el diagnostico. De acuerdo
a estos reportes cual es la conducta ms adecuada para este
paciente?
RESPUESTA
a.- IBP por 3 meses
b.- Esofagectoma
c.- Endoscopia cada 3 meses
d.- Inhibidores H2

Pharmed Solutions Institute

PGINA 423

MANUAL DE TRABAJO DEL CURSO ENARM CMN SIGLO XXI


CANCER DE ESOFAGO
CIENCIAS BASICAS: Se origina en la capa ms interna (la mucosa) y crece hacia fuera (a travs de la submucosa y la capa muscular). Debido a que dos
tipos de clulas pueden cubrir el esfago, existen dos tipos principales: el carcinoma de clulas escamosas (epidermoide) y el adenocarcinoma. Los
cnceres que se originan de clulas glandulares se llaman adenocarcinomas. Este tipo de clula no es normalmente parte del revestimiento interno
del esfago. Antes de que se pueda desarrollar un adenocarcinoma, las clulas glandulares tienen que reemplazar un rea de las clulas escamosas,
como en el caso del esfago de Barrett. Esto ocurre principalmente en la parte inferior del esfago, donde ocurren la mayora de los
adenocarcinomas. Los cnceres que se originan en el rea donde el esfago se une al estmago (la unin GE), lo que incluye aproximadamente las
primeras 2 pulgadas del estmago (llamada cardias), suelen comportarse como cnceres de esfago de manera que son agrupados como cnceres
esofgicos. Clasificados como tumores benignos esofgicos se encuentran, entre otros, el papiloma escamoso, el adenoma esofgico, y los plipos.
SALUD PUBLICA: El cncer de esfago es el cuarto tumor ms frecuente del aparato digestivo (por detrs del cncer colorrectal, gstrico y heptico)
y se sita entre los diez cnceres ms frecuentes en el mundo. Su incidencia es mayor en areas de alta frecuencia Asia y frica Central y del Sur. Es
ms frecuente en el hombre en relacin 3-10 hombres por cada mujer. La edad habitual de presentacin es entre los 55 y los 70 aos. En los ltimos
aos la incidencia del adenocarcinoma del tercio distal del esfago y de la unin GE se ha incrementado de forma paralela a la ERGE, especialmente
en personas con alto ndice de masa corporal. PATOGENIA: Los factores de riesgo asociados al desarrollo de carcinoma escamoso de esfago son:
tabaco, alcohol, extraccin social baja, dieta: dficit nutritivos, hipovitaminosis, ingesta de alimentos con alto contenido de nitrosaminas, acalasia,
esfago de Barrett, lesiones por custicos, Sndrome de Plummer-Vinson, neoplasias de cabeza y cuello, cncer de mama cuyo tratamiento haya
comprendido la radioterapia, ingesta de bebidas calientes. Los factores de riesgo para adenocarcinoma de esfago son: RGE, obesidad, infeccin por
H. pylori, sexo masculino, estrato social bajo, tabaco, alimentos con alto contenido de nitrosaminas. El adenocarcinoma de esfago distal y de la
unin gastroesofgica surge tpicamente en un epitelio con metaplasia, circunstancia conocida como esfago de Barret. Aproximadamente el 60% de
los casos de adenocarcinoma de esfago distal o de la unin gastroesofgica presentan evidencia de esfago de Barret. DIAGNOSTICO: En las etapas
iniciales de la enfermedad, el cncer de esfago habitualmente es asintomtico. Los sntomas ms frecuentemente asociados al cncer de esfago
son: Disfagia (primero, solidos luego a lquidos), prdida de peso, dolor retroesternal, sntomas respiratorios (tos e infecciones respiratorias), otros
sntomas: sangrado, disfona (ronquera) e hipo, hemorragia intensa o parlisis de cuerdas vocales. Es importante resaltar que estos sntomas
tambin pueden estar causados por otras enfermedades del esfago, o por otras causas menos serias. El diagnostico se hace con endoscopia y toma
de biopsia, biopsias tanto de la mucosa del esfago como de ganglios linfticos regionales sospechosa. TAC de trax y abdomen: permite conocer,
basndose en su tamao, la extensin local del tumor, la posible afectacin de ganglios cercanos o a distancia, y la extensin a otros rganos
(pulmn, hgado.). La estadificacin debe incluir TAC de trax y abdomen y en candidatos a tratamiento quirrgico se debe agregar esofagograma y
ultrasonido transendoscopico, para definir con mayor precisin T y N. Si se tiene disponible la tomografa por emisin de positrones (PET), se puede
utilizar con el fin de detectar metstasis a distancia ocultas. Clasificacin simplificada: Estadio 0: tambin denominado carcinoma in situ. El cncer
se localiza slo en la parte ms superficial de la mucosa esofgica, sin sobrepasarla. Estadio I: Tumores bien diferenciados, sin infiltracin ganglionar
locorregional por metstasis, que alcanzan como mximo la capa adventicia. Estadio II: El tumor no alcanza la capa adventicia, asociando un mximo
de dos ganglios linfticos locorregionales infiltrados por metstasis; alcanza sin sobrepasar la capa adventicia sin presentar ganglios linfticos con
metstasis. Estadio III: El tumor afecta a la capa adventicia, asociando un mximo de 2 ganglios linfticos locorregionales infiltrados por metstasis;
presenta afectacin ganglionar con ms de 2 ganglios infiltrados. Estadio IV: El tumor se ha extendido a distancia, a otros rganos del cuerpo.
TRATAMIENTO: Existen tres opciones fundamentales de tratamiento para el cncer de esfago: Ciruga: Su objetivo es extirpar el tumor con
mrgenes libres, es decir, sin dejar enfermedad residual. Se considera indicada en estadios iniciales de la enfermedad (resecable). Es el tratamiento
ms frecuentemente utilizado para el cncer de esfago. El tipo de ciruga vara en funcin del tamao, localizacin y extensin de la enfermedad. La
esofagectoma es la extirpacin quirrgica de parte o la totalidad del esfago, conectando la parte sana restante al estmago, para as permitir la
alimentacin del paciente. Es factible la utilizacin de parte del intestino para realizar la conexin. En la esofagectoma se incluye la extirpacin de
los ganglios linfticos cercanos al esfago, para su estudio al microscopio, con el fin de conocer el grado de extensin de la enfermedad. En
ocasiones puede ser necesaria la realizacin de una gastrostoma de alimentacin, ante imposibilidad para la ingesta de alimentos (condicionada por
una estrechez infranqueable del esfago). Esta tcnica puede realizarse va endoscpica o de manera quirrgica. Radioterapia: se puede emplear
como tratamiento nico formando parte de una estrategia multidisciplinar (combinada con la ciruga y/o la quimioterapia). Se utiliza con intencin
curativa, en estadios iniciales de la enfermedad, con intencin paliativa, en estadios avanzados, para aliviar sntomas tales como el dolor y la
disfagia. Radioterapia externa: consiste en la utilizacin de una mquina fuera del cuerpo para enviar la radiacin al rea concreta que se desea
tratar. Radioterapia interna: en sta, una sustancia radioactiva se coloca (mediante un sistema sellado) muy cerca o dentro del tumor, permitiendo
as alcanzar ms dosis de radiacin con menores efectos secundarios sobre los tejidos sanos. En el cncer de esfago generalmente se utiliza para el
tratamiento de la disfagia. Quimioterapia: Consiste en la introduccin de un frmaco al torrente sanguneo (quimioterapia sistmica) para eliminar
las clulas cancerosas de todo el cuerpo (dentro y fuera del esfago). La quimioterapia se puede emplear como tratamiento nico o formando parte
de una estrategia multidisciplinar (combinada con la ciruga y/o la radioterapia). Su utilizacin de forma aislada, previa a la ciruga, se considera un
tratamiento en investigacin. Otros tratamientos con posible indicacin en el cncer de esfago son la terapia lser (destruccin del tumor con
lser), y la colocacin de prtesis esofgicas (stent), consistente en la implantacin endoscpica de un tubo expandible. Esta ltima tcnica se utiliza
para resolver la estrechez esofgica provocada por el tumor y permitir as la adecuada alimentacin del paciente.
CASO CLINICO
Mujer de 51 aos, con antecedente de tabaquismo y asma. Consult
por disfagia, nuseas, vmitos y baja de peso. La endoscopia mostr
una lesin polipoidea, mamelonada y ulcerada en el tercio medio del
esfago que comprometa casi todo el lumen esofgico. El estudio
anatomopatolgico de la lesin mostr un tumor slido, infiltrante,
con caractersticas morfolgicas e inmunohistoqumicas compatibles
con un melanoma maligno. La tomografa computada de trax
mostr una lesin tumoral, con crecimiento intraluminal, en el tercio
distal del esfago. La lesin comprometa el cardias y comprima la
aurcula izquierda. No hubo evidencia de compromiso de rganos
adyacentes ni metstasis. La tomografa computada de abdomen y
pelvis fue normal.

CURSO ENARM CMN SIGLO XXI TEL: 36246001

PREGUNTA
Cual es el pronstico del caso?
RESPUESTA
a.- 6 a 12 meses de vida.
b.- 12 a 18 meses de vida.
c.- 18 a 24 meses de vida.
d.- 24 a 30 meses de vida.
CASO CLINICO
Hombre de 59 aos, sin antecedentes mrbidos. Consult por
disfagia asociada a baja de peso. La endoscopia digestiva alta
demostr una lesin polipoidea de 5 cm, pigmentada, en el tercio
distal del esfago. El estudio anatomopatolgico de la biopsia

Pharmed Solutions Institute

PGINA 424

MANUAL DE TRABAJO DEL CURSO ENARM CMN SIGLO XXI


endoscpica diagnostic un tumor maligno indiferenciado. El
examen fsico y los exmenes de laboratorio fueron normales.

c.- >90%
d.- <5%

PREGUNTA
Cual es el tipo mas probable de neoplasia que presente el caso?
RESPUESTA
a.- Leiomioma.
b.- Escamoso mucoide.
c.- Adenocarcinoma.
d.- Melanoma.

CASO CLINICO
Masculino de 56 aos, fumador, diagnosticado con acalasia hace 6
aos, sin tratamiento por decisin del paciente, refiere que hace 3
meses presenta disfagia a liquidos y solidos adems perdida de peso,
se realiza endoscopia donde se apresia abundante liquido, con
paredes dilatadas, mucosa con aspecto desquebrajado, friable. La
biopsia reporta positivo para neoplasia.

CASO CLINICO
Hombre 58 aos con disfagia. Comienza hace 2 meses con disfagia
progresiva sin remisiones, primero para slidos y luego para
semislidos. Actualmente ingiere lquidos y papillas blandas.
Sensacin de detencin del bolo alimenticio en regin esternal baja.
Desde hace meses sialorrea sobretodo nocturna. En algunas
oportunidades ha tenido regurgitaciones. No present dolor torcico
de ningn tipo. Adelgazamiento de 10kg en los ltimos meses. No
astenia, ni anorexia. AP: Fumador intenso de 2 cajillas/da.
Bronqutico crnico, etilista y enolista moderado. Se aporta una
fibrogastroscopia que informa lesin vegetante e infiltrante, en el
esfago, cuya biposia inform carcinoma epidermoide.

PREGUNTA
Cual de las siguientes tipos de neoplasia es mas frecuente
observar?
RESPUESTA
a.- Leiomioma.
b.- Tumor estroma gastrointestinal.
c.- Carcinomas de clulas escamosas
d.- Carcinomas de clulas pequeas

PREGUNTA
Radiologa del trax frente y perfil, valorando elementos de
remodelacin torcica (horizontalizacin costal, aplanamiento
diafragmtico) y rarefaccin pulmonar, a su vez descartar la
presencia de metstasis pulmonares y/u seas poco frecuentes
siendo este estudio poco sensible para su deteccin. Cual es el
tratamiento mas adecuado para el caso?
RESPUESTA
a.- Quimioterapia.
b.- Radioterapia.
c.- Reseccion quirrgica.
d.- Protesis expansible
CASO CLINICO
Paciente masculino, raza negra, de 59 aos de edad, de oficio
carpintero, diabtico, alcohlico y fuma 2 cajetillas de cigarros al da,
refiere disfagia de 6 meses de evolucin y prdida de 12 kg de peso,
dolor torcico y caquexia. Tiene antecedente de estenosis esofgica
y esofagitis, diagnosticada por endoscopia hace 4 aos. Actualmente
con diseminacin a hueso y pulmn.
PREGUNTA
Cul es la etiologa ms probable considerando los antecedentes y
sntomas de este paciente?
REPUESTA
a.- Carcinoma epidermoide de esfago
b.- Adenocarcinoma de esfago
c.- Leiomioma
d.- Tumor de Abrikosov
PREGUNTA
La superviviencia a 5 aos para este paciente si se encuentra en
estadio IV es de?
RESPUESTA
a.- 40-80%
b.- 15-25%

CURSO ENARM CMN SIGLO XXI TEL: 36246001

CASO CLINICO
Masculino de 66 aos de edad acude con su medico de atencion
primaria por dificultad para deglutir. Informa que hace alrededor de
8 meses empez a comer exclusivamente alimentos blandos debido
a que la sensacion de los alimentos solidos se quedaban pegados
detrs del pecho. Informa cierta anorexia y perdida involuntaria de
11Kgs de peso durante los meses pasados. Reconoce que a fumado
una cajetilla de cigarros al dia durante los ltimos 35 aos y que ha
bebido un aproximado de 2 cervezas diario desde los 18 aos de
edad. EF: revela T 37C, pulso 86/min, TA 136/86mmHg. Tiene
aspecto caquctico.
PREGUNTA
Cul es el diagnotico mas probable para este caso?
RESPUESTA
a.- Acalacia
b.- Esofago de Barret
c.- Cancer de e425sfago
d.- Cancer gastrico
PREGUNTA
Cul de los siguientes no es un dato de extensin de la
enfermedad?
RESPUESTA
a.- Tos
b.- Neumonia por 425spiracin
c.- Cambios en la voz
d.- Reflujo gastroesofagico
PREGUNTA
Cul es el pronostico para este paciente a cinco aos?
RESPUESTA
a.- Menos de 5%
b.- 15%
c.- 30%
d.- Mas de 40%

Pharmed Solutions Institute

PGINA 425

MANUAL DE TRABAJO DEL CURSO ENARM CMN SIGLO XXI


VARICES ESOFAGICAS
CIENCIAS BASICAS: Son vasos colaterales porto sistmicos, es decir, canales vasculares que unen la circulacin venosa porta y la sistmica. Se
forman como consecuencia de la hipertensin portal (una complicacin progresiva de la cirrosis), preferencialmente en la submucosa del esfago
inferior. La ruptura y el sangrado de las vrices esofgicas son complicaciones mayores de la hipertensin portal (HTP) y se acompaan de una alta
tasa de mortalidad. SALUD PUBLICA: El sangrado varicoso representa 1030% de todos los casos de sangrado gastrointestinal alto. Es una
complicacin frecuente de hipertensin portal 30-50%. Elevada mortalidad hospitalaria 20-50%. Riesgo elevado de resangrado, 30% en primera
semana y 50% durante el primer ao. 15-20% de pacientes cirrticos que sufren una hemorragia por varices esofagogastricas fallecen en las 6 sem
siguientes. PATOGENIA: La cirrosis, el estadio terminal de la hepatopata crnica, es la causa ms comn de HTP. La presin venosa portal (P) es el
producto de la resistencia vascular (R) y el flujo sanguneo (Q) en el lecho de la vena porta (Las varices esofgicas se rompen cuando la tensin
ejercida sobre su Las VE se rompen cuando la tensin ejercida sobre su pared es excesiva y sobrepasa un valor umbral pared es excesiva y sobrepasa
un valor umbral). En la cirrosis hay un aumento tanto de la resistencia vascular intraheptica como del flujo porta. La HTP lleva a la formacin de
colaterales porto-sistmicas. Sin embargo, debido a su mayor resistencia y
mayor flujo aferente venoso porta, estas colaterales no logran disminuir la
hipertensin. La mejor manera de evaluar la hipertensin portal
(indirectamente) es utilizando la medicin de la presin venosa heptica
enclavada (PVHC). Para que se formen las vrices se necesita una diferencia de
presin entre la circulacin portal y sistmica (gradiente de la presin venosa,
GPVH) de 1012 mmHg (aunque no es suficiente). La GPVH normal es 35
mmHg. Cuando la HTP es clnicamente significativa, hay formacin de varices
esofgicas con una presin >10mmHg, y hemorragia de varices esofgicas
>12mmHg. Correlacin entre la presencia de vrices y severidad de la
hepatopata: Pacientes ChildPugh A: 40% tienen vrices. Pacientes ChildPugh
C: 85% tienen vrices. Algunos pacientes pueden presentar vrices y hemorragia
en las primeras etapas de la enfermedad, an en ausencia de cirrosis. Los
pacientes con hepatitis C y fibrosis en puente: 16% tienen vrices esofgicas. La
presencia de vrices gastroesofgicas se correlaciona con la gravedad de la
enfermedad heptica. La severidad de la cirrosis puede clasificarse utilizando el sistema de clasificacin de ChildPugh. DIAGNOSTICO: La
esofagogastroduodenoscopa es el patrn oro para el diagnstico de vrices esofgicas. Estndar de oro para diagnstico: Vrices esofgicas
Soehendra: GRADO I: Replecin leve, dimetro < 2 mm, apenas se eleva del esfago relajado, se acentan en posicin cabeza abajo. GRADO II:
Replecin moderada, curso serpenteante, dimetro de 3-4 mm, restringidas a la mitad inferior del esfago. GRADO III: Replecin completa, tensa,
con dimetro > 4 mm, paredes finas, fenmeno de vrice sobre vrice, pasan al fondo gstrico. GRADO IV: Replecin completa, tensa, ocupan todo
el esfago, a menudo combinadas con vrices gstricas o duodenales. Si no se dispone de patrn oro, otros posibles pasos diagnsticos seran la
ecografa con Doppler de la circulacin sangunea (no la ecografa endoscpica). Si bien es una mala segunda opcin, ciertamente puede demostrar
la presencia de vrices. Entre otras alternativa, se puede utilizar la radiografa baritada (con ingestin de bario) del esfago y estmago, y angiografa
y manometra de la vena porta. Es importante valorar la ubicacin (esfago u estmago) y el tamao de las vrices, los signos de sangrado
inminente, que puede ser un primer sangrado agudo, o recurrente, y (si corresponde) debe considerarse la causa y la gravedad de la hepatopata.
TRATAMIENTO: Sangrado por varices esofgicas: Actuaciones iniciales, catteres venosos: 1-2 vas perifricas gruesas. Reserva de concentrados de
hemates. Analtica con pruebas de coagulacin. Reposicin de la volemia. En hemorragia grave, va central y sonda vesical. Reposicin cuidadosa de
la volemia (Hcto 21-27%). Prevencin de las infecciones: Norfloxacina VO 400mg cada 12hs (7 das). Ceftriaxona IV 1g da (7 das), principalmente en
pacientes Child C. Terapia farmacolgica: Vasoconstrictores esplcnicos; vasopresina, somatostatina, -bloqueantes no cardioselectivos (propanolo,
nanodol). Terlipresina Primera opcin; derivado sinttico de la Vasopresina pero con menos efectos secundarios, administracin en bolos, nico
frmaco que ha demostrado disminucin de la mortalidad. Somatostatina, segunda opcin es eficaz para detener la hemorragia (por lo menos
transitoriamente) en hasta 80% de los pacientes. La somatostatina puede ser superior a su anlogo octreotida. A pesar de emplearse en dosis
adecuadas, alrededor de 30% de los pacientes no responden a -bloqueantes con una reduccin en el gradiente de presin venosa heptica (GPVH).
Adems, los -bloqueantes pueden provocar efectos colaterales tales como fatiga e impotencia, que podran disminuir la adherencia al tratamiento
(especialmente con los varones jvenes), o pueden estar contraindicados los -bloqueantes por otras razones. Venodilatadores: Nitratos solos no
estn recomendados. El 5-MNI reduce la presin porta, pero su uso en los pacientes cirrticos est limitado por sus efectos vasodilatadores. La
terapia de combinacin lleva a un efecto sinrgico en reducir la presin porta. Se ha demostrado que la combinacin de 5-MNI con -bloqueantes no
selectivos tiene efectos aditivos al reducir la presin porta y es particularmente eficaz en pacientes que no responden a la terapia inicial con bloqueante solo. Sin embargo, estos efectos beneficiosos pueden verse contrarrestados por los posibles efectos deletreos sobre la funcin renal y
la mortalidad a largo plazo, especialmente en los pacientes mayores de 50 aos. Por lo tanto no se recomienda el uso de rutina del tratamiento
combinado. Terapias locales: Escleroterapia o ligadura endoscpica de las vrices (EVL). No tiene efecto sobre el flujo o la resistencia porta. Terapia
de derivacin. Quirrgica o radiolgica (derivacin portosistmica intraheptica transyugular, TIPS). Reduce la presin porta. La escleroterapia
endoscpica y la ligadura varicosa logran interrumpir el sangrado en hasta 90% de los pacientes. La ligadura endoscpica con banda es una
escleroterapia eficaz, pero se acompaa de menos efectos colaterales. Sin embargo esta tcnica puede ser ms difcil de aplicar que la escleroterapia
en los pacientes con sangrado activo severo. La derivacin TIPS es una buena alternativa cuando fracasan el tratamiento endoscpico y la
farmacoterapia. El uso de taponamiento con baln est disminuyendo ya que existe un alto riesgo de resangrado despus de desinflar el baln y hay
un riesgo de complicaciones mayores. Sin embargo, el taponamiento con baln logra detener la hemorragia por lo menos temporalmente en la
mayora de los casos, y puede ser utilizado en regiones del mundo donde no se disponga fcilmente de EGD y TIPS. Puede ayudar a estabilizar al
paciente para ganar tiempo y acceder a EGD y/o TIPS ms adelante. La hemorragia aguda de las vrices a menudo se acompaa de infeccin
bacteriana debido a translocacin intestinal y trastornos de la motilidad. La antibiticoterapia profilctica ha demostrado aumentar la tasa de
sobrevida.
CASO CLINICO
Masculino de 43 aos de edad que consulta por distencin
abdominal y episodios de sincope. El paciente tiene antecedentes de
cirrosis heptica de causa alcohlica (estadio C de Child-Pugh),
hipertensin portal con multiples episodios de sangrado por varices

CURSO ENARM CMN SIGLO XXI TEL: 36246001

esofgicas y sndrome asctico-edematoso. El paciente continuaba


con el consumo de alcohol sin cumplir la medicacin de
espironolactona y propanolol. Al Ingreso se encontraba vigil y
orientado con TA de 85/50 mmHg y FC 100 lpm. FR 18.

Pharmed Solutions Institute

PGINA 426

MANUAL DE TRABAJO DEL CURSO ENARM CMN SIGLO XXI


PREGUNTA
Cual es la conducta a seguir en este momento?
RESPUESTA
a.- Reposicin de liquidos y hemoderivados.
b.- Realizar TAC para buscar sangrado.
c.- Conducta expectante y medidas antiamonio.
d.- Preparar para laparatomia exploratoria.

Acude a urgencias paciente de 61 aos de edad la cual refiere dolor


ardoroso en la regin media del trax de predominio anterior, el
dolor ha incrementado en las ltimas semanas, refiere vomito en
tres ocasiones con trazas de sangre. Tiene antecedentes de diabetes
y osteoartritis bajo tratamiento, en ltimas fechas han incrementado
el dolor de sus manos por lo que aumento su dosis de medicamente.

PREGUNTA
Se realizan estudios y medidas de rutina para la patologa de base,
asi como bsqueda de hemorragia, por TAC se observa sangrado
peritoneal, y se prepara al paciente para ciruga. cual de las
siguientes causas de hemoperitoneo es mas frecuente?
RESPUESTA
a.- Rotura de vrices esofgicas.
b.- Rotura de canales linfticos
c.- Carcinoma hepatocelular
d.- Metstasis hepticas
PREGUNTA
Cual es la mortalidad del paciente en caso de recurrir a TIPS
considerando particularmente su estadio C de Child-Pugh?
RESPUESTA
a.- 100 %.
b.- 90 %.
c.- 80 %.
d.- 70 %.
PREGUNTA
Cual de la siguientes medidas es mas apropiada para reducir el
riesgo de sangrado por varices esofgicas en el caso presentado?
RESPUESTA
a.- Espironolactona.
b.- Propanolol.
c.- Propanolol e isosorbide.
d.- Espironolactona y propanolol.
CASO CLINICO
Paciente varn de 58 aos de edad fumador de 40 cigarrillos al da
desde los 15 aos, bebedor de unos 70 gramos etanol/ da. Presenta
de forma sbita 3 vmitos muy abundantes de color rojo oscuro con
restos semejantes a posos de caf. Exploracin fsica: TA: 86/60 y
frecuencia cardiaca 140 lpm. Consciente, orientado con intensa
palidez cutneo mucosa, sudoracin y frialdad cutnea. Eupneico.
Desnutrido. Telangiectasias malares e hipertrofia parotdea. AP: mvc,
AC ritmico a 140 lpm sin soplos ni extratonos. Abdomen: RHA
conservados no se palpan masas hepatoesplenomegalia de tres
traveses de dedo, circulacin colateral periumbilical, no signos de
irritacin peritoneal. Extremidades: pulsos perifricos presentes,
simtricos y dbiles, eritema palmar. Neurolgico normal sin
flapping ni signos de encefalopata. Hemograma: hematocrito 28%,
hemoglobina 10.3 g/dl, VCM 104 fl, leucocitos 10.3 mil/mcL,
plaquetas 100 mil/mcL. Perfil heptico: GPT 60 UI/L, GOT 93 UI/L,
fosfatasa alcalina 183 UI/L, gamma GT 200 UI/L. Estudio de
coagulacin: normal. Endoscopia digestiva: varices esofgicas con
signos de sangrado reciente, resto hemticos en estmago con
mucosa gstrica normal, duodeno hasta tercera porcin normal.
PREGUNTA
Cual es la conducta a seguir mas adecuada?
RESPUESTA
a.- Realizar escleroterapia.
b.- Restablecimiento de liquidos.
c.- Aminas vasoactivas.
d.- Colocacion de sonda de balones.
CASO CLINICO VARICES ESOFAGICAS.

CURSO ENARM CMN SIGLO XXI TEL: 36246001

PREGUNTA
Cul es el manejo farmacolgico ms adecuado?
RESPUESTA
a.- Ranitidina.
b.- Omeprazol.
c.- Sales de Bismuto.
d.- Hidroxido de aluminio y magnesio.
CASO CLINICO VARICES ESOFAGICAS
Se trata de paciente masculino de 45 aos de edad, alcoholismo
desde hace 20 aos el cual refiere vomito con sangre en 3 ocasiones
posterior a ingestin de alcohol durante 5 das, a la exploracin fsica
se observa diafortico, ansioso y desorientado, durante su estancia
presenta dos ocasiones ms vomito con trazas de sangre, sus
constantes vitales son TA 110/70 mmHg, FC 87, FR 19. 150,000
Plaquetas
PREGUNTA
Cul es la conducta ms adecuada para controlar el sangrado en
este caso?
RESPUESTA
a.- Colocacin de sonda de balones.
b.- Escleroterapia.
c.- Vitamina K.
d.- Inhibidores de Bomba H+
CASO CLINICO
Paciente varn de 63 aos con antecedentes de cirrosis alcohlica e
internacin en dos ocasiones por hemorragia variceal. Consulta por
vmito y tos con sangre, mareo, pirosis y disfagia as como
palpitaciones. Presenta ascitis leve, FC 110x, T/A 90/60. Laboratorio:
anemia, plaquetopenia, prolongacin de los tiempos de coagulacin,
Bilirrubina 2.5mg/dl, albumina 29 g/l, glucosa 145mg/dl.
PREGUNTA
En qu clase de la clasificacin de Child-Pugh se encuentra el
paciente?
RESPUESTA
a.- Clase A
b.- Clase B
c.- Clase C
d.- Clase D
PREGUNTA
Cul es la conducta teraputica ms adecuada para prevenirle
infecciones a este paciente?
RESPUESTA
a.- Norfloxacino c/12 hrs por 7 das
b.- Ceftriaxona 1 g c/24hrs por 7 das
c.- Penicilina 1,200, 000U c/24hrs por 3 das
d.- Azitromicina 500mgs c/24 hrs por 3 das
PREGUNTA
Despus de estabilizar al paciente e iniciar terlipresina Cul es la
conducta teraputica ms adecuada para prevenir la recidiva de
hemorragia?
REPUESTA
a.- Escleroterapia + beta bloqueadores
b.- Derivacin portosistmica de Warren
c.- Ligadura + beta bloqueadores
d.- Escleroterapia + 5 mononitrato de isosorbide

Pharmed Solutions Institute

PGINA 427

MANUAL DE TRABAJO DEL CURSO ENARM CMN SIGLO XXI


RUPTURA ESOFAGICA
CIENCIAS BASICAS: Patologa relativamente rara/grave, que sin tratamiento puede progresar a mediastinitis y shock sptico. Causas; traumtica,
iatrogenia, cuerpos extraos, agentes corrosivos, trauma torcico, espontanea, Sndrome de Mallory-Weiss, hematoma intramural, Sndrome de
Boerhaave, neoplasias, infecciosa. DIAGNOSTICO: Representan un difcil reto diagnstico, originando frecuentes retrasos en su tratamiento.
Manifestaciones clnicas clsicas son: a) Vmitos. b) Dolor retroesternal. c) Enfisema mediastnico (Trada de Macler). Signos clnicos sutiles e
inespecficos; hipotensin, sepsis, fiebre. SINDROME DE MALLORY-WEISS; Este sndrome se caracteriza por hemorragia digestiva superior despus
del vomito repetido o esfuerzos fuertes y prolongados para vomitar o toser. Aproximadamente 4 casos por cada 100.000 personas. Ms a los
hombres que a las mujeres y puede aparecer a cualquier edad. 5% de las hemorragias de la parte alta del tracto gastrointestinal. Suele ocurrir en
varones con historia de consumo de alcohol y se presenta clnicamente como vmitos repetidos seguidos de hematemesis. Desrdenes alimenticios
y en algunas evidencias se demuestran la presencia de hernia de hiato como una condicin pre-disponente. Puede estar asociado a la ingesta
repentina de salicilatos. El mecanismo es similar a la perforacin esofgica espontanea, o sea el aumento sbito en la presin intrabdomional contra
la glotis cerrada en un paciente con hernia hiatal. Los desgarros de Mallory-Weiss se caracterizan por hemorragia arterial que puede ser masiva.
Puede aparecer con cierta frecuencia en el punto de unin entre el esfago y el estmago. Para establecer el diagnstico es preciso mantener un
alto ndice de sospecha en un paciente que presenta hemorragia digestiva alta despus de vmito y arqueo prolongado. La endoscopia confirma el
diagnostico al identificar una o ms fisuras longitudinales en la mucosa del estmago herniado. En casi todos los pacientes la hemorragia cede en
forma espontnea (24-48hrs) con tratamiento mdico. Cicatrizacin en aproximadamente 10 das. Solo en ocasiones se requiere la operacin para
detener la prdida de sangre. El procedimiento consiste en laparotoma y gastrostoma alta con sutura del desgarro lineal. SNDROME DE
BOERHAAVE: Consiste en una rotura transmural completa del esfago, generalmente en el lado izquierdo a unos 2-3 cms por encima de la unin
gastroesofgica, con salida de la sangre hacia el mediastino. Afecta con ms frecuencia a hombres (60-78%) y se relaciona con vmitos violentos y
consumo de alcohol. Clnicamente se presenta con un cuadro dramtico de dolor, vmitos, neumotrax, enfisema subcutneo y shock. No suele
haber hematemesis (a diferencia del Mallory-Weiss). Un signo radiolgico clsico es la V de Naclerio. Enfisema mediastnico lineal en ngulo
costovertebral izq (entre aorta inferior y diafragma). A menudo existe derrame pleural de predominio izquierdo.
CASO CLINICO RUPTURA DE ESOFAGO.
Se trata de masculino de 34 aos de edad el cual se encuentra en
urgencias trado por ambulancia ya que en su trabajo presento
vomito con sangre fresca y abundante en ms de 4 ocasiones, a la
exploracin fsica se encuentra diafortico, plido, con pulsos
disminuidos, taquicardico e hipotenso, se estabiliza, su hematocrito
es de 41 % y plaquetas de 45,000.
PREGUNTA
Cul es el tratamiento adecuado para mejorar el pronstico del
paciente?
RESPUESTA
a.- Endoscopia y escleroterapia.
b.- Sonda de balones.
c.- Transfusin sangunea.
d.- Desmopresina y Inhibidor de H+.
CASO CLINICO RUPTURA DE MUCOSA DE ESOFAGO.
Se encuentra en urgencias por sangrado de tubo digestivo alto,
refiere familiar que encontr al paciente inconsciente en el bao,
refiere que tambin presento dos evacuaciones ftidas abundantes
mas vomito con sangre fresca, agrega que es alcohlico desde hace
25 aos, no ha recibido tratamiento previo hasta la fecha.
PREGUNTA
Cul es la complicacin asociada que presenta peor pronstico?
RESPUESTA
a.- Trastorno renal asociado.
b.- Trastorno plaquetario secundario.
c.- Hipertensin portal.
d.- Estado de choque.
CASO CLINICO
Paciente femenino de 23 aos de edad, con antecedentes de
Hipotiroidismo en tratamiento con levotiroxina, refiere cefaleas
frecuentes que mejoraban con la toma de aspirina. Amenorrea de 8

CURSO ENARM CMN SIGLO XXI TEL: 36246001

sem aproximadamente. No precisa la fecha de su ltima


menstruacin. Lleva una semana con vmitos frecuentes
pospandriales precoces, alrededor de 8 a 10 por da. Un da antes de
acudir a consulta y el da de la misma tuvo vomito con sangre roja
como la de una herida, en cantidad moderada, por lo que es
remitida a un hospital. FR 20. FC 90 TA 110/60. Abdomen, ligero
dolor a la palpacin profunda en epigastrio. Laboratorios Hto de
45%.
PREGUNTA
Cul es la conducta diagnostica ms adecuada en esta paciente?
RESPUESTA
a.- Tomografa
b.- Radiografa
c.- Endoscopa
d.- Ultrasonido
PREGUNTA
Cul es la conducta teraputica ms adecuada para esta paciente?
RESPUESTA
a.- Ranitidina
b.- Omeprazol
c.- Gel de aluminio y magnesio
d.- Sucralfato
PREGUNTA
De persistir la hemorragia a pesar de tratamiento mdico y con la
paciente estable. Cul sera la conducta teraputica ms adecuada?
RESPUESTA
a.- Sonda de balones
b.- Escleroterapia endoscpica
c.- Transfusin sangunea
d.- Ciruga abierta

Pharmed Solutions Institute

PGINA 428

MANUAL DE TRABAJO DEL CURSO ENARM CMN SIGLO XXI


ENFERMEDAD ACIDO-PEPTICA
CIENCIAS BASICAS: Es una enfermedad inflamatoria aguda o crnica de la mucosa gstrica producida por factores exgenos o endgenos que
produce sntomas disppticos atribuibles a la enfermedad y cuya existencia se sospecha clnicamente, se observa endoscpicamente y se requiere
confirmacin histolgica. La gastritis es etiolgicamente multifactorial, de los que el ms comn es la infeccin por Helicobacter Pylori. Factores
exgenos; Helicobacter pylori, AINES, imitantes gstricos, drogas, alcohol, tabaco. Factores endgenos; acido gstrico, pepsina, bilis, jugo
pancretico, uremia, inmunes SALUD PUBLICA: Es una entidad de elevada morbilidad a nivel mundial. PATOGENIA: El dao de la mucosa gstrica
depende del tiempo de permanencia del factor o factores lesionates, jugando un rol importante la capacidad que tiene la mucosa gstrica a travs
de la denominada barrear gstrica para resistir a estos factores o a los efectos deletreos de sus propias secreciones. La barrea gstrica est
constituida por componentes pre epiteliales (barrera de moco, bicarbonato, fosfolpidos), epiteliales (capacidad de restitucin de epitelio, gradiente
elctrico que previenen la acidificacin celular, los transportadores acidobsicos que transportan el bicarbonato hacia el moco, prostaglandinas, y
oxido ntrico) y sub epiteliales (flujo sanguneo que descarga nutrientes y bicarbonato en el epitelio y adherencia y extravasacin de los leucocitos,
que inducen lesin histica y quedan suprimidos por la prostaglandinas endgenas). El trastorno de uno u ms de los componentes defensivos,
originan lesin de la mucosa permitiendo la accin del cido, proteasas y cidos biliares en mayor o menor grado y que pueden llegar hasta la
lmina propia, sitio en el que producen lesin vascular, estimulan las terminaciones nerviosas y activan la descarga de histamina y otros
mediadores. Las causas ms comunes de lesin son por H. pylori, lesiones por estrs y AINES. DIAGNOSTICO: Clnicas; Las gastritis pueden ser
totalmente asintomticas y en caso de existir sntomas estos no son propios, sino atribuibles a ella, como es la presencia de ardor, o molestias
postprandiales en epigastrio, llenura precoz, RGE, nausea, distensin abdominal, pirosis, sntomas que tambin pueden estar presentes en dispepsia
no ulcerosa, lceras o neoplasias gstricas o duodenales y an en el colon irritable. Adems pueden manifestarse con hemorragias crnicas o agudas
que podran llegar a ser masivas con hematemesis y melena. Hallazgos endoscpicos: Los signos endoscpicos asociados a esta entidad incluyen
edema, eritema, mucosa hemorrgica, punteados hemorrgicos, friabilidad, exudados, erosiones, nodularidad, pliegues hiperplsicos, presencia de
signos de atrofia de la mucosa dada por visualizacin de vasos submucosos con aplanamiento o prdida de los pliegues acompaados o no de placas
blanquecinas que corresponden a reas de metaplasma intestinal. Estos signos endoscpicos pueden localizarse topogrficamente a nivel del antro,
cuerpo o en todo el estmago, denominndose gastritis antrales, gastritis corporal o pangastritis respectivamente. Hallazgos histolgicos: No se
debe abusar del diagnstico de gastritis, por lo que se requiere realizar la biopsia para confirmacin histolgica, establecer la presencia o ausencia
de Helicobacter pylori o de otras formas de gastritis especficas. Exmenes de laboratorio: Las pruebas de laboratorio pueden usarse para
determinar algunas causas de gastritis, como en el caso del Helicobacter pylori a travs de mtodos invasivos como la endoscopa y biopsias para el
estudio histolgico, realizar la tcnica de la ureasa rpida, el cultivo y o el empleo de mtodos no invasivos como la serolgica para Ig G, la deteccin
de antgeno en las deposicin, y la prueba del aliento del C13 o C14 espirado con sensibilidades / especificidades de o ms de 90/90 % a excepcin
de la serolgica 80/90% y el cultivo 50/100%. TRATAMIENTO: Dieta sin sustancias irritantes (caf, tabaco, alcohol, aj ) as como tambin drogas que
contrarresten la agresin de la barrera gstrica indicando ya sea anticidos orales, citoprotectores de la mucosa gstrica (sucralfato, bismuto,
misoprostol), antagonistas de receptores H2, Inhibidores de la bomba de protones, a los que se puede aadir gastrocinticos (metoclopramida,
domperidona, cisaprida, mosaprida, cinitaprida) si existe evidencias de trastornos de motilidad gastroesofgica o gastroduodenal. Los inhibidores de
bomba de protones son ms efectivos que los bloqueadores H2 en el manejo de gastritis IHQ abierta totalmente el manejo debe de ser hospitalario.
Omeprazol o pantoprazol 80 mg intravenosos en bolo seguidos de infusin continua de 8 mg/h por 72 horas). El mantener el PH por arriba de 4
ayuda a restituir la mucosa gstrica. El omeprazol es el frmaco de primera eleccin para el tratamiento de la Gastritis Aguda. Los esquemas de
primera lnea pueden ser triples o cudruples (cuando se agrega una sal de bismuto). El basado en las tasas de xito alcanzadas en prcticamente
todas las regiones del mundo se recomienda el uso de 1 g de amoxicilina dos veces al da, 500 mg de claritromicina dos veces al da y dosis doble de
un IBP durante 14 das. El esquema de segunda lnea recomendado para la erradicacin de Helicobacter pylori es levofloxacino 500 mgs cada 24 hrs,
amoxicilina 1 gr cada 12 hrs, inhibidor de bomba de protones dosis estndar cada 12 hrs por 10 a 14 das o moxifloxacino 400mg al da, amoxacilina
1 gr cada 12 hrs e inhibidor de bomba de protones cada 12 hrs. En el caso de alergia a la amoxicilina, se puede emplear como alternativa tetraciclina
(500 mg cuatro veces/da) o metronidazol (250 mg cuatro veces/da). Las sales de bismuto, en forma de subsalicilato o subcitrato, deben
administrarse en cuatro tomas al da y en dosis promedio de 525 mg. Para erradicar Helicobacter pylori hay que confirmar la erradicacin de la
bacteria. H.pylori: La infeccin por Helicobacter pylori se calcula que afecta la mitad de la poblacin mundial. Todos los sujetos que presentan
Helicobacter Pylori desarrollan gastritis, un 15 a 20% .
CASO CLINICO GASTRITIS
Se trata de paciente femenino de 21 aos de edad la cual acude a
servicio mdico de la escuela refiriendo presentar nauseas con
vomito con mnimo contenido gstrico, niega sangrados, agrega
gases y eructos ocasionales, se alimenta dos veces al da y come lo
que puede al salir de la escuela rumbo a su trabajo.

Cul es el mtodo diagnostico ms adecuado?


RESPUESTA
a.- Serie gastroesofagica.
b.- Endoscopia.
c.- La clnica es suficiente.
d.- Biometria hemtica.

PREGUNTA
Cul de las siguientes medidas mejora el pronstico?
RESPUESTA
a.- Tratamiento para H. pilory.
b.- Cambio de hbitos dietticos.
c.- Endoscopia con biopsia.
d.- Doble esquema anticido.

CASO CLINICO GASTRITIS CON H. PILORY


Se trata de masculino de 31 aos de edad el cual acude a consulta
por cefalea tensional, al interrogatorio refiere que presenta gastritis
recurrente y ha sido sometido a diversos tratamientos
farmacolgicos para este problema pero han sido incompletos, fue
sometido a endoscopia donde se diagnostico gastritis crnica con
presencia de H. pilory, existen antecedente en la familia de cncer
gstrico por lo que el paciente se encuentra ms preocupado.
PREGUNTA
Considerando el cuadro clnico y el diagnostico endoscpico. cual es
la conducta farmacolgica ms adecuada?
RESPUESTA
a.- Metronidazol, claritromicina y omeprazol.
b.- Claritromicina, amoxicilina y omeprazol.
c.- Metronidazol, sales de bismuto y ranitidina.
d.- Amoxicilina, metronidazol y pantoprazol.

CASO CLINICO GASTRITIS CRONICA


Se ingresa a paciente femenino de 51 aos de edad la cual acude a
urgencias por presencia de dolor epigstrico, vomito y nauseas,
adems agrega disminucin del apetito, agrega que desde hace 3
meses ha recibido tratamiento por anemia previamente diagnostica
para lo cual indicaron hierro.
PREGUNTA

CURSO ENARM CMN SIGLO XXI TEL: 36246001

Pharmed Solutions Institute

PGINA 429

MANUAL DE TRABAJO DEL CURSO ENARM CMN SIGLO XXI


ULCERA GASTRICA Y DUODENAL
CIENCIAS BASICAS: La ulceras duodenales casi siempre estn a 1-2cm del ploro. A menudo se acompaan de hipersecrecin de cido. Las ulceras
gstricas se dividen en varios tipos: Tipo I; la ms frecuente, en la parte proximal del antro o del cuerpo. Tipo II; secundaria a ulcera duodenal con
estenosis pilrica. Tipo III; ulcera prepilrica o del canal pre pilrico, sus causas son similares a las de la ulcera duodenal. SALUD PUBLICA: La mayor
incidencia de la ulcera gstrica es entre los 50-65 aos de edad. La mayor parte de casos de ulcera duodenal se presenta en la cuarta dcada de la
vida. PATOGENIA: 3 causas bsicas: hipersecrecin de cido, colonizacin por H. pylori y AINES. H.pylori se encuentra en 95% de los casos de
enfermedad ulcerosa duodenal y en 80% de enfermedad por ulcera gstrica. Sin embargo resulta difcil probar la relacin causa-efecto. Los AINES
suprimen la sntesis de prostaglandinas y debilitan la barrear mucosa. Entre 10-30% de los consumidores crnicos de estos frmacos presenta ulcera
pptica. Patogenia de ulcera duodenal: En general los pacientes secretan ms acido por aumento en la cantidad de clulas parietales, principales o
factores trficos (gastrina). H. pylori incrementa la liberacin de gastrina y produce duodenitis. Algunos pacientes tambin tienen trastornos de la
motilidad con vaciamiento gstrico rpido de lquidos. Lo que expone al duodeno a una cantidad de cido mayor de la normal. La anormalidad
fisiolgica ms frecuente es la disminucin de la secrecin de bicarbonato. Patogenia de ulcera gstrica: El defecto bsico generalizado radica en la
defensa de la mucosa gstrica contra el cido y la pepsina. El gasto acido es normal o apenas detectable, en contraste con la ulcera duodenal. La
anormalidad bsica suele ser el reflujo del contenido duodenal. Es posible que exista disfuncin pilrica, tambin secundaria al tabaquismo. El
reflujo de cidos biliares, lisolecitina y secreciones pancreticas ejerce un efecto nocivo y daan la mucosa. El ASA tiene un efecto similar.
DIAGNOSTICO: Clnica; dolor es lo ms frecuente, es epigstrico agudo o clico. El dolor de la ulcera duodenal se presenta muchas horas despus de
una comida, cuando el bulbo duodenal esta vaco; se alivia con alimentos y sustancias alcalinas. Por el contrario el dolor de la ulcera gstrica, se
intensifica con la ingesta de alimentos. El dolor suele ser crnico y recurrente. Otras manifestaciones son nausea, prdida de peso y ligera
sensibilidad. Por lo general el diagnstico se establece por endoscopia de tubo digestivo superior o una serie esofagogastroduodenal. La endoscopia
es necesaria en todos los casos de ulcera gstrica, debido al riesgo de cncer, aqu se obtienen el material para la deteccin de H. pylori.
TRATAMIENTO: En pacientes jvenes se basa solo en los sntomas, sin estudios. En ancianos es necesario realizar endoscopia por riesgo de tumor
maligno. Con una serie radiolgica de tubo digestivo superior de doble contraste se detecta 90% de las ulceras gstricas y duodenales. Medico; 1)
Neutralizar el cido gstrico. 2) Inhibir la secrecin. 3) Proteger la mucosa gstrica de la lesin. Los anticidos neutralizan el cido del estmago. Los
antagonistas de los recetores H2 bloquean dichos receptores en las clulas parietales. Los frmacos ms potentes son los bencimidazoles sustituidos
que bloquean la H+/K+ ATPasa (IBP) Y que funcionan incluso en pacientes con gastrinoma. La mucosa gstrica puede protegerse con anlogos de las
prostaglandinas E2 (misoprostol) que incrementa el flujo sanguneo de la mucosa y la produccin de bicarbonato y moco. El sucralfato se une a las
protenas de los crteres ulcerosos, por lo que favorece la cicatrizacin. El bismuto cubre la protena expuesta y tiene actividad contra H. pylori.
Quirrgico: Ulcera duodenal; vagotoma ultra selectiva COMPLICACIONES: Perforacin (5-10%), Hemorragia y obstruccin (<5%). Entre 15-20% de
las ulceras ppticas producen hemorragia macroscpica (gstrica o duodenal). Los pacientes refieren melena o hematemesis (gstrica). SINDROME
DE ZOLLINGER-ELLISON: Se identifica en 0.1-1% de todos los pacientes con enfermedad ulcerosa pptica y cerca de 20% de estos padece tambin
sndrome de neoplasia endocrina mltiple tipo I. Los sntomas incluyen hipergastrinemia y ulceracin pptica grave secundaria a un gastrinoma. Es
probable que las ulceras sean mltiples y ms distales. El gasto basal de acudo es muy alto, el nivel de gastrina puede ser superior a 1000pg/ml. Es
posible inducir niveles equvocos de gastrina con la administracin IV de calcio o secretina. Los gastrinomas son neoplasias de enrome malignidad
(>90%), pero de crecimiento lento; 50% de los tumores tiene metstasis ganglionares para el momento del diagnstico.
CASO CLINICO ULCERA DUODENAL.
Acude paciente femenino de 41 aos de edad para revisin de
tratamiento previo por anemia perniciosa, los resultados de
laboratorio no fueron trados por la paciente, sin embargo la
endoscopia afirmo el diagnostico, refiere que ha presentado dolor
en el cuadrante superior derecho, se realizo un USG de vas biliares
con presencia de lito en vescula biliar y resto de vas normales.
PREGUNTA
Cul es el diagnostico diferencial para iniciar tratamiento correcto?
RESPUESTA
a.- Serie gastroesofagica.
b.- Frotis de sangre perifrica.
c.- Tomografa abdominal.
d.- Biopsia gstrica.
CASO CLINICO
Varn de 58 aos de edad, fumador de 20 cigarrillos al da, como
nico antecedente. Consult por presentar desde haca ms de 15
das dolor en epigastrio, que se calmaba con la ingesta,
reapareciendo dos horas despus, disfagia. En dos ocasiones el dolor
se haba presentado por la noche, despertndole. Refera episodios
similares previos (pero de menor intensidad) en los ltimos dos
aos, que el paciente relacionaba con transgresiones dietticas y
que le obligaban a tomar bicarbonato. Negaba ingesta de alcohol y
de frmacos gastrolesivos, en concreto AINE y AAS. A la exploracin
fsica, presentaba buen estado general, estaba bien nutrido; la
auscultacin cardiopulmonar y neurolgica bsica fueron normales.
A la palpacin, el abdomen era blando y depresible, ligeramente

CURSO ENARM CMN SIGLO XXI TEL: 36246001

doloroso en epigastrio, pero sin defensa y sin identificar masas ni


visceromegalias.
PREGUNTA
Cul es la conducta diagnostica ms adecuada?
RESPUESTA
a.- Pautar tratamiento para erradicar H. Pylori
b.- Pautar tratamiento con IBP y vigilar
c.- Test de aliento
d.- Endoscopia digestiva alta
PREGUNTA
Cul es el diagnstico clnico ms probable para este paciente?
RESPUESTA
a.- Gastritis
b.- Ulcera duodenal
c.- Ulcera gstrica
d.- Cncer de estomago
PREGUNTA
Cul es la complicacin ms frecuente?
RESPUESTA
a.- Perforacin
b.- Hemorragia
c.- Obstruccin
d.- Cncer

Pharmed Solutions Institute

PGINA 430

MANUAL DE TRABAJO DEL CURSO ENARM CMN SIGLO XXI


CANCER GASTRICO
CIENCIAS BASICAS: Es un tipo de crecimiento tisular maligno producido por la proliferacin contigua de clulas anormales con capacidad de invasin
y destruccin de otros tejidos y rganos, en particular el esfago y el intestino delgado. En las formas metastasicas las clulas tumorales pueden
infiltrar los vasos linfticos de los tejidos, diseminarse a los ganglios linfticos y, sobrepasando esta barrera, penetrar en la circulacin sangunea y
diseminarse a cualquier rgano del cuerpo. Factores de riesgo: Ambientales; exceso de consumo de sal, nitritos, nitrosaminas, aminas aromticas,
bajo consumo de frutas y verduras, infeccin por H. pylori, VEB (ca de cardias), trabajadores del metal mineros caucho, polvo asbesto. Genticos;
historia familiar, grupo sanguneo A. Otros; gastrectoma subtotal, anemia perniciosa, plipos, Enf de Menetrier (pliegues engrosados,
hipersecrecin de mucina, con lo cual hay perdida de protenas). Factores de riesgo definitivos y de vigilancia sugerida: Displasia de Alto Grado (75100% evoluciona a cncer), poliposis Adenomatosa Familiar, adenomas, esfago de Barrett. Definitivos: Metaplasia Intestinal (80% se asocian a
cncer), Gastritis Crnica Atrfica, infeccin por Helicobacter pylori carcingeno tipo I, cncer Colorrectal Hereditario sin poliposis. Probables: post
Gastrectoma, vagotomia Ploroplastia. Condiciones premalignas: Mentrier y ulcera gstrica. SALUD PUBLICA: El cncer gstrico se reporta como la
segunda causa de muerte a nivel mundial con aproximadamente 600,000 muertes al ao. El
adenocarcinoma representa 95%. 3er lugar en incidencia, mayores de 50 aos (69.7%). 5to
lugar en incidencia, mayores de 55 aos (55.2%). Clases socioeconmicas bajas. Ms varones
2/1. El riesgo de infeccin por H.pylori a lo largo de toda la vida en pases desarrollados es del
40 a 60%. Pases en desarrollo puede alcanzar hasta el 90%. PATOGENIA: Secuencia de
carcinognesis gstrica: Infeccin por H. pylori, toxinas (n.nitrosos), predisposicin
geneticagastritis crnicaatrofia gstrica (aclorhidria)metaplasia intestinal (hasta aqu
puede haber retroceso) displasia bajo/alto gradocncer gstrico precoz o avanzado
(adencarcinoma)metastasis. ADENOCARCINOMA: Edad media: 65-74 aos, en hombres a
edades ms jvenes, la localizacin ms frecuente es en antropilorico. Tipos: 1.
Adenocarcinoma tipo intestinal: Es el mas frecuente (antro, curvatura menor), de crecimiento
expansivo, prevalente en poblaciones de alto riesgo, hombres de mayor edad (55aos), relacin H:M 2:1, largo proceso de pre cncer, rara
diseminacin, mejor pronstico. 2. Adenocarcinoma tipo difuso: metstasis tempranas, se presenta por igual en toda la poblacin, ms frecuente en
poblaciones jvenes (48aos), relacin H:M 1:1 relacionada con factores genticos, infiltra toda pared, en el diagnostico histopatlogico lo
caracterstico son clulas en anillo de sello, diseminacin es muy frecuente. Carcinoma gstrico precoz: Confinado a mucosa y submucosa,
respetando la muscular, independiente de la presencia o ausencia de metstasis ganglionares, operado tiene buen pronostico. Carcinoma avanzado:
Infiltra la pared muscular como mnimo, usualmente asociado con extensin distante, escasa posibilidad de curacin. Enfermedad metastasica al
momento de diagnstico en el 50% de pacientes que se encuentran en estadios III o IV, las ms frecuentes: Hgado (40%) por va hematgena a
suuperficie peritoneal y/o ndulos locoregionales o distantes. Aspecto Macroscpico de adenocarcinoma: Ulcerativo 75%, polipoideo (vegetante)
10%, infiltrante 10%, superficial 5% (ver clasificacin de Borrmann). DIAGNOSTICO: Clnica; Cncer que no penetra la muscular propia, asintomtico
80% de los casos. Enfermedad avanzada al momento del diagnstico: prdida de peso 62%, dolor abdominal persistente 52%, nusea y vmito,
anorexia, disfagia (tumor de cardias), melena, saciedad temprana (tumores de antro), sangrado, emesis fecal o alimentos no digeridos en heces.
Histologa: Adenocarcinoma: 90% de las neoplasias gstricas malignas. Linfoma gstrico 10% de estos 95% linfoma no Hodgkin. Tumor Carcinoide
gstrico 0.2%. GIST Tumores estromales GI 50-60% ocurren en el estmago. Leiomiosarcomas. Metstasis: Mama y melanoma. Laboratorio y
gabinete: Rx esfago gastroduodenal, endoscopia y biopsia. Rx de trax. Ecografia de abdomen. Eco endoscopa (til para determinar profundidad).
TAC de abdomen con y sin contraste (til para Mets a distancia). Laparoscopia diagnostica estadificacin. RNM grado de invasin extraserosa y
compromiso linftico, PET. Gen p53 (mutado) cromosoma 17 alterado en 35%. ENFERMEDAD POR EXTENSION: Ndulo periumbilical (ndulo de la
hermana mara jos). Ovarios (tumor de krukennberg). Ganglio supraclavicular izquierdo (Virchow). Ndulo axilar izquierdo (ndulo irlands).
LINFOMA GASTRICO: Es el segundo tumor gstrico mas frecuente, es el linfoma extraganglionar mas frecuente, la mayora son MALT de clulas tipo
B, la gran mayora esta vinculado a infeccion por H. pylori, generalmente son submucosos, la supervivencia a 5 aos con tratamiento es de 25%.
TRATAMIENTO: Operables: Tumores mviles sin extensin mtts o carcinomatosis. Inoperables: mtts pulmonares con insuficiencia respiratoria,
carcinomatosis peritoneal, ascitis, insuficiencia ponderal y/o metablica no corregibles. Irresecables: carcinomatosis no diagnosticada en
preoperatorio, fijacin por adherencias tumorales a otras vsceras, hgado y colon no son contraindicacin absoluta. Ciruga: El tratamiento
laparoscpico del cncer gstrico temprano con linfadenectomia, es factible de realizar, con resultados similares a las tcnicas abiertas. Laparoscopia
diagnostica para estadificacin. Los procedimientos quirrgicos: A) Gastrectoma total 27%. B) Gastrectoma Subtotal 35,7%. C)
Gastroenteroanastomosis 12%. D) Lap. Exploradora 18,3%. E) Gastrect. Polar Superior 7%. Radioterapia: Tumores irresecables, con reseccin
incompleta, negativa a la cirugia y / o qt, dolor, hemorragia. Quimioterapia: Estadio II o ms: 3 ciclos de epirrubicina, cisplatino, 5-FU (ECF), pre y
postoperatorio. Otros esquemas: 5FU-Doxorubicina Cisplatino o Etopsido-5FU-Leucovorina.
CASO CLINICO CANCER GASTRICO
Se trata de paciente masculino de 41 aos de edad, originario de
Hermosillo, inicia padecimiento hace 6 meses, caracterizado por
disminucin de peso de forma continua, agrega fatiga, adinamia,
sensacin de plenitud temprana, dolor en cuadrante superior
izquierdo, a la exploracin se observa palidez generalizada,
deshidratacin moderada, disneico con tos seca, niega tabaquismo
alcoholismo, existen antecedentes de tuberculosis y cncer en la
familia, resto sin datos que agregar. En el laboratorio se reporto
sangre oculta en heces y anemia por deficiencia de hierro.
PREGUNTA
Cul es el tratamiento ms adecuado para el padecimiento?
RESPUESTA
a.- Reseccin quirrgica.
b.- Radioterapia.
c.- Quimioterapia.
d.- Reseccin quirrgica y quimioterapia.

CURSO ENARM CMN SIGLO XXI TEL: 36246001

PREGUNTA
Cul es el signo o mtodo diagnostico ms certero para este
padecimiento?
RESPUESTA
a.- Sangre en heces.
b.- Ganglio de Virchow.
c.- Serie gastrointestinal.
d.- Marcadores tumorales.
CASO CLINICO
Paciente 66 aos de edad, sexo masculino, con antecedentes
mrbidos de hipertensin arterial y diabetes mellitus (DM) tipo 2 en
tratamiento. Ingresa por cuadro clnico de 4 meses de evolucin
caracterizado por astenia, adinamia, ardor epigstrico, pirosis y
regurgitacin, asociado a lesiones drmicas, de aparicin sbita en
ambos pies, no pruriginosas. El examen fsico revel un paciente en
buen estado general, en el que destacaba nicamente presencia de

Pharmed Solutions Institute

PGINA 431

MANUAL DE TRABAJO DEL CURSO ENARM CMN SIGLO XXI


lesiones queratcicas, seborreicas e hiperpigmentadas en ambos
pies lo cual asociado a historia clnica sugera signo de Leser-Trlat.
PREGUNTA
Cual es la conducta a seguir mas adecuada?
RESPUESTA
a.- Realizar endoscopia.
b.- Realizar TAC.
c.- Realizar IRM.
d.- Realizar Biopsia de lesin drmica.
PREGUNTA
Posterior a diversos estudios incluyendo biopsia de lesin drmica se
diagnostico adenocarcinoma gstrico, el cual se encontraba en
condiciones de tratamiento quirrgico. cual es el rea mas
frecuente donde encontrar estas lesiones en caso de observarse?
RESPUESTA
a.- Pies o manos.
b.- Torax.
c.- Cabeza.
d.- Abdomen.
CASO CLINICO
Paciente mujer de 68 aos de edad, fumadora y bebedora ocasional,
madre de 3 hijos, con partos normales. No otros antecedentes de
inters. Desde hace 3 meses presenta epigastralgia asociada a la
ingesta de forma intermitente, sensacin de saciedad temprana,
refiere en ocasiones alimentos no digeridos en heces y astenia. Su
mdico le prescribi tratamiento sintomtico y solicit hemograma
de rutina. Continu con la misma sintomatologa a pesar del
tratamiento y en la analtica se detect anemia microctica e
hipocrmica. Fue derivada a consultas externas de Digestivo para
estudio, donde se le solicit una endoscopia. En el ltimo mes el
dolor epigstrico se hace continuo, empeora con la ingesta y
presenta vmitos alimenticios frecuentes que se hacen diarios en la
ltima semana y prcticamente tras cada ingesta, por lo que
finalmente acude a urgencias. Su peso habitual hace 3 meses era de
unos 77 kg, ha presentado una prdida de peso del 11.6% progresiva
por anorexia, dolor y vmitos, especialmente en la ltima semana
cuando los vmitos se han vuelto ms frecuentes. Pruebas
complementarias: Se le practica una Rx simple de abdomen y se
objetiva estmago muy dilatado y en la analtica discreta
deshidratacin y alcalosis metablica. Se ingresa a la paciente para
completar estudio.
PREGUNTA
Cul es la conducta diagnostica menos probable para este
paciente?
RESPUESTA
a.- Tomografa
b.- Endoscopia y biopsia
c.- Laparoscopia
d.- Ecografa
PREGUNTA
Cul es el diagnstico ms probable en este caso?
RESPUESTA
a.- Gastritis erosiva
b.- Ulcera duodenal
c.- Adenocarcinoma de estmago
d.- Tumor carcinoide gstrico
PREGUNTA
Qu procedimiento diagnstico es el ms adecuado para la
estadificacin?
RESPUESTA
a.- Tomografa

CURSO ENARM CMN SIGLO XXI TEL: 36246001

b.- Endoscopia y biopsia


c.- Laparoscopia
d.- Ecografa

CASO CLINICO
Paciente de sexo masculino, de 68 aos, con antecedentes de
diabetes mellitus tipo 2, insulino requiriente, hipertensin arterial
desde hace 15 aos y sndrome ulceroso en tratamiento mdico
regular, sin historia familiar de neoplasias. Consult por cuadro de 2
meses de evolucin de intolerancia alimentaria, vmitos
postprandiales progresivos y baja de peso de 8 kilos. La ecografa
abdominal describi slo microlitiasis vesicular. En endoscopia
digestiva alta se observ gran formacin cardial, nodular, ulcerada,
de bordes mamelonados y zona central necrtica con restos
fibrinohemticos.
PREGUNTA
Cul es el diagnotico mas probable en este caso?
RESPUESTA
a.- Ulcera gastrica
b.- Adenocarcinoma gstrico
c.- Poliposis adenomatosa
d.- Diverticulosis complicada
PREGUNTA
Cul es la conducta diagnotica definitiva para esta patologa?
RESPUESTA
a.- Endoscopico
b.- Biopsia de lesion
c.- pHmetria
d.- Trago de Bario
PREGUNTA
Cul es la conducnta teraputica mas adecuada a seguir en este
momento?
RESPUESTA
a.- Gastrectomia total
b.- Tratamiento medico y nueva endoscopia
c.- Laparoscopia exploradora
d.- Endoscopia y biopsia
CASO CLINICO
Varn A.C. de raza blanca de 65 aos de edad con historia de ingreso
hospitalario por presentar deposiciones "oscuras" (negruzcas),
refiriendo dolor abdominal, as como anorexia, prdida de peso y
que es remitido a nuestro Servicio imagenologa para la realizacin
un estudio Tomogrfico simple toracoabdominal que revel la
presencia de una lesin deaspecto de masa, polipoidea bien
delimitada de unos 2,5 cm de dimetro y 44 UH localizada en el
cuerpo gstrico yengrosamiento focal de la pared gstrica anterior e
izquierda. Se llev a cabo la endoscopia y biopsia de la lesin.
Durante la realizacin del estudio endoscpico se comprob la
presencia de una masa vegetante, ulcerada que requiri la
realizacin de toma de biopsia.
PREGUNTA
Cul es la frecuencia del linfoma gstrico?
RESPUESTA
a.- 90%
b.-60%
c.- 30%
d.- 15%

Pharmed Solutions Institute

PGINA 432

MANUAL DE TRABAJO DEL CURSO ENARM CMN SIGLO XXI


POLIPOS
CIENCIAS BASICAS: Son protuberancias en la mucosa intestinal, protruyen de la pared hacia la luz intestinal. Hay de origen mucoso que pueden ser
no neoplsicos que forman el 90% (hiperplasicos, plipos glabdulares fundicos, poliposis hamartomosa de Peutz Jeghers, plipo fibroso
inflamatorio). Neoplasicos: adenomas. Tambien de origen submucoso. HAMARTOMAS: Es un crecimiento con proliferacin excesiva de un tipo de
tejido sin cambios neoplsicos verdaderos. SINDROME DE PEUTZ-JEGHERS: Sndrome autosmico dominante que se manifiesta por pigmentacin de
zonas mucocutneas y plipos hamartomosos en el intestino delgado y grueso compuestos de cantidades excesivas de muscular de la mucosa. Hay
descripcin de progresin de los plipos a neoplasia maligna, pero no es frecuente. Se observa mayor incidencia de tumores malignos en otros
rganos, como la mama y el ovario. Los sntomas incluyen hemorragia y obstruccin intestinal secundaria a invaginacin. POLIPOS JUVENILES: Por lo
general ocurren en nios, pero tambin se observan en adultos. Son hamartomas compuestos de glndulas dilatadas y lmina propia anormal. Es
posible que ocurra hemorragia secundaria a la autoamputacin o invaginacin. POLIPOSIS FAMILIAR JUVENIL: Sndrome autosmico domnate con
mltiples plipos juveniles en todo el colon. Pueden manifestarse por hemorragia u obstruccin y los pacientes tienen mayor riesgo de presentar
tumores malignos en el tubo digestivo. El tratamiento es la colectoma total. HIPERPLASICOS: Son muy frecuentes y casi siempre miden menos de
5mm; asociados a gastritis crnica en bordes de las ulceras, son pequeosny ssiles, al examen histolgico, muestra ausencia de maduracin e
hiperplasia sin displasia nuclear. No se consideran premalignos. ADENOMATOSOS: Son los ms importantes a causa de su frecuencia y potencial de
malignidad (25-70%). Desde el punto de vista morfolgico estos plipos se describen como pediculados (con un tallo) o ssiles (planos). El examen
histolgico, los clasifica como tubulares (87%), vellosos (5%) o tubulovellosos (8%) mixtos conforme el patrn dominante. Mientras mayor sea el
plipo y mayor componente velloso posea, mas potencial maligno tiene. Como regla general, los plipos adenomatosos deben extirparse por su
potencial maligno. La mayor parte puede extirparse con una trampa por colonoscopia, casi siempre en una pieza. Polipectoma aun sea un
tratamiento suficiente si se cubren los siguientes criterios: 1. El plipo es pediculado. 2. El tallo no est afectado y los mrgenes de reseccin estn
libres. No hay invasin vascular, linftica o neural y la lesin est bien diferenciada. DIAGNOSTICO: La colonoscopia, es la tcnica de eleccin para el
diagnstico de plipos colorectales, es el patrn de oro, adems permite la exerecis o la realizacin de una biopsia, en los que la exceresis no es
posible. Tan importante es la colonoscopia como el estudio histolgico del plipo. TRATAMIENTO: Todos los plipos de colon deben tratarse
mediante polipectoma endoscpica completa, ya que su aspecto macroscpico los hace indistinguibles, deben ser recuperados y proceder a anlisis
histolgico. La polipectoma endoscpica de los adenomas colorectales se ha demostrado que disminuye la incidencia de cncer colorectal y es el
tratamiento definitivo cuando se efecta una reseccin completa. La perforacin y el sangrado son las complicaciones ms frecuentes descritas.
CASO CLINICO
Paciente femenino de 18 aos de edad, con antecedente de
hiperpigmentacin de retina de ojo derecho y dermatitis atpica.
Acude por sangrado de tubo digestivo bajo, comenzando 3 meses
antes de la consulta, el sangrado es rojo, escaso que acompaa de
deposiciones normales diarias, sin dolor defecatorio, ni pujo o
tenesmo, agrega que antes del sangrado, presentaba dolor
abdominal en zona epigstrica de forma espordica, que calma
espontneamente, finalmente refiere dolor articular desde hace 4
meses, sin fiebre ni causa aparente. El EF es normal, se observa
tumoracin en zona retroauricular derecha, redondeada de 1 cm de
dimetro de consistencia firme, superficie lisa e indolora, adems de

tumoraciones en cara interna de ambos pies, sin dolor, redondeada


de .5 cm de dimetro consistente y firme, lisa e indoloro, inspeccin
perineal y anorectal normal.
PREGUNTA
Cual es la conducta a seguir mas adecuada?
RESPUESTA
a.- Transito gastro intestinal.
b.- Panendoscopia.
c.- Polipectomia endoscpica.
d.- Biopsia de plipo.

VESICULA BILIAR, LITOGENESIS:


CIENCIAS BASICAS: La vescula biliar es un rgano piriforme de aproximadamente 7 - 10 cms. de largo por 3 cms. de ancho, con una capacidad
habitual de 30-35cc, pudiendo contener volmenes muy superiores en condiciones patolgicas. Se le reconocen tres partes principales: fondo,
cuerpo y cuello. Desde ste emerge el conducto cstico, mediante el cual la vescula se une al conducto heptico comn, para dar origen al conducto
coldoco. La longitud de ste es de aproximadamente 7,5 cms., variable segn el punto de desembocadura del conducto cstico. Alcanza un
dimetro normal de 5,5 mm, el cual es mucho mayor en caso de patologa obstructiva. Se reconocen 4 porciones bien definidas: 1. Porcin
Supraduodenal, 2. Porcin Retroduodenal, 3. Porcin Pancretica, 4. Porcin intramural duodenal. El coldoco suele unirse al conducto pancretico
justo al interior de la pared duodenal en el 89% de los vasos. Formacin de clculos biliares: Existen dos tipos de clculos, de acuerdo a su
composicin bioqumica: clculos puros y clculos mixtos. Los clculos puros son de origen exclusivamente metablico, y estn compuestos de
colesterol o bilirrubina. Los clculos mixtos presentan en su etiopatogenia una causa inflamatoria, y estn formados por una mezcla de colesterol,
sales de calcio y pigmentos biliares, lo que se deposita sobre una base de naturaleza orgnica formada por clulas epiteliales, material proteico y
bacterias. El clculo de bilirrubinato de calcio (mixto) se forma a partir de un ncleo, de preferencia en los conductos biliares. Corresponden al 82%
de los clculos. El aumento de bilirrubina libre en la bilis (lo normal es 20%) determinar el fenmeno de litognesis primaria que corresponde a la
formacin de bilirrubinato de calcio. Los clculos de colesterol se producen por una alteracin del equilibrio bioqumico de los componentes de la
bilis: sales biliares, fosfolpidos (lecitina), colesterol y cidos biliares. Para que el colesterol no precipite, la bilis debe estar bajo la forma de micelas
constituidas por un centro de colesterol y una cubierta de sales biliares y fosfolpidos. En esta forma micelar el colesterol es hidrosoluble. Las
alteraciones tanto en la calidad como en cantidad de las sales biliares y la lecitina van a determinar la precipitacin y formacin posterior de
clculos. A esta bilis alterada en su composicin la denominaremos Bilis litognica (Bilis formadora de clculos). El aumento de la excrecin de
colesterol se ha asociado a una mayor incidencia de litiasis biliar. Por otro lado la ectasia biliar, la infeccin biliar y factores metablicos tambin se
relaciona con formacin de colelitiasis. Los estrgenos aumentan el ndice de saturacin de colesterol, favoreciendo colelitiasis. Tambin habra
factores genticos relacionados con litiasis biliar. Se ha observado mayor frecuencia de litiasis biliar en: diabticos, obesos, embarazadas y mujeres
que usan anticonceptivos de tipo andrognico.

CURSO ENARM CMN SIGLO XXI TEL: 36246001

Pharmed Solutions Institute

PGINA 433

MANUAL DE TRABAJO DEL CURSO ENARM CMN SIGLO XXI


COLANGITIS
CIENCIAS BASICAS: La colangitis aguda es la infeccin severa de la bilis dentro de los conductos biliares intra y extra hepticos. Est producida por la
existencia de bacterias en la bilis, en una situacin de obstruccin biliar, como consecuencia de la obstruccin del flujo biliar, se produce un aumento
en la presin intraductal que favorece el paso de grmenes a la circulacin portal y linftica, produciendo episodios de bacteriemia con septicemia o
sin ella. Su etiologa es debido a enfermedades concomitantes (colangitis primarias: coledocolitiasis (70%) parasitosis, estenosis, quistes de coldoco,
enfermedad de Caroli, colangitis esclerosante y tumores) o por actuaciones directas o indirectas, diagnsticas o teraputicas, sobre el hgado o la va
biliar (colangitis secundaria). La colangitis ser supurada o no supurada si la bilis en la va biliar est contaminada o es purulenta; los grmenes ms
frecuentes involucrados n son E. Coli, Kleibsella, pseudomona estreptococo fecalis, enterobacter, bacteroides y clostridium. DIAGNSTICO: Clnico:
Se basa en la asociacin de signos y sntomas de infeccin con los propios de una obstruccin biliar. Dolor abdominal, en el cuadrante superior
derecho en el centro del abdomen superior, puede ser intermitente, agudo, tipo clico o sordo, puede irradiarse a la espalda o debajo del
omplato derecho, escalofros, fiebre generalmente alta, con escalofros e ictericia (triada de Charcot). Cuando se aade confusin mental y shock se
denomina Pentada de Reynolds, con menos frecuencia y que se asocia con una colangitis supurada grave. Puede presentar heces color arcilla, orina
oscura as como nuseas y vmitos. Laboratorio y gabinete hallazgos de obstruccin biliar con aumento variable de las bilirrubinas ms de 2md/dl
(80%) y enzimas de colestasis (FA, TGO, TGP). Habitual encontrar leucocitosis con desviacin a la izquierda (80%) siendo por lo general las cifras ms
altas en torno a 20.000 mm3, reflejo de las formas ms severas, hemocultivos positivos durante los escalofros o picos febriles. La ecografa es la
tcnica de eleccin para detectar la existencia de obstruccin biliar por su elevada eficacia y versatilidad, pudindose detectar tambin
complicaciones en la colangitis como absceso heptico. La colangio-resonancia magntica con la sensibilidad de 90% y una especificidad 100%.La
colangiopancreatografa retrgrada endoscpica (CEPRE) y la colangiografa transheptica percutnea. TRATAMIENTO: Piperacilina 4grs i.v./8h.
Tazobactam 4.5grs i.v./8h. Ciprofloxacino 200mgsi.v./12h. Ceftriaxona 1gr i.v/24h. Metronidazol 500mgs i.v/8h. Analgsicos, antiinflamatorios y
antipirticos. Medidas higinicas y dietticas: El consumo de alimentos ser limitado durante su hospitalizacin. Se recomienda dieta baja en grasas.
Durante su internamiento depender de la situacin clnica y evolucin el inicio de la dieta, puede requerir de nutricin parenteral en los casos
graves por sepsis. Bajar de peso, desintoxicacin del hgado y coln. Evitar estrs, ejercicio regular ayuda a tonificar la vescula. Mantener una dieta
adecuada, realizar ejercicio, evitar alimentos condimentados y grasosos. Indicaciones de hospitalizacin: Tener fiebre, ictericia, vmitos recurrentes
o se presenta dolor despus del diagnstico o tratamiento, se incluye disnea, diaforesis y nuseas. Deterioro progresivo y mal estado general.
Criterios de evaluacin de severidad leve (Grado I), colangitis que responde al tratamiento mdico inicia. Moderado (Grado II) colangitis aguda que
no responde al tratamiento mdico inicial y no se acompaa de disfuncin orgnica mltiple. Severa (Grado III) colangitis aguda que se asocia con la
aparicin de disfuncin de al menos uno de los siguientes rganos / sistemas: cardiovascular, hipotensin que requiere de aminas, alteraciones de la
consciencia, relacin PaO2/Fio2 <300,Cr srica > 2 mg/dl, TP-RIN>1.5, Plaquetas < 100.000/ul.
CASO CLINICO COLANGITIS
Paciente de 59 aos sin antecedentes personales de inters que
acudi a urgencias por dolor epigstrico y febrcula. En la
exploracin se observo ictericia mucocutanea y ocupacin del
hemiabdomen superior derecho. La analtica demostr: bilirrubina
total, 5,3 mg/dl; transaminasa glutmico pirvica (GPT), 311 U/l;
transaminasa glutmico oxalactica (GOT), 166 U/l, y fosfatasa
alcalina (FA), 176 U/l; el resto fue normal. Se realiz una ecografa y
una tomografa computarizada (TC) abdominal que confirmaron la
existencia de 2 grandes quistes hidatdicos en el lbulo heptico
derecho (de 13 y 9 cm) y otro quiste hidatdico calcificado en el
lbulo heptico izquierdo de 5 cm. Las ramas biliares principales
derecha e izquierda y el coldoco en toda su longitud estaban
ocupadas por un material con mltiples ecos lineales. La serologa
para hidatidosis fue positiva (tuvo contacto con perros y ganado
hace aos). Se inici tratamiento con albendazol antes de la ciruga y
se realiz una colangiopancreatografa retrograda endoscpica
(CPRE), que confirm la dilatacin y la ocupacin de la va biliar con
salida espontnea de material hidatdico.
PREGUNTA
Cul es los siguientes patologas no es diagnostico diferencial.
RESPUESTA
a.- Hepatitis aguda.
b.- Nefrolitiasis.
c.- Obstruccin intestinal.
d.- Ulcera duodenal.
CASO CLINICO
Femenino de 70 aos, previamente colecistectomizada, a los 60 aos
e histerectomizada a los 54 aos por miomatosis uterina. Paciente
refiere cuadro de dolor abdominal epigstrico de seis das de
evolucin, no bien caracterizado, asociado a nuseas y vmitos. Tres
das previos a su ingreso se agregan ictericia de escleras y mucosas y,
coluria franca. Dolor persistente con exacerbaciones clicas.

CURSO ENARM CMN SIGLO XXI TEL: 36246001

Ecografa abdominal muestra va biliar de 18mm y con clculos en su


interior.
PREGUNTA
Cul es la complicacin ms frecuente para este caso?
RESPUESTA
a.- Perforacin
b.- Piocolecisto
c.- Colangitis
d.- Empiema
PREGUNTA
Cul es la conducta diagnostica ms adecuada para este paciente?
RESPUESTA
a.- CEPRE
b.- Ultrasonido endoscpico
c.- Gamagrama
d.- Tomografa
PREGUNTA
La paciente ya hospitalizada a los 2 das, desconoce a su familiar, se
encuentra desorientada, con fiebre (39C), ictericia, as como
progresin de dolor en cuadrante superior derecho, FC 100x, FR
23x, TA 90/60mmHg. BH leucocitos 15,000 con desviacin a la
izquierda, Fosfatasa alcalina 350 UI/L, BT 6.5mg/dl, enzimas
hepticas elevadas. Cul de las siguientes condiciones de la
paciente, no indica un abordaje quirurgico?
RESPUESTA
a.- Dilatacin de conductos extrahepticos
b.- Ictericia
c.- Escalofri y fiebre recurrente sugestiva de colangitis
d.- Hepatitis

Pharmed Solutions Institute

PGINA 434

MANUAL DE TRABAJO DEL CURSO ENARM CMN SIGLO XXI


COLECISTITIS
CIENCIAS BASICAS: Es la inflamacin de la vescula biliar ocasionada principalmente por clculos (litos) y con menor frecuencia por barro (lodo)
biliar, en raras ocasiones ninguna de estas condiciones est presente. Colelitiasis: presencia de litos en la vesicula biliar y con menor frecuencia de
lodo biliar. SALUD PUBLICA: Es una de las principales causas de consulta en el servicio de urgencias y en la consulta externa de ciruga general. La
colecistectoma electiva es la intervencin quirrgica ms frecuente en los centros hospitalarios. Se presenta en el 5-20% de los pacientes con
colelitiasis. La colecistitis aguda se presenta con ms frecuencia en mayores de 40 aos, ms frecuente en sexo femenino 2:1. PATOGENIA: Los
factores que pueden condicionar el desarrollo de Colecistitis y Colelitiasis: Edad: ms frecuente a partir de los 40 aos, cerca del 20% de los adultos
a partir de esta edad y del 30% en los mayores de 70 aos, sexo femenino, embarazo, sobre todo para el desarrollo de clculos de colesterol,
normalmente son formas asintomticas de litiasis biliar, anticonceptivos orales y terapia hormonal sustitutiva con estrgenos, en este caso en
mujeres menores de 40 aos y las que reciben una dosis mayor de 50 microgramos de estrgenos. Otros frmacos como los fibratos y la ceftriaxona,
obesidad, antecedentes familiares de litiasis biliar, prdida rpida de peso, nutricin parenteral, DM, cirrosis heptica, enfermedades del leon,
enfermedad de Crohn, enfermedades hepticas y metablicas. CLASIFICACION: La colecistitis aguda se clasifica en tres grados, de acuerdo a la
afectacin de otros rganos, medidos por pruebas de laboratorio: Grado I - leve, Grado II - moderada, Grado III grave. DIAGNOSTICO: Clnico: de
colecistitis y colelitiasis son: Murphy positivo, dolor y resistencia en cuadrante superior derecho, nausea, vomito. Manifestaciones agudas; vescula
palpable, fiebre >39, escalofros., inestabilidad hemodinmica. La perforacin con peritonitis generalizada se sospecha cuando; existen signos de
irritacin peritoneal difusa, distensin abdominal, taquicardia, taquipnea, acidosis metablica, hipotensin, choque. No hay pruebas de laboratorio
especficas para colecistitis aguda, pero pueden ser de utilidad: biometra hemtica: se puede encontrar leucocitosis. Protena C Reactiva la cual
puede encontrarse elevada y es de utilidad para confirmar el proceso inflamatorio. Ante la sospecha de colecistitis o colelitiasis aguda el ultrasonido
es la prueba no invasiva de primera eleccin 98% de sensibilidad, los hallazgos que reporta son: engrosamiento de la pared vesicular mayor de 5
mm. Liquido perivesicular, signo de Murphy ultrasonogrfico positivo, alargamiento vesicular 8 cm. Axial y 4 cm. Diametral, lito encarcelado, imagen
de doble riel, sombra acstica, ecos intramurales. Para identificar la gravedad de la Colecistitis y Colelitiasis se solicitar: Bilirrubinas, BUN,
creatinina, tiempo de protrombina, la amilasa srica es de utilidad para identificar complicaciones como coledocolitiasis. TRATAMIENTO: para la
disolucin de los clculos biliares en pacientes en situaciones especilaes (pacientes con alto riesgo quirrgico y aqueloos que se rehsan a la ciruga),
el tratamiento de eleccin es con acidos biliares como: Acido ursodesoxicolico, cido quenodeoxicolico, por un periodo de 1-2 aos. Depende del
nivel de gravedad, se prefiere una colecistectoma temprana y de seleccin adecuada de la tcnica con la que se realizar la ciruga. La litiasis
vesicular asintomtica no es indicacin de colecistectoma excepto que la pared vesicular este engrosada (4mm o ms) o calcificada o que los
clculos sean mayores a 3 cm de dimetro. La colecistectoma puede indicarse ocasionalmente en pacientes con dolor vesicular sin litiasis
(colecistitis alitisica) o porque la vescula no funcione adecuadamente (discinesia biliar). En estos casos el diagnostico se realiza mediante una
colecistografa oral o un gamagrama de vescula. La colecistectoma laparoscpica temprana es el tratamiento de eleccin en la mayora de los
pacientes, porque: la recuperacin es rpida, requiere menor estancia hospitalaria, reduce costos de tratamiento, reincorporacin rpida al trabajo.
Se considera temprana cuando se realiza de 1 a 3 das despus del ataque inicial y tardo si se realiza de 6 A 15 das despus del ataque. Factores que
dificultan la colecistectoma por laparoscopia son: ciruga abdominal previa, presencia o antecedentes de ictericia. La combinacin de extraccin de
litos por endoscopia durante una colangio-pancreatografa retrograda endoscpica (CPRE) y la colecistectoma por laparoscopia, es de utilidad en el
tratamiento de pacientes con colecisto y coledocolitiasis. El intervalo entre estos dos procedimientos es de pocos das (ambos procedimientos se
realizarn durante la misma estancia hospitalaria). En la Colecistectomia abierta no complicada los das de estancia hospitalaria posquirrgica son de
2 a 3 das. Y el tiempo promedio de recuperacin es de 21 das. En la colecistectomia laparoscpica no complicada dos das de estancia hospitalaria
es suficiente. MANEJO PERIOPERATORIO: Correccin control de los factores de riesgo, estudios de laboratorio e imagen solicitados en forma
particular a cada caso, en mayores de 45 aos electrocardiograma, telerradiografa de trax y valoracin cardiovascular preoperatoria, el uso
profilctico de antibiticos se selecciona en base al tipo de herida quirrgica. En todos los pacientes se debe llevar a cabo prevencin
tromboemblica. La administracin de AINEs se recomienda a pacientes con clico biliar, sin embargo no es til para la remisin de la enfermedad.
Es recomendable el uso de diclofenaco a razn de 75 mgr. IM para el manejo de los clicos biliares. El manejo del dolor es pacientes con colecistitis
aguda incluye narcticos como meperidina, en presencia de dolor intenso. El tratamiento de antibitico se indicar dependiendo de la gravedad de
la colecistitis: para colecistitis leve- un antibitico, para colecistitis moderada y grave se recomienda el uso de dos antibiticos, cuando se sospecha o
se detecta la presencia de anaerobios agregar metronidazol. Litotripsia extracorprea: Indicada en pacientes con litiasis nica, no calcificada, con
dimetros de 20-30mm. Contraindicada en pancreatitis, alteraciones de la coagulacin, quistes o aneurismas en el trayecto de las ondas de choque.
El cido ursodeoxicolico se indica como coadyuvante cuando se realiza litotripsia, ya que de esta forma se logra mayor efectividad en la destruccin
de litos. No aprobado por FDA como tratamiento definitivo para litiasis vesicular. COMPLICACIONES: de la colecistectoma laparoscpica
generalmente son debidas a instrumental inapropiado o caduco, y tambin por la falta de pericia del cirujano como son: lesin del conducto biliar,
lesin del intestino, lesin heptica. En la ciruga abierta y laparoscpica adems puede haber como complicacin: las infecciones, leo, hemorragia
intraperitoneal, atelectasia, trombosis de venas profundas, infeccin de vas urinarias.
CASO CLINICO
Varn de 32 aos, que acude a Urgencias por dolor en hipocondrio
derecho de 72h de evolucin tipo clico y vmitos tras la ingesta que
no calman con analgesia convencional y antiemticos. Siendo dado
de alta a las 48h. Sin antecedentes mdicos. No Sd constitucional.
Ictericia cutnea y mucosa evidente sin prurito asociado. No fiebre.
Hemodinmicamente estable. T. 36,8C. Abdomen blando,
depresible, doloroso en hipocondrio derecho, sin masas ni megalias.
No peritonismo. Ictericia mucocutnea. La analtica muestra 10,9
10E3/l leucocitos y discreta eosinofilia (0,7 10E3/l). Bilirrubina
total de 7,9mg/dl, con bilirrubina conjugada de 5,2mg/dl y aumento
de fosfatasa alcalina de 853 y ALT de 268. Lipasa de 346.
PREGUNTA
Cul es la conducta a seguir mas adecuada?
RESPUESTA
a.- Conducta conservadora.

CURSO ENARM CMN SIGLO XXI TEL: 36246001

b.- Preparar para colecistectoma.


c.- Realizar USG abdominal.
d.- Realizar TAC abdominal.
CASO CLINICO
Mujer de 17 aos que acude a Urgencias por un cuadro de abdomen
agudo. Como antecedentes, destacaba un ingreso un ao antes para
estudio por un sndrome febril y cefalea. Fue diagnosticada de una
infeccin por citomegalovirus y por virus Coxsackie B6, desde
entonces permaneciendo asintomtica y sin tratamiento. La
paciente consult por dolor abdominal de 6 horas de evolucin,
iniciado en epigastrio y posterior migracin a fosa ilaca derecha,
intenso, de caractersticas continuas, con vmitos, sin fiebre ni otros
sntomas. A la exploracin presentaba febrcula, defensa y signos de
irritacin peritoneal en hemiabdomen derecho. Analtica: leucocitos
16.330/l, neutrfilos 76,7%, hematocrito 39%, ALT 59 UI/l, GGT 19
UI/l, amilasa 354 UI/l, PCR 0,7mg/dl

Pharmed Solutions Institute

PGINA 435

MANUAL DE TRABAJO DEL CURSO ENARM CMN SIGLO XXI


PREGUNTA
Cual es la conducta inmediata mas adecuada?
RESPUESTA
a.- Realizar USG plvico.
b.- Realizar prueba de embarazo.
c.- Realizar RX de abdomen.
d.- Mantener medidas generales.

PREGUNTA
Cul es la conducta diagnostica a seguir ms adecuada para esta
paciente?
RESPUESTA
a.- CEPRE
b.- Laparoscopia
c.- Ecografa
d.- Endoscopia

PREGUNTA
Posterior a la medida realizada se realizar RX de abdomen siendo
negativa, y USG con liquido libre en cavidad, cual es la conducta a
seguir?
RESPUESTA
a.- Mantener conducta expectante.
b.- Realizar TAC.
c.- Preparar para LAPE.
d.- Lavado peritoneal
PREGUNTA
Cual es su impresin diagnostica mas probable en este momento?
RESPUESTA
a.- Colecistitis.
b.- Apendicitis.
c.- Embarazo ectpico.
d.- Ulcera perforada.
CASO CLINICO
Se trata de femenino de 85 aos, ingresa por dolor epigstrico de 12
horas de evolucin de inicio brusco y continuo, antecedentes de
hipertensin arterial y bronquitis crnica sin tratamiento con
corticoides, EF no signos de sepsis, abdomen blando depresivo,
doloroso en la regin superior, no datos de irritacin peritoneal, Hto
41, leucos 11,900 glucosa 128, urea 23, CK 78, DHL 385, resto
normal. USG abdominal mostro microlitiasis con vesicula biliar sin
dilatacin.
PREGUNTA
Cual es la conducta a seguir mas adecuada?
RESPUESTA
a.- Realizar endoscopia.
b.- Conducta expectante.
c.- Liquidos, analgsico y antibiticos.
d.- Colecistectomia laparoscpica.
CASO CLINICO
Se trata de mujer de 38 aos procedente de Monterrey, sin
antecedentes de inters salvo obesidad, en tratamiento con
anticonceptivos orales. Acude a urgencias de madrugada
presentando desde hace 2 horas: dolor agudo localizado en
epigastrio e irradiado a hipocondrio derecho y espalda, que ha ido
aumentando, as como vmitos recurrentes y diaforesis. Hace 1 mes
episodio similar que cedi espontneamente en 30 minutos.
Exploracin Fsica: Orientada, tinte ictrico +, T 39C. TA 150/90
mmHg. Abdomen: dolor en epigastrio e HCD a la palpacin profunda
sin signos de peritonismo y Murphy negativo. Exploraciones
complementarias: 13500 leucocitos (85 %PMN), Bilirrubina total 2,5
mgr /dL (urobilingeno en orina positivo), ALT/AST 128/65 UI/L, GGT
y FA aumentadas.
PREGUNTA
Cul es la conducta teraputica ms adecuada a seguir?
RESPUESTA
a.- Dar tratamiento y enviar a su domicilio
b.- Hospitalizar, iniciar protocolo de estudio
c.- Pasar analgsico y antiemtico IV
d.- Laparotoma exploradora

CURSO ENARM CMN SIGLO XXI TEL: 36246001

PREGUNTA
Cul es el diagnostico menos probable para el caso?
RESPUESTA
a.- Coledocolitiasis
b.- Colangitis esclerosante
c.- Quistes de coldoco
d.- Colecistitis
CASO CLINICO
Madre de 52 aos con Diabetes mellitus de 10 aos de evolucin
bajo tratamiento, obesidad. Inicia por la maana posterior al
desayuno con dolor tipo clico, de inicio sbito, de intensidad
moderada a grave, continuo, localizado en el cuadrante superior
derecho del abdomen, se irradia a la regin interescapular, y que no
remite ante ninguna posicin ni actividad. Aunado a esto presenta
nausea y vmito en una ocasin de contenido gstrico. Por la tarde
presenta fiebre no cuantificada, astenia, adinamia y anorexia,
adems de postracin e inmovilidad por incremento del dolor ante
los cambios de posicin. Refiere que en ocasiones anteriores
durante los ltimos tres aos tuvo un dolor parecido pero de menor
intensidad, que duraba de 3 a 4 horas y luego remita, y que en la
mayora de los casos se asociaba a comidas abundantes y/o
grasosas. EF: facies dolorosa, alerta, reactiva, en posicin antilgica,
diafortica, con tinte ictrico leve en conjuntivas, taquicrdica,
taquipneica, con hipersensibilidad abdominal en el cuadrante
superior derecho, donde se palpa una masa dolorosa de consistencia
renitente, signo de Murphy positivo, timpanismo generalizado,
peristalsis disminuida.
PREGUNTA
Cul es el diagnstico ms probable de acuerdo al cuadro clnico y
antecedentes de la paciente?
RESPUESTA
a.- Colangitis aguda
b.- Sndrome de Mirizzi
c.- Colecistitis aguda
d.- Coledocolitiasis
PREGUNTA
Considerando el caso Cul de las siguientes condiciones no sera
indicativo de procedimiento quirrgico?
RESPUESTA
a.- Colecistitis alitisica
b.- Discinesia biliar
c.- Colecistitis asintomtica
d.- Colecistitis litiasica aguda
PREGUNTA
Cul sera la utilidad de un AINE en la colecistitis?
RESPUESTA
a.- Accin espasmoltica sobre la musculatura lisa, modifica historia
natural
b.- Modifica historia natural, reduce prostaglandinas, disminuye
presin intraluminal
c.- Accin miorrelajante y dilatacin del esfnter de Oddi para un
mejor vaciamiento del lquido biliar
d.- Reduce prostaglandinas, accin espasmolitica del musculo liso

Pharmed Solutions Institute

PGINA 436

MANUAL DE TRABAJO DEL CURSO ENARM CMN SIGLO XXI


COLEDOCOLITIASIS
CIENCIAS BASICAS: El factor comn presente en la gran mayora de las enfermedades de la va biliar es la Colelitiasis. La complicacin ms
frecuente, la Coledocolitiasis, se ha reportado con cifras hasta de un 20%. Se define coledocolitiasis como la presencia de clculos biliares en el
conducto Coldoco y/o en conducto heptico comn. SALUD PUBLICA: Segn datos internacionales los clculos biliares se encuentran en el 12% de
los hombres y el 24% de las mujeres. La prevalencia aumenta con la edad. Ms de un 10% de pacientes portadores de colelitiasis presentan
coledocolitiasis. La asociacin entre colecistitis crnica litisica y coledocolitiasis es de aprox un 15%. La asociacin entre colecistitis aguda y
coledocolitiasis puede alcanzar hasta un 25%. PATOGENIA: La mayora de los clculos coledocianos se originan de la vescula biliar, de hecho su
forma y composicin son similares a la de los clculos vesiculares creciendo en el coldoco por aposicin de colesterol; simultneamente se produce
una dilatacin gradual de la va biliar que con los aos puede llegar a un dimetro de 2 cm o ms. Con menor frecuencia los clculos coledocianos se
originan en la misma va biliar, ello se observa en casos de estenosis del heptico comn o el coldoco en los que se desarrollan clculos mixtos o de
bilirrubinato de calcio. Este fenmeno desaparece si se corrige la estrechez (dilatacin endoscpica) o se deriva la va biliar dilatada al duodeno o
yeyuno. En un paciente colecistectomizado puede detectarse coledocolitiasis en el postoperatorio alejado, lo cual se debe a que durante la
operacin no se sospech ni se diagnostic una coledocolitiasis concomitante. En este caso hablamos de Coledocolitiasis residual cerrada. Cuando se
detecta la coledocolitiasis en el postoperatorio de un paciente sometido a una colecistectoma y adems coledocostoma y por lo tanto, portando
una sonda T situada en el coldoco, hablamos de Coledocolitiasis residual abierta. Cuando han pasado varios aos despus de realizada la
colecistectoma en un paciente y se detecta coledocolitiasis, hablamos de coledocolitiasis cerrada de Neoformacin. DIAGNOSTICO: Puede producir
obstruccin del coldoco con impedimento parcial o total del paso normal de bilis desde hgado al duodeno, dando lugar al denominado Sndrome
de Ictericia Obstructiva. La mayora de estos pacientes tienen antecedentes biliares, ya sea de clicos biliares en reaccin a ingestin de alimentos
grasos, colelitiasis asociada o demostrada antecedentes de colecistectoma. Es caracterstico, la presencia de dolor abdominal de tipo clico ubicado
en epigastrio o hipocondrio derecho, que precede a la aparicin de coluria e ictericia de piel y escleras. Posteriormente, puede aparecer hipocolia. Se
asocia a estado nauseoso y vmitos rebeldes. Puede prolongarse durante varias horas, y si se alivia con antiespasmdicos tiende a desaparecer
precozmente. La ictericia es fluctante, debido al mecanismo valvular que determina la obstruccin de la va biliar por l o los clculos flotantes que
se desplazan y eventualmente se impactan y se desimpactan sucesivamente dentro de ella. Se le puede confundir con un clico ureteral derecho; es
til tener presente en este caso que el paciente se encuentra muy inquieto, mientras que si el dolor es de origen biliar, tiende a permanecer
postrado en la cama. La coluria precede a la ictericia y el paciente ictrico orienta a etiologa obstructiva extraheptica, ya que esta se produce
debido a que se elimina bilirrubina directa o conjugada a travs de la orina. El enclavamiento de un clculo en la Ampolla de Vater da origen a una
ictericia prolongada, que en los enfermos ancianos puede complicarse con una enfermedad tubular aguda. A estos sntomas y signos se le agrega
prurito, por depsitos de sales biliares en la piel. En los pacientes seniles, la coledocolitiasis suele ser causa de anorexia y prdida de peso. Conviene
insistir en que muchos enfermos con clculos en el coldoco no tienen ningn sntoma que revele su presencia. Por ello es tan importante, durante
la colecistectoma electiva o de urgencia, la cuidadosa exploracin radiolgica de la va biliar. Los mtodos de diagnstico por imagen de la
coledocolitiasis pueden ser preoperatorios e intraoperatorios. Dentro de los preoperatorios se encuentran: Ecografa (eco), Colangioresonancia (CRMN), Tomografa computarizada (TC), Colangiopancreatografa retrgrada endoscpica (CPRE), Ecoendoscopa (ECO-E), Colangiografa endovenosa
(CIV) y Colangiografa transparietoheptica (CTPH). Los intraoperatorios: Colangiografa intraoperatoria (CIO) y ecografa por laparoscopa (ECO-L).
TRATAMIENTO: Si el paciente presenta coledocolitiasis asociada a colelitiasis y la presencia de clculos coledocianos ha sido establecida
previamente a la intervencin quirrgica, el procedimiento ms aceptado hoy da es intentar la extraccin de los clculos va endoscpica (CPER
asociada a Papilotoma y extraccin endoscpica de clculos), para luego proceder con una colecistectoma laparoscpica. Si el diagnstico de
coledocolitiasis ha sido establecido durante la realizacin de una colecistectoma (mediante colangiografa intraoperatoria), se puede programar una
extraccin diferida de clculos coledocianos por va endoscpica. En algunos centros se realiza ambos procedimientos en un solo tiempo en forma
intraoperatoria. Si no se cuenta con CPER o el paciente tiene contraindicacin para ciruga laparoscpica, se puede resolver ambas situaciones con
una ciruga clsica, realizando una coledocostoma intraoperatoria para extraer los clculos. Esta alternativa requiere dejar en la va biliar un drenaje
(sonda T) para evitar una complicacin post operatoria (biliperitoneo).
CASO CLINICO
Mujer de 56 aos, sin antecedentes de inters, que ingresa por
cuadro febril que se acompaa de dolor en hipocondrio derecho y
vmitos repetidos. Se diagnostica de colecistitis aguda y por
ecografa se comprueba colelitiasis. Se trata con pauta antibitica y
la paciente mejora, quedando afebril y asintomtica. Una nueva
ecografa de control demuestra la persistencia de la colelitiasis, por
lo que se decide tratamiento quirrgico, realizndose
colecistectoma. La intervencin quirrgica transcurre sin problemas
y el postoperatorio inmediato es correcto. La evolucin en piso
transcurre sin problemas, pero a la maana del 5 da del
postoperatorio la paciente presente de manera sbita un cuadro
clnico de disnea, taquipnea y taquicardia, apareciendo pocas horas
despus cianosis. EF: Consciente y orientada, colaboradora, pero
inquieta y con importante dificultad respiratoria. Cianosis ligera de
piel y mucosas. Auscultacin pulmonar y cardiaca sin hallazgos
patolgicos. Abdomen blando y depresible, con apsitos de
laparotoma limpios y drenajes ya retirados. Extremidades sin
edemas.

RESPUESTA
a.- Nemonia.
b.- Edema agudo pulmonar
c.- TEP
d.- SDRA.
CLINICAL CASE
Following an open cholecystectomy and common duct exploration
for multiple gallstones in a 75 years old female, the cholangiogram
obtained 10 days post operatively reveals an impacted 1.5cm
gallstone at the ampulla. The bilirrubin is 5.0.
QUESTIONS
What is the procedure of choice?
ANSWERS
a.- Repeat the cholangiogram in 1 week
b.- T-tube flush with capmul 8210
c.- Open sphincteroplasty
d.- Choledochoduodenostomy

PREGUNTA
Cual es la complicacin ms probable en este caso?

CURSO ENARM CMN SIGLO XXI TEL: 36246001

Pharmed Solutions Institute

PGINA 437

MANUAL DE TRABAJO DEL CURSO ENARM CMN SIGLO XXI


CANCER DE PANCREAS
CIENCIAS BASICAS: El carcinoma pancretico es el tumor periampular ms comn; las manifestaciones son similares para todos. La gran mayora de
los tumores malignos son carcinomas ductales, adenocarcinomas. Es un tumor de gran agresividad biolgica. SALUD PUBLICA: Ocupa cuarto lugar
como causa de muerte en Estados Unidos. Edad promedio de inicio es de 60 aos, afecta ms sexo masculino (2:1). Menos de 20% de los carcinomas
es resecable al momento del diagnstico. Tras ciruga curativa: 20% sobreviven al ao y <5% a los 5 aos. Segundo tumor digestivo ms
frecuente. El T. digestivo con peor pronstico, es el ms letal y el fallecimiento se produce en 95% de los casos. Supervivencia media del no tratado:
4-6 meses. ADENOCARCINOMA: Surge de los conductos en 90% de los casos y de los acinos en 10%. A menudo la mayor parte del tumor es estroma
fibroso con una zona de pancreatitis. Cerca del 75 % se origina en la cabeza o el proceso uncinado del pncreas y produce obstruccin biliar que hace
posible el diagnstico ms temprano, 15% en el cuerpo y 10% en la cola del pncreas. El tumor puede
CLASIFICACION CANCER DE PANCREAS
invadir la vena porta o los rganos adyacentes o producir metstasis al hgado o al peritoneo. Los
Tumores slidos de pncreas exocrino:
adenocarcinoma (90%), carcinoma acinar y
ganglios linfticos son positivos en 90% de los pacientes. A veces los carcinomas ampular y duodenal se
pancreatoblastoma
diagnostican como pancreticos; que es posibles que sean pequeos al momento de detectarlos.
Tumores qusticos del pncreas exocrino:
DIAGNOSTICO: Clnica; cerca de 75% con carcinoma de la cabeza pancretica acude por ictrica
neoplasia mucinosa qustica, neoplasia mucinosa
obstructiva (82%), prdida de peso (92%) y dolor abdominal (72%). El dolor es sordo, como
papilar intraductal (TMPI*), neoplasia serosa
adolorimiento, medioepigstrico y a menudo se irradia a espalda, de intensidad severa y predominio
qustica, neoplasia slida seudopapilar.
Tumores del pncreas endcrino (de los islotes):
nocturno, empeora con las comidas y con el decbito supino. El dolor en espalda sugiere invasin
insulinomas,
gastrinomas,
glucagonomas,
retroperitoneal. Son frecuentes la anorexia, la fatiga y el prurito. En la exploracin se encuentra ictericia,
somatotatinomas, VIPomas, PPomas, carcinoides
hgado palpable (50-70%) y vescula palpable (30%, si no es sensible y hay ictericia, el diagnstico es de
Mesenquimticos
y
hematopoyticos:
cncer pancretico: Signos de Courvoisier). Puede aparecer tromboflebitis migratoria recurrente (signo
liposarcoma, leiomiosarcoma, fibrosarcoma,
de Trousseau). Esplenomegalia e hipertensin porta. El 20% de los pacientes padece diabetes de inicio
linfoma, Swwanoma
Secundarios: metstasis de carcinoma de mama,
reciente. En caso de carcinoma ampular el dolor es menos frecuente, a menudo clico, y la ictericia es
pulmn, renal y melanoma.
intermitente. Los tumores de cuerpo y cola pancreticos producen sntomas en una fase ms tarda de la
enfermedad, por lo que la etapa es ms avanzada al momento del diagnstico. Las metstasis al ganglio supraclavicular izquierdo (ganglio de
Virchow), umbilical (ganglio de la hermana Mara Jos) y del piso plvico (repisa de Blumer), indican enfermedad incurable. Metstasis: Frecuentes
en hgado, seguido de los ganglios linfticos regionales, peritoneo y pulmones. En las pruebas de laboratorio hay aumento en el nivel de bilirrubinas,
en el de fosfatasa alcalina y solo un aumento ligero en transaminasas. El CA19-9, con marcador tumor srico con sensibilidad de 80% y especificidad
de 90%, a veces ayuda. Muchos pacientes se valoran primero con ultrasonido abdominal, pero la TAC espiral es ms precisa. En algunos casos resulta
til la CEPRE, sobre todo cuando no se encuentra masa alguna en la TAC. La aspiracin con aguja fina est indicada cuando hay evidencia topogrfica
de irresecabilidad de manera que el diagnostico tisular puede comprobarse sin una operacin (la aspiracin negativa, no descrata la enfermedad).
TRATAMIENTO: Se har en funcin de poder o no extirparlos quirrgicamente: Resecable: 10-20%. Avanzado localmente (no resecable): 40%.
Metastsico: 40%. Es necesario corregir pronto el estado nutricional, la anemia y estado de volumen, adems de valorar la funcin renal. La
pancreatoduodenectoma (operacin de Wipple) representa la nica esperanza de curacin para los tumores de la cabeza pancretica y es ms til
en caso de carcinoma localizado, ya sea ampular, duodenal o del coldoco distal. Los pacientes con tumores irresecables obtienen cierto beneficio
de la colecistoduodenostoma o coledocoduodenostoma y gastroyeyunostomia paliativas. La inyeccin transoperatoria en el plexo celiaco alivia el
dolor. El paciente con alto riesgo obtienen beneficios con la CEPRE o el drenaje transheptico. La radioterapia combinada con 5-fluoracilo tiene
cierto valor como coadyuvante y paliativo. La Gemcitabina es el medicamento de quimioterapia ms comnmente utilizado para tratar el cncer de
pncreas. Seguimiento de los pacientes: Cada 3 meses los primeros 2 aos. Cada 4 meses en el 3er ao. Cada 6 meses en el 4to y 5to aos.
NEOPLASIAS QUSTICAS: El 5% de todos los tumores pancreticos, estos tumores se originan en las clulas de los conductos, tienen revestimiento
epitelial. Las neoplasias serosas (microquisticas), son benignas como el CISTADENOMA SEROSO compuesto por mltiples quistes pequeos de hasta
2 cm de dimetro con lquido claro, ms en mujeres, alrededor de los 65 aos, en general asintomtico localizacin ms frecuente en el cuerpo y
cola del pncreas. EL CISTADENOMA MUCINOSO es premaligno, lesiones multiloculares y papilares >5cm, ms frecuente en mujeres y puede
evolucionar a CISTADENOCARCINOMA 1% de los cnceres de pncreas, son tumores grandes de 20- 30cm, deben extirparse, ya que presentan
buena supervivencia a los 5 aos, son tumores de crecimiento lento, tienen mejor pronstico y deben tratarse en forma agresiva. A menudo se
manifiestan por molestia vaga, menos de 10% presentan ictericia con anorexia y prdida de peso. Todas las lesiones macroquisticas deben
extirparse. NEOPLASIA INTRADUCTAL PAPILAR MUCINOSA: Proliferacin de clulas mucinosas en forma de papilas a nivel ductal con
hiperproduccin de moco. Ms en hombres, 68 aos. Sntomas sugestivos de pancreatitis crnica. En la cabeza y proceso uncinado, aunque puede
afectar difusamente la glndula pancretica. 30-60% son malignas en el momento de su diagnstico. Asociada a otros tumores malignos en un 2336% de los casos. NEOPLASIAS ENDOCRINAS: Se originan en las clulas de los islotes pancreticos, representan aproximadamente el 15% de las
neoplasias pancreticas, casi todas son malignas. Segregan hormonas, dependiendo de las clulas de las que proceden son muy raros. Ms
frecuente el gastrinoma, que adems representa el 10% de todos los tumores endocrinos gastropancreticos, el segundo en frecuencia es el
insulinoma. INSULINOMA: Tumor pancretico endocrino funcionante ms frecuente 50-60%, proviene de clulas pancreticas, la mayora en la
cabeza del pncreas. 80% benignos. Habitualmente se presenta en la 5ta y 6ta dcada de la vida, ms en mujeres que en hombres (2:1). La
hiperinsulinemia produce hipoglucemia grave y origina convulsiones, depresin y coma. Los sntomas desaparecen poco despus de administrar
glucosa. Los criterios diagnostico clsicos, la triada de Wipple, incluyen hipoglucemia en ayuno (<50mg/dl) durante los ataques, cambios en el SNC y
sntomas de hipoglucemia desencadenado por el ayuno, as como reversin de los cambios con administracin de glucosa. La ubicacin del tumor se
facilita con la angiografa, toma de muestras venosas selectivas, imgenes por TAC o centellografa con octretido. El tratamiento es quirrgico
(Pancreatectoma distal, dejando 20-30% del pncreas), excepto en enfermedad metastsica avanzada. GASTRINOMA: La triada original de ZollingerEllison incluye ulceras ppticas fulminantes de localizacin atpica, hipersecrecin gstrica extrema y un tumor de clulas no beta de islote
pancretico. El gastrinoma puede comenzar como una enfermedad sencilla y terminar con complicaciones graves como perforacin, obstruccin,
hemorragia e intratabilidad. La enfermedad no responde a tx., medico, ni quirrgico. Es posible que los pacientes presentes diarrea de gasto alto con
esteatorrea. Dx., hay hipergastrinemia en ayuno (>200pg/ml). Su potencial maligno es >90%. El crecimiento es lento y la metstasis se produce en
etapa tarda. Con frecuencia las lesiones se encuentran en el tringulo del gastrinoma: 1) unin de los conductos cstico y coldoco, 2) unin de la
segunda y tercera porcin del duodeno, 3) unin del cuello y el cuerpo pancreticos. VIPomas: Tumor productor de pptido intestinal vasoactivo,
procedentes de las clulas D1. Ocasiona diarrea, clera pancretico y un sndrome compuesto por diarrea acuosa, hipopotasemia y aclorhidria
gstrica. Cerca de 50% corresponde a malignos, ms comn en hombres, promedio 45 aos, de gran tamao. Provoca el Sx de Werner-Morrison o
clera pancretica: Diarrea hipersecretora + rubicundez (20%) + hipokalemia + aclorhidria. 50% de los pacientes con hiperglucemia (glucogenlisis
heptica inducida por el VIP y la hipopotasemia). Diagnstico: Concentraciones sricas de VIP >200 pg/ml. TAC y USG. Tratamiento: Enucleaciones,
pancreatectomas distales laparoscpicas. GLUCAGONOMAS: Procedentes de clulas A, ms frecuentes en mujeres, entre 50-70 aos de edad, ms

CURSO ENARM CMN SIGLO XXI TEL: 36246001

Pharmed Solutions Institute

PGINA 438

MANUAL DE TRABAJO DEL CURSO ENARM CMN SIGLO XXI


en cuerpo y cola. Causan lesiones cutneas (eritema necroltico migratorio, muy pruriginoso), diabetes, glositis, prdida de peso depresin y
trombosis venosa. Las tres cuartas partes son malignos. El mejor tratamiento es la reseccin. Sx de las 4 Ds (Diabetes, Depresin, Dermatitis, Deep
vein trombosis). Diagnstico: Glucagn srico>500pg/ml. Tambin puede detectarse hipoalbuminemia, hipoaminoacidemia, anemia e
hipocolesterolemia. TAC. El mejor tratamiento es la reseccin, Pancreatectoma distal con preservacin esplnica o pancreatoduodenectoma.
SOMATOSTINOMAS: Extremadamente raros, en pncreas proximal, 60% mpula (en clulas D). Se manifiestan por diabetes, diarrea, esteatorrea,
aclorhidria, calculos biliares, malabsorcin, y dolor abdominal. Por lo general se tratan con estreptozocina, dacarbacina y doxirrubicin. Diagnstico:
concentraciones sricas de somatostatina >100 pg/dl. TAC, RNM. Tratamiento reseccin del tumor. PPoma: Es productor de polipptido
pancretico, es el tumor no funcionante ms frecuente. No se asocian con sndromes clnicos de hiperfuncin hormonal. Entre la 5ta-6ta dcada de
la vida. En el 50% se ubican en la cabeza, proceso uncinado y cuello del pncreas.
CASO CLINICO CANCER DE PANCREAS.
Se trata de paciente masculino de 59 aos de edad con
antecedentes de DM 2 y tabaquismo positivo, el cual refiere diarrea
crnica de ms de 6 meses de evolucin, disminucin de peso del 15
%, ha disminuido su ingesta de alimento ya que tiene sensacin de
plenitud y vomito en varias ocasiones, se observa ictericia y piel
verdosa, refiere heces grasosas y flatulencias, fue diagnosticado
previamente con sndrome de Peutz - Jeghers.
PREGUNTA
Cul es la manifestacin clnica ms importante para distinguir
cncer pncreas de cabeza vs cuerpo y cola?
RESPUESTA
a.- Dolor de espalda.
b.- Coluria y acolia.
c.- Constipacin.
d.- Dispepsia.
CASO CLINICO
Acude paciente de 61 aos de edad el cual presenta prdida de peso
y sensacin de plenitud abdominal, cuenta con antecedentes de
consumo de alcohol y tabaco de forma social sin llegar a la
embriaguez, aproximadamente cada semana, cuenta con
antecedentes de cncer en la familia sin especificacin, adems
refiere que hace 20 aos sufre accidente automovilstico con
transfusin sangunea, hace 5 aos fue diagnosticado con hepatitis
C, actualmente presenta distencin abdominal, sobrecarga de
volumen y ascitis, confirmando cirrosis desde entonces, se
encuentra con tratamiento con espironolactona mas furosemida, se
realiza radiografia abdominal donde se aprecia lesin en lbulo
derecho de 5 cm aproximadamente, se indica TAC donde se
confirma lesin de 4,6 cm en lbulo derecho cercana a pediculos
portales principales, se solicita fetoproteina alfa con 384 ng/ml de
resultado, se programa biopsia percutnea.
PREGUNTA
Considerando los antecedentes y el cuadro clnico actual, cual es el
diagnostico ms probable.
RESPUESTA
a.- Adenocarcinoma heptico.
b.- Carcinoma hepatocelular.
c.- Carcinoma metastasico.
d.- Colangiocarcinoma.
CASO CLINICO
Varn 68 aos, con antecedente de hipertensin, miocardiopata
hipertrfica del VI, y FA permanente controlada con digoxina. AHF:
Sin inters. De su padecimiento actual refiere debilidad y sudoracin
leves que ceden con la ingesta de alimentos, se presentan 1-2
veces/mes desde hace 3 aos, no interferan con su vida diaria. La
clnica aparece mientras realiza algn esfuerzo fsico y ha tenido una
ganancia ponderal de 10Kg. Fue trado al servicio de urgencias por
un cuadro de inicio con visin borrosa, diplopa, prdida de
conciencia con sudoracin profusa y temblores generalizados, de
unos 10 minutos de duracin. En la exploracin fsica nicamente
destacaba su regular estado general, palidez cutneo-mucosa y un
soplo sistlico, ms intenso en foco artico. Resto de la exploracin
sin hallazgos de inters, incluida la exploracin neurolgica. P: 69Kg;

CURSO ENARM CMN SIGLO XXI TEL: 36246001

T: 162cm; IMC: 26.29Kg/m2. BQ: Cr 1.7 mg/dl, GB 87mg/dl, resto


normal. Estudio hormonal: IGF-I 153ng/ml; GH 2.33 ng/ml, T4L
1ng/dl; TSH 2.3 mcUI/ml, Cortisol basal 12.9 mcg%., Glucosa
36mg/dl, Insulina: 8.1 mUI/ml. El paciente no tena acceso a ningn
tipo de medicacin hipoglucemiante.
PREGUNTA
Cul es la conducta teraputica ms adecuada a seguir en este
momento?
RESPUESTA
a.- Anticonvulsivo y solucin glucosada al 5%
b.- Iniciar perfusin de glucosa al 5% y aportes extras de glucosa al
50%
c.- Iniciar vasodilatador y solucin fisiolgica al 0.9%
d.- Iniciar perfusin con glucosa al 50%
PREGUNTA
Cul es la conducta teraputica quirrgica ms adecuada a seguir
en este momento?
RESPUESTA
a.- Pancreatodudenectomia (dejando 50-60% del pncreas)
b.- Pancreatectoma distal (dejando 50-60% de pncreas)
c.- Pancreatodudenectomia (dejando 20-30% del pncreas)
d.- Pancreatectoma distal (dejando 20-30% de pncreas)
PREGUNTA
Al hacer el estudio histopatolgico. Cul es el porcentaje de
malignidad ms probable?
RESPUESTA
a.- 80%
b.- 10%
c.- 50%
d.- 30%
CASO CLINICO
Masculino de 57 aos, refiere padre con antecedente de cncer de
hipfisis. Hipertenso de 10 aos de evolucin tratada con enalapril
10 mg, diabtico tratado con metformina 850 mg, quien present
hace 2 aos pancreatitis aguda grave, de etiologa no precisada. Para
esa fecha se efectu endoscopia digestiva superior que report
gastroduodenitis erosiva y lceras gstricas y duodenales de 1cm
cada una, las biopsias revelaron ausencia de Helicobacter pylori, las
cuales fueron tratadas con IBP por 6 meses. Actualmente el
paciente acude a urgencias por presentar dolor abdominal de fuerte
intensidad en epigastrio, tipo urente, irradiado en banda, con
nuseas y vmitos en mltiples oportunidades as como
evacuaciones diarreicas abundantes y frecuentes mocosas,
esteatorrea. Refiere perdida de 5Kg de peso, y cuadros diarreicos
anteriores, pero leves. Al examen fsico de ingreso se encontraba
con TA: 110/72 mmHg, FC: 106x, FR: 16x, levemente deshidratado,
hemodinmicamente estable, con palidez muco-cutnea moderada.
Cardiopulmonar sin alteraciones. Abdomen globoso, distendido, con
peristaltismo lento, dolor difuso con nfasis en epigastrio y
mesogastrio. En los laboratorios de ingreso se encontr: Leucocitos:
11.6 103 U/L, Neutr: 84%, Linf: 9%, Amilasa 1675mg/dl, Lipasa 687
mg/dl, gastrina reportaron 1680 pg/ml, AST: 46 U/L, ALT: 31 U/L,
ALP: 102 U/L, PCR: 8.3 mg/dl. Se plantean los diagnsticos de ingreso

Pharmed Solutions Institute

PGINA 439

MANUAL DE TRABAJO DEL CURSO ENARM CMN SIGLO XXI


pancreatitis crnica reagudizada, enfermedad lcero pptica,
hipertensin arterial y diabetes mellitus.
PREGUNTA
Cul es la conducta diagnostica ms adecuada a seguir?
RESPUESTA
a.- Tomografa axial computada
b.- Resonancia magntica
c.- Endoscopia de abdomen
d.- Radiografia de abdomen
PREGUNTA
Dnde esperaramos encontrar metstasis ms probablemente en
este paciente?
RESPUESTA
a.- Hgado y ganglios linfticos regionales
b.- Bazo, huesos
c.- Huesos, hgado
d.- Ganglios linfticos regionales, hueso
PREGUNTA
Despus de 3 meses el paciente inicia con hiperparatiroidismo, se
realizan estudios en los cuales se detecta hipercalciuria y litiasis
renal. As como intensificacin de sntomas previos. Cul es la
conducta diagnostica ms adecuada a seguir?
RESPUESTA
a.- Resonancia magntica hipofisiaria, TAC abdominal, rastreo
gentico a familiares directos
b.- Ultrasonido renal, TAC abdominal rastreo gentico a familiares
directos
c.- QS, resonancia magntica, rastreo abdominal
d.- Rastreo gentico a familiares directos, TAC de crneo.
CASO CLINICO
Se trata de paciente masculino de 65 aos de edad, con antecedente
de hipertensin arterial, diabetes y dislipidemia, de
aproximadamente 10 aos de evolucin en tratamiento irregular. En
los ltimos aos curs con cuadros autolimitados de diarrea afebril.
Clnicamente en los ltimos tres meses present en forma secuencial
episodios de dorsalgia moderada que no cedan con
antiinflamatorios y relajantes musculares y que se intensificaron
progresivamente. Adems, present dolor sbito nivel de
hipocondrio derecho, mesogastrio y epigastrio de moderada
intensidad y escasa mejora con antiespasmdicos. En el ltimo mes,
ictericia de piel, baja de peso progresiva, dispepsia a grasas y prdida
del apetito. Al examen fsico se observa un paciente adelgazado con
ictericia severa de piel y mucosas, sin presencia de estigmas
hepticos, se palpa esplenomegalia. Exmenes auxiliares: un mes
antes del ingreso tuvo una ecografa renal con resultado normal y
examen completo de orina evidencindose glucosuria: 3+,
hemoglucotest: 330 mg/dl, Hb A1c : 8,7 % , creatinina: 0,65 mg/dl,
radiografa de trax: normal. Al ingreso, hemoglobina: 14,4 g/dl,
leucocitos: 10,380 cel/mm3, linfocitos: 15%, eosinfi los: 2%,
plaquetas: 363,000 cel/mm3, TP: 13,9 seg, glucosa 149: mg/dl, TGO:
290 UI/L, TGP: 660 UI/L, fosfatasa alcalina: 282 UI/L, bilirrubina total:
8,5 mg/dl, bilirrubina directa: 7,1 mg/dl, bilirrubina indirecta: 1,4
mg/dl
PREGUNTA
Cul es la conducta teraputica quirrgica ms adecuada a seguir
en este momento?
RESPUESTA
a.- Pancreatodudenectomia cefalica (dejando 50-60% del pncreas)
b.- Pancreatectoma distal (dejando 50-60% de pncreas)
c.- Pancreatodudenectomia ceflica (dejando 20-30% del pncreas)
d.- Pancreatectoma distal (dejando 20-30% de pncreas

CURSO ENARM CMN SIGLO XXI TEL: 36246001

PREGUNTA
Qu conducta teraputica coadyuvante es la ms adecuada para
este paciente?
RESPUESTA
a.- 5-fluorouracilo
b.- Gemcitabina
C.- Estreptomicina
d.- Adriamicina
PREGUNTA
Cul es el pronstico ms probable para este paciente a 5 aos?
RESPUESTA
a.- 4%
b.- 10%
c.- 20%
d.- 30%
CASO CLINICO
Mujer de 29 aos de edad acude con su medico de atencion primaria
por dolor intenso en el cuadrante inferior izquierdo que ha
persistido mas de una hora. Califica el dolo como 10/10 y no puede
sentrase erguida sobre la mesa de exploracin. Seala que endias
previos experimento esxpasmos en la espalda, pero que no fueron
intensos y no busco atencion medica. Sus antecedentes
ginecolgicos son evodentes por amenorrea de tres meses de
evolucin; no hay embarazo. La semana pasada se le diagnostico
enfermedad ulcerosa pptica, y recibi una prescripcin de
omeprazol que ha tomado segn las indicaciones. No puede
acostarse en posicin plana sobre la mesa de exploracin y al
principio rehsa a que el medico palpe su abdomen. TA
110/70mmHg, FC120 lpm, FR 19rpm. El examen es marcado por
hipersensibilidad en los cuadrantes superior e inferior izquierdos, asi
como en la parte baja de la espalda. Los ruidos intestinales son
normales. Los resultados de laboratorio importante son: glucosa
100mg/dl, Ca 12.3mg/dl, Gastrina 253pg/ml.
PREGUNTA
Cul es el diagnostico mas probable para esta paciente?
RESPUESTA
a.- Cancer gstrico
b.- Sindrome de MEN1
c.- Ulcera perforada
d.- Cancer de pancreas
PREGUNTA
Qu estructura es menos probable que este daada, de acuerdo al
cuadro clnico?
RESPUESTA
a.- Pancreas
b.- Estmago
c.- Paratiroides
d.- Higado
PREGUNTA
Qu probabilidad hay de que el paciente desarrolle cncer
pancretico?
RESPUESTA
a.- 20%
b.-30%
c.-50%
d.-80%

Pharmed Solutions Institute

PGINA 440

MANUAL DE TRABAJO DEL CURSO ENARM CMN SIGLO XXI


COLANGIOCARCINOMA
CIENCIAS BASICAS: Es un tumor maligno originado en el epitelio de los conductos biliares intra o extrahepticos, se puede ubicar a cualquier nivel de
la via biliar, desde los canalculos hasta la ampolla de Vater. Cerca de dos terceras partes de las lesiones, se localizan en los conductos proximales, a
menudo en la confluencia de los conducto heptico derecho e izquierdo) La mayora de los colangiocarcinomas son del tipo histolgico
adenocarcinoma ductal, sin embargo, suelen verse otros como los adenocarcinomas papilar, mucinoso, mucoepidermoide y el cistoadenocarcinoma.
SALUD PUBLICA: Es poco frecuente, representa menos del 2% de todos los tumores malignos de diagnstico reciente. La incidencia en Estados
Unidos es baja, de 1.0/100.000 por ao y es un poco ms alta en Israel y Japn. Se presenta especialmente despus de la sexta dcada de la vida y es
ligeramente ms frecuente en hombres que en mujeres. CLASIFICACION: Anatmicamente distinguimos el colangiocarcinoma intraheptico (20% a
25%), el perihiliar (50% a 60%), el extraheptico distal (20% a 25%) y el multifocal (5%). El extraheptico tambin se clasifica en el del tercio superior
de la va biliar (heptico comn, confluencia hiliar o hepticos derecho e izquierdo), el del tercio medio (coldoco hasta un plano dado por el borde
superior del duodeno) y el del tercio inferior (desde el plano del borde superior del duodeno hasta la ampolla de Vater). Se denomina TUMOR DE
KLATSKIN al colangiocarcinoma situado en la confluencia hiliar. Segn su patrn de crecimiento, puede ser exofitico, polipoideo e infiltrativo. El
tumor de Klatskin es de tipo infiltrativo con invasin maligna periductal. El polipoideo tiene crecimiento endoluminal. PATOGENIA: La etiologa no
est bien definida, hay algunos procesos patolgicos predisponentes, entre ellos la colangitis esclerosante primaria, la litiasis intraheptica, la
enfermedad de Caroli y malformaciones como la atresia de va biliar y los quistes de coldoco. Tambin existe asociacin con txicos cancergenos
de origen industrial como digoxinas, nitrosaminas y asbesto . Adems, se le asocia a infestacin crnica de la va biliar con parsitos endmicos del
sudeste asitico como clonorchs sinensis y opisthorchis viverrin. La infeccin crnica con salmonella typhi sera otro factor predisponente. Es
dudoso que la colelitiasis y la colecistectoma previa predispongan al desarrollo de colangiocarcinoma. Las alteraciones moleculares tambin se han
asociado al colangiocarcinoma, como la inactivacin de oncosupresores (p53, pl6, bel-2) y la mutacin de oncogenes (K-ras, c-myc, c.erbB-2). Sin
embargo, no son hallazgos especficos. DIAGNOSTICO: Clnica; dolor en hipocondrio derecho, ictericia y baja de peso. El signo ms importante es la
ictericia progresiva, presente en alrededor de 90%, muchas veces precedida de prurito. Tambin hay dolor no muy intenso en epigastrio e
hipocondrio derecho. El signo de Courvosier-Terrier puede estar presente en los colangiocarcinomas ubicados en el tercio inferior del coldoco. En
algunos pacientes asintomticos, las fosfatasas alcalinas estn aumentadas de 1 a 5 veces y las transaminasas de 1 a 2 veces. Los marcadores
tumorales CA 19-9, CA-125 y CEA estn aumentados en el colangiocarcinoma en 85%, 40% a 50% y 30%, respectivamente. No obstante, estos
marcadores tumorales tambin se elevan en otras patologas malignas y en lesiones hepticas graves. Un estudio comparativo entre CA 19-9 y CEA
realizado por autores chinos, demuestra que el CA 19-9 es ms til para el diagnstico del colangiocarcinoma. Actualmente el diagnstico se ha
facilitado por la disponibilidad de variados procedimientos imagenolgicos y endoscpicos. La colangiografa, localiza el sitio exacto de la
obstruccin. El hgado y los ganglios linfticos son los sitios ms comunes de metstasis. TRATAMIENTO: Lo ideal es la reseccin quirrgica completa
del tumor con mrgenes histolgicos negativos es la nica terapia curativa. Desafortunadamente, la mayora de las veces el tumor es irresecable.
Para esos casos hay posibilidad de paliacin mediante la colocacin de endoprtesis por va endoscpica o percutnea. Los pacientes con
colangiocarcinoma hiliar en un bajo porcentaje alcanzan a ser tratados con ciruga potencialmente curativa, fundamentalmente por tres motivos:
por diagnstico tardo, por infiltracin neoplsica de la vasculatura del hilio heptico, terapia de eleccin la reseccin de la va biliar con o sin
reseccin heptica ms linfadenectoma y anastomosis biliodigestiva. La mortalidad operatoria de las diferentes series de ciruga resectiva es de
alrededor de 10%. El trasplante heptico, ltimamente ha surgido como otra opcin vlida para tratar pacientes con colangiocarcinoma hiliar y
perifrico considerados irresecables pero no diseminados. La curacin de los tumores de la parte distal del coledoco mejoro con la linfadenectomia
radical y la pancreatoduadenectoma (Procedimiento de Wipple).
CASO CLINICO COLANGIOCARCINOMA
Se trata de paciente femenino de 56 aos de edad la cual acude a
consulta por malestar generalizado, fatiga y coloracin amarillenta,
la paciente es originaria del distrito federal, fumadora durante 15
aos as como consumo de alcohol tipo social cada fin de semana,
cuenta con antecedente de intolerancia a la glucosa con IMC de 27 la
cual se encuentra con tratamiento con dieta, ejercicio y metformida
con resultados leves, el padecimiento actual inicia hace una semana
caracterizado por deposiciones con perdida de consistencia y
coloracin disminuida, palidas, agrega prurito generalizado,
coloracin amarillenta de piel y escleras, a la exploracin fsica se
encuentra aptica, afebril, con signos vitales dentro de rango
normal, se palpa hepatomegalia dudosa, se ingresa a observacin
donde se realiza estudios de laboratorio con AST 119, ALT 158,
Fosfatasa Alcalina de 476 UI/L, bilirrubina total de 23 mg/dl, y
bilirrubina directa de 20 mg/dl, se indica ultrasonografia no se
observa vesicula biliar, se observa dilatacin de conductos biliares
intrahepaticos, pero no se muestra dilatacin del coldoco, resto de
estructuras sin datos por agregar.
PREGUNTA
Cul es su impresin diagnostica ms probable con los estudios
realizados?
RESPUESTA
a.- Cncer pancretico.
b.- Cncer de vesicula.
c.- Cncer heptico.
d.- Colangiocarcinoma.
CASO CLINICO

CURSO ENARM CMN SIGLO XXI TEL: 36246001

Se trata de paciente femenino de 47 aos de edad, inicia


padecimiento hace 2 aos caracterizado por dolor en cuadrante
superior derecho, que se presentaba posterior a la ingesta de
alimentos, ha sido tratada con analgesicos y antiespamodicos
diversos sin embargo desde hace 6 meses ya no han sido
controlados e incluso se encuentra con un estado nauseoso
continuo, tolera poco alimento, a la exploracin se observa IMC 17,
se observa ictericia importante.

PREGUNTA
Cul es la conducta a seguir ms adecuada?
RESPUESTA
a.- Endoscopia.
b.- Serie gastroesofagica.
c.- USG.
d.- Radiografa simple de abdomen.
CASO CLINICO
Se trata de paciente femenino de 21 aos de edad la cual presenta
malformacin cstica del coldoco la cual se identifico por USG
cuando la paciente refiri dolor en cuadrante superior derecho.
PREGUNTA
Se decide realizar reseccin quirrgica. Cual es el motivo ms
importante para la decisin?
RESPUESTA
a.- Degeneracin maligna.
b.- Colangitis recurrente.
c.- Cirrosis biliar.
d.- Coledocolitiasis.

Pharmed Solutions Institute

PGINA 441

MANUAL DE TRABAJO DEL CURSO ENARM CMN SIGLO XXI


PANCREATITIS AGUDA Y CRNICA:
PANCREATITIS AGUDA (PA). CIENCIAS BASICAS: Enfermedad sistmica caracterizada por un proceso inflamatorio agudo no bacteriano que resulta
de la liberacin de las enzimas pancreticas dentro de la propia glndula, que puede comprometer por continuidad otros tejidos y rganos vecinos.
Usual curso benigno. PAL: mnima disfuncin multiorgnica, evolucin sin complicaciones. PAG: presencia de falla orgnica o sistmica y/o presencia
de complicaciones locales. SALUD PUBLICA: 20% desarrolla enfermedad severa. 20% de mortalidad. 4-6ta. Dcada de la vida. Cerca de 40% de los
casos se debe a clculos biliares, otro 40% se debe a alcoholismo.
PATOGENIA: El pncreas normalmente secreta una gran cantidad de
enzimas, entre ellas la tripsina, la quimiotripsina, las amilasas, lipasas,
elastasas, carboxipeptidasa A y B, etc. La gran mayora de ellas son enzimas
lticas que normalmente se secretan en forma de precursores inactivos, los
cuales junto con inhibidores enzimticos presentes en el jugo pancretico,
protegen al pncreas de su autodigestin. Los eventos iniciales patognicos
de la Pancreatitis Aguda no estn del todo aclarados. Existen evidencias de
que la presencia de uno o ms de los siguientes factores, aumento de la
presin intraductal en el sistema excretor pancretico (litiasis biliar,
tumores, pncreas divisum o anular), reflujo de bilis al Wirsung,
hipertrigliceridemia aguda o preexistente, frmacos (aziatropina,
clortiazida, estrgenos, furosemida, sulfamidas, tetraciclinas), infecciones
(hepatitis, parotiditis, CMV, candida, scaris), dao isqumico, post-CEPRE,
disrupcin de algn conducto excretor (trauma), etc., desencadena una
cascada inflamatoria que se asocia a la activacin enzimtica intraglandular, con las consecuencias de dao microvascular, trombosis, necrosis
tisular, saponificacin del tejido graso, liberacin de radicales libres y eventualmente gatillamiento de una respuesta inflamatoria sistmica. PA Y
ALCOHOL: La ingesta alcohlica excesiva de tipo inveterada es un factor asociado a crisis de PA a repeticin, que si bien el primer episodio puede ser
de gravedad, las crisis posteriores son clnicamente menos importantes y que llevan al cuadro de la pancreatitis crnica recurrente. En las teoras se
ha postulado: (1) Un aumento de la secrecin gstrica que conduce a un aumento exagerado de la secrecin pancretica; (2) Inflamacin duodenal y
periampular con obstruccin de los conductos bilio-pancreticos; (3) La hiperlipidemia aguda inducida por el alcohol. En la mayora de los casos la PA
se produce 24 a 48 horas despus de la ingesta, siendo poco frecuente el encontrar niveles altos de alcoholemia simultneamente al evento clnico
de la PA. 4. P.A. e hipertrigliceridemia: DIAGNOSTICO: Clnica; produce dolor intenso epigstrico (estiramiento de la capsula pancretica)
transfictivo, que se irradia a la espalda, se alivia al sentarse y a menudo se acompaa de nausea intensa, puede haber relacin con ingesta de
alcohol o grasas. Tambin se observa sensibilidad abdominal superior y defensa muscular. Alrededor de 90% de los pacientes tiene fiebre,
leucocitosis y taquicardia. El leo es usual y tambin es posible que ocurra choque por secuestro de
CRITERIOS DE RANSON PARA PA
lquido y depresin miocrdica. La ictrica se presenta en 20-30% de los pacientes, por obstruccin o
Presentes al momento del ingreso:
Edad >55aos
compresin del coldoco. En ocasiones hay espasmo carpopedal por hipocalcemia. Alrededor de 1% de
Cuenta leucocitaria >16 000/l
los pacientes tiene sangre retroperitoneal alrededor de la cicatriz umbilical (Signo de Cullen) o en los
Glucosa sangunea >200mg/dl
flancos (Signo de Grey- Turner). Pancreatitis + Fiebre persistente + Resp. Inflamatoria sistmicapensar
Deshidrogenasa lctica srica >350 UI/L
en absceso o infeccin. Laboratorio y gabinete: pueden presentar hiperamilasemia (3 veces lo normal,
AST >250UI/dl
Se desarrollan durante las primeras 48 hrs
aumento 2-5 das), manifestacin muy inespecfica (elevada en colangitis, colecistitis, ulcera pptica
Descenso del hematocrito >10%
perforada, salpingitis, IRC, etc). La determinacin srica de lipasa es ms especfica (aumenta a partir de
Aumento >8mg/dl en BUN
las 72 h, regresa lentamente, mejor prueba diagnstica). La depuracin urinaria de amilasa puede ser
Calcio srico menos de 8mg/dl
til. El nivel de calcio disminuye a veces (<7.5mg/dl, mal pronstico, secuestro de calcio en las reas de
PO2 arterial >60mmHg
necrosis grasa). Hiperglucemia (aumento transitorio, liberacin de glucagn aumentada, glucemia > 200
Deficiencia de base >4mEq/l
Secuestro estimado de lquido >600ml
mg/dL signo de mal pronstico = necrosis extensa). PCR: su elevacin va en relacin con el mal
0-2 = 0.9% de mortalidad; 3-4 = 16% ;
pronstico. La radiografa puede mostrar un asa centinela con aire intestinal. Paciente con diagnstico
5-6 = 40% ; >6 = 91 de mortalidad
de PA: hacer ecografa abdominal (litiasis biliar) en las primeras 24-48 horas. La imagen por TAC ayuda a
predecir la gravedad y a establecer el diagnostico de las complicaciones. ndice de severidad por TAC, CRITERIOS DE BALTHAZAR: A Pncreas normal
(0 puntos). B Aumento de tamao focal o difuso del pncreas (1 punto). C Inflamacin del pncreas y/o grasa peri/pancretica (2 puntos). D
Coleccin pancretica nica (3 puntos). E dos o ms colecciones peripancreticas y/o gas retroperitoneal (4 puntos). Porciento de necrosis: 0% =0
puntos, 30% = 2 puntos, 30-50% = 4 puntos, >50% =6 puntos. Suma de puntos en TAC = ndice de severidad. 0-3 bajo, 4-6 medio, 7-10 alto.
TRATAMIENTO: La reposicin de las perdidas hidroelectrolticas es parte crucial de la atencin a estos pacientes, as como la vigilancia de volumen
vascular, las valoraciones repetidas de hematocrito y electrolitos y el reposo intestinal. El uso rutinario de antibiticos en pancreatitis no complicada
no se justifica. La colecistectoma est indicada en la pancreatitis biliar. El lavado peritoneal puede disminuir las complicaciones cardiopulmonares,
pero no reduce la mortalidad. La intervencin quirrgica es necesaria en casos de clculos impactados, desbridacin de necrosis y drenaje de
abscesos. COMPLICACIONES: La complicacin ms frecuente es el pseudoquiste (coleccin liquida), que suele presentarse despus de 2-3 semanas,
puede ser nico o mltiple, tx., drenaje quirrgico, el 50% se resuelven de manera espontnea a las 6 semanas. Necrosis pancretica temprana y
tarda (Dx: puncin por aspiracin, tratamiento: desbridamiento quirrgico). Absceso pancretico (4-6 semanas, Dx: puncin aspiracin, tratamiento:
drenaje percutneo). Infeccin: principal causa de morbimortalidad, aparece de 2-3 semanas despus, deben usarse antibiticos que penetren en
pncreas, primera eleccin imipenem, tambin cefalosporinas de tercera generacin y quinolonas, mnimo 14 das, o hasta que no haya
complicaciones sistmicas. Las complicaciones sistmicas incluyen SDRA, insuficiencia renal y depresin miocrdica. PANCREATITIS CRONICA:
CIENCIAS BASICAS: La definicin es vaga, pero por lo general incluye cambios secundarios a los episodios repetidos de pancreatitis aguda. El
pncreas se vuelve, pequeo, indurado y nodular, con acinos e islotes rodeados por tejido fibroso. Se observa estrechamiento y dilacin de los
conductos; la calcificacin es frecuente, los cambios estructurales son irreversibles y progresivos. SALUD PUBLICA: Incidencia oscila entre 5-10 casos
nuevos anuales por 100,000 hab y prevalencia de 30-40 por 100,000 hab. Algunos estudios elevan estas cifras hasta 5% de la poblacin general.
CLASIFICACION: TIGAR-O; Toxica-metablica que es la ms frecuente porque incluye el alcohol (70-90%), tambin la hipercalcemia, y algunos
mencionan la falla renal crnica. Idioptica alcanza de 10-30% se han agrupado en una forma juvenil que se presenta en las primeras dcadas de la
vida y una forma en 4ta-5ta dcada. Gentica, se han descrito algunas mutaciones, en el tripsinogeno catinico, CFTR, SPINK1. Autoinmune es una
entidad poco frecuente con hallazgos clnicos y paraclnicos definidos e histolgicamente su caracterstica es el extenso infiltrado linfoplasmocitario.
Recurrente, en el que se incluyen pacientes que posiblemente debido a mltiples episodios de pancreatitis aguda pueden conducir a pancreatitis
crnica, sin embargo existe controversia. Obstructiva, plantea el dao primario sobre los conductos pancreticos, posiblemente por dao

CURSO ENARM CMN SIGLO XXI TEL: 36246001

Pharmed Solutions Institute

PGINA 442

MANUAL DE TRABAJO DEL CURSO ENARM CMN SIGLO XXI


inmunolgico sobre el epitelio, en forma similar a los que se observa en la colangitis esclerosante primaria. Pncreas divisum, deficiencia del esfnter
de Oddi, traumatismos. PATOGENIA: La clulas estrelladas tienen un papel en la fibrosis pancretica, en el pncreas se localizan en la porcin
exocrina (espacios periacinar, perivascular), en condiciones normales, por sus propiedades
contrctiles ayudan a regular, las presiones en estos compartimentos, adems pueden contribuir
a mantener la matriz extracelular y tienen capacidades limitadas de migracin y proliferacin. De
DAO POR
acuerdo a la teora conocida como necrosis-fibrosis, se plantea que la necrosis y la inflamacin
ALCOHOL
pancretica son procesos que activan a las clulas estrelladas, por factores denominados
autocrinos por medio de molculas proinflamatorias que adems aumentan la secrecin de
colgeno y la expresin de TGF-B1 y endotelina, que estimulan la migracin, la contraccin y la
liberacin de sustancias proinflamatorias, perpetuando sus efectos aun cuando el estmulo inicial
haya terminado, favoreciendo el desarrollo progresivo de fibrosis, el dao repetido y persistente
en la pancreatitis crnica lleva a una activacin continua de clulas estrelladas que finalmente
conduce a fibrosis de la glndula y perdida de sus funciones exocrinas. Existe una evidencia clara
sobre la capacidad del alcohol para inducir lesin pancretica directa. Los alcohlicos ingieren
dietas deficientes en numerosos micronutrientes, especialmente selenio y zinc, lo que podra ser
origen de mecanismos celulares de lesin pancretica inducidos por la accin de radicales libres
de oxgeno. Adems tambin produce fenmenos de hipertensin intraductal, disminucin del
flujo sanguneo pancretico, toxicidad directa sobre la clula acinar, cambios en la sntesis
proteica, incremento de la respuesta inflamatoria y estimulacin de la fibrogenesis pancretica.
DIAGNOSTICO: Clnica; Los sntomas comprenden dolor epigstrico o en espalada continuo o intermitente, anorexia y maldigestin manifestada por
prdida de peso, esteatorrea. La diabetes puede desarrollarse a lo largo de la enfermedad y se caracteriza por destruccin de clulas productoras de
insulina y productoras de glucagn; la coexistencia de este dao aumenta el riesgo de hipoglicemia, por la deficiencia concomitante de sntesis de
glucagn. Durante los ataques agudos hay vomito. A menudo se forman pseudoquistes. Tambin son frecuentes las alteraciones de la personalidad.
Segn su etiologa pueden ocurrir otras manifestaciones como fenmenos de autoinmunidad u hepatopata. Laboratorios y gabinete: la presencia de
cambios morfolgicos en pruebas de imagen y/o en la demostracin de insuficiencia pancretica. La CPRE y las imgenes con TAC, RMN ayudan y es
probable que muestren dilatacin ductal, clculos y estrechamientos. Los niveles sricos de amilasa y lipasa tienen poco valor. La calcificacin
pancretica es patognomnica. Histolgicamente se requiere la presencia de fibrosis y atrofia acinar que se acompaa de un componente variable
de infiltrado inflamatorio crnico (se dispone pocas veces de estudio histolgico). TRATAMIENTO: Suprimir el consumo de alcohol. Analgsicos en
forma escalonada: AINES, opiceos dbiles (dextropropoxifeno, tramadol), combinaciones: dextropropoxifeno + dipirona (Klosidol);
dextropropropoxifeno + ibuprofeno (Supragesic). Si no hay respuesta: agregar amitriptilina. Tratamiento de la maldigestin: Reemplazo enzimtico:
25.000 a 50.000 U de lipasa en cada comida. IBP (Omeprazol). La pancreatoyeyunostoma (procedimiento de Puestow), es conveniente en casos de
pancreatitis alcohlica crnica, dolor crnico y alternancia de dilatacin con estrechamientos ductales (cadena de lagos). Se obtienen buenos
resultados con la pancreatoduodenectoma cuando la pancreatitis crnica se acompaa de estrechamiento del conducto en la cabeza del pncreas.
Si el conducto es pequeo, se obtienen cierto beneficio con la pancreatectoma de 95%, pero la mortalidad es alta. La abstinencia de alcohol resulta
primordial. El dolor crnico es difcil de tratar, si los sntomas persisten o recurren se puede hacer un bloqueo celiaco. COMPLICACIONES:
Pseudoquistes complicados; la estenosis biliar, duodeno o colon; la trombosis venosa (esplnica, mesentrica-portal); los pseudoaneurismas; la
hemorragia intraquistica, intrabdominal o digestiva; las fistulas internas o externas; las ascitis y el cncer de pncreas.
CASO CLINICO PANCREATITIS
Ingres paciente de sexo masculino de 68 aos de edad, con
antecedentes de Diabetes Mellitus en tratamiento con
hipoglicemiantes orales, colecistectoma y alcoholismo en remisin
desde haca 15 aos. Presentaba dolor abdominal, inicialmente en
epigastrio e hipocondrio izquierdo de 20 h de evolucin, el cual
haba ido progresando en intensidad y se extenda ya al resto del
abdomen. La aparicin del dolor fue brusca y se asoci a sudoracin
fra y sensacin de desvanecimiento, sin prdida de conciencia. No
se acompa de vmito ni diarrea. En el examen fsico del ingreso
destacaba un paciente consciente, algo plido, afebril, con
taquicardia leve, una presin arterial de 110/60 mm de Hg sin
apremio respiratorio y adecuada perfusin perifrica. No haba
estigmas de dao heptico crnico. En el examen pulmonar haba
disminucin del murmullo pulmonar en ambas bases, con
predominio izquierdo. En el examen abdominal se evidenciaba cierto
abombamiento, la pared era depresible pero la palpacin provocaba
intenso dolor con signo de Blumberg positivo en hemiabdomen
superior. Los ruidos hidroareos estaban ausentes. No se palpaban
masas pulstiles.
PREGUNTA
Considerando el cuadro clnico cual es su impresin diagnostica
inicial?
RESPUESTA
a.- Ulcera perforada.
b.- Pancreatitis aguda.
c.- Sindrome hepatorrenal.
d.- Ruptura de varices esofgicas.

CURSO ENARM CMN SIGLO XXI TEL: 36246001

PREGUNTA
Se obtuvieron los siguientes resultados de laboratorio
posteriomente, hemoglobina 9 g/L, recuento leucocitario de
8000/L, protrombina 55%, bilirrubina 1,4 mg/dL, albmina 2,3
mg/dL; amilasemia, lipasemia y protena C reactiva en rangos
normales. Los estudios radiolgicos de trax mostraron atelectasia
en ambas bases pulmonares, sin neumoperitoneo y la radiografa de
abdomen simple mostr aire en el colon, escasa cantidad en
intestino delgado y asas no dilatadas, considerando los datos
observados que puntaje presenta de la clasificacin de Child-Pugh.
RESPUESTA
a.- Menos de 5 puntos.
b.- De 5 a 7 puntos.
c.- De 7 a 10 puntos.
d.- Mas de 10 puntos.
PREGUNTA
El paciente se manej con aportes de cristaloides y evolucion con
disminucin del dolor y sin compromiso hemodinmico. En los
exmenes de control al segundo da de hospitalizacin se observ
disminucin de la hemoglobina a 5 g/L, protrombina 54% y recuento
plaquetario de 59.000/L. DHL 450 U/l, AST 140, Glucosa de 250
mgs/dl. Cuantos criterios de Ramson presenta?.
RESPUESTA
a.- 2 criterios.
b.- 3 criterios.
c.- 4 criterios.
d.- 5 criterios.
PREGUNTA

Pharmed Solutions Institute

PGINA 443

MANUAL DE TRABAJO DEL CURSO ENARM CMN SIGLO XXI


Cual de los siguientes diagnosticos diferenciales es el menos
probable en este caso clnico?
RESPUESTA
a.- Hemoperitoneo espontneo en cirrosis.
b.- Rotura de vrices o canales linfticos.
c.- Rotura de carcinoma hepatocelular
d.- Rotura de aneurisma abdominal.
CASO CLINICO
Masculino de 47 aos, casado, maestro. Originario de Monterrey, N.
L. Con antecedente de alcoholismo: ocasional (34g de alcohol),
Tabaquismo: ocasional (10 paquetes/ao), alrgicos: negados, HTA
de tres meses de evolucin tratada con lisinopril 10mg cada 24 h,
Quirrgicos: negados, Transfusionales: negados. Motivo de consulta:
inicia su padecimiento alrededor de 12 h previas a su ingreso al
presentar dolor de tipo ardoroso en epigastrio adems de ser
tambin retroesternal que se irradiaba hacia ambos hipocondrios y
que no estaba relacionado a la ingesta de alimentos. El dolor no
tena exacerbantes ni atenuantes. Al cuadro se le agregan cuatro
episodios de vomito de contenido gastroalimentario motivo por el
cual acude a consulta. Exploracin fsica: abdomen: blando,
depresible, globoso por panculo adiposo, doloroso a la palpacin
media y profunda en epigastrio, sin datos de irritacin peritoneal
(Mc Burney, Murphy, Giordano negativos). Peristalsis disminuida, no
se palpo organomegalias, mucosas: subhidratadas. Laboratorios al
ingreso Hb 20.44, Crea 0.88, VCM 76.12, BUN 25, HCM 34.14 amilasa
280, lipasa 210, Pla 396, BT 1.20, Leu 20.21, BD o.89, Na 128, TGO
84, K 4.16, TGP 25, Cl 96.9 Alb 3.6, Glu 269, Proteinas totales 5.4
PREGUNTA
Cules de los siguientes medicamentos es menos probable que
ocasionen pancreatitis aguda?
RESPUESTA
a.- Azatioprina
b.- Estrgenos
c.- Tetraciclinas
d.- Metoclopramida
PREGUNTA
De los valores de laboratorio del paciente. Cul es el ms probable
que nos oriente de mal pronstico y necrosis extensa?
RESPUESTA
a.- Lipasa 210 U/L
b.- Amilasa 280 U/L
c.- Glucosa >269mg/dl
d.- TGO 84
PREGUNTA
Cul es la causa menos probable de la hipocalcemia en este
paciente?
RESPUESTA
a.- Disminuye el calcio unido a protenas por la hipoalbuminemia
b.- Secuestro de calcio en las reas de necrosis grasa
c.- Liberacin de fosfato intracelular
d.- Aumento de glucagn con el consiguiente aumento de la
tirocalcitonina
CASO CLINICO
Una mujer de 48 aos acude a urgencias por cuadro caracterizado
por nauseas, vmitos y dolor abdominal intermitente que ha
persistido por dos das. El dolor se irradia desde el epigastrio y el
hipocondrio izquierdo hacia el dorso. Nada alivia el dolor y adems
aumenta al comer. La paciente refiere haber tenido, durante los
ltimos dos das, dos o tres deposiciones de muy mal olor,
disgregadas, pero sin elementos patolgicos como sangre o
mucosidades. No ha tenido fiebre. Dentro de sus antecedentes
mdicos destaca el tener diabetes mellitus 2 que requiere terapia
con insulina, alcoholismo crnico, muchos aos de dolor abdominal

CURSO ENARM CMN SIGLO XXI TEL: 36246001

crnico y diarrea crnica. Solo tiene como antecedente quirrgico


una colecistectoma hace muchos aos. EF: afebril, TA 130/80
mmHg, FC 96 lpm, FR 16 rpm. La paciente parece estar desnutrida,
refiere sensibilidad aumentada en el epigastrio e hipocondrio
izquierdo, pero no presenta aumento de la resistencia muscular ni
rigidez abdominal. Tiene ruidos hidroareos aumentados en
nmero, pero no se palpan masas ni ruidos abdominales anormales.
Al examen cardiopulmonar no destaca nada en especial. Se le
tomaron varios exmenes, dentro de los cuales destaca una glicemia
de 250 mg/dL, una albmina de 2,5 gr/dL y una prealbmina de 8
mg/dl y una hemoglobina glicosilada de 7,5%. Sus pruebas hepticas,
perfil lipdico, amilasa, lipasa y hemograma son todos normales.
PREGUNTA
Cul es el diagnstico ms probable para este paciente?
RESPUESTA
a.- Isquemia mesentrica
b.- Pancreatitis crnica
c.- Obstruccin intestinal
d.- Diverticulitis
PREGUNTA
Cul es la causa menos probable de desarrollo de pancreatitis
crnica?
RESPUESTA
a.- Fibrosis intracelular
b.- Calcificacin de material proteico degenerado en los ductos
c.- Infiltrados celulares
d.- Obstruccin biliar
PREGUNTA
Cul es la principal causa de morbimortalidad tarda en pancreatitis
aguda?
RESPUESTA
a.- Pseudoquiste
b.- Absceso
c.- Infeccin
d.- Necrosis
CASO CLINICO PANCREATITIS
Varn de 66 aos, con antecedentes de obesidad, HTA, diabetes
mellitus tipo 2, ex fumador, SAOS, con espirometra con patrn
mixto, hiperplasia benigna de prstata, HDA por lcera prepilrica y
pancreatitis litisica leve. Acude por dolor abdominal intenso, de 2 h
de evolucin, de inicio brusco en hemiabdomen superior, que no
cede con analgsicos y se acompaa de vmitos. Mantiene buenas
constantes vitales (TA 130/80 mmHg; FC 96 lat/min; FR 20/min, y T
36C). Laboratorios: 18.000 leucocitos; amilasa, 5.900; AST, 169; ALT,
105, y BT, 2,5g/dl; PCR, 0,8. En la ecografa abdominal se visualiza
una vescula distendida con barro biliar sin dilatacin de la va biliar.
No se observa bien el pncreas, pero se ve aumentado de tamao e
hipoecoico. Tras 12 h, empeora sbitamente su estado clnico, con
agitacin, sudor, taquipnea, livideces y agudizacin del dolor
abdominal (TA 90/60 mmHg; FC 160 lat/min). En la analtica
presenta 15.500 leucocitos; glucosa, 332 mg/dl; creatinina, 1,7
mg/dl; PCR, 6,8 mg/dl. El abdomen est distendido sin peritonismo.
Su situacin clnica se agrava con hipotensin, mala perfusin
perifrica, bradicardia extrema y asistolia que precisa RCP. Hay
deterioro analtico: leucocitos, 4.470; creatinina, 2,5mg/dl; pH 7,23,
y bicarbonato, 10,9mmol/l, y precisa noradrenalina a altas dosis (30)
y FiO2 al 100%.
PREGUNTA
Cul es su impresin diagnostica ms probable con los estudios
realizados?
RESPUESTA
a.- Necrosis pancretica grave con acidosis metabolica.
b.- Pancreatitis crnica agudizada de origen alcohlica
c.- Colangitis aguda bacteriana secundaria.
d.- Colecistitis crnica litiasica con obstruccin pancretica.

Pharmed Solutions Institute

PGINA 444

MANUAL DE TRABAJO DEL CURSO ENARM CMN SIGLO XXI


ISQUEMIA MESENTERICA AGUDA Y CRNICA:
AGUDA: CIENCIAS BASICAS: Entidad clnica multietiolgica, producida por la interrupcin brusca del aporte sanguneo a un determinado segmento
intestinal, lesionndolo al principio de forma reversible pero que si se mantiene el tiempo suficiente, hace que deje de ser viable y evolucione hacia
la necrosis completa de su pared. SALUD PUBLICA: Supone un tercio de las isquemias intestinales. Proceso con elevada mortalidad (60-80%) y en el
cual es vital la sospecha clnica temprana, ya que la supervivencia depende directamente de la rapidez con que se aplique el tratamiento.
PATOGENIA: Oclusin Arterial (embolia arterial, trombosis arterial): La arteria ms frecuentemente implicada es la mesentrica superior (AMS), el
tronco celiaco y la mesentrica inferior (AMI) tambin pueden ocluirse, pero gracias a la circulacin colateral, no suelen producir lesin isqumica
aguda, a menos que se afecte a la vez la AMS. Oclusin Venosa (trombosis venosa mesentrica): La imposibilidad de retorno venoso produce edema
y aumento de presin venosa que cuando se iguala a la arterial produce isquemia
e infarto hemorrgico. Isquemia no oclusiva (IMNO): el bajo gasto, la hipotensin
o la vasoconstriccin local mantenidos pueden impedir el aporte sanguneo
mnimo necesario para la viabilidad intestinal. CLASIFICACION: 1. EMBOLIA
ARTERIAL: es la causa ms frecuente de IMA (50%) Su origen en un 90% de los
casos es una cardiopata embolgena y puede existir historia de embolias previas a
otros niveles. La zona ms frecuente de embolizacin es la AMS, distalmente a la
salida de la A. Clica Media, afectando al colon derecho y rea leo-cecal. La
isquemia que se produce es muy grave, por la brusquedad con que se instaura y
por la no existencia de colaterales. 2. TROMBOSIS ARTERIAL: supone un 25% de
las IMA. Son pacientes con antecedentes vasculares (es caracterstica la historia
previa de Isquemia Mesentrica Crnica) cuyas arterias tienen disminuido el flujo
y han desarrollado circulacin colateral. La trombosis de la AMS suele producirse
en su origen o en sus 3 primeros centmetros y la mortalidad es muy alta, porque
a pesar de que la circulacin colateral es capaz de mantener cierto flujo, el
territorio afectado es muy extenso3. TROMBOSIS VENOSA MESENTRICA: ha de
afectarse de forma muy severa el eje porto-esplnico-mesentrico e incluso en
estos casos es excepcional que una trombosis venosa produzca un infarto intestinal agudo. Antecedente habitual de hipercoagulacin o TVP. 4.
Insuficiencia Mesentrica No Oclusiva (IMNO): produce el 20% de las IMA. Son pacientes con el flujo esplcnico de base disminuido por la
arteroesclerosis generalizada o el consumo de frmacos vasoactivos como la digital y que ante una disminucin de perfusin generalizada,
desencadenada por un proceso agudo, no pueden mantener el aporte sanguneo necesario en el territorio intestinal. Los precipitantes de la IMNO
van desde el sock cardiognico hasta una deshidratacin que desencadene hipovolemia. Excepcionalmente el consumo de txicos con actividad
simptica como la cocana o los ergotamnicos, pueden producir vasoconstriccin visceral aislada y desencadenar IMNO. DIAGNOSTICO: Sospecha
clnica: es la base del diagnstico y se establece ante un paciente con perfil de riesgo definido, que a primera vista impresiona de gravedad y
presenta dolor abdominal severo, acompaado de forma ms o menos frecuente de distensin abdominal, nauseas, vmitos, diarrea y/o
rectorragia. Llama la atencin la normalidad de la exploracin fsica, sin signos de irritacin peritoneal, en contraste con la gran intensidad del dolor.
1. Embolia Arterial: el dolor es de comienzo brusco y localizacin periumbilical. 2. Trombosis Arterial: inicio gradual del dolor, incluso de das de
evolucin, con aumento progresivo de intensidad y localizacin difusa. 3. Trombosis Venosa Mesentrica: dolor variable segn la extensin de la
trombosis, habitualmente difuso de instauracin progresiva e intensidad ms leve que en las de causa arterial.4. IMNO: el dolor abdominal es difuso
y de rpida instauracin, aunque en ocasiones, por la patologa desencadenante son pacientes graves que se encuentran en cuidados intensivos y
sedoanalgesiados en los que el dolor no es valorable. Se sospecha cuando existe episodio reciente de bajo gasto e hipotensin mantenida y aparece
fiebre y leucocitosis, distensin abdominal o sangrado digestivo. Si se produce la necrosis completa de la pared intestinal aparecen defensa
abdominal y signos de irritacin peritoneal con hiperperistaltismo primero y luego silencio abdominal total. Son pacientes inquietos, sudorosos y que
no se dejan explorar por la gran intensidad del dolor. Aparecen tambin signos de deterioro hemodinmico: taquicardia, hipotensin, oligoanuria,
etc. Laboratorio y gabinete: Radiologa simple: es normal inicialmente. Cuando aparece la necrosis podemos ver edema de pared, distensin de asas,
y gas intramural (infarto intestinal) o libre en peritoneo (perforacin). BH de inicio discreta leucocitosis. Posteriormente aparece: leucocitosis por
encima de 20000 y con gran desviacin izquierda. Acidosis metablica severa y/o coagulopata que en este contexto de dolor abdominal sugieren
isquemia o sepsis grave. Hiperamilasemia y elevacin de CPK, LDH, GOT y GPT que reflejan la presencia de importante destruccin celular. Puede
aparecer hemoconcentracin por presencia de importante edema y tercer espacio abdominal o anemia en los casos que cursan con importante
sangrado digestivo. La arteriografa es mtodo diagnstico tpico, pero es una prueba menos disponible que el TC o el ECO-doppler, que han
aumentado mucho su sensibilidad adems de ser ms tiles como primera prueba diagnstica, porque ayudan orientar el diagnstico diferencial. TC
Helicoidal con contraste IV: muy til para el diagnstico diferencial y es muy sensible para detectar signos indirectos de infarto intestinal: neumatosis
intestinal, dilatacin de asas, edema de pared. Trombosis Venosa Mesentrica: hoy da es el Gold standart. Se aprecia un retraso del paso de
contraste al sistema venoso, una pared engrosada y la falta de opacificacin de la porta. Oclusin emblica o trombtica Arterial: arteriografa sigue
siendo superior al TC. TRATAMIENTO: Inicial: Infusin vigorosa de cristaloides sonda de aspiracin gstrica, analgesia y antibiticos intravenosos
(Cefotaxima1 2gr + Metronidazol 500mg cada 8h o Piperacilina-tazobactam 4gr / 6h). T. definitivo: si existe infarto intestinal establecido el
tratamiento siempre es quirrgico, para reseccin del segmento necrosado, con la mxima urgencia posible. 1. Oclusin emblica o trombtica
Arterial: tratamiento quirrgico para repermeabilizar los segmentos aun viables mediante embolectoma o By-pass 2. Trombosis Venosa
Mesentrica: si no hay infarto intestinal establecido se realiza tratamiento conservador anticoagulando con heparina. 3. IMNO: lo fundamental es la
correccin de los factores desencadenantes. Se puede realizar infusin de vasodilatador (papaverina) intrarterial, durante la arteriografa. Hay
autores que recomiendan el empleo de IECA por la relacin del eje renina-angiotensina en la fisiopatologa del cuadro. ISQUEMIA MESENTERICA
CRONICA (IMC): Tambin conocida como angina intestinal se produce cuando existe una desproporcin entre las demandas de oxgeno, la cual
ocurre fundamentalmente durante la digestin, y el flujo sanguneo proporcionado por el sistema vascular. Generalmente se produce en personas
con marcada ateroesclerosis y representa menos del 5% de los casos de isquemia intestinal. Clnica; dolor abdominal que aparece precozmente tras
la ingesta y cede en el plazo de 2 hrs. La intensidad es mayor tras la ingesta de comidas copiosas con alta proporcin en grasa. Los sntomas suelen
aumentar con la consecuente prdida de peso, descrita hasta en el 80% de los casos debido a la aversin de la comida por miedo al dolor. En algunos
casos puede producirse una IMA sobre una IMC, debido a la formacin de trombos sobre placas ateroesclerosas. El diagnostico se establece por el
cuadro clnico compatible, en pacientes con coexistencia de otras enfermedades vasculares (cardiaca, cerebral o perifrica). La demostracin
arteriografica de obstruccin de al menos dos de los vasos esplcnicos y la exclusin de otras patologas que cursen con sntomas similares. La
demostracin por s sola no constituye una prueba inequvoca de IMC, puesto que estas lesiones se pueden encontrar en sujetos asintomticos. La

CURSO ENARM CMN SIGLO XXI TEL: 36246001

Pharmed Solutions Institute

PGINA 445

MANUAL DE TRABAJO DEL CURSO ENARM CMN SIGLO XXI


rentabilidad de las pruebas bioqumicas reside en la identificacin de factores de riesgo para desarrollo de IMC, como son la dislipidemias o la DM,
as como la identificacin de datos que apoyen la malabsorcin intestinal que pueden presentar estos pacientes. El tratamiento incluye construccin
quirrgica y ATP (angioplastia transluminal percutnea) con o sin la colocacin de un stent. La eleccin va a depender de la experiencia de cada
centro, de la edad del paciente, comorbilidad asociada y del nmero y severidad de la oclusin vascular. Revascularizacin quirrgica: como la
reimplantacin de la AMS en la aorta abdominal, la endarterectoma mesentrica y el bypass. INFARTO MESENTERICO. CIENCIAS BASICAS: El infarto
intestinal, isquemia intestinal o necrosis intestinal es la muerte del tejido intestinal debido a una interrupcin del suministro de sangre, de una o ms
de las arterias mesentricas, principales arterias que suministran sangre al intestino delgado. Suele venir precedida de sintomatologa de angina
intestinal, por insuficiencia vascular del mesenterio, que degenera en isquemia. Existen varias causas posibles del infarto intestinal: En una hernia, si
la irrigacin del intestino queda ocluida, esto puede llevar a isquemia intestinal. El intestino tambin puede quedar atrapado en tejido cicatricial de
una ciruga previa. Una embolia que bloquee los vasos principales que irrigan el intestino, as como la trombosis arterial, pueden impedir el
suministro adecuado, generalmente a partir de una enfermedad ateroesclertica (acumulacin de colesterol). Recientemente, se ha sugerido que
este bloqueo podra ser en parte causado por un remodelado vascular anmalo. Una trombosis de las venas que evacuan la sangre del intestino, que
se pueden obstruir con cogulos de sangre. sta es una afeccin ms frecuente en personas con insuficiencia heptica, cncer o trastornos de la
coagulacin. Una hipotensin, ya que la presin arterial muy baja en pacientes con obstruccin previa de las arterias tambin puede ocasionar
isquemia intestinal. Clnica; dolor abdominal de inicio sbito, diarrea, fiebre, vmitos, leucocitosis, leve acidosis. El tratamiento generalmente
requiere ciruga, conocida como enterectoma, en la que se extirpa la porcin necrosada y se unen los cabos seccionados. Segn la regin implicada,
se denomina colostoma o ileostoma. Si es posible, se corrige la obstruccin de las arterias que irrigan el intestino. El tratamiento tambin puede
incluir medicamentos para prevenir los cogulos, disolver los cogulos ya existentes, o la dilatacin de los vasos sanguneos. Si la angiografa se
realiza para diagnosticar el problema, es posible eliminar al mismo tiempo un cogulo de sangre o abrir una pequea arteria con la angioplastia. La
angioplastia consiste en utilizar un globo en la punta de un catter para comprimir la arteria y la extensin de los depsitos de grasa, haciendo un
camino ms amplio para que la sangre fluya sin problemas.

CASO CLINICO ISQUEMIA E INFARTO MESENTERICO


Mujer de 83 aos con antecedentes de cardiopata hipertensiva,
diabetes mellitus tipo 2, hipertensin arterial, flutter auricular y
episodio de pancreatitis aguda de origen biliar. Acude a urgencias
por episodio de dolor abdominal localizado en epigastrio que se
irradia hacia ambos hipocondrios, acompaado de nuseas, vmitos
y distensin abdominal. La analtica muestra leucocitosis (17,68 x 103/ul) con predominio de neutrfilos (92,8% N), as como un aumento
marcado de LDH (1082 U/l). La gasometra venosa, los parmetros
bioqumicos de perfil heptico y biliar, as como los niveles de
amilasa, se encuentran dentro de los lmites normales. Se realiza una
RX de abdomen donde se aprecia dilatacin de asas de delgado, con
presencia de gas distal, sugiriendo suboclusin intestinal y una
ecografa abdominal que mostraba quistes simples renales, sin
evidencia de lesiones ni lquido libre intraperitoneal.
PREGUNTA
Cul es la patologa ms probable con los datos clnicos?
RESPUESTA
a.- Isquemia mesentrica aguda.
b.- Isquemia mesentrica crnica.
c.- Colitis isqumica.
d.- Oclusin intestinal recurrente.

PREGUNTA
Cual es la conducta a seguir mas adecuada?
RESPUESTA
a.- Eco-dopler
b.- Angio-TAC.
c.- Angio- IRM.
d.- Angiografia.

CASO CLINICO ISQUEMIA MESENTERICA


Mujer de 61 aos diagnosticada de obesidad mrbida e hipertensin
arterial, que acude a urgencias por dolor periumbilical de 2 das de
evolucin. En la ecografa abdominal se observa escaso lquido libre
y conglomerado de asas intestinales sin herniacin. 24 horas
despus la paciente presenta un cuadro de fallo multiorgnico. En la
tomografa computarizada de abdomen con contraste intravenoso,
se observa neumatosis intestinal de la totalidad de asas del intestino
delgado con eventracin abdominal anterior incarcerada y presencia
de gas en venas mesentricas, perihepticas, periesplnicas y ramas
distales intrahepticas. En la ciruga se evidencia isquemia
mesentrica masiva y colon transverso incarcerado.
PREGUNTA
Cul es el antecedente ms probable con los datos clnicos y el
cuadro clnico actual?
RESPUESTA
a.- Colitis isqumica.
b.- Isquemia focal segmentaria.
c.- Trombosis venosa mesentrica.
d.- Isquemia mesenterica crnica.

CURSO ENARM CMN SIGLO XXI TEL: 36246001

CASO CLINICO
Paciente mujer de 55 aos, con antecedentes de tabaquismo
crnico, hipertensin arterial moderada y dislipidemia en
tratamiento, cuyo cuadro comenz en el ao 2012, caracterizado
por dolor abdominal tipo clico postprandial precoz, acompaado
de diarrea frecuente, alrededor de 10 episodios diarios y vmitos
ocasionales, adems de baja de peso de alrededor de 15 kilos,
siendo su peso habitual de 55 kg, llegando a un ndice de masa
corporal de 17,3. Estudiada en el contexto de diarrea crnica y
bsqueda de enfermedad inflamatoria intestinal, se someti a
endoscopia alta y baja, adems de enteroscopia, trnsito intestinal y
estudio de mala absorcin, lo que slo concluy gastritis y colitis
inespecfica, no respondiendo a tratamiento habitual, esto es,
rgimen
alimenticio,
antiespasmdicos
y
medicamentos
antiulcerosos.

CASO CLINICO
Paciente varn de 75 aos de edad, con antecedente de
dislipidemia, diabetes mellitus de 10 aos de evolcuicon, as como
hipertensin de 30 aos de evolucin, as como insuficiencia
cardiaca, adems refiere infarto de miocardio hace 5 aos. Ingresa al
servicio de guardia, con cuadro clnico de 10 horas de evolucin,
caracterizado por dolor abdominal de comienzo en epigastrio,
sostenido, con migracin a fosa iliaca derecha sin defensa ni
peritonismo. Se acompaa de naseas, sin vmitos, fiebre, diarrea,
con apetito conservado. Se solicita ecografa abdominal que informa
asas de intestino delgado distendidas en fosa ilaca derecha (FID). Rx
de trax: normal, Rx de abdomen: nivel hidroareo en flanco y fosa
ilaca derecha. Laboratorio: leucocitos 21700, resto: normal, orina:
normal, signos vitales al ingreso: Fc: 73 x, Fr: 17 x, TA:
130/90mmHg, T: 38,6C. En la exploracin de abdomen, timpanismo
y peristalsis disminuida.
PREGUNTA
Cul es el diagnstico ms probable para este paciente?
RESPUESTA

Pharmed Solutions Institute

PGINA 446

MANUAL DE TRABAJO DEL CURSO ENARM CMN SIGLO XXI


a.- Pancreatitis aguda
b.- Isquemia mesentrica aguda
c.- Pancreatitis crnica
d.- Diverticulitis

d.- Fiebre reumtica

PREGUNTA
Cul es el porcentaje de mortalidad para este paciente?
RESPUESTA
a.- 15%
b.- 30%
c.- 65%
d.- 90%
PREGUNTA
De los factores de riesgo que tienen el paciente. Cul es el menos
probable que influyo en el desarrollo de su patologa?
RESPUESTA
a.- Infarto agudo al miocardio
b.- Insuficiencia cardiaca
c.- Arterioesclerosis sistmica
d.- Diabetes mellitus
CASO CLINICO
Paciente de 43 aos, masculino, blanco, acude a emergencia por
dolor abdominal de 2 das de comienzo, el cual se inici
periumbilical, posterior a la ingestin de alimentos, dolor clico en
sus inicios, que a las 12 horas se localiz en hemiabdomen derecho,
especfico en fosa ilaca y flanco derecho, fiebre de 38, anorexia,
nuseas. Al examen fsico se encontr taquicardia; abdomen
doloroso a los golpes de tos y cada sobre los talones en fosa ilaca
derecha, palpacin dolorosa, con contractura muscular y signos de
irritacin peritoneal en dicha zona. Se realiza hemograma
encontrando leucocitosis. Con estos sntomas y signos se decide
intervenir quirrgicamente con el diagnstico de apendicitis aguda,
se realiza laparotoma encontrando apndice normal, presencia de
necrosis de 20 cm de leon a ms de 30 cm de la vlvula ileocecal, se
reseca segmento lesionado y anastomosis trmino-terminal, durante
el proceder se observan la salida de pequeos cogulos, mltiples a
medida que se reseca y se liga el mesenterio, encontrndonos en
presencia de una isquemia mesentrica segmentaria de intestino
delgado, aguda por trombosis venosa.
PREGUNTA
Cul es el tratamiento mdico ms adecuado para este caso?
REPUESTA
a.- Estreptoquinasa
b.- Papaverina
c.- Heparina
d.- Activador de plasminogeno
PREGUNTA
Cul es la causa menos probable de isquemia mesentrica aguda no
oclusiva?
RESPUESTA
a.- Sepsis
b.- Hemorragia
c.- Vasoespasmo esplcnico paradjico
d.- Hipercoagulabilidad
PREGUNTA
Cul es el factor de riesgo menos probable para la formacin de
mbolos en isquemia mesentrica?
RESPUESTA
a.- Hipotensin
b.- Lesin valvular
c.- Infarto agudo al miocardio reciente

CURSO ENARM CMN SIGLO XXI TEL: 36246001

CASO CLINICO
Paciente varn de 27 aos de edad, con cuadro clnico de 10 horas
de evolucin, caracterizado por dolor abdominal de comienzo en
epigastrio, sostenido, con migracin a FID sin defensa ni
peritonismo. Se acompaa de naseas, sin vmitos ni fiebre con
apetito conservado. Rx de abdomen: nivel hidroareo en flanco y
fosa ilaca derecha. Laboratorio: > GB 21700 (89%N) resto: normal,
se diagnostica apendisitis, se realiza apendisectomia, egresando dos
das despus sin datos anormales. El paciente re-ingresa, con cuadro
clnico de 48 hs de evolucin caracterizado por sndrome febril, que
cede parcialmente con la ingesta de antipirticos (Ibuprofeno VO).
Examen fsico: > TA 120/70, Fc 133x, Fr 16x, T 39,1C, > Leve dolor
abdominal, mayor sobre zona operada, signos de flogosis y
supuracin serohemtica en cicatriz de Mc Burney, mantiene
trnsito intestinal, sin dificultad al orinar. > Laboratorio: GB 21100
(92%N) resto normal.
PREGUNTA
Cual es su impresin diagnostica?
RESPUESTA
a.- Sepsis abdominal.
b.- Absceso abdominal.
c.- Trombosis mensenterica.
d.- Diverticulitis aguda.
PREGUNTA
Cual es la conducta a seguir mas adecuada para establecer el
diagnostico?
RESPUESTA
a.- LAPE.
b.- Angio-TAC.
c.- Eco-dopler.
d.- USG abdominal.
PREGUNTA
En esta patologa en su forma aguda. cual de los siguientes
sntomas es el menos frecuente?
RESPUESTA
a.- Dolor abdominal
b.- Anorexia.
c.- Diarrea.
d.- Nuseas y vmitos.
CASO CLINICO
aciente de sexo masculino de 50 aos quin present cuadro de
dolor abdominal de 36 h de evolucin, localizado en flanco derecho,
de inicio insidioso y con exacerbaciones de carcter clico. No
present fiebre y al examen fsico slo destacaba dolor a la
palpacin en regin periumbilical y flanco derecho. Sus exmenes de
laboratorio eran normales. Se realiz tomografa axial computada
helicoidal en fase arterial de abdomen que evidenci imagen
sugerente de diseccin de la AMS, sin otro hallazgo patolgico
evidente.
PREGUNTA
Cul es la conducnta diagnostica mas adecuada a seguir en este
caso?
RESPUESTA
a.- Resonancia magntica
b.- TAC con medio de contraste
c.- Angiografia
d.- Dopler color

Pharmed Solutions Institute

PGINA 447

MANUAL DE TRABAJO DEL CURSO ENARM CMN SIGLO XXI


DIVERTICULITIS
CIENCIAS BASICAS: Enfermedad diverticular: Enfermedad de colon asociado a cambios en el hbito intestinal, dolor y distensin abdominal.
Diverticulosis: Paciente con divertculos en ausencia de cualquier sntoma. Diverticultis: Complicacin inflamatoria de la enfermedad diverticular.
Factores de riesgo: AINES, edad, inmunocompromiso, tabaquismo y alcoholismo, bajo consumo de fibra, gentica, Sndrome de Williams, Enf.
Poliqustica Renal, Sndrome de Ehlers- Danlos. SALUD PUBLICA: Aproximado de 50-65% en >60 aos. 10-15% diverticulosis desarrollaran
Diverticultis. PATOGENIA: Anatoma patolgica: Sitio de debilidad en la pared colnica. Disminucin en la colgena tipo 1, incremento en
colgena tipo 3, incremento en la presin intraluminal 90mmHg, hay una respuesta exagerada sistema simptico, motilidad y electrofisiologa, hay
aumento en los movimientos de segmentacin y retropulsin. Histopatologa: Hipertrofia e hiperplasia de los segmentos afectados, debilidad y
adelgazamiento en el sitio de protucin o salida del divertculo. Teora de Ryan
Tipo 1: anormalidad muscular clsica por presin aumentada, confinada al colon
izquierdo, dolor inflamacin y complicaciones inflamatorias. Tipo 2: sin
anormalidades musculares, tejido conectivo afectado, colon derecho, sangrado es
el sntoma ms comn. Teora de Mann: incremento en la presin intraluminal
secundaria a falta de relajacin de la unin recto-sigmoidea, hipertrofia muscular.
Localizacin: Mayo Clinic Sigmoides 29%, sigmoides y otros 68%, otros 3%.
CLASIFICACION: de Hinchey modificada: Estadio I; I a Flemn, I b Diverticultis con
absceso periclico o mesentrico. Estadio II; Absceso plvico tabicado, II a Absceso distal capaz de drenarse percutneamente, II b Absceso complejo
asociado con fstula. Estadio III: Peritonitis purulenta generalizada. Estadio IV: Peritonitis fecal. Estadios clnicos: Diverticulosis: Presencia de
divertculos asintomtico, incidentalomas, no requiere tratamiento ni seguimiento. Enfermedad Diverticular no complicada: Presencia de
sintomatologa asociada al hbito intestinal, distensin abdominal, meteorismo, se confunde con SII. Enfermedad Diverticular complicada:
Sangrado, colon derecho, hematoquezia (vinoso), manejo mdico y resolucin en 80%, probabilidad de resangrado del 20-30%, indicacin quirrgica
hasta el segundo evento. Fstulas colo-vesical es la ms frecuente, ms frecuente en hombres 3:1, complicacin menos frecuente de la ED.
Diverticultis no complicada: Pacientes sin afectacin sistema por datos inflamatorios Hinchey I y II, respuesta al tratamiento mdico hasta 90%,
repeticin del cuadro en 25%, indicacin quirrgica electiva en el 1er cuadro con patologa sistmica asociada. Diverticulitis complicada: Hinchey II y
III, patologa que involucra absceso plvico o peritonitis, afectacin sistmica, tratamiento quirrgico, indicado siempre la programacin electiva si
mejora con tratamiento conservador. DIAGNOSTICO: Colon por enema sensibilidad del 99%, especificidad del 96%. Colonoscopia sensibilidad del
88%, especificidad del 74%. TAC sensibilidad del 95%, especificidad del 86%. Colonoscopia virtual. TRATAMIENTO: Tratamiento mdico
dependiendo de la severidad, identificar complicaciones, valoracin del estado hidroelecroltico, valoracin del estado acido-base, valorar
tratamiento quirrgico. Medico: Ayuno, soluciones, antibiticos (ciprofloxacino, metronidazol, analgsicos, ketorolaco, paracetamol, tramadol).
Quirrgico: Indicado en falla al tratamiento mdico, sepsis, alteraciones en el estado hemodinmico. Procedimiento 3 tiempos (three times
procedure= Colostoma de transverso y drenaje, reseccin subsecuente, cierre de colostoma). Procedimiento de Miculicz (exteriorizacin y
reseccin con colostoma y fstula mucosa. Restitucin del trnsito en segundo tiempo). Procedimiento de Hartman (Reseccin de sigmoides. Bolsa
de Hartmann o mun rectal. Restitucin del trnsito en segundo tiempo). Procedimiento en 1 solo tiempo (reseccin del foco sptico y restitucin
del trnsito en un solo tiempo quirrgico). Lavado de absceso. Drenaje percutneo. COMPLICACIONES: Sangrado 4-17%, perforacin 32.4%, absceso
10.9%, Fstula 5-33% (colovesical, colocutnea, colovaginal), estenosis 13.4%
PREGUNTA
Cual es el mtodo diagnostico mas adecuado para el caso descrito?
CASO CLINICO
RESPUESTA
Un hombre de 45 aos, hasta ese momento sano, consulta por dolor
a.- Colonoscopia.
intenso en la fosa ilaca izquierda, que comenz 35 horas antes. Ya
b.- Transito intestinal.
antes haba notado malestar leve y peridico en esta regin pero no
c.- TAC abdominal.
solicito tratamiento mdico. Tambin presentaba nuseas, anorexia
d.- USG abdominal.
y vmitos, que los relacionaba con las comidas. En el examen fsico
estaba febril (38,5C) y taquicrdico (110/min). Dolor a la palpacin
PREGUNTA
en la fosa ilaca izquierda, sin signos peritoneales.
Considerando el caso clnico, cual es la conducta a seguir mas
adecuada?
PREGUNTA
RESPUESTA
Las manifestaciones clnicas de la diverticulitis colnica aguda varan
a.- Ingreso, ayuno, hidratacin y antibiticos.
con la extensin de la enfermedad. Cual de las siguientes
b.- Preparar al paciente para ciruga laparoscpica.
observaciones es menos frecuente?
c.- Preparar al paciente para ciruga abierta.
RESPUESTA
d.- Tratamiento ambulatorio.
a.- Constipacin importante.
b.- Dolor abdominal.
PREGUNTA
c.- Sensacin de plenitud perirrectal.
Cual es la cantidad de fibra que deber consumir posteriormente el
d.- La prueba del guayaco en las heces es positiva.
paciente para evitar recaidas?
RESPUESTA
PREGUNTA
a.- 10 gr/dia.
Cual de los siguiente diagnosticos diferenciales es el menos
b.- 20 gr/dia.
frecuente?
c.- 30 gr/dia.
RESPUESTA
d.- 40 gr/dia
a.- Apendicits aguda
b.- Enfermedad intestinal inflamatoria
c.- Enfermedad de Crohn
d.- Colitis infecciosa

CURSO ENARM CMN SIGLO XXI TEL: 36246001

Pharmed Solutions Institute

PGINA 448

MANUAL DE TRABAJO DEL CURSO ENARM CMN SIGLO XXI


APENDICITIS
CIENCIAS BASICAS: Es la inflamacin del apndice vermiforme, que inicia con obstruccin de la luz apendicular; es un padecimiento grave, con
importantes complicaciones que pueden llevar a la muerte, en particular cuando se retrasan el diagnstico y la teraputica oportuna. La apendicitis
es la patologa quirrgica actual ms frecuente. SALUD PUBLICA: Edad promedio 31.3 aos, mediana 22 aos, error en diagnostico 15.3 %. El 40 %
10-29 aos. 260 000 apendicectomas. PATOGENIA: Etiologa: Obstruccin de la luz, **Hiperplasia de tejido linfoide. (55%), fecalito 40 %, parsitos
intestinales, tumores, semillas, bacterias aerobias (E.coli, P. aeruginosa, Klebsiella, Streptococcus), anerobios (Bacteroides fragilis, fusobacterium,
peptostreptococcus). Al obstruirse la luz y se forma un asa cerrada en la porcin distal del apndice que se distiende rpidamente al aumentar la
secrecin mucosa. Se eleva la presin hasta 50 o 60 cmH2O y la distensin estimula las terminaciones nerviosas, con aparicin de dolor sordo y
difuso; esto incrementa la peristlsis y aparece el clico caracterstico. El ensanchamiento de la luz apendicular sigue en aumento por la
multiplicacin bacteriana, que produce gas, pus, o necrosis. Se excede la presin de las venas y stas se colapsan e impiden el retorno de la sangre;
en tanto, la circulacin contina por las arterias y provoca congestin hemtica, edema y estasis. La distensin progresiva del apndice emite
reflejos por va nerviosa y ocasiona nusea, vmito y aumento del dolor. El proceso inflamatorio involucra pronto a la serosa apendicular y al
peritoneo parietal, con lo cual el dolor se traslada a la fosa iliaca derecha; la mucosa apendicular es invadida por bacterias y stas se extienden a
capas ms profundas; todo ello desencadena necrosis y absorcin de sustancias provenientes de tejidos muertos y de toxinas bacterianas, lo que
produce fiebre, taquicardia y leucocitosis. Al continuar la distensin apendicular se ocluye la circulacin de arteriolas, se agrava la necrosis, las
paredes no resisten y finalmente el apndice se perfora. El organismo reacciona rodeando la zona con asas de intestino y epipln, para limitar la
diseminacin a un rea restringida, pero esto origina absceso apendicular; entonces se presenta el leo paraltico para focalizar el proceso. Sin
embargo, otras veces el proceso infeccioso no se localiza y con la rotura apendicular se diseminan diversos lquidos en la cavidad abdominal,
ocasionando peritonitis generalizada. CLASIFICACION: A) Apendicitis aguda: 1. Apendicitis aguda no perforada (Fase hipermica, edematosa,
necrtica o gangrenada). 2. Apendicitis aguda perforada (Fase purulenta, de absceso, con peritonitis local, con peritonitis generalizada). 3.
Apendicitis aguda reactiva. 4. Apendicitis aguda complicada. B) Apendicitis crnica. DIAGNOSTICO: Clnica: El sntoma ms comn es el dolor (tipo,
localizacin, migracin, variaciones anatmicas), la anorexia, que acompaa a la apendicitis en 50 a 90 % de las veces; tambin son frecuentes
nuseas, vmito, diarrea (10 % de los casos), estreimiento, fiebre. Casi todos los pacientes sufren al principio anorexia y despus dolor abdominal y
vmito. Los hallazgos de la exploracin fsica son decisivos, especialmente los datos abdominales. En el cuadrante inferior derecho se presentan
hiperestesia e hiperbaralgesia, dolor intenso bien definido en el punto de McBurney, rigidez muscular y dolor a la descompresin. Otros datos
menos frecuentes son la positividad a las maniobras del psoas (dolor del cuadrante inferior derecho a la extensin del muslo en decbito lateral
izquierdo) y de Rovsing (la palpacin del cuadrante inferior izquierdo suele producir dolor en el cuadrante inferior derecho). En el apndice ubicado
en la pelvis los signos abdominales son escasos; es ms probable detectar una masa palpable al tacto rectal o vaginal. La apendicitis en el paciente
peditrico es ms grave por la alta incidencia de perforacin y por la peritonitis que se desarrolla al demorar el diagnstico; adems, la enfermedad
progresa con mayor rapidez. La apendicitis del anciano tambin es grave, estos pacientes manifiestan pocos sntomas ya que el dolor es menos
intenso y los leucocitos casi no se incrementan; adems, las enfermedades intercurrentes deterioran al paciente, riesgo de perforacin 49 %,
mortalidad 21 %. La apendicitis es comn durante el embarazo (uno de cada 2 200). Se relaciona con dolor en posicin ms ceflica que la normal,
sin dejar de ser compatible con la migracin del ciego, a partir del cuadrante inferior derecho a la posicin subcostal, conforme evoluciona el
embarazo. La apendicitis perforada durante la gestacin se acompaa de mayor riesgo para la madre y feto a causa de complicaciones spticas.
Laboratorio: El dato ms constante es la leucocitosis (entre 10 000 y 16 000 mm3), acompaada de un predominio porcentual de los neutrfilos y
presencia de formas en banda; sta es una reaccin inespecfica del proceso infeccioso y no es exclusiva de la apendicitis. Cifras mayores a 18 000 se
observan en el absceso apendicular, la peritonitis generalizada o en el absceso heptico. El EGO es normal excepto en el apndice retrocecal, que
involucra al urter o a la vejiga; esto puede hacer pensar al clnico en una infeccin de vas urinarias. Las placas simples de abdomen en posicin de
pie y en decbito, muestran una o dos asas distendidas por gas, ubicadas en el cuadrante inferior derecho del abdomen; puede apreciarse el fecalito
y tambin el borramiento de la sombra del msculo psoas derecho, la ausencia de gas en colon y una zona de opacidad en el cuadrante inferior
derecho, imagen en vidrio despulido. Ultrasonido: Sensibilidad 85 90 %, Especificidad 92 96 %, til en embrazo y pacientes ginecolgicas. TAC:
Sensibilidad 92 97 %. Especificidad 90-98 %, observamos masas, abscesos, signo de punta de flecha. TRATAMIENTO: Prequirrgico; El periodo de
preparacin debe ser menor de cuatro horas. Ayuno, reparar deficiencia de lquidos y electrolitos y trastornos cido-base. Iniciar antibiticos: 1.
Ampicilina, 2. Aminoglucsido, 3. Metronidazol o clindamicina, 4. Ampicilina-sulbactam o cefoxitina. Analgsico no opiceo. Antipirticos. Catter
central, sonda vesical, SNG (paciente critico). La ciruga laparoscpica es til tanto para la exploracin diagnstica como para el tratamiento; sigue
los mismos principios de la tcnica quirrgica tradicional y su empleo es cada vez ms frecuente. COMPLICACIONES: Perforacin, peritonitis focal,
peritonitis difusa, pileflebitis. En primer da posquirrgico: Hemorragia. Evisceracin por mala tcnica. leo adinmico. Tarda infertilidad (33%).
CASO CLINICO APENDICITIS
Masculino de 21 aos de edad. Consult en la madrugada, tras ms
o menos 16 h de evolucin de un dolor abdominal vago que habra
comenzado tras el desayuno, ubicado en la regin periumbilical, era
de baja intensidad, se mantuvo varias horas para luego localizarse en
fosa ilaca derecha. No hubo compromiso del estado general, pudo
ingerir comida liviana, no tuvo vmitos pero s algo de nuseas. Por
la tarde consult mdico quien sospech una apendicitis aguda. Con
diagnstico de apendicitis aguda se hospitaliz y al ingreso se
observo dolor espontneo y a la palpacin en fosa ilaca derecha con
signo de Blumberg. Se coloco en observacin y se realizaron
laboratorio pero el paciente refera ya no tener dolor abdominal y al
ser examinado no se presento dolor ni resistencia abdominal.
PREGUNTA
Debido a la presentacin y evolucin del cuadro clnico. Cual es la
conducta a seguir?
RESPUESTA
a.- Explorar posible patologa urinaria.
b.- Egresarlo y evaluar en 24 hrs.

CURSO ENARM CMN SIGLO XXI TEL: 36246001

c.- Realizar laparatomia exploratoria.


d.- Realizar apendicectomia laparoscpica.
CASO CLINICO APENDICITIS
Masculino de 17 aos de edad, que inici con sntomas de dolor
abdominal difuso, de gran intensidad que l atribua a una
trasgresin alimentara. Se mantuvo en reposo y dieta, no refera
vmitos ni diarrea; 24 h ms tarde el dolor an persista pero de
menor intensidad y se localizaba en abdomen bajo. La descripcin y
conclusin fueron compatibles con apendicitis aguda, demostrando
dimetros apendiculares de 11 y 8 mm, sensible, no deformable con
aumento de ecogenicidad de la grasa adyacente. Al ingreso el
paciente refera poco dolor espontneo y a la palpacin, con un
Blumberg positivo. La temperatura axilar era de 37,2C y el recuento
de leucocitos de 6.700.
PREGUNTA
Cul es sntoma ms relevante en el diagnostico de apendicitis?
RESPUESTA
a.- Dolor tpico.

Pharmed Solutions Institute

PGINA 449

MANUAL DE TRABAJO DEL CURSO ENARM CMN SIGLO XXI


b.- Resistencia muscular.
c.- Leucocitosis con predominio neutrofilos.
d.- Nausea y vomito.

a.- Hemorragia, evisceracin por mala tcnica, ileo adinmico


b.- Dehiscencia del mun, atelectasia, absceso intrabdominal
c.- Neumona, fistula cecal
d.- Hemorragia, fistula cecal, infeccin de herida

CASO CLINICO APENDICITIS


Mujer de 17 aos que acude a Urgencias por un cuadro de abdomen
agudo. Como antecedentes, destacaba un ingreso un ao antes para
estudio por un sndrome febril y cefalea. Fue diagnosticada de una
infeccin por citomegalovirus, desde entonces permaneciendo
asintomtica y sin tratamiento. La paciente consult por dolor
abdominal de 6 horas de evolucin, iniciado en epigastrio y posterior
migracin a fosa ilaca derecha, intenso, de caractersticas continuas,
con vmitos, sin fiebre ni otros sntomas. A la exploracin
presentaba febrcula, defensa y signos de irritacin peritoneal en
hemiabdomen derecho. Analtica: leucocitos 16.330/l, neutrfilos
76,7%, hematocrito 39%, ALT 59 UI/l, GGT 19 UI/l, amilasa 354 UI/l,
PCR 0,7mg/dl. Test de embarazo: negativo. Radiologa simple:
anodina. No signos de oclusin. Ecografa abdominal: vescula sin
alteraciones. Va biliar de calibre normal. Pequea cantidad de
lquido libre en pelvis.
PREGUNTA
Cul es el mtodo diagnstico ms certero para este padecimiento?
RESPUESTA
a.- Ultrasonido.
b.- Tomografa.
c.- Exploracin fsica.
d.- Rx de pie y decbito de abdomen
CASO CLINICO
Paciente de 20 aos de edad, sin alergias medicamentosas
conocidas, con antecedente patolgico de fractura de codo. Acudi
al servicio de urgencias por un dolor epigstrico con migracin
periumbilical. En la valoracin inicial, la exploracin tanto fsica
como radiolgica y la analtica efectuada no revelaron alteraciones
patolgicas, por lo que el paciente fue enviado a su domicilio. En la
evolucin, se autolimit el dolor abdominal, pero se aadi fiebre y
polaquiuria. El paciente fue visitado por un facultativo de
ambulatorio que prescribi tratamiento con amoxicilina-cido
clavulnico. A los 3 das del inicio del cuadro, volvi a acudir al
servicio de urgencias, refiriendo anorexia, vmitos, estreimiento y
dolor hipogstrico. En la exploracin, se encontr a un paciente con
buen estado general, consciente y orientado, con palidez +, con una
TA 92/63 mmHg, una FC de 77 lpm y una temperatura axilar de
37.9, a la exploracin de abdomen, signo de Holman, Lapinsky
positivos, y resistencia muscular, resto sin alteraciones.
PREGUNTA
Cul es la causa ms comn para esta patologa?
RESPUESTA
a.- Fecalito
b.- Hiperplasia de tejido linfoide
c.- Parsitos intestinales
d.- Semillas
PREGUNTA
Cules son los agentes etiolgicos ms frecuentes en el desarrollo
del proceso infeccioso?
RESPUESTA
a.- Pseudomona aeruginosa, Bacteroides species
b.- Bacteroides fragilis, Clostridium species
c.- Escherichia coli, Klebsiella species
d.- Bacteroides fragilis, Escherichia coli
PREGUNTA
Cules son las complicaciones ms probables en el primer da de
posquirrgico en el paciente?
RESPUESTA

CURSO ENARM CMN SIGLO XXI TEL: 36246001

CASO CLINICO
Femenino de24 aoos de edad es llevado a la sala de urgencias por
su compaero de cuarto, por dolor abdominal de conco horas de
evolucin. Nunca haba presentado ese dolor e informa que desde
que inicio el dolor no ha tenido apetito. Vomito en una ocasin
alrededor de una hora antes. Refiere que le duele todo el abdomen,
pero mas del lado derecho. El dolor es de 9 /10, sordo y no hay
factores que hagan que se aumente o se disminuya. Niega
antecedentes personales patolgicos, quirrgicos o familiares de
sntomas similares. EF: T 38.3C, TA 120/70mmHg, pulso 80/min,
FR10rpm. Resalta falata de ruidos intestinales e hipersensibilidad
intensa a la palpacin profunda de los cuadrantes inferiores tanto
derecho como izquierdo. Existe hipersensibilidad a la rotacin
interna pasiva de la cadera derecha. Laboratorios: leucocitos
10000/mm3, EGO pocos eritrocitos y leucocitos.
PREGUNTA
Cul es el diagnostico diferencial menos probable para este caso?
RESPUESTA
a.- Embarazo ectpico
b.- Torsion ovrica
c.- EPI
D.- Qusite ovrico roto
PREGUNTA
Cul es el estudio de gabinete mas til para esta patologa?
RESPUESTA
a.- Ecografia
b.- Resonancia magntica
c.- Tomografia
d.- Radiografia
PREGUNTA
Cul es la causa menos probable para esta patologa?
RESPUESTA
a.- Incremento de presin luminal
b.- Isquemia
c.- Crecimiento bacteriano excesivo
d.- Fibrosis
CASO CLINICO
Paciente masculino de 55 aos de edad el cual ingresa al servicio de
urgencias con antecedentes de haber sufrido agresin por terceras
personas de 6 das previos, presentando contusin abdominal y
torcica, el paciente refiere dolor abdominal difuso, motivo por el
cual acude a recibir atencin mdica en un dispensario, con
analgsicos y antibiticos, no especificados, se agrega la presencia
de dolor torcico y dificultad respiratoria hace tres das, adems de
astenia y adinamia, disfuncin abdominal, falta de canalizacin de
gases, as como tambin falta de evacuaciones, motivo por el
cualacude nuevamente a recibir atencin mdica. A pesar de todas
las medidas realizadas el paciente muere. El reporte de patologa
que se recibe posteriormente verifica la presencia de una apendicitis
aguda supurada, lo que confirma un proceso inflamatorio agudo que
coincide simultneamente con un antecedente de trauma
abdominal.
PREGUNTA
Cul es la causa mas frecuente de dolor abdominal en el anciano?
RESPUESTA
a.- Enfermedades biliares
b.- Obstruccin intestinal
c.- Tumores
d.- Causas vasculares

Pharmed Solutions Institute

PGINA 450

MANUAL DE TRABAJO DEL CURSO ENARM CMN SIGLO XXI


COLITIS
CIENCIAS BASICAS: Sndrome gastrointestinal que incluye dolor o molestias abdominales asociado al hbito intestinal en un periodo de 3 meses.
SALUD PUBLICA: 10-20% poblacin general. 20-35% en poblacin econmicamente activa. PATOGENIA: Multifactorial; Sobrecrecimiento bacteriano
en intestino delgado, variacin en niveles de serotonina en la mucosa intestinal, incremento en la Colgena Tipo 5. DIAGNOSTICO: Clnico; Dolor
abdominal (postprandial, preevacuatorio, asociado al estado emocional), distencin abdominal (postprandial, asociado al estado emocional,
evolutivo durante el da). Cambios en el hbito intestinal; calibre, frecuencia, morfologa, consistencia. Sntomas extraintestinales; cefalea,
lumbalgia, fatiga, mialgias, urgencias urinaria, psiquitricos. Sntomas de alarma; >50 aos, prdida de peso, AHF de Cncer, fiebre, sangrado,
lesiones drmicas. CRITERIOS DE ROMA III: Dolor o molestia abdominal al menos 3 das al mes en los ltimos 3 meses asociado a los siguientes
sntomas. Mejora con la defecacin. Inicio asociado a cambio en la frecuencia y forma (habito intestinal). Criterios revisados en los ltimos 3 meses
e inicio de la sintomatologa en los ltimos 6 meses. TRATAMIENTO: Tratamiento mdico: Dieta, fibra, actividad fsica (ejercicio en grupo). Diarrea;
loperamida, rifaximina, colestiramina, amitriptilina, fibra. Estreimiento; cinitaprida, mebeverina, tegaserod, dimeticona, simeticona, fibra. Dolor
abdominal; antidepresivos tricclicos: imipramina, amitriptilina. Probiticos. COLITIS PESEUDOMEMBRANOSA: Este padecimiento se observa en
pacientes que reciben antibiticos de amplio-espectro (clindamicina, penicilinas, semisintticas, cefalosporinas).Factores de riesgo: Diabetes
Mellitus, VIH, oncolgicos, trasplantados, postquirrgico inmediato. Salud pblica: 7% de adultos son portadores asintomticos. 13-21% adultos
asintomticos hospitalizados. Hasta 40% de los neonatos <6 meses. Ms comn en mujeres. Mortalidad de 2-4%. Pacientes >70 aos con +5 das con
AB riesgo de 34%. PATOGENIA: Agente causal ms frecuente en diarrea nosocomial (C. difficile) asociada a antibiticos. La alteracin de la flora
colnica normal permite el crecimiento excesivo de C. difficile, un anaerobio que produce una exotoxina, con efecto nocivo sobre la mucosa del
colon. El uso de antibiticos, induce esporulacin y crecimiento bacteriano logartmico, la toxina A y B producen dao citolgico y lesin epitelial,
generando paseudomembranas. Este sndrome puede presentarse hasta 6 semanas despus del tratamiento antibitico. DIAGNOSTICO: El espectro
clnico vara desde diarrea 93-98% (acuosa, moco, sangre) ligera autolimitada, fiebre 65-70%, dolor abdominal 85-90% (clico, abdomen agudo)
hasta inflamacin transmural intensa, colon toxico y perforacin. La leucocitosis no guarda proporcin con los datos clnicos. El diagnostico se
establece mediante el anlisis fecal en busca de la toxina y por endoscopia, que demuestra las paseudomemebranas amarillentas tpicas, ms en
recto y sigmoides. TAC: Engrosamiento de la pared colnica, pliegues internos a la luz, signo de T-bone Stea . CLASIFICACION: Tipo 1 (summit
lesion); Alteracin focal del epitelio superficial interglandular, infiltracin de clulas de reaccin en la lmina propia, fibrina subepitelial. Tipo 2
(volcano lesion); Prdida del patrn glandular, abundante infiltracin de la lmina propia, aparicin de psuedomembranas superficiales.
TRATAMIENTO: Ayuno. Suspender Antibiticos. Correccin de balance hdrico y electroltico. Metronidazol (500mg c/6hrs por 10-14 das) por va
oral o IV (750mg c/8hrs por 10-14 das), primera eleccin. La vancomicina (125mg c/6hrs por 10-14 dias) es una alternativa, pero debe evitarse
siempre que sea factible a causa del riesgo del surgimiento de enterococos resistentes a este antibitico. Indicadas cuando no se tenga disponible,
alergia o sensibilidad al Metronidazol o Vancomicina: Bacitracina, teicoplanina, colestiramina, colestipol. Los pacientes con colon toxico o
perforacin se someten a reseccin. El ndice de recurrencia despus del final del tratamiento se acerca al 20%.
CASO CLINICO
Mujer de 66 aos de edad, alrgica a la penicilina. Ingresa para
ciruga electiva de divertculos colnicos. Se le practic
sigmoidectoma asistida por laparoscopia. Como profilaxis
antibitica se utiliz clindamicina y gentamicina. Durante el
postoperatorio presenta un cuadro de abdominalgia, febrcula y
diarrea. En los anlisis aparece leucocitosis y desviacin a la
izquierda. Se practica tomografa computarizada, con resultado
normal. Se decide reoperar ante la progresin del cuadro, y no se
halla dehiscencia de sutura ni isquemia. La evolucin es trpida, y la
paciente fallece. Los cultivos de heces fueron positivos para la
toxina. La anatoma patolgica mostr los hallazgos tpicos de colitis
seudomembranosa.
PREGUNTA
El agente etiolgico de la colitis pseudomembranoso es:
RESPUESTA
a.- C. difficile.
b.- E. coli.
c.- Enteroco fecalis.
d.- Enterobacter.
CASO CLINICO
Varn de 64 aos ingres por diarrea infecciosa. Destacan tambin
como antecedentes varias infecciones respiratorias en los 2 meses
previos, por los que haba recibido tratamiento con cefepime,
amikacina y amoxicilina-clavulnico, y portador de una sonda de
nutricin enteral. Al ingreso presentaba 10-15 deposiciones
diarreicas/da, acuosas, acompaadas de dolor abdominal de tipo
clico y febrcula. En la analtica al ingreso slo destacaba
leucocitosis de 13,000 y anemia leve. Se inici tratamiento con
metronidazol i.v. 500mg/8 h ante la sospecha de infeccin por C.
difficile, tras toma de muestras para toxina. Se realiz rectoscopia al
segundo da de ingreso que confirma colitis seudomembranosa. Al
tercer da del ingreso, el paciente presenta deterioro clnico

CURSO ENARM CMN SIGLO XXI TEL: 36246001

importante, con disminucin del nmero de deposiciones y aumento


del dolor abdominal, que se hizo continuo, difuso, acompaado de
distensin abdominal e hipotensin. En la analtica destaca
leucocitosis de 23,000, insuficiencia renal, con creatinina de 190
mg/dl y urea de 12,6 mg/dl, y lactato srico de 4 mmol/l. Una TC de
abdomen muestra engrosamiento y edema difuso de todo el colon,
ms evidente en sigma y colon descendente.
PREGUNTA
El tratamiento mdico ideal para colitis pseudomembranosa es?
RESPUESTA
a.- Amikacina
b.- Ciprofloxacino
c.- Penicilina
d.- Vancomicina
CASO CLINICO
Mujer de 29 aos de edad acude con su medico de atencion primaria
por episodios repetidos de dolor en abdomen, que los ha tenido
desde que estaba en la universidad, pero que han empeorado, luego
de su divircio hace un ao. El dolor por lo general se localiza
alrededor del ombligo y lo describe como de tipo colico. Agrega que
los episodios de dolor se exacerban al comer y solo se alivian con la
defecacin. A si mismo suele experimentar meteorismo. Las
defecaciones recientes han sido aguadas y acuosas con episodios
interpuestos de estreimiento. Niega fiebre o perdidavde peso.
PREGUNTA
Cul es la conducta diagnotica mas adecuada para este caso?
RESPUESTA
a.- Ultrasonido abdominal
b.- Diagnostico de exclusin
c.- Coproparacitoscopico
d.- Radiografa de abdomen

Pharmed Solutions Institute

PGINA 451

MANUAL DE TRABAJO DEL CURSO ENARM CMN SIGLO XXI


ENFERMEDAD INTESTINAL INFLAMATORIA (CUCI, CROHN):
CIENCIAS BASICAS: La enfermedad intestinal inflamatoria, incluye colitis ulcerosa y enfermedad de Crohn, ambas se atribuyen a un defecto en la
regulacin inmunolgica del tubo digestivo que origina una reaccin inmunolgica descontrolada a diferentes antgenos. CUCI: La colitis ulcerativa
casi siempre se limita al intestino grueso, afecta la mucosa (colitis mucosa), y es continua desde el recto hacia arriba, y por lo general afecta solo el
colon y parte ms distal del leon. Las fistulas son raras. Una forma ms ligera de colitis ulcerativa puede afectar la porcin distal del intestino grueso,
el recto o el recto y el colon sigmoide. Se conoce como proctitis ulcerativa o proctosigmoiditis ulcerativa. 90% de estos pacientes responden al
tratamiento mdico y nunca experimentan pancolitis. CROHN: Enfermedad de Crohn puede daar cualquier parte del tubo digestivo, desde la boca
hasta el ano y suele comprometer todo el grosor de la pared abdominal, la mayora de los pacientes presenta compromiso del intestino delgado a
menudo con una distribucin discontinua (segmentos saltados). Los estrechamientos son usuales, la enfermedad perianal (fistulas, abscesos,
fisuras), son comunes y tienen un potencial maligno mayor que la CUCI. DIAGNOSTICO: Los cuadros clnicos se sobreponen y en 15% de los
pacientes la colitis es indeterminada. Por lo general los sntomas se deben a la inflamacin intestinal e incluyen diarrea, hemorragia rectal, tenesmo,
dolor abdominal, fiebre y prdida de peso. Es posible que se afecten otros rganos como el sistema musculoesqueltico (espondilitis, artritis), la piel
(eritema nodoso, pioderma gangrenosa), el ojo (iritis), sistema hematopoytico, riones, vas biliares. TRATAMIENTO: Es similar para ambos; la
sulfazalacina, base de tratamiento anterior, ya casi se sustituy por el cido 5-aminosalicilico (5-ASA), el cual tiene muchos menos efectos
colaterales y es mejor tolerado por los pacientes, las etapas agudas se controlan con esteroides, casi siempre prednisona en dosis 20-80mg/da, con
reduccin rpida de dosis. Los pacientes ms graves deben hospitalizarse e iniciar soluciones intravenosas, reposo intestinal y esteroides
intravenosos. La operacin casi siempre colectoma abdominal, se realiza si no se observa respuesta al tratamiento mdico en unos cuantos das. La
funcin de los inmunomoduladores como la 6-mercaptopurina (6MP) y aziatropina se incrementa conforme su eficacia y seguridad se establecen
cada vez ms. Su aplicacin principal es el tratamiento de mantenimiento y como ahorradores de esteroides en los pacientes con sntomas
persistentes mientras reciben solo preparaciones de 5-ASA. Tratamiento quirrgico: En CUCI, ya que la inflamacin se limita al intestino grueso, la
reseccin del rgano es curativa, a diferencia de la enfermedad de Crohn, que no puede erradicarse por medios quirrgicos y tienen un alto ndice
de recurrencia despus de la reseccin, la ciruga es la proctocolectoma reconstitutiva (se extirpa colon y parte superior de recto, se quita la mucosa
del recto restante y se construye un reservorio ileal (saco), con el leon terminal; luego se une con el reservorio a nivel de la lnea dentada. En
enfermedad de Crohn la ciruga est reservada para complicaciones que no responden a tratamiento conservador, casi siempre implica reseccin del
segmento intestinal, afectado.
CASO CLINICO
Paciente masculino de 33 aos que ingresa con el idx de brucelosis
crnica con tratamiento emprico ciprofloxacino durante 4 ocasiones
por periodos de 2 semanas con recidiva de la misma sintomatologia
(fiebre intermitente sin predominio de horario, dolor abdominal en
region del epigastrio y rara vez con irradiacion a marco colonico,
nauseas, vomito al inicio siempre del padecimiento y hematoquezia
sin diarrea. EF: abdomen blando depresible no doloroso en el
momento, no se identifica masa megalias. no hay signos de irritacin
peritoneal. Peristaltismo positivo de caracteristicas normales. Se
realiza colonoscopia ciego, el cual es de caractersticas
macroscpicas normales, se observa valvula ileocecal, se canula la
misma y se exploran 15cms del ileon terminal el cual es normal, el
colon ascendente y transverso conservan su morfologa y patrn
vascular, el colon descendente y rectosigmoides en algunas reas
conservan su patron vascular y en otral esta perdido, con imagen
tubular. Se toma biopsia de de colon ascendente y transverso, as
como de colon descendente y recto de reas afectadas. Se
diagnostica CUCI.
PREGUNTA
Cual de los siguientes manifestaciones son ms caractersticas de
CUCI?
RESPUESTA
a.- Estenosis, perdida de la vascularidad, ulceras, seudopolipos y
granularidad.
b.- Estenosis, perdida de la vascularidad, ulceras, polipos y
granularidad.
c.- Estenosis, perdida de la vascularidad, ulceras, seudopolipos y
agranularidad.
d.- Estenosis, aumento de la vascularidad, ulceras, seudopolipos y
granularidad
PREGUNTA
En la escala de Baron. En que estado se encuentra el caso clnico?
RESPUESTA
a.- 0.
b.- 1.
c.- 2.
d.- 3.
PREGUNTA

CURSO ENARM CMN SIGLO XXI TEL: 36246001

Cual de las siguientes afecciones extra-intestinales es la menos


frecuente en el CUCI?
RESPUESTA
a.- Artritis.
b.- Axial Espodilitis Anquilosante.
c.- Estomatitis.
d.- Glomerunefritis.
CASO CLINICO
Paciente de 19 aos que consulta por un cuadro de fiebre de 39,
acompaada de anorexia, prdida de peso de aprox., 3 Kg a lo largo
de 4 meses, dolor abdominal periumbilical no clico, sin nuseas, ni
vmitos; adems presenta episodios de diarrea, en nmero de dos a
tres al da, de consistencia blanda, no acompaada de productos
patolgicos (moco, sangre o pus). Presenta amenorrea desde hace 5
meses y no tolera ingesta de hierro. Como antecedentes personales
destaca apendicectoma que no fue confirmada histolgicamente. A
la exploracin fsica presenta buen estado general y se encuentra
bien hidratada. Peso 40,3Kg, talla 163 cm (IMC: 15). Palidez cutnea
+,
PREGUNTA
Cul es el diagnstico ms probable para este caso?
REPUESTA
a.- Colitis ulcerativa crnica inespecfica
b.- Linfoma
c.- Enfermedad de Crohn
d.- Infeccin por Clostridium difficile
PREGUNTA
Qu reas del tracto digestivo y que capas del mismo, es ms
probable que se afecten en este paciente?
RESPUESTA
a.- Colon y recto, capa mucosa y submucosa
b.- Todo el tracto digestivo, capa mucosa y submucosa
d.- Colon y recto, capas transmural
c.- Todo el tracto digestivo, capas transmura
PREGUNTA
Cul de las siguientes manifestaciones extraintestinales es la ms
probablemente relacionada con este cuadro?
RESPUESTA

Pharmed Solutions Institute

PGINA 452

MANUAL DE TRABAJO DEL CURSO ENARM CMN SIGLO XXI


a.- Pioderma gangrenoso
b.- Cirrosis
c.- Eritema nodoso
d.- Nefrolitiasis

quirrgica en la que se le realiza una amputacin abdominoperineal,


resecndole el recto. La evolucin en el postoperatorio transcurre
sin complicaciones hasta que en el dcimo da sufre una convulsin
tnico-clnica generalizada de 3 minutos de duracin, sin
incontinencia de esfnteres.

CASO CLINICO
Femenina de 63 aos, ama de casa. Antecedentes personales
patolgicos: DM tipo 2 hace 20 aos en tratamiento con Insulina
NPH SC 40 U a.m. y 18 U p.m., Metformina 500 mg VO. Hipertensin
arterial hace 25 aos en tratamiento con Atenolol 50 mg VO/da.
Antecedentes personales no patolgicos: Tabaquismo por ms de 30
aos. Antecedentes heredo familiares: Madre DM2 y Asma
Persistente Moderada. Padre Cardiopata Isqumica. Antecedentes
Quirrgicos: Salpingooforectoma Bilateral hace 20 aos,
Colecistectoma hace 20 aos. Padecimiento Actual: Paciente
presenta rectorragia de 2 aos de evolucin, asociada a moco y pujo,
de leve a moderada cantidad, continuo, frecuentemente matutino,
tanto a la miccin como a la defecacin, adems de diarrea
secretora ocasional. Niega otra sintomatologa. Al examen fsico no
se encontraron datos importantes. Especuloscopa: normal. Tacto
vaginal: tero en AVF, no aumentado de tamao, anexos libres.
Tacto rectal: Esfnter normo tnico, heces escasas en mpula, no se
palpan hemorroides internas, no se palpan masas, sangrado
moderado en guante. Paciente es referida para colonoscopa.
PREGUNTA
Cul es el diagnstico ms probable para este caso?
a.- Colitis ulcerativa crnica inespecfica
b.- Cncer de colon
c.- Enfermedad de Crohn
d.- Infeccin por Clostridium difficile
PREGUNTA
Cul es la complicacin ms comn que podra presentar este
paciente?
RESPUESTA
a.- Fisuras anales
b.- Perforacin
c.- Megacolon toxico
d.- Hemorragia masiva
PREGUNTA
Cul de las siguientes manifestaciones extraintestinales es la ms
probablemente relacionada con este cuadro?
RESPUESTA
a.- Pioderma gangrenoso
b.- Cirrosis
c.- Eritema nodoso
d.- Nefrolitiasis
CASO CLINICO
Varn de 44 aos con enfermedad de Crohn diagnosticado 15 aos
antes, que ingresa en el servicio de Ciruga General por un cuadro de
dolor abdominal y diarrea de una semana de evolucin, compatible
con un brote de su enfermedad. En tratamiento domiciliario con
corticoides y mesalazina, el paciente es portador de una ileostoma
desde hace 2 aos tras realizarle una colectoma subtotal, con
mun rectal cerrado a raz de un brote de su enfermedad.
Previamente se le haban realizado varias operaciones resectivas
como la extirpacin de los ltimos 40 centmetros de leon, vlvula
ileocecal y ciego, tras una perforacin ileocecal, as como otras
intervenciones por fstula anal con absceso en glteo, para su
drenaje correspondiente. Durante el ingreso actual desarrolla una
infeccin perianal polimicrobiana resistente al tratamiento mdico
con antibiticos, por lo que es sometido a una intervencin

CURSO ENARM CMN SIGLO XXI TEL: 36246001

PREGUNTA
Qu electrolito es mas probable que este participando en la
presentacin de crisis convulsivas de este paciente?
RESPUESTA
a.- Sodio
b.- Cloro
c.- Magnesio
d.- Potasio
PREGUNTA
Despues de un tiempo el paciente empieza a recibir tratamiento
con infliximab, por resistencia a metilprednisolona. Qu efecto
adverso pudiera presentar el paciente con el nuevo medicamento?
RESPUESTA
a.- Diarrea
b.- Fiebre y escalofros
c.- Cafalalgia
d.- Hipertension
CASO CLINICO
Femenino de 28 aos de edad acude con su medico, por dolor
recurrente en el abdomen. Informa que durante 6 meses ha tenido
episodios diarios de dolor de tipo colico en la parte baja del
abdomen y diarrea. La diarrea en ocasiones ha sido sanguinolenta y
la paciente ha experimentado perdida involuntaria de 5 kilogramos,
de peso durante los 3 meses pasados. Aade que tuvo fiebre
subjetiva varias noches de la semana anterior. EF: Revela
temperatura de 37.9C, pulso de 90/min, TA 110/70mmHg. A la
exploracin de abdomen hay hipersensibilidad difusa y el hallazgo
importante en un examen rectal es sangre manifiesta.
PREGUNTA
Cul es el diagnotico mas probable para esta paciente?
RESPUESTA
a.- Colitis membranosa
b.- Enfermedad de Crohn
c.- Colitis ulcerosa
d.- Colon irritable

PREGUNTA
Cul es la conducnta diagnostica mas adecuada para un diagnostico
definitivo?
RESPUESTA
a.- Colonoscopia
b.- Coprocultivo
c.- Biopsia
d.- Enema de bario
PREGUNTA
Cul es la conducnta teraputica mas adecuada para inducir
remisin en enfermedad moderada a grave?
RESPUESTA
a.- Sulfasalazina
b.- Mesalamina
c.- Corticoesteroides
d.- Quirurgico

Pharmed Solutions Institute

PGINA 453

MANUAL DE TRABAJO DEL CURSO ENARM CMN SIGLO XXI


HERNIAS DIAFRAGMATICAS, HIATAL, DE PARED E INGUINAL
DE PARED BDOMINAL. CIENCIAS BASICAS: Una hernia es la protrusin de una vscera a travs de una abertura en la pared (natural o adquirido) de
la cavidad que la contiene, las caractersticas importantes de una hernia son el orificio defecto en la capa aponeurtica ms interna del abdomen) y
el saco herniarios (protrusin del peritoneo). El cuello del saco herniario corresponde al orificio. Sitios de herniacin: son la ingle (inguinal), la cicatriz
umbilical (umbilical), la lnea alba, la lnea semilunar de Spiegel, el diafragma y las incisiones quirrgicas (postincisional). SALUD PUBLICA: Incidencia
10% poblacin general. 90% de hernias inguinales/incisionales. CLASIFICACION: La hernia es externa si el saco protruye completo a travs de la
pared abdominal; es interno si el saco est dentro de la cavidad visceral. Una hernia es reducible cuando la vscera herniada puede regresar al
abdomen; es irreducible cuando esta resulta imposible. Una hernia estrangulada es aquella en la que la vascularidad de la vscera que protruye se
compromete hay sufrimiento del contenido (isquemia), necrosis y puede llegar a sepsis (translocacin bacteriana). El estrangulamiento ocurre en
hernias con orificios pequeos y sacos grandes. Una hernia incarcerada es una que no puede reducirse pero que no est estrangulada. En una hernia
de Richter el contenido del saco consiste en solo un lado de la pared del intestino (siempre es antimesentrica). PATOGENIA: La herniacin puede
abarcar la grasa preperitoneal, rganos retroperitoneales y un saco herniado de peritoneo con estructuras intraperitoneales (por ejemplo, epipln u
rganos). Son poco comunes las hernias clnicamente significativas que carecen de saco peritoneal. Las hernias pueden complicarse cuando la
inclusin de una vscera forma una pared del saco herniario. Ello comprende un rgano parcialmente retroperitoneal y recibe el nombre de hernia
por deslizamiento, de las cuales las ms comunes afectan al colon. Etiologa: multifactorial causas enfermedad de tejido conectivo, tabaquismo,
EPOC, actividad fsica, edad avanzada. Por debilitamiento de la fascia transversalis. Aumento de la presin intraabdominalen: Embarazo, ascitis,
obesidad, obstruccin intestinal, esfuerzo defecatorio, prostatismo, ejercicio (Pesas). DIAGNOSTICO: Clnica; la historia natural de las hernias es un
crecimiento lento hasta llegar a ser irreducibles y desfigurantes, con riesgo de estrangulaciones. Las molestias que producen siempre son peores al
final del da y se alivian por la noche cuando el paciente se acuesta y la hernia se reduce. El dolor inguinal sin una hernia demostrable, casi nunca
indica el inicio de una hernia. La mayor parte tiene un inicio insidioso, pero algunas se desencadenan por esfuerzo muscular intenso. Por lo general
un saco herniario con su contenido crece y transmite un impulso palpable cuando el paciente hace esfuerzos o tose. El paciente debe ponerse de pie
en la exploracin, porque suele resultar imposible palpar una hernia inguinal reducida con el paciente en posicin supina. Las hernias que no son
detectables mediante exploracin fsica pueden demostrarse mediante ultrasonido o TAC. El estrangulamiento produce dolor intenso en la hernia
seguido muy pronto por hipersensibilidad, obstruccin intestinal y manifestaciones de sepsis. La reduccin de una hernia estrangulada est
contraindicada cuando hay sepsis o si se cree que el contenido del saco presenta gangrena. TRATAMIENTO: Casi todas las hernias deben repararse a
menos de que las condiciones locales o sistmicas del paciente impidan un resultado favorable. La posible excepcin es una hernia con cuello amplio
y saco poco profundo, en cuyo caso se espera que crezca despacio. Los bragueros son tiles en el tratamiento de hernias pequeas cuando la
operacin esta contraindicada, pero estn contraindicados en pacientes con hernias femorales. Reparacin laparoscpica: Actualmente indicada en
prcticamente cualquier tipo de hernia de pared, hernias incisionales extensas, hernias incarceradas con o sin crisis de oclusin. Contraindicaciones
absolutas: Eventraciones supergigantes con obesidad asociada y ausencia de elementos musculoaponeurticos, fstulas enterocutneas o foco
sptico intraabdominal, cirrosis heptica con ascitis libre, hernias con estrangulacin intestinal en fase temprana. HERNIAS INGUINALES: La ingle es
una de las reas dbiles por naturaleza de la pared abdominal y es el sitio ms frecuente de herniacin. Los varones tienen una probabilidad de 25
veces mayor de padecer una hernia inguinal. Las hernias que surgen por arriba del pliegue abdominocrural son inguinales, las que emergen por
debajo de ese pliegue son femorales. Las hernias femorales ocurren ocasionalmente en las mujeres, pero no con tanta frecuencia como las
inguinales; en los varones son raras. Las hernias femorales suelen presentarse como una masa irreducible en la base medial del tringulo femoral de
Scarpa. La incidencia aceptada es de 3-4 %. La estrangulacin se produce en 1.3-3.0% de las hernias inguinales. Las hernias inguinales se dividen en
directas e indirectas. Indirecta: Un saco herniario indirecto es un proceso vaginal con dilatacin persistente, su saco pasa en sentido oblicuo o en
forma indirecta hasta llegar al escroto, son dos veces ms frecuentes en los varones Directa: Los sacos se originan en el piso del trayecto inguinal
(tringulo de Hesselbach), protruye en formas directa y quedan contenidos por la aponeurosis del musculo oblicuo externo. Es frecuente que la
vejiga sea un componente deslizante de un saco herniario directo. Casi todas las hernias estranguladas son inguinales indirectas, pero las femorales
tienen el mayor ndice de estrangulacin. Diagnstico: Se diagnostica mediante la exploracin fsica del canal inguinal y regin crural. Aumento de la
regin inguinal. Aumenta con el esfuerzo, disminuye con el reposo. Se asocia a dolor en la regin inguinal, distinguir si es reductible o irreductible.
Las hernias inguinales pueden ser congnitas o adquiridas. Todas las inguinales indirectas son congnitas. El objetivo de la hernioplastia inguinal es
prevenir la protrusin peritoneal a travs del orificio miopectneo. La integridad se recupera de dos formas fundamentales: 1) cierre aponeurtico
del orificio miopectneo hasta la magnitud necesaria y 2) reposicin de la fascia transversal defectuosa con unja prtesis sinttica grande. A veces se
combinan ambos mtodos. Las hernias se reparan por va anterior a travs de una incisin inguinal o posterior por una incisin abdominal. El
abordaje anterior es la incisin ms popular para la hernioplastia inguinal. Las reparaciones posteriores se denominan hernioplastias
properitoneales. Las protecciones sintticas de malla tienen una funcin importante en el tratamiento de las hernias inguinales. Las prtesis de
malla se usan como parche o tapn del orificio miopectneo para reforzar una reparacin clsica y para reponer la fascia transversal. HERNIA
UMBILICAL: Son ms frecuentes en mujeres. Constituyen el 2.4% de los casos de Hernias de pared. 10% en adultos con antecedente de hernia en la
infancia. La obesidad y los embarazos repetidos son los precursores usuales. Suele observarse estrangulacin de colon y epipln. Las hernias de este
tipo que tienen un defecto parietal pequeo solo se cierran con sutura de polipropileno; aquellas con grandes defectos parietales se corrigen con
una prtesis. HERNIA DE SPIEGEL: Son hernias ventrales que ocurren a lo largo de la porcin subumbilical de la lnea semilunar de Spiegel y a travs
de la fascia del mismo nombre. Estas hernias son raras (1%) y a menos que sean grandes, su diagnstico se dificulta porque son interparietales y
estn contenidas por la aponeurosis del musculo oblicuo externo. Son ms frecuentes en el rea entre la cicatriz umbilical y la lnea que conecta las
espinas ilacas anterosuperiores, y el rea por debajo de la lnea arqueada y por arriba de los vasos epigstricos inferiores. 21% incareceradas.
HERNIA EPIGASTRICA: Es una protrusin de grasa properitoneal y peritoneo a travs de las fibras cruzadas de la vaina del recto en la lnea media
(alba), entre el apndice xifoides y la cicatriz umbilical. Las hernias epigstricas a menudo son irreducibles, siempre tienen defectos aponeurticos
pequeos, pueden ser mltiples. HERNIAS INCISIONALES: Son problemas quirrgicos graves. La obesidad y la infeccin son las causas principales, de
este trastorno, otras dehiscencia de la herida, hematoma o seroma, tcnica deficiente en el cierre de la herida, desnutricin. El peso del panculo
separa la incisin quirrgica y la infeccin obstaculiza la cicatrizacin de la herida. Una hernia incisional grande produce movimiento abdominal
respiratorio paradjico similar al trax inestable. La funcin diafragmtica se vuelve ineficiente. La mayor parte de las hernias incisionales pequeas
se trata con cierre simple de defecto aponeurtico. Las hernias con defectos aponeurticos mayores de 10 cm tienen ndices de recurrencia hasta de
50%. Por tanto muchas de las hernias incisionales y todas las recurrentes requieren una prtesis para conseguir una reparacin exitosa. Se prefiere
la hernioplastia de Stoppa (se implanta una prtesis muy grande de mersilene en la profundidad de los msculos de la pared abdominal sobre la
vaina posterior del recto o el peritoneo). La infeccin es una complicacin grave que se presenta en un 10% de los pacientes. La infeccin temprana
se trata mediante exposicin rpida y completa de la prtesis. Con tratamiento antimicrobiano intenso local y sistmico puede esperarse la
integracin completa de la prtesis. La reintegracin de la prtesis infectada casi nunca se logra cuando hay infecciones tardas y debe retirarse la

CURSO ENARM CMN SIGLO XXI TEL: 36246001

Pharmed Solutions Institute

PGINA 454

MANUAL DE TRABAJO DEL CURSO ENARM CMN SIGLO XXI


porcin secuestrada de la prtesis. HERNIA DE AMYAND: Aquella que contiene el apndice cecal inflamado a travs de un defecto herniario
inguinal. HERNIA PARAESTOMAL: Estas interfieren con las irrigaciones de colostoma y la adhesin de los artculos para el cuidado de la estoma. Las
hernias paracolostomales son ms frecuentes que las hernias paraileostomales y la ocurrencia de ambas es ms probable cuando el estoma emerge
por la lnea semilunar en lugar de la vaina del recto. Por lo general las hernias parastomales son laterales a la ostoma. Se prefiere el cambio de sitio
de la estoma sobre la reparacin local. La reparacin local falla con frecuencia porque los msculos cinturn laterales a la ostoma carecen de
aponeurosis suficiente, por ello se prefiere implantar un trozo grande de mersilene con una hendidura para acomodar el estoma. HERNIA HIATAL.
CIENCICAS BASICAS: Es el prolapso del estmago proximal hacia el trax a travs del hiato esofgico del diafragma. La hernia hiatal es una condicin
esencialmente adquirida que no slo es la ms frecuente de las hernias diafragmticas, sino una de las anormalidades ms frecuentes que afectan el
TGI superior. El hiato esofgico del diafragma se encuentra localizado a la izquierda de la lnea media a nivel de la dcima vrtebra torcica; es un
anillo musculotendinoso compuesto por fibras de los pilares derecho e izquierdo del diafragma, que provienen de la cara anterior y discos
intervertebrales de las cuatro primeras vrtebras lumbares, rodean la aorta y el esfago y se insertan en el centro tendinoso del diafragma. Aunque
existen variaciones anatmicas, se ha encontrado que en ms del 80% de los casos, el hiato esofgico se forma principalmente de fibras del pilar
derecho del diafragma. CLASIFICACION: Tipo I o hernia hiatal por deslizamiento (axial) donde se presenta desplazamiento superior de la unin
esfago-gstrica hacia el mediastino posterior. Se denomina hernia por deslizamiento ya que tiene un saco de peritoneo parietal parcial, cuya
pared posterior est formada por el estmago. Tipo II o hernia paraesofgica caracterizado por desplazamiento superior del fondo gstrico, anterior
y lateral al esfago, con la unin esfago-gstrica localizada en su posicin intraabdominal normal. Tipo III o mixta donde hay desplazamiento
superior tanto de la unin esfago -gstrica como del fondo gstrico. El 85-90% de las hernias hiatales son de tipo I, en tanto que las hernias
paraesofgicas puras son encontradas muy infrecuentemente. SALUD PUBLICA: La incidencia de hernia hiatal se estima en 5 por 1000 en la
poblacin general, aunque una verdadera incidencia es difcil de determinar debido a que un gran nmero de pacientes son asintomticos. La edad
de presentacin ms frecuente es entre 4 a 6 dcada de la vida y no existe diferencia en cuanto al sexo, aunque las hernias paraesofgicas son ms
frecuentes en mujeres. DIAGNOSTICO: Cuando se hacen manifiestas, la sintomatologa est dada bsicamente por pirosis, regurgitacin y dolor
retroesternal, sntomas clsicos del RGE que es la manifestacin ms significante en los pacientes con hernia hiatal. Puede presentarse disfagia
generalmente asociada a esofagitis, estenosis pptica, anillo de Schatzki (pliegue mucoso en el esfago distal, dentro de los 3 mm proximal a la
unin escamocolumnar, que siempre est asociada con hernia hiatal y constituye una causa de disfagia en estos pacientes) o por la compresin de
los pilares diafragmtica en la porcin herniada del estmago. Las hernias paraesofgicas en general son asintomticas, incluso cuando alcanzan
grandes tamaos. Grandes herniaciones pueden producir dolor retroesternal o disnea por disminucin de la reserva respiratoria. Se puede presentar
disfagia secundaria a la compresin esofgica por la hernia o a la rotacin de la UEG dentro del saco herniario. Sntomas de RGE se presentan
tambin en un alto porcentaje. En pacientes con ERGE documentado, se ha encontrado adems una relacin directa entre el tamao de la hernia
hiatal y el grado de disfuncin del EEI, la duracin de los episodios de reflujo, el aclaramiento cido y por consiguiente con la severidad de la
esofagitis. La funcin como esfnter del diafragma est dada principalmente durante la inspiracin y situaciones de estrs dinmico, como durante la
deglucin y en incrementos sbitos de la presin intraabdominal (tos), de tal forma que los episodios de reflujo relacionados a incremento de la
presin intraabdominal son ms probables que ocurran en pacientes con hernia hiatal. Sangrado oculto o moderado puede encontrarse hasta en un
tercio de los pacientes con hernia hiatal por deslizamiento sintomtica, siendo el sangrado masivo poco frecuente. El volvulus gstrico puede ser
organoaxial cuando el estmago rota a lo largo de su eje longitudinal o mesoentero axial cuando rota sobre el eje que une la curvatura menor con la
mayor. La laxitud o ausencia de los ligamentos de fijacin gstrica son la causa primaria del volvulus gstrico. Al incrementarse la rotacin del
estmago se produce compromiso del flujo sanguneo y del retorno venoso del estmago herniado, llevando a necrosis, perforacin, sepsis e incluso
la muerte. Radiografa de trax se puede evidenciar una masa dependiente de tejidos blandos en el mediastino posterior, con nivel hidroareo en el
caso de grandes hernias. Los estudios con medio de contraste baritados son ms exactos si se logra definir la relacin del hiato esofgico del
diafragma con la UEG. La presencia de una indentacin por encima del diafragma, lo que usualmente se considera como el sitio de transicin entre el
cardias gstrico y el vestbulo esofgico, implica la existencia de una hernia hiatal. En el caso de hernias paraesofgicas, los estudios con medio de
contraste muestran la porcin del fondo gstrico situado por encima del diafragma con la UEG ubicada en posicin normal a nivel del diafragma;
adicionalmente puede demostrarse la presencia de un volvulus gstrico. Endoscopia: Normalmente la unin de la mucosa escamocolumnar (lnea Z),
que corresponde aproximadamente a la localizacin de la UEG, se encuentra a menos de 2 cm por encima del hiato diafragmtico, de tal forma que
una distancia mayor entre estas estructuras es consistente con la presencia de una hernia hiatal. La posicin del hiato diafragmtico puede hacerse
ms evidente durante una inspiracin profunda. TRATAMIENTO: Modificacin en el estilo de vida, uso de drogas supresoras de la secrecin cida y
agentes proquinticos. El manejo quirrgico de la hernia hiatal sintomtica tiene varias indicaciones, y/o por manifestaciones directas de la hernia
hiatal, como: 1. hernia hiatal encarcelada con disfagia, 2. dolor torcico asociado a hernia hiatal gigante, 3. pacientes con severa deficiencia de hierro
secundaria a las erosiones o ulceraciones en la hernia hiatal, 4. hernia paraesofgica. Existen varias opciones quirrgicas de funduplicatura total o
parcial, que pueden ser realizadas por va transabdominal (funduplicatura de Nissen, Hill, Toupet) o transtorcica; o por manifestaciones directas de
la hernia hiatal, como: 1. hernia hiatal encarcelada con obstruccin sintomtica y disfagia, 2. dolor torcico asociado a hernia hiatal gigante, 3.
pacientes con severa deficiencia de hierro secundaria a las erosiones o ulceraciones en la hernia hiatal, 4. hernia paraesofgica. Existen varias
opciones quirrgicas de funduplicatura total o parcial, que pueden ser realizadas por va transabdominal (fundoplicatura de Nissen, Hill, Toupet) o
transtorcica (funduplicatura de Nissen o Belsey - Mark IV) ya sea en forma abierta o por ciruga mnimamente invasiva, pero que en general
cumplen los mismos principios quirrgicos que son: reduccin de la hernia hiatal, cierre del hiato esofgico, restablecer la funcin del EEI,
reposicionar el esfago intraabdominal y crear un mecanismo de vlvula antirreflujo. En aquellos casos de acortamiento esofgico se debe realizar
una gastroplastia de Collis para alargar el esfago y posteriormente la funduplicatura total o parcial, sin tensin sobre el esfago.
CASO CLINICO HERNIA HIATAL
Paciente varn de 44 aos que consulta por cuadro de pirosis,
ardores y dolor epigstrico irradiado a zona retroesternal de dos
aos de evolucin. Fue tratado con omeprazol, metoclopramida sin
resultados adecuados, En estudio endoscpico se observa hernia
hiatal sin signos de esofagitis, que se confirma en trnsito
esofagogastricoduodenal donde adems se observa reflujo
gastroesofgico espontneo. En la manometra esofgica se aprecia
disminucin del tono del esfnter esofgico inferior. Se realiza
radiografa de trax donde se observa la existencia de masa de

CURSO ENARM CMN SIGLO XXI TEL: 36246001

densidad grasa que ocupa todo el ngulo cardiofrnico derecho,


hallazgos que se confirman en TAC toracoabdominal.
PREGUNTA
Considerando el cuadro clnico asi como los estudios de gabinete
cual es el tipo de hernia mas probable?
RESPUESTA
a.- Hernia hiatal tipo I
b.- Hernia hiatal tipo II.
c.- Hernia hiatal y de morgagni.
d.- Hernia hiatal y de bochdalek

Pharmed Solutions Institute

PGINA 455

MANUAL DE TRABAJO DEL CURSO ENARM CMN SIGLO XXI


PREGUNTA
Cual de las siguientes manifestaciones es mas frecuente encontrar
para pensar en esta entidad nosolgica?
RESPUESTA
a.- Dolor abdominal.
b.- Oclusin intestinal.
c.- Dificultad respiratoria.
d.- Regurgitacion gastrointestinal.
PREGUNTA
Se ha reportado esta asociacin en menos del 3 % de casos de
hernias hiatales, cual de los siguientes rganos es el menos
frecuente que se comprometa?
RESPUESTA
a.- Jejuno.
b.- Colon.
c.- Higado.
d.- Epipln.
PREGUNTA
De los siguientes paraclinicos es mas habitual para su diagnostico?
RESPUESTA
a.- TAC.
b.- Radiografia de torax.
c.- Endocopia.
d.- Estudio de transito esfago-gastro-duodenal.
PREGUNTA
Considerando el caso clnico asi como el diagnostico confirmado,
porque esta indicado el tratamiento quirugico definitivo y a la
brevedad.
RESPUESTA
a.- Riesgo a malignizacin.
b.- Riesgo de estrangulacin.
c.- Riesgo de broncoaspiracin.
d.- Riesgo a la invaginacin.
CASO CLINICO
Paciente del sexo masculino, de 61 aos de edad, con estatura de
1.80mts y 58Kg quien consult por presentar sintomatologa
digestiva alta de 9 aos de evolucin, caracterizada por
epigastralgia, acidez, sensacin de repletes gstrica y discreta
disnea; nuseas y vmito solo al ingerir comidas en cantidades un
poco mayores a las acostumbradas. Cuadro que haba progresado en
los ltimos meses, principalmente en lo que se refera al dolor en
epigastrio y retroesternal, por lo que acude a la consulta de ciruga
general, donde se le indica una EGD la cual reporta estudio
alteraciones en la visualizacin normal, encontrando un vlvulo
gstrico con zonas hipermicas en relacin a gastritis hemorrgica.
Se decide realizar estudio contrastado Rx. EED, encontrando toda la
cmara gstrica intratorcica y confirmando el vlvulo rganoaxial
del estmago.
PREGUNTA
Cul es la conducta teraputica ms a seguir en este caso?
RESPUESTA
a.- Tratamiento mdico ms endoscopia
b.- Tratamiento quirrgico por laparoscopia
c.- Tratamiento quirrgico por ciruga convencional
d.- Tratamiento mdico ms laparoscopia

PREGUNTA
Qu anomalas congnitas estn asociadas con el defecto
posterolateral congnito de diafragma del lado izquierdo?
RESPUESTA
a.- Hipoplasia pulmonar, cardiopata
b.- Hipertensin pulmonar, insuficiencia respiratoria
c.- Hipoplasia pulmonar, atresia de esfago
d.- Atresia de esfago, trasposicin de los grandes vasos
CASO CLINICO
Se presenta el caso de un paciente de sexo masculino, de 77 aos de
edad, que presenta un bulto irreductible en regin inguinal izquierda
y que refiere aisladamente molestias a dicho nivel. Antecedente de
de ciruga por un cncer de colon transverso con intencin curativa
hace 6 aos. La estadificacin referida fue un adenocarcinoma de
colon Estadio III C. El paciente fue perdido en el seguimiento
postoperatorio por lo que no realiz ningn tipo de adyuvancia
postoperatoria. Al examen fsico, se palpaba una masa slida duroptrea indolora y que no pudo reducirse con las maniobras
habituales. No tena signos de atascamiento o estrangulacin de la
hernia. El ultrasonido revel a nivel de la regin inguinal izquierda un
saco herniario de contenido heterogneo, principalmente epipln
mayor, lquido y engrosamiento peritoneal. En este mtodo de
imagen no se detect contenido intestinal.
PREGUNTA
Cul es la conducta diagnostica ms adecuada para el diagnstico
definitivo de una hernia de pared?
RESPUESTA
a.- Clnica
b.- Radiografa
c.- Tomografa
d.- Ultrasonido
PREGUNTA
Cul es la contraindicacin menos probable para realizar una
ciruga laparoscpica para reparacin de una hernia?
RESPUESTA
a.- Hernia incacerada
b.- Eventracion supergigante con obesidad asociada
c.- Cirrosis heptica con ascitis libre
d.- Fistulas enterocutaneas
PREGUNTA
Cul es la hernia que ms probablemente contenga el apndice
cecal inflamado a travs de un defecto herniario inguinal?
RESPUESTA
a.- Hernia de Spiegel
b.- Hernia de Littre
c.- Hernia de Amyand
d.- Hernia de Richter
CASO CLINICO HERNIAS
Paciente de 21 aos de edad que acude a consulta por presencia de
dolor en abdomen sealando la zona umbilical, a la exploracin fsica
se observa abdomen con masa depresible levemente dolorosa, que
protruye espontneamente al pujar y al incorporarse.

PREGUNTA
Con que presin del esfnter esofgico inferior es ms probable
que se indique la reparacin quirrgica?
RESPUESTA
a.- 3-6mmHg
b.- 10-15mmHg

CURSO ENARM CMN SIGLO XXI TEL: 36246001

c.- 15-20mmHg
d.- 20-25mmHg

PREGUNTA
Cul de las siguientes medidas no est indicado en este paciente?
RESPUESTA
a.- Confirmar con laboratorio y gabinete con ECG.
b.- Realizacin de estudios preoperatorios.
c.- Envi a segundo nivel y seguimiento posoperatorio.
d.- Confirmar hernia umbilical no complicada.

Pharmed Solutions Institute

PGINA 456

MANUAL DE TRABAJO DEL CURSO ENARM CMN SIGLO XXI


HEMORROIDES
CIENCIAS BASICAS: Las hemorroides son dilataciones de los plexos hemorroidarios superior e inferior. Estn localizadas en los ltimos centmetros
del recto, en el conducto anal y en el recto. Forman parte de la anatoma normal de la regin y cuando sufren alteraciones y producen sntomas se
establece la enfermedad. Factores de riesgo: Multifactorial, estreimiento, dieta, embarazo, edad, herencia?. SALUD PBLICA: En Mxico, 5% de la
poblacin general presenta sntomas relacionados con las hemorroides. Son raras antes de los 20 aos de edad; su frecuencia aumenta con la edad y
es posible que el 50% de los adultos de 50 aos tenga o haya sufrido sintomatologa hemorroidaria. Incidencia de 4 a 36%. Incidencia mxima de 3060 aos. Sin diferencia racial. Ms en mujeres pero mayor severidad en hombres. PATOGENIA: Estreimiento, al implicarse mayor esfuerzo al
evacuar, se produce congestin de los cojinetes hemorroidales. Al persistir el pujo constante se distienden los soportes de dichos cojinetes y va
provocando el prolapso hemorroidario por debajo de la lnea anorectal hacia fuera del conducto anal. Alteraciones morfolgicas del Traitz.
Elongacin de las estructuras hemorroidales. Cambio del tejido elstico. Hipertona del EAI. Cambio irreversible en los vasos. CLASIFICACIN:
Anatmica 1. Internas (cubiertas por mucosas): I-IV, grado I; son cuando stas se exteriorizan hasta el conducto anal y sangran. Grado II; son aquellas
que sangran y se prolapsan al momento de la defecacin a nivel de ano pero se reducen espontneamente. Grado III; son aquellas que sangran y se
prolapsan ms all del ano al momento de la defecacin y requieren reduccin manual. Grado IV; son las que sangran y se prolapsan en forma
permanente y son irreductibles. 2. Externas (cubiertas por piel): vascular, pliegue. 3. Mixtas. De acuerdo a su situacin: Complicadas; edema,
trombosis, sangrado, absceso. No complicadas; sangrado 89% (Rutilante, ordea, no doloroso, postevacuatorio, abundante en el papel), prolapso
36% (Pujo, asociado a la esfuerzo, reduccin), prurito 27% (Posterior a la evacuacin, aseo deficiente, aseo excesivo), dolor 11% (solo con
complicaciones). DIAGNOSTICO: Clnico; Hemorroides externas, sntomas y signos principales dolor, tumoracin y prurito anal. Hemorroides
internas, fundamentalmente son dos: hemorragia y prolapso hemorroidario, la hemorragia: es el signo ms frecuente y ms constante. La sangre por
lo regular es roja, rutilante, expulsada con la metera fecal, en forma de estras en las heces fecales o en gotas al final de la evacuacin. Inspeccin;
Identificar hemorroides externas o prolapso de internas. Palpacin; Pliegues externos, tejido hemorroidal redundante. Anoscopa, Identificacin y
visin directa de los paquetes. TRATAMIENTO: Mdico: Incremento en el consumo de agua, consumo de Fibra (30 gr), flebotnicos: Hidrosmina
(sies) 200mg cada 8 horas 5 das, Diosmina (daflon) 450mg cada 8 horas 5 das, Hisperidina (eletec) 50mg cada 8 horas 5 das. Ligadura con banda
elstica. Escleroterapia: Esta consiste en la inyeccin de pequeas cantidades de hidroxipolietoxidodecanol al 3% aplicado en la submucosa y
extravascular por arriba del paquete vascular externo. Crioterapia: Se utiliza xido nitroso que congela a una temperatura de menos 60 C y menos
80 C o con nitrgeno lquido que congela a una temperatura de menos 180 C. El principio es la destruccin del tejido por medio de la coagulacin.
Electrocoagulacin: Se basa en un rayo calrico que evapora el agua intracelular y coagula las protenas. Se recomienda su aplicacin de 1.5
segundos en nmero de 3 a 4 en la base del tejido hemorroidario interno. Tratamiento Quirrgico: Milligan- Morgan. Tipo Parks (semiabierta).
Ferguson (cerrada). PPH (Procedure for Prolapse and Hemhorroids). El tratamiento quirrgico est indicado en todos los pacientes en los que se ha
fracasado el tratamiento mdico, en aquellos pacientes que tienen sintomatologa de hemorroides externas, en hemorroides internas grado III y IV y
en las complicaciones de la enfermedad hemorroidaria. COMPLICACIONES: Trombosis hemorroidaria nica externa; Es una complicacin frecuente
de causa desconocida. En forma clsica el paciente refiere dolor de forma brusca, acompaado de tumoracin perianal. El tratamiento puede ser
mdico o si el dolor es muy intenso, est indicada la trombectoma bajo anestesia local. Trombosis hemorroidaria mltiple o masiva; Consiste en la
trombosis de dos o ms paquetes hemorroidarios, acompaada de edema y dolor intenso. Dependiendo del tiempo de evolucin y su
sintomatologa, puede ser tributario o de tratamiento mdico o quirrgico. Hemorroides internas estranguladas. El paciente refiere dolor anal
intenso de corta duracin, acompaado de mltiples hemorroides que no se reducen con las maniobras habituales. Si continua su desarrollo se
convierte en hemorroides gangrenosas. El tratamiento siempre debe ser quirrgico y de urgencia

CASO CLINICO
Hombre de 47 aos de edad quien consult por dolor anal de dos
das de evolucin. Es casado, tiene 3 hijos y vive. Dentro de los
aspectos ms relevantes de la historia: se le dificulta sentarse, razn
por la cual prefiere mantenerse de pie, ha tomado analgsicos pero
no le han aliviado. Fiebre no cuantificada por termmetro. Ha tenido
episodios de sangrado. No hay historia de prdida de peso. Tiene el
antecedente de que hace dos aos le documentaron hemorroides
pero con tratamiento mdico alivi los sntomas. EF: signos vitales
estables, luce quejumbroso. El examen cardiopulmonar se encuentra
dentro de lmites normales. Abdomen y extremidades sin
alteraciones. Tacto Rectal: doloroso, tono esfintrico normal, se
palpa masa dura, lisa dolorosa, a escasos 2 cm de margen anal de
aproximadamente 1.5 cm de dimetro localizada en cuadrante rectal
lateral derecho, que abomba hacia el interior del recto.
PREGUNTA
Actualmente se encuentra estabilizado y se prepara para realizar
procedimiento quirrgico electivo. Considerando la preparacin del
paciente, cual es la indicacin ms apropiada?
RESPUESTA
a.- Indica aumento de fibra a la dieta.
b.- Prepara con polietilenglicol por via oral.
c.- Se requiere enema baritado.
d.- Indica lactulosa por via oral.

CURSO ENARM CMN SIGLO XXI TEL: 36246001

CASO CLINICO
Mujer de 49 aos de edad, desde hace 3 meses episodios de
aparicin de sangre roja junto con las heces a las que recubre. Como
antecedente, hemorroides desde su ltimo parto, en forma
intermitente le sangran despus de las deposiciones. Antecedentes
familiares: madre fallecida hace 10 aos por un ca. De mama, padre
operado hace 5 aos de ca. de colon. La paciente consult en
primera instancia a su ginecloga, quin le solicit un hemograma
con VSG, los cuales fueron normales, y un colon por enema con
doble contraste evidenciando lesin polipoidea pediculada de 1.5 cm
en colon sigmoide. Es derivada para proseguir la evaluacin. Examen
fsico: regin anal externa 3 cordones de hemorroides, uno
impresiona trombosado. Tacto renal s/p.
PREGUNTA
Qu diagnstico considera ms probable?
a.- Rectorragia por hemorroides
b.- Enterorragia por polipo sigmoideo
c.- Rectorragia por cancer de recto
d.- Enterorragia por lesin en ciego
PREGUNTA
Qu conducta diagnostica seria la mas adecuada tomara?
a.- Rectosigmoidoscopa flexible
b.- Colonoscopia izquierda
c.- Colonoscopia completa
d.- Ciruga

Pharmed Solutions Institute

PGINA 457

MANUAL DE TRABAJO DEL CURSO ENARM CMN SIGLO XXI


CANCER DE COLO RECTAL (CCR)
CIENCIAS BASICAS: El CCR es un tumor maligno que se origina dentro de las paredes del intestino grueso, incluye los segmentos; ciego, colon
ascendente, colon transverso, colon descendente, sigmoides, recto. El CCR no incluye los tumores en los tejidos del ano o del intestino delgado. Los
adenomas son tumores benignos epiteliales que se consideran lesiones precancerosas. Los adenomas pueden tener diferentes grados de displasia o
diferentes caractersticas histolgicas: tubular, tbulo-vellosas, asociados con el aumento de potencial malignidad. Factores de riesgo: Incluye
factores ambientales y genticos. Aproximadamente el 75% se presentan en pacientes sin factores de riesgo conocidos. Hay dos sndromes
genticos que predisponen al CCR: poliposis adenomatosa familiar (PAF), con riesgo de 60-90% y cncer colo rectal hereditario no asociado a
poliposis (CCHNP), considerar en familiares de primer grado de pacientes con diagnstico de CCR, pero representan menos del 10% de todos los
CCR. Otros factores son CUCI, obesidad, sedentarismo, exceso de alcohol, tabaquismo dieta alta en grasas y/o baja en frutas y vegetales o fibra.
SALUD PUBLICA: El CCR es el cncer ms comn del tracto gastrointestinal, la tercera causa ms comn de cncer y la cuarta causa de muerte por
cncer en el mundo. El CCR se presenta con mayor frecuencia a partir de los 50 aos de edad. Ms de 95% de CCR son adenocarcinomas. SINDROME
DE LYNCH: Sndrome de carcter hereditario, autosmico dominante y se caracteriza por el desarrollo precoz (antes de 50aos de edad) de CCR, de
predominio en colon derecho, y una elevada tendencia a presentar lesiones sincrnicas o metacrnicas, as como neoplasias en otros rganos
(endometrio, estomago, pncreas, sistema urinario, ovario, vas biliares). El diagnostico se establece a partir de la historia familiar segn los criterios
de Amsterdam. Esta entidad se caracteriza por la existencia de mltiples mutaciones somticas en fragmentos repetitivos de ADN. DIAGNOSTICO:
Clnica; dependen de la localizacin y extensin del tumor, no suele dar sntomas hasta etapas avanzadas. Las neoplasias en el hemicolon derecho
producen hemorragia oculta que el paciente no detecta y es probable que ocasionen sntomas de anemia ferropriva crnica. Este hallazgo en un
varn adulto o mujer postmenopusica, es indicacin para colonoscopia. Los tumores en el hemicolon izquierdo pueden manifestarse con
hemorragia visible (rectorragia), cambio en los hbitos intestinales y dolor abdominal clico secundario a obstruccin parcial. Tambin es probable
que las primeras manifestaciones del cncer del intestino grueso sean, metstasis heptica, ascitis y metstasis pulmonares. No es infrecuente que
se detecte una masa en la exploracin en el cuadrante inferior derecho del abdomen. Por ultimo una complicacin poco frecuente del cncer de
colon pero que empeora el pronstico es la perforacin intestinal. El cncer de recto puede comportarse como un sndrome anorrectal, con
urgencia rectal, tenesmo y diarrea con moco y sangre. Cunado su extensin rebasa la pared rectal, el paciente puede aquejar sntomas urinarios
atribuibles a invasin vesical como hematuria y polaquiuria. Si se ha establecido una fistula rectovesical, hay neumaturia e infecciones urinarias
recidivantes. Adems de los sntomas locales, CCR puede causar sntomas generales, como astenia, anorexia o prdida de peso. Puede aparecer
clnica secundaria a la presencia de metstasis a distancia, como ictericia, trastornos neurolgicos, dolores seos, disnea, ascitis. Ante la sospecha de
CCR es imperativo practicar una colonoscopia con la que adems de observar el tumor, se podrn tomar biopsias para su diagnstico histolgico y
descartar la presencia de lesiones sincrnicas. En caso de tumores estenosantes es recomendable completar la exploracin con una colonografa
por TAC o RNM. Por elevado porcentaje de presentaciones diseminadas, las exploraciones complementarias incluyen radiografas de torax, USG o
TAC abdominal, gammagrafa seas, cistoscopia, ultrasonografa endoscpica y RM plvica. La determinacin de antgeno carcinoembrionario,
tienen una reducida sensibilidad y especificidad para CCR. Sin embargo, posee valor pronstico y su monitorizacin es til para seguimiento del
paciente tras ciruga. TRATAMIENTO: La ciruga es usualmente el primer tratamiento o el tratamiento primario para los cnceres de colon que no se
han propagado a partes distantes. Tambin se puede usar la terapia adyuvante (adicional). En la mayora de los casos, la terapia adyuvante se
administra por aproximadamente 6 meses. Etapa 0: Puesto que estos cnceres no han crecido ms all del revestimiento interno del colon, todo lo
que se necesita es ciruga para extirpar el cncer. Esto se puede lograr en la mayora de los casos con una polipectoma (extirpacin del plipo) o
escisin local a travs de un colonoscopio. Si el tumor es demasiado grande para poder extirparlo mediante escisin local, es posible que
ocasionalmente sea necesario realizar una reseccin del colon (colectoma). Etapa I: Ha crecido a travs de varias capas del colon, pero no se han
propagado fuera de la pared del colon en s, ni ganglios. La colectoma parcial (ciruga para extirpar la seccin del colon que contiene cncer y los
ganglios linfticos cercanos) es el tratamiento convencional. Usted no necesita terapia adicional. Etapa II: Ha crecido a travs de la pared del colon y
se puede extender hacia el tejido vecino, no a ganglios linfticos. Puede que la ciruga (colectoma) sea el nico tratamiento que se necesita. Se
puede recomendar quimioterapia adyuvante si hay riesgos de recurrencia como: es de alto grado, ha crecido hacia los rganos cercanos, el cirujano
no extirp al menos 12 ganglios linfticos, se encontr cncer en o cerca del margen (borde) de la muestra quirrgica, lo que significa que algo de
cncer pudo haber quedado, ha bloqueado (obstruido) el colon, caus una perforacin (orificio) en la pared del colon. Las principales opciones de
quimioterapia para esta etapa incluyen 5-FU y leucovorn (solo) o capecitabina, aunque tambin se pueden usar otras combinaciones. Se puede
administrar radioterapia al rea de su abdomen donde est creciendo el cncer. Etapa III: Se ha propagado a los ganglios linfticos cercanos, pero
todava no se ha extendido a otras partes del cuerpo. Ciruga (colectoma parcial) seguida de quimioterapia adyuvante. El rgimen FOLFOX (5-FU,
leucovorn, y oxaliplatino) o el rgimen CapeOx (capecitabina y oxaliplatino) se usan con ms frecuencia, aunque algunos pacientes pueden recibir 5FU con leucovorn o capecitabina sola segn la edad y sus necesidades de salud. La radioterapia y/o la quimioterapia pueden ser opciones para las
personas que no estn lo suficientemente saludables como para someterse a ciruga. Etapa IV: Se propag desde el colon hasta rganos y tejidos
distantes. Con ms frecuencia, el cncer de colon se propaga al hgado, aunque tambin se puede propagar a otros lugares, tal como a los pulmones,
el peritoneo (la membrana que recubre la cavidad abdominal) o a los ganglios linfticos distantes. Es poco probable que la ciruga cure estos
cnceres. Si las metstasis no se pueden extirpar quirrgicamente debido a que son muy grandes o hay demasiadas, se puede tratar primero
quimioterapia para reducir el tamao de los tumores con el fin de permitir la ciruga. Entonces, se puede administrar nuevamente quimioterapia
despus de la ciruga. Otra opcin sera destruir los tumores en el hgado con criociruga, ablacin por radiofrecuencia, u otros mtodos no
quirrgicos. Si el cncer se ha propagado demasiado como para tratar de curarlo con ciruga, puede que en algunos casos an sea necesario realizar
operaciones, tales como una colectoma o una colostoma de derivacin (cortar el colon sobre el nivel del cncer y se adhiere el extremo a una
abertura en la piel del abdomen para permitir que los desechos salgan del cuerpo). Esto puede aliviar o prevenir el bloqueo del colon, lo que puede
evitar ciertos problemas. Algunas veces, se puede evitar tal ciruga al insertar un stent en el colon durante la colonoscopia con el fin de mantenerlo
abierto. La mayora de los pacientes con cncer en etapa IV recibir quimioterapia y/o terapias dirigidas para controlar el cncer. Los regmenes ms
comnmente usados incluyen: FOLFOX: leucovorn, 5-FU, y oxaliplatino (Eloxatin). FOLFIRI: leucovorn, 5-FU, e irinotecn (Camptosar). CapeOX:
capecitabina (Xeloda) y oxaliplatino. Cualquiera de las combinaciones anteriores ms bevacizumab (Avastin) o cetuximab (Eribitux) (pero no
ambos).PRONOSTICO: Se relaciona con la etapa del tumor. Se prefiere el sistema de Dukes por su sencillez: Dukes A: tumor limitado a submucosa.
Dukes B1: tumor que se extiende a la capa muscular. Dukes B2: tumor que se extiende ms all de la serosa intestinal. Dukes C1: compromiso de
ganglios linfticos; el tumor no se extiende fuera de la pared intestinal. Dukes C2; ganglios linfticos positivos; el tumor rebasa pared intestinal.
Dukes D: metstasis a distancia. La sobrevida no corregida a los 5 aos para etapas A= 85%, B= 65%, C = 45%, los valores corregidos son 100, 78 y
54% respectivamente

CURSO ENARM CMN SIGLO XXI TEL: 36246001

Pharmed Solutions Institute

PGINA 458

MANUAL DE TRABAJO DEL CURSO ENARM CMN SIGLO XXI


CASO CLINICO CANCER DE COLON
Se trata de paciente masculino de 71 aos de edad, refiere que
desde hace 6 meses presenta reduccin de peso, astenia, adinamia,
a la exploracin fsica se palpa masa no dolorosa en fosa iliaca
derecha, con presencia de hematoquesia, se reporto biometra
hemtica con hematocrito 31 %, y hemoglobina de 10, el frotis
mostro eritrociticos microcitico normocromicos, resto normal.
PREGUNTA
Cul es el factor de riesgo de mayor importancia para el desarrollo
de cncer de clon
RESPUESTA
a.- Consumo de alcohol.
b.- Dieta baja en fibra y alta en carnes.
c.- Tabaquismo positivo.
d.- Colitis o Diverticulitis previa.
CASO CLINICO CANCER DE COLON
Acude al servicio de urgencias un paciente masculino de 69 aos de
edad el cual refiere malestar generalizado y fiebre desde hace 3
semanas de evolucin, el paciente es originario de la ciudad de
mexico, administrador retirado hace 4 aos, tiene el antecedente de
tabaquismo desde la edad de 20 aos a la fecha en promedio 20
cigarrillos al da, adems hipertensin arterial sistmica controlada
con IECAS mas diurtico tiazidicos, a la exploracin fsica se observa
con palidez moderada, asi como ndice de masa corporal de 21, la
regin cardiaca se ausculta murmullo sistlico con predominio
mitral, los estudios de laboratorio se detecta hemoglobina de 11
g/dl, 34% de hematocrito, leucocitos y eritrocitos en orina, se
realizaron cultivos siendo positivo nicamente el hemocultivo para
streptococcus bovis, debido a los resultado se enva a
ecocardiograma, sin tener actualmente los resultados, pero se
sospecha vegetaciones.
PREGUNTA
Cual es la conducta a seguir ms adecuada en el paciente para
identificar la causa inicial.
RESPUESTA
a.- Realizar una biopsia renal.
b.- Esperar resultado de Ecocardiograma.
c.- Valoracion toxicolgica.
d.- Realizar colonoscopia.
CASO CLINICO
Se trata de una mujer de 40 aos de edad, de raza blanca, sin
antecedentes patolgicos, que se present a la consulta por cambios
en el ritmo evacuatorio. Al interrogarla sobre sus antecedentes
familiares, refiere madre y to con antecedentes de cncer de colon.
En la anamnesis sistmica manifest sntomas de distensin
abdominal leve y cambios en el hbito intestinal, con alternancia de
perodos de constipacin y diarrea de escasa duracin y cantidad. No
refiri prdida de peso, hematoquezia o dolor abdominal. El examen
fsico revel una paciente de mediana estatura, normonutrida,
ansiosa, afebril y con signos vitales dentro de los lmites normales. El
abdomen se hall blando, depresible, no doloroso a la palpacin
superficial, pero s levemente a la profunda, sobre todo en la zona
del hipocondrio derecho. Los ruidos hidroareos se hallaron algo
aumentados. El tacto rectal no revel ninguna alteracin.

d.- c-myc
PREGUNTA
Cul es el estudio diagnostico gold standard que realizaramos en
este paciente?
RESPUESTA
a.- Colonoscopia
b.- Rectosigmoidoscopia
c.- Enema baritado con doble contraste
d.- Prueba de guayaco
PREGUNTA
En caso de que la colonoscopia fuera incompleta Qu estudio
estara indicado realizar como complemento?
RESPUESTA
a.- Colonoscopia
b.- Rectosigmoidoscopia
c.- Enema baritado con doble contraste
d.- Prueba de guayaco
CASO CLINICO
Mujer de 68 aos remitida con Rx de Colon por Enema que muestra
estenosis concntrica a 6 cms del reborde anal con escaso paso del
medio de contraste a proximal. A la rectoscopia se encuentra lesin
ulcerada, necrosada, fcilmente sangrante y con estenosis
infranqueable. No hay antecendentes familiares de Cncer
Colorrectal. El estudio de patologa de esta paciente report
Adenocarcinoma de Recto.
PREGUNTA
Cul factor de riesgo es menos probable que influya en esta
patologa?
RESPUESTA
a.- Dieta rica en verduras, frutas, fibra y la ingesta de carbonato de
calcio y aspirina se han asociado a disminucin en el riesgo de
desarrollar adenomas y carcinomas colorrectales.
b.- El mayor riesgo de desarrollar cncer colorrectal cuando hay
antecedente familiar, hace de sta una enfermedad especial en
cuanto a los programas de deteccin y seguimiento
c.- El 90% de los cnceres colorrectales se presentan en mayores de
50 aos , con mayor incidencia en la raza blanca que en la negra
d.- El riesgo de desarrollar cncer colorrectal es mayor en pacientes
con Colitis Ulcerativa y con Enfermedad de Crohn
PREGUNTA
Los pacientes sometidos a resecciones rectocolnicas por Cncer
deben ser vigilados en forma cercana para la deteccin de
recurrencia o persistencia tumoral. Cul de las siguientes es menos
probable?
RESPUESTAS
a.- Anomalas en clnica, laboratorios o control endoscpico
ameritan la realizacin de Rx de Trax y TAC de Abdomen
b.- Dada las posibilidades de imagenologa y laboratorio, es poco
relevante el seguimiento con historia clnica y examen fsico
c.- El objetivo del control endoscpico es detectar Cncer
metacrnico, plipos adenomatosos vellosos o con displasia de alto
grado
d.- Realizar Sigmoidoscopia o Colonoscopia cada 6 a 12 meses
durante los primeros 3 aos

PREGUNTA
Cul es el gen ms probablemente afectado en esta patologa?
RESPUESTA
a.- BRCA-1
b.- p53
c.- HER-2/neu

CURSO ENARM CMN SIGLO XXI TEL: 36246001

Pharmed Solutions Institute

PGINA 459

MANUAL DE TRABAJO DEL CURSO ENARM CMN SIGLO XXI


FISURA ANORECTOPERINEAL
CIENCIAS BASICAS: Comunicacin anmala entre dos epitelios diferentes. La fstula es el conducto de paredes fibrosas infectadas, que comunica una
cripta anal con la piel o con la luz del recto; el orificio localizado en la cripta se denomina primario o interno y el cutneo mucoso o secundario. El
orificio primario es donde habitualmente se origina el proceso. Etiologa: Origen criptoglandular en 95%. Secuela de
un absceso. Debut en procesos menores. Trauma. Infecciosas (TB, gonorrea). Enfermedades de intestino irritable
(Crohn, CUCI). Radiacin. SALUD PUBLICA: Es una afeccin muy frecuente en la prctica colproctolgica ms
comn en el sexo masculino y en edades que fluctan entre 30 a 50 aos CLASIFICACION: A) Interesfinteriana, B)
Transesfintrica, C) Supraesfintrica, D) Extraesfintrica. DIAGNOSTICO: Clnico; Orificio secundario, dolor,
secrecin, sangrado, absceso. Realizar un adecuado interrogatorio del cuadro clnico y los antecedentes. secrecin
purulenta persistente y escasa , que procede de una elevacin cutnea del tejido inflamatorio ( orificio secundario),
a veces dolorosa al tacto, que puede ser nica o mltiple, situado en cualquiera de los dos cuadrantes perianales ,
con mayor frecuencia en los posteriores a distancias variables del ano , algunas tan lejanas como 12 a 13 cm y an
ms . Aproximadamente en la mitad de los casos se encuentra un aumento en el volumen e hiperpigmentacin de la piel en la regin que rodea el
orificio fistuloso, que traducen la presencia de un absceso que no fue totalmente vaciado. En conjunto el absceso y la fstula siguen una de las
siguientes evoluciones Exploracin Fsica: Orificio Secundario, tacto rectal. Gabinete: La fistulografa con el propsito de diagnstico y conocer el
trayecto de la fstula si el material de contraste logra pasar con facilidad los trayectos fistulosos, lo podr hacer el estilete durante la operacin, si
por el contrario es introducido a presin puede romper la pared de la fstula y causar error diagnstico. El ultrasonido endoanal de 360es til para
anatoma de fstulas complejas. CRITERIOS DIAGNSTICOS: Historia de absceso perianal. Drenaje anal o perianal de secrecin purulenta o
intermitente. Presencia de orificio fistuloso secundario con trayecto fistuloso dirigido hacia la cripta de origen. TRATAMIENTO: Tratar el absceso y la
fstula en la misma operacin. Fistulectoma o fistulotoma. La fistulectomia reseca la totalidad del trayecto fistuloso por lo que es ms radical y
disminuye la recidiva mayor riesgo de incontinencia; la fistulotoma solo una porcin del trayecto. La incontinencia esfinteriana por fistulectoma es
posible en fstulas muy altas que requieren divisin del anillo rectoanal. COMPLICACIONES: Fase aguda (abscesos) sepsis graves y muerte,
destruccin del esfnter e incontinencia anal (5-10%), deformidad anal (Keyhole) (3-5%), recidiva (3-5%), cambios neoplsicos malignos en fstulas de
larga evolucin, complicacin post- quirrgica: incontinencia anal en cirujanos de poca experiencia.
CASO CLINICO FISTULA ANAL
Se trata de paciente femenino de 31 aos de edad la cual acude a
revisin posterior a parto hace 45 das, al interrogatorio la paciente
refiere que presenta dolor al evacuar, con salida de sangre fresca
leve, niega sensacin de pensantes, ni descarga rectal espontnea,
niega ardor o prurito, a la exploracin no se observan hemorroides
externas.
PREGUNTA
Cul es la patologa ms probable en esta paciente?
RESPUESTA
a.- Lo ms frecuente es la fisura anal.
b.- Desgarro secundario a parto.
c.- Las hemorroides internas por mltiples embarazos.
d.- Un absceso anal por infeccin de episiotoma
CASO CLINICO
Paciente de 42 aos, consulta a medicina general hace
aproximadamente cinco aos por presentar prurito anal intenso,
exacerbado por comidas picantes. Es manejado inicialmente con
medidas locales y corticoide tpico. Persiste con prurito asociado a
evacuaciones diarias, sin esfuerzo y con sangrado en el ltimo ao,
adems de dolor perianal aun estando de pie. Antecedentes
mdicos: enfermedad pulmonar obstructiva crnica, hace un ao
drenaje de absceso perianal. Buen estado general. No presenta
lesiones drmicas. No hay afectacin en otras localizaciones. No hay
masas abdominales ni es doloroso a la palpacin. En el examen
proctolgico, el cual se realiza en posicin Sims a la inspeccin, se
observa una lesin elevada perianal rojiza simtrica con bordes
delimitados con salida de exudado a la presin, con signos de
rascado, sin ulceraciones. En el examen dinmico se observa una
adecuada contraccin del aparto esfinteriano, sin prolapso a la
maniobra de Valsalva.
PREGUNTA
Cul es la etiologa ms probable de la patologa de este paciente?
RESPUESTA
a.- Criptoglandular
b.- Infecciosa
c.- Trauma
d.- Neoplasia

CURSO ENARM CMN SIGLO XXI TEL: 36246001

PREGUNTA
Cul es la conducta diagnostica ms adecuada para este paciente?
RESPUESTA
a.- Resonancia magntica
b.- Ultrasonido endoanal 360
c.- Radiografa plvica
d.- TAC
PREGUNTA
Cul es la complicacin ms probable que presente el paciente,
secundario a la fistulotoma?
REPUESTA
a.- Recidiva
b.- Mayor destruccin de tejidos
c.- Incontinencia
d.- Absceso perianal
CASO CLINICO
Se trata de una paciente de sexo femenino de 76 aos edad con
buen nivel socioeconmico, heterose xual sin actividad sexual, que
consulta por presentarproctorragia e importante dolor anal que se
exacerba durante las evacuaciones. Al examen fsico presenta
esfnter hipertnico y no se observa patologa hemo rroidal. Se
realiza una rectosigmoideoscopa rgida (RSC) que muestra una fisura
anal en hora 6 y se to ma una biopsia de rutina. Se instaura durante
15 das tratamiento.
PREGUNTA
Cul es la conducta teraputica mas adecuada a seguir?
RESPUESTA
a.- Baos de asiento y rgimen higienicodietetico
b.- Ungento local con lidocana y corticoides, y rgimen
c.- Solo corticoides
d.- Reposo realativo mas alto consumo de fibra

Pharmed Solutions Institute

PGINA 460

MANUAL DE TRABAJO DEL CURSO ENARM CMN SIGLO XXI


ORQUIEPIDIDIMITIS
CIENCIAS BASICAS: La epididimitis, orquitis y orquiepididimitis se definen como la inflacin del epiddimo, testculo y del testculo/epiddimo
respectivamente. Hablaremos de orquiepididimitis aguda si el proceso dura menos de 6 semanas y crnica si es mayor. La causa ms frecuente en
pre-pberes son las bacterias coliformes que infectan la va urinaria, en hombres sexualmente activos predominantemente se encuentra infeccin
por Chlamidya trachomatis y N. gonorrehae, en hombres mayores con menor actividad sexual. Y con antecedentes de patologa obstructiva urinaria
o instrumentacin reciente presentan infeccin por patgenos urinarios como E. coli y paseudomonas. En homosexuales suele estar causada por
coliformes y haemophilus influenzae. Otras causas son tuberculosis, brucelosis, secundaria a traumatismos y medicamentos (amiodarona,
acumulacin de desetilamiodarona) y con menor frecuencia idioptica, antecedente de enfermedad de Behcet (vasculitis multiorgnica idioptica,
con ulceras genitales, ulceras aftosas recurrentes y uvetis). SALUD PUBLICA: Constituye la causa ms comn (80-90%) de escroto agudo en
pacientes mayores de 18 aos. El 56% de los hombres >60aos que presentan epididimitis se asocia con obstruccin de tracto urinario. 50% de
pacientes con antecedentes de instrumentacin, uretral, catter permanente o intermitente pueden presentar epididimitis infecciosa y de estos el
80% es de origen bacteriano (E. coli). PATOGENIA: El principal mecanismo patognico, lo constituye el reflujo de orina infectada de forma
ascendente (retrograda) hacia los conductos eyaculadores y deferente. La orquitis se produce por contigidad. DIAGNOSTICO: Clnica; Dolor escrotal
intenso de instauracin gradual con signos de inflamacin en el hemiescroto afectado, incluyendo el epiddimo y el conducto deferente. Suele
acompaarse de afeccin del estado general con fiebre (en ms de 50% de los casos, >38) y sntomas urinarios (disuria, secrecin uretral). Puede
evolucionar a absceso escrotal cuyo tratamiento es el drenaje. La inflamacin comienza en la cola del epiddimo y se extiende hacia el testculo. El
cordn espermtico esta engrosado y tumefacto y es frecuente la existencia de hidrocele reactivo. La elevacin del testculo produce disminucin
del dolor (signo de Prehn positivo), reflejo cremasteriano presente, la trasniluminacion, puede ser positivo por hidrocele reactivo. El diagnstico es
fundamentalmente clnico, debemos realizar una BH en el que se encontrara leucocitosis y neutrofilia as como un sedimento de orina que puede ser
patolgico con piuria y/o bacteriuria. El urocultivo es negativo en 40-90% de los casos. En pacientes con prcticas sexuales de riesgo y con flujo
uretral se solicita: exudado uretral, tincin gramm si se sospecha gonorrea, prueba de ELISA si se sospecha VIH. En caso de duda diagnostica
podemos recurrir a la ecografa doppler que detectara un flujo testicular conservado o aumentado y signos sugestivos de orquiepididimitis como
acentuacin de septos, existencia de focos hiperecogpnicos y epiddimo aumentado de tamao. TRATAMIENTO: Instaurar un tratamiento emprico
sin esperar resultados de cultivos. En caso de sospecha de infeccin de transmisin sexual (antecedente de contacto sexual sospechosos, uretritis,
infeccin diagnosticada en pareja sexual) se deber iniciar tratamiento con azitromicna o ceftriaxona (1 g IM o IV) en dosis nica junto con
doxiciclina 100mgs/12hrs. Como alternativa tenemos levofloxacino 500mgs c/12 hrs. Ambas pautas deben mantenerse por 10 das. En pacientes sin
prctica sexual trimetropim/sulfametoxaxol por 10-14 das. Debe tratarse a los compaeros sexuales del ltimo mes y evitar el coito sin proteccin
hasta haber terminado tratamiento. Si sospecha infeccin bacteriana inespecfica disponemos de varias pautas: ciprofloxacino 500mg/ 12 hrs,
norfloxacino 400mgs/ 12 hrs o amoxicilina-clavulanico 875mgs/ 8 hrs durante 10-14 das. En casos floridos debemos asociar aminoglucocidos:
tobramicina 100mgs IM o IV c/12 hrs o gentamicina 240mgs IM o IV cada 24 hrs. Adems del tratamiento antibitico, existe una serie de normas
generales que favorecen el alivio de los sntomas; actividad fsica limitada, suspensorio testicular, aplicacin de frio local. Para el control, de dolor se
recomienda el uso de AINEs (naproxen o diclofenaco por 2 semanas) que adems favorecen la disminucin del proceso inflamatorio. El edema
hemiescrotal puede persistir durante 4-6 semanas despus del tratamiento y de forma residual puede persistir una induracin epididimaria
indefinida. COMPLICACIONES: Absceso en epiddimo, oligoatenosospermia, oligoatenosteratospermia, azoospermia, dolor crnico.
CASO CLINICO ORQUIEPIDIDIMITIS.
Varn de 35 aos de edad. Consult por presentar desde haca tres
meses, un aumento del testiculo derecho y por manchar la cama
con lquido como pus. En la exploracin testicular destacaba un
aumento irregular del tamao escrotal, de consistencia dura, con
ulceracin escrotal y secrecin de tipo purulento. La analtica
sangunea re ej una leucocitosis como nico hallazgo a resear (la
bioqumica y marcadores tumorales fueron negativos).
PREGUNTA
Cul de los siguientes antecedentes es ms frecuente en este caso?
RESPUESTA
a.- Enfermedad de transmisin sexual recurrente.
b.- Presencia de bacterias anaerobias.
c.- Enfermedad de Behcets.
d.- Varicocele no tratado
CASO CLINICO
Masculino de 62 aos de edad, que acude a urgencias por referir
dolor testicular izquierdo de 24 horas de evolucin, acompaado de
fiebre 40, junto con escalofros y sndrome miccional acompaante
(disuria, polaquiuria, tenesmo y urgencia miccional, disminucin del
calibre del chorro), ausencia de dolor lumbar, adems refiere
nuseas y vomito en dos ocasiones de contenido alimentario.
Durante la anamnesis el paciente revela su profesin, veterinario. A
la exploracin se observa testiculo derecho de caractersticas
normales, y en el teste izquierdo, se visualiza un aumento de
tamao, edematizado as como eritema y dolor al tacto sin
fluctuacin de la zona afecta aunque con dolor a la movilizacin
testicular, signo de Prehn +. El tacto rectal resulta sin hallazgos
patolgicos y la exploracin abdominal as como la percusin renal

CURSO ENARM CMN SIGLO XXI TEL: 36246001

tambin descart afecciones acompaantes al ser normales.


Analticamente destaca leucocitosis de 22.240 con neutrofilia
(neutrfilos 90%). En la Ecografa testicular realizada previa al
ingreso muestra aumentos homogneos de la ecogenicidad del
testculo afecto.
PREGUNTA
Cul es el diagnstico ms probable para este paciente?
RESPUESTA
a.- Prostatitis
b.- Absceso escrotal
c.- Orquiepididimitis
d.- Tumor testicular
PREGUNTA
Cul es el factor de riesgo que ms probable influyo en la patologa
del paciente?
RESPESTA
a.- Hiperplasia prosttica
b.- IVU
c.- Edad
d.- Prcticas sexuales de riesgo
PREGUNTA
Cul es la conducta teraputica ms adecuada a seguir en este
caso?
RESPUESTA
a.- Azitromicina 1gr IM
b.- Doxiciclina 100mg VO c/12 hrs
c.- Ciprofloxacino 500mg VO c/12 hrs
d.- Amoxicilina/Ac. Clavulanico 875m/125mg c/8hrs

Pharmed Solutions Institute

PGINA 461

MANUAL DE TRABAJO DEL CURSO ENARM CMN SIGLO XXI


TORCION TESTICULAR
CIENCIAS BASICAS: Consiste en la rotacin del cordn espermtico sobre s mismo ocasionando una disminucin del aporte sanguneo al testculo y
al epiddimo. SALUD PUBLICA: Presenta una incidencia de 1/4000 varones menores de 25 aos. Es la causa ms frecuente de prdida testicular en el
varn joven. Puede aparecer a cualquier edad pero presenta dos picos de incidencia, uno durante el primer ao de vida y otro en la pubertad.
PATOGENIA: La etiologa es muy variada aunque suele existir una alteracin bilateral de la suspensin que permite que al testculo y cordn
espermtico rotar sobre s mismos. La tasa de viabilidad cae desde un 85-100%, cuando se resuelve en las primeras horas, hasta un 20% a las 12
hrs. En las 4 primeras horas se afectan las clulas de la espermatognesis. Las clulas de Leydig son ms resistentes aproximadamente unas 10 hrs.
Existen 2 formas anatomoclinicas: torsin extravaginal; constituye un 5% de las torsiones testiculares. Es prcticamente exclusiva del neonato.
Torsin intravaginal; forma tpica del nio, adolescente y adulto joven, con un pico entre 8-15 aos. No siempre es de aparicin brusca. Existen
formas incompletas. Es la forma ms frecuente y constituye el 95% de los casos. DIAGNOSTICO: Se caracteriza por dolor testicular de inicio sbito o
gradual que puede irradiarse a regin inguinal o suprapbica, acompaado de tumefaccin escrotal. Pueden aparecer nauseas, vmitos o
hiperpirexia. El 50% de las causas ocurre durante el sueo. En la exploracin fsica el testculo suele estar elevado hacia el anillo inguinal superficial y
horizontalizado (signos de Governeur), ser doloroso al tacto e incluso estar aumentado de tamao. La piel del escroto puede estar edematosa y
enrojecida, lo que dificulta la exploracin testicular. Al elevar el testculo hacia la snfisis del pubis, no disminuye o aumenta el dolor (signo de Prehn
negativo) y existe ausencia de reflejo cremasterico (reflejo cutneo que aparece al rozar la cara superointerna del muslo, produciendo elevacin del
testculo homolateral o la contraccin de la pared abdominal). La identificacin del epiddimo en posicin anterior o lateral y la palpacin de un
cordn espermtico de consistencia blanda y algo congestivo son indicios muy sugestivos de torsin testicular. Si en la exploracin fsica detectamos
un testculo ascendido y horizontalizado con el epiddimo en posicin anterior debemos intentar, tras la administracin de sedacin, la destorsin
manual. En caso de que exista duda diagnostica est indicada la realizacin de una ecografa doppler; que constituye el. TRATAMIENTO: La torsin
testicular constituye una autentica urgencia urolgica dado que la viabilidad testicular es inversamente proporcional a la duracin de la torsin, para
restablecer cuanto antes el flujo vascular al testculo evitando as la necrosis y atrofia posterior. El tratamiento definitivo es quirrgico, si bien como
medida temporal puede intentarse la destorsion manual. Se trata de una medida rpida, segura y no invasiva. Tras esta maniobra debe realizarse
una ecografa para comprobar la reperfusin testicular. Si esta medida es efectiva, la oquidopexia (fijacin del testculo a la pared escrotal), que
constituye el tratamiento de eleccin de la torsin podr realizarse de forma electiva. Se recomienda que la orquidopexia sea bilateral ya que la
causa anatmica que predispone a la torsin suele serlo. Algunos autores recomiendan tambin la orquidopexia contralateral en caso necesario
practicar una orquiectoma, en pacientes con historia previa de torsiones.
CASO CLINICO TORCION TESTICULAR
Masculino de 18 aos de edad, present con dolor testicular
izquierdo pocos das de duracin. No hay otros sntomas. El examen
fsico revel una induracin en el polo superior del testculo
izquierdo. Los marcadores tumorales fueron normales. Ecografa
demostr una lesin intratesticular mal definida, hipoecoica, 13,2 x
12,6 mm. En ecografa Doppler de potencia, una zona hipoecoica
apareci completamente avascular en contraste con el resto del
testculo.

CURSO ENARM CMN SIGLO XXI TEL: 36246001

PREGUNTA
Cual se las siguientes manifestacin considera ms relevante para
el diagnostico?
RESPUESTA
a.- Signo de Prehn positivo.
b.- Reflejo cremasterico.
c.- Trasluminacin positiva.
d.- USG testicular

Pharmed Solutions Institute

PGINA 462

MANUAL DE TRABAJO DEL CURSO ENARM CMN SIGLO XXI


CANCER TESTICULAR
CIENCIAS BASICAS: El tumor maligno de testculo es una de las neoplasias solidas ms comn y curable en el hombre entre 15-35 aos. El 95% de los
tumores malignos de testculo se origina del epitelio germinal primordial y 5% son del estroma gonadal. Factores de riesgo: Historia familiar de
cncer en familiares de primer grado, criptorquidea (principal factor), infertilidad, sndrome de Klinefelter, tumor testicular contralateral, orquitis
viral por sarampion. SALUD PUBLICA: Ocupa el 2% de todas las neoplasias. Tiene 3 picos de incidencia importante en la infancia, edades de 25-40
aos y a los 60 aos. CLASIFICACION: Los tumores malignos de testculo de origen germinal se clasifican en semnoma puros 50% (extremadamente
sensibles a radioterapia) y no seminoma (responden a quimioterapia basada en platino) 35% son puros y 15% mixtos. Los no seminomatosos
RIESGO
NO SEMINOMA
SEMINOMA
incluyen los subtipos histolgicos de senos endodrmicos, coriocarcinoma,
Riesgo
Tumor primario testicular o
Cualquier sitio primario y
carcinoma embrionario y teratoma maduro e inmaduro. DIAGNOSTICO: Temprano;
bajo
retroperitoneal y sin
sin metstasis
la presencia de una masa testicular solida indolora, es patognomnica de tumor
(bueno)
metstasis pulmonares o
pulmonares o viscerales
testicular y se puede acompaar de dolor local, dolor de espalda y en flanco
viscerales. Cualquier valor de
y AFP: normal. hCG
ipsilateral, tos, hemoptisis. Se pueden presentar signos y sntomas de enfermedad
los siguientes: AFP
cualquier valor. LDH
<1,000ng/ml. hCG <5,000iu/l.
cualquier valor.
metastsica. Se debe realizar una exploracin detallada del rea genital incluida la
LDH <1.5 por arriba del lmite
transiluminacion escrotal, abdomen en bsqueda de masas palpables, en trax y
normal
abdomen buscando ginecomastia o ganglios supraclaviculares. El 20% de los casos
Riesgo
Tumor primario testicular o
Cualquier sitio primario y
presentan Hidrocele secundaria. En neoplasia hormonosecretante: ginecomastia,
interme
retroperitoneal y sin
con metstasis
macrogenitosomia. Laboratorio y gabinete: Se debe efectuar determinacin srica
dio
metstasis pulmonares o
pulmonares o viscerales
viscerales, con AFP 1,000y AFP: normal. Hcg
de marcadores tumorales: alfafetoproteina, normal <15ng/ml (AFP), se eleva en
10,000ng/ml. hCG 5,000cualquier valor. LDH
tumores de senos endodermicos y en carcinoma embrionario, nunca se eleva en
50,000iu/l. LDH1.5-10 por
cualquier valor.
seminomas puros. Fraccin beta de la hormona gonadotropina corionica Humana
arriba de limite normal.
(HGC), se eleva principalmente en coriocarcinoma. Deshidrogenasa lctica (DHL), se
Riesgo
Tumor primario en mediastino
No existen pacientes
alto
y sin metstasis pulmonares o
clasificados con riesgo
eleva en 60% de pacientes con tumores no seminomatosos y en el 80% de los
viscerales con AFP >10,000
alto
seminomas en etapas avanzadas. Se debe realizar ultrasonido testicular para
ng/ml. hCG >50,000iu/l. LDH
confirmar diagnstico de tumor slido. Para la etapificacin del tumor conformado el
>19 por arriba de limite
diagnstico de cncer se debe realizar: TAC abdominopelvica y Rx de trax PA y
normal
lateral. Confirmacin mediante histologa. El PET-CT es til para evaluar viabilidad de
tumor residual posterior a tratamiento con quimioterapia en pacientes con tumores seminomatosos. TRATAMIENTO: El cncer testicular es
altamente curable, incluso en etapas avanzadas. La orquiectoma inguinal radical y ligadura del cordn espermtico en el anillo inguinal profundo
respetando as el drenaje linftico, es el estndar de tratamiento para los tumores testiculares. SEMINOMA: La etapa clnica I tres opciones de
manejo: vigilancia, radioterapia, quimioterapia (carboplatino). Etapa IIA: radioterapia. Etapa IIB: radioterapia o quimioterapia (bleomicina,
etoposido, platino=BEP). Etapa IIC y III: se considera enfermedad avanzada y debe manejarse con quimioterapia (BEP). NO SEMINOMA: Etapa clnica
I: de bajo riesgo, con vigilancia, quimioterapia, linfadenectioma retroperitonela neuropreservadora. Etapa I: de alto riesgo, quimioterapia (BEP),
linfadenectiomia retroperitonela neuropreservadora, vigilancia si la quimioterapia est contraindicada y no se desea ciruga. Etapa IS: (marcadores
sricos sin evidencia radiolgica de tumor) deber recibir quimioterapia (BEP). Etapa IIA y IIB con marcadores tumorales negativos puede manejarse
con vigilancia, linfadenectiomia retroperitonela neuropreservadora. Etapa IIA y IIB, con marcadores tumorales elevados deber manejarse con
quimioterapia. Etapa IIC y III, deber manejarse con quimioterapia. PRONOSTICO: El de mejor pronstico seminoma. El de peor pronstico
coriocarcinoma. Seminomas estadios I o IIA operados y en irradiacin ulterior o quimioterapia; sobrevida mayor a 5 aos. No seminomatosos 75 a
80% de xito. Estatus libre de enfermedad: primeros dos aos: determinacin de marcadores biolgicos, RX de trax cada 2 meses y TAC cada 3
meses. 3 aos: marcadores biolgicos, RX trax cada 3 meses y TAC cada 6 meses. 4 aos: controles semestrales. 5 aos: controles anuales.
TUMORES GERMINALES: CORIOCARCINOMA: Afortunadamente infrecuentes (<2%). Presenta elementos cito y sincitiotrofoblasticos,
extremadamente infrecuentes. Elevada tasa metastasica. Se presenta en forma de lesin pequea y con centro necrohemorragico. Su diseminacin
es rpida y por va hematgena habitualmente. Presenta casi 100% de metstasis al momento del diagnstico. TERATOCARCINOMA Y TERATOMAS:
Derivan de una o ms hojas embrionarias, contiene restos de las tres capas embrionarias (ecto, meso y endodermo). La forma inmadura contiene
elementos celulares primitivos indiferenciados. Alta resistencia a los tratamientos radio y quimioterapicos. Metastasis en 15% al momento del
diagnstico. CARCINOMA EMBRIONARIO: Forma el 25% de los tumores germinales. Existe en 2 formas; La del adulto (Ca. Embrionario propiamente
dicho). La infantil (tumor de Yolk, del saco vitelino o del seno endodermico), ms frecuente tumor testicular en nios. Predominan las formas mixtas
y necrohemorragicas. Elevada tasa metastsica (60%), ganglios regionales. El tumor el saco vitelino es el considerado ms agresivo de la lnea
germinal. TUMORES NO GERMINALES (Estromales). En general derivan del conducto de Wolf, son por regla benignos. Se presentan en los tejidos de
soporte y en los productores hormonales de los testculos. TUMOR DE CELULAS DE LEYDIG: Suponen el 1-3 % del total de todos los tumores
testiculares. Es el ms comn de los tumores testiculares de clulas no germinales. Su origen deriva de las clulas intersticiales del testculo, clulas
productoras de androgeneos. La bilateralidad en los tumores testiculares es excepcional, solo ocurre en el 0,7% sincrnicamente, y con carcter
metacrnico (hasta en los 5 aos posteriores al diagnstico del contralateral) en el 1,5%. Sin embargo en los tumores de Leydig y Sertoli puede
elevarse al 5-10%. El tumor de Leydig en el adulto provoca manifestaciones endocrinas en el 20-40% de los casos. Estos tumores son
hormonoproductores por lo que pueden dar manifestaciones endocrinas, y cuando las dan sern distintas dependiendo de la edad de los pacientes:
Adulto: ginecomastia, disfuncin erctil y disminucin de la libido, oligozoospermia y atrofia del otro testculo. Nios: A virilizacin precoz con
desarrollo de caracteres sexuales secundarios: aumento del tamao de los genitales externos, presencia de vello pbico, cambios de la tonalidad de
la voz. etc.; solo en el 10% de stos se acompaa de ginecomastia. Ninguna hormona va a ser secretada en niveles suficientes para que pueda ser til
como marcador tumoral.
CASO CLINICO CA TESTICULO.
Masculino, de 35 aos, con antecedentes de agenesia testicular
derecha, que asisti a consulta externa por un aumento del volumen
de la fosa ilaca derecha (FID). En el examen fsico se constat un
tumor a este nivel, interpretndose como plastrn apendicular, por
lo que se le administr antibioticoterapia. Posteriormente se le
realiz ultrasonido abdominal, visualizndose en el hipogastrio y la
FID, en correspondencia con el aumento de volumen, una imagen
tumoral heterognea vascularizada (al aplicarle Doppler color), de

CURSO ENARM CMN SIGLO XXI TEL: 36246001

13813cm, que recordaba la ecoestructura de un testculo


aumentado de tamao. Se explor, adems, el testculo izquierdo,
que tena ecoestructura y tamao normales.
PREGUNTA
Cul es el factor de riesgo ms importante para el cncer testicular.
RESPUESTA
a.- Paciente con Klinefelter.
b.- Infertilidad.

Pharmed Solutions Institute

PGINA 463

MANUAL DE TRABAJO DEL CURSO ENARM CMN SIGLO XXI


c.- Criptorquidia no reparada en la pubertad.
d.- Familiar con cncer testicular
CASO CLINICO
Paciente masculino de 26 aos, es llevado a consulta porque nunca
ha tenido testculo en la bolsa escrotal derecha y presenta
tumoracin redondeada de 6cm., de dimetro en regin inguinal
derecha. Hace 15 das refiere dolor en la misma, secundario a un
golpe que recibi accidentalmente en regin inguinal. Madre
diabtica, hipertensa con ca de mama, padre con hipertensin
arterial. Al examen fsico presenta facies tpica del Sndrome de
Down y en el examen genital se constata una bolsa escrotal derecha
atrfica y una tumoracin redondeada dura, en la proyeccin del
anillo inguinal superficial derecha de 3x4cm., de dimetro
ligeramente dolorosa y fija a planos profundos. Los exmenes
paraclnicos resultaron normales (AFP 12ng/dl, hCG normal) con
diagnstico de testculo no descendido derecho.
PREGUNTA
Cul es la conducta diagnostica ms adecuada a seguir en este
momento?
RESPUESTA
a.- Lmpara de Wood
b.- Ultrasonografa convencional
c.- TAC abdominopelvica
d.- Ultrasonografia doppler color
PREGUNTA
En estudio previamente realizado se observa lesin de 3x3cm, de
eco textura homognea. Cul es la conducta teraputica ms
adecuada a seguir en este momento, de acuerdo a la sospecha
diagnostica?
RESPUESTA
a.- Tumor seminomatoso- Orquipexia
b.- Tumor testicular seminomatoso- Orquiectomia radical
c.- Coriocarcinoma-orquiectomia inguinal
d.- Seminoma- orquipexia-radioterapia
PREGUNTA
Cul es el factor de riesgo ms importante en este paciente para su
diagnstico actual?
RESPUESTA
a.- Sndrome de Down
b.- Criptorquidea
c.- Antecedente familiar de cncer
d.- Traumatismo en testculo
CASO CLINICO
En un paciente que tiene un tumor testicular se efctua un anlisis de
la sangre que muestra concentracin srica de alfa-fetoproteina de
30g/l.
PREGUNTA
Cul es el rango de edad mas probable de este paciente?
RESPUESTA
a.- Lactancia a 3 aos
b.- 10-20 aos
c.- 20 a 30 aos
d.- 30 a 40 aos
PREGUNTA
Qu tumor es mas probable que se este presentando?
RESPUESTA
a.- Tumor de clulas de Leydig
b.- Tumor del saco vitelino
c.- Coriocarcinoma
d.- Teratocarcinoma

CURSO ENARM CMN SIGLO XXI TEL: 36246001

CASO CLINICO
Masculino de 18 aos de edad. Sin antecedentes de importancia y
de ocupacin estudiante. El paciente refiere en su evaluacin, un
cuadro clnico de 5 meses de evolucin, cuando nota la presencia de
una lesin indurada, indolora, sobre el testculo del lado de recho de
lento crecimiento. Se asocia con prdida ponderal de
aproximadamente 10 kilogramos de peso, astenia, adinamia,
malestar general, dilatacin varicosa del testculo izquierdo. El
examen fsico revela una masa ptrea de transiluminacin negativa,
sin extensin a pared escrotal y con el cordn libre. No masas
palpables en el abdomen y sin ganglios supraclaviculares. Las
pruebas inician les muestran elevacin de marcadores tumorales de
testculo elevados. La fraccin B de la hormo na gonadotropina
corinica (HCG) de 5400 y alfa fetoprotena (AFP) de 780. La
telerradiografa de trax muestra algunas lesiones nodulares, sobre
todo en el parnquima pulmonar del lado derecho y en la
tomografa de abdomen se observa un gran conglomerado de
ganglios linfticos retroperito neales, paracavos, inter-cavo-articos,
e inclusive pre y paraarticos.
PREGUNTA
Cul es la conducnta teraputica mas adecuada a seguir en este
caso?
RESPUESTA
a. Quimioterapia mas biopsia
b.- Radioterapia
c.- Orquiectomia radical mas estudio de patologa
d.- Solo sintomtico ya no hay nada que hacer
PREGUNTA
En el seguimiento que se le da al paciente. Por cunto tiempo hay
que darle secuencia por el riesgo de recurrencia?
RESPUESTA
a.- Por 7 meses
b.- Por 2 ao
c.- Por 3 aos
d.- Por 5 aos
CASO CLINICO
Paciente de 29 aos, acude al centro de especialidades por
tumoracin indolora en su testculo izquierdo de un mes de
evolucin sin otra sintomatologa. Se le detecta en la ecografa un
testculo izquierdo hipognico y disminuido de tamao que dificulta
su valoracin, y un testculo derecho con parnquima heterogneo
sospechoso de neoplasia. EF: se palpa un teste izquierdo hipotrfico.
El teste derecho se presenta algo aumentado de tamao. Ambos no
dolorosos, de consistencia normal y de superficie regulares.
Epiddimos y cordones sin hallazgos patolgicos. Se le solicitan
marcadores tumorales que resultan en una alfa-feto protena de 2.7
ng/ml con una beta-hCG de 4.2 mUI/ml. Ecografa con flujo Doppler
se aprecia un teste derecho de 49mm x 36mm x 26mm con lesin
hipoecoica, de bordes lobulados, heterognea, que ocupa
prcticamente todo el testculo, y que presenta aumento del flujo
Doppler. Y un teste izquierdo de 32x22x20mm hetrogneo con
lesin focal ligeramente hipoecoica de aproximadamente 9 mm con
sutil flujo Doppler. Adems, microlitiasis testiculares bilaterales. Al
TAC toraco-abdomino-plvico con contraste, donde no se
identificaron adenopatas ni metstasis en ninguno de los territorios
visualizados. Con el diagnstico de tumor testicular bilateral, se le
realiz una orquiectoma bilateral va inguinal.
PREGUNTA
Cul es la condnducta teraputica mas adecuada a seguir junto con
el procedimiento quirrgico?
a.- Terapia con esteroides
b.- Radioterapia
c.- Manejo con antidepresivos
d.- Tratamiento hormonal sustitutivo.

Pharmed Solutions Institute

PGINA 464

MANUAL DE TRABAJO DEL CURSO ENARM CMN SIGLO XXI


HIPERTROFIA PROSTATICA BENIGNA (HBP)
CIENCIAS BASICAS: En la clnica puede significar: 1) deteccin microscpica de la hiperplasia, es decir la proliferacin del estroma y el epitelio; b)
crecimiento de la prstata detectado por el examen rectal digital o por ultrasonido y c) un grupo de sntomas asociados con la hiperplasia prosttica
y definidos con el trmino sntomas de tracto urinario inferior (STUI). Las diferentes definiciones surgen por que el tamao de la prstata no
siempre correlaciona con los sntomas (solo 30-50% de los hombres con HBP detectada por tacto rectal o ultrasonido, presentan sntomas), as el
termino HBP implica uno o ms de los hallazgos ya mencionados. La HBP es considerada en la actualidad una enfermedad progresiva con origen
hormonal, en donde la dihidrotestosterona (DHT), producto de la accin de la enzima 5-afa-reductasa tipo 2 sobre la testosterona, es responsable.
SALUD PUBLICA: La prevalencia de HBP aumenta en forma lineal con la edad, en todos los grupos tnicos. En general afecta a los hombres mayores
de 45 aos y la presentacin de los sntomas suele darse a los 60-65 aos de edad. Su prevalencia es de >50% y 90% a los 60 y 85 aos
respectivamente. La mortalidad y la frecuencia de complicaciones serias asociada con la HBP bajas, reportndose en 1-3% incluyendo estas
complicaciones la retencin urinaria aguda. CLASIFICACION: Desde el punto de vista histolgico se pueden distinguir al menos los siguientes cinco
tipos de hiperplasia prosttica benigna: Estromal. Fibromuscular. Muscular. Fibroadenomatosa. Fibromioadenomatosa. PATOGENIA: Los STUI
asociados a HBP, tienen origen tanto anatmico como neuro-hormonal, ya que pueden ser producto de la obstruccin mecnica al flujo urinario
secundaria al crecimiento prosttico y/o por el aumento de tono y presin del musculo liso en la uretra, estroma y cuello de la vejiga, mediado por
alfa-1-adrenoreceptores. En los hombres entre los 50 y 70 aos, la masa hiperplstica e hipertrfica de la prstata est formada en un 14% por
ndulos y en un 86% es difusa, que ocurre principalmente en la zona de transicin. En las primeras fases de la hiperplasia predomina ampliamente el
componente estromal de la zona de transicin, donde actuaran al menos tres factores con accin inductora mesenquimatosa embrionario-smil:
Factor bsico de crecimiento fibroblstico (bFGF), factor de crecimiento transformador tipo B1 (TGF-B1), factor de crecimiento transformador tipo
B2 (TGF-B2). Los tres factores, perfectamente identificados, actan sinrgicamente llevando el estroma a un estado mesenquimtico. Adems bFGF
es mitognico, lo que significa crecimiento glandular; ste es regulado por TGF-B2. Es de conocimiento muy antiguo que para que haya hiperplasia
prosttica benigna es indispensable la presencia del testculo, lo que afirma la participacin de los andrgenos, ya sea en forma directa, permisiva o
activadora, como es la accin del KGF (keratinocitic growth factor), que es el primer factor probado como estimulante del crecimiento epitelial
prosttico. Adems se necesita de tiempo, es decir, envejecimiento. El fenmeno histolgico de hiperplasia prosttica benigna es propia del hombre
que envejece. DIAGNOSTICO: Los sntomas de HBP pueden ser secundarios a: 1. Obstruccin; disminucin en el grosor y fuerza del chorro urinario,
pujo al iniciar la miccin, goteo terminal, sensacin de vaciado incompleto o retencin aguda de orina. 2. Disminucin de la capacidad de
almacenamiento en la vejiga; nicturia, urgencia e incontinencia urinaria de urgencia y por rebosamiento. No se ha encontrado una relacin directa
entre la severidad de los sntomas y el tamao de la prstata; algunos hombres presentan sntomas severos de obstruccin urinaria con un
crecimiento prosttico mnimo, mientras que otros presentan sntomas mnimos pero con prstatas grandes. El International Prostate Symptom
Score (IPSS) permite evaluar los sntomas de manera objetiva y reproducible de acuerdo a su severidad en leves, moderados o graves. Para realizar
el diagnostico clnico de HBP, elabore la historia clnica y un examen fsico que incluya: 1.Exploracion de abdomen para detectar vejiga palpable. 2.
Tacto rectal para evaluar las caractersticas de la prstata: tamao, forma, simetra, textura y consistencia. Utilice IPSS y evalu la afeccin de la
calidad de vida del paciente con sospecha inicial de HBP, para mejorar la deteccin y catalogar los STUI de acuerdo a severidad. En pacientes con
STUI y sospecha de HBP, solicite un EGO para descartar la presencia de infeccin urinaria y/o hematuria. Glicemia: con el objetivo de excluir DM. El
ultrasonido vesical y prosttico con medicin de orina residual puede ayudar a determinar el tamao prosttico y el volumen urinario residual, lo
que ayuda a predecir los beneficios de un tratamiento mdico o guiar al tratamiento quirrgico. Tambin puede ser til examinar riones y vejiga
para descartar litiasis en caso de infeccin de vas urinarias repetidas y tumores en caso de hematuria persistente, as como la repercusin de la
obstruccin prosttica sobre el tracto urinario superior. Hay una relacin directa de los niveles de APE y el volumen de la prstata, sin embargo los
pacientes con HBP no tienen mayor riesgo de cncer de prstata. Los valores altos de APE, se relacionan fuertemente con la probabilidad de tener
cncer de prstata. Una prstata con un tamao >30ml, un flujo urinario dbil y APE >1.4ng/ml, son factores de riesgo para progresin de HBP con
un riesgo mayor de presentar retencin aguda de orina (RAO) y necesidad de ciruga, por la tanto deben de recibir tratamiento. La endoscopia
permite evaluar muy bien el tamao prosttico, el cuello vesical, las repercusiones de la uropata obstructiva sobre el detrusor y orificios ureterales.
Si bien se puede hacer en forma ambulatoria y bajo anestesia local, hay que ser cauto en su indicacin, pues en pacientes muy obstruidos y muy
sintomticos existe el riesgo de provocar retencin completa por edema y dolor, adems de infeccin. En muchos pacientes se practica la
endoscopia como una primera parte del procedimiento teraputico endoscpico bajo anestesia. TRATAMIENTO: La terapia de observacin y
vigilancia, es segura para la mayora de los pacientes con sntomas leves a moderados de HBP. Es tambin un tratamiento adecuado en los casos de
pacientes con sntomas moderados a severos pero, que no han desarrollado complicaciones secundarias a la obstruccin del flujo urinario
(insuficiencia renal, retencin urinaria aguda o infecciones recurrentes). Los pacientes con HBP, son sntomas leves a moderados o moderados a
severos pero sin complicaciones y que no presenten impacto en su calidad de vida, pueden ser manejados con vigilancia estrecha que incluya
valoracin anual con TR, APE y US vesical y prosttico con medicin de orina residual. La cafena y alcohol tienen un efecto diurtico e irritante que
aumenta la frecuencia miccional, la urgencia y la nicturia. La reduccin en la ingesta de lquidos reduce la frecuencia miccional en periodos
convenientes. Para mejorara los sntomas, disminuya la ingesta de lquidos libres por la tarde-noche y de irritantes como la cafena y el alcohol en
cualquier presentacin. Las ventajas del tratamiento farmacolgico incluyen: conveniencia y el evitar la morbilidad potencial asociada a la ciruga. La
desventaja es que es de forma indefinida. Los alfabloqueadores tamsulosina, terazosina y doxazosina tienen una eficacia similar sobre el alivio o
mejora de los STUI (relajan el musculo liso en la prstata y cuello de la vejiga), son bien tolerados y su eficacia se mantiene despus de 6-12 meses
de tratamiento. No reduce el tamao de la prstata ni alteran la progresin de HBP. Actualmente existen alfabloqueadores con mejor tolerancia por
su uroselectividad sin disminuir su eficacia. Tal es el caso de alfuzosina y silodosina, estos son ms efectivos que los inhibidores de la 5-alfa
reductasa, para mejorar los sntomas en el primer ao de tratamiento y su efecto se manifiesta desde el primer mes de inicio de tratamiento. Los
alfabloqueadores presentan efectos adversos como hipotensin ortostatica, cansancio, mareos, problemas eyaculatorios y congestin nasal, se
recomiendan en todos los pacientes candidatos a tratamiento mdico. Las dosis recomendadas son: alfazulozina 10mgs diarios, tamsulosina 0.4mg
diarios, terazosina 2-5mg diarios, doxazosina 2-4 mgs diarios. Los I-5ARs inhiben la enzima alfa reductasa, lo que da una disminucin de las
concentraciones de dihidrotestosterona a nivel prosttico, que resulta en una reduccin del tamao de la prstata, una mejora de los sntomas,
menor frecuencia de RAO y de intervenciones quirrgicas, se recomiendan en pacientes con STUI y crecimiento prosttico >40cc. Finasteride 5mg
diarios. En la actualidad se considera a la RTUP, como el tratamiento ms efectivo para la HBP. Los pacientes que son sometidos a tratamientos
quirrgicos transuretrales mnimamente invasivos (TUMT, HIFU, HoLEP, PVP), tienen una mejora menor en el puntaje del IPSS y de la tasa de flujo
urinario mximo, que los pacientes a los que se les realiza RTUP. La RTUP no reseca todo el tejido prosttico susceptible a cncer, por lo tanto deben
evaluarse anualmente con TR y APE. La RTUP tiene los siguientes riesgos: mortalidad >0.25%, intoxicacin hdrica 2%, estenosis de uretra y
contractura de cuello vesical 3.8%, eyaculacin retrograda 65-70%, incontinencia urinaria de esfuerzo, disfuncin erctil. La prostactectomia abierta

CURSO ENARM CMN SIGLO XXI TEL: 36246001

Pharmed Solutions Institute

PGINA 465

MANUAL DE TRABAJO DEL CURSO ENARM CMN SIGLO XXI


est indicada en pacientes con prstatas de >80cc y en aquellos con complicaciones asociadas, como litiasis vesical o que ameriten reseccin
diverticular.
CASO CLINICO
Se trata de paciente masculino de 62 aos de edad el cual acude a
consulta de control ya que padece de diabetes mellitus desde hace
15 aos, actualmente con depuracin de creatinina menor a 60 ml/
min. refiere que desde hace 7 das ha presentado ardor al orinar as
como dificultad, adems de tardar en terminar de miccionar, al
interrogatorio refiere que desde hace 6 meses presenta disminucin
del chorro de la orina as como goteo, niega hematuria usted
considera que actualmente cursa con IVU la cual controla y cita para
valorar la prstata, 15 dias porterior al tratamiento por IVU evalua
nuevamente al paciente observando al tacto rectal crecimiento no
indurado, levemente doloroso, solicita antgeno prosttico
reportado en 5.0 ng/ml. Urocultivo negativo, EGO negativo.

nicturia. En sus antecedentes personales no existen hbitos txicos


ni alergias a frmacos, desde hace aproximadamente cinco aos est
a tratamiento con Nifedipino por Hipertensin arterial y con
Simvastatina por dislipemia.

PREGUNTA
Cul es la conducta a seguir para establecer el diagnostico.
RESPUESTA
a.- USG renal.
b.- USG vesical.
c.- USG prostatico.
d.- Enviar a segundo nivel

PREGUNTA
Qu exploraciones estaran indicadas inicialmente para realizar un
diagnstico clnico en nuestro paciente?
a.- Sedimento y cultivo de orina
b.- Determinacin de PSA
c.- Anamnesis adecuada y tacto rectal
d.- Hematimetra y bioqumica

CASO CLINICO
Paciente masculino de 60 aos, diabtico e hipertenso de 10 aos
de evolucin. Acude a urgencias refiriendo que se encontraba en
una fiesta e inicio con dolor suprapbico, e iba a orinar casi cada
media hora y al ir a orinar presento pujo, tenesmo y hematuria asi
como aumento leve de la temperatura. Al interrogatorio refiri que
haca 2 meses inicio con nicturia y disminucin de la fuerza del
chorro de orina. EF: Paciente con aliento alcohlico, con
incoordinacin motora, cardiopulmonar sin compromiso, abdomen
blando depresible con dolor en regin de hipogastrio, Giordano
negativo, resto sin alteraciones.

PREGUNTA
Realizamos una historia clnica lo ms completa posible. En el tacto
rectal se objetiva una prstata agrandada de tamao de forma
uniforme, de consistencia elstica y superficie lisa. En las pruebas
complementarias obtenemos los siguientes resultados: Glucemia
basal 98 mg/dl, Hematimetra y funcin renal normales. Sedimento y
urocultivo normales - PSA 2.5ng/ml y porcentaje de PSA libre
(PSAlibre/ PSA total) 30%.
Cual sera tu diagnstico con estos datos clnicos y
complementarios?
RESPUESTA
a.- Hipertrofia benigna de prstata
b.- Cncer de prstata
c.- Prostatitis crnica abacteriana
d.- Prostatitis mas uretritis

PREGUNTA
Cul es el diagnstico ms probable en este caso?
RESPUESTA
a.- Cncer de prstata
b.- Hiperplasia prosttica benigna
c.- Glomerulonefritis
d.- Cistitis
PREGUNTA
Cul es la conducta teraputica ms adecuada a seguir en este
caso?
RESPUESTA
a.- Quimioterapia ms prostactectomia
b.- Doxaxozina mas finasteride
c.- Ciprofloxacino mas fenazopiridina
d.- IECA mas esteroide
PREGUNTA
Cul de las siguientes complicaciones es menos probable que
aparezca secundaria a una RTUP?
a.- Eyaculacion retrograda
b.- Estenosis de uretra
c.- Infecciones de vas urinarias recurrentes
d.- Incontinencia urinaria

PREGUNTA
En este paciente con esta situacin clnica Cul es el diagnostico
diferencial menos probable?
RESPUESTA
a.- Patologa prosttica
b.- Insuficiencia cardaca congestiva
c.- Diabetes Mellitus
d.- Efecto secundario a frmacos

PREGUNTA
Con este diagnostico. Cul sera la actitud teraputica adecuada?
RESPUESTA
a.- Derivacin a Urologa
b.- Medidas higinico-dietticas y tratamiento con fitoterapia
c.- Medidas higinico-dietticas y tratamiento con fitoterapia mas
terazosina
d.- Antibioticoterepia y medidas higienico dietticas
PREGUNTA
Cundo citaramos al paciente para la revisin?
a.- A los dos meses para realizar tacto rectal y analtica de orina
b.- Al ao, si ha existido mejora clnica, con tacto rectal, analtica de
sangre con PSA y orina
c.- Cada seis meses con tacto rectal, analtica de sangre con PSA y
orina
d.- A los 15 dias, para interrogar mejora clnica

CASO CLINICO
Paciente varn de 65 aos de edad que acude a nuestra consulta por
presentar desde hace varios meses clnica miccional de polaquiuria y

CURSO ENARM CMN SIGLO XXI TEL: 36246001

Pharmed Solutions Institute

PGINA 466

MANUAL DE TRABAJO DEL CURSO ENARM CMN SIGLO XXI


CANCER DE PROSTATA
CIENCIAS BASICAS: Es el crecimiento anormal de las clulas de la glndula prosttica, que adems tienen la capacidad, de invadir otros rganos. Es
un tumor que nace del epitelio acinar y/o ductal de la prstata que puede variar considerablemente en su diferenciacin glandular, anaplasia y
comportamiento. Es una de las neoplasias malignas ms frecuente en los hombres y se incrementa conforme aumenta la edad. En la prstata
fundamentalmente se distinguen 3 zonas: la zona Central cruzada por los conductos eyaculadores que supone un 25% de la glndula, la zona
Transicional que rodea a la uretra posterior con un 5% del volumen glandular y la zona Perifrica que ocupa un 70% del volumen glandular. Ms de
95% de todos los canceres de prstata son adenocarcinomas y desde el punto de vista anatomopatologico, el 97% de los tumores se originan en la
periferia. La histologa del cncer de prstata es muy importante, se ha visto que un 95% son adenocarcinomas acinares originados en la porcin
glandular de la prstata y solo un 4% es transicional, escamoso o endometroide, estos derivan de los conductos y por lo tanto son
hormonodependientes. El 1% son sarcomas. SALUD PUBLICA: El 87.6% de las defunciones se registraron en la poblacin mayor de 65 aos. El cncer
de prstata se ubic en segundo lugar en el grupo de edad postproductiva, con una tasa de 71.7 por 100 mil habitantes. Existe diferencia notable en
la incidencia del cncer entre el mundo oriental y occidental, atribuibles entre otros factores a la ingesta de grasas en la dieta. Un ejemplo lo
constituyen los chinos, que de una incidencia de 0.8 por 100 000 hab en su vida nativa, pasan a tener tasas de 18.6 cuando se integran a la
civilizacin occidental. La raza negra en EE.UU tiene la mayor tasa de incidencia con 100.2 por 100 000 hab. PATOGENIA: En la zona Perifrica se
desarrollan el 68% de los cnceres, en la zona Central un 8% y en la zona Transicional un 24%. La zona Perifrica es accesible al tacto rectal mientras
que los tumores que se desarrollan en las zonas Central y Transicional se valoran mejor con la ecografa. Se han postulado factores genticos, las
razas negras y los escandinavos tienen una alta incidencia, mientras que los japoneses tienen la ms baja. Tambin existira una predisposicin
familiar, pariente en primer grado de caso ndice tiene un riesgo 10% mayor de tener un cncer prosttico. Otros factores implicados son la
alimentacin y el ambiente. Se ha visto que el aceite de soya es un protector ya que se transforma en un estrgeno dbil; que los habitantes de
zonas rurales tienen una mayor incidencia atribuida a los insecticidas agrcolas; las prostatitis vrales por citomegalovirus o virus herpes a repeticin
tambin aumentaran las posibilidades de desarrollar un cncer de prstata. Un factor clave es el factor hormonal, se relaciona claramente con la
presencia de testculos funcionales; los eunucos no tienen cncer de prstata. La presencia de receptores esteroides en las clulas tumorales y la
respuesta positiva a la supresin de los andrgenos as como los altos niveles de Dihidrotestosterona en las clulas cancerosas, apoyan la hiptesis
hormonal. DIAGNOSTICO: En general los cnceres de prstata son silenciosos, crecen lentamente, invaden primero la cpsula prosttica, luego
ganglios pelvianos (obturatrices e iliacos) y dan metstasis preferentemente al hueso. Slo dan sintomatologa obstructiva. Las lesiones en el hueso
son osteoblsticas (se ve ms blanco y algodonoso el hueso en la radiografa) y los sitios ms frecuentes de metstasis son: Columna sacra, crestas
ilacas, columna lumbar, columna dorsal, crneo, etc. En hombres con mayor riesgo, con antecedentes familiares directos de cncer de prstata o de
mama, se debe iniciar un abordaje de deteccin a los 40 aos. El diagnstico se establece a travs de estudios clnicos y de gabinete: tacto rectal,
ultrasonografa, TAC, RMN, y marcadores sricos del tumor; entre estos, sin duda el ms importante es el antgeno prosttico especfico (APE) el cual
se produce nicamente en el citoplasma de clulas prostticas benignas y malignas. Su nivel srico se correlaciona con el volumen de ambos tejidos,
maligno y benigno. El APE, se determina como normal <4ng/ml, intermedio 4.1-10ng/ml y altamente sospechoso arriba de 10ng/ml. Estos dos
ltimos requieren de biopsia, ya que las dos terceras partes de cncer prosttico se ubican en personas con resultados de 10 y ms ng/ml. El APE ha
demostrado ser til para supervisar la eficacia del tratamiento y para controlar la recada despus de que el tratamiento ha terminado. Los niveles
de la fosfatasa acida prosttica (FAP) en la sangre pueden elevarse en pacientes que tienen ciertas enfermedades benignas de la prstata o cuyo
cncer est en la fase temprana. En hombres entre 50 y 55 aos se da una forma poco frecuente de cncer prosttico con Gleason 8 a 10, de
crecimiento rpido y aneploides, en general se desarrollan en dos o tres meses. Es importante que en el informe que se recibe del patlogo se
establezca el grado de Gleason, que es una clasificacin basada en la diferenciacin celular y la relacin estroma-glndula del cncer (no
mideanaplasia), para esto se le asigna un puntaje de 1 a 5 a la zona ms "mala" del cncer y otro a la ms "buena" y se suman. La escala de Gleason
va del 2 al 10, siendo el 2 el cncer ms benigno. En general los cnceres con Gleason del 2 al 4 tienen buen pronstico, del 5 al 7 son de pronstico
intermedio y del 8 al 10, son los de peor pronstico. El Gleason es importante porque determina el pronstico y evolucin. La biopsia de prstata se
considera como "el estndar de oro" pero generalmente se realiza luego de constatar anormalidades en el tacto rectal o elevacin del APE, por lo
que la sensibilidad del mtodo est sobrestimada. La Sociedad Americana de Cncer recomienda a todos los varones mayores de 50 aos se
sometan a tacto rectal (TR) y deteccin de APE. En los casoso de antecedentes familiares, el control se deber iniciar a los 40 aos. Si el TR y el APE
son normales se recomienda control anual. Si el TR es normal y el APE est entre 4.1 y 10 ng/ml se realizar Ecografa Transrectal (ETR) para
determinar la densidad del APE y detectar ndulos con el fin de realizar biopsia prosttica dirigida o aleatoria. Si el TR es normal y el APE mayor de
10 se realizar biopsia dirigida o aleatoria mediante ETR. Si el TR es sospechoso se realizar biopsia de dicha zona, independientemente del APE.
TRATAMIENTO: El tratamiento depende del TNM y de la edad del paciente. Las personas con cncer T1-T2 tempranos, localizados, que no han
invadido la cpsula, debern ser sujetos a prostatectoma radical, resecando la prstata y cuello vesical, ganglios obturatrices, ganglios ilacos y
vesculas seminales. La sobre vida de estos pacientes, en general es buena; es decir el 90% sobrevive a los 15 aos, con tratamiento precoz. Las
complicaciones de la prostatectoma radical son la impotencia que se produce en casi el 100% de los tratados y la incontinencia de orina se ve en un
30% aproximadamente. Si se diagnostica cncer TNM IV, con metastasisi a distancia solo se realiza tratamiento paliativo con supresin del estimulo
hormonal, de estos pacientes con o sin tratamiento 10% se muere antes de ao y otro 10% vive mas de 10 aos. El promedio de sobrevida es dos a
tres aos. No es un cncer tan maligno responde a la supresin de testosterona. Para obtener un bloqueo hormonal completo se realiza:
Orquiectoma. Frmacos bloqueadores andrognicos. Para el bloqueo andrognico se pueden utilizar anlogos de LHRH como el Leuprolide y
Decapeptil que tienen el problema de su alto costo. Tambin se puede utilizar flutamida. Se ha ensayado la administracin de estrgenos como el
dietilestilbestriol en dosis de 2.5 mg al da, es menos costoso, acta bloqueando los receptores perifricos de testosterona. En general con el
tratamiento paliativo se obtiene una mejora de la sintomatologa general y tambin de la uropata obstructiva. No se realiza quimioterapia.
Radioterapia solo local con implantes en el cncer TNM I o II. Tambin se utiliza como tratamiento local de las metstasis de columna, pero slo es
paliativo. Tratamiento del tumor incidental: En los pacientes con expectativa de vida menor de 10 aos y con un tumor focal A1, se les aplicar
conducta conservadora efectuando controles de APE, TR y EcoTR cada 6 meses. En los pacientes con expectativa mayor de 10 aos o tumores
focales A2 se realizar ciruga radical o radioterapia local. Tratamiento del tumor localizado, el tratamiento de eleccin es la prostatectomia radical,
siempre que el estado general sea bueno y la esperanza de vida sea superior a 10 aos. En los casos con mal estado general, edad avanzada o
negativa al tratamiento quirrgico puede ser til la radioterapia radical o bien el bloqueo anbdrogenico. Tratamiento del tumor con extensin
extraprosttica: Los pacientes con afectacin capsular no son buenos candidatos para la ciruga radical pues ms del 50% van a tener afectacin
ganglionar y la exresis puede ser incompleta. El tratamiento con bloqueo hormonal completo logra disminuir el volumen tumoral y facilita la
ciruga, pero pocos pacientes logran disminuir el estadio tumoral y la supervivencia no se modifica. El tratamniento adecuado depender de la
presencia de sntomas obstructivos para instaurara el bloqueo andrognico completo o la desobstruccin mediante RTU. Tratamiento del tumor
diseminado: El tratamiento es mediante la supresin hormonal, bien sea por castracin quirrgica o bien con anlogos de la LHRH. Si se quiere

CURSO ENARM CMN SIGLO XXI TEL: 36246001

Pharmed Solutions Institute

PGINA 467

MANUAL DE TRABAJO DEL CURSO ENARM CMN SIGLO XXI


actuar sobre los andrgenos suprarrenales se asociar un antiandrgeno al tratamiento, efectundose un bloqueo completo. Prostatectomia radical:
En esta ciruga se extirpan completamente la prstata, vesculas seminales y se restituye la va urinaria mediante anastomosis vesicouretral. Se
asocia a linfoadenopata de ganglios iliacos internos (principal indicacin: cncer prosttico localizado T2a y T2b), complicaciones: mortalidad (0,2%),
incontinencia urinaria (2 7%), disfuncin erctil (100%), estenosis uretrovesical (1-3%), linfocele (1-3%). La disfuncin erctil que puede ocurrir
despus de esta operacin puede ser evitada al respetar los haces neurovasculares localizados en hojas de la fasia plvica lateral y que estn muy
vecinos a la glndula prosttica. Se considera que no existe enfermedad residual despus de la ciruga cuando el APE desciende al cabo de algunas
semanas a cifras indetectables (<0,02 ng/ml). Por el contrario, el APE que se mantiene detectable o el APE en sostenido ascenso son indicadores de
recidiva tumoral o recidiva bioqumica. Radioterapia: Usada como terapia definitiva en alrededor de 1/3 de los pacientes. Se emplea radioterapia
externa y la radioterapia interticial (braquiterapia). Es eficaz con resultados similares a la ciruga al tratar tumores pequeos (intra prostticos).
Complicaciones: disfuncin erctil (30-45%), toxicidad gastrointestinal (3-5%), incontinencia urinaria (menos del 5%). Terapia hormonal: Pilar del
tratamiento de cncer prosttico metastsico (etapa IV). Los andrgenos (testosterona) son fundamentales para el crecimiento y funcionamiento de
las clulas prostticas. Son producidos mayoritariamente en los testculos (95%) y en la glndula suprarrenal (5%). La forma ms simple de emplear
la hormoneoterapia es la castracin quirrgica. Hormonoterapia de 1 lnea; orquiectoma, agonistas LHRH (zoladex, lupron), antagonistas LHRH
(abarelix), antiandrgenos (casodex) (bicalutamida), antiandrgenos/inhibidores de 5alfa-reductasa tipo II (flutamida). Hormonoterapia de 2 lnea:
bloquear sntesis adrenal de andrgenos (hodrocortisona, ketoconazol, aminoglutemida), basados en el hecho de que las glndulas adrenales
continan produciendo andrgenos despus de una castracin mdica o quirrgica, estrategia conocida como bloqueo andrognico mximo.
PRONSTICO: Cncer prosttico localizado, muestra supervivencia de 65 75 % a los 10 aos. Cncer prosttico diseminado; supervivencia del 55%
a los 10 aos.- Cncer prosttico con metstasis seas; supervivencia a los 10 aos: 15%. Medio de sobrevida = 30 meses (aunque un 10% vive ms
de 10 aos)
CASO CLINICO CANCER DE PROSTATA
Se trata de masculino de 81 aos de edad el cual se encuentra en
cuidados paliativos por cncer de prstata con metstasis, el cual
recibe tratamiento con goserelina, es ingresado a urgencias debido a
que presenta en casa estado confusional con tendencia a letargia, a
la exploracin fsica se observa con estado avanzado de emaciacin,
mucosas orales secas, deshidratacin moderada, pobre respuesta a
estmulos dolorosos pero presente, el examen mental es difcil
realizar por embotamiento, las constantes vitales son TA 110/50
mmHg, FC de 112 lpm, se observa afebril, debido a su estado actual
y antecedentes se considera que el paciente presenta hipercalcemia,
se realizan medidas de estabilizacin en espera de laboratorios.

disminucin de 15 kg de peso; se realizaron exmenes de imgenes


donde se encontr un proceso infiltrativo de hueso sacro. Se
present al servicio de urgencias de nuestro hospital mostrando
retencin aguda de orina de cinco horas de evolucin, por lo que se
solicita interconsulta al servicio de urologa. Se informa de hallazgo
de globo vesical, tacto rectal con prstata grado III, fcilmente
palpable al tacto bimanual, de supercie nodular, ptrea, ja, no
dolorosa, sin aumento de temperatura. Se instala sonda Foley,
laboratorios: Hb: 14.4; urea: 31; creatinina srica: 1.0; EGO:
eritrocitos: incontables, leucocitos: 0-2 x c; APE de 3.4 ng/ml;
ultrasonido transrectal de prstata con reporte de prstata de 80 cc,
con una medicin de: 66.3 x 44.2 x 52.5.

PREGUNTA
Cual es la conducta menos apropiada a seguir considerando el
estado de hipercalcemia.
RESPUESTA
a.- Solucion salina.
b.- Pamidronato.
c.- Furosemida.
d.- Dexametazona.

PREGUNTA
Cul de los siguientes es factores es menos probable que nos eleve
el antgeno prosttico?
RESPUESTA
a.- Tacto rectal
b.- Biopsia de prstata
c.- Prostatitis
d.- RAO

CANCER DE PROSTATA
Se trata de masculino de 81 aos de edad el cual se encuentra en
cuidados paliativos por cncer de prstata con metstasis, el cual
recibe tratamiento con goserelina, es ingresado a urgencias debido a
que presenta en casa estado confusional con tendencia a letargia, a
la exploracin fsica se observa con estado avanzado de emaciacin,
mucosas orales secas, deshidratacin moderada, pobre respuesta a
estmulos dolorosos pero presente, el examen mental es difcil
realizar por embotamiento, las constantes vitales son TA 110/50
mmHg, FC de 112 lpm, se observa afebril, debido a su estado actual
y antecedentes se considera que el paciente presenta hipercalcemia,
se realizan medidas de estabilizacin en espera de laboratorios.

PREGUNTA
Cul es la conducta diagnostica ms adecuada a seguir, de acuerdo
a los hallazgos de tacto rectal?
RESPUESTA
a.- Biopsia de prstata
b.- Ultrasonido prosttico
c.- Resonancia magntica
d.- Tomografa

PREGUNTA
Cual es la conducta menos apropiada a seguir considerando el
estado de hipercalcemia?
RESPUESTA
a.- Solucion salina.
b.- Pamidronato.
c.- Furosemida.
d.- Dexametazona
CASO CLINICO
Paciente de 67 aos que presenta dolor en el glteo derecho que se
irradia hacia los muslos afectando la bipedestacin y la marcha. Con
tiempo de enfermedad de cuatro meses, asociado con la

CURSO ENARM CMN SIGLO XXI TEL: 36246001

PREGUNTA
En qu sitio anatmico es menos probable encontrar METs
secundarias a ca de prstata?
RESPUESTA
a.- Ganglios linfticos
b.- Hueso
c.- Pulmn
d.- Pncreas
PREGUNTA
Cul de los siguientes factores es menos probable que nos oriente
sobre un mal pronstico, para este paciente?
RESPUESTA
a.- Dolor
b.- METs viscerales
c.- Hematuria
d.- Anemia con Hb <13mg/dl

Pharmed Solutions Institute

PGINA 468

MANUAL DE TRABAJO DEL CURSO ENARM CMN SIGLO XXI


TRAUMA CRANEOENCEFALICO (TCE)
CIENCIAS BASICAS: En adultos, es definido como un intercambio brusco de energa mecnica que genera deterioro fsico y/o funcional del contenido
craneal. Se consigna como alteracin del contenido enceflico el compromiso de conciencia, la amnesia postraumtica y/o un sndrome vertiginoso
o mareos persistentes. Tambin debe considerarse como un signo de disfuncin del contenido craneal la aparicin de una cefalea holocrnea
persistente y progresiva que puede o no acompaarse de vmitos. Se distingue de la Contusin de crneo, que corresponde a un impacto mecnico
sobre la bveda craneana que no produce alteracin del contenido craneano, y que puede asociarse a dolor
CLASIFICACION DE TCE SEGN OMS
local. SIGNOS DE ALARMA, en la evaluacin prehospitalaria o la llegada a urgencias: Deterioro progresivo de la
Fracturas de crneo:
conciencia (disminucin de Glasgow), signos de focalidad neurolgica, cefalea progresiva, vmitos explosivos
Fracturas de la bveda
recurrentes, agitacin psicomotora, convulsiones, amnesia antergrada de ms de 30 min, cambio en el tamao
Fracturas de la base
de las pupilas, sospecha de herida craneal penetrante, intoxicacin, evidencia clnica o radiolgica de fractura de
Fracturas de los huesos de la cara
crneo, sat. O2 <80%, hipotensin. SALUD PUBLICA: Constituye una de las principales causas de mortalidad e
Otras y las fracturas inclasificables
Mltiples fracturas que afectan el
incapacidad en la poblacin menor de 40 aos. Cada ao fallece 1,000 000 de personas vctimas de trauma
crneo o la cara con otros huesos
craneal severo. Los accidentes con vehculo automotor son la principal causa de traumatismos. El trauma causa
Lesin intracraneal
150,000 muertes en EU y un tercio se acompaa por TCE severo. Entre 2-10% de pacientes con TCE tienen
Conmocin
asociada lesin cervical. PATOGENIA: Se distinguen 2 mecanismos bsicos: la colisin o traumatismo directo, en
Laceracin cerebral y contusin
Hemorragia subaracnoidea,
el cual actan como formas lesivas la energa cintica y la deformante. Y el traumatismo indirecto en el que no
subdural y extradural
existe contacto previo, cabeza/objeto agresor, siendo las fuerzas lesivas la aceleracin angular pura y la
Hemorragias intracraneales
hiperpresin transmitida pura. El impacto mecnico origina la degeneracin neuronal mediante 3 mecanismos
postraumticas inespecficas
bsicos: 1. Mecanismo lesional primario, son lesiones nerviosas y vasculares producidas inmediatamente por la
Lesin intracraneal de naturaleza
agresin biomecnica, las lesiones resultantes de la agresin primaria son: fracturas craneales, contusiones,
inespecfica
laceraciones, hematomas intracerebrales, lesin axonal difusa. 2. Mecanismo lesional secundaria; se destacan el
GRAVEDAD DE TCE
edema cerebral (con el trauma se activa el edema vasognico, que ocurre por aumento de la permeabilidad de los
Leve
14-15 puntos
capilares, con ruptura local de la barrera hematoenceflica (BHE), aumento del lquido intersticial, y el edema
Moderado
9-13 puntos
citotxico, que ocurre por dao isqumico de la clula nerviosa), isquemia local, gluclisis anaerobia, edema de los
Severo
Menos de 9 puntos
astrositos, activacin de cascadas enzimticas y de las endotelinas, por formacin y liberacin de radicales libres de
oxgeno. En los traumatismos cerrados, el impacto de las fuerzas se produce en las zonas donde el crneo se pone en contacto directamente con el
encfalo, pudiendo producir lesiones en la zona de impacto (lesin por golpe) y en la zona diametralmente opuesta (lesin por
contragolpe).,tambin lesiones por cizallamiento, las lesiones resultantes son; hipotensin, hipercapnia, hipoxemia, hipertermia, hipoglucemia,
acidosis, hiponatremia, hipertensin intracraneal, hematoma cerebral tardo, edema cerebral, convulsiones, vasoespasmo. 3. Mecanismo lesional
terciario; engloban una serie de procesos neuroqumicos y fisiopatolgicos complejos, concatenados, con posibilidad de retroalimentacin positiva,
entre s que inician inmediatamente tras el TCE. CLASIFICACION: En la escala de coma de Glasgow se puede obtener una calificacin mxima de 15 y
un mnimo de 3. Un paciente que presenta cualquiera de los siguientes signos debe considerarse que sufre un TCE severo: Anisocoria, dficit motor
localizado, fractura abierta del crneo con exposicin de masa enceflica o salida de LCR, deterioro neurolgico, fractura deprimida de la bveda del
crneo. Otra clasificacin es en: BAJO RIESGO; Asintomtica, cefaleas, mareos, hematoma, laceracin o scalp de cuero cabelludo, ausencia de
criterios de moderado o alto riesgo. Actitud a seguir; Si no presentan otras lesiones asociadas que requieran ingreso hospitalario u observacin se
envan a su domicilio siempre que una segunda persona pueda observar la evolucin del paciente en las prximas horas. Debe informarse por escrito
sobre signos o sntomas de alarma, ante la presencia de las cuales debe consultar de nuevo al hospital. MODERADO RIESGO; Historia de perdida
transitoria de la conciencia, intoxicacin por alcohol y drogas, cefalea progresiva, vmitos persistentes, amnesia peritraumatica, politraumatismo
que impide adecuada valoracin clnica del TCE, traumatismo facial severo, sospecha de nio maltratado, edad menor de 2 aos excepto lesin
trivial. Actitud a seguir; deben permanecer en observacin al menos 24 hrs, pueden reducirse a 12 hrs la observacin si no hay sintomatologa
neurolgica, TAC de crneo normal, no existen otras lesiones asociadas que requieran ingreso hospitalario. Si existe sintomatologa neurolgica
evidente, el periodo de observacin debe prolongarse y valorar TAC craneal de control a las 12-24 hrs. ALTO RIESGO; Disminucin del nivel de
conciencia actual o progresivo no claramente debido a otras causas (metablico, epilepsia), signos neurolgicos de focalidad, hundimiento o herida
penetrante de crneo, sospecha de fractura de la base del crneo (otorrea, hemotmpano, rinorrea, hematoma en anteojos, hematoma
retroauricular), convulsiones postraumticas, respiracin irregular o apnica. Actitud a seguir Una vez diagnosticados y estabilizados, deben pasar al
rea de tratamiento definitivo (quirfano o UCI). Desde el punto de vista patolgico, pueden existir tres tipos fundamentales de lesiones cerebrales:
1. Conmocin o concusin cerebral; caracterizada clnicamente por una breve perdida de conciencia, con un corto periodo de amnesia seguida de
una recuperacin rpida y total, sin ningn signo neurolgico focal. No hay lesin estructural macroscpica del cerebro, tan solo se producen
lesiones por estiramiento de los tractos axonales de la sustancia blanca, con perdida reversible de su funcin, responsables de la prdida de
conciencia transitoria. 2. Contusin cerebral; sobre todo en polos frontales y temporales, por el contacto entre la superficie cerebral y el interior del
crneo, que abarcan desde una simple magulladura en una pequea rea cortical, hasta lesiones extensas, a menudo hemorrgicas, de gran parte de
la superficie cerebral, con dao en la sustancia blanca y el mesencfalo, clnicamente alteracin del nivel de conciencia, desde confusin, inquietud y
delirio y grados variables de coma, estos pacientes deben ser hospitalizados para observacin, dado el desarrollo tardo de edema cerebral. Realizar
TAC, valora necesidad de iniciar tratamiento para HIC. Las contusiones que producen efecto de masa requieren ciruga urgente. 3. Lesin cerebral
difusa; presencia de un coma prolongado de das o semanas, lesin frecuente con alta mortalidad. Su diagnstico es presumible cuando en la TAC
cerebral no se aprecia una lesin ocupante de espacio en un paciente con coma profundo, adems de la situacin de coma suelen presentar
posturas de descerebracin o decorticacin, y frecuentemente presentan signos de disfuncin autonmica. DIAGNOSTICO: Los complementarios
ms frecuentemente utilizados para decidir el diagnstico y tratamiento de este tipo de traumatismo son los siguientes: Radiografa de crneo: Se
deben indicar tres vistas fundamentales: anteroposterior, lateral y Towne. Las dos primeras nos permiten apreciar lesiones en la bveda craneal
(fracturas, presencia de cuerpos extraos intracraneales, desplazamientos de la pineal calcificada, distasis de suturas). La vista de Towne nos
permite visualizar la fosa craneal posterior, y parcialmente la proyeccin posterior de los peascos del hueso temporal.Angiografa carotdea: Es un
estudio invasivo, que requiere de cierta demora en su ejecucin, actualmente ha sido desplazada por la tomografa axial computarizada (TAC),
aunque en los centros que no cuentan con este ltimo recurso, mantiene toda su vigencia e importancia. Permite apreciar desplazamientos de los
patrones vasculares normales debido a lesiones intracraneales con efecto de masa (hematomas, higromas, contusiones). Tambin permite visualizar
zonas con vasoespasmo, fstulas vasculares y lesiones vasculares posteriores al trauma.Tomografa axial computarizada: Constituye en la actualidad
el estudio ecogrfico ms eficaz frente a este grupo de trastornos. Permite la visualizacin directa de lesiones intracraneales, su volumen y forma, as
como las estructuras que afecta, y la visualizacin indirecta de otras lesiones, como el edema cerebral. A continuacin, se expone la clasificacin de
las lesiones enceflicas detectadas por la TAC craneal en los pacientes con TCE descrita por Marshall, tambin denominada como clasificacin del

CURSO ENARM CMN SIGLO XXI TEL: 36246001

Pharmed Solutions Institute

PGINA 469

MANUAL DE TRABAJO DEL CURSO ENARM CMN SIGLO XXI


Traumatic Coma Data Bank (TCDB): Distingue 4 tipos de lesiones difusas y 2 tipos de lesiones focales: Clasificacin de los hallazgos de la primera
tomografa tras el TCE: A) Lesin difusa I; sin patologa visible. B) Lesin difusa II; cisternas visibles, con desviacin de la lnea media hasta 5mm y/o
sin lesin mayor de 25 ml. C) Lesin difusa III; cisternas comprimidas o ausentes, con desviacin de la lnea media hasta 5 mm y/o sin lesin mayor
de 25ml. D) Lesin difusa IV; desviacin de la lnea media mayor a 5 mm, sin lesin mayor de 25ml. E) Toda lesin evacuada quirrgicamente. F)
Lesin mayor de 25ml no evacuada quirrgicamente. TRATAMIENTO: Inicial de las situaciones de amenaza vital: 1. Asegurar la permeabilidad de la
va area con control de la columna cervical. 2. Oxigenacin y ventilacin adecuada. 3. Control de la hemorragia externa y mantener la presin
arterial. 4. Evaluacin del estado neurolgico. 5 Investigar otras lesiones traumticas. Objetivo del tratamiento en TCE: Posicin de la cabeza a 30
sobre el plano horizontal, analgesia eficaz, normotermia, PaO2 >70mmHg, normocapnia, presin arterial media >90mmHg, euvolemia, Hb >10mg/dl,
osmolaridad plasmtica >290mOsm, glucemia <200mg/dl, profilaxis de convulsiones precoces. Para la conmocin cerebral y sndrome
postraumtico que pueden ser manejados por la guardia de Ciruga: vigilar cada 4 horas posibles signos de alarma neurolgica (alteraciones de
conciencia, pupilas y motilidad de extremidades), los cuales deben ser tratados sintomticamente (analgsicos y antigravitatorios segn necesidad)
durante perodos prudenciales de tiempo en la sala de observaciones, y de ser preciso o requerirse, sern evaluados por la guardia de Neurociruga a
solicitud de la guardia de Ciruga General. Si el paciente presenta agitacin severa, debe ser sedado, de preferencia con Midazolam 0,10- 0,15 mg
endovenoso en bolo inicial, seguido por infusin endovenosa de 0,2- 0,4 mg x Kg/h. Puede utilizarse tambin morfina 0,10 mg endovenoso de inicio,
seguido por infusin de 1- 2 mg/h. Farmacologa de TCE; MANITOL, su efecto rpido sobre la PIC es especialmente til en situaciones de urgencia, su
accin es rpida y fugaz, obtenindose el efecto mximo a los 40 minutos de infusin en bolo, en urgencias, redujo con mayor frecuencia la
dilatacin pupilar y mejoro la evolucin a los 6 meses, tienden a mostrar mayor sobrevida y mejor pronstico. Indicaciones: a todo paciente
hemodinamicamente estable con signos de herniacin cerebral (anisocoria, signos de decorticacin-descerebracin) se le debe administrar manitol
en bolo rpido, realizando a continuacin una TAC craneal urgente. Dosis se comienza con un bolo de 1-2 g/kg, se puede repetir cada 6 hrs.
FUROSEMIDA, su ventaja sobre el manitol es su efecto sobre la osmolaridad es menos marcado, aunque puede producir trastornos electrolticos, la
desventaja es que es menos efectivo en el tratamiento del edema cerebral. Indicaciones: lesiones hemorrgicas cerebrales, debera reservarse para
pacientes en los que la expansin de volumen producida por el manitol pudiera resultar perjudicial (cardiopatas, insuficiencia renal). CORTICOIDES,
son eficaces para disminuir el edema cerebral en los procesos tumorales o inflamatorios pero en el edema, debido al traumatismo, no parecen ser
tiles. La administracin de pre-hospitalaria de sol.de cloruro de Na al 7.5% para trauma e hipotensin se asocia con un incremento
significativamente mayor en la presin arterial comparado con la infusin de Ringer-lactato. Profilaxis antibitica, siempre en los traumatismos
abiertos y penetrantes, cuando existan signos clnicos cardiolgicos de fractura de la base de crneo, se emplearan antibiticos de amplio espectro.
Profilaxis de crisis convulsivas; se establecer precozmente un tratamiento de fondo preventivo de las mismas siempre que la lesin sea
supratentorial con afectacin del parnquima cerebral (hundimientos, contusiones, dislaceraciones, hematomas intraparenquimatosos). Los
anticomiciales usados son; fenobarbital (100mg c/8hrs) tanto enteral como parenteral y fenitoina (100mg c/8 hrs). TCE grave; el exceso de tono
simptico en la dinmica vascular cerebral y sus efectos sobre la PC debe ser controlado con una adecuada sedacin y analgesia; opiceos
8fentanilo, morfina), benzodiacepinas, propofol. La intubacin debe ir siempre acompaada de una adecuada sedacin, una correcta relajacin
muscular ya que si esta no se consigue durante la maniobra de la PIC esta aumentara, el relajante muscular ideal para estos pacientes es la
succinilcolina. COMPLICACIONES: Hematoma extradural o epidural: coleccin de sangre entre el crneo y la duramadre cuya causa ms frecuente es
la lesin traumtica de la arteria menngea, sospechar en paciente que ha sufrido TCE, que tras un periodo de 1-24 hrs entra en estado de coma,
pudiendo haber dilatacin pupilar del lado lesionado y hemiparesia contralateral. Este cuadro requiere ciruga inmediata. Hematoma subdural:
Debidos a roturas de las venas comunicantes entre corteza cerebral y duramadre, incidencia mayor en pacientes etlicos y ancianos. Si aparecen en
las primeras 24 hrs son agudos, entre 24hrs y 2 semanas subagudos y crnicos cuando aparecen ms tardamente. El hematoma subdural agudo
requiere ciruga urgente. Hemorragia subaracnoidea: se acompaa a menudo de hematoma subdural concomitante o de una contusin cerebral, el
diagnostico se realiza mediante TAC y si es normal el diagnostico se realizara mediante la demostracin de un LCR hemorrgico, no requiere tx.,
quirrgico urgente. Hematoma intraparenquimatoso: pueden manifestarse como lesiones rpidamente expansivas o ser asintomticas, en la
mayora de los casos existe fractura craneal asociada por golpe o contragolpe. FRACTURAS CRANEALES: Las fracturas de la bveda craneal no
presentan un cuadro clnico propio, sino que dependen de la afectacin o no de estructuras neurolgicas. Son diagnosticadas con relativa facilidad
mediante la ayuda de los rayos X (en cambio, las fracturas de la base del crneo generalmente no son visibles por estos); poseen un cuadro clnico
propio de cada una de sus variedades, lo que favorece la realizacin del diagnstico clnicamente: Fosa craneal anterior: Hematoma periorbitario uni
o bilateral: en gafas o "en espejuelos" o signo del mapache (si es bilateral, afectando las dos hemifosas anteriores), o del oso Panda (si es
unilateral, que afecta slo una hemifosa anterior). Rinorragia o rinorraquia (uni o bilateral, en dependencia de si afecta una hemifosa anterior, o a
las dos). Es importante establecer diagnstico diferencial, al exponer el goteo en una sbana blanca o un papel, suele separarse el componente
hemtico del lquido cefalorraqudeo, signo del salvavidas o del huevo frito. Puede existir anosmia (por lesin del primer par craneal,
generalmente a nivel de la lmina cribosa del etmoides, menos frecuente por afectacin del bulbo o del nervio olfatorio), alteracin de la motilidad
de la musculatura periocular (pares craneales III-IV-VI), defecto visual por lesin del nervio ptico (II par craneal), anisocoria perifrica, o desigualdad
pupilar con midriasis unilateral, seudo-anisocoria central por lesin de ramos del III par o lesin del esfnter constrictor del iris, o por la presencia
de hematoma intraorbitario extracraneal, que comprima o irrite al ganglio ciliar en el polo posterior del ojo). En resumen, los signos y sntomas
suelen estar ms frecuentemente ocasionados por lesin de los pares craneales I-II-III-IV-VI. Fosa craneal media: Sensacin de "odo ocupado" o
"lquido en el odo", hipoacusia, hemotmpano durante la otoscopia, otorragia u otorraquia, signo de Batlle (hematoma retroauricular, o preesternocleidomastoideo generalmente visible despus de transcurridas las primeras 24 h), vrtigos intensos por lesin de los conductos
semicirculares, paresia o parlisis facial perifrica por lesin del nervio facial en su curso a travs del peasco (ms frecuente en las fracturas
transversas). En resumen, los signos y sntomas suelen estar ms frecuentemente ocasionados por lesin de los pares craneales VII-VIII. Las fracturas
de fosa posterior no tienen un cuadro clnico bien definido, y pueden ser visualizadas mediante rayos X (preferiblemente en la vista de Towne).
CASO CLINICO
Masculino de 35 aos de edad con cuadro de ms o menos una hora
de evolucin sufre trauma en la cabeza al caer de un vehculo en
movimiento con prdida de conocimiento de pocos segundos y
amnesia del hecho, no ha vomitado. Antecedentes patologa
negado, quirrgicos negado, alrgicos negados. A la EF; TA 130/80
FC 90 x FR 25 x T 36.9, escala de coma de Glasgow de 14/15.
Paciente alerta, con aliento alcohlico, se encuentra desorientado
en espacio y tiempo, cabeza presenta hematoma que se extiende de

CURSO ENARM CMN SIGLO XXI TEL: 36246001

regin frontal a regin occipital, cara: presenta excoriacin en


hemicara izquierda, cuello: mvil no dolor, trax: se observa
escoriaciones y equimosis en espalda, entrada y salida de aire
adecuadas, abdomen: normal, extremidades: escoriaciones en codos
miembros inferiores sin alteracin SN: confuso, sensibilidad y
motricidad conservadas.
PREGUNTA
Cul es la conducta ms adecuada a seguir en este momento?

Pharmed Solutions Institute

PGINA 470

MANUAL DE TRABAJO DEL CURSO ENARM CMN SIGLO XXI


RESPUESTA
a.- Analgsico suave, reposo en casa y alta con datos de alarma
b.- ABC, Hospitalizacin y TAC simple
c.- ABC, hospitalizacin 12hrs, alta con datos de alarma
d.- Observacin breve, analgsico y alta con datos de alarma
PREGUNTA
El paciente fue dado de alta y a las 12 hrs regresa por presentar
vomito en proyectil, cefalea intensa y somnolencia. Se le toma TAC
cerebral muestra fractura lineal frontoparietal, pequeo hematoma
epidural frontal de 15mm. Cul es la conducta ms adecuada a
seguir en este momento?
RESPUESTA
a.- Drenaje quirrgico, hemostasia
b.- No quirrgico, hospitalizacin, medidas antiedema
c.- Hospitalizacin y manejo con analgsico y antiemtico
d.- No quirrgico, valorar a las 12 hrs
PREGUNTA
Qu porcentaje del gasto cardiaco recibe el cerebro?
RESPUESTA
a.- 3%
b.- 8%
c.- 15%
d.- 25%
PREGUNTA
Si este paciente presentara anisocoria. Cul es el sitio que ms
probable estara afectado?
RESPUESTA
a.- Mesencfalo
b.- Protuberancia
c.- Compresin de III par craneal
d.- Compresin de VII par craneal
CASO CLINICO
Masculino de un ao dos meses de edad, el cual se encontraba
dormido y mientras lo haca rod sobre su cuerpo hasta caerse de la
cama; al pie de la misma se encontraba un cuchillo de cocina dentro
de un vaso en forma vertical, que sirvi de base para sostener un
raidolito, sufriendo el impacto del cuchillo en la regin parietal
derecha del crneo, ante el grito del nio la madre acude en su
ayuda. Los servicios de urgencias arribaron inmediatamente al
domicilio del paciente y lo trasladaron al Hospital Infantil a su
ingreso a Urgencias se le encuentra consciente, escala de Glasgow
de 14 puntos, reactivo, irritable al manejo y con pre sencia de un
cuchillo enclavado en la regin parietal posterior derecha, con salida
a nivel de parietal supero anterior izquierdo y huellas de sangrado
del trayecto del arma. Fue valorado por neurociruga quien despus
de observar las radiografas, decide someter a ciruga para
extraccin del arma blanca, hasta estos momentos sus constantes
vitales y su estado neurolgico se mantenan estables.
PREGUNTA
De quirfano es egresado a UCIP en donde se mantiene con
asistencia ventilatoria mecnica, le fue aplicado toxoide tetnico, se
hemotransfucion. Cul conducta teraputica es ms adecuada a
seguir?
RESPUESTA
a.- Sedacion
b.- Coma barbitrico
c.- Prevencion de crisis convulsivas con difenilhidantoina
d.- Suministrar altas cantidades de oxigeno

CURSO ENARM CMN SIGLO XXI TEL: 36246001

PREGUNTA
Cul es la conducnta mas adecuada a seguir?
RESPUESTA
a.- Ceftriaxona, ciprofloxacino
b.- Ceftriaxona, amikacina
c.- Ampicilina, gentamicina
d.- Penicilina, vacomicina
PREGUNTA
Cul es la complicacin menos probable en este paciente?
RESPUESTA
a.- Hemtaoma subdural
b.- Edema cerebral
c.- Convulsiones
d.- Absceso cerebral
CASO CLINICO
Masculino de 38 aos de edad con ductor de maquinaria pesada, sin
antecedentes personales patolgi cos de importancia; se encontraba
haciendo obras de reparacin en su trailer,cuando un aumento re
pentino de la velocidad del viento provoc su cada desde una altura
de aproximadamente tres metros, sufriendo un politraumatismo con
especial nfasis en un trauma cra neoenceflico con prdida de con
El paciente fue trasladado de inmediato al centro hospitalario ms
cercano, donde ingres con inmovilizacin cervical, en tabla rgida y
apertura ocular al dolor, respuesta verbal de quejidos sola mente y
respuesta motriz solamente al sentir dolor. Al examen fsico se
evidencia la presencia de un trauma contuso en regin parietal
derecha con la presencia de una fractura deprimida de un espesor
mayor al del grosor del crneo; adems evidencia escoriaciones
mltiples sin importancia y una posible fractura de antebrazo
derecho. El resto de la valoracin osteomuscular no evidencia
mtraumatismos severos.
PREGUNTA
Cul es el Glasgow que presenta el paciente en este momento?
RESPUESTA
a.- 10
b.- 14
c.- 8
d.- 6
PREGUNTA
Cul es la conducta teraputica mas adecuada a seguir?
RESPUESTA
a.- Intubacion y mantener oxigenacin al 100%
b.- Acceso intravenoso
b.- Tomografia computada
c.- Fluidoterapia
PREGUNTA
Dado que el hospital en que se encuentra no cuenta con todos los
servicios, se estabiliza y en el traslado se reporta que elpaciente
convulsiono en dos ocasiones, al llegar a hospital (4 horas despus)
se relaiza la TAC la cual reporta hematoma epidural en regin
parietal derecha. Contina con el mismo Glasgow. Cual es la
conducta mas adecuada a seguir?
RESPUESTA
a.- Coma Barbiturico
b.- Intervencion quirrgica
c.- Medidas de sostn
d.- Suministrar manitol

Pharmed Solutions Institute

PGINA 471

MANUAL DE TRABAJO DEL CURSO ENARM CMN SIGLO XXI


TRAUMA FACIAL
CIENCIAS BASICAS: Trauma se define como el dao que sufren los tejidos y rganos por accin de una energa que puede actuar en forma aguda o
crnica. El trauma maxilofaciales aqul que compromete tanto partes blandas como seas de la regin facial y ocurre en aproximadamente el 30%
de los politraumatizados. Su riesgo es la muerte por asfixia, hemorragias y asociacin de lesiones de columna vertebral y SNC. Las secuelas pueden
ser estticas y funcionales. SALUD PUBLICA: La principal causa de este tipo de pacientes politraumatizados son los accidentes de trfico, en el 40%
de los casos; seguidos por los accidentes domsticos con un 20% de la totalidad; las cadas casuales se presentan en el 14%; los accidentes laborales
suponen un 3% y otras causas el8% restante. Existe un claro predominio por el sexo masculino presentando un porcentaje del 68%, con una edad
media de 32 aos y un rango de edad entre 20 y 45 aos. PATOGENIA: La energa involucrada en un trauma es directamente proporcional a la mitad
de la masa y al cuadrado de la velocidad (E= 1/2Mx V2). Con una masa constante, al doblar la velocidad, la energa liberada es 4 veces ms y cuando
se triplica es 9 veces ms. Segn la energa los traumatismos se clasifican en: baja energa: determina fracturas ms simples, con menor
desplazamiento disyunciones y escaso compromiso de partes blandas. Alta energa: fracturas ms complejas, con mayor desplazamiento, extensin
y conminucion, y con gran compromiso de partes blandas. Las fracturas son el resultado de una sobrecarga mecnica (energa) que una fraccin de
segundo supera la resistencia sea y determina disrupcin. Esta lesin establece la interrupcin del flujo sanguneo tanto en el hueso (cabos de
fractura), como en los tejidos adyacentes. DIAGNOSTICO: La base del diagnstico, en una buena anamnesis y exploracin. Es importante preguntar
sensaciones disestesicas o anestsicas faciales, caractersticas del dolor, alteraciones subjetivas de la oclusin, alteraciones de la visin, obstruccin
nasal, epifora, alteraciones auditivas. Inspeccin; heridas, abrasiones, equimosis, orientan a zona de impacto. Identificar asimetras, falta de fuerza,
deformidades. Palpacin; de ceflico a caudal, terminando con una palpacin de la mandbula y de las regiones de las articulaciones
temporomandiblares. Examen intraoral: hematomas, equimosis, cuerpos extraos, ausencias dentarias, escalones seos y muy importante la
relacin oclusal. Palpacin de columna cervical, y tener siempre oculoscopio nasal. El examen gold standard del trauma facial grave es el scanner, en
especial en los que se sometern a ciruga. La radiologa convencional: 1) Radiografa de Waters: con cabeza hiperextendida, lo que permite
desproyectar la base del crneo del tercio medio facial. til para pirmide nasal, cuerpos cigomticos, maxilares superiores, arcos cigomticos y
rebordes infraorbitarios. Generalmente se completa con hueso propio de la nariz. 2) Radiografa de Towne: til para regin condilea y subcondilea
de la mandbula, como piso de orbita. 3) Radiografa de Hirtz: para arcos cigomticos. 4) craneo lateral y PA. 5) placas de mandbula 6)
ortopantomografia. TERCIO SUPERIOR: Se refiere a las fracturas frontales, nasales, naso orbito etmoidales (NOE), rbita, complejo cigomtico
maxilar, maxilar y paladar. FRACTURAS NASALES: Son las ms frecuentes (40-50%) de las fracturas faciales. La mayora incluye el tercio distal de los
huesos nasales, con el margen etmoidal intacto, generalmente las desviaciones del eje nasal obedecen a desviaciones del septum por fracturas de
este en forma exclusiva o asociadas a fracturas de los huesos propios, el tratamiento de reduccin puede ser diferido hasta 12 dias. FRACTURAS DEL
COMPLEJO NASO-ORBITO-ETMOIDAL: Se localizan en el tercio medio central, con alteracin en la confluencia de los pilares vertical medial y
transversal superior, as como de sus extensiones posteriores a lo largo de la pared medial de la rbita y el piso. Lo que podra corresponder a una
fractura conminuta del pilar vertical maxilar medial, en especial de la fosa lagrimal, donde se inserta el ligamento acantal. TERCIO MEDIO:
FRACTURAS MAXILARES Le Fort I, II, III, NOE, nasales, naso maxilares y aisladas del maxilar. cigoma, arco cigomtico, complejo cigomtico maxilar y
combinadas o especiales (panfaciales). Rene Le Fort clasifico las fracturas maxilares en tres tipos, segn el trayecto de la lnea de Fractura. LE FORT I:
(horizontal) puede resultar de una fuerza dirigida de lesiones bajo en el borde alveolar del maxilar en direccin hacia abajo. La fractura se extiende
desde el tabique nasal hasta el piriforme lateral llantas, viaja horizontalmente por encima de los pices de los dientes, cruza por debajo de la unin
zygomaticomaxillary, y atraviesa la unin pterigomaxilar para interrumpir los platos pterigoideos.; LE FORT II: (Fractura piramidal) La lnea de
fractura se extiende a travs de los huesos propios-nasales y el septum hacia abajo y hacia atrs por la pared medial de la rbita, cruza el reborde
infraorbitario y pasa por el arbotante cigomatico-maxilar. LE FORT III: (Disyuncin craneofacial) Es una verdadera separacin de los huesos de la base
del crneo. El trazo de la fractura pasa por la sutura nasofrontal, por la pared medial de la orbita hasta la fisura orbitaria superior, de esta a la fisura
orbitaria inferior y por la pared-lateral de la rbita hasta la sutura cigomtico-frontal y cigomtico-temporal. Hacia atrs-se fracturan las apfisis
pterigoides del esfenoides, normalmente a un nivel superior al que aparecen en las otras fracturas de Le Fort. Clinica: Hematoma periorbitario
bilateral o en antifaz, epistaxisis, equimosis conjuntival. Aumento de la longitud del tercio medio facial (cara larga, cara de plato). Mala oclusin.
Mordida abierta anterior: contacto prematuro de los molares respecto al segmento anterior. Vigilar rinolicuorrea y otorrea. Tratamiento: Cobertura
antibitica y analgsica. Vigilancia de la va area, control de la hemorragia y remitir a centro especializado. Si existe rinolicuorrea no realizar
taponamiento nasal y prescribir tratamiento antibitico. FRACTURAS CIGOMATICO-MAXILARES: Caracterizado por la presencia de hematoma,
equimosis, y depresin de la eminencia malar, asociado o no a limitacin de la apertura oral. FRACTURAS ORBITARIAS: Estas pueden ser de
cualquiera de sus cuatro paredes, siendo ms frecuente el piso y las paredes lateral (en fracturas malares) y medial, lo mas importante es efectuar
estudio del globo ocular, para descratar heridas que puedan peligrar la visin. En el segmento posterior de la rbita, se unen el piso con la pared
medial, formando lo que se conoce como la zona clave, la cual se ve frecuentemente fracturada, su diagnstico es por scanner, y es la causa ms
frecuente de enoftalmo post traumtico, de tal manera que se debe buscar en forma dirigida. TERCIO INFERIOR: FRACTURAS DE MANDIBULA: Muy
frecuentes, el paciente generalmente presenta impotencia funcional mandibular y/o alteraciones de la oclusin. Es un hueso de consistencia dura
que presenta tres zonas dbiles que son el cuello del cndilo mandibular, la zona canina-agujero mentoniano y el ngulo mandibular debido a la
confluencia de las ramas horizontal y vertical y la presencia de la muela del juicio. Es ms frecuente en varones jvenes entre 20 y 30 aos. La zona
de fractura ms frecuente es el cndilo mandibular, seguido del ngulo mandibular y la regin parasinfiaria. Es aconsejable la valoracin en conjunto
con el odontlogo, por posibles fracturas dentoalveolares asociadas. Clinica: Sntomas: dolor, tumefaccin, disconfort, hematoma, deformidad,
movilidad anormal, crepitacin, salivacin, mal olor, mal oclusin y trismus Signos: mala oclusin, movilidad del foco de fractura, disfuncin a la
deglucin y masticacin, crepitacin y tumefaccin. Tratamiento: Preservar va area, control de la hemorragia, sutura de heridas, limpieza de la
cavidad oral, remitir a centro especializado, cobertura antibitica y analgsica. TRATAMIENTO: Debe ser efectuado idealmente una vez que el
paciente se haya estabilizado y se haya realizado el estudio radiolgico adecuado, la ciruga facial se puede retrasar hasta 10 das. Indicaciones de
traqueostomia: fracturas panfaciales. Fractura conminuta de la mandbula. Edema masivo de tejidos blandos faciales (quemadura). Las hemorragias
de partes blandas, se recomienda compresin, ya que los pinzamientos a ciegas pueden producir graves daos a estructuras nerviosas. En el caso de
epistaxis, el taponamiento nasal anterior continua vigente. El taponamiento posterior puede efectuarse con gasas lo ms adecuado o con sonda
Foley, en caso de persistir ligadura arterial selectiva. Como norma general el paciente traumatizado facial asocia heridas de las partes blandas de la
cara, heridas que son complejas por su mecanismo de accin, su tratamiento y su repercusin esttica posterior. Las heridas: en primer lugar, deben
de ser lavadas profusamente con suero salino; un cuidadoso y exhaustivo examen para intentar eliminar cuerpos extraos en su totalidad, bajo
anestesia local las manchas de los bordes de las heridas tienen que ser eliminados para evitar la aparicin del tatuaje postraumtico cepillado
enrgico de los bordes de la herida. Posteriormente se procede a eliminar aquellos fragmentos de tejido necrtico que puedan dificultar una
correcta cicatrizacin, valorar lesiones de tipo nerviosos por seccin de ramas motoras o sensitivas y lesin de conductos lacrimales y salivares y

CURSO ENARM CMN SIGLO XXI TEL: 36246001

Pharmed Solutions Institute

PGINA 472

MANUAL DE TRABAJO DEL CURSO ENARM CMN SIGLO XXI


finalmente se realiza hemostasia y sutura de las mismas. Profilaxis antitetnica y cobertura antibitica de amplio espectro. Analgsicos. Dada la
riqueza vascular presente en el macizo facial la sutura de las heridas en este territorio puede demorarse de 6 a 24 horas.
CASO CLINICO
Se trata de paciente masculino 36 aos, el cual sufri accidente al ir
en su bicicleta transitando por una calle inclinada, perdi el control
impactndose contra el pavimento, con prdida del estado de alerta,
por lo cual es llevado en ambulancia al rea de urgencias. A la
exploracin clnica se observ: T/ A 130/80mmHg, FC 90x, FR 24x,
sat 89%, ECG 14, gran edema en cara y equimosis periorbitaria
bilateral que impide separar los prpados para explorar el globo
ocular, hipertelorismo, hay separacin de los huesos de la base del
crneo, hay descenso del maxilar y del paladar blando (fascies en
cara de plato).

espontneamente al llegar al rea de urgencias. En la valoracin


inicial por el Servicio de Ciruga Maxilo facial se observ edema
importante de la regin facial, dermoabrasiones en hemicara
derecha, depresin y movilidad de la pirmide nasal, equimosis
periorbitaria del lado derecho (sin compromiso ocular aparente),
crepitacin en ambos rebordes orbitarios, limitacin a la apertura
bucal y movilidad de todo el segmento maxilar, con trazo sagital en
paladar duro; no se apreci rinorrea ni otorrea. En la exploracin
intraoral se encontr laceracin de mucosa del labio superior que
llegaba al plano muscular, y laceracin de mucosa de paladar duro
que respetaba paladar blando, sin compromiso de va area.

PREGUNTA
Cul es la conducta teraputica ms adecuada a seguir en este
momento?
RESPUESTA
a.- Ciruga
b.- Iniciar cristaloides
c.- Intubacin orotraqueal
d.- Valoracin por neurologa

PREGUNTA
Se decidi realizar tomografa computarizada con ventana sea y
reconstruccin del maxilar y rebordes orbitatarios, para establecer
con precisin el grado de lesin. Se identific fractura
nasoorbitoetmoidal grado III, piso de rbitas, fractura de Le Fort I y
lnea media del paladar. A cul de las siguientes corresponde una
fractura nasoorbitofrontal grado III?
RESPUESTA
a.- Segmento central nico, sin compromiso del canto interno
b.- Segmento central conminuto, sin compromiso del canto interno
c.- Segmento central conminuto, con desinsercin del canto interno
d.- Fractura horizontal sobre lnea alveolar superior

PREGUNTA
Cul es la conducta diagnostica ms adecuada a seguir en este
momento?
RESPUESTA
a.- Watters y Rx. panoramica
b.- Rx. Lateral y anteroposterior de crneo
c.- Hirtz y Cadwell-Towne
d.- Watters y Cadwell- Towne
PREGUNTA
Qu tipo de fractura facial es ms probable que presente este
paciente?
RESPUESTA
a.- Le Fort II
b.- Fractura naso-orbito-etmoidal
c.- Le Fort III
d.- Fractura de mandbula
CASO CLINICO
Masculino de 24 aos de edad, con traumatismo facial por golpes
con un tubo de metal, que le ocasionaron prdida de la conciencia
de aproximadamente 90 minutos (segn lo referido por el familiar
que lo acompaaba) y hemorragia profusa, que cedi

CURSO ENARM CMN SIGLO XXI TEL: 36246001

PREGUNTA
Cul de las siguientes caractersticas es menos probable que se
presente en una fractura, Le Fort I?
RESPUESTA
a.- Fractura horizontal sobre lnea alveolar superior
b.- Movilidad de toda la porcin dento-alveolar del maxilar
c.- Movilidad patolgica de huesos propios nariz
d.- Equimosis vestbulo-palatina en herradura; gran edema en el
labio superior
PREGUNTA
Cul signo es ms probable encontrar en la fractura facial de base
de crneo de piso anterior?
RESPUESTA
a.- Signo de Battle
b.- Signos de ojos de mapache
c.- Signo de la gota
d.- Signo de la mordida anterior

Pharmed Solutions Institute

PGINA 473

MANUAL DE TRABAJO DEL CURSO ENARM CMN SIGLO XXI


TRAUMA RAQUIMEDULAR
CIENCIAS BASICAS: Toda lesin traumtica que afecta, conjunta o aisladamente, las diferentes estructuras de la columna vertebral en cualquier de
sus niveles. Es toda lesin de la columna vertebral asociada a una lesin neurolgica que puede comprometer a la medula espinal, races o cauda
equina. Esta lesin neurolgica puede ser reversible o irreversible. Factores predisponentes: Espondilosis cervical, inestabilidad atloaxoidea,
malformaciones vertebrales congnitas, osteoporosis, artropatas inflamatorias. Recuerdo anatmico: 33 vrtebras: 7 cervicales, 12 torcicas, 5
lumbares, 5 sacras, 4 coccgeas, 23 discos intervertebrales. Trauma vertebromedular: Transmisin abrupta de energa sobre la columna vertebral y
su contenido. Lesin medular: Alteracin temporal o permanente de la funcin motora, sensitiva o autonmica. SALUD PBLICA: La enfermedad
traumtica es la 4 causa de muerte. 20-30 casos/100.000 habitantes/ao. Mortalidad global 4%. Los traumatismos ocasionan severas incapacidades
y secuelas invalidantes permanentes. Ms 1/3 accidentes de trfico. Ms de 1/3 politrauma: 5-15% dao neurolgico, 30% otras lesiones, 5%
fracturas ocultas. Distribucin anatmica: 55% cervical, 10% torcica, 15-20% toracolumbar, 15-20% lumbosacra. 5% de los pacientes con TCE,
tienen TRM. 25% de TRM tienen TCE. PATOGENIA: La zonas ms vulnerables son la columna cervical baja y la columna toracolumbar, dos regiones
ms predispuestas en el sentido que son zonas de mayor movilidad en movimientos de flexoextensin y rotaciones, por lo tanto son las zonas donde
actan los vectores luego de la aplicacin de alguna fuerza que daa las estructuras ya sean seas o discoligamentarias de la columna vertebral.
Entre las causas ms frecuentes est el trauma automovilstico, generalmente asociado al alcohol. Estos dos factores estn presentes en por lo
menos 50% de los casos de trauma espinal. Otras causas son los accidentes industriales, deportivos y el trauma como resultado de la inseguridad:
heridas cortopunzantes y heridas por arma de fuego. Los mecanismos de lesin pueden ser directos o indirectos, los directos son los menos
frecuentes ocurren debido a la aplicacin de energa en el mismo punto donde se produce la lesin de columna vertebral. Los mecanismos
indirectos, los ms frecuentes, producen lesin no necesariamente en el sitio de aplicacin de la energa. Existen 4 mecanismos principales de
trauma espinal: 1. Extensin, 2. Flexin, 3. Rotacin, 4. Compresin vertical o carga axial. Estos son movimientos tolerados normalmente por la
columna y las estructuras nerviosas espinales siempre y cuando se mantengan dentro de rangos fisiolgicos. De lo contrario se presenta disrupcin
anatmica y lesin estructural del continente (columna) y contenido (medula-races nerviosas). Con frecuencia en una sola lesin espinal intervienen
dos o ms de los cuatro mecanismos. El trauma espinal producido por flexin forzad, la posibilidad de lesiona es causa frecuente de lesin sea,
ligamentaria y de medula y races nerviosas. La flexin forzada asociada o no a carga vertical (axial) se presenta por ejemplo en casos de clavados
en piscina o ros pandos. La hiperextensin puede producir lesin espinal, en esta se produce disminucin del espacio intraespinal. En pacientes de
edad avanzada en los cuales por cambios degenerativos el espacio interior del canal espinal ya esta disminuido, la posibilidad de lesin de las
estructuras contenidas dentro del canal espinal aumenta. Desde el punto de vista fisopatologico hay eventos despus del trauma espinal que llevan
finalmente a la desintegracin de la membrana celular, es decir, a la peroxidacin lipdica de la membrana celular. Algunos de estos eventos son la
entrada de calcio al espacio intracelular, el aumento de radicales libres, ac. Araquidnico, prostaglandinas y tromboxano, la liberacin de endorfinas
y aumento de norepinefrina. La peroxidacin lipdica lleva a desctruccion celular irreversible. DIAGNOSTICO: Sospechar traumatismo raquimedular
en todo paciente politraumatizado, inconsciente o intoxicado. El examen neurolgico es de gran importancia para determinar tanto en el paciente
consciente como en el inconsciente, si existe compromiso neurolgico de medula, de races nerviosas o de ambas, como resultado de trauma
espinal; se debe valorar la fuerza, sensibilidad, reflejos, compromiso de esfnteres y deformidad espinal. Las lesiones medulares pueden ser
completas o incompletas. Indicios de trauma de columna: Dolor, hematomas, prdida del patrn respiratorio, hipotensin y bradicardia, globo
vesical, hipotona de esfnter anal, priapismo, debilidad, hipoestesia. Se pueden manifestar como sndrome medular anterior, central, posterior o de
hemiseccin medular. La seccin medular completa se caracteriza por prdida total de movimientos, sensibilidad, reflejos y control de esfnteres. En
su grado ms extremo, el trauma espinal se asocia a seccin anatmica o fisiolgica de la medula espinal, con cuadro clnico como shock espinal. En
el examen se encuentra parlisis (paraplejia-cuadriplejia), nivel de anestesia, arreflexia, ausencia de funcin esfinteriana y sock neurognico
producido por desconexin del sistema simptico (simpatectoma) y que se caracteriza por hipotensin arterial y bradicardia. Hacen parte de la
lesin simptica vasodilatacin, perdida de calor por esta razn e hipotermia. Una vez estabilizado el paciente desde el punto de vista
hemodinmico y respiratorio, el primer estudio que se debe realizar en el paciente politraumatizado es la radiografa lateral de columna cervical.
Esta determinara la presencia o no de lesiones traumticas de columna cervical en 90% de los casos y permitir establecer si se puede o no
inmovilizar. Las radiografas AP y transoral, pueden complementar, el estudio imagenolgico inicial. Opcionalmente en el paciente TCE, que se tome
TAC cerebral, se puede aprovechar este estudio para realizar de forma complementaria cortes de TAC de las 3 primeras vertebras cervicales. La RMI,
es un excelente mtodo diagnstico para lesiones espinales traumticas, puede mostrar contusin medular, edema o seccin medular, una zona de
hiperintensidad medular generalmente implica la presencia de sangre dentro de la medula (hematomielia). De igual manera puede demostrar la
presencia descompresin de medula y races por disco, hematoma epidural, fragmentos de vertebra. TRATAMIENTO: Se debe seguir un protocolo
secuencial: 1. Inmovilizacin desde el sitio y momento del accidente. 2. Diagnstico clnico e imagenolgico. 3 intervencin farmacolgica para
disminuir la severidad de la lesin inicial y las secuelas neurolgicas 4. Reduccin de luxaciones, descompresin de tejido nervioso si hay fragmentos
de hueso, disco, ligamento, sobre races o medula espinal. 5. Fijacin espinal en caso de inestabilidad. 6. Rehabilitacin del paciente. Las lesiones
traumticas de columna que produzcan desplazamiento anormal de sus elementos requieren de reduccin. La inmovilizacin y reduccin de lesiones
espinales cervicales puede hacerse mediante la colocacin de halo craneano que tracciona el crneo e indirectamente la columna cervical, alineando
las vrtebras. En el trauma medular se debe ante todo evitar el dao secundario, el cual puede estar causado por hipotensin arterial, bradicardia,
hipoxemia, hipercarbia, acidosis y aumento de la viscosidad sangunea. Se han empleado compuestos para disminuir o anular la peroxidacin
lipdica. La metilprednisolona (30 mg/kg en bolo, 5.4 mg/kg/h por 23 hrs) es la sustancia ms aceptada hasta el momento ya que tiene efectos
inversos, disminuye la entrada de calcio a la clula, aumenta el flujo sanguneo e inhibe a la PG 2 alfa y el tromboxano. El estudio NASCIS II demostr
que la administracin de esta droga logra una mejor recuperacin funcional motora y sensitiva. Su administracin constituye la base de la
intervencin farmacolgica en los pacientes con trauma espinal. Conservador: Traccin, inmovilizacin, rehabilitacin. Quirrgico: Remocin de
fluidos o tejidos que comprimen (medula o racices), fusin sea, inestabilidad (que requiere estabilizacin mediante fijacin o artrodesis),
compresin medular. NEURONA MOTORA SUPERIOR: Parlisis, hipertona (espasticidad), hiperrreflexia, babinski. NEURONA MOTORA INFERIOR:
Parlisis, hipotona, hiporrreflexia, atrofia. SINDROME MEDULAR ANTERIOR: Se caracteriza principalmente por lesin motora asociada a signos
piramidales, con conservacin de diferente magnitud de la sensibilidad del paciente. SINDROME MEDULAR POSTERIOR: Se manifiesta clnicamente
por una mayor lesin de tipo sensitiva y de diferentes grados de conservacin de la funcin motora. SINDROME DE BROWN-SEQUARD:
Caracterizado por la lesin anatmica de la mitad de la medula espinal, se manifiesta clnicamente por pedida de la funcin motora ipsilateral y
perdida de la funcin sensitiva contralateral (dolor, termoalgesia). SINDROME DE SECCION MEDULAR TRANSVERSA: Se caracteriza clnicamente por
presentar una prdida total de la funcin sensitiva motora y de los reflejos desde el nivel de la lesin hacia distal (no necesariamente se asocia a una
seccin medular anatmica). La lesin de las races generalmente se produce por una traccin por los diferentes mecanismos de lesin o por un
desplazamiento de las vrtebras que produce un estrechamiento del dimetro de los orificios de conjuncin con lo cual se produce una compresin
de las races en diferentes magnitudes. La lesin de la cauda equina se puede producir en forma total o parcial en relacin con los desplazamientos

CURSO ENARM CMN SIGLO XXI TEL: 36246001

Pharmed Solutions Institute

PGINA 474

MANUAL DE TRABAJO DEL CURSO ENARM CMN SIGLO XXI


anteroposteriores de la columna vertebral lumbar desde el nivel L2 hacia distal; tambin puede lesionarse la cauda equina por protrusin hacia
posterior y ocupamiento del canal raqudeo de fragmentos seos o fragmentos discales. SNDROME CENTROMEDULAR DE SCHNEIDER: Paresia
flcida en miembros superiores (afectacin de 2 neurona), disminucin termoalgsica en miembros superiores, dolores por desaferentizacin en
miembros superiores, sin afectacin de miembros inferiores o menor que en extremidades superiores. Esta paresia es espstica (afectacin 1
neurona). MEDULAR CENTRAL (SIRINGOMIELIA): bilateral, sensibilidad termoalgsica segmentaria, disociacin termoalgsica. Exploracin clnica:
C5 flexin de codo (bceps), C6 extensin mueca (ECRB, ECRL), C7 extensin codo (trceps), C8 flexor dedo medio (FDP), T1 abduccin meique, L2
flexin cadera (psoas iliaco), L3-L4 extensin rodilla (cudriceps), L4 dorsiflexin tobillo, L5 dorsiflexin hallux, S1 flexin plantar (hallux, gemelos,
sleo).
CASO CLINICO
Paciente masculino, de 47 aos de edad, sufre cada de ms o menos
dos metros de altura en vertical, con prdida momentnea de
conocimiento segn refieren sus familiares. Es trasladado por la
ambulancia al servicio de urgencias En el informe inicial de los
paramdicos y con el cual se present el paciente en el hospital dice
lo siguiente: paciente encontrado consciente en estado confuso,
escala de Glasgow 13/15, TA 85/56 mmHg, FR: 14 x, dficit motor
en miembros inferiores, con mltiples laceraciones en diferentes
partes del cuerpo, abdomen edematizado y doloroso, quejumbroso,
manifiesta incapacidad para movilizar los miembros; es movilizado
en bloque y con cuello Filadelfia. Al ingreso a urgencias permanece
inmvil, se observa ansioso, irritable, manifiesta preocupacin por
no poder mover los miembros inferiores, ha presentado retencin
urinaria con globo vesical doloroso, por orden se realiza cateterismo
vesical permanente con sonda Foley.
PREGUNTA
Cul es el diagnstico ms probable para este caso?
RESPUESTA
a.- Hematoma subdural
b.- Isquemia cerebral
c.- Lesin medular
d.- TCE grave

PREGUNTA
Cul de las siguientes alteraciones es menos probable que se
presente en lesin de neurona motora inferior?
RESPUESTA
a.- Parlisis
b.- Hipotona
c.- Babinski
d.- Atrofia
PREGUNTA
A qu nivel es ms probable que encontremos el cono medular?
RESPUESTA
a.- L3-L4
b.- L1-L2
c.- L3-L4
d.- L4-S1
CASO CLINICO
Un varn de 18 aos, sin antecedente importante, es agredido con
un picahielo, en la regin torcica y cervical, produciendo dolor
cervical y disminucin de fuerza de hemicuerpo derecho. Al ingreso
a urgencias, el paciente se encontraba alerta, orientado, con
lenguaje fluido, funciones mentales superiores conservadas. Sin
compromiso de pares craneanos. Al examen del sistema motor se
evidenci una hemiparesia derecha predominio crural 1/5 con
alteracin de la sensibilidad profunda y vibratoria derecha y una
sensacin termoalgsica disminuida izquierda, con un nivel sensitivo
C7-D1, adems de un globo vesical y una herida penetrante en
regin paravertebral cervical derecha.
PREGUNTA
Cul es el diagnstico ms probable para este caso?
RESPUESTA

CURSO ENARM CMN SIGLO XXI TEL: 36246001

a.- Sndrome espinal anterior


b.- Sndrome de Brown-Sequard
c.- Siringomelia
d.- Sndrome centromedular
PREGUNTA
Cules son los fragmentos que ms probablemente se daaron?
RESPUESTA
a.- Tracto corticoespinal y tracto espinotalmico derecho
b.- Tracto corticoespinal y tracto espinotalmico izquierdo
c.- Tracto corticoespinal derecho, tracto espinotalmico izquierdo
d.- Tracto corticoespinal izquierdo, tracto espinotalmico derecho
PREGUNTA
Qu es menos probable que conduzca el haz espinotalmico?
RESPUESTA
a.- Temperatura
b.- Vibracin
c.- Motor
d.- Dolor
CASO CLINICO
Femenino de 55 aos de edad con antecedentes de diabetes
mellitus no insulino dependiente de aos de evolucin e
hipertensin arterial de 6 meses la cual posterior a sufrir accidente
automovilstico, presento dolor torcico intenso, al cual se le aade
disminucin importante de la fuerza en extremidades inferiores que
le imposibilit la deambulacin a las 6 horas de iniciado el cuadro;
prdida de control de esfnteres. A la EF: Miembros inferiores
hipotonicos y arreflcticos, prdida de sensibilidad a unos dos dedos
por encima de la regin umbilical, hacia abajo. Se conserva el
sentido de propiocepcin y la vibracin.
PREGUNTA
Cul es el diagnstico ms probable en este caso?
RESPUESTA
a.- Sndrome de cordn posterior
b.- Sndrome de cordn anterior
c.- Hemiseccin
d.- Sndrome centromedular
PREGUNTA
De acuerdo al cuadro clnico. A qu nivel es ms probable que se
presente la lesin en este paciente?
RESPUESTA
a.- T 8-T 9
b.- T 12-L 1
c.- T 5-T 6
d.- L 2-L 3
PREGUNTA
Cul es la causa ms probable por la que se conserva la
propiocepcin y la vibracin?
RESPUESTA
a.- Porque el haz espinotalmico no se lesiona
b.- Porque el haz corticoespinal no se lesiona
c.- Porque el cordn posterior no se lesiona
d.- Porque no se lesionan las neuronas motoras

Pharmed Solutions Institute

PGINA 475

MANUAL DE TRABAJO DEL CURSO ENARM CMN SIGLO XXI


TRAUMA TORACICO CERRADO Y ABIERTO:
CIENCIAS BASICAS: Es cualquier agresin o trauma sobre las paredes del trax que producir un dao en las estructuras slidas y partes blandas
comprendidas en la caja torcica. Los traumatismos torcicos pueden ser cerrados (contusos) o abiertos (penetrantes). Traumatismo abierto: se
denomina a lesin que rompe la integridad del tejido (atraviesa pleura parietal). Traumatismo cerrado: resulta por aplicacin de energa que provoca
lesin sobre los tejidos sin daar su integridad. SALUD PUBLICA: En Mxico los traumas cerrados son mayoritariamente por accidente de trnsito,
los penetrantes son por arma blanca, aunque han aumentado las heridas por
TRAUMA TORACICO CERRADO
TRAUMA TORACICO ABIERTO
Asociada a compresin y aceleracinAsociada a heridas por arma blanca
armas de fuego. 8 de cada 100.000 son letales. Principales causas de
desaceleracin
y arma de fuego
traumatismo torcico asociadas: Accidentes de trnsito (43%), Suicidios (29%),
Hay fracturas costales mltiples
Puede o no haber fracturas costales
Homicidios (22%.). CLASIFICACION: Segn el estado hemodinmico: Normales o
Puede haber hemo o neumotrax
Hemo o neumotrax inmediato.
compensados hemodinmicamente Presin Arterial: 90/60 , Frecuencia
tardo (>24hrs del trauma)
Cardiaca: 110 , diuresis satisfactoria. Anormales o descompensados
El trauma de va area superior se
El trauma de va area cursa con
hemodinamicamente Presin Arterial: 90/60 o , Frecuencia Cardaca: 120 o ,
manifiesta como estenosis
gran escape areo
Tratamiento quirrgico: requerido en
Tratamiento quirrgico: requeridos
oligoanuria o anuria. Ambos pueden ser estables (se mantienen los parmetros
menos del 10% de lesionados
entre 15-30% de lesionados.
con el correr del tiempo) o inestables. Segn grado de penetracin torcica:
Herida de arma blanca: Punzantes,
Accin directa ( golpe o choque
Grado 1: No comprometen pleura. Grado 2: Penetran pleura parietal, penetran
cortantes
directo)
pleura parietal y parnquima pulmonar. PERFORANTES O TRANSFIXIANTES
Heridas de proyectil de arma de
Accin indirecta: Compresin,
fuego: De baja velocidad (< 750 m/s)
alteracin
de
la
velocidad,
(Entran y salen del trax y comprometen otras cavidades). Grado 3: Penetran
De alta velocidad (> 751 m/s)
Aceleracin brusca, Desaceleracin
pleura, pulmn, mediastino o abdomen o regin cervical (Dos cavidades). Grado
Miscelneas ( Otros elementos lesivos
brusca, Torsin, Deslizamiento,
4: Penetran pleura, pulmn, mediastino y el otro hemitorax o abdomen o regin
internos o externos)
Inmersin
cervical (tres cavidades o regiones) Nos da gravedad en un Trauma torcico:
Impactos de alta energa: cada mayor a 6 metros. Impactos de alta velocidad. Pasajeros despedidos del vehculo. Atropello. Lesin penetrante de
cabeza, cuello, trax, abdomen o regin inguinal. Dos o ms fracturas proximales de huesos largos. Quemaduras mayores al 15% de SC o que
afecten cara o vas areas. Trax inestable. TIPOS DE TRAUMA TORCICO: Seis lesiones rpidamente fatales (Se detectan en evaluacin primaria):
Obstruccin de la va area. Neumotrax Hipertensivo. Neumotrax Abierto. Taponamiento cardaco. Hemotrax masivo. Trax inestable. Seis
lesiones potencialmente letales u ocultas (Detectadas en la evaluacin secundaria): Lesiones traqueobonquiales. Ruptura diafragmtica. Lesin
esofgica. Contusin pulmonar. Ruptura artica. Contusin miocrdica. Lesiones no necesariamente letales: Neumotrax simple, evidente u oculto.
Hemotorax simple. Enfisema. Fracturas costales. Fractura de Esternn o Escpula. 1.-OBSTRUCCIN DE LA VA AREA; se puede producir por
cuerpos extraos, secreciones, sangre, el manejo puede ser invasivo o no invasivo, otra clasificacin es va area quirrgica y no quirrgica.
Consisten en el manejo manual de la va area, intubacin oro o nasotraqueal, cricotiroidotomia por puncin o quirrgica y la traqueotoma. 2.
NEUMOTORAX HIPERTENSIVO; Ocurre cuando una lesin permite la entrada de aire al espacio pleural durante la inspiracin, sin que pueda salir
durante la espiracin. Esto lleva al colapso pulmonar, con desplazamiento mediastinal hacia el lado opuesto, se disminuye el retorno venoso por
angulacin de las cavas, compromete el gasto cardaco y el otro pulmn.El diagnstico es clnico: taquipnea, dificultad respiratoria aguda, tiraje
supraclavicular e intercostal, hipersonoridad y ausencia de murmullo vesicular, distencin de venas del cuello, desviacin de la trquea cianosis.
Tratamiento, descomprimir inmediatamente, con puncin catter teflonado calibre 12 o 14, en 2 espacio intercostal lnea medio clavicular, con lo
cual lo convierte en normotensivo. Proceder a la colocacin de avenamiento pleural bajo agua en 4 espacio intercostal lnea medio clavicular. 3.NEUMOTRAX A TENSIN; es el escape de aire hacia la cavidad pleural, mas comn neumotrax espontneos, el mediastino y la trquea se
desplazan hacia al lado opuesto, comprometiendo la posibilidad de respuesta ventilatoria por parte del pulmn sano, y afectando el retorno venos.
Clnicamente se manifiesta por dificultad respiratoria, taquicardia, hipotensin, desviacin de la trquea, ausencia unilateral de MV, timpanismo del
pulmn afectado, ingurgitacin yugular y cianosis tarda. El tratamiento inicial consiste en insertar una aguja, en 2 EIC en LMC del hemitrax
afectado, siempre por el borde superior de la 3 costilla. El definitivo es la insercin de un tubo de trax en el 5 EIC anterior a la LAM, siempre por el
reborde costal superior de la 6 costilla, en el hemitrax afectado. 4.-NEUMOTRAX ABIERTO; Al producirse una herida en la pared torcica el aire
penetrar preferentemente por la herida al igualarse las presiones intratorcica y ambiental y ofrecer menor resistencia al paso del aire por la
herida. El tratamiento inicial ser cubrir la herida con un apsito fijado en tres puntas. El tratamiento definitivo consiste en la instalacin de un
catter intercostal y conectarse a un equipo de succion a 10-25 cmH2O. Si el neumotrax es estable puede mantenberse bajo vigilancia y se
reabsorber a un ritmo de 1.25% por dia. , distante de la lesin, y el cierre de la herida ser quirrgico. Complicaciones: Inadecuada conexin
drenaje-aspirador. Inadecuada colocacin del drenaje. Oclusin bronquial. (Cuerpo extrao, cogulo, rotura). Roturas traqueobronquiales. Grandes
laceraciones pulmonares. Severa disminucin de la distensibilidad pulmonar. 5.- HEMOTRAX MASIVO: Es el resultado de la acumulacin de sangre
en la cavidad pleural, igual o superior a 1500 ml o mas de 200ml/h durante 4 hrs. La principal causa: es la lesin de vasos hiliares y mediastinicos
generalmente por heridas penetrantes. Clnicamente encontrar un paciente en shock, con colapso de los vasos del cuello por hipovolemia o con
ingurgitacin de estos por efecto mecnico de las cavidades. El tratamiento ser simultneamente con reposicin de volumen de forma agresiva
(cristaloides, coloides y sangre), y descompresin del hemitrax lesionado con un tubo de trax nico N 28-32 f. Se har toracotoma s: Deterioro
hemodinmico sin otra justificacin. Persiste un drenaje superior a 1500 ml en las primeras 12-24 horas. Drenaje superior a 200 ml/hora en 4 horas.
Persistencia de la ocupacin torcica (hemotrax coagulado). 6.- TAPONAMIENTO CARDIACO: Producto de una herida penetrante, en su gran
mayora, pero tambin puede aparecer por lesiones de los vasos pericardicos o traumatismo cardaco en un traumatismo cerrado. Desde el punto de
vista clnico se manifiesta por la trada de Bec , que consiste en el hallazgo de: aumento de la presin venosa central, disminucin de la presin
arterial, apagamiento de los ruidos cardiacos. Ingurgitacin yugular con la inspiracin en un paciente ventilando espontneamente es signo
inequvoco de taponamiento cardiaco (signo de Kussmaul). En el tratamiento, la pericardiocentesis por va subxifoidea es de eleccin en el
prehospitalario, para descompresionar el pericardio, basta extraer 15-20 ml, pero es una medida temporal, actualmente se postula que en l SU y
Hospital no deben hacerse pericardiocentesis a menos que sea para dar tiempo a la preparacin del pabelln. 7.- TRAX INESTABLE: El trax se
torna inestable cuando un traumatismo contuso produce fractura en 2 puntos de 4 o ms costillas. El paciente es incapaz de generar la presin
negativa suficiente para manetener la ventilacin; la intubacin y la ventilacin con presin positiva son imperativos. Clnicamente puede no ser
detectado en primera instancia por la hipoventilacin reactiva al dolor, y por los movimientos del trax. El tratamiento se basa fundamentalmente
en una buena ventilacin, eventualmente mecnica, oxigenacin, y tratamiento para el dolor. Si el paciente no est en shock la infusin de fluidos
debe ser cuidadosa para evitar la sobrehidratacin y el consiguiente edema pulmonar. Un adecuado y controlado balance hdrico. Cuando coexisten
fracturas costales mltiples en varias costillas consecutivas se produce una inestabilidad de la pared con movimiento paradojico y alteracin de la
mecnica respiratoria, con la consiguiente hipoxia. La gravedad de la lesin es directamente proporcional al grado de alteracin del parnquima
pulmonar en combinacin con el dao de la pared. 8.- CONTUCION PULMONAR: Lesin del parnquima pulmonar que causa hemorragia y edema
localizado, producto de traumas en los que hay rpida compresin y descompresin del trax. Si se afecta un rea considerable es probable que se

CURSO ENARM CMN SIGLO XXI TEL: 36246001

Pharmed Solutions Institute

PGINA 476

MANUAL DE TRABAJO DEL CURSO ENARM CMN SIGLO XXI


necesite asistencia mecnica, pero casi nunca durante ms de 48-72 hrs. Se observa falla respiratoria tarda, lenta, progresiva y sutil, con o sin trax
inestable.9.- CONTUCION MIOCRDICA: Difcil de diagnosticar, se sospecha por alteraciones al ECG (arritmias, extrasstoles mono o bifocales,
taquicardia sinusal inexplicable, FA, bloqueo de rama, o claramente un infarto), Eco cardiografa bidimensional e historia compatible. El tratamiento
tratara la manifestacin clnica o la arritmia especfica. 10.- RUPTURA DE AORTA: Producto de traumatismos cerrados, por laceracin o
arrancamiento de los puntos de fijacin de la Aorta. Signos clnicos alertan el diagnstico: Mediastino ensanchado en Rx de Tx. Fx de 1 y 2
costillas. Desviacin y elevacin del bronquio principal, de la trquea y el esfago hacia la derecha. Depresin del bronquio principal izquierdo.
Opacidad pleural apical. Tratamiento: quirrgico (reparacin o implante). 11.- RUPTURA DIAFRAGMTICA: Presente con ms frecuencia en el lado
izquierdo ya que se carece de la proteccin del hgado. Se sospecha el diagnstico por la presencia de intestino, estomago en el hemitrax
izquierdo. Los traumatismos penetrantes por arma blanca o de fuego a veces pasan inadvertidos y solo se detectan aos despus cuando aparece la
hernia diafragmtica. 12.- LESIN TRAQUEOBRONQUIAL: Laringe Diagnstico: ronquera, enfisema subcutneo y crepitacin palpable de fractura.
Manejo, en caso de va area obstruida: instalacin de IOT o Traqueotoma. Trquea: las lesiones penetrantes son ms obvias que las provocadas
por trauma. Se asocia a lesin de esfago y grandes vasos. Diagnostico a travs de broncoscopa. Bronquios: La lesin de un bronquio mayor es rara
y mortal, y ocurren a 2-3 cm. de la carina. Diagnstico: hemoptisis y enfisema subcutneo, y se sospecha en neumotrax a tensin con gran escape
de aire y es confirmado por broncoscopa. Una vez confirmado el diagnstico de lesin traqueobronquial el tratamiento es la reparacin
quirrgica. 13.- RUPTURA ESOFGICA: Se sospechara cuando exista neumo o hemotrax a izquierda sin fracturas costales, trauma directo al
esternn o epigastrio con dolor y shock no explicado, adems puede haber salida de partculas de contenido digestivo por tubo de trax.
Tratamiento: La conducta ser la reparacin directa, esofagostoma cervical de escape y yeyunostoma de alimentacin. 14. FRACTURAS
ESTERNALES: Suelen ser transeversas, se encuentran en el manubrio o cerca de l y son dolorosas. Resulta esesncial descratar la lesion de
estructuras adyacentes, en especial el corazn. INSERCIN DE PLEUROTOMA; cuidados postoperatorios: El paciente deber permanecer en el
hospital hasta que el tubo torcico sea retirado. Debe quedar conectado a una trampa de agua donde el pivote del sello de agua debe quedar
sumergido a 2cm, y oscilar cuando est conectado al paciente. Mientras el tubo est colocado en el trax del paciente, los enfermeros verifican con
cuidado que no haya escapes de aire, dificultades o problemas al respirar o necesidad de administrar oxgeno adicional al paciente. El paciente tosa y
respire profundo para facilitar que los pulmones se expandan de nuevo, ayudar con el drenaje y prevenir que los fluidos normales se alojen en los
pulmones. ATENCIN Y MANEJO: Las muertes relacionadas con Trauma, en la actualidad siguen una curva bimodal (2 picos de frecuencia
diferentes). El primer pico, ocurre dentro de la primera hora. El segundo pico en las primeras 24 a 48 horas despus del trauma. La ciruga de control
de daos es en etapas: PRIMERA ETAPA: Control de hemorragia y fuga area. SEGUNDA ETAPA: Resucitacin en la UCI, manejo agresivo de la
hipotermia, acidosis, coagulopatia. TERCERA ETAPA: Tratamiento definitivo de las lesiones. REGLA DE ORO (GOLD STANDARD): Efectuar las mnimas
reparaciones definitivas, usando tcnicas que sean rpidas y fciles, para acortar al mximo los tiempos quirrgicos de este modo. En base a ATLS: A.
Va area y control de columna cervical. Permeabilidad, estabilidad y seguridad de la va area (eventual IOT), aspiracin de secreciones, fijacin de
columna cervical firme y segura, cricotiroidotoma por aguja o quirrgica. B. Respiracin: Oxigenacin, movimientos respiratorios, ventilacin
asistida, oclusin de heridas torcicas abiertas, toracocentesis y drenajes torcicos. C. Circulacin y control de hemorragias. Compresin directa de
los sitios de hemorragia, evaluacin de los pulsos, masaje cardiaco externo, instalacin de dos vas venosas perifricas proximales de grueso calibre
para alto flujo, reposicin de volumen y uso de frmacos endovenosos, analgesia y sedacin, monitorizacin cardiaca. D. Dficit neurolgicos. E.
Exposicin corporal y abrigo. En el caso de focos hemorrgicos en cavidades, es imprescindible su deteccin y esto se utiliza el FAST (Focused
Abdominal Sonography for Trauma), que permite la evaluacin del abdomen y del pericardio, del lquido pleural. La mayora de los traumatismos
torcicos 75-85 % se solucionan con maniobras salvadoras de vida y o procedimientos quirrgicos menores: 1) Toracocentesis 2) drenaje pleural 3)
Pericardiocentesis 4) Ventana xifopericrdica 5) Cricotiroidotoma 6) Traqueostoma. Alteraciones que requieren toracotoma urgente: 1. Fuga
masiva de aire. Indica rotura de la trquea o de un bronquio principal. Ms de 80% de las lesiones se localizas a menos de 2-5cm de la caria. 2.
Taponamiento pericrdico en presencia de traumatismo. 3. Perforacin esofgica.

CASO CLINICO TRAUMA TORACICO


Paciente de 43 aos, fumador habitual, acude a urgencias por dolor
costal, despus de una cada de 3 metros de altura
aproximadamente. A su llegada al hospital el paciente presenta PA
de 130/70 mmHg, FC de 65lpm y saturacin de oxgeno del 98%
basal. En la exploracin fsica destaca la presencia de crepitacin
sea a la altura de la parrilla costal derecha baja, la radiografa de
trax es normal. A las 48h el paciente consulta nuevamente a
urgencias por aumento del dolor torcico asociado a dificultad
respiratoria y empeoramiento del estado general. La PA fue de
90/60 mmHg, la FC de 105lpm y la saturacin basal del 89%. La
radiografa de trax demuestra derrame pleural importante y el
estudio analtico muestra los siguientes valores: Hto del 26% Hb de 7
g/l, plaquetas de 430.000/mm3. Estudio de coagulacin: normal.
PREGUNTA
La primera maniobra en este paciente al recibirlo en urgencias es:
RESPUESTA
a.- Colocacin de sello pleural.
b.- Intubacion orotraqueal.
c.- Iniciar con cristaloides.
d.- Paquete globular.
CASO CLINICO TRAUMA TORACICO
Varn de 33 aos que sufri un traumatismo torcico severo tras
atropello. Precis ventilacin mecnica y frmacos inotrpicos.
Presentaba un soplo de insuficiencia artica. Se realiz una

CURSO ENARM CMN SIGLO XXI TEL: 36246001

tomografa computarizada (TC) torcica, que objetiv neumotrax y


fracturas costales bilaterales, sin datos de diseccin artica. El
ecocardiograma transtorcico evidenci insuficiencia artica severa
con dilatacin ligera de la raz artica. En el ecocardiograma
transesofgico (ETE) se observ una imagen filiforme,
correspondiente a velo coronariano derecho roto, que protrua hacia
el tracto de salida del ventrculo izquierdo, e insuficiencia artica
severa. En la pared anterior de aorta ascendente proximal, haba una
imagen de seudoaneurisma con rotura de capas ntima y media y se
observaban colgajos medio-intimales protruyendo hacia la luz.
PREGUNTA
La triada de Beck consiste en:
RESPUESTA
a.- Hipotensin, ruidos cardiacos velados y distensin venas del
cuello.
b.- Hipertensin, edema facial y ruidos cardiacos velados.
c.- Hipotensin, torax inestable y disfagia.
d.- Ruidos cardiacos velados, distensin venas del cuello y disfona.
CASO CLINICO
Paciente de 53 aos que es trado a urgencias tras sufrir un
accidente de trfico, al parecer es el paciente el que conduca,
refiere que se ha salido de la carretera y recuerda todo lo que ha
pasado. Sin antecedentes patolgicos. Fumador pero sin otros
hbitos txicos. No alergias conocidas. Llega consciente y orientado,
mueve todos los miembros y las pupilas son normales. Refiere
intenso dolor torcico, especialmente en hemitrax izquierdo.

Pharmed Solutions Institute

PGINA 477

MANUAL DE TRABAJO DEL CURSO ENARM CMN SIGLO XXI


Respira espontneamente con gran trabajo respiratorio, taquipnea y
sudoracin. La auscultacin pulmonar muestra hipofonesis en la
mitad inferior del hemitrax izquierdo y la percusin es mate. El
resto de campos pulmonares es normal. No tiene dolor a otros
niveles y la exploracin cardiaca, de abdomen y de miembros es
normal. TA: 105/60; FC 110; FR 36; Temp. 36,1C. Analtica: Hb
11,5 g/dL; Hto. 34%; Glucemia, urea e iones normales. Gases
arteriales: pH 7,41; PCO2 38; PO2 50; HCO3 25. Radiografa de
trax: Fracturas costales izquierdas (de 3 a 6 ) a nivel del arco
lateral. Imgenes de derrame pleural izquierdo. Parnquima
pulmonar del resto de campos normal.
PREGUNTA
Cul es la conducta teraputica ms adecuada a seguir en este
momento?
RESPUESTA
a.- Toracocentesis, ms lquidos intravenosos
b.- O2 con mascarilla, ms analgesia epidural
c.- Lquidos intravenosos, mas O2 con mascarilla
d.- Analgesia epidural, ms lquidos intravenosos
PREGUNTA
La evolucin del paciente inicialmente fue buena, mejorando su
disnea y corrigindose la hipoxemia.. Se mantuvo consciente y con
buena hemodinmica. Pero 48 horas ms tarde el paciente empeora
su cuadro respiratorio, con disnea progresiva y cada de la SaO2 que
obliga a colocar VMNI (CPAP) y posterior IOT + VM invasiva. Cul es
la causa ms probable de la hipoxemia del paciente?
RESPUESTA
a.- Recidiva de derrame pleural
b.- Contusin pulmonar
c.- Trax inestable
d.- Neumotrax
PREGUNTA
Cul de las siguientes es menos probable que sea una indicacin
para intubara a este paciente?
RESPUESTA
a.- FR >30x
b.- PaO2 <60mmHg
c.- PaCO2 >45mmHg
d.- Shock, lesin neurolgica
CASO CLINICO
Mujer de 39 aos, cursando con embarazo gemelar bicorial
biamnitico de 27 semanas, sufre accidente automovilstico viajando
como copiloto sin cinturn de seguridad. Se golpea hemitrax
derecho contra el parabrisas, sin prdida del conocimiento. Es
ingresada al servicio de urgencias, donde refiere intenso dolor en
zona dorsal derecha a la altura de la escpula ipsilateral. Evaluada
por traumatlogo quien no evidencia lesiones de su competencia. Es
enviada al servicio de urgencias obsttricas realizndose ecografa
donde ambos fetos presentan latidos cardiacos presentes, sin
desprendimiento de placenta ni alteracin en el lquido amnitico.
La paciente se encuentra consciente, hemodinmicamente estable.
Se realiza radiografa de trax, que evidencia fractura costal de 4, 5

CURSO ENARM CMN SIGLO XXI TEL: 36246001

y 6 costillas derechas. EF: destaca aumento de volumen escapular


con erosiones superficiales y herida cortante de 3 cm en codo
derecho. Se maneja con analgesia, oxigenoterapia, hidratacin, y
debido a su gravidez y falta de cama en UCI se hospitaliza en la
unidad de de alto riesgo obsttrico. Veinticuatro horas posterior a su
ingreso, cursa con cuadro de inicio sbito caracterizado por disnea
de pequeos esfuerzos, polipnea, aumento de dolor costal,
normotensa, destacando al examen pulmonar murmullo vesicular
disminuido a derecha, asociado a crepitaciones ipsilaterales.
Evoluciona sudorosa, con mayor dificultad respiratoria con murmullo
pulmonar conservado a izquierda pero ausente a derecha,
impresiona diafortica, hipotrmica, taquicrdica, hipertensa,
saturando 76 % con O2 ambiental y 98 % con FiO2 50%.
PREGUNTA
Cul es la sospecha diagnostica ms probable en este momento?
RESPUESTA
a.- Neumotorax
b.- Derrame pleural
c.- Derrame 478ericrdico
d.- Hemotorax
PREGUNTA
Se reevala tubo de drenaje pleural, que da salida a 1000 ml de
sangre adicionales, por lo que se indica ciruga de urgencia por
hemotrax masivo e inestabilidad hemodinmica. Cul es la
conducnta mas adecuada a seguir en este momento?
RESPUESTA
a.- Uso de aminas vasoactivas
b.- Colocar un nuevo tubo de drenaje pleural
c.- Toracotomia exploradora
d.- Administrar corticoides para maduracin fetal y esperar 24hrs
PREGUNTA
Por la evolcuon de la paciente. Qu estructura es mas probable que
se haya lesionado?
RESPUESTA
a.- Aorta
b.- Diafragma
c.- Esofago
d.- Ventriculo derecho
PREGUNTA
Cul es la complicacin menos frecuente que podra presebtar la
paciente?
RESPUESTA
a.- Hemorragia pulmonar
b.- Mediastinitis
c.- Hemotrax masivo
d.- Infeccion por traslocacion bacteriana

Pharmed Solutions Institute

PGINA 478

MANUAL DE TRABAJO DEL CURSO ENARM CMN SIGLO XXI


TRAUMA ABDOMINAL ABIERTO Y CERRADO:
CIENCIAS BASICAS: Lesin orgnica producida por la suma de la accin de un agente externo junto a las reacciones locales y generales que provoca
el organismo ante dicha agresin. Todo paciente puede presentar lesiones en mltiples rganos abdominales y, por tanto, debe ser considerado
como un paciente con traumatismo grave, desde el momento del ingreso en la unidad de urgencias. Un tercio de los pacientes que requieren una
exploracin abdominal urgente tienen un examen fsico inicial anodino, tener en cuenta que puede tener un comportamiento impredecible y
desestabilizarse en el momento ms inesperado. Importante conocer el mecanismo lesional con el fin de anticipar las lesiones esperables.
ANATOMA: Abdomen anterior: se define como el rea localizada entre una lnea superior que cruza por las mamilas, los ligamentos inguinales y la
snfisis del pubis como la lnea inferior, y las lneas axilares anteriores lateralmente. Flanco: rea entre las lneas axilares anteriores y posteriores y
desde el cuarto espacio intercostal hasta la cresta ilaca. El espesor de la musculatura de la pared abdominal a este nivel, ms que las capas
aponeurticas ms delgadas de la pared anterior, acta como una barrera parcial a las heridas penetrantes, particularmente por arma blanca.
Espalda: localizada atrs de las lneas axilares posteriores, desde la punta de la escpula hasta las crestas ilacas, el espesor de la espalda y los
msculos paravertebrales actan como una barrera parcial a las heridas penetrantes. Anatoma interna del abdomen: Cavidad peritoneal: cubierto
por la parte baja de la parrilla costal, el abdomen superior incluye el diafragma, hgado, bazo, estmago y colon transverso. Debido a que el
diafragma en una espiracin total se eleva hasta el 4 espacio intercostal, las fracturas de costillas inferiores o heridas penetrantes en la misma rea
pueden involucrar estas vsceras abdominales. El abdomen inferior contiene el intestino delgado y el colon ascendente, descendente y sigmoides.
Cavidad plvica: rodeada por los huesos plvicos, corresponde a la parte baja del espacio retroperitoneal y contiene el recto, la vejiga, los vasos
ilacos, y en la mujer los genitales internos. Espacio retroperitoneal: contiene la aorta abdominal, la vena cava inferior, la mayor parte del duodeno,
el pncreas, los riones, los urteres, as como segmentos del colon ascendente y descendente. Las lesiones en las vsceras retroperitoneales son
muy difciles de reconocer porque el rea es de difcil acceso al examen fsico y sus lesiones no son detectadas por medio del lavado peritoneal
diagnstico, y difcilmente valoradas por la ecografa, adems de ser de difcil exploracin fsica. SALUD PUBLICA: Traumatismos de los ms
frecuentes, estimndose en 1 por cada 10 ingresos por traumatismo en los servicios de urgencias. Las principales causas de muerte en los pacientes
con traumatismo abdominal son: 1. Por lesin de algn vaso principal, como vena cava, aorta, vena porta o alguna de sus ramas, o arterias
mesentricas. Las lesiones destructivas de rganos macizos, como hgado, bazo o rin, o sus asociaciones, pueden originar una gran hemorragia
interna. 2. Sepsis: la perforacin o rotura de asas intestinales o estmago, supone la diseminacin en la cavidad peritoneal de comida apenas
digerida o heces, con el consiguiente peligro de sepsis. Los trastornos de vascularizacin de un asa intestinal por contusin de la pared intestinal o de
su meso pueden manifestarse tardamente como necrosis puntiforme parietal y contaminacin peritoneal con sepsis grave. CLASIFICACIN:
Abiertos (penetrantes y no penetrantes), presentan solucin de continuidad en la piel. Cerrados; la piel no tiene solucin de continuidad.
PATOGENIA: Las principales causas de abiertos son las heridas por arma blanca (lesiones intrabdominales de 20-30%) y arma de fuego cuya
frecuencia es creciente. La principal causa de cerrados son los accidentes de trfico. Otras causas son los accidentes de trabajo, accidentes
domsticos, accidentes deportivos, siendo estos mucho ms frecuentes que los abiertos. TRAUMATISMO ABDOMINAL ABIERTO: Las heridas por
arma blanca y de fuego de baja velocidad (< 600 m/seg) causan dao al tejido por laceracin o corte. Ceden muy poca energa y el dao se localiza
en la zona perilesional, afectando habitualmente rganos adyacentes entre s, siguiendo la trayectoria de, objeto que penetra. Las heridas por
proyectiles de alta velocidad (> 600 m/seg) transfieren gran energa cintica a las vsceras abdominales, teniendo un efecto adicional de cavitacin
temporal y adems causan lesiones adicionales en su desviacin y fragmentacin, por lo que es impredecible las lesiones esperadas. TRAUMATISMO
ABDOMINAL CERRADO: Impacto directo; la energa cintica a los rganos adyacentes a la pared abdominal, puede provocar lesin. Desaceleracin:
mientras el cuerpo es detenido bruscamente los rganos intra abdominales animados an por la energa cintica tienden a continuar en movimiento
producindose una sacudida, especialmente acusada a nivel de los puntos de anclaje, vasos y mesenterio que sufren desgarros parciales o totales.
Compresin o aplastamiento: entre dos estructuras rgidas, estas fuerzas deforman los rganos slidos o huecos y pueden causar su ruptura o
estallido de estos. Este es el mecanismo tpico de lesin del duodeno, en un accidente de automvil con impacto frontal, donde aquel es comprimido
entre el volante y la columna vertebral. VALORACIN INICIAL: El objetivo en evaluacin primaria es evidenciar o descartar lesiones de riesgo vital e
instaurar las medidas necesarias de soporte vital para preservar la vida del paciente. El paciente con traumatismo abdominal debe ser considerado
como traumatismo grave o potencialmente grave y por lo tanto, el manejo de estos pacientes debe seguir las recomendaciones del ABC: A. Asegurar
la permeabilidad de la va area, con control cervical. B. Asegurar una correcta ventilacin/oxigenacin. Descartar neumotrax a tensin, abierto,
hemotrax masivo. Valorar la necesidad de soporte ventilatorio. Si no es necesario administrar oxgeno a alto flujo con mascarilla (10-15 l/min). C.
Control de la circulacin. Detener la hemorragia externa. Identificacin y tratamiento del shock. Identificacin de hemorragia interna.
Monitorizacin ECG estable. D. Breve valoracin neurolgica. E. Desnudar completamente al paciente, controlando el ambiente y previniendo la
hipotermia. DIAGNOSTICO: Valoracin clnica del estado de shock: aumento de la frecuencia del pulso, pulso dbil y filiforme, piel plida, fra y
sudorosa, disminucin de la presin del pulso, retardo en el relleno capilar, alteracin de la conciencia, taquipnea, hipotensin y oligo anuria., dado
que la hemorragia intraabdominal es la causa ms frecuente de shock hipovolmico en estos pacientes. Inicialmente, se asume que el estado de
shock es el resultado de la prdida aguda de sangre y se la trata con una infusin rpida de volumen: un bolo inicial de 1-2 litros para un adulto de
SF al 09 % o de solucin de Ringer lactato. La restitucin por medio de catteres intravenosos perifricos de calibre grueso (14G 16G), dada su
mayor rapidez de canalizacin. El shock refractario a la infusin rpida de cristaloides sugiere sangrado activo y requiere de una laparotoma
urgente. Primeras medidas: Colocar sonda gstrica: cuyo objetivo es aliviar la dilatacin gstrica aguda, descomprimir y reducir el riesgo de
broncoaspiracin. Si existen graves fracturas faciales o la sospecha de una fractura de la base del crneo, la sonda debe introducirse por la boca para
evitar el riesgo del paso del tubo hacia el cerebro a travs de la lmina cribiforme. Canalizacin de dos vas venosas perifricas con catter de gran
calibre. Sonda vesical para aliviar la retencin de orina y descomprimir la vejiga, comprobar la presencia de diuresis, su flujo horario, descartando
previamente la presencia de lesin uretral. Se deben sacar muestras de sangre y realizar una determinacin de BH, tiempos de coagulacin,
gasometra, amilasa, niveles de alcohol, grupo sanguneo y pruebas cruzadas. Evaluacin Secundaria: Historia clnica, a partir del propio paciente,
de sus familiares y de los profesionales que han llevado a cabo la atencin prehospitalaria. Mecanismo de produccin del traumatismo: a) En
traumatismos cerrados es importante el tipo de impacto, dao del vehculo, uso de sistemas de seguridad, el estado de otras vctimas. b) Para las
heridas penetrantes, puede ser til una descripcin del arma y de la cantidad de sangre perdida en el lugar del hecho. Tiempo de evolucin desde el
trauma hasta la recepcin del paciente. APP: alergias, patologas previas, medicacin habitual, ciruga previa, ingesta de drogas. Maniobras
realizadas por los profesionales de la atencin prehospitalaria: volumen infundido, vas canalizadas, necesidad de resucitacin cardiopulmonar.
Exploracin: Inspeccin; observar el trax, abdomen, espalda, pelvis, perin, espalada. Hay que observar las huellas en la piel y pared de los puntos
de impacto del agente agresor. Auscultacin; confirmar la presencia o ausencia de ruidos intestinales. La presencia de sangre libre intra peritoneal
o contenido gastrointestinal pueden producir un leo que produce una ausencia de ruidos intestinales. Las lesiones en estructuras adyacentes, por
ejemplo, costillas, columna o pelvis, tambin pueden producir leo. Percusin; detectar matidez (presencia de lquidos) en caso de hemoperitoneo;
timpanismo (presencia de aire) si hay dilatacin gstrica o desaparicin de la matidez heptica por neumoperitoneo. Palpacin; debe repetirse

CURSO ENARM CMN SIGLO XXI TEL: 36246001

Pharmed Solutions Institute

PGINA 479

MANUAL DE TRABAJO DEL CURSO ENARM CMN SIGLO XXI


peridicamente, por lo que debe ser realizado por la misma persona para poder evaluar las diferencias que se originen. Primero, debe dirigirse al
plano parietal buscando la presencia de hematomas, o contusiones musculares. Luego debe investigar la presencia de contractura abdominal refleja,
que es un signo fiable de irritacin peritoneal, al igual que el signo del rebote positivo. Finalmente, hay que realizar una palpacin ms profunda
buscando la presencia de puntos o zonas dolorosa cuya topografa nos oriente a relacionarlas con los posibles rganos lesionados. Evaluacin
estabilidad plvica; La exploracin del anillo pelviano debe realizarse mediante una cuidadosa compresin lateral y antero posterior, siendo dolorosa
cuando hay fractura plvica. Una fractura de pelvis puede ser causa de shock hipovolmico, en ocasiones muy severo. Produce hematoma perineal
y genital a las 24-48 horas del traumatismo y puede acompaarse de hematoma retroperitoneal y ausencia de hemoperitoneo. Examen del perin y
genitales; lesiones externas y la presencia de signos de lesin uretral como sangre en el meato, hematoma escrotal o desplazamiento hacia arriba de
la prstata. La laceracin de la vagina puede ocurrir en heridas penetrantes o por fragmentos seos de una fractura plvica. PRUEBAS
COMPLEMENTARIAS: Analtica: bioqumica, hemograma, tiempo de coagulacin, pruebas cruzadas, niveles de alcoholemia y anlisis de orina. ECG
y monitorizacin de constantes vitales. Radiografa de abdomen. Radiografa de trax: es importante para descartar la presencia de hemotrax,
neumotrax o fracturas costales. Radiografa de pelvis. Exmenes complementarios: Lavado peritoneal diagnstico (LPD); procedimiento invasivo
que puede ser realizado de forma rpida presenta una sensibilidad del 68% y una especificidad del 83%. La indicacin principal del LPD son Hallazgos
abdominales equvocos. Exploracin fsica no realizable por traumatismo raqudeo concomitante o alteracin de la conciencia (traumatismo crneo
enceflico o txico). Imposibilidad de reevaluacin contina. Hipotensin inexplicable. Prdida progresiva de sangre (descenso progresivo del
hematocrito). Puede realizarse mediante un mtodo abierto o cerrado. Un LPD negativo no excluye la presencia de lesiones retroperitoneales o
desgarros diafragmticos. El ultrasonido puede detectar presencia de hemoperitoneo, es un medio rpido, no invasivo y seguro en el diagnstico de
lesiones intra abdominales (cerrada o penetrante) y puede ser repetido frecuentemente. Es ms sensible que el lavado peritoneal diagnstico para la
determinacin de lesiones de vsceras macizas, aunque no lo es tanto como la TAC. Su indicacin es absoluta en casos de embarazo, cicatrices
abdominales por cirugas previas y alteracin de la coagulacin. La TAC requiere el transporte del paciente a la sala de rayos X, administracin oral e
intravenosa de contraste. Consume tiempo y es utilizado nicamente en pacientes hemodinmicamente estables en los que no existe la indicacin
inmediata de laparotoma. La TAC proporciona informacin relativa a la lesin especfica de un rgano en particular y tambin puede diagnosticar
lesiones en el retroperitoneo u rganos plvicos que son difciles de evaluar en la exploracin fsica o en el LPD. Laparoscopia la utilidad de la
laparoscopia efectuada bajo anestesia local para identificar lesiones diafragmticas y cuantificar la cantidad de sangre intraperitoneal. MANEJO DEL
PACIENTE TRAS LA VALORACIN INICIAL: 1. Traumatismo Abdominal Cerrado, Hemodinmicamente inestable: (imposibilidad de mantener una TA
sistlica por encima de 90 mm Hg, frecuencia cardiaca inferior a 100 lpm o diuresis de 50 ml/h adulto) Si el paciente presenta signos abdominales
patolgicos (distensin abdominal, peritonismo, neumoperitoneo en la radiografa simple), entonces la indicacin de laparotoma debe ser
inmediata. Si los signos abdominales son dudosos y el paciente presenta un traumatismo craneoenceflico o espinal severo, alteraciones de la
conciencia por toxicidad, traumatismos toraco-abdominales, debemos llevar a cabo un estudio rpido del abdomen que nos ayude a descartar la
presencia de patologa abdominal, fundamentalmente lquido libre. Para ello disponemos de dos pruebas que no son excluyentes: Ecografa
abdominal. Lavado peritoneal diagnstico. Otras pruebas radiolgicas que precisan mayor infraestructura y tiempo, como la TAC, no son posibles en
el paciente inestable. Hemodinmicamente estable. Se deben tener en cuenta las siguientes premisas a la hora de manejar un paciente con un
traumatismo abdominal cerrado: Las vsceras macizas se lesionan con ms frecuencia que las huecas. Traumatismo Abdominal Abierto; Arma
Blanca. - Si el paciente est inestable o presenta signos de irritacin peritoneal, debe ser sometido a una laparotoma urgente. - Si est estable
hemodinmicamente y no presenta signos de irritacin peritoneal, la primera maniobra que se debe realizar es la exploracin del orificio de entrada
del arma, comprobando si la herida es penetrante o no. Arma de Fuego; Puesto que la trayectoria de una bala es difcil de predecir y dado que el 8090% de los traumatismos por arma de fuego se asociaran a una o ms lesin visceral, el tratamiento de estos pacientes ser quirrgico,
realizndoseles una laparotoma urgente. Recomendaciones basadas en evidencia, Recomendaciones NIVEL I: Laparotoma exploratoria est
indicada para pacientes con LPD (+) TAC est recomendado para pacientes hemodinmicamente estables con un EF equvoco (lesin neurolgica,
lesiones extrabdominales). TAC es la modalidad diagnstica de eleccin para manejo no operatorio de lesiones de vsceras slidas. En pacientes
hemodinmicamente estables, LPD y TAC son modalidades diagnsticas complementarias.
CASO CLINICO TRAUMA ABDOMINAL
Varn de 28 aos, trado a urgencias tras haber tenido una cada en
bicicleta y con traumatismo cerrado periumbilical, refiriendo dolor
intenso, se observa distencin abdominal, adems se observa aire
subdiafragmatico en RX, signos vitales TA 100/70, FC 105, FR 28, se
apresia diafortico con palidez de tegumentos.
PREGUNTA
Cual es la conducta a seguir?
RESPUESTA
a.- Lavado peritoneal.
b.- Restitucin volumtrica.
c.- Laparatomia exploratoria.
d.- Conducta expectante.
CASO CLINICO
Femenino de 66 aos de edad, sana cay de su propia altura. Su
miembro inferior derecho estaba en flexin, rotacin externa y
abduccin con extensin de la rodilla durante la lesin. La paciente
experimenta dolor en la cadera grave y no pudo incorporarse.
Describi la sensacin de que su cadera derecha haba sido
desplazada de su articulacin momentneamente.
PREGUNTA
Cual de las siguientes medidas posoperatorias no es necesaria?
RESPUESTA

CURSO ENARM CMN SIGLO XXI TEL: 36246001

a.- Control estricto de liquidos.


b.- Analgesico, antibitico y enoxaparina.
c.- Realizar laboratorio y vigilar sangrado.
d.- Interconsulta a medicina interna
CASO CLINICO
Un paciente ingres por herida de arma de fuego (bala) en la regin
abdominal. Se encuentra lcido, TA 100/60, FC 120/min. La entrada
de aire est disminuida en el hemitrax izquierdo y la FR es de
17/min. Presenta orificio de entrada en la fosa ilaca derecha sin
orificio de salida. Refiere dolor abdominal que se exacerba con la
palpacin superficial y profunda, presenta defensa muscular.
PREGUNTA
Cul es la conducta diagnostica ms adecuada a seguir en este
caso?
RESPUESTA
a.- TAC de abdomen y trax
b.- FAST
c.- Radiografa, trax, abdomen y pelvis
d.- RMN abdominoplvica
PREGUNTA
Cul es la conducta teraputica ms adecuada a seguir en este
caso?
RESPUESTA

Pharmed Solutions Institute

PGINA 480

MANUAL DE TRABAJO DEL CURSO ENARM CMN SIGLO XXI


a.- Lavado peritoneal diagnstico y drenaje pleural
b.- Drenaje pleural y laparotoma exploradora
c.- Tomografa y laparotoma exploradora
d.- Drenaje pleural y tomografa

d.- FAST

PREGUNTA
Qu estructura de cavidad abdominal sera la ms probable de
encontrar lesionada en este paciente?
RESPUESTA
a.- Hgado
b.- Estructuras vasculares
c.- Intestino delgado
d.- Bazo
CASO CLINICO
Masculino de 62 aos sin antecedentes de importancia. Ingresa al
servicio de urgencias tras sufrir un accidente automovilstico de
impacto frontal y alta velocidad. Era el conductor y tena colocado
un cinturn de seguridad de 3 puntos. Ingres con disnea y dolor en
hipocondrio y hemitorax izquierdo. A la exploracin fsica
presentaba signos vitales inestables, datos de dificultad respiratoria
moderada-severa, con ausencia de murmullo vesicular en tercio
inferior de hemitrax izquierdo. No presenta datos de abdomen
agudo. Se realiz una radiografa simple de trax en la cual se
encontr imagen radioopaca de bordes regulares que ocupaba un
60% de hemitrax izquierdo y desplazaba el mediastino hacia
hemitrax derecho.
PREGUNTA
Cul es la conducta teraputica inmediata ms adecuada para este
paciente?
RESPUESTA
a.- Va area permeable
b.- Toracotoma
c.- Laparotoma
d.- Drenaje pleural
PREGUNTA
Qu estructura de cavidad abdominal sera la ms probable de
encontrar lesionada en este paciente?
RESPUESTA
a.- Hgado
b.- Estructuras vasculares
c.- Intestino delgado
d.- Bazo
PREGUNTA
Cul es la conducta diagnostica, ya con el paciente estable ms
adecuada a seguir?
RESPUESTA
a.- Resonancia magntica
b.- Lavado peritoneal diagnostico
c.- Tomografa

CURSO ENARM CMN SIGLO XXI TEL: 36246001

CASO CLINICO
Paciente de 21 aos, estudiante de magisterio, nulpara, afebril.
Antecedentes patolgicos personales y familiares no destacables. La
paciente recuerda que: hace cerca de dos meses al ser perseguida
en la Salida del Metro por un posible ladrn, le fall su rodilla
derecha y cay sobre el filo de cemento en la escalera, recibiendo el
golpe principal en las costillas y la parte derecha superior del vientre,
no se pudo levantar hasta ser socorrida por los Bomberos. Acude por
referir Malestar bajo las costillas derechas, astenia, discreta
palidez cutneo mucosas, inapetencia. Abarca y seala con su mano
el malestar, dolor en el hipocondrio derecho y fatiga, sin referir
antecedentes previos, su madre la llev a cuatro Instituciones de
salud e incluso agotaron el uso del Seguro Mdico, sin tener
respuestas. A su ingreso a urgencias en la EF: Mucosas
hipocoloreadas, tensin arterial: 100/60, FC: 102/min., orina oscura,
lengua seca, decaimiento general. En abdomen, se aprecia malestar
a la palpacin profunda del cuadrante superior derecho hacia la zona
sub-costal, no precisamos tumor palpable, no visceromegalias,
percusin algo molesta, ruidos abdominales presentes. La paciente
nicamente cuenta con BH la cual reporta Hb de 10.1mg/dl,
leucocitos 12,000.
PREGUNTA
Cul es la estructura que ms probablemente se haya lesionado en
este caso?
RESPUESTA
a.- Bazo
b.- Higado
c.- Diafragma
d.- Vesicula biliar
PREGUNTA
Posteriormente se decide realizar una Resonancia Magntica y se
informa de una posible coleccin subdiafragmtica derecha. Cul es
la conducta teraputica ms adecuada a seguir en este momento?
RESPUESTA
a.- Drenaje endoscpico
b.- Laparotoma
c.- Observacin y tx medico
d.- Lavado peritoneal
PREGUNTA
Cul es la indicacin menos probable para realizar una laparotoma
exploradora?
RESPUESTA
a.- Trauma cerrado con lavado peritoneal positivo
b.- Trauma cerrado con hipotensin persistente a pesar de adecuada
resucitacin
c.- Trauma cerrado con lquido libre en cavidad
d.- Datos tempranos de peritonitis

Pharmed Solutions Institute

PGINA 481

MANUAL DE TRABAJO DEL CURSO ENARM CMN SIGLO XXI


PIE DIABETICO Y COMPLICACIONES QUIRURGICAS
CIENCIAS BASICAS: Es una complicacin crnica de la DM, con etiologa multifactorial, teniendo como factores principales la infeccin, isquemia,
neuropata y mal control metablico. SALUD PUBLICA: La incidencia y prevalencia de la lcera en gente con DM vara de 2,4 a 2,6 y de 4 a 10%,
respectivamente.68 Aproximadamente 15% de todos los pacientes con DM desarrollar una lcera en el pie o en la pierna durante el transcurso de
su enfermedad. PATOGENIA: Las dos lesiones bsicas del PD son la neuropat a (pie indoloro) y las angiopapata (pie sin pulso). Estas son origen de
las complicaciones isqumicas e infecciosas. La hiperglucemia crnica es el factor de riesgo ms importante para el desarrollo de la neuropata.
Existen dos teoras etiopatognicas: una metablica y otra vascular, aunque probablemente sea la interaccin de ambas lo que la produzca. La
polineuropata simtrica distal de distribucin en calcetn es la forma de neuropata ms frecuente que predispone a la aparicin de lceras en el
pie. Inicialmente se alteran las fibras nerviosas pequeas (fibras sensitivas) provocando la prdida de la sensacin de dolor y temperatura y, ms
tarde, las fibras nerviosas grandes, disminuyendo el umbral de percepcin de vibracin. La neuropata motora produce hipotrofia muscular
apareciendo dedos en martillo y en garra que favorecen la protrusin de las cabezas de los metatarsianos. La neuropata autonmica produce
sequedad y fisuras en la piel por afectacin de las fibras simpticas posganglionares de las glndulas sudorparas, aumento del flujo sanguneo por
apertura de comunicaciones arteriovenosas que disminuyen la perfusin de la red capilar y aumentan la temperatura de la piel, lo que da lugar a un
trastorno postural en la regulacin del flujo con una respuesta inflamatoria anormal
CLASIFICACIN DE WAGNER.
frente a la agresin a los tejidos. Tambin hay edema neuroptico y aumento de la
0
Ninguna.
Pie de riesgo Callos, hallux valgus,
presin venosa. La osteoartropata neuroptica o PIE DE CHARCOT es una de las peores
dedos en garra
consecuencias de la DM en el pie. Puede encontrarse en una fase aguda que puede
I
lceras superficiales
Destruccin del espesor total de la piel
II
lcera profunda
Penetra piel, grasa y ligamentos sin
confundir con infeccin o en una fase deformante crnica y progresiva caracterizada
afectar hueso. Infectada
por la destruccin indolora de huesos y articulaciones. En esta situacin, con los
II
lcera profunda ms
(osteomielitis)
traumatismos se produce distensin ligamentosa y microfracturas, y con el peso
absceso
Profunda, secrecin, mal olor
corporal ocurre una destruccin articular progresiva que da lugar a fracturas y
IV
Gangrena limitada
Necrosis de una parte del pie
subluxaciones que son acelerados por el aumento de la reabsorcin sea osteoclstica,
V
Gangrena extensa
Todo el pie afectado, efectos
sistmicos
secundaria a la hiperemia causada por la denervacin simptica de la microcirculacin.
Por estas razones, el pie debe estar inmovilizado con frulas, muleta o silla de ruedas. La neuropata, con o sin isquemia asociada, est implicada en
la fisiopatologa de la lcera del PD en el 85% a 90% de los casos. Por tanto, el pie insensible, sea bien perfundido o isqumico, debe considerarse
como de alto riesgo de lcera. DIAGNOSTICO: Tiene caractersticas clnicas frecuentes como lceras, gangrenas y amputaciones en las extremidades
inferiores, ocasionando en el paciente discapacidad parcial o definitiva. Los pacientes con este problema de salud deben ingresar a un programa de
PD que debe ser permanente, estratgico y multidisciplinario incluyendo la prevencin y el tratamiento del PD. Infeccin: Es frecuenteencontrar
manifestaciones de calor, rubor o eritema, edema, dolor, olor ftido y secrecin purulenta. Tambin puede manifestarse con necrosis de los bordes
de la herida, aumento del flujo y tejido desvitalizado. Es
multifactorial: La neuropata y la consiguiente disminucin de la
sensibilidad predispone a las lceras a igual que la disminucin del
reflejo vasomotor que lleva a disminucin de sudoracin y a
sequedad de la piel y a la aparicin de grietas. La micosis
interdigital produce fisuras. La macroangiopata lleva a
insuficiencia arterial con retardo en la cicatrizacin y deficiente
llegada de antibiticos. El desarrollo de derivaciones
arteriovenosas de capilares nutricios tambin predispone a
lesiones de los tejidos. Traumas. La infeccin superficial aguda
(lceras no complicadas, celulitis): adquiridas en la comunidad y
sin tratamiento antibacteriano, en su mayora son
monomicrobianas, Staphylococcus y Streptococcus spp. No
siempre es necesario el uso de antibiticos ni de la toma de
cultivos, especialmente si es superficial. Las Infecciones profundas
y/o crnicas: son polimicrobianas en ms de 50% de los casos,
encontrndose dos a tres grmenes. Hay que tomar cultivo de
tejido profundo. A las cocceas grampositivas de las superficiales
se agregan bacilos gramnegativos y anaerobios.La lesin de fibras sensoriales lleva a disminucin de liberacin de histamina, con la consiguiente
disminucin de signos inflamatorios y de la respuesta inflamatoria. La respuesta inflamatoria es fundamental en los procesos de defensa y de la
cicatrizacin. TRATAMIENTO: Compromiso del estado general: Indicar si el manejo es ambulatorio o de hospitalizacin. Paciente con herida Wagner
3 o grado mayor por lo general necesita hospitalizacin. Control metablico: La hiperglicemia se controla principalmente con insulina, 30 U/d en
promedio, se regula segn su demanda y se usa sin o con antidiabticos orales, principalmente metformina, 1 700 mg/d. En estos pacientes, para el
manejo de la dislipidemia, se ha obtenido buenos resultados con atorvastatina, 40 mg/d. Si hay hipertensin, generalmente se controla con losartn
o enalapril solos o asociados con hidroclorotiazida, y si hay edemas se recomienda furosemida. Los pacientes obesos logran bajar de peso con
educacin, dieta de 1 200 kcal y ejercicios moderados de al menos 30 min/d. Con estas medidas tambin mejoran los problemas metablicos
descritos anteriormente. Adems del cambio de estilo de vida permanente, es de ayuda temporal el uso de orlistat, 120 mg, tres veces al da.
Cuidados de la lcera: Desbridamiento quirrgico, con bistur (Figura 1) y, en algunos casos, debridamientos enzimtico y biolgico (larvas). El uso de
factores de crecimiento y de injertos han logrado la cicatrizacin en un menor tiempo y se ha evitado las amputaciones. El tratamiento tpico con
propiedades antibiticas o regenerativas tambin son importantes. Tratamiento antibitico: Primer evento y sin compromiso de la extremidad.
Manejo ambulatorio con antibiticos va oral por dos semanas: Cefalosporina primera generacin (cefalexina), 500 mg, cada 6 horas, Clindamicina,
300 mg, cada 8 horas, Dicloxacilina, 500 mg, cada 6 horas, Levofloxacino, 500 a 750 mg, cada 24 horas. lcera recidivante, celulitis extensa en
miembro inferior, compromiso articular u seo. Manejo hospitalario con antibitico intravenoso. El tiempo vara dependiendo de la condicin clnica
del paciente: Ceftazidima, 1 g, EV, cada 8 horas, ms clindamicina, 600 mg, EV, cada 8 horas. Ceftriaxona, 1 g, EV, cada 12 horas, ms clindamicina,
600 mg, EV, cada 8 horas. Ciprofloxacino, 400 mg, EV, cada 12 horas, ms clindamicina, 600 mg, EV, cada 8 horas. Infecciones que amenazan la vida,
sospecha de meticilinorresistencia o multirresistencia y Pseudomonas Vancomicina, 1 g, EV, cada 12 h, ms ceftazidima, 1 g, EV, cada 8 h, ms
clindamicina, 600 mg, EV, cada 8 h. Vancomicina, 1 g, EV, cada 12 h, ms meropenem, 500 mg, EV, cada 8 h, ms moxifloxacino, 400 mg, VO, cada 24
h. Osteomielitis: El periodo mnimo de tratamiento antibitico es tres meses y puede durar seis meses a una ao. Para acortar este tiempo se debe
realizar curetaje o extraccin de huesos infectados y desvitalizados. Tratamiento hipolipemiante: Existen varias alternativas pero se prefiere

CURSO ENARM CMN SIGLO XXI TEL: 36246001

Pharmed Solutions Institute

PGINA 482

MANUAL DE TRABAJO DEL CURSO ENARM CMN SIGLO XXI


atorvastatina, 20 a 40 mg/comida. Tratamiento del dolor neuroptico: Una de las combinaciones que ha dado buenos resultados es asociar
pregabalina con tramadol, titulando la dosis. Amputacin: Es el ltimo recurso para evitar la complicacin o muerte del paciente con PD. Toda
amputacin que se realiza en el pie se denomina menor y las que se realizan por arriba del tobillo, incluyendo la infracondlea y supracondlea, se
denominan mayores. Es preferible una amputacin menor porque con esta el paciente casi siempre puede caminar por s solo. Cuando es
amputacin mayor, el paciente deber usar en forma permanente una muleta, prtesis o silla de ruedas si quiere desplazarse. Con la amputacin
mayor, el paciente entra a una discapacidad que deteriora su salud y calidad de vida por lo que necesita mayor apoyo de su familia y de la sociedad.
Se indica amputacin mayor cuando un miembro isqumico no se puede revascularizar o es afectado por osteomielitis difusa que no responde al
tratamiento y amenaza la vida del paciente. Cuando el miembro infectado constituye un foco sptico que no responde al tratamiento antibitico y
desbridamientos, para salvar la vida del paciente se debe tomar la decisin de una amputacin mayor.
CASO CLINICO PIE DIABETICO
Acude a consulta masculino de 58 aos de edad el cual refiere que
desde hace una semana inicia con una lesin en tipo ulcera plantar
de 3.5 cm aproximadamente, como antecentes de importancia, se
trata de diabtico controlado con hipoglucemiantes orales, como
moderado apego a tratamiento, sin embargo debido a su nivel
socioeconmico primero trato de manejar su lesin por medio
tradicionales, sin embargo usted no percibe datos sugerentes de
proceso por clostridium o agentes anaerobios, la extremida presenta
temperatura normal, sus pulso estn presentes y solo se observa
hiperbaralgesia.
PREGUNTA
Considerando este cuadro clnico, cual es el manejo de primera
intensin para este caso antes de realizar un envo a segundo nivel
de atencin?
RESPUESTA
a.- Es un paciente que tendra una rpida mejora en cmara
hiperbarica.
b.- Indica antiagregante plaquetario, pentoxifilina y vendaje de
jhonson.
c.- Lo ms indicado es debridar el area y antibiticos.
d.- Indica debridacion, pentoxifilina, antibitico y vendaje
CASO CLINICO
Paciente de sexo masculino de 57 aos de edad que ingresa por
guardia con diagnstico de pie diabtico infectado. Antecedentes
personales patolgicos: Diabtico tipo 2: De 6 aos de evolucin en
tratamiento con metformina 850 mg/da (sin controles mdicos).
Hipertensin arterial: Enalapril 10 mg/da. Obesidad IMC: 30.5.
Tabaquista: 2 cigarrillos /da desde hace 20 aos. Refiere que hace 1
mes comenz como una lesin ulcerosa en planta de pie izquierdo a
nivel de 3 falange, fue tratado con amoxicilina 500 mg c/ 8 hs en las
ltimas 2 semanas sin mejora, presentando al ingreso gran
inflamacin en el pie, con cianosis de 2 y 3 falange, crepitantes en
dorso de pie, flictenas, placas de necrosis y abundante secrecin
purulenta. Pulsos pedio y tibial posterior (+). Signos vitales: TA:
130/90 FC: 90 FR: 20 T: 38.9. Glucemia: 482 mg/dl, GB: 14500
Neutrfilo:80%, Urea: 60mg/dl, Hemoglobina: 11 g/dl, Creatinina:
0.97 mg %. GOT: 25, Sodio (Na): 137 mEq/L. GPT: 38. Potasio (K): 4.8
mEq.
PREGUNTA
Cul es el diagnostico mas probable para este paciente?
RESPUESTA
a.- Fascitis necrosante
b.- Gangrena gaseosa
c.- Celulitis anaerobia
d.- Gangrena Fournier
PREGUNTA
Cul es la conducnta teraputica mas adecuada a seguir?
RESPUESTA
a.- Ciprofloxacino mas clindamicina
b.- Vancomicina

CURSO ENARM CMN SIGLO XXI TEL: 36246001

c.- Ceftriaxona mas amikacina


d.- Ciprofloxacino
CASO CLINICO
Paciente masculino de 48 aos de edad, originario y residente de la
ciudad de Celaya, que fue hospitalizado en el servicio de Medicina
Interna. Su abuelo paterno y padre murieron debido a
complicaciones de diabetes mellitus tipo 2. Se sabe diabtico desde
hace 13 aos y su tratamiento mdico es con sulfonilureas
biguanidas, porque tiene poco apego a la dieta. Desde hace un ao
le han indicado insulina intermedia en dos dosis. Sus glucemias
centrales o capilares en la consulta externa oscilan de 140 a 280
mg/dL. Padece hipertensin arterial sistmica desde hace cuatro
aos. En el ao 2000 present necrobiosis del primero, tercero y
cuarto dedos del pie derecho. Se le propuso amputacin
supracondlea femoral porque la radiografa mostraba neumatosis
en el tejido subcutneo en los mrgenes del pie. El paciente rehus
la ciruga.
PREGUNTA
Cul es la conducnta teraputica mas adecuada seguir?
RESPUESTA
a.- Realizar la ciruga con el consentimiento de un familiar
b.- Solo tratamiento medico
c.- Amputacion parcial de dedos afectados, si lo acepta el paciente
d.- Medidas higienco-dieteticas
PREGUNTA
Tres aos despus tuvo una lcera plantar del mismo pie y
osteomielitis de metatarsianos segundo y tercero. Se le volvi a
proponer la amputacin supracondlea femoral. El paciente
nuevamente la rechaz.
Cul es la conducnta mas adecuada a seguir en este caso?
RESPUESTA
a.- Realizar la ciruga con el consentimiento de un familiar
b.- Solo tratamiento medico
c.- Amputacion parcial de dedos afectados, si lo acepta el paciente
d.- Medidas higienco-dieteticas
PREGUNTA
Tres das previos a su ingreso al hospital le apareci una ampolla e
inflamacin del quinto dedo del pie izquierdo, posterior al uso de
zapato nuevo una semana antes. Despus advirti cambio de
coloracin del dedo, acudi a urgencias y se hospitaliz EF: En el pie
izquierdo se muestran las lesiones del quinto dedo que motivaron su
ltimo ingreso. Los pulsos pedios de ambos pies de buena intensidad
y el llenado capilar distal de los dedos se estim retrasado (ms de
tres segundos). La temperatura de la piel se juzg normal. Cul es la
manera mas adecuada de evaluar la sensibilidad?
RESPUESTA
a.- Digitalmente
b.- Monofilamento
c.- Diapason
d.- Cambios de temperatura

Pharmed Solutions Institute

PGINA 483

MANUAL DE TRABAJO DEL CURSO ENARM CMN SIGLO XXI


RETINOPATIA DIABETICA (RD), RETINOPATIA HIPERTENSIVA:
RETINOPATIA DIABETICA (RD). CIENCIAS BASICAS: Es una microangiopata, que aparece como complicacin crnica de DM se caracteriza por la
presencia de lesiones microvasculares, microaneurismas, hemorragias, exudados duros, manchas algodonosas, alteraciones microvasculares,
arrosariamiento venosos, neovasos y tejido fibroso. Factores de aparicin: Control metablico pobre HbA1c, embarazo, HAS, nefropata,
alcoholismo, antigedad de la DM >5aos, dislipidemia, obesidad. SALUD PUBLICA: Principal causa de ceguera legal 20-65 aos. Primera causa de
ceguera prevenible. Casi todos los pacientes con DM tipo II tendrn evidencia de RD a los 20 aos de evolucin y ms del 21% presentan RD al
momento del diagnstico. La RD es la causa de 5% de los casos de ceguera en el mundo. PATOGENIA: Se produce por una microangiopatia el
mecanismo ms probable es que la hiperglicemia produce alteraciones del metabolismo intracelular que llevan, como resultado, a un aumento del
sorbitol. Esto produce el engrosamiento de la membrana basal endotelial y la prdida de los Pericitos, los cuales son clulas que envuelven a los
capilares retinales, la prdida de pericitos producira, a su vez, dos secuencias de eventos paralelas: a) Alteracin de la barrera hematoretinal,
filtracin al espacio extravascular, edema retinal, exudados lipdicos o creos formados por lipoprotenas. b) Formacin de microaneurismas por
debilidad estructural de la pared de los capilares retinales, activacin de la coagulacin en los microaneurismas, trombosis intracapilar, obstruccin y
cierre capilar. Lo anterior ser responsable de la produccin de isquemia retinal, con el consecuente desarrollo de manchas algodonosas, (que
corresponden a infartos de la capa de fibras nerviosas) neovascularizacin, hemorragias y, en ltimo trmino, complicaciones tales como
desprendimiento de retina traccional, glaucoma y, en definitiva, ceguera. El crecimiento de neovasos, tanto a nivel retinal como en el iris, se
producira debido a la liberacin por parte de la retina isqumica de un factor soluble estimulador del crecimiento vascular (VEGF) y a su efecto
sinrgico junto a un factor de crecimiento vascular presente en la retina (Factor de Crecimiento de Fibroblastos Bsico, bFGF). CLASIFICACION: En
una etapa temprana o Retinopata Diabtica No Proliferativa (RDNP) y una ms avanzada o Retinopata Diabtica Proliferativa (RDP). La RDNP se
subdivide a su vez en: 1. Leve (Presencia de al menos un microaneurisma, hemorragia retineana, exudado duro o exudado blando (mancha
algodonosa), 2. Moderada (Exudados duros, rosarios venosos, manchas algodonosas en menor grado que en la severa), 3. Severa (Regla 4-2-1, 4
cuadrantes con microaneurismas o hemorragias, 2 cuadrantes con rosarios venosos, manchas algodonosas, 1 cuadrante con IRMA) y 4. Muy severa
(2 criterios). La aparicin de estos neovasos es lo que define a la Retinopata Diabtica Proliferativa. Los neovasos se observan con mayor frecuencia
en el nervio ptico o cercano a las arcadas vasculares, pero se pueden encontrar en cualquier parte del fondo de ojo. La presencia de un
desprendimiento de Retina crnico en un paciente con RDP es un factor de riesgo para la neovascularizacin iridiana y para el Glaucoma neovascular
secundario, cuyo pronstico es muy sombro. El Edema Macular es un evento que puede suceder en cualquier momento de la progresin de la RD
los hallazgos son: minimo (algo de engrosamiento de retina o exudados duros en el polo posterior de la macula), moderado (engrosamiento de
retina o exudados duros que se acercan a ala macula), severo (engrosamiento de retina o exudados duros que involucran el centro de la macula).
DIAGNOSTICO: Clnica; Microaneurismas (relacionados con zonas de baja perfusin. Signos: puntos rojos), hemorragias (en forma de flama
(arteriolas precapilares), intraretineanas: venas, forma de puntos), edema macular (fuga capilar extensa), exudados duros (edema crnico localizado,
macrfagos con depsitos lipdicos y lipoproteinas PE, por lo general rodean microaneurismas), exudados algodonosos o blandos (infartos,
obstruccin de flujo axoplsmico en la capa de fibras nerviosas, inflamacin). Agudeza visual: Es de importancia saber que para la clasificacin de la
retinopata diabtica no se toma en cuenta la agudeza visual, sin embargo, cuando est se ve afectada nos habla principalmente de que la macula
est comprometida. Realizar exploracin de fondo de ojo bajo dilatacin pupilar por un oftalmlogo en DM tipo I: a partir de los 12 aos de edad. A
partir del 5to ao de diagnstico y cada ao, cuando el diagnostico sea posterior a la pubertad, debe comenzar cada 3 aos. En DM tipo II: al
momento del diagnstico, realizado por un oftalmlogo y debe repetirse anualmente. La ecografa es una prueba muy til para los diabticos con
medios opacos. Esta prueba debe ser considerada cuando los medios no permiten la observacin mediante oftalmoscopio indirecto. TRATAMIENTO:
Educacin del Paciente que tome conciencia de su enfermedad, de sus riesgos potenciales, y que acuda a controles peridicos con su diabetlogo y
con su oftalmlogo. Durante esta etapa se debe optimizar el control metablico de los pacientes, corregir la hiperlipidemia, lograr un adecuado
control de la hipertensin arterial, tratar la nefropata y hacer controles oftalmolgicos ms frecuentes en las mujeres embarazadas, en los casos
que corresponda. Fotocoagulacin con Lser. Vitrectoma va pars plana. Antiangiognicos. La Panfotocoagulacin consiste en hacer aplicaciones de
lser trmico sobre la superficie retinal. Estas quemaduras destruyen la retina en el lugar en que son aplicadas, creando una cicatriz. La racionalidad
de este tratamiento se basa en que, al destruir la retina isqumica, sta sera incapaz de producir el Factor de Crecimiento Vascular Endotelial, el que
sera el responsable de la formacin de los neovasos. La disminucin de la produccin de este factor soluble lograra la regresin de la
neovascularizacin existente y la prevencin de su desarrollo en el futuro. Este tratamiento no es inocuo. La panfotocoagulacin disminuye en un
50% el riesgo de prdida visual severa, especialmente en los pacientes con RDP de alto riesgo. El edema macular ya sea focal o difuso, en Edema
Macular Clnicamente Significativo (EMCS) y Edema Macular No Clnicamente Significativo (EMNCS). Esta definicin se basa exclusivamente en
aspectos oftalmoscpicos que dicen relacin con la cercana del edema al centro de la fvea y no considera la agudeza visual del paciente, es decir,
un paciente puede tener visin normal y aun as tener un EMCS, el tratamiento del EMCS focal con Fotocoagulacin Focal disminuye en un 50% el
riesgo de prdida visual moderada en los pacientes tratados versus los controles, y mejora la agudeza visual en el 16% de los pacientes. El EMCS
difuso, hasta hace poco no tena un tratamiento efectivo. Actualmente, con el advenimiento de la Triamcinolona intravtrea, como tratamiento de
este tipo de edema, se han logrado mejoras espectaculares en la disminucin del edema. RETINOPATIA HIPERTENSIVA. CIENCIAS BASICAS: La
Hipertensin Arterial (HTA), por el hecho de ser una enfermedad sistmica, compromete rganos como el cerebro, el corazn, el rin y tambin el
ojo. La retinopata hipertensiva es la principal manifestacin ocular de la HTA, pero no la nica. El compromiso conjuntival, la coroidopata
hipertensiva y la neuropata hipertensiva son otras manifestaciones oculares de la HTA. Aunque parece haber una asociacin clara entre las
alteraciones retinieanas y el aumento de las cifras tensionales medias, parece que existen otros factores asociados a la retinopata como
arteriosclerosis, edad avanzada, enfermedad de la arteria cartida. SALUD PUBLICA: La prevalencia en pacientes hipertensos oscila entre 7.8-11%.
PATOGENIA: Las manifestaciones oculares de la HTA son producidas por el dao que esta causa en los vasos arteriales conjuntivales, retinales,
coroideos y de la cabeza del nervio ptico. La HTA produce vasoconstriccin y arterioloesclerosis. Vasoconstriccin: Los vasos arteriales retinales
poseen la capacidad de autoregulacin y no estn comandados por el tono simptico. Esta autoregulacin permite que en la eventualidad de un
aumento de la presin arterial, las arteriolas retinales se contraigan manteniendo constante el flujo. Por esto en pacientes con HTA Crnica, uno de
los hallazgos ms tempranos es una disminucin difusa del calibre arteriolar, perdindose la relacin AV normal de 2:3. La vasoconstriccin tambin
puede ser focal. Esta vasoconstriccin se produce por espasmo vascular reversible. En los casos de vasoconstriccin focal permanente, la causa
podra ser edema de la pared del vaso o reas localizadas de fibrosis. Arterioloesclerosis: Cuando observamos el fondo de ojo y vemos los vasos
retinales, lo que en realidad estamos viendo es la columna de sangre en el interior del vaso. La pared arteriolar es, en condiciones normales,
transparente. En la arterioloesclerosis se produce inicialmente un aumento del tejido muscular de la media y un aumento del tejido elstico de la
ntima, para luego ser reemplazada por fibras colgenas, la media, y sufrir engrosamiento hialino, la ntima. Esto ocasiona, en un primer momento,
que el brillo arteriolar que produce la luz incidente del oftalmoscopio en la pared del vaso, pierda su aspecto normal (delgado y en la mitad de la
columna de sangre), tornndose menos brillante, ms ancho y ms difuso. Este es el signo ms temprano de arterioloesclerosis. En una segunda

CURSO ENARM CMN SIGLO XXI TEL: 36246001

Pharmed Solutions Institute

PGINA 484

MANUAL DE TRABAJO DEL CURSO ENARM CMN SIGLO XXI


etapa, al aumentar la arterioloesclerosis, disminuye la visibilidad de la columna de sangre y el brillo arteriolar se torna caf-rojizo. Es lo que los
clnicos antiguos llamaban vasos en "Hilo de Cobre". En una tercera etapa, la pared del vaso pierde por completo su transparencia, perdindose la
visibilidad de la columna de sangre y adquiriendo el vaso un aspecto blanquecino. Esto se denominaba vasos en "Hilo de Plata". Aunque
clnicamente estos vasos parecieran estar sin flujo. En la Hipertensin Maligna se produce necrosis fibrinoide de las arteriolas retinales, aun cuando
la necrosis fibrinoide es ms comn en los vasos coroideos. En estos casos se puede apreciar una estrella macular, que es el aspecto en estrella que
adquiere la mcula por la acumulacin de macrfagos llenos de lpidos, dispuestos en forma radial, en la capa plexiforme externa de la retina.
DIAGNOSTICO: Manifestaciones oftalmoscopicas: En la hipertensin crnica se produce esclerosis arteriolar por el efecto mantenido de la presin
sangunea aumentada sobre las paredes vasculares. Se genera un estrechamiento generalizado o localizado, se ha descrito como arterias en hilo de
cobre, arterias en hilo de plata, aumento de la tortuosidad de las arteriolas esclerosadas. Se produce oclusin de arteriolas finas y generacin de
manchas isqumicas retinianas profundas denominadas infiltrados algodonosos. En la hipertensin aguda puede verse una imagen denominada
espasmo arteriolar. Trasudados periarteriolares focales intrarretinianos (TPIF): Son muy precoces y especficos de la hipertensin maligna. Tienen
forma redondeada u oval y el tamao es variable: desde puntiformes hasta la mitad del tamao del disco ptico. Son de color blanco mate cuando
son recientes y luego palidecen hasta resolverse. Se localizan en las zonas profundas de la retina. Las lesiones se desarrollan totalmente en 2-3
semanas y tras su resolucin no quedan secuelas oftalmoscpicas. Manchas isqumicas retinianas profundas (infiltrados algodonosos o exudados
blandos): Son reas focales, esponjosas, opacas. Tienen formas muy irregulares, comienzan como una pelcula griscea que se vuelve como una
nube blanca y se resuelve fragmentndose en manchas menores. Cambios capilares retinianos: microaneurimas y shunts arteriovenosos. Se
observan como vasos retorcidos y colaterales venosos. Cambios retinianos venosos: Signos de Gunn: la vena tiene dos zonas plidas, iguales a ambos
lados del cruce, con prdida del reflejo y adelgazamiento de los dos extremos. La vena se ve como partida. Signo de Salus: el curso venoso sufre un
cambio de trayecto. Aspecto de U, si el cruce es perpendicular y de S, si es oblicuo. Signo de Bonnet: junto al cruce se observa una hemorragia o
exudado. Hemorragias retinianas: tarda y poco frecuente, adoptan forma de llama. Edema retinal y macular: los signos precoces de papiledema son
rebosamiento de las venas, perdida de la pulsacin venosa, hiperemia de la cabeza del nervio y borrado de los mrgenes del disco. Depsitos
lipdicos retinianos (exudados duros): son amarillos, redondeados generalmente y de bordes bien delimitadosLa aguda: se caracteriza por
disminucin de la agudeza visual, la que puede ser desde visin borrosa (lo ms frecuente) hasta la ceguera. Caractersticamente esta disminucin
de agudeza visual es transitoria. El aspecto ms caracterstico del fondo de ojo es una Vasoconstriccin Generalizada, se pueden apreciar
Hemorragias Retinales, Manchas Algodonosas, Estrella Macular, Manchas de Elschnig y Edema de Papila. Con mucha menor frecuencia se aprecian
Desprendimientos de Retina Serosos. La crnica: los hallazgos ms caractersticos en el fondo de ojo son: Vasoconstriccin difusa, Aumento del Brillo
Arteriolar, Cruces AV Patolgicos, Microaneurismas y Macroaneurismas. En casos ms graves pueden apreciarse: Manchas Algodonosas, Edema de
Retina, Hemorragias Retinales, Estrella Macular, Manchas de Elschnig y Edema de Papila. Como podemos ver, en las manifestaciones clnicas de
casos severos se aprecian signos de Coroidopata Hipertensiva y Neuropata Hipertensiva, adems de los de la Retinopata Hipertensiva.
COROIDOPATA HIPERTENSIVA: La coroides, a diferencia de los vasos retinales, est comandada por el tono simptico. En la HTA la coroides sufre
fenmenos de isquemia, observndose lbulos coroideos sin perfusin debido a necrosis fibrinoide de los vasos. Estas zonas de ausencia de
perfusin coroidea producen focos de necrosis isqumica en el epitelio pigmentario retinal suprayacente, que se denominan Manchas de Elschnig.
La isquemia coroidea tambin es responsable de desprendimientos de retina serosos por producir isquemia en las capas externas de la retina. Las
manchas de Elschnig, en su etapa aguda, se aprecian como manchas amarillentas redondeadas en el epitelio pigmentario retinal. Posteriormente, en
su etapa crnica, se aprecian como lesiones redondeadas hiperpigmentadas en su centro y con un halo de hipopigmentacin en su periferia.
CASO CLINICO RETINOPATIA DIABETICA
Un hombre de 31 aos de edad, con antecedentes de diabetes
mellitus insulino-dependiente de 12 aos de evolucin con parcial
apego a tratamiento, con alcoholismo ocacional y tabaquismo un
paquete al dia, se presenta para su examen oftlmico. Historia
oftalmolgica fue notable por desprendimiento total de retina de su
ojo derecho y la fotocoagulacin con lser de su ojo izquierdo
debido a la retinopata diabtica y edema macular diabtico.
Examen de dilatacin del fondo del ojo derecho mostr etapa final
retinopata diabtica proliferativa con desprendimiento de retina
traccional. El ojo izquierdo tena la neovascularizacin extensiva y la
proliferacin fibrovascular de la retina secundaria a la retinopata
diabtica proliferativa avanzada. Estos nuevos vasos anormales
mostraron fuga extensa durante la angiografa con fluorescena.
PREGUNTA
Cual de los siguientes factores es de mayor importancia para el
desarrollo de la retinopata diabtica?
RESPUESTA
a.- Ms de 5 aos de evolucin de la DM.
b.- Tabaquismo positivo intenso.
c.- Progresin rpida de la enfermedad.
d.- Depuracin de creatinina menos de 60 ml/ minuto.
CASO CLINICO RETINOPATIA DIABETICA
Un hombre de 56 aos de edad con diabetes presenta una historia
de varios meses de disminucin de la visin en el ojo izquierdo. El
examen del ojo izquierdo revel la neovascularizacin retiniana
extensa superior a la del disco ptico.

CURSO ENARM CMN SIGLO XXI TEL: 36246001

PREGUNTA
Cual de los siguientes metas son factores que no modifican el curso
de la enfermedad?
RESPUESTA
a.- Glucosa menor a 130 mg/dl
b.- Hb glucosilada menor a 7.
c.- Presin arterial menor a 130/80 mmHg.
d.- Uso profilctico de triamcinolona.
CASO CLINICO RETINOPATIA HIPERTENSIVA
Femenino de 42 aos de edad, con hipertensin arterial de 10 aos
de evolucin actualmente con disminucin en el aclaramiento de
creatinina (<60 ml/min), con un evento de isquemia cerebral
transitoria, presentando alteraciones del ritmo cardiaco bajo
tratamiento, su nivel de glucosa fue de 210 mg/dl, Colesterol 324
mg, triglicridos 450 mg, hemoglobina glucosilada de 9.2, acudi a
consulta debido a que presenta diminucin de la agudeza visual, asi
como alteraciones en la campimetrial, fue enviada a evaluacin
oftalmolgica con el siguiente reporte. Estrechamiento y
constriccin focal, edema retiniano, exudados y hemorragias.
PREGUNTA
Considerando el reporte cual es el estadio de Keith-Wagener?
RESPUESTA
a.- Keith-Wagener 1.
b.- Keith-Wagener 2.
c.- Keith-Wagener 3.
d.- Keith-Wagener 4.

Pharmed Solutions Institute

PGINA 485

MANUAL DE TRABAJO DEL CURSO ENARM CMN SIGLO XXI


URGENCIAS OFTALMOLOGICAS, PERFORACION, GLAUCOMA
CIENCIAS BASICAS: Hay que distinguir dos grandes categoras de afecciones, ante un ojo rojo, el anlisis semiolgico debe pasar por el examen de
tres signos mayores: Dolor, fotofobia y agudeza visual. Normas generales: 1. No utilizar colirios anestsicos, salvo para facilitar la exploracin. 2. No
presionar el globo ocular traumatizado. 3. Tras el diagnstico de perforacin ocular, abstenerse de continuar con la exploracin; ocluir ambos ojos
con gasa estril sin instilar colirios ni aplicar pomadas. 4. Para la exploracin corneal es de gran ayuda la instilacin de colirio de Fluorescena al 2% y
posterior lavado con suero fisiolgico, ya que con la luz azul cobalto evidenciar lesiones y defectos epiteliales dando una coloracin amarilloverdosa. 5. Para una midriasis exploratoria (con colirio de Tropicamida) o teraputica en pacientes de edad y, sobre todo hipermtropes, se debe
explorar previamente el signo "del sol naciente". Este signo consiste en la proyeccin tangencial desde el lado temporal de un foco luminoso que
iluminar la mayor parte del iris ante una cmara anterior profunda. 6. Utilizar guantes desechables ante procesos traumticos o infecciosos y, en
todo paciente al que se le vayan a tocar los prpados. 7. No realizar oclusin ocular en conjuntivitis o queratoconjuntivitis, ni en lceras corneales
con infeccin local. Hay que sealar cinco procesos oculares que son de asistencia inmediata, ya que el pronstico en cuanto a la funcin visual y/o
Integridad del globo ocular, depender de la rapidez en el diagnstico y tratamiento: Oclusin aguda de la arteria central de la retina. Causticacion
ocular por alcalis. Herida perforante del globo ocular. Glaucoma agudo. Endoftalmitis. OJO ROJO NO DOLOROSO: 1. Hiposfagma (Hemorragia
subconjuntival): Habitualmente son espontneas, de tamao variable y curan sin tratamiento. Si es secundario a traumatismo ocular, buscar signos
de perforacin, descartarse siempre crisis de hipertensin arterial. 2. CONJUNTIVITIS: Bacteriana; mas frecuente bilateral. Existe secrecin purulenta
o mucopurulenta y lagrimeo, tx., aplicar colirio de Tobramicina cada 3 horas o aureomicina cada 2 horas, dura aprox 2 sem. Si es unilateral,
purulenta, con dolor y adenopata preauricular, debe pensarse en una conjuntivitis gonoccica y se tratar con colirio de Aureomicina cada 2 horas y
Penicilina por va parenteral. Si es unilateral, con poca secrecin, adenopata preauricular, presencia de folculos en conjuntiva tarsal, es decir, con
caractersticas ms propias de una conjuntivitis vrica, debe sospecharse conjuntivitis de inclusin (Chlamydia oculogenitalis). Vrica: unilateral o
bilateral en pocos das, poca secrecin, lagrimeo y presencia de folculos en las conjuntivas tarsales. Habitualmente, muestra adenopatas
preauriculares y submaxilares, cursar con fotofobia y dolor (queratoconjuntivitis). Etiologia los adenovirus, no existe tratamiento etiolgico eficaz,
pero es aconsejable el colirio de Aureomicina cada 2 horas y la pomada de Aureomicina por la noche. Alrgica: Cursa con secrecin blanquecina
fibrinosa (estril), abundante lagrimeo, presencia de papilas en la conjuntiva tarsal superior y, sobre todo, predomina el picor como sntoma
fundamental y caracterstico de esta afeccin. Es bilateral y, excepcionalmente, unilateral. Se trata con colirio de Dexametasona cada 6-8 horas y
antihistamnicos. OJO ROJO DOLOROSO: 1. EPISCLERITIS: Inflamacin de la episclera, enrojecimiento violceo, localizado en un sector de la
conjuntiva bulbar, por lo general con un ndulo amarillento en su centro. Realmente, la dilatacin vascular afecta tanto a la episclera como a la
conjuntiva. La instilacin de colirio de Fenilefrina no modifica esta congestin profunda, sino que blanquea slo una hiperemia puramente
conjuntival (diagnstico diferencial). Afecta a sujetos jvenes de forma recidivante. Cursa con dolor a los movimientos oculares y, sobre todo, a la
presin. Con frecuencia es de origen alrgico. A veces forma parte de enfermedades granulomatosas y reumticas. Rara vez es signo de tumor
intraocular. Tratamiento: Colirio de Dexametasona cada 4 horas. Si no es suficiente, se recurrir a la corticoterapia sistmica. 2. QUERATITIS.- La
infeccin o inflamacin de la crnea cursa con dolor (que ser ms intenso cuanto mayor sea la denudacin de las terminaciones nerviosas),
fotofobia, lagrimeo, blefarospasmo, inyeccin periquertica, deslustramiento de la crnea y alteracin dispar de la agudeza visual. A veces existe
hipopion (nivel de pus en la cmara anterior), que traduce una reaccin uveal anterior. Tratamiento: Ciclopljico: Colirio de Atropina al 1% cada 8
horas. Antibitico: Tpico: Cada hora. Gentamicina: 9.1 mg/ml. Ceftazidima: 50 mg/ml. Subconjuntival: Cada 12-24 horas. Gentamicina: 20 mg (0.5
ml.). 3. IRITIS O IRIDOCICLITIS (UVEITIS ANTERIOR): Se trata de una inflamacin del tracto uveal anterior (iris y cuerpo ciliar). Sus sntomas son:
Dolor ocular y periocular (irradiaciones en el territorio del trigmino), lagrimeo, fotofobia y disminucin de la agudeza visual (borrosidad). Signos:
Inyeccin periquertica, precipitados en la cara posterior de la crnea, enturbiamiento del humor acuoso (efecto Tyndall), iris deslustrado, pupila
oco reactiva a la luz y presencia de sinequias posteriores. La tensin ocular por regla general est disminuida, medida teraputica urgente
encaminada a mitigar el dolor y evitar o liberar las sinequias posteriores, debemos administrar un colirio ciclopljico cada 6-8 horas y un colirio de
Dexametasona cada 2 horas. 4. GLAUCOMA AGUDO: Cierre total y sbito del ngulo camerular, que ocasiona una elevacin severa de la presin
intraocular. Slo ocurre en ojos a menudo hipermtropes y nunca miopes, de crnea pequea, de cmara anterior estrecha y de ngulo estrecho.
Estas condiciones anatmicas y el aumento del volumen del cristalino a lo largo de la vida, son responsables del bloqueo pupilar. Es ms frecuente
en mujeres (4: 1) y ocurre, generalmente, a partir de los 65 aos. Pueden desencadenar la crisis las emociones, la anestesia general, los derivados
atropnicos, los simpaticomimticos (jarabes para la tos), los antidepresivos, etc. La forma tpica; unilateral, aunque el ojo congnere presentar las
mismas condiciones de predisposicin. Tener dolor ocular intenso, similar a una cefalea retrobulbar, acompaado de nuseas y vmitos. Visin
borrosa y percepcin de halos de colores en el inicio. Posteriormente, la visin se reduce a percepcin de luz. Al examen con la lmpara de
hendidura: Inyeccin ciliar y conjuntival (gran congestin). Deslustramiento corneal por el edema. Cmara anterior muy aplanada, casi inexistente.
Pupila en midriasis media, oval, vertical, arreactiva. La tensin ocular est muy elevada, por encima de 50 mmHg. ("dureza ptrea").
Tratamiento,inhibidores de la produccin del humor acuoso y agentes hiperosmticos. Colirio de Pilocarpina al 2%+ Colirio de Dexametasona cada 5
minutos durante la primera media hora y cada 10 minutos durante la hora siguiente para ir espaciando las instilaciones paulatinamente.
Acetazolamida, 500 mg. por va IM. o IV. La va IV. debe manejarse con cautela debido al contenido en Lidocana que lleva el preparado. Manitol al
20%: 250 cc. por va IV. (Aproximadamente 90 gotas por minuto). El tratamiento se comenzar al unsono con todos los frmacos y, pasadas 3-4
horas del inicio del mismo, se proceder a realizar una IRIDOTOMIA PERIFERICA LASER, aunque no se haya resuelto el bloqueo pupilar. Si el edema
corneal no permite visualizar bien el iris, presionaremos en la crnea central repetidas veces, a razn de 30 seg. cada una de ellas, para conseguir
entreabrir el ngulo camerular y facilitar el drenaje del humor acuoso, con lo que aumentar la transparencia corneal. TRAUMATISMOS OCULARES:
LESIONES NO PENETRANTES: ABRASIN CORNEAL: La instilacin de un colirio anestsico nos facilitar la apertura palpebral. El tratamiento se hace
con colirio de Tobramicina cada 4 horas, colirio ciclopljico cada 8 horas y oclusin semicompresivo. Si observamos erosiones superficiales lineales o
geogrficas en la mitad superior de la crnea, se procede a evertir el prpado superior y retirar el cuerpo extrao que se aloja en la conjuntiva tarsal.
Ante un cuerpo extrao enclavado en la crnea, se procede a su extraccin y a la limpieza de xido si lo tuviera, con una aguja hipodrmica o IM. con
poco bisel, siempre previa anestesia tpica y utilizando la lmpara de hendidura. QUEMADURAS FSICAS: Pueden ser ocasionadas por un cigarrillo.
En este caso, la zona corneal afecta se blanquea al instante. El tratamiento es el mismo que el de la abrasin corneal. Tambin pueden ser causadas
por radiaciones ultravioletas (queratoconjuntivitis actnica): Afectacin punteada superficial por estallido de las clulas epiteliales tras la exposicin
a los rayos UV. (arco voltaico, radiacin solar, aparatos UV.,etc.). QUEMADURAS QUMICAS: Son muy graves, en especial las quemaduras producidas
por lcalis y constituyen una causa frecuente de ceguera. Las quemaduras por cido ejercen su efecto nocivo durante las primeras horas, precipitan
rpidamente las proteinas tisulares levantando barreras contra su propia penetracin. Secundariamente, la neutralizacin por las proteinas del
tejido circundante, tiende a localizar la lesin en el rea de contacto. Los lcalis se combinan con los lpidos de las membranas celulares,
produciendo as la total desestructuracin de las clulas con reblandecimiento del tejido. CONTUSIONES OCULARES: Directas; Producidas por
balonazos, pelotazos, puetazos, etc. Son lesiones conjuntivales el Hiposfagma (H. subconjuntival). Una lesin corneal es la Queratitis traumtica por

CURSO ENARM CMN SIGLO XXI TEL: 36246001

Pharmed Solutions Institute

PGINA 486

MANUAL DE TRABAJO DEL CURSO ENARM CMN SIGLO XXI


afectacin parenquimatosa, con edema (deslustramiento) y pliegues o estras en la Descemet. Se tratar con colirio de Dexametasona cada 6 horas y
colirio ciclopljico cada 8 horas. Las lesiones del iris y del ngulo irido-corneal, implican con frecuencia hiphemas. Los Hiphemas consisten en la
presencia de sangre fresca en la cmara anterior. Pueden condicionar aumento de presin intraocular. Si es total y coexiste con hipertensin ocular,
puede aparecer una tincin hemtica de la crnea. Esta situacin exige la valoracin urgente por el oftalmlogo. Es ms peligroso el resangrado al
tercer- quinto da. El tratamiento consiste en el reposo en posicin de antitrendelemburg, oclusin binocular, colirio ciclopljico cada 8 horas, colirio
de Dexametasona cada 6 horas e hipotensores oculares si est aumentada la presin intraocular. La prednisona oral (1 mg/Kg/da), evita el
resangrado. En cuanto a las modificaciones pupilares, podemos encontrar irregularidades del contorno por ruptura del esfnter. Sin embargo,
hallaremos ms frecuentemente midriasis, nunca mxima, oval-oblicua que no responde a la luz, pero s a la convergencia. La miosis es rara. La iritis
e iridociclitis, ocurren por ruptura de la barrera hematoacuosa. Aunque mnima, siempre existe en cualquier contusin. Se tratar con colirio de
Dexametasona cada 6 horas y colirio ciclopljico cada 8 horas. Tambin pueden producirse modificaciones de la presin intraocular. Se puede
encontrar tanto hipertensin como hipotensin, en relacin con las diversas afectaciones. Debe sospecharse subluxacin o luxacin cristaliniana por
la iridodonesis (iris trmulo) y por la variacin de profundidad de la cmara anterior. La luxacin del cristalino a cmara anterior exige ciruga
urgente. En cuanto a la posible aparicin de catarata, inicialmente puede apreciarse alguna opacidad cristaliniana, aunque tardar habitualmente
das en formarse. El edema retiniano ocasiona una importante disminucin de la agudeza visual, sobre todo si afecta a la mcula. Al examen del
fondo de ojo, se aprecia un aspecto blanquecino- lechoso de la retina. Se trata con colirio ciclopljico cada 8 horas y reposo absoluto con oclusin
binocular. HEMORRAGIAS RETINOVTREAS: La afectacin de la agudeza visual depende de la extensin y localizacin de stas. Se tratar con colirio
ciclopljico cada 8 horas y reposo absoluto. Si la hemorragia vtrea es total, interesa realizar ecografa para determinar si coexiste un
desprendimiento de retina. Los agujeros y desgarros retinianos se producen ms frecuentemente en ojos miopes. Se tratan con laserterapia o
ciruga. Cuando se observa un desprendimiento de retina, existe amputacin de campo visual. Requiere ingreso del paciente, con reposo absoluto,
oclusin binocular y aplicacin de colirio de Atropina al 1% cada 8 horas. Se realizar intervencin quirrgica programada. Las rupturas coroideas son
concntricas a la papila. No requieren tratamiento. Sin embargo, si es posterior, debemos atender a los siguientes signos: Prdida brusca de la
agudeza visual inmediatamente despus del traumatismo, marcada hipotona ocular, aumento acusado de la profundidad de la cmara anterior con
iridodonesis, hemorragia vtrea masiva y hematoma subconjuntival de aspecto gelatinoso. Requiere ciruga urgente. Indirectas: Cuando el
traumatismo es de vecindad o a distancia (traumatismo craneal, aplastamientos torcicos, explosiones, fractura de huesos largos...), puede
condicionar hemorragia y exudados en el segmento posterior (angiopata traumtica de la retina y embolia grasa de la retina). LESIONES
PENETRANTES: En las heridas palpebrales, debe comprobarse si son transfixivas. No existe duda cuando se observa contenido intraocular
prolapsado, pero en el resto de los casos se deben buscar signos indirectos de perforacin. Estos signos son los siguientes: 1. Prdida de agudeza
visual. 2. Hipotona ocular. 3. Hundimiento o aplanamiento de la cmara anterior. 4. Quemosis marcada5. Laceracin corneal o escleral. Para
comprobar si existe disrupcin total, se proceder a instilar una gota de colirio de fluorescena y, con la luz azul cobalto, comprobaremos si la
fluorescena es apartada por la salida del humor acuoso, bien espontneamente o a la presin en casos de herida valvulada. Este signo se denomina
SEIDEL. 6. Hiposfagma: A veces uncuerpo extrao se impacta en el globo ocular atravesando la esclertica y quedando la puerta de entrada
enmascarada por una hemorragia subconjuntival. Nos cercioraremos con el signo de Seidel. Siempre que se asista a una herida penetrante en rbita,
se debe recurrir a exploraciones radiolgicas: Radiografa simple frontal y lateral de rbita, as como TAC, que nos descartarn o confirmarn la
existencia de cuerpo extrao. El tratamiento se basa en los siguientes puntos: Siempre exige asistencia especializada. Se evitarn las manipulaciones
innecesarias. Abstenerse de medicacin tpica, debiendo estar ambos ojos ocluidos con no compresivo. Administracin de analgsicos. Profilaxis
antitetnica. Profilaxis antibitica por va IV: Gentamicina, 80 mg. cada 8 horas y Ceftazidima, 1 g. cada 8 horas. Reparacin quirrgica bajo anestesia
general
CASO CLINICO PERFORACION CORNEAL
Varn de 83 aos con enfermedad de Alzheimer avanzada, diabtico
insulinodependiente de 35 aos de evolucin, que acudi al servicio
de urgencias por presentar cuadro de dolor ocular en ojo derecho de
2 das de evolucin. Se envio al servicio de oftalmologa que refiere
que el segmento anterior presenta lesin ulcerativa corneal de
2,5mm de dimetro con un poro endotelial perforante central de
aproximadamente 1mm, sin proceso infeccioso o inflamatorio
concomitante. El paciente no presentaba antecedentes traumticos.
PREGUNTA
Cul es la conducta a seguir en esta paciente?
RESPUESTA
a.- Cubrir el ojo.
b.- Colgajo pediculado, tipo Gundersen.
c.- Autoplastia libre conjuntival.
d.- Queratoplastia penetrante.
CASO CLINICO GLAUCOMA DE ANGULO CERRADO
Paciente de 42 aos que acude a Urgencias por cuadro de
disminucin de agudeza visual (AV) de un da de evolucin. Como
antecedentes personales destaca trastorno depresivo en
tratamiento. No presentaba antecedentes oftalmolgicos de inters
y negaba la utilizacin previa de gafas. Su tratamiento consiste en
olanzapina 10mg, lorazepam 2mg y venlafaxina 150mg retard desde
haca 12 meses. Una semana antes haba comenzado a tomar
topiramato para perder peso. Al examen se presenta edema corneal
y quemosis conjuntival, una cmara anterior estrecha y cierre
angular evidenciado mediante gonioscopia. La presin intraocular
(PIO) es de 55mmHg en AO.

CURSO ENARM CMN SIGLO XXI TEL: 36246001

PREGUNTA
Cul es la conducta a seguir en esta paciente?
RESPUESTA
a.- Maleato de timolol.
b.- Manitol 3 g.
c.- Pilocarpina al 3 %.
d.- Acetazolamida 10 mg.

CASO CLINICO DIPLOPIA Y PTOSIS


Mujer de 34 aos de edad que inici de forma progresiva durante
dos semanas visin doble binocular, ptosis palpebral de ambos
prpados superiores y cefalea. En el examen neurolgico, se objetiv
una parlisis bilateral del nervio motor ocular comn, con afectacin
pupilar en el ojo derecho; el resto del examen neurolgico fue
normal. El estudio mediante resonancia magntica (RM) mostr la
presencia de mltiples focos en la sustancia blanca de hiperseal en
las secuencias basadas en T2, compatibles con lesiones de origen
desmielinizante; una de estas lesiones afectaba a la lnea media del
tegmento del mesencfalo, y se extenda anteriormente a la fosa
interpeduncular y posteriormente hasta el acueducto de Silvio. Esta
lesin, que presentaba realce en las secuencias T1 tras la
administracin intravenosa de gadolinio, era probablemente la
responsable de los sntomas, ya que afectaba a la localizacin de
ambos ncleos del III par craneal.
PREGUNTA
Cul es la conducta a seguir en esta paciente?
RESPUESTA

Pharmed Solutions Institute

PGINA 487

MANUAL DE TRABAJO DEL CURSO ENARM CMN SIGLO XXI


a.- Acetato de glatirmero.
b.- Interfern (1a o 1b).
c.- Azatioprina.
d.- Natalizumab.
CASO CLINICO
Paciente varn de 36 aos sin antecedentes familiares ni personales
de inters que acude a urgencias por presentar ojo rojo y dolor
ocular y fotofobia. En la exploracin oftalmolgica presenta una
agudeza visual en ojo derecho de 0.9 y en ojo izquierdo de 0.4. Su
motilidad extrnseca era normal, presentando una inyeccin
periquertica intensa, precipitados endoteliales finos y difusos,
tyndall ++, PIO de 46 mmHg y una hemorragia prerretiniana en
arcada superior sin vitritis asociada.
PREGUNTA
Cul es el diagnstico ms probable para este caso?
RESPUESTA
a.- Glaucoma
b.- Iridociclitis
c.- Uveitis
d.- Queratitis

PREGUNTA
Cul es la causa ms probable del cuadro clnico de este paciente?
RESPUESTA
a.- Diabetes mellitus
b.- Uso de esteroides
c.- Hipertensin arterial
d.- Uso de gentamicina
PREGUNTA
Cul es la conducta teraputica ms adecuada para disminuir
sbitamente la PIO?
RESPUESTA
a.- Latanoprost
b.- Pilocarpina
c.- Acetazolamida
d.- Manitol

PREGUNTA
Cul es la conducta teraputica ms adecuada para este caso?
RESPUESTA
a.- Esteroides tpicos
b.- Aciclovir
c.- Ceftriaxona
d.- Metotrexate
PREGUNTA
Cul sera la causa ms probable del diagnstico en este paciente?
RESPUESTA
a.- Espondilitis anquilosante
b.- Idiopatica
c.- Bacteriana
d.- Reiter

CURSO ENARM CMN SIGLO XXI TEL: 36246001

CASO CLINICO
Presentamos el caso de un paciente varn de 58 aos, con
antecedentes de diabetes mellitus e hipertensin arterial, que acude
a urgencias de oftalmologa por prdida de visin OI, asi como dolor
agudo en mismo ojo. Refiere molestias oculares que trata con colirio
de Gentamicina y Dexametasona desde hace un ao. A la
exploracin se objetiva PIO (OD) 42 mmHg y PIO (OI) 64 mmHg, ojo
en midriasis arreactiva. En campimetra muestra dao glaucomatoso
moderado en OD y severo en OI. En fondo de ojo presenta
neuropata glaucomatosa en AO con E/P 100% en OI.

PREGUNTA
Cul es el tratamiento quirrgico de eleccin para este paciente, si
no responde a tratamiento tpico mximo?
RESPUESTA
a.- Trabeculotomia
b.- Goniotomia
c.- Vlvula de Anet
d.- Cirugia Lasser

Pharmed Solutions Institute

PGINA 488

MANUAL DE TRABAJO DEL CURSO ENARM CMN SIGLO XXI


INGLES
CASE REPORT
A recently married 29-year-old nulliparous African-American female
presents with uncomplicated cystitis. She is otherwise healthy. She
reports that she is currently using barrier birth control methods but
plans to discontinue this soon, as she would like to become
pregnant.
QUESTION
Which one of he following supplemental vitamins or minerals would
you advise this patient to begin taking?
ANSWER
a) Thiamine
b) Iodine
c)
Folate
d) Calcium
QUESTION
Which one of the following is true regarding testing the rotator cuff
for injury?
ANSWER
a) The subscapularis is tested with lateral rotation against
resistance
b) The teres minor is tested with medial rotation against
resistance
c)
The infraspinatus is tested with medial rotation against
resistance
d) The supraspinatus is tested with abduction against resistance
CASE REPORT
You have recently begun caring for a 25-yerar-old white female who
has multiple complaints. You have seen her 3 times for walk-in office
visits over the past month. She has shown appreciation for your
work during the enconunters, but has been critical of your care
when talking to the office staff. At times she has been kind and
charming, and at other times she has been rude and verbally abusive
to your staff. She has a string of multiple relationships in the past,
none of wich has lasted very long. Durin times of intense stress, she
has sometimes engaged in self-mutilation. She frequently changes
jobs and living arrangements.
QUESTION
Which one of the following strategies would be most appropriate in
the care of this patient?
ANSWER
a) Strive to develop a close relationship with the patient.
b) Ignore verbal attacks on staff members.
c)
Provide detailed, technical explanations for. any therapies
provided.
d) Schedule frequent office visits for follow-up.
QUESTION
In an 11-year-old male with dark brown urine and hand and foot
edema. Which one of the following would be most suggestive of
glomerulonephritis?
ANSWER
a) RBC casts in the urine
b) Elevated C3 and C4 complement levels
c)
Eosinophils in the urine
d) Positive serum antinuclear antibody levels
A 32-year-old white female at 16 weeks gestation presents to your
office with right lower quadrant pain.
QUESTION
Which one of the following imaging studies would be most
appropriate for initial evaluation of this patient?
ANSWER

CURSO ENARM CMN SIGLO XXI TEL: 36246001

a)
b)
c)
d)

MRI of the Abdomen


Intravenous pyelography
CT of the abdomen
Ultrasonography of the abdomen

QUESTION
Which one of the following is the leading cause of blindness in
individuals over age 65?
ANSWER
a) Narrow angle glaucoma
b) Diabetic retinopathy
c)
Macular degeneration
d) Ophthalmic artery occlusion
CASE REPORT
A 48-year-old white female complains of anxiety and difficulty
concentrating at home and at work. She reports that the symptoms
have increased over the last 2 months because of her daughters
marital difficulties. She has had similar symptoms along with
intermittent depression since she was a teenager. She admits to a
loss of pleasure in work and recreational activities.
QUESTION
Which one of the following is LEAST likely to help her coexistent
depressive symptoms?
ANSWER
a) Nortriptyline.
b) Buspirone.
c)
Escitalopram
d) Venlafaxine
CASE REPORT
An 82-year-old white female visits your office. She reports a 2-week
history of shoulder stiffness that is severe in the morning, and a 3day history of pain on the left side of the face while chewing food.
She denies dental pain or sensitivity. Her family history is negative
for rheumatologic disorders. Your evaluation reveals a 5-lb weight
loss and evidence of mild depression. Her temperature is 38.0 C
(100.4 F), she has no dental decay or carotid bruit, and her left
temporal scalp is tender. Her lungs are clear to auscultation, there
are no abnormal heart sounds, and her abdomen is nontender. She
has no joint swelling or warmth, but experiences discomfort with
shoulder and hip range of motion. Laboratory analysis reveals a
hemoglobin level of 11.0 g/dL (N 11.716.1), an erythrocyte
sedimentation rate of 80 mm/hr (N 030), a serum glucose level of
120 mg/dL, and a BUN level of 24 mg/dL (N 823).
QUESTION
Which one of the following is the greatest immediate risk for this
patient?
ANSWER
a) Monocular vision loss
b) Hemiparesis
c)
Sudden death
d) Adhesive capsulitis of the shoulder
CASE REPORT
A 73-year-old male with COPD presents to the emergency
department
with
increasing
dyspnea.
Examination reveals no sign of jugular venous distention. A chest
examination
reveals
decreased
breath sounds and scattered rhonchi, and the heart sounds are very
distant
but
no
gallop
or
murmur
is noted. There is +1 edema of the lower extremities. Chest
radiographs
reveal
cardiomegaly
but
no pleural effusion. The patients B-type natriuretic peptide level is
850
pg/mL
(N
<100)
and
his
serum creatinine level is 0.8 mg/dL (N 0.61.5).

Pharmed Solutions Institute

PGINA 489

MANUAL DE TRABAJO DEL CURSO ENARM CMN SIGLO XXI


QUESTION
Which one of the following would be the most appropriate initial
management?
ANSWER
a) Prednisone, 20 mg twice daily for 1 week
b) Furosemide, 40 mg intravenously
c)
Intravenous heparin
d) Tiotropium
CASE REPORT
A 28-year-old primigravida is at 20 weeks gestation by dates but her
fundal height is consistent with a 26-week gestation. She has had
episodes of vomiting during the pregnancy that were more severe
than the physiologic vomiting typically seen in pregnancy. A
sonogram performed at about 5 weeks gestation for vaginal
bleeding was normal and showed a single fetus.
QUESTION
Which one of the following would be most appropriate at this point?
ANSWER
a) Expectant management
b) A repeat sonogram
c)
MRI of the pelvis
d) A serum hCG leve

CASE REPORT
A 24-year-old male, new to your practice, presents for a mental
health evaluation. The patient has a past history of schizophrenia,
diagnosed several years ago.
QUESTION
Which one of the following, if present, would lead to a
reconsideration of this diagnosis?
ANSWER
a) Loose associations
b) Elated mood
c)
Social dysfunction
d) Auditory hallucinations
CASE REPORT
A previously healthy 22-year-old female presents for her regular
prenatal checkup at 38 weeks gestation. She has a blood pressure of
145/95 mm Hg today and this is unchanged 1 hour later. Her blood
pressure was normal before pregnancy. She is otherwise feeling
well. She has moderate edema at the ankles and 3+ reflexes at the
knees and ankles. A urinalysis for protein is normal.
QUESTION
Given this presentation, which one of the following is the most likely
diagnosis?
ANSWER
a) Hemolysis, elevated liver enzymes, low platelets syndrome
b) Preeclampsia
c)
Essential hypertension
d) Gestational hypertension
CASE REPORT
A 72-year-old female with longstanding diabetes mellitus presents to
your office. During the review of systems, she complains of difficulty
voiding and frequent dribbling. A urinalysis is negative for infection
and her post-void residual volume is 250 mL.
QUESTION
Which one of the following is the most likely cause of this patients
urinary incontinence?
ANSWER

CURSO ENARM CMN SIGLO XXI TEL: 36246001

a)
b)
c)
d)

A grade II cystocele
Atrophic vaginitis
Asymptomatic bacteriuria
Autonomic neuropathy

CASE REPORT
A 3-year-old male is brought to your office the day after he was
stung by a honeybee. He has developed a significant local reaction,
with redness and swelling around the site of the sting on his
forearm, and also has had some swelling of his lips for a couple of
hours. His mother removed the stinger and gave him some oral
diphenhydramine The local reaction has now almost resolved and he
has not had any hives or respiratory distress.
QUESTION
Which one of the following is true concerning this situation?
ANSWER
a) An antibiotic that covers Streptococcus pyogenes and
Streptococcus aureus should be administered
b) If venom immunotherapy is begun, it should be stopped after 2
years of treatment
c)
The child is at high risk for a systemic reaction if he is stung
again in the future
d) Skin tests are not helpful in confirming the presence of insect
sting allergy
CASE REPORT
A 65-year-old white female develops a burning pain in the left lateral
thorax, followed 2 days later by an erythematous vesicular rash.
QUESTION
Of the following, the best treatment is?
ANSWER
a) topical capsaicin (Zostrix)
b) oral corticosteroids
c)
topical corticosteroids
d) topical acyclovir (Zovirax)
CLINIC CASE
A patient on peritoneal dialysis has received six (6) units of red blood
cells for a bleeding duodenal ulcer. Dialysis was discontinued 3 day
ago and an emergency gastric surgery is now necessary. The patient
arrests following induction of general anesthesia whit propofol and
succinylcholine.
QUESTION
Which of the following is the likely cause of this cardiac event?
ANSWERS
a.- Hypothermia.
b.- Hypovolemic shock.
c.- Hypocalcemia.
d.- Hyperkalemia.
CLINIC CASE
A 79 year old man with a one pack for day smoking history is
incidentally found to have a right lower lung mass with moderate,
free-flowing, right-side pleural effusion. On further questioning he
admits to some symptoms of dyspnea and a 20-pound weight loss
over 3 months, but denies other symptoms.
QUESTION
What should be done next?
ANSWERS
a.- Mediastinoscopy.
b.- Bronchoscopy.
c.- Thoracentesis.
d.- CT of the chest.

Pharmed Solutions Institute

PGINA 490

MANUAL DE TRABAJO DEL CURSO ENARM CMN SIGLO XXI


CLINIC CASE
A 21 year old woman with a history of cystic fibrosis is admitted to
the ICU for pneumonia. Her CXR shows right lower lung zone dense
consolidations with pleural effusion, in addition to previous seen
bronchiectasis. She has no drug allergies, and her previous
respiratory culture results are not available at this time.

ANSWER
a.- A sharp demarcation at the area of obstruction.
b.- Gray thick-walled small bowel matted together.
c.- Multiple areas of stricture in normal appearing small bowel.
d.- Inter loop abscesses and fistula.

QUESTION
What empiric antibiotics should be started?
ANSWERS
a.- Imipenem.
b.- Ceftriaxone and metronidazole.
c.- Cefepime.
d.- Piperacilin and Tazobactam.

CLINIC CASE
A 75 years old female is evaluated for reflux esophagitis. At
endoscopy the mucosa is salmon pink with a definite demarcation
between normal and abnormal esophageal mucosa. Biopsy
indicates no dysplasia. Three weeks following a Nissen
Fundoplication, the patient has a severe episode of retching
following a heavy meal and has vomiting with blood. Two hours
later, he has severe epigastric pain and a spiking temperature of 102
F.

CLINIC CASE
A 66 year old man complains of gradual onset of dyspnea. His PFT
shows: FEV1/FVC 104, FEV1 73% predicted, and FVC 70% of
predicted, TLC 66% of predicted, and DLCO 52% of predicted. The
patient denies any known medical problems. He has smoked a few
cigarettes a day since age 18, and reports that he use to work in a
shipyard from age 18 to 58. On exam his vitals are normal, and the
lung exam is significant for some bibasular crackies. No clubbing of
the fingernails is seen.

QUESTION
Which of the following is the problem?
ANSWER
a.- The Nissen has slipped.
b.- Fundoplication disruption has occurred with perforation.
c.- A perforated ulcer has developed.
d.- Acute pancreatitis.
CLINIC CASE
A 62 year old male presents with dyspepsia. An UGI shows
thickening of the rugal folds, at upper endoscopy, no mass or lesion
is identified. The gastric wall is thickened but no ulceration is noted.

QUESTION
What should be done next to evaluate this patients dyspnea?
ANSWERS
a.- Transthoracic echocardiogram.
b.- Bronchoscopy for transbronchial biopsy.
c.- High resolution CT scan of the chest.
d.- Right heart catheterization.
CLINIC CASE (HYPOMAGENSEMIA)
A 70 year old patient with hypercalcemic crisis has an emergency
resection of a large parathyroid adenoma. Immediately post
operation, the patient develops convulsions. Calcium IV is
administered with minimal response.
QUESTION
What is the etiology of the convulsions?
ANSWER
a.- Hypocalcemia.
b.- Hypernatremia
c.- Cerebral ischemia.
d.- Hypomagensemia.
CLINIC CASE
A 45 year old male weighing 80 kg is seen in the ER for two episodes
of hematochezia within the last 2 hours. He has pulse of 80 bpm, a
BP of 100/60 mmHg, a respiratory rate of 30, and urine output of 20
ml. he is anxious and confused.
QUESTION
What is his estimated blood loss?
ANSWER
a.- 500 ml
b.- 1000 ml.
c.- 2000 ml.
d.- 3000 ml.
CLINIC CASE
A 60 year old woman, who was previously treated with radiation
therapy for uterine cancer at age 50, presents with a small bowel
obstruction.
QUESTION
Intraoperatively, one would expect to see?

CURSO ENARM CMN SIGLO XXI TEL: 36246001

QUESTION
Following endoscopic biopsy, this patient is best treated by?
ANSWER
a.- Radiation.
b.- Chemotherapy.
c.- Chemotherapy and radiation.
d.- Subtotal gastrectomy.
CLINIC CASE
Following an open cholecystectomy and common duct exploration
for multiple gallstone in a 75 year old female, the colangiogram
obtained 10 days post operatively reveals an impacted 1.5 cm
gallstone at the ampulla. The bilirubin is 5.0.
QUESTION
What is the procedure of choice?
ANSWER
a.- Repeat the cholangiogram in 1 week.
b.- T-tube flush with Capmul 8210.
c.- Open sphincteroplasty.
d.- Choledochoduodenostomy.
CLINIC CASE
A 46 year old male has had intermittent flares of ulcerative pancolitis
for 10 years. He has increasing diarrhea, blood per rectum, and
fever. He responds to medical management.
QUESTION
Proctocolectomy is indicated for?
ANSWER
a.- Length of colon involved.
b.- Long periods of persistent disease.
c.- Severe dysplasia on initial biopsy.
d.- Persistent stricture.
CLINIC CASE
A 75 year old woman suffered from sudden onset left eye blindness.
No pain was associated with her blindness, and no other
abnormalities were seen in the neurological examination.

Pharmed Solutions Institute

PGINA 491

MANUAL DE TRABAJO DEL CURSO ENARM CMN SIGLO XXI


c.- Admit to the hospital.
d.- No further intervention.

QUESTION
What is the most likely diagnosis?
ANSWER
a.- Small lacunar stroke involving the occipital lobe.
b.- Ischemic lesion involving the lateral geniculate nucleus.
c.- Compressive lesion on the optical chiasm.
d.- Retinal ischemia due to retinal artery occlusion.

CLINIC CASE
A 65 year old make with pneumonia and septic shock develops acute
kidney injury. He has been running a low blood pressure (80/45
mmHg) on several vasopressor agents. Urine output start to fall and
his creatinine begins to rise. His urine sodium is high; he has no
proteinuria.

CLINIC CASE
A 51 year old male started to have a gradual decline in his
intellectual functions associated with depressive symptoms and
personality changes. The patient obtained a score of 23/30 on minimental status examination and on physical examination chorea was
noted. The diagnosis of Huntingtons disease was suspected.
QUESTION
What is typical inheritance pattern of Huntingtons disease?
ANSWER
a.- X-linked
b.- Autosomal dominant.
c.- Autosomal recessive.
d.- No familial pattern.

QUESTION
The most common cause of his kidney failure in this setting is:
ANSWER
a.- Obstruction.
b.- Acute tubular necrosis.
c.- Acute interstitial nephritis.
d.- Goodpastures disease.
CLINIC CASE
A 54 yearold woman reports a bothersome cough to 6 months
duration, She has never smoked, and denies history of wheezing or
heartburn. She has diabetes and hypertension, and has been on
metformin and lisinopril for the past 3 years.

CLINIC CASE
A 36 year old woman was referred to the neurology clinic to be
evaluated for her progressing headaches. The headaches were
described as dull, mild to moderate in severity, and located on the
left side of the head.
The patient has no family history of migraine headaches.
Neurological examinations was mostly normal except for right-sided
hyperreflexia.

QUESTION
What is the next appropriate step in management?
ANSWER
a.- Discontinue lisonopril.
b.- Start a proton pump inhibitor.
c.- Esophageal pH monitoring.
d.- Barium swallow evaluation.
CLINIC CASE
A 48 year old man presents with hemoptysis. He reports that he has
also had bloody nasal discharge over the past month. His CXR shows
bilateral nodules. Laboratory data is notable for creatinine of 4.7
mg/dl, and urine analysis shows 250 RBC/hpf

QUESTION
What is the most likely diagnosis?
ANSWER
a.- Left middle cerebral artery aneurysm.
b.- Arterio-Venous malformation.
c.- Vascular brain tumor.
d.- Sagittal sinus thrombosis.
CLINIC CASE
A 25 year old man with HIV presented to the ED with right-side
weakness and confusion. An MRI brain showed a few ring enhancing
lesions. The differential diagnosis is narrowed to toxoplasmosis and
primary CNS lymphoma.
QUESTION
What is the appropriate next step of management?
ANSWER
a.- Open brain biopsy.
b.- Stereotectic brain biopsy.
c.- Empiric treatment of toxoplasmosis with reimaging to evaluate
the response.
d.- High dosis steroids.

QUESTION
Which of the following serum studies should be ordered next?
ANSWER
a.- ANCA, anti-GBM antibodies.
b.- Anti-dis-DNA and anti-cardiolipin antibodies.
c.- ESR, C3, C4 leves
d.- RF, anti-citrulline antibodies.
CLINIC CASE
A 62 year old woman presents for evaluation of dizziness. It occurs
often when shes reaching overhead into her cupboards. She also
notice that her face has been swollen, and she has recently
developed a cough. On exam, she is noted to have some fullness in
her neck, but the rest of the physical exam is unremarkable. A CXR
shows a sharply circumscribed, dense anterior mediastinal mass that
displaces the trachea to the left.

CLINICAL CASE
A 32 year old primigravida presents to her obstetrician for routine
follow-up at 34 weeks gestation. She has a sonogram of the
developing baby which also shows that she has developed right
sided moderate hydronephrosis, Serum creatinine concentration is
checked and is 0.8 mg/dl. She has no symptoms. Urinalysis normal.

QUESTION
What is the next appropriate step in management?
ANSWER
a.- Radiation therapy.
b.- Radioactive iodine scanning.
c.- Referral for chemotherapy.
d.- Referral for surgical resection.

QUESTION
At this point, you should:
ANSWER
a.- Send urine for culture assuming infection.
b.- Consult urology immediately.

CLINIC CASE
A 38 year old woman with a history of asthma has had multiple
admissions in the past year for asthma exacerbation. She was
treated during the most recent hospitalization for a communityrequired pneumonia that was seen on CXR. She comes to you for

CURSO ENARM CMN SIGLO XXI TEL: 36246001

Pharmed Solutions Institute

PGINA 492

MANUAL DE TRABAJO DEL CURSO ENARM CMN SIGLO XXI


post-discharge follow-up. She reports she is compliant with her
combination steroid/long-acting beta agonist inhaler, but still needs
her rescue inhaler several times a day, and still has a cough
occasionally productive of brown plug. Her blood work on the day of
discharge was only significant for an eosinophilic count of 750/mm3.
QUESTION
Which of the following serum studies should be considered?
ANSWER
a.- Chronic eosinophilic pneumonia.
b.- Allergic brochopulmonary aspergillosis.
c.- Steroid non responsive asthma.
d.- Viral bronchitis.
CLINIC CASE
A 70 year old man who previously worked in stonecutting has had
bilateral upper lung zone interstitial markings and small nodules on
his CXR for several years. However, on his most recent CXR, it is
noted that some of the nodules in the right upper lobe have
coalesced into a mass, and cavitation is noted.
QUESTION
What is the next appropriate step in management?
ANSWER
a.- Sputum induction to rule out tuberculosis.
b.- Bronchoscopy with transbronchial biopsy to rule out malignancy.
c.- Refer to thoracic surgery for resection.
d.- Start antibiotics for treatment of community acquired
pneumonia.
CLINIC CASE
A 60 year old man with severe COPD is found to be difficult to
arouse after undergoing an outpatient, elective screening
colonoscopy. The following lab data are available: ABG on room air
pH 7.24, PaO2 64, PaCO2 78, Electrolytes: Na 140, HCO3 35, Cl 100.
QUESTION
What is his current acid-base status?
ANSWER
a.- Metabolic acidosis with respiratory compensation.
b.- Acute respiratory acidosis with metabolic compensation.
c.- Acute on chronic respiratory acidosis
d.- Chronic acidosis with metabolic compensation.
CLINIC CASE
A 50 year old man recently sustained a fall while shoveling snow;
Four weeks later he began to experience shortness of breath, rightsided chest discomfort, and a dry cough. He also feels extreme
fatigue and feels less productive at work. The patient has no other
significant past medical history and does not smoke. Other than
using the indoor hot tub in his condominium community three times
a week since his fall, there are no changes in his usual activities.

What antibiotic should be started?


ANSWER
a.- Doxycycline.
b.- Trimethoprim/Sulfamethoxazole.
c.- Cephalexine.
d.- Amoxicillin/Clavulanate.

CLINIC CASE
A 46 year old woman with a history of asthma calls you for a refill of
her albuterol inhaler. Her asthma has always been well-controlled on
a daily dose of moderate-dose inhaled corticosteroid and shes long
misplaced her albuterol inhaler. However 3 weeks ago she had an
upper respiratory tract infection. All her acute symptoms have
resolved but since then she has felt more wheezing, and often
coughs at night. Shes been using her daughters albuterol inhaler
and finally decides to call you to get her own prescription.
QUESTION
What is the next appropriate step in management?
ANSWER
a.- Prescribe a course of oral fluoroquinolone.
b.- Prescribe albuterol inhaler for as needed rescue use.
c.- A short course of oral corticosteroid therapy.
d.- Double her moderate-dose inhaled corticosteroid therapy.
CLINIC CASE
A 52 year old Hispanic female is seen for consultation in the hospital.
She states that she had a complete physical with laboratory testing 2
months earlier that was completely normal. She feels poorly now for
the past few weeks. Her legs have become swollen and she is having
shortness of breath. Laboratory testing reveals a positive ANA 1:360,
an elevated ds-DNA low C3 and C4 leves, a 24 hour urine protein of
2.5 grams with significant RBCs on her urinalysis, and renal failure.
An echocardiogram shows a significant pericardial effusion, but no
evidence for tamponade. A kidney biopsy reveals diffuse
proliferative lupus nephritis.
QUESTION
Which of the following is the most appropriate therapy at this time?
ANSWER
a.- Cyclophosphamide.
b.- Azothioprine.
c.- Solumedrol.
d.- Plasmaferesis.
CLINIC CASE
A 44 year old Hispanic female presents with inflammatory arthritis of
one of her wrists and MCPs interstitial lung disease, a non specific
rash, and Raynauds phenomenon. Lab test reveal and alevated
AntiNuclear Antibody 1:640, Rheumatoid Factor of 50 (Normal < 14),
and a normal anti-U1 RNP.

QUESTION
What is the most likely diagnosis?
ANSWER
a.- Sarcoidosis
b.- Aspergillosis
c.- Hypersensitivity pneumonitis due to mycobacterium avium.
d.- Active Infection with mycobacterium avium.

QUESTION
Which of the following is the diagnosis?
ANSWER
a.- Undifferentiated connective tissue disease.
b.- Mixed connective tissue disease.
c.- Systemic lupus erythematosus.
d.- Rheumatoid arthritis.

CLINIC CASE
A 48 year old man presents with a week of fever, cough, and
malaise. He has no significant past medical history and he takes no
medications. His vital signs are normal but an exam reveals right
sided crackies. A CXR shows a right lower lobe infiltrate.

CLINIC CASE
A 30 year old patient with sjogrens syndrome is placed on a
cytotoxic agent as well as oral steroids. At 3 months she begins to
have right thigh and knee pain that is increased with walking and
standing. Physical examination reveals normal range of motion of
her knee, but mild pain on internal and external rotation of her hip.

QUESTION

CURSO ENARM CMN SIGLO XXI TEL: 36246001

Pharmed Solutions Institute

PGINA 493

MANUAL DE TRABAJO DEL CURSO ENARM CMN SIGLO XXI


QUESTION
What is the most sensitive and appropriate test to diagnose his
condition early?
ANSWER
a.- Plain film radiography.
b.- Technetium-99m bone scanning.
c.- Magnetic resonance imaging.
d.- CT scan.
CLINIC CASE
A 32 year old white female with systemic lupus erythematosus is 32
week pregnant and presents to your office with a headache. Physical
examination is remarkable for a blood pressure of 200/110 mmHg.
Laboratory studies show a hemoglobin of 8.0 g/dl, platelet count of
38,000, a low haptoglobin level, and an elevated LDH, AST, and ALT.
QUESTION
What test would be best to determine the cause of her problems?
ANSWER
a.- Complement levels.
b.- Erythrocyte sedimentation rate.
c.- Anti-Smith antibody.
d.- C-reactive protein.

QUESTION
What pharmacological agent would be most appropriate for her?
ANSWER
a.- Prednisone.
b.- Hydroxychloroquine.
c.- Cyclophosphamide.
d.- Metotrexate.
CLINIC CASE
A 70 year old mal presents with weakness and low grade
temperature for a few month. He has no Raynauds. He has
tightening and erythema of the skin of his upper arms and legs with
prominence along the course of the superficial veins. There is
considerable muscle weakness. Laboratory testing reveals a normal
CK, ANA, RF and a significantly elevated CRP and ESR. A CBC reveals
a hemoglobin of 11.2 g/dl with a moderate increase in monocytes
and eosinophils.
QUESTION
Which of the following is the most likely diagnosis?
ANSWER
a.- Parasitic Involvement.
b.- Occult neoplasm.
c.- Alergic reaction.
d.- Eosinophillic fasciitis.
CLINIC CASE
A 29 year old male has lower back pain without radiation for the
past 3 weeks. He does not recall any specific instigating event. Pain is
severe and worse with sitting as well as standing. His neurological
examinations is unremarkable.

CURSO ENARM CMN SIGLO XXI TEL: 36246001

CLINIC CASE
A 69 year old morbidly obese white female states that she has
severe disabling right knee pain. The pain has been ongoing for the
past two weeks and has been keeping her awake at night. She does
not recall any preceding trauma and has been using a pillow
between her knees with some benefit. Physical examination reveals
crepitus and mild tenderness of the knee with severe pain 4 cm
below the medial joint line.
QUESTION
What is the most appropriate next step?
ANSWER
a.- X-ray of the knee.
b.- MRI of the Knee.
c.- Corticosteoid injection.
d.- Local supportive therapies.
CLINIC CASE
A 76 year old black male with a history of chronic gout on uric acid
lowering therapy and severe Class IV renal disease presents with
multiple swollen joints. Aspiration of his knee reveals uric acid
crystals intracellularly.

CLINIC CASE
A 24 year old female has diagnosed systemic lupus erythematosus
for the past 5 years. She has had episodes of discoid rashes as well
as polyarthritis and mild hematologic abnormalities, but none in the
past year.

QUESTION
Which of the following is the most appropriate action at this time?
ANSWER
a.- Check x-ray of his lower back.
b.- Check an MRI of his lower back.
c.- Start physical therapy and NSAIDs.

d.- An EMG evaluation of his lower extremities.

QUESTION
What is the most appropriate therapy for him a this time?
ANSWER
a.- Start indomethacin.
b.- Start prednisone.
c.- Discontinue allopurinol.
d.- Start febuxastat.
CLINIC CASE
A 42 year old male comes in with a history of recurrent episodes of
hives over the past 5 day that are intensely pruritic on the arms,
chest, and back. He has a medical history of Diabetes Type II,
coronary artery disease and a recent urinary tract infection. His
medications include glucophage, lisinopril, baby aspirin and
trimethoprim- sulfamethoxazole. He has applied over the counter
Calamine lotion and has been taking Benadryl without much relief.
He denies any new use of soaps or products and has not been out of
the country recently. He is not in contact with any plants or
chemicals at work.
QUESTION
What is the mechanism of hypersensitivity in this case?
ANSWER
a.- Overabundance of IgA.
b.- Overabundance of IgG.
c.- Prostanglandin synthesis.
d.- Mas cell degranulation.
CLINIC CASE
A 47 year old white female with diabetes mellitus type II presents to
the clinic with an erythematous rash in her axillae and groin of one
month duration. Over the counter cortisone has failed to clear her
rash. She denies using any deodorants or creams.
QUESTION
What is the best treatment option for this diagnosis?
ANSWER
a.- Erythromcin.
b.- Trimethoprim-sulfamethoxazole.
c.- Ketoconazole cream.
d.- Ciprofloxacin.
CLINIC CASE

Pharmed Solutions Institute

PGINA 494

MANUAL DE TRABAJO DEL CURSO ENARM CMN SIGLO XXI


A 18 year old male comes to your office after returning home from
college for summer break. He complains of an intensely pruritic rash
all over his body. He notes that one week ago he went to the
student health clinic with complaints of fever, fatigue, sore throat,
neck pain, and generalized body aches. He informs you that he was
given an antibiotic but does no remember the name. He denies any
other symptoms today. On exam, you note a generalized
erythematous morbilliform rash from the base of the neck to the
dorsal aspect of the feet bilaterally. You note his tongue and palate.
QUESTION
What do you suspect the original diagnosis was in this patient?
ANSWER
a.- Influenza.
b.- Adenovirus.
c.- Mononucleosis.
d.- Bacterial pneumonia.
CONGESTIVE HEART FAILURE
Entrance to emergency room female patient aged 48, originally from
Veracruz, agricultural employee, married 17 years, gravida 5, for 3, 1
abortion, 1 cesarean, hysterectomy for fibroids 38, does not smoke
or drink alcoholic beverages, apparently healthy, takes no
medications, dental cavities previously presented but were treated
for 7 months however until three months ago that started current
condition characterized by fatigue, dyspnea on moderate exertion,
drowning sensation predominantly nocturnal, is entered because it
has a cough with sputum thin pink, mild effort dyspnea, physical
examination patient shows serious condition, swollen with central
and peripheral cyanosis, jugular venous distention, breath
mediodiastolico III / IV which predominates in the upright position,
edema pelvic limbs with ischemic lesions in fingers 1 and 2 of the

CURSO ENARM CMN SIGLO XXI TEL: 36246001

right foot, treated and contaminated local remedies, perceives no


odor or pus, vital signs are as follows TA 125/85 mmHg, HR 102, RR
26, temperature 37.5 C. The patient receives a diagnosis of
congestive heart failure.
QUESTION
What is the most likely cause for congestive heart failure in this
case?
ANSWER
a. - Bacterial endocarditis with valvular insufficiency.
b. - Mitral valve stenosis.
c. - Mixioma handset.
d. - Metastatic melanoma.
CONGESTIVE HEART FAILURE
Stabilize the patient and imaging studies are performed which
identifies an intracavitary tumor of 6 cm, in the studies were CT, MRI
and catheterization, 4 hours after the patient has severe respiratory
distress, abundant presence of cough with sputum asalmonelado, so
is intubated for ventilatory support, peripheral cyanosis is observed
with loss of distal pulses of lower limbs, absence of bowel sounds
followed by cardiac arrest twice, declaring his death two hours later.
QUESTION
What is the intervention was more likely that culminated in massive
embolism in this patient?
ANSWER
a. - No prophylactic anticoagulation.
b. - Cardiac catheterization unnecessary.
c. - The mixioma detachment.
d. - The patient already had signs of stroke.

Pharmed Solutions Institute

PGINA 495

Potrebbero piacerti anche